{ "Tag": [ "calculus", "algebra unsolved", "algebra" ], "Problem": "Let $ u_n$ satisfying $ u_0 > 0,u_{n + 1} = u_n - e^{\\frac {- 1}{u_n^2}}$\r\nProve that $ \\limits_{n\\to \\infty} (u_n^2 ln n) = 1$", "Solution_1": "[quote=\"Allnames\"]Let $ u_n$ satisfying $ u_0 > 0,u_{n + 1} = u_n - e^{\\frac { - 1}{u_n^2}}$\nProve that $ \\limits_{n\\to \\infty} (u_n^2 ln n) = 1$[/quote]\r\nNo help for me :o :o :o", "Solution_2": "I think It has some problems!\r\n$ (U_n)$ is increase,don't exeist limit -> $ (U_{n}^2)\\minus{}>\\infty$ and $ ln_n \\minus{}>\\infty$\r\n->don't exist limit of your sequence! :wink:", "Solution_3": "[quote=\"Allnames\"]No help for me :o :o :o[/quote]\r\nIt would have had a better chance had you posted it where it belongs. You could try asking a moderator to move it to the calculus-analysis section.", "Solution_4": "[quote=\"Math pro\"]I think It has some problems!\n$ (U_n)$ is increase,don't exeist limit -> $ (U_{n}^2) \\minus{} > \\infty$ and $ ln_n \\minus{} > \\infty$\n->don't exist limit of your sequence! :wink:[/quote]\nthank for your comment but my proof is true,and your idea is clearly wrong try it with the sequence $ x_n \\equal{} \\frac {1}{n}$\n[quote=\"Kent Merryfield\"]It would have had a better chance had you posted it where it belongs. You could try asking a moderator to move it to the calculus-analysis section.\n\n[/quote]\r\nthanks,but here is only a problem in a contest,and we must kill it by only all thing that we learned\r\nNow .I proved it but my proof is really and used much hard knowledge ,,who can help me", "Solution_5": "It is clearly wrong?I don't think so. with $ U_n\\equal{}\\frac{1}{n} \\minus{}> (U_n)$ have limit -> not satisfy your problem!\r\nI can't find any mistake inmy ideal!I explained you in my class but why you don't understand?\r\nI know that you r problem from a book!Can you detail the solution from the book? :maybe:", "Solution_6": "Uhm,ok,I have just check carefully your idea,and I knew where your mistake is\r\nCan you detail why $ U_n$ doesn't have limit,I know it is increase,and you told me if exist $ c$ satisfies\r\n$ U_n\\ge c$ for all $ n$,we had the wrong thing\r\nBut you should note $ lim_{n\\to \\infty } u_n\\equal{}0$ and it is a small part in my ugly proof(it is not obivious,and to proof it is quite hard)", "Solution_7": "but if $ (U_n)$ exist limit $ L \\minus{} > L \\equal{} L \\minus{} e^{ \\minus{} \\frac {1}{L^2}} \\minus{} > e^{ \\minus{} \\frac {1}{L^2}} \\equal{} 0$ Is it true? :wink:", "Solution_8": "[quote=\"Math pro\"]but if $ (U_n)$ exist limit $ L \\minus{} > L \\equal{} L \\minus{} e^{ \\minus{} \\frac {1}{L^2}} \\minus{} > e^{ \\minus{} \\frac {1}{L^2}} \\equal{} 0$ Is it true? :wink:[/quote]\r\nwrong,note $ L\\equal{}L\\minus{}e^{lim \\minus{}\\frac{1}{u_n^2}}$ and if $ lim u_n\\equal{}0$ it is until true.To prove it,we only need Lagrangre theorem,only a few lines" } { "Tag": [ "algebra", "polynomial", "number theory", "Diophantine equation", "relatively prime", "number theory unsolved" ], "Problem": "Find x,y:\r\n$x+x^{2}=y+y^{2}+y^{3}$\r\n[b]that x is natural.[/b]", "Solution_1": "The Diophantine equation \r\n\r\n$(1) \\;\\;\\; x \\:+\\: x^{2}\\;=\\; y \\:+\\: y^{2}\\:+\\: y^{3},$\r\n\r\nwhere $x$ is a natural number and $y$ is an integer, is equivalent to\r\n\r\n$(2) \\;\\;\\; (x \\:-\\: y)(x \\:+\\: y \\:+\\: 1) \\;=\\; y^{3}.$\r\n\r\nThe polynomial $y \\:+\\: y^{2}\\:+\\: y^{3}$ is strictly increasing in ${\\bf R}$. Futhermore $y \\:+\\: y^{2}\\:+\\: y^{3}\\; \\geq \\; 2$ by (1) and the fact that $x \\geq 1,$ hence $y > 0.$ Consequently $x \\: >\\: y \\: >\\: 0$ by (2).\r\n\r\nNext assume $p$ is a prime divisor of both $x \\:-\\: y$ and $x \\:+\\: y \\:+\\: 1$. Thus \r\n\r\n$p|(x \\:+\\: y \\:+\\: 1) \\:-\\: (x \\:-\\: y) \\;=\\; 2y \\:+\\: 1.$\r\n\r\nMoreover $p|y^{3}$ by (2), so $p|y.$ Therefore \r\n\r\n$p|(2y \\:+\\: 1) \\:-\\: 2y \\;=\\; 1,$\r\n\r\nwhich means that $x \\:-\\: y$ and $x \\:+\\: y \\:+\\: 1$ are relatively prime. Then according to (2)\r\n\r\n$(3) \\;\\;\\; x \\:-\\: y \\;=\\; u^{3},$ \r\n$(4) \\;\\;\\; x \\:+\\: y \\:+\\: 1 \\;=\\; v^{3},$\r\n$(5) \\;\\;\\; y \\;=\\; uv,$\r\n\r\nder $u < v$ are two relatively prime natural numbers. Subtracting (3) from (4) we obtain $2y \\:+\\: 1 \\;=\\; v^{3}\\:-\\: u^{3},$ so \r\n\r\n$(6) \\;\\;\\; v^{3}\\:-\\: u^{3}\\:-\\: 2uv \\;=\\; 1$\r\n\r\nby (5). Let $a = v-u > 0.$ Substituting $v$ with $u \\:+\\: a$ in (6), the result is\r\n\r\n\\begin{eqnarray*}1 & = & (u \\:+\\: a)^{3}\\:-\\: u^{3}\\:-\\: 2u(u \\:+\\: a) \\\\ & = & a^{3}\\:+\\: 3au(u \\:+\\: a) \\:+\\: u^{3}\\:-\\: u^{3}\\:-\\: 2u(u \\:+\\: a) \\\\ & = & a^{3}\\:+\\: (3a \\:-\\: 2)(u \\:+\\: a)u, \\\\ \\end{eqnarray*}\r\n\r\ni.e.\r\n\r\n$(7)\\;\\;\\; (3a \\:-\\: 2)uv \\;=\\; 1 \\:-\\: a^{3}.$\r\n\r\nNow $1 \\:-\\: a^{3}\\leq 0$ since $a \\geq 1$. Hence $(3a \\:-\\: 2)uv \\; \\leq \\; 0$ by (7). The fact that $v > u > 0$ implies that $a \\leq 0$. This contradiction leads to the conclusion that equation (1) has no solutions in natural numbers.\r\n\r\n[u][b]Comment:[/b][/u] By moderating this proof it is easy to show that the only solution of (1) in integers is $(x,y) = (0,0).$" } { "Tag": [], "Problem": "[color=blue][i]\u039a\u03ac\u03c0\u03bf\u03c5 \u03c4\u03b7\u03bd \u03be\u03b1\u03bd\u03b1\u03b5\u03af\u03b4\u03b1 \u03c4\u03b5\u03bb\u03b5\u03c5\u03c4\u03b1\u03af\u03b1 \u03ba\u03b1\u03b9 \u03c4\u03b7 \u03b8\u03b5\u03c9\u03c1\u03ce \u03ba\u03b1\u03bb\u03ae \u03ac\u03c3\u03ba\u03b7\u03c3\u03b7.[/i][/color]\r\n\r\n\u0394\u03af\u03bd\u03b5\u03c4\u03b1\u03b9 \u03b9\u03c3\u03bf\u03c3\u03ba\u03b5\u03bb\u03ad\u03c2 \u03c4\u03c1\u03af\u03b3\u03c9\u03bd\u03bf $ ABC$ \u03bc\u03b5 $ AB \\equal{}AC$ \u03ba\u03b1\u03b9 \u03c4\u03bf \u03bc\u03ad\u03c3\u03bf $ M$ \u03c4\u03b7\u03c2 \u03c0\u03bb\u03b5\u03c5\u03c1\u03ac\u03c2 $ AB$ .\r\n\r\n\u0391\u03c0\u03cc \u03c4\u03bf $ A$ \u03c6\u03ad\u03c1\u03bd\u03bf\u03c5\u03bc\u03b5 \u03ba\u03ac\u03b8\u03b5\u03c4\u03b7 \u03c0\u03c1\u03bf\u03c2 \u03c4\u03b7\u03bd $ CM$ \u03c0\u03bf\u03c5 \u03c3\u03c5\u03bd\u03b1\u03bd\u03c4\u03ac \u03c4\u03b7\u03bd \u03ba\u03ac\u03b8\u03b5\u03c4\u03b7 \u03c4\u03b7\u03c2 $ AB$ ,\u03c3\u03c4\u03bf $ B$ , \u03c3\u03b5 \u03c3\u03b7\u03bc\u03b5\u03af\u03bf $ D$. \r\n\r\n\u039d\u03b1 \u03b1\u03c0\u03bf\u03b4\u03b5\u03af\u03be\u03b5\u03c4\u03b5 \u03cc\u03c4\u03b9 $ DC \\equal{} DA$\r\n\r\n \u039c\u03c0\u03ac\u03bc\u03c0\u03b7\u03c2", "Solution_1": "\u039b\u03cd\u03bd\u03b5\u03c4\u03b1\u03b9 \u03b5\u03cd\u03ba\u03bf\u03bb\u03b1 \u03bc\u03b5 \u03bc\u03b9\u03b3\u03b1\u03b4\u03b9\u03ba\u03bf\u03cd\u03c2 , \u03b1\u03c1\u03ba\u03b5\u03af \u03bd\u03b1 \u03b4\u03b5\u03af\u03c4\u03b5 \u03cc\u03c4\u03b9 $ z_D\\equal{}ki(z_C\\minus{}z_M)\\equal{}mi(z_A\\minus{}z_B)\\plus{}z_B$", "Solution_2": "\u03a3\u03b5 \u03bc\u03b5\u03c1\u03b9\u03ba\u03ac \u03c0\u03c1\u03bf\u03b2\u03bb\u03ae\u03bc\u03b1\u03c4\u03b1 \u03b7 \u03bb\u03cd\u03c3\u03b7 \u03bc\u03c0\u03bf\u03c1\u03b5\u03af \u03bd\u03b1 \u03b5\u03af\u03bd\u03b1\u03b9 \u03c7\u03c9\u03c1\u03af\u03c2 \u03bb\u03cc\u03b3\u03b9\u03b1. \u03a4\u03bf \u03c3\u03c7\u03ae\u03bc\u03b1 \u03bc\u03cc\u03bd\u03bf, \u03b5\u03af\u03bd\u03b1\u03b9 \u03b1\u03c1\u03ba\u03b5\u03c4\u03cc.\r\n\r\n\u039a\u03ce\u03c3\u03c4\u03b1\u03c2 \u0392\u03ae\u03c4\u03c4\u03b1\u03c2.", "Solution_3": "[quote=\"vittasko\"]\u03a3\u03b5 \u03bc\u03b5\u03c1\u03b9\u03ba\u03ac \u03c0\u03c1\u03bf\u03b2\u03bb\u03ae\u03bc\u03b1\u03c4\u03b1 \u03b7 \u03bb\u03cd\u03c3\u03b7 \u03bc\u03c0\u03bf\u03c1\u03b5\u03af \u03bd\u03b1 \u03b5\u03af\u03bd\u03b1\u03b9 \u03c7\u03c9\u03c1\u03af\u03c2 \u03bb\u03cc\u03b3\u03b9\u03b1. \u03a4\u03bf \u03c3\u03c7\u03ae\u03bc\u03b1 \u03bc\u03cc\u03bd\u03bf \u03b5\u03af\u03bd\u03b1\u03b9 \u03b1\u03c1\u03ba\u03b5\u03c4\u03cc.\n\n\u039a\u03ce\u03c3\u03c4\u03b1\u03c2 \u0392\u03ae\u03c4\u03c4\u03b1\u03c2.[/quote]\r\n\u03a3\u03c5\u03bc\u03c6\u03c9\u03bd\u03ce \u03b1\u03c0\u03cc\u03bb\u03c5\u03c4\u03b1 \u039a\u03ce\u03c3\u03c4\u03b1,\u03bc\u03cc\u03bd\u03bf \u03c0\u03bf\u03c5 \u03c3\u03b5 \u03b1\u03c5\u03c4\u03ae \u03c4\u03b7\u03bd \u03c0\u03b5\u03c1\u03af\u03c0\u03c4\u03c9\u03c3\u03b7 \u03b8\u03b5\u03c9\u03c1\u03b5\u03af\u03c2 \u03c4\u03bf \u03c3\u03b7\u03bc\u03b5\u03af\u03bf \u039d \u03ba\u03b1\u03b9 \u03c6\u03ad\u03c1\u03bd\u03b5\u03b9\u03c2 \u03c4\u03bf\u03bd \u03ba\u03cc\u03ba\u03ba\u03b9\u03bd\u03bf \u03ba\u03cd\u03ba\u03bb\u03bf (\u03b6\u03b1\u03b2\u03bf\u03bb\u03b9\u03ac?). \u0397 \u03bb\u03cd\u03c3\u03b7 \u03bc\u03b5 \u03b1\u03bd\u03b1\u03bb\u03c5\u03c4\u03b9\u03ba\u03ae \u03b5\u03af\u03bd\u03b1\u03b9 \u03b5\u03be\u03af\u03c3\u03bf\u03c5 \u03b5\u03cd\u03ba\u03bf\u03bb\u03b7,\u03b1\u03bb\u03bb\u03ac \u03c0\u03ac\u03bd\u03c4\u03b1 \u03c0\u03b9\u03bf \u03cc\u03bc\u03bf\u03c1\u03c6\u03b7 \u03b5\u03af\u03bd\u03b1\u03b9 \u03b7 \u0395\u03c5\u03ba\u03bb\u03b5\u03af\u03b4\u03b5\u03b9\u03b1 \u03bb\u03cd\u03c3\u03b7. \u03a0\u03c1\u03cc\u03c4\u03b5\u03b9\u03bd\u03b1 \u03c4\u03bf\u03c5\u03c2 \u03bc\u03b9\u03b3\u03b1\u03b4\u03b9\u03ba\u03bf\u03cd\u03c2 \u03c3\u03b1\u03bd \u03b5\u03be\u03ac\u03c3\u03ba\u03b7\u03c3\u03b7 \u03c4\u03b7\u03c2 \u0393 \u03bb\u03c5\u03ba\u03b5\u03af\u03bf\u03c5 \u03c3' \u03b1\u03c5\u03c4\u03bf\u03cd\u03c2", "Solution_4": "\u0396\u03b7\u03c4\u03ce \u03c3\u03c5\u03b3\u03bd\u03ce\u03bc\u03b7 \u03a1\u03bf\u03b4\u03cc\u03bb\u03c6\u03b5, \u03b1\u03bb\u03bb\u03ac \u03ba\u03b1\u03bc\u03b9\u03ac \u03c6\u03bf\u03c1\u03ac \u03bd\u03bf\u03bc\u03af\u03b6\u03bf\u03c5\u03bc\u03b5 \u03cc\u03c4\u03b9 \u03ba\u03ac\u03c4\u03b9 \u03c6\u03b1\u03af\u03bd\u03b5\u03c4\u03b1\u03b9 \u03b5\u03cd\u03ba\u03bf\u03bb\u03b1 \u03b1\u03c0\u03cc \u03c4\u03bf \u03c3\u03c7\u03ae\u03bc\u03b1, \u03b5\u03c0\u03b5\u03b9\u03b4\u03ae \u03ad\u03c4\u03c5\u03c7\u03b5 \u03bd\u03b1 \u03c4\u03bf \u03b4\u03bf\u03cd\u03bc\u03b5 \u03b5\u03bc\u03b5\u03af\u03c2.\r\n\r\n\u0393\u03b9\u03b1 \u03c4\u03bf \u03c4\u03b5\u03c4\u03c1\u03ac\u03c0\u03bb\u03b5\u03c5\u03c1\u03bf $ CDEN,$ \u03c0\u03c1\u03bf\u03ba\u03cd\u03c0\u03c4\u03b5\u03b9 \u03cc\u03c4\u03b9 \u03b5\u03af\u03bd\u03b1\u03b9 \u03b5\u03b3\u03b3\u03c1\u03ac\u03c8\u03b9\u03bc\u03bf, \u03b1\u03c0\u03bf \u03c4\u03b7 \u03c3\u03c7\u03ad\u03c3\u03b7 $ (AE)\\cdot (AD) \\equal{} (AM)\\cdot (AB) \\equal{} (AN)\\cdot (AC),$ \u03b1\u03c0\u03cc \u03c4\u03bf \u03b5\u03b3\u03b3\u03c1\u03ac\u03c8\u03b9\u03bc\u03bf $ BDEM$ \u03ba\u03b1\u03b9 \u03c4\u03bf \u03b9\u03c3\u03bf\u03c3\u03ba\u03b5\u03bb\u03ad\u03c2 $ \\bigtriangleup ABC.$ \r\n\r\n\u0398\u03b1 \u03b2\u03ac\u03bb\u03c9 \u03b1\u03c1\u03b3\u03cc\u03c4\u03b5\u03c1\u03b1 \u03c0\u03b5\u03c1\u03b9\u03c3\u03c3\u03cc\u03c4\u03b5\u03c1\u03b5\u03c2 \u03bb\u03b5\u03c0\u03c4\u03bf\u03bc\u03ad\u03c1\u03b5\u03b9\u03b5\u03c2, \u03b1\u03bd \u03c7\u03c1\u03b5\u03b9\u03b1\u03c3\u03c4\u03b5\u03af.\r\n\r\n\u039a\u03ce\u03c3\u03c4\u03b1\u03c2 \u0392\u03ae\u03c4\u03c4\u03b1\u03c2." } { "Tag": [ "ratio", "quadratics", "algebra" ], "Problem": "AB,CD are two chord of a circle .and P,Q are the midpoint of them.prove that;\r\n\r\nif AB bisects the angle CPD,CD will bisect the angle AQB.", "Solution_1": "Let the chords AB, CD of the circle (O) intersect at a point X, obviously inside this circle. Denote a = XA, b = XB, c = XC, d = XD. The power of the point X to the circle (O) is equal to ab = cd. Let the lines QA, QB, PC, PD intersect the circle (O) at points B', A', D', C' different from B, A, D, C, respectively. Since P, Q are the midpoints of AB, CD, we have QA' = QA, QB' = QB, PC' = PC, PD' = PD and\r\n\r\n$QA \\cdot QB = QA \\cdot QB' = QC \\cdot QD = \\left(\\dfrac{CD}{2}\\right)^2 = \\dfrac{(c + d)^2}{4}$\r\n\r\n$PC \\cdot PD = PC \\cdot PD' = PA \\cdot PB = \\left(\\dfrac{AB}{2}\\right)^2 = \\dfrac{(a + b)^2}{4}$\r\n\r\nWLOG, assume that the line AB bisects the angle $\\angle CPD$. Then $\\dfrac{PC}{PD} = \\dfrac{XC}{XD} = \\frac c d$. Knowing both the product and the ratio of the segments PC, PD, we can calculate their lenghts:\r\n\r\n$PC = \\dfrac{a + b}{2} \\sqrt{\\frac c d},\\ \\ PD = \\dfrac{a + b}{2} \\sqrt{\\frac d c}$. Knowing the sides CD, PC, PD of the triangle $\\triangle CPD$, we can calculate the length of its median m = PQ:\r\n\r\n$4m^2 = 2(PC^2 + PD^2) - CD^2 = \\dfrac{(a + b)^2}{2} \\left(\\frac c d + \\frac d c \\right) - (c + d)^2$\r\n\r\nWe have to show that $x = \\dfrac{QA}{QB} = \\dfrac{XA}{XB} =\\frac a b$. Knowing $QA \\cdot QB = \\dfrac{(c + d)^2}{4}$, we have\r\n\r\n$x = \\dfrac{QA}{QB} = \\dfrac{(c + d)^2}{4QB^2},\\ \\ \\frac 1 x = \\dfrac{QB}{QA} = \\dfrac{(c + d)^2}{4QA^2}$\r\n\r\n$QA^2 = \\dfrac{(c + d)^2}{4} \\cdot x,\\ \\ QB^2 = \\dfrac{(c + d)^2}{4} \\cdot \\frac 1 x$\r\n\r\nThe segment m = PQ is also a median of the triangle $\\triangle AQB$, hence,\r\n\r\n$4m^2 = 2(QA^2 + QB^2) - AB^2 = \\dfrac{(c + d)^2}{2} \\left(x + \\frac 1 x \\right) - (a + b)^2$\r\n\r\nComparing the 2 expressions for the median m = PQ, we get a simple equation for $y = x + \\frac 1 x$:\r\n\r\n$\\dfrac{(c + d)^2}{2} \\left(x + \\frac 1 x \\right) - (a + b)^2 = \\dfrac{(a + b)^2}{2} \\left(\\frac c d + \\frac d c \\right) - (c + d)^2$\r\n\r\nSolving this equation for y,\r\n\r\n$y = x + \\frac 1 x = \\left(\\dfrac{a + b}{c + d}\\right)^2 \\left( \\frac c d + \\frac d c \\right) - 2 + 2\\left(\\dfrac{a + b}{c + d}\\right)^2 =$\r\n\r\n$= \\left(\\dfrac{a + b}{c + d}\\right)^2 \\dfrac{c^2 + 2cd + d^2}{cd} - 2 =$\r\n\r\n$= \\dfrac{(a + b)^2}{ab} - 2 = \\dfrac{a^2 + b^2}{ab} = \\frac a b + \\frac b a$\r\n\r\nIt is now obviuus that $x = \\frac a b$ or $\\frac b a$, but let's calculate it. Since $y = x + \\frac 1 x$, it follows that $x^2 - xy + 1 = 0$ and\r\n\r\n$x^2 - \\left(\\frac a b + \\frac b a \\right) x + 1 = 0$\r\n\r\nThe product of the 2 roots of this quadratic equation is 1 and their sum $\\frac a b + \\frac b a$ hence, they are equal to $\\frac a b$ or $\\frac b a$. Neglecting the trivial degenerated case, when a = b and both roots are equal to 1, we have one root greater than and the other less than 1. But it is clear that $x = \\dfrac{QA}{QB} > 1\\ \\Longleftrightarrow\\ \\frac a b > 1$ and the other way around. Consequently, only one root $x = \\frac a b$ is acceptable.", "Solution_2": "a very nice solution ;)", "Solution_3": "Dear Mathlinkers,\nyou can see\nhttp://www.artofproblemsolving.com/Forum/viewtopic.php?f=46&t=610405\niincerely\nJean-Louis" } { "Tag": [ "ceiling function" ], "Problem": "In a 150-year period, what is the maximum possible number of\nleap years?", "Solution_1": "Isn't there some weird rule that if the year is divisible by 100, it doesn't count as a leap year,\r\nhowever, if the year is divisible by 400, it is considered a leap year?\r\n\r\nFor example:\r\n\r\n1900 is not a leap year while 2000 is a leap year.", "Solution_2": "Yeah so you just want to make sure 2000 is the only turn of the century. \r\n\r\nIf you start on a leap year, say 1920, you get 1920,1924....2068\r\n\r\nDividing all by 4:\r\n480,481....517\r\n\r\nSubtracting 479:\r\n\r\n1,2,3...38\r\n\r\nso 38 is the maximum.", "Solution_3": "Also, since a leap year occurs every four years, we have $ \\left \\lceil \\frac{150}{4} \\right \\rceil$, or $ \\boxed{38}$.", "Solution_4": "That doesn't always work. What if I say in a 200 year period?", "Solution_5": "That is, if $ n < 200$. (I think.)" } { "Tag": [ "algebra", "polynomial", "superior algebra", "superior algebra unsolved" ], "Problem": "why is it true that f(x) is irreducible iff f(x+1) is irreducible.", "Solution_1": "Because $ s_c: A[x] \\to A[x]$, $ f(x) \\mapsto f(x\\plus{}c)$ for $ c \\in A$ is an automorphism of $ A[x]$ wih inverse $ s_{\\minus{}c}$. And automorphisms preserve irreducibility.", "Solution_2": "[quote=\"ZetaX\"]And automorphisms preserve irreducibility.[/quote]\r\nProof by dogmatic assertion. :)\r\n\r\nIf $ f(x) \\equal{} p(x)q(x)$, then $ f(x \\plus{} 1) \\equal{} p(x \\plus{} 1)q(x \\plus{} 1)$. And vice versa. So factoring $ f(x)$ is the same as factoring $ f(x \\plus{} 1)$. (note that it is important in this argument to observe that $ \\rm{deg}\\ f(x) \\equal{} \\rm{deg}\\ f(x \\plus{} 1)$)" } { "Tag": [ "inequalities", "real analysis", "real analysis theorems" ], "Problem": "What are the suffice and necessary conditions for a sequance like $\\{a_i\\}$ such that \r\n\r\n1.$\\sum_{i=1}^{\\infty} x_i=$ convergent.\r\n\r\n2.$\\sum_{i=1}^{\\infty}(-1)^i x_i=$ convergent\r\n\r\n\r\n\r\n2. ,my teacher told me : these 2 conditions are enought but i didnt managed to proof.So would someone confirme the rightness of this?\r\n1.$lim|a_i| \\longrightarrow 0$ when $i \\longrightarrow \\infty$\r\n2.$|a_{i+1} | \\le |a_i|$", "Solution_1": "The answer you have for (1) is wrong - the classical counterexample being $a_i=\\frac{1}{i}$, which is decreasing and limits to zero as in the conditions you mention, but the sum of whose terms diverges.", "Solution_2": "[quote=\"Diarmuid\"]The answer you have for (1) is wrong - the classical counterexample being $a_i=\\frac{1}{i}$, which is decreasing and limits to zero as in the conditions you mention, but the sum of whose terms diverges.[/quote]\r\nI ment this for $\\sum_{i=1}^{\\infty} (-1)^i x_i$ ,i have edited the post .\r\n\r\nBy the way ,For $a_i=\\frac1i$ what will be $\\sum_{i=1} (-1)^i a_i$ :?:", "Solution_3": "it will be $ln 2$ ;)", "Solution_4": "Well it's not really right for the second one either - e.g. $a_i=\\frac{(-1)^i}{i}$ gives $\\sum_{i=1}^\\infty(-1)^ia_i=\\sum_{i=1}^\\infty\\frac{1}{i}$, which is the same thing.\r\n\r\nIf you additionally assume that all the terms are positive (or negative), then you'd have a sufficient condition.", "Solution_5": "1) The only reasonable sufficient condition you're going to get is something like the Cauchy criterion, unless you impose much tighter restrictions on $\\{a_i\\}$. In particular, $a_i\\to 0$ is not strong enough.\r\n\r\n2) If each $a_i$ is positive, then it is enough to have $a_i\\geq a_j$ for $i\\leq j$, and $a_i\\to 0$. This is the alternating series test; you may prove it using partial summation.", "Solution_6": "i have a related question: knowing that $\\sum_{i=1}^{\\infty} |a_i|$ converges, how do you prove that $\\sum_{i=1}^{\\infty} a_i$ converges?", "Solution_7": "Triangle inequality on the partial sums?\r\n\r\n(i.e. $\\left|\\sum a_i\\right|\\le\\sum|a_i|$).", "Solution_8": "[quote=\"Diarmuid\"]Triangle inequality on the partial sums?\n\n(i.e. $\\left|\\sum a_i\\right|\\le\\sum|a_i|$).[/quote]\r\n\r\nis this inequality sufficient to prove it? i didn't manage to prove it from here :( :blush:", "Solution_9": "Yes, it is enough. Try using the Cauchy criterion." } { "Tag": [ "inequalities", "search", "inequalities proposed" ], "Problem": "Let $ a,b,c>0$ find the best $ t$ such that\r\n\r\n\\[ \\frac{a\\plus{}b}{c}\\plus{}\\frac{a\\plus{}c}{b}\\plus{}\\frac{c\\plus{}b}{a}\\plus{}t(\\frac{ab\\plus{}bc\\plus{}ca}{a^2\\plus{}b^2\\plus{}c^2})\\ge 6\\plus{}t\\]", "Solution_1": "I think $ k\\equal{}2\\sqrt{2}$", "Solution_2": "[quote=\"tranvanluan\"]I think $ k \\equal{} 2\\sqrt {2}$[/quote]\r\nNo, $ k\\equal{}4 \\sqrt{2}$.\r\n:)", "Solution_3": "Yes, you right, can_hang2007 :) Setting:\r\n\\[ f(a,b,c,t)\\equal{}LHS\\minus{}RHS\\]\r\nWe can assume that $ a\\plus{}b\\plus{}c\\equal{}1$. Hence we have:\r\n\\[ f(1\\minus{}2x,x,x,t)\\ge 0\\to k\\equal{}4\\sqrt{2}\\]\r\nEasy to prove this one :wink: Setting $ p\\equal{}a\\plus{}b\\plus{}c\\equal{}1,q\\equal{}ab\\plus{}bc\\plus{}ca,r\\equal{}abc$ so we have \r\n\\[ q\\le 1/3,r\\le \\frac{q^2(1\\minus{}q)}{2(2\\minus{}3q)}\\]\r\nThe inequality is equivalent to:\r\n\\[ \\frac{q}{r}\\plus{}\\frac{4q\\sqrt{2}}{1\\minus{}2q}\\minus{}9\\minus{}4\\sqrt{2}\\ge 0\\]\r\n\\[ \\iff \\frac{2(2\\minus{}3q)}{q(1\\minus{}q)}\\plus{}\\frac{4q\\sqrt{2}}{1\\minus{}2q}\\minus{}9\\minus{}4\\sqrt{2}\\ge 0\\]\r\n\\[ \\iff \\frac{(1\\minus{}3q)[(4\\sqrt{2}\\plus{}6)q^2\\minus{}(11\\plus{}4\\sqrt{2})q\\plus{}4]}{q(1\\minus{}q)(1\\minus{}2q)}\\ge 0,\\forall q\\le 1/3\\]\r\n\r\nWe are done :wink:", "Solution_4": "[quote=\"zaizai-hoang\"]Yes, you right, can_hang2007 :) Setting:\n\\[ f(a,b,c,t) \\equal{} LHS \\minus{} RHS\n\\]\nWe can assume that $ a \\plus{} b \\plus{} c \\equal{} 1$. Hence we have:\n\\[ f(1 \\minus{} 2x,x,x,t)\\ge 0\\to k \\equal{} 4\\sqrt {2}\n\\]\nEasy to prove this one :wink: Setting $ p \\equal{} a \\plus{} b \\plus{} c \\equal{} 1,q \\equal{} ab \\plus{} bc \\plus{} ca,r \\equal{} abc$ so we have\n\\[ q\\le 1/3,r\\le \\frac {q^2(1 \\minus{} q)}{2(2 \\minus{} 3q)}\n\\]\nThe inequality is equivalent to:\n\\[ \\frac {q}{r} \\plus{} \\frac {4q\\sqrt {2}}{1 \\minus{} 2q} \\minus{} 9 \\minus{} 4\\sqrt {2}\\ge 0\n\\]\n\n\\[ \\iff \\frac {2(2 \\minus{} 3q)}{q(1 \\minus{} q)} \\plus{} \\frac {4q\\sqrt {2}}{1 \\minus{} 2q} \\minus{} 9 \\minus{} 4\\sqrt {2}\\ge 0\n\\]\n\n\\[ \\iff \\frac {(1 \\minus{} 3q)[(4\\sqrt {2} \\plus{} 6)q^2 \\minus{} (11 \\plus{} 4\\sqrt {2})q \\plus{} 4]}{q(1 \\minus{} q)(1 \\minus{} 2q)}\\ge 0,\\forall q\\le 1/3\n\\]\nWe are done :wink:[/quote]\r\nDear Hoang, in your proof, I dont see when does the second equality hold?", "Solution_5": "The best $ t$ :$ 4\\sqrt{2}$\r\n\r\nLet $ a\\plus{}b\\plus{}c\\equal{}1$ and $ ab\\plus{}bc\\plus{}ca\\equal{}\\frac{1\\minus{}s^2}{3}$such that $ 0\\le s\\le 1$ and $ r\\equal{}abc$\r\nthen we have $ r\\le \\frac{(1\\minus{}s)^2(1\\plus{}2s)}{27}$\r\n\r\nThe problem is equivalent to\r\n\r\n\\[ s^2((2\\plus{}\\sqrt{2})s\\plus{}1\\minus{}\\sqrt{2})^2\\ge 0\\]\r\n\r\nWhich is true :wink:", "Solution_6": "To zaizai-hoang:\r\nYou wrote \" We can assume that $ a\\plus{}b\\plus{}c\\equal{}1$.\" But it refers to a special case, doesn't it?? :huh: \r\n\r\nCan u post ur proof can_hang? :)", "Solution_7": "[quote=\"Sunkern_sunflora\"]To zaizai-hoang:\nYou wrote \" We can assume that $ a \\plus{} b \\plus{} c \\equal{} 1$.\" But it refers to a special case, doesn't it?? :huh: \n\nCan u post ur proof can_hang? :)[/quote]\r\nDear Sunkern_sunflora,\r\nThe reason we can assume $ a\\plus{}b\\plus{}c\\equal{}1$ is due to the following property\r\n\\[ f(ka,kb,kc) \\equal{}k^n f(a,b,c)\\]\r\nfor all $ k$.\r\nSo, in here, if we let $ k\\equal{}\\frac{1}{a\\plus{}b\\plus{}c}$, we will have\r\n\\[ f\\left( \\frac{a}{a\\plus{}b\\plus{}c},\\frac{b}{a\\plus{}b\\plus{}c},\\frac{c}{a\\plus{}b\\plus{}c}\\right) \\equal{}\\frac{1}{(a\\plus{}b\\plus{}c)^n} f(a,b,c)\\]\r\nSo if $ a,b,c>0$ or $ n$ is even, and our question is to prove $ f(a,b,c) \\ge 0$, it suffices for us to prove that\r\n\\[ f\\left( \\frac{a}{a\\plus{}b\\plus{}c},\\frac{b}{a\\plus{}b\\plus{}c},\\frac{c}{a\\plus{}b\\plus{}c}\\right) \\ge 0\\]\r\nwhich is equivalent to\r\n\\[ f(a_1,b_1,c_1) \\ge 0\\]\r\nfor $ a_1\\plus{}b_1\\plus{}c_1\\equal{}1$.\r\nNow, in our original problem, if we denote $ f(a,b,c) \\equal{}LHS\\minus{}RHS$, then you will have\r\n\\[ f(ka,kb,kc) \\equal{}f(a,b,c)\\]\r\nSo we can apply our above property. :)\r\nThis is called \"homogeneous\". :)\r\nBy the way, you can try to prove this inequality by SOS or vornicu-schur or mixing variables or more...\r\nRegards\r\n:)", "Solution_8": "Thank u can_hang2007! There's so much to learn from u! :)\r\nCould u also tell me what is vornicu-schur, what is SOS or what is mixing variables, please! :(", "Solution_9": "[quote=\"Sunkern_sunflora\"]Thank u can_hang2007! There's so much to learn from u! :)\nCould u also tell me what is vornicu-schur, what is SOS or what is mixing variables, please! :([/quote]\r\nFor the Vornicu-Schur: http://www.mathlinks.ro/viewtopic.php?t=49427\r\nFor the SOS: http://www.mathlinks.ro/viewtopic.php?t=80127\r\nFor the mixing variables, I am sorry, I cant find the link for you.", "Solution_10": "[quote=\"can_hang2007\"]For the Vornicu-Schur: http://www.mathlinks.ro/viewtopic.php?t=49427\nFor the SOS: http://www.mathlinks.ro/viewtopic.php?t=80127\nFor the mixing variables, I am sorry, I cant find the link for you.[/quote]\r\nFirst of all, THANK U VERY MUCH for the two links u provided. I could understand the SOS technique, but not Valentin-Schur for so sure. I'll have to study on that link further.\r\nAs for the mixing variables, could u please jist it a bit for me?? U explain subjects very good. Thank u very much. In our bengali, we call it \"DHANYOBAD\"! :)", "Solution_11": "[quote=\"Sunkern_sunflora\"][quote=\"can_hang2007\"]For the Vornicu-Schur: http://www.mathlinks.ro/viewtopic.php?t=49427\nFor the SOS: http://www.mathlinks.ro/viewtopic.php?t=80127\nFor the mixing variables, I am sorry, I cant find the link for you.[/quote]\nFirst of all, THANK U VERY MUCH for the two links u provided. I could understand the SOS technique, but not Valentin-Schur for so sure. I'll have to study on that link further.\nAs for the mixing variables, could u please jist it a bit for me?? U explain subjects very good. Thank u very much. In our bengali, we call it \"DHANYOBAD\"! :)[/quote]\r\nI have just found the following lecture on mixing variables of Garbriel Dospinescu and Tran Nam Dung. :)", "Solution_12": "Thank u! I have downloaded your source, but it shows that it is an RAR file. What do I need to open a RAR file?", "Solution_13": "[quote=\"Sunkern_sunflora\"]Thank u! I have downloaded your source, but it shows that it is an RAR file. What do I need to open a RAR file?[/quote]\r\nYou just need to go to http://www.google.com. And search for the winrar program, then install it. :)" } { "Tag": [ "abstract algebra", "linear algebra", "matrix", "absolute value", "linear algebra unsolved" ], "Problem": "Let A be an ortogonal matrix and b an eigenvalue of A. Prove that module(b)=1.", "Solution_1": "The English word would be \"modulus.\" The word \"module\" means something else. (I'd more likely say \"absolute value,\" anyway.)\r\n\r\nSo suppose $ A$ is orthogonal, which means that $ A^TA\\equal{}AA^T\\equal{}I,$ and that $ \\lambda$ is an eigenvalue of $ A.$\r\n\r\nThen $ Ax\\equal{}\\lambda x$ for some $ x\\ne0.$ However, it is possible that $ \\lambda$ and $ x$ are complex valued.\r\n\r\nIn what lies below, let $ x^*$ and $ A^*$ be the conjugate transpose of the matrix in question, and for real $ A,$ we have $ A^*\\equal{}A^T.$\r\n\r\n$ |\\lambda|^2\\|x\\|^2\\equal{}\\overline{\\lambda}x^*x\\lambda \\equal{}(\\lambda x)^*(\\lambda x)\\equal{}(Ax)^*(Ax)\\equal{}x^*A^*Ax\\equal{}x^*Ix\\equal{}x^*x\\equal{}\\|x\\|^2.$\r\n\r\nIf we divided by $ \\|x\\|^2,$ which is nonzero, we get $ |\\lambda|^2\\equal{}1$ which gives us that $ |\\lambda|\\equal{}1.$\r\n\r\nNote that we didn't actually use that $ A$ was real; we only used that $ A^*A\\equal{}I.$ Hence we may change the hypothesis of \"orthogonal\" to \"unitary.\"" } { "Tag": [ "geometry", "calculus", "calculus computations" ], "Problem": "Let $ h(t)$ and $ V(t)$ represent the height and volume of water in a tank at time $ t$. If water leaks through a hole with area a at the bottom of the tank, then Torricelli\u2019s Law says that $ \\frac{dV}{dt}\\equal{}\\minus{}a\\sqrt{2gh}$, where $ g$ is the acceleration due to gravity. So, the rate at which water flows from the tank is proportional to the square root of the water height.\r\n\r\nSuppose the tank is cylindrical with height $ 6$ feet and radius $ 2$ feet. The hole is circular with radius $ 1$ inch. If $ g \\equal{} 32$ feet per second squared, show that $ \\frac{dh}{dt}\\equal{}\\minus{}\\frac{1}{72}\\sqrt{h}$.\r\n\r\nAssuming that the tank is full at time t = 0, show that $ h(t)\\equal{}\\left(\\minus{}\\frac{1}{144}t\\plus{}\\sqrt{6}\\right)^2$ is a solution to the differential equation in (a).\r\n\r\nHow long will it take for the water in the tank to drain completely?", "Solution_1": "I'm not deriving the solution but I'll do the second part. Set $ h(t)\\equal{}0$ so we get\r\n$ \\frac{1}{144}t\\equal{}\\sqrt{6}\\Rightarrow t\\equal{}144\\sqrt{6}$\r\nSo at that time, the water is drained." } { "Tag": [ "geometry", "parallelogram", "analytic geometry", "complex numbers", "geometric transformation" ], "Problem": "Prove, using [u]only geometry[/u] that the center of the squares constructed on the sides of a parallelogram form a square as well.", "Solution_1": "Do you mean Euclidean geometry? There's a relatively simple proof [hide=\"using\"]complex numbers.[/hide]", "Solution_2": "[hide=\"The Relevant Diagram\"]Consider the image below [may take a few moments to load]. We hope to show that the four red segments form a square. To aid the process, I've created four light green triangles out of various segments. Can you see their relevance?\n\n[img]http://img.photobucket.com/albums/v194/mukmasterxps/pww_para_square.png[/img][/hide]", "Solution_3": "We could alternatively use the fact that given a triangle ABC with squares constructed on AB and AC, if M is the midpoint of AC and X and Y are the centers of the squares on AB and AC, respectively, then XMY is an isosceles right triangle. \r\n\r\nBut honestly, complex numbers would be my choice. =P", "Solution_4": "Does the Mean Geometry Theorem count? Since that can be proven using spiral similarity...", "Solution_5": "Got it from TZF's picture.\r\n\r\nAll you need to use is the basic congruence principle: If two triangles have the same side , angle in between, and the other side equal, then those triangles are congruent.\r\n\r\nThe whole point is to use most basic form of geometry, defeats the purpose using complex numbers.\r\n\r\nThanks for the attention guys, appreciate it", "Solution_6": "What is the proof of using complex numbers? I tried it (first assigning the coordinates) but it gets very messy...\r\nAlso, what is the Mean Geometry Theorem? I couldn't find it on the internet... :?:", "Solution_7": "I'm not sure why...the MGT just says that the weighted average of 2 directly similar figures is similar to the original figure...its a result of spiral similarity that makes the proof of Napoleon's theorem, Van Aubel's Theorem and a ton of other results easier I guess...its not really anything new in my opinion" } { "Tag": [], "Problem": "If $ a\\plus{}b\\equal{}8$, $ b\\plus{}c\\equal{}\\minus{}3$, and $ a\\plus{}c\\equal{} \\minus{}5$, what is the value of the product $ abc$?", "Solution_1": "We are given that\r\n\\begin{align} a + b & = 8 \\\\\r\nb + c & = - 3 \\\\\r\na + c & = - 5 \\end{align}\r\nSubtracting $ (2)$ from $ (1)$ and then adding $ (3)$, we get \\[ 2a = 6\\Rightarrow a = 3\\] Substituting, we find that \\[ b = 5 \\\\ c = - 8\\] \r\nThe product is $ (3)(5)( - 8) = \\boxed{ - 120}$", "Solution_2": "hello, solving the equation system\r\n$ a\\plus{}b\\equal{}8$\r\n$ b\\plus{}c\\equal{}\\minus{}3$\r\n$ a\\plus{}c\\equal{}\\minus{}5$\r\nwe get $ a\\equal{}3,b\\equal{}5,c\\equal{}\\minus{}8$, so our product has the value $ abc\\equal{}\\minus{}120$.\r\nSonnhard." } { "Tag": [ "advanced fields", "advanced fields unsolved" ], "Problem": "Let I = [0,1] and A = {0,1,1/2,1/3,...,}. Show that there is no continuous retraction \r\n$ I \\times I \\to I \\times {0} \\cup {A \\times I}$.\r\n\r\nI am guessing we want to use continuous on a compact set implies uniform continuity and then consider points near the y axis. I can't seem to make it rigorous though.", "Solution_1": "$ I \\times I$ is locally connected, but $ I \\times 0 \\cup A \\times I$ not, and the retraction is closed", "Solution_2": "[quote=\"myan\"]$ I \\times I$ is locally connected, but $ I \\times 0 \\cup A \\times I$ not, [/quote]\n\nI agree.\n\n[quote=\"myan\"]and the retraction is closed[/quote]\r\n\r\nCan you elaborate? Why is the retraction closed? Do closed retractions preserve local connectedness? I can't find that theorem in Munkres..." } { "Tag": [ "calculus", "integration", "logarithms", "parameterization", "derivative", "calculus computations" ], "Problem": "Compute $\\int_{0}^{1}\\left( x-\\frac12 \\right) \\cdot \\ln \\left(-\\ln x \\right) \\, dx$.", "Solution_1": "Your integral equals: $-\\frac{\\ln 2}{2}$. \r\n\r\nRecall that $\\int_{0}^{\\infty}e^{-x}\\ln x dx=-\\gamma$.\r\n\r\nLet $I$ be the value of your integral. Make the substitution $x=e^{-t}$ and you get that\r\n\r\n$I=\\int_{0}^{\\infty}\\left(e^{-2t}-\\frac{1}{2}e^{-t}\\right)\\ln t dt=\\int_{0}^{\\infty}e^{-2t}\\ln t dt-\\frac{\\gamma}{2}$.\r\n\r\nThe substitution $2t=y$ shows that the integral equals $-\\frac{\\ln 2}{2}$.", "Solution_2": "Nice solution, didilica. A few comments:\r\n\r\n1. We don't need to know what the \"exact\" value of $\\int_{0}^\\infty e^{-x}\\ln x \\, dx$ is. The fact that it converges is enough.\r\n\r\n2. How do you prove that $\\int_{0}^\\infty e^{-x}\\ln x \\, dx =-\\gamma$ anyway?\r\n\r\n3. By the way, I was computing $\\int_{0}^{1}\\frac{x-1}{\\ln x}\\, dx$ (I found it in a post of Jimmy, together with a [color=white]\"parameter differentiation\"[/color] solution) and I reached this integral after an integration by parts.", "Solution_3": "Here is a method of attack.\r\n$\\Gamma(a)=\\int_{0}^{\\infty}x^{a-1}e^{-x}dx$, valid for $Re(a)>0$. \r\n\r\n$\\Gamma'(a)=\\int_{0}^{\\infty}x^{a-1}\\ln x e^{-x}dx$.\r\n\r\nit is known that $\\Gamma'(1)=-\\gamma$\r\n\r\n$\\Gamma'(1)=\\int_{0}^{\\infty}e^{-x}\\ln(x)dx=-\\gamma$.\r\n\r\n\r\n\r\n$\\psi(z)=-\\frac{1}{z}-\\gamma-\\sum_{n=1}^{\\infty}\\left(\\frac{1}{z+n}-\\frac{1}{n}\\right)$, $z\\neq 0,-1,-2,\\cdots$.\r\n\r\n$\\psi(1)=-\\Gamma'(1)=-\\gamma$." } { "Tag": [ "inequalities", "inequalities unsolved" ], "Problem": "1. Let $a, b, c \\geq 0$. Show that $a^5 + b^5 + c^5 \\geq 5abc(b^2 - ac)$.", "Solution_1": "[quote=\"Centy\"]1. Let $a, b, c \\geq 0$. Show that $a^5 + b^5 + c^5 \\geq 5abc(b^2 - ac)$.\n[/quote]\r\nTake$\\frac{a}{b}=m,\\frac{c}{n}$(if $b=0$ the ineq. is done)\r\n$m,n>0$\r\n$a^5 + b^5 + c^5 \\geq 5abc(b^2 - ac)$<=>$m^5+n^5+5m^2n^2+1\\geq 5mn$\r\n$m^5+n^5\\geq 2\\sqrt{m^5n^5}=2\\sqrt{t^2}$=> we must only prove that \r\n$f(t)=2t^5+5t^4-5t^2+1\\geq 0$ when $t\\geq 1$=>$5t^4\\geq 5t^2$=>$f(t)\\geq 0$.\r\nNow let's find the minimum valu of $f$ in $[0;1]$.\r\n$f'(t)=10t(t+1)(t-\\frac{\\sqrt{5}-1}{2})(t+\\frac{\\sqrt{5}+1}{2})$\r\n$f(0)=1>0$,so we only need to see that$f(\\frac{\\sqrt{5}-1}{2})\\geq 0$,I think that isn't hard to do :lol: ;) .", "Solution_2": "[quote=\"Tiks\"] we only need to see that$f(\\frac{\\sqrt{5}-1}{2})\\geq 0$ [/quote]\r\nTiks, I checked it:\r\n$f(t)=2t^5+5t^4-5t^2+1=(t^2+t-1)^2(2t+1).$ :? :D :)", "Solution_3": "[quote=\"arqady\"][quote=\"Tiks\"] we only need to see that$f(\\frac{\\sqrt{5}-1}{2})\\geq 0$ [/quote]\nTiks, I checked it:\n$f(t)=2t^5+5t^4-5t^2+1=(t^2+t-1)^2(2t+1).$ :? :D :)[/quote]\r\nThat is nicer way :lol: ,I couldn't find any root of $f$ :) .", "Solution_4": "[quote=\"Centy\"]1. Let $a, b, c \\geq 0$. Show that $a^5 + b^5 + c^5 \\geq 5abc(b^2 - ac)$.[/quote]\r\n\r\nI remember posting and solving this, a very nice and unusual inequality. My solution is [url=http://www.mathlinks.ro/Forum/viewtopic.php?p=318045#p318045]here.[/url]", "Solution_5": "Why is $b = 1$ without loss of generality?", "Solution_6": "The inequality is homogenous, so we can divide or multiply $a$, $b$ and $c$ by any factor we want - it doesn't change anything. Ok?", "Solution_7": "How did I not see that. :blush:" } { "Tag": [ "number theory proposed", "number theory" ], "Problem": "Find all odd positive integers (a, b, c) such that \r\nab(a2+b2) = 2c4.\r\n\r\nI thought of this an hour ago. I am not sure if anyone else has ever made this exact problem before, so if they have, my apologies.", "Solution_1": "I came to this while solving this problem: $(e^4-f^4)^2+(e^4+f^4)^2=(2g^2)^2$ where $e,f$ are both odd and prime to each other. Is it easier from this point?", "Solution_2": "We can assume that $(a,b)=(b,c)=(c,a)=1$\r\nThen Exist $x,y$ integer:\r\n$ab=x^4$\r\n$a^2+b^2=2y^4$\r\n$U=a+b,V=a-b$\r\n$\\rightarrow 2y^4+2x^4=U^2,2y^4-2x^4=V^2$\r\n$\\rightarrow u^2+v^2=4y^4$\r\n$U^2-V^2=4x^4$\r\nThen $V=0$ and $a=b=c$.", "Solution_3": "sorry for my ignorance, but why does the last fact imply that V must be 0?", "Solution_4": "he use the fact that : there are no distinct positive integers $x,y$ for which $x^2+y^2$ and $x^2-y^2$ are both squares", "Solution_5": "Yes,And the general problem is:\r\n$xy(x+y)=2z^2$\r\n(Instead of $2z^4$)", "Solution_6": "thank you pluricomplex, and actually upon looking at it again it is not very hard to deduce from what we have, since:\r\n\r\n$(u^2-v^2)(2^2+v^2) = u^4 - v^4 = (2xy)^4$ so by Fermat-Wiles, we deduce v = 0 since 2xy > 0 :P" } { "Tag": [ "geometry", "rectangle", "incenter", "geometry unsolved" ], "Problem": "Let $ ABC$ be a rectangle triangle in A. Where $ r$ is the radius of the circle inscribed in ABC, prove that: \r\n\r\n$ r\\equal{}\\frac{AB\\plus{}AC\\minus{}BC}{2}$\r\n\r\n[img]http://img43.imageshack.us/img43/7697/aosdjas.jpg[/img]", "Solution_1": "Let $ O$ be the incenter of $ \\triangle ABC$, and points $ D,E,F$ on $ BC,AC,AB$ respectively such that $ OD,OE,OF$ are the radius of the incircle. \r\n\r\nWe have $ AE\\equal{}r$, then $ DE\\equal{}AC\\minus{}r$, and it is obvious that $ DE\\equal{}CD$ so $ CD\\equal{}AC\\minus{}r$\r\nAnalogously , $ FB\\equal{}BD\\equal{}AB\\minus{}r$\r\n$ BD\\plus{}CD\\equal{}BC$\r\n$ AC\\minus{}r\\plus{}AB\\minus{}r\\equal{}BC$\r\n$ r\\equal{}\\frac{AC\\plus{}AB\\minus{}BC}{2}$\r\n\r\nQED", "Solution_2": "Thanks ;)\r\nIt's a nice method. Thanks very much." } { "Tag": [ "geometry", "geometric transformation", "reflection", "geometry proposed" ], "Problem": "[b]Easy modified enunciation.[/b]\r\n\r\nConsider an $ A$-isosceles triangle $ ABC$ and the points : $ \\boxed M$ for which $ AM\\perp BC$ ; $ \\boxed N\\in AB$ for which \r\n\r\n$ N\\not\\equiv B$ and $ MN \\equal{} MC$ ; $ \\boxed P\\in AM\\cap CN$ ; $ \\boxed R\\in CM$ for which $ BR\\parallel AC$ . Prove that $ PR \\equal{} PC$ .\r\n\r\n[b]Notations[/b] : $ AB$ - [u]line[/u] and $ [AB]$ - [u]segment[/u] !", "Solution_1": "Here is my solution. :) \r\n\r\n[img]http://i524.photobucket.com/albums/cc322/khanhto-photo/untitled-109.jpg[/img]\r\n\r\nDenote by $ T \\equal{} (M,MC) \\cap AC (T\\not\\equiv C)$ \r\nWe see that :$ BT,CN,AM$ concur at $ P$.\r\nWe have :$ \\angle {PNM} \\equal{} \\angle{PCM} \\equal{} \\angle {PBM}$ ,i.e $ N,P,M,B$ lie on a circle . (1)\r\nBut: $ \\angle {NMC} \\equal{} 2 \\angle {NBC} \\equal{} 2 ( 90^0 \\minus{} \\frac {\\angle {A}}{2}) \\equal{} 180^0 \\minus{} \\angle {A} \\equal{} \\angle {NBR}$,so $ N,B,R,M$ lie on a circle.. (2)\r\n\r\nFrom (1) and (2) ,we have $ N,P,M,R$ lie on a circle ,so $ \\angle {PRM} \\equal{} \\angle {MNP} \\equal{} \\angle {PCM}$,i.e $ PR \\equal{} PC$", "Solution_2": "[b]Here is another approach:[/b]\r\nThanks to $ MN\\equal{}MC\\Longrightarrow \\angle MNC\\equal{}\\angle MCN$. In the other hand, $ B$ is the reflection of $ C$ respect to $ AM$, thus $ \\angle NCM\\equiv \\angle PCM\\equal{}\\angle PBM$, which implies that $ \\angle PNM\\equal{}\\angle PBM\\Longrightarrow \\{N,P,M,B\\}$ is the set of concyclic point.\r\n$ BR//AC\\Longrightarrow \\angle BRC\\equiv \\angle BRM\\equal{}\\angle ACR\\equiv \\angle ACM\\equal{}\\angle ABM$.\r\nBut due to the fact that $ MN\\equal{}MC\\equal{}MB\\Longrightarrow \\angle MNB\\equal{}\\angle MBN\\equiv \\angle ABM$, which implies that $ \\angle BRM\\equal{}\\angle BNM\\Longrightarrow R\\in (NBM)\\Longrightarrow \\{R,N,P,M,B\\}$ is the set of concyclic point. Thus, $ \\angle PRM\\equal{}\\angle PNM\\equal{}\\angle PCM$, which implies that $ \\triangle PRC$ is $ P$-isosceles $ \\Longrightarrow PR\\equal{}PC$.\r\nOur proof is completed then.", "Solution_3": "Dear Mathlinkers,\r\n1. Note (0) the circle centered at M and passing through C, T, N, B (se message from ma 29)\r\n1. the isoceles triangles MTB and MCN are similar\r\n2. M, P, T, C lie on circle (1)\r\n M, P, B, C lie on circle (2) ; note that these two circles intersect at M and P ; we have working wrt M\r\n3. By a converse of Reim's theorem applied to (1) and (2), R is on (2)\r\n4. Now consider again (1) and (2) and work wrt P ; then PCR is isoceles at P...\r\nSincerely\r\nJean-Louis", "Solution_4": "I liked very much this problem. Thanks to all for interest and nice proofs. \r\nHere is and my proof which can be similarly with one from these.\r\n\r\n[b][u]Proof[/u].[/b] $ m(\\angle MCN)=90^{\\circ}-B=m(\\angle MAC)$ $ \\implies$ $ \\widehat {MCP}\\equiv\\widehat {MAC}$ $ \\Longleftrightarrow$ ${ \\triangle MCP\\sim \\triangle MAC}$ $ \\implies$ $ \\widehat {MPC}\\equiv \\widehat {MCA}$ $ \\Longleftrightarrow$ \r\n\r\n$ \\widehat {MPB}\\equiv\\widehat {MRB}$ $ \\Longleftrightarrow\\ MPRB$ is cyclically $ \\Longleftrightarrow$ $ \\widehat {PRM}\\equiv\\widehat {PBM}$ $ \\Longleftrightarrow$ $ \\widehat {PRC}\\equiv\\widehat {PCM}$ $ \\Longleftrightarrow$ $ \\widehat {PRC}\\equiv\\widehat {PCR}$ $ \\Longleftrightarrow$ $ PR=PC$ ." } { "Tag": [], "Problem": "How many digits are in the number $ 25^{21}\\times2^{48}$?", "Solution_1": "First, let's see how many 0's it ends in. We have plenty of 2's, so let's see how many fives we have. $ 25^{21} \\equal{} 5^{2 \\cdot 21} \\equal{} 5^{42}$(coincidence? I think not.) Therefore, there will be 42 0's. Remaining is $ 2^6 \\equal{} 64$, which has two digits, so our final answer is $ 42 \\plus{} 2 \\equal{} \\boxed{44}$." } { "Tag": [ "logarithms", "ratio", "arithmetic sequence", "geometric sequence" ], "Problem": "$\\boxed{1}$\r\n\r\n(1) Find the $n$ th term of the sequence $\\{a_{n}\\}$ such that $a_{1}=2,\\ a_{n+1}=2a_{n}+1\\ (n=1,\\ 2,\\ 3,\\ \\cdots).$\r\n\r\n(2) Find the $n$ th term of the sequence $\\{b_{n}\\}$ such that $b_{1}=2,\\ b_{n+1}=2b_{n}+n\\ (n=1,\\ 2,\\ 3,\\ \\cdots).$\r\n\r\n[url]http://www.mathlinks.ro/Forum/viewtopic.php?t=108627[/url]", "Solution_1": "$\\boxed{2}$\r\n\r\nGiven the sequence $\\{a_{n}\\}$ which is defined by $a_{1}=1,\\ a_{n+1}=2a_{n}+2^{n}\\ (n=1,\\ 2,\\ 3,\\ \\cdots).$ Find the $n$ th term $a_{n}$ and sum $\\sum_{k=1}^{n}a_{k}.$\r\n\r\n[url]http://www.mathlinks.ro/Forum/viewtopic.php?t=108644[/url]", "Solution_2": "$\\boxed{3}$\r\n\r\nFind the $n$ th term of the sequence $\\{a_{n}\\}$ such that $a_{1}=1,\\ a_{n+1}=na_{n}+n-1\\ (n=1,\\ 2,\\ 3,\\ \\cdots).$\r\n\r\n[url]http://www.mathlinks.ro/Forum/viewtopic.php?t=108680[/url]", "Solution_3": "$\\boxed{4}$\r\n\r\nFind the $n$ th term of the sequence $\\{a_{n}\\}$ such that $a_{1}=1,\\ a_{n+1}=\\frac{a_{n}}{2a_{n}+3}\\ (n\\geq 1).$\r\n\r\n[url]http://www.mathlinks.ro/Forum/viewtopic.php?t=108782[/url]", "Solution_4": "$\\boxed{5}$\r\n\r\nFind the $n$ th term of the sequence $\\{a_{n}\\}$ such that $a_{1}=\\frac{1}{2},\\ (n-1)a_{n-1}=(n+1){a_{n}}\\ (n\\geq 2).$\r\n\r\n[url]http://www.mathlinks.ro/Forum/viewtopic.php?t=108784[/url]", "Solution_5": "$\\boxed{6}$\r\n\r\nFind the $n$ th term of the sequence $\\{a_{n}\\}$ such that $a_{1}=1,\\ a_{n+1}=2a_{n}^{2}\\ (n=1,\\ 2,\\ 3,\\ \\cdots).$\r\n\r\n[url]http://www.mathlinks.ro/Forum/viewtopic.php?t=108785[/url]", "Solution_6": "$\\boxed{7}$\r\n\r\nFind the $n$ th term of the sequence $\\{x_{n}\\}$ such that $x_{1}=2,\\ x_{n+1}=(n+1)^{2}\\left(\\frac{2x_{n}}{n^{2}}-1\\right)\\ (n=1,\\ 2,\\ 3,\\ \\cdots).$\r\n\r\n[url]http://www.mathlinks.ro/Forum/viewtopic.php?t=108786[/url]", "Solution_7": "$\\boxed{8}$\r\n\r\nThe positive sequence $\\{a_{n}\\}$ satisfies the following conditions $[A],\\ [B].$\r\n\r\n$[A]\\ \\ \\ a_{1}=1.$\r\n\r\n$[B]\\ \\ \\ \\log a_{n}-\\log a_{n-1}=\\log (n-1)-\\log (n+1)\\ \\ (n\\geq 2).$\r\n\r\nFind $\\sum_{k=1}^{n}a_{k}.$\r\n\r\n[url]http://www.mathlinks.ro/Forum/viewtopic.php?t=108787[/url]", "Solution_8": "$\\boxed{9}$\r\n\r\nFind the $n$ th term of the sequence $\\{a_{n}\\}$ such that $a_{1}=1,\\ a_{2}=3,\\ a_{n+1}-3a_{n}+2a_{n-1}=2^{n}\\ \\ (n\\geq 2).$\r\n\r\n[url]http://www.mathlinks.ro/Forum/viewtopic.php?t=108911[/url]", "Solution_9": "$\\boxed{10}$\r\n\r\nThe operation $*$ which makes two non zero integers $m,\\ n$ correspond to the integers $m*n$ satisfies the following three conditions.\r\n\r\n[A] $0*n=n+1.$\r\n\r\n[B] $m*0=m+1.$\r\n\r\n[C] $m*n=(m-1)*(m*(n-1)) \\ \\ (m\\geq 1,\\ n\\geq 1).$\r\n\r\n(1) Find $1*n.$\r\n(2) Find $2*n.$\r\n(3) Find $3*n.$\r\n\r\n[url]http://www.mathlinks.ro/Forum/viewtopic.php?t=108919[/url]", "Solution_10": "$\\boxed{11}$\r\n\r\nFind the $n$ th term of the sequence $\\{a_{n}\\}$ which is defined by $a_{1}=0,\\ a_{n}=\\left(1-\\frac{1}{n}\\right)^{3}a_{n-1}+\\frac{n-1}{n^{2}}\\ (n=2,\\ 3,\\ \\cdots).$\r\n\r\n[url]http://www.mathlinks.ro/Forum/viewtopic.php?t=109050[/url]", "Solution_11": "$\\boxed{12}$\r\n\r\nFind the $n$ th term of the sequence $\\{a_{n}\\}$ such that $a_{1}=1,\\ a_{n+1}^{2}=-\\frac{1}{4}a_{n}^{2}+4\\ \\ (a_{n}>0,\\ n\\geq 1).$\r\n\r\n[url]http://www.mathlinks.ro/Forum/viewtopic.php?t=109051[/url]", "Solution_12": "$\\boxed{13}$\r\n\r\nFind the $n$ th term of the sequence $\\{a_{n}\\}$ such that $a_{1}=1,\\ a_{n+1}=2a_{n}-n^{2}+2n\\ (n=1,\\ 2, \\ 3,\\ \\cdots).$\r\n\r\n[url]http://www.mathlinks.ro/Forum/viewtopic.php?t=109055[/url]", "Solution_13": "$\\boxed{14}$\r\n\r\nFind the $n$ th term of the sequence $\\{a_{n}\\}$ such that $\\ a_{1}=1,a_{n+1}=\\frac{1}{2}a_{n}+\\frac{n^{2}-2n-1}{n^{2}(n+1)^{2}}\\ (n=1,2,3,....) .$\r\n\r\n[url]http://www.mathlinks.ro/Forum/viewtopic.php?t=109138[/url]", "Solution_14": "$\\boxed{15}$\r\n\r\nFind the $n$ th term of the sequence $\\{x_{n}\\}$ such that $x_{n+1}=x_{n}(2-x_{n})\\ (n=1,\\ 2,\\ 3,\\ \\cdots)$ in terms of $x_{1}.$\r\n\r\n[url]http://www.mathlinks.ro/Forum/viewtopic.php?t=109309[/url]", "Solution_15": "$\\boxed{16}$\r\n\r\nFind the $n$ th term of the sequence $\\{a_{n}\\}$ such that $\\frac{a_{1}+a_{2}+\\cdots+a_{n}}{n}=n+\\frac{1}{n}\\ \\ (n=1,\\ 2,\\ 3,\\ \\cdots).$\r\n\r\n[url]http://www.mathlinks.ro/Forum/viewtopic.php?t=109310[/url]", "Solution_16": "$\\boxed{17}$\r\n\r\nFind the $n$ th term of the sequence $\\{a_{n}\\}$ such that $\\sum_{k=1}^{n}a_{k}=n^{3}+3n^{2}+2n.$ and Calculate $\\sum_{k=1}^{n}\\frac{1}{a_{k}}.$\r\n\r\n[url]http://www.mathlinks.ro/Forum/viewtopic.php?t=109312[/url]", "Solution_17": "$\\boxed{18}$\r\n\r\nFind the $n$ th term of the sequence $\\{a_{n}\\}$ such that $\\sum_{k=1}^{n}a_{k}=3n^{2}+4n+2\\ (n=1,\\ 2,\\ 3,\\ \\cdots)$ and calculate $\\sum_{k=1}^{n}a_{k}^{2}.$\r\n\r\n[url]http://www.mathlinks.ro/Forum/viewtopic.php?t=109318[/url]", "Solution_18": "$\\boxed{19}$\r\n\r\nFind the $n$ th term of the sequence $\\{a_{n}\\}$ such that $a_{1}=0,\\ a_{2}=1,\\ (n-1)^{2}a_{n}=\\sum_{k=1}^{n}a_{k}\\ \\ (n\\geq 1).$\r\n\r\n[url]http://www.mathlinks.ro/Forum/viewtopic.php?t=109320[/url]", "Solution_19": "$\\boxed{20}$\r\n\r\nLet $a_{n}$ be the $n$ th term of the arithmetic sequence with $a_{1}=7,$ the common difference $2,$ and $b_{n}$ be \r\n\r\n the $n$ th term of the geometric sequence with $b_{1}=\\frac{1}{3},$ the common ratio $\\frac{1}{3}.$ \r\nFor the sequence $\\{c_{n}\\},$ If $\\sum_{k=1}^{n}a_{k}b_{k}c_{k}=\\frac{1}{3}(n+1)(n+2)(n+3)$ holds, then find $c_{n}$ and evaluate $\\sum_{n=1}^{\\infty}\\frac{1}{c_{n}}.$\r\n\r\n[url]http://www.mathlinks.ro/Forum/viewtopic.php?t=109325[/url]", "Solution_20": "$\\boxed{21}$\r\n\r\nFind the $n$ th term of the sequence $\\{a_{n}\\}$ such that $a_{1}=a\\ (\\neq-1),\\ a_{n+1}=\\frac{1}{2}\\left(a_{n}+\\frac{1}{a_{n}}\\right)\\ (n\\geq 1).$\r\n\r\n[url]http://www.mathlinks.ro/Forum/viewtopic.php?t=109472[/url]", "Solution_21": "$\\boxed{22}$\r\n\r\nFind the $n$ th term of the positive sequence $\\{a_{n}\\}$ such that $a_{1}=1,\\ a_{2}=10,\\ a_{n}^{2}a_{n-2}=a_{n-1}^{3}\\ (n=3,\\ 4,\\ \\cdots).$\r\n\r\n[url]http://www.mathlinks.ro/Forum/viewtopic.php?t=109473[/url]", "Solution_22": "$\\boxed{23}$\r\n\r\nFind the $n$ th term of the sequence $\\{a_{n}\\}$ such that $a_{1}=1,\\ na_{n}=(n-1)\\sum_{k=1}^{n}a_{k}\\ (n=2,\\ 3,\\ \\cdots).$\r\n\r\n[url]http://www.mathlinks.ro/Forum/viewtopic.php?t=109474[/url]", "Solution_23": "$\\boxed{24}$\r\n\r\nFind the $n$ th term of the seuence $\\{a_{n}\\}$ such that $a_{1}=\\frac{1}{2},\\ a_{2}=\\frac{1}{3},\\ a_{n+2}=\\frac{a_{n}a_{n+1}}{2a_{n}-a_{n+1}+2a_{n}a_{n+1}}.$\r\n\r\n[url]http://www.mathlinks.ro/Forum/viewtopic.php?t=109475[/url]", "Solution_24": "$\\boxed{25}$\r\n\r\nDefine the sequence $\\{a_{n}\\}$ such that $a_{1}=-4,\\ a_{n+1}=2a_{n}+2^{n+3}n-13\\cdot 2^{n+1}\\ (n=1,\\ 2,\\ 3,\\ \\cdots).$\r\nFind the value of $n$ for which $a_{n}$ is minimized.\r\n\r\n[url]http://www.mathlinks.ro/Forum/viewtopic.php?t=109479[/url]", "Solution_25": "$ \\boxed{26}$\r\n\r\nFind the $ n$ th term of the sequence $ \\{a_{n}\\}$ such that $ a_{1} \\equal{} \\frac 32,\\ \\ a_{n \\plus{} 1} \\equal{} 2a_{n}(a_n \\plus{} 1)\\ (n \\geq 1).$\r\n\r\n[url]http://www.mathlinks.ro/Forum/viewtopic.php?t=329978[/url]" } { "Tag": [ "inequalities" ], "Problem": "Let $M,N,P$ be arbitrary points on the sides $BC,CA,AB$ respectively of an acute-angled triangle $ABC$. Prove that at least one of the following inequalities is satisfied: $NP\\geq \\frac 12 BC,PM\\geq \\frac 12 CA, MN\\geq \\frac 12 AB$.", "Solution_1": "[quote]bebakhshd ke saal ro engilisi ferestadam[/quote]\r\n\r\n :P", "Solution_2": "azizan lotfan ide bedahid , masaleye asooniye ha ;)" } { "Tag": [ "binomial coefficients", "Pascal\\u0027s Triangle" ], "Problem": "Can someone put Pascal's identity in layman's terms instead of binomials and coefficients?", "Solution_1": "Well binomial coefficients aren't too complicated if you think of $ \\dbinom nk$ as $ n$ \"choose\" $ k$ or the number of ways to choose $ k$ things from $ n$ things, where the order of the $ k$ things doesn't matter.\r\n\r\nThen there are several ways to remember this.\r\n\r\nOne way is to use Pascal's Triangle, where the zeroth row is $ \\left\\{\\dbinom00\\right\\} \\equal{} \\{1\\}$, the first row is $ \\left\\{\\dbinom10,\\dbinom11\\right\\} \\equal{} \\{1,1\\}$, and so on so that the $ n^{th}$ row is $ \\left\\{\\dbinom n0,\\dbinom n1,\\dbinom n2,\\ldots,\\dbinom nn\\right\\}$. But each element in Pascal's Triangle is the sum of the two elements directly above it! This quickly gives us $ \\dbinom{n \\minus{} 1}{k \\minus{} 1} \\plus{} \\dbinom{n \\minus{} 1}k \\equal{} \\dbinom nk$, as desired.\r\n\r\nAnother way is a combinatorial argument. In the simplest manner, we can choose $ k$ items from $ n$ things in $ \\dbinom nk$ ways, where order doesn't matter. Alternatively, we can mark one item as \"bad,\" so that there are $ \\dbinom{n \\minus{} 1}k$ ways to choose $ k$ items from the $ n$ items without the \"bad\" item in the group, and $ \\dbinom{n \\minus{} 1}{k \\minus{} 1}$ ways to choose $ k$ items from the $ n$ items with the \"bad\" item in the group. However, every group must have or not have that \"bad\" item! Thus, $ \\dbinom nk \\equal{} \\dbinom{n \\minus{} 1}{k \\minus{} 1} \\plus{} \\dbinom{n \\minus{} 1}k$.\r\n\r\nFinally, we can just use the algebraic definition $ \\dbinom nk \\equal{} \\frac {n!}{k!(n \\minus{} k)!}$, but this is hardly as interesting as the other two arguments. It doesn't help to remember it either.\r\n\r\nThe first method is the main reason why it's called Pascal's Identity, and is (in my opinion) the easiest way to remember it." } { "Tag": [ "LaTeX" ], "Problem": "A total of $ 28$ handshakes was exchanged at the conclusion of a party. Assuming that each participant was equally polite toward all the others, the number of people present was:\r\n\r\n$ \\textbf{(A)}\\ 14 \\qquad\\textbf{(B)}\\ 28 \\qquad\\textbf{(C)}\\ 56 \\qquad\\textbf{(D)}\\ 8 \\qquad\\textbf{(E)}\\ 7$", "Solution_1": "[hide]\n\nThe number of handshakes with $ n$ people is $ \\binom{n}{2}$. Hence, $ n(n\\minus{}1)/2\\equal{}28$ and $ n\\equal{}8\\Longrightarrow\\mathrm{ (D) }$\n\n[/hide]", "Solution_2": "Another way: think that if there are n people at a party, then the first person can shake hands with (n-1) people. Then, the next person can shake hands with all but the first person, or (n-2) people. Then the third person can shake hands with (n-3) and etc. until the second to last person shakes hands with the last person and all handshakes have been completed. Realizing this, we can simply add consecutive integers to get 28: 1+2+3+4+5+6+7 = 28 and so there are 8 people at the party.", "Solution_3": "Since each person shook some one's hand once, 1 handshake counts for 1 for each pair of people. The first would have x, the second x-1 (b/c we already counted the one he had w/ the first person), the 3rd x-2, etc. The sum ends up being 1+2+3+...+x-1+x. The equation x(x+1)/2=28 where x+1 is the number of people (x is the no. of handshakes the 1st person has, and since that person didn't shake hands w/ himself, the no. of people is n+1). We get x(x+1)=56, and x^2+x=56; x^2+x-56=0; (x-7)(x+8)=0. Since x can't be negative, x=7, and x=1=[u]8 people[/u].\nI wish i knew more latex, but oh well...", "Solution_4": "This means that there were $1+2+...+(n-1)$ handshakes, which in turn is\n\n$\\frac{n(n-1)}{2}=28$ \n\nSimplifying gives,\n\n$n^2-n-56=0$, which is $(n-8)(n+7)=0$.\n\nThe only positive solution is $n=\\boxed{8}$, which is $\\boxed{\\text{D}}$", "Solution_5": "[quote=\"Brut3Forc3\"]A total of $ 28$ handshakes was exchanged at the conclusion of a party. Assuming that each participant was equally polite toward all the others, the number of people present was:\n\n$ \\textbf{(A)}\\ 14 \\qquad\\textbf{(B)}\\ 28 \\qquad\\textbf{(C)}\\ 56 \\qquad\\textbf{(D)}\\ 8 \\qquad\\textbf{(E)}\\ 7$[/quote]\n\nThe number of people determines the number of handshakes. There is an equation: n people shake hands with n-1 people. Therefore, $\\frac{n(n-1)}{2}=28$, which becomes $n=8$." } { "Tag": [ "arithmetic series" ], "Problem": "[b]Formulas:[/b]\r\n tn=t1+(n-1)d . . . to find the position number of a term, I believe\r\n Sn=(n/2)[(2(t1))+(n-1)d] . . . to find the sum of an arithmetic series when the last term is not given\r\n Sn=(n/2)(t1+tn) . . . to find the sum of an arithmetic series when given the first [i]and[/i] last terms\r\n \r\n[b]The Problem:[/b]\r\n\"Find the sum of the multiples of 7 from 84 to 371, inclusive\". So, I need to find the sum of all multiples of 7 from 84 up to 371. When I asked my teacher how to do this, she said I needed to find the postion number, because I don't know how many terms there are between 84 and 371 that are multiples of 7.\r\n\r\nShe showed me...\r\n\r\ntn=t1+(n-1)d\r\n371=84+(n-1)7\r\n\r\n84 is the first term, or t1\r\nI do not know n, because n is the number of terms, which I need to find\r\nd is 7, since it is the common difference between each number, making multiples of seven\r\n\r\nBut why is tn equal to 371? What is tn? Any help would be greatly appreciated.", "Solution_1": "What you're trying to evaluate is $ \\sum_{k\\equal{}12}^{53}7k$, right?\r\n\r\nYou seem like you're not understanding how these formulas are derived; but to solve this problem, all you need is the simple $ \\sum_{k\\equal{}0}^{n}k\\equal{}\\frac{n(n\\plus{}1)}{2}$.\r\n\r\nYour sum is equivalent to $ 7\\left(\\sum_{k\\equal{}0}^{53}k\\minus{}\\sum_{k\\equal{}0}^{11}k\\right)$. Make sure you see why; the $ 7$ is a constant multiplier, so summing $ 84\\plus{}96\\plus{}\\cdots\\plus{}371$, you're summing $ 7(12\\plus{}13\\plus{}\\cdots\\plus{}53)$, which is the same thing. Additionally, to simplify things, we add everything from $ 7\\times 0$ to $ 7\\times 53$, then get rid of the excess.\r\n\r\nIf you understand the way these formulas are derived, you shouldn't be so confused.\r\n\r\n[b]EDIT:[/b]\r\nOh, just in case you're not familiar with sigma notation, an example:\r\n\\[ \\sum_{k\\equal{}a}^{b}f(k)\\equal{}f(a)\\plus{}f(a\\plus{}1)\\plus{}\\cdots\\plus{}f(b).\\]", "Solution_2": "Um...I'm sorry, but I have no idea what you're talking aboiut. I'm trying to solve the problem I posted under the bolded text titled \"The Problem\" and I have never seen my teacher, or anyone else, work an arithmetic series the way you just did.\r\n\r\nI apologize for my ignorance.", "Solution_3": "Here's how I would do it:\r\n\r\nTake 84-371 and divide it by 7. You now have 12-53.\r\n\r\nPairing the numbers, you get 53+12=65, 52+13=65, etc. You have 42 numbers or 21 pairs, so 21*65=1365. Now multiply this answer by 7 to get what you originally had.", "Solution_4": "Ok, I'll explain it as simply as possible.\r\n\r\nAn arithmetic series looks like $ S\\equal{}a\\plus{}(a\\plus{}r)\\plus{}(a\\plus{}2r)\\plus{}\\cdots\\plus{}(a\\plus{}nr)$. If we want to find a \"closed form\" for $ S$ (that is, an expression without \"...\" that gives the same answer), we'll use a couple of tricks.\r\n\r\nFirst off, let's find a closed form for $ 1\\plus{}2\\plus{}\\cdots\\plus{}n$.\r\nLet $ S\\equal{}1\\plus{}2\\plus{}\\cdots\\plus{}n$. Now, look at this:\r\n$ S\\equal{}1\\plus{}2\\plus{}\\cdots\\plus{}n$\r\n$ S\\equal{}n\\plus{}(n\\minus{}1)\\plus{}\\cdots\\plus{}1$.\r\nAdd them together:\r\n$ 2S\\equal{}\\underbrace{(1\\plus{}n)\\plus{}(1\\plus{}n)\\plus{}\\cdots\\plus{}(1\\plus{}n)}_{n\\text{ times}}$.\r\n\r\nTherefore, $ 2S\\equal{}n(1\\plus{}n)$, and $ S\\equal{}\\frac{n(n\\plus{}1)}{2}$.\r\n\r\nTo summarize, $ \\sum_{k\\equal{}0}^{n}k\\equal{}\\frac{n(n\\plus{}1)}{2}$. That fancy sigma notation is just saying $ 0\\plus{}1\\plus{}\\cdots\\plus{}n$.\r\nThis is an extremely important result, so know it well.\r\n\r\nNow what you want is $ \\sum_{k\\equal{}12}^{53}7k$. That's summing a bunch of multiples of 7 together, so we can just pull the 7 out (as I showed in the other post), so it's equivalent to $ 7\\sum_{k\\equal{}12}^{53}k$.\r\n\r\nNow you should be feeling a bit better, because $ \\sum k$ is familiar. However, those bounds are weird; we're used to sums from 0 to $ n$, but when the lower bound is not $ 0$, the sum becomes more complicated.\r\n\r\nBut we can be clever and recognize that $ \\sum_{k\\equal{}a}^{b}f(k)\\equal{}\\sum_{k\\equal{}0}^{b}f(k)\\minus{}\\sum_{k\\equal{}0}^{a}f(k)$. Chew on that for a while, list it out as terms instead of using sigma notation if it helps, and make sure you see why it works.\r\n\r\nThen you get the clean expression $ 7\\left(\\sum_{k\\equal{}0}^{53}k\\minus{}\\sum_{k\\equal{}0}^{11}k\\right)$ for your arithmetic series. Now, substitute $ \\sum_{k\\equal{}0}^{n}k\\equal{}\\frac{n(n\\plus{}1)}{2}$ and you're done.", "Solution_5": "You have 7*12, 7*13, 7*14 ... 7*53. There are (53-12) + 1 = 42 terms in this arithmetic sequence. Now you can apply the summation formula for an arithmetic sequence. \r\n\r\n(84 + 371)/2 times 42 = 9555. \r\n\r\nThis formula, for clarity, is simply (first term + last term)/2 times the number of terms.", "Solution_6": "That's equivalent to what I'm saying.\r\n\r\n$ 7\\left(\\sum_{k\\equal{}0}^{53}k\\minus{}\\sum_{k\\equal{}0}^{11}k\\right)\\equal{}7\\left(\\frac{53\\cdot 54}{2}\\minus{}\\frac{11\\cdot 12}{2}\\right)\\equal{}\\boxed{9555}$.\r\n\r\nI just went through the derivation of the formula." } { "Tag": [ "Miscellaneous Problems" ], "Problem": "For how many positive integers $n$ is \\[\\left( 1999+\\frac{1}{2}\\right)^{n}+\\left(2000+\\frac{1}{2}\\right)^{n}\\] an integer?", "Solution_1": "$ \\Leftrightarrow 2^n|3999^n + 4001^n$\r\nThe first we has $ n$ is odd .\r\nConsider $ n\\equiv 1(\\mod 2)$\r\nThen we has :\r\n$ 3999^n + 4001^n = 8000(\\frac {3999^n + 4001^n}{8000})$\r\nLemma: $ \\gcd(a + b,\\frac {a^n + b^n}{a+b}) = \\gcd(n,a + b)$ where $ \\gcd(a,b) = 1$\r\nSo $ \\gcd(8000,(\\frac {3999^n + 4001^n}{8000}) = \\gcd(800,n)$ \r\nSo $ 2^n|8000$ so $ n\\in = {1,3,5\\}}$\r\n[u]Result [/u]There are 3 number in this sequence is interger.", "Solution_2": "Your solution looks correct, but can you please add a proof of the lemma in your post? :)", "Solution_3": "Yes it from Newton express.\r\n$ \\gcd(a\\plus{}b,\\frac{a^n\\plus{}b^n}{a\\plus{}b})\\equal{}\\gcd(a\\plus{}b,na^n)\\equal{}\\gcd(n,a\\plus{}b)$", "Solution_4": "We can rephrase the problem as: Solve for $n$ such that $$v_2(3999^n+4001^n) \\ge n.$$ \n\nNote that if $n$ is an odd integer, we have $$3999^n+4001^n=(3999+4001)*(3999^{n-1}4001+3999^{n-2}4001^2 \\dots +4001^{n-1}).$$ Clearly the latter part of the expansion is not divisible by $2$, because modulo $2$ on gives $1+1 \\dots +1$ iterated $n$ times, which is odd. Now computing gives $v_2(3999^n+4001^n)=6v_2(n)=6$. This gives the solutions $n=1,3,5$.\n\nFor evens, we can simply note that for even $n$, $2$ trivially divides $3999^n+4001^n$ but $4$ does not divide $3999^n+4001^n$, as modulo $4$ gives $(-1)^n+(1)^n \\equiv 2 \\pmod{4}$. Thus no even $n$ holds." } { "Tag": [ "inequalities", "inequalities proposed" ], "Problem": "Prove that there existed a positive number $C,$ irrelevant to $n$ and $a_1,\\ a_2,\\ \\cdots a_n,$ satisfying the following condition.\r\n\r\nCondition: For arbiterary positive numbers $n$ and arbiterary real numbers $a_1,\\ \\cdots a_n,$ the following inequality holds.\r\n\r\n\\[ \\max_{0\\leq x\\leq 2}\\prod_{j=1}^n |x-a_j|\\leq C^n \\max_{0\\leq x\\leq 1}\\prod_{j=1}^n |x-a_j|. \\]", "Solution_1": "This ineq is hard. \r\nThis one was posted two times in unsolved section, by hxtung and flip2004. Try to search.\r\nIn flip2004's topic, he prove it for $C=6+\\sqrt{5}$." } { "Tag": [ "inequalities" ], "Problem": "x is a reral numbr $\\sqrt{\\frac{x-6}{4}}+\\sqrt{\\frac{x-7}{3}}+\\sqrt{\\frac{x-8}{2}}=\\sqrt{\\frac{x-4}{6}}+\\sqrt{\\frac{x-3}{7}}+\\sqrt{\\frac{x-2}{8}}$ :idea:", "Solution_1": "We have that $\\sqrt{\\frac{x-m}{10-m}}\\geq \\sqrt{\\frac{x-(10-m)}{m}}$ if and only if $2(x-10)(m-5) \\geq 0$.\r\nNow suppose we have that $x\\geq 10$. Then by our 'lemma' the following inequalities hold: \\[\\begin{aligned}& \\sqrt{\\frac{x-6}{4}}\\geq \\sqrt{\\frac{x-4}{6}}\\\\ & \\sqrt{\\frac{x-7}{3}}\\geq \\sqrt{\\frac{x-3}{7}}\\\\ & \\sqrt{\\frac{x-8}{2}}\\geq \\sqrt{\\frac{x-2}{8}}\\end{aligned}\\] with equality iff $x=10$. But by this inequalities $LHS \\geq RHS$, so we have equality. Therefore, $x=10$ if $x\\geq 10$.\r\nIf $x\\leq 10$, we use the same argument to show that $LHS \\leq RHS$ with equality iff $x=10$, so $x=10$.\r\n\r\nOh yeah, don't forget to use a more significant title than \"easy equation\" and \"hyper-easy\" next time :)", "Solution_2": "[quote=\"Kurt G\u00f6del\"]\nOh yeah, don't forget to use a more significant title than \"easy equation\" and \"hyper-easy\" next time :)[/quote]\r\nMe thinks [b]arkhammedos[/b] is following in the footsteps of [b]boxedexe[/b] :)", "Solution_3": "Hey $Kurt$ my solution is $extra$ easyr than urs , we rewrite the equoition in this form\r\n$\\sqrt{\\frac{x-6}{4}}-\\sqrt{\\frac{x-4}{6}}+\\sqrt{\\frac{x-7}{3}}-\\sqrt{\\frac{x-3}{7}}+\\sqrt{\\frac{x-8}{2}}-\\sqrt{\\frac{x-2}{8}}=0$\r\n We have $\\sqrt{\\frac{x-a}{b}}-\\sqrt{\\frac{x-b}{a}}=\\frac{(a-b)[x-(a+b)]}{ab[\\sqrt{\\frac{x-a}{b}}+\\sqrt{\\frac{x-b}{a}}]}$,then by substituting a and b by 2;3,4,6,7,8 we will find $(x-10)[S]$ wish $S\\neq 0$ then $x=10$\r\n :rotfl:" } { "Tag": [ "Euler", "ratio", "geometry", "perpendicular bisector", "geometry unsolved" ], "Problem": ":( :( :(", "Solution_1": "The line perpendicular to $ AE_a$ through $ M_a$ contains side $ BC$. Let the intersection be $ H_a$, the foot of the altitude from $ A$. Extend $ AE_a$ through $ E_a$ to $ H$ so that $ AE_a\\equal{}E_aH$. $ H$ is the orthocenter. (The Euler point $ E_a$ is the midpoint of $ AH$.)\r\nThe circle with diameter $ E_aM_a$ is the nine-point circle. Notice that $ \\triangle E_aE_bE_c$ is homothetic to $ \\triangle ABC$, with the center at $ H$ and the ratio being $ \\frac12$. Then $ HH_a\\perp BC\\Rightarrow HH_a\\perp E_bE_c$ and $ E_bE_c$ bisects $ HH_a$. Since $ E_b$ and $ E_c$ also lie on the nine-point circle, they are the intersections of the perpendicular bisector of $ HH_a$ with the nine-point circle.\r\n$ B$ and $ C$ are the intersections of $ HE_b$ and $ HE_c$ with $ BC$." } { "Tag": [ "search", "function", "complex numbers", "irrational number" ], "Problem": "Choose the answer that is both irrational and complex.\r\n\r\nA. $\\pi i$ \r\nB. $\\sqrt{3136}$ \r\nC. $\\frac{-e}{4}$\r\nD. $\\sqrt3-2i$\r\nE. $NOTA$\r\n\r\nEDITED.", "Solution_1": "[quote=\"surge\"]Solve for all real solutions of $x$.\n\n$2^x=x^2$\n\n[hide=\"hint\"]there are 3 solutions[/hide][/quote]\r\n\r\nuse search function, this problem was posted many times.....", "Solution_2": "[hide]D[/hide]", "Solution_3": "[hide=\"Official Answer\"]C[/hide]", "Solution_4": "Why cant it be A?", "Solution_5": "I guess because $0+\\pi i$ is not irrational... while $\\frac{-e}{4}$ can be expressed as complex? I dunno, that's why I posted it (from Mu Alpha Theta competition).", "Solution_6": "[quote=\"surge\"]I guess because $0+\\pi i$ is not irrational...[/quote]\r\n\r\n$\\pi i$ not irrational? But that would mean that it's rational... And the answer is that $\\frac{-e}4$ is irrational and [b]complex[/b]? Isn't $\\frac{-e}4$ real?\r\n\r\nMu Alpha Theta must've made a mistake...", "Solution_7": "But all real numbers are also complex numbers with imaginary part equal to zero...\r\nFrom most definitions I have met before, an irrational number is a real number that is not rational. So irrationals are real numbers but I have never seen them called \"complex irrationals\"....\r\nFor $\\pi i$, this is not a real number. Maybe you can call $i$ as a unit but that's not how it's used in most high school's definition of irrational??? :ninja: So maybe because of this, A is ruled out. B is an integer. D is definitely a complex number." } { "Tag": [ "logarithms", "number theory", "prime factorization", "rational number", "irrational number" ], "Problem": "Is $\\log_{10}{8}$ rational?", "Solution_1": "Suppose not. Then for some coprime positive integers x,y we have $10^{x/y} = 8$. It implies $10^x = 8^y$. By ideas from prime factorization, x = 3y. But this doesnt work.\r\n\r\nedit: explanation for gopher :: We suppose that it is rational. It implies this: ($10^{x/y} = 8$) [x,y integer]. We then show this cant be satisfied in integers. So our assumption is incorrect.", "Solution_2": "That only proves that it is not algebraic. Couldn't it be transcendental? Also, your proof is a bit vague. Could you go through it a bit slower? Thanks", "Solution_3": "[quote=\"gopherhole112\"]That only proves that it is not algebraic. Couldn't it be transcendental? Also, your proof is a bit vague. Could you go through it a bit slower? Thanks[/quote]I'm not sure where $x=3y$ came from, but I think \"suppose not\" was a typo, since we really want to suppose it is rational, and then find the contradiction.", "Solution_4": "[quote=\"AntonioMainenti\"][quote=\"gopherhole112\"]That only proves that it is not algebraic. Couldn't it be transcendental? Also, your proof is a bit vague. Could you go through it a bit slower? Thanks[/quote]I'm not sure where $x=3y$ came from, but I think \"suppose not\" was a typo, since we really want to suppose it is rational, and then find the contradiction.[/quote]\r\n$10^x=8^y$\r\n$2^x5^x=2^{3y}$\r\n$x=3y$ by comparing the power of $2$", "Solution_5": "[quote=\"gopherhole112\"]That only proves that it is not algebraic. Couldn't it be transcendental? Also, your proof is a bit vague. Could you go through it a bit slower? Thanks[/quote]\r\nWhether it is algebraic or transcendental does not matter here; the question only tells us to prove whether it is rational or not. (In this case, we are going to prove it is irrational.) And singular's proof is indeed quite clear at all.", "Solution_6": "[ I do not know the method of writing 10 a bit lower than log and writing exponent a bit higher than the base.So I have written '10'\r\n wherever I have wanted to mean 10 based log and used \"q\"\r\nwhenever I have wanted to mean q as a exponent :oops: :? :? :) ] \r\n\r\nSuppose log'10'8=x \r\n10\u00b0=1 , 10\u00b9=10 and 10\"x\" =8.\r\nSo 00, q\u22601 and p, q are relatively prime.\r\n\r\n\r\n log'10'8 = p/q\r\n\r\n Or, log'10'8+ log'10'(5/4)=p/q+ log'10'(5/4)\r\n\r\n Or, log'10'[(8)(5/4)]= p/q+ log'10'(5/4)\r\n\r\n Or, log'10'10= p/q+ log'10'(5/4)\r\n\r\n Or, 1= p/q+ log'10'(5/4)\r\n\r\n Or, [p+ qlog'10'(5/4)]/q=1\r\n\r\nOr, p+ [log'10'(5/4)\"q\"]/q =1\r\n\r\nOr, p+ [log'10'(5/4)\"q\"]= q\r\n\r\n As q is a whole number so (5/4) \"q\" is a fraction .So [log'10'(5/4)\"q\"] is a fraction.\r\nSo p+ [log'10'(5/4)\"q\"] is a fraction because p is a whole number.\r\nBut q is obviously a whole number.\r\nSo p+ [log'10'(5/4)\"q\"]\u2260 q\r\nSo log'10'8 is not a rational number. \r\nSo log'10'8 is irrational.\r\n [Proved]\r\n[please help me in writing exponent and 10 based log correctly :) ]", "Solution_7": "Of course, it is rational, for everyone who works in hexadecimal system. It is indeed $0.C$ which is nothing nut $3/4$. :)", "Solution_8": "So my proof that it was irrational was wrong?????\r\n\r\nPlease answer soon and clearly.", "Solution_9": "Sorry - Your proof was correct , I was saying it in a little jest In Hexadicmal system $10$ is $16$ in Decimal system, and of ccourse, $16^{3/4} = 8$ ....If one worked with computers (specially IBM architecture - etc) - Hexadecimal system is quite common, in fact many carry calculators which do all calcualtions in hexadecimal..\r\n\r\nAgain sorry for confusion, but hope that helps.", "Solution_10": "Of course, it is rational\n", "Solution_11": "[quote=leepakhin][quote=\"AntonioMainenti\"][quote=\"gopherhole112\"]That only proves that it is not algebraic. Couldn't it be transcendental? Also, your proof is a bit vague. Could you go through it a bit slower? Thanks[/quote]I'm not sure where x=3y came from, but I think \"suppose not\" was a typo, since we really want to suppose it is rational, and then find the contradiction.[/quote]\n10^x=8^y\n2^x5^x=2^{3y}\nx=3y by comparing the power of 2[/quote]\nBut this is incorrect since 2^x5^x=2^{3y} does not lead to the conclusion x=3y, in fact it leads to the conclusion 5^x=2^{3y-x} since we are not comparing only the powers of 2 on both sides of the equation, right? (Sorry, as a new user here, I am not allowed to post images so I have to make do without the Latex code.)", "Solution_12": "(My solution was incorrect)\n\n[s]$\\log_{10}{8}$ is not rational.\n\nLet's assume it is rational. Then there must be integers x and y where $10^{x/y} = 8$.\n\nWhenever we take 10 to the power of 1/y, we get an irrational number because 10 is not a perfect square/cube/etc. We multiply $10^{1/y}$ by $10^x$, which is rational. Since 8 is rational, it's impossible for it to be the product of one rational and one irrational number. Therefore, $\\log_{10}{8}$ cannot be rational.\n\nLet me know if you think my reasoning is incorrect.[/s]", "Solution_13": "[quote=basketballguy]\\log_{10}{8} is not rational.\n\nLet's assume it is rational. Then there must be integers x and y where 10^{x/y} = 8$.\n\nWhenever we take 10 to the power of 1/y, we get an irrational number because 10 is not a perfect square/cube/etc. We multiply 10^{1/y} by 10^x, which is rational. Since 8 is rational, it's impossible for it to be the product of one rational and one irrational number. Therefore, \\log_{10}{8} cannot be rational.\n\nLet me know if you think my reasoning is incorrect.[/quote]\nIf I'm not wrong, though, I think 10^{1/y} multiplied by 10^x is not equal to 10^{x/y}, in fact it is only equal to 10^{1/y+x}, and likewise 10^{x/y} can only be simplified or expressed as (10^{1/y})^x.", "Solution_14": "@pear333 you are correct. Here is a new solution:\n\n$\\log_{10}{8}$ is not rational.\n\nLet's assume it is rational. Then there must be integers x and y where $10^{x/y} = 8$. We rewrite $10^{x/y} = 8$ as $10^x = 8^y$. Since $10 = 2*5$ and $8 = 2^3$, we can write that as:\n\n$2^x * 5^x = 2^{3y}$\n\nThe above expression becomes:\n\n$2^{3y-x} = 5^x$.\n\nHowever, there are no integers x and y that satisfy $2^{3y-x} = 5^x$ except for x = 0 and y = 0. That's because there's no way to get a power of 2 through multiplying 5 by itself over and over again (and vice versa). The only way that works is if you set both x and y equal to 0, but $\\log_{10}{8}$ is clearly not $0/0$.\n\nSince we had originally set x/y to be the solution to $\\log_{10}{8}$, and x and y cannot be integers, $\\log_{10}{8}$ is not rational." } { "Tag": [], "Problem": "Prove that $\\left\\{ \\frac{x}{2}\\right\\}$ is either equal to $\\frac{\\{x\\}}{2}$ or $\\frac{\\{x\\}}{2}+\\frac{1}{2}$.", "Solution_1": "let $x=2a+b$ where $a\\in\\mathbb{Z}$ and $0\\le b<2$" } { "Tag": [ "calculus", "integration", "limit", "trigonometry", "calculus computations" ], "Problem": "Evaluate\r\n\\[ \\lim_{x\\rightarrow 0}\\frac {1}{x}\\int_{0}^{x}\\left(1 \\plus{} \\sin{2t}\\right)^{1/t}\\,dt.\\]\r\nI know how to do this via l'Hoptal's rule, but is there another approach?", "Solution_1": "Use the Mean Value theorem for the integral and the problem reduces to calculate the following limit\r\n\r\n$ \\lim\\limits_{\\theta\\rightarrow 0^{\\plus{}}}(1\\plus{}\\sin 2\\theta)^{1/\\theta}$.", "Solution_2": "Ah okay, I know the integrand is continuous. So just to make sure, the mean value theorem for the integral guarantees a number c in [0,x] such that our limit is basically the limit as x approaches 0 of f(c), where f is basically the integrand except at 0, where it is the value that makes the integrand continuous (removable discontinuity). But as x goes to zero, so does c, and hence the same result as if we had applied l'Hopital in the first place. Is this right? Thanks." } { "Tag": [ "vector", "linear algebra", "linear algebra unsolved" ], "Problem": "Let $ W$ and $ Y$ be subspaces of a finite dimensional vector space $ V$.\r\n\r\nProve that $ \\dim\\left(W\\plus{}Y\\right) \\equal{} \\dim W \\plus{} \\dim Y \\minus{} \\dim\\left(W \\cap Y\\right)$.", "Solution_1": "[quote=\"Cloak\"]Let W and Y be subspaces of a finite dimensional vector space V.\n\nProve that dim(W+Y) = dimW + dimY - dim(W intersect Y)[/quote]\r\nPick a basis $ B_0$ of $ Z \\equal{} W \\cap V$.\r\n\r\nAdd basis vectors until you get a basis $ B_1$ of $ W$ (Steinitz).\r\n\r\nAdd basis vectors to $ B_0$ until you get a basis $ B_2$ of $ V$ (Steinitz).\r\n\r\nCheck that $ B_1 \\cup B_2$ is a basis of $ V\\plus{}W$.\r\n\r\nCount.", "Solution_2": "[quote=\"hpe\"][quote=\"Cloak\"]Let W and Y be subspaces of a finite dimensional vector space V.\n\nProve that dim(W+Y) = dimW + dimY - dim(W intersect Y)[/quote]\nPick a basis $ B_0$ of $ Z \\equal{} W \\cap V$.\n\nAdd basis vectors until you get a basis $ B_1$ of $ W$ (Steinitz).\n\nAdd basis vectors to $ B_0$ until you get a basis $ B_2$ of $ V$ (Steinitz).\n\nCheck that $ B_1 \\cup B_2$ is a basis of $ V \\plus{} W$.\n\nCount.[/quote]\r\n\r\nHow do you establish linear independence to show it's a basis?", "Solution_3": "$ B_{0} \\equal{} \\{u_{1},\\ldots,u_{i}\\}$\r\n$ B_{1} \\equal{} \\{u_{1},\\ldots,u_{i}, w_{1},\\ldots,w_{j}\\}$\r\n$ B_{2} \\equal{} \\{u_{1},\\ldots,u_{i}, v_{1},\\ldots,v_{k}\\}$\r\n\r\n$ \\alpha_{1}w_{1} \\plus{} \\ldots \\plus{} \\alpha_{j}w_{j} \\plus{} \\beta_{1}u_{1} \\plus{} \\ldots \\plus{} \\beta_{i}u_{i} \\plus{} \\gamma_{1}v_{1} \\plus{} \\ldots \\plus{} \\gamma_{k}v_{k} \\equal{} 0$\r\n\r\n$ \\Leftrightarrow \\alpha_{1}w_{1} \\plus{} \\ldots \\plus{} \\alpha_{j}w_{j} \\equal{} \\minus{} \\left(\\beta_{1}u_{1} \\plus{} \\ldots \\plus{} \\beta_{i}u_{i} \\plus{} \\gamma_{1}v_{1} \\plus{} \\ldots \\plus{} \\gamma_{k}v_{k}\\right)$\r\n\r\n$ \\Rightarrow \\alpha_{1}w_{1} \\plus{} \\ldots \\plus{} \\alpha_{j}w_{j}\\in W$ and $ \\alpha_{1}w_{1} \\plus{} \\ldots \\plus{} \\alpha_{j}w_{j}\\in V$\r\n\r\n$ \\Rightarrow \\alpha_{1}w_{1} \\plus{} \\ldots \\plus{} \\alpha_{j}w_{j}\\in W \\cap V$\r\n\r\n$ \\Rightarrow \\alpha_{1}w_{1} \\plus{} \\ldots \\plus{} \\alpha_{j}w_{j} \\equal{} \\beta^{\\prime}_{1}u_{1} \\plus{} \\ldots \\plus{} \\beta^{\\prime}_{i}u_{i}$ \r\n\r\nSince $ u_{1}, \\ldots, u_{i}, w_{1}, \\ldots, w_{j}$ are lineary independent, we get $ \\alpha_{1} \\equal{} \\alpha_{2} \\equal{} \\ldots \\equal{} \\alpha_{i} \\equal{} 0$. Now we can conclude also, that all $ \\beta$-s and $ \\gamma$-s are $ 0$.", "Solution_4": "[b]Lemma:[/b] The complement $ R$ of the subspace $ Q$ in the vector space $ P$ is a subspace, for which $ P$ is the direct sum of $ Q$ and $ R$; $ P\\equal{}Q \\oplus R$. The complement is not unique. In fact there are infinitely many of them for a non-trivial subspace $ Q$. But they are all isomorphic and $ \\dim P\\equal{}\\dim Q \\plus{} \\dim R$.\r\n[b]\nProblem:[/b]\r\nLet us denote some complements of $ W\\cap Y$ in $ W, Y$ and $ W\\plus{}Y$ by $ W^{\\prime}, Y^{\\prime}$ and $ (W\\plus{}Y)^{\\prime}$.\r\n\r\nSince $ (W\\cap Y) \\cap W^{\\prime}\\equal{} \\{0\\}, (W\\cap Y) \\cap Y^{\\prime}\\equal{}\\{0\\}$ and $ W^{\\prime} \\cap Y^{\\prime}\\equal{}\\{0\\}$, we can take $ (W\\plus{}Y)^{\\prime}\\equal{}W^{\\prime}\\oplus Y^{\\prime}$.\r\n\r\nHence $ W \\equal{} (W\\cap Y) \\oplus W^{\\prime}, Y \\equal{} (W\\cap Y) \\oplus Y^{\\prime}$ and $ W\\plus{}Y \\equal{} (W\\cap Y) \\oplus W^{\\prime} \\oplus Y^{\\prime}$.\r\n\r\nTherefore $ \\dim W \\equal{} \\dim (W\\cap Y) \\plus{} \\dim W^{\\prime}, \\dim Y \\equal{} \\dim (W\\cap Y) \\plus{} \\dim Y^{\\prime}$ and $ \\dim W\\plus{}Y \\equal{} \\dim (W\\cap Y) \\plus{} \\dim W^{\\prime} \\plus{} \\dim Y^{\\prime}$. \r\n\r\nIt remains to express $ \\dim W^{\\prime}, \\dim Y^{\\prime}$ from the first two equalities and put them into the third one.\r\nWe get $ \\dim(W\\cap Y) \\plus{} \\dim (W\\plus{}Y)\\equal{} \\dim W\\plus{}\\dim Y$, which was wanted." } { "Tag": [], "Problem": "[i]question deleted -- Admin[/i]", "Solution_1": "Look at it again, re-read it.", "Solution_2": "We cannot clarify the problems while the round is in progress. Please read the problems carefully." } { "Tag": [ "algebra proposed", "algebra" ], "Problem": "Prove that there are $ a_0 ,a_1 ,...,a_n \\in \\mathbb{Z}$ not all of them equal to $ 0$ ,$ |a_i| \\leq 9$ ,$ i\\equal{}\\overline{1,n}$ ,such that \r\n\r\n$ |a_0\\plus{}a_1\\sqrt{2}\\plus{}...\\plus{}a_n \\sqrt{n\\plus{}1}|<\\frac{1}{10^n\\minus{}1}$ \r\n\r\n[i]author :Lavinia Savu[/i]", "Solution_1": "Consider all sums $ S(b)\\equal{}S(b_0,b_1,...,b_n)\\equal{}\\sum_{k\\equal{}0}^nb_k\\sqrt{k\\plus{}1}$ with $ 0\\le b_i\\le 9$.\r\nWe had $ 10^{n\\plus{}1}$ means of sum and all of them $ 0\\le 9A, \\ A\\equal{}\\sum_{k\\equal{}1}^{n\\plus{}1}\\sqrt k$\r\nTherefore exist 2 means of nabors $ b^1\\equal{}(b_0^1,b_1^1,...,b_n^1), b^2\\equal{}(b_0^2,...,b_n^2)$ suth that $ 0= ^2?\r\n\r\n\r\n: /", "Solution_3": "You can express $ p$ as a linear combination of $ p^2$ and $ d$. In the other direction, it's easy to show that the squares are divisible by $ p$.", "Solution_4": "oh ok. I did that.\r\n\r\nI guess I need to show that is a prime.\r\n\r\nif O/ is a domain then the ideal is prime. this is the part I'm having trouble with.", "Solution_5": "There's no need to bring domains into this; just use the definition of a prime ideal directly. Multiply two elements and ask under what conditions their product can lie in this ideal.", "Solution_6": "It may be useful to consider the norm $ N(a\\plus{}b\\sqrt{d})\\equal{}a^2\\minus{}b^2d$.", "Solution_7": "Ok. I think i got it.\r\n\r\nSo, if we have two elemens, r and s.\r\n\r\nr= a+b* (sqrt(d)) and s=(c+e*sqrt(d))\r\n\r\nthen rs= (ac+bed) + (ae+bc) (sqrt(d))\r\n\r\nand for this to be in the ideal generated by p and sqrt(d), then p|(ac+bed) and p| (ae+bc)\r\n\r\nsince p|d, the p|a or p|c. \r\n\r\nIf p|a then on the left side p|b so that r is factor. And it works the same way for the other side.\r\n\r\ncorrect?", "Solution_8": "I don't know what \"r is factor\" means. Do you mean to say $ r\\in \\langle p, \\sqrt{d}\\rangle$? These distinctions are super-duper important.", "Solution_9": "sry. yes that's what i mean." } { "Tag": [ "ratio", "algebra", "polynomial", "induction", "blogs", "vector", "function" ], "Problem": "Prove Binet's Formula. (i.e. Find the formula for the Fibonacci Numbers)", "Solution_1": "What do you mean the formula for the Fibonacci numbers? The golden ratio between them?", "Solution_2": "Prove that $ F_n \\equal{} \\frac1{\\sqrt5}\\left(\\left(\\frac {1 \\plus{} \\sqrt5}2\\right)^n \\minus{} \\left(\\frac {1 \\minus{} \\sqrt5}2\\right)^n\\right)$, for all integers $ n\\in\\mathbb{Z}^ \\plus{}$.", "Solution_3": "Is this supposed to be related to the [url=http://www.artofproblemsolving.com/Wiki/index.php/Binet%27s_formula]WoW?[/url] [i]Edit[/i]: an explanation (by me, lol) similar to nateharman1234's is at that link \r\n\r\nSince $ F_{n \\plus{} 1} \\equal{} F_{n} \\plus{} F_{n \\minus{} 1}$, we have a linear recurrence with characteristic polynomial $ x^2 \\minus{} x \\minus{} 1 \\equal{} 0 \\equal{} \\left(x \\minus{} \\frac {1 \\plus{} \\sqrt {5}}{2}\\right)\\left(x \\minus{} \\frac {1 \\minus{} \\sqrt {5}}{2}\\right)$. We know that $ F_0 \\equal{} 0$, $ F_1 \\equal{} 1$, and by the characteristic we have $ F_n \\equal{} c_1 \\left(\\frac {1 \\plus{} \\sqrt {5}}{2}\\right)^n \\plus{} c_2 \\left(\\frac {1 \\minus{} \\sqrt {5}}{2}\\right)^n$. So now we have a system of two equations (if we let $ n \\equal{} 0,1$), $ 0 \\equal{} c_1 \\plus{} c_2 \\Longrightarrow c_2 \\equal{} \\minus{} c_1$ and $ 1 \\equal{} c_1\\left(\\frac {1 \\plus{} \\sqrt {5}}{2}\\right) \\plus{} c_2\\left(\\frac {1 \\minus{} \\sqrt {5}}{2}\\right)$ $ \\Longrightarrow 1 \\equal{} c_1\\left(\\frac {1 \\plus{} \\sqrt {5}}{2} \\minus{} \\frac {1 \\minus{} \\sqrt {5}}{2}\\right)$ $ \\Longrightarrow c_1 \\equal{} \\frac {1}{\\sqrt {5}}$.\r\n\r\nThus, $ F_n \\equal{} \\frac {1}{\\sqrt {5}}\\left(\\frac {1 \\plus{} \\sqrt {5}}{2}\\right)^n \\minus{} \\frac {1}{\\sqrt {5}}\\left(\\frac {1 \\minus{} \\sqrt {5}}{2}\\right)^n$.", "Solution_4": "Many people do the induction proof of this at some point in their life, and while it may be a good induction exercise, it gives no insight into why this works. Heres a proof I know, and for another you can read one of t0rajir0u's recent blog posts.\r\n\r\n[hide]\n\nConsider the set S of sequences of the form An= A(n-1)+A(n-2), A0= x and A1 = y for real x,y (real works here, but for different sequences you may need complex)\n\nNote that if a sequence An is in S and a sequence Bn is in S, then (An+Bn) is in S.\n\nAlso note that this addition of sequences is associative and commutative and all that nice stuff as real additon is\n\nOh and of course if k is a real number and An is in S than so is the sequence k*An, and this multiplication distributes over the sequence addition I talked about before. and note that 1*An=An\n\nOk then note the sequence An=0 is in S and for all An in S -An is in S too.\n\nWell if you haven't realized already I am trying to establish that S forms a vector space, I think thats just about everything if I forgot an axiom I appologize.\n\nAnyway, now that we have a nice vector space it is of course nice to find a basis for it. Well i hope it is painfully obvious that S is two dimensional.\n\nOk now lets see if there are any nicely behaved sequences in S that have a closed form, perhaps any geometric sequences...\n\nSolve X^n = X^(n-1)+X^(n-2), x =/= 0...\nOoh... two linearly independent geometric sequences, that would make a lovely basis, now to just express the fibbonacci numbers in terms of them...\n\nI'll stop there, because I don't feel like doing any of the actual computations.\n[/hide]", "Solution_5": "The proof nateharman is talking about can be found [url=http://www.artofproblemsolving.com/Forum/weblog_entry.php?t=175257]here[/url]. \r\n\r\n[hide=\"Another solution by generating functions\"] Let $ G(x) \\equal{} \\sum_{k\\equal{}0}^{\\infty} F_k x^k$, where $ F_0 \\equal{} 0, F_1 \\equal{} 1$. The Fibonacci relation implies (where $ \\phi, \\varphi$ are the positive and negative roots of $ x^2 \\equal{} x \\plus{} 1$): \n\n$ x G(x) \\plus{} x^2 G(x) \\equal{} G(x) \\minus{} x \\Leftrightarrow$\n$ G(x) \\equal{} \\frac{x}{1 \\minus{} x \\minus{} x^2} \\equal{} \\frac{x}{(1 \\minus{} \\phi x)(1 \\minus{} \\varphi x)} \\equal{} \\frac{1}{\\phi \\minus{} \\varphi} \\left( \\frac{1}{1 \\minus{} \\phi x} \\minus{} \\frac{1}{1 \\minus{} \\varphi x} \\right)$\n\n(The last step is partial fraction decomposition.) Using the expansion $ \\frac{1}{1 \\minus{} ax} \\equal{} 1 \\plus{} ax \\plus{} a^2 x^2 \\plus{} ...$, we conclude that\n\n$ \\sum_{k\\equal{}0}^{\\infty} F_k x^k \\equal{} \\frac{1}{\\phi \\minus{} \\varphi} \\left( \\sum_{k\\equal{}0}^{\\infty} (\\phi^k \\minus{} \\varphi^k) x^k \\right) \\Leftrightarrow$\n$ F_k \\equal{} \\frac{\\phi^k \\minus{} \\varphi^k}{\\phi \\minus{} \\varphi}$\n\nas desired (this is the nice form of Binet's formula). [/hide]", "Solution_6": "Let $ \\tau \\equal{} \\frac {(1\\plus{}\\sqrt{5})}{2}$ and $ \\sigma \\equal{} \\frac {1\\minus{}\\sqrt{5}}{2}$\r\n\r\nSince $ \\tau$ and $ \\sigma$ are the roots of $ x^2\\equal{}x\\plus{}1$ then $ \\tau^2\\equal{}\\tau\\plus{}1$ and similarly with $ \\sigma$.\r\n\r\nNow multiplying both by their corresponding n-powers, $ \\tau^{n\\plus{}2} \\equal{} \\tau^{n\\plus{}1}\\plus{}\\tau^n$, and $ \\sigma^{n\\plus{}2} \\equal{} \\sigma^{n\\plus{}1}\\plus{}\\sigma^n$\r\n\r\nSubtracting the first from the latter yeilds $ \\tau^{n\\plus{}2}\\minus{}\\sigma^{n\\plus{}2}\\equal{} \\tau^{n\\plus{}1}\\plus{}\\tau^n\\minus{}\\sigma^{n\\plus{}1}\\minus{}\\sigma^n$\r\n\r\nso $ (\\tau^{n\\plus{}2}\\minus{}\\sigma^{n\\plus{}2})\\equal{} (\\tau^{n\\plus{}1}\\minus{}\\sigma^{n\\plus{}1})\\plus{}(\\tau^n\\minus{}\\sigma^n)$.\r\n\r\nSince $ \\tau\\minus{}\\sigma \\equal{} \\sqrt{5}$, and $ \\tau^2\\minus{}\\sigma^2\\equal{}\\sqrt{5}$ we must divide by $ \\tau\\minus{}\\sigma$\r\n\r\nso $ \\frac {\\tau^{n\\plus{}2}\\minus{}\\sigma^{n\\plus{}2}}{\\tau\\minus{}\\sigma}\\equal{} \\frac{\\tau^{n\\plus{}1}\\minus{}\\sigma^{n\\plus{}1}}{\\tau\\minus{}\\sigma}\\plus{}\\frac{\\tau^n\\minus{}\\sigma^n}{\\tau\\minus{}\\sigma}$. \r\n\r\nIf you would like it in your form with the denomiator being the radical, \r\n\r\nso $ \\frac {\\tau^{n\\plus{}2}\\minus{}\\sigma^{n\\plus{}2}}{\\sqrt{5}}\\equal{} \\frac{\\tau^{n\\plus{}1}\\minus{}\\sigma^{n\\plus{}1}}{\\sqrt{5}}\\plus{}\\frac{\\tau^n\\minus{}\\sigma^n}{\\sqrt{5}}$. \r\n\r\nLet $ u_n \\equal{} \\frac{\\tau^n\\minus{}\\sigma^n}{\\tau\\minus{}\\sigma}$, then the sequence is found as $ u_{n\\plus{}2}\\equal{}u_{n\\plus{}1}\\plus{}u{n}$\r\n\r\nand this completes the proof." } { "Tag": [], "Problem": "when $ \\text{1,3,5,5\\minus{}tetramethyl cyclohexa\\minus{}1,3\\minus{}diene}$ is disssolved in cold conc. $ \\text{H}_2\\text{SO}_4$ then a lowering of Freezing point is observed that correspond to 2 particles for each molecule of diene.When the solution is treated with water the diene is regenerated.Describe what happens with relevant equations", "Solution_1": "The part \"that correspond to 2 particles for each molecule of diene\" means that for each molecule of diene that reacts with H2SO4, two species are formed. Well, the diene has double bonds and H2SO4 is an acid: what do you think that might happen?\r\n\r\n[hide=\"Answer\"]A salt of the form cyclohexadiene+HSO4- is formed.[/hide]\r\n\r\n\r\n[b]Question[/b]: what would happen if the 1,3,5,5-tetramethylcyclohexa-1,3-diene had a C=O bond in carbon 6 and were treated with sulphuric acid?", "Solution_2": "i got the salt but couldn't know how on adding water that get's converted to diene", "Solution_3": "In the salt formation reaction, the diene acts as a base, accepting an H+ ion from sulphuric acid. When the solution is treated with water, the diene acts as an acid with a molecule of water acting as a base, regenerating the diene. These are simply acid-base reactions." } { "Tag": [ "algebra", "polynomial", "induction", "algebra unsolved" ], "Problem": "Any polynomial $g(x)$ of degree d or less satisfies the recursion \r\n\r\n$\\sum_{i=0}^{d+1}(-1)^i\\binom{d+1}{i}g(x+i)=0$\r\n\r\nCan someone explain this to me?", "Solution_1": "It sufficies to prove the identity for $g(x)=x^k$, $k\\leq d$.\r\nWe will use an induction.\r\n1. For $d=1$ the assertion is true.\r\n2. We have \\[\\sum_{i=0}^d(-1)^i\\binom{d+1}{i}(x+i)^k=\\sum_{i=0}^d(-1)^i\\binom{d+1}{i}(x+i)^{k-1}(x+i)=\\] \\[=x\\cdot \\sum_{i=0}^d(-1)^i\\binom{d+1}{i}(x+i)^{k-1}+\\sum_{i=0}^d(-1)^ii\\binom{d+1}{i}(x+i)^{k-1}.\\]\r\nWe know that $\\sum_{i=0}^d(-1)^i\\binom{d+1}{i}(x+i)^{k-1}=0$ using induction proposition for the pair $(d,k-1)$. Moreover, \\[\\sum_{i=0}^d(-1)^ii\\binom{d+1}{i}(x+i)^{k-1}=\\sum_{i=0}^{d-1}(d+1)(-1)^i\\binom{d}{i}((x+1)+i)^{k-1}=0\\] using induction proposition for pair $(d-1,k-1)$.", "Solution_2": "If you're inducting on d how can you assume the pair (d,k-1) is true?", "Solution_3": "Or like this... For polynomial $g(x)$ let $\\Delta g(x)=g(x+1)-g(x)$. Observe that $\\Delta g(x)$ is polynomial which degree is smaller than degree of $g(x)$. Also let $\\Delta_2 g(x)=\\Delta(\\Delta g(x))$ and $\\Delta_n g(x)=\\Delta (\\Delta_{n-1} g(x) )$. It is very easy to proove (by induction) that $\\Delta_n g(x)=\\sum_{i=1}^n (-1)^n {n \\choose i}g(x+i)$. We need to proove that $\\Delta_{d+1} g(x) =0$. But, this is now trivial, because $\\deg \\Delta g(x) \\leq d-1$, $\\deg \\Delta_2 g(x) \\leq d-2$, and so on, and $\\deg \\Delta_d g(x) =0$ so it is constant and that is why $\\Delta_{d+1} g(x) =0$", "Solution_4": "How do you prove $\\Delta_n g(x)=\\sum_{i=1}^n (-1)^n {n \\choose i}g(x+i)?$\r\n\r\nFor n=1 wouldn't you have $\\Delta g(x)=-g(x+1)?$", "Solution_5": "[quote=\"pilot\"]If you're inducting on d how can you assume the pair (d,k-1) is true?[/quote]\r\nSorry, I forgot to say that we use the double induction on pairs $(d,k)$ and for $k=0$ we get the well known equality $\\sum_{i=0}^{d+1}(-1)^i\\binom{d+1}{i}=0$.", "Solution_6": "I typed it wrong. It should be $\\Delta_n g(x)=\\sum_{i=0}^n (-1)^{n-i} {n \\choose i}g(x+i)$. For $n=1$ it becomes $\\Delta g(x) = g(x+1)-g(x)$, for $n=2$ it should be $\\Delta_2 g(x)=(g(x+2)-g(x+1))-(g(x+1)-g(x))=g(x+2)-2g(x+1)+g(x)$ and so on. If it's true for $n$ then $\\Delta_{n+1} g(x) = \\Delta_n g(x+1) - \\Delta_n g(x)=\\sum_{i=0}^n (-1)^{n-i} {n \\choose i}g(x+1+i) - \\sum_{i=0}^n (-1)^{n-i} {n \\choose i}g(x+i)=g(x+n+1)-g(x+n)({n \\choose n-1}+{n \\choose n})+g(x+n-1)({n \\choose n-2}+{n \\choose n-1})+\\cdots =\\sum_{i=0}^{n+1} (-1)^{n+1-i} {n+1 \\choose i}g(x+i)$.\r\n\r\nI hope I didn't make any mistake again..." } { "Tag": [ "quadratics", "Pythagorean Theorem", "geometry" ], "Problem": "Quadrilateral ABCD is inscribed in a circle with diameter AD=4. If sides AB and BC each have length 1, then find CD.", "Solution_1": "[hide=\"Solution\"]By Ptolemy's Theorem, $ AB \\cdot CD \\plus{} BC \\cdot AD \\equal{} CA \\cdot BD$, we get\n$ 4 \\plus{} CD \\equal{} CA \\cdot BD$.\nBecause $ AD$ is the diameter, we know that triangles $ ABD$ and $ DCA$ are right. We then use the Pythagorean Theorem to find the values of CA and BD.\n$ 4 \\plus{} CD \\equal{} \\sqrt {15} \\cdot \\sqrt {16 \\minus{} CD}$.\nThis is just a quadratic, so we solve and get $ CD \\equal{} \\frac {7}{2}$.[/hide]", "Solution_2": "[hide=\"solution\"][img]http://i.imgur.com/063bPwZ.png[/img]\nAs $AB=BC=1$, $ \\overarc{AB} = \\overarc{BC} \\Rightarrow \\angle{BDA}=\\angle{BDC}=\\frac{1}{2}\\angle{CDA}$\n$\\frac{CD}{AD}=\\cos(CDA)=\\cos(2BDA)=\\cos^{2}(BDA)-\\sin^{2}(BDA)=1-2\\sin^{2}(BDA)=1-2(\\frac{AB}{AD})^2$\n$\\Rightarrow CD=\\frac{7}{2}$[/hide]", "Solution_3": "[hide=Nice problem!] We use Pot Theorem to get AB*CD+BC*AD=CA*BD, therefore 4+CD=CA*BD. Because AD is diameter, we know triangles ABD and DCA are right. By Pythagorean Theorem, to find CA and BD. 4+CD=root(15)*root(16-CD). This is just a quadratic, so we solve and get CD=7/2. [/hide]\n\nOh no! Isn't this the same thing as sup3rcrash3r's solution? Oh well post count inflation is fun, so I won't delete.", "Solution_4": "It should be 4+CD=root(15)*root(16-CD^2)." } { "Tag": [ "email" ], "Problem": "can anyone give an exact number of how many AoPS users participated at nationals?\r\nThere seems like an awful lot.", "Solution_1": "[quote=\"236factorial\"]can anyone give an exact number of how many AoPS users participated at nationals?[/quote]No, I don't think anyone can give you an [b]exact[/b] number :)", "Solution_2": "AoPS is getting really big. There were a lot of AoPS shirts (I saw about 7), plus you shouldn't forget the people like me who don't have AoPS shirts. I would say there were a good 20 there. Then again, I really have no sense for how many there are. Maybe more, maybe less.", "Solution_3": "too many. Two off my team mates are here on aops and so is (was) my coach. then one of them (Secret Asian) went around asking everyone for their usrn.", "Solution_4": "Hey! Not everyone....er....ok yes everyone. I just wanted to see who was who. It was mathgeek who decided to go around and also tell people his username.", "Solution_5": "mathgeek? All he ever do is jump around in the halls and hit you. And through that you say he found time to ask people for their usrn?? \r\n\r\nI said Secret Asian cause I remebered at that moment you asking kingof21penguins.", "Solution_6": "Ok, I asked: New Jersey(which one is treething/aopser), Entropywins(but he was wearing an aops shirt, hence me asking) the whole reason I started asking texas was because asdf4(something like that, he's from tenessee) asked [b]me[/b] if I was an aopser...I asked more people than that, but I can't remember all of them", "Solution_7": "lol, nice signature secret asian", "Solution_8": "thanks g-unit.", "Solution_9": "That is hilarious signatur", "Solution_10": "The only people I saw were Naga, Xantos. C. Guin, and Solafidefarms. :( :( :(", "Solution_11": "You didn't see any of the TX or CA teams?", "Solution_12": "Well yeah but I didnt know who was who.", "Solution_13": "Yeah, a lot of people were running up to me asking \"Are you Treething?\"\r\n\r\nI'M A CELEBRITY :P !", "Solution_14": "I saw jb05, xantos, xxreddevilxx, zepelinator, smiley, smartnerd666, treething, biffandoc, 09husbri, entropywins, Hexahedron, mathgeek2006, Secret Asian, Dnas(though I didn't know it was him), kchande, aznness, bob123, piggypi, cutiepi, nosoupforyou, falconwing64, shlomadapenguin, jaeminlee, natedawg, mdean09, topper(who was at Word Power nats too), rorrimimage, jasonho007, dengmi, MooGoesCow?, RC-7th, and seungsoo. Oh, yes, and rcv, naga, frost13, and rrusczyk for the adult AoPSersAt least those are the ones I know the usernames for. 35. I didn't meet asdf4 from Tennessee, or if I did, I didn't ask if he was a AoPSer. Would someone please PM me his real name?\r\n\r\nBilly", "Solution_15": "[quote=\"solafidefarms\"]I saw jb05, xantos, xxreddevilxx, zepelinator, smiley, smartnerd666, treething, biffandoc, 09husbri, entropywins, Hexahedron, mathgeek2006, Secret Asian, Dnas(though I didn't know it was him), kchande, aznness, bob123, piggypi, cutiepi, nosoupforyou, falconwing64, shlomadapenguin, jaeminlee, natedawg, mdean09, topper(who was at Word Power nats too), rorrimimage, jasonho007, dengmi, MooGoesCow?, RC-7th, and seungsoo. Oh, yes, and rcv, naga, frost13, and rrusczyk for the adult AoPSersAt least those are the ones I know the usernames for. 35. I didn't meet asdf4 from Tennessee, or if I did, I didn't ask if he was a AoPSer. Would someone please PM me his real name?\n\nBilly[/quote]\r\n\r\nHoly Monkey!!\r\n\r\nThat's even more than I saw. Can you email me that list of everyone's emails and AoPS usernames?", "Solution_16": "It was nice meeting people who I knew only by username before. Now I can see some faces while reading posts.", "Solution_17": "Ohh, yes, Monkey, and Kingof21 penguins. Did I forget anymore???", "Solution_18": "WOW 35 AoPSers including 3 adults\r\nI know mdean09 he's at my school. Doesn't post much though. (sry dean)\r\nLet's see...\r\n57*5=285\r\n285/38=7.5%!!!\r\nplus the undoubtable 10 poeple we missed or so,\r\nwe have about 10%!!!!!!!!!\r\nWoohoo!\r\n~1 out of 10 national people was an AoPSer!!!! :D :D :D :D", "Solution_19": "Billy - since you do have my e-mail, could I just request that you not give it out to anybody unless you know for a fact that they were at nationals? Thanks!\r\n\r\nAnd everybody else, if you want my e-mail and you fit the above prerequisite, Billy has it!", "Solution_20": "[quote=\"solafidefarms\"]I saw jb05, xantos, xxreddevilxx, zepelinator, smiley, smartnerd666, treething, biffandoc, 09husbri, entropywins, Hexahedron, mathgeek2006, Secret Asian, Dnas(though I didn't know it was him), kchande, aznness, bob123, piggypi, cutiepi, nosoupforyou, falconwing64, shlomadapenguin, jaeminlee, natedawg, mdean09, topper(who was at Word Power nats too), rorrimimage, jasonho007, dengmi, MooGoesCow?, RC-7th, and seungsoo. Oh, yes, and rcv, naga, frost13, and rrusczyk for the adult AoPSersAt least those are the ones I know the usernames for. 35. I didn't meet asdf4 from Tennessee, or if I did, I didn't ask if he was a AoPSer. Would someone please PM me his real name?\n\nBilly[/quote]\r\n\r\nYou saw me too, remember at Joe DUmars", "Solution_21": "OK OK, I forgot some people. ... Does that make 36 now?", "Solution_22": "I think you saw me... although I was being swarmed by people who wanted that extremely rare and valuable CT pin.", "Solution_23": "[quote=\"Treething\"]Yeah, a lot of people were running up to me asking \"Are you Treething?\"\n\nI'M A CELEBRITY :P ![/quote]\r\n\r\n...I just wanted to know. Basically my rules were if you were wearing an Aops shirt you were fair game...or if I knew where you were from. I think Asod and mathgeek told people who I was though. I wanted to stay \"Secret\"", "Solution_24": "hmmm. I've been thinking, and I don't think that I saw xxreddevilzxx, since he wasn't from Washington. hmmmm. Sorry!" } { "Tag": [], "Problem": "These question(s) are from [i]Gravity from the Ground up[/i] - I want to know if my answer(s) are in fact solutions.\r\n\r\nThe first question is: Suppose that at the $ n^{th}$ time-step $ t_n$, the vertical speed is $ v_n$ and the vertical distance above the ground is $ h_n$. Show that at the next time-step $ t_{n\\plus{}1}$ $ \\equal{}$ $ t_n$ $ \\plus{}$ $ \\Delta$$ t$, the vertical speed is $ v_{n\\plus{}1}$ $ \\equal{}$ $ v_n$ $ \\minus{}$ $ g$ $ \\Delta$ $ t$.\r\n\r\nMy answer:\r\n\r\n$ v_{n\\plus{}1}$ = $ v_n$ $ \\plus{}$ $ v_1$\r\n\r\n$ v_1 \\equal{} at$ \r\n\r\nIn this case $ a \\equal{} g$ (gravity)\r\n\r\nSo $ v_n$ + $ v_1$ $ \\equal{}$ $ v_n$ $ \\minus{}g$ $ \\Delta$ $ t$\r\n\r\n\r\nI'm not sure why, but my answer doesn't seem complete to me.\r\n\r\nI'll post the rest of the questions/my answers tomorrow :)", "Solution_1": "Seems logical... :maybe:" } { "Tag": [ "quadratics" ], "Problem": "If the base $8$ representation of a perfect square is $ab3c$, where $a\\ne 0$, then $c$ equals \n\n$\\textbf{(A) } 0\\qquad \\textbf{(B) }1 \\qquad \\textbf{(C) } 3\\qquad \\textbf{(D) } 4\\qquad \\textbf{(E) } \\text{not uniquely determined}$", "Solution_1": "[hide=\"NT Solution\"]Recall that a perfect square must be equivilant to $0$, $1$, or $4$ mod $8$. Thus, c=0,1, or 4.\n\n$c=0$ is impossible because we're left with $8(8^2a+8b+3)$, which cannot be a square because $8$ isn't a square.\n\nIf $c=4$, we are left with $4(8N +7)$, where N is $4\\cdot 8^2a+32b$. This is once again impossible, because numbers of the form $8N+7$ cannot be a perfect square. (8 isn't a quadratic residue mod 7)\n\nThus, the answer is $n=1$\n[/hide]", "Solution_2": "[quote=\"4everwise\"]If the base $8$ representation of a perfect square is $ab3c$, where $a\\ngeq 0$, then $c$ equals \n\n$\\text{(A)} 0\\qquad \\text{(B)}1 \\qquad \\text{(C)} 3\\qquad \\text{(D)} 4\\qquad \\text{(E)} \\text{not uniquely determined}$[/quote]\r\n\r\nyou might want to edit the \r\n\r\n$a0$, $\\$$a\\neeq 0$\\$$ to \r\n$a\\ne 0$, $\\$$a\\ne 0$\\$$ i did not notice this until i looked at the code\r\n\r\nanyway, your solution is good, i was thinking mod 4, but that is not as easy, but also 25 is 31 base 8 so 1, answers are never \"not determined\"..." } { "Tag": [ "puzzles" ], "Problem": "There are five friends. Josh says that his birth date is 31st dec 1878. Rick says that his birth date is 14th nov 1902, Derik says his birth date is 12th july 1898, Usha says her birth date is 29th feb 1900 and Mike says his birth date is 15th May 1904.In which year will be the sum of their ages is equal to a prime number between 175 and 240.", "Solution_1": "Considering that this is probably homework, I'll just give you a hint.\r\n\r\n[hide]Start from the beginning of 1905 and note that each year the sum of the ages increase by 5.\n\nBtw, I think there might be more than one solution, but I'll have to check that. [/hide]", "Solution_2": "Never; February 29 1900 did not happen because 1900 is not a leap year.", "Solution_3": "[quote=\"1=2\"]Never; February 29 1900 did not happen because 1900 is not a leap year.[/quote]\r\n\r\nUhh...... yes it is. Any year divisible by 4 is a leap year...", "Solution_4": "Except for those divisible with 100 and not divisible with 400.\r\n\r\n[quote=\"Wikipedia\"]However, some exceptions to this rule are required since the duration of a solar year is slightly less than 365.25 days. Years that are evenly divisible by 100 are not leap years, unless they are also evenly divisible by 400, in which case they are leap years.[1][2] For example, 1600 and 2000 were leap years, but 1700, 1800 and 1900 were not. Similarly, 2100, 2200, 2300, 2500, 2600, 2700, 2900, and 3000 will not be leap years, but 2400 and 2800 will be. By this rule, the average number of days per year will be 365 + 1/4 \u2212 1/100 + 1/400 = 365.2425, which is 365 days, 5 hours, 49 minutes, and 12 seconds.[/quote]", "Solution_5": "[quote=\"hsiljak\"]Except for those divisible with 100 and not divisible with 400.\n\n[quote=\"Wikipedia\"]However, some exceptions to this rule are required since the duration of a solar year is slightly less than 365.25 days. Years that are evenly divisible by 100 are not leap years, unless they are also evenly divisible by 400, in which case they are leap years.[1][2] For example, 1600 and 2000 were leap years, but 1700, 1800 and 1900 were not. Similarly, 2100, 2200, 2300, 2500, 2600, 2700, 2900, and 3000 will not be leap years, but 2400 and 2800 will be. By this rule, the average number of days per year will be 365 + 1/4 \u2212 1/100 + 1/400 = 365.2425, which is 365 days, 5 hours, 49 minutes, and 12 seconds.[/quote][/quote]\r\n\r\nOh, yes. You are right. I've never heard that before.... I wonder why....", "Solution_6": "There are four solutions that I found, 1931, 1935, 1941, and 1943" } { "Tag": [ "trigonometry", "complex analysis", "function", "logarithms", "real analysis", "real analysis unsolved" ], "Problem": "Let $ \\bold{f}: \\mathbb{R}^2\\to\\mathbb{R}^2$ be defined by $ \\bold{f}(x,y)\\equal{}(e^x\\cos y, e^x\\sin y)$\r\nShow that $ \\bold{f}$ is injective on the strip $ \\{(x,y): \\minus{}\\pi=0.\r\n\r\nThe m,n,z are positive reals.\r\n\r\nWho can do?\r\n\r\nBQ", "Solution_1": "But\r\n$ \\sqrt {r_aw_a}(\\frac {1}{w_b} \\plus{} \\frac {1}{w_c}) \\geq 2$\r\nis easy.\r\nBQ", "Solution_2": "[quote=\"xzlbq\"]prove nice triangle Inequality:\n\n$ \\sqrt {w_bw_c}(\\frac {1}{r_a} \\plus{} \\frac {1}{w_a}) \\geq 2 \\minus{} \\minus{} \\minus{} \\minus{} \\minus{} \\minus{} \\minus{} \\minus{} \\minus{} \\minus{} \\minus{} \\minus{} \\minus{} \\minus{} \\minus{} (*)$\n\nNeed to prove:\n\n(65536*n^2+65536*m^2-65536*m*n)*z^12+(-98304*m^2*n+573440*m^3+98304*m*n^2+212992*n^3)*z^11+(2262016*m^4-47104*m^2*n^2+487424*m*n^3+1388544*m^3*n+279552*n^4)*z^10+(7234560*m^4*n+189440*n^5+435200*m*n^4+5309440*m^5+1863680*m^3*n^2-163840*m^2*n^3)*z^9+(8241408*m^6+17186304*m^5*n+69888*n^6+188416*m^3*n^3-1061120*m^2*n^4+44544*m*n^5+10274560*m^4*n^2)*z^8+(24886272*m^6*n-1989632*m^3*n^4+13312*n^7+8889344*m^7+23788544*m^5*n^2-1336320*m^2*n^5+6988800*m^4*n^3-119808*m*n^6)*z^7+(-677888*m^2*n^6+32251904*m^6*n^2+1024*n^8-1501184*m^3*n^5+3236864*m^4*n^4+24075264*m^7*n-66560*m*n^7+19113984*m^5*n^3+6810624*m^8)*z^6+512*m*(m+n)*(-27*n^7-249*m*n^6-59*m^2*n^5+5060*m^3*n^4+19379*m^4*n^3+31525*m^5*n^2+24303*m^6*n+7266*m^7)*z^5+128*m*(11180*m^7+37212*m^6*n+49695*m^5*n^2+33230*m^4*n^3+11088*m^3*n^4+1490*m^2*n^5-5*m*n^6-8*n^7)*(m+n)^2*z^4+256*m^2*(2*m+n)*(11*n^5+230*m*n^4+1264*m^2*n^3+2660*m^3*n^2+2335*m^4*n+734*m^5)*(m+n)^3*z^3+64*m^2*(248*m^4+944*m^3*n+663*m^2*n^2+116*m*n^3+4*n^4)*(2*m+n)^2*(m+n)^4*z^2+64*m^3*(6*n^3+59*m*n^2+85*m^2*n+12*m^3)*(2*m+n)^3*(m+n)^4*z+16*m^4*(m^2+14*m*n+9*n^2)*(m+n)^4*(2*m+n)^4>=0.\n\nThe m,n,z are positive reals.\n\nWho can do?\n\nBQ[/quote]\r\n\r\n\r\n\r\ncollect(%,m);\r\n256*m^14\r\n+(6144*z+5120*n)*m^13\r\n+(63488*z^2+77312*n*z+27776*n^2)*m^12\r\n+(375808*z^3+347904*n^2*z+75136*n^3+559104*n*z^2)*m^11\r\n+(2028800*n^2*z^2+1431040*z^4+2510848*n*z^3+823168*n^3*z+121488*n^4)*m^10+(4053760*n^3*z^2+3720192*z^5+7237376*n^2*z^3+126624*n^5+1177536*n^4*z+7625216*n*z^4)*m^9\r\n+(6810624*z^6+11733248*n^3*z^3+4960192*n^4*z^2+16163328*n*z^5+17318272*n^2*z^4+87536*n^6+1080768*n^5*z)*m^8+\r\n(3874304*n^5*z^2+24075264*n*z^6+11688192*n^4*z^3+40000*n^7+8889344*z^7+648192*n^6*z+21738496*n^3*z^4+28583936*n^2*z^5)*m^7+\r\n(250624*n^7*z+32251904*n^2*z^6+7332608*n^5*z^3+1939584*n^6*z^2+24886272*n*z^7+26062848*n^3*z^5+16287104*n^4*z^4+11632*n^8+8241408*z^8)*m^6+(5309440*z^9+12512768*n^4*z^5+19113984*n^3*z^6+23788544*n^2*z^7+2848512*n^6*z^3+17186304*n*z^8+7282688*n^5*z^4+1952*n^9+604160*n^7*z^2+59328*n^8*z)*m^5+(7234560*n*z^9+1800064*n^6*z^4+2262016*z^10+2560512*n^5*z^5+6988800*n^3*z^7+108480*n^8*z^2+3236864*n^4*z^6+10274560*n^2*z^8+144*n^10+643328*n^7*z^3+7616*n^9*z)*m^4\r\n+(-1989632*n^4*z^7+1863680*n^2*z^9+9472*n^9*z^2+1388544*n*z^10+573440*z^11+188416*n^7*z^4+384*n^10*z-1501184*n^5*z^6-157696*n^6*z^5+72960*n^8*z^3+188416*n^3*z^8)*m^3\r\n+(-163840*n^3*z^9-1336320*n^5*z^7-98304*n*z^11+256*n^10*z^2+65536*z^12-47104*n^2*z^10-677888*n^6*z^6-2688*n^8*z^4-141312*n^7*z^5+2816*n^9*z^3-1061120*n^4*z^8)*m^2\r\n+(-13824*n^8*z^5+44544*n^5*z^8-65536*n*z^12-119808*n^6*z^7-1024*n^9*z^4+98304*n^2*z^11+435200*n^4*z^9+487424*n^3*z^10-66560*n^7*z^6)*m\r\n+65536*n^2*z^12+69888*n^6*z^8+212992*n^3*z^11+189440*n^5*z^9+13312*n^7*z^7+279552*n^4*z^10+1024*n^8*z^6\r\n\r\n\r\n\r\nwo only to prove \r\n\r\n\r\n(7234560*n*z^9+1800064*n^6*z^4+2262016*z^10+2560512*n^5*z^5+6988800*n^3*z^7+108480*n^8*z^2+3236864*n^4*z^6+10274560*n^2*z^8+144*n^10+643328*n^7*z^3+7616*n^9*z)*m^4\r\n+(-1989632*n^4*z^7+1863680*n^2*z^9+9472*n^9*z^2+1388544*n*z^10+573440*z^11+188416*n^7*z^4+384*n^10*z-1501184*n^5*z^6-157696*n^6*z^5+72960*n^8*z^3+188416*n^3*z^8)*m^3\r\n+(-163840*n^3*z^9-1336320*n^5*z^7-98304*n*z^11+256*n^10*z^2+65536*z^12-47104*n^2*z^10-677888*n^6*z^6-2688*n^8*z^4-141312*n^7*z^5+2816*n^9*z^3-1061120*n^4*z^8)*m^2\r\n+(-13824*n^8*z^5+44544*n^5*z^8-65536*n*z^12-119808*n^6*z^7-1024*n^9*z^4+98304*n^2*z^11+435200*n^4*z^9+487424*n^3*z^10-66560*n^7*z^6)*m\r\n+65536*n^2*z^12+69888*n^6*z^8+212992*n^3*z^11+189440*n^5*z^9+13312*n^7*z^7+279552*n^4*z^10+1024*n^8*z^6>=0" } { "Tag": [ "articles", "geometry" ], "Problem": "This isn't math-related problem I supposed. Anyway, imagine you have tons of mails with your addresses, sin#, all sorts of stuffs that you don't want other people to know. Obviously, you don't want to just dump them in a recycling bin due to the possibility of identity theft. And buying a shredder is expensive.\r\n\r\nI was thinking about soaking all those mails into water in order to make them easier to destroy. Maybe I could add some colourful ink to mess up all the writing in the mails. Unfortunately I haven't taken any chemistry, so I don't know which ink to use. \r\n\r\nI hope somebody could give me a suggestion. Hopefully I am posting in the right forum.", "Solution_1": "You are posting them in the right forum, or that's what I think, and your idea of adding ink seemed good.", "Solution_2": "Eat them. But seriously, you could just burn it.", "Solution_3": "there was an article once, in the Atlantic Monthly I think on the topic of destroying information and failing to do so(not really related to what you want) that was amusing and may also have had information on how to properly dispose of stuff if anyone remembers the article maybe they could describe it better.", "Solution_4": "I would suggest tearing them to pieces with your teeth. Not only will this mimic the effect of a conventional shredder, your saliva will both partially disintegrate the paper and smudge the ink.", "Solution_5": "since people are describing the best ways to destroy the documents i recommend feeding them to a sibling or letting (significantly) younger siblings shred them and then feed them to (older more annoying) sibling. This may be biased by the fact that i do not like the taste of paper maybe others do and want to eat themselves. alternatively if you dont have siblings you could cover the paper in birdseed you can let a flock of pigeons or crows do the dirty work.", "Solution_6": "is a regular shreder not the best option?", "Solution_7": "Umm....why spend money to buy a shredder when SCISSORS work just as well....or you can flush it down the toilet.", "Solution_8": "[quote=\"dogseatcheese\"]Umm....why spend money to buy a shredder when SCISSORS work just as well....or you can flush it down the toilet.[/quote]\r\nare you crazy?!?!\r\n1) do you know how long it would take to cut up a bunch of sheets of paper? \r\n2) you can get a shreder for like 20 dollars\r\n3) flush it? do you want to back up your pipes? or maybe that was after you have cut it up into small peices w/ your scissors.", "Solution_9": "Matches are cheap. I got 1,500 of them at Wal-Mart for $\\$$3. They lasted me almost two years!", "Solution_10": "Burning them would have to happen outside, and that can get nasty if it's a lot of paper\r\n\r\nIn Belgium you can't burn stuff less than hundred meters (about thirty feet) from houses.", "Solution_11": "I would suggest buying a good shredder. A shredder is an INVESTMENT. You'll be sorry after your identity gets stolen.", "Solution_12": "do you own a hamster?", "Solution_13": "Only sure way is to hurl it into a black hole. All information will be lost.", "Solution_14": "make information soup. ya know, make soup with it. >_>", "Solution_15": "hamsters are extremely handy if you spilled food, just let them walk over it and it's gone!\r\nbut i think it's really bad to feed paper to a hamster :(", "Solution_16": "i was thinking of lining the cage, not feeding paper to it.", "Solution_17": "tear it apart with your hands.\r\n\r\nJust fold it up and tear, repeat step until disposes of properly. If you are that worried, burning it is the best way to go.", "Solution_18": "[quote=\"Gyan\"]Only sure way is to hurl it into a black hole. All information will be lost.[/quote]\r\n\r\nDidn't Stephen Hawking just lose a bet (a set of encyclopaedias on baseball, I believe) because he decided that information is actually not lost?", "Solution_19": "[quote]Didn't Stephen Hawking just lose a bet (a set of encyclopaedias on baseball, I believe) because he decided that information is actually not lost?[/quote]\r\nYes, that is true. (to be honest, I was almost sure that some would point this out, when I posted the message .. I was just curious, how long willl it take :)\r\n\r\nAlso in 1978, before US embassy was taken over , embassy employees were able to shred most of the secret documents (with professional shredders) , but Iranians were able to put most of the documents back.. the information leaked thus, according to reports, was very significant.", "Solution_20": "What you do, is you get a professional shredder. Then you take the shredded paper and use it to line the cage of your rabbit/hamster/guinea pig. See? Then it's professionally nastied!", "Solution_21": "Well...I it isn't a lot of items....the SCISSORS. if a lot....SHREDDER. if some....HAMSTER", "Solution_22": "Their shredders cut the documents into 1/4 inch strips. For greater security use the cross cut type or the type that cuts the paper into strips that are about as wide as they are long.\r\n\r\nDoes anyone else remember that one of the soldiers helped the Iranians to piece together the documents, as well as telling them which documents were most important then sued when he didn't get a medal, like the other soldiers.", "Solution_23": "You can shred the papers split them up into 4 piles and travel to the four corners of the earth and hide them that should be good enough. :)", "Solution_24": "Fast - run over important information with a permanent marker or whiteboard marker. If you really want to be secure, run over the corresponding area on the other side of the paper too. Noone will see.", "Solution_25": "first of all, thank you all for the suggestions.\r\n\r\nIt looks like for paper, shredder would be the best option since it is fast (I'm not gonna spend a whole day tearing paper), cheap (hamster and whiteboard pen costs a fortune), and clean (burning is dirty + toxic too). \r\n\r\nThough I admit, I am quite phobic towards shredder after reading about some disturbing accidents in the past, which I would rather not talk about it.\r\n\r\nEven though I am really annoyed by the birds' regular fly-by, I don't want to feed those poor things with paper... yet.", "Solution_26": "Just cremate all the papers in your fireplace. :D", "Solution_27": "[quote=\"cheerful coffin\"]first of all, thank you all for the suggestions.\n\nIt looks like for paper, shredder would be the best option since it is fast (I'm not gonna spend a whole day tearing paper), cheap (hamster and whiteboard pen costs a fortune), and clean (burning is dirty + toxic too). \n\nThough I admit, I am quite phobic towards shredder after reading about some disturbing accidents in the past, which I would rather not talk about it.\n\nEven though I am really annoyed by the birds' regular fly-by, I don't want to feed those poor things with paper... yet.[/quote]\r\nEh? how would a whiteboard marker cost more than a shredder? Personally, I think burning is the best idea. Do it in the sink. And have a pail of water on standby.", "Solution_28": "Thermite ;)\r\n\r\nBut that would count as burning, wouldn't it....", "Solution_29": "[quote=\"nr1337\"]Thermite ;)\n\nBut that would count as burning, wouldn't it....[/quote]\r\n\r\nI made thermite thats stuff is pretty scary (but really cool) but it would be effective.", "Solution_30": "Hmmm...If it's not too many pages, burning it seems best. Use a bunsen burner...advancement of science...", "Solution_31": "[quote=\"dogseatcheese\"]Umm....why spend money to buy a shredder when SCISSORS work just as well....or you can flush it down the toilet.[/quote]\r\n\r\nBecause a shredder is so much fun!" } { "Tag": [ "topology", "function" ], "Problem": "I need some help regarding the following questions; I have a solution to each of these but there's something missing to each of them:\r\n\r\n1) If X1xX2 is a topological space and t is in X1, then show that (t)xX2 considered with the subspace topology induced from the product topology on X1xX2 is homeomorphic to X2:\r\n\r\nThe subspace topology makes the mapping i:{t}\u00d7X_2\u2192X_1\u00d7X_2 continuous, and the product topology is the smallest topology that makes the mapping \u03c0:X_1\u00d7X_2\u2192X_2 continuous, so their composition mapping \u03c0\u2219i:{t}\u00d7X_2\u2192X_2 is also continuous. I think I just need to prove their inverse is also continuous, which I'm trying to figure out how to.\r\n\r\n2) Prove X is connected iff the only continuous functions f:X\u2192Z are constant.\r\n\r\nI can prove the 'only if' bit. With the 'if,' I tried contradiction by supposing X is not connected and so can be split into 2 non-trivial closed and open disjoint sets, but I just can't prove they cannot be non-trivial in the end.\r\n\r\nCan someone please help me with these questions and tell me if I'm going down the right direction?\r\nThanks", "Solution_1": "1. let $ f : X\\to X\\times Y$ st $ f(x) \\equal{} (x,y_{0})$ then $ f$ is continuous \r\nthen clearly $ f(X) \\equal{} X\\times y_{0}$, $ f$ is 1-1 and onto $ X\\times y_{0}$, \r\nto prove that $ g \\equal{} f^{ \\minus{} 1}$ is continuous, take $ O$ to be open in $ X$ then $ g^{ \\minus{} 1}(O) \\equal{} O\\times y_{0} \\equal{} (O\\times Y)\\cap (X\\times y_{0})$ which is open in $ X\\times y_{0}$\r\n\r\n2. $ f : X\\to \\mathbb{Z}$ is continuous then $ f(X)$ is connected in $ \\mathbb{Z}$, and it is known that connected components of $ \\mathbb{Z}$ is just one-point sets , so $ f$ must be constant . \r\n\r\nnow consider the other direction, $ X \\equal{} A\\cup B$ where $ A, B$ are non-empty [b]disjoint[/b] subset of $ X$ \r\ndefine $ f : X \\equal{} A\\cup B \\to \\mathbb{Z}$ , with $ f(A) \\equal{} \\{0\\}, f(B) \\equal{} \\{1\\}$ which are open in $ \\mathbb{Z}$ then clearly $ f$ is continuous, not constant." } { "Tag": [ "USAMTS", "LaTeX", "geometry" ], "Problem": "Does anyone know how to make an image that looks nice (isn't pixilated) for a wide range of zoom levels? (Like 100%, 125%, 150%.)\r\n\r\nWhenever I use an image in one of my PDF documents, it looks great at 100%. However, it becomes ugly as soon as you get past 115%.\r\n\r\nIf you're not sure what I'm talking about, try zooming in a lot on this document. The images still look really nice.\r\n[url=http://www.usamts.org/Tests/USAMTSProblems_18_1.pdf]USAMTS Round 1 Problems[/url]", "Solution_1": "But this document was created using LaTeX, so images are .eps, that's why they looks nice... This is a type of images called [b]Vectorial Images[/b].", "Solution_2": "Correct -- the images in the USAMTS problem document that you cite are EPS (Encapsulated PostScript) files created using [url=http://www.tug.org/metapost.html]MetaPost[/url]. So they will look sharp at any resolution.", "Solution_3": "what is the best program (but not really complicated and involved) to use to make image files and geometry renderings to include in latex files?\r\n\r\nis that program MetaPost useful or is it too complicated to learn just for the sake of making a few good drawings?", "Solution_4": "Hmmm...Just a word of warning...if you build files with pictures in JPG, it seems the pictures get progressively worse. I used paint on this one picture...and saved it in JPG (actually...maybe it's JPEG...uh...no sure what's right...) and after building my latex document 4 times, it was like someone sprinkled pepper on the image...I guess that's the best way I can describe it. Has anyone else had this problem?", "Solution_5": "I use a french software called [url=http://perso.orange.fr/Fradin.Patrick/TeXgraph/Telecharger.html]TeXgraph[/url], but there is other soft like gnplot\r\nhttp://www.mathlinks.ro/Forum/viewtopic.php?t=111694", "Solution_6": "[quote=\"DPatrick\"]Correct -- the images in the USAMTS problem document that you cite are EPS (Encapsulated PostScript) files created using [url=http://www.tug.org/metapost.html]MetaPost[/url]. So they will look sharp at any resolution.[/quote]\r\nHow can I obtain MetaPost?\r\nI've visited the link you've given and downloaded the file, but my computer can't seem to open a .tar file. Is there any program I can obtain that can do this, or is there another way I can obtain the program?\r\n(Also, I have Windows XP, in case you were wondering.)\r\nThanks!", "Solution_7": "[url=http://www.winzip.com/]WinZip[/url], [url=http://www.7-Zip.org/]7-zip[/url], [url=http://www.zipgenius.it/eng/index.php]ZipGenius[/url] and other compression programs should be able to extract from a tar file. They are also a lot faster at zipping & unzipping than XP!" } { "Tag": [ "search", "inequalities proposed", "inequalities" ], "Problem": "Prove that the following ineq holds for all positive reals $ x,y,z$\r\n\r\n$ \\frac{x}{\\sqrt{x\\plus{}y}}\\plus{}\\frac{y}{\\sqrt{y\\plus{}z}}\\plus{}\\frac{z}{\\sqrt{z\\plus{}x}}\\leq \\frac{5}{4} \\sqrt{x\\plus{}y\\plus{}z}$", "Solution_1": "Already posted. See [url]http://www.mathlinks.ro/viewtopic.php?t=152848[/url].", "Solution_2": "[quote=\"FOURRIER\"]Prove that the following ineq holds for all positive reals $ x,y,z$\n\n$ \\frac {x}{\\sqrt {x \\plus{} y}} \\plus{} \\frac {y}{\\sqrt {y \\plus{} z}} \\plus{} \\frac {z}{\\sqrt {z \\plus{} x}}\\leq \\frac {5}{4} \\sqrt {x \\plus{} y \\plus{} z}$[/quote]\r\nSee at http://www.mathlinks.ro/viewtopic.php?search_id=1688548402&t=1459" } { "Tag": [ "linear algebra", "matrix", "inequalities", "combinatorics proposed", "combinatorics" ], "Problem": "One tornament have n players. Each player play with other exactly 1 time.\r\nFor each 2 players A,B exists exactly t players that loses to A and B.\r\nProve that n = 4t + 3.\r\n\r\nPlease, give me a solution for this problem. :)", "Solution_1": "[quote=\"Lopes\"]\nFor each 2 players A,B exists exactly t times that loses to A and B.\n[/quote]\r\n\r\nDo you mean $t$ players instead of $t$ times? :?", "Solution_2": "[quote=\"grobber\"][quote=\"Lopes\"]\nFor each 2 players A,B exists exactly t times that loses to A and B.\n[/quote]\n\nDo you mean $t$ players instead of $t$ times? :?[/quote]\r\n\r\nsorry,sorry.. \r\nsee again and help me, please.", "Solution_3": "Please, give me a solution :(", "Solution_4": "This problem is really hard? :( \r\nPlease, help me :(", "Solution_5": "Hmm...I don't think that to hurry people is the best way to receive an answer...\r\n\r\nPierre.", "Solution_6": "I think we should be able to prove, that every team won the same number of games and therefore lost the same number of games, so we would have, every team won $(n-1)/2$ games. So counting the number of trios $(a,b,c)$ such that $a$ and $c$ defeated $b$, we would have $\\binom{n}{2}t=n\\binom{\\frac{n-1}{2}}{2}$ from where we would obtain $n=4t+3$. However, i cant prove anything about the regularity of the degrees, so i guess I will have to wait until someone post a Linear Algebra solutions, becuase it is the only that seems to work, and it seems because it can be traslated into nice matrix condition I guess becuase i dont now LA at all.", "Solution_7": "ok..a thanks :) \r\nwhen you solve post your solution, please :)", "Solution_8": "ok here is one inequality:\r\n count the number of triples $(a,b,c)$ such that $a \\rightarrow c,b \\rightarrow c$.choosing $c$ first the number of such triples equals $\\displaystyle \\sum_{v \\in V} (d^-(v))^2$(where $d^-(v)$ denotes the indegree).on the other hand, we can choose $a$ and $b$ first.IF $a=b$ then the number of such triples is $\\displaystyle \\sum_{v \\in V} d^-(v)=n(n-1)/2$.if $a \\neq b$, the number of triples is $n(n-1)t$,so we have \r\n $\\displaystyle \\sum_{v \\in V} (d^-(v))^2=n(n-1)t+n(n-1)/2$. now \r\n $\\displaystyle \\frac{\\sum_{v \\in V} (d^-(v))^2}{n}\\geq (\\frac{\\sum_{v \\in V} d^-(v)}{n})^2$,so we have $(n-1)t+(n-1)/2 \\geq (\\frac{n-1}{2})^2 \\Rightarrow t+1/2\\geq (n-1)/4 \\\\ \\Rightarrow 4t+3 \\geq n$.\r\nNow if there is equality attained then there must be equality in the inequality above.which comes to saying what Pascual2005 said.", "Solution_9": "Maybe I am missing something but if the equality attaines then we are done :?", "Solution_10": "This solution is not complete..\r\nplease, give me a complete solution.", "Solution_11": "I have got a weak lower bound :(\r\nIndeed, $\\frac{n(n-1)(n-2)}{6}\\geq \\frac{n(n-1)}{2}\\cdot t$, so $n\\geq 3t+2$.", "Solution_12": "Please, help me :(", "Solution_13": "ok here is the remaining part of the solution:\r\n firstly we already have $n \\leq 4t+3$.now we'll prove that for every vertex $v,d^+(v) \\geq 2t+1$;then summing over all the vertices we get $n(n-1)/2 \\geq n(2t+1) $ and that is the reverse inequality.\r\n to see this consider any vertex $v$ and the set $N^+(v)=\\{w|v\\rightarrow w\\}$;for each fixed $w \\in N^+(v)$ we have that there are exactly $t$ vertices $u$ such that $v\\rightarrow u,w\\rightarrow u$. but then since $N^+(v)$ includes all the vertices such that $v\\rightarrow u$ it follows that in the induced subgraph on $N^+(v)$, every vertex has outdegree atleast $t$.then again summing over all $w \\in N^+(v)$ we have $\\displaystyle \\sum_{w \\in N^+(v)} |N^+(w)\\cap N^+(v)| \\geq d^+(v)t \\Rightarrow d^+(v)(d^+(v)-1)/2 \\geq d^+(v)t \\\\ \\Rightarrow d^+(v) \\geq 2t+1$ \r\nand we are through." } { "Tag": [ "geometry proposed", "geometry" ], "Problem": "$1^{\\circ}.\\ \\triangle ABC\\Longrightarrow \\underline {\\overline {\\left| A=90^{\\circ}\\Longleftrightarrow \\ b+c=2\\cdot (R+r)\\Longleftrightarrow 1+cosB+cosC=\\frac{r_a}{R}\\right| }}. $\r\nI ask for a geometrical interpretation.\r\n\r\n$2^{\\circ}$. Ascertain the form of the triangle ABC for which exists the relation $cs(s-c)=S(a+b-R).$", "Solution_1": "I believe the only one you can get is by actually constructing triangle s etc....", "Solution_2": "When I will return here (15.o3.o6, U.S.A.) I will offer the geometrical construct. \r\nIs it O.K. ? Good luck !" } { "Tag": [ "number theory unsolved", "number theory" ], "Problem": "does anyone know how to compute cyclic generators?\r\nfor example\r\nhow many cyclic generators of (Z/227Z)* are there?", "Solution_1": "[quote=\"mat12\"]does anyone know how to compute cyclic generators?\nfor example\nhow many cyclic generators of (Z/227Z)* are there?[/quote]\r\n\r\nI can't understand your notation can you explain better?\r\nThx ;) \r\n\r\nTCM", "Solution_2": "I think he wants to know how many generators the multiplicative group of invertible elements of $\\mathbb Z_{227}$ has. For a prime $p$, there are $\\varphi(p-1)$ generators for its multiplicative group of non-zero residues (in general, there are $\\varphi(d)$ elements of order $d$ for all divisors $d$ of $p-1$ in that group). \r\n\r\nP.S. $227$ is a prime, right? :)", "Solution_3": "mat12: DON'T post same topic in five different subforums !! :mad:" } { "Tag": [ "search", "number theory proposed", "number theory" ], "Problem": "Let $ k$ and $ d$ be integers such that $ k>1$ and $ 0 \\le d<9$.\r\nProve that there exists a positive integer $ n$ such that the decimal representation of the number $ 2^n$ has the digit $ d$ in the position $ k$(from the right) :)", "Solution_1": "A more general result here (post #3) :\r\nhttp://www.mathlinks.ro/Forum/viewtopic.php?search_id=1604429076&t=23417\r\n\r\nPierre.", "Solution_2": "Do anyone has a simple solution?\r\n(I don't understand in #2 solution,It's too hard for me!)", "Solution_3": "Pierre, the post you are referring to doesnt seem to tell much about the above problem!" } { "Tag": [ "LaTeX", "calculus", "real analysis", "trigonometry", "algebra", "function", "domain" ], "Problem": "I'm teaching an undergraduate Fourier Analysis course this spring, and I'm using my own notes. If any of you are interested, I have put the notes on line at the following address: http://www.csulb.edu/~kmerry/\r\n\r\nWhen you get there, click on \"Notes for Fourier Analysis.\" There's a separate file for each chapter, and the whole thing is about 150 pages (of fairly kludgy $ \\text{\\LaTeX},$ from a file originally written in MS Word.) It's posted in .pdf form.\r\n\r\nSome comments:\r\n\r\n1. The notes are heavily influenced by and borrow somewhat from a textbook by David Kammler.\r\n\r\n2. The official prerequisites for the course are just a differential equations course, although a student should have a little more than that to get something from the course. The audience usually includes a physics major or two - they're not all math majors. In particular, I am not insisting on a real analysis or advanced calculus course as prerequisite, and I'm certainly not expecting students to know the Lebesgue integral. (Which doesn't stop me from using norms: $ \\|f\\|_1,\\|f\\|_2,\\|f\\|_\\infty.$)\r\n\r\n3. The notation depends on complex exponentials, not sines and cosines. But I include a section up front introducing that.\r\n\r\n4. I would characterize my attitude towards serious analysis in these notes as \"Eulerian\": calculate first, justify later, if at all.\r\n\r\n5. The notes attempt to do Fourier analysis on four different domains simultaneously, as one subject. The four domains are $ \\mathbb{R},\\mathbb{T}\\ ( = \\mathbb{R}/(P\\mathbb{Z})),\\mathbb{Z},$ and $ \\mathbb{Z}/(N\\mathbb{Z}).$\r\n\r\n6. I include a section on distributions.\r\n\r\nIf you find the places in which I've responded on this site to a question about Fourier series or the Fourier transform, you'll see that I've generally used language and attitudes that come from these notes.", "Solution_1": "I fixed a few typos and posted the cleaned-up versions in the same place.", "Solution_2": "Perhaps this is a lot to ask, but would it be possible to provide solutions for the problems, or at least sketches of solutions?" } { "Tag": [ "combinatorics unsolved", "combinatorics" ], "Problem": "Prove that: $ \\left ( C_n^1\\right)^2\\plus{}2\\left ( C_n^2\\right)^2\\plus{}...\\plus{}n\\left ( C_n^n\\right)^2\\equal{}\\frac{(2n\\minus{}1)!}{\\left [ (n\\minus{}1)!\\right]^2}$", "Solution_1": "$ \\sum_{i\\equal{}1}^{n} i \\binom{n}{i}^2 \\equal{} n\\sum_{i\\equal{}1}^{n} \\binom{n\\minus{}1}{i\\minus{}1}\\binom{n}{n\\minus{}i}$\r\nAnd this is just $ n$ multiplied by the coefficient of $ x^{(i\\minus{}1)\\plus{}(n\\minus{}i)}\\equal{}x^{n\\minus{}1}$ in the product $ (1\\plus{}x)^{n\\minus{}1}(1\\plus{}x)^{n}\\equal{}(1\\plus{}x)^{2n\\minus{}1}$\r\nWhich is $ \\binom{2n\\minus{}1}{n\\minus{}1}$.\r\nSo we have $ n \\cdot \\binom{2n\\minus{}1}{n\\minus{}1} \\equal{} n\\frac{(2n\\minus{}1)!}{(n\\minus{}1)!n!} \\equal{} \\frac{(2n\\minus{}1)!}{\\left [ (n\\minus{}1)!\\right]^2}$.\r\n\r\n(after the first manipulation it becomes just a special case of Vandermonde's identity)" } { "Tag": [ "geometry", "circumcircle", "geometry proposed" ], "Problem": "In an acute triangle ABC bisector AD is equal to AC and AD is perpendicular to OH, where O is circumcenter of triangle ABC and H is orthocenter. Find angles of triangle.", "Solution_1": "who wants to know solution, i will post", "Solution_2": "[quote=\"Ulanbek_Kyzylorda KTL\"]In an acute triangle ABC bisector AD is equal to AC and AD is perpendicular to OH, where O is circumcenter of triangle ABC and H is orthocenter. Find angles of triangle.[/quote]\r\n\r\nLet's $ \\angle A\\equal{}4\\alpha$\r\nwe have $ \\angle CAD\\equal{}\\angle BAD\\equal{}2\\alpha$\r\n$ \\angle CAH\\equal{}\\angle DAH\\equal{}\\alpha$\r\n$ \\angle ACB\\equal{} 90^o\\minus{}\\alpha$\r\n$ \\angle ABC\\equal{}90^o\\minus{}3\\alpha$\r\nIt's known that $ \\angle DAH\\equal{}\\angle DAO\\equal{}\\alpha$(becouse points $ H$ and $ O$ are isogonal conjugate with respect to $ \\triangle ABC$) and we have $ AD \\perp OH$ therefore $ AH\\equal{}AO\\equal{}R$\r\nalso it's known that $ AH\\equal{}2R \\cdot cos(4\\alpha)$\r\nSo $ R\\equal{}2R\\cdot cos(4\\alpha)$\r\nor $ cos(4\\alpha)\\equal{}\\frac{1}{2}$\r\n$ 4\\alpha\\equal{}60^o$\r\n$ \\alpha\\equal{}15^o$\r\n$ \\angle BAC\\equal{}60^o$, $ \\angle ACB\\equal{}75^o$, $ \\angle ABC\\equal{}45^o$", "Solution_3": "AC=AD :arrow: AN is height x \\minus{} 1 \\right) \\wedge \\left( y > \\minus{}x \\plus{} 1 \\right) \\}$\r\n\r\nsorry ! I don't know how to paste the image on forum :D", "Solution_2": "I will check your answer later. :lol:", "Solution_3": "[quote=\"brahman\"][quote=\"kunny\"]Draw the domain of $ (a,\\ b)$ for which the quadratic equation with real coefficients : $ x^2 \\plus{} ax \\plus{} b \\equal{} 0$ has more than one solution in $ \\minus{} 1 < x < 1$.[/quote]\n\nThe domain is V : \n\n$ V \\equal{} \\{ M \\left( x \\; ; \\; y \\right) \\in \\Re ^2 \\; : \\; \\left( \\minus{} 1 < x < 1 \\right) \\wedge \\left( y < \\frac {x^2}{4} \\right) \\wedge \\left( y > x \\minus{} 1 \\right) \\wedge \\left( y > \\minus{} x \\plus{} 1 \\right) \\}$\n\nsorry ! I don't know how to paste the image on forum :D[/quote]\r\n\r\nI think is $ V \\equal{} \\{ M \\left( x \\; ; \\; y \\right) \\in \\Re ^2 \\; : \\; \\left( \\minus{} 2 < x < 2 \\right) \\wedge \\left( y < \\frac {x^2}{4} \\right) \\wedge \\left( y > x \\minus{} 1 \\right) \\wedge \\left( y > \\minus{} x \\plus{} 1 \\right) \\}$", "Solution_4": "[quote=\"shyong\"][quote=\"brahman\"][quote=\"kunny\"]Draw the domain of $ (a,\\ b)$ for which the quadratic equation with real coefficients : $ x^2 \\plus{} ax \\plus{} b \\equal{} 0$ has more than one solution in $ \\minus{} 1 < x < 1$.[/quote]\n\nThe domain is V : \n\n$ V \\equal{} \\{ M \\left( x \\; ; \\; y \\right) \\in \\Re ^2 \\; : \\; \\left( \\minus{} 1 < x < 1 \\right) \\wedge \\left( y < \\frac {x^2}{4} \\right) \\wedge \\left( y > x \\minus{} 1 \\right) \\wedge \\left( y > \\minus{} x \\plus{} 1 \\right) \\}$\n\nsorry ! I don't know how to paste the image on forum :D[/quote]\n\nI think is $ V \\equal{} \\{ M \\left( x \\; ; \\; y \\right) \\in \\Re ^2 \\; : \\; \\left( \\minus{} 2 < x < 2 \\right) \\wedge \\left( y < \\frac {x^2}{4} \\right) \\wedge \\left( y > x \\minus{} 1 \\right) \\wedge \\left( y > \\minus{} x \\plus{} 1 \\right) \\}$[/quote]\r\n\r\nuhm .... may be ... you're right ! :D\r\n\r\n$ \\minus{}1 < \\minus{} \\frac{a}{2} < 1 \\;\\; \\Leftrightarrow \\;\\; \\minus{}2 < a < 2 \\;\\; \\left( it's \\; not\\; \\Leftrightarrow \\;\\; \\minus{}1 < a < 1 \\right)$", "Solution_5": "Sorry, What does $ \\wedge$ mean?", "Solution_6": "It means \"and.\" For example, $ \\{(x, y) \\mid (x > 0) \\wedge (y > 0)\\}$ is the first quadrant of the Cartesian plane.", "Solution_7": "Thank you. Regrettably I haven't received the correct answer. First I thought this problem should be posted in Intermidate Forum, but this problem doesn't seem to be easy, so I have posted in the forum.", "Solution_8": "$ x^2 \\plus{} ax \\plus{} b \\equal{} 0 \\Rightarrow x \\equal{} \\frac { \\minus{} a \\pm \\sqrt {a^2 \\minus{} 4b}}{2}$\r\nIn order for $ \\minus{} 1 < x < 1$ for both zeros:\r\n\r\n[hide]$ \\minus{} 1 < \\frac { \\minus{} a \\pm \\sqrt {a^2 \\minus{} 4b}}{2} < 1$\n$ \\minus{} 2 < \\minus{} a \\pm \\sqrt {a^2 \\minus{} 4b} < 2$\n$ a \\minus{} 2 < \\pm \\sqrt {a^2 \\minus{} 4b} < a \\plus{} 2$\n\n$ \\Leftrightarrow a \\minus{} 2 < \\minus{} \\sqrt {a^2 \\minus{} 4b} \\leq 0$ and $ 0 \\leq \\sqrt {a^2 \\minus{} 4b} < a \\plus{} 2$\n$ \\Leftrightarrow a^2 \\minus{} 4a \\plus{} 4 > a^2 \\minus{} 4b \\geq 0$ and $ 0 \\leq a^2 \\minus{} 4b < a^2 \\plus{} 4a \\plus{} 4$ and $ \\minus{}2 < a < 2$\n$ \\Leftrightarrow a \\minus{} 1 < b \\leq \\frac {a^2}{4}$ and $ \\minus{} a \\minus{} 1 < b \\leq \\frac {a^2}{4}$ with $ \\minus{}2 < a < 2$\n\nSo the domain, (a,b), is the set of points such that: $ b \\leq \\frac {a^2}{4}$, $ b > \\minus{} a \\minus{} 1$, $ b > a \\minus{} 1$, and $ \\minus{}2 < a < 2$.[/hide]", "Solution_9": "Your solution isn't logic.", "Solution_10": "Which part is incorrect?", "Solution_11": "Sorry, now your solution seems to be true, when I saw your solution, I couldn't see last line. So I tried to delete my post, but I couldn't that, that is inconvinient.\r\nAnyway, you should consider the case for which the qudratic equation has only one solution." } { "Tag": [ "algebra open", "algebra" ], "Problem": "find all positive integers n such that \\[d_{5}^{2}+d_{6}^{2}-1 = n\\] ,where \\[\\begin{array}{l}1 = d_{1}< d_{2}< ... < d_{k}= n \\\\ \\\\ \\end{array}\\] are all the positive divisors of n.", "Solution_1": "Let $n=p_{1}^{k_{1}}*p_{2}^{k_{2}}*...*p_{s}^{k_{s}}$. From $d_{5}^{2}+d_{6}^{2}-1=n$ we have $(d_{5},d_{6})=1\\Longrightarrow d_{5}d_{6}|n$ and $n=ld_{5}*d_{6},(k_{1}+1)(k_{2}+1)...(k_{s}+1)\\ge 12$.\r\nIt is easy to show $l\\not =1$. If $l>2$ then $d_{5}^{2}+d_{6}^{2}-1 p prime (1-1/p^2)^-1=:pi::^2:/6. But that's irrational! Thus there are infinitely many primes in the product, which is all the primes. Yay!", "Solution_2": "Interesting. This problem appeared in a problem-solving section of a calculus textbook that an instructor posed. The solution was found in about 3 minutes, to his surprise.\r\nDarn you ComplexZeta :)", "Solution_3": "In one of our ARML practices, we had a problem that was just like this. I think it was from the TRML." } { "Tag": [ "inequalities unsolved", "inequalities" ], "Problem": "Let $ a$, $ b$, $ c$ be three positive real numbers such that $ a\\plus{}b\\plus{}c\\equal{}3$.\r\nProve that \\[ \\frac{a}{ab\\plus{}1}\\plus{}\\frac{b}{bc\\plus{}1}\\plus{}\\frac{c}{ca\\plus{}1}\\ge\\frac{3}{2}\\]", "Solution_1": "[quote=\"April\"]Let $ a$, $ b$, $ c$ be three positive real numbers such that $ a\\plus{}b\\plus{}c \\equal{} 3$.\nProve that\n\\[ \\frac{a}{ab\\plus{}1}\\plus{}\\frac{b}{bc\\plus{}1}\\plus{}\\frac{c}{ca\\plus{}1}\\ge\\frac{3}{2}\\]\n[/quote]\r\nSee here:\r\nhttp://www.artofproblemsolving.com/Forum/viewtopic.php?t=21689" } { "Tag": [], "Problem": "Whenever __________________________________, God kills a kitten.\r\n\r\nFill the blank!\r\n\r\n\r\n\r\n\r\nWhenever you say \"I lost\" more times than necessary to make as many people lose the game as possible, God kills a kitten.", "Solution_1": "every time thales or fambior posts, god kills a kitten", "Solution_2": "you jump off a cliff", "Solution_3": "this is way too easy\r\nway way way too easy", "Solution_4": "Whenever someone abuses the priveledge to massload bots, God kills a kitten.", "Solution_5": "Whenever I can't think of a legitimate forum game idea, God kills a kitten.", "Solution_6": "[quote=\"randomdragoon\"]Whenever __________________________________, God kills a kitten.\n\nFill the blank!\n\n\n\n\nWhenever you say \"I lost\" more times than necessary to make as many people lose the game as possible, God kills a kitten.[/quote]\r\nomg you evil\r\nI lost", "Solution_7": "Whenever you write the sentence, \"Whenever you write the sentence, \"Whenever you write the sentence, \"Whenever you write the sentence ...\", God kills a kitten\", God kills a kitten\", God kills a kitten\", God kills a kitten\", God kills a kitten", "Solution_8": "[quote=\"Sly Si\"]Whenever you write the sentence, \"Whenever you write the sentence, \"Whenever you write the sentence, \"Whenever you write the sentence ...\", God kills a kitten\", God kills a kitten\", God kills a kitten\", God kills a kitten\", God kills a kitten[/quote]\r\n\r\nThat's not a well defined sentence ... recall Russel's paradox! :roll: \r\n\r\nEverytim god is bored, god kills a kitten.", "Solution_9": "Whenever someone fails a math test, God kills a kitten.", "Solution_10": ":mad: :mad: :mad:", "Solution_11": "[quote=\"now a ranger\"] :mad: :mad: :mad: [/quote]\r\n\r\nwe were trying to consciously avoid the obvious answer\r\n\r\ngood job sir", "Solution_12": "Whenever you get a math problem wrong, god kills a kitten. :o", "Solution_13": "Whenever you fight MISSINGNO, God kills a kitten.", "Solution_14": "whenever you PULL, HARDER, STRINGS, MARTYR, god kills a kitten", "Solution_15": "[quote=\"vulgarfraction\"]every time someone pronounces 'euler' wrong god kills a kitten :P[/quote]\r\new, ler", "Solution_16": "Everytime someone steals my library card, God kills a kitten.", "Solution_17": "when thames (the guy who screws up GFF) posts. God kills a kitten\r\n\r\nWHY KITTENS.", "Solution_18": "Everytime you kill a kitten, :mad: :mad: :mad:", "Solution_19": "ok... uncalled for...", "Solution_20": "When the kitten asks for it, god kills a kitten... :D", "Solution_21": "[quote=\"vulgarfraction\"]every time someone pronounces 'euler' wrong god kills a kitten :P[/quote]\r\nThen I killed a couple of kittens in eighth grade.\r\n :mad: :mad: :mad: \r\n\r\nWhat? Just keeping up the running gag.", "Solution_22": ":mad: :mad: :mad:", "Solution_23": "When God kills a kitten, God kills a kitten.", "Solution_24": "Whenever Treething makes a \"treething's favorite number problem\", God kills a kitten", "Solution_25": "Each time you breathe, a ninja tries to do a backflip. If the ninja fails, God kills a kitten.", "Solution_26": "Whenever my friend Tommy yells at me, God kills a kitten. \r\n\r\nKittens must be dying at an alrming rate. :huh: :maybe:", "Solution_27": "especiially with that pet food recall in USA because of poisoning...\r\n\r\n\r\n\r\nhurt junggi heal klebian\r\n\r\nwait, this isn't the hurt-heal.\r\n\r\nWhenever I become constipated, God kills a kitten.", "Solution_28": "when kitten convert to atheism, god kills a kitten", "Solution_29": "When the kitty proves that pi is rational, God kills that kitty" } { "Tag": [ "geometry", "trapezoid", "geometry proposed" ], "Problem": "The diagonals of the trapezoid $ ABCD$ with bases $ AB$ and $ CD$ are mutually perpendicular and intersect at point $ O$.Let $ M$ and $ B$ be points on the half lines $ OA$ and $ OB$,respectively,such that the angles $ \\angle{ANC}$ and $ \\angle{BMD}$ are right angles.Let $ E$ be the midpoint of the line segment $ MN$.Prove that \r\nthe line $ OE$ is perpendicular to the line $ AB$.", "Solution_1": "$ AB \\parallel DC \\Longrightarrow$ $ \\frac{OC}{OA} \\equal{} \\frac{OD}{OB} \\Longrightarrow$ the $ \\triangle ANC \\sim \\triangle BMD$ with $ \\angle ANC \\equal{} \\angle BMD$ and altitudes NO, MO are (oppositely) similar, $ \\angle MCN \\equal{} \\angle MDN \\Longrightarrow$ CDMN is cyclic with perpendicular diagonals $ CM \\perp DN,$ its diagonal intersection is identical with its anticenter, the intersection of maltitudes, and $ OE \\perp (DC \\parallel AB)$ is one of its maltitudes.", "Solution_2": "[quote=\"yetti\"]$ AB \\parallel DC \\Longrightarrow$ $ \\frac {OC}{OA} \\equal{} \\frac {OD}{OB} \\Longrightarrow$ the $ \\triangle ANC \\sim \\triangle BMD$ with $ \\angle ANC \\equal{} \\angle BMD$ and altitudes NO, MO are (oppositely) similar, $ \\angle MCN \\equal{} \\angle MDN \\Longrightarrow$ CDMN is cyclic with perpendicular diagonals $ CM \\perp DN,$ its diagonal intersection is identical with its anticenter, the intersection of maltitudes, and $ OE \\perp (DC \\parallel AB)$ is one of its maltitudes.[/quote]\r\nWhy $ \\frac{OC}{OA}\\equal{}\\frac{OD}{OB} \\rightarrow \\triangle ANC \\sim \\triangle BMD$?", "Solution_3": "[quote=\"apollo\"][quote=\"yetti\"]$ AB \\parallel DC \\Longrightarrow$ $ \\frac {OC}{OA} \\equal{} \\frac {OD}{OB} \\Longrightarrow$ the $ \\triangle ANC \\sim \\triangle BMD$ with $ \\angle ANC \\equal{} \\angle BMD$ and altitudes NO, MO are (oppositely) similar, $ \\angle MCN \\equal{} \\angle MDN \\Longrightarrow$ CDMN is cyclic with perpendicular diagonals $ CM \\perp DN,$ its diagonal intersection is identical with its anticenter, the intersection of maltitudes, and $ OE \\perp (DC \\parallel AB)$ is one of its maltitudes.[/quote]\nWhy $ \\frac {OC}{OA} \\equal{} \\frac {OD}{OB} \\rightarrow \\triangle ANC \\sim \\triangle BMD$?[/quote]\r\nSorry,I saw it." } { "Tag": [ "USAMTS" ], "Problem": "Round 3 of the USAMTS is now available at [url]http://www.usamts.org[/url]", "Solution_1": "Alice and Bob...again... :lol:" } { "Tag": [ "algebra proposed", "algebra" ], "Problem": "Let $\\omega_1,\\omega_2, . . . ,\\omega_k$ be distinct real numbers with a nonzero sum. Prove that there exist integers $n_1, n_2, . . . , n_k$ such that $\\sum_{i=1}^k n_i\\omega_i>0$, and for any non-identical permutation $\\pi$ of $\\{1, 2,\\dots, k\\}$ we have\n\\[\\sum_{i=1}^k n_i\\omega_{\\pi(i)}<0.\\]", "Solution_1": "Discussed before [url=http://www.artofproblemsolving.com/Forum/viewtopic.php?t=52906]here[/url].", "Solution_2": "Without loss of generality, let $\\omega_1 > \\omega_2 > \\ldots > \\omega_k$, and let $S = \\omega_1 + \\omega_2 + \\ldots + \\omega_k$. It is clear that of all sums of the form $2 \\omega_{\\pi(1)} + \\omega_{\\pi(2)} + \\ldots + \\omega_{\\pi(n)}$, $2\\omega_1 + \\omega_2 + \\ldots + \\omega_n$ is greatest and $\\omega_1 + 2\\omega_2 + \\omega_3 + \\ldots + \\omega_n$ is the second greatest. Because the rationals are dense over the reals, we can find some rational number $\\epsilon = \\frac{a}{b}, b>0$ such that \n \\[ \\omega_1 + 2\\omega_2 + \\ldots + \\omega_n + S \\epsilon < 0 < 2\\omega_1 + \\omega_2 + \\ldots + \\omega_n + S \\epsilon\\]\nThen \n\\begin{align*}\n(2+\\epsilon)\\omega_1 + (1+\\epsilon)\\omega_2 + \\ldots + \\omega_n \n&> 0 > (1 + \\epsilon)\\omega_1 + (2+\\epsilon)\\omega_2 + (1+\\epsilon)\\omega_3 + \\ldots + (1+\\epsilon)\\omega_n \\\\ \n&\\geq (2+\\epsilon)\\omega_{\\pi(1)} + (1+\\epsilon)\\omega_{\\pi(2)} + \\ldots + (1+\\epsilon)\\omega_{\\pi(n)} \\end{align*}\nfor any permutation $\\pi$ distinct from the identity. Multiplying both sides by $b$ shows that the sequence $n_1 = 2b + a = b(2 + \\epsilon)$ and $n_i = b + a = b(1 + \\epsilon)$ for $2 \\leq i \\leq k$ suffices." } { "Tag": [ "inequalities", "trigonometry", "function" ], "Problem": "Prove that\r\n$sec^{2}\\theta+sin2\\theta+2tan\\theta\\sqrt{1+sin2\\theta}> 9sin^{\\frac{4}{3}}\\theta$\r\nfor all $\\theta$.", "Solution_1": "If $\\theta=\\frac{2\\pi}{3}$ then $\\sec^{2}\\theta+\\sin \\theta+2 \\tan \\theta \\sqrt{1+\\sin 2\\theta}<9 \\sin^{2}\\theta$\r\nBecause,\r\n$\\sec^{2}\\theta+\\sin \\theta+2 \\tan \\theta \\sqrt{1+\\sin 2\\theta}= 4-\\frac{\\sqrt{3}}{2}-\\sqrt{12-6\\sqrt{3}}<4$\r\n$9 \\sin^{2}\\theta = \\frac{27}{4}= 6.75$", "Solution_2": "You misread the question. It's $sin2\\theta$, not $sin\\theta$.", "Solution_3": "I'm sorry. \r\nI have done a typo.\r\n\r\n$\\sec^{2}\\theta+\\sin 2\\theta+2 \\tan \\theta \\sqrt{1+\\sin 2\\theta}= 4-\\frac{\\sqrt{3}}{2}-\\sqrt{12-6\\sqrt{3}}<4$", "Solution_4": "Blue line is $\\sec^{2}\\theta+\\sin 2\\theta+2 \\tan \\theta \\sqrt{1+\\sin 2\\theta}$\r\nRed line is $9\\sin^{2}\\theta$", "Solution_5": "Sorry, let me edit.", "Solution_6": "Oops, the question had another mistype. It should say \"for $0 < \\theta < \\frac{\\pi}{2}$.\" It's too late to edit it.", "Solution_7": "[hide=\"Solution\"]Simplify the left side.\n$1+tan^{2}\\theta+2sin\\theta cos\\theta+2tan\\theta \\sqrt{cos^{2}\\theta+sin^{2}\\theta+2sin\\theta cos\\theta}$\n$cos^{2}\\theta+sin^{2}\\theta+2sin\\theta cos\\theta+2tan\\theta(sin\\theta+cos\\theta)+tan^{2}\\theta$\n$(cos\\theta+sin\\theta)^{2}+2(tan\\theta)(sin\\theta+cos\\theta)+tan^{2}\\theta$\nSo we have\n$(cos\\theta+sin\\theta+tan\\theta)^{2}> 9sin^{\\frac{4}{3}}\\theta$\n$cos\\theta+sin\\theta+tan\\theta > 3sin^{\\frac{2}{3}}\\theta$\n$\\frac{cos\\theta+sin\\theta+tan\\theta}{3}> \\sqrt[3]{sin^{2}\\theta}$\n$\\frac{cos\\theta+sin\\theta+tan\\theta}{3}> \\sqrt[3]{sin\\theta cos\\theta tan\\theta}$\nwhich is true by AM-GM if $sin\\theta$, $cos\\theta$, and $tan\\theta$ are all positive. The restrictions said for $0 < \\theta < \\pi$, for which all trigonometric functions are positive. However, AM-GM states $AM \\ge GM$, not $AM > GM$. For our problem to work, we simply need to show that the equality case ($sin\\theta = cos\\theta = tan\\theta$) is impossible. It is easy to show that $cos\\theta$ and $sin\\theta$ are only equal when the are $\\frac{\\sqrt{2}}{2}$. In this case $tan\\theta = 1$ and thus the equality case is impossible.\n[/hide]" } { "Tag": [], "Problem": "Find the lowest $ n\\in\\mathbb{N}$ such that $ 2008|\\underbrace{888...888}_{\\text{2008n}}$", "Solution_1": "It is equavalent to\r\n $ n\\in\\mathbb{N}$ such that $ 251|\\underbrace{1111...111}_{\\text{2008n}}$\r\nor $ 251|10^{2008n}\\minus{}1$ or $ 50|2008n$ or $ 25|n$.", "Solution_2": "How to proof if $ n<25$ then $ 2008\\not|\\underbrace{888...888}_{\\text{2008n}}$ :(", "Solution_3": "[quote=\"zai\"]How to proof if $ n < 25$ then $ 2008\\not|\\underbrace{888...888}_{\\text{2008n}}$ :([/quote]\r\n\r\nShow $ 2008\\not|\\underbrace{8888}$\r\nand $ 2008\\not|\\underbrace{88888}$", "Solution_4": "What is a convenient way to find the smallest $ n$?\r\n\r\nI tried by hand and cant find a solution for $ n< 10^{13}$", "Solution_5": "minimal T, suth that $ 251|10^{T}\\minus{}1$ is 50, therefore $ 50|2008n$ or $ 25|n$.", "Solution_6": "[quote=\"Rust\"]$ 251|10^{2008n}\\minus{}1$ or $ 50|2008n$ or $ 25|n$.[/quote]\r\n\r\nMay I ask, how did you make that connection?\r\n\r\nAnd does that mean...\r\n[hide]From $ 25|n$, that implies $ min(n)\\equal{}25$?[/hide]" } { "Tag": [], "Problem": "The motto is:\r\n\r\n\"Christmas Spirit is about giving,\r\nFriendship is about sharing,\r\nRelationship about receiving\"", "Solution_1": "Um, what are we supposed to post on this topic?", "Solution_2": "People, stop reviving old topics! (Especially you, armalite46!)" } { "Tag": [ "inequalities proposed", "inequalities" ], "Problem": "x+y+z=1, x,y,z are positive reals\r\nFind the minimum of\r\n\r\n$\\frac{{\\sqrt x }}{{1 - x}} + \\frac{{\\sqrt y }}{{1 - y}} + \\frac{{\\sqrt z }}{{1 - z}}$", "Solution_1": "I think this is an easy and well-known problem . \r\n We will prove $ \\frac{\\sqrt{x}}{1-x} \\geq \\frac{3\\sqrt3}{2}x $ \r\n It's equivalent to $ \\frac{2}{3\\sqrt3} \\geq \\sqrt{x}(1-x) $ \r\n It is followed by using AM-GM with : 2x , 1-x , 1-x \r\n So min = $ 3\\sqrt{3}/2 $", "Solution_2": "[quote=\"nttu\"]I think this is an easy and well-known problem . \n We will prove $ \\frac{\\sqrt{x}}{1-x} \\geq \\frac{3\\sqrt3}{2}x $ \n It's equivalent to $ \\frac{2}{3\\sqrt3} \\geq \\sqrt{x}(1-x) $ \n It is followed by using AM-GM with : 2x , 1-x , 1-x \n So min = $ 3\\sqrt{3}/2 $[/quote]\r\n\r\nthis is exactly my solution, while i am not sure the ans is 3sqrt 3/2 actually...\r\nthanks for verifying it for me. :D", "Solution_3": "[quote=\"nttu\"]\n It is followed by using AM-GM with : 2x , 1-x , 1-x [/quote]\r\n\r\nbut how ? I get only\r\n$\\frac{2x+(1-x)+(1-x)}{3}=\\frac{2}{3} \\geq \\sqrt[3]{2x(1-x)^2}$\r\nwhere's my mistake ?", "Solution_4": "Megus, your expression $A \\ge B$ is equal to Nttu expression.\r\n\r\nYou must do $A^{3/2} \\geq B^{3/2}$", "Solution_5": "thanks - I must've been blind :blush: :blush: :blush:", "Solution_6": "Alternately, if one doesn't see the AM-GM right away, we have that:\r\n\r\n$\\displaystyle \\frac {\\sqrt x}{1-x} \\geq \\frac {3\\sqrt 3}{2} x \\Rightarrow 0\\geq (3x-1)^2(3x-4)x$\r\n\r\nwhich is clearly true for $x,y,z \\leq 1$" } { "Tag": [ "superior algebra", "superior algebra solved" ], "Problem": "I got this as homework:\r\n\r\nIf we have x3=x for all x in a ring A then the ring is commutative.\r\n\r\n[i]unknown source[/i]\r\n\r\nMy teacher likes to discourage us, so he said: \" this is awfully hard\" :D I think I might have seen it before, but i don't know where.", "Solution_1": "We have \r\ni) 6A={0}\r\nii) 2A and 3A are ideals and 2A+3A = A; 2A \\cap 3A={0}\r\n \r\n---------------------------------------------------------------------------------\r\n\r\ni) 2x=x+x=(x+x)^3=x^3+3x^3+3x^3+x^3=8x^3=8x\r\n6x=0 \r\nii)It is just a verification that 2A and 3A are ideals of A\r\n2A+3A is included in A\r\nWith i) x=-5x=3(-x)+2(-x) so 2A+3A=A\r\nLet x \\in 2A \\cap 3A\r\nx=2y=3z but 2x=6z=0, 3x=6y=0 and x=3x-2x=0 =>2A \\cap 3A={0}\r\n\r\n--------------------------------------------------------------------------\r\n\r\nIf we have the commutativity for A'=A/2A and A\"=A/3A then for x,y \r\nin A xy-yx \\in 2A and xy-yx \\in 3A with ii) xy=yx \r\n\r\nLet's prove if A is a ring s.t 2A={0} then A is commutative:\r\n1+x=(1+x)^3=1+3x^2=> x=3x^2=x^2, then A is Boolean ring \r\nit is known that Boolean ring is commutative.\r\n\r\n------------------------------------------------------------------------------\r\n\r\nIf A is a ring s.t 3A={0} \r\nx+y=(x+y)^3=> \r\nx^2y+xyx+xy^2+yx^2+yxy+y^2x=0 (1)\r\n\r\nwith (x-y)^3 we have \r\n-x^2y-xyx+xy^2-yx^2+yxy+y^2x=0 (2)\r\n\r\ndifference (1) & (2)\r\n\r\n2x^2y+2xyx+2yx^2=0 but 3A={0} 2t=-t \r\n\r\nx^2y+xyx+yx^2=0 (3) \r\n\r\nmultiply (3) by x right or left\r\n\r\nxy+x^2yx+xyx^2=0\r\nx^2yx+xyx^2+yx=0 \r\n\r\ndifference => xy-yx=0", "Solution_2": "More generally, a theorem of Jacobson states that :\r\nIf for each x in a ring A there exists an integer n > 1, which may depend on x, such that x n = x, then the ring is commutative.\r\n\r\nYou will find a solution of your problem and references on Jacobson's theorem at :\r\nhttp://www.mathematik.uni-bielefeld.de/~sillke/PUZZLES/herstein\r\n\r\nPierre.", "Solution_3": "nice theorem :) but I guess it's not that easy to prove. we have had in Romania in contests the cases n=2,3,4,5,6,8 (this are the ones I've seen) :)", "Solution_4": "Romanians don't like number 7??? :D \r\n\r\nPierre.", "Solution_5": "[quote=\"pbornsztein\"]Romanians don't like number 7??? :D \n\nPierre.[/quote]\r\n\r\n7 is my favorite number. Even my phone number has lots of 7 in it.\r\nMy birthday has a 7 in it. And I'm pretty sure I'm romanian. I'm not proud of it, but that is what I am :D (a big smile).", "Solution_6": "Thanks for the solution, Mihai! \r\n\r\nI did find a solution myself:\r\n\r\nLemma 1: If we have a ring s.t x2 =0 =>x=0 then every idempotent element is in Z(A):\r\n\r\nAssume x is idempotent, i.e. x2 =x. We have \r\n(xyx-xy)2 =0 and (xyx-yx)2 =0, so xyx-xy=xyx-yx, so xy=yx, for any y in A.\r\n\r\nLemma 2: If in a ring xn=x then xn-1 is in Z(A). \r\n\r\nWe just apply Lemma 1 to the ring.\r\n\r\nNow to our problem. From Lemma 2 we get x*y2=y2*x for all x and y in A. \r\n\r\nWe have (x+1)3=x+1 for all x, so 3x2+3x=0 for all x. (*)\r\n\r\n(-x2-x)2 = x4 + x2 +2x3 = 2x2+2x=\r\n-x2-x from (*). This means that -x2-x is idempotent, so it's in Z(A) for all x in A (Lemma 1), and since x2 is also in Z(A), it means that x is in Z(A) for all x, so we're done.", "Solution_7": "[quote=\"grobber\"]Thanks for the solution, Mihai! \n[/quote]\r\n\r\nI would have liked to solve this problem, but I didn't. I think your thanks are for Moubinool.\r\nI think this problem was given in a Titeica contest, and I think I had it too in the form:p prime and x p=x for all x, than A is commutative, but I'm not sure.\r\nI had this or a similar problem in my 11th grade at the Titeica teams contest. But the team had a 12th grade member, 11&12 grade problems, and I couldn't touch the problem. It was too hot for me back then:D. It is a 12th grade problem and I've expected the 12th grade member to solve it. If I remember it correctly, no team solved that problem. However, my teammate solved the problem for p=2 and 3!!! and the team got some precious points that allowed us to win the contest.", "Solution_8": "Sorry Moubinool! I didn't take a good look to see who solved the problem! :D" } { "Tag": [ "geometry", "geometric transformation", "reflection" ], "Problem": "So I've been thinking about this for far too long...I'd like to know how to trisect a line segment with a straight-edge and non-collapsing compass in 6 steps (or less). Trisection means finding BOTH trisection points - though, of course, if you find one, you're one step away from getting the other one too. (A step is drawing a new line or a new circle. Picking a random point and extending an existing line do not count as steps.) There's a book that tells me this is possible. I can do it 1 way in 7 steps, about 6 ways in 8 steps, and several more ways in even more steps. No one in university seems to know anything about purely Euclidean geometry, so I'm referring the question to this forum.", "Solution_1": "What do you mean by 'non-collapsing compass' :?: \r\n\r\nIf you mean that it is of fixed length, I can't solve it in 6 steps\r\nIf it is a usual compass, then it's ok", "Solution_2": "No, a nice compass. As in, you can draw a circle with any radius that's a distance between two points you already have (not only a distance from the center of the circle to another point). So if you already have points A, B, and C, you can draw a circle centered at A with radius BC. Sorry if \"non-collapsing compass\" was confusing...that was way more explanation than it was worth. So how do you do it in 6 steps?", "Solution_3": "ok then..\r\n\r\nLet $ AM$ be the given segment. We'll construct a triangle $ ABC$ with $ M,N$ being the midpoints of $ BC$ and $ CA$ respectively\r\n\r\nPick an arbitrary point $ N$ out of the segment $ AM$ (not a step)\r\n\r\n[b]Step 1: [/b] Draw the ray $ AN$\r\n[b]Step 2: [/b] Reflect $ A$ through $ N$ to get the point $ C$ (requires a circle centered at $ N$, passing $ A$)\r\n[b]Step 3: [/b] Draw the ray $ CM$\r\n[b]Step 4: [/b] Reflect $ C$ through $ M$ to get the point $ B$\r\n[b]Step 5: [/b] Draw the line $ BN$, This intersects $ AM$ at the centroid $ G$\r\n[b]Step 6: [/b] Reflect $ M$ through $ G$ to get the point $ G'$\r\n\r\nThen of course $ MG=GG'=G'A$", "Solution_4": "Ahhh...ok, thanks! :lol: I was beating around the bush as usual.", "Solution_5": "I just saw this yesterday.\r\n\r\nTo trisect segment $ AB$,\r\n1) Construct circle $ \\omega_{1}$ with centre $ A$ and radius $ AB$,\r\n2) Construct circle $ \\omega_{2}$ with centre $ B$ and radius $ AB$,\r\nLet $ C$ be an intersection of $ \\omega_{1}$ and $ \\omega_{2}$,\r\nExtend AB to meet $ \\omega_{2}$ at E,\r\n3) Extend CA to meet $ \\omega_{1}$ at D,\r\n4) Let $ F$ lie on $ \\omega_{2}$ and $ DE$,\r\n5) Let $ G$ lie on $ CF$ and $ AB$,\r\n6) Let $ G'$ lie on $ AB$ and the circle with centre $ G$ and radius $ AG$ such that $ G'$ $ \\neq$ $ A$,\r\n\r\nThen $ AG$ = $ GG'$ = $ G'B$.\r\n\r\nI am wondering if anyone has an elegant proof that this is in fact a trisection. I have a trigonometric proof which is not so bad but it hides the elegance of the construction.", "Solution_6": "given line AB\r\n1 build square ABCD\r\n2 bisect CD at E\r\n3 draw diagonal AC\r\n4 draw line BE intersects AC at F\r\n5 project F to AB then we have BF = 1/3 AB", "Solution_7": "chichi, how do you build a square in one step?\r\n\r\ncarpo, here's a proof:\r\n\r\nLet $ \\angle DCF=x$ and let $ AB=r$, the radius of $ w_{1}$ and $ w_{2}$. We know $ \\triangle ABC$ is equilateral, so $ \\angle BAC=60^\\circ$. Then in $ \\triangle AGC$, $ \\angle AGC=120^\\circ-x$. Now $ \\angle BAD=120^\\circ$, and $ AB=AD$, so $ \\triangle ABD$ is isosceles and $ \\angle ABD=\\angle ADB=30^\\circ$. Let $ CF$ meet $ DB$ at $ I$. Then in $ \\triangle GBI$, $ \\angle BGI=120^\\circ-x$, and $ \\angle GBI=30^\\circ$, so $ \\angle GIB=30^\\circ+x$. Now $ CD$ is a diameter of $ w_{1}$, so $ \\angle CBI=\\angle CBD=90^\\circ$. $ AE$ is a diameter of $ w_{2}$, so $ \\angle DCE=\\angle ACE=90^\\circ$. $ \\angle CEB=\\frac{1}{2}\\angle ABC=30^\\circ$, as $ \\angle CBE$ and $ \\angle ABC$ are the subtended and sector angles of chord $ AC$ in $ w_{2}$. $ \\angle AEF=\\angle ACF=x$, since both are subtended by chord $ AF$ in $ w_{2}$. So $ \\angle CED=30^\\circ+x$, and $ \\triangle DCE \\sim \\triangle CBI$ (AA). From $ \\triangle ACE$, $ CE=\\sqrt{3}r$. So $ \\frac{BI}{CE}=\\frac{BC}{CD}=\\frac{r}{2r}=\\frac{1}{2}$. So $ BI=\\frac{\\sqrt{3}}{2}r$. But from $ \\triangle CBD$, $ BD=\\sqrt{3}$. So $ I$ is the midpoint of $ BD$. So in $ \\triangle CBD$, $ CI$ and $ AB$ are medians, so $ AG=2BG$ as desired.", "Solution_8": "The construction with the centroid is so cool! I love it." } { "Tag": [ "algebra", "polynomial", "algebra proposed" ], "Problem": "hi,\r\nProve, if $ a,b,c \\in \\mathbb{R}$, $ a > 0$ and \r\n\\[ T(x) \\equal{} ax^{2} \\plus{} bcx \\plus{} b^{3} \\plus{} c^{3} \\minus{} 4abc,\\]\r\nhas complex roots, than one of the polynoms $ T_{1}(x) \\equal{} ax^{2} \\plus{} bx \\plus{} c,$ $ T_{2}(x) \\equal{} ax^{2} \\plus{} cx \\plus{} b,$ has only strict positive values.", "Solution_1": "[quote=\"doicu\"]hi,\nProve, if $ a,b,c \\in \\mathbb{R}$, $ a > 0$ and\n\\[ T(x) \\equal{} ax^{2} \\plus{} bcx \\plus{} b^{3} \\plus{} c^{3} \\minus{} 4abc,\n\\]\nhas complex roots, than one of the polynoms $ T_{1}(x) \\equal{} ax^{2} \\plus{} bx \\plus{} c,$ $ T_{2}(x) \\equal{} ax^{2} \\plus{} cx \\plus{} b,$ has only strict positive values.[/quote]\r\n$ T(x) \\equal{} ax^{2} \\plus{} bcx \\plus{} b^{3} \\plus{} c^{3} \\minus{} 4abc$ has complex roots,so \r\n$ b^2c^2\\minus{}4a(b^3\\plus{}c^3\\minus{}4abc)<0$\r\n<=>$ (b^2\\minus{}4ac)(c^2\\minus{}4ab)<0$\r\n=>$ b^2\\minus{}4ac<0$ or $ c^2\\minus{}4ab<0$\r\nNote that $ a>0$ so\r\nIf $ b^2\\minus{}4ac<0$ then $ T_{1}(x) \\equal{} ax^{2} \\plus{} bx \\plus{} c$ has only strict positive values\r\nIf $ c^2\\minus{}4ab<0$ then $ T_{2}(x) \\equal{} ax^{2} \\plus{} cx \\plus{} b$ has only strict positive values\r\nDone!", "Solution_2": "But every polynomial has complex roots!!", "Solution_3": "Abusedly so, here the proposer meant by [b]complex[/b] roots to say that the polynomial has [b]no real [/b]roots." } { "Tag": [ "LaTeX", "Gauss", "videos" ], "Problem": "Rules of the game:\r\n1) You must say three words, and only three words.\r\n2) The three words must make sense when connected to the three words above you.\r\n3) No sick/perverted things.\r\n\r\nOk I'll start\r\n\r\nOnce upon a...", "Solution_1": "time - or maybe", "Solution_2": "three penguins lived", "Solution_3": "together in a", "Solution_4": "house made of", "Solution_5": "circles cut out", "Solution_6": "When you say make sense, do you mean just grammatically correct or not weird.\r\n\r\nLike, am I allowed to say\r\n\r\nHe lick desks\r\n\r\n\r\nAnyway....\r\n\r\nof very tasty", "Solution_7": "of a pi\r\n\r\n(The message is too small. Please make the message longer before submitting.)", "Solution_8": "One day however.........", "Solution_9": "He saw a...\r\nAnd I mean it has to make sense with the three words above you.", "Solution_10": "hypothermic weasel with", "Solution_11": "A lot of...", "Solution_12": "underpayed nomad muffins.", "Solution_13": "and so he...", "Solution_14": "committed suicide gracefully.", "Solution_15": "Two households, both with yongyi781 in naughty positions with cranberry juice, went to the fair. At the fair, this bump is containing a baby. Yongyi meets tinytim over some food. Tinytim then challenges Yongyi to a views contest. Tinytim then decides that his avatar is really cool and shames Yongyi because he did not make a blueberry pie for the hobos. Yongyi takes out (^_^)'s blue shoes, and burns them. Then, laughing evilly, they started crying over the hills and tripped on gauss1181's head. Fortunately, pacman2812 was there playing soccer with myyellowducky82 and xoangieexo. xoangieexo kicks the ball into the pit next to the emo club that ends the scenario to receive payment for work at the emomelon where emo people dance hokey pokeys and make IMO. IMO was bloody because a bomb-exam with spikes exploded and no one was alive after. And then they, the emo-zombies, bit dannyhamtx and killed Me, so I, the very greatest zombie killed Lawrence Johnson because he did not gave really bad grammar book to his parents, therefore he", "Solution_16": "Two households, both with yongyi781 in naughty positions with cranberry juice, went to the fair. At the fair, this bump is containing a baby. Yongyi meets tinytim over some food. Tinytim then challenges Yongyi to a views contest. Tinytim then decides that his avatar is really cool and shames Yongyi because he did not make a blueberry pie for the hobos. Yongyi takes out (^_^)'s blue shoes, and burns them. Then, laughing evilly, they started crying over the hills and tripped on gauss1181's head. Fortunately, pacman2812 was there playing soccer with myyellowducky82 and xoangieexo. xoangieexo kicks the ball into the pit next to the emo club that ends the scenario to receive payment for work at the emomelon where emo people dance hokey pokeys and make IMO. IMO was bloody because a bomb-exam with spikes exploded and no one was alive after. And then they, the emo-zombies, bit dannyhamtx and killed Me, so I, the very greatest zombie killed Lawrence Johnson because he did not gave really bad grammar book to his parents, therefore he haves an F", "Solution_17": "Two households, both with yongyi781 in naughty positions with cranberry juice, went to the fair. At the fair, this bump is containing a baby. Yongyi meets tinytim over some food. Tinytim then challenges Yongyi to a views contest. Tinytim then decides that his avatar is really cool and shames Yongyi because he did not make a blueberry pie for the hobos. Yongyi takes out (^_^)'s blue shoes, and burns them. Then, laughing evilly, they started crying over the hills and tripped on gauss1181's head. Fortunately, pacman2812 was there playing soccer with myyellowducky82 and xoangieexo. xoangieexo kicks the ball into the pit next to the emo club that ends the scenario to receive payment for work at the emomelon where emo people dance hokey pokeys and make IMO. IMO was bloody because a bomb-exam with spikes exploded and no one was alive after. And then they, the emo-zombies, bit dannyhamtx and killed Me, so I, the very greatest zombie killed Lawrence Johnson because he did not gave really bad grammar book to his parents, therefore he haves an F in math exam", "Solution_18": "Two households, both with yongyi781 in naughty positions with cranberry juice, went to the fair. At the fair, this bump is containing a baby. Yongyi meets tinytim over some food. Tinytim then challenges Yongyi to a views contest. Tinytim then decides that his avatar is really cool and shames Yongyi because he did not make a blueberry pie for the hobos. Yongyi takes out (^_^)'s blue shoes, and burns them. Then, laughing evilly, they started crying over the hills and tripped on gauss1181's head. Fortunately, pacman2812 was there playing soccer with myyellowducky82 and xoangieexo. xoangieexo kicks the ball into the pit next to the emo club that ends the scenario to receive payment for work at the emomelon where emo people dance hokey pokeys and make IMO. IMO was bloody because a bomb-exam with spikes exploded and no one was alive after. And then they, the emo-zombies, bit dannyhamtx and killed Me, so I, the very greatest zombie killed Lawrence Johnson because he did not gave really bad grammar book to his parents, therefore he haves an F in math exam. Meanwhile, the world", "Solution_19": "Two households, both with yongyi781 in naughty positions with cranberry juice, went to the fair. At the fair, this bump is containing a baby. Yongyi meets tinytim over some food. Tinytim then challenges Yongyi to a views contest. Tinytim then decides that his avatar is really cool and shames Yongyi because he did not make a blueberry pie for the hobos. Yongyi takes out (^_^)'s blue shoes, and burns them. Then, laughing evilly, they started crying over the hills and tripped on gauss1181's head. Fortunately, pacman2812 was there playing soccer with myyellowducky82 and xoangieexo. xoangieexo kicks the ball into the pit next to the emo club that ends the scenario to receive payment for work at the emomelon where emo people dance hokey pokeys and make IMO. IMO was bloody because a bomb-exam with spikes exploded and no one was alive after. And then they, the emo-zombies, bit dannyhamtx and killed Me, so I, the very greatest zombie killed Lawrence Johnson because he did not gave really bad grammar book to his parents, therefore he haves an F in math exam. Meanwhile, the world is being attacked", "Solution_20": "Two households, both with yongyi781 in naughty positions with cranberry juice, went to the fair. At the fair, this bump is containing a baby. Yongyi meets tinytim over some food. Tinytim then challenges Yongyi to a views contest. Tinytim then decides that his avatar is really cool and shames Yongyi because he did not make a blueberry pie for the hobos. Yongyi takes out (^_^)'s blue shoes, and burns them. Then, laughing evilly, they started crying over the hills and tripped on gauss1181's head. Fortunately, pacman2812 was there playing soccer with myyellowducky82 and xoangieexo. xoangieexo kicks the ball into the pit next to the emo club that ends the scenario to receive payment for work at the emomelon where emo people dance hokey pokeys and make IMO. IMO was bloody because a bomb-exam with spikes exploded and no one was alive after. And then they, the emo-zombies, bit dannyhamtx and killed Me, so I, the very greatest zombie killed Lawrence Johnson because he did not gave really bad grammar book to his parents, therefore he haves an F in math exam. Meanwhile, the world is being attacked by mutant aliens", "Solution_21": "who hate pianogirl\r\n\r\nEDIT: Two households, both with yongyi781 in naughty positions with cranberry juice, went to the fair. At the fair, this bump is containing a baby. Yongyi meets tinytim over some food. Tinytim then challenges Yongyi to a views contest. Tinytim then decides that his avatar is really cool and shames Yongyi because he did not make a blueberry pie for the hobos. Yongyi takes out (^_^)'s blue shoes, and burns them. Then, laughing evilly, they started crying over the hills and tripped on gauss1181's head. Fortunately, pacman2812 was there playing soccer with myyellowducky82 and xoangieexo. xoangieexo kicks the ball into the pit next to the emo club that ends the scenario to receive payment for work at the emomelon where emo people dance hokey pokeys and make IMO. IMO was bloody because a bomb-exam with spikes exploded and no one was alive after. And then they, the emo-zombies, bit dannyhamtx and killed Me, so I, the very greatest zombie killed Lawrence Johnson because he did not gave really bad grammar book to his parents, therefore he haves an F in math exam. Meanwhile, the world is being attacked by mutant aliens who hate pianogirl", "Solution_22": "Two households, both with yongyi781 in naughty positions with cranberry juice, went to the fair. At the fair, this bump is containing a baby. Yongyi meets tinytim over some food. Tinytim then challenges Yongyi to a views contest. Tinytim then decides that his avatar is really cool and shames Yongyi because he did not make a blueberry pie for the hobos. Yongyi takes out (^_^)'s blue shoes, and burns them. Then, laughing evilly, they started crying over the hills and tripped on gauss1181's head. Fortunately, pacman2812 was there playing soccer with myyellowducky82 and xoangieexo. xoangieexo kicks the ball into the pit next to the emo club that ends the scenario to receive payment for work at the emomelon where emo people dance hokey pokeys and make IMO. IMO was bloody because a bomb-exam with spikes exploded and no one was alive after. And then they, the emo-zombies, bit dannyhamtx and killed Me, so I, the very greatest zombie killed Lawrence Johnson because he did not gave really bad grammar book to his parents, therefore he haves an F in math exam. Meanwhile, the world is being attacked by mutant aliens who hate pianogirl and destroy splatyango", "Solution_23": "Two households, both with yongyi781 in naughty positions with cranberry juice, went to the fair. At the fair, this bump is containing a baby. Yongyi meets tinytim over some food. Tinytim then challenges Yongyi to a views contest. Tinytim then decides that his avatar is really cool and shames Yongyi because he did not make a blueberry pie for the hobos. Yongyi takes out (^_^)'s blue shoes, and burns them. Then, laughing evilly, they started crying over the hills and tripped on gauss1181's head. Fortunately, pacman2812 was there playing soccer with myyellowducky82 and xoangieexo. xoangieexo kicks the ball into the pit next to the emo club that ends the scenario to receive payment for work at the emomelon where emo people dance hokey pokeys and make IMO. IMO was bloody because a bomb-exam with spikes exploded and no one was alive after. And then they, the emo-zombies, bit dannyhamtx and killed Me, so I, the very greatest zombie killed Lawrence Johnson because he did not gave really bad grammar book to his parents, therefore he haves an F in math exam. Meanwhile, the world is being attacked by mutant aliens who hate pianogirl and destroy splatyango because he/she", "Solution_24": "Two households, both with yongyi781 in naughty positions with cranberry juice, went to the fair. At the fair, this bump is containing a baby. Yongyi meets tinytim over some food. Tinytim then challenges Yongyi to a views contest. Tinytim then decides that his avatar is really cool and shames Yongyi because he did not make a blueberry pie for the hobos. Yongyi takes out (^_^)'s blue shoes, and burns them. Then, laughing evilly, they started crying over the hills and tripped on gauss1181's head. Fortunately, pacman2812 was there playing soccer with myyellowducky82 and xoangieexo. xoangieexo kicks the ball into the pit next to the emo club that ends the scenario to receive payment for work at the emomelon where emo people dance hokey pokeys and make IMO. IMO was bloody because a bomb-exam with spikes exploded and no one was alive after. And then they, the emo-zombies, bit dannyhamtx and killed Me, so I, the very greatest zombie killed Lawrence Johnson because he did not gave really bad grammar book to his parents, therefore he haves an F in math exam. Meanwhile, the world is being attacked by mutant aliens who hate pianogirl and destroy splatyango because he/she wants to die.", "Solution_25": "Then 1=2 decides", "Solution_26": "Two households, both with yongyi781 in naughty positions with cranberry juice, went to the fair. At the fair, this bump is containing a baby. Yongyi meets tinytim over some food. Tinytim then challenges Yongyi to a views contest. Tinytim then decides that his avatar is really cool and shames Yongyi because he did not make a blueberry pie for the hobos. Yongyi takes out (^_^)'s blue shoes, and burns them. Then, laughing evilly, they started crying over the hills and tripped on gauss1181's head. Fortunately, pacman2812 was there playing soccer with myyellowducky82 and xoangieexo. xoangieexo kicks the ball into the pit next to the emo club that ends the scenario to receive payment for work at the emomelon where emo people dance hokey pokeys and make IMO. IMO was bloody because a bomb-exam with spikes exploded and no one was alive after. And then they, the emo-zombies, bit dannyhamtx and killed Me, so I, the very greatest zombie killed Lawrence Johnson because he did not gave really bad grammar book to his parents, therefore he haves an F in math exam. Meanwhile, the world is being attacked by mutant aliens who hate pianogirl and destroy splatyango because he/she wants to die. But the aliens", "Solution_27": "Two households, both with yongyi781 in naughty positions with cranberry juice, went to the fair. At the fair, this bump is containing a baby. Yongyi meets tinytim over some food. Tinytim then challenges Yongyi to a views contest. Tinytim then decides that his avatar is really cool and shames Yongyi because he did not make a blueberry pie for the hobos. Yongyi takes out (^_^)'s blue shoes, and burns them. Then, laughing evilly, they started crying over the hills and tripped on gauss1181's head. Fortunately, pacman2812 was there playing soccer with myyellowducky82 and xoangieexo. xoangieexo kicks the ball into the pit next to the emo club that ends the scenario to receive payment for work at the emomelon where emo people dance hokey pokeys and make IMO. IMO was bloody because a bomb-exam with spikes exploded and no one was alive after. And then they, the emo-zombies, bit dannyhamtx and killed Me, so I, the very greatest zombie killed Lawrence Johnson because he did not gave really bad grammar book to his parents, therefore he haves an F in math exam. Meanwhile, the world is being attacked by mutant aliens who hate pianogirl and destroy splatyango because he/she wants to die. But the aliens were like wussup", "Solution_28": "um...\r\n\r\nwhy did you two revive a topic from almost a year ago when there's already a different 3 word game going on?", "Solution_29": "Two households, both with yongyi781 in naughty positions with cranberry juice, went to the fair. At the fair, this bump is containing a baby. Yongyi meets tinytim over some food. Tinytim then challenges Yongyi to a views contest. Tinytim then decides that his avatar is really cool and shames Yongyi because he did not make a blueberry pie for the hobos. Yongyi takes out (^_^)'s blue shoes, and burns them. Then, laughing evilly, they started crying over the hills and tripped on gauss1181's head. Fortunately, pacman2812 was there playing soccer with myyellowducky82 and xoangieexo. xoangieexo kicks the ball into the pit next to the emo club that ends the scenario to receive payment for work at the emomelon where emo people dance hokey pokeys and make IMO. IMO was bloody because a bomb-exam with spikes exploded and no one was alive after. And then they, the emo-zombies, bit dannyhamtx and killed Me, so I, the very greatest zombie killed Lawrence Johnson because he did not gave really bad grammar book to his parents, therefore he haves an F in math exam. Meanwhile, the world is being attacked by mutant aliens who hate pianogirl and destroy splatyango because he/she wants to die. But the aliens were like wussup and promptly ate" } { "Tag": [ "limit" ], "Problem": "Find $ lim \\sqrt{2\\plus{}\\sqrt{2\\plus{}\\sqrt{2\\plus{}...\\plus{}\\sqrt{2\\plus{}\\sqrt{2008}}}}}$", "Solution_1": "If it equals x, then $ x \\equal{} \\sqrt {2 \\plus{} x}$ hence x=2 :wink:", "Solution_2": "Oh! You will post full solution!", "Solution_3": "[quote=\"thanhnam2902\"]Find $ lim \\sqrt {2 + \\sqrt {2 + \\sqrt {2 + ... + \\sqrt {2 + \\sqrt {2008}}}}}$[/quote]\r\nI question what this problem means. Are you saying that the \"...\" implies an infinite continuation? If so, then I don't think the problem is possible. You're saying that in an infinite chain of $ \\sqrt{2+\\sqrt{2+...}}$, the chain ends with a $ \\sqrt{2008}$. Clearly, an infinite chain does not end, so this can't be true.\r\nI understand that a limit is being sought, but you're asking for an infinite limit of something finite and infinite at the same time (which is impossible).\r\nA more appropriate problem would be to find $ \\lim\\sqrt{2+\\sqrt{2+\\sqrt{2+...}}}$, with the \"...\" implying an infinite continuation. Either way, a $ \\sqrt{2008}$ would be meaningless.", "Solution_4": "How about the following:\r\nLet $ f(x)\\equal{}\\sqrt{2\\plus{}x}$. Find $ \\lim_{n\\to\\infty} f^n(\\sqrt{2008})$." } { "Tag": [ "Euler", "Recursive Sequences" ], "Problem": "The sequence $ \\{a_{n}\\}_{n \\ge 1}$ is defined by \\[ a_{1}=1, \\; a_{2}=2, \\; a_{3}=24, \\; a_{n}=\\frac{ 6a_{n-1}^{2}a_{n-3}-8a_{n-1}a_{n-2}^{2}}{a_{n-2}a_{n-3}}\\ \\ \\ \\ (n\\ge4).\\] Show that $ a_{n}$ is an integer for all $ n$, and show that $ n|a_{n}$ for every $ n\\in\\mathbb{N}$.", "Solution_1": "I take liberty to start sequence from 0 index:\r\n\\[ a_{0}=1, \\; a_{1}=2, \\; a_{2}=24, \\; a_{n}=\\frac{ 6a_{n-1}^{2}a_{n-3}-8a_{n-1}a_{n-2}^{2}}{a_{n-2}a_{n-3}}.\\]\r\n\r\nRewriting the recurrent relation, we have\r\n\\[ 4 a_{n-1}b_{n-1}= a_{n-3}b_{n},\\quad\\mbox{where}\\ b_{n}= a_{n}a_{n-2}-2 a_{n-1}^{2}.\\]\r\nIt follows from the recurrent relation for $ b_{n}$ that $ b_{n}= 2^{2n-1}a_{n-1}a_{n-2}$ and, thus:\r\n\\[ 2^{2n-1}a_{n-1}a_{n-2}= a_{n}a_{n-2}-2 a_{n-1}^{2}\\]\r\nor\r\n\\[ a_{n-2}c_{n}= 2 a_{n-1}c_{n-1}\\quad\\mbox{where}\\ c_{n}= 2^{2n}a_{n-1}-a_{n}.\\]\r\nIt follows from the recurrent relation for $ c_{n}$ that $ c_{n}= 2^{n}a_{n-1}$ and, thus:\r\n\\[ 2^{n}a_{n-1}= 2^{2n}a_{n-1}-a_{n}\\]\r\nor\r\n\\[ a_{n}= (2^{2n}-2^{n}) a_{n-1},\\]\r\nimplying that\r\n\\[ a_{n}= \\prod_{k=1}^{n}(2^{2k}-2^{k}) = 2^{\\frac{n(n+1)}{2}}\\prod_{k=1}^{n}(2^{k}-1),\\]\r\nwhich is obviously integer.\r\n\r\n[modedit: the $ n|a_{n}$ part has been added since this post, someone please solve it :)]", "Solution_2": "[quote=\"Peter\"]The sequence $ \\{a_{n}\\}_{n \\ge 1}$ is defined by\n\\[ a_{1} \\equal{} 1, \\; a_{2} \\equal{} 2, \\; a_{3} \\equal{} 24, \\; a_{n} \\equal{} \\frac { 6a_{n \\minus{} 1}^{2}a_{n \\minus{} 3} \\minus{} 8a_{n \\minus{} 1}a_{n \\minus{} 2}^{2}}{a_{n \\minus{} 2}a_{n \\minus{} 3}}\\ \\ \\ \\ (n\\ge4).\n\\]\nShow that $ a_{n}$ is an integer for all $ n$, and show that $ n|a_{n}$ for every $ n\\in\\mathbb{N}$.[/quote]\r\n$ v_n \\equal{} \\frac {a_n}{a_{n \\minus{} 1}}$\r\n$ v_1 \\equal{} 2,v_2 \\equal{} 12$\r\n$ v_n \\equal{} 6v_{n \\minus{} 1} \\minus{} 8v_{n \\minus{} 2}$\r\nThen we has $ v_n \\equal{} 4^n \\minus{} 2^n$\r\n$ a_n \\equal{} (4^n \\minus{} 2^n)a_{n \\minus{} 1}$\r\nSo $ a_n \\equal{} 2^{\\frac {n(n \\plus{} 1)}{2}}\\prod _{k \\equal{} 1}^n(2^k \\minus{} 1)$\r\nResult $ n|a_n$ follow from Euler theorem." } { "Tag": [], "Problem": "How many zeros are at the end of (100!)(200!)(300!) when multiplied out?", "Solution_1": "Post problems using the \"Ask on Forum\" feature, not before you've attempted it :wink:", "Solution_2": "We start by thinking of the ways we \"create\" zeros. This is achieved simply by noting that 2*5=10. Since clearly in the given product there are fewer fives than twos, we are counting the number of fives in the product.\r\n\r\nIn 100!, there are 100/5=20 + 4 (because each multiple of 25 produces 2 fives) = $ 24$ fives produced.\r\n\r\nIn 200!, there are 200/5=40 + 8 +1 (because $ 125\\equal{}5^3$ produces a 3rd five) = $ 49$ fives produced.\r\n\r\nFollowing a similar method, we find 300! = $ 74$ fives produced.\r\n\r\nTherefore, our final answer is simply $ 24\\plus{}49\\plus{}74\\equal{}\\boxed{147}$.\r\n\r\nHope this made sense!" } { "Tag": [], "Problem": "I was reading this short story and came up with two things that I don't understand. Please help (I used dictionary to help me but didn't get much out of it).\r\n\r\n1. What is the difference between creased and wrinkled? The sentence had something like \"He was creased but not wrinkled.\"\r\n\r\n2. What does this sentence mean? -> John considered with puckered brows.", "Solution_1": "1. To crease means to just fold. For example, when you mail a letter you crease the letter twice so it will fit in the envelope. To wrinkle is more like crumpling that letter. Creasing is more gentle, deliberate, and neat. Wrinkling is more rugged and disorganized.\r\n\r\n2. Well 'puckered brows' means that his eyebrows were kinda cringing/wrinkled. Generally, I think of 'pucker' (or 'puckered') in terms of a person eating a lemon.", "Solution_2": "[quote=\"Silverfalcon\"]I was reading this short story and came up with two things that I don't understand. Please help (I used dictionary to help me but didn't get much out of it).\n\n1. What is the difference between creased and wrinkled? The sentence had something like \"He was creased but not wrinkled.\"\n\n2. What does this sentence mean? -> John considered with puckered brows.[/quote]\r\n\r\nI think that in these instances, it is actually the connotation of the word(s) that is important, not the denotation. \r\n\r\nI'm not sure of the context, but I would guess that \"He was creased but not wrinkled\" means that he has some age/experience under his belt, but he isn't old. The word \"creased\" suggests that he's probably experienced some defining event at one point, leaving a permanent mark on him. On the other hand, \"wrinkled\" would suggest that he's old and worn. (It's just a guess; there's no way for me to know without the text in front of me.) \r\n\r\n\"Puckered\" in itself means that it has become \"gathered, contracted.\" In this case, his \"puckered brows\" probably suggest some sort of perplexity. (Another wild guess.)\r\n\r\nIf you give me more context, I can probably give you a better answer." } { "Tag": [ "induction", "USAMTS", "modular arithmetic", "LaTeX" ], "Problem": "What did you score?\r\n\r\nI got 5 [b]5[/b] 4 5 5.\r\n\r\nI missed number 3 because it wasn't in a closed form.", "Solution_1": "I got 22 (5 2 5 5 5).......\r\nno commended solution.. :(", "Solution_2": "24 55455 no commended. I got a point taken off of 3 'cause I subtracted the original vertices for some unknown reason.", "Solution_3": "I got a [b]5[/b] 3 5 [b]5 5[/b], and 23 composite\r\n\r\nI got three commended solutions: #1, #4, and #5\r\n\r\ni wish i got a 24 though so I could be on the leaderboard though :(", "Solution_4": "25, but none of them were commended.", "Solution_5": "I got 5 5 5 5 5.\r\nI'm really happy I got a 25 but not so happy I didn't get any comended solutions.", "Solution_6": "5 5 [b]5[/b] 5 5\r\n\r\nApparently, my solution for #4 was one the grader hadn't seen before. The comments for number three were \"Nice job on the induction!\"\r\nThe rest were just \"Good job\", etc.", "Solution_7": "I hate all of you.\r\n\r\n5 4 5 2 0", "Solution_8": "5 4 5 5 [b]5[/b]\r\nI'm so glad they only deducted 1 point for number 2.", "Solution_9": "25, no commended. I'm happy, especially since this is my first time doing the USAMTS. Now that I'm more comfortable with the problems, I'm aiming for commended solutions in later rounds.", "Solution_10": "5 4 5 1 -\r\n\r\nActually better than I expected! I thought that I would get an extra point or two off on number 2.", "Solution_11": "5 3 5 [b]5[/b] 5\r\n\r\n23 total, Better than I expected, thought I'd get a 1 on number 2. Has USAMTS put logged Entry Forms yet? They still haven't loggged mine.[/b]", "Solution_12": "Got a 25. Guess I better appeal...=s\r\n\r\n\r\n5 [b][color=darkred]5[/color][/b] [b][color=darkred]5[/color][/b] 5 5\r\n\r\n#3 I had a nice solution to that one, I kind of expected it.\r\n#2 was a surprise though - I thought that was one of my less-nice solutions (I mean, come on, I used casework!)", "Solution_13": "5+5+[color=red]5[/color]+[color=red]5[/color]+2 = 22 \r\nI'm not complainging since my solution to #5 was BS.", "Solution_14": "4 [b]5[/b] 5 5 5\r\n\r\nWow, I got commended on the one I thought I lost points on and lost a point for the one I didn't think I had to think about! Oh well, I stlil have a chance for gold! :D", "Solution_15": "[b]5 5 5 5 -[/b]\r\n\r\nSo yeah, I turned in only 4 problems, but all of them got commended.\r\n\r\nHooah.", "Solution_16": "5 [b]5[/b] 5 [b]5 5[/b]\r\n\r\nI guess I really don't have anything to complain about. Someday I'll get 5 commendations in one round though :P", "Solution_17": "25, no commended.\r\n\r\nThat's right.", "Solution_18": "[quote=\"Ciaccona\"]\nPerhaps you need an unusal approach in addition to a well-written solution? I got a commended, but I used a different method than the MathJam and everyone I know.[/quote]\r\n\r\nwoah....your solution owns :10:", "Solution_19": "i think i have a pretty unique #1... (aka no mods)\r\n\r\nLet the number we are looking for be $S$. We have that $S=d_{n}d_{n-1}d_{n-2}...d_{2}4$. Where each $d_{k}$ represents the digit of the decimal expansion of $S$. Let us construct a rational repeating decimal $N$ such that $N=0.d_{n}d_{n-1}d_{n-2}...d_{2}4d_{n}d_{n-1}...$. For the condition of the problem, we have that $4N$ must equal $0.4d_{n}d_{n-1}d_{n-2}...d_{3}d_{2}...$. \r\n\r\n\r\n\r\nThus, we have the following equations:\r\n\r\n$(1)$ $N=0.d_{n}d_{n-1}d_{n-2}...d_{2}4d_{n}d_{n-1}...$. \r\n\r\n$(2)$ $4N=0.4d_{n}d_{n-1}d_{n-2}...d_{3}d_{2}...$. \r\n\r\nLet $(3)$ be $10 \\cdot 4N=40N$. Thus, we now have:\r\n\r\n$(1)$ $N=0.d_{n}d_{n-1}d_{n-2}...d_{2}4d_{n}d_{n-1}...$. \r\n\r\n$(3)$ $40N=4.d_{n}d_{n-1}d_{n-2}...d_{2}4...$\r\n\r\n$(3)-(1)$ gives us $39N=4$ and $N=.102564102564102564...$. \r\n\r\nThe repeating part of $N$ is the smallest such number that we desire. Thus, $S=102564$.\r\n\r\nChecking, we see that $4 \\cdot 102564=410256$ and indeed satisfies our conditions.", "Solution_20": "55555, no commended but im happy.\r\nthis is the first time ive gotten a perfect.", "Solution_21": "[quote=\"mustafa\"]Yeah, I'm planning on getting a 25 with 5 commended solutions, so watch out....... :rotfl:[/quote]\r\n\r\nYeah we're going to bounce back from our mediocre scores and end up with 15 commended solutions at the end of the year...", "Solution_22": "i got 25, none commended. wow it seems like massive amounts of people got 25.", "Solution_23": "Sigh, I was hoping for at least one point on number 4, though I didn't anticipate getting 3 on number 2...", "Solution_24": "[quote=\"Iversonfan2007\"]i think i have a pretty unique #1... (aka no mods)[/quote]\r\n\r\nWow...that's a really, really nice solution.", "Solution_25": "[quote=\"perfectnumber628\"]i got 25, none commended. wow it seems like massive amounts of people got 25.[/quote]\r\nAs predicted... right after we turned in solutions, people started a poll about how they thought they did and something like 18 people predicted 25's... it was like 40% of the vote... and everyone said theres no way those people are right\r\nI think most of us were, though", "Solution_26": "[quote=\"me@home\"][quote=\"perfectnumber628\"]i got 25, none commended. wow it seems like massive amounts of people got 25.[/quote]\nAs predicted... right after we turned in solutions, people started a poll about how they thought they did and something like 18 people predicted 25's... it was like 40% of the vote... and everyone said theres no way those people are right\nI think most of us were, though[/quote]\r\nThat someone=me.\r\nI guess I was wrong. There have been 10 25s so far...", "Solution_27": "5,3,3,2,2\r\n\r\ntotal=15 :blush: \r\n\r\nThat was better than what I expected!!", "Solution_28": "I didn't even turn in a paper. I didn't have time to latex it up. :(", "Solution_29": "[quote=\"anirudh\"]I didn't even turn in a paper. I didn't have time to latex it up. :([/quote]\r\n\r\nYou should have just written it, that's what I did. And they gave you like 6 weeks. Anyway, what do you think you would have gotten? I'm curious because you're in middle school and want to know what middle schoolers got." } { "Tag": [ "inequalities", "inequalities unsolved" ], "Problem": "for all real $x,y,z$ prove or disprove that\r\n\r\n$x^4+y^4+z^4+3(x^2y^2+x^2z^2+y^2z^2)\\geq 2(x^3(y+z)+y^3(x+z)+z^3(x+y))$", "Solution_1": "It is a particular case of the more general inequality\r\n\r\n$\\sum x^4+r(r+2)\\sum y^2z^2 +(1-r^2)xyz\\sum x \\geq (r+1)\\sum yz(y^2+z^2)$,\r\n\r\nwhere $r$ is a real number.", "Solution_2": "$\\sum x^4+r(r+2)\\sum y^2z^2 +(1-r^2)xyz\\sum x \\geq (r+1)\\sum yz(y^2+z^2)\\Leftrightarrow$\r\n$(\\sum(x^2y^2-x^2yz))r^2+$\r\n$+(\\sum(2x^2y^2-x^3y-x^3z))r+\\sum(x^4-x^3y-x^3z+x^2yz)\\geq0.$ This is true since\r\n$(\\sum(2x^2y^2-x^3y-x^3z))^2-4(\\sum(x^2y^2-x^2yz))(\\sum(x^4-x^3y-x^3z+x^2yz))=$\r\n$=-3([6,2,0]-2[6,1,1]-2[4,4,0]+2[4,3,1]-2[3,3,2])=$\r\n$=-3(x-y)^2(y-z)^2(z-x)^2(x+y+z)^2.$ :)", "Solution_3": "[quote=\"shyong\"]for all real $x,y,z$ prove or disprove that\n$x^4+y^4+z^4+3(x^2y^2+x^2z^2+y^2z^2)\\geq 2(x^3(y+z)+y^3(x+z)+z^3(x+y))$[/quote]\r\n$x^4+y^4+z^4+3(x^2y^2+x^2z^2+y^2z^2)-2(x^3(y+z)+y^3(x+z)+z^3(x+y))=$\r\n$=(x^2+y^2+z^2-xy-xz-yz)^2=\\frac{1}{2}((x-y)^4+(y-z)^4+(z-x)^4).$ ;)", "Solution_4": "obviously, it's ture by muirhead inequality.", "Solution_5": "We may write also the inequality\r\n \r\n$\\sum x^4+r(r+2)\\sum y^2z^2 +(1-r^2)xyz\\sum x \\geq (r+1)\\sum yz(y^2+z^2)$,\r\n\r\nas\r\n\r\n$\\frac{1}{2}\\sum (y-z)^2(x+rx-y-z)^2 \\geq 0$." } { "Tag": [ "algebra", "system of equations" ], "Problem": "Find all triples $\\text{(x, y, z)}$ of integers that satisfy the following system of equations: \\[x^{3}-4x^{2}-16x+60 = y\\] \\[y^{3}-4y^{2}-16y+60 = z\\] \\[z^{3}-4z^{2}-16z+60 = x\\]", "Solution_1": "Abridged solution\n\n\n\n[hide]\n\n\n\nAdd four to each side, and factor, the first equation factors as , and the rest similarly. Multiply all together, and you get the solutions follow quickly from this if you remember that they must all be integers.\n\n\n\n[/hide]", "Solution_2": "Yeah I did the same." } { "Tag": [], "Problem": "\u039d\u03b1 \u03bb\u03c5\u03b8\u03b5\u03af \u03b7 \u03c4\u03c1\u03b9\u03b3\u03c9\u03bd\u03bf\u03bc\u03b5\u03c4\u03c1\u03b9\u03ba\u03ae \u03b5\u03be\u03af\u03c3\u03c9\u03c3\u03b7\r\n$ sin^{2008}(x)\\plus{}cos^{2009}(x)\\plus{}cos^{2}(x)\\equal{}2$", "Solution_1": "\u03bb\u03c5\u03c3\u03b7 \u03c3\u03b5 hidden\r\n[hide]\u03b7 \u03b5\u03be\u03b9\u03c3\u03c9\u03c3\u03b7 \u03b3\u03c1\u03b1\u03c6\u03b5\u03c4\u03b1\u03b9\n$ sin^{2008}{x} \\minus{} sin^2{x} \\equal{} 1 \\minus{} cos^{2009} {x}$\n\n$ \\leftrightarrow sin^2{x}( sin^{2006}{x} \\minus{} 1 ) \\equal{} 1 \\minus{} cos^{2009}{x}$\n\n\u03bf\u03bc\u03c9\u03c2,\n\n$ sin^2{x}( sin^{2006}{x} \\minus{} 1 ) \\leq0$\n\n$ 1 \\minus{} cos^{2009}{x} \\geq 0$\n\n\u03c0\u03bf\u03c5 \u03c3\u03b7\u03bc\u03b1\u03b9\u03bd\u03b5\u03b9 \u03bf\u03c4\u03b9 \u03b5\u03bc\u03b5\u03b9\u03c2 \u03b1\u03bd\u03b1\u03b6\u03b7\u03c4\u03bf\u03c5\u03bc\u03b5 \u03c4\u03b1 x \u03c4\u03b5\u03c4\u03bf\u03b9\u03b1 \u03c9\u03c3\u03c4\u03b5\n$ sin^2{x}( sin^{2006}{x} \\minus{} 1 ) \\equal{} 0$\n$ 1 \\minus{} cos^{2009}{x} \\equal{} 0$\n\n\u03c3\u03c5\u03c3\u03c4\u03b7\u03bc\u03b1 \u03bc\u03b5 \u03bb\u03c5\u03c3\u03b7 \n$ cosx \\equal{} 1 \\equal{} > x \\equal{} 2kp$ \u03ba \u03b1\u03ba\u03b5\u03c1\u03b1\u03b9\u03bf\u03c2, $ p \\equal{} 3.141...$[/hide]" } { "Tag": [ "number theory unsolved", "number theory" ], "Problem": "Find all $n\\in \\mathbb{N}$ such that:\r\n\\[\\frac{3^n-1}{2^n-1}\\in \\mathbb{N}\\]\r\nMaybe it is classical! ;)", "Solution_1": "Deleted", "Solution_2": "$7=6\\cdot 1+1|6\\cdot 2+2=14$. \r\n\r\n(it's meant to be a counterexample, jpsteenstra :))", "Solution_3": "I'm sorry to be wrong.", "Solution_4": "here is a proof using Jacobi Symbols and Jacobi Reciprocity: \r\n\r\nfor notational purposes, [tex]\\left(\\frac{A}{B}\\right)[/tex] is a Jacobi Symbol.\r\n\r\nNow for n = 1, it is obvious, so consider n > 1 and assume for the sake of contradiction, that for some n > 1, [tex]2^n-1|3^n-1[/tex]\r\n\r\nthen n must be odd, for if n were even, then 2^n-1 must be divisible by 3, while 3^n-1 is not.\r\n\r\nthen we have [tex]3^n-1\\equiv 0\\mod (2^n-1)[/tex] which implies [tex] 3^n\\equiv 1\\mod (2^n-1)[/tex] and so 3 must be a square [tex]\\mod (2^n-1) [/tex] since n is odd, and \r\n\r\n[tex]3^\\left(\\frac{n+1}{2}\\right)^2[/tex][tex] \\equiv 3 \\mod (2^n-1)[/tex] \r\n\r\n\r\ntherefore [tex]\\left(\\frac{3}{2^n-1}\\right) = 1[/tex] and since n is odd, we also have that [tex]\\left(\\frac{2^n-1}{3}\\right)=1[/tex]\r\n\r\nand by Jacobi Reciprocity:\r\n\r\n[tex]1=\\left(\\frac{3}{2^n-1}\\right)*\\left(\\frac{2^n-1}{3}\\right) = (-1)^{\\frac{2^n-2}{2}*\\frac{3-1}{2}} = -1[/tex] contradiction", "Solution_5": "I think another proof can be given using Newton's Formula for $(2+1)^n$ and $2^n$ but the results won't be probably so nice in appearence" } { "Tag": [ "algebra proposed", "algebra" ], "Problem": "The sequence $ a_1, a_2, a_3, ...$ satisfies $ a_{4n\\plus{}1} \\equal{} 1, a_{4n\\plus{}3} \\equal{} 0, a_{2n} \\equal{} a_n$. Show that it is not periodic.", "Solution_1": "[quote=\"tdl\"]The sequence $ a_1, a_2, a_3, ...$ satisfies $ a_{4n \\plus{} 1} \\equal{} 1, a_{4n \\plus{} 3} \\equal{} 0, a_{2n} \\equal{} a_n$. Show that it is not periodic.[/quote]\r\n\r\nAssume $ a_{n\\plus{}p}\\equal{}a_n$ $ \\forall n$ and for some period $ p>0$.\r\n\r\nLet then $ p\\equal{}2^aq$ with odd $ q$.\r\nLet then $ n\\equal{}2^{a\\plus{}2}(q\\plus{}2)$\r\n\r\nUsing periodicity, we have $ a_{n\\plus{}p}\\equal{}a_n\\equal{}a_{q\\plus{}2}$\r\nBut $ a_{n\\plus{}p}\\equal{}a_{2^a(q\\plus{}4(q\\plus{}2))}\\equal{}a_{q\\plus{}4(q\\plus{}2)}$\r\n\r\nBut $ q\\plus{}2$ and $ q\\plus{}4(q\\plus{}2)$ are two odd numbers different modulus 4. So $ a_{q\\plus{}2}\\neq a_{q\\plus{}4(q\\plus{}2)}$ (one is 0, the other is 1).\r\n\r\nAnd so no such $ p>0$ exists and the sequence is not periodic." } { "Tag": [ "real analysis", "real analysis unsolved" ], "Problem": "Find all $f: Q\\to R$ satisfying $f(xy)=f(x)f(y)$ and $f(x+y)\\le\\max(f(x),f(y))$", "Solution_1": "$f=0$ and $f=1$ are obvious solutions. Assume that $f$ is another solution. Then $f(1)=1,f(0)=0$; if $x\\not=0$ then $f(x)\\not=0$, $f(x)f(\\frac{1}{x})=1$; therefore $f(-1)\\in\\{-1,1\\}$ and if $p,q\\in\\mathbb{Z}^{*}$ then $f(\\frac{p}{q})=\\frac{f(p)}{f(q)}$. \r\ni) $\\forall{n}\\in\\mathbb{N}^{*}f(n)\\in]0,1]$.\r\nProof: $f(x_{1}+\\cdots+{x_{n}})\\leq Max\\{f(x_{1}),\\cdots,f(x_{n})\\}$ (easy). $00$ then $f(x)=1$, if $x<0$ then $f(x)=-1$,$f(0)=0$.\r\niii) $2^{nd}$ case: $f(-1)=1$.\r\n$\\forall{n}\\in\\mathbb{Z}^{*}f(n)\\in]0,1]$. Let $p,q\\in\\mathbb{N}^{*}$ s.t. $gcd(p,q)=1$; then $\\exists{a,b}\\in\\mathbb{Z}^{*}$ s.t. $ap+bq=1$; $1=f(ap+bq)\\leq{Max}\\{f(ap),f(bq)\\}\\leq{Max}\\{f(p),f(q)\\}$; therefore $f(p)=1$ OR $f(q)=1$.\r\nNecessarily $\\exists{n}\\in\\mathbb{N}^{*}$ s.t. $f(n)<1$ and there exists $u$ a prime divisor of $n$ s.t. $f(u)<1$.\r\nCONCLUSION:\r\nIf $u$ isn't a divisor of $m\\in\\mathbb{Z}^{*}$ then $f(m)=1$.\r\nIf $m=u^{r}v$ with $gcd(u,v)=1$ then $f(m)=f(u)^{r}$.\r\n Remark: this solution is convenient because if $l\\leq{k}$ with $l,k\\in\\mathbb{Z}$ and if $gcd(u,v_{i})=1;gcd(u,w_{i})=1;v_{i},w_{i}\\in\\mathbb{Z}^{*}$ then $f(u^{k}\\frac{v_{1}}{v_{2}}+u^{l}\\frac{w_{1}}{w_{2}})=f(u^{l}(u^{k-l}\\frac{v_{1}}{v_{2}}+\\frac{w_{1}}{w_{2}}))=f(u)^{l}f(u^{k-l}\\frac{v_{1}}{v_{2}}+\\frac{w_{1}}{w_{2}})\\leq{f(u)^{l}}=Max\\{f(u^{k}\\frac{v_{1}}{v_{2}}),f(u^{l}\\frac{w_{1}}{w_{2}})\\}$." } { "Tag": [ "absolute value" ], "Problem": "solve:\r\n$|q+7|-4=2$", "Solution_1": "This problem isn't to hard because the absolute value is always positive\r\n[hide]\n\n$ \\mid q + 7 \\mid - 4 = 2 $\n\nadd 4 on each side:\n\n$ \\mid q + 7 \\mid = 6 $\n\nSo:\n\n$ q = -1 $[/hide]", "Solution_2": "[quote=\"math92\"]solve:\n$|q+7|-4=2$[/quote]\r\n\r\n[hide]$|q+7|-4=2$\n\n$|q+7|=6$\n\n---------------------------------------------------------------\n$q+7=6$\n\n$q=-1$\n\n---------------------------------------------------------------\n\n$q+7=-6$\n\n$q=-13$[/hide]", "Solution_3": "[hide]$|q+7|-4=2$\n$|q+7|=6$\n$q+7=6$ or $q+7=-6$\n$q=-1$ or $-13$[/hide]", "Solution_4": "[hide]$|q-7|=6$\n\n$q-7=6$ or $q-7 = -6$\n\n$q=13, 1$[/hide]", "Solution_5": "[hide]$|q+7| -4 = 2$\n$|q+7| = 6$\n\n$q+7 = -6$\n$q=-13$\nor\n$q+7 = 6$\n$q=-1$[/hide]" } { "Tag": [ "vector", "linear algebra", "matrix" ], "Problem": "Let T be a linear transformation of an n dimensional vectorspace into itself such that T has n+1 eigen vectors, (every n of them are linearly independent). Prove that T is a multiple of the identity.", "Solution_1": "I don't really understand what you meant.\r\n\r\nFirst, $ T$ has $ n\\plus{}1$ eigenvectors and each $ n$ of them are linearly independent is equivalent to the fact that the matrix representing $ T$ is diagonalizable and one of the eigenvector is a multiple of one of the others.\r\n\r\nSo, I can take any diagonalizable matrix (just choose any diagonal matrix $ D$ and let $ T \\equal{} P^{\\minus{}1} D P$) and it's done: $ T$ satisfies your condition and is not a multiple of identity.", "Solution_2": "If one of the eigenvectors is a multiple of the others, then it's not true that any $ n$ of them are linearly independent.\r\n\r\nIt is true that $ T$ is diagonalizable - what can you deduce about its eigenvalues?" } { "Tag": [ "Ring Theory", "superior algebra", "superior algebra unsolved" ], "Problem": "[color=olive]Let f : R ! S be a homomorphism. Let T = { f(r) : r 2 R}.\nT is called the image of f. Show that T is a subring of S.\nHint: In this problem, you can use the following facts: f(0R) = 0S, f(\u2212x) = \u2212f(x)\nfor x 2 R, and f(x \u2212 y) = f(x) \u2212 f(y) for x, y 2 R. These facts were stated in\nclass for isomorphisms only but the same properties (with the same proofs) hold\nfor homomorphisms.[/color]\r\n\r\nThanks very much", "Solution_1": "Why does this belong to the olympiad section?" } { "Tag": [ "function", "algebra proposed", "algebra" ], "Problem": "Consider the function $ f(x) \\equal{} 2x \\minus{} sinx$. Prove that there exist a real number $ \\large b \\in R$ and functions $ g,h$ such that :\r\ni) $ \\large f(g(x)) \\equal{} x \\forall x \\in R$.\r\nii) $ h$ is a periodic function.\r\niii) $ g(x) \\equal{} b.x \\plus{} h(x) \\forall x \\in R$\r\n\r\nI have a nice solution, and I will become happy if someone give more solution, thank!!! (my English is so poor!) :blush:", "Solution_1": "Because f -bijection, g is bijection too. Therefore work $ b\\equal{}0,h\\equal{}g$.", "Solution_2": "Im' sorry.\r\nI've just edit that $ h$ is a periodic function.", "Solution_3": "Obviosly work $ b\\equal{}\\frac 12, h\\equal{}g\\minus{}\\frac 12 x$.", "Solution_4": "Yes, that right!\r\nBut I want a clear solution :blush: (my solution is so clearly!.. :blush: )", "Solution_5": "Dtrung, please give your post a relevant subject title. The title \"...\" does not say anything about your post." } { "Tag": [ "function", "limit", "real analysis", "real analysis unsolved" ], "Problem": "Let $ a\\geqslant 0$, $ x_1 \\equal{} 1 \\plus{} a$ and $ x_{n\\plus{}1} \\equal{} x_n \\plus{} \\frac{n}{x_n}$. \r\n1. Show that $ \\lim_{n\\to \\infty} n(x_n \\minus{} n)$ exists and is non negative. Denote that limit $ b$.\r\n2. Denote by $ f : \\mathbb{R}_\\plus{} \\to \\mathbb{R}_\\plus{}$ the function defined by $ b \\equal{} f(a)$. Is $ f$ continuous?", "Solution_1": "This is question Q650 in RMS Mars 2009 page 100\r\nthe author of this question is Alain Tissier\r\n\r\nIf any one of you have a solution you can send in PDF or TEX at \r\n\r\nal.tissier@laposte.net\r\n\r\nfor publication in RMS" } { "Tag": [ "number theory unsolved", "number theory" ], "Problem": "Show there are infinitely many non-primes in $ f(\\mathbb{N})$ for any $ f \\in \\mathbb{Z}[x]$ of degree $ \\geq 1$.\r\n\r\nWhat if $ f \\in \\mathbb{Z}[x_1,...,x_n]$", "Solution_1": "Let $ f$ is non constandt and $ A\\equal{}|f(x_1,x_2,...,x_n)|>1$, then exist prime p, suth that $ p|A$. \r\nIf $ deg_{x_i}f>0$, then for all k $ p|f(x_1,..,x_i\\plus{}kpp$.", "Solution_2": "$ f(x) | f(kf(x) \\plus{} x)$ for $ k \\in \\mathbb{Z}$." } { "Tag": [ "real analysis", "real analysis unsolved" ], "Problem": "Knowing that p + q = 2, proves that:\r\n\r\n \r\n$ \\ 3p^qq^p + p^pq^q \\le 4$\r\n\r\nI believe that this is impossible ( $ \\ p,q\\in \\Re$)", "Solution_1": "Not true. For $ p \\equal{} \\minus{} 2$ and $ q \\equal{} 4$ you get the value of 67.. However, it is true for $ p,q \\geq 0$.." } { "Tag": [ "group theory", "abstract algebra", "function", "number theory", "prime numbers", "superior algebra", "superior algebra unsolved" ], "Problem": "Let $p_{1},p_{2},...,p_{n}$ be prime numbers and $G$ a group such that $p_{i}|card(G), \\forall i=1,2,3,...,n$ such that for every $p_{i}$ there exist $x_{i},y_{i}\\in G$ where $y_{i}$ is not a power of $x_{i}$ such that $ord(x_{i})=ord(y_{i})=p_{i}$.\r\nProve that: $|G|\\geq 1+\\prod_{i=1}^{n}(p_{i}^{2}-1)$", "Solution_1": "For all $i=1,\\ldots,n$ let $H_{i}=\\langle x_{i}\\rangle$ and $K_{i}=\\langle y_{i}\\rangle$. \r\nWe have $|H_{i}|=|K_{i}|=p_{i}$ and since $y_{i}$ is not a power of $x_{i}$ and the primes $p_{i}$ are distinct we also have\r\n\r\n$i)$ $H_{i}\\cap K_{i}=(1)$ and \r\n\r\n$ii)$ $H_{i}\\cap H_{j}=K_{i}\\cap K_{j}=(1)$ for all $i\\neq j$.\r\n\r\nHence, there exists a one--to-one map from\r\n\r\n\\[\\prod_{i=1}^{n}(H_{i}\\times K_{i})\\setminus\\{(1,1)\\}\\longrightarrow G\\]\r\n\r\nSince $|H_{i}\\times K_{i}|=p_{i}^{2}$ we obtain $|G|\\geq 1+\\prod_{i=1}^{n}(p_{i}^{2}-1)$.", "Solution_2": "[quote=\"anenciu\"]\nHence, there exists a one--to-one map from\n\\[\\prod_{i=1}^{n}(H_{i}\\times K_{i})\\setminus\\{(1,1)\\}\\longrightarrow G \\]\n.[/quote]\r\nCan you kind show the one-to-one map explicitly? Thank you", "Solution_3": "I think I made it harder and unclearer than I wanted. The idea is that $A_{i}=H_{i}K_{i}$ is a subgroup of $G$ of order $p_{i}^{2}$ for all $i=1,\\ldots,n$ (here we use the fact that $y_{i}$ is not a power of $x_{i}$). Furthermore, for all $i\\neq j$ we have $A_{i}\\cap A_{j}=(1)$. So $G$ cotains subgroups $A_{1},\\ldots, A_{n}$ each of order $p_{i}^{2}$ and any two have in common just the identity. So, if we let $B_{i}=A_{i}\\setminus (1)$ then the $B_{i}$'s have no elemnts in comon and they have order $p_{i}^{2}-1$.Since $G$ is a group it will contain elements of the form $\\prod_{i=1}^{n}b_{i}$ where $b_{i}\\in B_{i}$ and 1 which is not of the above form (product of elements from B_i's). Hence, the conclusion follows now.", "Solution_4": "[quote=\"anenciu\"][...] The idea is that $A_{i}=H_{i}K_{i}$ is a subgroup of $G$ [...][/quote]\r\n\r\nThat is wrong in general.\r\n\r\nHere is another approach: Sylows theorem tells us, that the number of subgroups of order $p^{k}$ is congruent 1 mod p (provided $p^{k}$ is a divisor of $|G|$).\r\nEspecially we have at least $p_{i}+1$ subgroups of order $p_{i}$ (if there was only one, $y_{i}$ would be power of $x_{i}$). They contain together $(p_{i}-1)(p_{i}+1)$ elements of order $p_{i}$. So G has at least $\\prod_{i=1}^{n}{(p_{i}^{2}-1)}$ elements of prime order. Add the neutral element and we're done.", "Solution_5": "[quote]Here is another approach: Sylows theorem tells us, that the number of subgroups of order $p^{k}$ is congruent 1 mod p (provided $p^{k}$ is a divisor of $|G|$).\nEspecially we have at least $p_{i}+1$ subgroups of order $p_{i}$ (if there was only one, $y_{i}$ would be power of $x_{i}$). They contain together $(p_{i}-1)(p_{i}+1)$ elements of order $p_{i}$. So G has at least $\\prod_{i=1}^{n}{(p_{i}^{2}-1)}$ elements of prime order. Add the neutral element and we're done.[/quote]\r\nBut from what your wrote the elements is at least $1+\\sum_{i=1}^{n}{(p_{i}^{2}-1)}$...", "Solution_6": "Actually I think that Gockel's proof is very much the same like anenciu's one from the point he has shown that $G$ has $p_{i}^{2}-1$ elements of order $p_{i}$, we should find a injective function $\\prod A_{i}\\to G$ where $A_{i}=\\{ x\\in G | ord(x)=p_{i}\\}$", "Solution_7": "Maybe I'm blind :blush: ,But can somebody kindly show me it is an injective?", "Solution_8": "[quote=\"zhaobin\"][quote]Here is another approach: Sylows theorem tells us, that the number of subgroups of order $p^{k}$ is congruent 1 mod p (provided $p^{k}$ is a divisor of $|G|$).\nEspecially we have at least $p_{i}+1$ subgroups of order $p_{i}$ (if there was only one, $y_{i}$ would be power of $x_{i}$). They contain together $(p_{i}-1)(p_{i}+1)$ elements of order $p_{i}$. So G has at least $\\prod_{i=1}^{n}{(p_{i}^{2}-1)}$ elements of prime order. Add the neutral element and we're done.[/quote]\nBut from what your wrote the elements is at least $1+\\sum_{i=1}^{n}{(p_{i}^{2}-1)}$...[/quote]\r\n\r\nf***... \r\nYou're right, I proved that G contains at least the sum, not the product...", "Solution_9": "[quote=\"Gockel\"][quote=\"anenciu\"][...] The idea is that $A_{i}=H_{i}K_{i}$ is a subgroup of $G$ [...][/quote][/quote]\r\n\r\nThat is wrong in general.\r\n\r\nYou are right. I made a mistake thinking that $x_{i}$ and $y_{i}$ commmute and we DO NOT KNOW this for this problem. Sorry! I will have to think of another proof.", "Solution_10": "It's too late now, but there's no point wasting your time when the smallest possible counterexample works ($ A_4$ has 3 elements of order 2 and 8 of order 3, in a group of order 12).\r\n\r\nSee also [url=http://www.artofproblemsolving.com/Forum/viewtopic.php?t=172678]here[/url]." } { "Tag": [ "modular arithmetic", "algebra", "binomial theorem" ], "Problem": "Find the last two digits of N=11^10-9", "Solution_1": "[quote=\"mdk\"]Find the last two digits of N=11^10-9[/quote]\r\n[hide=\"Not Sure\"]$ 11^{1}=11$\n$ 11^{2}=121$\n$ 11^{3}=1331$\n$ 11^{4}=14641$ You see the pattern? Every time its $ 11^{n}$, the last two digits are going to be n, and 1. So, since its 10, its going to be 01. Subtracting that from nine, taking one from the hundreds place, we have the two last digits are 92. [/hide]", "Solution_2": "[hide]the last digit is always going to be 1\n\nsuppose the tens digit of $ 11^{n}$ is $ a$. \n\n...a 1\nX 1 1\n-----\n(a+1) 1 mod 100\n\nso it just adds one to the tens digit every time[/hide]", "Solution_3": "[hide]Binomial theorem:\n\n$ (10+1)^{10}=\\binom{10}{10}10^{10}+\\binom{10}{9}10^{9}+...+\\binom{10}{1}10^{1}+\\binom{10}{0}*10^{0}$\n$ \\equiv \\binom{10}{1}10^{1}+\\binom{10}{0}*10^{0}\\mod 100\\equiv 1$\n\nConsequently, $ 11^{10}-9\\equiv 1-9\\equiv \\boxed{92}\\mod 100$.\n\nThis is a common technique.[/hide]", "Solution_4": "[hide=\"A bit more work\"]$ 11^{2}\\equiv 21\\pmod{100}$\nSo $ 11^{4}\\equiv 21^{2}\\equiv 41\\pmod{100}$\n$ 11^{8}\\equiv 41^{2}\\equiv 81\\pmod{100}$\nthus $ 11^{10}\\equiv 11^{8}\\cdot 11^{2}\\equiv 81\\cdot 21\\equiv 1\\pmod{100}$\nHence $ 11^{10}-9\\equiv-8\\equiv \\boxed{92}\\pmod{100}$.[/hide]", "Solution_5": "[hide=\"Approach requiring the least effort and the most theory\"] $ gcd(n, 100) = 1 \\implies n^{20}\\equiv 1 \\bmod 100$ by Carmichael's Theorem. Hence we know that $ 11^{10}\\equiv 01, 99 \\bmod 100$. Because we also know that $ 11^{10}\\equiv 1 \\bmod 10$, it follows that \n\n$ 11^{10}\\equiv 01 \\bmod 100 \\implies$\n$ 11^{10}-9 \\equiv \\boxed{92}\\bmod 100$ [/hide]" } { "Tag": [ "induction", "number theory proposed", "number theory" ], "Problem": "$ a)$ Prove that there isn't any sequence $ a_1 0$. Take now $ a'_i \\equal{} \\frac {a_i} {a_i \\plus{} 1} \\prod_{k \\equal{} 1}^p (a_k \\plus{} 1)$ for all $ 1 \\leq i \\leq p$, and $ a'_{p \\plus{} 1} \\equal{} \\prod_{k \\equal{} 1}^p (a_k \\plus{} 1)$. We still have $ 0 < a'_1 < a'_2 < \\cdots < a'_p < a'_{p \\plus{} 1}$. Then, for $ 1 \\leq j < i \\leq p$ we have $ a'_i \\minus{} a'_j \\equal{} (\\frac {a_i} {a_i \\plus{} 1} \\minus{} \\frac {a_j} {a_j \\plus{} 1})\\prod_{k \\equal{} 1}^p (a_k \\plus{} 1) \\equal{} \\frac {a_i \\minus{} a_j} {(a_i \\plus{} 1)(a_j \\plus{} 1)}\\prod_{k \\equal{} 1}^p (a_k \\plus{} 1)$ and also $ a'_i \\plus{} a'_j \\equal{} (\\frac {a_i} {a_i \\plus{} 1} \\plus{} \\frac {a_j} {a_j \\plus{} 1})\\prod_{k \\equal{} 1}^p (a_k \\plus{} 1) \\equal{} \\frac {a_i \\plus{} a_j \\plus{} 2a_ia_j} {(a_i \\plus{} 1)(a_j \\plus{} 1)}\\prod_{k \\equal{} 1}^p (a_k \\plus{} 1)$. But $ a_i \\plus{} a_j \\plus{} 2a_ia_j \\equal{} (a_i \\plus{} a_j)(1 \\plus{} a_j) \\plus{} (a_i \\minus{} a_j)a_j$, divisible by $ a_i \\minus{} a_j$ by the induction hypothesis, therefore $ a'_i \\minus{} a'_j \\mid a'_i \\plus{} a'_j$.\nNow, for $ 1 \\leq i \\leq p$, we have $ a'_{p \\plus{} 1} \\minus{} a'_i \\equal{} (1 \\minus{} \\frac {a_i} {a_i \\plus{} 1})\\prod_{k \\equal{} 1}^p (a_k \\plus{} 1) \\equal{} \\frac {1} {a_i \\plus{} 1}\\prod_{k \\equal{} 1}^p (a_k \\plus{} 1)$, while $ a'_{p \\plus{} 1} \\plus{} a'_i \\equal{} (1 \\plus{} \\frac {a_i} {a_i \\plus{} 1})\\prod_{k \\equal{} 1}^p (a_k \\plus{} 1) \\equal{} \\frac {2a_i \\plus{} 1} {a_i \\plus{} 1}\\prod_{k \\equal{} 1}^p (a_k \\plus{} 1)$, so clearly $ a'_{p \\plus{} 1} \\minus{} a'_i \\mid a'_{p \\plus{} 1} \\plus{} a'_i$. Induction is complete. Since the sequence changes altogether when increasing the number of terms, there is no contradiction with the result at point a).\n[/hide]" } { "Tag": [ "geometry", "geometric transformation", "reflection", "circumcircle", "trigonometry", "homothety", "parallelogram" ], "Problem": "In an acute angled triangle $ABC$, $\\angle A = 30^{\\circ}$, $H$ is the orthocenter, and $M$ is the midpoint of $BC$. On the line $HM$, take a point $T$ such that $HM = MT$. Show that $AT = 2 BC$.", "Solution_1": "[quote=\"Rushil\"]In an acute angled triangle $ABC$, $\\angle A = 30 ^{\\circ}$, $H$ is the orthocenter, and $M$ is the midpoint of $BC$. On the line $HM$, take a point $T$ such that $HM = MT$. Show that $AT = 2 BC$.[/quote]\r\n\r\nSince HM = MT, the point T is the reflection of the point H in the point M. On the other hand, since M is the midpoint of the segment BC, the point C is the reflection of the point B in the point M. Since reflection in a point maps any line to a parallel line, we thus have CT || BH. But since H is the orthocenter of triangle ABC, we have $BH\\perp CA$, and thus $CT\\perp CA$. Thus, < ACT = 90\u00ba. Similarly, < ABT = 90\u00ba. Hence, the points C and B lie on the circle with diameter AT. In other words, the circle with diameter AT passes through the points A, B, C; thus, this circle is the circumcircle of triangle ABC. Hence, the segment AT is a diameter of the circumcircle of triangle ABC, so that we have AT = 2R, where R is the radius of the circumcircle of triangle ABC. But on the other hand, by the extended law of sines, $BC=a=2R\\sin A=2R\\sin 30^{\\circ}=2R\\cdot\\frac12=R$, so that $AT=2R=2\\cdot BC$. That's all.\r\n\r\n darij", "Solution_2": "good solution!", "Solution_3": "intresting problem thanks", "Solution_4": "Just for the end of the solution : instead of use of the law of sines, we have $\\angle BOC=2\\angle BAC$ (angle inscribed !) and then $\\angle BOC=60^{\\circ}$. Then triangle $BOC$ is equilateral, so $BC=BO=\\frac{AT}{2}$\n($O$ is the center of the cercle circumscribed to $ABC$.)", "Solution_5": "[quote=\"Rushil\"]In an acute angled triangle $ABC$, $\\angle A = 30^{\\circ}$, $H$ is the orthocenter, and $M$ is the midpoint of $BC$. On the line $HM$, take a point $T$ such that $HM = MT$. Show that $AT = 2 BC$.[/quote]\nWell, the homothety $H(H,2)$ take the nine point circle to the circumcircle, since the midpoints of $AH,BH,CH$ lie on the nine point circle. Also, it is well known that if $P$ is the midpoint of $AH$, then $PM$ is a diameter of the nine point circle. Therefore, $M$ goes to the point diametrically opposite to $A$. Thus, $AT = 2R$ and since $BC = R$ by easy law of sines, we're done. :)", "Solution_6": "My solution:\nATC is right triangle;\nTC=BH...(TBCH is a parallelogram)\nBH=2OE (O is circumcenter;E is midpoint of AC)\nOAE is a right traingle\nOE^2=R^2-1/4*b^2\nAT^2=AC^2+CT^2\n =b^2+4(OE^2)\n =b^2+4(R^2-b^2/4)\n =4R^2\nAT=2R", "Solution_7": "Let the circumcentre of $\\Delta ABC$be at the origin.If $z_1,z_2,z_3$ be the complex numbers representing $A,B,C$ respectively.Then, $H=z_1+z_2+z_3$ and $M=\\frac{z_2+z_3}{2}$.If $x$ represents the point $T$,then $\\frac{x+z_1+z_2+z_3}{2}=\\frac{z_2+z_3}{2}$\n$\\Rightarrow x=-z_1$.Therefore $AT=2R$\nSince $BC=2RSinA=R$.\n$AT=2BC$", "Solution_8": "$BTCH$ is a parallelogram so $\\angle TBM=\\angle MCH$,but $\\angle MCH =\\angle ABH$,so $BT$ is a tangent of $(ABD)${$D=AH \\cap BC$},so $AB$ is perpendicular to $BT$,so $AT$ is the diameter of $ABTC$, ($ABTC$ cyclic because $\\angle BTC=\\angle BHC$),also $BOC$ is regular($O$ is the circumcenter),so $BO=BC$,hence done.", "Solution_9": "BM=MC and HM=MT.\nso,BHCT is a parallelogram. \nlet X be a point on AC passes through the point H.\nangle BXC=90 degree.\nso, angle TCA is 90 degree.\nAnd Y is a point on AB also passes through point H.\nangle TYB=90 degree.\nso,angle TBA=90 degree\nNow we know that ABTC is cyclic.\nAT is the diameter of that circle.\nLet O be the circum center of that circle.so, angle BOC=2*angle BAC=60 degree.\nOB=OC. and angle BOC=60 degree.\nThen angle OBC=angle OCB=60 degree.\nNow,OBC is a equilateral triangle.\nAT=2*AO\nAO=BO=CO.,\nBC=OB=OC.\nso,AT=2*BC.(proved)", "Solution_10": "Take a point F to be the midpoint of AH.Then AT = 2FM. So the problem reduces to show FM = BC.Now consider the Circumcentre O of the Triangle.\nThen, 2OM = AH => OM = FH. Also angle chasing shows angle AMH = 90. Therefore, FM = FH = OM = BC (BM = MC = OM/2).So we are done.", "Solution_11": "It follows directly from EGMO lemma", "Solution_12": "Given, $HM=MT,\\quad BM=MC$.\nThus, if we join $HC,CT,TB,BH,$ the diagonals of quadrilateral $CTBH$ bisect each other $\\implies HCTB$ is a parallelogram.\nIn $\\Delta ABE, \\angle AEB= 90^{\\circ}$ (as $BE$ is altitude). By angle sum property, $\\angle ABE= 90^{\\circ}-A$.\nSimilarly in $\\Delta CEB$, $\\angle CBE=90^{\\circ}-C$. In $\\Delta FCB, \\angle FCB=90^{\\circ}-B$.\n\n$BH \\parallel CT$ (parallelogram), $HC\\parallel BT$\n$\\implies \\angle HCB=\\angle CBT=90^{\\circ}-B$ (alternate angles)\n$\\implies \\angle HBC=\\angle TCB=90^{\\circ}-C$ (alternate angles)\nTherefore, $\\angle ACT=270^{\\circ}-(A+B+C)=90^{\\circ}$\n$\\angle ABT= 90^{\\circ} \\implies ABTC$ is cyclic.\nAs $\\angle ACT=90^{\\circ}$, diameter= $AT=2R$. $T$ lies on the circumcircle of $\\Delta ABC$.\n$\\angle A=30^{\\circ}$\nBy sine rule, $\\frac{a}{\\sin A}=2R \\implies \\frac{BC}{\\sin 30^{\\circ}}=AT$\n\n$\\implies BC=AT/2 \\implies AT=2BC$. Hence, proved!", "Solution_13": "Observe that from the Orthocentre Reflection Lemma in EGMO Ch 1, $AT$ is nothing but the circumdiameter of $\\Delta ABC$. Now letting $R$ be the circumradius, and $a$ to be the length $BC$, we get $$2BC = 2a = 2 \\cdot (2R \\sin A) = 2 \\cdot \\left(2R \\sin \\frac{\\pi}{6} \\right) = 2R = AT,$$ so we are done. $\\square$" } { "Tag": [], "Problem": "Solve for x: $ 2^{x} \\equal{} x2^{x\\plus{}3}$", "Solution_1": "if you divide both sides by $ 2^x$, then wouldn't the problem reduce to\r\n$ 1\\equal{}x2^x\\minus{}3\\minus{}x$ because of division by exponents\r\n\r\nthis would reduce to $ 1\\equal{}x2^3$\r\nso $ 1\\equal{}x8$\r\nso $ x\\equal{}1/8$\r\n\r\nim gonna guess this is dead wrong, but im gonna go with it for now...", "Solution_2": "if you divide both sides by $ 2^x$, then wouldn't the problem reduce to\r\n$ 1\\equal{}x2^x\\minus{}3\\minus{}x$ because of division by exponents\r\n\r\nthis would reduce to $ 1\\equal{}x2^3$\r\nso $ 1\\equal{}x8$\r\nso $ x\\equal{}1/8$\r\n\r\nim gonna guess this is dead wrong, but im gonna go with it for now...", "Solution_3": "if you divide both sides by $ 2^x$, then wouldn't the problem reduce to\r\n$ 1\\equal{}x2^x\\minus{}3\\minus{}x$ because of division by exponents\r\n\r\nthis would reduce to $ 1\\equal{}x2^3$\r\nso $ 1\\equal{}x8$\r\nso $ x\\equal{}1/8$\r\n\r\nim gonna guess this is dead wrong, but im gonna go with it for now...", "Solution_4": "Triple post? Really?\r\n\r\nI have no idea what you did, but you're right. :huh:\r\n\r\n$ 1\\equal{}x\\cdot 2^{x\\minus{}x\\plus{}3}\\equal{}8x\\implies \\boxed{x\\equal{}\\frac{1}{8}}$.", "Solution_5": "holy crap...sry!\r\nhow did i do that? :huh:", "Solution_6": "Perhaps you did triple click on \"Submit\" button." } { "Tag": [ "geometry solved", "geometry" ], "Problem": "Just with compass (no ruler) find the center of a given circle.", "Solution_1": "See http://www.mathlinks.ro/Forum/viewtopic.php?t=24888 .\r\n\r\n darij", "Solution_2": "thanks mr.darij for answering this much fast(wow)" } { "Tag": [ "linear algebra", "matrix", "abstract algebra", "superior algebra", "superior algebra unsolved" ], "Problem": "prove that for any $n\\in \\mathbb{N}$ and prime number p,there exist a group with order $p^n$ such that its center contains $p$ or $p^2$ or $p^3$ members (i'm not sure that i translate center correctly).", "Solution_1": "Is the center the set $\\{x: xy=yx$ $\\forall y\\}$? If so, you translated correctly.\r\n\r\nHere is a construction:\r\nLet $G_m$ be the set of $m\\times m$ matrices $A$ over the field of $p$ elements satisfying $A=I+N$ with $N$ strictly upper-triangular. This is a group under matrix multiplication, and its center has order $p$. Also, $G_m$ has $p^{\\frac{m(m-1)}{2}}$ elements. For completeness, let $G_1$ be the trivial group.\r\n(The center of $G_m$ is the subset of matrices which are zero except on the diagonal and in the upper right corner.)\r\n\r\nBy a well-known result, we can express $n$ as a sum of at most three triangular numbers; $n=\\frac{a(a-1)}{2}+\\frac{b(b-1)}{2}+\\frac{c(c-1)}{2}$ (with the possibility that some of $a,b,c$ are $1$). Let $H_n=G_a \\times G_b \\times G_c$; then the center of $H_n$ is a product of one, two, or three cyclic groups each of order $p$.", "Solution_2": "Here's an improved result: for all $n>4$, there is a group with $p^n$ elements and $p$ elements in its center.\r\n\r\nAgain, consider $m \\times m$ matrices over the field of $p$ elements of the form $I+N$, where $N$ is strictly upper-triangular. We restrict this further; the potentially nonzero elements $a_{ij}$ of $N$ satisfy $f(i) \\le j$, with $f(2)=1$, $f(m)=m-1$, $f(j) < j$, and $f(j) \\le f(j+1)$. By adjusting $f$, we can give this group $p^n$ elements if $2m-1 \\le n \\le \\frac{m(m-1)}{2}$. In all cases, its center is the same group of $p$ elements.\r\nThe only unresolved case here is order $p^4$; groups of order $p^2$ are abelian so their center cannot have only $p$ elements, but that is not guaranteed for $p^4$." } { "Tag": [ "AMC", "AIME", "AIME II" ], "Problem": "Any idea what the cutoff will be this year for qualifying to AIME?", "Solution_1": "My guess is that the cutoff will be 68.", "Solution_2": "I hope it's lower than 68... if it is, then I have to get perfect scores on rounds 2 and 3 :(", "Solution_3": "Just take the AMC....it is [i]so[/i] much easier to qualify.", "Solution_4": "[quote=\"chess64\"]I hope it's lower than 68... if it is, then I have to get perfect scores on rounds 2 and 3 :([/quote]\r\n\r\nWhy not Round 4?", "Solution_5": "The deadline for the fourth round is March 13, and AIME and AIME II take place on March 7 and March 22, respectively.\r\n\r\nThe cutoff is determined by the combined score of the first 3 rounds.", "Solution_6": "Oh, so that's why it's so \"low.\" :roll:" } { "Tag": [ "inequalities proposed", "inequalities" ], "Problem": "For positive reals $a,b,c$ with $a^2+b^2+c^2=1$ prove:\r\n\\[ a+b+c+\\frac{1}{abc}\\geq 4\\sqrt{3} \\]", "Solution_1": "[hide]for AM-GM, we have\n\n$a+b+c+\\frac{1}{9abc}+\\frac{1}{9abc}+\\frac{1}{9abc}+\\frac{1}{9abc}+\\frac{1}{9abc}+\\frac{1}{9abc}+\\frac{1}{9abc}+\\frac{1}{9abc}+\\frac{1}{9abc}$ $\\geq12\\sqrt[12]{\\frac{1}{3^{18}a^8b^8c^8}}$ $=\\frac{4}{\\sqrt{3}\\sqrt[3]{a^2b^2c^2}}$\n\nit remains only to show that\n\n$\\frac{4}{\\sqrt{3}\\sqrt[3]{a^2b^2c^2}}\\geq4\\sqrt{3}$\n\n$\\frac{1}{3}\\geq \\sqrt[3]{a^2b^2c^2}$\n\n$\\frac{a^2+b^2+c^2}{3}\\geq \\sqrt[3]{a^2b^2c^2}$\n\ntrue for AM-GM.[/hide]\r\n\r\nbye bye", "Solution_2": "[quote=\"Yimin Ge\"]For positive reals $ a,b,c$ with $ a^{2}\\plus{}b^{2}\\plus{}c^{2}\\equal{} 1$ prove:\n\\[ a\\plus{}b\\plus{}c\\plus{}\\frac{1}{abc}\\geq 4\\sqrt{3}\\]\n[/quote]\r\nWe have $ abc\\leq\\frac{1}{3\\sqrt{3}}$ =>$ \\frac{1}{abc}\\geq 3\\sqrt{3}$\r\n$ a\\plus{}b\\plus{}c\\plus{}\\frac{1}{9abc}\\geq\\frac{4}{\\sqrt{3}}$\r\nDone :P !", "Solution_3": "[quote=Yimin Ge]For positive reals $a,b,c$ with $a^2+b^2+c^2=1$ prove:\n\\[ a+b+c+\\frac{1}{abc}\\geq 4\\sqrt{3} \\][/quote]\nBy AM-GM, we have\n$a+b+c+\\frac{1}{abc}\\geq12\\sqrt[12]{abc(\\frac{1}{9abc})^9}=\\frac{4\\sqrt{3}}{3\\sqrt[3]{a^2b^2c^2}}\\geq\\frac{4\\sqrt{3}}{a^2+b^2+c^2}=4\\sqrt{3}.$\n[url=http://www.artofproblemsolving.com/community/c6h611053p3632912]Strengthen of Macedonia 1999[/url]" } { "Tag": [ "function", "integration", "real analysis", "real analysis unsolved" ], "Problem": "Is there a family of continuous functions $f_n:[0,1]->[0,1]$ such that $\\int_{0}^{1}{f_n(x)dx}$ goes to 0 and the sequence $ (f_n(x))$ diverges for all $x$?", "Solution_1": "Of course, but it's much easier to draw than to explain. I'll modify this post if I decide to do it :)." } { "Tag": [ "geometry open", "geometry" ], "Problem": "prove that the sum of the reciprocals of the lengths of the altitudes of a triangle equals the reciprocal of the length of the inradius.", "Solution_1": "I don't know if you want complete geometric reasoning, but this is my proof in any case:\r\n\r\n[hide]$ \\frac{1}{h_a}\\plus{}\\frac{1}{h_b}\\plus{}\\frac{1}{h_c}\\equal{}\\frac{1}{2S}\\left(AB\\plus{}BC\\plus{}CA\\right)$\n\n$ \\equal{}\\frac{2s}{2S}\\equal{}\\frac{1}{r}$[/hide]" } { "Tag": [ "limit", "algebra unsolved", "algebra" ], "Problem": "Let $ \\lambda\\geq 1$ be a natural number. Find the limit\r\n\r\n$ \\lim\\limits_{n\\rightarrow \\infty}\\left(\\frac{1}{n}\\right)^{n\\lambda}\\plus{}\\left(\\frac{2}{n}\\right)^{n\\lambda}\\plus{}\\cdots\\plus{}\\left(\\frac{n\\minus{}1}{n}\\right)^{n\\lambda}.$", "Solution_1": "For $ \\lambda >0$ it equal $ \\frac{1}{e^{\\lambda}\\plus{}1}.$", "Solution_2": "No, answer is $ \\frac {1}{e^{\\lambda} \\minus{} 1 }$\r\n :roll:\r\n[hide=\"hint\"] approx ... http://www.mathlinks.ro/viewtopic.php?t=200869 [/hide]", "Solution_3": "Yes. \\[ \\sum_{k\\ge 1}e^{\\minus{}k\\lambda}\\equal{}\\frac{1}{e^{\\lambda}\\minus{}1}.\\]", "Solution_4": "[quote=\"Rust\"]Yes.\n\\[ \\sum_{k\\ge 1}e^{ \\minus{} k\\lambda} \\equal{} \\frac {1}{e^{\\lambda} \\minus{} 1}.\n\\]\n[/quote]\r\nand how can we use it to solve this problem ? :huh:" } { "Tag": [ "ARML", "analytic geometry" ], "Problem": "did anyone get this resolved?? Can you post the details as to how to get admitted in writing the SATs on the reserved day (and where?)", "Solution_1": "Try to talk to your guidance counselor; he should be able to help you if he wants, especially since it sounds like a lot of people from your school will be going to ARML. I didn't really get the situation resolved. I'm just taking it with another nearby team.", "Solution_2": "Yeah, I was planning on taking SATII Math on ARML day... I didn't exactly get it resolved, I'm just going to take this SAT in the fall. I don't know what grade you're in, but I guess this isn't really an option for you... try to see if it is, though (I mean, check other SAT dates...though you've probably done this already...)", "Solution_3": "unfortunately, tis a burden for all of us ARMLers...I was planning on taking chem this year :-(", "Solution_4": "Caroline, did you, or others from your school, try to talk to your guidance counselors? I'm pretty surprised AAST won't arrange something.", "Solution_5": "[quote=\"cats...\"]Caroline, did you, or others from your school, try to talk to your guidance counselors? I'm pretty surprised AAST won't arrange something.[/quote]\r\n\r\nWell as far as I know, it's not the school that organizes it, it's the testing board thingy... They mandate the dates the tests have to be administered and stuff. I'm not even sure which tests my school administers, as I'll probably be taking the regular SAT at my town high school which I live right next to. ;)", "Solution_6": "[quote]Well as far as I know, it's not the school that organizes it, it's the testing board thingy...[/quote]\r\n\r\nActually, the school coordinates the rescheduling with Collegeboard, so I would just ask your school about it (and contact Collegeboard, informing them of the conflict).", "Solution_7": "the college board does not reschedule SATs for conflicts unless they're religious or something...tough luck...u gotta take them next fall i guess...\r\n\r\nim taking mine this may (i.e. in 3 days.)", "Solution_8": "[quote=\"**********\"]the college board does not reschedule SATs for conflicts unless they're religious or something...tough luck...u gotta take them next fall i guess...\n\nim taking mine this may (i.e. in 3 days.)[/quote]\r\n\r\nNot true. Just ask any of the seasoned ARML takers around here.", "Solution_9": "[quote=\"**********\"]the college board does not reschedule SATs for conflicts unless they're religious or something...tough luck...u gotta take them next fall i guess...\n\nim taking mine this may (i.e. in 3 days.)[/quote]\r\n\r\nYep, Fiery is correct -- your school can reschedule for sanctioned academic conflicts as well, such as taking ARML. I happen to know this because I've taken a rescheduled SAT twice due to ARML, as have many other people who have been on the NYC team.", "Solution_10": "Ooh thanks, I wasn't aware of this - I'll talk to guidance tomorrow and see if they can do anything :)", "Solution_11": "hmmm...i still think ur wrong...\r\n\r\nMAYBE it's because i've already talked to both my school and Collegeboard and they specifically said there weren't any alt. testing dates (except like THREE MONTHS from now)... but i dunno, maybe both my college counselor and the collegeboard rep i talked to are utterly incompetent, which could very well be the case\r\n\r\nanyways, tell me how it goes for you...", "Solution_12": "How can people who have taken the test on an alternate date because of ARML be wrong about the possibility of getting it changed?", "Solution_13": "Maybe the rule has changed since then, or maybe it was just a special exception made for them.", "Solution_14": "[quote=\"gauss202\"]Maybe the rule has changed since then, or maybe it was just a special exception made for them.[/quote]\r\n\r\nThey're doing it again this year, unless I am sorely mistaken. I think they passed out forms about it when I was at NYSML. Special exception seems possible but rather unlikely, to be honest. Unfortunately, I don't have any knowledge of how to go about finding out about how NYC does it.", "Solution_15": "i was just told that no team can take it anymore. I was going to take it with the Eastern MA team, but the coach was just called by ETS today, saying that they can't give an alternate date anymore." } { "Tag": [ "inequalities", "LaTeX", "inequalities unsolved" ], "Problem": "Let $ a,b,c$ be non-negative numbers, such that $ a\\plus{}b\\plus{}c\\plus{}abc\\equal{}4$\r\n\r\nProve that: \r\n\\[ 3abc \\le ab \\plus{} bc \\plus{} ca \\le abc \\plus{} 4\\]", "Solution_1": "The first part is easy:\r\nBy AM-GM, we know $ \\frac {ab \\plus{} bc \\plus{} ca}{3}\\geqslant\\sqrt [3]{a^2b^2c^2}$ and it suffices to prove that $ abc\\leqslant 1$. This is clear from the condition because if $ abc > 1$, then also $ a \\plus{} b \\plus{} c > 3$ (from AM-GM), and, adding these two, $ a \\plus{} b \\plus{} c \\plus{} abc > 4$. Thus, $ abc > 1$ is impossible and the first part is proved. We have equality iff $ a\\equal{}b\\equal{}c\\equal{}1$.", "Solution_2": "My solution:\r\n\r\nFirst Part:\r\n\r\nBy AM-GM we have: \r\n $ \\frac{a\\plus{}b\\plus{}c\\plus{}abc}{4}\\geq\\sqrt[4]{(abc)^2}\\Rightarrow{1\\geq abc}$ (1)\r\n\r\nIn the inequality we have: \r\n\r\n $ 3\\leq\\frac{1}{a} \\plus{} \\frac{1}{b} \\plus{} \\frac{1}{c}$ or $ \\frac{3}{\\frac{1}{a} \\plus{} \\frac{1}{b} \\plus{} \\frac{1}{c}} \\leq \\sqrt[3]{abc}\\leq 1$ \r\n\r\nBy HM-GM and (1)", "Solution_3": "Solution to this problem\r\n[b]\nFirst Part[/b]\r\n\r\nBy using AM - GM inequality, we have\r\n\r\n$ ab \\plus{} bc \\plus{} ca \\ge 3\\sqrt [3]{a^2b^2c^2}$\r\n\r\nSo it suffices to prove that\r\n\r\n$ \\sqrt [3]{a^2b^2c^2} \\ge abc$ or $ latex abc \\le 1$\r\n\r\nBy using AM - GM inequality we have\r\n\r\n$ 1 \\equal{} \\dfrac{a \\plus{} b \\plus{} c \\plus{} abc}{4} \\ge \\sqrt [4]{a^2b^2c^2} \\Leftrightarrow abc \\le 1$\r\n\r\nThe proof is completed. Equality holds if and only if $ a \\equal{} b \\equal{} c \\equal{} 1$\r\n\r\n[b]Second Part[/b]\r\n\r\nWithout loss of generality, we may assume that $ c \\equal{} max${$ a,b,c$}. From the condition, it is clear that $ c \\ge 1$ because if $ c < 1$ then also $ a, b < 1$, and $ a \\plus{} b \\plus{} c \\plus{} abc < 4$ (which is a contradiction)\r\n\r\nBy using AM - GM inequality, we have\r\n\r\n$ 4 \\equal{} a \\plus{} b \\plus{} c(1 \\plus{} ab) \\ge 2\\sqrt {c(a \\plus{} b)(1 \\plus{} ab)}$\r\n\r\n$ \\Leftrightarrow c(a \\plus{} b)(1 \\plus{} ab) \\equal{} bc \\plus{} ca \\plus{} abc(a \\plus{} b) \\le 4$\r\n\r\n$ \\Leftrightarrow ab \\plus{} bc \\plus{} ca \\le 4 \\plus{} ab \\minus{} abc(a \\plus{} b)$\r\n\r\nSo it suffices to prove that\r\n\r\n$ ab \\minus{} abc(a \\plus{} b) \\le abc$\r\n\r\n$ \\Leftrightarrow ab[c(a \\plus{} b \\plus{} 1) \\minus{} 1] \\ge 0$ (which is true)\r\n\r\nThe proof is completed. Equality holds if $ (a,b,c) \\equal{} (0,2,2)$ or any cyclic permutation.", "Solution_4": "[quote=\"leviethai\"]Let $ a,b,c$ be non-negative numbers, such that $ a \\plus{} b \\plus{} c \\plus{} abc \\equal{} 4$\n\nProve that:\n\\[ 3abc \\le ab \\plus{} bc \\plus{} ca \\le abc \\plus{} 4\\]\n[/quote]\r\nThe second part can be strengthened. Actually, the stronger inequality holds:\r\n\r\nIf $ a,$ $ b,$ $ c$ are nonnegative real numbers such that $ a\\plus{}b\\plus{}c\\plus{}abc\\equal{}4,$ then\r\n$ ab\\plus{}bc\\plus{}ca \\le a\\plus{}b\\plus{}c.$\r\n\r\nThis inequality is equivalent to Vietnamese TST 1996 problem.", "Solution_5": "[quote=\"can_hang2007\"][quote=\"leviethai\"]Let $ a,b,c$ be non-negative numbers, such that $ a \\plus{} b \\plus{} c \\plus{} abc \\equal{} 4$\n\nProve that:\n\\[ 3abc \\le ab \\plus{} bc \\plus{} ca \\le abc \\plus{} 4\\]\n[/quote]\nThe second part can be strengthened. Actually, the stronger inequality holds:\n\nIf $ a,$ $ b,$ $ c$ are nonnegative real numbers such that $ a \\plus{} b \\plus{} c \\plus{} abc \\equal{} 4,$ then\n$ ab \\plus{} bc \\plus{} ca \\le a \\plus{} b \\plus{} c.$\n\nThis inequality is equivalent to Vietnamese TST 1996 problem.[/quote]\r\n\r\nThere is simple solution for this problem.\r\n\r\nAssume for the sake of contradiction that $ ab \\plus{} bc \\plus{} ca>a\\plus{}b\\plus{}c$. Using Schur:\r\n\\[ \\frac {9abc}{a \\plus{} b \\plus{} c}\\ge 4(ab \\plus{} bc \\plus{} ca) \\minus{} (a \\plus{} b \\plus{} c)^2 > abc(a \\plus{} b \\plus{} c)\\]\r\nHence $ a \\plus{} b \\plus{} c < 3$, but using again the condition you get $ a \\plus{} b \\plus{} c\\ge 3$.", "Solution_6": "[quote=\"can_hang2007\"][quote=\"leviethai\"]Let $ a,b,c$ be non-negative numbers, such that $ a \\plus{} b \\plus{} c \\plus{} abc \\equal{} 4$\n\nProve that:\n\\[ 3abc \\le ab \\plus{} bc \\plus{} ca \\le abc \\plus{} 4\\]\n[/quote]\nThe second part can be strengthened. Actually, the stronger inequality holds:\n\nIf $ a,$ $ b,$ $ c$ are nonnegative real numbers such that $ a \\plus{} b \\plus{} c \\plus{} abc \\equal{} 4,$ then\n$ ab \\plus{} bc \\plus{} ca \\le a \\plus{} b \\plus{} c.$\n\nThis inequality is equivalent to Vietnamese TST 1996 problem.[/quote]\r\n\r\nThanks to Can hang2007 to reveal my mistake :P :lol:", "Solution_7": "[quote=\"Abdek\"]\nOn the other hand we have the identity \n\n$ \\frac {(x \\plus{} y \\minus{} z)(y \\plus{} z \\minus{} x)}{xz} \\plus{} \\frac {(y \\plus{} z \\minus{} x)(z \\plus{} x \\minus{} y)}{xy} \\plus{} \\frac {(z \\plus{} x \\minus{} y)(x \\plus{} y \\minus{} z)}{yz} \\plus{} \\frac {(x \\plus{} y \\minus{} z)(y \\plus{} z \\minus{} x)(z \\plus{} x \\minus{} y)}{xyz} \\equal{} 4$[/quote]\nI think you have some mistakes. This identity is related to $ ab \\plus{} bc \\plus{} ca \\plus{} abc \\equal{} 4,$ while the original condition is $ a \\plus{} b \\plus{} c \\plus{} abc \\equal{} 4.$\n\n[quote=\"Pain rinnegan\"]Assume for the sake of contradiction that $ a \\plus{} b \\plus{} c > ab \\plus{} bc \\plus{} ca$. [/quote]\r\nThere is a little typo in your writing, Pain. It must be $ ab \\plus{} bc \\plus{} ca > a \\plus{} b \\plus{} c.$" } { "Tag": [ "number theory unsolved", "number theory" ], "Problem": "Let $ N \\equal{}$ {1,2,...}. Find all $ n \\in N$ such that :\r\nIf $ (x,n) \\equal{} 1$ then $ x^2 \\equiv 1 (mod n)$", "Solution_1": "If prime $ p|n$, then for any $ 0 0$, we have some $ r$ and $ s$ in $ T$ such that $ |r \\minus{} s| < \\delta$. It is easy to see that otherwise, $ T$ must be finite. Therefore, for any $ \\delta > 0$, there exists some $ t$ in $ T$ such that $ |t| < \\delta$. Since $ T$ is closed under addition and subtraction, we may add/subtract as many arbitrarily small real numbers as we like, so it follows easily that $ T$ is dense, as desired. (I am aware this isn't rigorous, but I'm slightly too lazy to get that done now.)\n\nTherefore, it is impossible for there to exist any interval $ ( \\minus{} \\epsilon, 0)$ where $ g$ has infinitely many roots. We must have that there exists some interval $ ( \\minus{} a, \\minus{} b)$ where $ g$ has no roots, $ a > b > 0$. Assume now for the sake of contradiction that there is some $ x \\in ( \\minus{} a, \\minus{} b)$ with $ g(x) \\neq 0$ such that $ g( \\minus{} x) \\equal{} x$. It follows by that that $ g( \\minus{} y) \\equal{} g(y)$ for all $ y$ in $ ( \\minus{} a, \\minus{} b)$, since $ g( \\minus{} y) \\equal{} \\pm g(y)$, and $ \\minus{} g(y)$ and $ g(x)$ have different signs, so by the intermediate value theorem, there must be some $ c$ in $ ( \\minus{} a, \\minus{} b)$ such that $ g(c) \\equal{} 0$, a contradiction. \n\nBut then, for any $ x$ in $ ( \\minus{} a, \\minus{} b)$, set $ y \\equal{} \\minus{} x$; we get that $ f(2x) \\equal{} f(x)^2 \\plus{} g(x)^2 \\equal{} 1$! Thus, $ f(x) \\equal{} 1$ for all $ x$ in $ ( \\minus{} a/2, \\minus{} b/2)$. Furthermore, $ g$ must be constant in this interval as well. We can easily use the original functional equation to show that since $ f$ and $ g$ are continuous (I don't even think that's necessary), $ f$ and $ g$ must be constant everywhere, a contradiction. \n\nThus, ina ny interval $ ( \\minus{} a, \\minus{} b)$ with $ a > b > 0$, and $ g( \\minus{} a) \\equal{} g( \\minus{} b) \\equal{} 0$, $ g( \\minus{} x) \\equal{} \\minus{} g(x)$ in that interval. This covers all the negative intervals, so we are done. \n\nAs you can see, it was a bit... long-winded, and it used some analysis. There's probably an easier way, and I'm really curious as to what it is. Any response would be appreciated. [/hide]", "Solution_1": "[hide]If all you wanted to was show $ g$ was odd, consider the given equation for $ (x,y)$ and $ (\\minus{}x,\\minus{}y)$. This implies $ g(x)g(y) \\equal{} g(\\minus{}x)g(\\minus{}y)$ (*). $ x \\equal{} y$ shows $ g(x) \\pm g(\\minus{}x)$. (*) also shows if we find $ g(x) \\equal{} \\minus{}g(\\minus{}x)$ (or $ g(x) \\equal{} g(\\minus{}x)$) for one $ x$ with $ g(x) \\neq 0$, then it holds for every $ x$.\n\nSo suppose $ g(x) \\equal{} g(\\minus{}x)$. Then $ f(2x) \\equal{} f(x)^2 \\plus{} g(x)^2$, so $ f(2x) \\equal{} 1$, contradicting $ f$ nonconstant. And so we get $ g$ odd.[/hide]", "Solution_2": "Darn. I now vow to myself never to use calculus on functional equations ever so I don't miss little jewels like this. \r\n\r\nThank you Palmer.", "Solution_3": "Where are the solutions $f(x)=cosax, g(x)=sinax$? :o ", "Solution_4": "OP said that if he could prove (which he could) that $g$ is odd, then $P(x,y)+P(x,-y)$ gives the D'alembert's functional equation, which, when combining with non-constant and ontinuous constraint, give solutions $f(x)\\equiv cos(ax)$ and $f(x)\\equiv cosh(ax)$, the latter ruled out as it's unbounded.\n", "Solution_5": "Yes, I want to see a solution where is proved this for continuous fonction only, because I know a proof only if $f$ is differentiable.", "Solution_6": "It's easy to prove by induction that any non-constant continuous function $f$ that satisfied the D'Alembert functional equation will also satisfied the equation $f(0)=1,$ $f(-x)=f(x),$ and $f(ny)=T_n(f(y))$ for all $n\\in\\mathbb{N},$ where $T_n$ is the Chebyshev polynomial of degree $n.$ In particular, $f(2x)=2f(x)^2-1.$\n\nSince $f$ is continuous, there is an $\\epsilon>0$ such that $f(x)>0$ for all $x$ such that $|x|<\\epsilon.$ Choose a random $k$ in the interval $(0,\\epsilon)$ and let $a=\\arccos(f(k))/k$ or $a=arccosh(f(k))/k$, whichever is defined. (From now on, we will assume that $ak=\\arccos(f(k))$ as the another case can be handled similary.)\n\nSince $f(x)>0$ for all $054$.", "Solution_1": "Check this link for minimum $ n\\ge 48$\r\n[url]http://www.mathlinks.ro/Forum/viewtopic.php?t=307546[/url]" } { "Tag": [ "analytic geometry", "graphing lines", "slope" ], "Problem": "1. a. How many photons (wavelength = 620 nm) must be absorbed to melt a 2.0-kg block of ice at 0 degrees C into water at 0 degrees C? \r\n\r\nb. On the average, how many H20 molecules does one photon convert from the ice phase to the water phase? \r\n\r\n2. An incident X-ray photon of wavelength 0.2800 nm is scattered from an electron that is initially at rest. The photon is scattered at an angle of :theta: = 180.0 degrees and has a wavelength of 0.2849 nm. Use the conservation of linear momentum to find the momentum gained by the electron. \r\n\r\nCan anyone provide some help on these 2 problems? Especially for the first one, do I have to set E = hf = mc (delta T)?", "Solution_1": "[quote=\"keta\"]1. a. How many photons (wavelength = 620 nm) must be absorbed to melt a 2.0-kg block of ice at 0 degrees C into water at 0 degrees C? \n\nb. On the average, how many H20 molecules does one photon convert from the ice phase to the water phase? \n\n2. An incident X-ray photon of wavelength 0.2800 nm is scattered from an electron that is initially at rest. The photon is scattered at an angle of :theta: = 180.0 degrees and has a wavelength of 0.2849 nm. Use the conservation of linear momentum to find the momentum gained by the electron. \n\nCan anyone provide some help on these 2 problems? Especially for the first one, do I have to set E = hf = mc (delta T)?[/quote]\r\n\r\nketa,\r\n\r\nI had thought I answered this yesterday, but apparently not. I shall try again.\r\n\r\n1. When water, or any other chemical compound, undergoes a phase change, e.g. solid-to-liquid or liquid-to-solid, its temperature does not change during the transition. One has to wait for the entire sample to undergo the transition before its temperature is free to change again. As such, the regions of the Energy vs. Temperature graph where the slope is zero, are called latent heats; the solid/liquid transition, the latent heat of fusion ($L_f$), and the liquid/gas transition, the latent heat of vaporization ($L_v$). [b]Latent heats are dependent only upon the mass of a given sample, not its temperature.[/b]\r\n\r\nThe latent heat of fusion for water is: $L_f = 6.013 kJ/mol$.\r\n\r\nThe trick here is to equate the photon energy with the latent heat of fusion.\r\n\r\n2. Problem #2 is an example of [url=http://hyperphysics.phy-astr.gsu.edu/hbase/quantum/comptint.html]Compton scattering[/url].\r\n\r\nI hope this helps." } { "Tag": [ "inequalities", "inequalities unsolved" ], "Problem": "Let $m\\geq n\\geq1$ be real number. Prove or disprove that: for any $a,b,c>0$ then we must have \r\n\\[ \\frac{(a^m+b^m)(b^m+c^m)(c^m+a^m)}{(abc)^m}\\geq \\frac{(a^n+b^n)(b^n+c^n)(c^n+a^n)}{(abc)^n} \\]\r\n\r\nHow about $1\\geq m\\geq n\\geq0$ ?", "Solution_1": "By multiplying it out you get:\r\n\\[ \\sum_{sym}a^{2m+n}b^{m+n}c^n\\geq\\sum_{sym}a^{2n+m}b^{n+m}c^m \\]\r\nWhich is true by Muirhead (sorry, whats the correct spelling of this name?), cause $2m+n,m+n,n$ majorize $2n+m,n+m,m$ :)" } { "Tag": [ "\\/closed" ], "Problem": "According to the Wiki, the words of the week for March 1-8 are \"Gravity\" and \"AMC 8.\" However, the forum says the words are still \"Momentum, and CTY.\"", "Solution_1": "My fault; I fixed it." } { "Tag": [], "Problem": "3 books on maths 4 books on physics , 6 books on chemistry, ,, how many sets of books can be choosen so that each set has 2 books of different subjects", "Solution_1": "[quote=\"nust2007\"]3 books on maths 4 books on physics , 6 books on chemistry, ,, how many sets of books can be choosen so that each set has 2 books of different subjects[/quote]\r\n\r\n[hide]he has to choose 1 from two categories so $ 3\\cdot 4+4\\cdot 6+3\\cdot 6=54$.[/hide]", "Solution_2": "[quote=\"nust2007\"]3 books on maths 4 books on physics , 6 books on chemistry, ,, how many sets of books can be choosen so that each set has 2 books of different subjects[/quote]\r\nquick question are the physics (and the other subject) books dictinct from eachother?", "Solution_3": "I got a different answer for this..\r\nI assumed they were different because it said the books were \"on math\" so they can be different from eachother." } { "Tag": [ "function", "algebra proposed", "algebra" ], "Problem": "$ f: Z \\to R$ is a function such that :\r\n \\[ f(n) = \\{\\begin{array}{cc}n - 1, & \\mbox{if}n > 100 \\\\\r\nf(f(n + 1)) , & \\mbox{ if } n \\leq 100 \\end{array}\r\n\\]\r\nprove : $ \\forall 100 \\geq n$ f(n)=91", "Solution_1": "[quote=\"stvs_f\"]$ f: Z \\to R$ is a function such that :\n\\[ f(n) = \\{\\begin{array}{cc}n - 1, & \\mbox{if}n > 100 \\\\\nf(f(n + 1)) , & \\mbox{ if } n \\leq 100 \\end{array}\n\\]\nprove : $ \\forall 100 \\geq n$ f(n)=91[/quote]\r\n\r\ni think you forget somthing,\r\nif we take $ f(n)=n-1: \\ \\forall n\\in Z$ we have $ f(n) = \\{\\begin{array}{cc}n - 1, & \\mbox{if}n > 100 \\\\\r\nf(f(n + 1)) , & \\mbox{ if } n \\leq 100 \\end{array}$\r\n\r\n$ f(n)\\neq 91$ for $ n<80$", "Solution_2": "why $ f(n) \\neq 91$", "Solution_3": "[quote=\"stvs_f\"]why $ f(n) \\neq 91$[/quote]\r\n$ g(n)\\equal{}n\\minus{}1$ for all $ n\\in N$ is solution,because $ g(n)\\equal{}gog(n\\plus{}1)$\r\nbut for exemple $ g(79)\\equal{}78\\neq 91$" } { "Tag": [], "Problem": "Does string theory touted as TOE offer a possible explanation of what happens beyond the event horizon?\r\n\r\n\r\nIf not what may?", "Solution_1": "hep-th archive (arxiv.org) has papers/proposals by Samir Mathur on this.", "Solution_2": "Thank's for the location of the article." } { "Tag": [ "geometry", "AMC", "AIME", "USA(J)MO", "USAMO" ], "Problem": "Are there any other books besides Volume 1/2 that can help me with AIME-level geometry?", "Solution_1": "what do people think of geometry revisited?", "Solution_2": "It seems more like an olympiad preperation book. Also, the material covered doesn't seem to come up too often in competitions. Although, if you just want to learn more geometry, not necessarily for competitions, it's a great book.\r\n\r\nChallenging problems in geometry is a good book if you're looking for problems. It doesn't cover much material by itself however. I haven't looked at in a while; it's probably somewhere in my room, mixed in with all the books and paper on the floor :P \r\n\r\nOh, and my opinion doesn't matter much, since I haven't actually worked through most, or indeed, much, of these books, since I thought that geometry revisited was too challenging for me at the time (a few months back) and I guess I was too lazy to do CPIG. Maybe I should actually get back to work on them....", "Solution_3": "[quote=\"Jongao\"]Are there any other books besides Volume 1/2 that can help me with AIME-level geometry?[/quote]\r\n\r\nIntroduction to Geometry? AMC-easy AIME level, I think. Some MC level too.", "Solution_4": "For me, I'm just going to do the online geometry course that AoPS offers on Feb 4.\r\nThis is a good preparation for AIME / USAMO\r\n\r\n( I hope I make USAMO... )", "Solution_5": "Hmm... if you want a geometry book that focuses on OLYMPIADS, there is a stickied link to Kiran Kedlaya's Geometry Unbound in the Olympiad Geometry Thereoms forum." } { "Tag": [ "AMC", "AIME" ], "Problem": "Ruth picks pairs of numbers from the set $ S$ which is : $ S \\equal{} [1,2,3,4,5,6,7,8,9,10]$ and writes in her notebook the biggest element of the pair that she chose. After picking every possible pairs (without repeating any), Ruth added all the numbers she wrote. What is the answer that she got? \r\n\r\nI need a help on where to start with this problem...", "Solution_1": "Try to construct pairs in an orderly manner. You'll see a pattern.", "Solution_2": "It's weird. I keep trying to do it, yet, my answer is not in the options...\r\n\r\nI keep getting $ 325$\r\n\r\nand the answer options are:\r\n\r\n$ (a)$ $ 250$ $ (b)$ $ 330$ $ (c)$ $ 350$ $ (d)$ $ 430$ $ (e)$ $ 450$\r\n\r\nAnyone can do this please?", "Solution_3": "[hide=\"Solution\"]\nRuth can choose (1,2)~(1,10), (2,3)~(2,10), ..., (8,9)~(8,10), (9,10)\n\nRuth will have 9 10's, 8 9's, ..., and 1 2's in her notebook.\n\nThus, the sum will be $ 90\\plus{}72\\plus{}56\\plus{}42\\plus{}30\\plus{}20\\plus{}12\\plus{}6\\plus{}2\\equal{}330$\n\nAnswer: b(330)[/hide]", "Solution_4": "Good solution FantasyLover. xcenario, this is what I meant by constructing pairs in an orderly manner and counting them.\r\n\r\n-rts2007", "Solution_5": "This is almost the exact same problem as AIME 2003, Problem 3." } { "Tag": [ "ARML" ], "Problem": "Compute the number of positive integers that have the same number of digits expressed in base 3 and base 5.", "Solution_1": "For a given number of digits, the lower bound is dictated by powers of 5 and the upper bound is dictated by powers of 3.\r\n\r\nOne digit: $[5^{0},3^{1}) \\to \\{ 1,2 \\}$\r\nTwo digits: $[5^{1},3^{2}) \\to \\{ 5,6,7,8 \\}$\r\nThree digits: $[5^{2},3^{3}) \\to \\{ 25,26 \\}$\r\nFour digits: $5^{3}>3^{4}$, so nothing with 4 digits or above.\r\n\r\nThe intersection set, $\\{ 1,2,5,6,7,8,25,26 \\}$, has $\\boxed{8}$ elements.", "Solution_2": "This was number 1 on the ARML Individual Round this year." } { "Tag": [ "calculus", "integration", "logarithms", "trigonometry", "real analysis", "real analysis unsolved" ], "Problem": "find:\r\n$ \\int_{0}^{\\infty}\\frac{\\ln(x) \\cos(x) \\plus{}\\frac{\\pi}{2}\\sin(x)}{\\ln^{2}x\\plus{}\\frac{\\pi^{2}}{4}}dx$", "Solution_1": "I'm just playing with ideas here, I don't have an answer yet...\r\n\r\nIt almost looks like a trig substitution problem...\r\n$ \\ln(x) \\equal{} \\frac{\\pi}{2}\\tan(\\theta)$\r\n$ x \\equal{} e^{\\frac{\\pi}{2}\\tan(\\theta)}$\r\n$ dx \\equal{} e^{\\frac{\\pi}{2}\\tan(\\theta)}\\frac{\\pi}{2}\\sec^{2}(\\theta)d\\theta$\r\nSubstituting and simplifying:\r\n$ \\int\\frac{\\frac{\\pi}{2}\\tan(\\theta) \\cos(e^{\\frac{\\pi}{2}\\tan(\\theta)}) \\plus{}\\frac{\\pi}{2}\\sin(e^{\\frac{\\pi}{2}\\tan(\\theta)})}{(\\frac{\\pi}{2}\\tan(\\theta))^{2}\\plus{}\\frac{\\pi^{2}}{4}}e^{\\frac{\\pi}{2}\\tan(\\theta)}\\frac{\\pi}{2}\\sec^{2}(\\theta)d\\theta$\r\n$ \\equal{} \\int (\\tan(\\theta) \\cos(e^{\\frac{\\pi}{2}\\tan(\\theta)}) \\plus{}\\sin(e^{\\frac{\\pi}{2}\\tan(\\theta)})e^{\\frac{\\pi}{2}\\tan(\\theta)})d\\theta$\r\nBecause:\r\n$ tan^{2}(\\theta)\\plus{}1 \\equal{} sec^{2}(\\theta)$\r\n\r\nThat will get rid of the denominator, but you can see things turn ugly in the cos(x) and sin(x).", "Solution_2": "I don't think that there exist solution without complex analysis. :(" } { "Tag": [ "modular arithmetic", "function", "arithmetic sequence", "number theory", "prime numbers" ], "Problem": "Prove that, if the terms of an infinite arithmetic progression of natural numbers are not all equal, they cannot all be primes.", "Solution_1": "$ \\Delta a_{n}: \\equal{} d\\implies a_{n}\\equal{} a_{0}\\plus{}(n\\minus{}1)d\\implies a_{(a_{0}\\plus{}1)}\\equal{} (1\\plus{}d) a_{0}\\equiv 0\\pmod{1\\plus{}d}$\r\n\r\nThus, if $ d\\in\\mathbb{N}$, $ a$ cannot be a sequence of primes only.", "Solution_2": "[quote=\"mdk\"]Prove that, if the terms of an infinite arithmetic progression of natural numbers are not all equal, they cannot all be primes.[/quote]\r\n\r\n[hide=\"eh.\"]\nLet $ p$ be a prime and let it be the first term of our progression, with $ d$ being the common difference. Then the $ p\\plus{}1$st terms of the sequence is $ p\\plus{}pd\\equal{}p(d\\plus{}1)$ which is composite since $ d$ is not 0 (which would make all terms equal)\n[/hide]", "Solution_3": "Interestingly enough, there do exist arbitrarily long arithmetic progressions of primes, so here is an interesting question:\r\n\r\nWhat constraints on $ d$ are necessary for an arithmetic progression of primes of length at least $ L$?", "Solution_4": "$ d > A(69,L)$ should be satisfactory.", "Solution_5": "[quote=\"Lazarus\"]$ d > A(69,L)$ should be satisfactory.[/quote]\r\n\r\nHe said necessary, not sufficient.", "Solution_6": "[quote=\"Lazarus\"]$ d > A(69,L)$ should be satisfactory.[/quote]\r\n\r\nWhat does the $ A(69,L)$ mean?", "Solution_7": "He's probably talking about the [url=http://mathworld.wolfram.com/AckermannFunction.html]Ackermann Function[/url]. In any case, the problem I proposed is not terribly difficult; try working it out for small examples, say $ L \\equal{} 3, 4, 5$.", "Solution_8": "[hide=\"solution\"]Say that the first term is a, and the common difference is d.\n\nIf a\u22601, then the a+1st term is a(1+d), which is composite.\nIf a=1, it works for d=1. If d=k+1, where k\u22651, then the k-1st term is k^2, which works. So if the terms of an infinite arithmetic progression of natural numbers are not all equal, they cannot all be primes.[/hide]", "Solution_9": "[quote=\"mdk\"]Prove that, if the terms of an infinite arithmetic progression of natural numbers are not all equal, they cannot all be primes.[/quote]\n[hide=\"Solution\"]If $ a$ and $ d$ are respectively the first term and the common difference, then the $ (a\\plus{}1)$-th term is $ a\\plus{}(a\\plus{}1\\minus{}1)d \\equal{} a(d\\plus{}1)$, which cannot be prime for $ a > 1$. On the other hand, if $ a \\equal{} 1$ then the sequence is $ 1,d\\plus{}1,2d\\plus{}1,3d\\plus{}1,\\dots$, of which the $ (d\\plus{}2)$-th term is $ (d\\plus{}2)d\\plus{}1 \\equal{} (d\\plus{}1)^{2}$, which is not prime either. Hence the proof is complete.[/hide] \n\n[quote=\"t0rajir0u\"]What constraints on $ d$ are necessary for an arithmetic progression of primes of length at least $ L$?[/quote]\n\n$ \\gcd(d,L) > 1$. Is this correct? :maybe: \n\n[hide=\"Proof\"] It is well known (and can be proven easily) that if $ (d,L) \\equal{} 1$ then the numbers $ d,2d,3d,\\dots, (L\\minus{}1)d$ are all non-congruent modulo $ L$. Thus $ a,a\\plus{}d,a\\plus{}2d,\\dots, a\\plus{}(L\\minus{}1)d$ are also non-congruent modulo $ L$, which implies one of them must be divisible by $ L$. So in order to get $ L$ consecutive prime numbers in the A.P. we must have $ (d,L)\\neq 1$.[/hide]", "Solution_10": "Yes, that is a necessary condition, but you can do better. Think about it. An arithmetic progression of primes of length $ L$ also contains a progression of length $ L\\minus{}1$, $ L\\minus{}2$... ;)", "Solution_11": "I had to think about this for some time and finally came to this. I'm not sure if this is what you wanted though.\r\n\r\n[hide]So from my work and your hint we must have $ (k,d) > 1$, for $ k \\equal{} 1,2,\\dots,L$. This clearly implies that $ p_{i}\\mid L$ for all primes $ p_{1},p_{2},\\dots p_{n}\\le d$. And thus $ p \\equal{} p_{1}p_{2}\\cdots p_{n}\\mid L$.[/hide]\r\n\r\nActually I found this long ago, and I did some problems with it too. For example about one year or so ago I found the six primes $ 7,37,67,97,127,157$ in A.P. using this fact. But it seems that I forgot it so easily. :blush:\r\n\r\nEDIT: Yes t0rajir0u is right. Sorry about that. :oops:", "Solution_12": "[quote=\"nayel\"]I had to think about this for some time and finally came to this. I'm not sure if this is what you wanted though.\n\n[hide]So from my work and your hint we must have $ (k,L) > 1$, for $ k \\equal{} 1,2,\\dots,d$. This clearly implies that $ p_{i}\\mid L$ for all primes $ p_{1},p_{2},\\dots p_{n}\\le d$. And thus $ p \\equal{} p_{1}p_{2}\\cdots p_{n}\\mid L$.[/hide]\n\nActually I found this long ago, and I did some problems with it too. For example about one year or so ago I found the six primes $ 7,37,67,97,127,157$ in A.P. using this fact. But it seems that I forgot it so easily. :blush:[/quote]\r\n\r\nYou seem to have your variables switched... I think you mean $ (d, k) > 1, k \\equal{} 1, 2, ... L$. :)" } { "Tag": [ "induction", "inequalities" ], "Problem": "Using induction, prove a_1 <= a_n. We also know that a_n<=a_(n+1). I am not so familiar with induction so anything would help. Thanks", "Solution_1": "Base of induction: $ a_1\\le a_2$.\r\nThe inductive step: assuming $ a_1\\le a_{n\\minus{}1}$ (the induction hypothesis) and using the inequality $ a_{n\\minus{}1}\\le a_{n}$, we obtain $ a_1\\le a_n$.\r\n\r\n[url=http://en.wikipedia.org/wiki/Mathematical_induction]Mathematical induction[/url]" } { "Tag": [ "LaTeX", "linear algebra", "matrix", "conics", "ellipse", "algorithm", "function" ], "Problem": "Hi\r\nat first i want to say that i am very happy that i have found this forum! now to my question: i am 14 and im very interested in programming and math. two weeks ago i read \"the da vinci code\" and so i found the fibonacci numbers. since then im fascinated in them. so i made a small php programm to generate them, but php isnt very good for math and so i want to know which language you would recommend me.\r\nflo", "Solution_1": "Matlab is a very nice language and environment for math.\r\n\r\nAmong applied mathematicians, Matlab and C++ are both very popular.", "Solution_2": "What about LaTeX? (I think it counts as a language...)\r\n\r\nFortran is meant for math, too, but it's *ahem* [i]slightly[/i] outdated. *ahem*", "Solution_3": "of course! It has a very pure mathematical structure.", "Solution_4": "I think $\\LaTeX$ is just a way to type stuff (more specifically, math) up, so it'd be more like a font than a language. \r\n:Read above statement\r\n:If \r\n:you disagree\r\n:Then\r\n:write a program in $\\LaTeX$ that computes the $n^{th}$ term of the Fibonacc sequence using Binet's formula (or a similar/explicit one)\r\n:Else\r\n:My point is made and I win!\r\n:End\r\n :D", "Solution_5": "I also recommend using Matlab. It depends on what you want to do. I am thinking from the statistician and machine learning perspective where you need a lot of matrix computations to do all kind of fancy transformations of fancy datasets. But Matlab is also strange because you have to get rid of thinking in loops but have to try to implement iterations by means of matrix operations which takes some time to get used to if coming from a different world as me, more or less having applied languages as Ruby, C, C# and Java. Loops are very inefficient in Matlab.\r\nAs said before in Matlab you basically get a lof of matrix stuff for free as computation of eigenvalues/eigenvectors, Cholesky Factorization, Singular Value Decomposition and many more. But on the other hand it is like programming in the 70s. Matlab does not have any convincing object-oriented approach. C++ and those others languages are mentioned above are rather suited for traditional application development. My advice is to start learning Java. The best book to learn Java, in my opinion, can be found [url=http://www.amazon.co.uk/exec/obidos/ASIN/0131249339/202-7873790-0543068]here.[/url] It treats the beginner gently. It does not teach you hacking but to write good programs in the way of how to structure your programs, when to re-factor classes, when it is reasonable to apply inheritance etc. Or in other words don't forget about software engineering. And the most important point is that it is easy to get started. You don't need programming environments (integrated development environment) as Eclipse or anything like that which is confusing for beginners due to its broad range of settings. So you are starting with the BlueJ environment. \r\nAnd you should not forget about the fact that Matlab is not a free software, e.g. you have to buy it first. But, of course, you could use its free alternative Octave.\r\nBasically you should know different kind of languages, e.g. knowing Java it is easy to learn C. If you want to take a look at a functional programming language I recommend OCaml.\r\nHope this helps.", "Solution_6": "[quote]It does not teach you hacking but to write good programs in the way of how to structure your programs[/quote]\r\n\r\nWhat do you mean by \"hacking\" here?\r\n\r\nA lot of people talk about \"hacking\" at AoPS but I'm never quite sure what that is.", "Solution_7": "[quote]I think $\\LaTeX$ is just a way to type stuff (more specifically, math) up, so it'd be more like a font than a language [/quote]\r\n\r\nFYI: Languages which are merely used to display certain texts/fonts (like LaTeX and HTML) are called markup languages whereas languages that are used to do computations (like C++ and Matlab) are called programming langauges. :)", "Solution_8": "[quote=\"Aunt Sally\"][quote]It does not teach you hacking but to write good programs in the way of how to structure your programs[/quote]\n\nWhat do you mean by \"hacking\" here?\n\nA lot of people talk about \"hacking\" at AoPS but I'm never quite sure what that is.[/quote]\r\n\r\nHacking = writing really quick and poorly structured and commented and not that re-usable code. Also known as the sort of programs that everyone writes for Topcoder (if this sentence confuses you, ignore it).\r\n\r\nTo answer original question, C++ = best all-purpose language to use most likely.", "Solution_9": "I like C, it is a great balance between speed and ease of use. But I guess C++ kind of makes it obselete. And Matlab? That's just very slow BASIC like programming, even though its computational speeds are quick, its structures like loops are quite slow and inefficient.", "Solution_10": "thank you very much for the answers. i posted this question in an other forum and they recommended haskell. what do you say about that language?", "Solution_11": "[quote=\"schaf88\"]thank you very much for the answers. i posted this question in an other forum and they recommended haskell. what do you say about that language?[/quote]\r\n\r\nYeah Haskell is a good choice as well. It is a functional programming language that can be learned quite easily. Many universities start with a mixture of Java and Haskell. At first they teach Haskell and then they swap over to Java during the second part of the semester when algorithms come into play. If you want to learn some Haskell look [url=http://www.doc.ic.ac.uk/~ajf/Teaching/Haskell2005.html]here.[/url] I don't recommend C, C++ or C# when starting to learn a programming language. You can learn it later.\r\n\r\n[url=http://mitpress.mit.edu/sicp/full-text/book/book.html]\"Structure and Interpretation of Computer Programs\"[/url] is a must reading when starting to program. \r\n\r\n@pieterminate: I guess you never really needed to work out fancy matrix computations. Thus you just see the disadvantages which I already mentioned. I promise that nobody ever wanted to do that in C or whatever language you like.", "Solution_12": "thx. do you think oo-languages (like ruby) are better for maths or functional languages? I think ruby is qite easier to understand.", "Solution_13": "[quote=\"schaf88\"]thx. do you think oo-languages (like ruby) are better for maths or functional languages? I think ruby is qite easier to understand.[/quote]\r\n\r\nWell Ruby or Haskell are not quite mathematical programming languages. Well a program in a functional programming language is a (nested) function but of course you can do imperative stuff by means of side-effects in those languages.\r\nOf course, you can program recursion easily as you intended to do. But for the sake of math I am experimenting around a lot of time and you quickly want to have something plotted (displayed) and that is really easy in Matlab, e.g. you want to have a look at the growth of the Fibonacci numbers etc. Ruby is a pure object oriented languages and basically it is advancement compared to Java. You can program in Ruby like Java. But that is not quite the purpose you intend to follow. Ruby offers so many things that are very tedious in Java. And with the event of the framework \"Ruby on Rails\" Ruby gets more and more widely spread. But if I were you in the course of some self-study I would try learning Java.\r\n\r\nMaybe anyone wants to comment on his experiences with Mathematica or Maple. If you are doing statistics I recommend R." } { "Tag": [ "Support", "geometry", "\\/closed" ], "Problem": "Hi guys. \r\n\r\nUnfortunately it seems that phpBB does not (yet) support daylight savings time settings, so for now, all users in GMT 0,+1,+2,+3 areas, are urged to change their time zones with +1, that is in GTM +1,+2,+3,+4 respectively.\r\n\r\nto be more exact, all users from Romania, who want the correct time displayed on stuff, should go to Profile -> Date Zone -> and select +3 GMT. \r\n\r\nthis will be in effect untill the next daylight saving time change occurs.", "Solution_1": "Why is it that people in North America don't have that problem?", "Solution_2": "[quote=\"RC-7th\"]Why is it that people in North America don't have that problem?[/quote]\r\n\r\nWe don't have that problem here in the U.S. because we're the ones who live the anachronistic timezone nation." } { "Tag": [], "Problem": "In how many ways can you divide 6 people into two teams, with each team having at least one member?\r\n\r\nIf the number of teams were 7, C(7,1)+C(7,2)+c(7,3)=63 provides the correct answer. I'm not sure why that works and can someone help me figure how to do the problem with six people?", "Solution_1": "For each player, you can pick him for the first team, or for the second team. That gives 2^6=64 possible combinations of teams, eliminating the two possiblilties of all the players on one team or the other, gves 62 possibilites.", "Solution_2": "[quote=\"rofler\"]For each player, you can pick him for the first team, or for the second team. That gives 2^6=64 possible combinations of teams, eliminating the two possiblilties of all the players on one team or the other, gves 62 possibilites.[/quote]\r\n\r\nyou overcounted that two teams are indistinguishable..\r\nit is $6c1+6c2+6c3=41$\r\n\r\nbecause if theres 1 people on a team, then there will be 5 people on the other team, so it is 6 choose 1 to ensure that happen\r\n\r\nif there are 2 people onm a team, there will be 4 people on the other team. 6 choose 2...\r\n\r\netc..", "Solution_3": "The series goes individuals-ways\r\n2-1\r\n3-3\r\n4-7\r\n5-15\r\n6-31\r\n7-63\r\nI think (2^n-2)/2 gets you the right answer each time.\r\nLike (2^6-2)/2=31.\r\nThe C(7,1)+C(7,2)+C(7,3) works for all odd number of individuals such as C(5,1)+C(5,2)=15. I'm not sure why C(6,1)+C(6,2)+C(6,3)=41 doesn't work. That's what I'm wondering about. Thanks!" } { "Tag": [ "quadratics", "geometry", "number theory" ], "Problem": "how would an 8th grader do at the camp, like which track would i be on?", "Solution_1": "It depends, there were people going into 9th grade in pretty much all tracks, don't worry about it too much, once you're there, if your track is too easy/hard then they'll let you change.", "Solution_2": "Tracks don't necessarily depend on ability, mostly interests.", "Solution_3": "[quote=\"CatalystOfNostalgia\"]Tracks don't necessarily depend on ability, mostly interests.[/quote]\r\nThey depend on ability somewhat too though, I mean, track 1 Number Theory (Combinatorial Nullstellensatz, root flipping, Dirichlet series, Hensel's Lemma, Quadratic Reciprocity, primitive roots etc.) was a lot lot harder than track 2 Number Theory.\r\n\r\nNOTE: The track system has been abolished, people keep making references to it, but it won't be the system for this year.\r\nhttp://www.awesomemath.org/AMSP_dailysch.pdf", "Solution_4": "Hence the word necessary. What I'm hinting at is that if you are interested in geometry, you shouldn't take track 1 just because it is harder than track 2, even if you can handle track 1.", "Solution_5": "[quote=\"CatalystOfNostalgia\"]Hence the word necessary. What I'm hinting at is that if you are interested in geometry, you shouldn't take track 1 just because it is harder than track 2, even if you can handle track 1.[/quote]\r\nExcept that since they don't have tracks, it doesn't matter anymore karel." } { "Tag": [], "Problem": "Find the number of 10-letter permutations comprising 4 $ a$'s, 3 $ b$'s and 3 $ c$'s such that no two adjacent letters are identical.", "Solution_1": "I'm not sure if this is right, but...\r\n\r\n[hide]\nLet's start with the sequence abc. There are 6 initial ways to arrange this. We will build adding an a, b, c, a, b, c, a in that order. There are two places to put the first a, 3 places to put the first b, 4 places to put the first c, three places to put the second a, etc... In the end, it will be\n\n$ 6 \\cdot 2 \\cdot 3 \\cdot 4 \\cdot 3 \\cdot 4 \\cdot 5 \\cdot 4 \\equal{} 34560$\n\nBut we overcounted for the 4 a's, 3 b's, and 3 c's. Thus we divide by $ 4! \\cdot 3! \\cdot 3!$ to arrive at an answer of $ \\boxed {40}$.\n\nI'm not sure if that's right.[/hide]", "Solution_2": "My approach was a bit different.\r\n\r\n[hide]Consider how we can arrange 3 c's and 3 b's such that no two identical letters are adjacent. There are two, either bcbcbc or cbcbcb. Now we have 4 A's to distribute so that they are not paired with each other. There are seven possible places to place the a's either to the left or right of the letters b and c, and we want to find how we can choose four of them. This becomes 7C4, or 35, and since there are two original sequences of b's and c's, we arrive at an answer of 70.[/hide]\r\n\r\nI think I'm missing some solutions when bc and cb can be interchanged, but I am not sure.", "Solution_3": "Yongyi, the sequence is not necessarily abc; it could be aba.\r\n\r\nazn, there are a lot more ways than u found; u could have cbbcbc, for example, and then add an a between the 2 adjacent b's.\r\n\r\n[hide]\nI used pretty complicated casework to get $ \\boxed{248}$...\nProbably wrong.[/hide]", "Solution_4": "That could be remedied by putting the a between the b and c like abc -> abac.\r\n\r\nI DID say that you could arrange the abc 6 ways, right?", "Solution_5": "I'll try to add on to my answer of 70, which occurred when b and c were already separated.\r\n\r\nCase 1: The string has a pair of b's but no other pairs.\r\n\r\nThere are two strings of letters where this occurs, and since an a must divide the pair, there are only 6C3 strings now, or 2X20 more words. We multiply this by two again because the same holds true for a pair of c's, resulting in 80 more words.\r\n\r\nCase 2: There is a pair of c's and a pair of b's in the string.\r\n\r\nIf we look at the sequence bcbc, there are two places each for the c and for the b to form two pairs of letters, and the same holds true for cbcb, so we have 8 new possibilities. 2 a's will be used breaking these pairs, so we have 5C2 places left for the others, resulting in 8x(5C2), or another 80 words.\r\n\r\nCase 3: There is a triple of one letter and a pair of the other letter.\r\n\r\nThere are two ways to arrange this for one triple (the pair comes before or after it), so we have 2X2X4, or 16 more words.\r\n\r\nCase 4: There are two triples.\r\n\r\nThis occurs twice and every a must break a pair, resulting in 2 new words.\r\n\r\nSo the answer is 248. Yay!", "Solution_6": "Shucks I have 239 :blush:", "Solution_7": "[hide=\"this has got to be wrong, probably missing something obvious\"]Define $ p_{a,b,c}$ as the number of permutations with $ a$ as, $ b$ bs, and $ c$ cs. Adding a new letter to a string, there are $ a\\plus{}b\\plus{}c\\minus{}2$ ways to place it, so $ p_{a,b,c}\\equal{}(a\\plus{}b\\plus{}c\\minus{}2)p_{a\\minus{}1,b,c}$. Using this recursion, we have that the answer is $ 9!\\cdot6$, since $ p_{1,1,1}\\equal{}6$.[/hide]", "Solution_8": "A counterexample is $ p_{3, 1, 1}$. Clearly, there are only 2 ways to arrange aaabc, which are abaca and acaba. And $ p_{4, 1, 1} \\equal{} 0$." } { "Tag": [ "algebra", "polynomial", "algebra proposed" ], "Problem": "A trigonometric polynomial is nonnegative on R. Then it is of the form $ abs( P^2(cosx+isinx))$ for a polynomial P. Here abs is the modulus function.", "Solution_1": "It was also question 1a/ in part IV of written problem \r\nEcoles Normales Suprieures sujet commun Paris/Lyon 1999." } { "Tag": [ "AMC" ], "Problem": "okay, so just a few days ago i began doing AoPS vol 1. yesterday i was doing the end of chapter problems for the proportions section. one of the problems (an AHSME from some year) brought up a question for me.\r\nif some City A has 1000 dollars in its budget and has 100 citizens and each citizen gets the same amount of money, then to find the wealth of each citizen you would divide the budget by the # of citizens. this is obvious.\r\nhowever, if you had what percentage City A has of the entire world's population and what percentage City A has of the entire world's wealth, could you do the same thing and divide the percentage of population by the percentage of wealth to get the wealth of each citizen. (this is assuming the problem doesn't give you any numerical values for either). I did this and got the right answer to the problem, but the solutions manual gave a different method. i just want to be sure if i got the answer correct by luck or you can use my method in such problems.\r\n\r\nSorry if the questions are a little confusing.\r\nThanks!", "Solution_1": "you're right. they probably did it more algebraically.\r\n\r\nwait, shouldn't it be wealth/population? instead of population/wealth\r\ni'm guessing that's a typo", "Solution_2": "you're right indianamath, it was a typo.\r\n\r\nthanks!" } { "Tag": [ "logarithms" ], "Problem": "If $\\log_{b^2}x+\\log_{x^2}b=1,b>0,b\\neq 1,x\\neq 1,$ then $x$ equals:\r\n(A): $1/b^2$\r\n(B): $1/b$\r\n(C): $b^2$\r\n(D): $b$\r\n(E): $\\sqrt{b}$", "Solution_1": "all into base b:\r\n\r\nwe have $ \\frac{log_bx}{2}+\\frac{1}{2log_bx}=1$\r\n\r\n let $y=log_bx$\r\n\r\nso $ \\frac{y}{2}+\\frac{1}{2y}=1$\r\n \r\n $y^2-2y+1=0$\r\n $(y-1)^2=0$\r\n\r\n\r\n\r\nhence $log_bx=1$ $x=b$\r\n\r\nans is $D$", "Solution_2": "You can apply change of base formula, and have them all in log base b:\r\n\r\nSo:\r\n\r\n$\\frac{\\log_{b}{x}}{\\log_{b}{b^2}}+\\frac{\\log_{b}{b}}{\\log_{b}{x^2}}=1$\r\n$\\frac{\\log_{b}{x}}{2}+\\frac{1}{2(\\log_{b}{x})}=1$\r\nFrom here, it's fairly easy to deduce that both of those fractions being added together must equal $1$ and since they are both $\\frac{1}{2}$, then $x$ must equal $b$ to cancel the logs out. So $D$.", "Solution_3": "any more log problems??? :? :(", "Solution_4": "[quote=\"riddler\"]any more log problems??? :? :([/quote]\r\n\r\ncalm down and relax :sleep2:", "Solution_5": "what Ive only just started." } { "Tag": [ "vector", "function", "induction", "floor function", "algebra", "functional equation", "algebra unsolved" ], "Problem": "Find all $ f: R\\rightarrow R$ such that\r\ni)$ f(xy)=f(x)f(y)$ for all real x,y\r\nii)$ f(x+1)=f(x)+1$ for all real x", "Solution_1": "Obviosly $ f(1)=1\\to f(n)=n \\ \\forall n\\in Z \\to f(r)=r \\ \\forall r\\in Q$.\r\nBut there are infinetely many solutions in R.", "Solution_2": "Why there are infinite solutions?", "Solution_3": "Algebra R(Q) had infinetely many automorfisms. For example $ Q[\\sqrt 2 ]$ had 2 automorfisms.", "Solution_4": "If $ f$ is continuous, are there finite solution(s)?", "Solution_5": "[quote=\"snowyowl\"]If $ f$ is continuous, are there finite solution(s)?[/quote]\r\n\r\nIf $ f(x)$ is continuous in all $ \\mathbb{R}$ (including $ 0$), it is rather well known that the functional equation $ f(xy)=f(x)f(y)$ has only solutions $ f(x)=0$ and $ f(x)=x^{a}$ for any real $ a\\geq 0$.\r\n\r\nIf we add then the requirement $ f(x+1)=f(x)+1$, the only available continuous solution is $ f(x)=x$.\r\n\r\nBut I don't understand Rust's demo saying that it exist infinitely many non continuous solutions (at least we need axiom of Choice, and even with this axiom, I don't see). \r\n\r\nRust, I would be very interested in some examples.", "Solution_6": "[quote=\"pco\"]\nBut I don't understand Rust's demo saying that it exist infinitely many non continuous solutions (at least we need axiom of Choice, and even with this axiom, I don't see). \n\nRust, I would be very interested in some examples.[/quote]\r\nAll automorfism $ f: R/Q\\to R/Q$ ($ f|_{Q}=id$) is solution. By axiom of Choice we get infinetely many automorfisms $ R/Q\\to R/Q$. construct", "Solution_7": "[quote=\"Rust\"][quote=\"pco\"]\nBut I don't understand Rust's demo saying that it exist infinitely many non continuous solutions (at least we need axiom of Choice, and even with this axiom, I don't see). \n\nRust, I would be very interested in some examples.[/quote]\nAll automorfism $ f: R/Q\\to R/Q$ ($ f|_{Q}=id$) is solution. By axiom of Choice we get infinetely many automorfisms $ R/Q\\to R/Q$. construct[/quote]\r\n\r\nI'm sorry but I still don't understand. May be a language problem\r\nAssume (with axiom of Choice) that it exist a $ \\mathbb{Q}$-vector space $ A$ such that $ \\mathbb{R}=A+\\mathbb{Q}$ and $ A\\cap\\mathbb{Q}=\\{0\\}$.\r\nThen every real $ x$ can be written in a unique way as $ x=q(x)+a(x)$ with $ q(x)\\in\\mathbb{Q}$ and $ a(x)\\in A$\r\n\r\nLet then $ f(x)=q(x)+3a(x)$\r\n\r\n$ f(x)$ is an automorphism from the $ \\mathbb{Q}$-vector space $ \\mathbb{R}$ into itself with restriction of f in $ \\mathbb{Q}$ being the identity function (as you seemed to say) but $ f(x)$ is absolutely not a solution of the problem, isn't it?", "Solution_8": "[quote=\"pco\"]I'm sorry but I still don't understand. May be a language problem\nAssume (with axiom of Choice) that it exist a $ \\mathbb{Q}$-vector space $ A$ such that $ \\mathbb{R}=A+\\mathbb{Q}$ and $ A\\cap\\mathbb{Q}=\\{0\\}$.\nThen every real $ x$ can be written in a unique way as $ x=q(x)+a(x)$ with $ q(x)\\in\\mathbb{Q}$ and $ a(x)\\in A$\n\nLet then $ f(x)=q(x)+3a(x)$\n\n$ f(x)$ is an automorphism from the $ \\mathbb{Q}$-vector space $ \\mathbb{R}$ into itself with restriction of f in $ \\mathbb{Q}$ being the identity function (as you seemed to say) but $ f(x)$ is absolutely not a solution of the problem, isn't it?[/quote]\r\nBy transfinite induction we had $ Q=A_{0}\\subset A_{1}\\subset A_{2}\\subset ...=R.$\r\nWere $ A_{i+1}=A_{i}[x_{i+1}], \\ x_{i+1}\\not \\in A_{i}$ and for infinite ordinal i: $ A_{i}=\\bigcup_{j1$ which is in contradiction with $ f(nx)\\geq \\lfloor nx\\rfloor$. So $ f(x)\\geq x$ $ \\forall x\\in\\mathbb{R}$.\r\n\r\nThen $ f(-1)=-1$ implies $ f(-x)=-f(x)$ and so $ f(-x)\\geq-x$ implies $ -f(-x)\\leq x$ and so $ f(x)\\leq x$\r\n\r\nSo we have $ f(x)\\geq x$ and $ f(x)\\leq x$ and the only solution is $ f(x)=x$\r\n\r\nThanks Harazi for your demo.", "Solution_11": "Yes, I am wrong. There are unique automorfism $ R/Q\\to R/Q$.\r\nMy method work only for C." } { "Tag": [ "inequalities proposed", "inequalities" ], "Problem": "For $a\\geq 0,\\ b\\geq 0,\\ c\\geq 0$, Prove the following inequality.\r\n\r\n$a^{5}+b^{5}+c^{5}+2(a^{2}b^{3}+b^{2}c^{3}+c^{2}a^{3})\\geq a^{4}b+b^{4}c+c^{4}a+2(ab^{4}+bc^{4}+ca^{4})$.", "Solution_1": "[quote=\"kunny\"]For $a\\geq 0,\\ b\\geq 0,\\ c\\geq 0$, Prove the following inequality.\n\n$a^{5}+b^{5}+c^{5}+2(a^{2}b^{3}+b^{2}c^{3}+c^{2}a^{3})\\geq a^{4}b+b^{4}c+c^{4}a+2(ab^{4}+bc^{4}+ca^{4})$.[/quote]\r\n$a^{5}+b^{5}+c^{5}+2(a^{2}b^{3}+b^{2}c^{3}+c^{2}a^{3})\\geq a^{4}b+b^{4}c+c^{4}a+2(ab^{4}+bc^{4}+ca^{4})\\Leftrightarrow$\r\n$\\Leftrightarrow\\sum_{cyc}(2a+3b)(a-b)^{4}\\geq0.$", "Solution_2": "O.K. My solution is as follows.\r\n\r\n$a^{5}+b^{5}+c^{5}+2(a^{2}b^{3}+b^{2}c^{3}+c^{2}a^{3})\\geq a^{4}b+b^{4}c+c^{4}a+2(ab^{4}+bc^{4}+ca^{4})\\Longleftrightarrow$\r\n$\\Longleftrightarrow\\sum_{cyc}(b+c)(a-b)^{2}(b-c)^{2}\\geq0.$", "Solution_3": "How do you find the motivation for such factorizations?", "Solution_4": ":P [quote=\"kunny\"]O.K. My solution is as follows.\n\n$a^{5}+b^{5}+c^{5}+2(a^{2}b^{3}+b^{2}c^{3}+c^{2}a^{3})\\geq a^{4}b+b^{4}c+c^{4}a+2(ab^{4}+bc^{4}+ca^{4})\\Longleftrightarrow$\n$\\Longleftrightarrow\\sum_{cyc}(b+c)(a-b)^{2}(b-c)^{2}\\geq0.$[/quote]\r\n\\[=\\sum (b+c)(a-b)^{2}[(a-b)^{2}+(2b-a-c)(a-c)]=\\]\\[=\\sum (b+c)(a-b)^{4}+(b+c)(a-b)^{2}(2(b-a)+a-c)(a-c)=\\sum 2(b+c)(b-a)^{3}+(a-b)^{2}(b+c)(a-c)^{2}+(b+c)(a-b)^{4}\\]" } { "Tag": [ "calculus", "derivative", "real analysis", "real analysis unsolved" ], "Problem": "Find the sum, using derivative $ {C_n}^1\\plus{}2{C_n}^2\\plus{}3{C_n}^3\\plus{}...\\plus{}n{C_n}^n$", "Solution_1": "Let $ S(x)\\equal{}\\sum_{k\\equal{}1}^n\\binom{n}{k}x^k\\equal{}(1\\plus{}x)^n\\minus{}1$. Then, $ S'(x)\\equal{}n(1\\plus{}x)^{n\\minus{}1}$. We want $ S'(1)\\equal{}n2^{n\\minus{}1}$. And the conclusion follows." } { "Tag": [ "set theory", "real analysis", "real analysis theorems" ], "Problem": "PROVE :\r\n\r\nIf.$ x\\in A\\Longrightarrow y\\in A$,THEN x=y ,where $ A\\neq\\emptyset$", "Solution_1": "And A lives where?", "Solution_2": "where all the sets live.\r\n\r\nit is not so hard to find it :)", "Solution_3": "This problem makes no sense. As written, the set $ A \\equal{} \\{x, y \\}$ satisfies the hypotheses of the question without satisfying the conclusion.", "Solution_4": "I finally figured out what the question is supposed to mean:\r\n\r\nFix $ x$. Suppose that $ \\forall A$, $ x \\in A$ implies $ y \\in A$. Then $ x \\equal{} y$.\r\n\r\n(Quantifiers are important!!)\r\n\r\nAnyhow, since $ x \\in A$ implies $ y \\in A$ for all $ A$, we have $ y \\in \\{x\\}$ and so $ y \\equal{} x$. (It shouldn't be hard to justify each of those steps directly in terms of ZF, but I don't think I really want to go to the effort.)", "Solution_5": "Maybe in math my favorite subject is logic, but not strickt one (justifying facts due to axioms etc.). What this problem asks to prove is very \"logical\". I mean that if y was not equal to x then how could we know that if x belongs to A then y belongs as well? :wink:", "Solution_6": "[quote=\"JBL\"]I finally figured out what the question is supposed to mean:\n\nFix $ x$. Suppose that $ \\forall A$, $ x \\in A$ implies $ y \\in A$. Then $ x \\equal{} y$.\n\n(Quantifiers are important!!)\n\nAnyhow, since $ x \\in A$ implies $ y \\in A$ for all $ A$, we have $ y \\in \\{x\\}$ and so $ y \\equal{} x$. (It shouldn't be hard to justify each of those steps directly in terms of ZF, but I don't think I really want to go to the effort.)[/quote]\r\n\r\n :) :) [b]But toraiir0u thinks otherwise[/b]\r\n\r\nIn proving the forward implication of : (SxT)xU = Sx(TxU)$ \\Longleftrightarrow S\\equal{}\\emptyset\\vee T\\equal{}\\emptyset U\\equal{}\\emptyset$ i was forced to accept the above problem. you can check it up maybe i am wrong.\r\n\r\nAnyway ,next step to check your proof i suppose will be a formal proof justifying each step like you said", "Solution_7": "t0r doesn't think otherwise. He interpreted your problem literally (with no quantifier) and demonstrated (correctly) that it's false, while I gave a guess about what non-false statement you might have meant (which involved adding a quantifier) and gave a proof of it.", "Solution_8": "[quote=\"JBL\"]t0r doesn't think otherwise. He interpreted your problem literally (with no quantifier) and demonstrated (correctly) that it's false, while I gave a guess about what non-false statement you might have meant (which involved adding a quantifier) and gave a proof of it.[/quote]\r\n\r\n\r\nYes ,the right formula for the above problem is:\r\n\r\n$ \\forall A\\forall x\\forall y[(x\\in A\\Longrightarrow y\\in A)\\Longrightarrow x\\equal{}y]$\r\n\r\nBut whenever i write this kind of formula in a proof people get annoyed.\r\n\r\nDo you thing formalization will solve many mistakes in mathematics??", "Solution_9": "You don't have to write it that formally. There is a balance between over formalization and insufficient formalization. Your first post had insufficient formalization, your last post had too much. The way that JBL formulated the problem was clear, easy to understand, and sufficiently precise to comunicate the problem.\r\n\r\nAs a rule of thumb, just try to keep your mathematical writing as simple as possible without leaving it open to misinterpretation.", "Solution_10": "WHY ? \r\n\r\nIf i write the problem that formally ,will mathematics loose the power of its axioms??\r\n\r\nI REALLY do not understand why complete formalization is the red cloth for mathematicians ,while at the same time they try to formalize a problem as possible??\r\n\r\nIs not that a schizophrenic behavior??\r\n\r\nBesides [b]only[/b] complete formalization can give a formal proof .\r\n\r\nTaking out of a formula the logical symbols will prevent the formal proof show the rules of logic involved in the proof", "Solution_11": "You [u]could[/u] write your mathematics that formally, and in some instances you might want to -- for instance, if you wanted your proof to be checked by a computerized proof-checking system (yes, such things do actually exist). However, over formalization and added symbols often make math harder to understand and slower to read. So if your goal is to communicate your ideas (which is often the case in mathematical writing), then you have to strike a balance with your intended audience in mind. If it is communicated well, then the audience should be able to implicitly fill in the needed formalities without being led to any possible misinterpretations of what you are trying to say.", "Solution_12": "[quote=\"\u03c4\u03c1\u03b9\u03ba\u03bb\u03b9\u03bd\u03bf\"]Yes ,the right formula for the above problem is:\n$ \\forall A\\forall x\\forall y[(x\\in A\\Longrightarrow y\\in A)\\Longrightarrow x \\equal{} y]$[/quote]\n\nNo, it's not. This is the incorrect statement that t0r disproved. (In English, it says, \"For any set and any pair $ (x, y)$, we have that \"$ x \\in A$ implies $ y \\in A$\" if and only if $ x \\equal{} y$,\" and this is demonstrated false by the triple $ A \\equal{} \\{1, 2\\}$, $ x \\equal{} 1$, $ y \\equal{} 2$.) The correct formalization of the statement is\n\\[ \\forall x \\forall y ((\\forall A(x \\in A \\Longrightarrow x \\in y)) \\Longrightarrow x \\equal{} y).\n\\]This adds nothing at all to my phrasing of the statement, and in fact my earlier statement could be \"deformalized\" by replacing the symbolic quantifier $ \\forall$ with its word-equivalent with no loss. (Indeed, it might have been better had I written it that way to begin with.)\n\n[quote=\"\u03c4\u03c1\u03b9\u03ba\u03bb\u03b9\u03bd\u03bf\"]But whenever i write this kind of formula in a proof people get annoyed.\nDo you thing formalization will solve many mistakes in mathematics??[/quote] Perhaps this is because you are sloppy and/or not very good at correctly formalizing what you mean. Formalization is useful in theory because it shows that our intuitive notions of proof really can be mechanically checked. In practice, it serves almost no purpose: it would take a huge amount of time to properly formalize even fairly simple statements in many branches of mathematics, and doing so largely obscures the actual meaning of what you're trying to say. This is particularly try if you formalize improperly, as in this case. Your original problem was wrong because you were imprecise, not because you were informal.\r\n\r\nTo the extent that I haven't actually rewritten what he wrote, I agree with gauss.", "Solution_13": "I'd say that i also agree that over-formalization helps noone and serves no purpose. Mathematics is all about using your mind. When you make a mathematical statement your only goal is to commute your thought. As far as this happens there is no need of further formalizations. Further formalizations are a matter of literature not math(at least thats my opinion).\r\n\r\n[u]However[/u], i do not understand why the initial statement of this problem statement is wrong. My proof(scroll up) that uses just common sense seems to prove the statement(or do i miss something).\r\n\r\nP.S. I'd like to give an example of the logic i like.\r\n\r\n[u]Statement:[/u] All criminals are bad people and all bad people are on jail.\r\n\r\nThen it follows using common sense that all criminals are in jail. One could say that i haven't referred to ZF etc. but what would by the point of doing so? Would this require thought or sharpen my mind :?:", "Solution_14": "[quote=\"filosofimenos\"] i do not understand why the initial statement of this problem statement is wrong. My proof(scroll up) that uses just common sense seems to prove the statement(or do i miss something).[/quote]\r\nThis is a perfect demonstration of why it's problematic to rely on common sense. The original problem doesn't have a quantifier on $ A$ and you interpreted it as having a quantifier on $ A$ in the correct location (the statement JBL gave).", "Solution_15": "Your \"proof\" was not a proof at all. It was a question. Your explaination was vacuous because it did not address the original issue. It simply restated the problem in contrapositive form.\r\n\r\nThis shows where logical formalization can have some advantages. It can be helpful as a teaching tool to train you to see logical fallacies and to teach you how to avoid bad and/or vacuous logical arguments.", "Solution_16": "Chill guys, don't shoot. :surrender: Yea, OK i restated the problem but i did it in order to reveal the obvious of the claim. That, i call proof.\r\n\r\nWe are all brothers :) :cool:", "Solution_17": "[quote=\"gauss202\"]You [u]could[/u] write your mathematics that formally, and in some instances you might want to -- for instance, if you wanted your proof to be checked by a computerized proof-checking system (yes, such things do actually exist).\r\n\r\n\r\n\r\n[b]Why let the computer check my proof ,when i can write my own formal proof??[/b]", "Solution_18": "\"Check\" = \"Verify that it is correct after it is written down.\"", "Solution_19": "[quote=\"JBL\"]I finally figured out what the question is supposed to mean:\n\nFix $ x$. Suppose that $ \\forall A$, $ x \\in A$ implies $ y \\in A$. Then $ x \\equal{} y$.\n\n(Quantifiers are important!!)\n\nAnyhow, since $ x \\in A$ implies $ y \\in A$ for all $ A$, we have $ y \\in \\{x\\}$ and so $ y \\equal{} x$. (It shouldn't be hard to justify each of those steps directly in terms of ZF, but I don't think I really want to go to the effort.)[/quote]\r\n\r\nHERE is a formal proof of the above:\r\n\r\n1)$ \\forall A(x\\in A\\rightarrow y\\in A)$........................................................assumption ( to start a conditional proof)\r\n\r\n\r\n2)$ x\\in${x}$ \\rightarrow y\\in${x}............................................................From (1) and using Universal Elimination where we put A={x}\r\n\r\n3)$ \\forall z(z\\in\\{x\\}\\leftrightarrow z\\equal{}x)$...................................................Theorem in set theory\r\n\r\n\r\n4)$ x\\in\\{x\\}\\leftrightarrow x\\equal{}x)$.................................................................From (3) and using Universal Elimination where we put z=x\r\n\r\n\r\n5)$ \\forall x( x\\equal{}x)$..................................................................................Axiom in equality\r\n\r\n\r\n6) x=x ..................................................................................................From (5) and using Universal Elimination where we put x=x\r\n\r\n\r\n7)$ x\\in\\{x\\}$............................................................................................From (4) and (6) and using M.Ponens for the backwards implication of (4).....(<=====)\r\n\r\n\r\n8)$ y\\in\\{x\\}$...........................................................................................From (2) and (7) and using M.Ponens\r\n\r\n\r\n9)$ y\\in\\{x\\}\\leftrightarrow y\\equal{}x$................................................................From (3) and using Universal Elimination where we put z=y\r\n\r\n\r\n10) y=x..................................................................................................From (8) and (9) and using M.Ponens for the forwards implication (=====>) of (9)\r\n\r\n\r\n11)$ \\forall A(x\\in A\\rightarrow y\\in A)\\rightarrow y\\equal{}x$...............................From (1) to (10) and using the deduction theorem ,or the rule of conditional proof\r\n\r\n\r\n12$ \\forall x\\forall y[\\forall A(x\\ A\\rightarrow y\\in A)\\rightarrow y\\equal{}x]$..........From (11) and generalization for x any y" } { "Tag": [ "MATHCOUNTS", "Putnam", "geometry", "AMC", "USA(J)MO", "USAMO", "AIME" ], "Problem": "Pretty much every community has an introduction topic, so I thought, why not start one here?\r\nJust say your name, where you live, your school, or anything you want to say about yourself.\r\n\r\n\r\n\r\nI'm Kriti, and I live in Cupertino. I want to be on the school MathCounts team in 7th grade.", "Solution_1": "Hi guys! Name is Ricardo, and I am an undergraduate student at Stanford.\r\n\r\nYep -> getting ready for PUTNAM 09 and IMC 09 =)", "Solution_2": "I'm Fat.\r\n\r\nI live in California (Bay Area, to be more specific... any more specificness would be stalkerish).", "Solution_3": "It's kindof obvious where I live from my username... :D I'm going to ninth grade. I'm female and would rather not say my name here. Made USAMO but failed it.", "Solution_4": "Um...I'm going to 10th grade...from San Jose, CA...kind of failed USAMO, but that's okay...", "Solution_5": "I'm going to be a junior, and I go to a school in Menlo Park.", "Solution_6": "Hi, my name is (name censored), nice to meet you!\r\n\r\nI'm going into 9th grade, near San Jose.\r\n\r\n\r\n\r\nif you went to AMP 08, you probably heard of me, one way or another.", "Solution_7": "[quote=\"Ubemaya\"]I'm Fat.\n\nI live in California (Bay Area, to be more specific... any more specificness would be stalkerish).[/quote]\r\nDitto.", "Solution_8": "[quote=\"Nerd_of_the_Ages\"][quote=\"Ubemaya\"]I'm Fat.\n\nI live in California (Bay Area, to be more specific... any more specificness would be stalkerish).[/quote]\nDitto.[/quote]\r\n\r\nYou are definitely not Fat.", "Solution_9": "ubey you should really get some more self-esteem\r\n\r\nfor example: \r\na) you are NOT THAT FAT\r\nb) you are NOT BOTTOM FIVE IN RED MOP\r\nc) you do NOT SUCK AT PROGRAMMING AS MUCH AS I DO", "Solution_10": "Hehe I've seen a good deal of people say he's not that fat, but sorry Albert I still think you're really fat :P\r\n\r\nHmmm so why did someone suddenly revive this 3 year old thread?\r\nCirclesquared I'll put something into the name censored for you shall I?\r\n\r\nI'm Andrew and I failed AIME so I didn't get to take USAMO (I think I would've gotten like around 14).", "Solution_11": "Actually, my name is albert, too (different last name, though). I'm pretty much Ubemya except I suck at math and typing, and am even fatter.\r\n\r\nThe guy revived it, since he's a new member and doesn't know about this forum yet.", "Solution_12": "We'll have to see a picture of that.\r\nUbey is pretty fat, but I've seen worse so it's possible", "Solution_13": "[quote=\"Nerd_of_the_Ages\"]Actually, my name is albert, too (different last name, though). I'm pretty much Ubemya except I suck at math and typing, and am even fatter.\n\nThe guy revived it, since he's a new member and doesn't know about this forum yet.[/quote]\r\nYou're Albert Wu right?\r\nAlbert Gu is pretty hard to beat in fatness, at least that's what it seems to me.", "Solution_14": "After MOP and this summer, I'm fatter than anyone here can even imagine. The only person fatter in the whole Bay Area at this moment would be my sister. (At this moment, she is sitting beside me patting her belly).", "Solution_15": "Ubey is everyone fat in your family? It kinda be ironic if you parents were anorexic.", "Solution_16": "[quote=\"hunter34\"]Ubey is everyone fat in your family? It kinda be ironic if you parents were anorexic.[/quote]\r\n\r\nNo they'd just stuff the excess food in ubey's room or something.", "Solution_17": "Hello I have a story about Ubey and food.\r\n\r\nUbey and Toan were roommates. On one of the last few nights of MOP, he, Toan, Jeffrey S, and Charles X were in our room talking about some problem on our door given by Hamster. I hadn't finished my inequalities write-ups, and the teacher (Ali) was trying to get me to do them. Toan and Ubey kept trying to steal my granola bars, and succeeded. Then I made them open their door so I could search their room. I failed miserably to find it, and then said that they could each have one if they returned the box (with all the rest of them). It turns out that the granola bar was in one of the drawers I had searched 3 times :ninja:", "Solution_18": "oh man\r\ndude \r\nthat is a good story", "Solution_19": "[quote=\"serialk11r\"][quote=\"Nerd_of_the_Ages\"]Actually, my name is albert, too (different last name, though). I'm pretty much Ubemya except I suck at math and typing, and am even fatter.\n\nThe guy revived it, since he's a new member and doesn't know about this forum yet.[/quote]\nYou're Albert Wu right?\nAlbert Gu is pretty hard to beat in fatness, at least that's what it seems to me.[/quote]\r\nWho's Albert Wu?", "Solution_20": "Nope he's not Albert Wu. Albert Wu is most definitely not that fat.", "Solution_21": "[quote=\"not_trig\"]Toan[/quote]\r\nOH MAN TOAN WAS SO AWESOME\r\n\r\n\"NO NO WAIT, FAT\" (talking to David Zeng) \"LEMME CONFISTICATE YOUR CRACK\"", "Solution_22": "Name- Matthew\r\nLive- East Side San Jose. Don't attempt to visit.\r\nSchool- Chaboya Middle School (Besides, only school in East San Jose to have a math program!) Eighth grade", "Solution_23": "Basically, what Ubemaya said in his first post. Except I'm way, way worse at math", "Solution_24": "[quote=\"xscapezaer\"]Basically, what Ubemaya said in his first post. Except I'm way, way worse at math[/quote]\r\n\r\nNo, you're way better.", "Solution_25": "The fattest mopper eh? How many pounds do you weigh? Less than me I bet. I weigh over 9000", "Solution_26": "David Shi wasn't on the MOP list of 2008...", "Solution_27": "where does it say that I ever went to mop? You misread.", "Solution_28": "Fattest mopper doesn't necessarily mean really fat, so it would make more sense to compare if you did make mop.", "Solution_29": "I know. The whole post was a joke, just like most of the bay area threads you know?", "Solution_30": "I know, I was trying to add to the joke (in a failing manner).", "Solution_31": "I never visit this forum much, but anyways...\r\n\r\nname: dragon96\r\nlocation: refer to location (antarctica)\r\nphone number: (911)-911-0000\r\nemailaddress: PM->dragon96@artofproblemsolving.com", "Solution_32": "Name: Eugene\r\nSchool harvest park middle school\r\ngrade 7\r\nwise-ness STUPID\r\nanyone go to san jose math circle here?", "Solution_33": "[quote=\"dragon96\"]I never visit this forum much, but anyways...\n\nname: dragon96\nlocation: refer to location (antarctica)\nphone number: (911)-911-0000\nemailaddress: PM->dragon96@artofproblemsolving.com[/quote]\r\n\r\nNotice this is the bay area forum?\r\nNo peopel from antarctica should be here.", "Solution_34": "[quote=\"Ubemaya\"][quote=\"xscapezaer\"]Basically, what Ubemaya said in his first post. Except I'm way, way worse at math[/quote]\n\nNo, you're way better.[/quote]\r\n\r\nHeh heh... Ubey, you enjoy your little joke...\r\n\r\nI lost.", "Solution_35": "Rofl xscape...didn't u make usamo?\r\n\r\nanyway, do you go to san jose math circle?\r\n\r\nor berkely math circl", "Solution_36": "hi i am rt_08\r\ni live in india \r\nand i will be giving my rmo for my region", "Solution_37": "[quote=\"AIME15\"]Rofl xscape...didn't u make usamo?\n\nanyway, do you go to san jose math circle?\n\nor berkely math circl[/quote]\n\nHe did it seems\n[quote]\nSucceeded - AIME: 9(8); USAMO - Qualified\nFailed - AMC10:120(135); AMC12:114(132)[/quote]" } { "Tag": [ "linear algebra", "matrix", "function", "superior algebra", "superior algebra solved" ], "Problem": "If $A,B \\in M_n(C)$ such that $e^A = e^B$ \r\n\r\nProve or disprove that \r\n\r\nfor any $x \\in R \\; e^{xA}=e^{xB}$", "Solution_1": "That isn't going to work. Let\r\n\r\n$A=\\left[\\begin{array}{cc} 0&-2\\pi \\\\ 2\\pi&0\\end{array}\\right]$ and $B=\\left[\\begin{array}{cc} 0&-4\\pi \\\\ 4\\pi&0\\end{array}\\right].$\r\n\r\nThen $e^A=e^B=I$ but for real $x$,\r\n\r\n$e^{xA}=\\left[\\begin{array}{cc} \\cos(2\\pi x)&-\\sin(2\\pi x) \\\\ \\sin(2\\pi x)&\\cos(2\\pi x)\\end{array}\\right]$ and\r\n\r\n$e^{xB}=\\left[\\begin{array}{cc} \\cos(4\\pi x)&-\\sin(4\\pi x) \\\\ \\sin(4\\pi x)&\\cos(4\\pi x)\\end{array}\\right].$\r\n\r\nThose aren't the same functions of $x$." } { "Tag": [ "calculus", "derivative", "geometry", "trigonometry", "function", "calculus computations" ], "Problem": "I'm not sure how to go about doing related rates problems where you have to take the derivative of 3 variables being multipled (i.e. $ D_x(xyz)$). If anyone can help me walk through how to do these types of problems, I would be very thankful:\r\n\r\n1. If $ a$ and $ b$ are lengths of two sides of a triangle, and $ \\theta$ the measure of the included angle, the area $ A$ of the triangle is $ A\\equal{}\\frac{1}{2}ab \\sin \\theta$. How is $ \\frac{dA}{dt}$ related to $ \\frac{da}{dt}$, $ \\frac{db}{dt}$, and $ \\frac{d\\theta}{dt}$?\r\n\r\n2. Suppose that the edge lengths $ x$, $ y$, and $ z$ of a closed rectangular box are changing at the following rates:\r\n\r\n$ \\frac{dx}{dt} \\equal{} 1$ m/s\r\n\r\n$ \\frac{dy}{dt} \\equal{} \\minus{}2$ m/s\r\n\r\n$ \\frac{dz}{dt} \\equal{} 1$ m/s\r\n\r\nFind the rates at which the box's volume, surface area, and diagonal length $ s\\equal{}\\sqrt{x^2\\plus{}y^2\\plus{}z^2}$ are changing at the instant when $ x\\equal{}4$, $ y\\equal{}3$, and $ z\\equal{}2$.", "Solution_1": "For the first question: $ a$, $ b$ and $ \\theta$ are functions of $ t$. You just need to use the product and chain rules. Likewise for the others. If you're not comfortable with the product rule for a product of three terms, just remember associativity: $ xyz \\equal{} (xy)z$, so $ D_t(xyz) \\equal{} D_t((xy)z) \\equal{} D_t(xy) \\cdot z \\plus{} xy \\cdot D_t(z)$, and then just use the (two-term) product rule again. This idea should make it easy to state the rule in general (for a product of any number of terms).\r\n\r\nWith that hint, why don't you show how you would proceed?" } { "Tag": [ "algebra", "polynomial", "geometry", "superior algebra", "superior algebra unsolved" ], "Problem": "We discussed this problem in class and the solution was really anything but easy. Does anyone have a natural solution (ok, the one I have is also natural, but not very...)?\r\n If you have $n 2$ a prime, and $ p \\not\\equiv 1\\mod{q}$. Write $ c \\equal{} b\\minus{}a$, so that $ b \\equal{} a\\plus{}c$. Then we have $ q \\equal{} \\minus{}ac$ and $ (a\\plus{}c^2 \\equal{} p\\plus{}nq$.\r\n\r\nAs $ q$ is a prime, we must have $ \\{a,c\\} \\equal{} \\{q,\\minus{}1\\}$ or $ \\{a,c\\} \\equal{} \\{\\minus{}q,1\\}$. In either case $ (a\\plus{}c)^2 \\equal{} (q\\minus{}1)^2$. Hence we have $ (a\\plus{}c)^2 \\equal{} q^2\\minus{}2q\\plus{}1 \\equal{} p\\plus{}nq$, so that $ p \\equiv 1\\mod{q}$. Contradiction; hence such $ \\frac{p}{q}$ are not in $ S$.\r\n\r\nThe error, which is quite subtle, comes from the fact that we may not force $ q$ to be positive: that is, we may have $ q \\equal{} ac$ and $ b^2 \\equal{} \\minus{}(p\\plus{}nq)$, in which case we need $ p \\not\\equiv \\minus{}1\\mod{q}$ as well.\r\n\r\nHow much credit is to be expected? Obviously, the argument is mostly sound, but there is a fine error made. I realize that it shouldn't be more than two points, but am wondering whether I will get nothing or something.", "Solution_22": "[quote=\"Teki-Teki\"]The error, which is quite subtle, comes from the fact that we may not force $ q$ to be positive: that is, we may have $ q \\equal{} ac$ and $ b^2 \\equal{} \\minus{} (p \\plus{} nq)$, in which case we need $ p \\not\\equiv \\minus{} 1\\mod{q}$ as well.[/quote]\r\nIn my opinion this was the whole point of the problem, so I would not expect too much.", "Solution_23": "I think $ \\frac{2}{3} \\not \\in S$ is also true.", "Solution_24": "[quote=\"cefer\"]I think $ \\frac {2}{3} \\not \\in S$ is also true.[/quote]\n[quote=\"jmerry\"]That doesn't work. $ \\frac23$ is reachable: $ 0\\to 3\\to\\frac16\\to\\frac{\\minus{}36}{5}\\to\\frac{4}{5}\\to\\frac{25}{\\minus{}4}\\to\\frac34\\to\\frac{16}{\\minus{}3}\\to\\frac23$.[/quote]\r\n(The quote is from a PM.)", "Solution_25": "[quote=\"cefer\"]I think $ \\frac {2}{3} \\not \\in S$ is also true.[/quote]\r\n\r\nwe already know why have all $ \\frac {1}{l}$ so try $ x\\equal{}\\frac {1}{4}$ get $ \\frac {16}{\\minus{}3}$ then adjust to $ \\frac {2}{3}$ (actually in addition to all $ \\frac{1}{l}$ all $ \\frac{\\minus{}1}{l} \\in S$ as well)", "Solution_26": "I think you can show $ \\frac {3}{14}$ cannot be in $ S$, since $ 3$ is not a quadratic residue mod $ 7$.\r\n(More detail: like in collegebookworm's solution, discriminant is a perfect square, so $ (p\\minus{}nq)^2\\plus{}4q(p\\minus{}nq)\\equal{}(p\\minus{}nq)(p\\minus{}nq\\plus{}4q)$ is a perfect square. But if $ p\\equal{}3$, $ q\\equal{}14$ then $ \\gcd(p\\minus{}nq, p\\minus{}nq\\plus{}4q)\\equal{}\\gcd(3\\minus{}14n, 4*14)\\equal{}1$ so $ p\\minus{}nq\\equal{}3\\minus{}14n$ is a perfect square, so $ 3$ is a quadratic residue mod $ 7$, a contradiction).", "Solution_27": "collegebookworm's solution is incorrect, as far as I can tell.", "Solution_28": "Maybe the last part is incorrect, but the part about discriminant being a square is correct.\r\nSay you start with $ S_0 \\equal{} {0}$ and define $ S_{i \\plus{} 1}$ as everything you can get from $ S_{i}$ using exactly one operation (i.e. $ x \\Rightarrow x \\plus{} 1, x\\Rightarrow x \\minus{} 1, x \\Rightarrow \\frac {1}{x(x \\minus{} 1)}$). Then if you assume $ \\frac {3}{14}$ is in the set, you look at the last time the third operation was used and you will actually have $ \\frac {p}{q} \\equal{} \\frac {1}{x(x \\minus{} 1)} \\plus{} n$ for some integer $ n$, since any composition of the first two operations is just adding/subtracting an integer. Also obviously $ x$ is rational.\r\nFrom there you show discriminant must be a square; the specific choice of $ (3,14)$ allows to show $ \\gcd$ must be $ 1$ and then reduce to the fact when $ p \\minus{} nq$ is a perfect square.\r\nIn collegebookworm's solution I think the problem is he wrote the expression $ q^2n^2 \\plus{} 2nq(p \\plus{} 2q) \\plus{} (p^2 \\plus{} 4pq)$ is a square of a polynomial of degree $ 1$, which does not have to be true; it could just be a square for one specific value of $ n$. The point is to find $ p,q$ for which that expression is never a square.", "Solution_29": "The given conditions describe properties that elements of S satisfy. However, there is no requirement that S be constructed using only those rules. The set Q of all rational numbers satisfies these conditions, so S could be Q. In other words, the conditions do not uniquely determine such a set S.\r\n\r\nThus a proof that says that 2/5, for example, *cannot* be in S is probably not a complete solution to this problem. I believe that Albanian Eagle's solution is the only complete one I have seen posted so far.\r\n\r\nColin", "Solution_30": "I think what is usually meant is that $ \\frac{2}{5}$ is not in the intersection of all such $ S$.", "Solution_31": "Yes, however, what if we do construct $ S$ according to those rules?\r\nSo we take $ S \\equal{} \\bigcup _{i \\equal{} 0}^{\\infty}S_{i}$; then $ S$ will satisfy the conditions of the problem however will not contain $ \\frac {4}{13}$. I think this works now.", "Solution_32": "[quote=\"Kent Merryfield\"]Let $ S$ be a set of rational numbers such that \n\n(a) $ 0\\in S;$\n\n(b) If $ x\\in S$ then $ x \\plus{} 1\\in S$ and $ x \\minus{} 1\\in S;$ and\n\n(c) If $ x\\in S$ and $ x\\notin\\{0,1\\},$ then $ \\frac {1}{x(x \\minus{} 1)}\\in S.$\n\nMust $ S$ contain all rational numbers?[/quote]\r\n\r\nFrom (a) and (b), all non-negative integers are $ \\in S$. From (3), if $ 2\\in S$ then $ \\frac{1}{2}\\in S$, and $ \\frac{1}{\\left(\\minus{}\\frac{1}{2} \\frac{1}{2}\\right)}\\equal{}\\minus{}4\\in S$.\r\n\r\nNow, suppose we had $ \\minus{}\\frac{1}{4}$. Is this an element of $ S$? To find that, we need to find $ x|Z\\plus{}\\frac{1}{x\\left(x\\minus{}1\\right)}\\equal{}\\minus{}\\frac{1}{4}$, where $ Z$ is an integer.\r\n\\[ 4Z(x^2\\minus{}x)\\plus{}4\\equal{}\\minus{}x^2\\plus{}x\\]\r\n\\[ (4Z\\plus{}1)x^2\\minus{}(4Z\\plus{}1)x\\plus{}4\\equal{}0\\]\r\n\\[ x\\equal{}\\frac{(4Z\\plus{}1)\\pm \\sqrt{16Z^2\\minus{}56Z\\minus{}15}}{8Z\\plus{}2}\\]\r\nEvaluate the discriminant such that it equals 0.\r\n\\[ 16Z^2\\minus{}56Z\\minus{}15\\equal{}0\\]\r\n\\[ Z\\equal{}\\frac{56\\pm \\sqrt{3136\\plus{}960}}{32}\\equal{}\\frac{56\\pm \\sqrt{4096}}{32}\\equal{}\\frac{56\\pm 64}{32}\\equal{}\\frac{7}{4} \\pm 2\\notin \\textbf{Z}\\]\r\nThis is the maximum value the discriminant can take. The only other integer discriminant that arises from $ 16Z^2\\minus{}56Z\\minus{}15\\equal{}k^2, k\\in \\textbf{Z}$ is 0 at $ k\\equal{}8$, which produces $ Z\\equal{}\\frac{7}{4}$. Therefore, an x value that satisfies the equation does not exist, which means that $ \\minus{}\\frac{1}{4}\\notin S$. Therefore, it is not necessary that $ S$ contain all rational numbers.", "Solution_33": "$S$ will contain only numbers of the form $\\frac{a}{b}$, where $b\\in\\mathbb N$, $\\gcd(a,b)=1$ and $a$ or $-a$ is a quadratic residue modulo $b$. Hence for instance $\\frac{2}{5}$ is not achievable.", "Solution_34": "Any ????????" } { "Tag": [ "linear algebra", "matrix", "linear algebra unsolved" ], "Problem": "How to prove that for a symmetric matrix A, if it have two eigenvalues a1 and a2 with corresponding eigenvectors v1 and v2, then v1 and v2 must be orthogonal to each other?", "Solution_1": "Of course we are assuming that $ a_1\\ne a_2.$\r\n\r\nWe're given three things: $ Av_1\\equal{}a_1v_1,Av_2\\equal{}a_2v_2,$ and $ A^T\\equal{}A.$\r\n\r\n$ a_1v_1^Tv_2\\equal{}a_1v_2^Tv_1$ (since any $ 1\\times 1$ matrix is symmetric)\r\n\r\n$ \\equal{}v_2^T(a_1v_1)\\equal{}v_2^TAv_1$\r\n\r\n$ \\equal{}(v_2^TAv_1)^T$ (again, since any $ 1\\times 1$ is symmetric)\r\n\r\n$ \\equal{}v_1^TA^Tv_2\\equal{}v_1^TAv_2$ (since $ A^T\\equal{}A$)\r\n\r\n$ \\equal{}v_1^T(a_2v_2)\\equal{}a_2v_1^Tv_2.$\r\n\r\nSo $ a_1v_1^Tv_2\\equal{}a_2v_1^Tv_2$ which implies that either $ a_1\\equal{}a_2$ or $ v_1^Tv_2\\equal{}0.$ If we're given that $ a_1\\ne a_2,$ then $ v_1$ and $ v_2$ must be orthogonal.\r\n\r\nNote that the matrix product $ v_1^Tv_2$ is simply another way of writing the inner product $ \\langle v_1,v_2\\rangle$ or $ v_1\\cdot v_2.$\r\n\r\n----\r\n\r\nThis whole argument was written assuming you were talking about real symmetric matrices and real inner products. A very similar argument will work for Hermitian matrices and complex inner products, but we will have to make some careful adjustments to the notation." } { "Tag": [ "group theory", "abstract algebra", "linear algebra", "matrix", "superior algebra", "superior algebra unsolved" ], "Problem": "Let $ K$ be a normal subgroup of order 2 in the group $ G$. Show that $ K$ lies in the centre of $ G$, that is $ kg \\equal{} gk$ for all $ k \\in K$ and $ g \\in G$.\r\n\r\nDescribe a surjective homomorphism of the orthogonal group $ O(3)$ onto $ C_2$ and another onto the special orthogonal group $ SO(3)$.\r\n\r\nFor the first part I have that since $ geg^{\\minus{}1} \\equal{} e \\quad \\forall \\quad g \\in G$, $ e$ is conjugate to itself, so the other element $ k$ in $ K$ must be conjugate to itself. So $ \\{e, k\\}$ is a union of conjugacy classes of size 1 and is contained in the center of $ G$.\r\n\r\nFor the second part, can we map all elements $ A \\in O(3)$ with $ \\det (A) \\equal{} 1$ to $ I$, and all elements $ B \\in O(3)$ with $ \\det (B) \\equal{} \\minus{}1$ to $ \\minus{}I$? Since matrix multiplication is the operation concerned and $ \\det(AB) \\equal{} \\det(A) \\cdot \\det(B) \\quad \\forall \\quad A, B \\in O(3)$, the conditions for homorphism are satisfied.\r\n\r\nAny clues for the surjective homomorphism onto the special orthogonal group?", "Solution_1": "For the second problem, note that $ O_3\\cong SO_3\\times C_2$ by the map $ O_3\\to SO_3\\times C_2$: $ A\\mapsto (\\frac{1}{\\det A}A, \\det A)$. So surjective maps to $ SO_3$ and $ C_2$ are given by projection, i.e. $ A\\mapsto\\frac{1}{\\det A} A$ and $ A\\mapsto \\det A$.", "Solution_2": "It's rather simple in hindsight :maybe: Thanks!" } { "Tag": [ "search", "logarithms", "calculus", "complex numbers", "geometric series", "real analysis", "real analysis unsolved" ], "Problem": "Find all $ z \\in \\mathbb{C}$ such that $ \\sum_{n\\equal{}1}^\\infty \\frac{z^n}{n}$ converges.", "Solution_1": "So what have you done so far?", "Solution_2": "[quote=\"WWW\"]So what have you done so far?[/quote]\r\nI am pretty sure that I can use the Abel's Test, and from that test usually the solution is $ | z | < 1$ (I used Mathematica to check for random complex numbers and it is true) . But I don't know exactly how to apply the Abel's Test.", "Solution_3": "Easy tests tell us: convergence when |z|<1 and divergence when |z|>1. So the main part of the problem is to analyze the case |z|=1 .", "Solution_4": "I've posted here on this in detail, although it might have been several years ago and my own posts aren't always the easiest things to look for. Possible key words include \"summation by parts.\"", "Solution_5": "[quote=\"Kent Merryfield\"]I've posted here on this in detail, although it might have been several years ago and my own posts aren't always the easiest things to look for. Possible key words include \"summation by parts.\"[/quote]\r\n\r\nhttp://www.mathlinks.ro/viewtopic.php?search_id=1150242020&t=176509\r\nUsing Mathematica i got the same, that the series converges to $ \\minus{}\\log (1\\minus{}z)$. Is there an easy way to prove it?", "Solution_6": "Thanks. That's one of my more complete explanations, so it's the right link.\r\n\r\nGeneral principle: as long as you stay strictly inside the radius of convergence, you can integrate and differentiate power series term by term as if they were polynomials.\r\n\r\nLet $ f(z)\\equal{}\\sum_{n\\equal{}1}^{\\infty}\\frac{z^n}{n}$ and suppose $ |z|<1.$\r\n\r\nThen $ f'(z)\\equal{}\\sum_{n\\equal{}1}^{\\infty}z^{n\\minus{}1}.$\r\n\r\nThat is a geometric series, and we know how to sum geometric series. $ f'(z)\\equal{}\\frac1{1\\minus{}z}.$\r\n\r\nIntegrate that (with the appropriate knowledge of the calculus of the logarithm in the complex plane) to get that $ f(z)\\equal{}\\minus{}\\log(1\\minus{}z)\\plus{}C.$ Then choose a branch of the logarithm such that $ \\log(1)\\equal{}0$ and evaluate the series at $ z\\equal{}0$ to nail down that constant of integration. We then get that $ f(z)\\equal{}\\minus{}\\log(1\\minus{}z)$ for $ |z|<1.$\r\n\r\nTo extend that back to the appropriate values on the boundary circle, $ |z|\\equal{}1$ but $ z\\ne 1,$ quote Abel's Theorem.", "Solution_7": "[quote=\"Kent Merryfield\"]Thanks. That's one of my more complete explanations, so it's the right link.\n[/quote]\r\nThanks a lot for the extensive explanation." } { "Tag": [ "inequalities", "function", "parameterization", "analytic geometry", "geometry", "3D geometry" ], "Problem": "Let $ x$, $ y$, and $ z$ be non-negative real numbers such that\r\n\\[ 4^{\\sqrt {5x \\plus{} 9y \\plus{} 4z}} \\minus{} 68 \\cdot 2^{\\sqrt{5x \\plus{} 9y \\plus{} 4z}} \\plus{} 256 \\equal{} 0\r\n\\] What is the product of the minimum and maximum values of $ x \\plus{} y \\plus{} z$?", "Solution_1": "[hide=\"Solution\"]Let $ y\\equal{}2^{\\sqrt{5x\\plus{}9y\\plus{}4z}}$. We have $ y^2\\minus{}68y\\plus{}256\\equal{}0 \\implies y\\equal{}4, \\; y\\equal{}64$\n\nThus, $ \\sqrt{5x\\plus{}9y\\plus{}4z} \\equal{} 2$ or $ 6$, and $ 5x\\plus{}9y\\plus{}4z\\equal{}4$ or $ 36$.\n\nBy elementary linear programming, we must only test the endpoints of the regions.\n\n$ \\blacktriangleright$ The minimum value occurs when $ 5x\\plus{}9y\\plus{}4z\\equal{}4$, and $ (x,y,z)\\equal{} \\left(0, \\; \\frac{4}{9}, \\; 0 \\right) \\implies x\\plus{}y\\plus{}z\\equal{} \\frac{4}{9}$\n\n$ \\blacktriangleright$ The maximum value occurs when $ 5x\\plus{}9y\\plus{}4z\\equal{}36$, and $ (x,y,z) \\equal{} (0,0,9) \\implies x\\plus{}y\\plus{}z\\equal{}9$\n\nThe product is $ \\boxed{4}$\n\n[/hide]", "Solution_2": "[quote=\"The Zuton Force\"]By elementary linear programming, we must only test the endpoints of the regions.\n[/quote]\r\nCould you elaborate a little more on this? :P I'm not well-versed with linear programming", "Solution_3": "You don't need linear programming:\r\n\r\n[hide]For the minimum, $ 9x\\plus{}9y\\plus{}9z\\ge 5x\\plus{}9y\\plus{}4z\\ge 4$, so $ x\\plus{}y\\plus{}z\\ge 4/9$.\n\nFor the maximum, $ 4x\\plus{}4y\\plus{}4z\\le 5x\\plus{}9y\\plus{}4z\\le 36$, so $ x\\plus{}y\\plus{}z\\le 9$.\n\nWe can see that both of these are achievable, so they are the actual min and max.[/hide]", "Solution_4": "Does this mean to say that in constructing such inequalities it is alright to drop some terms or add them in? And does this prove that no better bound can be achieved? :P", "Solution_5": "[quote=\"aidan\"]Does this mean to say that in constructing such inequalities it is alright to drop some terms or add them in? And does this prove that no better bound can be achieved? :P[/quote]\r\n\r\nI'm not sure exactly what you're asking, but for example $ 4x\\plus{}4y\\plus{}4z\\le 5x\\plus{}9y\\plus{}4z$ is true because we're adding in non-negative terms ($ x$ and $ y$). The bounds $ 4/9$ and $ 9$ are the best bounds because there are values that achieve these bounds, namely $ (0,4/9,0)$ and $ (0,0,9)$. The reason we can get away with adding terms without ending up with weak bounds is because the variables we're adding are 0 in the equality case. If you know the equality case you should always keep it in mind when trying to prove inequalities - if you make a step that changes the equality case, that step probably isn't going to work to prove the inequality.", "Solution_6": "[u]To elaborate on what I meant by \"linear-programming\"[/u]\r\n\r\nSuppose you have a linear function of $ x,y,z$ of the form $ F(x,y,z) \\equal{} a_1 x \\plus{} a_2 y \\plus{} a_3 z$. Now, we want to evaluate this along some line in 3-space. The line $ \\ell$ can be parametrized in the form $ (x,y,z) \\equal{} (b_1 t, b_2 t, b_3 t)$ with respect to some parameter $ t$. Therefore, if we want to evaluate the linear function along the line, we can write it in terms of the single paramter $ t$ as follows:\r\n\r\n$ F_{\\ell} (t) \\equal{} F(x(t),y(t),z(t)) \\equal{} a_1 (b_1 t) \\plus{} a_2 (b_2 t) \\plus{} a_3 (b_3 t) \\equal{} (a_1 b_1 \\plus{} a_2 b_2 \\plus{} a_3 b_3) t$\r\n\r\nSo, as we move along $ \\ell$ (as we increase or decrease), the function $ F_{\\ell} (t)$ is either increasing with $ t$ or decreasing with $ t$, depending on the sign of $ a_1 b_1 \\plus{} a_2 b_2 \\plus{} a_3 b_3$. As a consequence, [b][color=red]this means that if we wish to evaluate a linear function on the coordinates over a line segment, one endpoint will produce the minimum and one will produce the maximum.[/color][/b] There is a qualifier to this, though: if $ a_1 b_1 \\plus{} a_2 b_2 \\plus{} a_3 b_3$ as shown in the original demonstration, is equal to $ 0$, then we have a constant value across the line. However, while optimization does not occur only at the endpoints, it does occur at the endpoints.\r\n\r\nWhat does this mean? Suppose we have a convex n-D polytope shaped region in n-space. Clearly, optimization cannot occur in the interior of this, because we can draw a segment connecting two boundary points which contains this interior point, and optimization will occur on one of the two end-points; namely, optimization must occur on the boundary. Now, consider the (n-1)-D polytopes that make up the boundary, and execute the same argument. Continue doing so, and you will find that [b][color=blue]optimization, if possible must occur at one of the vertices[/color][/b].\r\n\r\n[hide=\"A concrete example\"]Say we have a cube. Take any point in the cube. We can create a line segment connecting two opposite faces that contains this interior point. Since optimization must occur on one of the endpoints, which are on the faces of the cube, optimization does not occur in the interior, but on the boundary (the collection of faces, edges, and vertices).\n\nNow, on each face, any interior point of the face is also an interior point of a segment that connects two edges. Thus, optimization does not occur on the interior of the faces, but on the edges.\n\nNow, the edge is a line segment, so optimization on the edge must occur at a vertex.\n________________________________________________________________________[/hide]\r\nIn this case, we have two triangular regions. Each has 3 vertices, and we wish to optimize a linear function over these two triangles. The principle of linear programming implies that we only need to test the 6 vertices.", "Solution_7": "Thanks a lot guys! That was really helpful :)" } { "Tag": [ "linear algebra", "matrix", "vector", "combinatorics unsolved", "combinatorics" ], "Problem": "this nice problem is from a link that grobber introduced us.\r\nThere are 120 members in the Israeli parliament, and 40 committees. A committee can be assembled only when the number of committee members that are now present in the parliament hall, is odd (so that votes would be decisive). Show a way to assign the parliament members to committees, so that in all times, except for when the hall is completely empty, there would be at least one committee that can be assembled, or show that it is impossible.", "Solution_1": "Hi!\r\nCan there be a person in more than one committee", "Solution_2": "If each person had to be in only one committee then we could just put an even number of members from each committee in the hall so the problem would be trivial. That's why I think it's crucial for some people to be members of more than one committee.", "Solution_3": "Actually, this appears to be rather easy, unless I'm making yet another one of my stupid mistakes :?\r\n\r\nWe construct a $40\\times 120$ matrix $A=(a_{ij})_{i,j}$ s.t. $a_{ij}$ is $1$ if the $j$'th person is a member of the $i$'th committee and $0$ otherwise. We now work in $\\mathbb F_2$, the field with $2$ elements. The problem asks for such a matrix $A$ that the system $A\\cdot {\\bf x}={\\bf 0}$ has the null vector as its unique solution, where ${\\bf x}$ is a column vector with $120$ entries, and ${\\bf 0}$ is a column vector with $40$ entries.\r\n\r\nWe must thus construct a matrix $A$ which has rank $40$ (maximal rank, since it has $40$ rows). We can construct such a matrix by appending an all-$1$ $40\\times 80$ matrix to the right of a $I_{40}$ (the identity $40\\times 40$ matrix). It's clear that the rows of $A$ are independent (over $\\mathbb F_2$), as we intended.\r\n\r\nAm I missing something here? :?", "Solution_4": "Maybe I am the one who understood wrong, but it seems that the answer is completely opposite. I think $A$ must have rank $120$ not $40$ -- which is impossible.\r\nFor example, consider you example matrix $A$: if we get $x$ to be the column vector with $118$ first entries $0$ and only the last $2$ being $1$ then we obviously have $A$[b]x[/b]$=0$.\r\nAm I right?", "Solution_5": "Actually, you are right. I don't know what the heck I was thinking :? So the answer is negative, then.\r\n\r\nEdit: I forgot to say this wheh I replied, but I am [b]very[/b] embarrassed about this. :blush:", "Solution_6": "So problem is solved or not?", "Solution_7": "Yes, as far as I can tell, the problem is solved now and the answer is negative: the members of the parliament cannot form committees so that they respect the conditions of the problem.", "Solution_8": "Now that I read the thread, I think that too.\r\nCool, one more that we will move soon into the solved problem section :D \r\n\r\nPierre.", "Solution_9": "[quote=\"iura\"]I think $A$ must have rank $120$ not $40$ -- which is impossible.[/quote]\r\nI don't know why I didn't realize it from the beginning :blush:" } { "Tag": [ "trigonometry", "trig identities", "Law of Cosines" ], "Problem": "In triangle ABC, AB = 7 units, BC = 8 units, and AC = 9 units. A point D is located on side AC so that BD = 7 units. Find AD. Express your answer as a rational number.", "Solution_1": "My Answer: [hide]Let x be the side AD; in the triangle 7: 7+x is proportional to 8 : 16, so $\\frac{7}{7+x} = \\frac{8}{16}$, so $112 = 56 + 8x$, $56 = 8x$ and $x = 7$.[\\hide][/hide]\r\nOops I got that wrong...", "Solution_2": "[hide=\"Solution\"]Denote $x=AD.$ Double use of the Cosine Law:\n\nFrom $\\triangle ABC$:\n\n$\\cos A={{7^2+9^2-8^2}\\over{2\\cdot 7\\cdot 9}}={11\\over 21}$\n\nFrom $\\triangle ABD$:\n\\begin{eqnarray*}7^2=7^2+x^2-2\\cdot 7\\cdot x\\cdot\\cos A &\\implies& 0=x^2-14\\cdot{11\\over 21}x\\\\ &\\implies& x=14\\cdot{11\\over 21}={22\\over 3}.\\end{eqnarray*}[/hide]", "Solution_3": "Wow nice job, that is right....", "Solution_4": "[hide]\n\nStewart's Theorem: $AB^2\\cdot CD+BC^2\\cdot AD=AC(BD^2+AD\\cdot CD)$\n\nSo $7^2\\cdot (9-x)+8^2\\cdot x=9(7^2+x(9-x))$, which gives $x=\\frac{22}{3}$.\n\nBy the way, the theorem can be proven by using Law of Cosines twice, just like Farenhajt's solution. [/hide]" } { "Tag": [ "inequalities" ], "Problem": "If $p^3+q^3=2$, prove $p+q \\leq 2$.", "Solution_1": "By the Power-mean inequality,\r\n\r\n$4(p^3+q^3)\\geq (p+q)^3$\r\n\r\nHence $8\\geq (p+q)^3$, or $p+q\\leq 2$\r\n\r\nThat's if you want it to hold for nonnegative reals\r\n\r\nFor the case of real numbers, say that $q<0$, so that we may rewrite as\r\n\r\n$p^3-q^3=2$ and prove $p-q\\leq 2$ (where $p,q>0$)\r\n\r\nThen we get\r\n\r\n$2=p^3-q^3=(p-q)(p^2+pq+q^2)$\r\n\r\nSay $p-q>2$.\r\n\r\nThen $p^2+pq+q^2<1$, so that $p<1$ and hence $p^3-q^3<1$, a contradiction.", "Solution_2": "You can use Discriminant. ;)", "Solution_3": "Or :\r\n\r\n$(p+q)^3 = 4*(p^3 + q^3) - 3*(p+q)*(p-q)^2$\r\n\r\nIf $p+q < 0$ then it's true obviously and else above equality makes it" } { "Tag": [ "floor function", "number theory", "greatest common divisor", "analytic geometry", "symmetry", "number theory unsolved" ], "Problem": "Show that for all positive integers $m$ and $n$,\r\n\r\n$\\gcd(m,n)=m+n-mn+2\\sum_{k=0}^{m-1}\\lfloor \\frac{kn}{m}\\rfloor$", "Solution_1": "Let $gcd(m,n)=k$ $m=dp$ $n=dq$. $gcd (p,q)=1$. Call $r_{p}(j)$ for any $j\\in\\mathbb N$ the unique number $0\\leq r_{p}(j)\\leq p$ such that $r_{p}(j)\\equiv j\\;\\;(mod\\;p)$.\r\n\r\n$2\\sum_{k=0}^{m-1}\\lfloor \\frac{kn}{m}\\rfloor =2\\sum_{k=0}^{dp-1}\\lfloor \\frac{kq}{p}\\rfloor=$\r\n\r\n$=2\\sum_{k=0}^{dp-1}(\\frac{kq}{p}-\\left\\{ \\frac{kq}{p}\\right\\})= 2\\sum_{k=0}^{dp-1}(\\frac{kq}{p}-\\frac{r_{p}(kq)}{p})=$\r\n\r\n$=\\frac{2}{p}[q\\sum_{k=0}^{dp-1}(k)-\\sum_{k=0}^{dp-1}r_{p}(kq)]= \\frac{2}{p}[\\frac{q(dp)(dp-1)}{2}-\\sum_{j=0}^{d-1}\\sum_{k=0}^{p-1}r_{p}((jp+k)q))]=$\r\n\r\n$=\\frac{2}{p}[\\frac{q(dp)(dp-1)}{2}-\\sum_{j=0}^{d-1}\\sum_{k=0}^{p-1}k]=\\frac{2}{p}[\\frac{q(dp)(dp-1)}{2}-\\sum_{j=0}^{d-1}\\frac{p(p-1)}{2}]=$\r\n\r\n$=\\frac{2}{p}[\\frac{q(dp)(dp-1)}{2}-\\frac{dp(p-1)}{2}]=dq(dp-1)-d(p-1)=mn-m-n+d$\r\n\r\n$m+n-mn+2\\sum_{k=0}^{m-1}\\lfloor \\frac{kn}{m}\\rfloor= m+n-mn+mn-m-n+d=d=gcd(m,n)$", "Solution_2": "it is in TST Taiwan :lol: very :P", "Solution_3": "Consider the grid of the $(m+1)(n+1)$ integer points with coordinate $(x,y)$ such that $0 \\leq x \\leq m$ and $0 \\leq y \\leq n.$\r\nLet $\\Delta$ be the line with equation $y= \\frac{nx}{m}$ which then goes through the origin and the upper-right vertex $(m,n)$.\r\n\r\nLet $d = \\gcd (m,n)$. Let $m=da, n=db.$\r\nIt is easy to verify that the integer points of the grid which belong to $\\Delta$ are those of the form $(ka,kb)$ where $k=0,1,...,d.$ Thus there exactly $d+1$ integer points on the line.\r\n\r\nNow, the number of integer points which are under or on the line and with $x=k$ is exactly $[ \\frac{kn}{m}]+1.$ Summing over $k=0,1,...,m$ this leads to the total number of points of the grid which are under or on the line $\\Delta.$\r\nBy symmetry there are as many points above or on the line. Only the points which belong to $\\Delta$ are counted twice.\r\nIt follows that :\r\n$2 \\sum_{k=0}^{m}( [\\frac{kn}{m}]+1) = (m+1)(n+1)+d+1$.\r\nThis is the desired result.\r\n\r\nPierre." } { "Tag": [ "summer program", "MathPath" ], "Problem": "This is a problem from the 2004 qualifying quiz for the Mathpath camp.\r\n\r\nFor positive integers m, n define m \u2191 n as follows:\r\nm \u2191 1 = m\r\nm \u2191 (n+1) = m^(m\u2191n).\r\n\r\na) Find the smallest integer n so that 2 \u2191 n is greater than a googol (1 followed by 100 zeros).\r\n\r\nb) Find the smallest integer n so that 2 \u2191 n is greater than a googolplex (10^googol).\r\n\r\nI got the first part, but I just can't think about numbers as big as a googolplex. Can someone help me with this problem?", "Solution_1": "This question was on the MathPath Qualifying Quiz, wasn't it?", "Solution_2": "[quote=\"nr1337\"]This question was on the MathPath Qualifying Quiz, wasn't it?[/quote]\r\nIt says so at the very beginning of the question...\r\nmaybe you could use the fact that 1024 is close to 1000?\r\nand i have no idea how mathpath works so are you allowed to use a calculator?", "Solution_3": "OK, so for part 2:\r\n\r\nOur 2 \u2191 n go 2, 2^2, 2^4, 2^16, 2^65536, 2^(2^65536)...\r\n \r\nWe know 2^65536 is the least one greater than a googol, so we do not need to check the cases below it for the googolplex. So, checking for this one, we get 2^65536 ?> 10^(10^100). But, the base on the left is less than the base on the right, and the exponent on the left is less than the exponent on the right, so this clearly is not true.\r\n\r\nWe check the next case: 2^(2^65536) ?> 10^(10^100).\r\nTaking log of both sides, \r\nwe get 2^65536 * log(2) ?> 10^100.\r\nWe know 2^7 > 10^2, so we may substitute 2^350 for 10^100 on the right side --> \r\n2^65536 * log(2) ?> 2^350 --> \r\nlog(2) ?> 2^(350-65536).\r\nlog(2) > 0.25, \r\nwhich is clearly greater than 2^(-65186). \r\n\r\nThus, n = 6.\r\n\r\nI hope my explanation is OK.", "Solution_4": "[quote=\"Scrambled\"][quote=\"nr1337\"]This question was on the MathPath Qualifying Quiz, wasn't it?[/quote]\nIt says so at the very beginning of the question...\n[/quote]\r\n\r\nOops... :oops:" } { "Tag": [ "calculus", "number theory" ], "Problem": "It struck me the other day that many introductory combinatorics and number theory topics show up in some introduction to discrete mathematics courses. \r\n\r\nPeople who wrote the earliest math contests could not base their ideas and tests off previous math competitions because they didn't exist, right? Where do most adults who write non-calculus high school math contests learn their math to write the contests? Some topics like number theory and combinatorics are only lightly touched in some high school curricula so would they have had to either do one of the following? \r\n\r\n1) be a math major and study outside high school topics like number theory and combinatorics in depth\r\n2) somehow remember number theory or combinatorics touched on in college courses such as discrete mathematics?", "Solution_1": "I realize this is somewhat evasive, but in fact, the earliest math contest writers did not make as interesting problems as the contest writers do now. Try comparing the IMO's in the 70's to the IMO's now." } { "Tag": [ "\\/closed" ], "Problem": "I have a doubt regarding sending private messages?\r\n\r\n\r\nWhen do I know---- that the message I had 'typed' and sent, is actually sent to the desired person? I have tried sending two messages just now, but I still see them in the 'outbox' , and not in the 'sent box', does it take time to send these messages? \r\n\r\nHow should I know how to correct this problem.", "Solution_1": "They stay in the outbox until the addressee reads them. This is the way for you to know when your message has been actually read, not just delivered to the other person's mailbox :)." } { "Tag": [ "trigonometry", "algebra unsolved", "algebra" ], "Problem": "How can we prove that, for any n >= 2,\r\n\r\n (sin(pi/n))*(sin(2pi/n))*...*(sin((n-1)pi/n)) = n/(2^(n-1))?", "Solution_1": "See here.\r\n\r\n[url]http://www.mathlinks.ro/Forum/viewtopic.php?p=819011#819011[/url]" } { "Tag": [ "limit", "inequalities", "logarithms", "factorial", "calculus", "integration", "geometry" ], "Problem": "$ \\lim_{n\\to\\infty} \\sqrt[n\\plus{}1] { \\prod_{k\\equal{}0}^{n} {n \\choose k} }$", "Solution_1": "The limit is $ \\infty.$\r\n\r\nTempting as it is to get something out of the AM-GM inequality, the inequality sign has the wrong sense for that.\r\n\r\nLet $ A_n\\equal{}\\prod_{k\\equal{}0}^n\\binom{n}{k}.$ We're trying to find $ \\lim_{n\\to\\infty}\\sqrt[n\\plus{}1]{A_n}.$\r\n\r\nIf I can show that $ \\lim_{n\\to\\infty}\\frac{A_{n\\plus{}1}}{A_n}\\equal{}\\infty,$ it will follow that $ \\lim_{n\\to\\infty}\\sqrt[n\\plus{}1]{A_n}\\equal{}\\infty.$\r\n\r\n$ \\frac{A_{n\\plus{}1}}{A_n}\\equal{}\\prod_{k\\equal{}0}^n\\frac{\\binom{n\\plus{}1}{k}} {\\binom{n}{k}}\\equal{} \\prod_{k\\equal{}0}^n\\frac{n\\plus{}1}{n\\plus{}1\\minus{}k}\\equal{} \\prod_{k\\equal{}0}^n\\left(1\\minus{}\\frac{k}{n\\plus{}1}\\right)^{\\minus{}1}.$\r\n\r\nTake the logarithm of that:\r\n\r\n$ \\ln\\left(\\prod_{k\\equal{}0}^n\\left(1\\minus{}\\frac{k}{n\\plus{}1}\\right)^{\\minus{}1}\\right)\\equal{} \\minus{}\\sum_{k\\equal{}0}^n\\ln\\left(1\\minus{}\\frac{k}{n\\plus{}1}\\right)$\r\n\r\n$ >\\sum_{k\\equal{}0}^n\\frac{k}{n\\plus{}1}\\equal{}\\frac{n}2$\r\n\r\nSince this tends to infinity, we are done. We have that $ \\frac{A_{n\\plus{}1}}{A_n}>e^{n/2}.$", "Solution_2": "So, a stronger question: what does $ \\ln \\prod_{k\\equal{}0}^n \\binom{n}{k}$ look like asymptotically?\r\nUsing the factorial representation of the binomial coefficients, it's $ \\ln\\frac{(n!)^{n\\plus{}1}}{\\prod_{k\\equal{}0}^n (k!)^2}\\equal{}\\sum_{j\\equal{}1}^n (2j\\minus{}n\\minus{}1)\\ln j$\r\n\r\nWe estimate that sum with an integral: $ \\int_1^n (2x\\minus{}n\\minus{}1)\\ln x\\,dx\\approx \\sum_{j\\equal{}1}^n (2j\\minus{}n\\minus{}1)\\ln j\\minus{}\\frac12\\ln 1\\minus{}\\frac12(n\\minus{}1)\\ln n$ by the trapezoid rule. The error is a sum of terms involving the second derivative $ \\frac2x\\plus{}\\frac{n\\plus{}1}{x^2}$ evaluated on each interval, which is $ O(n)$ as $ n\\to\\infty$. Most of that error comes from the first few terms of the sum, and from the $ \\frac{n\\plus{}1}{x^2}$ term of the second derivative.\r\n\r\nThe integral $ \\int_1^n (2x\\minus{}n\\minus{}1)\\ln x\\,dx$ evaluates to\r\n$ \\left[x^2\\ln x\\minus{}\\frac12x^2\\minus{}(n\\plus{}1)(x\\ln x\\minus{}x)\\right]_1^n$\r\n$ \\equal{} n^2\\ln n\\minus{}\\frac12n^2\\minus{}(n^2\\plus{}n)\\ln n\\plus{}(n^2\\plus{}n)\\plus{}(n\\plus{}1)$\r\n$ \\equal{}\\frac12n^2\\plus{}n\\ln n\\plus{}2n\\plus{}1$\r\n\r\nCombining with the error estimate and the additional $ \\frac12(n\\minus{}1)\\ln n$ term, $ \\ln\\left( \\prod_{k\\equal{}0}^n \\binom{n}{k}\\right)\\equal{}\\frac12n^2\\plus{}\\frac32n\\ln n\\plus{}O(n)$ as $ n\\to\\infty$. It shouldn't be hard to get a decent bound on the $ O(n)$ term as well.\r\nKent's method essentially found the dominant $ \\frac12n^2$ term of the logarithm.", "Solution_3": "$ \\lim_{n\\to\\infty} \\sqrt[n\\plus{}1] { \\prod_{k\\equal{}0}^{n} {n \\choose k} }\\equal{}\\sqrt{e}$ \r\nwhy?", "Solution_4": "That's false. You'll only get that if you take an $ n^2$ root.\r\n\r\nPlease make better titles for your posts. \"Limit\" says nothing about the content, and multiples of the same useless title are even worse.", "Solution_5": "Hey int, ZetaX posted some good guidelines for making posts. They're here:\r\n\r\nhttp://www.artofproblemsolving.com/Forum/viewtopic.php?t=135914&sid=b3cd8382b67f0eb870ad0db0f35691e8\r\n\r\nMaybe we could make a sticky in each forum about such guidelines. How about this, let me try: \"Limit of n'th root of combinatorial products\"; it'll help with searching if the title is more meaningful." } { "Tag": [ "real analysis", "real analysis unsolved" ], "Problem": "$0\\frac{1}{|i-j|}$. And for $a=1$ ?", "Solution_1": "Look like an olympiad problem", "Solution_2": "Where does $a$ (which, by the way, cannot be an integer, as you wrote, if $00$.\n[/hide]", "Solution_3": "Yes, and why can't the nominator (edit: I mean numberator, sorry) be divisible by 2 too?\r\n\r\n1/2 + 1/3 + 1/4 = (12+8+6)/24", "Solution_4": "if $h_n=1+\\frac{1}{2}+\\frac{1}{3}+...+\\frac{1}{n}$\r\nLet $k$ be the largest power or $2$ less than or equal to $n$ \r\n\r\nNow if sum is $h_n=p$, multiply both side by $\\frac{k}{2}*$(product of all odd numbers up to $n$ ) then except for the term corresponding to $\\frac{1}{k}$ every other term would be an integer so: contradiction. \r\n\r\nbesides if harmanic sequence didn't start with $1$, then the sum would always be a odd number over an even number", "Solution_5": "[quote=\"Stijn\"]Yes, and why can't the nominator be divisible by 2 too?\n\n1/2 + 1/3 + 1/4 = (12+8+6)/24[/quote]\r\n\r\nI didn't tell that it isn't so, but the denominator is divisible by two everytime! (and so different from one)", "Solution_6": "I don't really get the point ZetaX is trying to make, but amirhtlusa's proof makes perfectly sense, well done!", "Solution_7": "I am telling exactly what amirhtlusa did explicitly!\r\nIf the denominator of a fraction contains a factor $2$ in it's prime factorisation ( independent from having coprime numerator or not!, or in other words: even when they are coprime ) it can't be an integer.", "Solution_8": "[quote=\"ZetaX\"]I am telling exactly what amirhtlusa did explicitly!\nIf the denominator of a fraction contains a factor $2$ in it's prime factorisation ( independent from having coprime numerator or not!, or in other words: even when they are coprime ) it can't be an integer.[/quote]\r\n\r\nI'm kind of not getting this. 8/4 is an integer...", "Solution_9": "sorry when my explaination is bad: I mean: when independently how you write that rational number as fraction a factor $2$ occurs in the denominator, then it can't be an integer. Equivalent to that is the statement that when $2$ divides the denominator of the reduced fraction it can't be an integer.\r\nYour example doesn't work, since $\\frac{8}{4} = \\frac{2}{1}$ and $1$ is not even.", "Solution_10": "[quote=\"ZetaX\"]sorry when my explaination is bad: I mean: when independently how you write that rational number as fraction a factor $2$ occurs in the denominator, then it can't be an integer. Equivalent to that is the statement that when $2$ divides the denominator of the reduced fraction it can't be an integer.\nYour example doesn't work, since $\\frac{8}{4} = \\frac{2}{1}$ and $1$ is not even.[/quote]\r\n\r\nAh, you mean a fully reduced/simplified fraction.", "Solution_11": "Oh, sorry for my bad english :blush:", "Solution_12": "[quote=\"amirhtlusa\"]\nbesides if harmanic sequence didn't start with $1$, then the sum would always be a odd number over an even number[/quote]\r\n\r\nWell ZetaX pointed out that even if it does start with $1$, the sum is odd over even. It's basically the same as what you said, by considering the highest power of $2$.", "Solution_13": "[quote=\"paladin8\"][quote=\"amirhtlusa\"]\nbesides if harmanic sequence didn't start with $1$, then the sum would always be a odd number over an even number[/quote]\n\nWell ZetaX pointed out that even if it does start with $1$, the sum is odd over even. It's basically the same as what you said, by considering the highest power of $2$.[/quote]\r\n :yup: \r\n(i never disagreed :) )", "Solution_14": "Guys, read the question. It dosnt nescilarrly start at 1. It \"consuctuive terms\" you can prove that starting at one, consider the higher power of 2 under n. I think the same approach might work for the actual question", "Solution_15": "[quote=\"seamusoboyle\"]Guys, read the question. It dosnt nescilarrly start at 1. It \"consuctuive terms\" you can prove that starting at one, consider the higher power of 2 under n. I think the same approach might work for the actual question[/quote]\r\n\r\nYeah. Check ZetaX's first post. It's a summation from $n$ to $n+k$ and we can show it cannot be an integer the same way as amirhtlusa showed the harmonic series from $1$ cannot be an integer.", "Solution_16": "Whoops... :blush:" } { "Tag": [ "function", "logarithms", "calculus", "calculus computations" ], "Problem": "How to solve this function in respect to x: [b]c*x^d = a*b^x[/b], where a, b, c and d are constants.\r\n\r\nThanks in advance!", "Solution_1": "That is $ \\ln c\\plus{}d\\ln x\\equal{}\\ln a\\plus{}x\\ln b$. :maybe:", "Solution_2": "There is no general solution in terms of elementary functions.", "Solution_3": "I think should be $ b>0$\r\nI solve the case $ x>0$ (the case $ x<0$ have same method.\r\nBecause $ ac>0$ so enogh to solve \r\n$ \\Longleftrightarrow \\ln{c}+d\\ln{x}=\\ln{a}+x\\ln{b}$\r\n$ f(x)=\\ln{b}x+\\ln{a}-\\ln{c}-d\\ln{x}$\r\n$ f'(x)=\\ln{b}-\\frac{d}{x}$\r\nUse this to find out the inteval of root.\r\nThe case $ x<0$ change sign of $ a,c$ and dolve with the same method.", "Solution_4": "Sorry but i don't understand this point:\r\n\r\n[quote=\"TTsphn\"]\n$ \\Longleftrightarrow \\ln{c} + d\\ln{x} = \\ln{a} + x\\ln{b}$\n$ f(x) = \\ln{b}x + \\ln{a} - \\ln{c} - d\\ln{x}$\n$ f'(x) = \\ln{b} - \\frac {d}{x}$\n[/quote]", "Solution_5": "You take ln and consider this function.", "Solution_6": "I should prove this but i don't see how is it possible:\r\n\r\n[b]lnb = x(lnb - (lnx)/x)[/b]" } { "Tag": [ "number theory unsolved", "number theory" ], "Problem": "Find all integer triples $(a,b,c)$ satisfying: $2^a + 3^b = 5^c$ .", "Solution_1": "[quote=\"Lovasz\"]Find all integer triples $(a,b,c)$ satisfying: $2^a + 3^b = 5^c$ .[/quote]\r\n$3^b \\ and \\ 5^c \\ are \\ olds \\Rightarrow 2^a \\ is \\ even$\r\nIf a=1: $c\\neq0$ if c=1 $\\Rightarrow$ (1,1,1) is a solution\r\nif $c\\ge2$ In mod 25 result: $b\\equiv13(mod \\ 25)$\r\n$\\Rightarrow b\\equiv3(mod \\ 5)$\r\n$\\Rightarrow 3^b\\equiv5(mod \\ 11)$\r\n$\\Rightarrow 5^c\\equiv7(mod \\ 11)$(impossible)\r\nIf $a\\ge2$ $\\Rightarrow 5^c\\equiv1(mod \\ 4)$\r\n$\\Rightarrow 3^b\\equiv1(mod \\ 4)$\r\n$\\Rightarrow \\ b \\ is \\ even$\r\nIf c is old $\\Rightarrow 5^c\\equiv5(mod \\ 8)$\r\nsince b is even $\\Rightarrow 3^b\\equiv1(mod \\ 8)$\r\n$\\Rightarrow 2^a\\equiv4(mod \\ 8)$\r\n$\\Rightarrow$ a=2\r\n$\\Rightarrow$ $4+3^b\\equiv1(mod \\ 3)$\r\n$\\Rightarrow 5^c\\equiv1(mod \\ 3)$\r\n$\\Rightarrow$ c is even(contradiction)\r\n$\\Rightarrow$ c is even \r\n$\\Rightarrow$ $b=2x \\ and c=2y$\r\n$\\Rightarrow 2^a=(5^x+3^y)(5^x-3^y)$\r\n$\\Rightarrow 5^x+3^y=2^m \\ and \\ 5^x-3^y=2^n$\r\n$\\Rightarrow 2. 3^y=2^m-2^n; \\ m>n$\r\n$\\Rightarrow 2^n \\ is \\ even$\r\n$\\Rightarrow 3^y=2^{m-1}-2^{n-1}$ m-1>n-1\r\n$\\Rightarrow 2^{n-1} \\ is \\ old$\r\n$\\Rightarrow n=1$\r\n$\\Rightarrow 5^x-3^y=2$\r\nBy the proof above x=y=1\r\n$\\Rightarrow b=c=2$ $\\Rightarrow a=4$\r\n(4,2,2) is other solution\r\nHence the solutions are (1,1,1) and (4,2,2)" } { "Tag": [], "Problem": "\u0394\u03b5\u03af\u03be\u03c4\u03b5 \u03cc\u03c4\u03b9 $\\sum_{k=0}^n\\binom{n+k}{k}\\frac{1}{2^{k}}=2^n$", "Solution_1": "\u03a3\u03b9\u03bb\u03bf\u03c5\u03b1\u03bd \u03c0\u03bf\u03bb\u03c5 \u03bf\u03bc\u03bf\u03c1\u03c6\u03b7 \u03b1\u03c3\u03ba\u03b7\u03c3\u03b7 \u03b1\u03bb\u03bb\u03b1 \u03bc\u03b7\u03c0\u03c9\u03c2 \u03b5\u03b9\u03bd\u03b1\u03b9 \u03bb\u03b1\u03b8\u03bf\u03c2? \u0391\u03bb\u03b7\u03b8\u03b5\u03b9\u03b1 \u03b5\u03b9\u03b4\u03b5 \u03ba\u03b1\u03bd\u03b5\u03b9\u03c2 \u03c4\u03bf \u039d\u03b9\u03ba\u03bf \u03a1\u03b1\u03c0\u03b1\u03bd\u03bf \u03c3\u03c4\u03b7\u03bd \u03c4\u03b7\u03bb\u03b5\u03bf\u03c1\u03b1\u03c3\u03b7?\u039c\u03c0\u03c1\u03b1\u03b2\u03bf \u039d\u03b9\u03ba\u03bf \u03c0\u03b1\u03bd\u03c4\u03b1 \u03c4\u03b5\u03c4\u03bf\u03b9\u03b1 :) :) :)", "Solution_2": "\u039d\u03b1\u03b9, \u03b5\u03af\u03bd\u03b1\u03b9 \u03bb\u03ac\u03b8\u03bf\u03c2. \u03a4\u03bf \u03b4\u03b5\u03be\u03b9\u03cc \u03bc\u03ad\u03c1\u03bf\u03c2 \u03c0\u03c1\u03ad\u03c0\u03b5\u03b9 \u03bd\u03b1 \u03b5\u03af\u03bd\u03b1\u03b9 1.", "Solution_3": "\u039f\u03bc\u03c9\u03c2 \u03b3\u03b9\u03b1 \u03bd=3 \u03bd\u03bf\u03bc\u03b9\u03b6\u03c9 \u03b4\u03b5 \u03b2\u03b3\u03b1\u03b9\u03bd\u03b5\u03b9 1", "Solution_4": "\u039c\u03b5 \u03c3\u03c5\u03b3\u03c7\u03c9\u03c1\u03b5\u03af\u03c4\u03b5 :oops: \u0388\u03ba\u03b1\u03bd\u03b1 \u03c4\u03c5\u03c0\u03bf\u03b3\u03c1\u03b1\u03c6\u03b9\u03ba\u03cc \u03bb\u03ac\u03b8\u03bf\u03c2. \u03a4\u03ce\u03c1\u03b1 \u03c4\u03b7\u03bd \u03b4\u03b9\u03cc\u03c1\u03b8\u03c9\u03c3\u03b1 \u03be\u03b1\u03bd\u03b1\u03ba\u03bf\u03b9\u03c4\u03ac\u03be\u03c4\u03b5 \u03c4\u03b7\u03bd...... :P", "Solution_5": "\u0391\u03bd \u03ba\u03b1\u03b9 \u03b4\u03b5 \u03b2\u03bb\u03b5\u03c0\u03c9 \u03c4\u03b7\u03bd \u03b1\u03c3\u03ba\u03b7\u03c3\u03b7 \u03c3\u03c5\u03bc\u03c6\u03c9\u03bd\u03c9 \u03c1\u03b5 \u03c6\u03b9\u03bb\u03b5:\u03c0\u03c1\u03b5\u03c0\u03b5\u03b9 \u03bd\u03b1 \u03b5\u03c4\u03bf\u03b9\u03bc\u03b1\u03b6\u03b5\u03c3\u03b1\u03b9 \u03b3\u03b9\u03b1 \u03c4\u03bf \u03ba\u03b1\u03bb\u03c5\u03c4\u03b5\u03c1\u03bf \u03ba\u03b1\u03b9 \u03bd\u03b1 \u03b5\u03b9\u03c3\u03b1\u03b9 \u03b5\u03c4\u03bf\u03b9\u03bc\u03bf\u03c2 \u03b3\u03b9\u03b1 \u03c4\u03bf \u03c7\u03b5\u03b9\u03c1\u03bf\u03c4\u03b5\u03c1\u03bf. \u03c3\u03c9\u03c3\u03c4\u03bf\u03c2 :) :)", "Solution_6": "\u0394\u03b5\u03af\u03be\u03c4\u03b5 \u03cc\u03c4\u03b9 $\\sum_{k=0}^n\\binom{n+k}{k}\\frac{1}{2^{k}}=2^n$\r\n\r\n. \u03a4\u03b7\u03bd \u03b5\u03af\u03c7\u03b1 \u03b4\u03b9\u03bf\u03c1\u03b8\u03ce\u03c3\u03b5\u03b9 \u03c0\u03ac\u03bd\u03c9 ;)", "Solution_7": "\u03b4\u03b5\u03bd \u03c0\u03b9\u03c3\u03c4\u03b5\u03c5\u03c9 \u03bd\u03b1 \u03bb\u03c5\u03bd\u03b5\u03c4\u03b1\u03b9(\u03b1\u03bd \u03bb\u03c5\u03bd\u03b5\u03c4\u03b1\u03b9 \u03bc\u03bf\u03bd\u03bf \u03bc\u03b5 \u03b5\u03c0\u03b1\u03b3\u03c9\u03b3\u03b7). \u03c4\u03bf \u03bb\u03b5\u03c9 \u03b3\u03b9\u03b1\u03c4\u03b9 \u03c4\u03b7\u03bd \u03b1\u03c0\u03bf\u03c6\u03b5\u03c5\u03b3\u03c9 \u03c3\u03b5\u03c4\u03b9\u03c2 \u03c0\u03b5\u03c1\u03b9\u03c3\u03bf\u03c4\u03b5\u03c1\u03b5\u03c2 \u03b1\u03c3\u03ba\u03b7\u03c3\u03b5\u03b9\u03c2 :lol: :lol:", "Solution_8": "\u03cc\u03c7\u03b9 \u03b4\u03b5\u03bd \u03bb\u03cd\u03bd\u03b5\u03c4\u03b1\u03b9 \u03bc\u03b5 \u03b5\u03c0\u03b1\u03b3\u03c9\u03b3\u03ae (\u03c4\u03bf\u03c5\u03bb\u03ac\u03c7\u03b9\u03c3\u03c4\u03bf\u03bd \u03b7 \u03bb\u03cd\u03c3\u03b7 \u03bc\u03bf\u03c5 \u03b4\u03b5\u03bd \u03b5\u03af\u03bd\u03b1\u03b9 \u03bc\u03b5 \u03b5\u03c0\u03b1\u03b3\u03c9\u03b3\u03ae ;) ) \u03ba\u03b1\u03b9 \u03bb\u03cd\u03bd\u03b5\u03c4\u03b1\u03b9 \u03c3\u03b5 \u03bb\u03b9\u03b3\u03cc\u03c4\u03b5\u03c1\u03bf \u03b1\u03c0\u03cc \u03c0\u03ad\u03bd\u03c4\u03b5 \u03c3\u03b5\u03b9\u03c1\u03ad\u03c2.....", "Solution_9": "[hide=\"\u0395\u03c0\u03b1\u03b3\u03c9\u03b3\u03ae\"] \u039d\u03b1 \u03bc\u03b9\u03b1 \u03b1\u03c0\u03cc\u03b4\u03b5\u03b9\u03be\u03b7 \u03bc\u03b5 \u03b5\u03c0\u03b1\u03b3\u03c9\u03b3\u03ae \u03c3\u03c4\u03bf $n$ :\n\n\u0398\u03b1 \u03b4.\u03bf. $\\sum_{k=0}^{n+1}\\binom{n+1+k}{k}\\frac{1}{2^{k}}=2^{n+1}$ .\n\n$\\\\ A =\\sum_{k=0}^{n+1}\\binom{n+1+k}{k}\\frac{1}{2^{k}}=\\sum_{k=0}^{n+1}\\frac{(n+1+k)!}{k!(n+1)!}\\frac{1}{2^{k}}= \\\\ \\\\ \\\\ \\sum_{k=0}^{n+1}\\frac{(n+1+k) \\cdot (n+k)!}{k! \\cdot (n+1)\\cdot n!}\\frac{1}{2^{k}}= \\sum_{k=0}^{n+1} \\left( 1+\\frac{k}{n+1} \\right) \\binom{n+k}{k} \\frac{1}{2^{k}}=\\\\ \\\\ \\\\ \\sum_{k=0}^{n+1} \\binom{n+k}{k} \\frac{1}{2^{k}} + \\sum_{k=0}^{n+1} \\frac{k}{n+1} \\binom{n+k}{k} \\frac{1}{2^{k}}=\\\\ \\\\ \\\\ \\sum_{k=0}^{n} \\binom{n+k}{k} \\frac{1}{2^{k}}+ \\binom{2n+1}{n+1} \\frac{1}{2^{n+1}} + \\sum_{k=0}^{n+1} \\frac{k}{n+1} \\binom{n+k}{k} \\frac{1}{2^{k}}=\\\\ \\\\ \\\\ 2^{n}+ \\frac{1}{2} A$\n\n\u03ac\u03c1\u03b1 $\\boxed{ A =2^{n+1}}$ :stretcher: \n[/hide]\r\n :)", "Solution_10": "\u0391\u03c0\u03bb\u03ac \u03ba\u03b1\u03c4\u03b1\u03c0\u03bb\u03b7\u03ba\u03c4\u03b9\u03ba\u03cc . :lol: . \u03a0\u03bf\u03bb\u03cd \u03c9\u03c1\u03b1\u03af\u03b1......\r\n\u0397 \u03bb\u03cd\u03c3\u03b7 \u03bc\u03bf\u03c5 \u03c7\u03c1\u03b7\u03c3\u03b9\u03bc\u03bf\u03c0\u03bf\u03b9\u03b5\u03af \u03cc\u03c4\u03b9 \r\n $\\sum_{k=0}^n\\binom{n+k}{k}\\frac{1}{2^{n+k}}=\\sum_{k=0}^np_k=1$", "Solution_11": "[quote=\"silouan\"]\u0391\u03c0\u03bb\u03ac \u03ba\u03b1\u03c4\u03b1\u03c0\u03bb\u03b7\u03ba\u03c4\u03b9\u03ba\u03cc . :lol: . \u03a0\u03bf\u03bb\u03cd \u03c9\u03c1\u03b1\u03af\u03b1......\n\u0397 \u03bb\u03cd\u03c3\u03b7 \u03bc\u03bf\u03c5 \u03c7\u03c1\u03b7\u03c3\u03b9\u03bc\u03bf\u03c0\u03bf\u03b9\u03b5\u03af \u03cc\u03c4\u03b9 \n $\\sum_{k=0}^n\\binom{n+k}{k}\\frac{1}{2^{n+k}}=\\sum_{k=0}^np_k=1$[/quote]\r\n\r\n\u03a3\u03c4\u03c1\u03b1\u03b2\u03ce\u03b8\u03b7\u03ba\u03b1 \u03b3\u03b9\u03b1 \u03bd\u03b1 \u03c4\u03bf \u03b3\u03c1\u03ac\u03c8\u03c9... \r\n\r\n\u03a0\u03ce\u03c2 \u03b1\u03c0\u03bf\u03b4\u03b5\u03b9\u03ba\u03bd\u03cd\u03b5\u03c4\u03b1\u03b9 ??? :?", "Solution_12": "\u03a4\u03bf \u03b1\u03c1\u03b9\u03c3\u03c4\u03b5\u03c1\u03cc \u03bc\u03ad\u03bb\u03bf\u03c2 \u03b5\u03af\u03bd\u03b1\u03b9 \u03ac\u03b8\u03c1\u03bf\u03b9\u03c3ma \u03c0\u03b9\u03b8\u03b1\u03bd\u03bf\u03c4\u03ae\u03c4\u03c9\u03bd $p_k=\\binom{n+k}{k}\\frac{1}{2^{n+k}}=\\frac{1}{2}\\binom{n+k}{k}\\frac{1}{2^{n+k}}+\\frac{1}{2}\\binom{n+k}{k}\\frac{1}{2^{n+k}}=P(A_k)+P(B_k)$", "Solution_13": "\u03a0\u03b9\u03b8\u03b1\u03bd\u03bf\u03c4\u03b7\u03c4\u03b5\u03c2???!!!! \u03c1\u03b5 \u03bc\u03b5\u03b3\u03b1\u03bb\u03b5 \u03c4\u03bf \u03b5\u03be\u03b7\u03b3\u03b5\u03b9\u03c2 \u03bb\u03b9\u03b3\u03bf \u03ba\u03b1\u03bb\u03c5\u03c4\u03b5\u03c1\u03b1?\u03ba\u03b1\u03b9 \u03b3\u03b9\u03b1\u03c4\u03b9 \u03be\u03b1\u03bd\u03b1\u03b3\u03c1\u03b1\u03c6\u03b5\u03b9\u03c2 \u03c4\u03bf n+k \u03b3\u03b9\u03b1 \u03b5\u03ba\u03b8\u03b5\u03c4\u03b7? :)" } { "Tag": [ "inequalities", "inequalities proposed" ], "Problem": "Let $ x,y,z$ be positive numbers satisfying $ x(x\\plus{}y\\plus{}z)\\equal{}3yz$.Prove that \r\n\\[ (x\\plus{}y)^3\\plus{}(x\\plus{}z)^3\\plus{}3(x\\plus{}y)(y\\plus{}z)(z\\plus{}x)\\le 5(y\\plus{}z)^3\\]", "Solution_1": "[quote=\"Allnames\"]Let $ x,y,z$ be positive numbers satisfying $ x(x \\plus{} y \\plus{} z) \\equal{} 3yz$.Prove that\n\\[ (x \\plus{} y)^3 \\plus{} (x \\plus{} z)^3 \\plus{} 3(x \\plus{} y)(y \\plus{} z)(z \\plus{} x)\\le 5(y \\plus{} z)^3\\] (*)\n[/quote]\r\n\r\n$ 3(x\\plus{}y)(x\\plus{}z)(y\\plus{}z)\\equal{}12yz(y\\plus{}z)$\r\n\r\nWe have :\r\n\r\n$ (*) \\Leftrightarrow 4(y^3\\plus{}z^3)\\plus{}3yz(y\\plus{}z) \\geq 2x^3\\plus{}3x^2(y\\plus{}z)\\plus{}3x(y^2\\plus{}z^2)$ \r\n\r\nBy AM-GM , we have : \r\n\r\n$ 3yz\\equal{}x(x\\plus{}y\\plus{}z) \\geq 3x\\sqrt[3]{xyz} \\Leftrightarrow yz \\geq x^2$\r\n\r\nWe have : \r\n\r\n* $ 3yz(y\\plus{}z) \\geq 3x^2(y\\plus{}z)$\r\n\r\n* $ y^3\\plus{}z^3 \\geq 2\\sqrt{(yz)^3} \\geq 2x^3$\r\n\r\n* $ (y^3\\plus{}z^3)^2(2) \\geq (y^2\\plus{}z^2)^2(y^2\\plus{}z^2) \\geq (y^2\\plus{}z^2)^{2}2yz \\geq2x^2(y^2\\plus{}z^2)^2$\r\n\r\n$ \\Leftrightarrow 3(y^3\\plus{}z^3) \\geq 3x^(y^2\\plus{}z^2)$\r\n\r\nDone ! \r\n\r\n:)", "Solution_2": "If $ x\\plus{}y\\equal{}2a , x\\plus{}z\\equal{}2b , y\\plus{}z\\equal{}2c$ $ \\Longrightarrow$ $ a^2\\minus{}ab\\plus{}b^2\\equal{}c^2$ \r\n\r\nInequality equivalents to $ (a\\plus{}b)c\\plus{}3ab \\leq 5c^2 \\Longleftrightarrow \\sqrt{a^2\\minus{}ab\\plus{}b^2}.(a\\plus{}b) \\leq 5a^2\\minus{}8ab\\plus{}5b^2 \\Longleftrightarrow$\r\n\r\n$ [\\frac{(a\\plus{}b)^2}{ab}\\minus{}4][24. \\frac{(a\\plus{}b)^2}{ab}\\minus{}81]\\geq0$ which is obviously true. :wink:", "Solution_3": "[quote=\"Materazzi\"][quote=\"Allnames\"]Let $ x,y,z$ be positive numbers satisfying $ x(x \\plus{} y \\plus{} z) \\equal{} 3yz$.Prove that\n\\[ (x \\plus{} y)^3 \\plus{} (x \\plus{} z)^3 \\plus{} 3(x \\plus{} y)(y \\plus{} z)(z \\plus{} x)\\le 5(y \\plus{} z)^3\\]\n(*)[/quote]\n\nBy AM-GM , we have : \n\n$ 3yz \\equal{} x(x \\plus{} y \\plus{} z) \\geq 3x\\sqrt [3]{xyz} \\Leftrightarrow yz \\geq x^2$\n\n\n:)[/quote]\r\nCan you explain this step? It means that you assume $ x\\equal{}min\\{x,y,z\\}$ so \r\n$ 3yz \\equal{} x(x \\plus{} y \\plus{} z) \\geq 3x\\sqrt [3]{xyz} \\ge 3x^2 \\Leftrightarrow yz \\geq x^2$", "Solution_4": "[quote=\"zaizai-hoang\"][quote=\"Materazzi\"][quote=\"Allnames\"]Let $ x,y,z$ be positive numbers satisfying $ x(x \\plus{} y \\plus{} z) \\equal{} 3yz$.Prove that\n\\[ (x \\plus{} y)^3 \\plus{} (x \\plus{} z)^3 \\plus{} 3(x \\plus{} y)(y \\plus{} z)(z \\plus{} x)\\le 5(y \\plus{} z)^3\\]\n(*)[/quote]\n\nBy AM-GM , we have : \n\n$ 3yz \\equal{} x(x \\plus{} y \\plus{} z) \\geq 3x\\sqrt [3]{xyz} \\Leftrightarrow yz \\geq x^2$\n\n\n:)[/quote]\nCan you explain this step? It means that you assume $ x \\equal{} min\\{x,y,z\\}$ so \n$ 3yz \\equal{} x(x \\plus{} y \\plus{} z) \\geq 3x\\sqrt [3]{xyz} \\ge 3x^2 \\Leftrightarrow yz \\geq x^2$[/quote]\r\n\r\nNo ! it is just a simplified step and you do not need to assume x min", "Solution_5": "I understand this one, everything comes from the condition :maybe: I have another question. Can we assume that without loss of generally $ x \\equal{} min\\{x,y,z\\}$ ? I think we can not :maybe: Right or wrong? Who can answers my question? Thank u so much :)", "Solution_6": "Let $ x,y,z$ be positive numbers satisfying $ x(x\\plus{}y\\plus{}z) \\equal{} 3yz$ .Prove that \r\n\\[ (x\\plus{}y)^3\\plus{}(x\\plus{}z)^3\\le 2(z\\plus{}y)^3\\]", "Solution_7": "[quote=\"zaizai-hoang\"]I understand this one, everything comes from the condition :maybe: I have another question. Can we assume that without loss of generally $ x \\equal{} min\\{x,y,z\\}$ ? I think we can not :maybe: Right or wrong? Who can answers my question? Thank u so much :)[/quote]\r\nOf course the answer is No, dear zaizai, Because The triple $ (2;6;1)$ satisfies the condition and neither of these triples $ (1,6,2)$,$ (6,2,1)$ does.But $ x$ is not maximal number.", "Solution_8": "[quote=\"zaizai-hoang\"]Let $ x,y,z$ be positive numbers satisfying $ x(x \\plus{} y \\plus{} z) \\equal{} 3yz$ .Prove that\n\\[ (x \\plus{} y)^3 \\plus{} (x \\plus{} z)^3\\le 2(z \\plus{} y)^3\\]\n[/quote]\r\nThe inequality is homogeneous, thus we can assume that $ x\\plus{}y\\plus{}z\\equal{}1$. Hence $ x\\equal{}3yz\\equal{}1\\minus{}y\\minus{}z$.\r\nYour inequality is equivalent to \r\n\\[ (y\\plus{}z)^2(3(y\\plus{}z)\\minus{}2)\\ge 0\\]\r\nWhich is obvious since $ 1\\minus{}y\\minus{}z\\le \\frac{3(y\\plus{}z)^2}{4} \\leftrightarrow y\\plus{}z\\ge \\frac{2}{3}$.\r\nIt is also the method I used to prove the first problem!" } { "Tag": [], "Problem": "Hi guys!\r\nI am looking for a Physics book in the net!\r\nI want something like \"Introduction to Physics\" :) .\r\nI hope you can help me!\r\nThnks beforhand!! :wink:", "Solution_1": "Physics for scientists and engineers, by Tipler and Mosca:\r\n\r\nhttp://lib.org.by/info/P_Physics/PG_General%20courses/Tipler%20P.A.,%20Mosca%20G.%20Physics%20for%20scientists%20and%20engineers%20(5ed.,%20extended%20version)(no%20p.399)(600dpi)(T)(C)(1395s).djvu\r\n\r\nClick http://windjview.sourceforge.net/ to get a djvu reader to read this scanned book with." } { "Tag": [ "logarithms", "complex numbers" ], "Problem": "Well, everyone knows that complex numbers can be expressed in exponential form with modulus $ r$ and argument $ \\theta$, written as $ e^{i\\theta}$. But $ e\\equal{}e(1)\\equal{}e(e^{2i\\pi})\\equal{}e^{2i\\pi\\plus{}1}$, but that seems to imply that $ e^{i\\theta}\\equal{}e^{(2i\\pi\\plus{}1)(i\\theta)}\\equal{}e^{\\minus{}2\\pi\\theta\\plus{}i\\theta}\\equal{}ke^{i\\theta}$ where $ k\\equal{}e^{\\minus{}2\\pi\\theta}$ is generally not 1, and thus the equality doesn't usually establish. Did I have a mistake in the above equations (or inequations) or is this actually a loophole of the set of complex numbers? If so, are there tacit assumptions that decrease ambiguity?", "Solution_1": "I think your mistake is assuming that $ 1 \\equal{} 2i\\pi \\plus{} 1$ because $ e \\equal{} e^{2i\\pi \\plus{} 1}$. This assumption gives $ i \\equal{} 0$ which is not true.\r\n\r\nEdit: Somebody correct me if I'm wrong but here's why I think the assumption doesn't work:\r\n$ e$ is a purely real number and when you multiply it by a complex number, $ z\\equal{}e^{i\\theta}$, only $ |z|$ is affected. In other words, the multiplication has no effect on $ \\arg(z) \\equal{} \\theta$.", "Solution_2": "[quote]I think your mistake is assuming that $ 1 \\equal{} 2i\\pi \\plus{} 1$ because $ e \\equal{} e^{2i\\pi \\plus{} 1}$. This assumption gives $ i \\equal{} 0$ which is not true.[/quote]\r\n\r\nNot quite. It tells you that $ 2\\pi i \\equal{} 0$, which can also imply, with equal incorrectness, that $ \\pi \\equal{} 0$.\r\n\r\n$ e$, as a complex number, can be written as $ e \\bullet e^{i\\pi}$, which basically brings about the same problem that Sorcerer presents since expanding would result in a totally different (and ugly) complex number, namely $ e^{i(2\\pi \\plus{} \\frac {1}{i})} \\equal{} \\cos(2\\pi \\plus{} \\frac {1}{i}) \\plus{} i\\sin(2\\pi \\plus{} \\frac {1}{i})$. As kcn2rivers points out, this cannot be simplified, for reasons that probably come down to the inability to use regular exponentiation laws when working with such numbers.", "Solution_3": "The complex exponential doesn't satisfy $ (e^a)^b \\equal{} e^{ab}$ for complex $ b$. This is because there is no unambiguous way to define $ x^y$ for complex $ x, y$; the problem boils down to describing the [url=http://en.wikipedia.org/wiki/Complex_logarithm]complex logarithm[/url]." } { "Tag": [ "inequalities", "function", "logarithms", "inequalities proposed" ], "Problem": "Let $k > 1$ and $n > 1$ be natural numbers. Let $x_1, x_2, ..., x_n$ be $n$ positive real numbers whose product is 1.\r\n\r\nShow that $\\sum {x_i}^{k+1} \\ge \\sum {x_i}^k$.\r\n\r\n[i]Edited by Myth[/i]", "Solution_1": "chebyshev.", "Solution_2": "Sorry for the ignorant question: What is 'Chebyshev' and how would you apply it here?", "Solution_3": "Pure Murheid... :)", "Solution_4": "http://planetmath.org/encyclopedia/ChebyshevsInequality.html\r\nChebyshev inequality is here.\r\nNow, as $x_i^k$ and $x_i$ are similarly sorted, you may apply chebyshev inequality and then amgm to get the result.", "Solution_5": "Ahh! It makes it so simple!\r\n\r\nFor a challenge, try not using the Chebyshev inequality... only AM-GM suffices for this problem.", "Solution_6": "[quote=\"Aryabhatta\"]Let $k > 1$ and $n > 1$ be natural numbers. Let $x_1, x_2, ..., x_n$ be $n$ positive real numbers whose product is 1.\n\nShow that $\\sum {x_i}^{k+1} \\ge \\sum {x_i}^k$.[/quote]\r\n\r\nThis can be proven without the use of Chebyshev. We can prove, more generally:\r\n\r\n[color=blue][b]Theorem.[/b] If $x_1$, $x_2$, ..., $x_n$ are n positive real numbers such that $x_1x_2...x_n\\geq 1$, then the function\n\n$f\\left(t\\right)=x_1^t+x_2^t+...+x_n^t$\n\nis monotonically increasing for $t\\geq 0$.[/color]\r\n\r\n[i]Proof.[/i] We have to prove that if u and v are two nonnegative reals such that $u\\geq v$, then $f\\left(u\\right)\\geq f\\left(v\\right)$.\r\n\r\nWell, let $q=\\frac{u-v}{n}$. Since $u\\geq v$, we have $u-v\\geq 0$ and thus $q=\\frac{u-v}{n}\\geq 0$.\r\n\r\nNow,\r\n\r\n$\\frac{v+q}{u}+\\underbrace{\\frac{q}{u}+\\frac{q}{u}+...+\\frac{q}{u}}_{n-1\\text{ times}}=\\frac{\\left(v+q\\right)+\\left(n-1\\right)q}{u}$\r\n$=\\frac{v+nq}{u}=\\frac{v+n\\frac{u-v}{n}}{u}=\\frac{v+\\left(u-v\\right)}{u}=1$;\r\n\r\nthus, we can apply the weighted AM-GM inequality to get\r\n\r\n$\\frac{v+q}{u}x_1^u+\\frac{q}{u}x_2^u+\\frac{q}{u}x_3^u+...+\\frac{q}{u}x_n^u\\geq \\left(x_1^u\\right)^\\frac{v+q}{u}\\left(x_2^u\\right)^\\frac{q}{u}\\left(x_3^u\\right)^\\frac{q}{u}...\\left(x_n^u\\right)^\\frac{q}{u}$\r\n$=x_1^{v+q}x_2^qx_3^q...x_n^q=x_1^v\\cdot\\left(x_1^qx_2^qx_3^q...x_n^q\\right)=x_1^v\\cdot\\left(x_1x_2...x_n\\right)^q$.\r\n\r\nSince $x_1x_2...x_n\\geq 1$, this becomes\r\n\r\n$\\frac{v+q}{u}x_1^u+\\frac{q}{u}x_2^u+\\frac{q}{u}x_3^u+...+\\frac{q}{u}x_n^u\\geq x_1^v$.\r\n\r\nBy cyclic permutation, this inequality yields\r\n\r\n$\\frac{v+q}{u}x_2^u+\\frac{q}{u}x_3^u+\\frac{q}{u}x_4^u+...+\\frac{q}{u}x_1^u\\geq x_2^v$;\r\n$\\frac{v+q}{u}x_3^u+\\frac{q}{u}x_4^u+\\frac{q}{u}x_5^u+...+\\frac{q}{u}x_2^u\\geq x_3^v$;\r\n...;\r\n$\\frac{v+q}{u}x_n^u+\\frac{q}{u}x_1^u+\\frac{q}{u}x_2^u+...+\\frac{q}{u}x_{n-1}^u\\geq x_n^v$.\r\n\r\nAfter summing up all these inequalities, we arrive at\r\n\r\n$\\left(\\frac{v+q}{u}+\\underbrace{\\frac{q}{u}+\\frac{q}{u}+...+\\frac{q}{u}}_{n-1\\text{ times}}\\right)\\left(x_1^u+x_2^u+...+x_n^u\\right)\\geq x_1^v+x_2^v+...+x_n^v$.\r\n\r\nBut since $\\frac{v+q}{u}+\\underbrace{\\frac{q}{u}+\\frac{q}{u}+...+\\frac{q}{u}}_{n-1\\text{ times}}=1$, $x_1^u+x_2^u+...+x_n^u=f\\left(u\\right)$ and $x_1^v+x_2^v+...+x_n^v=f\\left(v\\right)$, this becomes $1\\cdot f\\left(u\\right)\\geq f\\left(v\\right)$, so that $f\\left(u\\right)\\geq f\\left(v\\right)$, and we are done.\r\n\r\n Darij", "Solution_7": "[quote=\"Aryabhatta\"]Let $k > 1$ and $n > 1$ be natural numbers. Let $x_1, x_2, ..., x_n$ be $n$ positive real numbers whose product is 1.\n\nShow that $\\sum {x_i}^{u} \\ge \\sum {x_i}^v$ for $u>v$.[/quote]\r\nTo present a short way...\r\nWe have $\\sum {x_i}^{u} \\geq \\sum {x_i}^v(x_1...x_n)^{\\frac{u-v}{n}}$ by Murheid. Finally, $ \\sum {x_i}^v(x_1...x_n)^{\\frac{u-v}{n}}= \\sum {x_i}^v$.", "Solution_8": "You guys are good! \r\n\r\nAnyway, an 'elementary' solution, if you are willing to call it that :-)\r\n\r\nWe show that for _any_ positive reals $x_1, x_2, ..., x_n$\r\n\r\n$(\\frac{\\sum x_i^{k+1}}{\\sum x_i^k})^{\\sum x_i ^k} \\ge x_1^{x_1^k}x_2^{x_2^k}...x_n^{x_n^k} $\r\n\r\nFirst assume each $x_i$ is a natural number.\r\n\r\nWriting $x_i^{k+1}$ as $x_i + x_i + ...+ x_i, x_i^k$ times and applying AM-GM we get the result. Now it is easy to move to rationals and hence to reals.\r\n\r\n\r\nNow given that $x_1x_2...x_n = 1$, all we need to show is $x_1^{x_1^k}x_2^{x_2^k}...x_n^{x_n^k} \\ge 1$ \r\n\r\nWlog assume $x_1 \\ge x_2 \\ge ... \\ge x_n$. \r\nLet $r$ be the smallest such that $x_r < 1$.\r\nRewriting using $x_r = {\\frac {1} {x_1...x_{r-1}x_{r+1}...x_n}}$ we have\r\n\r\n$x_1^{x_1^k}x_2^{x_2^k}...x_n^{x_n^k} = x_1^{x_1^k - x_r^k}...x_n^{x_n^k - x_r ^k} \\ge 1$ as each term $x_i^{x_i^k - x_r^k} \\ge 1$", "Solution_9": "the same as darij said put $f(y)=x_{1}^{y}+...+x_{n}^{y}$ we want to show that for every $u,v$ such that $u\\geq v$ we have $f(u)\\geq f(v)$ so we have to show that $f$ is increasing,so its sufficient to show that $f'\\geq 0$ (note that $f$ is the sum of $n$ derivable functions so $f$ it self is derivable too)\r\nwe have $f'(y)=(x_{1}^{y})'+...+(x_{n}^{y})'=(x_{1}^{y}.\\ln x_{1})+...+(x_{n}^{y}.\\ln x_{n})=\\ln (x_{1}^{x_{1}^{y}}...x_{n}^{x_{n}^{y}})$\r\nin the last equation we used the facts that $\\log_{c}^{a}+\\log_{c}^{b}=\\log_{c}^{ab}$ and $b\\log_{c}^{a}=\\log_{c}^{a^{b}}$\r\nnow we know that the $\\log$ function is increasing in the interval $[1,+\\infty)$ and we know that $\\ln 1=0$ so the only thing that remains is to show that $x_{1}^{x_{1}^{y}}...x_{n}^{x_{n}^{y}}\\geq 1$...\r\n[hide][b]lemma:for every positive numbers $a_{1},...,a_{n}$ we have $a_{1}^{a_{1}}...a_{n}^{a_{n}}\\geq (\\frac{a_{1}+...+a_{n}}{n})^{a_{1}+...+a_{n}}$\nproof:WLOG assume that $a_{1}+...+a_{n}=1$ so we have to show that $a_{1}^{a_{1}}...a_{n}^{a_{n}}\\geq \\frac{1}{n}$ but from weighted AM-GM we know that $a_{1}^{\\alpha_{1}}...a_{n}^{\\alpha_{n}}\\geq \\frac{1}{\\frac{\\alpha_{1}}{a_{1}}+...+\\frac{\\alpha_{n}}{a_{n}}}$ if we put $\\alpha_{i}=a_{i}$;for every $i=1,...,n$;we get that $\\frac{1}{n}\\leq a_{1}^{a_{1}}...a_{n}^{a_{n}}$...[/b]\nnow in the lemma above put $a_{i}=x_{i}^{y}$;$i=1,...,n$; so we get that:\n$x_{1}^{yx_{1}^{y}}...x_{n}^{yx_{n}^{y}}\\geq (\\frac{x_{1}^{y}+...+x_{n}^{y}}{n})^{x_{1}^{y}+...+x_{n}^{y}}\\geq (\\sqrt[n]{x_{1}^{y}...x_{n}^{y}})^{x_{1}^{y}+...+x_{n}^{y}}\\geq 1$\n[/hide]", "Solution_10": "$kx^{k+1}+1\\ge \\ (k+1)x^{k}$\r\n$k\\sum{x_{i}}^{k+1}+n\\ge (k+1)\\sum{x_{i}}^{k}=k\\sum{x_{i}}^{k}+\\sum{x_{i}}^{k}\\ge k\\sum{x_{i}}^{k}+n$\r\n$k\\sum{x_{i}}^{k+1}\\ge k\\sum{x_{i}}^{k}$\r\n$\\sum{x_{i}}^{k+1}\\ge\\sum{x_{i}}^{k}$" } { "Tag": [ "algebra", "polynomial", "number theory unsolved", "number theory" ], "Problem": "Let $k$ be even number; let $p>2$ be prime number. Prove that iff $p-1$ doesnot divide $k$ then $p^{k+1}$ divides $\\displaystyle \\sum_{i=1}^{p-1}(\\binom {p}{i})^k$.", "Solution_1": "A similar problem has been solved on the forum recently. In fact, this is a very natural generalization of that problem. [url=http://www.mathlinks.ro/Forum/viewtopic.php?t=15433&highlight=]Here[/url]'s the link.\r\n\r\nWe need to show that, in $\\mathbb Z_p$, $S=\\displaystyle \\sum_{i=1}^{p-1}(\\frac{\\binom pi}p)^k=0$. Because $k$ is even, we can show fairly easily that $\\displaystyle(\\frac{\\binom pi}p)^k=i^{-k}$ (for a proof check out that link), so $S=\\displaystyle\\sum_{i=1}^{p-1}i^{-k}=\\sum_{i=1}^{p-1}i^k$. This is $0$. We prove it in the same way the following well-known theorem about the roots of unity is proven: if $z_i,\\ i\\in\\overline{0,n-1}$ are the roots of the polynomial $x^n-1$, then $\\displaystyle\\sum_{i=0}^{n-1} z_i^k= \\{\\begin{array}{c}0,\\ n\\not |\\ k\\\\n,\\ n\\ |\\ k\\end{array}$. We just work with the polynomial $x^{p-1}-1$ in $\\mathbb Z_p$ instead of $x^n-1$ in $\\mathbb C$.\r\n\r\nFor a quick proof, we can use the [url=http://mathworld.wolfram.com/Newton-GirardFormulas.html]Newton-Girard formulas[/url].", "Solution_2": "Nice solution!", "Solution_3": "since $S=\\sum_{i=1}^{p-1}i^k$ taking a primitive root of $a\\in\\mathbb{Z}_p$ ($a^k\\neq 1$ for $k0$ such that $f''(x)>\\frac{M}x$ for all $x>a$. Integrating, $f'(x)>M\\ln\\left(\\frac xa\\right)+f'(a)\\to\\infty.$ (That's only true for $x>a$, but it's all we need)", "Solution_3": "I'm sorry :blush: \r\nI wanted $xf(x) \\to 0, xf''(x)\\to 0$ when $x\\to \\infty$ and $f\\in C^2(R)$ and we need to prove that $xf'(x)\\to 0$ when $x\\to \\infty$\r\nSorry one more :blush:", "Solution_4": "[b]This solution is entirely wrong[/b]\r\n[hide]\n$f'(x) - f'(x-1) = f''(\\xi)$ for some $\\xi \\in (x-1,x)$. We have then that $\\xi (f'(x) - f'(x-1)) \\to 0$ as $x \\to \\infty$. Hence, $f'(x) - f'(x-1) = O(\\xi^{-p}) = O\\left(\\frac{1}{(x-1)^p}\\right)$ for some p > 1.\n\nWe also have $f(x) - f(x-1) = f'(\\eta) = O\\left(\\frac{1}{(\\eta-1)^p}\\right) + f'(\\eta - 1)$, where $\\eta \\in (x-1,x)$. Multiplying by x, we have that $xf(x) \\to 0$ and $xO\\left(\\frac{1}{(\\eta-1)^p}\\right) = xO\\left(\\frac{1}{(x-2)^p}\\right) \\to 0$, so we get that the limits of $-xf(x-1)$ and $xf'(\\eta - 1)$ are the same.\n\n\nWe have that $xf(x-1) = (x-1)f(x-1) + f(x-1) \\to 0$ as $x\\to \\infty$, where the limit of $f(x-1)$ is obvious (it must be 0). So we have that $\\lim_{x\\to\\infty}xf'(\\eta-1) = 0$, which must mean that $\\lim_{x\\to \\infty}xf'(x) = 0$ (I think you could make a formal argument, but I reason that $f'(x)$ goes to zero more quickly than $f'(\\eta-1)$.[/hide]", "Solution_5": "[quote=\"Kalle\"] We have then that $\\xi (f'(x) - f'(x-1)) \\to 0$ as $x \\to \\infty$. Hence, $f'(x) - f'(x-1) = O(\\xi^{-p}) = O\\left(\\frac{1}{(x-1)^p}\\right)$ for some p > 1.\n[/quote]\r\n\r\nYou have $\\xi f \\to 0.$ when $\\xi \\to \\infty$And you make a conclusion that $\\exists p>1$ such that $\\frac{f}{\\xi ^{-p}}$ is bounded?\r\nBut if $f(\\xi)=\\frac{1}{\\xi\\ln \\xi}$ than $\\frac{f}{\\xi ^{-p}}=\\frac{\\xi^p}{\\xi\\ln \\xi} = \\frac{\\xi^{p-1}}{\\ln\\xi}$ and it not bounded for any $p-1>0$. \r\nOr, maybe i not undestand you :?:", "Solution_6": "Hmm. What you say is indeed a counterexample. What I wanted to get at is that when you multiply $f'(x) - f'(x-1)$ by $x$, then the limit is 0. Maybe that is not correct either? Then it doesn't work.", "Solution_7": "This proof is now in a more clear version a couple of posts down :P\r\n\r\n[hide]$f(x) - f(x-1) = f'(\\eta(x))$ where $\\eta(x) \\in (x-1,x)$ by the mean value theorem. From the given conditions we have that $(x-1)(f(x) - f(x-1)) = xf(x) - (x-1)f(x-1) - f(x) \\to 0$ when $x \\to \\infty$ since the limit of $f(x)$ has to be 0. This means that $(x-1)f'(\\eta(x)) \\to 0$ when $x \\to \\infty$. Since $x$ goes to infinity more quickly than $\\eta(x)$, this bears the implication that $(x-1)f'(x) \\to 0$ when $x \\to \\infty$*.\n\nNow, $f'(x) - f'(x-1) = f''(\\xi(x))$ where $\\xi(x) \\in (x-1,x)$. From the given conditions we have that $|(x-1)f''(\\xi)| < |\\xi(x)f''(\\xi(x))| \\to 0$ as $x\\to \\infty$. So, rearranging and multiplying by $(x-1)$ we have that\n\n$\\lim_{x\\to\\infty}(x-1)f'(x-1) = \\lim_{x\\to\\infty}\\left((x-1)f'(x) - (x-1)f''(\\xi(x))\\right) = 0$\n\n*Proof: We have that $\\lim_{x\\to\\infty}(\\eta(x) - 1)f(\\eta(x)) = 0$ since $0 < |(\\eta(x) - 1)f(eta(x))| < |(x - 1)f(eta(x))| \\to 0$. Since $(y-1)f(y)$ is a continous function the statement follows.\n\nedit: FINALLY. This has to be right. Only took the entire day for me to come up with a full solution :D :P [/hide]", "Solution_8": "I don't understad your argument. can you be more detalied? Write it in the logic order. Thank you.", "Solution_9": "Hmm. Ok. I've spent a lot of time on it anyway, so why not.\r\n\r\nFor a constant $b$, the mean value theorem states that there exists a $\\eta \\in (b-1,b)$ such that $f(b) - f(b-1) = f'(\\eta)$. Therefore, there exists a function $\\eta(x)$ defined by the equality $f(x) - f(x-1) = f'(\\eta(x))$. This function does not have to be continous or anything, but it exists according to the mean value theorem. For this function, it must also be true that $x-1 < \\eta(x) < x$ for every x. \r\n\r\nWe must have that $\\lim_{x\\to\\infty}f(x) = 0$, or it would not be possible that $\\lim_{x\\to\\infty}xf(x) = 0$. From this we can conclude that:\r\n$(x-1)f'(\\eta(x)) = (x-1)(f(x) - f(x-1)) = xf(x) - f(x) - (x-1)f(x-1) \\to 0$ when $x\\to\\infty$, since every term on the right hand side goes to zero. From this, it is quite trivial to prove that $\\lim_{x\\to\\infty}(x-1)f'(x) = 0$, which is a result from the fact that $x-1 < \\eta(x) < x$. The proof is marked under * at the end of this message. \r\n\r\nNote how we've now proven a very similar thing to what's asked for, but it is slightly weaker. $(x-1)$ grows slower than $x$. So we need to strengthen the conditions a little.\r\n\r\nWith similar reasoning to in the beginning of the proof, there exists a function $\\xi(x)$ which fulfills the equality $f'(x) - f'(x-1) = f''(\\xi(x))$ and the inequality $x - 1 < \\xi(x) < x$. This inequality shows that $\\lim_{x\\to\\infty}\\xi(x) = \\infty$. Since $f''(x)$ is a continous function, we have that $\\lim_{x\\to\\infty}\\xi(x)f''(\\xi(x)) = 0$ by the initial conditions. From this we can prove that $\\lim_{x\\to\\infty}(x-1)f''(\\xi(x)) = 0$, since we have the inequality $|(x-1)f''(\\xi(x))| < |\\xi(x)f''(\\xi(x))|$ at least for large x. \r\n\r\nNow we have enough information to prove what is required. From multiplying the equality $f'(x) - f'(x-1) = f''(\\xi(x))$ by $(x-1)$ we find that\r\n$\\lim_{x\\to\\infty}(x-1)f'(x-1) = \\lim_{x\\to\\infty}\\left((x-1)f'(x) - (x-1)f''(\\xi(x))\\right) = 0$.\r\n\r\nWhich is an equivalent statement to what's asked for. Is it clear now? I think it is! I realize that it's a very long and messy solution, but I am fairly confident that it is right.\r\n\r\n\r\n*We have that $\\lim_{x\\to\\infty}(\\eta(x) - 1)f'(\\eta(x)) = 0$ since $0 < |(\\eta(x) - 1)f'(\\eta(x))| < |(x - 1)f'(\\eta(x))|$ at least for large x, and we've shown that $\\lim_{x\\to\\infty}(x-1)f'(\\eta(x)) = 0$. The inequality $x-1 < \\eta(x)$ shows that $\\lim_{x\\to\\infty}\\eta(x) = \\infty$. Since $f'(y)$ is a continous function we can make the variable substiton $y = \\eta(x)$ to get that $\\lim_{y\\to\\infty}(y - 1)f'(y) = 0$", "Solution_10": "[quote=\"Kalle\"]\n*We have that $\\lim_{x\\to\\infty}(\\eta(x) - 1)f'(\\eta(x)) = 0$ since $0 < |(\\eta(x) - 1)f'(\\eta(x))| < |(x - 1)f'(\\eta(x))|$ at least for large x, and we've shown that $\\lim_{x\\to\\infty}(x-1)f'(\\eta(x)) = 0$. The inequality $x-1 < \\eta(x)$ shows that $\\lim_{x\\to\\infty}\\eta(x) = \\infty$. Since $f'(y)$ is a continous function we can make the variable substiton $y = \\eta(x)$ to get that $\\lim_{y\\to\\infty}(y - 1)f'(y) = 0$[/quote]\r\nHmm... \r\nDenote $E=\\{\\eta(x)|x>0\\}$\r\nwe have $\\lim_{x\\to \\infty}(\\eta(x) - 1)f'(\\eta(x)) =0$\r\nIf $\\eta(x)$ is not continuous then it is possible case when $\\forall N\\ \\; (N;\\infty)\\setminus E \\ne \\emptyset$.\r\nthen we can prove only that \\[ \\forall \\varepsilon \\exists N \\; (x>N)\\bigwedge(x \\in E) \\Rightarrow |(x-1)f'(x)|<\\varepsilon \\] but we need \\[ \\forall \\varepsilon \\exists N \\; (x>N) \\Rightarrow |(x-1)f'(x)|<\\varepsilon \\]\r\n :?: :?:", "Solution_11": ":(\r\n\r\nIf you don't see a way to save it, I probably don't either. There is no way to get around that problem with some continuity arguments? It is possible that the solution could be saved, since the \"1\" was chosen arbitrarily and we could repeat the argument with $f(x) - f(x-\\varepsilon)$, but I don't know. I retire. I've spent a lot of time on this problem, probably like 5+ hours, so that's it. \r\n\r\nPost the real solution.", "Solution_12": "I'm don't understand why it's wrong. If we have that $f$ is continous and that $\\lim_{x\\to\\infty}g(x) = \\infty$. Do we not have that\r\n$\\lim_{x\\to\\infty}f(g(x)) = \\lim_{y\\to\\infty}f(y)$ ?\r\nI thought it was so. \r\n\r\nWould it be true if $\\lim_{x\\to\\infty}g(x) = A$ so that $\\lim_{x\\to\\infty}f(g(x)) = \\lim_{y\\to A}f(y)$ ?", "Solution_13": "[quote=\"Kalle\"]I'm confused as to why it doesn't work. If we have that $f$ is continous and that $\\lim_{x\\to\\infty}g(x) = \\infty$. Do we not have that\n$\\lim_{x\\to\\infty}f(g(x)) = \\lim_{y\\to\\infty}f(y)$ ?\nI thought it was so. \n[/quote]\r\ni try to construct counterexample...\r\nlet $g(x)=[x] \\to \\infty,$ when $x\\to \\infty$ and let for example $f(x)=\\sin(\\pi x)$. \r\nthen we have $f(g(x))=\\sin(\\pi [x]) = 0 \\to 0$, but $\\sin (x) \\nrightarrow 0$ when $x\\to \\infty$...", "Solution_14": "[quote=\"Kalle\"]Would it be true if $\\lim_{x\\to\\infty}g(x) = A$ so that $\\lim_{x\\to\\infty}f(g(x)) = \\lim_{y\\to A}f(y)$ ?[/quote]\r\n\r\nlet $g(x)=\\frac{1}{[x]\\pi}$ and $f(x)=\\sin \\frac{1}{x}$. Then $g(x)\\to 0$,$f(g(x))=\\sin([x]\\pi)=0 \\to 0$ and $f(x)\\nrightarrow 0$ when $x\\to 0$... :(", "Solution_15": "Yes, I see you are right. My book actually does write $\\lim_{x\\to\\infty}f(g(x)) = \\lim_{y\\to\\infty}f(y)$ at places. Do they actually mean that if $\\lim_{y\\to\\infty}f(y) = A$ where $A$ is a constant, \"$\\infty$\" or \"$-\\infty$\" then we know that $\\lim_{x\\to\\infty}f(g(x)) = A$? But if the first limit diverges in some other way, we don't know anything about the second. And if we know something about the second we don't know anything about the first limit?\r\n\r\nI thought I understood limits, but now I don't understand simple calculations anymore :(", "Solution_16": "if there $\\exists \\lim_{x\\to \\infty} f(x)$ and $\\lim_{x\\to \\infty} g(x) =\\infty$ and $\\lim_{x\\to \\infty} f(g(x))=A$\r\nthen $\\lim_{x\\to \\infty} f(x)=A.$ but when we not knew $\\exists \\lim_{x\\to \\infty}f(x)$ or not, then existence $\\lim f(g(x))$ can't guarantee existence $\\lim f(x)$... ;)\r\nand when $\\exists \\lim f(x)$ then $\\forall g$ such that $g \\to\\infty$ when $x\\to \\infty$ we have \\[ \\lim f(g(x)) = \\lim f(x) \\]", "Solution_17": "Ok, thanks for the help! I am going to think through limits and how they work tomorrow. I am sorry if you feel that I have wasted your time. I thought my solution was correct.\r\n\r\nDo you know a real solution to this problem? What \"level\" is this problem meant for? All I know is the mean value theorem ;) Would it be possible to show that the limit actually exists in some way, so that my solution could work?", "Solution_18": "[quote=\"Kalle\"]Ok, thanks for the help! I am going to think through limits and how they work tomorrow. I am sorry if you feel that I have wasted your time. I thought my solution was correct.\n\nDo you know a real solution to this problem? What \"level\" is this problem meant for? [/quote]\r\n\r\nQuite the contrary Kalle. Thank you because you think about my problem :) \r\nReason is that i don't knew solution :( This problem is give me my friend who is a student of first year of study in our university...", "Solution_19": "Two related problems\r\n1) Let $A=\\sup |f(x)|$ $B=\\sup|f''(x)|$, then $\\sup |f'(x)|\\le 2\\sqrt{AB}$ (Landau ineq).\r\n2) If $f\\to 0$ as $x\\to \\infty$ and $f''$ is bounded then $f'(x)\\to 0$ as $x\\to \\infty$.(Hardy-Littlewood th)\r\n\r\nI know the proof for the first, and I think that the first serve in the proof of the second problem. The second problem is quite similar to our problem. I think that those two problems will help you to find a solution of the problem." } { "Tag": [ "LaTeX" ], "Problem": "Hi!\r\n\r\nI have a \\newcounter command for definitions in my paper. I use newcounter, because i don't like to have italic text in my definitions, as it would be if i used \\newtheorem command. The problem is that i can't use \\label and \\ref commands on my definitions. OK, i can use them, but it doesn't work properly :lol: .\r\nCan somebody please help me with this. Or is there a way to define newtheorem command that it wouldn't have italic text. (But just for definitions, not for theorems, lemmas, ...)\r\n\r\nThank you for your answer(s)!", "Solution_1": "[quote=\"brokenbone\"]Hi!\n\nI have a \\newcounter command for definitions in my paper. I use newcounter, because i don't like to have italic text in my definitions, as it would be if i used \\newtheorem command. The problem is that i can't use \\label and \\ref commands on my definitions. OK, i can use them, but it doesn't work properly :lol: .\nCan somebody please help me with this. Or is there a way to define newtheorem command that it wouldn't have italic text. (But just for definitions, not for theorems, lemmas, ...)\n\nThank you for your answer(s)![/quote]\r\n\r\nUsually what I did is to put\r\n\\theoremstyle{definition}\r\n\r\nSo for example\r\n\\newtheorem{theorem}{Theorem}\r\n\\theoremstyle{definition}\r\n\\newtheorem{remark}{Remark}\r\n\\newtheorem{definition}{Definition}\r\nabove all \\newtheorem that you want to write in definition style (not italic text).\r\nBut I forgot whether I should put \\usepackage{amsthm} or \\usepackage{theorem}", "Solution_2": "Thank you!\r\n\r\nThat works (with amsthm package)! The thing that bothers me now (never satisfied ) :D is that definitions and theorems use the same counter. \r\n\r\nFor example:\r\n[b]Theorem 1.1[/b]\r\n...\r\n[b]Theorem 1.2[/b]\r\n...\r\n[b]Definition 1.3[/b]\r\n\r\nI'd like to have [b]Definition 1.1[/b] in this example.", "Solution_3": "I'm sorry, it doesn't count them like that, it works fine. \r\nThanks again for your answer" } { "Tag": [ "algebra", "polynomial", "logarithms", "Gauss", "algebra proposed" ], "Problem": "let $k,n\\in \\mathbb{N}$ and $P(x)$ apolynomial of degree $n$ whose coefficients are all in the set $\\{-1,0,1\\}$. We are given that $(x-1)^{k}|P(x)$ and that there exists a $q$ prime such that $\\frac{q}{\\ln q}<\\frac{k}{\\ln (n+1)}$. Show that at least one root of unity of index $q$ will be a root of $P(x)$.", "Solution_1": "As Arqady would say: Unsolved in 2006!\r\nanyway this problem is not so difficult, I'm surprised it recieved no solution (or attention :P )", "Solution_2": "I guess it's the presence of those $ \\ln$ that made the problem sound difficult :D . Anyway, it is more or less trivial: assuming the contrary and denoting $ x_{1},x_{2},...,x_{q}$ the roots of order $ q$ of unity, we have on one hand $ |P(x_{1})...P(x_{q})|\\leq n^{q}$ because of the coefficients and degree conditions. On the other hand, write $ P\\equal{}(x\\minus{}1)^{k}Q$ then observe that $ |P(x_{1})...P(x_{k})|\\equal{}q^{k}|Q(x_{1})...Q(x_{k})|\\geq q^{k}$, because by fundamental theorem of symmetric polynomials $ Q(x_{1})...Q(x_{k})$ is an integer (ok, Gauss lemma too).", "Solution_3": "That's OK, :lol: \r\nAnd you meant $ |P(x_{1})\\cdots P(x_{q})|\\le (n\\plus{}1)^{q}$ of course :)" } { "Tag": [ "AMC", "USA(J)MO", "USAMO", "AIME", "trigonometry", "vector", "geometry" ], "Problem": "Great Idea Rep!\r\n\r\nThat'll give someone time to prepare the Mock AIME also. Really appreciate it! Will this test be in the same format as the previous one by JSRosen?", "Solution_1": "Very good idea!\r\n\r\n(thinks to self: The more people we have, the more likely we are to have a Mock USAMO! muahahaha....)\r\n\r\nEdit: Actually, assuming that I don't end up writing the Mock AIME, I'd like to co-write this test. It'd probably help me more than taking it...", "Solution_2": "I MUST participate! I mad SOO many stupid mistakes on this one that I need to make a better score.", "Solution_3": "pkZKMbyW4UPdkRsgJ5xP+j8i+qckkW5ZznguEmtF5pAz8FyMwxsvrm2qBbia", "Solution_4": "[quote=\"Rep123max\"]Ya, it would be the same way (since that way worked really well). I'll do it if we have a minimum of 15 people. And is a .pdf of the test okay?[/quote]\r\n\r\nRep: Honestly, I want to know if I could co-write the test. I may have to captain (and possibly train) my math team this year, so I need experience in problem-writing.", "Solution_5": "I'm in! It'll be my chance to redeem myself after a very VERY embarrassing 53.5 :)", "Solution_6": "I have to redeem myself!", "Solution_7": "I'd like to take it...\r\n\r\neven though I made the Mock AIME already (barely... i was 14th/29 I think)\r\n\r\nI need all the practice I can get though... so bring it on!", "Solution_8": "Will this one be a bit easier than the first mock AMC contest?", "Solution_9": "I'm in. Lol. I messed up on the first one. Trigs and number theory beat me on the face good. :( \r\n\r\nP.S. Is yours gonna be easier than JSRosens'? I hope it is..", "Solution_10": "Yeh...Rep...you might want to think about making this AMC contest lean a bit more to AMC 10 than AMC 12 since this is after all a bonus qualifier for the AIME.", "Solution_11": "I suppose I already made mock AIME, but I'd like to take it anyway.", "Solution_12": "If your going to make another AMC, please make it easier. Try make the first half of the test the same difficulty as about the first 5 questions in JRosen's test.\r\n\r\nMabye then I can get at least a 100 on something besides math class.", "Solution_13": "[quote=\"kool_dudy\"]Yeh...Rep...you might want to think about making this AMC contest lean a bit more to AMC 10 than AMC 12 since this is after all a bonus qualifier for the AIME.[/quote]\r\n\r\nIt may turn out that way, as I can't write trig problems. :D\r\n\r\nWe'll work on it.", "Solution_14": "I'm definitiely up for it...I have to beat yif man12 for lowest score!", "Solution_15": "Rep123max wrote:I wrote their scores on a piece of scratch paper, but since I am 99% positive I will lose it, I thought to put it on Excel.\n\n\n\nThis can lead rise to a math problem: if there is a 99% chance that Rep123max will lose a piece of scratch paper with scores on it, what is the fewest number of copies Rep123max must make in order to be at least 50% sure he will still be able to find at least one of them (the original sheet doesn't count as a copy)?\n\n\n\nAnswer (in spoler):[hide]68.[/hide]", "Solution_16": "somebody has too much free time on their hands. :lol:", "Solution_17": "I somehow doubt it took him very long.\r\n\r\nPlus, I found it quite amusing.\r\n\r\n(Yay, AMC10B is tomorrow!)", "Solution_18": "[quote=\"genius1601\"]somebody has too much free time on their hands. :lol:[/quote]\r\n\r\nProbably you, David.", "Solution_19": "pkZKMbyW4UPdkRsgJ5xP+j8i+qckkW5ZznguEmtF5pAz8FyMwxsvrm2qBbia", "Solution_20": "I should be fine with Wed. 6PM EST. If not the hopefully I'll be able to PM you before or during to let you know I can't do it.", "Solution_21": "I'd advise against that, simply because it's generally not good to have the person who got the lowest score know they got the lowest score...it can be very demoralizing :(", "Solution_22": "I can't find the the topic Problems of Mock AMC B, can someone post the link to me?", "Solution_23": "pkZKMbyW4UPdkRsgJ5xP+j8i+qckkW5ZznguEmtF5pAz8FyMwxsvrm2qBbia", "Solution_24": "Yeah I can't find it either.\r\nWhere'd it go?", "Solution_25": "Good Luck everyone!!!!", "Solution_26": "Good Luck!!", "Solution_27": "i'll need the luck. :D", "Solution_28": "yup good luck everyone!", "Solution_29": "[quote=\"JSRosen3\"]I'd advise against that, simply because it's generally not good to have the person who got the lowest score know they got the lowest score...it can be very demoralizing :([/quote]\r\n\r\nhardy har har. :)" } { "Tag": [ "probability", "function", "probability and stats" ], "Problem": "I have come across a problem in A N Shiryaev, Probability page no. 387 . There he is saying the following obvious.\r\n$ \\sum \\mathbb{E}\\left(\\frac{X_n ^2}{1\\plus{}|X_n|}\\right) < \\infty \\Rightarrow \\sum \\mathbb{E}(X_n ^2 I_{\\{|X_n| \\leq 1\\}} \\plus{} |X_n| I_{\\{|X_n| > 1\\}}) < \\infty$\r\nwhere $ X_n$ are independent random variables with $ \\mathbb{E}(X_n) \\equal{}0$\r\nCan anyone justify this? Please...\r\n\r\n\r\nThank you.", "Solution_1": "Let $ f$ be the function $ f(x) \\equal{} \\frac{x^2}{1 \\plus{} |x|}$. Let $ g$ be the function such that $ g(x) \\equal{} x^2$ for $ |x| \\le 1$ and $ g(x) \\equal{} |x|$ for $ |x| > 1$. Then $ g(x) \\le 2f(x)$. That should justify the implication.", "Solution_2": "Thank you very much." } { "Tag": [ "geometry", "circumcircle", "Euler", "incenter", "geometric transformation", "homothety" ], "Problem": "Let $(I),(O)$ be the incircle, and, respectiely, circumcircle of $ABC$. $(I)$ touches $BC,CA,AB$ in $D,E,F$ respectively. We are also given three circles $\\omega_a,\\omega_b,\\omega_c$, tangent to $(I),(O)$ in $D,K$ (for $\\omega_a$), $E,M$ (for $\\omega_b$), and $F,N$ (for $\\omega_c$).\r\n\r\n[b]a)[/b] Show that $DK,EM,FN$ are concurrent in a point $P$;\r\n\r\n[b]b)[/b] Show that the orthocenter of $DEF$ lies on $OP$.", "Solution_1": "I don't have time to finish it right now, but I guess the following approach works.\r\nFor part a, just note that DK bisects the arc BC than containing A.\r\nFor part b, use the well known fact that O lies on the Euler line of triangle DEF.", "Solution_2": "$D$ - point which is center of omothety of one $\\omega_a$ and incircle,\r\n$K$ - point which is center of omothety of one $\\omega_a$ and circumcircle.\r\nIt rezults that point which is center of omothety of circumcircle and incircle lies on $DK$.\r\nHence $P$ is exactly point of similitude of incircle and circumcircle (I think so called).\r\nSo it lies on $OI$ and from Iran 1995 it contains ortocenter of $DEF$.", "Solution_3": "Can anyone give a clear solution please ?? :?", "Solution_4": "For Erdos, I am rewriting Prowler's solution:\r\n\r\n[quote=\"grobber\"]Let $(I),(O)$ be the incircle, and, respectiely, circumcircle of $ABC$. $(I)$ touches $BC,CA,AB$ in $D,E,F$ respectively. We are also given three circles $\\omega_a,\\omega_b,\\omega_c$, tangent to $(I),(O)$ in $D,K$ (for $\\omega_a$), $E,M$ (for $\\omega_b$), and $F,N$ (for $\\omega_c$).\n\n[b]a)[/b] Show that $DK,EM,FN$ are concurrent in a point $P$;\n\n[b]b)[/b] Show that the orthocenter of $DEF$ lies on $OP$.[/quote]\r\n\r\n[i]Solution.[/i] Let Q be the internal center of similitude of the circumcircle (O) and the incircle (I) of triangle ABC. Then, of course, this point Q lies on the line OI which joins the centers O and I of these two circles.\r\n\r\nBy the Monge theorem, the internal center of similitude of the circles (O) and (I), the internal center of similitude of the circles (I) and $\\omega_a$, and the external center of similitude of the circles $\\omega_a$ and (O) are collinear. But the internal center of similitude of the circles (O) and (I) is the point Q; the internal center of similitude of the circles (I) and $\\omega_a$ is the point D (since these two circles externally touch at D), and the external center of similitude of the circles $\\omega_a$ and (O) is the point K (since these two circles internally touch at K). Hence, the points Q, D and K are collinear; in other words, the point Q lies on the line DK. Similarly, the point Q lies on the lines EM and FN. Thus, the lines DK, EM and FN are concurrent at the point Q. This solves part [b](a)[/b] of the problem, and shows that the point of concurrence P of the lines DK, EM and FN is the point Q.\r\n\r\nNow, in order to prove part [b](b)[/b] of the problem, it is enough to show that the orthocenter of triangle DEF lies on the line OQ. Since the point Q lies on the line OI, the line OQ is the line OI; hence, it remains to show that the orthocenter of triangle DEF lies on the line OI. In other words, it remains to show that the orthocenter of triangle DEF, the incenter I of triangle ABC and the circumcenter O of triangle ABC are collinear. But this was shown in http://www.mathlinks.ro/Forum/viewtopic.php?t=6228 .\r\n\r\nProblem solved.\r\n\r\n Darij", "Solution_5": "Thanks Darij :lol:", "Solution_6": "Also $AK,BM,CN$ are concurrent.", "Solution_7": "Dear Mathlinkers,\nan article concerning the \u201cAyme\u2019s theorem\u2019\u2019 and this question have been put on my website.\nhttp://perso.orange.fr/jl.ayme vol. 20 p. 17, 18\nYou can use Google translator\nSincerely\nJean-Louis", "Solution_8": "[hide=\"a\"]By Monge d'Alembert, they concur at the insimilicenter of $(I), (O)$.[/hide]\n\n[hide=\"b\"]By a, $P\\in OI$, so it suffices to show that $H\\in OI$, where $H$ is the orthocenter of triangle $DEF$. Let $XYZ$ be the orthic triangle of triangle $DEF$. Angle chasing yields triangles $XYZ$, $ABC$ are homothetic, so let $\\mathcal{H}$ be the homothety that sends them to one another. $\\mathcal{H}$ sends their incenters $H$, $I$ and their circumcenters $N$, $O$ to one another, where $N$ is the nine-point center of triangle $DEF$; hence $H\\in OI$ as desired. $\\blacksquare$[/hide]\n\nNotice the similarity to [url=http://www.artofproblemsolving.com/Forum/viewtopic.php?p=268390]2003 Vietnam TST #2[/url], which concerns the incenter of the orthic triangle rather than the orthocenter of the intouch triangle.", "Solution_9": "Here is my solution for this problem \n[b]Solution[/b] \na) We define again point $K$ is second intersection of $\\bigodot(AI)$ with ($O$) \nLet $X$, $Y$, $Z$ be midpoint of $\\stackrel\\frown{BC}$ which not containing $A$, $\\stackrel\\frown{CA}$ which not containing $B$, $\\stackrel\\frown{AB}$ which not containing $C$ \nThen: $K$ is Miquel point of quadrilateral $BCEF$ \nSo: $\\triangle$ $KBF$ $\\stackrel{+}{\\sim}$ $\\triangle$ $KCE$ \nHence: $\\dfrac{KB}{KC}$ = $\\dfrac{BF}{CE}$ = $\\dfrac{BD}{CD}$ or $KD$ is internal bisector of $\\widehat{BKC}$ \nLet $J$ $\\equiv$ $OK$ $\\cap$ $ID$ \nWe have: ($KD$; $KJ$) $\\equiv$ $\\dfrac{\\pi}{2}$ $-$ ($CK$; $CX$) $\\equiv$ $\\dfrac{\\pi}{2}$ $-$ ($DK$; $DB$) $\\equiv$ ($DJ$; $DK$) (mod $\\pi$) \nSo: ($J$; $JK$) $\\equiv$ $\\omega_a$ \nThen: $DK$ passes through $X$ \nSimilarly: $Y$ $\\in$ $EM$, $Z$ $\\in$ $FN$ \nBut: $EF$ $\\parallel$ $YZ$, $FD$ $\\parallel$ $ZX$, $DE$ $\\parallel$ $XY$ then: $DX$, $EY$, $FZ$ concurrent at point $P$ is external homothetic center of ($I$) and ($O$) or $DK$, $EM$, $FN$ concurrent at $P$ \nb) Let $H$ be orthocenter of $\\triangle$ $DEF$ \nSince: $I$ is orthocenter of $\\triangle$ $XYZ$, we have: $I$, $H$, $P$ are collinear \nBut: $P$, $I$, $O$ are collinear then $I$, $H$, $P$, $O$ are collinear of $H$ $\\in$ $PO$" } { "Tag": [], "Problem": "six-sevenths in a fraction of the party delegates were proclaiming \r\ntheir positions. If 1100 were not involved in his behavior, how many delegates were there in all?", "Solution_1": "[quote=\"sharkman\"]six-sevenths in a fraction of the party delegates were proclaiming \ntheir positions. If 1100 were not involved in his behavior, how many delegates were there in all?[/quote]\r\n\r\n[hide]\n1100 is $\\frac{1}{7}$ th of the total. so 7700 is the total number of delagates. \n[/hide]", "Solution_2": "[quote=\"sharkman\"]six-sevenths in a fraction of the party delegates were proclaiming \ntheir positions. If 1100 were not involved in his behavior, how many delegates were there in all?[/quote]\r\nNote that it doesn't specify behavior.\r\nUsing Izebiglips's Lemma, we see that the result is 7700.", "Solution_3": "[quote=\"diophantient\"][quote=\"sharkman\"]six-sevenths in a fraction of the party delegates were proclaiming \ntheir positions. If 1100 were not involved in his behavior, how many delegates were there in all?[/quote]\nNote that it doesn't specify behavior.\nUsing Izebiglips's Lemma, we see that the result is 7700.[/quote]\r\n\r\nwhat exactly is Izebiglips's Lemma? Is it a type of formula of some kind?\r\n\r\nFrom what I can tell, the answer is 7700, which makes this problem way too easy for this forum if I can figure it out that quickly", "Solution_4": "[quote=\"rd5493\"][quote=\"diophantient\"][quote=\"sharkman\"]six-sevenths in a fraction of the party delegates were proclaiming \ntheir positions. If 1100 were not involved in his behavior, how many delegates were there in all?[/quote]\nNote that it doesn't specify behavior.\nUsing Izebiglips's Lemma, we see that the result is 7700.[/quote]\n\nwhat exactly is Izebiglips's Lemma? Is it a type of formula of some kind?\n\nFrom what I can tell, the answer is 7700, which makes this problem way too easy for this forum if I can figure it out that quickly[/quote]\r\n\r\nFrom my sources, there is no Izebiglips's Lemma.", "Solution_5": "Izebiglips is probably an AoPS user." } { "Tag": [ "analytic geometry", "graphing lines", "slope" ], "Problem": "How far is the line x+y=0 from the circle centered at (7,4) having a radius of 3 times the square root of 2?", "Solution_1": "[hide]In slope-intercept form, the first line (line $m$), is $y=-x.$ The line perpendicular to line $m$ that goes through $(7,\\,4)$ is $y=x-3.$ The intersection of these two lines is at $\\left(\\frac{3}{2},\\,-\\frac{3}{2}\\right).$ The distance from this point to the center of the circle is $D=\\sqrt{\\left(4+\\frac{3}{2}\\right)^{2}+\\left(7-\\frac{3}{2}\\right)^{2}}=\\sqrt{\\frac{242}{4}}=\\frac{11\\sqrt{2}}{2}.$ The distance from the circle is $\\frac{11\\sqrt{2}}{2}-3\\sqrt{2},$ or $\\boxed{\\frac{5\\sqrt{2}}{2}}.$[/hide]", "Solution_2": "[quote=\"i_like_pie\"][hide]In slope-intercept form, the first line (line $m$), is $y=-x.$ The line perpendicular to line $m$ that goes through $(7,\\,4)$ is $y=x-3.$ The intersection of these two lines is at $\\left(\\frac{3}{2},\\,-\\frac{3}{2}\\right).$ The distance from this point to the center of the circle is $D=\\sqrt{\\left(4+\\frac{3}{2}\\right)^{2}+\\left(7-\\frac{3}{2}\\right)^{2}}=\\sqrt{\\frac{242}{4}}=\\frac{11\\sqrt{2}}{2}.$ The distance from the circle is $\\frac{11\\sqrt{2}}{2}-3\\sqrt{2},$ or $\\boxed{\\frac{5\\sqrt{2}}{2}}.$[/hide][/quote]\r\n\r\nit didn't show up right the first time, but now i see it. thanks" } { "Tag": [], "Problem": "prove that the square of any prime, $p>3$, is one more than a multiple of 24.\r\nhave fun. i think this is really easy, just want to see how different people solve it.", "Solution_1": "[hide]Any prime greater than 3 is $1\\mod{3}$. Hence, the square of a prime is $1\\mod{3}$. Thus, any square of a prime is already one more than a multiple of 3.\nNow take primes mod 8. Clearly, a prime has to be one of $1\\mod{8}$, $3\\mod{8}$, $5\\mod{8}$ and $7\\mod{8}$. Any of these yields that the square of a prime is $1\\mod{8}$. Thus, any square of a prime is one more than a multiple of 8.\nSo, being one more than than a multiple of 8 and one more than a multiple of 3, the square of a prime must be one more than a multiple of 3*8=24, because 3 and 8 are relatively prime. QED[/hide]", "Solution_2": "p^2 = 24n + 1\r\np^2 - 1 = (p+1)(p-1) = 24n\r\n24|(p-1)(p+1)\r\n\r\np-1 and p+1 are both even numbers because p must be odd, and one of them will also be a multiple of 4, so we know 8|(p-1)(p+1).\r\n\r\nAlong with that, because p cannot be a multiple of 3, p-1 or p+1 has to be divisible. We could use casework with mods to see this.\r\n\r\nSince 8 and 3 are both divisors of (p-1)(p+1), 24 is a factor of p^2 - 1. Therefore, p^2 is one greater than a multiple of 24 for all primes p." } { "Tag": [ "number theory unsolved", "number theory" ], "Problem": "1. Show that $ 1987$ divides $ 2903^n\\minus{}803^n\\minus{}464^n\\plus{}261^n$ for all positive integers $ n$.\r\n\r\n2. Show that $ S\\equal{}5^n\\plus{}2*3^(n\\minus{}1)\\plus{}1$ is divisible by $ 8$ for all integers $ >\\equal{}1$.\r\n\r\n3. Let $ n$ be a positive integer. Show that any number greater than $ frac{n^4}{16}$ can be written in at most one way as the product of two of its divisors having difference not exceeding $ n$. \r\n\r\n4. Let $ k$ and $ n$ be positive integers. Show that the only solutions $ (k,n)$ of the equation $ (n\\minus{}1)!\\equal{}n^k\\minus{}1$ are $ (1,2), (1,3)$ and $ (2,5)$", "Solution_1": "[quote=\"kl2836\"]1. Show that $ 1987$ divides $ 2903^n \\minus{} 803^n \\minus{} 464^n \\plus{} 261^n$ for all positive integers $ n$.\n\n2. Show that $ S \\equal{} 5^n \\plus{} 2*3^(n \\minus{} 1) \\plus{} 1$ is divisible by $ 8$ for all integers $ > \\equal{} 1$.\n\n3. Let $ n$ be a positive integer. Show that any number greater than $ frac{n^4}{16}$ can be written in at most one way as the product of two of its divisors having difference not exceeding $ n$. \n\n4. Let $ k$ and $ n$ be positive integers. Show that the only solutions $ (k,n)$ of the equation $ (n \\minus{} 1)! \\equal{} n^k \\minus{} 1$ are $ (1,2), (1,3)$ and $ (2,5)$[/quote]\r\n\r\n\r\n2. $ n\\equal{}2k$ ----> $ 5^{2k}\\plus{}2*3^{2k\\minus{}1}\\plus{}1\\equiv25^{k}\\plus{}6*9^{k\\minus{}1}\\plus{}1\\equiv1^{k}\\plus{}6*1^{k\\minus{}1}\\plus{}1\\equiv 0 (mod8)$\r\n $ n\\equal{}2k\\plus{}1$ ----> $ 5^{2k\\plus{}}\\plus{}2*3^{2k}\\plus{}1\\equiv5*25^{k}\\plus{}2*9^{k}\\plus{}1\\equiv5*1^{k}\\plus{}2*1^{k\\minus{}1}\\plus{}1\\equiv 0 (mod8)$" } { "Tag": [], "Problem": "Find the sum of the digits of (100000+10000+1000+100+10+1)^2", "Solution_1": "[quote=\"mdk\"]Find the sum of the digits of (100000+10000+1000+100+10+1)^2[/quote]\r\n\r\n\r\n$ (10^{5}+10^{4}+10^{3}+10^{2}+10^{1}+10^{0})^{2}$ has 36 terms, and each of them are powers of 10. Since no single power of 10 is counted more than nine times, the answer is 36.", "Solution_2": "Note the identity:\r\n\r\n\\begin{tabular}{lcl}(1+x+...+x^{n})^{2}&=&\\\\ && 1+2x+3x^{2}+....+(n+1)x^{n}+\\\\ &&nx^{n+1}+\\\\ &&(n-1)x^{n+2}+(n-2)x^{n+3}....+x^{2n}\\\\ \\end{tabular}" } { "Tag": [ "ARML", "geometry" ], "Problem": "The only known virginia ARML team is TJ's. But this blocks many good math students from competing. Is there any way to go without going to TJ?\r\n\r\nThis questions has also been posted in the ARML forum.", "Solution_1": "last years arml sucked(for both of us but for me more than you)", "Solution_2": "last year's ARML owned. XD Can't believe I lost to allen and Kevin by 1 T.T", "Solution_3": "it was really hard (for me at least).\r\n\r\nwas it just me or was there an abnormally large number of geometry problems?", "Solution_4": "anybody interested in trying to make a VA ARML team not from TJ?" } { "Tag": [], "Problem": "What is 9,883 \u00d7 2,739?", "Solution_1": "[hide=\"Answer\"]27,0695,537[/hide]", "Solution_2": "[quote=\"math92\"]What is 9,883 \u00d7 2,739?[/quote]\r\n\r\n[hide=\"Answer\"] 27, 069, 537 [/hide]" } { "Tag": [ "geometry proposed", "geometry" ], "Problem": "For which real numbers $ x>1$ is there a triangle with side lengths $ x^4\\plus{}x^3\\plus{}2x^2\\plus{}x\\plus{}1, 2x^3\\plus{}x^2\\plus{}2x\\plus{}1,$ and $ x^4\\minus{}1$?", "Solution_1": "$ (x^4 \\plus{} x^3 \\plus{} 2x^2 \\plus{} x \\plus{} 1) \\plus{} (x^4 \\minus{} 1) > (2x^3 \\plus{} x^2 \\plus{} 2x \\plus{} 1)\\Rightarrow 2x^4 \\minus{} x^3 \\plus{} x^2 \\minus{} x \\minus{} 1 > 0$\r\n$ (2x^3 \\plus{} x^2 \\plus{} 2x \\plus{} 1) \\plus{} (x^4 \\minus{} 1) > (x^4 \\plus{} x^3 \\plus{} 2x^2 \\plus{} x \\plus{} 1)\\Rightarrow x^3 \\minus{} x^2 \\plus{} x \\minus{} 1 > 0$\r\n$ (2x^3 \\plus{} x^2 \\plus{} 2x \\plus{} 1) \\plus{} (x^4 \\plus{} x^3 \\plus{} 2x^2 \\plus{} x \\plus{} 1) > (x^4 \\minus{} 1)\\Rightarrow 3x^3 \\plus{} 3x^2 \\plus{} 3x \\plus{} 3 > 0$\r\n\r\nThese are all true for $ x > 1$\r\n\r\nOr, one can realize that $ x^4\\plus{}x^3\\plus{}2x^2\\plus{}x\\plus{}1$ is the largest side, so only the second line is actually necessary." } { "Tag": [ "function", "geometry", "pigeonhole principle", "ARML" ], "Problem": "I thought it was significantly easier this year.\r\n\r\n1: almost trivial, just parity analysis.\r\n2: also very easy, definition of the functions\r\n3: pretty simple, I expressed areas in terms of side lengths and solved for AB*BC. My friend used guess and check to solve it, which I thought was unfortunate...\r\n4: I thought this was the hardest one, I think i got it though\r\n5: (a) and (b) were easy, (c) was tough, I don't think I got (c).", "Solution_1": "#4 was really complicated, and I just gave up. I got everything else though.", "Solution_2": "How did you do 5(c)?\r\n\r\nFor 4, I showed that there existed a one-to-one correspondence between any arbitrary 7x7 board and 8x8 boards that satisfy the conditions. So I got an answer of $ 2^{49}$?", "Solution_3": "That seems right, now that I think about it. :wallbash: \r\n\r\nFor 5c, I basically considered the strategies of the $ 2009$ people when they had to consider the cases when they saw $ 1004$ red hats and $ 1004$ blue hats. By pigeonhole, at least $ 1005$ people make the same choice. Give $ 1005$ people that do this the hat that they would not expect in such a situation, and give the other $ 1004$ people the other color hat. Since $ 1005$ people are guaranteed to be wrong, there is no such strategy for $ 2009$ people.", "Solution_4": "I was confused as to how we know for sure that for 2008, 1004 is the optimum number of correct hats the people can get. I might just be being dumb right now. :( And why are 1005 people \"guaranteed to be wrong\"?", "Solution_5": "It's optimum in that $ 2009 \\minus{} 1005 \\equal{} 1004$.\r\n\r\nBy pigeonhole, at least $ 1005$ people will decide on, assuming WLOG, choosing a red hat in such a case. Give an arbitrary subset of $ 1005$ of those people a blue hat, and the other $ 1004$ people red hats. Those $ 1005$ people with blue hats will see $ 1004$ red hats and $ 1004$ blue hats, and deduce they are wearing red hats. Therefore, they are all wrong.", "Solution_6": "Why will the 1005 people deduce they are wearing red hats, from the fact that they see 1004 red and 1004 blue hats? Sorry, I don't think I'm following you... :maybe:", "Solution_7": "Consider the strategies of the $ 2009$ people when they have to decide on the color hat they're wearing when they see $ 1004$ people wearing red hats and $ 1004$ people wearing blue hats. By the pigeonhole principle, at least $ 1005$ people must share the same choice in the color hat they choose. Assume without loss of generality that these people guess that they're wearing a red hat when confronted with this situation. Take $ 1005$ people that make this decision, and give them blue hats. The other $ 1004$ people are given red hats.\r\n\r\nWhen it is time to guess, the $ 1005$ blue-hat-wearers will observe that $ 1004$ people are wearing red hats, and $ 1004$ people are wearing blue hats. Therefore, they will all guess they are wearing red hats. Since $ 1005$ people are already wrong, a maximum of $ 1004$ people can be right, so no set of strategies exists that consistently yields $ 1005$ or more people guessing correctly.", "Solution_8": "Oh, so the people are using Pigeonhole. I see what you mean. That was clever, to take a specific case of hat distribution, because that's all that was really needed....", "Solution_9": "Can anyone post the problems for part2, please?", "Solution_10": "http://www.math.umd.edu/highschool/mathcomp/2009.html", "Solution_11": "Also, solutions are up! Looks like $ 2^{49}$ is right for number 4. :)", "Solution_12": "Congratulations! Maybe the first place winner won't come from Montgomery County this year. :P", "Solution_13": "Haha no, I definitely won't be first place. I'm sure there's someone out there who will ace part 2. And do better on part 1.", "Solution_14": "I'm rooting for Linus Hamilton. :D", "Solution_15": "Oh, I don't know who that is!", "Solution_16": "He's a 10th grader at Eleanor Roosevelt. We were on the same ARML team.\r\n\r\n(I'm just hoping we can continue this trend of not having a Blair person win. Not that I have anything against Blair, but they did keep us from a first place finish at UMD HSPC. :P)", "Solution_17": "I know what you mean... do you go to Roosevelt?", "Solution_18": "Nope. I left a hint in my previous post as to what school I go to. :oops:", "Solution_19": "Annapolis High?", "Solution_20": ":yup::yup::yup:" } { "Tag": [ "function", "vector", "limit", "abstract algebra", "real analysis", "real analysis unsolved" ], "Problem": "Let $ a$ be a stationary point of a twice continuously differentiable function $ f, f: \\mathbb{R}^{n}\\rightarrow \\mathbb{R}$. Show, that if $ \\det{Hf(a)}\\neq 0$, then $ a$ is an isolated stationary point, that means there does not exist any other stationary point in the small neighbourghood of $ a$.", "Solution_1": "More general statement: Suppose $ g_1,\\dots,g_n$ are continuously differentiable functions such that \r\n(I) $ g_i(a)\\equal{}0$ for all $ i$; \r\n(II) the vectors $ \\nabla g_i(a)$ are linearly independent. \r\nThen $ a$ is an isolated point of the set $ Z\\equal{}\\{x\\colon g_i(x)\\equal{}0\\ \\forall i\\}$. \r\n\r\nIdea of proof: suppose there is a sequence $ x_i\\to a$ of other elements of $ Z$. Passing to a subsequence, we may assume that $ \\frac{x_i\\minus{}a}{\\parallel{}x_i\\minus{}a\\parallel{}}$ converges to some unit vector $ v$. Prove that $ v$ is orthogonal to $ \\nabla g_i$ for every $ i$.", "Solution_2": "Vectors $ \\nabla g_{i}(a)$ $ \\forall i$ are linearly independent. Therefore $ \\det(Df(a))\\neq 0$ ($ f\\equal{}(g_1,g_2,\\ldots,g_n)$). Hence $ Z$ is a manifold.\r\n\r\nBut then $ v: \\equal{}\\lim_{i\\to \\infty} \\frac{x_{i}\\minus{}a}{\\parallel x_{i}\\minus{}a\\parallel }$ lies in the tangent space of $ Z$ for the point $ a$. Indeed, we can make a smooth path $ \\xi: (\\minus{}\\delta,\\delta)\\rightarrow M$, which goes through $ a$ and $ x_i$. Since $ x_i$ can come arbitrarily close to $ x$, $ x\\minus{}a_i$ will be a tangent vector on $ Z$ in $ a$ for $ i \\to \\infty$. We know, that in the tangent space all vectors are orthogonal to $ \\nabla g_{i}(a)$ $ \\forall i$. But since $ v\\neq 0$, we get a contradiction with nonsingularity of $ Df(a)v$, $ Df(a)v\\equal{}0$.", "Solution_3": "There shall also exist a solution to my problem using Taylor series for $ \\frac {\\partial f}{\\partial x_i} \\forall i$ in the point $ (a \\plus{} h)$.\r\n\r\nI get $ \\frac {\\partial f}{\\partial x_i}(a \\plus{} h) \\equal{} \\sum^{n}_{j \\equal{} 1}\\frac {\\partial^2 f}{\\partial x_j\\partial x_i}(a \\plus{} th) h_j$, where $ t\\in (0,1)$. Using continuity of $ \\frac {\\partial^2 f}{\\partial x_j\\partial x_i}$ in $ a$, I get\r\n$ \\frac {\\partial f}{\\partial x_i} (a \\plus{} h) \\equal{} \\sum^{n}_{j \\equal{} 1}\\left(\\frac {\\partial^2 f}{\\partial x_j\\partial x_i}(a) \\plus{} \\epsilon_{ji}\\right)h_j.$\r\n\r\nNow if $ \\frac {\\partial f}{\\partial x_i} (a \\plus{} h) \\equal{} 0 \\forall i$ and some arbitrarily small $ h$, then $ \\sum^{n}_{i \\equal{} 1}\\frac {\\partial f}{\\partial x_i} (a \\plus{} h) \\equal{} 0$. But then\r\n\r\n$ \\sum^{n}_{i \\equal{} 1}\\frac {\\partial f}{\\partial x_i} (a \\plus{} h) \\equal{} \\sum^{n}_{i \\equal{} 1}\\sum^{n}_{j \\equal{} 1}\\left(\\frac {\\partial^2 f}{\\partial x_j\\partial x_i}(a) \\plus{} \\epsilon_{ji}\\right)h_j \\equal{} \\\\\r\n\\sum^{n}_{i \\equal{} 1}\\sum^{n}_{j \\equal{} 1}\\frac {\\partial^2 f}{\\partial x_j\\partial x_i}(a)h_j \\plus{} \\sum^{n}_{i \\equal{} 1}\\sum^{n}_{j \\equal{} 1}\\epsilon_{ji}h_j \\equal{} 0.$ \r\n\r\nNow I do not see , why this is necessarily a contradiction. Starting a little bit different, we can have a control over $ \\sum^{n}_{i \\equal{} 1}\\sum^{n}_{j \\equal{} 1}\\epsilon_{ji}h_j$, which means, that we can make it arbitrarily small. However, I do not see, why $ \\sum^{n}_{i \\equal{} 1}\\sum^{n}_{j \\equal{} 1}\\frac {\\partial^2 f}{\\partial x_j\\partial x_i}(a)h_j$ is not $ 0$. How is this connected with a nonzero determinant?", "Solution_4": "You can't arrive at a contradiction starting from $ \\sum^{n}_{i \\equal{} 1}\\frac {\\partial f}{\\partial x_i} (a \\plus{} h) \\equal{} 0$, because this equality may well occur in a lot of places.\r\n\r\nIf you don't sum over $ i$, your argument should yield $ \\sum^{n}_{j \\equal{} 1}\\frac {\\partial^2 f}{\\partial x_j\\partial x_i}(a)h_j\\equal{}0$ for every $ i$. And this is a contradiction: $ h$ is in the kernel of the Hessian matrix.", "Solution_5": "Thank you for both replies, mlok." } { "Tag": [], "Problem": "Each day, two people go to a 2 mile circular track. One walks around it once and the other jogs around it once. They each go a constant speed. \r\nOne day, they went in the same direction. They both started in the same place but at different times. The walker had gone 3520 ft. when the jogger started. The jogger passed the walker 10 minutes later. After the jogger had passed him, the walker took 30 minutes to finish.\r\nHow fast is the jogger in mph?", "Solution_1": "[hide]3520/5280 = 2/3\nanything sub 1 = walker\nanuthing sub 2 = jogger\nRate = miles/hour\n\nD=RT\nD = RT\nR_2 = R_1+(2/3)/(1/6) = R_1+4\n\n2 miles for walker = 2/3 mile + 10 min + 30 min\n4/3 mile for walker = 40 min\n1 mile for walker = 30 min\n2 = R_1*1/2\nR_1 =4\n\nR_2 = 4+4\nR_2 = 8\n>>8<<[/hide]", "Solution_2": "sorry, that's wrong :( ... the rate of the walking person is wrong in your answer, too." } { "Tag": [ "limit", "function", "algebra unsolved", "algebra" ], "Problem": "Find all $ f: R_{\\plus{}}\\rightarrow R_{\\plus{}}$ that satisfy two conditions:\r\nI)$ \\lim_{x\\to \\infty}f(x)\\equal{}0$\r\nII)$ \\forall_{x,y, \\in R_{\\plus{}}} f(xf(y))\\equal{}yf(x)$\r\nUnfortunately we don't have the continuity.\r\nWell, I proved few things:\r\n$ f(1)\\equal{}1$ \r\n$ f(x)\\equal{}f^{\\minus{}1}(x)$\r\nf is an injection\r\n$ \\lim_{x\\to 0}f(x)\\equal{}\\infty$\r\nUnfortunately I couldn't think of anything more :blush: \r\nThe quite obvious function would be $ f(x)\\equal{}\\frac{1}{x}$, but are there any more? If the answer is positive would the continuity be enough to prove $ f(x)\\equal{}\\frac{1}{x}$ is the only function? \r\nMany questions, not many answers... :wink:", "Solution_1": "[quote=\"polskimisiek\"]Find all $ f: R_{ \\plus{} }\\rightarrow R_{ \\plus{} }$ that satisfy two conditions:\nI)$ \\lim_{x\\to \\infty}f(x) \\equal{} 0$\nII)$ \\forall_{x,y, \\in R_{ \\plus{} }} f(xf(y)) \\equal{} yf(x)$\nUnfortunately we don't have the continuity.\nWell, I proved few things:\n$ f(1) \\equal{} 1$ \n$ f(x) \\equal{} f^{ \\minus{} 1}(x)$\nf is an injection\n$ \\lim_{x\\to 0}f(x) \\equal{} \\infty$\nUnfortunately I couldn't think of anything more :blush: \nThe quite obvious function would be $ f(x) \\equal{} \\frac {1}{x}$, but are there any more? If the answer is positive would the continuity be enough to prove $ f(x) \\equal{} \\frac {1}{x}$ is the only function? \nMany questions, not many answers... :wink:[/quote]\r\n\r\nLet $ P(x,y)$ be the property $ f(xf(y)) \\equal{} yf(x)$\r\n\r\n$ P(1,x)$ implies $ f(f(x))\\equal{}f(1)x$ and so $ f(x)$ is bijective (since $ f(1)\\neq 0$)\r\nThen $ P(1,1)$ gives $ f(f(1))\\equal{}f(1)$ and so $ f(1)\\equal{}1$ since $ f(x)$ is injective. As a consequence, we have $ ff(x))\\equal{}x$\r\nThen, $ P(x,f(y))$ implies $ f(xy)\\equal{}f(x)f(y)$ and $ f(x)$ is multiplicative.\r\nThen, Let $ x>1$ : $ f(x^n)\\equal{}f(x)^n$ and so $ f(x)<1$, else $ \\lim_{x\\to \\plus{}\\infty}f(x) \\equal{} 0$ would be false (take $ n\\rightarrow \\plus{}\\infty$)\r\nSo $ x>y$ implies $ \\frac{x}{y}>1$ and so $ f(\\frac{x}{y})<1$ and so $ f(y)\\equal{}f(x\\frac{y}{x})\\equal{}f(x)f(\\frac{x}{y}) 1$. But this contradicts that every fixed point $ t$ must satisfy $ t < 1$. So only fixed point of $ f$ is $ 1$. Plugging $ x\\equal{}y$ to the initial equation gives $ f(x)\\equal{}\\frac {1}{x}$.", "Solution_3": "Thank you very much :wink: Nice proof, well done :wink: That problem was not done by a doctor of mathematics, but it seems not so terrible after all :wink:" } { "Tag": [], "Problem": "http://www.mathlinks.ro/Forum/topic-66332-20.html\r\nbai 6 nam 1993 cua trung quoc", "Solution_1": "sao ko ai giup em vay huhuhuhu" } { "Tag": [ "quadratics", "algebra" ], "Problem": "Can anyone solve this question??\r\n\r\n(x+2)(x+3)(x+5)(x+6)=10", "Solution_1": "[hide=\"Solution\"]$ (x\\plus{}2)(x\\plus{}6) \\equal{} (x\\plus{}4\\plus{}2)(x\\plus{}4\\minus{}2) \\equal{} (x\\plus{}4)^2\\minus{}4$\n$ (x\\plus{}3)(x\\plus{}5) \\equal{} (x\\plus{}4\\plus{}1)(x\\plus{}4\\minus{}1) \\equal{} (x\\plus{}4)^2 \\minus{} 1$\n\nLet $ y \\equal{} (x\\plus{}4)^2$. Then:\n\n$ (y\\minus{}4)(y\\minus{}1) \\equal{} 10$. This is a quadratic equation, and we can solve it (or inspect) to determine that we must have $ y\\equal{}6$ or $ y\\equal{}\\minus{}2$.\n\nIn the former case, $ (x\\plus{}4)^2 \\equal{} 6 \\implies x\\plus{}4 \\equal{} \\pm \\sqrt{6} \\implies x \\equal{} \\minus{}4 \\pm \\sqrt{6}$\nIn the latter case, $ (x\\plus{}4)^2 \\equal{} \\minus{}3 \\implies x\\plus{}4 \\equal{} \\pm i \\sqrt{2} \\implies x \\equal{} \\minus{} 4 \\pm i \\sqrt{2}$\n\nThus, those are the four solutions to your first equation.[/hide]", "Solution_2": "There is a mistake in the second pair of solutions. It should be\r\n\r\n[hide] $ y \\equal{} \\minus{} 1$, so the solutions are $ x \\equal{} \\minus{} 4 \\pm \\sqrt {6}$ and $ x \\equal{} \\pm i \\minus{} 4$ [/hide]", "Solution_3": "This type of problem is pretty popular in math competitions. Here is a slightly different method\r\n\r\n[hide=\"Method 2\"]\n\n\"Pair up\" the equations as $ (x\\plus{}2)(x\\plus{}6)$ and $ (x\\plus{}3)(x\\plus{}5)$. We like to do this because...\n\n$ (x^2\\plus{}8x\\plus{}12)$ and $ (x^2\\plus{}8x\\plus{}15)$ is what we get.\n\nLet $ x^2 \\plus{} 8x \\plus{} 12 \\equal{} y$ then:\n\n$ y(y\\plus{}3) \\equal{} 10$\n\nFrom here, it's relatively simple quadratic. [/hide]", "Solution_4": "[quote=\"Silverfalcon\"]This type of problem is pretty popular in math competitions. Here is a slightly different method\n\n[hide=\"Method 2\"]\n\n\"Pair up\" the equations as $ (x \\plus{} 2)(x \\plus{} 6)$ and $ (x \\plus{} 3)(x \\plus{} 5)$. We like to do this because...\n\n$ (x^2 \\plus{} 8x \\plus{} 12)$ and $ (x^2 \\plus{} 8x \\plus{} 15)$ is what we get.\n\nLet $ x^2 \\plus{} 8x \\plus{} 12 \\equal{} y$ then:\n\n$ y(y \\plus{} 3) \\equal{} 10$\n\nFrom here, it's relatively simple quadratic. [/hide][/quote]\r\n\r\ncan you complete it all the way. I still get only 2 answers. Thanks", "Solution_5": "[hide=\"I propose this way\"]\n1st lets make substitution $ k\\equal{}x\\plus{}4$\nthen:\n$ (k\\minus{}2)(k\\minus{}1)(k\\plus{}1)(k\\plus{}2)\\equal{}10$\n\n$ (k^2\\minus{}4)(k^2\\minus{}1)\\equal{}10$\n\n$ k^4\\minus{}5k^2\\minus{}6\\equal{}0$\n\nagain lets substitute: $ k^2\\equal{}t$\n\n$ t^2\\minus{}5t\\minus{}6\\equal{}0$\n\nand when we solve t, we substitute until we get x, so\n\n$ x\\equal{}\\minus{}4\\pm\\sqrt{6}$ and $ x\\equal{}\\minus{}4\\pm i$[/hide]", "Solution_6": "[quote=\"tony77\"][quote=\"Silverfalcon\"]This type of problem is pretty popular in math competitions. Here is a slightly different method\n\n[hide=\"Method 2\"]\n\n\"Pair up\" the equations as $ (x \\plus{} 2)(x \\plus{} 6)$ and $ (x \\plus{} 3)(x \\plus{} 5)$. We like to do this because...\n\n$ (x^2 \\plus{} 8x \\plus{} 12)$ and $ (x^2 \\plus{} 8x \\plus{} 15)$ is what we get.\n\nLet $ x^2 \\plus{} 8x \\plus{} 12 \\equal{} y$ then:\n\n$ y(y \\plus{} 3) \\equal{} 10$\n\nFrom here, it's relatively simple quadratic. [/hide][/quote]\n\ncan you complete it all the way. I still get only 2 answers. Thanks[/quote]\n\n[hide=\"Rest\"]\n\n$ y^2\\plus{}3y\\minus{}10 \\equal{} (y\\plus{}5)(y\\minus{}2) \\equal{} 0$\n\nSo, $ y \\equal{} \\minus{}5, 2$. Now, substituting it back:\n\n$ x^2\\plus{}8x\\plus{}12 \\equal{} \\minus{}5 \\rightarrow x^2\\plus{}8x\\plus{}17 \\equal{} 0$ \n$ x^2\\plus{}8x\\plus{}12 \\equal{} 2 \\rightarrow x^2\\plus{}8x\\plus{}10 \\equal{} 0$\n\nFrom here, it's just quadratic formula. [/hide]" } { "Tag": [ "inequalities" ], "Problem": "Let $ a,b,c$ be a real numbers such that $ 0 3/4$, that is, $ 1 \\minus{} abc \\minus{} (1 \\minus{} a)(1 \\minus{} b)(1 \\minus{} c) > 3/4$. Rearranging, $ abc \\plus{} (1 \\minus{} a)(1 \\minus{} b)(1 \\minus{} c) < 1/4$. \r\n\r\nI'm a total failure at inequalities, so I'll let someone else find a contradiction.", "Solution_2": "[hide=\"solution\"]We have $ \\left(\\frac {1}{2} \\minus{} a\\right)^2\\ge 0, \\left(\\frac {1}{2} \\minus{} b\\right)^2\\ge 0,$ and $ \\left(\\frac {1}{2} \\minus{} c\\right)^2\\ge 0$.\n\nWe expand each of these: $ a^2 \\minus{} a\\ge \\minus{} \\frac {1}{4}$ and symmetric.\n\nWe reverse the sign and inequality symbol: $ a(1 \\minus{} a)\\le \\frac {1}{4}$ and symmetric.\n\nWe then take the product of all these: $ abc(1 \\minus{} a)(1 \\minus{} b)(1 \\minus{} c)\\le \\left(\\frac {1}{4}\\right)^3$\n\nThis is equal to $ b(1 \\minus{} a)\\cdot c(1 \\minus{} b)\\cdot a(1 \\minus{} c)$. Now none of these multiplicands are negative, so there must be one that is less than $ \\frac {1}{4}$. (For if they were all greater than 1/4, then their product would be greater than 1/64.) $ \\boxed{\\text{QED}}$.[/hide]\r\n\r\nEdit: Thanks, Farenhajt.", "Solution_3": "Just a typo: you should have $ \\left({1\\over 4}\\right)^3$" } { "Tag": [ "function", "trigonometry", "limit", "calculus", "calculus computations" ], "Problem": "Determine all real numbers $ a$, for which the function $ \\tan(x) \\plus{} a\\tan(3x)$ has a finite limit at $ \\frac {\\pi}{2}$ and calculate the limit for those values of $ a$.", "Solution_1": "Now I got it.\r\n\r\nWe know that $ \\cos(3x)=\\cos^{3}(x)-3\\sin^{2}(x)\\cos(x)$ and $ \\sin(3x)=-\\sin^{3}(x)+3\\cos^{2}(x)\\sin(x)$.\r\n\r\nSo we have \r\n\r\n\\begin{eqnarray*}\r\n \\tan(x)-a\\tan(3x) &=& \\frac{\\sin(x)\\cos(3x)+a\\sin(3x)\\cos(x)}{\\cos(3x)\\cos(x)} = \\\\\r\n &=& \\frac{\\sin(x)\\cos^{3}(x)-3\\sin^{3}(x)\\cos(x)-a\\sin^{3}(x)\\cos(x)+a3\\cos^{3}(x)\\sin(x)}{\\cos^{2}(x)(\\cos^{2}(x)-3\\sin^{2}(x))}= \\\\\r\n &=& \\frac{(1+3a)\\sin(x)\\cos^{2}(x)-(3+a)\\sin^{3}(x)}{\\cos(x)(\\cos^{2}(x)-3\\sin^{2}(x))}\r\n\\end{eqnarray*}\r\n\r\nNow since the denominator goes toward $ 0$, also the numerator must go toward $ 0$ in order for the finite limit. Since $ (1+3a)\\sin(x)\\cos^{2}(x)$ in the numerator goes toward $ 0$, also $ (3+a)\\sin^{3}(x)$ must go toward $ 0$. Hence $ a$ must equal $ -3$.\r\n\r\n$ a=-3$: \r\n\r\n$ \\lim_{x \\to \\frac{\\pi}{2}} \\frac{-8\\sin(x)\\cos^{2}(x)}{\\cos(x)(\\cos^{2}(x)-3\\sin^{2}(x))}=\\lim_{x \\to \\frac{\\pi}{2}} \\sin(x) \\lim_{x \\to \\frac{\\pi}{2}} \\frac{-8\\cos(x)}{\\cos^{2}(x)-3\\sin^{2}(x)}= 0$" } { "Tag": [ "LaTeX" ], "Problem": "How do you make the conjugate line longer over longer expressions like a-bi? When I use the bar command I get a shorter line:\r\n$\\bar{a-bi}$", "Solution_1": "Use \\overline: $\\overline{a+bi}=a-bi$." } { "Tag": [ "number theory solved", "number theory" ], "Problem": "We say that a natural number n is [i]nice[/i] if there exist integers a1, a2, . . . , an such that \r\na1 + a2 + ... + an = a1a2...an = n.\r\nFind all nice numbers.", "Solution_1": "Is it allowed to use negative integers a_i ?\r\n\r\nPierre.", "Solution_2": "[quote=\"pbornsztein\"]Is it allowed to use negative integers a_i ?\n\nPierre.[/quote]\r\n\r\nI think the answer is yes, because if a_i > 0 then only n=1 have this property.\r\n\r\nI didnt solve this prb, but I found that if n=4k+1 then n is nice.\r\n\r\nFor example 5 = 5.1.1.(-1).(-1) = 5 + 1 + 1 + (-1) + (-1).\r\n\r\nAlso, we can see that if n is prime then n is nice <=> n = 4k+1.\r\n\r\nTry to continue, friends ..." } { "Tag": [], "Problem": "Suppose we have a number in base 3 between 0 and 1. Suppose this number has the form\r\n$ 0.a_{1}a_{2}\\cdots022\\cdots\\equal{}0.a_{1}a_{2}\\cdots100\\cdots$\r\nNaturally, these two numbers are the same. I need to prove that such a number has two representations in binary. Any help? Thank you.", "Solution_1": "I hope you don't need to prove this, because it's not true: $ \\frac {1}{3} \\equal{} 0.1_3 \\equal{} 0.0222\\ldots_3$, but $ \\frac {1}{3} \\equal{} 0.01010101\\ldots_2$ has a unique binary representation. I think you are misinterpreting something in the problem that this came from.", "Solution_2": "Okay, that is what I want to hear. In my problem, I am using the Cantor set and the Cantor function. Obviously, $ f(1/3)\\equal{}f(2/3)$. The problem asks to prove that if $ x5 since 5 is the third prime. Also let the common difference be d with d = 2, 4, or 6. Then, since p, p+d, p+2d, p+3d, p+4d are all different modulo 5, one of them is divisible by 5. Since p>5, we have a composite number among these 5 terms. contradiction[/hide]", "Solution_2": "zabelman, I don't think the problem precludes having composite numbers in the list. It's just that it is necessary for there to be 1999 primes. But[hide]there aren't 1999 positive primes under 12345.[/hide]", "Solution_3": "I'm not entirely sure what the question is asking. Although I'd be inclined to agree that complexzeta's objection is the stronger of the two." } { "Tag": [ "calculus", "integration", "trigonometry", "logarithms", "parameterization", "function", "limit" ], "Problem": "show\r\n\r\n$\\int_{0}^{\\infty}\\;\\frac{2\\;-\\;2\\cos{x}}{x\\;e^{x}}\\;dx\\;=\\;\\ln{2}$", "Solution_1": "$\\frac{1-\\cos x}{x}=\\int_{0}^{1}\\sin(xt)\\,dt$.\r\n\r\nThe integral becomes $2\\int_{0}^\\infty\\int_{0}^{1}\\sin(xt)e^{-x}\\,dx\\,dt$.\r\nApply Fubini and switch the order of integration. $\\int_{0}^{\\infty}\\sin(tx)e^{-x}\\,dx=\\text{Im}\\int_{0}^{\\infty}e^{-(1-it)x}\\,dx =\\text{Im}\\frac1{1-it}=\\frac{t}{1+t^{2}}$\r\n\r\nNow $\\int_{0}^{1}\\frac{2t}{1+t^{2}}\\,dt=\\left.\\ln(1+t^{2})\\right|_{0}^{1}=\\ln 2$.", "Solution_2": "you dawg! :thumbup:", "Solution_3": "Introduce a parameter $ t > 0$:\r\n\\[ I(t) = \\int_{0}^\\infty \\frac{2-2 \\cos x}{x}\\cdot e^{-tx}\\, dx . \\]\r\nWe have that\r\n\\begin{eqnarray*}I^\\prime (t) &=&-\\int_{0}^\\infty \\left( 2-2 \\cos x \\right) e^{-tx}\\, dx \\\\ \\ &=&-2 \\cdot \\text{Re}\\int_{0}^\\infty \\left( e^{-tx}-e^{-(t-i)x}\\right) \\, dx \\\\ \\ &=&-2 \\cdot \\frac1{t}+\\frac{2t}{t^{2}+1}.\\end{eqnarray*}\r\nHence, $ I(t) = \\log \\left( 1+\\frac1{t^{2}}\\right)+\\mathcal C$, where $ \\mathcal C$ is some constant.\r\nNow notice that $ x \\mapsto \\frac{2-2 \\cos x}{x}$ is a continuous, bounded function on $ \\left[ 0, \\infty \\right)$ and that $ \\lim_{t \\to \\infty}\\int_{0}^\\infty e^{-tx}\\, dx = 0$. Hence, $ \\lim_{t \\to \\infty}I(t) = 0$. From this we infer that $ \\mathcal C = 0$, so $ I(1) = \\log 2$.\r\n\r\nPS: As always, I don't know how to justify the differentiation under the integral sign.", "Solution_4": "[quote=\"perfect_radio\"]As always, I don't know how to justify the differentiation under the integral sign.[/quote]\r\nThat's one of the big reasons why I like writing this sort of thing in terms of iterated integrals- it's much easier to see when you can apply a Fubini-type theorem.\r\n\r\nWith that idea, we write $ I(t)=\\int_{0}^{\\infty}\\int_{0}^{t}(2-2\\cos x)e^{-sx}\\,ds\\,dx$\r\nThe interchange is easy to justify, and leads to your computation.", "Solution_5": "[quote=\"perfect_radio\"]PS: As always, I don't know how to justify the differentiation under the integral sign.[/quote]\r\n\r\ni personally think leibniz's integral rule i.e. \"differentiation under the integral sign\" kicks \r\nsome major butt !! :coolspeak: :yup:" } { "Tag": [ "conics", "parabola", "analytic geometry", "graphing lines", "slope", "algebra unsolved", "algebra" ], "Problem": "Find value $ m \\in R$ for the equation later existence of root $ x \\in R$\r\n\r\n$ \\left( {\\sqrt {1 \\plus{} x^2 } \\plus{} \\sqrt {1 \\minus{} x^2 } \\plus{} 1} \\right).m \\equal{} 2\\sqrt {1 \\minus{} x^4 } \\plus{} \\sqrt {1 \\plus{} x^2 } \\plus{} \\sqrt {1 \\minus{} x^2 } \\plus{} 3$", "Solution_1": "Answer: $ 2\\sqrt {2} \\minus{} 1\\leq m\\leq \\frac {7}{3}$. \r\n\r\nLet $ t \\equal{} \\sqrt {1 \\plus{} x^2} \\plus{} \\sqrt {1 \\minus{} x^2}\\Longrightarrow 2\\sqrt {1 \\minus{} x^4} \\equal{} t^2 \\minus{} 2$, thus the given equation become $ m(t \\plus{} 1) \\equal{} t^2 \\plus{} t \\plus{} 1$ where $ 2\\leq t^2\\leq 4\\Longleftrightarrow \\sqrt {2}\\leq t\\leq 2$. We are to find the condition that the parabola $ y \\equal{} t^2 \\plus{} t \\plus{} 1\\ (\\sqrt {2}\\leq t\\leq 2)$ has one more intersection points with the line $ y \\equal{} m(t \\plus{} 1)$ which passes through the point $ ( \\minus{} 1,\\ 0)$ regardless any slope $ m$. Let $ f(t) \\equal{} t^2 \\plus{} t \\plus{} 1$, we have $ f(\\sqrt {2})\\leq m\\leq f(2)$, yielding the answer $ 2\\sqrt {2} \\minus{} 1\\leq m\\leq \\frac {7}{3}$.", "Solution_2": "[quote=\"kunny\"]Answer: $ 2\\sqrt {2} \\minus{} 1\\leq m\\leq \\frac {7}{3}$. \n\nLet $ t \\equal{} \\sqrt {1 \\plus{} x^2} \\plus{} \\sqrt {1 \\minus{} x^2}\\Longrightarrow 2\\sqrt {1 \\minus{} x^4} \\equal{} t^2 \\minus{} 2$, thus the given equation become $ m(t \\plus{} 1) \\equal{} t^2 \\plus{} t \\plus{} 1$ where $ 2\\leq t^2\\leq 4\\Longleftrightarrow \\sqrt {2}\\leq t\\leq 2$. We are to find the condition that the parabola $ y \\equal{} t^2 \\plus{} t \\plus{} 1\\ (\\sqrt {2}\\leq t\\leq 2)$ has one more intersection points with the line $ y \\equal{} m(t \\plus{} 1)$ which passes through the point $ ( \\minus{} 1,\\ 0)$ regardless any slope $ m$. Let $ f(t) \\equal{} t^2 \\plus{} t \\plus{} 1$, we have $ f(\\sqrt {2})\\leq m\\leq f(2)$, yielding the answer $ 2\\sqrt {2} \\minus{} 1\\leq m\\leq \\frac {7}{3}$.[/quote]\r\n\r\nThank kunny very much. It's nice. :P" } { "Tag": [ "geometry", "circumcircle", "geometry proposed" ], "Problem": "O, A, B, C, D are five distinct points on a circle.\r\n\r\n$L_1$, $L_2$, $L_3$, $L_4$, are the Simson lines of O with respect to the triangles BCD, CDA, DAB, ABC respectively.\r\n\r\nProve that OD is a diameter of the circumcircle of the triangle formed by the lines $L_1$, $L_2$, $L_3$.\r\n\r\n\r\nProve also that the feet of the perpendiculars from O to $L_1$, $L_2$, $L_3$, $L_4$ are collinear.", "Solution_1": "Let $N_{12}, N_{13}, N_{14}, N_{23}, N_{24}, N_{34}$ be the feet of normals from the point $O$ to the lines $AB, AC, AD, BC, BD, CD$, respectively.\r\n\r\nThe Simson lines $l_2, l_3$ of the triangles $\\triangle CDA, \\triangle DAB$ meet at the point $N_{14} \\equiv l_2 \\cap l_3$ on their common side $DA$. Since the angle $\\measuredangle ON_{14}D = 90^o$ is right, the locus of $N_{14}$ is a circle with diameter $OD$.\r\n\r\nThe Simson lines $l_3, l_1$ of the triangles $\\triangle DAB, \\triangle CBD$ meet at the point $N_{24} \\equiv l_3 \\cap l_1$ on their common side $BD$. Since the angle $\\measuredangle ON_{24}D = 90^o$ is right, the locus of $N_{24}$ is a circle with diameter $OD$.\r\n\r\nThe Simson lines $l_1, l_2$ of the triangles $\\triangle BCD, \\triangle CDA$ meet at the point $N_{34} \\equiv l_1 \\cap l_2$ on their common side $CD$. Since the angle $\\measuredangle ON_{34}D = 90^o$ is right, the locus of $N_{34}$ is a circle with diameter $OD$.\r\n\r\nThus the vertices of the triangle $\\triangle N_{14}N_{24}N_{34}$ formed by the Simson lines $l_1, l_2, l_3$ all lie on the circle with diameter $OD$, the circumcircle of this triangle.\r\n\r\nIn an entirely similar way, it can be shown that the circumcircle of the triangle $\\triangle N_{13}N_{23}N_{34}$ formed by the Simson lines $l_1, l_2, l_4$ is a circle with diameter $OC$, the circumcircle of the triangle $\\triangle N_{12}N_{23}N_{24}$ formed by the Simson lines $l_1, l_3, l_4$ is a circle with diameter $OB$, and the circumcircle of the triangle $\\triangle N_{12}N_{13}N_{14}$ formed by the Simson lines $l_2, l_3, l_4$ is a circle with diameter $OA$.\r\n\r\nLet $M_1, M_2, M_3, M_4$ be the feet of normals from the point $O$ to the Simson lines $l_1, l_2, l_3, l_4$. Since the point $O$ is on the circumcircle with diameter $OD$ of the $\\triangle N_{14}N_{24}N_{34}$, $M_1M_2M_3$ is a Simson line of this triangle. Since the point $O$ is on the circumcircle with diameter $OC$ of the triangle $\\triangle N_{13}N_{23}N_{34}$, $M_1M_2M_4$ is a Simson line of this triangle. Since the point $O$ is on the circumcircle with diameter $OB$ of the triangle $\\triangle N_{12}N_{23}N_{24}$, $M_1M_3M_4$ is a Simson line of this triangle. Finally, since the point $O$ is on the circumcircle with diameter $OA$ of the $\\triangle N_{12}N_{13}N_{14}$, $M_2M_3M_4$ is a Simson line of this triangle. As a result, the point $M_1, M_2, M_3, M_4$ are all collinear.\r\n\r\nThis is one of the possible proofs of the following theorem:\r\n\r\nThe 4 circumcircles of 4 triangles formed by any 4 lines of general placement are concurrent." } { "Tag": [ "videos" ], "Problem": "is anyone else addicted to this game? :lol:", "Solution_1": "[hide=\"I was\"]back in 1995 when I saw Win95 for the first time :D[/hide]", "Solution_2": "Mine swipper rox.", "Solution_3": "I like minesweeper but almost always, especially in expert you have to guess (at least that is what it seems to me) and so its hard to beat it. Its doesnt seem to be too mathematically challenging.", "Solution_4": "Depends how good you are at guessing then..\r\nReally -theres the basic cases where it looks like you have to guess but can actually logically work out which one must be a mine etc from other squares, but even if there is a guess involved usually one of the squares can be figured out to be a lot better to guess than the other.", "Solution_5": "you gan get $1000000 if you crack the code", "Solution_6": "[quote=\"Valentin Vornicu\"][hide=\"I was\"]back in 1995 when I saw Win95 for the first time :D[/hide][/quote]\r\n\r\ni thought 95 still had chip's challenge on it? now THAT was an addicting game.", "Solution_7": "[quote=\"hello\"]I like minesweeper but almost always, especially in expert you have to guess (at least that is what it seems to me) and so its hard to beat it. Its doesnt seem to be too mathematically challenging.[/quote]\r\n\r\nAs the $1M comment suggests, it is NP-complete (in some deterministic version of the problem that can be formulated as a Yes/No decision problem).", "Solution_8": "Yes! I am sooo addicted to that game. I play it way to much its just addicting I guess.", "Solution_9": "Wow... I was so addicted back when I was 6 or 7 - my parents yelled at me cuz I was hogging the computer so much.", "Solution_10": "Look below to see how good I am at Minesweeper :D", "Solution_11": "so im not the only one :P just out of curiosity, what are your best times?", "Solution_12": "Well done h_s_potter2002!\r\n\r\nI never played that game, but the MathChauffeuress had a 95 and a bundle of 97-99s at the expert level.\r\n\r\nBack then I did not know whether to sue the authors for taking so much of her from me, or to write them a thank you note for keeping her away from interfering with my own addictions :D", "Solution_13": "i finished in 999 seconds but for somereason, it woulnd't let me put my high score up :(", "Solution_14": "[quote=\"jli\"]i finished in 999 seconds but for somereason, it woulnd't let me put my high score up :([/quote] According to \"reliable source\", high score is defined as 998 or less.", "Solution_15": "When I was an actuarial technician 8 or 9 years ago, I would play for 10 to 20 minutes every day -- until I won once going at a fairly fast pace. I can't recall for sure, but my best time was somewhere in the high double digits, but that involved hitting around 10 or 12 random squares to start each game.\r\n\r\nAt some point I got very bored of it as I eventually have all video games.", "Solution_16": "I was about a year ago. My best score was 126 :(.", "Solution_17": "on the mindsweeper that came w/ windows 98 and earlier, the minesweeper clock could be stopped by clicking both mouse buttons and escape all at the same time. i used that trick and beat expert in 2 seconds. too bad it doesn't work on the newer windows. it's kinda impressive to show ppl that. anywho, i also don't like the fact that you do have to usually guess on expert - that takes the fun out of it.", "Solution_18": "[quote=\"furious\"]on the mindsweeper that came w/ windows 98 and earlier, the minesweeper clock could be stopped by clicking both mouse buttons and escape all at the same time. i used that trick and beat expert in 2 seconds. too bad it doesn't work on the newer windows. it's kinda impressive to show ppl that. anywho, i also don't like the fact that you do have to usually guess on expert - that takes the fun out of it.[/quote]It's good that you showed people 2 seconds. If you showed them 67, they might believe the score was real and would be scared away from you till eternity. :D \r\n\r\nWe found the 67 score posted on the net by somebody a few years back. Judging the way the story was told, we believed the score was real. We wished we had kept the story and his name, just to see what he\u2019s up to these days. Math may be too easy for him :)", "Solution_19": "102 and proud of it", "Solution_20": "[quote=\"MathChauffeur\"][quote=\"furious\"]on the mindsweeper that came w/ windows 98 and earlier, the minesweeper clock could be stopped by clicking both mouse buttons and escape all at the same time. i used that trick and beat expert in 2 seconds. too bad it doesn't work on the newer windows. it's kinda impressive to show ppl that. anywho, i also don't like the fact that you do have to usually guess on expert - that takes the fun out of it.[/quote]It's good that you showed people 2 seconds. If you showed them 67, they might believe the score was real and would be scared away from you till eternity. :D \n\nWe found the 67 score posted on the net by somebody a few years back. Judging the way the story was told, we believed the score was real. We wished we had kept the story and his name, just to see what he\u2019s up to these days. Math may be too easy for him :)[/quote]\r\n\r\nthe current highscores are somewhere in the low 40s iirc although i did hear of someone breaking the 40 barrier for expert (may have been a fake)\r\n\r\nmy personal best is 100, which i got several days ago :)", "Solution_21": "[quote=\"alan\"]the current highscores are somewhere in the low 40s...[/quote] WOW! It would be something to see those minds and fingers in action. I don't even know how to imagine somebody doing it that fast :D", "Solution_22": "You should see the masters of tetris and speed rubix cubing. Gods of players.", "Solution_23": "um.... whats minesweeper...\r\n\r\ni've never heard of it, never played it, never got addicted to it, etc, etc\r\n\r\nthe only thing im addicted to now is spamming :D (j/k) and neopets", "Solution_24": "it is impossible to get a mine on your first try", "Solution_25": "[quote=\"sunnyray\"]um.... whats minesweeper...\n\ni've never heard of it, never played it, never got addicted to it, etc, etc\n\nthe only thing im addicted to now is spamming :D (j/k) and neopets[/quote]\r\n\r\ngo to start and games and wala! there your go. Minesweeper is there.\r\n\r\nBTW, i played neotpets for a while", "Solution_26": "[quote=\"MathChauffeur\"][quote=\"jli\"]i finished in 999 seconds but for somereason, it woulnd't let me put my high score up :([/quote] According to \"reliable source\", high score is defined as 998 or less.[/quote]\r\n\r\nohh... :blush: :blush:", "Solution_27": "[quote=\"alan\"]is anyone else addicted to this game? :lol:[/quote]\r\n\r\nWell, I like to play once in a while, and is a good time killer, but it is not like an addiction, I really don't play it that much.\r\n\r\nThe games that I find addictive are those that include some \"strategy\" issue, for example, FREECIV, I have spent 26 hours in a row playing it, that is why I forbided to my self to play it ever if I am not in vacations (or spring break or something like that).\r\n\r\nBest regards,", "Solution_28": "i was an addicted to...\r\ni was quite good in the intermediate, with a best time of 31 seconds... and quite good also at the expert: 93 seconds (if i recall correctly)...\r\nbut it's not a silly game... it's just recognizing some recurring paths, and be able to see much more than it's shown...\r\n ;)", "Solution_29": "I like minesweeper but i'm bad it it... :blush: \r\nI don't think i've ever beat expert but i don't try to often. ;)", "Solution_30": "I love it but not very good at it..........\r\nMy best score is only 150.......... :(", "Solution_31": "[quote=\"jli\"]it is impossible to get a mine on your first try[/quote]\r\n\r\nWhy is that?", "Solution_32": "[quote=\"RC-7th\"][quote=\"jli\"]it is impossible to get a mine on your first try[/quote]\n\nWhy is that?[/quote]\r\n\r\nthats how the program works. i think they want to give you a fighting chance :)", "Solution_33": "Can anyone say ........... 4 in 5 years?", "Solution_34": "[quote=\"alan\"][quote=\"RC-7th\"][quote=\"jli\"]it is impossible to get a mine on your first try[/quote]\n\nWhy is that?[/quote]\n\nthats how the program works. i think they want to give you a fighting chance :)[/quote]\r\n\r\nWell i always fail at the second......" } { "Tag": [ "modular arithmetic" ], "Problem": "For $ n>\\equal{}1$, use congruence to establish \r\n\r\n1.) $ 27|(2^{5n\\plus{}1} \\plus{} 5^{n\\plus{}2})$\r\n\r\nFor $ n>\\equal{}0$, \r\n2.) $ 13| (4^{2n \\plus{} 1} \\plus{} 3^{n\\plus{}2})$\r\n3.) $ 7|(2^{n\\plus{}2} \\plus{} 3^{2n \\plus{}1})$", "Solution_1": "These are all essentially the same problem :P \r\n\r\n[hide=\"Solution for 1\"]$ 2^{5n\\plus{}1} \\equal{} 2 \\cdot (2^5)^n \\equiv 2 \\cdot 5^n \\pmod{27}$\n$ 5^{n\\plus{}2} \\equal{} 25 \\cdot 5^n$\n$ \\therefore 2^{5n\\plus{}1} \\plus{} 5^{n\\plus{}2} \\equiv 27 \\cdot 5^n \\equiv 0 \\pmod{27}$[/hide]\n[hide=\"Solution for 2\"]$ 4^{2n\\plus{}1} \\equal{} 4 \\cdot (4^2)^n \\equiv 4 \\cdot 3^n \\pmod{13}$\n$ 3^{n\\plus{}2} \\equal{} 9 \\cdot 3^n$\n$ \\therefore3^{n\\plus{}2} \\plus{} 4^{2n\\plus{}1} \\equiv 13 \\cdot 3^n \\equiv 0 \\pmod{13}$[/hide]\n[hide=\"Solution for 3\"]$ 3^{2n\\plus{}1} \\equal{} 3 \\cdot (3^2)^n \\equiv 3 \\cdot 2^n \\pmod{7}$\n$ 2^{n\\plus{}2} \\equal{} 4 \\cdot 2^n$\n$ \\therefore 2^{n\\plus{}2} \\plus{} 3^{2n\\plus{}1} \\equiv 7 \\cdot 2^n \\equiv 0 \\pmod{7}$[/hide]" } { "Tag": [ "geometry", "3D geometry", "icosahedron", "USAMTS", "pyramid", "trigonometry", "AMC" ], "Problem": "I'm wonding what scores you think you got in round 2.\r\nI think i had 21, 52554", "Solution_1": "I'm guessing, probably on the better side though.\r\n\r\n1. 3 or 4\r\n2. 5\r\n3. 5\r\n4. 3\r\n5. 5\r\n\r\nTotal: 21-22", "Solution_2": "Hmm...\r\n\r\n4/5\r\n5\r\n5\r\n5\r\n1/2/3 (hope I get a nice grader!)\r\n\r\nAt worst, 20. At best, 23.", "Solution_3": "Hopefully, something like 52555\r\n\r\nI got all the answers right but basically guessed on problem 2. I'm confident that i nailed the other problems, even though i actually had to make an icosahedron to visualize number 5. I'll be happy with anything that beats the 20 I got on round 1.", "Solution_4": "either 55455 or 55555", "Solution_5": "I'm thinking 55545", "Solution_6": "I'm thinking 55555, but I'm not sure if my number 3 was rigorous enough...so possibly 55455. I'll be angry if I get anything below 24 though, because I'm in 10th grade, and I really want to use USAMTS as a safety net.\r\n\r\nDid anybody else write a program to brute-force test all 512 combinations of N=9?\r\n\r\nWhich problem did you guys think was hardest on round 2?", "Solution_7": "Without doubt #5 was the hardest because unless you already knew what an icosahedron looked like, you had to look it up in a book or the internet. However, you could also say #1 was the hardest to find an elegant solution to without tons of casework.", "Solution_8": "55555 or possibly 55545", "Solution_9": "I'm thinking 25000", "Solution_10": "53152=16", "Solution_11": "Hopefully 5 5 5 5 4, depends how nice my grader is for number 5. \r\n\r\nHm.. last year round 2 I had $ 0$ commends, round 3, $ 1$ commend, round 4, $ 2$ commends, this year round 1, $ 3$ commends. Therefore my commends go in an arithmetic progression, and I will have four commends this round, and round 4 this year I will be getting 6 commendeds. $ \\square$\r\n\r\n\r\nOh, and about which problem was the hardest, I go with #2. I finished #4 before the other problems somehow .. and #1 (just a couple of cases), 5 (pyramids) weren't too bad.", "Solution_12": "Compared to previous years, are this year's problems harder or easier?", "Solution_13": "I guess thats based upon opinion, here's my feeling for rounds 2:\r\n\r\n1: Both are a lot of casework, but I think this years was slightly easier.\r\n2: I found this years rather difficult, but last year's wasn't very hard\r\n3: Neither was very difficult, I suppose last year's was easier proof-wise\r\n4: Both were fairly brutal (another coloring problem w/ contradictions!). I think last year's was harder.\r\n5: Last year's geometry problem was pretty easy, so I go with this year's being harder.\r\n\r\nAdding them up, that makes this years harder.\r\n\r\nAs for round 1, I did terrible last yr's round 1, this yr I finished in two days, but that's not a very accurate representation of the difficulty of the problems.", "Solution_14": "Hmm, I made two typos on #5... wrote $ \\sin$ as $ \\cos$, and forgot to -not- type a square root. :huh: Perhaps 55554, if I'm not lucky...\r\n\r\n#1, I was like, dude! casework! and there were only 3 cases... WAIT #$ *$(#^@# I DID NOT SHOW FOR N<8... w/e... argh\r\n#2, Newton's Sums.\r\n#3, I was lucky that I found one where 120 works... I thought it was 120, and then I was like \"oh no I keep getting 240\"... but yay, 120 works.\r\n#4, uh? what?\r\n#5, made two typos.\r\n\r\nbleh.", "Solution_15": "Are problems randomized in rounds, or are they categorized per round? Like, for Round 3s of different years, are the problem types similar?", "Solution_16": "[quote=\"123456789\"]Are problems randomized in rounds, or are they categorized per round? Like, for Round 3s of different years, are the problem types similar?[/quote]\r\n\r\nI'm fairly certain that problem types being similar would just be coincidences. I imagine that the writers would try to balance the types of questions on each round, which would probably cause the coincidences.", "Solution_17": "Initially USAMTS was sorted by subject, but in recent years this hasn't been true (now it's more sorted in terms of difficulty).", "Solution_18": "[quote=\"diophantient\"]Initially USAMTS was sorted by subject, but in recent years this hasn't been true (now it's more sorted in terms of difficulty).[/quote]\r\n\r\nWhen you say \"difficulty\", do you mean that Round 1 is easier than Round 2, which is easier than Round 3? Or do you mean the problems on each round (#1 is easier than #2, is easier than #3, etc.)?", "Solution_19": "[quote=\"123456789\"][quote=\"diophantient\"]Initially USAMTS was sorted by subject, but in recent years this hasn't been true (now it's more sorted in terms of difficulty).[/quote]\n\nWhen you say \"difficulty\", do you mean that Round 1 is easier than Round 2, which is easier than Round 3? Or do you mean the problems on each round (#1 is easier than #2, is easier than #3, etc.)?[/quote]\r\nBoth, I think.", "Solution_20": "#1 wasnt that hard i thought. heres what i did: (see if you can decipher my diagram :) )\r\n\r\nATTACHED", "Solution_21": "That's essentially the same as the brute force argument.", "Solution_22": "Well it took all of five minutes, which is not, in my opinion, long enough to be awarded the title of \"brute forced\".", "Solution_23": "It is just listing out all of the cases in a shorthand way. Yes, it does not take as long as writing them out one at a time, but how long a solution takes to find does not determine whether or not a solution is brute force, the actual solution in comparison to what is percieved as an elegant and concise solution determines whether or not it is brute force.", "Solution_24": "Um, listing them all out is \"brute force.\" Definitely better brute force than programming though, but nevertheless brute force.\r\n\r\nEnough arguing.", "Solution_25": "I'm thinking 54421 = 16 at worst, and 55543 = 22 at best.\r\n\r\nThat means I'll need a 21-27 next round to make AIME :( .", "Solution_26": "I thought you said you had handwaved #3?", "Solution_27": "i think I did but i'm not sure, hence my predictions" } { "Tag": [ "inequalities" ], "Problem": "Let $ n$ be a natural number. Prove that there exists integers $ a_1, a_2, a_3,...,a_k > 1$, such that $ a_1\\plus{}a_2\\plus{}...a_k \\equal{} n(\\frac{1}{a_1}\\plus{} \\frac{1}{a_2}\\plus{}...\\plus{}\\frac{1}{a_k})$. \r\n\r\nMy attempt:\r\n\r\n[hide]By AM-HM,\n $ \\frac{a_1\\plus{}a_2\\plus{}...a_k}{k} \\geq \\frac{k}{\\frac{1}{a_1}\\plus{} \\frac{1}{a_2}\\plus{}...\\plus{}\\frac{1}{a_k}}$\n\nSince we're exists a set of numbers, there is the case when $ n\\equal{}k$, since n is just any natural number. So,\n\n$ \\frac{a_1\\plus{}a_2\\plus{}...a_k}{n} \\geq \\frac{n}{\\frac{1}{a_1}\\plus{} \\frac{1}{a_2}\\plus{}...\\plus{}\\frac{1}{a_k}}$\n\nI don't know what happens after this though..[/hide]", "Solution_1": "Inequalities tell you that things can't happen; generally, they don't say much about whether things [b]can[/b] happen. You should try a different approach entirely.\r\n\r\n[hide=\"Big hint\"] What happens when you try to pair the terms on the LHS with the terms on the RHS? [/hide]", "Solution_2": "What if $ n \\equal{} 1$? Then we have $ a_1 \\plus{} \\dots \\plus{} a_k \\equal{} \\frac 1{a_1} \\plus{} \\dots \\plus{} \\frac 1{a_k}$, but since $ a_j > 1$, $ a_j^2>1$ as well, so it follows that $ a_j > \\frac 1{a_j}$ for all $ j$, so equality can't be achieved. :maybe:", "Solution_3": "hmm, the solution from the book i got it from is stupid. It says \"since n is a composite number, it can be written as $ n\\equal{}a*b$, where a and b are integers greater than 1. Consider $ a*b(\\frac{1}{a}\\plus{}\\frac{1}{b})$. Notice how it's equal to $ (a\\plus{}b)$, which satisfies the conditions of the problem\"... I forgot to put that n is composite but I didn't know it'd be that crucial for the solution :blush:", "Solution_4": "[quote=\"modularmarc101\"]I forgot to put that n is composite but I didn't know it'd be that crucial for the solution :blush:[/quote]\r\nI was about to say... anyway, why is that a \"stupid\" solution?", "Solution_5": "Well it was the last problem of an Olimpic test and the solution is like 3 sentences when the problem looks so hard (at least for me)", "Solution_6": "OK, so the solution given in the book shows that just 2 nos. are enough for any $ n$.\r\nHow does one think of something like that? I was trying for a generic proof (any no. of nos.)\r\nThe solution given in the book certainly didn't strike me.\r\nHow does one avoid traps like that, and think of the simplest way possible? Is experience the key?\r\n\r\nEdit: Oh yeah, I didn't know that $ n$ was composite, but anyway, could someone answer my question with a general perspective (not for this particular prob.)", "Solution_7": "The problem itself is vague; do you mean \"there exists a $ k$\" or \"for any $ k$\"? I'm pretty sure it's \"there exists a $ k$\", seeing as the \"any\" statement is not true." } { "Tag": [ "function", "combinatorics theorems", "combinatorics" ], "Problem": "I want to find a closed expression for $ \\sum_{n \\equal{} 0}^\\infty B_n \\frac {x^n}{n!}$, with $ B_n$ being the $ n^{th}$ Bell number. How do I go about that?\r\n\r\nThanks in advance,\r\n\r\n\r\n\r\nLos", "Solution_1": "The following is a very good reference on generating functions:\r\n[url]http://www.math.upenn.edu/~wilf/DownldGF.html[/url]", "Solution_2": "Thanks! I found what I needed!\r\n\r\nLos", "Solution_3": "I already heard that generating function is the most powerful tools to solve combinatorics, is this right? Who can give me some example for this", "Solution_4": "See for example: http://www.artofproblemsolving.com/Forum/viewtopic.php?t=141592" } { "Tag": [ "linear algebra", "matrix", "quadratics" ], "Problem": "Find a matrix whose determinant, when set equal to zero, represents the equation of a circle passing through the points $ (\\minus{}3,1)$, $ (2,4)$, and $ (5,\\minus{}2)$.\r\n\r\n--------\r\n\r\nIf a determinant is a number, how could it represent the equation of a circle?", "Solution_1": "Some of the entries of the matrix will involve $ x$ and $ y$, so the determinant will be a quadratic expression, not a constant. As a hint, there is a 4x4 matrix that will work for any 3 given points.", "Solution_2": "[quote=\"Ravi B\"]Some of the entries of the matrix will involve $ x$ and $ y$, so the determinant will be a quadratic expression, not a constant. As a hint, there is a 4x4 matrix that will work for any 3 given points.[/quote]\r\n\r\nOh, I see how the determinant can yield a circle's equation now. However, how can you tell that it will be a 4x4 matrix, and what numbers should go where? Is it trial and error?\r\n\r\nThanks!", "Solution_3": "I've seen this kind of problem before, which is how I knew about the 4x4 matrix. As a further hint, here's the first row of the matrix:\r\n\\[ \\begin{bmatrix} x^2 \\plus{} y^2 & x & y & 1 \\\\ ? & ? & ? & ? \\\\ ? & ? & ? & ? \\\\ ? & ? & ? & ? \\end{bmatrix}\\]\r\nDo you see how to complete the matrix so that plugging in each of the given points for $ x$ and $ y$ makes the determinant zero? Remember that a matrix with two identical rows automatically has determinant zero.", "Solution_4": "There was a discussion of this a few weeks ago [url=http://www.artofproblemsolving.com/Forum/viewtopic.php?p=1582110#1582110]here[/url].", "Solution_5": "Ok, I got it now. Thanks a lot, everyone!" } { "Tag": [ "floor function", "limit", "function", "algebra", "calculus", "calculus computations" ], "Problem": "Is $ {Lim}_{n \\to \\infty} 2.9999...$( 9 repeated n Times)$ \\equal{}3$?", "Solution_1": "Yes. What gave you reason to doubt this?", "Solution_2": "$ 2.9^{\\cdot}\\equal{}2\\plus{}0.9\\plus{}0.09\\plus{}0.009\\plus{}\\cdots \\equal{}2\\plus{}\\frac{0.9}{1\\minus{}0.1}\\equal{}3$?", "Solution_3": "Then what is the answer to the following problem:\r\n\r\nfind $ [lim_{n \\to \\infty}2.999....]$ where [.] denotes the greatest integer (floor) function.", "Solution_4": "$ \\left\\lfloor \\lim_{n\\to\\infty}(2 \\plus{} .9 \\plus{} .09 \\plus{} \\cdots 9(.1)^n)\\right\\rfloor \\equal{} \\lfloor 3\\rfloor \\equal{} 3.$\r\n\r\n$ \\lim_{n\\to\\infty}\\lfloor 2 \\plus{} .9 \\plus{} .09 \\plus{} \\cdots 9(.1)^n\\rfloor \\equal{} \\lim_{n\\to\\infty}2 \\equal{} 2.$\r\n\r\nBeing able to say that $ \\lim f(x_n) \\equal{} f\\left(\\lim x_n\\right)$ lies very near the heart of what it means for a function $ f$ to be continuous. The floor function is not continuous.", "Solution_5": "[quote=\"Kent Merryfield\"]$ \\left\\lfloor \\lim_{n\\to\\infty}(2 \\plus{} .9 \\plus{} .09 \\plus{} \\cdots 9(.1)^n)\\right\\rfloor \\equal{} \\lfloor 3\\rfloor \\equal{} 3.$\n\n$ \\lim_{n\\to\\infty}\\lfloor 2 \\plus{} .9 \\plus{} .09 \\plus{} \\cdots 9(.1)^n\\rfloor \\equal{} \\lim_{n\\to\\infty}2 \\equal{} 2.$\n\nBeing able to say that $ \\lim f(x_n) \\equal{} f\\left(\\lim x_n\\right)$ lies very near the heart of what it means for a function $ f$ to be continuous. The floor function is not continuous.[/quote]\r\n\r\nI understand your argument. but my problem was visualising the floor.\r\nIf $ \\alpha$ denotes $ \\lim_{n\\to\\infty}(2 \\plus{} .9 \\plus{} ...)$, isn't it true that $ \\alpha \\to 3$ but never $ \\equal{} 3$ and so $ \\alpha$ is $ < 3$?", "Solution_6": "So now we have established the issue: you fundamentally do not understand the meaning of \"limit.\"\r\n\r\nWhen we say that $ \\lim_{n\\to\\infty}x_n=L,$ we are not saying that any $ x_n$ must equal $ L.$ We are saying that we can [i]eventually[/i] make $ x_n$ [i]arbitrarily close[/i] to $ L.$\r\n\r\n(The words \"eventually\" and \"arbitrarily close\" do have precise mathematical definitions.)\r\n\r\nEven though $ \\frac1n>0$ for all $ \\n\\in\\mathbb{N},$ we still say that $ \\lim_{n\\to\\infty}\\frac1n=0.$ We say that because, by making $ n$ large enough, we can makes $ \\frac1n$ as close to zero as we please.\r\n\r\nYou said:\r\n\r\n[quote]If $ \\alpha$ denotes $ \\lim_{n\\to\\infty}(2 + .9 + ...)$, isn't it true that $ \\alpha\\to3\\dots$[/quote]\r\n\r\nTime out! You can't use that language. If we say that $ \\alpha$ equals the limit of something (assuming the limit exists), then $ \\alpha$ is just a number. One number, fixed and planted. You can't then talk about $ \\alpha$ tending to anything.\r\n\r\nMathematics depends on precise language, and what you've given us is not precise language. Even your expressions: $ \\lim_{n\\to\\infty}(2+.9+\\cdots)$ or $ \\lim_{n\\to\\infty}2.999\\dots$ - did you notice that I didn't quote those directly in my first response? That I felt compelled to create more precise notation just to make clear what we were talking about?\r\n\r\nImprecise language is not a source of profound paradoxes. It's merely imprecise language.", "Solution_7": "[quote=\"Kent Merryfield\"]So now we have established the issue: you fundamentally do not understand the meaning of \"limit.\"\n\nWhen we say that $ \\lim_{n\\to\\infty}x_n = L,$ we are not saying that any $ x_n$ must equal $ L.$ We are saying that we can [i]eventually[/i] make $ x_n$ [i]arbitrarily close[/i] to $ L.$\n\n(The words \"eventually\" and \"arbitrarily close\" do have precise mathematical definitions.)\n\nEven though $ \\frac1n > 0$ for all $ \\n\\in\\mathbb{N},$ we still say that $ \\lim_{n\\to\\infty}\\frac1n = 0.$ We say that because, by making $ n$ large enough, we can makes $ \\frac1n$ as close to zero as we please.\n\nYou said:\n\n[quote]If $ \\alpha$ denotes $ \\lim_{n\\to\\infty}(2 + .9 + ...)$, isn't it true that $ \\alpha\\to3\\dots$[/quote]\n\nTime out! You can't use that language. If we say that $ \\alpha$ equals the limit of something (assuming the limit exists), then $ \\alpha$ is just a number. One number, fixed and planted. You can't then talk about $ \\alpha$ tending to anything.\n\nMathematics depends on precise language, and what you've given us is not precise language. Even your expressions: $ \\lim_{n\\to\\infty}(2 + .9 + \\cdots)$ or $ \\lim_{n\\to\\infty}2.999\\dots$ - did you notice that I didn't quote those directly in my first response? That I felt compelled to create more precise notation just to make clear what we were talking about?\n\nImprecise language is not a source of profound paradoxes. It's merely imprecise language.[/quote]\r\n\r\nThanks for clearing my confusion. :oops: But perhaps you could be a little less harsh. :oops: :blush:", "Solution_8": "I assure you, Kent is one of the most helpful people on this forum. He's not being harsh, he's just trying to emphasize an important point." } { "Tag": [ "algebra", "polynomial", "complex analysis", "complex analysis theorems" ], "Problem": "We know that every polynomial $p(z)$ has a root in $C$. Could you give us a proof without using Liouville theorem? (An geometric proof is my requirement)", "Solution_1": "See http://en.wikipedia.org/wiki/Fundamental_theorem_of_algebra#Complex-analytic_proofs" } { "Tag": [ "inequalities", "algebra proposed", "algebra" ], "Problem": "Let sequence of numbers $ (u_n)$ such that\r\n\r\n$ u_0\\equal{}0$\r\n$ u_1\\equal{}1$\r\n$ u_{n\\plus{}1}\\equal{}2u_n\\minus{}u_{n\\minus{}1}\\plus{}1$ for all n be natural number, $ n\\ge 1$.\r\n\r\nProve that $ A\\equal{}4u_n.u_{n\\plus{}2}\\plus{}1$ be a square number.", "Solution_1": "Define $ v_n : \\equal{} u_n \\minus{} u_{n \\minus{} 1}$ for $ n\\ge 1$. Then the recursive relation implies\r\n\\[ v_{n \\plus{} 1} \\equal{} v_n \\plus{} 1,\\,\\forall n\\ge 1.\r\n\\]\r\nCombining this with $ v_1 \\equal{} 1$ we conclude that $ v_i \\equal{} i\\,\\forall i$. Thus\r\n\\[ u_n \\equal{} u_0 \\plus{} \\sum_{i \\equal{} 1}^nv_i \\equal{} \\frac {n(n \\plus{} 1)}{2}.\r\n\\]\r\nTherefore $ A \\equal{} n(n \\plus{} 1)(n \\plus{} 2)(n \\plus{} 3) \\plus{} 1 \\equal{} (n^2 \\plus{} 3n \\plus{} 1)^2$. Q.E.D.\r\n\r\nWhy is this in the Inequalities forum?", "Solution_2": "Very nice, thank you, nayel." } { "Tag": [], "Problem": "an 84 meter lenght of cable is cut so that one peisce is 18 meters longer than the other. find the lenght of each peice", "Solution_1": "Call the shorter piece $ \\alpha$. We then have the equation:\r\n\r\n$ \\alpha \\plus{} (18 \\plus{} \\alpha) \\equal{} 84$\r\n\r\nRemove the parentheses.\r\n\r\n$ 18 \\plus{} 2 \\alpha \\equal{} 84$\r\n\r\nIsolate the variable.\r\n\r\n$ 2 \\alpha \\equal{} 84 \\minus{} 18$\r\n\r\n$ 2 \\alpha \\div 2 \\equal{} 66 \\div 2$\r\n\r\n$ \\alpha \\equal{} 33$\r\n\r\nThus the smaller piece is 33 meters long, and the longer piece is $ 33 \\plus{} 18 \\equal{} 51$ meters long." } { "Tag": [], "Problem": "Ok, i have another one :D\r\n\r\n$-1 = (-1)^1 = (-1)^{\\frac{2}{2}} =(-1)^{2\\frac{1}{2}}=((-1)^2)^{\\frac{1}{2}}=1^{\\frac{1}{2}}=1$ :P", "Solution_1": "you could shorten it down\r\n$1^2=(+-1)^2$\r\n$1=-1$\r\n :P \r\nthis is wrong a bit because $a^2=b^2$\r\ndoesnt mean $a=b$" } { "Tag": [], "Problem": "If xyz=6 minimize 2z+2sqrt(x^2+y^2).\r\n\r\nThe original problem was: A ribbon is tied around a box with volume 6 such that it goes along an edge than along the diagonal of the top side than down the opposite edge and along the diagonal of the bottom side.", "Solution_1": "Does minimize mean finding the lower or upper bound....? :| \r\n$ 2z\\plus{} \\sqrt {x^2\\plus{}y^2} \\geq 2z \\plus{} \\frac{x\\plus{}y}{\\sqrt {2}} \\geq 3(xyz)^\\frac {1}{3}\\equal{}3(6)^\\frac {1}{3}$", "Solution_2": "lower bound" } { "Tag": [ "probability and stats" ], "Problem": "I need to proof this theorem or find e-book with this proof, please help me:\r\n\r\n$ t_{1}: (X,B) \\longrightarrow(T_{1},T_{1})$ is sufficient statistic for $ \\theta$ in experiment $ \\varepsilon \\equal{} (X,B,P_{\\theta}: \\theta \\in \\Theta )$\r\n\r\n$ t_{2}: (T_{1},T_{1}) \\longrightarrow(T_{2},T_{2})$ is sufficient statistic for $ \\theta$ in experiment which is generate by $ T_{1}$\r\n\r\nthen proof that $ T_{2}(T_{1}(x))$ is sufficient for $ \\theta$ in $ \\varepsilon$", "Solution_1": "I can't edit so i posted again. Sorry. :blush: Please help.\r\n\r\nI need to proof this theorem or find e-book with this proof, please help me:\r\n\r\n$ t_{1}: (X,B) \\longrightarrow(T_{1},T_{1})$ is sufficient statistic for $ \\theta$ in experiment $ \\varepsilon \\equal{} (X,B,P_{\\theta}: \\theta \\in \\Theta )$\r\n\r\n$ t_{2}: (T_{1},T_{1}) \\longrightarrow(T_{2},T_{2})$ is sufficient statistic for $ \\theta$ in experiment which is generate by $ t_{1}$\r\n\r\nthen proof that $ t_{2}(t_{1}(x))$ is sufficient for $ \\theta$ in $ \\varepsilon$", "Solution_2": "The problem is that your terminology is unclear :( What is $ (T_1,T_1)$ for example? And what is \"an experiment generated by $ t_1$\"?", "Solution_3": "[quote=\"fedja\"]The problem is that your terminology is unclear :( What is $ (T_1,T_1)$ for example?And what is \"an experiment generated by $ t_1$\"? [/quote]\r\n\r\n\r\nThis are sets from statistic definition.\r\n\r\nI have following statistic definition:\r\n\r\nStatistic $ t$ is measurable transformation $ t: (X,B) \\longrightarrow(T,F)$, where $ (X,B,P_{\\theta}: \\theta \\in \\Theta )$ is experiment.\r\n\r\nThen experiment $ (T,F,P_{\\theta}: \\theta \\in \\Theta )$ we call induced (generated) by statistic $ t$.\r\n\r\nIn this theorem we have two the same sets $ (T_{1},T_{1})$ instead $ (T,F)$\r\n\r\n\r\nBecause $ B$ is $ \\sigma$-algebra Borel sets and in this theorem $ t_{2}$ is a statistic so i think $ T_{1}$ and $ T_{2}$ are $ \\sigma$-algebras" } { "Tag": [ "geometry proposed", "geometry" ], "Problem": "(A. Myakishev, 8--9) In the plane, given two concentric circles with the center $ A$. Let $ B$ be an arbitrary point on some of these circles, and $ C$ on the other one. For every triangle $ ABC$, consider two equal circles mutually tangent at the point $ K$, such that one of these circles is tangent to the line $ AB$ at point $ B$ and the other one is tangent to the line $ AC$ at point $ C$. Determine the locus of points $ K$.", "Solution_1": "Locus is a circle with center A and radius of sqrt[(R^2 + r^2)/2].", "Solution_2": "How did you determine the locus? Proof?", "Solution_3": "Anyone???", "Solution_4": "\nLet $A(0,0)$; choose the x-axis through the given point $B$, then $B(r,0)$.\nPoint $C(a,b)$ on the circle with radius $R$, so $a^{2}+b^{2}=R^{2} \\quad\\quad\\quad (1)$.\n\nThe first circle, touching $AB$ in $B$, has midpoint $M_{1}(b,n)$ and radius $n$.\nThe second circle, touching $AC$ in $C$, has midpoint $M_{2}$ and also radius $n$.\nEquation of the line $CM_{2}\\ :\\ y-b=-\\frac{a}{b}(x-a)$;\nequation of the circle, midpoint $C$ and radius $n\\ :\\ (x-a)^{2}+(y-b)^{2}=n^{2}$.\nSolving the system of this two equations, gives us $M_{2}(a+\\frac{bn}{R},b-\\frac{an}{R})$.\n\nIt is possible to calculate $n$ from $M_{}M_{2}=2n$ or $(a+\\frac{bn}{R}-r)^{2}+(b-\\frac{an}{R}-n)^{2}=4n^{2} \\quad\\quad\\quad (2)$.\n\nPoint $K(x,y)$ is the midpoint of $M_{}M_{2}$:\n$a+\\frac{bn}{R}+r=2x$ and $b-\\frac{an}{R}+n=2y \\quad\\quad\\quad (3)$.\n\n$a+\\frac{bn}{R}=2x-r$ and $b-\\frac{an}{R}=2y-n$.\nFrom (2): $(2x-r-r)^{2}+(2y-n-n)^{2}=4n^{2}$ and $n = \\frac{(x-r)^{2}+y^{2}}{2y} \\quad\\quad\\quad (4)$.\n\nSolving $a$ and $b$ from (3): $a = \\frac{R(2Rx-2ny+n^{2}-rR)}{n^{2}+R^{2}}$ and $b=\\frac{R(2nx+2Ry-nr-nR)}{n^{2}+R^{2}}$.\n\nUsing (1), we have $4x^{2}-4rx+4y^{2}-4ny+r^{2}-R^{2}=0$.\nUsing (4), we have $2x^{2}+2y^{2}-r^{2}-R^{2}=0$.\n" } { "Tag": [ "search", "advanced fields", "advanced fields unsolved" ], "Problem": "For which cardinalities $ \\kappa$ do antimetric spaces of cardinality $ \\kappa$ exist? \r\n$ (X,\\varrho)$ is called an $ \\textit{antimetric space}$ if $ X$ is a nonempty set, $ \\varrho : X^2 \\rightarrow [0,\\infty)$ is a symmetric map, $ \\varrho(x,y)\\equal{}0$ holds iff $ x\\equal{}y$, and for any three-element subset $ \\{a_1,a_2,a_3 \\}$ of $ X$ \\[ \\varrho(a_{1f},a_{2f})\\plus{}\\varrho(a_{2f},a_{3f}) < \\varrho(a_{1f},a_{3f})\\] holds for some permutation $ f$ of $ \\{1,2,3 \\}$. \r\n\r\n\r\n[i]V. Totik[/i]", "Solution_1": "I posted this one [url=http://www.mathlinks.ro/Forum/viewtopic.php?search_id=1438234035&t=74263]here[/url]." } { "Tag": [ "inequalities", "geometry", "circumcircle", "trigonometry", "function", "Euler", "inequalities proposed" ], "Problem": "Let $ ABC$ be a triangle such that: $ A \\ge B \\ge \\frac{\\pi}{3} \\ge C$.Prove that:\r\n$ \\sqrt{\\frac{b\\plus{}c\\minus{}a}{a}}\\plus{}\\sqrt{\\frac{c\\plus{}a\\minus{}b}{b}}\\plus{}\\sqrt{\\frac{a\\plus{}b\\minus{}c}{c}} \\ge 3$\r\nFurthermore,we have:\r\n$ \\sqrt{\\frac{b\\plus{}c\\minus{}a}{a}}\\plus{}\\sqrt{\\frac{c\\plus{}a\\minus{}b}{b}}\\plus{}\\sqrt{\\frac{a\\plus{}b\\minus{}c}{c}} \\ge \\sqrt{9\\plus{}\\frac{R\\minus{}2r}{R}}$\r\nWhere $ r,R$ be the radius and circumradius of a triangle $ ABC$\r\nAnd ,we can make stronger.I will post later :)", "Solution_1": "[quote=\"quykhtn-qa1\"]Let $ ABC$ be a triangle such that: $ A \\ge B \\ge \\frac {\\pi}{3} \\ge C$.Prove that:\n$ \\sqrt {\\frac {b \\plus{} c \\minus{} a}{a}} \\plus{} \\sqrt {\\frac {c \\plus{} a \\minus{} b}{b}} \\plus{} \\sqrt {\\frac {a \\plus{} b \\minus{} c}{c}} \\ge 3$\nFurthermore,we have:\n$ \\sqrt {\\frac {b \\plus{} c \\minus{} a}{a}} \\plus{} \\sqrt {\\frac {c \\plus{} a \\minus{} b}{b}} \\plus{} \\sqrt {\\frac {a \\plus{} b \\minus{} c}{c}} \\ge \\sqrt {9 \\plus{} \\frac {R \\minus{} 2r}{R}}$\nWhere $ r,R$ be the radius and circumradius of a triangle $ ABC$\nAnd ,we can make stronger.I will post later :)[/quote]\r\nDear Virgil Nicula,hope you like this inequality :) .\r\nMay be it's easy :(", "Solution_2": "[color=darkblue][b]This inequality is very nice and difficult. I will also present a stronger inequality than the proposed one. But first, let's prove the following lemmas :[/b][/color]\n\n[quote][color=darkred][u][b]Lemma 1.[/b][/u] Let $ABC$ be a triangle so that its angles satisfy a relation of the type : $A\\ge B\\ge 60^{\\circ}\\ge C$. Prove that: \n\n\\[\\boxed{\\ s\\ \\ge\\ \\sqrt 3\\cdot (R+r)\\ }\\ .\\][/color]\n[/quote]\n\n[color=darkblue][u][b]Proof.[/b][/u] Since the angles of $\\triangle ABC$ satisfy $A\\ge B\\ge 60^{\\circ}\\ge C \\implies \\left\\{\\begin{array}{ccc}\n\\tan\\frac A2-\\frac {\\sqrt 3}3\\ \\ge\\ 0 \\\\ \\\\ \n\\tan\\frac B2-\\frac {\\sqrt 3}3\\ \\ge\\ 0 \\\\ \\\\ \n\\tan\\frac C2-\\frac {\\sqrt 3}3\\ \\le\\ 0 \\end{array}\\right|\\bigodot\\implies $\n\n$\\implies\\ \\prod\\left(\\tan\\frac A2-\\frac {\\sqrt 3}3\\right)\\ \\le\\ 0\\iff\\prod\\tan\\frac A2$ $-\\frac {\\sqrt 3}3\\cdot\\sum\\tan\\frac B2\\tan\\frac C2+\\frac 13\\cdot\\sum\\tan\\frac A2-\\frac{\\sqrt 3}9\\ \\le\\ 0\\iff$\n\n$\\iff\\frac rs-\\frac {\\sqrt 3}3+\\frac 13\\cdot\\frac{4R+r}s$ $-\\frac{\\sqrt 3}9\\ \\le\\ 0\\iff 9r+3(4R+r)-4s\\sqrt 3\\ \\le\\ 0 \\iff\\boxed{\\ s\\ \\ge \\sqrt 3\\cdot (R+r)\\ }\\ .$[/color]\n\n[quote][color=darkred][u][b]Lemma 2.[/b][/u] In any triangle $ABC$ the following inequality holds: \n\n\\[\\boxed{\\sqrt{\\frac {b+c-a}a}+\\sqrt{\\frac {c+a-b}b}+\\sqrt{\\frac {a+b-c}c}\\ge \\frac {s+3r\\cdot (2-\\sqrt 3)}{\\sqrt {2Rr}}}\\ .\\][/color]\n[/quote]\n\n[color=darkblue][u][b]Proof.[/b][/u] Apply the [u][b]Jensen[/b]'s inequality[/u] for the [u]convex[/u] function $f(x)=\\tan\\frac x4$ on the interval $\\left(0\\ ,\\ \\frac {\\pi}4\\right)$ : \n\n$3\\cdot\\tan\\left(\\frac {\\frac A4+\\frac B4+\\frac C4}3\\right)\\ \\le\\ \\tan\\frac A4+\\tan\\frac B4+\\tan\\frac C4\\color{white}{.}$ $\\iff \\boxed{\\ 3\\cdot (2-\\sqrt 3)\\ \\le\\ \\tan\\frac A4+\\tan\\frac B4+\\tan\\frac C4\\ }\\ \\ (\\ast)$ .\n\nOn the other hand, we have: $\\sum\\tan\\ \\frac A4=\\sum\\frac{1-\\cos\\frac A2}{\\sin\\frac A2}=\\sum\\frac{1-\\frac{s-a}{AI}}{\\frac{r}{AI}}=\\sum\\frac{AI-s+a}{r}=\\frac{AI+BI+CI-s}{r}$.\n\nThus, the $(\\ast)$ is now equivalent to: $\\boxed{AI+BI+CI \\ge s+3r\\cdot (2-\\sqrt 3)}\\ (\\ast\\ast)$. Since $\\boxed{AI=\\sqrt {2Rr}\\cdot\\sqrt{\\frac {b+c-a}a}}$.\n\nand so on, the inequality $(\\ast\\ast)$ finally becomes : $\\boxed{\\ \\sqrt{\\frac {b+c-a}a}+\\sqrt{\\frac {c+a-b}b}+\\sqrt{\\frac {a+b-c}c}\\ \\ge\\ \\frac {s+3r\\cdot (2-\\sqrt 3)}{\\sqrt {2Rr}}\\ }$[/color]\n\n[quote][color=darkred][u][b]Problem.[/b][/u] Prove that in any triangle $ABC$ that satisfies a relation of type $A\\ge B\\ge 60^{\\circ}\\ge C$, the following inequality holds:\n\n\\[\\boxed{\\sqrt{\\frac {b+c-a}a}+\\sqrt{\\frac {c+a-b}b}+\\sqrt{\\frac {a+b-c}c}\\ \\ge\\ \\frac{6r+\\sqrt 3\\cdot (R-2r)}{\\sqrt{2Rr}}}\\ \\ge\\ \\sqrt{9+\\frac {R-2r}R}\\ \\ge\\ 3\\ .\\][/color]\n[/quote]\n\n[color=darkblue][u][b]Proof.[/b][/u] \n\n$\\blacktriangleright$ Apply [u]lemma [b]1[/b][/u] [b]&[/b] [u]lemma [b]2[/b][/u] : $\\implies\\sum\\sqrt{\\frac {b+c-a}a}\\ \\stackrel{(2)}{\\ge}\\ \\frac {s+3r\\cdot (2-\\sqrt 3)}{\\sqrt {2Rr}}\\ \\stackrel{(1)}{\\ge}\\ \\frac{6r+\\sqrt 3\\cdot (R-2r)}{\\sqrt{2Rr}}$ .\n\n$\\blacktriangleright$ The inequality $\\frac{6r+\\sqrt 3\\cdot (R-2r)}{\\sqrt{2Rr}}\\ge\\sqrt{9+\\frac{R-2r}R}$ reduces to : $6r+\\sqrt 3\\cdot (R-2r)\\ge\\sqrt{2r(10R-2r)}$\n\n$\\iff 3(R-2r)^2+12\\sqrt 3\\cdot r(R-2r)+36r^2\\ \\ge\\ 20Rr-4r^2\\color{white}{.}$ $\\iff (R-2r)\\cdot\\left[3R+(12\\sqrt 3-26)\\cdot r\\right]\\ge 0$ .\n\nThe last inequality is obvious according to [u][b]Euler[/b]'s inequality[/u] i.e. $\\boxed{\\ R\\ \\ge\\ 2r\\ }$ .Therefore, the problem is now solved :lol: .[/color]" } { "Tag": [ "trigonometry" ], "Problem": "Let $ x,y,z > 0$ be such that $ xy \\plus{} yz \\plus{} zx \\plus{} 2xyz \\equal{} 1$.\r\n\r\nProve that there exist $ p,q,r > 0$ such that $ x \\equal{} \\frac {p}{q \\plus{} r},y \\equal{} \\frac {q}{r \\plus{} p},z \\equal{} \\frac {r}{p \\plus{} q}$", "Solution_1": "[hide=\"Large hint\"] This should remind you of the identity $ \\cos^2 \\alpha \\plus{} \\cos^2 \\beta \\plus{} \\cos^2 \\gamma \\plus{} 2 \\cos \\alpha \\cos \\beta \\cos \\gamma \\equal{} 1$, where $ \\alpha \\plus{} \\beta \\plus{} \\gamma \\equal{} \\pi$. What are $ x, y, z$ in terms of these cosines? :) [/hide]", "Solution_2": "[b]t0rajir0u\n[/b]\r\n\r\nCondition is not $ x^{2} \\plus{} y^{2} \\plus{} z^{2} \\plus{} 2xyz \\equal{} 1$ :roll:", "Solution_3": "[quote=\"delegat\"][b]t0rajir0u\n[/b]\n\nCondition is not $ x^{2} \\plus{} y^{2} \\plus{} z^{2} \\plus{} 2xyz \\equal{} 1$ :roll:[/quote]\r\n\r\nWat t0rajir0u means is $ x\\equal{}\\frac {\\cos A \\cos B}{\\cos C}, y\\equal{}\\frac {\\cos B \\cos C}{\\cos A}, z\\equal{}\\frac {\\cos C \\cos A}{\\cos B}$", "Solution_4": "actually one could have $ \\sqrt{xy}\\equal{}p$ :wink:", "Solution_5": "[quote=\"radio\"]Let $ x,y,z > 0$ be such that $ xy \\plus{} yz \\plus{} zx \\plus{} 2xyz \\equal{} 1$.\n\nProve that there exist $ p,q,r > 0$ such that $ x \\equal{} \\frac {p}{q \\plus{} r},y \\equal{} \\frac {q}{r \\plus{} p},z \\equal{} \\frac {r}{p \\plus{} q}$[/quote]\r\nLet $ x \\equal{} \\frac {p}{q \\plus{} r}$ and $ y \\equal{} \\frac {q}{r \\plus{} p}.$ It's probably because $ xy\\neq1.$ :wink: \r\nThen $ z \\equal{} \\frac {1 \\minus{} xy}{x \\plus{} y \\plus{} 2xy} \\equal{} \\frac {r}{p \\plus{} q}.$ :wink:" } { "Tag": [ "calculus", "derivative", "integration", "real analysis", "real analysis theorems" ], "Problem": "hi\r\ndoes anyone knows the proof (or where I can fin it) of that:\r\n\r\n$\\frac{\\delta f}{\\delta x \\delta y}=\\frac{\\delta f}{\\delta y \\delta x}$\r\n\r\n(not really the right symbol for second derivate $\\delta$ , what s the command?)\r\n\r\nthanks!\r\n\r\nledurt", "Solution_1": "[quote](not really the right symbol for second derivate \\delta , what s the command?) [/quote]\r\nThe right symbol is \\partial, and you need a square in there:\r\n\r\n$\\frac{\\partial^2f}{\\partial x\\partial y}=\\frac{\\partial^2f}{\\partial y\\partial x}$\r\n\r\nAs for the theorem: you need a hypothesis that both of these partial derivatives are continuous in an open set. I found it in Rudin's [i]Principles of Mathematical Analysis[/i] as Theorem 9.34. The proof involves taking integrals of these derivatives and interchanging the order of integration.", "Solution_2": ":blush: \r\ngeez...i'm ashame now, i forgot the $^2$.... :blush: \r\n\r\nand thanks for the info, i ll look it up!\r\n\r\nledurt", "Solution_3": "[quote=\"Kent Merryfield\"]$\\frac{\\partial^2f}{\\partial x\\partial y}=\\frac{\\partial^2f}{\\partial y\\partial x}$\n\nAs for the theorem: you need a hypothesis that both of these partial derivatives are continuous in an open set.[/quote]\r\nA weaker condition: both partial derivatives exist and one of them is continuous.", "Solution_4": "eee, what do you mean by \"weaker condition\" ?\r\n\r\nledurt", "Solution_5": "[quote=\"Ledurt\"]eee, what do you mean by \"weaker condition\" ?\nledurt[/quote]\r\nKent Merryfield says that $\\frac{\\partial^2 f}{\\partial y\\partial x}=\\frac{\\partial^2 f}{\\partial x\\partial y}$ if both of these partial derivatives are continuous;\r\nand in fact, the condition can be a little weaker: We do not need the existence of both, just the existence of one partial derivative is okay.", "Solution_6": "[quote=\"liyi\"]We do not need the [i]existence[/i] [u][b]continuity[/b][/u] of both, just the [i]existence[/i] [u][b]continuity[/b][/u] of one partial derivative is okay.[/quote]\r\nYou meant to say \"continuity\", not \"existence\", right?", "Solution_7": "[quote=\"Kent Merryfield\"][quote=\"liyi\"]We do not need the [i]existence[/i] [u][b]continuity[/b][/u] of both, just the [i]existence[/i] [u][b]continuity[/b][/u] of one partial derivative is okay.[/quote]\nYou meant to say \"continuity\", not \"existence\", right?[/quote]\r\nYes....sorry for the serious typo...", "Solution_8": "ok, i get it now\r\n\r\nthanks a lot!\r\n\r\nledurt", "Solution_9": "[quote=\"liyi\"][quote=\"Ledurt\"]eee, what do you mean by \"weaker condition\" ?\nledurt[/quote]\nKent Merryfield says that $\\frac{\\partial^2 f}{\\partial y\\partial x}=\\frac{\\partial^2 f}{\\partial x\\partial y}$ if both of these partial derivatives are continuous;\nand in fact, the condition can be a little weaker: We do not need the existence of both, just the existence of one partial derivative is okay.[/quote]\r\n\r\n Hey, that's Schwarz's criterion.If you want a proof,i'm here.I also know a similar theorem which is due to Young. ;)" } { "Tag": [ "inequalities", "calculus", "function", "inequalities proposed" ], "Problem": "Let $a,b,c $ be the sides of a triangle, $s=\\frac{a+b+c}{2}$ is the semiperimeter of triangle. Prove that:\r\n\\[\\frac{3}{s^3}\\ge \\frac{1}{as^2+9(s-a)^3}+\\frac{1}{bs^2+9(s-b)^3}+\\frac{1}{cs^2+9(s-c)^3}\\]", "Solution_1": "Edit: I will get back to my initial idea. putting a=x+y, cyclic, we get to prove:\r\n\r\nPutting x+y+z=1: $3 \\ge \\sum_{\\text{cyc}} \\frac{1}{1 - z + 9z^3}$.\r\n\r\nOr putting x+y+z=3: $1 \\ge \\sum_{\\text{cyc}} \\frac{1}{3 - z + z^3}$.\r\n\r\nI can solve this the ugly calculus-way, or the long muirhead-way, but I guess you have a much nicer solution in thoughts?", "Solution_2": "[quote=\"Peter VDD\"]I think your inequality should have the reversed sign. :) I will prove it:\n\n\\[\\frac{3}{s^3}\\le \\frac{1}{as^2+9(s-a)^3}+\\frac{1}{bs^2+9(s-b)^3}+\\frac{1}{cs^2+9(s-c)^3}\\]\n\nPut $a=x+y, b=y+z, c=z+x$. We get: \\[\\frac{3}{(x+y+z)^3} \\le \\sum_{cyc}\\frac{1}{(x+y)(x+y+z)^2+9z^3}\\]\n\nThe inequality is homogenous so we can assume $x+y+z=30$. We get: \\[\\frac{1}{9000} \\le \\sum_{cyc}\\frac{1}{900(x+y)+9z^3} = \\sum_{cyc}\\frac{1}{27000-900z+9z^3} \\text{, thus } {\\sum_{cyc}\\frac{1}{3000-100x+x^3} \\ge \\frac{1}{1000}}\\]\n\nBut the function $f(x) = \\frac{1}{3000-100x+x^3}$ is convex, so Jensen gives us: \\[\\dfrac{f(x)+f(y)+f(z)}{3} \\ge f\\left(\\dfrac{x+y+z}{3}\\right)=f(10)\\]\n\nWhich is, written out: \\[\\sum_{cyc}\\frac{1}{3000-100x+x^3} \\ge 3 \\cdot f(10) = \\frac{3}{3000}=\\frac{1}{1000}\\]\n\nQ.e.D.[/quote]\r\n\r\nThe inequality is true\r\nThe function $f(x) = \\frac{1}{3000-100x+x^3}$ is not convex\r\n\r\n.", "Solution_3": "Ah darn, it's convex from 0 to 1 I confused the intervals again :D\r\n\r\nI'm sorry, I'll check for a better proof.", "Solution_4": "[quote=\"xtar\"]Let $a,b,c $ be the sides of a triangle, $s=\\frac{a+b+c}{2}$ is the semiperimeter of triangle. Prove that:\n\\[\\frac{3}{s^3}\\ge \\frac{1}{as^2+9(s-a)^3}+\\frac{1}{bs^2+9(s-b)^3}+\\frac{1}{cs^2+9(s-c)^3}\\][/quote]\r\n\r\nNormalize $s$ to $1$, so: given $x + y + z = 1$, we want to prove: \r\n$3 \\ge \\sum_{\\text{cyc}} \\frac{1}{1 - z + 9z^3}$\r\nFirst, if all of the variables are between $0$ and $\\frac{2}{3}$, we have: $-\\frac{(2x+1)(3x-2)(3x-1)^2}{3} \\ge 0 \\Rightarrow \\frac{1}{1-x+9x^3} \\le \\frac{5}{3} - 2x$, so adding these inequalities yields the desired. Now, let $f(t) = \\frac{1}{1 - t + 9t^3}$. Suppose there is some variable, WLOG $x$, such that $x \\ge \\frac{2}{3}$. Then $f(x) \\le \\frac{1}{3}$, so if $f(x) + f(y) + f(z) > 3$, for sake of contradiction, then WLOG $f(y) > \\frac{4}{3}$, i.e. $1 - y + 9y^3 < \\frac{3}{4}$, i.e. $1 - 4y + 36y^3 < 0$, and we can easily see this never happens (define $g(y) = 1 - 4y + 36y^3$, then note $\\frac{dg}{dy} = 108y^2 - 4$, so calculate the critical points, etc.)\r\n\r\nhope this works", "Solution_5": "Yes, but that was exactly what I wanted to avoid. ;) I think if he asks this problem, then there'll be a nicer solution for $3 \\ge \\sum_{\\text{cyc}} \\frac{1}{1 - z + 9z^3}$.\r\n\r\n(Or when putting x+y+z=3, you get $1 \\ge \\sum_{\\text{cyc}} \\frac{1}{3 - z + z^3}$, which also seems nice, but opposes us to the same problem as above)", "Solution_6": "it's pretty nice ... the only part I actually take offense with is resorting to calculus to prove $36t^3 - 4t + 1 > 0$ but calculus really wasn't necessary for that: simply note that if $t < \\frac{1}{4}$, we have $1 > 4t$. Then if $t > \\frac{1}{3}$, we have $36t^3 > 4t$. Finally, if $\\frac{1}{4} \\le t \\le \\frac{1}{3}$, we have $36t^3 > 2t$ and $1 > 2t$, so in all cases, we're done.", "Solution_7": "Here is my solution for the problem : \r\n $ x+y+z =3 $ . prove that : $ \\frac{1}{x^3-x+3} + \\frac{1}{y^3-y+3} + \\frac{1}{z^3-z+3} \\leq 1 $ \r\n \r\n [b]Case 1: [/b] $ x, y , z \\leq 2 $ \r\n We have : $ (x-1)^2(2-x)(2x+3) \\geq 0 $ \r\n So $ \\frac{1}{x^3-x+3} \\leq \\frac{5}{9} - \\frac{2}{9}x $ \r\n Similarly , we have another 2 ineqs\r\n Add them all ---> Q.E.D \r\n [b]Case 2: [/b] Suppose $ x,y \\leq 1 , z \\geq 2 $ \r\n We have $ x^3-x+3 \\geq 2.5 $ \r\n So $ \\frac{1}{x^3-x+3} + \\frac{1}{y^3-y+3} \\leq 0.8 (1) $ \r\n Due to $ z \\geq 2 $ , we have $ \\frac{1}{z^3-z+3} \\leq \\frac{1}{9} (2) $ \r\n (1)+(2) ---> Q.E.D", "Solution_8": "[quote=\"xtar\"][quote=\"Peter VDD\"]I think your inequality should have the reversed sign. :) I will prove it:\n\n\\[\\frac{3}{s^3}\\le \\frac{1}{as^2+9(s-a)^3}+\\frac{1}{bs^2+9(s-b)^3}+\\frac{1}{cs^2+9(s-c)^3}\\]\n\nPut $a=x+y, b=y+z, c=z+x$. We get: \\[\\frac{3}{(x+y+z)^3} \\le \\sum_{cyc}\\frac{1}{(x+y)(x+y+z)^2+9z^3}\\]\n\nThe inequality is homogenous so we can assume $x+y+z=30$. We get: \\[\\frac{1}{9000} \\le \\sum_{cyc}\\frac{1}{900(x+y)+9z^3} = \\sum_{cyc}\\frac{1}{27000-900z+9z^3} \\text{, thus } {\\sum_{cyc}\\frac{1}{3000-100x+x^3} \\ge \\frac{1}{1000}}\\]\n\nBut the function $f(x) = \\frac{1}{3000-100x+x^3}$ is convex, so Jensen gives us: \n\\[\\dfrac{f(x)+f(y)+f(z)}{3} \\ge f\\left(\\dfrac{x+y+z}{3}\\right)=f(10)\\]\n\nWhich is, written out: \\[\\sum_{cyc}\\frac{1}{3000-100x+x^3} \\ge 3 \\cdot f(10) = \\frac{3}{3000}=\\frac{1}{1000}\\]\n\nQ.e.D.[/quote]\n\nThe inequality is true\nThe function $f(x) = \\frac{1}{3000-100x+x^3}$ is not convex\n\n.[/quote]\r\n \r\n I'm very interested in your assumption $ x+y+z =30 $ , Peter :lol:", "Solution_9": "Well, I was looking for f to be convex on [0,1]... so that was true for 30 onwards :P but I forgot I then needed convexity on [0,30] instead :blush:" } { "Tag": [ "MATHCOUNTS", "ARML" ], "Problem": "(chances are you're here and not in our own forum).", "Solution_1": "because of Missouri's awesomeness compared to Kansas...", "Solution_2": "Oh definitely. Pew pew. *fires lasers across state line*", "Solution_3": "And in terms of post count, I am now at the end of WWII. Hmm.", "Solution_4": "Oh hello there.", "Solution_5": "*AHH- gets hit by laser pianoforte shot across MO-KS border*\r\n\r\n*falls down to the floor and dies*", "Solution_6": "Ouch. :( :maybe: :maybe:", "Solution_7": "*comes back to life*\r\n\r\n*shoots laser across the state of Missouri at painoforte's house.*\r\n\r\nRemember to play the [url=http://www.artofproblemsolving.com/Forum/weblog_entry.php?t=262852]Mathcounts Lottery[/url].", "Solution_8": "then pianoforte retaliates with an atom bomb...\r\n\r\nbut the beastliness of tinytim, justinahmann, and math154 hold it back and completely demolish Kansas \r\n\r\nYAY! :D", "Solution_9": "So... like, when you get back (which I know is like late evening), could someone post the results, like top indivs, name + school, and top school(s).\r\n\r\n(aiming for 2000+ posts)", "Solution_10": "1st- Mewto55555 \r\n2nd- tinytim \r\n3rd- math154 \r\n4th- mewto's sister \r\n\r\nLadue- 1st place team", "Solution_11": "Congratulations to mewto, math154, tinytim, and mewto's sister (lol).\r\n\r\nI'm honored to know all four of you in real life :)", "Solution_12": "Kansas State Results\r\n\r\n1. Shyam Narayanan (Owned everyone in CD and had a substantially higher written score than anyone else)\r\n2. Kevin Cao (Hope I spelled that write)\r\n3. Michael Zhou \r\n4. Jack Chen \r\n\r\nSadly I was fourth written, but Jack killed me in CD because I blanked out and now I'm fifth. :(\r\n\r\nNow that I failed Mathcounts, does anyone know any high school competitions (other than the AMC) that I could do?", "Solution_13": "\"Mewto's sister\" does have a name!", "Solution_14": "ARML,Mandelbrot,etc.", "Solution_15": "[quote=\"mathemagician1729\"]Kansas State Results\n\n1. Shyam Narayanan (Owned everyone in CD and had a substantially higher written score than anyone else)\n2. Kevin Cao (Hope I spelled that write)\n3. Michael Zhou \n4. Jack Chen \n\nSadly I was fourth written, but Jack killed me in CD because I blanked out and now I'm fifth. :(\n\nNow that I failed Mathcounts, does anyone know any high school competitions (other than the AMC) that I could do?[/quote]\r\n\r\nAlso, our pwnage team (3rd plce winner, mathemagician1729, me, and other indian guy) got 1st. lakewood got 2nd. oxford got 3rd.", "Solution_16": "Thanks for the info. All BV kids too, should make for easy practices.\r\n\r\nMathe: Sorry for that, that kinda sucks. Do MO ARML, GPML and... there's really not that much.", "Solution_17": "math154, do you know jeff boyd?", "Solution_18": "[quote=\"tinytim\"]1st- Mewto55555 \n2nd- tinytim \n3rd- math154 \n4th- mewto's sister \n\nLadue- 1st place team[/quote]\r\n\r\nMeet u at nats :wink: .", "Solution_19": "I do not believe your state has had its competition yet. Cocky much?", "Solution_20": "I think so as well. :wink:", "Solution_21": "Thank you" } { "Tag": [ "geometry", "perimeter", "inequalities", "congruent triangles" ], "Problem": "How many non-congruent triangles with perimeter 7 have integer side lengths?\r\n\r\n$\\left(A\\right) 1$\r\n$\\left(B\\right) 2$\r\n$\\left(C\\right) 3$\r\n$\\left(D\\right) 4$\r\n$\\left(E\\right) 5$", "Solution_1": "[hide=\"solution\"]$(1, 3, 3)$, $(2, 2, 3)$, there are 2 answers or $\\boxed{B}$.[/hide]", "Solution_2": "[hide] for the number 7, we can list the possibilities (since there aren't that many).\n115\n124\n132\n\n214\n223\n\n331\n\nso... we also find out that\nby triangular inequalities, there aren't any triangles for which two sides added together is less or equal to the third side.\nso, we have\n223, and 331,\ngiving the answer 2,\nor $B$\n[/hide]", "Solution_3": "[quote=\"turboturtle\"][hide] for the number 7, we can list the possibilities (since there aren't that many).\n115\n124\n132\n\n214\n223\n\n331\n\nso... we also find out that\nby triangular inequalities, there aren't any triangles for which two sides added together is less or equal to the third side.\nso, we have\n223, and 331,\ngiving the answer 2,\nor $B$\n[/hide][/quote]\n[hide]We can reduce the number of possibilites we need to check. The largest length a side length could be is 3, because if a sidelength is 4, than the sum of the other two sides is 3, which contradicts Triangle Inequality.\nTesting, 2,3,3 and 3,3,1 are the only solutions. $B$.[/hide]", "Solution_4": "[hide]1,3,3 \n2,2,3\n\n[b]2[/b][/hide]" } { "Tag": [ "inequalities" ], "Problem": "The product of a number with 4 is equal to the sum of the number and 36. What is the number?", "Solution_1": "[hide]you can write the equation: $4x=x+36$. Solve for x: $3x=36$. Divide by 3 on both sides: $\\boxed{x=12}$[/hide]", "Solution_2": "[hide]12[/hide]\r\n\r\n[color=darkred][size=75][Hide your answers and show some work!][/size][/color]", "Solution_3": "Remember to hide your answers and show your work! :)\r\n\r\n[hide=\":D - Click to reveal hidden answer...\"]\n\nEquation: \n\n4x = x + 36 ????\n\n4x - x = 36\n\n3x = 36\n\nx = 12\n\nThe number \"x\" is 12.\n\n[/hide]", "Solution_4": "[quote=\"MCrawford\"]The product of a number with 4 is equal to the sum of the number and 36. What is the number?[/quote]\r\n[hide]i think the number is $12$,and i think it is true.[/hide]", "Solution_5": "the solutions are enough i think...\r\n\r\nbut the problem is so nice ;)", "Solution_6": "[quote=\"ashegh\"]the solutions are enough i think...\n\nbut the problem is so nice ;)[/quote]\r\nyes i am agree with you ashegh. :idea:", "Solution_7": "mecrawford, do we have any contest for the basics?", "Solution_8": "Eival Ashegh and Jensen you 2 are so Strong because after a long calculation I reached to Number $12$.Excellent", "Solution_9": "Kheili khafanit baba :D :D :D :D :D :D :D :D :D :D :D :D \r\n[color=blue]It means[/color]:So Strong", "Solution_10": "eivaly az khodete amir joon.to khodet strong hasti ke ma ro strong mibini. :lol:", "Solution_11": "thanks alot dear amir ;) \r\n\r\njensen means:thanks alot dear amir.ure eyes see us strong,but we are not too...", "Solution_12": "why are you guys posting here? ;) go back to the olympiad section where you belong :P", "Solution_13": "[hide]4x=x+36\n3x=36\nx=12[/hide]" } { "Tag": [ "analytic geometry", "calculus", "calculus computations" ], "Problem": "A particle is moving along the curve $ y\\equal{}\\sqrt{x}$. As the particle passes through the point $ (4.2)$, its x-coordinate increases at a rate of $ 3 \\text{cm/s}$. How fast is the distance from the particle to the origin changing at this point?\r\n\r\nI got an answer of $ \\frac{9}{\\sqrt{5}} \\text{cm/s}$ but I don't have the solutions manual so I don't know if this is right. Can someone just let me know what they get?\r\n\r\nI used $ x^{2}\\plus{}y^{2}\\equal{}z^{2}$ with $ y\\equal{}\\sqrt{x}$ so that $ x^{2}\\plus{}x\\equal{}z^{2}$.", "Solution_1": "I differentiated $ z\\,\\equal{}\\,\\sqrt{x^2\\,\\plus{}\\,x}$ by $ t$ for the solution and arrived at $ \\frac{dz}{dt}\\,\\equal{}\\,\\frac{27}{4\\sqrt{5}}.$" } { "Tag": [ "trigonometry", "algebra proposed", "algebra" ], "Problem": "$ \\sum_{i\\equal{}1}^{n}\\sum_{j\\equal{}1}^{n}(\\cos{\\frac{2\\pi (i^2\\plus{}j^2)}{n}})\\equal{}n\\sin{\\frac{n\\pi}{2}}$", "Solution_1": "It is known Gaus's summ\r\n\\[ \\sum_{i \\equal{} 1}^{n}\\sum_{j \\equal{} 1}^{n}(\\cos{\\frac {2\\pi (i^2 \\plus{} j^2)}{n}}) \\equal{}Re(\\sum_{j\\equal{}1}^n\\exp(\\frac{2\\pi i j^2}{n}))^2\\equal{} n\\sin{\\frac {n\\pi}{2}}\\]", "Solution_2": "[quote=\"Rust\"]It is known Gaus's summ\n\\[ \\sum_{i \\equal{} 1}^{n}\\sum_{j \\equal{} 1}^{n}(\\cos{\\frac {2\\pi (i^2 \\plus{} j^2)}{n}}) \\equal{} Re(\\sum_{j \\equal{} 1}^n\\exp(\\frac {2\\pi i j^2}{n}))^2 \\equal{} n\\sin{\\frac {n\\pi}{2}}\\]\n[/quote]\r\nanother nice identity here\r\n$ \\sum_{i \\equal{} 1}^{n}\\sum_{j \\equal{} 1}^{n}(cos(\\frac {2\\pi(i^2 \\minus{} j^2)}{n})) \\equal{} n \\plus{} n\\cos{\\frac {n\\pi}{2}}$" } { "Tag": [ "real analysis", "real analysis unsolved" ], "Problem": "Let f(n) = ((4n-2)/(n+1))*f(n-1), n >1\r\nSupposed f(2) and f(3) are prime numbers. Prove that f(n) is Not prime for any other integer values n>1.", "Solution_1": "[hide=\"solution\"]\nWe notice that $ f(3) \\equal{} \\frac {5}{2}f(2)$ so the only way both $ f(2)$ and $ f(3)$ are prime is if $ f(2) \\equal{} 2$ and $ f(3) \\equal{} 5$. We can then write the unique solution to the recurrence equation:\n\\[ f(n) \\equal{} 2^n\\frac {\\prod _{k \\equal{} 1}^n{(2k \\minus{} 1)}}{(n \\plus{} 1)!}\n\\]\nThe numerator is a product of all odds up to $ 2n \\minus{} 1$, and therefore certainly contains primes that the denominator does not.\n\nA question one might ask is if $ f(n)\\in\\mathbb{Z}$ for all $ n$. It is, though that requires a few more steps.\n[/hide]" } { "Tag": [ "Columbia", "algebra unsolved", "algebra" ], "Problem": "If $ x\\neq1$, $ y\\neq1$, $ x\\neq y$ and\r\n\\[ \\frac{yz\\minus{}x^{2}}{1\\minus{}x}\\equal{}\\frac{xz\\minus{}y^{2}}{1\\minus{}y}\\]\r\nshow that both fractions are equal to $ x\\plus{}y\\plus{}z$.", "Solution_1": "$\\frac{yz-x^2}{1-x} = \\frac{xz-y^2}{1-y}$\r\n$\\frac{yz-x^2}{xz-y^2} = \\frac{1-x-1+y}{1-y}$\r\n$\\frac{yz-x^2-xz+y^2}{xz-y^2} = \\frac{y-x}{1-y}$\r\n$\\frac{(x+y+z)(y-x)}{xz-y^2} = \\frac{y-x}{1-y}$\r\n$\\frac{xz-y^2}{1-y} = x+y+z = \\frac{yz-x^2}{1-x}$", "Solution_2": "Just this:\r\n$\\frac{yz-x^2}{1-x}=\\frac{zx-y^2}{1-y}=\\frac{(yz-x^2)-(xz-y^2)}{(1-x)-(1-y)}=x+y+z(x\\neq y)$" } { "Tag": [ "trigonometry", "inequalities", "quadratics" ], "Problem": "if A. B, C are the angles of a triangle, then sin A + sin B + sin C <= 3 :rt3: /2. this is directly proved from jensen, since sin is concave. how do we prove cos A + cos B + cos C <=3/2? cos is not convex or convace on [0, :pi: ]?", "Solution_1": "One approach is to apply Jensen to the two smallest angles.", "Solution_2": "Another approach is to use the identity cos A + cos B = 2 cos((A+B)/2) cos((A-B)/2) :le: 2 cos((A+B)/2), and its permutations. We just need to show that cos((A+B)/2) + cos((A+C)/2) + cos((B+C)/2) :le: 3/2. But those angles are all acute, so we can apply Jensen after all.", "Solution_3": "Sorry, what's Jensen's Inequality?", "Solution_4": "It's basically an application of convexity. Here it is in the special case we were using: Let f be a concave function. Then f(x) + f(y) + f(z) :le: 3 f((x + y + z) / 3).", "Solution_5": "[quote=\"Ravi B\"]Another approach is to use the identity cos A + cos B = 2 cos((A+B)/2) cos((A-B)/2) :le: 2 cos((A+B)/2), and its permutations. We just need to show that cos((A+B)/2) + cos((A+C)/2) + cos((B+C)/2) :le: 3/2. But those angles are all acute, so we can apply Jensen after all.[/quote]\r\n\r\nWhy are these angles acute? \r\n\r\nAlso, how does one prove cot A + cot B + cot C :ge: :sqrt:3, where A, B, and C are the angles of a triangle?", "Solution_6": "[quote]Why are these angles acute?[/quote]\r\n\r\nBecause (a + b + c)/2 = 90 so (a + b)/2 < 90 and thus is acute.", "Solution_7": "MysticTerminator wrote:how does one prove cot A + cot B + cot C :ge: :sqrt:3, where A, B, and C are the angles of a triangle?\n\nHint in spoiler:[hide]Note that cotangent is convex between 0 and 90 degrees. Assume that angles A and B are acute. By convexity, we can assume that A = B. We can then express the inequality purely in terms of C. Use the identity connecting cot C and cot(C/2) to express the inequality as a quadratic in cot(C/2).[/hide]\n\n\n\nI don't know if there is a better method.", "Solution_8": "Another way to prove \r\n[tex]\\cos{A} + \\cos{B} + \\cos{C} \\le 3/2[/tex]\r\nis to use\r\n[tex]\\displaystyle\\cos{A} + \\cos{B} + \\cos{C} = \\frac{r}{R} + 1[/tex]\r\nand\r\n[tex]2r \\le R[/tex].", "Solution_9": "[quote=\"MysticTerminator\"]cot A + cot B + cot C :ge: :sqrt:3, where A, B, and C are the angles of a triangle?[/quote]\r\n\r\nAn other solution :\r\n\r\nCot A=(b^2 +c^2 - a^2 )/4S by laws of sines + cosines\r\n\r\nSo :Sigma: Cot A = :Sigma: a^2 / 2S (S = surface)\r\n\r\n:Sigma: a >= ab + bc+ ca = 2S (:Sigma: 1/sinA)\r\n\r\nNow 1/sin(x) is convex so (:Sigma: 1/sinA) >= 3/sin(:pi:/3) = 2 :rt3:" } { "Tag": [ "inequalities", "induction", "graph theory", "combinatorics unsolved", "combinatorics" ], "Problem": "The plane is divided into regions by $ n \\ge 3$ lines, no two of which are parallel, and no three of which are concurrent. Some regions are coloured , in such a way that no two coloured regions share a common segment or half-line of their borders. Prove that the number of coloured regions is at most $ \\frac{n(n\\plus{}1)}{3}$", "Solution_1": "Note that two regions share a common segment or half-line only if they also share a vertex. Furthermore, every pair of lines forms exactly one vertex and there are $ 1 \\plus{} \\binom{n \\plus{} 1}{2}$ regions. Let $ v_R$ be the number of vertices of region $ R$ and denote $ V \\equal{} \\sum v_R$. Noticing that each intersection adds four vertices to $ V$ and that each line intersects every other line in distinct points, there $ v \\equal{} 4\\binom{n}{2}$, and this is $ \\le$ the number of sides with double counting (it would be $ \\equal{}$, but we have the unbounded regions). Finally, we observe that every side is counted exactly twice, so there are at least $ 2\\binom{n}{2}$ sides. \r\n\r\nSuppose there were more than $ \\frac {n(n \\plus{} 1)}{3}$ colored regions. Since no side is counted twice here, there are exactly $ \\frac {2n(n \\plus{} 1)}{3}$ sides counted. But this is $ > 2\\binom{n}{2}$; contradiction.\r\n\r\nThis seems to make the bound seem relatively weak, so I probably screwed up somewhere. It's late though, so I don't have time to check.\r\n\r\nEDIT:\r\n\r\nThe above is flawed in two ways. Firstly, I only proved there are at least $ 2\\binom{n}{2}$ sides with double counting, when I need an upper bound rather than zero bound. It turns out there are *exactly* $ 2\\binom{n}{2} \\plus{} 2n$ sides.\r\n\r\nFurthermore, the last inequality I used isn't actually true. :oops: I have a solution now, but unfortunately once again it's late and I don't have time to type it up. It rests on the lemma that there are always exactly three regions with 2 sides.\r\n\r\nIt's worth noting that the desired bound is 2/3 the total number of regions, so this may lead to a nicer proof.", "Solution_2": "Now that it's morning and I'm more sane, here's my solution:\r\n\r\n[b]Lemma:[/b] There are always $ 2n$ unbounded regions.\r\n[b]Proof:[/b] Induction. Note that all other regions have the same number of vertices and sides and unbounded regions have one more side than vertex; hence the total side sum count is exactly $ 2\\binom{n}{2}+2n=n(n+1)$ as I noted above. \r\n\r\n[b]Lemma:[/b] There are always exactly three regions with two sides.\r\n[b]Proof:[/b] They must be unbounded; induction by considering cases of what unbounded regions the $ n+1$th line cuts through.\r\n\r\nSo suppose we have chosen $ \\frac{n(n+1)}{3}$ regions and none share a side; we want to prove that this consumes more than the number of sides itself. To do this, we use a weighted average. Let $ x$ be the number of regions with exactly three sides; if we can show that $ 6+3x+4(\\frac{n(n+1)}{3}-x-3)> n(n+1)$, or rearranging, $ 3x (The picture was semi-pornographic)\nI kick Teammate C and say, \"I'll work on the rest of the front page, you guys check my answers to the backpage.\"\nAfter I finished the front page, they just told me they didn't know how to do it but one of them did do #5. So I checked their answers and had to change #1 and #5.\nThen, I rechecked all the answers while they fooled around for the rest of the time.\n[/hide]\r\n\r\nIt turns out that we got 18/20 on the Team Round but placed 6th at Chapters anyway. I'll let the school's scores speak for themselves why in format of \r\nSprint Right/2*Target Right\r\nMe - 28/14\r\nTeammate A - 20/6\r\nTeammate B - 12/4\r\nTeammate C - 12/8\r\nKid from our school 1 - 12/10\r\nKid from our school 2 - 12/0" } { "Tag": [], "Problem": "what is the sum of all values of $ x$ for which $ |x\\minus{}3| \\plus{} |3\\minus{}x| \\minus{}1 \\equal{} 3$?", "Solution_1": "We have $ |x \\minus{} 3| \\plus{} |3 \\minus{} x| \\minus{} 1 \\equal{} 3\\implies2|x \\minus{} 3| \\equal{} 4\\implies|x \\minus{} 3| \\equal{} 2\\implies x \\equal{} 3\\pm2$\r\n$ \\implies x_1 \\plus{} x_2 \\equal{} \\boxed6$." } { "Tag": [ "floor function", "combinatorics unsolved", "combinatorics" ], "Problem": "How much permutations of (1,2....,n) we have that they satisfies:\r\n$s_{i}\\leq s_{\\lfloor{i/2}\\rfloor}$ for all $2\\leq i\\leq n$ \r\nwhere $s_{i}$ is the value of the $i$ position in the permutation.\r\nEx:\r\n$n=3$, $S=2 1 3$ so $s_{2}=1$\r\nthanks!", "Solution_1": "nobody??????\r\ntkz!", "Solution_2": "See http://www.research.att.com/~njas/sequences/A056971 or [url]http://mathworld.wolfram.com/Heap.html[/url]. A recurrence and proof is given at the first link." } { "Tag": [ "algebra", "polynomial", "AMC", "USA(J)MO", "USAMO" ], "Problem": "Prove that any monic polynomial of degree $ n$ with real coefficients is the average of two monic polynomials of degree $ n$ with $ n$ real roots.\r\n\r\nI'm not sure of the difficulty of this problem so I appologize if it isn't in the right forum. Thanks for the help.", "Solution_1": "[[2002 USAMO Problems/Problem 3]]", "Solution_2": "[url]http://www.artofproblemsolving.com/Forum/viewtopic.php?p=337849#337849[/url]\r\n\r\nAdditionally, please use a more descriptive name than \"Problem\" - there are plenty of \"problems\" posted each day." } { "Tag": [ "number theory unsolved", "number theory" ], "Problem": "Find all pairs of positive integers (x,y) such that:\r\n$x^2+y^2-5xy+5=0$", "Solution_1": "[b] Lemma :[/b] The equation $x^2+y^2-pxy+p = 0$ $(p \\in Z^+)$ has positive integer roots if and only if $p=5$ \r\n \r\n [i]Proof : [/i] \r\n Let $T$ be the set of all positive integer roots of the equation : $x^2+y^2-pxy+p=0$ (1) . \r\n Call $(x_0,y_0)$ be the smallest root of (1) . This means $x_0+y_0 \\leq x+y$ , $\\forall (x,y) \\in T$ \r\n Suppose $x_0 \\leq y_0$ \r\n If $x_0=y_0$ , then after some computations , we get a contradiction ! \r\n So $x_0 0$ \r\n So $(x_0,y_1)$ is also a root of (1) \r\n By the definition of $(x_0,y_0)$ , we infer $y_1 >y_0 > x_0$\r\n We have : \r\n $x_0^2+p-px_0 =y_1y_0 -y_1-y_0 = (y_1-1)(y_0-1)-1 \\geq x_0^2-1$ \r\n $\\iff 1 \\geq p(x_0-1)$ \r\n From here , we infer $x_0 =1 \\to y_1 =3 , y_0 =2$ and $p=5$ [/i]", "Solution_2": "All solution are x1=1;x2=2;x_n=5x_(n-1)-x_(n-2)" } { "Tag": [ "limit", "logarithms", "real analysis", "real analysis unsolved" ], "Problem": "Compute $\\lim_{n\\rightarrow\\infty}{\\frac{1}{(n!)^{\\frac{1}{n}}}}\\cdot\\log_{2}{\\left(1+\\frac{1}{2}+\\frac{1}{3}+...+\\frac{1}{2^{2^{n+1}}-1}\\right)}$", "Solution_1": "We have that $1+\\frac 1 2+\\frac 1 3+...+\\frac 1 n\\approx \\ln n$ and $\\frac{n}{\\sqrt[n]{n!}}\\approx e$. The limit is e.", "Solution_2": "Oh :blush: ! I'm not clear about the log sum. Can you explain a bit? Thank you.", "Solution_3": "i think u can get the answer here :D [url]http://www.mathlinks.ro/Forum/post-361063.html[/url]" } { "Tag": [], "Problem": "As you drive down the road at 13 m/s you press on the gas pedal and speed up with a uniform acceleration of 1.33 m/s2 for 0.62 s. If the tires on your car have a radius of 33 cm, what is their angular displacement during this period of acceleration?\r\nThe answer should be in rad.\r\n\r\nThanks for helping!", "Solution_1": "We have $ v_0\\equal{}13ms^{\\minus{}1},a\\equal{}1.33ms^{\\minus{}2},t\\equal{}0.62s$. Thus $ s\\equal{}v_0t\\plus{}\\frac{at^2}{2}$ from this we can find $ s$. If the angular displacement is $ \\theta$ we have that $ s\\equal{}r\\theta$. Thus we can find $ \\theta$. Sorry I'm in a hurry so skipped the calculations." } { "Tag": [ "probability", "function", "advanced fields", "advanced fields unsolved" ], "Problem": "I don't understand the difference between the Weak Law of Large Numbers and the Strong Law of Large Numbers. Can someone explain the difference? Also, why does the latter imply the former?\r\n\r\nWeak Law of Large Numbers:\r\n\\[\\mu_{L}(B_{N})\\rightarrow0\\qquad\\text{as}\\qquad N\\rightarrow\\infty\\]\r\nStrong Law of Large Numbers:\r\nLet $s_{n}(\\omega)$ be the number of heads in the sequence $\\omega$ of $n$ flips of a fair coin and $N=\\left\\{\\omega; \\frac{s_{n}(\\omega)}{n}\\rightarrow\\frac{1}{2}\\text{ as }n\\rightarrow\\infty\\right\\}$. Then $\\mu_{L}(N^{c})=0$.\r\n\r\n($\\mu_{L}(A)$ is the measure of $A$.)", "Solution_1": "The Strong Law of Large Numbers says that the sequence of random variables $S_{n}/n$ converges to $1/2$ [b]almost surely[/b]. The Weak Law of Large Numbers says that the sequence of random variables $S_{n}/n$ converges to $1/2$ [b]in probability[/b]. \r\n\r\nHere is an example that shows that the convergence in probability doesn't imply the convergence almost surely.\r\n\r\nLet $I_{[a,b]}: [0,1]\\to\\mathbb R$ be the indicator function for the segment $[a,b]$:\r\n\\[I_{[a,b]}(t) = \\begin{cases}1, & \\text{ if }t\\in[a,b],\\\\ 0, &\\text{ otherwise}. \\end{cases}\\]\r\nConsider the following sequence of functions\r\n\\[I_{[0,1]}, I_{[0,1/2]}, I_{[1/2,1]}, I_{[0,1/3]}, I_{[1/3,2/3]}, I_{[2/3,1]}, I_{[0,1/4]}, \\dots \\]\r\nProve that they converge to $0$ in probability (i.e. $\\mu(\\{t: f_{n}(t) > \\varepsilon\\}) \\to 0$ as $n\\to\\infty$ for every $\\varepsilon > 0$). However, these functions do [b]not[/b] converge to $0$ almost surely (in fact, for every $t_{0}\\in [0,1]$, $f_{n}(t_{0})$ doesn't tend to $0$).", "Solution_2": "In crude terms, the Strong Law tells you that the places where things screw up are the same for all of the functions and get small. On the other hand, the Weak Law tells you that the places where things screw up get small, but it doesn't tell you that they're the same places for each function (or random variable if you like). The bad places could be all over, like in Yury's example." } { "Tag": [ "probability", "quadratics", "function", "symmetry", "ratio", "conditional probability" ], "Problem": "For the November Team one.. \"Random...\"", "Solution_1": "The official contest dates run through the 18th; some teams will be taking it this coming weekend. All public discussion is off limits until at least then.", "Solution_2": "Can we discuss it now!? Only 20 minutes left... Not like any team can start after reading a post right now...", "Solution_3": "It's probably been long enough. All of the answers should be in the mail by now.\r\n\r\nI think our guys thought this one was difficult.", "Solution_4": "Having Adam on the team would've helped... it took us (Western Washington) a little too long to get a conceptual grip on this one :| Our solutions were all pretty rushed.", "Solution_5": "Problem 6a) and the argument for it was the key to nearly everything. Here's a set of short answers to check against:\r\n\r\n1. a) $ P(A) \\equal{} \\frac23,\\ P(B) \\equal{} \\frac13$\r\n\r\n1. b) $ P(A) \\equal{} \\frac34,\\ P(B) \\equal{} \\frac14$\r\n\r\n1. c) $ P(A) \\equal{} \\frac45,\\ P(B) \\equal{} \\frac15$\r\n\r\n2. $ P(A) \\equal{} \\frac {n}{n \\plus{} 1},\\ P(B) \\equal{} \\frac {1}{n \\plus{} 1}$\r\n\r\n3. a) $ P(A) \\equal{} \\frac23,\\ P(B) \\equal{} \\frac13$\r\n\r\n3. b) $ P(A) \\equal{} \\frac47,\\ P(B) \\equal{} \\frac37$\r\n\r\n4. $ P(A) \\equal{} \\frac {b}{a \\plus{} b},\\ P(B) \\equal{} \\frac {a}{a \\plus{} b}$\r\n\r\n5. a) $ P(A) \\equal{} \\frac12,\\ P(B) \\equal{} \\frac14,\\ P(C) \\equal{} \\frac14$\r\n\r\n5. b) $ P(A) \\equal{} \\frac15,\\ P(B) \\equal{} \\frac25,\\ P(C) \\equal{} \\frac25$\r\n\r\n5. c) $ P(A) \\equal{} \\frac13,\\ P(B) \\equal{} \\frac16,\\ P(C) \\equal{} \\frac13,\\ P(D) \\equal{} \\frac16$\r\n\r\n6. a) $ P(A_k) \\equal{} a_k^{ \\minus{} 1}\\left(\\sum_{i \\equal{} 1}^na_i^{ \\minus{} 1}\\right)^{ \\minus{} 1}$\r\n\r\n6. b) $ P(A) \\equal{} \\frac14,\\ P(B) \\equal{} \\frac38,\\ P(C) \\equal{} \\frac38$\r\n\r\n7. $ P(A) \\equal{} \\frac3{14},\\ P(B) \\equal{} \\frac {17}{28},\\ P(C) \\equal{} \\frac1{14},\\ P(D) \\equal{} \\frac3{28}$\r\n\r\n8. $ \\frac1{1007013}$\r\n\r\n9. $ P(A) \\equal{} \\frac5{19},\\ P(B) \\equal{} \\frac5{19},\\ P(C) \\equal{} \\frac6{19},\\ P(D) \\equal{} \\frac3{19}$", "Solution_6": "[quote=\"Kent Merryfield\"]8. $ \\frac1{1007013}$[/quote]\r\n\r\nI gave the answer in terms of $ k$. Did the problem imply it wanted the lowest probability among all $ k$? I didn't think it was clear...", "Solution_7": "Here's the exact statement:\r\n\r\n[quote]8. A tree has the same shape as the tree in Figure 6a. If $ a_1\\plus{}a_2\\plus{}a_3\\plus{}\\cdots\\plus{}a_k\\equal{}2007,$ what is the smallest possible value for $ A_1.$[/quote]\r\nYou're right to say that that's not totally clear, but to me it doesn't imply that $ k$ is fixed, and hence it does imply that we seek the overall minimum.\r\n\r\nFor a given $ k,$ the smallest $ A_1$ would be obtained by setting $ a_1$ as large as possible. That is, we let $ a_1\\equal{}2008\\minus{}k$ and we let the other $ k\\minus{}1$ branches each have length $ 1.$ By the result in 6a), we get \r\n\r\n$ P(A_1)\\equal{}\\frac{\\frac1{2008\\minus{}k}}{\\frac1{2008\\minus{}k}\\plus{}\\frac{k\\minus{}1}1}\\equal{}\r\n\\frac1{\\minus{}k^2\\plus{}2009k\\minus{}2007}$\r\n\r\nNow, for what $ k$ is this as small as possible? We can minimize the fraction by maximizing the denominator. The quadratic in the denominator would be maximized by letting $ k\\equal{}1004.5,$ but since $ k$ must be an integer, the actual extreme value is achieved by letting $ k$ be either $ 1004$ or $ 1005$ and is the value reported above.", "Solution_8": "Well, yes, once I got the expression for a given $ k$ it occurred to me that they might want me to minimize it, but even if I had decided on that as the correct expression, we ran out of time. :|", "Solution_9": "t0rajir0u - did you guys get that expression for #6a)? That's the \"master key\" to the set, as it covers every problem except #9.\r\n\r\nI'll have to look again at the copies I kept of what we sent, but I don't think we had that.", "Solution_10": "Yeah. It was pretty clear after we looked at the numbers on #7. I don't think we defended any of our answers particularly well, though.", "Solution_11": "An argument for 6a):\r\n\r\nWe have a tree with $ n$ edges connected to the root, and each of those $ n$ edges starts a straight branch that leads to a single leaf. We have leaves $ A_1,A_2,\\dots,A_n,$ and there are $ a_i$ edges between the root and $ a_i.$\r\n\r\nLet's focus on the probability that a random walk starting at the root ends at leaf $ A_1.$ Let $ p$ be that probability.\r\n\r\nLet $ x_{i,j}$ be the conditional probability of ending at leaf $ A_1$ given that the current location is the $ j$th node on the $ i$th branch. By convention, let $ x_{i,0} \\equal{} p$ for all $ i.$ (That is, think of the root as belonging to all of the branches.) And we will use that $ x_{1,a_1} \\equal{} 1$ and $ x_{i,a_i} \\equal{} 0$ for $ i\\ne0.$\r\n\r\nConsidering the results of one step leads us to the following equations:\r\n\r\nFor $ 1\\le j\\le a_i \\minus{} 1,\\ x_{i,j} \\equal{} \\frac12\\,x_{i,j \\minus{} 1} \\plus{} \\frac12\\,x_{i,j \\plus{} 1}$\r\n\r\n$ p \\equal{} \\frac1n\\sum_{i \\equal{} 1}^nx_{i,1}.$\r\n\r\nThe first equation can be rearranged to this:\r\n\r\n $ x_{i,j \\plus{} 1} \\minus{} x_{i,j} \\equal{} x_{i,j} \\minus{} x_{i,j \\minus{} 1}.$\r\n\r\nHence the successive differences are constant and, for each $ i,$ $ x_{i,j}$ is a linear function of $ j.$ (It's an arithmetic sequence.) Considering the boundary values, we get the following:\r\n\r\n$ x_{1,j} \\equal{} p \\plus{} \\frac {(1 \\minus{} p)j}{a_1}$\r\n\r\n$ x_{i,j} \\equal{} p \\minus{} \\frac {pj}{a_i}$ for $ i\\ne 1.$\r\n\r\nThat means that $ x_{1,1} \\equal{} p \\plus{} \\frac {1 \\minus{} p}{a_1} \\equal{} p\\left(1 \\minus{} \\frac1{a_1}\\right) \\plus{} \\frac1{a_1}$\r\n\r\nand $ x_{i,1} \\equal{} p \\minus{} \\frac {p}{a_i} \\equal{} p\\left(1 \\minus{} \\frac1{a_i}\\right)$ for $ i\\ne 1.$\r\n\r\nThe probability equation around the root node gives\r\n\r\n$ p \\equal{} \\frac1n\\left(\\sum_{i \\equal{} 1}^np\\left(1 \\minus{} \\frac1{a_i}\\right) \\plus{} \\frac1{a_1}\\right)$\r\n\r\n$ p \\equal{} p \\minus{} \\frac {p}{n}\\sum_{i \\equal{} 1}^n\\frac1{a_i} \\plus{} \\frac1{na_1}$\r\n\r\n$ \\frac {p}{n}\\sum_{i \\equal{} 1}^n\\frac1{a_i} \\equal{} \\frac1{na_1}$\r\n\r\n$ p\\sum_{i \\equal{} 1}^n\\frac1{a_i} \\equal{} \\frac1{a_1}$\r\n\r\nSo, $ p \\equal{} \\frac {\\frac1{a_1}}{\\sum_{i \\equal{} 1}^n\\frac1{a_i}}$\r\n\r\nSince there was nothing special about the first branch, we have our general answer:\r\n\\[ \\boxed{P(A_k) \\equal{} \\frac {\\frac1{a_k}}{\\sum_{i \\equal{} 1}^n\\frac1{a_i}}}\r\n\\]", "Solution_12": "Assuming I understand the problem correctly, does the following argument work?\r\n\r\nLet's assign a weight of 0 to the root and all vertices not on the first two branches. Let's assign a weight of $ \\minus{}1$, $ \\minus{}2$, ..., $ \\minus{}a_1$ to the vertices on the first branch (as we go away from the root). Let's assign a weight of $ 1$, $ 2$, ..., $ a_2$ to the vertices on the second branch (as we go away from the root). We can check that each step, the expected weight does not change. Thus, when we finish the random walk, the expected weight will be 0. (The tree being finite helps us here.)\r\n\r\nBut the expected weight at the end will be $ \\minus{}a_1 P(A_1) \\plus{} a_2 P(A_2)$. Because that expected weight is 0, we have $ a_1 P(A_1) \\equal{} a_2 P(A_2)$. By symmetry, all the $ a_k P(A_k)$ must equal each other. In other words, $ P(A_k)$ is inversely proportional to $ a_k$. Because the probabilities add up to 1, we get\r\n\\[ P(A_k) \\equal{} \\frac{a_k^{\\minus{}1}}{\\sum_i a_i^{\\minus{}1}} \\, .\r\n\\]", "Solution_13": "Ravi: you understood the problem correctly. And that's a very nice argument.\r\n\r\nLet me try something related with problem #9.\r\n\r\nDescription of the tree: The root is connected to node $ x$ on the left and node $ y$ on the right. Node $ x$ is further connected to leaves $ A$ and $ B.$ Node $ y$ is further connected to leaf $ C$ and node $ z.$ Node $ z$ is further connected to leaf $ D.$\r\n\r\nLet's assign weights to each node so as to keep the balance about each point. Let the root have weight $ 0.$ Assign weight $ 1$ to node $ x$ and $ \\minus{}1$ to node $ y.$\r\n\r\nNow to keep the balance about node $ x,$ the three outcomes must average to $ 1.$ To do that, assign weight $ \\frac32$ to each of leaves $ A$ and $ B.$\r\n\r\nWe do the same thing about node $ y.$ Leaf $ C$ has weight $ \\minus{}\\frac32$ and node $ z$ has weight $ \\minus{}\\frac32.$ And then to keep the balance about $ z,$ we assign weight $ \\minus{}2$ to leaf $ D.$\r\n\r\nThe expected weight at the end will be\r\n\r\n$ \\frac32(P(A)\\plus{}P(B))\\minus{}\\frac32P(C)\\minus{}2P(D)\\equal{}0.$\r\n\r\nThat's one equation for the four probabilities. We need two more (we always have that the sum of the probabilities is $ 1.$) By symmetry, $ P(A)\\equal{}P(B),$ so we need just one more.\r\n\r\nTry zeroing the whole $ x,A,B,$ branch. Assign weight $ 0$ to node $ y.$ Then assign weight $ 1$ to leaf $ C$ and weight $ \\minus{}1$ to node $ z.$ We get balance around $ z$ by assigning weight $ \\minus{}2$ to leaf $ D.$ We now have that\r\n\r\n$ P(C)\\minus{}2P(D)\\equal{}0$ or $ P(C)\\equal{}2P(D).$\r\n\r\nReplace $ P(B)$ by $ P(A)$ and $ P(C)$ by $ 2P(D)$ and return to our first equation:\r\n\r\n$ 3P(A)\\minus{}5P(D)\\equal{}0.$\r\n\r\nThus the ratio $ P(A): P(D)\\equal{}5: 3.$ More completely, $ P(A): P(B): P(C): P(D)\\equal{}5: 5: 6: 3.$ \r\n\r\nSince $ 5\\plus{}5\\plus{}6\\plus{}3\\equal{}19,$ that gives us probabilities of $ \\frac5{19},\\frac5{19},\\frac6{19},\\frac3{19}.$\r\n\r\n---\r\n\r\nIt looks like to make this work, we'll need to do several assignments of weights, selectively zeroing various parts of the tree. But the assignment of weights is not difficult; each assignment is strictly local.", "Solution_14": "[quote=\"t0rajir0u\"][quote=\"Kent Merryfield\"]8. $ \\frac1{1007013}$[/quote]\n\nI gave the answer in terms of $ k$. Did the problem imply it wanted the lowest probability among all $ k$? I didn't think it was clear...[/quote]\r\n\r\nD'oh!\r\n\r\nWell, I hope the grader understands the ambiguity in the problem statement. When I wrote this event (well over a year ago; I had to reacquaint myself with the problems), I must admit I overlooked that interpretation.\r\n\r\nSee, this is why I should have students proofread these before they're submitted.\r\n\r\n...waaait a second, that'd be a bad idea :P", "Solution_15": "[quote=\"Kent Merryfield\"]\n\nSince there was nothing special about the first branch, we have our general answer:\n\\[ \\boxed{P(A_k) \\equal{} \\frac {\\frac1{a_k}}{\\sum_{i \\equal{} 1}^n\\frac1{a_i}}}\n\\]\n[/quote]\r\n\r\nI like that approach! Mine was to reduce to a previous problem: note that if you want to compare $ P(A_r)$ to $ P(A_s)$ in a $ k$-endpoint graph, you can disregard whatever the walk does before it restricts itself to the $ 2$-endpoint graph from the earlier problems:\r\n\r\n\\[ P(A_r | A_r \\textrm{ or } A_s) \\equal{} \\frac{P(A_r)}{P(A_r \\textrm{ or } A_s)} \\quad \\textrm{and} \\quad P(A_s | A_r \\textrm{ or } A_s) \\equal{} \\frac{P(A_s)}{P(A_r \\textrm{ or } A_s)},\\]\r\nbut $ P(A_r | A_r \\textrm{ or } A_s) \\equal{} \\frac{1/a_r}{1/a_r\\plus{}1/a_s}$ and $ P(A_s | A_r \\textrm{ or } A_s) \\equal{} \\frac{1/a_s}{1/a_r\\plus{}1/a_s}$, since we can restrict every $ A_r$-or-$ A_s$ walk to the part contained only in those two branches -- any earlier back-and-forths on other branches don't affect whether we finish at $ A_r$ or $ A_s$. Comparing the conditional probabilities, we get $ P(A_r): P(A_s) \\equal{} a_s: a_r$.", "Solution_16": "That turns out to be quite a bit like Ravi's \"weights\" argument. Notice that he also cut down to two branches.", "Solution_17": "Tom Kilkelly emailed a solution set to all of the sponsors. I'll post it here.", "Solution_18": "If allowed, could you post the problems as well?" } { "Tag": [ "function", "AMC", "AIME", "number theory", "totient function" ], "Problem": "I was doing one of the Mock AIMEs and the problem 4 was: Find the remainder when $1+7+7^2+7^3+...+7^{2004}$ is divided by 1000. The solution used Euler's totient theorem that\r\n$n^{\\phi(p)}\\equiv1(mod p)$\r\nIs this common on the AIME?", "Solution_1": "I doubt any of the solutions for actual AIME problems would REQUIRE the use of the totient function; it is however a handy tool in problems such as that one.", "Solution_2": "Well how do you do that WITHOUT the totient function then?", "Solution_3": "[quote=\"WarpedKlown1335\"]Well how do you do that WITHOUT the totient function then?[/quote]\r\n\r\nWell that's why it was on a mock AIME instead of a real one =P.", "Solution_4": "Don't take the Mock AIME's too seriously. If you really want to prepare for the AIME, look at real AIME problems.\r\n\r\nOr just learn the toitent function.", "Solution_5": "[quote=\"WarpedKlown1335\"]Well how do you do that WITHOUT the totient function then?[/quote]\r\n\r\nWith more patience.\r\n\r\n[hide]We want to calculate $\\frac{7^{2005}-1}{6}\\bmod1000$, meaning we're gonna have to know $7^{2005}-1\\bmod 2000$. (We can't simply calculate mod 1000 and then divide by 6 because 6 and 1000 are not relatively prime.) So we start looking for patterns mod 2000:\n\n7^1=7\n7^2=49\n7^3=343\n7^4=2401=401 mod 2000\n\nThat 1 is kinda nice, so we keep it:\n\n7^8=401^2=801\n7^12=801*401=1201\n7^16=1201*401=1601\n7^20=1601*401=1 mod 2000.\n\nYay, that's really good. So $7^{2005}-1\\equiv(7^{20})^{100}\\cdot7^5-1\\equiv7^5-1\\equiv806\\equiv4806\\bmod2000$, so the answer is $\\frac{4806}{6}\\equiv801\\bmod1000$.[/hide]", "Solution_6": "If memory serves, there WAS a problem on last year's AIME that was most easily solved with the totient function (but not Euler's theorem.)\r\n\r\n\r\n8. A regular n-star is the union of n equal line segments P_1P_2, P_2P_3, ... , P_nP_1 in the plane such that the angles at P_i are all equal and the path P_1P_2...P_nP_1 turns counterclockwise through an angle less than 180 degrees at each vertex. There are no regular 3-stars, 4-stars or 6-stars, but there are two non-similar regular 7-stars. How many non-similar regular 1000-stars are there?" } { "Tag": [ "calculus", "derivative", "Euler", "LaTeX", "calculus computations" ], "Problem": "(x+x^.5)^.5\r\nsorry i dont have those math symbols :(", "Solution_1": "Let $f(x)=\\sqrt{x+\\sqrt x}$. You need to use the chain rule on this bad boy.\r\n\r\nSo, $f'(x)=\\frac{1}{2} ({x+\\sqrt x})^{-\\frac{1}{2}}\\frac{d}{dx}\\left(x+\\sqrt x \\right).$\r\nAnd hence, \r\n\r\n$f'(x)=\\frac{1}{2} ({x+\\sqrt x})^{-\\frac{1}{2}}(1+\\frac{1}{2}{x^{-\\frac{1}{2}}} ).$\r\n\r\nAnd I shall leave the simplification to you.", "Solution_2": "[quote=\"=s\"](x+x^.5)^.5\nsorry i dont have those math symbols :([/quote]\r\n\r\nyou want derivative of $(x+x^5)^5$\r\n\r\n=$5(1+5x^4)(x+x^5)^4$", "Solution_3": "dont u need to chain it again? since its x+x^.5 after the second step", "Solution_4": "Is this $()^5$ or $()^{.5}$? \r\n\r\nIf it is the latter, see my post. If it is the first, see Euler-Fermat's post.", "Solution_5": "[quote=\"Fermat -Euler\"][quote=\"=s\"](x+x^.5)^.5\nsorry i dont have those math symbols :([/quote]\n\nyou want derivative of $(x+x^5)^5$\n\n=$5(1+5x^4)(x+x^5)^4$[/quote]\r\nno is to the square root=^.5", "Solution_6": "[quote=\"=s\"][quote=\"Fermat -Euler\"][quote=\"=s\"](x+x^.5)^.5\nsorry i dont have those math symbols :([/quote]\n\nyou want derivative of $(x+x^5)^5$\n\n=$5(1+5x^4)(x+x^5)^4$[/quote]\nno is to the square root=^.5[/quote]\r\nsorry, i dont see it", "Solution_7": "k thx", "Solution_8": "[quote=\"=s\"]sorry i dont have those math symbols[/quote]\r\nIf you can post here at all, you do- the math symbols here are $\\LaTeX$, and they're built into the forum software. You only need to use appropriate codes between dollar signs." } { "Tag": [ "calculus", "integration", "geometry", "rectangle", "function", "real analysis", "real analysis unsolved" ], "Problem": "Prove that if R is an aligned rectangle and f is a continuous function on R, then f is integrable on R.", "Solution_1": "I'll assume this is a Riemann integral question, so we should be discussing Riemann sums, or upper sums and lower sums. I'll also assume that \"aligned\" means \"with sides parallel to the axes.\" That's a convenience in the proof but hardly a vital condition.\r\n\r\nSomewhere in the proof you're going to use that a continuous function on a compact set is uniformly continuous.", "Solution_2": "1. For $\\epsilon>0$ choose $\\delta$ as in the definition of uniform continuity. \r\n2. Prove that any two choices of sample points in a partition with norm $<\\delta$ result in double Riemann sums that differ by less than $\\epsilon\\,\\mathrm{area}(R)$. \r\n3. Given two partitions $P_{1},P_{2}$ with norm $<\\delta$, let $P$ be their common refinement.\r\n4. Observe that any Riemann sum for $P$ is between the upper and lower Darboux sums for $P_{1}$. [The upper/lower Darboux sum is the Riemann sum with sample points being points of maximum/minimum of $f$ within a subrectangle]. \r\n5. Conclude from 2 and 4 that the difference of Riemann sums for $P_{1}$ and $P$ is less than $\\epsilon\\mathrm{area}(R)$. Same conclusion for $P_{2}$ and $P$.\r\n6. From 5, the difference of Riemann sums for $P_{1}$ and $P_{2}$ is less than $2\\epsilon\\mathrm{area}(R)$.", "Solution_3": "Since f is continuous almost everywhere on R, f is Riemann integrable on R.\r\n\r\n :P :D" } { "Tag": [ "blogs", "Columbia" ], "Problem": "[b][size=150][color=red]PE[/color][color=white]RU[/color][color=red] 2007[/color][/size][/b]\r\n\r\n[b] PER1. Daniel Soncco\n PER2. Fernando Manrique\n PER3. Jossy Alva\n PER4. Cesar Cuenca\n PER5. Amilcar Velez\n PER6. Ricardo Ramos [/b]\r\n\r\nFelecitaciones a todos,\r\n\r\nBLog de Procesos Selectivos de PERU: http://selectivos-peru.blogspot.com/\r\n\r\nSaludos,", "Solution_1": "[color=blue]Acabo de enterarme de que Uruguay no asistir\u00e1 a la I.M.O. este a\u00f1o. Seg\u00fan se dice, ninguno de los posibles participantes alcanza el nivel requerido para esta competencia y, adem\u00e1s, Viet Nam queda muy lejos.\nPor lo tanto, ni se molestaron en realizar un examen de clasificaci\u00f3n.\n\nTriste noticia, considerando que aportaba materiales a algunos participantes para que pudieran mejorar su nivel\u2026[/color]", "Solution_2": "[size=150][b][color=yellow]VEN[/color][color=blue]EZU[/color][color=red]ELA[/color][/b][/size]\r\n\r\nComo nadie ha posteado la delegaci\u00f3n venezolana ac\u00e1 va:\r\n\r\n :arrow: [b]Sof\u00eda Taylor\n :arrow: Carmela Acevedo\n :arrow: Gilberto Urdaneta[/b]\r\n\r\nsaludos! :lol:", "Solution_3": "[color=blue]\u00a1Las sorpresas que depara la vida! Al final, seg\u00fan una fuente fidedigna, s\u00ed habr\u00e1 un examen de selecci\u00f3n para definir la delegaci\u00f3n uruguaya que participar\u00e1 en la I.M.O..\nCualquier cambio imprevisto avisar\u00e9.\n\n[b]P.D.:[/b] El examen clasificatorio de Paraguay ser\u00e1 ma\u00f1ana (26-5-2.007).[/color]", "Solution_4": "[color=blue]\u00a1Una buena noticia!\n\u00a1Es seguro que yo voy a Viet Nam!\n\nL\u00e1stima que, por culpa de un joven muy molesto (que casi me mata a golpes), corro el riesgo de no ir como Paraguay 1.\n\nCuando tenga la lista completa, la publicar\u00e9.\n\nSaludos a todos.[/color]", "Solution_5": "[quote=\"R.G.A.M.\"][color=blue]\u00a1Una buena noticia!\n\u00a1Es seguro que yo voy a Viet Nam!\n\nL\u00e1stima que, por culpa de un joven muy molesto (que casi me mata a golpes), corro el riesgo de no ir como Paraguay 1.\n\nCuando tenga la lista completa, la publicar\u00e9.\n\nSaludos a todos.[/color][/quote]\r\n\r\nq fue lo q te paso con ese muchacho rafael?? :rotfl:\r\n\r\nbueno hace varias semanas se hizo el selectivo y la delegacion salvadore\u00f1a para esta imo es:\r\n\r\nESA-1: Gabriel Alexander Chicas Reyes (ese soy yo :wink:)\r\nESA-2: Mario Jos\u00e9 Castro Hern\u00e1ndez\r\nESA-3: Alfonso Abraham Alvarenga Gamero\r\nESA-4: Otto Tang Chen\r\n\r\nespero q amigos q ya nos conocemos nos encontremos de nuevo en vietnam :thumbup:", "Solution_6": "[color=blue]\u00a1AFortunado eres de no conocerlo!\nHe sido uno de los menos afectados por su molestia, he comprobado.\nChicos que estaban a cinco metros de \u00e9l se quejaron al finalizar el examen, pues result\u00f3 ser DEMASIADO molesto.\n\u00a1El criminal se pasaba silbando y gritando porque no quer\u00eda decirle cu\u00e1ntos problemas ya ten\u00eda resueltos!\nHe comentado el problema, y espero que tomen medidas, sino\u2026 (supongo que no necesito aclarar qu\u00e9 suceder\u00e1)\n\n\u00a1Felicitaciones al equipo salvadore\u00f1o![/color]", "Solution_7": "Finalmente salio el equipo de colombia:\r\n\r\n1) Diego Cifuentes (IMO 2006, OIM 2006) \r\n2) Santiago Cuellar (IMO 2006, OCCM 2005) (Skuge en Mathlinks)\r\n3) Jos\u00e9 Alejandro Samper (Yo!!!) (IMO 2006, OIM 2005/2006)\r\n4) Miguel Acosta (IMO 2006, OCCM 2004)\r\n5) Jorge Olarte (OCCM 2006)\r\n6) Federico Castillo $^{2}$ (OIM 2005)", "Solution_8": "Luego de 10 a\u00f1os Chile vuelve a la IMO... esperemos que hagan una buena actuaci\u00f3n :blush: \r\n\r\n1) Francisco Campos\r\n2) Cristobal Parraguez\r\n3) Roberto Villaflor\r\n4) Sebastian Pacheco\r\n\r\nMucha suerte!!", "Solution_9": "[quote=\"Killua\"]Luego de 10 a\u00f1os Chile vuelve a la IMO... esperemos que hagan una buena actuaci\u00f3n :blush: \n\n1) Francisco Campos\n2) Cristobal Parraguez\n3) Roberto Villaflor\n4) Sebastian Pacheco\n\nMucha suerte!![/quote]\r\n\r\nBien por chile amigo mio,\r\nespero la delegacion pueda cumplir sus objetivos \r\nen esta edicion de la IMO.\r\n\r\n\r\nCarlosbr\r\nLima -PERU", "Solution_10": "[size=150][b][color=red]DEL[/color][color=white]EGA[/color][color=blue]CI\u00d3N[/color] [color=red]PAR[/color][color=white]AGU[/color][color=blue]AYA[/color][/b][/size]\r\n\r\n[b][i]Participantes:[/i][/b]\r\n1. Ratti Bittinger, Gabriela Mar\u00eda.\r\n2. Arias Michel, Rafael Guillermo.\r\n3. Cal\u00f3 Caligaris, Maurizio.\r\n4. Jacquet Montiel, Jorge Daniel.\r\n\r\n[b][i]Suplente:[/i][/b]\r\n1. Hazevich Mulka, Iv\u00e1n Alexander.\r\n\r\n\r\n[color=blue][b][i]Observaci\u00f3n:[/i][/b] \u00c9sta lista se present\u00f3 sin tener en cuenta el orden de los puntajes, debido a que el tiempo apremiaba. Seg\u00fan o\u00ed hablar, bien pod\u00eda merecer estar en la posici\u00f3n primera de la lista de participantes (a pesar del desastre\u2026).\n\n[b]P.D.:[/b] Ma\u00f1ana es el examen de clasificaci\u00f3n para la Olimpiada \u00cdberoamericana de Matem\u00e1ticas en mi pa\u00eds.[/color]" } { "Tag": [ "trigonometry" ], "Problem": "All in the title\r\n$ \\displaystyle \\frac{\\tan x}{1\\minus{}\\cot x} \\plus{} \\frac{\\cot x}{1\\minus{}\\tan x} \\equal{} \\sec x \\csc x \\plus{}1$\r\n\r\nMy solution is rather long so if anyone could do it in less than 10+ steps that would be great.", "Solution_1": "[hide=\"Discussion\"] A general strategy for identities of this type is to write everything in terms of sines and cosines. I'm also going to do this proof \"forward,\" which is unmotivated (run the steps backwards to see the reasoning): \n\n$ \\frac{\\sin^3 x \\minus{} \\cos^3 x}{\\sin x \\cos x (\\sin x \\minus{} \\cos x)} \\equal{} \\frac{\\sin^2 x \\plus{} \\sin x \\cos x \\plus{} \\cos^2 x}{\\sin x \\cos x} \\implies$\n$ \\frac{\\sin^2 x}{\\cos x (\\sin x \\minus{} \\cos x)} \\plus{} \\frac{\\cos^2 x}{\\sin x(\\cos x \\minus{} \\sin x)} \\equal{} \\frac{1 \\plus{} \\sin x \\cos x}{\\sin x \\cos x} \\implies$\n$ \\frac{\\tan x}{1 \\minus{} \\cot x} \\plus{} \\frac{\\cot x}{1 \\minus{} \\tan x} \\equal{} \\sec x \\csc x \\plus{} 1$\n\nTwo steps okay? :) [/hide]", "Solution_2": "Awesome . This was a porblem we had to do in class and here's my overly detailed (as in every step) solution. From a comparison between t0rajir0u's abridged solution and mine I think it's quite obvious that some methods are better than others :( (sigh...all that time wasted time I spent coming up with this...)\r\n\r\n[hide]\n\\[ \\frac {\\tan x}{1 - \\cot x} + \\frac {\\cot x}{1 - \\tan x} = \\frac {(1 - \\tan x)\\tan x + (1 - \\cot x)\\cot x}{(1 - \\cot x)(1 - \\tan x)}\n\\]\n\n\\[ (1 - \\cot x)(1 - \\tan x) = 1 - \\cot x - \\tan x + \\cot x \\tan x = 2 - \\cot x - \\tan x\n\\]\n\n\\[ \\frac {(1 - \\tan x)\\tan x + (1 - \\cot x)\\cot x}{2 - \\cot x - \\tan x} = \\frac {\\tan x + \\cot x - (\\tan^2 x + \\cot^2 x)}{2 - (\\cot x + \\tan x)}\n\\]\n\n\\[ \\tan x + \\cot x = \\frac {\\sin x }{\\cos x} + \\frac {\\cos x}{\\sin x} = \\frac {\\sin^2 x + \\cos^2 x}{\\cos x \\sin x} = \\sec x \\csc x\n\\]\n\n\\[ \\tan^2 x + \\cot^2 x = \\frac {\\sin^2 x}{\\cos^2 x} + \\frac {\\cos^x}{\\sin^2 x} = \\frac {1 - \\cos^2 x}{\\cos^2 x} + \\frac {1 - \\sin^2 x}{\\sin^2 x} = \\frac {1}{\\cos^2 x} - \\frac {\\cos^2 x}{\\cos^2 x} + \\frac {1}{\\sin^2 x} - \\frac {\\sin^2 x}{\\sin^2 x} \\\\\n= \\sec^2 x + \\csc^2 x - 2\n\\]\n\n\\[ \\sec^2 x + \\csc^2 x = \\frac {1}{\\cos^2 x} + \\frac {1}{\\sin^2x} = \\frac {\\sin^2 x + \\cos^2 x}{\\sin^2 x \\cos^2 x} = \\sec^2 x \\csc^2 x\n\\]\n\n\\[ \\tan^2 x + \\cot^2 x = \\sec^2 x \\csc^2 x - 2\n\\]\n\n\\[ \\frac {\\tan x + \\cot x - (\\tan^2 x + \\cot^2 x)}{2 - (\\cot x + \\tan x)} = \\frac {\\sec x \\csc x - \\sec^2 x \\csc^2 x + 2}{2 - \\sec x \\csc x}\n\\]\n\n\\[ \\frac { - 1}{ - 1} \\cdot \\frac {\\sec x \\csc x - \\sec^2 x \\csc^2 x + 2}{2 - \\sec x \\csc x} = \\frac {\\sec^2 x \\csc^2 x - \\sec x \\csc x - 2}{2 - \\sec x \\csc x}\n\\]\n\n\\[ \\frac {\\sec^2 x \\csc^2 x - \\sec x \\csc x - 2}{2 - \\sec x \\csc x} = \\frac {(\\sec x \\csc x - 2)(\\sec x \\csc x + 1)}{\\sec x \\csc x - 2} = \\sec x \\csc x + 1\n\\]\n[/hide]", "Solution_3": "Well, the first reason that solution's longer than it has to is that $ 1 \\minus{} \\cot x$ and $ 1 \\minus{} \\tan x$ both have a common \"factor\" $ \\sin x \\minus{} \\cos x$. When you write everything in terms of sines and cosines, this becomes much clearer, so you don't have to put as much effort into adding the fractions on the left :)", "Solution_4": "[quote=\"emjay285\"]All in the title\n$ \\displaystyle \\frac {\\tan x}{1 \\minus{} \\cot x} \\plus{} \\frac {\\cot x}{1 \\minus{} \\tan x} \\equal{} \\sec x \\csc x \\plus{} 1$\n\nMy solution is rather long so if anyone could do it in less than 10+ steps that would be great.[/quote]\r\n\r\n$ \\frac{a}{1\\minus{}a^{\\minus{}1}}\\plus{}\\frac{a^{\\minus{}1}}{1\\minus{}a}\\equal{}1\\plus{}a\\plus{}a^{\\minus{}1}$ and $ \\tan x\\plus{}\\cot x\\equal{}(\\sin^2 x\\plus{}\\cos^2 x)\\cdot (\\sec x\\csc x)\\equal{}\\sec x\\csc x$. Using $ a\\equal{}\\tan x$ finishes the problem." } { "Tag": [ "modular arithmetic", "combinatorics proposed", "combinatorics" ], "Problem": "Let $ f : \\mathbb{N} \\to \\mathbb{N}$ be a bijection. Let $ \\text{Fix}(f)$ denote the number of fixed points of $ f$ if it is finite and let $ s_n \\equal{} \\text{Fix}(f^n)$. Show that there does not exist $ f$ such that $ s_n \\equal{} \\varphi(n)$.\r\n\r\nWhat if $ f$ is not required to be a bijection?", "Solution_1": "$ Fix(f^p) \\equiv Fix(f) \\pmod p$ for any prime $ p$ - any fixed point of $ f^p$ is either a fixed point of $ f$ or a number in a cycle of length $ p$ in $ f$, and then there are $ p \\minus{} 1$ more members who also become fixed.\r\nBut it trivializes the problem [if we put $ n \\equal{} 1,3$ for example], so I might be wrong....", "Solution_2": "No, that's correct; this wasn't supposed to be hard, I just thought it was cute :) What I was thinking was that if $ c_k$ denoted the number of cycles of length $ k$ then $ s_n \\equal{} \\sum_{d | n} dc_d$, but the Mobius inverse of $ \\varphi(n)$ isn't non-negative. But your argument just takes the case $ n \\equal{} p$, which makes it even easier than I thought it was." } { "Tag": [ "trigonometry", "geometry unsolved", "geometry" ], "Problem": "let a, b, c sides triangle that a+b=3c and $\\alpha$ and $\\beta$ are the opposite angle to the sides a and b. show that\r\n$tan\\left(\\frac{\\alpha}{2}\\right)\\cdot tan\\left(\\frac{\\beta}{2}\\right)=\\frac{1}{2}$", "Solution_1": "We have $ \\tan\\frac{\\alpha}{2}=\\sqrt{\\frac{(p-b)(p-c)}{p(p-a)}}$ and $ \\tan\\frac{\\beta}{2}=\\sqrt{\\frac{(p-a)(p-c)}{p(p-b)}}$, where $p$ is the semiperimeter of triangle.\r\nSo $ \\tan\\frac{\\alpha}{2}\\tan\\frac{\\beta}{2}=\\frac{p-c}{p}$, but $a+b+c=4c\\Rightarrow p=2c$.\r\nThen $ \\tan\\frac{\\alpha}{2}\\tan\\frac{\\beta}{2}=\\frac{2c-c}{2c}=\\frac{1}{2}$.", "Solution_2": "$\\tan(\\frac{\\alpha}{2})\\tan(\\frac{\\beta}{2})$\r\n$\\tan(\\frac{\\alpha}{2})=\\frac{r}{s-a}$ where $s$ is the semiperimeter in fact $s=\\frac{a+b+c}{2}=\\frac{3c+c}{2}=2c$. $\\tan(\\frac{\\beta}{2})=\\frac{r}{s-b}$. It's known $r=\\sqrt{\\frac{(s-a)(s-b)(s-c)}{s}}$. Moving members from the LHS to the RHS, we only need to prove that: $r^{2}=\\frac{(s-a)(s-b)}{2}$ Substituting yields $P.D.$ that $r^{2}=\\frac{(s-a)(s-b)(s-c)}{s}$ be $\\frac{(s-a)(s-b)}{2}$ so we only need that $\\frac{s-c}{s}=\\frac{1}{2}$ but $s=2c$, so it's true.", "Solution_3": "we well-know that $tan\\frac{A}{2}=\\frac{r}{u-a},tan\\frac{B}{2}=\\frac{r}{u-b}$.So, \r\n$tan\\frac{A}{2}.tan\\frac{B}{2}=\\frac{r^{2}}{(u-a)(u-b)}=\\frac{u.u.(u-c)r^{2}}{u.u.(u-a)(u-b)(u-c)}=\\frac{u-c}{u}=\\frac12$\r\nLokman G\u00d6K\u00c7E" } { "Tag": [ "number theory open", "number theory" ], "Problem": "Can you solve this problem?\r\nFind integers a,b,c,n,m such that $a^m+ a^n=b^2+c^2$?", "Solution_1": "A solution: a=5, b=11, c=3, n=1, m=3.\r\n\r\nLet a = u 2 + v 2 , choose n and k arbitrarily (with n>0,k >= 0), and let m = n+2k. Then let b + ci = (u+vi)^n*(a^k+i) where i = :sqrt: (-1).\r\n\r\nAnother example: a=73, b=41,c=61,m=1,n=2" } { "Tag": [], "Problem": "Two hundred students participated in a mathematical contest. They had 6 problems to solve. It is known that each problem was correctly solved by at least 120 participants. Prove that there must be two participants such that every problem was solved by at least one of these two students.", "Solution_1": "graph theory surely permit to solve this problem easily.though...", "Solution_2": "If one student has all of them, or 5/6, it's trivial.\r\n\r\nIf one got 4/6, then the other 199 will have 120 correct solutions among them for each of the other two problems, so there are many pupils with the other 2 correct.\r\n\r\nIf none of them got 4/6, the maximum score is 3/6, so the average cannot be above 50% -- but the average was 60%+ for each question. Contradiction.\r\n\r\nI may be wrong, but I think that the number 200 is still far from optimized :?\r\nVery easy question in any way :)" } { "Tag": [ "linear algebra", "matrix" ], "Problem": "1)Let $ A,B,C \\in GL_n{\\mathbb{(C)}}$ are Hermitian matrices ans such that :\r\n $ A^3B^5 \\equal{} B^5A^3$,$ (A^{2n \\plus{} 1} \\plus{} B^{2n \\plus{} 1} \\plus{} C^{2n \\plus{} 1})(A^{ \\minus{} (2n \\plus{} 1)} \\plus{} B^{ \\minus{} (2n \\plus{} 1)} \\plus{} C^{ \\minus{} (2n \\plus{} 1)}) \\equal{} I_n$,$ A^{2m \\plus{} 1} \\plus{} B^{2m \\plus{} 1} \\plus{} C^{2m \\plus{} 1} \\equal{} 2009I_n$ with $ m,n,k \\in \\mathbb{N^{*}}$\r\nCaculate $ P \\equal{} A^{2k \\plus{} 1} \\plus{} B^{2k \\plus{} 1} \\plus{} C^{2k \\plus{} 1}$\r\n [hide=\"Answer\"]$ \\sqrt [2m \\plus{} 1]{2009^{2k \\plus{} 1}}I_n$[/hide]\n2) Let $ a,b,c$ are real numbers and $ X,Y,Z \\in GL_n{\\mathbb{(C)}}$ are Hermitian matrices satisfying the conditions \n $ aX^{2009} \\equal{} bY^{2009} \\equal{} cZ^{2009}$, $ XY \\plus{} YZ \\plus{} ZX \\equal{} XYZ$\n Calculate $ P \\equal{} aX^{2008} \\plus{} bY^{2008} \\plus{} cZ^{2008}$\n[hide=\"Answer of me\"]$ (\\sqrt [2009]{a} \\plus{} \\sqrt [2009]{b} \\plus{} \\sqrt [2009]{c})^{2009}I_n$[/hide]", "Solution_1": "1)\r\n$ A^3,B^5$ commute and $ 3,5$ are odd then $ A,B$ commute and we may assume $ A\\equal{}diag(\\lambda_i),B\\equal{}diag(\\mu_i)$ where $ \\lambda_i,\\mu_i$ are non zero reals. Thus $ C^{2m\\plus{}1}$ is a diagonal matrix and $ C$ is also diagonal. We have reduced the problem to the case $ n\\equal{}1$. \r\n :mad: \r\n2)\r\nAssume $ a,b,c$ non all zero ; $ X,Y,Z$ invertible implies $ a,b,c\\not\\equal{}0$. $ 2009$ is odd then $ Y\\equal{}\\alpha{X},Z\\equal{}\\beta{X}$ and the second relation can be written $ (\\alpha\\plus{}\\beta\\plus{}\\alpha\\beta)X^2\\equal{}\\alpha\\beta{X^3}$. $ X$ is invertible then $ X$ is a scalar matrix.........\r\n re :mad:" } { "Tag": [ "trigonometry", "algebra proposed", "algebra" ], "Problem": "Prove that among any $ 7$ real numbers there exist two, say $ x$ and $ y$, such that: $ 0 \\le \\frac{x\\minus{}y}{1\\plus{}xy} \\le \\frac{1}{\\sqrt{3}}$.", "Solution_1": "[quote=\"moldovan\"]Prove that among any $ 7$ real numbers there exist two, say $ x$ and $ y$, such that: $ 0 \\le \\frac {x \\minus{} y}{1 \\plus{} xy} \\le \\frac {1}{\\sqrt {3}}$.[/quote]\r\n\r\nLet $ x\\equal{}\\tan a, y\\equal{}\\tan b$, where $ a,b \\in (\\minus{} \\frac{\\pi}{2}, \\frac{\\pi}{2})$, the question becomes the existance of $ a,b$ such that $ 0 \\le tan(a\\minus{}b) \\le \\frac{1}{\\sqrt{3}}) \\Leftrightarrow$ existance of $ a,b$ such that $ |a\\minus{}b| \\le \\frac{\\pi}{6}$ which is true by dividing $ (\\minus{} \\frac{\\pi}{2}, \\frac{\\pi}{2})$ into six sub-intervals : $ (\\minus{} \\frac{\\pi}{2}, \\minus{}\\frac{\\pi}{3}],[\\minus{} \\frac{\\pi}{3}, \\minus{}\\frac{\\pi}{6}],[\\minus{} \\frac{\\pi}{6}, 0],[0, \\frac{\\pi}{6}],[ \\frac{\\pi}{6}, \\frac{\\pi}{3}],[\\frac{\\pi}{3}, \\frac{\\pi}{2})$ and use pigeon-hole principle.", "Solution_2": "exactly exist a,b, suth that $ 0\\le tan(a\\minus{}b)\\le tan\\frac{\\pi}{7}$.", "Solution_3": "Nice solution. It is a good example for the using of the pigeon-hole principle." } { "Tag": [ "calculus", "calculus computations" ], "Problem": "A highway patrol plane flies 3 miles above a level, straight road at a constant rate of 120 mph. The pilot sees an oncoming car and with radar determines that at the instant the line-of-sight distance from plane to car is 5 miles the line-of-sight distance is decreasing at the rate of 160 mph. Find the car's speed along the highway.\r\n\r\nMy trouble is that I don't see a clear way to relate the speeds of the plane and the car.", "Solution_1": "Let $ L$ be the horizontal distance from the plane to the car, and let $ h$ be the vertical distance. Let $ z$ be the line-of-sight distance. Then we know that\r\n\\[ L^2 \\plus{} h^2 \\equal{} z^2.\\]\r\nWe are given that at this instance, $ h \\equal{} 3$ and $ z \\equal{} 5$. Then $ L \\equal{} 4$. Differentiating with respect to time, we get\r\n\\[ L \\dfrac{dL}{dt} \\plus{} h \\dfrac{dh}{dt} \\equal{} z \\dfrac{dz}{dt}\\]\r\nThe rest should be simple calculations.", "Solution_2": "Sorry, but I don't quite understand how to substitute the variables back into the equation. Isn't $ \\frac{dh}{dt}$ the change in height? If so, then isn't that just 0?", "Solution_3": "Yes -- what's the problem with that?", "Solution_4": "from there we have $ 4x\\plus{}0h\\equal{} 5(\\minus{}160),$$ x\\equal{}\\frac{\\minus{}800}{4}\\equal{}\\minus{}200$\r\nThe car is approaching the helicopter at 200mph" } { "Tag": [ "inequalities", "number theory open", "number theory" ], "Problem": "Find the smallest $n$ such that the following inequality is false: $\\sigma(n) \\le H_{n}+\\ln(H_{n})e^{H_{n}}$ \r\nwhere $\\sigma(n)$ is the sum of the divisors of $n$ and $H_{n}$ is the $1+\\frac12+\\frac13+\\frac14+...+\\frac1n$.\r\n\r\n[color=red]Edit by Megus: I've moved your topic to a proper subforum :D [/color]", "Solution_1": "See http://www.mathlinks.ro/Forum/viewtopic.php?t=35163, so its equivalent to Riemann's hypothesis (and you should not expect a counterexample)." } { "Tag": [], "Problem": "fie $ABC$ un triunghi si $\\omega$ cercul sau inscris, cu centrul in $I$. dreapta $l$ este tangenta la $\\omega$ si taie $BC,CA,AB$ in $P,Q,R$. fie $L$ punctul de intersectie al dreptelor $BQ$ si $CR$.\r\nsa se arate ca $IL\\bot AP$.", "Solution_1": "$IL \\perp AP \\Longleftrightarrow AP$ este polara lui $L$ fata de cerc , deci $A \\in$ polara lui $L$ si $P \\in$ polara lui $L \\Longleftrightarrow L \\in$ polara lui $A$, $L \\in$ polara lui $P$.\r\n\r\nacum fie $D,E,F,H$ punctele de tangenta ale cercului cu $BC,CA,RQ,AB$, deci polara lui $A$ este $EH$, si polara lui $P$ este $DF$, dar $L$ fiind intersectia diagonalelor patrulaterului circumscriptibil $BCQR$, rezulta conform teoremei lui newton , ca $L \\in EH \\cap DF$.", "Solution_2": "@pohoatza: Nu vad de ce ai folosit primul $\\Longleftrightarrow$...eu zic ca e adevarat doar intr-un sens. In rest solutia e buna :) \r\nAcum s-a dat o solutie cu polare. Problema iese destul de repede si cu complexe ( in caz ca te plictisesti prea rau :wink: ). Exista si o solutie pur sintetica, fara polare? (bine, e evident ca faza cu polare se poate traduce sintetic dar ma gandeam la alta metoda :) )", "Solution_3": ":maybe: , eh, ma rog , nu asta conta , am scris in graba.", "Solution_4": "Data viitoare sa nu te mai grabesti. Nu de alta, dar imi este teama sa nu-ti faci o obisnuinta." } { "Tag": [ "algebra unsolved", "algebra" ], "Problem": "P(x)Q(x+1)=P(x+100)Q(x)", "Solution_1": "yes, and...?", "Solution_2": "[quote=pluristiq]yes, and...?[/quote]\nFind all such a polynomial.\nCoefficients are real by the way.", "Solution_3": "Multiply bith sides by\n$(P\\left(x+1 \\right))(P\\left(x+2 \\right))\\dots(P\\left(x+99 \\right))$" } { "Tag": [], "Problem": "who will win?\r\n\r\nyou pick, see if youre right!:lol:", "Solution_1": "[b]guess federer[/b], i don't follow tennis :D", "Solution_2": "Go Rafael Nadal!! :)", "Solution_3": "I pick Nadal. :)", "Solution_4": "Through a billion days after it is already over,\r\n\r\n(1) Rafael Nadal ESP def. (2) Roger Federer SUI, 7-5 3-6 7-6(3) 3-6 6-2.\r\n\r\nYes, I memed that.", "Solution_5": "Necro...", "Solution_6": "Rafael Nadal is so much more beastlier than Roger Federer", "Solution_7": "Yeah but Shoujit Banerjee can pwn them all =O", "Solution_8": "[quote=\"yaofan\"]Rafael Nadal is so much more beastlier than Roger Federer[/quote]\r\n\r\nFederer has had better days. The outcome was expected.", "Solution_9": "both me and rafael are left handed.\r\n\r\nI already know who won.", "Solution_10": "SCREW ALL OF YOU. TIGER WOODS WILL WIN.", "Solution_11": "Obviously you know a lot about tennis. :wink: \r\n\r\nBTW, Federer got married...", "Solution_12": "THAT'S WHAT SHE SAID" } { "Tag": [ "function", "real analysis", "real analysis unsolved" ], "Problem": "Function $g: R^{n}\\rightarrow R$ is concace. \r\n\r\nFind the condition for function $f: R\\rightarrow R$ under which composite function $f(g((x))$ is also concave.", "Solution_1": "$f$ is concave and increasing?\r\n$f(g(\\lambda x+(1-\\lambda)y))\\gneq f(\\lambda g(x)+(1-\\lambda)g(y))\\gneq \\lambda f(g(x))+(1-\\lambda)f(g(y))$", "Solution_2": "That can be one condition of this problem. \r\n\r\nBut is it represent all condition? \r\n\r\ni.e. $g(x)=x^{\\frac{1}{4}}, f(x) = x^{2}$\r\n\r\nthen, although $f(x)$ is convex, $f(g(x))$ is concave.", "Solution_3": "I don't know what does your \"all condition\" mean.\r\nBesides, $f\\circ g$ may be not concave for some concave function $g$, e.g. $x^{\\frac34}$\r\nIf $f\\circ g$ is concave for any concave function $g$, then $f$ must be concave and increasing.\r\n1. Take $g(x)=x$, then $f$ is concave.\r\n2. $f$ must be continuous. Should $f$ be not increasing. Try to prove that there is $p\\in\\mathbb{R}$ such that $f$ is increasing on $(-\\infty, p)$ and decreasing on $(p, \\infty)$ (may not be strictly but by no means it could be increasing by assumption). I believe you can find a $g$ such that $f\\circ g$ is not concave.", "Solution_4": "Oh, yes.. you're right. I miss understood your comment. \r\n\r\nAt first, I wanted to find a nice condition that can define $f$ according to \r\n\r\nthe specific form of $g$. Now, however, I realized that it's impossible to \r\n\r\nfind that condition. So, as you said, problem must be changed to \r\n\r\n\"\" for all concave function $g$\"\" and then $f$ must be increasing and \r\n\r\nconcave function. \r\n\r\nThanks for your comment, and I'm sorry to my strange problem. ^^" } { "Tag": [ "analytic geometry", "geometry solved", "geometry" ], "Problem": "let $ABCD$ be a quadrilateral such that length of $AB,BC,CD,AD,AC,BD$ are positive integer numbers.prove that one of these length is $3t$. :D", "Solution_1": "We will denote $BD, AB, AD, DC, CB$ as $m, n, p, r, q$. Putting coordinates so that $B(0,0), D(m,0)$ is easy to find the length $AC$:\r\n\\[ n^2-\\frac{(m^2+n^2-p^2)^2}{4m^2} +q^2-\\frac{(m^2+q^2-r^2)^2}{4m^2} +2\\sqrt{n^2-\\frac{(m^2+n^2-p^2)^2}{4m^2}}\\sqrt{q^2-\\frac{(m^2+q^2-r^2)^2}{4m^2}} \\]\r\nNow suppose that none of $m, n, p, r, q$ is divisible by 3. As $1^2\\equiv 1 (\\mod 3)$ and $2^2\\equiv 3(\\mod 3)$,\r\nand $\\mathbb{Z}/3\\mathbb{Z}$ is a field, is inmediate that the sum of the terms outside the square roots is\r\n$\\equiv 0(\\mod 3)$, and the terms inside the root are also $\\equiv 0(\\mod 3)$, and being the result and integer,\r\nit is $\\equiv 0(\\mod 3)$.\r\n\r\nQED", "Solution_2": "nice solution i relly enjoed it.good job.thanks. :D", "Solution_3": "See also http://www.mathlinks.ro/Forum/viewtopic.php?t=16468 post #14 problem [b]a)[/b].\r\n\r\n darij", "Solution_4": "thanks darij. :D" } { "Tag": [], "Problem": "Just for people beginning modular stuff, here's a couple of easy proofs to do. All of these assume that n is a positive integer.\r\n\r\nProve that the sum formulas will yield integers:\r\nn(n+1)/2\r\n(2n+1)(n+1)n/6\r\n\r\nAnd a tad harder:\r\n(n^2)(n^2-1) will always be divisible by 24 for n>2", "Solution_1": "First one is really easy, n :^2: and n are either both odd or both even, thefore, their sum is even.", "Solution_2": "Demon wrote:First one is really easy, n :^2: and n are either both odd or both even, thefore, their sum is even.\n\n\n\nOr you could just note that [hide]either n is even or n+1 is even[/hide]", "Solution_3": "[quote=\"magnara\"](n^2)(n^2-1) will always be divisible by 24 for n>2[/quote]\r\n\r\nAre you sure? What about n=6? (For it to be divisible by 24, you need three factors of 2 and one factor of 3.. if n is even, but not a multiple of 4, you only have two factors of 2 in the product (n^2)(n-1)(n+1) )", "Solution_4": "You need 4 consecutive integers (which makes it (n^2 + 2n)(n^2 - 1) or some variant) to ensure the product is divisible by 24.", "Solution_5": "Yeah, let's just change that n :^2: into an n :^3: . I was trying to remember an old problem and did so very poorly.", "Solution_6": "I'll try the second part: (2n+1)(n+1)n/6\n\n[hide]\n\nfirst let a mod 6 :equiv: 0\n\n\n\nif n mod 6 :equiv: a mod 6 then n/6 is an integer\n\n\n\nif n mod 6 :equiv: a mod 6 +1 then (2(a +1)+1)(a+1+1)(a+1)/6=(2a+3)(a+2)(a+1)/6 (2a+3) mod 6 :equiv: 3 (a+2) mod 6 :equiv: 2 a multiple of 2 and of 3 so you have a multiple 6\n\n\n\nif n mod 6 :equiv: a mod 6 +2 then (2(a+2)+1)(a+2+1)(a+2)/6=(2a+5)(a+3)(a+2)/6 (a+3) mod 6 :equiv: 3 and (a+2) mod 6 :equiv: 2 again you have a multiple of 6\n\n\n\nif n mod 6 :equiv: a mod 6+3 then (2a+7)(a+4)(a+3)/6 4*3 mod 6 :equiv: 0 so it's divisable\n\n\n\nif n mod 6 :equiv: a mod 6 +4 then (2a+9)(a+5)(a+4)/6 (2a+9) mod 6 :equiv: 3 and (a+4) mod 6 :equiv: 2 so it's divisable\n\n\n\nif n mod 6 :equiv: a mod 6+5 then (2a+11)(a+6)(a+5)/6 this one's the simplest because (a+6) mod 6 :equiv: 0\n\n[/hide]\n\nThis (AoPS) is my first experence with algabraic proofs any critique on my format/method would be appreciated.", "Solution_7": "Good - though it's not necessary to consider n mod 6. Try breaking 6 up into 2 times 3, then considering n mod 2 and n mod 3 separately.", "Solution_8": "NAS wrote:I'll try the second part: (2n+1)(n+1)n/6\n[hide]\nfirst let a mod 6 :equiv: 0\n\nif n mod 6 :equiv: a mod 6 then n/6 is an integer\n\nif n mod 6 :equiv: a mod 6 +1 then (2(a +1)+1)(a+1+1)(a+1)/6=(2a+3)(a+2)(a+1)/6 (2a+3) mod 6 :equiv: 3 (a+2) mod 6 :equiv: 2 a multiple of 2 and of 3 so you have a multiple 6\n\nif n mod 6 :equiv: a mod 6 +2 then (2(a+2)+1)(a+2+1)(a+2)/6=(2a+5)(a+3)(a+2)/6 (a+3) mod 6 :equiv: 3 and (a+2) mod 6 :equiv: 2 again you have a multiple of 6\n\nif n mod 6 :equiv: a mod 6+3 then (2a+7)(a+4)(a+3)/6 4*3 mod 6 :equiv: 0 so it's divisable\n\nif n mod 6 :equiv: a mod 6 +4 then (2a+9)(a+5)(a+4)/6 (2a+9) mod 6 :equiv: 3 and (a+4) mod 6 :equiv: 2 so it's divisable\n\nif n mod 6 :equiv: a mod 6+5 then (2a+11)(a+6)(a+5)/6 this one's the simplest because (a+6) mod 6 :equiv: 0\n[/hide]\nThis (AoPS) is my first experence with algabraic proofs any critique on my format/method would be appreciated.\n\n\n\nAs Osiris said, you can work mod 2 and 3. However lets decide you want to use this mod 6 method, thats fine. But half of the thing about mods is that it reduces the algebra you need to do. Here you don't need to use the letter a at all. As you said, a is 0 mod 6, so you can just use the number 0 wherever a is. \n\n\n\nFor example, taking one of your lines:\n\n[hide]\n\nif n mod 6 :equiv: a mod 6 +1 then (2(a +1)+1)(a+1+1)(a+1)/6=(2a+3)(a+2)(a+1)/6 (2a+3) mod 6 :equiv: 3 (a+2) mod 6 :equiv: 2 a multiple of 2 and of 3 so you have a multiple 6[/hide]\n\n\n\nThis can be written as\n\n[hide]\n\nIf n:equiv:1 mod 6, then (2n+1)(n+1)n :equiv: (3)(2)(1) :equiv: 6 :equiv: 0 mod 6. \n\n[/hide]\n\n\n\nSo your proof could be drastically simplified. But you've got the main idea.", "Solution_9": "I was looking back at an old MA :theta: test and ran across this problem:\n\nFind the smalest positive ineger x that the solution to the following system of congruencies: x :equiv: 2 mod 3 x :equiv: 3 mod 5 x :equiv: 5 mod 7.\n\n\n\nI solved it by finding all the numbers :equiv: 5 mod 7 then looking for 3 mod 5 and 2 mod 3 I got [hide] 68 [/hide] but there must be a quicker way to do this.\n\n\n\nbtw thanks for the adivce TripleM and Osiris", "Solution_10": "Basically, you can't get much better than checking the way you did, though you do it with a bit of algebra. I'm not sure if people want to try your question, so I'll show you how I would do a different example:\n\n\n\nx = 3 mod 4, x = 1 mod 5, x = 6 mod 11.\n\n\n\n[hide]\n\nStart with the highest modulo, 11. We get x = 11a+6. \n\n\n\nThen we go to the next modulo 5. 11a+6 = 10a+a+5+1 = a+1 mod 5. So a must be 0 mod 5. Let a = 5b, and we get x = 55b+6. \n\n\n\nThen we go to the last modulo 4. x = 55b+6 = 52b+3b+4+2 = 3b+2 (mod 4). So 3b+2 = 3 (mod 4), so 3b = 1 (mod 4). 3x0=0, 3x1=3, 3x2=1, 3x3=4, so b is 2 mod 4. Finally, we let b = 4c+2, and thus x = 55(4c+2)+6 = 220c + 116.\n\n\n\nSo x is 116, 336, 556, etc.\n\n[/hide]\n\n\n\nThere is another way I think but this is what I do." } { "Tag": [ "inequalities", "inequalities proposed" ], "Problem": "Let be $ x_1,x_2,x_3,...,x_n\\in (0,\\infty)$ . Show that :\r\n\r\n$ \\frac{x_1}{x_1\\plus{}x_2\\plus{}x_3...\\plus{}x_n}\\plus{}\\frac{2x_2}{x_1\\plus{}2x_2\\plus{}x_3\\plus{}...\\plus{}x_n}\\plus{}...\\plus{}\\frac{nx_n}{x_1\\plus{}x_2\\plus{}\\plus{}...\\plus{}nx_n}>1$", "Solution_1": "Very nice. \r\n[hide]\nCase 1: At least one of the fractions $ \\dfrac{ix_i}{(x_1\\plus{}x_2\\plus{}...\\plus{}x_n)\\plus{}(i\\minus{}1)x_i}$ is greater than one.\n\nThe inequality is evident. \n\n\nCase 2: $ \\dfrac{ix_i}{(x_1\\plus{}x_2\\plus{}...\\plus{}x_n)\\plus{}(i\\minus{}1)x_i}\\le 1\\forall i\\in\\{1,2,...,n\\}$\n\nIf $ a$, $ b$ and $ x$ are positive numbers we have that $ 1\\ge\\dfrac{a}{b}\\Leftrightarrow\\dfrac{x\\plus{}a}{x\\plus{}b}\\ge\\dfrac{a}{b}$ with equality if and only if $ a\\equal{}b$.\n\nThen\n$ \\sum_{i\\equal{}1}^n{\\dfrac{ix_i}{(x_1\\plus{}x_2\\plus{}...\\plus{}x_n)\\plus{}(i\\minus{}1)x_i}}\\ge\\sum_{i\\equal{}1}^n{\\dfrac{x_i}{x_1\\plus{}x_2\\plus{}...\\plus{}x_n}}\\equal{}1$ \n\nBut equality cannot hold in any inequality since it would imply $ x_1\\plus{}...\\plus{}x_n\\minus{}x_i\\equal{}0$, which can't be true because all of the $ x_k$ are positive.\n\nThen the inequality is strict, and we have the result we wanted.\n[/hide]\r\n\r\n:)" } { "Tag": [ "search", "\\/closed" ], "Problem": "How can I help add problems to the archives?\r\n\r\nDo I just need to contact a mod and tell them what/where I've posted something?", "Solution_1": "Contact 4everwise or Silverfalcon.", "Solution_2": "[quote=\"SnowStorm\"]How can I help add problems to the archives?\n\nDo I just need to contact a mod and tell them what/where I've posted something?[/quote]\r\n\r\nUm. Here is what I recommend you to do:\r\n\r\n1. Post problem ONLY (i.e. no other wording like \"this is a nice problem.. etc\")\r\n2. In source field, put type of contest (AHSME, COMC, etc..), year, problem # in the source field so I can find them through searching (I already used some problems that you posted, SnowStorm).\r\n\r\nI don't really recommend by giving links because when I post problems, I ALWAYS use search button (yeah.. I know not a lot of people do this but this is in Valentin's signature and I really follow them. :) ) so if you posted them, I'll be able to find them." } { "Tag": [ "invariant", "number theory proposed", "number theory" ], "Problem": "Let [tex]n>1[/tex]. Let's shuffle the digits of [tex]n^m[/tex] ([tex]m\\in\\mathbb{N}[/tex]), and let's denote the new number by [tex]l[/tex]. Prove that for any [tex]k\\neq m[/tex] we have [tex]n^k \\neq l[/tex].\r\n\r\nP.S. In original problem we had n=2.", "Solution_1": "consider residue mod 3... it is clear the only case is $n=3$.", "Solution_2": "Can you explain your ideas better? I can't understand what you are trying to do. ;)", "Solution_3": "Consider: the relation between the new number and the old number is that one is 2-9 times the other (as it is not equal, so not 1 time, and cannot be 10 or more because they have the same number of digits). Rearranging the digits keeps it invariant $\\mod 9$. Thus, the either the multiplier is $1\\mod 9$ which cannot be, or the number was originally $0\\mod 9$. This means that n is a multiple of 3 (not 3, i made a mistake..)", "Solution_4": "Kueh! You forgot about leading zeros!", "Solution_5": "oh dear :blush: :blush:" } { "Tag": [ "rotation", "geometry", "incenter", "trigonometry", "trig identities", "Law of Cosines" ], "Problem": "A point $ D$ is drawn inside an equilateral triangle $ ABC$ such that $ D$ is $ 8$, $ 7$, and $ 5$ from points $ A$, $ B$, and $ C$, respectively. Compute a side length of the triangle.", "Solution_1": "[hide=\"Hint\"]Create a rotated version of the triangle that's rotated 60 degrees about point A.[/hide]\r\n\r\n[b]@ Fuuckin Kunt:[/b]\r\n1. Unfortunately, the incenter has little to do with it.\r\n2. I don't like your user name.", "Solution_2": "Any more hints?", "Solution_3": "That's really the hint for solving the problem. Try a little bit harder and you'll say 'Ah'.", "Solution_4": "I got it using the Zuton's Force hint.\r\n\r\nBut what's the motivation behind the hint? And that the angles would work that way?\r\n\r\n[hide]If you revolve the triangle about point $ A$ by $ \\minus{}60^\\circ$ and draw a line segment from $ D$ to $ D'$, you'll see that an equilateral triangle with side length $ 8$ is formed. It appears that $ DC'$ is a line segment, but I don't know why. Anyways, using the law of cosines, we have $ x^2\\equal{}8^2\\plus{}5^2\\minus{}2\\cdot 8\\cdot 5\\cdot \\cos 120^\\circ$, so $ \\boxed{x\\equal{}\\sqrt{129}}$.[/hide]", "Solution_5": "[quote=\"123456789\"]\n[hide]It appears that $ DC'$ is a line segment, but I don't know why.[/hide][/quote]\n[hide]I think you mean $ DD'C'$. Note that $ m\\angle D'DC \\equal{} \\arccos\\left(\\frac {5^2 \\plus{} 8^2 \\minus{} 7^2}{2\\cdot 5\\cdot 8}\\right)\\equal{}\\arccos\\frac12$, which means it's a 60 degree angle. So $ \\angle ADC\\cong\\angle AD'C'$ is supplementary to $ \\angle AD'D$.[/hide]\r\nI hope our pictures are the same.", "Solution_6": "Is there a way to see that without using the Law of Cosines?" } { "Tag": [ "inequalities", "function", "logarithms", "inequalities unsolved" ], "Problem": "Find all real x, such that: $ 1\\leq(\\frac {x}{2} \\plus{} 1)^{1 \\minus{} x^2}$", "Solution_1": "a)If $ x < 0$, then the inequality isn't well defined (the exponential function is defined for positive base).\r\nb)If $ x \\equal{} 0$, then the inequality holds (case of equality).\r\nc)If $ x > 0$, then $ (\\frac {x}{2} \\plus{} 1) > 1$. Logarithm the inequality to get $ 0\\le(1 \\minus{} x^2)ln(\\frac {x}{2} \\plus{} 1)$. But $ ln(\\frac {x}{2} \\plus{} 1) > 0$, so $ 0\\le(1 \\minus{} x^2)$, that is, $ x \\le1$.\r\nSo the final answer is $ 0\\le x \\le 1$.", "Solution_2": "Wait, can't we define exponential function for negative bases like this:\r\n\r\n$ a^b\\equal{}\\frac{1}{a^{\\minus{}b}}$?", "Solution_3": "I didn't say for negative exponents, I said for negative bases: in what you say, a is the base." } { "Tag": [], "Problem": "How long should it take me to complete the AoPS Volumes 1 and 2? GIven that I spend 7 hours/week on the material, do you think that I can complete it by the end of this year? (so that I can use the strategies before the AMCs are given).", "Solution_1": "The amount of time required to go through them relies heavily on your current level as well as other factors. The amount of time it takes you to absorb material is also a big factor. In addition, you will get through it much quicker if you go on to the next chapter after obtaining only a slight understanding of the last chapter. I think that it took me about 2 months to get through volume 1. I can't remember how much time a day I spent on it but it was a few hours(like 1-3 hrs i think). I still have a lot of volume 2 to do and I doubt I will get through it anytime soon. I have school and cross country to do first before I have free time to do extra math. When I do have free time i usually spend it on the AoPS classes I am taking.\r\n\r\nYou should be able to get through volume 1. I don't know about volume 2 though. Understanding(and mastering) all the material in 2 takes a lot of time and focus. If you are just starting vol. 1 now and you have a fairly good grounding in math I think that getting through vol 1 and about 1/2 of 2 is a good goal.", "Solution_2": "Well for the first AMC round you'll do pretty good with only the Volume 1. But basicly the message is (for all competitions) the more you'll train, the better the result. Regardless of whether or not you finished the book." } { "Tag": [ "geometry" ], "Problem": "A circle with center $ O$ and its chord $ AB$ is given. $ C$ is an arbitrary point on the circle.\r\n\r\ni) Prove that the locus of the orthocenter of $ \\triangle ABC$ is a circle.\r\n\r\nii) Find the radius of the loci circle in terms of $ OA \\equal{} R$.\r\n\r\niii) Find the distance from $ O$ to the center of the loci circle in terms of $ OA \\equal{} R$ and $ AB \\equal{} x$.", "Solution_1": "[hide]Use complex numbers taking O in the origin of the complex plane.[/hide]", "Solution_2": "Call H the orthocenter of ABC , because of = 0 by the relations a_(n+1) = 2a_n + 3b_n and b_(n+1) = a_n + 2b_n for \nn >= 0 given the values a_0 and b_0.\n\n(a) prove that the quantity (a_n)^2 - 3(b_n)^2 is a constant not dependant on n.\n(b) if a_0 = 1 and b_0 = 0, show that a_n and b_n are relatively prime over all n.\n(c) let r_n = a_n/b_n. Find a recursion for r_n.\n(d) if a_0 = 1 and b_0 = 0, compute the first few terms of the three sequences a_n, b_n and r_n. Use these values to conjecture a limiting value for r_n as n grows without bound and a recursive relation for a_n and b_n. Prove as many of your conjectures as possible.[/color]", "Solution_1": "For a) induction works like a charm :D This is actually similar to the way we prove that there are infinitely many solns for Pell's eqn.\r\n\r\nFor b) we use a) and we see that a_n^2-3b_n^3=1 for all n, so (a_n, b_n)=1.\r\n\r\nStill thinking of the others..." } { "Tag": [], "Problem": "Some people might have seen this problem before, but here goes:\n\n\n\nA guy went to cash a check. However, the teller misread the check, and gave the man cents for dollars, and dollars for cents (Either lucky or unlucky, depending on the amount of the check.) The man put the money in his pocket without checking to make sure he has the right amount (or could he be just greedy?). On his way home, the man gave 5 cents to a beggar in the subway.(Praise his charity!) When he got home, he found out that he has twice the amount of money stated by the check in his pockets. (He's really lucky). Before going to the bank, the man had no money at all in his pockets. What is the amount of the check?\n\n\n\nCan anyone help with a simple solution to this problem?\n\n\n\n[hide]Answer: $31.63[/hide]", "Solution_1": "Okay I'll try:\n\n[hide]\n\n\n\nStarts out A dollars and B cents, or A+B/100 dollars or 100A+B cents. Then at the bank comes out w/ B dollars and A cents, B+A/100 dollars total. He gives 5 cents to a beggar, leaving him B+A/100 - 5/100, and finds when he gets home that:\n\n\n\n2(A+B/100)=B+A/100-5/100\n\n2A+B/50=B+A/100-5/100\n\n199/100 * A + -49/50 *B=-5/100\n\n199A-98B=-5\n\n\n\nB=XY (digits) = 10X+Y\n\nA=CD (digits) = 10C + D\n\n\n\n1990X+199Y-980B-98D+5=0\n\n\n\nC, D, X, Y are integers 0 through 9\n\n\n\nSo you just have to play around w/it until you find 4 values that satisfy the equation.\n\n\n\n[/hide][/quote]", "Solution_2": "Good job so far, Julie. My only suggestions:\n\n\n\n(1) When working in dollars and cents, hours and minutes, or any other similar set of objects, it's almost always simpler to use the smaller unit. Thus, your calculations are easier and look nicer in this problem if you use cents as your unit instead of dollars, because you have expressions like (100A + B) instead of (A + B/100).\n\n\n\n(2) I wouldn't have switched over to the digits when you did. I think it's actually easier to solve when you're just looking for A and B. Also, that is really not as simple a task as you made it look. Try solving the equation you found, [hide]199A = 98B - 5.[/hide] I suggest as the simplest way to solve it [hide]looking at it mod 98, or in other words finding a value of A such that the left side is 5 less than a multiple of 98.[/hide]", "Solution_3": "I got[hide] A=31 and B=63[/hide] therefore, the check was [hide]$31.63[/hide].\n\n\n\ncorrect?", "Solution_4": "Is there a simpler way to do it than solving the equation [hide]98B-5=199A[/hide]?", "Solution_5": "JS1527- the answer is under the problem in spoiler. your answer is correct.", "Solution_6": "BobofBobs wrote:Is there a simpler way to do it than solving the equation [hide]98B-5=199A[/hide]?\n\n\n\nNot that I know of. That equation isn't too hard to solve, though, because 199 is very close to a multiple of 98.", "Solution_7": "[quote]JS1527- the answer is under the problem in spoiler. your answer is correct.[/quote]\r\n\r\nWhoa, I didn't even THINK to look there. I am absent minded, forgive me." } { "Tag": [ "HCSSiM" ], "Problem": "In this game, each of the 50 states starts out with 10 points. Each time you post, you get 2 points to spend(help your state or attack other states). You may post at most one time each hour. The only exception is that you may only add points to the state live in (please don't cheat). But you can form alliances which allow you to help other states(explained later).You may post at most one time each hour.\r\n\r\nPoints: Each state starts out with 10 points. If your state reaches 0 points, then it is out of the game and the state that destroyed it gets 2 extra points. When your state has enough points, you can choose to spend some of those points on special items(these items are listed below). \r\n\r\nAlliances: During the game, 2 or more states can choose to form an alliance. The maximum number of states there can be in an alliance is 5(name it whatever you want). In order to form one, one person from each state must first agree to it and then each state that wants to be in the alliance must give up 5 points.\r\n\r\nAdvantages of Alliances: \r\n\r\n1) States that are in the same alliance can help each other(add points to each other)\r\n\r\n2) When one of your allies destroys another state, your state gets 1 extra point(the state that destroyed it gets 2 points)\r\n\r\n3) You can split the cost of buying special items\r\n\r\n4) If you form an alliance with 3, 4, or 5 states you get that many points to spend(for example, if Hawaii, Alaska, and California form an alliance, then each person from those states gets 3 points to spend each time they post instead of 2. If 4 states form an alliance then each gets 4 points to spend)\r\n\r\nYou may not attack your allies unless they agree. And any state in an alliance can choose to leave at any time(you dont get the 5 points back).\r\n\r\nSpecial items:\r\n\r\nAtom bomb- Choose any state(except your allies) to drop the bomb on. That state is automatically destroyed no matter how many points it has. Each state that borders that state loses 5 points.\r\n\r\nCost: 50 points\r\n\r\nShield- Opponents may not attack your state for 3 hours.\r\n\r\nCost:15 points\r\n\r\nStronger Shield- Opponents may not attack your state for 5 hours.\r\n\r\nCost: 25 points\r\n\r\nNuclear defense system: 50 points - 1 use, protects you from nuke\r\n\r\nBiohazard suits- 25 points-1 use, protects you from radioactive fallout (if the state next to you is nuked)\r\n\r\n*taking suggestions for other items*\r\n\r\nAnd PLEASE hide the score so it doesn't take forever to scroll. :D \r\n\r\n[hide=\"score\"]Alabama-10 pts\nAlaska-10 pts\nArizona-10 pts\nArkansas-10 pts\nCalifornia-10 pts\nColorado-10 pts\nConnecticut-10 pts\nDelaware-10 pts\nFlorida-10 pts\nGeorgia-10 pts\nHawaii-10 pts\nIdaho-10 pts\nIllinois-10 pts\nIndiana-10 pts\nIowa-10 pts\nKansas-10 pts\nKentucky-10 pts\nLouisiana-10 pts\nMaine-10 pts\nMaryland-10 pts\nMassachusetts-10 pts\nMichigan-10 pts\nMinnesota-10 pts\nMississippi-10 pts\nMissouri-10 pts\nMontana-10 pts\nNebraska-10 pts\nNevada-10 pts\nNew Hampshire-10 pts\nNew Jersey-10 pts\nNew Mexico-10 pts\nNew York-10 pts\nNorth Carolina-10 pts\nNorth Dakota-10 pts\nOhio-10 pts\nOklahoma-10 pts\nOregon-10 pts\nPennsylvania-10 pts\nRhode Island-10 pts\nSouth Carolina-10 pts\nSouth Dakota-10 pts\nTennessee-10 pts\nTexas-10 pts\nUtah-10 pts\nVermont-10 pts\nVirginia-10 pts\nWashington-10 pts\nWest Virginia-10 pts\nWisconsin-10 pts\nWyoming-10 pts[/hide]", "Solution_1": "[hide=\"score\"]Alabama-10 pts\nAlaska-10 pts\nArizona-10 pts\nArkansas-10 pts\nCalifornia-10 pts\nColorado-10 pts\nConnecticut-10 pts\nDelaware-10 pts\nFlorida-10 pts\nGeorgia-10 pts\nHawaii-10 pts\nIdaho-10 pts\nIllinois-10 pts\nIndiana-10 pts\nIowa-10 pts\nKansas-10 pts\nKentucky-10 pts\nLouisiana-10 pts\nMaine-10 pts\nMaryland-10 pts\nMassachusetts-10 pts\nMichigan-10 pts\nMinnesota-10 pts\nMississippi-10 pts\n[color=green]Missouri-12 pts[/color]\nMontana-10 pts\nNebraska-10 pts\nNevada-10 pts\nNew Hampshire-10 pts\nNew Jersey-10 pts\nNew Mexico-10 pts\nNew York-10 pts\nNorth Carolina-10 pts\nNorth Dakota-10 pts\nOhio-10 pts\nOklahoma-10 pts\nOregon-10 pts\nPennsylvania-10 pts\nRhode Island-10 pts\nSouth Carolina-10 pts\nSouth Dakota-10 pts\nTennessee-10 pts\nTexas-10 pts\nUtah-10 pts\nVermont-10 pts\nVirginia-10 pts\nWashington-10 pts\nWest Virginia-10 pts\nWisconsin-10 pts\nWyoming-10 pts[/hide][/quote]", "Solution_2": "take out the states where no one lives first. :D \r\n\r\n[hide=\"score\"]Alabama-10 pts\nAlaska-[color=red]8 pts[/color]\nArizona-10 pts\nArkansas-10 pts\nCalifornia-10 pts\nColorado-10 pts\nConnecticut-10 pts\nDelaware-10 pts\nFlorida-10 pts\nGeorgia-10 pts\nHawaii-10 pts\nIdaho-10 pts\nIllinois-10 pts\nIndiana-10 pts\nIowa-10 pts\nKansas-10 pts\nKentucky-10 pts\nLouisiana-10 pts\nMaine-10 pts\nMaryland-10 pts\nMassachusetts-10 pts\nMichigan-10 pts\nMinnesota-10 pts\nMississippi-10 pts\nMissouri-12 pts\nMontana-10 pts\nNebraska-10 pts\nNevada-10 pts\nNew Hampshire-10 pts\nNew Jersey-10 pts\nNew Mexico-10 pts\nNew York-10 pts\nNorth Carolina-10 pts\nNorth Dakota-10 pts\nOhio-10 pts\nOklahoma-10 pts\nOregon-10 pts\nPennsylvania-10 pts\nRhode Island-10 pts\nSouth Carolina-10 pts\nSouth Dakota-10 pts\nTennessee-10 pts\nTexas-10 pts\nUtah-10 pts\nVermont-10 pts\nVirginia-10 pts\nWashington-10 pts\nWest Virginia-10 pts\nWisconsin-10 pts\nWyoming-10 pts[/hide]", "Solution_3": "oh man this looks slightly silly... but I will help NC at least once\r\n\r\n[hide=\"score\"]Alabama-10 pts\nAlaska-8 pts\nArizona-10 pts\nArkansas-10 pts\nCalifornia-10 pts\nColorado-10 pts\nConnecticut-10 pts\nDelaware-10 pts\nFlorida-10 pts\nGeorgia-10 pts\nHawaii-10 pts\nIdaho-10 pts\nIllinois-10 pts\nIndiana-10 pts\nIowa-10 pts\nKansas-10 pts\nKentucky-10 pts\nLouisiana-10 pts\nMaine-10 pts\nMaryland-10 pts\nMassachusetts-10 pts\nMichigan-10 pts\nMinnesota-10 pts\nMississippi-10 pts\nMissouri-12 pts\nMontana-10 pts\nNebraska-10 pts\nNevada-10 pts\nNew Hampshire-10 pts\nNew Jersey-10 pts\nNew Mexico-10 pts\nNew York-10 pts[color=green]\nNorth Carolina-12 pts[/color]\nNorth Dakota-10 pts\nOhio-10 pts\nOklahoma-10 pts\nOregon-10 pts\nPennsylvania-10 pts\nRhode Island-10 pts\nSouth Carolina-10 pts\nSouth Dakota-10 pts\nTennessee-10 pts\nTexas-10 pts\nUtah-10 pts\nVermont-10 pts\nVirginia-10 pts\nWashington-10 pts\nWest Virginia-10 pts\nWisconsin-10 pts\nWyoming-10 pts[/hide]", "Solution_4": "Suggestions:\r\n\r\nNuclear defense system: 50 points - 1 use, protects you from nuke\r\nBiohazard suits- 25 points-1 use, protects you from radioactive fallout (if the state next to you is nuked)\r\n\r\nNote: for those 2 items, all members of the team can use it, but once it is used once, it can't be used again by any member of the team.\r\n\r\nDissolving alliance: After an alliance is made, you may choose to dissolve the alliance. There is a 1-day ceasefire (between members of the alliance) and then fighting between those states can resume.", "Solution_5": "Hopefully more people from my state will see this.\r\n\r\n[hide=\"score\"]Alabama-10 pts\nAlaska-8 pts\nArizona-10 pts\nArkansas-10 pts\nCalifornia-10 pts\nColorado-10 pts\nConnecticut-10 pts\nDelaware-10 pts\nFlorida-10 pts\nGeorgia-10 pts\nHawaii-10 pts\nIdaho-10 pts\nIllinois-10 pts\nIndiana-10 pts\nIowa-10 pts\nKansas-10 pts\nKentucky-10 pts\nLouisiana-10 pts\nMaine-10 pts\nMaryland-10 pts\n[color=green]Massachusetts-12 pts[/color]\nMichigan-10 pts\nMinnesota-10 pts\nMississippi-10 pts\nMissouri-12 pts\nMontana-10 pts\nNebraska-10 pts\nNevada-10 pts\nNew Hampshire-10 pts\nNew Jersey-10 pts\nNew Mexico-10 pts\nNew York-10 pts\nNorth Carolina-12 pts\nNorth Dakota-10 pts\nOhio-10 pts\nOklahoma-10 pts\nOregon-10 pts\nPennsylvania-10 pts\nRhode Island-10 pts\nSouth Carolina-10 pts\nSouth Dakota-10 pts\nTennessee-10 pts\nTexas-10 pts\nUtah-10 pts\nVermont-10 pts\nVirginia-10 pts\nWashington-10 pts\nWest Virginia-10 pts\nWisconsin-10 pts\nWyoming-10 pts[/hide]", "Solution_6": "I will LOSE but oh well\r\n\r\n[hide=\"score\"]Alabama-10 pts \nAlaska-8 pts \nArizona-10 pts \nArkansas-10 pts \nCalifornia-10 pts \nColorado-10 pts \nConnecticut-10 pts \nDelaware-10 pts \nFlorida-10 pts \nGeorgia-10 pts \nHawaii-10 pts \nIdaho-10 pts \nIllinois-10 pts \nIndiana-10 pts \nIowa-10 pts \nKansas-10 pts \nKentucky-10 pts \nLouisiana-10 pts \nMaine-10 pts \nMaryland-10 pts \nMassachusetts-12 pts \nMichigan-10 pts \nMinnesota-10 pts \nMississippi-10 pts \nMissouri-12 pts \nMontana-10 pts \nNebraska-10 pts \nNevada-10 pts \nNew Hampshire-10 pts \nNew Jersey-10 pts \nNew Mexico-10 pts \nNew York-10 pts \nNorth Carolina-12 pts \nNorth Dakota-10 pts \nOhio-10 pts \nOklahoma-10 pts \nOregon-10 pts \nPennsylvania-10 pts \nRhode Island-10 pts \nSouth Carolina-10 pts \nSouth Dakota-10 pts \nTennessee-10 pts \nTexas-10 pts \nUtah-10 pts \nVermont-10 pts \nVirginia-10 pts \n[color=green]Washington-12 pts [/color]\nWest Virginia-10 pts \nWisconsin-10 pts \nWyoming-10 pts [/hide]", "Solution_7": "[hide=\"score\"] Alabama-10 pts\nAlaska-8 pts\nArizona-10 pts\nArkansas-10 pts\nCalifornia-10 pts\nColorado-10 pts\nConnecticut-10 pts\nDelaware-10 pts\nFlorida-10 pts\nGeorgia-10 pts\nHawaii-10 pts\nIdaho-10 pts\nIllinois-10 pts\nIndiana-10 pts\nIowa-10 pts\nKansas-10 pts\nKentucky-10 pts\nLouisiana-10 pts\nMaine-10 pts\nMaryland-10 pts\nMassachusetts-12 pts\nMichigan-10 pts\nMinnesota-10 pts\nMississippi-10 pts\nMissouri-[color=green]14 pts[/color]\nMontana-10 pts\nNebraska-10 pts\nNevada-10 pts\nNew Hampshire-10 pts\nNew Jersey-10 pts\nNew Mexico-10 pts\nNew York-10 pts\nNorth Carolina-12 pts\nNorth Dakota-10 pts\nOhio-10 pts\nOklahoma-10 pts\nOregon-10 pts\nPennsylvania-10 pts\nRhode Island-10 pts\nSouth Carolina-10 pts\nSouth Dakota-10 pts\nTennessee-10 pts\nTexas-10 pts\nUtah-10 pts\nVermont-10 pts\nVirginia-10 pts\nWashington-12 pts\nWest Virginia-10 pts\nWisconsin-10 pts\nWyoming-10 pts[/hide]\n\n[hide=\"updated items list\"]Atom bomb- Choose any state(except your allies) to drop the bomb on. That state is automatically destroyed no matter how many points it has. Each state that borders that state loses 5 points.\n\nCost: 50 points\n\nShield- Opponents may not attack your state for 3 hours. Shields do not protect from nukes or radiation.\n\nCost:15 points\n\nStronger Shield- Opponents may not attack your state for 5 hours.\n\nCost: 25 points\n\nNuclear defense system: 50 points - 1 use, protects you from nuke\n\nBiohazard suits- 25 points-1 use, protects you from radioactive fallout (if the state next to you is nuked)[/hide]", "Solution_8": "[hide]Alabama-10 pts \nAlaska-8 pts \nArizona-10 pts \nArkansas-10 pts \nCalifornia-10 pts \nColorado-10 pts \nConnecticut-10 pts \nDelaware-10 pts \nFlorida-10 pts \nGeorgia-10 pts \nHawaii-10 pts \nIdaho-10 pts \nIllinois-10 pts \nIndiana-10 pts \nIowa-10 pts \nKansas-10 pts \nKentucky-10 pts \nLouisiana-10 pts \nMaine-10 pts \nMaryland-10 pts \nMassachusetts-12 pts \nMichigan-10 pts \nMinnesota-10 pts \nMississippi-10 pts \n[color=green]Missouri-16 pts[/color] \nMontana-10 pts \nNebraska-10 pts \nNevada-10 pts \nNew Hampshire-10 pts \nNew Jersey-10 pts \nNew Mexico-10 pts \nNew York-10 pts \nNorth Carolina-12 pts \nNorth Dakota-10 pts \nOhio-10 pts \nOklahoma-10 pts \nOregon-10 pts \nPennsylvania-10 pts \nRhode Island-10 pts \nSouth Carolina-10 pts \nSouth Dakota-10 pts \nTennessee-10 pts \nTexas-10 pts \nUtah-10 pts \nVermont-10 pts \nVirginia-10 pts \nWashington-12 pts \nWest Virginia-10 pts \nWisconsin-10 pts \nWyoming-10 pts \n[/hide]", "Solution_9": "[hide=\"abooga\"]\nAlabama-10 pts \nAlaska-[color=red]6 pts[/color] \nArizona-10 pts \nArkansas-10 pts \nCalifornia-10 pts \nColorado-10 pts \nConnecticut-10 pts \nDelaware-10 pts \nFlorida-10 pts \nGeorgia-10 pts \nHawaii-10 pts \nIdaho-10 pts \nIllinois-10 pts \nIndiana-10 pts \nIowa-10 pts \nKansas-10 pts \nKentucky-10 pts \nLouisiana-10 pts \nMaine-10 pts \nMaryland-10 pts \nMassachusetts-12 pts \nMichigan-10 pts \nMinnesota-10 pts \nMississippi-10 pts \nMissouri-16 pts \nMontana-10 pts \nNebraska-10 pts \nNevada-10 pts \nNew Hampshire-10 pts \nNew Jersey-10 pts \nNew Mexico-10 pts \nNew York-10 pts \nNorth Carolina-12 pts \nNorth Dakota-10 pts \nOhio-10 pts \nOklahoma-10 pts \nOregon-10 pts \nPennsylvania-10 pts \nRhode Island-10 pts \nSouth Carolina-10 pts \nSouth Dakota-10 pts \nTennessee-10 pts \nTexas-10 pts \nUtah-10 pts \nVermont-10 pts \nVirginia-10 pts \nWashington-12 pts \nWest Virginia-10 pts \nWisconsin-10 pts \nWyoming-10 pts \n[/hide]", "Solution_10": "[hide]Alabama-10 pts\nAlaska-6 pts\nArizona-10 pts\nArkansas-10 pts\nCalifornia-10 pts\nColorado-10 pts\nConnecticut-10 pts\nDelaware-10 pts\nFlorida-10 pts\nGeorgia-10 pts\nHawaii-10 pts\nIdaho-10 pts\nIllinois-10 pts\nIndiana-10 pts\nIowa-10 pts\nKansas-10 pts\nKentucky-10 pts\nLouisiana-10 pts\nMaine-10 pts\nMaryland-10 pts\nMassachusetts-12 pts\nMichigan-10 pts\nMinnesota-10 pts\nMississippi-10 pts\nMissouri-16 pts\nMontana-10 pts\nNebraska-10 pts\nNevada-10 pts\nNew Hampshire-10 pts\nNew Jersey-10 pts\nNew Mexico-10 pts\nNew York-10 pts\nNorth Carolina-12 pts\nNorth Dakota-10 pts\nOhio-10 pts\nOklahoma-10 pts\nOregon-10 pts\nPennsylvania-10 pts\nRhode Island-10 pts\nSouth Carolina-10 pts\nSouth Dakota-10 pts\nTennessee-10 pts\nTexas-12 pts\nUtah-10 pts\nVermont-10 pts\nVirginia-10 pts\nWashington-12 pts\nWest Virginia-10 pts\nWisconsin-10 pts\nWyoming-10 pts [/hide]", "Solution_11": "which state di you do, budi????? its hard to tell without colors.", "Solution_12": "texas, washington, massachusetts, north carolina alliance?\r\n\r\n\r\n[hide=\"score\"]Alabama-10 pts\nAlaska-[color=red]4 pts[/color]\nArizona-10 pts\nArkansas-10 pts\nCalifornia-10 pts\nColorado-10 pts\nConnecticut-10 pts\nDelaware-10 pts\nFlorida-10 pts\nGeorgia-10 pts\nHawaii-10 pts\nIdaho-10 pts\nIllinois-10 pts\nIndiana-10 pts\nIowa-10 pts\nKansas-10 pts\nKentucky-10 pts\nLouisiana-10 pts\nMaine-10 pts\nMaryland-10 pts\nMassachusetts-12 pts\nMichigan-10 pts\nMinnesota-10 pts\nMississippi-10 pts\nMissouri-16 pts\nMontana-10 pts\nNebraska-10 pts\nNevada-10 pts\nNew Hampshire-10 pts\nNew Jersey-10 pts\nNew Mexico-10 pts\nNew York-10 pts\nNorth Carolina-12 pts\nNorth Dakota-10 pts\nOhio-10 pts\nOklahoma-10 pts\nOregon-10 pts\nPennsylvania-10 pts\nRhode Island-10 pts\nSouth Carolina-10 pts\nSouth Dakota-10 pts\nTennessee-10 pts\nTexas-12 pts\nUtah-10 pts\nVermont-10 pts\nVirginia-10 pts\nWashington-12 pts\nWest Virginia-10 pts\nWisconsin-10 pts\nWyoming-10 pts[/hide]\r\nbudi lives in TX", "Solution_13": "[hide=\"score\"]Alabama-10 pts\nAlaska-4 pts\nArizona-10 pts\nArkansas-10 pts\nCalifornia-10 pts\nColorado-10 pts\nConnecticut-10 pts\nDelaware-10 pts\nFlorida-10 pts\nGeorgia-10 pts\nHawaii-10 pts\nIdaho-10 pts\nIllinois-10 pts\nIndiana-10 pts\nIowa-10 pts\nKansas-10 pts\nKentucky-10 pts\nLouisiana-10 pts\nMaine-10 pts\nMaryland-10 pts\nMassachusetts-12 pts\nMichigan-10 pts\nMinnesota-10 pts\nMississippi-10 pts\n[color=green]Missouri-18 pts[/color]\nMontana-10 pts\nNebraska-10 pts\nNevada-10 pts\nNew Hampshire-10 pts\nNew Jersey-10 pts\nNew Mexico-10 pts\nNew York-10 pts\nNorth Carolina-12 pts\nNorth Dakota-10 pts\nOhio-10 pts\nOklahoma-10 pts\nOregon-10 pts\nPennsylvania-10 pts\nRhode Island-10 pts\nSouth Carolina-10 pts\nSouth Dakota-10 pts\nTennessee-10 pts\nTexas-12 pts\nUtah-10 pts\nVermont-10 pts\nVirginia-10 pts\nWashington-12 pts\nWest Virginia-10 pts\nWisconsin-10 pts\nWyoming-10 pts[/hide]", "Solution_14": "[hide=\"score\"]Alabama-10 pts\nAlaska-4 pts\nArizona-10 pts\nArkansas-10 pts\nCalifornia-10 pts\nColorado-10 pts\nConnecticut-10 pts\nDelaware-10 pts\nFlorida-10 pts\nGeorgia-10 pts\nHawaii-10 pts\nIdaho-10 pts\nIllinois-10 pts\nIndiana-10 pts\nIowa-10 pts\nKansas-10 pts\nKentucky-10 pts\nLouisiana-10 pts\nMaine-10 pts\nMaryland-10 pts\nMassachusetts-12 pts\nMichigan-10 pts\nMinnesota-10 pts\nMississippi-10 pts\nMissouri-18 pts\nMontana-10 pts\nNebraska-10 pts\nNevada-10 pts\nNew Hampshire-10 pts\nNew Jersey-10 pts\nNew Mexico-10 pts\nNew York-10 pts\nNorth Carolina-[color=green]14 pts[/color]\nNorth Dakota-10 pts\nOhio-10 pts\nOklahoma-10 pts\nOregon-10 pts\nPennsylvania-10 pts\nRhode Island-10 pts\nSouth Carolina-10 pts\nSouth Dakota-10 pts\nTennessee-10 pts\nTexas-12 pts\nUtah-10 pts\nVermont-10 pts\nVirginia-10 pts\nWashington-[color=green]14 pts[/color]\nWest Virginia-10 pts\nWisconsin-10 pts\nWyoming-10 pts[/hide]\r\n\r\nEDIT: forgot about alliance!", "Solution_15": "Alabama-6 pts\r\nHawaii-14 pts\r\nMinnesota-5 pts\r\nOhio- 15 pts\r\n\r\n-5 MT, -3 AL => +2 HI", "Solution_16": "[quote=\"james4l\"]Alabama-6 pts\nHawaii-14 pts\nMinnesota-5 pts\nOhio- 15 pts\n\n-5 MT, -3 AL => +2 HI[/quote]\r\n\r\nLol you killed our ally.", "Solution_17": "whoops\r\n\r\noh well no one's posting there...", "Solution_18": "ernie, you know that you are cheating right?", "Solution_19": "Alabama-3 pts\r\nHawaii-14 pts\r\nMinnesota-5 pts\r\nOhio- 15 pts \r\n\r\nAL -3", "Solution_20": "Alabama-3 pts\r\nHawaii-4 pts\r\nMinnesota-5 pts\r\nOhio- 5 pts", "Solution_21": "Hawaii-8 pts\r\nOhio- 5 pts\r\n\r\nOWNED", "Solution_22": "Hawaii-8 pts+7pts=15pts-15pts=0pts,PWNT\r\nOhio- 5 pts \r\nJUST TO SAVE POSTS", "Solution_23": "Hawaii-8 pts+7pts=15pts-15pts=0pts,PWNT\r\nOhio- 5 pts\r\nJUST TO SAVE POSTS\r\n\r\nAnd how would you do that?", "Solution_24": "Ohio-8+2=10pts", "Solution_25": "[quote=\"ternary0210\"]Ohio-8+2=10pts[/quote]\r\n\r\nyou know that Ohio is an ally :roll:", "Solution_26": "Ohio-10+4=14pts", "Solution_27": "Ohio-14+1=15pts-15pts=0pts(shield) PWNT\r\n\r\n\r\nOk, now lock this forum.", "Solution_28": "Whoa, my state won???\r\n\r\nSick...", "Solution_29": "Interesting game! Should we start again?" } { "Tag": [ "algebra", "polynomial", "calculus", "derivative", "algebra unsolved" ], "Problem": "Find all polynomials $ P \\in \\mathbb{R}[X]$ with the propriety that :\r\n$ P^n(x)\\equal{}P(x^n) , \\forall x \\in \\mathbb{R}$ where $ n \\in \\mathbb{N} , n >1$ is fixed .", "Solution_1": "If $ P(x) \\equal{} c$ then \r\n$ c^n \\equal{} c\\Leftrightarrow c\\in \\{0,1\\}$\r\nSo $ P(x) \\equal{} 0$ or $ P(x) \\equal{} 1$ \r\nConsider case $ \\deg P \\equal{} m > 0$ \r\nCall $ a_m$ is the leading coefficient of $ P(x)$ then \r\n$ a_n^m \\equal{} a_n$ so $ a_n \\equal{} 1$\r\nConsider $ G(x) \\equal{} P(x) \\minus{} x^m$ then $ \\deg (g) \\leq {m \\minus{} 1}$\r\nSo $ (G(x) \\plus{} x^m)^n \\equal{} G(x^n) \\plus{} x^{mn}$\r\nIf $ \\deg G > 0$ then $ deq (G(x) \\plus{} x^m)^n \\minus{} x^{mn} \\geq n(m \\minus{} 1) \\plus{} 1$\r\nIt gives contradiction . \r\nSo $ G(x) \\equal{} c$ \r\nEasy to check that $ c \\equal{} 0$ \r\nSo $ P(x) \\equal{} x^m$", "Solution_2": "Let P(x)=x^k. Q(x) where Q(0) not equal to zero\r\n\r\nWe have\r\n\r\nQ(x^n)=Q(x)^n\r\n\r\nTake the derivative we have\r\n\r\nn.x^(n-1). Q'(x^n)= n.Q'(x).Q(x)^(n-1)\r\n\r\nFrom there we must have x^(n-1)| Q'(x) --> Q'(x)=c.x^(n-1) as deg Q' =n-1\r\n\r\n......" } { "Tag": [ "function", "induction", "real analysis", "real analysis unsolved" ], "Problem": "I need help solving the following problem:\r\n\r\nIf f(x) is continuous on (a,b) and f((x+y)/2) <= 1/2f(x) + 1/2f(y) for all x,y in (a,b), \r\nthen f(x) is convex.", "Solution_1": "By induction prove that\r\n\\[ f\\left(\\frac{x_1+x_2+\\ldots+x_{2^n}}{2^n}\\right)\\leq \\frac{f(x_1)+f(x_2)+\\ldots+f(x_{2^n})}{2^n} \\]\r\nfor all $n\\geq 1$. Hence conclude that\r\n\\[ f(px+qy)\\leq pf(x)+qf(y) \\] for all $x,y$ and $p,q\\in\\mathbb{Q}^+$ such that $p+q=1$. By continuity it follows that\r\n\\[ f(\\lambda x+(1-\\lambda)y)\\leq \\lambda f(x)+(1-\\lambda)f(y) \\]\r\nfor all $x,y$ and $\\lambda\\in (0,1)$; this is convexity." } { "Tag": [ "probability" ], "Problem": "a box of candy hearts contains 52 hearts of which 19 are white, 10 are tan, 7 are pink, 3 are purple, 5 are yellow, 2 are orange, and 6 are green. if you select 9 pieces of candy randomly from the box, without replacement, give the probability that \r\n\r\n(a) Exactly Three of the hearts are white.\r\n(b) Three are white, two are tan, one is pink, one is yello, and two are green.", "Solution_1": "[hide=\"A\"]\n$ \\binom{9}{3} \\times \\frac{19 \\times 18 \\times 17 \\times 33 \\times 32 \\times 31 \\times 30 \\times 29 \\times 28}{52 \\times 51 \\times 50 \\times 49 \\times 48 \\times 47 \\times 46 \\times 45 \\times 44}\\equal{}\\boxed{\\frac{102486}{351325}}$\n[/hide]\r\n\r\nUse the same method for (B).", "Solution_2": "EDIT: Haha ok i get it now, thanks. so (a) shouuld be 19C3 times 33C6 all divided by 52C9, and b is the same...", "Solution_3": "Yes, that works faster than what I did. Make sure you understand why it works, though." } { "Tag": [], "Problem": "Phosphoric acid is of great importance in fertiliser production. Besides, phosphoric acid and its various salts have a number of applications in metal treatment, food, detergent and toothpaste industries.\r\n\r\na) Write the Lewis structure of phosphoric acid.\r\n\r\nb) The pK values of the three successive dissociations of phosphoric acid at 25\u00b0C are: pK1 = 2.12, pK2 = 7.21, and pK3 = 12.32. Write down the conjugate base of dihydrogen phosphate ion and determine its pKb value.\r\n\r\nc) Small quantities of phosphoric acid are extensively used to impart the sour or tart taste to many soft drinks such as colas and root beers. A cola having a density of 1.00 g/mL contains 0.05 % by weight of phosphoric acid. Determine the pH of the cola (ignoring the second and the third dissociation steps for phosphoric acid). Assume that the acidity of the cola arises only from phosphoric acid.\r\n\r\nd) Phosphoric acid is used as a fertiliser for agriculture. A $ 1.00 \\times 10^{-3}$ M solution of phosphoric acid is added to an aqueous soil suspension and the pH is found to be 7.00. Determine the fractional concentrations of all the different phosphate species present in the solution. Assume that no component of the soil interacts with any phosphate species.\r\n\r\ne) Zinc is an essential micronutrient for plant growth. Plants can absorb zinc in water soluble form only. In a given soil water with pH = 7.0, zinc phosphate was found to be the only source of zinc and phosphate. Calculate the concentration of $ Zn^{2+}$ and $ PO_{4}^{3-}$ ions in the solution. Ksp for zinc phosphate is $ 9.1 \\times 10^{-33}$.", "Solution_1": "a) Just look it up.\r\n\r\nb) Conjugate base is $ HPO_{4}^{2-}$. For a conjugate acid-base pair, $ pK_{a}+pK_{b}=14$, so $ pK_{b}=6.79$ for hydrogen phosphate. \r\n\r\nc) The cola is $ 5.10\\times 10^{-6}M$.\r\n\r\n$ K_{a}=\\frac{[H_{2}PO_{4}^{-}][H^{+}]}{[H_{3}PO_{4}]}=\\frac{x^{2}}{5.10\\times 10^{-6}-x}=7.59\\times 10^{-3}$.\r\n\r\nEDIT: WRONG MOLARITY\r\n\r\nI'll try the rest later.", "Solution_2": "Your answer for b) is correct. However, your answer for c) is wrong.", "Solution_3": "Oops, the cola is actually $ 5.10\\times 10^{-4}M$. \r\n\r\n$ \\frac{x^{2}}{5.10\\times 10^{-4}-x}=7.59\\times 10^{-3}$. \r\n\r\nSolve to get $ x=4.80\\times 10^{-4}$. Is the pH 3.32?\r\n\r\nIf so, I'll move on to d) and e).", "Solution_4": "Still wrong. How are you calculating the concentration of phosphoric acid in the cola? Your problem is there.", "Solution_5": "Let the cola be $ x$ mL. Then it weighs $ x$ grams. Since it contains $ 0.00005x$ grams of phosphoric acid, and the molar mass of phosphoric acid is 98 g/mol, the cola contains $ (5.1\\times 10^{-7})x$ mol. Divide by the volume which is $ x\\cdot 10^{-3}$ liters and get $ 5.1\\times 10^{-4}M$.", "Solution_6": "[quote=\"LeFromage\"]Since it contains $ 0.00005x$ grams of phosphoric acid[/quote]\r\n\r\nYou have an extra zero there. 0.05% by weight means that in 100 g of solution there are 0.05 g of pure solute.", "Solution_7": "...wow that was really clumsy of me. \r\n\r\nSo in fact it is $ 5.1\\times 10^{-3}M$.\r\n\r\nAnd the pH turns out to be 2.46.", "Solution_8": "Yes, that's right. Let's move one to (d) and (e)." } { "Tag": [ "geometry", "trigonometry", "calculus", "calculus computations" ], "Problem": "Find the area inside the lemniscate $ r^2\\equal{}6\\cos 2\\theta$ and outside the circle $ r\\equal{}\\sqrt{3}$", "Solution_1": "hello\r\nsee this prob. may be it would be helpful.\r\n[url]http://curvebank.calstatela.edu/lemniscate/lemniscate.htm[/url]\r\nthank u" } { "Tag": [ "group theory", "abstract algebra", "superior algebra", "superior algebra solved" ], "Problem": "Has anyone something nice for the following problem:\r\n Find all morphisms from the symmetric group $S_4$ into $S_3$.", "Solution_1": "Welcome back,harazi.\r\n\r\nWe are glad to see you! :) \r\n\r\nkunny", "Solution_2": "Thanks, Kunny. Now, let's solve the problem... :(", "Solution_3": "There are precisely two nontrivial normal subgroups of $S_4$. One is $A_4$, and taking the quotient gives a copy of $\\mathbb{Z}_2$ that can be embedded into $S_3$. The other has order 4, and taking the quotient gives a copy of $S_3$, since there can be no elements of order 6.\r\n\r\nUp to automorphism of $S_3$, these and the trivial map are the only homomorphisms.", "Solution_4": "But it's nice to see an explicit permutation representation. In the case where the image is $C_2$, you're permuting the sign of $\\prod_{1\\le i R such that supp(f) = C?\r\n\r\nNow I know that the strong Urysohn's lemma says that if A & B are closed in X, there exists a continuous function g: X -> [0,1] such that g^-1(0) = A and g^-1(1) = B iff A and B are both g-delta. So I thought perhaps that given the condition on C above, perhaps this is enough to conclude that that C compliment is a g-delta, and then find some smaller closed set contained in the interior of C that is also g-delta and I would be done but that is really just a shot in the dark and I haven't been able to get anywhere on it.\r\n\r\nAnyways any insight you could offer would be greatly appreciated. Thanks", "Solution_1": "[quote=\"Spacecow\"]So I thought perhaps that given the condition on C above, perhaps this is enough to conclude that that C compliment is a g-delta, and then find some smaller closed set contained in the interior of C that is also g-delta and I would be done but that is really just a shot in the dark and I haven't been able to get anywhere on it.[/quote]\r\nIf $ C$ is additionally assumed to have a countably locally finite basis, the topology of $ C$ is metrizable, by [url=http://en.wikipedia.org/wiki/Nagata-Smirnov_metrization_theorem]Nagata-Smirnov[/url]. This implies that the interior of $ C$ can be exhausted by countably many closed subsets." } { "Tag": [], "Problem": "Find the median of this set of data: \\[ \\{68,12,28,41,45,29,33,67,17,19,22,34,54,58,25\\}.\\]", "Solution_1": "Beasting with guessing, we see that $ \\boxed{33}$ is the middle number (there's an odd number of elements).", "Solution_2": "\\[ \\{68,12,28,41,45,29,33,67,17,19,22,34,54,58,25\\}\r\n\\]\r\n\r\nEliminating the top 3 big (68,67,58) and the smallest 3 (12,17,19)\r\n\\[ \\{28,41,45,29,33,22,34,54,25\\}\r\n\\]\r\n\r\nEliminating 54 and 22\r\n\r\n\\[ \\{28,41,45,29,33,34,25\\}\r\n\\]\r\n\r\n25 and 45, 28 and 41\r\n\r\n\\[ \\{29,33,34\\}\r\n\\]\r\n\r\nAnd we see the median is 33." } { "Tag": [ "geometry", "inequalities", "circumcircle", "triangle inequality", "geometry proposed" ], "Problem": "It is given a regular hexagon in the plane. Let P be a point of the plane. Define s(P) as the sum of the distances from P to each side of the hexagon, and v(P) as the sum of the distances from P to each vertex.\r\na) Find the locus of points P that minimize s(P)\r\nb) Find the locus of points P that minimize v(P)", "Solution_1": "If $a$ is the sidelenght of the regular hexagon $A_{1}...A_{6}$then $a.s(P)=\\sum_{i=1}^{6}[PA_{i}A_{i+1}]\\geq[A_{1}...A_{6}]=const$ ($[XYZ]=$area($XYZ$)) therefore the locus of $P$ when $s(P)$ is minimized is the regular hexagon.\r\nFor the second one put the hexagon in the complex plane such that its surcumcircle is the unit curcle. Then use the triangle inequality to obtain that $v(P)$ is minimized when $P$ is the center.", "Solution_2": "Other way for the second with regular polygon $A_{1}..A_{n}$ center $(O)$ circumradius $R$ we have for all $M$ in plane $R(\\sum MA_{i})=\\sum OA_{i}MA_{i}\\ge\\sum\\vec{OA_{i}}\\vec{MA_{i}}=\\sum\\vec{OA_{i}}(\\vec{MO}+\\vec{OA_{i}})=\\vec{MO}(\\sum \\vec{OA_{i}})+nR^{2}=nR^{2}$ because in regular polygon $\\sum\\vec{OA_{i}}=\\vec{0}$\r\nThus $\\sum MA_{i}\\ge nR$ equality hold iff $M\\equiv O$" } { "Tag": [], "Problem": "$ f(\\frac {x \\minus{} 1}{x \\plus{} 1}) \\plus{} f(\\frac {1}{x}) \\plus{} f(\\frac {1 \\plus{} x}{1 \\minus{} x}) \\equal{} x$ \r\n\r\n$ f(x)$=?", "Solution_1": "[hide=\"solution\"]\n$ f\\left(\\frac {x \\minus{} 1}{x \\plus{} 1}\\right) \\plus{} f\\left(\\frac {1}{x}\\right) \\plus{} f\\left(\\frac {1 \\plus{} x}{1 \\minus{} x}\\right) \\equal{} x \\qquad(1)$\n\nlet $ y \\equal{} \\frac {x \\minus{} 1}{x \\plus{} 1} \\Longrightarrow x \\equal{} \\frac {y \\plus{} 1}{1 \\minus{} y}$\n\n$ \\therefore f(y) \\plus{} f\\left(\\frac {1 \\minus{} y}{y \\plus{} 1}\\right) \\plus{} f\\left(\\frac { \\minus{} 1}{y}\\right) \\equal{} \\frac {y \\plus{} 1}{1 \\minus{} y}$\nThen set $ y \\equal{} \\minus{} x$\n$ \\therefore f( \\minus{} x) \\plus{} f\\left(\\frac {1 \\plus{} x}{1 \\minus{} x}\\right) \\plus{} f\\left(\\frac {1}{x}\\right) \\equal{} \\frac {1 \\minus{} x}{1 \\plus{} x} \\qquad (2)$\n\nlet $ y \\equal{} \\frac {1 \\plus{} x}{1 \\minus{} x} \\Longrightarrow x \\equal{} \\frac {y \\minus{} 1}{y \\plus{} 1}$\n\n$ \\therefore f\\left(\\frac { \\minus{} 1}{y}\\right) \\plus{} f\\left(\\frac {1 \\plus{} y}{y \\minus{} 1}\\right) \\plus{} f(y) \\equal{} \\frac {y \\minus{} 1}{1 \\plus{} y}$\nthen set $ y \\equal{} \\minus{} x$\n$ \\therefore f\\left(\\frac {1}{x}\\right) \\plus{} f\\left(\\frac {x \\minus{} 1}{x \\plus{} 1}\\right) \\plus{} f( \\minus{} x) \\equal{} \\frac {x \\plus{} 1}{x \\minus{} 1} \\qquad(3)$ \n\nNow $ (2) \\minus{} (1)$ gives\n$ f( \\minus{} x) \\minus{} f\\left(\\frac {x \\minus{} 1}{x \\plus{} 1}\\right) \\equal{} \\frac {x \\minus{} 1}{x \\plus{} 1} \\minus{} x \\Leftrightarrow f( \\minus{} x) \\minus{} ( \\minus{} x) \\equal{} f\\left(\\frac {x \\minus{} 1}{x \\plus{} 1}\\right) \\minus{} \\left(\\frac {x \\minus{} 1}{x \\plus{} 1}\\right) \\qquad(*)$\n\nSimilarly $ (3) \\minus{} (2)$ gives\n$ f\\left(\\frac {x \\minus{} 1}{x \\plus{} 1}\\right) \\minus{} \\left(\\frac {x \\minus{} 1}{x \\plus{} 1}\\right) \\equal{} f\\left(\\frac {x \\plus{} 1}{1 \\minus{} x}\\right) \\minus{} \\left(\\frac {x \\plus{} 1}{1 \\minus{} x}\\right) \\qquad(**)$\n\n\nFrom (*) and (**) we find\n\n$ 3\\left(f\\left(\\frac {1}{x}\\right) \\minus{} \\frac {1}{x}\\right) \\equal{} x \\minus{} \\left(\\frac {x \\minus{} 1}{x \\plus{} 1} \\plus{} \\frac {1}{x} \\plus{} \\frac {1 \\plus{} x}{1 \\minus{} x}\\right)$\n\n$ \\therefore f\\left(\\frac {1}{x}\\right) \\equal{} \\frac {x^4 \\plus{} 5x^2 \\minus{} 2}{3x(x^2 \\minus{} 1)}$\n\n$ \\therefore f(x) \\equal{} \\frac {2x^4 \\minus{} 5x^2 \\minus{} 1}{3x(x^2 \\minus{} 1)}$ :lol: \n[/hide]" } { "Tag": [ "Asymptote" ], "Problem": "The vertical asymptotes of the graph of $ y\\equal{}\\frac{x^3\\minus{}5x^2\\minus{}17x\\plus{}21}{3x^2\\plus{}36x\\plus{}81}$ can be written in the form $ x\r\n\\plus{}Ny\\equal{}S$ and $ x\\equal{}A$, where $ N,S,$ and $ A$ are rational, real numbers. Find $ N * A *S *A$", "Solution_1": "Oblique symptotes are considered vertical?", "Solution_2": "Apparently anything not horizontal is vertical. (Come to think of it, a teacher in Kansas for five years taught her students that a kilometer is greater than a mile...)\r\n\r\nTo find the vertical asymptote, first factor both the numerator and denominator and cancel out a common factor. The remaining factor gives you the vertical asymptote, where the one that was cancelled out gives you a hole, but that is not needeed here. Then do long division to find the oblique asymptote." } { "Tag": [], "Problem": "Hi,\r\n\r\nI am competing in an Olympiad in 2 days (the AIMO - Australia), and they usually have 1 or 2 motion questions, like rates and really wordy motion problem solving questions. I would really appreciate it if anyone has any tips for me on these types of questions =) \r\n\r\nThanks", "Solution_1": "Average velocity is given by total distance traveled divided by the time. Remember, this is average velocity. Acceleration is given by the change in velocity divided by the time passed.\r\n\r\nThe first few equations on [url=http://emsc32.nysed.gov/osa/reftable/reftablearch/physics06tbl.pdf]page 6[/url] are the fundamental equations of motion in Newtonian mechanics.", "Solution_2": "[quote=\"JRav\"]Average velocity is given by total distance traveled divided by the time. Remember, this is average velocity. Acceleration is given by the change in velocity divided by the time passed.\n\nThe first few equations on [url=http://emsc32.nysed.gov/osa/reftable/reftablearch/physics06tbl.pdf]page 6[/url] are the fundamental equations of motion in Newtonian mechanics.[/quote]\r\n\r\nThanks for your help but i was more referring to mathematical problem solving questions on \"motion\" like this one\r\n\r\n[i]\"Two candles of equal length start burning at the same time. One of the candles will burn down in 4 hours, the other in 5 hours. How many hours will they burn before one candle is 3 times the length of the other?\"[/i]\r\n\r\nIts just these questions that i find difficult", "Solution_3": "Your goal on any problem of that type should be to turn the given information into a system of equations. Once you get those equations, there will be various ways to solve them, but the first step is to write out all of the information as an equation and identify the algebraic expression that you want to find.\r\n\r\nFor your problem:\r\n\r\nLet the candles burn for $ x$ hours and let $ y$ be the initial height of the candles. Then, after $ x$ hours, the first candle will be at height $ y\\minus{}\\frac{y\\cdot x}4\\equal{}y\\left(1\\minus{}\\frac{x}4\\right)$. The second candle will be at height $ y\\minus{}\\frac{y\\cdot x}5\\equal{}y\\left(1\\minus{}\\frac{x}5\\right)$.\r\n\r\nSince the first candle burns faster than the other candle, it will be at a lower height than the second candle, meaning that the second candle must be 3 times as tall as the first candle. We can write this as an equation to solve for $ x$:\r\n\r\n$ 3y\\left(1\\minus{}\\frac{x}4\\right)\\equal{}y\\left(1\\minus{}\\frac{x}5\\right)$\r\n$ 60\\minus{}15x\\equal{}20\\minus{}4x$\r\n$ 11x\\equal{}40$\r\n$ x\\equal{}\\frac{40}{11}$\r\n\r\nOnce you get everything expressed algebraically, the problem should become fairly simple to finish." } { "Tag": [ "geometry", "circumcircle", "algebra", "system of equations", "algebra open" ], "Problem": "$ \\theta _{1}, \\cdots ,\\theta _{n} , R$ are variables and $ d_{1}, \\cdots ,d_{n}$ are given.\r\nWe have following equations. Can we find a formula for $ \\theta _{1}, \\cdots ,\\theta _{n} , R$?\r\n(All variables and $ d_{i}$s are positive.)\r\n\r\n$ sin\\frac {\\theta_{1}}{2} \\equal{} \\frac {d_{1}}{2R}, \\cdots ,sin\\frac {\\theta_{n}}{2} \\equal{} \\frac {d_{n}}{2R}, \\theta_{1} \\plus{} \\cdots \\plus{} \\theta_{n} \\equal{} 2\\pi$", "Solution_1": "[hide=\"observation\"]The system of equations has a solution if and only if there exists a cyclic $ n$-gon with sides of length $ d_1,d_2,...,d_n$ and its circumcenter is within it. $ R$ is then the radius of its circumscribed circle, and $ \\theta _n \\equal{} 2\\arcsin \\frac {d_n}{2R}$[/hide]" } { "Tag": [ "vector" ], "Problem": "Ok, i think i found a counterexemple. Here it goes:\r\n\r\nSuppose that $ n \\equal{} 2$, $ X \\equal{} {x_1,...,x_r}$,$ A_1 \\equal{} {x_1,...,x_{r \\minus{} 1} }$ and $ f(x_i) \\equal{} x_i$, for $ i \\equal{} 1,r \\minus{} 1$ and $ f(x_r) \\equal{} x_1$. Then the nice vectors are $ (x_1,x_1),...,(x_{r \\minus{} 1},x_{r \\minus{} 1}),(x_1,x_r),(x_r,x_1)$ .The number of nice vectors is $ r \\plus{} 1$ while $ \\frac {|X|^n}{\\prod\\limits_{i \\equal{} 1}^{n \\minus{} 1} |A_i|}$=$ \\frac {r^2}{r \\minus{} 1}$, contradiction. \r\n\r\nI hope i understand correctly the problem. Maybe is there another condition? :wink:\r\n\r\n\r\nEdit: Yes, I'm the idiot...", "Solution_1": "i think you left out $ (x_r,x_r)$ ..." } { "Tag": [], "Problem": "The double-bar graph shows the number of home runs hit by McGwire and Sosa during each month of the 1998 baseball season. At the end of which month were McGwire and Sosa tied in total number of home runs?\n\n[asy]draw((0,0)--(28,0)--(28,21)--(0,21)--(0,0)--cycle,linewidth(1));\n\nfor(int i = 1; i < 21; ++i)\n{\n\tdraw((0,i)--(28,i));\n}\n\nfor(int i = 0; i < 8; ++i)\n{\n\tdraw((-1,3i)--(0,3i));\n}\n\nlabel(\"0\",(-1,0),W);\nlabel(\"3\",(-1,3),W);\nlabel(\"6\",(-1,6),W);\nlabel(\"9\",(-1,9),W);\nlabel(\"12\",(-1,12),W);\nlabel(\"15\",(-1,15),W);\nlabel(\"18\",(-1,18),W);\nlabel(\"21\",(-1,21),W);\n\nfor(int i = 0; i < 8; ++i)\n{\n\tdraw((4i,0)--(4i,-1));\n}\n\nfilldraw((1,0)--(2,0)--(2,1)--(1,1)--(1,0)--cycle,gray,linewidth(1));\nfilldraw((5,0)--(6,0)--(6,10)--(5,10)--(5,0)--cycle,gray,linewidth(1));\nfilldraw((9,0)--(10,0)--(10,16)--(9,16)--(9,0)--cycle,gray,linewidth(1));\nfilldraw((13,0)--(14,0)--(14,10)--(13,10)--(13,0)--cycle,gray,linewidth(1));\nfilldraw((17,0)--(18,0)--(18,8)--(17,8)--(17,0)--cycle,gray,linewidth(1));\nfilldraw((21,0)--(22,0)--(22,10)--(21,10)--(21,0)--cycle,gray,linewidth(1));\nfilldraw((25,0)--(26,0)--(26,15)--(25,15)--(25,0)--cycle,gray,linewidth(1));\n\nfilldraw((6,0)--(7,0)--(7,6)--(6,6)--(6,0)--cycle,black,linewidth(1));\nfilldraw((10,0)--(11,0)--(11,7)--(10,7)--(10,0)--cycle,black,linewidth(1));\nfilldraw((14,0)--(15,0)--(15,20)--(14,20)--(14,0)--cycle,black,linewidth(1));\nfilldraw((18,0)--(19,0)--(19,9)--(18,9)--(18,0)--cycle,black,linewidth(1));\nfilldraw((22,0)--(23,0)--(23,13)--(22,13)--(22,0)--cycle,black,linewidth(1));\nfilldraw((26,0)--(27,0)--(27,11)--(26,11)--(26,0)--cycle,black,linewidth(1));\n\nlabel(\"Mar\",(2,0),S);\nlabel(\"Apr\",(6,0),S);\nlabel(\"May\",(10,0),S);\nlabel(\"Jun\",(14,0),S);\nlabel(\"Jul\",(18,0),S);\nlabel(\"Aug\",(22,0),S);\nlabel(\"Sep\",(26,0),S);[/asy]\n\n[asy]draw((30,6)--(40,6)--(40,15)--(30,15)--(30,6)--cycle,linewidth(1));\nfilldraw((31,7)--(34,7)--(34,10)--(31,10)--(31,7)--cycle,black,linewidth(1));\nfilldraw((31,11)--(34,11)--(34,14)--(31,14)--(31,11)--cycle,gray,linewidth(1));\n\nlabel(\"McGwire\",(36,12.5));\nlabel(\"Sosa\",(36,8.5));[/asy]", "Solution_1": "Keep counting and counting... it's August." } { "Tag": [ "analytic geometry", "geometry", "circumcircle", "conics", "geometry unsolved" ], "Problem": "[color=darkblue]Let $M$ be a point with the barycentrical coordinates $(x,y,z)$ w.r.t. the given triangle $ABC$ with the incircle $C(I,r)$ and the circumcircle $C(O,R)\\ .$ \nProve that the point $M$ belongs to the incircle if and only if there is the relation $\\boxed{\\ \\sum bcx=\\sum yza^{2}+r(4R+r)\\ }\\ .$\n[b]Remark 1.[/b] The power $p(M)$ of the point $M$ w.r.t. the circumcircle of the given triangle is $p(M)=-\\sum yza^{2}\\ .$\n[b]Remark 2.[/b] If $a=b=c$, i.e. the triangle $ABC$ is equilateral, then $MI=r$ $\\Longleftrightarrow$ $xy+yz+zx=\\left(\\frac{a}{2}\\right)^{2}\\ .$\n[b]Can anyone[/b] (using, if need be, the above result), for example [u]M4rio[/u], [u]Shobber[/u], [u]Yetti[/u], [u]Darij G.[/u] a.s.o., generalize the nice problem from the post http://www.mathlinks.ro/Forum/viewtopic.php?t=106446 ?[/color]", "Solution_1": "In an equilateral triangle $ABC$, with lenght:$L$, a point $P$ belongs to the incircle if: $\\boxed{(AP)^{2}+(BP)^{2}+(CP)^{2}=\\frac{5L^{2}}{4}}$", "Solution_2": "[quote=\"M4RI0\"]In an equilateral triangle $ABC$, with lenght $a$, a point $P$ belongs to the incircle if $\\boxed{(AP)^{2}+(BP)^{2}+(CP)^{2}=\\frac{5a^{2}}{4}}$[/quote]\r\n[color=darkblue]Sorry, M4rio, I am interested by the general case of characterization in an any triangle $ABC$ when the point $P$ belongs to the incircle.\n\nThe mentioned property in a equilateral triangle is easily. Generally, $PA^{2}+PB^{2}+PC^{2}=3\\cdot PG^{2}+\\frac{a^{2}+b^{2}+c^{2}}{3}\\ .$\n\nParticularly, if $a=b=c$, then $r=\\frac{a\\sqrt 3}{6}$, $G\\equiv I$ and $P\\in C(I,r)$ $\\Longleftrightarrow$ $PG=r$ $\\Longleftrightarrow$ $\\sum PA^{2}=\\frac{5a^{2}}{4}\\ .$[/color]", "Solution_3": "[quote=\"Virgil Nicula\"][b]Can anyone[/b] (using, if need be, the above result), for example [u]M4rio[/u], [u]Shobber[/u], [u]Yetti[/u], [u]Darij G.[/u] a.s.o., generalize the nice problem from the post http://www.mathlinks.ro/Forum/viewtopic.php?t=106446 ?[/color][/quote]\r\n\r\nIn fact, these properties follow from the barycentric equation of an inscribed conic in triangle ABC:\r\n\r\nLet P be a point with barycentric coordinates $P\\left(p: q: r\\right)$ with respect to triangle ABC. Let $A^{\\prime}=AP\\cap BC$, $B^{\\prime}=BP\\cap CA$, $C^{\\prime}=CP\\cap AB$. Then, there exists one and only one conic touching the lines BC, CA, AB at the points A', B', C', and the barycentric equation of this conic is\r\n\r\n$\\frac{x^{2}}{p^{2}}+\\frac{y^{2}}{q^{2}}+\\frac{z^{2}}{r^{2}}-\\frac{2yz}{qr}-\\frac{2zx}{rp}-\\frac{2xy}{pq}=0$.\r\n\r\nThis equation can be rewritten as\r\n\r\n$\\left(\\sqrt{\\frac{x}{p}}+\\sqrt{\\frac{y}{q}}+\\sqrt{\\frac{z}{r}}\\right)\\left(-\\sqrt{\\frac{x}{p}}+\\sqrt{\\frac{y}{q}}+\\sqrt{\\frac{z}{r}}\\right)\\left(\\sqrt{\\frac{x}{p}}-\\sqrt{\\frac{y}{q}}+\\sqrt{\\frac{z}{r}}\\right)\\left(\\sqrt{\\frac{x}{p}}+\\sqrt{\\frac{y}{q}}-\\sqrt{\\frac{z}{r}}\\right)$\r\n$=0$.\r\n\r\nIf P is the Gergonne point of triangle ABC, with its barycentric coordinates $\\left(p: q: r\\right)=\\left(\\frac{1}{s-a}: \\frac{1}{s-b}: \\frac{1}{s-c}\\right)$, then the conic becomes the incircle of triangle ABC. Hence, the barycentric equation of the incircle is\r\n\r\n$\\left(\\sqrt{\\left(s-a\\right)x}+\\sqrt{\\left(s-b\\right)y}+\\sqrt{\\left(s-c\\right)z}\\right)\\left(-\\sqrt{\\left(s-a\\right)x}+\\sqrt{\\left(s-b\\right)y}+\\sqrt{\\left(s-c\\right)z}\\right)$\r\n$\\left(\\sqrt{\\left(s-a\\right)x}-\\sqrt{\\left(s-b\\right)y}+\\sqrt{\\left(s-c\\right)z}\\right)\\left(\\sqrt{\\left(s-a\\right)x}+\\sqrt{\\left(s-b\\right)y}-\\sqrt{\\left(s-c\\right)z}\\right)$\r\n$=0$.\r\n\r\nThis generalizes M4rio's result at http://www.mathlinks.ro/Forum/viewtopic.php?t=106446 .\r\n\r\nFor a systematic treatise of inscribed conics, see \u00a710.2 of\r\n\r\n[url=http://www.math.fau.edu/yiu/geometry.html]Paul Yiu, [i]An Introduction to the Geometry of the Triangle[/i], 2001[/url].\r\n\r\n darij", "Solution_4": "well It follows that $IM=r$ so $S_{A}(a/2p-x)^{2}+S_{B}(b/2p-y)^{2}+S_{C}(c/2p-z)^{2}=r^{2}$", "Solution_5": "Is there any free book for download on barycentric coordinates?", "Solution_6": "See [url=http://www.math.fau.edu/yiu/geometry.html]Paul Yiu's \"An Introduction to the Geometry of the Triangle\"[/url]. I know it from Darij's posts.", "Solution_7": "[quote=\"dule_00\"]Is there any free book for download on barycentric coordinates?[/quote]\r\n\r\nI have made one but it's in Spanish", "Solution_8": "perfect_radio, thank you for a book!!! :)" } { "Tag": [ "limit", "trigonometry", "LaTeX", "real analysis", "real analysis unsolved" ], "Problem": "Compute the value of the limit \r\n $lim_{x\\to\\{0}}\\sum_{i=1}^{n}\\ (i^{sin^{2}x)^{\\frac{1}{sin^{2}x}}$", "Solution_1": "${\\lim_{x\\to 0}\\, \\left(\\sum_{i=1}^{n}i^{\\sin^{2}(x)^{\\frac{1}{\\sin^{2}(x)}}}\\right)}$\r\n\r\n${\\sin (x)=k}$\r\n${i^{\\sin^{2}(x)^{\\frac{1}{\\sin^{2}(x)}}}=i^{\\left(k^{2}\\right)^{\\frac{1}{k^{2}}}}}$\r\n\r\nSubstituing for the new limit\r\n\r\n${\\sin (x)=k}$\r\n${\\sin (0)=k}$\r\n${k\\to 0}$\r\n\r\n${\\lim_{x\\to 0}\\, \\left(\\sum_{i=1}^{n}i^{\\sin^{2}(x)^{\\frac{1}{\\sin^{2}(x)}}}\\right)=\\lim_{k\\to 0}\\, \\left(\\sum_{i=1}^{n}i^{\\left(k^{2}\\right)^{\\frac{1}{k^{2}}}}\\right)}$\r\n${\\lim_{x\\to 0}\\, \\left(\\sum_{i=1}^{n}i^{\\sin^{2}(x)^{\\frac{1}{\\sin^{2}(x)}}}\\right)=\\sum_{i=1}^{n}\\lim_{k\\to 0}\\, i^{\\left(k^{2}\\right)^{\\frac{1}{k^{2}}}}}$\r\n\r\nIts Easy to see\r\n\r\n${\\lim_{k\\to 0}\\, i^{\\left(k^{2}\\right)^{\\frac{1}{k^{2}}}}=1}$\r\n\r\n${\\lim_{x\\to 0}\\, \\left(\\sum_{i=1}^{n}i^{\\sin^{2}(x)^{\\frac{1}{\\sin^{2}(x)}}}\\right)=\\sum_{i=1}^{n}1=n}$", "Solution_2": "[quote=\"ashwinkrishnan\"]Compute the value of the limit \n $lim_{x\\to\\{0}}\\sum_{i=1}^{n}\\ (i^{sin^{2}x)^{\\frac{1}{sin^{2}x}}$[/quote]\r\n\r\nhttp://ivanisvili.narod.ru/Extr2.doc", "Solution_3": "The exponent $\\frac{1}{sin^{2}x}$ is over the whole summation and not just over each term.I couldn't do that using Latex", "Solution_4": "[quote=\"ashwinkrishnan\"]The exponent $\\frac{1}{sin^{2}x}$ is over the whole summation and not just over each term.I couldn't do that using Latex[/quote]\r\nI don't understand what is wrong? the limit of the sum is equal the sum of limit. and we use it for each ... it is easy", "Solution_5": "Right , doesn't make any difference" } { "Tag": [ "number theory", "least common multiple" ], "Problem": "How many positive integers less than 100 and divisible by 3 are also divisible by 4?", "Solution_1": "[hide]if a number is divisible by 4 and 3, then its divisible by 12. so, there are 8 numbers less than 100.[/hide]", "Solution_2": "[hide]numbers divisible by 3&4 are divisible by 12, and 8*12 is the highest under 100, so the answer is 8[/hide]", "Solution_3": "[hide]Since 4*3 is twelve, I have to find how many multiples of twelve there are below 100. The answer is 8.[/hide]", "Solution_4": "[quote=\"joejia\"]How many positive integers less than 100 and divisible by 3 are also divisible by 4?[/quote]\r\n\r\n8, since the lcm of 3 an d4 are 12, and there are 8 multiples of 12 under 100.", "Solution_5": "anirudh, can you [i]please[/i] hide your answer ;)", "Solution_6": "[hide]\n100/3 is about 33. So 33 numbers are divisible by 3. 33/4 is about 8. So 8 of the 33 numbers divisble by 3 are divisble by 4.\n[/hide]" } { "Tag": [ "inequalities", "logarithms", "inequalities proposed" ], "Problem": "Let $a$, $b$, $c$ be positive real numbers such that $abc=1$. Prove that\n\\[\\frac a{a^{2}+2}+\\frac b{b^{2}+2}+\\frac c{c^{2}+2}\\leq 1 \\]", "Solution_1": "$\\sum{\\frac{a}{a^2+2}}\\leq\\sum{\\frac{a}{2a+1}}$.\r\n\r\nWe shall prove that $\\sum{\\frac{a}{2a+1}}\\leq{1}$ or\r\n $\\sum{\\frac{2a}{2a+1}}\\leq{2}$ or \r\n $\\sum{\\frac{1}{2a+1}}\\geq{1}$.\r\n\r\nClearing the denominator we have to prove that:\r\n\r\n $2\\sum{a}\\geq{6}$ wich is true by AM-GM.", "Solution_2": "a/a^2+2=0,abc\\equal{}1\\iff{k_1\\le{k}\\le{k_2}}$\r\n\r\nin which\r\n\r\n $ k_1\\equal{}{\\frac{{59\\minus{}9\\sqrt{59}\\minus{}3{\\sqrt{646\\minus{}118{\\sqrt{29}}}}}}{4}}\\equal{}0 .19725809...,$\r\n\r\n$ k_2\\equal{}{\\frac{{59\\minus{}9\\sqrt{59}\\plus{}3{\\sqrt{646\\minus{}118{\\sqrt{29}}}}}}{4}}\\equal{}5.06950026....$", "Solution_25": "[quote=\"can_hang2007\"][quote=\"Vasc\"][quote=\"arqady\"] \nBy the way, $ \\frac a{a^{2} \\plus{} 5} \\plus{} \\frac b{b^{2} \\plus{} 5} \\plus{} \\frac c{c^{2} \\plus{} 5}\\leq \\frac {1}{2}$ is wrong.[/quote]\nIt is true, arqady. :)[/quote]\nI found now a proof by Cauchy Schwarz Inequality for this hard inequality! :idea:[/quote]\r\n\r\nCongratulation can_hang2007! :lol: I really want to see your Cauchy-Schwarz proof. Could you post it?", "Solution_26": "Does this way work for this problem:\r\n$ \\frac{4a}{a^2\\plus{}5} \\le \\frac{x\\plus{}1}{x^2\\plus{}x\\plus{}1}$\r\nwith $ x\\equal{}\\frac{1}{a^{\\frac{3}{4}}}$", "Solution_27": "[quote=\"speciallove\"]Does this way work for this problem:\n$ \\frac {4a}{a^2 \\plus{} 5} \\le \\frac {x \\plus{} 1}{x^2 \\plus{} x \\plus{} 1}$\nwith $ x \\equal{} \\frac {1}{a^{\\frac {3}{4}}}$[/quote]\r\n\r\nYour way is a very interesting, but this inequality is not hold, try $ a\\equal{}2,$ this inequality become\r\n$ 8x^{2}\\le x\\plus{}1,$ where $ x\\equal{}\\frac{1}{2^{\\frac{3}{4}}}<1$\r\n$ \\Rightarrow2\\sqrt{2}\\equal{}8x^{2}\\le x\\plus{}1<2$\r\ncontradiction.", "Solution_28": "sorry,I want to say $ k\\equal{} \\frac{\\minus{}4}{3}$ not $ \\frac{\\minus{}3}{4}$\r\nBecause I will find k for the inequality such that:\r\n$ \\frac{4a}{a^2\\plus{}5} \\le \\frac{a^k\\plus{}1}{a^{2k}\\plus{}a^k\\plus{}1}$\r\nor $ f(k)\\equal{}(a^k\\plus{}1)(a^2\\plus{}5)\\minus{}4a(a^{2k}\\plus{}a^k\\plus{}1) \\ge 0$\r\nf('1)=6k+8=0[/tex] or $ k\\equal{} \\frac{\\minus{}4}{3}$\r\nwith $ k\\equal{}\\frac{\\minus{}4}{3}$ we have $ x\\equal{}\\frac{1}{a^{\\frac{4}{3}}}$", "Solution_29": "[quote=\"speciallove\"]sorry,I want to say $ k \\equal{} \\frac { \\minus{} 4}{3}$ not $ \\frac { \\minus{} 3}{4}$\nBecause I will find k for the inequality such that:\n$ \\frac {4a}{a^2 \\plus{} 5} \\le \\frac {a^k \\plus{} 1}{a^{2k} \\plus{} a^k \\plus{} 1}$\nor $ f(k) \\equal{} (a^k \\plus{} 1)(a^2 \\plus{} 5) \\minus{} 4a(a^{2k} \\plus{} a^k \\plus{} 1) \\ge 0$\nf('1)=6k+8=0[/tex] or $ k \\equal{} \\frac { \\minus{} 4}{3}$\nwith $ k \\equal{} \\frac { \\minus{} 4}{3}$ we have $ x \\equal{} \\frac {1}{a^{\\frac {4}{3}}}$[/quote]\r\n\r\nBut this inequality is still not hold for all $ a>0,$ try $ a\\equal{}\\frac{1}{4},$ this inequality become\r\n$ 16x^{2}\\le65x\\plus{}65,$ where $ x\\equal{}4^{\\frac{4}{3}}>6$\r\n$ \\Rightarrow 65x\\plus{}65<65x\\plus{}11x\\equal{}76x<16x^{2}$\r\ncontradiction.", "Solution_30": "[quote=\"Vasc\"][quote=\"arqady\"] \nBy the way, $ \\frac a{a^{2} \\plus{} 5} \\plus{} \\frac b{b^{2} \\plus{} 5} \\plus{} \\frac c{c^{2} \\plus{} 5}\\leq \\frac {1}{2}$ is wrong.[/quote]\nIt is true, arqady. :)[/quote]\r\n\r\nThis is just... so easy :lol: \r\nHere is a pretty elementary proof.", "Solution_31": "Very nice proof, Inequalities Master :)", "Solution_32": "[quote=\"fjwxcsl\"][quote=\"n0vakovic\"]Let $ a$, $ b$, $ c$ be positive real numbers such that $ abc \\equal{} 1$. Proove that\n\\[ \\frac a{a^{2} \\plus{} 2} \\plus{} \\frac b{b^{2} \\plus{} 2} \\plus{} \\frac c{c^{2} \\plus{} 2}\\leq 1\\]\n[/quote]\n\n$ \\frac a{a^{2} \\plus{} k} \\plus{} \\frac b{b^{2} \\plus{} k} \\plus{} \\frac c{c^{2} \\plus{} k}\\leq {\\frac{3}{1\\plus{}k}}$\n\nholds $ \\forall {a,b,c}>0,abc\\equal{}1\\iff{k_1\\le{k}\\le{k_2}}$\n\nin which\n\n $ k_1\\equal{}{\\frac{{59\\minus{}9\\sqrt{59}\\minus{}3{\\sqrt{646\\minus{}118{\\sqrt{29}}}}}}{4}}\\equal{}0 .19725809...,$\n\n$ k_2\\equal{}{\\frac{{59\\minus{}9\\sqrt{59}\\plus{}3{\\sqrt{646\\minus{}118{\\sqrt{29}}}}}}{4}}\\equal{}5.06950026....$[/quote]\nBut can we use a way to get the best k ?", "Solution_33": "[quote=\"fjwxcsl\"][quote=\"n0vakovic\"]Let $ a$, $ b$, $ c$ be positive real numbers such that $ abc \\equal{} 1$. Proove that\n\\[ \\frac a{a^{2} \\plus{} 2} \\plus{} \\frac b{b^{2} \\plus{} 2} \\plus{} \\frac c{c^{2} \\plus{} 2}\\leq 1\\]\n[/quote]\n\n$ \\frac a{a^{2} \\plus{} k} \\plus{} \\frac b{b^{2} \\plus{} k} \\plus{} \\frac c{c^{2} \\plus{} k}\\leq {\\frac{3}{1\\plus{}k}}$\n\nholds $ \\forall {a,b,c}>0,abc\\equal{}1\\iff{k_1\\le{k}\\le{k_2}}$\n\nin which\n\n $ k_1\\equal{}{\\frac{{59\\minus{}9\\sqrt{59}\\minus{}3{\\sqrt{646\\minus{}118{\\sqrt{29}}}}}}{4}}\\equal{}0 .19725809...,$\n\n$ k_2\\equal{}{\\frac{{59\\minus{}9\\sqrt{59}\\plus{}3{\\sqrt{646\\minus{}118{\\sqrt{29}}}}}}{4}}\\equal{}5.06950026....$[/quote]\nLet me ask you this: Why?", "Solution_34": "[quote=\"n0vakovic\"]Let $a$, $b$, $c$ be positive real numbers such that $abc=1$. Prove that\n\\[\\frac a{a^{2}+2}+\\frac b{b^{2}+2}+\\frac c{c^{2}+2}\\leq 1 \\][/quote]\n\nLet $a=\\dfrac{x}{y}, b=\\dfrac{y}{z}, =\\dfrac{z}{x}$\n\n$\\sum{\\frac{a}{a^2+2}}=\\sum{\\dfrac{\\dfrac{x}{y}}{\\dfrac{x^2}{y^2}+2}=\\sum{\\dfrac{xy}{x^2+2y^2}}\\leq\\sum{\\dfrac{xy}{2xy+y^2}}=\\sum{\\dfrac{x}{2x+y}}}$\n\n$\\sum{\\dfrac{x}{2x+y}}\\leq1$\n\n$\\Longleftrightarrow\\sum{\\dfrac{2x}{2x+y}}\\leq 2$\n\n$\\Longleftrightarrow\\sum{-\\dfrac{2x}{2x+y}}\\geq-2$\n\n$\\Longleftrightarrow\\sum{1-\\dfrac{2x}{2x+y}}\\geq1$\n\n$\\Longleftrightarrow\\sum{\\dfrac{y}{2x+y}}\\geq1$\n\n$\\Longleftrightarrow \\dfrac{y}{2x+y}+\\dfrac{z}{2y+z}+\\dfrac{x}{2z+x}\\geq1$\n\n$\\Longleftrightarrow \\dfrac{y^2}{2xy+y^2}+\\dfrac{z^2}{2yz+z^2}+\\dfrac{x^2}{2xz+x^2}\\geq\\dfrac{(x+y+z)^2}\n{x^2+y^2+z^2+2xy+2xz+2yz}=1$ and we are done.", "Solution_35": "for #1 \nWith condition $abc=1$,We have\n\\[1-\\sum{\\frac{a}{a^2+2}}=\\frac{\\sum{(9+16c^2)(a-b)^2}+\\sum{(23+8c+20c^2)(a+b-2)^2}}{36(a^2+2)(b^2+2)(c^2+2)}\\]\n\nNotice:\nThe condition $abc=1$ can be replaced by $a+b+c=1(or a^2+b^2+c^2=1 or ab+bc+ca=1)$, inequality still holds!", "Solution_36": "[quote=zaizai-hoang]\nThe following inequality is also holds:\nLet $ a$, $ b$, $ c$ be positive real numbers such that $ abc \\equal{} 1$. Proove that\n\\[ \\frac a{a^{2} \\plus{} 3} \\plus{} \\frac b{b^{2} \\plus{} 3} \\plus{} \\frac c{c^{2} \\plus{}3}\\leq \\frac{3}{4}\n\\]\n\nI think it is more difficult than above inequality!!![/quote]\nSee here:\nhttp://www.artofproblemsolving.com/community/c6h1195493p5852610\n\n", "Solution_37": "solution here:", "Solution_38": "Let $a$, $b$, $c$ be positive real numbers such that $ab+bc+ca=1$. Prove that\n$$ \\frac{a}{a^{2}+2}+\\frac{b}{b^{2}+2}+\\frac{c}{c^{2}+2}\\leq \\frac{3\\sqrt 3}{7}$$" } { "Tag": [], "Problem": "Hi. Im from Michigan so I dont get all of this canadian stuff. Can someone explain:\r\n\r\n1) What are some contests that the COMC leads to? Ive heard of stuff like Repechage and CMOR. What are those contests, and how can you be invited to them? Are there any similar contests that the COMC leads to? Are they only for Canadian people? \r\n\r\n2) What do you guys think the CMO cutoff will be this year? And when do we get our COMC scores?\r\n\r\nThanks", "Solution_1": "[quote=\"Thunder365\"]1) What are some contests that the COMC leads to? Ive heard of stuff like Repechage and CMOR. What are those contests, and how can you be invited to them? Are there any similar contests that the COMC leads to? Are they only for Canadian people? \n\n2) What do you guys think the CMO cutoff will be this year? And when do we get our COMC scores?[/quote]\r\n\r\n1) The COMC leads to the CMO... the CMOR (CMO Repechage) is a chance for the top 75 students who didn't make it by default to try again to get to the CMO - it's a one-week take-home contest that you do on your own time and submit through email. Only the top 25 get in, and based on what I remember from last year, you pretty much need to get every single question right.\r\n\r\n2) The CMO cutoff is probably not going to change much - it'll probably be in the 72 to 74 range. For the repechage, it might be as low as 64, or as high as 69.", "Solution_2": "[quote=\"sdkudrgn88\"][quote=\"Thunder365\"]1) What are some contests that the COMC leads to? Ive heard of stuff like Repechage and CMOR. What are those contests, and how can you be invited to them? Are there any similar contests that the COMC leads to? Are they only for Canadian people? \n\n2) What do you guys think the CMO cutoff will be this year? And when do we get our COMC scores?[/quote]\n\n1) The COMC leads to the CMO... the CMOR (CMO Repechage) is a chance for the top 75 students who didn't make it by default to try again to get to the CMO - it's a one-week take-home contest that you do on your own time and submit through email. Only the top 25 get in, and based on what I remember from last year, you pretty much need to get every single question right.\n\n2) The CMO cutoff is probably not going to change much - it'll probably be in the 72 to 74 range. For the repechage, it might be as low as 64, or as high as 69.[/quote]\r\nwell...the only way for u to make CMO from CMOR and did get perfect on it is to have atleast 1 or 2 questions that had some really mind blowing answers. but getting perfect is alot easier =D", "Solution_3": "So basically 65(?) kids make CMO and 75 more are invited to take the CMOR, of which only 25 kids get selected to take the CMO?", "Solution_4": "[quote=\"Thunder365\"]So basically 65(?) kids make CMO and 75 more are invited to take the CMOR, of which only 25 kids get selected to take the CMO?[/quote]\r\nuhh...about 75 for CMO and about 200ish for CMOR", "Solution_5": "actually, about 50 for CMO and about 100 - 150 for CMOR ...\r\nI had 72 last year.. putting me in 49th.. and I was given the CMOR :(\r\nI didn't make CMO.. :(", "Solution_6": "[quote=\"azn_rocker0322\"]actually, about 50 for CMO and about 100 - 150 for CMOR ...\nI had 72 last year.. putting me in 49th.. and I was given the CMOR :(\nI didn't make CMO.. :([/quote]\r\nwell...50 directly from COMC then 25 from from CMOR so total for 75.", "Solution_7": "[quote=\"azn_rocker0322\"]actually, about 50 for CMO and about 100 - 150 for CMOR ...\nI had 72 last year.. putting me in 49th.. and I was given the CMOR :(\nI didn't make CMO.. :([/quote]\r\nDo you know the cutoff for last year? I think the cutoff this year is somewhat similar, if not slightly lower(like 72-74).", "Solution_8": "What was the cutoff for the repechage last year?", "Solution_9": "Wait, let me get this straight...If I live in Michigan, can I qualify for the Repechage???\r\n\r\nAnd how good of a shot would a 68 be at making repechage?", "Solution_10": "[quote=\"Thunder365\"]Wait, let me get this straight...If I live in Michigan, can I qualify for the Repechage???\n\nAnd how good of a shot would a 68 be at making repechage?[/quote]\r\n\r\nOf course! Your chance of making Repechage is very high.", "Solution_11": "Thunder, congratulations.\r\nYou have qualified for CMO. Not Repechage.\r\n :D", "Solution_12": "Ummm the cliu, the reason khetan posted the pictures with the cutoff of 68 was because he said LAST year the cutoff was 68... but imo this year was harder, so idk, theres still a chance 68 is cmo..", "Solution_13": "Hmm...weird.\r\n\r\nWell this is because I'm not in III.\r\n\r\nAnd this is why Khetan should move me to III :mad:" } { "Tag": [ "LaTeX" ], "Problem": "Where can I download the free version of Latex?\r\n\r\ncheers! :D", "Solution_1": "the instructions you should follow in order to use LaTeX to create a .pdf file are in this thread: [url=http://www.mathlinks.ro/viewtopic.php?t=38]http://www.mathlinks.ro/viewtopic.php?t=38[/url]", "Solution_2": "the interface is very similiar to Microsoft Visual C++" } { "Tag": [], "Problem": "The deadline for submitting solutions via the website is now [b]3 PM Eastern / Noon Pacific[/b] of the due date. \r\n\r\nThe reason for this is twofold:\r\n\r\nFirst, now if you have any difficulty submitting your solutions by the web, you still have time to go to the post office and mail your solutions later that day by the postmark deadline.\r\n\r\nSecond, if there is a website problem or other problem with the web submission near the deadline, we can be aware of it and try to fix it during the day while people are in the office, instead of it being late at night when no one is here.\r\n\r\nThis is not a big change: by my calculations we've reduced the number of hours that you have to work on Round 3 from 975 to 963, a 1.2% reduction.", "Solution_1": "Ha ha ha. Of course on a forum like this everyone does the percentages. Two very good reasons for the change, though. Also, maybe this will help some people procrastinate 2-3% less by actually getting it done at least a day before.", "Solution_2": "Yeah, now I'm going to have to pretend the due date is January 11th so I don't procrastinate. Not a big deal at all though. :)" } { "Tag": [ "algebra unsolved", "algebra" ], "Problem": "Let $ a,b,c,d$ be the real numbers.\r\nProve that:\r\nIf the roots of equation $ ax^{2}\\plus{}(b\\plus{}c)x\\plus{}d\\plus{}e \\equal{} 0$ in $ [1;\\plus{}\\infty)$\r\nthen the equation $ ax^{4}\\plus{}bx^{3}\\plus{}cx^{2}\\plus{}dx\\plus{}e\\equal{}0$ so have roots in $ [1;\\plus{}\\infty)$.\r\n( :maybe: )\r\n[i]Louis Latin and Vicky[/i]", "Solution_1": "A link from diendantoanhoc.net and not English :lol: \r\n\r\nhttp://diendantoanhoc.net/forum/index.php?showtopic=35517\r\n\r\nHave fun :)", "Solution_2": "Sorri .But this is a exciting prolems.Can you show the solutions of them !", "Solution_3": "[quote=\"Vicky and Louis Latin\"]Let $ a,b,c,d$ be the real numbers.\nProve that:\nIf the roots of equation $ ax^{2} \\plus{} (b \\plus{} c)x \\plus{} d \\plus{} e \\equal{} 0$ in $ [1; \\plus{} \\infty)$\nthen the equation $ ax^{4} \\plus{} bx^{3} \\plus{} cx^{2} \\plus{} dx \\plus{} e \\equal{} 0$ so have roots in $ [1; \\plus{} \\infty)$.\n( :maybe: )\n[i]Louis Latin and Vicky[/i][/quote]\r\n\r\nUnfortunately, this is wrong :\r\n\r\nLet $ a \\equal{} 1$, $ b \\equal{} 1$, $ c \\equal{} \\minus{} 5$, $ d \\equal{} 4$ and $ e \\equal{} 0$\r\n\r\nThen $ f(x) \\equal{} ax^{2} \\plus{} (b \\plus{} c)x \\plus{} d \\plus{} e \\equal{} x^2 \\minus{} 4x \\plus{} 4 \\equal{} (x \\minus{} 2)^2$ has its two roots (2 and 2) in $ [1; \\plus{} \\infty)$\r\nBut $ g(x) \\equal{} ax^{4} \\plus{} bx^{3} \\plus{} cx^{2} \\plus{} dx \\plus{} e \\equal{} x^4 \\plus{} x^3 \\minus{} 5x^2 \\plus{} 4x$ has only two real roots, one is in $ ( \\minus{} 4, \\minus{} 3)$ and the other is $ 0$, so both are outside of $ [1; \\plus{} \\infty)$." } { "Tag": [ "function", "algebra", "polynomial", "calculus", "derivative", "induction", "functional equation" ], "Problem": "Find all real continuous functions $f$ which verify $ f(x+2y)+f(x-2y)+6f(x)=4(f(x+y)+f(x-y))$ for all reals $x,y$.", "Solution_1": "Polynomials of degree no more than three. That's a fourth difference, so functions with the fourth derivative identically zero satisfy it. Conversely, we can use the values at $a,a+b,a+2b,a+3b$ to find the values at $a+4b$ and $a-b$, and build the whole arithmetic sequence. Since the interpolating polynomial of our original data satisfies the functional equation, the values on this infinite arithmetic sequence are a cubic polynomial. Now, you can do the same thing with the values at $a,a+\\frac bn,a+2\\frac bn,a+3\\frac bn$. Since the arithmetic sequence this generates shares infinitely many members with the previous, the function must be the same cubic polynomial on this set. By induction, $f(x)=p(x)$ for all rational $x$, where $p(x)$ is the interpolating polynomial that matches it at $0,1,2,3$. By continuity, $f(x)=p(x)$ everywhere." } { "Tag": [ "geometry", "function", "trigonometry", "circumcircle", "perpendicular bisector" ], "Problem": "Let a convex qurdilateral $ABCD$ such that $AD=BC$ and $AD,BC$ non parallel.\r\nand $O$ the point of intersection of $AC,BD$. Prove that there exist point $P$ different from $O$ such $\\frac{[PBD]}{[PAC]}=(\\frac{PB}{PA})^2$ where $[...]$ we symbolize the area", "Solution_1": "Let $M$ be the midpoint of $AB$. Consider a point $P$ on $\\ell$, the perpendicular bisector of $AB$, whose location can be determined uniquely by the (directed) distance $t = MP$.\r\n\r\n\r\nConsider the function $f(t) = [PBD] - [PAC]$. It is obvious that $\\frac{\\delta[PBD]}{\\delta t}$ (and $\\frac{\\delta[PAC]}{\\delta t}$) is $\\frac{BD\\sin \\beta}{2}$ (and $\\frac{AC\\sin \\alpha}{2}$), where $\\beta$ , ($\\alpha$) is the acute angle between $BD, \\ell$, ($AC, \\ell$). As long as $\\frac{BD}{AC} \\ne \\frac{\\sin \\alpha}{\\sin \\beta}$, $\\frac{\\delta f}{\\delta t}$ is some nonzero constant, then by geometric continuity, some $t$ exists for which $f(t) = 0$, which has a bijection to a unique point $P$.\r\n\r\nOtherwise, $\\frac{BD}{AC} = \\frac{\\sin \\alpha}{\\sin \\beta}$ (*), and sine law in $ABD, ABC$ imply $\\frac{AD}{\\cos \\beta} = \\frac{AB}{\\sin ADC}$, $\\frac{BC}{\\cos \\alpha} = \\frac{AB}{\\sin ACB}$, implying $\\frac{\\sin ACB}{\\sin ADB} = \\frac{\\cos \\alpha}{\\cos \\beta}$.\r\n\r\nNow observe that:\r\n\r\n$\\frac{\\cos \\alpha}{\\cos \\beta} = \\frac{\\sin ACB}{\\sin ADB} = \\frac{(1/2)AC*BC*\\sin ACB}{(1/2)BD*AD*\\sin ADB}$\r\n\r\n$= \\frac{[ABD]}{[ABC]} = \\frac{(1/2)AB*AC*\\cos \\alpha}{(1/2)AB*BD*\\cos \\beta} \\Rightarrow \\frac{AC}{BD} = 1$\r\n\r\nimplying immediately $\\sin \\alpha = \\sin \\beta$ and by $\\alpha, \\beta \\in [0, \\frac{\\pi}{2}]$, $\\alpha = \\beta \\Rightarrow O \\in \\ell$.\r\n\r\nFinally, $O \\in \\ell$ implies $f(0) = 0$, and together with $\\frac{\\delta f}{\\delta t} = 0$, implies $f(t) = 0$. So any point $P \\ne O, P \\in \\ell$ will suffice in this case.", "Solution_2": "The official solution was rather simpler .\r\n\r\nThe circumcircles of the triangles $OAD$ and $OBC$ have two points of intersection \r\nbecause $AD$ not parallel to $BC$. Notice that the circumcircles of the triangles $OAD$ and $OBC$ are equal because $AD=BC$. So the triangles $PBD$ and \r\n$PAC$ are similar so we are done", "Solution_3": "I don't understand this solution. Where is P? Also, I think my solution is easy too.", "Solution_4": "[quote=\"silouan\"]\nThe circumcircles of the triangles $OAD$ and $OBC$ have two points of intersection \nbecause $AD$ not parallel to $BC$[/quote]\r\n\r\nThe point $P$ is the second point of intersection of the circumcircles of the triangles $OAD$ and $OBC$ ;)" } { "Tag": [ "algebra", "polynomial", "superior algebra", "superior algebra unsolved" ], "Problem": "Prove that if a field $F$ is formally real (meaning $-1$ cannot be expressed as a sum of squares in $F$), $P$ is an irreducible polynomial of $F[X]$ with odd degree, and $\\alpha$ one of its roots, then $F[\\alpha]$ is also formally real.", "Solution_1": ":blush: deleted since it was rubbish :oops:", "Solution_2": "In your attempted proof, $R=-1$, which does not allow you to conclude anything (actually, you did not use any hypothesis, but it took me several readings before I could find the flaw ...).", "Solution_3": "so actually i reached at the end where i started :blush: :rotfl: too pathetic,i wonder why did all these to prove $h^2 +f^2$ is not divisible by $P$ while it was very evident from the assumption :oops: .", "Solution_4": "Actually I saw in the math forum website a proof for the fact that if a homogenouos polynomial of degree 2 has a non trivial solution in an odd degree extension of a field $K$ then it has one in $K$,\r\nso i think the proof for our question follows if you apply this to $X^2 + Y^2 + Z^2$,but anyway i could'nt do it :(,If you have a solution different from this,plz do post it.", "Solution_5": "It follows pretty quickly from the Artin-Schreier Theorem, stating that a formally real field has an extension which is a real closed field:\r\n\r\nDenote the real closure of $F$ by $\\overline F$. $P$ has at least one root $\\beta$ in $\\overline F$. Now, $F(\\beta)$ is an ordered field, and the canonical isomorphism $F(\\beta)\\to F(\\alpha)$ transports this order to $F(\\alpha)$. This means that $F(\\alpha)$ can be organized as an ordered field, and that's that." } { "Tag": [ "inequalities", "trigonometry", "inequalities unsolved" ], "Problem": "Given $a,b,c$ are three side of a triangle such that $a\\ge b\\ge c$ prove that\r\n$a^{2}+c^{2}\\ge b^{2}+bc$", "Solution_1": "no, it's not always true....\r\nuse the cosine law, transform a bit and you'll get 2h(b)>=b, which doesn't have to hold...\r\n\r\nh is an altitude", "Solution_2": "just take the case $a=b>c$ :)" } { "Tag": [ "function", "trigonometry", "calculus", "derivative", "complex analysis" ], "Problem": "I'm really struggling in my complex variables class, and it seems as though even the simplest concepts are eluding me. I'm trying to understand how to find the order of a zero of an analytic function. I feel like I have the basic idea, but when it comes to doing practice problems, I just feel lost. Could somebody walk me through the following excercise:\r\n\r\nGive the order of each of the zeros of the given function: sin(z)/z.\r\n\r\nI know that the answer is \"zeros of order one at n(pi), n = ...-2, -1, 1, 2,...\", but I need help with the actual steps needed to get there.", "Solution_1": "If $f(a)=0$ but $f'(a)\\ne0$, then $f$ has a zero of order $1$ at $a$. This is enough to answer the question about $\\frac{\\sin z}{z}$. More generally, the order of $a$ is $k$ if the derivatives of orders less than $k$ vanish at $a$, but $f^{(k)}$ does not.", "Solution_2": "I would use Laurent series expansion, which should give you the answer you want instantly." } { "Tag": [ "search", "quadratics", "geometry", "topology", "complex numbers" ], "Problem": "I have a sneaking suspicion that this has been discussed here before, but I did a search for it and it didn't show up, so I'll assume it's not. I had an interesting discussion with a friend of mine a while ago about what mathematics really [i]is[/i], and I was wondering what you all thought about it. How would you define mathematics? Or, if you were describing the body of stuff that makes up math, what would be the major features you would point out?\r\n\r\nI think this is related to the discovered vs. invented question, but I find it to be a more compelling question. So what do you all think?", "Solution_1": "There is an interesting [url=http://www.amazon.com/exec/obidos/tg/detail/-/0195105192/qid=1079401532/sr=8-1/ref=sr_8_xs_ap_i1_xgl14/002-3538802-1523255?v=glance&s=books&n=507846]book[/url] about this.", "Solution_2": "I'm not sure it really answers the question I am asking. Anyhow, \"there is a book about this\" does not constitute an opinion :-p", "Solution_3": "[quote=\"My Dictionary\"]\nA group of related sciences, including algebra, geometry, and calculus, concerned with the study of number, quantity, shape and space and their interrelationships by using a specialized notation.\n[/quote]\r\n\r\nBut yeah.", "Solution_4": "I hate these kind of philosophical questions, so I'll just state my opinion, and then try to keep my mouth shut. To me, mathematics is deductive reasoning. In other words, the feature of mathematics that distinguishes it from all other subjects is the notion of proof.", "Solution_5": "What does your dictionary give for the definition of \"science?\" And, doesn't it bother you that it essentially boils down to \"mathematics is the use of funny letters normal people don't understand\"?", "Solution_6": "Mathematics is what mankind created as a necessity which also turns out to be a recreational activity. Therefore mathematics is either necessity to mankind, is recreational, or a series of logical conclusions deduced from facts which have surfaced because of the above two. At any rate the third kind can be a waste of time but at least the first two serves a purpose.\r\n\r\nIn order to determine if it is mathematics or not, just simply keeps playing around with it by either going backwards to theorems/statements/etc. it had originally developed for or view all the aspects of that fact until you figure out if it is either useful or fun. Course the second is opinionated and therefore the definition of mathematics is opinionated, therefore there is no strict definition of mathematics. Q.E.D.", "Solution_7": "Well, Joel understandably didn't like my last answer, so I thought about it a bit more. Suppose we start with the integers and keep asking more difficult questions about what we have. If we try to divide 7 by 3 when we only have integers, we can't do it. So we create the rationals to help us solve that problem. What if we have a square and we want to draw the diagonal? Then we can't use the rationals, and we have to create at least quadratic surds. In general, it's not too difficult to ask questions that can generate all surds. What if we want to find the area of a circle? Then we need pi. (Or we could put a topology on it and generate all reals by using completeness, but that's getting ahead of where we need to be.) Anyway, suppose we have created all reals. We can generate complex numbers by asking how to solve equations such as x2+1=0. I think in general, all of mathematics can be generated by asking questions that can't be answered with what we already have. The best response that I can come up with therefore is that the mathematics is the questions we have to ask to come up with new abstract structures.", "Solution_8": "Ummm...math to me is just a hobby I do in my spare time but I'd think it's the study of relationships.", "Solution_9": "The definition of science is: systenatized knowledge derived from observation, study, and experimentation carried on in order ti determin the nature or principles of what is being studied. That is one definition, and it seems to me that mathmatics fits in there quite nicely. So to me mathmatics is a science because it is a study that we experiment with to find answers to different questions.", "Solution_10": "What I think for mathematics is not dictionary philosophies..\r\nThey are telling just what they are:\r\nWhat I'm think is that mathematic is society of human brain survivor..\r\nMost people think really hard on Math than any other subjects they do in their life... Math isn't just plain fun.. It could be but for more advanced people math is the language to them... Even in science, math is used... If you see in history, there are many parts that have mathematical signs such as in many towers.. If you see the shape of arc, you could see that it's shape that has cut which is the shape of circle...\r\n\r\nSo, math is the life...Not just philosophical definitions.. That's just what I think, though... Do anybody disagree on it?", "Solution_11": "Mathematics is sole power in the universe. Nothing exists without having something having to do with math in it.", "Solution_12": "[quote=\"Ragingg\"]Mathematics is sole power in the universe. Nothing exists without having something having to do with math in it.[/quote]\r\n\r\nHmm ... I think I disagree with this. Mathematics is a very effective tool for modelling many aspects of the physcial universe - but the physcial universe does not depend on mathematics. It would carry on quite happily if there was no-one around to try to model, understand or conceptualise it. \r\n\r\nIn other words ... if the universe contained no sentient beings, it would contain no mathematics. It's like ... bananas would still exist even if we had no word for \"banana\".", "Solution_13": "These are all good responses so far. Ragging brought up a particularly interesting point. But I think the best answer given thus far is Ravi B's. Mathematics is just a subfield of logic. In particular, it's a subfield of logic built on certain axioms and definitions that attempt to model quantitative properties of the physical Universe.\r\n\r\nIn other words, it's just a deductive science built from axioms culled from our intuitive understanding of quantitative and geometric truths. This view of mathematics really began with the Greeks some 2000 years ago. They recognized that, because mathematics is deductive in nature (via the rules of logic), it must have some axiomatic basis.\r\n\r\nThis idea was really brought to a head in the late 1800s and early 1900s when contradictions were found in some of the axioms that mathematicians had been using. And they began to find that the axioms could be changed to give different kinds of mathematics that had aspects modeling the real world!!\r\n\r\nSo to put it shortly, modern mathematics is the study of the logical deductive consequences of a set of axioms. Once those axioms are chosen - you have a type of mathematics that follows from that. Whether the Universe has a natural basic set of axioms that can be expressed in a finite language? That is the grand question! No one knows!\r\n\r\nGodol's Incompleteness theorem (one of the greatest theorems in the history of mathematics) puts this question in a particularly interesting light. I'm not an expert on this, but I believe GIT implies that if the Universe does have some cononical axiomatic basis then it cannot be finite. Meaning that mathematics can, at best, only ever be an approximation to modeling the features of the Universe.", "Solution_14": "Mathematics is a science that is incapable of completely describing itself and is an extension of logic such that, given a finite set of axioms,\r\n\r\n1) It is impossible to prove the truth of all of the axioms from the given axioms in a completely rigorous manner, and\r\n\r\n2) All questions that can be proven either true or false from the given axioms have a definite truth value.", "Solution_15": "I think that math is something that was both invented and discovered. Some things seem like they were found because they were there but just \"lost\" at the time. I also think that some things could have been made to fit the needs of something (\"This looks like it works, but I think that it needs something to make my point more valid\"). I'm not really sure of how people first made up the rules for solving something, just that it could have been determined by someone thinking that the way they thought of first was the way that it would always be done. Sometimes I wonder if they could be done differently somehow, but the answers wouldn't match the \"right way\" we use, but the other method would have its own unique set of solutions. I just don't know...", "Solution_16": "Math is the systematic study of categories. No, functors. Wait, natural transformations. Argh.\r\n\r\nActually, ignoring metaphysical concerns of ontology and epistemology, math's the rigorous study of abstract structures and the relationships between them. I'm pretty sure that this makes a nice if-and-only-if, because it definitely covers all of math and whenever other fields of human endeavor start messing with abstract structures, they definitely become mathematical. I'm glad to have had this opportunity to clear things up for everyone.\r\n\r\nAnd nothing is invented, if anything at all is discovered. How could you possibly draw a line between them that didn't pass through arbitrary social standards (not that there's anything wrong with arbitrary social standards, of course)?", "Solution_17": "[quote=\"kine\"]And nothing is invented, if anything at all is discovered.[/quote]\r\n\r\nI don't think this is at all the case. Unless we're going to talk about a Platonic world where there is an ideal copy of everything that ever could be floating around in some ideal place, there is a clear distinction between physical objects (the gasoline-powered internal-combustion engine, for example) which can definitely be invented, and ideas, which may or may not be able to be invented. We can talk about the Platonic ideal for ideas without having one for objects.", "Solution_18": "[quote=\"JBL\"][quote=\"kine\"]And nothing is invented, if anything at all is discovered.[/quote]\n\nI don't think this is at all the case. Unless we're going to talk about a Platonic world where there is an ideal copy of everything that ever could be floating around in some ideal place, there is a clear distinction between physical objects (the gasoline-powered internal-combustion engine, for example) which can definitely be invented, and ideas, which may or may not be able to be invented. We can talk about the Platonic ideal for ideas without having one for objects.[/quote]\r\n\r\nOnly if one can invent/there exists a sufficiently strong definition of \"idea.\" :P", "Solution_19": "Mathematics is deductive reasoning at its best and purest.\r\n\r\nYou start with axioms that seemingly come out of nowhere. They are broad and general, but often we consider them to be correct. Then we find everything possible from these axioms to find out the implications of our axioms: to see if they completely describe what we are looking at (proving the fundamentals), and if so, to find out and test new ideas about our system. \r\n\r\nOf course, there are times when axioms go wrong (can anyone say unrestricted comprehension?), which is why we must be VERY careful.", "Solution_20": "the dictionary's definition: The study of the measurement, properties, and relationships of quantities, using numbers and symbols.\r\n\r\nHowever, I think that they forgot about logical reasoning. \r\n\r\nArithmetic, the very basics of mathematics, is really invented by human beings ( because we can never prove why 1+1=2)\r\n\r\nAlso, mathematics is really abstract that it talks about things that do not really exist in real world, like a perfect line, perfect plane...\r\n\r\nwe created rules of mathematics really, just like we created language.", "Solution_21": "2 is defined to be 1 + 1 .\r\n\r\n3 is defined to be 2 + 1 . . . etc.\r\n\r\n1 is defined to be the multiplicative identity\r\n\r\n0 is defined to be the additive identity.\r\n\r\nYou have to prove there are no integers between 0 and 1.", "Solution_22": "Like Thomas Edison asked, why 1+1=3 is not true?", "Solution_23": "math = numbers, shapes, and graphs", "Solution_24": "Mathematics is just the evolution of common sense (i.e. simple axioms changed to things that almost nobody in the world can understand.)", "Solution_25": "all i know is that math is essential for my survival. if i dont do good at math, my mom would kill me" } { "Tag": [ "combinatorics unsolved", "combinatorics" ], "Problem": "Hi, this is regarding Problem 2 on the Romanian 2005 TST on March 31st 2005 http://www.artofproblemsolving.com/Forum/viewtopic.php?highlight=Romania+TST+2005&t=31951\r\n\r\nLet $n\\geq 1$ be an integer and let $X$ be a set of $n^2+1$ positive integers such that in any subset of $X$ with n+1 elements there exist two elements $x\\neq y$ such that $x\\mid y$. Prove that there exists a subset $\\{x_1,x_2,\\ldots, x_{n+1} \\}$ $\\in X$ such that $x_i \\mid x_{i+1}$for all $i=1,2,\\ldots, n$.\r\n\r\nWhile there is a solution given to this problem, it requires the use of chains and dilworth particularization. For somebody say who doesnt know dilworth particularization, and is familiar with basic combinatorial tools, is there an elementary solution for this?\r\n\r\nYou have my gratitude", "Solution_1": "Look here:\r\n\r\n[url=http://www.mathlinks.ro/Forum/viewtopic.php?t=31425]www.mathlinks.ro/Forum/viewtopic.php?t=31425[/url]" } { "Tag": [ "counting", "distinguishability" ], "Problem": "Can someone explain why the number of ways to place n indistinguishable objects into m categories is (n+m-1) choose n?", "Solution_1": "Think of the equation $x_{1}+...+x_{m}=n,\\ x_{i}\\in\\mathbb{N}_{0},\\ n\\in\\mathbb{N}$. :wink: \r\n\r\nMasoud Zargar" } { "Tag": [ "geometry", "3D geometry", "sphere" ], "Problem": "There's just so much to say about the 12 (soon to be 13 :wink: ) game series. So who here plays? Which was your favorite game? Favorite character? Anything at all you want to say?\r\n\r\nPersonally, you can't beat X. The graphics and battle system were the best of any of the games I thought. Sephiroth was the greatest character, too, he was so realistic and in depth.", "Solution_1": "no, 6 is the best. plus, one of the characters has my name! :o", "Solution_2": "yes, the graphics are getting amazing nowadays. althoguh i miss the traditional final fantasy gameplay. The music was the best when Nobuo Oematsu was around. The famous victory theme, and all that.\r\n\r\nBut simply because it's still final fantasy, it's still awesome", "Solution_3": "[quote=\"EggyLv.999\"]no, 6 is the best. plus, one of the characters has my name! :o[/quote]\r\n\r\nOne of the characters' names is EggyLv.999? :P \r\n\r\n@seph: Yea, Nobuo really contributed to the enjoyment of the games. I can recall off the top of my head any song from VII or X, because he made them so memorable. Then when I played XII it was like: dude, what's with this music? That was the one reason that I disliked XII compared to the rest. Nice avatar btw, that stupid little cactar made me use up like 300 phoenix downs!\r\n\r\nAlso, anyone played X-2? I'm in the middle of it right now and what did they do to all the characters?????? They're all so different...not in a good way either (except the attractiveness of course :wink: )", "Solution_4": "my REAL name...", "Solution_5": "I just started playing FFX two days ago, and it's the first Final Fantasy game I've played, so I can't compare it to other games in the series.\r\nHowever, the music is awesome, graphics are awesome (especially considering it's a PS2), storyline so far is awesome, cutscenes are awesome, and it definitely isn't like any cliche RPG, especially with the sphere grid being used.\r\nThe main complaints I have are that you can't adjust the camera, which puts you in hard to see spots sometimes, and the game so far has been extremely linear." } { "Tag": [ "algebra unsolved", "algebra" ], "Problem": "Find all real $\\alpha$ s.t.\r\n\\[ [ \\sqrt{n + \\alpha} + \\sqrt{n} ] = [ \\sqrt{4n+1} ] \\] holds for all natural numbers $n$", "Solution_1": "1,2,3,4 ???", "Solution_2": "Is $ \\alpha\\in [1,2]$ :?:", "Solution_3": "http://www.mathlinks.ro/viewtopic.php?t=150706", "Solution_4": "Anyone with a solution for this?", "Solution_5": "I have proved that $\\alpha \\in \\left(\\frac{1}{1+2\\sqrt { \\frac{2}{9} } },2 \\right)$\nAny $\\alpha$ in this interval will satisfy the above relation.\nDoes anybody know the answer, is this correct. ?" } { "Tag": [], "Problem": "The King and the Queen invited n other married couples to the palace for a round-table dinner. In order that each person (including the King and the Queen) had a chance to sit beside every person except his/her spouse during the dinner, the King and the Queen decided to have a different sitting arrangement for each course of food served. Prove that they can achieve their purpose if n courses of food are served, but not fewer. :D", "Solution_1": "huh? i dont understand..\r\n\r\n'had a chance to sit beside every person except his/her spouse during the dinner,'\r\n\r\nif there are n other married couples, then any person needs to sit beside 2n people, which cannot be done in fewer than n courses, else a person sits adjacent to fewer than 2n people.", "Solution_2": "maybe he means next to each of the n other people of the opposite gender?", "Solution_3": "I seriously don't know what that statement means either..... :blush: I was wondering if any of you could figure it out...", "Solution_4": "No, I think that's right, but Singular hasn't shown that n courses is sufficient. That is, it's easy to see why n is the least, but you still must show that it is possible to do it in n courses." } { "Tag": [ "logarithms", "quadratics", "function", "LaTeX", "algebra", "quadratic formula" ], "Problem": "In these exercises solve the logarithmic equation algebraically. Roung the result to three decimal places\r\n\r\n96. ln(x + 1) - ln(x - 2) = ln x^2\r\n\r\n98. log 4x - log (12 + radical x) = 2\r\nthe first part of the equations w/ the log have a base of 10\r\n\r\nENJOY", "Solution_1": "[hide]96. Using log properties,\n\n$\\ln (x+1)-\\ln (x-2) = \\ln x^{2}$\n\n$\\ln \\frac{x+1}{x-2}= \\ln x^{2}$\n\n$\\frac{x+1}{x-2}= x^{2}$\n\n$x^{3}-2x^{2}-x-1 = 0$\n\nMy calculator tells me that this cubic has only one real solution, $x \\approx 2.547$. I'm not sure that this can be solved without a calculator.\n\n98. Similarly, the equation becomes:\n\n$\\log \\frac{4x}{12+\\sqrt x}= 2$\n\n$\\frac{4x}{12+\\sqrt x}= 100$\n\n$4x = 1200+100\\sqrt x$\n\n$(x-300)^{2}= 625x$\n\n$x^{2}-925x-90000 = 0$\n\nQuadratic formula, etc. yields $x \\approx-88.777$ or $1013.777$, but the first (as can easily be shown) is extraneous. Thus, $x \\approx 1013.777$.[/hide]", "Solution_2": "[quote=\"I Am Me\"][hide]96. Using log properties,\n\n$\\ln (x+1)-\\ln (x-2) = \\ln x^{2}$\n\n$\\ln \\frac{x+1}{x-2}= \\ln x^{2}$\n\n$\\frac{x+1}{x-2}= x^{2}$\n\n$x^{3}-2x^{2}-x-1 = 0$\n\nMy calculator tells me that this cubic has only one real solution, $x \\approx 2.547$. I'm not sure that this can be solved without a calculator.\n\n98. Similarly, the equation becomes:\n\n$\\log \\frac{4x}{12+\\sqrt x}= 2$\n\n$\\frac{4x}{12+\\sqrt x}= 100$\n\n$4x = 1200+100\\sqrt x$\n\n$(x-300)^{2}= 625x$\n\n$x^{2}-925x-90000 = 0$\n\nQuadratic formula, etc. yields $x \\approx-88.777$ or $1013.777$, but the first (as can easily be shown) is extraneous. Thus, $x \\approx 1013.777$.[/hide][/quote]\r\nHow do you go from $4x = 1200+100\\sqrt x$ to $(x-300)^{2}= 625x$?", "Solution_3": "$4x=1200+100\\sqrt(x)$\r\nSubtract 1200 from each side\r\n$4x-1200=100\\sqrt(x)$\r\nDivide each side by 4\r\n$x-300=25\\sqrt(x)$\r\nSquare both sides\r\n$(x-300)^{2}=625x$", "Solution_4": "thankx guys\r\nit really helped a lot\r\n\r\ni was completely stuck at the cuvic function\r\n\r\nand by the way\r\nhow do u guys solve the equation\r\nwith those equation\r\nit lookz really ncie\r\nand lot eazier to read if i am dividing somehting\r\n\r\nplz\r\ntell", "Solution_5": "What? Do you mean $\\LaTeX$?", "Solution_6": "$\\LaTeX$ is the code you write your math equations\r\n\r\nso instead of x+y=5 you can say\r\n\r\n$x+y=5$ which is much better to look at :)", "Solution_7": "[quote=\"Micalix\"]$4x=1200+100\\sqrt(x)$\nSubtract 1200 from each side\n$4x-1200=100\\sqrt(x)$\nDivide each side by 4\n$x-300=25\\sqrt(x)$\nSquare both sides\n$(x-300)^{2}=625x$[/quote]\r\nor you could just solve for $\\sqrt{x}$ and square that", "Solution_8": "[quote=\"kryms3n\"]and by the way\nhow do u guys solve the equation\nwith those equation\nit lookz really ncie\nand lot eazier to read if i am dividing somehting\n\nplz\ntell[/quote]\r\n\r\n\r\n[url]http://www.artofproblemsolving.com/LaTeX/AoPS_L_About.php[/url]\r\n\r\nHere is an example:\r\n\r\n$\\sum_{i=1}^{n}i^{n}=1^{n}+2^{n}+3^{n}+\\cdots+(n-1)^{n}+n^{n}$" } { "Tag": [ "geometry", "perpendicular bisector", "geometry unsolved" ], "Problem": "Two circles meet at $ A$ and $ B$.The tangent at a point $ P$ of one of the circles meets the other circle at $ Q$ and \r\n\r\n$ R$.The points $ X,Y,Z$ are the feet of the perpendiculars from $ P$ to $ AB , AQ,BR$ respectively.\r\n\r\nProve that the circle $ XYZ$ passes through the pidpoint $ M$ of $ AB$.\r\n\r\nBabis", "Solution_1": "how do you proove it?", "Solution_2": "Let $ C \\equiv AQ \\cap BR$ and $ M$ be the midpoint of $ AB.$ By simple angle chasing we have \n\n$ \\angle ABP \\equal{} \\angle APQ \\ , \\ \\angle CBA \\equal{} \\angle AQP $ \n\n$\\Longrightarrow \\angle PBR \\equal{} 180^{\\circ} \\minus{} \\angle CBA \\minus{} \\angle ABP \\equal{} \\angle QAP.$\n\nThis implies that $ \\angle APY \\equal{} \\angle BPZ,$ but the quadrilaterals $ AXPY$ and $ BXPZ$ are cyclic $\\Longrightarrow$ $ \\angle AXY \\equal{} \\angle BPZ$ $\\Longrightarrow$ lines $ XP$ and $ AB$ bisect $ \\angle YXZ$ internally and externally $ ( \\star).$ Let $ U$ and $ V$ be the midpoints of $ PA$ and $ PB.$ Then $ \\triangle MUV$ is the medial triangle of $ \\triangle PAB$ $ \\Longrightarrow$ $ MV \\equal{} UA \\equal{} UY,$ $ MU \\equal{} VB \\equal{} VZ$ and $ \\angle MVZ \\equal{} \\angle MUY \\equal{} \\angle APB \\plus{} 2\\angle BPZ.$ Thus, $ \\triangle MUY$ and $ \\triangle ZVM$ are congruent by SAS. Which yields $ MY \\equal{} MZ,$ i.e. $ M$ lies on the perpendicular bisector of $ YZ.$ Together with $ (\\star),$ we conclude that $ M \\in \\odot(XYZ).$" } { "Tag": [ "inequalities proposed", "inequalities" ], "Problem": "Let a, b, c, d be non-negative real numbers such that a^2+b^2+c^2+d^2 = 1.\r\n\r\nProve that\r\n\r\n(1 \u2212 a)(1 \u2212 b)(1 \u2212 c)(1 \u2212 d) \u2265 abcd.", "Solution_1": "[quote=\"ABRORBEK\"]Let a, b, c, d be non-negative real numbers such that a^2+b^2+c^2+d^2 = 1.\n\nProve that\n\n(1 \u2212 a)(1 \u2212 b)(1 \u2212 c)(1 \u2212 d) \u2265 abcd.[/quote]\r\nSee here:\r\nhttp://www.artofproblemsolving.com/Forum/viewtopic.php?t=111267" } { "Tag": [], "Problem": "1) Assume that A=B\r\n2) Then, AB=A :^2: \r\n3) Therefore, AB-B :^2: =A :^2:-B :^2: \r\n4) [i]Next I will show you that[/i] A :^2: -B :^2: =(A+B)(A-B)\r\n5) (A+B)(A-B)=(A+B)A-(A+B)B\r\n6) =A :^2: +AB-AB-B:^2: \r\n7) =A :^2: -B :^2: \r\n8) Therefore, AB-B :^2: =(A+B)(A-B)\r\n9) Also, AB-B :^2: =BA-B :^2: \r\n10) =B(A-B)\r\n11) Thus, B(A-B)=(A+B)(A-B)\r\n12) B=A+B\r\n13) 2B=A\r\n14) Since, A=B\r\n15) 2B=B\r\n16) 2=1\r\n\r\nWhat is wrong with that proof?", "Solution_1": "You like these kind of problems, don't you :) \r\n\r\nThis one's famous, so probably everyone knows the answer, but if anyone needs a hint, just ask.", "Solution_2": "[quote=\"MathFiend\"]11) Thus, B(A-B)=(A+B)(A-B)\n12) B=A+B\n13) 2B=A\n14) Since, A=B\n15) 2B=B\n16) 2=1\n\nWhat is wrong with that proof?[/quote]\r\n\r\n\r\nthe top part. you divided both sides by (A-B) to get B=A+B, but if A=B, A-B=0. and you cant divide by 0. Q.E.D.", "Solution_3": "Ummm... Neal... since A=B then A-B does equal 0... so basically you're right.", "Solution_4": "Thank you.", "Solution_5": "[quote=\"Chinaboy\"]Ummm... Neal... since A=B then A-B does equal 0... so basically you're right.[/quote]\r\n\r\n\r\nyou made it seem like i was wrong. where did the basically part come in?", "Solution_6": "sigh... u were right overall.. but you just missed a part.", "Solution_7": "[quote=\"Syntax Error\"]\n\n\nthe top part. you divided both sides by (A-B) to get B=A+B, but if A=B, A-B=0. and you cant divide by 0. Q.E.D.[/quote]\r\n\r\n\r\npfft, i included it right there.", "Solution_8": "I think the only thing missing there is an apostrophe :)", "Solution_9": "lol, and other stuff like capitalizations. but im too lazy to think more than i wamt", "Solution_10": "ah... oops. my apologies to you.", "Solution_11": "lol, its aight mr. man", "Solution_12": "if you think this proof is cool, i can make a \"pseudoproof\" claiming all triangles are isosceles. (reply if u want to see it). For this triangle proof, the error is even harder to find then in the previous.", "Solution_13": "I know that one too. Can I post it?", "Solution_14": "[color=cyan]There's also a nice proof that all obtuse angles are right, but it relies very much on a diagram that I can't create here. I bet you could find it on the web, though.[/color]", "Solution_15": "You could update it as your avatar, if it's simple enough.", "Solution_16": "I think I posted the isosceles triangle one ages ago, under \"paradoxes\" not sure which forum its in now.. I'm assuming thats the same one. There were heaps of things like this posted under there..", "Solution_17": "OK, now let's try to prove some other things like this:\r\nProve that you can not make two numbers, a and b, from a pool of 100 digits (digits means all integers between 0 and 9, inclusive) such that a:^2:=b", "Solution_18": "I did a little cleaning in this thread . . .", "Solution_19": "Thank you. I was afraid to do it myself since people dislike it when I delete their post, just like I get angry when other people delete my posts.\r\n\r\nI don't think it was that serious, though...I mean, it was \"on topic\" and had to do with math...oh well, you're the boss, we're your white-collars :)", "Solution_20": "4 years later...", "Solution_21": "don't revive old threads, especially ones that are 4 years old... also, you're not really saying anything important", "Solution_22": "yes i know, but that is not my purpose", "Solution_23": "geez i posted that 2=1 and everyones making a big deal about it.\r\n\r\n[hide]the problem with this is that since a=b a-b=0 and you cant divide by 0[/hide]" } { "Tag": [ "linear algebra", "matrix", "limit", "linear algebra unsolved" ], "Problem": "Let $ M_1,\\ldots ,M_k$ be $ n\\times n$ positive Hermitian matrices whose largest eigenvalue is 1.\r\nShow that $ \\lim_{m\\to \\infty} (M_1\\cdots M_k)^m$ exists and equals 0 $ \\iff \\ker(M_1 \\cdots M_k \\minus{} I_n) \\equal{} \\{0\\}$.", "Solution_1": "Let$ \\parallel{}.\\parallel{}$ be the $ 2$-norm.\r\ni) for all $ i,x$ $ \\parallel{}M_ix\\parallel{}\\leq\\parallel{}x\\parallel{}$ and $ \\parallel{}M_ix\\parallel{}=\\parallel{}x\\parallel{}$ implies $ M_ix=x$.\r\nProof: $ \\parallel{}M_ix\\parallel{}^2=x^T{M_i}^2x\\leq{x^Tx}$ with equality only if $ x$ is in the eigenspace associated to the eigenvalue $ 1$ of $ M_i^2$.\r\nii) ${ \\parallel{}M_1\\cdots{M_k}x\\parallel{}=\\parallel{}x\\parallel{}}\\Rightarrow{x\\in\\bigcap_iker(M_i-I)}$.\r\nProof: $ \\parallel{}M_1\\cdots{M_k}x\\parallel{}\\leq{\\parallel{}M_2\\cdots{M_k}x\\parallel{}}\\leq{\\parallel{}M_{k-1}M_kx\\parallel{}}\\leq{\\parallel{}M_kx\\parallel{}}\\leq{\\parallel{}x\\parallel{}}$ with equality if $ \\parallel{}M_kx\\parallel{}=\\parallel{}x\\parallel{}$ that is $ M_kx=x$ and if $ \\parallel{}M_{k-1}x\\parallel{}=\\parallel{}x\\parallel{}$ that is $ M_{k-1}x=x$ and if $ \\cdots$.\r\niii) $ ker(M_1\\cdots{M_k}-I)=\\bigcap_iker(M_i-I)$.\r\nProof :$ x\\in{ker}(M_1\\cdots{M_k}-I)\\Leftrightarrow{M_1\\cdots{M_k}x=x}\\Rightarrow{\\parallel{}M_1\\cdots{M_k}x\\parallel{}=\\parallel{}x\\parallel{}}$\r\n$ \\Rightarrow{x\\in\\bigcap_iker(M_i-I)}\\Rightarrow{M_1\\cdots{M_k}x=x}$.\r\niv) $ (M_1\\cdots{M_k})^m$ tends to $ 0$ iff $ ker(M_1\\cdots{M_k}-I)=\\{0\\}$.\r\nProof: $ (M_1\\cdots{M_k})^m$ tends to $ 0$ iff $ \\{$for all eigenvalues $ \\lambda$ of $ M_1\\cdots{M_k}$ : $ |\\lambda|<1\\}\\Rightarrow{ker(M_1\\cdots{M_k}-I)=\\{0\\}}$.\r\nConversely let $ \\lambda$ be an eigenvalue of $ M_1\\cdots{M_k}$ s.t. $ |\\lambda|\\geq{1}$.\r\nBy the proofs of ii),iii) $ |\\lambda|=1$, $ \\lambda=1$ and $ ker(M_1\\cdots{M_k}-I)\\not=\\{0\\}$." } { "Tag": [ "modular arithmetic" ], "Problem": "Given that we have a number with a decimal representation with only 1s (ex: 1 is a string of one 1, 111 is a string of three ones, 111111 is a string of 6 ones, etc.) then \r\n\r\na) prove that a string of 2001 1s will be divisible by 2003\r\nb) prove that there is a smaller string of 1s that will be divisible by 2003", "Solution_1": "[hide=\"a\"]the string of 1's is $\\frac{10^{2002}-1}{9}$. Division by 9 won't affect divisibility, and since 2003 is prime, $10^{2002}\\equiv 1\\pmod{2003}$ by Fermat's Little Theorem. Then the string is divisible by 2003.[/hide]", "Solution_2": "[quote=\"Scrambled\"]Given that we have a number with a decimal representation with only 1s (ex: 1 is a string of one 1, 111 is a string of three ones, 111111 is a string of 6 ones, etc.) then \n\na) prove that a string of 2001 1s will be divisible by 2003\nb) prove that there is a smaller string of 1s that will be divisible by 2003[/quote]\r\n\r\n[hide=\"a\"]\n\n$2003$ is prime. so we can rewrite the string of 2001 1s as:\n$\\frac{10^{2002} - 1}{10-1}$ we don't care about the 9 on the bottom because it doesn't divide 2003 so if 2003 divides $10^{2002} -1$ then we're set, and that's trivially true by fermat's little theorem.\n[/hide]\n\n[hide=\"b\"]\n$10^d \\equiv 1 \\pmod{2003}$ for $d | 2002$ because you have $(10^d)^{\\text{oddpower}} \\equiv 1 \\pmod{2003}$\n[/hide][/hide]", "Solution_3": "[quote=\"agolsme\"][quote=\"Scrambled\"]Given that we have a number with a decimal representation with only 1s (ex: 1 is a string of one 1, 111 is a string of three ones, 111111 is a string of 6 ones, etc.) then \n\na) prove that a string of 2001 1s will be divisible by 2003\nb) prove that there is a smaller string of 1s that will be divisible by 2003[/quote]\n\n[hide=\"a\"]\n\n$2003$ is prime. so we can rewrite the string of 2001 1s as:\n$\\frac{10^{2002} - 1}{10-1}$ we don't care about the 9 on the bottom because it doesn't divide 2003 so if 2003 divides $10^{2002} -1$ then we're set, and that's trivially true by fermat's little theorem.\n[/hide]\n\n[hide=\"b\"]\n$10^d \\equiv 1 \\pmod{2003}$ for $d | 2002$ because you have $(10^d)^{\\text{oddpower}} \\equiv 1 \\pmod{2003}$\n[/hide][/hide][/quote]\r\n\r\nFor b, d would have to divide 2002. But that does not necessarily happen (what about d=1?)" } { "Tag": [ "linear algebra", "matrix", "calculus", "calculus computations" ], "Problem": "This is a worked example\r\n\r\nThe objective is to prove\r\n\r\n$ - 1 \\leq \\rho(X,Y) \\leq 1$\r\n\r\nThen the book uses this formula...\r\n\r\n(2) $ 0 \\leq Var( \\frac {X}{\\sigma_x} + \\frac {Y}{\\sigma_y} )$\r\n\r\n(3) $ = \\frac {Var(X)}{{\\sigma_x}^2} + \\frac {Var(Y)}{{\\sigma_y}^2} + \\frac {2Cov(X,Y)}{\\sigma_x \\sigma_y}$\r\n\r\nThe question is, how does 2 lead to 3? Namely, how does $ Var(\\frac {X}{\\sigma_x} ) = > \\frac {Var(X)}{{\\sigma_x}^2}$?\r\n\r\nAlso, how does one get the idea to use formula (2) to prove (1)? It doesn't seem like a natural step", "Solution_1": "Variance scales quadratically- just look at the definition, and think of units.\r\n\r\nThe key fact we know if we want to prove (1) is that the covariance matrix is positive definite; to use this, we either study the matrix directly and calculate its determinant, or look at the variance of various linear combinations.", "Solution_2": "oh true\r\n\r\nactually I got it, due to the simple Var(aX) = a^2 Var(X) formula. I encountered it before. my attention lapses are bad... =/" } { "Tag": [ "AMC" ], "Problem": "For any set $S$, let $|S|$ denote the sum of elements in $S$, and let $n(S)$ be the number of subsets of $S$, including the empty set and the set $S$ itself. If $A,B$ and $C$ are sets for which \r\n\r\n$n(A)+n(B)+n(C) = n(A \\cup B \\cup C) \\ \\text{and} \\ |A| = |B| = 100,$\r\n\r\nthen what is the minimum possible value of $|A \\cap B \\cap C|$?\r\n\r\n$\\text{(A)} \\ 96 \\qquad \\text{(B)} \\ 97 \\qquad \\text{(C)} \\ 98 \\qquad \\text{(D)} \\ 99 \\qquad \\text{(E)} \\ 100$", "Solution_1": "Obviously $n(S) = 2^{|S|}$. The equation boils down to $2^{101} + 2^{|C|} = 2^{|A\\cup B \\cup C|}$.\r\n\r\nFor the LS to be a power of two, it must be the case that $|C| = 101$; then $|A\\cup B \\cup C| = 102$.\r\n\r\nWe can have $|A\\cap B\\cap C| = 97$, $|A\\cap B|, |B\\cap C|, |C\\cap A| = 1,2,2$. If we had 96, we would need to use 6 more elements to achieve a gain of 6 in C and 4 in A and B. It's not possible.\r\n\r\nAnswer B.", "Solution_2": "Silverfalcon, I think you have a typo...$|S|$ should be the NUMBER of elements in $S$, NOT the SUM of the elements in $S$. Anyways, this is a fun problem in AoPS Volume II :lol: :lol:" } { "Tag": [ "linear algebra", "linear algebra unsolved" ], "Problem": "What is the maximal number of points in $R^n$ such that the distance between any two is an odd integer?", "Solution_1": "Not long ago i was solving this problem for $n=2$.In this case $n< 4$.", "Solution_2": "Yes, and even that one is hard in my humble opinion. But for general n, I have no idea. I thing I can play a little bit arround with the idea for $n=2$ in case $n$ has a special form modulo 8, but even there I cannot solve it completely." } { "Tag": [], "Problem": "Se dau numerele n, n+1, n+2 si n+3, n natural.\r\na) Sa se arate ca exista un singur trapez si numai unul avand ca lungimi ale laturilor sale aceste numere.\r\nb) Aflati n astfel incat trapezul sa fie dreptunghic.\r\nc) Aflati n astfel incat inaltimea trapeuzlui sa fie egala cu baza mica a trapezului.", "Solution_1": "Haide, haide, putina inviorare!Chiar nu vrea nimeni sa o faca?Sau e prea simpla?Sau prea grea?", "Solution_2": "La clasa a VIIa s-a dat, nu?", "Solution_3": "Da, s-a dat la a VII-a.Problema e cam grea totusi pentru clasa a VII-a. :ninja: Ai cumva o solutie?Btw, care e numele tau? :)", "Solution_4": "[hide=\"Hint\"]Se arata, prin reducere la absurd, ca bazele au lungimile $n$ si $n+3$.[/hide]", "Solution_5": "Da, domnule profesor.Asa e.Dar eu am pierdut cam o jumatate de ora sa gasesc solutia asta.M-am chinuit sa incerc toate variantele posibile,ca, poate gresesc altcumva.Si cred ca dupa asta problema devine cam simpla...asa cred.Problema asta mi-a dat-o doamna profesoara a mea pentru selectia la Concursul Pitagora.", "Solution_6": "Problema asta am rezovat-o pe vremuri :lol: cand eram a VIIa. \r\n\r\nar trebui sa o rezolve cei de a VIIa...dar nu se prea inghesuie nimeni...whatever...\r\n\r\no idee asemanatoare\r\n\r\n[hide]Daca ABCD este trapezul cerut, cu AB baza mare, se duce paralela CE la AD. Se aplica inegalitatea triunghiului in triunghiul BCE si se vede luand toate cazurile ca AB=n+3, CD=n. Punctele b) si c) sunt usoare.[/hide]" } { "Tag": [ "geometry", "3D geometry", "calculus", "derivative", "calculus computations" ], "Problem": "A right circular cone has base radius $ r$, height $ h$ and slant length $ l$. Its volume $ V$, and the area $ A$ of ts curved surface, are given by \r\n\r\n$ V\\equal{} \\frac{1}{3}\\pi r^{2}h, \\,A \\equal{} \\pi rl$\r\n\r\n(i) Given that $ A$ is fixed and $ r$ is chosen so that $ V$ is at its stationary value, show that $ A^{2} \\equal{} 3\\pi^{2}r^{4}$ and that $ l\\equal{}\\sqrt{3}r$.\r\n\r\n(ii) Given instead, that $ V$ is fixed and $ r$ is chosen so that $ A$ is at its stationary value, find $ h$ in terms of $ r$.", "Solution_1": "[hide=\"(i)\"]\n$ l \\equal{} \\sqrt{r^2 \\plus{} h^2}$, so $ A \\equal{} \\pi r \\sqrt{r^2 \\plus{} h^2}$ gives $ h \\equal{} \\frac1r\\sqrt{\\frac{A^2}{\\pi^2} \\minus{} r^4}$.\n\n$ \\frac{dV}{dr}\\equal{} \\frac{d}{dr}\\left[ \\frac13 \\pi r \\sqrt{\\frac{A^2}{\\pi^2} \\minus{} r^4}\\right] \\equal{} \\frac{\\pi}3 \\left[\\sqrt{\\frac{A^2}{\\pi^2} \\minus{} r^4} \\plus{} \\frac12 (\\minus{} 4r^3) r \\frac1{\\sqrt{\\frac{A^2}{\\pi^2}\\minus{} r^4}}\\right]$\n$ \\equal{} \\frac{\\pi}3 \\sqrt{\\frac{A^2}{\\pi^2} \\minus{} r^4}\\left[1 \\minus{} \\frac{2 r^4}{\\frac{A^2}{\\pi^2} \\minus{} r^4}\\right] \\equal{} 0$\n\nGives $ 0 \\equal{} 1 \\minus{} \\frac{2 r^4}{\\frac{A^2}{\\pi^2} \\minus{} r^4}$ so $ A^2 \\equal{} 3 \\pi^2 r^4$.\n\nAnd then also $ l \\equal{} \\sqrt{3} r$ as then $ A^2 \\equal{} \\pi^2 r^2 l^2 \\equal{} \\pi^2 r^2 3 r^2 \\equal{} 3 \\pi^2 r^4$.\n[/hide]\n\n[hide=\"(ii)\"]\n$ h \\equal{} \\frac{3 V}{\\pi r^2}$.\n\nSo $ A \\equal{} \\pi r \\sqrt{r^2 \\plus{} h^2} \\equal{} \\pi r \\sqrt{r^2 \\plus{} \\frac{9 V^2}{\\pi^2 r^4}} \\equal{} r^2 \\sqrt{\\pi^2 \\plus{} \\frac{9 V^2}{r^6}}$.\n\nso $ \\frac{dA}{dr} \\equal{} 2 r \\sqrt{\\pi^2 \\plus{} \\frac{9 V^2}{r^6}} \\minus{} \\frac1{2 r^5} \\frac{45 V^2}{ \\sqrt{\\pi^2 \\plus{} \\frac{9 V^2}{r^6}}}$\n$ \\equal{} \\frac{2}{r^2} \\sqrt{\\pi^2 r^6 \\plus{} 9 V^2} \\minus{} \\frac1{2r^2} \\frac{45 V^2}{ \\sqrt{\\pi^2 r^6 \\plus{} 9 V^2}} \\equal{} 0$\ngives\n$ \\pi^2 r^6 \\plus{} 9 V^2 \\equal{} \\frac{45}4 V^2$ (or $ r \\equal{} 0$).\n\nThis then implies $ r \\equal{} \\left(\\frac{3 V}{2 \\pi}\\right)^{\\frac13}$.\n\n$ h \\equal{} \\frac{3 V}{\\pi r^2} \\equal{} \\frac{3 V}{\\pi} \\left(\\frac{2 \\pi}{3 V}\\right)^{\\frac23} \\equal{} 3 \\left(\\frac{4 V}{\\pi}\\right)^{\\frac13}$.\n[/hide]", "Solution_2": "Thanks Tyl.", "Solution_3": "Actually this is wrong. I apparently thought the derivative of $ \\frac1{r^6}$ was $ \\minus{} 5 \\frac1{r^7}$ at some point.\r\n\r\nWe instead get $ \\pi^2 r^6 \\plus{} 9 V^2 \\equal{} \\frac {9 \\cdot 6}4 V^2$ which gives $ r \\equal{} \\left(\\frac {9 V^2}{2 \\pi^2}\\right)^{\\frac13}$ and then $ h \\equal{} \\left(\\frac {3 V}{2 \\pi}\\right)^{\\frac13}$." } { "Tag": [], "Problem": "For $\\{1,2,3,...,n\\}$ and each of its nonempty subsets, a unique [b]alternating sum[/b] is defnined as follows: Arrange the numbers in the subset in decreasing order and then, beginning with the largest, alternately subtract and add successive numbers. (For example, the alternating sum for {1,2,4,6,9} is 9-6+4-2+1=6 and for {5} it is simply 5.) For each $n$, find a formula for the sum of all the alternating sums of all the subsets.", "Solution_1": "[hide=\"hint\"]consider pairing a set $S$ that does not have n, with the set that has all the elements of $S$ and 7[/hide]", "Solution_2": "[hide]Recursion would be another way.[/hide]" } { "Tag": [], "Problem": "\u0393\u03b9\u03b1 \u03ba\u03b1\u03b8\u03b5 n \u03c6\u03c5\u03c3\u03b9\u03ba\u03bf \u03bd\u03b1 \u03b2\u03c1\u03b5\u03b8\u03b5\u03b9 \u03bf \u03bc\u03b5\u03b3\u03b9\u03c3\u03c4\u03bf\u03c2 \u03c6\u03c5\u03c3\u03b9\u03ba\u03bf\u03c2 k \u03b5\u03c4\u03c3\u03b9 \u03c9\u03c3\u03c4\u03b5 $ 2^k|[(3 + \\sqrt{11}){}^2{}^n{}^-{}^1]$", "Solution_1": "\u03a0\u03bf\u03bb\u03cd \u03c9\u03c1\u03b1\u03af\u03b1 \u03ba\u03b1\u03b9 \u03b4\u03cd\u03c3\u03ba\u03bf\u03bb\u03b7 \u03ac\u03c3\u03ba\u03b7\u03c3\u03b7. :!: \u03a4\u03b7 \u03bb\u03cd\u03c3\u03b7 \u03b1\u03c5\u03c4\u03ae\u03c2 \u03b2\u03c1\u03ae\u03ba\u03b1 \u03ba\u03b1\u03b8\u03ce\u03c2 \u03be\u03b5\u03c6\u03cd\u03bb\u03bb\u03b9\u03b6\u03b1 \u03c4\u03bf \u03b2\u03b9\u03b2\u03bb\u03af\u03bf \"\u03a0\u03b1\u03b3\u03ba\u03cc\u03c3\u03bc\u03b9\u03b1 \u0398\u03b5\u03bc\u03b1\u03c4\u03bf\u03b3\u03c1\u03b1\u03c6\u03af\u03b1 \u039f\u03bb\u03c5\u03bc\u03c0\u03b9\u03ac\u03b4\u03c9\u03bd \u039c\u03b1\u03b8\u03b7\u03bc\u03b1\u03c4\u03b9\u03ba\u03ae\u03c2 \u0391\u03bd\u03ac\u03bb\u03c5\u03c3\u03b7\u03c2\" \u03c4\u03bf\u03c5 \u0393\u03b9\u03ac\u03bd\u03bd\u03b7 \u039c\u03c0\u03b1\u03ca\u03bb\u03ac\u03ba\u03b7. \u039c\u03af\u03b1 \u03c5\u03c0\u03cc\u03b4\u03b5\u03b9\u03be\u03b7:[hide]\u0398\u03b5\u03c9\u03c1\u03bf\u03cd\u03bc\u03b5 \u03c4\u03b7\u03bd \u03b1\u03ba\u03bf\u03bb\u03bf\u03c5\u03b8\u03af\u03b1 \u03bc\u03b5 \u03bd-\u03bf\u03c3\u03c4\u03cc \u03cc\u03c1\u03bf $ (3+\\sqrt{11})^n$+$ (3-\\sqrt{11})^n$, \u03b3\u03b9\u03b1 \u03c4\u03b7\u03bd \u03bf\u03c0\u03bf\u03af\u03b1 \u03b5\u03af\u03bd\u03b1\u03b9 \u03b5\u03cd\u03ba\u03bf\u03bb\u03bf \u03bd\u03b1 \u03b2\u03c1\u03bf\u03cd\u03bc\u03b5 \u03b1\u03bd\u03b1\u03b4\u03c1\u03bf\u03bc\u03b9\u03ba\u03cc \u03c4\u03cd\u03c0\u03bf. \u0395\u03c0\u03af\u03c3\u03b7\u03c2 \u03b1\u03c0\u03bf\u03b4\u03b5\u03af\u03be\u03c4\u03b5 \u03cc\u03c4\u03b9 \u03c4\u03bf $ [(3+\\sqrt{11}){}^{2}{}^{n}{}^{-}{}^{1}]$ \u03b5\u03af\u03bd\u03b1\u03b9 \u03bf $ 2n-1$ \u03cc\u03c1\u03bf\u03c2 \u03c4\u03b7\u03c2 \u03b1\u03ba\u03bf\u03bb\u03bf\u03c5\u03b8\u03af\u03b1\u03c2 \u03b1\u03c5\u03c4\u03ae\u03c2. \u0395\u03c0\u03b1\u03b3\u03c9\u03b3\u03b9\u03ba\u03ac \u03b4\u03b5\u03af\u03c7\u03bd\u03bf\u03c5\u03bc\u03b5 \u03cc\u03c4\u03b9 \u03c4\u03bf $ 2^n$ \u03b4\u03b9\u03b1\u03b9\u03c1\u03b5\u03af \u03c4\u03bf\u03c5\u03c2 \u03cc\u03c1\u03bf\u03c5\u03c2 $ 2n-1$ \u03ba\u03b1\u03b9 $ 2n-2$ \u03c4\u03b7\u03c2 \u03c0\u03b1\u03c1\u03b1\u03c0\u03ac\u03bd\u03c9 \u03b1\u03ba\u03bf\u03bb\u03bf\u03c5\u03b8\u03af\u03b1\u03c2 \u03ba\u03b1\u03b9 \u03cc\u03c4\u03b9 \u03c4\u03bf $ 2$^(n+1) \u03b4\u03b5\u03bd \u03b4\u03b9\u03b1\u03b9\u03c1\u03b5\u03af \u03c4\u03bf\u03bd $ 2n-1$ \u03cc\u03c1\u03bf \u03b1\u03c5\u03c4\u03ae\u03c2.[/hide]", "Solution_2": "\u0388\u03bd\u03b1\u03c2 \u03c4\u03c1\u03cc\u03c0\u03bf\u03c2 \u03bd\u03b1 \u03bb\u03cd\u03c3\u03bf\u03c5\u03bc\u03b5 \u03c4\u03b7\u03bd \u03ac\u03c3\u03ba\u03b7\u03c3\u03b7 \u03b1\u03c5\u03c4\u03ae \u03b5\u03af\u03bd\u03b1\u03b9 \u03bf \u03b5\u03be\u03ae\u03c2:\r\n\r\n- \u039a\u03b1\u03c4\u03b1\u03c1\u03c7\u03ac\u03c2 \u03b8\u03ad\u03c4\u03bf\u03c5\u03bc\u03b5 $ a\\equal{}3\\plus{}\\sqrt{11}$ \u03ba\u03b1\u03b9 \u03c0\u03b1\u03c1\u03b1\u03c4\u03b7\u03c1\u03bf\u03cd\u03bc\u03b5 \u03cc\u03c4\u03b9 \u03bf \u03b1\u03c1\u03b9\u03b8\u03bc\u03cc\u03c2 $ a^{2n\\minus{}1}$ \u03b4\u03b5\u03bd \u03b5\u03af\u03bd\u03b1\u03b9 \u03c6\u03c5\u03c3\u03b9\u03ba\u03cc\u03c2. (\u03b1\u03bd \u03ae\u03c4\u03b1\u03bd \u03c6\u03c5\u03c3\u03b9\u03ba\u03cc\u03c2 \u03c4\u03cc\u03c4\u03b5 \u03c4\u03bf \u03b4\u03b9\u03ce\u03bd\u03c5\u03bc\u03bf \u03c4\u03bf\u03c5 \u039d\u03b5\u03cd\u03c4\u03c9\u03bd\u03b1 \u03bc\u03b1\u03c2 \u03b4\u03af\u03bd\u03b5\u03b9 \u03b5\u03cd\u03ba\u03bf\u03bb\u03b1 \u03cc\u03c4\u03b9 \u03bf $ \\sqrt{11}$ \u03b5\u03af\u03bd\u03b1\u03b9 \u03c1\u03b7\u03c4\u03cc\u03c2)\r\n\r\n- \u0392\u03c1\u03af\u03c3\u03ba\u03bf\u03c5\u03bc\u03b5 \u03bc\u03af\u03b1 \u03ad\u03ba\u03c6\u03c1\u03b1\u03c3\u03b7 \u03b3\u03b9\u03b1 \u03c4\u03bf \u03b1\u03ba\u03ad\u03c1\u03b1\u03b9\u03bf \u03bc\u03ad\u03c1\u03bf\u03c2 \u03c4\u03bf\u03c5 $ a^{2n\\minus{}1}$. \u0391\u03c5\u03c4\u03cc \u03b3\u03af\u03bd\u03b5\u03c4\u03b1\u03b9 \u03b4\u03bf\u03c5\u03bb\u03b5\u03cd\u03bf\u03bd\u03c4\u03b1\u03c2 \u03bc\u03b5 \u03c4\u03bf\u03bd \u03b1\u03c1\u03b9\u03b8\u03bc\u03cc $ b\\equal{}3\\minus{}\\sqrt{11}$. \u039f \u03b1\u03c1\u03b9\u03b8\u03bc\u03cc\u03c2\r\n\u03b1\u03c5\u03c4\u03cc\u03c2 \u03b2\u03c1\u03af\u03c3\u03ba\u03b5\u03c4\u03b1\u03b9 \u03bc\u03b5\u03c4\u03b1\u03be\u03cd -1 \u03ba\u03b1\u03b9 0. \u03a4\u03bf \u03af\u03b4\u03b9\u03bf \u03b9\u03c3\u03c7\u03cd\u03b5\u03b9 \u03ba\u03b1\u03b9 \u03b3\u03b9\u03b1 \u03c4\u03bf\u03bd $ b^{2n\\minus{}1}$. \u03a3\u03c4\u03b7 \u03c3\u03c5\u03bd\u03ad\u03c7\u03b5\u03b9\u03b1 \u03c4\u03bf \u03b4\u03b9\u03ce\u03bd\u03c5\u03bc\u03bf \u03c4\u03bf\u03c5 \u039d\u03b5\u03cd\u03c4\u03c9\u03bd\u03b1 \u03bc\u03b1\u03c2 \u03b4\u03af\u03bd\u03b5\u03b9 \u03cc\u03c4\u03b9 \u03bf\r\n\u03b1\u03c1\u03b9\u03b8\u03bc\u03cc\u03c2 $ a^{2n\\minus{}1}\\plus{}b^{2n\\minus{}1}$ \u03b5\u03af\u03bd\u03b1\u03b9 \u03c6\u03c5\u03c3\u03b9\u03ba\u03cc\u03c2 \u03ba\u03b1\u03b9 \u03c3\u03c5\u03bd\u03b5\u03c0\u03ce\u03c2 \u03ad\u03c7\u03bf\u03c5\u03bc\u03b5 \u03c4\u03b7\u03bd \u03b5\u03be\u03ae\u03c2 \u03ba\u03b1\u03c4\u03ac\u03c3\u03c4\u03b1\u03c3\u03b7. \u03a0\u03c1\u03bf\u03c3\u03b8\u03ad\u03c3\u03b1\u03bc\u03b5 \u03ad\u03bd\u03b1 \u03b1\u03c1\u03b9\u03b8\u03bc\u03cc \u03bc\u03b5\u03c4\u03b1\u03be\u03cd -1 \u03ba\u03b1\u03b9 0 \u03c3\u03b5\r\n\u03ad\u03bd\u03b1 \u03b8\u03b5\u03c4\u03b9\u03ba\u03cc \u03b1\u03c1\u03b9\u03b8\u03bc\u03cc \u03ba\u03b1\u03b9 \u03b2\u03c1\u03ae\u03ba\u03b1\u03bc\u03b5 \u03ad\u03bd\u03b1 \u03c6\u03c5\u03c3\u03b9\u03ba\u03cc \u03b1\u03c1\u03b9\u03b8\u03bc\u03cc. \u039f \u03c6\u03c5\u03c3\u03b9\u03ba\u03cc\u03c2 \u03b1\u03c5\u03c4\u03cc\u03c2 \u03b1\u03c1\u03b9\u03b8\u03bc\u03cc\u03c2 \u03c0\u03c1\u03ad\u03c0\u03b5\u03b9 \u03bd\u03b1 \u03b5\u03af\u03bd\u03b1\u03b9 \u03c4\u03bf \u03b1\u03ba\u03ad\u03c1\u03b1\u03b9\u03bf \u03bc\u03ad\u03c1\u03bf\u03c2 \u03c4\u03bf\u03c5 \u03b8\u03b5\u03c4\u03b9\u03ba\u03bf\u03cd \u03b1\u03c1\u03b9\u03b8\u03bc\u03bf\u03cd \u03c0\u03bf\u03c5\r\n\u03c0\u03c1\u03bf\u03b1\u03bd\u03b1\u03c6\u03ad\u03c1\u03b1\u03bc\u03b5.\r\n\r\n- \u03a3\u03c4\u03b7 \u03c3\u03c5\u03bd\u03ad\u03c7\u03b5\u03b9\u03b1 \u03b8\u03b1 \u03bc\u03b5\u03bb\u03b5\u03c4\u03ae\u03c3\u03bf\u03c5\u03bc\u03b5 \u03c4\u03b7\u03bd \u03b1\u03ba\u03bf\u03bb\u03bf\u03c5\u03b8\u03af\u03b1 $ x_n\\equal{}a^n\\plus{}b^n$. \u03a0\u03c1\u03bf\u03ba\u03b5\u03b9\u03c4\u03b1\u03b9 \u03b3\u03b9\u03b1 \u03bc\u03b9\u03b1 \u03b1\u03ba\u03bf\u03bb\u03bf\u03c5\u03b8\u03af\u03b1 \u03c6\u03c5\u03c3\u03b9\u03ba\u03ce\u03bd \u03b1\u03c1\u03b9\u03b8\u03bc\u03ce\u03bd \u03b7 \u03bf\u03c0\u03bf\u03af\u03b1 \u03c0\u03b5\u03c1\u03b9\u03b3\u03c1\u03ac\u03c6\u03b5\u03c4\u03b1\u03b9 \u03b1\u03c0\u03cc \u03c4\u03b7\u03bd \u03b1\u03bd\u03b1\u03b4\u03c1\u03bf\u03bc\u03b9\u03ba\u03ae \u03c3\u03c7\u03ad\u03c3\u03b7:\r\n\r\n\\[ x_0\\equal{}2\\quad x_1\\equal{}6\\quad x_{n\\plus{}2}\\equal{}6x_{n\\plus{}1}\\plus{}2x_n\\]\r\n\r\n\u0397 \u03b8\u03b5\u03c9\u03c1\u03af\u03b1 \u03c4\u03c9\u03bd \u03b1\u03ba\u03bf\u03bb\u03bf\u03c5\u03b8\u03b9\u03ce\u03bd \u03b1\u03c5\u03c4\u03ce\u03bd \u03b5\u03af\u03bd\u03b1\u03b9 \u03c3\u03c4\u03bf\u03b9\u03c7\u03b5\u03b9\u03ce\u03b4\u03b7\u03c2 \u03ba\u03b1\u03b9 \u03c3\u03c5\u03bd\u03b1\u03bd\u03c4\u03ac\u03c4\u03b1\u03b9 \u03ba\u03b1\u03b9 \u03c3\u03c4\u03b7 \u03bc\u03b5\u03bb\u03ad\u03c4\u03b7 \u03c4\u03b7\u03c2 \u03b1\u03ba\u03bf\u03bb\u03bf\u03c5\u03b8\u03af\u03b1\u03c2 Fibonacci. \u039c\u03b9\u03b1 \u03b1\u03ba\u03bf\u03bb\u03bf\u03c5\u03b8\u03b9\u03b1 \u03bc\u03b5 \u03c4\u03b7\u03bd \r\n\u03c0\u03b1\u03c1\u03b1\u03c0\u03ac\u03bd\u03c9 \u03b1\u03bd\u03b1\u03b4\u03c1\u03bf\u03bc\u03b9\u03ba\u03ae \u03c3\u03c7\u03ad\u03c3\u03b7 \u03ad\u03c7\u03b5\u03b9 \u03c0\u03ac\u03bd\u03c4\u03b1 \u03c4\u03b7 \u03bc\u03bf\u03c1\u03c6\u03ae $ \\lambda_1\\omega_1^n\\plus{}\\lambda_2\\omega_2^n$ \u03cc\u03c0\u03bf\u03c5 $ \\omega_1$ kai $ \\omega_2$ \u03b5\u03af\u03bd\u03b1\u03b9\r\n\u03bf\u03b9 \u03bb\u03cd\u03c3\u03b5\u03b9\u03c2 \u03c4\u03b7\u03c2 \u03c7\u03b1\u03c1\u03b1\u03ba\u03c4\u03b7\u03c1\u03b9\u03c3\u03c4\u03b9\u03ba\u03ae\u03c2 \u03b5\u03be\u03af\u03c3\u03c9\u03c3\u03b7\u03c2 $ \\omega^2\\minus{}6\\omega\\minus{}2\\equal{}0$. \u0395\u03b4\u03ce \u03c6\u03c5\u03c3\u03b9\u03ba\u03ac \u03ad\u03c7\u03bf\u03c5\u03bc\u03b5 $ \\omega_1\\equal{}a$ \u03ba\u03b1\u03b9 $ \\omega_2\\equal{}b$.\r\n\r\n- \u03a3\u03c4\u03bf \u03c4\u03b5\u03bb\u03b5\u03c5\u03c4\u03b1\u03af\u03bf \u03c3\u03c4\u03ac\u03b4\u03b9\u03bf \u03c4\u03b7\u03c2 \u03bb\u03cd\u03c3\u03b7\u03c2 \u03b8\u03b1 \u03c7\u03c1\u03b7\u03c3\u03b9\u03bc\u03bf\u03c0\u03bf\u03b9\u03ae\u03c3\u03bf\u03c5\u03bc\u03b5 \u03c4\u03b7 \u03c3\u03c5\u03bd\u03ac\u03c1\u03c4\u03b7\u03c3\u03b7 \u03b4\u03c5\u03b1\u03b4\u03b9\u03ba\u03ae\u03c2 \u03c4\u03af\u03bc\u03b7\u03c3\u03b7\u03c2. \u03a3\u03c4\u03b7\u03bd \u03c0\u03c1\u03bf\u03ba\u03b5\u03b9\u03bc\u03ad\u03bd\u03b7 \u03c0\u03b5\u03c1\u03af\u03c0\u03c4\u03c9\u03c3\u03b7 \u03bc\u03c0\u03bf\u03c1\u03bf\u03cd\u03bc\u03b5 \u03bd\u03b1\r\n\u03c4\u03b7 \u03b8\u03b5\u03c9\u03c1\u03ae\u03c3\u03bf\u03c5\u03bc\u03b5 \u03c3\u03b1 \u03bc\u03b9\u03b1 \u03c3\u03c5\u03bd\u03ac\u03c1\u03c4\u03b7\u03c3\u03b7 \u03bc\u03b5 \u03c0\u03b5\u03b4\u03af\u03bf \u03bf\u03c1\u03b9\u03c3\u03bc\u03bf\u03cd \u03c4\u03bf\u03c5\u03c2 \u03c6\u03c5\u03c3\u03b9\u03ba\u03bf\u03cd\u03c2 \u03ba\u03b1\u03b9 \u03c3\u03cd\u03bd\u03bf\u03bb\u03bf \u03c4\u03b9\u03bc\u03ce\u03bd \u03c4\u03bf\u03c5\u03c2 \u03bc\u03b7 \u03b1\u03c1\u03bd\u03b7\u03c4\u03b9\u03ba\u03bf\u03cd\u03c2 \u03b1\u03ba\u03b5\u03c1\u03b1\u03af\u03bf\u03c5\u03c2. \u0391\u03bd $ n$ \u03b5\u03af\u03bd\u03b1\u03b9\r\n\u03ad\u03bd\u03b1\u03c2 \u03c6\u03c5\u03c3\u03b9\u03ba\u03cc\u03c2 \u03b1\u03c1\u03b9\u03b8\u03bc\u03cc\u03c2 \u03c4\u03cc\u03c4\u03b5 \u03bf\u03c1\u03af\u03b6\u03bf\u03c5\u03bc\u03b5 $ v(n)$ \u03c9\u03c2 \u03c4\u03bf \u03bc\u03ad\u03b3\u03b9\u03c3\u03c4\u03bf \u03bc\u03b7 \u03b1\u03c1\u03bd\u03b7\u03c4\u03b9\u03ba\u03cc \u03b1\u03ba\u03ad\u03c1\u03b1\u03b9\u03bf \u03c0\u03bf\u03c5 \u03b5\u03af\u03bd\u03b1\u03b9 \u03c4\u03ad\u03c4\u03bf\u03b9\u03bf\u03c2 \u03ce\u03c3\u03c4\u03b5 $ 2^{v(n)}|n$. \u0393\u03b9\u03b1 \u03c0\u03b1\u03c1\u03ac\u03b4\u03b5\u03b9\u03b3\u03bc\u03b1\r\n$ v(1)\\equal{}0$, $ v(2)\\equal{}1$, $ v(3)\\equal{}0$, $ v(4)\\equal{}2$ \u03ba\u03bb\u03c0. \u0398\u03b1 \u03c7\u03c1\u03b5\u03b9\u03b1\u03c3\u03c4\u03bf\u03cd\u03bc\u03b5 \u03b4\u03cd\u03bf \u03b2\u03b1\u03c3\u03b9\u03ba\u03ad\u03c2 \u03ba\u03b1\u03b9 \u03c3\u03c4\u03bf\u03b9\u03c7\u03b5\u03b9\u03ce\u03b4\u03b5\u03b9\u03c2 \u03b9\u03b4\u03b9\u03cc\u03c4\u03b7\u03c4\u03b5\u03c2 \u03c4\u03b7\u03c2 \u03c3\u03c5\u03bd\u03ac\u03c1\u03c4\u03b7\u03c3\u03b7\u03c2 \u03b1\u03c5\u03c4\u03ae\u03c2. \u0397 \u03c0\u03c1\u03ce\u03c4\u03b7 \u03b5\u03af\u03bd\u03b1\u03b9 \u03cc\u03c4\u03b9 $ v(nm)\\equal{}v(n)\\plus{}v(m)$ \u03b3\u03b9\u03b1 \u03bf\u03c0\u03bf\u03b9\u03bf\u03c5\u03c3\u03b4\u03ae\u03c0\u03bf\u03c4\u03b5 \u03b4\u03cd\u03bf \u03c6\u03c5\u03c3\u03b9\u03ba\u03bf\u03cd\u03c2 \u03b1\u03c1\u03b9\u03b8\u03bc\u03bf\u03cd\u03c2 $ m$ kai $ n$ \u03ba\u03b1\u03b9 \u03b7 \u03b4\u03b5\u03cd\u03c4\u03b5\u03c1\u03b7 \u03b5\u03af\u03bd\u03b1\u03b9 \u03cc\u03c4\u03b9 $ v(m\\plus{}n)\\geq \\min(v(m),v(n))$, \u03ba\u03b1\u03b9 \u03b1\u03bd $ v(m)\\neq v(n)$ \u03c4\u03cc\u03c4\u03b5 $ v(m\\plus{}n)\\equal{} \\min(v(m),v(n))$. \u0391\u03be\u03af\u03b6\u03b5\u03b9 \u03bd\u03b1 \u03c3\u03c7\u03bf\u03bb\u03b9\u03ac\u03c3\u03bf\u03c5\u03bc\u03b5 \u03bb\u03af\u03b3\u03bf \u03c4\u03b7 \u03b4\u03b5\u03cd\u03c4\u03b5\u03c1\u03b7 \u03b9\u03b4\u03b9\u03cc\u03c4\u03b7\u03c4\u03b1. \u0391\u03c2 \u03c5\u03c0\u03bf\u03b8\u03ad\u03c3\u03bf\u03c5\u03bc\u03b5 \u03cc\u03c4\u03b9 \u03ad\u03c7\u03bf\u03c5\u03bc\u03b5 \u03b5\u03bd\u03b1 \u03c0\u03bf\u03bb\u03bb\u03b1\u03c0\u03bb\u03ac\u03c3\u03b9\u03bf \u03c4\u03bf\u03c5 8 \u03ba\u03b1\u03b9 \u03ad\u03bd\u03b1 \u03b1\u03ba\u03c1\u03b9\u03b2\u03ad\u03c2 \u03c0\u03bf\u03bb\u03bb\u03b1\u03c0\u03bb\u03ac\u03c3\u03b9\u03bf \u03c4\u03bf\u03c5 4. \u039c\u03b5 \u03b1\u03c5\u03c4\u03cc \u03b5\u03bd\u03bd\u03bf\u03bf\u03cd\u03bc\u03b5 \u03cc\u03c4\u03b9 \u03bf \u03b4\u03b5\u03cd\u03c4\u03b5\u03c1\u03bf\u03c2 \u03b1\u03c1\u03b9\u03b8\u03bc\u03cc\u03c2 \u03b4\u03b9\u03b1\u03b9\u03c1\u03b5\u03af\u03c4\u03b1\u03b9 \u03bc\u03b5 \u03c4\u03bf 4 \u03b1\u03bb\u03bb\u03ac \u03cc\u03c7\u03b9 \u03bc\u03b5 \u03c4\u03bf 8. \u03a4\u03cc\u03c4\u03b5 \u03c4\u03bf \u03ac\u03b8\u03c1\u03bf\u03b9\u03c3\u03bc\u03ac \u03c4\u03bf\u03c5\u03c2 \u03b5\u03af\u03bd\u03b1\u03b9 \u03ad\u03bd\u03b1 \u03b1\u03ba\u03c1\u03b9\u03b2\u03ad\u03c2 \u03c0\u03bf\u03bb\u03bb\u03b1\u03c0\u03bb\u03ac\u03c3\u03b9\u03bf \u03c4\u03bf\u03c5 4.\r\n\r\n- \u039f \u03c3\u03ba\u03bf\u03c0\u03cc\u03c2 \u03c4\u03b7\u03c2 \u03ac\u03c3\u03ba\u03b7\u03c3\u03b7\u03c2 \u03b5\u03af\u03bd\u03b1\u03b9 \u03bd\u03b1 \u03c5\u03c0\u03bf\u03bb\u03bf\u03b3\u03af\u03c3\u03bf\u03c5\u03bc\u03b5 \u03c4\u03bf $ v(x_{2n\\minus{}1})$. \u03a7\u03c1\u03b7\u03c3\u03b9\u03bc\u03bf\u03c0\u03bf\u03b9\u03ce\u03bd\u03c4\u03b1\u03c2 \u03c4\u03b7\u03bd \u03b1\u03bd\u03b1\u03b4\u03c1\u03bf\u03bc\u03b9\u03ba\u03ae \u03c3\u03c7\u03ad\u03c3\u03b7 \u03bc\u03c0\u03bf\u03c1\u03bf\u03cd\u03bc\u03b5 \u03bd\u03b1 \u03b4\u03bf\u03cd\u03bc\u03b5 \u03cc\u03c4\u03b9:\r\n\r\n\\[ x_0\\equal{}2\\quad x_1\\equal{}6 \\quad x_2\\equal{}40 \\quad x_3\\equal{}252\\]\r\n\r\n\u03b1\u03c0' \u03cc\u03c0\u03bf\u03c5 \u03bb\u03b1\u03bc\u03b2\u03ac\u03bd\u03bf\u03c5\u03bc\u03b5:\r\n\r\n\\[ v(x_0)\\equal{}1\\quad v(x_1)\\equal{}1\\quad v(x_2)\\equal{}3 \\quad v(x_3)\\equal{}2\\]\r\n\r\n\u0398\u03b1 \u03c7\u03c1\u03b7\u03c3\u03b9\u03bc\u03bf\u03c0\u03bf\u03b9\u03ae\u03c3\u03bf\u03c5\u03bc\u03b5 \u03b5\u03c0\u03b1\u03b3\u03c9\u03b3\u03ae \u03b3\u03b9\u03b1 \u03bd\u03b1 \u03b4\u03b5\u03af\u03be\u03bf\u03c5\u03bc\u03b5 \u03cc\u03c4\u03b9:\r\n\r\n\\[ v(x_{4k})\\geq 2k\\plus{}1 \\quad v(x_{4k\\plus{}1})\\equal{}2k\\plus{}1 \\quad v(x_{4k\\plus{}2})\\geq 2k\\plus{}2 \\quad v(x_{4k\\plus{}3})\\equal{}2k\\plus{}2\\]\r\n\r\n\u03b3\u03b9\u03b1 \u03ba\u03ac\u03b8\u03b5 $ k$ \u03bc\u03b7 \u03b1\u03c1\u03bd\u03b7\u03c4\u03b9\u03ba\u03cc \u03b1\u03ba\u03ad\u03c1\u03b1\u03b9\u03bf.\r\n\r\n\u03a0\u03c1\u03b1\u03b3\u03bc\u03b1\u03c4\u03b9\u03ba\u03ac \u03b1\u03bd \u03b9\u03c3\u03c7\u03cd\u03bf\u03c5\u03bd \u03bf\u03b9 \u03c0\u03b1\u03c1\u03b1\u03c0\u03ac\u03bd\u03c9 \u03c3\u03c7\u03ad\u03c3\u03b5\u03b9\u03c2 \u03b8\u03b1 \u03b4\u03b5\u03af\u03be\u03bf\u03c5\u03bc\u03b5 \u03cc\u03c4\u03b9 \u03b9\u03c3\u03c7\u03cd\u03bf\u03c5\u03bd \u03b1\u03bd \u03c4\u03bf $ k$ \u03b1\u03bd\u03c4\u03b9\u03ba\u03b1\u03c4\u03b1\u03c3\u03c4\u03b1\u03b8\u03b5\u03af \u03bc\u03b5 $ k\\plus{}1$. (H $ k\\equal{}0$ \u03b5\u03af\u03bd\u03b1\u03b9 \u03c0\u03c1\u03bf\u03c6\u03b1\u03bd\u03ae\u03c2\r\n\u03b1\u03c0\u03cc \u03c4\u03bf\u03bd \u03b1\u03c1\u03c7\u03b9\u03ba\u03cc \u03bb\u03bf\u03b3\u03b1\u03c1\u03b9\u03b1\u03c3\u03bc\u03cc.)\r\n\r\n\u039e\u03ad\u03c1\u03bf\u03c5\u03bc\u03b5 \u03cc\u03c4\u03b9 $ x_{4k\\plus{}4}\\equal{}6x_{4k\\plus{}3}\\plus{}2x_{4k\\plus{}2}$. \u0397 \u03b5\u03c0\u03b1\u03b3\u03c9\u03b3\u03b9\u03ba\u03ae \u03c5\u03c0\u03cc\u03b8\u03b5\u03c3\u03b7 \u03c3\u03c5\u03bd\u03b5\u03c0\u03ac\u03b3\u03b5\u03c4\u03b1\u03b9 \u03cc\u03c4\u03b9\r\n$ v(6x_{4k\\plus{}3})\\equal{}2k\\plus{}3$ \u03ba\u03b1\u03b9 $ v(2x_{4k\\plus{}2})\\geq 2k\\plus{}3$. \u0397 \u03b4\u03b5\u03cd\u03c4\u03b5\u03c1\u03b7 \u03b9\u03b4\u03b9\u03cc\u03c4\u03b7\u03c4\u03b1 \u03c4\u03b7\u03c2 \u03c3\u03c5\u03bd\u03ac\u03c1\u03c4\u03b7\u03c3\u03b7\u03c2 \u03c4\u03af\u03bc\u03b7\u03c3\u03b7\u03c2 \u03c3\u03c5\u03bd\u03b5\u03c0\u03ac\u03b3\u03b5\u03c4\u03b1\u03b9 \u03cc\u03c4\u03b9 $ v(4k\\plus{}4)\\geq 2k\\plus{}3$.\r\n\r\n\u039f \u03c3\u03c5\u03bb\u03bb\u03bf\u03b3\u03b9\u03c3\u03bc\u03cc\u03c2 \u03b3\u03b9\u03b1 \u03c4\u03b7\u03bd \u03b5\u03c0\u03cc\u03bc\u03b5\u03bd\u03b7 \u03c3\u03c7\u03ad\u03c3\u03b7 \u03b5\u03af\u03bd\u03b1\u03b9 \u03c0\u03b1\u03c1\u03cc\u03bc\u03bf\u03b9\u03bf\u03c2:\r\n\r\n\u0399\u03c3\u03c7\u03cd\u03b5\u03b9 $ x_{4k\\plus{}5}\\equal{}6x_{4k\\plus{}4}\\plus{}2x_{4k\\plus{}3}$. \u0391\u03c0\u03cc \u03c4\u03b9\u03c2 $ v(6x_{4k\\plus{}4})\\geq 2k\\plus{}4$ \u03ba\u03b1\u03b9 $ v(2x_{4k\\plus{}3})\\equal{} 2k\\plus{}3$, \u03ad\u03c0\u03b5\u03c4\u03b1\u03b9 $ v(x_{4k\\plus{}5})\\equal{}2k\\plus{}3$.\r\n\r\n\u03a0\u03b1\u03c1\u03b1\u03bb\u03b5\u03af\u03c0\u03bf\u03c5\u03bc\u03b5 \u03c4\u03b9\u03c2 \u03b4\u03c5\u03bf \u03c4\u03b5\u03bb\u03b5\u03c5\u03c4\u03b1\u03af\u03b5\u03c2 \u03c3\u03c7\u03ad\u03c3\u03b5\u03b9\u03c2 \u03c4\u03c9\u03bd \u03bf\u03c0\u03bf\u03af\u03c9\u03bd \u03b7 \u03b1\u03c0\u03cc\u03b4\u03b5\u03b9\u03be\u03b7 \u03b5\u03af\u03bd\u03b1\u03b9 \u03c0\u03b1\u03c1\u03cc\u03bc\u03bf\u03b9\u03b1.\r\n\r\n\u0391\u03c5\u03c4\u03cc \u03c0\u03bf\u03c5 \u03bc\u03b1\u03c2 \u03b5\u03bd\u03b4\u03b9\u03b1\u03c6\u03ad\u03c1\u03b5\u03b9 \u03b5\u03af\u03bd\u03b1\u03b9 \u03cc\u03c4\u03b9:\r\n\r\n\\[ v(x_{4k\\plus{}1})\\equal{}2k\\plus{}1 \\quad v(x_{4k\\plus{}3})\\equal{}2k\\plus{}2\\]\r\n\r\n\u039f\u03b9 \u03b4\u03cd\u03bf \u03b1\u03c5\u03c4\u03ad\u03c2 \u03c3\u03c7\u03ad\u03c3\u03b5\u03b9\u03c2 \u03c3\u03c5\u03bd\u03bf\u03c8\u03af\u03b6\u03bf\u03bd\u03c4\u03b1\u03b9 \u03c3\u03c4\u03b7\u03bd $ v(x_{2n\\minus{}1})\\equal{}n$ \u03b7 \u03bf\u03c0\u03bf\u03af\u03b1 \u03b5\u03af\u03bd\u03b1\u03b9 \u03ba\u03b1\u03b9 \u03b7 \u03b1\u03c0\u03ac\u03bd\u03c4\u03b7\u03c3\u03b7 \u03c3\u03c4\u03bf \u03b5\u03c1\u03ce\u03c4\u03b7\u03bc\u03b1 \u03c4\u03b7\u03c2 \u03ac\u03c3\u03ba\u03b7\u03c3\u03b7\u03c2." } { "Tag": [ "trigonometry", "rotation", "geometry", "trig identities", "Law of Sines" ], "Problem": "Two lighthouses are located on a north-south line. From lighthouse A, the baring of a ship 3742 m away is 129 degrees and 43 minutes. From lightouse B, the bearing is 39 degrees and 43 minutes. Find the distance between the lighthouses.\r\n\r\nI'm not sure how to set up the diagram. I drew a y-axis, and put lighthouses A and B on it. Then I rotated 129 degrees and 43 minutes clockwise. I'm not sure if I'm drawing it right. Will someone please help me?", "Solution_1": "i believe that this is a law of sines problem, try drawing the lighthouses, and then drawing another point for the ship so they form those angles\r\n\r\nA\r\n|\r\n|\r\n|\r\n............Ship\r\n|\r\n|\r\n|B" } { "Tag": [ "topology" ], "Problem": "let $ A\\subset \\mathbb{R}^2$ be a countable subset , prove that $ \\mathbb{R}^2\\setminus A$ is path-connected ?\r\n\r\nthe hint in the book is : how many lines pass through a point in $ \\mathbb{R}^2$ ? \r\n\r\nBut I don't really get it, could some one point it out for me ? thanks", "Solution_1": "Fix $ x\\in\\mathbb{R}^{2}\\setminus A$, and consider the set of all (straight) lines through $ x$. There are uncountably many of these and they meet in the single point $ x$ , so not all of them contain a point of $ A$ . Choose one that doesn't and move along it: your distance from $ p$ takes on uncountably many values, and hence at some point this distance $ r$ from $ p$ is not shared by any point of $ A$ . The whole of the circle with radius $ r$ and center $ p$, lies in $ \\mathbb{R}^{2}\\setminus A$ so we may move around it freely.\r\n\r\nConsider all lines through $ p$: these all intersect this circle, and there are uncountably many of them so we may choose one, say $ L$ , that contains no point of $ A$. Moving around the circle until we meet $ L$ and then following it inwards completes our path from $ x$ to $ p$.", "Solution_2": "The one-step version: to get from $ A$ to $ B$, consider all circular arcs from $ A$ to $ B$; there are two for each possible center, and they're all disjoint. There are uncountably many of these, so some miss all of the points in $ A$.", "Solution_3": "Let $ x,y\\in\\mathbb{R}^{2}\\setminus A$; then, as was alraedy said, there is an uncountable number of lines passing through x that doesn't contain any points from $ A$ (with the possible exception of x, but this doesn't matter); if one of these lines also contains $ y$, then we're done. If not, consider the similar lines passing through $ y$; if one of them intersects another passing through $ x$, we're done again; if not, they must be paralell, so pick a point $ z$ in one of them, a line through z that doesn't intersect $ A$ and is different from both lines previously chosen; this line must intersect the other lines and, from the three, you may construct a path from $ x$ to $ y$." } { "Tag": [ "Putnam", "superior algebra", "superior algebra unsolved" ], "Problem": "Let $G$ be a infinite group with the following propertis:\r\n(I) :if $a\\in{G}$ and $n\\in{\\mathbb{N}}$ and $a^{n}=e$ then $a=e$ \r\n(II):if $x,y\\in{G}$ then $(xy)^{2}=(yx)^{2}$\r\nprove that $G$ is commutative.", "Solution_1": "Multiplying by $(y^{-1}x^{-1})^{2}$ we have\r\n\r\n$(y^{-1}x^{-1})^{2}(xy)^{2}= e = (y^{-1}x^{-1}yx)^{2}= y^{-1}x^{-1}yx$\r\n\r\nMultiplying by $xy$ we have\r\n\r\n$yx = xy$\r\n\r\nAs desired. QED.", "Solution_2": "[quote=\"t0rajir0u\"]Multiplying by $(y^{-1}x^{-1})^{2}$ we have\n\n$(y^{-1}x^{-1})^{2}(xy)^{2}= e = (y^{-1}x^{-1}yx)^{2}$[/quote]\r\nthanks t0rajir0u;\r\nI think Multiplying by $(y^{-1}x^{-1})^{2}$ give:\r\n$(y^{-1}x^{-1})^{2}(xy)^{2}= e = (y^{-1}x^{-1})^{2}(yx)^{2}$\r\ncan you tell me why we have:$(y^{-1}x^{-1})^{2}(yx)^{2}=(y^{-1}x^{-1}yx)^{2}$", "Solution_3": "Hi my friend :) \r\nLet$z = xyx^{-1}$ then $z^{2}= (xyx^{-1})(xyx^{-1})=xy^{2}x^{-1}$,\r\nbut$z^{2}= (xyx^{-1})^{2}= (x^{-1}xy)^{2}= y^{2}$\r\nThis implies$xy^{2}= y^{2}x$ for all x and y lie in $G$\r\nNow assume $xy = w(yx)$ then we conclude that:\r\n\r\n$(xy)^{2}= (wyx)(xy) =wyx^{2}y=wy^{2}x^{2}=(yx)^{2}=(yx)(yx)$\r\n$wy^{2}x=yxy$\r\n$wxy^{2}=yxy$\r\n$wxy = yx$\r\n$w^{2}(yx)=yx$\r\n$w^{2}= e$ \r\nthen $w=e$ and$xy = yx$", "Solution_4": "[quote=\"ali666\"]can you tell me why we have:$(y^{-1}x^{-1})^{2}(yx)^{2}=(y^{-1}x^{-1}yx)^{2}$[/quote]\r\n\r\nAssociativity. At least, I hope it's just associativity.", "Solution_5": "Is there a problem source because this problem looks familiar, possibly from an old Putnam?", "Solution_6": "t0rajir0u...that's not associativity... in order to have that should know that the group is comutative :maybe:", "Solution_7": "Oops. I see now. My mistake :oops:", "Solution_8": "[quote=\"me@home\"]Is there a problem source because this problem looks familiar, possibly from an old Putnam?[/quote]\r\n sorry,I don't know the original source.", "Solution_9": "[quote=\"sina rezazadeh baghal\"]Hi my friend :) \nLet$z = xyx^{-1}$ then $z^{2}= (xyx^{-1})(xyx^{-1})=xy^{2}x^{-1}$,\nbut$z^{2}= (xyx^{-1})^{2}= (x^{-1}xy)^{2}= y^{2}$\nThis implies$xy^{2}= y^{2}x$ for all x and y lie in $G$\nNow assume $xy = w(yx)$ then we conclude that:\n\n$(xy)^{2}= (wyx)(xy) =wyx^{2}y=wy^{2}x^{2}=(yx)^{2}=(yx)(yx)$\n$wy^{2}x=yxy$\n$wxy^{2}=yxy$\n$wxy = yx$\n$w^{2}(yx)=yx$\n$w^{2}= e$ \nthen $w=e$ and$xy = yx$[/quote]\r\nvery nice!\r\nThx a lot my friend :roll:" } { "Tag": [ "combinatorics unsolved", "combinatorics" ], "Problem": "We have an equilateral triangle with the edge sized 2.\r\nin each generation,we will erect a new equilateral triangle inside every existing triangle ,by drawing lines between the midpoints of it's edges...,so,let n be the number of generation,then:\r\nif n = 1 , we have 1 triangle\r\nif n = 2 , we will have 5 triangle (4 triangles with the edge sized 1,and 1 triangle with the edge sized 2)\r\nif n = 3 , we will have 27 triangles (16 (1/2)-sized-edge triangles,7 1-sized-edge triangles,3 (3/2)-sized-edge triangles,and 1 4-sized-edge triangle)\r\n...\r\ndetermine the number of all triangles in the generation number n,in terms of n.", "Solution_1": "Well..if I do understand, you just want to determine the number of equilateral triangles (of any size) formed when each side of an equilateral triangle is divided by by points into $ n$ equal parts, and when we draw lines parallel to the sides of the triangle and joining these points by pairs?\r\n\r\nPierre.", "Solution_2": "That appears to be the question, although the O.P. has asked only for the answer in the case $ n$ is a power of $ 2$. I suggest splitting into the \"right side up\" and \"upside down\" cases -- the former are easy while the latter are a little more complicated.\r\n\r\nThis is sequence number [url=http://www.research.att.com/~njas/sequences/A002717]A002717[/url] in the [url=http://www.research.att.com/~njas/sequences/]OEIS[/url].", "Solution_3": "$ \\displaystyle{\\sum_{i \\equal{} 1}^{2^{n \\minus{} 1}} \\sum_{j \\equal{} 1}^{2^{n \\minus{} 1} \\plus{} 1 \\minus{} i}j} \\plus{} {\\sum_{i \\equal{} 1}^{2^{n \\minus{} 2}} \\sum_{j \\equal{} 1}^{2^{n \\minus{} 1} \\plus{} 1 \\minus{} 2i}j}$", "Solution_4": "yes pbornsztein,that's what i was trying to state... :) \r\ncan you please post your solutions too? (i mean not just the answer)" } { "Tag": [ "geometry", "parallelogram", "geometric transformation", "reflection", "similar triangles", "geometry unsolved" ], "Problem": "Given a parallelogram $ ABCD$ ($ BD>AC$). A circle with diameter $ AC$ cut $ BD$ at $ P,Q$ ($ BQ>BP$) A tangent to that circle at $ C$ meet $ AB,AD$ at $ X,Y$ respectively. Prove that $ P,Q,X,Y$ are concyclic.", "Solution_1": "Let $ O$ be the center of the parallelogram and $ M \\equiv{} DB \\cap XY.$ $ C'$ is the reflection of $ C$ about $ Y.$ Then $ AC' \\parallel DB$ and let $ P_{\\infty}$ be their infinite point. Since $(D,B,O,P_{\\infty})=-1,$ it follows that $( X,C,Y,C')=-1$ $\\Longrightarrow$ $ MC^2 \\equal{} MY \\cdot MX,$ but $ MC^2$ is the power of $ M$ to the circle with diameter $\\overline{AC}$ $\\Longrightarrow$ $ MY \\cdot MX \\equal{} MP \\cdot MQ.$\n\nP.S. We also can go for a couple of similar triangles to get the same relation.", "Solution_2": "[quote] We also can go for a couple of similar triangles to get the same relation.[/quote]\nI don't think it's that easy. Can you show it?", "Solution_3": "$\\triangle MBY \\sim \\triangle MDC \\Longrightarrow \\ \\frac {MB}{MD} \\equal{} \\frac {MY}{MC} \\ \\ (1)$\n\n$\\triangle MBC \\sim \\triangle MDX \\Longrightarrow \\ \\frac {MD}{MB} \\equal{} \\frac {MX}{MC} \\ \\ (2)$\n\nCombining $(1)$ and $(2)$ yields $ MC^2 \\equal{} MX \\cdot MY,$ and the conclusion follows." } { "Tag": [ "geometry", "quadratics", "algebra", "polynomial", "superior algebra", "superior algebra unsolved" ], "Problem": "Suppose that F is a subfield of the field of real numbers and that c is a real number whose square root is not in F. Show that any number d in the square root extension field F[c] is a root of a linear or quadratic polynomial P(x) with coefficients in F.", "Solution_1": "This is not (really) geometry, Olympiad, or an Inequality. It belongs [url=http://www.artofproblemsolving.com/Forum/index.php?f=61]here[/url].", "Solution_2": "This is standard algebra, and true for any field $ F$ and any solution $ c$ of a quadratic equation in $ F$.", "Solution_3": "[quote=\"t0rajir0u\"]This is not (really) geometry, Olympiad, or an Inequality. It belongs [url=http://www.artofproblemsolving.com/Forum/index.php?f=61]here[/url].[/quote]\r\n\r\nI am new and I could not find this place for my posting, but I am not sure if I got an answer from you or anyone else yet. Please advise. Thanks", "Solution_4": "Well, I assume that that should be $ \\sqrt{c}$. It is indeed a general fact that if $ \\alpha$ is a root of an irreducible polynomial $ p$ over $ K$, every $ x\\in K(\\alpha)$ is a root of some polynomial over $ K$ with degree dividing the degree of $ p$.\r\nIn this quadratic case, you can explicitly construct a polynomial that $ a\\plus{}b\\sqrt{c}$ satisfies.", "Solution_5": "[quote=\"ZetaX\"]This is standard algebra, and true for any field $ F$ and any solution $ c$ of a quadratic equation in $ F$.[/quote]\r\n\r\nDo you have a more explicit proof, because that is what I need. Thanks", "Solution_6": "$ (x\\minus{}(a\\plus{}b\\sqrt{c}))(x\\minus{}(a\\minus{}b\\sqrt{c}))\\equal{}x^2\\minus{}2ax\\plus{}(a^2\\minus{}b^2c)$.", "Solution_7": "[quote=\"jmerry\"]$ (x \\minus{} (a \\plus{} b\\sqrt {c}))(x \\minus{} (a \\minus{} b\\sqrt {c})) \\equal{} x^2 \\minus{} 2ax \\plus{} (a^2 \\minus{} b^2c)$.[/quote]\r\n\r\nDid I get it right here?\r\nI assume that the number d should be sqrt(c). It is a general fact that if d is a root of an irreducible polynomial p over F, every element of F(d) is a root of some polynomial over K with degree dividing the degree of p. \r\nIn this case, we can explicitly construct a quadratic polynomial satisfied by a+b*sqrt(c) and its conjugate, where a, b, and c are in F, based on the fact that P(x)= (x-1st root)*(x-2nd root).\r\nP(x) = [x-(a+b*sqrt(c))]*[x-(a-b*sqrt(c))]=x^2-2ax+(a^2-(b^2)*c), which is the quadratic polynomial P(x) with coefficients 1, -2a, and a^2-(b^2)*c in F.\r\nThanks jmerry.", "Solution_8": "$ d$ is one of the two roots; it doesn't matter which one. You do have to verify that all numbers in the field have that form." } { "Tag": [ "algebra", "polynomial", "algorithm" ], "Problem": "The remainder when $x^5-3x^2+ax+b$ is divided by $(x-1)(x-2)$ is $11x-10$. Find $a, b$.", "Solution_1": "[hide]dividing $x^5-3x^2+ax+b$ by $x^2-3x+2$ we obtain the result by $a-14=36-11$ $b-24=-10 \\rightarrow a=-11$ $b=14$ looking at the remainders in the division[/hide]", "Solution_2": "yep thats right :lol:", "Solution_3": "[hide=\"hint\"]\nLet $g(x)$ be the quotient of the division. Then by the division algorithm we have \\[ x^5-3x^2+ax+b=g(x)\\cdot (x-1)(x-2)+(11x-10). \\] and plug in $x=1,2$.\n[/hide]" } { "Tag": [ "probability", "geometry", "3D geometry" ], "Problem": "A sequence $ a_1,a_2,a_3$ is formed by selecting $ a_1$ at random from $ [1,5]$, $ a_2$ at random from $ [11,15]$, $ a_3$ at random from $ [21,25]$.\r\n\r\nWhat is the probability that $ a_i$ is either an arithmetic or geometrical sequence?", "Solution_1": "[hide]For an arithmetic:\n\nPut $ a_1\\equal{}1\\plus{}x, a_2\\equal{}11\\plus{}y, a_3\\equal{}21\\plus{}z$ where $ x,y,z\\in[0,4]$. Then we must have\n\n$ a_1\\plus{}a_3\\equal{}2a_2\\iff x\\plus{}z\\equal{}2y$\n\nSo, within a cube defined by $ x,y,z\\in[0,4]$ the space of favorable outcomes is a subset of a [i]plane[/i] $ x\\minus{}2y\\plus{}z\\equal{}0$. However, the volume of a planar figure is zero, so is our probability.\n\nFor a geometric:\n\nPut $ a_1\\equal{}x, a_2\\equal{}11y, a_3\\equal{}21z$ where $ x\\in[1,5], y\\in[1,{15\\over 11}], z\\in[1,{25\\over 21}]$ to get\n\n$ a_1a_3\\equal{}a_2^2\\iff 21xz\\equal{}121y^2$\n\nand apply similar argument.\n\nTherefore, since the probability of both events is zero, the probability of their disjunction is also zero.[/hide]", "Solution_2": "Do you mean that $ a_1, a_2, a_3$ are integers? As Farenhajt describes, the problem is trivial otherwise.", "Solution_3": "Yes, trivialization. Sorry, I should have been more clear. :oops:" } { "Tag": [ "summer program", "PROMYS" ], "Problem": "This question was proposed by Belgium :) \r\n\r\nFind all numbers $x \\in \\mathbb{Z}$ for which the number \\[x^{4}+x^{3}+x^{2}+x+1\\] is a perfect square.\r\n\r\nEnjoy! :lol:", "Solution_1": "Well for positive x,\r\n\r\n$(x^{2})^{2}< x^{4}+x^{3}+x^{2}+x+1 < (x^{2}+1)^{2}$\r\n\r\nDo something similar for negative to get that the only answer is 0.", "Solution_2": "Sorry, that's not true...\r\n\r\n$x^{4}+x^{3}+x^{2}+x+1 < (x^{2}+1)^{2}$ implies that $x^{3}+x 3$. Equality on the left for $x = 3$. [/hide]\r\n\r\nThe cases for $x$ negative (edit:) is similar." } { "Tag": [ "geometry", "analytic geometry", "circumcircle", "Pythagorean Theorem" ], "Problem": "Three different points A, B, C lie on a circle [i]o[/i]. The tangent lines to [i]o[/i] at the points A and B intersect in the point P. The tangent line to o at C intersects the line AB in the point Q. \r\nProve that PQ^2 = PB^2 + QC^2.", "Solution_1": "I didnt understand can you give a figure please", "Solution_2": "here is a pic that i think it describes the geometry above\r\n\r\n[img]http://www.mathlinks.ro/Forum/album_page.php?pic_id=20[/img]", "Solution_3": "[quote=\"Palytoxin\"]here is a pic that i think it describes the geometry above\n\n[img]http://www.mathlinks.ro/Forum/album_page.php?pic_id=20[/img][/quote]\r\n\r\nwhich program did you use to make that drawing? also, do you know why those pics don't display...? mine don't display either x__x", "Solution_4": "[quote=\"Andreas\"]Three different points A, B, C lie on a circle [i]o[/i]. The tangent lines to [i]o[/i] at the points A and B intersect in the point P. The tangent line to o at C intersects the line AB in the point Q. \nProve that PQ^2 = PB^2 + QC^2.[/quote]\r\n\r\nI have solution using coordinate geometry\r\n\r\nLet the equation to the circle is $x^2 + y^2=r^2$\r\nP is $(x_1, y_1)$ and Q is $(x_2, y_2)$ \r\nFrom the given information, AB is the chord of contact of P w.r.t. the Circle.\r\nso, its equation is given by \r\n$xx_1+yy_1=r^2$. Since it passes through Q, we have \r\n\r\n$x_2x_1+y_2y_1=r^2$....(I)\r\n\r\n$PB^2$ and $QC^2$ are the lengths of tangents from P and Q respectively, hence \r\nare given by $x_1^2 + y_1^2-r^2$ and $x_2^2 + y_2^2-r^2$ respectively.\r\n\r\nSo, $PQ^2=(x_2-x_1)^2+(y_2-y_1)^2$ gives \r\n$x_1^2 + y_1^2+x_2^2 + y_2^2-2(x_2x_1+y_2y_1)$ \r\nUsing (I), we can write it as \r\n\r\n$x_1^2 + y_1^2-r^2+x_2^2 + y_2^2-r^2$ which is equal to $PB^2+QC^2$\r\n\r\nhence the proof.", "Solution_5": "Let O be the circumcenter of the triangle $\\triangle ABC$, the center of the circle $o \\equiv (O)$. The line OP cuts the chord AB at its midpoint F and $OP \\perp AB$. Using Pythagorean theorem for the right angle triangles $\\triangle PQF, \\triangle PBF$,\r\n\r\n$PQ^2 = PF^2 + QF^2,\\ \\ PB^2 = PF^2 + BF^2$\r\n\r\nSubtracting and using the fact that F is the midpoint of the chord AB, i.e., AF = BF,\r\n\r\n$PQ^2 - PB^2 = QF^2 - BF^2 = (QF + BF)(QF - BF) =$\r\n\r\n$= (QF + BF)(QF - AF) = QB \\cdot QA$\r\n\r\nThis is the power of the point Q to the circle $o \\equiv (O)$, hence,\r\n\r\n$PQ^2 - BF^2 = QB \\cdot QA = QC^2$\r\n\r\n$PQ^2 = PB^2 + QC^2$", "Solution_6": "[quote=\"mathclass\"][quote=\"Palytoxin\"]here is a pic that i think it describes the geometry above\n\n[img]http://www.mathlinks.ro/Forum/album_page.php?pic_id=20[/img][/quote]\n\nwhich program did you use to make that drawing? also, do you know why those pics don't display...? mine don't display either x__x[/quote]\r\nhaha i use cheap program MSPaint :blush: \r\ni will try to get Geometry Sketchpad\r\nand yea, Valentin said he will come back to fix it after IMO2005" } { "Tag": [ "USAMTS", "LaTeX", "AMC", "AIME", "email", "geometry", "Asymptote" ], "Problem": "I have a few questions:\r\n\r\n1) Will the answer to a problem ever be \"no solutions\"?\r\n\r\n2) If you wish to use LaTeX, do you have to own it?\r\n\r\n3) [quote=\"The person who made the USAMTS website\"]The USAMTS is one of the ways to enter the process of selecting the USA Mathematical Olympiad team, which participates in the International Mathematical Olympiad. Participants who score well after 3 rounds will be invited to participate in the American Invitational Mathematics Examination (AIME), the second step in the process of selecting the USA Mathematical Olympiad team.[/quote]\r\n\r\nHow well do you have to score to \"score well\" and make it to the AIME?\r\n\r\nThanks in advance! :)\r\n\r\nEDIT:\r\n\r\n4) One last one: If we plan to use LaTeX, can we send the solutions via mail? If so, we must use the USAMTS solutions form, correct?", "Solution_1": "1. Sometimes - For an example, see the last problem set from year 18, the answer to part of the first question was that it had no solution, and the same for others. Just make sure you prove that there are no solutions.\r\n\r\n2. No, it is a free download.\r\n\r\n3. Last year the cutoff was 68 points after the 3rd round, it might be different this year, but that should give a general suggestion to how well.\r\n\r\n4. I don't know.", "Solution_2": "3. It has indeed been 68 for the past two years.\r\n\r\n4. I believe you'd be allowed to if you print the LaTeX file that you would have e-mailed. Of course, if you're going to type up your solutions on the computer, you might as well send it by e-mail.", "Solution_3": "alright, thanks everyone!", "Solution_4": "[quote=\"buzzer11\"]1) Will the answer to a problem ever be \"no solutions\"? [/quote]\n[quote=\"monkeymatt\"]1. Sometimes - For an example, see the last problem set from year 18, the answer to part of the first question was that it had no solution, and the same for others. Just make sure you prove that there are no solutions.[/quote]\r\nmonkeymatt is correct. However, note that Problem 1/4/18 part (a) that he used as an example never said that there had to be a solution. It just asked the reader to find all solutions, and in that case, the set of all solutions was the empty set. And part (b) did have solutions. In contrast, if a problem asks a reader to find a particular solution, it will have a solution.\r\n\r\nThe goal of the USAMTS is to challenge the participants mathematically. Posting misleading phrasing would interfere in that goal, so we try to be as clear and honest as possible.\r\n\r\nErin Schram\r\nUSAMTS grader", "Solution_5": "Yes, I am a newbie. :) To USAMTS, at least.\r\n\r\nHow exactly does the email system work? I have downloaded all the files and put them in a single folder... then what do we do?", "Solution_6": "[quote=\"buzzer11\"]Yes, I am a newbie. :) To USAMTS, at least.\n\nHow exactly does the email system work? I have downloaded all the files and put them in a single folder... then what do we do?[/quote]\r\n\r\nAre those two separate questions?\r\n\r\nHow exactly does the email system work?\r\nEmail your .tex file, .pdf file, and any other pictures that you used in your solution to solutions@usamts.org.\r\n\r\n\r\nI have downloaded all the files and put them in a single folder... then what do we do?\r\nStart writing your solutions! In the solutions template, there are directions on where you should put your solutions. If you experience the \"I can't write on the file solutions.tex\" while you compile, you may need to close any existing pdf files.", "Solution_7": "Yes, those were two seperate questions, and thank you.\r\n\r\nDoes anyone have any idea how to reference points in the 18-gon when writing our solutions?", "Solution_8": "Include a diagram. Geometry solutions without diagrams make graders sad.", "Solution_9": "do they have the diagram as they grade? for instance, if we say, \"the point under X\", will they know which point it is?\r\n\r\nor i'll just include a diagram... :)", "Solution_10": "They might, but then they might not and take off. \r\n\r\nI have another question to add. I were to be working on the second part of a question, are all the results from the first part assumed?", "Solution_11": "i would think so. im sure they wont make you prove the first part all over again.\r\n\r\nto attach a diagram, is all you have to do make one and then attach it to the email? or do we have to reference it in some way in our solutions.tex file?", "Solution_12": "[quote=\"buzzer11\"]i would think so. im sure they wont make you prove the first part all over again.\n\nto attach a diagram, is all you have to do make one and then attach it to the email? or do we have to reference it in some way in our solutions.tex file?[/quote]\r\n\r\nWhat do you mean by \"reference it in some way\"?\r\n\r\nI recommend that you make your picture a .png file, and then use the command:\r\n\r\n\\includegraphics{file.png}\r\n\r\nif you saved it as file.png [b]in the same folder as you solutions[/b]. For more detail on how to put a picture in LaTeX, visit:\r\n[url]http://www.artofproblemsolving.com/Wiki/index.php/LaTeX:Pictures[/url]\r\n\r\n\r\nI don't like making separate pictures, though. I prefer asymptote.", "Solution_13": "how do we convert a file into a .png file? will the \\includegraphics{} command work for other types of files (.jpg, .gif, etc)? and do we have to attach this image to the email when we send in the solutions?", "Solution_14": "It will work for a .jpg:\r\n\r\n\\includegraphics[scale=.5]{Problem2Diagram3.jpg}\r\n\r\nAnd I believe you do have to attach all images you include. It says: http://www.usamts.org/Problems/U_ProblemsSoln.php to include all image files you use.", "Solution_15": "If you're emailing solutions, do you need to post your username or name on every page?", "Solution_16": "it does it for you... if you use the solutions.tex template.", "Solution_17": "[quote=\"Tenoreoz\"]If you're emailing solutions, do you need to post your username or name on every page?[/quote]Regardless of how you are submitting your solutions -- whether by mail, email, or fax -- you should have your name and ID# on every page.", "Solution_18": "hm i am using the solutions.tex template but it's still placing my information only on the first page. How do I fix this?", "Solution_19": "how do i convert a picture i made with asymptote into a .png?", "Solution_20": "[quote=\"le00327146\"]how do i convert a picture i made with asymptote into a .png?[/quote]\r\n\r\nWell, if you're going to use asymptote, you might as well do it in TeXnicCenter, instead of converting them into .png format (the advantage of this is that you don't need to make your pictures separately). If you would like to learn how this works, click below:\r\n\r\n[url]http://www.artofproblemsolving.com/Wiki/index.php/Asymptote:_Advanced_Configuration#Using_Asymptote_in_LaTeX[/url]\r\n\r\nBut, if you really want to make your pictures separately (the only thing that's nice with this is that LaTeX takes less time to compile):\r\n\r\n[url]http://www.artofproblemsolving.com/Wiki/index.php/Asymptote:_Advanced_Configuration#PDF_output_configuration[/url]\r\n\r\nand download ImageMagick." } { "Tag": [ "probability", "expected value" ], "Problem": "On the popular game show [i]The Price is Right[/i], in the showcase showdown, players spin a big wheel in turn. This wheel has all multiples of 5 from 5 to 100. There are 3 players, twice during the game, and each player gets to spin twice if they want. The object is to get the closest to 100, without going over. The two winners' scores are compared to see who will get to choose the showcase first. What is the expected value of this person's score?", "Solution_1": "[quote=\"tenniskidperson3\"]each player gets to spin twice if they want.[/quote]\r\n\r\nWhat's the probability of this? Does each player follow their optimal strategy?", "Solution_2": "I suppose they would, so they would spin any time they get under 47.5. :)", "Solution_3": "In order to talk about expected values we need to first fix their strategies. There is really only one type of strategy for this game, \"the $ n$ strategy\": choose some $ n$. If your first spin is less than $ n$, spin again. (You didn't state this, but I assume that if you spin again, we add the two values that you've spun, and if you get a sum larger than 100, we set it equal to 0. Also, that the second player has no knowledge of how the first player has done. Otherwise, the strategies depend rather much on the order of spins -- AABB will be quite different than ABAB from the point of view of player A.) So, we can then ask, suppose we have two players, one of whom follows the $ m$ strategy and one of whom follows the $ n$ strategy. What is the expected value of the score of the winning player? Is this along the lines of what you're asking?", "Solution_4": "Well, the order actually is AABBCC more games DDEEFF. And yes, the other players know how previous players have played. You are right in suspecting the 105-and-up rule, that it goes to 0.", "Solution_5": "[quote=\"tenniskidperson3\"]There are 3 players, twice during the game, and each player gets to spin twice if they want. [/quote]\r\nWhat does \"twice during the game\" mean? :huh:" } { "Tag": [], "Problem": "This is an open marathon very much like the 24 game marathon. \r\n\r\nWhat you do is post an equation with one total on one side and 4-10 numbers that add, subtract, multiply, or divide to the total, but you take out all the signs, and the solution will be the equation with signs like this: \r\n4 1 1 3 2 = 0\r\n\r\nSolution: 4+1+1-3x2=0 \r\n\r\nOrder of ops always applies. The answer must be with a new problem.\r\n\r\nI'll start easy, with 3 5 6 4 2 = 10", "Solution_1": "$ 3\\minus{}5\\plus{}6\\plus{}4\\plus{}2\\equal{}0$\r\n\r\n1 4 7 5 1 3 = 5", "Solution_2": "1 + 4 + 7 \u2013 5 + 1 \u2013 3 = 5\r\n\r\nNP: 2 5 4 3 5 1 = 9", "Solution_3": "$ 2\\plus{}5\\plus{}4\\plus{}3\\minus{}5\\times1\\equal{}9$\r\n\r\n9 9 7 9 = 9", "Solution_4": "$ (9\\div 9)^7*9\\equal{}9$\r\n4 2 4 2 4 2 4 2 = 69", "Solution_5": "$ 4 \\cdot 2 \\cdot 4 \\cdot 2 \\plus{} 4 \\plus{} 2 \\div 4 \\cdot 2 \\equal{} 69$, I think.\r\n\r\n1 2 3 4 5 6 = 1", "Solution_6": "$ 1\\plus{}2\\plus{}3\\minus{}(4\\minus{}5)\\minus{}6\\equal{}1$\r\n\r\n2 7 7 4 6 4 4 2=0", "Solution_7": "$ 2 \\minus{} 7 \\minus{} 7\\minus{} 4 \\plus{} 6 \\plus{} 4 \\plus{} 4 \\plus{} 2\\equal{}0$\r\n\r\n1 2 3 4 5 6 7 = 4", "Solution_8": "1^2+3+4-5-6+7=4\r\n\r\n1 3 3 7 4 2 = 69", "Solution_9": "1*3*3*7+4+2=69\r\n\r\n8 2 5 10 3 5 1 2 = 48 \r\nha!", "Solution_10": "[quote=\"Bugi\"]1*3*3*7+4+2=69\n\n8 2 5 10 3 5 1 2 = 48 \nha![/quote]\r\n\r\n8*2+5+10+3*5-1+2=48\r\n\r\neasy one\r\n1 2 3 4 5 = 15", "Solution_11": "1+2+3+4+5=15\r\n9 6 1 5 1=10", "Solution_12": "$ 9\\minus{}6\\plus{}1\\plus{}5\\plus{}1\\equal{}10$\r\n\r\n5 5 3 2 6 =4", "Solution_13": "Are parentheses allowed? If so, $ (5\\plus{}5) \\cdot (3\\minus{}2) \\minus{}6\\equal{}4$.\r\n\r\n3 6 4 4 8 5 = 10", "Solution_14": "$ 3\\minus{}6\\plus{}4\\minus{}4\\plus{}8\\plus{}5 \\equal{} 10$\r\n\r\n3 1 4 1 5 9 2 6 = 9", "Solution_15": "$ 3\\plus{}1\\plus{}4\\plus{}1\\plus{}5\\minus{}9\\minus{}2\\plus{}6\\equal{}9$\r\n\r\n\r\n6 5 4 2 = 14400", "Solution_16": "(6*5*4)^2=14400\r\n\r\nMy problem wasn't solved, you people got 47\r\n\r\n8 2 5 10 3 5 1 2=48", "Solution_17": "$ 8^2\\minus{}5\\minus{}10\\plus{}3\\minus{}5\\minus{}1\\plus{}2\\equal{}48$\r\n\r\n\r\n5 3 3 10 2=157", "Solution_18": "$ 5^3\\plus{}3*10\\plus{}2\\equal{}157$\r\n\r\n2 6 3 6 2 = 512", "Solution_19": "$ 2^{[(6 \\minus{} 3)\\times6]/2}\\ \\equal{} \\ 512$\r\n\r\n3 6 100 75 50 25 ~ 952", "Solution_20": "[quote=\"ValentineA\"]3 6 100 75 50 25 ~ 952[/quote]\r\nNobody?\r\n\r\n[hide][youtube]6mCgiaAFCu8[/youtube]\n\nSo it\u2019s $ [3\\times(6 \\plus{} 100)\\times75 \\minus{} 50]/25$ \u2013 and the numbers I myself gave are in their correct order. Hopefully I\u2019m playng by the rules of the game.[/hide]" } { "Tag": [ "inequalities", "LaTeX", "calculus", "integration", "inequalities open" ], "Problem": "Can anyone show me a proof of the inequlaity:\r\n\r\n0 \r\n\r\n[code]\n\\displaystyle{ {(\\sum_{i=1}^m (\\sum_{j=1}^n ({A_{i,j}}^s)^{r/s})}^s\n \\leq {(\\sum_{j=1}^n (\\sum_{i=1}^m ({A_{i,j}}^r)^{s/r})}^r }[/code]\r\nWhat would be the condition for equality?", "Solution_1": "and i thought my latex skills were bad... :P", "Solution_2": "Theorem \r\n\r\n0 M_s^n ( M_r^m A_{i,j} ) \\leq M_r^m (M_s^n A_{i,j} )\r\n\r\nwhere M_r^m {x_k} = (\\sum_{i=1}^m (x_i)^r)^{1/r) \r\n\r\nIn particulr when s= -\\infinity , r= +\\infinity we have the Minimax Inequality \r\n\r\nThe proof is already done by myself! :lol:", "Solution_3": "Plus: Thanks for someone remind the inequality follows from Minkowski Inequality :roll:", "Solution_4": "Note that the inequality can also be written into the form\r\n\r\n${01$. Prove that \\[\\frac{1}{2}+\\cdots+\\frac{1}{n}\\] is not an integer.", "Solution_1": "[quote=\"nicetry007\"]Let $ 2^{k}\\;\\leq \\; n \\;<\\; 2^{k+1}$. \n\nIt is to be noted that every number $ m \\;\\neq\\; 2^{k}$ and $ m \\;\\leq\\;n$ has a power of $ 2$ strictly smaller than $ k$. \n\nSuppose not. Let $ m \\; = \\; 2^{l}\\cdot s$ where $ l \\;\\geq \\;k$ and $ s$ is odd and $ s \\;\\geq\\; 1$.\n\nWe have $ m \\; = \\; 2^{l}\\cdot s \\;\\leq \\;n\\;< \\;2^{k+1}\\;\\Rightarrow s \\;< \\;2^{k+1-l}\\;\\leq \\; 2\\;$ ( as $ l \\;\\geq\\; k$) $ \\Rightarrow s = 1$ (as $ s$ is odd ) $ \\Rightarrow m \\; = \\; 2^{l}\\;<\\; 2^{k+1}\\;\\Rightarrow \\; l \\;\\leq \\; k$.\n\nBut $ l\\;\\geq \\;k$. Hence, $ l \\; = \\; k \\;\\Rightarrow \\; m \\; = \\; 2^{k}$, which is a contradiction as $ m \\;\\neq\\; 2^{k}$.\n\nThus, we can conclude that every $ m \\; \\neq \\; 2^{k}$ and $ m \\;\\leq \\; n$ has a power of $ 2$ strictly smaller than $ k$.\n\nThe denominator of the sum is the $ lcm(2\\;,\\;3\\;,\\;\\cdots\\;,\\;n)$ which has $ 2^{k}$ as a factor. Thus, when we take the sum of all the unit fractions, the numerator of every fraction other than $ \\frac{1}{2^{k}}$ \ngets multiplied by a power of $ 2$ and the numerator of $ \\frac{1}{2^{k}}$ gets multiplied by an odd number. Hence, the numerator of the sum of all the fractions is an odd number as $ odd+even \\; = \\; odd$.\n\nTherefore, the sum can never be an integer as the numerator is odd and the denominator is even.[/quote]", "Solution_2": "Sorry to revive this but I remember there was a proof using Bertrand's postulate and \nfactorials , but I can't remember the whole thing .Can anyone post it?", "Solution_3": "[quote=\"aham\"]Sorry to revive this but I remember there was a proof using Bertrand's postulate and \nfactorials , but I can't remember the whole thing .Can anyone post it?[/quote]\n\nConsider \n\n$\\frac{1}{2}+\\frac{1}{3}+\\cdots +\\frac{1}{n}=\\frac{\\frac{n!}{1}+\\frac{n!}{2}+\\cdots+\\frac{n!}{n}}{n!}$\n\nBy Bertrand's postulate, there exists a prime $p$ such that $\\lfloor{\\frac{n}{2}}\\rfloor n$\n\nOr in other words, $p\\nmid \\frac{n!}{p}$\n\nNow Note that $p|\\frac{n!}{k} \\forall k \\in \\mathbb{N}, 1\\le k \\le n, k\\ne p$\n\nSo, ${p\\nmid \\frac{n!}{1}+\\frac{n!}{2}+\\cdots+\\frac{n!}{n}}$ but $p|n!$ \n\n$\\Longrightarrow \\frac{\\frac{n!}{1}+\\frac{n!}{2}+\\cdots+\\frac{n!}{n}}{n!}$ is never an integer.", "Solution_4": "[quote=\"gouthamphilomath\"]By Bertrand's postulate, there exists a prime $p$ such that $\\left\\lfloor{\\frac{n}{2}}\\right\\rfloor 0 and q>0 because the minor arc MA is in the first quadrant. If T(u,v) then up+vq=0 because MX is perpendicular to MT.\r\nAfter a lot of computations one get\r\ntan(MTB-CTM) = a/h, which in fact does not depends on X(p,q).", "Solution_2": "Denote E be the midpoint of BT, then the quadrilateral XMET inscribed the circle whose diameter is XT. Since ME || CT, \r\n$ \\angle MTB \\minus{} \\angle MTC \\equal{} \\angle MTE \\minus{} \\angle EMT \\equal{} \\angle MXE \\minus{} \\angle EXT \\equal{} \\angle MXE \\minus{} \\angle EXB \\equal{} \\angle MXB \\equal{} \\angle MAB$", "Solution_3": "Let the perpendiculers from $ B$ and $ C$ to $ XM$ and $ TM$ respectively intersect at $ P$.\r\n$ \\angle BPC \\equal{} 90^0\\implies BM \\equal{} CM \\equal{} PM$. So $ P$ is the reflection of $ C$ at $ TM$ and of $ B$ at $ XM$.\r\n$ XT \\equal{} XB \\equal{} XP\\implies$ $ X$ is the circumcenter of $ TBP$. \r\n$ \\angle MTB \\minus{} \\angle CTM \\equal{} \\angle MTB \\minus{} \\angle PTM \\equal{} \\angle BTP \\equal{} \\frac {1}{2}\\angle BXP \\equal{}$\r\n$ \\equal{} \\angle BXM \\equal{} \\angle BAM\\ \\ \\text{(Const.)}$", "Solution_4": "\u039dotice here that the condition AB=AC is absolutely redundant.... Hence the problem can be generalized to any triangle ABC,using the same method of proof as mr.danh's idea,which is also the same as mine.\n\nCheers,\nNick", "Solution_5": "Sorry for the revival; why is XMET cyclic in mr.danh's solution?\n\n[quote=\"mr.danh\"]then the quadrilateral XMET inscribed the circle whose diameter is XT[/quote]", "Solution_6": "[quote=\"AwesomeToad\"]Sorry for the revival; why is XMET cyclic in mr.danh's solution?\n[/quote]\n\n$TX=BX$\nso, $\\triangle TXB$ is isosceles\nsince $E$ is the midpoint\n$XE$ is the median as well as altitude of the triangle\nSo, $\\angle TEX=90^\\circ$\nit is given $\\angle TMX=90^\\circ$\nSo, $TEMX$ is cyclic with $TX$ being the diameter", "Solution_7": "[hide=\"Solution\"]\nLet $N$ be the midpoint of $BT$. Since $M$ is the midpoint of $BC$, we have $MN||CT$. Therefore, $\\angle{CTM}=\\angle{TMN}$. \n\nSince $TX=BX$, $\\triangle{BXT}$ is isosceles, so $\\angle{TXN}=\\angle{BXN}$ and $XN\\perp BT$. Then $\\angle{TNX}=90=\\angle{TMX}$, so $XMNT$ is cyclic. Thus, $\\angle{MTB}=\\angle{MXN}$ and $\\angle{CTM}=\\angle{TMN}=\\angle{TXN}=\\angle{BXN}$. Finally, observe that from cyclic quadrilateral $ABMX$, \n\\[ \\angle{BAM}=\\angle{BXM}=\\angle{MXN}-\\angle{BXN}=\\angle{MTB}-\\angle{CTM}.\\]\nBecause $\\angle{BAM}$ is independent of $X$, $\\angle{MTB}-\\angle{CTM}$ is also not dependent on the choice of $X$. // [/hide]", "Solution_8": "Let $C' $ be the symetrical of $C$ wrt $TM$ and $L=AM\\cap TC'$ and $Y$ the reflexion of $X$ wrt the midpoint of $AB$. and $E$ the midpoint of $TB$\nthen $ \\angle{MTB}-\\angle{CTM} = \\angle LTB$\nit is enough to show that $ATLB$ is cyclic to conclude that $\\angle{MTB}-\\angle{CTM} = \\angle MAB$\nwe have: $\\angle ALB=\\frac{\\pi}{2}-\\angle TMB - \\angle MTC = \\frac{\\pi }{2}- \\angle AYX -\\angle TME = \\frac{\\pi}{2}-\\angle AYX -\\frac{\\angle TXB}{2}=\\angle XBT -\\angle ABX =\\angle ABT $\nHence $ATBL$ is cyclic which leads to \n\\[ \\angle{MTB}-\\angle{CTM}= \\angle LTB = \\angle MAB \\]", "Solution_9": "We will prove the result for an arbitrary $\\triangle ABC.$ Define $\\Gamma \\equiv \\odot(X, XB)$ and $\\omega \\equiv \\odot(ABM).$ Let $S$ be the reflection of $T$ in $M$ and let $MT$ meet $\\omega$ for a second time at $Y.$ \n[asy]\n /* Geogebra to Asymptote conversion, documentation at artofproblemsolving.com/Wiki, go to User:Azjps/geogebra */\nimport graph; size(15cm); \nreal labelscalefactor = 0.5; /* changes label-to-point distance */\npen dps = linewidth(0.7) + fontsize(10); defaultpen(dps); /* default pen style */ \npen dotstyle = black; /* point style */ \nreal xmin = 937.3756443093395, xmax = 1122.3822298074242, ymin = 958.746170568615, ymax = 1051.0863181805378; /* image dimensions */\npen qqwuqq = rgb(0.,0.39215686274509803,0.); \n\ndraw(arc((981.6398411686986,1019.9779690940252),4.894354113705946,-41.70511190447346,-22.045779161991184)--(981.6398411686986,1019.9779690940252)--cycle, qqwuqq); \ndraw(arc((1036.6505840120194,970.956380225791),4.894354113705946,138.29488809552655,157.95422083800884)--(1036.6505840120194,970.956380225791)--cycle, qqwuqq); \ndraw(arc((976.8060711531035,995.1907717416409),4.894354113705946,78.96519747630964,98.62453021878137)--(976.8060711531035,995.1907717416409)--cycle, qqwuqq); \nLabel laxis; laxis.p = fontsize(10); \nxaxis(xmin, xmax, Ticks(laxis, Step = 10., Size = 2, NoZero),EndArrow(6), above = true); \nyaxis(ymin, ymax, Ticks(laxis, Step = 10., Size = 2, NoZero),EndArrow(6), above = true); /* draws axes; NoZero hides '0' label */ \n /* draw figures */\ndraw((976.8060711531035,995.1907717416409)--(1041.4843540276145,995.7435775781753)); \ndraw(circle((992.8090020683957,1014.8258301910363), 25.330403040433268), linetype(\"2 2\") + blue); \ndraw(circle((1013.9145017344058,1000.8191459995926), 37.532841845441226), linetype(\"2 2\") + blue); \ndraw((1036.6505840120194,970.956380225791)--(971.7035024023855,1028.83251438248)); \ndraw((976.8060711531035,995.1907717416409)--(981.6398411686986,1019.9779690940252), red); \ndraw((1041.4843540276145,995.7435775781753)--(981.6398411686986,1019.9779690940252), red); \ndraw((976.8060711531035,995.1907717416409)--(1036.6505840120194,970.956380225791), red); \ndraw((1036.6505840120194,970.956380225791)--(1041.4843540276145,995.7435775781753), red); \ndraw((971.7035024023855,1028.83251438248)--(976.8060711531035,995.1907717416409)); \n /* dots and labels */\ndot((990.,1040.),linewidth(3.pt) + dotstyle); \nlabel(\"$A$\", (990.7241041487343,1041.9713196788784), NW * labelscalefactor); \ndot((976.8060711531035,995.1907717416409),linewidth(3.pt) + dotstyle); \nlabel(\"$B$\", (974.5727355735047,993.1697945016816), SW * labelscalefactor); \ndot((1041.4843540276145,995.7435775781753),linewidth(3.pt) + dotstyle); \nlabel(\"$C$\", (1042.4411126168939,993.1697945016816), SE * labelscalefactor); \ndot((1009.145212590359,995.4671746599081),linewidth(3.pt) + dotstyle); \nlabel(\"$M$\", (1008.5069240951992,992.1909236789404), S * labelscalefactor); \nlabel(\"$\\omega$\", (960,1010), NE * labelscalefactor,blue); \ndot((1013.9145017344058,1000.8191459995926),linewidth(3.pt) + dotstyle); \nlabel(\"$X$\", (1015.5221649915111,1000.1850353979937), E * labelscalefactor); \ndot((971.7035024023855,1028.83251438248),linewidth(3.pt) + dotstyle); \nlabel(\"$Y$\", (967.3520911855517,1029.8774503544778), NE * labelscalefactor); \nlabel(\"$\\Gamma$\", (1042,1032), NE * labelscalefactor,blue); \ndot((981.6398411686986,1019.9779690940252),linewidth(3.pt) + dotstyle); \nlabel(\"$T$\", (981.6513640015929,1022.2730164414478), N * labelscalefactor); \ndot((1036.6505840120194,970.956380225791),linewidth(3.pt) + dotstyle); \nlabel(\"$S$\", (1037.7099036403113,967.3928628361625), SE * labelscalefactor); \nclip((xmin,ymin)--(xmin,ymax)--(xmax,ymax)--(xmax,ymin)--cycle); \n /* end of picture */\n[/asy]\nNote that $BTCS$ is a parallelogram because its diagonals bisect one another. Hence, $\\measuredangle CTM = \\measuredangle BST.$ Meanwhile, $S \\in \\Gamma$ because $XM$ is the perpendicular bisector of $\\overline{ST}.$ Then since $\\measuredangle YBX = \\measuredangle YMX = 90^{\\circ}$, it follows that $YB$ is tangent to $\\Gamma.$ Thus, the Alternate Segment Theorem yields $\\measuredangle BST = \\measuredangle YBT.$ Hence,\n\\begin{align*}\n\\measuredangle MTB - \\measuredangle CTM = \\measuredangle MTB - \\measuredangle YBT = \\measuredangle BYT.\n\\end{align*}\nBut clearly $\\measuredangle BYT = \\measuredangle BYM = \\measuredangle BAM$ is fixed, as desired. $\\square$", "Solution_10": "Here is an analytic solution:\n\nWe use complex numbers and set $(ABM)$ to be the unit circle. Furthermore, let $a=-1,b=1$. Then it is clear that $c=2m-1$. Now since $TM \\perp XM$, we have \\[\\frac{t-m}{x-m}=-\\frac{\\bar t - \\frac{1}{m}}{\\frac{1}{x}-\\frac{1}{m}} \\implies t-m=mx \\bar t - x \\implies \\bar t = \\frac{t-m+x}{mx}\\] Now since $XT=XB$, we have \\[(t-x)(\\bar t - \\tfrac{1}{x})=(1-x)(1-\\tfrac{1}{x}) \\implies t \\bar t - \\frac{t}{x}- \\bar t x + x +\\frac{1}{x} -1=0\\] Plugging in our first equation into our second, we get \\[t\\left(\\frac{t-m+x}{mx}\\right)-\\frac{t}{x}-\\left(\\frac{t-m+x}{m}\\right) + x + \\frac{1}{x} -1=0 \\implies t^2 - 2mt + (m-1)x^2+m=0\\] Now let $r$ be the root that lies within angle $AMB$ and call the other root $s$. It is clear that these roots are reflections about $M$ (and thus $r,m,s$ are collinear), so now consider the following complex number which has argument $\\angle MTB - \\angle CTM$ \\[\\frac{\\frac{s-r}{1-r}}{\\frac{2m-1-r}{s-r}}=-\\frac{(s-r)^2}{(1-r)(1-s)}=\\frac{4m^2-4((m-1)x^2+m)}\n{(m-1)(1-x^2)}=\\frac{4(m-x^2)}{1-x^2}\\] Now notice that since $|x|=1$, $x^2$ lies on the unit circle, and so we see that the argument of this complex number is $\\angle MXB=\\angle MAB$ which is independent of $X$, as desired. $\\Box$", "Solution_11": "We do not use complex numbers\n\n[hide=Solution]\nLet $N$ be the midpoint of $BT$. Since $XB=XT$, $\\angle XNT=90^{\\circ}=\\angle XMT$, so $XMNT$ is cyclic, yielding $\\angle BTM=\\angle NTM=\\angle NXM$.\nNow $MN$ is a midline in $\\triangle BCT$, so \n\\[ \\angle MTC=\\angle BTC - \\angle MTC=\\angle BNM-\\angle MXN=\\angle MXT-\\angle MXN=\\angle NXT=\\angle NXB \\]\nHence, $\\angle BTM-\\angle CTM=\\angle BXN - \\angle MXN=\\angle BXM=\\angle BAM$ which is fixed.\n[/hide]\n\nWhen I started working on this problem, I misread it as $TB=TX$. If we characterize $T$ as such, can we find any interesting properties of this new configuration? All my progress so far for this configuration is rather trivial.", "Solution_12": "[hide=Solution]\nLet $N$ be the midpoint of $BT$. Then $MN \\parallel CT$. Also, $XN \\perp NT$, so $XMNT$ is cyclic. Then \\[\\begin{aligned} \\angle MTB - \\angle CTM &= \\angle MTB - (\\angle CTB - \\angle MTB) \\\\ &= 2\\angle MTB - \\angle CTB \\\\ &= 2\\angle MTN - \\angle MNB \\\\ &= 2\\angle MXN - (\\pi - \\angle MNT) \\\\ &= 2\\angle MXN - \\angle MXT \\\\ &= 2\\angle MXN - (\\angle MXN + \\angle NXT) \\\\ &= \\angle MXN - \\angle NXT \\\\ &= (\\angle MXB + \\angle BXN) - \\angle NXT \\\\ &= (\\angle MXB + \\angle NXT) - \\angle NXT \\\\ &= \\angle MXB \\\\ &= \\angle MAB \\end{aligned}\\] which does not depend on $X$.\n[/hide]", "Solution_13": "My solution:(similar MStang and mr.danh)\nLet $Q$ be the midpoint of $BT,$ then $\\angle XQT =90,$ also $\\angle XMT=90\\to XMQT$ is cyclic.\nAlso $MQ$ is the midline of $\\triangle CBT\\to MQ\\parallel CT.$\nThen $\\angle MTB-\\angle CTM=\\angle MXQ-\\angle QMT=\\angle MXQ-\\angle QXT=\\angle MXQ-\\angle BXQ=\\angle MXB=\\angle MAB.$ As desired.", "Solution_14": "I generally don't like geometry, but the locus of $T$ as $X$ varies on $\\omega_{ABM}$ is (showed in black) a cute shape, which surprisingly is independent of $\\angle BAM$: \n\n[asy] /* Geogebra to Asymptote conversion, documentation at artofproblemsolving.com/Wiki, go to User:Azjps/geogebra */\nimport graph; size(27.1cm); \nreal labelscalefactor = 0.5; /* changes label-to-point distance */\npen dps = linewidth(0.7) + fontsize(10); defaultpen(dps); /* default pen style */ \npen dotstyle = black; /* point style */ \nreal xmin = -4.3, xmax = 22.8, ymin = -5.76, ymax = 6.3; /* image dimensions */\npen ffxfqq = rgb(1.,0.4980392156862745,0.); \n /* draw figures */\ndraw(circle((6.38,1.), 4.011234224026315), linewidth(2.) + blue); \ndraw((9.02,4.02)--(3.74,-2.02), linewidth(2.) + green); \ndraw((3.74,-2.02)--(14.408602941635356,-1.9233120679801736), linewidth(2.) + green); \ndraw((9.02,4.02)--(9.074301470817678,-1.9716560339900866), linewidth(2.) + green); \ndraw((xmin, -0.5299145299145296*xmin + 4.380854700854699)--(xmax, -0.5299145299145296*xmax + 4.380854700854699), linewidth(2.) + linetype(\"2 2\") + ffxfqq); /* line */\ndraw(circle((9.924343193909214,-0.8781989574561639), 6.288863996049319), linewidth(2.) + linetype(\"2 2\") + blue); /* locus construction */\n\ndraw((3.666532459319288,1.7158227254556473)^^(5.685493539125569,0.9999999930549354)--(5.6854935026018385,0.9999999861098701)--(5.685493429554378,0.9999999722197372)--(5.685493283459452,0.9999999444394625)--(5.685492991269605,0.9999998888788786)--(5.685492406889907,0.9999997777575703)--(5.685491238130497,0.9999995555143926)--(5.685488900611638,0.9999991110257929)--(5.685484225573751,0.9999982220396174)--(5.685474875497324,0.9999964440313623)--(5.685456175341818,0.9999928878712353)--(5.685418775020227,0.9999857749765105)--(5.685343974334768,0.9999715468891729)--(5.685194372795103,0.9999430815228921)--(5.6848951690436405,0.9998861140234769)--(5.684296758875018,0.999772031953789)--(5.683099928057738,0.9995432795034654)--(5.680706225965126,0.9990834211396069)--(5.675918671647202,0.9981542888147239)--(5.66634305611945,0.9962583479987558)--(5.647190547285527,0.9923156543167926)--(5.608886427386223,0.9838262051167292)--(5.532330009850903,0.9644252800129192)--(5.45593496758696,0.9417860264693532)--(5.379812586571335,0.9159008409444593)--(5.304074679375265,0.8867657406276426)--(5.228833469397423,0.8543803858449063)--(5.154201474284849,0.8187480987858757)--(5.080291388658231,0.7798758785249891)--(5.007215966259028,0.7377744123133211)--(4.935087901636658,0.6924580831212156)--(4.8640197114945805,0.6439449734156475)--(4.794123615814696,0.5922568651599956)--(4.725511418879817,0.5374192360276844)--(4.658294390314279,0.4794612518249427)--(4.592583146262918,0.41841575512172113)--(4.528487530828694,0.3543192500936243)--(4.466116497889135,0.2872118835815003)--(4.405577993411586,0.21713742237914313)--(4.346978838386987,0.14414322676336178)--(4.290424612501343,0.06828022028443304)--(4.236019538663634,-0.010397144161248928)--(4.183866368508116,-0.09183092195079112)--(4.134066268988286,-0.17595971801737553)--(4.086718710178719,-0.2627187317628561)--(4.041921354400151,-0.35203980556364733)--(3.999769946781858,-0.4438514768296642)--(3.9603582073742345,-0.5380790335723598)--(3.9237777249230215,-0.6346445734344237)--(3.8901178524153184,-0.7334670661300633)--(3.874410431075394,-0.783698508657428)--(3.8594656045055804,-0.8344624192415226)--(3.845293867670346,-0.8857478192080364)--(3.8319055569285347,-0.9375435473137967)--(3.8193108470060984,-0.9898382620819577)--(3.8075197480037692,-1.042620444186364)--(3.7965421024407617,-1.0958783988843006)--(3.786387582334997,-1.1496002584973328)--(3.777065686320849,-1.2037739849394553)--(3.768585736804999,-1.258387372292142)--(3.760956877161276,-1.3134280494255997)--(3.7541880689650657,-1.3688834826657588)--(3.7482880892683106,-1.4247409785061471)--(3.7432655279153932,-1.480987686364395)--(3.7391287849011388,-1.5376106013822852)--(3.735886067771019,-1.5945965672692508)--(3.733545389064971,-1.6519322791880227)--(3.732114563804804,-1.709604286682412)--(3.7316012070264213,-1.767598996646019)--(3.7320127313571203,-1.825902676331583)--(3.7333563446389872,-1.884501456399909)--(3.7356390475986863,-1.9433813340079813)--(3.7817729391770887,-2.0169625784216554)--(3.8406614889536357,-2.011884343848216)--(3.8991536309809525,-2.0059079212844435)--(3.9572363309402707,-1.9990420087628293)--(4.014896712864063,-1.9912954933108074)--(4.072122061911019,-1.9826774482532565)--(4.128899827096567,-1.9731971304735236)--(4.185217623978182,-1.9628639776339127)--(4.2410632372949255,-1.9516876053562344)--(4.296424623560574,-1.9396778043632736)--(4.351289913609769,-1.926844537581855)--(4.40564741509662,-1.913197937208281)--(4.459485614945187,-1.8987483017369464)--(4.512793181751267,-1.8835060929528697)--(4.565558968135011,-1.8674819328889105)--(4.617772013043776,-1.8506866007485507)--(4.66942154400475,-1.8331310297949206)--(4.7204969793267955,-1.8148263042070127)--(4.770987930251089,-1.7957836559037932)--(4.870175801073993,-1.755530238254017)--(4.96690598349996,-1.7124634643729049)--(5.061102520338164,-1.6666781772839392)--(5.152692729714776,-1.6182712687308367)--(5.241607261134152,-1.567341570728622)--(5.327780148019183,-1.5139897455534361)--(5.411148856686603,-1.458318174284143)--(5.491654331716694,-1.4004308440101652)--(5.569241037680472,-1.3404332338210294)--(5.643856997191119,-1.2784321996941987)--(5.715453825250149,-1.21453585839865)--(5.783986759862463,-1.1488534705325133)--(5.849414688898338,-1.0814953228136701)--(5.911700173184018,-1.01257260974289)--(5.970809465806485,-0.9421973147594456)--(6.026712527621715,-0.8704820910094742)--(6.079383038959575,-0.7975401418476298)--(6.128798407522286,-0.7234851011925337)--(6.1749397724772415,-0.6484309138566487)--(6.217792004748711,-0.5724917159708548)--(6.257343703516804,-0.4957817156239312)--(6.29358718893584,-0.41841507383657733)--(6.326518491088016,-0.3405057859892265)--(6.356137335192053,-0.26216756382215917)--(6.382447123090203,-0.18351371812575248)--(6.405454911040664,-0.1046570422378223)--(6.425171383846217,-0.025709696463980317)--(6.441610825353404,0.05321690646413829)--(6.45479108536028,0.1320122147790795)--(6.464733542974214,0.21056664980396)--(6.471463066464841,0.28877171670411605)--(6.475007969660631,0.366520113754667)--(6.475399964940974,0.44370584001635704)--(6.472674112879078,0.5202243013137076)--(6.46686876859422,0.5959724144114407)--(6.45802552487513,0.6708487092870767)--(6.446189152139483,0.7447534293997062)--(6.43140753529755,0.8175886298570284)--(6.4137316075910915,0.8892582733851142)--(6.393215281481533,0.9596683240076236)--(6.369915376664348,1.028726838343676)--(6.343891545289346,1.0963440544361516)--(6.315206194469325,1.1624324780247315)--(6.283924406162099,1.2269069661808207)--(6.250113854513555,1.2896848082241383)--(6.213844720751744,1.3506858038437304)--(6.1751896057244515,1.4098323383490716)--(6.134223440174887,1.4670494549799202)--(6.091023392852363,1.5222649242066486)--(6.045668776556834,1.5754093099560182)--(5.998240952218204,1.6264160327004318)--(5.948823231113102,1.6752214293520855)--(5.897500775323634,1.7217648099067209)--(5.84436049654423,1.7659885107850446)--(5.789490953344301,1.8078379448233224)--(5.732982246995787,1.8472616478681394)--(5.6749259159760195,1.884211321933829)--(5.615414829257601,1.9186418748845964)--(5.554543078497945,1.9505114566070063)--(5.492405869242279,1.979781491642084)--(5.429099411254618,2.006416708249943)--(5.364720808092082,2.0303851638835315)--(5.2993679460385,2.051658267051775)--(5.233139382513753,2.070210795556103)--(5.166134234075729,2.0860209110881094)--(5.098452064132063,2.0990701701797354)--(5.0301927704789,2.109343531501242)--(4.961456472784042,2.1168293595058554)--(4.892343400131779,2.1215194244237745)--(4.822953778746377,2.1234088986119635)--(4.753387720011027,2.1224963492698428)--(4.683745108898547,2.1187837275347285)--(4.614125492929577,2.1122763539745906)--(4.5446279717733296,2.10298290049932)--(4.475351087605282,2.090915368715359)--(4.406392716335074,2.0760890647522405)--(4.337849959817122,2.0585225705930683)--(4.269819039155097,2.0382377119446)--(4.2023951892103675,2.0152595226860868)--(4.1356725544229915,1.9896162059395197)--(4.069744086052536,1.961339091807299)--(4.0047014409442525,1.9304625918268503)--(3.9406348819246984,1.89702415019487)--(3.8776331799288593,1.861064191817328)--(3.815783517959146,1.8226260672444372)--(3.7551713969745366,1.7817559945530228)--(3.6958805438061364,1.7385029982417912)--(3.637992821193168,1.692918845208002)--(3.5815881400311973,1.6450579778770307)--(3.5267443739220026,1.5949774445591358)--(3.473537276112087,1.5427368271105628)--(3.422040398904174,1.4883981659788486)--(3.372325015623585,1.4320258827147827)--(3.324460045218516,1.3736867000360564)--(3.2785119795705544,1.3134495595300808)--(3.2345448135888564,1.2513855370858298)--(3.192619978158516,1.1875677561468216)--(3.152796276010604,1.1220712988795545)--(3.115129820578321,1.0549731153537794)--(3.0796739779005318,0.9863519308329873)--(3.046479311630807,0.9162881512753641)--(3.0155935312067754,0.8448637671472466)--(2.9870614432313367,0.7721622556527938)--(2.9609249061138847,0.6982684814851486)--(2.937222788016291,0.6232685962058302)--(2.915990928144988,0.5472499363604465)--(2.897262101426889,0.4703009204400513)--(2.8810659866035255,0.39251094479861454)--(2.867429137773995,0.313970278638062)--(2.8563749594139387,0.23476995817327012)--(2.8479236848940026,0.1550016800901659)--(2.84209235851762,0.07475769441076185)--(2.83889482109435,-0.005869303120494607)--(2.838341699061271,-0.08678627901423397)--(2.8404403971612093,-0.1678999702624202)--(2.845195094683023,-0.24911699269799303)--(2.852606745265287,-0.33034394937248246)--(2.862673080261134,-0.4114875388362289)--(2.8753886156582973,-0.4924546632059379)--(2.8907446625446607,-0.5731525359046548)--(2.908729341106011,-0.6534887889595333)--(2.92932759813894,-0.7333715797433715)--(2.9525212280582744,-0.8127096970464502)--(2.9782888973747674,-0.8914126663659339)--(3.006606172615117,-0.9693908543010348)--(3.037445551652917,-1.0465555719429918)--(3.0707764984155395,-1.1228191771500586)--(3.1065654809284435,-1.198095175598918)--(3.144776012654988,-1.272298320505147)--(3.1853686970863646,-1.345344710906856)--(3.228301275532933,-1.4171518884071133)--(3.273528678064902,-1.4876389322723633)--(3.321003077547001,-1.5567265527858622)--(3.370673946708675,-1.6243371827568658)--(3.4224881181880713,-1.6903950670883643)--(3.4763898474850774,-1.7548263503081132)--(3.532320878755609,-1.8175591619698683)--(3.5902205133764484,-1.8785236998339527)--(3.6500256812069916,-1.9376523107386412)--(3.7116710144714986,-1.9948795690761667)--(3.775088924182728,-2.0501423527897904)--(3.840209679025163,-2.103379916810797)--(3.906961486613503,-2.154533963857111)--(3.9752705770396304,-2.203548712517821)--(4.045061288618811,-2.2503709625508614)--(4.1162561557437085,-2.2949501573239077)--(4.188775998752504,-2.3372384433315023)--(4.262540015715336,-2.3771907267246117)--(4.3374658760412705,-2.414764726791693)--(4.41346981580614,-2.4499210263337146)--(4.490466734699709,-2.4826231188787085)--(4.568370294489033,-2.5128374526847153)--(4.647093018893145,-2.5405334714833425)--(4.726546394762882,-2.565683651919534)--(4.806640974458132,-2.5882635376465752)--(4.887286479313617,-2.608251770038825)--(4.968391904083121,-2.6256301154881836)--(5.049865622251042,-2.6403834892538685)--(5.131615492099235,-2.652499975838608)--(5.213548963416232,-2.661970845867995)--(5.2955731847353205,-2.668790569453413)--(5.377595110987289,-2.6729568260224825)--(5.459521611453228,-2.6744705106047943)--(5.541259577902418,-2.673335736564246)--(5.622716032800073,-2.669559834773075)--(5.703798237469634,-2.663153349226357)--(5.7844138000942795,-2.654130029099403)--(5.864470783442447,-2.642506817254245)--(5.943877812202401,-2.628303835205039)--(6.022544179811227,-2.6115443645559555)--(6.100379954664131,-2.5922548249287236)--(6.177296085590451,-2.570464748400741)--(6.253204506483588,-2.5462067504781927)--(6.328018239972782,-2.5195164976323614)--(6.401651500025676,-2.490432671430731)--(6.474019793371595,-2.458996929298212)--(6.5450400196366925,-2.4252538619472173)--(6.614630570083332,-2.389250947518787)--(6.682711424847496,-2.3510385024805487)--(6.749204248569482,-2.310669629330468)--(6.814032484314764,-2.268200161158872)--(6.877121445683596,-2.2236886031244434)--(6.938398407009734,-2.177196070903121)--(6.997792691550595,-2.128786226172008)--(7.055235757573147,-2.0785252091935535)--(7.11066128224197,-2.026481568568283)--(7.1640052432181225,-1.9727261882273293)--(7.21520599787973,-1.9173322117389469)--(7.2642043600776605,-1.8603749640060485)--(7.31094367434207,-1.80193187043449)--(7.355369887458228,-1.7420823736546087)--(7.397431617332653,-1.6809078478810122)--(7.4370802190733345,-1.6184915109982243)--(7.474269848210662,-1.5549183344621311)--(7.508957520988518,-1.4902749511095918)--(7.54110317165801,-1.4246495609707153)--(7.570669706709337,-1.3581318351805618)--(7.5976230559803755,-1.2908128180889764)--(7.621932220583759,-1.2227848276692956)--(7.643569317597443,-1.1541413543284293)--(7.662509621467036,-1.0849769582226687)--(7.678731602071531,-1.0153871651850954)--(7.692216959407448,-0.9454683613720967)--(7.702950654849841,-0.8753176867378674)--(7.710920938952139,-0.8050329274470911)--(7.716119375750251,-0.7347124073372292)--(7.718540863540035,-0.6644548785428993)--(7.718183652100688,-0.5943594113958361)--(7.715049356340395,-0.5245252837147606)--(7.70914296634409,-0.4550518696002611)--(7.700472853806974,-0.3860385278503894)--(7.6890507748410455,-0.31758449011322704)--(7.674891869145676,-0.24978874889302194)--(7.658014655536929,-0.182749945526834)--(7.638441023834116,-0.11656625824875011)--(7.616196223105752,-0.0513352904587574)--(7.591308846280891,0.012846040686644855)--(7.563810811135435,0.07588161524531412)--(7.533737337666901,0.13767622090523002)--(7.501126921874691,0.19813566272911132)--(7.466021305966696,0.2571668721123279)--(7.428465445016741,0.31467801483921676)--(7.388507470101018,0.37057859812373284)--(7.346198647945297,0.42477957652124854)--(7.301593337118333,0.47719345659930035)--(7.2057258562395825,0.576318326578249)--(7.145670669463989,0.6305820984210833)--(7.082824779499289,0.681924453783592)--(7.017299996009972,0.7302208882615004)--(6.949213262109829,0.775350986483673)--(6.878686505069395,0.8171986242525913)--(6.80584647928422,0.8556521651896496)--(6.730824601707877,0.8906046515942045)--(6.653756779964498,0.9219539892328772)--(6.574783233366126,0.9496031257836874)--(6.494048307070418,0.9734602226679547)--(6.411700279624169,0.993438820011705)--(6.3278911641477285,1.009457994487441)--(6.24277650342457,1.0214425097966837)--(6.156515159169266,1.0293229595634736)--(6.069269095755586,1.0330359024192304)--(5.981203158694583,1.032523989069822)--(5.892484848160368,1.0277360811464995)--(5.80328408786853,1.0186273616533879)--(5.713772989619189,1.00515943683559)--(5.691367083097633,1.0011074497453225)--(5.685764594958424,1.000051514851744)--(5.6855895118414,1.0000182399576059)--(5.6855019701664835,1.0000015962158717)--(5.685496498809231,1.0000005558426774)--(5.6854937631304905,1.000000035649933)--(5.685493592150569,1.0000000031377505)^^(3.666532459319288,1.7158227254556468), linewidth(2.)); \n /* dots and labels */\ndot((6.38,1.),dotstyle); \ndot((9.02,4.02),dotstyle); \ndot((9.074301470817678,-1.9716560339900866),dotstyle); \ndot((3.74,-2.02),dotstyle); \ndot((14.408602941635356,-1.9233120679801736),dotstyle); \ndot((9.924343193909214,-0.8781989574561639),dotstyle); \ndot((4.367476888693955,2.066465238469869),linewidth(4.pt) + dotstyle); \nclip((xmin,ymin)--(xmin,ymax)--(xmax,ymax)--(xmax,ymin)--cycle); \n /* end of picture */[/asy]", "Solution_15": "Let $N$ be the midpoint of $BT$. Now since $M$ is the midpoint of $BC$, we hace $MN||CT$ so $\\angle CTM = \\angle TMN$. Also $\\angle TNX = 90 = \\angle XMT$, hence $\\omega_{TNMX}$ exists. So coupled with this, we get $\\angle CTM = \\angle TMN = \\angle TXN$ (call this $\\spadesuit$)\n\nNow, $\\angle BTM $ $$= \\angle BTX - \\angle XTM = \\angle BTX + \\angle TXM - 90 = \\angle TXM + (\\angle BTX - 90)$$ $$= \\angle TXM - \\angle TXN = \\angle NXM = \\angle NXB + \\angle BXM = \\angle TXN + \\angle BXM \\overset{\\spadesuit}{=} \\angle CTM + \\angle BAM$$, hence $\\angle BTM - \\angle CTM = \\angle BAM$, independent of $X$, as desired. ", "Solution_16": "Let $N$ be [i]not[/i] the midpoint of $BT$. ;)\n\n[hide=Solution]Let $N$ be the reflection of $T$ across point $M$ and note that $BNCT$ is a parallelogram. Let $\\angle{TNX}=\\beta$ and $\\angle{BNT}=\\angle{NTC}=\\alpha$. Then since $XT=XN$ and $M$ is the midpoint of $TN$, $\\angle{XTM}=\\angle{XNM}=\\beta$. Since $XT=XB=XN$ is the radius of the circumcircle of $\\triangle{TBN}$, $\\angle{BXT}=2\\angle{BNT}=2\\alpha$. Since $\\triangle{BTX}$ is isosceles, $\\angle{BTX}=90^{\\circ}-\\alpha$ so $\\angle{BTN}=\\angle{BTX}-\\angle{NTX}=90^{\\circ}-\\alpha-\\beta$. So we want to find the value of $\\angle{BTN}-\\angle{CTN}=90^{\\circ}-2\\alpha-beta$.\n\nSince $\\triangle{XNT}$ is a right triangle, $\\angle{TXN}=90^{\\circ}-\\beta$ and $\\angle{BTX}=2\\alpha$ so $\\angle{BXM}=\\angle{TXN}-\\angle{BTX}=90^{\\circ}-\\beta-2\\alpha=\\angle{BAM}$ as $BMXA$ is cyclic. Therefore, $\\angle{BTM}-\\angle{CTM}=\\angle{BAM}$, which is independent of $X$.[/hide]", "Solution_17": "[quote=delegat]Denote by $ M$ midpoint of side $ BC$ in an isosceles triangle $ \\triangle ABC$ with $ AC = AB$. Take a point $ X$ on a smaller arc $ \\overarc{MA}$ of circumcircle of triangle $ \\triangle ABM$. Denote by $ T$ point inside of angle $ BMA$ such that $ \\angle TMX = 90$ and $ TX = BX$. \n\nProve that $ \\angle MTB - \\angle CTM$ does not depend on choice of $ X$.\n\n[i]Author: Farzan Barekat, Canada[/i][/quote]\nLet $N$ be the midpoint of $AB.$ Define $\\omega,\\Omega$ to be the circles $(AMB), \\mathcal{C}(X,XB).$ Let $Y$ be the antipode of $X$ in $\\omega.$ Clearly $T \\in \\overline{MY}, T \\in \\Omega$ by the conditions. Also, $\\measuredangle XBY=\\pi/2$ implies that $YB$ is tangent to $\\Omega.$\n\n[color=#f00]Claim:[/color] $\\measuredangle YBT=\\measuredangle CTM.$\n[color=#00f]Proof:[/color] Let $L$ be the reflection of $T$ over $M.$ Then $TBLC$ is a parallelogram. Further, since $TM=ML$ and $\\measuredangle XMT=\\pi/2,$ hence $TX=XL.$\n\nSo, $L \\in \\Omega.$ Since $YB$ is tangent to this circle, hence $\\measuredangle YBT=\\measuredangle BLT=\\measuredangle CTL.$ $\\square$\n-------\nThus we have\n$$\\measuredangle MTB-\\measuredangle CTM=\\measuredangle MTB-\\measuredangle YBT=\\measuredangle TYB=\\measuredangle MAB$$\nwhich is independent of the position of $X.$ $\\blacksquare$\n[asy]\n /* Geogebra to Asymptote conversion, documentation at artofproblemsolving.com/Wiki go to User:Azjps/geogebra */\nimport graph; size(13cm); \nreal labelscalefactor = 0.5; /* changes label-to-point distance */\npen dps = linewidth(0.7) + fontsize(10); defaultpen(dps); /* default pen style */ \npen dotstyle = black; /* point style */ \nreal xmin = -1086.5975564873966, xmax = 1190.1243804115663, ymin = -879.117740078539, ymax = 608.3405920287947; /* image dimensions */\n\n\ndraw(arc((-586.1713229414271,-304.6162131768956),108.41533032852203,92.83654263451801,108.26645574818497)--(-586.1713229414271,-304.6162131768956)--cycle, linewidth(0.2) + red); \ndraw(arc((-603.0349055618302,35.73566225078713),108.41533032852203,-35.66215225285588,-20.232239139188955)--(-603.0349055618302,35.73566225078713)--cycle, linewidth(0.2) + red); \n /* draw figures */\ndraw(circle((-353.0915854617429,10.809060298607557), 392.19799485807476), linewidth(0.4)); \ndraw((-586.1713229414271,-304.6162131768956)--(-120.01184798205874,326.2343337741107), linewidth(0.4)); \ndraw((-120.01184798205874,326.2343337741107)--(337.5007033357593,-310.9150475089905), linewidth(0.2)); \ndraw((337.5007033357593,-310.9150475089905)--(-586.1713229414271,-304.6162131768956), linewidth(0.4)); \ndraw((-120.01184798205874,326.2343337741107)--(-124.3353098028339,-307.76563034294304), linewidth(0.4)); \ndraw((-728.1424189633996,125.50927590610821)--(21.959248039913632,-103.89115530889305), linewidth(0.2)); \ndraw((-586.1713229414271,-304.6162131768956)--(-728.1424189633996,125.50927590610821), linewidth(0.4)); \ndraw((-728.1424189633996,125.50927590610821)--(-124.3353098028339,-307.76563034294304), linewidth(0.4)); \ndraw(circle((21.959248039913632,-103.89115530889305), 640.4009214688908), linewidth(0.2) + linetype(\"2 2\")); \ndraw((-603.0349055618302,35.73566225078713)--(354.3642859561624,-651.2669229366732), linewidth(0.4)); \ndraw((337.5007033357593,-310.9150475089905)--(354.3642859561624,-651.2669229366732), linewidth(0.4)); \ndraw((-603.0349055618302,35.73566225078713)--(337.5007033357593,-310.9150475089905), linewidth(0.4)); \ndraw((-586.1713229414271,-304.6162131768956)--(354.3642859561624,-651.2669229366732), linewidth(0.4)); \ndraw((-603.0349055618302,35.73566225078713)--(-586.1713229414271,-304.6162131768956), linewidth(0.4)); \ndraw((-124.3353098028339,-307.76563034294304)--(337.5007033357593,-310.9150475089905), linewidth(0.2)); \ndraw((-603.0349055618302,35.73566225078713)--(-728.1424189633996,125.50927590610821), linewidth(0.2)); \n /* dots and labels */\ndot((-353.0915854617429,10.809060298607557),dotstyle); \nlabel(\"$N$\", (-377.5612961388625,42.4125677139054), NE * labelscalefactor); \ndot((-120.01184798205874,326.2343337741107),dotstyle); \nlabel(\"$A$\", (-110.85958353069825,348.1437992403399), NE * labelscalefactor); \ndot((-120.0118479820587,326.2343337741107),linewidth(4pt) + dotstyle); \ndot((-586.1713229414271,-304.6162131768956),linewidth(4pt) + dotstyle); \nlabel(\"$B$\", (-622.5799426813222,-360.89246110819965), NE * labelscalefactor); \ndot((-124.3353098028339,-307.76563034294304),dotstyle); \nlabel(\"$M$\", (-141.2158760226844,-365.2290743213406), NE * labelscalefactor); \ndot((337.5007033357593,-310.9150475089905),linewidth(4pt) + dotstyle); \nlabel(\"$C$\", (346.6531104556648,-293.6749563045155), NE * labelscalefactor); \ndot((21.959248039913632,-103.89115530889305),dotstyle); \nlabel(\"$X$\", (56.1000251752257,-98.52736171317432), NE * labelscalefactor); \ndot((21.95924803991367,-103.89115530889306),linewidth(4pt) + dotstyle); \ndot((-728.1424189633996,125.50927590610821),linewidth(4pt) + dotstyle); \nlabel(\"$Y$\", (-783.0346315675349,129.14483197672368), NE * labelscalefactor); \ndot((-603.0349055618302,35.73566225078713),linewidth(4pt) + dotstyle); \nlabel(\"$T$\", (-652.9362351733084,12.056275221919), NE * labelscalefactor); \ndot((354.3642859561624,-651.2669229366732),linewidth(4pt) + dotstyle); \nlabel(\"$L$\", (368.33617652136917,-696.9799851266205), NE * labelscalefactor); \nclip((xmin,ymin)--(xmin,ymax)--(xmax,ymax)--(xmax,ymin)--cycle); \n /* end of picture */\n[/asy]", "Solution_18": "[hide=Solution]Spent Whole day solving $BX=BT$ :wallbash: \n[quote=ISL 2007 G2]Denote by $ M$ midpoint of side $ BC$ in an isosceles triangle $ \\triangle ABC$ with $ AC = AB$. Take a point $ X$ on a smaller arc $ \\overarc{MA}$ of circumcircle of triangle $ \\triangle ABM$. Denote by $ T$ point inside of angle $ BMA$ such that $ \\angle TMX = 90$ and $ TX = BX$. \n\nProve that $ \\angle MTB - \\angle CTM$ does not depend on choice of $ X$.\n[/quote]\n[b][color=#000]Solution:[/color][/b] Let $T'$ be reflection of $T$ over $M$ $\\implies$ $X$ is center of $\\odot (BTT')$. Hence, $$ \\angle BTM-\\angle CTM=\\angle BXM-\\tfrac{1}{2} \\angle BXT-\\angle BT'T=\\angle BXM=\\angle BAM \\qquad \\blacksquare$$\n[/hide]", "Solution_19": "The structure of the most un-elegant solution in this topic.\n\nI will compute these step by step:\n\n1) Let $a,b,c$ be the side lengths of $\\Delta ABC$.We can compute $AM$ by Pythagoras.\n\n2) Let $AX=x$, We can compute $BX$ by Pythagoras.\n\n3) We can find $MX$ by Ptolemy over $ABMX$.\n\n4)We can compute $MT$ by Pythagoras over $\\Delta MTX$($TX=BX$ will be useful here.)\n\n5) Note that $\\angle XMA =\\angle TMB$. The side lengths of $\\Delta MXA$ is known, hence both the angles is known.\n\n6) By cosine rule we can find BT and TC and hence $\\angle BTM \\& \\angle CTM$.\n\n7) If we subtract these angles, we will get a relation in which there is not $x$ ! That means it is independent of $X$.\n\nI know it is near impossible to do but It is the first thing that came to my mind :-D (Geo is a lie without bash)", "Solution_20": "[asy]\nsize(9cm);\ndefaultpen(fontsize(10pt));\ndefaultpen(linewidth(0.4));\n\npair A = dir(55), B = dir(235), M = dir(305), C = 2M-B, X = dir(355), D = dir(175), T = intersectionpoint(D--M,arc(X,abs(B-X),90,225)), E = 2M-T;\n\ndot(\"$A$\", A, dir(55));\ndot(\"$B$\", B, dir(235));\ndot(\"$M$\", M, dir(270));\ndot(\"$C$\", C, dir(20));\ndot(\"$X$\", X, dir(5));\ndot(\"$D$\", D, dir(175));\ndot(\"$T$\", T, dir(210));\ndot(\"$E$\", E, dir(280));\n\ndraw(A--B--C--A--M--X^^unitcircle, purple);\ndraw(T--X^^E--X^^B--X, blue);\ndraw(T--B--E--C--T, orange);\ndraw(X--A--D--2B-D^^D--E, heavygreen);\ndraw(arc(circumcenter(T,B,E),abs(circumcenter(T,B,E)-T),165,315), magenta);\n[/asy]\nLet $\\overline{MT}$ meet $(ABM)$ again at $D$, and $E$ be the reflection of $T$ over $M$. \n\n[b]Claim:[/b] $X$ is the circumcenter of $(TBE)$. \n\n[i]Proof.[/i] Note that $\\triangle XMT \\cong \\triangle XME$ by SAS, so $TX = EX$. Also we are given that $TX = BX$ so we are done. $\\square$\n\nIn addition, $\\angle DBX = \\angle DMX = 90^\\circ$, so $\\overline{DB}$ is tangent to $(TBE)$. It follows that \\begin{align*}\\angle MTB - \\angle CTM &= \\angle MTB - \\angle BEM = \\angle MTB - \\angle DBT \\\\ &= \\angle MDB = \\angle BAM,\\end{align*} which does not depend on $X$ as desired. $\\blacksquare$", "Solution_21": "Solution from [i]Twitch Solves ISL[/i]:\n\nConstruct parallelogram $CTBS$ whose diagonals meet at $M$. Also, let $N$ be the midpoint of $\\overline{BT}$.\n[asy] pair A = dir(180); pair B = dir(0); pair M = dir(35); pair C = 2*M-B; pair X = dir(100);\n\nfilldraw(A--B--C--cycle, invisible, red); draw(A--M, red); filldraw(unitcircle, invisible, blue);\n\npair Y = -X; draw(CP(X, B), mediumgreen); pair T = IP(M--Y, CP(X, B)); pair S = 2*M-T; draw(S--T, mediumgreen); draw(B--X--M, mediumgreen);\n\npair N = midpoint(B--T); draw(B--T, deepgreen); draw(T--X--S, mediumgreen); draw(X--N, deepgreen);\n\nfilldraw(circumcircle(X, M, T), invisible, dotted+deepgreen);\n\ndot(\"$A$\", A, dir(A)); dot(\"$B$\", B, dir(B)); dot(\"$M$\", M, dir(M)); dot(\"$C$\", C, dir(C)); dot(\"$X$\", X, dir(X)); dot(\"$T$\", T, dir(T)); dot(\"$S$\", S, dir(S)); dot(\"$N$\", N, dir(N));\n\n/* TSQ Source:\n\nA = dir 180 B = dir 0 M = dir 35 C = 2*M-B X = dir 100\n\nA--B--C--cycle 0.1 lightred / red A--M red unitcircle 0.1 cyan / blue\n\nY := -X CP X B mediumgreen T = IP M--Y CP X B S = 2*M-T S--T mediumgreen B--X--M mediumgreen\n\nN = midpoint B--T B--T deepgreen T--X--S mediumgreen X--N deepgreen\n\ncircumcircle X M T 0.1 yellow / dotted deepgreen\n\n*/ [/asy]\nWe first eliminate $C$ from the diagram by noting that \\[ \\angle CTM = \\angle MSB = \\angle TSB = \\frac{1}{2} \\angle TXB = \\angle NXB. \\] Also, noting that $XMTN$ are cyclic (as $\\angle XMT = \\angle XNT = 90^{\\circ}$), we have \\[ \\angle MTB = \\angle MTN = \\angle NXM. \\] Thus $\\angle MTB - \\angle CTM = \\angle NXM - \\angle NXB = \\angle MXB$ which is fixed.", "Solution_22": "Let $N$ be the midpoint of $BT$. Observe that since $\\angle XMT = \\angle XNT = 90$, the quadrilateral $XMNT$ is cyclic. Furthermore, $NM \\parallel CT$.\n\nOn the one hand, note that $$\\angle MTB = \\angle MTN = MXN.$$\n\nOn the other hand, note that $$\\angle CTM = \\angle NMT = \\angle NXT = \\angle NXB.$$ Now, note that $$\\angle MTB - \\angle CTM = \\angle MXN - \\angle NXB = \\angle BXM = \\angle BAM$$ which does not depend on $X$, as desired.\n", "Solution_23": "[asy]\n\t\tunitsize(3cm);\n\t\tpair A, B, C, M, T, T1, X;\n\t\t\n\t\tA = dir(90);\n\t\tB = dir(210);\n\t\tC = dir(330);\n\t\tM = (B + C) / 2;\n\t\tX = rotate(20, (A + B) / 2) * M;\n\t\tT = IP(CP(X, B), Line(M, rotate(90, M) * X, 10), 0);\n\t\tT1 = 2 * M - T;\n\t\t\n\t\tdraw(A--B--C--A^^unitcircle, heavyred);\n\t\tdraw(circumcircle(A, B, M), heavygreen);\n\t\tdraw(CP(X, T), blue);\n\t\tdraw(X--T^^X--B^^X--M^^X--T1^^T--T1, orange);\n\t\t\n\t\tdot(\"$A$\", A, N);\n\t\tdot(\"$B$\", B, SW);\n\t\tdot(\"$C$\", C, SE);\n\t\tdot(\"$M$\", M, S);\n\t\tdot(\"$X$\", X, E);\n\t\tdot(\"$T$\", T, NW);\n\t\tdot(\"$T'$\", T1, SE);\n[/asy]\n\nLet $T'$ be the reflection of $T$ over $M$. Note that $TBT'C$ is a parallelogram. Additionally, as $\\angle TMX = 90^\\circ$, $\\triangle XTT'$ is isosceles, so we have $XT = XB = XT'$. Hence, $X$ is the circumcenter of $\\triangle BTT'$. Now, \n\\begin{align*}\n\t\\angle BTM - \\angle CTM &= \\angle BTT' - \\angle CTT' \\\\\n\t&= \\angle BTT' - \\angle BT'T \\\\\n\t&= \\frac{1}{2}(\\angle BXT' - \\angle BXT) \\\\\n\t&= \\frac{1}{2}((\\angle BXM + \\angle MXT) - (\\angle MXT - \\angle BXM)) \\\\\n\t&= \\frac{1}{2}(2\\angle BXM) \\\\\n\t&= \\angle BAM,\n\\end{align*}\nwhich is fixed, as desired. $\\Box$\n", "Solution_24": "[quote=AMN300]We do not use complex numbers\n\n[hide=Solution]\nLet $N$ be the midpoint of $BT$. Since $XB=XT$, $\\angle XNT=90^{\\circ}=\\angle XMT$, so $XMNT$ is cyclic, yielding $\\angle BTM=\\angle NTM=\\angle NXM$.\nNow $MN$ is a midline in $\\triangle BCT$, so \n\\[ \\angle MTC=\\angle BTC - \\angle MTC=\\angle BNM-\\angle MXN=\\angle MXT-\\angle MXN=\\angle NXT=\\angle NXB \\]\nHence, $\\angle BTM-\\angle CTM=\\angle BXN - \\angle MXN=\\angle BXM=\\angle BAM$ which is fixed.\n[/hide] yes I think the locus of T is a circle passing through $A,M$ and midpoint of $AB$\n ;) \n\nWhen I started working on this problem, I misread it as $TB=TX$. If we characterize $T$ as such, can we find any interesting properties of this new configuration? All my progress so far for this configuration is rather trivial.[/quote]\n\n", "Solution_25": "Dunno if this has been posted before.\n\nComplex bash with $a=-1, b=1$ so that $|m|=|x|=1$ and $c=2m-1$. Since the foot from $T$ to $\\overline{MX}$ is $M$, $$m=\\frac{1}{2}(t+m+x-mx\\overline{t}) \\implies \\overline{t}=\\frac{t-m+x}{mx}.$$ Since $\\overline{TX}=\\overline{BX}$ we should have $\\frac{t-x}{b-x}$ on the unit circle, and upon substituting our expression for $\\overline{t}$ $$\\frac{t-x}{b-x}=\\frac{\\frac{1}{b}-\\frac{1}{x}}{\\overline{t}-\\frac{1}{x}} \\implies t^2-2mt+x^2m+m-x^2.$$ Let $r$ be the unwanted root. By Vieta's, it follows that the argument of $\\angle{MTB}-\\angle{CTM}$ is (for $k \\in \\mathbb{R}$) $$\\frac{\\frac{m-t}{1-t}}{\\frac{2m-1-t}{m-t}}=k_1 \\cdot \\frac{\\frac{r-t}{1-t}}{\\frac{r-1}{r-t}}=k_1 \\cdot \\frac{(r-t)^2}{-(rt-r-t+1)} = k_1 \\cdot \\frac{4m^2-4m-4x^2m+4x^2}{-(x^2m-x^2-m+1)} = k_2 \\cdot \\frac{m-x^2}{1-x^2}$$ or $\\frac{m-x^2}{1-x^2}$. Since $x^2 \\in (ABM)$, this is equivalent to $\\angle{MXB}=\\angle{MAB}$ which does not depend on $X$, done.", "Solution_26": "Construct $N$ the midpoint of $TB$. Since $\\overline{MN} \\parallel \\overline{TC}$, we can calculate \\begin{align*} \\angle MTB - \\angle MTC = \\angle MTN - \\angle TMN = \\angle MXN - \\angle NXB = \\angle MXB = \\angle MAB, \\end{align*} which is constant done yay kill me now\n\nmisread this problem for a good 50 minutes :D", "Solution_27": "Solved with [b][url=https://artofproblemsolving.com/community/user/599649]MrOreoJuice[/url][/b] and [b][url=https://artofproblemsolving.com/community/user/785149]Fakesolver19[/url][/b].\n[asy]\n//meh asy, as always...\nsize(210);\npair A=(0,6.5),B=(-3,0),C=(3,0),M=(0,0),T=(-3.246,3.0711);\npair X=(1.828,1.9321), TT=(3.246,-3.0711);\ndraw(A--B--C--cycle, orange);\ndraw(circumcircle(A,B,M), orange);\ndraw(T--B--TT--C--cycle, magenta);\ndraw(X--T, red);\ndraw(X--B, red);\ndraw(X--TT, red);\ndraw(X--M, orange);\ndraw(circumcircle(B,T,TT), red);\ndraw(T--TT, magenta);\ndot(\"$A$\",A,N);\ndot(\"$B$\",B,W);\ndot(\"$C$\",C,E);\ndot(\"$M$\",M,S);\ndot(\"$T'$\",TT,S);\ndot(\"$X$\",X,NE);\ndot(\"$T$\",T,NW);\n[/asy]\nLet $T'$ be the reflection of $T$ over $M$. Note that $TBT'C$ is a parallelogram and $X$ is circumcenter of $(TBT')$.\n\\begin{align*}\n \\angle MTB - \\angle CTM &= \\angle T'TB - \\angle TT'B \\\\\n &= \\dfrac{\\angle T'XB - \\angle TXB}{2} \\\\\n &= \\dfrac{\\angle TXT' - 2\\angle TXB}{2} \\\\\n &= \\dfrac{2\\angle TXM - 2\\angle TXB}{2}\\\\\n &= \\dfrac{2\\angle BXM}{2}\\\\\n &= \\angle BAM\n\\end{align*}\nwhich does not depend on $X$ as desired. $\\blacksquare$", "Solution_28": "Let $N$ be the midpoint of $BT$. We have that $\\angle XMT=\\angle XNT=90$ so $XMNT$ is cyclic. Also $CT\\parallel MN$ thus $\\angle CTM=\\angle TMN$. In turn $\\angle TMN=\\angle TXN=\\angle BXN$. Also $\\angle MTB=\\angle MXN$, so we can get that $$\\angle MTB-\\angle CTM= \\angle MXN-\\angle BXN = \\angle MXB=\\angle MAB$$ which doesn't depend on $X$. ", "Solution_29": "Denote by $N$ midpoint of $BT.$ Clearly $CT\\parallel MN, XN\\perp BT,$ hence $$\\angle BTM-\\angle CTM=\\angle NTM-\\angle NMT=\\angle NXM-\\angle NXT=\\angle BXM=\\angle BAM.$$", "Solution_30": "Let $N$ be midpoint of $TB$. $BXT$ is isosceles so $\\angle XNT = 90 = XMT$ so $XMNT$ is cyclic.\n$\\angle MTB - \\angle MTC = \\angle MXE - \\angle EXT = \\angle MXE - \\angle BXE = \\angle MXB = \\angle MAB$ and $\\angle MAB$ is fixed and independent from $X$.\nwe're Done.", "Solution_31": "Let $N$ be the midpoint of $\\overline{BT}$ and note $MNTX$ is cyclic and $\\overline{MN}\\parallel\\overline{CT}.$ Hence, $\\angle BTM=\\angle BXM$ and $$\\angle MTC=\\angle NMB=\\angle TXN=\\angle NXB$$ so $$\\angle MTB-\\angle CTM=\\angle BXM=\\tfrac{1}{2}\\angle A.$$ $\\square$", "Solution_32": "All angles are in degrees.\n\nLet $MT$ intersect the circle again at $U$. Note that $\\angle XBU = \\angle XMU = 90$, thus $AXBU$ is a rectangle. Since $\\angle XBM + \\angle TMB = \\angle XBM + \\angle UMB = \\angle XBM + \\angle ABX = \\angle MBA$, we have $$\\angle MTB = 180 - \\angle TBX - \\angle XBM - \\angle TMB = 180 - \\angle MBA - \\angle TBX.$$\n\nLet $N$ be the midpoint of $BT$. Notice that $\\angle XMT = \\angle XNT = 90$, so $XMNT$ is cyclic. Since $MN\\parallel CT$, we have $$\\angle MTC = \\angle TMN = \\angle TXN = 90 - \\angle XBT.$$ This implies that $\\angle BTM - \\angle MTC = 90 - \\angle MBA$, a constant value.", "Solution_33": "[img width=50]https://media.discordapp.net/attachments/986049049834713139/986428810968825886/ISL2007G2.png[/img]Let $N$ denote the midpoint of $BT.$ Then, $\\angle TNX=\\angle TMX=90^\\circ$ so $TNMX$ is cyclic. Then, $\\angle MTB-\\angle CTM=\\angle NTM-\\angle TMN$ because $NM || TC.$ Now, $\\angle NTM-\\angle TMN=\\angle NXM-\\angle TXN=\\angle BXM=\\angle BAM$ which is fixed so we're done.", "Solution_34": "Let $T_1$ be the reflection of $T$ in $M$, ray $MT$ meet $(ABM)$ again at $Y$, and the circle centered at $X$ with radius $XB = XT$ meet $(ABM)$ again at $Q$.\n\nBecause $XM \\perp TT_1$, we know $XT = XT_1$, so $BTQT_1$ is cyclic. Now, observe $$\\angle YBX = \\angle YMX = \\angle TMX = 90^{\\circ}$$ which implies $YB$ is tangent to $(BTQT_1)$. Similarly, we deduce that $YQ$ is tangent to $(BTQT_1)$, so $\\overline{TT_1Y}$ is the $T$-Symmedian of $BQT$. \n\nNow, since $M$ is the midpoint of $TT_1$, we know it's the $T$-Dumpty point of $BQT$, yielding $MBT \\overset{+}{\\sim} MTQ$. Thus, because $BTCT_1$ is clearly a parallelogram, $$\\angle MTB - \\angle CTM = \\angle MQT - \\angle BT_1T$$ $$= \\angle MQT - \\angle BQT = \\angle MQB = \\angle MAB$$ which finishes. $\\blacksquare$\n\n\n[b]Config Stuff:[/b] Let ray $XT$ meet $(ABM)$ again at $P$. The Incenter-Excenter Lemma implies $T$ is the incenter of $BPQ$. Moreover, since $\\angle XMT = 90^{\\circ}$, we know the $P$-Mixtilinear Incircle of $BPQ$ touches $(BPQ)$ at $M$. In fact, $MBT \\overset{+}{\\sim} MTQ$ is actually a well-known mixtilinear lemma.", "Solution_35": "Let $D$ be the midpoint of $BT$. So $TDMX$ is cyclic and $MD\\parallel CT$. So the desired angle quantity is $\\angle BAM$ which is fixed.", "Solution_36": "Let $E = \\overline{TM} \\cap (X)$. Notice that $\\angle MTB - \\angle CTM = \\angle MTB - \\angle BET = \\frac 12(\\widehat{BE} - \\widehat{TB})$ because $TCEB$ is a parallelogram. However, setting $D = \\overline{XM} \\cap (X)$ as the arc midpoint of $\\widehat{TE}$, we have $$\\frac 12(\\widehat{BE} - \\widehat{TB})= \\widehat{BD} = \\angle BXM = \\angle BAM$$ is fixed regardless of $X$, as needed.", "Solution_37": "time to go through my writeup list :P\n\nLet $N$ be the midpoint of $BT$, with $XMNT$ cyclic (since $\\angle TNX=\\angle TMX=90^\\circ$) and $MN \\parallel TC$ (since $MN$ is the $B$-midsegment in $\\triangle BTC$). Now we angle chase:\n\\[\\angle MTC=\\angle NMT=\\angle TXN=\\angle NXB\\]\nand $\\angle BTM=\\angle NXM$, so we are done since $\\angle BTM - \\angle MTC = \\angle BXM = 90 - \\angle ACM$. $\\square$", "Solution_38": "[quote]When I started working on this problem, I misread it as $TB=TX$. If we characterize $T$ as such, can we find any interesting properties of this new configuration? All my progress so far for this configuration is rather trivial.[/quote]\nLOL i made the exact same mistake on ggb, even though i READ TX=BX I didn't construct it properly and constructed it as on the circle with radius BX but with center B, so i was like \"why is this not working\"\n\nMy solution is the same as above, but without typos: @above I think it should be MTC=NMT=..., and it should also be BTM=NXM, hence BTM-MTC=BXM=BAM, which is independent.", "Solution_39": "Let $\\overline{MT}$ intersect $(ABM)$ again at $X'$ (the $X$-antipode) and the circle $\\omega$ centered at $X$ passing through $B$ and $T$ at $D$. Then since $\\overline{XM} \\perp \\overline{TD}$, $M$ is the midpoint of chord $\\overline{DT}$, hence $BDCT$ is a parallelogram. On the other hand, since $\\angle X'BX=90^\\circ$, $\\overline{X'B}$ is tangent to $\\omega$, hence $\\angle X'BT=\\angle BDT=\\angle CTM$, hence $\\angle MTB-\\angle CTM=\\angle BX'M=\\angle BAM$ which is fixed. $\\blacksquare$", "Solution_40": "Let $S$ be such that $SBTC$ is a parallelogram. Then\n\\[2(\\angle MTB-\\angle CTM)=\\angle BXS-\\angle BTX=(\\angle BXM+\\angle MXS)-(\\angle MXT-\\angle BXM)=2\\angle BXM=2\\angle BAM\\]\ndone. ", "Solution_41": "Let Omega be circle with center X and radius XB.\nLet TM intersect Omega at Y.\nLet P be point on Omega(Arc TBY) such that Angle PTY=CTY\nAfter that observe TY||BP (Angle chase for this)\nWe are done" } { "Tag": [ "combinatorics proposed", "combinatorics" ], "Problem": "Suppose that there are n people, and they each have their favourite numbers between 1 and n. Person #1 starts at 1 and moves towards n, and takes his favourite number. Person #2,... repeats the steps. If his favourite number is already taken, then he would move on and take the next available number. However, if all the numbers between the person's favourite number and n are taken, the person gets angry and leaves. Find the number of pairs of the (person, favourite number) so that no one gets angry and every number gets taken.\r\n\r\n[Edit] Note that, two people may have the same favourite number. For instance, the arrangement (n, f(n)), where f(n) is the favourite number of nth person, may be (1,5), (2,5), ..., (6,5), (7,5) (assuming n=10). Then this arrangement is not possible, because the 2nd person would take 6, 3rd would take 7, etc., and 6th taking 10. Then the 7th person has nothing to take, so he leaves for good.\r\n\r\nI hope I have made the problem more understandable.", "Solution_1": "I've got $(n+1)^{n-1}$.", "Solution_2": "Sorry, I haven't been checking on my problems lately.\r\n\r\nMaxal, do you want to share your solution? The reason I posted this problem is because it has an ingenious way of solving it, which I can't remember anymore (maybe only vaguely).\r\n\r\nAs soon as I remember mine, I'll post it as well, in maybe a week or so, so that others can try it as well." } { "Tag": [], "Problem": "A wooden raft (density=775 kg/m^3) is supporting a 72.7 kg man entirely above the water when he stands on it at Lake Travis. Find the minimum volume of the raft.", "Solution_1": "I'm so happy to finally see an easy problem! :)" } { "Tag": [], "Problem": "It might be a little early for me to be thinking about this, but should I go straight to grad school after college? Is it all right to work for maybe 2 years, and then apply for grad? Or will it be more difficult that way?", "Solution_1": "I have read some graduate stats for schools in the ivy league and such and it seems that quite a few people take time off before continuing on to graduate school. I would assume this is the same throughout most schools. What are you studying, by the way?", "Solution_2": "I might be doing Physics and CS... \r\nCollege will (hopefully) determine which field I want to go into, though. I'm very indecisive.", "Solution_3": "You have (at least) 4 years -- don't worry about it yet." } { "Tag": [], "Problem": "[quote=\"Vutang\"]Given $a, b, c$ are three real numbers such that : $a 2$\n$a+b=6-c$\n$ab+c(a+b)=9$ we see that $a,b$ satisfy\n$a+b=6-c$ and $ab=(c-3)^2$ so $a$ and $b$ are the roots of $f(x)=x^2+(c-6)x+(c-3)^2=0$ the discriminant is $D=-3c(c-4)$ $D>0$ when $c<4$\n$c$ is outside the roots of $f(x)=0$ so $f(c)>0$ but $f(c)=3(c-1)(c-3)$ so $c>3$\nAs $f(1)=(c-1)(c-4)$ we see that $f(1)<0$ so $a<10$ and $b>1$ we conclude that $a>0$ and as $a+c>3$ $b<3$\n[/hide]" } { "Tag": [ "induction" ], "Problem": "For each positive integer $k>1$, define the sequence ${a_{n}}$ by\r\n$a_{0}=1$ and $a_{n}=kn+(-1)^{n}a_{n-1}$ for each $n \\geq 1$.\r\n\r\nDetermine all values of $k$ for which 2000 is a term of the sequence.", "Solution_1": "Is [hide]$\\ k=3,23,667,2001$[/hide] correct?", "Solution_2": "I think that [hide=\"k can also equal\"]87[/hide] :maybe:", "Solution_3": "You're right. I'll outline my solution:\r\n[hide]So you can prove by induction or whatever that if \n$\\ n \\equiv 0 \\mod 4 \\rightarrow a_{n}=nk+1$\n$\\ n \\equiv 1 \\mod 4 \\rightarrow a_{n}=k-1$\n$\\ n \\equiv 2 \\mod 4 \\rightarrow a_{n}= (n+1)k-1$\n$\\ n \\equiv 3 \\mod 4 \\rightarrow a_{n}= 1$\n\nSo, clearly when $\\ n \\equiv 3 \\mod 4$ $\\ a_{n}$ can never be 2001. \nAlso, if $\\ n \\equiv 0 \\mod 4$ then $\\ 2000=4mk+1$ however $\\ 4$ does not divide $\\ 1999$ so $\\ 2000$ will not appear when $\\ n$ is divisible by $\\ 4$.\nSo, when $\\ n \\equiv 1 \\mod 4$ then $\\ a_{n}= k-1$ so if $\\ k=2001$ $\\ 2000$ will appear in the series.\nWhen $\\ n$ is in the form $\\ 4m+3$ then we can right $\\ 2000=(4m+3)k-1$ or $\\ 2001=(4m+3)k$. Now the factors of $\\ 2001$ which appear in the form $\\ 4m+3$ are $\\ 3, 23, 87, 667$ and the corresponding $\\ k$ values are $\\ 667, 87, 23, 3$\nSo, $\\ k= 3, 23, 87, 667, 2001$\n [/hide]" } { "Tag": [ "function", "calculus", "algebra", "domain", "linear algebra", "calculus computations" ], "Problem": "Let g: A - B, f: B - C, f o g: A - C be functions.\r\nIf f and f o g are one to one, does it follow that g is one to one?", "Solution_1": "Suppose $g$ is not 1-1. Then there exists $x,y\\in A$ such that $g(x)=c=g(y)$ for some $c\\in B$. So $f(g(x))=f(c)$ and $f(g(y))=f(c)$, but this is a contradiction since $f\\circ g$ is 1-1. Therefore $g$ is 1-1.\r\n\r\n...I didn't use $f$ is 1-1. Do you only need $f\\circ g$ to be 1-1 to get $g$ 1-1?", "Solution_2": "Yes, you only need $f \\circ g$ to be injective. \r\nProof: choose $a_{1}, a_{2}\\in A$. Suppose $g(a_{1}) = g(a_{2})$, we wish to show that $a_{1}= a_{2}$. Applying f to $g(a_{1}) = g(a_{2})$, and use the fact that this composition is injective, we get $a_{1}= a_{2}$, as it was required. So you did not need f to be injective.", "Solution_3": "This is clearly not calculus. This is elementary \"functionspeak.\" Problems of this ilk also sometimes show up in the linear algebra forum. I'd move them, except I'm stumped as to where to move them to.", "Solution_4": "At my university at least, the first time you would see this material and this terminology would be in a second year analysis course (it is also covered in a third-year abstract algebra course).\r\n\r\nI think it's fine where it is?", "Solution_5": "[quote=\"blahblahblah\"]At my university at least, the first time you would see this material and this terminology would be in a second year analysis course (it is also covered in a third-year abstract algebra course).[/quote]\r\nAn abstract algebra course is a bit late in the game for this. I'm using some of this language right now teaching a (college sophomore level) linear algebra course; we also quite explicitly deal with it in a sophomore level \"transition\" course (discrete structures/introduction to proof and abstraction.) Some questions one might ask - say about counting certain kinds of functions between finite sets - would properly belong to the combinatorics forum here. Ideally, students should come to a basic understanding of this language, and be able to write short proofs and construct counterexamples, before they even start calculus - but in the real world, that's a lot to ask.\r\n\r\nIt's really a matter of vocabulary: relation, function, domain, codomain, range, injective (1-1), surjective (onto), composition, identity map, inverse function." } { "Tag": [ "LaTeX", "number theory proposed", "number theory" ], "Problem": "Let $ k_1,k_2,...,$k_304 be the positive integers satisfying $ k_1^3\\plus{}k_2^3\\plus{}...\\plus{}$(k_304)^3\u22642003.\r\nFinh the largest value of A=$ k_1\\plus{}k_2\\plus{}k_3\\plus{}...\\plus{}$k_ 304", "Solution_1": ":blush: The answer is 304*3\u221a2003/304", "Solution_2": "[quote=\"New player\"]:blush: The answer is 304*3\u221a2003/304[/quote]\r\nNew player, use $ LaTeX$\r\nAlso $ k_1, k_2, \\dots \\dots ,k_{304}$ are positive integers." } { "Tag": [ "combinatorics unsolved", "combinatorics" ], "Problem": "Each of seven boys went to a cafe 3 times, such that every pair met there. Prove that at some moment 3 of the boys were at the cafe.", "Solution_1": "Take boy 1.\r\nHe meets the 6 other boys in his 3 visits to the cafe.\r\nSo in at least one of his visits, he meets more than one of the boys. \r\nAnd at that time, there are 3 boys at the cafe. \r\n\r\nOr have i misread the question? Do you mean [b]exactly[/b] three boys in the cafe?", "Solution_2": "You understood the problem well, but the thing is, I thought of the same solution and it's wrong. Namely, while one of the 3 boys is in the cafe, one of the other 2 might have come and left before the third one arrived.\r\n______ ___________\r\n ____________ Here it is visually, reperesented on time line, where easch of these 3 lines represents the time each of the three boys were present.", "Solution_3": "Consider the intervals when boys are in cafe as line-segments on a line. Consider the 21 left ends of this segments. If two boys meet they also meet in some of this 21 points(not hard to prove). If some of this points coincide then we may think that there are 20 points with this condition. If not take the leftmost point. Obviously there is no meeting there so again we may think the points are 20. But there are at least $ \\binom{7}{2}= 21$ meetings so some two meetings coincide, i.e. some 3 boys are in the cafe at the same time." } { "Tag": [ "summer program", "PROMYS", "Ross Mathematics Program" ], "Problem": "Ive got into both PROMYS, SSP, and the ROSS (unfortunately no RSI).\r\nwhich one should it be?", "Solution_1": "[quote=\"xxreddevilzxx\"]Ive got into [b]both[/b] PROMYS, SSP, and the ROSS (unfortunately no RSI).\nwhich one should it be?[/quote] \r\n\r\n :P \r\n\r\nPROMYS and ROSS are more math related than SSP, ( i think ;) ), \r\n\r\ngo to the one that gives you more money, so you have to pay less :lol:", "Solution_2": "where are you going?", "Solution_3": "Well Qiaochu will be at PROMYS, Edward at Ross, so maybe you'll see one of them there.", "Solution_4": "well im still undecided to attend PROMYS or ROSS.\r\ni was wondering where Young will go?", "Solution_5": "Young is going to PROMYS I believe. They've offered him a good chunk of money.\r\n\r\nYeah...I'm going somewhere else. You Washingtonians have fun!!! And say no to drugs! :ninja:", "Solution_6": "Actually i might be going somewhere else. My schedule just totally changed 360 degrees due to some personal reasons.", "Solution_7": "[quote=\"xxreddevilzxx\"]somewhere else. [/quote]\r\n\r\nas in? :P", "Solution_8": "Ewww... I don't want anybody like Young or Amir to be at Ross with me. They're just icky. :P And we all know that QC's a loser for choosing PROMYS over SIMUW, since SIMUW rules over all else. :D", "Solution_9": "[quote=\"Uh oh!\"]Ewww... I don't want anybody like Young or Amir to be at Ross with me. :P [/quote]\r\n\r\nerr...you're the one who made me apply in the first place!! :D" } { "Tag": [ "induction", "inequalities", "strong induction", "number theory unsolved", "number theory" ], "Problem": "the problem like following is far from my ability maybe masters would help :) \r\nPut $a_k=p_k^2-p_k,k\\geq 2$.($p_k$ is kth prime number)then there exists an absolute constant $A$ so that every even integer greater than $A$ is the sum of distinct $a_k$'s.", "Solution_1": "Wow, that's too beautiful! Thanks for the dedication, but this problem seems far too much for my poor head.", "Solution_2": "Here's a way to reduce the problem to a finite problem. Suppose we can find an even integer $A$ and an integer $k_0 \\ge 2$ with the following properties:\r\n\r\n(1) $A+2$, $A+4$, $A+6$, ..., $A + a_{k_0}$ are representable (as the sum of distinct $a_k$'s).\r\n\r\n(2) $2 a_k > a_{k+1} + A$ for every integer $k \\ge k_0$. (Presumably we can prove such a bound using the Prime Number Theorem or something similar.)\r\n\r\nThen I claim that every even integer $n$ bigger than $A$ is representable. The proof is by strong induction on $n$. The cases $n \\le a_{k_0} + A$ follow from property 1. So assume $n > a_{k_0} + A$. Let $j$ be the maximum integer such that $a_j + A < n$. We know that $j \\ge k_0$. By Property 2 and maximality, we have\r\n\\[\r\n 2 a_j > a_{j+1} + A \\ge n \\, .\r\n\\]\r\nBy induction, $n - a_j$ is representable. Because $2 a_j > n$, the representation of $n - a_j$ cannot use $a_j$. Thus $n = (n - a_j) + a_j$ is representable. That completes the inductive proof.\r\n\r\nAlas, I haven't done the computations needed to find $A$ and $k_0$.", "Solution_3": "Okay, I did some computations on an Excel spreadsheet. The choice $A = 3106$ and $k_0 = 19$ seems to work. We have $p_{k_0} = 67$ and $a_{k_0} = 4422$. The spreadsheet says that 3108, 3110, 3112, ..., 7528 are representable, so Property 1 is true. The inequality $2 a_k > a_{k+1} + 3106$ is true for the first few $k \\ge 19$, and can presumably be shown for all $k \\ge 19$ because $p_{k+1} / p_k$ (and thus $a_{k+1}/a_k$) approaches 1.\r\n\r\nEdit: I found a way to weaken Property 2 (at the expense of strengthening Property 1). That should make it easier to prove Property 2. Here are the new properties.\r\n\r\n(1') The numbers $A+2$, $A+4$, $A+6$, $A + a_{k_0}$ are representable with the numbers $a_2$, ..., $a_{k_0 - 1}$.\r\n\r\n(2') $a_{k+1} \\le 2 a_k$ for all $k \\ge k_0$.\r\n\r\nAccording to my spreadsheet, the new Property 1' is true for $A = 3106$ and $k_0 = 17$. So we just need to prove Property 2' that $a_{k+1} \\le 2 a_k$ for all $k \\ge 17$. In fact, the inequality seems to be true for all $k \\ge 5$. We can probably prove it by using inequalities such as $p_{k+1} \\le 1.3 p_k$ for all sufficiently large $k$." } { "Tag": [ "function", "algebra", "floor function", "number theory open", "number theory" ], "Problem": "If $ x\\in R$ then find all solution of the given floor - function $ [\\frac {2x \\minus{} 1}{3}] \\plus{} [\\frac {4x \\plus{} 1}{6}] \\equal{} \\frac {5x \\minus{} 4}{3}$", "Solution_1": "[quote=\"ertanrock\"]If $ x\\in R$ then find all solution of the given floor - function $ [\\frac {2x \\minus{} 1}{3}] \\plus{} [\\frac {4x \\plus{} 1}{6}] \\equal{} \\frac {5x \\minus{} 4}{3}$[/quote]\r\n\r\n$ 5x\\equal{}3 ([\\frac {2x \\minus{} 1}{3}] \\plus{} [\\frac {4x \\plus{} 1}{6}] ) \\plus{}4\\equal{}a \\in \\mathbb{Z} \\Rightarrow x\\equal{}\\frac{a}{5}$\r\n$ a\\equal{}3 ([\\frac {4a\\minus{}10}{30}] \\plus{} [\\frac {4a\\plus{}5}{30}]) \\plus{}4$\r\n\r\n$ a\\equal{}30m\\plus{}n$ with $ 0 \\le n < 30$\r\n$ 30m\\plus{}n\\equal{}3 ([\\frac {4(30m\\plus{}n)\\minus{}10}{30}] \\plus{} [\\frac {4(30m\\plus{}n)\\plus{}5}{30}]) \\plus{}4$\r\n$ 30m\\plus{}n\\equal{}24m\\plus{}3 ([\\frac {4n\\minus{}10}{30}] \\plus{} [\\frac {4n\\plus{}5}{30}]) \\plus{}4$\r\n$ 6m\\plus{}n\\equal{}3 ([\\frac {4n\\minus{}10}{30}] \\plus{} [\\frac {4n\\plus{}5}{30}]) \\plus{}4$\r\ntaking mod $ 3$ then $ n \\equiv 1 (3)$ therefore only solutions for $ n \\in \\{1,4,7, 10,28 \\}$ \r\nThus $ x \\in \\{\\frac{1}{5},\\frac{4}{5},\\frac{7}{5},2,\\minus{}\\frac{2}{5} \\}$", "Solution_2": "[quote=\"mszew\"][quote=\"ertanrock\"]If $ x\\in R$ then find all solution of the given floor - function $ [\\frac {2x \\minus{} 1}{3}] \\plus{} [\\frac {4x \\plus{} 1}{6}] \\equal{} \\frac {5x \\minus{} 4}{3}$[/quote]\n\n$ 5x \\equal{} 3 ([\\frac {2x \\minus{} 1}{3}] \\plus{} [\\frac {4x \\plus{} 1}{6}] ) \\plus{} 4 \\equal{} a \\in \\mathbb{Z} \\Rightarrow x \\equal{} \\frac {a}{5}$\n$ a \\equal{} 3 ([\\frac {4a \\minus{} 10}{30}] \\plus{} [\\frac {4a \\plus{} 5}{30}]) \\plus{} 4$\n\n$ a \\equal{} 30m \\plus{} n$ with $ 0 \\le n < 30$\n$ 30m \\plus{} n \\equal{} 3 ([\\frac {4(30m \\plus{} n) \\minus{} 10}{30}] \\plus{} [\\frac {4(30m \\plus{} n) \\plus{} 5}{30}]) \\plus{} 4$\n$ 30m \\plus{} n \\equal{} 24m \\plus{} 3 ([\\frac {4n \\minus{} 10}{30}] \\plus{} [\\frac {4n \\plus{} 5}{30}]) \\plus{} 4$\n$ 6m \\plus{} n \\equal{} 3 ([\\frac {4n \\minus{} 10}{30}] \\plus{} [\\frac {4n \\plus{} 5}{30}]) \\plus{} 4$\ntaking mod $ 3$ then $ n \\equiv 1 (3)$ therefore only solutions for $ n \\in \\{1,4,7, 10,28 \\}$ \nThus $ x \\in \\{\\frac {1}{5},\\frac {4}{5},\\frac {7}{5},2, \\minus{} \\frac {2}{5} \\}$[/quote] great solution." } { "Tag": [], "Problem": "I'm taking Sequences and Series at nationals, so I was studying and I've found that certain number sequences appear like Pentagonal numbers, Catalan numbers, and Lucas numbers. If someone knows offhand several types of numbers and the formulas to find them, it'd be greatly appreciated. Or at least compiling a list of different types of numbers", "Solution_1": "Try searching on [url=http://mathworld.wolfram.com/]MathWorld[/url]." } { "Tag": [], "Problem": "Topsoil costs $ \\$6$ per cubic foot. What is the cost, in dollars, of\n5 cubic yards of topsoil?", "Solution_1": "Well there are 3x3x3=27 cubic feet in a cubic yard. Thus there are 5x27=135 cubic feet, so its 6x135=810." } { "Tag": [ "geometry", "trigonometry", "function", "algebra", "polynomial", "quadratics", "analytic geometry" ], "Problem": "Hey guys,\r\ni'm 8th grade and about preparing for the euclid,\r\ni have these books:\r\n\"AoPS 2\"\r\n\"Challenging Problems in Geometry\"\r\nAlfred S. Posamentier\r\n\"Contest Problem Books (ahsme) 1-4\"\r\nHow to Solve It\"\r\nG. Polya\r\n\"Five Hundred Mathematical Challenges\"\r\nEdward J. Barbeau\r\n\"Mathematics of Choice\"\r\nIvan Morton Niven\r\n\"Introduction to Geometry\"\r\nCoxeter\r\n\r\nAre they helpful?\r\n\r\nAnd of course i did the old tests and my marks ranged from 90-100 and an 86. (I know on the real contest, i'll do crap :( )\r\n \r\nAnyway, thanks for helping :)", "Solution_1": "i'm sure thats enough..lol..u'll probably do fine if u can tackle most of the questions in past euclid contests", "Solution_2": "especially since youre in grade 8!", "Solution_3": "I found out that I can write the euclid as well as the hypatia :D\r\n\r\nBut what would you guys think the best topics to review would be, theorems that kinda thing?", "Solution_4": "For Euclid? This is the official list of topics (and it's pretty thorough IMO):\r\n\r\n-Euclidean and Analytic Geometry \r\n-Trigonometry, including functions, graphs, identities, sine and cosine laws\r\n-Exponential and logarithmic functions\r\n-Functional notation\r\n-Systems of equations\r\n-Polynomials, including relationships involving the roots of quadratic and cubic equations, the remainder theorem.\r\n-Sequences and series\r\n-Simple counting problems\r\n-Properties of numbers\r\n\r\nBasically anything you'd see in the high school basics forum extending into a bit of intermediate forum stuff.", "Solution_5": "[quote=\"max_tm\"]For Euclid? This is the official list of topics (and it's pretty thorough IMO):\n\n-Euclidean and Analytic Geometry \n-Trigonometry, including functions, graphs, identities, sine and cosine laws\n-Exponential and logarithmic functions\n-Functional notation\n-Systems of equations\n-Polynomials, including relationships involving the roots of quadratic and cubic equations, the remainder theorem.\n-Sequences and series\n-Simple counting problems\n-Properties of numbers\n\nBasically anything you'd see in the high school basics forum extending into a bit of intermediate forum stuff.[/quote]\r\ni dont get anything by looking at the topics :rotfl:", "Solution_6": "i already knew the topics and saw through old contests and most of the difficulty of the topics were kind of basic.\r\n\r\nthanks anyway" } { "Tag": [], "Problem": "A clock chimes two times 15 minutes after the hour, four times 30 minutes after the hour and 6 times 45 minutes after the hour. The clock also chimes eight times on each hour in addition to chiming the number of times equal to the hour. (So at 2:00 p.m. the clock chimes 8+2=10 times). Starting at 12:05 a.m., how many times does the clock chime in a 24 hour period?", "Solution_1": "[quote=\"myspacesucks\"]A clock chimes two times 15 minutes after the hour, four times 30 minutes after the hour and 6 times 45 minutes after the hour. The clock also chimes eight times on each hour in addition to chiming the number of times equal to the hour. (So at 2:00 p.m. the clock chimes 8+2=10 times). Starting at 12:05 a.m., how many times does the clock chime in a 24 hour period?[/quote]\r\n[hide=\"There is probably a shorter way...\"]For every hour (x:00-x:59), the number of chimes is $2+4+6+8+x=20+x$\nThen you solve for each value (1-12)\nThe values are then 21,22, 23, 24, 25, 26, 27, 28, 29, 30, 31, 32\n$21+22+23+24+25+26+27+28+29+30+31+32=6(21+32)=6(53)$\nMultiply by 2 for am/pm to get a final answer of $318*2=\\boxed{636}$[/hide]\r\nThat's an annoying clock!", "Solution_2": "[quote=\"Quevvy\"][quote=\"myspacesucks\"]A clock chimes two times 15 minutes after the hour, four times 30 minutes after the hour and 6 times 45 minutes after the hour. The clock also chimes eight times on each hour in addition to chiming the number of times equal to the hour. (So at 2:00 p.m. the clock chimes 8+2=10 times). Starting at 12:05 a.m., how many times does the clock chime in a 24 hour period?[/quote]\n[hide=\"There is probably a shorter way...\"]For every hour (x:00-x:59), the number of chimes is $2+4+6+8+x=20+x$\nThen you solve for each value (1-12)\nThe values are then 21,22, 23, 24, 25, 26, 27, 28, 29, 30, 31, 32\n$21+22+23+24+25+26+27+28+29+30+31+32=6(21+32)=6(53)$\nMultiply by 2 for am/pm to get a final answer of $318*2=\\boxed{636}$[/hide]\nThat's an annoying clock![/quote]\n[hide=\"I have a much easier way\"]Each hour, excluding the clock chiming for the number of hours in the time, there are $2+4+6+8=20$ chimes. Since there are $24$ hours, there are $480$ chimes. Next we account for the clock chiming for the number of hours in the time. It is not hard to check that it is: $2(1+2+...+12)=156$. Add these two up to get a final answer of $636$. [/hide]", "Solution_3": "You do realize that your ways are pretty much the same, right?", "Solution_4": "[quote=\"Ubemaya\"]You do realize that your ways are pretty much the same, right?[/quote]\r\n\r\nI'm just curious, but: Ubemaya, which method would you use?\r\n\r\nI realize that they are practically the same ways, but I'm more of a \"casework\" kind of person.", "Solution_5": "Uhh... well if I didn't read your solutions I probably couldn't have solved this.", "Solution_6": "You guys are right, the answer is 636, Vishalarul, your method is pretty much the same with a minute difference in the middle when you add them together. Nice job you guys." } { "Tag": [ "AMC", "USA(J)MO", "USAMO", "email", "AIME", "AMC 10", "MATHCOUNTS" ], "Problem": "I write to correct a number of false statements and incorrect\r\nunderstandings in this forum about the time it takes the AMC office\r\nto complete the USAMO selection process.\r\n\r\nIn the 2009 USAMO Teachers' Manual, on page 4, Section I, paragraph 6\r\nit says: \"The selection of the USAMO participants will be made on or\r\nbefore April 10th. All students ... will receive information...no\r\nlater than April 10.\" We published the USAMO invitation list and sent\r\nthe email invitations on April 10. We met our announced date. We\r\nwere not late or delayed in making the invitations.\r\n\r\nThe 2009 USAMO Teachers' Manual was written and printed months ago.\r\nAfter more than 30 years of administering the USAMO, we know our\r\nbusiness and we know how long it takes to complete our business. We\r\nmet our announced date. We were not late or delayed in making our\r\ninvitations. \r\n\r\nThere are two primary reasons that it takes us approximately 9 days\r\nfrom the alternate AIME date to the announcement of USAMO\r\ninvitations.\r\n (1) We do not receive the AIME II score sheets from some schools \r\n until as late as 1 week after the AIME II. On Friday, April 3\r\n we had received from about 60% of participating\r\n schools. Even on Wednesday, April 8 the AMC office still\r\n received AIME score sheets from some schools. This is in spite\r\n of requiring schools to send by some kind of express mail or\r\n package delivery. In some cases, it still takes 5 calendar days\r\n or more for the answer forms to reach us.\r\n\r\n (2) Second, and far more time consuming for us is that fact that\r\n many students incorrectly fill out the answer sheets. The most\r\n common error is to not use the same name when taking the AMC\r\n contest and the AIME contest, or when taking the AMC contests on\r\n both the A-date and the B-date. If the student uses the same\r\n name each time, our scoring processes automatically match the\r\n names. If a student did not use the same name, or incorrectly\r\n filled out the answer from, we have to laboriously match the\r\n student names and scores. This is tedious and time-consuming,\r\n particularly if the students took the contest at different\r\n locations. If students correctly and consistently filled out\r\n the answer forms, we might be able to save 2 to 3 days on the\r\n selection process.\r\n\r\n (3) A third time consuming effort is when students take the AMC 10\r\n or AMC 12 at more than one location, but request that scores and\r\n affiliations be reported from, say, their high school. It takes a\r\n lot of searching, matching, cross-checking and validation to make\r\n sure that we have reported the scores correctly.\r\n\r\nOne potential solution to the second and third problems would be to allow\r\nstudents to take only one AMC contest on the A-date or the B-date but\r\nnot both. Of course, a simpler solution would be for all students to\r\nconsistently and correctly enter their information on the answer\r\nforms. The AMC office continues to explore technological solutions\r\nto the first problem that would easily adapt to all schools nationally\r\nand internationally. \r\n\r\nSteve Dunbar\r\nMAA Director, American Mathematics Competitions", "Solution_1": "Thanks for the time and effort you spend responding to concerns on this forum.", "Solution_2": "Yes, I agree with worthawholebean, thank you.", "Solution_3": "And thanks for all the effort on sorting out tests and making sure that the results were on time. Just a thought, but maybe the AMC office was too nice last year with the preliminary list? :D", "Solution_4": "Thank you for your efforts. Sometimes people are just a bit eager to find out whether they made USAMO =p but we appreciate your work.", "Solution_5": "[quote](3) A third time consuming effort is when students take the AMC 10\nor AMC 12 at more than one location, but request that scores and\naffiliations be reported from, say, their high school. It takes a\nlot of searching, matching, cross-checking and validation to make\nsure that we have reported the scores correctly. [/quote]\r\nI'm sure this is a massive problem and a thankless one to try to overcome. And very, very, very hard to get everyone on the first pass. Do not be surprised if the list of USAMO qualifiers grows slightly as more multiple-site names are matched up. (In fact, I personally know of one such case, and I will inform the AMC office of what I know.)", "Solution_6": "[quote=\"AMCDirector\"]\n (2) Second, and far more time consuming for us is that fact that\n many students incorrectly fill out the answer sheets. The most\n common error is to not use the same name when taking the AMC\n contest and the AIME contest, or when taking the AMC contests on\n both the A-date and the B-date. If the student uses the same\n name each time, our scoring processes automatically match the\n names. If a student did not use the same name, or incorrectly\n filled out the answer from, we have to laboriously match the\n student names and scores. This is tedious and time-consuming,\n particularly if the students took the contest at different\n locations. If students correctly and consistently filled out\n the answer forms, we might be able to save 2 to 3 days on the\n selection process.\n\n (3) A third time consuming effort is when students take the AMC 10\n or AMC 12 at more than one location, but request that scores and\n affiliations be reported from, say, their high school. It takes a\n lot of searching, matching, cross-checking and validation to make\n sure that we have reported the scores correctly.\n[/quote]\r\n\r\nDear AMCDirector,\r\nHave you thought about assigning an unique number for each student (like using SSN)? I know many students took AMC 10 / AMC 12 at more than one location, unique number for each student may help to identify the correct score for each student.", "Solution_7": "[quote=\"AMCDirector\"]One potential solution to the second and third problems would be to allow\nstudents to take only one AMC contest on the A-date or the B-date but\nnot both. [/quote]\r\n\r\nA better solution is to modifying the answer sheets for students to write in their CEEB# and scores earned at the specific CEEB location:\r\n\r\nAMC 10/12A: CEEB#, AMC10 or AMC 12, your score\r\nAMC 10/12B: CEEB#, AMC 10 or AMC 12, your score\r\n\r\nIt will make the automatic matching easier.", "Solution_8": "[quote=\"shtsxc12\"][quote=\"AMCDirector\"]\nDear AMCDirector,\nHave you thought about assigning an unique number for each student (like using SSN)? I know many students took AMC 10 / AMC 12 at more than one location, unique number for each student may help to identify the correct score for each student.[/quote][/quote]\r\n\r\nThat will not work as AMC/AIME exams are international. Students in China, Taiwan, Singapore, etc. do not have SSN!", "Solution_9": "[quote=\"zmli\"]A better solution is to modifying the answer sheets for students to write in their CEEB# and scores earned at the specific CEEB location:\n\nAMC 10/12A: CEEB#, AMC10 or AMC 12, your score\nAMC 10/12B: CEEB#, AMC 10 or AMC 12, your score\n\nIt will make the automatic matching easier.[/quote]\r\n\r\nProblem is, people from International locations don't have a CEEB. (Or, could you explain what a CEEB is, if I'm wrong?)", "Solution_10": "[quote=\"zmli\"][quote=\"shtsxc12\"][quote=\"AMCDirector\"]\nDear AMCDirector,\nHave you thought about assigning an unique number for each student (like using SSN)? I know many students took AMC 10 / AMC 12 at more than one location, unique number for each student may help to identify the correct score for each student.[/quote]\n\nThat will not work as AMC/AIME exams are international. Students in China, Taiwan, Singapore, etc. do not have SSN![/quote][/quote]\r\n\r\nI think he means a special identification number just for AMC, not the actual SSN itself.", "Solution_11": "[quote=\"not_trig\"]I think he means a special identification number just for AMC, not the actual SSN itself.[/quote]\r\n\r\nSo something along the lines of what they do with USAMO student numbers? The problem is that the numbers would have to be centrally managed, and that takes a long time to administer. An [i]extremely[/i] long time.", "Solution_12": "The problems with an AMC-assigned special identification number:\r\n\r\nIt would have to have enough digits to distinguish several hundred thousand individuals - so it would have to be a 6 or 7 digit number, not the easiest thing to remember.\r\n\r\nThe onus would be on the individual student to remember/bring that number to the B date. There would have to be a mechanism for giving each student a piece of paper or sticker with that number on it (extra printing cost), and even if you did that, many hundreds of students would forget to bring that sticker to the B exam, in which case you're no better off than before, perhaps worst off.\r\n\r\nThere are a dozen different reasons why the AMC shouldn't and won't use Social Security numbers. I won't elaborate, other than to say this is a complete non-starter.\r\n\r\nThe only scheme I can see with a slight amount of hope: phone numbers. Put a place for a 10 - digit number, and ask student to fill in a number of their own choosing that they will remember, with the suggestion that it be a phone number (but the student is allowed to invent some other number). There would be some duplicates, both among siblings and among people that invented numbers, and there would still be students forgetting which number it was they had used the first time.\r\n\r\nI still think that there really isn't a better scheme than name-matching with address-matching as a backup. Unless the AMC wants to prohibit students from taking more than one AMC, or from taking AMC and AIME at different locations (which actions would ignite their own protests), then this problem is going to be there.", "Solution_13": "Personally, I really like the idea of restricting the AMC to one taking.", "Solution_14": "I would like to continue having the opportunity to take two AMCs. Apart from the mathematical side, AMC is very error-prone; by that, I include scantron error and teacher error. Being able to take two time greatly reduces such problems.\r\n\r\nA possible modification: if one choose to take the second AMC, then he or she automatically forfeits the first AMC score. It would be best if the first AMC score comes out before this decision is made.", "Solution_15": "I can only imagine the logistical challenge facing the AMC every year! Kudos to them for managing as well as they do.\r\n\r\nAccording to the published statistics, thousands of students don't even bother to bubble in their grade level on the AMC exams, and hundreds of students don't bubble in their grade levels on the AIME. Also, students with common names present challenges as well. (At our state MATHCOUNTS competition, there were ~200 students competing, and when the emcee read off the list of the top twelve students to call them down for Countdown, 14 students materialized at the front of the room, due to multiplicity of names.)\r\n\r\nSo any suggestions we can make can only mitigate the logistical challenge, not make it go away. I completely agree with Kent that the idea of SSN's is totally a non-starter.\r\n\r\n[quote]The only scheme I can see with a slight amount of hope: phone numbers. Put a place for a 10 - digit number, and ask student to fill in a number of their own choosing that they will remember, with the suggestion that it be a phone number (but the student is allowed to invent some other number).[/quote]\r\n\r\nHow about including a field for date of birth month/day/year on the AMC and AIME answer sheets, as a supplement to the phone number?\r\n\r\nThe date of birth should at least be easy for students to remember and replicate consistently each time. The phone number is a good idea because there may be some occasional instances when the easiest way for the AMC to get some piece of information is just to call and ask.\r\n\r\nAlso, how about designing the AIME answer form to ask the student about both their A date and B date scores, along with a place for the student to bubble in a field to designate if one of the AMC scores comes from a different location than the place where s/he is taking the AIME? \r\n\r\nAgain, these suggestions are only designed to make life a little bit easier for the AMC. I am completely in awe of their ability to manage the process as quickly as they do.\r\n\r\nOne more thought: perhaps there could be a special incentive for complete, consistent, and correctly filled out answer sheets. \r\n\r\nEach year, students whose AMC and AIME forms provide clear, consistent, and correct information that cause no problems for the AMC staff could be eligible for a drawing for a small prize, for example, a subscription to the MAA publication, [url=http://www.maa.org/mathhorizons/]Math Horizons[/url]. Students whose AIME forms wound up in the \"problem pile\" would not be eligible for the drawing.\r\n\r\nAlternatively, the incentive could operate at a school level. Schools that had no students winding up in the \"problem pile\" could be eligible for a drawing for a bulk subscription of Math Horizon magazines.\r\n\r\nThe drawing could be done in late summer, when the AMC is not quite so busy as at other times of year. The winning school could get recognized on the AMC website and in the MAA Messenger as a way to remind contest managers and students to be careful with their data entry.\r\n\r\nI have previously donated to the MAA to recognize the hard work done by the AMC Lincoln staff--there's a brick in the [url=http://www.maa.org/development/riverofbricks.html]MAA headquarters \"River of Bricks\" walkway[/url]celebrating the AMC staff, with a special shout-out to Donita Bowers, who retired a couple years ago.\r\n\r\nI would be happy to make an additional donation to fund subscriptions to a great magazine as a small incentive to students who do their part to make the AMC staff's work just a little bit easier.", "Solution_16": "[quote=\"timwu\"]A possible modification: if one choose to take the second AMC, then he or she automatically forfeits the first AMC score. It would be best if the first AMC score comes out before this decision is made.[/quote]\nCorrection: this should only be applicable if the first score comes out before the second test. You need the information first. What if you got better on the first one and didn't know it until after you did worse on the second one? It's too risky to do it without the information (and isn't in the spirit of the AMC. I don't think it will promote dishonesty either).\n\n[quote=\"sophia\"]The date of birth should at least be easy for students to remember and replicate consistently each time. The phone number is a good idea because there may be some occasional instances when the easiest way for the AMC to get some piece of information is just to call and ask. [/quote]\r\nThis is an extremely good idea. They do this on the Euclid contest (not sure about other CEMC contests, but alright) as well.", "Solution_17": "[quote=\"sdkudrgn88\"][quote=\"zmli\"]A better solution is to modifying the answer sheets for students to write in their CEEB# and scores earned at the specific CEEB location:\n\nAMC 10/12A: CEEB#, AMC10 or AMC 12, your score\nAMC 10/12B: CEEB#, AMC 10 or AMC 12, your score\n\nIt will make the automatic matching easier.[/quote]\n\nProblem is, people from International locations don't have a CEEB. (Or, could you explain what a CEEB is, if I'm wrong?)[/quote]\r\n\r\nThis is not a problem. AMC office automatically assign a test site a CEEB number if they don't have one. So, every test site has a CEEB number as far as AMC is concerned.", "Solution_18": "The problem is that the CEEB number of the location where the student took a previous exam is not something that most students carry around in their heads. The diligent ones will write it down and carry a note with them, but if you could trust everyone to do that, you could go ahead an assign a unique identifier to each individual. \r\n\r\nStudents don't always remember their previous scores correctly, either. I'm sure the AMC policy about that is \"Thanks for the hint, but if you don't mind, we'll look that up ourselves.\"", "Solution_19": "For starters, why not just print a warning message?\r\n\r\n\"If you are also taking the AMC 10/12B, or if you qualify for the AIME, note that [b]you must give exactly the same name on each contest that you participate in[/b].\"\r\n\r\nAnd similarly for each contest.", "Solution_20": "[quote=\"worthawholebean\"]Personally, I really like the idea of restricting the AMC to one taking.[/quote]\r\n\r\nI certainly agree with you relative to taking 2 AMC-12s or 2 AMC-10s. Getting two bites at the apple here doesn't seem to make things more fair for anyone.\r\n\r\nHowever, given all the fuss on these forums about whether underclassmen should take the AMC-12 or the AMC-10 (maximizing AIME chances (and taking a more challenging test) vs. potentially maximizing USAMO chances), I think a great solution there is to have the gifted underclassmen take 1 AMC-12 test and and 1 AMC-10 test. I realize this won't work for everyone (e.g. school doesn't/won't offer both test dates, money concerns, etc.) and some will have to choose (or have their school choose for them), but it does present a remedy for a large number of the borderline cases. I wouldn't want to give this up unless there is a good solution to the other problem that doesn't compare AMC-10 scores as equal to AMC-12 scores for the underclassmen USAMO index calculation.\r\n\r\nP.S. On a 'rabbit-trail' side note, I went through the system in the early 90s where we had 1 ASHME and 1 AIME (no alternate date tests, and no AMC-10), and consideration to underclassmen wasn't given until MOP selection. I find it a bit ironic that the process has certainly bent over backwards to afford underclassmen a lot of opportunity (a whole AMC-10 test, a generous portion of 'underclassmen-only' slots in the USAMO, and the large number of MOP spots reserved for freshmen and underclassmen), yet we spend time on the forums debating how the process isn't fair to those underclassmen. I'm not suggesting the debate isn't warranted, but there is a level of irony here that is quite inescapable.", "Solution_21": "I'm pretty sure we're debating on how it's not fair to the upperclassmen, not the other way around.", "Solution_22": "No, I thought everyone was saying that some underclassmen are kicked out of the second cut because of large amount of underclassmen who take the 10 instead of the 12. The upperclassmen actually get the advantage of not having to compete with many underclassmen.", "Solution_23": "[quote]\ncould you explain what a CEEB is\n[\\quote]\n\nCEEB stands for College Entrance Exam Board, now just known as the College Board. They are the folks behind the SAT, the AP Exams and several other things. They have a 6-digit code for each high school in the US, the CEEB number, that helps identify their information. In the 1980s and 1990s when the AMC went to centralized scoring, we adopted the numbers as a way to uniquely identify high schools. With many home schools, math circles, colleges and universities and schools outside the US now administering the AMC contests, we have created many new numbers, so the \"CEEB\"s that many of you use do not actually exist at the CB, just at the AMC.\n\n[/quote][quote]\nperhaps there could be a special incentive for complete, consistent, and correctly filled out answer sheets. \n[\\quote]\n\nInteresting idea. We'll explore it. Philosophically, I like positive incentives rather than negative enforcements. Might be worth considering.\n\n[quote]I have previously donated to the MAA to recognize the hard work done by the AMC Lincoln staff--there's a brick in the MAA headquarters \"River of Bricks\" walkway celebrating the AMC staff, with a special shout-out to Donita Bowers, who retired a couple years ago. \n[\\quote]\n\nThanks! Next time I'm in Washington at MAA HQ, I'll take a picture of it, and we'll post it at the website.\n\n\n\n[quote]\nfield for date of birth month/day/year on the AMC and AIME answer sheets, as a supplement to the phone number\n[/quote]\n\nThe date of birth and the phone number are both possibly feasible ideas. We will consider using them. I wonder about students outside the US, where phone numbers can be variable length.....\n\nOne thing to balance here is that we try to collect as little personal information as reasonably possible about each student. That's better for you, and and better for us. This maybe a place where we reasonably have to increase what we collect.\n\n[quote]For starters, why not just print a warning message?\n\n\"If you are also taking the AMC 10/12B, or if you qualify for the AIME, note that you must give exactly the same name on each contest that you participate in.\" \n[/quote][/quote][/quote][quote][/quote]\r\n\r\nWe will consider adding some kind of warning like that, perhaps in the instructions in the Teachers' Manual.\r\n\r\nOur experience is that such warning messages are ignored. Most students seem to be focused on solving the problems and getting the responses bubbled in, not on studying detailed instructions on the answer sheet. And that's okay, I understand that. \r\n\r\nIn any case, the students who did not follow the instructions to use the same name each time would still write or call us, and ask that we match their scores. In that case, we might not have a significant increase in efficiency. \r\n\r\nWe work about 12-24 months in advance planning for contests. We might be able to make these changes for 2010, more likely for 2011.\r\n\r\nSteve Dunbar\r\nMAA Director, American Mathematics Competitions", "Solution_24": "I think the date of birth/phone number method is the best idea here so far. I don't think people will mind having that extra personal information collected as long as they know it's only being used to match up tests. And international phone numbers shouldn't be a problem--just allow the phone number to have variable length.\r\n\r\nIt still leaves room for ambiguity, though--you'd have to remember if you put your home or cell number. And international students, I suppose, would have to remember if they included country code and all that.", "Solution_25": "[quote=\"m1sterzer0\"]P.S. On a 'rabbit-trail' side note, I went through the system in the early 90s where we had 1 ASHME and 1 AIME (no alternate date tests, and no AMC-10), and consideration to underclassmen wasn't given until MOP selection. [/quote]\r\nIn the late 90's, there was only 1 ASHME and 1 AIME, but there was definitely a multi-tiered selection system. A main selection (\"first cut\") with everyone eligible, followed by a few more 11th grade and below, followed by a few more 10th grade and below, followed by a few more 9th grade and below.\r\n\r\nI still find that to be an attractive scheme, provided there is a single basis of selection for all of these tiers, such as \"M.M. index.\"\r\n\r\nOf course, when [i]I[/i] went through the system (1969-71), there was no AIME and no USAMO. Just the AHSME. And the U.S. did not participate in the IMO - that was something that communist countries in Eastern Europe did.", "Solution_26": "I think the change might be at 1996- that's when the one qualifier per state rule was introduced, at least. Since then, the AMC has had various systems, all of which made it easier to qualify as an underclassman." } { "Tag": [ "Stanford", "college" ], "Problem": "So an interesting idea hit me the other day. What if we as members of AOPS got together and formed a global secret society where we aspire to take over the world....muhuhahaha. Think about it for a second; AOPS consists of perhaps some of the most intelligent and gifted young students in the world, and we could do some great things. The society could consist of different chapters in various regions, with some sort of a hierarchy in which each region would have a head person and some sub-leaders. We could have virtual meetings online via webcam/webchat (perhaps Skype?) where all of the leaders would meet each week online to discuss the \"operations\" of the society and to see how each region is doing. It could be quite the organization. We could even design some sort of a stock market operation for investments and whatnot. What do you guys think of the idea?", "Solution_1": "hey neato I call dibs on ultimate world leader okay", "Solution_2": "hm.we probably could talk about random stuff for 5 seconds and then get bored and then go do the same on FTW...", "Solution_3": "how much time did you spend planning this thread and thinking about all the e-props that you were going to get for it?", "Solution_4": "imo you need a life\r\n\r\nthe fact that you posted it in round table kinda implies that you're taking this seriously\r\n\r\nwhich... is a scary thought", "Solution_5": "Haha, no not at all. I just found it to be quite an amusing idea. Believe me, I definitely have a life ;)", "Solution_6": "[quote=\"Lazarus\"]imo you need a life\n\nthe fact that you posted it in round table kinda implies that you're taking this seriously\n\nwhich... is a scary thought[/quote]\r\n\r\nThis isn't round table. This is Games & Fun Factory.", "Solution_7": "It was originally posted in Round Table.", "Solution_8": "Yeah, that was my bad. I was unsure of which one I should post it in. Obviously I chose the wrong one though.", "Solution_9": "[quote=\"MysticTerminator\"]hey neato I call dibs on ultimate world leader okay[/quote]\r\n\r\nHow did I know that MysticTerminator was going to be the first person to say this?", "Solution_10": "lol I'll join the ring... I call Planner!", "Solution_11": "I'll join ring... I call [i][b]Whatever[/b][/i].", "Solution_12": "I'll join, I'll ask people in EPGY OHS (gifted online high school run by stanford) to join :)", "Solution_13": "EHEM! this is [size=200][u][i][b][color=darkred]AOPS[/color][/b][/i][/u][/size] secret society!!! :mad:\r\n\r\nI join! I can be the entering platform to Latin America", "Solution_14": "I call snacks provider. Anyone care for a cookie?", "Solution_15": "[quote=\"mathblitz\"]i join as prez advisor me is so not smart! o and lol googlism is so funny!id join but my parents wont let me[/quote]\r\n\r\nJust join it secretly.", "Solution_16": "um... wait so what was the original purpose of this discussion again???\r\n\r\nanyways i is ok with revealing who i really am to the rest of the secret society. like who i am other than an anime-obsessed computer freak.", "Solution_17": "I was showing how great a religion Googlism was :)", "Solution_18": "yes, i completely agree.", "Solution_19": "hm...well, I guess I'll be the person who pretends not to be a part of this, therefore allowing me to retrieve outside information from certain connections... =D \r\n\r\njk, I'll do whatever.", "Solution_20": "An AOPS Secret Society FAILED!!!!!!!!\r\n\r\nIt is supposed to be SECRET. This is NO LONGER SECRET. So THIS FAILED!!! QED\r\n\r\nSorry", "Solution_21": "How do YOU know it's not secret...\r\n\r\nYou aren't in the society...", "Solution_22": "anyway, i also proclaim myself church co-minister (with chenhsi) and that the official religion is Googlism. Usually im an atheist, but googlism sounds good. :D", "Solution_23": "No one else wants to join?", "Solution_24": "i think if we got any more members it REALLY wouldnt be secret any more...", "Solution_25": "I join! But considering the fact that I don't exist, this secret organization is still a secret, right? :roll:", "Solution_26": "[quote=\"MysticTerminator\"]hey neato I call dibs on ultimate world leader okay[/quote]\r\n\r\nOMG THAT IS NOT ARNAV HE IS NOT POSTING IN CAPS OMG HELP ARNAV!\r\n\r\nAlso, this is a dumb idea, since some people *cough*me*cough* live in the middle of nowhere, and the nearest user isn't even in their county.", "Solution_27": "um actually same here, don't get depressed\r\nunless you count all the ppl in seattle\r\nbut there still in a different county", "Solution_28": "[quote=\"splatyango\"]um actually same here, don't get depressed\nunless you count all the ppl in seattle\nbut there still in a different county[/quote]\r\nHey! What about me?!! (I'm her brother)\r\nyeah........", "Solution_29": "oh sry lol :blush: \r\n\r\nbut u dont come on often, excluding today\r\n\r\nso yeah.\r\n\r\nEDIT: putting subjects is ok, but most ppl dont, so unless you have a really important point, i would advise you not to..." } { "Tag": [ "search" ], "Problem": "What is the sum of the digits of the sum of the digits of the sum of the digits of $ 4444^{4444}$", "Solution_1": "Let $ A=4444^{4444}< 4444^{10000}$\r\nThen $ A$ has less than $ 4444*\\lg10000 = 17776$ digits.\r\n$ \\implies \\text{the sum of digits of A}< 9*17776 = 159984$\r\nSo the sum of the digits of the sum of digits A is less than $ 9*6 = 54$\r\nThen X is the sum of the digits of the sum of the digits of the sum of digits A $ \\leq 4+9 = 13$\r\nWe know that $ X \\equiv A \\equiv (-2)^{4444}\\equiv 2^{3*1481+1}\\equiv-2 (mod 9)$\r\nThat means $ X=7$\r\n\r\nHope I don't make any mistakes.", "Solution_2": "http://www.artofproblemsolving.com/Forum/viewtopic.php?search_id=1299623712&t=133303\r\n\r\nPlease, if you're going to insist on posting so many questions, at least check to make sure they haven't been posted before.", "Solution_3": "[quote=\"bibobeo\"]Let $ A = 4444^{4444}< 4444^{10000}$\nThen $ A$ has less than $ 4444*\\lg10000 = 17776$ digits.\n$ \\implies\\text{the sum of digits of A}< 9*17776 = 159984$\nSo the sum of the digits of the sum of digits A is less than $ 9*6 = 54$\nThen X is the sum of the digits of the sum of the digits of the sum of digits A $ \\leq 4+9 = 13$\nWe know that $ X\\equiv A\\equiv (-2)^{4444}\\equiv 2^{3*1481+1}\\equiv-2 (mod 9)$\nThat means $ X = 7$\n\nHope I don't make any mistakes.[/quote]\r\n\r\nWhere did you get $ X\\equiv A\\equiv (-2)^{4444}$ from?", "Solution_4": "Since $ 4446$ is a multiple of $ 9$, $ 4444$ must be equivalent to $ 7$, or $ \\minus{}2\\text{ mod }9$. Raising both $ 4444$ and $ \\minus{}2$ to the power of $ 4444$, we must obtain that the results would still be congruent $ \\text{ mod }9$. So $ 4444^{4444}\\equiv (\\minus{}2)^{4444}\\text{ mod }9$. \r\n\r\nConsidering the numbers mod 9 in these types of problems is very helpful, because if you keep taking the sum of digits till there is only one digit left, you get the [i]digital root[/i], which is constant mod 9." } { "Tag": [ "\\/closed" ], "Problem": "[url]http://www.artofproblemsolving.com/Forum/profile.php?mode=viewprofile&u=66008[/url]\r\n\r\nIt seems like this person is an impersonator of the user r15s11z55....(something) or some existing user's multi. Suspicions/thoughts?", "Solution_1": "Please pm an admin directly about these.\r\n\r\nThis user is just bored and has nothing better to do. Please ignore him." } { "Tag": [ "function", "trigonometry", "algebra", "functional equation", "real analysis", "real analysis unsolved" ], "Problem": "Prove or disprove the following:\r\nlet f,g be real functions s.t g(x-y)=g(x)g(y)+f(x)f(y) and g(0)=1 then g'(x)=-f(x)f'(0) & f'(x)=g(x)f'(0)", "Solution_1": "$f(x)=\\sin x$ \r\n$g(x)=\\cos x$", "Solution_2": "Yes, but can that be concluded from the functional equation (assuming continuity or measurability), for instance by constructing an additive function from $f$ and $g$? Some trigonometric equations are available, such as $f^2 + g^2 = 1$ and the angle-doubling formula for $g(x)$, but it is not immediately clear whether e.g. $f + ig$ is a homomorphism.", "Solution_3": "Let $\\alpha\\left(x\\right)$ be any function satisfies $\\alpha\\left(x+y\\right)=\\alpha\\left(x\\right)+\\alpha\\left(y\\right)$, then $f\\left(x\\right)=\\sin\\left(\\alpha\\left(x\\right)\\right),\\ g\\left(x\\right)=\\cos\\left(\\alpha\\left(x\\right)\\right)$ is a solution, which is not necessarily continous.", "Solution_4": "Yes, but given $f$ and $g$ can one construct a solution of $\\alpha(x+y) = \\alpha(x) + \\alpha(y)$?\r\nThis is the point mentioned above. \r\n\r\nThis problem is relatively easy if also given the functional equation for $f(x+y)$. Given only the equation for $g(x+y)$, is it possible?", "Solution_5": "You're so greedy. The Captain wrote f'(0) in his post, so f'(0)=k exists, we're able show $f\\left(x\\right)=\\sin\\left(k x\\right),\\ g\\left(x\\right)=\\cos\\left(k x\\right)$.", "Solution_6": "I interpreted the problem as to prove or disprove that the functional equation implies the differentiability and the formulas for $f', g'$. Nowhere is it stated that $f', g'$ are assumed to exist, it makes things easier to assume that, but it makes things more interesting to not assume that.", "Solution_7": "fleeting_guest is right. \r\nGiven g(x-y)=g(x)g(y)+f(x)+f(y) where f,g are real functions can we say that \r\n(( g(0)=1 )) & ((there exists an a s.t f(a)=1 & g(a)=0)) are equivalent ?\r\nI think this will help a lot.", "Solution_8": "The condition on $g(0)$ or $f(0)$ is kinda redundant, we can derive this from $g(x-y)=g(x)g(y)+f(x)f(y)$ except for some trivial case, in which $f,\\ g$ are constant functions.\r\nSince we can find solution $g(x)\\equiv 1,\\ f(x)\\equiv 0$, thus $g(0)=1$ and $\\exists a$ s.t $f(a)=1,\\ g(a)=0$ are not equivalent.\r\nIn order to get the differentiability, we need some extra conditions, for example, the existence of $f'(0)$.", "Solution_9": "[quote=\"fleeting_guest\"]Yes, but can that be concluded from the functional equation (assuming continuity or measurability), for instance by constructing an additive function from $f$ and $g$? Some trigonometric equations are available, such as $f^2 + g^2 = 1$ and the angle-doubling formula for $g(x)$, but it is not immediately clear whether e.g. $f + ig$ is a homomorphism.[/quote]\r\nI have a basic question.\r\nwhat is the f+ig ? :?:", "Solution_10": "$f+ig = f + g\\sqrt{-1}$ is a homomorphism (from $\\mathbb{R}^\\ast \\to \\mathbb{C}$) if $ (f(x) + ig(x))(f(y) + ig(y)) = f(x+y) + ig(x+y)$. This would be true if $f = \\cos (kx), g = \\sin (kx)$.", "Solution_11": "[quote=\"fleeting_guest\"]$f+ig = f + g\\sqrt{-1}$ is a homomorphism (from $\\mathbb{R}^\\ast \\to \\mathbb{C}$) if $ (f(x) + ig(x))(f(y) + ig(y)) = f(x+y) + ig(x+y)$. This would be true if $f = \\cos (kx), g = \\sin (kx)$.[/quote]\r\n\r\nRight ;but the captain said that f & g are real functions :!: :huh:", "Solution_12": "If $f$ and $g$ are real functions, $f+ig$ is a complex function. This complex function is often a useful thing to consider when solving problems about real $f,g$ satisfying functional equations similar to those of $\\sin$ and $\\cos$.", "Solution_13": "[quote=\"Buffalo\"]Since we can find solution $g(x)\\equiv 1,\\ f(x)\\equiv 0$, thus $g(0)=1$ and $\\exists a$ s.t $f(a)=1,\\ g(a)=0$ are not equivalent.\r\nBuffalo!Is'nt it fairly clear that in the case of constant functions the question is trivial?" } { "Tag": [], "Problem": "cho da thuc$P(x)$ he so nguyen thoa man :\r\n $P(2006)=2006!$\r\n $xP(x-1)=(x-2006)P(x)$ voi moi x thuoc $R$\r\n CMR: $G(x)=P(x)^2 +1$ bat kha qui trong$Z[x]$", "Solution_1": "De dang chung minh duoc rang:$ P(0)\\equal{}P(1)\\equal{}P(2)\\equal{}...\\equal{}P(2005)\\equal{}0$.\r\nDat $ P(x)\\equal{}x(x\\minus{}1)(x\\minus{}2)...(x\\minus{}2005)Q(x)$, thay vao gia thiet ta co $ Q(x)\\equal{}Q(x\\minus{}1)$\r\nsuy ra $ Q(x)\\equal{}k$ voi k la hang so.\r\nMat khac $ P(2006)\\equal{}2006!$ nen suy ra $ P(x)\\equal{}x(x\\minus{}1)(x\\minus{}2)...(x\\minus{}2005)$\r\nTa da dua ve bai toan quen thuoc:\r\n\"Chung minh rang: $ R(x)\\equal{}x^2(x\\minus{}1)^2(x\\minus{}2)^2...(x\\minus{}n)^2\\plus{}1$ bat kha quy tren $ Z[x]$ voi moi n nguyen duong\"" } { "Tag": [ "AMC", "AIME", "USA(J)MO", "USAMO", "AMC 10" ], "Problem": "I'm just wondering...\r\ncoz there's just a \"small\" little chance for me to get enroll...\r\nlol...\r\nI can't say no chance...but I know the truth :P", "Solution_1": "There have been lots of discussion on this topic. Please check the American Mathematics Competitions board on this website.\r\n\r\nIf you're in 11th or 12th grade, then you will need a good index to qualify for the USAMO. Your index score is AMC + 10*AIME, and you qualify for the AIME by scoring at least 100 on the AMC 12 (AMC 10 = 120). It is in your best interest to do the best you can on the AMC 12.\r\n\r\nHowever, if you're in 10th grade or below, then just getting a 100 on the AMC 12 or something will do. Just make sure you do your best on the AIME.\r\n\r\nI believe I've answered your question?\r\n\r\nBottom line, do well on the AMCs.", "Solution_2": "Does she mean if the AMC is good for CMO?\r\n\r\nIf that is what she asks I want to know the answer too.", "Solution_3": "I mean I may never can get enroll to the Olympiad\r\nso...what's the point of doing the AMC??", "Solution_4": "Sigh, alright. I'll give you some reasons.\r\n\r\nIdealistically: You enjoy doing Math and you want the opportunity to show all you've got, but also have fun and get a free pretty coloured pamphlet after the test.\r\n\r\nRealistically: A good AMC score can get the attention of college adcoms. Making the AIME would be ideal.\r\n\r\nSuper Idealistically: You never know, you just might make the USAMO. I know a kid who went from a 61 on the COMC last year to USAMO qualifier that same spring. Huge improvements can be made in a short amount of time; it just depends on how well you prepare I guess. And evidently, Canadians are known very much so for improving vastly in very short amounts of time.\r\n\r\nAs for the COMC, if you didn't score about a 69 or a 68 this time...strive for a better future! :)", "Solution_5": "I wish it can be anything to canadian universities....", "Solution_6": "since I've registered :P \r\n\r\ngoal for this yr is 152 hope it's not too hard for me :huh:", "Solution_7": "[quote=\"british8985\"]since I've registered :P \n\ngoal for this yr is 152 hope it's not too hard for me :huh:[/quote]Oh 152....", "Solution_8": "too low??\r\nthat's the best I can do I think..", "Solution_9": "[quote=\"british8985\"]too low??\nthat's the best I can do I think..[/quote]Uh, no.\r\nIsn't 150 the perfect score? :?", "Solution_10": "I think she means 152 index, AMC + 10*AIME score. That's not a bad goal.", "Solution_11": "for me, i am wondering what is the right goal for a scarcely educated 12th grader, gone through little math study, first time writing AMC? :P", "Solution_12": "maybe...130~140\r\n\r\ncoz I'm in gr11..so..." } { "Tag": [ "geometry", "trapezoid" ], "Problem": "Find the area of trapezoid ABCD if AB || CD, AB = 24, CD = 12, the measure of angle A is 30 and the measure of angle B is 60.", "Solution_1": "[hide=\"ans\"]\nwe need to find the height of the trapezoid and in order to do that we need to find the side length of one of our 30-60-90 triangles\n\nlet x and y equal the top lengths of the two triangles, x is for the angle 30, and y the angle 60\n\n$x+y = 12$ and $x\\tan30 = y\\tan60$\nthus,\n\n$x\\tan30 = (12-x)\\tan60$\n\n$x\\tan30 = 12\\tan60-x\\tan60$\n\n$x(\\tan30+\\tan60) = 12\\tan60$\n\n$x = \\frac{12\\tan60}{\\tan30+\\tan60}= 9$\n\nNow we can find the height of our trapezoid, using the 30-60-90 theory\n\nsince x = 9 and this side is $\\sqrt{3}$ in the theorem, then it has been mutliplied by $3\\sqrt{3}$, thus the height of the trapezoid is $3\\sqrt{3}$\n\nOR, we use the fact that $x\\tan30$ = the height of the trapezoid which comes out to 5.196 which is exactly the same as $3\\sqrt{3}$\n\ncomputing the area is now easy\n\n$area = 3\\sqrt{3}* (\\frac{24+12}{2})= 93.53$\n[/hide]" } { "Tag": [ "LaTeX" ], "Problem": "Jim has a certain amount of people in front of him. First he calculates the number of ways he can take six people from the group in front of him. Then he calculates the number of ways he can take ten people from the group in front of him. He then realizes that the number of ways he can take six or ten people are exactly the same. How many people are in front of him?", "Solution_1": "[hide]$16!/10!/6!=16!/6!/10!$\n\n$16?$[/hide]", "Solution_2": "[hide=\"hint\"]\nRemember that $\\binom nr=\\binom{n}{n-r}$[/hide]", "Solution_3": "eheehee is right\r\n[hide=\"solution\"]Take the mean of 10 and 6\n8 is the center number. Doubling it brings [b]16[/b][/hide]", "Solution_4": "The correct answer is...\r\n\r\n[hide=\"Solution\"]\n(n r) = (n n-r)\n\n(n 6) = (n 10)\n(n 6) = (n n - 10)\n6 = n - 10\nn = [b]16[/b]\n[/hide]\r\n\r\n(sorry about the lack of latex)" } { "Tag": [], "Problem": "Solve for $ x: 3^{2x} \\plus{} 19 \\equal{} 10^x$.", "Solution_1": "look at it. x=2. simplest way to do it.", "Solution_2": "[quote=\"nikeballa96\"]look at it. x=2. simplest way to do it.[/quote]\r\n\r\nI seriously do not see the simplicity of brute force/guessing here. How would you know to try x=2?", "Solution_3": "$ 3^{2x}\\plus{}19\\equal{}10^x\\implies9^x\\plus{}19\\equal{}10^x$", "Solution_4": "try multiples of 10. 10 gives a negative value for 3^2x. 10^2 gives an answer. also try math's method.", "Solution_5": "/bump can anyone explain how to get this? Logically.\n", "Solution_6": "[hide=One Solution]\nWe have $3^{2x}+19=10^x\\implies 9^x+19=10^x$. Suppose $x=2y$. The equation becomes $9^{2y}+19=10^{2y}$. Subtracting $9^{2y}$ over and factoring, we get $19=10^{2y}-9^{2y} \\implies 19=(10^y-9^y)(10^y+9^y)$. Since $19$ is prime, the only two positive integers that multiply to equal $19$ are $1$ and $19$. Since $1<19$, we have $10^y-9^y=1$ and $10^y+9^y=19$. It is clear that $y=1$ satisfies both of these, and since $x=2y=2(1)=\\boxed{2}$. \nWe can check this by plugging in $x=2$ to the original equation: $3^4+19=10^2 \\implies 81+19=100$, which is true. [/hide]" } { "Tag": [ "percent", "geometry", "geometric transformation", "reflection", "vector", "email", "search" ], "Problem": "just about 4 days left for the exam\r\n\r\ni m a bit confused abt cut offs\r\n\r\nLAst yr total marks 300 (part A-180,PART B-120)\r\nthis yr total marks 240 (part A-180,PART B-60)\r\n\r\nsome 1 told me cut offs were 120-130 and 150 safe zone\r\nwell was it out of 180 :P , 240 or 300 :o", "Solution_1": "What about NSEJS?", "Solution_2": "check on hbcse", "Solution_3": "do u think HBCSE wud publish cutoffs?? :( \r\n\r\nofc i forgot NSEJS and NSEA :P", "Solution_4": "i presume it would", "Solution_5": "i had a doubt regarding NSEP \r\nis it that we have $ 2$ and a $ \\frac {1}{2}$ hours for both the sections A and B :( or only section A", "Solution_6": "u r giving the exam?? :oops: \r\n\r\nsorry to disappoint u but we hav just 2 hrs ....for both section A and B", "Solution_7": "and that too difficult questions", "Solution_8": "best of luck to everybody giving the exam :) \r\n\r\n@rt:these exams are for students of class 12th,not for 11thies :rotfl:", "Solution_9": "wat ??????????\r\nNSEP AND NSEC ARE FOR XII?????????????/", "Solution_10": "ofc eleventhies lik u and skand will get thru the 1st round :)", "Solution_11": "all the best to all for the test going on...................... :P :rotfl: \r\n\r\nsrry for the late wishes :maybe:", "Solution_12": "well how many of you wrote NSEA????\r\nand what are you expecting???\r\nMine expected score is anything between 100 - 120", "Solution_13": "thats really great.. :thumbup: \r\n\r\nit became bad 2 worse frm me :(", "Solution_14": ",i answered 35+\r\n\r\nin end it all went in hurry\r\n\r\n@ritu,NSEP and NSEC were meant 4 timepass only 4 me\r\n\r\nhow did you all performed in them", "Solution_15": "congrats to those who are selected :) \r\n\r\nas expected I am not selected\r\n\r\nnever mind,i have next chance :D\r\nand one in INMO too :rotfl:", "Solution_16": "@anand...\r\nnice to see u dude\r\nthanks and grats to u\r\ni lost ur phone no :(\r\nand can u mail it to me?\r\nand i have ur arihant organice chem...didnt do a page :((\r\nbtw..why din ya write the last base exam??", "Solution_17": "[quote=\"sumanth_d\"]@anand...\n\nand i have ur arihant organice chem...[b]didnt do a page[/b] :((\n[/quote]\r\n\r\n\r\neven i havent done a page.......and i dont plan to do it also :P", "Solution_18": "Here is the site for chem papers. They are Icho but icho and incho are not very different.\r\n[url=http://nhb.topcities.com/Chemclub/Icho.html]IChO[/url]\r\n\r\n[hide]Made NSEC, didnt give astro, and failed in nsep[/hide]\n\n[hide=\"@Da Vinci\"]Don't give up on chem da, especially organic chem. I am sure you also are aware of your talent in chem. [/hide]", "Solution_19": "@ritu\r\nDo not get confused betn ICO, INChO and INCO. The first one stands for International Chemistry Olympiad and rest two stands for Indian Chemistry Olympiad though INChO is more frequently used. Got it????", "Solution_20": "i not getting confused neways :P ...there r millions of exceptions in chem :rotfl: \r\n\r\nbut just cant believe how i got thru Chem while other JOI'ians didn't :maybe:", "Solution_21": "Can any1 plz tell the following details of the INCho paper ???? \r\n1. Total marks\r\n2. Subjective or objective\r\n3. Any negative marking process followed\r\nPlz. give me at least some idea. I'm totally blank!!!!", "Solution_22": "Do the same for INPhO :D :)", "Solution_23": "Is any1 interested in buying the book/s provided by HBCSE with contains the theory problems of past years INPho and INCho ? Btw what are your views regarding open tests held by various institutes like Fiitjee, Resonance, etc. for IITJEE???", "Solution_24": "I just got my NSEP certificate! :rotfl: Statewise 1% thing.\r\nAnyone getting it so late?\r\nEdit: do they send 11th and 12th certificates separately?", "Solution_25": "i got to know that i have cleared neep/c/a only after the results of ino's were out[the equivalent ino concept] :rotfl: i don't think there can b anything funnier than this", "Solution_26": "[quote=\"VP_91\"]i got to know that i have cleared neep/c/a only after the results of ino's were out[the equivalent ino concept] :rotfl: i don't think there can b anything funnier than this[/quote]\r\nOh yeah! In comparison this is nothing....", "Solution_27": "I havent got NSEP certificate yet :rotfl: \r\n\r\nand there was 1 more funnier thing\r\nTuhin got 71.5 in INChO(initially)\r\nHe didnt send the paper 4 reevaluation\r\nThe cutoff was written 72.5\r\nStill his name was there on the list as selected :rotfl:", "Solution_28": "@rituraj\r\nall papers were rechecked due to mistake in q7.8 soln\r\n\r\neven i got a new performance card without sending for rechecking\r\n\r\ni got also nsec certificate\r\n\r\n[hide]\ni want to tear it to pieces burn it and drink the ashes \nolympiads are a big lie :furious: \nbeing top 1% in chem olympiad doesn't guarantee being in even top 10% in jee chem \n[/hide]", "Solution_29": "dont say u r not in top 10% in JEE chem :mad: \r\n\r\n3.98 lakhs sat for JEE\r\nthat means top 10% implies being in top 40,000.. :rotfl:" } { "Tag": [], "Problem": "Without dividing, determine the remainder when f(x) = x^3 + 5x^2 - x -21 is divided by x^2 + 4x - 10.", "Solution_1": "[quote=\"king_23\"]Without dividing, determine the remainder when \n $f(x) = x^3 + 5x^2 - x -21$ \nis divided by \n$x^2 + 4x - 10$ [/quote]\r\nsorry but i made it clearlier", "Solution_2": "[quote=\"inom\"][quote=\"king_23\"]Without dividing, determine the remainder when \n $f(x) = x^3 + 5x^2 - x -21$ \nis divided by \n$x^2 + 4x - 10$ [/quote]\nsorry but i made it clearlier[/quote]\n\nsolution,\n$x^2+4x^2-10=10$\n$x^2=10-4x$\n$F(x^2)=69-26x$ so\n$R(x)=69-26x$[/quote]", "Solution_3": "Sorry, I don't quite understand how you did solution. Your remainder is possible, but I don't understand how you derived it.", "Solution_4": "By division (as a check), I got 5x-1.\r\n\r\nBut I am intriqued by your method! More info?" } { "Tag": [ "calculus", "integration" ], "Problem": "Solve the following simultaneous equation for all positive integer $a,b,c$ \r\n\r\n$a+b+c=38$\r\n\r\n$a^2+b^2+c^2=722$", "Solution_1": "We see that $a^2+b^2+c^2=2ab+2ac+2bc$", "Solution_2": "I got that much, but where do you go from there?", "Solution_3": "Well from brute force I got 4,9,25 as a solution. Iono if its coincidence or not that those are all perfect squares themselves.", "Solution_4": "[quote=\"silouan\"]We see that $a^2+b^2+c^2=2ab+2ac+2bc$[/quote]\n\nya , silouan , can you show your solution ? ( I believe you wouldnt brute force it out ...)\n\n[quote=\"krustyteklown\"]\nWell from brute force I got 4,9,25 as a solution. Iono if its coincidence or not that those are all perfect squares themselves.[/quote]\r\n\r\nWell , I think it can be shown that they are all perfect square . :)", "Solution_5": "[quote=\"silouan\"]We see that $a^2+b^2+c^2=2ab+2ac+2bc$[/quote]\r\n\r\n$ab+bc+ca=361$\r\n\r\n$\\Rightarrow ab + (a + b)( - a - b + 38) = 361$\r\n\r\n$\\Rightarrow (a + b)^2 - 38(a + b) + 19^2 = ab \\Rightarrow (a + b - 19)^2 = ab$\r\n\r\n$\\Rightarrow (19 - c)^2 = ab$\r\n\r\nThis gives $(a - b)^2 = c(76 - 3c)$\r\n\r\nThen I tried for values of $c$ which make $c(76 - 3c)$ a perfect square.\r\n\r\nI found $c= 4, 9, 25$ (25 is maximum for c)are the only numbers which satisfy.\r\n\r\nHence the solution set is any permutation of the numbers $4, 9, 25$ and are $6$ in number.", "Solution_6": "well done [b]vidyamanohar[/b] :) I have a method to show that $a,b,c$ must be prefect square \r\n\r\nWLOG let $a\\geq b\\geq c$ . Since we know\r\n\r\n$a^2+b^2+c^2=2(ab+ac+bc)$ \r\n :arrow: $c^2-2c(a+b)+(a-b)^2$ which mean \r\n\r\n$c=(a+b)-2\\sqrt{ab}$ *we take minus sign here since $c0$ prove that\r\n$(2(b-c)^{2}+2a^{2}+bc)(2(c-a)^{2}+2b^{2}+ca)(2(a-b)^{2}+2c^{2}+ab)\\ge a^{2}+b^{2}+c^{2}\\ge ab+bc+ca$", "Solution_1": "[quote=\"skywalker\"]Given $a,b,c>0$ prove that\n$(2(b-c)^{2}+2a^{2}+bc)(2(c-a)^{2}+2b^{2}+ca)(2(a-b)^{2}+2c^{2}+ab)\\ge a^{2}+b^{2}+c^{2}\\ge ab+bc+ca$[/quote]\r\nWhat is this?", "Solution_2": "He must mean + on the left hand side which was left out in the latex, I'm guessing :wink:", "Solution_3": "Sorry it isn't homogeneous it must be \r\nGiven $a,b,c>0$ prove that\r\n$(2(b-c)^{2}+2a^{2}+bc)(2(c-a)^{2}+2b^{2}+ca)(2(a-b)^{2}+2c^{2}+ab)\\ge (a^{2}+b^{2}+c^{2})^{3}\\ge (ab+bc+ca)^{3}$\r\n :)", "Solution_4": "[quote=\"skywalker\"]\nGiven $a,b,c>0$ prove that\n$(2(b-c)^{2}+2a^{2}+bc)(2(c-a)^{2}+2b^{2}+ca)(2(a-b)^{2}+2c^{2}+ab)\\ge (a^{2}+b^{2}+c^{2})^{3}$\n [/quote]\r\nLet $a^{2}+b^{2}+c^{2}=1.$ Then your inequality equivalent to the following inequality:\r\n$(2-3ab)(2-3ac)(2-3bc)\\geq1,$ which was posted many time ago by Vasc or hungkhtn.\r\nMy ugly proof:$(2-3ab)(2-3ac)(2-3bc)\\geq1\\Leftrightarrow$\r\n$\\Leftrightarrow2\\cdot\\sum_{cyc}(a-b)^{6}+\\frac{3}{2}\\cdot\\sum_{cyc}(a-b)^{2}(a^{4}+2a^{3}b-3a^{2}b^{2}+2ab^{3}+b^{4})+$\r\n$+6\\cdot\\sum_{cyc}abc(a^{3}-a^{2}b-a^{2}c+abc)+\\sum_{cyc}(a^{3}b^{3}-a^{2}b^{2}c^{2})\\geq0.$", "Solution_5": "Yes thus you recognized it, I am finding a beautiful solution, I known solution of Vasc but it is same of you (about beauty) :D", "Solution_6": "[quote=\"skywalker\"]I am finding a beautiful solution...[/quote]\r\nIt's very interesting.", "Solution_7": "I found a solution but perhaps it's ugly as well (but it's fast for me 'cause I'm familar with such kind of ways using Schur Inequality); let\r\n$x=a+b+c,\\ y=ab+bc+ca,\\ z=abc$, and let\r\n$a^{2}+b^{2}+c^{2}=x^{2}-2y=1$\r\nthen we need to prove\r\n$(2-3ab)(2-3bc)(2-3ca)\\geq0\\Longleftrightarrow 7-12y+3\\cdot 3z(2x-3z)\\geq0$\r\nAnd we see that if\r\n$f(t)=t(u-t)\\ \\left(t\\leq\\frac{u}{2}\\right)$\r\nthen\r\n$f'(t)=u-2t\\geq0$\r\nSo with\r\n$x-3z=(a+b+c)(a^{2}+b^{2}+c^{2})-3abc\\geq6abc\\geq0$\r\nand with Schur Inequality\r\n$a^{3}+b^{3}+c^{3}+3abc\\geq ab(a+b)+bc(b+c)+ca(c+a)\\Longleftrightarrow z\\geq\\frac{4xy-x^{3}}{9}$\r\n(notice that $x^{2}=1+2y$) we have\r\n$7-12y+3\\cdot 3z(2x-3z) \\geq 7-12y+3\\cdot\\frac{4xy-x^{3}}{3}\\left(2x-\\frac{4xy-x^{3}}{3}\\right)=7-12y+\\frac{1}{3}(1+2y)(2y-1)(7-2y)=\\frac{1}{3}(1-y)(4y^{2}-10y+7)\\geq0$\r\n(because\r\n$y=ab+bc+ca\\leq a^{2}+b^{2}+c^{2}=1$)\r\nThen it's done.", "Solution_8": "I think, it's better:\r\n$(2-3ab)(2-3ac)(2-3bc)\\geq1\\Leftrightarrow$\r\n$\\Leftrightarrow7-12(ab+ac+bc)+18(a+b+c)abc-27a^{2}b^{2}c^{2}\\geq0.$\r\n$3(a+b+c)abc\\geq27a^{2}b^{2}c^{2}.$ Hence, remain to prove that\r\n$7-12(ab+ac+bc)+15(a+b+c)abc\\geq0,$ which is easily killed by SOS. :wink:", "Solution_9": "[quote=\"arqady\"][quote=\"skywalker\"]\nGiven $a,b,c>0$ prove that\n$(2(b-c)^{2}+2a^{2}+bc)(2(c-a)^{2}+2b^{2}+ca)(2(a-b)^{2}+2c^{2}+ab)\\ge (a^{2}+b^{2}+c^{2})^{3}$\n [/quote]\nLet $a^{2}+b^{2}+c^{2}=1.$ Then your inequality equivalent to the following inequality:\n$(2-3ab)(2-3ac)(2-3bc)\\geq1,$ which was posted many time ago by Vasc or hungkhtn.\nMy ugly proof:$(2-3ab)(2-3ac)(2-3bc)\\geq1\\Leftrightarrow$\n$\\Leftrightarrow2\\cdot\\sum_{cyc}(a-b)^{6}+\\frac{3}{2}\\cdot\\sum_{cyc}(a-b)^{2}(a^{4}+2a^{3}b-3a^{2}b^{2}+2ab^{3}+b^{4})+$\n$+6\\cdot\\sum_{cyc}abc(a^{3}-a^{2}b-a^{2}c+abc)+\\sum_{cyc}(a^{3}b^{3}-a^{2}b^{2}c^{2})\\geq0.$[/quote]\r\n\r\nYou are right, Arqady. More exactly, this problem is of VASC and in Hungkhtn's book (vietnamese), following nice solution are show:\r\n\r\nLet $a,b,c \\ge 0$ verify that $a^{2}+b^{2}+c^{2}=3$. Prove that\r\n\\[(2-ab)(2-bc)(2-ca) \\ge 1. \\]\r\nProof.\r\nLet $x=2-ab,y=2-bc,z=2-ca$. \r\nWe need to prove $xyz \\ge 1$ if $x,y,z$ are three positive numbers satisfying that\r\n\\[\\frac{(2-x)(2-y)}{2-z}+\\frac{(2-y)(2-z)}{2-x}+\\frac{(2-z)(2-x)}{2-y}=3. \\]\r\nThe above condition can be rewritten as follow\r\n\\[(2-x)(2-y)+(2-y)(2-z)+(2-z)(2-x)\\&=3(2-x)(2-y)(2-z)\\]\r\n\\[\\Leftrightarrow 8(x+y+z)+3xyz\\&=12+5(xy+yz+zx).\\]\r\nAssume that it's false with some numbers $x,y,z$, or $xyz <1$. We will prove\r\n\\[8(x+y+z)+3xyz <12+5(xy+yz+zx). \\]\r\nThis last one is easy by Mixing variable or SOS or Schur. Notice that we can assume $xyz=1$.", "Solution_10": "[quote=\"GacKiem\"]\n\\]\nAssume that it's false with some numbers $ x,y,z$, or $ xyz < 1$. We will prove\n\\[ 8(x \\plus{} y \\plus{} z) \\plus{} 3xyz < 12 \\plus{} 5(xy \\plus{} yz \\plus{} zx).\n\\]\nThis last one is easy by Mixing variable or SOS or Schur. Notice that we can assume $ xyz \\equal{} 1$.[/quote]\r\n\r\nI think it is wrong. :P", "Solution_11": "[quote=\"arqady\"][quote=\"skywalker\"]\nGiven $ a,b,c > 0$ prove that\n$ (2(b \\minus{} c)^{2} \\plus{} 2a^{2} \\plus{} bc)(2(c \\minus{} a)^{2} \\plus{} 2b^{2} \\plus{} ca)(2(a \\minus{} b)^{2} \\plus{} 2c^{2} \\plus{} ab)\\ge (a^{2} \\plus{} b^{2} \\plus{} c^{2})^{3}$\n [/quote]\nLet $ a^{2} \\plus{} b^{2} \\plus{} c^{2} \\equal{} 1.$ Then your inequality equivalent to the following inequality:\n$ (2 \\minus{} 3ab)(2 \\minus{} 3ac)(2 \\minus{} 3bc)\\geq1,$ which was posted many time ago by Vasc or hungkhtn.\n[/quote]\r\npqr kill it easily :maybe:" } { "Tag": [ "Divisibility Theory" ], "Problem": "Let $a$ and $b$ be positive integers. When $a^{2}+b^{2}$ is divided by $a+b,$ the quotient is $q$ and the remainder is $r.$ Find all pairs $(a,b)$ such that $q^{2}+r=1977$.", "Solution_1": "$ \\frac{a\\plus{}b}{2}\\le [\\frac{a^2\\plus{}b^2}{a\\plus{}b}]\\equal{}q0$ :)", "Solution_3": "The sequence $a_{n}$ is (up to a constant multiple) the sine coefficients of the Fourier series for the function $f(x)$ such that $f(x)=\\frac1{\\sqrt{x}}$ for $0 0$), wlog say $ f(0) \\equal{} \\minus{} 1$\r\n\r\nThen $ P(0,x)$ $ \\implies$ $ g(x) \\equal{} f(x) \\plus{} 1$ and $ P(x,y)$ becomes $ f(xy) \\equal{} f(x)f(y) \\plus{} f(x) \\plus{} f(y)$ and so $ f(xy) \\plus{} 1 \\equal{} (f(x) \\plus{} 1)(f(y) \\plus{} 1)$\r\n\r\nand so $ g(xy) \\equal{} g(x)g(y)$ with $ g(x)$ strictly increasing and so $ g(x) \\equal{} x^a$ with $ a > 0$ and such that $ x^a$ is an increasing function defined even for $ x < 0$ \r\n\r\nAnd it is easy to check back that this indeed is a solution.\r\n\r\nHence the result :\r\n\r\n$ f(x) \\equal{} \\alpha(x^a \\minus{} 1)$\r\n$ g(x) \\equal{} x^a$\r\n\r\nFor any $ \\alpha > 0$ and any $ a > 0$ such that $ x^a$ is an increasing function defined even for $ x < 0$ (example : $ 2p \\plus{} 1, \\frac 1{2q \\plus{} 1}, ...$)\r\n\r\n(be careful that acceptance of $ x^a$ for $ x < 0$ when $ a\\notin\\mathbb N$ may vary in different countries. But $ f(x) \\equal{} \\sqrt [3] x \\minus{} 1$ is indeed a solution, for example).", "Solution_2": "[quote=\"NikolayKaz\"]It was supposed to be strictly increasing, i just mistranslated the task. I have a question regarding your solution. \n[quote=\"pco\"]\n$ g(xy) \\equal{} g(x)g(y)$ with $ g(x)$ strictly increasing and so $ g(x) \\equal{} x^a$\n[/quote]\n\nHow do we know that we can assume any strictly increasing function here? I mean what shows us that all strictly increasing functions g are the solution?[/quote]\r\n\r\nI dont understand your question.\r\n\r\n$ f(x)$ is increasing, so $ g(x)\\equal{}f(x)\\plus{}1$ is increasing too.", "Solution_3": "I understand this but my question is how do we know that it holds for ANY increasing function.", "Solution_4": "[quote=\"NikolayKaz\"]I understand this but my question is how do we know that it holds for ANY increasing function.[/quote]\r\n\r\n[u]Problem [/u]: $ g(xy) = g(x)g(y)$ and $ g(x)$ increasing.\r\nLet $ P(x,y)$ be the assertion $ g(xy) = g(x)g(y)$\r\n\r\nIf $ g(u) = 0$ for some $ u\\ne 0^$, $ P(u,\\frac xu)$ $ \\implies$ $ g(x) = 0$ $ \\forall x$. Impossible since $ g(x)$ is increasing. So $ g(x)\\ne 0$ $ \\forall x\\ne 0$\r\n$ P(x,1)$ $ \\implies$ $ g(x)(g(1) - 1) = 0$ and so $ g(1) = 1$ else $ g(x) = 0$ $ \\forall x$, in contradiction with $ g(x)$ increasing.\r\n$ P(x,0)$ $ \\implies$ $ g(0)(g(x) - 1) = 0$ and so $ g(0) = 0$ else $ g(x) = 1$ $ \\forall x$, in contradiction with $ g(x)$ increasing.\r\n$ P( - 1, - 1)$ $ \\implies$ $ 1 = g( - 1)^2$ and so $ g( - 1) = - 1$ since $ g(x)$ is increasing and $ g(0) = 0$\r\n$ P(x, - 1)$ $ \\implies$ $ g( - x) = - g(x)$\r\n\r\nLet then $ h(x)$ : $ \\mathbb R\\to\\mathbb R$ such that $ h(x) = \\ln(g(e^x))$ : $ h(x)$ is increasing and $ h(x + y) = h(x) + h(y)$\r\nThis is Cauchy's equation and the only monotonic solutions are $ h(x) = cx$ for any $ c > 0$ \r\n\r\nSo $ g(x) = x^c$ $ \\forall x > 0$ and $ g(x) = - ( - x)^c$ $ \\forall x < 0$\r\n\r\nAnd so [u][b]I was wrong : there are some other solutions [/b][/u]!\r\n\r\nThe general solution for $ g(x)$ is $ g(x) = sign(x)|x|^c$ for any $ c > 0$\r\n\r\nSo for example $ g(x) = sign(x)x^2$ is a solution (that I did not give in my previous posts).\r\n\r\nHence the result for the original problem :\r\n\r\n$ f(x) = \\alpha(sign(x)|x|^a - 1)$\r\n$ g(x) = sign(x)|x|^a$\r\n\r\nFor any $ \\alpha > 0$ and any $ a > 0$\r\n\r\nAnd thanks, NikolayKaz, for your question. :)", "Solution_5": "@pco sorry to revive... but how do you get $g(x)=x^a$ for some $a>0$ with $g(xy)=g(x)g(y)$ and $g$ monotonic?", "Solution_6": "I dont see where.\nWe are speaking, I suppose about post #5 (and not post #2)\nDont hesitate to quote the first two or three lines you dont understand.\n" } { "Tag": [ "LaTeX", "integration" ], "Problem": "Say I want to create a document with a lot of problems, each followed by a solution. I want to make it so that I can print\r\n(1) The document with both problems and solutions (interleaved) and\r\n(2) The document with only the problems, and the solutions taken out.\r\n\r\nNow, is there an easy way to do this WITHOUT having to create two LaTeX documents?\r\n\r\n\r\nI thought that something like\r\n\r\n\\newenvironment{solution}{ \\begin{comment} } {\\end{comment} }\r\n\r\nmight work, but it gives an error (try it yourself). So any other suggestions?", "Solution_1": "You could always type out the entire document, and then copy/cut/paste the solutions. I think it might be a better idea to just type out all the problems, start a new page, and then post the solutions.", "Solution_2": "I believe the [i]comment[/i] package (different from the command), the [i]version [/i]package, and the [i]versions[/i] package can do what you want. See\r\n[url]http://www.tex.ac.uk/cgi-bin/texfaq2html?label=conditional[/url].", "Solution_3": "Here's what's up I think with the comment package:\r\n\r\n1. First load the package:\r\n\r\n[code]\\usepackage{comment}\n[/code]\n2. Define inclusion and exclusion environment names. For example, suppose want to selectively include/exclude all the problems. Then define a \"problems\" environment name and add the command:\n\n[code]\\includecomment{problems} \n[/code]\nNow can create an environment like:\n\n[code]\\begin{problems}\nThis is problem 1:\n$$\\int_0^1 xdx$$\n\\end{problems}[/code]\n\nSince the environment \"problems\" is an included environment, this block of latex is included in the document.\n\n3. Now define exclusion environments. For example, define an \"answers\" environment:\n\n[code]\\excludecomment{answers}\n[/code]\nNow can place \"answers\" environments with the problems like:\n\n[code]\\begin{answers}\nThis is answer to problem 1\n\\end{answers}[/code]\n\nand since \"answers\" is an excluded environment, it won't be included in the document.\n\n4. If want to disable excluding all the answers and print them out, just comment out the \\excludecommand{answers}\n\n[code]%\\excludecommand{answers}[/code]", "Solution_4": "Perhaps another solution is to use test style file\r\nhttp://www.math.rochester.edu/people/faculty/cmlr/Latex-exam-ur/\r\nor\r\nanswers.sty\r\nmechanics.civil.tohoku.ac.jp/~bear/bear-collections/style-files/answers.sty\r\nor \r\n\r\nhttp://www-hep2.fzu.cz/tex/texmf-dist/doc/latex/probsoln/probsoln.html\r\n\r\nI hope that it help \r\n\r\nGenick Bar-Meir\r\nhttp://www.potto.org\r\nwhere free textbooks roam!" } { "Tag": [ "calculus", "integration", "real analysis", "real analysis theorems" ], "Problem": "From what I've seen in learning numerical analysis, it looks like a lot of numerical integration methods are based on the principle that you can use your k-step methods for solving ODEs to approximate Riemann integrals. This got me thinking about the possibility of a numerical Lebesgue integral but that sounds ridiculous. However, I don't see any obstruction to numerical Henstock-Kurzweil integration. I don't know anything about adaptive numerical methods but it sounds like this would be a natural framework for them. Has this ever been done or is it stupid?", "Solution_1": "In a way, this is already done by [url=http://en.wikipedia.org/wiki/Adaptive_quadrature]adaptive quadrature[/url] methods, which evaluate the integrand on a mesh that becomes denser near a singularity. \r\n[quote=\"Maple 10\"]evalf(Int((1/x)*sin(1/x^3),x=0..1));\n0.2082377521[/quote]" } { "Tag": [ "calculus", "integration", "function", "analytic geometry", "geometry", "calculus computations" ], "Problem": "Can anyone help me evaluate the following improper integral?:\r\n\r\n$\\int_{0}^{\\infty} x e^{-x^2} dx$\r\n\r\nI tried using integration by parts along with the error function but I am getting $\\infty$ for an answer. May be because I don't have much experience in using integration by parts for improper integrals.", "Solution_1": "Try substitution. What might be a good choice of a function to substitute? A common strategy is to split your integrand into $f(u)$ and $du$ so that $f(u)$ is something you know how to integrate and $du$ equals $dx$ times the leftover junk not included in $f(u)$.", "Solution_2": "Yeah, I did that. I had $u = x \\Rightarrow du = dx$ and $dv = e^{-x^2} dx \\Rightarrow v = \\int_{0}^{\\infty} e^{-x^2} dx$. Thus $v = \\frac{{\\pi}^{1/2}}{2}$ by the error function. But then you get $uv = x\\frac{{\\pi}^{1/2}}{2}$ evaluated from $0$ to $\\infty$ which is equal to $\\infty$!!", "Solution_3": "I think you want to make the substitution $u = x^2$. \r\n\r\nI don't mean integrating by parts, either. I mean plain old substitution. Try it, see what happens! :)", "Solution_4": "Oh Wow! I never realized that. I was unnecessarily trying to make the problem hard! :roll: \r\n\r\nWell, now that this problem is solved, what about this one:\r\n\r\n$\\int_{0}^{\\infty} x^2 e^{-x^2} dx$\r\n\r\nI don't think any u substitution would do the trick this time. :(", "Solution_5": "You're right. See the \"marathon\" thread -- we've been discussing a very similar problem.", "Solution_6": "I read the thread, but that just labels this function as non-elementary and moves on. I wanna know how you would use the error function to evalue this integral.", "Solution_7": "Ah, that's a commonly discussed integral. Have you had calculus of several variables yet? \r\n\r\nThe simplest way to evaluate the integral is to say \r\n\r\n$I = \\int_0^\\infty e^{-x^2}dx$\r\n\r\nso \r\n\r\n$I^2 = \\int_0^\\infty e^{-x^2}dx \\cdot \\int_0^\\infty e^{-y^2}dy = \\iint_{QI}e^{-x^2-y^2}dA$\r\n\r\nThat is, we write the two integrals as the integral over the first quadrant of the product of the two functions. This is an application of Fubini's theorem, which you can't always use with impunity, but I don't think this is the right place for such subtleties. Anyway, then we change variables to polar coordinates. The limits of integration in polar coordinates are $0$ to $\\infty$ for $r$ and $0$ to $\\frac{\\pi}{2}$ for $\\theta$. The differential area in polar coordinates is $rdrd\\theta$ (which you can obtain from the Jacobian of the change of variables, but you can also see it geometrically). \r\n\r\nThen we have\r\n\r\n$I^2 = \\int_0^\\frac{\\pi}{2}\\int_0^\\infty e^{-r^2}rdrd\\theta = \\frac{\\pi}{4}$.\r\n\r\nSo $I = \\frac{\\sqrt{\\pi}}{2}$, and you can get the integral of your function using integration by parts and that formula.", "Solution_8": "That is my problem. When I use integration by parts along with the error function on this new integral (like I used it on the previous integral), I get infinity for an answer. I mean, I have always been used to do integration by parts on indefinte integrals but for some reason I am getting confused using integration by parts for definite integrals. \r\n:huh:", "Solution_9": "We have $I = \\int_0^\\infty x^2e^{-x^2}dx$.\r\n\r\nLet $u = x$ and $dv = xe^{-x^2}dx$. Then we get \r\n\r\n$I = \\left[\\frac{-x}{2}e^{-x^2}\\right]_0^\\infty + \\frac{1}{2}\\int e^{-x^2}dx$\r\n\r\nI think your mistake was in the first part, where you evaluate at the bounds after you integrate as if it were an indefinite integral. You should be able to finish it from here. Nothing diverges.", "Solution_10": "Thanks a lot Xevarion! :lol: I never realized that setting $u = x$ would simplify the problem so elegantly. I was actually setting $u = x^2$ and so I just wansn't able to evalute the integral.\r\n\r\n(Also, you were right about me being evaluating at the bounds after me integrate :oops: )\r\n\r\n-Swapnil" } { "Tag": [ "number theory", "relatively prime" ], "Problem": "if I have \\[1776^{1492!} \\equiv 1 (mod 125)\\]\r\nhow do I calculate \\[1776^{1492!} (mod 2000)\\] \r\n$2000=2^4 \\times 5^3$", "Solution_1": "maybe you can use the fact that\r\n\r\n$qa \\equiv qb (mod p) \\Rightarrow a \\equiv b (mod \\displaystyle\\frac{p}{\\gcd(p,q)})$\r\n\r\nor... at least i think that's how the division thing goes...", "Solution_2": "Since $1776$ and $2000$ are both divisible by $16$, the remainder left over must also be divisible by $16$. By looking at the possibilities ($126, 251, 376,...$) we see that it must be $1376$.", "Solution_3": "I heard this question has something to do with Chinese remainder theorem, can somebody fill me in?", "Solution_4": "The Chinese Remainder Theorem states that given simultaneous congruences\r\n\r\n$ x \\equiv a_i (mod m_i)$ for i=1,...r, and $m_i$ relatively prime,\r\n\r\nthe set of solutions:\r\n\r\n$x= a_1 b_1 \\frac{M}{m_1}+...a_r b_r \\frac{M}{m_r}$\r\n\r\nwhere $ M \\equiv m_1 m_2 ...m_r$\r\n\r\nand $b_i \\frac{M}{m_i} \\equiv 1 (mod m_i)$", "Solution_5": "which is equivalent to solving \r\n\r\n$x\\equiv 1\\mod 125$\r\n$x\\equiv 0\\mod 16$ \r\n\r\nwhere $x=1776^{1492!}$ \r\n\r\nBy using Chinese Remainder Theorem\r\n\r\nwe have $x=125p+1$ and $x=16q$ for some integer $p,q$ \r\n\r\nSolving $125p+1=16q$ by diophantine gives us one of the solution is $(p,q)=(11,86)$\r\n\r\nSo $p=11+16k$ , $q=86+125k$ for some integer $k$ .Hence\r\n\r\n$x=16(86+125k)=1376+2000k$ which is $x\\equiv 1376 \\mod 2000$", "Solution_6": "[quote=\"liangchene\"]how do I calculate \\[1776^{1492!} (mod 2000)\\] [/quote]\r\n\r\n[hide=\"simple solution\"]\n$1776^{1} \\equiv 1776 \\mod {2000}$\n$1776^{2} \\equiv 176 \\mod {2000}$\n$1776^{3} \\equiv 576 \\mod {2000}$\n$1776^{4} \\equiv 976 \\mod {2000}$\n$1776^{5} \\equiv 1376 \\mod {2000}$\n$1776^{6} \\equiv 1776 \\mod {2000}$\n\nIt can be proved that $1776^{5j+k} \\equiv 1776^{k}$\nOf course $1492! \\equiv 0 \\mod {5}$, so $1776^{1492!} \\equiv 1376 \\mod {2000}$.\n[/hide]" } { "Tag": [], "Problem": "I started out with 1 and 2 and took and took the arithmetic mean and the geometric mean of both. Then I did the same with the resulting two numbers. I kept doing the same over and over. Both numbers approached\r\n[hide=\"Click\"] 1.456791031...[/hide]\r\nCan anyone figure out what this number is equal to, or is there no way to express it?", "Solution_1": "Well, what you discovered is called the arithmetic-geometric mean of the two numbers (1 and 2 in this case). As far as I know, those sequences converge, but the closed form is in terms of elliptic integrals. \r\nI found about this at http://mathworld.wolfram.com/Arithmetic-GeometricMean.html, and you might want to take a look ( or maybe not :P )", "Solution_2": "Interesting.... Thank you for the link. And from the same website, the limit of the arithmetic mean and the harmonic mean is the geometric mean. There is also the geometric-harmonic mean on wikipedia [url]http://en.wikipedia.org/wiki/Geometric-harmonic_mean[/url]." } { "Tag": [ "modular arithmetic", "number theory open", "number theory" ], "Problem": "Is it true that if $ q$ is prime, $ n$ is prime, and $ q | 3^n \\minus{} 2^n$, then $ n | q \\minus{} 1$?", "Solution_1": "work fermat's little theorem,i feel u ll get it", "Solution_2": "I can't seem to get far with Fermat's little theorem. $ q | 3^n \\minus{} 2^n \\implies \\left(3/2\\right)^n \\equiv 1 \\pmod q$. From this, we get that $ \\mbox{ord}_q(3/2) | n$, but if I'm not mistaken, $ q \\minus{} 1 | n$ only follows if $ \\frac {3}{2}$ is a primitive root modulo $ q$. Am I missing something?", "Solution_3": "When you have $ \\textrm{ord} (3/2)\\mid n$, since $ n$ is prime, it follows that $ \\textrm{ord}(3/2)\\equal{}n$. Since the order of anything divides $ q\\minus{}1$, we have $ n\\mid q\\minus{}1$.", "Solution_4": "$ n\\equal{}p$,$ q|3^{p}\\minus{}2^{p}$\r\n\r\n$ q>3$ which is obvious\r\n\r\n$ 3^{p}\\equiv 2^{p}\\pmod{q}$\r\n\r\n$ 3^{q\\minus{}1}\\equiv 2^{q\\minus{}1}\\pmod{q}$\r\n\r\nClaim:$ x^{a}\\equiv y^{a}\\pmod{q},x^{b}\\equiv y^{b}\\pmod{q}$,$ (x,q)\\equal{}(y,q)\\equal{}1$\r\n\r\n$ \\implies$ $ x^{(a,b)}\\equiv y^{(a,b)}\\pmod{q}$\r\n\r\n$ \\frac{x}{y}\\equal{}w$ $ \\implies$ $ w^{a}\\equiv 1\\pmod{q},w^{b}\\equiv 1\\pmod{q}$\r\n\r\n$ w^{(a,b)}\\equiv 1\\pmod{q}$ \r\n\r\n$ \\implies$ $ 3^{(p,q\\minus{}1)}\\equiv 2^{(p,q\\minus{}1)}\\pmod{q}$\r\n\r\n$ (p,q\\minus{}1)\\equal{}1$ or $ (p,q\\minus{}1)\\equal{}p$\r\n\r\nif $ (p,q\\minus{}1)\\equal{}1$ $ \\implies$ $ q|1$ Contradiction!\r\n\r\n$ \\implies$ $ p|q\\minus{}1$ :wink:", "Solution_5": "Oh wow, I completely forgot the condition that $ n$ was prime. :blush: How silly of me. Thanks for all the responses." } { "Tag": [ "algebra", "polynomial", "superior algebra", "superior algebra unsolved" ], "Problem": "Let $p$ be a prime,$k$ a field, $\\beta$ a non-zero elemnt of $k$, and $K$ a splitting filed over $k$ for the polynomial $f(x)=X^p-\\beta$.\r\n(i) Prove that $K$ contains a subfield $K_1$ which is a splitting field over $k$ for the polynomial $f(X)=X^p-1$, and that if $\\alpha$ is a root of $f(X)$ in $K$, then $K=K_1(\\alpha)$.\r\n(ii) Show that eother $f(X)$ is irredicuble in $K[x]$, or $f(X)$ has a root in $k$.", "Solution_1": "Let $a_1,a_2...a_p$ be the roots of $x^p-B$ in $L$. Then $a_ia_1^{-1}$ is a root for $x^p-1$ and they are all in $L$, so $x^p-1$ has its $p$ roots on $L$, So $L$ contains its splitting field. Also if $L: M: K$ and $M$ is the splitting field for $x^p-1$ then it is easy to see that $M=K(a)$ where $a$ acts on $K$ as an n primitive root of unity (because all other roots are powers of $a$ then)\r\n\r\nNote we asume Char $K$ is not $p$\r\n\r\nSecond part is Known as Abels theorem." } { "Tag": [ "MATHCOUNTS" ], "Problem": "This problem is from back in the old days when Martin was around...(no offense :| )\r\n\r\nHow many cards must be selected from a standard deck to be certain that three cards from the same suit are drawn?", "Solution_1": "hint: Pigeon Hole Principle\r\n\r\n\r\n[hide]\n9\n[/hide]", "Solution_2": "This is for their mathcount members :wink: \r\nBut since my school only makes us do workouts and warm ups :sleeping: , and since you already posted the answer, I'll provide a solution.\r\n[hide]Since there are 4 suits, the worst case scenario is when you have 8 cards, 2 from each suit. Thus, the smallest is 9. Which is also true by Pidgeonhole.[/hide]", "Solution_3": "By the way, you will be back with Martin again starting next week and will probably be there for most of the rest of the time.", "Solution_4": "[quote=\"xpmath\"]This is for their mathcount members :wink: \nBut since my school only makes us do workouts and warm ups :sleeping: , and since you already posted the answer, I'll provide a solution.\n[hide]Since there are 4 suits, the worst case scenario is when you have 8 cards, 2 from each suit. Thus, the smallest is 9. Which is also true by Pidgeonhole.[/hide][/quote]\n\nThanks xpmath! :lol: \n\n[quote=\"dan80\"]By the way, you will be back with Martin again starting next week and will probably be there for most of the rest of the time.\n[/quote]\r\nAlrighty.. :lol:", "Solution_5": "lol, that makes me feel old!\r\n\r\nBut frankly, I don't remember the question because there are so many like that.", "Solution_6": "[quote=\"Chinaboy\"]lol, that makes me feel old!\n\nBut frankly, I don't remember the question because there are so many like that.[/quote]\r\n\r\nYeah.. I noticed that there were whole bunch of those in back in the old days....(no offense, again..) And you answered almost all of them..!! :lol:" } { "Tag": [ "algebra", "system of equations", "algebra unsolved" ], "Problem": "Solve the system of equations\r\n$x^{2}(y+z)^{2}= (3x^{2}+x+1)y^{2}z^{2}$\r\n$y^{2}(z+x)^{2}= (4y^{2}+y+1)z^{2}x^{2}$ \r\n$z^{2}(x+y)^{2}= (5z^{2}+z+1)x^{2}y^{2}$.", "Solution_1": "$(x,y,z)=(0,0,z)$ or $(0,y,0)$ or $(x,0,0)$ or $({9\\over 13},{3\\over 4},{9\\over 11})$ or $(-{5\\over 6},-1,-{5\\over 4})$.", "Solution_2": "Can you show me?", "Solution_3": "1) $x^{2}(y+z)^{2}= (3x^{2}+x+1)y^{2}z^{2}$\r\n2) $y^{2}(z+x)^{2}= (4y^{2}+y+1)z^{2}x^{2}$ \r\n3) $z^{2}(x+y)^{2}= (5z^{2}+z+1)x^{2}y^{2}$\r\n\r\nIf $x=0$ or $y=0$ or $z=0$, then 1) gives $y^{2}z^{2}=0$ or $x^{2}z^{2}=0$ or $x^{2}y^{2}=0$.\r\nSo $(x,y,z)=(0,0,z)$ or $(0,y,0)$ or $(x,0,0)$.\r\nAssume therefore $x,y,z\\not =0$ and set $a={1\\over x},b={1\\over y}$ and $c={1\\over z}$.\r\nThen dividing 1),2) and 3) with $x^{2}y^{2}z^{2}$ gives\r\n4) $(b+c)^{2}=3+a+a^{2}$\r\n5) $(c+a)^{2}=4+b+b^{2}$\r\n6) $(a+b)^{2}=5+c+c^{2}$.\r\n\r\nAdding those equations gives\r\n$2a^{2}+2b^{2}+2c^{2}+2ab+2ac+2bc=12+a+b+c+a^{2}+b^{2}+c^{2}$\r\nand $(a+b+c)^{2}=12+a+b+c$, and so $a+b+c\\in\\{4,-3\\}$.\r\n\r\nIf $a+b+c=4$, then 4),5) and 6) give\r\n$(4-a)^{2}=(b+c)^{2}=3+a+a^{2}\\Rightarrow a={13\\over 9}$ and similar $b={4\\over 3}$ and $c={11\\over 9}$. This gives $(x,y,z)=({9\\over 13},{3\\over 4},{9\\over 11})$.\r\nIf $a+b+c=-3$ we get analog $(a,b,c)=(-{6\\over 5},-1,-{4\\over 5})$ and $(x,y,z)=(-{5\\over 6},-1,-{5\\over 4})$." } { "Tag": [ "logarithms", "Gauss", "real analysis", "real analysis unsolved" ], "Problem": "$ \\sum_{n\\equal{}1}^{\\infty}(2\\minus{}x)(2\\minus{}x^\\frac{1}{2})...(2\\minus{}x^{\\frac{1}{n}})$\r\n\r\nwhere $ x>0$\r\n\r\nDiscuss the above series converges or not", "Solution_1": "Pag.49 of Bromwich's book (not exactly the same exercise but very close). If $ a_n \\equal{} (2 \\minus{} x)(2 \\minus{} x^\\frac {1}{2})...(2 \\minus{} x^{\\frac {1}{n}}),$ \r\n$ {a_n\\over a_{n \\minus{} 1}} \\equal{} (2 \\minus{} x^{\\frac {1}{n}}) \\equal{} 1 \\minus{} {\\ln x\\over n} \\plus{} O(1/n^2)$ and by Gauss' criterion the series converges \r\nif and only if $ \\ln x > 1$ hence $ x > e.$ \r\n\r\n[color=green]Redundant quote deleted.[/color]", "Solution_2": "Not so simple. When does the general term tend to $ 0$? It certainly does not for all $ x > e$.", "Solution_3": "From $ x^{\\frac1k}\\approx 1 \\plus{} \\frac1k\\ln x$, we have $ (2 \\minus{} x)(2 \\minus{} x^{\\frac12})\\cdots (2 \\minus{} x^{\\frac1n})\\approx c\\cdot \\exp\\left( \\minus{} \\ln x \\minus{} \\frac {\\ln x}{2} \\minus{} \\cdots \\minus{} \\frac {\\ln x}{n}\\right)$\r\n$ \\approx c'\\exp( \\minus{} \\ln x\\ln n) \\equal{} c'\\cdot n^{ \\minus{} \\ln x}$; that $ c$ comes from the deviations in the early terms from $ \\exp( \\minus{} \\frac1k\\ln x)$.\r\nThose terms certainly go to zero for all $ x > 1$.\r\n\r\nNow, there is one value missed. At $ x \\equal{} 2$, all terms are equal to zero, and the sum converges trivially.", "Solution_4": "If $ x\\equal{}2^N$ for some integer $ N,$ the general term is definitively zero. Otherwise \r\n$ \\prod_{k\\equal{}1}^n (2\\minus{}x^{1/k}) \\to0\\ \\iff\\ \\prod_{k\\equal{}1}^n \\vert (2\\minus{}x^{1/k})\\vert \\to 0 \\ \\iff\\ \r\n\\sum_{k\\equal{}1}^n \\ln \\vert (2\\minus{}x^{1/k}) \\vert \\to\\minus{}\\infty.$ Now break \r\n$ \\sum_{k\\equal{}1}^n \\ln \\vert (2\\minus{}x^{1/k}) \\vert \\equal{} \\sum_{k\\equal{}1}^{n_0} \\ln \\vert (2\\minus{}x^{1/k})\\vert \\plus{} \r\n\\sum_{k\\equal{}n_0\\plus{}1}^n \\ln (2\\minus{}x^{1/k})$ where evidently $ 2>x^{1/k}$ for any $ n_0\\plus{}1 \\le k \\le n.$ Now \r\n$ \\sum_{k\\equal{}n_0\\plus{}1}^n \\ln (2\\minus{}x^{1/k}) \\equal{} \\sum_{k\\equal{}n_0\\plus{}1}^n (\\minus{}{\\ln x\\over k} \\plus{} O(\\ln^2 x/k^2))$ and this quantity goes to $ \\minus{}\\infty$ if and only if $ x>e.$ Moreover the general term has definite sign (a fact that I skipped when applying the Gauss's criterion) because $ 2>x^{1/k}$ definitively. \r\n\r\nDo you agree?\r\n\r\n\r\n[quote=\"milin\"]Not so simple. When does the general term tend to $ 0$? It certainly does not for all $ x > e$.[/quote]" } { "Tag": [ "real analysis", "real analysis unsolved" ], "Problem": "Let $x_i\\ge -1$ and $\\sum^n_{i=1}x_i^3=0$. Prove $\\sum^n_{i=1}x_i \\le \\frac{n}{3}$.", "Solution_1": "Let $a_i=x_i+1\\ge 0,\\;1=1,2,\\ldots,n$ so \n$ \\sum^n_{i=1}(a_i-1)^3=0 \\Longleftrightarrow \\sum^n_{i=1}a_i^3-3\\sum^n_{i=1}a_i^2+3\\sum^n_{i=1}a_i-n=0$\nWe must prove that\n$\\sum^n_{i=1}a_i\\le \\frac{4n}{3}\\Longleftrightarrow 3\\sum^n_{i=1}a_i\\le 4\\sum^n_{i=1}a_i^3-12\\sum^n_{i=1}a_i^2+12\\sum^n_{i=1}a_i\\Longleftrightarrow$ $\\sum^n_{i=1}a_i(2a_i-3)^2\\ge 0$\nwhich is true." } { "Tag": [ "calculus", "integration", "geometry", "algebra", "function", "domain", "real analysis" ], "Problem": "attached the question, and what progress I had towards a solution.\r\nnow I'm not sure how to calculate the last integral.\r\nis it the surface of the torus?\r\nshould it be taken as a torus of 2 unit circles?", "Solution_1": "it [i]is[/i] the area of the torus, which is 1 (with respect to this coordinates).\r\nyou can see it by considering a fundamental domain (e.g. $ Q\\equal{}[0,1]\\times[0,1]\\subset\\mathbb{R}^2$) for the projection $ \\mathbb{R}^2\\to T$, and observing that integrating the pull-back of $ dx_1\\wedge dx_2$ over $ Q$ gives you the lebesgue measure of $ Q$ (or, if you prefer, the \"standard\" area of $ Q$)." } { "Tag": [ "calculus", "derivative", "calculus computations" ], "Problem": "for the life of me I can't solve it.\r\n\r\nIt's \r\n\r\ny=(x+1)^(x+1)\r\n\r\nI did\r\nlny= (x+1)ln(x+1)\r\nthen\r\ny'/y= (1)ln(x+1)+ 1/X (x+1)\r\ny'= [ln(x+10) +(x+1)/x) ] (x+1)^(x+1)\r\n\r\nthe solution in my textbook says something else...what did i do wrong?", "Solution_1": "You are wrong! Here is it :)\r\n[quote=\"rayray123\"]\ny'/y= (1)ln(x+1)+ 1/X (x+1)\n\n\n[/quote]", "Solution_2": "If you know the derivative of $x^{x}$, you can find the derivative of $(x+1)^{x+1}$ by replacing $x$ with $x+1$. (A special case of the Chain Rule)." } { "Tag": [ "function", "superior algebra", "superior algebra unsolved" ], "Problem": "I'm sorry if this is an absurd question, but I haven't had any class in algebra (especially not noncommutative algebra) so I have no idea what's what. \r\n\r\nAnyway, de Bruijn is explaining big-O notation, and he explains that sometimes you write $A=B$ when you really mean $A$ is of type $B$. Then he gives the example of $n=L(m)$ meaning $n < m$ (which you might write if you didn't have \"<\" in your typewriter, he says). Then we have some interesting properties of this, such as $L(3) = L(5)$ but $L(5) \\ne L(3)$. Is there any interest or value at all in this, or am I just entertained by a parlor novelty that doesn't have any substance?\r\n\r\nedit: I guess it isn't really noncommutative either. I was just thinking of the fact that $a=b$ is not the same as $b=a$.... Not quite the same I guess.", "Solution_1": "Hi.\r\n\r\nYou have a binary relation \"a = L(b)\" or \"a = O(b)\" and are thinking about the property $a = L(b) \\Rightarrow b = L(a)$, which is called \"symmetry\". The O notation is not symmetric, and it is also not antisymmetric: The property $a = O(b) \\wedge b = O(a) \\Rightarrow a = b$ is called \"antisymmetry\".\r\n\r\nTherefore some authors prefer to use the notation $a \\in O(b)$, which makes it clear that we do not have an equation, but some arbitrary relation (indeed any binary relation $\\sim$ can be written in the form $a \\sim b \\Leftrightarrow a \\in L(b)$ with an appropriate function $L$).\r\nBut most of the time you will see $a = O(b)$, and have to remember that \"$=O$\" is one single relation symbol.\r\n\r\nLiMa" } { "Tag": [ "geometry", "circumcircle", "geometry proposed" ], "Problem": "Let $AA'$ be the line tangent in $A$ to the circumcircle of $ABC$ ($A'\\in BC$). Show that the orthic axis of $ABC$ passes through the midpoint of $AA'$.\r\n\r\n[The orthic axis is the following line: let $H_a,H_b,H_c$ be the feet of the altitudes from $A,B,C$ respectively. Now let $A^*=H_bH_c\\cap BC$, and define $B^*,C^*$ similarly. $A^*,B^*,C^*$ are collinear, and the line they lie on is called the orthic axis]", "Solution_1": "That is on page 6 (at the very end of paragraph 11) of\r\n\r\nWilliam Gallatly, [i]The Modern Geometry of the Triangle[/i], Second Edition London 1913.\r\n\r\nThere is an affine generalization of the problem:\r\n\r\n[i]Let P be a point in the plane of a triangle, and let the lines AP, BP, CP intersect the lines BC, CA, AB at the points X, Y, Z, respectively. The parallel to the line YZ through the point A intersects the line BC at A'. Let the lines YZ, ZX, XY meet the lines BC, CA, AB at the points X', Y', Z', respectively. It is well-known that the points X', Y', Z' lie on one line, the so-called [b]tripolar[/b] of the point P with respect to triangle ABC. Prove that this tripolar passes through the midpoint of the segment AA'.[/i]\r\n\r\nHere is my solution:\r\n\r\nSince the lines AX, BY, CZ concur at one point (namely, the point P), we have by Ceva\r\n\r\n$\\frac{BZ}{ZA}\\cdot \\frac{AY}{YC}\\cdot \\frac{CX}{XB}=1$.\r\n\r\nSince the points X, Y, Z' are collinear, Menelaos yields\r\n\r\n$\\frac{BZ^{\\prime }}{Z^{\\prime }A}\\cdot \\frac{AY}{YC}\\cdot \\frac{CX}{XB}=-1$.\r\n\r\nComparing these two equations, we see that\r\n\r\n$\\frac{BZ^{\\prime }}{Z^{\\prime }A}=-\\frac{BZ}{ZA}$.\r\n\r\nNow, since AA' || YZ, we have by Thales\r\n\r\n$\\frac{BZ}{ZA}=\\frac{BX^{\\prime }}{X^{\\prime }A^{\\prime }}$.\r\n\r\nThus,\r\n\r\n$\\frac{BZ^{\\prime }}{Z^{\\prime }A}=-\\frac{BZ}{ZA}=-\\frac{BX^{\\prime }}{X^{\\prime }A^{\\prime }}=-\\frac{X^{\\prime }B}{A^{\\prime }X^{\\prime }}$.\r\n\r\nNow, if M is the midpoint of the segment AA', then we clearly have $\\frac{AM}{MA^{\\prime }}=1$. Hence,\r\n\r\n$\\frac{BZ^{\\prime }}{Z^{\\prime }A}\\cdot \\frac{AM}{MA^{\\prime }}\\cdot \\frac{A^{\\prime }X^{\\prime }}{X^{\\prime }B}=\\left( -\\frac{X^{\\prime }B}{A^{\\prime }X^{\\prime }}\\right) \\cdot 1\\cdot \\frac{A^{\\prime }X^{\\prime }}{X^{\\prime }B}=-1$.\r\n\r\nThus, by Menelaos, the points Z', X' and M are collinear, i. e. the line Z'X' passes through the point M. But the line Z'X' is the tripolar of the point P with respect to triangle ABC, and the point M is the midpoint of the segment AA'. Hence, we see that the tripolar of the point P with respect to triangle ABC passes through the midpoint of the segment AA'. Proof complete.\r\n\r\nNow, your problem is a special case of this since, if we take P to be the orthocenter of triangle ABC, then the tripolar of P is the orthic axis of triangle ABC, while the line AA' is the tangent to the circumcircle of triangle ABC at the vertex A (in fact, the line AA' is parallel to the line YZ, and since the line YZ, being a side of the orthic triangle XYZ of triangle ABC, is antiparallel to the side BC, the line AA' must be antiparallel to the side BC, too; but the antiparallel to BC through A is the tangent to the circumcircle at A, and therefore, the line AA' is the tangent to the circumcircle at A). And hence we get your problem.\r\n\r\n Darij", "Solution_2": "Well, my solution to the initial problem fits the generalization as well: $(A,C;Y,Y')=-1$, so the lines $X'A,X'C,X'Y,X'Y'$ form a harmonic quadruple, and $AA'$ is parallel to one of the lines of this quadruple, so the other three cut equal segments on it." } { "Tag": [ "integration", "vector", "calculus", "real analysis", "real analysis theorems" ], "Problem": "I've noticed that $\\int_{x_{a}}^{x_{b}}y dx+\\int_{y_{a}}^{y_{b}}x dy = x_{b}y_{b}-x_{a}y_{a}$, and I think I can prove that it holds for vector line integrals when $x$ and $y$ are vector fields on the plane. Is this a well-known theorem? Is there a multivariable generalization? (like n variables instead of x,y)", "Solution_1": "Yes, this is well known. It's a special case of the fundamental theorem for line integrals; note that $(y,x)$ is the gradient of $xy$.\r\n\r\nThere is a large family of \"fundamental theorem\" results in multivariable calculus- most of them go by the name of Stokes' theorem." } { "Tag": [ "modular arithmetic" ], "Problem": "For how many positive integers, $n < 1000$, is $n^4 \\equiv 9 \\pmod {16}$?", "Solution_1": "None. Since $2^4=16$, any even integer will automatically result in $n^4\\equiv 0 \\bmod{16}$. Next, we check odd integers, using the form $(n^2)^2\\equiv x \\bmod{16}$ for easier calculations, and we see that all odd integers squared will result in either a $1$ or a $9$ when taken $\\bmod{16}$; from there, we have either $1\\bmod{16}$ or $81\\bmod{16}$, both of which will always be $1$. Therefore, if $n^4\\equiv x\\bmod{16}$, then $x=0$ for all even integers or $x=1$ for all odd integers.", "Solution_2": "I agree with JesusFreak197.\r\n\r\nIf $n$ is even, then $n=2k$ for some integer $k$.\r\n$(2k)^4=16k^4 \\equiv 0\\pmod{16}$\r\n\r\nIf $n$ is odd, then $n=2k+1$ for some integer $k$.\r\n$(2k+1)^4=16k^4+32k^3+24k^2+8k+1=16(k^4+2k^3)+8k(3k+1)+1$\r\nNote that $k(3k+1)$ is always even.\r\n[hide=\"reason\"]when $k$ is even, $k(3k+1)$ is $(even)(odd)=(even)$\nwhen $k$ is odd, $k(3k+1)$ is $(odd)(even)=(even)$[/hide]\r\nThus $8k(3k+1)$ is a multiple of 16 and we have\r\n$16(k^4+2k^3)+8k(3k+1)+1 \\equiv 1\\pmod{16}$\r\n\r\nSo there does not exist any integer $n$ such that $n^4 \\equiv 9\\pmod{16}$.", "Solution_3": "$0$ is the correct answer. :)", "Solution_4": "Now, if it was $n^2\\equiv 9 \\bmod{16}$, you would actually have some that work. :) I challenge people to find all integers $n<1000$ that satisfy this equation. ;)", "Solution_5": "My pathetic attempt so far:\r\n\r\n$n^2\\equiv9(mod 16)$\r\n$n^2-9\\equiv0(mod 16)$\r\n$(n+3)(n-3)\\equiv0(mod 16)$\r\n\r\nWhat do do now, is just a mystery :( (without trial and error that is) ooh...$5$ works!", "Solution_6": "[hide]$(n+3)(n-3)$ must be a multiple of $16$, so it must contain at least four $2$'s.[/hide]", "Solution_7": "[hide=\"Hint\"]There might be a formula for finding the answer, but I just found a pattern.[/hide]", "Solution_8": "I used the same method as I solved the previous problem :) \r\n\r\nClearly, even number doesn't work, so we just have to check for odd numbers.\r\nAll odd numbers are in one of the following forms:\r\n$8k+1$, $8k+3$, $8k+5$, or $8k+7$.\r\n\r\nIf $n=8k+1$, then $(8k+1)^2=64k^2+16k+1 \\equiv 1\\pmod{16}$\r\nIf $n=8k+3$, then $(8k+3)^2=64k^2+48k+9 \\equiv 9\\pmod{16}$\r\nIf $n=8k+5$, then $(8k+5)^2=64k^2+80k+25 \\equiv 9\\pmod{16}$\r\nIf $n=8k+7$, then $(8k+7)^2=64k^2+112k+49 \\equiv 1\\pmod{16}$\r\n\r\nTherefore, all numbers in the form $8k+3$ or $8k+5$ ($3$, $5$, $11$, $13$,...) are equivalent to 9 in mod 16, and there are $1000\\times \\frac{2}{8}=250$ of them for $n<1000$.", "Solution_9": "Alright, now that the answer has been posted... the pattern that I found is essentially the one that you found, although I didn't think about it in the same way; I took consecutive odd numbers, squared them, divided by 16, and found the pattern $1$, $9$, $9$, $1$, $1$, $9$, etc." } { "Tag": [ "number theory unsolved", "number theory" ], "Problem": "Let x, y be positive integers such that 3x + 7y is divisible by 11. Prove that 4x - 9y is also divisible by 11.\r\n\r\nThanks,", "Solution_1": "[quote=\"futurus\"]Let x, y be positive integers such that 3x + 7y is divisible by 11. Prove that 4x - 9y is also divisible by 11.\n\nThanks,[/quote]\r\n$3(4x-9y)=4(3x+7y)-55y.$" } { "Tag": [ "algebra", "polynomial", "Galois Theory", "superior algebra", "superior algebra unsolved" ], "Problem": "Let $ p\\text{ be a prime},n\\in\\mathbb{N}, (p,n)\\equal{}1$, \r\nand let $ o_n(p)$ be the order of $ p\\in(\\mathbb{Z}/n\\mathbb{Z})^*$\r\nProve that the $ n$-th cyclotomic polynomial $ \\Phi_n(x)$ over $ \\mathbb{F}_p$\r\nfactors into irreducible factors of degree $ o_n(p)$.", "Solution_1": "pick a primitive $ n$-root of unity $ \\zeta$ in $ \\overline{\\mathbb{F}_p}$ and let $ G$ be the galois group of $ \\Phi_n$ over $ \\mathbb{F}_p$, which is generated by the frobenius $ x \\mapsto x^p$. the equation $ g(\\zeta) \\equal{} \\zeta^{\\alpha(g)}$ yields an injective homomorphism $ \\alpha : G \\to (\\mathbb{Z}/n)^*$, which maps the frobenius to $ p$. thus $ ord(G) \\equal{} o_n(p)$.\r\n\r\nnow if $ f$ is an irreducible factor of $ \\Phi_n$, $ f$ is the minimal polynomial of $ \\zeta$ (or another choice of $ \\zeta$), so that its degree equals $ ord(G) \\equal{} o_n(p)$.", "Solution_2": "Ok, thank you for reply." } { "Tag": [ "real analysis", "real analysis unsolved" ], "Problem": "For solving the Riccati equations, can u guess the particular solution or it should be given in the question. \r\n\r\n\r\ndy/dx= y + y^2/x^2 - x(2x-1) \r\n\r\nIf it is not given, how can I find a particular solution for this equation?\r\n\r\nIf you can give me a particular solution, that would be helpful and can u tell me why too...", "Solution_1": "Try the variable substitution defined by $y = -x^2\\frac{w'}{w}$ where $w(x)$ is some function.", "Solution_2": "hello, Maple 13 gives the following solution\r\n$ y \\left( x \\right) \\equal{}1/2\\,{\\frac { \\left( \\minus{}7\\,x{\\it \\_C1}\\plus{}ix\\sqrt {7}{\r\n{\\rm e}^{\\minus{}i\\sqrt {7}x}}\\plus{}14\\,{\\it \\_C1}\\minus{}2\\,i\\sqrt {7}{{\\rm e}^{\\minus{}i\\sqrt \r\n{7}x}}\\minus{}7\\,i\\sqrt {7}x{\\it \\_C1}\\plus{}7\\,x{{\\rm e}^{\\minus{}i\\sqrt {7}x}} \\right) x\r\n}{\\minus{}7\\,{\\it \\_C1}\\plus{}i\\sqrt {7}{{\\rm e}^{\\minus{}i\\sqrt {7}x}}}}$ \r\nSonnhard." } { "Tag": [ "linear algebra", "matrix", "vector" ], "Problem": "Let A be an nxn matrix of rank (n-1). So some row/column a_i will be a linear combination of the rest. Why would the value of every determinant of size (n-1)x(n-1) formed from the cofactor expansion on A would be zero, except when we expand along a_i? Why is the linear dependence of a_i preserved even when we take out one of the columns/rows that participates in forming a_i?", "Solution_1": "Ok, can I reason myself in the following way:\r\n\r\nIf A is an nxn of rank (n-1), then there are n-1 vectors that can serve as a basis of the space of dimension n-1. So, if one of these vectors is taken out, there will be n-2 linearly independent vectors in the subspace of dimension n-2 which will span it. So, from this it follows that the linearly dependent a_i in A will still be linearly dependent in the subspace of dimension n-2 (which will be any cofactor matrix not created with expansion on a_i) and could be expressed as a combination of the vectors left.", "Solution_2": "I think you're trying to prove something that isn't true. We can easily construct $n$ vectors so that any $n-1$ are linearly independent." } { "Tag": [ "geometry", "inequalities", "geometry solved" ], "Problem": "Try solving this please beacause I want to know your opinion.\r\n\r\nLet ABC be a triangle with the standard notations.\r\nsin2A+sin2B+sin2C=1/2 \r\nProve that:\r\n\r\np/3 \\leq \\sqrt 3S", "Solution_1": "Solution:\r\n\r\nwe know that:\r\nsin 2A+sin 2B+sin 2C=4sinA*sinB*sinC => \r\nsinA*sinB*sinC=1/8\r\n\r\nnow we use this identity: S=2R^2*sinA*sinB*sinC => S=R^2/4\r\nand the inequality becomes:\r\np/3<=sqrt(3*R^2/4) => p<=3Rsqrt(3)/2 which is Mitrinovic's inequality\r\nso p/3<=sqrt(3S) is proved.\r\n\r\nbye!\r\n:D :D" } { "Tag": [ "number theory proposed", "number theory" ], "Problem": "A number consists of 3 different digits. The sum of the 5 other numbers formed with those digits is 2003. Find the number.", "Solution_1": "217.\r\n\r\nTrivial casework...did I misunderstand the question?", "Solution_2": "Trivial casework is possible indeed. Although there is a nicer solution. \r\nDon't expect it to be too hard anyways.\r\n\r\nBut the other problems is not open for posting to non-mods, and I guess they'll complain about this in intermediate :?", "Solution_3": "Well, I don't there is a problem to post easy problems into the advanced section especially if they are coming from a national olympiad (in that case, the mods do not want them into the intermediate forum).\r\n\r\nDont worry about it, there are many others superhard problems here to maintain the average level :D \r\n\r\nPierre.", "Solution_4": "[quote=\"pbornsztein\"]Well, I don't there is a problem to post easy problems into the advanced section especially if they are coming from a national olympiad (in that case, the mods do not want them into the intermediate forum).\n\nDont worry about it, there are many others superhard problems here to maintain the average level :D \n\nPierre.[/quote]\r\n\r\nPierre, I don't think Flanders MO is the level people expect to get when they hear \"National Olympiad\" :D\r\n\r\nBut you're right. Anyway someone got ideas for a beatiful solution here?", "Solution_5": "not beautiful, perhaps the same thought process:\r\n\r\nthe six numbers formed by the digits a,b,c are\r\n[tex]100a+10b+c[/tex]\r\n[tex]100a+10c+b[/tex]\r\n[tex]100b+10a+c[/tex]\r\n[tex]100b+10c+a[/tex]\r\n[tex]100c+10a+b[/tex]\r\n[tex]100c+10b+a[/tex]\r\n\r\nAdding these 6 gives [tex]222(a+b+c)[/tex]\r\n\r\nSince the 5 arrangements other than our solution adds to 2003, we have that [tex]10 \\leq a+b+c \\leq 13[/tex]\r\n\r\nFrom here we test the number forced by a+b+c=10,11,12,13\r\na+b+c=10 --> 2220-2003 = 217 (our solution)\r\na+b+c=11--> 2442-2003 = 439 (sum is 16 - bad)\r\na+b+c=12 --> 2664-2003 = 661 (sum is 13 - close, but bad)\r\na+b+c=13 --> 2886-2003 = 883 (sum is 19 - worst)", "Solution_6": "That's the best solution indeed. But instead of case checking you can show the sum doesn't divide 3, doesn't divide 11, etc. But that one is far the most efficient.", "Solution_7": "Let the number be abc.\nThen 2003=acb+bac+bca+cab+cba=122a+212b+221c.\nSince a is not equal to 0,easy calculation shows that a=2,b=1 and c=7.\nThus,the number is 217. :lol:", "Solution_8": "Then another problem similar to that.Hope it would not be so easy.Find\n$\\overline {abcd}$ such that $4|\\overline {abcd},7|\\overline {bacd},$$9|\\overline {abdc},5|\\overline {acbd}$" } { "Tag": [ "Reaper", "combinatorics unsolved", "combinatorics" ], "Problem": "We have a stack of $ n$ books piles on each other, and labeled by $ 1,2,...,n$. In each round we make $ n$ moves in the following manner: In the $ i$-th move of each turn, we turn over the $ i$ books at the top, as a single book. After each round we start a new round similar to the previous one. Show that after some moves, we will reach the initial arrangement.", "Solution_1": "this one is from [b]iran 2006 second round[/b] to,like the other problmes you posted...\r\njust note that we have $ n!$ different arrangements,and also using the moves you told,there are at most $ 2^n.n!$ possibilities...\r\nnow try to use pigeon hole principle to complete the proof...", "Solution_2": "Well, Mr.Babak Ghalebi, can you please provide the full solution? :)", "Solution_3": "ok here it goes,I'd try to make it easier to understand...\r\nassume the sequence $ \\{a_j^i\\}$ we will give any POSITION of books a member of this sequence in the following way:\r\n\r\nthe index $ j$ in $ a_j^i$ introduces the arrangement of the books,or in other words it shows that [b]which book is on top,which one is second,which one is third,...[/b] and also each book is faced up or faced down...\r\nhence $ 1\\leq j\\leq 2^n.n!$ (I)...\r\n\r\nthe index $ i$ in $ a_j^i$ introduces the MOVE which was taken to reach this POSITION,in other words we have [b]turned the $ i$ books at top[/b] to reach this POSITION...\r\nhence $ 1\\leq i\\leq n$ (II)...\r\n\r\nso now every POSITION of books has it's member of the sequence,now lets start to solve the problem...\r\n\r\nas the problem says,we start the moves,in each step write down the [b]member of the sequence[/b] which is given to this position...\r\nso we would have an infinite sequence of $ a_j^i$s,now this special sequence has the following properties:\r\n\r\n$ i)$the index $ i$ in this sequence starts from $ 1$ goes on to $ n$ and then turns $ 1$ again,like the following one:\r\n\r\n$ a_{j_1}^1,a_{j_2}^2,a_{j_3}^3,\\ldots,a_{j_n}^n,a_{j_{n \\plus{} 1}}^1,...$\r\n\r\n$ ii)$let the $ k^{th}$ member of this sequence be $ a_{j}^i$ then the $ k \\plus{} 1^{th}$ member is unique...\r\n\r\n$ iii)$if $ a_j^i \\equal{} a_{j}^{i'}$ then THE two positions which these members introduce,are the same...\r\n\r\nnow note that we have $ n.2^n.n!$ different $ a_j^i$s (according to (I),(II)) but our sequence is infinite so there must be a repeat in our sequence,let the first repeat be $ a_s^r$ i.e.:\r\n\r\n$ a_{j_1}^1,\\ldots,a_k^{r \\minus{} 1},a_s^r,\\ldots,a_t^{r \\minus{} 1},a_s^r,\\ldots$\r\n\r\nand assume that $ r > 1$...\r\n\r\nnow according to the property $ ii)$ we must have:\r\n\r\n$ a_k^{r \\minus{} 1} \\equal{} a_t^{r \\minus{} 1}$\r\n\r\nbut this contradicts the fact that $ a_s^r$ was the first reapet,so the assumption $ r > 1$ is incorrect so we must have $ r \\equal{} 1$ and also from $ iii)$ we get that $ s\\equal{}j_1$,which means that the first POSITION repeats after finitiely many moves..." } { "Tag": [], "Problem": "If the smaller angle between the hour and minute hands of a 12 hour analog clock is 84 degrees and the sum of the hours and minutes is 17, what time between noon and midnight is it?", "Solution_1": "Use $ |5.5m\\minus{}30h|$ is angle where $ m$ is minutes and $ h$ hours.\r\n\r\n$ |5.5m\\minus{}30h|\\equal{}84$ or $ 360\\minus{}84$\r\nand $ m\\plus{}h\\equal{}17$ \r\n$ |5.5m\\minus{}510\\plus{}30m|\\equal{}|35.5m\\minus{}510|$\r\nSolutions are $ m\\equal{}12$ and $ m\\equal{}6.59...$ so :\r\n$ m\\equal{}12$ and $ h\\equal{}5$\r\nTime: $ 5: 12am$" } { "Tag": [ "algebra", "polynomial", "number theory", "IMO", "prime numbers", "IMO 1987", "IMO Shortlist" ], "Problem": "Let $n\\ge2$ be an integer. Prove that if $k^2+k+n$ is prime for all integers $k$ such that $0\\le k\\le\\sqrt{n\\over3}$, then $k^2+k+n$ is prime for all integers $k$ such that $0\\le k\\le n-2$.", "Solution_1": "Lemma:If m<(2b+1)^2 and m in relatively prime to b+1...2b then m is prime\r\nWe have n+k^2+k is prime for k=1->r;r=[\\sqrt{n\\3}] and need to show n+k^2+k is prime for k=r+1->n-2.Applying the lemma for m=n+(r+s)^2+r+s;b=r+s(s=1->n-s-r)The computation is simple", "Solution_2": "I have not still yet checked ur solution,but perhaps,my one is quite similar.\r\nCall y is miniamal number st p(y) is composite,p is minimal prime divisor of p(y).We need to show that: y>n-2.\r\nSupposing that y\u2264n-2.We will show that:p\u22652y+1 ,then p(y)=y\u00b2+y+n\u2265p\u00b2> 4y\u00b2+4y+1->n>3y\u00b2->y<\u221a(n/3).It's absurd!\r\n Indeed suppoing that p<2y+1.Consider p(y)-p(x)=(y-x)(y+x+1) when x in {1,2,..y} then (y-x) and y+x+1 will run over all values of set{1,2,...,2y+1} then there exists x in that set such that p(y)-p(x) is divisible by p ->p(x) is too.\r\n Accoring definition of y we deduce that p(x)=p.But :\r\ny-xA contradiction!!And we are done!", "Solution_3": "Consider the least prime $ q$ dividing a composite value $ k^2 \\plus{} k \\plus{} n$ with $ \\sqrt {n/3} < k\\leq n \\minus{} 2$. Clearly $ q < n$ so every residue class modulo q is represented between 0 and n-2. Thus both roots of $ k^2 \\plus{} k \\plus{} n \\equal{} k^2 \\minus{} (q \\minus{} 1)k \\plus{} n \\equal{} 0$ mod q are in the desired range, so we may some root $ k < \\equal{} \\frac {q \\minus{} 1}{2}$.\r\n$ k > \\sqrt {n/3}$ implies $ q > 2\\sqrt {n/3} \\plus{} 1$\r\n$ q$ being minimal implies $ q^2 \\leq k^2 \\plus{} k \\plus{} n \\leq \\frac {q^2 \\minus{} 1}{4} \\plus{} n$, so\r\n\\[ q^2 \\leq \\frac {4n \\minus{} 1}{3} < (2\\sqrt {n/3} \\plus{} 1)^2 < q^2\r\n\\]\r\n.\r\nSo no such $ q$ exists.", "Solution_4": "q \\sqrt{a_k}&\\iff 4k^2-4k+1> k^2-k+s \\\\\n&\\iff 3k^2-3k+1> s \\\\\n&\\longleftarrow 3(\\sqrt{s/3}+1)^2-3(\\sqrt{s/3}+1)+1> s \\\\\n&\\iff s+3\\sqrt{s/3}+1>s,\n\\end{align*}\nwhich is true, so $2k-1> \\sqrt{a_k}\\ge p$, so $p+1-kk\\ge k-n$, so $a_n>k-n$. Therefore, $a_n\\mid k+n-1$. But $2a_n\\ge s+s>k+n>k+n-1$, so the only possibility is $a_n=k+n-1$. But then, $n^2-n+s=k+n-1$, or $k=s+(n-1)^2\\ge s$, which is a contradiction. Therefore, our assumption that $a_k$ is not prime is wrong, so $a_k$ is prime. Therefore, we finish by induction on $k$. ", "Solution_17": "Let $f(x)=x^2+x+n$ and assume that there exists $\\sqrt\\frac{n}{3}< k\\leq n-2$ such that $f(k)$ is not prime and choose the smallest such $k.$ Let $p$ be a prime divisor of $f(k),$ such that $p\\leq \\sqrt{f(k)}.$ Note that $$p\\leq \\sqrt{f(k)}=\\sqrt{k^2+k+n}<\\sqrt{4k^2+k}<2k+1\\Rightarrow p<2k.$$\n\nLet's first look at the case $kn-2,$ contradiction$.$\n\nFinally, assume $0k>p,$ contradiction$. \\blacksquare$", "Solution_18": "[hide = rewording]Let $m$ and $s$ be positive integers with $2 \\leq s \\leq 3m^2$. Define a sequence $a_1, a_2, \\ldots$ recursively by $a_1 = s$ and\\[a_{n+1} = 2n + a_n \\text{ }(\\text{for } n = 1, 2, \\ldots).\\]Prove that if the numbers $a_1, a_2, \\ldots, a_m$ are prime, then $a_{s-1}$ is also prime.[/hide]\n\nNote that in general, we may write $a_n = n^2 - n + s$. \n\nLet $t$ be the smallest positive integer for which $a_t$ is not prime; we wish to prove $t \\geq s$. Let $p$ be the smallest prime dividing $a_t$. We have\\[p \\leq \\sqrt{a_t} = \\sqrt{t^2 - t + s} \\leq \\sqrt{4t(t - 1)} < 2t - 1\\]since clearly $s = 3m^2 \\leq 3t(t-1)$ as $t \\geq m + 1$. Hence $p \\in [0, 2t - 2]$. Next, we observe that \\[a_t \\equiv 0 \\pmod p \\implies a_{t-p} \\equiv 0 \\pmod p.\\]So in the case $p \\leq t-1$ then we can subtract sufficiently many copies of $p$ from $t$ to obtain an index $t - \\ell p = i$ for which $1 \\leq i \\leq m$ and $a_i \\equiv 0 \\pmod p$ is prime. Then, $a_i$ must be $p$ and then we have $a_t \\geq p^2 = a_i^2 \\geq a_1^2 = s^2 = a_s \\implies t \\geq s$ as desired. \n\nIn the case that $p \\in [t+1, 2t - 2]$, we can let $p = t + r$ where $r \\in [1, t-2]$ so that\\[a_t \\equiv a_{t - p} = a_{-r} = r^2 + r + s \\equiv a_{r-1} \\equiv 0 \\pmod{p}\\]and clearly $r - 1 \\leq t - 1 \\leq m$ so $a_{r-1}$ is prime, so it follows that\\[a_t \\geq p^2 = a_{r-1}^2 \\geq a_1^2 = s^2 = a_s \\implies t \\geq s\\]as desired. \n\nLastly, if $p = t$, then $a_t = a_p = p^2 - p + s$ is disivible by $p$ hence $p \\mid s$. However, we have $s = a_1$ is prime so $p = s \\implies t = s$ which is what we wanted to prove.\n\nWe have exhausted all three cases so we are done; indeed, $t \\geq s$ always. $\\blacksquare$", "Solution_19": "Let $F(X)=X^2+X+n$ and define $p$ as the smallest prime factor of the numbers $F(0),F(1),.....$\nFor the sake of contradiction assume the opposite and proceed.\nSuppose for $X=k2A \\implies A^2+A+n\\geq p(p+2)>4A(A+1) \\implies n>3A^2+3A,$$\nbut the RHS is at least $3A^2>n$, which is a contradiction. As such no $A$ exist so all the terms are prime. $\\blacksquare$" } { "Tag": [ "algebra", "polynomial", "LaTeX" ], "Problem": "15. Let a,b, and c be real constants such that Code: \r\nx^2+x+2 \r\nis a factor of Code: \r\nax^3+bx^2+cx+5 \r\n, and \\2x-1 is a factor of ax^3+bx^2+cx-(25/16). Find a+b+c. \r\n\r\n21. Let k be a positive integer. A positive integer is said to be a k-flip if the digits of n are reversed in order when it is multiplied by k. For example, 1089x9 = 9801, and 21978 is a 4-flip because 21978x4=87912. Explain why there is no 7-flip integer? \r\n\r\n22. Let ABC be an acute-angled triangle? Let D and E be the points on BC and AC, respectively, such that AD is perpendicular to BC and BE is perpendicular to AC. Let P be the point where AD meets the semicircle constructed outwardly on BC, and Q the point where BE meets the semicircle constructed outwardly on AC. Prove that PC=QC. \r\n\r\n23. Two friends, Marco and Ian, are talking about their ages. \r\n\r\nIan says, \"My age is a zero of a polynomial with integer coefficients.\" \r\n\r\nHaving seen the polynomial p(x) Ian was talking about, Marco exclaims, \"You mean, you are seven years old? Oppps, sorry I miscalculated! p(7) = 77 and not zero\" \r\n\r\n\"Yes I am older than that,\" Ian's agreeing reply. \r\n\r\nThen Marco mentioned a certain number, but realizes after a while that he was wrong again because the value of the polynomial at that number is 85. \r\n\r\nIan sighs, \"I am even older than that number.\" \r\n\r\nDetermine Ian's age. \r\n[/code]", "Solution_1": "plz write in $ LATEX$.... its not readable.", "Solution_2": "[quote=\"geoffrels\"]15. Let a,b, and c be real constants such that Code: \nx^2+x+2 \nis a factor of Code: \nax^3+bx^2+cx+5 \n, and \\2x-1 is a factor of ax^3+bx^2+cx-(25/16). Find a+b+c. \n\n21. Let k be a positive integer. A positive integer is said to be a k-flip if the digits of n are reversed in order when it is multiplied by k. For example, 1089x9 = 9801, and 21978 is a 4-flip because 21978x4=87912. Explain why there is no 7-flip integer? \n\n22. Let ABC be an acute-angled triangle? Let D and E be the points on BC and AC, respectively, such that AD is perpendicular to BC and BE is perpendicular to AC. Let P be the point where AD meets the semicircle constructed outwardly on BC, and Q the point where BE meets the semicircle constructed outwardly on AC. Prove that PC=QC. \n\n23. Two friends, Marco and Ian, are talking about their ages. \n\nIan says, \"My age is a zero of a polynomial with integer coefficients.\" \n\nHaving seen the polynomial p(x) Ian was talking about, Marco exclaims, \"You mean, you are seven years old? Oppps, sorry I miscalculated! p(7) = 77 and not zero\" \n\n\"Yes I am older than that,\" Ian's agreeing reply. \n\nThen Marco mentioned a certain number, but realizes after a while that he was wrong again because the value of the polynomial at that number is 85. \n\nIan sighs, \"I am even older than that number.\" \n\nDetermine Ian's age. \n[/code][/quote]\r\n\r\n1. Use $ \\text{\\LaTeX}$\r\n2. Never ever ever doublepost anything because it will end in noone replying (or even getting you banned)\r\n3. I'll just give you some hints regardless...\r\n\r\n21: The \"flipped\" number has first digit 7, 8 or 9, so the other one has first digit 1. This means that it has last digit 3 which is impossible.\r\n23: Use the very useful lemma: If $ p(x)$ is a polynomial with integer coefficients, then $ (x-y)|(p(x)-p(y)).$ The second number Marco guessed is a, Ian's age is b. Then $ (a-7)|8,(b-7)|77,(b-a)|85.$ This yields finitely many possibilites for a and b and you just have to go through them to find the proper one." } { "Tag": [ "trigonometry", "function", "integration", "geometry", "rectangle", "linear algebra", "matrix" ], "Problem": "Calculate $ \\sum_{n \\equal{} 1}^{\\infty}\\frac {\\cos n}{n^2 \\plus{} 1}$ :)", "Solution_1": "First hint: it's actually easier to compute $ f(x)\\equal{}\\sum_{n\\equal{}1}^{\\infty}\\frac{\\cos nx}{n^2\\plus{}1}.$ \r\n\r\nSecond hint: what can you say about $ f''(x)\\minus{}f(x)?$ (You do have to stop worrying about such details as having series converge any way except weakly as distributions.)", "Solution_2": "$ \\sum_1^{\\infty}\\frac {\\cos{nx}}{1 \\plus{} n^2}$ is very close to the Fourier series of $ f(x) \\equal{} e^x \\plus{} e^{2\\pi}e^{ \\minus{} x}$ on $ [0,\\pi]$. I did not compute everything carefully, but to see this i integrated the given series twice and then noticed that $ \\sum_1^{\\infty}\\frac {\\cos{nx}}{n^2(1 \\plus{} n^2)} \\equal{} \\sum_1^{\\infty}\\frac {\\cos{nx}}{n^2} \\minus{} \\sum_1^{\\infty}\\frac {\\cos{nx}}{1 \\plus{} n^2}$, the first part is known to be linear function. Then it is clear that the solution can be found between the solutions of equation $ F'' \\minus{} F \\equal{} 0$, which are of the form $ F \\equal{} c_1e^x \\plus{} c_2e^{ \\minus{} x}$.\r\nP.S. oops, as always i'm late :wink:", "Solution_3": "Actually, they are of the form $ f(x) \\equal{} c_1 cos(x) \\plus{} c_2 sin(x)$ but that's basically the same if you calculate non-reals!\r\n\r\n\r\n[b]EDIT:[/b]\r\n\r\nI was about to delete this post when I saw what I wrote, but since jmerry referenced this post, I'll just state the obvious - this is wrong!", "Solution_4": "No, that's different. $ \\cos$ and $ \\sin$ are solutions to $ f''\\plus{}f\\equal{}0$, which is a different differential equation. $ \\cosh$ and $ \\sinh$ might be useful here, except that there's a corner at zero; we're better off translating horizontally and using $ \\cosh(x\\minus{}\\pi)$ on $ [0,2\\pi]$.", "Solution_5": "Consider a function:\r\n\r\n$ f(x) = \\sum^{\\infty}_{n = 1}\\frac {1}{\\frac {n^2}{x^2} + 1}$\r\n\r\n$ f(x) = - \\sum^{\\infty}_{n = 1}\\sum^{\\infty}_{k = 1}( - 1)^k \\frac {x^{2k}}{n^{2k}}$\r\n\r\n$ f(x) = - \\sum^{\\infty}_{k = 1}\\sum^{\\infty}_{n = 1}( - 1)^k \\frac {x^{2k}}{n^{2k}}$\r\n\r\nnow\r\n\r\n$ \\sum^{\\infty}_{n = 1}( - 1)^k \\frac {x^{2k}}{n^{2k}} = ( - 1)^k x^{2k} \\zeta(2k)$\r\n\r\nso we get:\r\n\r\n$ f(x) = \\sum^{\\infty}_{k = 1}( - 1)^{k - 1} x^{2k} \\zeta(2k)$\r\n\r\nwe know that:\r\n\r\n${ B_{2k} = 2\\zeta(2k) \\frac {( - 1)^{k - 1} (2k)!} {(2 \\pi)^{2k}}}$\r\n\r\nwhere $ B_n$ is the $ n$-th Bernoulli nuber. From this we get:\r\n\r\n$ ( - 1)^{k - 1}\\zeta(2k) = \\frac {B_{2k}}{2}\\frac {(2\\pi)^{2k}} {(2k)! }$\r\n\r\nand\r\n\r\n\r\n$ f(x) = \\sum^{\\infty}_{k = 1}x^{2k} \\frac {B_{2k}}{2}\\frac {(2\\pi)^{2k}} {(2k)! }$\r\n\r\n$ f(x) = \\frac {1}{2}\\sum^{\\infty}_{k = 1}B_{2k}\\frac {(2\\pi x)^{2k}} {(2k)! }$\r\n\r\n$ f(x) = \\frac {1}{2}\\sum^{\\infty}_{k = 0}B_{2k}\\frac {(2\\pi x)^{2k}} {(2k)! } - \\frac {1}{2}$\r\n\r\nand since\r\n\r\n$ \\sum^{\\infty}_{k = 0}B_{2k}\\frac {x^{2k}} {(2k)! } = \\frac {x}{2} \\coth(\\frac {x}{2})$\r\n\r\nwe get\r\n\r\n$ f(x) = \\frac {\\pi x}{2} \\coth(\\pi x) - \\frac {1}{2}$\r\n\r\nplugging $ x = 1$ yields:\r\n\r\n$ f(1) = \\sum^{\\infty}_{n = 1}\\frac {1}{n^2 + 1} = \\frac {\\pi}{2} \\coth(\\pi) - \\frac {1}{2}$", "Solution_6": "[hide=\"Answer\"]Provided I have made no mistake, \n\n$ \\sum_{n\\equal{}1}^{\\plus{} \\infty}\\frac{\\cos(nx)}{n^2 \\plus{} a^2} \\equal{} \\frac{\\pi}{2a}\\left[\\coth(a \\pi) \\cosh(ax) \\minus{} \\sinh(ax)\\right] \\minus{} \\frac{1}{2a^2}$[/hide]", "Solution_7": "Carcul, I calculated LHS and RHS for some special $ x$ and $ a$ with Maple.\r\n\r\nYour formula seems to be true. :) \r\n\r\nCan you tell us please how you deduced this formula.", "Solution_8": "By using Fourier series.", "Solution_9": "Let $ 00$ .\r\n\r\nAfter Poisson summation formula \r\n\r\n$ \\sum_{n\\in\\Bbb{Z}} \\frac{e^{inx}}{n^2\\plus{}\\alpha^2}\\equal{}\\sum_{n\\in\\Bbb{Z}} \\int_{\\minus{}\\infty}^\\infty \\frac{e^{itx}}{t^2\\plus{}\\alpha^2}\\, e^{2\\pi i n t}\\, dt\\equal{}\\frac{\\pi}{\\alpha} \\sum_{n\\in\\Bbb{Z}} e^{\\minus{}\\alpha |2\\pi n\\plus{}x|}$\r\n\r\n$ \\equal{}\\frac{\\pi}{\\alpha} \\left(\\sum_{n\\equal{}\\minus{}\\infty}^{\\minus{}1} e^{\\alpha (2\\pi n\\plus{}x)}\\plus{}\\sum_{n\\equal{}0}^\\infty e^{\\minus{}\\alpha(2\\pi n\\plus{}x)}\\right)$\r\n\r\n$ \\equal{}\\frac{\\pi}{\\alpha} \\left(e^{\\alpha x}\\sum_{n\\equal{}1}^\\infty e^{\\minus{}2\\pi\\alpha n}\\plus{}e^{\\minus{}\\alpha x}\\sum_{n\\equal{}0}^\\infty e^{\\minus{}2\\pi \\alpha n}\\right)$\r\n\r\n$ \\equal{}\\frac{\\pi}{\\alpha} \\left(\\frac{e^{\\alpha x}}{e^{2\\pi\\alpha}\\minus{}1}\\plus{}\\frac{e^{\\minus{}\\alpha x}}{1\\minus{}e^{\\minus{}2\\pi\\alpha}}\\right)\\equal{}\\frac{\\pi}{\\alpha}\\, \\frac{e^{\\alpha(x\\minus{}\\pi)}\\plus{}e^{\\minus{}\\alpha(x\\minus{}\\pi)}}{e^{\\pi\\alpha}\\minus{}e^{\\minus{}\\pi\\alpha}}$\r\n\r\nSo $ \\sum_{n\\in\\Bbb{Z}} \\frac{\\cos(nx)}{n^2\\plus{}\\alpha^2}\\equal{}\\frac{\\pi}{\\alpha}\\, \\frac{\\cosh\\big(\\alpha(x\\minus{}\\pi)\\big)}{\\sinh(\\alpha \\pi)}$\r\n\r\nAnd similarly $ \\sum_{n\\in\\Bbb{Z}} \\frac{(\\minus{}1)^n \\, \\cos(nx)}{n^2\\plus{}\\alpha^2}\\equal{}\\frac{\\pi}{\\alpha} \\,\\frac{\\cosh(\\alpha x)}{\\sinh(\\alpha \\pi)}$ for $ \\minus{}\\pi 0 \\end{matrix}\\right\\} \\plus{} 2\\pi i \\sum_{k \\equal{} 0}^\\infty \\left\\{\\begin{matrix} 0 & \\text{Re}(\\alpha) < 0 \\\\\n \\\\\ne^{ \\minus{} \\alpha x}\\, e^{ \\minus{} 2\\pi k\\alpha} & \\text{Re}(\\alpha) > 0 \\end{matrix}\\right\\}$\n\nSo for $ \\text{Re}(\\alpha) < 0 : \\quad S \\equal{} \\minus{} 2\\pi i \\, e^{ \\minus{} \\alpha x} \\sum_{k \\equal{} 1}^\\infty e^{2\\pi \\alpha k} \\equal{} 2\\pi i \\,\\frac {e^{ \\minus{} \\alpha x}}{1 \\minus{} e^{ \\minus{} 2\\pi \\alpha}}$\n\nAnd for $ \\text{Re}(\\alpha) > 0 : \\quad\\!\\! S \\equal{} 2\\pi i \\, e^{ \\minus{} \\alpha x} \\sum_{k \\equal{} 0}^\\infty e^{ \\minus{} 2\\pi \\alpha k} \\equal{} 2\\pi i\\, \\frac {e^{ \\minus{} \\alpha x}}{1 \\minus{} e^{ \\minus{} 2\\pi \\alpha}}$\n\nHence for $ 0 < x < 2\\pi$ and $ \\alpha\\in\\Bbb{C}\\setminus \\Bbb{R}$\n\n$ \\sum_{k\\in\\Bbb{Z}} \\frac {e^{ikx}}{k \\plus{} \\alpha} \\equal{} 2\\pi i\\, \\frac {e^{ \\minus{} i\\alpha x}}{1 \\minus{} e^{ \\minus{} 2\\pi i\\alpha}} \\equal{} \\pi \\, \\frac {e^{ \\minus{} i(x \\minus{} \\pi) \\alpha}}{\\sin \\alpha \\pi}$\n\nAfter shifting $ x$ to $ x \\plus{} \\pi \\quad \\sum_{k\\in\\Bbb{Z}} \\frac {( \\minus{} 1)^k\\, e^{ikx}}{k \\plus{} \\alpha} \\equal{} \\pi\\, \\frac {e^{ \\minus{} i\\alpha x}}{\\sin \\alpha\\pi}$ for $ \\minus{} \\pi < x < \\pi$ .\n\n[/hide]\nor with the [hide=\"residue theorem\"]\nFor $ \\minus{}\\pin+1$ implies $A=I$.", "Solution_1": "If it had an eigenvalue different from $1$, then it would have to have all $p-1$ primitive $p$-th roots of unity as eigenvalues, because $x^{p-1}+\\ldots+x+1$ is irreducible. This, however, is impossible, because $n 4$ should be $ n \\geq 4$; the first image they have there uses degree 4.", "Solution_2": "It might be that the image doesn't represent a hyperelliptic curve though - it's not clear which parts of the article to trust and which not to. For example - twice it is written $ n>4$ and once it is written \"if $ f$ is a cubic or quartic...\".", "Solution_3": "Yes, you may be right -- the talk page thinks it's a bad article:\r\nhttp://en.wikipedia.org/wiki/Talk:Hyperelliptic_curve\r\nAt least some of it (like one of the sentences you quote) is just pulled from Mathworld (or, I suppose, vice versa). Probably it is advisable to find a more authoritative source.", "Solution_4": "$ n > 4$ is correct. When $ f$ has degree $ d$ and distinct roots, the genus of the resulting curve is $ \\left\\lfloor \\frac{d\\minus{}1}{2} \\right\\rfloor$; in particular when $ d \\equal{} 4$ the genus is $ 1$, not $ 3$. You can verify this for yourself by constructing the corresponding [url=http://lamington.wordpress.com/2009/09/23/how-to-see-the-genus/]Newton polygon[/url]. Alternately, the genus is a birational invariant and you can show directly that if $ f$ has degree $ 2n$, then $ y^2 \\equal{} f$ is birational to a curve of the form $ y^2 \\equal{} g$ where $ g$ has degree $ 2n\\minus{}1$. (Essentially you send one of the roots of $ f$ to infinity.)\r\n\r\nYou may be being misled by the usual claim that a generic curve of degree $ d$ has genus $ \\frac{(d\\minus{}1)(d\\minus{}2)}{2}$. This formula only holds for smooth curves, and a projective curve of the form $ y^2 z^2 \\equal{} x^4 \\plus{} ax^3 z \\plus{} bx^2 z^2 \\plus{} cxz^3 \\plus{} dz^4$ has a singularity at $ (0 : 1 : 0)$, which you can verify by computing the partial derivatives there. (Similar singularities appear in higher degree.)", "Solution_5": "Thanks for the clarification, it seems the wiki page is in serious need of some attention.\r\n@ t0rajir0u: if we have a plane curve given by the equation $ y^{2}\\equal{}g(x)$ with the degree of $ g$ equal to $ 2n\\minus{}1$ then we may as well assume that 0 isn't a root of $ g$, and then the map sending $ (x,y)$ to $ (x^{\\minus{}1},yx^{\\minus{}n})$ is the map to which you refer, yes? (Also - Newton Polygons; very cool.)" } { "Tag": [ "calculus", "geometry unsolved", "geometry" ], "Problem": "one more for ya.....can't use calculus and have to use geometry. \r\n\r\nA rectangular sheet of paper is to contain 72 square inches of printed matter with 2-inch margins at the top and bottom and 1-in margis on each side. What dimensions for the sheet will use the least paper?", "Solution_1": "We have $ab=72$ and want to minimize $(a+2)(b+4)=ab+2b+4a+6=78+2b+4a$\r\nSo we must minimize $b+2a$. Note $b=\\frac{72}{a}$. Then we have to minimize $2a+\\frac{72}{a} \\implies a=6$, so $b=12$ , giving dimensions 6 x 12.\r\n\r\njmccray, I don't think this problem belongs in the olympiad section either. This one should probably go in getting started...", "Solution_2": "[quote=\"K81o7\"]We have $ab=72$ and want to minimize $(a+2)(b+4)=ab+2b+4a+6=78+2b+4a$\nSo we must minimize $b+2a$. Note $b=\\frac{72}{a}$. Then we have to minimize $2a+\\frac{72}{a} \\implies a=6$, so $b=12$ , giving dimensions 6 x 12.\n\njmccray, I don't think this problem belongs in the olympiad section either. This one should probably go in getting started...[/quote]Sorry for posting in the wrong forum, but I do have one last question. How do we get from minimizing 78+2b+4a to minimizing b+2a?", "Solution_3": "[quote=\"jmccray\"]Sorry for posting in the wrong forum, but I do have one last question. How do we get from minimizing 78+2b+4a to minimizing b+2a?[/quote]\r\n\r\nWell, $78+2b+4a = 78 + 2(b+2a)$. If you are able to find the maximum of $b+2a$, then that will correspond to the maximum of $2(b+2a)$ and hence also to the maximum of $78 + 2(b+2a)$. All operations you're doing with the number $b+2a$ preserve its condition of being maximum. Hope you undertand now\r\n\r\nDaniel" } { "Tag": [ "geometry", "geometry theorems" ], "Problem": "Is there something special for quadrilateral ABCD that has incircle and can be also inscribed in a circle (cyclic)\r\n(Espacially relations between sides and other special things about this kind of quadrilaterals). :?:", "Solution_1": "I think it must be cyclic (i.e. the sum of opposite angles \u00e0 180\u00b0) and exscribed (i.e. the sum of opposite sides are equal ). :roll:", "Solution_2": "[quote=\"mstoenescu\"]I think it must be cyclic (i.e. the sum of opposite angles \u00e0 180\u00b0) and exscribed (i.e. the sum of opposite sides are equal ). :roll:[/quote]\r\n\r\nVERY good answer. :rotfl:\r\n\r\nIn fact, such quadrilaterals are called \"bicentric quadrilaterals\", and searching for \"bicentric\" will yield a number of nice problems about them.\r\n\r\nhttp://www.mathlinks.ro/Forum/viewtopic.php?t=146867 (dear mods, this does NOT belong into Intermediate Topics)\r\nhttp://www.mathlinks.ro/Forum/viewtopic.php?t=31693\r\nhttp://www.mathlinks.ro/Forum/viewtopic.php?t=5510\r\nhttp://www.mathlinks.ro/Forum/viewtopic.php?t=23192\r\n\r\nare maybe the most important ones.\r\n\r\n Darij", "Solution_3": "[quote]VERY good answer. \n\n[/quote]\r\n\r\nI haven't lough this much for a long time before but when I saw this....\r\nMaybe he didn't understand me well (he thought that I am begginer).\r\n\r\nMany thanks to Darij and especially to mstoenescu.", "Solution_4": "Many thanks to Darij who opens to me a fantastic world of these quadrilaterals by the topics founded !\r\n\r\nHave a lot of fun delegat ! The beginner is ME and I'm glad to be ! And long live to Sarajevo ! :blush:", "Solution_5": "[quote]Many thanks to Darij who opens to me a fantastic world of these quadrilaterals by the topics founded ! \n\nHave a lot of fun delegat ! The beginner is ME and I'm glad to be ! And long live to Sarajevo ! [/quote]\r\n\r\nOK I didn't want to write anything bad to you \r\nMy question was about these bicentric quadrialterals but you gave answer to cyclic quadrialteral and then to quadrilateral that can be exscribed (no relation between them) \r\nYour answer didn't make me laufhing Darij's answer especially :rotfl: did it.\r\nMaybe you wrongly understood my question so your answer was...\r\n\r\nWhat do you have against Sarajevo?", "Solution_6": "Sorry for leading this topic into a flame, but I mistook Mstoenescu's post as a spoof of Delegat. I hope everything is settled now.\r\n\r\n Darij", "Solution_7": "[quote=\"delegat\"]\n\nWhat do you have against Sarajevo?[/quote]\r\n\r\nI think you misunderstood me about that ! On the contrary, I'm sympathizer of Sarajevo !", "Solution_8": "[quote]I think you misunderstood me about that ! On the contrary, I'm sympathizer of Sarajevo !\n[/quote]\r\n\r\nOK Then it is obvious that I misunderstood you I though that you don't like Sarajevo or something like that.\r\n\r\nI think that we can close this topic (either if somebody has something about bicetric quadrilaterals)." } { "Tag": [ "SFFT" ], "Problem": "Find three natural numbers a, b, c such that:\r\n\\[ 4(a\\plus{}b\\plus{}c) \\plus{}14 \\equal{} abc\\]", "Solution_1": "got lucky and found the answer\r\n[hide=\"answer\"]\n2, 5, 7[/hide]\r\nnow trying to find a reason", "Solution_2": "[hide=\"Mathematica\"]\n[code]Timing[For[a =\n 2, a \u2264 500, a += 2, {For[b = 1, b \u2264 500, b++, {For[c = \n b, c \u2264 500, c++, If[4(a + b + c) + 14 == a b c, Print[{a, b, c}]]]}]}]][/code]\n\n{2,3,17}\n{2,5,7}\n{6,1,21}\n{38,1,5}\n\n{464.027 Second,Null}[/hide]", "Solution_3": "[hide=\"Solution inspired by above\"]WLOG, let $ a\\leq b\\leq c$. Putting $ a \\equal{} 1$ then $ a \\equal{} 2$ and using SFFT gives the 4 solutions from above: $ (1,5,38), (1,6,21), (2,3,17), (2,5,7)$.\nThe case $ a \\equal{} 3$ gives $ 26 \\plus{} 4b \\plus{} 4c \\equal{} 3bc$. Let $ b \\equal{} x \\plus{} 2$, $ c \\equal{} y \\plus{} 2$, so that we can solve for $ x,y$ in naturals instead of for values $ \\geq 3$. Also, for easier solving, drop the restriction that $ x\\geq y$.\nThen $ 30 \\plus{} 4x \\plus{} 4y \\equal{} 3xy \\plus{} 6x \\plus{} 6y \\plus{} 12 \\iff 18 \\equal{} 3xy \\plus{} 2x \\plus{} 2y$. Examining this mod 2, we can WLOG let $ x \\equal{} 2z$. Then $ 18 \\equal{} 6yz \\plus{} 4z \\plus{} 2y \\iff 9 \\equal{} 3yz \\plus{} y \\plus{} 2z \\equal{} y(3z \\plus{} 1) \\plus{} 2z$. Examining this mod 2, we can let $ z \\equal{} 2w$. Then $ 6 \\equal{} 6wy \\plus{} y \\plus{} 4w\\geq11$. Thus, no solutions exist for $ a \\equal{} 3$.\nFor the case $ a \\equal{} 4$, let $ a \\equal{} t \\plus{} 4$, where $ t\\in \\mathbb{N}_0$.\n(Edit: Then let $ x\\equal{}t$. :mad: )\nThen the equation is equivalent to $ 34 \\plus{} 4x \\equal{} 4(b \\minus{} 1)(c \\minus{} 1) \\plus{} xbc$. We have $ 4(b \\minus{} 1)(c \\minus{} 1)\\geq 4\\cdot3\\cdot3 \\equal{} 36 > 34$, and $ xbc\\geq16x\\geq4x$, so this equation can never be satisfied. Thus, the only solutions are those 4 solutions and permutations.[/hide]\r\n:P Mathematica pwns." } { "Tag": [ "induction" ], "Problem": "Find all [tex]f:\\mathbb{N}\\,_0\\to\\mathbb{N}\\,_0[/tex],\r\nsuch that\r\n[tex]f(f(n)) + f(n) = 2n+6[/tex],\r\nwhere\r\n[tex]\\mathbb{N}\\,_0=\\mathbb{N}\\,\\cup\\left\\{0\\right\\}[/tex]", "Solution_1": "Well, heres a way that will work but is a little bit heavy on the algebra side, so I haven't worked 100% of the way through it:\n\n[hide]\n\nLet f(a) = b for any a. Then substitute a into the equation to get f(b) = 2a-b+6. Then substitute b into the equation to get f(2a-b+6) = 3b-2a. \n\nWe then continue this repeatedly, and prove by induction the pattern we find. This isn't as bad as it looks, to get the a and b terms involve doubling the previous one and possibly adding an a or b, the constant term i'm not 100% sure about, but there will be a pattern.\n\n\n\nAnyway, after working out the formula by induction, set each term >=0. Every second term will give us a lower bound for b, every other term an upper bound. Take these to infinity, and I'm 99.9% sure that they will both converge to b = a+2.\n\n[/hide]", "Solution_2": "That is similar to one of the solutions I have seen, although [hide] it does not take it to infinity, but rather stops it after a finite number of steps[/hide]. See what you can do with that. There are four solutions in this book I have, along with one that I came up with.\n\n\n\nAnd any solution you can find will be algebra-heavy, or case-heavy.\n\n\n\nMine is case-heavy. (By heavy I mean about 7 small proofs by contradiction)" } { "Tag": [ "topology", "superior algebra", "superior algebra unsolved" ], "Problem": "How can I show Gal (E/F) where F 2 and n > 1. Prove that if the set {1,2,3...mn} be partitioned into m sets each of size n in any manner whatsoever, then we can select 1 element from each set such that the sum of [m/2] of the elements equals the sum of the remaining elements.\r\n\r\nElse find a counterexample.\r\n\r\nThe problem is solved only for the cases m = 3 and m = 4", "Solution_1": "m=6 gives a counterexample.\r\nConsider a partition $\\{1,2,\\dots,mn\\}= P_{0}\\cup P_{1}\\cup\\dots\\cup P_{m-1}$ where $P_{i}$ contains all elements $x$ such that $x\\equiv i\\pmod m$.\r\nSuppose that we selected elements $x_{0}, x_{1}, \\dots, x_{m-1}$ from the sets $P_{0}, P_{1}, \\dots, P_{m-1}$ respectively. Then \\[x_{0}+x_{1}+\\dots+x_{m-1}\\equiv 0+1+\\dots+m-1 \\equiv \\frac{m(m-1)}{2}\\pmod{m}\\] implying that $x_{0}+x_{1}+\\dots+x_{m-1}$ is odd (for $m=6$). Therefore, a set $\\{ x_{0}, x_{1}, \\dots, x_{m-1}\\}$ cannot be split into two with equal sums.", "Solution_2": "Yes, your counterexample does work for $n=6$. It works for any number of the form $4n+2$.\r\n\r\nCan we make then prove the positive result for numbers that are not of the form $4n+2$? Or find a counterexample there as well?" } { "Tag": [ "linear algebra", "matrix", "complementary counting", "algebra", "system of equations" ], "Problem": "How many $ 0\\minus{}1$ sequences of length $ n$ there are, which don't have three consecutive $ 1$?\r\n\r\nFor $ n\\equal{}4$ such sequence is for example $ 1011$, but $ 0111$ doesn't fulfill conditions.", "Solution_1": "By complementary counting and the principle of inclusion-exclusion, I have $ 2^{n}\\minus{}2^{n\\minus{}3}(n\\minus{}2)\\plus{}2^{n\\minus{}4}(n\\minus{}3)\\minus{}2^{n\\minus{}5}(n\\minus{}4)\\plus{}...$.", "Solution_2": "[hide=\"Recursion approach\"]For $ n\\equal{}3$ we have $ a_3\\equal{}2^3\\minus{}1\\equal{}7$ that work. These are:\n\n000\n001\n010\n100\n011\n101\n110\n\nNow we'll think about $ a_4$. In the cases ending with 00, 01, or 10, we may tack on either a 1 or a 0 and create two more numbers that work. In the cases ending with 11, we can add a 0 and get a number that works.\n\nSo now let's define a sequence $ b_n$ of the number that end with 00, or 10. We define $ c_n$ as the number that end in $ 11$. Also, let $ d_n$ be the number that end in $ 01$. (This'll make sense in a second). We have that $ a_n\\equal{}b_n\\plus{}c_n\\plus{}d_n$, of course. We also have that $ a_{n\\plus{}1}\\equal{}2(b_n\\plus{}d_n)\\plus{}c_n$.\n\nWe see that $ b_3\\equal{}4$, $ c_3\\equal{}1$, and $ d_3\\equal{}2$ Based on that, $ a_4\\equal{}13$, which we can easily verify is correct, so we're on the right track.\n\nNow we can see that if a number ends with 00 or 10, if we get two $ b$ values from it and two $ d$ values. If it ends with 01 or 11 we'll get a $ c$ value and a $ b$ value but no $ d$.\n\nSo $ b_{n\\plus{}1}\\equal{}2b_n\\plus{}c_n\\plus{}d_n$\n\nAnd $ c_{n\\plus{}1}\\equal{}c_n\\plus{}d_n$\n\nAnd $ d_{n\\plus{}1}\\equal{}2b_n\\plus{}c_n$.[/hide]\r\n\r\nRight, so, that got kind of ugly... Finding a closed form based on that would probably be difficult, but if you needed to find the value for some small $ n>3$ you could use this to generate values somewhat quickly.", "Solution_3": "[quote=\"grn_trtle\"][hide=\"Recursion approach\"]For $ n \\equal{} 3$ we have $ a_3 \\equal{} 2^3 \\minus{} 1 \\equal{} 7$ that work. These are:\n\n000\n001\n010\n100\n011\n101\n110\n\nNow we'll think about $ a_4$. In the cases ending with 00, 01, or 10, we may tack on either a 1 or a 0 and create two more numbers that work. In the cases ending with 11, we can add a 0 and get a number that works.\n\nSo now let's define a sequence $ b_n$ of the number that end with 00, or 10. We define $ c_n$ as the number that end in $ 11$. Also, let $ d_n$ be the number that end in $ 01$. (This'll make sense in a second). We have that $ a_n \\equal{} b_n \\plus{} c_n \\plus{} d_n$, of course. We also have that $ a_{n \\plus{} 1} \\equal{} 2(b_n \\plus{} d_n) \\plus{} c_n$.\n\nWe see that $ b_3 \\equal{} 4$, $ c_3 \\equal{} 1$, and $ d_3 \\equal{} 2$ Based on that, $ a_4 \\equal{} 13$, which we can easily verify is correct, so we're on the right track.\n\nNow we can see that if a number ends with 00 or 10, if we get two $ b$ values from it and two $ d$ values. If it ends with 01 or 11 we'll get a $ c$ value and a $ b$ value but no $ d$.\n\nSo $ b_{n \\plus{} 1} \\equal{} 2b_n \\plus{} c_n \\plus{} d_n$\n\nAnd $ c_{n \\plus{} 1} \\equal{} c_n \\plus{} d_n$\n\nAnd $ d_{n \\plus{} 1} \\equal{} 2b_n \\plus{} c_n$.[/hide]\n\n[/quote]\r\n\r\nlet $ a_n$ be the number of sequences of length $ n$.\r\n\r\nIf we take any of the $ a_n$ sequences of length $ n$ we can add a $ 0$ to the start to make a sequence length $ n \\plus{} 1$\r\n\r\nWe can also add a $ 1$ to the front of the sequence given it doesn't start with $ 11$\r\n\r\nThe number of sequences that start with $ 11$ is $ a_{n \\minus{} 3}$: because if a seqence starts with $ 11$ then it must start $ 110$ then if we append any of the $ a_{n \\minus{} 3}$ sequences to $ 110$ we get our set.\r\n\r\nHences $ a_{n \\plus{} 1} \\equal{} 2a_n \\minus{} a_{n \\minus{} 3}$", "Solution_4": "[quote=\"grn_trtle\"]Right, so, that got kind of ugly... Finding a closed form based on that would probably be difficult, but if you needed to find the value for some small $ n > 3$ you could use this to generate values somewhat quickly.[/quote]\r\nIt's not ugly at all! The system of equations you write down can be written in matrix form, and then the question is how to compute arbitrary powers of a fixed matrix. This is done by finding its eigenvalues and then diagonalizing, which is a standard computation in linear algebra." } { "Tag": [ "\\/closed" ], "Problem": "I want this as my sig:\r\n\r\n[code]There are 10 kinds of mathematicians. Those who can think binarily and those who can't.\n\n[url]http://www.vf11.com/nebula/nebula.html[/url][/code]\nBut it says:\n\n[code]Signature: the value is too long.[/code]", "Solution_1": "Well, I don't know why this happened, but you should probably try to be more polite. :wink:", "Solution_2": "We've shortened the length of allowable signatures. This is only for new signatures -- if you had a long signature before we made the change, it's grandfathered in (although we still, as always, reserve the right to edit your signature without warning if it's inappropriate). \r\n\r\nAlthough the signature that you wrote above is not abusive nor objectionable (the link is OK, since it's not a blind link -- anybody who clicks on it knows that it is taking them off-site), it is too long under the new policy. It must be shorter than 128 characters." } { "Tag": [], "Problem": "Hey everybody, I have a couple questions regarding the USNCO free response section.\r\n\r\nWhat kind of criteria do they grade the free response on?\r\nDo they pay a lot of attention to sig figs?\r\nDo they want you to write out every equation you use?\r\nShould you write out all your reasoning behind using the equation?\r\nShould states of matter be written for equation writing, and just for substances in general?\r\nIf anyone has any tips or pointers regarding the free response section regarding the kind of format the graders are looking for, it'd be greatly appreciated. Thanks!", "Solution_1": "does anybody know the answer to this question?", "Solution_2": "ACS has managed to never reveal any details about the national exam...\r\nMy only advice is therefore to be as accurate as possible... obviously dont write down something without explaining it... and write as much MEANINGFUL information you can. Im assuming that some old grumpy chemistry teachers will be grading this and after grading hundreds of papers, they arent going to be happy seeing a paper with sloppy handwriting and little explanation." } { "Tag": [ "geometry", "3D geometry", "algebra", "factorization", "difference of cubes", "special factorizations" ], "Problem": "I understand that when simplifying $\\frac{\\sqrt{x+h}-\\sqrt{x}}{h}$ , you use the conjugate to rationalize the numerator. Likewise when taking the cube root of x+h - the cube root of x, you use the difference of cubes formula. Or, when taking the fourth root, you use the conjugate concept twice.\r\n\r\nBut, I am confused when it comes to fifth roots.\r\nI want to rationalize $\\sqrt[5]{(x+h)^{2}}-\\sqrt[5]{x^{2}}$, but I cannot figure out how to do it. Help?", "Solution_1": "[hide=\"My initial thoughts on this\"]The difference of cubes formula can be extended.\n\ne.g.\n$x^{5}-y^{5}=(x-y)(x^{4}+x^{3}y+x^{2}y^{2}+xy^{3}+y^{4})$\n\nOr\n\n$x^{n}-y^{n}=(x-y)(x^{n}+x^{n-1}y+x^{n-2}y^{2}+\\ldots+xy^{n-2}+y^{n-1})$\n\n[/hide]", "Solution_2": "Try using rational exponents:\r\n$(x+h)^{2/5}-x^{2/5}$\r\n$((x+h)^{1/5})^{2}-(x^{1/5})^{2}$\r\nThis is a conjugate using squares.\r\nSay our whole fraction\r\n$\\frac{(x+h)^{2/5}-x^{2/5}}{h}$\r\nneeds to be rationalized.\r\nWell, we see that:\r\n$\\frac{((x+h)^{1/5})^{2}-(x^{1/5})^{2}}{h}$\r\nWe can use a conjugate of squares.\r\n\r\nMultiply by the conjugate of ${((x+h)^{1/5})^{2}-(x^{1/5})^{2}}$. \r\nThe conjugate is:\r\n${((x+h)^{1/5})^{2}+(x^{1/5})^{2}}$\r\nThen work on eliminating the fifth roots." } { "Tag": [ "inequalities" ], "Problem": "Triangle ABC is acute\r\n\r\n\r\nProve SQRT(a^2 +b^2 -c^2) + SQRT (b^2 +c^2 -a^2) +SQRT(c^2+a^2-b^2)\r\n\r\n<= a+b+c", "Solution_1": "Applying the AM-QM inequality to the numbers $\\sqrt{c^{2}+a^{2}-b^{2}}$ and $\\sqrt{a^{2}+b^{2}-c^{2}}$, we get\r\n\r\n$\\frac{\\sqrt{c^{2}+a^{2}-b^{2}}+\\sqrt{a^{2}+b^{2}-c^{2}}}{2}\\leq \\sqrt{\\frac{\\left( c^{2}+a^{2}-b^{2}\\right) +\\left( a^{2}+b^{2}-c^{2}\\right) }{2}}=\\sqrt{\\frac{2a^{2}}{2}}=\\sqrt{a^{2}}=a$,\r\n\r\nand thus\r\n\r\n$\\sqrt{c^{2}+a^{2}-b^{2}}+\\sqrt{a^{2}+b^{2}-c^{2}}\\leq 2a$.\r\n\r\nSimilarly,\r\n\r\n$\\sqrt{a^{2}+b^{2}-c^{2}}+\\sqrt{b^{2}+c^{2}-a^{2}}\\leq 2b$\r\n\r\nand\r\n\r\n$\\sqrt{b^{2}+c^{2}-a^{2}}+\\sqrt{c^{2}+a^{2}-b^{2}}\\leq 2c$.\r\n\r\nSumming up, we get\r\n\r\n$2\\cdot \\left( \\sqrt{b^{2}+c^{2}-a^{2}}+\\sqrt{c^{2}+a^{2}-b^{2}}+\\sqrt{a^{2}+b^{2}-c^{2}}\\right) \\leq 2\\cdot \\left( a+b+c\\right) $,\r\n\r\nand thus\r\n\r\n$\\sqrt{b^{2}+c^{2}-a^{2}}+\\sqrt{c^{2}+a^{2}-b^{2}}+\\sqrt{a^{2}+b^{2}-c^{2}}\\leq a+b+c$,\r\n\r\nproving your inequality.\r\n\r\n Darij" } { "Tag": [ "geometry", "geometric transformation", "rotation", "Gamebot" ], "Problem": "An international meeting is held between England, Germany, and France. Three representatives attend from England, four from Germany, and two from France. How many ways can all nine representatives sit around a circular table, if representatives of the same country sit together?", "Solution_1": "this problem makes me feel really stupid... :!:", "Solution_2": "Well, there are $ 3$ different countries, so we can order them in $ 3!$ or $ 6$ ways. There are $ 3!$ or $ 6$ ways to rearrange the English. There are $ 4!$ or $ 24$ ways to arrange the German. There are $ 2!$ or $ 2$ ways to arrange the French. However, we must divide by $ 9$ to account for rotations, so we have $ 6 \\times 6 \\times 24 \\times 2 \\div 9$ or $ \\boxed{192}$ ways.", "Solution_3": "wow. ernie. that's the same answer/solution i got....and guess what? WRONG.\r\ni don't get why my solution is wrong....help please? :lol:", "Solution_4": "Well, what's the answer?\r\nAnd,\r\nErnie,why did you divide by nine?\r\n\r\nEdit: Oh, is it because the table is round? You would only have 2 ways of seating the countries.", "Solution_5": "Yes, it is because the table is round. Also, the answer isn't two, because you need to worry about how everyone sits within their country group.\r\n\r\nAre you sure it's not just $ 4!*3!*2!/9 \\equal{} 32$? In my opinion, this would make sense. Why even multiply by 6? I don't think there is anymore than 1 way to organize the country groups.\r\n\r\nBwu- what was the answer?", "Solution_6": "Er....the answer was $\\boxed{576}$. You start out by considering the countries in blocks, like Germany one block, England on block, and France one block. You have $(3-1)!=2$ ways to order them around the table. Then you consider how many ways to order the English group, $3!=6$, then $4!=24$ for Germans, and then $2!=2$ for the French. There for we have $2!\\times3!\\times4!\\times2!=2\\times6\\times24\\times2=\\boxed{576}$\n\n[hide=\"Click here for the official answer\"]To begin, consider the number of ways to arrange the three countries around the circle. We can consider the English representatives a block, the Germans another block, and the French a third block. There are $(3-1)!=2$ ways to arrange these three blocks around a circle. Within the English group, there are $3!=6$ ways to arrange the three representatives. Similarly, there are $4!$ ways to arrange the Germans and $2!$ ways to arrange the French representatives. Overall, the total number of ways to seat the 9 representatives is: $2!\\times3!\\times4!\\times2!=2\\times6\\times24\\times2=\\boxed{576}$[/hide]", "Solution_7": "Why would it not be 576/9 accounting for rotations or 64 ways to seat people at the table?", "Solution_8": "Can someone explain this? My logic was that treating each country as a block, there are 3! ways to orient the blocks, 3! ways to orient the representatives from England, 4! ways for the representatives from Germany, and 2! ways for the representatives from France. This gives us 3! x 3! x 4! x 2!, but we must divide by 9 to account for rotations, giving us 192. I still don't see what's wrong with this solution.", "Solution_9": "Dividing by 9 is accounting for individual rotations. However, note that we are rotating the \"blocks\" or countries, not the individual representatives themselves. You already multiplied by 3! to find the ways to rotate the countries. ", "Solution_10": "[quote=pinkmuskrat]Yes, it is because the table is round. Also, the answer isn't two, because you need to worry about how everyone sits within their country group.\n\nAre you sure it's not just $ 4!*3!*2!/9 \\equal{} 32$? In my opinion, this would make sense. Why even multiply by 6? I don't think there is anymore than 1 way to organize the country groups.\n\nBwu- what was the answer?[/quote]\n\nwait this one is not in the contry groups you mutiplyed", "Solution_11": "Sorry for the bump, but [hide=here's an alternative.]We can think of grouping the representatives from England, the ones from Germany, and the ones from France. We have $3!$ ways for England, $4!$ ways for Germany, and $2!$ ways for France. We can arrange these groups in $3!$ ways, however, because of rotational symmetry we have to divide by $3.$ Thus, we get an answer of \n$$\\frac{3!4!2!3!}{3}=\\boxed{576}.$$[/hide]", "Solution_12": "I had a different answer. :) \n\n[hide=solution]just counted the amount of ways to rearrange them around the circle. This becomes 2, counterclockwise, and clockwise. So then there are 3! = 6 ways to arrange England, 4! = 24 ways to arrange Germany, and 2! = 2 ways to arrange France. Therefore, there are $2\\cdot6\\cdot24\\cdot2 = 576$ ways to arrange them around the round table. Tip: The formula that AoPs posted in the solution for finding how many ways to rearrange $n$ amount of people is very helpful! [/hide]", "Solution_13": "[quote=pilover123]Can someone explain this? My logic was that treating each country as a block, there are 3! ways to orient the blocks, 3! ways to orient the representatives from England, 4! ways for the representatives from Germany, and 2! ways for the representatives from France. This gives us 3! x 3! x 4! x 2!, but we must divide by 9 to account for rotations, giving us 192. I still don't see what's wrong with this solution.[/quote]\n\neven though this user posted 3 years ago\nthe mistake was you had to multiply that by the number of ways to arrange them\nthere are 3!=6 ways to arrange them and you have to divide by 2 to account for rotation so the answer is 576", "Solution_14": "Redact or hide please. Thank you!", "Solution_15": "[quote=MinecraftArchitect]Redact or hide please. Thank you![/quote]\n\nYou don't really have to in a GameBot thread, but thanks. :D\n\nAlso, I think it is $\\frac{2*6*6*24}{3}=\\boxed{576}$", "Solution_16": "uh the first post here is from January 2009. This is still going on??!!!!?? :what?: :wacko: ", "Solution_17": "[quote=ThunderStrike314][quote=MinecraftArchitect]Redact or hide please. Thank you![/quote]\n\nYou don't really have to in a GameBot thread, but thanks. :D\n\nAlso, I think it is $\\frac{2*6*6*24}{3}=\\boxed{576}$[/quote]\n\nNot redacting or hiding technically counts as cheating.", "Solution_18": "This was made 8 days after gamebot joined" } { "Tag": [ "number theory", "least common multiple" ], "Problem": "Find the least common multiple of 36, 48, and 27.", "Solution_1": "36=3*3*2*2\r\n48=2*2*2*2*3\r\n27=3*3*3\r\nlcm=3*3*3*2*2*2*2=432" } { "Tag": [ "geometry", "3D geometry", "rectangle", "conics", "ellipse", "analytic geometry", "graphing lines" ], "Problem": "1. Find the equation of a perpendicular bisector of a segment with endpoints 7,1 and 3, -6.\r\n\r\n3. A circular Wheel with radius 6/ pi is rolled 216 units. How many revolutions occured?\r\n\r\n2. Given:Triangle ABC where angle A = 60 degrees, AB = AC and BC=4. Find the square of the area of a triangle\r\n\r\n11. A cube has a side length of 10. Find the length of tis longest internal diagonal\r\n\r\n10. Find the number of diagonals in a 12-gon.\r\n\r\n12. circle is circumscribed about a right triangle with legs 7 and 24, What is the circle's area?\r\n\r\n9.Find the area of a pentagon if all sides equal 5 and 2 of the adjacent angles are both right\r\n\r\n15. A solid box is polaced on the floor. ITs dimensions are 11x1x12. If an insect lands on one corner of the box, what is the length of the shortest route to the corner diagonally opposite the starting corner? \r\n\r\n17. A circle has a circumference of 10. Find the area of an inscribed hexagon.\r\n\r\n21. Cowboy bob is an outhouse 10 units south of from a linear river that travels east to west. He needs to pick up his horse which is 14 units south of the river. If wBob would like to wash his hands in the river before picking up his horse, what is the least distance he could travel in the entire trip from his outhouse to his horse?\r\n\r\n23. Mason builds a fence for his dog Nathan. It is shaped like a rectangle with width 30 and lenght 3 root 3. The entrance has been closed. If nathaniel is attached to an inside corner in this fence by a leash of length 6, how much area can he cover?\r\n\r\n6. In a plane the set of all points distance 3 from a line forms: \r\n\r\nan ellipse, circle, line , line segment, or NOTA", "Solution_1": "#1[hide]The midpoint is $(5,-\\frac{5}{2})$. The equation of the line, which we can determine by fairly straightforward algebra, is $y=\\frac{7}{4}x-\\frac{45}{4}$. Thus, the slope of the perpendicular bisector is $-\\frac{4}{7}$. Since it passes through the point $(5,-\\frac{5}{2})$, we set up the line $y=mx+b$ and substitute, to get $y=-\\frac{4}{7}+\\frac{5}{14}$.[/hide]\n#3[hide]The radius is $\\frac{6}{\\pi}$, so the diameter is 12. Thus the the number of revolutions is $216/12=18$.[/hide]\n#2[hide]By the isosceles triangle theorem (I don't know the name, I think it's Euclid I.5), the base angles are equal, and the triangle is equilateral. We are given that one side is 4, so the area is $4\\sqrt{3}$.[/hide]\n#11[hide]We use the 3-D Pythagorean theorem, to get the interior diagonal as $10\\sqrt{3}$. [/hide]\n#10[hide]The formula for this is $\\frac{n(n-3)}{2}$. Subsituting $n=12$, we have 54 diagonals. [/hide]\n#12[hide]Since this is a right triangle, the hypotenuse is the diameter of the circle. By the Pythagorean Theorem, the hypotenuse is $\\sqrt{7^{2}+24^{2}}=25$, so the radius of the circle is $12.5$, making the area of the circle $156.25\\pi$.[/hide]\n#9-I don't really understand the wording of this question.\n\n#15[hide]Unfold the box. We have three possible routes that make right triangles, one has legs 1, 23, one has legs 11, 13, the other has legs 12, 12. We can easily check that the one with legs 12, 12 is the most efficient route. Thus the shortest possible distance is $\\sqrt{12^{2}+12^{2}}=12\\sqrt{2}$.[/hide]\n#17[hide]The circumference is 10, so the radius is $\\frac{5}{/pi}$. If we draw six radii 60 degrees apart and connect the points in which they intersect the circle, we get a regular hexagon. Thus, the side length of the regular hexagon is the same as the radius, $\\frac{5}{\\pi}$, and the area is $\\frac{75}{2\\pi^{2}}$[/hide]\n#21-I think the problem needs to be explained better...\n#23-I'm gonna have to think about this a little more.\n\n#6[hide]I believe it's 2 lines, to NOTA.[/hide]", "Solution_2": "Ehh I hope these are right...\r\n[hide=\"9\"]Basically one of those house style pentagons, with the two base right angles. So it's an equilateral triangle with length five on top of a square with length five, the total area is $\\frac{25\\sqrt{3}}{4}+100.$[/hide]\n\nFor 21, they don't specify how far away the horse is...I'm not sure if that matters or not...\n\n[hide=\"22\"] See the attachment at the bottom. The area of the larger 3/4 circle is $36\\pi \\times \\frac{3}{4}=27\\pi$ The area of the smaller 1/4 circle is $\\frac{(6-3\\sqrt{3})^{2}\\pi}{4}=(4-\\sqrt{3})\\pi.$ Adding these together, we have the answer as $(31-\\sqrt{3})\\pi$ Problems like these come on practically every Vestavia test.[/hide]\r\nI'll post the others' as I figure them out...\r\n13375P34K, you should check your answer for number 2, it was asking for the square of the area.\r\nI believe you have a typo in number 17.", "Solution_3": "and then some... nice explanations by these guys =) I am learning\r\n\r\n4. Find the difference between the area of the sold and shaded regions of this parallelogram with an area of 36. It has two sides of 12 and two sides of 5, like a rombus and 2 diagonals.\r\n\r\n5. Find the length of the common external tangent of 2 circles of radii 5 and 17 with distance of 20 between their centers.\r\n\r\n7. A cone standing on its vertex of height 4 and radius 3 is filled halfway by volume with kool aid. What is the cube of the height of the filled portion?\r\n\r\n8. Find the sum of the altitudes in a triangle of sides 5, 12, 13. \r\n\r\n13. A right triangle has one leg of length 10 and another of a random length between 0 and 10. What is the probability that the square units of area exceeds the units in the length of the hypotenuse?\r\n\r\n14. How many triangles exist with integral sides , two sides of length 6 and an area greater than 18. \r\n\r\n16. Ozymandias wants to build a set of pyramids to rival those at Giza. To do this first builds a regular tetrahedron with edge length of 1 the first year. Every following year he builds a pyramid of edge length twice the previous year's pyramid. he planned to do this forever but his rule only lasted 3 years. Assuming SOLID pyramids, what volume of stone was required for 3 regular tetrahedrons? \r\n\r\n18. y=[x] represents the greatest integer function where y is equal to the greatest integeer less than or equal to x. Find the shortest distance from the graph of y equals x to the point (13/8,13/8)\r\n\r\n20. Find the \uff56\uff4flume of the #d graph represented by all points distance 3 from the x axis enclosed by the planes x equals 999 and x equals 1002.\r\n\r\n24. A woman builds a cubical pool of side 12 and fills it 3/4 of the way. Then she marks the water line on the side of the pool. She takes a large solid cylinder with height 6 and radius 1 and dips it into the pool such that its base is parallel to the pool's floor and that its lower base is 3 units below her marked water line. What volume of the cylinder is beneath the water?", "Solution_4": "WHOA maybe I'll do this tomorrow...", "Solution_5": "after this , I might post some problems I had on a theta geometry test \r\n\r\n\r\nI got number 5, it is 16 after applying formula d squared minus 2 radiis absolute value squared = ext tangent squared" } { "Tag": [ "logarithms", "limit", "integration", "calculus" ], "Problem": "A friend of mine gave me this one (among a couple others)\r\n\r\nLet G(n) be the the gemetric mean of the sequence...\r\n\r\n\r\n nC0, nC1, ... nCn-1\r\n\r\nProve:\r\n lim(n->infinity) G(n)^(1/n) = e^(1/2)", "Solution_1": "After taking [tex]\\ln[/tex] and do some calculation,\r\n\r\n[tex]\\displaystyle \\lim_{n\\rightarrow\\infty}\\frac{1}{n}\\ln{G(n)} =2\\int_0^1x\\ln{x}\\,dx-\\int_0^1\\ln{x}\\,dx = \\frac{1}{2}.[/tex]", "Solution_2": "[quote=\"fuzzylogic\"]After taking [tex]\\ln[/tex] and do some calculation,\n\n[tex]\\displaystyle \\lim_{n\\rightarrow\\infty}\\frac{1}{n}\\ln{G(n)} =2\\int_0^1x\\ln{x}\\,dx-\\int_0^1\\ln{x}\\,dx = \\frac{1}{2}.[/tex][/quote]\r\n\r\ncould you show some more steps?\r\n\r\nalso, isn't the last integral improper?", "Solution_3": "[quote=\"VeCTer\"]could you show some more steps?\n[/quote]\r\n\r\n[tex]\\displaystyle G^n(n) = {n\\choose{0}}{n\\choose{1}}\\cdots{n\\choose{n-1}}=\\frac{(n!)^{n+1}}{(1!2!\\cdots n!)^2} = \\frac{(n!)^{n+1}}{(1^n2^{n-1}\\cdots n^1)^2}[/tex]\r\n\r\n[tex]\\displaystyle {n}\\ln{G(n)} = (n+1) \\sum_{i=1}^n\\ln{i} - 2\\sum_{i=1}^n (n+1-i)\\ln{i}[/tex] \r\n[tex]\\displaystyle = 2\\sum_{i=1}^n i\\ln{i} - (n+1)\\sum_{i=1}^n \\ln{i}[/tex] \r\n[tex]\\displaystyle = 2\\sum_{i=1}^n i\\ln{\\frac{i}{n}} - (n+1)\\sum_{i=1}^n \\ln{\\frac{i}{n}}[/tex]\r\n\r\n[tex]\\displaystyle {\\frac{1}{n}}\\ln{G(n)} = 2\\left(\\frac{1}{n}\\sum_{i=1}^n\\frac{i}{n}\\ln{\\frac{i}{n}}\\right) - \\frac{n+1}{n}\\left(\\frac{1}{n}\\sum_{i=1}^n \\ln{\\frac{i}{n}}\\right)[/tex]\r\n\r\n[tex]\\displaystyle \\lim_{n\\rightarrow\\infty}\\frac{1}{n}\\ln{G(n)} =2\\int_0^1x\\ln{x}\\,dx-\\int_0^1\\ln{x}\\,dx = \\frac{1}{2}.[/tex]", "Solution_4": "[quote=\"VeCTer\"]\nalso, isn't the last integral improper?[/quote]\r\n\r\nBoth of the integrals are improper, but they're both finite.", "Solution_5": "[quote=\"gauss202\"]Both of the integrals are improper, but they're both finite.[/quote]\r\n\r\nI wouldn't say the first one is improper since x(ln x)->0 when x->0.", "Solution_6": "Good point. I didn't notice that. So only the second one is improper - but it still converges." } { "Tag": [], "Problem": "If $ \\frac {xy}{x \\plus{} y} \\equal{} a, \\frac {xz}{x \\plus{} z} \\equal{} b, \\frac {yz}{y \\plus{} z} \\equal{} c$, where $ a,b,c$ are other than zero, then $ x$ equals:\r\n\r\n$ \\textbf{(A)}\\ \\frac {abc}{ab \\plus{} ac \\plus{} bc} \\qquad\\textbf{(B)}\\ \\frac {2abc}{ab \\plus{} bc \\plus{} ac} \\qquad\\textbf{(C)}\\ \\frac {2abc}{ab \\plus{} ac \\minus{} bc}$\r\n$ \\textbf{(D)}\\ \\frac {2abc}{ab \\plus{} bc \\minus{} ac} \\qquad\\textbf{(E)}\\ \\frac {2abc}{ac \\plus{} bc \\minus{} ab}$", "Solution_1": "[quote=\"Brut3Forc3\"]If $ \\frac {xy}{x \\plus{} y} \\equal{} a, \\frac {xz}{x \\plus{} z} \\equal{} b, \\frac {yz}{y \\plus{} z} \\equal{} c$, where $ a,b,c$ are other than zero, then $ x$ equals:\n$ \\text{(A)}\\, \\frac {abc}{ab \\plus{} ac \\plus{} bc} \\qquad\\text{(B)}\\, \\frac {2abc}{ab \\plus{} bc \\plus{} ac} \\qquad\\text{(C)}\\, \\frac {2abc}{ab \\plus{} ac \\minus{} bc} \\\\\n\\text{(D)}\\, \\frac {2abc}{ab \\plus{} bc \\minus{} ac} \\qquad\\text{(E)}\\, \\frac {2abc}{ac \\plus{} bc \\minus{} ab}$[/quote]\r\n$ \\frac{1}{x} \\plus{} \\frac{1}{y} \\equal{} \\frac{1}{a}...$\r\n$ x \\equal{} \\frac{{2abc}}{{bc \\plus{} ac \\minus{} ab}}$", "Solution_2": "[hide]\nNotice that;\n$ \\frac {1}{a} \\equal{} \\frac {1}{x} \\plus{} \\frac {1}{y}$ and now its easy\n\n$ \\frac {1}{a} \\plus{} \\frac {1}{b} \\minus{} \\frac {1}{c} \\equal{} \\frac {2}{x}$\n\n$ x \\equal{} \\frac {2abc}{bc \\plus{} ac \\minus{} ab} \\longrightarrow \\boxed{E}$[/hide]\r\n\r\nEDIT: ninja'd by T.T.Hung!! :ninja:", "Solution_3": "X has to be symmetric to a and b.\r\nSo it's E!!!!! :D" } { "Tag": [ "integration", "calculus", "limit", "real analysis", "real analysis theorems" ], "Problem": "Given $ \\alpha\\in BV[a,b]$ (bounded variation), show that there is a unique $ \\beta\\in BV[a,b]$ with $ \\beta(a)\\equal{}0$ such that $ \\beta$ is right-continuous on $ (a,b)$ and $ \\int_{a}^{b}f\\,d\\alpha\\equal{}\\int_{a}^{b}f\\,d\\beta$ for every $ f\\in C[a,b]$.\r\n\r\nI assumed some $ \\gamma$ that also works, and used integration by parts to get that $ f(b)[\\beta(b)\\minus{}\\gamma(b)]\\equal{}\\int_{a}^{b}(\\beta\\minus{}\\gamma)\\,df$, but I'm stuck here. But I never used any assumptions about $ \\alpha$, so I'm not completely sure I'm heading in the right direction.", "Solution_1": "Hint: $ \\beta(t) \\equal{} \\lim_{y \\to a^\\plus{}}\\int_y^t d\\beta$", "Solution_2": "$ \\beta(b)\\equal{}\\lim_{y\\rightarrow a^{\\plus{}}}\\int_{y}^{b}d\\beta$ and $ \\gamma(b)\\equal{}\\lim_{y\\rightarrow a^{\\plus{}}}\\int_{y}^{b}d\\gamma$, so $ f(b)(\\beta(b)\\minus{}\\gamma(b))\\equal{}f(b)\\lim_{y\\rightarrow a^{\\plus{}}}\\int_{y}^{b}d(\\beta\\minus{}\\gamma)\\equal{}\\int_{a}^{b}(\\beta\\minus{}\\gamma)\\,df$. Since this holds for all $ f\\in C[a,b]$, then the integral must be 0, so $ \\beta\\equal{}\\gamma$. Is this correct?" } { "Tag": [ "topology", "real analysis", "real analysis unsolved" ], "Problem": "Give an example of a set in the plane that has an internal point that is not an interior point.", "Solution_1": "[hide=\"Example\"]$ 0\\in\\{re^{it}|0 < t\\le 2\\pi, 0\\le r\\le e^{ \\minus{} \\frac {1}{t}}\\}$. Note that every closed set in $ \\mathbb R^2$ with an internal point has non-trivial interior - as proven [url=http://www.mathlinks.ro/viewtopic.php?t=227831]here[/url][/hide]" } { "Tag": [ "geometry", "function" ], "Problem": "Suppose we have a perfect tap of transverse area $A$. a circular one. the water flux is constant.\r\n\r\nThe tap is $h$ meters to the ground. does not exist Air and the matter is not discrete.\r\n\r\nSo we can determine the Pressure $P$ of the water in the ground in function of the velocity of the water and others constants.\r\n\r\nHow do you calculate this?!", "Solution_1": "Are you looking for the pressure that the water exerts on the ground when it hits it? Then you need some more data, i.e. whether the colision of the water and the ground is ellastic.\r\n\r\nOn the other hand, it's easy to calculate the pressure [i]in[/i] the water just before it hits the ground, although I don't see any use of this fact...", "Solution_2": "[quote]Are you looking for the pressure that the water exerts on the ground when it hits it? [/quote]\r\n\r\nyes.\r\n\r\nAnd i dont think we need more data. we have the velocity of the particles (very smal, infinitesimal) of the water in the ground. we can easily know the area of collision in the ground level.\r\n\r\nthe only think i dont know if the weight of the water that hits the ground, what's portion of the water that is hiting.\r\n\r\nand this is very useful!!!\r\nSuppose we dont have a Weighing scale and dont want to buy one.\r\nwe can make one easily using this experiment. because we can easily know the flux of the water, the area and the gravity.\r\nwe dont need to work in Newtons, but we can have a result in newtons.\r\n\r\nthats my purpose", "Solution_3": "Ok then, let's assume that the collision is not elastic at all. \r\n\r\nFirst of all, let's find the pressure in the water when it hits the ground. Bernoulli's equation says that \\[p_{0}+\\rho g h+\\rho \\frac{v_{0}^{2}}{2}= p_{0}+\\rho \\frac{v^{2}}{2},\\] so $v = \\sqrt{v_{0}^{2}+2gh}$.The equation of continuity requires that $Av = A_{0}v_{0}$ ($A_{0}$ is the area of the tap) so $A = A_{0}v_{0}/\\sqrt{v_{0}^{2}+2gh}$. During an infinitesimal interval $\\Delta t$, the water exerts the force \\[F = \\Delta P/\\Delta t = \\rho v^{2}A = \\rho A_{0}v_{0}\\sqrt{v_{0}^{2}+2gh}.\\] I think this is what you wanted.\r\n\r\nHowever, the water that has fallen down doesn't just disappear; if the collision is inelastic, it's weight will afflict an additional force, and if the collision is somewhat elastic the above reasoning needs to be altered a bit.", "Solution_4": "I did not understand one thing\r\n\r\n$\\Delta P/\\Delta t = \\rho v^{2}A$\r\n\r\nwhere did you come this?!\r\n\r\nBecause $P$ depend on $v$ and not $v^{2}$.\r\nSo, where did come the addictional $v$?\r\n\r\nthe dimensional analysis of the equation is true. but. explain it\r\n\r\nthanks", "Solution_5": "In time $\\Delta t$ the impulse $\\Delta P = \\Delta m v$ is transferred to the ground. The mass $\\Delta m = \\rho \\Delta V = \\rho A v \\Delta t$..." } { "Tag": [ "inequalities", "inequalities proposed" ], "Problem": "If $a,b,c$ are positive numbers , prove that\r\n\r\n $\\frac{ab}{c(b+c)}+\\frac{bc}{a(c+a)}+\\frac{ca}{b(a+b)}\\geq \\frac{3}{2}$\r\n\r\n\r\n[u] Babis[/u]", "Solution_1": "[quote=\"stergiu\"]If $a,b,c$ are positive numbers , prove that\n\n $\\frac{ab}{c(b+c)}+\\frac{bc}{a(c+a)}+\\frac{ca}{b(a+b)}\\geq \\frac{3}{2}$\n\n\n[u] Babis[/u][/quote]\r\n\r\n\\[\\frac{ab}{c(b+c)}+\\frac{bc}{a(c+a)}+\\frac{ca}{b(a+b)}\\geq 3 \\sqrt[3]{ \\frac{abc}{(a+b)(b+c)(c+a)}}\\geq 3 \\sqrt[3]{ \\frac{abc}{2 \\sqrt{ab}\\cdot 2 \\sqrt{bc}\\cdot 2 \\sqrt{ca}}}= \\frac{3}{2}\\]\r\n\r\nQ.E.D", "Solution_2": "[quote=\"JANKRI\"]\n\\[3 \\sqrt[3]{ \\frac{abc}{(a+b)(b+c)(c+a)}}\\geq 3 \\sqrt[3]{ \\frac{abc}{2 \\sqrt{ab}\\cdot 2 \\sqrt{bc}\\cdot 2 \\sqrt{ca}}}= \\frac{3}{2}\\]\n[/quote]\r\n\r\nThis step is wrong.\r\nI'll post a solution.\r\n\r\nThe given inequality is equivalent to\r\n$\\sum \\frac{a^{2}b^{2}}{b+c}\\geq \\frac{3}{2}abc$ \r\nBut $\\sum \\frac{a^{2}b^{2}}{b+c}\\geq \\frac{(ab+bc+ca)^{2}}{2(a+b+c)}\\geq\\frac{3}{2}abc$.\r\nIf we denote $x=ab$ ,$y=bc$ and $z=ca$ the last inequality is equivalent to the well-known :$(x+y+z)^{2}\\geq 3(xy+yz+xz)$", "Solution_3": "[quote=\"stergiu\"]If $a,b,c$ are positive numbers , prove that\n\n $\\frac{ab}{c(b+c)}+\\frac{bc}{a(c+a)}+\\frac{ca}{b(a+b)}\\geq \\frac{3}{2}$\n\n\n[u] Babis[/u][/quote]\r\nSvejk, I think, it easier:\r\n$\\sum_{cyc}\\frac{ab}{c(b+c)}=\\sum_{cyc}\\frac{a^{2}b^{2}}{b^{2}ac+c^{2}ab}\\geq\\frac{(ab+ac+bc)^{2}}{2abc(a+b+c)}\\geq\\frac{3}{2}.$ :wink:", "Solution_4": "[quote=\"Svejk\"][quote=\"JANKRI\"]\n\\[3 \\sqrt[3]{ \\frac{abc}{(a+b)(b+c)(c+a)}}\\geq 3 \\sqrt[3]{ \\frac{abc}{2 \\sqrt{ab}\\cdot 2 \\sqrt{bc}\\cdot 2 \\sqrt{ca}}}= \\frac{3}{2}\\]\n[/quote]\n\nThis step is wrong.\nI'll post a solution.\n\nThe given inequality is equivalent to\n$\\sum \\frac{a^{2}b^{2}}{b+c}\\geq \\frac{3}{2}abc$ \nBut $\\sum \\frac{a^{2}b^{2}}{b+c}\\geq \\frac{(ab+bc+ca)^{2}}{2(a+b+c)}\\geq\\frac{3}{2}abc$.\nIf we denote $x=ab$ ,$y=bc$ and $z=ca$ the last inequality is equivalent to the well-known :$(x+y+z)^{2}\\geq 3(xy+yz+xz)$[/quote]\r\n\r\nI see, sorry :blush:", "Solution_5": "Let x = a/b, y = b/c, z = c/a.", "Solution_6": "[quote=\"X-men\"]Let x = a/b, y = b/c, z = c/a.[/quote]\r\n What do we gain , by homogenizing the inequality in this way? I think we must again use BCS(Engel form).\r\n\r\n[b]Alternative[/b]\r\n \r\n Darij Grinberg has made in the past a solution using Rearrangement.\r\n\r\n Babis" } { "Tag": [ "calculus", "integration", "trigonometry", "logarithms", "calculus computations" ], "Problem": "[color=red] Calculate the integral :\n $ I\\equal{}\\int\\frac{dx}{x^8\\plus{}1}$\n[/color]", "Solution_1": "First, this should be in the calculus forum. Second, it's not necessarily hard-just really, really annoying. Get the denominator in a more friendly form and then just use partial fractions. That is, factor it so that you can use partial fractions. Then it's just a lot of arctangents and logarithms.", "Solution_2": "Although $ \\int^\\infty_{\\minus{}\\infty} \\frac{dx}{x^8\\plus{}1}$ would be fairly easy to do..", "Solution_3": "$ \\boxed{\\int_{\\minus{}\\infty}^{\\infty} \\frac{1}{x^{2n}\\plus{}1}\\ dx\\equal{}\\frac{\\pi}{n\\sin \\frac{\\pi}{2n}}\\ (n\\equal{}1,\\ 2,\\ \\cdots)}$", "Solution_4": "I would just solve a problem like that using residues. But getting the indefinite integral...", "Solution_5": "Exactly my point.. :)", "Solution_6": "hello, i have got \r\n$ \\int \\frac{1}{x^8\\plus{}1}dx\\equal{}\\\\\r\n\\frac{1}{4} \\sin \\left(\\frac{\\pi }{8}\\right) \\tan ^{\\minus{}1}\\left(\\left(x\\minus{}\\cos \\left(\\frac{\\pi }{8}\\right)\\right) \\csc \\left(\\frac{\\pi }{8}\\right)\\right)\\minus{}\\frac{1}{8}\r\n \\cos \\left(\\frac{\\pi }{8}\\right) \\log \\left(x^2\\minus{}2 \\cos \\left(\\frac{\\pi }{8}\\right) x\\plus{}1\\right)\\plus{}\\frac{1}{8} \\cos \\left(\\frac{\\pi }{8}\\right) \\log \\left(x^2\\plus{}2 \\cos\r\n \\left(\\frac{\\pi }{8}\\right) x\\plus{}1\\right)\\plus{}\\frac{1}{4} \\tan ^{\\minus{}1}\\left(\\left(x\\plus{}\\cos \\left(\\frac{\\pi }{8}\\right)\\right) \\csc \\left(\\frac{\\pi }{8}\\right)\\right) \\sin\r\n \\left(\\frac{\\pi }{8}\\right)\\minus{}\\frac{1}{8} \\log \\left(x^2\\minus{}2 \\sin \\left(\\frac{\\pi }{8}\\right) x\\plus{}1\\right) \\sin \\left(\\frac{\\pi }{8}\\right)\\plus{}\\frac{1}{8} \\log\r\n \\left(x^2\\plus{}2 \\sin \\left(\\frac{\\pi }{8}\\right) x\\plus{}1\\right) \\sin \\left(\\frac{\\pi }{8}\\right)\\plus{}\\frac{1}{4} \\tan ^{\\minus{}1}\\left(\\sec \\left(\\frac{\\pi }{8}\\right)\r\n \\left(x\\minus{}\\sin \\left(\\frac{\\pi }{8}\\right)\\right)\\right) \\cos \\left(\\frac{\\pi }{8}\\right)\\plus{}\\frac{1}{4} \\tan ^{\\minus{}1}\\left(\\sec \\left(\\frac{\\pi }{8}\\right) \\left(x\\plus{}\\sin\r\n \\left(\\frac{\\pi }{8}\\right)\\right)\\right) \\cos \\left(\\frac{\\pi }{8}\\right)$.\r\nSonnhard.", "Solution_7": "That's just from Mathematica. It would be more important to get the partial fraction decomposition.", "Solution_8": "hello, note that $ \\frac{1}{x^8\\plus{}1}\\equal{}\\frac{1}{4}\\frac{x^2\\sqrt{2}\\minus{}2}{\\minus{}x^4\\plus{}x^2\\sqrt{2}\\minus{}1}\\plus{}\\frac{1}{4}\\frac{2\\plus{}x^2\\sqrt{2}}{x^4\\plus{}x^2\\sqrt{2}\\plus{}1}$.\r\nSonnhard.", "Solution_9": "We have:\r\n$ \\frac{1}{x^8 \\plus{} 1} \\equal{} \\frac{1}{8} \\sum_{k \\equal{} 1}^{8} \\frac{a_{k}^{\\minus{}7}}{x \\minus{} a_{k}}$, where $ a_{k} \\equal{} e^{\\frac{i(2k \\minus{} 1)\\pi}{8}}$.", "Solution_10": "[color=red] Thank you very much, Dr Sonnhard Graubner,but could you please tell me why do you get the nice notation :\n $ \\frac{1}{x^8\\plus{}1}\\equal{}\\frac{1}{4}\\frac{x^2\\sqrt{2}\\minus{}2}{\\minus{}x^4\\plus{}x^2\\sqrt{2}\\minus{}1}\\plus{}\\frac{1}{4}\\frac{2\\plus{}x^2\\sqrt{2}}{x^4\\plus{}x^2\\sqrt{2}\\plus{}1}$\nIt is which I want, thank you again.\nRegard,\nArsenaler.\n[/color]", "Solution_11": "hello, we have $ x^8\\plus{}1\\equal{}(x^4\\minus{}\\sqrt{2}x^2\\plus{}1)(x^4\\plus{}\\sqrt{2}x^2\\plus{}1)$.\r\nSonnhard." } { "Tag": [ "geometry", "AMC", "USA(J)MO", "USAMO", "geometric transformation", "homothety", "vector" ], "Problem": "Do some problems on the USAMO require the knowledge of \"\"nontraditional\" geometry - e.g. inversion, projections, homothecy, transformational geometry, constructions, vectors, cross/dot products, etc.? And do any USAMO problem require knowledge of graph theory? Thanks in advance.", "Solution_1": "These types of questions are very hard to answer.\r\n\r\n[b]Theoretically[/b], you don't need anything beyond what they teach in high school. Nothing will [i]require[/i] the use of anything fancy. Everything can be solved with basic theorems and ingenuity.\r\n\r\nBut everybody knows that the more you know, the better are your chances at solving those problems.\r\n\r\nOlympiads tend not to be tests of knowledge (i.e. no problem will ask you to specificly use inversion, and no USAMO problem will be phrased in graph theory terminology). Olympiads are tests of techniques. Most of the things you listed there are well-known techniques. It will not help you much by simply \"knowing\" them. You will have to see them in action and use them yourself.", "Solution_2": "I've looked at some past USAMO problems, and it seems to me that tools like these are useful but not necessary for solving the problems. I've seen complicated vector-based solutions to geometry problems, and then simpler solutions to the same problems based on pure geometry.", "Solution_3": "ouch, graph theory and geometry. basically everyone's worst subject" } { "Tag": [ "function", "quadratics", "algebra", "polynomial", "algebra theorems" ], "Problem": "Given that \r\n\r\n$ ax^4 \\plus{} bx^3 \\plus{} cx^2 \\plus{} dx \\plus{} e \\equal{} 0$\r\n\r\n1) Find the discriminant of quartic function.\r\n\r\n2) As you know from quadratic funtion, that if discriminant of quadratic function is negative, then there are no real roots and function has strictly positive or negative value. Can you say the same to quartic function? \r\n\r\nThank you.", "Solution_1": "See [url=http://en.wikipedia.org/wiki/Discriminant#Discriminant_of_a_polynomial]here[/url] for the generic formula, assuming you know some linear algebra. Mostly just finding determinants.", "Solution_2": "Sorry, i don't know linear algebra. Therefore, it is hard to me to understand the generalization in Wikipedia. \r\nI just want you to write me discriminant of quartic function with coefficients of terms ($ a, b, c, d, e$). \r\nFor example, the discriminant of quadratic is $ b^2 \\minus{} 4ac$. Give me the one for quartic.\r\n\r\nPlease, also answer question 2)\r\n\r\nThanks.", "Solution_3": "Equation (10) [url=http://mathworld.wolfram.com/PolynomialDiscriminant.html]here[/url]. There is no point in actually giving the explicit formula since the quartic discriminant is too complicated for straightforward computation.\r\n\r\nThe answer to 2) is \"no.\" The discriminant of a monic quartic with roots $ r_1, r_2, r_3, r_4$ is $ \\prod_{i < j} (r_i \\minus{} r_j)^2$. All a negative discriminant tells you is that [i]some[/i] of the roots are non-real. It's possible for two of the roots to be complex and two of the roots to be real." } { "Tag": [ "LaTeX", "quadratics" ], "Problem": "Do there exist integers a and b for which $ a^2\\plus{}b^2$ and $ a^2\\minus{}b^2$ are square numbers?", "Solution_1": "Well, let's assume for the moment it is possible. Therefore, we have that\r\n$ a^{2}\\plus{}b^{2}\\equal{}s^{2}$ for some $ s$ and\r\n$ a^{2}\\minus{}b^{2}\\equal{}k^{2}$ for some $ k$. Adding these, we get that\r\n2$ a^{2}k^{2}\\plus{}s^{2}$ \r\nor $ a^{2}\\equal{}frac{k^{2}\\plus{}s^{2}}{2}$.\r\nPlugging this into our first equation,we get that\r\n$ b^{2}\\equal{}frac{s^{2}\\minus{}k^{2}}{2}$. \r\nThis is just the same problem.\r\n\r\nSo, this problem looks kind of circular, but I'm willing to guess\r\nthat there do exist such numbers.", "Solution_2": "@Z-coli\r\nI'm sorry, but don't understand what your post is saying. Your third line is $ 2a^2 \\equal{} k^2 \\plus{} s^2$, ok. But what does $ frack$ and $ fracs$ mean? Also, $ k^2$ and $ 4k^2$ is a square number, but not $ 2k^2$. Could you clarify?", "Solution_3": "It was a $ \\text{\\LaTeX}$ typo. Here is the corrected post.\r\n\r\n[quote=\"Z-coli\"]Well, let's assume for the moment it is possible. Therefore, we have that\n$ a^{2} + b^{2} = s^{2}$ for some $ s$ and\n$ a^{2} - b^{2} = k^{2}$ for some $ k$. Adding these, we get that\n$ 2a^{2} = k^{2} + s^{2}$ \nor $ a^{2} = \\frac {k^{2} + s^{2}}{2}$.\nPlugging this into our first equation,we get that\n$ b^{2} = \\frac {s^{2} - k^{2}}{2}$. \nThis is just the same problem.\n\nSo, this problem looks kind of circular, but I'm willing to guess\nthat there do exist such numbers.[/quote]\r\n\r\nYes, there do. Infinitely many, in fact. (Hint: let $ b$ be zero...)", "Solution_4": "Sorry, [i]positive[/i] integers. \r\n\r\nOtherwise, Yongiyi781's solution works, and this is a non-problem", "Solution_5": "Edit: Scratch that. Fermat's infinite descent actually doesn't work. (There is no bound...)\r\n\r\nLook [url=http://en.wikipedia.org/wiki/Pythagorean_triple]here[/url], and especially look at the last line in the section \"Elementary properties of primitive Pythagorean triples\":\r\n\r\n[quote]There are no Pythagorean triplets in which the hypotenuse and one leg are the legs of another Pythagorean triple.[/quote]\r\n\r\nThis essentially answers your question.\r\n\r\nSuppose that the problem was false. Then let $ a^2\\plus{}b^2\\equal{}c^2$ and $ a^2\\minus{}b^2\\equal{}d^2$. WLOG let $ \\gcd(a,b)\\equal{}1$ because otherwise, we have a smaller solution by dividing both sides of each equation by $ [\\gcd(a,b)]^2$. Now, it is not possible for both $ a$ and $ b$ to be odd because mod 4, both $ a^2$ and $ b^2$ are 1 mod 4, so $ c^2$ is 2 mod 4, but 2 is not a quadratic residue mod 4.\r\n\r\nSo one of $ a$ and $ b$ must be even.\r\n\r\nCase 1: $ a$ is even and $ b$ is odd. Then $ c^2\\minus{}d^2\\equal{}2b^2$. But since $ b^2$ is odd, $ 2b^2$ has only one factor of 2, and thus can't be the difference of two squares. (Hint: factor $ c^2\\minus{}d^2\\equal{}(c\\plus{}d)(c\\minus{}d)$. Each factor must have the same parity.)\r\n\r\nCase 2: $ a$ is odd and $ b$ is even. This is the crux of the proof which I will leave to you." } { "Tag": [ "inequalities", "linear algebra", "linear algebra unsolved" ], "Problem": "Let $A, \\, B$ be real symmetric positive definite $n \\times n$ matrices with the same determinant. Show that\r\n\\[trace \\left(A B^{-1}\\right) \\ge n\\]\r\nwith equality iff $A = B$.", "Solution_1": "(In exchange for asking questions, I should try answering one...)\r\n\r\nLet ${\\lambda_{i}}$ be the eigenvalues of $AB^{-1}$. Then the problem is equivalent to showing $\\frac{\\sum_{i=1}^{n}\\lambda_{i}}{n}\\geq 1$. But this follows fairly quickly from the Arithmetic-Geometric Mean Inequality:\r\n\r\n$\\frac{\\sum_{i=1}^{n}\\lambda_{i}}{n}\\geq (\\prod_{i=1}^{n}\\lambda_{i})^{1/n}= (det(AB^{-1}))^{1/n}=(\\frac{det(A)}{det(B)})^{1/n}=1$.", "Solution_2": "[quote=\"edoo\"](In exchange for asking questions, I should try answering one...)\n\nLet ${\\lambda_{i}}$ be the eigenvalues of $AB^{-1}$. [/quote]How do you know that these are real numbers? :wink:", "Solution_3": "[quote]How do you know that these are [color=green][positive][/color] real numbers?[/quote]\r\nSee the solution I posted to problem #3 in [url=http://www.artofproblemsolving.com/Forum/viewtopic.php?t=147726]this topic.[/url]" } { "Tag": [ "inequalities", "modular arithmetic", "number theory proposed", "number theory" ], "Problem": "For each positive integer $ n$,denote by $ S(n)$ the sum of all digits in decimal representation of $ n$.\r\nFind all positive integers $ n$,such that $ n\\equal{}2S(n)^3\\plus{}8$.", "Solution_1": "if $ d$ is the number of digits in $ n$, then $ S(n)\\leq 9d$, whereas $ n\\geq 10^{d \\minus{} 1}$. so we get the inequality\r\n\\[ 10^{d \\minus{} 1}\\leq 1458d^3 \\plus{} 8\r\n\\]\r\nwhich is false for $ d\\geq 8$. in particular, this means that $ S(n)\\leq 63$.\r\n\r\nnow in light of the fact that $ S(n)\\equiv n\\pmod{9}$, we must have $ n\\equiv 2n^3 \\plus{} 8\\pmod{9}$, whose only solution is $ n\\equiv 1\\pmod{9}$. so the only possible values of $ S(n)$ are 1, 10, 19, 28, 37, 46, 55. a quick check now shows that the only solutions are $ n \\equal{} 10, 2008, 13726$.", "Solution_2": "[quote=\"pleurestique\"]if $ d$ is the number of digits in $ n$, then $ S(n)\\leq 9d$, whereas $ n\\geq 10^{d \\minus{} 1}$. so we get the inequality\n\\[ 10^{d \\minus{} 1}\\leq 1458d^3 \\plus{} 8\n\\]\nwhich is false for $ d\\geq 8$. in particular, this means that $ S(n)\\leq 63$.\n\nnow in light of the fact that $ S(n)\\equiv n\\pmod{9}$, we must have $ n\\equiv 2n^3 \\plus{} 8\\pmod{9}$, whose only solution is $ n\\equiv 1\\pmod{9}$. so the only possible values of $ S(n)$ are 1, 10, 19, 28, 37, 46, 55. a quick check now shows that the only solutions are $ n \\equal{} 10, 2008, 13726$.[/quote]\r\nIn fact,you only need to check $ n\\equal{}1,10,19,28,37,46$,the value $ 55>9\\cdot 6$.\r\nMy solution and official solutions is the same. :wink: \r\nThank you.", "Solution_3": ".............", "Solution_4": "[quote=Superguy]S(n)<=54 not 63 bto[/quote]\n\nthat bump though...\n\nbtw how far did you have to scroll to get to this thread :rotfl: ", "Solution_5": "please explain me why $S(n)$ = $1$ ( $mod 9$)?", "Solution_6": "inequality \nk<7" } { "Tag": [ "number theory", "greatest common divisor", "least common multiple", "AMC", "USA(J)MO", "USAMO", "summer program" ], "Problem": "What was your first Olympiad experience?\r\n\r\nAt first, I saw some Diophantine Equation and it says find all solutions or something. I just went and plugged heaps of values in and when I got to like $ 30$ I concluded no more solutions exist. I was wrong, there was a few bigger values.\r\n\r\nSo, I thought 'How the [censored] am I suppose to know that!' so I looked at the solution which made use of everything I know (e.g. Prime Factorization, Divisibility, gcd, lcm) but never thought about integrating these 'fudamental' concepts.\r\n\r\nThen, the rest is history. Here I am! (I don't even know how I found the website, I think I was googling random terms).\r\n\r\nTell me about yours.", "Solution_1": "First USAMO problem was at the end of my Intro to NT class. Rather easy, actually.\r\nI think it was something like:\r\n\r\nProve or dissprove that it there is a set of 14 consecutive integers where each number is a divisor at least one prime number 2, 3, 5, 7, 11. If you prove it, give an example.\r\n\r\nMight have remembered it wrong. But is it really hard enough to be an Olympiad question>", "Solution_2": "My first experience was... the 1997 USAMO. I don't think I ever saw an Olympiad problem before the actual contest.\r\n\r\nI did pretty well, too- I think my score was in the 7-13 points range. That wasn't enough for MOP at the time, but it would be now with \"Red MOP\".\r\n\r\nI think I misclicked the poll- it's not showing the answer I intended.", "Solution_3": "My first Olympiad experience was the 2007 USAMO. I realized that the problems weren't so terrible, although I messed up on some of them. I ended up getting 10 points, and I felt rather pleased.", "Solution_4": "It's hard to pinpoint my first olympiad experience. I participated in olympiad practices throughout freshman year, including some actually olympiad-style tests. I then proceeded to qualify for USAMO 2007, scoring 18 - a very good score. At that point, I realized that it was not actually very difficult to score 14+ on olympiads since you only need problems 1 and 4 to do so. After MOP (red, but I qualified for blue. Oh well), I went to MathCamp and there I participated in a practice olympiad consisting of the 6 problems from IMO 2007. I scored a 14 on that, the equivalent of the low end of a bronze medal (50% mark). All of these were pretty good experiences, but it's hard to say what would be my actual first olympiad (not the practice IMO of course).", "Solution_5": "By the way, it doesn't have to be going to an actual Olympiad or campe etc. Just the first time you encountered Olympiad-like problems.", "Solution_6": "The first time I saw Olympiad problems was from a booklet. I was like \"...what...hmm this is pretty interesting, but really hard...\"\r\nThey were old USAMO problems which were pretty bad.\r\n\r\nThe first time I officially took an Olympiad was this year's USAMO.\r\n\r\nBut I've taken Olympiad exams unofficially quite a bit...for example, last year's USAMO (I got like 3 problems) or this year's IMO (5 problems, don't ask how). I don't think the pressure of an actual Olympiad can be simulated through practice, though. No matter how many problems you do, you'll always do significantly worse on the real thing. That's how it is for me, at least...", "Solution_7": "For me, in an actual Olympiad examination (not the IMO) I tend to do better than I expect (also better than when I practice). I know the answers are not there and there are no hints so my mind tend to be a bit open on interpretations of problems. Especially geometry problems, I find that when I open a hard geometry book to work through I manage to solve less than desired but during an exam I zoom through most like crazy.", "Solution_8": "[quote=\"BanishedTraitor\"]For me, in an actual Olympiad examination (not the IMO) I tend to do better than I expect (also better than when I practice). I know the answers are not there and there are no hints so my mind tend to be a bit open on interpretations of problems. Especially geometry problems, I find that when I open a hard geometry book to work through I manage to solve less than desired but during an exam I zoom through most like crazy.[/quote]\r\n\r\nWell okay, I should elaborate:\r\n\r\nWhen I just casually look at problems and try to solve them, I don't try as hard or solve as many as on a real Olympiad. Then again, on a real Olympiad, I don't solve as many as if I simulate taking the exam at home.", "Solution_9": "[quote=\"Nerd_of_the_Ages\"]First USAMO problem was at the end of my Intro to NT class. Rather easy, actually.\nI think it was something like:\n\nProve or dissprove that it there is a set of 14 consecutive integers where each number is a divisor at least one prime number 2, 3, 5, 7, 11. If you prove it, give an example.\n\nMight have remembered it wrong. But is it really hard enough to be an Olympiad question>[/quote]\r\n\r\n...are you sure that was \"rather easy\"?\r\n\r\nMy first encounter with Olympiad-level problems was 2001 IMO #3, a pretty nice inequality which I solved using Jensen's. As of yet, the only Olympiads I've been able to solve are inequalities and a geometry problem.", "Solution_10": "[quote=\"Temperal\"][quote=\"Nerd_of_the_Ages\"]First USAMO problem was at the end of my Intro to NT class. Rather easy, actually.\nI think it was something like:\n\nProve or dissprove that it there is a set of 14 consecutive integers where each number is a divisor at least one prime number 2, 3, 5, 7, 11. If you prove it, give an example.\n\nMight have remembered it wrong. But is it really hard enough to be an Olympiad question>[/quote]\n\n...are you sure that was \"rather easy\"?[/quote]\r\n\r\nWell, I didn't think is was THAT hard... and it couldnt have been either, because we covered it in like 20 minutes in the Intro to NT class.\r\n\r\nEDIT: Actually, it wasnt a rigorous proof, but yeah....", "Solution_11": "My experience.... does it have to be official? [hide=\"If so\"]I got a 3 on last year's AIME. I sucked then.[/hide]", "Solution_12": "The point was that it doesn't have to be official- just the first Olympiad problems you tried to solve. It just so happens that those were part of an official contest for many of us." } { "Tag": [ "Asymptote", "geometry" ], "Problem": "Since I suck at asymptote, try your best to imagine the question:\r\n\r\nTwo circles, circle $ O$ and circle $ C$, with radius 1 are created. The center of circle $ O$ lies on circle $ C$, and the center of circle $ C$ lies on circle $ O$. What is the area of the intersection of $ O$ and $ C$?\r\n\r\nIm thinking this has a bit of PIE in its solution, but I really dont know.\r\n\r\nAlso, how would you do this with 3 circles of radius 1 and the same restrictions (each circles center is on the other circles circumference)?", "Solution_1": "The area is 2pi/3", "Solution_2": "How so? And how do you do it for 3 circles?", "Solution_3": "[asy]size(8cm,0);\nimport math;\nimport graph;\nreal r,s;\npair a,b, common;\npath circ1, circ2;\nr=1; s=1;\na=(0,0);\nb=(1,0);\ncirc1=circle(a,r);\ncirc2=circle(b,s);\ndraw(circ1,linewidth(1bp));\ndraw(circ2,linewidth(1bp));\npair [] x=intersectionpoints(circ1, circ2);\ndot(x[0],3bp+blue);\ndot(x[1],3bp+blue);\nlabel(\"C\",(0,0),SW);\nlabel(\"O\",(1,0),SE);\ndot((0,0),red);\ndot((1,0),red);[/asy]\r\n\r\nFor two circles correct?\r\n\r\n[hide=\"Solution\"]\n[asy]size(8cm,0);\nimport math;\nimport graph;\nreal r,s;\npair a,b, common;\npath circ1, circ2;\nr=1; s=1;\na=(0,0);\nb=(1,0);\ncirc1=circle(a,r);\ncirc2=circle(b,s);\ndraw(circ1,linewidth(1bp));\ndraw(circ2,linewidth(1bp));\npair [] x=intersectionpoints(circ1, circ2);\ndot(x[0],3bp+blue);\ndot(x[1],3bp+blue);\nlabel(\"C\",(0,0),SW);\nlabel(\"O\",(1,0),SE);\ndot((0,0),red);\ndot((1,0),red);\ndraw((0,0)--x[0]--(1,0),linewidth(1));\ndraw((0,0)--x[1]--(1,0),linewidth(1));\ndraw((0,0)--(1,0),blue+linewidth(1));[/asy]\nIt's kinda easy to see that the area of the football-shaped region is twice the area above the blue line. You can find that area by taking the sum of two $ 60^{\\circ}$ sections of the circle, then subtracting out the area of the equilateral triangle. This would be \\[ 2\\left(\\frac{1^2\\pi}{6}\\right)\\minus{}\\frac{1^2\\sqrt{3}}{4}\\equal{}\\frac{\\pi}{3}\\minus{}\\frac{\\sqrt3}{4}.\\] The area of the whole football is two times this, or \\[ \\boxed{\\frac{2\\pi}{3}\\minus{}\\frac{\\sqrt3}{2}}.\\][/hide]\r\n\r\nYou can do a similar thing for 3 circles.", "Solution_4": "yes, for three circles it's just one equilateral triangle and 3 of those arc minus triangle things. bad description, i know :)", "Solution_5": "[quote=\"algebra1337\"]yes, for three circles it's just one equilateral triangle and 3 of those arc minus triangle things. bad description, i know :)[/quote]\r\n\r\nWhat youre saying is take the area of three 60 degree circles but then subtract 2 triangles?\r\n\r\nThanks guys, I get it now :lol:" } { "Tag": [ "ratio", "real analysis", "real analysis unsolved" ], "Problem": "Prove that for a sequence of positive terms, (a_k), if there is N with (a_n+a)/(a_n)>=1 for all n >=N, then the series \r\n\r\nsummation(a_n) from n=1 to n=infinity diverges. Show that this result implis one direction of the Ration test.", "Solution_1": "Hint: prove that $(a_n)$ does not converge to $0$. In particular, this implies that the series diverges." } { "Tag": [ "inequalities", "inequalities unsolved" ], "Problem": "[i]Let be a, b and c real > 0. Show that[/i]\r\n\r\n$(a^{2}-bc)\\sqrt{b+c}+(b^{2}-ac)\\sqrt{a+c}+(c^{2}-ab)\\sqrt{a+b}\\geq0$\r\n\r\nBecause a square root is strictly positive, we need to prove \r\n\r\n$\\{\\begin{array}{rcl}a^{2}-bc\\geq0\\\\ b^{2}-ac\\geq0\\\\ c^{2}-ab\\geq0 \\end{array} \\Leftrightarrow a^{2}+b^{2}+c^{2}-ab-ac-bc \\geq0$\r\n\r\nBut,\r\n\r\n$(a-b)^{2}+(b-c)^{2}+(c-a)^{2}\\geq0 \\Leftrightarrow a^{2}+b^{2}+c^{2}-ab-ac-bc \\geq0$\r\n\r\nDone.\r\n\r\n[u]Is it correct ? :blush: [/u]\r\n\r\nEDIT : I think, because the ineguality is cyclic, we need to set a >= b >= c, no ?", "Solution_1": "Let b+c=x^2,c+a=y^2,a+b=z^2(x,y,z>0),then\r\n\r\n a = 1/2*z^2+1/2*y^2-1/2*x^2,\r\n b = 1/2*z^2-1/2*y^2+1/2*x^2,\r\n c = -1/2*z^2+1/2*y^2+1/2*x^2.\r\ntherefore\r\n\r\n (a^2-b*c)*sqrt(b+c)+(b^2-c*a)*sqrt(c+a)+(c^2-a*b)*sqrt(a+b)\r\n\r\n= 1/2*x*z^4-1/2*z^2*x^3+1/2*x*y^4-1/2*y^2*x^3+1/2*y*z^4-1/2*z^2*y^3\r\n\r\n-1/2*y^3*x^2+1/2*y*x^4-1/2*z^3*y^2-1/2*z^3*x^2+1/2*z*y^4+1/2*z*x^4\r\n\r\n= 1/2*[y^2*(z+x)*(y-x)*(y-z)+x^2*(y+z)*(x-y)*(x-z)+z^2*(x+y)*(z-y)*(z-x)]\r\n\r\n+[ y*x*z*(y-x)*(y-z)+x*y*z*(x-y)*(x-z)+z*y*x*(z-y)*(z-x)]>=0.", "Solution_2": "[quote=\"zweig\"]\n\n$\\{\\begin{array}{rcl}a^{2}-bc\\geq0\\\\ b^{2}-ac\\geq0\\\\ c^{2}-ab\\geq0 \\end{array} \\Leftrightarrow a^{2}+b^{2}+c^{2}-ab-ac-bc \\geq0$\n\n[/quote]\r\nWrong here! :D", "Solution_3": "fjwxcsl > Thanks for your solution, I will s\u00ea it more precisly later ^^\r\n\r\nN.T.TUAN > Why is it wrong ? :)", "Solution_4": "[quote=\"zweig\"]fjwxcsl > Thanks for your solution, I will s\u00ea it more precisly later ^^\n\nN.T.TUAN > Why is it wrong ? :)[/quote]\r\n\r\n$\\Leftrightarrow$ means if and only if. The 'only if' part is true in this case, but not the 'if':\r\nAssume that the inequalities on the left are true. Then the inequality on the right is clearly true as well. So the left inequalities are true [b]only if[/b] the right hand side inequality is true.\r\n\r\nHowever, [b]if[/b] the inequality is on the right is true, then perhaps:\r\n$a^{2}-bc=3$\r\n$b^{2}-ac=7$\r\n$c^{2}-ab=-5$\r\n\r\nSo in this case, the right inequality is correct, and the left inequalities are not. So we cannot say that the left inequalities are true [b]if[/b] the inequality on the right is.\r\n\r\nWhat you have said about the inequality is similar to saying:\r\n\"You are a dog if and only if you are an animal.\"\r\nWhich we can break down into:\r\n\"You are a dog only if you are an animal\" (true)\r\n\"You are a dog if you are an animal\" (false)\r\n\r\nI may have over-stressed the point, but this problem has lost me too many points on Olympiad exams down the years :(" } { "Tag": [ "\\/closed" ], "Problem": "Perhaps a user who has given out his password to other people should have his screen name blocked completely by the administrators (the real person can always sign up for a new screen name, and is very welcome to solve problems or otherwise to contribute to any forum I moderate) so that we don't have to deal with this problem of spam posts anymore. \r\n\r\nJust a thought, as I don't have moderator powers on this forum, and I don't administer the site.", "Solution_1": "This was split from an original thread on spam with the title that I've put in the subject line.\r\n\r\nI've moved the split to this forum because it's more relevant here, and I believe it's an issue that should be discussed. Thanks for tokenadult for bringing it up.\r\n\r\nThe original thread can be found in Fun and Games.", "Solution_2": "Who exactly did that? \r\nBy the way, any such instances are to be reported A.S.A.P. to us (the admins).", "Solution_3": "The original \"having lots of posts is not . . .\" thread on Games and Fun Factory asserts, plausibly, that the holder of the I.D. that started that thread (gameworld7) has shared his/her I.D. and password with other persons, and thus is being impersonated online by spammers. Once a screen name is no longer secure and indentifiable with one person, it's probably a good idea to block that screen name entirely, right?", "Solution_4": "I split that topic from another discussion to explain to him why he some of his posts were deleted. Did he say that someone has taken over his account?", "Solution_5": "[quote=\"Valentin Vornicu\"]Did he say that someone has taken over his account?[/quote]\r\n\r\nNot explicitly, but I suppose someone else posting from his account should be enough evidence.", "Solution_6": "Could it have been himself posting that as sort of an excuse? Not too likely, but there's a possibility.\r\n\r\nBut that also opens the story where somebody who has his account can say \"yes, nobody took over my account\"", "Solution_7": "[quote=\"yif man12\"]Could it have been himself posting that as sort of an excuse? Not too likely, but there's a possibility.\n\nBut that also opens the story where somebody who has his account can say \"yes, nobody took over my account\"[/quote]\r\n\r\nYou know, I say just close the account...if he wants access to this site, nobody's preventing him from making another one.\r\n\r\nBut then again, that's probably the reason i'm not an admin ;-)", "Solution_8": "Good point. I was thinking the same thing.\r\n\r\nBut, it's up to the admin. :)", "Solution_9": "[quote=\"yif man12\"]Could it have been himself posting that as sort of an excuse? Not too likely, but there's a possibility.\n\nBut that also opens the story where somebody who has his account can say \"yes, nobody took over my account\"[/quote]\r\nno.\r\ni was one of em\r\nand he gave it away to try to prevent himself from going back to AoPS to waste more time on the other topics forum\r\n\r\nBut it was, of course, doomed to failiure" } { "Tag": [ "geometry", "geometric transformation", "rotation", "reflection", "geometry proposed" ], "Problem": "two squares are given (O,M,N,P) and (O,S,R,T)[O is the only point they have in common]. If A is the middle point of PS prove that r(AO) is perpendicular to r(MT).\r\nthe letters are clocwise\r\n\r\ngood luck :)", "Solution_1": "perform a 90 degree clockwise rotation around point $O$. then $M$ goes to $P$ and $T$ goes to, say, $T'$. it suffices to show that $PT' \\parallel AO$. but this is easy: $O$ is the midpoint of $ST'$, and since $A$ is the midpoint of $SP$, $AO$ is definitely parallel to $PT'$", "Solution_2": "Let $S'$ be the reflection of $T$ in $S$. $OMS'$ is the image of $OPS$ through a rotation centered at $O$ of angle $-\\frac\\pi 2$, so, if $A'$ is the midpoint of $MS'$, then $OA'\\perp OA\\ (*)$. \r\n\r\nOn the other hand, $A'$ is the midpoint of $MS'$, while $O$ is the midpoint of $TS'$, so $OA'\\|MT\\ (**)$. $(*)$ and $(**)$ prove the desired result.\r\n\r\nEdit: Looks like someone else was here first :)." } { "Tag": [ "inequalities unsolved", "inequalities" ], "Problem": "Let $\\ a,b,c$ be positive real numbers \r\nProve that $\\frac{a^{3}}{b}$+$\\frac{a^{3}}{b}$+$\\frac{a^{3}}{b}$ $\\geq$ $a\\sqrt{ac}$+$b\\sqrt{ba}$+$c\\sqrt{cb}$\r\n Good luck :D", "Solution_1": "[quote=\"2006\"]Let $\\ a,b,c$ be positive real numbers \nProve that $\\frac{a^{3}}{b}$+$\\frac{a^{3}}{b}$+$\\frac{a^{3}}{b}$ $\\geq$ $a\\sqrt{ac}$+$b\\sqrt{ba}$+$c\\sqrt{cb}$\n Good luck :D[/quote]\r\n\r\nI think it should be $\\frac{a^{3}}{b}$+$\\frac{b^{3}}{c}$+$\\frac{c^{3}}{a}$ $\\geq$ $a\\sqrt{ac}$+$b\\sqrt{ba}$+$c\\sqrt{cb}$ :wink:\r\n\r\nAnd this is the solution: \r\nby $AM-GM$, $\\frac{a^{3}}{b}+\\frac{b^{3}}{c}+\\frac{c^{3}}{a}= \\sum_{cyc}(\\frac{15}{26}\\cdot \\frac{a^{3}}{b}+\\frac{5}{26}\\cdot \\frac{b^{3}}{c}+\\frac{3}{13}\\cdot \\frac{c^{3}}{a}) \\geq \\sum_{cyc}a\\sqrt{ac}=a\\sqrt{ac}+b\\sqrt{ba}+c\\sqrt{cb}$ :D" } { "Tag": [ "logarithms" ], "Problem": "Can an irrational exponent be reduced to something more manageable? (example: 2^sqrt3)...? :?:", "Solution_1": "You can use logarithms but that doesn't really make it any simpler." } { "Tag": [ "search", "linear algebra" ], "Problem": "\u0398\u03ad\u03bb\u03c9 \u03c4\u03b7 \u03b2\u03bf\u03ae\u03b8\u03b5\u03b9\u03ac \u03c3\u03b1\u03c2 \u03c3\u03b5 2 \u03c0\u03c1\u03bf\u03b2\u03bb\u03ae\u03bc\u03b1\u03c4\u03b1.\r\n\r\n1) \u0388\u03c3\u03c4\u03c9 \u03c3\u03cd\u03bd\u03bf\u03bb\u03bf $ A$ \u03bc\u03b5 \u03c0\u03bb\u03b7\u03b8\u03b9\u03ba\u03cc\u03c4\u03b7\u03c4\u03b1 $ n$ \u03ba\u03b1\u03b9 \u03ad\u03c3\u03c4\u03c9 $ S$ \u03ad\u03bd\u03b1 \u03c3\u03cd\u03bd\u03bf\u03bb\u03bf \u03c0\u03bf\u03c5 \u03ad\u03c7\u03b5\u03b9 \u03b3\u03b9\u03b1 \u03c3\u03c4\u03bf\u03b9\u03c7\u03b5\u03af\u03b1 \u03c4\u03bf\u03c5 $ n \\plus{} 1$ \u03c5\u03c0\u03bf\u03c3\u03cd\u03bd\u03bf\u03bb\u03b1 \u03c4\u03bf\u03c5 $ A$. \u039d\u03b1 \u03b1\u03c0\u03bf\u03b4\u03b5\u03b9\u03c7\u03b8\u03b5\u03af \u03cc\u03c4\u03b9 \u03bc\u03c0\u03bf\u03c1\u03bf\u03cd\u03bc\u03b5 \u03bd\u03b1 \u03b2\u03c1\u03bf\u03cd\u03bc\u03b5 2 \u03be\u03ad\u03bd\u03b1 \u03bc\u03b5\u03c4\u03b1\u03be\u03cd \u03c4\u03bf\u03c5\u03c2 \u03c5\u03c0\u03bf\u03c3\u03cd\u03bd\u03bf\u03bb\u03b1 $ S1$ \u03ba\u03b1\u03b9 $ S2$ \u03c4\u03bf\u03c5 $ S$ (\u03bd\u03b1 \u03ad\u03c7\u03bf\u03c5\u03bd \u03b4\u03b7\u03bb\u03b1\u03b4\u03ae \u03b3\u03b9\u03b1 \u03c3\u03c4\u03bf\u03b9\u03c7\u03b5\u03af\u03b1 \u03c4\u03bf\u03c5\u03c2 \u03ba\u03ac\u03c0\u03bf\u03b9\u03b1 \u03b1\u03c0\u03cc \u03c4\u03b1 $ n \\plus{} 1$ \u03b1\u03c5\u03c4\u03ac \u03c5\u03c0\u03bf\u03c3\u03cd\u03bd\u03bf\u03bb\u03b1 \u03c4\u03bf\u03c5 $ A$) \u03ce\u03c3\u03c4\u03b5 union $ S1$ \u03bd\u03b1 \u03b5\u03af\u03bd\u03b1\u03b9 \u03af\u03c3\u03b7 \u03bc\u03b5 union $ S2$.\r\n\r\n\u0395\u03af\u03bd\u03b1\u03b9 \u03b5\u03cd\u03ba\u03bf\u03bb\u03bf \u03bd\u03b1 \u03b1\u03c0\u03bf\u03b4\u03b5\u03b9\u03c7\u03b8\u03b5\u03af \u03cc\u03c4\u03b9 \u03c5\u03c0\u03ac\u03c1\u03c7\u03bf\u03c5\u03bd \u03c4\u03ad\u03c4\u03bf\u03b9\u03b1 \u03c3\u03cd\u03bd\u03bf\u03bb\u03b1 \u03c0\u03bf\u03c5 \u03b4\u03b5\u03bd \u03b5\u03af\u03bd\u03b1\u03b9 \u03be\u03ad\u03bd\u03b1 \u03bc\u03b5\u03c4\u03b1\u03be\u03cd \u03c4\u03bf\u03c5\u03c2 \u03b1\u03bb\u03bb\u03ac \u03c0\u03ce\u03c2 \u03b5\u03be\u03b1\u03c3\u03c6\u03b1\u03bb\u03af\u03b6\u03c9 \u03cc\u03c4\u03b9 \u03c5\u03c0\u03ac\u03c1\u03c7\u03bf\u03c5\u03bd \u03be\u03ad\u03bd\u03b1 \u03bc\u03b5\u03c4\u03b1\u03be\u03cd \u03c4\u03bf\u03c5\u03c2 \u03bc\u03b5 \u03c4\u03b7\u03bd \u03af\u03b4\u03b9\u03b1 \u03b9\u03b4\u03b9\u03cc\u03c4\u03b7\u03c4\u03b1? \u03a0\u03c1\u03bf\u03c6\u03b1\u03bd\u03ce\u03c2 \u03b4\u03b5\u03bd \u03bc\u03c0\u03bf\u03c1\u03ce \u03bd\u03b1 \u03b1\u03c6\u03b1\u03b9\u03c1\u03ad\u03c3\u03c9 \u03c4\u03b7\u03bd \u03c4\u03bf\u03bc\u03ae \u03c4\u03bf\u03c5\u03c2 \u03b3\u03b9\u03b1\u03c4\u03af \u03bc\u03c0\u03bf\u03c1\u03b5\u03af \u03ba\u03ac\u03c0\u03bf\u03b9\u03bf \u03c3\u03c4\u03bf\u03b9\u03c7\u03b5\u03af\u03bf \u03ba\u03ac\u03c0\u03bf\u03b9\u03bf\u03c5 \u03c5\u03c0\u03bf\u03c3\u03c5\u03bd\u03cc\u03bb\u03bf\u03c5 \u03c4\u03b7\u03c2 \u03c4\u03bf\u03bc\u03ae\u03c2 \u03bc\u03c0\u03bf\u03c1\u03b5\u03af \u03bd\u03b1 \u03b1\u03bd\u03ae\u03ba\u03b5\u03b9 \u03ba\u03b1\u03b9 \u03c3\u03b5 \u03ba\u03ac\u03c0\u03bf\u03b9\u03bf \u03ac\u03bb\u03bb\u03bf \u03c5\u03c0\u03bf\u03c3\u03cd\u03bd\u03bf\u03bb\u03bf \u03c0\u03bf\u03c5 \u03b4\u03b5\u03bd \u03b5\u03af\u03bd\u03b1\u03b9 \u03c3\u03c4\u03b7\u03bd \u03c4\u03bf\u03bc\u03ae... (\u03b4\u03b7\u03bb\u03b1\u03b4\u03ae \u03b4\u03b5\u03bd \u03b5\u03af\u03bd\u03b1\u03b9 \u03c4\u03bf \u03af\u03b4\u03b9\u03bf \u03bc\u03b5 \u03c4\u03bf \u03c0\u03b1\u03c3\u03af\u03b3\u03bd\u03c9\u03c3\u03c4\u03bf \u03c0\u03c1\u03cc\u03b2\u03bb\u03b7\u03bc\u03b1 \u03bc\u03b5 \u03c4\u03bf\u03c5\u03c2 10 \u03b1\u03c1\u03b9\u03b8\u03bc\u03bf\u03cd\u03c2 \u03ba\u03b1\u03b9 \u03c4\u03b1 \u03b1\u03b8\u03c1\u03bf\u03af\u03c3\u03bc\u03b1\u03c4\u03b1)\r\n\r\n2) \u039d\u03b1 \u03b1\u03c0\u03bf\u03b4\u03b5\u03b9\u03c7\u03b8\u03b5\u03af \u03cc\u03c4\u03b9 \u03ba\u03ac\u03b8\u03b5 linear map $ T$ \u03b1\u03c0\u03cc \u03c4\u03bf $ V$ \u03c3\u03c4\u03bf $ V$ \u03bc\u03c0\u03bf\u03c1\u03b5\u03af \u03bd\u03b1 \u03b3\u03c1\u03b1\u03c6\u03c4\u03b5\u03af \u03c9\u03c2 \u03c4\u03bf \u03b5\u03c5\u03b8\u03cd \u03ac\u03b8\u03c1\u03bf\u03b9\u03c3\u03bc\u03b1 (direct sum) \u03b5\u03bd\u03cc\u03c2 invertible \u03ba\u03b9 \u03b5\u03bd\u03cc\u03c2 nilpotent operator.\r\n\r\n\u0391\u03c1\u03ba\u03b5\u03af \u03bb\u03bf\u03b9\u03c0\u03cc\u03bd \u03bd\u03b1 \u03b4\u03b5\u03af\u03be\u03bf\u03c5\u03bc\u03b5 \u03cc\u03c4\u03b9 \u03bc\u03c0\u03bf\u03c1\u03bf\u03cd\u03bc\u03b5 \u03bd\u03b1 \u03b3\u03c1\u03ac\u03c8\u03bf\u03c5\u03bc\u03b5 \u03c4\u03bf $ V$ \u03c9\u03c2 \u03c4\u03bf \u03b5\u03c5\u03b8\u03cd \u03ac\u03b8\u03c1\u03bf\u03b9\u03c3\u03bc\u03b1 \u03b4\u03cd\u03bf \u03c5\u03c0\u03bf\u03c7\u03ce\u03c1\u03c9\u03bd $ A$ \u03ba\u03b1\u03b9 $ B$ \u03b1\u03bd\u03b1\u03bb\u03bb\u03bf\u03af\u03c9\u03c4\u03c9\u03bd \u03c9\u03c2 \u03c0\u03c1\u03bf\u03c2 \u03c4\u03bf $ T$ \u03ce\u03c3\u03c4\u03b5 \u03c4\u03bf restriction \u03c4\u03bf\u03c5 $ T$ \u03c3\u03c4\u03bf $ A$ \u03bd\u03b1 \u03b5\u03af\u03bd\u03b1\u03b9 nilpotent (\u03b4\u03b7\u03bb\u03b1\u03b4\u03ae $ T^n \\equal{} 0$ \u03b3\u03b9\u03b1 \u03ba\u03ac\u03c0\u03bf\u03b9\u03bf $ n$) \u03ba\u03b1\u03b9 \u03c4\u03bf restriction \u03c4\u03bf\u03c5 $ T$ \u03c3\u03c4\u03bf $ B$ \u03bd\u03b1 \u03b5\u03af\u03bd\u03b1\u03b9 invertible. \u0394\u03b9\u03b1\u03b9\u03c3\u03b8\u03b7\u03c4\u03b9\u03ba\u03ac \u03ba\u03b1\u03b9 \u03bc\u03cc\u03bd\u03bf \u03b1\u03bd\u03c4\u03b9\u03bb\u03b1\u03bc\u03b2\u03ac\u03bd\u03bf\u03bc\u03b1\u03b9 \u03cc\u03c4\u03b9 \u03b1\u03c5\u03c4\u03cc \u03ad\u03c7\u03b5\u03b9 \u03c3\u03c7\u03ad\u03c3\u03b7 \u03bc\u03b5 \u03c4\u03bf Jordan Canonical Form \u03b5\u03bd\u03cc\u03c2 \u03c0\u03af\u03bd\u03b1\u03ba\u03b1 (\u03b4\u03b9\u03cc\u03c4\u03b9 \u03bf \u03c0\u03af\u03bd\u03b1\u03ba\u03b1\u03c2 \u03c4\u03bf\u03c5 nilpotent \u03ad\u03c7\u03b5\u03b9 \u03bc\u03b7\u03b4\u03b5\u03bd\u03b9\u03ba\u03ac \u03c0\u03b1\u03bd\u03c4\u03bf\u03cd \u03b5\u03ba\u03c4\u03cc\u03c2 \u03af\u03c3\u03c9\u03c2 \u03b1\u03c0\u03cc \u03c4\u03bf \u03ac\u03bd\u03c9 \u03c4\u03c1\u03af\u03b3\u03c9\u03bd\u03bf \u03ba\u03b1\u03b9 \u03bf \u03c0\u03af\u03bd\u03b1\u03ba\u03b1\u03c2 \u03c4\u03bf\u03c5 invertible \u03b4\u03b5\u03bd \u03ad\u03c7\u03b5\u03b9 eigenvalue zero, \u03b4\u03b7\u03bb\u03b1\u03b4\u03ae \u03c3\u03c4\u03b7 \u03b4\u03b9\u03b1\u03b3\u03ce\u03bd\u03b9\u03bf \u03c4\u03bf 0) \u03b1\u03bb\u03bb\u03ac \u03b4\u03b5\u03bd \u03bc\u03c0\u03bf\u03c1\u03ce \u03bd\u03b1 \u03b4\u03ce\u03c3\u03c9 \u03b1\u03c0\u03cc\u03b4\u03b5\u03b9\u03be\u03b7.\r\n\r\n\u0395\u03c5\u03c7\u03b1\u03c1\u03b9\u03c3\u03c4\u03ce \u03c0\u03bf\u03bb\u03cd.", "Solution_1": "\u03a4\u03bf \u03c0\u03c1\u03ce\u03c4\u03bf \u03c0\u03c1\u03cc\u03b2\u03bb\u03b7\u03bc\u03b1 \u03c4\u03bf \u03bb\u03c5\u03c3\u03b1. \u0398\u03b1 \u03b1\u03bd\u03b1\u03c1\u03c4\u03ae\u03c3\u03c9 \u03bb\u03cd\u03c3\u03b7 \u03b3\u03b9\u03b1 \u03c4\u03bf\u03bd \u03ba\u03ac\u03b8\u03b5 \u03b5\u03bd\u03b4\u03b9\u03b1\u03c6\u03b5\u03c1\u03cc\u03bc\u03b5\u03bd\u03bf \u03b1\u03c1\u03b3\u03cc\u03c4\u03b5\u03c1\u03b1.\r\n\r\n\u0393\u03b9\u03b1 \u03c4\u03bf \u03b4\u03b5\u03cd\u03c4\u03b5\u03c1\u03bf \u03b1\u03ba\u03cc\u03bc\u03b1 \u03b4\u03b5\u03bd \u03ad\u03c7\u03c9 \u03b1\u03c5\u03c3\u03c4\u03b7\u03c1\u03ae \u03b1\u03c0\u03cc\u03b4\u03b5\u03b9\u03be\u03b7.", "Solution_2": "\u039a\u03ac\u03c4\u03b9 \u03bc\u03bf\u03c5 \u03b8\u03c5\u03bc\u03af\u03b6\u03b5\u03b9 \u03c4\u03bf \u03c0\u03c1\u03ce\u03c4\u03bf... :roll: :roll: :roll: \r\n\r\n\u0394\u03b5\u03c2 \u03b5\u03b4\u03ce \u03b8\u03ad\u03bc\u03b1 3\u03bf...", "Solution_3": "\u0397 \u03bb\u03cd\u03c3\u03b7 \u03bc\u03bf\u03c5 \u03c3\u03c4\u03bf \u03c0\u03c1\u03ce\u03c4\u03bf:\r\n\r\n\u0388\u03c3\u03c4\u03c9 $ A\\equal{}{a1,.....an}$\r\n\r\n\u0388\u03c3\u03c4\u03c9 $ S\\equal{}{A1, A2,.....A(n\\plus{}1)}$\r\n\r\n\u03a3\u03b5 \u03ba\u03b1\u03b8\u03ad\u03bd\u03b1 \u03b1\u03c0\u03cc \u03b1\u03c5\u03c4\u03ac \u03c4\u03b1 \u03c5\u03c0\u03bf\u03c3\u03cd\u03bd\u03bf\u03bb\u03b1 $ A(i)$ \u03b1\u03bd\u03c4\u03b9\u03c3\u03c4\u03bf\u03b9\u03c7\u03af\u03b6\u03c9 \u03ad\u03bd\u03b1 \u03b4\u03b9\u03ac\u03bd\u03c5\u03c3\u03bc\u03b1 $ v(j)\\equal{}(v(1j), v(2j), ... v(nj))$ \u03ce\u03c3\u03c4\u03b5:\r\n\r\n$ v(ij)\\equal{}1$ \u03b1\u03bd \u03c4\u03bf $ a(i)$ \u03b1\u03bd\u03ae\u03ba\u03b5\u03b9 \u03c3\u03c4\u03bf $ A(i)$\r\n\r\n\u03ae\r\n\r\n$ v(ij)\\equal{}0$ \u03b1\u03bd \u03c4\u03bf $ a(i)$ \u03b4\u03b5\u03bd \u03b1\u03bd\u03ae\u03ba\u03b5\u03b9 \u03c3\u03c4\u03bf $ A(i)$\r\n\r\n\u0391\u03c6\u03bf\u03cd \u03c4\u03b1 \u03b4\u03b9\u03b1\u03bd\u03cd\u03c3\u03bc\u03b1\u03c4\u03b1 \u03b1\u03c5\u03c4\u03ac \u03b5\u03af\u03bd\u03b1\u03b9 $ n\\plus{}1$ \u03c5\u03c0\u03ac\u03c1\u03c7\u03b5\u03b9 \u03b3\u03c1\u03b1\u03bc\u03bc\u03b9\u03ba\u03cc\u03c2 \u03c4\u03bf\u03c5\u03c2 \u03c3\u03c5\u03bd\u03b4\u03c5\u03b1\u03c3\u03bc\u03cc\u03c2 (\u03cc\u03c7\u03b9 \u03cc\u03bb\u03b1 \u03c4\u03b1 scalars \u03af\u03c3\u03b1 \u03bc\u03b5 \u03c4\u03bf $ 0$) \u03c0\u03bf\u03c5 \u03b9\u03c3\u03bf\u03cd\u03c4\u03b1\u03b9 \u03bc\u03b5 $ 0$.\r\n\r\n\u039e\u03b5\u03c7\u03c9\u03c1\u03af\u03b6\u03c9 \u03c4\u03ce\u03c1\u03b1 \u03c4\u03bf\u03c5\u03c2 \u03b8\u03b5\u03c4\u03b9\u03ba\u03bf\u03cd\u03c2 \u03ba\u03b1\u03b9 \u03b1\u03c1\u03bd\u03b7\u03c4\u03b9\u03ba\u03bf\u03cd\u03c2 scalars \u03ba\u03b1\u03b9 \u03ad\u03c7\u03c9\r\n\r\n$ \\sum(m(i)v(i))\\equal{}\\sum(m(j)v(j))$\r\n\r\n(\u03bf\u03b9 \u03b4\u03b5\u03af\u03ba\u03c4\u03b5\u03c2 $ i, j$ \u03b1\u03bb\u03bb\u03ac\u03be\u03b1\u03bd \u03bd\u03cc\u03b7\u03bc\u03b1)\r\n\r\n\u03a4\u03ce\u03c1\u03b1 \u03b5\u03af\u03bd\u03b1\u03b9 \u03c3\u03c7\u03b5\u03c4\u03b9\u03ba\u03ac \u03b1\u03c0\u03bb\u03cc \u03bd\u03b1 \u03b4\u03bf\u03cd\u03bc\u03b5 \u03b3\u03b9\u03b1\u03c4\u03af \u03c4\u03bf union \u03c4\u03c9\u03bd $ A(i)$ \u03b5\u03af\u03bd\u03b1\u03b9 \u03af\u03c3\u03bf \u03bc\u03b5 \u03c4\u03bf union \u03c4\u03c9\u03bd $ A(j)$\r\n\r\n\u03a4\u03bf \u03b1\u03c6\u03ae\u03bd\u03c9 \u03c3\u03b1\u03bd \u03ac\u03c3\u03ba\u03b7\u03c3\u03b7 \u03b1\u03bb\u03bb\u03ac \u03b1\u03bd \u03ba\u03ac\u03c0\u03bf\u03b9\u03c2 \u03b8\u03ad\u03bb\u03b5\u03b9 \u03c4\u03b7 \u03bb\u03cd\u03c3\u03b7 \u03c4\u03b7 \u03b3\u03c1\u03ac\u03c6\u03c9.", "Solution_4": "[quote=\"Protonios\"]2) \u039d\u03b1 \u03b1\u03c0\u03bf\u03b4\u03b5\u03b9\u03c7\u03b8\u03b5\u03af \u03cc\u03c4\u03b9 \u03ba\u03ac\u03b8\u03b5 linear map $ T$ \u03b1\u03c0\u03cc \u03c4\u03bf $ V$ \u03c3\u03c4\u03bf $ V$ \u03bc\u03c0\u03bf\u03c1\u03b5\u03af \u03bd\u03b1 \u03b3\u03c1\u03b1\u03c6\u03c4\u03b5\u03af \u03c9\u03c2 \u03c4\u03bf \u03b5\u03c5\u03b8\u03cd \u03ac\u03b8\u03c1\u03bf\u03b9\u03c3\u03bc\u03b1 (direct sum) \u03b5\u03bd\u03cc\u03c2 invertible \u03ba\u03b9 \u03b5\u03bd\u03cc\u03c2 nilpotent operator.\n\n\u0391\u03c1\u03ba\u03b5\u03af \u03bb\u03bf\u03b9\u03c0\u03cc\u03bd \u03bd\u03b1 \u03b4\u03b5\u03af\u03be\u03bf\u03c5\u03bc\u03b5 \u03cc\u03c4\u03b9 \u03bc\u03c0\u03bf\u03c1\u03bf\u03cd\u03bc\u03b5 \u03bd\u03b1 \u03b3\u03c1\u03ac\u03c8\u03bf\u03c5\u03bc\u03b5 \u03c4\u03bf $ V$ \u03c9\u03c2 \u03c4\u03bf \u03b5\u03c5\u03b8\u03cd \u03ac\u03b8\u03c1\u03bf\u03b9\u03c3\u03bc\u03b1 \u03b4\u03cd\u03bf \u03c5\u03c0\u03bf\u03c7\u03ce\u03c1\u03c9\u03bd $ A$ \u03ba\u03b1\u03b9 $ B$ \u03b1\u03bd\u03b1\u03bb\u03bb\u03bf\u03af\u03c9\u03c4\u03c9\u03bd \u03c9\u03c2 \u03c0\u03c1\u03bf\u03c2 \u03c4\u03bf $ T$ \u03ce\u03c3\u03c4\u03b5 \u03c4\u03bf restriction \u03c4\u03bf\u03c5 $ T$ \u03c3\u03c4\u03bf $ A$ \u03bd\u03b1 \u03b5\u03af\u03bd\u03b1\u03b9 nilpotent (\u03b4\u03b7\u03bb\u03b1\u03b4\u03ae $ T^n \\equal{} 0$ \u03b3\u03b9\u03b1 \u03ba\u03ac\u03c0\u03bf\u03b9\u03bf $ n$) \u03ba\u03b1\u03b9 \u03c4\u03bf restriction \u03c4\u03bf\u03c5 $ T$ \u03c3\u03c4\u03bf $ B$ \u03bd\u03b1 \u03b5\u03af\u03bd\u03b1\u03b9 invertible. \u0394\u03b9\u03b1\u03b9\u03c3\u03b8\u03b7\u03c4\u03b9\u03ba\u03ac \u03ba\u03b1\u03b9 \u03bc\u03cc\u03bd\u03bf \u03b1\u03bd\u03c4\u03b9\u03bb\u03b1\u03bc\u03b2\u03ac\u03bd\u03bf\u03bc\u03b1\u03b9 \u03cc\u03c4\u03b9 \u03b1\u03c5\u03c4\u03cc \u03ad\u03c7\u03b5\u03b9 \u03c3\u03c7\u03ad\u03c3\u03b7 \u03bc\u03b5 \u03c4\u03bf Jordan Canonical Form \u03b5\u03bd\u03cc\u03c2 \u03c0\u03af\u03bd\u03b1\u03ba\u03b1 (\u03b4\u03b9\u03cc\u03c4\u03b9 \u03bf \u03c0\u03af\u03bd\u03b1\u03ba\u03b1\u03c2 \u03c4\u03bf\u03c5 nilpotent \u03ad\u03c7\u03b5\u03b9 \u03bc\u03b7\u03b4\u03b5\u03bd\u03b9\u03ba\u03ac \u03c0\u03b1\u03bd\u03c4\u03bf\u03cd \u03b5\u03ba\u03c4\u03cc\u03c2 \u03af\u03c3\u03c9\u03c2 \u03b1\u03c0\u03cc \u03c4\u03bf \u03ac\u03bd\u03c9 \u03c4\u03c1\u03af\u03b3\u03c9\u03bd\u03bf \u03ba\u03b1\u03b9 \u03bf \u03c0\u03af\u03bd\u03b1\u03ba\u03b1\u03c2 \u03c4\u03bf\u03c5 invertible \u03b4\u03b5\u03bd \u03ad\u03c7\u03b5\u03b9 eigenvalue zero, \u03b4\u03b7\u03bb\u03b1\u03b4\u03ae \u03c3\u03c4\u03b7 \u03b4\u03b9\u03b1\u03b3\u03ce\u03bd\u03b9\u03bf \u03c4\u03bf 0) \u03b1\u03bb\u03bb\u03ac \u03b4\u03b5\u03bd \u03bc\u03c0\u03bf\u03c1\u03ce \u03bd\u03b1 \u03b4\u03ce\u03c3\u03c9 \u03b1\u03c0\u03cc\u03b4\u03b5\u03b9\u03be\u03b7.[/quote]\r\n\r\nProtonios, sorry, \u03ae\u03b8\u03b5\u03bb\u03b1 \u03bd'\u03b1\u03c0\u03b1\u03bd\u03c4\u03ae\u03c3\u03c9 \u03ba\u03b1\u03b9 \u03be\u03b5\u03c7\u03ac\u03c3\u03c4\u03b7\u03ba\u03b1...\r\n\r\nAnyway, \u03c3\u03b5 \u03c0\u03b5\u03c1\u03af\u03c0\u03c4\u03c9\u03c3\u03b7 \u03c0\u03bf\u03c5 \u03c3\u03bf\u03c5 \u03b5\u03af\u03bd\u03b1\u03b9 \u03b1\u03ba\u03cc\u03bc\u03b1 \u03c7\u03c1\u03ae\u03c3\u03b9\u03bc\u03bf,\u03b7 \u03b4\u03b9\u03b1\u03af\u03c3\u03b8\u03b7\u03c3\u03ae \u03c3\u03bf\u03c5 \u03ba\u03b1\u03bb\u03ac \u03c3\u03b5 \u03c0\u03ae\u03b3\u03b5. \u03a0\u03c1\u03ad\u03c0\u03b5\u03b9 \u03b2\u03ad\u03b2\u03b1\u03b9\u03b1 \u03bd\u03b1 \u03c0\u03c1\u03bf\u03c3\u03b4\u03b9\u03bf\u03c1\u03af\u03c3\u03b5\u03b9\u03c2 \u03b1\u03bd \u03bc\u03b9\u03bb\u03ac\u03c2 \u03b3\u03b9\u03b1 \u03c7\u03ce\u03c1\u03bf\u03c5\u03c2 \u03c0\u03b5\u03c0\u03b5\u03c1\u03b1\u03c3\u03bc\u03ad\u03bd\u03b7\u03c2 \u03ae \u03ac\u03c0\u03b5\u03b9\u03c1\u03b7\u03c2 \u03b4\u03b9\u03ac\u03c3\u03c4\u03b1\u03c3\u03b7\u03c2 \u03ba\u03b1\u03b9 \u03b1\u03bd \u03c4\u03bf \u03c3\u03ce\u03bc\u03b1 \u03c4\u03c9\u03bd \u03c7\u03ce\u03c1\u03c9\u03bd \u03c3\u03bf\u03c5 \u03b5\u03af\u03bd\u03b1\u03b9 \u03b1\u03bb\u03b3\u03b5\u03b2\u03c1\u03b9\u03ba\u03ac \u03ba\u03bb\u03b5\u03b9\u03c3\u03c4\u03cc \u03ae \u03cc\u03c7\u03b9, \u03b1\u03bb\u03bb\u03ac \u03b1\u03c5\u03c4\u03cc \u03c0\u03bf\u03c5 \u03c8\u03ac\u03c7\u03bd\u03b5\u03b9\u03c2 \u03bb\u03ad\u03b3\u03b5\u03c4\u03b1\u03b9 \u03b1\u03bd\u03ac\u03bb\u03c5\u03c3\u03b7 Jordan-Chevalley.\r\n\u0395\u03af\u03bd\u03b1\u03b9 \u03b3\u03b5\u03bd\u03b9\u03ba\u03ac \u03b1\u03c1\u03ba\u03b5\u03c4\u03ac \u03b3\u03bd\u03c9\u03c3\u03c4\u03ae \u03ba\u03b1\u03b9 \u03bd\u03bf\u03bc\u03af\u03b6\u03c9 \u03c5\u03c0\u03ac\u03c1\u03c7\u03b5\u03b9 \u03c3\u03c4\u03bf Linear algebra done right \u03ae \u03c3\u03c4\u03b7\u03bd \u03ac\u03bb\u03b3\u03b5\u03b2\u03c1\u03b1 \u03c4\u03bf\u03c5 Lang... \u03b1\u03bb\u03bb\u03ac \u03b1\u03bd \u03b8\u03ad\u03bb\u03b5\u03b9\u03c2 \u03ba\u03ac\u03c4\u03b9 \u03b3\u03c1\u03ae\u03b3\u03bf\u03c1\u03bf \u03ba\u03b1\u03b9 \u03b1\u03c0\u03bb\u03cc, \u03b2\u03c1\u03ae\u03ba\u03b1 \u03b1\u03c5\u03c4\u03cc \u03bc\u03b5 \u03ad\u03bd\u03b1 google search:\r\n\r\n[url]http://www.math.kochi-u.ac.jp/docky/bourdoki/NAS/nas006/node5.html[/url]\r\n\r\n\u039d\u03bf\u03bc\u03af\u03b6\u03c9 \u03ba\u03ac\u03c4\u03b9 \u03b1\u03bd\u03ac\u03bb\u03bf\u03b3\u03bf \u03c5\u03c0\u03ac\u03c1\u03c7\u03b5\u03b9 \u03ba\u03b1\u03b9 \u03b3\u03b9\u03b1 \u03c7\u03ce\u03c1\u03bf\u03c5\u03c2 Banach, \u03b1\u03bb\u03bb\u03ac \u03b4\u03b5 \u03b8\u03c5\u03bc\u03ac\u03bc\u03b1\u03b9 \u03c0\u03ce\u03c2 \u03b1\u03ba\u03c1\u03b9\u03b2\u03ce\u03c2 \u03c0\u03ac\u03b5\u03b9 \u03ba\u03b1\u03b9 \u03b1\u03bd \u03c3\u03c5\u03bc\u03b2\u03b1\u03af\u03bd\u03b5\u03b9 \u03ba\u03ac\u03c4\u03b9 \u03b4\u03b9\u03b1\u03c6\u03bf\u03c1\u03b5\u03c4\u03b9\u03ba\u03cc...\r\n\r\nA\u03bd \u03c7\u03c1\u03b5\u03b9\u03b1\u03c3\u03c4\u03b5\u03af\u03c2 \u03b2\u03bf\u03ae\u03b8\u03b5\u03b9\u03b1, let me know :) Cheerio,\r\n\r\nDurandal 1707", "Solution_5": "\u0395\u03c5\u03c7\u03b1\u03c1\u03b9\u03c3\u03c4\u03ce \u03c0\u03bf\u03bb\u03cd. \u03a0\u03bb\u03ad\u03bf\u03bd \u03ac\u03bc\u03b5\u03c3\u03b1 \u03c7\u03c1\u03ae\u03c3\u03b9\u03bc\u03bf \u03b4\u03b5\u03bd \u03bc\u03bf\u03c5 \u03b5\u03af\u03bd\u03b1\u03b9 :P \r\n\r\n\u0391\u03bb\u03bb\u03ac \u03b3\u03b5\u03bd\u03b9\u03ba\u03ac \u03c7\u03c1\u03ae\u03c3\u03b9\u03bc\u03bf \u03c6\u03c5\u03c3\u03b9\u03ba\u03ac \u03b5\u03af\u03bd\u03b1\u03b9. \u039d\u03b1 \u03c3\u03b1\u03b9 \u03ba\u03b1\u03bb\u03ac." } { "Tag": [ "calculus", "inequalities", "three variable inequality" ], "Problem": "Let be given positive reals $a$, $b$, $c$. Prove that: $\\frac{a^{3}}{\\left(a+b\\right)^{3}}+\\frac{b^{3}}{\\left(b+c\\right)^{3}}+\\frac{c^{3}}{\\left(c+a\\right)^{3}}\\geq \\frac{3}{8}$.", "Solution_1": "[quote=\"orl\"]Let be given $a,b,c>0$. Prove that: $\\sum \\frac{a^3}{(a+b)^3} \\geq \\frac{3}{8}$.[/quote]\r\n.$\\sum \\frac{a^3}{(a+b)^3}=\\sum\\frac{a^4}{a(a+b)^3}$. By Cauchy's inequlity .", "Solution_2": "[quote=\"xiaozg\"].$\\sum \\frac {a^3}{(a+b)^3}=\\sum\\frac{a^4}{a*(a+b)^3}$. By Cauchy's inequlitity and A-G inequlitity.[/quote]\r\nEhhh... sorry but how can you prove that with cauchy ?? :? :?", "Solution_3": "Well, Cauchy (in engel form) gives us\r\n\r\n$\\sum \\frac {a^4}{a(a+b)^3}\\geq \\frac {(a^2+b^2+c^2)^2}{\\sum a(a+b)^3}$\r\n\r\nbut I believe this is too weak.", "Solution_4": "Lets do the work:\r\n\r\n$8(a^4 + b^4 + c^4) + 16(a^2b^2 + b^2c^2 + c^2a^2) \\geq \\sum 3(a^4 + 3a^3b + 3a^2b^2 + ab^3)$\r\n\r\n$\\sum 5a^4 + 7a^2b^2 \\geq \\sum 9a^3b + 3ab^3)$\r\n\r\nNow $a^4 + a^2b^2 \\geq 2a^3b$.\r\n\r\n$\\sum a^4 + 3a^2b^2 \\geq \\sum a^3b + 3ab^3$\r\n\r\n$\\sum 2a^2b^2 \\geq \\sum ab(a^2 + b^2)$\r\n\r\n$\\sum 2ab \\geq \\sum a^2 + b^2$\r\n\r\nso it is bad, for sure.", "Solution_5": "it is not necesary bad...\r\nin fact you can use better inequalities.. for example, use that $\\sum{a^4}+\\sum{a^2bc} \\geq \\sum{a^3b}+\\sum{ab^3}$ and it will sufice to prove that $2\\sum{a^4}+4\\sum{a^2b^2} \\geq 6\\sum{a^3b}$ which doesn't look so obviously wrong...\r\n\r\n apart from that, you can not cancel terms when you have cyclic sums..", "Solution_6": "firstly, my cancelling was correct.\r\n\r\n--\r\n\r\nat the second step, we have only expanded and cancelled. So lets go there.\r\n\r\nworking with: $\\displaystyle \\sum 5a^4 + 7a^2b^2 \\geq \\sum 9a^3b + 3ab^3$... \r\n\r\nhow did you find a a^2bc term?", "Solution_7": "^\r\n\r\nI got the same thing when I expanded as singular did, the sums $\\sum a^4$ and $\\sum a^2b^2$ are not really cyclic, its only the $\\sum a^3b$ terms.", "Solution_8": "Does anyone has a better solution for this problem ?", "Solution_9": "can you read the other thread of this problem?\r\na solution is posted already.", "Solution_10": "yes I know but I just asked for a better solution, not one which is using calculus", "Solution_11": "[quote=\"erdos\"]yes I know but I just asked for a better solution, not one which is using calculus[/quote]\r\n\r\nI think the one I saw is not using calculus?\r\nPower mean + an old ineq on the forum.", "Solution_12": "Yes, in ML circles, the inequality\r\n\r\n$a,b,c,d\\geq0$, $abcd=1$,\r\n\r\n$\\sum \\frac {1}{(1+a)^2}\\geq 1$\r\n\r\nis about as famous as it gets", "Solution_13": "It's really true Singular\r\n$\\sum 5a^4+\\sum 7a^2b^2\\ge \\sum 9a^3b+\\sum 3ab^3$\r\nWe have $4a^4+b^4+7a^2b^2-9a^3b-3ab^3=(a-b)^2(4a^2-ab+b^2)\\ge 0$\r\nAnd the same with the 2 other.", "Solution_14": "My student suggested following proof:\r\n\r\n$\\sum a^3/(a+b)^3 * \\sum c^3(a+b)^3 \\ge (\\sum ac^{3/2})^2$\r\n\r\nThen, this enough to show that\r\n\r\n$8(\\sum ac^{3/2})^2 \\ge 3(\\sum c^3(a+b)^3) (1) $ \r\n\r\nPut $\\sqrt(ab) = x, \\sqrt(cb) = y, \\sqrt(ac) = z$ (1) becomes\r\n\r\n$8(\\sum x^3)^2 \\ge 3(\\sum (x^2+y^2)^3) $\r\n\r\nwhich leads to an trivial inequality.\r\n\r\nBut I think, the nicest proof it's the one that uses lemma\r\n\r\n$ 1/(1+x)^2 + 1/(1+y)^2 \\ge 1/(1+xy) $", "Solution_15": "utkarshgupta I posted a solution regarding this problem:\n\nProve that for all $a,b,c$ positive real numbers the following statement is true, $ \\frac{a^{3}}{\\left(a+b\\right)^{3}}+\\frac{b^{3}}{\\left(b+c\\right)^{3}}+\\frac{c^{3}}{\\left(c+a\\right)^{3}}\\geq\\frac{3}{8} $. What is wrong?", "Solution_16": "nkalosidhs, you are trying to prove another inequality. :wink:", "Solution_17": "Oh...excuse me...I saw the first inequality on page 1 and then tried to post my solution without realizing that it is posted on third page! Well, I would like to just check whether my solution is correct! :wink:", "Solution_18": "Read your post please.\n[quote=\"nkalosidhs\"][quote=\"orl\"]Let be given positive reals $a$, $b$, $c$. Prove that: $\\frac{a^{3}}{\\left(a+b\\right)^{3}}+\\frac{b^{3}}{\\left(b+c\\right)^{3}}+\\frac{c^{3}}{\\left(c+a\\right)^{3}}\\geq \\frac{3}{8}$.[/quote]\n\nLet denote $A=\\frac{a^3}{(b+c)^3}+\\frac{b^3}{(a+c)^3}+\\frac{c^3}{(a+b)^3}$\nthen we should prove that $A\\geq\\frac{3}{8}$\n[/quote]\nWe don't need to prove that $A\\geq\\frac{3}{8}$. :wink:", "Solution_19": "Well, I don't think that the logic in my solution changes dramatically now...", "Solution_20": "I think I would agree but rechecking would be helpful !", "Solution_21": "\n[quote=nkalosidhs][quote=\"orl\"]Let be given positive reals $a$, $b$, $c$. Prove that: $\\frac{a^{3}}{\\left(a+b\\right)^{3}}+\\frac{b^{3}}{\\left(b+c\\right)^{3}}+\\frac{c^{3}}{\\left(c+a\\right)^{3}}\\geq \\frac{3}{8}$.[/quote]\n\nLet denote $A=\\frac{a^3}{(a+b)^3}+\\frac{b^3}{(b+c)^3}+\\frac{c^3}{(c+a)^3}$\nthen we should prove that $A\\geq\\frac{3}{8}$\nWe have by holder's inequality, $A[(a+b)+(b+c)+(c+a)][(a+b)^2+(b+c)^2+(c+a)^2]\\geq(a+b+c)^3$\nSo, $2A(a+b+c)[(a+b)^2+(b+c)^2+(c+a)^2]\\geq(a+b+c)^3$\nwhich means that,\n$2A[(a+b)^2+(b+c)^2+(c+a)^2]\\geq(a+b+c)^2$\nwe multiply both sides with $3(a^2+b^2+c^2)$\nSo we have,\n\n$2A3(a^2+b^2+c^2)[(a+b)^2+(b+c)^2+(c+a)^2]\\geq3(a^2+b^2+c^2)(a+b+c)^2$ $(1)$\nSo due to the fact that, $3(a^2+b^2+c^2)\\geq(a+b+c)^2$\nwhich means that from $(1)$ we get that\n\n$2A[(a+b)^2+(b+c)^2+(c+a)^2]\\geq3(a^2+b^2+c^2)$\n\n[/quote]\n\nI think this is wrong!\n\nHow can you deduce from $(1)$ to\n\n$2A[(a+b)^2+(b+c)^2+(c+a)^2]\\geq3(a^2+b^2+c^2)$?\n ", "Solution_22": "Clearing the denominator, then we use Muirhead inequality(Bunching),\n\n$a^5bc^3+a^3b^5c+ab^3c^5 \\geq a^4b^3c^2+a^2b^4c^3+a^3b^2c^4$\n\nObviously this holds by AM-GM inequality. :)", "Solution_23": "Let $a+b+c=1$, by Rearrangement,\n\n$\\sum_{cyc}\\frac{a^3}{(a+b)^3}\\geq \\sum_{cyc}\\frac{a^3}{(b+c)^3}=\\sum_{cyc}\\frac{a^3}{(1-a)^3}$\n\nWe're going to using Tangent Line method.\n\nLet $f(x)=\\frac{x^3}{(1-x)^3}(0 Y if X and Y have at most 2 different terms. It also showed that for any two sequences that X > Y, we can find X > Z1 > Z2 > ... >Zn > Y where (X,Z1), (Z1,Z2),...,(Zn,Y) differ by at most two terms. This shows that we only need to prove karamata for two variables, which is simple.", "Solution_12": "Yes, I know, but I am unable to read it ;) My solution in other thread can be remade to showing that you can make one vector to another by finite number of compressions (I saw Darij calling it compressions), it's the same what you are mentioning Siuhochung. With these so-called compressions you can also prove Muirhead inequality easily ;)", "Solution_13": "[quote=\"suweijie\"]Now I have found a proof in a book,let me post it here.Maybe my English is poor. :blush: \nSuppose [img]http://www.artofproblemsolving.com/Forum/latexrender/pictures/be86156d54f526b5b670467980553c24.gif[/img]\nthen for arbitrary convex function f the ineq(below) is true.\n[img]http://www.artofproblemsolving.com/Forum/latexrender/pictures/4994a74cc03f1a13857dd38211c94364.gif[/img]\ndefined [img]http://www.artofproblemsolving.com/Forum/latexrender/pictures/ea70490be4bc30dde73e57c84467bde3.gif[/img]\nsince f is convex,hence [img]http://www.artofproblemsolving.com/Forum/latexrender/pictures/f4bbc08bcddb4e0b8282adc7f5b41517.gif[/img]\n\n\n[/quote]\n\nwhy is it true?\n\n\n\n[quote=\"ondrob\"]Yes, I know, but I am unable to read it ;) My solution in other thread can be remade to showing that you can make one vector to another by finite number of compressions (I saw Darij calling it compressions), it's the same what you are mentioning Siuhochung. With these so-called compressions you can also prove Muirhead inequality easily ;)[/quote]\r\n\r\ni'm eager to see the proof of the much used muirhead", "Solution_14": "It's true because of the convexity of functions. It is somehow a property of convex functions. I remember blahblahblah posted a proof somewhere but i cannot find it.\r\n\r\nFor muirhead, Peter Scholze and darij proved it.\r\nhttp://www.mathlinks.ro/Forum/viewtopic.php?p=140365#140365\r\nA generalized version.", "Solution_15": "[quote=\"siuhochung\"]It's true because of the convexity of functions. It is somehow a property of convex functions. I remember blahblahblah posted a proof somewhere but i cannot find it.[/quote]\r\n\r\ni found in kedlaya's notes that we must have $a \\geq b \\geq c \\geq d$ for such an inequality to hold. here i think this is not always the case", "Solution_16": "I wonder why anyone has mentioned: [url=http://www.mathlinks.ro/Forum/topic-41966.html]www.mathlinks.ro/Forum/topic-41966.html[/url] with an inductive proof....as far as I know there are few versions of these majorization inequalities.", "Solution_17": "[quote=\"Rafal\"]I wonder why anyone has mentioned: [url=http://www.mathlinks.ro/Forum/topic-41966.html]www.mathlinks.ro/Forum/topic-41966.html[/url] with an inductive proof....as far as I know there are few versions of these majorization inequalities.[/quote]\r\nThis version is weaker than the known version on the forum (the one mahbub posted).", "Solution_18": "[quote=\"suweijie\"]Now I have found a proof in a book,let me post it here.Maybe my English is poor. :blush: \nSuppose [img]http://www.artofproblemsolving.com/Forum/latexrender/pictures/be86156d54f526b5b670467980553c24.gif[/img]\nthen for arbitrary convex function f the ineq(below) is true.\n[img]http://www.artofproblemsolving.com/Forum/latexrender/pictures/4994a74cc03f1a13857dd38211c94364.gif[/img]\ndefined [img]http://www.artofproblemsolving.com/Forum/latexrender/pictures/ea70490be4bc30dde73e57c84467bde3.gif[/img]\nsince f is convex,hence [img]http://www.artofproblemsolving.com/Forum/latexrender/pictures/f4bbc08bcddb4e0b8282adc7f5b41517.gif[/img]\nthen we have [img]http://www.artofproblemsolving.com/Forum/latexrender/pictures/1e846364bd39b5598116910af06f77f5.gif[/img]\nwhere [img]http://www.artofproblemsolving.com/Forum/latexrender/pictures/2863342235680af68717d091cf076f24.gif[/img]\nhence [img]http://www.artofproblemsolving.com/Forum/latexrender/pictures/3432a6589d995c7ca4933f484d51c0ce.gif[/img]\nso we obtain [img]http://www.artofproblemsolving.com/Forum/latexrender/pictures/6f4fccd694bfd367db11a59cd31fb263.gif[/img]\nsince [img]http://www.artofproblemsolving.com/Forum/latexrender/pictures/c20f21e98dd8c86966e9208f21c4fe13.gif[/img]\nso [img]http://www.artofproblemsolving.com/Forum/latexrender/pictures/163a50a2be565707cb62c3fb273e6547.gif[/img]\nnow we have proved it.Cheers! :D[/quote]\r\n\r\nBAH...\r\n\r\nDo you have the proof stored on your computer?\r\nPlease don't link to LaTeXrender graphics, they are not kept online for long!\r\n\r\n Darij", "Solution_19": "A very straight forward and elegant proof can be found here:\r\n\r\n[url]http://www.imocompendium.com/tekstkut/ineq_im.pdf[/url]\r\n\r\nA lot of goodies in this article and on the website too." } { "Tag": [ "algebra", "polynomial", "induction", "modular arithmetic", "algebra proposed" ], "Problem": "Let $n,\\ k$ be positive integers and let $P(x)$ be polynomial with degree $\\geq n.$ If all the coefficients of the term with degree $\\leq n$ of the polynomial $(1+x)^{k}P(x)$ are integers, then prove that all the coefficients of the term with degree $\\leq n$ of $P(x)$ are integers.Note that you may regard constant term itself as coefficient.", "Solution_1": "The case $k=1$ is easy, and the rest follows trivially by induction on $k$.\r\n\r\nEDIT another solution would be\r\n$(1+x)^{k}P(x)\\equiv Q(x)\\pmod{x^{n+1}}$ for some $Q\\in\\mathbb{Z}[x]$.\r\nThen \r\n$\\begin{array}{rcll}P(x)&\\equiv & (1-x^{2n+2})^{k}P(x)&\\pmod{x^{n+1}}\\\\ &=&(1+x^{2}+\\cdots+x^{2n})^{k}(1-x)^{k}(1+x)^{k}P(x)&\\\\ &\\equiv & (1+x^{2}+\\cdots+x^{2n})^{k}(1-x)^{k}Q(x) & \\pmod{x^{n+1}}\\end{array}$\r\nwith $(1+x^{2}+\\cdots+x^{2n})^{k}(1-x)^{k}Q(x)\\in\\mathbb{Z}[x]$." } { "Tag": [ "inequalities", "trigonometry", "function", "AMC", "USA(J)MO", "USAMO", "inequalities unsolved" ], "Problem": "Prove that in triangle $ABC$ we have the following inequality:\r\n\r\n\r\n$1+\\frac{r}{R}\\leq \\sin\\frac{A}{2}+\\sin\\frac{B}{2}+\\sin\\frac{C}{2}\\leq \\sqrt{2+\\frac{r}{2R}}$", "Solution_1": "i'll prove just the LHS inequality...\r\n\r\nrecall that \r\n$1+\\frac{r}{R}=\\cos A+\\cos B+\\cos C=\\sin (90-A)+\\sin (90-B)+sin(90-C)$\r\n\r\nthen, it's easy to verify that $[90-A,90-B,90-C]$ majorizes $[A/2, B/2, C/2]$, so the result follows from the karamata inequality applied to the function $\\sin x$. :D", "Solution_2": "minutes later i came with some ideas for the RHS inequality\r\n\r\nagain, let's use the identity \r\n\r\n$1+\\frac{r}{R}=\\cos A+\\cos B+\\cos C=2(\\cos^{2}A/2+\\cos^{2}B/2+\\cos^{2}C/2)-3$\r\n\r\nthen,\r\n\r\n$\\sqrt{2+\\frac{r}{2R}}=\\sqrt{\\cos^{2}A/2+\\cos^{2}B/2+\\cos^{2}C/2}$.\r\n\r\nwe could try squaring it, and then the inequality is equivalent to\r\n\r\n$(\\sin A/2+\\sin B/2)^{2}+(\\sin B/2+\\sin C/2)^{2}+(\\sin C/2+\\sin A/2)^{2}\\leq 3$\r\n\r\nthis looks similar to the inequality $\\sum_{cyc}(\\cos A+\\cos B)^{2}\\leq 3$, which turned to be very famous in the forum some time ago and which can be proven to be equivalent with the usamo 2001 inequality...\r\n\r\nhope somebody can continue this :lol:", "Solution_3": "just to finish this out... i didn't wonder how close I was :P \r\n\r\naccording to [url]http://www.mathlinks.ro/Forum/viewtopic.php?t=14567[/url] we have that \r\n\r\n$\\sum_{cyc}(\\cos A+\\cos B)^{2}\\leq 3$, for $A,B,C$ acute angles.\r\n\r\nthen, make the $A,B,C=90-X/2, 90-Y/2, 90-Z/2$, with $X,Y,Z$ angles of a triangle, and then\r\n\r\n$\\sum_{cyc}(\\sin X/2+\\sin Y/2)\\leq 3$, and we're done." } { "Tag": [ "quadratics", "modular arithmetic", "number theory solved", "number theory" ], "Problem": "can anyone show me how to prove the four identities in the end of this page: http://mathworld.wolfram.com/LegendreSymbol.html", "Solution_1": "I guess(3) follows from the following:\r\n\r\n(a/p)=a^((p-1)/2) (mod p)\r\n\r\n[b]Solution:[/b]\r\n\r\nif p|a then a^((p-1)/2)=0=(a/p) (mod p)\r\n\r\nLets consider that not ( p|a) then because of the fact that:\r\n\r\n(a^((p-1)/2)-1)(a^((p-1)/2)+1)=a^(p-1)-1=0(mod p) we get that\r\na^((p-1)/2)= \\pm1 (mod p). \r\nif a is a quadratic residue mod p then we have that:\r\na^((p-1)/2)=1=(a\\p)(mod p)\r\nif it's not a quadratic residue mod p then \r\na^((p-1)/2)<>1(mod p) so \r\na^((p-1)/2)=-1=(a/p) (mod p)\r\n\r\ncheers! :D :D", "Solution_2": "(3) comes if u take in the above a=-1;\r\n\r\n(2) (ab/p)=(a/p)(b/p)\r\ncomes from the relation (a/p)=a^((p-1)/2) (mod p) \r\n\r\nwe have that (a/p)(b/p)=a^((p-1)/2)b^((p-1)/2)=(ab)^((p-1)/2)=\r\n(ab/p)(mod p) so\r\n(a/p)(b/p)=(ab/p) (mod p)\r\nbut because the only values of (a/p);(b/p);(ab/p) can only be 0 or \\pm1 and p>=3 then we have the relation above! :D \r\n\r\ncheers!", "Solution_3": "(4) \r\n\r\n(2/p)=(-1)^((p^2-1)/8)\r\n\r\nLet e be a root of order 8. so that e^8=1 and e^4<>1\r\nso we can take e=cos(2pi/8)+isin(2pi/8)=sqrt(2)/2(1+i)\r\nwe have that e^4=-1 and e^2+e^(-2)=0.\r\nlets denote t=e+e^(-1) then t^2=2.\r\nwe can see that e,t are algebric integers and we shall use congruence in the algebric integer rings omega.\r\nwe have that: t^(p-1)=(t^2)^((p-1)/2)=2^((p-1)/2)=(2/p)(mod p)\r\nso: t^p=(2/p)*t(mod p)\r\nand also:\r\nt^p=e^p+e^(-p)(modp)\r\nbecause e^8=1 we have for p= \\pm1(mod 8); e^p+e^(-p)=e+e^(-1)\r\nand for p= \\pm3(mod p) we have e^p+e^(-p)=-(e+e^(-1)).\r\nso \r\ne^p+e^(-p)= \r\nt if p= \\pm1 (mod 8 )\r\n-t if p= \\pm3 (mod 8 )\r\nwhich means e^p+e^(-p)=(-1)^(eps(p))*t\r\nwhere:\r\neps(p)=\r\n1 if p= \\pm1 (mod 8 )\r\n-1 if p= \\pm3 (mod 8 )\r\n\r\nso we get that \r\n(2/p)*t=t^p=e^p+e^(-p)=(-1)^(eps(p))*t(mod p)\r\nnow in the last relation we lumtiply it with t and we know that t^2=2 we get that:\r\n(2/p)*2=(-1)^(eps(p))*2(mod p)\r\nso (2/p)=(-1)^(eps(p))\r\nnow it's easy to see that eps(p)=(p^2-1)/8 (mod 2)\r\nso we got the indetity!!!\r\n\r\ncheers! :D :D", "Solution_4": "Nice proof for (4), Lagrangia. I have sen this before in a book somewhere. You can also find a proof here:\r\n\r\n[url]http://www.mathlinks.ro/phpBB/viewtopic.php?t=2154[/url]\r\n\r\nIt's somewhat messier than what Lagrangia posted." } { "Tag": [ "analytic geometry", "function", "real analysis", "real analysis unsolved" ], "Problem": "f:[0,1]->R^n is a continuous path of finite length. (by the way, what is a path? Each coordinate is a function of t, t in [0,1]?). Show that if n>1 then the n-dimentional Leb measure of the path is zero. (what about n=1, still measure zero?) Thanks.", "Solution_1": "Using your definition of path and assuming $n>1$:\r\n\r\nFor each $\\epsilon>0$ you can find a partition of $[0,1]$ by points $t_{i}$ such that the length of the arc between $f(t_{i-1})$ and $f(t_{i-1})$ is less than $\\epsilon$ for all $i$. The balls of radius $|f(t_{i})-f(t_{i-1})|$ centered at $f(t_{i})$ cover the path. Since $\\sum_{i}|f(t_{i})-f(t_{i-1})|\\le L$ ($L$ is the length), we have $\\sum_{i}|f(t_{i})-f(t_{i-1})|^{n}\\le L\\epsilon^{n-1}$, which is good." } { "Tag": [], "Problem": "The United States Mint produced 13.5 billion pennies in 1995. If the value of these pennies is exchanged for the same amount in dimes, how many dimes would be needed?", "Solution_1": "There are 13.5 billion pennies.\r\n\r\n10 pennies equals 1 dime.\r\n\r\nSo divide 13.5 by 10 to get the answer.\r\n\r\nThe answer is 1.35 billion dimes." } { "Tag": [ "inequalities", "function", "inequalities unsolved" ], "Problem": "Let $ a,b,c>0 $ with $ a^3+b^3+c^3=2$\r\nProve that:\r\n\r\n\\[ \\frac{a^2}{b^2+bc+c^2} + \\frac{b^2}{c^2+ca+a^2} + \\frac{c^2}{a^2+ab+b^2}\\geq\\frac{3}{1+3abc} \\]", "Solution_1": "[b]This is not true![/b]\r\n\r\nLet $a=b$ and $c$ goes to 0, then both $a,b$ go to 1. So LHS goes to 2 and RHS goes to 3.", "Solution_2": "Maverick, I don't think your inequality is true. For if it would be true, then we could \"de-homogenize\" it and obtain the inequality\r\n\r\n[tex]\\frac{a^{2}}{b^{2}+bc+c^{2}}+\\frac{b^{2}}{c^{2}+ca+a^{2}}+\\frac{c^{2}}{a^{2}+ab+b^{2}}\\geq \\frac{3\\left( a^{3}+b^{3}+c^{3}\\right) /2}{\\left( a^{3}+b^{3}+c^{3}\\right) /2+3abc}[/tex]\r\n\r\nfor all positive numbers a, b, c.\r\n\r\nBut taking a = 1, b = 1 and $c=\\sqrt{2}-1$, we get\r\n\r\n[tex]\\frac{a^{2}}{b^{2}+bc+c^{2}}+\\frac{b^{2}}{c^{2}+ca+a^{2}}+\\frac{c^{2}}{a^{2}+ab+b^{2}}-\\frac{3\\left( a^{3}+b^{3}+c^{3}\\right) /2}{\\left( a^{3}+b^{3}+c^{3}\\right) /2+3abc}\\approx -4.\\,5\\cdot 10^{-2}<0[/tex].\r\n\r\nOn the other hand, the inequality with reversed sign is not true either, since a = 1, b = 1 and c = 2 gives\r\n\r\n[tex]\\frac{a^{2}}{b^{2}+bc+c^{2}}+\\frac{b^{2}}{c^{2}+ca+a^{2}}+\\frac{c^{2}}{a^{2}+ab+b^{2}}-\\frac{3\\left( a^{3}+b^{3}+c^{3}\\right) /2}{\\left( a^{3}+b^{3}+c^{3}\\right) /2+3abc}\\approx 0.255>0[/tex].\r\n\r\nWhat, I think, is valid would be the inequality\r\n\r\n[tex]\\frac{a^{2}}{b^{2}+bc+c^{2}}+\\frac{b^{2}}{c^{2}+ca+a^{2}}+\\frac{c^{2}}{a^{2}+ab+b^{2}}\\geq \\frac{\\left( a^{3}+b^{3}+c^{3}\\right) /2}{\\left( a^{3}+b^{3}+c^{3}\\right) /2+3abc}+\\frac{2}{3}[/tex].\r\n\r\nBut I have no idea how to prove it.\r\n\r\n Darij", "Solution_3": "[quote=\"darij grinberg\"]What, I think, is valid would be the inequality\n\n[tex]\\frac{a^{2}}{b^{2}+bc+c^{2}}+\\frac{b^{2}}{c^{2}+ca+a^{2}}+\\frac{c^{2}}{a^{2}+ab+b^{2}}\\geq \\frac{\\left( a^{3}+b^{3}+c^{3}\\right) /2}{\\left( a^{3}+b^{3}+c^{3}\\right) /2+3abc}+\\frac{2}{3}[/tex].\n\nBut I have no idea how to prove it.\n\n Darij[/quote]\r\n\r\nPost it in the [b]Open Questions[/b] section, see whether somebody can prove or disprove it!", "Solution_4": "[quote=\"fuzzylogic\"]Post it in the [b]Open Questions[/b] section, see whether somebody can prove or disprove it![/quote]\r\n\r\nWell, I have not spent any time with trying to prove it - I have not much time now -, so it may be very easy and I don't want to spam the Open Questions section with such problems...\r\n\r\n Darij", "Solution_5": "Darij, your examples are wrong: $(1,1,\\sqrt{2}-1)\\,\\ (1,1,2)$ do not satisfy $a^3+b^3+c^3=2$ and also fuzzylogic $c$ cannot be 0.\r\nI'm not sure if the problem is wrong or not, but here is the way I thought it:\r\n\r\n$\\sum{\\frac{a^2}{b^2+bc+c^2}}= \\sum{\\frac{a^3}{ab^2+abc+ac^2}}\\\\\r\n\\frac{a^3}{ab^2+abc+ac^2}+\\frac{b^3}{bc^2+bca+ba^2}+\\frac{c^3}{ca^2+cab+cb^2}\\geq \\frac{a^3+b^3+c^3}{3} \\cdot \\sum{\\frac{1}{ab^2+abc+ac^2}}\\\\\r\n\\frac{2}{3} \\cdot \\sum{\\frac{1}{ab^2+abc+ac^2}}\\geq \\frac{6}{\\sum{ab(a+b)}+3abc}\\geq \\frac{6}{a^3+b^3+c^3+6abc}=\\\\\r\n\\frac{6}{2+6abc}=\\frac{3}{1+3abc}$", "Solution_6": "Fuzzylogic is right.\r\nFirst inequality written by Maverick in his last message is wrong, since\r\nfor [i]a[/i] goes to 0 obtain $\\displaystyle finite\\geq \\infty$.", "Solution_7": "[quote=\"Maverick\"]Darij, your examples are wrong: $(1,1,\\sqrt{2}-1)\\,\\ (1,1,2)$ do not satisfy $a^3+b^3+c^3=2$[/quote]\n\nMaverick, you probably skipped the first line of my message. I wrote that I \"de-homogenized the inequality\". If you really want to have a counterexample with $a^3+b^3+c^3=2$, then take $\\displaystyle a=\\root{3}\\of{\\frac{2}{1^{3}+1^{3}+\\left( \\sqrt{2}-1\\right) ^{3}}}\\cdot 1$, $\\displaystyle b=\\root{3}\\of{\\frac{2}{1^{3}+1^{3}+\\left( \\sqrt{2}-1\\right) ^{3}}}\\cdot 1$ and $\\displaystyle c=\\root{3}\\of{\\frac{2}{1^{3}+1^{3}+\\left( \\sqrt{2}-1\\right) ^{3}}}\\cdot \\left( \\sqrt{2}-1\\right) $, then you have $a^{3}+b^{3}+c^{3}=2$ but still\n\n[tex]\\frac{a^{2}}{b^{2}+bc+c^{2}}+\\frac{b^{2}}{c^{2}+ca+a^{2}}+\\frac{c^{2}}{a^{2}+ab+b^{2}}-\\frac{3}{1+3abc}\\approx -4.5\\cdot 10^{-2}<0[/tex].\n\n[quote=\"Maverick\"]and also fuzzylogic $c$ cannot be 0.[/quote]\n\nBut it can get arbitrarily close to 0 and the function is continuous, so...\n\n[quote=\"Maverick\"]$\\frac{a^3}{ab^2+abc+ac^2}+\\frac{b^3}{bc^2+bca+ba^2}+\\frac{c^3}{ca^2+cab+cb^2}\\geq \\frac{a^3+b^3+c^3}{3} \\cdot \\sum{\\frac{1}{ab^2+abc+ac^2}}$[/quote]\r\n\r\nIf you applied Chebyshev here, then it's wrong: The number arrays $\\left( a^{3},\\;b^{3},\\;c^{3}\\right) $ and $\\displaystyle \\left( \\frac{1}{ab^{2}+abc+ac^{2}},\\;\\frac{1}{bc^{2}+bca+ba^{2}},\\;\\frac{1}{ca^{2}+cab+cb^{2}}\\right) $ are (in general) not equally sorted.\r\n\r\n Darij", "Solution_8": "yes you are right. Sorry!!" } { "Tag": [ "integration", "calculus", "derivative", "probability", "algebra", "polynomial", "conditional probability" ], "Problem": "a funny question given to a friend during an interview for a job:\r\n\r\nA man picks a number $ x$ (uniformly) in $ [0;1]$, and writes it down on a pice of paper. On another piece of paper, he writes $ x^2$. Then you enter the room and choose one of the two pieces of paper. You have the choice between: \r\n1/ keep this piece of paper, and the man will give you $ y$ dollars ($ y$ is what is written on the piece of paper you chose).\r\n2/ decide to take the other piece of paper, and the man will give you what is written on the other paper.\r\n\r\nWhat is your strategy ?", "Solution_1": "[hide=\"Something I do not believe any more.\"]I get the following optimal strategy: examine the number on the first slip. If it is greater than $ \\frac14,$ keep it. If it is less than $ \\frac14,$ switch to the other slip.[/hide]\n\nMy method is probably unnecessarily complicated; there's probably an easier argument.\n\nLet $ X$ be the original number that is uniform on $ [0,1].$ Let $ Y$ be the number written on the slip you choose first. Let $ L$ be the event that $ Y \\equal{} X.$\n\n$ P(Y\\in [y,y \\plus{} \\Delta y]\\text{ and }L) \\equal{} P(X\\in [y,y \\plus{} \\Delta y]\\text{ and }L) \\equal{} \\frac {\\Delta y}2.$\n\n$ P(Y\\in [y,y \\plus{} \\Delta y]\\text{ and }L^C) \\equal{} P(X\\in \\left[\\sqrt {y},\\sqrt {y \\plus{} \\Delta y}\\right]\\text{ and }L^C)\\approx \\frac {\\Delta y}{4\\sqrt {y}}.$\n\nFrom that I get that \n\n$ P(L\\mid Y \\equal{} y) \\equal{} \\frac {\\frac12}{\\frac12 \\plus{} \\frac1{4\\sqrt {y}}} \\equal{} \\frac {2\\sqrt {y}}{1 \\plus{} 2\\sqrt {y}}.$\n\nNow, let us explore a strategy of switching for $ Y < b$ and keeping for $ Y > b$ for some $ b$ to be determined. If you switch, you get $ Y^2$ if $ L$ and $ Y$ if not $ L.$ If you keep, the payoffs reverse. I get that the expected payoff for this strategy is\n\n[hide=\"An argument not to trust.\"]$ g(b) \\equal{} \\int_0^b\\frac {y^22\\sqrt {y}}{1 \\plus{} 2\\sqrt {y}} \\plus{} \\frac {y}{1 \\plus{} 2\\sqrt {y}}\\,dy \\plus{} \\int_b^1\\frac {y^2\\sqrt {y}}{1 \\plus{} 2\\sqrt {y}} \\plus{} \\frac {y^2}{1 \\plus{} 2\\sqrt {y}}\\,dy$\n\nWe can differentiate this by the fundamental theorem of calculus. The derivative simplifies to\n\n$ g'(b) \\equal{} \\frac {(b \\minus{} b^2)(1 \\minus{} 2\\sqrt {b})}{1 \\plus{} 2\\sqrt {b}}.$\n\nFrom this we see that we have a maximum at $ b \\equal{} \\frac14,$ hence my answer.[/hide]", "Solution_2": "I just hid some parts of the last post because I realized that they depended on the assumption that $ Y$ is uniformly distributed - and it's not.\r\n\r\nThe probability of $ L,$ which is $ \\frac12,$ should be the expectation of the conditional probability of $ L.$ But\r\n\r\n$ \\int_0^1\\frac{2\\sqrt{y}}{1\\plus{}2\\sqrt{y}}\\,dy\\equal{}\\frac12\\ln3,$ which is not $ \\frac12.$\r\n\r\nSome parts of the work I did might have uses, but otherwise, it's time to try again.", "Solution_3": "There's a borderline number $ b$ which is approximately $ 0.31944846.$\r\n\r\nI get the following optimal strategy: examine the number on the first slip. If it is greater than $ b,$ keep it. If it is less than $ b,$ switch to the other slip.\r\n\r\nLet $ X$ be the original number that is uniform on $ [0,1].$ Let $ Y$ be the number written on the slip you choose first. Let $ L$ be the event that $ Y = X.$\r\n\r\n$ P(Y\\in [y,y + \\Delta y]\\text{ and }L) = P(X\\in [y,y + \\Delta y]\\text{ and }L) = \\frac {\\Delta y}2.$\r\n\r\n$ P(Y\\in [y,y + \\Delta y]\\text{ and }L^C) = P(X\\in \\left[\\sqrt {y},\\sqrt {y + \\Delta y}\\right]\\text{ and }L^C)\\approx \\frac {\\Delta y}{4\\sqrt {y}}.$\r\n\r\nFrom that I get that \r\n\r\n$ P(L\\mid Y = y) = \\frac {\\frac12}{\\frac12 + \\frac1{4\\sqrt {y}}} = \\frac {2\\sqrt {y}}{1 + 2\\sqrt {y}}.$\r\n\r\nNote also that the probability density of $ y$ is simply $ \\frac12 + \\frac1{4\\sqrt {y}}.$ As a consequence, we get that\r\n\r\n$ P(L\\mid Y = y)\\cdot f_Y(y) = \\frac12$ and $ P(L^C\\mid Y = y)\\cdot f_Y(y) = \\frac1{4\\sqrt {y}}.$\r\n\r\nNow, let us explore a strategy of switching for $ Y < b$ and keeping for $ Y > b$ for some $ b$ to be determined. If you switch, you get $ Y^2$ if $ L$ and $ \\sqrt{Y}$} if not $ L.$ If you keep, you get $ Y.$ I get that the expected payoff for this strategy is\r\n\r\n$ g(b) = \\int_0^by^2\\cdot \\frac12 + \\sqrt{y}\\cdot\\frac1{4\\sqrt {y}}\\,dy + \\int_b^1y\\left(\\frac12 + \\frac1{4\\sqrt {y}}\\right)\\,dy$\r\n\r\nWe can differentiate this by the fundamental theorem of calculus. The derivative becomes\r\n\r\n$ g'(b) = \\frac{b^2}2+\\frac14-\\frac{b}2-\\frac{\\sqrt{b}}{4}.$\r\n\r\nSetting this equal to zero results in a quartic polynomial equation for $ \\sqrt{b}.$ $ b=1$ is one root; factoring this out gives a cubic equation. The number given at the top is a numerical approximation to the root of this equation.", "Solution_4": "[url=http://www.artofproblemsolving.com/Forum/viewtopic.php?t=147596]Two[/url] [url=http://www.artofproblemsolving.com/Forum/viewtopic.php?t=32063]threads[/url] on a related \"paradox\".", "Solution_5": "another way to write the same proof as Kent Merryfield:\r\n\r\nLet $ X$ and $ \\epsilon$ be two independant random variables and suppose that $ X$ is uniformly distributed in $ [0;1]$, and that $ \\epsilon$ is uniformly distributed in $ \\{0,1\\}$. One can modelize the number written on the slip you choose first by $ Y_1 \\equal{} \\epsilon X \\plus{} (1\\minus{}\\epsilon) X^2$, and the second number by $ Y_2 \\equal{} (1\\minus{}\\epsilon) X \\plus{} \\epsilon X^2$. Then the good strategy is to choose the second number when $ E[Y2 | Y1] \\geq Y1$. But standart calculations show that $ E[Y2 | Y1] \\equal{} \\frac{Y_1^2 \\plus{} \\frac{1}{2}}{1 \\plus{} \\frac{1}{2 \\sqrt{Y1}}}$. Hence one has to find the interval where $ \\frac{x^2 \\plus{} \\frac{1}{2}}{1 \\plus{} \\frac{1}{2 \\sqrt{x}}} \\geq x$." } { "Tag": [ "function", "integration", "limit", "real analysis", "calculus", "derivative", "search" ], "Problem": "Let $ \\phi (x) , x\\in\\mathbb{R}^n$ be a bounded measurable function, such that $ \\phi (x) \\equal{} 0$ for $ \\parallel{}x\\parallel{}\\geq 1$, and $ \\int \\phi \\equal{} 1$. For $ \\epsilon > 0$ let $ \\phi _{\\epsilon}(x): \\equal{} \\epsilon ^{ \\minus{} n}\\phi (\\frac {x}{\\epsilon})$ (approximation to the identity) . \r\nIf $ f\\in L(\\mathbb{R}^n)$ show that $ \\lim_{\\epsilon \\rightarrow 0}(f\\ast \\phi_{\\epsilon})(x) \\equal{} f(x)$ in the Lebesgue set of $ f$.", "Solution_1": "Here's a start. Note that $ \\int\\phi_{\\epsilon} \\equal{} 1$ and that $ |\\phi_{\\epsilon}(x)|\\le\\epsilon^{ \\minus{} n}M.$\r\n\r\n$ (f*\\phi_{\\epsilon})(x) \\minus{} f(x) \\equal{} \\int_{\\mathbb{R}^n}\\phi_{\\epsilon}(x \\minus{} y)f(y)\\,dy \\minus{} \\int_{\\mathbb{R}^n}\\phi_{\\epsilon}(x \\minus{} y)f(x)\\,dy$\r\n\r\n$ |(f*\\phi_{\\epsilon})(x) \\minus{} f(x)|\\le\\int_{B(x;\\epsilon)} |\\phi_{\\epsilon}(x \\minus{} y)|\\,|f(y) \\minus{} f(x)|\\,dy$\r\n\r\n$ \\le \\epsilon^{ \\minus{} n}M\\int_{B(x;\\epsilon)}|f(y) \\minus{} f(x)|\\,dy$\r\n\r\nCan you finish it from there?\r\n\r\nNext up: weakening the hypotheses on $ \\phi$ from bounded and supported on a ball to majorized by a decreasing radial integrable function.", "Solution_2": "Thanks, I have two question: why is this function ($ \\phi_{\\epsilon}$) called \"approximation to the identity\" (I just realized I wrote it wrong the last post :blush: )? My opinion is,the problem is put there out of context ( it's an intro to harmonic analysis book). \r\nNext one, what do you mean by [b]radial[/b]? I'm lost on terminology.", "Solution_3": "\"radial\" means \"radially symmetric- the value depends only on the distance from the origin.\r\nAs for why this is an \"approximation to the identity\"- functions form a ring, with convolution as the multiplication. This sequence converges weakly to the multiplicative identity in that ring, although we need to extend to distributions for that identity ($ \\delta$) to actually exist.", "Solution_4": "For a textbook or course that studies Euclidean harmonic analysis (that is, harmonic analysis on $ \\mathbb{R},\\mathbb{T},\\mathbb{Z},\\mathbb{Z}/N\\mathbb{Z}$ and cartesian products of those) and which assumes a knowledge of the Lebesgue integral, this exercise is absolutely and totally [b]in[/b] context. This result and its relatives are things that any student of the subject should be instantly and intensely familiar with.", "Solution_5": "This problem is not out of Harmonic Analysis. It's wellknown that there's connection between results on maximal functions and results on differentiability and pointwise convergence, say [url=http://planetmath.org/encyclopedia/LebesgueDifferentiationTheorem.html]Lebesgue's Differentiation theorem[/url]. As you see the first step in hints which Kent Merryfield showed, you could finish it by using this theorem. The pattern of the above comments were reproduced to obtain other differentiability results, from estimates on maximal functions. This motivates the search for extensions of the orginal [url=http://planetmath.org/encyclopedia/HardyLittlewoodMaximalTheorem.html]maximal theorem of Hardy Littlewood[/url], namely $ f\\to \\sup_{\\epsilon}|\\phi_{\\epsilon}*f(x)|$ is bounded in $ L^p$ for $ 1 < p < \\infty$.", "Solution_6": "Kent Merryfield, would you please tell me something about Harmonic Analysis on $ {\\mathbb Z}/\\mathbb NZ$ ?", "Solution_7": "Let $ \\omega\\equal{}e^{2\\pi i/N}$ - a primitive $ N$th root of unity. Suppose $ f$ is a complex-valued function defined on $ \\mathbb{Z}_N$ (another notation for $ \\mathbb{Z}/N\\mathbb{Z}$). Then it has a Fourier transform $ \\widehat{f},$ also a function on $ \\mathbb{Z}/N,$ defined by\r\n\\[ \\widehat{f}(k)\\equal{}\\frac1N\\sum_{n\\equal{}0}^{N\\minus{}1}\\omega^{\\minus{}kn}f(n)\\]\r\nfrom which we can recover $ f$ by\r\n\\[ f(n)\\equal{}\\sum_{k\\equal{}0}^{N\\minus{}1}\\omega^{kn}\\widehat{f}(k).\\]\r\n(There are other possible conventions for handing the factor of $ N.$)\r\n\r\nAlternately, this can be seen as a matrix operation. In all of our vectors and matrices, let the indices run from $ 0$ to $ N\\minus{}1.$ Consider the $ N\\times N$ matrix $ F$ whose $ (k,n)$th entry is $ \\omega^{kn}.$ Then $ \\widehat{f}\\equal{}\\frac1NF^*f,$ and $ f\\equal{}F\\widehat{f}.$ Here, by $ F^*,$ we mean the Hermitian adjoint, or conjugate transpose. $ F$ is within a constant of being a unitary matrix: $ F^*F\\equal{}NI.$\r\n\r\nWe can now build all of the usual structures of harmonic analysis: convolution (with the idea that convolution on one side is pointwise multiplication on the other), norms and norm inequalities, including a 2-norm equality (a version of Plancherel's Theorem), diagonalization of translation-invariant linear operators, and so on. We don't need much in the way of heavy theorems of analysis, since for the most part we are working with finite sums.\r\n\r\nThe four domains $ \\mathbb{R},\\mathbb{T},\\mathbb{Z},$ and $ \\mathbb{Z}_N$ can be related to each other. Start with a function $ f$ on $ \\mathbb{R}.$ We can create a function on $ \\mathbb{Z}$ by periodically sampling it: $ \\varphi(n)\\equal{}f(nh)$ for some $ h>0.$ Or we can create a function on $ \\mathbb{T}$ by summing it in a particular way: $ g(x)\\equal{}\\sum_{m\\equal{}\\minus{}\\infty}^{\\infty}f(x\\plus{}mP),$ where $ P$ is the period. Sample and sum both (in either order) and you get a function on $ \\mathbb{Z}_N.$ These sampling and summing operations have predictable and useful consequences for the Fourier transforms - this is called the Poisson summation formula, along with a number of related theorems.\r\n\r\nThe operation that takes $ f$ to $ \\widehat{f}$ over $ \\mathbb{Z}_N$ is called the [i]discrete Fourier transform[/i] (DFT). By the raw definition of matrix multiplication, it requires $ O(N^2)$ arithmetic operations to execute. However, if $ N$ is a highly composite integer (nothing but small factors), this can be speeded up to $ O(N\\log N)$ arithmetic operations. Such an algorithm is a [i]fast Fourier transform[/i] (FFT).", "Solution_8": "[quote=\"pluricomplex\"]Kent Merryfield, would you please tell me something about Harmonic Analysis on $ {\\mathbb Z}/\\mathbb NZ$ ?[/quote]\r\n\r\nI refer you to the book entitled \"An Introduction to Harmonic Analysis (Cambridge Mathematical Library) \" by Katznelson.", "Solution_9": "I don't recall that much about the finite case in Katznelson. But, returning to Mr. Doe's original question, Katznelson would be an excellent textbook or resource for the course of study that his question suggests. For something with a little more emphasis on the $ \\mathbb{R}^n$ case, also consider the Stein and Weiss book (from the Princeton series).", "Solution_10": "I don't see much speacial DFT appear in book of Katznelson but FT on $ \\mathbb T,R$ or on LCA. \r\nThanks Kent for excellent post. :)" } { "Tag": [ "USAMTS" ], "Problem": "I have written my USAMTS solutions in pencil. Is pencil fine, or do we have to use pen?", "Solution_1": "Pencil is fine, but I'd strongly suggest mailing it instead of faxing it (pencil doesn't fax too well). And I'd suggest photocopying it and keeping a copy for yourself.", "Solution_2": "thanks for your suggestion-I'll do that" } { "Tag": [ "real analysis", "real analysis unsolved" ], "Problem": "Hello! :) \r\nLet $\\mu: 2^N\\to [0,+\\infty]$ and $\\mu(E)=card E$. Let $f: N\\to R$. Proof that $f$ is $\\mu$ - integrable if and only if series $\\sum_{n=1}^{+\\infty}f(n)$ is absolutely summable. Thanks.", "Solution_1": "isn't it straightfoward ? what's the definition of $\\mu$-integrable .." } { "Tag": [ "algebra unsolved", "algebra" ], "Problem": "What's a good book for learning amc/aime algebra?", "Solution_1": "Any AOPS book. This question is in the wrong forum by the way." } { "Tag": [], "Problem": "Which natural numbers $t$ can be represented as \\[t = \\sum^n_{k=1} x_k\\] where $ x_k \\in \\mathbb{N}_0$ with fixed $n,$ $n = 4 \\cdot z, z \\in \\mathbb{N}$ and $x_i \\cdot x_{i+1} = x_{i+2} \\cdot x_{i+3}$ where $i = 4 \\cdot q + 1, q \\in \\mathbb{N}_0$ and $i = 1, \\ldots, n-3.$", "Solution_1": "Ugh? :? What a complex wording! Did I understand it correct if I'm rewriting it like this? :\r\n\r\n$\\displaystyle t = \\sum^{4n}_{k=1}x_k \\ (x_k \\in \\mathbb{N}_0)$\r\n\r\nand $\\displaystyle x_{4m+1}\\cdot x_{4m+2} = x_{4m+3} \\cdot x_{4m+4}$ for $m3$, the sum of squares of all residues is $ 0 \\mod p$ by http://www.mathlinks.ro/Forum/viewtopic.php?t=40171 . Here we counted every quadratic residue twice, thus the sum of all quadratic residues is $ 0 \\mod p$.", "Solution_2": "Oh, I know that the sum is \u2261 0 mod p. I want to know the sum as a number ( in Z not in Zp)", "Solution_3": "For $ p \\equiv 1 \\mod 4$ it's not hard:\r\n$ x$ is a quadratic residue iff $ p \\minus{} x$ is one. Thus we have $ \\frac {p \\minus{} 1}4$ such pairs of quadratic residues, and every pair sums to $ p$. Thus the sum is $ \\frac {p(p \\minus{} 1)}4$.\r\nFor $ p \\equiv \\minus{} 1 \\mod 4$, it's more difficult:\r\n\r\nLet in general be $ s(p)$ be the desired sum. Then $ s(p) \\equal{} \\sum_{i \\equal{} 1}^{p \\minus{} 1} \\frac 12 (\\left( \\frac ip \\right) \\plus{} 1)i$, where $ \\left( \\frac ip \\right)$ is the Legendre-symbol (it is $ 1$ for quadratic residues and $ \\minus{} 1$ for nonresidues). So in fact $ s(p) \\equal{} \\frac 12 \\cdot \\left( \\sum_{i \\equal{} 1}^{p \\minus{} 1} \\left( \\frac ip \\right) i \\plus{} \\sum_{i \\equal{} 1}^{p \\minus{} 1} i \\right) \\equal{} \\frac {p(p \\minus{} 1)}4 \\plus{} \\frac 12 \\cdot \\sum_{i \\equal{} 1}^{p \\minus{} 1} \\left( \\frac ip \\right) i$.\r\nSo we are interested in $ h(p) \\equal{} \\sum_{i \\equal{} 1}^{p \\minus{} 1} \\left( \\frac ip \\right) i$.\r\nFor $ p \\equiv 1 \\mod 4$, this is well known to be $ 0$, giving the result from above. But if $ p \\equiv \\minus{} 1 \\mod 4$ and $ p \\neq 3$, this is known to be $ \\minus{} p$ times the class number $ h_{ \\minus{} p}$ of $ \\mathbb Q [\\sqrt { \\minus{} p}]$. Thus we have $ s(p) \\equal{} p \\cdot \\left( \\frac {p \\minus{} 1}4 \\minus{} \\frac {h_{ \\minus{} p}}2 \\right)$.", "Solution_4": "ZetaX :\r\nI think that this sum is p(p-3)/4 when p\u22613 (mod 4). It is posisble?", "Solution_5": "That would mean $ h_{ \\minus{} p} \\equal{} 1$, but this is only true for $ p \\equal{} 3, 7, 11, 19, 43, 67, 163$. In fact, Gau\u00df' class number conjecture (which is proven) says that for every $ n$ there are only finitely many primes $ p$ such that $ h_{ \\minus{} p} \\equal{} n$.", "Solution_6": "My mistake, sorry\r\n\r\nInitialy I try to solve this problem:\r\n(American Mathematical Monthly, 1999) Let p be a prime number with p \u0011\u22617 (mod 8), and let Lp =\r\n{1, 2, . . . , (p \u2212 1)/2}. Prove that the sum of the quadratic residues (i.e. the squares) modulo p in Lp\r\nequals the sum of the quadratic nonresidues modulo p in Lp. (The sums are taken in Z)\r\n\r\nAnd I make a mistake that make necessay that the sum of the quadratics was p(p-3)/4 \r\njavascript:emoticon(':blush:')\r\nBlush\r\nHowever.\r\nYou can help me to solve that (the AMM)?" } { "Tag": [ "Olimpiada de matematicas" ], "Problem": "todos sabemos que las lineas que unen los vertices respectivos de 2 triangulos homot\u00e9ticos concurren en el centro de homotcia. pero.. :\r\n\r\n como lo pruebo si no supiera nada de homotecia utilizando ceva??", "Solution_1": "Bueno por lo que a mi respecta :\r\n\r\nSe llama homotecia de centro O y raz\u00f3n k (distinto de cero) a la transformaci\u00f3n geometrica que hace corresponder a un punto A otro A\u00b4, alineado con A y O, tal que: OA\u00b4=k\u00b7OA. Si k>0 se llama homotecia directa y si k<0 se llama homotecia inversa. ( o sea no habria nada que probar ) \r\n\r\nAhora la pregunta seria la siguiente :\r\n\r\nDados dos triangulo semejantes ABC y A'B'C' y ademas se cumple : AB // A'B' ; BC // B'C' y CA // C'A' . Probar que : AA', BB' , CC' son concurrentes .\r\n\r\n\r\nBueno eso creo que seria asi :\r\n\r\nsea O el punto de interseccion de AA' y BB' luego como los triangulo ABC y A'B'C' son semejantes entonces : AB/A'B' = BC/B'C' = CA/C'A' = k . entonces por semejanza de ABO con A'B'O se cumple que : OB/OB' = k y el angulo entre OB y BC es congruente angulo entre OB'/B'C' o sea los triangulos OBC y OB'C' son semejantes . con ello los angulos BOC y B'OC' son congruentes y esto garantiza que C , O y C' sean colineales .\r\n\r\n\r\nNota : la condicion de que AB // A'B' ; BC // B'C' y CA // C'A' implica que los triangulo ABC y A'B'C' sean semejantes . asi que se podria replantear la situacion .", "Solution_2": "Bueno .. \r\npodemos encontrar algo de informacion adiconal sobre [color=darkblue]CENTRO DE HOMOTECIA[/color]....\r\naqui pongo un link desde http://mathworld.wolfram\r\n\r\n[url=http://mathworld.wolfram.com/HomotheticCenter.html]Homotecia - english version![/url]\r\n\r\ny su version traducida:\r\n\r\n[url=http://translate.google.com/translate?u=http%3A%2F%2Fmathworld.wolfram.com%2FHomotheticCenter.html&langpair=en%7Ces&hl=es&ie=UTF-8&oe=UTF-8&prev=%2Flanguage_tools]Centro de Homotecia (espa\u00f1ol)[/url]\r\n\r\nCarlos Bravo :)\r\nLima -PERU" } { "Tag": [ "topology" ], "Problem": "Suppose $ X$ is a metric space such that every open cover has a finite subcover. I'm proving that this implies that every sequence has a convergent subsequence.\r\n\r\nSo take a sequence $ x_n$ of points in $ X$. Assume that it has no convergent subsequence. Thus for every $ x \\in X$, we choose an open ball $ U_x$ such that $ U_x$ contains finitely many terms of the sequence. But we can cover $ X$ with finitely many of these balls, and since each contains only finitely many terms of the sequence, it follows that the sequence has finitely many terms, a contradiction.\r\n\r\nYet this proof appears to work for any topological space, which I believe is not true (just replace $ U_x$ with an arbitrary open neighborhood around $ x$ containing finitely many terms of the sequence). Where did I go wrong?", "Solution_1": "In a space with the right separation properties, i.e. one that lets you pick $ U_x$ to be small enough, this result should still hold. I'm not very clear on the details, though, so I couldn't tell you exactly which separation axiom(s) you need. (For example, the argument fails if there exists a point $ x$ such that the smallest open set containing $ x$ is the whole space.)\r\n\r\nEdit: See [url=http://en.wikipedia.org/wiki/Compact_space#Other_forms_of_compactness]the Wiki article[/url].", "Solution_2": "I actually believe I see the issue. It doesn't always hold that if every open set around a point contains infinitely many terms of the sequence, then there exists a convergent subsequence. For a metric space, we could consider balls of radius $ 2^{\\minus{}n}$ and arbitrarily choose an element of the sequence there. In fact, this argument can be applied to all first countable spaces (and I was previously aware that the implication is supposed to hold for first countable spaces)." } { "Tag": [ "algebra", "polynomial" ], "Problem": "Show that $ x^7\\minus{}2x^5\\plus{}10x^2\\minus{}1$ has no roots greater than 1.\r\n\r\nI have tried making the substitution $ x\\equal{}y\\plus{}1$ and showing that the new equation has no positive roots but don't seem to be getting anywhere.\r\nAny suggestions?", "Solution_1": "Just a thought\r\n\r\n[hide]$ f(x)\\equal{}x^7 \\minus{} 2x^5 \\plus{} 10x^2 \\minus{} 1$\n$ f(1)\\equal{}8$\n\nThen show $ |f(x)|>|f(x\\minus{}1)|$[/hide]", "Solution_2": "[hide]Try factoring it like this\n\n$ (x^{5} \\plus{} 10)(x^{2} \\minus{} 2) \\equal{} \\minus{} 19$\n\nand every $ x \\ge 2$ would make the left side positive.[/hide]", "Solution_3": "[quote=\"SimonM\"]Just a thought\n\n[hide]$ f(x) \\equal{} x^7 \\minus{} 2x^5 \\plus{} 10x^2 \\minus{} 1$\n$ f(1) \\equal{} 8$\n\nThen show $ |f(x)| > |f(x \\minus{} 1)|$[/hide][/quote]\r\n\r\nThis would only prove that no [i]integer[/i] roots greater than $ 1$ exist. To prove for all reals greater than $ 1$, you would have to prove that $ f(x) > f(x\\minus{}\\epsilon)$, which is essentially proving that $ f'(x) \\ge 0$ (i.e. calculus).", "Solution_4": ":!: :!: :!: This may be one of the rare instances where [hide] Descartes' Rule of Signs [/hide] is useful, although you would have to check that the coefficients are indeed all positive. Of course, you don't need Descartes' to see that a polynomial with all positive coefficients can have only negative roots.", "Solution_5": "[quote=\"SnipedYou\"][hide]Try factoring it like this\n\n$ (x^{5} \\plus{} 10)(x^{2} \\minus{} 2) \\equal{} \\minus{} 19$\n\nand every $ x \\ge 2$ would make the left side positive.[/hide][/quote]\r\nsorry but i do not see how this helps. \r\nalso I thought about using Descartes but realized that I would have to expand after the substitution, but i guess that is what i will have to do.", "Solution_6": "[quote=\"t0rajir0u\"][quote=\"SimonM\"]Just a thought\n\n[hide]$ f(x) \\equal{} x^7 \\minus{} 2x^5 \\plus{} 10x^2 \\minus{} 1$\n$ f(1) \\equal{} 8$\n\nThen show $ |f(x)| > |f(x \\minus{} 1)|$[/hide][/quote]\n\nThis would only prove that no [i]integer[/i] roots greater than $ 1$ exist. To prove for all reals greater than $ 1$, you would have to prove that $ f(x) > f(x \\minus{} \\epsilon)$, which is essentially proving that $ f'(x) \\ge 0$ (i.e. calculus).[/quote]\r\n\r\nYeah, I should have written\r\n\r\n$ f(x)>f(y), x>y$", "Solution_7": "[hide]\nLet $ r>1$. Then $ r^7\\plus{}10r^2\\equal{}3\\cdot \\frac{r^7}{3}\\plus{}2\\cdot 5r^2\\geq 5r^5\\sqrt[5]{\\frac{25}{27}}>3r^5>2r^5\\plus{}1$, so $ r^7\\minus{}2r^5\\plus{}10r^2\\minus{}1>0$.\n[/hide]", "Solution_8": "Here is another seemingly unmotivated solution... $ x^7\\minus{}2x^5\\plus{}10x^2\\minus{}1\\equal{}8\\plus{}(x\\minus{}1)(17\\plus{}(x\\minus{}1)(11\\plus{}(x\\minus{}1)(x^4\\plus{}3x^3\\plus{}4x^2\\plus{}4x\\plus{}3) ) )$" } { "Tag": [ "Polynomials" ], "Problem": "Suppose $p(x) \\in \\mathbb{Z}[x]$ and $P(a)P(b)=-(a-b)^2$ for some distinct $a, b \\in \\mathbb{Z}$. Prove that $P(a)+P(b)=0$.", "Solution_1": "[quote=\"Peter\"]Suppose $p(x) \\in \\mathbb{Z}[x]$ and $P(a)P(b)=-(a-b)^{2}$ for some distinct $a, b \\in \\mathbb{Z}$. Prove that $P(a)+P(b)=0$.[/quote]\r\n\r\n$P(x)=a_{n}x^{n}+...+a_{1}x+a_{0}$\r\n=>\r\n\\[P(a)-P(b)=a_{n}(a^{n}-b^{n})+...+a_{1}(a-b)\\]\r\n=>\r\n\\[(a-b)|(P(a)-P(b))\\]\r\n=>\r\n\\[P(a)P(b)|(P(a)-P(b))^{2}\\]\r\n(we suppose that $P(a)P(b)$ is not equal $0$,otherwise we would have that $P(a)=P(b)=0$)=>\r\n\\[P(a)P(b)|(P(a)^{2}+P(b)^{2})\\]\r\n=>\r\n\\[P(a)|P(b)\\]\r\n\\[P(b)|P(a)\\]\r\n=>\r\n\\[|P(a)|=|P(b)|\\]\r\nThe case $P(a)=P(b)$ again gives us that $P(a)=P(b)=0$,the other case is $P(a)=-P(b)$ <=> $P(a)+P(b)=0$ :yes: :wink: .", "Solution_2": "[quote=\"Tiks\"]we suppose that $P(a)P(b)$ is not equal $0$[/quote]In fact, we are even given that (since $a,b$ are distinct).", "Solution_3": "[quote=\"Peter\"][quote=\"Tiks\"]we suppose that $P(a)P(b)$ is not equal $0$[/quote]In fact, we are even given that (since $a,b$ are distinct).[/quote]\r\nOoops,I forgot that :oops: .", "Solution_4": "$P(a)-P(b)$ is divided $a-b$\nthe rest is easy. :thumbup: " } { "Tag": [ "algebra", "function", "domain", "articles", "number theory" ], "Problem": "Two questions:\r\n\r\n1. Do PDFs on the web written by reliable people like [url=http://www.artofproblemsolving.com/Resources/Papers/SatoNT.pdf]Naoki Sato's Number Theory[/url] count as reliable sources to cite without proof?\r\n\r\n2. Suppose you find a completely correct proof (not just a statement, a full proof) on an \"unreliable\" source like AoPS forums or Wikipedia. Can you use the proof?", "Solution_1": "2. I think citing the proof would be a bad idea, but copy-pasting never hurt anyone... (I'm sure wiki is in the public domain, but I'm not sure about AoPS.)", "Solution_2": "is this for USAMT's? wats the point of using someone elses proof!?", "Solution_3": "[quote=\"Wikipedian1337\"]1. Do PDFs on the web written by reliable people like [url=http://www.artofproblemsolving.com/Resources/Papers/SatoNT.pdf]Naoki Sato's Number Theory[/url] count as reliable sources to cite without proof?[/quote]No. Although Naoki is reliable, it doesn't fit the criteria of rule #8:\n[quote=\"USAMTS Rule #8\"]Students may cite (without proof) results from a reputable source, such as a widely-accepted book or peer-reviewed journal, or from a professionally-maintained reference website such as Wolfram MathWorld.[/quote]\n\n[quote]2. Suppose you find a completely correct proof (not just a statement, a full proof) on an \"unreliable\" source like AoPS forums or Wikipedia. Can you use the proof?[/quote]Yes, a proof is a proof. If it's valid, then it's legitimate. Of course, if you find such a proof, you need to cite when you found it (otherwise you're stealing someone else's work without credit). However, it pretty much defeats the purpose of participating. The point of the contest is not to do internet research to try to find a proof--the point is to try to solve the problem yourself.", "Solution_4": "[quote]2. Suppose you find a completely correct proof (not just a statement, a full proof) on an \"unreliable\" source like AoPS forums or Wikipedia. Can you use the proof?[/quote]Yes, a proof is a proof. If it's valid, then it's legitimate. Of course, if you find such a proof, you need to cite when you found it (otherwise you're stealing someone else's work without credit). However, it pretty much defeats the purpose of participating. [b]The point of the contest is not to do internet research to try to find a proof[/b]--the point is to try to solve the problem yourself.[/quote]\r\n\r\n\r\nThats what I thought!", "Solution_5": "[quote=\"USAMTS Rule #8\"]Students may cite (without proof) results from a reputable source, such as a widely-accepted book or peer-reviewed journal, or from a professionally-maintained reference website such as Wolfram MathWorld.[/quote]\r\n\r\nReally? I cited Wolfram functions last year for the fact $ \\sum_{n\\equal{}1}^{\\infty} \\frac{\\phi(n)}{n^4} \\equal{} \\frac{\\displaystyle \\sum_{n\\equal{}1}^\\infty \\frac{1}{n^3}}{\\displaystyle \\sum_{n\\equal{}1}^\\infty \\frac{1}{n^4}}$ yet still lost a point for it. Is this rule new? Because I thought there was a rule like this when I cited it.", "Solution_6": "Last year, for the same problem, I cited that fact, and also provided the proof for it, and still got a point off.", "Solution_7": "[quote=\"facis\"][quote=\"USAMTS Rule #8\"]Students may cite (without proof) results from a reputable source, such as a widely-accepted book or peer-reviewed journal, or from a professionally-maintained reference website such as Wolfram MathWorld.[/quote]\n\nReally? I cited Wolfram functions last year for the fact $ \\sum_{n \\equal{} 1}^{\\infty} \\frac {\\phi(n)}{n^4} \\equal{} \\frac {\\displaystyle \\sum_{n \\equal{} 1}^\\infty \\frac {1}{n^3}}{\\displaystyle \\sum_{n \\equal{} 1}^\\infty \\frac {1}{n^4}}$ yet still lost a point for it. Is this rule new? Because I thought there was a rule like this when I cited it.[/quote]\r\n\r\nWell, I was able to get to $ \\sum_{n \\equal{} 1}^{\\infty} \\frac {\\phi(n)}{n^4}$ in like 1 line so proving that this is equal to $ \\frac {\\displaystyle \\sum_{n \\equal{} 1}^\\infty \\frac {1}{n^3}}{\\displaystyle \\sum_{n \\equal{} 1}^\\infty \\frac {1}{n^4}}$ is the only non-trivial part of the proof. Citing it makes the entire part of the proof trivial... I think. That should be a guideline to citing stuff..", "Solution_8": "Xaero, I slightly agree with you, but as I am completely clueless about how to manipulate Dirilecht series... Anyway, for me that was not \"line 1\" but one of my first steps once I got past proving that the circles cannot overlap and started the second part of the problem.\r\n\r\nSo I'd like to learn more about Dirilecht series, but I've had very little luck... any ideas would be appreciated.", "Solution_9": "[quote=\"facis\"]Xaero, I slightly agree with you, but as I am completely clueless about how to manipulate Dirilecht series... Anyway, for me that was not \"line 1\" but one of my first steps once I got past proving that the circles cannot overlap and started the second part of the problem.\n\nSo I'd like to learn more about Dirilecht series, but I've had very little luck... any ideas would be appreciated.[/quote]\r\n\r\nlearning Dirichlet series need results from some somewhat difficult to understand number theory, but for that particular problem, we can look at the wikipedia article and try to understand the proof.\r\n\r\n[url]http://en.wikipedia.org/wiki/Phi_function#Generating_functions[/url]\r\nYou can sort of intuitively check that the proof is correct by writing out the series and multiplying them instead of using convolution. Oh, I think it's necessary to prove\r\n$ \\sum_{d|n} \\varphi(d) \\equal{} n$ to get full credit.", "Solution_10": "Yeah, I just looked back at that and it makes a lot more sense now. I'm still not sure what $ *$ is supposed to mean but the key step seems to be that the product of the two dirilecht series can be written as a single dirilecht series: $ \\sum_{g \\equal{} 1}^\\infty \\frac {1}{g^s} \\cdot \\sum_{f \\equal{} 1}^\\infty \\frac {\\phi(f)}{f^s} \\equal{} \\sum_{h \\equal{} 1}^\\infty \\frac{\\displaystyle\\sum_{f|h} \\phi(f)}{h^s}$ if we let $ fg \\equal{} h \\implies f|h$ and then the sum of the totients of the divisors...\r\n\r\nIf I had had the inspiration to look in this direction, I might have been able to conjecture that $ \\sum_{d|n} \\phi(d) \\equal{} n$ but I probably wouldn't have figured out how to prove that... anyway, I'll know better for next time ;-)", "Solution_11": "[quote=\"123456789\"]Last year, for the same problem, I cited that fact, and also provided the proof for it, and still got a point off.[/quote]\r\n\r\nHow did you get marked off if you provided the proof? I think you should have been able to protest that (but there's nothing you can do about it now..)" } { "Tag": [ "algebra", "polynomial", "number theory proposed", "number theory" ], "Problem": "let $x_1,x_2,\\dots,x_m\\in \\mathbb{Z}$ be distinct numbers and $P(x)$ be polynomials with $m-1$ degree. prove that at least one of the numbers $|P(x_1)|,\\dots,|P(x_m)|$ is not less than $\\frac{(m-1)!}{2^{m-1}}$", "Solution_1": "You can get easy proof, using Lagrang's interpolational polynom." } { "Tag": [ "AMC", "AIME", "AMC 10" ], "Problem": "On the AIME answer form, there is a space where one bubbles which previous AMC exam was taken. If someone, such as myself, took both the AMC 10, and the AMC 12, which bubble(s) should we fill in.", "Solution_1": "I'm not positive, but I think it would be the one that you did better on (in your case AMC 12). But if my memory is really accurate, it actually doesn't matter, as they will check both scores anyway. Maybe bubbling in the better one will reduce the likelihood of error though.", "Solution_2": "What should you get on the AMC 12 to be considered \"better\" than a passing AMC 10 score?", "Solution_3": "The test-taker I coach here will just put in his AMC 12 score, which is lower NUMERICALLY than his AMC 10 score (as one would expect) but probably more significant for someone of the age to take either test.", "Solution_4": "> the one that you did better on (in your case AMC 12). But if my memory is really accurate, it actually doesn't matter, as they > will check both scores anyway. \r\n\r\nThis is correct. Fill in the higher of the two scores. We check both.\r\n\r\nSteve Dunbar\r\nDirector, American Mathematics Competitions", "Solution_5": "[quote=\"AMCDirector\"]Fill in the higher of the two scores. We check both.[/quote]\r\n\r\nThat's clear. The higher score it is then.", "Solution_6": "[quote=\"randomdragoon\"]What should you get on the AMC 12 to be considered \"better\" than a passing AMC 10 score?[/quote]\r\n\r\nI guess you could weight it so that 120 (on the 10) = 100 (on the 12) and then go up proportionally there to 150. Too much work though ;) .", "Solution_7": "It would be too much work to proportionatly see which one is a better score. Just put down the one you like the best and well they check both anyway.", "Solution_8": "[quote=\"dogseatcheese\"]It would be too much work to proportionatly see which one is a better score. Just put down the one you like the best and well they check both anyway.[/quote]\r\n\r\nAcutally I realized not really:\r\n\r\nTake your AMC 10 score. subtract 120. (Only if you got above 120) Then multiply by 3/5. Then add 100. And there you go.\r\n\r\nKK that's not that accurate but hey it's not bad.", "Solution_9": "Someone should make a converter for AMC 10 and 12 scores....what if someone.....*me* cough got less than 120 on one of the AMC 10s and over on the other.....then I would have a negative score after using your operation." } { "Tag": [ "search", "probability", "function", "ratio", "MATHCOUNTS", "expected value", "conditional probability" ], "Problem": "Now that you have all seen the \"lets make a deal\" or monty hall problem, here is one that is of a similar confusing nature. Please vote in the poll what you personally think the answer is, don't go search for it on the web or something.\r\n\r\nWe plan to put Beauty to sleep by chemical means, and then well flip a fair coin. If the coin lands Heads, we will awaken Beauty on Monday afternoon and interview her. If it lands Tails, we will awaken her Monday afternoon, interview her, put her back to sleep, and then awaken her again on Tuesday afternoon and interview her again. The (each?) interview is to consist of the one question : what is your credence now for the proposition that our coin landed Heads? When awakened (and during the interview) Beauty will not be able to tell which day it is, nor will she remember whether she has been awakened before. She knows about the above details of our experiment. What credence should she state in answer to our question?", "Solution_1": "gah. i'm tempted to say [hide]1/3[/hide] because [hide]there's a 1/3 chance that it's the interview for which the coin landed heads, and 2/3 chance it's one of the tail interviews[/hide] but it seems like she should just say [hide]1/2[/hide] because, [hide]well, there's a 1/2 chance the coin will land heads.[/hide]. gah. you have ruined my perfect life, stephen. i think i'll vote [hide]1/3[/hide].", "Solution_2": "Uhh...[hide]1/3, since an interview can be of three different natures:\n\n- it could be on the Monday, and we flipped a heads\n\n- it could be on the Monday, and we flipped a tails\n\n- it could be on the Tuesday, and we flipped a tails.\n\n\n\nSleeping Beauty doesn't know which one of these it is. But she does know that if you selected a random interview that may have taken place, 1 in 3 times it will be because we flipped a heads and 2 in three times it will be because we flipped a tails. So the chance that we did flip a heads is 1/3.[/hide]\n\n\n\nOops, I'm supposed to vote.", "Solution_3": "I think its [hide]1/3 [/hide] because the probability that she [hide]wakes up and its heads [/hide]is[hide] 1/3[/hide]... and the probability [hide]that she wakes up is 1[/hide]... so [hide](1/3)/1 = 1/3[/hide]", "Solution_4": "MysticTerminator wrote:gah. i'm tempted to say [hide]1/3[/hide] because [hide]there's a 1/3 chance that it's the interview for which the coin landed heads, and 2/3 chance it's one of the tail interviews[/hide] but it seems like she should just say [hide]1/2[/hide] because, [hide]well, there's a 1/2 chance the coin will land heads.[/hide]. gah. you have ruined my perfect life, stephen. i think i'll vote [hide]1/3[/hide].\n\n\n\nBut [hide]it doesn't matter what the coin's chances are. She knows she is in an interview, and she knows that the chances that any given interview is a \"heads\" interview is 1/3.[/hide]", "Solution_5": "Theres two types of people in this world - halfers, who think the answer is obviously 1/2, thirders who think its obviously 1/3. Both are convinced they are right.. and I have never seen a convincing proof that either is wrong.", "Solution_6": "Ok, here is an argument for each case. Don't read these until you've made up your own mind.. and then see if you can disprove either.\n\n[hide]\n\n1) For the halfers: The probability the coin toss came up heads was 1/2. SB gains absolutely no new information from being interviewed, as this was always going to happen. Therefore the probability must be 1/2.\n\n\n\n2) For the thirders: 2/3 of the interviews take place when the coin has come up tails. Therefore the probability must be 1/3.\n\n[/hide]\n\n\n\nThere are various other arguments too..", "Solution_7": "[quote=\"TripleM\"]Theres two types of people in this world - halfers, who think the answer is obviously 1/2, thirders who think its obviously 1/3. Both are convinced they are right.. and I have never seen a convincing proof that either is wrong.[/quote]\r\nlook at my post, and you'll see i'm not either. i think the question itself maybe slightly ambigous, but i can't quite pin it down.", "Solution_8": "i'll give a reason for my last comment: for the monty hall problem, we could actually enact it with dice \"choosing\" random curtains, etc, and find the probability is indeed close to 2/3 if we switch (or w/e, i can't remember exactly). for this problem, we can't do any such thing. i think it depends on how we define probability again.", "Solution_9": "Okay, here is my confusing take on it:\r\nThat probability doesn't exist. Probability measures the expected number of times an event will occur when a trial is repeated indefinitely. This is always the definition of probability. The problem is that the way this problem is stated, it [i]explicitly forbids[/i] doing an unlimited number of trials -- she has to deduce probability from a single trial. This is not possible. In essence, the question she is being asked (I believe) is meta-mathematical, and not truly mathematical. The question she can answer is \"What do you think the coin flip landed?\" to which it is in her best interest to answer \"tails.\" But the way the question is phrased, it doesn't really have an answer. [/confusion]", "Solution_10": "Ok, so how about we repeat this experiment as many times as you want to get this probability. Each time, she still gains no information from waking up and being interviewed, and the past experiments can have no effect on later tosses of the coin. So we get the same thing..", "Solution_11": "Exactly -- each repitition is completely blind. She [i]cannot[/i] compute a probability because, as a person [i]in[/i] the problem, [i]she[/i] cannot collect any repeated data. And she's the one who needs the data, because of the way the problem is phrased, not us.", "Solution_12": "My answer is she should answer 1/2 the first time and 0 if she is waken up the second time; I believe even if you are put to sleep by chemical means your memory will not degenerate in such a short period of time. Of course this is not the case here so the answer is 1/2.\r\n\r\n1/3 is incorrect because the three times she wake up does not have anything to do with the previously flipped coin, those two events are completely independant. No matter what the coin was flipped whether she's awakened, sleeping, or dead. According to this logic she could have been woken 5 times when the coin showed tails and the probability would've been 1/5, and so on until you get 0, which is obviously not possible. Therefore the answer is 1/2.\r\n\r\n\r\nBy the way if the problem asked for her credence on whether or not this interview was [b]because [/b]the coin showed heads then the answer is 1/3, see the difference?", "Solution_13": "Joel, where did you get the idea that we can only define probability if we can repeat an experiment? If I just bought a fair coin, the probability of getting heads on my very first toss is 1/2, even though I can't repeat the experiment (because later tosses won't be the first).\n\n\n\nI believe the answer to the original question is [hide]1/3[/hide], using conditional probability. I will leave out my proof until more people have thought about the question.", "Solution_14": "[quote]If I just bought a fair coin . . .[/quote]\r\n\r\nRavi -- how can you buy a fair coin? What does that mean? It means that someone, at some point, did something which either (a) flipped the coin some infinite number of times to determine that it does in fact land heads half the time or (b) carried out some experiment or calculation that determined the same thing. Either way, someone had to do something, before you ever bought the coin, that determined that it was fair. There had to have been such a determination for you to buy a fair coin in the first place.\r\n\r\nI still think the question is fundamentally unsolvable.", "Solution_15": "Ok, let us look at the begging, chance of A, B and c is each 1/3, we chose b, thus the chance of a or c being right is 2/3, no we have been told c isn't right, so a's chance is 2/3? am i wrong somewhere?", "Solution_16": "ml2, if you're thinking of the \"let's make a deal\" game, this one is different. In that situation, the game show host purposefully reveals which answer is incorrect based on which one you picked (i.e. if you had picked the correct one, he will reveal either of the two wrong ones, but if you had picked an incorrect one, he will reveal the OTHER incorrect one). In this situation, the teacher is unaware of your current response, so his saying that choice C is incorrect is completely random.\r\n\r\nSo I'd say it doesn't matter if you switch or not.", "Solution_17": "Yeah, I also get that it makes no difference whether you swap or not. \r\n\r\nSo now where does the paradox come from?", "Solution_18": "Mathfanatic's explaination is exactly right. The way the question is worded, you have to infers that the teacher's announcement was independent of your answer choice. So in theory they [i]could[/i] have announced the same answer choice that you had put, making the sample space of possibile outcomes larger than in the Monty Hall problem.\r\n\r\nThe paradox comes from the fact that this problem sounds so similar to the Monty Hall Problem, but a small inference makes such a big difference in the outcome. The physical situation of the two problems are the same, but from psychological inference the two problems become different. The psychology of infering that the teacher was not making the announcment just to you is what changes the problem!\r\n\r\nKeep in mind that a paradox isn't a problem without an answer. A paradox is a problem that seemingly has different reasonble interpretation leading to different answers. The fact that I didn't explicitly [b]say[/b] whether the teacher's announcement was independent of my answer leaves that determination up to an inference. That ambiguity of interpretation is where the paradox comes from.", "Solution_19": "i still don't see the difference, if you had siad that you picked [i]a door[/i] and the teacher told you that c isn't right then maybe, but since you did pick B and the teacher did (indepndently or not) do the same thing that the game host does.", "Solution_20": "Yes but in one case you *know* what Monty Halls strategy is, but not the teachers.", "Solution_21": "could you explain that more clearly, i can't quite understand what you mean", "Solution_22": "OK.\r\n\r\nIn Monty hall problem, he says he will open a wrong door, but not the one you have chosen.\r\nSuppose you chose door A.\r\n\r\nThe probabilities are: \r\nA correct, open B: 1/3 x 1/2 = 1/6\r\nA correct, open C: 1/3 x 1/2 = 1/6\r\nB correct, open C: 1/3 x 1 = 1/3\r\nC correct, open B: 1/3 x 1 = 1/3.\r\n\r\nSo given that door C was opened, the probability A is correct is 1/6 / (1/6 + 1/3) = 1/3.\r\n\r\nNow for the teachers problem. She doesn't know you have chosen B and will tell you one of the incorrect answers.\r\nThe probabilities are:\r\nA correct, told B: 1/3 x 1/2 = 1/6\r\nA correct, told C: 1/3 x 1/2 = 1/6\r\nB correct, told A: 1/3 x 1/2 = 1/6\r\nB correct, told C: 1/6\r\nC correct, told A: 1/6\r\nC correct, told B: 1/6.\r\n\r\nSo the probability that B is correct, given that you were told C is incorrect, is 1/6 / (1/6 + 1/6) = 1/2.", "Solution_23": "I am still undecided as to whether I think the best answer is 1/2 or 2/3, and I keep arguing myself from one side to the other. Here are my answers to Ravi's questions that I would make if i were defending the answer of 1/2:\r\n1. 1/2\r\n2. 3/4 - This is probably wrong, but this is my reasoning: there is a half of a chance to get a heads - and if this is the case there is a 100% chance of being woken up on monday. (I think this question may be just as much a source of debate as the original one.)\r\n3. 1/2 - Given that today is monday there is a 2/3 probability that the coin landed heads, hence the answer is 3/4*2/3.\r\n4. 1/4 - 1/3 probability given that today is monday.\r\n5. 0\r\n6. 1/4 - The probability of a tails is 1 if we know that today is tuesday.\r\n\r\n\r\nI think that I used these last four to answer the first ones (is that a valid method?). Anyways, I will answer as best I can for now and possibly change them later.\r\n7. 2/3\r\n8. 0\r\n9. 1\r\n10. 1/2", "Solution_24": "This problem is one of my favorite in all of mathematics because it demonstrates how difficult it is to even define probability.\r\n\r\nFirst, let us consider what probability means.\r\n\r\nThe frequentist interpretation of probability is based on repeated experiments. I personally find this interpretation superficial because it doesn't relate to reality. We can never reset the initial conditions of an experiment. It also doesn't jive with events that we can examine such as sporting events. How is it that bookies can set odds that seem to work so well in the long run [i]before each event is played out[/i]?\r\n\r\nI personally prefer the subjectivist interpretation of probability. All probabilities are subject to the information that I have about the events being considered.\r\n\r\nThis interpretation has the benefit of making problems with information in the form of frequencies solvable directly through mathematical reasoning.\r\n\r\nHowever, this interpretation also allows me to make statements such as \"I think the probability that the Cardinals will beat the Padres when next they play is 3/4 because I am [i]indifferent[/i] toward betting for or against the Cardinals as 3:1 money favorites.\"\r\n\r\nThis may seem a little goofy at first, but we can extend this seemingly blurry idea of subjective probability to problems of quantum physics -- what do we mean when we say that an electron has a 1/3 probability of being in some region of its orbital around an atomic nucleus? We can never reset initial conditions because we can never even determine what those initial conditions were (due to the Heisenberg Uncertainty Principle). We can however make a statement such as \"I am indifferent toward the proposition of betting for or against the event given 2:1 money odds against the proposition\".\r\n\r\nAs the subjectivist interpretation relates to the Sleeping Beauty problem, she should answer 1/3, not 1/2. If she were to make a bet, she would want 2:1 odds because she'll be asked the question twice following one result and only once following the other. She is not answering the question \"what was the probability that the coin heads?\" She is answering the question \"given the circumstance that we are interviewing you, what odds would you need us to lay such that you are indifferent toward betting on heads (vs. tails)?\"\r\n\r\nThis jives nicely with the Bayesian (dependent probability) arguments put forth numerous times in this thread.\r\n\r\nThe only problem with the subjectivist interpretation is that is doesn't take into account utility concerns over volatility. Volatility destroys utility and every time a person makes a wager they are subjecting themselves to volatility (this is a good reason not to play games of [i]chance[/i]). For instance, who makes more money, a person who earns 100% on some years and loses 50% on other years with approximately equal frequency (for an expected yearly return of 25%) or the person who earns 10% every year without fail?\r\n\r\nThis problem again might be analogous to real world physics. Why is it that given all other information about an electron, we can only pinpoint its location to within an interval?\r\n\r\nWhy is it that if I take volatility into consideration that I can only set a range on what odds I would take for the Yankees to win the World Series?", "Solution_25": "MCrawford, those are interesting points, and I basically agree with you.\r\n\r\nIan, you are the first halfer to at least give self-consistent answers to my set of 10 questions. Congratulations. But that doesn't mean I agree with those answers. :) Let me focus on question 7: \"Now what is the conditional probability that the coin landed heads given that today is Monday?\". Your answer was 2/3. Do other halfers agree with that answer? Let me argue that the answer should be 1/2. Basically, here we are telling Sleeping Beauty the day, and then asking her the probability of heads. It happens to be Monday. But Sleeping Beauty can reason that, whether the coin was heads or tails, she would be interviewed on Monday. Thus her being woken on a Monday has given her no information on whether the coin is heads or tails. So she should answer 1/2.\r\n\r\nIn other words, if she knows today is Monday, it is irrelevant that she sometimes gets interviewed on Tuesdays. She should answer the same as if someone had flipped a coin in front of her eyes: 1/2.", "Solution_26": "Hmmm..... I think I need something clarified about conditional probability.\r\n\r\nYour question was \"What is the probability that the coin landed heads given that today is monday?\"\r\n\r\nIm wondering -- which of the following situations is that analagous to:\r\n\r\n1. The sleeping beauty is told beforehand that she would be told what day it is when she is awoken.\r\n\r\n2. The sleeping beauty is not told what day she will be awoken but happens to notice a calendar with the day marked on it as Monday.\r\n\r\nThese are different situations, I believe -- in the first I would say the probability that the coin landed on heads would be 1/2; whereas in the second I maintain the probability that the coin landed heads would be 2/3.", "Solution_27": "Well, I had been thinking of the first situation. But the second situation seems equivalent to me, for the purpose of calculating probability. Unless Sleeping Beauty knew something like \"the interviewers make calendars available only when the coin came out heads\", both situations just filter out Tuesday outcomes. A third situation to think about conditional probability is that no one tells Sleeping Beauty what day it is; instead she just makes a conditional statement such as \"If [or given that] today is Monday, then the probability of heads is ...\". All three situations are equivalent to me. Whichever situation you prefer, it should make the formula Pr(Monday and heads) = Pr(Monday) Pr(heads | Monday) work out, i.e., let you go from Question 7 to Question 3.\r\n\r\nBasically, conditional probability means to filter out those outcomes that do not meet the condition, and then scale up the probabilities of the remaining outcomes so that they sum to 1.\r\n\r\nTo hopefully clarify our differences, let me broaden the discussion. To solve any question about \"applied\" probability, there is usually a two-step process:\r\n(1) Decide on a probability space that models the question. A probability space just means a sample space (set of outcomes) and a probability distribution on those outcomes.\r\n(2) With the probability space in Step 1 and the basic definitions of probability, calculate the probability of the event in question. In other words, add up the probabilities of those outcomes that satisfy the event.\r\n\r\nIt seems to me that we disagree on the appropriate probability space in step 1. To make our differences explicit, I will state here the probability space that I am using.\r\n\r\nHad there been no restriction against heads interviews on Tuesdays, the sample space would consist of 4 outcomes {Mh, Mt, Th, Tt}, where Mh stands for \"an interview on Monday and the coin came up heads\", and similarly for the other three outcomes. The probability distribution would be 1/4 for each. But in fact there is a restriction against heads interviews on Tuesdays; that is, the original problem is itself a conditional probability. Following the definition of conditional probability, our sample space becomes {Mh, Mt, Tt}, and the probability of each outcome scales up to 1/3 for each. That is the sample space I use. That explains why I think the answer to the original question is 1/3. It also explains why my answer to say Question 7 (the probability of heads given Monday) is 1/2. In fact, this sample space explains all my answers.\r\n\r\nSo, Ian (and other halfers), let me ask you: What is the probability space that you are using? The one you use to answer the original question and all the other questions. Thanks.", "Solution_28": "Sorry that it took me so long to respond to this thread.\r\n\r\nFirst to clear up the difference (that I think exists, but may be mistaken in doing so) between the two cases that I gave:\r\n\r\n[quote]1. The sleeping beauty is told beforehand that she would be told what day it is when she is awoken. \n\n2. The sleeping beauty is not told what day she will be awoken but happens to notice a calendar with the day marked on it as Monday. [/quote]\n\n\nThe difference that I see between these two situations is the same as the difference between the Monty-Hall problem and the one gauss202 mentioned earlier in this thread:\n\n[quote]Suppose that you're taking a multiple choice test with 3 equally likely answer choices, A, B, and C. Towards the end of the test, your teacher announces to the class that the answer to problem 1 is NOT C. You had previously marked answer choice B (but not having any idea what the question was even asking you really just chose it at random). Should you stick with your answer choice B, or should you switch your answer to A? [/quote]\r\n\r\n\r\nThe answer to gauss's problem is that it doesnt matter whether you switch or not -- whereas in the Monty-Hall problem you have a higher chance of winning if you switch. The difference between these two problems is that while you are taking the test, the teacher announces the answer is not C [i]without knowing which one you have picked[/i], but in the Monty Hall problem the person who shows you a door that doesnt contain the prize [i]purposely avoids[/i] picking the door that you chose.\r\n\r\n\r\nI think that, for the same reason that gauss's problem is not equivalent to the Monty-Hall problem, the knowledge of whether she will be told what day it is [i]does[/i] change the conditional probability that the coin landed heads.\r\n\r\n\r\n\r\n\r\nNext, my probability space is the same as yours {Mh, Mt, Th, Tt} except with probabilities as follows:\r\nMh: 1/2\r\nMt: 0\r\nTh: 1/4\r\nTt: 1/4\r\n(The different outcomes are permitted to have different wights, right?)", "Solution_29": "I agree that if the interviewers strategically determine when to make calendars available, then they can fool Sleeping Beauty. For that matter, they can fool Sleeping Beauty even more easily by not waking her according to the rules. I was assuming that the interviewers were honest. That's why I added the caveat about \"unless Sleeping Beauty knew something like the interviewers make calendars available only when the coin came out heads\".\r\n\r\nTo avoid such complications, we can stipulate that for calculating conditional probability, Sleeping Beauty knows in advance that she will be told the date. Your answer for conditional probability in that case was 1/2, which I agree with. What about the third possibility I mentioned, where Sleeping Beauty just makes conditional statements to herself, such as \"If today is Monday, then the probability of heads is ...\". How would she complete such a sentence? (Again, I think she would say 1/2.)\r\n\r\nThanks also for listing your probability distribution. I don't see where your numbers come from, but at least I know where you stand. By the way, I think you made some typos in your probability distribution. For example, you wrote that the probability of Mt is 0. But I think you meant that the probability of Th is 0. \r\n\r\nLet me try another tack to explain why I think the answer to the original Sleeping Beauty problem can't be 1/2. When we say that the probability of an event is 1/2, we generally mean that we are indifferent (as MCrawford would say) to betting for or against the event. For example, for halfers, Sleeping Beauty would be indifferent to betting that the coin was heads. In other words, she would be indifferent to gaining a dollar whenever she was awoken because of heads, and losing a dollar whenever she was awoken because of tails.\r\n\r\nNow consider the following revision of the original problem. Here she is guaranteed to be interviewed on both Monday and Tuesday, regardless of the coin outcome. (She knows about the new rules.) Here I think we would all agree that the probability of heads is 1/2. That is, Sleeping Beauty would be indifferent to gaining a dollar whenever she was awoken because of heads, and losing a dollar whenever she was awoken because of tails. \r\n\r\nNow compare the two games. The only difference between them is what happens on Tuesday heads. In the original problem, she neither gains nor wins a dollar on such days. In the revised problem, she gains a dollar. So how can she be indifferent to both games? Wouldn't she much prefer the second game to the first?" } { "Tag": [ "number theory solved", "number theory" ], "Problem": "Solve in positive integers:\r\n\r\n$x^{2}+tx=y^{2}+ty$\r\n\r\nwhere $t$ is given positive integer.\r\n\r\n[hide=\"some notes\"]related to http://www.mathlinks.ro/Forum/viewtopic.php?t=103637[/hide]", "Solution_1": "Err... $(x-y)(x+y+t)=0$?", "Solution_2": "[quote=\"TomciO\"]Err... $(x-y)(x+y+t)=0$?[/quote]\r\n\r\n :oops: \r\n\r\nso all the solutions are:\r\n\r\n$(x,y)\\in\\{(s,s),(r,t-r);s,r\\in N,r0$ and ${\\rm Res}(f,g)<0$? Can we conclude anything from ${\\rm Res}(f,g)>0$?", "Solution_1": "[quote=\"bchui\"]If the resultant polynomial ${\\rm Res}(f,g)$ of two polynomials $f$ and $g$ equals to zero then we know that $f$ and $g$ has a common root. How about the meanings of ${\\rm Res}(f,g)>0$ and ${\\rm Res}(f,g)<0$? Can we conclude anything from ${\\rm Res}(f,g)>0$?[/quote]\r\n\r\nTaking into account the sign and degree of the highest term in each polynomial, the sign of Res indicates the number of crossings between roots of $f$ and roots of $g$ when slides one set of roots across the other (pulling $f$ in the positive direction and $g$ to the negative).", "Solution_2": "So, ${\\rm Res}(f,f')=0$ means $f(x)=0$ has a repeated root. What does it mean for ${\\rm Res}(f,f')>0$ and ${\\rm Res}(f,f')<0$?", "Solution_3": "[quote=\"bchui\"]So, ${\\rm Res}(f,f')=0$ means $f(x)=0$ has a repeated root. What does it mean for ${\\rm Res}(f,f')>0$ and ${\\rm Res}(f,f')<0$?[/quote]\r\n\r\nIn that case the relative position of $f$ and $f'$ on the real line is fixed by the number of real roots of $f$, because the roots interlace (Rolle's theorem). The sign of Res is something like $(-1)^{k(k-1)/2}$ where $k$ is the number of real roots of $f$. This is after taking into account the sign and degree of the highest degree term of $f$.", "Solution_4": "By the way, do we have a definition of \"resultant\" for two multinomials $f(x_{1},.\\ldots,x_{n})$ and $g(x_{1},\\ldots,x_{n})$?" } { "Tag": [ "parameterization", "function", "probability", "probability and stats" ], "Problem": "A continuous random variable is said to have an exponential distribution with parameter $ \\lambda$ if its density function is $ f(t) \\equal{} \\lambda e^{\\minus{}\\lambda t}$ ($ 0 \\leq t < \\infty$). If $ X_1$ and $ X_2$, which are independent random variables, have exponential distributions with parameters $ \\lambda_1$ and $ \\lambda_2$ respectively, find an expression for the probability that either $ X_1$ or $ X_2$ (or both) is less than $ x$. Prove that if $ X$ is the random variable whose value is the lesser of the values of $ X_1$ and $ X_2$, then $ X$ also has an exponential distribution.\r\n\r\nRoute A and Route B buses run from my house to my college. The time between buses on each route has an exponential distribution and the mean time between busses is 15 minutes for Route A and 30 minutes for Route B. The timings of buses on the two routes are independent. If I emerge from my house one day to see a Route A bus and a Route B bus just leaving the stop, show that the median wait for the next bus to my college will be approximately 7 minutes.", "Solution_1": "I have only looked at the first part.\r\n\r\nIf X = min(X1,X2) then \r\nP[Xx] = 1-P[x1>x;X2>x].\r\nBecause of independence of X1 and X2, we have this equals to:\r\n1 - P[X1>x]P[X1>x]\r\n\r\nFrom here , you can easily deduce the distribution function of X and differentiating this function will give you the density function of X which is of a Exponential distribution as expected.", "Solution_2": "ok! Thanks! But what I do not understand is the second part:\r\n\r\ndensity function of Route A distribution: $ \\lambda_1 e^{\\minus{}\\lambda_1 t}$ where $ \\lambda_1 \\equal{} \\frac{1}{15}$\r\n\r\ndensity function of Route B distribution: $ \\lambda_2 e^{\\minus{}\\lambda_2 t}$ where $ \\lambda_2 \\equal{} \\frac{1}{30}$\r\n\r\nHence the density function of the minimum time by part one is:\r\n\r\n$ f(t) \\equal{} (\\lambda_1 \\plus{} \\lambda_2)e^{\\minus{}(\\lambda_1 \\plus{} \\lambda_2)t}\r\n \\equal{} \\frac{1}{10} e^{\\minus{}\\frac{1}{10}t}$\r\n\r\nBut the mean of this distribution is 10 minutes and not 7 as stated in the problem. Where's my mistake?", "Solution_3": "Well, it says that you have to find the median, not the mean.." } { "Tag": [ "calculus", "integration", "trigonometry", "limit", "search", "real analysis", "real analysis unsolved" ], "Problem": "Prove that\r\n$ \\int_{0}^{\\infty} \\frac{\\sin x}{x} \\mathrm{d}x \\equal{} \\frac{\\pi}{2}$\r\nusing real analytic methods.\r\n\r\nI know a proof using Residue Theorem but I want to see a straightforward real analytic approach.\r\n\r\nThanks.", "Solution_1": "[hide]$ \\mathcal{L} \\left(\\frac{\\sin x}{x}\\right) \\equal{} \\int_0^{\\plus{} \\infty} \\frac{\\sin x}{x} e^{\\minus{}sx}\\,dx \\equal{} \\arctan \\left(\\frac{1}{s}\\right)$.\n\nTherefore, \n\n$ \\int_0^{\\plus{} \\infty} \\frac{\\sin x}{x}\\,dx \\equal{} \\lim_{s \\to 0^\\plus{}} \\int_0^{\\plus{} \\infty} \\frac{\\sin x}{x} e^{\\minus{}sx}\\,dx \\equal{} \\lim_{s \\to 0^\\plus{}} \\arctan \\left(\\frac{1}{s}\\right) \\equal{} \\frac{\\pi}{2}$.[/hide]", "Solution_2": "hello, this may help you\r\nhttp://www.physicsforums.com/archive/index.php/t-22531.html\r\nSonnhard.", "Solution_3": "Carcul's proof requires an analogue for integrals of Abel's theorem for series - see [url=http://www.artofproblemsolving.com/Forum/viewtopic.php?search_id=1409951753&t=19202]here[/url]\r\n\r\nI've posted my own favorite version of this here many times, but am not sure how to find it. Here's the outline:\r\n\r\nStep 1: Integrate by parts to show $ \\int_0^{\\infty}\\frac{\\sin x}x\\,dx\\equal{}\\int_0^{\\infty}\\frac{1\\minus{}\\cos x}{x^2}\\,dx.$\r\n\r\n(The significance of step 1 is that we now have an absolutely convergent integral with a nonnegative integrand.)\r\n\r\nStep 2: Replace $ \\frac1{x^2}$ with $ \\int_0^{\\infty}te^{\\minus{}tx}\\,dt.$\r\n\r\nStep 3: Interchange the order of integration (justified by Tonelli's theorem - nonnegative integrands).\r\n\r\nStep 4: Do some calculus and algebra.", "Solution_4": "Thanks.\r\n\r\nWhen I was reading some Fourier Analysis today, I came across a proof using the following:\r\n$ \\frac {1}{2} a_{0} \\plus{} \\sum_{n \\equal{} 1}^{\\infty}a_n \\equal{} \\frac {f(0 \\plus{} ) \\plus{} f(0 \\minus{} )}{2} \\int_{0}^{\\infty} \\frac {\\sin x}{x} \\mathrm{d}x$\r\nwhere $ a_i$'s are the Fouriers coefficients involving the $ \\cos$ terms, and $ f$ satisfies certain conditions like boundedness and integrability in $ [ \\minus{} \\pi,\\pi]$, monotonicity in $ [\\minus{}\\delta,0]$ and $ [0,\\delta], 0 < \\delta < \\pi$.\r\n\r\nUsing that for $ f(x) \\equal{} 1, \\forall x$ yields our required result.", "Solution_5": "Lemma: If $ \\lim_{x \\to \\infty} f(x) \\equal{} 0$, then $ \\int_{0}^{\\infty} \\sin x f(x) dx \\equal{} \\lim_{h \\to 0} h \\sum_{n \\equal{} 1}^{\\infty} f(nh) \\sin nh$*.\r\n\r\nApplying the above lemma and noting that $ \\sum_{n \\equal{} 1}^{\\infty} \\frac {\\sin \\theta n}{n} \\equal{} \\frac {\\pi \\minus{} \\theta}{2}$ for $ \\theta \\in (0, 2\\pi)$ give the desired result for the integral in question.\r\n\r\n\r\n\r\n\r\n\r\n\r\n* sine can be replaced with cosine too.", "Solution_6": "romanovodka: my preferred pathway is to reverse the logic on your post #5. That is, first compute the integral in question by some other means, and then use that to establish the truth of the theorem you quote (or something similar to it).", "Solution_7": "Another proof I came across:\r\n$ \\lim I_n \\equal{} \\lim_{n \\rightarrow \\infty} \\int_{0}^{\\delta} \\frac {\\sin nx}{\\sin x} \\mathrm{d}x \\equal{} \\lim_{n \\rightarrow \\infty} \\int_{0}^{\\delta}\\frac {\\sin nx}{x} \\mathrm{d}x$ where $ \\delta > 0$\r\n(which follows by applying the Riemann Lebesgue lemma on $ \\frac {1}{\\sin x} \\minus{} \\frac {1}{x}$ defining as $ 0$ at $ 0$).\r\n\r\nIts easy to see that with a change of variable the RHS becomes,\r\n$ \\lim_{n \\rightarrow \\infty} \\int_{0}^{n \\delta }\\frac {\\sin x}{x} \\mathrm{d}x \\equal{} \\int_{0}^{\\infty} \\frac {\\sin x}{x} \\mathrm{d}x$\r\n\r\nWe can prove easily that the limit in the LHS exists i.e. $ \\lim_{n \\rightarrow \\infty} I_n$ exists for any $ \\delta$.(use Cauchy's principle of convergence after change of variable as before)\r\nProve that:\r\n$ \\lim_{n \\rightarrow \\infty} I_n \\equal{} \\lim_{n \\rightarrow \\infty} \\int_{0}^{\\frac {\\pi}{2}}\\frac {\\sin nx}{\\sin x} \\mathrm{d}x$\r\nProof:\r\n$ \\lim_{n \\rightarrow \\infty} \\int_{0}^{\\frac {\\pi}{2}}\\frac {\\sin nx}{\\sin x} \\mathrm{d}x \\equal{} \\lim_{n \\rightarrow \\infty} \\int_{0}^{\\delta}\\frac {\\sin nx}{\\sin x} \\mathrm{d}x \\plus{} \\lim_{n \\rightarrow \\infty} \\int_{\\delta}^{\\frac {\\pi}{2}}\\frac {\\sin nx}{\\sin x} \\mathrm{d}x$\r\nThe 2nd part of the RHS sum goes to $ 0$ by the Riemann-lebesgue lemma.\r\nSo done.\r\n\r\nWe take n=2m+1 for ease in evaluation.\r\n$ \\frac {\\sin nx}{\\sin x} \\equal{} 2(\\frac {1}{2} \\plus{} \\cos 2x \\plus{} \\cos 4x \\cdots \\plus{} \\cos 2mx)$\r\n$ \\Rightarrow \\int_{0}^{\\frac {\\pi}{2}} \\frac {\\sin nx}{\\sin x} \\mathrm{d}x \\equal{} \\frac {\\pi}{2}, \\forall m \\in \\mathbb{N}$\r\n\r\nQED.\r\n\r\n@Kent Merryfield\r\nI made an error in mentioning the theorem. The denominator should have $ \\pi$ instead of $ 2$.\r\nI think the proof I referred to is not circular logic. You can derive it without using value of the integral. A more standard result that is usually mentioned, I guess, is \r\n$ \\frac {1}{2}a_{0} \\plus{} \\sum_{n \\equal{} 1}^{\\infty}a_{n} \\equal{} \\frac {f(0 \\plus{} ) \\plus{} f(0 \\minus{} )}{2}$ which can be derived from the theorem by substituting the value of the integral. But nonetheless, it does not make it circular." } { "Tag": [ "trigonometry", "quadratics", "algebra" ], "Problem": "If $ \\alpha,\\beta,\\gamma$ are acute angles such that $ cos\\alpha\\equal{}tan\\beta,cos\\beta\\equal{}tan\\gamma,cos\\gamma\\equal{}tan\\alpha$ ,prove that $ sin\\alpha\\equal{}sin\\beta\\equal{}sin\\gamma\\equal{}\\frac{\\sqrt{5}\\minus{}1}{2}$", "Solution_1": "[hide=\"My solution\"]\n$ sin\\alpha \\equal{} cos\\alpha\\times tan\\alpha \\equal{} cos\\alpha\\times cos\\gamma$\nLet $ sin^2\\alpha \\equal{} x, sin^2\\beta \\equal{} y$ and $ sin^2\\gamma \\equal{} z$.\nThen $ x \\equal{} (1 \\minus{} x)(1 \\minus{} z)$, so $ z \\equal{} \\frac {1 \\minus{} 2x}{1 \\minus{} x}$\nSimilarly, $ y \\equal{} \\frac {1 \\minus{} 2z}{1 \\minus{} z}$ and $ x \\equal{} \\frac {1 \\minus{} 2y}{1 \\minus{} y}$.\nIn those 3 expressions we perform next:\nInto expression for $ y$ we plug in $ z$ from 1. expression and then we got $ y$ in terms of $ x$ and $ x$ in terms of $ y$. It's easy to plug in one into another and get quadratic equation:\n$ x^2 \\minus{} 3x \\plus{} 1 \\equal{} 0$ which solution is $ x \\equal{} \\frac {3 \\minus{} \\sqrt {5}}{2}$ and same for $ y$ and $ z$.\nEDIT:Then $ \\sqrt{x}\\equal{}\\frac{1\\plus{}\\sqrt{5}}{2}$\n\n[/hide]", "Solution_2": "[quote=\"Bugi\"]Are you sure?\nI got that $ sin\\alpha \\equal{} sin\\beta \\equal{} sin\\gamma \\equal{} \\frac {3 \\minus{} \\sqrt {5}}{2}$\n[hide=\"My solution-Correct it!\"]\n$ sin\\alpha \\equal{} cos\\alpha\\times tan\\alpha \\equal{} cos\\alpha\\times cos\\gamma$\nLet $ sin^2\\alpha \\equal{} x, sin^2\\beta \\equal{} y$ and $ sin^2\\gamma \\equal{} z$.\nThen $ x \\equal{} (1 \\minus{} x)(1 \\minus{} z)$, so $ z \\equal{} \\frac {1 \\minus{} 2x}{1 \\minus{} x}$\nSimilarly, $ y \\equal{} \\frac {1 \\minus{} 2z}{1 \\minus{} z}$ and $ x \\equal{} \\frac {1 \\minus{} 2y}{1 \\minus{} y}$.\nIn those 3 expressions we perform next:\nInto expression for $ y$ we plug in $ z$ from 1. expression and then we got $ y$ in terms of $ x$ and $ x$ in terms of $ y$. It's easy to plug in one into another and get quadratic equation:\n$ x^2 \\minus{} 3x \\plus{} 1 \\equal{} 0$ which solution is $ x \\equal{} \\frac {3 \\minus{} \\sqrt {5}}{2}$ and same for $ y$ and $ z$.\n\n[/hide][/quote]\r\nLooks like what you have is $ \\sin^2 \\alpha \\equal{} \\sin^2\\beta \\equal{} \\sin^2\\gamma \\equal{} \\frac {3 \\minus{} \\sqrt {5}}{2}$.", "Solution_3": "Yes, than solution is correct.\r\nThanks!", "Solution_4": "[quote=\"Bugi\"][hide=\"My solution\"]\n$ sin\\alpha \\equal{} cos\\alpha\\times tan\\alpha \\equal{} cos\\alpha\\times cos\\gamma$\nLet $ sin^2\\alpha \\equal{} x, sin^2\\beta \\equal{} y$ and $ sin^2\\gamma \\equal{} z$.\nThen $ x \\equal{} (1 \\minus{} x)(1 \\minus{} z)$, so $ z \\equal{} \\frac {1 \\minus{} 2x}{1 \\minus{} x}$\nSimilarly, $ y \\equal{} \\frac {1 \\minus{} 2z}{1 \\minus{} z}$ and $ x \\equal{} \\frac {1 \\minus{} 2y}{1 \\minus{} y}$.\nIn those 3 expressions we perform next:\nInto expression for $ y$ we plug in $ z$ from 1. expression and then we got $ y$ in terms of $ x$ and $ x$ in terms of $ y$. It's easy to plug in one into another and get quadratic equation:\n$ x^2 \\minus{} 3x \\plus{} 1 \\equal{} 0$ which solution is $ x \\equal{} \\frac {3 \\minus{} \\sqrt {5}}{2}$ and same for $ y$ and $ z$.\nEDIT:Then $ \\sqrt {x} \\equal{} \\frac {1 \\plus{} \\sqrt {5}}{2}$\n\n[/hide][/quote]\r\n\r\nhow can you say that The last line?", "Solution_5": "[quote=\"Euclid_great\"][quote=\"Bugi\"][hide=\"My solution\"]\n$ sin\\alpha \\equal{} cos\\alpha\\times tan\\alpha \\equal{} cos\\alpha\\times cos\\gamma$\nLet $ sin^2\\alpha \\equal{} x, sin^2\\beta \\equal{} y$ and $ sin^2\\gamma \\equal{} z$.\nThen $ x \\equal{} (1 \\minus{} x)(1 \\minus{} z)$, so $ z \\equal{} \\frac {1 \\minus{} 2x}{1 \\minus{} x}$\nSimilarly, $ y \\equal{} \\frac {1 \\minus{} 2z}{1 \\minus{} z}$ and $ x \\equal{} \\frac {1 \\minus{} 2y}{1 \\minus{} y}$.\nIn those 3 expressions we perform next:\nInto expression for $ y$ we plug in $ z$ from 1. expression and then we got $ y$ in terms of $ x$ and $ x$ in terms of $ y$. It's easy to plug in one into another and get quadratic equation:\n$ x^2 \\minus{} 3x \\plus{} 1 \\equal{} 0$ which solution is $ x \\equal{} \\frac {3 \\minus{} \\sqrt {5}}{2}$ and same for $ y$ and $ z$.\nEDIT:Then $ \\sqrt {x} \\equal{} \\frac {1 \\plus{} \\sqrt {5}}{2}$\n\n[/hide][/quote]\n\nhow can you say that The last line?[/quote]\r\n\r\nWell $ {(\\frac {1 \\plus{} \\sqrt {5}}{2})}^2\\equal{}\\frac{3\\minus{}\\sqrt{5}}{2}$, and the negative solution is impossible because angles are acute." } { "Tag": [ "number theory", "least common multiple" ], "Problem": "[b][u]What is the smallest positive integer greater than 1 which leaves a remainder of 1 when divided by 2,3,4,5,6 or 7?[/u][/b]", "Solution_1": "[hide]LCM of those numbers, plus 1. Which I am too lazy to figure out.[/hide]", "Solution_2": "LCM=420\r\n420+1=421", "Solution_3": "just find the LCM of them and add one which is like 420ish\r\n\r\n\r\n421 apparently by everyone else" } { "Tag": [], "Problem": "John and Judith are guinea pig farmers. They need to make enclosures for their many species of guinea-pig, but unfortunately, they live in a country where there is \"fence tax\". Consequently, they can only afford to erect a maximum of 24 fences. The enclosures can have any number of sides and can have any shape, so long as the fences are straight, and a fence can join another fence only at end points. What is the largest number of enclosures they can make?\r\n\r\nSource: Australian Math Competition, S29, 2000", "Solution_1": "I got [hide=\"Guess\"] 13. :? [/hide]", "Solution_2": "Unfortunately that's wrong. :)", "Solution_3": "[hide=\"New and Improved Guess, though still probably wrong\"] 14. :D [/hide]\n\n[hide=\"Explanation, just in case I'm right\"] The most number of sections you can make dissecting a polygon by drawing lines to the vertices is $n$ with $2n$ lines. So the minimum answer is 12. You can make this more efficient by adding on to an existing side. The more additions, the more efficient. We can make lots of figures to make it as efficient as possible, therefore we use triangles. We can keep adding on to the side of a triangle and dissecting it. We can form 4 triangles, each including 3 sections, with 21 fences. We can finish off the 3 fences any way we want, but the maximum is always $\\boxed{14}$. [/hide]", "Solution_4": "Err that's still not right (nearly there though). However you were correct about adding on to an existing side. \r\n\r\nConsider using triangles (polygon with least sides) everywhere and you've pretty much got it." } { "Tag": [ "summer program", "Mathcamp", "Ross Mathematics Program", "PROMYS" ], "Problem": "Is there a special section in the college resume that asks what you did over the summer? (listing names of camps, activities, etc each year) :?", "Solution_1": "yes, I believe so.", "Solution_2": "I always just listed mine under \"seminars, etc\"... that's where I listed AoPS class, my equine reproduction seminar, and my transportation engineering internship (which was horrible, fyi). When in doubt, put it under whatever's most general.", "Solution_3": "The [url=http://app.commonapp.org/index.cfm?APP=AppOnline&ACT=Display&DSP=Forms]Common Application[/url] has a space for asking about summer activities.", "Solution_4": "Having looked at the common application...I'm concluding that we don't list all the camps we've attended during summer?\r\nAlso, what does the resume cover? \r\nThank you!", "Solution_5": "[quote=\"thisismysn\"]Having looked at the common application...I'm concluding that we don't list all the camps we've attended during summer?[/quote]\r\n\r\nWell, you could. The first general principle to consider is that you list activities since eighth grade. Some middle school accomplishments are impressive, but generally colleges want to know what you did at high school age. So start with the summer after eighth grade, usually. If you run out of space, you can write \"see attachment\" and put on an extra page to list all the summer programs you did. That is an issue of setting priorities: list the summer programs that were most impressive (that is, the ones that you had to take a test to get into) and maybe don't mention unknown local programs except in such terms as \"local chess champ, 2003-2006\" or something like that. But you DEFINITELY want to mention something like MOP, MathCamp, Ross, or PROMYS, so make space for it if it doesn't fit in the standard blanks on the form." } { "Tag": [ "vector", "linear algebra" ], "Problem": "Let V be a vector space over the reals R. Let F(V) be the set of all linear maps T: V ->R (linear functionals), with standard operations:\r\n(T+S)(v) = T(v)+S(v) and (cT)(v)=c(T(v)).\r\n\r\na) Check that F(V) is a vector space with these operations (this one I can do)\r\nb) Prove that if V is finite dimensional, then F(V) is finite dimensional too, and dim F(V) = dim(V).", "Solution_1": "For part (b): Choose a basis $b_{i}$ for $V$. A basis for the dual $V^{*}$ ($F(V)$ in your problem) is $u_{i}$ so that $u_{i}(v_{i})=1$ and $u_{i}(v_{j})=0$ for $i\\neq j$." } { "Tag": [ "geometry", "circumcircle" ], "Problem": "O is the orthocentre of $ \\triangle ABC$. $ \\angle BOC$ =$ 90^o$, find $ \\angle BAC$.\r\n\r\nThank you.", "Solution_1": "Let X be the perpendicular from B to AC, so BX passes through O. Since $ CO\\perp BX$ and $ CX \\perp BX$, we must have $ X \\equal{} O$ (they are the same point), because there is only one perpendicular from C to BX. Thus X is the orthocenter, so $ CX \\perp AB$, so $ \\angle BAC \\equal{} 90$.", "Solution_2": "Another way:\r\n\r\nThe image of the orthocentre lies on the circumcircle of ABC, so $ \\angle BHC\\equal{}180^\\circ\\minus{}\\alpha$" } { "Tag": [ "geometry", "AMC", "AIME", "quadratics", "USA(J)MO", "USAMO", "probability" ], "Problem": "Suppose that you are on the last problem of the AMC. You have solved every other problem and are fairly certain you got them all without any mistakes etc. The last problem is stumping you and you only have less than a minute left...not nearly enough time to solve the problem. Do you guess and hope that you are lucky OR do you leave it blank and convince yourself that you are content with a 146.5?", "Solution_1": "I would guess. That kind of mentality helped me go from virtually guaranteed 143 to 138 this year :). Still, there isn't that much to lose on the AMC under those circumstances, so I say why not.", "Solution_2": "As Bob puts it, \"it's a sin to get a 146.5\".\r\n\r\nSeriously, what's the difference between a 146.5 and a 144 (don't say 2.5 points)? Everyone will know that one person chose to leave a blank and the other took a guess. I'd much rather have a 20% chance of getting a perfect and my picture on the AMC website than a 100% chance of not getting a perfect and no picture on the AMC website.", "Solution_3": "For me, definately the guess. First, you can always, always narrow down answer choices, sometimes with math, sometimes with logic, sometimes with a calculator... anything!\r\n\r\nOdd analysis: Lets say that you have a 30% of being right, which isn't unreasonable. If you are a perfect scorer on the AMC, you will be remembered - the prestige is great. You're guaranteed at the top of your nation, state, school, etc.... There's a lot in it. \r\nIf you lose - you lost 2.5 points, from 146.5 -> 144. Big deal? No.\r\n\r\nSo guess!\r\n\r\n\r\nBy the way - On last year's AMC-10, this happened to me. (But not on 25, I'm thinking on 24.) I decided to solve a geometry problem by measuring the diagram on the test. I got the problem right. I didn't get the 150 because of other fun mistakes, like on problem 3 or something like that, but it was worth it to guess.", "Solution_4": "Haha, I wouldn't describe a 146.5 as sinful, having had one myself once. I did guess the next time I took it (no gain in getting two 146.5s in the same year, obviously), but sadly to no avail. I say leave it blank -- what if you're wrong about having gotten all the others? (And, as Kent Merryfield said in another post, guessing is a losers game -- the wrong answers are sure to look more attractive than the right one.) Sorry for being unromantic, or whatever.", "Solution_5": "[quote=\"JBL\"](And, as Kent Merryfield said in another post, guessing is a losers game -- the wrong answers are sure to look more attractive than the right one.)[/quote]\r\n\r\nI have a TI-30xIIS calculator (in addition to a TI-89)... if I was randomly guessing, I'd probably use the random number generator to give me an integer from 1 to 5, inclusive (unless, of course, I could eliminate a choice or two for certain; then, I'd use the generator to make an integer from 1 to 3 or something :D).", "Solution_6": "DEFINITELY GUESS... You are only gonna get 2.5 pts lower than what you are supposed to get, and the 2.5 pts is not important at all. You can easily gain it back by getting 1 more AIME question right :) .\r\n\r\nAlso, say you leave the question blank, and you get 7 problems wrong:\r\n\r\nYou'll end up with a 104.5\r\n\r\nBut if you guess, you still have a 102, you still qualify for AIME.\r\n\r\n!", "Solution_7": "People like me count on the AIME as opposed to the AMC anyway, so I just guess--whatever :)", "Solution_8": "Definitely guess. Getting a 146.5 (I'm sad to admit I have) makes you feel very unmanly. Sort of like a 0 on the USAMO. But I won't go there.", "Solution_9": "[quote=\"ednerd\"]Definitely guess. Getting a 146.5 (I'm sad to admit I have) makes you feel very unmanly. Sort of like a 0 on the USAMO. But I won't go there.[/quote]\r\n\r\nWhat if you're a girl?", "Solution_10": "150 ALL THE WAY!!! (I got a 144 =P, but that wasn't from a guess so yeah)\r\n\r\nLet's see... \r\n(Glory of 150) * (1/5) ? (Closeness of 144) * (4/5)\r\n(Glory of 150) ? 4(Closeness of 144)\r\n\r\nWell since the Glory of 150 includes a picture on the AMC Website, we will proceed to add 1000 points.\r\n\r\nHence\r\n\r\n(Glory of 150) = 150+1000 = 1150 > 576 = 4(Closeness of 144)\r\n\r\nThus guessing is worth it.", "Solution_11": "This has actually happened to me. On the 2004 10A, I couldn't solve a problem (I think it was #19), so I just picked one that looked attractive -- I think it was 8/9. It turned out that was the correct answer!\r\n\r\nBut, of course, I missed a different problem, so I got a 144. The moral of this story: check your work.", "Solution_12": "[quote]What if you're a girl?[/quote]\r\n\r\nIt makes you feel unwomanly?", "Solution_13": "[i]If[/i] I ever come that close, I'll mull it over on the spot.", "Solution_14": "Actually, it doesn't just make you feel unmanly. You just find yourself regretting not solving it/yelling at yourself. Especially if not solving it involves solving a quadratic equation wrong and wondering why none of the answer choices are right.", "Solution_15": "Because I would never, ever, ever, ever have any confidence in my answers for the other 24, I'd stay. I wouldn't be happy with a perfect due to random guessing anyway. I'd feel like I didn't deserve it.", "Solution_16": "[quote=\"yif man12\"]Miths, if you ever get a perfect, just send in a picture of BoA. Please. It would be so sick.[/quote]\r\n\r\nHaha...perfect score?...wow...we'll see. ;) It would actually be pretty cool if I sent in a picture of BoA though!! :D\r\n\r\nAs far as my grade level...I'm a sophomore, which makes me pretty much a newb in terms of years of experience compared to many of the users here.\r\n\r\nAs a side note...for token and qvc and anyone else who's interested (and hasn't been keeping up with my xanga :-P ), [url=http://img.photobucket.com/albums/v452/MithsApprentice/SCCHE.jpg]here's a picture[/url] from like last October. That the picture makes my face look fatter than it should be though...so...*cough*. ^_^", "Solution_17": "Well... if you let your hair grow you might look like BoA.", "Solution_18": "[quote=\"tetrahedr0n\"]Well... if you let your hair grow you might look like BoA.[/quote]\r\n\r\nLol. Nice.", "Solution_19": "whoa. if I ever get a perfect score, I'll need to send in a picture of .... something. hmm. now I need ideas. just in case.", "Solution_20": "[quote=\"MysticTerminator\"]whoa. if I ever get a perfect score, I'll need to send in a picture of .... something. hmm. now I need ideas. just in case.[/quote]\r\n\r\nwhatever that is on your avatar! :D", "Solution_21": "[quote=\"MysticTerminator\"]whoa. if I ever get a perfect score, I'll need to send in a picture of .... something. hmm. now I need ideas. just in case.[/quote]\r\n\r\nwhatever that is on your avatar! :D \r\n\r\nbtw, if i get one, i am sending in Homer Simpson... :lol:", "Solution_22": "[quote=\"MysticTerminator\"]whoa. if I ever get a perfect score, I'll need to send in a picture of .... something. hmm. now I need ideas. just in case.[/quote]\r\n\r\n[img]http://home.wanadoo.nl/r.de.sain/geitebreier/catsniper.jpg[/img]\r\n\r\nUse that picture!", "Solution_23": "Double LOL (literally) from tokenadult and A+MATH here.", "Solution_24": "First time AMC has seen a sniper kitten who's good at math. I would send something like that in, if only my parents would let me.", "Solution_25": "In the unlikely event that I get a 150 on the AMC-12 next year, I promise you folks to send in that picture.", "Solution_26": "All the perfect scorers should send in the SAME picture :P", "Solution_27": "If that's the case, they'd definitely have to send in pictures of BoA. Tis only fair to the viewers.", "Solution_28": "What is BoA?", "Solution_29": "I think this has been discussed several times now... use the search or ask Miths." } { "Tag": [ "LaTeX" ], "Problem": "well, sometimes TeXNiCcenter used to notify me when i need to install a package, but now it does not appear such notification, and it's not nice because every package i need i have to download it and install it by myself.\r\n\r\ni'm using 2.6 version, since i couldn't install the 2.7.", "Solution_1": "1) Go to MiKTeX, Settings, Install packages on-the-fly and choose Ask me first.\r\nOr\r\n2) Download the complete version of MiKTeX and then you won't need to keep installng." } { "Tag": [ "calculus", "integration", "logarithms", "real analysis", "calculus computations" ], "Problem": "I have a few problems, the first one, I have no idea whatsoever how to convert a Riemann sum to a definite integral:\r\n\r\n1. lim(n->infin.) (1/n) [sum of (n, i=1) 1/(1+(i/n))^2]\r\n\r\nThe other two are integrals that I don't understand the process for:\r\n\r\n2. int((x^5)(e^(x^2))\r\n\r\n3. int_1^4(e^(sqrt(x)))\r\n\r\nHelp would be much appreciated!", "Solution_1": "1:\r\n\r\n$\\frac{1}{n}\\sum_{i=1}^n\\frac{1}{1+(i/n)^2}$\r\n\r\nThe $i/n$ in the sum goes from $1/n$ to $1$. As $n\\to\\infty$, this is $\\int_0^1\\frac{dx}{1+x^2}=\\mathrm{arctan}\\;1=\\frac{\\pi}{4}$\r\n\r\n2:\r\n\r\n$I=\\int x^5e^{x^2} dx$\r\n\r\nSubstitute $u=x^2$. Then $x\\;dx=\\frac{1}{2}du$, so $I=\\frac{1}{2}\\int u^2e^udu$. This can be done with partial integration.\r\n\r\n3:\r\n\r\n$I=\\int_1^4e^{\\sqrt{x}}dx$\r\n\r\nWe can substitute again. $u=\\sqrt{x}$ gives $dx=2u\\;du$ so we have\r\n\r\n$I=2\\int_1^2ue^{u}du$\r\n\r\nAnd this can be done with partial integration again.", "Solution_2": "Thanks so much for your help... one question though, in the Riemann sum, the whole 1+ i/n term is squared, not just the i/n term... how does this affect the result?", "Solution_3": "Oh, sorry. Well, we get\r\n\r\n$\\int_0^1\\frac{dx}{(1+x)^2}$\r\n\r\nI reckon. This is not as easy to evaluate though...", "Solution_4": "That shouldn't be hard; it's $(1+x)^{-2}$, which can be done by the general power rule.", "Solution_5": "the part that confuses me is the dx being over the term... I have another similar problem that I'm not sure how to work through because of this:\r\n\r\nint( dx/(1+e^x) )\r\n\r\nany solutions or suggestions?", "Solution_6": "That's just notation to save space. If you feel more comfortable with it that way, write the $dx$ off to the side: $\\int \\frac1{1+e^x}\\,dx$.\r\n\r\nFor this, try the substitution $v=1+e^x$, or the partial substitution $u=e^x$. You'll have to pull $du$ or $dv$ out of thin air, so it might be helpful to write the substitutions backward ($x=\\ln(v-1)$ or $x=\\ln u$).", "Solution_7": "That makes sense, I feel kinda stupid for not realizing that.... oh well... I'll work through it and see what I come up with... thanks for your help!", "Solution_8": "Remark that $\\frac{1}{1+e^x}+\\frac{e^x}{1+e^x}=1.$ :)" } { "Tag": [ "AMC", "AMC 10", "AMC 10 B", "search" ], "Problem": "15. There are 100 players in a singles tennis tournament. The tournament is single elimination, meaning that a player who loses a match is eliminated. In the 1rst round, the strongest 28 players are given a bye, and the remaining 72 players are paired off to play. After each round, the remaining players play in the next round. The match continues until only one player remains unbeaten. The total number of matches played is \r\n\r\n(A) a prime number (B) divisible by 2 (C) divisible by 5\r\n(D) divisible by 7 (E) divisible by 11\r\n\r\n[b]The first part of the problem about \"28 strongest players are given a bye\" is very confusing. I don't understand this problem.[/b]", "Solution_1": "that part doesnt matter. All you need to know is that there are 100 players\r\n\r\nsince there are 100 players, 99 have to be elliminated, so you find the letter that satisfies 99, which is E :P. I think that part was meant to confuse you", "Solution_2": "Um, hello, you know you can always search for the full solution in the resources section:\r\n\r\n[url]http://www.artofproblemsolving.com/Forum/viewtopic.php?p=1370108#1370108[/url]\r\n\r\n(Or look up \"2003 AMC 10B Problems/Problem 15\" in the wiki)\r\n\r\nBut as to your question, \"the 28 strongest players are given a bye\" means that those 28 players automatically progress to the second round without playing a first round match, so it's like an \"automatic\" win against an imaginary player (by that, that's the definition of a bye) That means those 28 players are technically not involved in any matches for the first round. But the 72 other players are paired, so 36 matches are played between those players in Round 1. Then the losers get eliminated, and the 36 winners of those matches, along with the 28 \"bye\" players are paired for Round 2, and so on for the rest of the rounds, until only one player remain as the winner.", "Solution_3": "Basically a bye is that they pretty much sit out for one round.\r\n\r\nSince it is elimintion, 1 person gets out for every match played. So it is 99 matches, which is $ E$ divisible by 11", "Solution_4": "Also, please try to keep discussion about a problem in that problem's original thread. That way, if someone else has the same question, they don't have to start a new thread about it." } { "Tag": [ "inequalities proposed", "inequalities" ], "Problem": "Prove the following ineq:\r\n$ (3a\\plus{}2b\\plus{}c)(\\frac{1}{a}\\plus{}\\frac{1}{b}\\plus{}\\frac{1}{c}) \\leq \\frac{45}{2}$ with $ a;b;c$ satisfying $ 1 \\leq a;b;c \\leq 2$", "Solution_1": "http://www.mathlinks.ro/viewtopic.php?p=1066439#1066439\r\nReffering to pvthuan that problem is from an ongoing contest. Please lock the topic..." } { "Tag": [ "integration", "algebra", "polynomial", "function", "real analysis", "real analysis unsolved" ], "Problem": "$ f\\in R[a,b]$ and $ \\int_a^b{f(x)dx} > 0$, $ P$ is a polynomial s.t. $ \\int_a^b{P^2(x)f(x)dx} \\equal{} 0$.\r\nShow that $ P\\equiv 0$", "Solution_1": "This doesn't appear to be true. For example, if $ a \\equal{} \\minus{}b$ and $ f(x)$ is a positive even function then the first condition is clearly true but $ x f(x)$ is odd.", "Solution_2": "Sorry, I made a typing mistake.\r\nActually it should be $ \\int_a^b{P^2(x)f(x)dx} \\equal{} 0$.", "Solution_3": "It's still not true: consider $ f(x) \\equal{} x$, $ [a, b] \\equal{} [ \\minus{} 1, 2]$ and $ P(x) \\equal{} x \\minus{} c$ with $ c \\equal{} \\frac {1 \\minus{} \\sqrt {6}}{2}$. (This is what you get when you solve the equation $ \\int_{ \\minus{} 1}^2 x(x \\minus{} c)^2 dx \\equal{} 0$ for $ c$. If I were a textbook, I would have chosen $ a$ and $ b$ so that $ c$ came out rational :wink: ) Perhaps there's supposed to be a quantifier somewhere that makes this not false? (The conclusion looks like the statement should begin, \"Suppose $ P$ is a polynomial such that for all functions $ f$ with the property ....\")" } { "Tag": [ "Euler", "geometry", "circumcircle", "geometric transformation", "reflection", "geometry proposed" ], "Problem": "prove that there is a point $ P$ on euler line of $ \\triangle ABC$ that $ AG_A\\equal{}BG_B\\equal{}CG_C$.where $ G_A,G_B,G_C$ are centroids of $ \\triangle BPC,\\triangle APC,\\triangle APB$", "Solution_1": "Thank you dear [b][size=100]Amir.S[/size][/b], for a nice problem.\r\n\r\nI can only this time to present the construction of the point $ P,$ as the problem states and I will post here later more details.\r\n\r\nThe basic idea is that for every point $ P,$ lies on the Euler line $ OH$ of a given triangle $ \\bigtriangleup ABC,$ where $ O,$ $ H,$ are its circumcenter and orthocenter respectively, the line segments $ AGa,$ $ BGb,$ $ CGc,$ as the problem states, are concurrent at one point so be it $ Q,$ which lies also on the line segment $ OH.$\r\n\r\nSo, if we consider $ Q\\equiv O,$ we can define two points $ Ga,$ $ Gb,$ on the line segments $ AO,$ $ BO$ respectively, such that $ GaGb\\parallel MaMb,$ where $ Ma,$ $ Mb,$ are the midpoints of the side-segments $ BC,$ $ AC$ respectively and simultaneously $ GaGb \\equal{} \\frac {2}{3}\\cdot (MaMb)$.\r\n\r\nThe line segments $ MaGa,$ $ MbGb,$ intersect each other at one point $ P,$ lies also on the Euler line $ OH$ of $ \\bigtriangleup ABC,$ which has the property as the problem states.\r\n\r\nKostas Vittas.", "Solution_2": "[quote=\"Amir.S\"] [color=darkred]Prove that there is a point $ P$ on [b]Euler's line[/b] of $ \\triangle ABC$ that $ AG_a \\equal{} BG_b \\equal{} CG_c$ ,\n\nwhere $ G_a,G_b,G_c$ are the [i]centroids[/i] of $ \\triangle BPC$ , $ \\triangle APC$ , $ \\triangle APB$ .[/color] [/quote]\r\n[color=darkblue][b][u]Proof.[/u][/b] Denote the [i]projection[/i] $ D$ of $ A$ to $ BC$ and the [i]reflection[/i] $ D_1$ of $ D$ w.r.t.\n\nthe [i]midpoint[/i] of $ [BC]$ ( $ D$ and $ D_1$ are [i]isotomic[/i] w.r.t. $ [BC]$ ). Prove easily metrically\n\n( [b]I\"ll post if you wish ![/b] ) that the required point $ P$ has the property $ PD_1\\perp BC$ .\n\n[b]Remark.[/b] Denote the [i]projections[/i] $ D$ , $ E$ , $ F$ of the [i]orthocenter[/i] $ H$ on the sidelines $ BC$ , $ CA$ , $ AB$ respectively and the its [i]isotomic[/i] points $ D_1$ , $ E_1$ , $ F_1$ w.r.t. the sides $ [BC]$ , $ [CA]$ , $ [AB]$ respectively. Prove easily ( using the [b]Carnot's lemma[/b] ) that the perpendicular lines in the points $ D_1$ , $ E_1$ , $ F_1$ on the sidelines $ BC$ , $ CA$ , $ AB$ respectively are concurrently (let $ P$ be this point). The upper proof certifies that the point $ P$ belongs to the [b]Euler's line[/b] of the triangle $ ABC$ .[/color]" } { "Tag": [ "real analysis", "real analysis unsolved" ], "Problem": "Given two convergent series of real numbers, $ \\sum_{j\\equal{}0}^{\\infty} a_j$ and $ \\sum_{k\\equal{}0}^{\\infty} b_k$, we define their product as $ \\sum_{n\\equal{}0}^{\\infty} c_n$ where $ c_n\\equal{}\\sum_{j\\plus{}k\\equal{}n} a_j b_k$. The series $ \\sum_{n\\equal{}0}^{\\infty} c_n$ may be divergent. Rudin gives this example: if $ a_j\\equal{}b_j\\equal{}\\frac{(\\minus{}1)^j}{\\sqrt{j\\plus{}1}}$, then each of the $ n\\plus{}1$ products in the formula for $ c_n$ has the same sign and is at least $ \\frac{1}{n\\plus{}1}$ in absolute value. Hence $ |c_n|\\ge 1$ for all $ n$.\r\nOn the other hand, the product converges if at least one of the series $ \\sum_{j\\equal{}0}^{\\infty} a_j$ and $ \\sum_{k\\equal{}0}^{\\infty} b_k$ is absolutely convergent. \r\n\r\nWhat about the converse? Given a conditionally (=non-absolutely) convergent series $ \\sum_{j\\equal{}0}^{\\infty} a_j$, can one find a convergent series $ \\sum_{k\\equal{}0}^{\\infty} b_k$ for which the product diverges? To begin with, take $ a_j\\equal{}\\frac{(\\minus{}1)^j}{j\\plus{}1}$.", "Solution_1": "We can take care of your $ a_j\\equal{}\\frac{(\\minus{}1)^j}{j\\plus{}1}$ example by letting $ b_j\\equal{}\\frac{(\\minus{}1)^j}{\\ln(j\\plus{}2)}.$\r\n\r\nSlightly more generally, suppose $ a_j\\equal{}(\\minus{}1)^jp_j$ for decreasing $ p_j>0$ and define $ q_k\\equal{}\\sum_{j\\equal{}0}^kp_j.$ By assumption, $ q_j\\to\\infty$ as $ j\\to\\infty.$\r\n\r\nLet $ b_j\\equal{}\\frac{(\\minus{}1)^j}{q_j}.$\r\n\r\nThen as in Rudin's example, all of the signs in any sum for $ c_n$ are equal, and\r\n\r\n$ |c_n|\\equal{}\\sum_{j\\equal{}0}^n\\frac{p_j}{q_{n\\minus{}j}}\\ge\\frac1{q_n}\\sum_{j\\equal{}0}^np_j\\equal{}\\frac{q_n}{q_n}\\equal{}1.$\r\n\r\nSo $ c_n$ doesn't tend to zero and the sum of the $ c_n$ diverges.\r\n\r\nThat's not a general proof, since it's all still tied into the specifics of the alternating series test, but it seems to bode well for the prospects of your proposed theorem being true.", "Solution_2": "Oh. I made a sloppy estimate and somehow decided that 1/log doesn't work. But it does. You don't even need $ p_j$ to be decreasing." } { "Tag": [ "geometry", "circumcircle", "trigonometry" ], "Problem": "Find the area of a regular polygon in terms of it's side length s.", "Solution_1": "If we're just given the side length, then it is impossible. But if we're given that it's a regular $n$-gon, then a formula in terms of $n$ and $s$ would be $\\frac{ns^{2}\\cot\\left(\\frac{180}{n}\\right)}{4}$.", "Solution_2": "That's the exact same formula I got! How did you get it? I just found a pattern that worked with 3-5.", "Solution_3": "We can divide a regular $n$-gon into $n$ congruent triangles. We'll find the area of one of the triangles and multiply it by $n$. \r\n\r\nLet $R$ denote the circumradius of the polygon. Then a formula for the triangle is $\\frac{1}{2}\\sin\\left(\\frac{360}{n}\\right)R^{2}$. To find $R$ in terms of $s$, we can write\r\n$\\sin\\left(\\frac{180}{n}\\right)=\\frac{s}{2R}$\r\n$\\Longrightarrow R=\\frac{s}{2\\sin\\left(\\frac{180}{n}\\right)}$. Substituting this in to our expression, we get $\\frac{ns^{2}\\cot\\left(\\frac{180}{n}\\right)}{4}$." } { "Tag": [ "geometry", "perpendicular bisector" ], "Problem": "On page 224 of Volume 2 (first page of Find It and Make It), I don't get the part where is says that we cannot conclude that the desired locus is the perpendicular bisector of AB, and that only the fact that every point in the bisector is in the locus was proven. It then goes on with a similar proof and then concludes that the locus is the perpendicular bisector of the segment. Could someone please clarify/explain this to me? Thanks in advance!", "Solution_1": "Here are three statements:\r\n\r\n1) Everything that is an element of $ X$ is an element of $ Y$.\r\n2) Everything that is not an element of $ X$ is not an element of $ Y$.\r\n3) $ X \\equal{} Y$.\r\n\r\nWhat is the logical relationship between these three statements?\r\n\r\n\r\nIn your case, $ X$ is the locus of points equidistant from two given points (at least this is my guess based on what you've said) and $ Y$ is the perpendicular bisector of the segment joining two given points.", "Solution_2": "[quote=\"JBL\"]Here are three statements:\n\n1) Everything that is an element of $ X$ is an element of $ Y$.\n2) Everything that is not an element of $ X$ is not an element of $ Y$.\n3) $ X \\equal{} Y$.\n\nWhat is the logical relationship between these three statements?[/quote]\n\nOh, so Vol. 2 first proved statement one with SSS. However, the fact that the locus is the perpendicular bisector of the segment was not proven until statement two was proven, which automatically proved statement three, which represents what we were trying to prove in the first place. Please correct me if this is not right.\n\n[quote=\"JBL\"]In your case, $ X$ is the locus of points equidistant from two given points (at least this is my guess based on what you've said)[/quote]\r\n\r\nYes, that's the locus I was referring to. :)\r\n\r\nThanks a lot!", "Solution_3": "Right: the third statement is equivalent to (1) and (2) together, and you need both (1) and (2) in order to conclude (3). In other words, the first result proved that the bisector is [i]contained in[/i] (not \"equal to\") the locus; you then need to show that the locus is contained in the bisector, or else the locus could consist of the bisector plus a bunch of other points somewhere." } { "Tag": [ "function", "complex analysis" ], "Problem": "What is the point of determining the differential rate law through experimentation when it is easier to determine the integrated rate law.\r\n\r\nFor $ A+B\\rightarrow C$, you would need at least 3 plots of concentration vs. time (normal, keep A constant, and keep B constant) to determine reaction orders. Then you can determine the rate constant and the actual rate law.\r\n\r\nHowever, you only need conc. vs time for 1 experiment to determine the integrated rate law.", "Solution_1": "Actually, I would not think it is so easy to get the integrated rate law out of experiment. For a reaction $ A+B\\rightarrow C$ where both $ A$ and $ B$ appear in the rate law, the integrated rate law is essentially a real-valued function of two variables. So at best you can find the equation of some curve on the surface defined by this function, but it is not possible to replicate the entire function from there.\r\n\r\nYou could probably go through a list of all possible rate laws and see if any are consistent with the data you obtained, but that is probably more work than doing 3 experiments (plus, I am not sure if you can really do this).\r\n\r\nAnd in any case, it is not always so easy to track the concentrations of all reactants and products. Unless you have some very high-level equipment, you will probably be doing this spectroscopically, in which case you probably only want one species to be absorbing. Keep in mind also that if you were able to keep track of all products and reactants, it is likely that intermediates would also absorb, which could throw things out of wack.\r\n\r\nCompared to these difficulties, I think the differential rate law sounds nice and easy to measure:).", "Solution_2": "Thanks, I see what you mean.\r\n\r\nSo I guess integrated rate law is easiest to find when there is only 1 reactant, like in $ A\\rightarrow B$?", "Solution_3": "Or if you know that the rate law depends only on one reactant. You could probably do it in one experiment this way, but again you would have to do curve fitting. If you have a computer, this might be easier, but I don't think you would have fun doing so by hand :).", "Solution_4": "[quote=\"gregx007\"]And in any case, it is not always so easy to track the concentrations of all reactants and products. Unless you have some very high-level equipment, you will probably be doing this spectroscopically, in which case you probably only want one species to be absorbing.[/quote]\r\n\r\nWhich reminds me, at IChO last year they had a lab problem where you did have to do spectroscopy of solutions with two dyes. It is possible to do, but unless you can make some approximations (as we got to do there) it can be a lot of grind work. And believe me, grind work is not fun on (competitive) labs!" } { "Tag": [ "inequalities", "LaTeX", "logarithms", "function", "inequalities proposed" ], "Problem": "Let $a,b,c >0$ and $a+b+c=1$ \r\n\r\nFind the maximum of\r\n$a^k(b+c)+b^k(c+a)+c^k(a+b)$\r\n\r\n$k$ is a positive real.\r\n\r\n------------------------------------------------------------------------------------------------\r\nThis problem I created last night and It take me all this morning to Find the solution!\r\nI hope It hasn't been post before!", "Solution_1": "$\\frac{b+c}{2(a+b+c)}+\\frac{c+a}{2(a+b+c)}+\\frac{a+b}{2(a+b+c)}=1$", "Solution_2": "What do you want to say,Kunny?", "Solution_3": "Who people can solve my Inequality?", "Solution_4": "I think we can use this ineq:\r\na^{k}b+b^{k}c+c^{k}a<= n^{n}/(n+1)^{n+1}.", "Solution_5": "I think we can use this ineq:\r\na^{k}b+b^{k}c+c^{k}a /lef n^{n}/(n+1)^{n+1}.", "Solution_6": "I don't think so.\r\nPlease,If you can,complete your answer!", "Solution_7": "I think the maximum is $\\frac{2}{3^k}$ - is it correct?", "Solution_8": "Sorry But my result isn't it! It's only special case.\r\n :( :(", "Solution_9": "Your problem is really interesting ,Hung.", "Solution_10": "my guess of answer: $max(\\frac{2}{3^k},\\frac{1}{2^k})$\r\n\r\nI think it's very similar to the fraction inequality.\r\nI used similar approach and i got stuck at the same thing again. I proved the case $k \\ge \\frac{2}{3}$ for fraction inequality and I prove the case $ k \\le \\frac{3}{2}$ for this one.", "Solution_11": "Ok,It's my result.But I think It's more difficult than The Fraction Inequality alot.\r\n :? :? \r\nPlease Explain your answer,Siuhochung.I don't think It can be solved in 2 cases of $k$.", "Solution_12": "[quote=\"hungkhtn\"]Ok,It's my result.But I think It's more difficult than The Fraction Inequality alot.\n :? :? \nPlease Explain your answer,Siuhochung.I don't think It can be solved in 2 cases of $k$.[/quote]\r\n\r\nnono, i dun hv any result yet. I can only prove the case $k \\leq \\frac{3}{2}$ for this inequality. (much weaker than the result of this question)", "Solution_13": "After 2 days of hard working ,i have found the solution \r\n\r\nIn my proof , i suppose that a<=b<=c .Then let a be a fixed number .Then we can prove that \r\nf ( a,b,c) <= f (a,(b+c)/2 ,(b+c)/2) = g(a).\r\nCase k<=1 and k>=2 we only need to work on g'(a) and it's easy.\r\nCase 1<=k<=2 , i worked on g'(a) and g''(a) .\r\n\r\nMy complete solution is long and i will write it when i have time ( and when i finish the Learning Latex course :D ) .", "Solution_14": "Suppose that you are right.What do you continue?Because the rest is already not easy,I think.\r\n :) :) \r\nI wait you post the solution after you finish Leaning Latex.", "Solution_15": "Hungkhtn,this is my proof:Because [tex]k\\geq\\ -1+1=0[/tex]\r\n[tex]a^k(b+c)+b^k(c+a)+c^k(a+b)\r\n=\\frac{a^k}{(b+c)^{-1}}+\\frac{b^k}{(a+c)^{-1}}+\\frac{c^k}{(a+b)^{-1}}\\leq\\frac{(a+b+c)^k}{3^{k+1-1}[2(a+b+c)]^{-1}}[/tex]\r\nHence:\r\n[tex]L.H.S\\leq\\frac{2(a+b+c)^{k+1}}{3^k}=\\frac{2}{3^k}[/tex]", "Solution_16": "Am I wrong", "Solution_17": "I think I have wrong result because this ineq is not easy.", "Solution_18": "Obvious,It's Wrong!", "Solution_19": "Can you post your solution,Hungkhtn?", "Solution_20": "By next 2 weeks!", "Solution_21": "I have the following results:\r\n\r\n(1) $\\sum{a^2(b+c) \\le \\frac{1}{4}(a+b+c)^3}$, with equality for $(a,b,c)=(0,1/2,1/2)$;\r\n(2) $\\sum{a^3(b+c) \\le \\frac{1}{8}(a+b+c)^4}$, with equality for $(a,b,c)=(0,1/2,1/2)$;\r\n\r\nAdditionally, I conjecture that\r\n(3) $\\sum{a^4(b+c) \\le \\frac{1}{12}(a+b+c)^5}$, with equality for $(a,b,c)=(0,\\frac{3-\\sqrt{3}}{6}, \\frac{3+\\sqrt{3}}{6})$.", "Solution_22": "[quote=\"hungkhtn\"]By next 2 weeks![/quote]\r\n\r\nhungkhtn: so now you can post your solution :)", "Solution_23": "Hungkhtn, I prove Siuhochung's result 2/(3^k)", "Solution_24": "1. 2/3^k is definitely wrong.\r\n2. Please clarify your last line of work.", "Solution_25": "We have 2 cases: \r\n(A) $0-1\r\n-0.2x+0.6y>-1\r\n-0.1x+0.05y+0.25z>-0.5\r\n\r\nThanks", "Solution_1": "[quote=\"claudiocamera\"]In order to solve an optimization problem I came across a inequality system, I think it is not dificult, but I dont remember how to solve this kind of systems, can you help me? The system is:\n\n0.4x -0.2y>-1\n-0.2x+0.6y>-1\n-0.1x+0.05y+0.25z>-0.5\n\nThanks[/quote]\r\n[hide]$0.4x-0.2y>-1;\\,-0.2x+0.6y>-1;\\,-0.1x+0.05y+0.25z>-0.5$\n\nFirst, make all coefficients integers: $2x-y>-5;\\,-x+3y>-5;\\,-2x+y+5z>-10$\n\nThe second inequality can be changed to $2x-y>-5+3x-4y.$ The LHS is the same as in the first inequality, so $-5=-5+3x-4y,$ or $3x=4y.$ Isolate one of the variables and substitute them into one of the first two inequalities. We find $x>-4$ and $y>-3.$\n\nI'm not sure how to find $z.$ :|[/hide]" } { "Tag": [ "number theory unsolved", "number theory" ], "Problem": "a) Prove that every multiple of 6 can be written as the sum of four third powers of integers.\r\nb) Prove that every integer can be written as the sum of five third powers of integers.", "Solution_1": "For (a), we have $6x^3=(x+1)^3+(x-1)^3-x^3-x^3$. Notice that (b) is an immediate corollary of (a) since that $6x+1=6x+1^3$, $6x \\pm 2 = 6(x \\mp 1) \\pm 8$, $6x+3=6(x-4)+27$.", "Solution_2": "... where for $6x+3 = 6(x-1)+9$ read $6(x-4) + 3^3$.", "Solution_3": "it says for EVERY multiple of 6 in a), which is clear nonsense...", "Solution_4": "Sorry for my mistake. It has been corrected." } { "Tag": [ "linear algebra", "matrix", "topology", "linear algebra unsolved" ], "Problem": "[b]Source: Homework[/b]\r\n\r\nShow that the matrices $M$ $\\in$ $M(n,n)$ on the form $M=$ $\\begin{pmatrix}A & B\\\\ C & D \\end{pmatrix}$ where $A$ $\\in$ $M_{k}(n,n)$ is invertible, make an open set $U$ $\\subseteq$ $M(n,n).$", "Solution_1": "The determinant of a matrix is continuous ..." } { "Tag": [ "algebra unsolved", "algebra" ], "Problem": "Prove that $tan^{2}1^{o}+tan^{2}3^{o}+tan^{2}5^{o}+...+tan^{2}85^{o}+tan^{2}87^{o}+tan^{2}89^{o}= 4005$.", "Solution_1": "Use the same method as in http://www.mathlinks.ro/Forum/viewtopic.php?t=63539 to find also the sum for those with $k$ even.", "Solution_2": "Reverse the sum, put 90-theta in for theta and it is the same sum" } { "Tag": [ "modular arithmetic", "function", "search", "geometry", "3D geometry", "combinatorics unsolved", "combinatorics" ], "Problem": "Define an overpartition of $ n$ to be a partition of $ n$ in which that last occurrence of a part may be overlined. For example, the overpartitions of $ 3$ are $ 3, \\overline{3}, 2 \\plus{} 1, \\overline{2} \\plus{} 1, 2 \\plus{} \\overline{1}, \\overline{2} \\plus{} \\overline{1}, 1 \\plus{} 1 \\plus{} 1, 1 \\plus{} 1 \\plus{} \\overline{1}$. Define the rank of an overpartition to be the largest part minus the number of parts. Let $ p_o(n)$ denote the number of overpartitions of odd rank and $ p_e(n)$ denote the number of overpartitions of even rank. Prove that, if $ n \\equiv 3 \\pmod{8}$, then \r\n\\[ (\\minus{}1)^n (p_e(n)\\minus{}p_o(n)) \\equal{} r_3(n)\r\n\\]\r\nwhere $ r_3(n)$ is the number of representations of $ n$ as a sum of three squares, taking order and sign into account (so $ 1^2 \\plus{} 1^2 \\plus{} 1^2$ is different from $ (\\minus{}1)^2 \\plus{} 1^2 \\plus{} 1^2$).\r\n\r\nAny ideas for approaches are very welcome. Thanks.", "Solution_1": "Looks generating-functional to me. The GF for $ r_3(n)$ is easy enough to write down (and is related to some known number-theoretic generating functions, I believe -- this all looks very Ramanujan-y), as is (I think) the generating function for overpartitions. And to modify this to take rank into account shouldn't be too hard. \r\n\r\nAs is obvious, I'm not thinking very long about this, but have you tried something like this? (What context did this come up in? Putting \"overpartition\" into my search engine brings up a lot of interesting-looking stuff, though I don't know if it's directly relevant. Putting it into google scholar might bring up more.)", "Solution_2": "The $ r_3(n)$ generating function is the cube of a certain [url=http://en.wikipedia.org/wiki/Theta_function#Jacobi_theta_function]theta function[/url], so I think the Jacobi triple product will be relevant, and I'm reasonably certain it can't be too hard to write down the generating function for the overpartitions of rank $ k$. (Then substitute $ \\minus{}1$ into the rank-weighting variable.)", "Solution_3": "This is a slightly altered form of the case $ n \\equiv 3 \\pmod{8}$ of a theorem that says:\r\n\\[ ( - 1)^n (p_e(n) - p_o(n)) = - 16H(n) - 1/3 r_3(n)\r\n\\]\r\nwhere $ H(n)$ denotes the Hurwitz Class Number of $ n$. I'm interested in proving the whole theorem, but since this case was so nice in appearance I thought there might be a slick combinatorial method (i.e. some sort of bijection from even overpartitions to odd ones that is not defined on a subset that can be corresponded with sums of squares). I've done some work with generating functions, but it seems difficult to encode the condition $ n \\equiv 3 \\pmod{8}$, and the general case is difficult because I know of no generating function for $ H(n)$ (that is simple). I'll post my work so far when I can.\r\n\r\nEDIT:\r\n\r\nHere it is. Let $ p_{k, m}(n)$ denote the number of overpartitions of $ n$ with $ k$ parts and rank $ m$. Then I claim that\r\n\\[ \\sum_{k, m, n} p_{k, m}(n) z^k r^ m q^n = \\frac {(1 + 1) ( - rq; q)_{k - 1} (zq)^k}{(rq; q)_k} r^{1 - k}\r\n\\]\r\nThe RHS can be explained as follows: $ (zq)^k$ represents $ k$ ones as the starting base for the partition. Then\r\n\\[ \\frac {1}{(rq; q)_k} = \\frac {1}{(1 - rq)(1 - rq^2)\\ldots (1 - rq^k)} = (1 + rq + r^2q^2 + ...)(1 + rq^2 + r^2q^4 + ...) ... (1 + rq^k + r^2q^{2k\r\n}\\]\r\nIf we choose $ rq^{\\alpha i}$ from the $ i$th bracket in the product, then we increment the first $ i$ terms of the partition by $ 1$. It is easy to see that this increases the rank by $ \\alpha$, hence the coefficient of $ r^{\\alpha}$. Furthermore, this process constructs a unique partition, and any partition can be uniquely decomposed into such steps of increments. \r\n\r\nNow, the term $ (1 + 1)( - rq;q)_{k - 1} = (1 + 1)(1 + rq)(1 + rq^2)\\ldots (1 + rq^{k - 1})$ generates a \"partition\" with distinct parts but allowing $ 0$ as a part. \r\n\r\nThus, the generating function generates two partitions: one ordinary one, and one with distinct nonnegative parts (by the way, this sort of \"direct product\" of partitions is the substance of Frobenius partitions), whose parts together add to $ n$. We establish a bijection between these and overpartitions as follows: for each part i$ \\lambda$ n the second partition (with distinct nonnegative parts), we augment the first $ \\lambda$ parts of the first partition and overline the $ \\lambda + 1$st part.\r\n\r\nSo, let $ p_m(n)$ denote the number of overpartitions of $ n$ with rank $ m$. Then clearly\r\n\\[ \\sum_{m, n} p_m(n) = \\sum_{k = 1}^{\\infty} \\sum_{k, m, n} p_{k, m}(n) z^k r^ m q^n = \\sum_{k = 1}^{\\infty} \\frac {(1 + 1) ( - rq; q)_{k - 1} (zq)^k}{(rq; q)_k} r^{1 - k}\r\n\\]\r\nIf we set $ z = 1$ and $ r = - 1$, then we get the generating function for the values in question, $ p_e(n) - p_o(n)$. \r\nThe latter looks similar to the LHS of Ramanujan's $ _1 \\psi_1$ formula, but the sum isn't bilateral. I'm looking for any identities that might help compute such a sum, as well as any identities relating to $ \\theta(q)^3$ (the generating function for sums of three squares). \r\n\r\nThe above discussion implies that the following is a generating function for partitions:\r\n\\[ \\sum_{k = 1}^{\\infty} \\frac {(q)^k}{(q; q)_k}\r\n\\]\r\nbut the product form is more well known:\r\n\\[ \\prod_{k = 1}^{\\infty} \\frac {1}{1 - q^k}\r\n\\]\r\nI suspect that there should be an alternate generating function for the rank difference. Can anybody come up with one? \r\n\r\nFinally, there is an analogous theorem for a different definition of the rank called the M2-rank. Let $ l$ be the largest part, $ m$ be the number of parts, $ m'$ be the number of overlined odd parts, and $ p = 1$ if the largest part is odd and non-overlined and $ 0$ otherwise. The M2 rank is $ l - m + m' - p$. Then\r\n\\[ ( - 1)^n (p_e(n) - p_o(n)) = - 8H(n) + 1/3 r_3(n)\r\n\\]\r\nA notable case is that if $ n \\equiv 3 \\pmod(8)$, then the number of overpartitions with even M2-rank minus the number of overpartitions with odd M2-rank is $ 0$. I wonder if there is a nice bijection that proves this." } { "Tag": [ "geometry", "search", "geometry unsolved" ], "Problem": "The incircle of triangle $ ABC$ is tangent to the sides $ BC,CA,AB$ at $ D,E,F$ respectively.\r\nSuppoe the incircle intersects $ AD$ again at $ X$ such that $ AX\\equal{}XD,XB$ and $ XC$ \r\nintersects the incircle again at points $ Y$ and $ Z$,respectively.Prove that $ EY\\equal{}FZ$\r\n\r\nPoon.", "Solution_1": "It is from Iberoamerican MO 1995 . See here for a solution http://www.mathlinks.ro/viewtopic.php?search_id=952312459&t=4954", "Solution_2": ":D :roll: Thank you" } { "Tag": [ "trigonometry", "logarithms", "floor function", "calculus", "calculus computations" ], "Problem": "This isn't too difficult, but someone asked me this in my office today, so why not?\r\n\r\nConsider $\\sum_{n=1}^{\\infty}\\frac{(-1)^{n}\\sin n}n$\r\n\r\nShow that the sum converges conditionally.\r\n\r\nCompute its exact value.", "Solution_1": "generally : :wink: \r\n$\\sum_{n=1}^{\\infty}\\frac{(-1)^{n}\\sin nx}n$\r\neasy we only find this sum easy \r\n$\\sum\\frac{1}{n}z^{n}$ where $z\\in C$ I mean complex number. and $|z|<1$. after use Abelia theorem", "Solution_2": "We have, by Fourier analysis, that:\r\n\r\n$x=2\\sum_{n=1}^{\\infty}(-1)^{n-1}\\frac{\\sin nx}{n}$, $-\\pi\\;0 \\; )$\r\n\r\nhence, the 2nd series will diverge.\r\n\r\nnow, for the 1st one, use dirichlet's test, i.e. let $a_{n}$ be terms of a sequence \r\nthat is monotonically convergent to 0, and let $b_{n}$ be terms of a sequence \r\nwhose partial sums, i.e. $\\sum_{n}^{p}\\;b_{n}\\;( \\; where\\; p\\; \\ll \\; \\infty )$, are bounded above by \r\nsome constant $M\\; \\ll \\; \\infty\\; ( \\; \\Rightarrow\\;Cauchy\\; series )$ ; \r\nit then follows $\\;\\sum_{n}\\; a_{n}\\; \\cdot \\; b_{n}\\;$ is convergent. \r\n[ PS: one can also use abel's test here. ]\r\n\r\nthis concludes \r\n\r\n$\\sum_{n}\\; | \\;(-1)^{n}\\;\\frac{\\sin{n}}{n}\\; |$ diverges.\r\n\r\n(ii) to show $\\sum_{n}\\;\\mathbb{a}_{n}\\;$ converges, we use a similar argument as above,\r\nbut this time using leibnitz's alternating series test, where\r\nthe partial sums of $\\sum_{n}^{p}\\;(-1)^{n}\\;\\sin{n}$ are bounded and $\\frac{1}{n}$ monotonically approaches 0.\r\n\r\nas for the sum, didi already showed (using fourier series):\r\n\r\n$\\sum_{n=1}^{\\infty}\\; (-1)^{n}\\;\\frac{\\sin{n}}{n}\\; =\\;-\\frac{1}{2}$\r\n\r\n(i used a different method, but i guess there is no need to post it anymore...)", "Solution_4": "In general we have:\r\n\r\n$\\sum_{n=1}^{\\infty}(-1)^{n-1}\\frac{\\sin(nx)}{n}= \\frac{x}{2}$." } { "Tag": [ "geometry", "geometry proposed" ], "Problem": "Let $ABCDE$ be a convex pentagon and let $F = BC \\cap DE, G = CD \\cap EA, H = DE \\cap AB, I = EA \\cap BC, J = AB \\cap CD$. Suppose that the areas of the triangles $AHI,BIJ,CJF,DFG,EGH$ are all equal. Then the lines $AF,BG,CH,DI,EJ$ are all concurrent.", "Solution_1": "See here: [url]http://www.mathlinks.ro/viewtopic.php?t=149421[/url]" } { "Tag": [ "inequalities", "inequalities proposed" ], "Problem": "Let $a,b,c >0$ such that their sum is $3.$ Prove that the following inequality holds:\r\n\r\n \\[ \\frac{a^{3}+abc}{(b+c)^{2}}+\\frac{b^{3}+abc}{(c+a)^{2}}+\\frac{c^{3}+abc}{(a+b)^{2}}\\geq\\frac{3}{2}. \\]", "Solution_1": "Cute. Assume $a\\geq b\\geq c$.\r\n\r\nBy Schur's inequality,\r\n\r\n$\\sum \\left(\\frac{a}{(b+c)^2}\\right)(a-b)(a-c)\\geq 0$\r\n\r\nIt follows that\r\n\r\n$LHS\\geq \\sum \\frac{a^2(b+c)}{(b+c)^2}=\\sum \\frac{a^2}{b+c}\\geq \\frac{a+b+c}{2}=\\frac{3}{2}$.", "Solution_2": "You mean Vornicu-Schur. :)", "Solution_3": "Sure. It's a nice problem, in any event.", "Solution_4": "Thx. I'm really happy you appreciate it. :lol:", "Solution_5": "Another proof:\r\n\r\nBy Chebyshev, then Iran 1996, then Schur\r\n\r\n\\begin{eqnarray*} LHS&\\geq&\\frac{1}{3}\\left(\\sum x^3+xyz\\right)\\left(\\sum \\frac{1}{(x+y)^2}\\right) \\\\ &\\geq& \\frac{3}{4(xy+yz+zx)}\\left(\\sum x^3+xyz\\right) \\\\ &\\geq& \\frac{1}{2}\\frac{(xy+yz+zx)(x+y+z)}{xy+yz+zx}\\\\ &=&\\frac{x+y+z}{2} \\\\ &=&\\frac{3}{2} \\end{eqnarray*}\r\n\r\nwhich yields the desired result.", "Solution_6": ":D I found a very simple way for this one. But I am not sure it is correct or not??\r\nWe know that $a^2+b^2+c^2 \\geq (ab+bc+ca)$ so $a^2+bc \\geq a(b+c)+(b-c)^2$\r\nThen we get $a^3+abc \\geq a^2(b+c)+a(b-c)^2$\r\nSo LHS $\\geq \\sum \\frac{a^2}{b+c} +\\sum a{(\\frac{b-c}{b+c})^2}$\r\n But $\\sum \\frac{a^2}{b+c} \\geq \\frac32$ so we are done! :rotfl:", "Solution_7": "[quote=\"Gibbenergy\"]:D I found a very simple way for this one. But I am not sure it is correct or not??\nWe know that $a^2+b^2+c^2 \\geq (ab+bc+ca)$ so $a^2+bc \\geq a(b+c)+(b-c)^2$\n[/quote]\r\nTHe above should be $a^2+bc \\geq a(b+c)-(b-c)^2$", "Solution_8": "Terrible sorry!\r\n I make a very silly mistake. But anyway, after long computation, I can prove it but It is very ugly(brutal force). And that is very interesting that I have found out \r\n$\\sum \\frac{a^3(2a^2+bc)}{(b+c)^2} \\geq \\frac94$\r\n But I am not sure if it is correctly or not since I sometimes make stupid mistakes!", "Solution_9": "We can do without Iran 1996 as follows: by Chebyshev and CBS,\r\n$\\sum_{cycl}\\frac{a^{3}+abc}{(b+c)^{2}}\\ge\\frac{a^3+b^3+c^3+3abc}{3}\\sum_{cycl}\\frac{1}{(b+c)^{2}} \\ge\\frac{3(a^3+b^3+c^3+3abc)}{2(a^2+b^2+c^2+ab+bc+ca)}$\r\nand note that using Schur we have\r\n$(ab+bc+ca)(a+b+c) \\le a^3+b^3+c^3+6abc$\r\nand\r\n$(a^2+b^2+c^2)(a+b+c)=a^3+b^3+c^3+a^2(b+c)+b^2(c+a)+c^2(a+b) \\le 2(a^3+b^3+c^3)+3abc$\r\nso that\r\n$(a+b+c)(a^2+b^2+c^2+ab+bc+ca) \\le 3(a^3+b^3+c^3+3abc)$", "Solution_10": "If $ a,b,c$ be nonnegative numbers such that $ bc + ca + ab > 0$, then when $ k\\leq3$ we have\r\n\r\n$ \\frac {a^3 + kabc}{(b + c)^2} + \\frac {b^3 + kabc}{(c + a)^2} + \\frac {c^3 + kabc}{(a + b)^2}\\geq\\frac {1 + k}{4}(a + b + c),$\r\n\r\nwith equality if $ k = 3,a = 0,b = c > 0.$\r\n\r\n[b]Proof[/b] $ \\sum{\\frac {a^3 + kabc}{(b + c)^2} - \\frac {1 + k}{4}\\sum{a}}$\r\n\r\n$ = \\sum{\\frac {(b - c)^2\\left[(b + c - a)^2(5a + 3b + 3c) + bc(17a + 4b + 4c)\\right]}{6(c + a)^2(a + b)^2}}$\r\n\r\n$ + \\frac {3 - k}{4}\\sum{\\frac {a(b - c)^2}{(b + c)^2}}\\geq0.$\r\n\r\n[b]Remark[/b] When $ - 1\\leq k\\leq 3$, it is stronger than another Cezar Lupu's inequality, see : \r\n\r\nhttp://www.mathlinks.ro/viewtopic.php?t=61944" } { "Tag": [ "inequalities", "inequalities proposed" ], "Problem": "Let $ a,b,c$ positive reals such that $ ab\\plus{}bc\\plus{}ca\\equal{}1$. Prove that\r\n\r\n$ \\sum\\frac {\\sqrt {a \\plus{} b \\plus{} c} \\plus{} \\sqrt {a \\plus{} b} \\plus{} \\sqrt {c}}{a \\plus{} b}\\geq\\frac {2 \\plus{} \\sqrt {2}}{2} \\plus{} \\frac {3(3 \\plus{} \\sqrt {3})}{2\\sqrt {a \\plus{} b \\plus{} c}}$", "Solution_1": "[quote=\"Ligouras\"]Let $ a,b,c$ positive reals. Prove that\n\n$ \\sum\\frac {\\sqrt {a \\plus{} b \\plus{} c} \\plus{} \\sqrt {a \\plus{} b} \\plus{} \\sqrt {c}}{a \\plus{} b}\\geq\\frac {2 \\plus{} \\sqrt {2}}{2} \\plus{} \\frac {3(3 \\plus{} \\sqrt {3})}{2\\sqrt {a \\plus{} b \\plus{} c}}$[/quote]\r\nThe inequality is not homogenous ! Try $ a\\equal{}b\\equal{}c \\geq \\frac{ 9(1\\plus{}\\sqrt{2}\\plus{}\\sqrt{3})^2 }{ (2\\plus{}\\sqrt{2})^2 }$", "Solution_2": "[quote=\"rachid\"][quote=\"Ligouras\"]Let $ a,b,c$ positive reals. Prove that\n\n$ \\sum\\frac {\\sqrt {a \\plus{} b \\plus{} c} \\plus{} \\sqrt {a \\plus{} b} \\plus{} \\sqrt {c}}{a \\plus{} b}\\geq\\frac {2 \\plus{} \\sqrt {2}}{2} \\plus{} \\frac {3(3 \\plus{} \\sqrt {3})}{2\\sqrt {a \\plus{} b \\plus{} c}}$[/quote]\nThe inequality is not homogenous ! Try $ a \\equal{} b \\equal{} c \\geq \\frac { 9(1 \\plus{} \\sqrt {2} \\plus{} \\sqrt {3})^2 }{ (2 \\plus{} \\sqrt {2})^2 }$[/quote]\r\n\r\nMy Friend\r\n\r\n$ ab \\plus{} bc \\plus{} ca \\equal{} 1$\r\n\r\nsorry" } { "Tag": [ "geometry", "trapezoid", "geometry unsolved" ], "Problem": "Let's take the trapezoid $\\ ABCD$ whith $\\ AB\\parallel CD$, $\\angle A=90, \\angle C=2\\angle B$ and $\\ AB=3CD$.\r\n$\\ a)$ Find the points $\\ M,N\\in (AD)$ such that $\\angle DMC=\\angle AMB$ and $\\angle DNC=\\angle ABN$.\r\n$\\ b)$ If we know $\\ MN=3 cm$, find the area of $\\ ABCD$.\r\n$\\text{Constantin Apostol}$", "Solution_1": "[quote=\"stancioiu sorin\"]Let's take the trapezoid $\\ ABCD$ whith $\\ AB\\parallel CD$, $\\angle A=90, \\angle C=2\\angle B$ and $\\ AB=3CD$.\n$\\ a)$ Find the points $\\ M,N\\in (AD)$ such that $\\angle DMC=\\angle AMB$ and $\\angle DNC=\\angle ABN$.\n$\\ b)$ If we know $\\ MN=3 cm$, find the area of $\\ ABCD$.\n$\\text{Constantin Apostol}$[/quote]\r\nMay you mean $\\ MN=3 CM$ :maybe:", "Solution_2": "I guess it mean \"centimeter\". So we can use it to find the area numericaly.", "Solution_3": ":blush: Thank you,I see.\r\nIt isn't hard problem,we just need to show that $AN=DN$ and $AM=3DM$,\r\nafter this it is trivial to find the area.", "Solution_4": "Ok, let me finish this problem.\r\n\r\nThe construction of $M$: $S(AD)$, then $C \\to C'$, let $M = C'B \\cap AD$, then $\\angle{CMD}=\\angle{BMA}$.\r\n\r\nThe construction of $N$: As Tiks put it, $N$ is the midpoint of $AD$. This is because $AN=DN= \\sqrt{3}DC=\\sqrt{3}a$, thus $\\frac{ND}{CD}=\\sqrt{3}=\\frac{AB}{AN}$, hence $\\triangle{CND}\\sim \\triangle{NBA}$, hence $\\angle{CND}=\\angle{NBA}$.\r\n\r\nBecause $\\triangle{DMC}\\sim \\triangle{AMB}$, so $AM=3DM$, thus $AD=4MN=12$ which implies $a=2\\sqrt{3}$. The area of the trapzoid is $48\\sqrt{3}$." } { "Tag": [ "topology" ], "Problem": "Let D be the decomposition of the plane into concentric circles about the origin. Prove that D is homeomorphic to ${ \\{x \\in \\mathbb{R}: x \\geq 0}$. Show also that D is upper semicontinuous.", "Solution_1": "For part 1, send each circle to its radius in $ \\mathbb{R}$. For part 2, I don't know what semi continuous means", "Solution_2": "It's true you can carry the quotient topology on to the quotient $ Y$ by $ X/Y$, but in that case $ D$ would be endowed with the quotient topology and we won't get $ X\\equal{}D\\times Y$ that easily since the product topology is different from the multiplied quotient topology." } { "Tag": [ "geometry", "perpendicular bisector", "angle bisector", "Law of Sines" ], "Problem": "In convex quadrilateral $ ABCD$, $ CB,DA$ are external angle bisectors of $ \\angle DCA,\\angle CDB$, respectively. Points $ E,F$ lie on the rays $ AC,BD$ respectively such that $ CEFD$ is cyclic quadrilateral. Point $ P$ lie in the plane of quadrilateral $ ABCD$ such that $ DA,CB$ are external angle bisectors of $ \\angle PDE,\\angle PCF$ respectively. $ AD$ intersects $ BC$ at $ Q.$ Prove that $ P$ lies on $ AB$ if and only if $ Q$ lies on segment $ EF$.", "Solution_1": "Hi, my name is dr. Ilham Kosasih, I am from Indonesia, if i have a questions about Math, can i ask u?? :) , would u pliz help me?? :lol: btw, my email is masterpool4ever@yahoo.com, i love math.", "Solution_2": "Seems that \n\n \n\n\n M.T.", "Solution_3": "Hello!\r\n\r\na) $ Q\\in EF$, $ \\left\\{P_{1}\\right\\}\\equal{}CP\\cap AB$ and $ \\left\\{P_{2}\\right\\}\\equal{}DP\\cap AB$.\r\n\r\n$ \\frac{P_{1}A}{P_{1}B}\\equal{}\\frac{CA\\cdot sin\\left(\\angle ACP_{1}\\right)}{CB\\cdot sin\\left(\\angle BCP_{1}\\right)}\\equal{}\\frac{CA}{CB}\\cdot\\frac{sin\\left(\\angle DCF\\right)}{sin\\left(\\angle QCF\\right)}\\equal{}$\r\n$ \\equal{}\\frac{CA}{CB}\\cdot\\frac{DF\\cdot sin\\left(\\angle CDF\\right)}{QF\\cdot sin\\left(\\angle CQF\\right)}\\equal{}\\frac{CA}{CB}\\cdot\\frac{DF}{QF}\\cdot\\frac{sin\\left(\\angle CEF\\right)}{sin\\left(\\angle CQE\\right)}\\equal{}$\r\n$ \\equal{}\\frac{CA}{CB}\\cdot\\frac{DF}{QF}\\cdot\\frac{CQ}{CE}$ (1)\r\n\r\n$ \\frac{P_{2}A}{P_{2}B}\\equal{}\\frac{DA\\cdot sin\\left(\\angle ADP_{2}\\right)}{DB\\cdot sin\\left(\\angle BDP_{2}\\right)}\\equal{}\\frac{DA}{DB}\\cdot\\frac{sin\\left(\\angle QDE\\right)}{sin\\left(\\angle CDE\\right)}\\equal{}$\r\n$ \\equal{}\\frac{DA}{DB}\\cdot\\frac{QE\\cdot sin\\left(\\angle DQE\\right)}{CE\\cdot sin\\left(\\angle DCE\\right)}\\equal{}\\frac{DA}{DB}\\cdot\\frac{QE}{CE}\\cdot\\frac{sin\\left(\\angle DQF\\right)}{sin\\left(\\angle DFE\\right)}\\equal{}$\r\n$ \\equal{}\\frac{DA}{DB}\\cdot\\frac{QE}{CE}\\cdot\\frac{DF}{DQ}$ (2)\r\n\r\n(1), (2) $ \\Rightarrow\\frac{P_{1}A}{P_{1}B}\\equal{}\\frac{P_{2}A}{P_{2}B}\\Leftrightarrow \\frac{CA}{CB}\\cdot\\frac{DF}{QF}\\cdot\\frac{CQ}{CE}\\equal{}\\frac{DA}{DB}\\cdot\\frac{QE}{CE}\\cdot\\frac{DF}{DQ}\\Leftrightarrow$\r\n$ \\Leftrightarrow\\frac{QC}{QE}\\cdot\\frac{QD}{QF}\\equal{}\\frac{CB}{DB}\\cdot\\frac{DA}{CA}\\Leftrightarrow$\r\n$ \\Leftrightarrow\\frac{sin\\left(\\angle QEC\\right)}{sin\\left(\\angle QCE\\right)}\\cdot\\frac{sin\\left(\\angle QFD\\right)}{sin\\left(\\angle QDF\\right)}\\equal{}\\frac{sin\\left(\\angle CDB\\right)}{sin\\left(\\angle DCB\\right)}\\cdot\\frac{sin\\left(\\angle DCA\\right)}{sin\\left(\\angle CDA\\right)}\\Leftrightarrow$\r\n$ \\Leftrightarrow\\frac{sin\\left(\\angle QCD\\right)}{sin\\left(\\angle QCE\\right)}\\cdot\\frac{sin\\left(\\angle QDC\\right)}{sin\\left(\\angle QDF\\right)}\\equal{}1$. \r\nThe last relation is true.\r\nSo $ P_{1}\\equal{}P_{2}\\equal{}P\\Rightarrow P\\in AB$.\r\n\r\nb) $ P\\in AB$, $ \\left\\{Q_{1}\\right\\}\\equal{}BC\\cap EF$ and $ \\left\\{Q_{2}\\right\\}\\equal{}AD\\cap EF$.\r\n\r\n$ \\frac{Q_{1}E}{Q_{1}F}\\equal{}\\frac{\\frac{Q_{1}C\\cdot sin\\left(\\angle Q_{1}CE\\right)}{sin\\left(\\angle Q_{1}EC\\right)}}{\\frac{Q_{1}C\\cdot sin\\left(\\angle Q_{1}CF\\right)}{sin\\left(\\angle Q_{1}FC\\right)}}\\equal{}\\frac{sin\\left(\\angle Q_{1}CE\\right)}{sin\\left(\\angle Q_{1}EC\\right)}\\cdot\\frac{sin\\left(\\angle Q_{1}FC\\right)}{sin\\left(\\angle Q_{1}CF\\right)}\\equal{}$\r\n$ \\equal{}\\frac{sin\\left(\\angle Q_{1}CD\\right)}{sin\\left(\\angle CDB\\right)}\\cdot\\frac{sin\\left(\\angle CDE\\right)}{sin\\left(\\angle BCP\\right)}\\equal{}\\frac{sin\\left(\\angle BCD\\right)}{sin\\left(\\angle CDB\\right)}\\cdot\\frac{sin\\left(\\angle BDP\\right)}{sin\\left(\\angle BCP\\right)}\\equal{}$\r\n$ \\equal{}\\frac{BD}{BC}\\cdot\\frac{sin\\left(\\angle BDP\\right)}{sin\\left(\\angle BCP\\right)}\\equal{}\\frac{BP\\cdot sin\\left(\\angle BPD\\right)}{BP\\cdot sin\\left(\\angle BPC\\right)}\\equal{}\\frac{sin\\left(\\angle BPD\\right)}{sin\\left(\\angle BPC\\right)}$ (3) \r\n\r\n$ \\frac{Q_{2}E}{Q_{2}F}\\equal{}\\frac{\\frac{Q_{2}D\\cdot sin\\left(\\angle Q_{2}DE\\right)}{sin\\left(\\angle Q_{2}ED\\right)}}{\\frac{Q_{2}D\\cdot sin\\left(\\angle Q_{2}DF\\right)}{sin\\left(\\angle Q_{2}FD\\right)}}\\equal{}\\frac{sin\\left(\\angle Q_{2}DE\\right)}{sin\\left(\\angle Q_{2}ED\\right)}\\cdot\\frac{sin\\left(\\angle Q_{2}FD\\right)}{sin\\left(\\angle Q_{2}DF\\right)}\\equal{}$\r\n$ \\equal{}\\frac{sin\\left(\\angle ADP\\right)}{sin\\left(\\angle DCF\\right)}\\cdot\\frac{sin\\left(\\angle DCA\\right)}{sin\\left(\\angle Q_{2}DC\\right)}\\equal{}\\frac{sin\\left(\\angle ADP\\right)}{sin\\left(\\angle ACP\\right)}\\cdot\\frac{sin\\left(\\angle DCA\\right)}{sin\\left(\\angle CDA\\right)}\\equal{}$\r\n$ \\equal{}\\frac{sin\\left(\\angle ADP\\right)}{sin\\left(\\angle ACP\\right)}\\cdot\\frac{AD}{AC}\\equal{}\\frac{AP\\cdot sin\\left(\\angle APD\\right)}{AP\\cdot sin\\left(\\angle APC\\right)}\\equal{}\\frac{sin\\left(\\angle APD\\right)}{sin\\left(\\angle APC\\right)}$ (4)\r\n\r\n(3), (4) $ \\Rightarrow\\frac{Q_{1}E}{Q_{1}F}\\equal{}\\frac{Q_{2}E}{Q_{2}F}\\Leftrightarrow\\frac{sin\\left(\\angle BPD\\right)}{sin\\left(\\angle BPC\\right)}\\equal{}\\frac{sin\\left(\\angle APD\\right)}{sin\\left(\\angle APC\\right)}$. \r\nThe last relation is true. \r\nSo $ Q_{1}\\equal{}Q_{2}\\equal{}Q\\Rightarrow Q\\in EF$.\r\n\r\nBest regards, Petrisor Neagoe :)", "Solution_4": "[quote=\"Petry\"]b) $ P\\in AB$, $ \\left\\{Q_{1}\\right\\}=BC\\cap EF$ and $ \\left\\{Q_{2}\\right\\}=AD\\cap EF$.[/quote]Nice proof! Just wanted to point out that once you've proven (a), you can just prove that $P\\in AB$ must be unique (for instance, by noting $\\angle{PDC}$ and $\\angle{PCD}$ increase with $EF$), so no need to go through all the law of sines again for (b). :P \n\n---\n\nHere's another solution. Let $T=DB\\cap CA$ so that $Q$ is the $T$-excenter of $\\triangle{TDC}$. The convexity of $ABCD$ yields $T>C,D$.\n\nWe are given that $P$ exists, so because $DA,CB$ are external bisectors of $\\angle{PDE},\\angle{PCF}$, $Q,T$ must lie on opposite sides of $DE$ and $CF$.\n\nLet $\\theta = \\angle{TDE}$. Since $DCEF$ is cyclic, $\\angle{PDC}=\\angle{TDE}=\\theta=\\angle{TCF}=\\angle{PCD}$. Hence $P$ varies (with $\\theta<90^\\circ$) along the perpendicular bisector $\\ell$ of $DC$, so $P\\in AB$ for exactly one value of $\\theta$ (say $\\alpha$). Similarly, we always have $\\angle{TFE}=C$ and $\\angle{TEF}=D$, so $Q\\in EF$ for a unique value of $\\theta$ (say $\\beta$). It clearly suffices to show that $\\alpha=\\beta$.\n\nFirst suppose $\\theta=\\alpha$, so $FE=r_T(\\csc{C}+\\csc{D}) = 2[TCD](\\csc{C}+\\csc{D})/(c+d-t)=t(d+c)/(c+d-t)$ (by the sine area formula). Then\n\\[ \\frac{FE}{\\sin\\theta} = \\frac{FE}{\\sin{FDE}}=\\frac{CD}{\\sin{CFD}} = \\frac{t}{\\sin(T+\\theta)} \\implies \\frac{\\sin T}{\\tan\\alpha}+\\cos{T} = \\frac{c+d-t}{c+d}, \\]where we use $\\sin(T+\\theta)=\\sin{T}\\cos\\theta+\\cos{T}\\sin\\theta$.\n\nNow suppose $\\theta=\\beta$, and let $A',P',B'$ be the feet from $A,P,B$ to $DC$. Then\n\\[ \\frac{t}{2}\\tan\\beta = DP'\\tan\\beta = PP' = \\frac{2[PA'B']}{A'B'} = \\frac{2[P'AB]}{A'B'} = \\frac{[DAB]+[CAB]}{TA\\cos{C}+TB\\cos{D}}. \\]By the angle bisector theorem, $\\frac{TA}{TC}=\\frac{TA}{CA-TA}=\\frac{TD}{CD-TD}$, so $TA = \\frac{dc}{t-c}$ and $CA = TA+d=\\frac{td}{t-c}$. Similarly, $TB = \\frac{cd}{t-d}$ and $DB = \\frac{tc}{t-d}$, so\n\\begin{align*}\n\\tan\\beta = \\frac{2}{t}\\frac{[DAB]+[CAB]}{TA\\cos{C}+TB\\cos{D}}\n&= \\frac{(t-c)(t-d)\\sin{T}}{tcd}\\frac{DB\\cdot TA+CA\\cdot TB}{TA\\cos{C}+TB\\cos{D}} \\\\\n&= \\frac{(t-c)(t-d)\\sin{T}}{tcd}\\frac{\\frac{tcd}{(t-c)(t-d)}(c+d)}{(t-d)\\cos{C}+(t-c)\\cos{D}} \\\\\n&= \\frac{(c+d)\\sin{T}}{t(\\cos{C}+\\cos{D}) - t}.\n\\end{align*}Thus\n\\begin{align*}\n\\frac{\\sin{T}}{\\tan\\beta}+\\frac{t}{c+d}\n&= \\frac{\\sin{T}\\cos{C}+\\sin{T}\\cos{D}}{\\sin{C}+\\sin{D}} \\\\\n&= \\frac{\\sin(T+C)-\\cos{T}\\sin{C}+\\sin(T+D)-\\cos{T}\\sin{D}}{\\sin{C}+\\sin{D}} = 1-\\cos{T} = \\frac{\\sin{T}}{\\tan\\alpha}+\\frac{t}{c+d},\n\\end{align*}so we're done (as $\\theta$ must be acute in order for $P$ to exist).", "Solution_5": "This can be done using harmonic range of points. And i wonder if the condition \"$CEFD$ is concyclic\" can be deleted?", "Solution_6": "[quote=P-H-David-Clarence]This can be done using harmonic range of points. And i wonder if the condition \"$CEFD$ is concyclic\" can be deleted?[/quote]\n\nYou are right, the condition \"$CEFD$ is cyclic\" can be deleted.\nHere's a short proof using barycentric coordinates.\n\nLet $AC \\cap BD = X$, and $X=(1,0,0), C=(0,1,0), D=(0,0,1)$, $a=CD, b=DX, c=XD$\n$Q$ is $X$- excenter of triangle $XCD$, so $Q=(-a:b:c)$\n$E$ is on $XC$, so let $E=(t, 1-t, 0)$ for some real number $t$, and similary let $F=(u, 0, 1-u)$ for some real number $u$\n$P' = CF \\cap DE$, so $P'=(ut:u(1-t):t(1-u))$\n$P$ is isogonal conjugate of $P'$, so $P=(a^2 : \\frac{t}{1-t} b^2 : \\frac{u}{1-u} c^2)$. ($u, t$ are not $0, 1$)\n$A = DQ \\cap XC$, so $A=(-a:b:0)$, and similary $B=(-a:0:c)$\n\n$P$ is on $AB \\iff \\det\\begin{pmatrix} -a & b & 0 \\\\ -a & 0 & c \\\\ a^2 & \\frac{t}{1-t} b^2 & \\frac{u}{1-u} c^2 \\end{pmatrix}=0 \\iff a+ b \\frac{t}{1-t} + c \\frac{u}{1-u} =0$\n$Q$ is on $EF $\n$\\iff \\det \\begin{pmatrix} -a & b & c \\\\ t & 1-t & 0 \\\\ u & 0 & 1-u \\end{pmatrix} =0 \\iff -a(1-t)(1-u)-bt(1-u)-cu(1-t)=0 \\iff a+ b \\frac{t}{1-t} + c \\frac{u}{1-u} =0$\n\nSo, $P$ is on $AB$ iff $Q$ is on $EF$, whether $CDFE$ is concyclic or not." } { "Tag": [], "Problem": "Google Harvey Mudd.\r\n\r\nHave you? Well now you are a geek for knowing what Harvey Mudd is. The all-Math school.\r\n\r\nSo, What are your thoughts? I really want to know. Not sure whether to apply there or not.", "Solution_1": "Whether or not to apply to Harvey Mudd depends on whether you'd like to attend there if you are admitted. If you wouldn't possibly accept an offer of admission there, there is no reason to apply. I hear that Harvey Mudd is a very good engineering school, and it's in a part of California with pleasant weather. I think my oldest son will only apply to research universities (that is, colleges with attached graduate schools), but he may still change his mind. My personal preference is strongly for research universities in large urban areas.", "Solution_2": "Harvey Mudd provides very generous merit aid for many students. Some students turn down more \"prestigious\" institutions to become Mudders.\r\n\r\n[quote]The Harvey S. Mudd Merit Award\n\nAll applicants for admission are considered for the Harvey S. Mudd Merit Award, including our foreign/international student applicants. Harvey S. Mudd Merit Award recipients are selected by the Office of Admission on the basis of their academic achievement and their leadership potential. Recipients of this academic award have the following credentials:\n\nSAT Critical Reading Score: 700 or above\n\nSAT Writing Score: 700 or above\n\nSAT Math Score: 750 or above\n\nSAT Math 2 Subject Exam Score: 750 or above\n\nHigh school rank within the top 10%\n\nEvidence of character and personal promise as reflected in admission application materials.[/quote]\r\nSee [url]http://www.hmc.edu/admin/finaid/grants.html#hmcmerit[/url]", "Solution_3": "Is that given to everyone who qualifies, or is it just the minimum requirements?", "Solution_4": "We have going to have a Math Jam with the Harvey Mudd Director of Admissions on Tuesday, October 24 at 7:30 Eastern / 4:30 Pacific. \r\n\r\n(I'm about to post an announcement in this forum.)", "Solution_5": "Harvery Mudd is also in the same city as 4 other colleges (Scripps, Pomona, Claremont McKenna, and Pitzer) as well as two graduate institutions. Most of the colleges offer cross-registration. They share a library system and some athletic facilities. I can see how that would easily change some peoples' minds about going to Harvery Mudd.\r\n\r\n[url]http://www.claremont.edu/[/url]", "Solution_6": "[quote=\"numberdance\"]Harvery Mudd is also in the same city as 4 other colleges (Scripps, Pomona, Claremont McKenna, and Pitzer) as well as two graduate institutions. Most of the colleges offer cross-registration. They share a library system and some athletic facilities. I can see how that would easily change some peoples' minds about going to Harvery Mudd.\n\n[url]http://www.claremont.edu/[/url][/quote]\r\n\r\nnot to mention i think caltech is close by", "Solution_7": "It's closer to HMC than most colleges, but it isn't literaly the same campus - not just city - like the Claremonts. I think cross-registration at Caltech wouldn't be very feasible, whereas the 5Cs readily offer everything numberdance mentioned.\r\n\r\nAnd no one has answered my question yet... To rephrase it: Is the Merit Award given to every qualifier or is it a competition among qualifiers?", "Solution_8": "The requirements listed above are the ONLY requirements for the merit award. Typically, 35-40% of the students here receive that award. Of course, you still have to do things like maintain a decent GPA, not flunk out, etc.\r\n\r\nAs a current HMC student, I may be somewhat biased in favor of Mudd, but I would like to point out that one of the biggest advantages of Mudd: the tight student body and student-faculty relationship. Since there are no graduate students here, undergrads are first in line when a prof is looking for a research assistant, and anyone who wants to do summer research is almost guaranteed at least 1 opportunities here in their 4 years. Furthermore, we have this thing where you and a group of friends can take a prof out to lunch/dinner, and Mudd will pay for most of it (something like 7 or 8 bucks per person, can't rememeber off the top of my head). The student body is also very friendly and warm, and it would not be uncouth to randomly go up to someone and strike up a conversation.\r\n\r\nOf course, the best way to decide if Mudd (or any college, for that matter) is right for you is to go and VISIT. Stay overnight if you can, but really, one needs to go and experience a little of the college before one can make an informed decision. It always makes me sad when people get acceptances to several awesome schools and decide solely based on the prestige of one, without actually visiting the campuses." } { "Tag": [ "geometry" ], "Problem": "If the length of a diagonal of a square is $ a \\plus{} b$, then the area of the square is:\r\n\r\n$ \\textbf{(A)}\\ (a \\plus{} b)^2 \\qquad\\textbf{(B)}\\ \\frac {1}{2}(a \\plus{} b)^2 \\qquad\\textbf{(C)}\\ a^2 \\plus{} b^2$\r\n$ \\textbf{(D)}\\ \\frac {1}{2}(a^2 \\plus{} b^2) \\qquad\\textbf{(E)}\\ \\text{none of these}$", "Solution_1": "[hide]\nThe area is $ \\frac{1}{2}d^2$ (because $ d\\equal{}s\\sqrt{2}$, and $ (s\\sqrt{2}^2\\equal{}2s^2\\equal{}2A)$, so $ \\boxed{B}$, $ \\frac{1}{2}(a\\plus{}b)^2$ is the correct answer.\n[/hide]" } { "Tag": [ "\\/closed" ], "Problem": "This has popped up a couple of times now, sometimes when submitting posts, this most recent one happened when I was trying to view a thread in the Round Table by author.\r\n\r\nThe error message is as follows\r\n[quote]\nSQL requests not achieved\n\nDEBUG MODE\n\nSQL Error: 1054 Unknown column 'pt.post_id' in 'on clause'\n\nSQL Request: SELECT p.*, pt.post_subject, pt.post_sub_title, pt.post_text, pt.bbcode_uid, u.username, u.user_level, u.user_allow_viewonline, u.user_session_time, u.user_session_logged, u.user_bot_ips, u.user_bot_agent, u.user_posts, u.user_from, u.user_interests, u.user_website, u.user_email, u.user_icq, u.user_aim, u.user_yim, u.user_regdate, u.user_msnm, u.user_viewemail, u.user_rank, u.user_sig, u.user_sig_bbcode_uid, u.user_avatar, u.user_avatar_type, u.user_allowavatar, u.user_allowsmile, u.user_weblogs, u.user_current_blog, u.user_albums, u.user_gender, u.user_post_rating, r.rating FROM phpbb_posts p, phpbb_posts_text pt, phpbb_users u LEFT JOIN phpbb_post_ratings r ON ( r.post_id = pt.post_id AND r.user_id = 17514) WHERE p.topic_id = 88374 AND pt.post_id = p.post_id AND u.user_id = p.poster_id ORDER BY u.username, p.post_id LIMIT 0, 20\nLine : 397\nFile : class_posts.php[/quote]", "Solution_1": "That happens once in a while, but usually goes away (gets fixed?) after a while. When you post and that error comes up, your post is posted.", "Solution_2": "[quote=\"i_like_pie\"]That happens once in a while, but usually goes away (gets fixed?) after a while. When you post and that error comes up, your post is posted.[/quote]That's a different error (not sure what causes that random error once in a while, but the thing is that the server executes the command (placing the post in). \r\n\r\nThis one is a bug (can be reproduced by trying to sort after author). Bug fixed :lol:" } { "Tag": [ "inequalities", "inequalities proposed" ], "Problem": "\\[\\triangle ABC\\Longrightarrow 2r\\le \\sqrt {\\frac {a}{s}\\cdot \\left[ a(s-a)+{(b-c)}^2\\right]}\\le 2(R-r).\\] [u][i]Example:[/i][/u] $a=5,\\ b=4,\\ c=3\\Longrightarrow s=6,\\ r=1,\\ R=\\frac 52\\Longrightarrow 2<\\sqrt 5<3.$", "Solution_1": "Interesting result!\r\n\r\nThe left side hand is quite easy: It can be prove by squaring both sides and use formula r = S/s. But the right side is quite complicated. I thought about the idea to show middle term is less or equal R <= 2(R-r) but I'm not sure about it's validity.", "Solution_2": "hello, after inserting $ R\\equal{}\\frac{abc}{4A}$, $ r\\equal{}\\frac{A}{s}$ , $ A\\equal{}\\sqrt{s(s\\minus{}a)(s\\minus{}b)(s\\minus{}c)}$ and squaring $ \\sqrt{\\frac{a}{s}\\left(a(s\\minus{}a)\\plus{}(b\\minus{}c)^2\\right)}\\le 2(R\\minus{}r)$ your inequality is equivalent with\r\n$ \\frac{(a^2b\\plus{}a^2c\\plus{}b^2c\\plus{}bc^2\\minus{}b^3\\minus{}c^3\\minus{}2abc)^2}{(b\\plus{}c\\minus{}a)(a\\plus{}b\\minus{}c)(a\\minus{}b\\plus{}c)(a\\plus{}b\\plus{}c)}\\geq 0$ which is obviously true.\r\nSonnhard." } { "Tag": [ "geometry", "geometric transformation", "rotation", "trigonometry" ], "Problem": "Because of earth's rotation, a pendolum isn't always perfectly aligned to the force of gravity. If $\\alpha$ is the angle between the gravity and the pendolum, $l$ is the latitude, $g$ the gravitational field, $R$ the radius of the Earth, $T$ the period of Earth's rotation:\r\n\r\n- Determine all values of $l$ that make $\\alpha=0$\r\n- Demonstrate that $\\alpha \\simeq \\frac{2 {\\pi}^2 R}{gT^2} \\cdot sin(2l)$\r\n- Determine which value of $l$ maximizes $\\alpha$\r\n\r\nIt isn't hard...", "Solution_1": "Hm, I remember doing something very similar to (2) in the [url=http://www.mathlinks.ro/Forum/viewtopic.php?t=55078]Balloooooneee[/url] topic. Too bad I only put the answer there. Anyway, (1) and (3) follow directly from your equation - $\\alpha=0^\\circ$ only for equator and poles and is maximal for $l=45^\\circ$.\r\n\r\nEDIT:\r\nBut I remember that it wasn't as straightforward as it first looked. I'll try to do it again and post here.", "Solution_2": "It's more or less all [url=http://www.mathlinks.ro/Forum/viewtopic.php?t=55078]there[/url], you only need to divide $a_\\text{tangential}/g$:\r\n\r\n$\\alpha\\approx\\tan\\alpha=a_\\text{tangential}/g=\\omega^2R\\cos\\phi\\sin\\phi/g$\r\n$\\alpha\\approx(\\frac{2\\pi}{T})^2R\\frac{1}{2}(2\\cos\\phi\\sin\\phi)\\frac{1}{g}$\r\n$\\alpha\\approx\\frac{2\\pi^2R}{gT^2}\\sin{2\\phi}$" } { "Tag": [ "inequalities", "geometry open", "geometry" ], "Problem": "in trigangle $ ABC$, the shortest side is $ BC \\equal{} a$ (that is,$ b>a, c>a$). Prove or disprove that:\r\n\r\n$ (c \\minus{} a)^2.(b \\minus{} a)^2 \\plus{} bc.[b^2 \\plus{} bc \\plus{} c^2 \\minus{} 2a(b \\plus{} c) \\plus{} a^2] > 0$\r\n\r\n Lokman G\u00d6K\u00c7E", "Solution_1": "This probem is a problem or a solution?\r\n\r\n (c - a)\u00b2.(b - a)\u00b2 + bc.[b\u00b2 + bc + c\u00b2 - 2a(b + c) + a\u00b2]\r\n\r\n=(c - a)\u00b2.(b - a)\u00b2 + bc.[(b - a)\u00b2 + c(b + c - 2a)] > 0" } { "Tag": [ "number theory unsolved", "number theory" ], "Problem": "solve equation in $N_{0}$ : $x^{3}-1=y^{2}$", "Solution_1": "Work in progress then :blush:", "Solution_2": "You are failed!\r\n$3a^{2}-b^{2}=3a^{2}-(a+c)^{2}=2a^{2}-2ac-c^{2}$ but not $2a^{2}-2ac+c^{2}$.", "Solution_3": "how can solve it??", "Solution_4": "[url]http://www.mathlinks.ro/Forum/viewtopic.php?t=127034[/url]" } { "Tag": [ "calculus", "integration", "LaTeX", "function", "real analysis", "real analysis unsolved" ], "Problem": "I will be honest, i don't have any ideea how to do this.\r\nBut i can guarantee you that this will be the only exercices that i don't know how to solve at all that i will post on this forum.\r\nSo, this is only one time help.\r\n\r\nI have an exam tommorow, i've only studied partial derivates, rotor and the gradient.\r\nI didn't had any time for integrals.\r\n\r\nThere are some chances that i will have to solve some type ( very close ) of this integral tommorow.\r\n\r\n\r\nThanks in advance.", "Solution_1": "Why don't you just type it in Latex? I can hardly read it. Do you mean $ \\int\\sqrt {\\frac {3}{x^2 \\plus{} 1}}dx$ ?\r\nWhat methods have you seen on computing integrals? Do you know the substitution rule?", "Solution_2": "If it's what Blue Velvet typed, then a trigonometric (or possibly hyperbolic) substitution will help.\r\n\r\nIf it's $ \\int\\sqrt {\\frac {3}{x^3 \\plus{} 1}}\\,dx,$ then I suspect that there is no elementary function (function that can be written in a finite number of steps starting with the usual formulas) that is the antiderivative of that.\r\n\r\nYou do know that the latter situation is common, right? You shouldn't be surprised by it.", "Solution_3": "Kent Merryfield - you are correct, that's the integral", "Solution_4": "Nobody ? \r\nI need it in about 12h maximum :(", "Solution_5": "You don't read very well, do you? At the very least you ignored the message I was trying to deliver. But since you insist on an \"answer,\" I'll give you one:\r\n\r\n$ F(x) \\equal{} \\int_0^x\\sqrt {\\frac {3}{t^3 \\plus{} 1}}\\,dt,$ to which you may add any constant." } { "Tag": [ "geometry", "circumcircle", "geometry unsolved" ], "Problem": "[img]http://www.mathlinks.ro/Forum/viewtopic.php?mode=attach&id=14865[/img]\r\n\r\n :(", "Solution_1": "$ M$ is the midpoint of $ BC.$ Internal bisector of the $ \\angle A$ cuts the circumcircle $ (O)$ again at $ N.$ $ P, Q$ are feet of perpendiculars from $ N \\in (O)$ to $ AB, AC,$ $ PQ$ is Simson line with the pole $ P$ passing through $ M.$ Assume $ b > c.$ The right $ \\triangle NBP \\cong \\triangle NCQ$ are congruent, because $ NB \\equal{} NC$ and $ \\angle NBP \\equal{} 180^\\circ \\minus{} \\angle B \\minus{} \\frac{\\angle A}{2} \\equal{} \\angle C \\plus{} \\frac{\\angle A}{2} \\equal{} \\angle NCQ.$ Therefore, $ BP \\equal{} CQ$ and it follows $ AP \\equal{} AQ \\equal{} \\frac{b \\plus{} c}{2}.$ Midpoint $ M$ of $ a \\equal{} BC$ is on $ PQ.$\r\n\r\nConstruct $ P, Q$ on the legs of the given angle $ \\angle A,$ such that $ AP \\equal{} AQ \\equal{} \\frac {b \\plus{} c}{2}.$ Draw circle $ (A)$ with center $ A$ and given radius $ m_a.$ $ M$ is also on $ (A).$ The circle cuts the segment $ PQ$ at $ M, M'$ (symmetrical WRT the internal bisector of the $ \\angle A$). Pick one, say $ M.$ Double the segment $ AM$ into $ AN \\equal{} 2\\ AM.$ Parallel to $ AQ$ through $ N$ cuts the ray $ AP$ at $ B$ and parallel to $ AP$ through $ N$ cuts the ray $ AQ$ at $ C.$", "Solution_2": "[img]http://www.mathlinks.ro/Forum/viewtopic.php?mode=attach&id=14873[/img]\r\n\r\n :thumbup:" } { "Tag": [ "inequalities proposed", "inequalities" ], "Problem": "$ Let,x,yz\\ge 0$,$ xy \\plus{} yz \\plus{} zx \\plus{} 2xyz \\equal{} 1$\r\nFind the best $ k > 0$ to this ineq is true for all $ x,y,z$\r\n\r\n$ x \\plus{} y \\plus{} z\\le \\frac {1}{4xyz} \\plus{} (12 \\minus{} 8k)xyz \\plus{} k \\minus{} 2$\r\nP\\s>I think it is a nice ineq,with that condition,we have many other ineq :)", "Solution_1": "[quote=\"Allnames\"]$ Let,x,yz\\ge 0$,$ xy \\plus{} yz \\plus{} zx \\plus{} xyz \\equal{} 2$\nFind the best $ k > 0$ to this ineq is true for all $ x,y,z$\n\n$ x \\plus{} y \\plus{} z\\le \\frac {1}{4xyz} \\plus{} (12 \\minus{} 8k)xyz \\plus{} k \\minus{} 2$\nP\\s>I think it is a nice ineq,with that condition,we have many other ineq :)[/quote]\r\nIt 'll be kill by CYH Put :$ a\\plus{}b\\plus{}c\\equal{}p;ab\\plus{}bc\\plus{}ca\\equal{} \\frac{p^2\\minus{}q^2}{3} ;abc\\equal{}r.$\r\n\r\nOr put $ a\\equal{}cosA;b\\equal{}cosB;c\\equal{}cosC$", "Solution_2": "No,no.my friend,i also think it can be killed by CYH ,but follow me,it is quite hard,can you detail it :wink: \r\nAnd the second way,i dont think your put is true,check care fully.my dear", "Solution_3": "Sorry,my problem has some mistakes,and i edited\r\nBy the way,i have just found my ineq is quite stong,and usefull :lol:", "Solution_4": "[quote=\"onlylove_math\"][quote=\"Allnames\"]$ Let,x,yz\\ge 0$,$ xy \\plus{} yz \\plus{} zx \\plus{} xyz \\equal{} 2$\nFind the best $ k > 0$ to this ineq is true for all $ x,y,z$\n\n$ x \\plus{} y \\plus{} z\\le \\frac {1}{4xyz} \\plus{} (12 \\minus{} 8k)xyz \\plus{} k \\minus{} 2$\nP\\s>I think it is a nice ineq,with that condition,we have many other ineq :)[/quote]\nIt 'll be kill by CYH Put :$ a \\plus{} b \\plus{} c \\equal{} p;ab \\plus{} bc \\plus{} ca \\equal{} \\frac {p^2 \\minus{} q^2}{3} ;abc \\equal{} r.$\n\nOr put $ a \\equal{} cosA;b \\equal{} cosB;c \\equal{} cosC$[/quote]\r\nI think denta is much nicer", "Solution_5": "Not comment my friend,tell me about your proof,at least,what is your result :wink:", "Solution_6": "[quote=\"Allnames\"]$ Let,x,yz\\ge 0$,$ xy \\plus{} yz \\plus{} zx \\plus{} 2xyz \\equal{} 1$\nFind the best $ k > 0$ to this ineq is true for all $ x,y,z$\n\n$ x \\plus{} y \\plus{} z\\le \\frac {1}{4xyz} \\plus{} (12 \\minus{} 8k)xyz \\plus{} k \\minus{} 2$\nP\\s>I think it is a nice ineq,with that condition,we have many other ineq :)[/quote]\r\nwith can_hang 's help,i have found some mistake in my problem :blush: \r\nbut how abuot $ k\\equal{}3\\sqrt 3 \\minus{}4$ :lol:" } { "Tag": [ "summer program", "Mathcamp", "PROMYS", "HCSSiM", "Ross Mathematics Program", "Mafia", "number theory" ], "Problem": "Hi, \r\n\r\nI'm a senior in high school this year and am interested in doing a math program next summer (before my freshman year of college); I particularly wanted to find one though where I wouldn't be the only person that old and everyone else would be in lower grades, but instead a place where it would be totally common to have people about to start college. Does anyone have any suggestions for what this might be? I was most interested in CanadaUSA Mathcamp and PROMYS, but have also heard good things about HCSSiM and Texas Honors Math Camp. Please let me know if you have any advice!\r\n\r\nThanks, M.", "Solution_1": "I don't think it's particularly common, at any of the programs you mentioned, for students to be about to start college. (Someone please correct me if I'm wrong!) Both Mathcamp and PROMYS explicitly allow graduating seniors to apply; HCSSiM almost never invites graduating seniors to participate.", "Solution_2": "I spent two summers at the Ross Math Program. I'm surprised it wasn't on your list. Last summer there were people in college who still came as first-years. In general, first-years are more likely to be in high school and not about to go to college. Having said that, there were at least a handful of people about to go to college among the first year and second year students alone. Beyond that, there are the junior counselors and counselors who stay in the same dorms as you. Since most of the days are spent in the dorms (you have to be pretty committed to doing a lot of math), you can choose to work/ hang out with any age group you want. I mean all counselors are required to be at least old enough to be going to college the next year and they are very very open to just chilling or helping. As a Rosser, I would also have to say that it seems like it is the most hardcore out of all these camps. It is 8-weeks long and it is also where I decided to be a math major in college. You don't have to worry about the quality of the program at all. The only thing that doesn't work as well for some people is the expectation that people work hard. It's up to you what you want out of your summer program. I highly recommend it!", "Solution_3": "PROMYS is really nice, though I went as a high school freshman... there were several graduated seniors, though, and really the age difference isn't that important, as the counselors are all college students and the junior counselors are graduated seniors as well. As long as you like math, you'll find yourself in good company.", "Solution_4": "PROMYS is fantastic. We had quite a few people who were transitioning between high school and college last year. Going to a math camp is great because you get to enter college with a peer group that you already know. \r\n\r\nBut like paladin8 says, the age difference really isn't that important. Most of the people there are at different stages of their life and social development anyway.", "Solution_5": "Hi Mathilde,\r\n\r\nLast year there were a dozen students at Mathcamp who were about to start college. Six of those were new Mathcamp students; the other six were students who had been to Mathcamp in previous years and came back. For applicants in grade 10 and above, we don't really look closely at exactly what grade they're in, so as far as admissions goes it doesn't help or hurt you.", "Solution_6": "I have to agree with Peter about the Ross Program. There is just the right amount of guidance from counselors and individual creativity involved to ensure a complete understanding of what you learn. Eight weeks does seem like long time, but, with math and frisbee, time flies.", "Solution_7": "PROMYS is very nice, I went there this summer and I learned a lot of number theory there.\r\n\r\nHehe you get to play mafia to like 4:00AM on weekends :D", "Solution_8": "[quote=\"beta\"]PROMYS is very nice, I went there this summer and I learned a lot of number theory there.\n\nHehe you get to play mafia to like 4:00AM on weekends :D[/quote]\r\n\r\nhehe....you didn't stay around for the 7:00am game, Feiqi?", "Solution_9": "[quote=\"mel\"][quote=\"beta\"]PROMYS is very nice, I went there this summer and I learned a lot of number theory there.\n\nHehe you get to play mafia to like 4:00AM on weekends :D[/quote]\n\nhehe....you didn't stay around for the 7:00am game, Feiqi?[/quote]\r\n\r\n5:00AM is good enough, no? :D", "Solution_10": "[quote=\"beta\"][quote=\"mel\"][quote=\"beta\"]PROMYS is very nice, I went there this summer and I learned a lot of number theory there.\n\nHehe you get to play mafia to like 4:00AM on weekends :D[/quote]\n\nhehe....you didn't stay around for the 7:00am game, Feiqi?[/quote]\n\n5:00AM is good enough, no? :D[/quote]\r\n\r\nWell, I guess by then, it's hard to tell the difference between 5am and 7am anyway. :P" } { "Tag": [ "trigonometry", "function", "limit", "calculus", "derivative", "trig identities", "calculus computations" ], "Problem": "I been having a problem with trig functions. One of which is: lim x ->0 (sin2x)/x. And also, what would I do if I had the same trig function like \r\nlim x->0 tan5x/tan2x? And for distribution, I had this problem: lim x->0 square root(x+1) -1/ x, I tried it in all different ways but somehow I cannot compute the answer which is 1/2, plz help.", "Solution_1": "[quote=\"red218\"]I been having a problem with trig functions. One of which is: lim x ->0 (sin2x)/x. And also, what would I do if I had the same trig function like \nlim x->0 tan5x/tan2x? And for distribution, I had this problem: lim x->0 square root(x+1) -1/ x, I tried it in all different ways but somehow I cannot compute the answer which is 1/2, plz help.[/quote]\r\n\r\n1)$\\lim_{x\\to 0}\\frac{\\sin 2x}{x}= \\frac{1}{2}\\lim_{x\\to 0}\\frac{\\sin 2x}{2x}= \\frac{1}{2}$\r\n\r\n2)$\\lim_{x\\to 0}\\frac{\\tan 5x}{\\tan 2x}= \\lim_{x\\to 0}\\frac{\\cos 5x}{\\cos 2x}\\cdot \\frac{\\sin 2x}{\\sin 5x}= \\lim_{x\\to 0}\\frac{\\cos 5x}{\\cos 2x}\\cdot \\lim_{x\\to 0}\\frac{\\sin 2x}{\\sin 5x}= \\lim_{x\\to 0}\\frac{\\sin 2x}{\\sin 5x}$\r\n$=\\lim_{x\\to 0}\\frac{2x}{\\sin 5x}\\cdot \\frac{\\sin 2x}{2x}= \\lim_{x\\to 0}\\frac{2x}{\\sin 5x}\\cdot \\lim_{x\\to 0}\\frac{\\sin 2x}{2x}= \\lim_{x\\to 0}\\frac{2x}{\\sin 5x}$\r\n$=2\\lim_{x\\to 0}\\frac{x}{\\sin 5x}= \\frac{2}{5}\\lim_{x\\to 0}\\frac{5x}{\\sin 5x}= \\frac{2}{5}$\r\n\r\n3)Just rationalize.", "Solution_2": "I think the answer of 1) is 2... :D", "Solution_3": "I understand now how to get 1 and 3, but in the back of the book for 2., somehow the answer is 5/2(I apologize I did not mention that before). I even got 2/5 by doing the same steps as #1. Was I suppose to switch one of the fractions upside down?", "Solution_4": "It's $\\frac52$, and the bad step is the first one. $\\tan=\\frac{\\sin}{\\cos}$.", "Solution_5": "Using the above method (correctly this time) it is not hard to show that:\r\n$\\lim_{x\\to 0}\\frac{\\tan nx}{\\tan mx}= \\frac{n}{m}$ with $m\\not = 0$.", "Solution_6": "Alternatively, use l'hospital... :P", "Solution_7": "[quote=\"me@home\"]Alternatively, use l'hospital... :P[/quote]\r\nUck, I hate that rule", "Solution_8": "I just want to know if I did these right... \r\n\r\nFor 2. I make it (cos2x/cos5x)*(sin5x/sin2x)=(5x/sin2x)*(sin5x/5x=1)=5 lim x->0 x/sin2x= 5/2 lim x->0= 1(basically)=5/2 ? The ans. is 5/2.\r\n\r\n\r\nFor lim x->0 (sec(x)-1)/x sec(x), I split the denominators in half (x)(sec x), =(Sec(x)/Sec(x)=1)-(1/x), make the x on the bottom like, 1-1=0 ?the ans. is 0\r\n\r\n\r\nTo be honest, I really do not know what to do with this: lim x->0 (1-cos(x))/sin(x)) . The ans. is 0. \r\n\r\n\r\nAnd if I did those on top wrong, I can take criticism. On a side note, The book did not explain the material well, is there any website where I can find the tools necessary to solve these types of problems( and beyond)?", "Solution_9": "Looking at those limits in your last post:\r\n$\\frac{\\sec x-1}{x\\sec x}=\\frac{(1-\\cos x)/\\cos x}{x/\\cos x}=\\frac{1-\\cos x}{x}$.\r\nYou know this limit- the fact that $\\lim_{x\\to 0}\\frac{1-\\cos x}{x}=0$ is an essential step in calculating the derivatives of trigonometric functions.\r\n\r\nThe second can also be built from pieces you know: $\\frac{1-\\cos x}{\\sin x}=\\frac{1-\\cos x}{x}\\cdot \\frac{x}{\\sin x}$.\r\nAlternately, we use trig identities: $\\frac{1-\\cos x}{\\sin x}=\\frac{1-\\cos^{2}x}{\\sin x(1+\\cos x)}=\\frac{\\sin x}{1+\\cos x}$\r\n\r\nBased on the content of these, I assume you haven't yet learned the theorem known as L'Hopital's rule. You don't need it here, so just ignore those comments." } { "Tag": [ "number theory proposed", "number theory" ], "Problem": "Hi, \r\n\r\nI don't know if I have already asked this question in this forum before but I think it's a nice problem...\r\n\r\nShow that for every [tex]q \\in \\mathbb{Q}^+[/tex] exist four positiv integers [tex]a,b,c,d[/tex] such that:\r\n\r\n[tex]q=\\frac{a^3+b^3}{c^3+d^3}[/tex]", "Solution_1": "http://www.mathlinks.ro/Forum/viewtopic.php?t=6232\r\nhttp://www.mathlinks.ro/Forum/viewtopic.php?t=19763\r\n\r\n:)" } { "Tag": [], "Problem": "Jennifer burns calories while doing activities..:\r\n\r\nSleeping - 10 calories/per hour\r\n\r\nWalking - 80 cph\r\n\r\nBasketball - 150 cph\r\n\r\nBicycling - 100 cph.\r\n\r\nShe spends 5 times as much time playing basketball as she does while walking, and half as much time bicycling as playing basketball. She spends 4 hours sleeping. She burns 1250 calories that day. How much time did she spend on each activity?\r\nRound all answers to nearest hundredth.", "Solution_1": "I don't think I should show my work because I used matrices.\r\n\r\n[hide] walking: 1.779 hours\nbasketball: 8.897 hours\ncycling: 4.449 hours\nsleeping: given[/hide]", "Solution_2": "[hide]\n\nbasketball = 5.601 hours\nwalking = 1.120\nbiking = 2.80\n[/hide]", "Solution_3": "[hide]\nused a one variable equation...[/hide]", "Solution_4": "[hide]she spends 4 hours sleeping burning 40 calories. The rest of the activities she burns 1210 cals.\nIn hours of x, if walking is x\nbasketball:5x\nwalking:x\nbiking:2.5x\n\n5x*150+2.5x*100+80x=1210\n750x+250x+80x=1210\n1080x=1210\nx=1210/1080\nx=121/108\n\nsleeping:4 hours\nwalking:1.12037037037037037037037037037037 hours\nbasketball:5.60185185185185185185185185185185 hours\nbiking:2.80092592592592592592592592592593 hours[/hide]", "Solution_5": "everyone got different answers. :?", "Solution_6": "xenon_nightmare152 and jimli are correct", "Solution_7": "how do you know :?: ToBe didn't say they were right" } { "Tag": [ "geometry", "3D geometry", "ratio" ], "Problem": "A cube with 3-inch edges is made using 27 cubes with 1-inch edges.\nNineteen of the smaller cubes are white and eight are black. If the\neight black cubes are placed at the corners of the larger cube, what\nfraction of the surface area of the larger cube is white?", "Solution_1": "The surface area of the cube counting every side is $ 54$ square inches. Every black cube has outside surface area of $ 3$, so the surface area of the larger cube that is black is $ 8*3 \\equal{} 24$, and $ 34$ square inches white. So the fraction of the surface area of the larger cube that is white is $ \\frac {34}{54} \\equal{} \\frac {17}{27}$. I think... :maybe:\r\n\r\nEdit: Oops, I seem to need to go back to kindergarten.. :blush:", "Solution_2": "[quote=\"$ LaTeX$\"]The surface area of the cube counting every side is $ 54$ square inches. Every black cube has outside surface area of $ 3$, so the surface area of the larger cube that is black is $ 8*3 \\equal{} 24$, and [color=red][b]34 [/b][/color] square inches white. So the fraction of the surface area of the larger cube that is white is $ \\frac {34}{54} \\equal{} \\frac {17}{27}$. I think... :maybe:[/quote]\r\n\r\nThe logic in the above is correct, except for a small mistake:\r\n\r\nThe surface area would be 54 square inches as by the formula 6s^2. The black cubes would take away 3 square inches per each of the 8. Thus 24 square inches would be black, leaving 54-24 or [b]30 square inches[/b] as white. Thus we have-\r\n\r\n$ \\dfrac{\\mbox{white area}}{\\mbox{total area}} \\equal{} \\dfrac{30}{54} \\equal{} \\boxed{\\dfrac{5}{9}}$", "Solution_3": "But why isn't it 29/54? 1 of the white cubes is inside the 3x3x3 cube so the surface area of that is 0.", "Solution_4": "Easy Way!\nOne face looks like this:\n$\\Box \\blacksquare \\Box$\n$\\blacksquare \\Box \\blacksquare$\n$\\Box \\blacksquare \\Box$\nAll the other faces look the same.\nThus, the ration of surface area is $5/9$" } { "Tag": [ "function", "superior algebra", "superior algebra theorems" ], "Problem": "I don't know if I remember this problem correctly but here it goes.\r\n\r\nLet $ S_n$ be the set of all permutations of $ S \\equal{} {\\{0, 1, 2, 3, \\ldots, n\\}}$. Show that if $ A$ is a permutation of $ S$, and $ B$ is a permutation of $ S$, then $ AB$ is also a permutation of $ S$.\r\n\r\nMy proof:\r\n\r\nSince $ A$ and $ B$ are permutations of $ S$, that means that they are one-to-one and onto with the elements of $ S$. Now $ AB$ is simply a mapping of the elements of $ S$ from $ A$ to $ B$, which means that $ AB$ is still a permutation of $ S$.\r\n\r\nIs my proof correct? Is there a more elegant proof?", "Solution_1": "The easiest way to show that a function is one-to-one and onto is to explicitly describe its inverse. As for terminology, usually $ S_n$ denotes the number of permutations of $ \\{ 1, 2, ... n \\}$.", "Solution_2": "[quote=\"guile\"]Now $ AB$ is simply a mapping of the elements of $ S$ from $ A$ to $ B$, which means that $ AB$ is still a permutation of $ S$.[/quote] This sentence doesn't have any discernable meaning, or at least, it could be connected in your mind with the right thing to put here, but it could equally as easily be connected with things that have nothing to do with the solution.\r\n\r\nIn fact, since we're working with a finite set it is equivalent to show that the map is injective, is surjective, or has an inverse. Proving any one of these three things is easy in this case, but you should be clear about which one you're doing and what intermediate steps you're making." } { "Tag": [ "LaTeX", "function" ], "Problem": "Is there a way in $ \\text{\\LaTeX}$ to define terms with two words inside single one curly brackets? I have typed the definitions as\r\nThe field is said to be \\operatorname{algebraic} \\operatorname{closed} if ...\r\n\\operatorname{algebraic closed} won't produce the space between terms.", "Solution_1": "An operator name is supposed to be a one-off [b]mathematical[/b] function defined in math mode, not a text definition. For example, (see The LaTeX Companion 8-2-11):\r\n$ \\operatorname{seg} (a,r) \\equiv \\{ z \\in \\mathbf{C} \\colon \\Im z < \\Im a, \\ |z\\minus{}a| < r\\}$\r\nUsing \\operatorname prevents seg being put into italics and gets the correct spacing. Here seg is only used in this formula so doesn't have to be defined elsewhere. \r\n\r\nIs [i]algebraic closed[/i] a mathematical function? No, then you just say:\r\nThe field is said to be algebraically closed if ...\r\nand use \\emph, \\textbf, \\textit, \\textsl etc for the algebraically closed words." } { "Tag": [ "probability", "octahedron", "Duke", "USA" ], "Problem": "Two cows play a game where each has one playing piece, they begin by having the two pieces on opposite vertices of an octahedron, and the two cows take turns moving their piece to an adjacent vertex. The winner is the first player who moves its piece to the vertex occupied by its opponent's piece. Because cows are not the most intelligent of creatures, they move their pieces randomly. What is the probability that the first cow to move eventually wins?\r\n\r\nSource: Duke Math Meet 2008\r\n\r\nI'm pretty sure this problem has been posted before, but I can't seem to find where it is...\r\n\r\nAny solution or link to this problem will be greatly appreciated. :)", "Solution_1": "[hide]Two vertices of an octahedron are either opposite each other or adjacent each other. Let $ p$ be the probability of a cow winning if it starts on a vertex opposite of the other cow, and let $ q$ be the probability of winning if it starts on a vertex adjacent to the other cow.\nWe have $ p\\equal{}1\\minus{}q$ since the first move brings the cow adjacent to the other cow, then the other cow's probability of winning becomes $ q$.\nWe also have $ q\\equal{}\\frac{1}{4}\\plus{}\\frac{1}{2}(1\\minus{}q)\\plus{}\\frac{1}{4}(1\\minus{}p)$ since a move has $ \\frac{1}{4}$ probability of an immediate win, $ \\frac{1}{2}$ probability of landing adjacent to the other cow, and $ \\frac{1}{4}$ probability of landing opposite of the other cow.\nMaking the substitution $ 1\\minus{}p\\equal{}q$ in the second equation gives $ q\\equal{}\\frac{1}{4}\\plus{}\\frac{1}{2}(1\\minus{}q)\\plus{}\\frac{q}{4} \\iff q\\equal{}\\frac{3}{5}$, so $ p\\equal{}\\boxed{\\frac{2}{5}}$.[/hide]" } { "Tag": [ "function", "integration", "limit", "calculus", "derivative", "real analysis", "real analysis unsolved" ], "Problem": "Hello, I know my level of math is too low compared to most of you, but I have the following problem with a GRE math subject. \r\n\r\nIf function f is strictly increasing, which of the following is necessarily wrong?\r\n\r\nA.$\\forall x f(2x)=f(x)$\r\nB.$\\int_{0}^{1}f(x)dx=\\int_{1}^{2}f(x)dx$\r\nC.$\\lim_{x\\to \\infty}f'(x)=0$\r\nD. $f'(1)=-f'(2)$\r\n\r\n\r\nI think both B and D are necessarily wrong, what do you think?", "Solution_1": "D is not necessarily false. A strictly increasing function can still have a zero derivative somewhere (example: $f(x)=x^{3}$). So we could have $f'(1)=f'(2)=0.$\r\n\r\nB is certainly false.\r\n\r\nAnd, as you've written it, A is also false.", "Solution_2": "But if function f has f'(x0)=0, it means it has a maximum or minimum value at x0. It contradicts the condition that f is strictly increasing. Besides, for f(x)=x^3, f'(1)!=0 and f'(2)!=0.", "Solution_3": "[quote=\"fredzhang0421\"]But if function f has f'(x0)=0, it means it has a maximum or minimum value at x0. It contradicts the condition that f is strictly increasing. [/quote]\r\nThat is absolutely and totally wrong. A \"critical point\" is not a point at which a function [i]does[/i] have a max or a min; rather it is a point at which a function [i]might[/i] have a max or a min. The example of $f(x)=x^{3}$ was intended to be a counterexample for that. $f'(0)=0$ and yet the function is strictly increasing at zero.\r\n\r\nI didn't intend for $x^{3}$ to itself be the example; the intent was to find a function whose local behavior around 1 and again around 2 was the same as the local behavior of $x^{3}$ around zero.\r\n\r\nFor instance, consider the primitive $\\int(x-1)^{2}(x-2)^{2}\\,dx.$ That should do it.", "Solution_4": "Yes, yes ,absolutely! I see! Sorry for the wrong understanding of the condition f'(x)=0" } { "Tag": [ "MATHCOUNTS", "geometry", "3D geometry", "factorial", "calculus", "ratio", "articles" ], "Problem": "This is actually a nice memory technique. I give a word, and you give another word related to that, [i]and state this relation clearly[/i]. The nest person gives a word related to your word, and the game moves on. As usual don't post after yourself. [b]Proper nouns[/b] [b]are allowed[/b].", "Solution_1": "umm... might we have a first word to start the game bubka? ;)", "Solution_2": "I am getting forgetful nowadays. Anyway let me give it an archetypal beginning:\r\n\r\n[b]maths[/b]\r\n\r\n(a bit banal, but works fine!)", "Solution_3": "pencils. Not just one pencil. Not a pen either. Pencils, a essential part of every maths exam.", "Solution_4": "eraser: an essential part of every pencil, particularly (in my case,) those used in every maths exam.", "Solution_5": "memory(gets erased)", "Solution_6": "calculators, specifically my TI-83 Plus which has almost no available memory because that's all being used for the list of 314-ish prime numbers", "Solution_7": "Target (round of Mathcounts)", "Solution_8": "abacus, the early calculator", "Solution_9": "ancient greeks, because they had abacuses (abacusi? abaci?)", "Solution_10": "philosophy\r\n\r\nbecause they started all that crap", "Solution_11": "my english teacher, because he seems to like philosophy", "Solution_12": "Poems, because that's all he's been talking about all week.", "Solution_13": "Rhymes.\r\n'Nuff said.", "Solution_14": "\"Touch It\", as in that new busta rhymes song", "Solution_15": "Prostitutes\r\n\r\nDon't ask", "Solution_16": "Wow, there are so many dirty words that I can think of right now...\r\n\r\nSTDs", "Solution_17": "SDT (Signal Detection Theory)", "Solution_18": "Stealth bombers", "Solution_19": "Uranium-238", "Solution_20": "Manhattan project", "Solution_21": "1945", "Solution_22": "war, guns and all that violence that makes us cry", "Solution_23": "Cold War\r\nThe message is too small. Please make the message longer before submitting. \r\n \r\nThe message is too small. Please make the message longer before submitting. \r\n \r\nThe message is too small. Please make the message longer before submitting. \r\n \r\nThe message is too small. Please make the message longer before submitting. \r\n \r\nThe message is too small. Please make the message longer before submitting. \r\n \r\nThe message is too small. Please make the message longer before submitting. \r\n \r\nThe message is too small. Please make the message longer before submitting. \r\n \r\nThe message is too small. Please make the message longer before submitting. \r\n \r\nThe message is too small. Please make the message longer before submitting. \r\n \r\nThe message is too small. Please make the message longer before submitting. \r\n \r\nThe message is too small. Please make the message longer before submitting. \r\n \r\nThe message is too small. Please make the message longer before submitting. \r\n \r\nThe message is too small. Please make the message longer before submitting. \r\n \r\nThe message is too small. Please make the message longer before submitting. \r\n \r\nThe message is too small. Please make the message longer before submitting. \r\n \r\nThe message is too small. Please make the message longer before submitting. \r\n \r\nThe message is too small. Please make the message longer before submitting.", "Solution_24": "Refridgerator.", "Solution_25": "Ice cream! :icecream:", "Solution_26": "the word [size=125] Frigid[/size]\r\n\r\nAnd for some reason, the wii", "Solution_27": "tiger woods", "Solution_28": "hot Swedish models.", "Solution_29": "ugly swedish models whom you somehow have a fetish for" } { "Tag": [ "logarithms", "MATHCOUNTS", "FTW" ], "Problem": "What is the value of $ a$ if $ 3^a2^a5^a \\equal{} 0.0081$?", "Solution_1": "30^n=.0081 Hm this will be nasty.", "Solution_2": "According to my calculator, it is $ \\boxed{\\approx \\minus{}1.4159}$.", "Solution_3": "hello, we can write $ 30^a\\equal{}\\frac{81}{10000}\\equal{}\\frac{3^4}{10^4}$, taking the logarithm of both sides we have $ a\\equal{}\\frac{4\\ln(3)\\minus{}4\\ln(10)}{\\ln(30)}$.\r\nSonnhard.", "Solution_4": "This was a mathcounts question...logarithms are DEFINITELY not needed here.", "Solution_5": "hello, then solve this problem without logarithm, please. Are calculators allowed?\r\nSonnhard.", "Solution_6": "This was a chapter target. Thus, you [b]were[/b] allowed to use a calculator.\r\n\r\nThere must be either:\r\n\r\n1) Some intutively obvious way that we all missed.\r\n2) A mistake in the problem.", "Solution_7": "hello, your topic (2) is the reason why i have solved this problem with logarithms. Topic (1) is meaningless.\r\nSonnhard.", "Solution_8": "I think it's 2, since the given number is 3^4*2^-4*5^-4.", "Solution_9": "Dr Sonnhard Graubner,\n\nDid you get -4?\n_________________\n~pi man!! :rotfl:", "Solution_10": "According to ftw, the answer is -4. How is that possible? 1/30^4 =1/8100!", "Solution_11": "hello, the solution of the equation\n$30^{a}=\\frac{81}{10000}$ is definitively\n$a=\\frac{\\ln\\left(\\frac{81}{10000}\\right)}{\\ln(30)}$\nSonnhard.", "Solution_12": "Yeah I've seen this problem come up and I do believe the \"FTW\" answer is -4, which we know is grossly inaccurate.", "Solution_13": "[quote=\"ernie\"]According to my calculator, it is $ \\boxed{\\approx \\minus{}1.4159}$.[/quote]\n\nAccording to my calculator and wolfram alpha, this is correct.", "Solution_14": "Has somebody reported the problem yet" } { "Tag": [ "modular arithmetic", "function" ], "Problem": "1) Prove that if $a|b$, then $2^{a}-1 | 2^{b}-1$. (What about the converse?)\r\n\r\n2) Prove that if $2a|b$, then $2^{a}+1|2^{b}-1$. (What about the converse?)", "Solution_1": "Statement 2 doesn't hold: $2|4$ but $2^{2}+1\\nmid 2^{4}+1$, $3|6$ but $2^{3}+1\\nmid 2^{6}+1$, etc.", "Solution_2": "[quote=\"Phelpedo\"]1) Prove that if $a|b$, then $2^{a}-1 | 2^{b}-1$. (What about the converse?)\n\n2) Prove that if $a|b$, then $2^{a}+1|2^{b}+1$. (What about the converse?)[/quote]\r\n[hide=\"1)\"]Notice that $2^{n}-1$ is a number built of $n$ ones in the binary system. Now we clearly see that 1) is true, for example $111_{(2)}\\mid 111111111_{(2)}$ (the quotient is $1001001$). Also the converse is true, because $\\underbrace{1\\ldots 1}_{b}\\equiv \\underbrace{1\\ldots 1}_{b\\mod a}\\pmod{\\underbrace{1\\ldots 1}_{a}}$ and $\\underbrace{1\\ldots 1}_{a}$ obviously does not divide $\\underbrace{1\\ldots 1}_{a'}$ for $1a+2r$\n$\\implies 2a>a+r=20$\n\nThus, the smallest a can be is 11, the largest it can be is 19 (since the triangle is non-equilateral),\n$19-11+1=\\boxed{9}$ triangles[/hide]" } { "Tag": [ "function", "probability", "binomial coefficients", "real analysis", "real analysis unsolved" ], "Problem": "The generating function for the central binomial coefficients is\r\n\r\n$ \\sum_{n \\ge 0} {2n \\choose n} x^n \\equal{} \\frac {1}{\\sqrt {1 \\minus{} 4x} }$.\r\n\r\nThis function has the following product expansion:\r\n\r\n$ \\frac {1}{\\sqrt {1 \\minus{} 4x} } \\equal{} (1 \\plus{} 2x)(1 \\plus{} 6x^2)(1 \\plus{} 8x^3)(1 \\plus{} 54x^4)(1 \\plus{} 96x^5)(1 \\plus{} 312x^6)(1 \\plus{} 1152x^7)...$.\r\n\r\nThe sequence of coefficients in each factor is not in the OEIS. What is it? (It may or may not be more appropriate to place this question in Combinatorics.)", "Solution_1": "Really random question: where do products like that ever come up, except as generating functions for things like partitions? I ask because once you've posed the problem, it sounds obvious, but it never occurred to me before :) Also, we can obviously find the associated sequence for any given sequence -- and none of them seem to appear in the OEIS. (In fact, looking at some obvious first choices -- Fibonacci, Catalan, Bell -- the only one that seems remotely easy to understand is the powers of 2.)\r\n\r\nEdit: a related idea (inspired by \"partitions with distinct parts\" versus \"partitions\") is to look at products $ \\prod_{i \\equal{} 1}^\\infty \\frac {1}{1 \\minus{} a_i x^i}$. This is called the [i]Somos transform[/i], and so what you're looking at is something like a \"inverse dual Somos transform.\" Some inverse Somos transforms are in the OEIS, like [url=http://www.research.att.com/~njas/sequences/A006177]A006177[/url], the inverse Somos transform of the Catalan numbers.\r\n\r\nEdit the second: It does seem like we should at least be able to write down some recursive summation formula for these, right?", "Solution_2": "A friend of mine asked me how I would go about writing a geometric distribution as a sum of Bernoulli distributions, and (unless I'm horribly wrong) this comes down to writing a generating function of the form $ \\frac {\\frac {x}{2} }{1 \\minus{} \\frac {x}{2} }$ as a product of terms of the form I gave above. Here, the relevant product (again, unless I'm horribly wrong) is\r\n\r\n$ \\frac {\\frac {x}{2} }{1 \\minus{} \\frac {x}{2} } \\equal{} x \\prod_{k \\equal{} 0}^{\\infty} \\left( \\frac {2^{2^k}}{2^{2^k} \\plus{} 1} \\plus{} \\frac {x^{2^k}}{2^{2^k} \\plus{} 1} \\right)$,\r\n\r\nthat is, each Bernoulli distribution controls one bit of the output. So that got me thinking about whether this idea generalizes, that is, whether there is a natural way to write distributions as a sum of Bernoulli distributions. Unfortunately, I don't know enough probability to know whether this is a meaningful question to ask (or whether my interpretation of the original question is even correct), which is why I posed it here.", "Solution_3": "Belatedly: I don't have a clue :) (though I am still curious about this writing-of-generating-functions-as-products idea). But this does sound like the sort of thing with the property that there are other people around here who [i]do[/i] have a clue about it. (Apologies for the tortured sentence structure there.) So perhaps one of them will take a look and see if there is any sense to be made out of it.\r\n\r\nEdit: here's one sort of vague idea: let $ E(t) \\equal{} \\sum_{n \\geq 0} e_n t^n$ where the $ e_i$ are the elementary symmetric functions. Then $ E(t) \\equal{} \\prod_{n \\geq 0}1 \\plus{} x_i t$, so this is a natural-seeming relationship. Since the $ e_i$ are algebraically independent, it's also well-suited to your situation. But whether this is \"useful\" in any sense (e.g., for determining the coefficients of the sequence you mentioned), I have no idea. This might be worth a trip to Professor Stanley's office ;)", "Solution_4": "A recursive relationship that can be written is:\r\n\r\n$ \\sum_{\\frac{p}{i} \\in \\mathbb{N}}\\frac{(\\minus{}1)^{i\\plus{}1}}{i}a^i_{\\frac{p}{i}}\\equal{}\\frac{2^{2\\,p\\minus{}1}}{p}$\r\n\r\nwhere\r\n\r\n$ \\prod_{k\\equal{}1}^\\infty(1\\plus{}a_k\\,x^k)\\equal{}\\frac{1}{\\sqrt{1\\minus{}4x}}$\r\n\r\n(I hope I got it right..)", "Solution_5": "A question:\r\n\r\nSuppose a function has infinitive number of zeros named $ z_k$ with multiplicity $ p_k$. Would it then be safe to say that this function can be written in a form similar to:\r\n\r\n$ f(x) \\sim \\prod_{k} (x\\minus{}z_k)^{p_k}$", "Solution_6": "That looks like the [url=http://en.wikipedia.org/wiki/Weierstrass_factorization_theorem]Weierstrass factorization theorem[/url], although I don't think it applies directly here.\r\n\r\nYour other recurrence looks almost like a Dirichlet convolution. It might be feasible to extract a summation identity for $ a_n$ that way.", "Solution_7": "Yes, Weierstrass was my base idea.. But the idea it self should work for this..", "Solution_8": "It shouldn't work, because it's the wrong sort of convergence. The products t0rajir0u is looking at converge in the formal power series sense: the coefficient of $ x^n$ is eventually constant. A Weierstrass-type product doesn't do that; if things go right, it converges as an analytic function on some disk. Of course, in order to make things go right, you have to multiply by various nonvanishing entire functions in each term." } { "Tag": [ "number theory proposed", "number theory" ], "Problem": "find x,y,z in positive integer such that z=4*q+3 and \r\n x^2+5=y^z. :D ;) :lol: :lol: :lol: :lol:", "Solution_1": "no one answer my question :( :( :( :( :( :(", "Solution_2": "it is easy to prove that x is even and y is odd.\r\nthen x^2+4=y^z-1=(y-1)(y^(z-1)+..+1);\r\nput A=y^(z-1)+...+1;it follow that A is odd such that y-1 divisible to 4\r\nhence you have the solution\r\ngood luck", "Solution_3": "it is easy to prove that x is even and y is odd.\r\nthen x^2+4=y^z-1=(y-1)(y^(z-1)+..+1);\r\nput A=y^(z-1)+...+1;it follow that a is odd such that y-1 divisible to 4\r\nhence you have the solution\r\ngood luck" } { "Tag": [ "modular arithmetic", "number theory proposed", "number theory" ], "Problem": "Hope did not posted before :? \r\nShow that if $a$ and $b$ are positive integers then:\r\n$(a+\\frac{1}{2})^n+(b+\\frac{1}{2})^n$ is an integer for only finetely many positive integers $n$.", "Solution_1": "Take $n$ so large that $2^n$ does not divide $u-v$, where $u=(2a+1)^2,v=(2b+1)^2$. If we assume that $2^n|(2a+1)^n+(2b+1)^n$, then we have $2^n|(2a+1)^n+(2b+1)^n|u^n-v^n$. \r\n\r\nSince the element $uv^{-1}$ is not the unit in the multiplicative group of invertible elements in $\\mathbb Z_{2^n}$, it must have an order which is a power of $2$ greater than $1$ (because th group has $\\varphi(2^n)=2^{n-1}$ elements), and this order must divide $n$ (because $(uv^{-1})^n=1$ in the mentioned group). However, it's easy to see that, in the given conditions, $n$ must be odd if it's larger than $2$ (because otherwise we would have $(2a+1)^n\\equiv(2b+1)^n\\equiv 1\\pmod 4$, so $2^n\\not |\\ (2a+1)^n+(2b+1)^n$). We thus have a contradiction." } { "Tag": [ "modular arithmetic" ], "Problem": "The word MATH is repeatedly written producing the pattern MATHMATHMATH\u2026 If the pattern is continued, what letter will occur in the 2009th position?", "Solution_1": "[hide] Since $ 2009\\equiv1\\pmod4$, the letter $ \\text{M}$ will be in that position.[/hide]", "Solution_2": "[hide]dsince there are 4 letters we divide 2009/4 andget a re,inder of 1 so it is [b]M.[/b][/hide]" } { "Tag": [ "geometry", "number theory proposed", "number theory" ], "Problem": "Let R be a convex region symmetrical about the origin with area greater than 4. \r\nShow that R must contain a lattice point different from the origin.\r\n\r\nThis is the 2-D case of Minkowski's theorem, right ?\r\n\r\nHow about the n-dimensional version ?\r\n\r\nThe n-dimensional version is : Given a convex region R symmetrical to the origin in the n-dimensional space. \r\n\r\nShow that if R has volume greater than 2^n, then R contains a lattice point different from the origin.", "Solution_1": "The proof is the same. You reduce the whole thing to an area of 1 and then apply Blichfeldt's Lemma." } { "Tag": [ "inequalities", "inequalities unsolved" ], "Problem": "Let $ x,y\\ge 0$. Prove that:\r\n\r\n$ x^4y \\plus{} x^2y^3 \\minus{} 2x^3y^2 \\plus{} x^4 \\plus{} y^4 \\plus{} xy^3 \\minus{} 2x^2y^2 \\minus{} x^3y\\plus{}2x^3 \\plus{} 2y^3 \\minus{} 3x^2y\\plus{}x^2 \\minus{} xy \\plus{} y^2\\ge 0$", "Solution_1": "Fix $ x\\plus{}y\\equal{}2t$ and set $ x\\equal{}t\\plus{}\\lambda$, $ y\\equal{}t\\minus{}\\lambda$ ($ x\\geq y$ is the worse case).", "Solution_2": "[quote=\"BG Yoda\"]Fix $ x \\plus{} y \\equal{} 2t$ and set $ x \\equal{} t \\plus{} \\lambda$, $ y \\equal{} t \\minus{} \\lambda$ ($ x\\geq y$ is the worse case).[/quote]\r\n\r\nCan you complete your solution?" } { "Tag": [ "induction", "algebra proposed", "algebra" ], "Problem": "Find all $f: \\mathbb Z \\to \\mathbb Z$ such that $f(x+y+f(y))=f(x)+2y$ for all $ x,y\\in \\mathbb Z$.", "Solution_1": "Hi :) !!!\r\n\r\n$ f(x + y + f(y)) = f(x) + 2y$ $ (E)$\r\n\r\n$ let$ $ f(0) = a \\in \\mathbb{Z}$\r\n\r\nthan $ let$ $ y = 0$:\r\n\r\n$ f(x + a) = f(x)$ \r\n\r\nput $ x = - a$ we find $ f(0) = f( - a) = a$ \r\n\r\nin $ (E)$ put $ x = a$ and $ y = - a$ we find $ f(a - a + f( - a)) = f(a) - 2a %Error. \"Rightlong\" is a bad command.\nf(a) = f(a) - 2a \\Rightarrow a = 0$\r\n\r\nthan $ f(0) = 0$ we returd to $ (E)$:\r\n\r\n$ let \\ \\ \\ x = 0$:\r\n\r\n$ (E)$ become's:\r\n\r\n$ f(y + f(y)) = 2y$ \r\n\r\n$ f$ are bijective\r\n\r\n$ let$ $ g(y) = f(y) + y$ than $ g(g(y)) = f(g(y)) + g(y) = 2y + g(y) = 3y + f(y) = 2y + g(y)$\r\n\r\nthan $ g^{[n]}(y) = \\frac {(2)^n - ( - 1)^n}{3}f(y) + \\frac {2^{n + 1} - ( - 1)^{n + 1}}{3}y$\r\n\r\nthan .....etc\r\n\r\nin conclusion we can find: \r\n\r\n$ f(x) = x$ and $ f(x) = - 2x$ $ \\forall x \\in \\mathbb{Z}$", "Solution_2": "[quote=\"DCTPKTCSPHN\"]Find all $ f: Z\\rightarrow Z$ such that $ : f(x \\plus{} y \\plus{} f(y)) \\equal{} f(x) \\plus{} 2y$ $ \\forall x,y\\in Z$[/quote]\r\nLet $ P(x,y)$ be the equation $ f(x\\plus{}y\\plus{}f(y))\\equal{}f(x)\\plus{}2y$\r\n\r\nLet $ a\\in\\mathbb{Z}$, let $ u\\equal{}a\\plus{}f(a)$ and $ v\\equal{}2a$. We have $ f(x\\plus{}u)\\equal{}f(x)\\plus{}v$ and so, with easy induction, $ f(x\\plus{}nu)\\equal{}f(x)\\plus{}nv$ $ \\forall n\\in\\mathbb{Z}$\r\nFor $ n\\equal{}v$, we get $ f(x\\plus{}uv)\\equal{}f(x)\\plus{}v^2$\r\n\r\nThen, $ P(x\\minus{}v,y\\plus{}u)$ $ \\implies$ $ f(x\\minus{}v\\plus{}y\\plus{}u\\plus{}f(y\\plus{}u))\\equal{}f(x\\minus{}v)\\plus{}2y\\plus{}2u$, and so $ f(x\\plus{}y\\plus{}f(y))\\plus{}v\\equal{}f(x\\minus{}v)\\plus{}2y\\plus{}2u$\r\n\r\nSo we have : $ f(x\\plus{}y\\plus{}f(y))\\equal{}f(x\\minus{}v)\\plus{}2y\\plus{}2u\\minus{}v$\r\nBut : $ f(x\\plus{}y\\plus{}f(y))\\equal{}f(x)\\plus{}2y$ \r\nSo : $ f(x)\\plus{}2y \\equal{} f(x\\minus{}v)\\plus{}2y\\plus{}2u\\minus{}v$\r\nSo : $ f(x)\\equal{}f(x\\minus{}v)\\plus{}2u\\minus{}v$ and so, with easy induction : $ f(x\\plus{}nv)\\equal{}f(x)\\plus{}n(2u\\minus{}v)$ $ \\forall n\\in\\mathbb{Z}$\r\nFor $ n\\equal{}u$, we get $ f(x\\plus{}uv)\\equal{}f(x)\\plus{}u(2u\\minus{}v)$\r\n\r\nBut : $ f(x\\plus{}uv)\\equal{}f(x)\\plus{}v^2$\r\n\r\nSo $ v^2\\equal{}u(2u\\minus{}v)$\r\n\r\nSo $ 4a^2\\equal{}2(a\\plus{}f(a))f(a)$\r\n\r\nSo $ (f(a)\\minus{}a)(f(a)\\plus{}2a)\\equal{}0$ $ \\forall a\\in\\mathbb{Z}$ and so either $ f(a)\\equal{}a$, either $ f(a)\\equal{}\\minus{}2a$\r\n\r\nSo, $ \\forall x$, either $ f(x)\\equal{}x$, either $ f(x)\\equal{}\\minus{}2x$\r\n\r\nAssume now that there exist $ x\\neq 0$ such that $ f(x)\\equal{}x$ and $ y\\neq 0$ such that $ f(y)\\equal{}\\minus{}2y$.\r\n\r\n$ P(x,y)$ $ \\implies$ $ f(x\\minus{}y)\\equal{}x\\plus{}2y$\r\nIf $ f(x\\minus{}y)\\equal{}x\\minus{}y$, we get $ x\\minus{}y\\equal{}x\\plus{}2y$ and so $ y\\equal{}0$\r\nIf $ f(x\\minus{}y)\\equal{}\\minus{}2(x\\minus{}y)$, we get $ \\minus{}2x\\plus{}2y\\equal{}x\\plus{}2y$ and so $ x\\equal{}0$\r\n\r\nSo, either $ f(x)\\equal{}x$ $ \\forall x$, either $ f(x)\\equal{}\\minus{}2x$ $ \\forall x$\r\n\r\nAnd it's easy to verify that these two expressions both are solutions of the initial equation.", "Solution_3": "[quote=\"pco\"][quote=\"DCTPKTCSPHN\"]Find all $ f: Z\\rightarrow Z$ such that $ : f(x \\plus{} y \\plus{} f(y)) \\equal{} f(x) \\plus{} 2y$ $ \\forall x,y\\in Z$[/quote]\nLet $ P(x,y)$ be the equation $ f(x \\plus{} y \\plus{} f(y)) \\equal{} f(x) \\plus{} 2y$\n\nLet $ a\\in\\mathbb{Z}$, let $ u \\equal{} a \\plus{} f(a)$ and $ v \\equal{} 2a$. We have $ f(x \\plus{} u) \\equal{} f(x) \\plus{} v$ and so, with easy induction, $ f(x \\plus{} nu) \\equal{} f(x) \\plus{} nv$ $ \\forall n\\in\\mathbb{Z}$\nFor $ n \\equal{} v$, we get $ f(x \\plus{} uv) \\equal{} f(x) \\plus{} v^2$\n\nThen, $ P(x \\minus{} v,y \\plus{} u)$ $ \\implies$ $ f(x \\minus{} v \\plus{} y \\plus{} u \\plus{} f(y \\plus{} u)) \\equal{} f(x \\minus{} v) \\plus{} 2y \\plus{} 2u$, and so $ f(x \\plus{} y \\plus{} f(y)) \\plus{} v \\equal{} f(x \\minus{} v) \\plus{} 2y \\plus{} 2u$\n\nSo we have : $ f(x \\plus{} y \\plus{} f(y)) \\equal{} f(x \\minus{} v) \\plus{} 2y \\plus{} 2u \\minus{} v$\nBut : $ f(x \\plus{} y \\plus{} f(y)) \\equal{} f(x) \\plus{} 2y$ \nSo : $ f(x) \\plus{} 2y \\equal{} f(x \\minus{} v) \\plus{} 2y \\plus{} 2u \\minus{} v$\nSo : $ f(x) \\equal{} f(x \\minus{} v) \\plus{} 2u \\minus{} v$ and so, with easy induction : $ f(x \\plus{} nv) \\equal{} f(x) \\plus{} n(2u \\minus{} v)$ $ \\forall n\\in\\mathbb{Z}$\nFor $ n \\equal{} u$, we get $ f(x \\plus{} uv) \\equal{} f(x) \\plus{} u(2u \\minus{} v)$\n\nBut : $ f(x \\plus{} uv) \\equal{} f(x) \\plus{} v^2$\n\nSo $ v^2 \\equal{} u(2u \\minus{} v)$\n\nSo $ 4a^2 \\equal{} 2(a \\plus{} f(a))f(a)$\n\nSo $ (f(a) \\minus{} a)(f(a) \\plus{} 2a) \\equal{} 0$ $ \\forall a\\in\\mathbb{Z}$ and so either $ f(a) \\equal{} a$, either $ f(a) \\equal{} \\minus{} 2a$\n\nSo, $ \\forall x$, either $ f(x) \\equal{} x$, either $ f(x) \\equal{} \\minus{} 2x$\n\nAssume now that there exist $ x\\neq 0$ such that $ f(x) \\equal{} x$ and $ y\\neq 0$ such that $ f(y) \\equal{} \\minus{} 2y$.\n\n$ P(x,y)$ $ \\implies$ $ f(x \\minus{} y) \\equal{} x \\plus{} 2y$\nIf $ f(x \\minus{} y) \\equal{} x \\minus{} y$, we get $ x \\minus{} y \\equal{} x \\plus{} 2y$ and so $ y \\equal{} 0$\nIf $ f(x \\minus{} y) \\equal{} \\minus{} 2(x \\minus{} y)$, we get $ \\minus{} 2x \\plus{} 2y \\equal{} x \\plus{} 2y$ and so $ x \\equal{} 0$\n\nSo, either $ f(x) \\equal{} x$ $ \\forall x$, either $ f(x) \\equal{} \\minus{} 2x$ $ \\forall x$\n\nAnd it's easy to verify that these two expressions both are solutions of the initial equation.[/quote]\r\nPco: you have nice solution ,I agree with you :)" } { "Tag": [ "algorithm", "function", "combinatorics unsolved", "combinatorics" ], "Problem": "On a blackboard there are $ n \\geq 2, n \\in \\mathbb{Z}^{\\plus{}}$ numbers. In each step we select two numbers from the blackboard and replace both of them by their sum. Determine all numbers $ n$ for which it is possible to yield $ n$ identical number after a finite number of steps.", "Solution_1": "I think the result holds for every n.\r\nLets go in reverse. Suppose we had n identical numbers on the board. Then , we could replace any two numbers such that the sum of the new numbers is equal to their initial occupants. Now here's a number set with the given property. This way, can't we say that every number n has the given property?\r\nHave I misunderstood the problem? Or is the claim supposed to be proved for a given arbitrary set of numbers?", "Solution_2": "Hi, lajanugen, I think they require solution for arbitrary set of numbers.\r\n\r\nMy Solution:\r\n[hide]\nClaim:\nIt is always possible to equate all numbers [b]iff[/b] $ n$ [b]is even[/b].\n\n\nFirst we prove that for all odd $ n$, there exists some $ n$-tuple such that it's impossible to equate all numbers.\n\nConsider the $ n$-tuple $ (2,2,\\cdots ,2,2,1)$ (total odd numbers)\n\nSuppose to the contrary that it is possible to yield equal numbers.\ni.e. $ m,m,\\cdots ,m,m,m$\n\nClearly $ m>2$\n\nNote that if the operation $ (a,b)\\to (a\\plus{}b,a\\plus{}b)$ can get $ (2,2,\\cdots ,2,2,1)\\to(m,m,\\cdots ,m,m,m)$\nThen the reverse operation $ (p,p)\\to (q,p\\minus{}q)$ can get $ (m,m,\\cdots ,m,m,m)\\to(2,2,\\cdots ,2,2,1)$\n\nNote that the operation $ (p,p)\\to (q,p\\minus{}q)$ decreases both numbers.\nThe operation must act on each $ m$ in the set $ (m,m,\\cdots ,m,m,m)$\n\nAlso this reverse operation requires both numbers to be equal.\nThus it will always act on the $ m$s in pairs.\nBut there are odd number of $ m$s.\nHence there will remain one $ m$ on which this reverse operation never acts.\n\nContradiction.\n\n\nNow it remains to prove that for all even $ n$ every $ n$-tuple can be transformed into equal numbers.\n\nLets treat the $ n$-tuple as a $ m$-tuple of pair of elements of the $ n$-tuple. ($ n\\equal{}2m$)\nThat is $ (a_1,a_2,\\cdots ,a_n)\\equal{}((a_1,a_2),(a_3,a_4),\\cdots ,(a_{2m\\minus{}1},a_{2m}))$\n\nLets do the operation on the 2 elements of each pair.\nLet $ b_i\\equal{}(a_{2i\\minus{}1}\\plus{}a_{2i},a_{2i\\minus{}1}\\plus{}a_{2i})$\n$ ((a_1,a_2),(a_3,a_4),\\cdots ,(a_{2m\\minus{}1},a_{2m}))\\to (b_1,b_2,\\cdots ,b_m)$\n(From now the two elements of each pair will never be different)\nSuppose $ b_i\\equal{}(a,a),b_j\\equal{}(b,b)$\n$ (b_i,b_j)\\to (b_i\\plus{}b_j,b_i\\plus{}b_j)$ is equivalent to $ ((a,a),(b,b))\\to ((a\\plus{}b,a\\plus{}b),(a\\plus{}b,a\\plus{}b))$.\nwhich is the composition of 2 original operations.\n\nActually now we treat the $ n$-tuple as an $ m$-tuple.\nalso note that we now have a extra operation: we can multiply some $ b_i$ by $ 2$ whenever we like, because $ (a,a)\\to (2a,2a)$\n\nThus at some point we can multiply powers of $ 2$ to some $ b_i$s so that the largest power of 2 dividing $ b_i$ are same for all $ b_i$\n\nLet $ o(x)$ denote the largest odd divisor of $ x$.\nIf $ b_i\\not \\equal{}b_j$ have the same largest power of $ 2$ then note that \n$ o(b_i\\plus{}b_j)<\\max(o(b_i),o(b_j))$\n(It's not hard to see, because the sum of odd numbers is even)\n\nAt some particular state of the blackboard define $ l,s$ such that for all $ i$, $ o(b_l)\\ge o(b_i)\\ge o(b_s)$\n\nNow the strategy of obtaining all equal numbers is:\n\n$ \\boxed{}$ operate pairs of the $ n$-tuple and make equal to get it into $ (b_1,b_2,\\cdots b_m)$ form.\n$ \\boxed{}$ get all numbers into equal largest powers of $ 2$\n$ \\boxed{}$ operate on $ (b_l,b_s)$\n$ \\boxed{}$ get all numbers into equal largest powers of $ 2$\n$ \\boxed{}$ operate on $ (b_l,b_s)$\n$ \\boxed{}$ get all numbers into equal largest powers of $ 2$\n$ \\boxed{}$ operate on $ (b_l,b_s)$\n\n$ \\cdots$\n\n\nNow it remains to prove that this strategy will yield equal numbers within finite number of steps.\n\nAfter all \"operate on $ (b_l,b_s)$\" steps, the-largest-odd-number-dividing-at-least-one-$ b_i$ decrease unless $ o(b_l)\\equal{}o(b_s)$\n\nBut the-largest-odd-number-dividing-at-least-one-$ b_i$ can't decrease indefinitely.\nHence, after finite number of steps, $ o(b_l)\\equal{}o(b_s)$\nBut $ o(b_l)\\equal{}o(b_s)$ implies $ o(b_i)$ is same for all $ i$\n\nAnd then we can equate the largest power of $ 2$ of all $ b_i$\nNow we have all equal numbers.\n\nExample:\n\n$ 3,10,7,9,4,6$\n$ 13,13,16,16,10,10$\n$ 26,26,16,16,20,20$\n\n$ \\cdots$\n\n$ 2^4\\cdot 13,2^4\\cdot 13,2^4\\cdot 1,2^4\\cdot 1,2^4\\cdot 5,2^4\\cdot 5$\n$ 2^4\\cdot 14,2^4\\cdot 14,2^4\\cdot 14,2^4\\cdot 14,2^4\\cdot 5,2^4\\cdot 5$\n(from now there won't be any unnecessary repeatations.)\n$ 2^5\\cdot 7,2^5\\cdot 7,2^5\\cdot 5$\n$ 2^5\\cdot 7,2^5\\cdot 12,2^5\\cdot 12$\n$ 2^7\\cdot 7,2^7\\cdot 3,2^7\\cdot 3$\n$ 2^7\\cdot 10,2^7\\cdot 10,2^7\\cdot 3$\n$ 2^8\\cdot 5,2^8\\cdot 5,2^8\\cdot 3$\n$ 2^8\\cdot 5,2^8\\cdot 8,2^8\\cdot 8$\n$ 2^{11}\\cdot 5,2^{11}\\cdot 1,2^{11}\\cdot 1$\n$ 2^{11}\\cdot 6,2^{11}\\cdot 6,2^{11}\\cdot 1$\n$ 2^{12}\\cdot 3,2^{12}\\cdot 3,2^{12}\\cdot 1$\n$ 2^{12}\\cdot 3,2^{12}\\cdot 4,2^{12}\\cdot 4$\n$ 2^{14}\\cdot 3,2^{14}\\cdot 1,2^{14}\\cdot 1$\n$ 2^{14}\\cdot 4,2^{14}\\cdot 4,2^{14}\\cdot 1$\n$ 2^{16}\\cdot 1,2^{16}\\cdot 1,2^{16}\\cdot 1$\n(Note that it might have terminated before reaching all powers of $ 2$, but that doesn't matter)[/hide]\r\n\r\n[b]The solution might look big. It looks big because I have tried to explain things in detail.\nPlease someone check if my solution is understandable/true.\nAnd reply.[/b]", "Solution_3": "[quote=\"Akashnil\"]Please someone check if my solution is understandable/true.[/quote] It looks good to me.", "Solution_4": "Thanks for checking. It took me a lot of time to type it. :roll:\r\n\r\nP.S. What's \"MEMO, Team\" :?:", "Solution_5": "MEMO = Middle European Math Olympiad\r\n\r\nand Team means its from the team competition.\r\n\r\n[hide=\"Solution\"]If $ n$ is odd, then start with the numbers $ (2,2,1,\\dots,1)$. Because we always have either a new maximal value which occurs twice or the total number of maximal values increases by two, there is always an even amount of maximal values, so there can't be $ n$ equal numbers ($ n$ is odd).\n\nIf $ n$ is even, group the numbers in pairs like Akashnil did. Now we define the permutation $ \\sigma$ as follows: If $ a_i$ is even, then $ \\sigma_i \\equal{} i$, and if $ a_i$ and $ a_j$ are both odd, then $ a_i > a_j \\iff a_{\\sigma_i} < a_{\\sigma_j}$ (that is, the odd numbers are sorted in the opposite way). Note that $ \\sigma\\circ\\sigma$ is the identity, so all cycles have length of no more than 2. Now, if $ a_i$ is odd, then replace it and $ a_{\\sigma_i}$ by $ a_i\\plus{}a_{\\sigma_i}$ (obviously, this is done only once for each pair). Then all numbers are even, and dividing by 2 won't change the result in any way. Now set $ f(a) \\equal{} \\sum_{i\\equal{}1}^m a_i^2$ and identify the algorithm with a function $ \\phi$. Then\n\n$ f(a)\\minus{}f(\\phi(a)) \\equal{} \\sum_{i\\equal{}1}^m a_i^2 \\minus{} \\left(\\sum_{a_i\\text{ even}} \\frac{a_i^2}{4}\\plus{}\\sum_{a_i\\text{ odd}} \\left(\\frac{a_i\\minus{}a_{\\sigma_i}}{2}\\right)^2\\right) \\equal{} \\frac{3}{4}\\sum_{a_i\\text{ even}} a_i^2 \\plus{} \\sum_{a_i\\text{ odd}} \\left(\\frac{a_i\\minus{}a_{\\sigma_i}}{2}\\right)^2 \\ge 0$\n\nshows that iteration of $ \\phi$ will make $ f$ decrease. However, $ f$ must be nonnegative; so $ f$ must be constant from some point on, and this is only the case if there are no even numbers and $ a_{\\sigma_i} \\equal{} a_i$, but by the definition of $ \\sigma$, this means that minimum and maximum must be equal, and thus that all numbers must be equal.[/hide]", "Solution_6": "This problem, where the numbers are real numbers instead of positive integers, and where the operation is addition instead of multiplication is also true - came out in a national TST." } { "Tag": [ "MATHCOUNTS", "geometry", "AMC", "USA(J)MO", "USAMO", "FTW" ], "Problem": "Answer the question.\r\n\r\nI slept at around 2:30.", "Solution_1": "I didn't sleep. Instead I played fruits and vegetables (which failed), swam in the pool for a while, etc. Then I completely crashed when I came back home. :D", "Solution_2": "Why are you not sleeping? It's not good for studying!", "Solution_3": "I didn't need sleep...I drank this like powerful energy drink, did random stuff like swam in the pool and threw water balloons at people...\r\n\r\nI did crash at like 2 AM, but I somehow was able to stay awake..", "Solution_4": "I decided to go to sleep at around 1:00 and got a whole 4 hours of sleep. I don't know anyone who slept more than I did. HA! :P", "Solution_5": "I stayed up the whole night with sjcamath!!! :D \r\n\r\nAlthough I did start dozing off in DBR's dorm at around 8 AM... :sleep2:", "Solution_6": "[quote=\"alexhhmun\"]I decided to go to sleep at around 1:00 and got a whole 4 hours of sleep. I don't know anyone who slept more than I did. HA! :P[/quote]\r\n\r\nI went to sleep at 2:30 and got up at 7:30... that's one hour more than you!!!!! HA! :P :P :P", "Solution_7": "oh yeah all of you guys have been owned i slept from 1 to 7 !!! and i woulda slept more if i hadnt been woken up by like 5 people to catch my shuttle bus which was scheduled at 6:50.", "Solution_8": "I slept like 4 hours because I was cleaning up after people when they were leaving in the middle of the night...", "Solution_9": "Well... it was easier for all of you guys to get more sleep since your shuttle left at 8:30 or something. Since my shuttle left around 5:30 it was a bit more difficult... I think not sleeping for a whole night had lasting effects. I think I'm still more tired than usual. :sleeping:", "Solution_10": "I got 6 hours, which seems to be the current record.", "Solution_11": "I tied you zephyredx (almost) \r\nI slept from 3:30 to 9:00\r\nMy bus was supposed to come at 9:00, so that's when I got up.\r\nExcept the bus was an hour late.\r\nSo I could have slept longer instead of having to sleep on the plane. :mad:", "Solution_12": "haha people actually slept on the last day?!?\r\nWHATT?!??\r\nyoure supposed to hang out&make the final memories with one anotherrrr!!\r\n:O\r\n\r\ni was in someone else's dorm the entiiireee night lol\r\nwatching movies online :P", "Solution_13": "[quote=\"sjcamath\"]Instead I played fruits and vegetables (which failed)[/quote]\r\n\r\nMy impression of Ice actually beat meenamathgirl! :D\r\n\r\nI tried to but I finally gave up and had some half-hour snoozes at the pool. I was packed and ready to go before midnight ( :o )", "Solution_14": "No sleep woot. but I did randomly venture into the pool area... and coincidentally met jeff and COUGH i wonder there... lol. i think they left their room after i randomly walked in and started watching that korean soap opera with them", "Solution_15": "Overachievers? You haven't seen Gunn...teachers here are on crack, yet there are people who have half A+s half As, and taking all APs. I'm probably the biggest slacker, my grade report generally looks like: A- A- A- A- A- A- B+ (when an 90.1 accidentally slips :()\r\n\r\nThat is a lot of caffeine. When I was at Mathcounts State a long time ago, I sucked really bad at math, but I somehow got much better than I ever could have without caffeine. In the morning, as soon as I got off the car I drank one, to cure my carsickness+sleepyness. Then before the contest, I got really thirsty and had no water, so I drank another. Then during the contest, I finished that one and got thirsty again, and drank a third. Halfway through the third I wasn't really thirsty anymore, but it felt cool to be sipping that kind of junk.\r\n\r\nThe crash happened at 4am in the morning, 20 hours later :O\r\n\r\nI'm pretty sure my caffeine saved me yesterday, since I had 3 finals and I slept 2 hours...", "Solution_16": "hahahahahaa dude i agree that Gunn's pretty insane too..\r\nbut try surviving at a school with asian population of 48%. :O at least Gunn isn't infected with asian to this extent. [but i gotta admit gunn has some hella smart matheletes. my school doesnt since they all graduated last year]\r\nwe're all loaded with APs...who isnt? [me! but thats only cuz i take the courses outside of school at a community college to save some of my sanity...]\r\n\r\nhaha i dont crash on daily basis but i crash like twice a week. my stress will just stack up to the point where i cant take it anymore and BOOM.\r\nlike last night when i almost fell asleep on jeff while talking on the phone xP\r\n\r\nHAHAHA I LOVE CAFFEINE. [shelly32494, dont shoot me! T_T roflmao xD]", "Solution_17": "*take out gun* jkjkjk. lolll. \r\njust try and cut the caffiene? \r\nlol serialk11r, is there a diff. between A-, A, A+ ?? \r\nhow bout the entire state of cali consists of hecka crazyyy asian overacheivers. and to relieve stress. move out to WI. XP\r\nwellll thxfully my 'rents ban caffeine/i cant drive/its too cold to walk so... no starbucks/energy drinks for me lol XP ive learned to live w/o it -.-", "Solution_18": "They misspelled my AoPS username in the yearbook. It's bpgbcg, not bpglrg. (My handwriting is terrible, which is probably what caused the mistake.)", "Solution_19": "i tried to live without it but my parents drink coffee quite frequently too, so there's infinite supply of coffee stuff at my house! :D\r\nyayyy ahahahahhahaa xP\r\nso bad.\r\nbut yeah.\r\ndude..90 degrees todayy x.X when i got in my car after my prep courses..it was BLAZING HOT in there. yikkkes. im so glad i dont have black interior [first of all it would look FUGLY with baby blue exterior ew ew ew]", "Solution_20": "bpgbcg: i found u in the yrbk!! lol. i just had to know -.-\r\n\r\nlol. it got warmer today!!! :D", "Solution_21": "Oh dang 48% is pretty crazy...Gunn is only around 35% I think. We also have 10% Jews, and a few racist Jewish teachers to go with them. However down in Cupertino there's Monta Vista which is 75% asian :)\r\nAt Gunn, there is no difference between A-, A, A+ GPA wise, but I know hella people who get disappointed when they get an A-. On person who lives on my street (Jeff also lives on my street actually...my street has hella Gunn kids) has parents who didn't let her go to homecoming dance because she got an A- in bio...", "Solution_22": "=_='' \r\nthen thats just stupiddddd. asian parents -.- srsly. \r\nan A's an A. good god. -_-;; lol i have an increased appreciation for my 'rents now lol XP\r\n... \r\ncrazy. amt. of. asians. xO", "Solution_23": "[quote=\"sooozyy\"]hahahahahaa dude i agree that Gunn's pretty insane too..\nbut try surviving at a school with asian population of 48%. :O at least Gunn isn't infected with asian to this extent. [but i gotta admit gunn has some hella smart matheletes. my school doesnt since they all graduated last year]\nwe're all loaded with APs...who isnt? [me! but thats only cuz i take the courses outside of school at a community college to save some of my sanity...]\n\nhaha i dont crash on daily basis but i crash like twice a week. my stress will just stack up to the point where i cant take it anymore and BOOM.\nlike last night when i almost fell asleep on jeff while talking on the phone xP\n\nHAHAHA I LOVE CAFFEINE. [shelly32494, dont shoot me! T_T roflmao xD][/quote]\r\n\r\nONLY 48%!!!!!!!!!!!!\r\n\r\nMy school's about 80% asian...hecka awesomeness", "Solution_24": "lol 80% is insane!\r\n\r\n\r\nwhich school?\r\n\r\ni know that uni high in irvine is close to 60%? [Dai, correct me if im wrong?]\r\n\r\nanyways irvine is asian...WE HAVE A KOREAN MAYOR.\r\nthat just blows it allllllll.", "Solution_25": "is dai that guy who sang,'everybody likes problem-solving'??becuz if he is,im still mad @ him for making briam do that stupid sing-a-long for 'everytime we touch'.i mean,EVERYTIME WE TOUCH?!?!?!?!WTF!!!!!!!!!o,and WAYYYYYYYYYYYYY bak on the 'did u sleep' i stayed up with katherine and sumun and carl and um............o yea sasha playing volleyball..........erik and meena were somewhere else...........o, and this is steve", "Solution_26": "omg. steveeee!!!!! \r\nyes that is that dai. \r\nnd omg. hey. i liked everytime we touch it was a good song. and basham was amazinggg at it :) \r\nlol. wow we got rly off topic. \r\n\r\nhey . well. my school's like. 80% white. ahaha.", "Solution_27": "lol typical us.\r\ndigressing :P\r\n\r\nOMG HI STEVE! :)))))))\r\n\r\n\r\n agreed with shelly32494, i liked that song! =]", "Solution_28": "Carbon57 goes to Lynbrook.\r\nAt Gunn there's a senior who's the nephew of the new Secretary of Energy :D\r\nBay Area is definitely like, the most asian place in the world outside of asia.", "Solution_29": "lol...Yeah...Lynbrook have A- or A+, just As, and nobody cares about it, since it doesn't go on our semester grades, but our school's still insanely competitive though...too many asians trying to get 4.0 and shiz\r\n\r\nYup...bay area is asian since it's the closest place (except Hawaii and Alaska) to Asia in America...and a lot of tech jobs" } { "Tag": [ "superior algebra", "superior algebra unsolved" ], "Problem": "Let A be a commutative ring with 1. We want to show that the Jacobson radical is an ideal in A. Also how would I show A/R (where R=nilradical of A) has no nonzero nilpotent elements. Furthermore, how can I show that an element a can be in the Jacobson radical iff 1-ab is invertible for all b in A. Also I want to show that the nilradical is contained the Jacobian. Any help would be great :)", "Solution_1": "What's your definition of the Jacobson radical? By the usual definition as intersection of all maximal left/right ideals, it's without question an ideal, therefore I ask. But anyway, you can find the proofs here :-)\r\n \r\nhttp://www.mathreference.com/ring-jr,intro.html\r\n\r\nBesides, what's the Jacobian radical? I have not found any definition by google :?", "Solution_2": "A Jacobson radical is the intersection of all maximal ideals of a commutative ring. I am still not clear on how to show an element a in the Jacobson(A) iff 1-ab is invertible for all b in A.", "Solution_3": "Just open the link. There's the proof.\r\n \r\nDo you mean with Jacobian the same as Jacobson? :-) Then the nilradical is of course in it because the nilradical is the intersection of all prime ideals." } { "Tag": [ "algebra proposed", "algebra" ], "Problem": "How could 'change' the way we think of 'cardinals' but even our 'universe' (sets) if we defined alternatively when 2 sets have are equinumerous? Do such definitions exist? I think that such a definition could 'work' for finite sets and the 'new' results could appear only when talking abut infinite sets. Any ideas?", "Solution_1": "In the future, please don't post the same topic in multiple forums, and see my comment in your other thread. Another comment:\r\n\r\nI think one of the problems with finding an alternate definition of cardinality is that, from a purely set-theoretic point of view, and ignoring any other properties of a set, cardinality (as it is usually defined) is the [i]only[/i] way we have to distinguish two abstract sets. So any alternate (in particular, any stricter) definition of cardinality would need to be specific to certain types of sets." } { "Tag": [ "algebra", "polynomial", "abstract algebra", "real analysis", "real analysis solved" ], "Problem": "Prove that the polynomial Xn + aXn-1 + ... + aX - 1, with n \\geq 2 and a \\in Z\\{0} is irreducible over Z[X].", "Solution_1": "well I've used this problem in MathLinks contest, 1st Edition, Round 3, problem 2 (the well-known Round 3 :D) ... :P", "Solution_2": "If I'm not wrong, a particular case of this problem (a=-1) appeared in AMM! So, imagine its difficulty!", "Solution_3": "Suppose WLOG a<0, b=|a|.\r\nLet p(x)=xn-bxn-1-bxn-2-...-bx-1.\r\nConsider f(x)=(x-1)p(x)=xn+1-(b+1)xn+(b-1)x+1, g(x)=xn+1-(b+1)xn.\r\nLet t>0 be a small number, z be complex number and |z|=1+t. We have |f(z)-g(z)|=|(b-1)z+1| \\leq (b-1)(1+t)+1. In the other hand |f(z)|+|g(z)| \\geq |g(z)|=(1+t)n(a-t). So |f(z)-g(z)|<|f(z)|+|g(z)| for all sufficiently small t.\r\nDue to Rushe's theorem f(z) and g(z) have equal number of roots inside |z|<1+t. It follows that f(z) has exactly n roots in |z|\\leq 1. \r\nIt is easy to show that z=1 is only root of f(z) on circle |z|=1, so p(z) has exactly (n-1) roots inside |z|<1 and single root outside of |z|<1.\r\n\r\nSuppose p(x)=u(x)v(x), where u,v \\in Z[x]. Then -1=u(0)v(0) ==> u(0),v(0)=\\pm 1 ==> both u(z) and v(z) have root in |z| \\geq 1==> p(z) has at least two roots outside of |z|<1 - contradiction.\r\n\r\n\r\nThere are common theorem:\r\nLet a1 \\geq a2 \\geq ... \\geq an \\geq 1, n \\geq 2. Then f(x)=xn-a1xn-1-...-an-1x-an is irreducibale over Z[x].", "Solution_4": "For a polinomial $p=\\sum\\limits_0^na_iX^{n-i}\\in Z[X],$ we define $q=\\sum\\limits_0^na_iX^i\\in Z[X],$ and $h=\\frac{p+q}{2}, k=\\frac{p-q}{2}.$\r\nI will use this facts:\r\n1. Every zero of p which lies on the unit circle is a common zero for h and k.(easy to prove)\r\n2.If $p_{\\lambda}=X^n+a_1(\\lambda)X^{n-1}+...+a_n(\\lambda)$ is a family of polynomials with the property that $p_{\\lambda}(z)\\neq 0,$\r\nfor z in the unit circle, then $p_{\\lambda}$ has the same number of zeros in the inetrior of the unit circle.(a little harder.)\r\nBack to problem. We consider a>0.\r\n$p=x^n+aX^{n-1}+...+aX-1.$ We consider the family of polynomials:\r\n$p_t=x^n+(at+1-t)X^{n-1}+...+(at+1-t)X-1, p_1=p,$ and \r\n$p_0=x^n+X^{n-1}+...+X-1.$ We show that if |z|=1 $p_t\\neq 0.$ Indeed, if exists z such that |z|=1 and $p_t(z)=1,$ with 1., we obtain thar z is a common zero for $h_t, k_t$ But\r\n$h_t=(at+1-t)(X^{n-1}+...+x=(at+1-t)X(X^{n-2}+...+1), and$\r\n$k_t=x^n-1.$ So z is a root of order n-1 fespectiv n of the unity. Because n and n-1 are relative prime we obtain that z=1, which is not a solution for $p_t.$ We can apply 2. to conclude that $p=p_1$ have the same number of roots in the interior of the unit circle as $p_1.$ I'll prove that $p_1$ have one zero in the interior of the unit circle.\r\nConsider the family $q_t=tx^n+X^{n-1}+...+x-1, q_1=p_1, q_0=X^{n-1}+...+x-1.$ In the same way we can prove that on yhe unit circle $q_t$ have no zeros, so the number of zeros in the unit circle for $p_1=q_1$ is the same with the number of zeros in the unit circle for $q_0.$ Apply the same reasoning n_2 times we conclude that the number of zeros in the unit circle for $p_1$ is the same with the number of zeros in the unit circle for $x^2+x-1$ which is 1.\r\nSo qur polynomial have n-1 roots with supraunitary module. If p is reducibile, one of the factors will have all zeros supraunitary in module, which is imposibile because the free term is one. :lol:" } { "Tag": [ "Euler", "geometry", "angle bisector", "power of a point", "radical axis", "geometry proposed" ], "Problem": "From a point $Q$ on a circle with diameter $AB$ different from $A$ and $B$, we draw a perpendicular to $AB$, $QH$, where $H$ lies on $AB$. The intersection points of the circle of diameter $AB$ and the circle of center $Q$ and radius $QH$ are $C$ and $D$. Prove that $CD$ bisects $QH$.", "Solution_1": "Mmm ! A really nice problem ! Please draw a gifure.\r\n\r\n Let $QH$ meets $CD$ at $M$ and $OQ$ meets $CD$ at $N$ , where $O$ is the center of the great circle. Let line $QO$ , produced , meets the circle $O$ at $E$. Then $OHMN$ is inscribed and :\r\n\r\n $QM \\cdot QH = QN\\cdot QO = \\frac{1}{2} QN \\cdot QE = \\frac{1}{2} QC^2 = \\frac{1}{2} QH^2$\r\n\r\n Hence \r\n\r\n $QM \\cdot QH = \\frac{1}{2} QH^2$ or $QM = MH$ \r\n\r\n[u]Babis[/u]", "Solution_2": "[quote=\"Valentin Vornicu\"]From a point $Q$ on a circle with diameter $AB$ different from $A$ and $B$, we draw a perpendicular to $AB$, $QH$, where $H$ lies on $AB$. The intersection points of the circle of diameter $AB$ and the circle of center $Q$ and radius $QH$ are $C$ and $D$. Prove that $CD$ bisects $QH$.[/quote]\r\n\r\nActually is rather simple provided inversion is applied... Center $Q$ and radius $QH$. The big circle turns into line $CD$ and viceversa. If $X$ is the other point where $QH$ meets big circle and $Y$ is the point of intersection of $CD$ and $QH$ then it must be\r\n$QY \\cdot QX = QH^2 \\Rightarrow \\frac{QY}{QH} = \\frac{QH}{QX} = \\frac{1}{2}$ \r\n\r\nDaniel", "Solution_3": "[quote=\"stergiu\"]\n$\\frac{1}{2} QN \\cdot QE = \\frac{1}{2} QC^2$ [/quote]\r\n\r\nDon't understand this part, could someone explain me? :blush:", "Solution_4": "[quote=\"M4RI0\"][quote=\"stergiu\"]\n$\\frac{1}{2} QN \\cdot QE = \\frac{1}{2} QC^2$ [/quote]\n\nDon't understand this part, could someone explain me? :blush:[/quote]\r\n\r\nThe triangle $ECQ$ is right and $CN$ is its height . In Greece we called it as <> and you can prove it by similar triangles.", "Solution_5": "[quote=\"M4RI0\"][quote=\"stergiu\"]\n$\\frac{1}{2} QN \\cdot QE = \\frac{1}{2} QC^2$ [/quote]\n\nDon't understand this part, could someone explain me? :blush:[/quote]\r\n\r\nThe similarity of the triangles gives us this relation. Since $QN$ is the diameter of the first cirlce we have $CN$ perpendicular to $CQ$. Aslo we know that the diagonals of kites are perpendicular to each other which is : the diagonals of a kite $QCOD$ are $QO$ and $CD$ are perpendicular to each other. Therefore we have a similarity of the triangles $QCP$ and $QNC$. In other words, $\\frac{QC}{QN}= \\frac{QP}{QC}$, quivalent to $QC^2=QN \\times QP$\r\n\r\nNote that the typos are according to my own figure\r\n- $O$ is the centre of the first circle\r\n- $Q$ is the centre of the second circle\r\n- $N$ is the intersection of point of $QO$ with the first circle \r\n- $P$ is the intersection point of $CD$ with $QO$\r\n\r\n[color=red][Maybe they are the same but i have no time check, sorry][/color]\r\n\r\nDavron", "Solution_6": "Let V be the intersection point of OH with circle O and let N be the intersection point of DC and QH. \r\nDenote angle QDN with x and angle HDC with y.\r\nSince QH=QD we have angleDQH=180-2(x+y) so angleDCH=1/2angleDQH=90-(x+y).\r\n ALso we have angleDCV=angleDCH+angleHCV=90-(x+y) + angleHCV so angleHCV=90-(x+y) so this means that HC is angle bisector of angleNCV in triangle NCV and from this we conclude that CN/CV=NH/HV or NH=CN/CV*HV (1)\r\nAlso we have angleQVC=angleQDC=angleQCD=x and since angleNQC=angleVQC it follows that triangles QNC and QVC are similar so QN/NC=QC/VC or QN=NC/VC*QC (2)\r\nPoint H is a feet of perpendicular from O to VQ in an triangle VOQ (VO=OQ) so QH=HV.\r\nAlso QH=QC (radius of circle with radius QH) so we conclude that HV=QC (*)\r\nNow, from (1), (2) and (*) we conslude that NH=NQ so it means that CD bisects QH. q.e.d. :lol: [/i]", "Solution_7": "[quote=\"delegat\"]Let $V$ be the intersection point of [b][color=red]QH[/color][/b] with circle $O$ - the center of the first circle - and let $N$ be the intersection point of $DC$ and $QH$. \nDenote $\\angle QDN = x$ and $\\angle HDC = y$.\nSince $QH=QD$ we have \n $\\angle DQH=180-2(x+y)$ so $\\angle DCH= \\frac{1}{2} \\angle DQH=90-(x+y)$.\n Also we have $\\angle DCV=angleDCH+ \\angle HCV=90-(x+y) + \\angle HCV$ , so \n $\\angle HCV=90-(x+y)$ .This means that $HC$ is angle bisector of $\\angle NCV$ in triangle $NCV$ and from this we conclude that\n $\\frac{CN}{CV} = \\frac{NH}{HV}$ or $NH= \\frac{CN}{CV} \\cdot HV$ [b](1)[/b]\nAlso we have\n $\\angle QVC= \\angle QDC= \\angle QCD=x$ \n and since $\\angle NQC= \\angle VQC$ , \n it follows that triangles $QNC$ and $QVC$ are similar so \n $\\frac{QN}{NC}= \\frac{QC}{VC}$ or $QN= \\frac{NC}{VC} \\cdot QC$ [b](2)[/b]\n\nPoint $H$ is a feet of perpendicular from $O$ to $VQ$ in $\\triangle VOQ , (VO=OQ)$ so $QH=HV$ .\n\nAlso $QH=QC$ (radius of circle with radius $QH$) so we conclude that $HV=QC$ [b](*)[/b]\nNow, from $(1), (2)$ and [b](*)[/b] we conslude that \n $NH=NQ$ , which means that $CD$ bisects $QH$. q.e.d. :lol: [/i][/quote]\r\n\r\n[b] I simply made a little correction(...typo) to this elementary and nice solution![/b]\r\n[u]Babis[/u]", "Solution_8": "[quote=\"stergiu\"]\n$QM \\cdot QH = \\frac{1}{2} QH^2$ or $QM = MH$ \n[u]Babis[/u][/quote]\r\n \r\nIt follows from Euler theorem in right triangles\r\nSince $QE$ is diameter so triangle QCE is right at $\\angle C$ and foot of altitude from $\\angle C$ is $N$\r\n So $CQ^2= QN\\times QE$", "Solution_9": "Let us denote the the circle with centre Q and radius QH by W. Let CD intersect QH at M. Extend QH to meet the circle with diameter AB at M'. (Note that since QH is perpendicular to the diameter AB, H is the midpoint of QM'.) \r\n\r\nNow, consider inversion with respect to the circle W. Circle with diameter AB passes through Q, the centre of W. Therefore that circle inverts into a line. Further, the points C and D are fixed under inversion because they lie on W. This implies that the circle with diameter AB inverts into line CD.\r\n\r\nThus the inverse of point M' is M. Now ,\r\nQM' = 2QH.\r\nQM * QM' = (QH) ^ 2 (M is inverse of M')\r\nSo, QM = 1/2 QH \r\nThus M is midpoint of QH. \r\n\r\nQED", "Solution_10": "Please see post 3 , by Daniel ;) \r\n\r\n [u]Babis[/u]", "Solution_11": "Is it so simple or am I doing something wrong?\n\nLet $M$ be the intersection of $CD$ and $QH$.\n$QD = QC \\Rightarrow \\angle QDC = \\angle QCD = \\angle QRC \\Rightarrow QH^2 = QC^2 = QM\\cdot QR = QM\\cdot 2QH \\Rightarrow QH = 2QM $", "Solution_12": "[quote=\"xeroxia\"]Is it so simple or am I doing something wrong?\n\nLet $M$ be the intersection of $CD$ and $QH$.\n$QD = QC \\Rightarrow \\angle QDC = \\angle QCD = \\angle QRC \\Rightarrow QH^2 = QC^2 = QM\\cdot QR = QM\\cdot 2QH \\Rightarrow QH = 2QM $[/quote]\n\nThis solution is correct, very nice and simple too, but probably not easier than the solution with inversion !\n\n This solution has two important points :\n\n- To take the whole circle and produce $AH$\n\n- To prove that $QC$ is tangent with the circle $(R,M,C) $ \n\nOf course it is not an national or IMO level olympiad problem, it is easy for senior students but is still an ellegant and intreresting problems for juniors.\n\n Thank you for sending your solution !\n\nBabis", "Solution_13": "It is a team selection test problem. I think the proposers missed the easy solution.", "Solution_14": "[hide=\"Picture\"][img]http://i.snag.gy/Mcw4T.jpg[/img][/hide]\n\nLet $M$ be the midpoint of $QH$, and let $QH$ cut $(AQB), (CHD)$ for a second time at $P,N$ respectively. Now $MQ\\cdot MP=(\\frac{1}{3} MN)(3 MH)=MN\\cdot MH$, so $M$ has equal powers with respect to both circles. Thus $M$ lies on the radical axis $CD$.", "Solution_15": "[quote=\"xeroxia\"]It is a team selection test problem. I think the proposers missed the easy solution.[/quote]\n\nYes !\n\nIn my notes I see that it is a problem in some Bulgarian olympiad, probably tst 2004.\n\nBut it is olympiad problem for some other country too !\n\nYour proof is original and I have not seen it before.\n\nBabis" } { "Tag": [ "function", "calculus", "algebra", "polynomial", "functional equation", "algebra unsolved" ], "Problem": "Find all functions $f : N \\Rightarrow N$ such that\r\n$f(f(f(n))) + 6f(n) = 3f(f(n)) + 4n + 2001$; $n \\in N$", "Solution_1": "Well I don't have much time so only the idea.... \r\ndefine sequence a_i as a_1=n, a_2=f(n), a_3=f(f(n)) etc.\r\nFrom the equation we'l get a recursive formula with characteistic polynom which has roots\r\n$1-\\sqrt 3i$,$1+\\sqrt 3i$ and twice 1, so $a_i=k.2^i.sin(i.\\pi/3+\\phi)+li+m$. For k not null it diverges very fast so k=0. Pluging $a_i=li+m$ into the equation\r\n4l+m + 6(2l+m) = 3(3l+m) + 4l+4m + 2001,\r\n3l = 2001, so l=667. \r\nBecause of that\r\nf(n)-n=a_2-a_1=667,\r\nf(n)=n+667.", "Solution_2": "[quote=\"Kondr\"]Well I don't have much time so only the idea.... \ndefine sequence a_i as a_1=n, a_2=f(n), a_3=f(f(n)) etc.\nFrom the equation we'l get a recursive formula with characteistic polynom which has roots\n$1-\\sqrt 3i$,$1+\\sqrt 3i$ and twice 1, so $a_i=k.2^i.sin(i.\\pi/3+\\phi)+li+m$. For k not null it diverges very fast so k=0. Pluging $a_i=li+m$ into the equation\n4l+m + 6(2l+m) = 3(3l+m) + 4l+4m + 2001,\n3l = 2001, so l=667. \nBecause of that\nf(n)-n=a_2-a_1=667,\nf(n)=n+667.[/quote]\r\n\r\nI really don't understand you .\r\n\r\nPlease post it clearier.\r\n\r\nDo you use calculus to do it?\r\n\r\nPlease , Post you solution without calculas!", "Solution_3": "Where did you stop?\r\nI mean, what can't you understand? The whole thing, or just from a certain point?", "Solution_4": "Ok The whole thing ! :)", "Solution_5": "Ok, then, let's follow and detail [b]Kondr[/b]'s reasoning :\r\nWe have : $f(f(f(n))) + 6f(n) = 3f(f(n)) + 4n + 2001$.\r\nNow, like he says :\r\n[quote=\"Kondr\"] \ndefine sequence a_i as a_1=n, a_2=f(n), a_3=f(f(n)) etc.[/quote]\r\nSo, our functional equation becomes :\r\n$a_4 - 3a_3 + 6a_2 - 4a_1 - 2001 = 0$\r\nSo, our recursive formula is :\r\n$a_{n+3} - 3a_{n+2} + 6a_{n+1} - 4a_n - 2001 = 0$.\r\nThe characteristic polynomial is thus :\r\n$r^3 - 3r^2 + 6r - 4 = 0$\r\nAnd, from this, if we solve in $\\mathbb{C}$, we get :\r\n$r = 1 - \\sqrt{3}i$ or $r = 1 + \\sqrt{3}i$ or $x=1$(1 being a double root).\r\n\r\n\r\nSo far, do you understand?\r\nIf yes, then I'll go on. ;)", "Solution_6": "Thank you very very very much ,mathmanman. You are very kinds of me!\r\n\r\nBut Ok!I don't anything know about characteristic polynomial.\r\n\r\nIs it hard or long ? Give me some definitions about that please.\r\n\r\nPs.Thank you very much", "Solution_7": "Ok! :)\r\n\r\nThen, I suggest you this :\r\nhttp://www.mathpages.com/home/kmath003/part3/sympart3.htm\r\n\r\nAnd, even better :\r\nhttp://mathcircle.berkeley.edu/BMC3/Bjorn1/Bjorn1.html\r\n\r\n\r\nYou're welcome. ;)" } { "Tag": [ "algebra unsolved", "algebra" ], "Problem": "$ A\\equal{}(1^{4}\\plus{}1/4)(3^{4}\\plus{}1/4)(5^{4}\\plus{}1/4)...(49^{4}\\plus{}1/4)$\r\n$ B\\equal{}(2^{4}\\plus{}1/4)(4^{4}\\plus{}1/4)(6^{4}\\plus{}1/4)...(50^{4}\\plus{}1/4)$\r\nFind $ A/B$", "Solution_1": "Hint: Use $ x^{4}\\plus{}\\frac{1}{4}\\cdot y^{4}\\equal{}(x^{2}\\plus{}\\frac{1}{2}\\cdot y^{2}\\minus{}xy)(x^{2}\\plus{}\\frac{1}{2}\\cdot y^{2}\\plus{}xy)$.", "Solution_2": "It was a problem from TTT 2 in Vietnam :D" } { "Tag": [ "inequalities", "function", "inequalities unsolved" ], "Problem": "Let $a,b,c$ be the sides of a triangle. Prove that:\r\n$\\sqrt{a+b-c}+\\sqrt{b+c-a}+\\sqrt{c+a-b}\\leq\\sqrt{a}+\\sqrt{b}+\\sqrt{c}$\r\n\r\nI made a part of the solving but cant get to its end :(. Here it is:\r\n$a=y^2+z^2;b=x^2+z^2;c=x^2+y^2$\r\nNow\r\n$x\\sqrt{2}+y\\sqrt{2}+z\\sqrt{2}\\leq\\sqrt{x^2+y^2}+\\sqrt{x^2+z^2}+\\sqrt{y^2+z^2}$\r\n$2xy+2yz+2xz\\leq\\sqrt{(x^2+y^2)(x^2+z^2)}+\\sqrt{(x^2+z^2)(y^2+z^2)}+\\sqrt{(x^2+y^2)(y^2+z^2)}$\r\nSince\r\n$\\sqrt{(x^2+y^2)(x^2+z^2)}+\\sqrt{(x^2+z^2)(y^2+z^2)}+\\sqrt{(x^2+y^2)(y^2+z^2)}\\geq\\sqrt{2xy(x^2+z^2)}+\\sqrt{2xz(y^2+z^2)}+\\sqrt{2yz(x^2+y^2)}$\r\nSomebody :roll: just has to prove that:\r\n$\\sqrt{2xy}(\\sqrt{2xy}-\\sqrt{x^2+z^2})+\\sqrt{2xz}(\\sqrt{2xz}-\\sqrt{y^2+z^2})+\\sqrt{2yz}(\\sqrt{2yz}-\\sqrt{x^2+y^2})$\r\n\r\n$xy+yz+zx\\leq x^2+y^2+z^2$ but I think this isnt enough to solve that problem :huh:", "Solution_1": "Here is a nicer solution for this one: We only use Cauchy-Schwarz inequalilty in the form $\\sqrt{x}+\\sqrt{y}\\leq\\sqrt{2(x+y)}$.Applying this inequality we get $\\sqrt{b+c-a}+\\sqrt{c+a-b}\\leq\\sqrt{2\\cdot 2c}$ and we write the similar ones and by adding them we get the desired result. ;)", "Solution_2": "[quote=\"Gohan\"]Let $x,y,z$ be the sides of a triangle. Prove that:\n$\\sqrt{a+b-c}+\\sqrt{b+c-a}+\\sqrt{c+a-b}\\leq\\sqrt{a}+\\sqrt{b}+\\sqrt{c}$\n\nI made a part of the solving but cant get to its end :(. Here it is:\n$a=y^2+z^2;b=x^2+z^2;c=x^2+y^2$\n [/quote]\r\n\r\nLet's go from the beginning, nice and easy.... what of these are sides of a triangle $a,bc$ or $x,y,z$ \r\n\r\n :) :) :) :) \r\n\r\nbye......", "Solution_3": "[quote=\"cezar lupu\"]We only use Cauchy-Schwarz inequalilty in the form $\\sqrt{x}+\\sqrt{y}\\leq\\sqrt{2(x+y)}$[/quote]\r\n\r\nHuh .. :huh: I wish I knew all tha formulas :P.", "Solution_4": "Come on,It is just Cauchy-Scwarz, :P", "Solution_5": "Its APMO 1996 . An easy solution is to consider [tex] a\\geq b\\geq c[/tex] . But then [tex](a+b-c,c+a-b,b+c-a)>(a,b,c).[/tex] Aply Karamata for function [tex] f(x)=\\sqrt x [/tex]", "Solution_6": "Maybe,you shouldn't call it Karamata's inequality because it isn't his. Actually this is Hardy-Littlewood-Polya inequality. ;)" } { "Tag": [], "Problem": "Cho m,n,x,y,z l\u00e0 c\u00e1c s\u1ed1 th\u1ef1c d\u01b0\u01a1ng.Ch\u1ee9ng minh r\u1eb1ng:\r\n (mx+ny)/(mz+nz)+(my+nz)/(mx+nx)+(mz+nx)/(my+ny)>=3\r\n Xin l\u1ed7i c\u00e1c b\u00e1c em kh\u00f4gn bi\u1ebft \u0111\u00e1nh k\u00ed hi\u1ec7u n\u00ean n\u00f3 h\u01a1i kh\u00f3 \u0111\u1ecdc. B\u00e1c n\u00e0o d\u1ea1y em c\u00e1i!!", "Solution_1": "B\u1ea5t \u0111\u1eb3ng th\u1ee9c \u0111\u00e3 cho t\u01b0\u01a1ng \u0111\u01b0\u01a1ng v\u1edbi:\r\nm( x/z + z/y + y/x) + n( y/z + z/x + x/y) >= 3(m + n)\r\n\u00c1p d\u1ee5ng B\u0110T C\u00f4si (v\u00ec x, y, z, l\u00e0 c\u00e1c s\u1ed1 th\u1ef1c d\u01b0\u01a1ng) ta \u0111\u01b0\u1ee3c : \r\n(x/z + z/y + y/x) >= 3 v\u00e0 (y/z + z/x + x/y) >= 3\r\nV\u1eady:\r\nm( x/z + z/y + y/x) + n( y/z + z/x + x/y) >= 3m + 3n = 3(m + n)\r\n\u0110\u1eb3ng th\u1ee9c x\u1ea3y ra khi v\u00e0 ch\u1ec9 khi x = y = z", "Solution_2": "Th\u1ebf c\u00f2n:\r\nx/(my+nz)+y/(mz+nx)+z/(mx+ny)>=3/(m+n)", "Solution_3": "[quote=\"tmathst\"]\n Xin l\u1ed7i c\u00e1c b\u00e1c em kh\u00f4gn bi\u1ebft \u0111\u00e1nh k\u00ed hi\u1ec7u n\u00ean n\u00f3 h\u01a1i kh\u00f3 \u0111\u1ecdc. B\u00e1c n\u00e0o d\u1ea1y em c\u00e1i!![/quote]\r\nC\u1ed1 th\u1eed d\u1ecbch c\u00e1i n\u00e0y nh\u00e9 b\u1ea1n http://www.artofproblemsolving.com/LaTeX/AoPS_L_About.php" } { "Tag": [ "absolute value" ], "Problem": "What is the positive difference between the two solutions of $ |x \\plus{} 5| \\equal{} 20$?", "Solution_1": "one solution is if x+5=20, so x=15. The other solution is if x+5= -20, so x=-25. The positive difference ie absolute value equals 15--25=40.", "Solution_2": "You can generalize and say that for any equation of this type $ |{mx \\plus{} a}| \\equal{} n$, the postive difference between the solutions is $ |\\frac {2n}m|$. This is because:\r\n\r\n$ |{mx \\plus{} a}| \\equal{} n$\r\nso\r\n$ mx \\plus{} a \\equal{} n$\r\n$ x \\equal{} \\frac {n \\minus{} a}m$\r\nor\r\n$ mx \\plus{} a \\equal{} \\minus{} n$\r\n$ x \\equal{} \\frac { \\minus{} n \\minus{} a}m$\r\nFinding the difference gives\r\n$ \\frac {n \\minus{} a}m \\minus{} \\frac { \\minus{} n \\minus{} a}m \\equal{} \\frac {2n}m$\r\nSo positive difference is $ |\\frac {2n}m|$\r\n\r\nSo the answer here is $ |\\frac {2*20}1| \\equal{} 40$", "Solution_3": "$ \\left|\\frac{2n}{m}\\right|$ looks nicer" } { "Tag": [], "Problem": "A farmer plants seeds for a $ 75$-acre field of yellow sweet clover. A $ 25$-pound bag of seed costs $ \\$24$. How much would it cost, in dollars, to seed the field if $ 12$ pounds of seed were used per acre?", "Solution_1": "[hide]\nMultiply $ 75$ by $ 12$ to find how many pounds were used then divide by 25 to find the amount of bags used and then multiply by 24.\n\\[ \\left(\\frac{75\\cdot12}{25}\\right)24\\equal{}36\\cdot24\\equal{}\\boxed{864}\n\\]\n[/hide]" } { "Tag": [ "function", "algebra unsolved", "algebra" ], "Problem": "Prove that there is NOT exist function at $R$ such that $f(f(x))=-x^3$ for each $x\\in R$", "Solution_1": "Are you sure that you do not miss the continuity?\r\n\r\nPierre.", "Solution_2": "I think that the problem is this", "Solution_3": "Ok, you are right. I thought that any orbit was infinite but this is not true...\r\nAssume that such a function does exist. Note that clearly $f$ is bijective from $\\mathbb {R}$ onto itself.\r\nLet $a=f(1).$\r\nThen $f(a)=-1,f(-1)=-a^3,f(-a^3)=1,f(1)=a^9.$\r\nTherefore $a=a^9$ which forces $a=0,-1$ or $1$.\r\n\r\nBut, if $a=1$ then $f(1)=1$ so that $1=f(f(1))=-1$, a contradiction.\r\nIf $a=-1$ then $f(-1)=f(f(1))=-1.$ This leads to $f(1)=f(-1)$, a contradiction.\r\nIf $a=0$ then $f(0)=f(f(1))=-1$ and $f(-1) = f(f(0))=0$ so that $f(1)=f(-1)$, a contradiction. And we are done.\r\n\r\nPierre.", "Solution_4": "Nice job" } { "Tag": [ "function", "algebra", "polynomial", "induction", "number theory proposed", "number theory" ], "Problem": "Let $f(n)=\\sum_{k=0}^{n-1}x^ky^{n-1-k}$ with, $x$, $y$ real numbers. If $f(n)$, $f(n+1)$, $f(n+2)$, $f(n+3)$, are integers for some $n$, prove $f(n)$ is integer for all $n$.", "Solution_1": "Nice!! Observe that $ f(n+1)f(n-1)-f^2(n)=-(xy)^{n-1}$ and so it follows that $ (xy)^n$ and $ (xy)^{n+1}$ are integers. Thus xy is rational and algebraic integer and so it is integer. Next, observe that you can write $f(n)$ as a polynomial with integer coefficients in xy and x+y (or, at least, it seems it is true by cheking small cases, so it must be true :D ) such that $ (x+y)$ appears with maximal power alone (not multiplied with xy) and with coefficient 1. It thus follows that x+y is algebraic integer and rational (since f(n), f(n+1), f(n+2) and xy are integers) and thus it is integer. The conclusion follows. This, if the claim with the polynomial is true. ;)", "Solution_2": "Sorry, I was idiot, the assertion with the polynomial comes by induction from the fact that $ f(n+1)=(x+y)f(n)-xyf(n-1)$ and xy integer. The assertion is: for all n, f(n) has the form $ f(n)=(x+y)^{n-1}+P(x+y,xy)$ where P is a polynomial with integer coefficients and such that all powers of x+y has exponent at most n-2.", "Solution_3": "This was also used in the 1995 Bulgarian olympiad." } { "Tag": [], "Problem": "What is the greatest positive integer $n$ such that $n^2$ is less than 95?", "Solution_1": "i remember answering a question like this...\r\n[hide]so 8^2=64\n9^2=81\n10^2=100\nso 9 is the answr.[/hide]", "Solution_2": "[hide]\n$9^2=81$\n$10^2=100$\n\nso $9$[/hide]", "Solution_3": "its 9. :D", "Solution_4": "[quote=\"venkat3848\"]its 9. :D[/quote]\r\nI know this is pretty obvious but please say WHY it's [hide=\"~\"]9[/hide]!", "Solution_5": "obvious its 9 :rotfl:" } { "Tag": [ "inequalities", "inequalities proposed" ], "Problem": "If $x,y,z>0$ and $x+y+z=1$ prove that \r\n\r\n$xy+yz+zx\\geq 4(x^2y^2+y^2z^2+z^2x^2)+5xyz$", "Solution_1": "Just homogenise and use muirhead", "Solution_2": "Serbia MO 2008:\nIf $x,y,z>0$ and $x+y+z=1$ prove that\\[x^2+y^2+z^2+3xyz\\ge\\frac{4}{9}\\].", "Solution_3": "[quote=\"sqing\"]Serbia MO 2008:\nIf $x,y,z>0$ and $x+y+z=1$ prove that\\[x^2+y^2+z^2+3xyz\\ge\\frac{4}{9}\\].[/quote]\n\n\nThe problem is equivalent to:\n$ x,y,z>0,x+y+z=3\\Rightarrow x^2+y^2+z^2+xyz\\ge 4 $\nFollowing result is known:\n(see http://www.artofproblemsolving.com/Forum/viewtopic.php?f=52&t=369554)\n$ x,y,z>0\\Rightarrow x^2+y^2+z^2+xyz+5\\ge 3(x+y+z) $\nUsing this result we obtain QED.\n___________\nSandu Marin", "Solution_4": "Inequality is equivalent to:\n1) $ x,y,z>0,x+y+z=3\\Rightarrow 9(xy+yz+zx)\\ge4(x^2y^2+y^2z^2+z^2x^2)+15xyz $\n$ xy+yz+zx=s\\&(x,y,z>0,(x+y+z)^2\\ge 3(xy+yz+zx))\\Rightarrow 00,00$ and $x+y+z=1$ prove that\\[x^2+y^2+z^2+3xyz\\ge\\frac{4}{9}\\].[/quote]\nLet $x\\ge y\\ge z$ , so $9\\sum x^2+27xyz-4=9 \\sum x \\sum x^2+27xyz-4(\\sum x)^3$\n\n$=\\frac{9}{4}(2x+2y-z)(x-y)^2+\\frac{1}{4}(2x+2y+5z)(x+y-2z)^2\\ge 0.$" } { "Tag": [ "function", "summer program", "MathPath" ], "Problem": "I recently read about proofs involving Ramsey numbers, and I understand a basic definition of Ramsey's Theorem, but I am not sure how to apply it. Could any of you please enlighten me as to where Ramsey Numbers become useful?\r\n\r\nPlease post some challenging problems, and tell me (as well as the entire AoPS community) what Ramsey Numbers exactly are. Post some interesting facts if you'd like :D", "Solution_1": "Suppose you have a regular n-gon with all the diagonals drawn in. You want to color each diagonal with one of k colors so that you don't have an a1-gon with all diagonals in color 1, a regular a2-gon with all diagonals in color 2, etc. What is the largest possible value of n?\r\n\r\nThat is the standard Ramsey Theory problem. You can make variations on it (for example, what if you only want the sides and not the diagonals to be not monochromatic).\r\n\r\nRamsey Theory problems tend to be incredibly hard. Here's a seemingly very simple problem that is actually unsolved (at least, as of a few years ago, and I doubt anyone has solved it since):\r\n\r\nWhat is the smallest value of n so that if you color all the sides and diagonals of a regular n-gon red or blue, you are guaranteed to have a regular pentagon with all diagonals in either red or blue?", "Solution_2": "In space, there are given p_n=[en!]+1 points, where [x] is the least integer function\r\nEach pair of points is connected by a line, and each line is colored with one of n colors. Prove that there is at least one triangle with sides of the same color", "Solution_3": "Lol. Ramsey numbers...brings back some pretty weird memories.\r\n\r\nI remember at MathPath 2003 we were introduced to Ramsey numbers. One of my friends and I spent about a week trying to come up with some formula to express them (the dichromatic Ramsey numbers, anyway.) I remember we came up with some pretty fair approximations." } { "Tag": [ "calculus" ], "Problem": "So I skimmed through Mathematical Olympiad Treasures on Amazon and my jaw dropped. Could I get a recommendation on a textbook for harder Algebra problems such as the ones I saw in MOT book.", "Solution_1": "Mathematical Olympiad Treasures is a problem-solving book. The subjects it handles are not meant to be approached in a \"textbook\" manner: you won't find any large theorems from which calculations can be done mechanically, such as in, say, differential calculus. \r\n\r\nThat having been said, [u]Problem-Solving Strategies[/u] by Arthur Engel is a good book that approaches Olympiad subjects in a somewhat more cohesive manner.", "Solution_2": "[quote=\"t0rajir0u\"]Mathematical Olympiad Treasures is a problem-solving book. The subjects it handles are not meant to be approached in a \"textbook\" manner: you won't find any large theorems from which calculations can be done mechanically, such as in, say, differential calculus. \n\nThat having been said, [u]Problem-Solving Strategies[/u] by Arthur Engel is a good book that approaches Olympiad subjects in a somewhat more cohesive manner.[/quote]Aw, that's sad to know. I am currently in Calculus 2 but I find myself encountering Algebra problems (student tutor) that I've never seen before and usually I can solve them but some definitely give me a hard time and some I can't even solve. I just love problem-solving, thanks!", "Solution_3": "Apparently, you already know some problem-solving, so I would recommend you Problem-Solving Strategies too... after all the book is written by a German, so it has to be great :P\r\n\r\n[/snobism]" } { "Tag": [ "algebra unsolved", "algebra" ], "Problem": "Prove that if a sequence $ \\{G(n) \\}_{n\\equal{}0}^{\\infty}$ of integers satisfies: $ G(0)\\equal{}0, G(n)\\equal{}n\\minus{}G(G(n))\\quad (n\\equal{}1,2,3,...)$ then:\r\n$ (a)$ $ G(k) \\ge G(k\\minus{}1)$ for any positive integer $ k$\r\n$ (b)$ no integer $ k$ exists such that $ G(k\\minus{}1)\\equal{}G(k)\\equal{}G(k\\plus{}1)$.", "Solution_1": "is easy to pove for all $ n > 0 G(n) < > 0$ because if not $ G(n) \\equal{} 0 \\equal{} n \\minus{} G(G(n)) \\equal{} n \\minus{} G(0) \\equal{} n$ so $ n \\equal{} 0$ absurde\r\n\r\nand for all $ n > 0 n G(n) \\equal{} n \\minus{} G(G(n)) \\le n \\minus{} 1$\r\n\r\n1) by recurence (fort) ( tha for all $ n$ $ G(n)\\geq G(n \\minus{} 1) \\geq ....\\geq G(0)$\r\n\r\nfor $ n \\equal{} 1$ $ G(1)\\in N$ so $ G(1) \\geq 0 \\equal{} G(0)$\r\n\r\nsuppose that is true for $ n$ and prove that is true for $ n \\plus{} 1$\r\n\r\n$ G(n \\plus{} 1) \\equal{} n \\plus{} 1 \\minus{} GG(n \\plus{} 1)$ you have $ G(n \\plus{} 1) \\le n$ so $ G(n) \\geq G(G(n \\plus{} 1))$ so \r\n\r\n$ G(n \\plus{} 1) \\equal{} n \\plus{} 1 \\minus{} GG(n \\plus{} 1) \\geq n \\plus{} 1 \\minus{} G(n) \\equal{} G(n) \\plus{} 1 > G(n)$\r\n\r\n2) if $ G(n) \\equal{} G(n \\minus{} 1)$ ===> $ n \\minus{} G(G(n)) \\equal{} n \\minus{} 1 \\minus{} G(G(n \\minus{} 1)) \\equal{} n \\minus{} 1 \\minus{} G(G(n))$ ===>$ 1 \\equal{} 0$ absurde so \r\n\r\n$ G(n)>G(n\\minus{}1)$", "Solution_2": "[quote=\"mohamed-01-01\"] ... $ G(n \\plus{} 1) \\equal{} n \\plus{} 1 \\minus{} GG(n \\plus{} 1) \\geq n \\plus{} 1 \\minus{} G(n) \\equal{} G(n) \\plus{} 1 > G(n)$ ...[/quote]\r\nHello mohamed-01-01!\r\n\r\nWhy is \"$ n \\plus{} 1 \\minus{} G(n) \\equal{} G(n) \\plus{} 1$\" true ?", "Solution_3": "[quote=\"pco\"][quote=\"mohamed-01-01\"] ... $ G(n \\plus{} 1) \\equal{} n \\plus{} 1 \\minus{} GG(n \\plus{} 1) \\geq n \\plus{} 1 \\minus{} G(n) \\equal{} G(n) \\plus{} 1 > G(n)$ ...[/quote]\nHello mohamed-01-01!\n\nWhy is \"$ n \\plus{} 1 \\minus{} G(n) \\equal{} G(n) \\plus{} 1$\" true ?[/quote]\r\n\r\noh sorry my solution is wrong but I think there is a mistake in this probleme\r\n\r\nbecause if $ G(n) \\geq G(n \\minus{} 1)$ \r\n\r\nand you have for all $ n$ $ G(n) < > G(n \\minus{} 1)$ because if $ G(n) \\equal{} G(n \\plus{} 1)$ ==> \r\n\r\n$ n \\plus{} 1 \\minus{} G(G(n \\plus{} 1)) \\equal{} n \\minus{} G(G(n)) \\equal{} n \\minus{} G(G(n \\plus{} 1))$ so $ 1 \\equal{} 0$\r\n\r\nso $ G(n \\plus{} 1) > G(n)$ ===>$ G(n) \\geq n$\r\n\r\nso $ G(n) \\equal{} n \\minus{} G(G(n)) \\le n \\minus{} G(n) \\le n \\minus{} n \\equal{} 0$ ???", "Solution_4": "[quote=\"mohamed-01-01\"] oh sorry my solution is wrong but I think there is a mistake in this probleme\n\nbecause if $ G(n) \\geq G(n \\minus{} 1)$ \n\nand you have for all $ n$ $ G(n) < > G(n \\minus{} 1)$ because if $ G(n) \\equal{} G(n \\plus{} 1)$ ==> \n\n$ n \\plus{} 1 \\minus{} G(G(n \\plus{} 1)) \\equal{} n \\minus{} G(G(n)) \\equal{} n \\minus{} G(G(n \\plus{} 1))$ so $ 1 \\equal{} 0$\n\nso $ G(n \\plus{} 1) > G(n)$ ===>$ G(n) \\geq n$\n\nso $ G(n) \\equal{} n \\minus{} G(G(n)) \\le n \\minus{} G(n) \\le n \\minus{} n \\equal{} 0$ ???[/quote]\r\n\r\nThere is a little mistake in your demo but your conclusion seems correct to me : $ G(n)\\ne G(n\\plus{}1)$ only for $ n>0$, so maybe $ G(1)\\equal{}G(0)$ So, if the required non decreasing assertion is true, we have $ G(n)\\geq n\\minus{}1$ (and not $ G(n)\\geq n$). So $ G(G(n))\\geq n\\minus{}2$ and $ n\\equal{}n\\minus{}G(G(n))\\leq 2$ !\r\nAnd so it seems indeed that there is a mistake in this problem ... :huuh:" } { "Tag": [ "geometry", "incenter", "circumcircle", "conics", "ellipse", "perpendicular bisector", "geometry unsolved" ], "Problem": "Let AD be the altitude of a triangle ABC and E , F be the incenters of the triangle ABD and ACD , respectively. line EF meets AB and AC at K and L. prove tht AK=AL if and only if AB=AC or A=90", "Solution_1": "Let I be the incenter of the triangle $\\triangle ABC$. We have to show that the bisector AI of the angle $\\angle A$ is perpendicular to EF iff either AB = AC or the angle $\\angle CAB = 90^\\circ$ is right. The bisectors $AI, BI \\equiv EI, CI \\equiv FI$ are concurent cevians of the triangle $\\triangle AEF$. Let $X \\in AF, Y \\in AE$ be the feet of EI, FI in this triangle.\r\n\r\n$\\angle EXA \\equiv \\angle BXA = 180^\\circ - \\frac{\\angle B}{2} - (90^\\circ - \\angle B) - \\frac{90^\\circ - \\angle C}{2} =$\r\n\r\n$= \\frac{90^\\circ + \\angle B + \\angle C}{2} = 90^\\circ + \\frac{90^\\circ - \\angle A}{2}$\r\n\r\n$\\angle FYA \\equiv \\angle CYA = 180^\\circ - \\frac{\\angle C}{2} - (90^\\circ - \\angle C) - \\frac{90^\\circ - \\angle B}{2} =$\r\n\r\n$= \\frac{90^\\circ + \\angle C + \\angle B}{2} = 90^\\circ + \\frac{90^\\circ - \\angle A}{2}$\r\n\r\nHence, $\\angle EXA = \\angle FYA$. If the angle $\\angle A = 90^\\circ$ is right, EX, FY are 2 altitudes of the triangle $\\triangle AEF$, I its orthocenter and $AI \\perp EF$ its remaining altitude. Assume now that the angle $\\angle A \\neq 90^\\circ$ is not right and that we still have $AI \\perp EF$, i.e., AI is the A-altitude of the triangle $\\triangle AEF$, but I is no longer its orthocenter. Since $\\angle EXA = \\angle FYA \\neq 90^\\circ$, then also $\\angle EXF = \\angle FYE \\neq 90^\\circ$, i.e., the quadrilateral EFXY is cyclic. Let (P) be its circumcircle, which is centered on the perpendicular bisector of the segment EF. Using parallel projection, we can project the triangle $\\triangle AEF$ into a triangle $\\triangle A'EF$, so that the point I is projected into its orthocenter I'. Then the cevians EX, FY are projected into its altitudes EX', FY', i.e., the quadrilateral EFX'Y' is also cyclic, with the circumcircle (O') centered at the midpoint O' of the segment EF. Thus the points X, Y lie both on the circle (P) and on an ellipse $o$ with the main axis EF (which is projected into the circle (O') in our parallel projection), not identical with the circle (P). Since both the ellipse $o$ and the circle (P) are symmetrical with respect to the perpendicular bisector of the segment EF, so are their intersections X, Y, i.e. EX = FY and EY = FX. Hence, the triangles $\\triangle EFX \\cong \\triangle FEY$ are congruent and it immediately follows that the triangle $\\triangle AEF$ is isosceles with AE = AF. This and $AD \\perp BC$ implies that the incircles $(E) \\cong (F)$ are congruent, which means that the triangle $\\triangle ABC$ itself is isosceles with AB = AC." } { "Tag": [ "trigonometry" ], "Problem": "How did you guys do?", "Solution_1": "Just a reminder that the problems should be not posted or discussed before they are officially posted on the website, since other cities may write the exam at a later date;", "Solution_2": "Out of curiousity, do you all take Senior A?", "Solution_3": "yes, I took senior A.", "Solution_4": "Any idea why they're written at so many times of the year? I mean, Calgary doesn't write them for another month or so.", "Solution_5": "What is this? Have a link? Thanks.\r\n\r\n-The Living in Closet all life person", "Solution_6": "[quote=\"billzhao\"]Just a reminder that the problems should be not posted or discussed before they are officially posted on the website, since other cities may write the exam at a later date;[/quote]\r\n\r\nI have no idea when Edmonton is writing this. :? Does anyone know? :D", "Solution_7": "Oh, William, you've never heard of Tournament of Town? I guess they don't hold it at Waterloo. Here is link for T of T in Toronto, [url]http://www.math.toronto.edu/oz/turgor/news.php[/url]. It's basically a 4/5 hour contest primarily organized by some Russian math commitee and hold in a few cities in Europe and North America. I think the contest is rather an informal one, but in a sense, make Olypiad level problems more approachable to beginner like me. The contest is fairly nice. O-Level is not that challenging, A-Level is a bit hard for me...\r\n\r\nBTW, Alex, I did horrible today. I solved only two problems and BSed for couple others. I wrote 3 pages of trigs for #5, and couldn't prove it in the end.... Was you there today? I didn't even see you.", "Solution_8": "Is this the same thing as mathbattle?\r\n\r\nEither way, I'm looking for activities for our math club to join as a team. Maybe this, mathbattle, and MCTIC (CARML) would be good candidates?\r\n\r\nBTW Alan good to hear from you again! Haven't seen you in so long :)", "Solution_9": "yes i was there; an hour into the end i left because i knew i couldnt break a max-O-level score which I am sure that i could get later anyway.\r\n\r\nbtw, solving 2 problems and bsing #5 is very good. Isn't this close to a perfect score? (assuming \"solving 2 problems\" doesn't include #1)\r\n\r\nI had very strong ideas on how to solve all, but couldn't bring them out.", "Solution_10": "[quote=\"TheDreamer\"]Is this the same thing as mathbattle?\n\nEither way, I'm looking for activities for our math club to join as a team. Maybe this, mathbattle, and MCTIC (CARML) would be good candidates?\n\nBTW Alan good to hear from you again! Haven't seen you in so long :)[/quote]\r\n\r\nI'm not sure what exactly mathbattle is, but I'm quite sure that it's different.\r\n This competition gives you 5~7 problems(I don't remember the exact number) and 4~5 hours(I don't remember this either) to solve them. Each problem is weighted differently and they only take the best three(right?) problems that you solved. There are two rounds in a year, fall and spring round. There are also two levels, O level for grade 10 and below and A level for grade 11 and above(but they have some overlapping problems). This is an individual competition. There are no prizes(except for a certificate for doing ok), but the problems are [i][b]very[/b][/i] beautiful.", "Solution_11": "[quote=\"lightrhee\"]There are also two levels, O level for grade 10 and below and A level for grade 11 and above(but they have some overlapping problems).[/quote]\r\n\r\nYou got it confused there. There are 2 levels, Junior for grade 10 and under, and Senior for grade 11 and over. There are two rounds, O-Level written first, the easier of the two, and A-Level which is harder.", "Solution_12": "Yea, this is different from mathbattle. This is more or less in a format similar to \r\nCMO(although much longer). T of T is individual contest, whereas mathbattle is a team competition. MathBattle is a lot more fun though. It's like you have a set of problems and your team will work on them for like 2.5 hrs. And you and your opposing team take turn in showing solution to problems by the other team. You will have rebuttals, judges too. It's strategic sometimes too, not always depend on how many prolbems you solved. There are more info. on the website I provided in the above post. Oh, we get free pizza and pop too. So free lunch, if that is an incentive for anyone to come.\r\n\r\nBTW, William you are going to have a team from your school come to Toronto to join the MathBattle? Well, Waterloo is not that far from Toronto after all. I would be glad to see you there. I knew couple others people from your school also.", "Solution_13": "Hmm that would be really cool. I'll see how much interest there is in our math club. We wouldn't really stand a chance though :s, so it may not be such a good idea, making ppl lose confidence lol", "Solution_14": "How large is your Math Club? We only have 10 members at most.", "Solution_15": "We had 25 the first time, with another 25 who said they'll come next time. Although prolly some of them will just keep procrastinating and won't come at all, so i'm guessing in the end we'll have around 30, which is about right i think" } { "Tag": [ "geometry", "perimeter", "trigonometry", "function", "complex analysis", "parameterization", "parametric equation" ], "Problem": "A mark is placed on the perimeter of a wheel where it touches the road. The wheel is rolled along the road without skidding though less than half a rotation. The mark has advanced horizontally from its starting position by x. Given x and the radius of the wheel (r) what is the height (y) of the mark above the road?\r\n\r\n\r\nIt seems a simple problem but as far as I can gather there is no solution to this question that does not involve numerical aproximation methods.", "Solution_1": "Imagine that the wheel was squeaky clean at the beginning, and now a part of it (i.e. a circular arc) is covered with road dirt. The length of this arc is $ x$, therefore ... \r\n\r\nAlso, this is not complex analysis, and not even calculus.", "Solution_2": "[quote=\"mlok\"]Imagine that the wheel was squeaky clean at the beginning, and now a part of it (i.e. a circular arc) is covered with road dirt. The length of this arc is $ x$, therefore ... \n\nAlso, this is not complex analysis, and not even calculus.[/quote]\r\n\r\nYou have mis-interpreted the question.\r\n\r\nThe dusty arc represents the distance the wheel the has rolled along the road and this distance (call it z) is given by $ R\\theta$ wher $ \\theta$ is the angle the wheel has turned through. The x in the question is the distance the mark has traveled along the x axis and this is given by $ x\\equal{}r(\\theta\\minus{}\\sin(\\theta))$ or $ x\\equal{}z\\minus{}\\sin(\\theta)$ and is not the same as the distance the wheel has rolled. (The mark traces out a cycloid). The parametric equation for a cycloid defines y as $ R(1\\minus{}\\cos(\\theta))$ but I am looking for y in terms of x when $ \\theta$ is not given. \r\n\r\nIf you think that is an easy question I'll give $ \\$$50 to a nominated charity or to whoever answers the question (if that is allowed) if you provide an equation for y in terms of x where the angle the the wheel has turned through $ (\\theta)$ is not given.\r\n\r\nRules:\r\n\r\nThe answer must be given in the form of a cartesian equation with y and only y on the left hand side and where $ \\theta$ or y do not appear on the right hand side of cartesian equation. \r\n\r\nBy the way, if you know y then there is a cartesian equation for x that does not include $ \\theta$:\r\n\r\n$ x\\equal{}\\pm\\arccos\\left({R\\minus{}y \\over R}\\right)R\\pm\\sqrt{2Ry\\minus{}y^2}$\r\n\r\nAll you have to do is solve that equation for y ;)\r\n\r\nWould have been nice if you had actually read the question properly before moving it.", "Solution_3": "Don't waste your time looking. The inverse function $ y(x)$ is not an elementary function. There is no reason to expect transcendental equations to be solvable in elementary terms.\r\n\r\nIn practice, you can do anything you want with the parametric form. Numerical methods also work well here." } { "Tag": [ "real analysis", "modular arithmetic", "function", "real analysis unsolved" ], "Problem": "A cute result :):\r\n\r\nLet $T$ be a measure preserving invertible transformation on the unit interval $I$ (with the usual Lebesgue measure), such that for almost all $x\\in I,\\ T^{n}x=x$, and $x,\\ Tx,\\ \\ldots,T^{n-1}x$ are distinct. \r\n\r\nProve that there is a measurable subset $E$ of $I$ with measure $\\frac{1}{n}$ such that $E,\\ T(E),\\ \\ldots,T^{n-1}(E)$ are pairwise disjoint.", "Solution_1": "While we're all trying to prove this, let me state, trivially, that the topic is not vacuous. It is possible to construct such a $T.$\r\n\r\nThe simplest such example I can think of is to let $T(x)\\equiv x+\\frac1n\\pmod{1}.$\r\n\r\n(And in that case, $E=\\left[0,\\frac1n\\right)$ can be the set we want.)", "Solution_2": "Here's an attempt. Let $J$ be the set of points $x$ that have period $n$. By the hypothesis, $J$ has measure 1.\r\n\r\nSay that a point $x$ is [i]early[/i] if $x \\le T^{k}x$ for all integers $k$. Let $E$ be the set of early points in $J$. Then $E$ is measurable because, um, well, I don't really know why. (How do I even show that the set of $x$ such that $x \\le Tx$ is a measurable set?) But I will assume that $E$ is measurable.\r\n\r\nBy the given property, it is easy to see that $E$, $TE$, $\\ldots$, $T^{n-1}E$ form a partition of $J$. All these $n$ sets have the same measure because $T$ is measure-preserving. Thus $E$ has measure $\\frac{1}{n}$.", "Solution_3": "Neat :). My proof was a bit more involved. Anyway, regarding your question:\r\n\r\nOne possible definition for the product $\\sigma$-algebra of Lebesgue measurable subsets of $I\\times I$ is: the smallest $\\sigma$-algebra that will make both projections $\\pi_{i},\\ i=1,2$ from $I\\times I$ onto $I$ (i.e. the projections on the two factors) measurable. This implies that a map $S: I\\to I\\times I$ is measurable iff $\\pi_{i}\\circ S$ are both measurable self-maps of $I$. \r\n\r\nNow let $Sx=(x,Tx)$. By the above characterization of measurable functions from $I$ to $I\\times I,\\ S$ is measurable. Your set $\\{x\\ |\\ x\\le Tx\\}$ is the preimage of the (measurable, of course) triangle $\\{(x,y)\\ |\\ x\\le y\\}$ through $S$, and is thus measurable. And when you do this for every $T^{k}$ and take intersections as $k$ goes from $1$ to $n-1$, you find the set of early points to be measurable, as desired.", "Solution_4": "Thanks for the help on measurability." } { "Tag": [ "modular arithmetic", "polynomial", "number theory unsolved", "number theory" ], "Problem": "If $ a^2\\plus{}ab\\plus{}b^2$ is a prime$ (a,b \\in Z)$,prove that $ a^2\\plus{}ab\\plus{}b^2\\equiv 1 \\pmod 3$", "Solution_1": "Try $ p \\equal{} 3$.\r\n\r\n$ p \\equal{} a^2 \\plus{} ab \\plus{} b^2 \\Leftrightarrow$\r\n$ 4p \\equal{} (2a \\plus{} b)^2 \\plus{} 3b^2 \\implies$\r\n$ (2a \\plus{} b)^2 \\equiv \\minus{} 3b^2 \\bmod p \\implies$\r\n$ \\left( \\frac {2a \\plus{} b}{b} \\right)^2 \\equiv \\minus{} 3 \\bmod p$\r\n\r\nwhich is possible if and only if $ p \\equal{} 3$ or $ p \\equiv 1 \\bmod 3$. (In the other direction: $ p \\equiv 1 \\bmod 3 \\implies \\exists x : p | x^2 \\plus{} 3 \\equal{} (x \\plus{} \\sqrt { \\minus{} 3})(x \\minus{} \\sqrt { \\minus{} 3})$, which implies that $ p$ is not prime in the [url=http://mathworld.wolfram.com/EisensteinInteger.html]Eisenstein integers[/url].)", "Solution_2": ":arrow: If $ 3|a \\equal{}> a\\equal{}3$.\r\nWe have $ A \\equal{} b^2\\plus{}3b\\plus{}9$\r\nIf $ b \\vdots 3 (b\\equal{}3)$, $ A\\equal{}27$, which is not a prime.\r\nSo b is not divisible by 3, then $ b^2 \\equiv 1 (mod 3) \\equal{}> A \\equiv 1 (mod 3).$\r\n\r\n :arrow: If a and b are not divisible by 3, $ a^2\\plus{}b^2 \\equiv 1\\plus{}1\\equal{}2 (mod 3).$\r\n$ ab \\equiv 1 or 2 (mod 3)$\r\nThen, $ A \\equiv 0 or 1 (mod 3)$.\r\nBecause A > 3, $ A \\equiv 1 (mod 3)$.\r\n\r\nHence, $ A \\equiv 1 (mod 3)$ if A is a prime.", "Solution_3": "Another problem\r\n\r\nIf $ 3k\\plus{}1$ is a prime , prove that it can be a form like \"$ a^2\\plus{}ab\\plus{}b^2(a,b\\in Z)$\" \r\n\r\n\r\n\r\nl'm sorry.My English is poor,so it's hard for me to translate the problems from Chinese into English. :fool:", "Solution_4": "Already solved that problem in the last line. See also my comments on the corresponding theorem for $ a^2 \\plus{} b^2$ [url=http://www.artofproblemsolving.com/Forum/weblog_entry.php?t=219017]here[/url].", "Solution_5": "[quote=\"skywalkerJ.L.\"]If $ a^2 \\plus{} ab \\plus{} b^2$ is a prime$ (a,b \\in Z)$,prove that $ a^2 \\plus{} ab \\plus{} b^2\\equiv 1 \\pmod 3$[/quote]\r\n\r\nif $ a\\equal{}b\\equal{}1$ ? then $ a^2\\plus{}b^2\\plus{}ab\\equal{}3$ !!!!!!!!" } { "Tag": [ "USAMTS", "search" ], "Problem": "So, what are some formulas that are usefull?", "Solution_1": "The reason nobody's responding to your post is that there aren't just a few formulas that are useful. Anyway, formulas are not the important thing. They really don't help much when solving problems, except for the formulas that we all know and use commonly.", "Solution_2": "Search Mathworld if you want to know something. It's much easier than memorizing obscure formulae." } { "Tag": [ "Gauss", "linear algebra", "matrix" ], "Problem": "There is something about matrices that I have found out... and I attempted to see if my idea has ever been thought of before, and I found nothing.\r\nI need someone to help me find out if this idea was pre-existant and I just came up with something that already existed.\r\n\r\nWhat its about: Matrices\r\n\r\nUses principles presented in: Guassian Elimination, Gauss-Jordan Elimnation", "Solution_1": "Here it is... Im posting it up\r\n\r\nBasic idea:\r\n\r\nGiven any sets of three matrices where AX=B, one may perform any identical Gaussian elimination operations to both A and B and the equation will remain equal.\r\n\r\nWhat this can be used for:\r\n\r\nInstead of needing to find the inverse of Matrix A, and multiply both sides by A, one can simply manipulate both sides until A is the identity. :P", "Solution_2": "Yes, this is well known. If you look at it closely you will see that it's just a slight extension of the Gauss-Jordan method for solving systems of equations.\r\n\r\nThe good thing about it is that it doesn't just work for equations of 2x2 or nxn size, but also for matrix equations of any size (something that Cramer's Rule doesn't do). It also ultimately turns out to be the fastest and most efficient way of solving matrix equations for large matrices.\r\n\r\nYou've re-discovered a really great thing here! See if you can use it to try and solve equations of non-square matrices." } { "Tag": [ "inequalities", "induction", "rearrangement inequality", "inequalities proposed" ], "Problem": "Let $ 0 < x_{1}\\leq\\frac {x_{2}}{2}\\leq\\cdots\\leq\\frac {x_{n}}{n}, 0 < y_{n}\\leq y_{n \\minus{} 1}\\leq\\cdots\\leq y_{1},$ Prove that $ (\\sum_{k \\equal{} 1}^{n}x_{k}y_{k})^2\\leq(\\sum_{k \\equal{} 1}^{n}y_{k})(\\sum_{k \\equal{} 1}^{n}(x_{k}^2 \\minus{} \\frac {1}{4}x_{k}x_{k \\minus{} 1})y_{k}).$ where $ x_{0} \\equal{} 0.$", "Solution_1": "The inequality to prove rewrites as $ A = (\\sum x_k^2y_k)(\\sum y_k) - (\\sum x_ky_k)^2\\ge B = \\frac14(\\sum y_k)(\\sum y_kx_kx_{k - 1})$. From the identity $ (\\sum a_i^2)(\\sum b_i^2) - (\\sum a_ib_i)^2 = \\displaystyle\\sum_{i < j}(a_ib_j - a_jb_i)^2$, we have $ A = \\sum_{i < j}y_iy_j(x_i - x_j)^2$. We are going to resort now to induction.\r\n\r\nFor $ n = 1,2$ the inequality can be verified easily. For the step of the induction, it is sufficient to prove that \r\n$ y_n\\displaystyle\\sum_{k = 1}^{n - 1}y_k(x_n - x_k)^2\\ge \\frac {y_nx_nx_{n - 1}(y_1 + \\ldots + y_{n - 1} + y_n)}4 +$ $ y_n\\cdot\\frac {y_2x_2x_1 + \\ldots + y_{n - 1}x_{n - 1}x_{n - 2}}4$. \r\n\r\nSince $ x_k\\le x_n$, it is enough to prove the last inequality for $ x_1 = \\frac {x_2}2 = \\ldots = \\frac {x_n}n$. Actually we can assume $ x_i = i$, $ i = 1,2,\\ldots,n$. Hence we are to prove \r\n$ \\displaystyle\\boxed{\\sum_{k = 1}^{n - 1}y_k(n - k)^2\\ge\\frac {n(n - 1)(y_1 + \\ldots + y_n)}4 + \\frac {y_2(2^2 - 2) + \\ldots + y_{n - 1}[(n - 1)^2 - (n - 1)]}4}$. \r\nLet ${ Y = y_1 + \\ldots + y_{n - 1}}$.\r\nFrom $ y_n\\le y_{n - 1}\\le\\ldots\\le y_1$ and rearrangement inequality we deduce the following:\r\n\r\n[b]A.[/b] $ y_1 + \\ldots + y_n\\le \\frac n{n - 1}Y$.\r\n\r\n[b]B.[/b] $ y_2 + \\ldots + y_{n - 1}\\le \\frac {n - 2}{n - 1}Y$.\r\n\r\n[b]C.[/b] $ \\displaystyle\\sum_{k = 1}^{n - 1}y_k(n - k)^2\\ge \\frac {Y[1^2 + 2^2 + \\ldots + (n - 1)^2]}{n - 1} = \\frac {n(2n - 1)}6Y$.\r\n\r\n[b]D.[/b] $ E = y_2(2^2 - 2) + \\ldots + y_{n - 1}[(n - 1)^2 - (n - 1)]\\le$ $ \\frac {(y_2 + \\ldots + y_{n - 1})[1^2 - 1 + 2^2 - 2 + \\ldots + (n - 1)^2 - (n - 1)]}{n - 2}$.\r\n\r\n[b]E.[/b] $ E\\le\\frac Y{n - 1}[\\frac {(n - 1)n(2n - 1) - 3n(n - 1)}6]$.\r\n\r\nHence, it would be enough to have that $ \\frac {n(2n - 1)}6Y = \\frac {n^2}4Y + Y[\\frac {n(2n - 1) - 3n}{24}]$, which is an identity." } { "Tag": [ "logarithms", "induction", "algebra unsolved", "algebra" ], "Problem": "given the array $ x_{n}$: that:\r\n$ x_{1} \\equal{} c$ ,c is an integer\r\n$ x_{n \\plus{} 1} \\equal{} cx_{n} \\plus{} \\sqrt {(c^2 \\minus{} 1)(x_{n}^2 \\minus{} 1)}$\r\nprove that $ x_{n}$ is the array of integers", "Solution_1": "Let $ c\\equal{}ch(a)$, were $ a\\equal{}\\ln {(c\\plus{}\\sqrt{c^2\\minus{}1})}$. Then by induction $ x_n\\equal{}ch(na)\\equal{}P_n(ch(a)$, were \r\n\\[ P_n(x)\\equal{}\\sum_{k\\equal{}0}^{[n/2]}C_n^{2k}x^{n\\minus{}2k}(x^2\\minus{}1)^k\\]- polinom with integer coefficients.\r\nTherefore \\[ x_n\\equal{}\\sum_{k\\equal{}0}^{[n/2]}C_n^{2k}c^{n\\minus{}2k}(c^2\\minus{}1)^k\\] - is integer.", "Solution_2": "thanks Rust\r\nbut ,I need a nice solution\r\n :) any idea???", "Solution_3": "If $ c\\le \\minus{} 1$, the sequence is periodic $ c,2c^2 \\minus{} 1,c,2c^2 \\minus{} 1,\\cdots$.\r\n\r\nIf $ c \\equal{} 0$, the sequence is periodic $ 0,1,0,1,\\cdots$\r\n\r\nIf $ c\\ge 1$, the sequence is $ x_{n \\plus{} 1} \\equal{} 2cx_n \\minus{} x_{n \\minus{} 1}$. It can be shown by induction.", "Solution_4": "[quote=\"xxp2000\"]If $ c\\le \\minus{} 1$, the sequence is periodic $ c,2c^2 \\minus{} 1,c,2c^2 \\minus{} 1,\\cdots$.\n\nIf $ c \\equal{} 0$, the sequence is periodic $ 0,1,0,1,\\cdots$\n\nIf $ c\\ge 1$, the sequence is $ x_{n \\plus{} 1} \\equal{} 2cx_n \\minus{} x_{n \\minus{} 1}$. It can be shown by induction.[/quote]\r\nthanks\r\n :)" } { "Tag": [ "inequalities unsolved", "inequalities" ], "Problem": "let $ a,d\\geq 0$,$ b,c>0$, such that $ a\\plus{}d\\leq b\\plus{}c$\r\n\r\nfind the minimum of \r\n\r\n$ \\frac{b}{c\\plus{}d}\\plus{}\\frac{c}{a\\plus{}b}$", "Solution_1": "hello, after my conjecture is ${ \\frac{b}{c+d}+\\frac{c}{a+b}\\geq\\frac{1}{2}(2\\sqrt{2}-1)}$. The equal sign will obtain for $ a=\\frac{1}{2}+\\frac{\\sqrt{2}}{2},b=\\frac{1}{2}(\\sqrt{2}-1),c=1,d=0$.\r\nSonnhard." } { "Tag": [ "function", "algebra unsolved", "algebra" ], "Problem": "prove that doesnot exist the function f: R -> R satisfy f(f(x)) = x^2 - 2008", "Solution_1": "[quote=\"caothujjj\"]prove that doesnot exist the function f: N -> N satisfy f(f(x)) = x^2 - 2008[/quote]\r\n\r\n$ x\\equal{}1$ implies $ f(f(1))\\equal{}\\minus{}2007<0$, which is impossible, since $ f(n)$ is from $ \\mathbb{N}$ in $ \\mathbb{N}$", "Solution_2": "[quote=\"pco\"][quote=\"caothujjj\"]prove that doesnot exist the function f: N -> N satisfy f(f(x)) = x^2 - 2008[/quote]\n\n$ x \\equal{} 1$ implies $ f(f(1)) \\equal{} \\minus{} 2007 < 0$, which is impossible, since $ f(n)$ is from $ \\mathbb{N}$ in $ \\mathbb{N}$[/quote]\r\n\r\n sorry, i have repaired it" } { "Tag": [ "inequalities proposed", "inequalities" ], "Problem": "Let $ a,b,c$ be positive number. Prove: \r\n$ \\frac{(a\\plus{}b)^2}{a\\plus{}b\\plus{}2c} \\plus{} \\frac{(b\\plus{}c)^2}{b\\plus{}c\\plus{}2a} \\plus{} \\frac{(c\\plus{}a)^2}{c\\plus{}a\\plus{}2b} \\ge \\sqrt{3(a^{2}\\plus{}b^{2}\\plus{}c^{2})}$", "Solution_1": "[quote=\"......\"]Let $ a,b,c$ be positive number. Prove: \n$ \\frac {(a \\plus{} b)^2}{a \\plus{} b \\plus{} 2c} \\plus{} \\frac {(b \\plus{} c)^2}{b \\plus{} c \\plus{} 2a} \\plus{} \\frac {(c \\plus{} a)^2}{c \\plus{} a \\plus{} 2b} \\ge \\sqrt {3(a^{2} \\plus{} b^{2} \\plus{} c^{2})}$[/quote]\r\nEasy SOS:\r\n$ \\sum_{cyc}\\frac {(a \\plus{} b)^2}{a \\plus{} b \\plus{} 2c}\\geq\\sqrt {3(a^{2} \\plus{} b^{2} \\plus{} c^{2})}\\Leftrightarrow$\r\n$ \\Leftrightarrow\\sum_{cyc}\\left(\\frac {(a \\plus{} b)^2}{a \\plus{} b \\plus{} 2c} \\minus{} c\\right)\\geq\\sqrt {3(a^{2} \\plus{} b^{2} \\plus{} c^{2})} \\minus{} a \\minus{} b \\minus{} c\\Leftrightarrow$\r\n$ \\Leftrightarrow\\sum_{cyc}\\frac {(a \\plus{} b)^2 \\minus{} c^2 \\minus{} c(a \\plus{} b \\plus{} c)}{a \\plus{} b \\plus{} 2c}\\geq\\sum_{cyc}\\frac {(a \\minus{} b)^2}{a \\plus{} b \\plus{} c \\plus{} \\sqrt {3(a^2 \\plus{} b^2 \\plus{} c^2)}}\\Leftrightarrow$\r\n$ \\Leftrightarrow\\sum_{cyc}\\frac {(a \\plus{} b \\plus{} c)(a \\plus{} b \\minus{} 2c)}{a \\plus{} b \\plus{} 2c}\\geq\\sum_{cyc}\\frac {(a \\minus{} b)^2}{a \\plus{} b \\plus{} c \\plus{} \\sqrt {3(a^2 \\plus{} b^2 \\plus{} c^2)}}\\Leftrightarrow$\r\n$ \\Leftrightarrow\\sum_{cyc}(a \\plus{} b \\plus{} c)\\left(\\frac {b \\minus{} c}{a \\plus{} b \\plus{} 2c} \\minus{} \\frac {c \\minus{} a}{a \\plus{} b \\plus{} 2c}\\right)\\geq$\r\n$ \\geq\\sum_{cyc}\\frac {(a \\minus{} b)^2}{a \\plus{} b \\plus{} c \\plus{} \\sqrt {3(a^2 \\plus{} b^2 \\plus{} c^2)}}\\Leftrightarrow$\r\n$ \\Leftrightarrow\\sum_{cyc}(a \\plus{} b \\plus{} c)\\left(\\frac {a \\minus{} b}{a \\plus{} c \\plus{} 2b} \\minus{} \\frac {a \\minus{} b}{b \\plus{} c \\plus{} 2a}\\right)\\geq$\r\n$ \\geq\\sum_{cyc}\\frac {(a \\minus{} b)^2}{a \\plus{} b \\plus{} c \\plus{} \\sqrt {3(a^2 \\plus{} b^2 \\plus{} c^2)}}\\Leftrightarrow$\r\n$ \\Leftrightarrow\\sum_{cyc}(a \\minus{} b)^2\\left(\\frac {a \\plus{} b \\plus{} c}{(a \\plus{} c \\plus{} 2b)(b \\plus{} c \\plus{} 2a)} \\minus{} \\frac {1}{a \\plus{} b \\plus{} c \\plus{} \\sqrt {3(a^2 \\plus{} b^2 \\plus{} c^2)}}\\right)\\geq0.$\r\nLet $ a\\geq b\\geq c.$ Hence, \r\n$ \\sum_{cyc}(a \\minus{} b)^2\\left(\\frac {a \\plus{} b \\plus{} c}{(a \\plus{} c \\plus{} 2b)(b \\plus{} c \\plus{} 2a)} \\minus{} \\frac {1}{a \\plus{} b \\plus{} c \\plus{} \\sqrt {3(a^2 \\plus{} b^2 \\plus{} c^2)}}\\right)\\geq$\r\n$ \\geq\\sum_{cyc}(a \\minus{} b)^2\\left(\\frac {a \\plus{} b \\plus{} c}{(a \\plus{} c \\plus{} 2b)(b \\plus{} c \\plus{} 2a)} \\minus{} \\frac {1}{2(a \\plus{} b \\plus{} c)}\\right) \\equal{}$\r\n$ \\equal{} \\sum_{cyc}\\frac {(a \\minus{} b)^2(c^2 \\plus{} ac \\plus{} bc \\minus{} ab)(a \\plus{} b \\plus{} 2c)}{2(a \\plus{} b \\plus{} c)(b \\plus{} c \\plus{} 2a)(a \\plus{} c \\plus{} 2b)(a \\plus{} b \\plus{} 2c)}\\geq$\r\n$ \\geq\\frac {(a \\minus{} b)^2(c^2 \\plus{} ac \\plus{} bc \\minus{} ab)(a \\plus{} b \\plus{} 2c)}{2(a \\plus{} b \\plus{} c)(b \\plus{} c \\plus{} 2a)(a \\plus{} c \\plus{} 2b)(a \\plus{} b \\plus{} 2c)} \\plus{}$\r\n$ \\plus{} \\frac {(a \\minus{} c)^2(b^2 \\plus{} ab \\plus{} bc \\minus{} ac)(a \\plus{} c \\plus{} 2b)}{2(a \\plus{} b \\plus{} c)(b \\plus{} c \\plus{} 2a)(a \\plus{} c \\plus{} 2b)(a \\plus{} b \\plus{} 2c)}\\geq$\r\n$ \\geq\\frac {(a \\minus{} b)^2((c^2 \\plus{} ac \\plus{} bc \\minus{} ab)(a \\plus{} b \\plus{} 2c) \\plus{} (b^2 \\plus{} ab \\plus{} bc \\minus{} ac)(a \\plus{} c \\plus{} 2b))}{2(a \\plus{} b \\plus{} c)(b \\plus{} c \\plus{} 2a)(a \\plus{} c \\plus{} 2b)(a \\plus{} b \\plus{} 2c)} \\equal{}$\r\n$ \\equal{} \\frac {(a \\minus{} b)^2(b^3 \\plus{} c^3 \\plus{} b^2a \\plus{} c^2a \\plus{} 2b^2c \\plus{} 2c^2b)}{(a \\plus{} b \\plus{} c)(b \\plus{} c \\plus{} 2a)(a \\plus{} c \\plus{} 2b)(a \\plus{} b \\plus{} 2c)}\\geq0.$", "Solution_2": ":( you said it easy...For such terrible caculation...\r\nAny nice method to this problem?", "Solution_3": "We can solve it easily by Am-Gm, my friend :)", "Solution_4": "Can you post your solution for this problem, nguoivn ?", "Solution_5": "Using the well-known result: $ \\sum\\ \\frac {x^2}{y \\plus{} z} \\geq\\ \\frac {5\\sum\\ x^2 \\minus{} 2\\sum\\ xy}{2(x\\plus{}y\\plus{}z)}$\r\n(we can prove easily this result by SOS).\r\nLet $ x \\equal{} a \\plus{} b$;... we obtain:\r\n$ \\sum\\ \\frac {(a \\plus{} b)^2}{a \\plus{} b \\plus{} 2c} \\geq\\ \\frac {2(a^2 \\plus{} b^2 \\plus{} c^2) \\plus{} ab \\plus{} bc \\plus{} ca}{a \\plus{} b \\plus{} c} \\equal{}$\r\n$ \\equal{} \\frac {1}{2}[a \\plus{} b \\plus{} c \\plus{} \\frac {3(a^2 \\plus{} b^2 \\plus{} c^2)}{a \\plus{} b \\plus{} c}] \\geq\\ \\sqrt {3(a^2 \\plus{} b^2 \\plus{} c^2)}$ (by Am-Gm)\r\nWe have done :)" } { "Tag": [ "floor function", "irrational number", "algebra proposed", "algebra" ], "Problem": "Here is something for everyone: part (i) --- for beginners; part (ii) --- for intermediate; part (iii) --- for advanced problem-solvers (I haven't yet found a solution for (iii); may be very difficult, but may as well be trivial, in case I missed something obvious).\r\n\r\n[b](i)[/b] (easy; many would perceive it as a joke :) ). Let $\\alpha > 1$ be a real number such that $n \\mid \\lfloor n\\alpha^n \\rfloor$ for all $n \\in \\mathbb{N}$. Must $\\alpha$ be an integer?\r\n[b](ii)[/b] (intermediate) Does there exist an irrational $\\alpha > 1$ such that $n \\mid \\lfloor \\alpha^n \\rfloor$ for infinitely many $n$?\r\n[b](iii)*[/b] (very difficult, I think; may be an open problem) Does there exist an $\\alpha > 1$ such that $n \\mid \\lfloor \\alpha^n \\rfloor$ for [b]all[/b] $n \\in \\mathbb{N}$?\r\n\r\nPerhaps a little surprisingly, I think that the answer to (iii) should be 'no.' I'd find quite unnormal the existence of an integer sequence $\\{a_n\\}$ such that $\\big( (a_{n+1}-1)^{\\frac{1}{n+1}}, (a_{n+1} + 1)^{\\frac{1}{n+1}} \\big) \\subset \\big( (a_n-1)^{\\frac{1}{n}}, (a_n + 1)^{\\frac{1}{n}} \\big)$ AND $n \\mid a_n$, because the length of the interval $\\big( (a_n-1)^{\\frac{n+1}{n}} + 1, (a_n + 1)^{\\frac{n+1}{n}} - 1 \\big)$ (from which we have to choose $a_{n+1}$) is $o(n)$ (if I did the estimates correctly). Any ideas are welcome! :lol: \r\n\r\nBesides, enjoy :D !\r\n\r\n--Vesselin", "Solution_1": "(i) Hmm... $\\alpha = \\frac 1 2 $ so that $0 \\leq n \\alpha^n < 1$ for all $n$.\r\nI like problems I can solve in 10 seconds :P \r\n\r\nPierre.", "Solution_2": "What exactly does that mean? :? He mentioned $\\alpha>1$ in all three problems.", "Solution_3": "Aaargh...10 seconds is not enough for me to understand the problem :blush: \r\n\r\nPierre.", "Solution_4": "(i) Hint: Construct [b]explicitly[/b] an irrational number $\\alpha > 1$ such that $\\lfloor n\\alpha^n \\rfloor = n \\lfloor \\alpha^n \\rfloor$ for all $n$ :lol: . This is why I said the problem is a 'joke,' but it's not [i]absolutely[/i] trivial --- though this hint itself is possibly the most non-trivial part :) ." } { "Tag": [], "Problem": "Suppose that $ a$ and $ b$ are positive real numbers such that:\r\n\r\n$ a^{3}\\equal{} a\\plus{}1$ and $ b^{6}\\equal{} b\\plus{}3a$\r\n\r\nShow that $ a > b$.", "Solution_1": "Look at [url=http://www.mathlinks.ro/Forum/viewtopic.php?t=79648]here[/url]." } { "Tag": [ "algorithm", "function", "algebra", "polynomial", "rational function", "complex analysis" ], "Problem": "Hello everyone! I am beginner, I have a questions(may be easy). Help me please.\r\n\r\nLet $ P(z)\\equal{}\\displaystyle\\sum_{k\\equal{}1}^{n}a_kz^k$ be a polynomial.\r\ni)Express $ P(z)$ as a power series around some point $ u\\neq 0$. \r\nii)Express the following rational map explicitly as a power series around $ z\\equal{}2$: $ f(z)\\equal{}\\frac{3z^4\\plus{}z^3\\plus{}2z^2\\plus{}7}{z^2\\minus{}z}$.\r\nii)Give an algorithm of expressing as a power series an arbitrary rational function $ f\\equal{}\\frac{P(z)}{Q(z)}$.", "Solution_1": "Step 1: Substitute $ w\\equal{}z\\minus{}u$ everywhere and expand. It's just easier to see things that way.\r\nIn the rational function cases, you should definitely do the long division to separate out the polynomial part. For the proper rational part, there are two basic approaches:\r\n- Build the coefficients recursively, getting the equations by clearing the denominator and matching coefficients. You don't usually get an explicit closed form this way, but it's pretty easy to calculate as far as you want.\r\n- Partial fractions. $ \\frac1{(1\\minus{}az)^n}\\equal{}\\sum_{k\\equal{}0}^{\\infty}\\binom{n\\plus{}k\\minus{}1}{k}a^k z^k$; use these for an explicit closed form for the coefficients." } { "Tag": [ "linear algebra", "matrix", "function", "Euler", "algebra", "polynomial", "linear algebra unsolved" ], "Problem": "Let $ \\mathbf{P}$ denote the \"infinite matrix\"\r\n\r\n$ \\left[ \\begin{array}{ccccc} 1 & 0 & 0 & 0 & \\hdots \\\\\r\n1 & 1 & 0 & 0 & \\hdots \\\\\r\n1 & 2 & 1 & 0 & \\hdots \\\\\r\n1 & 3 & 3 & 1 & \\hdots \\\\\r\n\\vdots & \\vdots & \\vdots & \\vdots & \\ddots \\\\\r\n\\end{array} \\right]$\r\n\r\nwith entries $ \\mathbf{P}_{ij} \\equal{} {i \\minus{} 1 \\choose j \\minus{} 1}$ and let $ \\mathbf{I}$ denote the \"infinite identity matrix.\" Compute the inverse of $ \\mathbf{P} \\plus{} \\mathbf{I}$.\r\n\r\n[hide=\"Remark\"] The computation of this inverse has apparently been the subject of several published papers, one of which has the thread title, even though it is relatively straightforward if looked at from the proper perspective. [/hide]\n\n[hide=\"Hint 1\"] $ \\mathbf{P}$ acts on $ \\mathbb{C}[[x]]$ (regarded as the ring of formal exponential generating functions over $ \\mathbb{C}$) by multiplication by $ e^x$. [/hide] \n\n[hide=\"Hint 2\"] Generalize to multiplication by $ e^{tx}$. [/hide]", "Solution_1": "[hide=\"Hint 3\"] It suffices to compute the power series of $ \\frac{1}{1 \\plus{} e^x}$. This computation can be done using [url=http://mathworld.wolfram.com/EulerPolynomial.html]Euler polynomials[/url] or you can write it as $ \\frac{1}{2} \\cdot \\frac{1}{1 \\plus{} \\frac{e^x \\minus{} 1}{2}}$ and expand directly. [/hide]" } { "Tag": [ "geometry", "rectangle" ], "Problem": "Find a recursion for $ a_{n}$, where $ a_{n}$ is the number of ways you can tile a $ 2\\times n$ rectangle with $ 1\\times 1$ squares and L-trominoes.\r\n\r\nI know problems like this have been posted, but for some reason my answer doesn't agree with JBL's from a post of his in 2004. \r\n\r\n[hide=\"My solution\"]\nThe standard procedure is to consider tiling the leftmost side. Using only $ 1\\times 1$ squares, there is 1 way. Using only L-trominoes, there are 2 ways.\n\n$ a_{n}=a_{n-1}+2b_{n-1}$,\n\nwhere $ b_{n-1}$ is the number of ways to tile a $ 2\\times (n-1)$ rectangle with a corner missing. \n\n$ b_{n-1}=a_{n-2}+a_{n-3}$.\n\nSo $ a_{n}=a_{n-1}+2a_{n-2}+2a_{n-3}$. \n\nJBL's answer was $ a_{n}= a_{n-1}+4a_{n-2}+2a_{n-3}$.[/hide]", "Solution_1": "I calculated the first four cases and got $ a_{1}= 1, a_{2}= 5, a_{3}= 11, a_{4}= 58$, which fits JBL's recursion.\r\n\r\nBut your solution makes sense too so I don't know what happened.", "Solution_2": "Oh I just realized I might have forgotten the case where you use 1 $ 1\\times 1$ square and 1 L-trominoe. \r\n\r\nSo $ a_{n}=a_{n-1}+2b_{n-1}+2a_{n-2}$.\r\n\r\nThen $ b_{n-1}=a_{n-2}+a_{n-3}$.\r\n\r\n$ a_{n}=a_{n-1}+4a_{n-2}+2a_{n-3}$, like JBL's answer." } { "Tag": [ "algebra", "polynomial", "function", "LaTeX", "calculus" ], "Problem": "Hey, this is a total shot in the dark, but... I just joined because I need help with a math problem [i]now[/i]. I have been working on it all night long and my brain is finally shot. I can't seem to do anything else with it. If you could show me how to solve for x I'd be forever indebted to you, but if you can't, I'll live and still thank you for reading this.\r\n\r\n[921,600x^10+1,105,920rt(3)x^8-2,525,184x^6-2,112,306rt(3)x^4+4,357,416.96x^2-1,901,076x^(-2)+268,738.56x^(-6)+597,196.8rt(3)]/[146,966,400x^4+62,985,600rt(5)x^4]=-(2/3)x^2+(36/25)x^(-2)-(6rt(3)/15)\r\n\r\nrt=square root, and I know it's long and messy, but I hope you could follow it...\r\n\r\nHow can I solve for x?", "Solution_1": ":o \r\n\r\nWhere on Earth is this problem from?", "Solution_2": "Well, just for screams and giggles, I'll translate this into latex and you can tell me if I got it right. Ok?\r\n$\\frac{[921600x^{10}+1105920\\sqrt{3}x^{8}-2525184x^{6}-2112306\\sqrt{3}x^{4}+4357416.96x^{2}-1901076x^{-2}+268738.56x^{-6}+597196.8\\sqrt{3}]}{146966400x^{4}+62985600\\sqrt{5}x^{4}}=$\r\n$=-\\frac{2}{3}\\cdot x^{2}+\\frac{36}{25}\\cdot x^{-2}-(\\frac{6\\sqrt{3}}{15})$\r\n\r\nIs that right?\r\n\r\nWell, unless you happen to have mathematica, I think you're hopelessly screwed. However, it's late at night, and this might be one of those trick questions that work out nicely. However, like Torajirou said... Where did this come from???", "Solution_3": "Yep, that's it. The problem came from my own awful mind. I had to create a story problem for AP Calculus, and I had about six pages of work to get to that point where I just totally blanked and couldn't solve anymore. I guess I'll just have to make an easier problem before third period today." } { "Tag": [ "Support", "geometry", "percent", "articles", "analytic geometry", "absolute value" ], "Problem": "My tributes to the man who did not bow to the US verdicts till the end.\r\nHe had his faults, but he is far better than tha people who hanged him.\r\n\r\nThe Iraqis and the Americans will pay for this. The fact that papa Bush now sits comfortably somewhere in his son's kingdom is especially stinging. :mad:", "Solution_1": "Yeah Saddam Hussein's defiance to the last minute is worthy of respect.", "Solution_2": "Killing people is not nice.\r\n\r\nInterpret the above as you wish...", "Solution_3": "look who's talking!\r\nwho told me once 'i have an instant urge to stab you'?", "Solution_4": "[quote=\"beta\"]Yeah Saddam Hussein's defiance to the last minute is worthy of respect.[/quote]\n\nIf non-defiance would have changed his situation in any aspect, this would make sense.\n\n[quote=\"bubka\"]He had his faults, but he is far better than tha people who hanged him.[/quote]\r\n\r\nPlease do specify the criteria for \"good\" you are using. Unless the post is meant just as another provocation to start a flamewar/spam topic.\r\n\r\n darij", "Solution_5": "[quote=\"beta\"]Yeah Saddam Hussein's defiance to the last minute is worthy of respect.[/quote]\r\nNothing about that man is worthy of respect. Have you seen the atrocities he and his son committed?\r\n\r\nThat said, killing people is still [i]not nice.[/i]\r\n\r\nEDIT: And what's up with Bubka subtitling this 'the martyr'? Honestly, this is a new low, even for Bubka.", "Solution_6": "we already had a discusion about the death penalty.\r\nsure there were some other ways than killing him...", "Solution_7": "He's no martyr. He committed genocide; he was a brutal tyrant and a dictator and in no way worthy of anyone's respect.\r\n\r\nI think the Iraq war was a bad idea from the start, and I oppose the death penalty (I think life imprisonment would be a better sentance), but I find it hard to get upset over his death, or feel any sympathy for him.", "Solution_8": "I have this question, how many of the people in this forum had heard about Sadam's crimes [i]before[/i] the begginning of hostilities that finally led to Iraq's war and Sadam's capture?\r\nThe question is relevant to me because I hear all these opinions about Sadam's being a brutal tyrant and all which I'm sure he was but were you people aware [i]before[/i] the western world campaign against Sadam started.\r\n\r\nThat trial was a fraud, pretty conveniently orchestrated from the US and of course taking place in Iraq where law allows these quick executions. Had the trial taken place in the US, there was no way you could convict Sadam and some days later execute him. That, besides the fact that the conviction came right on time for the elections, curious, pity it didn't do the republicans any good.\r\n\r\nAnd the Iraq war is for oil. The idea of US coming to save the world of the opression of the tyrant is pretty but c'mon!. The accusations made to Iraq prior to the war had nothing to do with the killing of many shii'te or kurds and were false and I don't buy they didn't know they were. And that's is despicable.\r\n\r\nAs for Sadam's hanging, I truly hope someday we won't have DP anymore. As other people have said I'm just against DP, whoever it is, I don't see the point. And it does not fall in the way of making our society better.\r\n\r\nAlso it is no joke to add the martyr thing in the title of this thread. That is also an insult to many people who suffered Sadam's regime. No matter [b]bubka[/b] what your political views on the subject are, which btw you can expose rightfully in the thread, I can't figure out the point of putting it in the title so it is probably only to make people angry about it. That is a very childish behaviour\r\n\r\nUrsula", "Solution_9": "[quote=\"deimos\"]we already had a discusion about the death penalty.\nsure there were some other ways than killing him...[/quote]\nYes but the Iraqi law did allow executions and there have been executions of ordinary criminals since the fall of the regime. So it would make no sense to pardon him and not those others. \n\n[quote=\"ursula\"]I have this question, how many of the people in this forum had heard about Sadam's crimes [i]before[/i] the begginning of hostilities that finally led to Iraq's war and Sadam's capture?\nThe question is relevant to me because I hear all these opinions about Sadam's being a brutal tyrant and all which I'm sure he was but were you people aware [i]before[/i] the western world campaign against Sadam started.[/quote]\nWell I had, but only after the first Gulf War erupted.\n\n[quote]That trial was a fraud, pretty conveniently orchestrated from the US and of course taking place in Iraq where law allows these quick executions. Had the trial taken place in the US, there was no way you could convict Sadam and some days later execute him. [/quote]\nThere was a proposal to let the trial take place in Sweden too, because they knew if he would have been sentenced to death, Sweden would have never given him back.\n\n[quote]That, besides the fact that the conviction came right on time for the elections, curious, pity it didn't do the republicans any good.[/quote]\nThat was silly.\n\n[quote]And the Iraq war is for oil. The idea of US coming to save the world of the opression of the tyrant is pretty but c'mon!. The accusations made to Iraq prior to the war had nothing to do with the killing of many shii'te or kurds and were false and I don't buy they didn't know they were. And that's is despicable.[/quote]\nIt's amazing how many people have forgotten that back in 1988,the Bush administration tried to blame Iran for the Halabja incident... :| \n\n\n[quote]Also it is no joke to add the martyr thing in the title of this thread. That is also an insult to many people who suffered Sadam's regime. No matter [b]bubka[/b] what your political views on the subject are, which btw you can expose rightfully in the thread, I can't figure out the point of putting it in the title so it is probably only to make people angry about it. That is a very childish behaviour[/quote]\r\nIt's the same kind of behavior as Chavez's, hanging out with other people who don't like the USA (he hangs with Lukashenko, the Iranian president, Castro,..)\r\nBubka's ideas tend to frighten me. He seems to defend communism at all costs, including democracy and human rights. And anti-USA sentiment, at all costs.", "Solution_10": "Okay here is a simple question:\r\nbetween saddam and papa bush, who deserves more to be hanged? (On second thoughts, I'll add drawn and quartered.)\r\n\r\nI'll come to the 'martyr' part later.", "Solution_11": "There are many ways for a man to be dead.\r\n\r\n\r\nJust think about it...\r\n\r\n\r\n\r\nIt doesn't mean just hanged or executed.\r\n\r\nHe could be killed in many other ways, not violently or with any weapons....\r\n\r\n\r\nmake him ashamed..", "Solution_12": "[quote=\"bubka\"]Okay here is a simple question:\nbetween saddam and papa bush, who deserves more to be hanged? (On second thoughts, I'll add drawn and quartered.)\n[/quote]\r\n\r\nI'd still have to say Saddam- Bush hasn't committed any sins that Saddam hasn't committed many times over.\r\n\r\nHowever I oppose the death penalty on general principle, and I would have to agree with bubka on the martyrdom thing- martyrdom is in the eyes of the beholder, and to be honest, Saddam's defiance, and his call for nonviolence at the end of his life will probably bring him to the status of martyr in the eyes of many.\r\n\r\nIn other words, the government probably shouldn't have given him the death penalty- life in prison would have been far more successful at erasing his influence.", "Solution_13": "oh dear. poor bush. mother's icicle. \r\n\r\nanyway what about donald ramsfeld who ordered the torture in abu ghraib?", "Solution_14": "[quote=\"bubka\"]oh dear. poor bush. mother's icicle. \n\nanyway what about donald ramsfeld who ordered the torture in abu ghraib?[/quote]\r\n\r\nCan you find evidence for that statement?", "Solution_15": "[quote=\"Go Around the Tree\"][quote=\"kstan013\"]In a land using the Code of Hammurabi, Saddam was treated, er, justly.\n@bubka What if I said when that the tsunami hit southern India, I was very happy although some Americans died. And the more frightening thing about 9/11 was the TERROR. Living in the US is not as happy jolly as you might think. :wink:[/quote]\n\n\nI quite agree. TERRORISTS ARE TERRIBLE.\n\nSADDAM WAS A TERRORIST. SADDAM=TERRIBLE\n\n\nGet the picture?\n\n\nI don't see why they hanged him so soon though. :maybe: \n\n\nwere you happy, guys, that the WTC was knocked down? \n\n\nYou support Saddam? \n\n\nWHYYY???? \n\nSADDAM IS A TERRORIST. ALTHOUGH BUSH IS A BAD PRESIDENT IN MY OPINION, SADDAM IS STILL MORE EVIL .[/quote]\nDoes Saddam have anything to do with the attacks on the World Trade Center?\n\n\n[quote]NUMBER OF PEOPLE KILLED ON THE 9/11 ATTACKS \n\n2973 \n\nNow... \n\nDarfur conflict, Sudan(ongoing): ~400 000(mostly civilians) \nSecond Sudanese Civil War, Sudan (1983-2005): ~2 000 000 deaths \nTutsis-Hutus conflict, Rwanda (three months of 1994): ~1 000 000 deaths (that's 10 000 per day) \nIraqi conflict, Iraq(ongoing): 3000+ US soldiers, the number of Iraqi civilian casualties differ greatly, from 30 000 to 600 000 deaths \nChechenyan War, Chechenya(1994-1996): 30 000 ~ 100 000 deaths many more thousands in the second conflict. \nGulf War, Kuwait: 100 000 Iraqi deaths \nKosovo War: ~12 000 deaths, NATO bombing in 1999, 2000 civilian deaths \nCroatian Independence War: ~15 000 deaths \nEritrea Ethiopia war (1998-2000): 70 000~170 000 deaths \nAlgerian Civil War, (1992-2001): 100 000 ~ 200 000 deaths \nSierra Leone Civil War(1991-2001): ~75 000 deaths [/quote]\r\n\r\nYou left out the Second Congo War(1998-2003) : more than 3 500 000 civilian casualties :ninja: \r\n\r\nIt's time we simply accept that people have double standards. Posters here are doing their best to express grief for the people who died in Manhattan but seem to forget that action like those are a consequence of US policy and MORE IMPORTANTLY : will lead to much more of the same suffering among others (the Iraqi people first)\r\nPeople get bombed everyday, I suggest you watch Fahrenheit 9/11 if you wanna see real wounded peopple, not the clean version. \r\nAnd now this has led to retaliations in Spain and London...", "Solution_16": "[quote=\"fredbel6\"][quote=\"Go Around the Tree\"][quote=\"kstan013\"]In a land using the Code of Hammurabi, Saddam was treated, er, justly.\n@bubka What if I said when that the tsunami hit southern India, I was very happy although some Americans died. And the more frightening thing about 9/11 was the TERROR. Living in the US is not as happy jolly as you might think. :wink:[/quote]\n\n\nI quite agree. TERRORISTS ARE TERRIBLE.\n\nSADDAM WAS A TERRORIST. SADDAM=TERRIBLE\n\n\nGet the picture?\n\n\nI don't see why they hanged him so soon though. :maybe: \n\n\nwere you happy, guys, that the WTC was knocked down? \n\n\nYou support Saddam? \n\n\nWHYYY???? \n\nSADDAM IS A TERRORIST. ALTHOUGH BUSH IS A BAD PRESIDENT IN MY OPINION, SADDAM IS STILL MORE EVIL .[/quote]\nDoes Saddam have anything to do with the attacks on the World Trade Center?\n\n\n[quote]NUMBER OF PEOPLE KILLED ON THE 9/11 ATTACKS \n\n2973 \n\nNow... \n\nDarfur conflict, Sudan(ongoing): ~400 000(mostly civilians) \nSecond Sudanese Civil War, Sudan (1983-2005): ~2 000 000 deaths \nTutsis-Hutus conflict, Rwanda (three months of 1994): ~1 000 000 deaths (that's 10 000 per day) \nIraqi conflict, Iraq(ongoing): 3000+ US soldiers, the number of Iraqi civilian casualties differ greatly, from 30 000 to 600 000 deaths \nChechenyan War, Chechenya(1994-1996): 30 000 ~ 100 000 deaths many more thousands in the second conflict. \nGulf War, Kuwait: 100 000 Iraqi deaths \nKosovo War: ~12 000 deaths, NATO bombing in 1999, 2000 civilian deaths \nCroatian Independence War: ~15 000 deaths \nEritrea Ethiopia war (1998-2000): 70 000~170 000 deaths \nAlgerian Civil War, (1992-2001): 100 000 ~ 200 000 deaths \nSierra Leone Civil War(1991-2001): ~75 000 deaths [/quote]\n\nYou left out the Second Congo War(1998-2003) : more than 3 500 000 civilian casualties :ninja: \n\nIt's time we simply accept that people have double standards. Posters here are doing their best to express grief for the people who died in Manhattan but seem to forget that action like those are a consequence of US policy and MORE IMPORTANTLY : will lead to much more of the same suffering among others (the Iraqi people first)\nPeople get bombed everyday, I suggest you watch Fahrenheit 9/11 if you wanna see real wounded peopple, not the clean version. \nAnd now this has led to retaliations in Spain and London...[/quote]\r\n\r\n\r\n\r\nBush really didn't have a GOOD reason. He just associated Terrorists with WTC with Saddam.\r\n\r\n\r\nEven though Saddam didn't cause WTC and wasn't involved, Bush uses the tone that makes us believe he is in with the same group and that is used as the reason why we are in Iraq.", "Solution_17": "I agree that the media takes what they think will sell the most papers and blows it out of perspective. Yet, despite this, many Americans are becoming aware of other problems, such as the Darfur conflicts. Recently, that has become a big thing here, and I have even seen a few disaster relief fund commercials.\r\n\r\nAlso, on a separate note, even as a conservative Republican, I do not support the war in Iraq, nor do I agree with many of Bush's policies. What a lot of foreign people don't get is that a strong majority of Americans do not support the war in Iraq. America is around 50% Republican (Bush's party) and 50% Democratic. However, Bush's approval rating dipped below 40% at the end of last month, and less than 35% of Americans approve of the war in Iraq.", "Solution_18": "[quote=\"fredbel6\"][\n\nYou left out the Second Congo War(1998-2003) : more than 3 500 000 civilian casualties :ninja: \n[/quote]\r\n\r\nYou are quite right. I firstly wrote the conflicts I remembered and then went through Internet looking for the death tolls I didn't knew about. But I knew that you were going to be there if I missed any :)\r\n\r\nUrsula", "Solution_19": "[quote=\"ursula\"][quote=\"fredbel6\"][\n\nYou left out the Second Congo War(1998-2003) : more than 3 500 000 civilian casualties :ninja: \n[/quote]\n\nYou are quite right. I firstly wrote the conflicts I remembered and then went through Internet looking for the death tolls I didn't knew about. But I knew that you were going to be there if I missed any :)\n\nUrsula[/quote]\r\n\r\nYou missed the toll in Bosnia too (150.000-200.000)... :!:", "Solution_20": "[quote=\"hsiljak\"][quote=\"ursula\"][quote=\"fredbel6\"][\n\nYou left out the Second Congo War(1998-2003) : more than 3 500 000 civilian casualties :ninja: \n[/quote]\n\nYou are quite right. I firstly wrote the conflicts I remembered and then went through Internet looking for the death tolls I didn't knew about. But I knew that you were going to be there if I missed any :)\n\nUrsula[/quote]\n\nYou missed the toll in Bosnia too (150.000-200.000)... :!:[/quote]\r\n\r\nOK thanks, :) I'm editing the post, so it is more complete. \r\n\r\nUrsula", "Solution_21": "THE AMERICAN WAY OF LIFE\r\naccording to bubka\r\n\r\nEveryone goes to the beach every day,\r\nnobody ever has to work,\r\nwe all sit in recumbent lawn chairs all day sipping pina coladas out of coconut shells,\r\noil is flowing out of our ears and we burn it for fun,\r\nwe worship Bush and never ever disagree with him,\r\nwe kill Iraqis in our free time\r\n\r\nBubka, we donated millions of dollars in relief efforts to help the tsunami victims and what did we get for 9/11? Not a dime.\r\n\r\nI realize that the Iraqi way of life was bad, don't get me wrong, but when I said [quote=\"kstan013\"]Living in the US is not as happy jolly as you might think. :wink: [/quote]I was referring to how you think Americans live.", "Solution_22": "[quote=\"kstan013\"]Bubka, we donated millions of dollars in relief efforts to help the tsunami victims and what did we get for 9/11? Not a dime.\n[/quote]\r\n\r\nI'm sorry my friend but that argument is ridiculous.\r\n1) A lot of developed countries and not developed gave help to the tsunami victims one way or another. I don't know about the actual numbers but certainly the US weren't the only good souls in the world. I'm not sure about this one but it tends to happen that the help the US gives is absurd compared with what other countries do.\r\n\r\n2) There are not a lot of countries who can presume of being able to give something to the US that they can not proveide themselves. If furthermore such countries observe that after 9/11 you rather spend the money in multiple wars - and that's a lot of money -, money that could be spent in reconstruction or in helping the families of the victims, or the families of the firemen now sick, or en fin preventing Katrina, in such a case, you question the point of helping the US.\r\n\r\n3) I remember you that a long time ago there was this UN agreement about donating 0.7 percent of the GIP of all developed countries to the developing ones. The greatest GIP of the world, that is, the US, set an example by donating about 0.15 percent some years. I don't know how the thing is now, but your behavior towards the developing countries is full of stains.\r\n\r\nUrsula", "Solution_23": "I don't understand why you all find a necessity to make juxtapositions with other items that have no implication on the absolute value of a deed.\r\n\r\nBush vs. Saddam\r\nHurricane deaths vs. trade center\r\n\r\nSimply put Bubka, let's take things for face value. 1 death due to evil is wrong. That's it. There is no need to compare it to a natural disaster or what Americans did or so on. Looking at everything from an individualistic standpoint ellicits a neutral position on the subject that is unbiased by the influence of needless comparisons. This is the most logical way of handling it.\r\n\r\nI am no judge, but if you consider yourself a good person you will admit that evil is evil at face value.", "Solution_24": "[quote=\"Potato Theory\"]\nI am no judge, but if you consider yourself a good person you will admit that evil is evil at face value.[/quote]\r\n\r\n[b]Yay for Ad Hominem arguments![/b]\r\n\r\nThough I am still not sure why this thread hasn't undergone serious deletion by mods.\r\n\r\nAt any rate, it would be kinda cool for people, instead of accusing everyone else of making stuff up, to give us credible sources that back their arguments (by credible I mean actually credible, if you don't know what I mean, well, at the very least don't cite random news articles or something that are all going to be obviously biased).", "Solution_25": "[quote=\"JSteinhardt\"]\nThough I am still not sure why this thread hasn't undergone serious deletion by mods.\n[/quote]\r\n\r\nWell, it is a serious discussion, although totally off topic thanks to our friend bubka (and me too for responding to him, I guess).", "Solution_26": "[quote=\"JSteinhardt\"]\n[b]Yay for Ad Hominem arguments![/b]\n[/quote]\r\n\r\nThe best kind! :wink:", "Solution_27": "[quote=\"ursula\"][quote=\"kstan013\"]Bubka, we donated millions of dollars in relief efforts to help the tsunami victims and what did we get for 9/11? Not a dime.\n[/quote]\n\nI'm sorry my friend but that argument is ridiculous.\n1) A lot of developed countries and not developed gave help to the tsunami victims one way or another. I don't know about the actual numbers but certainly the US weren't the only good souls in the world. I'm not sure about this one but it tends to happen that the help the US gives is absurd compared with what other countries do.[/quote]\n\n[url=http://en.wikipedia.org/wiki/Humanitarian_response_to_the_2004_Indian_Ocean_earthquake#List_of_Donors]link[/url]\n\n[quote]2) There are not a lot of countries who can presume of being able to give something to the US that they can not proveide themselves. If furthermore such countries observe that after 9/11 you rather spend the money in multiple wars - and that's a lot of money -, money that could be spent in reconstruction or in helping the families of the victims, or the families of the firemen now sick, or en fin preventing Katrina, in such a case, you question the point of helping the US.[/quote]\r\nTrue, after 9/11 we heard the usual \"stand as one\" talk... but what does it really mean? Lots of firemen are now sick and unable to provide for themselves. In the mean time 3000 US soldiers have died in Iraq in a war that has cost more than 350 $10^{9}$ US dollars.\r\n\r\nBut what you say about helping the US is not entirely correct. Even Afghanistan sent some money after Katrina even though that was probably symbolic. But Belgium sent some soldiers to Louisiana to help them coordinate the relief efforts... even though we were quite surprised that a superpower able to wage several costly wars at the same time was not able to help its own citizens in its backyard... The Dutch, masters when it comes to manipulating water, have also offered help reconstructing the levees.", "Solution_28": "EDIT: post removed...\r\n\r\nOne thing I would like to mention is that those of you who want to discuss \"hot\" issues like this should try to do so in a less abrasive manner. While I hate to say it, I have learned one thing from my learning argumentation in english. What you have to say is less important than how you say it. This is why I suggest [note I claim no authority] that people, like Bubka, consider rogerian argumentation. That way, you don't look like a troll who just wants to start a flame war [not only this topic, but the topic about the cost of WOOT].", "Solution_29": "[quote=\"fredbel6\"]But what you say about helping the US is not entirely correct. Even Afghanistan sent some money after Katrina even though that was probably symbolic. But Belgium sent some soldiers to Louisiana to help them coordinate the relief efforts... even though we were quite surprised that a superpower able to wage several costly wars at the same time was not able to help its own citizens in its backyard... The Dutch, masters when it comes to manipulating water, have also offered help reconstructing the levees.[/quote]\n\n[quote=\"fredbel6\"]But what you say about helping the US is not entirely correct. Even Afghanistan sent some money after Katrina even though that was probably symbolic. But Belgium sent some soldiers to Louisiana to help them coordinate the relief efforts... even though we were quite surprised that a superpower able to wage several costly wars at the same time was not able to help its own citizens in its backyard... The Dutch, masters when it comes to manipulating water, have also offered help reconstructing the levees.[/quote]\r\n\r\nI didn't actually meant the US didn't receive any help for Katrina :). Actually even [url=http://www.cnn.com/2005/WEATHER/09/03/katrina.castro/]Cuba[/url] proposed to send help to the US. But I see it can be misunderstood.\r\nOne point more in my argument could be that compared to the tsunami, 9/11 is a minor catastrophe in terms of human lives lost aswell as of economical damage.\r\n\r\nAlso [url=http://www.unmillenniumproject.org/involved/action07.htm]here you have[/url] the link about the 0.7 percent the rich countries agreed to donate to the developing ones. There you have data for year 2005. You see that the US is along with Portugal the developed country who gives [i]the least[/i], you can also see that is one of the few who have NOT set a timetable for doing it. Draw your own conclusions\r\n\r\nUrsula" } { "Tag": [ "ARML", "Support" ], "Problem": "Here's another change to the ARML rules for 2009:\r\n\r\n[quote]In order to keep the proportions of points as they were prior to the addition of the fifth individual round, the scoring structure will be altered as follows.\n\n * Individual Rounds: 1 pt each (total of 10 points possible per person, 150 points possible per team)\n\n\n * Team Round: 5 pts each (total of 50 possible points per team)\n\n\n * Power Round: 50 points possible points per team\n\n\n * Relay Rounds: correct answer submitted in 3 minutes 3 pts, correct answer submitted in 6 minutes 2 pts. (15 points possible per team for Relay 1, 15 points possible per team for Relay 2)[/quote]\r\n\r\nSo instead of 120/40/40/40, it's going to be 150/50/50/30. It doesn't actually keep the proportions of points the same, but I guess they only meant that to apply to individual, team, and power. Comments?\r\n\r\nI guess I like it. Relays always seem kind of unpredictable, and it's too easy to blow a big lead with a bad relay.\r\n\r\nBy the way, NYSML has made the same change, except relays are worth 50 points (5 for 3 minutes, 2 for 6 minutes).", "Solution_1": "I was under the impression that in order to both preserve proportions for the events and make the relay scoring gentler on B teams, the values were going to change to 5 pts / 3 minutes and 3 pts / 6 minutes.\r\n\r\nThe scoring Silas mentions indeed makes the 6 minutes less penalizing than before, since it is now worth more than half the previous scoring. I know that relays seem to elicit the most contentious opinions from ARML students and coaches, so I'm curious how reducing the overall impact of the relays will be met. \r\n\r\nI guess it may depend somewhat on how seriously you take the competition. I see ARML as more \"for fun\" than anything else at this level, so I don't mind the random element that the relays seem to impart. From a coaching standpoint, the possibility of a massive swing in the standings heading into the final event is a good motivator for my students... no matter how bad a day they might have had up to that point, they can still jump several spots if they buckle down and do a good job.", "Solution_2": "I think of all the events, the relays is the one that is hardest to \"carry,\" with a couple good people, that is, you need many smart people to score well on relays, whereas for team and power, a couple strong people (2 in the case of my ARML team last year) can lead the team to 30+ scores.", "Solution_3": "Yeah, I would have thought 5/3 also. I would also support that scoring. I would object to 3/1 or 5/2, because I've always thought 6-minute answers should be worth at least half as much as 3-minute answers.\r\n\r\nGoing to 5/3 relays would also give a way to make a rough comparison between new-format and old-format scores.", "Solution_4": "An excellent change! I am completely in favor of it.", "Solution_5": "I emailed the head writer and I think indeed that there was a miscommunication regarding the relay scoring rules. Hopefully this will be clarified soon :lol:", "Solution_6": "I heard from Dr. Merryfield that the relay score won't be changing at all... so not 40 to 30, but just 40 still.", "Solution_7": "Brut3Forc3, my source was this thread, which I seem to have misread. Believe what you see here.", "Solution_8": "A lot of ARML officials have been emailing each other voicing their opinions on the relay scoring change. It is sounding like the 3/2 scoring rubric sent out earlier was the result of some miscommunication, and that the intended scoring was 5/3. Hopefully that will be made official soon :)", "Solution_9": "Apparently things have been taken care of. It will in deed be 3 points for a 6 minute answer, 5 points for a 3 minute answer, and 23 points for a 36-second answer.", "Solution_10": "[quote=\"generating\"]Apparently things have been taken care of. It will in deed be 3 points for a 6 minute answer, 5 points for a 3 minute answer, [b]and 23 points for a 36-second answer.[/b][/quote]\r\n\r\nHuh? I hope you were kidding.", "Solution_11": "Note the date.", "Solution_12": "Which is why I was going to wait until tomorrow to ask him about the first part of that sentence." } { "Tag": [], "Problem": "hi, I'm new and I thought I would post a problem... at least to see what happens :D \r\n\r\nHere it is\r\n\r\nLaura is 3 times as old as Sara was when Laura was as old as Sara is now. In 2 years Laura will be twice as old as Sara was 2 years ago. How old are they now?", "Solution_1": "Well, what happens is people like me respond with a answer and a solution.\r\n[hide]We can deduct that Laura has to be a multiple of six, because her age is a multiple of 3 and has to be able to be divisible by two because an odd +2 is always odd.\nSo plug in 6 and see if it works, which it doesn't\nThen try 12 and see if it works, and it does, so \n[b]12[/b] is the answer[/hide]\r\nEDIT- I mean plug in 6 and 12 for the age of Laura.", "Solution_2": "me too! :) \r\n[hide]i found that for every 3 years Laura goes up, Sara goes up by two. so, i estimated and trial and error two or three times, and i got Laura = 18 and Sara = 12.[/hide]", "Solution_3": "[hide=\"an albegraic solution :)\"]\nCall Laura's age L\nSara's age S\n\nFor the second statement, it is easy to decode.\n$L+2=2(S-2)$\n$L+2=2S-4$\n$2S-L=6$\nFor the first, \nIf the difference in ages is $d$,\nthen when Laura was Sara's age, Sara was $S-d=S-(L-S)=2S-L$\nThus $L=3(2S-L)$\n$L=6S-3L$\n$4L-6S=0$\n$2L-3S=0$\n\nNow we have to solve.\nChange the first equation to \n$-2L+4L=12$\n$2L-3S=0$\n\nAdd\n$L=12$\n\nSubstitute back in to get $S=8$[/hide]" } { "Tag": [ "LaTeX", "USAMTS" ], "Problem": "for one of my usamts files i have 8 syntax errors but i cant find anyof them and the doc is only like 2 paragraphs long wut do i do i no i cant post it for people to look at because of the honor code\r\nalso is there a way to get a spell check in latex?", "Solution_1": "Usually, it's one error that causes several errors. For Example (not breaking honor code)...\r\n\r\nblablabla\r\n\\ begin*{eqnarray}\r\n\r\nblabla...\r\n\r\n\\end*{eqnarray}\r\n\r\nwill create about 100 errors while\r\n\r\nblablabla\r\n\\ begin*{eqnarray}\r\nblabla...\r\n\\end*{eqnarray}\r\n\r\nwill create none.", "Solution_2": "Usually, when you have a string or two of LaTeX, it's hard to determine where the error is. So I recommend deleting like the second row and see if the problem persists. If it does, then delete more until you have just the code that shows up right and try to fix from there. I used to look at entire string but what happens is sometimes, you actually make correct code to be wrong, thinking it was the wrong code and therefore, make the problem even more difficult.", "Solution_3": "are you saying that u dont put a spare line after \\begin{document}\r\nand before \\end{document} because i did\r\nill try removing the lines", "Solution_4": "i just found a section of the forum for latex so ill stop posting latex stuff here" } { "Tag": [ "number theory unsolved", "number theory" ], "Problem": "Solve in $ \\mathbb{P}$ the equation $ 2^{x\\plus{}1}\\plus{}y^2\\equal{}z^2$.", "Solution_1": "(z+y)(z-y)=2^(x+1). As z+y > z-y, z+y is a multiple of z-y\r\n\r\nAll primes greater than 3 can be expressed as 6k+1or 6k-1. So, either z+y is a multiple of 3 or \r\nz-y is a multiple of 3. So, not possible for y>3.\r\nFor y=3,\r\nz+3 is a multiple of z-3. So, z<7. The only possible prime value for z=5\r\nFor y=2,\r\nz<5. So, no possible primes for this case.\r\n\r\n(2,3,5) is the only solution in P.", "Solution_2": "i have a question\r\nwhy you said\r\n$ x\\plus{}y>x\\minus{}y\\Longrightarrow x\\minus{}y\\mid x\\plus{}y$\r\ni didnt understand :blush:", "Solution_3": "[quote=\"pelao_malo\"]i have a question\nwhy you said\n$ x \\plus{} y > x \\minus{} y\\Longrightarrow x \\minus{} y\\mid x \\plus{} y$\ni didnt understand :blush:[/quote]\r\nbecause x+y and x-y are both powers of 2, which means x+y is a greater power of 2 than x-y" } { "Tag": [ "calculus", "integration", "limit", "real analysis", "function", "calculus computations" ], "Problem": "What condition is necessary so that:\r\n\r\n$ \\lim_{x\\to\\infty}\\left(\\int_{a}^{b}f(x,y) \\ dy\\right) \\equal{} \\int_{a}^{b}\\lim_{x\\to\\infty}f(x,y) \\ dy$\r\n\r\n?", "Solution_1": "I don't want to get into asking about necessary conditions; I suspect there really isn't anything nice you can say. In other words, sometimes you \"shouldn't\" be able to interchange the limit and integral, but it works anyway.\r\n\r\nThere are several reasonable sufficient conditions.\r\n\r\n1. If $ f(x,y)\\to g(y)$ [b]uniformly[/b] as $ x\\to\\infty,$ then this is justified. That is, $ \\forall\\,\\epsilon>0\\,\\exists\\,N\\equal{}N(\\epsilon)$ such that if $ x>N,$ then $ |f(x,y)\\minus{}g(y)|<\\epsilon.$\r\n\r\n2. If there exists $ h(y)$ defined almost everywhere on $ [a,b]$ such that $ |f(x,y)|\\le h(y)$ and $ \\int_a^bh(y)\\,dy<\\infty,$ then this is justified. Of course, this is simply the Lebesgue Dominated Convergence Theorem, with some minor stuff about turning limits on arbitrary sequences into limits at infinity.", "Solution_2": "If the functions in question are non-negative, the interval is finite, the limit exists a.e. or in measure, and the integral of the limit function is finite, then eventual equiintegrability (or whatever the right term in English is) is necessary and sufficient. The property is just that for every $ \\varepsilon>0$ there exists $ \\delta>0$ such that whenever the measure of a set $ E\\subset[a,b]$ is less then $ \\delta$, one has $ \\int_E f(x,y)\\,dy<\\varepsilon$ for all sufficiently large $ x$. Another way to state the same is to say that there exists a function $ \\Psi: [0,\\plus{}\\infty)\\to[0,\\plus{}\\infty)$ such that $ \\lim_{t\\to\\plus{}\\infty}\\frac{\\Psi(t)}{t}\\equal{}\\plus{}\\infty$ and $ \\limsup_{x\\to\\infty}\\int_a^b \\Psi(f(x,y))\\,dy<\\plus{}\\infty$. For sign changing functions it is as close to a necessary and sufficient condition as you can possibly wish but, of course, as Kent said there is nothing that is absolutely necessary." } { "Tag": [ "number theory unsolved", "number theory" ], "Problem": "Find all pairs of positive integers $ (a, b)$ such that\r\n\\[ ab \\equal{} gcd(a, b) \\plus{} lcm(a, b).\r\n\\]", "Solution_1": "[quote=\"STARS\"]Find all pairs of positive integers $ (a, b)$ such that\n\\[ ab \\equal{} gcd(a, b) \\plus{} lcm(a, b).\n\\]\n[/quote]\r\n\r\n\r\n\r\n$ (a;b)\\equal{}k$----->$ a\\equal{}nk;b\\equal{}mk$-----$ (m,n)\\equal{}1$---->$ [a;b]\\equal{}nmk$----->$ ab \\equal{} gcd(a, b) \\plus{} lcm(a, b)\\equal{}(a;b)\\plus{}[a;b]$----->\r\n\r\n\r\n$ nmk^{2}\\equal{}k\\plus{}mnk$-----> $ mnk\\equal{}1\\plus{}mn$----->$ k\\equal{}\\frac{1}{mn}\\plus{}1$---->$ mn\\equal{}1$-----> $ m\\equal{}n\\equal{}1$ and $ k\\equal{}2$----> $ a\\equal{}b\\equal{}2$", "Solution_2": "It is well known that $ ab \\equal{} \\gcd(a,b)lcm(a,b)$, so\r\n\r\n$ \\gcd(a,b)lcm(a,b) \\equal{} \\gcd(a,b) \\plus{} lcm(a,b)$\r\n$ (\\gcd(a,b) \\minus{} 1)(lcm(a,b) \\minus{} 1) \\equal{} 1$\r\n\r\n$ \\gcd(a,b) \\equal{} lcm(a,b) \\equal{} 2$.\r\n\r\n$ (a,b) \\equal{} (2,2)$" } { "Tag": [], "Problem": "Four people think of different positive integers. They add them and their sum is 10. What is the greatest integer?", "Solution_1": "Lets say biggest integer is more then 5. Then other 3 integer have to sum up to equal or less then $ 5$.\r\nSmallest sum of different positive integer is $ 1 \\plus{} 2 \\plus{} 3 \\equal{} 6$ Therefore biggest integere has to be $ 4$ or less.\r\nWe can easily see that integers are $ 1,2,3,$ and $ 4$ Therefore $ 4$ is biggest integer" } { "Tag": [ "function", "algebra unsolved", "algebra" ], "Problem": "Determine all function $f: \\mathbb N \\to \\mathbb N$ such that\n\\[f(m+n)f(m-n) = f(m^{2}),\\]\nfor all $m,n \\in \\mathbb N$.", "Solution_1": "[quote=\"hoangclub\"]Denote $ Z^{+}$ is set of all positive integer numbers. \n determine all function $ f$:$ Z^{+}$ ->$ Z^{+}$ such that\n $ f(m+n)f(m-n)$=$ f(m^{2})$ for all m,n $ \\in$ $ Z^{+}$.[/quote]\r\n\r\n${ f(n)f(n+4)=f((n+2)^{2}})=f(n+1)f(n+3)$ and so $ \\frac{f(n+4)}{f(n+3)}=\\frac{f(n+1)}{f(n)}$ and so It exists $ u,a,b,c$ such that :\r\n$ f(3p+1)=ua^{p}b^{p}c^{p}$\r\n$ f(3p+2)=ua^{p+1}b^{p}c^{p}$\r\n$ f(3p+3)=ua^{p+1}b^{p+1}c^{p}$\r\n\r\nThen :\r\n$ f(1)f(3)=f(4)$ $ \\implies$ $ u uab=uabc$ and $ u=c$\r\n$ f(1)f(5)=f(9)$ $ \\implies$ $ u ua^{2}bc = ua^{3}b^{3}c^{2}$ and $ u=ab^{2}c$\r\n$ f(1)f(7)=f(16)$ $ \\implies$ $ u ua^{2}b^{2}c^{2}= ua^{5}b^{5}c^{5}$ and $ u=a^{3}b^{3}c^{3}$\r\n$ f(1)f(9)=f(25)$ $ \\implies$ $ u ua^{3}b^{3}c^{2}= ua^{8}b^{8}c^{8}$ and $ u=a^{5}b^{5}c^{6}$\r\n\r\nAnd so $ a=b=c=u=1$\r\n\r\nAnd $ f(n)=1$ $ \\forall n$" } { "Tag": [ "inequalities", "induction", "inequalities proposed" ], "Problem": "Prove that for all positive integers $0=n-i$\n2)$a_n>=n$", "Solution_4": "Hmmm strange... the identity $\\sum_{i=1}^n{i^3}=\\left(\\sum_{j=1}^n{j}\\right)^2$ kills this problem without thought. A curious choice of a TST problem..." } { "Tag": [ "combinatorics unsolved", "combinatorics" ], "Problem": "Can we color some $1\\times1$ squares in $1377\\times1377$ square that each $1\\times1$ square has odd colored neighbors.(Each square has at most 8 neighbors).", "Solution_1": "I solved this problem today.\r\nThis is very beautiful problem I suggest you solve it.", "Solution_2": "I think I solved it. The answer is no.\r\nMy solution is hard to understand without a sheet of paper.\r\nI prove that the number of coloured bordering squares must be odd, the even so reaching a contradiction.\r\nPlease tell me if my answer is correct.\r\nI would like to see your solution", "Solution_3": "I did'nt solved it in this way but I like to see your solution.\r\nI send my solution later.", "Solution_4": "Hi!\r\nI don't know about iura, but when I last asked him (10 May), he said his solution was wrong. Anyway, I would like to see you solution, Omid.\r\nI have an idea, similar to another problem.\r\nMake a graph with each square and put some numbers in each square and some numbers in each edge of the graph. The numbers must be picked in such a way, so that if one square is colored black, the parity doesn't change. In the initial situation, if it would be possible, then we would heve 1 parity, in the final, when all are colored, the sum must be another parity.\r\nThis is like another IRAN problem ,1998, Titu Andreescu Math Olymp Chalenges", "Solution_5": "I tell you ahint.\r\nChoose all cells (i,j) that both i and j are both odd.....", "Solution_6": "We prove the problem with contradiction. Suppose there exists such coloring. Consider set $S$ of cells of tables $(i,j)$ that both $i,j$ are odd. For each cell in $S$ consider the number of its black neighbors and let sum of these numbers to be $A$. Because $|S|=\\frac{1378^{2}}4=689^{2}$, thus $A$ is odd. Also each black cell is counted as the number of its neighbors that are in $S$. Because for each cell this number is even, so $A$ is even. Contradiction proves everything. :)", "Solution_7": "[quote]Also each black cell is counted as the number of its neighbors that are in . Because for each cell this number is even[/quote]\r\n\r\nI don't think this is true...", "Solution_8": "That's true and is obvious. Also you see each cell has even neigbors in $S$. (Read the problem carefully and note that each cell has 8 neighbors (We consider vertex neighbors.) )", "Solution_9": "could you write the problem one more time??", "Solution_10": "A black cell at (1, 1) only has one neighbor in S...", "Solution_11": "Of course $(1,1)\\in S$ and each cell is not neighbor of itself, and it does not have any other neighbors." } { "Tag": [ "algebra", "polynomial", "logarithms", "algebra unsolved" ], "Problem": "Let $x_1,x_2,x_3,...,x_{n-1}$, be the seros different from 1 of the polynomial $x^n-1$, here $n\\ge 2$. Prove that \\[\\frac{1}{1-x_1}+\\frac{1}{1-x_2}+\\frac{1}{1-x_3}+...+\\frac{1}{1-x_{n-1}}=\\frac{n-1}{2}\\].", "Solution_1": "For $x\\neq 1$, $x^n-1=(x-1)\\cdot \\frac{x^n-1}{x-1}=(x-1)(x^{n-1}+x^{n-2}+\\cdots\\cdots +x+1)$\r\n\r\nSince $x_1,x_2,\\cdots x_{n-1}$ are the roots of the equation of $x^{n-1}+x^{n-2}+\\cdots x+1=0$, we have\r\n\r\n$x^{n-1}+x^{n-2}+\\cdots x+1=(x-1)(x^{n-1}+x^{n-2}+\\cdots\\cdots +x+1)$. Taking logarithm of the both sides,\r\n\r\nwe have $\\log |x^{n-1}+x^{n-2}+\\cdots\\cdots +x+1|=\\log |x-x_1|+\\log |x-x_2|+\\cdots +\\log |x-x_n|$,\r\n\r\nDifferentiating of both sides with respct to $x$, for $n=2,3,\\cdots$, we get \r\n\r\n$\\frac{(n-1)x^{n-2}+(n-2)x^{n-3}+\\cdots +2x+1}}{x^{n-1}+x^{n-2}+\\cdots x+1}$\r\n\r\n\r\n$=\\frac{1}{x-x_1}+\\frac{1}{x-x_2}+\\cdots +\\frac{1}{x-x_{n-1}}$\r\n\r\n\r\nPlugging $x=1$ into the both sides of this equation, we obtain $\\frac{1}{1-x_1}+\\frac{1}{1-x_2}+\\cdots +\\frac{1}{1-x_{n-1}}=\\frac{\\frac{(n-1)n}{2}}{n}=\\frac{n-1}{2}$. Q.E.D.", "Solution_2": "I think I'm missing a huge shortcut, now that I read the title.\r\n\r\nBut it's really very simple.\r\n\r\nWe want to show that\r\n\r\n$\\sum_i \\prod_{j\\not=i} (1-x_j)=\\frac {n-1}{2}\\prod_k (1-x_k)$\r\n\r\nUsing Viete's formulae, it is very easy to show that \r\n\r\n$\\prod_k (1-x_k)=1-\\sum_k x_k+\\sum_{k{\\infty}} {(x {(1+ \\frac{1}{x})}^{x}-ex) }$\r\nPlease give me a general technic because I have lots of problems of this type and I can not solve them by using fundamental limits or by applying l'Hospital rule ( I get $\\infty - \\infty$)", "Solution_1": "[quote=\"nemesis\"]Could anyone explain how to determine the fallowing limit:\n$\\lim_{ x->{\\infty}} {(x {(1+ \\frac{1}{x})}^{x}-ex) }$\nPlease give me a general technic because I have lots of problems of this type and I can not solve them by using fundamental limits or by applying l'Hospital rule ( I get $\\infty - \\infty$)[/quote]\r\n\r\nI assume by ex you mean exp(x). You have:\r\n\r\nx*(1+1/x)^x-exp(x) = \r\nx*exp(x*ln(1+1/x))-exp(x) = \r\nx*exp(x*(1/x+o(1/x)))-exp(x) =\r\nx*exp(1+o(1))-exp(x), which goes to +oo as x->+oo. \r\n\r\nIf by ex you meant exp(1)*x then you need to develop further:\r\n\r\nx*exp(x*(1/x-1/(2x^2)+o(1/x^2)))-e*x =\r\nx*exp(1-1/(2x)+o(1/x))-e*x = \r\ne*x*(-1/(2x)+o(1/x))-e*x,\r\n\r\nand now the limit is -3e/2.\r\n\r\nP.S: I didn't check the numerical values above so be careful, but the idea is here: using taylor developments.", "Solution_2": "As $y\\to0,$ $\\ln(1+y)=y-\\frac{y^2}2+\\frac{y^3}3+\\cdots.$\r\n\r\nSo $\\ln\\left[\\left(1+\\frac1x\\right)^x\\right]=x\\ln\\left(1+\\frac1x\\right)$\r\n\r\n$=x\\left(\\frac1x-\\frac1{2x^2}+O(x^{-3})\\right)=1-\\frac1{2x}+O(x^{-2})$\r\n\r\nNow, $e^{1+u}=e\\cdot e^u=e\\left(1+u+O(u^2)\\right)$\r\n\r\nSo $\\exp\\left\\{\\ln\\left[\\left(1+\\frac1x\\right)^x\\right]\\right\\}=\\dots$\r\n\r\n(To be completed later, although you're welcome to finish it yourself.)", "Solution_3": "no, it is x*(1+1/x)^x-ex , \r\n$\\lim_{ x->{\\infty}} {(x {(1+ \\frac{1}{x})}^{x}-ex)}$", "Solution_4": "By the way, what is it $O(u^2)$ \r\n :?:", "Solution_5": "[quote=\"nemesis\"]no, it is x*(1+1/x)^x-ex ,[/quote]\nNot so literal minded! Can't you see that I'm working on parts of the problem, to be assembled in the end? Have a little imagination.\n\n[quote=\"nemesis\"]By the way, what is it $O(u^2)$ [/quote]\r\nSee [url=http://www.artofproblemsolving.com/Forum/viewtopic.php?t=31517]this topic.[/url]", "Solution_6": "julien_santini had the right idea and came close, but some of the details slipped.\r\n\r\nContinuing from #3,\r\n\r\n$\\left(1+\\frac1x\\right)^x= \\exp\\left\\{\\ln\\left[\\left(1+\\frac1x\\right)^x\\right]\\right\\}= \\exp\\left(1-\\frac1{2x}+O(x^{-2})\\right)$\r\n\r\n$=e\\cdot\\left(1-\\frac1{2x}+O(x^{-2})\\right)$\r\n\r\nSo, $x\\left(1+\\frac1x\\right)^x=ex-\\frac{e}2+O(x^{-1})$\r\n\r\nand $x\\left(1+\\frac1x\\right)^x-ex =-\\frac{e}2+O(x^{-1})$\r\n\r\nThe limit as $x\\to\\infty$ is $-\\frac{e}2.$\r\n\r\nIf the original problem had been $\\lim_{x\\to\\infty}x^2\\left(1+\\frac1x\\right)^x-ex^2+\\frac{ex}2$\r\n\r\nthen we would have had to extend our analysis by one more term, which would be quite a bit more work." } { "Tag": [ "vector", "geometry unsolved", "geometry" ], "Problem": "A,B,C are 3vectors,\r\nProve that $|A+B+C|+|A|+|B|+|C|>|A+B|+|A+C|+|B+C|$", "Solution_1": "This is $Hlawka$'s inequality." } { "Tag": [ "USAMTS" ], "Problem": "Ok here is the problem: There is a number that ends on 2 (the last digit is 2 and the number of digits is unknown)\r\nand if you place the last digit (2) in the begging of the number then the number doubles. Whats the number?\r\n\r\nI hope it makes sence, if it doesn't then let me know and I'll give you an example.", "Solution_1": "When you place the 2 at the beginning, do you take it away from the last digit?\r\nso it's like __________2 * 2 = 2__________? or 2________2?", "Solution_2": "Like if you had 12 it would become 21, does that asnwer your question?", "Solution_3": "Yes. lemme think for a minute.", "Solution_4": "I think you will be thinking for quite a few minutes, lol unless you just randomly pick a number ending on 2 and it turns out to be right, lol.", "Solution_5": "Ok got it.\r\n\r\n105263157894736842\r\nx2 =\r\n210526315789473684", "Solution_6": ":rotfl: Can't belive it, tell me your method please, master... lol", "Solution_7": "Ok, so since the second number has to start with 2.. and it's the first number multiplied by 2.. I just did long division. a number that starts with 2 divided by 2.. gets 1, and then so the second digit has to be 1.. and then the third 0, cause that comes out to 0 on top.. etc until you get the last digit to be 2.", "Solution_8": ":blush: thanks.", "Solution_9": "You know that the tens digits has to be 4. Then the hundreds is 8. So on, until you get a 1, which becomes a two when the number is doubled.", "Solution_10": "wow, that's very clever :D", "Solution_11": "Clever's my middle name. :lol: \r\n(no really it is.. just in a different language)\r\n\r\nand yeah, what k81o7 said was good.. I didn't think of that. well maybe i did but i didn't realize..", "Solution_12": "[quote=\"ambierona\"]Ok got it.\n\n105263157894736842\nx2 =\n210526315789473684[/quote]\r\n\r\nInteresting. If you take the original number move the 1 from the front to the back and place a decimal before the 0 and put a repetend sign over it (.052631578947368421 repeating) you get 1/19 in exact form.", "Solution_13": "You did do it the same way...you just went in the other direction. You went left to right and divided by 2 instead of multiplying by 2. So it's basically the same approach.", "Solution_14": "That's no accident Shogia. It's actually part of a more general method for solving problems like this.\r\n\r\nLet N be a repeating decimal number whose decimal expansion looks like $\\displaystyle N = .\\overline{a_1a_2a_3 \\ldots a_n2}$, and such that $\\displaystyle 2N = .2\\overline{a_1a_2a_3 \\ldots a_n2}$.\r\n\r\nIn other words, its a repeating decimal number such that multiplying the number by 2 moves the digit 2 from the last of the repeating part to the front of the repeating part.\r\n\r\nBut notice that $2N = .2 + \\frac{N}{10}$. So solving for N we get that $N = \\frac{2}{19}$. And sure enough if you look at the repeating part of the decimal expansion of 2/19 (using a computer because its too big to fit in a calculator) it is 105263157894736842.\r\n\r\nBonus Problem: See if you can apply this method to find the smallest positive integer such that when you move its last digit to the front of the number it equals the original number multiplied by 3. There was also a USAMTS problem similar to this on one of the rounds this year.", "Solution_15": "I'm actually quite aware of that. The only reason i noticed the 1/19 thing was because of the reading I have done on using Vedic mathematics (if anyone is familiar with it) to compute recurring decimals.", "Solution_16": "vedic mathematics?", "Solution_17": "It is a form of mathematics that is a bit different from the standard method. It makes some problems much easier to solve and the even the methods for multiplication and division are different. I would probably never even have heard of it if my friend did not have a book on it. I will try to find a site with it if you like.\r\n\r\nThe bit on recurring decimals allows you to calculate things like 1/19, 1/29, 1/23... in less than a minute with plenty of time to spare. One cool bit that I learned from it was that if you have 1/x you can determine what the last digit, or last digit in the repetend, is. If x ends in a 1 then it will 1/x will \"end\" in a 9, 2-->5, 3-->3, 5-->2, 7-->7 of course you can figure out the others since they arent prime." } { "Tag": [ "inequalities", "algebra", "polynomial", "Computer solutions", "4-variable inequality", "Symmetric inequality" ], "Problem": "Let $a,b,c,d >0$ and $abcd=1$. Prove that:\r\n\\[ \\frac{1}{(1+a)^2}+\\frac{1}{(1+b)^2}+\\frac{1}{(1+c)^2}+\\frac{1}{(1+d)^2} \\geq 1 \\]", "Solution_1": "shobber wasn't this problem already posted?since i can't find the link right now i will tell the main ideas of two solutions:one posted here(you can also find it in old and new inequalities since this is vasc's problem posted in \"gazeta matematica\")and one that i found.\r\none elegant idea would be to use the fact that $1/(a+1)^2 +1/(b+1)^2\\ge 1/(ab+1)$\r\nhowever if you are not inspired enough to see that you may also try to kill it with sturm and a little \"strategy\" since from sturm you get that $1/(a+1)^2 +1/(b+1)^2$is decreasing when $a-b$ decreases(when assuming that a>b)if $ab\\ge1$.so you always pick two numbers such that their product is bigger than one.if anybody wants to know more about this solution i will post it", "Solution_2": "yep. it has been posted before....\r\n\r\nhttp://www.artofproblemsolving.com/Forum/viewtopic.php?p=112435#p112435\r\nhttp://www.artofproblemsolving.com/Forum/viewtopic.php?p=190407#p190407", "Solution_3": "[quote=\"bodom\"]shobber wasn't this problem already posted?since i can't find the link right now i will tell the main ideas of two solutions:one posted here(you can also find it in old and new inequalities since this is vasc's problem posted in \"gazeta matematica\")and one that i found.\none elegant idea would be to use the fact that $1/(a+1)^2 +1/(b+1)^2\\ge 1/(ab+1)$\nhowever if you are not inspired enough to see that you may also try to kill it with sturm and a little \"strategy\" since from sturm you get that $1/(a+1)^2 +1/(b+1)^2$is decreasing when $a-b$ decreases(when assuming that a>b)if $ab\\ge1$.so you always pick two numbers such that their product is bigger than one.if anybody wants to know more about this solution i will post it[/quote]\r\n\r\nCan you post your solution bodom?", "Solution_4": "ok.i will do that in the evening because now i'm a little busy", "Solution_5": "[quote=\"shobber\"]Let $a,b,c,d >0$ and $abcd=1$. Prove that:\n\\[ \\frac{1}{(1+a)^2}+\\frac{1}{(1+b)^2}+\\frac{1}{(1+c)^2}+\\frac{1}{(1+d)^2} \\geq 1 \\][/quote]\r\n\r\n$\\frac{1}{(1+a)^2}+\\frac{1}{(1+b)^2}+\\frac{1}{(1+c)^2}+\\frac{1}{(1+d)^2} \\geq\\min\\{1,\\frac{2}{1+abcd}\\}$ \r\nfor all non-negative $a,$ $b,$ $c$ and $d.$\r\nIt is stronger. ;)", "Solution_6": "See \r\n[url]http://www.mathlinks.ro/Forum/viewtopic.php?t=98967[/url]\r\n[url]http://www.mathlinks.ro/Forum/viewtopic.php?t=66028[/url]\r\n[url]http://www.mathlinks.ro/Forum/viewtopic.php?t=83784[/url]\r\n[url]http://www.mathlinks.ro/Forum/viewtopic.php?t=32031[/url]\r\n[url]http://www.mathlinks.ro/Forum/viewtopic.php?t=30724[/url]\r\n[url]http://www.mathlinks.ro/Forum/viewtopic.php?t=16027[/url]", "Solution_7": "[quote=\"shobber\"]Let $ a,b,c,d > 0$ and $ abcd \\equal{} 1$. Prove that\n\n$ \\frac {1}{(1 \\plus{} a)^2} \\plus{} \\frac {1}{(1 \\plus{} b)^2} \\plus{} \\frac {1}{(1 \\plus{} c)^2} \\plus{} \\frac {1}{(1 \\plus{} d)^2} \\geq 1$.[/quote]Note $ f(r) \\equal{} \\frac {(1 \\plus{} r)^2}{(1 \\plus{} ra)^2} \\plus{} \\frac {(1 \\plus{} r)^2}{(1 \\plus{} rb)^2} \\plus{} \\frac {(1 \\plus{} r)^2}{(1 \\plus{} rc)^2} \\plus{} \\frac {(1 \\plus{} r)^2}{(1 \\plus{} rd)^2}$, the above iequality can be rewritten as $ f(1)\\geq f(0)$. \r\n\r\nFurther, the inequality $ f(p)\\leq f(q)$ holds if and only if $ \\left(0\\leq p\\leq q \\wedge q \\geq 1\\right)\\vee\\left(p\\leq q \\leq 1 \\wedge F(p,q)\\geq 0\\right)$, where \r\n\r\n$ F(p,q) \\equal{} 791297088768p^{41}q^{41}(p \\plus{} q)$\r\n\r\n$ \\plus{} 13824p^{40}q^{40}(427223039p^2 \\plus{} 968927717pq \\plus{} 427223039q^2)$\r\n\r\n$ \\plus{} 3456p^{39}q^{39}(p \\plus{} q)(6090377113p^2 \\plus{} 15189393892pq \\plus{} 6090377113q^2)$\r\n\r\n$ \\plus{} 3456p^{38}q^{38}(13722903675p^4 \\plus{} 64516122158p^3q \\plus{} 100768147674p^2q^2$\r\n\r\n$ \\plus{} 64516122158pq^3 \\plus{} 13722903675q^4)$\r\n\r\n$ \\plus{} 432p^{37}q^{37}(p \\plus{} q)(173688858761p^4 \\plus{} 838555981116p^3q \\plus{} 1344852329190p^2q^2$\r\n\r\n$ \\plus{} 838555981116pq^3 \\plus{} 173688858761q^4)$\r\n\r\n$ \\cdots$\r\n\r\n$ \\minus{} 162(135092061p^{16} \\minus{} 665629194p^{15}q \\plus{} 249716562p^{14}q^2 \\minus{} 2231356257p^{13}q^3$\r\n\r\n$ \\plus{} 6400603379p^{12}q^4 \\plus{} 15107734327p^{11}q^5 \\plus{} 42943235426p^{10}q^6 \\plus{} 23849200372p^9q^7$\r\n\r\n$ \\plus{} 71842248824p^8q^8 \\plus{} 23849200372p^7q^9 \\plus{} 42943235426p^6q^{10} \\plus{} 15107734327p^5q^{11}$\r\n\r\n$ \\plus{} 6400603379p^4q^{12} \\minus{} 2231356257p^3q^{13} \\plus{} 249716562p^2q^{14} \\minus{} 665629194pq^{15}$\r\n\r\n$ \\plus{} 135092061q^{16})$\r\n\r\n$ \\minus{} 209952(p \\plus{} q)^{12}$\r\n\r\nis an irreducible polynomial (degree = 83, 1770 expanding terms, 15 print pages). Specially, $ F(0,1) \\equal{} 0$;\r\n\r\n$ F\\left(\\frac {1}{4},\\frac {1}{3}\\right) \\equal{} \\frac {316365130337792711401324387061897099707046593}{32294785429008247205187499988123494907904}$;\r\n\r\n$ F\\left(\\frac {1}{5},\\frac {1}{3}\\right) \\equal{} \\minus{} \\frac {46735935121369674620811954200789126517628587200217088}{2764333080235003035568297491408884525299072265625}$;\r\n\r\n$ F\\left(\\frac {1}{7},\\frac {1}{2}\\right) \\equal{} \\frac {2219213864953238297522670210813327836751708984375}{20936185994239763297898479586227222883598336}$;\r\n\r\n$ F\\left(\\frac {1}{8},\\frac {1}{2}\\right) \\equal{} \\minus{} \\frac {14061357910842776796378414886758065820716927789808483}{730750818665451459101842416358141509827966271488}$.\r\n\r\nHence, $ f(r)$ rise monotonously on $ [s, \\plus{} \\infty)$, where $ s \\equal{} 0.28783\\cdots$ be a root of the following irreducible polynomial \r\n\r\n$ 114481639s^{16} \\plus{} 564075766s^{15} \\plus{} 1183750488s^{14} \\plus{} 1376656902s^{13} \\plus{} 742228566s^{12}$\r\n\r\n$ \\minus{} 999306606s^{11} \\minus{} 3588048396s^{10} \\minus{} 5291401734s^9 \\minus{} 4791695109s^8 \\minus{} 3224090940s^7$\r\n\r\n$ \\minus{} 1993194540s^6 \\minus{} 1007735328s^5 \\minus{} 187439320s^4 \\plus{} 105691796s^3 \\plus{} 43664832s^2$\r\n\r\n$ \\minus{} 3297024s \\plus{} 62208$.", "Solution_8": "The inequality is clearly equivalent to\n\n$\\frac16 \\sum_{sym}a^2b^2c^2+\\sum_{sym}a^2b^2c+\\frac12\\sum_{sym}a^2b^2+2\\sum_{sym}a^2bc+2\\sum_{sym}a^2b+\\frac43 \\sum_{sym}abc + \\frac12 \\sum_{sym}a^2+2\\sum_{sym}ab+\\sum_{sym}a+\\frac16\\sum_{sym}1 \\geq \n\\frac1{24}\\sum_{sym}a^2b^2c^2d^2+\\frac13\\sum_{sym}a^2b^2c^2d+\\frac16\\sum_{sym}a^2b^2c^2+\\sum_{sym}a^2b^2cd+\\sum_{sym}a^2b^2c+\\frac43\\sum_{sym}a^2bcd+\\frac14\\sum_{sym}a^2b^2+2\\sum_{sym}a^2bc+\\frac23\\sum_{sym}abcd+\\sum_{sym}a^2b+\\frac43\\sum_{sym}abc+\\frac16\\sum_{sym}a^2+\\sum_{sym}ab+\\frac13\\sum_{sym}a+\\frac1{24}\\sum_{sym}1$ by expansion.\n\nAfter homogenising to degree 8\n\n${\\frac16 \\sum_{sym}a^{5/2}b^{5/2}c^{5/2}d^{1/2}+\\sum_{sym}a^{11/4}b^{11/4}c^{7/4}d^{3/4}+\\frac12\\sum_{sym}a^3b^3cd+2\\sum_{sym}a^3b^2c^2d+2\\sum_{sym}a^{13/4}b^{9/4}c^{5/4}d^{5/4}+\\frac43 \\sum_{sym}a^{9/4}b^{9/4}c^{9/4}d^{5/4} + \\frac12 \\sum_{sym}a^{7/2}b^{3/2}c^{3/2}d^{3/2}+2\\sum_{sym}a^{5/2}b^{5/2}c^{3/2}d^{3/2}+\\sum_{sym}a^{11/4}b^{7/4}c^{7/4}d^{7/4}+\\frac16\\sum_{sym}a^2b^2c^2d^2 \\geq \n\\frac1{24}\\sum_{sym}a^2b^2c^2d^2+\\frac13\\sum_{sym}a^{9/4}b^{9/4}c^{9/4}d^{5/4}+\\frac16\\sum_{sym}a^{5/2}b^{5/2}c^{5/2}d^{1/2}+\\sum_{sym}a^{5/2}b^{5/2}c^{3/2}d^{3/2}+\\sum_{sym}a^{11/4}b^{11/4}c^{7/4}d^{3/4}+\\frac43\\sum_{sym}a^{11/4}b^{7/4}c^{7/4}d^{7/4}+\\frac14\\sum_{sym}a^3b^3cd+2\\sum_{sym}a^3b^2c^2d+\\frac23\\sum_{sym}a^2b^2c^2d^2+\\sum_{sym}a^{13/4}b^9/4}c^{5/4}d^{5/4}+\\frac43\\sum_{sym}a^{9/4}b^{9/4}c^{9/4}d^{5/4}+\\frac16\\sum_{sym}a^{7/2}b^{3/2}c^{3/2}d^{3/2}+\\sum_{sym}a^{5/2}b^{5/2}c^{3/2}d^{3/2}+\\frac13\\sum_{sym}a^{11/4}b^{7/4}c^{7/4}d^{7/4}+\\frac1{24}\\sum_{sym}a^2b^2c^2d^2$ which after simplification is conveniently equivalent to\n\n$\\frac14\\sum_{sym}a^3b^3cd+\\sum_{sym}a^{13/4}b^{9/4}c^{5/4}d^{1/4}+\\frac13\\sum_{sym}a^{7/2}b^{3/2}c^{3/2}d^{3/2} \\geq \\frac13 \\sum_{sym} a^{9/4}b^{9/4}c^{9/4}d^{5/4}+\\frac23 \\sum_{sym}a^{11/4}b^{7/4}c^{7/4}d^{7/4}+\\frac7{12}\\sum_{sym}a^2b^2c^2d^2$\n\nBut this is true since by Muirhead's inequality or weighted AM-GM we have\n$\\frac14\\sum_{sym}a^3b^3cd \\geq \\frac14\\sum_{sym}a^{9/4}b^{9/4}c^{9/4}d^{5/4}$\n$\\frac34\\sum_{sym}a^{13/4}b^{9/4}c^{5/4}d^{5/4} \\geq \\frac{1}{12}\\sum_{sym}a^{9/4}b^{9/4}c^{9/4}d^{5/4}+\\frac23 \\sum_{sym}a^{11/4}b^{7/4}c^{7/4}d^{7/4}$\n$\\frac13\\sum_{sym}a^{7/2}b^{3/2}c^{3/2}d^{3/2}+\\frac14 \\sum_{sym}a^{13/4}b^{9/4}c^{5/4}d^{5/4} \\geq \\frac{7}{12}\\sum_{sym}a^2b^2c^2d^2$.", "Solution_9": "These bashes are really ugly! :(", "Solution_10": "For prove the problem , we will prove that :\n\n\\[ \\frac {1}{(1+a)^2} + \\frac {1}{(1+b)^2} \\ge \\frac {1}{1+ab} \\]\n\\[ \\frac {1}{(1+c)^2} + \\frac {1}{(1+d)^2} \\ge \\frac {1}{1+cd} \\]\n\nProof :\n\n\\[ \\frac {1}{(1+a)^2} + \\frac {1}{(1+b)^2} \\ge \\frac {1}{1+ab} \\]\n\\[ \\Longleftrightarrow (1+ab)((1+a)^2+(1+b)^2) \\ge (1+a)^2(1+b)^2 \\]\n\\[ \\Longleftrightarrow a^3b+b^3a-a^2b^2-2ab+1 \\ge 0 \\]\n\\[ \\Longleftrightarrow 1+ab(a^2+b^2-ab-2) \\ge 0 \\]\n\nUse AM-GM on $LHS$ we have\n\n\\[ \\text {LHS} \\ge 1+ab(2ab-ab-2) = 1+ab(ab-2)\\1+a^2b^2-2ab \\ge 2ab - 2ab = 0 \\]\n\nSimilarly we can prove second one. Now use $abcd=1$. So we have :\n\n\\[ \\frac{1}{(1+a)^2}+\\frac{1}{(1+b)^2}+\\frac{1}{(1+c)^2}+\\frac{1}{(1+d)^2} \\ge \\frac {1}{ab+1} + \\frac {1}{cd+1} \\]\n\\[ = \\frac {cd+ab+2}{cd+ab+abcd+1} = \\frac {cd+ab+2}{cd+ab+2} = 1 \\]", "Solution_11": "The following inequality is also true.\nLet $a,b,c >0$ and $abc=1$. Prove that:\n\\[ \\frac{1}{(1+a)^2}+\\frac{1}{(1+b)^2}+\\frac{1}{(1+c)^2}\\geq\\frac{3}{4}.\\]", "Solution_12": "Just take $d=1$ in the general case: $\\frac{1}{(1+a)^2}+\\frac{1}{(1+b)^2}+\\frac{1}{(1+c)^2}+\\frac{1}{(1+d)^2}\\geq 1 $\nYou get \\[ \\frac{1}{(1+a)^2}+\\frac{1}{(1+b)^2}+\\frac{1}{(1+c)^2}+\\frac{1}{4}\\geq 1 \\]\nSo: \\[ \\frac{1}{(1+a)^2}+\\frac{1}{(1+b)^2}+\\frac{1}{(1+c)^2}\\geq \\frac{3}{4} \\]", "Solution_13": "Let a,b,c> 0 abc=1\nUsing the previous lemma $\\frac{1}{(1+a)^2}+\\frac{1}{(1+b)^2}\\geq \\frac{1}{1+ab}=\\frac{c}{c+1}$\nWe only need to prove that : $\\frac{1}{(1+c)^2} + \\frac{c}{c+1} \\geq\\frac{3}{4}$\nwhich's equivalente after expanding to $( c-1)^2\\geq0$", "Solution_14": "Thanks.\nLet $x_1,x_2,\\cdots,x_n>0$ and $x_1x_2\\cdots x_n=1$. Prove that:\n\\[ \\frac{1}{(1+x_1)^2}+\\frac{1}{(1+x_2)^2}+\\cdots+\\frac{1}{(1+x_n)^2} \\geq \\frac{n}{4}. \\]", "Solution_15": "see here http://www.artofproblemsolving.com/Forum/viewtopic.php?f=51&t=542038&p=3124726#p3124726", "Solution_16": "[quote=\"leminscate\"]These bashes are really ugly! :([/quote]\n\nNo, they're not. :lol:", "Solution_17": "[quote=\"sqing\"]The following inequality is also true.\nLet $a,b,c >0$ and $abc=1$. Prove that:\n\\[ \\frac{1}{(1+a)^2}+\\frac{1}{(1+b)^2}+\\frac{1}{(1+c)^2}\\geq\\frac{3}{4}.\\][/quote]\nLet $a=\\frac{y}{x},b=\\frac{z}{y},c= \\frac{x}{z},$ we have (\u300aMathematical communication\u300b(China Wuhan) N0.7\u30018(2014) Q183 ):\nLet $x,y,z$ be positive real numbers . Prove that \n\\[ \\frac{x^2}{(x+y)^2}+\\frac{y^2}{(y+z)^2}+\\frac{z^2}{(z+x)^2}\\geq\\frac{3}{4}.\\]", "Solution_18": "I think from the first we must use CS inequality, then Muirhead's inequality.", "Solution_19": "The following inequalities are also true.\nLet $a,b,c >0$ and $abc=1$. Prove that: \\[\\frac{1}{(a+1)^2} + \\frac{1}{(b+1)^2} + \\frac{1}{(c+1)^2} + \\frac{1}{{a + b + c +1}} \\geqslant 1.\\]\nLet $a,b,c,d >0$ and $abcd=1$. Prove that:\n\\[ \\frac{1}{(3a-1)^2}+\\frac{1}{(3b-1)^2}+\\frac{1}{(3c-1)^2}+\\frac{1}{(3d-1)^2} \\geq 1. \\]\n[url=http://www.artofproblemsolving.com/Forum/viewtopic.php?t=16027]here[/url]", "Solution_20": "we can prove:\n$ f(a,b,c,d)\\geq f(\\sqrt{ab},\\sqrt{ab},c,d)\\geq 0 $ with $ f(a,b,c,d)=\\sum (\\frac{1}{(1+a)^{2}}-\\frac{1}{4}) $", "Solution_21": "[quote=\"sqing\"]The following inequality is also true.\nLet $a,b,c,d >0$ and $abcd=1$. Prove that:\n\\[ \\frac{1}{(3a-1)^2}+\\frac{1}{(3b-1)^2}+\\frac{1}{(3c-1)^2}+\\frac{1}{(3d-1)^2} \\geq 1. \\]\n[/quote]\nIt's Jensen! :D", "Solution_22": "[quote=War-Hammer]\nProof :\n\n\\[ \\frac {1}{(1+a)^2} + \\frac {1}{(1+b)^2} \\ge \\frac {1}{1+ab} \\]\n\\[ \\Longleftrightarrow (1+ab)((1+a)^2+(1+b)^2) \\ge (1+a)^2(1+b)^2 \\]\n\\[ \\Longleftrightarrow a^3b+b^3a-a^2b^2-2ab+1 \\ge 0 \\]\n\\[ \\Longleftrightarrow 1+ab(a^2+b^2-ab-2) \\ge 0 \\]\n[/quote]\nwhich is $ab(a-b)^2+(ab-1)^2\\geq0$.\n\n", "Solution_23": "generalization to 6 variables:\nlet $x_1,x_2,\\dots ,x_6$ be 6 positive numbers such that $x_1x_2\\dots x_6=1$ and $\\prod_{1\\le i0$ and $abc=1$. Prove that:\n\\[ \\frac{1}{(1+a)^2}+\\frac{1}{(1+b)^2}+\\frac{1}{(1+c)^2}\\geq\\frac{3}{4}.\\][/quote]\nLet $a=\\frac{y}{x},b=\\frac{z}{y},c= \\frac{x}{z},$ we have (\u300aMathematical communication\u300b(China Wuhan) N0.7\u30018(2014) Q183 ):\nLet $x,y,z$ be positive real numbers . Prove that \n\\[ \\frac{x^2}{(x+y)^2}+\\frac{y^2}{(y+z)^2}+\\frac{z^2}{(z+x)^2}\\geq\\frac{3}{4}.\\][/quote]\n\nJust is my LBQ105.\nBQ", "Solution_28": "[quote=sqing]The following inequality is also true.\nLet $a,b,c >0$ and $abc=1$. Prove that:\n\\[ \\frac{1}{(1+a)^2}+\\frac{1}{(1+b)^2}+\\frac{1}{(1+c)^2}\\geq\\frac{3}{4}.\\][/quote]\n\nAlso substituting $ a=xy/z^2$ and likewise does the trick.", "Solution_29": "[quote=War-Hammer]For prove the problem , we will prove that :\n\n\\[ \\frac {1}{(1+a)^2} + \\frac {1}{(1+b)^2} \\ge \\frac {1}{1+ab} \\]\n\\[ \\frac {1}{(1+c)^2} + \\frac {1}{(1+d)^2} \\ge \\frac {1}{1+cd} \\]\n\nProof :\n\n\\[ \\frac {1}{(1+a)^2} + \\frac {1}{(1+b)^2} \\ge \\frac {1}{1+ab} \\]\n\\[ \\Longleftrightarrow (1+ab)((1+a)^2+(1+b)^2) \\ge (1+a)^2(1+b)^2 \\]\n\\[ \\Longleftrightarrow a^3b+b^3a-a^2b^2-2ab+1 \\ge 0 \\]\n\\[ \\Longleftrightarrow 1+ab(a^2+b^2-ab-2) \\ge 0 \\]\n\nUse AM-GM on $LHS$ we have\n\n\\[ \\text {LHS} \\ge 1+ab(2ab-ab-2) = 1+ab(ab-2)\\1+a^2b^2-2ab \\ge 2ab - 2ab = 0 \\]\n\nSimilarly we can prove second one. Now use $abcd=1$. So we have :\n\n\\[ \\frac{1}{(1+a)^2}+\\frac{1}{(1+b)^2}+\\frac{1}{(1+c)^2}+\\frac{1}{(1+d)^2} \\ge \\frac {1}{ab+1} + \\frac {1}{cd+1} \\]\n\\[ = \\frac {cd+ab+2}{cd+ab+abcd+1} = \\frac {cd+ab+2}{cd+ab+2} = 1 \\][/quote]\n\nthank you for Lemma", "Solution_30": "Let $a,b,c,d$ are positive reals such that $abcd = 1 .$ [url=https://artofproblemsolving.com/community/c6h1825235p13223004]Prove that[/url]$$\\frac{1}{5a^2-2a+1} +\\frac{1}{5b^2-2b+1} + \\frac{1}{5c^2-2c+1} + \\frac{1}{5d^2-2d+1} \\geq \\frac{1}{(a^2+1)^2} +\\frac{1}{(b^2+1)^2} +\\frac{1}{(c^2+1)^2} +\\frac{1}{(d^2+1)^2} \\geq 1.$$\n", "Solution_31": "[quote=shobber]Let $a,b,c,d >0$ and $abcd=1$. Prove that:\n\\[ \\frac{1}{(1+a)^2}+\\frac{1}{(1+b)^2}+\\frac{1}{(1+c)^2}+\\frac{1}{(1+d)^2} \\geq 1 \\][/quote]\n[url=https://artofproblemsolving.com/community/c6h1392755p7773884]here[/url] [url=https://artofproblemsolving.com/community/c6h1387835p7725948]here[/url] [url=http://www.360doc.com/document/13/0712/11/10693281_299362805.shtml]here:[/url]", "Solution_32": "Let $a,b,c >0$ and $abc=1$. Prove that\n$$\\dfrac{1}{(3a^2+1)^2}+\\dfrac{1}{(3b^2+1)^2}+\\dfrac{1}{(3c^2+1)^2}+\\dfrac{1}{(a+b+c+1)^2}\\geq\\dfrac{1}{4}$$\n[quote=shobber]Let $a,b,c,d >0$ and $abcd=1$. Prove that:\n\\[ \\frac{1}{(1+a)^2}+\\frac{1}{(1+b)^2}+\\frac{1}{(1+c)^2}+\\frac{1}{(1+d)^2} \\geq 1 \\][/quote]\n[url=https://om.mimuw.edu.pl/static/app_main/camps/oboz2018.pdf]Poland 2018, MO Science Camp[/url]\n[url=https://artofproblemsolving.com/community/c6h383409p2126599]h[/url]\n [url=https://artofproblemsolving.com/community/c6h2200804p16594994]Romanian magazine Gazeta Matematica, 1999, No. 11[/url]", "Solution_33": "[quote=sqing]The following inequality is also true.\nLet $a,b,c >0$ and $abc=1$. Prove that:\n\\[ \\frac{1}{(1+a)^2}+\\frac{1}{(1+b)^2}+\\frac{1}{(1+c)^2}\\geq\\frac{3}{4}.\\][/quote]\nLet $a,b,c\\in(0,2)$ and $abc=(1-a)(2-b)(3-c).$ Prove that$$\\dfrac{a^2}{3}+\\dfrac{b^2}{12}+\\dfrac{c^2}{27}\\geq\\frac{3}{4}.$$\n", "Solution_34": "Let $a,b,c,d >0$ and $abcd=1$. Prove that:\n\\[ \\frac{1}{(1+a)^2}+\\frac{1}{(1+b)^2}+\\frac{1}{(1+c)^2}+\\frac{1}{(1+d)^2}<3 \\]\n", "Solution_35": "[quote=sqing]Let $a,b,c,d >0$ and $abcd=1$. Prove that:\n\\[ \\frac{1}{(1+a)^2}+\\frac{1}{(1+b)^2}+\\frac{1}{(1+c)^2}+\\frac{1}{(1+d)^2}<3 \\][/quote]\nBecause $$\\frac{1}{(1+a)^2}+\\frac{1}{(1+b)^2}+\\frac{1}{(1+c)^2}+\\frac{1}{(1+d)^2}\\geq1.$$ :-D \n\n", "Solution_36": "[quote=sqing]\nLet $a,b,c >0$ and $abc=1$. Prove that: \\[\\frac{1}{(a+1)^2} + \\frac{1}{(b+1)^2} + \\frac{1}{(c+1)^2} + \\frac{1}{{a + b + c +1}} \\geqslant 1.\\][/quote]\n\n", "Solution_37": "[quote=arqady][quote=\"shobber\"]Let $a,b,c,d >0$ and $abcd=1$. Prove that:\n\\[ \\frac{1}{(1+a)^2}+\\frac{1}{(1+b)^2}+\\frac{1}{(1+c)^2}+\\frac{1}{(1+d)^2} \\geq 1 \\][/quote]\n\n$\\frac{1}{(1+a)^2}+\\frac{1}{(1+b)^2}+\\frac{1}{(1+c)^2}+\\frac{1}{(1+d)^2} \\geq\\min\\{1,\\frac{2}{1+abcd}\\}$ \nfor all non-negative $a,$ $b,$ $c$ and $d.$\nIt is stronger. ;)[/quote]\n\nLet us not forget this math problem", "Solution_38": "This problem is a special case of this inequality:\nLet $abcd=1$ as above. Then\n$\\frac{1}{(1+a)^k}+\\frac{1}{(1+b)^k}+\\frac{1}{(1+c)^k}+\\frac{1}{(1+d)^k} \\geq 2^{2-k}$.", "Solution_39": "Very interesting", "Solution_40": "[quote=sqing]Thanks.\nLet $x_1,x_2,\\cdots,x_n>0$ and $x_1x_2\\cdots x_n=1$. Prove that:\n\\[ \\frac{1}{(1+x_1)^2}+\\frac{1}{(1+x_2)^2}+\\cdots+\\frac{1}{(1+x_n)^2} \\geq \\frac{n}{4}. \\][/quote]\n\nThis is wrong for $n\\geq 5$. Try $x_1=...=x_{n-1}\\to +\\infty$." } { "Tag": [], "Problem": "$ (\\frac{a}{b})^ 3 \\plus{} (\\frac{c}{d})^ 3 \\equal{} 6$\r\n\r\n$ a, b, c, d$ are positive integers. a couple of them may or may not be equal.\r\n \r\nfind $ a, b, c, d$.", "Solution_1": "I think it can be proved with infinite descent, we have $ m^3\\plus{}n^3\\equal{}6p^3$, by mod 6 either m and n are both multiples of 3, thus p is too and it divides out?", "Solution_2": "dgreenb801, $ m$ and $ n$ do not need to be multiples of $ 3$. All residue in mod 6 are cubic residue.\r\n\r\nBut $ (ad)^3 \\plus{} (bc)^3 \\equal{} 6(bd)^3$ is a good start...\r\n\r\n$ (ad)^3 \\equal{} 6(bd)^3 \\minus{} (bc)^3$, so $ b|ad$. Similarly, $ d|bc$. Let $ bk\\equal{}ad$, $ dj\\equal{}bc$, then $ bkj\\equal{}adj\\equal{}abc \\implies kj\\equal{}ac$\r\n\r\nI'm a bit stuck here", "Solution_3": "[hide=\"Spoiler\"][url]http://www.math.niu.edu/~rusin/known-math/95/cubic.prize[/url][/hide]" } { "Tag": [ "geometry", "3D geometry", "MATHCOUNTS", "floor function", "AMC", "AIME" ], "Problem": "The increasing sequence $2,3,5,6,7,10,11,\\ldots$ consists of all positive integers that are neither the square nor the cube of a positive integer. Find the 500th term of this sequence.", "Solution_1": "[hide]Because there aren't that many perfect squares or cubes, let's look for the smallest perfect square greater than $500$. This happens to be $23^2=529$. Notice that there are $23$ squares and $8$ cubes less than or equal to $529$, but $1$ and $2^6$ are both squares and cubes. Thus, there are $529-23-8+2=500$ numbers in our sequence less than $529$. Magically, we want the 500th term, so our answer is the smallest non-square and non-cube less than $529$, which is $\\boxed{528}$.[/hide]", "Solution_2": "This was similar to a mathcounts question. \r\n\r\n[hide]500 is the what number term? After taking the 22 perfect squares and 7 perfect cubes away, and then adding the 2 perfect 6th powers that we eliminated twice, we get the 473rd. To get to the 500th term, we add 27 to 500 to get 527. However, we also have to skip over 512 in the process, so the actual 500th term is 528. [/hide]", "Solution_3": "[quote=\"236factorial\"]then adding the 2 perfect 4th powers that we eliminated twice[/quote]You mean 6th powers, right? :)", "Solution_4": "[hide=\"Answer\"]We have $23$ squares up to $575$, and we will not need to go beyond that. In that same space, we have $8$ cubes. However, we also have two sixth-powers which overlap: $1^2=1^3=1^6=1$ and $8^2=4^3=2^6=64$, so we subtract those. Therefore, we have $29$ terms which are not used. However, this sequence has the $29^{th}$ of those terms as $529$, so we actually want the number before, which is $\\boxed{528}$.[/hide]", "Solution_5": "[quote=\"4everwise\"][quote=\"236factorial\"]then adding the 2 perfect 4th powers that we eliminated twice[/quote]You mean 6th powers, right? :)[/quote]\r\n\r\nOops, only had 6 hours of sleep the previous night :blush:", "Solution_6": "We seek the number n such that\r\n$ n \\minus{} \\lfloor \\sqrt {n} \\rfloor \\minus{} \\lfloor \\sqrt [3] {n} \\rfloor \\plus{} \\lfloor \\sqrt [6] {n} \\rfloor \\equal{} 500$, in which case $ n \\equal{} 528$, which we find by trial and error.", "Solution_7": "[quote=\"dgreenb801\"]We seek the number n such that\n$ n \\minus{} \\lfloor \\sqrt {n} \\rfloor \\minus{} \\lfloor \\sqrt [3] {n} \\rfloor \\plus{} \\lfloor \\sqrt [6] {n} \\rfloor \\equal{} 500$, in which case $ n \\equal{} 528$, which we find by trial and error.[/quote]\r\n\r\nThis is an AIME question.. I don't think it would be a good idea nor is it a good use of time..", "Solution_8": "It's not nearly as bad as it looks, you see that there will be 22-23 squares less than it, 8 cubes less than it, and 2 6th powers less after only trying out one number, there's not that much to check.", "Solution_9": "[hide=Solution]We want the number satisfying $n - \\lfloor \\sqrt{n} \\rfloor - \\lfloor \\sqrt [3]{n} \\rfloor + \\lfloor \\sqrt [6]{n} \\rfloor = 500$. Testing $500$ shows it's too small, so we try $529$, it being a perfect square. This works, but it can't be in the sequence, so the answer is one below, or $\\boxed{528}$.[/hide]", "Solution_10": "this post is from over 11 years ago, i do not see why you are posting in a post that was from 11 years ago", "Solution_11": "[quote=xingzhe]this post is from over 11 years ago, i do not see why you are posting in a post that was from 11 years ago[/quote]\n\nI believe its an AIME problem, so it is allowed", "Solution_12": "$n - \\lfloor \\sqrt{n} \\rfloor - \\lfloor \\sqrt [3]{n} \\rfloor + \\lfloor \\sqrt [6]{n} \\rfloor = 500$, testing $500$ is too small, more testing gives us $n=\\boxed{528}.$" } { "Tag": [ "calculus", "function", "inequalities" ], "Problem": "Hi All,\r\n\r\nI have a question about a question that was given out in our multivariable class.\r\n\r\n\"Find the maximum value of 4xy(10-x-y) over positive numbers x and y.\"\r\n\r\nWe are supposed to do it 2 ways, one with calculus, and the other without.\r\n\r\nI was able to do it with calculus pretty easily by finding critical pts. by setting the partials to zero, and found f(5/2,5)=625 to be the maximum value of the function for positive x and y.\r\n\r\n\r\nHowever, I am stuck on how to approach this problem without calculus. My guess is that I have to somehow apply AM-GM to the expression, but b/c I am trying to find the maximum, I need the arithmetic side to become constant..\r\n\r\nAlso, since x and y are positive, out of the three terms in the expression right now, the second and third terms will both be negative, so is AM-GM even an option here??? :maybe:", "Solution_1": "Actually, if $ f(x,y) \\equal{} 4xy(10 \\minus{} x \\minus{} y)$, then $ f(5/2,5) \\equal{} 125$, and that is not the maximum value since $ f(10/3,10/3) \\equal{} 4000/27$\r\n\r\n\r\n[hide=\"AM-GM hint\"]\nApply AM-GM to $ x$, $ y$, and $ 10 \\minus{} x \\minus{} y$.[/hide]\n\n\n[hide=\"solution with AM-GM\"]\nSince $ x$ and $ y$ are positive, $ (10 \\minus{} x \\minus{} y)$ must also be positive in order for the function to reach a maximum.\n\nBy AM-GM, $ \\frac {x \\plus{} y \\plus{} (10 \\minus{} x \\minus{} y)}{3}\\ge\\sqrt [3]{xy(10 \\minus{} x \\minus{} y)}$\nnotice how the inequality is set up so that the x and the y cancel out on the left side.\n\nSome final manipulations:\n\n$ \\frac {10}{3}\\ge\\sqrt [3]{xy(10 \\minus{} x \\minus{} y)}$\n\n$ \\frac {1000}{27}\\ge xy(10 \\minus{} x \\minus{} y)$\n\n$ \\frac {4000}{27}\\ge 4xy(10 \\minus{} x \\minus{} y)$\n\nas desired.\n\nThis inequality reaches equality when $ x \\equal{} y \\equal{} 10\\minus{}x\\minus{}y$ or $ x \\equal{} y \\equal{} 10/3$.[/hide]", "Solution_2": "Clearly 10 - x - y must be positive or else we'll get a negative answer, which is less than a positive one.\r\n\r\nBy AM-GM,\r\n\\[ \\frac{x \\plus{} y \\plus{} (10 \\minus{} x \\minus{} y)}{3} \\ge (xy(10\\minus{}x\\minus{}y))^\\frac{1}{3}\\]\r\n\\[ \\therefore 4 \\left(\\frac{10}{3} \\right)^3 \\ge 4xy(10\\minus{}x\\minus{}y)\\]", "Solution_3": "Symmetrize. Let $ z\\equal{}10\\minus{}x\\minus{}y$; now you want to maximize $ 4xyz$ under the constraint that $ x\\plus{}y\\plus{}z\\equal{}10$, and at most one of the three is negative. (If two are negative, the product can be made arbitrarily large)\r\nWe can constrain further to $ x,y,z\\ge 0$, since having one negative obviously won't generate the maximum.\r\n\r\nYou might also want to see how Lagrange multipliers work for the symmetrized version.", "Solution_4": "O shoot, I was looking at your solutions and wondering why it didn't match mine when I realized that I had typed the original function wrong.\r\n\r\nit should be 4xy^2(10-x-y), which with calculus, should give you a mx of 625 at (5/2,5).\r\n\r\nI was able to verify it with AM-GM by splitting it up into 4 terms, 2x, y, y, and 2(20-x-y), and I also reached an answer of 625." } { "Tag": [ "geometry", "minimization", "triangle inequality", "Triangle", "optimization", "IMO", "IMO 1973" ], "Problem": "A soldier needs to check if there are any mines in the interior or on the sides of an equilateral triangle $ABC.$ His detector can detect a mine at a maximum distance equal to half the height of the triangle. The soldier leaves from one of the vertices of the triangle. Which is the minimum distance that he needs to traverse so that at the end of it he is sure that he completed successfully his mission?", "Solution_1": "Let $ABC$ the triangle, with side $a$. Let $h=a\\frac{\\sqrt{3}}{4}$ the half altitude\r\n\r\nThe soldier starts moving from $A$.\r\n\r\nLet $D,E,F$ the midpoints of $AB,BC,CA$ respectively.\r\n\r\nIf he moves on the segment $DE$ (after $A\\to D$), then he can reach the most points of the area but he will lose the vertice $C$.\r\n\r\nThe problem is of course at the vertices.\r\n\r\nWe construct the triangle $DEF$ and let $K,L$ the midpoints of $DE,EF$.\r\n\r\nFrom $K$, he can reach the point $B$ since $BK=h$. \r\nAlso, from $L$ he can reach the point $C$.\r\n\r\n\r\nIf he could start from $K$, then the better way would be $K\\to L\\to M$ where $M$ is the midpoint of $DF$.\r\nActually, each of these points covers a large percentage of the triangle, for example the circle $(K,h)$ includes the triangle $BDE,DEF$ and much more area.\r\n\r\nBy the way, if could \"jump\" from $K$ to $L$ and then to $M$, the area would be covered perfectly. :rotfl: \r\n\r\n\r\n\r\nI think that, since he must start from $A$, the most convinience way is to get first to the point $K$ and then go to the point $L$\r\n\r\nThe vertical distance from $A$ to $K$ is $2h-\\frac{h}{2}=\\frac{3h}{2}$\r\n\r\nThe horizontal distance is $\\frac{a}{8}$\r\n\r\n$AK^2 = (\\frac{3h}{2})^2+(\\frac{a}{8})^2 =$\r\n\r\n$(\\frac{3a\\sqrt{3}}{8})^2 + (\\frac{a}{8})^2 =$\r\n\r\n$= a^2 \\frac{9 \\cdot 3 +1}{8^2}=$\r\n\r\n$= a^2 \\frac{7}{16}\\Rightarrow$\r\n\r\n\r\n$AK = a\\frac{\\sqrt{7}}{4}$ and $KL = \\frac{a}{4}$\r\n\r\n$AK+KL = a\\frac{\\sqrt{7}+1}{4}$\r\n\r\n\r\n\r\n\r\nI don't know if this is the minimum or if this is the better way to face the problem. At least I tried :( \r\n\r\nMaybe it should be easier using calculus?", "Solution_2": "This solution is wrong!\r\nCheck http://www.mat.itu.edu.tr/gungor/IMO/www.kalva.demon.co.uk/imo/isoln/isoln734.html\r\nThe answer is $ a\\frac {2\\sqrt {7} \\minus{} \\sqrt {3}}{4}$ which is less then $ a\\frac {\\sqrt {7} \\plus{} 1}{4}$", "Solution_3": "Geometric Problems on MAXIMA and MINIMA (by Author \"Titu Andreescu\")\r\n\r\n\"Let h be the length of the altitude of the given equilateral triangle ABC. Assume that the soldier's path starts at the point A. Consider the circles k1 and k2 with centers B and C, respectively, both with radius h/2. In order to check the points B and C, the soldier's path must have common points with both k1 and k2. Assume that the total length of the path is t and it has a common point M with k2 first and then a common point N with k1. Denote by D the common point of k2 and the altitude through C in triangle ABC and by L the line through D parallel to AB. Adding the constant h/2 to t and using the triangle inequality, one gets t+(h/2)\u2265AM+MN+NB=AM+MP+PN+NB\u2265AP+PB, where P is the intersection point MN and L. On the other hand, Heron's problem shows that AP+PB\u2265AD+DB, where equality occurs precisely when P=D. This implies t+(h/2)\u2265AD+DB,\r\ni.e. , t\u2265AD+DE, where E is the point of intersection of DB and k1.\"\r\n\r\n\u21d2shortest path length=a\u00d7{2\u00d7(\u221a7/4) - (\u221a3/4)}, where a is the length of equilateral triangle ABC.", "Solution_4": "Let d be the distance from any of the vetices to the mid point of the triangle's altitude. Let h be the altitude of the equilateral triangle. In order to do his task, the soldier must travel the distance d plus the diference (d - h/2). Hence:\n\n displacement = d + d - h/2 ~ 0.8899 units of length,\n\n as 77ant suggested", "Solution_5": "Let h be the altitude of triangle ABC and t be the radius of the detector, $r=\\frac{h}{2}$\n\nLet the soldier start from A, so the critical points to detect are B and C --> the paths to be taken are $A\\to D$ and $D\\to E$, where D is a point with the distance $r=\\frac{h}{2}$ from B and E is a point with the distance $r=\\frac{h}{2}$ from C.\n\nThe length of the paths is $s=\\|A\\to D\\|+\\|D\\to E\\|$\n\nLet's imagine the soldier is already on D (although we don't know where). To take the shortest path from D to E, the soldier must walk in the direction to C \n--> E is on the line DC. So $\\|D\\to E\\|=\\|D\\to C\\| - \\|E\\to C\\|=\\|D\\to C\\| - \\frac{h}{2}$\n\n--> $s=\\|A\\to D\\|+\\|D\\to C\\|-r$\n\ns is minimal, if and only if $\\|A\\to D\\|+\\|D\\to C\\|$ is minimal. $\\|A\\to D\\|+\\|D\\to C\\|$ is minimal, if and only if $\\|A\\to D\\|=\\|D\\to C\\|$. \n--> D must be on the line from B to F, where F is the midpoint of line AC.\n\nBecause BF is an altitude of triangle ABC, so D must be the midpoint of line BF.\n\nWe can calculate s as a function of h, but it's not the answer from the question. The correct answer is:\n[i]The soldier must walk from A to D and then to E. D is the midpoint of line BF (the altitude of the triangle ABC from B). E is on line DC with $EC=\\frac{h}{2}$[/i]" } { "Tag": [ "geometry", "geometric transformation", "reflection", "Support", "ratio" ], "Problem": "http://en.wikinews.org/wiki/North_Korea_claims_it_has_conducted_a_nuclear_test\r\n\r\nSo, I'm wondering if China or Russia are going to do anything about it, or if everyone's just going to sit back and go \"ho-hum\". The US is sure way too unpopular and streached out to do anything. Early results look like a relatively low-magnitude bomb, and I don't think the North Koreans are planning on doing much with it at the moment. If they were to say, smuggle a nuke into Tokyo and set it off, they'd be able to take out a few thousand South Koreans before we bombed them back to the stone age. CNN projects that a war will have about 100,000 casualties.", "Solution_1": "[quote=\"PenguinIntegral\"]http://en.wikinews.org/wiki/North_Korea_claims_it_has_conducted_a_nuclear_test\n\nSo, I'm wondering if China or Russia are going to do anything about it, or if everyone's just going to sit back and go \"ho-hum\". The US is sure way too unpopular and streached out to do anything. Early results look like a relatively low-magnitude bomb, and I don't think the North Koreans are planning on doing much with it at the moment. If they were to say, smuggle a nuke into Tokyo and set it off, they'd be able to take out a few thousand South Koreans before we bombed them back to the stone age. CNN projects that a war will have about 100,000 casualties.[/quote]\r\nok fine be racist and make a \"koreans suck/are violent/are stupid\" topic :mad:", "Solution_2": "\"ho hum\" is what they'll do because \"ho hum\" is pretty much all the UN and any of its councils do... in the short term, anyway. \r\n\r\nbut don't worry, time is what we have lots of. i'm calling the nuclear test a bluff, or at least heavily exaggerated in terms of its success - SK has to brag and stand tall, cuz when you're small, that's what gets you attention. \r\n\r\nat any rate, it's irrational to think that nuclear attack is imminent in light of this development, since SK is neither irrational or suicidal. the biggest threat i can think of right now is the sale of nuclear technology to fanaticist organizations - which is still very far-fetched, since A) it'll have to be the sale of nuclear weapons, since having the technology won't let just any small group to build nuclear devices like they do cell-phone-rigged-bombs, and B) SK is not suicidal, and wont do anything that will have the slightest chance of getting blamed on them, causing massive retaliation - remember, a war w/ 100,000 causualties may be unwanted from our side, but it'd be downright devastating on theirs.\r\n\r\nand that concludes my first post here since like April :D", "Solution_3": "Why should we do anything but \"ho hum\" ? They have the right to make nukes, it's not our job to tell them what they can and can't make. We're not the rulers of them.", "Solution_4": "they have the right to make nukes as much as your neighbor down the street has the right to buy a pistol and a target with your picture on it.\r\n\r\np.s.: that profile pic is the weirdest game of Go i've ever seen... is that even a real endgame?", "Solution_5": "[quote=\"L_Li\"]they have the right to make nukes as much as your neighbor down the street has the right to buy a pistol and a target with your picture on it.\n\np.s.: that profile pic is the weirdest game of Go i've ever seen... is that even a real endgame?[/quote]\r\n\r\nYes, my neighbor definitely has that right to bear arms as long as he wasn't just released from prison for having illegal arms or something.\r\n\r\nAlso, your example is bad, my neighbor is under the jursidiction of the US and its laws, however, we don't control North Korea, they are their own country with their own soverignty.", "Solution_6": "[quote=\"Ignite168\"]\nYes, my neighbor definitely has that right to bear arms as long as he wasn't just released from prison for having illegal arms or something.[/quote]\n\nNo, but he never really got over that firefight he had w/ your friend and other neighbor a few years ago...\n\n[quote=\"Ignite168\"]Also, your example is bad, my neighbor is under the jursidiction of the US and its laws, however, we don't control North Korea, they are their own country with their own soverignty.[/quote]\r\n\r\nyour neighbor = NK\r\nyou = US\r\n\r\nNK not under US jurisdiction, not violating international law (which debatably even exist)\r\nneighbor not under your jurisdiction, not violating national law.\r\n\r\nget the analogy right before you post again... or do i have to reduce myself to a humorless brick before we even start debating?", "Solution_7": "If he had a firefight then it's definitely illegal for him to be having a gun.", "Solution_8": "fine, he has ninja throwing stars, or a compound bow, or really pointy darts. or maybe he had a fistfight w/ your neighbor. sorry if i didn't have the perfect analogy. please try to see the bigger picture of the analogy. back on topic, now, please.", "Solution_9": "The thing is, it can be deduced that he has intent to kill which is [i]not allowed[/i]. However, whatever North Korea does IS allowed as it has FULL SOVERIGNTY except for any resolutions the UN may pass because it is a member.", "Solution_10": "[quote=\"PenguinIntegral\"]If they were to say, smuggle a nuke into Tokyo and set it off, they'd be able to take out a few thousand South Koreans before we bombed them back to the stone age.[/quote]\r\n\r\nLast I checked, Tokyo was in Japan. I think you're looking for Seoul :wink:.", "Solution_11": "I'm quite worried about the situation... :maybe: \r\n\r\nI fear that Japan will use this event as an excuse to build their own nuclear weapons, leading to a nuclear arms race between Japan and China :o", "Solution_12": "[quote=\"PenguinIntegral\"]http://en.wikinews.org/wiki/North_Korea_claims_it_has_conducted_a_nuclear_test\n\nSo, I'm wondering if China or Russia are going to do anything about it, or if everyone's just going to sit back and go \"ho-hum\". The US is sure way too unpopular and streached out to do anything. Early results look like a relatively low-magnitude bomb, and I don't think the North Koreans are planning on doing much with it at the moment. If they were to say, smuggle a nuke into Tokyo and set it off, they'd be able to take out a few thousand South Koreans before we bombed them back to the stone age. CNN projects that a war will have about 100,000 casualties.[/quote]\r\n\r\nYou speak in such an interesting fashion. I intend to reply the whole thing but just can't help quoting this one first \"before we bombed them back to the stone age\". I'm just surprised you missed the next line \"Yeah baby, let's rock & bomb them!\". That's quite a spirit in the first place you know? I mean, I still find it hard to understand the state of the mind in which one sits and states 'let's bomb these asians to the stone age\". But that surely explains the US being unpopular.\r\n\r\nSo we have 'go ho-hum' as alternative number one, alternative number two, good ol' rock'n roll. I can perceive distaste for alternative number one, nevertheless, it is valuable maybe the fact that it doesn't implies war? Do you think it is a solution a 100 000 casualties belic 'incident'? I forget you are assuming they'll be giving the world a reason for it, the damn communists targeting Seoul or something. We've seen much already in this millenium and this might be an argument based ultimately on faith but the question I would ask inmediately would be, [i]what for?[/i]. In order to get their country levelled? Relations between Pyonyang and Seoul have seen a slight improvement before with the joint declaration and a reunification of Korea I can only imagine in the long run as good for both sides. So again, what makes you think NK will use its nuclear capability on actual targets? and are you sure they will be the first to strike in the case we're in the brink of war?\r\n\r\nIn the other hand, I think there must be criticism on this event, and it is not as simple as claiming sovereignity. Countries have indeed the right to develop a peaceful nuclear program but what is the intention of building a nuke? And this should be criticized and dealed with diplomatically. There are enough nukes in the world already, there's no need for more, be NK of the US who builds them. The nuclear bomb was a weapon in whose development historical motives played a significant role. Those motives are gone, in today's world there's no need for nukes, nor for war, for what matters. By withdrawing from the nuclear non proliferation treaty (which by the way was and maybe still is sistematically violated by western countries) and now by detonating a bomb NKorea is clearly stating that they ALSO consider the nuclear bombs a solution. This should be at least a concern for everyone who hopes for, in a few million years or so, a peaceful and balanced world.\r\n\r\nDaniel", "Solution_13": "[quote=\"ben12345_5\"]I'm quite worried about the situation... :maybe: \n\nI fear that Japan will use this event as an excuse to build their own nuclear weapons, leading to a nuclear arms race between Japan and China :o[/quote]\r\n\r\nNo, I don't think Japan will be doing any nuclear bomb development any time soon. \r\n\r\n[list]$\\bullet$Firstly, Japan's constitution prohibits use of its military other than in defense, and a nuclear bomb would not exactly be a defensive use of the military. \n\n$\\bullet$Secondly, I perceive Japan's culture to be fairly anti-war, at least among the younger generation. Japan is so used to being subservient to the US, their culture reflects this and the younger Japanese generation (I believe) as a whole would have little inclination to go to war. \n\n$\\bullet$Thirdly, because of this anti-war stance, the news that nuclear development was going on would result in loss of power by the ruling political party (which has been in power almost all of Japan's post-WWII history), and another political party would take control. And everyone knows that [i]politicians[/i] are assuredly spineless when it's their job on the line :D.[/list]\r\n\r\nI will admit that I'm not Japanese and I haven't experienced Japanese culture firsthand.", "Solution_14": "[quote=\"L_Li\"][quote=\"Ignite168\"]\nYes, my neighbor definitely has that right to bear arms as long as he wasn't just released from prison for having illegal arms or something.[/quote]\n\nNo, but he never really got over that firefight he had w/ your friend and other neighbor a few years ago...\n\n[quote=\"Ignite168\"]Also, your example is bad, my neighbor is under the jursidiction of the US and its laws, however, we don't control North Korea, they are their own country with their own soverignty.[/quote]\n\nyour neighbor = NK\nyou = US\n\nNK not under US jurisdiction, not violating international law (which debatably even exist)\nneighbor not under your jurisdiction, not violating national law.\n\nget the analogy right before you post again... or do i have to reduce myself to a humorless brick before we even start debating?[/quote]\r\n\r\nI think Ignite is not allowed to keep the gun himself either. And if he has, he can't complain about his neighbour. So if the US agree to hang up their nukes, and Russia, China, India, Pakistan, Iran all such nuclear powers follow suit, sure we shall take them off Seoul as well. No problem. See?\r\n\r\n[img]http://www.lokasenna.pe.kr/bb/denahi_spring.jpg[/img]", "Solution_15": "And also without the aid North Koreans would be starving and CCY I love how you just started saying that Ignite is a racist if you had not noticed Ignite said that the South Koreans were stupid after 2 of them started telling everyone that we should nuke Russia and China off the map because they exist and protest nuking North Korea as that would cause huge enviromental damage and probably kill millions of Chinese South Koreans and Russians from radiation. And the huge amounts of aid were proven to be a lot less then what you keep saying if you had not noticed.", "Solution_16": "But the thing is, you can't just say that after you see comments that were obviously not serious in their intent. Look at the context of his first comment\r\nhttp://www.artofproblemsolving.com/Forum/viewtopic.php?t=113976&start=60#648226\r\nand tell me how that is justified.\r\n\r\nCutting off aid is not just to please Kim Jong-Il so he won't attack China, whatever the chances of that may be--it's to slow the flow of N. Korean refugees over the border. China doesn't want to be responsible for those refugees overflowing into its borders. But in the end, it's a problem on the N. Korean side--not developing their agriculture properly and/or not distributing this food properly to their people.", "Solution_17": "That was because the 2 south koreans were saying to nuke North Korea off the map on the previous page. Yes I agree it was a bad thing to say but guess several posts later someone decides to blame the issue on China and Russia (which by the way even the person had no way to connect to this) and the koreans decided that they should be nuked also so I do not agree with what he said but what you are defending is worse", "Solution_18": "[quote=\"Ars\"]That was because the 2 south koreans were saying to nuke North Korea off the map on the previous page. Yes I agree it was a bad thing to say but guess several posts later someone decides to blame the issue on China and Russia (which by the way even the person had no way to connect to this) and the koreans decided that they should be nuked also so I do not agree with what he said but what you are defending is worse[/quote]\r\ndude, are you stupid?\r\n1. treething NEVER says anything serious about these matters.\r\n2. I was being sarcastic", "Solution_19": "yes but then Ignite is being accused of being a racist as he was also joking but some people did not understand it which makes it interesting as they see you guys joking but accuse Ignite for saying a joke. That is the problem", "Solution_20": "[quote=\"Ars\"]That is the problem[/quote]\r\nNo.\r\nThe only problem is that there are about 6 posts in this 6-page-thread seriously concerning with the topic! The rest is a near-to-flame discussion about chinese-japanese conflicts, accusations of being racist, how stupid or not Koreans are, how bad USA are, and whatever offtopic else.\r\n\r\nAm I the only one who has this impression\u00bf", "Solution_21": "[quote=\"ZetaX\"]\nThe only problem is that there are about 6 posts in this 6-page-thread seriously concerning with the topic! The rest is a near-to-flame discussion about chinese-japanese conflicts, accusations of being racist, how stupid or not Koreans are, how bad USA are, and whatever offtopic else.\n\nAm I the only one who has this impression\u00bf[/quote]\nYou're not alone.\n\nBut you forgot to mention Treething suggesting USA nukes Russia... :mad:\n\n[quote=\"Treething\"]well just nuke the bastards, nuke russia and china too if they get pissed or something[/quote]", "Solution_22": "[quote=\"i_like_pie\"][quote=\"ZetaX\"]\nThe only problem is that there are about 6 posts in this 6-page-thread seriously concerning with the topic! The rest is a near-to-flame discussion about chinese-japanese conflicts, accusations of being racist, how stupid or not Koreans are, how bad USA are, and whatever offtopic else.\n\nAm I the only one who has this impression\u00bf[/quote]\nYou're not alone.\n\nBut you forgot to mention Treething suggesting USA nukes Russia... :mad:\n\n[quote=\"Treething\"]well just nuke the bastards, nuke russia and china too if they get pissed or something[/quote][/quote]\r\nand you forgot that treething is never serious on these forums(ie its a dumb joke)", "Solution_23": "And also I tried to get this back towards the topic in my first post but that entirely failed.", "Solution_24": "i think that the n. koreans look like nazis \r\nif i were indiana jones i'd be fighting them\r\nreally who is the US to tell them what to do? :?: \r\nNOTE: Funcia is not setting off nukes in north korea if you think this you are among the fake-moustached and obtused-glassesed people who are plotting against him.", "Solution_25": "[quote=\"funcia\"]i think that the n. koreans look like nazis \n[/quote]\r\nI don't care if that's a joke\r\nGO DIE\r\n\r\n(wow this thread is so off topic)", "Solution_26": "My point was NOT that. But I am wrong about it in many ways but right in one way. And in that one way I was talking about the US too. That way is \"patriotism\". Which I don't really like and is how I say people look like nazis.\r\nThe US is acting like it owns the world. which i must point out is not really true.", "Solution_27": "This should be locked.", "Solution_28": "Yeah I don't want to fight with junggi or anyone. If someone is a seer and can see what they'll do with the nukes-fine. Tell us. But we all are biased in some way. Last post Funcia will make in this thread. Bye!", "Solution_29": "So long, [b]funcia[/b]! And thanks for all the fish!!!\r\n\r\nThe title of the thread promises a lot. But the posts disappoint. We should really have more order on ML/AoPS.\r\n\r\nOn a Romanian forum (there the moderators are oppressive: you can actually get banned for repeated off topic and/or flaming - I guess it's a remnant from the \"Communist Era\"), I found a similar discussion. One of the users is a fervent conspiracy theorist... and I just can't get enough.\r\n\r\nI have one question: Does anyone think that North Koreans are really THAT ignorant? Or you people just like to post (like me)?" } { "Tag": [ "MATHCOUNTS", "rotation", "analytic geometry", "geometry", "geometric transformation", "trigonometry" ], "Problem": "When the point (4,1) is rotated 90 degrees counterclockwise about the point (1,0), what are the coordinates of the point at which it lands?", "Solution_1": "I find that because these are Mathcounts, all you need to do is graph it, because it's all lattice points in MC. xD\r\n\r\nAnyway, a real solution:\r\nOK think about it this way. Draw the lines $ x\\equal{}1$ and $ y\\equal{}1$, and the line containing both points (that were given). We form a right triangle here. We know that the two legs are of length 1 and 3. If we rotate the point (4,1), this is essentially rotating the triangle: the legs will stay the same. Because we are just rotating lines perpendicular to the axes, this is not too difficult to do.", "Solution_2": "OK. Imagine you had 2 coordinate planes on transparent sheets of paper. Align them perfectly one on top of the other. On the top plane, mark the point (4,1). Fix the bottom plane to the table so that it doesn't move. Now put a pin at the point (1,0). Now it is possible to spin the top plane around the point (1,0). Rotate it 90 degrees counterclockwise.\r\n\r\nBefore the rotation, the point (4,1) one unit higher and 3 units to the right of (1,0). With some careful thinking, you will realize that after the rotation, the point is 3 units higher and one unit to the left of (1,0). That is, directly over the point labeled (0,3) on the bottom, fixed plane.\r\n\r\nIn general, when you rotate point (a,b) 90 degrees counterclockwise about point (c,d), you get point ( c-b+d , d+a-c ). (or at least I think so... :maybe: )", "Solution_3": "Yeah I suck with graphs...", "Solution_4": "Thanks math154 and leoxnlin. :) Leoxlin, the way you explained it really helped me understand.", "Solution_5": "In a Cartesian plane, any $ (x,y)$ rotated counterclockwise through any angle $ \\theta$ about the origin will result in $ (x',y')$...\r\n\r\n$ x'\\equal{}x\\cos(\\theta)\\minus{}y\\sin(\\theta)$\r\n\r\n$ y'\\equal{}x\\sin(\\theta)\\plus{}y\\cos(\\theta)$\r\n\r\nTo rotate a point $ (x_1,y_1)$ about $ (x_2,y_2)$ with an angle of $ \\theta$...\r\n\r\n$ x'\\equal{}(x_1\\minus{}x_2)\\cos(\\theta)\\minus{}(y_1\\minus{}y_2)\\sin(\\theta)\\plus{}x_2$\r\n\r\n$ y'\\equal{}(x_1\\minus{}x_2)\\sin(\\theta)\\plus{}(y_1\\minus{}y_2)\\cos(\\theta)\\plus{}y_2$\r\n\r\nSo the answer is $ (0,3)$.\r\n\r\nSorry if I'm over complicating things.", "Solution_6": "Yeah, but this is MC forum and we're not supposed to use trig. :noo: :o", "Solution_7": "Hey, it doesn't say anywhere in the MathCounts Rules that \"Competitors are neither allowed to use Trigonometry to solve the problems nor think about Trigonometry while the contest is in session.\" In fact, I used Trig for a lot of my MathCounts problems.", "Solution_8": "Yeah but you're not expected to use trig...there's basically always a better way to do the things without trig.", "Solution_9": "no problem in mathcounts [b]requires[/b] the use of trigonometry, but you can and in some problems, it may be beneficial.", "Solution_10": "[quote=\"herefishyfishy1\"]In a Cartesian plane, any $ (x,y)$ rotated counterclockwise through any angle $ \\theta$ about the origin will result in $ (x',y')$...\n\n$ x' \\equal{} x\\cos(\\theta) \\minus{} y\\sin(\\theta)$\n\n$ y' \\equal{} x\\sin(\\theta) \\plus{} y\\cos(\\theta)$\n\nTo rotate a point $ (x_1,y_1)$ about $ (x_2,y_2)$ with an angle of $ \\theta$...\n\n$ x' \\equal{} (x_1 \\minus{} x_2)\\cos(\\theta) \\minus{} (y_1 \\minus{} y_2)\\sin(\\theta) \\plus{} x_2$\n\n$ y' \\equal{} (x_1 \\minus{} x_2)\\sin(\\theta) \\plus{} (y_1 \\minus{} y_2)\\cos(\\theta) \\plus{} y_2$\n\nSo the answer is $ (0,3)$.\n\nSorry if I'm over complicating things.[/quote]\r\nwhy are the top formula's true?", "Solution_11": "Consider a rotation of the axes by $ \\theta$ degrees. Choose coordinates $ (x,y)$ such that they lie on one axis of the rotation. Basic trig will yield those formulas.", "Solution_12": "you can also convert to polar and back to show that.\r\n\r\nremember that x=rcostheta and y=rsintheta to convert" } { "Tag": [ "geometry", "3D geometry", "sphere", "icosahedron", "ratio", "rectangle", "geometry proposed" ], "Problem": "12 points are arranged on the surface of a sphere with radius 1. Let \"a\" be the shortest distance between any two of the points. What is the maximum possible value of \"a?\"", "Solution_1": "the regular icosahedron inscribed in the sphere has 12 vertices and gives an arrangement with maximal $ a$: $ a$ is the length of an edge of the icosahedron and is easy to compute via straightforward geometry and trig.", "Solution_2": "The ratio betweem the radius of the sphere and the value \"a\" can be determined by observing a rectangle inside the icosahedron that connects opposite edges. This rectangle happens to be the golden rectangle (this can be shown by observing pentagons within the icosahedron). From there, the diameter of the sphere is the rectangles diagonal, and the value \"a\" is it's shorter edge length. \"a\" can now be solved for using the pythagorean theorem." } { "Tag": [ "articles", "LaTeX" ], "Problem": "Hello,\r\n\r\nA friend and I have the next problem: we collected the drafts of some lectures, those have been given by different people, and the author of each lecture typed it. Now, everyone defines different commands, and some of those would overwrite the ones of other people. Is there a way to say to the compiler in $\\LaTeXe$ to take the macros on each document separately.\r\n\r\nLet me rephrase, because probably it is not totally clear. Suppose that I typed lecture1, and I defined\r\n[code]\\newcommand{\\R}{\\mathbb{R}}\n\\newcommand{\\r}{\\rightarrow}\n\\newcommand{\\rr}{\\Rightarrow}[/code]\nwhile John typed lecture2, and he defined\n[code]\\newcommand{\\rr}{\\mathbb{R}}\n\\newcommand{\\ra}{\\Rightarrow}[/code]\nthe command \\rr is rewrited, but I don't want to have to open the file typed by John and \"correct\" all the \\rr to \\R, and change al the \\ra to \\rr. Is there a way to just say something similar to\n\n[code]%\n% Something in the preamble\n%\n\\begin{document}\n\n\\include{lecture1}\n\n\\include{lecture2}\n\n\\end{document}[/code]\r\n\r\nand that the compiler understands that it has to take the macros in lecture1 just for lecture1, and the ones in lecture2 just for lecture2?\r\n\r\nIt should be possible, because the journals usually receive their articles (I suppose) typed with the personal commands of each author defined on the preamble of the article, and they compile all of them together!\r\n\r\nthanks :first:", "Solution_1": "It seems to me that the simplest solution is to change all subsequent redefinitions of the same command to \\renewcommand rather than \\newcommand. For instance, in your example above, all you would need to do is to change the \\newcommand{\\rr}{...} line in Lecture 2 to \\renewcommand{\\rr}{...}, and it should work fine.", "Solution_2": "[quote=\"DPatrick\"]It seems to me that the simplest solution is to change all subsequent redefinitions of the same command to \\renewcommand rather than \\newcommand. For instance, in your example above, all you would need to do is to change the \\newcommand{\\rr}{...} line in Lecture 2 to \\renewcommand{\\rr}{...}, and it should work fine.[/quote]Thank you! I will try that!" } { "Tag": [ "calculus", "integration", "number theory", "relatively prime" ], "Problem": "Transform the equation $ x^{5}\\plus{}25x^{4}\\plus{}250x^{3}\\plus{}1250x^{2}\\plus{}3125x\\plus{}3125\\equal{}0$, so that its roots may be increased by $ 5\\frac{1}{2}$. Find the sum of the numerical coefficients of the third degree term and the 1st degree term if the transformed equation has integral coefficients.\r\nA) 70\r\nB) 79\r\nC) 81\r\nD) 90\r\nE) 89", "Solution_1": "$ (x\\plus{}5)^5\\equal{}0$\r\n\r\nAll roots are equal to -5\r\n\r\n$ \\minus{}5\\plus{}5\\frac{1}{2}\\equal{}\\frac{1}{2}$\r\n\r\n$ (x\\minus{}\\frac{1}{2})^5\\equal{}x^5\\minus{}\\frac{5}{2}x^4\\plus{}\\frac{5}{2}x^3\\minus{}\\frac{5}{4}x^2\\plus{}\\frac{5}{16}x\\minus{}\\frac{1}{32}$\r\n\r\nI can't finish this problem, cause I just don't know what to do.", "Solution_2": "Just multiply it by 32 to get rid of the fractions:\r\n\r\n$ 32(x \\minus{} \\frac12)^5 \\equal{} 32x^5\\minus{}80x^4\\plus{}80x^3\\minus{}40x^2\\plus{}10x\\minus{}1$\r\n\r\n$ 10 \\plus{} 80 \\equal{} 90$.", "Solution_3": "So \"intergral coefficients\" means that all coefficients are relatively prime integers? :)", "Solution_4": "\"integral\" means that the coefficients are integers, not necessarily relatively prime. I picked the relatively prime solution because all the others would exceed the alternatives given (i.e. they would be higher than 90)." } { "Tag": [ "linear algebra", "matrix" ], "Problem": "Let\r\n\\[ \\mathsf{A} \\equal{} \\begin{pmatrix} 2 & 0 \\\\ 0 & 1 \\end{pmatrix} \\textup{ and } \\mathsf{P} \\equal{} \\begin{pmatrix} 1 & 0 \\\\ 1 & 1 \\end{pmatrix}\\]Do you expect $ \\mathsf{PAP}^{\\minus{}1}$ to be symmetric? Why?", "Solution_1": "$ P$ is not an orthogonal matrix, so at first sight there is no reason to expect $ B \\equal{} PAP^{\\minus{}1}$ to be symmetric. What you can say at a glance is that $ B$ is lower triangular.", "Solution_2": "So in general, for $ B \\equal{} PAP^{\\minus{}1}$ to be symmetric it is required that $ P$ is orthogonal?", "Solution_3": "Assume $ A\\equal{}diag(\\lambda_i)\\in\\mathcal{M}_n(\\mathbb{R})$ where the $ \\lambda_i$'s are pairwise distinct. Let $ P$ be an invertible matrix s.t. $ S\\equal{}PAP^{\\minus{}1}$ is symmetric. Then $ P\\equal{}OD$ where $ O$ is an orthogonal matrix and $ D$ is a diagonal matrix with positive entries. Conversely these matrices $ P$ work.\r\nProof: Let $ (e_i)$ be the canonical basis of $ \\mathbb{R}^n$ (it's an orthonormal basis). Then $ (P(e_i))$ is a basis of eigenvectors of $ S$. The eigenvalues of $ S$ are pairwise distinct. Hence $ (P(e_i))$ is an orthogonal basis. Thus the columns of $ P$ are pairwise orthogonal and we are done. The converse is easy." } { "Tag": [], "Problem": "Three ants 1,2,3 are placed at the three vertices of an equilateral triangle ABC of side length 'a' . They start moving towards each other with a velocity 'v' each. The three ants meet at the centroid of the triangle after a time of t= 2a/3v.\r\n\r\nFind the Angular Velocity and the Angular Acceleration of ant 2 with respect to ant 1 at time t=0.", "Solution_1": "YEH TO HC VERMA KA SOLVED PROBLEM HAI", "Solution_2": "anvay hindi mein baat muth karo.kuch logon ko hindi samajh mein nahin aaegi.", "Solution_3": "[quote=\"Anvay\"]YEH TO HC VERMA KA SOLVED PROBLEM HAI[/quote]\n\nTranslates to : THIS IS HC VERMA'S SOLVED PROBLEM!\n\n\n[quote=\"aadil\"]anvay hindi mein baat muth karo.kuch logon ko hindi samajh mein nahin aaegi.[/quote]\r\n\r\nSame goes for you! :P\r\n\r\n\r\n\r\n\r\n\r\n\r\n.", "Solution_4": "Look at the full question man! \r\nI have asked about the relative angular acceleration, No where in H C Verma is it solved! \r\n[b]Whatever is solved there, I have already supplied in the question![/b]" } { "Tag": [ "conics", "ellipse", "analytic geometry", "geometry unsolved", "geometry" ], "Problem": "recently i have came across this question which i dun think i can solve, \r\nIf the circle $ x^2\\plus{}y^2\\minus{}ax\\equal{}0$ meet the ellipse $ \\frac{x^2}{a^2}\\plus{}\\frac{y^2}{b^2}\\equal{}1$ at three points, prove the the x coordinate of the centroid of the three points is $ a(a^2\\plus{}b^2)/3(a^2\\minus{}b^2)$", "Solution_1": "I am enclosing a pdf file with the solution of this problem. It is not so hard as is said in the ttle..." } { "Tag": [ "linear algebra", "matrix", "algebra", "polynomial", "superior algebra", "superior algebra solved" ], "Problem": "Find out all X in M_2(R) so that we have X^2004-11X^2003+30X^2002=O_2.", "Solution_1": "X has eigenvalues 0,5 or 6.\r\nSo X is similar to a matrix A (X=TAT-1 for some invertible X) from the set:\r\n0, 5I, 6I, 6I+U, 5I+U, U\r\n\r\n0 0 \r\n0 5\r\n\r\n0 0\r\n0 6\r\n\r\n5 0\r\n0 6\r\n\r\nU is the matrix:\r\n0 1\r\n0 0", "Solution_2": "I have another solution, lets see if u find it! :D :D", "Solution_3": "the matrix has to have the proper values 0, 5 or 6, but 5 and 6 can appear at most one time. \r\n\r\nso basically the char. polynomial of X can only be x^2, x(x-5), x(x-6) or (x-5)(x-6).\r\n\r\nso the solutions of this eq. are exactly those matrices in M_2(R) which have the char. polynomial one of the 4 mentioned above. \r\n\r\nthey are fully (well almost :D) described by two elements from the first line." } { "Tag": [ "inequalities" ], "Problem": "Show that :\r\n\r\n$ \\frac{1}{1^2}\\plus{}\\frac{1}{2^2}\\plus{}\\frac{1}{3^2}\\plus{}...\\plus{}\\frac{1}{n^2}<\\frac{41}{24}\\ ,\\ (\\forall)n\\in \\mathbb{N}$", "Solution_1": "[hide=\"solution\"] Clearly $ \\frac{1}{3^2}\\plus{}\\frac{1}{4^2}\\plus{}...\\plus{}\\frac{1}{n^2}<\\frac{1}{4}\\plus{}\\frac{1}{8}\\plus{}\\frac{1}{16}\\plus{}...\\frac{1}{2^n}<\\frac{1}{4}\\plus{}\\frac{1}{8}\\plus{}\\frac{1}{16}\\plus{}...$, where the last sum continues to infinity. This is true firstly because each of the corresponding terms in the first are smaller than the corresponding in the second and secondly because the third is an infinite series, not a finite.\n\nThe sum of the infinite series $ \\frac{1}{4}\\plus{}\\frac{1}{8}\\plus{}...$ is $ \\frac{1}{2}$. Thus, $ \\frac{1}{1^2}\\plus{}\\frac{1}{2^2}\\plus{}\\frac{1}{3^2}\\plus{}...\\plus{}\\frac{1}{n^2}<\\frac{1}{1^2}\\plus{}\\frac{1}{2^2}\\plus{}\\frac{1}{4}\\plus{}\\frac{1}{8}\\plus{}....\\equal{}\\frac{5}{4}\\plus{}\\frac{1}{2}\\equal{}\\frac{7}{4}<\\frac{41}{24}$.[/hide]", "Solution_2": "[quote=\"xpmath\"]Clearly $ \\frac {1}{3^2} \\plus{} \\frac {1}{4^2} \\plus{} ... \\plus{} \\frac {1}{n^2} < \\frac {1}{4} \\plus{} \\frac {1}{8} \\plus{} \\frac {1}{16} \\plus{} ...\\frac {1}{2^n} < \\frac {1}{4} \\plus{} \\frac {1}{8} \\plus{} \\frac {1}{16} \\plus{} ...$, where the last sum continues to infinity. This is true firstly because each of the corresponding terms in the first are smaller than the corresponding in the second and secondly because the third is an infinite series, not a finite.\n\nThe sum of the infinite series $ \\frac {1}{4} \\plus{} \\frac {1}{8} \\plus{} ...$ is $ \\frac {1}{2}$. Thus, $ \\frac {1}{1^2} \\plus{} \\frac {1}{2^2} \\plus{} \\frac {1}{3^2} \\plus{} ... \\plus{} \\frac {1}{n^2} < \\frac {1}{1^2} \\plus{} \\frac {1}{2^2} \\plus{} \\frac {1}{4} \\plus{} \\frac {1}{8} \\plus{} .... \\equal{} \\frac {5}{4} \\plus{} \\frac {1}{2} \\equal{} \\frac {7}{4} < \\frac {41}{24}$.[/hide][/quote]\r\n\r\nNo! No!\r\n\r\n$ \\frac{7}{4} \\equal{} \\frac{42}{24} > \\frac{41}{24}$", "Solution_3": "Whoops. Anyway, just use $ \\frac{1}{9}\\plus{}\\frac{1}{27}\\plus{}...$ in place and add that to $ \\frac{1}{1^2}\\plus{}\\frac{1}{2^2}\\plus{}\\frac{1}{3^2}$ with the same argument.", "Solution_4": "$ \\frac{1}{k^2}<\\frac{1}{k(k\\minus{}1)}$\r\n\r\n$ \\Rightarrow LHS<1\\plus{}\\frac{1}{4}\\plus{}\\frac{1}{9}\\plus{}\\frac{1}{3\\cdot 4}\\plus{}\\frac{1}{4\\cdot 5}\\plus{}...\\plus{}\\frac{1}{(n\\minus{}1)n}\\equal{}\\frac{49}{36}\\plus{}\\frac{1}{3}\\minus{}\\frac{1}{n}<\\frac{61}{36}\\equal{}\\frac{122}{72}<\\frac{123}{72}\\equal{}\\frac{41}{24}$", "Solution_5": "the series $ \\frac1{1^2}\\plus{}\\frac1{2^2}\\plus{}\\frac1{3^2}\\plus{}...$ converges to $ \\frac{\\pi^{2}}{6}$", "Solution_6": "[quote=\"xpmath\"]Whoops. Anyway, just use $ \\frac {1}{9} \\plus{} \\frac {1}{27} \\plus{} ...$ in place and add that to $ \\frac {1}{1^2} \\plus{} \\frac {1}{2^2} \\plus{} \\frac {1}{3^2}$ with the same argument.[/quote]\r\n\r\nSo you believe that $ \\frac{1}{n^2} < \\frac{1}{3^n}$ for all $ n > 3$?", "Solution_7": "Ok disregard everything I said. I wasn't thinking at all :wallbash: . Sorry about that." } { "Tag": [ "linear algebra", "matrix" ], "Problem": "Can anyone help me with the following?\r\n\r\nIf $ P$ is a projection matrix, show from the infinite series that $ e^P \\sim I \\plus{} 1.718P$? \r\n\r\nFor this problem, I wrote out the infinite series expansion $ e^P \\equal{} I \\plus{} P \\plus{} \\frac{P^2}{2!} \\plus{} \\frac{P^3}{3!} \\plus{} ... \\plus{} \\frac{P^n}{n!}$, but the $ P$s are matrices and I need 1.718, a number. Also, how can I show that what I get is independent of any projection matrix?", "Solution_1": "Remember that $ P^n \\equal{} P$, so it's $ P\\left(\\sum_1^\\infty \\frac{1}{n!}\\right)$" } { "Tag": [ "function", "algebra", "polynomial" ], "Problem": "Yeah, exchanging the variables would work. Also remember that if g is the inverse of f, then\r\nf(g(x)) = x and g(f(x)) = x.\r\n\r\nFierytycoon", "Solution_1": "Ex: find the inverse of f(x) = 3x + 2\r\n\r\nHere's the sequence:\r\nf(x) = 3x + 2\r\ny = 3x + 2\r\nx = 3y + 2\r\nx - 2 = 3y\r\ny = (x-2)/3\r\nf^-1(x) = (x-2)/3\r\n\r\nSo f(f^-1(x)) = x", "Solution_2": "[quote=\"mathfanatic\"]So f(f^-1(x)) = 1[/quote]\r\n\r\nf(f^-1(x))=x actually.", "Solution_3": "Ug. I am having a bad day...\r\n\r\nHmm...maybe I can make a list of short phrases that I use to express disgust: \"ug, oops, erp, eek, etc.\"", "Solution_4": "Soon you realise that for most functions it is not easy to find an inverse function. Sometimes specific inverse functions have been created, i.e. arcsin, arccos, arctan. :)", "Solution_5": "Well, we were referring to polynomial functions. Exponential functions (and their inverses logarithmic functions), trigonometric functions, etc. naturally don't use this rule.", "Solution_6": "Find the inverse of a generic quintic equation that has df/dx>=0 for all x. That's still really hard (read: impossible with ``elementary'' functions)." } { "Tag": [ "factorial", "algebra", "binomial theorem" ], "Problem": "coeff of $x^{5}y^{4}$ in $(2x-3y^{2})^{7}$?", "Solution_1": "[quote=\"yeppyyep\"]coeff of $x^{5}y^{4}$ in $(2x-3y^{2})^{7}$?[/quote]\r\n\r\n[hide=\"solution\"]$\\text{Using the binomial theorem, the coefficient of}\\ x^{5}y^{4}\\ \\text{is}$ ${7\\choose 2}2^{5}(-3)^{2}=21\\cdot 32\\cdot 9=\\boxed{6048}$[/hide]", "Solution_2": "[quote=\"yeppyyep\"]coeff of $x^{5}y^{4}$ in $(2x-3y^{2})^{7}$?[/quote]\r\nObviously his way works but another way of looking at is (just taking his way to the next step) is:\r\n\r\n[hide]$\\frac{7! \\cdot (2x)^{5}\\cdot (-3y^{2})^{2}}{5! \\cdot 2!}$; Just take the exponents which each of your variables are raised to and put them as factorials on the bottom while their sum as a factorial on the top.\n$=6048 x^{5}y^{4}$\n\nThus it is $\\boxed{6048}$[/hide]" } { "Tag": [ "algebra", "polynomial", "national olympiad" ], "Problem": "1.$ ABCDEF$ is a convex hexagon such that all of its vertices are on a circle. Prove that $ AD$, $ BE$ and $ CF$ are concurrent if and only if $ \\frac {AB}{BC}\\cdot\\frac {CD}{DE}\\cdot\\frac {EF}{FA} = 1$.\r\n\r\n2. Determine whether there are two infinite point sequences $ A_1,A_2,\\ldots$ and $ B_1,B_2,\\ldots$ or not, such that for all $ i,j,k$ that satisfies $ 1\\le i < j < k$,\r\n i.$ B_k$ is on the line that passes through $ A_i$ and $ A_j$ if and only if $ k = i + j$.\r\n ii.$ A_k$ is on the line that passes through $ B_i$ and $ B_j$ if and only if $ k = i + j$.\r\n\r\n3. $ n$ is a positive integer. Calculate the sum $ \\sum_{k = 1}^{n}{\\sum_{1\\le i_1 < \\ldots < i_k\\le n}^{}{\\frac {2^k}{(i_1 + 1)(i_2 + 1)\\ldots (i_k + 1)}}}$\r\n\r\n4. The sequence of polynomials $ (a_n)$ is defined as, $ a_0 = 0$, $ a_1 = x + 2$ and for all $ n > 1$ integers, $ a_n = a_{n - 1} + 3a_{n - 1}a_{n - 2} + a_{n - 2}$. \r\n a.Show that for $ k,m$ positive integers, if $ k$ divides $ m$; $ a_k$ divides $ a_m$.\r\n b.Find all $ n$ positive integers such that sum of the roots of polynomial $ a_n$ is an integer.", "Solution_1": "number $ 1$ is VERY old,it has been discussed here to. :huh: \r\nhere for example:\r\n[url]http://www.mathlinks.ro/viewtopic.php?t=182839[/url]", "Solution_2": "1. It is very well-known :D\r\n2.[url]http://www.mathlinks.ro/viewtopic.php?t=197870[/url]\r\n3.[url]http://www.mathlinks.ro/viewtopic.php?p=1088260[/url]\r\n4.[url]http://www.mathlinks.ro/viewtopic.php?p=1088257[/url]" } { "Tag": [], "Problem": "Find the number with $97$ digits, all different from $0$, such that it is multiple of the sum of all its $97$ digits.", "Solution_1": "e.g. $111...11122$ is divided by 99 ($9\\cdot11$)" } { "Tag": [ "induction", "algebra proposed", "algebra" ], "Problem": "let $\\left\\{ x_n \\right\\}$ be a sequence given by : $x_1 = 1$ and $x_{n+1} = \\frac{x_n}{n} + \\frac{n}{x_n}$\r\nprove that :$[x_n^2] = n$", "Solution_1": "$x_{n+1} = \\frac{x_n}{n} + \\frac{n}{x_n} \\Longleftrightarrow x_2=2$$???$", "Solution_2": "Actually, this statement is true for all $n \\ge 4$.", "Solution_3": "$hard ?$", "Solution_4": "It is easy. Let $y_n=x_n^2, y_4=\\frac{169}{36}=4+\\frac{25}{36},y_{n+1}=f_n(y_n),f_n(x)=\\frac{x}{n^2}+2+\\frac{n^2}{x}.$\r\nIt is easy to prove by induction, that $n+\\frac{2}{n-1}n+1+\\frac 2n,$ and\r\n$f_n(n+\\frac{2}{n-1})=\\frac{(n-2)^2(n+1)^2+2n^2(n^2-1)(n-2)+n^4(n-1)^2}{n^2(n^2-1)(n-2)}0 \\right\\}$ .\r\n\r\nFor every sequence $ (z_n)$ with $ z_n\\in D$ and $ z_n\\to 0$\r\n\r\n$ w(z_n)\\to \\frac{\\sqrt{\\pi}}{2}$ and for all other $ z\\notin D \\quad w(z)$ is either undefined or $ 0$ iff $ z\\equal{}0$ ." } { "Tag": [ "function", "real analysis", "real analysis unsolved" ], "Problem": "Let $ f: R\\minus{}>R$ be a nonconstant function, periodical and with primitives. Prove that $ f$ has a minimal positive period.", "Solution_1": "since f has primitives, f is continuous. since f is periodical and nonconstant it has a positive period $ b$. Suppose f does not have a minimal positive period then $ b/2 ,...,b/n$ is a period of f for all integer n. $ \\forall x f(x\\plus{}b/n)\\equal{}f(x)$ then $ f(\\frac{kb}{n})\\equal{}f(0)$ | $ \\forall (n,k) \\in N^{*}.N$\r\nWe can suppose $ b \\in Q^{\\plus{}}$ , the case $ b \\notin Q$ is similar. So for all rational number x f(x)=f(0) and we conclude by the density of Q in R and the continuity of f, which extends the result for every real number x , f(x)=f(0). That is in contradiction with the nonconstantness of f.\r\nHence f has a minimal positive period", "Solution_2": "[quote=\"jedusor\"]since f has primitives, f is continuous[/quote]\r\nThis is wrong. There are discontinuous functions that have antiderivatives.", "Solution_3": "oh , the stupid mistake, the problem is not as trivial as i thought. :oops_sign:", "Solution_4": "[quote=\"jedusor\"] Suppose f does not have a minimal positive period then $ b/2 ,...,b/n$ is a period of f for all integer n. [/quote] I may put a stupid question too but why is $ b/n$ also period? :roll:", "Solution_5": "because when supposing that f doesn't have a minimal positive period every positive real number less than b is a period of f , it's the case for b/n. For example $ ]0;1]$ doesn't have a minimal positive element $ \\implies \\forall b \\in ]0;1] ,b/n \\in ]0;1]$" } { "Tag": [ "superior algebra", "superior algebra unsolved" ], "Problem": "if $ f: G\\rightarrow H$ is a homormorphism with kernal $ N$ and $ K\\leq G$ , then prove that $ f^{\\minus{}1}(f(K)\\equal{}KN$.\r\n\r\n :)", "Solution_1": "What progress do you have? Where are you stuck?\r\n[hide=\"hint\"]Just show that $ f^{\\minus{}1}(f(K) \\subseteq KN$ and $ KN \\subseteq f^{\\minus{}1}(f(K)$[/hide]", "Solution_2": "Delete this post please :)", "Solution_3": "Finally , I got the usual way .\r\n\r\nIs there another solution with subsets argument ? :wink:", "Solution_4": "What algebraic structures are those $ K,G,N,H$? Monoids or groups or something else?", "Solution_5": "Surely groups" } { "Tag": [ "linear algebra", "matrix", "algebra", "polynomial", "calculus", "derivative" ], "Problem": "Suppose $f(x)=\\left |\\begin{matrix}1&1&1\\\\ 3-x & 5-3x^{2}& 3x^{3}-1\\\\ 2x^{2}-1 & 3x^{5}-1 & 7x^{8}-1 \\\\ \\end{matrix}\\right |$. Prove that there exists $c$ such that $f'(c)=0$.", "Solution_1": "its easy to see that $c=0$ is a solution\r\n\r\n\r\nsince $f(x)$ is polynomial, its derivative is too.\r\n\r\nSo it need to have a root, real or complex.\r\n\r\n\r\nSince the determinat dont have a diagonal with just constants. so. $f(0) = 0$\r\n\r\nand the same thing to his derivative, because of the product of xs.... etc.. etc...", "Solution_2": "post your solution, please! :D I not understand.", "Solution_3": "${f(x)=\\left|\\left( \\begin{array}{lll}1 & 1 & 1 \\\\ 3-x & 5-3 x^{2}& 3 x^{3}-1 \\\\ 2 x^{2}-1 & 3 x^{5}-1 & 7 x^{8}-1 \\end{array}\\right)\\right|}$\r\n\r\n${f(x)=\\left(2 x^{2}-1\\right) \\left(3 x^{3}-1\\right)+\\left(3 x^{5}-1\\right) (3-x)+\\left(7 x^{8}-1\\right) \\left(5-3 x^{2}\\right)-}$\r\n${\\left(\\left(3 x^{3}-1\\right) \\left(3 x^{5}-1\\right)+\\left(7 x^{8}-1\\right) (3-x)+\\left(2 x^{2}-1\\right) \\left(5-3 x^{2}\\right)\\right)}$\r\n\r\n${f(x)=x^{2}\\left(x^{2}\\left(x \\left(-21 x^{5}+7 x^{4}+5 x^{3}-3 x+18\\right)+6\\right)-12\\right)}$\r\n\r\n${f'(x)=x \\left(x^{2}\\left(x \\left(-210 x^{5}+63 x^{4}+40 x^{3}-18 x+90\\right)+24\\right)-24\\right)}$\r\n\r\nOne possible solution is\r\n\r\n${f'(0)=0}$", "Solution_4": "This is my solution\r\n\r\nWe have $f(0)=f(1)=0$ and $f(x)$ is the polynomial therefore by Mean value theorem we have done :D", "Solution_5": "i did the same thingh and u dont understand\r\n\r\n\r\noh......\r\n\r\n\r\nthis is polynomial\r\n\r\ni said it!!!!\r\n\r\nafff......\r\n\r\ndont joke with me", "Solution_6": "[quote=\"Thales418\"]\n\ndont joke with me[/quote]\r\nI not joke with you, I not understand what you want speaked. That is truth :)" } { "Tag": [ "vector", "linear algebra", "linear algebra unsolved" ], "Problem": "Let $ V$ be a finite dimensional Euclidean or unitary space and $ W_1$\r\nand $ W_2$ two vector subspace of $ V$ . Prove that:\r\n\r\na) $ (W_1 \\plus{} W_2)^\\perp \\equal{} W_1^ \\perp \\cap W_2^\\perp$\r\nb) $ (W_1 \\cap W_2)^\\perp \\equal{} W_1^ \\perp \\plus{} W_2^\\perp$\r\n\r\nFitim", "Solution_1": "Don't you mean the ortho-complements of the first vector spaces mentioned in (a) and (b) to have the claim be true? ;)", "Solution_2": "I forgot to write \"T\" to. Look again please :)\r\n\r\nThank you\r\nFitim", "Solution_3": "To prove two sets are equal, show that each is a subset of the other.", "Solution_4": "Use transseries' hint to show (a) - here you do not need the assumption that $ V$ is finite dimensional.\r\n\r\nFor (b) show $ LHS\\subseteq RHS$ first, then that $ \\dim LHS \\equal{} \\dim RHS < \\infty$ using\r\n\\[ \\dim W \\plus{} \\dim W^\\perp \\equal{} \\dim V < \\infty\\]\r\nfor every sub-vector space $ W$ of $ V$ having [b]finite[/b] dimension." } { "Tag": [], "Problem": "assume: $a_n=a_{n-1}+\\frac{a^2_{n-2}}{a_{n-1}}$ and $b_n=\\frac{a_{n+1}}{a_n}$. if $b_n$ is convergant to $L$, prove: $10$\r\n\r\n$\\frac {d}{dL} L^3-L-1=3L^2-1$, so that it is increasing for $|L|^2>\\frac {1}{3}$.\r\n\r\nFurthermore, $L^3-L<0$ for $01$. Setting $L=\\frac {3}{2}$, we see that $L^3-L-1=\\frac {27}{8}-\\frac {3}{2}-1>0$, so \r\n\r\n$10$ such that $x_n\\geq C y_n$ for all $n$] \r\n\r\nSince the length of the range is greater than a constant times $x^{-1}$, we have\r\n\r\n\\[ h(x)\\gg x^{-1}\\cdot x^{3/2}=x^{1/2} \\] \r\n\r\nBut $|\\cot(\\frac{x}{2})|=\\left|\\frac{\\cos\\frac{x}{2}}{\\sin\\frac{x}{2}}\\right|\\gg\\frac{1}{|x|}$ as $x\\to 0$. Therefore $\\cot(\\frac{x}{2})h(x)\\gg |x|^{-1/2}$ is the bad guy.\r\n\r\nConclusion: \r\n\r\n$|f(x)|=|g(x)+\\cot(\\frac{x}{2})h(x)|\\gg |x|^{-1/2}$ as $x\\to 0$, which is what was needed. \r\n\r\nMore summation by parts probably gives $h(x)=O(|x|^{-1/2})$ as well, so we see that $|x|^{-1/2}$ is indeed the true order of magnitude of $f(x)$ as $x\\to 0$ (and not just a lower bound)." } { "Tag": [ "inequalities open", "inequalities" ], "Problem": "\\[\\begin{array}{l}a,b,c \\in R \\\\prove that (a^{2}+b^{2}+c^{2})^{2}\\ge 3(a^{3}b+b^{3}c+ac^{3}) \\\\ \\end{array}\\]", "Solution_1": "See [url]http://www.mathlinks.ro/Forum/viewtopic.php?t=6026[/url]." } { "Tag": [], "Problem": "I'm not sure how to create a sequence that is both arithmetic and geometric.", "Solution_1": "[hide]there is no one except 0,0,0...\nthe proof is that we want $\\frac{x+d}{x}=\\frac{x}{x-d}$ thus $d=0$ so k,k,k..satisfy this[/hide]", "Solution_2": "[quote=\"king_23\"]I'm not sure how to create a sequence that is both arithmetic and geometric.[/quote]\r\n\r\nWell, I can only consider the trivial cases.\r\n\r\n{a}\r\n{a,b}\r\nand\r\n{a,a,a,...}\r\n\r\nWhere a and b are real numbers. But I don't know if they could be imaginary.", "Solution_3": "suppose the first 3 terms are:\r\n\r\n$a-b$, $a$, $a+b$\r\n\r\nthen it is aritmetic, now for geometric\r\n\r\n$(a-b)(a+b)=a^2$\r\n$b=0$\r\n\r\ntherefore the sequence is a constant", "Solution_4": "i think there is such sequence that is both arithmetic and geometric since randomness and chaos exists.in my opinion everything has a sequence and that sequence must be arithmetical,geometrical,.....etc, loads of them in one sequence,\r\nthat makes it difficult for us to predict future.", "Solution_5": "I admit to being somewhat baffled as to the relevance of \"chaos,\" \"randomness,\" or \"predict[ing the] future\" to this question. \"Arithmetic\" and \"geometric\" are not some mystical phrases, the extent of which are unknown and unknowable -- they mean something very specific and well-defined. The post seems particularly odd because it follows directly a proof that the only arithmetic-geometric sequences with 3 or more terms are constant.", "Solution_6": "How is something like $\\frac{1}{2}, \\frac{2}{4}, \\frac{3}{8}, ..., \\frac{n}{2^n}$ categorized?", "Solution_7": "arithmetic/geometric." } { "Tag": [ "inequalities", "inequalities proposed" ], "Problem": "If there be three real nos. a,b,c lying in the interval [0,1], prove that\r\n\r\n [b] [a/(b+c+1)]+[b/(c+a+1)]+[c/(a+b+1)]+(1-a)(1-b)(1-c) is less than equal to 1.[/b]", "Solution_1": "[quote=\"Sunkern_sunflora\"]If there be three real nos. a,b,c lying in the interval [0,1], prove that\n\n [b] [a/(b+c+1)]+[b/(c+a+1)]+[c/(a+b+1)]+(1-a)(1-b)(1-c) is less than equal to 1.[/b][/quote]\r\nSuppose that $ a \\leq b \\leq c$\r\nthen $ \\frac{a}{b\\plus{}c\\plus{}1}\\plus{}\\frac{b}{c\\plus{}a\\plus{}1}\\plus{}\\frac{c}{a\\plus{}b\\plus{}1} \\leq \\frac{a\\plus{}b\\plus{}c}{a\\plus{}b\\plus{}1}$\r\n$ (1\\minus{}a)(1\\minus{}b)(1\\plus{}a\\plus{}b) \\leq 1$\r\nso $ (1\\minus{}a)(1\\minus{}b) \\leq \\frac{1}{1\\plus{}a\\plus{}b}$\r\n=>$ (1\\minus{}a)(1\\minus{}b)(1\\minus{}c) \\leq \\frac{1\\minus{}c}{1\\plus{}a\\plus{}b}$\r\n=>$ \\sum_{cyc} \\frac{a}{b\\plus{}c\\plus{}1} \\plus{}(1\\minus{}a)(1\\minus{}b)(1\\minus{}c) \\leq \\frac{a\\plus{}b\\plus{}c}{a\\plus{}b\\plus{}1}\\plus{}\\frac{1\\minus{}c}{a\\plus{}b\\plus{}1}\\equal{}1$\r\nDone!", "Solution_2": "Oh! That was a beauty chien than! Just I didn't understand the very first step, i.e.\r\n [a/(b+c+1)]+[b/(c+a+1)]+[c/(a+b+1)] < [(a+b+c)/(a+b+1)]\r\nCould you explain it?\r\nHere's another inequality for you, chien than!\r\n\r\n[i]Prove, for positive reals, [/i] [b](ab)^1/2+(cd)^1/2 < [(a+d)(b+c)]^1/2[/b] :wink:", "Solution_3": "[quote=\"Sunkern_sunflora\"]Oh! That was a beauty chien than! Just I didn't understand the very first step, i.e.\n [a/(b+c+1)]+[b/(c+a+1)]+[c/(a+b+1)] < [(a+b+c)/(a+b+1)]\nCould you explain it?\n[/quote]\r\n\r\nnote that $ a\\leq b\\leq c$ so:\r\n\r\n$ b\\plus{}c\\plus{}1\\geq a\\plus{}b\\plus{}1\\Rightarrow\\frac{a}{b\\plus{}c\\plus{}1}\\leq\\frac{a}{a\\plus{}b\\plus{}1}$\r\n\r\n$ c\\plus{}a\\plus{}1\\geq a\\plus{}b\\plus{}1\\Rightarrow\\frac{b}{c\\plus{}a\\plus{}1}\\leq\\frac{b}{a\\plus{}b\\plus{}1}$\r\n\r\n$ \\Rightarrow\\frac{a}{b\\plus{}c\\plus{}1}\\plus{}\\frac{b}{c\\plus{}a\\plus{}1}\\plus{}\\frac{c}{a\\plus{}b\\plus{}1}\\leq\\frac{a\\plus{}b\\plus{}c}{a\\plus{}b\\plus{}1}$", "Solution_4": "[quote=\"Sunkern_sunflora\"][i]Prove, for positive reals, [/i] [b](ab)^1/2+(cd)^1/2 < [(a+d)(b+c)]^1/2[/b] :wink:[/quote]\r\n\r\nI think it should be:\r\n\r\n$ \\sqrt{ab}\\plus{}\\sqrt{cd}\\leq\\sqrt{(a\\plus{}d)(b\\plus{}c)}$\r\n\r\nanyway,by squaring both sides we have to prove that:\r\n\r\n$ ab\\plus{}cd\\plus{}2\\sqrt{abcd}\\leq (a\\plus{}d)(b\\plus{}c)$\r\n\r\n$ \\iff 2\\sqrt{abcd}\\leq ac\\plus{}bd$\r\n\r\nwhich is a directly consequence of AM-GM." } { "Tag": [], "Problem": "Can you find a formula for the sum from i=1 to i=n of\r\n\r\n$\\Sigma\\frac{1}{(i)(i+1)(i+2)}$", "Solution_1": "Try decomposing it into partial fractions and then telescoping the terms to simplify things.", "Solution_2": "Yes, telescoping is one of my favorite algebraic tools.\r\n\r\nIn case you don't know what that is, its when you write out a sum, and lots of terms cancel, leaving like some amount of terms to add that you can deal with, like\r\n\r\n$(a-b)+(b-c)+(c-d)$\r\n\r\nor something.", "Solution_3": "And in general we know what\r\n\r\n$\\sum_{i=1}^{\\n}\\frac{1}{(i)(i+1)(i+2)\\cdots(i+m)}$\r\n\r\nis. Its one of the more well know formulas.", "Solution_4": "See [url]http://www.mathlinks.ro/Forum/viewtopic.php?t=76327[/url]." } { "Tag": [ "inequalities", "inequalities unsolved" ], "Problem": "Let $ a,b,c$ be positive numbers such that $ a \\plus{} b \\plus{} c \\equal{} 1$. Prove that $ (1 \\plus{} a)\\sqrt {\\frac {1 \\minus{} a}{a}} \\plus{} (1 \\plus{} b)\\sqrt {\\frac {1 \\minus{} b}{b}} \\plus{} (1 \\plus{} c)\\sqrt {\\frac {1 \\minus{} c}{c}}\\geq\\frac {3\\sqrt {3}}{4} \\frac {(1 \\plus{} a)(1 \\plus{} b)(1 \\plus{} c)}{\\sqrt {(1 \\minus{} a)(1 \\minus{} b)(1 \\minus{} c)}}$ :?:", "Solution_1": "Anyone has a solution...it looks rather difficult...", "Solution_2": "It's very easy :rotfl:", "Solution_3": "[quote=\"thepunisher123\"]It's very easy :rotfl:[/quote]\r\n\r\nI hope you can post your solution....Thanks in advance." } { "Tag": [ "inequalities unsolved", "inequalities" ], "Problem": "For every positive numbers $ a,b,c,x,y,z$, prove that\r\n\r\n$ \\sqrt[3]{a(b\\plus{}1)yz}\\plus{}\\sqrt[3]{b(c\\plus{}1)zx}\\plus{}\\sqrt[3]{c(a\\plus{}1)xy}\\leq \\sqrt[3]{(a\\plus{}1)(b\\plus{}1)(c\\plus{}1)(x\\plus{}1)(y\\plus{}1)(z\\plus{}1)}$", "Solution_1": "[quote=\"radio\"]For every positive numbers $ a,b,c,x,y,z$, prove that\n\n$ \\sqrt [3]{a(b \\plus{} 1)yz} \\plus{} \\sqrt [3]{b(c \\plus{} 1)zx} \\plus{} \\sqrt [3]{c(a \\plus{} 1)xy}\\leq$\n$ \\leq\\sqrt [3]{(a \\plus{} 1)(b \\plus{} 1)(c \\plus{} 1)(x \\plus{} 1)(y \\plus{} 1)(z \\plus{} 1)}$[/quote]\r\nSee here:\r\nhttp://www.artofproblemsolving.com/Forum/viewtopic.php?t=89595" } { "Tag": [ "inequalities", "function", "logarithms", "inequalities unsolved" ], "Problem": "Prove the inequality :$\\frac{1}{\\sqrt{1+x^{2}}}+\\frac{1}{\\sqrt{1+y^{2}}}\\leq \\frac{2}{\\sqrt{1+xy}}$\r\nfor $0\\leq x,y\\leq 1$", "Solution_1": "Russia 2000. Nice,very nice.", "Solution_2": "$f(x,y)={1 \\over \\sqrt{1+xy}}$ in $[0,1]^2$\r\n${d^2f \\over dxdy}={1-xy \\over 4(xy+1)^2\\sqrt{xy+1}}\\geq 0 $\r\n\r\nso we can use Tschebyschev inequality:\r\n$f(x,y)+f(x,y)\\geq f(x,x)+f(y,y)$\r\n \r\nOr i have mistake in my calculation", "Solution_3": "[quote=\"erdos\"]Prove the inequality :$\\frac{1}{\\sqrt{1+x^{2}}}+\\frac{1}{\\sqrt{1+y^{2}}}\\leq \\frac{2}{\\sqrt{1+xy}}$\nfor $0\\leq x,y\\leq 1$[/quote]\r\nActually, if I correctly remember we proved somewhere on the forum that it holds for $xy\\leq 1$. Try it! :) It is more hard, in my opinion, but who knows ;)", "Solution_4": "A trigonometric substitution $x=tg\\alpha,y=tg\\beta$ would be very helpful.Between can we use Weiershtrass theorem that the extremum must be at the endpoints and check putting just the values $(x,y)=(1,1);(1,0);(0,0).$", "Solution_5": "An other way : :lol: \r\nfor $x=0$ or $y=0$ the inequality is obvious\r\nsuppose $00.\n\nSo, from Jensen for every u,v>0:\n\n\\[\\frac{f(u)+f(v)}{2}\\gef(\\frac{u+v}{2})\\] or\n\\[\\frac{1}{\\sqrt{1+e^{-2u}}}+\\frac{1}{\\sqrt{1+e^{-2v}}}\\ge\\frac{1}{\\sqrt{1+e^{-(u+v)}}}\\]\n\nFor $u=-\\ln x, v=-\\ln y$, we get that:\n\n\\[\\frac{1}{\\sqrt{1+e^{2\\ln x}}}+\\frac{1}{\\sqrt{1+e^{2\\ln y}}}\\ge\\frac{1}{\\sqrt{1+e^{\\ln x +\\ln y}}}\\] or\n\\[\\frac{1}{\\sqrt{1+e^{\\ln x^2}}}+\\frac{1}{\\sqrt{1+e^{2\\ln y^2}}}\\ge\\frac{1}{\\sqrt{1+e^{\\ln(xy)}}}\\] or\n\\[ \\frac{1}{\\sqrt{1+x^{2}}}+\\frac{1}{\\sqrt{1+y^{2}}}\\leq\\frac{2}{\\sqrt{1+xy}} \\]", "Solution_11": "http://www.artofproblemsolving.com/Forum/viewtopic.php?f=151&t=576617\nLet $a, b $ be positive real numbers such that $ab\\ge 3$ , prove that\\[\\frac{1}{\\sqrt{1+a^2}}+\\frac{1}{\\sqrt{1+b^2}}\\ge \\frac{2}{\\sqrt{1+ab}}.\\]", "Solution_12": "[quote=erdos]Prove the inequality :$$\\frac{1}{\\sqrt{1+x^{2}}}+\\frac{1}{\\sqrt{1+y^{2}}}\\leq \\frac{2}{\\sqrt{1+xy}}$$\nfor $0\\leq x,y\\leq 1$[/quote]\nLet $a,b\\in [0,\\frac{1}{2}].$ Prove that$$\\frac{1}{\\sqrt{1+2a^2}}+\\frac{1}{\\sqrt{1+2b^2}}\\leq\\frac{2}{\\sqrt{1+2ab}}$$\n[size=50]p/6980885324[/size]", "Solution_13": "I think Jensen kills the inequality. It is not difficult to see:\n\n$\\frac{1}{\\sqrt{1+ xy}} \\geq \\frac{1}{\\sqrt{1+ (\\frac{x+y}{2})^2}}$\n\nBecause $(x + y)^2 \\geq 4xy$ and \n\n $ \\frac{1}{2.\\sqrt{1+ x^2}} + \\frac{1}{2.\\sqrt{1+ y^2}} \\leq \\frac{1}{\\sqrt{1+ (\\frac{x+y}{2})^2}} $\n\nBy Jensen taking $f(x) = \\frac{1}{\\sqrt{1+ x^2}}$\n\nthus $f'(x) = \\frac{- x}{\\sqrt{(1+ x^2)^3}}$\n\nAnd finaly\n\n $f''(x) = - \\frac{x^6 + 3x^4 + 1}{\\sqrt{(1+ x^2)^9}} < 0 $ obviuosly." } { "Tag": [ "vector", "trigonometry", "Pythagorean Theorem", "geometry" ], "Problem": "can you help me to trancelate the numbers to polar form?\r\n \r\n1) 1+j\r\n2) -4+3j\r\n3) (1+j)(-4+3j)\r\n4) e^(j*\u03c0/4) + 2e^(-j*\u03c0/4)\r\n5) e^j + 1\r\n\r\n\r\n thank you!!", "Solution_1": "What is j?", "Solution_2": "I think it is the unit vector $ \\hat{\\mathbf{j}}$.", "Solution_3": "It may as well be $ i$.\r\n\r\n[hide=\"Hints\"]1. The argument (angle) should be pretty obvious for this one. Pythagorean theorem for the radius.\n2. Same thing, though you'll need either the answer in terms of $ \\arctan$ or a calculator.\n3. De Moivre's Theorem. The answer to this is obtained if you multiply the radii of the answers to 1 and 2 and add their arguments.\n4. Simplify it into $ a \\plus{} bi$ form.\n5. $ \\cos 1$ and $ \\sin 1$ ($ 1$ radian) are pretty much meaningless numbers, so I'd put the answer in terms of them.[/hide]", "Solution_4": "In electrical engineering and some other applied fields, the imaginary unit is often called $ j$ instead of $ i$ to avoid confusion with current.\r\n\r\nIn specialized cases, $ \\iota$ is sometimes seen to denote the imaginary unit.", "Solution_5": "thank you a lot! Can you help me to transform please!!", "Solution_6": "A complex number in cartesian form is given by $ a \\plus{} ib$. Now, looking at this number in the complex plane as an oriented vector from the origin to the point $ a \\plus{} ib$, then its polar form is given $ \\sqrt{a^2\\plus{}b^2} \\cdot cis(\\theta)$, where $ cis$ means $ \\cos \\theta \\plus{} i \\sin \\theta$, and the angle $ \\theta$ is defined as $ \\theta \\equal{} \\arctan \\frac{b}{a}$, for a > 0, and as $ \\theta \\equal{} \\pi \\plus{} \\arctan \\frac{b}{a}$, for a < 0.", "Solution_7": "1) 1+j \r\n2) -4+3j \r\n3) (1+j)(-4+3j) \r\n4) e^(j*\u03c0/4) + 2e^(-j*\u03c0/4) \r\n5) e^j + 1 \r\n \r\n\r\n\r\n\r\nI would really appreciate your help on exersise above.\r\nAny suggestions-entries are most welcome.", "Solution_8": "???\r\n\r\nWe've given you plenty of help. We're not going to do the problems for you if that's what you're expecting." } { "Tag": [ "algebra", "polynomial", "induction", "function", "factorial", "abstract algebra", "number theory" ], "Problem": "If the polynom $ ax^{3} \\plus{} bx^{2} \\plus{} cx \\plus{} d$ is divisble by 5 for all x prove that $ 5|a,b,c,d$ :)", "Solution_1": "[quote=\"Elio (n)\"]If the polynom $ ax^{3} \\plus{} bx^{2} \\plus{} cx \\plus{} d$ is divisble by 5 for all x prove that $ 5|a,b,c,d$ :)[/quote]\r\nhi $ p(x) \\equal{} ax^3 \\plus{} bx^2 \\plus{} cx \\plus{} d$ \r\n$ \\forall x \\in R p(x)\\equiv 0(5)$ so \r\n$ p(0) \\equal{} 0(5)$ ==> $ d \\equal{} 0(5)$ \r\n$ p(2) \\equal{} 0(5)$ ==> $ 4a \\plus{} 2b \\plus{} c \\equal{} 0(5)$ (1)\r\n$ p(1/2) \\equal{} 0(5)$ ==> $ a \\plus{} 2b \\plus{} 4c \\equal{} 0(5)$ (2) \r\nfrom (1) and (2) $ b \\equal{} 0(5)$ and $ a \\minus{} c \\equal{} 0(5)$ \r\n$ p(1) \\equal{} 0(5)$==> $ a \\plus{} b \\plus{} c \\equal{} 0(5)$ ==> $ (a \\plus{} c) \\equal{} 0(5)$ \r\nfinally 5/(a.b.c.d)", "Solution_2": "What if we try to generalize?\r\n\r\nGiven a positive integer $ k$ and a polynomial $ P_n(x) \\equal{} a_0x^n \\plus{} a_1x^{n \\minus{} 1} \\plus{} \\dots \\plus{} a_{n \\minus{} 1}x \\plus{} a_n$ with integer coefficients such that $ k\\mid P_n(x)$ for all $ x$, prove that $ k\\mid a_i\\,\\forall i$.\r\n\r\nLemma: If $ k\\mid mn$ for all $ m$ then $ k\\mid n$.\r\n\r\nProof: Choose an $ m$ such that $ (k,m) \\equal{} 1$ and the conclusion follows.\r\n\r\nLet us proceed by induction on the degree of the polynomial. This is clearly true for a linear polynomial, since $ k\\mid P_1(0) \\equal{} a_1$ and so $ k\\mid a_0x\\,\\forall x\\implies k\\mid a_0$, by the lemma. Now assume that it is true for any polynomial $ P_{n \\minus{} 1}(x)$ of degree $ n \\minus{} 1$. Then $ k\\mid P_n(0) \\equal{} a_n$ implies $ k\\mid a_0x^n \\plus{} a_1x^{n \\minus{} 1} \\plus{} \\dots \\plus{} a_{n \\minus{} 1}x\\,\\forall x$. But this implies $ k\\mid x(a_0x^{n \\minus{} 1} \\plus{} a_1x^{n \\minus{} 2} \\plus{} \\dots \\plus{} a_{n \\minus{} 2}x \\plus{} a_{n \\minus{} 1}) \\equal{} xP_{n \\minus{} 1}(x)$. Hence by the lemma we conclude that $ k\\mid P_{n \\minus{} 1}(x)$ for all $ x$ and thus by the inductive hypothesis $ k\\mid a_i$ for $ i \\equal{} \\overline{0,n \\minus{} 1}$. Therefore $ k\\mid a_i$ for all $ i \\equal{} \\overline{0,n}$.\r\n\r\nEDIT: I think we can generalize even further. \r\n\r\nIf $ k\\mid P_n(x)$ for $ n \\plus{} 1$ consecutive integer values of $ x$ then $ k\\mid a_i\\,\\forall i$.\r\n\r\nThe proof should be similar I guess.", "Solution_3": "[quote=\"nayel\"]Given a positive integer $ k$ and a polynomial $ P_n(x) \\equal{} a_0x^n \\plus{} a_1x^{n \\minus{} 1} \\plus{} \\dots \\plus{} a_{n \\minus{} 1}x \\plus{} a_n$ with integer coefficients such that $ k\\mid P_n(x)$ for all $ x$, prove that $ k\\mid a_i\\,\\forall i$.[/quote]\r\nThis is far too strong a result. Let $ k \\equal{} r!$ and let $ P(x) \\equal{} x(x \\minus{} 1)...(x \\minus{} (r \\minus{} 1))$. Similarly, let $ k \\equal{} p$ (a prime) and let $ P(x) \\equal{} x^p \\minus{} x$. (Your proof is invalid because $ P_{n \\minus{} 1}(x)$ is not a constant, so your lemma is not applicable.)\r\n\r\nIn generalizing this result, it is essential to compare $ k$ to $ n$. Restricting $ k$ to prime values, the problem is this: we are given a polynomial with $ p$ roots in $ \\mathbb{Z}_p$ and we wish to prove that it is the zero polynomial. Since $ P(x) \\equal{} x^p \\minus{} x$ is a polynomial with $ p$ roots that is [b]not[/b] the zero polynomial, we know that $ n < p$ is a necessary condition. Now, can you prove that it is sufficient? How about generalizing to non-prime values of $ k$?\r\n\r\nEdit: The necessary lemma for prime values of $ k$ is [url=http://en.wikipedia.org/wiki/Lagrange%27s_theorem_%28number_theory%29]Lagrange's theorem[/url].", "Solution_4": "I see. Thank you very much for pointing it out. :)", "Solution_5": "The problem for non-prime values of $ k$ is quite interesting. The fact that Lagrange's theorem fails makes it harder to get a grip on $ n$, although we know that for $ k \\equal{} r!$ the smallest $ n$ that does not satisfy the problem criterion (which is $ r$) can be quite small.\r\n\r\n[hide=\"Further discussion\"] First of all, an explicit counterexample for some $ n$ for any value of $ k$ is as follows. Let $ e$ be the largest exponent of a prime in the prime factorization of $ k$. Then\n\n$ P(x) \\equal{} \\left( x^{\\lambda(k)} \\minus{} 1 \\right) x^e$\n\nis divisible by $ k$ for all values of $ x$ (where $ \\lambda(k)$ is the [url=http://en.wikipedia.org/wiki/Carmichael_function]Carmichael function[/url]), so we know that $ n < \\lambda(k) \\plus{} e$. The behavior of $ \\lambda(k)$ depends strongly on the prime factorization of $ k$, so while this bound is sharp when $ k$ is prime it does not guarantee much when $ k$ is composite. For example, when $ k \\equal{} 5! \\equal{} 2^3 \\cdot 3 \\cdot 5$ we get $ \\lambda(k) \\plus{} e \\equal{} 4 \\plus{} 3 \\equal{} 7$ but we know that $ 5$ is a stronger upper bound.\n\nHaving the factorial example at hand presents a second idea. Let $ M$ be the smallest positive integer such that $ k | M!$. Then\n\n$ P(x) \\equal{} x(x \\minus{} 1)...(x \\minus{} (M \\minus{} 1))$\n\nis divisible by $ k$ for all integer $ x$. Then $ n < M$. I believe this bound is tight when $ k \\equal{} M!$ (the case we considered earlier), and it is also tight when $ k$ is prime (because $ M \\equal{} p$), but the growth rate of $ M$ when $ k$ is not of either form again strongly depends on the prime factorization of $ k$ and I am again unsure whether it is tight in general. [/hide]\r\nThe next step after the case $ k \\equal{} p$ is the case $ k \\equal{} p^m$, which I think might be enough to solve the composite case." } { "Tag": [ "modular arithmetic", "number theory unsolved", "number theory" ], "Problem": "Determine all quadruples $ (a,b,c,d)$ of nonnegative integers satisfying:\r\n\r\n$ a^2\\plus{}b^2\\plus{}c^2\\plus{}d^2\\equal{}a^2 b^2 c^2$.", "Solution_1": "[quote=\"moldovan\"]Determine all quadruples $ (a,b,c,d)$ of nonnegative integers satisfying:\n\n$ a^2 \\plus{} b^2 \\plus{} c^2 \\plus{} d^2 \\equal{} a^2 b^2 c^2$.[/quote]\r\n\r\nClearly $ (a,b,c,d)\\equal{}(0,0,0,0)$ is a solution.\r\n\r\nAssume that other solution $ (a_1,b_1,c_1,d_1)$ exists with $ \\gcd(a_1,b_1,c_1,d_1)\\equal{}1$.\r\n\r\nThen any one of $ a_1,b_1,c_1$ even $ \\Rightarrow$ all of $ a_1,b_1,c_1,d_1$ even after mod 4 both sides of the equation, contradiction as now $ 2|\\gcd(a_1,b_1,c_1,d_1)$.\r\n\r\nall $ a_1,b_1,c_1$ odd $ \\Rightarrow a_1^2\\plus{}b_1^2\\plus{}c_1^2\\plus{}d_1^2 \\equiv 3\\plus{}d_1^2 \\equiv 3,0 \\pmod{4}$, contradiction as $ a_1^2b_1^2c_1^2 \\equiv 1 \\pmod{4}$\r\n\r\nSo $ (a,b,c,d)\\equal{}(0,0,0,0)$ is the only solution." } { "Tag": [ "probability" ], "Problem": "Two dice are thrown. What is the probability that the product\nof the two numbers is a multiple of 5? Express your answer as a common fraction.", "Solution_1": "That means one of the numbers is 5.\r\nThe other number we can choose in 6 ways, and we divide by 2 because there's no matter of order.\r\n15", "Solution_2": "The probability of no 5s is 5/6 ^2. This its 1-25/36=11/36", "Solution_3": "don't get something- if you just think about it, as long as there is a 5 for one dice, the product will automatically be a multiple of 5. So the prob is 1/6 then accounting for both its [u][i][b]1/3[/b][/i][/u]", "Solution_4": "Because you are overcounting when we get 5 on both dice.", "Solution_5": "Let's use...\n[b][size=150][center]BRUTE FORCE[/size][/center][/b]\n[hide]Dice combinations for $5$: 1 & 5 and 5 & 1 (2)\n Dice combinations for $10$: 2 & 5 and 5 &1 (2)\n Dice combinations for $15$: 3 & 5 and its inverse (2)\n Dice combinations for $20$: 5 & 4 & inverse (2)\n Dice combinations for $25$: 5 & 5 (1)\n Dice combinations for $30$: 5 & 6 & inverse (2)\n Add them all, and you get 11 in total.\n We have $6*6$ total cases, which gives us $[box]11/36[/box]$ cases[/hide]", "Solution_6": "[quote=Tuatramxo]Let's use...\n[b][size=150][center]BRUTE FORCE[/size][/center][/b]\n[hide]Dice combinations for $5$: 1 & 5 and 5 & 1 (2)\n Dice combinations for $10$: 2 & 5 and 5 &1 (2)\n Dice combinations for $15$: 3 & 5 and its inverse (2)\n Dice combinations for $20$: 5 & 4 & inverse (2)\n Dice combinations for $25$: 5 & 5 (1)\n Dice combinations for $30$: 5 & 6 & inverse (2)\n Add them all, and you get 11 in total.\n We have $6*6$ total cases, which gives us $[box]11/36[/box]$ cases[/hide][/quote]\n\nAgreed. ", "Solution_7": "P.S. Do you mean 5 & 2 & 5 & 2?" } { "Tag": [ "calculus", "integration", "function", "calculus computations" ], "Problem": "Find the indefinite integral $ \\int \\frac{x^3}{(x\\minus{}1)^3(x\\minus{}2)}\\ dx$.", "Solution_1": "with writing $ \\frac {x^3}{(x \\minus{} 1)^{3}(x \\minus{} 2)} \\equal{} \\frac {ax^{2} \\plus{} bx \\plus{} c}{(x \\minus{} 1)^{3}} \\plus{} \\frac {d}{x \\minus{} 2}$ and finding $ a,b,c,d$ :\r\n$ a \\equal{} \\frac { \\minus{} 2}{7} , b \\equal{} \\frac {27}{7} , c \\equal{} \\frac {\\minus{}9}{14} , d \\equal{} \\frac {9}{7}.$\r\nthen we have $ \\int (\\frac {\\frac { \\minus{} 2}{7}x^{2} \\plus{} \\frac {27}{7}x \\plus{} \\frac {\\minus{}9}{14}}{(x \\minus{} 1)^{3}} \\plus{} \\frac {\\frac {9}{7}}{x \\minus{} 2})\\ dx \\equal{} I_{1} \\plus{} I_{2} .$\r\n$ I_{1} \\equal{} \\int \\frac {\\frac { \\minus{} 2}{7}x^{2} \\plus{} \\frac {27}{7}x \\plus{} \\frac {27}{7}}{(x \\minus{} 1)^{3}}\\ dx \\equal{} \\frac { \\minus{} 2}{21}\\ln(x \\minus{} 1)^3 \\minus{} \\frac {69}{21(x \\minus{} 1)} \\plus{} \\frac {177}{42(x \\minus{} 1)^2}$ \r\nand $ I_{2} \\equal{} \\int \\frac {\\frac {9}{7}}{x \\minus{} 2}\\ dx \\equal{} \\frac {9}{7}\\ln(x \\minus{} 2).$\r\nright ?", "Solution_2": "Regrettably that is incorrect.", "Solution_3": "i think now it is right ... if not plz say my fault and correct answer", "Solution_4": "Still incorrect.", "Solution_5": "I got 8ln|x-2|-ln|x-1|+4/(x-1)", "Solution_6": "Partial fraction decomposition gives a function of the form:\r\n\r\n$ \\frac{A}{(x-1)^3}+\\frac{B}{(x-1)^2}+\\frac{C}{x-1}+\\frac{D}{x-2} = \\frac{x^3}{(x-1)^3(x-2)}$\r\n\r\nPutting over a common denominator and solving the resulting linear system gives:\r\n\r\n$ A=-1$\r\n$ B=-4$\r\n$ C=-7$\r\n$ D=8$\r\n\r\nThe integral is then:\r\n\r\n$ \\int{\\frac{x^3}{(x-1)^3(x-2)}=\\frac{1}{2(x-1)^2}+\\frac{4}{x-1}-7\\ln|x-1|+8\\ln|x-2|}$" } { "Tag": [ "number theory open", "number theory" ], "Problem": "Prove there exists infinitely many squares not of the form 4xy+y+x. I think this is the same as proving that there exists infinitely many primes of the form kk+1 so a simple proof may not be possible.", "Solution_1": "Closely related to http://www.mathlinks.ro/Forum/viewtopic.php?t=90831 ;)\r\n\r\nEqivalent this time: $(2z)^2+1 = (4x+1)(4y+1)$.\r\nFrom that point, it's easy to see that is equivalent to $\\infty$ many primes of type $(2z)^2+1$." } { "Tag": [], "Problem": "How many ordered triples of primes $ (a,b,c)$ exist such that $ a\\le b\\le c$ and $ a\\plus{}b\\plus{}c\\equal{}26$?", "Solution_1": "[quote=\"GameBot\"]How many ordered triples of primes $ (a,b,c)$ exist such that $ a\\le b\\le c$ and $ a \\plus{} b \\plus{} c \\equal{} 26$?[/quote]\r\n\r\nOne of the primes[b]must[/b] be 2 or else you couldn't have an even sum. Now we need two primes summing to 24. They are:\r\n\r\n(5,19)\r\n(7,17)\r\n(11,13)\r\n\r\nSo we have:\r\n\r\n$ 2 \\le 5 \\le 19\\\\\r\n2 \\le 7 \\le 17\\\\\r\n2 \\le 11 \\le 13$\r\n\r\nThus our answer is $ \\boxed{3}$" } { "Tag": [], "Problem": "(US) I just heard that the NSA is coupling up with companies like Horizon and spying on Americans' phone calls...We've got do have some sort of privacy around us, don't you think?", "Solution_1": "You can't. For all that bloody US Govt knows you could well be plotting against it. (sorry for the expletive)" } { "Tag": [ "number theory proposed", "number theory" ], "Problem": "it's an easy problem :\r\nif we have :\r\n\r\nxy / (x,y)^2 = z \r\nand xz / (x,z)^2 = y \r\nthen prove that : (y / (x,y) , (x,y) ) = 1", "Solution_1": "Denote $(x,y)=d, x=da, y=db, (a,b)=1$\r\n\r\nThen $xy=z(x,y)^{2}\\Rightarrow adbd=zd^{2}\\Rightarrow z=ab$\r\n\r\n$xz=y(x,z)^{2}\\Rightarrow adab=bd(ad,ab)^{2}\\Rightarrow (b,d)=1\\Rightarrow (\\frac{y}{(x,y)}, (x,y) ) = 1$" } { "Tag": [ "algebra unsolved", "algebra" ], "Problem": "What are the possible lenths of horizontal line segments that start and finish on the graph of $ x^3 \\minus{} x$", "Solution_1": "It is $ 2$. Let $ f(x) \\equal{} x^3 \\minus{} x$. The line that passes through the points $ (x, f(x))$ and $ (y, f(y))$ is horizontal iff $ f(x) \\equal{} f(y)$. Then its lenght is $ |x \\minus{} y|$. Suppose by absurd $ |x \\minus{} y| > 2$ and $ x^3 \\minus{} x \\equal{} y^3 \\minus{} y$. We have\r\n\r\n$ x^3 \\minus{} y^3 \\equal{} x \\minus{} y \\Longleftrightarrow x^2 \\plus{} xy \\plus{} y^2 \\equal{} 1 \\Longleftrightarrow (x \\minus{} y)^2 \\plus{} 3xy \\equal{} 1$\r\n\r\nWe have $ (x \\minus{} y)^2 > 4 \\Longrightarrow 3xy < \\minus{} 3$. Then $ \\minus{} 1 < \\frac {1}{xy} \\equal{} 1 \\plus{} \\frac {x}{y} \\plus{} \\frac {y}{x} \\Longrightarrow \\frac {x}{y} \\plus{} \\frac {y}{x} > \\minus{} 2$. So $ x$ and $ y$ have the same sign (otherwise we would have $ \\frac {x}{y} \\plus{} \\frac {y}{x} \\le \\minus{} 2$). But this contradicts $ xy < 0$.\r\n\r\nWe have $ |x \\minus{} y| \\equal{} 2$ for $ x \\equal{} 1$ and $ y \\equal{} \\minus{} 1$.", "Solution_2": "If you look at the graph of $ x^3\\minus{}x$, you will notice it is increasing on $ (\\minus{}\\infty,\\minus{}\\frac1{\\sqrt{3}})$ and $ (\\frac1{\\sqrt{3}},\\infty)$ and decreasing otherwise. So $ x^3\\minus{}x\\equal{}a$ has three real solutions when $ a\\in(\\minus{}\\frac{2\\sqrt{3}}9,\\frac{2\\sqrt{3}}9)$ and has two solutions when $ a\\equal{}\\pm\\frac{2\\sqrt{3}}9$. My calculation is this line segments may have any length in $ (0,2]$." } { "Tag": [ "ARML", "geometry", "email" ], "Problem": "Has anyone from here gone to the arml tryouts of East MA 2006? If so post here!\r\nI did pretty well but I made stupid mistakes on two of the questions. like for example there was one qeustion that asked for the area of a circle, but instead I put down the radius...\r\n\r\n(This forum looks empty)", "Solution_1": "yea i took the tryout\r\n\r\ni got a 15 :( , which would have a been a decent score if the problems were actually hard\r\n\r\ni hate those problems were you have to use a lot of ingenuity to get to the last step but then out a nowhere there is an extraneous solution that wastes ur effort\r\n\r\ni hope the problems at the practices are harder\r\n\r\nhmm.. i hear lexington might not coem with us\r\n\r\nif so...[hide]omg we're screwed!!!!!!!!![/hide]\r\n\r\nwe're all from Ma in this forum anyways, so no fear in using real names, my name is Norman Yu, Boston Latin School, Grade 11", "Solution_2": "Heh, yeah, I'm from Lex, and we would have gladly come had someone told us about the tryout. hmm . . . \r\n\r\nBut anyways, we might have our coach just try us out for the team at Lex, so try to not discuss problems on the forum.\r\n\r\nWe also have an extremely small chance of forming our own ARML team, but that probably won't happen.\r\n\r\nOh, yeah, if we make the team are practices required, and when are practices?", "Solution_3": "Hmm.. steve xu and sway chen did ARML last year... they should;ve known\r\n\r\nmr curry sent the tryouts to mr rahman, so he ahs the problems and should be able to admiinister them\r\n\r\nask sway about the practice schedule, to lazy to type", "Solution_4": "Yeah, none of us unfortunately can go to the practices, so we can't try out.", "Solution_5": "o man, so no one from lex is going?!?!??", "Solution_6": "Nope. No one at all. Unless someone has a change of heart, which they probably won't.", "Solution_7": "hello, Norman :D\r\nIt's me, James Sun from Brookline high.\r\n\r\nMy school also didn't receive an email for the tryouts (on time), so I would have missed it if I did not hear about it. that's a shame too, because I know a few people at my school who would have performed just as well.. \r\n\r\nAnyways, norman, how do you find out that z = -15 is extraneous?\r\nAlso, I can't figure out the dollar chang problem, tell me why the answer is 9 (or tell me what the problem asked for)", "Solution_8": "well... we shouldn't discuss the solutions to the problems yet, but i dont really care\r\n\r\nyou figure out z=15 is extraneous because on of the equations was x^2+y^2+z^2 = 49 or something, if z=15, then x^2+y^2 will be negative which cant happen since its states x,y,z are real\r\n\r\nthe dollar change one is just counting... hard to explain thr\\ourougjly", "Solution_9": "Noah, the EMass ARML tryouts were announced at the end of the 2/2 MML meet at Westford Academy. I heard the announcement. I know the Worcester Academy kids heard it, because those of them trying out for WMass ARML asked what it was all about.", "Solution_10": "I stand corrected :blush:", "Solution_11": "Our school heard about it, but I thought it would be too hard to qualify. Turns out everyone from my school who went qualified, and I should have gone :?" } { "Tag": [ "ratio", "limit", "geometric series" ], "Problem": "I was wondering about the proof that the sum of an infinite geometric series being $\\frac{a}{1-r}$ where $a$ is the first term and $r$ is the common ratio. What is it?", "Solution_1": "Consider $S-Sr$", "Solution_2": "while considering infinite geo. sum you should always mention that the mod of common ratio is less than 1\r\ni.e. $\\mid r\\mid \\leq 1$", "Solution_3": "[quote=\"varun\"]while considering infinite geo. sum you should always mention that the mod of common ratio is less than 1\ni.e. $\\mid r\\mid \\leq 1$[/quote]\r\n\r\nor we can just stick with the general formula, $\\frac{a(1-r^{n})}{1-r}$.\r\n\r\nPersonally, I like to prove geometric with the identity $x^{n}-1 = (x-1)(x^{n-1}+x^{n-2}+\\dots+1)$", "Solution_4": "[quote=\"varun\"]while considering infinite geo. sum you should always mention that the mod of common ratio is less than 1\ni.e. $\\mid r\\mid \\leq 1$[/quote]\r\nWell, duh. Otherwise the terms would grow and their sum would be plus or minus infinity (depepnding on the value of $r$). And if $r=-1$, you can't really determine it.", "Solution_5": "Basically you take the formula for regular geometric series $\\frac{a(1-r^{n})}{1-r}$, take the limit as $n\\to \\infty$ where $|r|<1$. What happens is $r^{n}$ \"becomes\" $0$, and we are left with $\\frac{a}{1-r}$.", "Solution_6": "[quote=\"eryaman\"]Basically you take the formula for regular geometric series $\\frac{a(1-r^{n})}{1-r}$, take the limit as $n\\to \\infty$ where $|r|<1$. What happens is $r^{n}$ \"becomes\" $0$, and we are left with $\\frac{a}{1-r}$.[/quote]\r\nOr, using mathematical syntax: When $|r|<1$, $\\lim_{n\\to\\infty}{r^{n}}=0$. Therefore, $\\lim_{n\\to\\infty}{\\frac{a(1-r^{n})}{1-r}}=\\frac{a}{1-r}$.", "Solution_7": "Okay, thanks, everyone. I was wondering about it.", "Solution_8": "A proof without really using limits:\r\n\r\n[quote=\"manuel\"]Consider $S-Sr$[/quote]\r\n\r\nLet $S=a+ar+ar^{2}+ar^{3}+...$\r\nThen $Sr=ar+ar^{2}+ar^{3}+...$\r\nThus $S-Sr=a$\r\n$S(1-r)=a$\r\n$S=\\frac{a}{1-r}$\r\nQ.E.D.", "Solution_9": "the one I have learned goes:\r\n\r\nLet S be the sum of the series.\r\nLet r be the rate.\r\nLet a be the first term.\r\n\r\n$S = a+ar+ar^{2}+ar^{3}+...$\r\nFactor out the r from $ar+ar^{2}+...$\r\n\r\n$S = a+r(a+ar+ar^{2}+ar^{3}+...)$\r\n\r\nSince $a+ar+ar^{2}...$ has already been accepted as S, substitute S in for the quantity in parenthesis.\r\n\r\n$S = a+rS$\r\nSo $S-rS = a \\Rightarrow S(1-r) = a \\Rightarrow S = \\frac{a}{1-r}$\r\n\r\nI hope it makes sense :)", "Solution_10": "yes the above 2 posts show the most common technique for this problem, however, you have to be careful of the way that you manipulate $S$, by subtracting, etc, you implicitly assume that S has a finite value, which is only true iff $|r|<1$", "Solution_11": "Oops sorry I didn't include it in my proof! I had figured that all of the proofs above mine had made it clear that |r| must be less than one.", "Solution_12": "[quote=\"mathnerd314\"]A proof without really using limits:[/quote]\r\nI know all I do in the intermediate forum is whine about lack of rigour, but anyway: The very definition of an infinite sum depends on limits. You can not just add up an infinite amount of numbers with the traditional addition.", "Solution_13": "Yeah, you can't really get this proof without limits.\r\n\r\nHere's how I'd prove it:\r\n$S=a+ar+ar^{2}+ar^{3}+\\cdots$\r\n$Sr=ar+ar^{2}+ar^{3}+ar^{4}+\\cdots$\r\n\r\nSo $S-Sr=a-ar^{\\infty+1}$. Since $\\lim_{k\\to\\infty+1}ar^{k}=0$ (since $|r|<1$), we have $S-Sr=a$. The rest is pretty easy.", "Solution_14": "$\\infty+1$. Interesting. :wink: :D" } { "Tag": [ "number theory unsolved", "number theory" ], "Problem": "Find all $ m,n$ such that $ 1\\plus{}5.2^{m}\\equal{}n^{2}$", "Solution_1": "$ 5 \\cdot 2^m\\equal{}(n\\plus{}1)(n\\minus{}1), 2\\nmid n, (n\\plus{}1,n\\minus{}1)\\equal{}2$, so $ n\\plus{}1$ is the form $ \\{5\\cdot 2,5\\cdot 2^{m\\minus{}1},2,2^{m\\minus{}1}\\}$ that leads to $ n \\in \\{9,3,1,11\\}$, but $ (m,n)\\equal{}(4,9)$ is the unique solution." } { "Tag": [ "trigonometry", "algebra", "polynomial", "Pythagorean Theorem", "geometry", "trig identities", "Law of Cosines" ], "Problem": "A machine-shop cutting tool has the shape of a notched circle, as shown. The radius of the circle is $\\sqrt{50}$ cm, the length of $AB$ is 6 cm, and that of $BC$ is 2 cm. The angle $ABC$ is a right angle. Find the square of the distance (in centimeters) from $B$ to the center of the circle.\n[asy]\nsize(150); defaultpen(linewidth(0.65)+fontsize(11));\nreal r=10;\npair O=(0,0),A=r*dir(45),B=(A.x,A.y-r),C;\npath P=circle(O,r);\nC=intersectionpoint(B--(B.x+r,B.y),P);\ndraw(Arc(O, r, 45, 360-17.0312));\ndraw(A--B--C);dot(A); dot(B); dot(C);\nlabel(\"$A$\",A,NE);\nlabel(\"$B$\",B,SW);\nlabel(\"$C$\",C,SE);\n[/asy]", "Solution_1": "[hide]Because we are given a right angle, we look for ways to apply the Pythagorean Theorem. Extend a perpendicular from $O$ to $AB$ and label it $D$. Additionally, extend a perpendicular from $O$ to the line $BC$, and label it $E$. Let $OE=x$ and $OD=y$. We're trying to find $x^2+y^2$.\n\nApplying the Pythagorean Theorem, $OA^2 = OD^2 + AD^2$, and $OC^2 = EC^2 + EO^2$.\n\nThus, $(\\sqrt{50})^2 = y^2 + (6-x)^2$, and $(\\sqrt{50})^2 = x^2 + (y+2)^2$. We solve this system to get $x = 1$ and $y = 5$, resulting in an answer of $1^2 + 5^2 = 26$.[/hide]", "Solution_2": "interesting solution...I actually destroyed this problem w/ trig, creating a right triangle with the hypotenuse as the diameter etc.", "Solution_3": "Wouldn't it be$\\sqrt{26}$? That's also how it is on Kalva.", "Solution_4": "[quote=\"NeverOddOrEven\"]Wouldn't it be$\\sqrt{26}$? That's also how it is on Kalva.[/quote]\r\n\r\nFind the square of the distance.\r\n\r\nKalva changes the questions a little.", "Solution_5": "oh ok gotcha", "Solution_6": "Is this question from Contest Book 5?", "Solution_7": "With Pythagorean theorem we get $ |AC|\\equal{}\\sqrt{40}$\r\nLets say $ \\angle BAC\\equal{}\\beta$\r\n $ \\angle OAC\\equal{}\\alpha$\r\n $ \\angle OAB\\equal{}\\phi$\r\n\r\nThen by Law of Cosines we get:\r\n$ \\cos \\beta\\equal{}\\frac{4\\minus{}36\\minus{}40}{\\minus{}2*6*\\sqrt{40}}\\equal{}\\frac{3}{\\sqrt{10}} \\Rightarrow \\sin \\beta\\equal{}\\frac{1}{\\sqrt{10}}$\r\n\r\n$ \\cos \\alpha\\equal{}\\frac{50\\minus{}50\\minus{}40}{\\minus{}2*\\sqrt{50}*\\sqrt{40}}\\equal{}\\frac{1}{\\sqrt{5}} \\Rightarrow \\sin \\beta\\equal{}\\frac{2}{\\sqrt{5}}$\r\n\r\nThen $ \\cos \\phi\\equal{}\\cos (\\alpha\\minus{}\\beta)\\equal{}cos\\alpha \\cos\\beta\\plus{}sin\\alpha \\sin\\beta\\equal{}\\frac{5}{\\sqrt{50}}$\r\n\r\nFinally $ |OB|^2\\equal{}|OA|^2|AB|^2\\minus{}2|OA\\parallel{}AB|\\cos\\phi\\equal{}50\\plus{}36\\minus{}2*6*\\sqrt{50}*\\frac{5}{\\sqrt{50}}\\equal{}\r\n26$\r\nTherefore answer is 26.", "Solution_8": "Sorry for reviving the old thread, but how did you find the values for the sines of the angles?", "Solution_9": "Draw $ \\overline{AC}$ and $ \\overline{OC}$. Now it's Law of Cosines, and then $ \\sin x \\equal{} \\sqrt {1 \\minus{} \\cos^2x}$ in triangles (since $ 0^\\circ0$, so $y|652$. We try testing small values of $y$ first, and we see that $y=2$. When we divide the LHS by $(y-2)$, \\[ (y-2)(y^3+16y^2+101y+326)=0 \\]\nThe polynomial to the right of $(y-2)$ must have the bound $y<0$ because it has positive coefficients. Therefore, there are no more positive solutions for $y$. And if $y=2$, then $x=4$. Substituting $x$ back into $OB=\\sqrt{50-6x}$ gives $OB=\\sqrt{26}$. The answer is $OB^2=\\boxed{26}$[/hide]", "Solution_13": "^That's how I would have done it. :P", "Solution_14": "Refer to attached diagram.\n\nDrop perpendiculars from $O$ to $AC$ and $AB$ and call their feet $D$ and $E$, respectively. Also connect $C$ to $A$ and $O$, and $O$ to $A$ and $B$. We will find $AE$ in two different ways.\n\nObserve that: \\[\\triangle{ADP}\\sim \\triangle{ABC}\\sim \\triangle{OEP}\\quad (*)\\] By the Pythagorean Theorem, $AC$ $=\\sqrt{40}$. Further, since $OA=OC=\\sqrt{50}$, it follows that $\\triangle{AOC}$ is isosceles, therefore $AD=AC/2=\\tfrac{\\sqrt{40}}{2}$. Thus, going back to (*) we have $\\tfrac{AD}{AB}$ $=\\tfrac{AP}{AC}$ $\\Rightarrow AP$ $=\\tfrac{10}{3}$. Therefore, applying the Pythagorean Theorem to $\\triangle{ADP}$ yields $PD=\\tfrac{\\sqrt{40}}{6}$; in the same way, $\\triangle{ADO}$ gives $OD=\\sqrt{40}$. As a result, $OP=\\tfrac{5\\sqrt{50}}{6}$, and consequently (*) implies $\\tfrac{PE}{PD}=\\tfrac{OP}{AP}$, that is, $PE=\\tfrac{5}{3}\\Rightarrow AE=5$.\n\nNow, consider the right triangles $\\triangle{OAE}$ and $\\triangle{OBE}$. By the Pythagorean Theorem, \\[OA^2-AE^2=OB^2-(AB-AE)^2\\] \\[\\Leftrightarrow 50-AE^2=OB^2-(6-AE)^2\\] \\[\\Leftrightarrow AE=\\frac{86-OB^2}{12}.\\] Lastly, equating our two values for $AE$ we obtain $5=\\tfrac{86-OB^2}{12}$ or $OB^2=\\boxed{26}$.", "Solution_15": "Here's a solution with coordinates.\n\n[hide=\"Solution\"]\n\nLet $B$ be at $(0,0)$ and consequently $A(0,6)$ and $C(2,0)$ are on a circle defined by $(x-h)^2+(y-k)^2=50$.\n\\[h^2+(6-k)^2=h^2+k^2-12k+36=50 \\Rightarrow h^2+k^2-12k=14.\\]\n\\[(2-h)^2+k^2=h^2+k^2-4h+4=50 \\Rightarrow h^2+k^2-4h=46.\\]\n\\[\\Rightarrow 3k-h=8 \\Rightarrow h=3k-8.\\]\n\\[\\Rightarrow (3k-8)^2+k^2-12k=14 \\Rightarrow 10k^2-60k+50=0 \\Rightarrow (k-1)(k-5)=0 \\Rightarrow k=1, h=5\\] and $k$ is not $5$, which is obvious by referring to a diagram (I won't prove it).\n\nSo by the distant formula, the square of the distance is $5^2+1^2=26$.", "Solution_16": "This is AIME (1983).\nhttps://www.artofproblemsolving.com/wiki/index.php?title=1983_AIME_Problems#Problem_4", "Solution_17": "[hide=power of a point]\nExtend $AB$ through $B$ to intersect the circle at point $D$. and Extend $CO$ through $O$ to intersect the circle at $E$. Now, $\\angle CED = \\angle CAB$ because they intercept the same arc, and both $\\triangle CED$ and $\\triangle CAB$ are right, so $\\triangle CED \\sim \\triangle CAB$.\n\nThen, $\\dfrac{CE}{CA} = \\dfrac{ CD}{CB} \\implies \\dfrac{2\\sqrt{50}}{\\sqrt{40}} = \\dfrac{CD}{2} \\implies CD = \\sqrt{20}$. So, $BD = \\sqrt{20 - 4} = 4$.\n\nLet $x$ be the desired length $OB$. Extend segment $OB$ to intersect the circle twice at $F$ and $G$ ($F$ further from $B$ than $O$), and applying power of a point to segments $FG$ and $AD$ gives us $(FB)(BG) = (AB)(BD) \\implies (\\sqrt{50} + x)(\\sqrt{50} -x) = (6)(4) = 24 \\implies 50 - x^2 = 24 \\implies x^2 = \\boxed{26}$.\n[/hide]", "Solution_18": "There\u2019s been like 1@ revives..", "Solution_19": "[hide]Refer to scuffed diagram below. We start off by letting $O$ be the center of the circle, drawing $\\overline{OC}$ and $\\overline{OA}$, and dropping the perpendicular from $O$ to $AB$, with their intersection being $H$, and doing the same for $O$ and $BC$, with their intersection being $J$. The reason we do this is that we can now get $\\overline{AH}$ and $\\overline{BH}$ in terms of $\\overline{OH}$, so we can get an expression for $\\overline{AB}$ in terms of $\\overline{OH}$ which we know is equal to $6$. Therefore, let $\\overline{OH}=x$. Then, $\\overline{AH}=\\sqrt{50-x^2}$ and $\\overline{JB}=x$, so $\\overline{OJ}=\\overline{HB}=\\sqrt{50-(x+2)^2}$. So we have the equation $\\sqrt{50-x^2}+\\sqrt{50-x^2-4x-4}=6$. We can move $\\sqrt{50-x^2}$ to the other side and square to get $50-x^2-4x-4=50-x^2+36-12\\sqrt{50-x^2}$. Cancelling, isolating the square root, and squaring again, we get $x^2+20x+100=450-9x^2$, which we can solve to yield the solutions of $-7$ and $5$; $5$ clearly being the one we want to use in this situation. Therefore, $OB=\\sqrt{26}$ so our answer is $26$. [/hide]" } { "Tag": [ "geometry", "inradius", "circumcircle", "geometry proposed" ], "Problem": "Show that a point $P$ lies on the incircle $(I)$ of $\\triangle ABC$ if and only if\n\n\\[a \\cdot PA^2 \\plus{} b \\cdot PB^2 \\plus{} c \\cdot PC^2 \\equal{} (4R \\plus{} 2r)[\\triangle ABC] \\] \n$R,r$ denote the inradius and circumradius of $\\triangle ABC$ and $[\\triangle ABC]$ stands for its area.", "Solution_1": "This problem based on that identity(i will try to post proof later):\n$ \\sum_{cyclic}{XA^2\\cdot a}\\equal{}XI^2\\sum_{cyclic}{a}\\plus{}abc$\n\nso if $ P$ is a point in the inscribed cirumference of a $ \\triangle ABC$ then we have:\n$ aPA^2 \\plus{} bPB^2 \\plus{} cPC^2 \\equal{} r^2(a\\plus{}b\\plus{}c)\\plus{}abc\\equal{}2r[ABC]\\plus{}4[ABC]R\\equal{}(4R\\plus{}2r)[ABC]$\n\nif we have:\n$ aPA^2 \\plus{} bPB^2 \\plus{} cPC^2 \\equal{} (4R \\plus{} 2r)[ABC]$\n$ \\leftrightarrow$\n$ PI^2(a\\plus{}b\\plus{}c)\\plus{}abc\\equal{}4R[ABC]\\plus{}2r[ABC]$\n$ \\leftrightarrow$\n$ PI^2(a\\plus{}b\\plus{}c)\\plus{}abc\\equal{}abc\\plus{}r^2(a\\plus{}b\\plus{}c)$ \n$ \\leftrightarrow$\n$ PI\\equal{}r$\nSo point $ P$ is in the inscribed circumference of $ \\triangle ABC$", "Solution_2": "[quote=SaYaT]This problem based on that identity(i will try to post proof later):\n$ \\sum_{cyclic}{XA^2\\cdot a}\\equal{}XI^2\\sum_{cyclic}{a}\\plus{}abc$\n[/quote]\n\nIs there any proof of this identity without using the barycentic coordinates?", "Solution_3": "[quote=Luis Gonz\u00e1lez]Show that a point $P$ lies on the incircle $(I)$ of $\\triangle ABC$ if and only if\n\n\\[a \\cdot PA^2 \\plus{} b \\cdot PB^2 \\plus{} c \\cdot PC^2 \\equal{} (4R \\plus{} 2r)[\\triangle ABC] \\] \n$R,r$ denote the inradius and circumradius of $\\triangle ABC$ and $[\\triangle ABC]$ stands for its area.[/quote]\n\nThis condition can be rewritten in the following form: $$|a\\cdot \\overline{PA}+b\\cdot \\overline{PB}+c\\cdot \\overline{PC}|=2S(ABC).$$ This condition holds iff $P$ lies on the incircle... Very interesting..." } { "Tag": [ "money" ], "Problem": "If the price of a stamp is 33 cents, what is the maximum number \nof stamps that could be purchased with $ \\$32$?", "Solution_1": "32 dollars is 3200 cents. Since each stamp is 33, we divide $ 3200 \\div 33 \\approx \\boxed{96}$." } { "Tag": [ "MATHCOUNTS" ], "Problem": "I am very nervous about state....grrrr....", "Solution_1": "Nervousness will lower your score. :(", "Solution_2": "For some odd reason, we don't have chapter until tomorrow. But the problem is that we only have 2 weeks to prepare for state.", "Solution_3": "[quote=\"NathanSkinner\"]I am very nervous about state....grrrr....[/quote]\r\n\r\nwhat grade are you in?", "Solution_4": "My States are next Saturday. I've found that nervousness does make me do worse, so I try not to be nervous. But most of the time, it doesn't work. :D", "Solution_5": "[quote=\"SoccerBrainy40\"]My States are next Saturday. I've found that nervousness does make me do worse, so I try not to be nervous. But most of the time, it doesn't work. :D[/quote]\r\n\r\nWow, my state isn't for a month", "Solution_6": "Lol, I guess MA States are really early. Lucky, you get more time to study! It's not like I'm even going to make it to the top 10, anyway. :blush:", "Solution_7": "Our states are on March 12, [size=200]one day before my birthday[/size], and I can't ask for a better birthday present than making the nationals!!! \r\n\r\nI think two weeks are pretty much enough for anyone to study, because its in those last two weeks when you're most efficient in your study habits..", "Solution_8": "[quote=\"hockeyadi23\"]Our states are on March 12, [size=200]one day before my birthday[/size], and I can't ask for a better birthday present than making the nationals!!! \n\nI think two weeks are pretty much enough for anyone to study, because its in those last two weeks when you're most efficient in your study habits..[/quote]\r\n\r\nSAY WHAAAAAAAT? States are on the 12th??? Damn, it used to be late March.", "Solution_9": "Just like Chapter, the dates for States vary from state to state. Ours is the 12th, and it is the first year it hasn't been the first Saturday in March.", "Solution_10": "My dilemma is that I have a very competitive, hard worked for history Day project, and it's on the SAME DAY SAME TIME as mathcounts STATE (Mar. 5 here)! The only stroke of luck is my partner can present without me, but he's almost clueless on our project and was pretty much deadweight during the task =/", "Solution_11": "we just had chapter today\r\n=P state in like two weeks", "Solution_12": "I'VE BEEN DENIED MY CHANCE TO GO TO STATES!!!!!! grrr :( our school is going on the east coast trip (yea us californains get to \"explore our rich history\") so i cant go to states... boohoo AND i was on the team... so our 6th graders are going on the team to replace two of us going on the trip", "Solution_13": "[quote=\"theaznkungfu\"]I'VE BEEN DENIED MY CHANCE TO GO TO STATES!!!!!! grrr :( our school is going on the east coast trip (yea us californains get to \"explore our rich history\") so i cant go to states... boohoo AND i was on the team... so our 6th graders are going on the team to replace two of us going on the trip[/quote]\r\nWait... I thought that the only substitutions allowed are for medical reasons... at least that's how it is here...", "Solution_14": "I was first in my school competition, and then our team made 2nd at our chapter competition (thanks to a person whose score of 14 out of 46 killed our team average). Someone now replaced me for the state team because I have travel to China, starting right on the day the competition is held! Boo hoo! :(" } { "Tag": [ "inequalities", "inequalities proposed" ], "Problem": "Can anybody prove or disprove the following inequality\r\nIf a, b and c are positive real numbers then\r\na*2b + b*2c +c*2a<= 1/9(a+b+c)*3\r\n(Here x*2 means x square)", "Solution_1": "[quote=\"delegat\"]If a, b and c are positive real numbers then\na*2b + b*2c +c*2a<= 1/9(a+b+c)*3\n(Here x*2 means x square)[/quote]\r\n\r\nWrong for a = 2, b = 2, c = 1.\r\n\r\n darij", "Solution_2": "Thanks for this", "Solution_3": "Thanks for this!!! :roll:" } { "Tag": [ "number theory proposed", "number theory" ], "Problem": "Suppose $ k\\in \\mathbb N$ be an arbitrary number and $ N$ be the set of numbers that $ k$ does not occur in their decimal representation. Prove that:\r\n\\[ \\sum_{n\\in N}\\frac1n\\]\r\nis finite.\r\n\r\nThis proves that there are infinitely many primes such as $ p$ that $ k$ occurs in their decimal representation as $ \\sum\\frac 1p$ is infinite.", "Solution_1": "Hi Omid! I posted my proof here:\r\n\r\n[url=http://bboyjordan.wordpress.com/2009/10/31/every-number-m2m-nm-has-all-digits/]Problem 11.Only a few element in harmonic sum determine its divergence[/url]\r\n\r\nHowever I am not sure that I have understood the quoted corollary:\r\n[quote=\"Omid Hatami\"]This proves that there are infinitely many primes such as $ p$ that $ k$ occurs in their decimal representation as $ \\sum\\frac 1p$ is infinite.[/quote]\r\nBy Dirichlet Theorem if $ 10^{m\\minus{}1}>k$ then $ \\displaystyle \\sum_{p \\in \\mathbb{P}\\text{ s.t. }10^m \\mid p\\minus{}10k\\minus{}1}{p^{\\minus{}1}}$ diverges, but I don't see the relation with the divergence of $ \\displaystyle \\sum_{p \\in \\mathbb{P}}{p^{\\minus{}1}}$ and with the above problem.. :huh:", "Solution_2": "Yes, Obviously Dirichlet proves the fact.\r\n\r\nBut this is an elementary proof. If except finitely many primes all other primes don't contain $ k$ in their decimal representation, then $ \\sum_{n\\in N}\\frac1n$ will be infinite as $ \\sum\\frac1p$ is infinite..." } { "Tag": [ "algebra", "polynomial", "factorial", "calculus", "derivative", "algebra unsolved" ], "Problem": "Prove that the polynomial $p(x) = \\sum_{k=1}^{n} \\frac{x^k}{k!}$ has $n$ different complex roots for every $n\\in \\mathbb{N}$", "Solution_1": "Clearly $p(x)-p'(x)=\\frac {x^k}{k!}$\r\n\r\nIf a polynomial has a double root $x=k$, then $p(k)-p'(k)=0$. But that is only possible at $x=0$, and $p$ does not vanish there.", "Solution_2": "What do you mean with $p'(x)$\r\n\r\ncan you explain your solution a little more", "Solution_3": "$p'(x)$ is the derivative of $p(x)$. If $p$ is a polynomial and it has a double root at $x=k$, then $p'(k)=0$.\r\n\r\nThis proves that there are no double roots. The fundamental theorem of algebra states that $p$ has $n$ roots in the complex plane. Therefore they are $n$ distinct roots, which is the desired conclusion.", "Solution_4": "Yes, i had tried that.. but just notice that $p(x)-p'(x) = \\frac{x^n}{n!}-1$ and didn't know what to do with that $-1$... Also, $c=0$ is a root of $p$.", "Solution_5": "Sorry, I misread the bounds on the summation. I have no idea how to proceed, save for showing that the $n$-th roots of $n!$ are not roots of $p$. I tried using Rouche's theorem, but had no success." } { "Tag": [ "geometry", "probability", "number theory" ], "Problem": "Hi, I am looking for a book that will help me with my Problem Solving so I am considering buying AoPs Vol. 1&2 + Solutions for $ \\$$73. \r\n\r\nFirstly, what currency is this? AUD or US? (I live in Australia)\r\n\r\nAlso, do you think these books would be suitable for me? I am in year 10 at the moment and am mostly competent with most of the syllabus, although I am weak in number theory (no lessons on it) and geometry as far as I know. I usually get distinctions in the AMC(Australian), even getting top 1% in year 7, but since then I've dropped to about top ~10% (either im getting dumber or examiners are separating the wheat from the chaff).\r\nThanks.", "Solution_1": "I think that it is US Dollars", "Solution_2": "I think you have the proper level for those books. We also offer a series of subject-orientated books, both Intro and Intermediate levels.", "Solution_3": "I'm pretty sure the prices are in US currency.\r\n\r\nI think AOPS Volume 1 would be great for your level. I remember first working out of Volume 1 when I was about 12 and I loved it. The introduction books would also be good for you if you really feel weak in a subject, but I recommend just working through Volume 1. Volume 2 is probably too much now but you'll want it later.", "Solution_4": "So a few days ago I ordered them, they should hopefully come in a few weeks... which is pretty awesome. :D Still a bother ordering overseas though, but whatever", "Solution_5": "One nice thing for our non-US customers right now is that the US Dollar is historically weak relative to most other currencies. So goods priced in US Dollars (like our books) are relatively cheap. \r\n\r\nFor example, our new [url=http://www.artofproblemsolving.com/Books/AoPS_B_Item.php?item_id=302]Intermediate Counting & Probability[/url] books are 47 USD, which is only 23.12 UK Pounds or 33.45 Euro! What a deal! :)\r\n\r\nAnd to our Canadian customers -- the Canadian dollar is now actually worth [b]more[/b] than the US dollar. (I never thought I'd see this in my lifetime -- I grew up in Buffalo, NY, and I remember when 1 USD = 1.50 CAD.) \r\n\r\nOn the down side, yes, overseas shipping is expensive for us. But we've recently switched our international shipping to the US Postal Service's new Priority Mail International service, so our overseas shipments should be both faster and more reliable than they were in the past.", "Solution_6": "Cool, they arrived, America to Australia in a week! :lol:" } { "Tag": [ "geometry unsolved", "geometry" ], "Problem": "Let $P$ be a point inside an equilateral triangle $ABC$, and let $d_{a}, d_{b}, d_{c}$ denote the distances of P from edges, respectively, of triangle. Prove or disprove\r\n$(1+\\frac{d_{a}}{PA})(1+\\frac{d_{b}}{PB})(1+\\frac{d_{c}}{PC})\\ge\\frac{27}{8}$", "Solution_1": "$d_{a}+d_{b}+d_{c}$ is the hight of the equilateral triangle.", "Solution_2": "sorry I type error, I have edited it" } { "Tag": [ "analytic geometry", "graphing lines", "slope", "Pythagorean Theorem", "geometry", "absolute value" ], "Problem": "What is the x-coordinate of the point on the x axis which is equidistant from (8,5) and (-2,3)?\r\n\r\nSource: Math Team team round quesiton", "Solution_1": "feel free to correct\r\n\r\n[hide]Ok the way i would do this is first find the midpoint of the points (8,5) and (-2,3) which is $ ({\\frac {8 \\minus{} 2}{2} , \\frac {5 \\plus{} 3}{2}})$ which is $ (3 , 4)$ then we figure out the line that passes through that point and is perpendicular to the line that contains (8,5) and (-2,3). The formula of this line is $ y \\equal{} 5x \\minus{} 11$ then we find the zeroes of this equation so we get $ 0 \\equal{} 5x \\minus{} 11$ therefore $ \\boxed{x \\equal{} \\frac {11}{5}}$[/hide]\r\n\r\nEDIT: didnt actually edit accidentally clicked edit box", "Solution_2": "the answer the teacher gave us was[hide]19/5[/hide] but a lot of times, she makes silly mistakes, so you might be right. \r\n\r\nI have no idea how to do this problem, so I have no idea myself. :(", "Solution_3": "[quote=\"zolojetto\"]feel free to correct\n\n[hide]Ok the way i would do this is first find the midpoint of the points (8,5) and (-2,3) which is $ ({\\frac {8 \\minus{} 2}{2} , \\frac {5 \\plus{} 3}{2}})$ which is $ (3 , 4)$ then we figure out the line that passes through that point and is perpendicular to the line that contains (8,5) and (-2,3). The formula of this line is $ y \\equal{} 5x \\minus{} 11$ then we find the zeroes of this equation so we get $ 0 \\equal{} 5x \\minus{} 11$ therefore $ \\boxed{x \\equal{} \\frac {11}{5}}$[/hide][/quote]\r\n\r\nits the negative recipricol, not the recipricol that is the slope of a perpendicular line.", "Solution_4": "Your teacher is correct. Let x = the x-coordinate of the desired point. Then (x, 0) is the desired point. Let d1 = the distance from (x, 0) to (8, 5). Let d2 = the distance from (x, 0) to (-2, 3). From the Pythagorean theorem or, alternatively, the definition of distance:\r\n\r\nd1 = ((x - 8)^2 + (0 - 5)^2)^(1/2)\r\nd2 = ((x - -2)^2 + (0 - 3)^2)^(1/2)\r\n\r\nSince d1 = d2 by hypothesis, set the two right-hand sides above equal to get\r\n((x - 8)^2 + (0 - 5)^2)^(1/2) = ((x - -2)^2 + (0 - 3)^2)^(1/2).\r\n\r\nSquare both sides to get\r\n(((x - 8)^2 + (0 - 5)^2)^(1/2))^2 = (((x - -2)^2 + (0 - 3)^2)^(1/2))^2.\r\n\r\nFrom the definition of absolute value, (x^(1/2))^2 = |x|. Therefore:\r\n|(x - 8)^2 + (0 - 5)^2)| = |(x - -2)^2 + (0 - 3)^2)|\r\n\r\nFrom the definition of distance, d1 >= 0 and d2 >= 0. Therefore, |x| = x, and\r\n(x - 8)^2 + (0 - 5)^2 = (x - -2)^2 + (0 - 3)^2.\r\n\r\nSimplify to get:\r\n(x - 8)^2 + 25 = (x + 2)^2 + 9\r\nx^2 - 16x + 64 + 25 = x^2 + 4x + 4 + 9\r\n - 16x + 89 = 4x + 13\r\n 76 = 20x\r\n\r\nTherefore, x = 76\u204420 = 19/5." } { "Tag": [], "Problem": "The positive integers A, B, A-B, and A+B are all prime numbers. The sum of these four primes is\r\n(A) even\r\n(B) divisible by 3\r\n(C) divisible by 5\r\n(D) divisible by 7\r\n(E) prime", "Solution_1": "[quote=\"Totally Zealous\"]The positive integers A, B, A-B, and A+B are all prime numbers. The sum of these four primes is\n(A) even\n(B) divisible by 3\n(C) divisible by 5\n(D) divisible by 7\n(E) prime[/quote]\r\n\r\n[hide]The sum is 17 which is E[/hide]", "Solution_2": "[hide]Let A=5, B=2 A+B=7, A-B=3 The sum is 17, which is a prime. THE ANSWER IS E.[/hide]", "Solution_3": "[hide]SUM IS 17 so the answer is E\n\n$E$[/hide]" } { "Tag": [ "function", "algebra unsolved", "algebra" ], "Problem": "Find continous functions $f: \\mathbb{R}\\to\\mathbb{R}$ satisfying:\r\n$f(f(x))=f(x)+2x$\r\n\r\n\r\n\r\n[color=red]Edit by Megus: splitted from another topic (don't post two problems in one topic!) and LaTeXed[/color]", "Solution_1": "It's obviously $f(x)=2x$ :D", "Solution_2": "have a look ther:\r\n[url=http://www.mathlinks.ro/Forum/viewtopic.php?highlight=continuous+injective&t=15554]www.mathlinks.ro/Forum/viewtopic.php?highlight=continuous+injective&t=15554[/url]" } { "Tag": [], "Problem": "A homogenous ladder of length L and mass M rests against a smooth, vertical wall. If the coefficient of static friction between the ladder and the ground is $\\mu_{s}$, find he minimum angle $\\theta_{min}$, such that the ladder does not slip. Express your answer in terms of L, M, $\\mu_{s}$ and g.", "Solution_1": "Hi, \r\n\r\nThe force from gravity would then be M(g)\r\nbut the frictional force on the ladder is :[mu_{s}]M * g\r\n\r\nThe weight of the ladder is the y-component of the resultant force of the ladder on the ground (Which is applied at a direction of {theta} below the horisontal)\r\n\r\nthus weight / sin({theta}) = (x-component) /cos({theta})\r\n\r\n(x -component)= (weight*cos({theta}))/sin({theta})\r\n(x component) = (M*g*cos(theta))/sin(theta)\r\n\r\nbut x-component must have a greater magnetude than [mu_{s}]*M*g\r\n\r\nthus (M*g)*[(cos(theta)/sin(theta))-[img]http://www.artofproblemsolving.com/Forum/latexrender/pictures/f14c87a6a6fe31a1391c59a761bbc511.gif[/img] ] > 0\r\n\r\nhence cos(theta)/sin(theta) - [mu_{s}] > 0 \r\n\r\nangle before slipping gives cos(theta)/sin(theta) = [img]http://www.artofproblemsolving.com/Forum/latexrender/pictures/f14c87a6a6fe31a1391c59a761bbc511.gif[/img]\r\ntheta = arccot([img]http://www.artofproblemsolving.com/Forum/latexrender/pictures/f14c87a6a6fe31a1391c59a761bbc511.gif[/img])" } { "Tag": [ "logarithms" ], "Problem": "Can anyone help me with this problem? \r\n\r\n(x2-5x+5)x2-9x+20=1\r\n\r\nI think you're supposed to start with logarithms, but I'm not sure. I'd appreciate any assistance.", "Solution_1": "x=1,4,5 ;) ;)", "Solution_2": "Anything which makes the exponent 0 makes the number 1. Hence, $x=4,5$ work.\r\nAlso, you get 1 if the base is 1. Thus, $x=1, 4$ fulfill this property, and so the answers are $x=1,4,5$, repectively yielding $1^{12}$, $1^0$, and $5^0$.", "Solution_3": "So you say roots are ( $1,4,5$ ) only .. But why not $x=2$ ? and for that matter $x=3$ They work too! :) \r\n\r\n(Hope that helps - )\r\n\r\n(Actually if you don't restrict $x$ to real values (eg if complex values are allowed for x there are infinite solutions ...:)", "Solution_4": "[quote=\"Ijo07\"]Can anyone help me with this problem? \n\n(x2-5x+5)x2-9x+20=1\n\nI think you're supposed to start with logarithms, but I'm not sure. I'd appreciate any assistance.[/quote]\r\n\r\nAlright so when does something equal 1? $y^0$ when $y \\not= 0$, $1^y$ for all $y$, and $-1^{2y}$ for all $y$.\r\nThis means $x^2-9x+20 = 0 \\rightarrow x=5,4$ and at $x=5,4$, $x^2-5x+5 \\not= 0$. $x^2-5x+5 = 1 \\rightarrow x = 4,1$. Finally, $x^2-5x+5 = -1 \\rightarrow x=3,2$, but we need to check that $x^2-9x+20$ is even for $x=2,3$, which is true so the roots are $x=1,2,3,4,5$.", "Solution_5": "Thanks, FMako. I didn't even conceive that you could take this approach to the problem. When I plugged this into [url=http://www.hostsrv.com/webmab/app1/MSP/quickmath/02/pageGenerate?site=quickmath&s1=equations&s2=solve&s3=basic]this algebra equation solver[/url], I only got 1, 4, and 5, and Gyan said that it could also be 2 and 3, but you proved why it could be 2 and 3. I guess the calculator didn't give me 2 and 3 because it can't calculate -12yas being positive.", "Solution_6": "For those of you who are interested, this question was a NYSML tiebreaker from two years ago. I've also been told it was on an AMC." } { "Tag": [ "inequalities unsolved", "inequalities" ], "Problem": "[color=brown]For any $ x,y,z> 0$ prove that\n\n$ \\sum \\dfrac{1}{(x\\plus{}y)^2}\\ge \\dfrac{9}{4}\\cdot \\dfrac{1}{\\sum xy}$[/color]", "Solution_1": "Er....1996 Iran with have posted many times ....", "Solution_2": "Sorry then...", "Solution_3": "See here:\r\nhttp://www.artofproblemsolving.com/Forum/viewtopic.php?t=3547", "Solution_4": "i am nnot sure if my solution is right but using titu's lemma ,i think the problem can be done. :maybe:", "Solution_5": "[quote=\"sophie germain\"]i am nnot sure if my solution is right but using titu's lemma ,i think the problem can be done. :maybe:[/quote]\r\nI wish to see it. Thank you! :)", "Solution_6": "[quote=\"sophie germain\"]i am nnot sure if my solution is right but using titu's lemma ,i think the problem can be done. :maybe:[/quote]\r\nI don't think its right :wink: \r\nBy titus lemma we have :\r\n$ LHS \\geq \\frac{9}{2(x^2\\plus{}y^2\\plus{}z^2\\plus{}xy\\plus{}yz\\plus{}zx)}$,which is not bigger from $ RHS$...\r\nIf you mean something else i would like to see it too :)", "Solution_7": "yes ,i did get that. but after that i got \r\n$ x^2\\plus{}y^2\\plus{}z^2$ $ \\ge$ $ xy\\plus{}yz\\plus{}zx$\r\nthen arent u done? :maybe:", "Solution_8": "[quote=\"sophie germain\"]yes ,i did get that. but after that i got \n$ x^2 \\plus{} y^2 \\plus{} z^2$ $ \\ge$ $ xy \\plus{} yz \\plus{} zx$\nthen arent u done? :maybe:[/quote]\r\nLook it better my friend :wink: \r\nIt is the opposite:\r\n$ \\frac{9}{2(x^2\\plus{}y^2\\plus{}z^2\\plus{}xy\\plus{}yz\\plus{}zx)} \\leq \\frac{9}{4(xy\\plus{}yz\\plus{}zx)}\\Leftrightarrow\r\nx^2\\plus{}y^2\\plus{}z^2\\plus{}xy\\plus{}yz\\plus{}zx \\geq 2(xy\\plus{}yz\\plus{}zx)$", "Solution_9": "oh yeah, i realise the mistake,.thank you :)" } { "Tag": [ "email", "AMC", "AIME" ], "Problem": "Hey i just want to know how can i get all my scores from past years if our school didn't keep everything?\r\n\r\nis there an email address i am email to ask for my scores?", "Solution_1": "For what it's worth, I have records of the last four years of AIME qualifiers in California - but only California, and only AIME qualifiers.", "Solution_2": "Can't you buy entire packets of info on AIME qualifiers from the AMC people?", "Solution_3": "Write a letter or an email to the AMC office with your request. Our postal address and our email address is on all of our literature. \r\n\r\nBe sure to specify your name, exactly as it was used on the contests you took, the years that you took the contests, the name of the school where you took the contest (the CEEB number will help us look it up faster, if you know the CEEB number) and the city and the state.\r\n\r\nRequests for scores are answered only for the individual requesting the scores.\r\n\r\nPlase allow up to two weeks for an answer.\r\n\r\nSteve Dunbar\r\nMAA Director, American Mathematics Competitions", "Solution_4": "thanks ! i just sent an email to amcinfo @ unl . edu\r\n\r\n( i hope that is the right email XP)", "Solution_5": "When are amc 12 perfect score plaques being delivered? Or are they still doing that this year?", "Solution_6": "Yes, we are still sending the plaques.\r\n\r\nMost of them were sent last week. If you do not receive the correct plaque, first contact your teacher to determine if your teacher or contest manager received the plaque, then contact our office.\r\n\r\nSteve Dunbar\r\nMAA Director, American Mathematics Competitions", "Solution_7": "[quote=\"xxazurewrathxx\"]When are amc 12 perfect score plaques being delivered? Or are they still doing that this year?[/quote]\r\n\r\nMust it be perfect score? I thought it was just winner of the region.\r\n\r\nAlso, are there plaques for state winners on 10A/10B as wel", "Solution_8": "I think it's just for AMC 12, but I could be wrong. Also, I think winners of regions get awards too, but aren't they different from perfect score plaques? I seem to recall that the top score in the region where there are no perfect scores receive some kind of special award, and everyone else get subscriptions or books and stuff. Are awards delivered to the home or the school? \r\n\r\nEDIT: Oh by the way, it looks like I posted my question on the wrong thread by accident. It was supposed to go on the AMC Awards thread. Sorry about that.", "Solution_9": "i sent an email to amcinfo @ unl . edu two weeks ago about getting all my scores\r\nbut i haven't heard any replies yet", "Solution_10": "[quote=\"rainynightstarz\"]i sent an email to amcinfo @ unl . edu two weeks ago about getting all my scores\nbut i haven't heard any replies yet[/quote]\r\n\r\nThe two regular answerers to that email address are 1) extremely busy with MOSP and 2) on vacation. MOSP ends Saturday, so give us another week to catch up on the backlog, and if you haven't heard anything, email again. I'd look into it for you, but I'm not involved in the registration/score processing end of things.\r\n\r\nElizabeth Claassen\r\nAMC Senior Accounting Clerk" } { "Tag": [ "geometry", "3D geometry", "sphere", "calculus", "integration", "trigonometry" ], "Problem": "A uniform sphere (Mass M, radius R) is rolling w/o slipping. Its centre of mass translates with a speed $ v_0$ The sphere encounters a \"stair\"(like you climb steps/stairs) of height $ h( mgh$ (K.E of solid sphere rolling without slipping is $ \\frac{7}{10}Mv^2$)\n\n$ \\Longrightarrow v^2>\\frac{7}{10}gh$\n\nnow,using (i),\n$ \\Longrightarrow v_0(\\frac{7R-5h}{7R}) >\\sqrt{\\frac{10}{7}gh}$\n$ \\Longrightarrow v_0>\\frac{R\\sqrt{70gh}}{7R-5h}$ .......(ii)\n\n[b]bound 2[/b]\nto roll without slipping,total inward force must be greater than total outward force,\nso,\n$ Mgcos{\\theta}>\\frac{Mv^2}{R}$\n$ \\Longrightarrow v<\\sqrt{gRcos{\\theta}}$\n$ \\Longrightarrow v<\\sqrt{gR(R-h)}$ as $ (cos{\\theta}=\\frac{R-h}{R})$\n\nnow using (i),\n$ \\Longrightarrow v_0(\\frac{7R-5h}{7R}) <\\sqrt{gR(R-h)}$\n$ \\Longrightarrow v_0 < \\frac{7R}{7R-5h}\\sqrt{gR(R-h)}$ .........(iii)\n\n\nusing (ii) and (iii),we can get bounds :) \n\nbut second bound may be wrong(its just logical thinking)\n$ {\\theta}$ is angle made by pt of contact with vertical\n\nsrry for latexing\n[/hide]", "Solution_2": "That is right. :D", "Solution_3": "Question about part 1 of question: (to Skand and anyone who read his solution).\r\n\r\n- I guess you used the system being the sphere. If so , how did you use conservation of angular momentum in part 1? There is a net torque due to gravity on the sphere between the initial rolling and final rolling, which causes a change in angular momentum?", "Solution_4": "Um... that solution looks very sketchy to me, for several reasons:\r\n\r\n1. COAM is applied incorrectly, as integration pointed out.\r\n\r\n2. In the limiting case $ h \\rightarrow R$, we clearly should have that the minimum velocity approaches infinity. Instead, skand's upper bound gives $ v < 0$ and $ v > \\frac {1}{2}\\sqrt {70gR}$, a finite and positive quantity. Huh?\r\n\r\n3. It doesn't take into account the fact that when the sphere impacts the stair, the stair will \"absorb\" most of its velocity, leaving only a component normal to the line connecting the contact point and the center of the sphere.\r\n\r\nThis is an interesting problem, but I believe it can be interpreted many different ways.", "Solution_5": "At the moment of impact, the step exerts a force on the ball. But the ball's angular velocity about the axis through the point of impact is not affected by the impact. Thus the ball's initial angular velocity (at and just after impact) about this axis is $ \\omega_0 \\equal{} \\frac{v_0 \\sin{\\theta}}{R} \\equal{} \\frac{R\\minus{}h}{R^2} v_0$ where $ \\theta$ is the angle the line between the center of the ball and the point of impact makes with the horizontal. After impact, the only torque affecting the system is due to gravity, so we just need to make sure that the initial rotational energy is sufficient for the ball to get over the step: $ \\frac{1}{2} I \\omega_0^2 \\geq M g h$. Solving we get $ v \\geq \\frac{R \\sqrt{5 g h}}{R \\minus{} h}$.", "Solution_6": "Yup, I totally agree with that solution.", "Solution_7": "Haha, when you said integration generic, I thought What? There's not even an integral in the solution?? :D \r\n\r\nAnyway, what's COAM?\r\n\r\nNice solution durt... I was stuck for a while on this one.", "Solution_8": "some explanations :) \r\n[hide=\"1\"]$ mg$is a constant(non impulsive) force,so,torque due to gravity can be neglected,here the axis is fixed to ground and not to the ball,that also explains the fact why torque due to pseudo forces are not considerd(there is no pseudo force)[/hide]\n\n[hide=\"3\"]it does take,the Conservation of energy is applied after change in velocity and angular velocity,and the values are after the impact(that are less than initial values)\nthe total Kinetic energy lost in collisionis $ \\frac{7}{10}M(v_0^2\\minus{}v^2)$[/hide]\n\n[hide=\"2\"]i couldn't understand what you are saying :( ,please explain,and i couldn't understand durt's soln(i am just a beginner)[/hide]", "Solution_9": "COAM = conservation of angular momentum.\r\n\r\nIn no. 2, skand, I'm just saying that it might be helpful to see whether your answer makes sense in special cases that are easy to analyze.", "Solution_10": "@ skand plz explain ur answer with some more detail, and also plz add a picture so that i can understand easily :D", "Solution_11": "[quote=\"befuddlers\"]@ skand plz explain ur answer with some more detail, and also plz add a picture so that i can understand easily :D[/quote]\n[quote=\"durt\"]so we just need to make sure that the initial rotational energy is sufficient for the ball to get over the step: $ \\frac {1}{2} I \\omega_0^2 \\geq M g h$.[/quote]\r\nYour method is alright. You also need to consider the translational Kinetic energy, besides the rotational energy here. One mistake that you have made, I believe, is considered the translational velocity of the ball to become $ 0$ on hitting the step. Note that the collision causes a change in only that component of velocity which is perpendicular to it. Hence $ v_0 \\sin \\theta$ remains the velocity of the ball just after collision and this velocity is perpendicular to Normal Reaction $ N$.\r\n\r\nMy solution goes as follows:\r\n\r\n\r\n\r\n(The ball should collide inelastically with the step, else it will bounce.)\r\nNow, as it hits the step, the velocity in the direction of the Normal at the point of contact will disappear $ (%Error. \"since\" is a bad command.\ne = 0)$.\r\nThe velocity that will remain is $ v_0 \\sin \\theta$ (see figure) perpendicular to the Normal reaction $ N$.\r\n\r\nNow, the ball climbs the step to gain height $ R(1 - \\sin \\theta)$.\r\n\r\n$ \\begin{align*} & \\therefore mgR(1 - \\sin \\theta) = \\frac {1}{2}mv_0^2 \\sin^2 \\theta + \\frac {1}{2} I \\omega ^2 \\\\\r\n& \\therefore gR(1 - \\sin \\theta) = \\frac {7}{10} v_0^2 \\sin^2 \\theta \\\\\r\n\\left( I = \\frac {2}{5} mR^2, \\; \\omega = \\frac {v_0^2 \\sin^2 \\theta}{R}\\right)$\r\n\r\n$ \\therefore \\boxed {\\color{red}{v_0 \\ge \\frac {R}{R - H} \\sqrt {\\frac {10Hg}{7}}}}$", "Solution_12": "Hey! No comments/suggestions about this one?" } { "Tag": [ "quadratics", "search", "number theory", "number theory open" ], "Problem": "Let $ C_N(p)$ be the number of $ N$ consecutive quadratic residues modulo some prime $ p$.\r\n\r\n[hide=\"For Example\"]\n$ C_1(p) \\equal{} (p \\minus{} 1)/2$\n$ C_2(p) \\equal{} \\frac {1}{4}(p \\minus{} 4 \\minus{} ( \\minus{} 1)^{(p \\minus{} 1)/2})$ ([i]Number Theory[/i], Andrews)\n[/hide]\r\n\r\nProve (or disprove) that $ C_N(p) \\equal{} O \\left (\\frac {p}{2^N} \\right )$.\r\n\r\nHopefully I am making the problem statement clear. It is not unrelated to [url=http://www.mathlinks.ro/viewtopic.php?search_id=1670343724&t=253789]This Problem[/url] and, if true, provides an alternate proof to it. I lack the ability to get near a solution, but would be interested in seeing one.", "Solution_1": "Hmm, [url=http://citeseer.ist.psu.edu/viewdoc/summary?doi=10.1.1.29.5334]yes, apparently quadratic residues really do behave pretty much like coin flips...[/url]", "Solution_2": "Do you really want $O$-notation, as this gives only upper bounds\u00bf\nI think you intended $C_N(p) \\sim \\frac{p}{2^N}$." } { "Tag": [ "Ring Theory", "algebra", "polynomial", "abstract algebra", "search", "calculus", "integration" ], "Problem": "$ A$ is subring of $ \\mathbb{C}$ generated by mth roots of unity, $ p$ is a prime number. Prove that there are finitely many prime ideals $ \\mathfrak{p}_1,....\\mathfrak{p}_h$ of $ A$ containing $ p$. The quotents $ A/\\mathfrak{p}_i$ are finite fields of characteristic $ p$, and there exists an integer $ N$ such that $ pA\\supset (\\mathfrak{p}_1\\cap...\\cap \\mathfrak{p}_h)^N$. Help me. Thank!", "Solution_1": "the $ \\mathfrak{p_i}$ correspond to prime ideals of $ A$/ $ pA$, which is a finite ring of characteristic $ p$. The first two assertions folow from this. This third follows from the fact that $ (\\mathfrak{p_1}\\cap{\\mathfrak{p_2}} \\cdots \\cap{\\mathfrak{p_n}})/pA$ is the radical of the artinian ring $ A/pA$, thus is nilpotent.", "Solution_2": "For the first part let $ A\\equal{}\\mathbb{Z}[u]$, where $ u$ is an mth primitive root of the unit and let $ \\phi_m$ be the mth cyclotomic polynomial. We know that $ \\phi_m$ is the irreducible monic polynomial of low degree in $ \\mathbb{Z}[x]$ that annihilates $ u$. If $ f\\in\\mathbb{Z}[x]$ satifaces $ f(u)\\equal{}0$ theh $ \\phi_m$ divides $ f$. This shows that the map $ \\mathbb{Z}[x]\\rightarrow \\mathbb{Z}[u]$ given by $ f\\rightarrow f(u)$ is an epimorphism whose kernel is $ (\\phi_m)$. Then the prime ideals in A that contian $ p$ are in correspondence with the prime ideals in $ Z[x]$ that contains $ p$ and $ (\\phi_m)$. Now it is convenient to read http://www.mathlinks.ro/viewtopic.php?search_id=1643396135&t=72993). \r\n\r\nLet $ \\phi_m\\equal{}f_1f_2...f_n$ the factorization in irreducbles in $ \\mathbb{Z}_p[x]$. Then $ f_i$ is irreducible in $ \\mathbb{Z}[x]$ since $ f$ is monic and is irreducible as a polynomial in $ \\mathbb{Z}_p[x]$. We claim that $ (p,f_1),....,(p,f_n)$ are all the prime ideals that contain $ (p,\\phi_m)$. In fact, any other prime ideal that contains $ (p,\\phi_m)$ is of the form $ (p,f)$ with $ f$ irreducible as a polynomial in $ \\mathbb{Z}_p[x]$ (see the link). Then $ f$ divides $ \\phi_m$ in $ \\mathbb{Z}_p[x]$. This implies $ (f)\\equal{}(f_i)$ for some $ f_i$ as ideals in $ \\mathbb{Z}_p[x]$ and then $ (p,f)\\equal{}(p,f_i)$ as ideals in $ \\mathbb{Z}[x]$", "Solution_3": "Thank you,\r\nBut I don't understand why $ A/pA$ is an Artin ring?", "Solution_4": "$ \\mathbb{Z}/p \\mathbb{Z} \\to A/pA$ is integral and of finite type, thus finite." } { "Tag": [ "vector", "abstract algebra", "group theory", "linear algebra", "linear algebra unsolved" ], "Problem": "Find an abelian group V and a field F for which V is a vector space over F in two\r\ndifferent ways, that is, there are two different definitions of scalar multiplication\r\nmaking V a vector space over F.", "Solution_1": "you can take any vector space and define scalar operation to take everything into the zero vector.", "Solution_2": "I'll assume that we have an axiom that $ 1\\cdot x\\equal{}x$- that rules out zero multiplication.\r\n\r\n$ \\mathbb{R}^2$ as a vector space over $ \\mathbb{C}$:\r\n(1) $ (a\\plus{}ib)\\cdot (x,y)\\equal{}(ax\\minus{}by,ay\\plus{}bx)$\r\n(2) $ (a\\plus{}ib)\\cdot (x,y)\\equal{}(ax\\plus{}by,ay\\minus{}bx)$\r\n\r\nThis is impossible if the field is $ \\mathbb{Q}$ or the field with $ p$ elements for some prime $ p$. For other fields, the two structures will always be related by a field automorphism- in the above case, the conjugate.\r\n\r\nFor a related problem, see [url=http://www.artofproblemsolving.com/Forum/viewtopic.php?&t=166604]this topic[/url].", "Solution_3": "my suggestion is wrong but i cannot edit. sorry" } { "Tag": [ "limit", "function", "calculus", "calculus computations" ], "Problem": "Let be sequence $ a_0 \\equal{} 1,\\,\\,a_{n \\plus{} 1} \\equal{} \\sum\\limits_{i \\equal{} 0}^n {a_i a_{n \\minus{} i} }$, ($ n\\ge 0$ )\r\nFind limit of sequence $ a_n$:\r\n$ \\mathop {\\lim }\\limits_{n \\to \\infty } \\frac {{a_n }}{{a_{n \\minus{} 1} }}$", "Solution_1": "[hide=\"Hint 1\"]Consider the generating function $ f(x) \\equal{} \\sum_{k\\equal{}0}^\\infty a_k x^k$. [/hide]\n[hide=\"Hint 2\"]Show that $ f(x) \\equal{} x f(x)^2 \\plus{} 1$. Conclude that \n\\[ f(x) \\equal{} \\frac{1 \\minus{} \\sqrt{1\\minus{}4x}}{2x}.\\]\n[/hide]\n[hide=\"Hint 3\"]Therefore, for $ k>0$\n\\[ a_k \\equal{} 2^k \\frac{(2k\\minus{}1)!!}{(k\\plus{}1)!},\\]\nand $ \\lim \\frac{a_{k\\plus{}1}}{a_k} \\equal{} 4$.\n[/hide]" } { "Tag": [ "Princeton", "college" ], "Problem": "Does anyone know the website with all the pumac problems of previous years?", "Solution_1": "http://seansoni.com/contests.html" } { "Tag": [ "Galois Theory", "superior algebra", "superior algebra unsolved" ], "Problem": "An exam typo suggested this cute (though not particularly deep) problem. I enjoyed solving it, and hope others will as well. :)\r\n\r\nLet $ K \\equal{} \\cup_{m \\equal{} 0}^\\infty \\mathbb{F}_{2^{2^m}}$, and let $ \\sigma: K\\to K$ be the Frobenius automorphism $ x\\mapsto x^2$. Prove that $ \\{\\sigma, \\sigma^{1 \\plus{} 2}, \\sigma^{1 \\plus{} 2 \\plus{} 3},\\ldots \\}$ is dense in the topological group $ \\rm{Gal}(K/\\mathbb{F}_2)$.\r\n\r\nFor a bonus, show that this is false over $ \\mathbb{F}_p$ for odd $ p$!", "Solution_1": "Hints:\r\n\r\n(1) If $ \\tau: K\\to K$ is an arbitrary automorphism, an elementary open neighborhood of $ \\tau$ consists in those automorphisms of $ K$ that agree with $ \\tau$, for some fixed $ \\mathbb{F}_{2^{2^m}}$.\r\n\r\n(2) In particular, every open neighborhood of $ \\tau$ contains some power of $ \\sigma$. Hence we may assume without loss of generality that $ \\tau \\equal{} \\sigma^k$.\r\n\r\n(3) The conclusion follows if we can find some $ j$ such that $ \\sigma^{\\frac {j(j \\plus{} 1)}{2}} \\equal{} \\sigma^k$ on $ \\mathbb{F}_{2^{2^m}}$. This is entirely a number-theoretic question.", "Solution_2": "The Galois group is $ \\mathbb Z_2$ ($ 2$-adics) and $ \\sigma$ corresponds to $ 1$. Thus it's equivalent to showing that $ n^2\\plus{}n$ is dense in $ 2\\mathbb Z_2$, which follows from Hensels Lemma.\r\n\r\nBut I never needed $ p\\equal{}2$ here... :maybe:", "Solution_3": "Well, you need $ p\\equal{}2$ to guarantee that $ (n^2\\plus{}n)'\\equal{}2n\\plus{}1$ is nonzero... but also you need to know first that $ n(n\\plus{}1)/2$ is dense mod p, which is not true for $ p>2$.\r\n\r\nFor those not familiar with the p-adics: you can alternatively just show that $ n(n\\plus{}1)/2 \\equal{} ((2n\\plus{}1)^2 \\minus{}1)/8$ is dense mod $ 2^k$ for all $ k$, which is equivalent to showing that $ 8l\\plus{}1$ is always a square mod $ 2^{k\\plus{}3}$. This can be done with, e.g. a counting argument.", "Solution_4": "No it's $ \\mathbb Z_2$ independent of $ p$ (if you work with $ K \\equal{} \\cup_{m \\equal{} 0}^\\infty \\mathbb{F}_{p^{2^m}}$). So the value of $ p$ is not needed.", "Solution_5": "I suppose I was thinking of the fields $ \\mathbb{F}_{p^{p^m}}$. You're right." } { "Tag": [ "algebra", "polynomial", "function", "symmetry" ], "Problem": "P(x) is of degree 7. P(x) is odd and monic. P(x) has a factor of $ (x\\minus{}5)^{2}$. The curve y=P(x) passes through $ (1,1152)$. P(x) has 5 real roots.\r\n\r\n----\r\n[hide=\"part of the soln. i dont understand\"] $ (x\\minus{}5)^{2}$ is a factor and since its odd then $ (x\\plus{}5)^{2}$ is a factor. \nOdd functions pass through the origin therefore x is also a factor. $ P(x) \\equal{} x(x\\minus{}5)^{2}(x\\plus{}5)^{2}$. \nThen the soln. says that P(x) cannot have any real fns hence let the final factor be $ (x^{2}\\plus{}k^{2})$. Why can't it have any more factors?[/hide]\r\n--\r\n\r\nalternative soln. are wlecome", "Solution_1": "Since $ P(x)$ is an odd function so we have that $ P(\\minus{}5) \\equal{}\\minus{}P(5) \\equal{} 0$ hence $ (x\\plus{}5)^{2}$ is a factor as well. Because it is an odd function so $ P(0) \\equal{}\\minus{}P(0)$ hence $ x$ is a factor. The question says that it has $ 5$ real roots and since we have already got $ 5$ factors, so no more real roots exist. So let us represent the last factor as $ x^{2}\\plus{}k^{2}$ (which we are assuming the roots are purely imaginary). $ P(x) \\equal{} x(x\\plus{}5)^{2}(x\\minus{}5)^{2}(x^{2}\\plus{}k^{2})$ an substitue $ x \\equal{} 1$ in and we see that $ k^{2}\\equal{} 1$.\r\n\r\n$ P(x) \\equal{} x(x\\plus{}5)^{2}(x\\minus{}5)^{2}(x^{2}\\plus{}1)$", "Solution_2": "oh right. Thanks for that man", "Solution_3": "[quote=\"BanishedTraitor\"]Since $ P(x)$ is an odd function so we have that $ P(\\minus{}5) \\equal{}\\minus{}P(5) \\equal{} 0$ hence $ (x\\plus{}5)^{2}$ is a factor as well. [/quote]\r\n\r\nThis argument only proves that $ (x\\plus{}5)$ is a factor. To prove that $ (x\\plus{}5)^{2}$ is a factor, we must do more work:\r\n\r\n$ P(x) \\equal{} x(x\\plus{}5)^{2}(x\\minus{}a)(x\\minus{}b)(x\\minus{}c)(x\\minus{}d) \\equal{}\\minus{}P(\\minus{}x) \\equal{} x(x\\minus{}5)^{2}(x\\plus{}a)(x\\plus{}b)(x\\plus{}c)(x\\plus{}d)$\r\n\r\nAlternatively, if $ P(x)$ is odd then $ P'(x)$ is even, and $ P'(\\minus{}5) \\equal{} P(5) \\equal{} 0$.", "Solution_4": "since there is a double root at 5, there is a double root at -5 since it is an odd function... [we have reflective symmetry wrt. the origin...]" } { "Tag": [ "number theory", "least common multiple", "greatest common divisor", "MATHCOUNTS", "relatively prime" ], "Problem": "what is the greatest integer than cannot be obtained by addining non negative multiples of 6, 10 , and 15?\r\n\r\nis there a quick formula for questions like these?", "Solution_1": "I think a fast way would be two recognize 6=2*3,10=5*2,15=3*5. But from here i am stuck, alternative-brute force it.", "Solution_2": "considering units digits can for some reason make it easier to find out", "Solution_3": "6, 10, and 15.\r\nWell, using 6 and 10, the largest we can form is infinitely large.\r\nHowever, we can form all even numbers larger than 14.\r\nNow consider if we have 15. Then, for all odd numbers we need one 15 and then some number of 6 and 10. However, 14+15 would be impossible and the largest number we can't form.\r\n14+15 = 30, just kidding, 29.", "Solution_4": "15+ (any even number>15)=any odd number>30, and we already got all even numbers, so 29.", "Solution_5": "any coincidence that its their LCM-1?", "Solution_6": "Yes, it is a coincidence.\r\n\r\nI think there was a formula posted here sometime for 3 values, but I forgot it and I wasn't sure if it was correct in the first place.", "Solution_7": "Well, their LCM has to be one, otherwise the problem is unsolvable... if they had a LCM of n, you would not be able to make any number congruent to mn-(n-q) for positive integers n, m, and q, q < n.", "Solution_8": "[quote=\"uldivad9\"]Well, their LCM has to be one, otherwise the problem is unsolvable... if they had a LCM of n, you would not be able to make any number congruent to mn-(n-q) for positive integers n, m, and q, q < n.[/quote]\r\n\r\nI think you mean GCF :wink:", "Solution_9": "[quote=\"unimpossible\"]any coincidence that its their LCM-1?[/quote]\r\n\r\nHey, that's what he said, and I forgot to correct it. You know what I mean. :P", "Solution_10": "[quote=\"unimpossible\"]any coincidence that its their LCM-1?[/quote]\r\nYes, it's a coincidence. A quick counterexample... Consider the three summands, $ 15,$ $ 21,$ and $ 35,$ which are pairwise products of three distinct primes, as in the original problem. I assert the largest number that cannot be achieved is $ 139.$\r\n\r\nFirst, let's assume you know the Chicken McNugget Theorem, which says if you have exactly [b]two[/b] relatively prime positive integers, $ s$ and $ t$, the largest number that cannot be achieved through summation is their product minus their sum, $ (s\\times t) \\minus{} (s \\plus{} t).$ So, if you can buy McNuggets only in cartons of 5 or 7, the largest number you cannot purchase is $ 35 \\minus{} 12 \\equal{} 23.$\r\n\r\nLet's consider how we might achieve the number $ 139,$ above. Let's repeatedly take away $ 35$, and consider what remains. $ 139,$ $ 104,$ $ 69,$ and $ 34.$ We must use 15's and 21's to sum to one of these numbers. But 15 and 21 are multiples of 3. The only multiple of 3 in our list is 69. But 69 is the largest number you cannot achieve with 15 and 21. (Divide everything by three. 23 is the largest number you cannot achieve with 5 and 7.)\r\n\r\nLet's illustrate a different way... We'll take away $ 21,$ and consider what remains. $ 139,$ $ 118,$ $ 97,$ $ 76,$ $ 55,$ $ 34,$ and $ 13.$ We must use 15's and 35's to sum to one of these numbers. But 15 and 35 are multiples of 5. The only multiple of 5 in our list is 55. But 55 is the largest number you cannot achieve with 15 and 35. (Divide everything by five. 11 is the largest number you cannot achieve with 3 and 7. $ 21 \\minus{} 10 \\equal{} 11.$\r\n\r\nLet's try a third way... We'll take away $ 15,$ and consider what remains. $ 139,$ $ 124,$ $ 109,$ $ 94,$ $ 79,$ $ 64,$ $ 49,$ $ 34,$ $ 19,$ and $ 4.$ We must use 21's and 35's to sum to one of these numbers. But 21 and 35 are multiples of 7. The only multiple of 7 in our list is 49. But 49 is the largest number you cannot achieve with 21 and 35. (Divide everything by seven. 7 is the largest number you cannot achieve with 3 and 5. $ 15 \\minus{} 8 \\equal{} 7.$\r\n\r\nAll three of these give us good patterns to work with. (Always look for patterns with MATHCOUNTS.) I will assert (but not prove) the formula for this special case:\r\n\r\n[hide]Given three distinct primes, $ p,$ $ q,$ and $ r,$ taken pairwise to make three summands, $ pq,$ $ qr,$ and $ rp\\ldots$\n\nThis formula follows directly from any of the above three examples:\n$ ((p\\times q) \\minus{} (p \\plus{} q))\\times r \\plus{} (p\\times q)\\times(r \\minus{} 1)$\n\nSimplify...\n$ pqr \\minus{} pr \\minus{} qr \\plus{} pqr \\minus{} pq$\n\n$ \\boxed{2pqr \\minus{} (pq \\plus{} qr \\plus{} rp)}$\n\nFor this problem, $ 2\\times 3\\cdot 5\\cdot 7 \\minus{} (3\\cdot 5 \\plus{} 5\\cdot 7 \\plus{} 7\\cdot 3) \\equal{} 2\\times 105 \\minus{} (15 \\plus{} 35 \\plus{} 21) \\equal{} 210 \\minus{} 71 \\equal{} 139$\n\nFor the original problem, $ 2\\times 2\\cdot 3\\cdot 5 \\minus{} (2\\cdot 3 \\plus{} 3\\cdot 5 \\plus{} 5\\cdot 2) \\equal{} 2\\times 30 \\minus{} (6 \\plus{} 15 \\plus{} 10) \\equal{} 60 \\minus{} 31 \\equal{} 29$[/hide]\r\n\r\nProblems involving two numbers are common in MATHCOUNTS, and high-level competitors should know the Chicken McNugget Theorem. Problems involving more than two numbers are uncommon in MATHCOUNTS. Such problems will almost surely be small enough to use brute force.\r\n\r\nA perfectly fair, State- or National-level problem: What is the largest sum that cannot be attained by adding non-negative multiples of 6, 14, and 21.", "Solution_11": "what if you fitted the numbers 8,14,19 would it be 39 then?" } { "Tag": [ "integration", "real analysis", "real analysis unsolved" ], "Problem": "Let $ a_1,a_2,..,a_n$ are $ n$ real number .\r\nCalculate \r\n $ \\int_{0}^{1}\\int_{0}^{1}{(x\\plus{}a_1y)(x\\plus{}a_2y)...(x\\plus{}a_ny)dxdy}$", "Solution_1": "Well, $ \\prod_i (x\\plus{}\\alpha_i y) \\equal{} x^n \\plus{} x^{n\\minus{}1}y\\sum_i \\alpha_i \\plus{} x^{n\\minus{}2}y^2 \\sum_{i17$ we may assume that $a>0$(if not then just take the polynomial -q(x)), then it is easy to see that $|c|\\leq 1$\nAlso if $|a|>17$ then the polynomial $r(x)=p(x)-\\frac{8}{a}\\cdot q(x)$ is a polynomial of degree not more than 1.\nBut $r(0)>0,r(1/2)<0,r(1)>0$ contradiction since $r$ has 2 distinct zeros.So $|a|<8$\nThen $|b|>8$.Also $b=-|b|$.Next $-1\\leq p(1/2)=a/4+b/2+c\\leq 3+b/2$ that gives $|b|\\leq 8$ contradiction so there's no such polynomial $q$.[/hide]", "Solution_3": "Let $f\\left( x \\right) = ax^2 + bx + c$ be a function, and we have: \r\n$\\left| {f\\left( x \\right)} \\right| \\le 1,\\forall x \\in \\left[ {0,1} \\right] \\Leftrightarrow f\\left( {\\left[ {0,1} \\right]} \\right) \\subseteq \\left[ { - 1,1} \\right]$\r\nIn this case, how we find $\\max \\left( {\\left| a \\right| + \\left| b \\right| + \\left| c \\right|} \\right)$ ???\r\n\r\n\r\n(I see that $\\max \\left( {\\left| a \\right| + \\left| b \\right| + \\left| c \\right|} \\right)=17$)", "Solution_4": "I didn't get you?What do you mean? :?" } { "Tag": [ "logarithms" ], "Problem": "If $2(4^{x})+6^{x}=9^{x}$ and $x=\\log_{2/3}a$, find the numerical value of $a$.", "Solution_1": "divide both sides by 9^x to get:\r\n\r\n2(4/9)^x+(2/3)^x-1=0...putting in a, we have\r\n\r\n2a^2+a-1=0, and a>0...so\r\n\r\n$a=\\frac{1+\\sqrt{3}}{2}$" } { "Tag": [ "inequalities", "trigonometry", "analytic geometry", "graphing lines", "slope" ], "Problem": "Prove that \r\n$ cos{1}\\plus{}(sin{1}\\minus{}1)(\\sqrt{2}\\plus{}1)>0$", "Solution_1": "$ \\cos{1} \\plus{} (\\sin{1} \\minus{} 1)(\\sqrt {2} \\plus{} 1) > 0$\r\n\r\n$ \\Longleftrightarrow \\frac {\\sin 1 \\minus{} 1}{\\cos 1} > 1 \\minus{} \\sqrt {2}$\r\n\r\nThe left side means the slope of the line passing through some two points and the right side means $ \\minus{}\\tan 22.5^\\circ$.", "Solution_2": "This is a hint or solution?" } { "Tag": [ "induction", "graph theory", "combinatorics unsolved", "combinatorics" ], "Problem": "Let $G$ be a graph such that if you remove any $k-1$ vertices the graph will still connected. Prove that for any 2 vertices in $G$ there exist $k$ disjoint paths between them.\r\n\r\nI proved a specific case for the 2-connected graph by induction on the distance between two vertics but i am having problems trying to generalize it.", "Solution_1": "I found this theorem with three proofs in book \"Graph Theory\" by R. Diestel:\r\n\r\nLet $G=(V, E)$ be a graph and $A, B \\subseteq V$. Then the minimum number of vertices separating $A$ from $B$ in $G$ is equal to the maximum number of disjont $A-B$ paths in $G$.\r\n\r\nAnd as your graph is $k$-connected the result follows. So I think you can google this theorem - as you wrote in source this is Menger's Theorem.", "Solution_2": "I have the diestel book.. I will look it there.. thanks ;)" } { "Tag": [ "number theory", "modular arithmetic" ], "Problem": "I remember taking a peek this problem:\r\n\r\n\"The spinner has only 3 and 8 on it. What is the largest possible value that can not be obtained?\"\r\n\r\nI remember the solution being 11 or 13 and it said that it can be solved by $ mn \\minus{} m \\minus{} n$ or something like that.\r\n\r\nI didn't get it. Can someone explain?", "Solution_1": "Maybe the problem was \"the largest possible value which [u]can't[/u] be obtained, with as many spins as you want. Then you would use mn-n-m to get 13. You can't get 13, but you can get any number higher (for example 14=8+3+3).", "Solution_2": "Isn't that supposed to be \"cannot be obtained\"?\r\n\r\nModular arithmetic is used kinda often here.\r\n[hide]\nAnyways, the $ mn\\minus{}m\\minus{}n$ is the answer for this, where $ m\\equal{}3$ and $ n\\equal{}8$. This is known as the Chicken McNugget Theorem.\n\nWe know that all numbers congruent to 0 mod 3 greater than or equal to 3 can be obtained with just 3s. If we have one 8, and then the rest of the numbers being 3s, then all numbers congruent to 2 mod 3 greater than or equal to 8 can be obtained. Then, by the same process, all numbers greater than or equal to 16 that are congruent to 1 mod 3 can be obtained. So subtract 3 from 16 to get [b]13[/b], which is the answer given by the formula above.\n[/hide]", "Solution_3": "[quote=\"alanchou\"]Isn't that supposed to be \"cannot be obtained\"?\n\nModular arithmetic is used kinda often here.\n[hide]\nAnyways, the $ mn \\minus{} m \\minus{} n$ is the answer for this, where $ m \\equal{} 3$ and $ n \\equal{} 8$. This is known as the Chicken McNugget Theorem.\n\nWe know that all numbers congruent to 0 mod 3 greater than or equal to 3 can be obtained with just 3s. If we have one 8, and then the rest of the numbers being 3s, then all numbers congruent to 2 mod 3 greater than or equal to 8 can be obtained. Then, by the same process, all numbers greater than or equal to 16 that are congruent to 1 mod 3 can be obtained. So subtract 3 from 16 to get [b]13[/b], which is the answer given by the formula above.\n[/hide][/quote]\r\n@lokito and alanchou: Sorry, my memory wasn't clear. My apology.\r\n\r\nOh, I see. But Chicken McNugget theorem.. :wink:" } { "Tag": [ "AMC", "AIME", "AMC 12", "geometry", "AIME I", "AIME II" ], "Problem": "OK, I know when we do AIMEs for practice, we wonder what is the relative difficulty level of the one we are taking. So rank your top 3 hardest AIMEs in order. It would help me a lot. :| \r\n1. 1994\r\n2. 1995\r\n3. 1999\r\n\r\nAlso, comment on other people's lists if you want.", "Solution_1": "I've only taken 83, 84, 85, 86, 87, 89, 2000A. Here's the order in which I'd rank them, hardest to easiest.\r\n\r\n1. 2000A\r\n2. 85\r\n3. 87\r\n4. 83, 84, 86, 89: all very easy.", "Solution_2": "1. 1994\r\n2. 1992\r\n3. 199-something?\r\n\r\nI only remember those two being particular difficult for me... that was a while ago though. I can say 1983-1990 are pretty much the easiest, though...", "Solution_3": "Anything from 1993-1997 were all fairly hard...my average on those is about 3 less than the others.", "Solution_4": "Hmm... \r\n\r\n 1st place: 1994, 1999, 1993\r\n\r\nOn these two AIME's I solved 12 problems\r\n\r\n2nd place: 1993, 1996, 1997, 1998, 2003A\r\n\r\nOn these I solved 13 problems total, by the time i figure out one of the problems left I ran out of time. \r\n\r\nSo yeah, difficulty of an AIME does not affect me very much\r\n\r\nIronically my highest score out of these is 1999(no careless mistakes!), the one I considered to be one of the hardest. So really careless mistakes control how well you do on AIME.", "Solution_5": "beta I don't think your personal story of what happened to you in terms of careless mistakes affects the actual difficulty of the contest....\r\n\r\n1. 1994\r\n\r\nCan't think of another one that really sticks out.", "Solution_6": "My philosophy is that you should train, in part, for problems more difficult than the ones you will encounter. With that in mind, the bunch of Mock AIMEs I wrote last year were very difficult. I tried to make them harder than the 1994 AIME (hardest real one in my opinion.)", "Solution_7": "Out of curiousity, what was the floor on the 1994? I'm guessing 5 or 6.", "Solution_8": "There was no such thing as floor until 2002, partly because the AMC 10 didn't exist until 2001? (Maybe 2000)", "Solution_9": "Ah. So it was determined by AMC12+AIME index?", "Solution_10": "You mean AHSME. AMC 12 and AMC 10 replaced the AHSME.", "Solution_11": "I'm confused. I've seen a number of problems from before 2001 posted on here that were titled AMC12 so-and-so. Did the poster just put the wrong name?", "Solution_12": "The name was changed in 2000, according to [url=http://www.unl.edu/amc/e-exams/e6-amc12/factoids12.html]their site[/url].", "Solution_13": "Now that people are preparing for the 2006 AIME I and AIME II, it would be interesting to hear more opinions about which previous AIMEs are most challenging, and which are most easy. If I remember correctly (please correct me if I'm wrong), last year's AIME I (that is, the 2005 AIME I) was quite approachable to test-takers who were familiar with the problem-solving literature. I remember that when I page through it as a coach it seemed to me that many of the problem types were familiar.", "Solution_14": "Personally, my favorite AIME is 1994. (Other than the 1980's, those are too easy :P). 1991 is really fun too, along with 1990 and 1992. 1993 is a bit harder than the others though, and 1995 is the trickiest for me. In fact, it seems that all the 90's AIMEs have good problems... maybe because of all the algebra problems and only a few geometry ones (with the exception of 1995, which is the one I think is hardest :lol:)\r\n\r\nIronically, everyone else thinks that 1994 was hardest. :huh: I haven't taken any of the 2000-2005 AIMEs recently but I think I got like 4 or 5 on 2001 and 2002 a few months ago. :blush:\r\n\r\nIf there's a bunch of geometry problems on AIME 2006, I'll do really bad. :( :(" } { "Tag": [], "Problem": "Okay, this is the player list. Order matters: \r\n\r\ncf249 \r\nmathelete \r\nchenshi\r\nWickedestjr\r\nernie\r\naleph0\r\n\r\nEach player has 3 grenades in their arsenal. There is no concept of turns. No consecutive turns. When you go, you have five options. Do only one of them. \r\n\r\n1. Choose another player, and subtract points. The number of points subtracted is the post number. \r\n\r\n2. Same as above, but add points instead. \r\n\r\n3. Throw a grenade. Convert the post number to binary, then add up the digits. The grenade will go off that many posts later. If it does not go off in 4 days, the grenade is defused. \r\n\r\n4. Bump the grenade to the player below you on the list. If you are at the bottom, it will go to the top. \r\n\r\n5. Pick up a defused grenade. It goes into your arsenal. \r\n\r\nEverybody starts off with 0 points, and they can have a negative number of points. After 75 posts, the game ends. When the grenade goes off, whoever it is by falls for the rest of the game. If a person brings another person's points back to 0, then that person earns a mushroom. If a person doubles a positive number of points, that person earns a star. These are the sixth and seventh options. A mushroom gives yourself points. When a star is used on post x, the person is invincible till post 2x. An invincible person automatically bumps a grenade without posting.", "Solution_1": "Are double posts allowed?", "Solution_2": "No double posting.\r\n\r\ncf249: 0, 3 grenades \r\nmathelete: 0, 2 grenades \r\nchenshi: 0, 3 grenades, GRENADE 5 BEEPING\r\nWickedestjr: 0, 3 grenades\r\nernie: 0, 3 grenades\r\naleph0: 0, 3 grenades\r\n\r\nThrow grenade at chenhsi. It will go off at post 5, so I will call it grenade 5.", "Solution_3": "cf249: 0, 2 grenades \r\nmathelete: 0, 2 grenades \r\nchenshi: 0, 3 grenades, GRENADE 5 BEEPING \r\nWickedestjr: 0, 3 grenades \r\nernie: 0, 3 grenades, Grenade 5b beeping\r\naleph0: 0, 3 grenades \r\n\r\nThrow grenade at ernie. It will also go off on post 5 (4=100 in binary), so I will call it Grenade 5b.", "Solution_4": "cf249: 5, 2 grenades \r\nmathelete: 0, 2 grenades \r\nWickedestjr: 0, 3 grenades \r\naleph0: 0, 3 grenades \r\n\r\nNice job helping with the double kill. For that you get 5 points.", "Solution_5": "wtf\r\n\r\nI didn't even realize I was in this game yet.\r\n\r\nAnd look at my comments in the other thread,", "Solution_6": "cf249: 12, 2 grenades \r\nmathelete: 0, 2 grenades \r\nWickedestjr: 0, 3 grenades \r\naleph0: 0, 3 grenades", "Solution_7": "cf249: 12, 2 grenades \r\nmathelete: 0, 1 grenade \r\nWickedestjr: 0, 3 grenades, GRENADE 9 BEEPING \r\naleph0: 0, 3 grenades\r\n\r\nThrow grenade 9 at wicked. Either wickedestjr or aleph0 will be killed.", "Solution_8": "cf249: 21, 2 grenades \r\nmathelete: 0, 1 grenade \r\nWickedestjr: PWNT\r\naleph0: 0, 3 grenades", "Solution_9": "cf249: 21, 2 grenades \r\nmathelete: 0, 1 grenade \r\naleph0: 10, 3 grenades\r\n\r\ngive aleph 10 points", "Solution_10": "cf249: 21, 2 grenades \r\nmathelete: 11, 1 grenade \r\naleph0: 10, 3 grenades", "Solution_11": "Ernie and chenshi tied for 5th place\r\nWickedestjr in 4th place\r\n\r\nLet us finish this\r\n\r\ncf249: 21, 2 grenades \r\nmathelete: 11, 1 grenade \r\naleph0: -2, 3 grenades\r\n\r\n-12 from aleph", "Solution_12": "cf249: 21, 2 grenades \r\nmathelete: 24, 1 grenade \r\naleph0: -2, 3 grenades", "Solution_13": "cf249: 21, 2 grenades \r\nmathelete: 24, 1 grenade \r\naleph0: -16, 3 grenades\r\n\r\nTake away 14 points from aleph0.", "Solution_14": "Hey! I want to know why I didn't get notified that it restarted. Don't you think it's unfair that someone can lose before they even knew that a game existed?", "Solution_15": "cf249: 21, 2 grenades , GRENADE 17\r\nmathelete: 24, 1 grenade \r\naleph0: -16, 2 grenades", "Solution_16": "Ernie and chenshi tied for 5th place \r\nWickedestjr in 4th place \r\ncf249 in 3rd place\r\n\r\nmathelete: 24, 1 grenade \r\naleph0: 1, 2 grenades\r\n\r\ngive aleph 17 points. Since grenades can be stalled in one-on-one, we will convert them to points. So:\r\n\r\nmathelete: 25 \r\naleph0: 3\r\n\r\nLet us speed this up by ending it at post 32", "Solution_17": "ANSWER MY QUESTION!!!", "Solution_18": "mathelete:25\r\naleph0:22\r\n\r\ngive 19 points to aleph\r\n\r\nchenhsi, I put it in the previous thread. You were too late. \r\n\r\nBTW, you can pass your turn if you want. just say pass", "Solution_19": "mathelete:45 \r\naleph0:22 \r\n\r\nHow does someone win?", "Solution_20": "[quote=\"aleph0\"]mathelete:45 \naleph0:22 \n\nHow does someone win?[/quote]\r\nBy post 36, get more points. \r\n\r\nBTW chenhsi took up a post, so this is really post 20. I pass", "Solution_21": "mathelete:24\r\naleph0:22", "Solution_22": "mathelete:24, star\r\naleph0:44\r\n\r\nSince this is post 22 (chenhsi took up a post by being mad), give aleph 22 points. I gave aleph the lead to earn a star. I plan to use it later.", "Solution_23": "grr...\r\n\r\nWe don't even get time to be notified that the game restarted? You don't even ask if we still want to play? \r\n\r\nAnd wth are you adding new rules in the middle of the game?", "Solution_24": "mathelete:48, star \r\naleph0:44, star\r\n\r\nI lose...", "Solution_25": "chenshi, you wasted another post. I ended up redoing everything because people did not follow directions. This is really post 25. I use the star. Since it would normally last the rest of the game, I will make it last till post 32 to be nice. Now you can only give me points till then.\r\n\r\nmathelete:48, invincible \r\naleph0:44, star(not used yet)", "Solution_26": "That's not my point. I have no objections with you restarting the game. My points is that you should have notified us before the second game started, and you should have allowed more players. And I dislike the fact that you ended a game without asking the players. What if we wanted to continue playing the game, even if we aren't using your rules?", "Solution_27": "Since I earned the star at 22, and aleph earned it at 24, we will convert them to points, so I win by 2!\r\n\r\n1st-mathelete\r\n2nd-aleph\r\n3rd-cf249\r\n4th-wicked\r\n5th-chenhsi and ernie\r\n\r\nI will post a new topic. Chenhsi, you are in.", "Solution_28": "I want to be in." } { "Tag": [ "search" ], "Problem": "There are positive integers $ k$, $ n$, and $ m$ such that $ \\frac{19}{20} < \\frac{1}{k}\\plus{}\\frac{1}{n}\\plus{}\\frac{1}{m} < 1$. What is the smallest possible value of $ k\\plus{}n\\plus{}m$?\r\n\r\nI brute forced my way though this problem during practice, but does anyone have a solution?\r\n :lol:", "Solution_1": "[url=http://www.artofproblemsolving.com/Forum/viewtopic.php?search_id=1742539080&t=77789]Here[/url]'s some discussion, but I didn't find the explanations too satisfying.\r\n\r\n[hide=\"My Solution\"]\nWLOG, let $ k\\le n\\le m$.\n$ k\\equal{}1$ will make this at least $ 1$, which doesn't work.\n$ k\\equal{}2$ means that $ m\\ge n\\ge3$, since if either of them were $ 2$ it would be at least $ 1$. If $ n\\equal{}3$, then some testing gives the smallest possible value of $ m$ that works as $ 7$, so $ (2,3,7)$ works. If $ n\\equal{}4$, then $ m\\equal{}4$ would make this equal to $ 1$ and $ m\\ge4$ would make this less than or equal to $ \\frac{19}{20}$, both of which don't work. $ n\\ge5$ would make this expression less than $ \\frac{19}{20}$.\n$ k\\equal{}3$ gives the maximum at $ (3,3,3)$, but this equals $ 1$ and doesn't work. If we change it to $ (3,3,4)$, it's a bit less than $ \\frac{19}{20}$, and any more increasing will make this value even smaller.\n$ k\\ge4$ makes the expression less than or equal to $ \\frac34$.\nWe conclude that $ 2\\plus{}3\\plus{}7\\equal{}\\boxed{12}$ gives the minimum value.\n[/hide]" } { "Tag": [ "geometry", "rhombus" ], "Problem": "The quesiton below is from '94 State competition, Team Round.\r\n\r\n[img]http://www.geocities.com/danb80/quest.6.jpg[/img]\r\n\r\nCan anyone help me figure out how to solve it?\r\n\r\n[hide]the answer is 91[/hide]\r\n\r\nThanks!", "Solution_1": "[hide=\"hint\"]inscribe the circles inside a rhombus. You should be able to find the length of one of the diagonals as well as one of the sides. Then find the other diagonal using rhombus properties and then the area of the rhombus can be found. [/hide]", "Solution_2": "Not [i]exactly[/i] sure about this but...\r\n\r\n[hide]\nLet's assume that the circles have radius 1.\n\nNow, draw an equilateral triangle with side length 2 from the 3 circles as shown. It should be clear that the whole plane can be tiled with that triangle. The area of this triangle would be $\\sqrt{3}$. Now find the area of the sector bound by the sides of the triangle in one of the circles: $\\frac{\\pi}{6}$. Since there are 3 such sectors multiply that by 3 to get $\\frac{\\pi}{2}$. Divide that by $\\sqrt{3}$ to get $\\frac{\\pi\\sqrt{3}}{6}\\approx0.90689968211710892529703912882108\\approx91\\%$\n[/hide]", "Solution_3": "Thanks!\r\n\r\nI thought of the rhombus solution, but it doesn't account for the other circles around.\r\n\r\nThe triangle method seems to work!", "Solution_4": "[hide]\nDo the triangle thingy like ccy said.\n\nYou can say that the circles have a radius of $1$\nThus, the area of the triangle is $\\frac{4\\sqrt{3}}{4}=\\sqrt{3}$\nThe amount of the circles that is inside the triangles is $\\frac 12$ of a circle which is $\\frac{\\pi}{2}$\nThus, for every $\\sqrt{3}$ triangle, $\\frac{\\pi}{2}$ is covered.\nThe answer is thus\n$\\frac{\\frac{\\pi}{2}}{\\sqrt{3}}\\approx.906\\approx91\\%$[/hide]", "Solution_5": "This problem has already been posted.\r\n\r\nhttp://www.artofproblemsolving.com/Forum/viewtopic.php?t=57395&highlight=" } { "Tag": [], "Problem": "The graph of $ y\\equal{}ax^2\\plus{}bx\\plus{}c$ passes through points $ (0,5)$, $ (1,10)$, and $ (2,19)$. Find $ a\\plus{}b\\plus{}c$.", "Solution_1": "We're given the answer: let $ f(x)\\equal{}ax^2\\plus{}bx\\plus{}c$; $ f(1)\\equal{}a\\plus{}b\\plus{}c\\equal{}\\boxed{10}$.", "Solution_2": "[quote=\"GameBot\"]The graph of $ y \\equal{} ax^2 \\plus{} bx \\plus{} c$ passes through points $ (0,5)$, $ (1,10)$, and $ (2,19)$. Find $ a \\plus{} b \\plus{} c$.[/quote]\r\n\r\nSecond point: $ 10\\equal{}a\\plus{}b\\plus{}c$." } { "Tag": [ "University of Chicago", "Stanford", "college", "MIT" ], "Problem": "Can anyone give any information about Northwestern U ? How good are the academics for math/physics and also the ISP programme? How does it compare with UChicago ?", "Solution_1": "I think I can help you out. I applied to and got into both Northwestern and ISP, however, I'm actually going to be a freshman this fall at Caltech, so I can't say I have a 100% accurate view. NU is a very well respected research university, with excellent facilities, especially in nanotechnology and bio-engineering related stuff. Their chemistry department seems to be pretty amazing, and ISP kids usually either double major in chemistry, or drop out altogether of ISP and do chemistry. Also, physics there seems to be pretty respected (one physics major I met was a goldwater scholar and going to Stanford for grad), though maybe not as much as chemistry. I don't know about math (but I imagine that it's pretty decent too, though not UChicago) \r\n\r\nAs for ISP itself, it'll give you a very solid background in the sciences, though moreso in math and physics than biology. It's a 3 year program that basically covers the same core class requirements of Caltech and MIT, and then goes a little beyond. It's almost as hard as those respective classes at MIT and Caltech, though no Techer will ever admit it. It's really easy to double major with ISP in math, chem, bio, physics, and materials science, and possible (though not as common) in other scientific or engineering majors. Also, ISP gives more respect and status in terms of getting attention for research at NU, and all ISP classes are taught by professors in classes of 20 or less people.\r\n\r\nThe kids I met at NU were all fairly bright and talented one way or another. The ISP kids I met were basically kids who were certainly smart enough to get into say MIT or Caltech but didn't due to the randomness of admissions. ISP seniors were mostly going to grad school, and Stanford seemed to be the popular choice among the ones I met. The acceptance rate for ISP applicants was 30% (and they had to get into NU before being considered for ISP). Also, some ISP kids do become Goldwater and Gates scholars and whatnot. \r\n\r\nNU has a huge rivalry with UChicago, and I'd say UChicago probably has a very slight edge on NU for science, though ISP seems to bridge that gap. UChicago math is pretty incredible. However, I only visited UChicago once, so what I know about them is mostly from rumors.\r\n\r\nSorry, it got to be a bit long. Hope this helps?", "Solution_2": "Thanks a lot. I was deciding whether to add NU to my already long list.\r\nI was also wondering, why is NU not that well known (on this forum for example)? UChicago also seems more popular than NU.", "Solution_3": "According to the website, it is need-blind for citizens and is a member of the common application. So, applying would make [i]some[/i] sense \r\nBut won't the high school GPA be very important there? We have low GPAs in high school. Is there any need-blind college where the high school GPA is [i]relatively[/i] less important especially when the gpa is from a foreign country?(in my case, India) The rest of my application (SAT, recommendations, etc) is alright, though not outstanding.", "Solution_4": "I'm sure plenty of the people here have heard of NU, it's just not talked about as much because most people here want to be math majors, and UChicago is considered a \"top 5 math school\", and thus many people here would probably look at UC for math, but not NU. NU however seems to be a more well rounded school, as it has an engineering school, and has plenty of special programs for applied math and science (like ISP, MMSS, etc.), that are especially valuable for later careers in the financial sector or MBA programs. Unless you want to go hard core theoretical math, I think you really can't go wrong choosing between NU and UC. \r\n\r\nI would think all need-blind schools consider GPA very important, as most of them are rigorous and need to see that you can do the work. That said, schools that are members of the common app are supposed to be committed to \"holistic admissions\", and look at your application in the context of your background, ie, they would see that schools in your country give lower GPA's, and would take that into account when considering your application. That said, it is very competitive for international students to gain admissions to most needblind schools, but I can't give you any specifics, as there are so many different schools out there with slightly different admissions policies.", "Solution_5": "Actually, I wanted to major in pure mathematics. So U.Chicago might be a better place for me than Northwestern. But I lack outstanding achievements like IMO (didn't do well at the selection camp) and my GPA is low. Hence, getting into a high ranked college might be difficult.\r\nHowever, I am [i]not[/i] an international student (although my school education has been from India). So getting into a need-blind college is what I have to worry about." } { "Tag": [], "Problem": "Other than using web.archive.org, can anybody tell me where the most complete compilation of old national competition results is? Thanks.", "Solution_1": "Please...I would really like these results. Thanks.", "Solution_2": "Why?", "Solution_3": "Maybe he wants to build a time machine, go back in time, make the winners really sick before the competition and make his state win..... :lol: :P" } { "Tag": [ "calculus", "integration", "inequalities", "function", "real analysis", "real analysis solved" ], "Problem": "The function $f(x)$ satisfies $f'(x)\\geqq0$ at $-1\\leqq x\\leqq 1$.\r\nProve the following inequality.\r\n\r\n\\[\\int^1_{-1}\\frac{x}{\\sqrt{1+x^2}}f(x)dx\\geqq0\\]", "Solution_1": "Sorry for my misstyping.\r\n\r\nI have just edited.\r\n\r\nBest regards.\r\n\r\nkunny", "Solution_2": "It's simply integration by parts. The LHS is $\\int^1_{-1}(\\sqrt{x^2+1})'f(x)dx=\\sqrt 2\\cdot(f(1)-f(-1))-\\int^1_{-1}\\sqrt{x^2+1}f'(x)dx$. We now use the fact that $f'$ is non-negative and the fact that $\\sqrt{x^2+1}\\le\\sqrt 2$ to prove that $\\int^1_{-1}\\sqrt{x^2+1}f'(x)dx\\le \\sqrt 2\\cdot\\int^1_{-1}f'(x)dx=\\sqrt 2\\cdot(f(1)-f(-1))$, Q.E.D.", "Solution_3": "You are right, grobber.\r\n\r\nThis problem is a piece of cake for you,isn't it?\r\n\r\nkunny" } { "Tag": [ "geometry", "trigonometry", "geometry unsolved" ], "Problem": "Given a triangle ABC and a point P on AB with AP>PB, Show how to construct a point Q on AC such that the triangle APQ will have one-half the area of triangle ABC.\r\n\r\nI have gotten to the point where I drew a perpendicular line from H on AB that goes to point C, then a parallel line above this line will intersect AC at Q and AB at F. I am now stuck trying to determine how to find the distance between H and F which would enable drawing a parallel line to find Q. Any ideas??", "Solution_1": "$()$ denotes the area:\r\n\r\n$(ABC) = \\frac{1}{2}AB \\cdot AC \\cdot \\sin{ \\widehat{A}}$\r\n\r\n$(APQ) = \\frac{1}{2}AP \\cdot AQ \\cdot \\sin{ \\widehat{A}}$\r\n\r\n$(ABC) = 2(APQ)\\Rightarrow$\r\n\r\n$AB \\cdot AC = 2AP \\cdot AQ \\Rightarrow$\r\n\r\n$\\frac{AQ}{AC} = \\frac{AB}{2AP}$\r\n\r\none way to find the point $Q$ is:\r\n\r\n[list][b]-[/b] find a point $F$ on the ray $(AB$ such that $AF=2AP$ \n\n (since $AP>PB$ you'll have $2AP>AB$, so $F$ is out of the segment $AB$)[/list]\n[list][b]-[/b] connect $F$ and $C$[/list]\n[list][b]-[/b] bring a parallel from $B$ to $FC$. This line will intersect the segment $AC$ at the point $Q$[/list]", "Solution_2": "look:\r\nhttp://www.mathlinks.ro/Forum/viewtopic.php?highlight=shown&t=26105" } { "Tag": [ "analytic geometry", "graphing lines", "slope", "geometry" ], "Problem": "A line with slope 6 bisects the area of the unit square with vertices (0,0); (0,1); (1,1); and (1,0). What is the y-intercept of this line?", "Solution_1": "[quote=\"4everwise\"]... bisects the area of the unit square with vertices (0,0); (0,1); (1,1); and (1,0) ...[/quote]\r\nBy this do you mean that the line with slope six runs through the square with given vertices :?", "Solution_2": "[hide]Since the line bisects the area of a square, it should pass throught center, which is $(\\frac{1}{2},\\frac{1}{2}$. \n\n$y=mx+b$\n\n$\\frac{1}{2}=3+b$\n\n$b=-2\\frac{1}{2}$[/hide]" } { "Tag": [], "Problem": "to continue the over-personal questions that would allow a stalker to find out more about you than should be known, how much does your car insurance cost per month. i have to pay $155 a month for minimum insurance. and i only drive a 96 neon. i think i'm getting ripped off.", "Solution_1": "errr...not old enough to drive yet :(", "Solution_2": "I'm a male 17 year old student and I been in an accident...\r\n\r\n\r\n :(", "Solution_3": "I have not got the chance to drive a car... ... :(", "Solution_4": "Can't drive, don't really want to. Well, I suppose that I could get a license, but I don't feel like paying for a car and insurance.", "Solution_5": "Im 13...can't drive", "Solution_6": "does anyone drive?", "Solution_7": "Of course not!! ;)", "Solution_8": "Haha...still waiting on that 06 model of the Mercedes SL65 AMG. *hop*", "Solution_9": "I drive a 94 Land Cruiser and am paying less than you; yeah, I think you're getting ripped off.\r\nBTW, have you submitted your grades to the insurance company? Some will give you a discount for that sort of thing. (I also had driver's ed, so mine is probably lower than normal).", "Solution_10": "[quote=\"miraculouspostmaster\"]I drive a 94 Land Cruiser and am paying less than you; yeah, I think you're getting ripped off.\nBTW, have you submitted your grades to the insurance company? Some will give you a discount for that sort of thing. (I also had driver's ed, so mine is probably lower than normal).[/quote]\r\n\r\nDo you currently live in the US. Here (according to some european friend, and my own experience) insurance is, first, in general, more expensive, second, a lot more expensive if you are younger than 25! (and so am I!)\r\n\r\nBest,", "Solution_11": "too young to drive...nearly 16 :(" } { "Tag": [ "real analysis", "function", "real analysis unsolved" ], "Problem": "Prove that if $ E \\subset \\mathbb{R}$ is a bounded set of positive Lebesgue measure, then for every $ u < 1/2$, a point $ x\\equal{}x(u)$ can be found so that \\[ |(x\\minus{}h,x\\plus{}h) \\cap E| \\geq uh\\] and \\[ |(x\\minus{}h,x\\plus{}h) \\cap (\\mathbb{R} \\setminus E)| \\geq uh\\] for all sufficiently small positive values of $ h$. \r\n\r\n\r\n[i]K. I. Koljada[/i]", "Solution_1": "for the first ques, i wonder if it's true. Consider the following measure supported on [0,1]. $ A_{1}\\equal{}[\\frac{1}{8},\\frac{2}{8}]\\bigcup [\\frac{6}{8},\\frac{7}{8}]$. $ A_{1}$ seperate [0,1] into three parts. $ B_{1}\\equal{}[\\frac{1}{64},\\frac{2}{64}]\\bigcup [\\frac{6}{64},\\frac{7}{64}]$, that is, analog to $ A_{1}$ respect to $ [0,\\frac{1}{8}]$.Make $ B_{2}$ in the same way from $ [\\frac{2}{8},\\frac{6}{8}]$, and B3\u2026\u2026 Generally, each step, add a set made in the above way from every section that has not been contained in E into E, cut a set (as we do to [0,1] in the first step) from every section that has already in E. It's obvious that the measure conserves to be $ \\frac{1}{4}$. But it seems for every x and h, if (x-h,x+h) $ \\in E$ then $ |(x\\minus{}h,x\\plus{}h)\\cap E|\\equal{}\\frac{1}{4}h$\r\n\r\nas to the second ques. Since E is bounded then every x far away enough from E satisfy $ |(x\\minus{}h,x\\plus{}h)\\cap (\\mathbb R \\setminus E)|\\equal{}2h$", "Solution_2": "Sory for the wrong answer. The set I made was not definite. Maybe you can try Rusin theory.", "Solution_3": "Solution:\nWe will prove problem for every $u<1/4$, let $u=1/4-\\varepsilon$ for $\\varepsilon>0$. First let's find interval $I^1:\\frac{|I\\cap E|}{|I|}=\\frac{|I\\cap (\\mathbb{R}\\setminus E)|}{|I|}=\\frac{1}{2}$. After Lebesgue's density theorem we get that there exists point $a:\\exists lim_{h\\rightarrow 0}\\frac{|(a-h,a+h)\\cap E|}{2h}=1$, so there exists $h:\\frac{|(a-h,a+h)\\cap E|}{2h}>\\frac{1}{2}$, E is bounded set, so there exists point $b:\\frac{|(a-h,a+h)\\cap E|}{2h}=0<\\frac{1}{2}$, denote function $\\phi:[a,b]\\rightarrow \\mathbb{R}$, where $\\phi(x)=\\frac{|(x-h,x+h)\\cap E|}{2h}$, $\\phi\\in C([a,b])$, so there exist point $c\\in (a,b):\\frac{|(c-h,c+h)\\cap E|}{2h}=\\frac{1}{2}$, so $I^1=(c-h,c+h)$. Let intervals $I_1,I_2:|I_1|=|I_2|=\\frac{|I^1|}{2},I_1\\sqcup I_2=I^1$, easy to see that $min(\\frac{I_1\\cap E}{|I_1|},\\frac{I_2\\cap E}{|I_2|})\\leq \\frac{1}{2}\\leq max(\\frac{I_1\\cap E}{|I_1|},\\frac{I_2\\cap E}{|I_2|})$, so there exist interval $I^2:\\frac{|I^2\\cap E|}{|I^2|}=\\frac{1}{2}-\\delta,|\\delta| \\leq2\\varepsilon,[I^2]\\subset I^1,|I^2|=|I_1|=|I_2|=\\frac{|I^1|}{2}$. Like the same construct intervals $I^2,I^3,...$, where $[I_{n+1}]\\subset I_n, |I_{n+1}|=\\frac{|I_n|}{2}\\ \\forall n$, let $x(u)=\\cap_{i=1}^{\\infty}I^{i}$, $x(u)\\in I^1$, so there exist $\\tau >0:O_{\\tau}(x(u))\\subset I^1$, for every $h<\\tau$ there exist $n_h\\in N:I^{n_h}\\subset O_h(x(u)),|I^{n_h}|\\geq \\frac{h}{2}$, so $|(x(u)-h,x(u)+h)\\cap E|\\geq |I^{n_h}\\cap E|\\geq (\\frac{1}{2}-2\\varepsilon)|I^{n_h}|\\geq (\\frac{1}{4}-\\varepsilon)h=uh$, like the same $|(x(u)-h,x(u)+h)\\cap (\\mathbb{R}\\setminus E)|\\geq uh$. done" } { "Tag": [ "superior algebra", "superior algebra solved" ], "Problem": "Show that the cyclotomic fiedl E8=Q(i, \u221a2).\r\n\r\nThank you very much!", "Solution_1": "It is easy to prove that the $8$-th cyclotomic field is contained in $\\mathbb Q(i,\\sqrt 2)$. Just write down the $8$-th roots of unity to see this.\r\nWe have then $\\mathbb Q\\subseteq E_8\\subseteq \\mathbb Q(i,\\sqrt 2)$. We have $[E_8: \\mathbb Q]=\\varphi(8)=4$ and $[\\mathbb Q(i,\\sqrt 2): \\mathbb Q]=[\\mathbb Q(i,\\sqrt2): \\mathbb Q(i)]\\cdot[\\mathbb Q(i): \\mathbb Q]=2\\cdot 2=4$. It follows $E_8=\\mathbb Q(i,\\sqrt 2)$.", "Solution_2": "Thank you very much Amfulger!" } { "Tag": [ "puzzles" ], "Problem": "Jerry wins the race, but he gets no trophy.\r\n\r\nHow is this possible?", "Solution_1": "[hide]Jerry is a horse. His rider gets the trophy[/hide]", "Solution_2": "[hide] He gets a medal or a cash prize or a certificate....or this is a horse race and jerry is the horse...i dunno! :D [/hide]" } { "Tag": [], "Problem": "just thought i'd get this started. \r\ni'm in ap bio. who feels ap has too much memorization? i know i do. hope everyone is ready for the ap exam or will be on exam day.", "Solution_1": "There are mountains of things to memerize in biology. It is not like math or physics, in which you can solve a lot of problems by talent. No hints of studying it, just memerize and do exercises.", "Solution_2": "[quote=\"furious\"]just thought i'd get this started. \ni'm in ap bio. who feels ap has too much memorization? i know i do. hope everyone is ready for the ap exam or will be on exam day.[/quote]\r\n\r\nYou just described why I lost interest in bio.\r\n\r\nHowever, I do think computers are going a long way to bringing mathematics and rigor into biology, not to mention the vast amounts of money in biotech drawing better and better people into the field. I know a handful of olympiad types who are now 'computational biologists', melding math, cs, and bio much the same way string theorists mix math and physics.", "Solution_3": "Unfortunately, a modern AP biology class is essentially a \"memorize a book\" class. This kind of class has eliminated any thoughts of a potential biology career for me.", "Solution_4": "Exactly why I don't like biology! It's way too much memorization (that's so unscientific, even history is a bit more flexible)!", "Solution_5": "I dont like biology at all (same reason)..........but I want to be a doctor. Not only is it boring to memorize everything, i feel its also not very interesting. I hope this forum could clear up some confusion.", "Solution_6": "All sciences have some amount of memorization required for study. Biology has a lot of this, not because it wants to bore you with its repetitiveness, but because it is an inherently structured science. It is also by far the most expansive science. Studying ecology and evolution can be extremely different from studying development and genetics, and most people find their biology niche (no pun intended) and then delve into that specific area. Others like the complete unification that biology presents because though all the different parts of biology can be vastly different to study, evolution and molecular chemistry and processes are prevalent motifs and unify the whole idea of life sciences very well.\r\n\r\nAP Biology is a class meant to get you to learn all the basics of biology and also to teach how all the different parts are unified by evolution. When you look into the specific topics that biology can present, you will understand the great magnitude of this science. How amazing is it that the pGLO gene present in some bioluminescent cnidarians can be transferred into Arabidopsis thaliana plants to make them \"glow\"? How amazing is it that the fusion of two haploid nuclei at fertilization can carry all of the information necessary to build a functional, living organism that has such complex cognition as man? These are the phenomena of biology and some things that AP Biology will present to you.\r\n\r\nDon't let AP Biology turn you off to the science. In college, biology (especially laboratory) becomes less memorization and a whole lot more chemistry and physics (especially in the submicroscopic fields of biology). If you choose to do zoology or ecology, however, you probably enjoy the memorization of systematics but the laboratory experiences in those fields are also amazing.\r\n\r\nIn conclusion, not only is biology an ever-growing frontier science, it is the science that is most applicable to everyday life and the science that growing amounts are learning to cherish for its widespread scope and intrigue.", "Solution_7": "Just wondering, I want to be doctor as well (not sure what kind) but then, should I have my major on biology?\r\n\r\nI guess biology is a lot of memorizing but it's actually pretty fun once you get the concept. Also, my biology teacher actually taught me one useful thing this year and that is \"look for the words you know.\"\r\n\r\nLike if you have \"sporophyte\" and you know \"phyte\" means plant and \"sporo\" means spore, you can see that the word \"sporophyte\" is a plant that produces spores for the reproduction. :) \r\n\r\nBut do I have to take major as biology if I want to be doctor? Is it recommended that way?", "Solution_8": "i'm not sure if your major has to biology, but i would assume it would be in a doctor's best interest to major in some branch of biology or science. \r\nanywho, in ap bio, i enjoyed the first half of the year when we were learning celluar processes like respiration and photosynethesis as well as genetics. but i hate the second half of the year because it is 100% memorization - memorize plants, memorize invertabrate, memorize the body systems, etc. o well.", "Solution_9": "you definitely do not have to be a biology major to become a doctor - even an english or music major can apply for med school provided they have the required courses... which are usually covered by the core requirements at top universities...", "Solution_10": "[quote=\"tetrahedr0n\"]Unfortunately, a modern AP biology class is essentially a \"memorize a book\" class. This kind of class has eliminated any thoughts of a potential biology career for me.[/quote]\r\nI couldn't agree more.", "Solution_11": "a science degree is not even required to become a medical doctor, though it would be quite helpful. pre-med is a very common major for aspiring physicians.", "Solution_12": "yes.... many people find that there is lot of stress on memomorisation in biology \r\ni agree with what silverfalcon said... :) if the teacher said it becomes easier if you know the terminology, therefore it depends a lot on the teachers who explain the concepts if a little effort is showwn towards the understanding of the terminology in the lower classes it is easy to catch what the teachres are talking in the higher classes especially in the chapters like systematics as addidasty ;)" } { "Tag": [ "LaTeX", "inequalities", "inequalities proposed" ], "Problem": "Let be $ a,b,c>0$ such that $ abc\\equal{}1$ . Show that :\r\n\r\n$ \\frac{(a^2\\plus{}2b^2)(b^2\\plus{}2c^2)(c^2\\plus{}2a^2)}{a^2b\\plus{}b^2c\\plus{}c^2a}\\ge 9$", "Solution_1": "Hello, :D \r\n\r\n\r\n$ {a^2b \\plus{} b^2c \\plus{} c^2a}\\geq 3 \\sqrt [3]{a^3 b^3 c^3} \\equal{} 3 \\implies \\frac {(a^2 \\plus{} 2b^2)(b^2 \\plus{} 2c^2)(c^2 \\plus{} 2a^2)}{a^2b \\plus{} b^2c \\plus{} c^2a}\\geq \\frac {(a^2 \\plus{} 2b^2)(b^2 \\plus{} 2c^2)(c^2 \\plus{} 2a^2)}{3}$\r\n\r\nNow it is enought to prove that $ (a^2 \\plus{} 2b^2)(b^2 \\plus{} 2c^2)(c^2 \\plus{} 2a^2)\\geq 27$\r\n\r\nBy AM-GM \r\n \r\n$ a^2 \\plus{} 2b^2 \\equal{} a^2 \\plus{} b^2 \\plus{} b^2 \\geq 3\\sqrt [3]{a^2 b^4} ,$ \r\n\r\n $ b^2 \\plus{} 2c^2 \\equal{} b^2 \\plus{} c^2 \\plus{} c^2 \\geq 3\\sqrt [3]{b^2 c^4} ,$\r\n\r\n $ c^2 \\plus{} 2a^2 \\equal{} c^2 \\plus{} a^2 \\plus{} a^2 \\geq 3\\sqrt [3]{c^2 a^4} ,$\r\n\r\nMultiplying we have $ {(a^2 \\plus{} 2b^2)(b^2 \\plus{} 2c^2)(c^2 \\plus{} 2a^2)}\\geq 27\\sqrt [3]{a^6 b^6 c^6} \\equal{} 27$ as desired.\r\n\r\n\r\nBy the way ,How to center LaTeX ?\r\n\r\nSorry: IGNORE THIS POST !", "Solution_2": "But you are wrong in the first step... :maybe:", "Solution_3": "[quote=\"Jianxing113725\"]But you are wrong in the first step... :maybe:[/quote]\r\n\r\nOkay!..It's not easy :lol: ..\r\nI will try", "Solution_4": "Can you tell me where exactly is my mistake.Is it typo,or it is wrong ?", "Solution_5": "[quote=\"enndb0x\"]Can you tell me where exactly is my mistake.Is it typo,or it is wrong ?[/quote]\r\n\r\n$ (a^2\\plus{}2b^2)(b^2\\plus{}2c^2)(c^2\\plus{}2a^2) \\ge 27$\r\nBut:\r\n$ 9(a^2b\\plus{}b^2\\plus{}c^2) \\ge 27$", "Solution_6": "i think the condition $ abc\\equal{}1$ is not correct... and i think the right condition should be $ abc\\ge 1$ for $ a,b,c>0$\r\n\r\n\r\n\r\nand that could be proven easily using enndb0x's method with some small improvements \r\n\r\nSo... that's it :D:D", "Solution_7": "You're right . can you post your solution ?", "Solution_8": "I will prove that this inequality is always true for $ a,b,c>0$ when $ abc\\ge 1$ .\r\n\r\n first by enndbox (with improvement)\r\n\r\n $ {a^{2}b+b^{2}c+c^{2}a}\\geq 3\\sqrt [3]{a^{3}b^{3}c^{3}}= 3 abc$\r\n\r\n Now we have to prove that \r\n\r\n $ (a^{2}+2b^{2})(b^{2}+2c^{2})(c^{2}+2a^{2})\\geq 27abc$\r\n\r\n By AM-GM\r\n\r\n $ a^{2}+2b^{2}= a^{2}+b^{2}+b^{2}\\geq 3\\sqrt [3]{a^{2}b^{4}},$\r\n\r\n $ b^{2}+2c^{2}= b^{2}+c^{2}+c^{2}\\geq 3\\sqrt [3]{b^{2}c^{4}},$\r\n\r\n $ c^{2}+2a^{2}= c^{2}+a^{2}+a^{2}\\geq 3\\sqrt [3]{c^{2}a^{4}},$\r\n\r\n Multiplying all of them we have \r\n\r\n $ {(a^{2}+2b^{2})(b^{2}+2c^{2})(c^{2}+2a^{2})}\\geq 27\\sqrt [3]{a^{6}b^{6}c^{6}}= 27{a^{2}b^{2}c^{2}}$\r\n\r\n by this we have :\r\n\r\n $ 27{a^{2}b^{2}c^{2} \\geq 27abc}$\r\n\r\n $ {a^{2}b^{2}c^{2} \\geq abc}$\r\n\r\n and finally from that we have :\r\n\r\n $ abc\\ge 1$\r\n\r\n So this is proven :D:D", "Solution_9": "[quote=\"SHP3ND1\"]I will prove that this inequality is always true for $ a,b,c > 0$ when $ abc\\ge 1$ .\n\n first by enndbox (with improvement)\n\n $ {a^{2}b + b^{2}c + c^{2}a}\\geq 3\\sqrt [3]{a^{3}b^{3}c^{3}} = 3 abc$\n\n Now we have to prove that \n\n $ (a^{2} + 2b^{2})(b^{2} + 2c^{2})(c^{2} + 2a^{2})\\geq 27abc$\n\n By AM-GM\n\n $ a^{2} + 2b^{2} = a^{2} + b^{2} + b^{2}\\geq 3\\sqrt [3]{a^{2}b^{4}},$\n\n $ b^{2} + 2c^{2} = b^{2} + c^{2} + c^{2}\\geq 3\\sqrt [3]{b^{2}c^{4}},$\n\n $ c^{2} + 2a^{2} = c^{2} + a^{2} + a^{2}\\geq 3\\sqrt [3]{c^{2}a^{4}},$\n\n Multiplying all of them we have \n\n $ {(a^{2} + 2b^{2})(b^{2} + 2c^{2})(c^{2} + 2a^{2})}\\geq 27\\sqrt [3]{a^{6}b^{6}c^{6}} = 27{a^{2}b^{2}c^{2}}$\n\n by this we have :\n\n $ 27{a^{2}b^{2}c^{2} \\geq 27abc}$\n\n $ {a^{2}b^{2}c^{2} \\geq abc}$\n\n and finally from that we have :\n\n $ abc\\ge 1$\n\n So this is proven :D:D[/quote]\r\nYOY ARE WRONG!!!Look again your first step :wink:\r\nYou prove that $ \\frac{(a^2+2b^2)(b^2+2c^2)(c^2+2a^2)}{3abc} \\geq 9$\r\nBut $ \\frac{(a^2+2b^2)(b^2+2c^2)(c^2+2a^2)}{3abc} \\geq \\frac{(a^2+2b^2)(b^2+2c^2)(c^2+2a^2)}{\\sum a^2b}$\r\nSo you didnt prove anything", "Solution_10": "Well ... if the expression above is greater or equal than $ 27{a^{2}b^{2}c^{2}}$ and the expression under that is greater or equal than $ 3 abc$ ... then if we divide first with the second we will have :\r\n\r\n$ \\frac { 27a^{2}b^{2}c^{2}}{3abc}$ which is greater or equal with $ 9$ for $ abc\\ge 1$ \r\n\r\n\r\nSo... this is right... i can't see any mistake here :P :rotfl:", "Solution_11": "What you actually say is that :\r\n\r\n$ A>B$ and $ A>C \\Rightarrow B>C$ (which can be false) , where $ A\\equal{}\\frac{(a^2\\plus{}2b^2)(b^2\\plus{}2c^2)(c^2\\plus{}2a^2)}{3abc}$ , $ B\\equal{}\\frac{(a^2\\plus{}2b^2)(b^2\\plus{}2c^2)(c^2\\plus{}2a^2)}{a^2b\\plus{}b^2a\\plus{}c^2a}$ and $ C\\equal{}9$", "Solution_12": "[quote=\"SHP3ND1\"]Well ... if the expression above is greater or equal than $ 27{a^{2}b^{2}c^{2}}$ and the expression under that is greater or equal than $ 3 abc$ ... then if we divide first with the second we will have :\n\n$ \\frac { 27a^{2}b^{2}c^{2}}{3abc}$ which is greater or equal with $ 9$ for $ abc\\ge 1$ \n\n\nSo... this is right... i can't see any mistake here :P :rotfl:[/quote]\r\nYou have a mistake my friend...\r\n$ a^2b + b^2c + c^2a \\geq 3abc$\r\nBut $ \\frac {1}{a^2b + b^2c + c^2a} \\leq \\frac {1}{3abc}$\r\nSo you prove that an expresion is greater that $ 9$,but simultanously this expresion is also greater that what you have to prove..\r\nLet me put another way:we have to prove that $ b \\geq c$...If we find a number $ a$ so that $ a \\geq b$ and $ a \\geq c$ it doesnt mean that $ b \\geq c$....", "Solution_13": "[quote=\"Dimitris X\"]\nLet me put another way:we have to prove that $ b \\geq c$...If we find a number $ a$ so that $ a \\geq b$ and $ a \\geq c$ it doesnt mean that $ b \\geq c$....[/quote]\r\n\r\nI guess you didn't see my post . I explained him the same thing :P", "Solution_14": "[quote=\"alex2008\"][quote=\"Dimitris X\"]\nLet me put another way:we have to prove that $ b \\geq c$...If we find a number $ a$ so that $ a \\geq b$ and $ a \\geq c$ it doesnt mean that $ b \\geq c$....[/quote]\n\nI guess you didn't see my post . I explained him the same thing :P[/quote]\r\nYes...I saw your post after my reply :P", "Solution_15": "[quote=\"alex2008\"]Let be $ a,b,c > 0$ such that $ abc \\equal{} 1$ . Show that :\n\n$ \\frac {(a^2 \\plus{} 2b^2)(b^2 \\plus{} 2c^2)(c^2 \\plus{} 2a^2)}{a^2b \\plus{} b^2c \\plus{} c^2a}\\ge 9$[/quote]\r\nAfter expanding and homogenizing:\r\n$ \\sum_{cyc} 4a^4b^2\\plus{}2a^2b^4\\plus{}3a^2b^2c^2 \\minus{} 9a^3b^2c \\ge 0 \\iff$\r\n$ \\left ( \\sum_{cyc} 8a^4b^2\\plus{}a^4c^2\\plus{}a^2b^4\\plus{}8a^2b^2c^2 \\minus{} 18a^3b^2c \\right ) \\plus{} 2\\left ( \\sum_{cyc} a^2b^4\\minus{}a^2b^2c^2 \\right)\\ge 0$ Which is true by Am-GM :)", "Solution_16": "Let $ a \\equal{} \\frac{x}{y}, b \\equal{} \\frac{y}{z}, c \\equal{} \\frac{z}{a}$ and substitute to the inequality, multiply numerator and denominator by $ (xyz)^4$. After expanding, the desired inequality becomes:\r\n\r\n$ 3 \\sum x^4y^4z^4 \\plus{} 2\\sum x^8y^2z^2 \\plus{} 4 \\sum x^6y^6 \\geq 9 \\sum x^6y^3z^3$ (where all summation is symmetric)\r\n\r\nNow, from Schur's:\r\n$ A^3\\plus{}B^3\\plus{}C^3\\plus{}3ABC \\geq \\sum A^2B$, and let $ A \\equal{} x^2y^2, B \\equal{} y^2z^2, C \\equal{} z^2x^2$ we have:\r\n\r\n$ 3\\sum x^6y^6 \\plus{} 3\\sum x^4y^4z^4 \\geq 6\\sum x^6y^4z^2$\r\n\r\nSo now we only need to prove:\r\n$ 2\\sum x^8y^2z^2 \\plus{} \\sum x^6y^6 \\plus{} 6 \\sum x^6y^4z^2 \\geq 9 \\sum x^6y^3z^3$\r\n\r\nBut each term in the inequality above is true by Muirhead / AM-GM.", "Solution_17": "Your solutions are better than nothing , but there's no solution without brute force ?\r\n\r\n[hide=\"P.S.\"]Actually hendrata01's substitution gave me an idea to corelate this inequality to another one which is well-known[/hide]", "Solution_18": "[quote=\"alex2008\"]Your solutions are better than nothing , but there's no solution without brute force ?\n[/quote]\r\n\r\nACtually my \"brute force\" isn't that bad, you just multiply 2 terms x 2 terms x 2 terms = 8 terms, and a lot of them are cyclic, so there's not that much algebra really. The reason the coefficients turned out that way is just because there's a \"2\" coefficient that gives the impression that it's a grungy algebra. It's not really", "Solution_19": "hendrata01 your method works in [url=http://www.mathlinks.ro/Forum/viewtopic.php?t=277327]this[/url] inequality ?", "Solution_20": "hey, so Hendrata still playing with this math problem.. :D :D \r\n\r\n(udah kerja kan ?? Masi sempet aja ya maen2 ke Mathlinks.. :P )", "Solution_21": "[quote=\"alex2008\"]Let be $ a,b,c > 0$ such that $ abc \\equal{} 1$ . Show that :\n\n$ \\frac {(a^2 \\plus{} 2b^2)(b^2 \\plus{} 2c^2)(c^2 \\plus{} 2a^2)}{a^2b \\plus{} b^2c \\plus{} c^2a}\\ge 9$[/quote]\r\nwith $ abc\\equal{}1$ the problem isn't wrong and this is my slution\r\n\r\nThe inequality equivalent\r\n\r\n$ (a^2 \\plus{} 2b^2)(b^2 \\plus{} 2c^2)(c^2 \\plus{} 2a^2)\\ge 9(a^2b \\plus{} b^2c \\plus{} c^2a)$\r\n\r\nsetting $ x\\equal{}\\frac{a}{c}$, $ y\\equal{}\\frac{b}{a}$, $ z\\equal{}\\frac{c}{b}$ $ \\Rightarrow xyz\\equal{}1$\r\n\r\nassume $ x\\equal{}min(x,y,z) \\Rightarrow x\\le 1$\r\n\r\nInequality became \r\n\r\n$ (1\\plus{}2x^2)(1\\plus{}2y^2)(1\\plus{}2z^2)\\ge 9(x\\plus{}y\\plus{}z)$\r\n\r\nsetting $ F(x,y,z)\\equal{}(1\\plus{}2x^2)(1\\plus{}2y^2)(1\\plus{}2z^2) \\minus{} 9(x\\plus{}y\\plus{}z)$\r\n\r\nwe have \r\n\r\n$ F(x,y,z)\\minus{}F(x,\\sqrt{yz},\\sqrt{yz})\\equal{}(\\sqrt{y}\\minus{}\\sqrt{z})^2[2(1\\plus{}2x^2)(\\sqrt{y}\\plus{}\\sqrt{z})\\minus{}y]$\r\n\r\nBy Am-Gm $ \\Rightarrow F(x,y,z)\\ge F(x,\\sqrt{yz},\\sqrt{yz})$\r\n\r\nwe will prove $ F(x,\\sqrt{yz},\\sqrt{yz})\\ge 0$\r\n\r\n$ \\Leftrightarrow F(1/t^2,t,t)\\ge 0$\r\n\r\n$ \\Leftrightarrow (t\\minus{}1)(4t^7\\plus{}4t^6\\plus{}8t^5\\plus{}8t^4\\plus{}17t^3\\minus{}t^2\\minus{}2t\\minus{}2)\\ge 0$ (right!!!)\r\n\r\nDoNe.", "Solution_22": "[quote=\"alex2008\"]Let be $ x,y,z > 0$ such that $ xyz \\equal{} 1$ . Show that :\n\n$ \\frac {(x^2 \\plus{} 2y^2)(y^2 \\plus{} 2z^2)(z^2 \\plus{} 2x^2)}{x^2y \\plus{} y^2z \\plus{} z^2x}\\ge 9$[/quote]\r\nLet $ x \\equal{} \\frac {a}{b}$... We need to prove:\r\n$ (2a^4 \\plus{} b^2c^2)(2b^4 \\plus{} c^2a^2)(2c^4 \\plus{} a^2b^2) \\geq\\ 9a^3b^3c^3(a^3 \\plus{} b^3 \\plus{} c^3)$\r\n<=> $ 4\\sum\\ a^3b^3 \\plus{} 9a^3b^3c^3 \\geq\\ 7a^3b^3c^3(a^3 \\plus{} b^3 \\plus{} c^3)$\r\nUsing Am-Gm and Schur ineq, we have: $ RHS \\geq\\ \\frac {7}{2}a^2b^2c^2 \\sum\\ a^4(b^2 \\plus{} c^2) \\geq\\ LHS$\r\n(obviously trues by Am-Gm)\r\nWe have done :)" } { "Tag": [], "Problem": "Chan's bank account contains $ \\$450$, which is $ 125\\%$ more than \nit contained at this time last year. How many dollars were in\nChan's bank account last year?", "Solution_1": "450=225% of last year\r\n\r\n$ 450\\div225\\times100\\equal{}200$\r\n\r\nanswer : 200", "Solution_2": "There is a typo in the question. The percentage in the question shows $ 125 $% while the correct percentage is $ 225$", "Solution_3": "No, there isn't. 125% [i]more[/i] is the same as 225% [i]of[/i]." } { "Tag": [], "Problem": "How many integers between 300 and 500 have the sum of their\ndigits equal to 16?", "Solution_1": "If the hundreds digit is 3, then the other two digits add to 13. There are 6 ways there. If the hundreds digit is 4, then the other two digits add to 12. There are 7 ways. Added up, 13 ways." } { "Tag": [ "ARML", "search" ], "Problem": "I'm getting ready for GPML state, and I was wondering if anybody knows where I can find old GPML problems. Also, does anyone know where to find ARML problems that are older than 2005", "Solution_1": "If your school has participated in GPML for a while, your math coach probably has copies of old tests. I know that my school's math coach has quite a few... Other than that, I don't think they're on the web anywhere, though if it's practice you want, going through old AMC 12s (available on this site) are about the same in difficulty, maybe a bit harder. Also don't know about old ARML problems (there are probably books you can buy with past problems). If it's just ARML-ish problems, though, and you've already done the ones since '05, there are quite a few mock ARMLs stored on AoPS... try the wiki, search the forums.", "Solution_2": "May I know what GPML is ?", "Solution_3": "There's a neat thing called [url=http://lmgtfy.com/?q=gpml&l=1]Google[/url] :wink:", "Solution_4": "Thanks AIME15!" } { "Tag": [ "FTW", "calculus", "integration", "counting", "distinguishability", "geometry", "geometric transformation" ], "Problem": "Yay, my fav math tourney's coming up! (bcuz its uber long and they give ciphering prizes. also its PROOF. very hardcore :O)\r\n\r\nSo, who's going? I'm going JV, slackers ftw :D", "Solution_1": "Uh, like no one from Northview's going, cuz science bowl, debate, and science olympiad tourneys are on the same day.\r\n\r\nUh, I'm going to be daring and say GT is not my favorite tournament cuz the only time I participated, I got a perfect on the first round, did okay on ciphering, and made over 9000 careless mistakes on proof round. The proof problems are more like \"find the value and justify that it's the answer,\" so they're not very hardcore. But that's just JV. I dunno about varsity. I guess it's just because I'm not 1337. :(", "Solution_2": "Question. I've heard rumors that there are no ciphering awards.\r\n\r\nO RLY? :O\r\n\r\nplz dun say YA RLY. :(", "Solution_3": "Ciphering is conducted towards team scores. I don't know if there's a prize\r\n\r\nDoes anyone want to form an alternate team since I'll probably be the only person going from Norcross?", "Solution_4": "Hey, so when I was doing the 2005 proof paper I noticed that the solutions file was missing on the website. If it exists could someone tell me where I can get it?\r\n\r\nIf it doesn't, maybe we should just go over or outline the solution to each question here :) \r\n(Especially #5, I keep getting 0 for c) and my proof for b) is a little iffy)\r\n\r\n[hide=\"Number 1\"]\n\n$ \\binom{2n}{n}$ is the coefficient of $ x^n$ in $ (x \\plus{} 1)^{2n}$\n\nBut\n\n$ (x \\plus{} 1)^{2n} \\equal{} [(x \\plus{} 1)^n]^2$\n\nand so\n\n$ \\binom{2n}{n} \\equal{} \\sum^{n}_{j \\equal{} 0}{\\binom{n}{j} \\binom{n}{n \\minus{} j}} \\equal{} \\binom{n}{j}^2$\n\n[/hide]", "Solution_5": "#5: This is pretty much \"Have you heard of the Cantor Set, and do you understand some of the math behind it?\" This is a rough outline of the solutions, sorry for not LaTeXing this. \r\n\r\n[hide]a) \nThere are just as many numbers in the Cantor Set as in the interval (0,1) \nProof: 1 to 1 correspondence: \nEvery number in the Cantor Set when written in base 3 only contains 0's and 2's. So for every number in the Cantor Set, write the number in base-3, change all the 2's to 1's, and then erase the base-3 and write base-2 without actually changing the digits. This will yield a distinct number in the interval (0,1). You can do the same thing in reverse on any number in (0,1) to get a number in the Cantor Set. So, there is a 1:1 correspondence, and thus the same number of points in both the Cantor Set and (0,1). \n\nb) \nNo, it is not possible. \nSuppose that it was possible to have a 1:1 correspondence. \nLet the number corresponding to m in the Cantor set be: \nM: 0.d_{m,1}d_{m,2}d_{m,3}...d_{m,n}... in base-3 \nAlso let d'{m,n} = 2-d_{m,n} (In other words, the compliment digit of d_{mn}) \nRemember that since numbers in the Cantor Set don't have 1's for digits in base-3, the compliment is a different digit than the original digit. \n\nConsider the number X = 0.d'_{1,1}d'_{2,2}d'_{3,3}...d'_{n,n}.... \nThe 1st digit after the decimal is d'_{1,1}, which is different from d_{1,1}, the 1st digit after the decimal of the number corresponding to 1. So, X doesn't correspond to 1. \nThe 2nd digit after the decimal is d'_{2,2}, which is different from d_{2,2}, the 2nd digit after the decimal of the number corresponding to 2. So, X doesn't correspond to 2. \nAnd in general, The nth digit after the decimal is d'_{n,n}, which is different from d_{n,n}, the nth digit after the decimal of the number corresponding to n. So, X doesn't correspond to n. \n\nSo, this number X does not correspond to any natural number. Thus, our assumption that there was a 1:1 correspondence is false. \n\nc) After each iteration, 1/3 of the length is removed, so 2/3 of the length remains. \nSo, the length after n iterations is (2/3)^n \nThus, the length of the Cantor Set is lim(n->infinity) (2/3)^n = 0. \nYes, the length is actually 0 despite the fact that some numbers, such as 1/4 and 10/13, are in the Cantor Set. I believe that numbers that are in (0,1) and are not in the Cantor are \"dense\" in the interval (0,1), but I am not sure that is the correct terminology.[/hide]", "Solution_6": "#2: I believe the simplest solution is casework on the number of quarters and dimes, and figure out the combination of nickels and dimes from this. Does anyone know of a better way? \r\n\r\n#3: This solution uses calculus: \r\n[hide]\nIf the original series converged, then its sum would be greater than the sum of the series $ \\displaystyle \\sum_{n = 1}^{\\infty} \\frac {1}{10n - 1} = \\frac {1}{9} + \\frac {1}{19} + \\frac {1}{29} + \\cdots + \\frac {1}{90} + \\frac {1}{99} + \\frac {1}{109} + \\cdots$ (sum of reciprocals of only the integers with 9 as the last digit)\n\nIf this series converged, then its sum would be greater than $ \\frac {1}{9} + \\frac {1}{10}\\displaystyle \\int^\\infty_9 \\frac {1}{x} \\, dx$. \n\nHowever, $ \\int^\\infty_9 \\frac {1}{x} \\, dx$ diverges, and thus, so does both $ \\displaystyle \\sum_{n = 1}^{\\infty} \\frac {1}{10n - 1}$, and the original series in the problem. [/hide]\n\n#4: This is just Fermat's Little Theorem. I don't feel like proving this now, so instead, I will tell you this story: \n[hide]\nIn a research base in Antarctica, $ p$ penguins (where $ p$ is a prime number) were held captive by researchers. One day when the researchers were particularly negligent, all of the penguins escaped. To celebrate their escape, these penguins decided to host a dinner party (as proper penguins would do.) \n\nThey all waddled over to the nearest tuxedo rental store, and found out that the store had $ a$ different colors of matching tuxedo and bow-tie packages. Since they were sick of being forced to conform to each other in the research base, they decided that each penguin could buy which ever color tuxedo and bow-tie that he or she wanted, as long as they wouldn't all be wearing the same color tuxedo. \n\nAfter this, they went to a restaurant, and sat down randomly at a perfectly round table for $ p$. During the dinner conversation, one of the penguins posed a problem: \"If the only distinguishable feature of the penguins was what color tuxedo they wore, and there was no way to distinguish rotations of the table, how many different dinner table arrangements could be made?\" \n\n\"Well,\" Waddles said, \"Since there are $ a$ tuxedo outfits and $ p$ of us, there are $ a^p$ different combinations of tuxedos that we could have bought.\" \n\nTux immediately interjected, \"Don't forget that we agreed that we couldn't all wear the same color, so that leaves $ a^p - a$ possible combinations of tuxedos that we could have bought. But we still need to account for rotations.\"\n\nWaddles replied, \"That is easy. Since there are a prime number of us, each possible arrangement can either be rotated to yield $ p$ different arrangements, or only be rotated to yield the same arrangement over and over. But that can only happen if we all wore the same color tuxedo. So, there are $ \\frac {a^p - a}{p}$ different table arrangements.\"\n\nAll of the penguins agreed that this was an interesting problem. After a few more hours of intellectual conversation, the penguins thanked their gracious host, and left the dinner party. \n[/hide]\nAnyway, now that I have finished the story, here is the rest of the proof. \n[hide]\nClearly, $ \\frac {a^p - a}{p}$, the number of different table arrangements, must be an integer. \n\nSo, $ a^p - a \\equiv 0 \\pmod{p} \\Rightarrow a^p \\equiv a \\pmod{p}$ \n\nSince $ p \\nmid a$, $ a \\not\\equiv 0 \\pmod{p}$. Thus, $ a^{p - 1} \\equiv 1 \\pmod{p}$ as desired. \n\nThe story is similar to the beads and necklace argument, however, I felt that penguins were more interesting. \n[/hide]", "Solution_7": "Your penguin proof of FLT is interesting! Did you see the proof of FLT that AoPSer t0rajiru came up with? Here's the link to that entry in his blog:\r\n\r\n[url]http://www.artofproblemsolving.com/Forum/weblog_entry.php?t=229442[/url]", "Solution_8": "Thanks, Santosh!\r\n\r\nYeah, for #2, I don't see any way you could've used generating functions without making a mess.\r\n\r\nFor #3, couldn't you just compare it to the harmonic sequence and prove that the harmonic sequence diverges by creating infinite groups of n>1/2? (yeah, that might take a little longer, though)\r\n\r\nI liked your counting solution for #4 :D \r\nI just ended up using $ (1\\plus{}(a\\minus{}1))^p \\equiv 1^p \\plus{} (k\\minus{}1)^p \\text{mod p}$", "Solution_9": "You don't need to prove that the harmonic series diverges; it's well known.\r\n\r\nYes, I think combinatorics and induction are two of the three well known elementary proofs of Fermat's little theorem. Here is the third one. We will let $ a$ be any integer not divisible by $ p$.\r\n\r\n[hide]Notice that the sets\n\\[ \\{ 1, 2, \\dotsc, p-1 \\} \\]\nand\n\\[ \\{ a, 2a, \\dotsc, (p-1) a \\} \\]\nare congruent mod $ p$, since $ a$ has a multiplicative inverse. Then their products are congruent mod $ p$; that is,\n\\[ (p-1)! = \\prod_{k=1}^{p-1} ka = a^{p-1} \\cdot (p-1)! . \\]\nSince $ (p-1)!$ has a multiplicative inverse, it follows that $ a^{p-1} \\equiv 1 \\pmod{p}$.[/hide]\n\nHere is a group-theoretic approach. If you read the background through the proof of Lagrange's theorem, it seems like kind of a lot, but know that it is a fundamental part of modern algebra, and if you ever take undergraduate algebra, you will learn all of it. Besides, it is pretty, and what comes afterward is very natural; by now it is the easiest of these four proofs for me to remember.\n\n[hide]\nFirst we need to prove Lagrange's Theorem (well, sort of a special case), for which we need some background. All group composition will be written multiplicatively, and the identity will be denoted $ e$.\n\n[hide=\"Background\"]\nLet $ G$ be a group. We call the the number of elements of $ G$ (the cardinality of the set of elements of $ G$) the [i]order[/i] of $ G$, and we denote it $ \\text{ord}(G)$, or sometimes $ \\lvert G \\rvert$.\n\nLet $ H$ be a subgroup of $ G$. We say that two elements $ x,y$ of $ G$ are [i]left equivalent[/i] mod $ H$ if $ x^{-1}y \\in H$. (For those interested, right equivalence is defined by the relation $ xy^{-1} \\in H$.)\n\n[b]Proposition.[/b] Left equivalence mod $ H$ is an equivalence relation on $ G$.\n\n[i]Proof.[/i] We need to establish that the relation is reflexive, symmetric, and transitive. The relation is reflexive because $ x^{-1}x$ is the identity, which is in $ H$ by definition of \"subgroup\". The relation is symmetric because if $ x^{-1}y \\in H$, then so is $ (x^{-1}y)^{-1} = y^{-1}x$. Finally, the relation is transitive because if $ x^{-1}y \\in H$ and $ y^{-1}z \\in H$, then\n\\[ x^{-1}z = (x^{-1}y)(y^{-1}z) \\in H, \\]\nas $ H$ is closed under composition. Thus left equivalence mod $ H$ is an equivalence relation on $ G$. $ \\blacksquare$\n\nA [i]left coset[/i] mod $ H$ is a set of the form\n\\[ \\{ x \\in G \\vert a^{-1}x \\in H \\} \\]\nfor some $ a \\in G$. In other words, it is the set of all $ x\\in G$ that are left equivalent (mod $ H$) to some fixed $ a \\in G$. More succinctly, a coset is an equivalence class under the relation of left equivalence mod $ H$.\n\nThe cardinality (number of elements) of the set of left cosets mod $ H$ is denoted $ (G: H)$. (It is the same as the cardinality of the set of right cosets, but that's irrelevant here, since we're only going to talk about commutative groups anyway.)\n\nOkay, that's all the background we need![/hide]\n\n[b]Theorem (Lagrange).[/b] Let $ G$ be a group, and let $ H$ be a subset of $ G$. Then\n\\[ \\lvert G \\rvert = (G : H) \\cdot \\lvert H \\rvert . \\]\n\n[hide=\"Proof\"]Since each element of $ G$ is an element of exactly one left coset mod $ H$, it is enough to show that every left coset mod $ H$ has the same size as $ H$.\n\nTo this end, let $ S$ be an arbitrary left coset mod $ H$, and let $ a$ be an element of $ S$.\n\nConsider the mapping $ A: x \\mapsto ax$ of $ H$ into $ S$. This is indeed a mapping into $ S$ because for all $ x\\in H$, $ a^{-1}(ax) = x \\in H$. It is injective, because if $ A(x) = A(y)$, then $ ax = ay$, so $ x=y$.\n\nFurthermore, if $ b$ is an arbitrary element of $ S$, then $ a^{-1}b \\in H$, and $ f(a^{-1}b) = a(a^{-1}b) = b$, so $ f$ is surjective. Hence $ f$ is a bijection between $ H$ and $ S$. Therefore all the cosets of $ S$ have the same size as $ H$, and we are done. $ \\blacksquare$[/hide]\n\nNow, let $ G$ be the group of nonzero integers modulo $ p$, under multiplication. Since $ G$ is finite, we must have $ a^{r} = a^s$ for some integers $ r>s$. Then for some positive integer $ N$ (equal to $ r-s$), $ a^N = e$. Then let $ n$ be the least integer such that $ a^n = e$.\n\nThe powers of $ a$ evidently form a subgroup of $ G$, and the order of this subgroup is evidently $ n$. Since the order of $ G$ is $ p-1$, it follows from Lagrange's theorem that $ n$ divides $ p-1$. It follows directly that $ a^{p-1} = e^{(p-1)/n} = e$, or, in the notation of modular arithmetic,\n\\[ a^{p-1} \\equiv 1 \\pmod{p} . \\qquad \\blacksquare \\]\n[/hide]\r\n\r\nNow, are you ready for the slickest proof of Wilson's theorem? Since the integers mod $ p$ form a commutative field, and all nonzero integers (mod $ p$) are zeros of the polynomial $ x^{p-1} - 1$, it follows that\r\n\\[ (x - 1)(x - 2) \\dots p \\bigl( x - (p-1) \\bigr) \\equiv x^{p-1} - 1 \\pmod{p}. \\]\r\nSet $ x$ equal to zero. The left-hand side of the equation evaluates to $ (-1)^{(p-1)}(p-1)! = (p-1)!$, and the right-hand side evaluates to $ -1$. So\r\n\\[ (p-1)! \\equiv -1 \\pmod{p} . \\]", "Solution_10": "Just as a general question, what can we assume that the graders will accept as being 'well known'?\r\n\r\nSpecifically, I was a little worried by my statement on a (practice) proof that:\r\n\r\nIt is well known that the Fermat point, which makes three 120 degree angles at the center of a triangle, minimizes the sum of the distances to the three sides.", "Solution_11": "In general, it is acceptable to cite any widely known result as \"well-known\" if it does not trivialize the problem at hand.\r\n\r\nThis is particularly true if the \"well-known\" results is indeed well known in an area of modern mathematics. The divergence of the harmonic series is a fairly basic fact in the field of analysis, a vibrant and mainstream field of research these days. The Fermat point is a fairly obscure corner of Euclidean geometry.\r\n\r\nIn any case, the Fermat point does [i]not[/i] minimize the sum of the distances from a point to the three sides of a triangle. That would be the vertex opposite the longest side.", "Solution_12": "Oops :blush: \r\nI meant the distances to the three vertices...\r\n\r\nBut thanks for clarifying that for me, I think I get the general idea of what I can cite now.", "Solution_13": "[hide=\"non-calculus way for #3\"]Note the series includes $ \\sum_{k \\equal{} 1}^{\\infty}\\sum_{n \\equal{} 9\\cdot 10^k}^{10^{k \\plus{} 1} \\minus{} 1}\\frac {1}{n} > \\sum_{k \\equal{} 1}^{\\infty}\\sum_{n \\equal{} 9\\cdot 10^k}^{10^{k \\plus{} 1} \\minus{} 1}\\frac {1}{10^{n \\plus{} 1}} \\equal{} \\sum_{k \\equal{} 1}^{\\infty}\\frac {10^{n}}{10^{n \\plus{} 1}} \\equal{} \\sum_{k \\equal{} 1}^{\\infty}\\frac {1}{10} \\equal{} \\infty$. Therefore, it diverges.[/hide]\r\n\r\nFor #5, can't you just say that since Cantor middle-third set has the same cardinality as the real numbers, it cannot form bijection with natural numbers? (its proof is essentially the same as yours)", "Solution_14": "So I was looking at the site and noted that Wolfram Research is a sponsor.", "Solution_15": "Yeah, I got an email from someone in charge of the sponsorships group at Wolfram Research. She wanted to know if I was attending, and gave me a bit of info on what they were trying to do.", "Solution_16": "Congrats Yubo and Oliver for winnng Georgia Tech.\r\n\r\nIt is late now.\r\n\r\nWill discuss about the problems tomorrow.", "Solution_17": "* irrelevant question about befuddler's initials *", "Solution_18": "Disclaimer : Problems not in correct order\r\n\r\n1. Prove that $ {2^{2m} \\plus{} 1}/5$ is composite\r\n\r\nif m is odd and > 3\r\n\r\n[hide]\nI used the identity of factorising $ a^4 \\plus{} 4b^4$ to do this problem\n\nPlease post any other methods\n[/hide]\r\n\r\nI am very slow in typing and latexing\r\n\r\nwill post the questions one by one\r\n\r\nBtw i used Cauchy's lemma for the inequality problem which the corrector probably did not understand or probably there was a mistake.", "Solution_19": "#5: Prove\r\n\\[ \\sum_{i \\equal{} 0}^{1111} \\frac {1}{\\sqrt {9i \\plus{} 1} \\plus{} \\sqrt {9i \\plus{} 4}} > 10\r\n\\]\r\n#1: ABC is a triangle. E is on AB such that AE=(1/2)EB. Given AB=6,AC=4,BC=5, find CE. (Evidently, from what I heard, lots of people have difficulty with computation on this one ;))\r\n\r\n#3: Let $ x_i,y_i,z_i$ be sequences with $ x_1,y_1,z_1 > 0$, $ x_i \\equal{} y_{i \\minus{} 1} \\plus{} 1/z_{i \\minus{} 1}$, $ y_i \\equal{} z_{i \\minus{} 1} \\plus{} 1/x_{i \\minus{} 1}$, $ z_i \\equal{} x_{i \\minus{} 1} \\plus{} 1/y_{i \\minus{} 1}$. \r\n(a) Prove all three increase without bound.\r\n(b) Show at least one of $ x_{200},y_{200},z_{200}$ is greater than 20.", "Solution_20": "[quote=\"solafidefarms\"]#5: Prove\n\\[ \\sum_{i \\equal{} 0}^{1111} \\frac {1}{\\sqrt {9i \\plus{} 1} \\plus{} \\sqrt {9i \\plus{} 4}} > 10\n\\]\n[/quote]\r\n[hide]Here's my solution(I'm curious why #5 in this year is so easy!):\nApplying the Cauchy-Schwartz's inequality we have: \n$ \\sum_{i \\equal{} 0}^{1111} \\frac {1}{\\sqrt {9i \\plus{} 1} \\plus{} \\sqrt {9i \\plus{} 4}} \\geq(11111)^2/sum_{i \\equal{} 0}^{1111}{\\sqrt {9i \\plus{} 1} \\plus{} \\sqrt {9i \\plus{} 4}}$(the equality actually doesn't occur in this inequality!)\nNow applying Cauchy-Schwartz's inequality we get:\n$ sum_{i \\equal{} 0}^{11111}{\\sqrt {9i \\plus{} 1} \\plus{} \\sqrt {9i \\plus{} 4}}$ = < $ sqrt((18i \\plus{} 55555)(11112))$\nNow we just need to prove: $ 11111^2$ > $ 10\\sqrt ((18.11111.11112/2 \\plus{} 55555)(11112))$ \nThis is equivalent to: $ 11111.sqrt{11111} > 10.sqrt((9.11112 \\plus{} 5).11112)$. The last step is just a simple inequality when everything is just a number![/hide]", "Solution_21": "[hide=\"This is one of many ways to do #5\"]\n$ \\sum_{i = 0}^{1111} \\frac {1}{\\sqrt {9i + 1} + \\sqrt {9i + 4}} = \\frac {1}{3} \\sum_{i = 0}^{1111} (\\sqrt {9i + 4} - \\sqrt {9i + 1})$\nby multiplying each term by its conjugate.\n\nThus it is equivalent to prove that\n\n$ \\sum_{i = 0}^{1111} (\\sqrt {9i + 4} - \\sqrt {9i + 1}) > 30$\n\nLet $ S_1 = \\sum_{i = 0}^{1111} (\\sqrt {9i + 4} - \\sqrt {9i + 1})$\nLet ${ S_2 = \\sum_{i = 0}^{1110} (\\sqrt {9i + 10} - \\sqrt {9i + 4})}$\n\nThen $ S_1 + S_2 = \\sqrt {10003} - \\sqrt {1} > 90 = 3 \\times 30$\n\nSo all we need to do is prove that\n\n$ 2S_1 \\geq S_2$\n\n(we will ignore the 1111th index of $ S_1$, since it only adds to the LHS)\n\nWe need to show that\n\n$ 2(\\sqrt {9k + 4} - \\sqrt {9k + 1}) \\geq (\\sqrt {9k + 10} - \\sqrt {9k + 4})$ for $ 1 \\leq k \\leq 1110$,\n\nor\n\n$ 3\\sqrt {9k + 4} \\geq 2\\sqrt {9k + 1} + \\sqrt {9k + 10}$\n\nSince both sides are positive, we can take the square and do some arithmetic to achieve\n\n$ \\frac {18k + 11}{2} \\geq \\sqrt {(9k + 1)(9k + 10)}$\n\nwhich follows directly from AM-GM\n\n[/hide]\n\nEDIT: Did 3a) as well\n\n[hide=\"3a)\"]\n\nWe will prove the statement indirectly; suppose that indeed one of the sequences is bounded.\n\nWLOG let it be $ x_n$.\n\nThen, since $ x_n = y_n + \\frac{1}{z_n}$, $ y_n$ must be bounded also.\nSimilarly, $ z_n$ must also be bounded.\n\nSince all three sequences are bounded, their product, $ x_n y_n z_n$, must also be bounded.\n\nHowever,\n\n$ x_{n+1} y_{n+1} z_{n+1}$\n\n$ = x_n y_n z_n + \\frac{1}{x_n y_n z_n} + ( x+ \\frac{1}{x})+( y+ \\frac{1}{y} )+( z+ \\frac{1}{z} )$\n\n$ > x_n y_n z_n + 6$\n\nfor all $ n$, implying that $ x_n y_n z_n$ is not bounded.\n\nContradiction.\n\n[/hide]", "Solution_22": "[hide=\"my method for #5\"]Let $ A$ be the l.h.s. Let $ B \\equal{} \\sum_{n \\equal{} 0}^{3333}\\frac {1}{\\sqrt {3n \\plus{} 1} \\plus{} \\sqrt {3n \\plus{} 4}}$.\nLemma 1: $ B > 33$.\nProof: Multiplying the denominator by its conjugate,\n$ B \\equal{} \\sum_{n \\equal{} 0}^{3333}\\frac {\\sqrt {3n \\plus{} 4} \\minus{} \\sqrt {3n \\plus{} 1}}{3} \\equal{} \\frac {\\sqrt {10003} \\minus{} 1}{3} > \\frac {\\sqrt {10000} \\minus{} 1}{3} \\equal{} 33$.\nLemma 2: $ 3A > B$\nProof: Note $ \\frac {1}{\\sqrt {3n \\plus{} 1} \\plus{} \\sqrt {3n \\plus{} 4}} > \\frac {1}{\\sqrt {3n \\plus{} 4} \\plus{} \\sqrt {3n \\plus{} 7}} > \\frac {1}{\\sqrt {3n \\plus{} 7} \\plus{} \\sqrt {3n \\plus{} 10}}$, which implies $ \\frac {3}{\\sqrt {3n \\plus{} 1} \\plus{} \\sqrt {3n \\plus{} 4}} > \\frac {1}{\\sqrt {3n \\plus{} 1} \\plus{} \\sqrt {3n \\plus{} 4}} \\plus{} \\frac {1}{\\sqrt {3n \\plus{} 4} \\plus{} \\sqrt {3n \\plus{} 7}} \\plus{} \\frac {1}{\\sqrt {3n \\plus{} 7} \\plus{} \\sqrt {3n \\plus{} 10}}$. Therefore,\n$ 3A \\equal{} \\sum_{n \\equal{} 0}^{1111}\\frac {3}{\\sqrt {9n \\plus{} 1} \\plus{} \\sqrt {9n \\plus{} 4}} > \\sum_{n \\equal{} 0}^{1111}\\frac {1}{\\sqrt {9n \\plus{} 1} \\plus{} \\sqrt {9n \\plus{} 4}} \\plus{} \\frac {1}{\\sqrt {9n \\plus{} 4} \\plus{} \\sqrt {9n \\plus{} 7}} \\plus{} \\frac {1}{\\sqrt {9n \\plus{} 7} \\plus{} \\sqrt {9n \\plus{} 10}} > \\sum_{n \\equal{} 0}^{9999}\\frac {1}{\\sqrt {3n \\plus{} 1} \\plus{} \\sqrt {3n \\plus{} 4}} \\equal{} B$. \nHence, $ A > B/3 > 11 > 10$.\n\n[/hide]\n\n[hide=\"my method for 3a\"]Let $ s_n \\equal{} x_n \\plus{} y_n \\plus{} z_n$. Suppose for some positive integer $ N$, $ x_n < N$ for all $ n$. Since $ x_n \\equal{} y_n \\plus{} \\frac {1}{z_n}$ and $ y_n \\equal{} z_n \\plus{} \\frac {1}{x_n}$, we have $ y_n, z_n < N$ for all positive $ n$. Thus, $ s_n < 3N$ is bounded.\nNote $ s_{n \\plus{} 1} \\equal{} x_n \\plus{} y_n \\plus{} z_n \\plus{} \\frac {1}{x_n} \\plus{} \\frac {1}{y_n} \\plus{} \\frac {1}{z_n}$, or $ s_{n \\plus{} 1} \\minus{} s_{n} \\equal{} \\frac {1}{x_n} \\plus{} \\frac {1}{y_n} \\plus{} \\frac {1}{z_n} > \\frac {3}{N}$. Thus, $ s_{n}$ is unbounded. Contradiction.\n[/hide]", "Solution_23": "[quote=\"solafidefarms\"]#5: Prove\n\n#1: ABC is a triangle. E is on AB such that AE=(1/2)EB. Given AB=6,AC=4,BC=5, find CE. (Evidently, from what I heard, lots of people have difficulty with computation on this one ;))\n.[/quote]\r\n[hide]Really? Anyway, here's my solution:\n$ cos B\\equal{}(36\\plus{}25\\minus{}16)/60\\equal{}3/4$\nNow we have: $ cosB\\equal{}EB^2\\plus{}25\\minus{}EC^2/(40)$ and $ EB\\equal{}4$, we get: $ 11\\equal{}EC^2$, which means $ EC\\equal{}sqrt{11}$[/hide]", "Solution_24": "[quote=\"befuddlers\"]\n\nBtw i used Cauchy's lemma for the inequality problem which the corrector probably did not understand or probably there was a mistake.[/quote]\r\n\r\n@ghjk i think cauchy's lemma is the same as cauchy scwartz\r\n\r\nbasically my method was the same.\r\n\r\nCauchy's lemma states that $ x^2/a \\plus{} y^2/b ..................................$\r\n>or equal to $ (x\\plus{}y\\plus{}........................)^2/(a\\plus{}b\\plus{}............................)$\r\nI was pretty shocked i did not place after i got this problem,the power of 2 problem and the cevian problem\r\n\r\nAh well i guess all those who placed did atleast 4 problems", "Solution_25": "[hide=\"Problem 3b\"]\nConsider $ a_n^2 \\equal{} (x_n \\plus{} y_n \\plus{} z_n)^2$\n$ a_2^2 \\equal{} (x_1 \\plus{} 1/x_1 \\plus{}.........\\plus{} z_1 \\plus{} 1/z_1)^2 \\geq 36 \\equal{} 2.18$\nBy induction we can prove $ a_{n\\plus{}1}^2 \\geq a_n^2 \\plus{} 18$\n$ a_n^2 > 18n$ for $ n>2$\nBy the way this is not my own solution. I found this in Problem Solving Strategies Chapter 9 Sequences Exercise problem 24. If any of you doesn't have this book and wants a more detailed solution, please tell me. I will post it here. \nAnd i am really surprised that the proof based exams are borrowing questions from such well known books. Even KSU proof exam had a question from the same book. This simply gives an unfair advantage to a person who has already seen this problem. [/hide]", "Solution_26": "For Problem one on the multiple choice, a direct application of Stewart's Theorem yields the answer. :jump: :icecream: :heli: \r\n\r\nYes I know that Stewart's Theorem results from a double application of the law of Cosines. But why not do one formula?", "Solution_27": "[quote=\"befuddlers\"][hide=\"Problem 3b\"]\nAnd i am really surprised that the proof based exams are borrowing questions from such well known books. Even KSU proof exam had a question from the same book. This simply gives an unfair advantage to a person who has already seen this problem. [/hide][/quote]\n\n[hide]You know what? Making a new problem that is good for such those contests are really hard. Most of the problems now are always derived from some \"original problems\" with some new insightful ideas added to it. Drawing a problem randomly from a book or any old contests(even from IMO) is the common thing. It tests how hard you work, and if you already saw that problem, will you actually remember how to do it, or you're stuck with it again?? Basically, I don't see any advantage for drawing such problems from the well-known book. Remember, there are thousands of books, and millions of problems are covered in those book! Just ask yourself why you didn't work hard enough to \"see\" that problem beforehand. Even I, myself, saw the problem #5 on BAMO contest this year before, and remember exactly it's on the APMO contest, but still miss it because I haven't done that problem before(just glance at it). \n\nAnyway, here's my solution for #3:\n\na)Since the equations are cyclic, W.L.O.G, assume that x_i=max(x_i,y_i,z_i) with every i. Then we have:\nx_i>x_(i-1)(since $ x_i>z_i\\equal{}x_(i\\minus{}1)\\plus{}1/y_(i\\minus{}1)>x_(i\\minus{}1)$). Therefore, we have:\ny_(i-1)+1/z_(i-1)>y_(i-2)+1/z_(i-2) (1)\nNow since z_(i-1)>x_(i-2) and x_(i-2)>z_(i-2), we have: z_(i-1)>z_(i-2). Therefore, from (1), we easily see that y_(i-1)>y_(i-2)\nTherefore, x_i>x_(i-1)>x_(i-2)>....>x_0>0 and similar to other two sequence $ y_i$ and $ z_i$.\nNow I just need to prove that x_i is unbounded.\nLet x_0,y_0 and z_0 approaches to 0, I have: x_1 approaches to infinity, y_1 approaches to infinity and z_1 approaches to infinity. Since x_i>x_1,y_i>y_1 and z_i>z_1, we easily see that they are all unbounded!\n\nb) We see that $ x_200\\plus{}y_200\\plus{}z_200\\equal{}x_199\\plus{}y_199\\plus{}z_199\\plus{}(1/x_199\\plus{}1/y_199\\plus{}1/z_199)$\n $ x_199\\plus{}y_199\\plus{}z_199\\equal{}x_198\\plus{}y_198\\plus{}z_198\\plus{}1/x_198\\plus{}1/y_198\\plus{}1/z_198)$\nDoing exactly the same for x_i+y_i+z_i(i runs down from 199 to 1) and adding all equations together we get:\n$ x_200\\plus{}y_200\\plus{}z_200\\equal{}(x_0\\plus{}y_0\\plus{}z_0)\\plus{}(1/x_199\\plus{}1/y_199\\plus{}1/z_199)\\plus{}...\\plus{}(1/x_0\\plus{}1/y_0\\plus{}1/z_0)>\n1999^2.9/(x_1\\plus{}x_1\\plus{}x_2\\plus{}...\\plus{}x_199\\plus{}y_1\\plus{}...\\plus{}y_199\\plus{}z_1\\plus{}...\\plus{}z_199)\\plus{}6$.\nSince $ x_1\\plus{}x_1\\plus{}...\\plus{}x_199<199x_200,y_1\\plus{}...\\plus{}y_199<199.y_200 and z_1\\plus{}...\\plus{}z_199<199z_200$(because of (a)), we get: $ x_200\\plus{}y_200\\plus{}z_200>1999.9/(x_200\\plus{}y_200\\plus{}z_200)\\plus{}6$, which means $ (x_200\\plus{}y_200\\plus{}z_200)^2\\minus{}6(x_200\\plus{}y_200\\plus{}z_200)\\minus{}1999.9>0$, which means $ 3x_200>x_200\\plus{}y_200\\plus{}z_200>137.146$. Dividing both sides by 3, we get that x_200>20. \nHmm, this inequality is quite strict! Good problem though! I'm not really sure about my solution to the unbounded part since it seems to trivialize part b) if my proof is true. :huh: [/hide]", "Solution_28": "[quote=\"befuddlers\"][quote=\"befuddlers\"]\n\nBtw i used Cauchy's lemma for the inequality problem which the corrector probably did not understand or probably there was a mistake.[/quote]\n\n@ghjk i think cauchy's lemma is the same as cauchy scwartz\n\nbasically my method was the same.\n\nCauchy's lemma states that $ x^2/a \\plus{} y^2/b ..................................$\n>or equal to $ (x \\plus{} y \\plus{} ........................)^2/(a \\plus{} b \\plus{} ............................)$\nI was pretty shocked i did not place after i got this problem,the power of 2 problem and the cevian problem\n\nAh well i guess all those who placed did atleast 4 problems[/quote]\r\n\r\nUh, the highest I know was 4.5 problems (or perhaps more, Oliver?).", "Solution_29": "[quote=\"timwu\"]\n\n[hide=\"my method for 3a\"]Let $ s_n \\equal{} x_n \\plus{} y_n \\plus{} z_n$. Suppose for some positive integer $ N$, $ x_n < N$ for all $ n$. Since $ x_n \\equal{} y_n \\plus{} \\frac {1}{z_n}$ and $ y_n \\equal{} z_n \\plus{} \\frac {1}{x_n}$, we have $ y_n, z_n < N$ for all positive $ n$. Thus, $ s_n < 3N$ is bounded.\nNote $ s_{n \\plus{} 1} \\equal{} x_n \\plus{} y_n \\plus{} z_n \\plus{} \\frac {1}{x_n} \\plus{} \\frac {1}{y_n} \\plus{} \\frac {1}{z_n}$, or $ s_{n \\plus{} 1} \\minus{} s_{n} \\equal{} \\frac {1}{x_n} \\plus{} \\frac {1}{y_n} \\plus{} \\frac {1}{z_n} > \\frac {3}{N}$. Thus, $ s_{n}$ is unbounded. Contradiction.\n[/hide][/quote]\r\nWait! How can $ x_n\\equal{}y_n\\plus{}1/z_n??$ Is it your typo mistake??", "Solution_30": "[quote=\"timwu\"]Uh, the highest I know was 4.5 problems (or perhaps more, Oliver?).[/quote]\r\n\r\nMy score is at most 4.6; I did not complete 2b)\r\n\r\n@ befuddlers:\r\n\r\nI think that someone who received full credit for 4 problems would have placed.\r\nIf you are confident that there are no holes in your proof, then perhaps there was a scoring error.\r\n\r\n@ghjk:\r\n\r\nI'm pretty sure he meant $ x_{n\\plus{}1}$.", "Solution_31": "[quote=\"azn6021023\"]For Problem one on the multiple choice, a direct application of Stewart's Theorem yields the answer. :jump: :icecream: :heli: \n\nYes I know that Stewart's Theorem results from a double application of the law of Cosines. But why not do one formula?[/quote]\r\n\r\nYep even #1 on the proof round was a direct application of Stewart's Theorem\r\n\r\nBut i was so scared i would lose marks so i brute forced the entire darn thing using cosine rule twice.", "Solution_32": "Can anybody mind checking my solution above for problem 3?? I think it has a serious flaw on part a), but I don't want to confirm myself that I did it wrong, which means I have to go and fix it :oops: . And 1 more questions anybody?? I think we have 5 problems but I just see 4 totally. Hmm... This year must be an easy year!", "Solution_33": "@ghjk:\r\n\r\n2 things...\r\n\r\n1. How can you assume that $ x_i \\equal{} \\text{max}\\{x_i,y_i,z_i\\}$ for all $ i$?\r\nand\r\n2. I think the question meant unbounded for any $ x_0, y_0, z_0$, so you cannot let them go to 0?", "Solution_34": "[quote=\"iniquitus\"]@ghjk:\n\n2 things...\n\n1. How can you assume that $ x_i \\equal{} \\text{max}\\{x_i,y_i,z_i\\}$ for all $ i$?[quote]\n\nConsider that these equations are cyclic, we just think of it in terms of a system of equations, where we have 3 variables and 3 equations. On the other hand, it's also true if we assume $ x_1 \\equal{} max(x_1,y_1,z_1)$, we still can prove that $ x_2 > x_1$, $ y_2 > y_1$ and $ z_2 > z_1$. So continue to do it again and again, we get the result I showed above.\n[quote]and\n\n2. I think the question meant unbounded for any $ x_0, y_0, z_0$, so you cannot let them go to 0?[/quote][/quote][/quote]\r\nThis one, I actually think my proof has a flaw since if it does, then the lower bound of x_200 in part b) will be infinity. I'll find another proof!(surely the contradiction method works for this one).\r\n\r\nBy the way, can you post the 2nd problem on this round?", "Solution_35": "Nope the max among $ x_n$,$ y_n$ and $ z_n$ change\r\n\r\nTake an example and u will get it.", "Solution_36": "#1: ABC is a triangle. E is on AB such that AE=(1/2)EB. Given AB=6,AC=4,BC=5, find CE. \r\n\r\n#2: Prove that $ \\frac{2^{2m}\\plus{}1}{5}$ is a composite integer if m is an odd integer greater than 3.\r\n\r\n#3: Let $ x_i,y_i,z_i$ be sequences with $ x_1,y_1,z_1 > 0$, $ x_i \\equal{} y_{i \\minus{} 1} \\plus{} 1/z_{i \\minus{} 1}$, $ y_i \\equal{} z_{i \\minus{} 1} \\plus{} 1/x_{i \\minus{} 1}$, $ z_i \\equal{} x_{i \\minus{} 1} \\plus{} 1/y_{i \\minus{} 1}$. \r\n(a) Prove all three increase without bound.\r\n(b) Show at least one of $ x_{200},y_{200},z_{200}$ is greater than 20.\r\n\r\n#4: Let S={1, 2, 3, 4, ..., 17}. How many subsets of S are there such that each subset has no two consecutive integers?\r\n\r\n#5: Prove\r\n\\[ \\sum_{i \\equal{} 0}^{1111} \\frac {1}{\\sqrt {9i \\plus{} 1} \\plus{} \\sqrt {9i \\plus{} 4}} > 10\r\n\\]\r\n\r\nIn some order. Thanks to solafidefarms for providing 1, 3, and 5.", "Solution_37": "Yeah, #4 was like pretty easy for me. (Except: be sure to count the empty subset!)", "Solution_38": "[quote=\"befuddlers\"]Nope the max among $ x_n$,$ y_n$ and $ z_n$ change\n\nTake an example and u will get it.[/quote]\r\nHmm... We don't need to care about the max among those 3 variables! We care about each sequence, which means x_2>x_1,y_2>y_1 and z_2>z_1. In these 3 \"new variables\", will there exists such a maximum variable? Yes it will. And in what order? Doesn't matter at all! As long as it exists a maximum variables among x_i,y_i and z_i(if you don't believe me, then there is only 1 case that can happen:x_i=y_i=z_i, right??), we can always prove that each sequence is increasing. Simple argument, isn't it??", "Solution_39": "[quote=\"ghjk\"]\na)\n\nSince the equations are cyclic, W.L.O.G, assume that $ x_i\\equal{}\\text{max}(x_i,y_i,z_i)$ with every $ i$. \n\nThen we have:\n\n$ x_i>x_{i\\minus{}1}$ (since $ x_i > z_i \\equal{} x_{i\\minus{}1}\\plus{}\\frac{1}{y_{i\\minus{}1}}>x_{i\\minus{}1}$). \n\nTherefore, we have:\n\n$ y_{i\\minus{}1}\\plus{}\\frac{1}{z_{i\\minus{}1}}>y_{i\\minus{}2}\\plus{}\\frac{1}{z_{i\\minus{}2}}$ (1)\n\nNow since $ z_{i\\minus{}1}>x_{i\\minus{}2}$ and $ x_{i\\minus{}2}>z_{i\\minus{}2}$, we have: $ z_{i\\minus{}1}>z_{i\\minus{}2}$.\n\nTherefore, from (1), we easily see that $ y_{i\\minus{}1}>y_{i\\minus{}2}$\n\nTherefore, $ x_i>x_{i\\minus{}1}>x_{i\\minus{}2}>....>x_0>0$ and similar to other two sequence and now I just need to prove that x_i is unbounded.\n\nLet $ x_0,y_0$ and $ z_0$ approaches to $ 0$, I have: $ x_1$ approaches to infinity, $ y_1$ approaches to infinity and $ z_1$ approaches to infinity. \n\nSince $ x_i>x_1$,$ y_i>y_1$ and $ z_i>z_1$, we easily see that they are all unbounded![/quote]\r\n\r\nOK so I cleaned up your Latex a bit to make it clearer...\r\n\r\nTo show your error (That you cannot assume WLOG that $ x_i$ is the maximum):\r\nYour first argument, $ x_i > x_{i\\minus{}1}$, is unfortunately false.\r\n\r\nLet $ x_1 \\equal{} 10, y_1 \\equal{} 1, z_1 \\equal{} 1$\r\nThen $ x_2 \\equal{} 1\\plus{}1 \\equal{} 2 < x_1$", "Solution_40": "[quote=\"iniquitus\"]\n\nOK so I cleaned up your Latex a bit to make it clearer...\n\nTo show your error (That you cannot assume WLOG that $ x_i$ is the maximum):\nYour first argument, $ x_i > x_{i \\minus{} 1}$, is unfortunately false.\nLet $ x_1 \\equal{} 10, y_1 \\equal{} 1, z_1 \\equal{} 1$\nThen $ x_2 \\equal{} 1 \\plus{} 1 \\equal{} 2 < x_1$[/quote]\r\n\r\nInteresting.... You're right :oops: . Hmm... But does your example also show the contradiction to what the problem asks(it asks that all 3 sequences are increasing but now we have: x_24$ the triangle that contains the blue side must also contain exactly one of the red sides adjacent to it. We get two cases. In each case by removing this triangle we can apply the induction step to get $ 2^{n-4}$ possibilities, for a total of $ 2^{n-3}$.\r\n\r\nLemma 2: A polygon with $ n$ vertices has all its sides colored red except two which are blue. It is known that we have $ k$, respectively $ l$ consecutive red sides ($ k+l=n-2$). Then the number of triangulations such that each triangle contains at least one red side if $ \\binom{k+l}{k}$.\r\n\r\nThe proof may be done by induction on $ n=k+l+2$. If we denote by $ f(k,l)$ the given function then as the triangle containing one of the blue sides must contain one of the red adjacent sides we get $ f(k,l)=f(k,l-1)+f(k,l-1)$. A combinatorial proof is also possible: Assign plus to a triangle that contains one red side from the $ k$ and minus to a triangle that contains one red side from the $ l$. Starting from the blue side, we can visit all triangles (we go from one triangle to the next that shares a diagonal with it and was not already visited). We shall encounter a defining sequence of $ k$ pluses and $ l$ minuses in some order which may be chosen in $ \\binom{k+l}k$ ways.\r\n\r\nAlso note that lemma 1 follows from lemma 2 if we cut the unique triangle containing two consecutive red sides. \r\n\r\nThese two lemmas provide the key to the solution. Color the sides of the polygon red and the diagonals of the two interior triangles blue. The polygon will be split into the two interior triangles, four regions that have one blue side and one region with two blue sides (all the other sides are red, the last region may be empty). Let the four regions considered have $ a,b,c,d$ red sides and the last region have $ i$ and $ j$ consecutive red sides, $ a+b+c+d+i+j=n$. Then according to the lemmas we have $ 2^{a-2}2^{b-2}2^{c-2}2^{d-2}\\binom{i+j}{j}=2^{n-i-j-8}\\binom{i+j}{j}$ ways to complete the triangulation. \r\n\r\nWe also design a plan to count the triangulations. We can firstly find the $ i$ sides then find $ j$ then compute $ a+b$ then $ c+d$ and finally get $ a,b,c,d$. Each triangulation will occur exactly two times (namely, we could swap the two interior triangles and get the same configuration). So let's compute the total number of possibilities following the plan.\r\n\r\nNow assume that we've found the two blue sides. We thus know $ k=a+b, l=c+d, i, j$. It is clear that $ a,b$ and $ c,d$ may be chosen in $ k-3, l-3$ ways because $ a,b,c,d\\geq 2$. So we get $ (k-3)(l-3)2^{n-i-j-8}\\binom{i+j}{j}$ ways. \r\n\r\nNow set $ h=i+j$, then $ l=n-h-k$ so $ (k-3)(l-3)=(k-3)(n-h-k-3)=-k^{2}+k(n-h)-3(n-h-3)$. Also $ 4\\leq k \\leq n-h-4$.\r\n\r\nNext, we can place the $ i$ consecutive red sides in $ n$ ways, then we may choose $ 4\\leq k\\leq n-h-4$. The next $ j$ red sides will form the next group and the remaining will form the last group.\r\nSo we shall have a total of $ n\\cdot 2^{n-h-8}\\binom{h}{i}\\sum_{k=4}^{k=n-h-4}(-k^{2}+k(n-h)-3(n-h-3))$ ways. It can be computed as $ n\\cdot 2^{n-h-8}\\binom{h}{i}\\frac{s(s-1)(s+1)}6$ where $ s=n-h-6$ (we may prove that $ \\sum_{a+b=s, a,b \\in N}ab=\\frac{s(s-1)(s+1)}6$). So we get $ n\\cdot 2^{n-h-8}\\binom{h}{i}\\frac{s(s-1)(s+1)}6$. Next as $ i$ may take values from 0 to $ h$ and $ \\sum_{i=0}^{h}\\binom{h}{i}=2^{h}$, the sum over all $ i$ will be $ \\cdot n2^{n-h-8}2^{h}\\frac{s(s-1)(s+1)}6=n\\cdot 2^{n-8}\\binom{s+1}3$. Finally $ s$ may take values from $ 2$ to $ n-6$. And we know $ \\sum_{s=0}{m}\\binom{s}{3}=\\binom{m+1}{4}$. So we get from here that the total sum is $ n\\cdot 2^{n-8}(\\binom{n-4}{4}-\\binom{3}{4})$. Next we need to divide by 2 because every triangulation was computed twice, as we could start with either of the two sets of consecutive red sides. \r\n\r\nWe get $ n\\cdot 2^{n-9}\\cdot \\binom{n-4}{4}$. The same result as yours :)", "Solution_3": "My solution avoids calculations almost completely: \r\n\r\nFirst some notation:\r\n\r\nDenote the interior triangles by $ A_{1}A_{2}A_{3}$ and $ B_{1}B_{2}B_{3}$ such that the sides $ A_{1}A_{2}$ and $ B_{1}B_{2}$ are facing each other. \r\n\r\nThere is a tower of (say $ i$) triangles in between $ A_{1}A_{2}$ and $ B_{1}B_{2}$ and there are towers of triangles on each of the remaining sides $ A_{2}A_{3}$, $ A_{3}A_{1}$, $ B_{2}B_{3}$ and $ B_{3}B_{1}$, say consisting of $ j+1,k+1,m+1$ and $ l+1$ triangles (every such tower contains at least one triangle. Otherwise the triangles $ A_{1}A_{2}A_{3}$ and $ B_{1}B_{2}B_{3}$ wold not be interior triangles.) \r\n\r\nNow first choose $ A_{3}$. This can be done in $ n$ ways. Furthermore we have to keep in mind that we will count each triangulation twice (interchanging the roles of the two interior triangles.) \r\n\r\nNow let us fix values of $ i,j,k,m,l$. The only condition these numbers are required to satisfy is that they should be integers $ \\ge 0$ and that the resulting polygon should have $ n$ points. By simple counting this can be seen to be equivalent to the condition\r\n\\[ 1+(j+1)+(k+1)+(i+2)+(l+1)+(m+1)+1 = n, \\]\r\ni.e. \r\n\\[ i+j+k+l+m = n-8. \\]\r\nFor fixed $ i,j,k,m,l$ satisfying these conditions the number of triangulations can easily be calculated: Each tower of triangles can be constructed from the bottom line by addinga triangle in each step and determining which of the remaining two sides will be the outer side (= side of the given polygon) (the other one has to be the interior side = diagonal of the given polygon), for the topmost triangle both remaining sides have to be outer sides. \r\n(Of course for the triangulation in between the two interior triangles there is no topmost triangle.)\r\nThus altogether the number of triangulations is \r\n\\[ 2^{i}\\cdot 2^{j}\\cdot 2^{k}\\cdot 2^{l}\\cdot 2^{m}= 2^{i+j+k+l+m}= 2^{n-8}. \\]\r\n\r\nHowever the number of possible choices for $ i,j,k,l,m$ is just the number of partitions of $ n-8$ into a sum of 5 natural numbers, which is just \r\n\\[ \\binom{n-8+4}{4}= \\binom{n-4}{4}. \\]\r\nTo summarize the number of triangulations of the given polygon with the given properties is \r\n\\[ n \\binom{n-4}{4}2^{n-8}/ 2 = n \\binom{n-4}{4}2^{n-9}. \\]" } { "Tag": [ "IMO", "IMO 2003" ], "Problem": "Here is my IMO 2003 trip diary I write for a website that I'm one of the internal members:\r\nhttp://db.math.ust.hk/articles/imo_charlie/e_imo_charlie.htm\r\nIt's more related to the trip than to math, but well you may still find it intersesting :)", "Solution_1": "Very interesting. I also was in Japan last summer and your diary remember me that nice period. Charlie, you are not only good at mathematics, but at writing. Congratulation!\r\n\r\nNamdung", "Solution_2": "You all played so much mahjong that word got around pretty quickly it seems :)\r\n\r\nThat red pillar in the lift lobby sure brings back a lot of memories...", "Solution_3": "haha... I remembered the time when the polish team wanted to take a photo on the day of the closing ceremony, but they didn't have their national flag, so they borrowed Singapore's flag, folded it a bit and turned it up-side-down...\r\nAll of us in the Singapore team was so amused that we took pictures with them too.. all those interesting memories... :)" } { "Tag": [ "modular arithmetic", "combinatorics unsolved", "combinatorics" ], "Problem": "There are c red, p blue, and b white balls on a table. Two players A and B play a game by alternately making moves. In every move, a player takes two or three balls from the table. Player A begins. A player wins if after his/her move at least one of the three colors no longer exists among the balls remaining on the table. For which values of c, p, b does A have a winning strategy?\r\nPlease help me solve this problem. It is very hard for me. :(", "Solution_1": "If one of $c, p, b$ is at most $3$ then obviously A wins.\r\n\r\nIf any of $c, p, b$ become $3$ or smaller at any point in the game, it's done. In other words, a player loses at once if he makes one of $c, p, b$ at most $3$. So we can assume $c, p, b \\geq 4$ and take $C=c-4, P=p-4, B=b-4$. Now we can imagine that there are $C, P, B$ balls and the player who can't take his balls (i.e. is left with $0$ or $1$) loses. Note how the colors become irrelevant! Now suppose that $C+P+B \\equiv 0, 1 \\pmod{5}$. Then if $A$ takes $2$ balls, $B$ takes $3$ and viceversa, always handing $A$ a number of balls $\\equiv 0, 1$. Eventually $A$ will get $0$ or $1$ balls and lose. On the other hand, if $C+P+B \\equiv 2, 3, 4$ then $A$ can take 2, 2 or 3 balls on his first move and then play B's previous strategy. Since $C+P+B=c+p+b-12$, this translates to: $A$ wins if one of $c, p, b$ is $<4$, or if $c+p+b \\equiv 0, 1, 4$, and $B$ wins if $c, p, b \\geq 4$ and $c+p+b \\equiv 2, 3$.", "Solution_2": "Thank you very much Severius!", "Solution_3": "You're wrong, severius : if for example $c=p=1$ and $b=5$, A loses.", "Solution_4": "Umm... does he? Unless I misunderstood the condition for winning, A can take one of the $c$ balls and anything else, and win immediately.", "Solution_5": "Sorry.\r\n\r\nI misread the problem." } { "Tag": [ "logarithms", "geometry", "algebra", "function", "domain", "AMC" ], "Problem": "23. Square $ABCD$ has area 36, and $\\overline{AB}$ is parallel to the x-axis. Vertices $A, B,$ and $C$ are on the graphs of $y = \\log_{a}x, y = 2\\log_{a}x,$ and $y = 3\\log_{a}x$, respectively. what is $a$?\r\n\r\n$(\\text{A})\\ \\sqrt[6]3 \\quad (\\text{B})\\ \\sqrt3\\quad (\\text{C})\\ \\sqrt[3]6\\quad (\\text{D})\\ \\sqrt6 \\quad (\\text{E})\\ 6$", "Solution_1": "[hide]\nClearly each side has length 6.\n\nIt's not hard to see that $A$ will be the lower right, $B$ will be the lower left, and $C$ will be the upper left. Let $B$ be $(x, y)$. We then have 3 equations for 3 variables.\n\n$y = 2\\log_{a}x$\n$y = \\log_{a}{(x+6)}$\n$y+6 = 3\\log_{a}x \\implies y = 3\\log_{a}x-6$\n\nFrom the first and the second:\n$2\\log_{a}x = \\log_{a}{(x+6)}$\n$\\log_{a}{\\frac{x^{2}}{x+6}}= 0$\n$\\frac{x^{2}}{x+6}= 1$\n$x^{2}= x+6$\n$x^{2}-x-6 = 0 = (x-3)(x+2)$ Since $x>0$ because of the domain of a logarithm, $x = 3$.\n\nNow, from the first and the third:\n$2\\log_{a}3 = 3\\log_{a}3-6$\n$6 = \\log_{a}3$\n$a^{6}= 3$\n$a = \\sqrt[6]3 = (\\text{A})$\n[/hide]", "Solution_2": "[hide=\"I liked this one.\"]\nSay that $A=(x,y)$. Then $B=(x+6,y)$ or $(x-6,y)$. $\\log_{a}x < 2\\log_{a}x+6$, clearly, so $B=(x-6,y)$. By the same reasoning, $C=(x-6,y+6)$. We thus have \n\n$y = \\log_{a}x$\n\n$y = 2\\log_{a}x-6$\n\n$y+6 = 3\\log_{a}x-6$\n\nSubtracting the second equation from the third, we get \n\n$6 = \\log_{a}x-6\\Rightarrow 2\\log_{a}x-6 = y = 12$\n\nWith the first equation, this gives $x = a^{12}$. \n\nThe first two equations give $(x-6)^{2}=x$, for which the solutions are 9 and 4. 4 runs into problems with $\\log_{a}x-6$, so $x=9$.\n\nThus $a^{12}=9=3^{2}$ and $a=\\sqrt[6]{3}$. A.\n\n[/hide]", "Solution_3": "I didn't like it because being the idiot I am I placed vertex A to the left of Vertex B. That yielded an answer of -3.5 :roll:", "Solution_4": "I liked it.\r\n\r\n[hide]\nLet A=(x,y). Then B=(x-6,y) and C=(x-6, y+6).\n\nFrom the problem we know that\n$\\log_{a}x = y \\qquad\\ \\iff \\qquad a^{y}=x$\n$2\\log_{a}{(x-6)}=y \\qquad\\iff\\qquad a^{y}=(x-6)^{2}$\n$3\\log_{a}{(x-6)}=y+6\\qquad\\iff\\qquad a^{y+6}=(x-6)^{3}$\nFrom the first two we know that $x=4$ or $x=9$.\nFrom the second and third:\n$a^{y+6}=\\left(a^{y}\\right)\\left(a^{6}\\right)=(x-6)^{3}$\n$(x-6)^{2}\\cdot a^{6}=(x-6)^{3}$\n$a^{6}=x-6=-2 \\text{ or }3$\n-2 doesn't work, so $a^{6}=3$ and $a=\\sqrt[6]3$\n\n[/hide]", "Solution_5": "[quote=\"1337h4x\"]I didn't like it because being the idiot I am I placed vertex A to the left of Vertex B. That yielded an answer of -3.5 :roll:[/quote]\r\n\r\nThe way I solved it, that didn't matter. Of course, I didn't finish solving it until 30 seconds after the test was over, but..." } { "Tag": [], "Problem": "How many different ways can the five letters of the word STATE be scrambled if the two T's cannot be consecutive?", "Solution_1": "There are $ \\frac{120}{2!}$ or $ 60$ ways to arrange the letters without restrictions. From these, there are $ 4$ cases we must subtract:\r\n\r\nCase 1: TTxxx- $ 6$ ways\r\n\r\nCase 2: xTTxx- $ 6$ ways\r\n\r\nCase 3: xxTTx- $ 6$ ways\r\n\r\nCase 4: xxxTT- $ 6$ ways\r\n\r\nThus, the answer is $ 90 \\minus{} 6(4)$, or $ \\boxed{66}$.", "Solution_2": "You can just note that there are $ 4$ spots to put the double T, and $ 6$ ways to arrange the remaining letters, for $ 24$ invalid cases. Also, it's $ 60\\minus{}6(4)\\equal{}\\boxed{36}$." } { "Tag": [ "calculus", "integration", "algebra", "function", "domain", "Ring Theory", "superior algebra" ], "Problem": "If $R=\\mathbb{Z}[\\sqrt{d}]$, then show that $F\\cong \\lbrace r+s\\sqrt{d}|r,s\\in\\mathbb{Q}\\rbrace$ where $R$ is an integral domain and $F$ is the field of quotients.\r\n\r\nIs there an easier way to do this other than defining a map from $F$ to $\\lbrace r+s\\sqrt{d}|r,s\\in\\qq \\rbrace$? I foresee this becoming no more than a tedious task of basic algebraic manipulation...", "Solution_1": "why tedious? it's obvious :-)\r\nand of course you can generalize it to adjunctions of integral elements." } { "Tag": [], "Problem": "What is the sum of all positive odd multiples of 3 that are less \nthan 100?", "Solution_1": "$ \\frac {100} {3}$ $ \\equal{}$ $ 33.333...\\sim 33$ positive multiples of 3 less than 100. Since we start with an odd and end with an odd, we add $ 33\\plus{}1$ $ \\equal{}$ $ 34$ and divide by $ 2$ in order to get the number of odd multiples of three. $ \\frac {34} {2}$ $ \\equal{}$ $ 17$. We add the first and last of such numbers. $ 3$ + $ 99$ $ \\equal{}$ $ 102$. Now we do $ 17\\minus{}1$ $ \\equal{}$ $ 16$ and divide by two. $ \\frac {16} {2}$ $ \\equal{}$ $ 8$ and multiply it by $ 102$. $ 8$ times $ 102$ $ \\equal{}$ $ 816$. But we still hhave the middle term of the sequence $ 3, 9, 15... 99$, so we divide $ 102$ in half (to average the first and last to get the middle). $ \\frac {102} {2}$ $ \\equal{}$ $ 51$, which we add to $ 816$ to get $ \\boxed{\\boxed{\\boxed {867}}}$.", "Solution_2": "here's easier way:\r\n\r\nthe answer is 3*1 + 3*3 + ... + 3*33, which simplifies as 3*(1+3+...+33).\r\n\r\nand there's total 17 odd numbers in the parentheses, which is $ 17^2$, 289.\r\n\r\nsoo the answer is 3*289=867." } { "Tag": [ "AMC", "AIME" ], "Problem": "Approximately how many people made it into the AIME in NY state this year? If you don't know, where can I find out?", "Solution_1": "You can find that by looking at the [url=http://www.unl.edu/amc/e-exams/e6-amc12/e6-1-12archive/2009-12a/09-1-1012-StateStats.shtml]state stats[/url]." } { "Tag": [ "geometry unsolved", "geometry" ], "Problem": "[color=darkblue]Let $ S_1$ and $ S_2$ be circles meeting at the points $ A$ and $ B$. A line through $ A$ meets $ S_1$ at $ C$ and $ S_2$ at $ D$. Points $ M,N,K$ lie on the line segments $ CD, BC, BD$ respectively, with $ MN$ parallel to $ BD$ and $ MK$ parallel to $ BC$. Let $ E$ and $ F$ be points on those arcs $ BC$ of $ S_1$ and $ BD$ of $ S_2$ respectively that do not contain $ A$ . Given that $ EN$ is perpendicular to $ BC$ and $ KF$ is perpendicular to $ BD$ .Prove that $ \\angle EMF \\equal{} 90^0$.[/color]\r\n\r\n[color=red][b]Pleases tell me another solution. Thank you very much.[/b][/color]", "Solution_1": "http://www.mathlinks.ro/viewtopic.php?p=118681#118681", "Solution_2": "Thank you. But I read it some times, now I want to see another solutions.", "Solution_3": "There are 2 solutions there, and no need to open a new thread for the same question (and you knew for its existance!)" } { "Tag": [], "Problem": "A college student drove his compact car 120 miles home for the weekend and\naveraged 30 miles per gallon. On the\nreturn trip the student drove his parents' SUV and averaged only 20 miles per gallon.\nWhat was the average gas mileage, in miles per gallon, for the round trip?", "Solution_1": "We see that $ \\frac{120}{30}\\equal{}4$, and $ \\frac{120}{20}\\equal{}6$. Adding $ 4\\plus{}6\\equal{}10$, we get he used 10 gallons. Now dividing $ 240$ by $ 10$, we get $ 24$." } { "Tag": [ "number theory unsolved", "number theory" ], "Problem": "Find all triples of natural numbers $(x, y, n)$ such that $\\frac{x!+y!}{n!}=3^{n}$", "Solution_1": "[hide] \n\nCase 1: $x,y 2n!$ for all $n>1$, we need only check the case $n=1$. This doesn't work though.\n\nCase 2: $xy$ then\r\n$[x(x-1)...(y+1)+1]y!=3^{n}n!$. Therefore $y=n$ or $y=n+1$.\r\nIf y=n we have $x(x-1)...(n+1)=3^{n}-1\\Longrightarrow x\\le n+2$.\r\nIn this case we have only one solution $x=2,y=n=1$.\r\nLet $y=n+1$. It give $x(x-1)...(n+2)+1=\\frac{3^{n}}{n+1}$. Therefore $n=3^{k}-1,x\\le n+3$. It give $(3^{k}+1)(3^{k}+2)+1=3^{3^{k}-k-1}$\r\nIn this case there are not solutions.", "Solution_3": "I'm sorry, but I do not really understand why $y=n$ or $y=n+1$. Could you explain?", "Solution_4": "[quote=\"pdiao\"]I'm sorry, but I do not really understand why $y=n$ or $y=n+1$. Could you explain?[/quote]\r\nBecause $[x(x-1)...(y+1)+1]y(y-1)...(n+1)=3^{n}$.\r\nIf $y>n+1$ then (n+1)(n+2) had prime divisor distinct 3.", "Solution_5": "Oh, right, doh.", "Solution_6": "Already posted, but I guess I'm a little bit too late :wink: \r\n\r\nhttp://www.mathlinks.ro/Forum/viewtopic.php?p=183463#183463" } { "Tag": [ "AMC", "AIME", "counting", "distinguishability" ], "Problem": "Okay, this is how my tournament works:\r\n\r\n1) Peeps will sign up.\r\n2) Those peeps will be sent the same problem, but with the numberings altered.\r\n3) Those peeps will send me back a solution and an answer.\r\n\r\nEDIT: Problems may not be the same, but I will try to have the difficulty the same. Maybe Group A will be sent this problem, and Group B the other, like that. (using random.org of course.)\r\n\r\nGrading:\r\n\r\nIf your answer is wrong, but I like your solution, I will award you 20 points. If both answer and solution are god, you will get 50 points. If the answer is good but not the solution, then you will get 10 points.\r\n\r\nRules:\r\n\r\nthe aid of a calculator is not permitted, neither is anything else.\r\n\r\nSo sign up by PMing me or posting here!!\r\n\r\nNOTE: I will try to end reminder PM's.", "Solution_1": "I'll join.", "Solution_2": "/in!!\r\n\r\ni completed step 1", "Solution_3": "I'll join!", "Solution_4": "Current sign-ups:\r\n\r\nhigh_flyer10\r\nAIME15\r\nghjk\r\nBOGTRO\r\nnikeballa96\r\nAIME_is_hard\r\n\r\nI've also added a new rule:\r\n\r\nInactivity will result in subtraction of points. BTW: Each round will have like 3 questions and each one is worth 50 points.", "Solution_5": "Eh, join but I might drop out later.", "Solution_6": "[hide=\"because i really feel like making a hide thing\"] NIKE JOINS! [/hide]\r\n\r\n=]", "Solution_7": "OK, but I might drop out", "Solution_8": "I join :starwars: :starwars:", "Solution_9": "I join.", "Solution_10": "Current sign-ups: \r\n\r\nhigh_flyer10 \r\nAIME15 \r\nghjk \r\nBOGTRO \r\nnikeballa96 \r\nAIME_is_hard \r\nisabella2296\r\nSuperMathBoy\r\nDojo\t\r\nTwin Prime Conjecture\r\n\r\n10 so far...need more!!", "Solution_11": "[quote=\"007math\"]Maybe Group A will be sent this problem, and Group B the other, like that. (using random.org of course.)\n[/quote]\r\n\r\ngroups??", "Solution_12": "[quote=\"AIME_is_hard\"][quote=\"007math\"]Maybe Group A will be sent this problem, and Group B the other, like that. (using random.org of course.)\n[/quote]\n\ngroups??[/quote]\r\n\r\nEven if there are groups, the scores fo that one group will not effect on another. You wouldn't even know what group you are in unless you figure out who has the same questions as you. I need 8+ more people!!!", "Solution_13": "Oh well what the heck\r\njoin", "Solution_14": "I'll join.", "Solution_15": "[quote=\"BOGTRO\"]can someone give me the probs?[/quote]\r\n\r\nI sent them to you, but I'll send them again.\r\n\r\nExtending until Wednesday.", "Solution_16": "athunder turned in his work and got 100/100!!", "Solution_17": "Hurry up and submit!!! You'll be eliminated if you don't!!", "Solution_18": "[quote=\"athunder\"]How many rounds are you planning on having?[/quote]\r\n\r\nUNtil I have a winner...", "Solution_19": "Round 2 ended today. Scores will be up either tomorrow and Round 3 will be out Tomorrow also.", "Solution_20": "Scores for Round 2:\r\n\r\n\r\n[hide=\"Round 1 Scores\"]\nBOGTRO: 150 \nathunder: 150 \nDiscrete_Math : 150 \nhigh_flyer10: 150 \nDojo: 150 \nAIME15: 150 \nghjk : 140 \nsomeperson01 : 135 \nisabella2296: 110 \nTwin Prime Conjecture: 110 \n[/hide]\n\n[hide=\"Round 2 Scores\"]\nTwin Prime Conjecture: 100\nhigh_flyer10: 100\nDiscrete_Math: 100\nathunder : 100\nBOGTRO: 100\nghjk : 85\n\n[/hide]\n\n[hide=\"Official Current Scores\"]\nBOGTRO: 250\nathunder: 250\nDiscrete_Math : 250\nhigh_flyer10: 250\nTwin Prime Conjecture: 210\nghjk : 225\nDojo: 150 \nAIME15: 150 \nsomeperson01 : 135 \nisabella2296: 110 \n[/hide]\r\n\r\nThe eliminated people are: \r\n\r\nsomeperson01\r\nisabella2296\r\nDojo (random.org got 4 heads for AIME and 1 for you....sorry...)\r\n\r\nNow the scores are:\r\nBOGTRO: 250\r\nathunder: 250\r\nDiscrete_Math : 250\r\nhigh_flyer10: 250\r\nTwin Prime Conjecture: 210\r\nghjk : 225\r\nAIME15: 150 \r\n\r\n\r\nRound 3 will be sent.", "Solution_21": "These are the Questions for Round 3...\r\n\r\n1. I go to school and open locker 1. Then I open locker two until I get to locker 50. I go back to locker 1 and close every locker number which is a multiple of 2. I go back to locker 1 and close a locker if it is open or open a locker if it is closed if the number is a multiple of 3. I do this until the multiples of 6. How many lockers are open?\r\n\r\n2. My friend has 6 apples. I have 9 banana. My other friend has 10 oranges. I decided I want to distribute these fruits among ourselves, how many ways are there to do this if each type of fruit is indistinguishable?\r\n\r\n3. One day, Billy99 decided to inscribe his name on a door and install it in his bedroom. The current door he has he can sell for 50 dollars. Billy99 will have to buy a special door costing 200 dollars, and for each capital letter, it costs him the positive value of $ x$ in $ x^2 \\minus{} 2x \\minus{} 143$. For every lowercase letter, it costs him $ xy \\minus{} y \\equal{} 108$ dollars where $ x$ is $ x$ from the previous equation and where $ y$ is the cost of a lowercase letter. For each different type of character (ex. a number) it costs him $ xy^2 \\plus{} x^2y$ dollars where $ x$ and $ y$ are the cost of uppercase letters and lowercase ones respectfully. But if Billy99 takes two of the same letter or number, he gets a 10% discount four those 2 letters or numbers. What is the amount of money it will cost him to get his name on a door if he sells his old one and buys the special one?\r\n\r\nThese are due on [b]January 9, 2009[/b]", "Solution_22": "For number 3, there are an infinite amount of possible values of x^2-2x-143. Do you mean x^2-2x-143=0", "Solution_23": "[quote=\"AIME15\"]For number 3, there are an infinite amount of possible values of x^2-2x-143. Do you mean x^2-2x-143=0[/quote]\r\n\r\nYeah... :fool:", "Solution_24": "To end this quickly, the person with the highest score will win. This time I will grade very aggressively, I'm going take off points for incorrect grammar too and more...", "Solution_25": "[quote=\"007math\"]To end this quickly, the person with the highest score will win. This time I will grade very aggressively, I'm going take off points for incorrect grammar too and more...[/quote]\r\nWhat? Grammar is also counted? By the way, you already graded me harshly from the beginning though. Will you show your solution to every problems you give us at the end? :)", "Solution_26": "If you miss a comma, misspell a few words is okay. But making it very hard to understand is what i mean (though many off you don't do that). Because of (someones's...) request, I will post solutions.\r\n\r\nLike see \"Writing a Solution\"...", "Solution_27": "Answers are due tomorrow!! I will not be extending the deadline, so turn in, or th current scores will be used to determine winner!!!", "Solution_28": "Todays the last day..you have just a few hours until this end!!!!!!!!!", "Solution_29": "Well, no one turned in answers, so...THE WINNERS OF 007MATH'S TOURNEY!! (I used time to put places)\r\n\r\n[hide=\"In 3rd Place came.....\"]\n\nDiscrete_Math :winner_third: Good Job!!\n\n[hide=\"IN 2nd place....\"]\n\nathunder :winner_second: Good Job\n\n[hide=\"And in 1st place, the person who turned in their answers first (being one of the first) was...\"]\n\nBOGTRO!!!! :first: GOOd Job!!\n\n\n[/hide][/hide][/hide]\r\n\r\nGood Job to everyone who participated. ANd for those solutions...no one turned in answers for Round 3, so I guess there is no reason to post them. They weren't that hard anyway." } { "Tag": [ "ratio", "trigonometry", "trig identities", "Law of Cosines" ], "Problem": "Source: Massachusetts Mathematics Leagues States 2004\r\n\r\nRegular hexagon $ABCDEF$ is inscribed in a circle. $X$ is the midpoint of arc $\\widehat{DC}$. Determine the exact value of $\\frac{AX}{DX}$.", "Solution_1": "obviously $2.5/.5$ or 5.", "Solution_2": "I think he means line segments", "Solution_3": "That is wrong.\r\n\r\n[hide]If the radius of the circle is $r$, then by the Cosine Law we have\n\n$AX^2=r^2+r^2-2r^2\\cos 150^\\circ=2r^2(1+\\frac{\\sqrt{3}}{2})=r^2(2+\\sqrt{3})$\n\n$DX^2=r^2+r^2-2r^2\\cos 30^\\circ=2r^2(1-\\frac{\\sqrt{3}}{2})=r^2(2-\\sqrt{3})$\n\nTherefore $\\frac{AX^2}{DX^2}=\\frac{2+\\sqrt{3}}{2-\\sqrt{3}}=\\frac{(2+\\sqrt{3})^2}{1}=(2+\\sqrt{3})^2$, giving\n\n$\\frac{AX}{DX}=2+\\sqrt{3}$[/hide]", "Solution_4": "[hide]$\\widehat{DC}=60 \\rightarrow \\widehat{DX}=30$. Inscribed angles are half of their intercepted arcs, so $\\angle{XAD}=15$, $\\angle{XDA}=75 \\rightarrow \\angle{AXD}=90$. Right triangle $ADX$ is formed, $\\frac{AX}{DX}=\\tan{75}=\\boxed{2+\\sqrt3}$[/hide]\r\n(edit: forgot hat)", "Solution_5": "This may be a little off topic, but does anyone have a link that explains the properties of a 15/75 right triangle? I never learned them and it might be a good idea.\r\n\r\nthanks", "Solution_6": "[quote=\"Pocket Sand\"]This may be a little off topic, but does anyone have a link that explains the properties of a 15/75 right triangle? I never learned them and it might be a good idea.\n\nthanks[/quote]\r\nI dont have a link but i can explain the basics....\r\n\r\nFirst, lets get those trig ratios (which can be found this way or by using the half-angle identities and denesting the radicals: \r\n$\\sin{75}=\\sin{45+30}=\\sin{45}\\cos{30} + \\cos{45}\\sin{30} = \\boxed{\\frac{\\sqrt{6}+\\sqrt{2}}{4}}$.\r\n$cos{75}=\\cos{45}\\cos{30}-\\sin{45}\\sin{30}=\\boxed{\\frac{\\sqrt{6}-\\sqrt{2}}{4}}$.\r\n$tan(75)=\\frac{\\sqrt{6}+\\sqrt{2}}{\\sqrt{6}-\\sqrt{2}}=\\boxed{2+\\sqrt{3}}$. (this one is the most useful)\r\n\r\nThe sides are then $x(\\frac{\\sqrt{6}-\\sqrt{2}}{4}),$ $x(\\frac{\\sqrt{6}+\\sqrt{2}}{4}),$ $x$. I'm sure you could google a web page that explains more properties, but what you can do if you need the properties and dont remember the numbers is draw a 30-75-75 triangle, call a leg a simple number like 1 or 2 (or whatever value you need), and then use the Law of Cosines to solve for the other values, dividing the base in two when you draw the altitude to get the desired triangle:\r\n$a^2=b^2+c^2-2bc\\cos{A} \\rightarrow \\boxed{a^2=2b^2(1-\\cos{30^\\circ)})}$, where a is the base and b is the isosceles legs. remember then that the 15 degree side of the 15-75-90 triangle then is $\\frac{a}{2}$, not $a$.\r\n\r\nHope this helps! :D" } { "Tag": [ "algebra", "binomial theorem" ], "Problem": "What is the coefficient of $ x^2y^2$ in the expansion of $ (x\\plus{}y)^4\\plus{}(x\\plus{}2y)^4$?", "Solution_1": "$ (x\\plus{}y)^4\\equal{}x^4\\plus{}4x^3y\\plus{}6x^2y^2\\plus{}4xy^3\\plus{}y^4$\r\n$ (x\\plus{}2y)^4\\equal{}x^4\\plus{}8x^3y\\plus{}24x^2y^2\\plus{}32xy^3\\plus{}16y^4$\r\nThus, $ 6\\plus{}24\\equal{}\\boxed{30}$.", "Solution_2": "This would be kinda hard to do during a countdown, though he is right. Instead, we can use the [url=http://www.artofproblemsolving.com/Wiki/index.php/Binomial_Theorem]Binomial Theorem[/url]. So we have $ \\binom{4}{2}(x)^2(y)^2\\plus{}\\binom{4}{2}(x)^2(2y)^2\\equal{}6x^2y^2\\plus{}6x^2\\cdot4y^2\\equal{}\\boxed{30}x^2y^2$. Sorry if it seems kind of confusing for those who don't know the Binomial Theorem, it's kind of hard to explain :oops:", "Solution_3": "hmmm... what is the binomial theorem?", "Solution_4": "Jasonion,\n\nthe Binomial Theorem states that\n\n$(x+y)^n=\\binom{n}{0}x^n+\\binom{n}{1}x^{n-1}y+\\binom{n}{2}x^{n-2}y^2+\\cdots+\\binom{n}{n}y^n$, where $\\binom{n}{k}=\\frac{n!}{k!(n-k)!}$\n\nEdit: Thanks for correction, BOGTRO!", "Solution_5": "[quote=\"usamo42j\"]Jasonion,\n\nthe Binomial Theorem states that\n\n$(x+y)^n=\\binom{n}{0}+\\binom{n}{1}+\\binom{n}{2}+\\cdots+\\binom{n}{n}$, where $\\binom{n}{k}=\\frac{n!}{k!(n-k)!}$[/quote]\n\nThe Binomial Theorem states that \n\n$(x+y)^n=\\binom{n}{0}x^ny^0+\\binom{n}{1}x^{n-1}y^1+\\hdots+\\binom{n}{n}x^0y^n$." } { "Tag": [ "geometry" ], "Problem": "[b]IMPORTANT[/b]: im looking for hints ONLY!!!\r\nTHat means no solutions and no answers!\r\n\r\nim trying to avoid looking at the solution and i dont want to put up the problem again but here it is\r\n\r\nhttp://www.artofproblemsolving.com/Forum/viewtopic.php?p=455310#455310", "Solution_1": "[quote=\"Smartguy\"][b]IMPORTANT[/b]: im looking for hints ONLY!!!\nTHat means no solutions and no answers!\n\nim trying to avoid looking at the solution and i dont want to put up the problem again but here it is\n\nhttp://www.artofproblemsolving.com/Forum/viewtopic.php?p=455310#455310[/quote]Since the solutions in that topic are all hidden, why would you start another topic? You can ask for hints in that topic and ask for no solutions or hide solutions. This topic can be locked, it's pointless and considered SPAM\r\n :spam:", "Solution_2": "mods: please dont lock this!!\r\ni meant this to be a post for hints, i didn't want to revive an old thread and have people be yelling at me for doing that...\r\n\r\ncan someone give some hints anyways?\r\n\r\n\r\nThanks!", "Solution_3": "[hide=\"A starting direction\"]Let $ \\theta \\equal{} \\text{m} \\angle B$, and let $ a_1$ and $ a_2$ be the sidelengths of squares $ S_1$ and $ S_2$ (do not compute $ a_1,2$ yet). Use these values to find expressions for the areas of the two triangles. What happens when you equate them?[/hide]", "Solution_4": "People should be ashamed of for yelling at other individual for reviving an old topic because he wanted hints and [u]needed the necessary help to learn[/u]. And this should be addressed to [u]everyone[/u] on AoPS/ML and [i]MUST BE reminded that there is NO such rule for not reviving old topics out of necessity[/i].", "Solution_5": "thank you for your insight TZF, i have solved it thanks to you and a friend :D \r\n :D \r\n :D \r\n\r\n\r\nThank you for spamming in this post, 10000th user!", "Solution_6": "[quote=\"Smartguy\"]thank you for your insight TZF, i have solved it thanks to you and a friend :D \n :D \n :D \n\n\nThank you for spamming in this post, 10000th user![/quote]Your welcome, creator of a spammable topic! :D" } { "Tag": [ "inequalities", "inequalities solved" ], "Problem": "For positive reals a , b show that\r\n[(a+1)/(b+1)]^(n+1) >=(a/b)^b\r\n\r\n24 th swedish mo , 1984\r\nthanks...", "Solution_1": "This can't be true.\r\nChoose any a,b > 0 such that a < b, and choose n sufficientely large, you will obtain the reverse inequality.\r\n\r\nPierre." } { "Tag": [ "inequalities unsolved", "inequalities" ], "Problem": "This is an older problem.", "Solution_1": "This problem is wrong...\r\nThe true problem is \r\n$\\frac{a}{b^{2}+1}+\\frac{b}{c^{2}+1}+\\frac{c}{a^{2}+1}\\geq\\frac{3(a\\sqrt{a}+b\\sqrt{b}+c\\sqrt{c})^{2}}{4}$\r\n\r\nand it is very easy just Cauchy in Engel form :wink: .", "Solution_2": "[quote=\"silouan\"]This problem is wrong...\nThe true problem is \n$\\frac{a}{b^{2}+1}+\\frac{b}{c^{2}+1}+\\frac{c}{a^{2}+1}\\geq\\frac{3(a\\sqrt{a}+b\\sqrt{b}+c\\sqrt{c})^{2}}{4}$\n\nand it is very easy just Cauchy in Engel form :wink: .[/quote]\r\n\r\n I think you are right. I found the same result , by writing $\\frac{a}{b^{2}+1}= \\frac{a^{3}}{a^{2}b^{2}+a^{2}}= \\frac{(a \\cdot \\sqrt a)^{3}}{a^{2}b^{2}+a^{2}}$ and then applying Caushy in Engel - Andreescu form. But I could not find the original source , to ensure the problem.\r\n Thank you Silouan for your nice information !", "Solution_3": "[quote=\"stergiu\"][quote=\"silouan\"]This problem is wrong...\nThe true problem is \n$\\frac{a}{b^{2}+1}+\\frac{b}{c^{2}+1}+\\frac{c}{a^{2}+1}\\geq\\frac{3(a\\sqrt{a}+b\\sqrt{b}+c\\sqrt{c})^{2}}{4}$\n\nand it is very easy just Cauchy in Engel form :wink: .[/quote]\n\n I think you are right. I found the same result , by writing \n $\\frac{a}{b^{2}+1}= \\frac{a^{3}}{a^{2}b^{2}+a^{2}}= \\frac{(a \\cdot \\sqrt a)^{2}}{a^{2}b^{2}+a^{2}}$ and then applying Caushy in Engel - Andreescu form. But I could not find the original source , to ensure the problem.\n Thank you Silouan for your nice information ![/quote]", "Solution_4": "I have the original text in my house .\r\nThe correct problem is the above...... :)" } { "Tag": [ "function", "inequalities unsolved", "inequalities" ], "Problem": "Prove that if $x,y,z\\geq0$, then\r\n\\[\\frac1{\\sqrt{4x^{2}+yz}}+\\frac1{\\sqrt{4y^{2}+zx}}+\\frac1{\\sqrt{4z^{2}+xy}}\\geq\\frac2{\\sqrt{xy+yz+zx}}.\\]", "Solution_1": "[quote=\"pvthuan\"]Prove that if $ x,y,z\\geq0$, then\n\\[ \\frac1{\\sqrt {4x^{2} \\plus{} yz}} \\plus{} \\frac1{\\sqrt {4y^{2} \\plus{} zx}} \\plus{} \\frac1{\\sqrt {4z^{2} \\plus{} xy}}\\geq\\frac2{\\sqrt {xy \\plus{} yz \\plus{} zx}}.\\]\n[/quote]\r\nLet $ x\\plus{}y\\plus{}z\\equal{}1.$ Since, $ f(t)\\equal{}\\frac{1}{\\sqrt t}$ is convex function we obtain:\r\n$ \\sum_{cyc}\\frac1{\\sqrt {4x^{2} \\plus{} yz}}\\equal{}\\sum_{cyc}\\frac{\\frac{y\\plus{}z}{2}}{\\sqrt {\\frac{(y\\plus{}z)^2(4x^{2} \\plus{} yz)}{4}}}\\geq\\frac{1}{\\sqrt{\\sum\\frac{(y\\plus{}z)^3(4x^2\\plus{}yz)}{8}}}.$\r\nId est, it remains to prove that \r\n$ 2(x\\plus{}y\\plus{}z)^3(xy\\plus{}xz\\plus{}yz)\\geq\\sum_{cyc}(y\\plus{}z)^3(4x^2\\plus{}yz),$\r\nwhich is equivalent to $ \\sum_{sym}(x^4y\\minus{}x^3y^2\\plus{}7x^3yz)\\geq0,$ which is obvious.", "Solution_2": "See also here:\nhttp://www.artofproblemsolving.com/Forum/viewtopic.php?f=52&t=422538" } { "Tag": [ "inequalities", "inequalities unsolved" ], "Problem": "$ a,b,c \\ge 0$\r\n$ a \\plus{} b \\plus{} c \\equal{} 1,F(a,b,c) \\equal{} \\sqrt {\\frac {1 \\minus{} a}{1 \\plus{} a}} \\plus{} \\sqrt {\\frac {1 \\minus{} c}{1 \\plus{} c}} \\plus{} \\sqrt {\\frac {1 \\minus{} c}{1 \\plus{} c}}$\r\nFind the max and min vaule of F(a,b,c).\r\n\r\n[hide]\nthe Min value is f(1,0,0),and Max is F(0.5,0.5,0)\n[/hide]", "Solution_1": "For max try $ a$ close to $ \\minus{}1$, $ b\\equal{}1,c$ close to $ 1$", "Solution_2": "I am sorry for missing a condition that $ a,b,c>0$", "Solution_3": "Take $ f(x)\\equal{}\\sqrt{\\frac{1\\minus{}x}{1\\plus{}x}}$ and this problem could be solved by Karamata Inequality (though not so obvious and direct).", "Solution_4": "Notice that\r\n\r\n$ f''(x) \\equal{} \\frac {1 \\minus{} 2x}{\\sqrt{(1 \\plus{} x)^{5}(1 \\minus{} x)^{3}}}$.\r\n\r\nWLOG we assume $ a\\le b\\le c$,\r\n\r\n[b]Case 1[/b]: $ a \\plus{} b\\le \\frac {1}{2}$.\r\n\r\nWe obtain\r\n\r\n$ f(a) \\plus{} f(b)\\le f(0) \\plus{} f(a \\plus{} b)$.\r\n\r\n[b]Case 2[/b]: $ a \\plus{} b > \\frac {1}{2}$.\r\n\r\nWe obtain\r\n\r\n$ f(a) \\plus{} f(b)\\le f\\left( a \\plus{} b \\minus{} \\frac {1}{2}\\right) \\plus{} f\\left( \\frac {1}{2} \\right)$.\r\n\r\nSo we may assume that one of $ a,b$ is $ 0$ or $ \\frac {1}{2}$.\r\n\r\n[b]Case 1[/b]: $ a \\equal{} 0$.\r\n\r\n$ f(a) \\plus{} f(b) \\plus{} f(c)\\le 0 \\plus{} 1 \\plus{} 1 < 1 \\plus{} \\frac {2}{\\sqrt {3}}$.\r\n\r\n[b]Case 2[/b]: $ a \\equal{} \\frac {1}{2}$.\r\n\r\n$ a \\plus{} b \\plus{} c\\ge \\frac {3}{2} > 1$ which contradicts.\r\n\r\n[b]Case 3[/b]: $ b \\equal{} 0$.\r\n\r\n$ f(a) \\plus{} f(b) \\plus{} f(c) \\equal{} 1 \\plus{} 1 \\plus{} 0 < 1 \\plus{} \\frac {2}{\\sqrt {3}}$.\r\n\r\n[b]Case 4[/b]: $ b \\equal{} \\frac {1}{2}$.\r\n\r\n$ f(a) \\plus{} f(b) \\plus{} f(c)\\le f(0) \\plus{} f(b) \\plus{} f(a \\plus{} c) \\equal{} 1 \\plus{} \\frac {2}{\\sqrt {3}}$." } { "Tag": [], "Problem": "I left my TI-83+ at school and im on spring break. I have an 89 forwhatever reason. Will they let me use it?\r\nThanks.", "Solution_1": "yes!", "Solution_2": "My friend has an nspire is he allowed to use that also?", "Solution_3": "[quote=\"abacadaea\"]My friend has an nspire is he allowed to use that also?[/quote]\r\nOh, yeah. That's why they gave those out in Nats last year...... :D", "Solution_4": "yay! Thank you." } { "Tag": [ "calculus", "integration", "ratio", "geometry", "perimeter", "quadratics", "LaTeX" ], "Problem": "Show that the number of integral-sided right triangles whose ratio of area to semi-perimeter is $p^{m}$, where $p$ is a prime and $m$ is an integer, is $m+1$ if $p=2$ and $2m+1$ if $p \\neq 2$.", "Solution_1": "The problem is equivalent to finding the number of solutions in positive integers to the following system of equations:\r\n\\[ \\begin{cases} a^2 \\plus{} b^2 \\equal{} c^2 \\\\\r\nab \\equal{} (a \\plus{} b \\plus{} c)p^m\\end{cases}\r\n\\]\r\nDenoting $ a \\plus{} b \\equal{} t$, we have $ t^2 \\equal{} c^2 \\plus{} 2(t \\plus{} c)p^m$ implying that $ (t \\plus{} c)(t \\minus{} c) \\equal{} 2(t \\plus{} c)p^m$ and\r\n\\[ c \\equal{} t \\minus{} 2p^m.\r\n\\]\r\nTherefore, the original system is equivalent to\r\n\\[ \\begin{cases} a \\plus{} b \\equal{} t \\\\\r\nab \\equal{} 2(t \\minus{} p^m)p^m\\end{cases}\r\n\\]\r\nimplying that $ a, b$ are the roots of the following quadratic equation:\r\n\\[ x^2 \\minus{} t x \\plus{} 2(t \\minus{} p^m)p^m \\equal{} 0\r\n\\]\r\nwhich is solvable in integers only if its discriminant $ t^2 \\minus{} 8 (t \\minus{} p^m)p^m \\equal{} (t \\minus{} 4p^m)^2 \\minus{} 8p^{2m}$ is a square. \r\n\r\nIf $ (t \\minus{} 4p^m)^2 \\minus{} 8p^{2m} \\equal{} q^2$ then $ 8p^{2m} \\equal{} (t \\minus{} 4p^m \\minus{} q)((t \\minus{} 4p^m \\plus{} q)$. All possible non-negative such $ q$ are obtained as $ q \\equal{} \\frac {d_2 \\minus{} d_1}{2}$ where $ d_1 d_2 \\equal{} 8p^{2m}$ and $ d_2 \\geq d_1$ are of the same oddness.\r\n\r\nIf $ p \\equal{} 2$ then $ 8p^{2m} \\equal{} 2^{2m \\plus{} 3}$ and $ q \\equal{} \\frac {2^k \\minus{} 2^{2m \\plus{} 3 \\minus{} k}}{2} \\equal{} 2^{k \\minus{} 1} \\minus{} 2^{2m \\plus{} 2 \\minus{} k}$ where $ k \\equal{} m \\plus{} 2\\dots,2m \\plus{} 2$. These $ m \\plus{} 1$ distinct values of $ k$ give the following solutions:\r\n\\[ \\begin{cases} a \\equal{} 2^{k \\minus{} 1} \\plus{} 2^{m \\plus{} 1} \\\\\r\nb \\equal{} 2^{2m \\plus{} 2 \\minus{} k} \\plus{} 2^{m \\plus{} 1} \\\\\r\nc \\equal{} 2^{k \\minus{} 1} \\plus{} 2^{2m \\plus{} 2 \\minus{} k} \\plus{} 2^{m \\plus{} 1}\\end{cases}\r\n\\]\r\nIf $ p > 2$ then $ q \\equal{} \\frac {4p^k \\minus{} 2 p^{2m \\minus{} k}}{2} \\equal{} 2 p^k \\minus{} p^{2m \\minus{} k}$ (for $ k \\equal{} m,\\dots,2m$) or $ q \\equal{} \\frac {2p^k \\minus{} 4 p^{2m \\minus{} k}}{2} \\equal{} p^k \\minus{} 2 p^{2m \\minus{} k}$ (for $ k \\equal{} m \\plus{} 1,\\dots,2m$). Therefore, the number of solutions here is $ 2m \\plus{} 1$. Explicit solutions can be derived as above.", "Solution_2": "There is a MUCH simpler proof knowing that all Pythagorean triples can be parametrized as (k(m^2-n^2), 2mnk, k(m^2+n^2) where m, n, and k are positive integers such that m - n is odd and gcd(m, n) = 1. I won't write it here because I don't know how to use Latex and it will look horrible, but that idea just about solves the problem instantly." } { "Tag": [ "vector", "linear algebra", "linear algebra unsolved" ], "Problem": "Let T be a linear transformation defined by a 4x5 matrix. Can T be one to one? can T be onto? why?", "Solution_1": "$T$ is a linear transformation $K^5 \\to K^4$. If $T$ is one-to-one, then it's image would be a $5$-dimensional subspace of the $4$-dimensional vector space $K^4$. Dou you think this is possible? ;-)\r\n \r\nFor the case that $T$ is onto: There are many examples, e.g. $(x_1 , x_2 , x_3 , x_4 , x_5) \\mapsto (x_1 , x_2 , x_3 , x_4)$." } { "Tag": [ "floor function", "combinatorics unsolved", "combinatorics" ], "Problem": "How many zeros has $ 1000!$ ?", "Solution_1": "$ \\sum_{n=1}^{\\infty}\\left\\lfloor \\frac{1000}{5^{n}}\\right\\rfloor= \\left\\lfloor \\frac{1000}{5}\\right\\rfloor+\\left\\lfloor \\frac{1000}{5^{2}}\\right\\rfloor+\\left\\lfloor \\frac{1000}{5^{3}}\\right\\rfloor+\\left\\lfloor \\frac{1000}{5^{4}}\\right\\rfloor+0+0+\\cdots$\r\n$ =200+40+8+1=\\boxed{249}$", "Solution_2": "[quote=\"kunny\"]$ \\sum_{n=1}^{\\infty}\\left\\lfloor \\frac{1000}{5^{n}}\\right\\rfloor= \\left\\lfloor \\frac{1000}{5}\\right\\rfloor+\\left\\lfloor \\frac{1000}{5^{2}}\\right\\rfloor+\\left\\lfloor \\frac{1000}{5^{3}}\\right\\rfloor+\\left\\lfloor \\frac{1000}{5^{4}}\\right\\rfloor+0+0+\\cdots$\n$ =200+40+8+1=\\boxed{249}$[/quote]\r\nCan you explain please? :blush:", "Solution_3": "From $ 10=2*5$, we are to consider how many $ 5, 5^{2}, 5^{3}, 5^{4}, \\cdots$ can be included in $ 1000.$", "Solution_4": "[quote=\"kunny\"]From $ 10=2*5$, we are to consider how many $ 5, 5^{2}, 5^{3}, 5^{4}, \\cdots$ can be included in $ 1000.$[/quote]\r\nthx a lot kunny... :lol:", "Solution_5": "Don't mention it. :) \r\n\r\nkunny", "Solution_6": "[quote=\"santosguzella\"]How many zeros has $ 1000!$?[/quote]\r\nDo you mean zeros at the end of $ 1000!$? That's what Kunny has found. How would you find the total number of zeros of $ 1000!$?" } { "Tag": [ "blogs", "\\/closed" ], "Problem": "Where do you go to change your shoutbox colors? I've been looking all over blog options and I can't find it! Anyone know? Thanks a million bajillion times pi!", "Solution_1": "Odd numbered shouts (from the top):\r\nProfile --> Blog Options --> Blog customization --> Block background color\r\n\r\nEven numbered shouts (from the top):\r\nProfile --> Blog Options --> Blog Style --> Entry background color", "Solution_2": "I would recommend using CSS rather than using the color options provided as this will give you much greater control over the look of your blog. There are numerous tutorials for CSS on the web.\r\n\r\nIn general you may have something like:\r\n\r\n[code]#shoutsbox .row1 {\n background-color: red;\n color: black;\n font-weight: bold;\n}\n\n#shoutsbox .row2 {\n background-color: #2937ab;\n color: #283def;\n font-weight: normal;\n}\n\n#shoutsbox .shoutfooter {\n color: #cf4;\n font-family: serif;\n text-decoration: underline;\n font-size: 16px;\n}[/code]", "Solution_3": "Thank you so very much!\r\nI would've never figured that out!" } { "Tag": [ "geometry", "inradius", "circumcircle", "LaTeX", "inequalities" ], "Problem": "1.Let AD, BE, CF be angle-bisectors of triangle ABC. I is a point in triangle ABC such that ID is perpendicular to BC and IE is perpendicular to AC. Prove that IF is perpendicular to AC and (IA+IB+IC) is smaller or equal to 3R/2 with R is radius of (ABC).", "Solution_1": "What type of radius are you claiming that R IS of triangle ABC?\r\n\r\nAre YOU claiming it to be an inradius or a circumradius?\r\n\r\nPlease go back and EDIT that into your post, if you are \r\n\r\nallowed to.", "Solution_2": "by default a capital 'R' is used to denote the circumradius; this is a common convention (a lowercase 'r' is used for the inradius).", "Solution_3": "[color=darkblue]Oh! This problem $ R$ must be circumradius of $ \\triangle ABC$. Hi [i][b]Nguyen Ngoc Linh[/b][/i]! Your problem is nice. Now I'm busy so I can't try to solve it. I think you should post it at forum Geometry Usolved Problem, you will get help more quickly.\n\nAnd you should use latex in your post. It's very easy if you want.\nYou can see how to use Latex at this link: http://www.mathlinks.ro/viewtopic.php?t=165835\n\nThis is your problem use latex:\n\nLet $ AD, BE, CF$ be angle-bisectors of triangle $ ABC$. Let $ I$ is a point in $ \\triangle ABC$ such that $ ID \\perp BC$ and $ IE \\perp AC$. Prove that $ IF \\perp AC$ and $ IA\\plus{}IB\\plus{}IC \\le \\frac{3R}{2}$ with $ R$ is radius of circumcircle of $ \\triangle ABC$.[/color]", "Solution_4": "[quote=\"thanhnam2902\"][color=darkblue]\n\nLet $ AD, BE, CF$ be angle-bisectors of triangle $ ABC$. Let $ I$ be a point in $ \\triangle ABC$ such that $ ID \\perp BC$ and $ IE \\perp AC$. Prove that $ IF \\perp AC$ and $ IA \\plus{} IB \\plus{} IC \\le \\frac {3R}{2}$ with $ R$, the circumradius of the circumcircle of $ \\triangle ABC$.[/color][/quote]\r\nI made a few changes for grammar, and I allowed for redundancy\r\nin describing the \"R\" variable to emphasize what it is.\r\n\r\nIA, IB, and IC are the line segments, but if you want to be\r\ncorrect in your inequality, you must have bars above each\r\nof these three to indicate lengths of the line segments.\r\n\r\nPerhaps you know the code(s) in Latex to place them there." } { "Tag": [ "algorithm" ], "Problem": "is there a proof for the shoelace algorithm?", "Solution_1": "I just googled it, so [url=http://www.mathreference.com/la-det,shoe.html]here[/url].", "Solution_2": "can someone please explain?", "Solution_3": "http://en.wikipedia.org/wiki/Shoelace_formula should help." } { "Tag": [ "geometry", "rotation", "3D geometry", "tetrahedron" ], "Problem": "Compute the area of tetragon if its sides are : $ a \\equal{} 21.4 ,b \\equal{} 13.8 .c \\equal{} 22.25 , d \\equal{} 14.5$\r\nIs there any general formula to compute the area of iregular polygons ?", "Solution_1": "Shoelace would work, but it's annoying to set up when you have decimals...\r\n\r\nHere's the link to the theorem:\r\nhttp://www.artofproblemsolving.com/Wiki/index.php/Shoelace_Theorem", "Solution_2": "Actually for a tetragon/quadrilateral, I don't think there is a definite answer to every single set of side lengths. Take, for example, one with side lengths of $ 6$, $ 6$, $ 4$, and $ 4$. This is obviously a kite, but is it the convex version (the normal kite) or the concave version (with the cave-in, forming an arrowhead)? I think that in order to find the area of a quadrilateral, you need more than four sides...either information about the concavity of the quadrilateral or the length of the variable diagonal.", "Solution_3": "Actually, the shape is determined by the sides, since three sides determine a triangle (i.e. each face), and every orientation of the sides can be rotated to give the same figure.\r\n\r\nYou can find the volume of a general tetrahedron with the [url=http://en.wikipedia.org/wiki/Scalar_triple_product]triple scalar product[/url].", "Solution_4": "Um...math154...enndb0x said tetraGON, meaning quadrilateral, not tetrahedron", "Solution_5": "Hmm... I think Brahmagupta's Formula is applicable here since the length of the sides were given.[hide=\"That is...\"]\n$ \\sqrt{(s\\minus{}a)(s\\minus{}b)(s\\minus{}c)(s\\minus{}d)}$\nWhere: $ s\\equal{} \\frac{a\\plus{}b\\plus{}c\\plus{}d}{2}$\n\nSo, By Solving I've got $ s\\equal{}35.975$\nand Area of the tetragon is $ 304.64\\approx305$\n\n[/hide]", "Solution_6": "Brahmagupta's formula only works for cyclic quadrilaterals. \r\nSo you would need to prove that it is a cyclic quadrilateral.\r\nIf it is not, you need to find angle measures for the formula to work.\r\nThe formula is \r\n\r\n$ \\sqrt {(s\\minus{}a)(s\\minus{}b)(s\\minus{}c)(s\\minus{}d)\\minus{}abcd cos^2 }\\theta$.\r\nWhere $ \\theta$ is half the sum of opposite angles.", "Solution_7": "sorry but I don't have the angles ...anyone any concret idea of solving this problem , or in general ? :maybe:", "Solution_8": "[quote=\"r15s11z55y89w21\"]Actually for a tetragon/quadrilateral, I don't think there is a definite answer to every single set of side lengths. ... \nI think that in order to find the area of a quadrilateral, you need more than four sides...either information about the concavity of the quadrilateral or the length of the variable diagonal.[/quote] Quoted for truth. (Almost any other piece of information would suffice: the length of a diagonal or the measure of a single angle, for example. It should be obvious that the edge-lengths don't determine the quadrilateral or its area: just think about the family of rhobuses, say.)" } { "Tag": [ "function", "inequalities", "integration", "calculus", "real analysis", "real analysis unsolved" ], "Problem": "Let $ f$ and $ g$ be increasing functions on $ [a,b]$. Prove that\r\n\\[ \\int_a^b f(x)g(a\\plus{}b\\minus{}x) dx \\le \\frac 1{b\\minus{}a} \\left( \\int_a^b f(x) dx \\right) \\left( \\int_a^b g(x) dx \\right) \\le \\int_a^b f(x)g(x) dx.\\]", "Solution_1": "This is an application of Chebyshev inequality. The last $ \\leqq$ is directly followed. The first $ \\leqq$ is obtained by a substitution.", "Solution_2": "I had not previously seen a proof of chebyshev's inequalities for integrals, and while you can probably get it from the discrete case using riemann sums, I labored for awhile to produce the following argument:\r\n\r\nIt suffices to assume that $ f(x)\\geq 0$, since if the inequality holds for $ f(x) \\plus{} c$, then it holds for $ f(x)$ as well.\r\n\r\nPut\r\n\\[ \\widetilde{g}(x) \\equal{} \\frac {1}{x \\minus{} a}\\int^x_a g(x)dx\r\n\\]\r\nand define $ \\widetilde{f}$ similarly.\r\n\r\nNow integrate $ (b \\minus{} a)[f(x)g(x) \\minus{} f(x)\\widetilde{g}(x)]$ by parts over $ [a,b]$ to get\r\n\\[ (b \\minus{} a)\\int^b_a f(x)g(x) \\minus{} \\int^b_a f(x)dx\\int^b_a g(x)dx \\equal{} (*)\r\n\\]\r\nwhere\r\n\\[ (*) \\equal{} \\minus{} (b \\minus{} a)\\int^b_a \\widetilde{f}(x)(g(x) \\minus{} \\widetilde{g}(x))dx\r\n\\]\r\nSince the integrand of $ (*)$ is clearly positive ($ \\widetilde{f}$ is positive by our assumptions on $ f$ and the other term is positive since $ g$ is increasing), the result follows instantly. A straightforward modification of the above shows that the opposite holds if $ g$ is decreasing, whence the inequality on the LHS follows by taking the decreasing function $ h(x) \\equal{} g(a \\plus{} b \\minus{} x)$.\r\n\r\nIs there a quicker proof?" } { "Tag": [], "Problem": "Let d and z be integers such that d2 divides z2. Prove that d divides z.", "Solution_1": "Write $kd^2=z^2$ and prove that k is a square.\r\n\r\nMisha", "Solution_2": "Look at the prime factors of d^2 and z^2. What do you know about them If $d^2|z^2$, then what do you know about d and z?", "Solution_3": "[quote=\"Misha123\"]Write $kd^2=z^2$ and prove that k is a square.\n\nMisha[/quote]\r\n\r\nkd^2 = z^2\r\nd:sqrt:k = z ?\r\nahhh I get it..\r\n\r\n :sqrt:k = z/d\r\n\r\nso :sqrt: k is rational and so k must be a square!", "Solution_4": "[quote=\"Misha123\"]Write $kd^2=z^2$ and prove that k is a square.\n\nMisha[/quote]\r\n\r\nkd^2 = z^2\r\nd:sqrt:k = z ?\r\nahhh I get it..\r\n\r\n :sqrt:k = z/d\r\n\r\nso :sqrt: k is rational and so k must be a square!", "Solution_5": "Exactly!!! :idea: \r\n\r\nMisha", "Solution_6": "[quote=\"Misha123\"]Exactly!!! :idea: \n\nMisha[/quote]\r\n\r\nThis can be extended to prove that if d^n divides z^n, d divides z, where n is any natural" } { "Tag": [ "MATHCOUNTS", "percent", "geometry", "summer program", "MathPath", "AMC", "USA(J)MO" ], "Problem": "I was wondering who made nationals after the state comp now that March is flying by. Only take this poll if you've already done your state comp or didn't make state. Thanks!", "Solution_1": "i think theres already another poll", "Solution_2": "I know, but a lot of people, including me, didn't wait until after state until they voted, hence the title", "Solution_3": "5th place -- read my rant in the other thread.", "Solution_4": "I know, I heard. I thoroughly feel your pain. The only thing worse than getting last is getting 5th when it comes to mathcounts.", "Solution_5": "don't get down on yourself. You should be proud to get 5th place.", "Solution_6": "yeah. and the ranting was MUCH worse. there were at least 50 nat mc polls like \"what place would i have come in you state\" and \"would i have won in your state\" and \"do you think its unfair that i didn't make nats\" and \"what was the top score in your state\" and on and on and on and on.", "Solution_7": "[quote=\"biffanddoc\"]yeah. and the ranting was MUCH worse. there were at least 50 nat mc polls like \"what place would i have come in you state\" and \"would i have won in your state\" and \"do you think its unfair that i didn't make nats\" and \"what was the top score in your state\" and on and on and on and on.[/quote]\r\n\r\n\r\nEasy for you to say. You live in one of the easiest states, from what I hear :roll: .", "Solution_8": "Well he's really smart so he deserves to go to nationals.", "Solution_9": "This is true. But there are also MANY other smart people (possibly smarter) that get weeded out in other states, that may have had bad luck, not been feeling well, or even had an off day.\r\n\r\nBesides, who are you to decide who \"deserves\" to go?", "Solution_10": "tenth 4 me...", "Solution_11": "[quote=\"sferics\"][quote=\"biffanddoc\"]yeah. and the ranting was MUCH worse. there were at least 50 nat mc polls like \"what place would i have come in you state\" and \"would i have won in your state\" and \"do you think its unfair that i didn't make nats\" and \"what was the top score in your state\" and on and on and on and on.[/quote]\n\n\nEasy for you to say. You live in one of the easiest states, from what I hear :roll: .[/quote]\r\neasy for me to talk about nat mc's ranting??? \r\nand yeah de is really easy", "Solution_12": "\"How are you to say\"\r\n\r\nWell it's pretty obvious -- I mean, just take a look at stats. Nat_mc got perfect AMC 10 last year and beat a lot of other people. \r\n\r\nAnd I'm this year's Indiana #5. Just wow, just wow. I had state and then missed #7 and #8. I CHANGED MY ANSWER FROM RIGHT TO WRONG ON 8 IN THE LAST SECOND. Literally, a SECOND. On the sprint, the only person I lost to was Nathan.\r\n\r\nI'm pretty sure that's the worst.", "Solution_13": "In terms of a scripted loss -- I don't know the details of his. He told me he missed #7 and #8. \r\n\r\nBut coming within A SECOND of getting 4th or 3rd. ONE SECOND. and then changing your answer. WOW JUST WOW.\r\n\r\nI'm sure that's worse if you take grade out of the equation.", "Solution_14": "Well, you did a good job. I'm over it now being in 9th grade (i just had to say it one more time), but you can still have a lot of fun if you go and watch it. Also, you were easily the top 7th grader, so you're almost a number 1 lock in the state, and maybe a top 12 finish in nats next year. Don't get down on yourself like I did, it doesn't help.", "Solution_15": "I will. I just got back like ten minutes ago.", "Solution_16": "hahaha :rotfl:", "Solution_17": "[quote=\"janieluvsmusic\"]also, I think this year was the 1st year Kelsey ever did Mathcounts. It was really surprising when cowpi beat her in countdown round, just because she seemed so good at chapter countdown and she was up 2-0 before cowpi got 3 in a row to beat her.[/quote]\r\n\r\nReally? Well that's too bad, because I think she can only compete one more time in Mathcounts. She's incredibly good though.", "Solution_18": "[quote=\"camathkid\"]hahaha :rotfl:[/quote] What's so funny? :?", "Solution_19": "[quote] Really? Well that's too bad, because I think she can only compete one more time in Mathcounts. She's incredibly good though.[/quote]\n\nyeah, she's usually first in math competitions. but I don't think she was in the top 10 at blaine this year and I know she was there. Last year she was 1st too. The only reason Kelsey knows about mathcounts is because she met Al Lippert at this math camp, and he formed a homeschool team this year.[/quote]", "Solution_20": "[quote=\"janieluvsmusic\"]she met Al Lippert at this math camp[/quote]\r\n\r\nWhat--MathPath?\r\n\r\nhttp://www.mathpath.org", "Solution_21": "yeah, their both from WA too.", "Solution_22": "[quote=\"ehehheehee\"]#1 I am in 8th grade\n#2 I was 6th in chapter because I misread 3 questions\n#3 I misread one question in target in state and missed another because I am bad at entering things into my calculator. Four points could of given me so much more\n\nMathcounts is not the only competition in the world[/quote]\r\n\r\nI did the same as you, I xxxx in chapter, xxx in state.\r\n\r\n[Mod edit: Declare what year you are talking about, or it is assumed you are talking about this year. Which is illegal right now. ]", "Solution_23": "[quote=\"janieluvsmusic\"]also, I think this year was the 1st year Kelsey ever did Mathcounts. It was really surprising when cowpi beat her in countdown round, just because she seemed so good at chapter countdown and she was up 2-0 before cowpi got 3 in a row to beat her.[/quote]\r\nIt is the first year Kelsey was with Seattle Chapter, but she had two years before this (2004 and 2005) in Lake Washington Chapter (e.g. In 2004 she was 30th place in Lake Washington on Official team)", "Solution_24": "[quote=\"janieluvsmusic\"]\r\nyeah, she's usually first in math competitions. but I don't think she was in the top 10 at blaine this year and I know she was there. Last year she was 1st too. The only reason Kelsey knows about mathcounts is because she met Al Lippert at this math camp, and he formed a homeschool team this year.[/quote]\r\nSee my other posting - Kelsey started MATHCOUNTS in 2004 with Lake Washington Chapter.\r\nThe big surprise this year at Washington is that Kerry Xing did not make to National. Kerry is a very strong mathlete both in written and in countdown (I watched Kerry edging out Samath in Countdown - both are incredible mathletes and made to this year's USAMO, the only two 8th graders made to that list from WA, I believe). It was a shock to everyone since Kerry was in the National last year.", "Solution_25": "yeah, it was also really weird that kerry was 10th at blaine. It was weird because it meant I beat him.", "Solution_26": "Indeed, Kelsey is certainly not a ninth grader. Fortunately for a few previous posters, she'll be in Nevada next year, thus not competing against you unless you make nationals. It's a bit surprising that she lost in the State Countdown, but Washington did have its best team ever.", "Solution_27": "Oh well, I made 19th chapters and placed 25th states.\r\n\r\nfunny quote: [quote]I\"m sorry. Crap is not the right answer.[/quote]", "Solution_28": "[quote=\"Kamior\"]Indeed, Kelsey is certainly not a ninth grader. Fortunately for a few previous posters, she'll be in Nevada next year, thus not competing against you unless you make nationals. It's a bit surprising that she lost in the State Countdown, but Washington did have its best team ever.[/quote]\r\n\r\nHow do you know she'll be in Nevada?", "Solution_29": "probably knows her personally or asked through AIM/MSN?" } { "Tag": [ "geometry", "number theory", "prime factorization", "area of a triangle", "Heron\\u0027s formula", "Pythagorean Theorem" ], "Problem": "(1) Find the number of inches in the length of the shortest altitude for a triangle whose side measure 5 inches, 7 inches and 8 inches. express your answer in simplest radical form.\r\n\r\n(2) The product of positive integers X, Y, and Z equals 2004. What is the minimum possible value for the sum of X, Y, and Z?\r\n\r\n(3) Using an array of 3x3 dots, how many different triangles can you make by picking and three of the nine dots at random and creating the vertices of a triangle?\r\n\r\n(4) How many liters of water must be added to 8 liters of a 30% acid solution to make a 12% acid solution.\r\n\r\nThankx!![/hide]", "Solution_1": "[hide=\"4\"]\n\nThe mixture is originally $ \\frac{2.4}{8}$ acid.\nWe want a 12% acid solution, so:\n$ \\frac{2.4}{8\\plus{}x}\\equal{}\\frac{3}{25}$\nCross multiplying, we have:\n$ 60\\equal{}24\\plus{}3x \\Rightarrow 36\\equal{}3x \\Rightarrow 12\\equal{}x$\nSo $ \\boxed{12 liters}$\n\n[/hide]", "Solution_2": "[hide=\"1\"]\nThe semiperimeter of this triangle is $ \\frac {5 \\plus{} 7 \\plus{} 8}2 \\equal{} 10$.\n\nBy Heron's, the area is $ \\sqrt {10(10 \\minus{} 5)(10 \\minus{} 7)(10 \\minus{} 8)} \\equal{} \\sqrt {2\\cdot5\\cdot5\\cdot3\\cdot2} \\equal{} 10\\sqrt3$.\n\nSince $ A \\equal{} \\frac12bh$, $ h \\equal{} \\frac {2A}b \\equal{} \\frac {20\\sqrt3}b$, and we quickly see that the shortest altitude occurs when $ b \\equal{} 8$, and\n\n$ h \\equal{} \\frac {20\\sqrt3}8 \\equal{} \\boxed{\\frac {5\\sqrt3}2}$.\n[/hide]\n[hide=\"2\"]\n$ XYZ \\equal{} 2004 \\equal{} 2^2\\cdot3\\cdot167$, and since the minimum value of $ X \\plus{} Y \\plus{} Z$ occurs when $ X,Y,Z$ are as close as possible and $ \\sqrt [3]{2004}\\approx12.6$, we see that\n\n$ 3\\cdot4\\cdot167$ gives the best result, $ 3 \\plus{} 4 \\plus{} 167 \\equal{} \\boxed{174}$.\n[/hide]\n[hide=\"3\"]\nFirst, we note that any three dots will form a triangle, so our preliminary count is $ \\dbinom{3\\cdot3}3 \\equal{} 84$ triangles.\n\nHowever, we are counting degenerate triangles, which can only be formed from all lines with three points in the array.\n\nThere are three rows, three columns, and two diagonals that form these degenerate triangles, so our final answer is\n\n$ 84 \\minus{} 3 \\minus{} 3 \\minus{} 2 \\equal{} \\boxed{76}$.\n[/hide]\n[hide=\"4\"]\nCurrently, the solution is $ 0.3\\cdot8 \\equal{} 2.4$ liters acid.\n\nSince we are only adding water, the amount of acid will stay the same, and we want\n\n$ \\frac {2.4}{8 \\plus{} x} \\equal{} 12\\% \\equal{} \\frac3{25}\\implies60 \\equal{} 24 \\plus{} 3x\\implies x \\equal{} \\boxed{12}$ liters.\n[/hide]", "Solution_3": "[hide=\"2\"]\n\nTaking the prime factorization of 2004, we have:\n\n$ 2004\\equal{}2 \\times 2 \\times 3 \\times 167$\n\nWe want to minimize the sum of three factors, so take $ 167, 2 \\times 2 \\equal{}4,$ and $ 3$.\n\n$ 167\\plus{}3\\plus{}4\\equal{}\\boxed{174}$\n\n[/hide]", "Solution_4": "I don't understand #1 still, could you explain it by either (1) easier way or (2) explain each step more carefully?", "Solution_5": "[url=http://www.artofproblemsolving.com/Wiki/index.php/Heron%27s_formula]Heron's Formula.[/url] (proof is not MC level, but the formula, at least, is good to know.)\r\n\r\nAs for the second part, i.e. minimizing $ \\frac {10\\sqrt3}b$, notice that the minimum of this occurs when $ b$ is at its maximum (which in this case, would be $ b \\equal{} 8$).", "Solution_6": "First we find the area using Heron's Formula. Heron's Formula states that the area of $ \\triangle ABC\\equal{}\\sqrt{s(s\\minus{}a)(s\\minus{}b)(s\\minus{}c)}$, where $ s\\equal{}\\frac{a\\plus{}b\\plus{}c}{2}$ and where $ a$, $ b$, and $ c$ are the side lengths of the triangle. Then we have $ [ABC]\\equal{}\\sqrt{10(10\\minus{}5)(10\\minus{}7)(10\\minus{}8)}\\equal{}\\sqrt{10 \\times 5 \\times 3 \\times 2}\\equal{}\\sqrt{300}\\equal{}10\\sqrt{3}$. \r\n\r\nRemember our formula $ A\\equal{}\\frac{1}{2}bh$. Then we have $ 10\\sqrt{3}\\equal{}\\frac{1}{2}bh$. Because we want $ h$ to be minimized, we maximize $ b$. The longest side is $ 8$, so we let that be $ b$. Then we have $ 10\\sqrt{3}\\equal{}4h \\implies \\boxed{\\frac{5\\sqrt{3}}{2}\\equal{}h}$.\r\n\r\nmath154, you forgot to multiply $ A$ by $ 2$.", "Solution_7": "[quote=\"math154\"][url=http://www.artofproblemsolving.com/Wiki/index.php/Heron%27s_formula]Heron's Formula.[/url] (proof is not MC level, but the formula, at least, is good to know.)\n\nAs for the second part, i.e. minimizing $ \\frac {10\\sqrt3}b$, notice that the minimum of this occurs when $ b$ is at its maximum (which in this case, would be $ b \\equal{} 8$).[/quote]\r\n\r\nAh...I got the Herons formula since I know it, but I had no idea where the 8 came from. It came from the original side lengths, right?", "Solution_8": "[quote=\"AIME15\"]math154, you forgot to multiply $ A$ by $ 2$.[/quote]\n\nWhat are you talking about? I even wrote $ \\frac{2A}b$. :P \n\n[quote=\"007math\"]Ah...I got the Herons formula since I know it, but I had no idea where the 8 came from. It came from the original side lengths, right?[/quote]\r\n\r\nYes. :)", "Solution_9": "You edited...", "Solution_10": "[hide=\"1 without herons\"]\nLabel the triangle ABC, such that AB=7, BC=8, and CA=5.\nClearly, the smallest altitude is the one from A to BC, because BC is the longest side. Let the point that the altitude meets BC be called D.\nNow, let the length CD be x, so that the length BD is 8-x.\nFrom the pythagorean theorem on ACD, we know that $ x^2\\plus{}AD^2\\equal{}25$, or $ AD^2\\equal{}25\\minus{}x^2$\nFrom the pythagorean theorem on ABD, we know that $ (8\\minus{}x)^2\\plus{}AD^2\\equal{}49$, or $ AD^2\\equal{}49\\minus{}(8\\minus{}x)^2$\nSetting these two equal, we have $ 25\\minus{}x^2\\equal{}49\\minus{}(8\\minus{}x)^2 \\iff 25\\minus{}x^2\\equal{}49\\minus{}(64\\minus{}16x\\plus{}x^2) \\iff 25\\minus{}x^2\\equal{}\\minus{}15\\plus{}16x\\minus{}x^2 \\iff 40\\equal{}16x \\iff x\\equal{}\\frac{5}{2}$.\nTherefore, $ AD^2\\equal{}25\\minus{}(\\frac{5}{2})^2\\equal{}25\\minus{}\\frac{25}{4}\\equal{}\\frac{75}{4}$, so $ AD\\equal{}\\sqrt{\\frac{75}{4}}\\equal{}\\frac{\\sqrt{75}}{2}\\equal{}\\boxed{\\frac{5\\sqrt{3}}{2}}$\n[/hide]" } { "Tag": [ "algebra", "function", "domain" ], "Problem": "Factorize:\r\n\r\n$ x^4 \\plus{} 7x \\plus{} 4y^2 \\minus{} 8y \\plus{} 4 \\equal{} 0$\r\n\r\n\r\nSecond problem:\r\n\r\nFind the domain and range of the following function:\r\n\r\n$ f(x) \\equal{} x^2 \\plus{} 2x \\plus{} 1$", "Solution_1": "For the second problem, the domain is $ \\mathbb{R}$ since the function works for any $ x\\in\\mathbb{R}$. The minimum value of the function occurs at $ x\\equal{}\\minus{}1$, where $ f(\\minus{}1)\\equal{}0$. So the range is $ [0,\\infty)$." } { "Tag": [ "linear algebra", "matrix", "combinatorics proposed", "combinatorics" ], "Problem": "Today is Barbara's birthday, and Alberto wants to give her a gift playing the following game. The numbers 0,1,2,...,1024 are written on a blackboard. First Barbara erases $2^{9}$ numbers, then Alberto erases $2^{8}$ numbers, then Barbara $2^{7}$ and so on, until there are only two numbers a,b left. Now Barbara earns $|a-b|$ euro.\r\nFind the maximum number of euro that Barbara can always win, independently of Alberto's strategy.", "Solution_1": "[quote=\"edriv\"][b]Problem 4[/b]\nToday is Barbara's birthday, and Alberto wants to give her a gift playing the following game. The numbers 0,1,2,...,1024 are written on a blackboard. First Barbara erases $2^{9}$ numbers, then Alberto erases $2^{8}$ numbers, then Barbara $2^{7}$ and so on, until there are only two numbers a,b left. Now Barbara earns $|a-b|$ euro.\nFind the maximum number of euro that Barbara can always win, independently of Alberto's strategy.[/quote]\r\n\r\nI assume numbers are $1,...,1024$ and not $0,...,1024$ as stated.\r\n\r\nI have a simple stategy for Barbara which gives her at least 48 at the end, but I'm not sure it is the best :\r\nBarbara just have to pick one number each two numbers for each of her 4 first turns and she takes the two middle numbers at her last turn. With this method :\r\nDifference between two consecutive numbers is exactly 2 after the first Barbara's turn (it remains 512 numbers)\r\nDifference between two consecutive numbers is at least 4 after the second Barbara's turn (it remains 128 numbers)\r\nDifference between two consecutive numbers is at least 8 after the third Barbara's turn (it remains 32 numbers)\r\nDifference between two consecutive numbers is at least 16 after the fourth Barbara's turn (it remains 8 numbers)\r\nDifference between two consecutive numbers is at least 16 before the last Barbara's turn (it remains 4 numbers) and so, by picking the two middle numbers, the remaining difference is at least 48.\r\n\r\nIt's possible on the contrary to show that it exist strategies for Alberto which prevents Barbara going over 64 (or even 56) : take at each turn the numbers which minimize the \"max - min\" of remaining numbers.\r\n\r\nSo I can't prove 48 is the best sure result.\r\n\r\n\r\n-- \r\nPatrick", "Solution_2": "i think that 32 its the best results.", "Solution_3": "[quote=\"mathrix\"]i think that 32 its the best results.[/quote]\r\n\r\nI'm sorry but the best result is between 48 and 64.\r\n\r\nIn my previous post, I gave a strategy for Barbara to have at least 48 and I said it exist a strategy for Alberto to leave at most 64 for Barbara (remove numbers in such a way max(reminder)-min(reminder) be minimized).\r\n\r\n-- \r\nPatrick", "Solution_4": "I haven't read your proof but mathrix is right, the best is 32.", "Solution_5": "[quote=\"Sepp\"]I haven't read your proof but mathrix is right, the best is 32.[/quote]\r\n\r\nOK. I thought that there was a mistake i the problem and that there was 1024 numbres (and not 1025).\r\nWith 1025, there is a round more and the strategy I gave lead to 32 min.\r\n\r\nThe strategy o Alberto divides max - min by two, and since Alberto now have 5 rounds, il leads to 32 max.\r\n\r\nAnd he result is 32.\r\n\r\nSorry mathrix, you were right, indeed. :blush: \r\n\r\n\r\n-- \r\npatrick" } { "Tag": [ "algebra", "partial fractions" ], "Problem": "Caculate the sum \\[\\frac{1\\cdot 3}{3\\cdot 5}+\\frac{2\\cdot 4}{5\\cdot 7}+...+\\frac{(n-1)(n+1)}{(2n-1)(2n+1)}+...+\\frac{1002\\cdot 1004}{2005\\cdot 2007}.\\]", "Solution_1": "[hide=\"notice that\"]$\\frac{n^{2}-1}{4n^{2}-1}=\\frac{1}{4}(1-\\frac{3}{(2n-1)(2n+1)})$[/hide]\r\nwe denote \r\n$A=\\frac{1.3}{3.5}+...+\\frac{1002.1004}{2005.2007}$\r\nconsider $4A=1002-\\frac{3}{3.5}+\\frac{3}{5.7}+...+\\frac{3}{2005.2007}$\r\nlet $B = \\frac{3}{3.5}+\\frac{3}{5.7}+\\frac{3}{2005.2007}$\r\nso that \r\n$\\frac{2}{3}B=\\frac{2}{3.5}+\\frac{2}{5.7}+...+\\frac{2}{2005.2007}$\r\n$=\\frac{1}{3}-\\frac{1}{5}+\\frac{1}{5}-\\frac{1}{7}+....+\\frac{1}{2005}-\\frac{1}{2007}$\r\n$=\\frac{668}{2007}$\r\nwe obtain $B=\\frac{334}{669}$ and A=....", "Solution_2": "[hide=\"Solution\"]\nFrom [b]HTA[/b]'s hint we have that \\[\\frac{n^{2}-1}{4n^{2}-1}= \\frac{1}{4}\\left( 1-\\frac{3}{(2n+1)(2n-1)}\\right) \\] So our summation becomes: \\[S = \\frac{1}{4}\\sum_{n=2}^{1003}\\left( 1-\\frac{3}{(2n+1)(2n-1)}\\right) = \\frac{501}{2}-3 \\sum_{n=2}^{1003}\\left( \\frac{1}{ (2n+1)(2n-1) }\\right) \\] Now, using the method of partial fractions we find that \\[\\frac{1}{ (2n+1) (2n-1) }= \\frac{1}{2}\\left( \\frac{1}{2n-1}-\\frac{1}{2n+1}\\right) \\] and thus our summation is equal to: \\[\\frac{501}{2}-\\frac{3}{2}\\sum_{n=2}^{1003}\\left( \\frac{1}{2n-1}-\\frac{1}{2n-1 }\\right) = \\frac{501}{2}-\\frac{3}{2}\\left( \\frac{1}{3}-\\frac{1}{2007}\\right) = \\boxed{ \\frac{334501}{1338}}\\] [/hide]", "Solution_3": "i don't see the need for multiple steps...this is just one partial fractions decomposition\r\n\r\n$\\frac{(n-1)(n+1)}{(2n-1)(2n+1)}=\\frac{3}{8}*( \\frac{1}{2n+1}-\\frac{1}{2n-1})+\\frac{1}{4}$" } { "Tag": [ "number theory", "algebra unsolved", "algebra" ], "Problem": "Find the remainder when $ 3^{5^{100}}$ is divided by 7", "Solution_1": "$ 5^{2} \\equiv 1 \\mod 6$, hence $ 5^{100} \\equiv 1 \\mod 6.$\r\n\r\nOn the other hand, Fermat's little theorem allows us to ascertain that \r\n\r\n$ 3^{6} \\equiv 1 \\mod 7.$\r\n\r\nTherefore, there exists $ k \\in \\mathbb{N}$ such that\r\n\r\n$ 3^{5^{100}} \\equal{} 3^{6k \\plus{} 1} \\equal{} (3^{6})^{k}(3) \\equiv 3 \\mod 7$\r\n\r\nand we're done: the remainder is [b]3[/b].", "Solution_2": "Thanks.\r\nI would want an answer without using congruence modulo concept as the problem is intended for students who haven't studied congruence modulo.", "Solution_3": "[quote=\"pankajsinha\"]Thanks.\nI would want an answer without using congruence modulo concept as the problem is intended for students who haven't studied congruence modulo.[/quote]\r\n\r\nYou can use coquitao's ideas without the congruence notation:\r\n\r\n1) Notice that $ 3^6\\equal{}729\\equal{}7\\cdot 104\\plus{}1$\r\n2) Notice that $ (7a\\plus{}1)(7b\\plus{}1)\\equal{}7(ab\\plus{}a\\plus{}b)\\plus{}1\\equal{}7j\\plus{}1$ then $ (7a\\plus{}1)^n\\equal{}7k\\plus{}1$ for all natural $ n$ (by induction). \r\n2*) Moreover using that approach then $ (ma\\plus{}1)^n\\equal{}mk\\plus{}1$\r\n3) Then $ (3^6)^l\\equal{}(7\\cdot 104\\plus{}1)^l\\equal{}7k\\plus{}1$\r\n\r\n4) Similarly $ 5^2\\equal{}25\\equal{}6\\cdot4\\plus{}1$\r\n5) Using 2*) then $ 5^{100}\\equal{}(5^2)^{50}\\equal{}(6\\cdot4\\plus{}1)^{50}\\equal{}6l\\plus{}1$\r\n6) All together now... $ 3^{{5}^{100}}\\equal{}3^{6l\\plus{}1}\\equal{}(3^6)^l \\cdot 3\\equal{}(7k\\plus{}1) \\cdot 3\\equal{} 7 (3k)\\plus{}3$" } { "Tag": [ "function", "algebra", "functional equation", "real analysis", "real analysis unsolved" ], "Problem": "Determine whether there exists a nonconstant power series $ y \\equal{} \\sum a_n \\frac{x^n}{n!}$ with $ y(0) \\equal{} 1$ that satisfies, formally, the functional equation\r\n\r\n$ y \\equal{} e^{y\\minus{}1}$.", "Solution_1": "[quote=\"t0rajir0u\"]Determine whether there exists a nonconstant power series $ y \\equal{} \\sum a_n \\frac {x^n}{n!}$ with $ y(0) \\equal{} 1$ that satisfies, formally, the functional equation\n\n$ y \\equal{} e^{y \\minus{} 1}$.[/quote]\r\nI assume you are interested in a formal power series. If such a power series existed with finite radius of convergence, then within this disk we'd have $ y' \\equal{} y' e^{y\\minus{}1}$ and hence, since $ y \\ne const$, $ 1 \\equal{} e^{y\\minus{}1}$ a.e., implying that $ y$ is constant after all.\r\n\r\nSuppose now a formal power series with $ y\\equal{}e^{y\\minus{}1}, \\, y(0) \\equal{} 1$ exists and $ a_1 \\equal{} a_2 \\equal{} \\dots \\equal{} a_{n\\minus{}1} \\equal{} 0 \\ne a_n.$. The $ 2n$th term of $ y$ is\r\n\\[ a_{2n} \\frac{x^{2n}}{(2n)!} \\]\r\nOn the other hand, the term of order $ 2n$ of $ e^{y\\minus{}1}$ is \r\n\\[ a_{2n} \\frac{x^{2n}}{(2n)!} \\plus{} \\frac12 \\left( a_n \\frac{x^n}{n!} \\right)^2 \\, .\r\n\\]\r\nThis implies $ a_n \\equal{} 0$, a contradiction. Hence no such formal power series exists.", "Solution_2": "Doesn't it follow from the fact that $ e^{x}\\equal{}1\\plus{}x \\implies x\\equal{}0$ that $ y \\equiv 1$ is the only solution?", "Solution_3": "It is slightly harder than that, but not by much: a formal power series can have radius of convergence $ 0$.", "Solution_4": "To emphasize that point, here's a functional equation that does have a (unique) formal power series solution: $ f\\left(\\frac{z}{1\\minus{}z}\\right)\\minus{}f(z)\\equal{}z$, $ f(0)\\equal{}1$. The function defined this way is meromorphic on $ \\mathbb{C}$ except at the origin, where it has an essential singularity and infinitely many nearby poles." } { "Tag": [ "function" ], "Problem": "Find the minimum value of the function $ f(x)\\equal{}3(x^2\\minus{}1)\\plus{}2$.", "Solution_1": "a graph of a function reaches maximum/minimum when $ x \\equal{} \\minus{} \\frac {b}{2a}$\r\n\r\n$ f(x) \\equal{} 3x^2 \\minus{} 1$\r\n\r\nEDIT: lol, $ x\\equal{}\\minus{}\\frac{b}{2a}$\r\n\r\n$ x\\equal{}0$\r\n\r\nplug it in, and the minimum of $ f(x)\\equal{}\\minus{}1$\r\n\r\nanswer : -1", "Solution_2": "multiplying the whole thing out and simplifying, we get $ f(x) \\equal{} 3x^2 \\minus{} 1$. If you have a sense of maxima and minima, you can pretty easily understand that the minimum of this graph will be at its vertex. Using the fact that the x-value of the vertex will always be $ x \\equal{} \\frac { \\minus{} b}{2a}$, we can determine that the x value of the vertex is 0. Plugging this back into our original function we see that our answer is $ \\boxed{ \\minus{} 1}$\r\n\r\nEDIT: BEATEN TO IT" } { "Tag": [ "Diophantine Equations", "pen" ], "Problem": "Prove that the equation $a^2 +b^2 =c^2 +3$ has infinitely many integer solutions $(a, b, c)$.", "Solution_1": "If $ a\\plus{}1\\equal{}n^{2}$, then $ (2n)^{2}\\plus{}(2a)^{2}\\equal{}(2a\\plus{}1)^{2}\\plus{}3$, since there are infinitely many $ a$ for which $ a\\plus{}1$ is a perfect square, this solves the problem.", "Solution_2": "$ a\\equal{}2k, b\\equal{}2k^2\\minus{}2,c\\equal{}2k^2\\minus{}1$", "Solution_3": "[quote=\"Johan Gunardi\"]$ a \\equal{} 2k, b \\equal{} 2k^2 \\minus{} 2,c \\equal{} 2k^2 \\minus{} 1$[/quote]I think that's not correct: look at the coefficient of $ k$...", "Solution_4": "$ (2k)^2\\plus{}(2k^2\\minus{}2)^2 \\minus{} (2k^2\\minus{}1)^2 \\equal{} 4k^2\\plus{}4k^4\\minus{}8k^2\\plus{}4\\minus{}4k^4\\plus{}4k^2\\plus{}1 \\equal{} 3$, so it works.", "Solution_5": "I'm sorry, must have missed the squares. Never mind.", "Solution_6": "Can we find all solutions ?" } { "Tag": [ "search", "combinatorics unsolved", "combinatorics" ], "Problem": "Consider the set of all discs in the plane with centres at lattice points and radius = $\\frac{1}{1000}$ each. \r\n\r\n(a) Prove that there exists an equilateral triangle whose vertices lie in distinct discs considered above.\r\n(b) Prove that the side lengths of any such equilateral triangle is greater than $100$", "Solution_1": "IMO 2003 Shortlist, C5 if I am not mistaken. Search button..." } { "Tag": [ "algebra", "polynomial", "algebra solved" ], "Problem": "$a_0, a_1,a_2,...a_n $ real numbers such that $a_0>=a_1>=a_2>=...>=a_n>0 $\r\nand r is a root of the polynomial \r\n$ P(z)=a_0z^n+a_1z^(n-1)+..+a_n $\r\nprove that $ r<=1 $", "Solution_1": "$P(z)>0$ if $z>0$ so $r\\leq0<1$. Or do you meant $r\\in C$ and $|r|\\leq 1$?\r\n\r\nMisha", "Solution_2": "[quote=\"Misha123\"]$P(z)>0$ if $z>0$ so $r\\leq0<1$. Or do you meant $r\\in C$ and $|r|\\leq 1$?\n\nMisha[/quote]\r\n$a_i, r $ and$ z $ are all real numbers" } { "Tag": [ "linear algebra", "matrix", "vector" ], "Problem": "Let $A$ and $B$ be matrices such that $A^{n}=B^{n}=0$ and $AB=BA$. Prove that $(A+B)^{n}=0$", "Solution_1": "Notice that we\u00b4ll have to assume that $n$ is the rank of our matrices $A$ and $B$. \r\n\r\nSo, we\u00b4ll have that all eigenvalues of $A$ will be equals to 0. That\u00b4s because if we considerate $x_{a}$ an eigenvector of $A$, we\u00b4ll have that $A^{n}x_{a}=0 x_{a}$ -> $t^{n}x_{a}=0$ -> $t^{n}=0$ -> $t=0$. That\u00b4s because $x_{a}$ is far from $0$.\r\n\r\nBy an analogue demonstration we\u00b4ll have that all eigenvalues of $B$ we\u00b4ll be equals to $0$.\r\n\r\nNow we considerate $x_{b}$ an eigenvector of $B$. Notice that $A^{n}x_{b}=0$ -> $A(A^{n-1}x_{b})=0$. But $det(A)=0$ and so we\u00b4ll have that $A^{n-1}x_{b}$ isn\u00b4t the null vector. And I\u00b4ll prove now that this vector is an eigenvector of $(A+B)$. To do this observe that $(A+B)A^{n-1}x_{b}=A^{n}x_{b}+BA^{n-1}x_{b}=A^{n}x_{b}+A^{n-1}Bx_{b}=0$.\r\n\r\nhere I did not considerate that $A$ and $B$ have an commun eigenvector. If they have this eigenvector will be an eigenvector of $(A+B)$.\r\n\r\nAnd we can make this for all eigenvector of $B$ and conclude now that all eigeinvalue of $(A+B)$ is zero and so by Cayley-Hamilton theorem we\u00b4ll have that $(A+B)^{n}=0$." } { "Tag": [ "summer program", "MathPath", "search" ], "Problem": "[b]For COP[/b]\r\n\r\nThe last round will be graded and result after about two weeks. The reason is because I won't have computer access at all. So as soon as I get my computer back, I'll grade the last round of COP and finish this.\r\n\r\n[b]For Getting Started[/b]\r\n\r\nOther moderator white_horse_king is currently in vacation. He sent me message on this matter and till now, it was okay since I was able to moderate here. But since I won't be able for about two weeks (hopefully this is it probably), I suggest you guys to do NOT SPAM.\r\n\r\nWhen I get back and see some unncessary posts made often by certain users, I'll notify the administrator. \r\n\r\nNot meaning that you can't post at all. Let's just not spam. :) \r\n\r\nHave a good summer!", "Solution_1": "but there is still moderator jli!", "Solution_2": "jli is at MathPath. He does not have ample access to a computer. None of us do.", "Solution_3": "[quote=\"IntrepidMath\"]jli is at MathPath. He does not have ample access to a computer. None of us do.[/quote]\r\nwhat is mathpath?", "Solution_4": "So, there will be no mods for a while? \r\n[hide]Hmmm.... [evil_music]:evil_laugh: I could take over the forum! :evil_laugh:[/evil_music] [hide]Nah.... That takes too much work.[/hide] [/hide]", "Solution_5": "those administrators will still be there \r\nnice tags Xantos", "Solution_6": "Mathpath is a math camp for middle schoolers. For more info about it, please do a search in the forum with that keyword.", "Solution_7": "[quote=\"IntrepidMath\"]jli is at MathPath. He does not have ample access to a computer. None of us do.[/quote]\r\n\r\ndo not fear...jli is here. :D", "Solution_8": "Yay! \r\njli is back.", "Solution_9": "No, he's still at Mathpath, as am I. But we have 15+ mins of computer time every day, and I guess that's enough for him to moderate??", "Solution_10": "I got a tab bit of computer access today, but the internet spontaneously shuts down... sigh.", "Solution_11": "Consider yourself lucky. The people at the campus we're at gave us computers that have monitors with the wrong power cords. So the monitors shut down after about two seconds and we have to keep turning them off and on. It's really annoying, but at least better than having the internet shut down spontaneously.", "Solution_12": "Wrong.\r\n\r\nIf you really want to use the computers, just go to the library after lunch or during free time. You lose.", "Solution_13": "I lose what?\r\n\r\nNot all of us can go to the library at arbitrary time x:yz, where x, y, and z are positive integers x<13, y<6, z<9. Some of us have other things we have to do, and some \r\nof us have to go to the library with someone else.\r\n\r\nCamp's almost over anyway.", "Solution_14": "Come on guys... chill out.", "Solution_15": "PMS affecting you, mathnerd?\r\n\r\nAd hominem aside, yeah, camp is almost over, so jli will be back to his moderater-ness in three or four days.", "Solution_16": "Treething, easy there.\r\n\r\nI'm currently in my local town library and eh.. let's just say it will take sometime for it to get fixed.\r\n\r\nThere was major attack on Windows itself and I can't do anything at all. No internet, no simple documenting, etc.. Turning on the computer gives me warnings on how it can't function..\r\n\r\nHopefully it will be okay so I can start the COP as I planned. (So far, it will but there is chance that it won't and if it doesn't, I'll notify in here).\r\n\r\nSilverfalcon" } { "Tag": [ "geometry", "rectangle", "calculus", "ratio", "trigonometry", "function" ], "Problem": "Sorry if my problem has been poorly evaluated considering difficulty. Here's a problem that I really enjoyed, and I hope you do too. Here it is.\r\n\r\nWhat is the maximum area of a rectangle circumscribed about a fixed rectangle of length 8 and width 4?\r\n\r\nMasoud Zargar", "Solution_1": "how do you define circumscribing about a rectangle", "Solution_2": "Calculus... *cough* Optimization *cough* *cough* :)", "Solution_3": "I have a question. Would you define very very basic defferentiation inappropriate for this forum?\r\n\r\nMasoud Zargar", "Solution_4": "[quote=\"Altheman\"]how do you define circumscribing about a rectangle[/quote]\r\n\r\nWithin a rectangle at an angle.", "Solution_5": "[quote=\"boxedexe\"]I have a question. Would you define very very basic defferentiation inappropriate for this forum?[/quote]Oh, not at all. I was just implying that I knew the calculus solution (didn't fail my math class). :)", "Solution_6": "[hide]\nCall the original rectangle ABCD and the new one WXYZ such that A lies on WX, B on XY, C on YZ, and D on WZ.\n\nLet $AW = x$, $WD = y$. Since $DWA$ and $AXB$ are both right and $\\angle WAD = 90 - \\angle BAX = \\angle ABX$, we have $DWA$ similar to $AXB$. But the hypotenuses are in ratio $1: 2$ so the sides are as well. Then $AX = 2y$, $BX = 2x$.\n\nSo $[WXYZ] = (AW+AX)(WD+BX) = (x+2y)(y+2x) = 2(x^2+y^2)+5xy$. Since $x^2+y^2 = 16$ and $xy \\le \\frac{x^2+y^2}{2} = 8$ by AM-GM, we have $[WXYZ] \\le 2(16) + 5(8) = 72$, which is satisfied when WXYZ is a square.[/hide]", "Solution_7": "Nice solution. I really like it. Here's my solution.\r\n[hide=\"Solution\"]\nLet $ABCD$ and $JKLM$ be the circumscribing rectangle and the fixed rectangle, respectively. $J$ lies on $AB$, $K$ lies on $BC$, $L$ lies on $CD$, and $M$ lies on $AD$. $AJ$ subtends $\\angle\\theta$, and $BK$ subtends another angle which is also $\\angle\\theta$. We know that $|ML|=|JK|=8$ and $|MJ|=|KL|=4$. We use trigonometry and get, $|AJ|=4\\sin\\theta$, $|BJ|=8\\cos\\theta$, $|AM|=4\\cos\\theta$, and $|MD|=|BK|=8\\sin\\theta$.\n\nIt is now clear that the area of the circumscribing rectangle is $A=(|AM|+|MD|)(|AJ|+|BJ|)$. Let us define $A$ as a function of $theta$.\n\\[ \\\\A(\\theta)=(4\\cos\\theta+8\\sin\\theta)(4\\sin\\theta+8\\cos\\theta)\\\\=16\\sin\\theta\\cos\\theta+32\\sin^2\\theta+32\\cos^2\\theta+64\\sin\\theta\\cos\\theta\\\\=80\\sin\\theta\\cos\\theta+32=40\\sin 2\\theta+32\\implies 0<\\theta<\\frac{\\pi}{2} \\]\n\\[ \\\\A'(\\theta)=80\\cos 2\\theta\\\\0=\\cos 2\\theta\\\\2\\theta=\\frac{\\pi}{2}\\\\\\theta=\\frac{\\pi}{4} \\]\n$\\\\|AJ|=4\\sin\\frac{\\pi}{4}=2\\sqrt{2}\\\\|BJ|=8\\cos\\frac{\\pi}{4}=4\\sqrt{2}\\\\|AM|=4\\cos\\frac{\\pi}{4}=2\\sqrt{2}\\\\|MD|=8\\sin\\frac{\\pi}{4}=4\\sqrt{2}$\n\nWe now determine the area of the circumscribing rectangle. $A=(|AM|+|MD|)(|AJ|+|BJ|)=(2\\sqrt{2}+4\\sqrt{2})(2\\sqrt{2}+4\\sqrt{2})=(6\\sqrt{2})^2=\\boxed{72}$.\nWe see that our dimensions satisfy teh dimensions of a square, which has the largest area.\n$\\mathbb{QED}$\n[/hide]\r\n\r\nMasoud Zargar" } { "Tag": [ "geometry", "3D geometry", "sphere" ], "Problem": "The area and volume of a sphere are given by $A\\pi$ and $V\\pi$ respectively.If the value of A and V are both 3 digit integers.What s the smallest possible radius of the sphere ?\r\n :?", "Solution_1": "$A\\pi=4\\pi r^2\\Longrightarrow A=4r^2$\r\n$V\\pi=\\frac{4}{3}\\pi r^3\\Longrightarrow V=\\frac{4}{3}r^3$\r\nSince $V$ is an integer, $r^3$ must be divisible by $3$, so $r$ must be divisible by $\\sqrt[3]{3}$. Also, $r^3$ must be an integer.\r\nSince $V$ is an integer, $r^2$ must be an integer.\r\nIf $r^3$ is an integer and $r^2$ is an integer, $r$ is an integer.\r\nBecause $4r^2\\ge 100$, $r\\ge 5$.\r\nSince $r$ must be divisible by $3$, the least possible value for $r$ is $\\boxed{6}$", "Solution_2": "???", "Solution_3": "D :?: :?: :?:" } { "Tag": [ "inequalities" ], "Problem": "Let be $ a,b,c$ the side lenghts of a triangle . Then prove that :\r\n\r\n$ \\frac{a\\plus{}b\\minus{}2c}{c^2\\plus{}ab}\\plus{}\\frac{b\\plus{}c\\minus{}2a}{a^2\\plus{}bc}\\plus{}\\frac{c\\plus{}a\\minus{}2b}{b^2\\plus{}ca}\\ge 0$", "Solution_1": "hello, your inequality is equivalent to\r\n$ ba^4\\minus{}b^3c^2\\plus{}b^4c\\minus{}c^2a^3\\plus{}ab^4\\minus{}c^3b^2\\plus{}c^4a\\minus{}a^2c^3\\minus{}a^3b^2\\minus{}b^3a^2\\plus{}c^4n\\plus{}a^4c\\geq0$ this can be simplifyied to $ ab(a\\minus{}b)(a^2\\minus{}b^2)\\plus{}bc(b^2\\minus{}c^2)(b\\minus{}c)\\plus{}ac(a\\minus{}c)(a^2\\minus{}c^2)\\geq0$\r\nthis is true.\r\nSonnhard." } { "Tag": [ "geometry", "3D geometry", "sphere", "puzzles" ], "Problem": "A helicopter took of from Leningrad and flew north. After five hundredkilometres it turned and flew 500 kilometres east. After that it turned south and covered another 500 kilometres. Then it flew 500 kilometres west, and landed. The question is, where did it land: west, east, north, or south of Leningrad?\r\n\r\n I'm not sure if this is too simple........", "Solution_1": "that depends. It can be anywhere but north of lenningrad.\r\n\r\nSay it were 500 miles south of the north pole. It can land either east or west of lenningrad or right on it.\r\n\r\nsay it were on the south pole. Then it would land right on lenningrad\r\n\r\nsay it were on the north pole. It can't go north, goes around the earth infinitely, goes south, goes west and is still south of lenningrad.\r\n\r\nbut if you mean anywhere else, it'll be east.", "Solution_2": "[hide=\"answer\"]Since Leningrad is in the Northern hemisphere, and less than 500 km from the pole, the helicopter would end up East of Leningrad. The reason for this is that the surface of the earth is a sphere, and the closer to the pole you are, the smaller the distance around the earth. Thus, the 500 km east that the helicopter flies, being at a distance closer to the North Pole than the 500 km west, actually take the helicopter further around the earth in angular measure.\n\nAn easier example to see this effect is as follows. Imagine you are on the equator and go north till you reach the north pole. There you spin around at random and go south til you reach the equator. Even though all you did was go north and then go south the same distance, you will not necessarily end up in the place yuo started from! In fact, if you don't spin and keep going straight then you will end up on the opposide side of the globe!\n\nHope this helps.[/hide]", "Solution_3": "oh....lenningrad is an acutal place? i didn't know that... :oops:", "Solution_4": "correct, i thought most people would fall for the \" :huuh: dosen't it land $in$ leningrad?\"", "Solution_5": "well that would be possible...\r\n\r\nif lenningrad were exactly 250 km south of the equator", "Solution_6": "would it land back in Leningrad?", "Solution_7": "[quote=\"math92\"]would it land back in Leningrad?[/quote]\r\ndid u say that just to say it lol? ... :P right after GUNIt\r\nsaid:\r\ncorrect, i thought most people would fall for the \" dosen't it land [b][i]in[/i][/b] leningrad?\"", "Solution_8": "you might want to read more of the topic before you post math92. but i agree with JanSiwanowicz it would land east of Leningrad since as you go north the distance around earth is less.", "Solution_9": "oh, i didn't see G-UNIT's post :blush:", "Solution_10": "how old is this problem...\r\n\r\nLeninsgrad doesn't even exist anymore....it is called St. Petersburg now....", "Solution_11": "right on it...\r\nyou guys take to many chances... :rotfl:" } { "Tag": [], "Problem": "In IMO2005 VISA form which the contestents (I) have to solve it is said that: on that the contestents have to sing. but i can't find any place for signing!\r\n\r\nin one place of second page it is written that:\r\nFIRMA DEL PROMOVENTE\r\n\r\ni can easily guess that: FIRMA = sign, DEL= of the but PROMOVENTE=? I am not sure! can anyone help?\r\n\r\nMahbub", "Solution_1": "I am sure David will help you, becuase I am not very sure about \"Promovente\", it could be the person requaesting the Visa, or it could also be the person who will read the request and accept it, I dont know...", "Solution_2": "From my side i am 99.99% sure ( :D ) that it is the reader of the VISA letter! :D \r\n\r\nLet me see if my guessing power is correct! :P", "Solution_3": "I think you are right!", "Solution_4": "[quote=\"Pascual2005\"]I am sure David will help you, becuase I am not very sure about \"Promovente\", it could be the person requaesting the Visa, or it could also be the person who will read the request and accept it, I dont know...[/quote]\r\n\r\nSorry for not being answering very fast lately, but my wrist was left wrist was almost broken in an accident. It is a little better tonight, but I still am too slow writting on a computer.\r\n\r\nI searched on several dictionaries (included the dictionary of the [b]Real Academia Espa\u00f1ola[/b]), and the word is not there, hence I suppose it may be a mistranslation. Now, by the roots it looks like it is either the \"Sponsor\" if it applies, or the person who would potentially take care of the debt in case you can't pay (I suppose that most of the IMO participants are too young for VISA to give them directly a credit account to them). If that does not make sence, then probably it is actually your firm the one that has to go there.\r\n\r\nBye" } { "Tag": [ "geometry" ], "Problem": "The largest square possible is inscribed in a regular octagon. The longest diagonal of the octagon is 20 cm. What is the number of square centimeters in the area of the square?", "Solution_1": "[hide=\"Hint\"]The biggest square is the one that connects four of the vertices. (I'm not sure how to prove this myself so if anyone knows how please tell :D )[/hide]", "Solution_2": "From the hint that you posted, the diagonal of the octagon would also be the diagonal of the square.\r\n[hide]\nOne side of the square would be [tex]\\frac {20}{\\sqrt 2}[/tex]\n\nSo the area would be [tex] 200 \\ cm^2[/tex]\n\n[/hide]", "Solution_3": "[quote=\"ritchjp\"]From the hint that you posted, the diagonal of the octagon would also be the diagonal of the square.\n[hide]\nOne side of the square would be [tex]\\frac {20}{\\sqrt 2}[/tex]\n\nSo the area would be [tex] 200 \\ cm^2[/tex]\n\n[/hide][/quote]\r\n\r\nRight. :)" } { "Tag": [ "counting", "derangement", "search", "floor function" ], "Problem": "Suppose a school has $2007$ students, each with a corresponding piece of paper with his/her name on it. How many ways can the papers be distributed to the students so that every student receives exactly one piece of paper and no student receives the paper with his/her name on it?", "Solution_1": "It seems like this is a little calculation intensive. Using the recurrence relation would take a long time to get up to 2007, and if I remember correctly there's a summation method of calculating it, but surely you have to have a calculator?\r\n\r\nA link for anyone who doesn't know what derangements are: [url]http://en.wikipedia.org/wiki/Derangement[/url]\r\nYou could also search it on the forum.", "Solution_2": "It's going to be approximately $\\frac{2007!}{e}$. Probably if you were to calculate that and then round to the nearest integer that would be the correct answer.", "Solution_3": "I agree with the approximation of $\\frac{2007!}{e}$, but is this what drunner wants?", "Solution_4": "I can't prove it but I've managed to convince myself that it's exactly $\\left\\lfloor\\frac{2007!}{e}\\right\\rfloor$, which is about $1.5835747690667932749504701216585 \\times 10^{5758}$.", "Solution_5": "This question isn't designed for a tournament. Its just a problem.\r\nThe approximation should be $2007!(1-e^{-1})$. There is an exact answer though. You don't need to think recursively to get it. Just apply PIE", "Solution_6": "The exact answer is $\\left\\lfloor\\frac{2007!}{e}\\right\\rfloor$ :P", "Solution_7": "yeah, I believe there's a non-recursive formula. Actually, what is the recursive formula??? (I only know the formula from PIE)\r\n[hide]\nwe'll count the number of ways that at least 1 student got his/her name. Using PIE we have $2007! - \\binom{2007}{2}2005! + \\binom{2007}{3}2004! - \\dots \\pm \\binom{2007}{k}(2007-k)! \\mp \\dots + 1$, and 2007! subtract this is $\\binom{2007}{2}2005! - \\binom{2007}{3}2004! + \\dots -1$.\n[/hide]", "Solution_8": "Let me post my answer. :) \r\n\r\nFirst the exact number of ways. It will be (total number of ways you can distribute paper to students)-(number of ways at least one person gets their own paper). Let's find the second quantity.\r\nThere are $\\binom{2007}{1}2006!$ ways to guarantee that at least one person receives their hat since there are $\\binom{2007}{1}$ ways to choose the person and $2006!$ ways to arrange the rest of the hats. But we've overcounted the cases where two people have received their hat. So we subtract $\\binom{2007}{2}2005!$. Then we add back in the cases where three people receive their hat: $\\binom{2007}{3}2004!$, and so on, giving $\\binom{2007}{1}2006!-\\binom{2007}{2}2005!+\\binom{2007}{3}2004!-\\ldots+\\binom{2007}{2007}0!$ as the number of ways that at least one person receives their hat.\r\nThe total number of ways we can distribute the hats is $2007!$. So the number of ways that we can distribute the hats without one person receiving their own hat is $2007!-(\\binom{2007}{1}2006!-\\binom{2007}{2}2005!+\\binom{2007}{3}2004!-\\ldots+\\binom{2007}{2007}0!)=\\binom{2007}{0}2007!-\\binom{2007}{1}2006!+\\binom{2007}{2}2005!-\\binom{2007}{3}2004!+\\ldots-\\binom{2007}{2007}0!$\r\n\r\nNow let's find the approximation. Factor out $2007!$ to get $2007!(\\binom{2007}{0}\\frac{1}{0!}-\\binom{2007}{1}\\frac{1}{2007}+\\binom{2007}{2}\\frac{1}{2007*2006}-\\binom{2007}{3}\\frac{1}{2007*2006*2005}+\\ldots-\\binom{2007}{2007}\\frac{1}{2007!}=2007!(\\frac{1}{0!}-\\frac{1}{1!}+\\frac{1}{2!}-\\frac{1}{3!}+\\ldots-\\frac{1}{2007!})$ which is approximately equal to $2007!(\\frac{1}{0!}-\\frac{1}{1!}+\\frac{1}{2!}-\\ldots)=2007!(e^{-1})$\r\n\r\nAh man, you were right! I accidentally wrote the last part as $2007!(1-e^{-1})$ the other day. Ok, I agree with you guys :lol:", "Solution_9": "Yeah, I type till mid-way then I'm like \"How do you continue simplifying again...???\" :rotfl: thx, drunner by posting the real formula.", "Solution_10": "you have to be careful with the derangement formula, it is\r\n\r\n$\\frac{n!}{e}$ rounded, not floor,\r\n\r\nthe proof has something to do with the identity:\r\n\r\n$e=\\frac{1}{0!}+\\frac{1}{1!}+\\frac{1}{2!}+...$\r\n\r\nand comparing it to the formula from the inclusion-exclusion formula", "Solution_11": "Yeah just look at my post", "Solution_12": "[quote=\"pkerichang\"]what is the recursive formula???[/quote]It's on the wikipedia page in my first post. $d_n=(n-1)(d_{n-1}+d_{n-2})$", "Solution_13": "[quote=\"Altheman\"]you have to be careful with the derangement formula, it is\n\n$\\frac{n!}{e}$ rounded, not floor,\n\nthe proof has something to do with the identity:\n\n$e=\\frac{1}{0!}+\\frac{1}{1!}+\\frac{1}{2!}+...$\n\nand comparing it to the formula from the inclusion-exclusion formula[/quote]\r\n\r\nWell, you can verify that for odd $n$, it's floor, and for even $n$, it's floor + 1. That amounts to the same thing as rounding, or the floor of $\\frac{n!}{e}+\\frac{1}{2}$, though." } { "Tag": [ "limit", "superior algebra", "superior algebra solved" ], "Problem": "Compute \\lim_{x->0}(x^1999 - (tgx)^1999)/x^2001\r\n\r\nHint: When x tend to 0 tgx = x + x^3/3+2/15.x^5 +o(x^5)", "Solution_1": "I update this problem \r\nCompute \\lim_{x->0} (x^2003 - (tgx)^2003)/x^2005 \r\nParis-2003", "Solution_2": "Rewrite the limit like so:\r\n\r\nlim(x-->0) [1-(tan(x)/x)^1999]/x^2=lim(x-->0) [x-tan(x)]/x^3[1+tan(x)/x+..+(tan(x)/x)^1998]=1999lim(x-->0) [x-tan(x)]/x^3=-1999/3\r\n\r\nlim(x-->0) (x-tan(x))/x^3=-1/3 by L'hopital.\r\n\r\nI think that the other problem has the answer:\r\nL=-2003/3\r\n\r\ncheers! :D :D", "Solution_3": "[quote=\"Moubinool\"]Compute \\lim_{x->0}(x^1999 - (tgx)^1999)/x^2001\n\nHint: When x tend to 0 tgx = x + x^3/3+2/15.x^5 +o(x^5)[/quote]\r\n\r\nI do it with Landau technic \r\nN(x)= x^1999 - (tgx)^1999=x^1999 - ( x + x^3/3+ o(x^3))^1999\r\n= x^1999 - x^1999(1+x^2/3 + o(x^2))^1999\r\n=x^1999 - x^1999(1 + 1999/3.x^2 + o(x^2))\r\n= -1999/3.x^2001 +o(x^2) \r\nN(x)/x^2001 = -1999/3+ o(1) \r\nthe lilmit is -1999/3" } { "Tag": [ "percent", "LaTeX", "function" ], "Problem": "How do you type a percent sign or approximation sign on latex? Thanks everybody.\r\n\r\nPercent sign: It doesn't show up!---> $ 30%\n$ (I typed \"30%\" in between 2 dollar symbols)\r\n\r\nApproximation...I can get ~ but not two of those.... :(", "Solution_1": "Did you try looking at http://www.artofproblemsolving.com/Wiki/index.php/LaTeX:Symbols ? :wink:", "Solution_2": "Specifically, it would be\r\n\r\n\\approx - $ \\approx$\r\n\r\n30 \\% - $ 30 \\%$", "Solution_3": "$ 30% x=y\n$\r\n\r\nI was just curious about what would happen on the forum; what I actually typed between the dollar signs there was 30% x=y. You'll notice that not only did the % sign disappear, but so also the x=y disappeared; that is what I expected to happen.\r\n\r\nIn a $ \\text{\\LaTeX}$ document - and remember, such a document is essentially a computer program - the % sign serves a function familiar to programmers: it starts a comment. Specifically, everything on a line after % stays in the source code as an annotation but is not executed and produces nothing on a document.\r\n\r\nPrecisely because the % symbol has that specialized use, you need to use \\% to actually use it as a symbol in a document.", "Solution_4": "Ok, thx everyone for the input. :D" } { "Tag": [], "Problem": "A move consists of joining two clusters, including clusters of just one piece. What is the minimum number of moves required to complete a two-thousand piece jigsaw puzzle that is separated into individual pieces?", "Solution_1": "Every time you join two clusters you reduce the number of clusters by $ 1$. We need to reduce it from $ 2000$ to $ 1$, so $ 2000\\minus{}1\\equal{}\\boxed{1999}$ moves are needed." } { "Tag": [ "calculus", "integration", "geometry", "parameterization", "trigonometry", "analytic geometry", "3D geometry" ], "Problem": "A hemisphere with radius $R$ lies in the $xy$-plane. \r\nLet $\\bar{A}$ be a vectorfield $\\bar{A}=-y\\hat{x}+x\\hat{y}+f(z)\\hat{z}$\r\n\r\nCalculate $\\oint_\\gamma \\bar{A}\\cdot d\\bar{r}$\r\n\r\nand $\\iint_\\Sigma (\\bar{\\nabla}\\times \\bar{A}) \\cdot dS$\r\n\r\nWhere $\\gamma$ is the large circle (base) of the hemisphere lying in the xy-plane (+ orientation) and $\\Sigma$ the surface of the hemisphere with $\\gamma=\\partial \\Sigma$. $f(z)$ is an arbitrary nice function.", "Solution_1": "Does anyone have an idea?", "Solution_2": "By Stokes' Theorem, the two integrals are equal, so we only need do one of them. Look at the line integral: since $\\hat{z}\\cdot\\vec{dr}=0$ on that curve, we may safely ignore $f(z).$ I'd suggest just parameterizing the curve and computing it - it's straightforward.\r\n\r\nBut an alternative: A third integral that is equal to the other two is the integral of $\\nabla\\times A$ over the flat disk in the $xy$-plane bounded by $\\gamma.$ That's also equal to the line integral by Stokes' Theorem. But since the normal to the surface is $\\hat{z},$ we need only compute the $\\hat{z}$ component of the curl (again, that ignores $f(z).$) This will turn into the integral of a constant over that area, once you compute that component of the curl.", "Solution_3": "But i get different values for each of them :( .\r\n\r\n[b]The first one:[/b]\r\nfirst I find a parametrization: $\\bar{r}(t)=(R\\cos{t},R\\sin{t},0)$, $t\\in[0,2\\pi]$\r\n\\[\\oint_\\gamma \\bar{A}\\cdot d\\bar{r}=\\int_{0}^{2\\pi}\\bar{A}(\\bar{r}(t))\\cdot(\\bar{r}'(t))=\\ldots \\]\r\nAnd we have: $\\bar{r}'(t)=R(-\\sin{t}, \\cos{t},0)$\r\nand also: $\\bar{A}(\\bar{r}(t))=R(-\\sin{t}, \\cos{t}, f(0))$\r\nso scalar prod. yields\r\n\\[\\ldots=\\int_{0}^{2\\pi}R^{2}(-\\sin{t}, \\cos{t}, f(0))\\cdot R(-\\sin{t}, \\cos{t},0)=2\\pi R^{2}\\]\r\n\r\n\r\n\r\n[b]The second one:[/b]\r\nHere we switch to spherical polar coordinates:\r\n$\\bar{r}(\\theta, \\varphi)=R[(\\sin{\\theta}\\cos{\\varphi}),(\\sin{\\theta}\\sin{\\varphi}),(\\cos{\\theta})]$\r\nAnd since $\\mbox{rot}\\bar{A}=(0,0,2)$ our integral becomes\r\n\\[\\int_{0}^{2\\pi}\\int_{0}^\\pi (0,0,2)\\cdot(|\\bar{r}'_\\theta\\times\\bar{r}'_\\varphi|d\\theta d\\varphi) \\]\r\nWhere $d\\bar{S}=|\\bar{r}'_\\theta \\times\\bar{r}'_\\varphi|d\\theta d\\varphi$\r\nThis can also be expressed as $\\bar{n}dS$ where $\\bar{n}$ is the normal to the sphere aand could simpler be calculated as $\\frac{\\bar{r}}{|\\bar{r}|}=\\frac{\\bar{r}}{R}$\r\n\r\nThus the integral becomes\r\n\\[\\int_{0}^{2\\pi}\\int_{0}^\\pi (0,0,2)\\cdot(\\frac{\\bar{r}}{R}d\\theta d\\varphi) =2\\int_{0}^{2\\pi}\\int_{0}^\\pi \\cos{\\theta}d\\theta d\\varphi=0 \\]\r\n\r\n\r\n[color=blue]So where is the problem?\nI appreciate som help with the details.[/color]", "Solution_4": "The first one, the line integral, is correct.\r\n\r\nYour problem in the second one is that by letting $\\varphi$ go from $0$ to $\\pi$ you integrated over the whole sphere. You should have only the upper hemisphere, in which $\\varphi$ ranges from $0$ to $\\frac{\\pi}2.$", "Solution_5": "[quote=\"Kent Merryfield\"]The first one, the line integral, is correct.\n\nYour problem in the second one is that by letting $\\varphi$ go from $0$ to $\\pi$ you integrated over the whole sphere. You should have only the upper hemisphere, in which $\\varphi$ ranges from $0$ to $\\frac{\\pi}2.$[/quote]\r\n\r\nOf course! yes! How stupid of me, I did not even consider that! :blush: :blush: Thanks a lot Mr. Merryfield !", "Solution_6": "[quote=\"Kent Merryfield\"]The first one, the line integral, is correct.\n\nYour problem in the second one is that by letting $\\varphi$ go from $0$ to $\\pi$ you integrated over the whole sphere. You should have only the upper hemisphere, in which $\\varphi$ ranges from $0$ to $\\frac{\\pi}2.$[/quote]\r\n\r\nis it correct to take abs signs in ($r\\times r \\ldots$)? And is the geometrical\r\nmethod $\\frac{\\bar{r}}{R}$ equivalent? Does $r\\times r \\ldots$ have anything to do with the jacobian for the parametrization?" } { "Tag": [ "logarithms", "function" ], "Problem": "I am taking an Algebra 2 course and just was told that you can't take the log of negative numbers. Can't you do $ log_\\minus{}3 \\minus{}27$? The answer would be negative 3. PLease explain why this is not allowed.", "Solution_1": "The answer would be $ 3$. However, note that your base is also negative. In general, the base of a logarithm is positive, and once we add that condition, $ \\log_3 (\\minus{}27)$, for example, is not defined.\r\n\r\nWhy are negative bases discouraged? The simple answer is that exponential functions with negative bases are very badly behaved. Try to graph $ y \\equal{} (\\minus{}3)^x$ on your calculator.\r\n\r\nThe more complicated answer is that in analysis, we define every logarithm in terms of the natural logarithm $ \\ln x \\equal{} \\log_e x$, and a negative base (or taking the logarithm of a negative number) requires you to define $ \\ln \\minus{}1$, which is not real." } { "Tag": [ "\\/closed" ], "Problem": "Hey, you know what's frustrating sometimes is trying to split topics but not being able to. People sometimes address two issues in one post and then you can't move that post b/c it's relevant to the old topid but you can't not move it b/c it's part of the new one.\r\n\r\nAnyways, what I'm saying is, is there a way we can make it so mods can split posts as well as threads? That would be most awesome.\r\n\r\nJust a suggestion ;-)", "Solution_1": "Hehe, splitting posts is and probably will not be possible very soon. We'll see maybe later about merging topics, and then, you split out a certain post and merge it with the previously splitted topic.", "Solution_2": "Merging topics would be good too. *nods* Oodles of convenience for us.", "Solution_3": "Merging topics would be great as there are so many places where the same question gets asked. It would be great if we could respond by tacking their post onto the previous answer with the response \"see above.\"\r\n\r\nThe only problem is that our priority list is currently 13.2 miles long.", "Solution_4": "[quote=\"MCrawford\"]The only problem is that our priority list is currently 13.2 miles long.[/quote]Actually it's $10 + \\pi$ long, but 13.2 will do as an approximation :P" } { "Tag": [ "inequalities", "induction" ], "Problem": "\\[ \\begin{array}{l}\r\n \\forall x_i \\in R,x_1 < x_2 < x_3 < ....... < x_n \\\\ \r\n find,the\\_{\\bf{rearrangement}}\\_x_{i_1 } ,x_{i_2 } ,x_{i_3 } ,......,x_{i_n } \\_of\\_the\\_given\\_nos\\_for \\\\ \r\n which \\\\ \r\n \\left( {x_{i_1 } \\minus{} x_{i_2 } } \\right)^2 \\plus{} \\left( {x_{i_2 } \\minus{} x_{i_3 } } \\right)^2 \\plus{} \\left( {x_{i_3 } \\minus{} x_{i_4 } } \\right)^2 \\plus{} ......... \\plus{} \\left( {x_{i_{n \\minus{} 1} } \\minus{} x_{i_n } } \\right)^2 \\plus{} \\left( {x_{i_n } \\minus{} x_{i_1 } } \\right)^2 \\\\ \r\n becomes\\_the\\_le{\\bf{a}}st \\\\ \r\n \\end{array}\r\n\\]", "Solution_1": "the \\[ x_i\\] are all distinct .\r\nso the given expression can never be 0.\r\n\r\n\r\ni think some sorta inequality will do the trick :wink:", "Solution_2": "We have the minimum of:\r\n\r\n$ ( {x_{i_1 } - x_{i_2 } } )^2 + ( {x_{i_2 } - x_{i_3 } } )^2 + ( {x_{i_3 } - x_{i_4 } } )^2 + ......... + ( {x_{i_{n - 1} } - x_{i_n } } )^2 + ( {x_{i_n } - x_{i_1 } } )^2$\r\n\r\nWhen $ A=x_{i_1 }.x_{i_2 }+x_{i_2 }.x_{i_3 }+...+x_{i_n }x_{i_1 } \\hspace{1cm}$ is maximum.\r\n\r\nWe will prove by induction that we have the maximum of $ A$ when:\r\n$ A=x_nx_{n-1}+x_{n-1}x_{n-3}+x_{n-3}x_{n-5}+...+x_1x_2+...+x_{n-4}x_{n-2}+x_{n-2}x_n$\r\n\r\nFor $ n=3$ is obvious.\r\n\r\nSuppose that it is true for $ n=k$\r\n\r\nLet see $ A$ like a string. We choose the position of $ x_1$ in any string with the numbers $ x_2,x_3,...,x_k$ such that $ A$ is maximum.\r\n\r\nSuppose that the string after we choose the position of $ x_1$ is $ ....x_j,x_1,x_p....$\r\n\r\nThen when we add $ x_1$ to the string the value of $ A$ increase by:\r\n$ B=x_jx_1+x_1x_p-x_jx_p$\r\n$ B=-(x_1-x_j)(x_1-x_p)+x_1^2$\r\nWe have the maximum of $ B$ when:\r\n$ C=(x_1-x_j)(x_1-x_p)$ is minimum\r\nThen we have the minimum of C when $ x_j=x_2$ and $ x_p=x_3$.\r\n\r\nBy the supposition, the maximum of $ A$ with the numbers $ x_2,x_3,...,x_k,x_{k+1}$ is:\r\n$ x_{k+1}x_k+x_kx_{k-2}+x_{k-2}x_{k-4}+...x_2x_3+...+x_{k-3}x_{k-1}+x_{k-1}x_{k+1}$\r\n\r\nAnd choosing the best position of $ x_1$:\r\n$ A=x_{k+1}x_k+x_kx_{k-2}+x_{k-2}x_{k-4}+...x_2x_1+x_1x_3+...+x_{k-3}x_{k-1}+x_{k-1}x_{k+1}$\r\nThen it is true for $ n=k+1$ and by the induction is true for all $ n$.\r\n\r\nThen the rearrange is:\r\n\r\n${ ( {x_{n } - x_{n-1 } } )^2 + ( {x_{n-1 } - x_{n-3 } } )^2 + ( {x_{n-3 } - x_{n-5 } } )^2 + ......... + ( {x_{n-4} } - x_{n-2 } } )^2 + ( {x_{n-2 } - x_{n } } )^2$" } { "Tag": [ "combinatorics unsolved", "combinatorics" ], "Problem": "3 countries $A, B, C$ participate in a competition where each country has 9 representatives. The rules are as follows: every round of competition is between 1 competitor each from 2 countries. The winner plays in the next round, while the loser is knocked out. The remaining country will then send a representative to take on the winner of the previous round. The competition begins with $A$ and $B$ sending a competitor each. If all competitors from one country have been knocked out, the competition continues between the remaining 2 countries until another country is knocked out. The remaining team is the champion.\r\n\r\n\r\n[b]I.[/b] At least how many games does the champion team win?\r\n\r\n[b]II.[/b] If the champion team won 11 matches, at least how many matches were played?", "Solution_1": "Suppose $A$ is the champion. $A$ plays $a$ times and loses $b$ times ($b\\le8$).\n\n$A$ does not play when: (1) the first game starts with $B$ and $C$; (2) $A$ loses. So the number of games $A$ does not play is $1+b$ or $b$. The total number of games is equal to the total number of loses, which is $b+9+9=b+18$. Thus we get $1+b+a=b+18$ or $b+a=b+18$, whence $a=18$ or $a=17$.\n\nI. The number of wins for $A$ is $a-b\\ge17-8=9$.\n\nConstruction: $B$ and $C$ starts the game. $B,C,A$ loses repeatedly in this order.\n\nII. We have $a-b=11$, so $b=a-11=6$ or $7$. Thus the number of games is $b+18\\ge24$.\n\nConstruction: $B$ and $C$ starts the game. $B$ beats 6 representatives of $A$ and 7 representatives of $C$ in the first 13 games. Then $A$ beats all 11 remaining representatives from $B$ and $C$ in the last 11 games." } { "Tag": [ "geometry", "parallelogram", "calculus", "integration", "trapezoid", "complex numbers", "national olympiad" ], "Problem": "I have post CWMO 2001, 2003 and 2004 problems, I'll post CWMO 2002 problem here, which is also in Chinese. \r\nHave a good day!", "Solution_1": "I'll translate it, but later perhaps, because I really should be asleep at this hour of the day :blush: .", "Solution_2": "I'll be waiting when you wake up ;)", "Solution_3": "There you go, Myth! :)\r\n\r\n1) Find all positive integers $n$ such that $n^4 - 4n^3 + 22n^2 -36n + 18$ is a perfect square.\r\n\r\n2) Let $O$ be the circumcentre of $\\triangle ABC$ and $P$ be a point in the interior of $\\triangle AOB$. Let $D, E, F$ be the feet of the perpendiculars from $P$ onto $BC, CA, AB$ respectively. Prove that the parallelogram with adjacent sides $FE$ and $FD$ is situated within $\\triangle ABC$.\r\n\r\n3) Consider a square in the complex plane. The four complex numbers that its vertices represent happen to be the four roots of the equation $x^4 + px^3 + qx^2 + rx + s = 0$ which has integral coefficients. Find the smallest possible area of such a square.\r\n\r\n(I really hope that I translated this problem correctly...)\r\n\r\n4) Let $n$ be a positive integer. The sets $A_1, A_2, \\ldots, A_{n+1}$ are $n + 1$ non-empty subsets of $\\{1, 2, \\ldots, n\\}$. Prove that there exist two disjoint subsets $\\{i_1, i_2, \\ldots, i_k\\}$ and $\\{j_1, j_2, \\ldots, j_m\\}$ of $\\{1, 2, \\ldots, n + 1\\}$ such that $A_{i_1} \\cap A_{i_2} \\cap \\cdots \\cap A_{i_k} = A_{j_1} \\cap A_{j_2} \\cap \\cdots \\cap A_{j_m}$.\r\n\r\n5) In a given trapezium $ABCD$, $AD$ is parallel to $BC$, $E$ is a variable point on $AB$, $O_1, O_2$ are the circumcentres of $\\triangle AED, \\triangle BEC$ respectively. Prove that $O_1O_2$ is a constant.\r\n\r\n6) Let $n$ ($n \\geq 2$) be a given integer. Find all sets of integers $\\{a_1, a_2, \\ldots, a_n\\}$ that satisfy the following conditions:\r\n(I) $a_1 + a_2 + \\ldots + a_n \\geq n^2$\r\n(II) $a_1^2 + a_2^2 + \\ldots + a_n^2 \\geq n^3 + 1$.\r\n\r\n7) Let $\\alpha, \\beta$ be the roots of the equation $x^2 - x - 1 = 0$, and let $a_n = \\frac{\\alpha^n - \\beta^n}{\\alpha - \\beta}, n = 1, 2, \\ldots$.\r\na) Prove that for any positive integer $n$, $a_{n+2} = a_{n+1} +a_n$\r\nb) Find all positive integers $a, b$ $(a < b)$ such that for any positive integer $n$, $b$ divides $a_n - 2na^n$.\r\n\r\n8) Let $S = (a_1, a_2, \\ldots, a_n)$ be a longest possible sequence formed from the numbers $0, 1$ satisfying the following condition: No $2$ pairs of $5$ consecutive numbers in $S$ are the same, that is for any $1 \\leq i < j \\leq n - 4, a_i, a_{i+1}, a_{i+2}, a_{i+3}, a_{i+4}$ is not the same as $a_j, a_{j+1}, a_{j+2}, a_{j+3}, a_{j+4}$. Prove that the first $4$ terms of $S$ are the same as the last $4$ terms of $S$.", "Solution_4": "I make me happy, Valiowk!!! :)" } { "Tag": [ "national olympiad" ], "Problem": "Do you know any TST's or Olympiads which had problems from Balkan shortlist?(I know that UK has problems from balkan shortlist)", "Solution_1": "Noone?Why?" } { "Tag": [ "factorial", "AMC", "AIME", "blogs", "LaTeX", "FTW", "ratio" ], "Problem": "Guess who will post next.\r\n\r\nKeep track of points in each post.\r\n\r\nYou get 1 point for guessing.\r\n\r\nThe person who is guessed get's 2 points.\r\n\r\nYou get 5 additional points for getting your guess right.\r\n\r\nNo double posting.\r\n\r\nNo guessing yourself.\r\n\r\n-----------------------------------------------------------\r\n\r\nI'll guess Zephyredx.\r\n\r\niLord: 1\r\nZephyredx: 2", "Solution_1": "iLord: 1 \r\nZephyredx: 2\r\nmanofdayear:1\r\n1=2: 2\r\n\r\n\r\nI guess 1=2", "Solution_2": "iLord: 1 \r\nZephyredx: 2 \r\nmanofdayear:1 \r\n1=2: 4 \r\nmyyellowducky82:1\r\n\r\ni guess 1=2", "Solution_3": "iLord: 1 \r\nZephyredx: 2 \r\nmanofdayear:2\r\n1=2: 4 \r\nmyyellowducky82:1 \r\nPerfect628: 2\r\n\r\nperfect628", "Solution_4": "iLord: 1 \r\nZephyredx: 2 \r\nmanofdayear:2 \r\n1=2: 4 \r\nmyyellowducky82:2 \r\nPerfect628: 2 \r\ncrazychinesemanic: 2\r\n\r\ni guess crazychinesemanic.", "Solution_5": "Ummm... manofdayear = crazychinesemanic ...\r\n\r\niLord: 1\r\nZephyredx: 2 \r\n1=2: 4 \r\nmyyellowducky82: 4\r\nPerfect628: 2 \r\ncrazychinesemanic: 4\r\nchenhsi: 1\r\n\r\ni guess myyellowducky", "Solution_6": "I'll guess chenhsi!\r\n\r\niLord: 2 \r\nZephyredx: 2 \r\n1=2: 4 \r\nmyyellowducky82: 4 \r\nPerfect628: 2 \r\ncrazychinesemanic: 4 \r\nchenhsi: 3", "Solution_7": "[quote=\"chenhsi\"]Ummm... manofdayear = crazychinesemanic ...[/quote]\r\n\r\n\r\nhow do u know my runescape sn...i dont even know you\r\nEDIT: O wait...my bad :oops: \r\n\r\n\r\niLord: 4\r\nZephyredx: 2 \r\n1=2: 4 \r\nmyyellowducky82: 4 \r\nPerfect628: 2 \r\ncrazychinesemanic: 5\r\nchenhsi: 3\r\n\r\niLord", "Solution_8": "I'll guess Chenhsi again.\r\n\r\niLord: 5 \r\nZephyredx: 2 \r\n1=2: 4 \r\nmyyellowducky82: 4 \r\nPerfect628: 2 \r\ncrazychinesemanic: 5 \r\nchenhsi: 5", "Solution_9": "w00t i guessed right!!!!\r\n\r\niLord: 7\r\nZephyredx: 2 \r\n1=2: 4 \r\nmyyellowducky82: 4 \r\nPerfect628: 2 \r\ncrazychinesemanic: 10\r\nchenhsi: 5\r\n\r\ni guess iLord again", "Solution_10": "Yep! I'll guess Crazychinesemaniac!\r\n\r\niLord: 8 \r\nZephyredx: 2 \r\n1=2: 4 \r\nmyyellowducky82: 4 \r\nPerfect628: 2 \r\ncrazychinesemanic: 17 \r\nchenhsi: 5", "Solution_11": "err...scoreboard is messed up. My fault, fixed score board is here.\r\n\r\n\r\niLord: 17 \r\nZephyredx: 2 \r\n1=2: 4 \r\nmyyellowducky82: 4 \r\nPerfect628: 2 \r\ncrazychinesemanic: 19 \r\nchenhsi: 5\r\n\r\nI guess...lol 3 times the charm iLord!", "Solution_12": "iLord: 18 \r\nZephyredx: 2 \r\n1=2: 4 \r\nmyyellowducky82: 4 \r\nPerfect628: 2 \r\ncrazychinesemanic: 26 \r\nchenhsi: 5 \r\n\r\nYeah, let's rack up points!\r\n\r\nI'm waiting for CCM!", "Solution_13": "[quote=\"iLord\"]iLord: 20 \nZephyredx: 2 \n1=2: 4 \nmyyellowducky82: 4 \nPerfect628: 2 \ncrazychinesemanic: 26 \nchenhsi: 5 \nalexhhmun: 1\n\nYeah, let's rack up points!\n\nI'm waiting for CCM![/quote]\r\n\r\nI guess iLord. Please? \r\n\r\nP.S. It's kinda sad that 1=2 has never posted here, yet is still beating me...", "Solution_14": "iLord: 20\r\nZephyredx: 2\r\n1=2: 4\r\nmyyellowducky82: 4\r\nPerfect628: 2\r\ncrazychinesemanic: 26\r\nchenhsi: 5\r\nalexhhmun: 1 \r\nBudi713: 3\r\n\r\nNo rule against double posting but that would be cheap so i will not do that but is will still \r\nguess that budi713 is next giving me 3 points.", "Solution_15": "nick42", "Solution_16": "tinytim.", "Solution_17": "r15 ", "Solution_18": "dojo\r\n\r\n\r\n\r\ni think so", "Solution_19": "r15somethingsomethingsomething", "Solution_20": "[sarcasm]Good job, guys. [/sarcasm]You have revived a thread that already has a revival thread. Not only that, there are no such AoPS users, at least for the moment being, named \"r15\" or \"r15somethingsomethingsomething.\"", "Solution_21": "uhhh dojo?", "Solution_22": "Some guy on AoPS..", "Solution_23": "Good job King!. You would've been in trouble if it was a girl but luckily for you, I'm male.\nMy guess: Powerofpi", "Solution_24": "Hi. 1/2 year bump. Don't post in dead threads.", "Solution_25": "This is a very old thread. It died for a reason. Let dead threads lie in peace. Can this be locked, please?", "Solution_26": "It the fun factory so they (mods) dont ussually do that.\nDo you poeple just trawle though the old threads and randomly revive one?", "Solution_27": "Phire's usually pretty good at locking old threads. \n\nNew members usually don't know the revival rules (I know when I was new I revived a lot), and it looks like this guy is new, so we can cut him some slack.", "Solution_28": "islander7", "Solution_29": "12 year bump lol" } { "Tag": [ "geometry unsolved", "geometry" ], "Problem": "Can have a triangle the lenghts of sides 3k+1,3l+1,3m+1 or 3k+2,3l+2,3m+2 where $ l,m,n \\in Z$?", "Solution_1": "Nobody,nothing?", "Solution_2": "Yes. For example when $ k \\equal{} l \\equal{} m$ then the triangle is equilateral." } { "Tag": [ "AMC", "AIME" ], "Problem": "2001! For the first time, someone has attained this truly remarkable rating. After AIME's defeat when he had 2000, he slowly worked his way back to 2000, culminating his epic climb to the top with a series of 11 cds against hurdler! Congratulations AIME, a true hero!", "Solution_1": "Now can he get 2008?", "Solution_2": "Depends. I think hurdy logged out but other people might \"help the cause\".", "Solution_3": "Gee...this is weird! Somehow, once I reached 2000 again, RG comes on! And plays mewto a whole bunch of times to get to 2004!\r\n\r\nSomebody is a hypocrite...", "Solution_4": "Huh? When did i play rg? I was just cding myself a bunch. RG is still asleep right now...", "Solution_5": "And mewto has 2008 now.\r\nAIME15, something is weird (OK, we all know what is it)." } { "Tag": [], "Problem": "Prove that the remainder when any prime number is divided by 30 is either 1 or a prime number.", "Solution_1": "[hide]\nSuppose $ p\\equal{}30k\\plus{}n$, where $ 0\\leq n<30$. Since $ p$ is prime, we have $ 30\\not|p$, so $ n\\neq 0$. We also have $ \\gcd(30,n)\\equal{}1$, so $ n$ has no factors of $ 2,3,$ or $ 5$. But then, any such composite $ n$ is at least $ 7\\cdot 7\\equal{}49$, which is larger than 30, so $ n$ must be prime or $ 1$.[/hide]", "Solution_2": "In fact, you can say more: $ 30$ is the largest modulus with this property." } { "Tag": [ "superior algebra", "superior algebra unsolved" ], "Problem": "Is it true that for any $ n\\geqslant 3$, there exist $ x,y\\in D_{n}$ such that $ \\left|x\\right|\\equal{}\\left|y\\right|\\equal{}2$ and $ \\left|xy\\right|\\equal{}n$? If so, how do we prove it?", "Solution_1": "Think of the group as the symmetries of a regular $ n$-gon, and look at two adjacent reflections." } { "Tag": [ "calculus", "integration", "trigonometry", "logarithms", "calculus computations" ], "Problem": "I'm having trouble with this integral. I usually find calculus fairly easy (classroom calculus :D ) but I'm not getting anywhere with this problem.\r\n\r\nFind: $\\displaystyle \\int \\tan^2 x \\sec x \\ dx$\r\n\r\nI tried many things such as U-Substitution, using $\\tan^2 x = \\sec^2 x - 1$ as substitution but I'm not getting anywhere. Can anyone help me? I appreciate this. Thanks.", "Solution_1": "Rewrite the integrand as $\\sec^3x-\\sec x$, and integrate $\\sec^3x$ by parts.", "Solution_2": "$\\begin{eqnarray*} \\int \\tan^2 x \\, \\sec x \\, dx &=& \\int \\sec^3 x -\\sec x \\, dx \\\\ &=& \\int \\sec^3 x \\, dx - \\int \\sec x \\, dx \\\\ &=& \\sec x \\tan x - \\int \\tan^2 x \\sec x\\, dx - \\ln |\\sec x + \\tan x| +C$\r\n\r\nSo $2 \\int \\tan^2 x \\, \\sec x \\, dx = \\sec x \\tan x - \\ln |\\sec x + \\tan x |$, and $\\int \\tan^2 x \\, \\sec x \\, dx = \\frac{\\sec x \\tan x - \\ln |\\sec x + \\tan x |}{2}$", "Solution_3": "Yeah that's what I did.\r\n\r\nOnly thing was that we didn't learn that rule in the class and when we went over it, our substitute teacher just stopped and said, \"We'll go over it after the final.\"\r\n\r\nI don't know why the problem was assigned for Final Review when it was going to be discussed AFTER the final but yeah. Thanks for the help. :)" } { "Tag": [ "percent", "LaTeX" ], "Problem": "If there are 100 users of a website, and 20 are banned, what is the percentage of those who remain?", "Solution_1": "[hide=\"Too Easy?\"]$80\\%$[/hide]", "Solution_2": "[hide=\"easy\"]1 user=1% there are 100-20=80 users who remain.=> 80%[/hide]", "Solution_3": "[hide=\"solution\"]There are 20% banned, so 80% remain[/hide]", "Solution_4": "Hix very easy\r\n[hide] $80\\%$ [/hide]\r\n\r\n[size=75][color=green]Use \\% for percents\nLaTeX edited[/color][/size]", "Solution_5": "[hide=\"easy\"]20=20%, 100-20=80 :moose: [/hide]", "Solution_6": "[hide][quote=\"anirudh\"]If there are 100 users of a website, and 20 are banned, what is the percentage of those who remain?[/quote]\n\n4/5 self explanatory[/hide]", "Solution_7": "100-20=80\r\n80/100=80%", "Solution_8": "[b] 80%[/b][i][u] :10: [/u][/i]", "Solution_9": "Very easy! 80%", "Solution_10": "wow, VERY easy! \r\n80% !!\r\nyay!! :10:", "Solution_11": "easy 80%................" } { "Tag": [ "function", "trigonometry", "real analysis", "real analysis unsolved" ], "Problem": "Consider the function $ f(x) \\equal{} \\sin{\\frac {1}{x}}$ mapping the open interval $ (0,1)$ into $ \\mathbb{R}$. Prove that it is not uniformly continuous.", "Solution_1": "Two possible approaches:\r\n\r\n1. If it were uniformly continuous, it could be extended to a continuous function on the closure of that domain.\r\n\r\n2. More explicitly: can you find a sequence $ x_n$ such that $ |x_n\\minus{}x_{n\\plus{}1}|\\to 0$ but $ |f(x_n)\\minus{}f(x_{n\\plus{}1})|$ does not tend to zero?" } { "Tag": [ "geometry", "3D geometry", "tetrahedron", "octahedron", "icosahedron", "graph theory", "advanced fields" ], "Problem": "Hi, can anyone help me with the following question in graph theory?\r\n\r\nWe want to consider the planar embedding of the tetrahedron, octahedron, and icosahedron, the three platonic solids with triangular faces. We know that it is a planar graph because they are convex polyhedra.\r\n\r\nA [i]Hamiltonian path[/i] is a path that visits each vertex exactly once. A [i]traceable graph[/i] is a graph that contains a Hamiltonian path.\r\n\r\nThe problem is to show that [b]if $ T$ is a tree contained in a planar embedding $ G$ of these planar graphs, then we can always find some [i]traceable subgraph[/i] $ H$ of of $ G$ with $ V(H) \\equal{} V(T)$ and $ E(H) \\subseteq E(G).$[/b]\r\n\r\n[i]i.e., [/i] We want to find some traceable subgraph $ H$ such that it spans the vertices of $ T$ with edges contained in the edges of $ G$.\r\n\r\nMy strategy is to first construct the closure $ \\text{cl}(T)$ of $ T$ by adding for all nonadjacent pairs of vertices $ u$ and $ v$ with $ (u,v) \\in E(G)$ the new edge $ (u,v)$ to the set $ E(T).$ And then use some theorem to show that we can always find a Hamiltonian path.", "Solution_1": "Actually thinking about it, there is a simpler way to put it.\r\n\r\nShow that any connected subgraph of a planar embedding of these planar graphs is also a traceable subgraph.", "Solution_2": "I'm a little confused here, so maybe I'm misunderstanding.\r\n\r\n[quote=\"lenny\"]\nThe problem is to show that [b]if $ T$ is a tree contained in a planar embedding $ G$ of these planar graphs [/quote]\n\nWhat's the point of looking at planar embeddings? A tree being contained in a graph has nothing to do with if or how the graph is embedded in the plane.\n\n[quote] then we can always find some [i]traceable subgraph[/i] $ H$ of of $ G$ with $ V(H) \\equal{} V(T)$ and $ E(H) \\subseteq E(G).$[/b] [/quote]\r\n\r\nso let's say the tree $ T$ consists of a a vertex $ v$ of degree 3 along with its 3 neighbors $ u_1,u_2,u_3$ (which obviously exists in the cube, tetrahedron, icosahedron or any other cubic graph). You want a subgraph $ H$ whose vertex set is precisely $ \\{v,u_1,u_2,u_3\\}$, which implies that it's either disconnected or it's the whole tree $ T$. Obviously this $ T$ contains to spanning graph with a Hamiltonian path.", "Solution_3": "[quote]What's the point of looking at planar embeddings? A tree being contained in a graph has nothing to do with if or how the graph is embedded in the plane. [/quote]\n\nYou have the point, there isn't much advantage in doing this. I guess it just makes things looks easier by reducing the problem to planar graphs.\n\n\n[quote]which implies that it's either disconnected or it's the whole tree .[/quote]\n\nNo, I can add edges besides the edges in $ T$ (i.e., besides $ vu_1, vu_2, vu_3$.) to be included in $ H,$ as long as those edges are in $ G.$\n\n\n\n[quote]Obviously this $ T$ contains to spanning graph with a Hamiltonian path.[/quote]\r\n\r\nThis $ T$ does not contain a spanning graph with a Hamiltonian path. Do you mean the closure of $ T$ as I defined in the first entry? Even so, I don't see why it is obvious and will be happy to see your proof of it. =)", "Solution_4": "never mind. I just found an counter example." } { "Tag": [ "function", "geometric sequence" ], "Problem": "is there a closed form for\r\n$\\sum_{k=1}^{n} k*y^{k}$ for $y \\ne 1$\r\ni.e. a function that gives the same result as the summation but as a function of n and y", "Solution_1": "$S = y + 2y^2 + 3y^3 + ... + ny^n$\r\n$yS = y^2 + 2y^3 + 3y^4 + ... + (n-1)y^n + ny^{n+1}$\r\n\r\nSubtracting...\r\n\r\n$S - yS = y + y^2 + y^3 + ... + y^n - ny^{n+1}$\r\n$S(1 - y) = y(1 + y^2 + y^3 + ... y^{n-1}) - ny^{n+1}$\r\n$S(1 - y) = y\\frac{1 - y^n}{1-y} - ny^{n+1}$\r\n$S = y\\frac{1 - y^n}{(1 -y)^2} - \\frac{ny^{n+1}}{1 - y}$\r\n\r\nI hope there is no mistakes... im soo sleepy :sleeping:", "Solution_2": "$S_n = \\sum\\limits_{k = 1}^n {k \\cdot y^k = y \\cdot \\sum\\limits_{k = 1}^n {\\left[ {\\left( {k - 1} \\right) + 1} \\right]} \\cdot y^{k - 1} = } $\r\n\r\n$ = y \\cdot \\sum\\limits_{k = 1}^n {\\left( {k - 1} \\right) \\cdot y^{k - 1} } + y \\cdot \\sum\\limits_{k = 1}^n {y^{k - 1} } = $\r\n\r\n$ = y \\cdot \\sum\\limits_{k = 1}^{n - 1} {k \\cdot y^k + y \\cdot \\sum\\limits_{k = 1}^n {y^{k - 1} } = } $\r\n\r\n$ = y \\cdot \\sum\\limits_{k = 1}^n {k \\cdot y^k } - ny^{n + 1} + y \\cdot \\frac{{1 - y^n }}{{1 - y}} = $\r\n\r\n$ = y \\cdot S_n + \\frac{{n \\cdot y^{n + 2} - \\left( {n + 1} \\right) \\cdot y^{n + 1} + y}}{{1 - y}}$\r\n\r\n\r\n\r\nThen:\r\n$S_n = \\frac{{n \\cdot y^{n + 2} - \\left( {n + 1} \\right) \\cdot y^{n + 1} + y}}{{\\left( {1 - y} \\right)^2 }}$", "Solution_3": "$\\sum_{k=1}^{n} k*y^{k}$\r\n$y* \\sum_{k=1}^{n} k*y^{k-1}$\r\nthink of $D_y(y^k) = k*y^{k-1}$, since derivitives are distributive over additon,\r\n$y* D_y(\\sum_{k=1}^{n} y^{k})$ since this is a geometric sequence,\r\n$y* D_y(\\frac{y-y^{n+1}}{1-y})$, using the derivitative rule for quotients...\r\n$y* \\frac{ (1-y)(1-(n+1)y^{n}) - (y-y^{n+1})(-1)}{(1-y)^{2}}$implifying gives\r\n$\\frac{y(n*y^{n+1} - (n+1)y^{n}+1)}{(y-1)^{2}}$ \r\ni don't think that this simplifies...somebody already got this, but i thought that this was a pretty clever solution" } { "Tag": [ "geometry" ], "Problem": "The radius of a circle is 2 inches. When the radius is doubled,\nby how many square inches is the area increased? Express your \nanswer in terms of $ \\pi$.", "Solution_1": "The original circle's area is $ 2^2\\pi \\equal{} 4\\pi$\r\n\r\nThe new radius is $ 2*2 \\equal{} 4$\r\nIts area is $ 4^2\\pi \\equal{} 16\\pi$\r\nSo the area increased is $ (16\\minus{}4)\\pi \\equal{} \\boxed{12\\pi}$", "Solution_2": "$\\text{Radius doubled meaning the area increased by a factor of }4\\text{ (because a circle is }2\\text{D).}$\n\n$\\text{Original area is }2^2\\pi=4\\pi$\n\n$4-1=3$\n\n$3\\times (4\\pi)=\\boxed{12\\pi}$" } { "Tag": [ "complex numbers", "algebra unsolved", "algebra" ], "Problem": "Z is a unit root.PRoof that the real part of Z donnot have the form of (k+1)^1/2 -(K)^1/2", "Solution_1": "I once want to use trigonometric formulae to do the problem but not sucess", "Solution_2": "What do k , K mean?", "Solution_3": "I think he means the following problem (I remember seeing it in some MOSP homework):\r\n\r\nProve that the real part of a root of unity cannot have the form $ \\sqrt {k \\plus{} 1} \\minus{} \\sqrt {k}$ for an integer $ k > 1$.\r\n\r\n darij\r\n\r\n[color=red][See this : http://www.mathlinks.ro/viewtopic.php?t=113831 -mathmanman][/color]" } { "Tag": [ "modular arithmetic", "combinatorics open", "combinatorics" ], "Problem": "Determine if it's possible to arrange the numbers 1,1,2,2,...,n,n, such that there are j numbers between two j's, 1<=j<=n, when n=2000, n=2001, and n=2002.", "Solution_1": "Let $a_1,a_2,\\ldots, a_n$ be the position in the sequence of the first $1$, the first $2$, and so on, respectively. We have $1+2+\\ldots+2n=2(a_1+\\ldots+a_n)+(2+3+\\ldots+(n+1))$. This means that $n$ and $\\frac{(n+1)(n+2)}2$ have different parities. A quick case analysis reveals that this implies $n\\equiv 0$ or $3\\pmod 4$, so $2001,2002$ are out. \r\n\r\nI think such an arrangement exists for $n=2000$, but I don't really have the time to think about a construction right now.", "Solution_2": "Tanks Alex! Check this one: Using your idea,\r\n\r\nLet's say: a permutation is good if it satisfies the above conditions. In general, there is a good permutation of the numbers 1,1,2,2,...,n,n if and only if n=4k or 4k-1 for some positive integers k.First we show there is a good permutation of numbers 1,1,2,2,...,n,n if n=1 or 2 modulo 4. We aproach indirectly by assuming that a good permutation a_1, a_2,...,a_2n exists. For each number k, let (i_k, j_k),where i_k < j_k. Denote the positions of two occurrences of k.Then The sum of i_k + the sum j_k =1+2+...+2n=n(2n+1)=S_1 which is odd if n=1 (mod4) and even if n=2(mod 4). \r\nOn the other hand j_k - i_k = k+1, so \r\n........The sum of i_k + the sum j_k =2+...+(n+1)=n(n+3)/2 =S_2 which is even if n=1 (mod 4) and odd \r\nif n=2 (mod 4).Obtained 2(the sum of j_k = S_1 + S_2, an even number equal to an odd number, which is impossible..." } { "Tag": [ "function", "inequalities", "number theory", "prime numbers", "number theory unsolved" ], "Problem": "Find every positive integer $ n$ such that:\r\n\r\n$ 1. \\sigma (n) \\vdots n\\plus{}1$\r\n$ 2. n\\minus{}1 \\vdots \\varphi (n)$\r\n\r\nwhere $ \\varphi(n)$ is the Euler's function and $ \\sigma (n)$ is the sum of positive divisors of $ n$ including itself.", "Solution_1": "[quote=\"Mashimaru\"]Find every positive integer $ n$ such that:\n\n$ 1. \\sigma (n) \\vdots n + 1$\n$ 2. n - 1 \\vdots \\varphi (n)$\n\nwhere $ \\varphi(n)$ is the Euler's function and $ \\sigma (n)$ is the sum of positive divisors of $ n$ including itself.[/quote]\r\nDo you mean that n satisfy two conditions ? I will post my solution for this case . The first if $ n > 2$ ,from the condition it is not hard to show that $ n$ must be an odd square free number . \r\nLet $ n = \\prod_{i = 1}^{r} p_i$. We will show that $ r = 1$ (which means that only prime numbers satisfy these conditions ) .If not ,assume $ r\\req 2$ . Two conditions can be rewritten in the form :\r\n\\[ p_1...p_r + 1|(p_1 + 1)...(p_r + 1)\r\n\\]\r\n\r\n\\[ (p_1 - 1)...(p_r - 1)|p_1...p_r-1\r\n\\]\r\nBecause $ r\\geq 2$ so $ 4|p_1...p_r - 1$ ,therefore $ 2\\parallel{}p_1...p_r + 1$ . Because all $ p_$ are greater than 2 so we can write $ p_i + 1 = 2t_i$ and $ t_i\\geq 1$ . \r\nThe condition i means that $ p_1...p_r + 1|2^r.\\prod_{i = 1}^rt_i$ .By the fact that $ 2\\parallel{}p_1...p_r + 1$ we have $ p_1...p_r + 1|2t_1...t_r$ or $ (2t_1 - 1)...(2t_r - 1)\\leq 2t_1...t_r$ .This inequality is equivalent to $ (2 - \\frac {1}{t_1})...(2 - \\frac {1}{t_r})\\leq 2$ . \r\nBecause $ r\\geq 2$ and $ t_i\\geq 2$ so $ \\prod_{i = 1}^r (2 - \\frac {1}{t_i})\\geq (\\frac {3}{2})^r > 2$ \r\nThe converse is obvious because all prime numbers satisfy two conditions . \r\nHence solutions are all prime number .", "Solution_2": "[quote=\"TTsphn\"][quote=\"Mashimaru\"]Find every positive integer $ n$ such that:\n\n$ 1. \\sigma (n) \\vdots n \\plus{} 1$\n$ 2. n \\minus{} 1 \\vdots \\varphi (n)$\n\nwhere $ \\varphi(n)$ is the Euler's function and $ \\sigma (n)$ is the sum of positive divisors of $ n$ including itself.[/quote]\nDo you mean that n satisfy two conditions ? I will post my solution for this case . The first if $ n > 2$ ,from the condition it is not hard to show that $ n$ must be an odd square free number . \nLet $ n \\equal{} \\prod_{i \\equal{} 1}^{r} p_i$.[/quote]\r\nSorry for my stupidness, but can you explain more clearer about the above property? :maybe:", "Solution_3": "If $ n$ has a divisor of form $ p^m$ and $ m>1$ then $ p|\\varphi(n)$ but $ p\\not |n\\minus{}1$", "Solution_4": "Yes, I meant the two statements both have to be satisfied. This is a problem of teacher \u0110\u1eb7ng H\u00f9ng Th\u1eafng.\r\n\r\n@[b]TTsphn[/b]: Are you Mr. Th\u1ecd T\u00f9ng? If not, I am sorry. If right, since you are very good at Number Theory and I am very bad at this kind, could you please share with me some exp of yourself :blush: ?", "Solution_5": "[i]A good solution , Tung . Congratulation :lol: [/i]", "Solution_6": "What is exactly the meaning of \r\n\r\n$ \\vdots$\r\n\r\n?", "Solution_7": "[quote=\"Ronald Widjojo\"]What is exactly the meaning of \n\n$ \\vdots$\n\n?[/quote]\r\nIt means \"is divisible by\"." } { "Tag": [ "calculus", "integration", "trigonometry", "limit", "calculus computations" ], "Problem": "The sequence $\\{a_{n}\\}$ is defined as follows. \\[a_{1}=\\frac{\\pi}{4},\\ a_{n}=\\int_{0}^{\\frac{1}{2}}(\\cos \\pi x+a_{n-1})\\cos \\pi x\\ dx\\ \\ (n=2,3,\\cdots)\\] Find $\\lim_{n\\to\\infty}a_{n}$.", "Solution_1": "We have $a_{n}=\\int_{0}^{1/2}\\cos^{2}\\pi x \\ dx+\\int_{0}^{1/2}a_{n-1}\\cos \\pi x \\ dx = \\frac{1}{4}+a_{n-1}\\frac{1}{\\pi}$\r\n\r\nAfter looking at a few values of $n$, we see that \r\n\r\n$a_{n}=\\frac{1}{4}\\left(\\frac{\\pi^{n-2}+\\pi^{n-3}+...+\\pi+2}{\\pi^{n-2}}\\right) = \\frac{1}{4}\\left(1+\\pi^{-1}+\\pi^{-2}+...+\\pi^{-(n-2)}+\\frac{2}{\\pi^{n-2}}\\right)$\r\n\r\n$\\lim_{n\\to\\infty}a_{n}= \\frac{1}{4}\\left(\\frac{1}{1-\\frac{1}{\\pi}}\\right) = \\frac{\\pi}{4(\\pi-1)}$", "Solution_2": "Your answer is correct. :)" } { "Tag": [ "articles", "Support", "LaTeX" ], "Problem": "i type \r\n[color=red]\n\n\\documentclass[11pt]{article}\n\\begin{document}\nHello, world!\n\\end{document}[/color]\r\n\r\nand compile, but when i open 'Hello.dvi' , i can only see some quotation marks\r\nthe basic page told me that i should see [color=red]Hello, world![/color]\r\n\r\ncan anyone tell me what the problem is?\r\n\r\n[i][ Admin Edit please do not use [size=184]this size[/size] of font when asking support ][/i]", "Solution_1": "first of all, use normal characters to display your problems. \r\nsecond of all, I just pasted that code into a new .tex file and compiled it, and I obtained a \"hello world!\" text on the .dvi file, so I can only assume that either \r\n\r\n1) you haven't compiled it correctly \r\n2) you haven't installed miktex correctly\r\n\r\nplease check for both.", "Solution_2": "[quote=\"Valentin Vornicu\"]\n1) you haven't compiled it correctly \n2) you haven't installed miktex correctly\n\n[/quote]\r\n\r\nthat takes time. i think i would rather do that later, coz my PC is very slow.\r\n\r\ni want to ask u a question:\r\nit seems that u are here everywhere (maybe becoz of your face)\r\nhow long do u stick online per day??\r\n\r\nwell, i made the fonts larger becoz i am still not used to read so many small english at a time. i simply didnt want anyone to get tired, esply for your eyes.", "Solution_3": "writing normally doesn't tire anyone. however, changing the color of the text in red, and making it larger, make tires me (at least). so please, do not do that again. \r\n\r\nre-install your latex, and re-read the documentation carefully. these two should do the trick ;)" } { "Tag": [ "calculus", "integration" ], "Problem": "How many two-digit positive integers $ N$ have the property that the sum of $ N$ and the number obtained by reversing the order of the digits of $ N$ is a perfect square?\r\n\r\n$ \\textbf{(A)}\\ 4\\qquad\r\n\\textbf{(B)}\\ 5\\qquad\r\n\\textbf{(C)}\\ 6\\qquad\r\n\\textbf{(D)}\\ 7\\qquad\r\n\\textbf{(E)}\\ 8$", "Solution_1": "[hide]If the first number is ab, then the reversal is ba, so the sum is 10a+b+10b+a=11a+11b=11(a+b). For this to be a perfect square, a+b must equal 11* a perfect square. But this means it must equal 11, for 11*4=44, which is too large for a sum of digits. So a+b=11. It is easy to check that the possibilities for N are 92, 83, 74, 65, 56, 47, 38, 29, so there are 8 possibilities. E.[/hide]", "Solution_2": "[hide]$10x+y+10y+x=11x+11y$. So $11(x+y)=k^2$ so therefore, $x+y=11$, so you can have $29, 38, 47, 56, 65, 74, 83, 92$.[/hide]\r\n\r\nBy the way, how do you add problems to the AMC archive thing? Do you just post the problems and give the year, etc, and the moderators do it for you?", "Solution_3": "Guys, please hide your answers.\r\n\r\nAs for your question, only specific moderators who were given permissions and administrators are allowed to add to the archives. I don't think you should simply post random problems and ask moderators to put them in the archive.\r\n\r\nWhat I think is a good idea is if you have years below, send the PDF or HTML document to me and I'll do the job. Please send the choices and exact wording if possible.\r\n\r\nYears 1990 through 1994\r\nYears 1950 through 1979", "Solution_4": "[hide]\n\nLet $ N\\equal{}10a\\plus{}b$, where $ a$ and $ b$ have only one digit. So, the sum of $ N$ and the reverse of $ N$ is $ 10a\\plus{}b\\plus{}a\\plus{}10b\\equal{}11a\\plus{}11b\\equal{}11(a\\plus{}b)$. Clearly, this is a square if $ a\\plus{}b\\equal{}11x^2$ for some integer $ x$. However, if $ x\\equal{}2$, $ 4(11)\\equal{}44$ is too large to obtain with the sum of two single digit numbers, so $ a\\plus{}b\\equal{}11$. There are $ 8$ ways of doing this. $ \\mathrm{ (E) \\ }$ [/hide]", "Solution_5": "[hide=\"Solution\"]$ N\\equal{}\\overline{ab}\\equal{}10a\\plus{}b$.\n$ 10a\\plus{}b\\plus{}10b\\plus{}a\\equal{}11a\\plus{}11b\\equal{}11(a\\plus{}b)$.\nThis is a perfect square iff $ a\\plus{}b\\equal{}11$ or $ a\\plus{}b\\equal{}11n^2$.\n$ a\\plus{}b\\equal{}11$ has $ 8$ solutions.\nThat's already $ 8$, so the answer is clearly $ \\boxed{\\textbf{(E)}\\ 8}$.\n\nHowever, for the sake of completeness...\n$ a\\plus{}b\\equal{}11n^2$ for $ n\\in\\mathbb{Z}$, $ n>1$.\nSuppose $ n\\equal{}2$. $ a\\plus{}b\\equal{}44$ has no integral solutions $ 10\\leq\\overline{ab}\\leq99$. Clearly, for $ n>1$, there are no good solutions, so we are done.[/hide]" } { "Tag": [ "group theory", "abstract algebra", "analytic geometry", "vector", "algebra", "function", "domain" ], "Problem": "Hi all,\r\n\r\nI'm an incoming high school senior, and took abstract algebra this past year. Obviously this isn't really general high school curiculum and it was only the second time that the teacher had taught the class. Anyway, I was working on isomorphism and there was something in the book about the complex plane with zero removed (punctured plane) being isomorphic to the group of all complex numbers with a magnitude of one (the unit circle). I have searched and searched for the proof to this and was wondering if anyone here had an idea. I have found other sources saying that they are isomorphic and the book even gave reference to the journal / edition in which the proof was published.\r\n\r\nSo it seems as though this isomorphism does exist, however elusive that it is. I also worked with my teacher for a few days trying to come up with the proof on our own, and had no such luck. It just doesn't make sense simply because the unit circle has one degree of freedom and the punctured plane has two, so it becomes very complicated trying to figure out how to create a one to one mapping.\r\n\r\nAnyway, if anyone can help, it would be much appreciated.\r\n\r\nChris", "Solution_1": "Some hints.\r\n\r\nLet $\\mathbb C^{*}=\\mathbb C\\setminus 0$ and let $U=\\{z\\in\\mathbb C\\colon \\mid z\\mid =1\\}$. \r\n\r\n1) Prove that $\\mathbb C^{*}\\cong U\\times\\mathbb R^{+}$, where $\\mathbb R^{+}$ is the group of positive reals with multiplication.\r\n\r\n2) Prove that $\\mathbb R^{+}\\cong(\\mathbb R,+)$ (reals with addition).\r\n\r\n3) Prove that $U\\cong(\\mathbb R/\\mathbb Z,+)$.\r\n\r\n4) Prove that $\\mathbb R=\\mathbb Q\\oplus\\Bigl(\\bigoplus_{i\\in I}A_{i}\\Bigr)$, where each subgroup $A_{i}$ is isomorphic to $\\mathbb Q$.\r\n\r\n5) Prove that $\\mathbb R/\\mathbb Z\\cong\\mathbb Q/\\mathbb Z\\oplus\\Bigl(\\bigoplus_{i\\in I}A_{i}\\Bigr)$\r\n\r\n6) Use the above results to prove that $\\mathbb C^{*}\\cong U$.", "Solution_2": "There's a more simple way.\r\n \r\nThe homomorphism theorem tells us $S^{1}\\cong \\mathbb{R}/ \\mathbb{Z}$, and the polar coordinates show $\\mathbb{C}^{*}\\cong \\mathbb{R}\\times \\mathbb{R}/ \\mathbb{Z}$. So, our aim is $\\mathbb{R}/ \\mathbb{Z}\\cong \\mathbb{R}\\times \\mathbb{R}/ \\mathbb{Z}$. Since $\\mathbb{R}$ is a infinite-dimensional vector space over $\\mathbb{Q}$, the vector spaces $\\mathbb{R}$ and $\\mathbb{R}\\times \\mathbb{R}$ are isomorphic. We can even find an isomorphism $f$ mapping $1$ to $(1,0)$ (basis extension etc.). Then $g : \\mathbb{R}/ \\mathbb{Z}\\to \\mathbb{R}\\times \\mathbb{R}/ \\mathbb{Z}, r+\\mathbb{Z}\\to (y,x+\\mathbb{Z}) ,~ f(r)=(x,y)$ is an isomorphism, which can be easiliy checked.\r\n\r\nBesides, a similiar proof shows $\\mathbb{R}/ \\mathbb{Z}\\cong \\mathbb{R}\\times \\mathbb{Q}/ \\mathbb{Z}$. Of course, these isomorphisms aren't continuous.", "Solution_3": "Thank you for the quick response.\r\n\r\nIt seems to me that lofar's way is pretty much a round about way of doing -oo-'s way, but anyway, that all makes sense (its 4:30 in the morning and im about to get on a plane, so I am not going to try to work through the proofs at the moment) except that how can you properly make the isomorphism from R to R x R? (sorry, I don't know mathBB) It seems like any mothed that you would use wouldn't be able to produce a 1-1 corrolation.\r\n\r\nI have to go, but thanks a lot for your response\r\n\r\nChris", "Solution_4": "It is well-knwon that $\\kappa+\\kappa = \\kappa$ for infinite cardinals $\\kappa$. So, if $V$ is a infinite-dimensional vector space, $\\dim(V \\times V)=\\dim(V)+\\dim(V) = \\dim(V)$ and hence $V \\times V \\cong V$.", "Solution_5": "Sorry my message got cut off, I had to leave for the airport...\r\n\r\nThis makes sense in proof, but I am trying to understand it in practice. I have never really had a great concept of infinity as it is mostly counter intuitive at first glance. Anyway it seems that if you have a one dimmensional set of real numbers it will have an infite order, and the same is true for a two dimmensional, but if you take any given sample domain there will always be twice as much data in the two dimmensional set as the one. I am just trying to understand how this mapping can take place such that you can say that R is isomorphic to R^2. If this seems a little trivial or whatnot, its fine, don't bother to respond, but if you have the time I would very much appreciate it.\r\n\r\nThanks\r\n\r\nChris" } { "Tag": [ "MATHCOUNTS" ], "Problem": "What is the remainder when 9^1995 is divided by 7?", "Solution_1": "[hide] we can change this into\n\n9^1995 (mod 7) which is\n\n2^1995 (mod7)which is\n\n8^665 (mod 7) which is\n\n1^665 (mod 7) which is\n\n1.[/hide]", "Solution_2": "lol, goode job to both of you" } { "Tag": [ "linear algebra", "matrix", "linear algebra unsolved" ], "Problem": "Here's a really nice problem: what is the maximal number of 1 in a nilpotent order $n$ matrix over the reals?", "Solution_1": "The answer is n^2-2 times \"1\" (incredible!)\r\nWe take A=[Aij] with Aij=1 except Ann=-n+1 (trace(A)=0) and A1,n=z.\r\nThen A^3=(z+n^2-n-1)A and A^2<>0 for all z.\r\nThus A is nilpotent iff z=-n^2+n+1.\r\nWe cannot do better than n^2-2 because trace(A)=0 and because the precedent \"iff\".", "Solution_2": "Nice example loup blanc.\r\n\r\nHowever I understand harazi's question, he asked for \"nilpotent order $n$ matrix\", [i]i.e.[/i] $A^n=0$, but $A^{n-1}\\not=0$.\r\n\r\nThen this example does not answer the question :?:", "Solution_3": "Hi JC_math,\r\nperhaps you are right, but your problem seems more difficult.\r\nHarazi, what is really the question?", "Solution_4": "Both of them. ;) Really, the first one I intended to put on the site, since it was the one I could solve. But indeed, the second one is much more interesting, so why not asking it too?", "Solution_5": "Now we want also A^(n-1)<>0.\r\nMy preceding proof shows that if n=2,3,4 the answers are 2,7,<=14.\r\nLet n=4. One cannot carry out the value 14:\r\n2 cases are considered (with Maple):\r\na) Exactly one of the Aij<>1 is on the diagonal. Then if A^n=0 then A^(n-1)=0.\r\nb) The Aij<>1 are both on the diagonal. Then A is never nilpotent.\r\nFact: the answer is 13 as the following example shows:\r\nA is a n,n matrix s.t. aij=1 except \r\nAnn=-n+1;A1,n-1=-n^2+2n+1; An-1,n=-n+1.\r\nThen A is nilpotent and A^3<>0.\r\n Perhaps the general answer is n^2-n+1 ????? (it's true for n=3,4 thus it's true for all n)." } { "Tag": [ "geometry", "circumcircle", "trigonometry", "power of a point", "radical axis", "geometry proposed" ], "Problem": "Let $ ABC$ be a triangle and $ A_1$, $ B_1$, $ C_1$ the mid-points of the sides $ BC$, $ CA$, $ AB$, respectively. Let $ H$ be the foot of altitude dropped from $ A$, and $ K$ the second intersection of the circumcircles of triangles $ BHC_1$, $ CHB_1$. Prove that $ AK$ is the $ A$-symmedian line of triangle $ ABC$.", "Solution_1": "[quote=\"April\"]Let $ ABC$ be a triangle and $ A_1$, $ B_1$, $ C_1$ the mid-points of the sides $ BC$, $ CA$, $ AB$, respectively. Let $ H$ be the foot of altitude dropped from $ A$, and $ K$ the second intersection of the circumcircles of triangles $ BHC_1$, $ CHB_1$. Prove that $ AK$ is the $ A$-symmedian line of triangle $ ABC$.[/quote]\r\nNice problem!\r\n[hide=\"Solution\"]\nNotice that\n\\[ \\angle C_1KB_1 \\equal{} 360 \\minus{} \\angle C_1KH \\minus{} \\angle B_1KH \\equal{} 360 \\minus{} (180 \\minus{} \\angle ABC) \\minus{} (180 \\minus{} \\angle ACB) \\equal{} \\angle ABC \\plus{} \\angle ACB \\equal{} 180 \\minus{} \\angle B_1AC_1\n\\]\nso it follows that $ C_1KB_1A$ is cyclic. Now, since $ C_1$ is the midpoint of the hypotenuse of right $ \\triangle AHC$, we see that $ HC_1 \\equal{} C_1C$. Similarly, $ HB_1 \\equal{} B_1B$. Now, since $ B_1$, $ C_1$, and $ A_1$ are midpoints of $ AC$, $ AB$, and $ BC$, we have that $ B_1C_1\\parallel BC$, $ A_1C_1\\parallel AC$, and $ A_1B_1\\parallel AB$. Therefore, $ \\angle CKC_1 \\equal{} \\angle C_1HC \\equal{} \\angle ACB \\equal{} \\angle AC_1B_1 \\equal{} \\angle AKB_1$. Furthermore, $ AC_1KB_1$ is cyclic, so $ \\angle KC_1C \\equal{} \\angle AB_1K$, so $ \\triangle AKB_1\\sim \\triangle CKC_1$. Since $ AC_1 \\equal{} CC_1$ and $ AB_1 \\equal{} BB_1$, we have that\\[ \\triangle AKB\\sim \\triangle CKA\\implies \\frac {AK}{CK} \\equal{} \\frac {AB}{AC} \\equal{} \\frac {AB_1}{B_1A_1}\\]Furthermore,\\[ \\angle AB_1A_1 \\equal{} 180 \\minus{} \\angle BAC \\equal{} \\angle AB_1C_1 \\plus{} \\angle AC_1B_1 \\equal{} \\angle AKC_1 \\plus{} \\angle ACB \\equal{} \\angle AKC_1 \\plus{} \\angle C_1HC \\equal{} \\angle AKC_1 \\plus{} \\angle C_1KC \\equal{} \\angle AKC\\]Thus, it follows that $ \\triangle AB_1A_1\\sim \\triangle AKC$ and so $ \\angle A_1AB \\equal{} \\angle KAC$, which means that $ AK$ the $ A$-symmedian of $ \\triangle ABC$. [/hide]", "Solution_2": "Nice solution too. I have a different approach. \r\n\r\nLet $ A^{\\prime}B^{\\prime}C^{\\prime}$ be the tangential triangle of $ ABC$ (i.e. $ A^{\\prime}$ is the intersection of the tangents at $ B$, and $ C$ to the circumcircle of $ ABC$, and so on). It is known that $ AA^{\\prime}$ is the $ A$-symmedian of triangle $ ABC$, and therefore it is suffice to prove that the second intersection $ K$ of circumcircles of triangles $ BHC_{1}$ and $ CHB_{1}$ lies on the line $ AA^{\\prime}$.\r\n\r\nConsider the inversion $ \\Psi$ with pole $ O$, the circumcenter of $ ABC$, with power $ R^{2}$, where $ R$ is the circumradius of triangle $ ABC$. The images of the midpoints $ A_{1}$, $ B_{1}$, $ C_{1}$ are $ A^{\\prime}$, $ B^{\\prime}$, $ C^{\\prime}$, and the image $ H_{i}$ of $ H$ under $ \\Psi$ is the second intersection of the circumcircle of triangle $ A^{\\prime}BC$ with the circumcircle of $ A^{\\prime}B^{\\prime}C^{\\prime}$. Hence, the circles $ BHC_{1}$ and $ CHB_{1}$ are mapped into the circles $ BH_{i}C^{\\prime}$, and $ CH_{i}B^{\\prime}$, and since $ H_{i}$ is the Miquel point of the quadrilateral $ BCB^{\\prime}C^{\\prime}$ (this is because the circumcircles of triangles $ A^{\\prime}BC$ and $ A^{\\prime}B^{\\prime}C^{\\prime}$ intersect for the second time at $ H_{i}$), we conclude that the circumcircles of $ BH_{i}C^{\\prime}$, and $ CH_{i}B^{\\prime}$ pass through the intersection of the lines $ BC$ and $ B^{\\prime}C^{\\prime}$, i.e. the intersection $ T$ of the tangent at $ A$ to the circumcircle of $ ABC$ with the sideline $ BC$. On other hand, $ T$ is the image of $ K$ under $ \\Psi$ (since $ AA^{\\prime}$ is mapped into the circle $ AOA_{1}T$, and circle $ A^{\\prime}BC$ into the line $ BC$), and therefore $ K$ lies on the line $ AA^{\\prime}$. This proves our problem.", "Solution_3": "Thank you dear [b][size=100]April[/size][/b] for a nice result, new to me.\r\n\r\n$ \\bullet$ It is easy to show that the points $ A,\\ C_{1},\\ K,\\ O,\\ B_{1},$ where $ O$ is the circumcenter of $ \\bigtriangleup ABC$ are concyclic and let $ (O_{1})$ be, their circumcircle.\r\n\r\n$ ($ From $ OB_{1}\\perp AC$ and $ OC_{1}\\perp AB$ $ \\Longrightarrow$ $ AC_{1}OB_{1}$ is cyclic with diameter $ AO$ and from\r\n\r\n$ \\angle AB_{1}K \\plus{} \\angle AC_{1}K \\equal{} \\angle KHC \\plus{} \\angle KHB \\equal{} 180^{o}$ $ \\Longrightarrow$ $ AC_{1}KB_{1}$ is cyclic and so, $ AK\\perp KO$ $ ).$\r\n\r\nWe will prove now that the quadrilateral $ BKOC,$ is also cyclic.\r\n\r\nLet be the point $ D\\equiv (O_{1})\\cap CO$ and because of $ \\angle KOD \\equal{} \\angle KB_{1}D$ $ ,(1)$ it is enough to prove that $ \\angle KB_{1}D \\equal{} \\angle KBC.$\r\n\r\n$ \\angle KB_{1}D \\equal{} \\angle KB_{1}A \\minus{} \\angle AB_{1}D \\equal{} \\angle KB_{1}A \\minus{} \\angle ACO \\minus{} \\angle B_{1}DO$ = $ \\angle KB_{1}A \\minus{} 2\\angle CAO$ $ \\Longrightarrow$ $ \\angle KB_{1}D \\equal{} \\angle KB_{1}A \\minus{} 2\\angle CAO$ $ ,(2)$\r\n\r\n$ \\angle KBC \\equal{} \\angle KHC \\minus{} \\angle BKH \\equal{} \\angle KB_{1}A \\minus{} \\angle BC_{1}H \\equal{} \\angle KB_{1}A \\minus{} 2\\angle BAH$ $ \\Longrightarrow$ $ \\angle KBC \\equal{} \\angle KB_{1}A \\minus{} 2\\angle BAH$ $ ,(3)$\r\n\r\nFrom $ (2),$ $ (3)$ and because of $ \\angle CAO \\equal{} \\angle BAH,$ we conclude that $ \\angle KB_{1}D \\equal{} \\angle KBC$ $ ,(4)$\r\n\r\nFrom $ (1),$ $ (4)$ $ \\Longrightarrow$ $ \\angle KOD \\equal{} \\angle KBC$ and so, we have that $ BKOC$ is cyclic and let $ (O_{2})$ be, its circumcircle.\r\n\r\n$ \\bullet$ In the configuration now, of the three circles $ (O),\\ (O_{1}),\\ (O_{2}),$ we have that the tangent line of $ (O)$ at point $ A$ and the line segments $ KO,\\ BC,$ as their radical axis, taken per two of them, are concurrent at one point, so be it $ S.$\r\n\r\nWe denote as $ A',$ the intersection point of the tangent lines of $ (O)$ at points $ B,\\ C$ and so, the line segment $ AA',$ is the $ A$-symmedian of $ \\bigtriangleup ABC,$ as well and it is enough to prove that $ AK\\equiv AA'.$\r\n\r\nBut it is true, because of the line segment $ AK,$ as the polar of $ S$ with respect to $ (O)$ $ ($ because of $ SA$ tangents to $ (O)$ at $ A$ and $ SO\\perp AK$ $ ),$ passes through the point $ A',$ since the line segment $ BC,$ as the polar of $ A'$ wrt $ (O),$ passes through the point $ S.$\r\n\r\nHence, the line segment $ AK$ is the $ A$-symmedian of $ \\bigtriangleup ABC$ and the proof is completed.\r\n\r\n$ \\bullet$ This proof is dedicated to [b][size=100]Dimitris Papadimitriou[/size][/b].\r\n\r\nKostas Vittas.", "Solution_4": "[quote=\"April\"][color=darkred]Let $ ABC$ be a triangle and denote the midpoints $ M$ , $ N$ of the sides $ [AB]$ , $ [AC]$ respectively.\n\nLet $ D\\in BC$ be the point for which $ AD\\perp BC$ . Denote be the second intersection $ P$ between \n\nthe circumcircles of $ \\triangle BMD$ , $ \\triangle CND$ . Prove that the point $ P$ belongs to the $ A$-symmedian in $ \\triangle ABC$.[/color][/quote]\n\n[color=darkblue][b][u]Proof.[/u][/b] Denote $ S\\in AP\\cap BC$ . Prove easily that the quadrilateral $ AMPN$ is cyclically (or it is well-known).\n\nThus, $ \\{\\begin{array}{ccccccc} \\|\\begin{array}{c} \\widehat {MPB}\\equiv\\widehat {MDB}\\equiv\\widehat {ABC} \\\\\n \\\\\n\\widehat {MPA}\\equiv\\widehat {MNA}\\equiv\\widehat {ACB}\\end{array}\\| & \\implies & \\frac {MB}{MA} = \\frac {PB}{PA}\\cdot\\frac {\\sin\\widehat {MPB}}{\\sin\\widehat {MPA}} & \\implies & \\frac {PB}{PA} = \\frac cb \\\\\n \\\\\n\\|\\begin{array}{c} \\widehat {NPC}\\equiv\\widehat {NDC}\\equiv\\widehat {ACB} \\\\\n \\\\\n\\widehat {NPA}\\equiv\\widehat {NMA}\\equiv\\widehat {ABC}\\end{array}\\| & \\implies & \\frac {NC}{NA} = \\frac {PC}{PA}\\cdot\\frac {\\sin\\widehat {NPC}}{\\sin\\widehat {NPA}} & \\implies & \\frac {PC}{PA} = \\frac bc\\end{array}\\|$ $ \\implies$\n\n$ \\boxed {\\ \\frac {PB}{PC} = (\\frac cb)^2\\ }$ . But $ \\{\\begin{array}{c} m(\\widehat {SPB}) = 180^{\\circ} - m(\\widehat {BPM}) - m(\\widehat {MPA}) = 180^{\\circ} - B - C = A \\\\\n \\\\\nm(\\widehat {SPC}) = 180^{\\circ} - m(\\widehat {CPN}) - m(\\widehat {NPA}) = 180^{\\circ} - C - B = A\\end{array}\\|$ $ \\implies$ \n\n$ \\widehat {SPB}\\equiv\\widehat {SPC}$ . Therefore, $ \\frac {SB}{SC} = \\frac {PB}{PC}$ $ \\implies$ $ \\boxed {\\ \\frac {SB}{SC} = (\\frac cb)^2\\ }$ , i.e. the line $ AP$ is the $ A$ - symmedian in $ \\triangle ABC$ .[/color]\n\n[quote=\"Virgil Nicula\"][color=darkred][b][u]An easy extension.[/u][/b] Let $ ABC$ be a triangle and denote the midpoints $ M$ , $ N$ of the sides $ [AB]$ , $ [AC]$ respectively.\n\nFor a point $ D\\in BC$ denote $ \\|\\begin{array}{c}\nm(\\widehat {BDM})=x\\\\\\\nm(\\widehat {CDM})=y\\end{array}$ , the second intersection $ P$ between the circumcircles \n\nof $ \\triangle BMD$ , $ \\triangle CND$ and $ S\\in AP\\cap BC$ . Prove that $ \\frac {SB}{SC}=(\\frac cb)^2\\cdot \\frac {\\cot C+\\cot x}{\\cot B+\\cot y}$ .[/color][/quote]", "Solution_5": "It can be solved with inversion also.\r\nLet $ P$ be the second intersection point of $ AB_1C_1$'s circumcircle and symmedian from $ A$. We will show that $ BHPC_1$ and $ CHPB_1$ are circumscribable. Make an inversion centered at $ A$ with radius $ \\sqrt {AB\\cdot AC}$. (inversion turnes point $ X$ to $ X'$) Now $ B'$, $ C'$ and $ P'$ are respectively the midpoints of $ AC_1'$, $ AB_1'$ and $ B_1'C_1'$. Also $ P'$ is the second intersection of $ AB'C'$'s circumcircle and the line that passes through $ A$ and $ AB'C'$'s circumcenter. Therefore $ P'$ is the circumcenter of $ AC_1'B_1'$. $ B'H'P'C_1'$ and $ C'H'P'B_1'$ are circumscribable. So are $ BHPC_1$ and $ CHPB_1$.", "Solution_6": "See [url=http://www.mathlinks.ro/viewtopic.php?t=201048][color=red][b]here[/b][/color][/url] !" } { "Tag": [ "trigonometry" ], "Problem": "Ok, well there haven't been any new problems for a little while, so here's some ones that are semi-intermediate (easier) from the Furman Senior test:\r\n\r\n1. What is the thousands' digit of 101^29?\r\n\r\n2. Four positive integers a, b, c, d are given. There are exactly 4 distinct ways to choose 3 of a, b, c, d. The mean of each of the 4 possible triples is added to the fourth integers, and the four sums 29, 23, 21, 17 are obtained. What are the original integers? \r\n\r\n3. Sin(a)=2/3 and Sin(b)=3/4 and a is in Quadrant II and B is in Quadrant IV, then sin(a - b) = ?\r\n\r\n4. The four digit number 2pqr is multiplied by four and the result is the 4 digit number rqp2. It follows that p+q is...", "Solution_1": "[hide] 3. since sin(a-b) = sin(a+(-b)) we can use the identity to get:\n\n\n\nsin(a)cos(-b) + sin(-b)cos(a) and because cos(-x) = cos(x) and sin(-x) = -sin(x) we get:\n\n\n\nsin(a)cos(b) + -sin(b)cos(a) = (2/3)( :sqrt: (7)/4) - (3/4)( :rt5: /3) = :sqrt: (7)/6 - :rt5: /4.[/hide]", "Solution_2": "[hide] 4. \n\nFirst, we know that r = 8, because 9*4 (mod 10) is not 2.\n\n\n\nx is the floor value function.\n\n\n\nSo, we set up these equations for p and q. (4q+3)(mod 10) = p and 4p + [[(4q+3)/10]] = q. \n\n\n\nSolving the second equation for p we get p = q - [[(4q+3)/10]]. Plug back into first equation and get:\n\n\n\n(4q+3)(mod 10) = q - [[(4q+3)/10]]\n\n\n\nBy the \"Theory of Guess-and-Check\" we find that 7 works. Plug it back into the first equation we get p = 1.\n\n\n\nTherefore, p+q = 8.[/hide]", "Solution_3": "1. [hide]i think the hundredth grows by 1 each time, thus 101^29 ends with 2901. thousands digit is 2.[/hide]\n\n2. [hide]adding up the means of 3 and the last, we get each integer twice , thus their sum is 45. multiplying each equation by 3 then derving everything is terms of 'a' for example, we get a=21 (if a is biggest). then b=12, c=9 and b=3.[/hide]", "Solution_4": "1. I disagree - [hide]thousands digit is always 0?[/hide]\n\n\n\nUpdate: I'm an idiot. This is just further reaffirmation... I should've started getting suspicious when it started getting asymmetric at the 5th power.", "Solution_5": "hmm no try 101^12 ends in [hide]1201, use comp calculator if you don't belive me.[/hide]", "Solution_6": "Ml2=1 and 2 are right.\r\n\r\nPaladin8: You got #4 right (Nice solution, btw. I did it differently.)\r\nI don't think you got number three right...well maybe you did...could you rationalize that please?", "Solution_7": "well its rationalized but on a common denominator it would be:\n\n[hide] (4 :sqrt:7 - 6 :rt5:)/24 [/hide]", "Solution_8": "That's incorrect. Try drawing triangles in those quadrants and see what you get. Besides, I think you used the wrong identity...it's sin(a - b) = sin(a)cos(b) - sin(b)cos(a).", "Solution_9": "MagnusRex wrote:3. Sin(a)=2/3 and Sin(b)=3/4 and a is in Quadrant II and B is in Quadrant IV, then sin(a - b) = ?\n\n\n\nPerhaps you meant sin(b) = -3/4? If b is in Quad IV, sines are negative.\n\n\n\nAnyhow I got [hide]cos(a) = - :rt5: /3, cos(b) = :sqrt:7/4 . \n\n\n\nsin(a-b) = (2 :sqrt:7 - 3 :rt5: ) / 12 [/hide]", "Solution_10": "ACK! I'm so sorry about that one. Here was what the problem was supposed to be: \r\n\r\n3. Sin(a)=2/3 and cos(b)=3/4 and a is in Quadrant II and b is in Quadrant IV, then sin(a - b) = ?\r\n\r\nBut you don't have to solve it if you don't want to..." } { "Tag": [ "algebra", "polynomial", "complex numbers" ], "Problem": "It is pretty well known that if you have a complex root of a polynomial with real coefficients that the conjugate is also a root. i.e. if f(a+bi)=0 then f(a-bi)=0.\n\n\n\nI had never seen a short easy proof of it.\n\n\n\nIt's a nice problem, but if you just want to see the proof, here is a sketch:\n\n[hide]\n\nLet f(a+bi)= a-bi\n\nShow that f(a+bi) + f(c+di) = f(a+c +(b+d)i)\n\nShow that f(a+bi) * f(c+di) = f((a+bi) * (c+di))\n\n\n\nNow prove that for any polynomial g with real coefficients\n\nf(g(a+bi)) = g(f(a+bi)).\n\n\n\nThen we get the desired result as a corollary.\n\n[/hide]\n\n\n\n--Dan", "Solution_1": "Is this right?\n\n\n\n[hide]The following are true of all complex numbers and their conjugates:\n\nc * (a + bi) is the conjugate of c * (a - bi)\n\n\n\n(a + bi) + (c + di) is the conjugate of (a - bi) + (c - di)\n\n\n\n(a + bi)^n is the conjugate of (a - bi)^n\n\n\n\nNow consider two complex numbers w and z such that w is the conjugate of z.\n\n\n\nFrom the first and third statements we know that c *z^n is the conjugate of c * w^n.\n\n\n\nNow using the second we know that c_0 * z^n + c_1 * z^(n-1) + ... + c_n * z^0 is the conjugate of c_0 * w^n + c_1 * w^(n-1) + ... + c_n * w^0.\n\n\n\nThis means that P(a + bi) is the conjugate of P(a - bi) so if one is zero then the other must be.[/hide]", "Solution_2": "I assume you mean that c is real in the first statement. If that is the case, it looks good!" } { "Tag": [ "function", "inequalities", "algebra", "domain", "combinatorics proposed", "combinatorics" ], "Problem": "Let $\\mathcal{F}$ be the set of all the functions $f : \\mathcal{P}(S) \\longrightarrow \\mathbb{R}$ such that for all $X, Y \\subseteq S$, we have $f(X \\cap Y) = \\min (f(X), f(Y))$, where $S$ is a finite set (and $\\mathcal{P}(S)$ is the set of its subsets). Find\r\n\\[\\max_{f \\in \\mathcal{F}}| \\textrm{Im}(f) |. \\]", "Solution_1": "[quote=\"Valentin Vornicu\"]Let $\\mathcal{F}$ be the set of all the functions $f : \\mathcal{P}(S) \\longrightarrow \\mathbb{R}$ such that for all $X, Y \\subseteq S$, we have $f(X \\cap Y) = \\min (f(X), f(Y))$, where $S$ is a finite set (and $\\mathcal{P}(S)$ is the set of its partitions). Find\n\\[\\max_{f \\in \\mathcal{F}}| \\textrm{Im}(f) |. \\]\n[/quote]\r\n\r\nSorry, your terminology is confusing.\r\n\r\nIs $\\mathcal{P}(S)$ the power set of $S$? i.e the set of all subsets of $S$?\r\n\r\nIf $\\mathcal{P}(S)$ is the set of partitions of $S$ and If $X \\subseteq S$ then $X \\not\\in \\mathcal{P}(S)$, so talking about $f(X)$ would not make sense.", "Solution_2": "$P(S)$ is indeed the set of subsets of $S$, and yes, you are right, it is a typo in the enuntiation, $X, Y \\in P(S)$, instead of $X, Y \\in S$.", "Solution_3": "${\\mathcal P}(S)$ is the powerset of $S$, i.e. the set of its subsets, but he [i]does[/i] mean $X, Y \\subseteq S$ (which is equivalent to $X, Y \\in{\\mathcal P}(S)$), not what pohoatza said.\r\n\r\n[hide=\"Answer but no proof\"]If $S$ has $n$ elements, then we can always achieve $n+1$ values, as follows: choose any chain $\\emptyset = X_{0}\\subset X_{1}\\subset \\ldots \\subset X_{n}= S$. Then define $f(Y) = \\max_{i}(X_{i}\\subseteq Y)$. Then for any $Y, Z$,\n\\begin{eqnarray*}f(Y \\cap Z) &=& \\max_{i}(X_{i}\\subseteq(Y\\cap Z)) \\\\ &=& \\max_{i}(X_{i}\\subseteq Y\\textrm{ and }X_{i}\\subseteq Z) \\\\ &=& \\min\\{\\max_{i}(X_{i}\\subseteq Y),\\max_{i}(X_{i}\\subseteq Z)\\}\\\\ &=& \\min\\{f(Y), f(Z)\\}, \\end{eqnarray*}\nas desired.[/hide]", "Solution_4": "Yes JBL, I meant $X,Y \\in P(S)$, anyway your answer is correct.", "Solution_5": "Here is the proof we cannot do better:\r\n\r\n[hide]Let $|S| = n$, and consider the values on the $n-1$-element subsets of $S$, $X_{1}, X_{2}, \\ldots, X_{n}$. Choose any set $X \\subset S$, $|X| = n-i$, and without loss of generality let $X_{1}, X_{2}, \\ldots, X_{i}$ be the $n-1$-element supersets of $X$ in $S$. Also without loss of generality, assume $f(X_{1}) \\leq f(X_{2}) \\leq\\ldots\\leq f(X_{i})$. Then\n\\[f(X) = f(X_{i}\\cap(X_{i-1}\\cap\\ldots(X_{2}\\cap X_{1}))) = f(X_{i-1}\\cap\\ldots(X_{2}\\cap X_{1})) = \\ldots = f(X_{2}\\cap X_{1}) = f(X_{1}), \\]\nwhere the leftmost equality follows from the equality of the sets involved while the other inequalities follow from right to left by the given property of $f$. Thus, the values of $f$ on all sets other than $S$ itself are among these $n$ values, so $f$ can take at most $n+1$ different values.[/hide]", "Solution_6": "i think the value for the question is positive infinte. As i haven't find the way send my prove, so pls wait me..", "Solution_7": "The domain of $f$ is finite, so I'm not sure how you expect to make its range infinite. In any event, I have already posted a complete proof.", "Solution_8": "ok, i am sure the value is n+1." } { "Tag": [ "modular arithmetic" ], "Problem": "The sequence $(a_{1},a_{2},a_{3}......)$ is defined by $a_{1}=1, a_{2}=1, a_{3}=1$ and\r\n\r\n$a_{n}_{+}_{3}= \\frac{1}{a_{n}}(a_{n}_{+}_{1}a_{n}_{+}_{2}+7)$.\r\n\r\nProve that each of its elements is an integer.", "Solution_1": "Correct me if I'm wrong:\r\n$a_{1}=1$\r\n$a_{2}=1$\r\n$a_{3}=1$\r\n$a_{4}=9$\r\n$a_{5}=16$\r\n$a_{6}=151$\r\n$a_{7}=\\frac{2423}{9}$", "Solution_2": "$a_{4}=8$, so from there on vishalarul is not correct.\r\n\r\n[hide=\"Hint\"] The problem is the same as showing that $a_{n+4}\\cdot a_{n+5}\\equiv{_{a_{n+3}}}7$ $\\forall n \\in \\mathbb{N}$.[/hide]", "Solution_3": "Would you mind showing us how please? :)", "Solution_4": "[hide=\"hint that gives answer\"]\nmod 7\n[/hide]\n\n[hide=\"answer\"]\nObviously the first three terms are 1 mod 7, evaluating the equation for $a_{n+3}$ in mod 7 gives $\\frac{1 \\cdot 1+7}{1}\\equiv 1 \\pmod 7$, therefore all terms of the sequence of 1 mod 7, which make all of them integers by the prior congruence.\n[/hide]", "Solution_5": "[quote=\"deej21\"]\n[hide=\"answer\"]\nObviously the first three terms are 1 mod 7, evaluating the equation for $a_{n+3}$ in mod 7 gives $\\frac{1 \\cdot 1+7}{1}\\equiv 1 \\pmod 7$, therefore all terms of the sequence of 1 mod 7, which make all of them integers by the prior congruence.\n[/hide][/quote]\r\n\r\nYour logic is incorrect. If we have two numbers $a,b\\equiv 1\\pmod{7}$, $b$ does not necessarily divide $a$. A simple counterexample is $a=15$, $b=8$.", "Solution_6": "What about proving that $a_{n+4}= 16 a_{n+2}-a_{n}$ for all $n$? This is an easy exercise (induction).\r\n\r\nBut this alternative definition for the sequence shows immediately that all terms are integers." } { "Tag": [ "number theory unsolved", "number theory" ], "Problem": "Given that $ p$ is prime, prove that $ _{p}C_j$ is divisible by $ p$ where $ j\\in [1,p\\minus{}1]$.", "Solution_1": "Because $ \\binom{p}{j}\\equal{}\\frac{p!}{j!(p\\minus{}i)!}\\in\\mathbb{N}$ and $ p\\not |j!(p\\minus{}j)!,p|p!$ we're done.", "Solution_2": "[quote=\"countdownmath\"]Given that $ p$ is prime, prove that $ _{p}C_j$ is divisible by $ p$ where $ j\\in [1,p \\minus{} 1]$.[/quote]\r\nThe following is also true: \r\n$ \\binom{kp}{j}$ is divisible by $ p$ when $ k$ is an integer, $ gcd(j,p)\\equal{}1$ and of course you define $ j \\frac k2$ and so $ a_n \\equal{} 1$ $ \\forall n > 1$ and the sequence is $ \\{a,1,1,1,...\\}$\r\nIn this case, question (b) is obvious and the best $ \\lambda$ is $ \\max(a,\\frac 1a)$\r\n\r\nConsider now that $ a_n\\neq 1$ $ \\forall n$ and let $ b_n$ be the sequence $ b_n \\equal{} \\frac {1 \\plus{} a_n}{1 \\minus{} a_n}$\r\n\r\nWe get $ a_n \\equal{} \\frac {b_n \\minus{} 1}{b_n \\plus{} 1}$ and the equation becomes $ b_nb_m \\equal{} b_pb_q$ $ \\forall m,n,pq\\in\\mathbb N$ such that $ m \\plus{} n \\equal{} p \\plus{} q$\r\n\r\nIt is then easy to solve this equation and to get $ b_n \\equal{} b_1^{2 \\minus{} n}b_2^{n \\minus{} 1}$ and so $ a_n \\equal{} \\frac {b_1^{2 \\minus{} n}b_2^{n \\minus{} 1} \\minus{} 1}{b_1^{2 \\minus{} n}b_2^{n \\minus{} 1} \\plus{} 1}$ $ \\equal{} \\frac {b_2^{n \\minus{} 1} \\minus{} b_1^{n \\minus{} 2}}{b_2^{n \\minus{} 1} \\plus{} b_1^{n \\minus{} 2}}$ and :\r\n\r\n[u]General expression for $ a_n$[/u] :\r\nIf $ a \\equal{} 1$, then $ b \\equal{} 1$ and $ a_n \\equal{} 1$ $ \\forall n$\r\nIf $ a\\neq 1$ and $ b \\equal{} 1$, then $ a_n \\equal{} 1$ $ \\forall n > 1$\r\nIf $ a\\neq 1$ and $ b\\neq 1$, then $ a_n \\equal{} \\frac {\\left(\\frac {1 \\plus{} b}{1 \\minus{} b}\\right)^{n \\minus{} 1} \\minus{} \\left(\\frac {1 \\plus{} a}{1 \\minus{} a}\\right)^{n \\minus{} 2}}{\\left(\\frac {1 \\plus{} b}{1 \\minus{} b}\\right)^{n \\minus{} 1} \\plus{} \\left(\\frac {1 \\plus{} a}{1 \\minus{} a}\\right)^{n \\minus{} 2}}$\r\n\r\nThen :\r\n[u]Question (a)[/u] : $ \\frac {1 \\plus{} a}{1 \\minus{} a} \\equal{} 3$ and $ \\frac {1 \\plus{} b}{1 \\minus{} b} \\equal{} 9$ and so $ a_n \\equal{} \\frac {9^{n \\minus{} 1} \\minus{} 3^{n \\minus{} 2}}{9^{n \\minus{} 1} \\plus{} 3^{n \\minus{} 2}}$ $ \\equal{} \\frac {3^n \\minus{} 1}{3^n \\plus{} 1}$\r\n\r\n[u]Question (b)[/u] : \r\nLet $ u \\equal{} \\frac {1 \\plus{} b}{1 \\minus{} b}$ and $ v \\equal{} \\frac {1 \\plus{} a}{1 \\minus{} a}$. In order to have $ a_n > 0$ $ \\forall n$, we need to have $ |u|\\geq |v| > 1$ and we can write $ a_n \\equal{} 1 \\minus{} \\frac {2}{u\\left(\\frac uv\\right)^{n \\minus{} 2} \\plus{} 1}$. \r\n\r\nIt's then rather easy to show, using that $ |v|\\equal{}\\frac{1\\plus{}\\min(a,\\frac 1a)}{1\\minus{}\\min(a,\\frac 1a)}$ $ \\equal{}\\minus{}\\frac{1\\plus{}\\max(a,\\frac 1a)}{1\\minus{}\\max(a,\\frac 1a)}$, that \r\n$ \\min(a,\\frac 1a)\\leq a_n\\leq\\max(a,\\frac 1a)$ and so $ \\boxed{\\lambda \\equal{} \\max(a,\\frac 1a)}$", "Solution_2": "I was going to write my solution when I found the nice pco's. I must however make the following comments.\r\n\r\nBy taking $ (m,n,p,q) \\equal{} (1,k\\plus{}1,2,k)$ one gets the recurrence relation $ a_{k\\plus{}1} \\equal{} \\frac {(1\\minus{}ab)a_k \\plus{} (b\\minus{}a)} {(b\\minus{}a)a_k \\plus{} (1\\minus{}ab)}$ for the sequence $ \\{ a_k \\}$, with $ (1\\minus{}ab)^2 \\minus{} (b\\minus{}a)^2 \\equal{} (1\\minus{}a^2)(1\\minus{}b^2)$. Take $ \\alpha \\equal{} 1\\minus{}ab$, $ \\beta \\equal{} b\\minus{}a$, and $ f(x) \\equal{} \\frac {\\alpha x \\plus{} \\beta} {\\beta x \\plus{} \\alpha}$. Then $ a_{k\\plus{}1} \\equal{} f(a_k)$, so $ a_k \\equal{} f^k(a)$.\r\nBut $ f$ is a homographic function (also known as Mobius transformation, or other names...), and the theory of its iterates is widely known, related to the eigenvalues of the associated matrix $ A(f) \\equal{} \\genfrac {(} {)} {0pt} {} {\\alpha \\ \\beta} {\\beta \\ \\alpha}$.\r\n\r\nSee for example [url]http://www.mathlinks.ro/viewtopic.php?t=4948[/url] in this very forum.\r\n\r\nThus the general form, and the ensuing discussions, are, from this point of view, a direct application of these classical techniques." } { "Tag": [ "inequalities", "calculus", "derivative", "algebra", "polynomial", "AMC" ], "Problem": "[img]http://img220.imageshack.us/img220/3761/inequalitydw8.jpg[/img]", "Solution_1": "[hide]$-\\frac{1}{8}$?[/hide]", "Solution_2": "Please send your solution.", "Solution_3": "I don't know if it is correct.\r\n\r\n[hide]\nSince $a^{3}+b^{3}+c^{3}-3abc=(a+b+c)[(a+b)^{2}+(b+c)^{2}+(c+a)^{2}]$, $3abc=a^{3}+b^{3}+c^{3}$. Then\n\n$f(a,b,c)=a^{4}+b^{4}+c^{4}-3abc$\n\n$=a^{4}+b^{4}+c^{4}-a^{3}-b^{3}-c^{3}$\n\n$=a^{3}(a-1)+b^{3}(b-1)+c^{3}(c-1)$\n\nWe can just put $a=1$ since we want the minimum and then $b+c=-1$. Then \n\n$f(1,b,c)=b^{3}(b-1)+(b+1)^{3}(b+2)$\n\nBut I don't feel like taking the derivative of that and I suck at inequalities, so I give up. :P\n[/hide]\r\n\r\nEDIT: The numbers didn't add up to 0 so I had to edit my solution. . .", "Solution_4": "An actual solution:\r\n\r\n[hide]Applying AM-GM, $\\frac{a^{4}+b^{4}+c^{4}-3abc}{4}\\ge \\sqrt[4]{-3a^{5}b^{5}c^{5}}$.\nSimplifying, we have $a^{4}+b^{4}+c^{4}-3abc \\ge 4abc \\sqrt[4]{-3abc}$.\nTherefore, if we know the value of maximum value of abc, we will cover all the cases and will have found the minimum value. Obviously, the maximum value of abc is 0. Thus, $a^{4}+b^{4}+c^{4}-3abc \\ge 0$.\n\nAnd to think I was about to generate a polynomial! :rotfl: Good thing I didn't.[/hide]", "Solution_5": "AM-GM only works for positive numbers.", "Solution_6": "What isn't positive?\r\nAnd I thought AM-GM worked for all nonnegative integers...", "Solution_7": "[quote=\"falconwing64\"]What isn't positive?\nAnd I thought AM-GM worked for all nonnegative integers...[/quote]\r\n\r\nthen you're already assuming $a,b,c=0$", "Solution_8": "My God! I didn't assume a,b,c = 0! I [b]proved[/b] it. Read the proof.", "Solution_9": "you can't use AM-GM on negative integers. therefore, by using it, you're assuming $a,b,c=0$ since $a+b+c=0$", "Solution_10": "[quote=\"falconwing64\"]My God! I didn't assume a,b,c = 0! I [b]proved[/b] it. Read the proof.[/quote]\r\n\r\nYour proof is completely wrong.", "Solution_11": "What's positive in my AM-GM Inequality? $a^{4}, b^{4}, c^{4}$ are all positive, and $-3abc$, which I am treating as a single term, must also be positive.", "Solution_12": "[quote=\"falconwing64\"]What's positive in my AM-GM Inequality? $a^{4}, b^{4}, c^{4}$ are all positive, and $-3abc$, which I am treating as a single term, must also be positive.[/quote]\r\n\r\nIf $-3abc$ is positive, then $a^{4}+b^{4}+c^{4}-3abc>0$", "Solution_13": "$-3abc$ can be negative or positive depending on the specific values of $a,b,c$.", "Solution_14": "oh.......ok...i'm wrong...i see what my mistake was...thx for pointing it out. Sorry for being so rude :blush:", "Solution_15": "What is AM-GM?", "Solution_16": "It's a very useful inequality. Goto:\r\n\r\n[url]http://en.wikipedia.org/wiki/Inequality_of_arithmetic_and_geometric_means[/url]", "Solution_17": "AM-GM states that the arithmetic mean of any finite number of positive reals is greater than or equal to the geometric mean of the same set of numbers, with equality if and only if the numbers are all equal.", "Solution_18": "If $a=b=c$, the minimum is $-\\frac{81}{256}$", "Solution_19": "what???\r\n\r\na=b=c can't be true, since $a\\ge1$", "Solution_20": ":wallbash_red: i forgot $a+b+c=0$", "Solution_21": "[quote=\"falconwing64\"]It's a very useful inequality. Goto:\n\n[url]http://en.wikipedia.org/wiki/Inequality_of_arithmetic_and_geometric_means[/url][/quote]\r\nIsn't AM-GM obvious?", "Solution_22": "That's a strange question, but I wouldn't say that AM-GM is \"obvious\" in the way that the trivial inequality is obvious (they don't call it the trivial inequality for nothing :P ). The proof is quite involved.", "Solution_23": "[hide=\"how about...\"]\n$a+b+c = 0 \\implies a^{3}+b^{3}+c^{3}= 3abc$\nsince $a^{3}+b^{3}+c^{3}-3abc = (a+b+c)(a^{2}+b^{2}+c^{2}-ab-bc-ca)$. In particular, unless all variables are equal this is basically biconditional. Anyways,\n\n$a^{3}(a-1)+b^{3}(b-1)+(-a-b)^{3}(-a-b-1) = a^{3}(a-1)+b^{3}(b-1)+(a+b)^{3}(a+b+1)$\n\nand the goal is basically to minimize this sum for $a \\geq 1$. This implies that $a^{3}(a-1) \\geq 0$, so\n\n$a^{3}(a-1)+b^{3}(b-1)+(a+b)^{3}(a+b+1) \\geq b^{3}(b-1)+(a+b)^{3}(a+b+1)$\n\nFinally, if $b$ is positive the above expression must also be positive. And if $b$ is negative, we can replace the original expression with an expression given as \n\n$a^{3}(a-1)+b^{3}(b+1)+(a-b)^{3}(a-b+1) = b^{4}+b^{3}+(a-b)^{4}+(a-b)^{3}$\n\nwhich is in turn equal to\n\n$2b^{4}+2a^{4}+6a^{2}b^{2}+3ab^{2}-4a^{3}b-4ab^{3}-3a^{2}b$\n\nso we need only show that for $a \\geq 1$,\n\n$2b^{4}+2a^{4}+6a^{2}b^{2}+3ab^{2}\\geq 4a^{3}b+4ab^{3}+3a^{2}b$\n\nBy AM-GM, $2a^{4}+2a^{2}b^{2}\\geq 4a^{3}b$.\nSimilarly, $2b^{4}+2a^{2}b^{2}\\geq 4ab^{3}$. Then we simply need\n\n$2a^{2}b+3ab \\geq 3a^{2}$\n\n$2b \\geq 3 \\iff b \\geq \\frac{2}{3}$, so we can assume $b < \\frac{2}{3}$.\n\nThat's a start, at least...\n[/hide]", "Solution_24": "AM-GM is pretty obvious for two numbers, but the proof of AM-GM for all sets of numbers is by no means obvious." } { "Tag": [ "inequalities unsolved", "inequalities" ], "Problem": "Let $ a,b,c>0$ such that $ abc\\equal{}1$. Prove that\r\n$ \\frac{a}{{a^2 \\plus{} k}} \\plus{} \\frac{b}{{b^2 \\plus{} k}} \\plus{} \\frac{c}{{c^2 \\plus{} k}} \\le \\frac{3}{{k \\plus{} 1}}$", "Solution_1": "[quote=\"thanhak5\"]Let $ a,b,c > 0$ such that $ abc \\equal{} 1$. Prove that\n$ \\frac {a}{{a^2 \\plus{} k}} \\plus{} \\frac {b}{{b^2 \\plus{} k}} \\plus{} \\frac {c}{{c^2 \\plus{} k}} \\le \\frac {3}{{k \\plus{} 1}}$[/quote]\r\nthe condition of $ k$???? :maybe:", "Solution_2": "[quote=\"thanhak5\"]Let $ a,b,c > 0$ such that $ abc \\equal{} 1$. Prove that\n$ \\frac {a}{{a^2 \\plus{} k}} \\plus{} \\frac {b}{{b^2 \\plus{} k}} \\plus{} \\frac {c}{{c^2 \\plus{} k}} \\le \\frac {3}{{k \\plus{} 1}}$[/quote]\n[quote=\"nguoivn\"]Let $ a \\equal{} \\frac {x}{y}$...\nIt can rewrite:\n$ \\sum\\ \\frac {xy}{x^2 \\plus{} 3y^2} \\leq\\ \\frac {3}{4}$\nWe have: $ x^2 \\plus{} y^2 \\plus{} 2y^2 \\geq\\ 2\\sqrt []{2}y\\sqrt []{x^2 \\plus{} y^2}$\nSo, $ \\frac {xy}{x^2 \\plus{} 3y^2} \\leq\\ \\frac {1}{2\\sqrt []{2}}\\sqrt []{\\frac {x^2}{x^2 \\plus{} y^2}}$\n :)[/quote]\r\nI think $ k \\ge\\ 3$\r\nsimilar to nguoivn 's proof \r\nWe have: $ x^2 \\plus{} y^2 \\plus{} (k\\minus{}1)y^2 \\geq\\ 2\\sqrt []{k\\minus{}1}y\\sqrt []{x^2 \\plus{} y^2}$" } { "Tag": [ "geometry", "3D geometry", "pyramid", "analytic geometry", "national olympiad" ], "Problem": "1. Find all solutions in positive integers to:\r\n\\begin{eqnarray*} a + b + c = xyz \\\\ x + y + z = abc \\end{eqnarray*} \r\n\t\t\r\n2. $F_n$ is the Fibonacci sequence $F_0 = F_1 = 1$, $F_{n+2} = F_{n+1} + F_n$. Find all pairs $m > k \\geq 0$ such that the sequence $x_0, x_1, x_2, ...$ defined by $x_0 = \\frac{F_k}{F_m}$ and $x_{n+1} = \\frac{2x_n - 1}{1 - x_n}$ for $x_n \\not = 1$, or $1$ if $x_n = 1$, contains the number $1$. \t\t\r\n\r\n3. $PABCDE$ is a pyramid with $ABCDE$ a convex pentagon. A plane meets the edges $PA, PB, PC, PD, PE$ in points $A', B', C', D', E'$ distinct from $A, B, C, D, E$ and $P$. For each of the quadrilaterals $ABB'A', BCC'B, CDD'C', DEE'D', EAA'E'$ take the intersection of the diagonals. Show that the five intersections are coplanar. \r\n\t\t\r\n4. Define the sequence $a_1, a_2, a_3, ...$ by $a_1 = 1$, $a_n = a_{n-1} + a_{[n/2]}$. Does the sequence contain infinitely many multiples of $7$? \r\n\t\t\r\n5. The points $D, E$ on the side $AB$ of the triangle $ABC$ are such that $\\frac{AD}{DB}\\frac{AE}{EB} = (\\frac{AC}{CB})^2$. Show that $\\angle ACD = \\angle BCE$. \t\r\n\t\r\n6. $S$ is a board containing all unit squares in the $xy$ plane whose vertices have integer coordinates and which lie entirely inside the circle $x^2 + y^2 = 1998^2$. In each square of $S$ is written $+1$. An allowed move is to change the sign of every square in $S$ in a given row, column or diagonal. Can we end up with all $-1$s by a sequence of allowed moves?", "Solution_1": "1. http://www.mathlinks.ro/Forum/viewtopic.php?p=366273#p366273\r\n2. http://www.mathlinks.ro/Forum/viewtopic.php?p=366275#p366275\r\n3. http://www.mathlinks.ro/Forum/viewtopic.php?p=366276#p366276\r\n4. http://www.mathlinks.ro/Forum/viewtopic.php?p=366277#p366277\r\n5. http://www.mathlinks.ro/Forum/viewtopic.php?p=366280#p366280\r\n6. http://www.mathlinks.ro/Forum/viewtopic.php?p=366282#p366282" } { "Tag": [ "geometry", "3D geometry", "prism", "MATHCOUNTS", "tetrahedron", "puzzles" ], "Problem": "I have a very pricky fish. It only wants to go into a 50 gallon tank, but the problem is that I only have two 60 gallon tanks. If I have no measurements of any kind, how can I have a tank that is exactly 50 gallons?", "Solution_1": "Are the tanks in the shape of rectangular prisms?", "Solution_2": "ckck,\r\n\r\nyou meant a \"very picky\" fish, didn't you?", "Solution_3": "I say \"Too bad fishy! You're going to the big tank!\"\r\n\r\nAnyhow, do you have anything to scoop the water from one tank to the other, or do you just dump it?", "Solution_4": "Go out and buy a 50 gallon tank", "Solution_5": "If your fish is being that picky you should just get rid of it and get something cool.............like a shark!", "Solution_6": "Yes tanks are in the shape of a retangular prism, and I meant picky fish. Sorry. :blush:", "Solution_7": "[hide]\nIs there a solution without using two different color, immiscible liquids in the two containers? I can see no other way of measuring out one third...[/hide]", "Solution_8": "There should be a solution like that.", "Solution_9": "[hide]\nIf you are referring to my last hide tag, then a solution is along the lines of: measure out one half a container, measure out one third a container by:\n\nputting equal amounts of the two immiscible multicolored liquids in one container such that the three volumes are equal\n\nthen the difference is one sixth a container\nwhich is subtracted.\n\n\nNote this isn't a real solution, which requires cunning, but a methd of the math behind the solution and what lengths are measurable[/hide]\r\n\r\nEDIT: post 38^2 or for mathcounts people that 1444 :P", "Solution_10": "[hide]\nFill a 60 gallon tank, then put the fish into it.\nThen scoop a little amount of water out, and ask the fish,\n\"Fishy, is this ok?\"\nIf the answer is no, repeat the procedure.\n\nSorry for the nonsense, but the picky fish should give a sign about whether the amount of water is 50 gallon.\n[/hide]", "Solution_11": "[quote=\"modeler\"][hide]\nFill a 60 gallon tank, then put the fish into it.\nThen scoop a little amount of water out, and ask the fish,\n\"Fishy, is this ok?\"\nIf the answer is no, repeat the procedure.\n\nSorry for the nonsense, but the picky fish should give a sign about whether the amount of water is 50 gallon.\n[/hide][/quote]\r\n :rotfl: :rotfl: Good try.", "Solution_12": "Get a gallon-sized milk bottle and fill it up 50 times", "Solution_13": "[quote=\"SoxFan34\"]Get a gallon-sized milk bottle and fill it up 50 times[/quote]\r\nYou have no kind of measurement at all.", "Solution_14": "Fill up a 60 gallon tank and then pour some of it into the other tank such that the remaining liquid forms a tetrahedron sharing 3 edges with the tank. If you know what I mean.", "Solution_15": "[quote=\"probability1.01\"]Fill up a 60 gallon tank and then pour some of it into the other tank such that the remaining liquid forms a tetrahedron sharing 3 edges with the tank. If you know what I mean.[/quote]\r\n\r\nClever... I assume you have found a way to stand the tank up on a vertex such that the plane containing the three adjacent vertices is parallel to the table... or make water go where you want it to go.", "Solution_16": "You mean a tank with 50 gallons of water in it, not a 50-gallon tank of water, correct?", "Solution_17": "Sixty gallons of water is really heavy IME.\r\n\r\nA gallon is 3.79 (approximately) liters. Sixty gallons is 227.4 liters, which is 227.4 kg\r\n\r\nCan [i]you[/i] lift 227.4 kg?", "Solution_18": "Not to mention the weight of the tank by itself you did not factor in." } { "Tag": [ "probability" ], "Problem": "Without replacement, what is the probability of drawing a 6 before a 9 when three cards are picked from a standard 52-card deck? (For example, 6 of diamonds, Queen of hearts, 9 of spades.)", "Solution_1": "Do you have to draw a 9?", "Solution_2": "yes you do.", "Solution_3": "[hide] If there is a 6 in the first card, then the probability of drawing exactly one 9 in the other 2 is $\\frac{4}{51}\\times \\frac{47}{50}+\\frac{47}{51}\\times \\frac{4}{50}=\\frac{188}{1275}$ The probability of drawing 2 9's is $\\frac{4}{51}\\times \\frac{3}{50}=\\frac{2}{425}$. Since there is a $\\frac{1}{13}$ chance of drawing a 6 on the first draw, this is a total probability of $\\frac{188+6}{1275}\\times \\frac{1}{13}=\\frac{194}{16575}$.\n\nIf the first card is a 9, the second a 6, and the third a 9, there is a probability of $\\frac{1}{13}\\times \\frac{4}{51}\\times \\frac{3}{50}=\\frac{2}{5525}$\n\nIf the first card is not a 6 or a 9, the second is a 6, and the 3rd is a 9, there is a probability of $\\frac{11}{13}\\times \\frac{4}{51}\\times \\frac{2}{25}=\\frac{88}{16575}$\n\nThis is a total probability of $\\frac{194+6+88}{16575}=\\frac{96}{5525}$.[/hide]\r\n\r\nI'm not sure if this is right because it seems to be too ugly of an answer.", "Solution_4": "[hide]\n\n First of all, we need to find out the total number of possible combinations that you can get by drawing from a deck of 52\nat three at a time.\n\n that would be $_{52}C_{3}= \\frac{52 \\bullet 51 \\bullet 50}{3 \\bullet 2 \\bullet 1}= 22100$\n\n The number of possibilities that a $6$, a $9$ and another card (including drawing another $6$ or $9$) are drawn is $50(_{3}P_{1}) = 300$\n\nThe number of possibilities that a $6$, a $9$ and another card (excluding another $9$) are drawn is $47(_{3}P_{1}) = 282$\n\nThe probability that a $6$ is the first card drawn in those possibilities is $\\frac{1}{3}$, so there are $100$ possibilities that a $6$ will be drawn first.\n\nThe second case is the probability of a $6$ being the second card drawn and the third card being $9$, provided that another $9$ isn't drawn as the first card which is $\\frac{1}{6}$ so there are $47$ possibilities that a number other than $9$ will be drawn first, a $6$ drawn second and a $9$ drawn third.\n\nThe total possible ways of drawing three cards with a $6$ drawn before a $9$ is $100+47 = 147$\n\nSo the probability of drawing three cards where a $6$ is drawn before a $9$ in a deck of 52 cards is $\\frac{147}{22100}$\n\n[/hide]", "Solution_5": "I got the same answer as samath.", "Solution_6": "remember\r\n\r\nif you see a 69, 96, or 666 in a treething question you know you're either correct or very close" } { "Tag": [ "articles", "Asymptote", "number theory" ], "Problem": "I try to insert a horizontal line into my pdf : \r\n[code]\\documentclass{article} \n\\usepackage[pdftex]{graphicx} \n\\usepackage{asymptote} \n\\begin{document} \n \nNumber theory maths\n \n\\begin{figure}[h] \n\\begin{asy} \n................ \n\\end{asy} \n\\end{figure} \nNumber theory \n \n\\end{document} [/code]\n\nThe length of the line is the textwidth, what is the code ? \n \nIf the line is a dotted line , say, longdashed, then how to get that? \n\n\nMany thanks! \n[/code]", "Solution_1": "If the length of the line is not \\textwidth , but half or one-third of \\textwidth , \r\nand the line is on the middle of the paper, what is the code? \r\n\r\nthangks!" } { "Tag": [ "number theory open", "number theory" ], "Problem": "If $a,b,c=(a^{2}+b^{2})/(ab+1)$ are integers, then prove that $c$ is a perfect square.", "Solution_1": "It's probl\u00e8me 6 IOM 1988\r\nsee\r\nhttp://www.mathlinks.ro/Forum/viewtopic.php?p=352683#p352683", "Solution_2": "And it's not an open problem ;)", "Solution_3": "I've written another post to apologize for accidentally having posted it in here, instead of the Unsolved Problems, but it was cancelled, I guess." } { "Tag": [ "algebra", "polynomial", "superior algebra", "superior algebra solved" ], "Problem": "To which famous ring is \r\n\r\n$\\frac{\\mathbb{Z}_{5}\\left[X,Y\\right]}{\\left(Y-X^{2}, XY+Y+2\\right)}$\r\n\r\nisomorphic?", "Solution_1": "We can substitute any $y$ to $x^{2}$.\r\nThus we look for $\\mathbb{Z}/(x^{3}+x^{2}+2)$.\r\nIf we would know that $\\mathbb{Z}_{5}$ is the field with $5$ elements, we would see (cause of the polynomial clearly being irreducible) that the ring we get is $\\mathbb{F}_{125}$, the field with $125$ elements.\r\nIf $\\mathbb{Z}_{5}$ are the $5$-adic integers, we get an unramified extension of degree $3$." } { "Tag": [], "Problem": "My post numbuh is 8! How about yours?", "Solution_1": "You do realize that you can look under somebody's avatar to see their post \"numbah\", right?", "Solution_2": "He's Treething. look at his sig, location, and avatar.", "Solution_3": "so if i created a new user with your signature, location, and avatar, you would appreciate it if other people claimed i was 1=2?", "Solution_4": "[quote=\"1=2\"]He's Treething. look at his sig, location, and avatar.[/quote]\r\nhuh\r\ni don't think treething knows funcia, and takes pictures of her\r\notherwise, i would be worried for funcia\r\nbut tat's not the case\r\n\r\nand junngi ist sezy", "Solution_5": "[quote=\"pianoforte\"][quote=\"1=2\"]He's Treething. look at his sig, location, and avatar.[/quote]\nhuh\ni don't think treething knows funcia, and takes pictures of her\notherwise, i would be worried for funcia\nbut tat's not the case\n\nand junngi ist sezy[/quote]\r\neh i'd be worried too\r\n\r\nasmallgoat is my sibling\r\n\r\ntreething->treething's profile\r\n\r\nthe converse isn't neccesarily true", "Solution_6": "[quote=\"Klebian\"]so if i created a new user with your signature, location, and avatar, you would appreciate it if other people claimed i was 1=2?[/quote]\r\n\r\nyou mean like this?", "Solution_7": "Ego boost ++ for Treething.\r\n\r\nIf this ever gets out of hand, though, keep in mind that the administrators can easily read IP addresses from posts. I'm pretty sure that asmallgoat doesn't sneak into my house and post on my laptop when I'm out, so I should be fine.", "Solution_8": "Too much off topic talk, that's the problem of this generation.\r\nMine is 274! A good deal more than yours.", "Solution_9": "[quote=\"bubka\"]Too much off topic talk, that's the problem of this generation.\nMine is 274! A good deal more than yours.[/quote]\r\n\r\nWasn't yours like 3275? You posted -3000 times in 6 months?!?!?", "Solution_10": "[quote=\"Ignite168\"][quote=\"bubka\"]Too much off topic talk, that's the problem of this generation.\nMine is 274! A good deal more than yours.[/quote]\n\nWasn't yours like 3275? You posted -3000 times in 6 months?!?!?[/quote]\r\n\r\nEver here of the great depression? Well, the lesser depression happened, and all posts in the G&FF didn't count. Bubka fell from 1000 something to 3 something(disturbing...) and I fell from 1000 something to 200. Now that I hae solved a bunch of problems, I'm back on my feet, :)", "Solution_11": "Back off topic, isn't there also a \"_funcia\" with the same exact info as funcia?", "Solution_12": "[quote=\"miyomiyo\"]Back off topic, isn't there also a \"_funcia\" with the same exact info as funcia?[/quote]\r\n\r\nbut that's funcia b/c my post number is 1337\r\n\r\nand my don't asmallgoat and _funcia have the same ip almost all the time :o \r\n\r\nand doesn't asmallgoat post pictures of me all the time\r\n\r\nthat's annoying but i am not sure what i can do about it other than secretly finding \r\ntheir pw one day...", "Solution_13": "my post count is 99999999999999", "Solution_14": "mine is 999999999999999999999999999999999999999999999999999999999.53", "Solution_15": "[quote=\"ra5249\"]mine is 999999999999999999999999999999999999999999999999999999999.53[/quote]\r\n\r\nEven a double-precision floating point number cannot store such a large number with so many nines!", "Solution_16": "[quote=\"1=2\"]He's Treething. look at his sig, location, and avatar.[/quote]\r\n\r\nDrink less Evian water please.", "Solution_17": "[quote=\"kyyuanmathcount\"][quote=\"1=2\"]He's Treething. look at his sig, location, and avatar.[/quote]\n\nDrink less Evian water please.[/quote]\r\n\r\nOr he could wait for his to drop before posting..." } { "Tag": [ "function", "vector", "algebra proposed", "algebra" ], "Problem": "Does there exist two periodic function f(x),g(x),R->R,which causes to any x, has f(x)+g(x)=x?", "Solution_1": "Any solution\uff1f", "Solution_2": "That's a [url=http://cornellmath.wordpress.com/2008/01/22/periodic-functions-problem/]pretty famous[/url] question. \r\n\r\nThe answer is \"yes\", and it's fairly easy to do using a Hamel basis. This is a set $ H$ of real numbers which forms a vector-space basis for $ \\mathbb{R}$ over $ \\mathbb{Q}$. Thus every real number can be expressed as a linear combination of elements of $ H$ with rational coefficients, in a unique way.\r\n\r\nIt's now an easy exercise using the definitions of \"basis\" for a vector space to prove the following facts:\r\n\r\n1. Every function $ f: H \\to \\mathbb{R}$ defines, in a unique way, a function $ \\tilde{f}: \\mathbb{R} \\to \\mathbb{R}$ such that $ \\tilde{f}(h)\\equal{}f(h)$ for all $ h \\in H$.\r\n\r\n2. If $ f: H \\to \\mathbb{R}$ satisfies $ f(h_0)\\equal{}0$ for some $ h_0 \\in H$, then $ \\tilde{f}$ is periodic: $ \\tilde{f}(x\\plus{}h_0) \\equal{} \\tilde{f}(x)$ for all $ x \\in \\mathbb{R}$. \r\n\r\nTo solve the problem, all that remains now is to define two functions $ f,g: H \\to \\mathbb{R}$ such that $ f(h_0)\\equal{}g(h_1)\\equal{}0$ for some arbitrary basis elements $ h_0 \\neq h_1 \\in H$, in such a way that $ f(h)\\plus{}g(h) \\equal{} h$ for all $ h \\in H$. This is easily done. Taking $ \\tilde{f}$ and $ \\tilde{g}$ yields two periodic functions which sum to the identity." } { "Tag": [ "articles", "geometry" ], "Problem": "Hi!\r\nis there anyone here who is or is thinking of studying maths at cambridge.\r\nI live in germany but i am british and I am thinking of applying...\r\n\r\ngot some questions:\r\n\r\n-how do I select the college?\r\n-what exactly do I need to state in the UCAS application?\r\n-what are the interviews like?\r\n-how good do i need to be in maths to have a good chance of being accepted?\r\n-is cambridge even a good place to study maths as a german student?\r\n\r\nappreciate help...", "Solution_1": "Hi aaton,\r\nI hope what follows is helpfull, although i'm only going to answer your questions partially.\r\n\r\n1) How do you select a college ? Well, that's the last thing you should be concerned with :-) There are lots of excellent colleges e.g. trinity, johns, kings and many others. My advice is to check them out (try their websites for example or some relevant forums) and see which one would suit you most, also bare in mind some are more competitive to get in than others. You can also make an 'open' application in case you cannot decide.\r\n\r\n2) What do you need to state in the UCAS app ? That's a bit special and the details change every year, so the best thing to try is to read carefully all the manuals, i'm sure they are designed to be accessible for everyone. \r\n\r\n3) The interviews are very cool ! They give you the opportunity to show them how capable you are for doing maths there. They are very important for the admission ! They only ask math stuff (at least most of the colleges, i think) ! They don't assume extra knoledge and are designed to fit all the candidates. They don't want to test your knowledge, but your ability to think mathematically. Usually, you will be given a set of problems an hour before the interview and then you discuss your solutions with the interviewers. \r\n\r\n4) How good do i need to be ? Very good question ! You don't need to be a genious neither particularly clever. But you need to be able and enthousiastic, the later more !\r\n\r\n5) From Germany ? Well, there are many students from Germany ! I think you should try to contact them (maybe you should write in the mathlinks-germany forum). Germany has excellent unis to do maths, but i know nothing more about them. There are brilliant students from Germany that decided to stay there ! I think whether you want to leave or not, it's a rather personal question. Something i can tell, is that Cambridge is also an excellent place to study maths, full a stimullating stuuf and students, very enthousiastic, it's really an amazing place. \r\n\r\nI suggest you go to the website : http://www.maths.cam.ac.uk, and http://www.dpmms.cam.ac.uk (go to people-> Prof Koerner and check his website, it has many articles that would interest you), also check the student representatives website and the relative forums there e.g. the archimidean's forum. You can find more information there, and if you post there you're likely to get a better and more detailed answer.\r\n\r\nGood luck with your decisions,\r\nStergios", "Solution_2": "thanks for your quick answere!\r\n\r\nStill got some questions, hope you don't mind:\r\nDo you think an open application will increase my chances of being accepted? I don't really care in which college I will end up in. Or are there big differences in the quality of tutoring?\r\n\r\nDo you still have some problems from the interview? I would be really interested in seeing them...\r\nSo if I understood you correctly, extracurricular activities like sport or community work have no influence on the admission? I think I am fairly strong in that area and it would be great if they also look at those kind of things.\r\nDid you have to take the STEP test or were you given an unconditional offer?\r\n\r\nI also read somewhere that you can change to physics after studying maths for a year. Have you also heard of this?", "Solution_3": "I don't think that an open application will increase your chances of getting in. I know ppl who got in both ways. The quality of tutoring from college to college is roughly the same, there are a few differences tho. I think it's better to chose a college you like and apply to it. By doing an open app they \"randomly\" sent you to a college and then the college decides, so you might as well apply directly to one. (I'm not precise here but it works roughly like that) \r\n\r\nAs for the interview, i don't have any problems but i'm sure you can find a sample on the relevant websites. \r\nAlso, i suppose they do take in mind extracurricular activities but definitely don't know how much. \r\nAs far as STEP is concerned, there are cases were students are given unconditional offers (usually based on exceptional academic performances or cause they are coming from countries where it is not possible to take the test) Personally, i don't know anyone who was given a STEP-conditional offer and failed ! That doesn't mean STEP is so easy, it does require work and preparation but it's nothing to be afraid of and also it's the last thing you should bother with !\r\n\r\nAt last, you can indeed change from maths to physics. There is a math-physics choice the first year. But i encourage you to do straight math. There is a big choice of applied math courses and theoretical physics courses in the tripos so that you don't have to worry in case your interests lie in physics. \r\n\r\nGood luck,\r\nStergios", "Solution_4": "In terms of cost, how does the maths and physics course differ from the maths (only) course?", "Solution_5": "Sorry to revive this old topic... I just wanted to know when the STEP is taken by the students... Is it like the SAT where people take it before they pass out of their school? Or, after one appears for the class 12 board exams?", "Solution_6": "Do you guys say \"maths\" in Britain?", "Solution_7": "Indeed. How crazy are we?" } { "Tag": [ "Inequality", "Sequence", "algebra", "IMO Shortlist" ], "Problem": "Let $ a_1 \\geq a_2 \\geq \\ldots \\geq a_n$ be real numbers such that for all integers $ k > 0,$\n\n\\[ a^k_1 \\plus{} a^k_2 \\plus{} \\ldots \\plus{} a^k_n \\geq 0.\\]\n\nLet $ p \\equal{}\\max\\{|a_1|, \\ldots, |a_n|\\}.$ Prove that $ p \\equal{} a_1$ and that\n\n\\[ (x \\minus{} a_1) \\cdot (x \\minus{} a_2) \\cdots (x \\minus{} a_n) \\leq x^n \\minus{} a^n_1\\] for all $ x > a_1.$", "Solution_1": "For each $ k$ which is odd, we have $ na_{1}^k \\geq \\sum_{i = 1}^{n}{a_{i}^k} \\geq 0$, therefore ${ a_{1} \\geq 0}$\r\n+ If $ a_{n} \\geq 0 \\rightarrow p = a_{1}$\r\n+ If $ a_{n} < 0 \\rightarrow - a_{n}^k \\leq \\sum_{i = 1}^{n - 1}{a_{i}^k} \\leq (n - 1)a_{1}^k \\rightarrow - a_{n} \\leq (n - 1)^{\\frac {1}{k}}a_{1}$\r\nBecause $ \\lim_{k \\to \\infty}{(n - 1)^{\\frac {1}{k}} = 1}$, therefore $ - a_{n} \\leq a_{1} \\rightarrow a_{1} = p$\r\nWith $ k = 1$, we get $ a_{1} \\geq - \\sum_{i = 2}^{n}{a_{i}}$\r\nApplying AM-GM:\r\n$ (x - a_2)\\cdot(x - a_3)\\cdots (x - a_n) \\leq (\\frac {(n - 1)x - \\sum_{i = 2}^{n}{a_i}}{n - 1})^{n - 1} \\leq (x + \\frac {a_1}{n - 1})^{n - 1}$\r\nBut \r\n$ (x + \\frac {a_1}{n - 1})^{n - 1} = x^{n - 1} + \\sum_{i = 2}^{n - 2}{\\binom{n}{i}x^{n - i}(\\frac {a_1}{n - 1})^i} + a_1^{n - 1} \\leq x^{n - 1} + \\sum_{i = 2}^{n - 2}{x^{n - i}a_1^i} + a_1^{n - 1} = \\frac {x^n - a_1^n}{x - a_1}$ \r\n(because $ \\binom{n}{i}\\cdot\\frac {1}{(n - 1)^i} < 1$)\r\nSo $ (x - a_1) \\cdot (x - a_2) \\cdots (x - a_n) \\leq x^n - a^n_1$", "Solution_2": "$(x - a_1) \\cdot (x - a_2) \\cdots (x - a_n) = x^n - a^n_1\\iff (n=1$ or $n=2\\wedge a_2=-a_1)$", "Solution_3": "Sorry to revive the post. I used induction instead.", "Solution_4": "[quote=WolfusA]$(x - a_1) \\cdot (x - a_2) \\cdots (x - a_n) = x^n - a^n_1\\iff (n=1$ or $n=2\\wedge a_2=-a_1)$[/quote]\n\nsorry but what do you mean? :blush: ", "Solution_5": "Condition for equality in the inequality is $$n=1\\vee (n=2\\wedge a_1=-a_2)$$", "Solution_6": "[quote=joepro]\nBut \n$ (x + \\frac {a_1}{n - 1})^{n - 1} = x^{n - 1} + \\sum_{i = 2}^{n - 2}{\\binom{n}{i}x^{n - i}(\\frac {a_1}{n - 1})^i} + a_1^{n - 1} \\leq x^{n - 1} + \\sum_{i = 2}^{n - 2}{x^{n - i}a_1^i} + a_1^{n - 1} = \\frac {x^n - a_1^n}{x - a_1}$ [/quote]\n\nI think you mean:\n\n$ (x + \\frac {a_1}{n - 1})^{n - 1} = x^{n - 1} + \\sum_{i = 2}^{n - 1}{\\binom{n}{i}x^{n - i}(\\frac {a_1}{n - 1})^i}\\leq ...$\n\nsince the last term is $\\displaystyle(\\frac{a_1}{n-1})^{n-1}$. But otherwise nice solution!", "Solution_7": "For the first part, it is easy to see that $k\\to \\infty$ for some $k,$ if the largest number $p$ is actually negative, then the first expression will not be true.\n\nFor the second part, could somebody help with me with the induction step. I managed to prove that if we assume that $n$ is true, then we just want to show that $a_{n+1}(x^n-a_1^n)+a_1^n(x-a_1)\\geq 0$ is true, but I am not sure how to proceed from here. :(", "Solution_8": "very nice problem , Solved with [b]twbnrftw[/b]\n\n[i]Solution:[/i] 1st part we observe that not all of the terms can be negative so at least one of the terms is positive, now since $a_1$ is the maximum of all $a_{i}'s$ we have $a_{1} \\geqslant 0$, now we also have for each odd integer $k$ , $\\sum_{i=1}^{n} a_{i}^{k} \\leqslant na_{1}^{k}$ , now we claim that $\\max{\\{|a_{1}|, |a_{2}|, \\cdots, |a_{n}|\\}}=a_{1}$. This is equivalent to show that $\\max{\\{|a_{1}|, |a_{2}|, \\cdots , |a_{n}|\\}} \\neq |a_{n}|$ , now FTSOC let us assume that is the case (which can only happen if $a_{n}<0$) , then we have for sufficiently large odd $k$, $a_{1}^{k}+a_{2}^{k}+\\cdots+a_{n-1}^{k}+a_{n}^{k} \\leqslant (n-1)a_{1}^{k}+a_{n}^{k} $ , now we show that for sufficiently large choosen $k$ we have $(n-1)a_{1}^{k}+a_{n}^{k} <0$, but that is actually true because if we had $(n-1) \\geqslant -\\frac{a_{n}}{a_{1}}^{k}$ for large such odd $k$ we have $n-1 \\rightarrow \\infty$ which is not possible since $n$ is fixed , now that implies for large enough odd $k$ we have $a_{1}^{k} +a_{2}^{k}+ \\cdots +a_{n}^{k} <0$ contradiction, hence $\\max{\\{|a_{1}|, |a_{2}|, \\cdots , |a_{n}|\\}}=a_{1}$. $\\square$\n-----\nNow we work on the second part clearly for for $x>a_{1}$ we have $x-a_{i}>0$ $\\forall$ $i \\in \\{1,2, \\cdots , n\\}$ , hence by $\\emph{AM} \\geqslant \\emph{GM}$ we have $(x-a_{2})(x-a_{3})\\cdots (x-a_{n}) \\leqslant \\left( x -\\frac{\\sum_{i=2}^{i=n} a_{i}}{n-1}\\right)^{n-1}$ , also for $k=1$ we have $a_{1} \\geqslant -(a_2+a_3+\\cdots +a_n) \\implies \\left( x -\\frac{\\sum_{i=2}^{i=n} a_{i}}{n-1}\\right)^{n-1} \\leqslant \\left(x+\\frac{a_{1}}{n-1}\\right)^{n-1}$ , Now:\\\\\n\n$\\left(x+\\frac{a_{1}}{n-1}\\right)^{n-1} =x^{n-1}+ a_{1}\\frac{\\binom{n-1}{1}}{n-1}+a_{2}^{2}\\frac{\\binom{n-1}{2}}{(n-1)^2}+\\cdots +a_{1}^{n-1}< x^{n-1}+x^{n-2}a_{1}+x^{n-3}a_{2}^{2}+\\cdots +a_{1}^{n-1}=\\frac{x-a_{1}^{n}}{x-a_{1}} \\implies (x-a_{1})(x-a_{2})(x-a_{3})\\cdots (x-a_{n}) \\leqslant x^{n}-a_{1}^{n}$ , equality holds at $n=1$ or $n=2$ with $a_{2}=-a_{1}$. $\\square$" } { "Tag": [ "inequalities", "inequalities proposed" ], "Problem": "$x,y,z>0$ such as $xy+yz+xz=3$.\r\nPove that : $\\sqrt{x} + \\sqrt{y} + \\sqrt{z} \\ge 3$", "Solution_1": "[quote=\"IP\"]$x,y,z>0$ such as $xy+yz+xz=3$.\nPove that : $\\sqrt{x} + \\sqrt{y} + \\sqrt{z} \\ge 3$[/quote]\r\nThis is not true.\r\nTry $x=y\\rightarrow\\sqrt3,$ $z\\rightarrow0.$ ;)", "Solution_2": "With the given condition the inequality $x^k+y^k+z^k\\ge3$ is only valid for $k\\ge2-\\frac{\\ln4}{\\ln3}\\approx.73814$.", "Solution_3": "[quote=\"spanferkel\"]With the given condition the inequality $x^k+y^k+z^k\\ge3$ is only valid for $k\\ge\\frac{\\ln3}{\\ln2}$.[/quote]\r\nI think it is your mistake: $x+y+z\\geq3.$ ;)\r\nThe following problem is very interesting to me:\r\nLet $x,y,z$ are non-negative real numbers and $xy+xz+yz=3.$\r\nProve that $\\sqrt[4]{x^3}+\\sqrt[4]{y^3}+\\sqrt[4]{z^3}\\geq3,$\r\nbut I do not manage to prove it. :(", "Solution_4": "Note to user IP that section \"Inequalities Proposed & Own Problems\" is here for \"Problems that you have already solved and you are interested in second opinions or solutions.\"\r\n\r\nIf you don't know any solution to some problem and you don't know whether the statement of your problem is true or not, post it \"Inequalities Open Questions\".\r\n\r\nPS.: Try first to get know with some rules, before you use the forum. :mad:\r\n\r\nNice day.", "Solution_5": "Not really a similar question but maybe useful:\r\nhttp://www.mathlinks.ro/Forum/viewtopic.php?t=35839", "Solution_6": "[quote=\"conceive\"]\nIf you don't know any solution to some problem and you don't know whether the statement of your problem is true or not, post it \"Inequalities Open Questions\".\n[/quote]\r\nActually, in this case, it should rather go into the \"Inequalities Unsolved Problems\". ;)\r\n\r\n\r\n[b]arqady[/b], your inequality is indeed interesting, I'll have a try at proving it! :)", "Solution_7": "[quote=\"mathmanman\"]\nActually, in this case, it should rather go into the \"Inequalities Unsolved Problems\". \n[/quote]\r\nThank you, mathmanman. I have already made it.", "Solution_8": "[quote=\"arqady\"][quote=\"spanferkel\"]With the given condition the inequality $x^k+y^k+z^k\\ge3$ is only valid for $k\\ge\\frac{\\ln3}{\\ln2}$.[/quote]\nI think it is your mistake: $x+y+z\\geq3.$ ;)\n :([/quote] right, I've edited :blush:", "Solution_9": "The general problem had been posted here, but not complete solution.\r\n\r\nhttp://www.mathlinks.ro/Forum/topic-57106.htmlhttp://www.mathlinks.ro/Forum/topic-57106.html\r\n\r\nThe solution's at http://diendantoanhoc.net/forum/index.php?showtopic=7982 but in VietNamese only.", "Solution_10": "Thank you hungkhtn. \r\nId est, this problem:\r\nLet $x,y,z$ are non-negative real numbers and $xy+xz+yz=3.$\r\nProve that $\\sqrt[4]{x^3}+\\sqrt[4]{y^3}+\\sqrt[4]{z^3}\\geq3$\r\n is still unsolved. ;)", "Solution_11": "I don't think so, this has solution in the links I post. Even that It had solution in other famous problem.", "Solution_12": "[quote=\"hungkhtn\"]I don't think so, this has solution in the links I post. Even that It had solution in other famous problem.[/quote]\r\nYes you are right, hungkhtn. I have understood it, at last.:D \r\nThank you! :lol:" } { "Tag": [ "LaTeX", "USAMTS", "articles" ], "Problem": "i downloaded the LaTeX months ago but i didnt actually use it\r\n\r\nbut i thought itd be useful for writing solutions for usamts so i started using it...\r\n\r\n[code]\\documentclass{article}\n\\begin{document}\nHello, world!\n\\end{document} [/code]\r\n\r\ni put this in my TeXnicCenter, saved it, and pressed ctrl-f7, but nothing comes out in my output.\r\n\r\nwhat is wrong?", "Solution_1": "What is in the log? How many pages did it say were created on the bottom line of the bottom window? You can see it in the window at the bottom of TexnicCenter and is also the file in the same place as your document; so if your file was called latex1.tex then the log is latex1.log. Is there a file called latex1.pdf?", "Solution_2": "even after i press ctrl+f7, there is nothing on the log...", "Solution_3": "Really? Are you saying there is no log or it is blank? Are any files created?" } { "Tag": [ "search", "\\/closed" ], "Problem": "When private messaging, if you click on find a username, the screen seems to \"overlap\" making it impossible to type in the line to input a username. I've tried it with IE, and I use windows XP. \r\n\r\n[url]http://www.artofproblemsolving.com/Forum/search.php?mode=searchuser[/url]", "Solution_1": "That was posted before [url=http://www.artofproblemsolving.com/Forum/viewtopic.php?t=99298]here[/url] but no one responded about it.", "Solution_2": "Whoops, I guess somebody delete this post or the other." } { "Tag": [ "factorial", "algebra", "polynomial", "number theory proposed", "number theory" ], "Problem": "Suppose that $P(x)\\in\\mathbb Z[x]$ is a non-constant polynomial.\r\nProve that there is a natural number like $m$ such that $P(m!)$ is a composite number.\r\n\r\nP.S. : I have a long solution, which uses some kind of asymptotics. Anyone has a short proof, or a proof without asymptotics?!", "Solution_1": "Nima Ahmadi Pour, could you please post your solution?\r\ni would be very interested in a solution to this difficult problem.", "Solution_2": "Could you please post a soln? i dont mind if it is long and complicated, i think this problem is very difficult." } { "Tag": [ "trigonometry", "trig identities", "Law of Cosines" ], "Problem": "Quadrilateral ABCD is inscribed in circle O with AB=3, BC=4, CD=5, and AD=6. Find, to the nearest hundredth, AC.\r\n\r\nI know shamefully little about cyclic quadrilaterals, so please help and please explain.", "Solution_1": "[hide][b]Hint\n\nPtolemy[/b][/hide]", "Solution_2": "[hide=\"Solution not using the above hint\"]Since the quadrilateral is cyclic, angle $ ABC$ + angle $ CDA$ = $ 180$ degrees, so the sum of the cosines of the two angles is zero.\n\nSo we desire $ x$ such that $ 3^2 \\plus{} 4^2 \\minus{} 2\\times3\\times4x \\equal{} 5^2 \\plus{} 6^2 \\minus{} 2\\times5\\times6(\\minus{}x)$.\n\nSo $ 25 \\minus{} 24x \\equal{} 61 \\plus{} 60x$. $ 84x \\equal{} \\minus{}36$. $ x \\equal{}\\minus{}\\frac{3}{7}$. So $ \\sqrt{25 \\plus{} 72/7} \\approx \\boxed{5.94}$.[/hide]", "Solution_3": "That's brilliant (as in the Harry Potter British English \"brilliant\" :P), thanks.", "Solution_4": "let $ AC \\equal{} c$ and $ m\\angle B \\equal{} \\theta$. then $ m\\angle D \\equal{} 180^{\\circ} \\minus{} \\theta$ (this is a key property of cyclic quadrilaterals). by law of cosines in triangles $ ABC$ and $ ADC$, we have (since $ \\cos(180^{\\circ} \\minus{} \\theta) \\equal{} \\cos \\theta$)\r\n\\[ x^2 \\equal{} 3^2 \\plus{} 4^2 \\minus{} 2\\cdot 3\\cdot 4\\cdot \\cos \\theta\\]\r\n\r\n\\[ x^2 \\equal{} 5^2 \\plus{} 6^2 \\plus{} 2\\cdot 5\\cdot 6\\cdot \\cos \\theta\\]\r\nmultiply the first by 5, the second by 2, and add...", "Solution_5": "Now I'm confused, we have conflicting answers. O_O", "Solution_6": "no reason to be confused. There was an error in my post.", "Solution_7": "[quote=\"Jebus McAzn\"]Now I'm confused, we have conflicting answers. O_O[/quote]\r\n\r\nActually, his answers matches mine.\r\n\r\nHe has $ x^2 \\equal{} 25 \\minus{} 24 \\cos \\theta$ and $ x^2 \\equal{} 61 \\plus{}60 \\cos \\theta$. The first equation is equivalent to $ 5x^2 \\equal{} 125 \\minus{} 120 \\cos \\theta$ and $ 2x^2 \\equal{} 122 \\plus{} 120 \\cos \\theta$. Add the two: $ 7x^2 \\equal{} 247$. $ x \\equal{} \\sqrt{\\frac{247}{7}}$, which is what I had.", "Solution_8": "[quote=\"pleurestique\"]let $ AC \\equal{} c$ and $ m\\angle B \\equal{} \\theta$. then $ m\\angle D \\equal{} 180^{\\circ} \\minus{} \\theta$ (this is a key property of cyclic quadrilaterals). by law of cosines in triangles $ ABC$ and $ ADC$, we have (since $ \\cos(180^{\\circ} \\minus{} \\theta) \\equal{} \\cos \\theta$)\n\\[ x^2 \\equal{} 3^2 \\plus{} 4^2 \\minus{} 2\\cdot 3\\cdot 4\\cdot \\cos \\theta\\]\n\n\\[ x^2 \\equal{} 5^2 \\plus{} 6^2 \\plus{} 2\\cdot 5\\cdot 6\\cdot \\cos \\theta\\]\nmultiply the first by 5, the second by 2, and add...[/quote]\r\n\r\nSo the typo would be that $ \\cos(180^{\\circ} \\minus{} \\theta) \\equal{} \\minus{}\\cos \\theta$ instead of $ \\cos \\theta$?", "Solution_9": "$ \\cos (180^{\\circ} \\minus{} \\theta) \\equal{} \\cos 180^{\\circ} \\cos \\theta \\plus{} \\sin 180^{\\circ} \\sin \\theta \\equal{} (\\minus{}1) \\cos \\theta \\plus{} (0) \\sin \\theta \\equal{} \\minus{} \\cos \\theta$.", "Solution_10": "yeah, that was a typo, but the equations I wrote are correct." } { "Tag": [ "function", "topology", "complex analysis", "complex analysis unsolved" ], "Problem": "Let $ G$ be a bounded open set in $ \\bf C$ and suppose $ f$ is continuous function on $ \\bar{G}$ which is analytic in $ G$. Then $ \\max \\{ |f(z)|: z \\in \\bar{G} \\} \\equal{} \\max \\{ |f(z)|: z \\in \\delta G \\}$.\r\n\r\nI need to prove this, using the previous theorem:\r\n\r\n[quote]\nIf $ f$ is analytic in a region $ G$ and $ a$ is a point in $ G$ with $ |f(a)| \\geq |f(z)|$ for all $ z$ in $ G$, then $ f$ must be constant.\n[/quote]\r\n\r\nClearly we have not assumed connectedness in the theorem I need to prove. However, I can apply the previous theorem to the connected components and take the maximum. But what if there are infinitely many connected components. Isn't this a flaw in my proof?", "Solution_1": "No, you are still fine. Every point in $ G$ lies in one of the components and the boundary of each component is a subset of the boundary of the entire set. You may be required to prove this simple topological statement though. :)", "Solution_2": "Mmm, I still haven't figured this one out...\r\n\r\n[quote]MMT (version 1) if $ f$ is analytic in a [b]region[/b] $ G$ and $ a$ is a point in $ G$ with $ |f(a)| \\geq |f(z)|$ for all $ z \\in G$ then $ f$ is constant.\n\nProof: let $ \\Omega \\equal{} f(G)$, put $ \\alpha \\equal{} f(a)$. If $ |\\alpha| \\geq |\\zeta|$ for all $ \\zeta \\in \\Omega$, then $ \\alpha \\in bound(\\Omega) \\cap \\Omega$. The set $ \\Omega$ can't be open. So the open mapping theorem yields that $ f$ must be constant.[/quote]\n\n\n\t\n[quote]MMT (version 2)\nLet $ G$ be a bounded open set in $ C$ and suppose $ f$ is continuous function on $ Cl(G)$ and is analytic in $ G$ . Then $ \\max \\{ |f(z)|: z \\in \\bar{G} \\} \\equal{} \\max \\{ |f(z)|: z \\in \\delta G \\}$.\n\nProof: Since $ G$ is bounded there is a point $ a \\in Cl(A)$ such that $ |f(a)|\\geq|f(z)|$ for all $ z \\in Cl(G)$. If $ f$ is constant we're done. If $ f$ is not constant the results follows from MMT version 1.[/quote]\n\n\nI don't understand the part\n\n[quote]\"If $ f$ is not constant the results follows from MMT version 1\"[/quote]\r\n\r\nWhere did the connectedness go? How do we use MMT version 1?", "Solution_3": "What is the problem when you have what fedja said? Anyway, the proof you give is technically not enough, I guess. I suspect they wanted $ G$ to be connected as well.", "Solution_4": "For suppose that $ G \\equal{} A \\cup B$ with $ A$ and $ B$ sets that are both disjoint and open. Then if we define $ f(z) \\equal{} 1$ on $ A$ and $ f(z) \\equal{} 0$ on $ B$ then $ f$ is analytic on the open set $ G$ but $ \\mid f(z) \\mid$ does not take its maximum on the boundary of $ G$." } { "Tag": [ "inequalities", "inequalities unsolved" ], "Problem": "if $ a,b,c \\in \\mathbb{R}^\\plus{}$ , prove that :\r\n\r\n$ (a^2\\plus{}2)(b^2\\plus{}2)(c^2\\plus{}2) \\geq 9(ab\\plus{}ac\\plus{}bc)$\r\n\r\n :D", "Solution_1": "[quote=\"Hidden Scofield\"]if $ a,b,c \\in \\mathbb{R}^ \\plus{}$ , prove that :\n\n$ (a^2 \\plus{} 2)(b^2 \\plus{} 2)(c^2 \\plus{} 2) \\geq 9(ab \\plus{} ac \\plus{} bc)$\n\n :D[/quote]\r\nThe stronger one:\r\nhttp://www.artofproblemsolving.com/Forum/viewtopic.php?t=76508", "Solution_2": "[hide=\"Hint\"]\nWlog $ (a\\minus{}1)(b\\minus{}1) \\ge 0$ then $ (a^2\\minus{}1)(b^2\\minus{}1) \\ge 0$ implying $ (a^2\\plus{}2)(b^2\\plus{}2) \\ge 3(a^2\\plus{}b^2\\plus{}1)$ then we can use CS[/hide]", "Solution_3": "thank arqady about the link.\r\n\r\nhey FOURRIER , Could you post a full proof.\r\n\r\n :D", "Solution_4": "See here:\r\nhttp://www.mathlinks.ro/viewtopic.php?p=474995#474995", "Solution_5": "Thank you ...\r\n\r\nthat wonderful link :D", "Solution_6": "[quote=\"Hidden Scofield\"]thank arqady about the link.\n\nhey FOURRIER , Could you post a full proof.\n\n :D[/quote]\r\n\r\nI did :P" } { "Tag": [ "AMC", "AIME", "geometry", "3D geometry", "email", "geometric transformation", "rotation" ], "Problem": "So here's part of an email I sent to the AMC people:\r\n\r\n\"I believe that question #5 on the 2008 AIME I can be interpreted in two valid ways which lead to different answers, and thus both answers should be accepted.\r\n \r\nThe question says that \"the cone first returns to its original position on the table after making 17 complete rotations\" but does not specify the axis/axes of rotation. The majority interpretation seems to be that this means 17 rotations around the cone's axis, which would make the answer 014.\r\n \r\nHowever, if the cone makes n rotations around its own axis and returns to its original position, it also makes another complete rotation around an axis which is perpendicular to the table and goes through the vertex of the cone--that is, it sweeps out a circle on the table. Thus, \"17 complete rotations\" could mean that the cone makes 16 rotations around its own axis and 1 around this second axis, so that msqrt(n)=1*sqrt(255) and the answer is 1+255=256.\r\n \r\nI would also argue that anyone who answers the problem assuming the second interpretation has already thought through the first interpretation and rejected it on the basis that it doesn't take the entirety of the cone's movement into account. They should not be punished for overthinking the problem. The second interpretation does not simplify the problem or change it in any substantial way.\"\r\n\r\nIn hindsight, I'm not sure what possessed me to interpret the problem this way despite the answer format and the fact that the AIME just doesn't use tricks like this...but at the time, I was pretty convinced that my answer was \"more correct\". I haven't found anything inherent in the problem that would eliminate this interpretation. So I guess my questions are 1) did anyone else do this/consider doing this, and 2) does my argument actually have any merit? Well, also 3) if my argument does have merit, is there anything else I should do with it to ensure that it's considered before grading starts?", "Solution_1": "I see where you're coming from\r\nThe question should have read \"17 complete rotations about the cone's altitude\" or something like that.\r\n\r\nI actually did think of that interpretation, but I immediately discarded it as incorrect because it didn't seem to be going anywhere in my head. I didn't bother to justify why I discarded it, but looking back, that interpretation does seem valid.", "Solution_2": "I couldn't interpret that problem at all at first, but I eventually settled around it rotating about its own axis.\r\n\r\nBut here's the problem with your interpretation, bookaholic:\r\n\r\nThe earth [b]rotates[/b] about its own axis,\r\nbut it [b]revolves[/b] around the sun.", "Solution_3": "http://en.wikipedia.org/wiki/Rotation\r\n\r\n\"Mathematically, a rotation is, unlike a translation, a rigid body movement which keeps a point fixed. This definition applies to rotations within both two and three dimensions (in a plane and in space, respectively.) A rotation in three-dimensional space keeps an entire line fixed, i.e. a rotation in three-dimensional space is a rotation around an axis.\"\r\n\r\nBoth of the rotations that bookaholic referred to are contained by this definition.\r\n\r\n\r\nThe first keeps the central axis of the cone fixed relative to the rest of the cone.\r\nThe second keeps the vertex of of the cone fixed relative to the table.", "Solution_4": "[quote=\"distracted523\"]But here's the problem with your interpretation, bookaholic:\n\nThe earth [b]rotates[/b] about its own axis,\nbut it [b]revolves[/b] around the sun.[/quote]\r\nYour version of rotation of an object is (to the rest of the mathematical world) a rotation about some point in the object itself, and your definition of revolution of an object is a rotation about some point outside the object. While such a distinction might be helpful in astronomy or something, in a mathematical context it is quite arbitrary.\r\n\r\nHere is one serious objection to bookaholic's interpretation : the cone was not engaged in simple rotation about this axis in the center of the tables. If you follow one point on the cone as it moves, it will not trace a circular path about this axis. Sure, topologically, the path is equivalent to a rotation, and in some sense the cone has rotated about this axis. But the same is true for any of the infinitely many oblique axes that pass through the cone's vertex, so wouldn't you have infinitely many rotations whatever you did? The only thing that distinguishes the axis perpendicular to the table is that the cone does indeed make a complete rotation about it if you identify all the cone's points with each other. But in this case, rotation about the cone's central axis doesn't make very much sense at all, so I think that the mainstream interpretation is the only solid interpretation.", "Solution_5": "[quote]Here is one serious objection to bookaholic's interpretation : the cone was not engaged in simple rotation about this axis in the center of the tables. If you follow one point on the cone as it moves, it will not trace a circular path about this axis. Sure, topologically, the path is equivalent to a rotation, and in some sense the cone has rotated about this axis. But the same is true for any of the infinitely many oblique axes that pass through the cone's vertex, so wouldn't you have infinitely many rotations whatever you did? The only thing that distinguishes the axis perpendicular to the table is that the cone does indeed make a complete rotation about it if you identify all the cone's points with each other. But in this case, rotation about the cone's central axis doesn't make very much sense at all, so I think that the mainstream interpretation is the only solid interpretation.[/quote]\r\nI'm not sure I see how I would be \"identifying all the cone's points with each other\". My argument is that from start to finish, each point on the cone moves exactly 360 degrees around the axis perpendicular to the table. The end result of this is the same as a simple rotation around the axis. The fact that the points do not trace a circular path is exactly accounted for by the 16 other rotations. \r\n\r\nAlso, the points on the cone's altitude do move in a circle. Since the problem doesn't specify an axis of rotation, I think it's problematic to insist that the points must stay on a single circle, because only the altitude points stay on a circular course without reference to a particular (moving) axis. That course goes around the axis perpendicular to the table. So the problem basically forces everyone to assume something about which axis/axes are being referred to--I just picked an unpopular assumption.", "Solution_6": "I think that answer is valid, but I don't think its the one that would be expected to be seen. (helping you bump too)", "Solution_7": "Thanks for the bump :D Yeah, I don't deny that I should have expected the obvious interpretation to be the correct one. But I don't think there's anything wrong with my interpretation outside of \"AIME writers wouldn't do that\", and I think relying on \"AIME writers wouldn't do that\" would set a bad precedent.", "Solution_8": "I understand your interpretation, but I think that is the least obvious of the three interpretations I came up with. I came up with the two that you came up with and a third. What if the 17 complete rotations is around the vertex of the cone and the original position means that the cone has to be in the exact same orientation aka the same part of the cone is touching the table? I am not sure whether or not that gets a legitimate answer because I ended up just doing the mainstream one, but it could work because all it would be is that the number of rotations necessary for the cone to make it back to its original orientation, position and rotation of cone, could be 17 rotations if after one rotation a different part of the cone was touching the horizontal plane.", "Solution_9": "[quote=\"watermaximillion\"]I understand your interpretation, but I think that is the least obvious of the three interpretations I came up with. I came up with the two that you came up with and a third. What if the 17 complete rotations is around the vertex of the cone and the original position means that the cone has to be in the exact same orientation aka the same part of the cone is touching the table? I am not sure whether or not that gets a legitimate answer because I ended up just doing the mainstream one, but it could work because all it would be is that the number of rotations necessary for the cone to make it back to its original orientation, position and rotation of cone, could be 17 rotations if after one rotation a different part of the cone was touching the horizontal plane.[/quote]\nAfter every integral number of rotations, the same part of the cone touches the table anyhow, so I'm pretty sure your new interpretation is the same as the mainstream one.\n\n[quote=\"bookaholic\"]I'm not sure I see how I would be \"identifying all the cone's points with each other\". My argument is that from start to finish, each point on the cone moves exactly 360 degrees around the axis perpendicular to the table.[/quote]\nHow can this be? Degrees are a planar measure, and most of the cone's points do not follow a planar path around the perpendicular axis.\n\n[quote]The end result of this is the same as a simple rotation around the axis.[/quote]\nIt is also the same as any integral number of rotations about this axis, as the cone returns to its original position. We could also perform several translations such that each point follows a path topologically equivalent to a circle, and which return each point to its original position. Although the end result is the same as a simple rotation about any axis we choose, I do not think anybody would call such a motion a rotation.\n\n[quote]The fact that the points do not trace a circular path is exactly accounted for by the 16 other rotations.[/quote]\nCould you please explain this sentence more thoroughly?\n\n[quote]Also, the points on the cone's altitude do move in a circle. Since the problem doesn't specify an axis of rotation, I think it's problematic to insist that the points must stay on a single circle, because only the altitude points stay on a circular course without reference to a particular (moving) axis. That course goes around the axis perpendicular to the table. So the problem basically forces everyone to assume something about which axis/axes are being referred to--I just picked an unpopular assumption.[/quote]\r\nThe trouble is that as a point is rotated about a given axis, it traces a circle in a plane perpendicular to the axis of rotation. No point on the cone (except points on the altitude, as you pointed out) traces a circlular path around your perpendicular axis. On the other hand, every point on the cone traces a circlular path around the altitude as the cone---with the altitude---spins around the table.\r\n\r\nAlso, I still do not see what is special about the axis perpendicular to the table. What distinguishes this axis from an arbitrary oblique axis passing through the cone's vertex? Why can't we apply your arguments there?", "Solution_10": "What bookaholic meant by \"The fact that the points do not trace a circular path is exactly accounted for by the 16 other rotations\" is that, if you took out the other 16 rotations (aka the cone is somehow sliding around the table in a circle, with the same side always touching the table), each point on the cone would trace a circular path. Add in the other 16 rotations around the cone's axis, and you get what is happening in the problem.\r\nYou can't do that with just any axis passing through the vertex.", "Solution_11": "[quote=\"Alumen\"]\nYou can't do that with just any axis passing through the vertex.[/quote]\r\nThe entirety of your argument rests on this point, but Boy Soprano's last two paragraphs prove it wrong. [i]Any[/i] point which is rotating will trace a circle around the axis.", "Solution_12": "Okay, now I think I understand the argument, but I still don't think it's really valid, because you do not have unique representation of any motion as a sum of rotations.\r\n\r\nFor an example of this issue, consider a point in the plane rotating a full turn counterclockwise. How many rotations is this? You say it is one rotation, but I say it is three, because I am applying two counterclockwise rotations and one clockwise rotations at the same time; therefore, three rotations.", "Solution_13": "But there is no functional difference between one counterclockwise rotation and two counterclockwise rotations + one clockwise rotation, while there is a functional difference between the cone moving as it does in the problem and rotating in place, which would get rid of the \"extra\" rotation I'm arguing for. The additional rotation is necessary to fully represent the cone's movement.\r\n\r\nP.S. Alumen, thanks for the great explanation!", "Solution_14": "I don't know if this really helps your argument, but a girl at my school interpreted it the same way as you did and got it wrong (answer 256) because of it.", "Solution_15": "[quote=\"Boy Soprano II\"]For an example of this issue, consider a point in the plane rotating a full turn counterclockwise. How many rotations is this? You say it is one rotation, but I say it is three, because I am applying two counterclockwise rotations and one clockwise rotations at the same time; therefore, three rotations.[/quote]\r\nThat's like saying the polynomial 2x+1 has three terms because you can write it as 3x-x+1.", "Solution_16": "[quote=\"bookaholic\"]But there is no functional difference between one counterclockwise rotation and two counterclockwise rotations + one clockwise rotation, while there is a functional difference between the cone moving as it does in the problem and rotating in place, which would get rid of the \"extra\" rotation I'm arguing for. The additional rotation is necessary to fully represent the cone's movement.\n\nP.S. Alumen, thanks for the great explanation![/quote]\r\nOkay, now I have a better idea of what you are saying. I think that your interpretation would be valid if the problem had said something like, \"Seventeen is the smallest value of $ n$ such that the motion of the cone can be expressed as the composition of $ n$ rotations proceeding at the same rate.\" However, I still think this is a bit of a stretch." } { "Tag": [ "logarithms", "real analysis", "real analysis unsolved" ], "Problem": "Can anyone help me with this problem? \r\n\r\n(x2-5x+5)x2-9x+20=1\r\n\r\nI think you're supposed to start with logarithms, but I'm not sure. I'd appreciate any assistance.", "Solution_1": "see here : http://www.mathlinks.ro/Forum/viewtopic.php?t=59286" } { "Tag": [ "inequalities", "function", "inequalities unsolved" ], "Problem": "if $a,b,c$ be positive find the minimum value of $S$\r\n$S=(\\frac {a+b} {c})^2 +(\\frac{a+c}{b})^2 +(\\frac {b+c}{a})^2$", "Solution_1": "[quote=\"payman_pm\"]if $a,b,c$ be positive find the minimum value of $S$\n$S=(\\frac {a+b} {c})^2 +(\\frac{a+c}{b})^2 +(\\frac {b+c}{a})^2$[/quote]\r\n\r\nAre you sure about your question? If it is this way round, then it's trivial:\r\n\r\nFor any three positive numbers a, b, c, we have\r\n\r\n$\\frac{b+c}{a}+\\frac{c+a}{b}+\\frac{a+b}{c}=\\left(\\frac{b}{a}+\\frac{c}{a}\\right)+\\left(\\frac{c}{b}+\\frac{a}{b}\\right)+\\left(\\frac{a}{c}+\\frac{b}{c}\\right)$\r\n$=\\left(\\frac{b}{c}+\\frac{c}{b}\\right)+\\left(\\frac{c}{a}+\\frac{a}{c}\\right)+\\left(\\frac{a}{b}+\\frac{b}{a}\\right)\\geq 2+2+2=6$\r\n\r\n(since $\\frac{x}{y}+\\frac{y}{x}\\geq 2$ for any two positive x and y). Hence,\r\n\r\n$\\frac{\\frac{b+c}{a}+\\frac{c+a}{b}+\\frac{a+b}{c}}{3}\\geq \\frac63=2$,\r\n\r\nand by the AM-QM inequality,\r\n\r\n$\\frac{\\left(\\frac{b+c}{a}\\right)^2+\\left(\\frac{c+a}{b}\\right)^2+\\left(\\frac{a+b}{c}\\right)^2}{3}\\geq \\left(\\frac{\\frac{b+c}{a}+\\frac{c+a}{b}+\\frac{a+b}{c}}{3}\\right)^2\\geq 2^2=4$,\r\n\r\nso that\r\n\r\n$\\left(\\frac{b+c}{a}\\right)^2+\\left(\\frac{c+a}{b}\\right)^2+\\left(\\frac{a+b}{c}\\right)^2\\geq 3\\cdot 4=12$.\r\n\r\nThus, 12 is the required minimum (it is achieved for a = b = c).\r\n\r\n Darij", "Solution_2": "If someone want to complicate it then he should apply Jensen's inequality to the convex fucntion $\\left(\\frac{1-x}{x}\\right)^2$. :lol:", "Solution_3": "the solution is very simple!\r\nwe have \r\n$S=\\frac {a^2}{c}+\\frac{b^2}{c}+\\frac{2ab}{c}+\\frac {a^2}{b}+\\frac{c^2}{b}+\\frac{2ac}{b}+\\frac{b^2}{a}\r\n+\\frac {c^2}{a}+\\frac{2bc}{a}$", "Solution_4": "and by $AM-GM$ inaquality we have \r\n$S\\ge 12$", "Solution_5": "Why did you make THREE posts on it?", "Solution_6": "sorry,it was a mistake !" } { "Tag": [ "geometry solved", "geometry" ], "Problem": "Actually, this problem is from a book of Valentin Vornicu:\r\n\"Olimpiada de Matematica\r\nDe la provocare la experienta\" at a Simulation test for IMO-BMO. Problem 1.\r\nIt's original formulation was:\r\n\" Prove that any triangle can be divided into n isosceles triangles, n>=4\".\r\nThe solution is quite simple, but it doesn't check this following case, and I didn't find such a partition:\r\n\r\nCan an equilateral triangle be partitioned into [i]5[/i] isosceles non- degenerate triangles?", "Solution_1": "Let $ABC$ be the equilateral triangle. Take $P$ inside the triangle s.t. $BPC$ is a right isosceles triangle ($\\angle BPC=\\frac {\\pi}2$). Now take $M\\in (AB),N\\in (AC)$ s.t. $\\angle MPB=\\angle NPC=\\frac {\\pi}{12}=15^{\\circ}$. The triangles we want are $BPC,PMB,PNC,PAM,PAN$." } { "Tag": [], "Problem": "\u039a\u03b1\u03c4\u03b1\u03c3\u03ba\u03b5\u03c5\u03ac\u03c3\u03c4\u03b5 \u03bc\u03b5 \u03c4\u03b7\u03bd \u03bc\u03ad\u03b8\u03bf\u03b4\u03bf \u03c4\u03b7\u03c2 \u03bf\u03bc\u03bf\u03b9\u03bf\u03b8\u03b5\u03c3\u03af\u03b1\u03c2 \u03ad\u03bd\u03b1 \u03b5\u03c5\u03b8\u03cd\u03b3\u03c1\u03b1\u03bc\u03bc\u03bf \u03c4\u03bc\u03ae\u03bc\u03b1 DE, \u03ad\u03c4\u03c3\u03b9 \u03ce\u03c3\u03c4\u03b5 \u03c4\u03bf \u03c3\u03b7\u03bc\u03b5\u03af\u03bf D \u03bd\u03b1 \u03b2\u03c1\u03af\u03c3\u03ba\u03b5\u03c4\u03b1\u03b9 \u03c0\u03ac\u03bd\u03c9 \u03c3\u03c4\u03b7\u03bd AB \u03ba\u03b1\u03b9 \u03c4\u03bf \u0395 \u03c0\u03ac\u03bd\u03c9 \u03c3\u03c4\u03b7\u03bd AC \u03c3\u03b5 \u03b4\u03bf\u03b8\u03ad\u03bd \u03c4\u03c1\u03af\u03b3\u03c9\u03bd\u03bf ABC, \u03ad\u03c4\u03c3\u03b9 \u03ce\u03c3\u03c4\u03b5 BD=DE=EC.", "Solution_1": "\u03a6\u03ad\u03c1\u03bd\u03bf\u03c5\u03bc\u03b5 $ EF\\equal{}\\parallel{}BD$\r\n\u03a4\u03cc\u03c4\u03b5 $ DBEF$ \u03b5\u03af\u03bd\u03b1\u03b9 \u03c1\u03cc\u03bc\u03b2\u03bf\u03c2 \u03ba\u03b1\u03b9 \u03c4\u03bf $ EFC$ \u03b9\u03c3\u03bf\u03c3\u03ba\u03b5\u03bb\u03ad\u03c2 \u03bc\u03b5 \u03ba\u03bf\u03c1\u03c5\u03c6\u03ae \u03c4\u03bf $ E$ \u03ac\u03c1\u03b1 \u03b7 \u03b3\u03c9\u03bd\u03af\u03b1 $ BLF\\equal{}90\\minus{}A/2\\minus{}C$\u03b5\u03af\u03bd\u03b1\u03b9 \u03c3\u03c4\u03b1\u03b8\u03b5\u03c1\u03ae \u03ba\u03b1\u03b9 \u03bf \u03bb\u03cc\u03b3\u03bf\u03c2 $ LF/FE\\equal{}LF/FB\\equal{}const$ \u03b1\u03c1\u03b1 \u03c4\u03bf $ F$ \u03b1\u03bd\u03ae\u03ba\u03b5\u03b9 \u03c3\u03c4\u03bf\u03bd \u03b1\u03c0\u03bf\u03bb\u03bb\u03ce\u03bd\u03b9\u03bf \u03ba\u03cd\u03ba\u03bb\u03bf \u03c4\u03c9\u03bd $ B,C$ \u03bc\u03b5 \u03bb\u03cc\u03b3\u03bf $ LF/FE$ \u03ba\u03b1\u03b9 \u03c4\u03b7\u03bd \u03b7\u03bc\u03b9\u03b5\u03c5\u03b8\u03b5\u03af\u03b1 $ LF$ \u03ba\u03bb\u03c0\r\n\u0398\u03ad\u03bc\u03b1 \u03c4\u03b7\u03c2 \u0395\u039c\u0395 1972 \u03c4\u03b7\u03bd \u03c7\u03c1\u03bf\u03bd\u03b9\u03ac \u03c0\u03bf\u03c5 \u03ad\u03b4\u03c9\u03c3\u03b1 \u03ba\u03b1\u03b9 \u03b5\u03b3\u03ce!", "Solution_2": "Suggnwmh alla mhpws mporeite na dieukrinhsete ti einai to L?\r\nEpishs, poios einai o appolwnios kuklos??? :blush:", "Solution_3": "\u03c6\u03af\u03bb\u03b5 salonikie \u03bf \u0391\u03c0\u03bf\u03bb\u03bb\u03ce\u03bd\u03b9\u03bf\u03c2 \u03ba\u03cd\u03ba\u03bb\u03bf\u03c2 \u03b5\u03af\u03bd\u03b1\u03b9 \u03bf \u03b3\u03b5\u03c9\u03bc\u03b5\u03c4\u03c1\u03b9\u03ba\u03cc\u03c2 \u03c4\u03cc\u03c0\u03bf\u03c2 \u03c4\u03c9\u03bd \u03c3\u03b7\u03bc\u03b5\u03af\u03c9\u03bd \u039c \u03c4\u03bf\u03c5 \u03b5\u03c0\u03b9\u03c0\u03ad\u03b4\u03bf\u03c5 \u03c0\u03bf\u03c5 \u03bf \u03bb\u03cc\u03b3\u03bf\u03c2 \u03c4\u03c9\u03bd \u03b1\u03c0\u03bf\u03c3\u03c4\u03ac\u03c3\u03b5\u03c9\u03bd \u03c4\u03bf\u03c5\u03c2 \u03b1\u03c0\u03cc 2 \u03c3\u03c4\u03b1\u03b8\u03b5\u03c1\u03ac \u03c3\u03b7\u03bc\u03b5\u03af\u03b1 \u0391 \u03ba\u03b1\u03b9 \u0392 \u03b5\u03af\u03bd\u03b1\u03b9 \u03c3\u03c4\u03b1\u03b8\u03b5\u03c1\u03cc\u03c2,\u03b2\u03bb\u03ad\u03c0\u03b5 \u03c3\u03b5\u03bb\u03af\u03b4\u03b1 161 \u03c3\u03c7\u03bf\u03bb\u03b9\u03ba\u03bf\u03cd \u03b3\u03b5\u03c9\u03bc\u03b5\u03c4\u03c1\u03af\u03b1\u03c2 \u0391 \u03b1\u03ba\u03b9 \u0392 \u039b\u03c5\u03ba\u03b5\u03af\u03bf\u03c5", "Solution_4": "r_boris \u03ba\u03b1\u03b9 \u03b3\u03c9 \u03b4\u03b5\u03bd \u03ba\u03b1\u03c4\u03ac\u03bb\u03b1\u03b2\u03b1 \u03c0\u03bf\u03b9\u03cc \u03c3\u03b7\u03bc\u03b5\u03af\u03bf \u03b5\u03af\u03bd\u03b1\u03b9 \u03c4\u03bf L\r\n\u03b5\u03c0\u03af\u03c3\u03b7\u03c2 \u03b4\u03b5\u03bd \u03ba\u03b1\u03c4\u03ac\u03bb\u03b1\u03b2\u03b1 \u03b3\u03b9\u03b1\u03c4\u03af \u03c4\u03bf \u0395F=//BD \u03bc\u03b1\u03c2 \u03b5\u03be\u03b1\u03c3\u03c6\u03b1\u03bb\u03af\u03b6\u03b5\u03b9 \u03cc\u03c4\u03b9 \u03c4\u03bf DBEF \u03b5\u03af\u03bd\u03b1\u03b9 \u03c1\u03bf\u03bc\u03b2\u03bf\u03c2, \r\n\u03b1\u03c5\u03c4\u03cc \u03bc\u03b1\u03c2 \u03b4\u03af\u03bd\u03b5\u03b9 \u03c4\u03bf \u03c0\u03b1\u03c1\u03b1\u03bb\u03bb\u03b7\u03bb\u03cc\u03b3\u03c1\u03b1\u03bc\u03bc\u03bf, \u03b5\u03ba\u03c4\u03cc\u03c2 \u03b1\u03bd \u03be\u03ad\u03c7\u03b1\u03c3\u03b5\u03c2 \u03ba\u03ac\u03c4\u03b9 \u03bd\u03b1 \u03bc\u03b1\u03c2 \u03c0\u03b5\u03b9\u03c2 \u03ae \u03b5\u03af\u03bd\u03b1\u03b9 \u03ba\u03ac\u03c4\u03b9 \u03c0\u03bf\u03c5 \u03b4\u03b5\u03bd \u03c4\u03bf \u03ad\u03c7\u03c9 \u03ba\u03b1\u03c4\u03b1\u03bb\u03ac\u03b2\u03b5\u03b9 \u03c0\u03c9\u03c2 \u03b9\u03c3\u03c7\u03cd\u03b5\u03b9! \r\n\u03cc\u03c0\u03bf\u03c4\u03b5 \u03b2\u03c1\u03b5\u03b9\u03c2 \u03c7\u03c1\u03cc\u03bd\u03bf, \u03c0\u03b5\u03c2 \u03bc\u03b1\u03c2 \u03bb\u03b9\u03b3\u03bf \u03c0\u03b9\u03bf \u03b1\u03bd\u03b1\u03bb\u03c5\u03c4\u03b9\u03ba\u03ac \u03c4\u03b9 \u03b5\u03bd\u03bd\u03bf\u03b5\u03af\u03c2, \r\n\u03c3\u03b5 \u03b5\u03c5\u03c7\u03b1\u03c1\u03b9\u03c3\u03c4\u03ce \u03ba\u03b1\u03b9 \u03c0\u03ac\u03bb\u03b9!\r\n\u03ba\u03b1\u03bb\u03cc \u03b1\u03c0\u03cc\u03b3\u03b5\u03c5\u03bc\u03b1", "Solution_5": "\u03a3\u03c5\u03b3\u03bd\u03ce\u03bc\u03b7 \u03b5\u03af\u03bd\u03b1\u03b9 \u03c1\u03cc\u03bc\u03b2\u03bf\u03c2 \u03c4\u03bf BDEF, \u03c4\u03ce\u03c1\u03b1 \u03c0\u03bf\u03c5 \u03c4\u03b1 \u03be\u03b1\u03bd\u03b1\u03b5\u03af\u03b4\u03b1, \r\n\u03b5\u03af\u03c7\u03b1 \u03c0\u03ac\u03c1\u03b5\u03b9 \u03c4\u03bf \u03c3\u03b7\u03bc\u03b5\u03af\u03bf F \u03c0\u03ac\u03bd\u03c9 \u03b1\u03c0\u03cc \u03c4\u03bf \u0395 \u03b5\u03bd\u03ce \u03b5\u03af\u03bd\u03b1\u03b9 \u03ba\u03ac\u03c4\u03c9 \u03b1\u03c0\u0384\u03c4\u03bf \u0395 \u03c3\u03c4\u03b7\u03bd \u03af\u03b4\u03b9\u03b1 \u03b5\u03c5\u03b8\u03b5\u03af\u03b1 \u03c0\u03bf\u03c5 \u03b5\u03af\u03bd\u03b1\u03b9 \u03c0\u03b1\u03c1\u03ac\u03bb\u03bb\u03b7\u03bb\u03b7 \u03bc\u03b5 \u03c4\u03bf BD \u03ba\u03b1\u03b9 \u03bc\u03bf\u03c5 \u03ad\u03b2\u03b3\u03b1\u03b9\u03bd\u03b5 \u03b5\u03bc\u03ad\u03bd\u03b1 \u03b7 DE \u03c3\u03b1\u03bd \u03b4\u03b9\u03b1\u03b3\u03ce\u03bd\u03b9\u03bf \u03c4\u03bf\u03c5 \u03c0\u03b1\u03c1\u03b1\u03bb\u03bb\u03b7\u03bb\u03bf\u03b3\u03c1\u03ac\u03bc\u03bc\u03bf\u03c5 BDEF \r\n\u03b5\u03bd\u03ce \u03b1\u03bd \u03c4\u03bf \u03c0\u03ac\u03c1\u03b5\u03b9\u03c2 \u03c4\u03bf F \u03c3\u03c4\u03b7\u03bd \u03af\u03b4\u03b9\u03b1 \u03b5\u03c5\u03b8\u03b5\u03af\u03b1 \u03ba\u03ac\u03c4\u03c9 \u03b1\u03c0\u03cc \u03c4\u03bf \u0395 \u03b2\u03b3\u03b1\u03af\u03bd\u03b5\u03b9 \u03c1\u03cc\u03bc\u03b2\u03bf\u03c2 \u03ba\u03b1\u03b9 \u03b7 DE \u03b5\u03af\u03bd\u03b1\u03b9 \u03c0\u03bb\u03b5\u03c5\u03c1\u03ac \u03c4\u03bf\u03c5", "Solution_6": "[quote=\"spinos\"]\u03a3\u03c5\u03b3\u03bd\u03ce\u03bc\u03b7 \u03b5\u03af\u03bd\u03b1\u03b9 \u03c1\u03cc\u03bc\u03b2\u03bf\u03c2 \u03c4\u03bf BDEF, \u03c4\u03ce\u03c1\u03b1 \u03c0\u03bf\u03c5 \u03c4\u03b1 \u03be\u03b1\u03bd\u03b1\u03b5\u03af\u03b4\u03b1, \n\u03b5\u03af\u03c7\u03b1 \u03c0\u03ac\u03c1\u03b5\u03b9 \u03c4\u03bf \u03c3\u03b7\u03bc\u03b5\u03af\u03bf F \u03c0\u03ac\u03bd\u03c9 \u03b1\u03c0\u03cc \u03c4\u03bf \u0395 \u03b5\u03bd\u03ce \u03b5\u03af\u03bd\u03b1\u03b9 \u03ba\u03ac\u03c4\u03c9 \u03b1\u03c0\u0384\u03c4\u03bf \u0395 \u03c3\u03c4\u03b7\u03bd \u03af\u03b4\u03b9\u03b1 \u03b5\u03c5\u03b8\u03b5\u03af\u03b1 \u03c0\u03bf\u03c5 \u03b5\u03af\u03bd\u03b1\u03b9 \u03c0\u03b1\u03c1\u03ac\u03bb\u03bb\u03b7\u03bb\u03b7 \u03bc\u03b5 \u03c4\u03bf BD \u03ba\u03b1\u03b9 \u03bc\u03bf\u03c5 \u03ad\u03b2\u03b3\u03b1\u03b9\u03bd\u03b5 \u03b5\u03bc\u03ad\u03bd\u03b1 \u03b7 DE \u03c3\u03b1\u03bd \u03b4\u03b9\u03b1\u03b3\u03ce\u03bd\u03b9\u03bf \u03c4\u03bf\u03c5 \u03c0\u03b1\u03c1\u03b1\u03bb\u03bb\u03b7\u03bb\u03bf\u03b3\u03c1\u03ac\u03bc\u03bc\u03bf\u03c5 BDEF \n\u03b5\u03bd\u03ce \u03b1\u03bd \u03c4\u03bf \u03c0\u03ac\u03c1\u03b5\u03b9\u03c2 \u03c4\u03bf F \u03c3\u03c4\u03b7\u03bd \u03af\u03b4\u03b9\u03b1 \u03b5\u03c5\u03b8\u03b5\u03af\u03b1 \u03ba\u03ac\u03c4\u03c9 \u03b1\u03c0\u03cc \u03c4\u03bf \u0395 \u03b2\u03b3\u03b1\u03af\u03bd\u03b5\u03b9 \u03c1\u03cc\u03bc\u03b2\u03bf\u03c2 \u03ba\u03b1\u03b9 \u03b7 DE \u03b5\u03af\u03bd\u03b1\u03b9 \u03c0\u03bb\u03b5\u03c5\u03c1\u03ac \u03c4\u03bf\u03c5[/quote]\r\n\u03a3\u03a5\u0393\u039d\u03a9\u039c\u0397 \u03a0\u0391\u0399\u0394\u0399\u0391 \u039f\u03a0\u039f\u03a5 \u0392\u039b\u0395\u03a0\u0395\u03a4\u0395 $ L$ \u0392\u0391\u039b\u03a4\u0395 $ C$", "Solution_7": "Wraia, dhladh to F einai ston appolwnio kuklo twn B,C me logw CF/FE omws den katalavainw pws 8a ginei h kataskeuh thn stimgh pou den gnwrizoume pou vrisketai to shmeio F (afou h diereunush egine me vash ta E,D dedomena). Mporeite na to e3hghsete ligo parapanw?\r\nHlias", "Solution_8": "[quote=\"salonikios\"]Wraia, dhladh to F einai ston appolwnio kuklo twn B,C me logw CF/FE omws den katalavainw pws 8a ginei h kataskeuh thn stimgh pou den gnwrizoume pou vrisketai to shmeio F (afou h diereunush egine me vash ta E,D dedomena). Mporeite na to e3hghsete ligo parapanw?\nHlias[/quote]\r\n\r\n\u0391\u03c6\u03bf\u03cd \u03b7 \u03b3\u03c9\u03bd\u03af\u03b1 BCF \u03b5\u03af\u03bd\u03b1\u03b9 \u03c3\u03c4\u03b1\u03b8\u03b5\u03c1\u03ae \u03b7 \u03b7\u03bc\u03b9\u03b5\u03c5\u03b8\u03b5\u03af\u03b1 CF \u03b5\u03af\u03bd\u03b1\u03b9 \u03b3\u03bd\u03c9\u03c3\u03c4\u03ae. \u0391\u03c5\u03c4\u03cc\u03c2 \u03b5\u03af\u03bd\u03b1\u03b9 \u03bf \u03b4\u03b5\u03cd\u03c4\u03b5\u03c1\u03bf\u03c2 \u03c4\u03cc\u03c0\u03bf\u03c2 \u03c4\u03bf\u03c5 F \u03bf\u03c0\u03cc\u03c4\u03b5 \u03c0\u03c1\u03bf\u03c3\u03b4\u03b9\u03bf\u03c1\u03af\u03b6\u03b5\u03c4\u03b1\u03b9", "Solution_9": "Tr to katalava auto..Alla mperdeuthka se ena allo shmeio. Ton logo CF/FE pws ton gnwrizoume? Afou den exoume prosdiorisei to shmeio F akoma...(Milaw gia thn kataskeuh)\r\nSorry an sas exw prh3ei!\r\n\r\nHlias", "Solution_10": "[quote=\"salonikios\"]Tr to katalava auto..Alla mperdeuthka se ena allo shmeio. Ton logo CF/FE pws ton gnwrizoume? Afou den exoume prosdiorisei to shmeio F akoma...(Milaw gia thn kataskeuh)\nSorry an sas exw prh3ei!\n\nHlias[/quote]\r\n\u03b1\u03c0\u03cc \u03c4\u03bf \u03b3\u03b5\u03b3\u03bf\u03bd\u03cc\u03c2 \u03cc\u03c4\u03b9 \u03c4\u03bf \u03c4\u03c1\u03af\u03b3\u03c9\u03bd\u03bf ECF \u03ad\u03c7\u03b5\u03b9 \u03c3\u03c4\u03b1\u03b8\u03b5\u03c1\u03ad\u03c2 \u03b3\u03c9\u03bd\u03af\u03b5\u03c2 \u03b1\u03c1\u03b1 \u03b1\u03c0\u03cc \u03c4\u03bf \u03b8\u03b5\u03ce\u03c1\u03b7\u03bc\u03b1 \u03b7\u03bc\u03b9\u03c4\u03cc\u03bd\u03c9\u03bd \u03ba\u03b1\u03b9 \u03c3\u03c4\u03b1\u03b8\u03b5\u03c1\u03cc \u03bb\u03cc\u03b3\u03bf \u03c0\u03bb\u03b5\u03c5\u03c1\u03ce\u03bd", "Solution_11": "Wraia to katalava twra! Euxaristw polu!!", "Solution_12": "[quote=\"stelmarg7\"]\u039a\u03b1\u03c4\u03b1\u03c3\u03ba\u03b5\u03c5\u03ac\u03c3\u03c4\u03b5 \u03bc\u03b5 \u03c4\u03b7\u03bd \u03bc\u03ad\u03b8\u03bf\u03b4\u03bf \u03c4\u03b7\u03c2 \u03bf\u03bc\u03bf\u03b9\u03bf\u03b8\u03b5\u03c3\u03af\u03b1\u03c2 \u03ad\u03bd\u03b1 \u03b5\u03c5\u03b8\u03cd\u03b3\u03c1\u03b1\u03bc\u03bc\u03bf \u03c4\u03bc\u03ae\u03bc\u03b1 DE, \u03ad\u03c4\u03c3\u03b9 \u03ce\u03c3\u03c4\u03b5 \u03c4\u03bf \u03c3\u03b7\u03bc\u03b5\u03af\u03bf D \u03bd\u03b1 \u03b2\u03c1\u03af\u03c3\u03ba\u03b5\u03c4\u03b1\u03b9 \u03c0\u03ac\u03bd\u03c9 \u03c3\u03c4\u03b7\u03bd AB \u03ba\u03b1\u03b9 \u03c4\u03bf \u0395 \u03c0\u03ac\u03bd\u03c9 \u03c3\u03c4\u03b7\u03bd AC \u03c3\u03b5 \u03b4\u03bf\u03b8\u03ad\u03bd \u03c4\u03c1\u03af\u03b3\u03c9\u03bd\u03bf ABC, \u03ad\u03c4\u03c3\u03b9 \u03ce\u03c3\u03c4\u03b5 BD=DE=EC.[/quote]\r\n\u039d\u03b1 \u03ba\u03b1\u03c4\u03b1\u03c3\u03ba\u03b5\u03c5\u03b1\u03c3\u03c4\u03b5\u03af \u03c4\u03c1\u03af\u03b3\u03c9\u03bd\u03bf $ ABC$ \u03b1\u03c0\u03cc \u03c4\u03b1 \u03c3\u03c4\u03bf\u03b9\u03c7\u03b5\u03af\u03b1 \u03b3\u03c9\u03bd\u03af\u03b1$ A$, \u03c0\u03bb\u03b5\u03c5\u03c1\u03ad\u03c2 $ a\\plus{}b , a\\plus{}c$\r\n\u0395\u03af\u03bd\u03b1\u03b9 \u03c3\u03c7\u03b5\u03b4\u03cc\u03bd \u03af\u03b4\u03b9\u03b1 \u03ac\u03c3\u03ba\u03b7\u03c3\u03b7", "Solution_13": "[quote=\"r_boris\"][quote=\"stelmarg7\"]\u039a\u03b1\u03c4\u03b1\u03c3\u03ba\u03b5\u03c5\u03ac\u03c3\u03c4\u03b5 \u03bc\u03b5 \u03c4\u03b7\u03bd \u03bc\u03ad\u03b8\u03bf\u03b4\u03bf \u03c4\u03b7\u03c2 \u03bf\u03bc\u03bf\u03b9\u03bf\u03b8\u03b5\u03c3\u03af\u03b1\u03c2 \u03ad\u03bd\u03b1 \u03b5\u03c5\u03b8\u03cd\u03b3\u03c1\u03b1\u03bc\u03bc\u03bf \u03c4\u03bc\u03ae\u03bc\u03b1 DE, \u03ad\u03c4\u03c3\u03b9 \u03ce\u03c3\u03c4\u03b5 \u03c4\u03bf \u03c3\u03b7\u03bc\u03b5\u03af\u03bf D \u03bd\u03b1 \u03b2\u03c1\u03af\u03c3\u03ba\u03b5\u03c4\u03b1\u03b9 \u03c0\u03ac\u03bd\u03c9 \u03c3\u03c4\u03b7\u03bd AB \u03ba\u03b1\u03b9 \u03c4\u03bf \u0395 \u03c0\u03ac\u03bd\u03c9 \u03c3\u03c4\u03b7\u03bd AC \u03c3\u03b5 \u03b4\u03bf\u03b8\u03ad\u03bd \u03c4\u03c1\u03af\u03b3\u03c9\u03bd\u03bf ABC, \u03ad\u03c4\u03c3\u03b9 \u03ce\u03c3\u03c4\u03b5 BD=DE=EC.[/quote]\n\u039d\u03b1 \u03ba\u03b1\u03c4\u03b1\u03c3\u03ba\u03b5\u03c5\u03b1\u03c3\u03c4\u03b5\u03af \u03c4\u03c1\u03af\u03b3\u03c9\u03bd\u03bf $ ABC$ \u03b1\u03c0\u03cc \u03c4\u03b1 \u03c3\u03c4\u03bf\u03b9\u03c7\u03b5\u03af\u03b1 \u03b3\u03c9\u03bd\u03af\u03b1$ A$, \u03c0\u03bb\u03b5\u03c5\u03c1\u03ad\u03c2 $ a \\plus{} b , a \\plus{} c$\n\u0395\u03af\u03bd\u03b1\u03b9 \u03c3\u03c7\u03b5\u03b4\u03cc\u03bd \u03af\u03b4\u03b9\u03b1 \u03ac\u03c3\u03ba\u03b7\u03c3\u03b7[/quote]\r\n\u0391\u03bd \u03c0\u03ac\u03c1\u03bf\u03c5\u03bc\u03b5 \u03b3\u03c9\u03bd\u03af\u03b1 $ xoy\\equal{}A$ \u03ba\u03b1\u03b9 \u03c3\u03c4\u03b9\u03c2 \u03c0\u03bb\u03b5\u03c5\u03c1\u03ad\u03c2 $ ox$ \u03c0\u03ac\u03c1\u03bf\u03c5\u03bc\u03b5 \u03bc\u03ae\u03ba\u03bf\u03c2 $ OB\\equal{}a\\plus{}b$ \u03b5\u03bd\u03ce \u03c3\u03c4\u03b7\u03bd $ oy$ \u03c0\u03ac\u03c1\u03bf\u03c5\u03bc\u03b5 \u03bc\u03ae\u03ba\u03bf\u03c2 $ OC\\equal{}a\\plus{}c$ \u03c4\u03cc\u03c4\u03b5 \u03b7 \u03c0\u03bb\u03b5\u03c5\u03c1\u03ac $ DE\\equal{}a$ \u03c0\u03c1\u03bf\u03c3\u03b4\u03b9\u03bf\u03c1\u03af\u03b6\u03b5\u03c4\u03b1\u03b9 \u03cc\u03c0\u03c9\u03c2 \u03c3\u03c4\u03bf \u03c0\u03c1\u03bf\u03b7\u03b3\u03bf\u03cd\u03bc\u03b5\u03bd\u03bf \u03c0\u03c1\u03cc\u03b2\u03bb\u03b7\u03bc\u03b1" } { "Tag": [], "Problem": "For integers $a,b,c,d > 1$, find the smallest $k$ such that it can be expressed as:\r\n\r\n$k=a^{1}+1=b^{3}+3=c^{5}+5=d^{7}+7$\r\n\r\nCan you generalize, for $n$ equivalent expressions? Will there always be a solution?", "Solution_1": "Well whatever $k$ is, it's something over $10^{25}$... :read:", "Solution_2": "Oh. :rotfl: How'd you find that? \r\n\r\n(What I'm more interested in, tho, is how to go about solving this problem. I have basically no clue, and the only thing I can think of is mods... but I havent gotten that far with them. Any help?)", "Solution_3": "sam, problems that you just make up tend not to have elegant solutions...", "Solution_4": ":rotfl: Aww. Fine. :P Im still interested, though, on any ideas for this problem." } { "Tag": [], "Problem": "After the episode with [url=http://artofproblemsolving.com/Forum/viewtopic.php?t=25734]mutton[/url] \r\nIt\u2019s strict diet for [url=http://www.shopnbc.com/media/products/R/R10427_200.jpg]Button[/url]\r\nAfter five-sevenths of his [url=http://www.crofter.com/graphics/sorrel.jpg]sup'[/url]\r\nHe is left with five-sevenths of a [url=http://www.drdandanner.com/Images/dog%20food.jpg]cup[/url].\r\nHow many cups of food \r\nIs he gettin\u2019 ?\r\n\r\n[url=http://www.anvari.org/db/fun/Cute_Kids/Eating_Dog_Food.jpg]hint: Click here.[/url]\r\n\r\n(You may like to click on the highlighted text - eg \"mutton\" etc above for more clarity ) :)", "Solution_1": "[hide]\nWell, x being the supper size, we have that he ate 5/7 x, and has 2/7 x left. Solving further, we get that 2/7 x = 5/7, and if we multiply each side by 7/2, we get 5/2, so Button only ate 2.5 cups of mutton today.\n[/hide]", "Solution_2": "[hide]\\frac{2}{7}d=5/7\nd=35/14=5/2 cup to start with.[/hide]\r\n\r\nBilly", "Solution_3": "Self deleted.", "Solution_4": "I forgot to reduce.", "Solution_5": "[hide]d=dinner\n2/7x=what he has left\n2/7x=5/7 (cups)\nx=5/2\n>>2.5<<[/hide]", "Solution_6": "[hide]\nX=Dinner\n2/7x=5/7\nx=[b]2.5[/b][/hide]" } { "Tag": [ "function", "integration", "real analysis", "real analysis unsolved" ], "Problem": "$ f: [0,1]\\minus{}>R$ with $ \\int_0^1f(x)dx\\equal{}0$ and let $ n$ be an odd number. Prove that there exists $ x$ s.t. $ f^n(x)\\equal{}\\int_0^xf(t)dt$.", "Solution_1": "I'll assume that $ f$ is continuous; otherwise the statement is false.\r\n\r\nSketch: Suppose that $ f^n(x)>\\int_0^x f(t)\\,dt$ for all $ x\\in [0,1]$. Since $ f(0)>0$, the function $ \\int_0^x f(t)\\,dt$ attains a positive maximum at some point $ x_0\\in (0,1)$. Then $ f(x_0)\\equal{}0$, which contradicts our assumption that $ f^n(x_0)>\\int_0^{x_0} f(t)\\,dt$." } { "Tag": [ "modular arithmetic" ], "Problem": "a and b are positive integers, a is a multiple of 19, b is a multiple of 47 and \r\n1000 = a + b. Find the positvie defference between a and b.", "Solution_1": "[hide]$ 1000\\equiv 13\\equiv 60 \\pmod{47}$. \n$ 19*3\\equiv 10 \\pmod{47}$ therefore\n$ 19*18\\equiv 60 \\pmod{47}$\na is then $ 9*18\\equal{}342$. \nb is $ 1000\\minus{}342\\equal{}658$\n$ 658\\minus{}342\\equal{}316$[/hide]" } { "Tag": [ "linear algebra", "matrix", "linear algebra unsolved" ], "Problem": "Two graphs that are isomorphic have the same vertices and edges. You can obviously draw them differently if you liked by just rearranging the vertices. But how can you prove that two isomorphic graphs can be drawn identically?", "Solution_1": "By isomorphism, if a drawing is a model of one graph, it is a model of the other. That's really the only meaningful question I can see here, and it's trivial.", "Solution_2": "Suppose you have a graph on $n$ vertices. We can associate to the graph a symmetric matrix $A$, with $1$ in the $(j,k)$ location if there's an edge from the $j$th vertex to the $k$th vertex.\r\n\r\n\"Redrawing\" the graph could involve renumbering the vertices. If we do that, then we have a permutation $P$ such that the newly represented graph has matrix $A'=P^TAP.$\r\n\r\nThat would seem to be an appropriate algebraic criterion for graph isomorphism - that the two matrices are similar by a permutation." } { "Tag": [ "inequalities", "trigonometry", "geometry proposed", "geometry" ], "Problem": "[color=darkblue]In any triangle $ABC$ exists the inequality : $8Rr\\le\\boxed{\\frac{a^{3}+b^{3}+c^{3}+abc}{a+b+c}\\le 4R^{2}}\\ ,$ where $2p=a+b+c$ and $R$- the its circumradius.\n\n[b]Remark.[/b] I like it ! The LHS is evidently. It belongs to the \"background\". [/color]", "Solution_1": "ok I have two solution for $LHS$ !Actually $LHS$ is very hard!\r\nThe first: trigonometric method\r\nBy law of sin we can prove that\r\n$\\sum\\sin A\\ge\\sum\\sin^{3}A\\prod\\sin A$\r\n$\\Leftrightarrow\\sum\\sin A\\ge\\frac{1}{4}(3\\sum\\sin A-\\sum\\sin 3A)+\\prod\\sin A\\Leftrightarrow\\frac{1}{4}\\sum\\sin A+\\frac{1}{4}\\sum\\sin 3A\\ge\\prod\\sin A\\Leftrightarrow\\prod\\cos\\frac{A}{2}-\\prod\\cos\\frac{3A}{2}\\ge\\prod\\sin A \\Leftrightarrow\\prod\\cos\\frac{A}{2}-\\prod(4\\cos^{3}\\frac{A}{2}-3\\cos\\frac{A}{2})\\ge\\prod(2\\sin\\frac{A}{2}\\cos\\frac{A}{2}) \\Leftrightarrow\\prod\\cos\\frac{A}{2}-\\prod\\cos\\frac{A}{2}.\\prod(4\\cos^{2}\\frac{A}{2}-3)\\ge 8\\prod\\sin\\frac{A}{2}\\cos\\frac{A}{2}\\Leftrightarrow 1-\\prod(2(1+\\cos A)-3)\\ge8\\prod\\sin\\frac{A}{2}\\Leftrightarrow 1+\\rod(1-2\\cos A)\\ge 2(\\sum\\cos A-1) \\Leftrightarrow 3+1-2\\sum\\cos A+4\\sum\\cos B\\cos C-8\\prod\\cosh\\ge\\sum\\cos A \\Leftrightarrow 4-4\\sum\\cos A+4\\sum\\cos B\\cos C-4\\prod\\cosh\\ge 4\\prod\\cos A \\Leftrightarrow \\prod(1-\\cos A)\\ge\\prod\\cos A(1)$\r\nif triangle $ABC$ isn't acute $\\Rightarrow \\prod\\cos A\\le 0\\le\\prod(1-\\cos A)$ wich is true.\r\nIf triangle $ABC$ acute\r\n$(1)\\Leftrightarrow \\prod(1-\\cos^{2}A)\\ge\\prod\\cos A\\prod(1+\\cos A) \\Leftrightarrow\\prod\\sin^{2}A\\ge\\prod\\cos A\\prod(2\\cos^{2}\\frac{A}{2}) \\Leftrightarrow\\prod\\tan A\\ge\\prod\\cot \\frac{A}{2}$\r\nwhich is true because\r\n$\\sum\\tan A=\\sum\\frac{\\tan B+\\tan C}{2}\\ge\\sum\\tan\\frac{B+C}{2}=\\sum\\cot\\frac{A}{2}$\r\nThe second solution and solution for $RHS$ I will post later, I think $RHS$ is more easlier than $LHS$ :)", "Solution_2": "I don't think the $LHS$ is hard. Here's my approach: \\[8Rr(a+b+c) \\leq a^{3}+b^{3}+c^{3}+abc\\] \\[\\iff 16RS=16R\\cdot \\frac{abc}{4R}\\leq a^{3}+b^{3}+c^{3}+abc\\] \\[\\iff a^{3}+b^{3}+c^{3}-3abc \\geq 0\\] So done.", "Solution_3": "ok Jan sorry I understand wrong between $RHS$ and $LHS$ :D", "Solution_4": "Here's what I would do with the RHS:\r\n\r\n$2R = \\frac{abc}{2S}$ \r\n\r\nSo use the formula of Heroon, and we have to prove that \\[(a^{3}+b^{3}+c^{3}+abc)(a+b-c)(a-b+c)(b+c-a) \\leq 4a^{2}b^{2}c^{2}\\] Now, use the Ravi substitution, and we have to prove that \\[\\sum_{cyc}{4x^{4}(y-z)^{2}}+8(\\sum_{cyc}x^{3}y^{3}+3x^{2}y^{2}z^{2}-\\sum_{sym}x^{3}y^{2}z) \\geq 0\\] And after substitution $xy=\\alpha, xz=\\beta, yz=\\gamma$ we can use Schur and $f(\\alpha)=\\alpha$ to prove that \\[\\sum_{cyc}x^{3}y^{3}+3x^{2}y^{2}z^{2}-\\sum_{sym}x^{3}y^{2}z \\geq 0\\] so we are ready! :lol:", "Solution_5": "[color=darkblue]Apply the inequality $\\sum a\\sin\\frac{A}{2}\\ge p\\mathrm{\\ \\ (*)\\ \\ (\\ see\\ }$ http://www.mathlinks.ro/Forum/viewtopic.php?t=80834 $)$\n\nto the orthic triangle $DEF$ of the triangle $ABC\\ ,$ where $D\\in BC\\ ,\\ AD\\perp BC$ a.s.o. Preliminary :\n\n$\\{\\begin{array}{c}abc=4RS=4Rpr\\\\\\\\ \\sum \\sin 2A=4\\prod\\sin A=\\frac{2S}{R^{2}}\\end{array}$ $\\ldots\\ \\maltese\\ \\ldots$ $\\{\\begin{array}{c}m(\\widehat{EDF})=180^{\\circ}-2A\\\\\\\\ EF=R\\sin 2A=a\\cos A\\end{array}\\ .$ Therefore, from the inequality $(*)$ obtain :\n\n$\\sum EF\\cdot\\sin\\frac{m(\\widehat{EDF})}{2}\\ge \\frac{1}{2}\\cdot\\sum EF$ $\\Longrightarrow$ $\\sum a\\cos A\\cdot\\cos A\\ge \\frac{R}{2}\\cdot\\sum \\sin 2A$ $\\Longrightarrow$ $\\sum a(1-\\sin^{2}A)\\ge \\frac{S}{R}\\ \\ \\|\\ \\bullet\\ 4R^{2}$ $\\Longrightarrow$\n\n$\\sum(4R^{2}a-a^{3})\\ge 4RS$ $\\Longrightarrow$ $\\sum a^{3}\\le 4R^{2}\\sum a\\--abc$ $\\Longrightarrow$ $\\boxed{\\ \\frac{a^{3}+b^{3}+c^{3}+abc}{a+b+c}\\le 4R^{2}\\ }\\ .$\n\n[b]Remark.[/b] With LHS (Left Hand Side) I pulled (up) ... the zip ![/color]" } { "Tag": [ "geometry", "circumcircle", "projective geometry", "power of a point", "radical axis", "geometry proposed" ], "Problem": "Let $ ABC$ be a triangle. A line parallel to $ BC$ intersects the lines $ AB$ and $ AC$ at $ D$ and $ E$. Let $ P$ be a point inside the triangle $ ADE$. The lines $ PB$ and $ PC$ intersect the line $ DE$ at $ M$ and $ N$. The circumcircle of triangle $ PDN$ intersects the circumcircle of triangle $ PEM$ at a point $ Q$ (apart from $ P$). Prove that the points $ A$, $ P$, $ Q$ are collinear.", "Solution_1": "[quote=\"romano\"]Let $ ABC$ be a triangle. A line parallel to $ BC$ intersects the lines $ AB$ and $ AC$ at $ D$ and $ E$. Let $ P$ be a point inside the triangle $ ADE$. The lines $ PB$ and $ PC$ intersect the line $ DE$ at $ M$ and $ N$. The circumcircle of triangle $ PDN$ intersects the circumcircle of triangle $ PEM$ at a point $ Q$ (apart from $ P$). Prove that the points $ A$, $ P$, $ Q$ are collinear.[/quote]\r\n\r\nUnfortunately, all what I have right now is a solution using projective geometry:\r\n\r\nConsider the following theorem which was proven by Grobber in post #2 of http://www.mathlinks.ro/Forum/viewtopic.php?t=5860 :\r\n\r\n[b]Theorem 1.[/b] If P is an arbitrary point in the plane of a triangle ABC, and if a line intersects the lines PA, PB, PC at the points K, M, N and the lines BC, CA, AB at the points F, E, D, then the circumcircles of triangles PKF, PME and PND have a common point apart from the point P.\r\n\r\nApplying this to your configuration, where the line is your line parallel to BC, we see that the point F, where this line intersects BC, is an infinite point, so that the circumcircle of triangle PKF degenerates into a line, namely the line PK, which is, of course, simply the line PA (since the point K lies on the line PA). Thus, instead of saying that the circumcircles of triangles PKF, PME and PND have a common point apart from the point P, we can say that the line PA and the circumcircles of triangles PME and PND have a common point apart from the point P. But the common point of the circumcircles of triangles PME and PND apart from the point P is the point Q; thus, we can conclude that the point Q lies on the line PA, and thus, the points A, P, Q are collinear. $ \\blacksquare$\r\n\r\nUnfortunately, proving Theorem 1 is not so easy (see the thread I linked to).\r\n\r\n Darij", "Solution_2": "The problem is not difficult. Let me give the solution :D \r\n Your goal is to prove $A$ lies on the radical axis of this two circles. Call $K, L$ the second intersection of $(PME)$ and $AC$, $(PDN)$ and $AB$. Now, it is sufficient to prove that $B, C, K, L$ lie on a circle.\r\n We have $(BK, KC) = (BK, KP) + (KP, KC) = (BC,CP) + (PB, BC) = (PB, PC)$, here we notice that $B, K, C, P$ lie on a circle since $PKME$ is cyclic and $ME//BC$. Analogously, we obtain $(BL,LC) = (PB, PC)$. Hence we 're done.", "Solution_3": "Another solution : \r\n Suppose $ DQ $ :cup: $ BP $ at $ R $ , $ EQ $ :cup: $ PC $ at $ S $.\r\n It's easy to show that : $ RS // DE // BC $\r\n Applying Menelaus 's theorem for triangle $ BDR $ ==> $ Q.E.D $", "Solution_4": "the problem still true if P inside the triangle ABC", "Solution_5": "[quote=\"nttu\"]$ RS // DE // BC $ [/quote]\r\n$nttu$ can you show me why this is true?\r\nand can anybody tell me what is the meaning by $(BK, KC)$? :(", "Solution_6": "[quote=\"zhaobin\"][quote=\"nttu\"]$ RS // DE // BC $ [/quote]\n$nttu$ can you show me why this is true?[/quote]\n\nWell, here is how I understand Nttu's solution:\n\nLet the lines DQ and BP meet at R, and let the lines EQ and CP meet at S.\n\nWe will use directed angles modulo 180\u00b0. Since the point Q lies on the circumcircle of triangle PDN, we have < QDN = < QPN; in other words, < (DQ; DE) = < (PQ; CP). Similarly, < (EQ; DE) = < (PQ; BP). Thus,\n\n < (DQ; EQ) = < (DQ; DE) - < (EQ; DE) = < (PQ; CP) - < (PQ; BP) = < (BP; CP).\n\nIn other words, < RQS = < RPS. Thus, the points R, S, Q and P lie on one circle. Consequently, < RSP = < RQP; in other words, < (RS; CP) = < DQP. But since the point Q lies on the circumcircle of triangle PDN, we have < DQP = < DNP, so that < (RS; CP) = < DNP. In other words, < (RS; CP) = < (DE; CP). Consequently, RS || DE. Since we also know that DE || BC, we thus have RS || DE || BC.\n\nNow, from here on, there are two possibilities to complete the solution, i. e. to prove that the points A, P, Q are collinear. Nttu does it somehow using the Menelaos theorem, applied to triangle BDR (I don't know how exactly he does it, but I haven't really tried to understand). Actually, there is a simpler argument: Since RS || DE || BC, the lines RS, DE, BC concur at an infinite point; hence, the triangles RDB and SEC are perspective. Hence, by the Desargues theorem, the points $DB\\cap EC$, $BR\\cap CS$ and $RD\\cap SE$ are collinear (hereby, the abbreviation $g\\cap h$ means the point of intersection of two lines g and h). But $DB\\cap EC = A$, $BR\\cap CS=P$ and $RD\\cap SE=Q$. Thus, the points A, P and Q are collinear. Proof complete.\n\n[quote=\"zhaobin\"]and can anybody tell me what is the meaning by $(BK, KC)$?[/quote]\r\n\r\nIn Treegoner's solution, $\\left(BK,KC\\right)$ means the directed angle between the lines BK and KC (Treegoner works with directed angles modulo 180\u00b0). In my opinion, it is better to use the notation $\\measuredangle\\left(BK,KC\\right)$ (or $\\measuredangle\\left(BK;\\;KC\\right)$) instead.\r\n\r\n darij", "Solution_7": "Lemma 1. \r\nLet X, Y, S, and T be four ciclyc points on a plain. Then if U lies on XS and V lies on YT it holds that UV//ST iff UVYX is ciclyc. \r\n\r\nproof \r\n\r\nEasy. \r\n\r\nGoing to the problem, let L = AB^c(PDN) and let K = AC^c(PME). From lemma 1, as DN//BC it results that LPBC is ciclyc and, for the same reason, KPBC is ciclyc too. Let's now define Q=c(PDN)^c(PME) and R=PQ^c(BCP). From lemma1 it follows that BR//DQ and CR//EQ, so the triangles DEQ and BCR are omothetics the center of this omothetie is A=BD^CE^RQ. Then the thesis.", "Solution_8": "$darij grinberg$,thank you very much.I'll study it when I'm back home." } { "Tag": [ "algebra unsolved", "algebra" ], "Problem": "$x^{5}+y^{5}=a^{2}+b^{2}$", "Solution_1": "It is well known that $t^{2}\\in${0,$\\pm1$}$(mod 5)$ and from Fermat's theorem we have x^5$\\equiv$x$(mod 5)$ $\\Rightarrow$ $x+y\\equiv \\alpha+\\beta (mpd 5)$ where $\\alpha , \\beta \\in{0,\\pm 1}$ $\\Rightarrow$ x+y $\\in$ {0,1,2} because x,y $\\geq$ 0 So we have (x,y) $\\in$ {(0,0),(1,0),(1,1),(2,0)} .\r\n I hope it's right :D", "Solution_2": "I don't think so- for all integers $t,u$, we can set\r\n$x=t^{2}$, $y=u^{2}$, $a=t^{5}$, $b=u^{5}$\r\nto give a solution. :maybe:", "Solution_3": "You only proved that x + y = 0, +/- 1, +/- 2 MOD 5. That is in fact true of every pair of integers!\r\n\r\nSome counterexamples to the result shown include: \r\n(i) x = -y, a = b = 0, so that one of x and y CAN be negative.\r\n(ii) x = y = 2, a = 8, b = 0. \r\n(iii) x = 3, y = 1, a = 10, b = 12.\r\n\r\nThis is a difficult problem I think.", "Solution_4": "Yes you're right ." } { "Tag": [ "algebra unsolved", "algebra" ], "Problem": "Solve the equation:\r\n\\[2\\sqrt{x^{2}+3}-\\sqrt{8+2x-x^{2}}=x\\]", "Solution_1": "[quote=\"nobody1\"]Solve the equation:\n\\[2\\sqrt{x^{2}+3}-\\sqrt{8+2x-x^{2}}=x \\]\n[/quote]\r\nSquare is $x^{2}+x+10=2\\sqrt{(x^{2}+3)(8+2x-x^{2})}.$ It give\r\n$0=5x^{4}-6x^{3}+x^{2}-4x+4=(x-1)^{2}(5x^{2}+4x+4)=(x-1)^{2}[(x+2)^{2}+4x^{2}]$. Therefore only $x=1$." } { "Tag": [ "SFFT", "special factorizations" ], "Problem": "What are all ordered triples of positive integers (x,y,z) whose product is 4 times their sum, if x < y < z?", "Solution_1": "Once x gets to 4, we can divide by 4 to get:\r\nyz=4+y+z \r\ny and z are going to have to be greater than x,\r\nso we think of using 5 and 6. From that, we see\r\nthat xyz will be greater than 4(x+y+z), and as\r\nwe go to bigger values of y and z we see\r\nthat xyz continues to get bigger and bigger\r\ncompared to 4(x+y+z) (when x is equal to 4).\r\nAnother way to see this is if we try 4 in the place of \r\nx, getting 4yz=4(4+y+z). The smallest possible value\r\nof y is 5, so trying this gives us that z=36/16=9/4<3.\r\nThus, we see that x=4 is not a possible case, as z\r\nis automatically smaller than y in the conditions given.\r\nWhen x is greater than 4, xyz is always going\r\nto be great than 4(x+y+z)(for simiilar reasons\r\nas when x=4), so we really only have 3 cases:\r\n[i]x=1[/i]: From this, we have that \r\nyz=4(1+y+z). We see that y has to be greater\r\nthan 4, so we try it for 5, 6, 7...\r\nFor y=6 we get [u]{1,6,14}[/u]\r\nFor y=5 we get: [u]{1,5,24}[/u]\r\nFor y=8 we get [u]{1,8,9}[/u]\r\ntrying y=9 gives us z=8. At this point, since z0, 1/x + 1/y + 1/z = 1\r\n\r\nprobably 3/root(2), need proof.\r\n\r\nLatex doesnt like my computer, sorry. here root(x) = x^0.5", "Solution_1": "[b]Q: find minimum of f(x,y,z) = (x+y+z)/((x-1)^(1/2) + (y-1)^(1/2) + (z-1)^(1/2)) with condition x,y, z > 0 and 1/x + 1/y + 1/z = 1.\nA: The answer is 3/sqrt(2).\nIt is not that hard to see Jensen's pattern here due to the cyclicity of the expression. So quite natural we try Jensen's for h(x) = (x-1)^(1/2) to get h''(x) = -1/4(x-1)^(-3/2) < 0 for x > 1. So Jensen's inequality gives:\nf(x,y,z) >= [(x+y+z)/3]/[((x+y+z)/3 - 1)^(1/2)]. But (x+y+z)(1/x + 1/y + 1/z) >= 9 which implies (x+y+z) >= 9 and hence t = (x+y+z)/3 >= 3.\nSo g(t) = t/((t-1)^(1/2)) has g'(t) = (t-2)/(2(t-1)^(3/2)) > 0 for t >= 3 and therefore f(x,y,z) >= g(t) >= g(3) = 3/sqrt(2) as claimed and that\n\"=\" occurs when t = 3 which yields x=y=z=3.\nand we're done.\n____________________________________________________________[/b]" } { "Tag": [ "counting", "distinguishability" ], "Problem": "Noah's favorite flower shop sells 5 different types of flowers. How many different ways are there to buy 5 flowers if there are no restrictions on how many of each flower may be purchased?", "Solution_1": "[url=http://www.artofproblemsolving.com/Forum/viewtopic.php?t=241220]Double post[/url].", "Solution_2": "You can solve this using the hockey stick identity. \r\n\r\nLet's pretend that Noah organizes the plants by putting the same type of flower on the same table, but no others on that table. Let l represent the tables, and let o represent the flowers. We'll use a diagram to solve this problem. The flowers that are to the left of a table are considered the flowers of that type. \r\n\r\nFor example, if we had the diagram ollloolool, this would mean that there are two tables that don't have any flowers on it, one table that has one flower on it, and two tables that have two flowers on it. \r\n\r\nNow we know that there are a total of $ 5$ l's. There are also a total of $ 5$ o's. We know that one of the tables has to go to the very right, because otherwise, that would mean that there is a flower that doesn't have a table. \r\n\r\nSo we have $ 5$ possible tables for the left most spot. Now there are nine symbols left, but the o's are the only ones that are indistinguishable, so we divide $ 9!$ by $ 5!$, and then multiply that by $ 5$ to get the answer. \r\n\r\nYou can factor out a $ 5!$ from the $ 9!$ to cancel out with the $ 5!$ that is dividing $ 9!$, and you are left with $ 6$, $ 7$, $ 8$, and $ 9$ multiplying each other. We also have to multiply this by five. The product turns out to be; \r\n\r\n\r\n$ 15120$", "Solution_3": "@Wickedestjr - Refer to the other thread for the correct answer and solution. Also, in the future, try not to post in a double-posted thread to avoid having a split discussion, as is the case here.\r\n\r\n[color=red]@mods - Please lock/delete this thread.[/color]" } { "Tag": [ "inequalities", "inequalities proposed" ], "Problem": "[img]http://i10.servimg.com/u/f10/11/07/90/04/sans_t10.jpg[/img]", "Solution_1": "See here http://www.mathlinks.ro/Forum/viewtopic.php?p=469045#469045 and please use English in your post :wink:" } { "Tag": [ "limit", "calculus", "integration", "advanced fields", "advanced fields unsolved" ], "Problem": "Let $ \\lambda_i \\;(i=1,2,...)$ be a sequence of distinct positive numbers tending to infinity. Consider the set of all numbers representable in the form \\[ \\mu= \\sum_{i=1}^{\\infty}n_i\\lambda_i ,\\] where $ n_i \\geq 0$ are integers and all but finitely many $ n_i$ are $ 0$. Let \\[ L(x)= \\sum _{\\lambda_i \\leq x} 1 \\;\\textrm{and}\\ \\;M(x)= \\sum _{\\mu \\leq x} 1 \\ .\\] (In the latter sum, each $ \\mu$ occurs as many times as its number of representations in the above form.) Prove that if \\[ \\lim_{x\\rightarrow \\infty} \\frac{L(x+1)}{L(x)}=1,\\] then \\[ \\lim_{x\\rightarrow \\infty} \\frac{M(x+1)}{M(x)}=1.\\] \r\n\r\n[i]G. Halasz[/i]", "Solution_1": "Solution in PDF\nWe will use upper Darboux integrals see here: http://mathworld.wolfram.com/DarbouxIntegral.html" } { "Tag": [ "MATHCOUNTS" ], "Problem": "Each of the four students on each of 16 MATHCOUNTS teams wins a math book valued at $ \\$125$. What is the total value, in dollars, of all of the math books received by the students?", "Solution_1": "$ 4\\cdot 16\\cdot 125$\r\n$ 4\\cdot 16\\cdot 5\\cdot 25$\r\n$ 4\\cdot 4\\cdot 20\\cdot 25$\r\n$ 4\\cdot 20\\cdot 100$\r\n8000\r\n\r\nassociative + commutative properties of multiplication to 'make' a 100. $ n\\cdot 25$ is just $ \\frac {n}{4}\\cdot 100$ it's obvious why but most people don't realize how useful it is.\r\n\r\nEdit: I fail", "Solution_2": "Since there are $ 4$ students on $ 16$ teams, we have $ 4(16)(125)\\equal{}64(125)\\equal{}\\boxed{8000}$." } { "Tag": [ "IMO", "IMO 2007" ], "Problem": "Dear friends:\r\n\r\nHere is the official website for IMO 2007.\r\n\r\nhttp://imo2007.edu.vn", "Solution_1": "See here http://www.mathlinks.ro/Forum/viewtopic.php?t=131283 :rotfl:", "Solution_2": "WELLCOM TO VIETNAM >_<", "Solution_3": "Hi friends,\r\n\r\nWelcome to IMO at Vietnam, I'm hopefully u will have a nice time and medal here for all of you.\r\n\r\nToday I took an examination that to choose guides for IMO participants. We have to take English test and I took Japanese test too.\r\nThere are many students to join this selection. Hope I will pass. Pray for me, ah aha ah.\r\n\r\nBut anyway, we are preparing for this special festival and we want to send you the message of WELCOME and contact us if you need anything.\r\n\r\nWant to become friends with u all.\r\nEnjoy your days.", "Solution_4": "toato, you have been studying Japanese, gan-batte-kudasai. :) \r\n\r\nkunny", "Solution_5": "Great thanks, hope I can become the Japanese guide. :P \r\nSo happy, see u then in Hanoi" } { "Tag": [ "geometry", "quadratics", "number theory unsolved", "number theory" ], "Problem": "Let $ a, b, c, d$ be positive integers such that $ ab \\equal{} cd$ and $ a\\plus{}b \\equal{} c \\minus{} d.$ Prove that there exists a right-angled triangle the measure of whose sides (in some unit) are integers and whose area measure is $ ab$ square units.", "Solution_1": "[hide]\nWe have $ ab \\equal{} cd$ and $ a \\plus{} b \\equal{} c \\minus{} d$. Substituting gives $ ab \\equal{} d(a \\plus{} b \\plus{} d)$, or $ d^2 \\plus{} (a \\plus{} b)d \\minus{} ab \\equal{} 0$. For this quadratic in $ d$ to be solvable over the integers, we must have the discriminant $ (a \\plus{} b)^2 \\minus{} 4( \\minus{} ab)$ be an integer. So\n\n$ a^2 \\plus{} 6ab \\plus{} b^2 \\equal{} k^2$\n$ (a \\plus{} 3b)^2 \\minus{} 8b^2 \\equal{} k^2$\n$ (a \\plus{} 3b)^2 \\minus{} k^2 \\equal{} 8b^2$\n$ (a \\plus{} 3b \\plus{} k)(a \\plus{} 3b \\minus{} k) \\equal{} 8b^2$\n\nThose two factors are of the same parity, so they must be even.\n\n$ \\left(\\frac {a \\plus{} 3b \\plus{} k}{2}\\right)\\left(\\frac {a \\plus{} 3b \\minus{} k}{2}\\right) \\equal{} 2b^2$\n\nNow if we let those two factors be $ p$ and $ q$, we have $ p \\plus{} q \\equal{} a \\plus{} 3b$ and $ pq \\equal{} 2b^2$. Then we must have $ p \\equal{} 2rs^2$ and $ q \\equal{} rt^2$ for some integers $ r,s,t$. So now $ b \\equal{} rst$ and $ a \\equal{} p \\plus{} q \\minus{} 3b \\equal{} 2rs^2 \\plus{} rt^2 \\minus{} 3rst \\equal{} r(2s \\minus{} t)(s \\minus{} t)$. Then\n\n$ ab \\equal{} r^2 st(2s \\minus{} t)(s \\minus{} t)$\n\nNow set $ m \\equal{} s$ and $ n \\equal{} s \\minus{} t$.\n\n$ ab \\equal{} r^2 m(m \\minus{} n)(m \\plus{} n)n \\equal{} r^2 mn(m^2 \\minus{} n^2)$\n\nNow if $ x \\equal{} r(m^2 \\minus{} n^2)$ and $ y \\equal{} 2mn$, we'll have $ ab \\equal{} \\frac {1}{2}xy$, so $ ab$ will be the area of the triangles whose legs are $ x$ and $ y$. Then the hypotenuse is an integer too by the classic identity\n\n$ r^2(m^2 \\minus{} n^2)^2 \\plus{} r^2(2mn)^2 \\equal{} r^2(m^2 \\plus{} n^2)^2$\n\nSo the triangle with legs $ x$ and $ y$ is the triangle we want.\n\nRemark. We had $ a \\equal{} n(m \\plus{} n) \\equal{} mn \\plus{} n^2$ and $ b \\equal{} m(m \\minus{} n) \\equal{} m^2 \\minus{} mn$. We can also find\n$ c \\equal{} m(m \\plus{} n) \\equal{} m^2 \\plus{} mn$ and $ d \\equal{} n(m \\minus{} n) \\equal{} mn \\minus{} n^2$. Then we have $ ab \\equal{} cd$ and $ a \\plus{} b \\equal{} c \\minus{} d$.\n[/hide]", "Solution_2": "[hide=\"Solution\"]\nBy the Four Numbers Theorem, we can say that $ a \\equal{} wx$, $ b \\equal{} yz$, $ c \\equal{} xy$, $ d \\equal{} wz$. Now, since $ a \\plus{} b \\equal{} c \\minus{} d$, we have that $ a \\plus{} d \\equal{} c \\minus{} b$, so $ w(x \\plus{} z) \\equal{} y(x \\minus{} z)$. By the Four Numbers Theorem, we can say that $ w \\equal{} kl$, $ x \\plus{} z \\equal{} mn$, $ y \\equal{} lm$, and $ x \\minus{} z \\equal{} nk$. Then, we get that $ x \\equal{} \\frac {nk \\plus{} mn}{2}$ and $ z \\equal{} \\frac {mn \\minus{} nk}{2}$. It follows that\\[ ab \\equal{} xyzw \\equal{} \\frac {(nk \\plus{} mn)(mn \\minus{} nk)(lm)(kl)}{4} \\equal{} \\frac {n^2l^2mk(m \\plus{} k)(m \\minus{} k)}{4}\\]Letting $ \\frac {m^2 \\minus{} k^2}{2} \\equal{} t$ and $ u \\equal{} mk$, so $ u$ and $ t$ are the legs of a right triangle. Scaling everything up by a factor of $ nl$, we get that $ t' \\equal{} \\frac {nl(m^2 \\minus{} k^2)}{2}$ and $ u' \\equal{} mkln$, so $ u'$ and $ t'$ are still the legs of a right triangle with integer sidelengths. Then, $ \\frac {t'u'}{2} \\equal{} \\frac {mkl^2n^2(m^2 \\minus{} k^2)}{4} \\equal{} ab$, so the area of the triangle is equal to $ ab$, as desired. [/hide]" } { "Tag": [ "calculus", "search", "calculus computations" ], "Problem": "see attachement", "Solution_1": "1) It was alredy disscused. Try a search about \"condensation test\"." } { "Tag": [], "Problem": "A store normally sells windows at $ \\$100$ each. This week the store is offering one free window for each purchase of four. Dave needs seven windows and Doug needs eight windows. How many dollars will they save if they purchase the windows together rather than separately?\r\n\r\n$ \\textbf{(A)}\\ 100 \\qquad \\textbf{(B)}\\ 200 \\qquad \\textbf{(C)}\\ 300 \\qquad \\textbf{(D)}\\ 400 \\qquad \\textbf{(E)}\\ 500$", "Solution_1": "[hide=\"Click for solution\"]\nWhen Dave buys seven windows separately he'll purchase six, and get one for free, for $ \\$600$. When Doug buys eight windows separately, he'll purchase seven and get one for free, for a total cost of $ \\$700$. The total cost for them purchasing separately is $ \\$1300$, whereas if they purchase fifteen together, they need to purchase $ 12$ windows for $ \\$1200$. The savings is $ \\$100$ or $ \\boxed{\\textbf{(A)}}$.\n[/hide]" } { "Tag": [ "geometry", "3D geometry", "factorial", "function", "algorithm", "symmetry", "calculus" ], "Problem": "Solve in integers :\r\n\r\n(1) 2n = a! + b! + c! ;\r\n(2) a! b! = a! + b! + c! ;\r\n(3) abc + ab + bc + ca + a + b + c = 1000 ;\r\n(4) 2a + 3b = c2 ;\r\n(5) a2 + b2 = abc - 1 ;\r\n(6) y2 = x5 - 4 ;\r\n(7) :Sigma:1:le:k:le:nk2 = m2n ;\r\n(8) m2 = n(n + p) where p is an odd prime ;\r\n(9) 49 + 20(61/3) = (m1/3 + n1/3 - 1)2 ;\r\n(0) x2 + 4y = s2, y2 + 4x = t2.\r\n\r\n\r\nHappy solving :)", "Solution_1": "um...wow. i'll take them one at a time\n\n\n\n(1) [hide]let a:le:b:le:c. Then we can factor out an a! to get a!+b!+c!=a!(1+(a+1)(a+2)...b+(a+1)(a+2)...c). Well, if this is to be a power of 2, then both factors must be powers of 2. So, if a! is a power of 2, a is either 0, 1, or 2. Alright. Note that a!+b!+c!:ge:3, so n cannot be 0 or 1. The only way n can be 2 is if a=b=0 or 1 and c=2. Okay, now mod 24, all 2^n, n:ge:3, are :equiv:8 or 16. a!:equiv:1 or 2, and b! and c!:equiv:1, 2, 6, or 0. The only possible solution here is a=2, b=3, and c:ge:4. c!=8(2^(n-3)-1). This means c! has exactly 3 factors of 2, i.e. 4:le:c:le:5. Thus, the only possible quadruples (a,b,c,n) are (0,0,2,2), (0,1,2,2), (1,1,2,2), (2,3,4,5), (2,3,5,7) and permutations of the first 3 variables. correct?[/hide]", "Solution_2": "u r pure eval", "Solution_3": "I won't pass up on an opportunity to use Simon's favorite factoring trick:\r\n\r\n3) I assume this should be solved in positive integers. (a+1)(b+1)(c+1)=1001. Assume a<=b<=c. a+1=7, b+1=11, c+1=13, so (a,b,c)=(6,10,12) (and its rotations). If it's supposed to be nonnegative integers, one would have to deal with the cases with a+1=1 and b+1=1. If it includes negative integers, there are several more cases, but the idea is the same.", "Solution_4": "(2) [hide]still working on it[/hide]\n\n\n\n(3) [hide]factor as (a+1)(b+1)(c+1)=7*11*13. just factor and subract one to get all possibilities...[/hide]\n\n\n\n(4) [hide]if a:ge:2, we can write it as c:^2:-3^b=2^a. A mod 4 analysis shows us that b is even, i.e. b=2x. Then (c-3^x)(c+3^x)=2^a. Then, if c-3^x=2^m and c+3^x=2^n, 3^x=(2^n-2^m)/2=2^(m-1)(2^(n-m)-1). This will only be a power of 3 if m=1 and n-m=1 or 2, i.e. if n=2 or 3. Thus, the only solutions in this case are (a,b,c)=(3,0,:pm:3), (4,2,:pm:5). \n\nNow, if a=0, we have (c-1)(c+1)=3^b, meaning (a,b,c) is (0,1,:pm:2). If a=1, we have 2+3^b=c:^2:. If b>0, mod 3 gives us c:^2::equiv:2 mod 3, which is impossible. b=0 gives us a nonsquare. So there are no solutions in this case. Therefore, all solutions are (3,0,:pm:3), (4,2,:pm:5), and (0,1,:pm:2) Correct?[/hide]", "Solution_5": "hmmm...(5)[hide](a+b):^2:+1=ab(2+c) => c=(a+b):^2:/ab - 2. If c is an integer, then ab|(a+b):^2:. Therefore a|(a+b):^2: and b|(a+b):^2: => a|b:^2: and b|a:^2: not sure where to go from there[/hide]\n\n\n\n(6)[hide]mod 5 analysis shows x:equiv:0, 3, 4 mod 5. Hmmm...what else can I say...[/hide]", "Solution_6": "(7) [hide]So, the sum of the squares from 1 to n is n(n+1)(2n+1)/6 = m:^2:n\n\nSo (n+1)(2n+1)/6 = m:^2:\n\nSo 6|(n+1)(2n+1). This means either (6|n+1) or (6|2n+1) or (3|2n+1 and 2|n+1). \n\nCase 1: 6|n+1\n\nNow, (n+1) and (2n+1) are going to be relatively prime, which means that (n+1)/6 and (2n+1) would have to be perfect squares. However, if 6|n+1, 2n+1 :equiv: 5 (mod 6) which is never a perfect square. Throw out this case.\n\nCase 2: 6|2n+1\n\nThis never works. That was easy.\n\nCase 3: 3|2n+1 and 2|n+1. This gives us that n :equiv: 1 (mod 6). \n\nn = 6j + 1. m:^2: = (6j + 2)(12j + 3)/6 = (3j + 1)(4j + 1). These two numbers are relatively prime to each other (since they are relatively prime to their common difference), so we have that 3j + 1 and 4j + 1 are both perfect squares. Hmm. What can I do with this? I'm sure someone else knows how to finish this off.[/hide]", "Solution_7": "(8) m:^2: = n(n + p) where p is an odd prime\n\n[hide]\n\n4m:^2: + p:^2: = 4n:^2: + 4np + p:^2:\n\np:^2: + (2m):^2: = (2n + p):^2:\n\n\n\nNow, we resort to the parametrized form of Pythagorean triples, which looks roughly like d(u:^2: - v:^2:), 2uvd, and d(u:^2: + v:^2:) for u, v relatively prime positive integers, u > v. Now, p is an odd prime and one of the legs, so p = 2uvd (which fails on odd=even) or \n\np = d(u:^2: - v:^2:), which factors as p = d(u+v)(u-v). This implies\n\n u + v = p, u - v = 1, d = 1. Then\n\n\np = 2v + 1\n\n2m = 2v(v+1)\n\nm = v:^2: + v\n\n2n + p = v:^2: + (v+1):^2:\n\n2n + 2v + 1 = 2v:^2: + 2v + 1\n\nn = v:^2:.\n\n\n\nThus, all solutions are (m, n, p) = (v:^2: + v, v:^2:, 2v + 1) provided that 2v + 1 is prime.[/hide]", "Solution_8": "(9) [hide]Let s = 6^(1/3). Okay, so, ignore the right side for a moment and just look for the square root of 49 + 20s. Guess that it has the form a + bs + cs:^2:, where a, b, and c are relatively nice looking numbers. I don't know if you can prove that this must work. I don't think you can. But anyhow, guess that it works. Square it out. Get the equations\n\nb:^2: + 2ac = 0\n\na:^2: + 12bc = 49\n\n3c:^2: + ab = 10\n\n\n\nAll right. So this is totally non-legit. I guessed that bc = 4, because that would make me very happy because it would give me a nice solid a = :pm:1. And shazaam! it works. We get solutions (a, b, c) = :pm:(-1, 2, 2). (This is not unique. There are more. But that's okay. They aren't nice-looking.) Now, take the member where a = -1, not positive 1, because we need m and n to be positive. This gives me the sums of the cube roots of m and n is equal to twice the cube root of 6 plus twice the cube root of 36. This should give you that {m, n} = {48, 288}. I would be willing to wager that this is unique, but I definitely can't prove it.[/hide]", "Solution_9": "7. [hide]n(n+1)(2n+1)/6=m:^2:n, as n:ge:1, (n+1)(2n+1)=6m:^2: as n is odd, we replace n by 2a-1 to get a(4a-1)=3m:^2: a:equiv:0 or 1 mod 3. If a:equiv:1 mod 3, we can write it as (3k+1)(4k+1)=m:^2:. well, gcd(k,3k+1)=1, so gcd(3k+1,4k+1)=1, so 3k+1 and 4k+1 must both be perfect squares. ok, so if 3k+1=x:^2:, then 4k+1=(4x:^2:-1)/3=(2x-1)(2x+1)/3 => if x=3y+1, then(6y+1)(2y+1) is a perfect square blah this isn't going anywhere.[/hide]", "Solution_10": "edit: wurg", "Solution_11": "8. [hide]either gcd(n,p) is 1 or gcd(n,p) is not 1. (duh) If gcd(n,p) is 1, then gcd(n,n+p) is also 1. Then n and n+p must both be perfect squares. So n+p=b:^2: and n=a:^2: => p=(b-a)(b+a) => a=(p-1)/2 => n=(p-1):^2:/4. This makes n+p=(p+1):^2:/4 and m=(p-1)(p+1)/4. This will be an integer, as p is odd so both p-1 and p+1 will have a factor of 2. So, one solution will be (m,n,p)=((p:^2:-1)/4,(p-1):^2:/4,p), where p can be any odd prime.\n\n\n\nNow, if gcd(n,p) is not 1, n=kp. That means m:^2:=kp(k+1)p=k(k+1)p:^2:. This means k(k+1) is a perfect square. However, k(k+1)=k:^2:+k is always between k:^2: and (k+1):^2:=k:^2:+2k+1, for positive k, so k(k+1) cannot be a perfect square. So no solutions in this case. Thus, all solutions are (m,n,p)=((p:^2:-1)/4,(p-1):^2:/4,p), where p is any odd prime. Correct?[/hide]", "Solution_12": "Seeing all those factorials reminded me of this cool little problem:\r\n\r\nProve that there are infinitely many solutions (positive integers) for a!*b!=c!. I've struggled with it for a long time ( :D :oops: believe it or not), but it's REALLy easy.", "Solution_13": "There's only one ``nontrivial'' solution as far as we know: 10!=7!6!. You can get a lot more by taking (n!)!=(n!-1)!*n!. I think there's another class that generates infinitely many, but I can't remember what it is.", "Solution_14": "1!*n! = n! ? Or are you thinking of something slightly less trivial?", "Solution_15": "Hehe ... some comments.\n\n\n\n(1) Taken from the dutch 'Pythagoras' contest 2003.\n\nSolved by Mysticterminator.\n\n\n\n(2) British Mathematical Olympiad 2003 (First round).\n\nNo solution not yet. \n\nHint : [hide]Consider the cases a < b and a = b.[/hide]\n\n\n\n(3) Taken from the dutch 'Pythagoras' contest 2002.\n\nEasy problem, solved by Complexzeta & Mysticterminator.\n\n\n\n(4) British Mathematical Olympiad 1996.\n\nSolved by Mysticterminator. \n\nhttp://www.kalva.demon.co.uk/bmo/bsoln/bsol961.html\n\n\n\n(5) I found this on the mathlinks forum. It's quite difficult.\n\nNo solution not yet. \n\nHint : [hide]First prove that c must equal 3.[/hide]\n\n\n\n(6) Aha ... Interesting. I will mention the source later \n\nNo solution not yet. \n\nHint : [hide]Modular arithmetic. If p is a prime then (p - 1)/2 is interesting ...[/hide]\n\n\n\n(7) Adapted from British olympiad 1994.\n\nPartial solution by JBL.\n\nMysticterminator is wrong here since n = 337 works.\n\n\n\n(8) British olympiad 1992.\n\nSolved by JBL and Mysticterminator.\n\n\n\n(9) British olympiad 1999, adapted. \n\nAn attempt by JBL.\n\nA hint : [hide]Compare rational and irrational parts of both sides.[/hide]\n\n\n\n(0) I will mention my source later \n\nNo solution not yet.", "Solution_16": "MysticTerminator -- your solution to number 7 is wrong because when you substituted n = 2a -1, you replaced (n + 1) with (2a - 1) instead of 2a.", "Solution_17": "for those you say you will mention the source later, they had better not be IMOs. ;) (well, at least not the recent ones)\r\n\r\nwurg i knew 7 must have been too easy. :(", "Solution_18": "Arne wrote:Solve in integers :\n(9) 49 + 20(61/3) = (m1/3 + n1/3 - 1)2 ;\n(0) x2 + 4y = s2, y2 + 4x = t2.\n\n(9)[hide]m and n can be written as a:^3:*b and c:^3:*d:^2:. I'm pretty sure that's the only way it would work. Then multiplying everything out gives you:\n\n49+20*6^(1/3)=a:^2:b^(2/3)+c:^2:dd^(1/3)+1\n\n+2acb^(1/3)d^(2/3)-2ab^(1/3)-2cd^(2/3). All the things with something to the (2/3) has to cancel, and we equate the things to the (1/3) and the real numbers:\n\n49=1. okay, this isn't right. this means that there is another term that produces a real number. this must be the 2acb^(1/3)d^(2/3) term, meaning that b=:pm:d. So we replace all the d's by :pm:b's, and now we set the real and irrational terms equal:\n\n\n\n49=1+2abc (we don't need to worry about :pm: since the d term is squared)\n\na:^2:=2c (again we don't need to worry about :pm:)\n\n20*6^(1/3)=b^(1/3)*(bc:^2:-2a) (again, we don't need to worry about :pm: since the d term is to the 4/3 power, which is even, sort of)\n\n\n\nokay, since there is only one term to the 1/3 power in the RHS of the last equation, b must equal :pm:6. Now we get:\n\n\n\n:pm:4=ac\n\na:^2:=2c\n\n:pm:20=:pm:6c:^2:-2a\n\n\n\nwow lot of :pm:s, but we can solve from here. I think all possible solutions are as follows (a,b,c)=(:pm:2,:pm:6,2) Now we remember that d=:pm:b=:pm:6 and get our original solutions:\n\n(m,n)=(:pm:48,288). correct?[/hide]\n\n\n\nedited: cool i followed your hint even though i didn't look at it. and i got the same answer as JBL, but i shouldn't have had the :pm:. Hmm...why did i have it then? my steps seem legit...\n\n\n\n(0)[hide](x-y)(x+y-4)=(s-t)(s+t)\n\n(x+2)^2+(y+2)^2=s^2+t^2+8\n\nx&s have same parity\n\ny&t have same parity [/hide]", "Solution_19": "(5) a:^2: + b:^2: = abc - 1\n\n[hide]c = (a:^2: + b:^2: + 1)/ab is an integer. Thus\n\nb | a:^2: + 1\n\na | b:^2: + 1.\n\n\n\nEither: wlog a>b or a=b.\n\nIf a=b, 2a:^2: = ca:^2:-1\n\n1 = (c - 2)a:^2:\n\na = b = 1, c = 3.\n\n\n\nIf a>b,\n\na:^2: > b:^2:\n\na:^2: >= b:^2: + 2b + 1 > b:^2: + 1\n\nk = (b:^2: + 1)/a < a.\n\n\n\nLet us now examine the ordered triple (k, b, c). I claim that this ordered triple also satisfies the given equation.\n\n\n\nProof:\n\nb:^4: + 2b:^2: + 1 + a:^2:b:^2: = (a:^2: + b:^2: + 1)(b:^2: + 1) - a:^2: by multiplication and other fun stuff.\n\n(b:^2: + 1):^2: + a:^2:b:^2: = abc(b:^2: + 1) - a:^2:, since (a, b, c) satisfies our given relation.\n\n((b:^2: + 1)/a):^2: + b:^2: = bc((b:^2: + 1)/a) - 1 by division\n\nk:^2: + b:^2: = kbc - 1\n\nWhich is what we wanted.\n\n\n\nSo. That means that from any ordered pair, we can get an ordered pair with a smaller value for one of the coefficients, provided they are not equal. Note that this process is exactly reversible -- if we use the process we used to generate (k, b, c) from (a, b, c) on (k, b, c), we get right back to (a, b, c). (That is, the function is its own inverse.) Now, start from any ordered pair with a > b. Use our process to generate a pair with a smaller value a' for a. Now, either a' > b, a' < b, or a' = b. In the latter case, we must now have the ordered triple (1, 1, 3). In the former cases, we can continue to repeat the algorithm to generate smaller triples. Since all members are integers, we must end up, no matter what, with the ordered triple (1, 1, 3). But, as noted above, the process is reversible. Thus, every solution can be found by using the inverse process starting with (1, 1, 3) and building up. Thus, our first few triples are (1, 1, 3), (2, 1, 3), (5, 2, 3), (13, 5, 3), (34, 13, 3), etc.\n\nThe pattern is easy to see (think Fibonacci numbers) and can be proven with an inductive argument that should present no problem.\n\n\n\nNote: if the problem read had a plus one instead of a minus, we would have solutions using the alternate pairs of Fibonacci numbers such as (8, 21, 3), although I don't know offhand if those are unique.[/hide]", "Solution_20": "You know what -- for number 9, I knew I could say that, and then I thought, \"Wait, but what if the cube root of m is the sum of an integer and a rational multiple of the cube root of six,\" because I forgot that it was a diophantine equation! Bother! I think if I had tossed that in, it would have essentially rigorized my solution, or at least half of it.", "Solution_21": "For (9) I did the following. The given relation is equivalent with\r\n\r\n48 + 20(61/3) = m2/3 + n2/3 + 2(mn1/3) - m1/3 - n1/3.\r\n\r\nConsider the following cases :\r\n\r\n(I) Both m and n are perfect cubes.\r\nContradiction since the LHS is irrational and the RHS is rational.\r\n\r\n(II) (WLOG) m is a perfect cube but n is not, write m = k3.\r\nThen 48 + 20(61/3) = k2 + n2/3 + 2k(n1/3) - k - n1/3.\r\nComparing rational and irrational parts we get :\r\nk2 - k = 48\r\n... = 20(61/3)\r\nbut there does not exist an integer k such that k2 - k = 48.\r\nSo we have a contradiction.\r\n\r\n(III) None of m and n is a perfect cube.\r\nThen mn MUST be a perfect cube. (See Joel's remark :)).\r\nComparing rational and irrational parts we get : \r\n48 = 2(mn1/3)\r\n20(61/3) = m2/3 + n2/3 - m1/3 - n1/3.\r\nNow we have mn = 2933.\r\nWrite m = 2a3b, n = 2c3d.\r\nNow only routine work remains. It's easy to check all cases.\r\nThen (m, n) = (288, 48) easily comes out as the only answer.\r\n\r\n\r\nPS. The problems for which I didn't mention the source are certainly not IMO problems ...", "Solution_22": "Still unsolved :\r\n\r\n(2)\r\n(6)\r\n(7) -> Partial solution\r\n(0)\r\n\r\nPS. Nice work on (5), JBL !", "Solution_23": "Thanks, Arne. The argument took me a long time playing around to get it to work -- then, when I was looking back at my notes, I couldn't even figure out how I had realized that b and (b:^2: + 1)/a would be a solution in the first place. \n\nI really like how the structure of the solution is, though -- the reductio argument plus a reversible algorithm . . . I thought it was very pretty.\n\n\n\n(0) x:^2: + 4y = s:^2:, y:^2: + 4x = t:^2:. \n\n[hide]So, I would like to claim that there are no solutions. As MysticTerminator said, we can see from the given equations that the parities of s and x must be the same, ditto for y and t. Also, being diophantine equations, x, y > 0, which implies that s > x, \n\nt > y. These two conditions give us that s = x + 2c, t = y + 2d. Substituting these values into the given equations, we get\n\nx:^2: + 4y = (x + 2c):^2:\n\nx:^2: + 4y = x:^2: + 4cx + 4c:^2:\n\ny = cx + c:^2: > x.\n\nBut by symmetry, we get that x > y. This is a contradiction. Thus, there are no positive integral solutions (x, y, s, t) to the given system.[/hide]", "Solution_24": "For problem (0), x and y can be negative or zero !", "Solution_25": "Darn. I thought that might be the case.", "Solution_26": "For (6), a hint : [hide]2(5) + 1 = ?[/hide]\n\n(Remember my previous hint for (6) :\n\n[hide]If p is prime then (p - 1)/2 is interesting[/hide].\n\nMaybe that hint was a bit unfair )\n\nTo conclude, worth remembering :\n\n[hide]If 2k + 1 is prime, then it can be useful to look at ak (mod 2k + 1).[/hide]", "Solution_27": "Arne wrote:Solve in integers :\n(2) a! b! = a! + b! + c!\nHappy solving \n\n\n\n(2) [hide]let us assume, wlog, that a:le:b. then we see that a:le:c:le:b, since a!|LHS and a!|(a!+b!); also, b!|LHS and the 2nd term of the RHS, but not the 1st term, so b cannot divide c!. okay, so we divide everything by a! we get b!=1+(a+1)*...*b+(a+1)*...*c. Well, let us assume for the moment that b:ge:2. Then the LHS is even. That means the sum of the last two terms of the RHS is odd. So one term is odd and one term is even. Well, if the 3rd term is even, the 2nd term is definitely even, so the 3rd term is odd and the 2nd term is even. ok, so the 3rd term will be odd if c=a or if c=a+1 and a is even. \n\n\n\nokay, if c=a+1, then b:ge:a+2 (else the 2nd term of the RHS would also be odd). Plugging in c=a+1 gives us b!=(a+2)+(a+1)*...*b . Cancelling an (a+2) gives us 1*2*...*(a+1)*(a+3)*....*b=1+(a+1)*(a+3)*...*b. The LHS is even, so the 2nd term of the RHS must be odd. Remember, a is even, so this means b=a+2 or a+3. b=(a+2) gives the solution a=0, and b=a+3 gives no solutions (the term a+3 has to divide every term, as it divides the LHS and the 2nd term of the RHS, but it obviously does not divide 1 for a:ge:0). \n\n\n\nOkay, now we go back and look at the case c=a. this gives us 2=[(a+1)*...*b](a!-1). the second factor in the RHS can only divide 2 if a=2. This gives 4=b!. darn no solutions.\n\n\n\nOkay, now we go back and check the cases b=0, 1. This gives us 1=1+1+c!. Okay, no solutions. \n\n\n\nIt could be cleaned up from there, but is that the general idea, or have I made a fatal mistake (which I suspect I did), like in #7? [/hide]", "Solution_28": "Here is my solution to (6) y2 = x5 - 4, following Arne's generous hint.\n\n [hide]To see why there are no integer solutions to the equation, take mod 11 of both sides. First, I claim that x5 is equivalent to -1, 0, or 1 mod 11. We can show that either by trying all x from -5 to 5, or by using Fermat's Little Theorem, which implies that 11 divides x11 - x = x(x5-1)(x5+1). Therefore the right-hand side of the original equation is equivalent to 6, 7, or 8 mod 11. Second, I claim that the left-hand side, y2, is equivalent to 0, 1, 3, 4, 5, or 9 mod 11, which we can see by trying all y from -5 to 5. Thus the two sides of the equation are different mod 11, so they can't be equal. [/hide]", "Solution_29": "Mysticterminator, you did something wrong for (2).\n\n\n\nVerify that (3, 3, 4) is a solution ! It is the only solution in fact.\n\n\n\nHint : [hide]If a < b then there it's quite easy to prove that there are no solutions at all. The case a = b is more difficult.[/hide]", "Solution_30": "(6) isn't IMO at all, it is problem 4 of the Balkan Olympiad 2000.\r\nThere's a solution at [url]http://www.kalva.demon.co.uk[/url].\r\nThat's why I didn't mention the source :)", "Solution_31": "The kalva site says that the problem is from Balkan 1998, not 2000.", "Solution_32": "Hm ... Sorry.", "Solution_33": "OK, Should I post solutions for the remaining problems ?\r\nI think no one is really interested anymore ...", "Solution_34": "wurg no don't. really. i still haven't given in and looked at your hints.", "Solution_35": "Thanks. Still unsolved :\r\n\r\n(2) C'mon guys ! It's not that difficult. \r\n(7) Partial solution.\r\n(0) Some preliminary work is done.", "Solution_36": "A hint for (0) :\r\n\r\nThe case where x, y > 0 is done by JBL.\r\nThe case x < 0 < y is not so difficult.\r\nIt is more difficult to deal with the case x, y < 0.\r\n\r\nA hint for (2) :\r\n\r\n[i]If a and b are distinct then WLOG a > b.\nSo a! does not divide b!.\nHence ... does not divide a!b! - a! - b!.\nHence ... does not divide ... (from the given relation).\nHence ... > ... .\nSo a!b! = a! + b! + c < ... or (...)(...) < ... .\nThis gives a few easy cases to check.\n...\nSo there are no solutions for distinct a and b.\nConsider now the case a = b.[/i]\r\n\r\nOK ?", "Solution_37": "I'll post solutions soon since no one seems to be still interested.", "Solution_38": "For (2) :\r\n\r\nSuppose a > b. Then a! does not divide b!.\r\nHence c! = a!b! - a! - b! is not divisible by a!.\r\nHence a > c. So a!b! = a! + b! + c! < 2a! + b!.\r\nHence (a! - 1)(b! - 2) < 2. \r\nIt is easy to check (routine) that this gives no solutions.\r\n\r\nSo we must have a = b. Then a! = 2a! + c!.\r\nHence a! divides c! and therefore a < c.\r\nSo we write c = a + k where k > 0 is an integer.\r\nDivision by a gives a! = 2 + (a + 1)(a + 2)...(a + k).\r\nObviously RHS > 2 so a! > 2 => a > 2 => 3|a!.\r\nSo we must have (a + 1)(a + 2)...(a + k) = 1 (mod 3).\r\nHence k < 3. (If k > 2 then 3|(a + 1)(a + 2)...(a + k).)\r\nIf k = 1 then we get a! = a + 3. This gives a = 3.\r\nIf k = 2 then we get a! = a + 3a + 4.\r\nThen a((a - 1)! - a - 3) = 4 => a|4. So a = 1, a = 2 or a = 4. \r\nIt is a routine job to check that these cases fail.\r\n\r\nTherefore the only solution is (a, b, c) = (3, 3, 4).", "Solution_39": "For (0) :\r\n\r\nA solution can be found at the kalva site :\r\n\r\nhttp://www.kalva.demon.co.uk/apmo/asoln/asol994.html\r\n\r\nIt is APMO 1999/4.", "Solution_40": "(7) is straightforward (but ugly) using Pell's equation.", "Solution_41": "So this topic is finally closed. What did you think of it ?", "Solution_42": "DANGIT YOU USE MOD 3 I WAS TOO STUCK ON MOD 2 TO USE MOD 3 RGGGGG!\r\n\r\n\r\nwurg i thought of using modulo 3, but I thought that might give too much away. sigh . . . \r\n\r\n\r\nthe argument for no solutions w/ a>b was much much much nicer than what I came up with though. \r\n\r\n\r\n\r\ntopic = fun; #2 = :evil:", "Solution_43": "I liked them. The topic is maybe not my favorite. It was a good set of questions, though." } { "Tag": [ "geometry", "rectangle", "probability", "search", "combinatorics unsolved", "combinatorics" ], "Problem": "1. In how many ways can four squares, not all in the same row or column be selected from an 8-by-8 chessboard to form a rectangle?\r\n\r\n2. A gardener plants three maple trees, four oak trees and five birch trees in a row. He plants them in random order, each arrangement being equally likely. Find the probability that no two birch trees are next to one another.\r\n\r\n3. 25 people sit around a circular table. Three of them are chosen randomly. What is the probability that at least two of the three are sitting next to one another?", "Solution_1": "Searching for \"birch trees\" gets\r\nhttp://www.artofproblemsolving.com/Forum/viewtopic.php?search_id=1518516264&t=277174\r\nhttp://www.artofproblemsolving.com/Forum/viewtopic.php?search_id=1518516264&t=66513\r\nhttp://www.artofproblemsolving.com/Forum/viewtopic.php?search_id=1518516264&t=135810\r\nhttp://www.artofproblemsolving.com/Forum/viewtopic.php?search_id=1518516264&t=126224\r\nhttp://www.artofproblemsolving.com/Forum/viewtopic.php?search_id=1518516264&t=87431\r\nhttp://www.artofproblemsolving.com/Forum/viewtopic.php?search_id=1518516264&t=48556\r\n\r\nExercise: repeat this experiment to find your other questions. (Hint: you're posting questions from a particular source. Many people include the sources when they post questions. Incidentally, this is something you could do, too, especially when the questions come from some established source like a collection of mathematical problems or a national olympiad.)" } { "Tag": [ "modular arithmetic" ], "Problem": "prove that for all $n\\geq 2$ \r\n$5^{5n+2}+ 3^{5n-1} - 7 \\equiv 0\\pmod{11}$", "Solution_1": "$(5^5)^n \\cdot 5^2 + (3^5)^n : 3 - 7 \\equiv 1 \\cdot 3 + 1 : 3 - 7 \\equiv 0$\r\n$3 + \\frac{1}{3} - 7 \\equiv 0$\r\n$-\\frac{11}{3} \\equiv 0$\r\n$-11 \\equiv 0$ which is true.\r\n\r\nBut I think for all $n$.", "Solution_2": "Firstly notice that:\r\n$5^5\\equiv{1}\\pmod{11}$\r\n$5^{5n}\\equiv{1}\\pmod{11}$\r\n$5^{5n+2}\\equiv{3}\\pmod{11}$\r\n$5^{5n+2}-7\\equiv{-4}\\pmod{11}$\r\nAnd now:\r\nWe have to prove, that:$5^{5n+2}+ 3^{5n-1} - 7 \\equiv 0\\pmod{11}$\r\nI will use induction.\r\nSo first I check if $1$ satisfy this, it's true. \r\nNow I will prove, that if $n$ satisfies this, so $n+1$ will also satisfy. \r\nFor $n+1$ our theorem looks like this:\r\n$5^{5(n+1)+2}+3^{5(n+1)-1}- 7 \\equiv 0\\pmod{11}$\r\nFrom our assumption we can conclude, that :$3^{5n-1}\\equiv{4}\\pmod{11}$.\r\n${5^{5(n+1)+2}-7}\\equiv{-4}\\pmod{11}$ for all $n$ (we proved it at the beginning)\r\nSo we have to prove, that if $3^{5n-1}\\equiv{4}\\pmod{11}$ then $3^{5(n+1)-1}\\equiv{4}\\pmod{11}$\r\n$3^{5n-1}\\equiv{4}\\pmod{11}$\r\n$3^{5n-1}\\cdot{3^5}\\equiv{4\\cdot{3^5}\\pmod{11}}$\r\n$3^{5(n+1)-1}\\equiv{4\\cdot{3^5}\\pmod{11}}$\r\n$3^{5(n+1)-1}\\equiv{4}\\pmod{11}$", "Solution_3": "[quote=\"Andreas\"]$(5^5)^n \\cdot 5^2 + (3^5)^n : 3 - 7 \\equiv 1 \\cdot 3 + 1 : 3 - 7 \\equiv 0$\n$3 + \\frac{1}{3} - 7 \\equiv 0$\n$-\\frac{11}{3} \\equiv 0$\n$-11 \\equiv 0$ which is true.\n[/quote]\r\nAndreas, you cannot use fractions in congruences.", "Solution_4": "[hide]since n is an integer, it is odd or even, so we have $n=2k+1$ and $n=2m+2$\nnote that $\\phi(11)=10$, so we can eliminate the k and m for each case, and just add up the results, and they are in fact, 0 mod 11[/hide]", "Solution_5": "[quote=\"dondigo\"][quote=\"Andreas\"]$(5^5)^n \\cdot 5^2 + (3^5)^n : 3 - 7 \\equiv 1 \\cdot 3 + 1 : 3 - 7 \\equiv 0$\n$3 + \\frac{1}{3} - 7 \\equiv 0$\n$-\\frac{11}{3} \\equiv 0$\n$-11 \\equiv 0$ which is true.\n[/quote]\nAndreas, you cannot use fractions in congruences.[/quote]\r\n\r\ndondigo, note that $11$ is prime. :)" } { "Tag": [ "modular arithmetic", "superior algebra", "superior algebra unsolved" ], "Problem": "Consider a prime p and let $ f(x) \\in \\mathbb{Z}[x] - {0}$ have degree $ \\leq p-2$. Then find $ \\sum_{i=0}^{p-1}f(i) \\mod p$", "Solution_1": "let $ g$ be a primitive root modulo $ p$. then $ 1^k\\plus{}2^k\\plus{}\\cdots\\plus{}(p\\minus{}1)^k\\equal{}g^k\\plus{}g^{2k}\\plus{}\\cdots\\plus{}g^{(p\\minus{}1)k}\\equal{}g^k\\cdot \\frac{g^{k(p\\minus{}1)}\\minus{}1}{g^k\\minus{}1}$. the numerator is divisible by $ p$ because $ g^{k(p\\minus{}1)}\\equiv 1\\pmod{p}$, and if $ k\\leq p\\minus{}2$, the denominator is not divisible by $ p$ by the fact that $ g$ is a primitive root. so $ \\sum_{i\\equal{}0}^{p\\minus{}1}f(i)\\equiv 0 \\pmod{p}$." } { "Tag": [ "algebra unsolved", "algebra" ], "Problem": "Let $ a, b, c, d$ be positive real numbers such that\r\n\r\n$ a\\plus{}b\\plus{}c\\plus{}d \\equal{} 12$\r\n\r\nand \r\n\r\n$ abcd\\equal{}27\\plus{}ab\\plus{}ac\\plus{}ad\\plus{}bc\\plus{}bd\\plus{}cd$.\r\n\r\nFind all possible values of $ a, b, c, d$ satisfying these equations.", "Solution_1": "By AM-GM, $ ab\\plus{}ac\\plus{}ad\\plus{}bc\\plus{}bd\\plus{}cd\\geq 6\\sqrt{abcd}$ so $ abcd\\equal{}27\\plus{}ab\\plus{}bc\\plus{}cd\\plus{}da\\plus{}ac\\plus{}bd\\geq 27\\plus{}6\\sqrt{abcd}$. This factorizes into $ (\\sqrt{abcd}\\minus{}9)(\\sqrt{abcd}\\plus{}3)\\geq 0$ so $ \\sqrt{abcd}\\geq 9$, or $ 4\\sqrt[4]{abcd}\\geq 12\\equal{}a\\plus{}b\\plus{}c\\plus{}d\\geq4\\sqrt[4]{abcd}$. Equality if $ a\\equal{}b\\equal{}c\\equal{}d\\equal{}3$." } { "Tag": [], "Problem": "\u0388\u03c3\u03c4\u03c9 \u03c4\u03bf \u03c4\u03b5\u03c4\u03c1\u03ac\u03b3\u03c9\u03bd\u03bf $ ABCD$ \u03c0\u03bb\u03b5\u03c5\u03c1\u03ac\u03c2 $ a$ \u03ba\u03b1\u03b9 $ M$ \u03c4\u03bf \u03bc\u03ad\u03c3\u03bf \u03c4\u03b7\u03c2 $ AB$. \u0391\u03bd \u03b7 $ DB$ \u03c4\u03ad\u03bc\u03bd\u03b5\u03b9 \u03c4\u03b7\u03bd $ MC$ \u03c3\u03c4\u03bf $ E$, \u03bd\u03b1 \u03c5\u03c0\u03bf\u03bb\u03bf\u03b3\u03af\u03c3\u03b5\u03c4\u03b5 \u03c4\u03b7\u03bd \u03b1\u03ba\u03c4\u03af\u03bd\u03b1 $ R$ \u03c4\u03bf\u03c5 \u03b5\u03b3\u03b3\u03b5\u03b3\u03c1\u03b1\u03bc\u03bc\u03ad\u03bd\u03bf\u03c5 \u03ba\u03cd\u03ba\u03bb\u03bf\u03c5 \u03c3\u03c4\u03bf \u03c4\u03c1\u03af\u03b3\u03c9\u03bd\u03bf $ DEC$ \u03c3\u03c5\u03bd\u03b1\u03c1\u03c4\u03ae\u03c3\u03b5\u03b9 \u03c4\u03bf\u03c5 $ a$. :)", "Solution_1": "[hide]An einai sostes oi prakseis:\n\n$ r\\equal{}\\frac{1}{\\sqrt{5}\\plus{}\\sqrt{2}\\plus{}\\frac{3}{2}}\\cdot \\alpha$[/hide]", "Solution_2": "\u0392\u03b3\u03b1\u03af\u03bd\u03bf\u03c5\u03bd $ DE \\equal{} \\frac {2a\\sqrt {2}}{3}$ ,...$ EC \\equal{} \\frac {a\\sqrt {5}}{3}$, ...$ (DEC) \\equal{} \\frac {a^2}{3}$ \r\n\r\n\u03ba\u03b1\u03b9 \u03bc\u03b5 \u03b4\u03b5\u03b4\u03bf\u03bc\u03ad\u03bc\u03bf \u03cc\u03c4\u03b9 DC = \u03b1, \u03ba\u03b1\u03b9 \u03c4\u03bf\u03bd \u03c4\u03cd\u03c0\u03bf \r\n\r\n\u0395 = \u03c4\u03c1 \u03c0\u03c1\u03bf\u03ba\u03cd\u03c0\u03c4\u03b5\u03b9 \u03c4\u03bf \u03b6\u03b7\u03c4\u03bf\u03cd\u03bc\u03b5\u03bd\u03bf ...\r\n\r\nYs. \u039d\u03af\u03ba\u03bf \u03bd\u03bf\u03bc\u03af\u03b6\u03c9 \u03cc\u03c4\u03b9 \u03ad\u03c7\u03b5\u03b9\u03c2 \u03c6\u03ac\u03b5\u03b9 \u03ad\u03bd\u03b1 2 \u03c3\u03c4\u03bf\u03bd \u03c0\u03b1\u03c1\u03bf\u03bd\u03bf\u03bc\u03b1\u03c3\u03c4\u03ae \u03c4\u03bf\u03c5 \u03c1\u03af\u03b6\u03b1 5.\r\n\r\n\u0391\u03bb\u03bb\u03ac \u03bc\u03c0\u03bf\u03c1\u03b5\u03af \u03bd\u03b1 \u03ad\u03c7\u03c9 \u03ba\u03ac\u03bd\u03b5\u03b9 \u03ba\u03b1\u03b9 \u03b5\u03b3\u03ce \u03bb\u03ac\u03b8\u03bf\u03c2 \u03c3\u03c4\u03b9\u03c2 \u03c0\u03c1\u03ac\u03be\u03b5\u03b9\u03c2 ( \u03c9\u03c2 \u03c3\u03c5\u03bd\u03ae\u03b8\u03c9\u03c2 )\r\n\u03b3\u03b9\u03b1\u03c4\u03af \u03c4\u03b9\u03c2 \u03ad\u03ba\u03b1\u03bd\u03b1 \u03b2\u03b9\u03b1\u03c3\u03c4\u03b9\u03ba\u03ac...", "Solution_3": "[quote=\"vangelismak\"]\u0392\u03b3\u03b1\u03af\u03bd\u03bf\u03c5\u03bd $ DE \\equal{} \\frac {2a\\sqrt {2}}{3}$ ,...$ EC \\equal{} \\frac {a\\sqrt {5}}{3}$, ...$ (DEC) \\equal{} \\frac {a^2}{3}$ \n\n\u03ba\u03b1\u03b9 \u03bc\u03b5 \u03b4\u03b5\u03b4\u03bf\u03bc\u03ad\u03bc\u03bf \u03cc\u03c4\u03b9 DC = 1 \u03ba\u03b1\u03b9 \u03c4\u03bf\u03bd \u03c4\u03cd\u03c0\u03bf \n\n\u0395 = \u03c4\u03c1 \u03c0\u03c1\u03bf\u03ba\u03cd\u03c0\u03c4\u03b5\u03b9 \u03c4\u03bf \u03b6\u03b7\u03c4\u03bf\u03cd\u03bc\u03b5\u03bd\u03bf ...\n\nYs. \u039d\u03af\u03ba\u03bf \u03bd\u03bf\u03bc\u03af\u03b6\u03c9 \u03cc\u03c4\u03b9 \u03ad\u03c7\u03b5\u03b9\u03c2 \u03c6\u03ac\u03b5\u03b9 \u03ad\u03bd\u03b1 2 \u03c3\u03c4\u03bf\u03bd \u03c0\u03b1\u03c1\u03bf\u03bd\u03bf\u03bc\u03b1\u03c3\u03c4\u03ae \u03c4\u03bf\u03c5 \u03c1\u03af\u03b6\u03b1 5.\n\n\u0391\u03bb\u03bb\u03ac \u03bc\u03c0\u03bf\u03c1\u03b5\u03af \u03bd\u03b1 \u03ad\u03c7\u03c9 \u03ba\u03ac\u03bd\u03b5\u03b9 \u03ba\u03b1\u03b9 \u03b5\u03b3\u03ce \u03bb\u03ac\u03b8\u03bf\u03c2 \u03c3\u03c4\u03b9\u03c2 \u03c0\u03c1\u03ac\u03be\u03b5\u03b9\u03c2 ( \u03c9\u03c2 \u03c3\u03c5\u03bd\u03ae\u03b8\u03c9\u03c2 )\n\u03b3\u03b9\u03b1\u03c4\u03af \u03c4\u03b9\u03c2 \u03ad\u03ba\u03b1\u03bd\u03b1 \u03b2\u03b9\u03b1\u03c3\u03c4\u03b9\u03ba\u03ac...[/quote]\r\n\r\nme auto ton tropo to ekana ki ego...tora gia tis prakseis-h episthmh shkonei ta xeria psila.", "Solution_4": "\u03a0\u03c1\u03b1\u03b3\u03bc\u03b1\u03c4\u03b9\u03ba\u03ac \u03bd\u03bf\u03bc\u03af\u03b6\u03c9 \u03cc\u03c4\u03b9 \u03b5\u03af\u03bd\u03b1\u03b9 \u03c4\u03bf \u03c0\u03b9\u03bf \u03b4\u03c5\u03c3\u03ba\u03bf\u03bb\u03bf \u03bc\u03ad\u03c1\u03bf\u03c2 \u03c4\u03b7\u03c2 \u03ac\u03c3\u03ba\u03b7\u03c3\u03b7\u03c2 :rotfl:", "Solution_5": "M\u03b9\u03b1 \u03bb\u03cd\u03c3\u03b7 \u03c0\u03bf\u03c5 \u03b4\u03b5\u03bd \u03b5\u03af\u03bd\u03b1\u03b9 \u03b2\u03b1\u03c3\u03b1\u03bd\u03b9\u03c3\u03c4\u03b9\u03ba\u03ae ... ( \u03bb\u03af\u03b3\u03b5\u03c2 \u03c0\u03c1\u03ac\u03be\u03b5\u03b9\u03c2 \u03bc\u03cc\u03bd\u03bf ... ) :| \r\n\r\n\r\n[hide]\u0388\u03c3\u03c4\u03c9 \u03c4\u03b1 \u03c3\u03b7\u03bc\u03b5\u03af\u03b1 \u0391(0,\u03b1), \u0392(\u03b1,\u03b1), C(\u03b1,0) \u03ba\u03b1\u03b9 D(0,0) \u03c0\u03bf\u03c5 \u03b5\u03af\u03bd\u03b1\u03b9 \u03ba\u03bf\u03c1\u03c5\u03c6\u03ad\u03c2 \u03c4\u03b5\u03c4\u03c1\u03b1\u03b3\u03ce\u03bd\u03bf\u03c5. \u03a4\u03cc\u03c4\u03b5 \u03b8\u03b1 \u03b5\u03af\u03bd\u03b1\u03b9 \u039c( $ \\frac {a}{2}$,\u03b1 ).\nE\u03cd\u03ba\u03bf\u03bb\u03b1 \u03b2\u03c1\u03af\u03c3\u03ba\u03bf\u03c5\u03bc\u03b5 \u03cc\u03c4\u03b9 \u03bf\u03b9 \u03b5\u03be\u03b9\u03c3\u03ce\u03c3\u03b5\u03b9\u03c2 \u03c4\u03c9\u03bd \u03b5\u03c5\u03b8\u03b5\u03b9\u03ce\u03bd BD \u03ba\u03b1\u03b9 CM \u03b5\u03af\u03bd\u03b1\u03b9 \nBD: y=x (1) \u03ba\u03b1\u03b9 CM: y= -2x+2\u03b1 (2).\u039b\u03cd\u03bd\u03bf\u03bd\u03c4\u03b1\u03c2 \u03c4\u03bf \u03c3\u03cd\u03c3\u03c4\u03b7\u03bc\u03b1 \u03c4\u03c9\u03bd (1) \u03ba\u03b1\u03b9 (2) \u03c0\u03c1\u03bf\u03ba\u03cd\u03c0\u03c4\u03b5\u03b9 \u03cc\u03c4\u03b9 \u03c4\u03bf \u03c3\u03b7\u03bc\u03b5\u03af\u03bf \u0395 \u03ad\u03c7\u03b5\u03b9 \u03c3\u03c5\u03bd\u03c4\u03b5\u03c4\u03b1\u03b3\u03bc\u03ad\u03bd\u03b5\u03c2 E$ \\left(\\frac {2a}{3},\\frac {2a}{3}\\right)$. \u0391\u03c0\u03cc \u03c4\u03bf\u03bd \u03c4\u03cd\u03c0\u03bf \u0395 = $ \\frac {1}{2}|det( \\vec{DE},\\vec{DC})|$ \u03c5\u03c0\u03bf\u03bb\u03bf\u03b3\u03af\u03b6\u03bf\u03c5\u03bc\u03b5 \u03c4\u03bf \u03b5\u03bc\u03b2\u03b1\u03b4\u03cc\u03bd \u03c4\u03bf\u03c5 \u03c4\u03c1\u03b9\u03b3\u03ce\u03bd\u03bf\u03c5 DEC, \nE = $ \\frac {a^2}{3}$\u03ba\u03b1\u03b9 \u03b1\u03c0\u03cc \u03c4\u03bf\u03bd \u03c4\u03cd\u03c0\u03bf \u03b1\u03c0\u03cc\u03c3\u03c4\u03b1\u03c3\u03b7\u03c2 \u03b4\u03c5\u03bf \u03c3\u03b7\u03bc\u03b5\u03af\u03c9\u03bd \u03b2\u03c1\u03af\u03c3\u03ba\u03bf\u03c5\u03bc\u03b5 \u03cc\u03c4\u03b9 DE=$ \\frac {2a}{3}\\sqrt {3}$.. \u03ba\u03b1\u03b9 CE = $ \\frac {a}{3}\\sqrt {5}$,..\u03ba\u03b1\u03b9 \u03b4\u03b5\u03b4\u03bf\u03bc\u03ad\u03bd\u03b7\u03c2 \u03c4\u03b7\u03c2 \u03b1\u03c0\u03cc\u03c3\u03c4\u03b1\u03c3\u03b7\u03c2 DC = a, \u03bc\u03c0\u03bf\u03c1\u03bf\u03cd\u03bc\u03b5 \u03bd\u03b1 \u03b2\u03c1\u03bf\u03cd\u03bc\u03b5 \u03c4\u03b7\u03bd \u03b7\u03bc\u03b9\u03c0\u03b5\u03c1\u03af\u03bc\u03b5\u03c4\u03c1\u03bf \u03c4\u03bf\u03c5 \u03c4\u03c1\u03b9\u03b3\u03ce\u03bd\u03bf\u03c5 DE,...\u03c4 = $ \\frac {1}{2}$$ \\left(\\frac {2a}{3}\\sqrt {2} \\plus{} \\frac {a}{3}\\sqrt {5} \\plus{} a\\right)$\n\n\u03a3\u03c4\u03b7 \u03c3\u03c5\u03bd\u03ad\u03c7\u03b5\u03b9\u03b1 \u03b1\u03c0\u03cc \u03c4\u03bf\u03bd \u03c0\u03bf\u03bb\u03cd \u03b3\u03bd\u03c9\u03c3\u03c4\u03cc \u03c4\u03cd\u03c0\u03bf \u0395 = \u03c4\u03c1 \u03c0\u03c1\u03bf\u03ba\u03cd\u03c0\u03c4\u03b5\u03b9 \u03c4\u03bf \u03b6\u03b7\u03c4\u03bf\u03cd\u03bc\u03b5\u03bd\u03bf ...[/hide]" } { "Tag": [ "induction", "function" ], "Problem": "I have been thinking about this a lot lately.\r\n\r\nJust recently I have seen proofs (on AoPS website and in my class), that are formal and prove why the theorem works, or prove that something is not true. For example, if someone makes a statement \"x^3-5>0 for all x\" and then you can say \"what about x=cuberoot(5)?\" and that is a contradiction, so the statement is proved false. However, they are of course way more complicated than this simple example.\r\n\r\nHowever, in math classes my teacher has always given some sort of justification as to why we use a certain theorem or formula. When you see it you think \"Ohhh, so that's why it works!\" This justification is not usually very formal.\r\n\r\nNow the first type of proof I was talking about does not really elicit the same reaction (for me at least). Do I just have to get used to proofs, or will this always be the case?\r\n\r\nHere's the best example that I could think of for this:\r\n\r\n[tex] g(x)= \\sqrt{f(x)} [/tex]\r\n\r\nNow, I am going to make the statement \"the maximum of g(x) is the same as the maximum of f(x).\"\r\n\r\nProof:\r\n\r\n[tex]g(x)\\ge g(x)_{min}[/tex]\r\n[tex]\\sqrt{f(x)} \\ge \\sqrt{f(x)_{min}}[/tex]\r\n[tex]f(x) \\ge f(x)_{min}[/tex]\r\n\r\nJustification:\r\n\r\nIf your highest value for f(x) is one number, then if you take the square root of all of the values of f(x), the square root of the maximum f(x) will still be the greatest, b/c sqrt(bigger #)>sqrt(smaller number), b/c of how square roots work.\r\n\r\nI am sure you guys can find lots of fundamental flaws in this, but I am just trying to provide an example. This is a bad one, I know. A good example might be the \"justification\" (dividing out b/c of overcounting, how it works w/multiplication counting principle) and \"proof\" (more formal, I don't know it, induction maybe) or the combinations and permutations formulas.", "Solution_1": "I don't quite see how what you are saying is true. Here's a counterexample: Say f(x)=4. Then g(x)=2. The maximum of f(x) is 4, but the maximum of g(x) is 2.\r\n\r\nThe main reason rigorous proofs are used is so that you are certain the proof is correct. The many false proofs out there show that it is important to be as rigorous as possible.\r\n\r\nYou are saying that a rigorous proof does not really explain to you why something works, unlike a less formal justification. The best thing to do is to think about what a statement in a rigorous proof really means.\r\n\r\nI guess a good example would be if you see:\r\n\r\n[tex]x=\\frac pq,[/tex] where [tex]p,q\\in\\mathbb Z,q>0,GCF(p,q)=1[/tex].\r\n\r\nYou know it's just saying that x is written as a fraction in lowest terms.", "Solution_2": "Oh I mean that f(x) was a polynomial.", "Solution_3": "I was under the impression she meant that the x at which the maximum for one was achieved was the same.\r\n\r\n\r\nJulie, I'm not really sure what you are asking/talking about. Your examples confuse me more, not less. What do you want to know/have people talk about?", "Solution_4": "I just want to know the difference between formal proofs and informal justifications. I don't know why informal justifications make more sense to me than proofs do. Will formal proofs ever help me understand why something is so it \"clicks\" when I am using it (a formula, theorem, etc)?\r\n\r\nYes and I realize my examples make no sense.", "Solution_5": "It depends on the skill and goals of the proof-writer. If the proof-writer's main objective is just to establish that something is true - then they may choose a proof that is quick and simple, but that gives little \"insight\" into why it is true.\r\n\r\nHowever if the proof-writer's main purpose is to [i]explain[/i] why something is true - then they may choose to provide either a more constructive or intuitive proof of the statement.\r\n\r\nYou often find this as the main difference between the style of proofs that you find in advanced textbooks or in journals, and the proofs that you find in elementary textbooks. In elementary textbooks, when authors are faced with the choice between styles of proof, they will often choose one that is more instinctive, or constructive, or visual.\r\n\r\nIn some cases though, early on in math, you are taught theorems that are important, but that you just don't have the mathematical machinery yet to prove. So the only option is to give at least a \"motivation\" for why the theorem is true.\r\n\r\nThe ability to come up with intuitive, though maybe not entirely formal, justifications for things is a great skill to have. Along with this goes an intuition as to when things are true or not true (especially the ability to come up with counterexamples). Mathematicians use this type of thinking all the time to come up with conjectures, or to lead them in the right direction. But also they realize that a statement is never really completely safe until a formal proof of it has been provided. Also formal proofs often reaveal the conditions under which a statement is true. That's something that is often hard to do with hand-wavy explainations.\r\n\r\nDeep within every proof though, is a good intuitive \"reason\" why the statement is true. It just depends on how deeply or abstractly you can look the proof to see it. In some cases, proof-writers intentionally obscure and hide their work and thought processes for the sake of a cleaner and more elegant final product. But even then, the ultimate reason behind the truth or the statement is in there.", "Solution_6": "[quote=\"JS1527\"]\n[tex] g(x)= \\sqrt{f(x)} [/tex]\n\nNow, I am going to make the statement \"the maximum of g(x) is the same as the maximum of f(x).\"\n\n[/quote]\r\n\r\nYou don't need to rely on shaky logic here.\r\n\r\nThe idea I'm going to use here is that of a bijection between two continuous functions - that is to say, that over the set R+ (the positive reals), the function p(x) = sqrt(x) does two things.\r\n\r\n1: It maps R+ onto R+ --> every element in R+ is mapped onto a new element in R+ by p(x).\r\n\r\n2: For every element a E R+, the equation p(x) = a has exactly one solution.\r\n\r\nSo this is nice. The second thing that we need to note is that sqrt(x) is monotonically increasing - it is increasing EVERYWHERE.\r\n\r\nSo what the function p(x) = sqrt(x) does is preserve order.\r\n\r\nGiven a > b, we have that sqrt(a) > sqrt(b). But we also know that we will not randomly have some number c < a such that sqrt(c) = sqrt(a), because it is a bijection.\r\n\r\nSo, let t = the value where f(x) assumes a maximum.\r\n\r\nThen we know that f(t) >= f(x), x E R+ by hypothesis, correct?\r\n\r\nSo we now know that sqrt(f(t)) >= sqrt(f(x)) Hence f(t) and sqrt(f(t)) are both maximized for the same value." } { "Tag": [ "vector", "FTW" ], "Problem": "", "Solution_1": "", "Solution_2": "", "Solution_3": "", "Solution_4": "", "Solution_5": "", "Solution_6": "Somewhat sinister.", "Solution_7": "Not bad, but somewhat phailish.", "Solution_8": "", "Solution_9": "", "Solution_10": "Nice attempt at vector art, much better than my first. A bit too contrasted, try lowering it.\r\n\r\nedit: Also it's very aliased.", "Solution_11": "I find this adorable :)\r\nSnorlax PWNS!", "Solution_12": "You look funny in that pic.", "Solution_13": "Yay! Aliens FTW", "Solution_14": "I never look funny. :wink:", "Solution_15": "", "Solution_16": "Yeah, she really liked this on Facebook.", "Solution_17": "", "Solution_18": "I like this one better than the other ones.\r\nThough the shades is a close second.", "Solution_19": "this is creepy.", "Solution_20": "You have friends?", "Solution_21": "May the force be with you.", "Solution_22": "", "Solution_23": "", "Solution_24": "Woah, this is pretty impressive! Did you just like mess with teh filter on gimp and merged the light saber from Yoda?", "Solution_25": "Actually, you use a Gaussian blur and colorizing. I was holding a dowel for the picture." } { "Tag": [ "parameterization", "number theory unsolved", "number theory" ], "Problem": "Prove that the equation $ x^2 \\equal{} y^3 \\plus{} z^5$ has infinitely many solutions in positive integer.\r\n\r\n[hide=\"Hint\"]A magical substitution(I mean constructing solutions by parameters) solves this problem, but I don't really understand what the motivation behind that substitution is... and also how to cast this type of magic...:( [/hide]", "Solution_1": "$ (3u^{15},2u^{10},u^6)$", "Solution_2": "[quote=\"Moonmathpi496\"]Prove that the equation $ x^2 \\equal{} y^3 \\plus{} z^5$ has infinitely many solutions in positive integer.\n\n[hide=\"Hint\"]A magical substitution(I mean constructing solutions by parameters) solves this problem, but I don't really understand what the motivation behind that substitution is... and also how to cast this type of magic...:( [/hide][/quote]\r\nOnce you have found the solution $ x\\equal{}3, y\\equal{}2, z\\equal{}1$, I think it is pretty easy to find the rest of the solutions, since there is no restriction, like they should be coprime :)", "Solution_3": "[quote=\"xxp2000\"]$ (3u^{15},2u^{10},u^6)$[/quote]\r\nThanks! Another family of solutions are $ (x_n,y_n,z_n)\\equal{}(n^{10}(n\\plus{}1)^8,n^7(n\\plus{}1)^5,n^4(n\\plus{}1)^3)$" } { "Tag": [ "inequalities" ], "Problem": "If a, b, c are positive numbers such that\r\n$(1+a)(1+b)(1+c) = 8$\r\nprove that abc <= 1.", "Solution_1": "[quote=\"PenguinIntegral\"]If a, b, c are positive numbers such that\n$(1+a)(1+b)(1+c) = 8$\nprove that abc <= 1.[/quote]\r\nI think the only solution to this problem is $a=b=c=1$.\r\nSo $abc=1$..\r\nbut i don't understand why there is an inequality $abc\\leq 1$..", "Solution_2": "[quote=\"kimby_102\"][quote=\"PenguinIntegral\"]If a, b, c are positive numbers such that\n$(1+a)(1+b)(1+c) = 8$\nprove that abc <= 1.[/quote]\nI think the only solution to this problem is $a=b=c=1$.\nSo $abc=1$..\nbut i don't understand why there is an inequality $abc\\leq 1$..[/quote]\r\nPositive NUMBERS, not only positive integers.", "Solution_3": "[quote=\"lotrgreengrapes7926\"][quote=\"kimby_102\"][quote=\"PenguinIntegral\"]If a, b, c are positive numbers such that\n$(1+a)(1+b)(1+c) = 8$\nprove that abc <= 1.[/quote]\nI think the only solution to this problem is $a=b=c=1$.\nSo $abc=1$..\nbut i don't understand why there is an inequality $abc\\leq 1$..[/quote]\nPositive NUMBERS, not only positive integers.[/quote]\r\nsorry, my mistake :blush: \r\n\r\nthen we apply AM-GM inequality.\r\n$1+a\\geq 2\\sqrt{a}$\r\n$1+b\\geq 2\\sqrt{b}$\r\n$1+c\\geq 2\\sqrt{c}$.\r\nNow multiply thses three inequal. we get:\r\n$(1+a)(1+b)(1+c)\\geq 8\\sqrt{abc}$.\r\nBut since $(1+a)(1+b)(1+c)=8$, $1\\geq abc$. :D", "Solution_4": "yes its a good solution." } { "Tag": [ "number theory", "prime numbers", "number theory unsolved" ], "Problem": "Suppose that $ p$ is a prime number .Find all $p$ s.t. the remainder of $p$ to all other prime numbers be the square-free . :huh: :o", "Solution_1": "well I have showed that for big primes, Not too big I guess $p>10$ will do, we must have one of the following situations:\r\n\r\n$p=3^k+4=2^t+9$, or $p=3^k+4=2^t.7^s+9=5^l.7.3^2+16$ or $p=3^k+4=7.2^t+9$ and some of them can be shown to have no nontrivial solutions." } { "Tag": [ "\\/closed" ], "Problem": "hmm links don't open in a new window/tab anymore...\r\nI'm on the ML skin/firefox", "Solution_1": "As required per web 2.0 standards links shouldn't open in new tabs / windows (basically the user should choose to do so by right-click open in a new tab/window). In plus the target attribute will not validate in xhtml." } { "Tag": [ "LaTeX", "linear algebra", "linear algebra unsolved" ], "Problem": "Are these ordered triples two distinct bases in Q^3?\r\n\r\n(4/5,3/4,2/3) and ( 11/13,7/11,5/7) ?\r\n\r\nIf they are, how would i go about proving that they span Q^3?\r\n\r\nIf they aren't-- what's a good example of an ordered triple that is a basis?", "Solution_1": "No. Each of those triples is [i]one[/i] element of $ \\mathbb{Q}^{3}$, and you need three elements for a basis.", "Solution_2": "How about these?\r\n\r\n([1,0,0],[0,1,0],[0,0,1]) and ([4/5,0,0],[0,3/4,0],[0,0,2/3]).\r\n\r\nare those two distinct bases in Q^3?\r\n\r\nAlso, how does someone go about writing tex in these forums?", "Solution_3": "Yes, those are bases.\r\n\r\nTo write $ \\text{\\LaTeX}$ math mode here, just put your code between dollar signs." } { "Tag": [ "real analysis", "real analysis unsolved" ], "Problem": "Prove that $\\sum_{n=0}^{\\infty}\\frac{(-1)^nn^3}{n!}=\\frac{1}{e}$", "Solution_1": "$n^3=n(n-1)(n-2)+3n(n-1)+n$, so $\\sum_{n=0}^{\\infty}\\frac{(-1)^n n^3}{n!}=-\\frac{1}{e}+\\frac{3}{e}-\\frac{1}{e}=\\frac{1}{e}$", "Solution_2": "[quote=\"Buffalo\"]$n^3=n(n-1)(n-2)+3n(n-1)+n$, so $\\sum_{n=0}^{\\infty}\\frac{(-1)^n n^3}{n!}=-\\frac{1}{e}+\\frac{3}{e}-\\frac{1}{e}=\\frac{1}{e}$[/quote]\r\n\r\nIt's cool! :) :)" } { "Tag": [ "function", "calculus", "derivative", "inequalities", "inequalities unsolved" ], "Problem": "If x, y, z > 0 and $ x \\plus{} y \\plus{} z \\le 1$ prove that:\r\n\r\n$ x^2y \\plus{} y^2z \\plus{} z^2x \\plus{} xyz \\le 4/27$", "Solution_1": "Basicly I don't quite have the energy to post my solution but it uses derivatives. You set z=const and you substitute x=1-y-z and you make a function of y. You can set y to be between 0 and 1/3 ( basicly the smallest of the 3, without loss of generality cuz it is a cyclic function). Next you should take the first derivative and see that one of the roots ( the maximum of the funcion) is not between those two numbers which means that one of them ( or two as the case may be) must be the value for which f(y) is maximum. Next by substituting y=0 you will see that x=2/3 and z=1/3 (easy again with derivative of the function (x^2)y since all other monomials are equal to 0 and that would lead you to one solution (0,2/3,1/3) ) and the second y=1/3 leads to x=1/3 z=1/3 again with derivative of a function.\r\n It is certainly not an easy way to do this and not the most pretty but certainly all BEAUTIFUL inequalities like AM, GM, Cauchy or anything else would lead you nowhere cuz equality is reached basicly when x=y=z and that is not the only case we are seeing in this problem so I don't see any way around derivatives or Lagrange. Hope you find an easier way. But at least I hope I saved everyone some time in checking for an elegant solutions with the above inequalities. THEY WILL NOT WORK!!! \r\n So there are two answers: (1/3;1/3;1/3) AND (0;1/3;2/3) and their permutations of course.", "Solution_2": "Hello. Here is a simple solution:\r\n\r\nWLOG suppose that $ x$ is the middle variable (i.e. $ y\\ge x\\ge z$ or $ z\\ge x\\ge y$). Then clearly, $ (x\\minus{}y)(x\\minus{}z)\\le 0$. So $ \\minus{}y(x\\minus{}y)(x\\minus{}z)\\ge 0$.\r\n\r\nNow let $ F[x,y,z]\\equal{}x^2 y \\plus{} y^2 z \\plus{} z^2 x \\plus{} x y z$. It is easy to verify that \r\n\r\n\\[ F[x, 0, y \\plus{} z] \\minus{} F[x, y, z]\\equal{}\\minus{}(x \\minus{} y) y (x \\minus{} z)\\ge 0\\]\r\n\r\nLet $ w\\equal{}y\\plus{}z$. Then $ x\\plus{}w\\le 1$, and we need to prove that $ w^2x\\le \\frac{4}{27}$.\r\n\r\nBut this is trivial by AM-GM because \\[ w^2x\\equal{}4\\cdot\\frac{w}{2}\\frac{w}{2}x\\le 4\\cdot \\left(\\frac{ \\frac{w}{2}\\plus{}\\frac{w}{2}\\plus{}x}{3}\\right)^3\\equal{}\\frac{4}{27}\\cdot (x\\plus{}w)^3\\le \\frac{4}{27}\\]", "Solution_3": "Ugh, Altheman beat me to it but i'm pretty sure i have the nicer proof anyway.\r\n\r\nLet $ x\\geq z \\geq y$, and clearly we may assume $ x\\plus{}y\\plus{}z\\equal{}1$. Then, $ 0\\geq (x\\minus{}z)(y\\minus{}z)\\iff z(x\\plus{}y)\\geq xy\\plus{}z^2\\qquad (*)$.\r\n\r\nNow,\r\n\\[ yz(x\\plus{}y)\\plus{}x(xy\\plus{}z^2) \\leq z(x\\plus{}y)^2\\qquad \\text{ by }(*)\\]\r\nAnd, by Am-Gm we have \\[ \\frac13\\equal{}\\frac{\\frac{x\\plus{}y}{2}\\plus{}\\frac{x\\plus{}y}{2}\\plus{}z}{3}\\geq \\sqrt[3]{\\frac{z(x\\plus{}y)^2}{4}}\\iff z(x\\plus{}y)^2\\leq \\frac{4}{27}\\]\r\nQED\r\n :D", "Solution_4": "It seems to me that our proofs are equivalent...you should be careful because you cannot assume $ x\\ge z\\ge y$ since the inequality is cyclic by not symmetric.", "Solution_5": "Yes, that was sort of the joke... and good poitn about the cyclicity, I should have caught that mistake.\r\nAnyway, I was thinking about the whole \"motivation\" behind comparing $ xy\\minus{}z^2$ to $ z(x\\plus{}y)$ and realized that when you substitute t+a,t-a = x,y we have $ |t\\minus{}z|\\leq a$ as the square root of $ xy\\minus{}z^2\\leq z(x\\plus{}y)$, so it's totally a topological statement", "Solution_6": "[quote=\"jumbogaba\"]If x, y, z > 0 and $ x \\plus{} y \\plus{} z \\le 1$ prove that:\n\n$ x^2y \\plus{} y^2z \\plus{} z^2x \\plus{} xyz \\le 4/27$[/quote]\r\n\r\n$ x^2y \\plus{} y^2z \\plus{} z^2x \\plus{} xyz \\le \\frac {4}{27}(x \\plus{} y \\plus{} z)^3 \\le \\frac {4}{27}$ :lol:" } { "Tag": [ "inequalities proposed", "inequalities" ], "Problem": "Given $ a, b, c > 0$ satisfy $ (4a\\plus{}5b)(4b\\plus{}5c)(4c\\plus{}5a)\\equal{}729$. \r\n\r\nProve that: $ abc(a^2\\plus{}bc\\plus{}ca)(b^2\\plus{}ca\\plus{}ab)(c^2\\plus{}ab\\plus{}bc) \\le 27$\r\n :)", "Solution_1": "[quote=\"nguoivn\"]Given $ a, b, c > 0$ satisfy $ (4a \\plus{} 5b)(4b \\plus{} 5c)(4c \\plus{} 5a) \\equal{} 729$. \n\nProve that: $ abc(a^2 \\plus{} bc \\plus{} ca)(b^2 \\plus{} ca \\plus{} ab)(c^2 \\plus{} ab \\plus{} bc) \\le 27$\n :)[/quote]\r\nWe can solve this problem by \"ony Am-Gm\" :)", "Solution_2": "[quote=\"nguoivn\"]Given $ a, b, c > 0$ satisfy $ (4a \\plus{} 5b)(4b \\plus{} 5c)(4c \\plus{} 5a) \\equal{} 729$. \n\nProve that: $ abc(a^2 \\plus{} bc \\plus{} ca)(b^2 \\plus{} ca \\plus{} ab)(c^2 \\plus{} ab \\plus{} bc) \\le 27$\n :)[/quote]\r\nSee it here: http://can-hang2007.blogspot.com/2010/01/inequality-67-t-q-anh.html\r\n(Note: Visitors need to use Firefox in order to see the Math Formulas in my blog.)", "Solution_3": "It also exists a shorter solution by AM-GM :)" } { "Tag": [ "function", "calculus", "integration", "LaTeX", "real analysis", "real analysis unsolved" ], "Problem": "Hi all, just studying for my analysis exam, I am stuck on this problem:\r\n\r\nSuppose mu is a Borel measure on X, f:X -> [0,infinity) is measurable and g:[0,infinity) -> [0,infinity) is an increasing C-one function st. g(0) = 0\r\n\r\nShow the integral over X of g(f(x)) d(mu) = the integral from 0 to infinity of g'(t)*mu({x:f(x)>t}) dt\r\n\r\nThanks for the help, sorry I don't know Latex, I intend to learn it over the summer.", "Solution_1": "You have\r\n\r\n\\[ \\int^\\infty_0 \\mu(\\{f>t\\})g'(t)dt \\equal{} \\int^\\infty_0 \\int_X g'(t) \\chi_{\\{f>t\\}}d\\mu dt\r\n\\]\r\n\r\nNow use Fubini.", "Solution_2": "Ah yes of course, thanks very much, I appreciate the help. I am stuck on another problem now, about Fourier coefficients of measures on the unit circle:\r\nProve that, if mu^(n) -> 0 as n->+infinity, then for any bounded Borel function f on S^1, the integral over S^1 of \r\nexp(-2Pi*i*n*t)f(t)dmu(t) ->0 as n->+infinity\r\n\r\n*Here mu^(n) means the nth Fourier coefficient of mu, namely the integral over S^1 of exp(-2Pi*i*n*t)dmu(t)\r\n\r\n\r\nAny advice would be appreciated." } { "Tag": [ "geometry", "3D geometry", "puzzles" ], "Problem": "How high would the column be if you were to put all the millimeter cubes contained in a cubic meter,on top of each other.", "Solution_1": "a cubic meter contains 1 billion milimeter cubes. So if you put those on top of eacother you get something 1 billion milimeters heigh, that is 1 million meters, or 1.000 kilometers. \r\nagain, far too easy. what was the point?" } { "Tag": [ "number theory unsolved", "number theory" ], "Problem": "How does one go about finding the first few digits of a mammoth number by just using a calculator and logs?. For instance, the newest discovered prime is 2^25,964,951-1. How can the first 3 digits be found by just using the methods above?.\r\n\r\nThanks,\r\nCody", "Solution_1": "This was fun. I came up with this method just now, then checked it to make sure I wasn't crazy.\r\n\r\n$log_2(2^{25,964,951}-1)$ is, for our purposes, about equal to ${log_2(2^{25,964,951}})=25,964,951$. Therefore, $log_{10}(2^{25,964,951}-1)$ is about $25,964,951*log_{10}2=7816229.0869454841986461649283156$. If we subtract an integer from that value, we are essentially just moving the decimal place over 1 point in our behometh number. Therefore, we take $10^.0869454841986461649283156$ to get $1.2216463006127794810775378964564$. Those should be the first couple of digits of the number.\r\n\r\nEdit: Those first few digits match up with what they posted on mathworld. It matched up to 9810, then mathworld stop posting the number. Presumably, the rest are correct as well.", "Solution_2": "Thanks. I was close. I got as far as 25964951*log2=7816229.88695, but I didn't multiply 10 times the decimal part. Clever. Thanks\r\n\r\nCody" } { "Tag": [ "number theory proposed", "number theory" ], "Problem": "$ xy|x^{2}\\plus{}y^{2}$. Proof that $ x\\equal{}y$", "Solution_1": "[quote=\"inny\"]$ xy|x^{2} \\plus{} y^{2}$. Proof that $ x \\equal{} y$[/quote]\r\n$ x^2 \\minus{} kxy \\plus{} y^2 \\equal{} 0$, where $ k\\in\\mathbb{N}$. Divide the equation by $ y^2$ and note $ t \\equal{} \\dfrac{x}y$. So: $ t^2 \\minus{} kt \\plus{} 1 \\equal{} 0$. In order to admit rational solution its discriminant must be rational. So $ k^2 \\minus{} 4 \\equal{} a^2$ or $ 4 \\equal{} (k \\minus{} a)(a \\plus{} k)$ So $ k \\equal{} 2, a \\equal{} 0$ which implies $ x \\equal{} y$", "Solution_2": "THanks a lot" } { "Tag": [ "number theory unsolved", "number theory" ], "Problem": "Let $ S(n)$ denote the sum of all digits of a positive integer $ n$. Do there exist positive integers $ a,b$ such that for any positive integer $ n$, $ S(an)0$ and $a^2+b^2+c^2+d^2>0$, in order to show that this is composite we must show that $a+d=b+c+1$ is not possible. Suppose it is possible. Then squaring yields \\[a^2+2ad+d^2=b^2+2bc+c^2+2b+2c+1\\implies a^2+ad+d^2=bc+2b+2c+1.\\] Using the fact that $ad=b^2+bc+c^2$ again gives \\[a^2+b^2+c^2+d^2+ad+bc=ad+bc+2b+2c+1\\implies a^2+(b-1)^2+(c-1)^2+d^2=1.\\] However, since $a^2+d^2\\geq 2$ we have a contradiction. Therefore $a+d-b-c\\geq 2$ and $a^2+b^2+c^2+d^2$ is composite, as desired. $\\blacksquare$", "Solution_4": "Note that \n\\[(a+d)^2 - (b+c)^2 = a^2+d^2 + 2(ad-bc) - b^2-c^2 = a^2+d^2 + 2(b^2+c^2)-b^2-c^2 = a^2+b^2+c^2+d^2\\]\nThus, $a^2+b^2+c^2+d^2 = (a+d-b-c)\\cdot (a+d+b+c)$. Thus, it suffices to show that $a+d\\geq b+c+2$. AFTSOC that $a+d\\leq b+c+1$, then \n\\[b^2+bc+c^2=ad\\leq \\frac{(a+d)^2}{4} \\leq \\frac{(b+c+1)^2}{4}\\]\nSo, \n\\[4b^2+4bc+4c^2\\leq b^2+c^2+2bc+2b+2c+1\\]\nThen,\n\\[3b^2+2bc+3c^2\\leq 2b+2c+1\\]\nbut since $b,c$ are positive, we have $3b^2>2b, 2bc>1, 3c^2>2c$, so this is absurd. Thus, $(a+d-b-c)\\geq 2$, so $a^2+b^2+c^2+d^2$ can be written as the product of two integers $\\geq 2$ and we're done." } { "Tag": [ "induction", "trigonometry", "algebra unsolved", "algebra" ], "Problem": "How to prove the identity\r\n\r\n${{\\sin^2(n+1)x}\\over {\\sin x}} = \\sin x+\\sin 2x +\\sin 3x+\\ldots +\\sin (2n+1)x$\r\n\r\nby induction?", "Solution_1": "I don't think it's correct :? (so we can't prove it by induction when assuming consistency of our axioms :P ).\r\nTake $n=1$ to get $\\frac{\\sin(2x)^2}{\\sin(x)} = \\sin(x)+\\sin(2x)+\\sin(3x)$, which is wrong for all $x \\neq \\frac{n\\pi}{2}$.", "Solution_2": "The correct formula should be\r\n$\\sin\\alpha+\\sin 2\\alpha+\\sin 3\\alpha+\\cdots +\\sin n\\alpha=\\frac{\\sin\\frac{n\\alpha}{2}\\sin\\frac{(n+1)\\alpha}{2}}{\\sin\\frac{\\alpha}{2}}$.", "Solution_3": "Indeed, and your formula can be easily proven using complex numbers ;) (if we don't know the RHS of course, else, induction!)", "Solution_4": "The correct identity should be:\r\n\r\n${{\\sin^2(n+1)x}\\over {\\sin x}}=\\sinh+\\sin3x +\\ldots+\\sin (2n+1)x$\r\n\r\nFor $n=1$:\r\n\r\nLHS${= {{\\sin^2 2x}\\over {\\sin x}}={{(2\\sin x\\cos x)^2}\\over {\\sin x}} ={{4\\sin^2 x\\cos^2 x}\\over {\\sin x}}=4\\sin x\\cos^2 x}$\r\n\r\n\r\nRHS$=\\sin x+\\sin3x =2\\sin{{3x+x}\\over 2}\\cos{{3x-x}\\over 2} =2\\sin 2x\\cos x=4\\sin x\\cos^2 x=$LHS" } { "Tag": [], "Problem": "1. Find all values of $ x,y \\in\\mathbb{N}$ that satisfy\r\n$ (x\\plus{}y)\\plus{}(x\\minus{}y)\\plus{}xy\\plus{}\\frac2{y}\\equal{}2009$\r\n\r\n2. Find the value of $ x$ if\r\n$ x^2\\minus{}2\\sqrt{x\\minus{}1\\minus{}\\frac1{x}\\plus{}\\frac1{x^2}}\\minus{}x\\plus{}\\frac2{x}\\equal{}1$, $ x\\neq 0$\r\n\r\n3. It is given that $ x\\equal{}\\frac1{2\\minus{}\\sqrt{3}}$. Find the value of\r\n$ x^6\\minus{}2\\sqrt{3}x^5\\minus{}x^4\\plus{}x^3\\minus{}4x^2\\plus{}2x\\minus{}\\sqrt{3}$\r\n\r\nthx", "Solution_1": "[quote=\"IW@IT\"]1. Find all values of $ x,y \\in\\mathbb{N}$ that satisfy\n$ (x \\plus{} y) \\plus{} (x \\minus{} y) \\plus{} xy \\plus{} \\frac2{y} \\equal{} 2009$\n[/quote]\r\n\r\n[hide=\"1.\"]\n$ \\because y\\in\\mathbb{N}$ then $ y\\in \\left\\{1,2\\right\\}$\n\nIf $ y\\equal{}1$ then $ 2x\\plus{}x\\plus{}2\\equal{}2009 \\implies x\\equal{}667$\n\nIf $ y\\equal{}2$ then $ 2x\\plus{}2x\\plus{}1\\equal{}2009 \\implies x\\equal{}502$\n\n$ \\therefore (x,y)\\in \\left\\{(667,1),(502,2)\\right\\}$\n[/hide]", "Solution_2": "[quote=\"IW@IT\"]\n2. Find the value of $ x$ if\n$ x^2 - 2\\sqrt {x - 1 - \\frac1{x} + \\frac1{x^2}} - x + \\frac2{x} = 1$, $ x\\neq 0$\n[/quote]\r\n\r\n[hide=\"2.\"]\n$ x^2 - x + \\frac2{x}-1- 2\\sqrt {\\frac{x^2(x-1)-(x-1)}{x^2}}=0$\n\n$ \\implies \\frac{x^3-x^2-x+2}{x}=\\frac{2(x-1)\\sqrt{x+1}}{x}$\n\n${ \\implies x^3-x^2-x+2=2(x-1)\\sqrt{x+1}}$\n\n${ \\implies (x^3-x^2-x+1)+1=2(x-1)\\sqrt{x+1}}$\n\n${ \\implies (x^2-1)(x-1)+1=2(x-1)\\sqrt{x+1}}$\n\n${ \\implies (x^2-1)(x-1)+1=2(x-1)\\sqrt{x+1}}$\n\n${ \\implies (x-1)^2(x+1)-2(x-1)\\sqrt{x+1}}+1=0$\n\n$ \\implies [(x-1)\\sqrt{x+1}-1]^2=0$\n\n$ \\implies (x-1)\\sqrt{x+1}-1=0$\n\n$ \\implies (x-1)^2(x+1)=1$\n\n$ \\implies x^3-x^2-x=0 \\implies x^2-x-1=0 \\implies x=\\frac{1\\pm\\sqrt{5}}{2}$\n\n[/hide] :P", "Solution_3": "[quote=\"IW@IT\"]\n3. It is given that $ x = \\frac1{2 - \\sqrt {3}}$. Find the value of\n$ x^6 - 2\\sqrt {3}x^5 - x^4 + x^3 - 4x^2 + 2x - \\sqrt {3}$\n[/quote]\r\n\r\n[hide=\"3.\"]\n$ x = \\frac1{2 - \\sqrt {3}}=\\frac1{2 - \\sqrt {3}}.\\frac{2+\\sqrt3}{2+\\sqrt3}=2+\\sqrt 3$\n\n\n$ x^6 - 2\\sqrt {3}x^5 - x^4 \\\\\n=x^5(x-2\\sqrt {3})-x^4 \\\\\n=x^5(2-\\sqrt 3)-x^4=x^4(2+\\sqrt 3)(2-\\sqrt 3)-x^4=x^4-x^4=0$\n\n$ x^3 - 4x^2 + 2x - \\sqrt {3}\\\\\n=x^2(x-4)+2x-\\sqrt {3}\\\\\n=x^2(2+\\sqrt 3-4)+2x-\\sqrt {3}\\\\\n=x^2(\\sqrt 3-2)+2x-\\sqrt {3}\\\\\n=x(\\sqrt 3+2)(\\sqrt 3-2)+2x-\\sqrt {3}\\\\\n=x(3-4)+2x -\\sqrt {3}\\\\\n=x-\\sqrt {3}\\\\\n=2+\\sqrt 3-\\sqrt {3}=\\boxed{2}$ :P \n\n\n\n[/hide]", "Solution_4": "[quote=\"IW@IT\"]\n$ x^6 \\minus{} 2\\sqrt {3}x^5 \\minus{} x^4 \\plus{} x^3 \\minus{} 4x^2 \\plus{} 2x \\minus{} \\sqrt {3}$\n[/quote]\r\n\r\nNinja'd by vinskman! ;)\r\n\r\n[hide=\"3\"]\n$ x \\equal{} 2 \\plus{} \\sqrt {3}$\n\nWe have $ x^5(x \\minus{} 2\\sqrt {3}) \\equal{} x^6 \\minus{} 2\\sqrt {3}x^5 \\equal{} x^4$.\n\nTherefore the expression is equivalent to $ x^4 \\minus{} x^4 \\plus{} x^3 \\minus{} 4x^2 \\plus{} 2x \\minus{} \\sqrt {3} \\equal{} x^3 \\minus{} 4x^2 \\plus{} 2x \\minus{} \\sqrt {3}$\n\nThis is not too hard to calculate, it's $ 26 \\plus{} 15\\sqrt {3} \\minus{} 28 \\minus{} 16\\sqrt {3} \\plus{} 4 \\plus{} 2\\sqrt {3} \\minus{} \\sqrt {3} \\equal{} 2$. \n[/hide]", "Solution_5": "[quote=\"ffao\"][quote=\"IW@IT\"]\n$ x^6 \\minus{} 2\\sqrt {3}x^5 \\minus{} x^4 \\plus{} x^3 \\minus{} 4x^2 \\plus{} 2x \\minus{} \\sqrt {3}$\n[/quote]\n\nNinja'd by vinskman! ;)\n\n[hide=\"3\"]\n$ x \\equal{} 2 \\plus{} \\sqrt {3}$\n\nWe have $ x^5(x \\minus{} 2\\sqrt {3}) \\equal{} x^6 \\minus{} 2\\sqrt {3}x^5 \\equal{} x^4$.\n\nTherefore the expression is equivalent to $ x^4 \\minus{} x^4 \\plus{} x^3 \\minus{} 4x^2 \\plus{} 2x \\minus{} \\sqrt {3} \\equal{} x^3 \\minus{} 4x^2 \\plus{} 2x \\minus{} \\sqrt {3}$\n\nThis is not too hard to calculate, it's $ 26 \\plus{} 15\\sqrt {3} \\minus{} 28 \\minus{} 16\\sqrt {3} \\plus{} 4 \\plus{} 2\\sqrt {3} \\minus{} \\sqrt {3} \\equal{} 2$. \n[/hide][/quote]\r\n\r\nHello , my guy :lol: what means \"Ninja'd by vinskman\"? would you tell me?", "Solution_6": "It means that you posted exactly what I would post, right before I posted.", "Solution_7": "[quote=\"ffao\"]It means that you posted exactly what I would post, right before I posted.[/quote]\r\n\r\n\r\nThanks.\r\n\r\nNo worries.You are welcome. :D :P", "Solution_8": "[quote=\"ffao\"]\nTherefore the expression is equivalent to $ x^4 \\minus{} x^4 \\plus{} x^3 \\minus{} 4x^2 \\plus{} 2x \\minus{} \\sqrt {3} \\equal{} x^3 \\minus{} 4x^2 \\plus{} 2x \\minus{} \\sqrt {3}$\n[/quote]\r\n[hide]$ 2\\plus{}\\sqrt{3}$ is a root of $ x^2\\minus{}4x\\plus{}1\\equal{}0$ so $ x^3 \\minus{} 4x^2 \\plus{} 2x \\minus{} \\sqrt {3}\\equal{}x(x^2\\minus{}4x\\plus{}1)\\plus{}x\\minus{}\\sqrt{3}\\equal{}x\\minus{}\\sqrt{3}\\equal{}2$[/hide]" } { "Tag": [ "number theory", "prime numbers", "number theory unsolved" ], "Problem": "Find all prime numbers $ p$ such that $ 1\\plus{}p\\plus{}p^2\\plus{}...\\plus{}p^{p\\minus{}1}$ and $ 1\\minus{}p\\plus{}p^2\\minus{}p^3\\plus{}....\\plus{}p^{p\\minus{}1}$ are prime numbers", "Solution_1": "http://www.mathlinks.ro/viewtopic.php?t=229499" } { "Tag": [ "geometry", "incenter", "circumcircle", "Euler", "geometric transformation", "reflection" ], "Problem": "Dear Mathlinkers,\r\nlet ABC be a triangle,\r\n1 the incircle , I the incenter, X the pointy of contact of 1 with BC, X' the antipode of X wrt 1, A\"B\"C\" the I-circumtriangle, U the point of intersection of B\"C\" and AI, and Fe the Feuerbach' s point of ABC.\r\nProve that Fe, U and X' are collinear.\r\nSincerely\r\nJean-Louis", "Solution_1": "Dear Jean-Louis\r\nWhat is \"$ I$-circumtriangle\"?", "Solution_2": "Dear Zhang Fangyu,\r\nthe I-curcumtriangle is the triangle forwicg the vertex are the second intersection od the cevians AI, BI, CI with the circumcircle of ABC.\r\nSincerely\r\nJean-Louis", "Solution_3": "A1,A2 are the projections of A onto BI,CI\r\n\u25b3AA1A2\u223d\u25b3ABC A1A2\u2225BC \r\nFrom here we (http://www.mathlinks.ro/viewtopic.php?t=219598) we know X\u2019Fe is the Euler Line of \u25b3AA1A2 \r\nSo now our goal is to prove U is on Euler Line of \u25b3AA1A2 \r\nNote that AA\u2019\u2019\u22a5B\u2019\u2019C\u2019\u2019 => U,A,C\u2019\u2019,A2 are concyclic\r\n\u2220UAA2=UA2A=\u2220CC\u2019\u2019B\u2019\u2019=\u2220AC\u2019\u2019B\u2019\u2019=1/2\u2220B\r\nWe have \u25b3AA2I is a right triangle , thus U is midpoint of IA => U is the circumcenter of AA2IA1 => U lies on the Euler Line of \u25b3AA1A2 => X\u2019, U , Fe are collinear[/url]", "Solution_4": "[quote]A1,A2 are the projections of A onto BI,CI\n\u25b3AA1A2\u223d\u25b3ABC A1A2\u2225BC\nFrom here we (http://www.mathlinks.ro/viewtopic.php?t=219598) we know X\u2019Fe is the Euler Line of \u25b3AA1A2\nSo now our goal is to prove U is on Euler Line of \u25b3AA1A2\nNote that AA\u2019\u2019\u22a5B\u2019\u2019C\u2019\u2019 => U,A,C\u2019\u2019,A2 are concyclic\n\u2220UAA2=UA2A=\u2220CC\u2019\u2019B\u2019\u2019=\u2220AC\u2019\u2019B\u2019\u2019=1/2\u2220B\nWe have \u25b3AA2I is a right triangle , thus U is midpoint of IA => U is the circumcenter of AA2IA1 => U lies on the Euler Line of \u25b3AA1A2 => X\u2019, U , Fe are collinear.[/quote]\r\n\r\nIndeed plane geometry, the ideas from previous messages keep coming back. \r\nYour short and good solution is even still a bit too long, because $ U$ can be immediately proven to be the desired circumcenter:\r\n\r\nSince $ I$ is the orthocenter of $ \\Delta A''B''C''$, it follows that $ U$ is the midpoint of $ A$ and $ I$, and hence the circumcenter of $ \\Delta AA_1A_2$. Since $ X'$ is on the Euler line of this triangle, the problem now follows from Hatzipolakis' theorem.\r\n\r\nBut anyway, your solution is good. I wonder if we can avoid all those overkill far-fetched theorems and find an independent olympiad solution. I bet we can use Monge-d'Alembert with some additional arguments or something. I don't have time right now to investigate further.", "Solution_5": "Could anyone show me why X\u2019Fe is the Euler Line of \u25b3AA1A2 :) \r\nI searched http://www.mathlinks.ro/viewtopic.php?t=219598 but I didn't find a specific proof\r\nThank you very much!", "Solution_6": "Dear Mathlinkers,\r\nsee for example :\r\nVonk J., The Feuerbach Point and Reflections of the Euler Line, Forum Geometricorum, Volume 9 (2009) 47\u201355 ;\r\nhttp://forumgeom.fau.edu/FG2009volume9/FG200905.pdf\r\nSincerely\r\nJean-Louis", "Solution_7": "Thank you so much :lol:", "Solution_8": "Dear jayme\uff0cyour problem actually is that $ Fe$ is on the $ Line\\minus{}OH$ what it the Euler-line of \u0394$ DPQ$ .\r\nBut your description prompt another way to prove the Problem.", "Solution_9": "Dear Lym and Mathlinkers,\r\nif I am not wrong, ithink that this way concerns another problem post on Mathlinks recently.\r\nYour observation is nice\r\nSincerely\r\nJean-Louis", "Solution_10": "jayme,I read the proof in your excellent article :) But I find that it used a tough property of $ X_{80}$\r\nIt states that $ X_{80}$ is the reflection of incenter wrt feuerbach's.I spent a whole afternoon to solve it,and I create a solution using inversion and a bit long calculation(if necessary I will post here).Do you have or know a syntheic proof for it?I am eager to see it :)", "Solution_11": "Dear Mathlinkers,\r\nit is not my proof but these of Jan Vonk.\r\nSincerely\r\nJean-Louis", "Solution_12": "Can anyone provide a synthetic proof :) \r\nMy solution is very long but I like it very much.", "Solution_13": "Dear Mathlinkers,\r\nfor more explanation, see for example\r\nhttp://perso.orange.fr/jl.ayme vol. 4 Symetriques de OI par rapport aux triangles de contact et m\u00e9dian p. 21\r\nSincerely\r\nJean-Louis" } { "Tag": [], "Problem": "Prove:\r\n$\\frac{1989}{2}-\\frac{1988}{3}+\\frac{1987}{4}-...+\\frac{1}{1990}= \\frac{1}{996}+\\frac{3}{997}+\\frac{5}{998}+...+\\frac{1989}{1990}$", "Solution_1": "Using a nice identity of the harmonic numbers:\r\n\r\n[hide=\"Solution\"]\n$h_{n}= \\sum_{i = 1}^{n}\\frac1i$\n$\\frac12 h_{n}= \\sum_{i = 1}^{n}\\frac1{2i}$\n$h_{2n}-\\frac12 h_{n}= \\sum_{i = 1}^{n}\\frac1{2i-1}$\n$h_{2n}-h_{n}=-\\sum_{i = 1}^{2n}\\frac{(-1)^{n}}{i}$\n\n$\\sum_{i = 2}^{2n}(-1)^{i}\\frac{2n-i+1}{i}=-1+\\sum_{i = 2}^{2n}(-1)^{i}\\frac{2n+1}{i}=-1+(2n+1)(h_{n}-h_{2n}+1)$\n\n$\\sum_{i =1}^{n}\\frac{2i-1}{n+i}= \\sum_{i =1}^{n}2-\\frac{2n+1}{n+i}= 2n-(2n+1)(h_{2n}-h_{n})$\n[/hide]" } { "Tag": [ "parameterization", "function", "calculus", "derivative", "conics", "ellipse", "limit" ], "Problem": "For which real $p$ has system of equations:\r\n$(x-y)^2+z^2=(z-y)^2+x^2=p^2$\r\n$xyz=p^3$\r\nat least two solutions $(x,y,z)$?", "Solution_1": "Subtracting the first 2 equations immediately yields $y(z - x) = 0$. But if $y = 0$, then from the 2nd equation $p = 0$ and substituting this to the first 2 equations leads to a single solution $x = y = z = 0$. Hence, it must be $z = x$ and we actually have only 2 equations:\r\n\r\n$2x^2 - 2xy + y^2 = p^2$\r\n$yx^2 = p^3$\r\n\r\nObviously, this set of 2 equations always has the solution $x = y = p$ for any $p \\neq 0$. The second equation represents the function $y = \\frac {p^3}{x^2}$, which has derivative at $x = p$ equal to\r\n\r\n$y' = -2\\frac{p^3}{x^3}, \\ y'(p) = -2$\r\n\r\nThe first equation represents an ellipse centered at the origin $[0, 0]$ (because the equation is symmetrical WRT the exchange $[x, y] \\leftrightarrow [-x, -y]$) and passing through the points $[-p, -p], [0, -p], [0, p], [p, p]$. Solving the ellipse equation for $y$ and differentiating:\r\n \r\n$y = x \\pm \\sqrt{p^2 - x^2}$\r\n$y' = 1 \\mp \\frac {x}\\sqrt{p^2 - x^2}$ $, \\ \\lim_{x\\to \\pm p}{y'} = \\infty$\r\n\r\nHence, this ellipse has vertical tangents at the points $[-p, -p], [p, p]$. This is sufficient to conclude that the ellipse intersects the graph of the function $y = \\frac {p^3}{x^2}$ at exactly one more point different from the point $[p, p]$ for any $p \\neq 0$.\r\n\r\nEdit:\r\n$x = y = -p$ is not a solution, error fixed.\r\n$p\\ \\#\\ 0$ replaced by $p \\neq 0$.\r\nThanks.\r\n\r\n...", "Solution_2": "correct me if i'm wrong, but it seems to be that $x=y=z=-p$ is not a solution?", "Solution_3": "You are right, I will edit out the error. Tx.", "Solution_4": "yetti you can use \\neq (not equal) command to make $p \\neq 0$ if you want ;)" } { "Tag": [ "inequalities", "inequalities proposed" ], "Problem": "(***)\r\n59. Let $ x,y$ be distinct positive reals. Determine the minimum value of\r\n\\[ \\frac {1}{(x \\minus{} y)^2} \\plus{} \\frac {16}{xy} \\plus{} 6(x \\plus{} y)\r\n\\]\r\nand determine when that minimum occurs.\r\n\r\n[hide=\"Comment:\"] A proof exists with only Cauchy and AM-GM.[/hide]", "Solution_1": "[quote=\"Altheman\"](***)\n59. Let $ x,y$ be distinct positive reals. Determine the minimum value of\n\\[ \\frac {1}{(x \\minus{} y)^2} \\plus{} \\frac {16}{xy} \\plus{} 6(x \\plus{} y)\n\\]\nand determine when that minimum occurs.\n\n[hide=\"Comment:\"] A proof exists with only Cauchy and AM-GM.[/hide][/quote]\r\nLet $ x \\ge y$ then : $ LHS \\ge (\\frac {1}{(x \\minus{} y)^2}\\plus{}(x\\minus{}y)\\plus{}(x\\minus{}y))\\plus{}(4x\\plus{}8y\\plus{}\\frac {16}{xy} ) \\ge (3)\\plus{}(24)\\equal{}27$;( by AM-GM )\r\nequality is for $ x\\equal{}2,y\\equal{}1;$ :)", "Solution_2": "Hey, I like your solution, it is very nice!!\r\n\r\nHere is my solution:\r\n\r\n$ \\frac{1}{(x\\minus{}y)^2}\\plus{}\\frac{64}{4xy}\\plus{}6(x\\plus{}y)\\ge \\frac{(1\\plus{}8)^2}{(x\\minus{}y)^2 \\plus{} 4xy}\\plus{}6(x\\plus{}y)$\r\n\r\n$ \\equal{}\\frac{81}{(x\\plus{}y)^2}\\plus{}3(x\\plus{}y)\\plus{}3(x\\plus{}y)\\ge 3\\sqrt[3]{81*3*3}\\equal{}27$.\r\n\r\nEquality occurs iff $ \\frac{1}{(x\\minus{}y)^2}\\equal{}\\frac{8}{4xy}$ and $ \\frac{81}{(x\\plus{}y)^2}\\equal{}3(x\\plus{}y)$. The first one factors as $ (x\\minus{}2y)(2x\\minus{}y)\\equal{}0$. The second equation is $ x\\plus{}y\\equal{}3$. So $ (x,y)\\equal{}(1,2),(2,1)$." } { "Tag": [], "Problem": "What is the maximum number of bishops that can be placed on any $8\\times 8$ chessboard such that at most three bishops lie on any diagonal?\r\n\r\nHere are my thoughts. Can they be made more rigorous? \r\n[hide=\"Approach\"]Consider all white squares. There are two 1 square diagonals, two 2 square diagonals, ... , and two 7 square diagonals. There is also one 8 square diagonal, for a total of 15 diagonals.\n\nEach bishop is contained in exactly two diagonals, and each diagonal (except the 1 and 2 squares) can contain 3 bishops.\n\nHence, the bishops can theoretically occupy $3(15)-4-2=39$ spots on these 15 diagonals (subtract 4 and 2 since there are two diagonals of length 1 and two of length 2), where one bishop occupies \"two spots\" (one on each diagonal). However, since one bishop occupies two spots at a time, there can only be $38$ occupied, so $19$ bishops on white squares. \n\nSimilarly, $19$ bishops on black squares, for a total of $38.$\n[/hide]\nIf this method works, I believe this generalization will folllow\n[hide=\"Generalization?\"]For each color, $3(2n-1)-4-2$ spots. Only $6n-10$ can be used, so $3n-5$ bishops on each color.\n\nIn total $6n-10$ bishops.[/hide]\r\nNot sure how to show that the maximum value is attainable though.", "Solution_1": "aren't you 4everwise?" } { "Tag": [ "ratio", "vector", "geometry", "trapezoid" ], "Problem": "Prove that two medians in a triangle are congruent iff the sides which they bisect are congruent.", "Solution_1": "[hide]I don't have a diagram, but...\n\nSince two sides of a triangle are congruent, the triangle, say ABC, is isosceles, and the medians are coming from the 2 base angles B and C. The medians intersect sides AB and AC at M and N, respectively.\n\nSince $ AB \\equal{} AC$, $ MB \\equal{} NC$, and since $ \\angle B \\equal{} \\angle C$ and $ BC \\equal{} BC$, $ \\triangle BMC\\cong\\triangle CNB$.\nThus, $ CM \\equal{} BN$\n\nNow, for the \"only if\" part:\n\nConsider triangle ABC, with medians CM and BN, with CM=BN. Denote the intersection of CM and BN as P. Because the intersection of the medians divide the medians into a ratio of 2:1, and since CM=BN, PM=PN and PB=PC. Since $ \\angle MPB \\equal{} \\angle NPC$, $ \\triangle MPB\\cong\\triangle NPC$.\n\nThus, $ \\angle ABN \\equal{} \\angle ACM$. Since $ \\angle A \\equal{} \\angle A$, and $ BN \\equal{} MC$, $ \\triangle AMC\\cong\\triangle ANB$, and thus AB=AC.\n\nQED[/hide]\r\n\r\nEDIT: Yeah, didn't notice it sad \"iff\", but added it now.", "Solution_2": "What about the other direction?", "Solution_3": "Let $ A$ be the origin, and let $ 2y$, $ 2z$ be vectors that denote points $ B$ and $ C$, respectively. Let $ N$ and $ P$ be the midpoints of $ AC$ and $ AB$, respectively. Then \\[ BN\\equal{}CP\\iff BN^2\\equal{}CP^2\\iff (2y\\minus{}z)\\cdot (2y\\minus{}z)\\equal{}(2z\\minus{}y)\\cdot (2z\\minus{}y)\\]\r\n\\[ \\iff 2y\\cdot 2y\\minus{}4y\\cdot z\\plus{}z\\cdot z\\equal{}2z\\cdot 2z\\minus{}4z\\cdot y\\plus{}y\\cdot y\\equal{}y\\cdot y\\equal{}z\\cdot z\\iff AB\\equal{}AC\\]\r\n\r\n\r\nyay", "Solution_4": "Let $ AA'$ and $ BB'$ be two medians in the triangle $ ABC$\r\n$ AA'\\equal{}BB'$,$ AG\\equal{}BG$, $ AB'A'B$ is a trapezoid therefore $ AB'\\equal{}BA'$ $ \\implies$ $ BC\\equal{}AC$", "Solution_5": "[hide] Let ABC be an isosceles triangle with AB=AC\n\nand BD and CE are the medians\n\nclearly AD=AE and \\equal{} 0$ to satisfy the equation(since $ 0 \\equal{} < {\\theta} \\equal{} < pi$) and $ cos{\\theta} \\equal{} 0$ is not a solution to this equation!\r\nSo, we just need to care about $ cos{\\theta} > 0$\r\nNow, dividing both sides by $ cos{\\theta}$ we get: $ \\frac {1}{cos^2{\\theta}} \\equal{} 2004(tan{\\theta} \\minus{} 1)$.\r\nThis equation is equivalent to: $ 1 \\plus{} tan^2{\\theta} \\equal{} 2004tan{\\theta} \\minus{} 2004$. \r\nThis is the quadratic equation, so we easily get: $ tan{\\theta} \\equal{} 2005$ or $ tan{\\theta} \\equal{} \\minus{} 1$. Since $ sin{\\theta}$ and $ cos{\\theta} > 0$, we eliminate all negative values of $ tan{\\theta}$. Finally, the sum of $ tan{\\theta} \\equal{} 2005$(Q.E.D)\r\nHmm... The 2nd one seems to be tougher than this one even though it looks familiar to me.I'll post the solution to it later.", "Solution_3": "whoops, forgot to post :huh: \r\n\r\nghjk: In the first question, $ 0 \\le \\theta < \\pi$, so $ \\tan \\theta$ can take any real. :wink: \r\n[hide=\"1\"]\nDividing both sides by $ \\cos^2 \\theta$ (clearly $ \\cos \\theta \\neq 0$), we have $ \\sec^2 \\theta = 1 + \\tan^2 \\theta = 2004 \\tan \\theta - 2004$, and by vieta's [each value occurs once] the sum is $ 2004$. \n[/hide]\n[hide=\"2\"]\nRepeatedly applying the product to sum formula yields\n\\begin{align*}8\\cos \\alpha \\cos 3\\alpha \\cos 5\\alpha \\cos 7\\alpha & = 2 \\left[\\cos 8 \\alpha + \\cos 2\\alpha\\right]\\left[\\cos 8\\alpha + \\cos 6 \\alpha \\right] \\\\\n& = 2\\left[\\cos^2 8\\alpha + \\cos 8\\alpha \\cos 6\\alpha + \\cos 8\\alpha \\cos 2\\alpha + \\cos 6\\alpha \\cos 2\\alpha\\right] \\\\\n& = \\left[\\cos 2\\alpha + \\cos 4\\alpha + \\cdots + \\cos 14\\alpha \\right] - \\cos 12\\alpha.\\end{align*}\nThus, we have $ \\frac {1}{2} = \\sum_{n = 1}^{7} \\cos 2n\\alpha = \\text{Re}\\left(\\sum_{n = 1}^7 e^{i2n\\alpha}\\right) = \\text{Re}\\left(\\frac {e^{i16 \\alpha} - 1}{e^{i2\\alpha} - 1}\\right) = \\frac {\\sin 7\\alpha \\cos 8\\alpha}{\\sin \\alpha}$. Then $ \\sin \\alpha = 2\\sin 7\\alpha \\cos 8\\alpha \\cdot \\frac {\\sin 8\\alpha}{\\sin 8\\alpha} = \\frac {\\sin 7 \\alpha \\sin 16\\alpha}{\\sin 8 \\alpha}$ $ \\Longrightarrow 2\\cos 9\\alpha = \\cos 23\\alpha + \\cos 7\\alpha$ $ \\Longrightarrow \\cos 9\\alpha = \\cos 15\\alpha \\cos 8 \\alpha$ .. shouldn't be too much harder from here ..\n[/hide]", "Solution_4": "[quote=\"azjps\"]whoops, forgot to post :huh: \n\nghjk: In the first question, $ 0 \\le \\theta < \\pi$, so $ \\tan \\theta$ can take any real. :wink: \n[/quote]\r\nHmm... Did you read my argument above? I already showed that $ cos{\\theta}>0$(I leave its proof for you to think of it first!). Therefore, $ tan{\\theta}>0$ with $ 0 \\le\\theta <\\pi$ :)", "Solution_5": "Edit: Actually, I'm right :lol: its $ \\tan^2 \\theta \\minus{} 2004\\tan \\theta \\plus{} 2005 \\equal{} 0$, the roots are both positive.", "Solution_6": "[quote=\"azjps\"]Edit: Actually, I'm right :lol: its $ \\tan^2 \\theta \\minus{} 2004\\tan \\theta \\plus{} 2005 \\equal{} 0$, the roots are both positive.[/quote]\r\nMy equation is right, but I made a dumb factorization. :o Since the quadratic equation gives two positive roots for tan, the sum is indeed 2004(you're lucky since it gives both positive roots, aren't u?) :oops:", "Solution_7": "[quote=\"ghjk\"]Beautiful problem! Where did you get this problem, my friend??\nI hope you like my solution.[/quote]\r\n\r\n :D I really like your solution!", "Solution_8": "[quote=\"azjps\"]\n[hide=\"2\"]\nRepeatedly applying the product to sum formula yields\n\\begin{align*}8\\cos \\alpha \\cos 3\\alpha \\cos 5\\alpha \\cos 7\\alpha & = 2 \\left[\\cos 8 \\alpha + \\cos 2\\alpha\\right]\\left[\\cos 8\\alpha + \\cos 6 \\alpha \\right] \\\\\n& = 2\\left[\\cos^2 8\\alpha + \\cos 8\\alpha \\cos 6\\alpha + \\cos 8\\alpha \\cos 2\\alpha + \\cos 6\\alpha \\cos 2\\alpha\\right] \\\\\n& = \\left[\\cos 2\\alpha + \\cos 4\\alpha + \\cdots + \\cos 14\\alpha \\right] - \\cos 12\\alpha.\\end{align*}\nThus, we have $ \\frac {1}{2} = \\sum_{n = 1}^{7} \\cos 2n\\alpha = \\text{Re}\\left(\\sum_{n = 1}^7 e^{i2n\\alpha}\\right) = \\text{Re}\\left(\\frac {e^{i16 \\alpha} - 1}{e^{i2\\alpha} - 1}\\right) = \\frac {\\sin 7\\alpha \\cos 8\\alpha}{\\sin \\alpha}$. Then $ \\sin \\alpha = 2\\sin 7\\alpha \\cos 8\\alpha \\cdot \\frac {\\sin 8\\alpha}{\\sin 8\\alpha} = \\frac {\\sin 7 \\alpha \\sin 16\\alpha}{\\sin 8 \\alpha}$ $ \\Longrightarrow 2\\cos 9\\alpha = \\cos 23\\alpha + \\cos 7\\alpha$ $ \\Longrightarrow \\cos 9\\alpha = \\cos 15\\alpha \\cos 8 \\alpha$ .. shouldn't be too much harder from here ..\n[/hide][/quote]\r\n\r\n\r\nIs there a better solution for this?", "Solution_9": "Lets solve the system $ a \\equal{} x(y \\minus{} x)$ $ x^2 \\plus{} y^2 \\equal{} 1$ where $ a$ is given.\r\n\r\nNaturally, we homogenize in $ x,y$ to get $ a(x^2 \\plus{} y^2) \\equal{} x (y \\minus{} x)$ and we easily solve for $ x: y$ by the quadratic formula and we are done.\r\n\r\n\r\nThis motivates the other solution that was given...\r\n\r\n\r\nThis works for any system in the form $ A \\equal{} (Bx \\plus{} Cy)(Dx \\plus{} Ey)$ and $ x^2 \\plus{} y^2 \\equal{} 1$...", "Solution_10": "[quote=\"Altheman\"]Lets solve the system $ a \\equal{} x(y \\minus{} x)$ $ x^2 \\plus{} y^2 \\equal{} 1$ where $ a$ is given.\n\nNaturally, we homogenize in $ x,y$ to get $ a(x^2 \\plus{} y^2) \\equal{} x (y \\minus{} x)$ and we easily solve for $ x: y$ by the quadratic formula and we are done.\n\n\nThis motivates the other solution that was given...\n\n\nThis works for any system in the form $ A \\equal{} (Bx \\plus{} Cy)(Dx \\plus{} Ey)$ and $ x^2 \\plus{} y^2 \\equal{} 1$...[/quote]\r\n\r\n\r\nWhat in the world does this have to do with trig?", "Solution_11": "He was just explaining the motivation for the solution to #1, namely homogenizing. (there should be a comma between $ a \\equal{} x(y\\minus{}x)$ and $ x^2 \\plus{} y^2 \\equal{} 1$, and now look at the substitution $ x \\equal{} \\cos \\theta, y \\equal{} \\sin \\theta$)." } { "Tag": [ "LaTeX", "advanced fields", "advanced fields theorems" ], "Problem": "What is it? \r\nHow to prove things with it?\r\nFor example, how to proof that (in this method)\r\nA -> B |-- (C V A) -> (C V B)\r\n\r\n(I can prove it with true-table only...)\r\n\r\nThank you", "Solution_1": "Please $ \\text{\\LaTeX}$ your formula. Is it $ (A \\rightarrow B) \\wedge (C \\vee A) \\rightarrow C \\vee B$ ?", "Solution_2": "I believe it's: $ A \\Rightarrow B \\vdash (C \\vee A) \\Rightarrow (C \\vee B)$\r\n\r\nThe symbol \"$ p, q, r, \\ldots \\vdash c$\" means \"given $ p, q, r, \\ldots$, prove $ c$\".", "Solution_3": "$ A \\Rightarrow B \\vdash (C \\vee A) \\Rightarrow (C \\vee B)$ Yes. \r\n\r\n(I am sorry, I relatively new user and I am not so good in Latex)" } { "Tag": [ "group theory", "abstract algebra", "geometry", "geometric transformation", "rotation", "superior algebra", "superior algebra solved" ], "Problem": "Without any computation show $S_{4}$ has a subgroup isomorphic to $Z_{4}$ and a subgroup isomorphic to $K_{4}$ (Klein four group). Extend this for $S_{n}.$", "Solution_1": "$S_{n}$ always contains an $n$-cycle $(1,2,3,...,n)$ clearly having order $n$.\r\nAnd $K_{4}$ is the group of permuations of two stacks of two elements without perturbation, thus also contained.\r\n$S_{n}$ also always contains the dihedral group $D_{n}$ since it is a group of permutations on $n$ objects. By this, the rotations give another way of getting an element of order $n$.", "Solution_2": "[quote=\"ZetaX\"]$S_{n}$ always contains an $n$-cycle $(1,2,3,...,n)$ clearly having order $n$.\nAnd $K_{4}$ is the group of permuations of two stacks of two elements without perturbation, thus also contained.\n$S_{n}$ also always contains the dihedral group $D_{n}$ since it is a group of permutations on $n$ objects. By this, the rotations give another way of getting an element of order $n$.[/quote]\r\n\r\nYou are right, but note that I said that without any computation!! What you say about this problem:\r\n\r\nWithout any computation show $S_{8}$ has subgroups isomorphic to $Z_{8}, Z_{2}\\times Z_{4},Z_{2}\\times Z_{2}\\times Z_{2}, Q$ and $D$ where $Q$ is the quaternion group of order 8 and $D$ dihedral group of order 8.", "Solution_3": "Where did I use computations (even for the cycle I gave a pure combi-algebraical proof).\r\nAnd the generalisation is simply that $S_{n}$ contains every group that is a permutation group on $\\leq n$ element set, so especially contains any group of order $\\leq n$ (Cayley's theorem).", "Solution_4": "Ok, this is my solution exactly.\r\n\r\nThanks" } { "Tag": [], "Problem": "Jest dany trojkat $ABC$ z $\\angle ABC = 3\\angle CAB$. Punkty $M$ i $N$ sa wybrane na boku $AC$ z $N$ pomiedzy $A$ i $M$ tak, ze $\\angle CBM =\\angle MBN = \\angle NBA$. Niech $L$ bedzie dowolnym punktem wewnetrznym odcinka $BN$, zas $K$ punktem na $BM$ takim, ze $LK || AC$. Udowodnij, ze proste $AL$,$NK$ i $BC$ przecinaja sie w jednym punkcie.\r\n\r\nP.S. reanimacja trwa :P", "Solution_1": "Najwyzszy czas podac rozwiazanie :P Niech $ NK\\cap BC\\equal{}P$, $ LK\\cap AC\\equal{}Q$ oraz $ LK\\cap AB\\equal{}R$. Z tw. Menelausa mamy $ \\frac{BP\\cdot QK\\cdot CN}{PQ\\cdot KL\\cdot BN}\\equal{}1\\iff \\frac{BP\\cdot BC\\cdot QC}{PQ\\cdot BN\\cdot BC}\\equal{}1\\iff \\frac{BP}{PQ}\\equal{}\\frac{BN}{QC}$. Teraz mamy $ \\frac{BN\\cdot QL\\cdot AR}{PQ\\cdot RL\\cdot AB}\\equal{}\\frac{BN\\cdot CN\\cdot CQ}{CQ\\cdot AN\\cdot BL}\\equal{}1$ qed", "Solution_2": "Ca\u0142kiem nie\u017ale: po blisko 3 latach rozwi\u0105zanie (nie sprawdza\u0142em czy poprawne...) \r\n\r\n :)", "Solution_3": "Sa literowki. Ma byc\r\n$ LK\\cap BC \\equal{} Q$ i w ostatniej rownosci$ \\frac{BP\\cdot QL\\cdot AR}{PQ\\cdot RL\\cdot AB}\\equal{}\\frac{BN\\cdot CN\\cdot CQ}{CQ\\cdot AN\\cdot BC}\\equal{}1$" } { "Tag": [ "algebra", "polynomial", "calculus", "derivative", "induction", "algebra proposed" ], "Problem": "If $b_{1}, b_{2}, \\ldots, b_{n}$ are non-negative reals not all zero, then prove that the polynomial \\[x^{n}-b_{1}x^{n-1}-b_{2}x^{n-2}-\\ldots-b_{n}=0\\] has only one positive root $p$, which is simple. Moreover prove that any root of the polynomial does not exceed $p$ in absolute value.", "Solution_1": "[quote=\"iura\"]If $b_{1}, b_{2}, \\ldots, b_{n}$ are non-negative reals not all zero, then prove that the polynomial\n\\[x^{n}-b_{1}x^{n-1}-b_{2}x^{n-2}-\\ldots-b_{n}=0 \\]\nhas only one positive root $p$, which is simple. [/quote]\nIt is obviosly, consider \n(*) $1-b_{1}y-b_{2}y^{2}-...-b_{n}y^{n}, \\ y=1/x$.\n[quote]Moreover prove that any root of the polynomial does not exceed $p$ in absolute value.[/quote]\r\nIt followed from (*).", "Solution_2": "Part 1: Use Descartes' Sign Rule .\r\nPart 2: If $x_{0}$ is a real root, then $|x_{0}|^{n}\\leq b_{1}|x_{0}|^{n-1}+...+b_{n}$. And we're done.", "Solution_3": "In book \"Polynomials\" by Prasolov, this is stated as a theorem (Cauchy's)", "Solution_4": "[quote=\"Goblin\"]In book \"Polynomials\" by Prasolov, this is stated as a theorem (Cauchy's)[/quote]\r\nYes, theorem 1.3, page 11. But prove in that not as of mine :P", "Solution_5": "can you please post an entire solution?", "Solution_6": "[quote=\"iura\"]If $b_{1}, b_{2}, \\ldots, b_{n}$ are non-negative reals not all zero, then prove that the polynomial\n\\[f(x)=x^{n}-b_{1}x^{n-1}-b_{2}x^{n-2}-\\ldots-b_{n}=0 \\]\nhas only one positive root $p$, which is simple. Moreover prove that any root of the polynomial does not exceed $p$ in absolute value.[/quote]\r\nThis is prove in that book.\r\n\r\nPut $F(x)=-\\frac{f(x)}{x^{n}}=\\frac{b_{1}}{x}+\\frac{b_{2}}{x^{2}}+...+\\frac{b_{n}}{x^{n}}-1.$ If $x\\not = 0$ then $f(x)=0$ is equivalent to $F(x)=0$. Now if $x\\to+\\infty$ then $F(x)\\to-1$ and if $x\\to 0^{+}$ then $F(x)\\to+\\infty$, by $F$ is continuous and monotone on $(0,+\\infty)$ we have $f(x)$ has only a positive root, denote it by $p$. We also have $-\\frac{f'(p)}{p^{n}}=F'(p)=-\\frac{b_{1}}{p^{2}}-...-\\frac{nb_{n}}{p^{n+1}}<0$, so $p$ is simple.\r\n\r\nAssume that $x_{0}$ is a root of $f$. We have $q=|x_{0}|\\leq p$. In fact, if $q>p$ we have $F(q)<0$ and so $f(q)>0$. But from $x_{0}^{n}=b_{1}x_{0}^{n-1}+...+b_{n}$ we have $q^{n}\\leq b_{1}q^{n-1}+...+b_{n}$ or $f(q)\\leq 0$. Contradiction!", "Solution_7": "You could use derivatives from the beginning:\r\nUse induction on $n$. If $f$ is the polynomial in the problem, then $f^\\prime$ satisfies more or less the induction hypothesis (you should watch out for the special case $x^{n}-\\alpha$). Let $x_{0}$ be the positive root of $f^\\prime$. Then, we have that $f$ is strictly decreasing on $[0,x_{0}]$ and strictly increasing on $\\left[ x_{0}, \\infty \\right)$. But $f(0) \\leq 0$, so $f$ has a unique positive root.", "Solution_8": "You must remember that we need prove $p$ is SIMPLE!", "Solution_9": "I thought that was clear. We have that $f \\left( x_{0}\\right) < 0$, so the positive root $u$ of $f$ must satisfy $u > x_{0}\\, \\Longrightarrow \\, f^\\prime (u) > 0$." } { "Tag": [], "Problem": "I cant seem to comprehend this type of questions. Thorough explanation would help me a lot.\r\n\r\n2. A flare is dropped from a plane flying over level ground at a velocity of $70 m/s$ in the\r\nhorizontal direction. At the instant the flare is released, the plane begins to accelerate horizontally\r\nat $0.75 m/s^{2}$. The flare takes $4.0 s$ to reach the ground. Assume air resistance is negligible.\r\nRelative to a spot directly under the flare at release, where does the flare land?", "Solution_1": "I don't see why the acceleration of the plane is important... The time the flare needs to reach the ground is $t$, so the distance it covers in the horizontal direction is $vt$, since there are no forces acting on it in that direction. That should be the answer.", "Solution_2": "thanks, I appreciate the help. :)" } { "Tag": [], "Problem": "find the last two digits of\r\n$2^{2004},3^{2004},4^{2004}$\r\nhow would you find the first two digits of the same numbers", "Solution_1": "the last two digits is basically:\r\n\r\n$\\varphi(100)=(2-1)\\cdot2^{1}(5-1)\\cdot5^{1}=40$\r\n\r\n$2^{2004}\\bmod100\\equiv2^{4}\\bmod100=16$\r\n$3^{2004}\\bmod100\\equiv3^{4}\\bmod100=81$\r\n$(2^{2004})^{2}\\bmod100\\equiv(2^{4})^{2}\\bmod100=56$\r\n\r\nthx..", "Solution_2": "SM4RT, Euler's totient theorem is only valid when the base is relatively prime to the modulus.", "Solution_3": "But it gets the correct answer for each problem, lol. \r\n\r\n[hide=\"$2^{2004}$\"]\nWe have that\n\\[16^{ \\varphi(25)}=16^{20}\\equiv 1 \\bmod 25 \\ \\ \\wedge \\ \\ 16^{20}\\equiv 0 \\bmod 4 \\stackrel{\\text{CRT}}{\\implies }16^{20}\\equiv 76 \\bmod 100 \\]\nso\n\\[2^{2004}=16^{501}\\equiv 16 \\cdot (76)^{25}\\bmod 100 \\]\nBut, notice that\n\\[76 \\bmod 100 \\equiv 76^{2}\\bmod 100 \\equiv 76^{3}\\mod 100 \\equiv \\ldots \\]\nso our expression is equivalent to:\n\\[16 \\cdot (76) \\bmod 100 \\equiv \\boxed{16 \\bmod 100}\\]\n[/hide]\n\n[hide=\"$3^{2004}$\"]\nWe have\n\\[(3,100)=1 \\implies 3^{ \\varphi(100)}=3^{40}\\equiv 1 \\bmod 100 \\]\n\n\\[\\implies 3^{2004}\\equiv 3^{4}\\equiv \\boxed{81 \\bmod 100}\\]\n[/hide]\n\n[hide=\"$4^{2004}$\"]\nFrom the first problem, we know that $2^{2004}\\equiv 16 \\mod 100$. But,\n\\[4^{2004}=\\left( 2^{2004}\\right)^{2}\\equiv 16^{2}\\bmod 100 \\equiv \\boxed{ 56 \\bmod 100}\\]\n[/hide]\r\n(Click on each expression to see the solution. :D )", "Solution_4": "What is CRT and how does it apply?" } { "Tag": [], "Problem": "How would you produce formic acid from acetic acid?\r\n\r\n[hide=\"Is this OK?\"]$ \\ce{CH3COOH}\\plus{}\\ce{NaOH}\\longrightarrow\\ce{CH3COONa}\\plus{}\\ce{H2O}\\\\\n\\ce{CH3COONa}\\plus{}\\ce{NaOH}\\,(\\ce{CaO},\\Delta )\\longrightarrow\\ce{CH4}\\plus{}\\ce{Na2CO3}\\\\\n\\ce{CH4}\\plus{}\\ce{I2}\\longrightarrow\\ce{CH3I}\\plus{}\\ce{HI}\\\\\n\\ce{CH3I}\\plus{}\\ce{NaOH}\\,(\\text{aq})\\longrightarrow\\ce{CH3OH}\\plus{}\\ce{NaI}\\\\\n\\ce{CH3OH}\\plus{}2[\\ce{O}]\\longrightarrow\\ce{HCOOH}\\plus{}\\ce{H2O}$[/hide]", "Solution_1": "2 mistakes in step 3 itself.\r\n\r\n[hide=\"1\"]$ \\ce {HI}$ is a reducing agent and you will get back the alkane, meaning to say, the reaction is not completely towards the right. Hence one must use an oxidising agent also, say $ \\ce {HIO_3}$ or $ \\ce {HNO_3}$[/hide]\n\n[hide=\"2\"]Even still, you cannot gaurantee a 100% pure product. You will get a mixture of products. ($ \\ce {CH_2I_2}$, $ \\ce {CHI_3}$ etc will also be formed.)[/hide]", "Solution_2": "May I ask what in God's name might be the purpose of preparing formic acid from acetic acid? If you want to have practice in organic reactions there are much more instructive examples to work with.\r\n\r\nHere's the method I would use:\r\n\r\n1) Sell a bottle of acetic acid.\r\n\r\n2) With the money earned in 1), buy a bottle of formic acid.", "Solution_3": "[b]Side question:[/b]\r\n\r\nWell I was working on Schmidt reaction but I cant proceed further: \r\n\r\n+$ N_3H$ (Schmidt reaction) to give $ CH_3NH_2$\r\n\r\nHow can we go on to produce acid from an amine.", "Solution_4": "amine to amide\r\nthen hydrolysis of amide formed", "Solution_5": "Amine -> Amide how? Note that I want to form HCOOH", "Solution_6": "Thanks all for your help. A friend of mine suggested creating $ \\ce{Ca}$ salt and decomposing it to get propanone. Then create $ \\ce{HCOONa}$ and iodoform. But I can't understand the last step. Could anyone please help?\r\n\r\n@hell_ever: Yes I thought about the second mistake, but I couldn't find any other way of doing it. Could you please help fix the mistake or find another way of doing this?\r\n\r\n[quote=\"Carcul\"]May I ask what in God's name might be the purpose of preparing formic acid from acetic acid? If you want to have practice in organic reactions there are much more instructive examples to work with.\n\nHere's the method I would use:\n\n1) Sell a bottle of acetic acid.\n\n2) With the money earned in 1), buy a bottle of formic acid.[/quote]\r\n\r\nYes I'm pretty amateur in organic chemistry and I've been trying to learn organic reactions. But from what you say above, I don't think I should learn anymore about these, I can go to a store and buy whatever I need. You helped me lose my interest in this subject and now I think it's nothing but memorizing a stupid bunch of reactions. Thank you very much for making me realize this.", "Solution_7": "[quote=\"nayel\"]i think it's nothing but memorizing a stupid bunch of reactions[/quote]\natleast organic is a lot more than this\n\nwat if suddenly a fault appears and the starting product used by most of industries disappear....sm1 will hav 2 divise new method :oops: \n\n[hide][quote=\"nayel\"]i think it's nothing but memorizing a stupid bunch of reactions[/quote]\nactually i also think so :rotfl: :P [/hide]", "Solution_8": "Nayel, I apologize if my post above looked a bit harsh, but, as you said, you are \"pretty amateur in organic chemistry\". Therefore, according to my experience, this doesn't look like a problem with pedagogic value. Even for starters, the problem BanishedTraitor has posted about protoanemonin is an excellent one for learning several important reactions, their mechanisms, and to start thinking like a synthetic organic chemist. I advise you to look at it.\r\n\r\nAlso, there was nothing im my post above that implies Organic Chemistry \"it's nothing but memorizing a stupid bunch of reactions\". It's much more than that, specially in the laboratory.\r\n\r\nFinally, don't forget you can always prove me wrong by providing a nice synthesis of formic acid from acetic acid.", "Solution_9": "[hide=\"@ Carcul\"]I understand; I also apologize for my comments. But as a friend once told me: we are here to help one another, so please make sure no one gets upset. :) [/hide]\r\n\r\nWhat if we reduce the acetic acid (with for example $ \\ce{LiAlH4}$) to ethanol, and dehydrate it to ethylene. Then ozonolysis of the ethylene in presence of water should produce formic acid.\r\n\r\nShouldn't it work? :maybe:", "Solution_10": "Actually, ozonolysis in the presence of water should produce formaldehyde. In the presence of hydrogen peroxide (or direct treatment with hot potassium permanganate) would, in principle, yield formic acid." } { "Tag": [ "LaTeX" ], "Problem": "If you consider the problem:\r\n\r\nHow many base-4 numbers with $ k$ digits have at least one digit a \"2\" AND have no \"0\"s before that \"2\"? (Note the number doesn't have to have a \"0\")?\r\n\r\nThis is the answer written recursively:\r\n\r\n[hide]\n$ k_1 \\equal{} 1$\n$ k_n \\equal{} 4^{n\\minus{}1} \\plus{} 2*k_{n\\minus{}1}$\nSo, for $ k\\equal{}1,2,3,4$, you get $ k_n \\equal{} 1, 6, 28, 120$\n[/hide]\n\nMy question is\n\n[hide]\nWhy are the solutions perfect numbers? What is the relationship (an algebraic one would be fine, but if anyone knows an intuitive relationship, that would be better)\n[/hide]", "Solution_1": "[quote=\"magixter\"]Why are the solutions perfect numbers?[/quote]\r\nThey are not all perfect numbers. For example, $ k_6 \\equal{} 2016$ is not a perfect number. :lol:\r\n\r\nThere is a theorem that states that an integer is an even perfect number if and only if it is of the form $ 2^{p \\minus{} 1}M_p$ for some prime $ p$ and Mersenne prime $ M_p \\equal{} 2^p \\minus{} 1.$ From the recursive definition, it is easy to easy why some of the $ k_i$ should be of this form and hence are even perfect numbers. ;)", "Solution_2": "[quote=\"ValentineA\"][quote=\"magixter\"]Why are the solutions perfect numbers?[/quote]\nThey are not all perfect numbers. For example, $ k_6 \\equal{} 2016$ is not a perfect number. :lol:[/quote] Or, uh, $ k_4 \\equal{} 120$ ;) To slightly restate what ValentineA wrote, you can solve that recurrence to get $ k_n \\equal{} 2^{n \\minus{} 1} \\cdot (2^n \\minus{} 1)$, so it follows from the theorem ValentineA quoted that every even number is of the form $ k_p$ for some $ p$.\r\n\r\nBy the way, a LaTeX suggestion for the O.P.: \\cdot makes a much nicer multiplication dot than the asterix. If you want a multiplication cross, you can use \\times.", "Solution_3": "Heh, I missed $ k_4.$ :blush:" } { "Tag": [], "Problem": "Prove that for all positive integers $n$,\\[0<\\sum^n_{k=1}\\frac{g(k)}{k}-\\frac{2n}{3}<\\frac{2}{3}\\]where $g(k)$ denotes the greatest odd divisor of $k$", "Solution_1": "can anyone solve this problem?", "Solution_2": "are you sur that this in\u00e9galit\u00e9 it's true ?? :huh:", "Solution_3": "Do you have a counterexample???", "Solution_4": "Well indeed is not dificult at all...first let\u00b4s see that $\\displaystyle 0<\\sum^n_{k=1}\\frac{g(k)}{k}-\\frac{2n}{3}<\\frac{2}{3}$ is iff ${\\displaystyle \\frac{2n}{3}<\\sum^n_{k=1}\\frac{1}{p(k)}<\\frac{2n}{3}+\\frac{2}{3}}$ where $p(k)$ is the largest power of 2 which divides k. Now lets call $Q(n)=\\sum^n_{k=1}\\frac{1}{p(k)}$ then lets see that if n fit in the equality then 2n also...that is because:\r\n\r\n$Q(2n)=\\sum^{n}_{x=1}\\frac{1}{p(2x-1)}+\\sum^{n}_{x=1}\\frac{1}{p(2x)}$ now see that in the first sume there are just 1\u00b4s...\r\n\r\n$Q(2n)=n+\\sum^{n}_{x=1}\\frac{1}{p(2x)}=n+\\frac{1}{2}\\sum^{n}_{x=1}\\frac{1}{p(x)}=n+\\frac{1}{2}Q(n)$\r\nand if n fit in the equality then $\\frac{2(2n)}{3}+\\frac{2}{3}>n+\\frac{n}{3}+\\frac{1}{3}>n+\\frac{1}{2}Q(n)>n+\\frac{n}{3}=\\frac{2(2n)}{3}$\r\n so 2n fits...\r\n\r\nalso y $2n$ fits then $2n+1$ and that is because $Q(2n+1)=Q(n)+\\frac{1}{p(2x+1)}=Q(2n)+1$\r\nand that make 2n+1 fits in the equality...just check i have to leave...but lets see that $Q(1)=1$ and it fits so every other natural fits...just multiply by 2 and sum 1" } { "Tag": [ "inequalities", "search", "inequalities open" ], "Problem": ":rotfl: :) :o \r\nplease prove this inequlity", "Solution_1": "I think it maybe classify number theory.\r\n\r\nSee this\r\nhttp://www.mathlinks.ro/Forum/viewtopic.php?search_id=605111911&t=6233" } { "Tag": [ "inequalities", "linear algebra", "matrix" ], "Problem": "Problem. Give $ A,B \\in M_n(k)$. Prove that:\r\n1.\r\n$ rank(A \\plus{} B) \\leq rank(A) \\plus{} rank(B)$\r\n2.\r\n$ rank(A) \\plus{} rank(B) \\minus{} n \\leq rank (AB) \\leq min(rank(A),rank(B))$", "Solution_1": "1. More generally, let $ A,B \\in M_{m \\times n} (\\mathbb{K})$, and let $ T: V \\to W$, $ L: V \\to W$ (with $ \\dim V \\equal{} n$, $ \\dim W \\equal{} m$) be linear transformations represented by matrices $ A, B$, respectively. Then, $ A \\plus{} B$ represents the linear transformation $ T \\plus{} L$. Now, since $ Im(T\\plus{}L), Im(T), Im(L)$ are subspaces of $ W$ and $ Im(T\\plus{}L) \\subseteq Im(T) \\plus{} Im(L)$, then $ Im(T\\plus{}L)$ is a subspace of $ Im(T) \\plus{} Im(L)$, and so\r\n\r\n$ \\dim Im(T\\plus{}L) \\leq \\dim [Im(T) \\plus{} Im(L)]$\r\n\r\n$ \\equal{} \\dim Im(T) \\plus{} \\dim Im(L) \\minus{} \\dim [Im(T) \\cap Im(L)]$\r\n\r\n$ \\leq \\dim Im(T) \\plus{} \\dim Im(L)$.\r\n\r\nSince for any linear transformation F, the dimension of its image equals the rank of any matrix that represents it, we conclude from the previous inequality that\r\n\r\n$ rank(A\\plus{}B) \\leq rank(A) \\plus{} rank(B)$. $ \\Diamond$", "Solution_2": "2. Let A,B be matrices such that the product AB is defined. Since the columns of AB are linear combinations of the columns of A, we have $ rank(AB) \\leq rank (A)$. On the other hand, the rows of AB are linear combinations of the rows of B, and so $ rank(AB) \\leq rank B$. Hence, $ rank(AB) \\leq min\\,\\,[rank(A),rank(B)]$. The other inequality, $ rank(A) \\plus{} rank(B) \\leq rank(AB) \\plus{} n$, is Sylvester's inequality, which can be proved directly or as a special case of the Frobenius inequality. In any case, you can also use the concept of linear transformation, and I will leave that to you.", "Solution_3": "[quote=\"vipCD_A1\"]Problem. Give $ A,B \\in M_n(k)$. Prove that:\n\n2.\n$ rank(A) \\plus{} rank(B) \\minus{} n \\leq rank (AB)$[/quote]\r\n\r\nCan you detail solve?\r\nI have thought about it but I can't solve it. Please, carcul!", "Solution_4": "For instance, start by assuming that $ A \\equal{} diag \\{I_{r(A)}, O\\}$, where $ r(A)$ is the rank of A. Then show that $ r(AB) \\geq r(A) \\plus{} r(B) \\minus{} n$. Next, if A does not have the form above, there are invertible matrices P,Q such that $ D \\equal{} diag \\{I_{r(A)}, O\\} \\equal{} PAQ$. Using the previous result we have $ r(DB) \\equal{} r(PAQB) \\geq r(D) \\plus{} r(B) \\minus{} n \\equal{} r(A) \\plus{} r(B) \\minus{} n$. Now show that $ r(PAQB) \\equal{} r(AB)$." } { "Tag": [ "pigeonhole principle" ], "Problem": "Im new to the pigeonhole prinicple and i think i understand the theory but i cant solve tough problems...e.g.\r\n\r\n(1)If {a1,a2,a3...a1995} (numbers are subscripts) be a sequence of positive integers whose sum is 3989, show that there is a block of r sucessive ai's (i is subscripts here) whose sum is 95.\r\n\r\n(2)A Chessmaster who has 11 weeks to prepare for a tournament decides to play at least one game everyday, but no more than 12 games during a week.Show that there exists a succession of days during which the chessmaster played exactly 21 games.\r\n\r\nCan someone please give me advice on how to attack such problems.....also it would be nice if someone posts the solution to the to probs.......", "Solution_1": "i was going to post something one this or might have already. but yeah" } { "Tag": [ "inequalities proposed", "inequalities" ], "Problem": "A,b,c,d are real nunbers .a^2+b^2+c^2+d^2=1\r\nProve that ab+ac+ad+bc+bd+cd<=4abcd+5/4", "Solution_1": "[quote=\"stevenwang1992\"]A,b,c,d are real nunbers .a^2+b^2+c^2+d^2=1\nProve that $ ab\\plus{}ac\\plus{}ad\\plus{}bc\\plus{}bd\\plus{}cd\\leq4abcd\\plus{}\\frac{5}{4}$[/quote]\r\nLet $ a\\equal{}\\min\\{a,b,c,d\\}.$\r\n$ ab\\plus{}ac\\plus{}ad\\plus{}bc\\plus{}bd\\plus{}cd\\leq4abcd\\plus{}\\frac{5}{4}\\Leftrightarrow$\r\n$ 5(a^2\\plus{}b^2\\plus{}c^2\\plus{}d^2)^2\\plus{}16abcd\\geq4(ab\\plus{}ac\\plus{}ad\\plus{}bc\\plus{}bd\\plus{}cd)(a^2\\plus{}b^2\\plus{}c^2\\plus{}d^2),$ \r\nwhich is easy to check after substitution $ b\\equal{}a\\plus{}x,$ $ c\\equal{}a\\plus{}y$ and $ d\\equal{}a\\plus{}z.$ :wink:" } { "Tag": [ "inequalities", "pigeonhole principle" ], "Problem": "Taken a point A in a regular polygon A1...An. Prove that there exists at least one angle A_i O A_j verifying the inequalities:\r\n\r\n pi(1-1/n) < A_i O A_j < pi\r\n[/hide]", "Solution_1": "it is pigeonhole on the n wells $(\\frac{k}{n}, \\frac{k+1}{n})$ for $1\\leq k\\leq n$" } { "Tag": [ "number theory open", "number theory" ], "Problem": "S={1,2,...,3^{2}-1}\r\n3^{2} | 1+2+...+3^{2}-1\r\n3^{1} | 1*2+2*3+...+(3^{2}-3)*(3^{2}-1) (just sum of 2-element combination of S)\r\n \r\nIf change S={1,2,...,3^{2}-1} to S={1,2,...,3^{3}-1}\r\n3^{3} | 1+2+...+3^{3}-1\r\n3^{2} | 1*2+2*3+...+(3^{3}-3)*(3^{3}-1) (just sum of 2-element combination of S)\r\n3^{1} | 1*2*3+2*3*4+...+(3^{3}-4)*(3^{3}-3)*(3^{3}-1) (just sum of 3-element combination of S)\r\n\r\nand so on\r\n\r\nIn the set S, changing 3 will not make diff situation\r\n\r\nFrom some iteration, above must be true, but why?\r\nplease let me know this simple thing...", "Solution_1": "$1+2+3+...+n=\\sum k$\r\n$1\\cdot 2+2\\cdot 3+...+n\\cdot (n+1)=\\sum k(k+1)=\\sum k^{2}+\\sum k$\r\n$1\\cdot 2\\cdot 3+2\\cdot 3\\cdot 4+...+n\\cdot (n+1) \\cdot (n+2)=\\sum k(k+1)(k+2)=\\sum k^{3}+3\\sum k^{2}+2\\sum k$\r\nAll the sums from the $RHS$ are known, and divisibility should give easy.", "Solution_2": "hmm\r\nsorry, my representation is somewhat wrong.\r\n\r\nexactly writting is following\r\n3^{2} | 1*2+1*3+...+2*3+...+(3^{3}-3)*(3^{3}-1) (just sum of 2-element combination of S) \r\n\r\nactually problem can be written like this...(hmm above rep was somewhat stupid :blush: )\r\n3^{3} | (x+1)(x+2)...(x+(3^{3}-1))'s 2th-coeff\r\n3^{2} | (x+1)(x+2)...(x+(3^{3}-1))'s 3th-coeff\r\n3^{1} | (x+1)(x+2)...(x+(3^{3}-1))'s 4th-coeff" } { "Tag": [ "geometry" ], "Problem": "Given that the largest triangle is equilateral and all the smaller triangles are formed by connecting the midpoints of the sides of the larger triangle that contains, them, how many equilateral triangles are there?\n[asy]import olympiad; import geometry_dev; import graph; size(100); defaultpen(linewidth(0.8));\n void drawEqTriangle(real side, pair blCorner, real degrees = 0){\n pair c1 = blCorner;\n pair c2 = c1 + side*dir(degrees);\n pair c3 = c2 + side*dir(120 + degrees);\n draw(c1--c2--c3--cycle);\n }\n drawEqTriangle(1,origin); drawEqTriangle(1,(1,0)); drawEqTriangle(1,(0.5,sqrt(3)/2));\n for(int i = 1; i <=3; ++i){\n for(int j = 0; j <i; ++j){\n drawEqTriangle(1/4, (1 - i/8 + 1/4*j, sqrt(3)/8*i));\n }\n }[/asy]", "Solution_1": "From smallest triangles to largest: 16+7+3+4+1=31" } { "Tag": [ "function", "real analysis", "real analysis unsolved" ], "Problem": "Show that if $ f: R \\rightarrow R$ is a monotone increasing function, then the set where $ f$ is discontinuous has measure zero.", "Solution_1": "The set where f is dicontinuous is countable, because they are all jump discontinuities. Uncountably many jump discontinuities of positive size on some interval $ (a,b)$ would make $ f$ increase more than $ f(b) \\minus{} f(a)$ on the interval", "Solution_2": "in fact, I saw what you said, but I don't know how to prove that [i]uncountably many jump discontinuities of positive size on some interval $ (a,b)$ would make increase $ f$ more than on the interval $ (f(a),f(b))$.[/i]", "Solution_3": "I was just loosely saying that an \"uncountable sum\" of positive numbers is infinite. Index all the discontinuities in $ (a,b)$ as $ x_j$ where $ j$ runs through some index, and let the corresponding jump size at $ x_j$ be $ K_j$. Then the set of $ x_j$ such that $ K_j \\geq 1/n$ is finite. The set of all $ x_j$ is the countable union of the former sets as $ n$ runs through the positive integers. The countable union of finite sets is countable, so the number of $ x_j$ is countable.", "Solution_4": "[quote=\"Kalle\"]I was just loosely saying that an \"uncountable sum\" of positive numbers is infinite. Index all the discontinuities in $ (a,b)$ as $ x_j$ where $ j$ runs through some index, and let the corresponding jump size at $ x_j$ be $ K_j$. Then the set of $ x_j$ such that $ K_j \\geq 1/n$ is finite. The set of all $ x_j$ is the countable union of the former sets as $ n$ runs through the positive integers. The countable union of finite sets is countable, so the number of $ x_j$ is countable.[/quote]\r\n\r\nThank you Kalle !", "Solution_5": "[quote=\"Kalle\"]I was just loosely saying that an \"uncountable sum\" of positive numbers is infinite. Index all the discontinuities in $ (a,b)$ as $ x_j$ where $ j$ runs through some index, and let the corresponding jump size at $ x_j$ be $ K_j$. Then the set of $ x_j$ such that $ K_j \\geq 1/n$ is finite. The set of all $ x_j$ is the countable union of the former sets as $ n$ runs through the positive integers. The countable union of finite sets is countable, so the number of $ x_j$ is countable.[/quote]\r\n\r\nwhy is the set of $ x_j$ s.t. $ K_j \\geq \\frac{1}{n}$ finite?" } { "Tag": [], "Problem": "For natural numbers $m, n$, set $a = (n+1)^{m}-n$ and $b = (n+1)^{m+3}-n$.\r\n\r\n(a) Prove that $a$ and $b$ are coprime if $3 \\nmid m$.\r\n\r\n(b) Find all numbers $m, n$ for which $a$ and $b$ are not coprime.", "Solution_1": "This is solve at\r\nhttp://www.mathlinks.ro/Forum/viewtopic.php?t=116060" } { "Tag": [ "geometry", "3D geometry", "sphere", "Pythagorean Theorem" ], "Problem": "Given two balls. The larger ball of radius 9 leans against the wall of a room, and the smaller ball, or radius 4, rests upon the larger ball. Each ball is tangent to the wall, and the larger ball is tangent to the floor. What is the distance from the floor to the point of tangency of the smaller ball?\r\n\r\nThe solution given is 21, but I'm getting 22.", "Solution_1": "The solution uses the picture.\r\n\r\n[hide=\"Agree with the book\"]By the Pythagorean Theorem, $ GE = 12$. $ 9 = DA = EB$. Hence, the length is $ 12+9 = 21$[/hide]", "Solution_2": "the correct answer is $ 21$.\r\n\r\n[hide=\"solution\"]\n[asy]size(200); draw((0,0)--(100,0)); draw((0,0)--(0,100)); pair A,B,C,D,R,F,M,P,Q,O; O=origin; A=(0,27); B=(27,0); C=(54,27); D=(0,63); R=(24,63); F=(12,51); draw(circumcircle(A,B,C)); draw(circumcircle(D,R,F)); label(\"D\",D,W); label(\"A\",A,W); P=(27,27); Q=(12,63); label(\"P\",P,E); label(\"Q\",Q,E); dot(P); dot(Q); draw(P--Q); M=(12,27); label(\"M\",M,S); dot(M); draw(A--P); draw(Q--M); label(\"O\",O,W);[/asy]\nfrom the diagram, we get using pythagorean theorem, $ PQ=9+4=13=\\sqrt{QM^{2}+(9-4)^{2}}$ thus $ QM=AD=12$.\nso $ OD=AD+AO=12+9=21$.\n[/hide]\r\n\r\nEDIT: sorry vishalarul, didn't see your post there.", "Solution_3": "Thanks for your help, I see my error! :lol:" } { "Tag": [ "trigonometry", "geometry", "function", "probability", "AMC" ], "Problem": "This is a stupid question and can be deleted at soon as there is one accurate response. How much time do you get for the AMC 10/12? I've heard 75 and I've heard 90.\r\n\r\nEdit: Perhaps I'll substantiate this post slightly by asking, what tips/strategies people have for maximizing productivity under limited and straining time conditions. Also, how can one reach the later level questions quickly (as they require more time than the earlier questions), without making silly careless errors?", "Solution_1": "AMC 10/12 has a 75 minute time limit. The old AHSME had a 90 minute time limit (AHSME turned into AMC).\r\n\r\nTips: \r\n\r\nDon't waste too much time on a problem. If, for example, there is a trig problem but you just learned trig last week in your precalc class, skip it (unless you are confident you can do it in a timely fashion). \r\n\r\nTry to stay organized. You don't want to be unable to correctly read what you wrote. It also helps to have organized work in case you have the chance to go back and check your work.\r\n\r\nThere aren't really any tips I can give for you to get through the 'easier' problems quicker. That mostly comes with practice (the more times you've seen a certain type of problem, the faster you'll recognize how to approach, and so forth).", "Solution_2": "Why should you skip a problem that you just learned how to do? That seems like it would be one of the problems you would want to make sure you solve.", "Solution_3": "He means skip it, since you were just recently introduced to the concept, so that you don't spend a disproportionate amount of time on it. Obviously, if you've finished all the problems that you can do easily, go back to that one and try your hand at it, but don't go after it when you get to it (unless it's at the end of the test, obviously :P). As joml said, if you're sure you can do it quickly, go for it, but make sure you assess your capabilities in that subject realistically.\r\n\r\nWow, I just used a whole lot of big words. :what?:", "Solution_4": "More tips:\r\n\r\n1. Always check as you go. Of course it's nice to check back at the end, especially if you've organized your work well, but often you don't have time anyway. Also, you might have forgotten some special cases that you noticed the first time. Being meticulous the first time is usually the most effective method.\r\n\r\n2. Don't be too hasty. Actually I'm not really sure if this helps, but a lot of times I dive into a problem because I immediately see a way, when in fact there's an easier way that I could have used to save up to a minute or so if I had just spent a few seconds longer before diving in.", "Solution_5": "After 3 minutes and you've gotten no where on a problem, it's not the end of the world, just skip it and come back to it after doing like 2 or 3 other problems.", "Solution_6": "i would do all the problems that i am good at (geometry, nt, functions,etc) and skip all but the easiest probability (probability and combinatorics make me mad :mad: ), then if i have time, go back to the problems i am bad at, also most of the early amc problems can be done with 2 sentences of explaination if you do them correctly, so don't attack a tedious solution, find a better one, or move on, if you find the best solution, very little work is needed, and don't hesitate to use lots of paper, writing big helps you check better", "Solution_7": "I'm certainly no expert but if you look at a problem and it looks hard to you dont waste time, do the easy problems first, and then investigate those problems..", "Solution_8": "I think the key is to focus correctly.\r\n\r\nFocus on reading every word of the problem, and taking in the meaning.\r\nThen just get everything clear in your mind exactly what you need to do.\r\nOrganize your work and write out as much as you can (without going too slowly). Continue to keep your mind clear as to what you're doing and what you need to do. Especially, don't go through sloppy calculations or thoughts in your head; write things down.\r\nOnce you have the answer just write it in and go on to the next problem.\r\nStill remember not to take too much time on each problem.\r\n\r\nOne example is that in a math competition, I had a factoring/geometry problem. I got three solutions, 0, 8, and 60. I quickly thought through in my head that 0 didn't work, and didn't bother to check 8 and 60, thinking they worked because they were positive. As it turned out, 8 didn't work. Only 60 did.\r\n\r\nAlthough that particular problem might not apply to AMC 12 directly, the idea behind it does. Rather than quickly thinking through in my head, I should have paid attention to each answer individually, doing the full process of checking each answer.\r\nAnyway, checking the answers should have only taken a few seconds apiece. One can still focus in this way and do a problem quickly." } { "Tag": [], "Problem": "Joyce wants to share six identical pieces of licorice equally among four people. What is the mnimum number of pieces that must be cut?", "Solution_1": "2\r\n :P :P", "Solution_2": "2 is correct, but if you don't see why, I will explain.\r\n\r\nSharing 6 pieces amoung 4 people means each person gets $\\frac64=1+\\frac12$. Give each person a stick, and you have 2 sticks remaining. Now, cut them in half so there are 4 halfs to give to the people. You only have to cut 2 of the sticks." } { "Tag": [ "number theory proposed", "number theory" ], "Problem": "Find all $ x,y\\in Z^\\plus{}$ such that $ 3^x\\equal{}2^x.y\\plus{}1$\r\nIf post before,please give links :)", "Solution_1": "Let $ x = m.2^k$, then\r\n${ 3^x - 1 = 3^{m.2^k} - 1 = (3^{2^k} - 1)(3^{2k.(m - 1}} + . . . + 1)$.\r\n${ (3^{2k.(m - 1}} + . . . + 1)$ is sum of $ m$ integers odd, so it's odd.\r\n$ 3^{2^k} - 1 = (3 - 1)(3 + 1)(3^2 + 1)(3^{2^2} + 1) . . . (3^{2^{k - 1}} + 1) \\parallel{} 2^{k + 1}$.\r\nSo $ m.2^k\\leq k + 1$.\r\n$ \\rightarrow m = 1,k = 0,1$.\r\n$ (x,y) = (1,1);(2,2)$.", "Solution_2": "Also, refer to the exponent lifting lemma" } { "Tag": [ "algebra proposed", "algebra" ], "Problem": "Let $ m, n \\in N$ and $ \\frac{m^{n}-1}{n-1}+\\frac{n^{m}-1}{m-1}\\in Z$\r\nshow that $ \\frac{m^{n}-1}{n-1}\\in Z$ and $ \\frac{n^{m}-1}{m-1}\\in Z$", "Solution_1": "$ n\\neq{1}$and$ m\\neq{1}$\r\nSuppose by absurd $ \\frac{m^{n}-1}{n-1}\\notin{Z}$ and $ \\frac{n^{m}-1}{m-1}\\notin{Z}$ then \r\n$ (m-1)\\frac{(m^{n}-1)}{n-1}\\in{Z}$ and $ (n-1)\\frac{n^{m}-1}{m-1}\\in{Z}$\r\n$ \\Leftrightarrow \\frac{m-1}{n-1}\\in{Z}$ and $ \\frac{n-1}{m-1}\\in{Z}\\Leftrightarrow{m=n}$absurd" } { "Tag": [ "trigonometry" ], "Problem": "If $ \\sin x\\plus{}a \\cos x\\equal{}b$, express $ |a \\sin x \\minus{}\\cos x|$ in terms of $ a,b$.", "Solution_1": "Let's calculate $ (\\sin x \\plus{} a \\cos x)^2\\plus{}|a \\sin x \\minus{} \\cos x|^2.$" } { "Tag": [ "superior algebra", "superior algebra unsolved" ], "Problem": "Show that $C$ has infinitely many field automorphisms. What is the cardinality of the set of these automorphisms?", "Solution_1": "There are as many as possible: $2^{2^{\\aleph_{0}}}$. This is because any permutation of a transcendence basis of $\\mathbb C$ over $\\mathbb Q$ (and all such bases have cardinality $2^{\\aleph_{0}}$) extends to an automorphism of $\\mathbb C$.", "Solution_2": "Grobber can you calrify ?" } { "Tag": [ "calculus", "integration", "limit", "logarithms", "geometry", "rectangle", "inequalities" ], "Problem": "Let $ n$ be natural number. Find the limit value of ${ \\lim_{n\\to\\infty} \\frac{1}{n}(\\frac{1}{\\sqrt{2}}+\\frac{2}{\\sqrt{5}}}+\\cdots\\cdots +\\frac{n}{\\sqrt{n^2+1}}).$", "Solution_1": "hello kunny, we get\r\n$ \\lim_{n\\to \\infty}\\frac{1}{n}\\left(\\sum_{i\\equal{}1}^{n}\\frac{i}{\\sqrt{i^2\\plus{}1}}\\right)\\equal{}1$\r\nSonnhard.", "Solution_2": "That's correct. How did you get it? :)", "Solution_3": "[quote=\"Dr Sonnhard Graubner\"]hello kunny, we get\n$ \\lim_{n\\to \\infty}\\frac {1}{n}\\left(\\sum_{i \\equal{} 1}^{n}\\frac {i}{\\sqrt {i^2 \\plus{} 1}}\\right) \\equal{} 1$\nSonnhard.[/quote]\r\nYour answer is circular answer.\r\nYou are the same as answering by feeling.\r\n\r\n[b]My Solution[/b]\r\nLet $ f(n) \\equal{} \\frac{n}{\\sqrt{n^2\\plus{}1}}$.\r\nSo,\r\n$ f(1) \\plus{} \\int_1^n f(x)dx \\le \\sum_{i \\equal{} 1}^{n} f(i) \\le \\int_1^n f(x)dx \\plus{} f(n)$\r\n$ \\iff \\frac{1}{\\sqrt{2}} \\plus{} \\sqrt{n^2\\plus{}1} \\minus{} \\sqrt{2} \\le \\sum_{i \\equal{} 1}^{n} f(i) \\le \\sqrt{1\\plus{}n^2} \\minus{} \\sqrt{2} \\plus{} \\frac {n}{\\sqrt {n^2\\plus{}1}}$\r\n$ \\iff \\sqrt{n^2\\plus{}1} \\minus{} \\frac{1}{\\sqrt{2}} \\le \\sum_{i \\equal{} 1}^{n} f(i) \\le \\frac {n^2\\plus{}n\\plus{}1}{\\sqrt {n^2\\plus{}1}} \\minus{} \\sqrt{2}$\r\n$ \\iff \\sqrt{1\\plus{}\\frac{1}{n^2}} \\minus{} \\frac{1}{n\\sqrt{2}} \\le \\frac{1}{n}\\left(\\sum_{i \\equal{} 1}^{n} f(i)\\right) \\le \\frac {1\\plus{}\\frac{1}{n}\\plus{}\\frac{1}{n^2}}{\\sqrt {1\\plus{}\\frac{1}{n^2}}} \\minus{} \\frac{\\sqrt{2}}{n}$\r\n$ \\Rightarrow 1 \\le \\frac{1}{n}\\left(\\sum_{i \\equal{} 1}^{n} f(i)\\right) \\le 1 \\ \\ \\ \\ \\ (n \\to \\infty)$\r\n\r\nThus, $ \\lim_{n \\to \\infty} \\frac{1}{n}\\left(\\sum_{i \\equal{} 1}^{n} f(i)\\right) \\equal{} \\boxed{1}$", "Solution_4": "We have:\r\n$ 1 \\geq \\frac{1}{n}\\sum_{i\\equal{}1}^{n}\\frac{i}{\\sqrt{i^2}} \\geq \\frac{1}{n}\\sum_{i\\equal{}1}^{n}\\frac{i}{\\sqrt{i^2 \\plus{} 1}} \\geq \\left(\\prod_{i\\equal{}1}^{n} \\frac{i}{\\sqrt{i^2 \\plus{} 1}}\\right)^{\\frac{1}{n}} \\geq \\left(\\prod_{i\\equal{}1}^{n} \\frac{i}{\\sqrt{\\left(i \\plus{} 1\\right)^2}}\\right)^{\\frac{1}{n}} \\equal{} \\left(\\prod_{i\\equal{}1}^{n} \\frac{i}{i \\plus{} 1}\\right)^{\\frac{1}{n}} \\equal{} \\left(1 \\plus{} n\\right)^{\\frac{1}{n}}$\r\n\r\nApplying the sandwich method we get the limit to be 1.", "Solution_5": "[quote=\"akech\"]We have:\n$ 1 \\geq \\frac {1}{n}\\sum_{i \\equal{} 1}^{n}\\frac {i}{\\sqrt {i^2}} \\geq \\frac {1}{n}\\sum_{i \\equal{} 1}^{n}\\frac {i}{\\sqrt {i^2 \\plus{} 1}} \\geq \\left(\\prod_{i \\equal{} 1}^{n} \\frac {i}{\\sqrt {i^2 \\plus{} 1}}\\right)^{\\frac {1}{n}} \\geq \\left(\\prod_{i \\equal{} 1}^{n} \\frac {i}{\\sqrt {\\left(i \\plus{} 1\\right)^2}}\\right)^{\\frac {1}{n}} \\equal{} \\left(\\prod_{i \\equal{} 1}^{n} \\frac {i}{i \\plus{} 1}\\right)^{\\frac {1}{n}} \\equal{} \\left(1 \\plus{} n\\right)^{\\frac {1}{n}}$\n\nApplying the sandwich method we get the limit to be 1.[/quote]\r\n\r\nyour solution and Kouichi Nakagawa's both are excellent. I do agree with Kouichi Nakagawa that Dr Sonnhard Graubner's answer is probably based on answering by feeling. \r\n\r\nI think instead of $ \\left(1 \\plus{} n\\right)^{\\frac {1}{n}}$\r\nwrite $ \\frac{1}{\\left(1 \\plus{} n\\right)^{\\frac {1}{n}}}$\r\n\r\nand $ \\lim_{n\\to\\infty}\\left(1 \\plus{} n\\right)^{\\frac {1}{n}}\\equal{}1$", "Solution_6": "[color=blue]\nMy solution as follows:\n\nAs $ \\ \\frac {r}{\\sqrt {(r\\plus{}1)^2}}<\\frac {r}{\\sqrt {r^2\\plus{}1}}<\\frac {r}{\\sqrt {r^2}}\\ \\forall \\ r \\equal{} 1,\\ 2,\\ 3,\\ \\dots$\n\n$ \\implies\\frac {r}{r\\plus{}1}<\\frac {r}{\\sqrt {r^2\\plus{}1}}<\\frac {r}{r}$\n\n$ \\implies1\\minus{}\\frac {1}{r\\plus{}1}<\\frac {r}{\\sqrt {r^2\\plus{}1}}<1$\n\n$ \\implies \\sum_{r \\equal{} 1}^{n}\\left(1\\minus{}\\frac {1}{r\\plus{}1}\\right)<\\sum_{r \\equal{} 1}^{n}\\frac {r}{\\sqrt {r^2\\plus{}1}}<\\sum_{r \\equal{} 1}^{n}1$\n\n$ \\implies n\\minus{}\\sum_{r \\equal{} 2}^{n\\plus{}1}\\frac {1}{r}<\\sum_{r \\equal{} 1}^{n}\\frac {r}{\\sqrt {r^2\\plus{}1}}< n$\n \nAs $ \\ \\int_{1}^{n\\plus{}1}\\frac{1}{x}\\ dx \\plus{} \\frac {1}{n\\plus{}1}\\ge \\sum_{r \\equal{} 2}^{n\\plus{}1}\\frac {1}{r}$\n\n$ \\implies n\\minus{}\\int_{1}^{n\\plus{}1}\\frac{1}{x}\\ dx \\minus{} \\frac {1}{n\\plus{}1}<\\sum_{r \\equal{} 1}^{n}\\frac {r}{\\sqrt {r^2\\plus{}1}}< n$\n\n$ \\implies 1\\minus{}\\frac{\\ln (n\\plus{}1)}{n} \\minus{} \\frac {1}{n(n\\plus{}1)}<\\frac{1}{n}\\sum_{r \\equal{} 1}^{n}\\frac {r}{\\sqrt {r^2\\plus{}1}}< 1$\n\nAs $ \\ \\lim_{n\\to\\infty}\\frac{\\ln (n\\plus{}1)}{n}\\equal{}0$ and $ \\ \\lim_{n\\to\\infty}\\frac{1}{n(n\\plus{}1)}\\equal{}0, $\n\nBy applying sandwich theorem we get,\n\n$ \\boxed {\\boxed {\\lim_{n\\to\\infty}\\frac{1}{n}\\sum_{r \\equal{} 1}^{n}\\frac {r}{\\sqrt {r^2\\plus{}1}}\\equal{}1}}$\n\n[/color]", "Solution_7": "Another approach: let $ a_n \\equal{} \\frac{n}{\\sqrt{n^2 \\plus{} 1}}$. As $ n$ approaches infinity, $ a_n$ approaches 1. So by Cesaro's theorem, $ \\frac{1}{n} \\sum_{i\\equal{}1}^n a_i$ approaches 1 too.", "Solution_8": "[quote=\"Ravi B\"]Another approach: let $ a_n \\equal{} \\frac {n}{\\sqrt {n^2 \\plus{} 1}}$. As $ n$ approaches infinity, $ a_n$ approaches 1. So by Cesaro's theorem, $ \\frac {1}{n} \\sum_{i \\equal{} 1}^n a_i$ approaches 1 too.[/quote]\r\n\r\nCan u explain Cesaro's theorem in detail", "Solution_9": "If you have a sequence of numbers $ a_n$ that converges to $ L$, then the sequence of partial averages $ \\frac{1}{n} \\sum_{i\\equal{}1}^n a_i$ also converges to $ L$.", "Solution_10": "Note that the sum is equivalent to the area of the rectangles above the curve $ y = \\frac {x}{\\sqrt {x^2 + 1}}$ on $ [0,n]$, where each rectangle has width $ 1$. Note also that each term of the sum is less than $ 1$, so these rectangles all lie below $ y=1$. Hence, we get the inequality\r\n\r\n$ \\int_0^n \\! \\frac {x}{\\sqrt {x^2 + 1}} \\, dx \\le \\sum_{i = 1}^{n}\\frac {i}{\\sqrt {i^{2} + 1}} \\le \\int_0^n \\! dx$\r\n\r\nand subsequently,\r\n\r\n$ \\frac {\\sqrt {n^2 + 1} - 1}{n} \\le \\frac {1}{n} \\sum_{i = 1}^{n}\\frac {i}{\\sqrt {i^{2} + 1}} \\le 1$\r\n\r\nSince $ \\lim_{n\\to\\infty}\\!\\frac {\\sqrt {n^2 + 1} - 1}{n}\\, = 1$ and $ \\lim_{n\\to\\infty}1 = 1$, by the Squeeze Theorem, we get\r\n\r\n$ {\\lim_{n\\to\\infty}\\frac {1}{n}\\sum_{r = 1}^{n}\\frac {r}{\\sqrt {r^{2} + 1}} = 1}$", "Solution_11": "Thank you for your replies! :lol: \r\n\r\nKouichi Nalagawa and Elixir Solution are a standard for Japanese high school students.\r\n\r\nMy solution is almost same as Elixir 's, but I write $ \\int_0^n f(x)\\ dx < \\sum_{k \\equal{} 1}^n \\frac {k}{\\sqrt {k^2 \\plus{} 1}} < \\boxed{\\int_1^{n \\plus{} 1}}\\ f(x)\\ dx$\r\n\r\nCesaro's theorem by Ravi B is very useful, but it usually appeares in University mathematics in Japan.", "Solution_12": "I am rather pleased to see that Japanese high school* students are taught some interesting theory and asked questions that actually require some thought - a \"hard\" question in the UK would be probably be off a STEP paper;\r\n\r\nhttp://www.admissionstests.cambridgeassessment.org.uk/adt/step\r\n\r\nI just remmbered this is a medical entrance exam - in the UK a medical student would also hopelessly struggle as mathematics is not a requirement for entrance into medical schools." } { "Tag": [ "inequalities", "linear algebra", "matrix", "linear algebra unsolved" ], "Problem": "Let $A_i \\in {\\mathbb C}^{n \\times n},i=1,2,...,n$. If there is at least a $k \\in \\{1,2,...,n\\}$ such that $A_k = O_n$ then show that the following inequality holds: $\\sum_{i=1}^{n} rank(A_i) \\leq n(n-1)$.\r\n :ninja:", "Solution_1": ":huh:\r\nIs $O_n$ the zero matrix (then it's trivial), or is it an orthogonal matrix (then it's false) ?", "Solution_2": "$O_n$ is zero matrix ;)" } { "Tag": [ "probability", "AMC", "AIME", "counting", "distinguishability", "summer program", "Mathcamp" ], "Problem": "What do ppl mean when they say u need to know the balls-and-urns argument in order to solve many of the probability problems in AIME? Can someone tell me what that argument is. :D", "Solution_1": "If you have $b$ indistinguishable balls and $u$ urns, the number of ways you can put $b$ balls in $u$ urns is $\\dbinom{b+u-1}{u-1}$. If there is the additional restriction that there must be at least one ball in each urn, the number of ways is $\\dbinom{b-1}{u-1}$.", "Solution_2": "What if u have different types of balls? Then does the same argument still hold or is it somewhat different?", "Solution_3": "Oh, I forgot, sorry, they have to be indistinguishable balls. Distinguishable balls have different methods.", "Solution_4": "A specific question: If you have $6$ indistinguishable balls that you need to pout into $4$ distinguishable urns, how many ways can you put the balls into the urns?\r\n\r\nAnswer:\r\nWe make a bijection. Instead of putting balls into urns, let us imagine that the balls are arranged in a straight line and that we are putting dividers between the balls. There will be 3 dividers.\r\n\r\nFor example, oo|o||ooo means put two balls in the first urn, one ball in the second urn, zero balls in the third urn, and 3 balls into the fourth urn.\r\n\r\nEach permutation of oooooo||| represents a different way to put the balls into the urns, and each way the balls can be put into urns can be represented as a permutation of oooooo|||. However, it is easy to count the number of permutations of oooooo|||, which is $\\binom{9}{3}$.\r\n\r\n\r\nIn general with b balls and u urns, you get what davidyko has.", "Solution_5": "woah.. i was about to ask the same question... So where did everyone learn the balls and urns argument from? Was it in Aops?1 or 2?", "Solution_6": "I learned in MOP, I think it's in AoPS, and the teacher in my school's honors math class even talked about it! This concept is amazingly useful. For example:\r\n\r\nHow many 4-digit numbers have the property that no digit is less than the digit to its left?", "Solution_7": "[quote=\"turtlecloud\"]woah.. i was about to ask the same question... So where did everyone learn the balls and urns argument from? Was it in Aops?1 or 2?[/quote]\r\nIt's in intro to C&P and AoPS vol. 2.", "Solution_8": "[quote=\"randomdragoon\"]I learned in MOP, I think it's in AoPS, and the teacher in my school's honors math class even talked about it! This concept is amazingly useful. For example:\n\nHow many 4-digit numbers have the property that no digit is less than the digit to its left?[/quote]\r\n\r\n[hide=\"hint\"]\n\nI interpret it as, \"How many way are there to pick 4 digits from 9 if we allow repetition?\"\n\n[/hide]", "Solution_9": "Where is it found in aops 2? i looked through the book but i still cant find it.", "Solution_10": "[quote=\"Kamil Witek\"]Where is it found in aops 2? i looked through the book but i still cant find it.[/quote]\r\nEh, in the counting in the twighlight zone section I think.\r\n\r\nNot really sure.", "Solution_11": "Meh, I don't think it is in there. Oh well, I learned it from these forums. :P", "Solution_12": "[quote=\"davidyko\"]Meh, I don't think it is in there. Oh well, I learned it from these forums. :P[/quote]\r\nI'm almost positive it's in there.\r\nIt was a problem about a person buying 8 german dogs with three breeds to choose from.", "Solution_13": "Never mind, you're right. :roll: \r\nChapter 17.2, Clever Correspondences, the first problem about 8 dogs and three breeds, just in case anyone wanted to know.", "Solution_14": "What about indistinguishable urns? How would you do that?", "Solution_15": "[quote=\"123s\"]What about indistinguishable urns? How would you do that?[/quote]\r\n\r\nI believe, correct me if I'm wrong, that that's counting the number of partitions, which has an extremely extremely complicated formula (Bell numbers) that I think was on the back of some mathcamp shirts and went all the way across.", "Solution_16": "For that you could be 1337 and use generating functions, then plug into mathematica...\r\n\r\ncoefficient of $x^{n}$ in $(1+x+x^{2}+\\cdots)^{\\infty}$ is the number of partitions of $n$.", "Solution_17": "[quote=\"13375P34K43V312\"]For that you could be 1337 and use generating functions, then plug into mathematica...\n\ncoefficient of $x^{n}$ in $(1+x+x^{2}+\\cdots)^{\\infty}$ is the number of partitions of $n$.[/quote]\r\n\r\nHeh Heh no.\r\n\r\nThat expression you gave is always $\\infty$. (look at the expansion for the coefficient of x).\r\n\r\nThe correct generating function would be\r\n$(1+x+x^{2}+\\cdots)(1+x^{2}+x^{4}+\\cdots)(1+x^{3}+x^{6}+\\cdots)(\\cdots)$\r\n\r\nwhich is equal to\r\n\r\n$\\prod_{n\\in\\mathbb{N}}\\frac{1}{1-x^{n}}$", "Solution_18": "[quote=\"randomdragoon\"]I learned in MOP, I think it's in AoPS, and the teacher in my school's honors math class even talked about it! This concept is amazingly useful. For example:\n\nHow many 4-digit numbers have the property that no digit is less than the digit to its left?[/quote]\r\n\r\nis the answer 12 choose 4? (which is 495)", "Solution_19": "It's also called stars and bars.", "Solution_20": "[quote=\"machack\"][quote=\"randomdragoon\"]I learned in MOP, I think it's in AoPS, and the teacher in my school's honors math class even talked about it! This concept is amazingly useful. For example:\n\nHow many 4-digit numbers have the property that no digit is less than the digit to its left?[/quote]\n\nis the answer 12 choose 4? (which is 495)[/quote]\r\n12 choose 3 is the answer. Remember that if you have n urns you will only have n-1 divider bars!\r\n\r\n\r\nEDIT: This post is wrong, see below", "Solution_21": "I'm feeling incredibly stupid after posting about balls-and-urns...\r\nWill someone enlighten me why it's [b]12[/b] choose 3?", "Solution_22": "I get 12C4 as well...isn't it analogous to the situation of having to give out 4 apples to 9 kids (we label them from 1 to 9) with no restrictions?", "Solution_23": "In that case, it would be 12C3.\r\nBut why 12? Why 9 kids? I think I'm missing something big here.", "Solution_24": "Perhaps what you are missing is that the first digit can't be zero. Since no digit is less than the one on its left, this means that no digit can be a zero. :roll:", "Solution_25": "i really think its 12 c 4 because it isn't exactly the concept of ball and urn (the numbers don't have to add up to a ceertain sum. and the \"dividing\" concept don't work in this example.\r\nAnyone agree with me?", "Solution_26": "[quote=\"randomdragoon\"]\n\nFor example, oo|o||ooo means put two balls in the first urn, one ball in the second urn, zero balls in the third urn, and 3 balls into the fourth urn.\n\nEach permutation of oooooo||| represents a different way to put the balls into the urns, and each way the balls can be put into urns can be represented as a permutation of oooooo|||. However, it is easy to count the number of permutations of oooooo|||, which is $\\binom{9}{3}$.\n[/quote]\r\n\r\ni thought that was the method behind hockey-stick identity? :huh:", "Solution_27": "Never mind, got it. Yep, it's $\\dbinom{12}{3}$.\r\nA good night's sleep can work wonders.", "Solution_28": "I don't see the conection between the 4-digit number problem and the balls-in-urns formula", "Solution_29": "Sorry it is $\\binom{12}{4}$. I needed a night's sleep as well. :oops: \r\n\r\n\r\nExplanation: There aren't 4 urns, there are 9 urns, and 4 balls." } { "Tag": [ "calculus", "integration", "algebra", "function", "domain", "superior algebra", "superior algebra unsolved" ], "Problem": "If $R$ is an integral domain which has only finitely many ideals, then show that $R$ is a field.\r\n\r\n Darij", "Solution_1": "Solution without thinking: finitely many ideals => artinian => zero-dimensional, and zero-dimensional + domain => field. owk", "Solution_2": "Is it the Krull dimension you are talking about? In fact, I am not that far in Atiyah-Macdonald...\r\n\r\n[hide=\"Solution without looking up\"][i]Solution.[/i] Let $ a$ be a nonzero element of $ R$. Denote the (finitely many) ideals of $ R$ (including both $ R$ itself and $ 0$) by $ I_{1}$, $ I_{2}$, ..., $ I_{n}$ such that $ I_{1} \\equal{} R$ and such that $ I_{i}\\neq I_{j}$ for any two integers $ i$ and $ j$ from the set $ \\left\\{1,2,...,n\\right\\}$ satisfying $ i\\neq j$. Then, $ aI_{1}$, $ aI_{2}$, ..., $ aI_{n}$ are also ideals of $ R$.\n\nAssume that $ aI_{i} \\equal{} aI_{j}$ holds for two integers $ i$ and $ j$ from the set $ \\left\\{1,2,...,n\\right\\}$. Then, for every $ s_{i}\\in I_{i}$, there exists some $ s_{j}\\in I_{j}$ satisfying $ as_{i} \\equal{} as_{j}$ (because $ as_{i}\\in aI_{i} \\equal{} aI_{j}$); this becomes $ a\\left(s_{i} \\minus{} s_{j}\\right) \\equal{} 0$, what yields $ s_{i} \\minus{} s_{j} \\equal{} 0$ because $ a$ is nonzero (and $ R$ is an integral domain). Hence, $ s_{i} \\equal{} s_{j}$, so that $ s_{i}\\in I_{j}$. Since this holds for every $ s_{i}\\in I_{i}$, we thus get $ I_{i}\\subseteq I_{j}$. Similarly, $ I_{j}\\subseteq I_{i}$. Thus, $ I_{i} \\equal{} I_{j}$, what yields $ i \\equal{} j$ (since we else would have $ I_{i}\\neq I_{j}$).\n\nSo we see that if $ i$ and $ j$ are two integers from the set $ \\left\\{1,2,...,n\\right\\}$, then $ aI_{i} \\equal{} aI_{j}$ holds only if $ i \\equal{} j$. On the other hand, $ aI_{1}$, $ aI_{2}$, ..., $ aI_{n}$ are ideals of $ R$ and thus elements of the set $ \\left\\{I_{1},I_{2},...,I_{n}\\right\\}$. Thus, $ aI_{1}$, $ aI_{2}$, ..., $ aI_{n}$ are $ n$ pairwise distinct elements of the set $ \\left\\{I_{1},I_{2},...,I_{n}\\right\\}$. This yields that the set $ \\left\\{aI_{1},aI_{2},...,aI_{n}\\right\\}$ is a permutation of the set $ \\left\\{I_{1},I_{2},...,I_{n}\\right\\}$. Thus, in particular, there exists some $ k\\in\\left\\{1,2,...,n\\right\\}$ such that $ I_{1} \\equal{} aI_{k}$. Since $ 1\\in R \\equal{} I_{1}$, this yields $ 1\\in aI_{k}$, so that there exists some $ b\\in I_{k}$ satisfying $ 1 \\equal{} ab$. Hence, $ a$ has a multiplicative inverse in $ R$. Since $ a$ can be any arbitrary nonzero element of $ R$, we have thus shown that every nonzero element of $ R$ has a multiplicative inverse, and thus $ R$ is a field, qed.[/hide]\r\n\r\n darij", "Solution_3": "just consider $(x) \\supseteq (x^{2}) \\supseteq ...$, and you're done. this also shows domain + artinian = field.", "Solution_4": "[quote=\"darij grinberg\"]Is it the Krull dimension you are talking about? In fact, I am not that far in Atiyah Macdonald...\n[/quote]\r\n\r\nYes, but zero-dimensional just means that all prime ideals are maximal. owk", "Solution_5": "[quote=\"darij grinberg\"]If $ R$ is an integral domain which has only finitely many ideals, then show that $ R$ is a field. [/quote]\r\n\r\nThe nonzero principal ideals form a group, being a finite cancellative monoid." } { "Tag": [ "number theory" ], "Problem": "Can anyone suggest me some good books for preparing for the RMO . i am currently in 12th and i need some advice on this", "Solution_1": "The book Excursion in Mathematics cover all the topics that could ever come in RMO.For the depth on those topics you can try books on a particular topic like \r\nBurton's Number Theory,Balakrishnan's Combinatorics and Sharygin's Geometry(Though out of print).If you are looking for a problem book then arthur engel is a unique collection of problems.But most problems are very difficult.There are other olympiad problem books like titu's Mathematical Olympiad Challenges but their coverage is incomplete.", "Solution_2": "Is Challenge and Thrill of Pre-College Mathematics by V. Krishnamurthy, C. R. Pranesachar, K. N. Ranganathan and B. J. Venkatachala(New Age International Publishers) good?Well;I find the questions pretty tough!", "Solution_3": "i had to bear much difficulties before ensuring myself a copy of'challenge & thrills.......'\r\nthe book is a nice one............", "Solution_4": "I want it to! Asking a friend in Delhi to mail it :P", "Solution_5": "u may also contact the publication office...", "Solution_6": "u may also contact the publication office...", "Solution_7": "My parents went to Delhi for 2 days, so they got it for me :)", "Solution_8": "I bought my copy at the Kolkata Book fair", "Solution_9": "I miss Kol :(", "Solution_10": "Are u from kol? :o", "Solution_11": "lol\r\nu guys shud be in pune , people complete CAT in their 8th :P", "Solution_12": "I am happier in kgp, :) \r\nover there,I would have suffered from a worse inferiority complex! :(", "Solution_13": "[quote=\"Scary math\"]Are u from kol? :o[/quote]\r\nLived there from 4-10 years of age." } { "Tag": [ "abstract algebra", "calculus", "integration", "group theory", "Ring Theory", "superior algebra", "superior algebra unsolved" ], "Problem": "Assume R is commutative. Prove that the set of prime ideals in R has a minimal element with respect to inclusion. (by Zorn's lemma)", "Solution_1": "the cut of a chain in the set of prime ideals is a obviously a lower bound for it, so zorn's lemma (applied to the \"dual\" inclusion relation) yields the result.", "Solution_2": "\"the cut of a chain in the set of prime ideals is a obviously a lower bound\"\r\n\r\n...can you explain a little bit more? I have no idea on how to construct a chain in it.....", "Solution_3": "zorn's lemma advises you to take an arbitrary chain (totally ordered subset) - it must not be constructed - and find a lower bound ..", "Solution_4": "related to this question..\r\nme and friend were wondering: is the proposition still true if you ask for [i]non-zero[/i] minimal prime ideals?\r\nof course, forget the case of $R$ field, or maybe (just in order to let the statement have a sense) consider also the whole field as a prime ideal.\r\nor maybe reformulate the statement for ideals which are non-zero primes or the whole field :P\r\n\r\nthe question reduces to this: does it exist a (non-noetherian, integral) ring with an infinite descending chain of non-zero prime ideals with trivial intersection?\r\nactually, i have some problem in finding a ring with an infinite descending chain of prime ideals :blush: :maybe: ...", "Solution_5": "Look what happened in [url=http://www.artofproblemsolving.com/Forum/viewtopic.php?t=125503]this[/url] thread: the algebraic question was transferred, via valuation rings, to a question on totally ordered abelian groups, and then the problem consisted simply of finding a totally ordered abelian group with certain properties. We can do the same thing here.\r\n\r\nLet $\\nu: K^{*}\\to\\Gamma$ be, just as in the other thread, a valuation of the field $K$ onto the totally ordered abelian group $\\Gamma$, and let $A$ be the valuation ring corresponding to $\\nu$, i.e. $A=\\{a\\in K\\ |\\ \\nu(a)\\ge 0\\}$. A subgroup $\\Delta\\le\\Gamma$ is called [i]isolated[/i] if whenever $0\\le\\beta\\le\\alpha$ and $\\alpha\\in\\Delta$, we also have $\\beta\\in\\Delta$. If $\\frak p$ is a prime ideal of $A$, then it can be shown that the set $\\nu(A\\setminus\\frak p)$ is the set of non-negative elements of an isolated subgroup of $\\Gamma$, and that this gives a bijection between prime ideals and isolated subgroups of $\\Gamma$. Now, the question of finding infinite descending chains of prime ideals in $A$ becomes, through this correspondence, equivalent to finding infinite ascending chains of isolated subgroups of $\\Gamma$.\r\n\r\nWe can choose $\\Gamma$ to be any totally ordered abelian group we want. Take it to be the set of infinite sequences $(\\alpha_{n})_{n\\in\\mathbb Z}$ (infinite in both directions) of integers such that $\\alpha_{n}$ is zero for small enough $n$, with componentwise addition. We order this set lexicographically, i.e. $(\\alpha_{n})_{n}<(\\beta_{n})_{n}$ if $\\alpha_{n}<\\beta_{n}$ for the smallest $n$ where the sequences differ. Now define $\\Delta_{n},\\ n\\le 0$ to be the set of elements in $\\Gamma$ whose components with index smaller than $n$ is zero. It is easy to check that $\\Delta_{n}$ for an infinite ascending chain of isolated subgroups of $\\Gamma$, so we\u2019re done. Moreover, this example can be modified to give descending chains of non-zero prime ideals of any cardinality we want." } { "Tag": [ "MATHCOUNTS" ], "Problem": "MATHCOUNTS 2004-2005 Warm-Up 13\r\n\r\nA two-pan balance scale comes with a collection of weights. Each weight weighs a whole number of grams. Weights can be put in either or both pans during a weighing. To ensure any whole number of grams up to 100 grams can be measured, what is the minimum number of weights needed in the collection?\r\n\r\n[hide=\"Answer\"]Here is how I did and wondering if there is a better way:\n\nWe start with 1 and 3 to cover number 1-4.\nTo get 5, we need 4+5=9, so 1, 3 and 9 can cover number 5-13.\nTo get 14, we need 13+14=27, so 1, 3, 9 and 27 can cover number 14-40.\nTo get 41, we need 40+41=81, so 1, 3, 9, 27 and 81 can cover number 41-121.\n\nSo, the answer is [b]5[/b].[/hide]", "Solution_1": "If you think of the famous base 2 numbers magic trick, you'll realize that you just need the powers of two, including 1. The answer is the number of powers of 2 less than 100, which are 1,2,4,8,16,32, and 64. That gives you the answer of 7. Also you could do log(2)64, which also equals 7.", "Solution_2": "[quote]\nIf you think of the famous base 2 numbers magic trick, you'll realize that you just need the powers of two, including 1.[/quote]\r\n\r\nBut that doesn't give you the [b]minimum[/b] number of weights, because you are allowed to put weights on both sides of the scale.\r\n\r\nYakko" } { "Tag": [ "logarithms", "function", "Euler", "complex analysis" ], "Problem": "[b]\nDetermine is this Conv. Or Div. and Is there any effect if $ z\\in \\mathbb {C}$ OR $ z\\in \\mathbb{R}$.\n\n$ \\prod\\limits_{n = 1}^\\infty {\\left( {1 - \\frac {z}{n}} \\right)} e^{\\frac {z}{n}}$\n\n\nI need details enough to understand why?\n\nMy thinking , it's not Con. , since no any factor of it's equal to 0 yes Or not ?\n\n[/b]", "Solution_1": "If one of the factors is $ 0$, it certainly converges to $ 0$. Otherwise, take logarithms and reduce the problem to that about convergence of a series (you need to know that $ \\log(1 \\minus{} w) \\equal{} \\minus{} w \\plus{} O(|w|^2)$ for small $ w$).\r\n\r\nBy the way, [b]entire posts in bold are slightly irritating[/b], so, please, use normal fonts switching to [b]bold[/b], [i]italic[/i], [color=red]color[/color], etc. only when you need to emphasize something :).\r\n\r\nAlso, please spell the words in the topic title according to English standards!", "Solution_2": "I'm sorry :blush: about Bold and my spelling . I'll use Standard English In titles and bold in something need to emphasize about it.\r\n\r\nThank u about Your Polite way. :) \r\n\r\nAnyway,\r\n\r\nCould you do more steps about my question please ? I think I still have warping about it.\r\n\r\n\r\nThank you.", "Solution_3": "Let's see. There are two possibilities for an infinite product $ \\prod_n a_n$ to converge.\r\n1) One of the factors is $ 0$. Then the entire product obviously converges to $ 0$.\r\n2) All $ a_n\\ne 0$, and $ \\sum_n \\log a_n$ converges. Then the limit of the partial products is just the value of the exponential function at the limit of the partial sums of the logarithms.\r\n\r\nSo, assume that none of the factors in your product is $ 0$. When $ n$ is much larger than $ |z|$, you can write \r\n\\[ \\log(1\\minus{}\\tfrac zn)\\plus{}\\frac zn\\equal{}\\minus{}\\frac zn\\plus{}O(n^{\\minus{}2})\\plus{}\\frac zn\\equal{}O(n^{\\minus{}2})\\]\r\nso the series of logarithms is dominated by the positive series $ \\sum_N \\frac 1{n^2}$, which converges. I hope that the rest is clear :)", "Solution_4": "That's clear dude.\r\n\r\nI'm So thankful.\r\n\r\n :D :wink:", "Solution_5": "[quote=\"fedja\"]\n1) One of the factors is $ 0$. Then the entire product obviously converges to $ 0$.\n[/quote]\r\nI'd just like to point out that typically the product is not considered to converge just because one of the factors is 0. This is because if you allow this for convergence, then you cannot say anything about the terms. As an example, you'd often like to know if the product converges to a continuous or differentiable functions, and in such discussion it becomes apparent that when factors are 0 you need to throw out the zero factors and look at the rest of the factors.\r\n\r\n(fedja knows this, others might not :wink: )", "Solution_6": "I'm going to go ahead and move this topic into the complex analysis forum. The standards for convergence in Kalle's post are those typically elaborated in a complex analysis course. And by those standards, this product converges as nicely as we could possibly ask for: uniformly on each compact subset of $ \\mathbb{C}.$ Hence, the function defined by this product is an entire function, and it has a simple zero at each positive integer.\r\n\r\nNow let me add a question: what does it converge to? Can we recognize and identify this entire function? Can we at least find some useful algebraic properties of this entire function?", "Solution_7": "Well, applying the logarithm to the function and doing some basic manipulation, we can get th expression:\r\n\r\n$ \\prod^\\infty_{n\\equal{}1}\\left(1\\minus{}\\frac{z}{n}\\right)e^{\\frac{z}{n}}\\equal{}\\frac{e^{z\\, \\gamma}}{\\Gamma(1\\minus{}z)}$\r\n\r\nwhere $ \\gamma$ is the Euler-Mascheroni constant." } { "Tag": [ "calculus", "calculus computations" ], "Problem": "Write an equation for the tangent line at (c,f(c)).\r\n\r\nf(x) =sqrt(x); c=4", "Solution_1": "Since I can flip open my calculus textbook and find many homework exercises (in a fairly early chapter) that look exactly like this, I'll have to ask: what have you done so far?", "Solution_2": "y - x/4 - 1 = 0", "Solution_3": "Yes, that's a correct answer." } { "Tag": [ "trigonometry", "inequalities", "geometry proposed", "geometry" ], "Problem": "Prove that\r\n\r\n$ a^4\\plus{}b^4\\plus{}c^4\\geq16s(s\\minus{}a)(s\\minus{}b)(s\\minus{}c)(\\cot A\\cot B \\plus{} \\cot B \\cot C \\plus{} \\cot C \\cot A)$", "Solution_1": "Same form like previous problems\r\nUse $ 4R\\equal{}\\frac{abc}{S}, 2R\\equal{}\\frac{a}{\\sin A}, \\cos A \\equal{}\\frac{b^{2}\\plus{}c^{2}\\minus{}a^{2}}{2bc}$\r\nThen it goes to obvious.", "Solution_2": "[quote=\"Heebeen, Yang\"]Same form like previous problems\nUse $ 4R \\equal{} \\frac {abc}{S}, 2R \\equal{} \\frac {a}{\\sin A}, \\cos A \\equal{} \\frac {b^{2} \\plus{} c^{2} \\minus{} a^{2}}{2bc}$\nThen it goes to obvious.[/quote]\r\n\r\n\r\nDear [color=red][b]Yang[/b][/color] can you describe better your solution? Thanks", "Solution_3": "If we use them,\r\n$ \\cot A\\equal{}\\frac{2R(b^{2}\\plus{}c^{2}\\minus{}a^{2})}{2abc}$and $ \\cot B, \\cot C$ defined similar.\r\nThen, above inequality is equivalent to \r\n$ a^{4}\\plus{}b^{4}\\plus{}c^{4} \\geq \\sum_{cyc}(b^{2}\\plus{}c^{2}\\minus{}a^{2})(c^{2}\\plus{}a^{2}\\minus{}b^{2})$\r\n<=>\r\n$ a^{4}\\plus{}b^{4}\\plus{}c^{4} \\geq a^{2}b^{2}\\plus{}b^{2}c^{2}\\plus{}c^{2}a^{2}$", "Solution_4": "$ 16S^2\\equal{}16s(s \\minus{} a)(s \\minus{} b)(s \\minus{} c) \\equal{} \\sum (a^2\\plus{}b^2\\minus{}c^2)(a^2\\plus{}c^2\\minus{}b^2)\\equal{} 2\\sum b^2c^2 \\minus{} \\sum a^4$ \r\n\r\nand $ \\sum\\cot B\\cot C \\equal{} 1$ . [b]Simply ![/b] The proposed inequality becomes $ \\sum a^4\\ge \\sum b^2c^2$ what is evidently.", "Solution_5": "Well.... That's right wow.\r\nI've forgottend that property.\r\nThank you for your advice ^^", "Solution_6": "[quote=\"Virgil Nicula\"]$ 16S^2 \\equal{} 16s(s \\minus{} a)(s \\minus{} b)(s \\minus{} c) \\equal{} \\sum (a^2 \\plus{} b^2 \\minus{} c^2)(a^2 \\plus{} c^2 \\minus{} b^2) \\equal{} 2\\sum b^2c^2 \\minus{} \\sum a^4$ \n\nand $ \\sum\\cot B\\cot C \\equal{} 1$ . [b]Simply ![/b] The proposed inequality becomes $ \\sum a^4\\ge \\sum b^2c^2$ what is evidently.[/quote]\r\n\r\nVery Nice my Dear Friend.. Thanks :)", "Solution_7": "[quote=\"Heebeen, Yang\"]If we use them,\n$ \\cot A \\equal{} \\frac {2R(b^{2} \\plus{} c^{2} \\minus{} a^{2})}{2abc}$and $ \\cot B, \\cot C$ defined similar.\nThen, above inequality is equivalent to \n$ a^{4} \\plus{} b^{4} \\plus{} c^{4} \\geq \\sum_{cyc}(b^{2} \\plus{} c^{2} \\minus{} a^{2})(c^{2} \\plus{} a^{2} \\minus{} b^{2})$\n<=>\n$ a^{4} \\plus{} b^{4} \\plus{} c^{4} \\geq a^{2}b^{2} \\plus{} b^{2}c^{2} \\plus{} c^{2}a^{2}$[/quote]\r\n\r\nThanks Dear [color=red][b]Yang[/b][/color]...." } { "Tag": [ "quadratics", "LaTeX", "trigonometry", "limit", "algebra unsolved", "algebra" ], "Problem": "Solve:\r\n$ e^{x\\minus{}y}\\equal{}\\frac{sinx}{siny}$\r\n$ 10\\sqrt{x^6\\plus{}1}\\equal{}3(y^4\\plus{}2)$\r\n$ \\pi $ e^{x}siny\\equal{}e^{y}sinx$ ----> $ e^{x}*\\frac{e^{iy}\\minus{}e^{\\minus{}iy}}{2i}\\equal{}e^{y}*\\frac{e^{ix}\\minus{}e^{\\minus{}ix}}{2i}$\r\n\r\n\r\n$ e^{x\\plus{}iy}\\minus{}e^{x\\minus{}iy}\\equal{}e^{y\\plus{}ix}\\minus{}e^{y\\minus{}ix}$-----> $ \\minus{}\\minus{}e^{x\\minus{}iy}\\equal{}\\minus{}e^{y\\minus{}ix}$-----> $ x\\minus{}iy\\equal{}y\\minus{}ix$----> $ (1\\plus{}i)(x\\minus{}y)\\equal{}0$-----> $ x\\equal{}y$\r\n\r\n\r\n---> $ 10\\sqrt {x^6 \\plus{} 1} \\equal{} 3(x^4 \\plus{} 2)$ -----> $ 9x^{8}\\minus{}100x^{6}\\plus{}36x^{4}\\minus{}64\\equal{}0$----> $ x\\equal{}?$", "Solution_2": "[quote=\"zaya_yc\"]$ 9x^{8} \\minus{} 100x^{6} \\plus{} 36x^{4} \\minus{} 64 \\equal{} 0\\implies x \\equal{} ?$[/quote]\r\nIt can be factorized as: \r\n$ (9x^{4} \\minus{} 10x^{2} \\plus{} 8)(x^{4} \\minus{} 10x^2 \\minus{} 8) \\equal{} 0$\r\nNow we can use the quadratic formula. :wink: \r\n\r\n[hide=\"Latex Tips\"]The latex code of \"$ \\implies$\" is \"\\implies \"\nAnd \"\\iff \" for \"$ \\iff$\"\nThe code \"\\sin \" gives \"$ \\sin$\" which looks better than \"$ sin$\", and similarly for $ \\cos$, $ \\lim$ etc.\nFor multiplication \"\\cdot \" gives \"$ \\cdot$\", it looks better than \"$ *$\"\nSometimes \"\\cdots \" is needed for \"$ \\cdots$\"[/hide]", "Solution_3": "Let $ f(x)\\equal{}\\frac{\\sin x}{e^x}$. $ e^{x\\minus{}y}\\equal{}\\frac{sinx}{siny} \\iff f(x)\\equal{}f(y)$\r\n$ f'(x)\\equal{}\\frac{\\cos x \\minus{}\\sin x}{e^x}<0$ for $ \\pi < x < \\frac{5\\pi}{4} \\implies x\\equal{}y$" } { "Tag": [ "function", "algebra unsolved", "algebra" ], "Problem": "hi!!\r\nLet f a function such that: $ f(x) \\equal{} x^{n} \\minus{} \\sum_{k \\equal{} 0}^{n \\minus{} 1}x^{k}$ with $ n\\ge2$ is even.\r\nprove that f has two solutions $ a$ and $ b$ such as: $ a < 0 < b$\r\n\r\n\r\nthanx.", "Solution_1": "[hide=\"Hint\"]$ f( \\minus{} 1) \\equal{} 1 > 0$, $ f(0) \\equal{} 0$, $ f(1) \\equal{} 1 \\minus{} n < 0$.[/hide]\r\n\r\nSorry. $ f( \\minus{} 1) \\equal{} 1 > 0$, $ f(0) \\equal{} \\minus{} 1 < 0$, $ f(2) \\equal{} 1 > 0$.", "Solution_2": "why $ f(0) \\equal{} 0$ and $ f(1) \\equal{} 1 \\minus{} n$?\r\nI think we have $ f(0) \\equal{} \\minus{} 1$.\r\n\r\nI forgot that $ n\\ge2$." } { "Tag": [ "calculus", "integration", "inequalities", "Euler", "function", "real analysis", "real analysis unsolved" ], "Problem": "Try to find max $C$ \r\nfor wich we have\r\nif $0<{a}\\leq{f(x)}\\leq{b}$ and $0<{c}\\leq{g(x)}\\leq{d}$ and ${1/p}+{1/q}=1$ then \r\n${C}({\\int^{l}_{k}{f^{p}(x)}})^{1/p}({\\int^{l}_{k}{g^{q}(x)}})^{1/q}\\leq{\\int^{l}_{k}{f(x)}{g(x)}}\\leq({\\int^{l}_{k}{f^{p}(x)}})^{1/p}({\\int^{l}_{k}{g^{q}(x)}})^{1/q}$\r\nwhere $f(x)$ and $g(x)$ in interval$[k,l]$ is continious derivetiv function.\r\nWhen we have equal?", "Solution_1": "[quote=\"Extremal\"]Try to find max $C$ \nfor wich we have\nif ${a}\\leq{f(x)}\\leq{b}$ and ${c}\\leq{g(x)}\\leq{d}$ and ${1/p}+{1/q}=1$ then \n${C}({\\int^{l}_{k}{f^{p}(x)}})^{1/p}({\\int^{l}_{k}{g^{q}(x)}})^{1/q}\\leq{\\int^{l}_{k}{f(x)}{g(x)}}\\leq({\\int^{l}_{k}{f^{p}(x)}})^{1/p}({\\int^{l}_{k}{g^{q}(x)}})^{1/q}$\nwhere $f(x)$ and $g(x)$ in interval$[k,l]$ is continious derivetiv function.\nWhen we have equal?[/quote]\r\nFew comment: right inequality is known It's calld Holders integral inequality :lol: , but it's know everyone.But you don't know leftintegral inequality, it's very nice and interesting. it's open many interesting things in real anaysis and not only here.It's can bu used to solve difficult problems posted in the extremal theories.Every problems wich solved by holders integral inequality now we can use this this and give problems different face. I don't know is it known or not!! But I will post Sulution of this problem after Conferenc Eulers in Saint Petersburg. \r\nThe sulution is hard. Good luck. \r\n :|", "Solution_2": "Very simple, very crude estimates give us that $\\int fg\\ge ac(k-l)$ while $\\|f\\|_{p}\\|g\\|_{q}\\le bd(k-l).$ Hence, $C=\\frac{ac}{bd}$ would work. \r\n\r\nI would guess that that's not best possible and that one can find $C$ larger than that.\r\n\r\nIncidentally, the condition for equality in H\u00f6lder's inequality - the right hand inequality - (assuming positive functions) is that $\\frac{f^{p}}{g^{q}}$ is constant.", "Solution_3": "your way is simple, but it's not best constant. I have more big than your :wink:", "Solution_4": "[quote=\"Kent Merryfield\"]Very simple, very crude estimates give us that $\\int fg\\ge ac(k-l)$ while $\\|f\\|_{p}\\|g\\|_{q}\\le bd(k-l).$ Hence, $C=\\frac{ac}{bd}$ would work. \n\nI would guess that that's not best possible and that one can find $C$ larger than that.\n\nIncidentally, the condition for equality in H\u00f6lder's inequality - the right hand inequality - (assuming positive functions) is that $\\frac{f^{p}}{g^{q}}$ is constant.[/quote]\r\n\r\nKent Merryfield, $\\int fg\\ge ac(k-l)$ it is true and equl when f=a,g=c,\r\nthen $\\|f\\|_{p}\\|g\\|_{q}\\le bd(k-l).$ it's true and equal when f=b, g=d. \r\nthen you write \r\n$(\\int fg)/{ac}\\ge{(k-l)}\\ge{(\\|f\\|_{p}\\|g\\|_{q})/{bd}}$ here can't be achievd first equal and seccond equal togather/. therefore this constant is wrong.", "Solution_5": "I knew all of that already, and I was already assuming this wasn't best possible. I was just putting a floor under it.", "Solution_6": "[quote=\"Kent Merryfield\"]I knew all of that already, and I was already assuming this wasn't best possible. I was just putting a floor under it.[/quote]\r\nOK. I have a question: Do you anyvere heardabout this inequality? I mean left :lol:", "Solution_7": "[quote][b] Assume:\n1) $p>1\\; ,\\; \\frac{1}{p}+\\frac{1}{q}=1$ ,\n2) $x\\to f(x)^{p}\\; ,\\; x\\to g(x)^{q}$ are integrable functions on $[a,b] ,$ \n3) $01\\; ,\\; \\frac{1}{p}+\\frac{1}{q}=1$ ,\n2) $x\\to f(x)^{p}\\; ,\\; x\\to g(x)^{q}$ are integrable functions on $[a,b] ,$ \n3) $0 dobi\u0161 ravninski graf in to celo drevo. Potem pa gruntaj kaj naprej... (a je to dost hitro?)\r\n\r\nBtw: Od kod si pa ti?", "Solution_19": "No ja, ta re\u0161itev je precej podobna \"predpisani\" sam sej po moje res ni kraj\u0161ega na\u010dina (razen Tom@\u017eevega ;) )\r\n\r\npr nas lahko gre pa mnda iz vsakega letnika sam po eden... drga\u010d je pa to I. gimnazija v Celju :)", "Solution_20": "[quote=\"majaa\"]No ja, ta re\u0161itev je precej podobna [color=blue]\"predpisani\"[/color] sam sej po moje res ni kraj\u0161ega na\u010dina (razen Tom@\u017eevega ;) )[/quote]\r\n\r\nhehe, sm si pa \u017ee skor ustvaru iluzijo, da sm kej novga proguntu... :oops:", "Solution_21": "no sej \u010de si to sam pogruntu je vseen zlo v redu ker si se o\u010ditno spomnu najbolj\u0161e mo\u017ene poti :)" } { "Tag": [ "Support" ], "Problem": "Which one?", "Solution_1": "Who's actually going to own up to IE? Seriously", "Solution_2": "What about the multibrowser option? I use Safari and Firefox. On the Windows machine, that's Firefox for everything, while it's Safari for almost everything (except AOPS) on my Mac.\r\n\r\n(I know Safari is now on the list of supported browsers here, but there were problems a year or two ago)", "Solution_3": "Apparently IE is by far the most used. Surprising...I now use Firefox, but personally I don't prefer either over the other. I'm just more used to clicking on the firefox icon on my desktop...and it's useful sometimes to have two different web browsers.", "Solution_4": "[quote=\"K81o7\"]Apparently IE is by far the most used. Surprising...I now use Firefox, but personally I don't prefer either over the other. I'm just more used to clicking on the firefox icon on my desktop...and it's useful sometimes to have two different web browsers.[/quote]\r\n\r\nIE is most used because anyone who doesn't know what a browser is uses it.", "Solution_5": "I use Firefox, but I keep IE handy just in case.", "Solution_6": "[quote=\"7h3.D3m0n.117\"]I use Firefox, but I keep IE handy just in case.[/quote]\r\n\r\nAre you sure thats not because MS have embedded it so deeply in the OS its too hard to get rid of?", "Solution_7": "No :wink: . I use IE7 whenever my Firefox doesn't support a certain plugin (too lazy to install).", "Solution_8": "[quote=\"SimonM\"][quote=\"7h3.D3m0n.117\"]I use Firefox, but I keep IE handy just in case.[/quote]\n\nAre you sure thats not because MS have embedded it so deeply in the OS its too hard to get rid of?[/quote]\r\n\r\nActually I got rid of it... you just remove it, but I reinstalled it again lol", "Solution_9": "I have a Mac. I use Safari more often, and use Firefox for AoPS classes and Java applets. If I have to, I go to Parallels and open Windows to go on IE or Firefox.", "Solution_10": "FIREFOX!!!!!!!!!!!!!!!!!!!!!!!!!!!!!!!!!!\r\n\r\nI use 10 extensions and 5 themes. Firefox is definitely the best.\r\n\r\nCurrent theme: Vista-Aero\r\n\r\nWell IE7 is, I admit, cool, but\r\n1. Vista-Aero can change Firefox to that identical look\r\n2. CSS can make it even cooler\r\n3. Vimperator\r\n4. 1337key\r\n5. AiOS\r\n6. A lot of things only people really deep into HTML would know.\r\n\r\nFF for all eternity!", "Solution_11": "[quote=\"math_explorer\"]4. 1337key[/quote]\r\nI see, the ultimative reason to use FireFox :wink: \r\n\r\nI use FireFox only, I never had big problems with it. Well, the pdf-plugin sometimes crashes...\r\nThe only real disadvantage is the big amount of RAM it occupies; especially, it's memory usage sometimes grows and grows without any reason.", "Solution_12": "Firefox.. definitely", "Solution_13": "But, 1337key is broken, AiOS feels granted, and Vimperator just isn't that useful. Firebug is useful. Download Statusbar, Gmail Manager, YSlow, Talkback, Google Notebook. That's all the extensions I have.\r\n\r\nYippee!", "Solution_14": "just wondering, what's the 1337key?\r\n\r\ni like firefox. just that my computer doesn't have it b/c my parents are like, \"why do you need firefox? you have IE.\"\r\n\r\ni hope to get a mac soon and get firefox on there and all my awesome programs that i'm not allowed to download.", "Solution_15": "[quote=\"kimmystar94\"]just wondering, what's the 1337key?[/quote]\n\nPlugin that turns everything into L337 so you don't have to manually translate and type as you go. \n\n[quote=\"kimmystar94\"]i like firefox. just that my computer doesn't have it b/c my parents are like, \"why do you need firefox? you have IE.\"[/quote]\r\n\r\nYour parents aren't very knowledgeable of computers and Internet, I see.", "Solution_16": "Firefox has better security...i think...", "Solution_17": "Yeah, and it supports pseudo-classes for everything. There are standards for how the web pages are supposed to be written and how they should then look. IE fails to follow a lot of these standards.\r\n\r\nSomething like that...\r\n\r\n\r\nPlus, if you don't like the default font (I despise it!), there's this wonderful part of Firefox where you get to change it. Just that you need to know some CSS and be good at Googling.\r\n\r\nAnd so on.", "Solution_18": "[quote=\"7h3.D3m0n.117\"][quote=\"kimmystar94\"]just wondering, what's the 1337key?[/quote]\n\nPlugin that turns everything into L337 so you don't have to manually translate and type as you go. \n\n[quote=\"kimmystar94\"]i like firefox. just that my computer doesn't have it b/c my parents are like, \"why do you need firefox? you have IE.\"[/quote]\n\nYour parents aren't very knowledgeable of computers and Internet, I see.[/quote]\r\n\r\nthanks 7h3.D3m0n.117... and my parents are not very knowledgeable of computers and internet.\r\nhowever, i want to be very knowledgeable.. i asked them and they're like, \"no\" you can always use IE for testing web layouts and programming...\r\nand i want a mac!!!", "Solution_19": "How about links? You can view some images, and the benefits of browsing without Javascript and Flash and Java are myriad. But I don't see links/lynx on the poll...\r\n.\r\n[hide=\".\"]/troll[/hide]", "Solution_20": "I often use firefox. I sometimes use safari if I'm using a website that I know works well in safari.", "Solution_21": "I use firefox on the family computer, but IE on my mom's laptop." } { "Tag": [], "Problem": "figure out as much as u could \r\n\r\n[img]http://img252.imageshack.us/img252/7154/untitledyy5.png[/img]", "Solution_1": "Which are the ones which are on the row? it isn't clear.\r\n Bharath", "Solution_2": "my guess is that its the horizontal rows which are in a 1-4-2-4-1 formation :maybe:", "Solution_3": "This is impossible because you cant have 2 different numbers on the first/last row that are the same.", "Solution_4": "hmm, good point. maybe columns?", "Solution_5": "Maybe its the diagonal rows/column/whatever.", "Solution_6": "i think its the sides of the two triangles... :maybe:" } { "Tag": [ "percent" ], "Problem": "How do I convert a mixed number into a proper fraction? Like for example 35 and 4/6th to a proper fraction?\r\n\r\nThanks.", "Solution_1": "Dude, read your book ...\r\n\r\n\r\nMultiply it by 6, then divide by 6.\r\n\r\nIf you multiply that number by 6, you get $ 35 \\times 6 \\plus{} 4 \\equal{} 214$\r\n\r\nNow divide by 6 to get $ \\frac {214}{6}$\r\n\r\nYou can simplify it by dividing the top and bottom by $ 2$, to get $ \\frac {107}{3}$", "Solution_2": "Actually that would be an improper fraction.\r\nIt is impossible to convert to a proper fraction.", "Solution_3": "I don't think he really meant proper. \r\n\r\nAs Zuton shows, simply multiply the whole number by the denominator in the fraction, and then add this product to the numerator in the fraction.", "Solution_4": "[quote=\"Ryuk\"]I don't think he really meant proper. \n\nAs Zuton shows, simply multiply the whole number by the denominator in the fraction, and then add this product to the numerator in the fraction.[/quote]\r\nI meant proper, because on this website I saw it done, but it didn't make sense to me.\r\nhttp://www.purplemath.com/modules/percents.htm\r\n\r\nHalfway down the page, under \"another example\", for 33 and 1/3 he converted that to 1/3.", "Solution_5": "[quote=\"leroyjenkens\"][quote=\"Ryuk\"]I don't think he really meant proper. \n\nAs Zuton shows, simply multiply the whole number by the denominator in the fraction, and then add this product to the numerator in the fraction.[/quote]\nI meant proper, because on this website I saw it done, but it didn't make sense to me.\nhttp://www.purplemath.com/modules/percents.htm\n\nHalfway down the page, under \"another example\", for 33 and 1/3 he converted that to 1/3.[/quote]\r\n\r\nOh, that was for percentages. If you meant percentages, just put your number over 100, and reduce.", "Solution_6": "its probably .33 not 33", "Solution_7": "[quote=\"Walk Around The River\"][quote=\"leroyjenkens\"][quote=\"Ryuk\"]I don't think he really meant proper. \n\nAs Zuton shows, simply multiply the whole number by the denominator in the fraction, and then add this product to the numerator in the fraction.[/quote]\nI meant proper, because on this website I saw it done, but it didn't make sense to me.\nhttp://www.purplemath.com/modules/percents.htm\n\nHalfway down the page, under \"another example\", for 33 and 1/3 he converted that to 1/3.[/quote]\n\nOh, that was for percentages. If you meant percentages, just put your number over 100, and reduce.[/quote]\r\nOh, I didn't think that mattered. Alright thanks. How do I reduce the percentage part of it? Like for example 24 and 2/4ths. How do I reduce the 2/4ths along with the 24?\r\nIsn't it still considered a mixed number if it's a percent?", "Solution_8": "A percent just means that the number is a numerator in a fraction where 100 is the denominator.\r\n\r\ne.g.\r\n\r\n$ a$% = $ \\frac{a}{100}$\r\n\r\n\r\nHere's an example of converting fractions:\r\n\r\n$ 35\\frac{4}{6}$\r\n\r\n$ \\equal{}35\\frac{2}{3}$\r\n\r\n$ \\equal{}\\frac{(35)(3)}{3}\\plus{}\\frac{2}{3}$\r\n\r\n$ \\equal{}\\frac{105}{3}\\plus{}\\frac{2}{3}$\r\n\r\n$ \\equal{}\\frac{105\\plus{}2}{3}$\r\n\r\n$ \\equal{}\\frac{107}{3}$\r\n\r\n\r\nBTW: As people have said, this is converting to an improper fraction. A proper fraction is one where the numerator is less than the denominator.", "Solution_9": "[quote=\"fishythefish\"]A percent just means that the number is a numerator in a fraction where 100 is the denominator.\n\ne.g.\n\n$ a$% = $ \\frac {a}{100}$\n\n\nHere's an example of converting fractions:\n\n$ 35\\frac {4}{6}$\n\n$ \\equal{} 35\\frac {2}{3}$\n\n$ \\equal{} \\frac {(35)(3)}{3} \\plus{} \\frac {2}{3}$\n\n$ \\equal{} \\frac {105}{3} \\plus{} \\frac {2}{3}$\n\n$ \\equal{} \\frac {105 \\plus{} 2}{3}$\n\n$ \\equal{} \\frac {107}{3}$\n\n\nBTW: As people have said, this is converting to an improper fraction. A proper fraction is one where the numerator is less than the denominator.[/quote]\r\nYeah, the reason I asked was because in the link I gave, the guy converts an improper fraction, which is a percentage (33 and 1/3 percent) into a proper fraction. He converted it to 1/3rd. I was wondering how he did that.\r\nThanks for the responses.", "Solution_10": "It's because $ 33\\frac{1}{3}$%\r\n\r\n$ \\equal{}\\frac{33\\frac{1}{3}}{100}$\r\n\r\n$ \\equal{}\\frac{1}{3}$\r\n\r\nThat's all he did.", "Solution_11": "[quote=\"fishythefish\"]It's because $ 33\\frac {1}{3}$%\n\n$ \\equal{} \\frac {33\\frac {1}{3}}{100}$\n\n$ \\equal{} \\frac {1}{3}$\n\nThat's all he did.[/quote]\r\nFor that one you can see that it's 1/3rd just by looking at it, but I don't know the method of turning it into a proper fraction. For example, one that's more complex, like 44 and 2/6ths percent. How do I turn that into a proper fraction?\r\nThanks.", "Solution_12": "multiply the top and bottom by the denominator of the fraction.\r\n\r\n(44 2/6)/100\r\n(44 1/3)/100\r\n133/300\r\n\r\ncan't be simplified.", "Solution_13": "[quote=\"Ihatepie\"]multiply the top and bottom by the denominator of the fraction.\n\n(44 2/6)/100\n(44 1/3)/100\n133/300\n\ncan't be simplified.[/quote]\r\nOh, well that's simple enough. Thanks. But just so I know, this can't be done with just a mixed number? It has to be a fraction? Thanks." } { "Tag": [ "combinatorics proposed", "combinatorics" ], "Problem": "Consider a $25 \\times 25$ grid of unit squares. Draw with a red pen contours of squares of any size on the grid. What is the minimal number of squares we must draw in order to colour all the lines of the grid?", "Solution_1": "It can be done trivially with 48 squares, just connect $(i,i)$ to $\\left(25\\left[\\frac{25-i}{25}\\right],25\\left[\\frac{25-i}{25}\\right]\\right)$ and $(i,-i)$ to $\\left(25\\left[\\frac{25-i}{25}\\right],-25\\left[\\frac{25-i}{25}\\right]\\right)$, where $[...]$ denotes round(...). \r\n\r\nNow can it be done with 47? No, consider the 24 inner horizontal lines and 24 inner vertical lines. None of them can be covered by just 1 square, so we need at least 2 squares per line, and since a square has 4 sides we need at the very least $\\frac{2.24.2}{2}=48$ lines." } { "Tag": [ "LaTeX", "calculus", "integration", "linear algebra", "matrix", "function" ], "Problem": "Hello All,\r\n I need to get a large integral sign,\r\nsay if i have to put an integral sign in front of a 3X3 matrix...\r\n\r\nthnx in advance\r\nPparakh", "Solution_1": "ok, here's what I found.\r\n\r\nTwo packages can vary the size of the integral sign. [b]mtpro[/b] and [b]bigint[/b]. None of them are in the miktex distribution. I tried [b]bigint[/b] but it didn't work, something about using the cmex10 font but I don't know. Look for bigint.sty in google.\r\n\r\n[b]mtpro[/b] seems much better but I couldn't find it on ctan (didn't try really hard though).\r\n\r\nIf you find a way, tell me.\r\n\r\nLedurt\r\n\r\nP.S.: Never seen an integral of a matrix but I like the idea;)", "Solution_2": "That would be some funny-looking notation, and I'm not sure I'd do it if I could. Of course you can take the integral of a matrix - the integral is linear, after all. But I'd probably use more than one line.\r\n\r\n$A(t) = $(matrix of functions, in detail), followed by\r\n\r\n$\\int_a^bA(t)\\,dt.$\r\n\r\nPart of the charm and power of matrix notation is its abilitity to compress notation into a small space, if you'll let it.", "Solution_3": "great solution Kent. Plus it's less scary for the readers to see \r\n$\\int A(t) dt$ \r\nthan the integral of a matrix.\r\n\r\ngood call\r\n\r\nledurt" } { "Tag": [ "function", "algebra unsolved", "algebra" ], "Problem": "1. Find, as a function of $n$, the sum of the digits of $9 \\times 99 \\times 9999 \\times \\cdots \\times \\left( 10^{2^n} - 1 \\right)$, where each factor has twice as many digits as the previous one.", "Solution_1": "It is $9*2^n$.\r\nLet \\[ m(n)=\\prod_{k=0}^n (10^{2^k}-1)=\\sum_{i=0}^{2^{n+1}-1} (-1)^{n+1-s_2(i)}10^i. \\] Were $s_2(i)$ is sum of 2-adic digits.\r\nWe have $s(m(n))=s(m(n-1))+s(10^{2^n}-m(n-1))=s(m(n-1))-1+(1+9*2^n-s(m(n-1))=9*2^n.$" } { "Tag": [ "integration", "calculus", "algebra", "polynomial", "real analysis", "real analysis unsolved" ], "Problem": "QUESTION: Following proposition is correct ?\r\n[color=green] [b] If $f , w \\in C [a,b], w \\ge 0$ on $[a,b], \\int\\limits_{a}^{b}w(x) \\; dx =1 ,$ then there exists $c , c\\in (a,b)$ such that $\\int\\limits_{a}^{b}f(x)w(x)\\; dx = f(c) .$\n [/b] [/color] If not, why ? If it is false , please a counterexample and give the correct assertion", "Solution_1": "it's true , because if we put m=min(f) and M=max(f) then \r\n for all x in [a,b] we have m.w(x)<=f(x).w(x)<=M.w(x) and by integral we get m<=integral(f(x).w(x)dx,a,b)<=M , f is continuous , the result follows (intermidiate values )", "Solution_2": "[quote=\"yassinus\"]it's true , because ..... [/quote] [b]The mean-value theorem, posted by proposer, [u] is'nt correct[/u] ! Why ?[/b]", "Solution_3": "i dont undertand what you want !! what i have write doesn't convaincu you ?", "Solution_4": "but by what you wrote,we can only obtain $c \\in [a,b]$\r\nnot $c \\in (a,b)$,right?", "Solution_5": "[quote=\"zhaobin\"]but by what you wrote,we can only obtain $c \\in [a,b]$ not $c \\in (a,b)$,right?[/quote] \r\n[color=blue] [b] [u]Yes[/u] , but please give a counterexample ! [/b] [/color] Thanks,for interest/proposer .", "Solution_6": ":blush: yes , i see , i'm thinking ...", "Solution_7": "Sorry,I can't give a couterexample,Because I think the proposition is correct ;) \r\n[b]Solution[/b]\r\nLet $m=\\min\\{f|x \\in[a,b]\\},M=\\max\\{f|x \\in[a,b]\\}$\r\nWe can easily get:\\[ m \\le \\int_{a}^{b}f(x)w(x)dx \\le M \\]\r\n(We may assume $M>m$,if not then f is constant on [a,b],the result follow quickly)\r\nIt is easy to prove the problem,if \\[ m<\\int_{a}^{b}f(x)w(x)dx0$\r\nthen because $w \\in C[a,b]$,we can get a intervuale $[d,e] \\subseteqq [a,b]$,such that \r\n\\[ f(x)>\\frac{f(\\xi)}{2},x \\in [d,e] \\]\r\nthen we get \r\n\\[ 0=\\int_{a}^{b}\\left(f(x)-m\\right)w(x)dx \\ge \\int_{d}^{e}\\left(f(x)-m\\right)w(x)dx \\ge \\frac{f(\\xi)}{2} \\int_{d}^{e}\\left(f(x)-m\\right)dx \\ge 0 \\]\r\nThat means $f(x)=m,x \\in [d,e]$,then we can choose every $c \\in [d,e]$ as desired.\r\nand similar when $M=\\int_{a}^{b}f(x)w(x)dx$\r\n\r\n\r\n[b]More generality:[/b]If $f \\in C[a,b]$,$w \\in R[a,b]$ and g doesn't change sign on $[a,b]$,then there exist $c \\in (a,b)$ such that:\r\n\\[ \\int_a^b f(x)w(x)dx=f(c)\\int_a^b w(x)dx \\] :)", "Solution_8": "[quote=\"zhaobin\"]... I think the proposition is correct ...\n[b]If $f \\in C[a,b]$,$w \\in R[a,b]$ and g doesn't change sign on $[a,b]$,then there exist $\\theta \\in (a,b)$ such that: [/b] \\[ \\int_a^b f(x)w(x)dx=f(\\theta)\\int_a^b w(x)dx \\] :)[/quote]\r\nOK, but $c =a$or $c=b$ is also possible ! \r\nLet $f \\in C[a,b]$ and consider the mean-value theorem \r\n$(\\pounds)\\; \\; \\; \\; \\frac{1}{b-a}\\int\\limits_{a}^{b}f(x) \\; dx= f(\\theta)\\; ,\\theta \\in [a,b] .$ (That is $w(x)=\\frac{1}{b-a}$ ).\r\n\r\nLet $[a,b]: =[0,1]$ , i.e. for the mean value theorem $\\int\\limits_{0}^{1}f(x)\\; dx =f(\\theta) \\; :$ \r\nTake $f_*(x)= x-\\frac{3}{4}x^2 ;$ In this case equation $\\int\\limits_{0}^{1}f_*(x)\\; dx =f_*(\\theta) \\; ,$ has solution $\\theta=1$ but also $\\theta=\\frac{1}{2}$ ( which belongs to $[0,1]$ ) verifies the equation.\r\n\r\n[b] Other remarks:[/b]\r\n[color=green][b]1) [/b][/color] If there exists $h\\in C^1[a,b]$ such that $F=h^{\\prime}$ on $[a,b]$, then (according to Lagrange mean value theorem of differential calculus) we have $\\theta \\in (a,b)$ (see $\\pounds$).\r\n\r\n[color=green][b]2)[/b] [/color] In ($\\pounds$) it's clear that [b] it's possible to have $\\theta= a$ or/and $\\theta=b .$ [/b]\r\nAnother example , (see also above), in case $[a,b]=[-1,2]$ may be : $f_1(x)=\\left\\{\\begin{array}{lcl} P_n(x) &, & x\\in [-1,1] \\\\ \\frac{4}{5}(1-x) +1 &, & x\\in (1,2] \\; ,\\\\ \\end{array}\\right.$ \r\nwhere $P_n(x)=\\frac{1}{2^n n!}\\left((x^2-1)^n\\right)^{(n)}$ is the Legendre polynomial of degree $n$ . Remind that \r\n$\\left|P_n(x) \\right| \\le 1$ for $x\\in [-1,1]$ , $\\int\\limits_{-1}^{1}P_n(x)\\; dx = 0$ , $(n\\ge 1),\\;$ $P_n(-x)=(-1)^nP_n(x)\\; ,\\; P_n(1)=1 .$\r\n If we consider the corresponding mean value \r\n$(\\pounds ')\\; \\; \\; \\int\\limits_{-1}^{2}f_1(x)\\; dx = 3f_1(\\theta)$ , then $\\theta=2$ is a possiblity (a solution) .\r\n [b] But $\\theta$ may be also any other solution , from $[-1,1]$ [/b], of equation $P_n(\\theta)-\\frac{1}{5}=0 .$ Because ,e.g. in case $n=2\\; ,\\; P_2(x) =\\frac{3}{2}x^2-\\frac{1}{2} ,$ and we conclude that $\\theta \\in \\left\\{-\\sqrt{\\frac{7}{15}},\\; \\sqrt{\\frac{7}{15}},\\; 2\\right\\}\\limits_{n=2} .$", "Solution_9": "Thanks,nice.(But I didn't say that It is impossible that $\\theta=a$ or $b$) :) :?" } { "Tag": [ "geometry", "3D geometry", "calculus", "calculus computations" ], "Problem": "2. A right cylinder is inscribed in a right circular cone with the radius 3 cm and height 5 cm. Find the dimensions of the cylinder of maximum volume.", "Solution_1": "$ V\\equal{}\\pi r^2 h$, we can make r depend on h since we are inscribed in the cone\r\n\r\n$ r(h)\\equal{}3\\minus{}\\frac{3}{5}h$ so $ V(h)\\equal{}9\\pi h (1\\minus{}\\frac{h}{5})^2$\r\n\r\nso $ V'(h)\\equal{}9\\pi[ (1\\minus{}\\frac{h}{5})^2\\minus{}2\\frac{h}{5}(1\\minus{}\\frac{h}{5})]\\equal{}9\\pi [1\\minus{}\\frac{3h}{5}](1\\minus{}\\frac{h}{5})$\r\n\r\nhas max at $ h\\equal{}\\frac{5}{3}$, which is $ V(\\frac{5}{3})\\equal{}9\\pi \\frac{5}{3} (1\\minus{} \\frac{1}{3})^2\\equal{}\\frac{20\\pi}{3}$" } { "Tag": [ "inequalities", "inequalities proposed" ], "Problem": "Given $k$ be a positive real;$x,y,z >0$ satisfying $xy+yz+zx=3$.Prove that:\r\n $(x+y)^{k}+(y+z)^{k}+(x+z)^{k}\\geq 3.2^{k}$", "Solution_1": "hint\r\n[hide]\n$(x+y)(y+z)(z+x) \\ge \\frac{8}{9}(x+y+z)(xy+yz+zx)$[/hide]", "Solution_2": "how do you get that inequality zhaobin?", "Solution_3": "because \\[(a+b)(b+c)(c+a)=(a+b+c)(ab+bc+ca)+abc\\]" } { "Tag": [ "factorial", "function", "MATHCOUNTS", "calculus", "integration" ], "Problem": "What happens when you have\r\n\r\n$-5!$?\r\n\r\nis it\r\n\r\n$-(5 \\times 4 \\times 3 \\times 2 \\times 1) =-120$?\r\n\r\nor\r\n\r\n$-5 \\times-4 \\times-3 \\times-2\\times-1$?", "Solution_1": "In the strictest sense, you can't take a factorial of a negative number.\r\n\r\nHowever, in advanced math there is a thing called the gamma function, which is basically a generalized factorial. But I'd better stop.\r\n\r\nWoot, 1200th post!", "Solution_2": "[quote=\"mathnerd314\"]In the strictest sense, you can't take a factorial of a negative number.\n[/quote]\r\n\r\nThats what I thought but I kept having nightmares that it will appear in the Mathcounts Chapters tommorow :maybe: \r\n\r\nGratz mathnerd314 :lol:", "Solution_3": "[quote=\"7h3.D3m0n.117\"][quote=\"mathnerd314\"]In the strictest sense, you can't take a factorial of a negative number.\n[/quote]\n\nThats what I thought but I kept having nightmares that it will appear in the Mathcounts Chapters tommorow :maybe: [/quote]\r\nFactorials don't appear very often, so I wouldn't worry about negative or non-integral factorials.", "Solution_4": "If they start putting negative factorials on mc I'm not going", "Solution_5": "If you actually tried to do this, (-1)! wouldn't be defined because it would end up as 1/0, same thing with all other negative factorials. The Gamma Theorem makes people believe it is possible, but the math greats have yet to determine negative factorials as something other than the number/0. If they put this on tests for mathcounts I would stop going to practices.", "Solution_6": "Factorial of any negative number would be undefined.\r\nI find it interesting, though, that 0!=1. I understand 1!=1 and, therefore, 0!=1/1=1. But some how it just doesn't make sense to me because n! is defined as 1*2*3*...*n.", "Solution_7": "[quote=\"myspacesucks\"]If you actually tried to do this, (-1)! wouldn't be defined because it would end up as 1/0, same thing with all other negative factorials. The Gamma Theorem makes people believe it is possible, but the math greats have yet to determine negative factorials as something other than the number/0. If they put this on tests for mathcounts I would stop going to practices.[/quote]\r\n\r\nIt's the gamma function, not the gamma theorem. Its relationship to the factorial is:\r\n\r\n$\\Gamma(x+1)=x!$\r\n\r\nif the factorial is defined. If it's not, you use calculus to get the answer.\r\n\r\nIf they put this on tests for Mathcounts, I would keep going to practices. I would just complain to the people who write the tests.", "Solution_8": "sorry, I meant the gamma function not theorem, but calculus is not at mc level. That's why I didn't talk about that.", "Solution_9": "[quote=\"7h3.D3m0n.117\"]What happens when you have\n\n$-5!$?\n\nis it\n\n$-(5 \\times 4 \\times 3 \\times 2 \\times 1) =-120$?\n\nor\n\n$-5 \\times-4 \\times-3 \\times-2\\times-1$?[/quote]\r\n\r\n$-5!=-(5 \\times 4 \\times 3 \\times 2 \\times 1) =-120$\r\n\r\nbut $(-5)!\\neq-5 \\times-4 \\times-3 \\times-2\\times-1$", "Solution_10": "[quote=\"7h3.D3m0n.117\"]What happens when you have\n\n$-5!$?[/quote]\r\n\r\nThis isn't a question about negative factorials as much as it is a question of [b]order-of-operations.[/b]\r\n\r\n$-5! =-(5!) =-120$\r\n\r\nThat's all there is to it.", "Solution_11": "uuhh...\r\n\r\nI thought that\r\n\r\n$(n-1)!=n!/n$\r\n\r\nBut, with n equal to 0, it will be undefined.", "Solution_12": "No. You get $\\frac{1}{1}$ which is clearly $1$.", "Solution_13": "I don't think we need to worry about this. This stuff definitely won't be at chapter, or state, probably not even nationals. Unless they explain to you how they want you to do it, which would make it very, very easy.", "Solution_14": "I guess they'd probly say $-(5!)$ if visharul is to be right", "Solution_15": "why r we still on this? i mean just forget about it" } { "Tag": [ "inequalities", "inequalities proposed" ], "Problem": "$ \\ a,b,c>0$ v\u00e0 $ \\ a\\plus{}b\\plus{}c\\equal{}1$ .\r\n\r\nProve that :\r\n\r\n$ \\frac{a^2\\plus{}b^3}{b\\plus{}3c^2}\\plus{}\\frac{b^2\\plus{}c^3}{c\\plus{}3a^2}\\plus{}\r\n\\frac{c^2\\plus{}a^3}{a\\plus{}3b^2}\\geq\\frac{2}{3}$.", "Solution_1": "You had a mistake in type, RHS is $ \\frac {3}{2}$, not $ \\frac{2}{3}$\r\nIndeed, it's a weak ineq :) \r\nWe have: $ \\frac {b^3}{b \\plus{} 3c^2} \\plus{} \\frac {c^3}{c \\plus{} 3a^2} \\plus{} \\frac {a^3}{a \\plus{} 3b^2}\\geq\\frac {3}{4}$\r\nSo, we only need to prove: $ \\frac {a^2}{b \\plus{} 3c^2} \\geq\\ \\frac {3}{4}$\r\nIt's weaker than my old result:\r\nGiven $ a, b, c > 0$ and $ a \\plus{} b \\plus{} c \\equal{} 3$. Prove that:\r\n$ \\frac {a}{\\sqrt {4b \\plus{} 4c^2 \\plus{} 1}} \\plus{} \\frac {b}{\\sqrt {4c \\plus{} 4a^2 \\plus{} 1}} \\plus{} \\frac {c}{\\sqrt {4a \\plus{} 4b^2 \\plus{} 1}} \\geq\\ 1$\r\n$ k \\equal{} \\frac {1}{4}$ is also the best constant for this ineq trues :)", "Solution_2": "[quote=\"nguoivn\"]You had a mistake in type, RHS is $ \\frac {3}{2}$, not $ \\frac {2}{3}$\nIndeed, it's a weak ineq :) \nWe have: $ \\frac {b^3}{b \\plus{} 3c^2} \\plus{} \\frac {c^3}{c \\plus{} 3a^2} \\plus{} \\frac {a^3}{a \\plus{} 3b^2}\\geq\\frac {3}{4}$\nSo, we only need to prove: $ \\frac {a^2}{b \\plus{} 3c^2} \\geq\\ \\frac {3}{4}$\nIt's weaker than my old result:\nGiven $ a, b, c > 0$ and $ a \\plus{} b \\plus{} c \\equal{} 3$. Prove that:\n$ \\frac {a}{\\sqrt {4b \\plus{} 4c^2 \\plus{} 1}} \\plus{} \\frac {b}{\\sqrt {4c \\plus{} 4a^2 \\plus{} 1}} \\plus{} \\frac {c}{\\sqrt {4a \\plus{} 4b^2 \\plus{} 1}} \\geq\\ 1$\n$ k \\equal{} \\frac {1}{4}$ is also the best constant for this ineq trues :)[/quote]\r\nI can solve this inequality with $ c \\ge\\ b \\ge\\ a$ :D" } { "Tag": [ "function", "calculus", "integration", "search", "number theory", "complex numbers", "complex analysis" ], "Problem": "hi, i'm trying to do some basic number theory , and the riemann function turns out to be very important\r\n\r\nsum(1/n^z, n=1..infinity) can easily be shown to be convergent for every z with real number greater than one\r\n\r\n1)but for which z in general does this series converge?\r\n\r\nif z is real then it is quite clear that is must be greater than 1, but how about imaginary numbers\r\n\r\n\r\n2) i often hear about extending this function to the entire plane except the the line with real part 1, by analytic continuation\r\n\r\nwhat is analytic continuation, \r\nit has something to do with power summands, but is it easy how exactly you must extend the function?", "Solution_1": "You have surely already got your question answered. There are several ways to make an analytic continuation of this function to the entire complex plane by making it a mermomorphic function with a pole of residue 1 at z = 1 and no other singularities. I believe Ahlfors covers this issue in his Complex Analysis book, but I don't have that book anymore. Anyways, I remember rewriting the \"normal\" sum-definition as a sum of a convergent sum for Re s > 0 or mabye it was >= 0 and an integral then you repeadily can use the functional equation etc etc. I think this should work, don't have time to check it. But a search on \"analytic continuation of zeta function\" on google would give answers.", "Solution_2": "For a proof, check out http://www.math.harvard.edu/~elkies/M259.02/zeta1.pdf\r\n\r\nAndrei" } { "Tag": [ "geometry", "geometric transformation", "reflection", "MATHCOUNTS", "\\/closed" ], "Problem": "Some students have noted that our classes start too early for them to participate. We are considering changing our schedule for the next set of classes we offer. Please only answer this poll if you would consider taking one of our classes at the times mentioned in the poll. Also, please make any comments about timing you might have.", "Solution_1": "I voted for 7-8:30 est, because this gives me an early enuff time that I'm not absolutely dead (waking up at 6:15 every morning doesnt agree with me) and enuff time to finish whatever afterward. I woudl like to make one appeal, though. Please stop scheduling classes that i want to take on Tuesdays. Actually, just reschedule all classes that have olympiad in front of them currently on tuesdays. Please? Really, pretty please? With a giant cherry and proportioned whip cream and your favorite food (whatever it may be) on top?", "Solution_2": "I just had A+MATH click on his preferred time, which reflects our schedule in the Central Time Zone. My only other comment would be that making sure whatever was on Thursday one semester is on some other day of the week the following semester would help us, as his evening accelerated math class meets on Thursdays (and an occasional additional Tuesday) this year. We are looking forward to him getting heavily into this semester's online classes.", "Solution_3": "Heh... I just read Anthem (Ayn Rand) the other day, thus all mention of 'we' makes me think again.", "Solution_4": "[quote=\"tokenadult\"]I just had A+MATH click on his preferred time, which reflects our schedule in the Central Time Zone. My only other comment would be that making sure whatever was on Thursday one semester is on some other day of the week the following semester would help us, as his evening accelerated math class meets on Thursdays (and an occasional additional Tuesday) this year. We are looking forward to him getting heavily into this semester's online classes.[/quote]\r\n\r\nWe intend to move class days around in most cases although there might be instances when we do not. A small part of our scheduling is dictated by circumstances, though we're fairly flexible.", "Solution_5": "Anything but 7:00-8:30 ET is good for me. Some of my other extra-curricular activities run until 7:00, so by the time I'm home it's 7:15, so I can't make it to the 7:00 ET class.", "Solution_6": "Yeah... I wouldn't want later than to finish at 9 EST, because I also have to get up around 5:45 every morning. Thanks for the poll!", "Solution_7": "I agree with EFuzzy. Extra curriculars force me to lean toward later times. I usually stay up to at least 11 anyways and get up at 5:30 so any of those times wouldn't have much of an effect except for the earlier times which might interfere with other stuff.", "Solution_8": "Interesting how people either like the earliest time or the latest time. Seems like someones going to be unhappy no matter what time is chosen.", "Solution_9": "[quote=\"RC-7th\"]Interesting how people either like the earliest time or the latest time. Seems like someones going to be unhappy no matter what time is chosen.[/quote]\r\n\r\nThat all depends on how the future of the school develops. At some point we could run back to back classes, but we can only do that if enrollment grows large enough. Perhaps next year we will offer some classes that way.", "Solution_10": "i like 7:30-9, because by then all my other activities are over, also its not tooo late, and its not too early for west coast people.", "Solution_11": "[quote=\"cutiepi\"]i like 7:30-9, because by then all my other activities are over, also its not tooo late, and its not too early for west coast people.[/quote]\r\n\r\nAH HA!\r\n\"i didnt take any AoPS classes\" she says.\r\nits like ur ashamed to admit it or something. ;)", "Solution_12": "[quote=\"cutiepi\"]i like 7:30-9, because by then all my other activities are over, also its not tooo late, and its not too early for west coast people.[/quote]\r\n\r\nI agree. \r\n\r\nSometimes at the later times, after a long day, I'm just not up for doing problem solving. I just feel like going to bed and get frustrated a lot easier.", "Solution_13": "I voted for 9:00 to 10:30 time because in winter, I don't have any sport going on and I rather want to finish my homeworks, tests, etc before classes so I can simply attend class and go to sleep after it.\r\n\r\nI might stay some after to review some school stuffs but still, I don't want class to be starting at too early like 7:30. :D", "Solution_14": "[quote=\"white_horse_king88\"]Yeah... I wouldn't want later than to finish at 9 EST, because I also have to get up around 5:45 every morning. Thanks for the poll![/quote]\r\n\r\nI would add that especially for the middle school level classes (MATHCOUNTS and Intro classes) ending after 9 would be difficult for many people. \r\n\r\nIt's quite difficult, I'm sure, to accomodate schedules in multiple timezones! I can see how starting at 4 or 4:30 could be difficult for those on the west coast.\r\n\r\nGood luck working it all out!" } { "Tag": [ "\\/closed" ], "Problem": "How would one invert the left-right layout of the wiki as you did?", "Solution_1": "Use CSS", "Solution_2": "[quote=\"#H34N1\"]Use CSS[/quote]\r\nOh, so there isn't a php variable you can edit or anything?", "Solution_3": "There is, but its easier to just use CSS.", "Solution_4": "But what is the variable, then?", "Solution_5": "I don't know; you have to look into the monobook template." } { "Tag": [ "calculus", "derivative", "induction", "algebra unsolved", "algebra" ], "Problem": "Find all functions $f: \\mathbb R \\to \\mathbb R$ such that\n \\[ f(x)=f\\left(\\frac{x}{2}\\right)+\\frac{x}{2}f'(x)\\]\nfor all $x \\in \\mathbb R$.", "Solution_1": "So we have to solve $ f(x) \\equal{} f(\\frac x2) \\plus{} \\frac x2f'(x)$ (and $ f(x)$ is continuous and it's derivative exists all over $ \\mathbb R$)\n\nLet $ f(x)$ any solution, let $ u\\neq 0$ and let $ g(x) \\equal{} f(x) \\plus{} (f(0) \\minus{} f(u))\\frac xu \\minus{} f(0)$. Obviously, $ g(x)$ is a solution and $ g(u) \\equal{} g(0) \\equal{} 0$\nLet then $ M \\equal{} \\max_{x\\in[0,u]}g(x)$ (I write $ [0,u]$ even if $ u < 0$) and any $ x_0\\in[0,u]$ such that $ g(x_0) \\equal{} M$ ($ M$ and $ x_0$ exist since $ g(x)$ is continuous)\nIf $ M\\neq 0$, $ x_0\\neq 0$ and $ f'(x_0) \\equal{} 0$. We have then $ f(x_0) \\equal{} f(\\frac {x_0}{2}) \\equal{} M$ and so $ f'(\\frac {x_0}{2}) \\equal{} 0$. \nAn immediate induction give us $ f(\\frac {x_0}{2^n}) \\equal{} M$ and so $ M \\equal{} 0$ (since $ g(0) \\equal{} 0$ and $ g(x)$ is continuous at $ 0$).\n\nSame, Let then $ m \\equal{} \\min_{x\\in[0,u]}g(x)$ and any $ x_1\\in[0,u]$ such that $ g(x_1) \\equal{} m$. The same method implies $ m \\equal{} 0$\n\nSo $ g(x) \\equal{} 0$ $ \\forall x\\in[0,u]$. and so $ f(x) \\equal{} \\frac {f(u) \\minus{} f(0)}{u}x \\plus{} f(0)$ $ \\forall x\\in[0,u]$\n\nSo $ \\frac {f(u) \\minus{} f(0)}{u} \\equal{} \\frac {f(v) \\minus{} f(0)}{v}\\equal{}a$ $ \\forall u,v$ with same sign.\n\nSo $ f(x) \\equal{} ax \\plus{} b$ $ \\forall x\\geq 0$ and $ f(x) \\equal{} cx \\plus{} d$ $ \\forall x\\leq 0$. But continuity at $ 0$ implies $ b \\equal{} d$ and existence of $ f'(0)$ implies $ a \\equal{} c$\n\nAnd so the only solutions are $ f(x) \\equal{} ax \\plus{} b$ $ \\forall x\\in\\mathbb R$ (and we easily verify that these solutions fit)." } { "Tag": [ "function", "algebra unsolved", "algebra" ], "Problem": "Find all three continuous functions $f,g,h: R\\to R$ such that $: f(xy)=g(x).g(y)-h(x+y)+1, \\forall x,y\\in R$", "Solution_1": "It seems like solutions are infinite... for example, $f(x) = g(x) = x, h(x) = 1$, and we can also have $f(x) = g(x) = h(x) = 1$.", "Solution_2": "thanks,please,you get the solution for this problem.I think this problem is very difficult." } { "Tag": [ "inequalities", "inequalities unsolved" ], "Problem": "Given 3 possitive real $a,b,c$ satisfying $abc+a+c=b$. Find the maximum value of\r\n$P=\\frac{2}{a^2+1}-\\frac{2}{b^2+1}+\\frac{3}{c^2+1}$", "Solution_1": "If ac = 1, then a = -c and 1/(a^2 + 1) - 2/(b^2 + 1) needs to be maximized, choose a, b tending to 0, infinity to get maximum of 1.\r\n\r\nFrom now on, ac isn't 1.\r\n\r\nWe have $b = \\frac{a+c}{1-ac}$ given.\r\n\r\nTake a look at the negative term: $\\frac{2}{b^2 + 1} = \\frac{2(1-ac)^2}{(a+c)^2 + (1-ac)^2} = \\frac{2(1-ac)^2}{(a^2 + 1)(c^2 + 1)}$.\r\n\r\nSo we can get a common denominator fairly easily. Let's do that.\r\n\r\nNow:\r\n\r\n$P(a^2 + 1)(c^2 + 1) = 2(c^2 + 1) + 3(a^2 + 1) - 2(1-ac)^2 = 3 + 3a^2 + 2c^2 + 4ac -2(ac)^2$.\r\n\r\ni dont see a nice way so im just going to give up, but ya you should be able to work from there since this is only 2 vars", "Solution_2": "Hmmm I don't think it's easy from there Singular. \r\nHere is my oponion : Let $b'=-b$ so we have $a+b'+c=ab'c$ thus we can let $a=tgA,b'=tgB,c=tgC$ with $A,B,C$ are 3 angle of a triangle and $\\angle B>\\pi/2$\r\nNow we have to find max of :\r\n$\\frac{2}{tg^2A+1}-\\frac{2}{tg^2B+1}+\\frac{3}{tg^2C+1}$\r\nBut I can't sovle it yet. What do you think?", "Solution_3": "[quote=\"Stun\"]Hmmm I don't think it's easy from there Singular. \nHere is my oponion : Let $b'=-b$ so we have $a+b'+c=ab'c$ thus we can let $a=cotgA,b'=cotgB,c=cotgC$ with $A,B,C$ are 3 angle of a triangle.\nNow we have to find max of :\n$\\frac{2}{cotg^2A+1}-\\frac{2}{cotg^2B+1}+\\frac{3}{cotg^2C+1}$\nBut I can't sovle it yet. What do you think?[/quote]\r\nI think it should be $a=tanA,b'=tanB,c=tanC$", "Solution_4": "I'm sorry it's a stupid mistake . I did corect it :D .\r\nAnd now do you get any idea? (Huh!!!! I'm really bad at this section :( )", "Solution_5": "The answer is $\\frac{10}{3}$ when $a=\\frac{1}{\\sqrt{2}},b=\\sqrt{2},c=\\frac{1}{2\\sqrt{2}}$. ;)", "Solution_6": "Oh really!!! Could we see some proof for this claim :lol:", "Solution_7": "hello, by inserting $c=\\frac{b-a}{1+ab}$ in $P$ we get the following expression:\r\n$\\frac{2-2a^2+6ab+2b^2+3a^2b^2}{\\left(1+a^2\\right)\\left(1+b^2\\right)}$.\r\nNow we will show, that \r\n$\\frac{2-2a^2+6ab+2b^2+3a^2b^2}{\\left(1+a^2\\right)\\left(1+b^2\\right)} \\le \\frac{10}{3}$. But this inequality is equivalent with\r\n$0 \\le 4\\left(2a-b\\right)^2+\\left(ab-1\\right)^2$.\r\nFor $a=\\frac{1}{\\sqrt{2}}$ and $b=\\sqrt{2}$ we get equality. \r\nThis completes the proof.", "Solution_8": "source:VietnamMO and it's true for a,b,c are real" } { "Tag": [ "probability", "analytic geometry", "geometry", "similar triangles" ], "Problem": "I'm having trouble with these four problems. I know the answers but I want detailed solutions on each of these. Thanks.\r\n\r\n21. A circle with a radius of 2 units has its center at (0, 0). A circle with a radius of 7 units has its center at (15, 0). A line tangent to both circles intersects the x-axis at (x, 0) to the right of the origin. What is the value of x? Express your answer as a common fraction.\r\n[hide]Answer: 10/3[/hide]\n\n26. Kevin will start with the integers 1, 2, 3, and 4 each used exactly once and written in a row in any order. Then he will find the sum of the adjacent pairs of integers in each row to make a new row, until one integer is left. For example, if he starts with 3, 2, 1, 4, and then takes sums to get 5, 3, 5, followed by 8, 8, he ends with the final sum 16. Including all of Kevin's possible starting arrangements of the integers 1, 2, 3, and 4, how many possible final sums are there?\n[hide]Answer: 5[/hide]\n\n29. Given that x^2-5x+8=1, what is the possible value of x^4-10x^3+25x^2-9?\n[hide]Answer: 40[/hide]\n\n30. An o-Pod MP3 player stores and plays entire songs. Celeste has 10 songs stored on her o-Pod. The time length of each song is different. When the songs are ordered by length, the shortest song is only 30 seconds long and each subsequent song is 30 seconds longer than the previous song. Her favorite song is 3 minutes, 30 seconds long. The o-Pod will play all the songs in random order before repeating any song. What is the probability that she hears the first 4 minutes, 30 seconds of music-there are no pauses between songs-without hearing every second of her favorite song? Express your answer as a common fraction.\n[hide]Answer: 79/90[/hide]", "Solution_1": "Huge hints:\r\n\r\n[hide=\"21\"]Draw the circles and line, and use similar triangles.[/hide]\n\n[hide=\"26\"]Write a, b, c, and d in a row, and if you add it up, you get that the top number is $ a\\plus{}3b\\plus{}3c\\plus{}d\\equal{}a\\plus{}d\\plus{}3(b\\plus{}c)$, so you just have to find the number of possible values of $ a\\plus{}d$, since the value of $ b\\plus{}c$ will be forced from that.[/hide]\n\nThere is already a topic on 29.\n\n[hide=\"30\"]Find the probability that she [b]will[/b] be able to hear [b]all[/b] of her favorite song (it's really not that complicated) and subtract it from 1. It might help to make a list all of the song lengths in seconds.[/hide]", "Solution_2": "I don't get using similar triangles for 21.\r\n\r\nI tried to, as well as finding the internal tangent's length as 12, but I still couldn't get it.", "Solution_3": "[hide=\"30\"]I spent like 30 seconds on #30 in the middle of the test and thought it was too easy. But I was right so yay.\n\nWe only need to worry about the first minute of music that plays.\n\nThere is a $ \\frac{1}{10}$ chance that the first minute is the song one minute long. Then the next song played must be one of the 8 other choices that is not her favorite song.\n\n$ \\frac{1}{10}*\\frac{8}{9}\\equal{}\\frac{8}{90}$\n\nAnother possibility is that the first song is 30 seconds long, $ \\frac{1}{10}$ of a chance.\n\nThen the next song can be any other song except her favorite song, $ \\frac{8}{9}$, giving us the same probability, $ \\frac{8}{90}$.\n\nNow the final probability is if the first song is any song except her favorite song and the two other songs, $ \\frac{7}{10}$.\n\nAdd these up to get $ \\boxed{\\frac{79}{90}}$[/hide]\n\n[hide=\"21\"]Let the center of the circle with radius 2 be $ C_2$ and the center of the other circle be $ C_7$\nLet the point tangent to the circle radius 2 be $ t_2$ and similarly denote it for the other circle.\nNow let the point with coordinate $ (x,0)$ be $ A$.\n\nThen $ \\bigtriangleup C_2t_2A\\sim \\bigtriangleup C_7t_7A$ because by vertical angles, $ \\angle C_2At_2\\equal{}\\angle C_7At_7$ and the tangent line is perpendicular to the radius at those points.\n\nThus $ \\frac{C_2t_2}{C_7t_7}\\equal{}\\frac{C_2A}{C_7A}$\n$ \\frac{2}{7}\\equal{}\\frac{x}{15\\minus{}x}$\n$ 7x\\equal{}30\\minus{}2x$\n$ x\\equal{}\\boxed{\\frac{10}{3}}$[/hide]\n[hide=\"26\"]\nConsider the arrangement $ a,b,c,d$\nThen the next row will be $ a\\plus{}b,b\\plus{}c,c\\plus{}d$\nThen the next row will be $ a\\plus{}2b\\plus{}c,b\\plus{}2c\\plus{}d$\nAnd the last row will be $ a\\plus{}3b\\plus{}3c\\plus{}d$\n\nThus, we only need to worry about the middle two numbers.\n\nThere are 4 numbers, and thus $ \\binom{4}{2}\\equal{}6$ ways to arrange them.\n\nBut $ 1\\plus{}4\\equal{}2\\plus{}3$, which is the overcounted because $ a\\plus{}d$ as well as $ b\\plus{}c$ is the same.\n\nThus there are $ \\boxed{5}$ sums.[/hide]", "Solution_4": "[hide=\"a much easier way for 26\"]\n\nIt can only be even numbers, after a little effort, you find 16 is lowest possible and 24 is highest, that is 5 even numbers between 16 and 24 inclusive.[/hide]", "Solution_5": "Ok. I get the first three, but I still don't get #30. Any faster ways?", "Solution_6": "For thirty, I counted the ways she DOES hear every second of her favorite song and then subtracted from 1. \r\n\r\nFirst off, since her favorite song is 3:30 long, the song must start within the first minute for here to hear all of it. So you only have to consider the first two songs that the iPod chooses.\r\n\r\nCase 1: iPod chooses her favorite song first and then any other song : $ \\frac{1}{10}\\cdot{1}\\equal{}\\frac{1}{10}$\r\n\r\nCase 2: iPod chooses the 30 sec or 1 minute song and then her favorite song: $ \\frac{2}{10}\\cdot\\frac{1}{9}\\equal{}\\frac{1}{45}$\r\n\r\n$ \\frac{1}{45}\\plus{}\\frac{1}{10}\\equal{}\\frac{11}{90}$\r\n\r\n$ 1\\minus{}\\frac{11}{90}\\equal{}\\boxed{\\frac{79}{90}}$", "Solution_7": "ok I get it now-thx a lot! :D", "Solution_8": "Sorry, but I still don't get #30. Could someone please explain it in greater detail? Thanks!", "Solution_9": "[quote=\"tinytim\"]For thirty, I counted the ways she DOES hear every second of her favorite song and then subtracted from 1. \n\nFirst off, since her favorite song is 3:30 long, the song must start within the first minute for here to hear all of it. So you only have to consider the first two songs that the iPod chooses.\n\nCase 1: iPod chooses her favorite song first and then any other song : $ \\frac {1}{10}\\cdot{1} \\equal{} \\frac {1}{10}$\n\nCase 2: iPod chooses the 30 sec or 1 minute song and then her favorite song: $ \\frac {2}{10}\\cdot\\frac {1}{9} \\equal{} \\frac {1}{45}$\n\n$ \\frac {1}{45} \\plus{} \\frac {1}{10} \\equal{} \\frac {11}{90}$\n\n$ 1 \\minus{} \\frac {11}{90} \\equal{} \\boxed{\\frac {79}{90}}$[/quote]\r\n\r\nIt's [b]o-pod[/b], not ipod. :P", "Solution_10": "[quote=\"314.math\"]Sorry, but I still don't get #30. Could someone please explain it in greater detail? Thanks![/quote]\r\nI'll try expanding upon a previous explanation.\r\n[hide]\nIf the very first song is Celeste's favorite, she gets to hear it all. Since there are 10 songs, the chance that her song plays first is $ \\frac{1}{10}.$\n\nOtherwise, in order for Celete's favorite song to play before the end of 4 minutes 30 seconds, the first song must be 1 minute or less. There are two such choices -- the 30 second song and the 1 minute song. If one of these is first, then Celeste's song must be second. The combined probability is the product of these two events: $ \\frac{2}{10}\\times\\frac{1}{9}\\equal{}\\frac{1}{45}.$\n\nEven if the two shortest songs played first, and Celeste's favorite played third, she won't hear every second of her favorite song. So the previous two ways are the only ways in which Celeste will hear every second of her favorite song. Adding the above... $ \\frac{1}{10}\\plus{}\\frac{1}{45}\\equal{} \\frac{9}{90}\\plus{}\\frac{2}{90}\\equal{} \\frac{11}{90}.$\n\nBut since we want the chance she [b]won't[/b] get to hear every second, we subtract from 1:\n$ 1\\minus{}\\frac{11}{90}\\equal{}\\boxed{\\frac{79}{90}}.$[/hide]", "Solution_11": "#26:\nIf Kevin's original four numbers are $a$, $b$, $c$, and $d$, then here are the numbers that he writes down:\n\\[\n\\begin{array}{ccccccc}\na & & b & & c & & d \\\\\n& a + b & & b + c & & c + d & \\\\\n& & a + 2b + c & & b + 2c + d & & \\\\\n& & & a + 3b + 3c + d & & &\n\\end{array}\n\\]\n\nSince Kevin always starts with the numbers 1, 2, 3, and 4 (in some order), we have that $a + b + c + d = 10$. Then Kevin's final number is\n\\[a + 3b + 3c + d = 2(b + c) + (a + b + c + d) = 2(b + c) + 10.\\]\nThe possible values of $b + c$ are 3, 4, 5, 6, and 7, so there are $\\boxed{5}$ possible final sums.", "Solution_12": "[quote=Max0815]#26:\nIf Kevin's original four numbers are $a$, $b$, $c$, and $d$, then here are the numbers that he writes down:\n\\[\n\\begin{array}{ccccccc}\na & & b & & c & & d \\\\\n& a + b & & b + c & & c + d & \\\\\n& & a + 2b + c & & b + 2c + d & & \\\\\n& & & a + 3b + 3c + d & & &\n\\end{array}\n\\]\n\nSince Kevin always starts with the numbers 1, 2, 3, and 4 (in some order), we have that $a + b + c + d = 10$. Then Kevin's final number is\n\\[a + 3b + 3c + d = 2(b + c) + (a + b + c + d) = 2(b + c) + 10.\\]\nThe possible values of $b + c$ are 3, 4, 5, 6, and 7, so there are $\\boxed{5}$ possible final sums.[/quote]\n\nlol gotta love that 9.5 year bump.... :)", "Solution_13": "ik, right?", "Solution_14": "Lol that is hilarious.... I bet he/she would look back at this forum and be like \"gee that bump lol xD troll\"", "Solution_15": "I love trolling.......I did that on several fourums..... xD\nthey probably were like...... ( -_-) bro....no", "Solution_16": "There seems to be solutions for everything but 29, so:\n[hide=Solution for 29]\nSince x^2-5x+8=1, then x^2-5x=-7. If we square both sides of the equation and simplify, we have x^4-10x^3+25x^2=49, so x^4-10x^3+25x^2-9=49-9=40.\n[hide]", "Solution_17": "for problems like 29, they are all engineered a special way, otherwise it would be too tedious. As my father always said, \"Find the grand truth.\" Translation: all competition problems are engineered specially, so translate the problem into a easier problem." } { "Tag": [], "Problem": "World is in financial crise, most of countries are in recession. For 2008, an estimated 2.6 million U.S. jobs were eliminated. In Russia only in december 2008 1.0 million jobs were eliminated. Learn how to survive and play virtual stock exchange trading with stocks and goods. Only the best wins!\r\n\r\n\r\nhttp://stockexchange.cjb.net\r\n\r\n\r\nSee rankings:\r\n\r\n\r\n[img]http://i88.servimg.com/u/f88/11/28/19/17/most_s10.jpg[/img]", "Solution_1": "Advertising is bad :wink:\r\n\r\nAnyway, I think that [url]updown.com[/url] is better, even though I haven't tried yours :D\r\n\r\nMod delete." } { "Tag": [ "function", "algebra", "polynomial", "calculus", "Germany" ], "Problem": "Find all functions $ f: \\mathbb{R} \\mapsto \\mathbb{R}$ such that $ \\forall x,y,z \\in \\mathbb{R}$ we have: If\r\n\\[ x^3 \\plus{} f(y) \\cdot x \\plus{} f(z) \\equal{} 0,\\]\r\nthen\r\n\\[ f(x)^3 \\plus{} y \\cdot f(x) \\plus{} z \\equal{} 0.\\]", "Solution_1": "Note first that the polynomial $ z^3 \\plus{} Az \\plus{} B \\equal{} 0$ has at least one real solution for any real $ A,B$, and this solution is unique when $ A\\geq0$ (the derivative of the polynomial never changes sign), so the function is surjective; indeed, for any real $ r$ take $ y > 0$ and $ z \\equal{} \\minus{} r^3 \\minus{} yr$, then $ x^3 \\plus{} xf(y) \\plus{} f(z) \\equal{} 0$ has at least one root whose image satisfies $ f(x)^3 \\plus{} yf(x) \\plus{} z \\equal{} 0$, with unique solution $ f(x) \\equal{} r$.\r\n\r\nTake now any nonzero $ x$, since $ f$ is surjective we may select $ y$ such that $ f(y) \\equal{} \\minus{} x^2 \\minus{} \\frac {f(x)}{x}$, or $ f(x)^3 \\plus{} yf(x) \\plus{} x \\equal{} 0$, and $ f(x)$ may not be zero. Since $ f$ is surjective, then $ f(0) \\equal{} 0$.\r\n\r\nTake now $ y \\equal{} f(y) \\equal{} 0$ and $ x \\equal{} \\minus{} \\sqrt [3]{f(z)}$. Then, $ z \\equal{} \\minus{} f(x)^3$ is uniquely determined by $ f(z)$, and $ f$ is injective. Moreover, taking $ z \\equal{} f(z) \\equal{} 0$ and $ f(y) \\equal{} \\minus{} r^2 < 0$ any negative real, then $ \\pm r$ are roots of $ x^3 \\plus{} f(y)x \\plus{} f(z) \\equal{} 0$, and $ \\pm f(r)$ are the roots of $ f(x)^2 \\equal{} \\minus{} y$; this proves two things: first, if $ f(y) < 0$, then $ y < 0$, and second, $ |f(r)| \\equal{} |f( \\minus{} r)|$ for any real $ r$, and since $ f$ is injective, then $ f(r) \\equal{} \\minus{} f( \\minus{} r) > 0$ for any positive real $ r$.\r\n\r\nFor any reals $ u,v$, take $ f(y) \\equal{} \\minus{} u^2 \\minus{} uv \\minus{} v^2 \\equal{} uv \\minus{} (u \\plus{} v)^2$ and $ f(z) \\equal{} uv(u \\plus{} v)$. Clearly $ x \\equal{} u,v, \\minus{} u \\minus{} v$ are the roots of $ x^3 \\plus{} f(y)x \\plus{} f(z) \\equal{} 0$, or $ f(u),f(v),f( \\minus{} u \\minus{} v) \\equal{} \\minus{} f(u \\plus{} v)$ are the roots of $ f(x)^3 \\plus{} yf(x) \\plus{} z \\equal{} 0$, and by Cardano, $ f(u \\plus{} v) \\equal{} f(u) \\plus{} f(v)$, as well as $ y \\equal{} f(u)f(v) \\minus{} f(u)f(u \\plus{} v) \\minus{} f(v)f(u \\plus{} v) \\equal{} \\minus{} f(u)^2 \\minus{} f(u)f(v) \\minus{} f(v)^2 \\equal{} f(u)f(v) \\minus{} f(u \\plus{} v)^2$ and $ z \\equal{} f(u)f(v)f(u \\plus{} v)$.\r\n\r\nTake now $ f(y) \\equal{} \\minus{} 1$, $ z \\equal{} 0$, $ x \\equal{} 1$. Clearly, $ y \\equal{} \\minus{} f(1)^2$, or $ 1 \\equal{} f(f(1)^2)$. Take also $ f(z) \\equal{} \\minus{} 1$, $ y \\equal{} 0$, $ x \\equal{} 1$. Clearly $ z \\equal{} \\minus{} f(1)^3$, or $ 1 \\equal{} f(f(1)^3)$. Since $ f$ is injective and $ f(0) \\equal{} 0$, then $ f(1)^3 \\equal{} f(1)^2$ or $ f(1) \\equal{} 1$.\r\n\r\nTake now in the previous result $ v \\equal{} 1$, then $ v \\equal{} \\minus{} 1$, resulting in $ f(y) \\equal{} \\minus{} u^2 \\minus{} u \\minus{} 1$ when $ y \\equal{} \\minus{} f(u)^2 \\minus{} f(u) \\minus{} 1$ and $ f(y') \\equal{} \\minus{} u^2 \\plus{} u \\minus{} 1$ when $ y' \\equal{} \\minus{} f(u)^2 \\plus{} f(u) \\minus{} 1$. Then $ f(2u)\\equal{}f(y'\\minus{}y)\\equal{}f(y')\\minus{}f(y)\\equal{}2u$ for all real $ u$, done!", "Solution_2": "As the equation $ x^{3} \\plus{} ax \\plus{} b \\equal{} 0$ always has at least one real solution (the solution is unique if $ a$ is nonnegative), we easily conclude the surjectivity of $ f$. (For any real $ m$, set $ y \\equal{} 1$, $ z \\equal{} \\minus{} m(m^{2} \\plus{} 1)$ and $ x$ such that $ x^{3} \\plus{} xf(y) \\plus{} f(z) \\equal{} 0$. Than from the given condition we have $ f(x) \\equal{} m$). If we take $ t$ such that $ f(t) \\equal{} 0$ and set $ (x,y,z) \\equal{} (0,a,t)$ in condition, we get that $ af(0)$ is fixed, which means $ f(0) \\equal{} 0$ and from that we get $ t \\equal{} 0$. It means that for any real $ x$, different from $ 0$, $ f(x)$ is also different from $ 0$. Assume there are some different $ a,b$ such that $ f(a) \\equal{} f(b)$. Than, plugging any $ z$, different from $ 0$ and such $ x$, that $ x^{3} \\plus{} xf(a) \\plus{} f(z) \\equal{} 0 \\equal{} x^{3} \\plus{} xf(b) \\plus{} f(z) \\equal{} 0$, the condition gives $ af(x) \\equal{} bf(x)$, but $ (a \\minus{} b)$ can't be $ 0$. So $ f(x) \\equal{} 0$, but this imply $ x \\equal{} 0$ and $ z \\equal{} 0$, which contradicts the assumption. Hence, $ f$ is injective. As $ f$ is surjective, we can easily prove that the equalities $ f(x)^{3} \\plus{} yf(x) \\plus{} z \\equal{} 0$ and $ x^{3} \\plus{} xf(y) \\plus{} f(z) \\equal{} 0$ are equivalent. That means that $ f( \\minus{} f(x)^{3} \\minus{} yf(x)) \\equal{} \\minus{} x^{3} \\minus{} xf(y)$. (1)\r\n When $ x \\equal{} 1$, we have: $ f(y) \\plus{} f(z) \\equal{} \\minus{} 1$ iff $ y \\plus{} z \\plus{} 1 \\equal{} 0$. So, $ f( \\minus{} y \\minus{} 1) \\equal{} \\minus{} f(y) \\minus{} 1$.(2) If $ y$ is such that $ f(y) \\equal{} \\minus{} 1$, than plugging in the last condition $ z \\equal{} 0$, we get $ y \\equal{} \\minus{} 1$.(3) Setting $ x \\equal{} \\minus{} 1$ in (1), we get: $ f(y \\plus{} 1) \\equal{} f(y) \\plus{} 1$. (4). It means that for all integers $ m$, $ f(x \\plus{} m) \\equal{} m \\plus{} f(x)$. (*) From (2) and (4) we conclude that $ f( \\minus{} x) \\equal{} \\minus{} f(x)$, for all $ x$ and as $ f(0) \\equal{} 0$, from $ (4)$ we get $ f(m) \\equal{} m$, for all integer values of $ m$. Substituting $ x$ by $ ( \\minus{} x)$ in (1), we get: $ f(f(x)^{3} \\plus{} yf(x)) \\equal{} x^{3} \\plus{} xf(y)$ (5). If $ x$ is integer in (5), we have \r\n$ x^{3} \\plus{} xf(y) \\equal{} f(x^{3} \\plus{} xy) \\equal{} x^{3} \\plus{} f(xy)$. Hence, for all integers $ m$: $ f(mx) \\equal{} mf(x)$. (6). If $ x \\equal{} \\frac {n}{m}$, where $ n,m$ are integers ($ m > 0$), (6) yields $ f(\\frac {n}{m}) \\equal{} \\frac {n}{m}$, which means that $ f(q) \\equal{} q$ for all rational $ q$. \r\n When $ y \\equal{} \\minus{} f(x)^{2}$, (5) leads to $ f( \\minus{} f(x)^{2}) \\equal{} \\minus{} x^{2}$, but $ \\minus{} f(x)^{2}$ can get any negative value. It means that for all $ x < 0$, we have $ f(x) < 0$, but as $ f$ is odd, we have $ f(x) > 0$ when $ x > 0$. From (6) and (*), we get: $ f(x \\plus{} \\frac {m}{n}) \\equal{} \\frac {f(nx \\plus{} m)}{n} \\equal{} \\frac {nf(x) \\plus{} m}{n} \\equal{} f(x) \\plus{} \\frac {m}{n}$. It means that for all rational $ q$, we have $ f(x \\plus{} q) \\equal{} f(x) \\plus{} q$. (**).\r\n Assume there exists such $ x$ that $ f(x) < x$. It's clear that there is such rational number $ q$ that $ f(x) < q < x$. In this case, $ f(x \\minus{} q) > 0$ and $ q > f(x) \\equal{} f(q \\plus{} (x \\minus{} q)) \\equal{} q \\plus{} f(x \\minus{} q) > q$, which is impossible. We will get the similar contradiction if we assume that $ f(x) > x$ holds for some real number $ x$. \r\n Finally, $ f(x)$ can't be neither bigger, nor smaller than $ x$, for any $ x$. So, $ f(x) \\equal{} x$ holds for any $ x$. It's easy to verify that the function $ f(x) \\equal{} x$ indeed satisfies the given conditions. Lasha Lakirbaia :)", "Solution_3": "[i]Yes , i have checked Lasha's solution and it's really a correct solution\n\n I don't have time , can anybody check Daniel's solution ?[/i] :lol:", "Solution_4": "Wow!! A wonderful problem!!", "Solution_5": "[quote=Germany TST 2009/3]Find all functions $ f: \\mathbb{R} \\mapsto \\mathbb{R}$ such that $ \\forall x,y,z \\in \\mathbb{R}$ we have: If\n\\[ x^3 \\plus{} f(y) \\cdot x \\plus{} f(z) \\equal{} 0,\\]\nthen\n\\[ f(x)^3 \\plus{} y \\cdot f(x) \\plus{} z \\equal{} 0.\\][/quote]\n\n\nNice problem :)\n\n[hide=solution] Answer: The only function which works is the identity. \n\nIt is clear to check that it works. I prove that no other function does, so for the sake of argument, let $f$ be a function which satisfies the given condition. Henceforth, we will refer to a triple $(x,y,z)$ satisfying $x^3+f(y)x+f(z)=0$ as [i]valid.[/i] We will use subtly, the fact that any cubic real polynomial has at least one real root.\n\n[b]Claim 1:[/b] There exists a real number $x_0$ such that $f(x_0)=0$.\n\n(Proof) Let $x_0$ be a real solution of $x^3+xf(0)+f(0)=0$. Evidently, $(x_0,0,0)$ is valid so $f(x_0)^3=0$ so our claim holds. $\\square$\n\n[b]Claim 2:[/b] $f(0)=0$. \n\n(Proof) Note that $(0, y, x_0)$ is valid where $x_0$ satisfies $f(x_0)=0$ so $f(0)^3+yf(0)+x_0=0$ for all reals $y$ which yields $f(0)=0$. $\\square$\n\n[b]Claim 3:[/b] $f$ is injective.\n\n(Proof) Let $f(a)=f(b)$ and take $x, y$ as real numbers for which $(x, y, a)$ is valid. Consequently, $(x,y,b)$ is also valid, so $$f(x)^3+yf(x)+a=f(x)^3+yf(x)+b=0 \\Longrightarrow a=b$$ so injectivity follows. $\\square$\n\nNow that we have proved the easy claims, it's time to make bolder ones.\n\n[b]Claim 4:[/b] $f$ is surjective. \n\n(Proof) Fix an arbitrary $t \\in \\mathbb{R}$. Consider the polynomial $$P(X)=(X-t)\\cdot \\left(X^2+tX+(1+t^2)\\right).$$ Set $z_0=-t(1+t^2)$ and $y_0=1$. Take $x_0$ to be a real number for which $(x_0, y_0, z_0)$ is valid and observe that $$P(x_0)=f(x_0)^3+y_0f(x_0)+z_0=0.$$ Since $P$ has exactly one real root $t$ we conclude $f(x_0)=t,$ establishing the claim. $\\square$\n\n[b]Claim 5:[/b] For all $y \\ge 0$ we have $f(y) \\ge 0$ as well.\n\n(Proof) For the sake of contradiction, let $y_0>0$ be a real number for which $f(y_0)<0$. Let $x_0$ be a solution to $x^2=-f(y)$ and see that $(x_0, y_0, 0)$ is a valid triple so $f(x_0)^3+yf(x_0)=0$ which fails to occur since $x_0 \\ne 0 \\Longrightarrow f(x_0) \\ne 0$ by injectivity and $f(x_0)^2+y_0>0$. Our initial assertion fails so we get a contradiction! $\\square$\n\n[b]Claim 6:[/b] $f$ is additive.\n\n(Proof) Fix any three distinct real numbers $a, b, c$ such that $a+b+c=0$. Note that $f$ is surjective so we can choose real numbers $y_0, z_0$ such that $a, b, c$ are the roots of the polynomial $f(X)=X^3+Xf(y)+f(z)=0$. Evidently, $f(a), f(b), f(c)$ are distinct real numbers (since $a, b, c$ are distinct), which happen to be the roots of the equation $X^3+yX+z=0$. It follows that $f(a)+f(b)+f(c)=0$ so for all real numbers $a, b$ we get $$f(-(a+b))=-(f(a)+f(b)).$$ Put $b=0$ to get $f(-x)=-f(x)$ for all $x$, yielding $f(a+b)=f(a)+f(b)$ for all distinct reals $a, b$. For $a=b$ simply consider $$f(2a)=f(a+m)+f(a-m)=(f(a)+f(m))+(f(a)-f(m))=2f(a)$$ where $m$ is a sufficiently large number. Thus, $f$ is completely additive. $\\square$\n\n[b]Claim 7:[/b] $f$ is the identity function.\n\n(Proof) By [b]Claim 5[/b] and [b]Claim 6[/b] it is clear that $f$ is additive and bounded on an interval, so $f$ is linear. Since $f(0)=0$ we get $f(x)=cx$ for all reals $x$. Notice that $c \\ne 0$ so we get from a valid triple $(1,y,z)$ that $z=\\left(\\frac{-1}{c}-y\\right)=-c^3-cy$. This clearly fails to hold for $y \\rightarrow \\infty$ unless $c=1$ in which case we are done! $\\square$\n\nThe End.\n\n\n[/hide]\n\n\n" } { "Tag": [ "logarithms", "real analysis", "real analysis unsolved" ], "Problem": "1) Find a sequence $ \\{a_k\\}_{k\\ge1}$ such that $ a_k\\ne 0,1$ and $ \\sum_{k \\equal{} 1}{1\\over a_k}$ converges but $ \\sum_{k \\equal{} 1}{1\\over a_k \\minus{} 1}$ diverges\r\n\r\n2) Find a conditionally convergent sequence $ \\sum a_k$ such that $ \\sum \\vert a_k\\vert^c \\cdot sgn(a_k)$ diverges \r\nfor any $ c > 0,$ $ c\\ne1.$", "Solution_1": "For Problem 1, I think the following sequence works: $ \\sqrt{2}$, $ \\minus{}\\sqrt{2}$, $ \\sqrt{3}$, $ \\minus{}\\sqrt{3}$, $ \\sqrt{4}$, $ \\minus{}\\sqrt{4}$, and so on. The first sum is an alternating sequence with decreasing terms that tend to zero, so by somebody's theorem, the series converges. (Or we can just see that its partial sums hover around zero.) For the second sum, we can group the terms into alternating pairs to see that the series diverges. \r\n\r\nAnother sequence that works for Problem 1 is $ a_n \\equal{} (\\minus{}1)^n n^{1/2}$. The proof is similar.", "Solution_2": "For the second problem:\r\n\r\n$ (a_k)\\equal{}\\frac2{\\ln 2},\\frac{\\minus{}1}{\\ln 2},\\frac{\\minus{}1}{\\ln 2},\\frac2{\\ln 3},\\frac{\\minus{}1}{\\ln 3},\\frac{\\minus{}1}{\\ln 3},\\frac2{\\ln 4},\\dots.$" } { "Tag": [], "Problem": "I have a quick question somebody can hopefully clarify for me. \r\n\r\nIf there is a contest that runs every hour for 40 days. I enter 30 entries during one hour of a each day for 40 days what are my odds of winning during the course of 40 days?", "Solution_1": "This is a math question. Moving to math forum.", "Solution_2": "what kind of contest?", "Solution_3": "[quote=\"peeta\"]what kind of contest?[/quote]\r\n\r\nTo win an xbox 360. \r\n\r\nThis guy believes that if I enter 30 a day and on average there are 2000 entrie so 30/2000 are mine that after 40 days my chances of winning are 40%", "Solution_4": "You have posted the same spamming message in different forums:\r\n\r\n :spam:\r\n\r\n[url=http://www.mathlinks.ro/Forum/viewtopic.php?t=55546]http://www.mathlinks.ro/Forum/viewtopic.php?t=55546[/url]\r\n[url=http://www.mathlinks.ro/Forum/viewtopic.php?t=55547]http://www.mathlinks.ro/Forum/viewtopic.php?t=55547[/url]\r\n[url=http://www.mathlinks.ro/Forum/viewtopic.php?t=55549]http://www.mathlinks.ro/Forum/viewtopic.php?t=55549[/url]\r\n\r\nI urgently ask you to stop. Thank you.", "Solution_5": "[quote=\"10000th User\"]You have posted the same spamming message in different forums:\n\n :spam:\n\n[url=http://www.mathlinks.ro/Forum/viewtopic.php?t=55546]http://www.mathlinks.ro/Forum/viewtopic.php?t=55546[/url]\n[url=http://www.mathlinks.ro/Forum/viewtopic.php?t=55547]http://www.mathlinks.ro/Forum/viewtopic.php?t=55547[/url]\n[url=http://www.mathlinks.ro/Forum/viewtopic.php?t=55549]http://www.mathlinks.ro/Forum/viewtopic.php?t=55547[/url]\n\nI urgently ask you to stop. Thank you.[/quote]\r\n\r\nAlright but answer the question please.", "Solution_6": "If a billion other people do the exact same thing, your chances would be one in a billion (and one). If you're the only person trying, your chances would be one in one.\r\nObviously, more information is needed. The best you could probably give are many assumptions that may not be based on fact, so it might not even be possible to calculate it. Would the odds be stated somewhere in the contest rules, like the fine print, or something like that?", "Solution_7": "[quote=\"Bictor717\"]If a billion other people do the exact same thing, your chances would be one in a billion (and one). If you're the only person trying, your chances would be one in one.\nObviously, more information is needed. The best you could probably give are many assumptions that may not be based on fact, so it might not even be possible to calculate it. Would the odds be stated somewhere in the contest rules, like the fine print, or something like that?[/quote]\r\n\r\n\r\nI'll do my best to explain this fully.\r\n\r\n\r\nOk there is a contest and they are giving away 1 xbox every hour for 40 days. In total there are like 900+ xbox's to give out. On average there are 3000 entries for every contest. \r\n\r\nYou put in 30 entries outta 3000 once a day for 40 days what are you odds of winning overall." } { "Tag": [], "Problem": "solve in integers the equation:\r\n$2^{(x+1)^{2}}+2^{(y+1)^{2}}=x+y$.", "Solution_1": "Any typo? There is no solution of course." } { "Tag": [ "inequalities", "inequalities unsolved" ], "Problem": "let x,y,z be positive real numbers.\r\nfind the minimum value of (x+y+z) \\sum \\sqrt (x 2 +xy+y 2 ) -1", "Solution_1": "[quote=\"wpolly\"]Let $ x,y,z$ be nonnegative real numbers such that $ yz \\plus{} zx \\plus{} xy > 0.$ \n\nFind the minimum value of $ \\sum{\\frac {x \\plus{} y \\plus{} z}{\\sqrt {y^2 \\plus{} yz \\plus{} z^2 }}}.$[/quote]$ \\sum{\\frac {1}{\\sqrt {y^2 \\plus{} z^2 \\plus{} yz}}}\\geq\\sum{\\frac {y^2 \\plus{} z^2 \\plus{} 2\\left(\\sqrt {3} \\minus{} 1\\right)yz}{(y \\plus{} z)\\left(y^2 \\plus{} z^2 \\plus{} yz\\right)}}\\geq\\cfrac{4 \\plus{} \\cfrac{2}{\\sqrt {3}}}{x \\plus{} y \\plus{} z},$\r\n\r\nwith equality if $ x \\equal{} y \\minus{} z \\equal{} 0.$" } { "Tag": [ "probability", "algebra", "polynomial", "geometry", "rectangle", "arithmetic sequence" ], "Problem": "Hello Everyone,\r\n\r\nSince marathons are such a great way to prepare for competitions and exams, I have decided to start one!\r\n\r\nThe following are the rules:\r\n1) If you are deriving the problems from previous competitions, then state the competition name, year, and question number. \r\n2) Provide a [i]full[/i]solution to your answers (extremely trivial steps may be skipped but it is good to include them). It is easier for people to understand your steps like that.\r\n3) Read the guidelines for posting problems in this forum and follow them.\r\n4) Post a problem when you submit a solution. This will keep the marathon moving.\r\n5) You may use calculators only on problems labeled as \"brute force\". \r\n6) You must simplify all fractions except those labeled as \"brute force\".\r\nThese are all basically the guidelines you find in other marathons so there should be no problem in following them.\r\n\r\nProblem:\r\n[b]Brute Force[/b]\r\nWhat is the value of $ x$ in the equation $ \\frac{97x}{83}\\equal{}\\frac{727}{863}$?", "Solution_1": "I'm just wondering here, I may not know, but to me problems like the one you just posted will not serve any purpose whatsoever except:\r\nA. Calculator training\r\nB. SIMPLE linear equations", "Solution_2": "Yes I do believe we should refrain from posting these simple problems, as they don't belong in this forum. But anyway... here's the answer and some new problems to work on. \r\n\r\n97x / 83 = 727/863 \r\n\r\n97x times 863 = 727 times 83 \r\n\r\n83711x = 60341 \r\n\r\nx = 60341/83711\r\n\r\nMy next problems \r\n\r\nAn easy one: \r\n\r\nLet a, b, c, d, and e be five consecutive terms in an arithmetic sequence and suppose that a + b+ c + d + e = 30. Which of the five terms can we find? \r\n\r\n\r\nA harder question: \r\n\r\nIntegers a, b, c, and d not necessarily distinct, are chosen independently and at random from 0 to 2007 inclusive. What is the probability that ad-bc is even?", "Solution_3": "Easy:\r\nYou can let $ x \\equal{} c$. And rewrite the equation given the fact that they are in an arithmetic sequence:\r\n\r\n$ x \\minus{} 2d \\plus{} x \\minus{} d \\plus{} x \\plus{} x \\plus{} d \\plus{} x \\plus{} 2d \\equal{} 30 \\\\\r\n5x \\equal{} 30\\\\\r\nx \\equal{} 5$\r\n\r\nSo $ \\fbox{c}$ can be found.", "Solution_4": "[hide=\"Harder question(Solution)\"]\nFirst,\nodd * odd = odd\neven * odd = even\neven * even = even\n\nodd - odd = even\nodd - even = odd\neven - odd = odd\neven - even = even\n\nO = Odd\nE = Even\nPossible even out comes:\nO*E - O*E (number of outcomes O*E*O*E = $ 1004^4$)\nO*E - E*O (number of outcomes ... = $ 1004^4$)\nE*O - O*E (number of outcomes ... = $ 1004^4$)\nE*O - E*O (number of outcomes ... = $ 1004^4$)\nO*E - E*E (number of outcomes ... = $ 1004^4$)\nE*O - E*E (number of outcomes ... = $ 1004^4$)\nE*E - O*E (number of outcomes ... = $ 1004^4$)\nE*E - E*O (number of outcomes ... = $ 1004^4$)\nE*E - E*E (number of outcomes ... = $ 1004^4$)\nO*O - O*O (number of outcomes ... = $ 1004^4$)\n\nTotal odd numbers from 0-2007: 1004\nTotal even numbers from 0-2007: 1004 \nTotal outcomes: $ 2008^4$\nProbability: $ \\dfrac{10*1004^4}{2^4*1004^4}\\equal{}\\dfrac{10}{2^4}\\equal{}\\dfrac{5}{8}$\nProbability I am wrong: $ 60$%\n[/hide]\r\n\r\nQuestion(Easy):\r\nThe Real Numbers: $ a\\not\\equal{}0$ and $ b\\not\\equal{}0$ have a specification such that $ ab\\equal{}a\\minus{}b$\uf02d. \r\n\r\nWhat is the value of $ \\dfrac{a}{b}\\plus{}\\dfrac{b}{a}\\minus{}ab$?\r\n\r\nQuestion(hard):\r\nSimplify the following Polinomial\r\n$ P(n)\\equal{}(2\\plus{}1)(2^2\\plus{}1)(2^4\\plus{}1)(2^8\\plus{}1)\\cdots(2^{2^n}\\plus{}1)$", "Solution_5": "[hide=\"Easier\"]\n$ \\frac{a}{b}\\plus{}\\frac{b}{a}\\minus{}ab\\equal{}\\frac{a^2\\plus{}b^2}{a\\minus{}b}\\minus{}(a\\minus{}b)\\equal{}\\frac{a^2\\plus{}b^2\\minus{}a^2\\minus{}b^2\\plus{}2ab}{ab}\\equal{}\\frac{2ab}{ab}\\equal{}2$\n[/hide]", "Solution_6": "[hide]\n\nFor the easier problem: \n\n(a^2 + b^2)/a-b - (a-b) \n\n= (a^2 + b^2 - (a-b)^2) / (a-b) \n\nThen you expand and combine terms on the numerator to get: \n\n2ab/(a-b) which is 2ab/ab = 2 [/hide]", "Solution_7": "Isn't that what I wrote?", "Solution_8": "Yes it is.\r\n[hide=\"Hint to harder question\"]\n$ (2 \\minus{} 1)P(n)$\n[/hide]\r\n\r\nAnother question:\r\nWhat digit does the number $ 2^{2008} \\plus{} 2^{2009}$ end in?", "Solution_9": "[hide]\n\nThis post addresses the problem posted earlier. \n\ntonypr: your answer is correct, but your method is somewhat too complicated \n\nYou can have ad and bc both odd or both even \n\nIf you have them both odd, then both a, d, b, and c must all be odd. The probability of this happening is (1/4) ^4 = 1/16. \n\nThen if you want ad and bc to be even, you just have to subtract the probability that both ad and bc are odd. This is equal to 1 - (1/4) which is 3/4. Squaring 3/4 (we're considering both ad and bc) yields 9/16. \n\n9/16 + 1/16 = 10/16 = 5/8. [/hide]", "Solution_10": "[hide]\n\n2^2008 ends in a 6. \n2^2009 ends in a 2. \n\n6+2 = 8[/hide]", "Solution_11": "$ P(n)\\equal{}(2\\minus{}1)P(n)\\equal{}2^{2^{n\\plus{}1}}\\minus{}1$ after doing $ a^2\\minus{}b^2\\equal{}(a\\plus{}b)(a\\minus{}b)$\r\n\r\nHarder question:\r\nIf we divide a square with 8 lines, 4 parallel to 2 sides, and other 4 to other 2 sides, then if there are 4 squares among 25 little rectangles, there's a 5th square among them. Prove." } { "Tag": [ "number theory proposed", "number theory" ], "Problem": "For a positive integer $n$ let $d(n)$ be the sum of its digits.when written in demical form.Prove that for each positive integer $k$ there exists a positive integer $m$ such that the equation $x+d(x)=m$ has exactly $k$ positive integer solutions.", "Solution_1": "This is a Romanian ?Year Unknown? Problem 6. \r\nIt was unsolved on the forum (if I remember right) so if you have a solution it will be a joy for us to know it !", "Solution_2": "http://www.mathlinks.ro/Forum/viewtopic.php?p=334405#p334405" } { "Tag": [ "geometry", "3D geometry", "sphere" ], "Problem": "What is the radius of the largest circle that can be contained in a cube of side length 1?", "Solution_1": "[hide=\"hint\"]\nThis circle would be inscribed in a regular hexagon whose sides lie on the faces of the cube\n[/hide]\nI'm actually not sure that i can prove that that case is the maximum, but I'm pretty sure.\nAssuming that this condition is ture...\n[hide]\nThis hexagon would be formed by taking the midpoints of 6 edges of the cube such that each face would have exactly 2 adjacent dges among the 6 chosed edges and connection them in order to make a regular hexagon.\nthis would lead to an edge length of $ \\frac{\\sqrt2}{2}$, and since the radius of a circle inscribed in a hexagon is the side length of teh hexagon, this is also our answer.\n(this fact is found by drawing the longest diagonals in a hexagon and forming equilateral triangles.\n[/hide]", "Solution_2": "I think you'll have to multiply the side length of the hexagon by $ \\frac {\\sqrt{3}}{2}$ to get the radius of the circle inscribed in the hexagon, so the correct answer would be $ \\frac{\\sqrt{6}}{4}$. I also strongly feel that this is the correct answer, but I don't know how to prove it.. How do we know that by tilting the axis of the circle elsewhere we can enlarge the circle a bit while keeping it within the cube?", "Solution_3": "I agree The answer should be $ r \\equal{} \\frac {\\sqrt {6}}{4}$, here's an idea for a proof...\r\n\r\n[hide=\"solution\"]\nFirstly, we must show that the center of the circle must also be the center of the cube(which i'll call the centroid for brievity). Suppose the center of the circle was not the centroid, then the circle would be closer to one face of the cube than the other (and also closer than the centroid is to that face). Therefore we would be able to shift the center of the circle onto the centroid and it would still be contained within the cube. Hence wlog we may assume that the center of the circle is also the center of the cube\n\nSuppose there is a sphere inscribed inside our unit cube. On this sphere there is an infinite number of circles - all with radius $ \\frac {1}{2}$\nNow suppose the sphere increases, but its center and the cube remains constant.\nas the sphere increases it protrudes from each face of the cube; creating a circles on each face \n\nEventually we wont be able to draw a complete circle arround the sphere such that it never leaves the interior of the cube, and we must find this point. \n\nNow, if we have a circle running arround the sphere then we can also extend and trace that circle onto the cube making a straight line. So let's fold the cube out\n [geogebra]aa5e4c448e18adc38ec839fb15691c0bc8e7a551[/geogebra] \n\nThe radius of our circle will be maximised when the circles on the face of each cube are maximised (and we can still draw a straight line between them)\nTherefore from here it is clear that the maximum size of the circles on the face of each cube occurs in the situation above - where they are tangent to line runing though the midpoints of each edge.\nThis happens to be the same situation described by [b]steven meow[/b]\nAnd the maximum radius is $ \\frac {\\sqrt {6}}{4}$\n[/hide]" } { "Tag": [ "logarithms", "factorial" ], "Problem": "What positive integer $n$ satisfies $\\log_{10} (225!) - \\log_{10} (223!) = 1 + \\log_{10} (n!)$?", "Solution_1": "It looked kinda scary at first when I saw the factorial symbol but got better when I saw the subtraction. :) \r\n\r\n[hide=\"answer\"]With some log rules, we have $\\log_{10} \\frac{225!}{223!}=1 + \\log_{10} n!$ \n\nNext, $\\log_{10} 5040 = \\log_{10} n!$\n$n!=5040$ and $n=7$\n[/hide]", "Solution_2": "Solution:\r\n[hide]\n$\\\\\\log 225!-\\log 223!=1+\\log (n!)\\\\\\log 50400=1+\\log (n!)\\\\n!=10^{\\log 50400-1}=5040\\\\n=7\\\\\\mathbb{QED}$\n[/hide]\r\n\r\nEDIT: 4everwise was faster. :(\r\n\r\nMasoud Zargar", "Solution_3": "[quote=\"Silverfalcon\"]What positive integer $n$ satisfies $\\log_{10} (225!) - \\log_{10} (223!) = 1 + \\log_{10} (n!)$?[/quote]\r\n[hide]\nThis turns into\n$\\log {224*225}=\\log {10*n!}$\n$224*225=10*n!$\n$50400=10*n!$\n$n!=5040$\nn=7.\n\n\n[/hide]", "Solution_4": "answer is 7(n=7)" } { "Tag": [ "inequalities solved", "inequalities" ], "Problem": "x,y,z,t>0 x+y+z=4-t\r\n\r\n(x+z)(y+t) \\leq 1/x 2 +1/y 2 +1/z 2 +1/t 2", "Solution_1": "We have 1/x^2 + 1/z^2 >= 8/(x+z)^2 (trivial).\r\n\r\nSo, 1/x^2+1/y^2+1/z^2+1/t^2 >= 8/(x+z)^2 + 8/(y+t)^2\r\n\r\nNow, is enough to show that \r\n 8/(x+z)^2 + 8/(y+t)^2 >= (x+z)(y+t)\r\n<=> 8/a^2 + 8/b^2 >= ab with a+b = 4\r\n<=> 8(a^2+b^2) >= a^3.b^3\r\n<=> (a+b)^4(a^2+b^2)>= 32a^3.b^3\r\n\r\nwhich is trivial by AM-GM.\r\n\r\nNamdung", "Solution_2": "Another proof:\r\n We have \\sum 1/x 2 \\geq 4/ \\sqrt xyzt \\geq 4. But 4(xy+yz+zt+tx) \\leq (x+y+z+t) 2 (well known)=16. So, xy+yz+zt+tx \\leq 4 \\leq \\sum 1/x 2" } { "Tag": [ "geometry", "parallelogram", "geometry proposed" ], "Problem": "Points $ K,M$ are taken on diagonal $ AC$ at points $ P,T$ on diagonal $ BD$ of convex quadrilateral $ ABCD$ such that $ AK=MC=\\frac{1}{4}AC$ and $ BP=TD=\\frac{1}{4}BD$. Prove that the line passing through the midpoints of $ AD,BC$ bisects the segments $ PM$ and $ KT$.", "Solution_1": "[quote=\"cancer\"][color=darkred]Points $ K,\\,M$ are taken on diagonal $ AC$ at points $ P,\\,T$ on diagonal $ BD$ of convex quadrilateral $ ABCD$ such that $ AK=MC=\\frac{1}{4}AC$ and $ BP=TD=\\frac{1}{4}BD.$ \nProve that the line passing through the midpoints of $ AD,\\,BC$ bisects the segments $ PM$ and $ KT$[/color][/quote]\r\n[color=indigo][b]Proof.[/b] Denote $ G,\\,H,\\,I,\\,J,\\,S$ be the midpoints of $ AD,\\,BC,\\,AC,\\,BD,\\,IJ,$ respectively.\nWe have $ GIHJ$ is a parallelogram, so $ S\\in GH\\quad (*)$\n\nWe also have: $ MHPS$ is a parallelogram, so $ SH$ bisects the segment $ PM$\nSimilarly, $ SG$ bisects the segment $ TK$\nHence the line passing through the midpoints of $ AD,\\,BC$ bisects the segments $ PM$ and $ KT$[/color]" } { "Tag": [ "calculus", "integration", "geometry", "3D geometry", "sphere", "analytic geometry", "calculus computations" ], "Problem": "In Euclidean space, take the point $ N(0,\\ 0,\\ 1)$ on the sphere $ S$ with radius 1 centered in the origin. For moving points $ P,\\ Q$ on $ S$ such that $ NP \\equal{} NQ$ and $ \\angle{PNQ} \\equal{} \\theta \\left(0 < \\theta < \\frac {\\pi}{2}\\right),$ consider the solid figure $ T$ in which the line segment $ PQ$ can be passed.\r\n\r\n(1) Show that $ z$ coordinates of $ P,\\ Q$ are equal.\r\n\r\n(2) When $ P$ is on the palne $ z \\equal{} h,$ express the length of $ PQ$ in terms of $ \\theta$ and $ h$.\r\n\r\n(3) Draw the outline of the cross section by cutting $ T$ by the plane $ z \\equal{} h$, then express the area in terms of $ \\theta$ and $ h$.\r\n\r\n(4) Pay attention to the range for which $ h$ can be valued, express the volume $ V$ of $ T$ in terms of $ \\theta$, then find the maximum $ V$ when let $ \\theta$ vary.", "Solution_1": "[quote=\"kunny\"]consider the solid figure $ T$ by forming the line segment $ PQ$ passes.[/quote]\n\nI don't understand this statement.\n\n[quote=\"kunny\"](4) Pay attention to the range for which $ h$ can be valued, express the volume $ V$ of $ T$ in terms of $ \\theta$, then find the maximum $ V$ when let $ \\theta$ move.[/quote]\r\n\r\nI also don't understand this statement.", "Solution_2": "Sorry for my poor English.I have just edited. How about this time?" } { "Tag": [ "inequalities", "graph theory", "combinatorics proposed", "combinatorics" ], "Problem": "Given a graph with $n\\ge 2$ vertices s.t. the degree of every vertex is $>\\frac n2$, prove that any two vertices are the ends of some hamiltonian chain.", "Solution_1": "It comes straightforward from the Ore's Theorem [url=http://mathworld.wolfram.com/OresTheorem.html]mathworld.wolfram.com/OresTheorem.html[/url].\r\nAn easy, indirect proof can be found in many discrete math books.", "Solution_2": "Could you still post a complete proof though? I don't think it's [b]that[/b] obvious how it follows from Ore's Theorem :). And I didn't even apply Ore's Theorem \"per se\".", "Solution_3": "Ok, soon I'll place a proof of this theorem, note that the graph you're talking about fulfills that theorem's conditions.", "Solution_4": "Since every vertex has degree more than $n/2$ then the graph is connected.\r\n\r\nConsider the maximum path $P=v_0,...,v_k$. Since $P$ is maximm all the neighbors of $v_0$ and of $v_k$ lie in $P$. SO at least $n/2$ of the vertices $v_0,...,v_{k-1}$ are adjacent to $v_k$ and at least $n/2$ of these same vertices $v_i$ are such that $v_0 v_{i+1}$ is an edge. By pigeon hole principle, there exists a vertex say $v_i$ that has both properties, so we have that $v_0 v_{i+1}$ and $v_i v_k$ are both edges. So now it is easy to see that the cycle $C=v_0 v_{i+1}Pv_kv_iPv_0$ is a hamilton cycle in the graph. Otherwise, since the graph is conncted, $C$ would have a neighbor in $G_C$ but this neighbor plus a spannig path of $C$ makes a path longer than $P$ contradicting its maximality. So the graph must conain a hamilton cycle.", "Solution_5": "People, did you understand what the problem I posted asks? :)\r\n\r\nIt does not ask for a proof that the graph is hamiltonian. For this, it suffices (it follows immediately from Ore's Theorem, cited above) to have all the degrees $\\ge\\frac n2$. The hypothesis here is stronger: all the degrees are $>\\frac n2$, and we have to prove something else: given [b]any[/b] two vertices $x_1,x_n$ of the graph, there is a hamiltonian chain (not cycle, i.e. $x_1$ need not be connected to $x_n$) $x_1,x_2,\\ldots,x_n$ beginning with $x_1$ and ending with $x_n$.", "Solution_6": "Oops! :blush: \r\nI'll think on it.\r\nGrobber, is a halmilton chain the same thing as a spannig path right?", "Solution_7": "here is an approach(i still haven't finished it):\r\nthe question is equivalent to showing that in a graph where $\\delta(G)>n/2$, every edge is contained in a Hamiltonian cycle(if $u,v$ are not adjacent, then consider the graph $G\\cup\\{e\\}$ where $e=\\{u,v\\}$.then this new graph would have a hamiltonian cycle containing the edge $e$ and hence the original graph has a hamiltonian path between $u$ and $v$).\r\n for such a pair of vertices $u,v$ consider all the cycles of $G$ that contain the edge $\\{u,v\\}$. this isn't empty since the degree condition implies that $N(u)\\cap N(v)\\neq \\emptyset$.Let $C$ be a maximum such cycle. Now every vertex $x\\in V(G)\\setminus V(C)$ can have atmost $\\frac{|C|+1}{2}$ neighbors in $C$(or else there are 2 'consecutive' vertices,other than $u,v$ of $C$ adjacent to $x$ and this admits a larger cycle containing $\\{u,v\\}$) and so every vertex $x\\in V(G)\\setminus V(C)$ has $\\geq (n/2-c/2)=(n-c)/2$ neighbors off $C$ and so by Ore's thm $G\\setminus C$ is hamiltonian.\r\nnow we seek to enlarge the cycle $C$ to get a contradiction. if 2 consecutive vertices of $C$(other than $u,v$) have neighbors off $C$ then such an enlargement of $C$ comes immediately(since $G\\setminus C$ is Hamiltonian).", "Solution_8": "after a few futile attempts along that direction i proposed(which seemed promising) i was more and more convinced that there must be another way out...!\r\n Let us do the following: For a given pair of vertices $u\\neq v$ we form a new graph $G_{u,v}$ with vertex set $V(G)\\cup\\{x_{uv}\\}$ and the edges being the edges of $G$ along with a couple of new edges-joining $u,v$ to the new vertex $x_{uv}$.now this new graph on $n+1$ vertices is Hamiltonian iff the original graph $G$ has a hamiltonian path between $u$ and $v$.\r\n To see that this new graph $G_{u,v}$ is hamiltonian we take its Hamiltonian closure,i.e. for every pair of nonadjacent vertices $x,y$ such that $d(x)+d(y)\\geq (n+1)$ we add the edge $\\{x,y\\}$ and keep doing this till we no longer can add newer edges.\r\nnow a graph is hamiltonian iff its closure is.denoting by $\\overline{G_{u,v}}$ the closure, we note that in the closure, since for every $x,y\\in V(G)$,nonadjacent, we have $d(x)+d(y)>n\\Rightarrow d(x)+d(y)\\geq n+1$, so $\\overline{G_{u,v}}$ restricted to $V(G)$ gives the complete graph $K_n$, and hence $G_{u,v}$ is hamiltonian and that finishes the proof.\r\nnote that all this proof needs is that for every pair of nonadjacent vertices $x\\neq y$ of $G$, we have $d(x)+d(y)>n$.(so the given condition is stronger).", "Solution_9": "sorry, I've mistaken the definition of a chain :)\r\n\r\nHere is my proposal for a solution:\r\n\r\nLet's use induction. For $n=3$ it's obvious.\r\nNow let's have a graph with $n (n>3)$ vertices. Suppose that there exists a pair of vertices $x,y$ which are not the ends of a a hamiltonian chain. The number of edges equals $k < n $. Let's name the group of vertices belonging to the chain by $X$, let $Y$ be the group of vertices which not belong to $Y$.\r\nLet's note that the degree of every vertice in $Y$ is bigger than $\\frac{n-k}{2}$ cause if there is a vertice with a smaller degree in $Y$ then it is connected with at least $\\frac{k}{2}$ vertices from the chain. That means that there exist two vertices from the chain conected with that vertice from $Y$ and wich are consecutive edges in the chain. That means that we can expand our chain with that element from $Y$ - contradiction coming from maximality of $k$. Apllying the inductive assumption subset $Y$ satisfies the theorem of the problem (inductive condition) -.\r\nNow let's solve two cases (which solve the problem):\r\n(1)If $ k=|X| \\leq \\frac{n}{2}$ then it's easy cause picking two consecutive edges $a,b$ from the chain - \r\neach of them is connected with an edge with some elements in $Y$ so we can expand our chain replacing $a,b$ with\r\n$a,k_1,...,k_l,b$ were $k_1,...,k_l$ is the fragment of the hamilton cycle in $Y$ - so we have \r\na contradiction because of the maximality of $k$.\r\n(2)If $ k > \\frac{n}{2}$ then each vertice from $Y$ is conected with more then $\\frac{n}{2}-(n-k)+1$\r\n vertices in the chain in $X$. Because $Y$ satisfies the inductive condition - in $Y$ for every two vertices \r\nthere exists a hamiltonian chain with ends in those two vertices. So if there exist two vertices from $Y$ \r\n$a,b$ when $a$ is connected with $c$ from the chain and $b$ is connected with $d$ from the chain ($c,d$ are from $X$ and in addition there are less then $n-k$ vertices in the chain which lie between $a$ and $b$ - then we can expand our chain adding to $X$ the chain from $Y$ - so we have a contradiction with the maximality of $k$.\r\nWe will show that such $c,d$ in chain $X$ must exist. There are more than $(n-k)(\\frac{n}{2}-(n-k))$ vertices conecting $X$ with $Y$. The maximal value of vertices connecting $X$ with $Y$ for which there is no pair of vertices in $X$ which have less than $n-k$ vertices between them in the chain is $\\frac{(n-k)k}{n-k+1}$ (every vertice from $X$ connected with some vertice from $Y$ is connected with all from $Y$) but(!!) $(n-k)(\\frac{n}{2}-(n-k)+1) \\geq \\frac{(n-k)k}{n-k+1}$. In order to show that let's put $k=|X|=a(t+1)+r$, $|Y|=t$ for $t,a$ positive integer and $0 \\leq r \\leq t$. The inequality is equivalent to $\\frac{a(t+1)+r+t+1}{2}-a(t+1)-r-t+a(t+1)+r+1 \\geq \\frac{a(t+1)+r}{t+1}\\\"=\\\"a+1$-if $r>0$,$a$ if $r=0$. What is equivalent to $\\frac{a(t+1)+r+t+1}{2}+1 \\geq a+1+t$ or $a+t$, but $(a+1)(t+1) \\geq 2a+2t$ - this inequality holds for all $a,t$ positive integers. That means that we have have a contradiction (it's against the maximality of $k$) - we can indeed expand our chain $X$ adding the chain $Y$ - of lenght $t$.", "Solution_10": "suppose that it isn't true, so there exist $a,b$ such that there doesn't exist hamiltonian chain\r\nif it is not true, we can add more edges untill every next adding of edge will give us hamiltonian chain beween $a$ and $b$, thus let us add such number of edges, we have now\r\n$a \\rightarrow x_1 \\rightarrow x_2 \\rightarrow ... \\rightarrow x_{n-3} \\rightarrow b$\r\nbut $x_n$ is connected with more than $\\frac{n}{2}$ vertices so it must be connected with one of pairs \r\n$\\{a,x_1\\},\\{x_1,x_2\\},..,\\{x_{n-3},b\\}$. contradiction, becouse there shouldn't be any chain between $a,b$" } { "Tag": [ "calculus", "integration", "real analysis", "real analysis unsolved" ], "Problem": "Compare the integrals $\\int_{0}^{1}x^{x}dx$ and $\\int_{0}^{1}\\int_{0}^{1}(xy)^{xy}dxdy.$ \r\n(V. Radchenko)", "Solution_1": "$2\\int_{0}^{1}x^{x}dx=\\int_{0}^{1}\\int_{0}^{1}(x^{x}+y^{y}) dxdy \\ge 2\\int_{0}^{1}\\int_{0}^{1}(xy)^{xy/2}dxdy \\ge 2\\int_{0}^{1}\\int_{0}^{1}(xy)^{xy}dxdy$\r\nObvious,the equation can't be hold.\r\nSo:$\\int_{0}^{1}x^{x}dx>\\int_{0}^{1}\\int_{0}^{1}(xy)^{xy}dxdy$ :)", "Solution_2": "[quote=\"zhaobin\"]$\\int_{0}^{1}\\int_{0}^{1}(x^{x}+y^{y}) dxdy \\ge 2\\int_{0}^{1}\\int_{0}^{1}(xy)^{xy/2}dxdy$\n[/quote]Why???", "Solution_3": "Sorry,I am wrong :blush:" } { "Tag": [ "floor function", "algebra proposed", "algebra" ], "Problem": "Hi,\r\n\r\n$n\\in{\\Bbb N}\\setminus\\{0\\}$, $x\\in{\\Bbb R}$. Prove that :\r\n\\[\\lfloor x\\rfloor^{2}+\\left\\lfloor x+{1\\over n}\\right\\rfloor^{2}+\\cdots+\\left\\lfloor x+{{n-1}\\over{n}}\\right\\rfloor^{2}=\\lfloor nx\\rfloor^{2}\\]\r\nif and only if $\\lfloor nx\\rfloor=\\lfloor x\\rfloor$.", "Solution_1": "[quote=\"Michael\"]Hi,\n\n$n\\in{\\Bbb N}\\setminus\\{0\\}$, $x\\in{\\Bbb R}$. Prove that :\n\\[\\lfloor x\\rfloor^{2}+\\left\\lfloor x+{1\\over n}\\right\\rfloor^{2}+\\cdots+\\left\\lfloor x+{{n-1}\\over{n}}\\right\\rfloor^{2}=\\lfloor nx\\rfloor^{2}\\]\nif and only if $\\lfloor nx\\rfloor=\\lfloor x\\rfloor$.[/quote]\r\nLet $\\lfloor x\\rfloor =a\\in Z$ and $\\frac{k}{n}\\le\\{x\\}<\\frac{k+1}{n},0\\le k1, then this equation had only solution $a=k=0$, because $D=k^{2}(n-2)^{2}<4n(n-1)k(k-1)$ for k>1, k=0 give a=0, k=1 had non integer solution.\r\n$a=0=k$ mean $\\lfloor x\\rfloor =\\lfloor nx\\rfloor$." } { "Tag": [], "Problem": "$ (4^3)\\plus{}(6^2)\\equal{}10^x$. Find $ x$.", "Solution_1": "$ 4^3\\plus{}6^2\\equal{}64\\plus{}36\\equal{}100\\equal{}10^{\\boxed{2}}$", "Solution_2": "hello, $ 4^3\\plus{}6^2\\equal{}64\\plus{}36\\equal{}100\\equal{}10^2$, so we get $ 10^2\\equal{}10^x \\Leftrightarrow x\\equal{}2$.\r\nSonnhard." } { "Tag": [ "MATHCOUNTS", "videos" ], "Problem": "for those who know more about the history of mathcounts than i do, has a score of 46 ever been achieved by an individual at states? at national? if not what are the highest scores ever for state and national?\r\n\r\nalso, what was the highest team score ever achieved, state and national?", "Solution_1": "im pretty sure there were people with perfects mathcounts aint the hardest math competition", "Solution_2": "but, mathcounts is only US, and not like china or something.\r\n\r\nand if bobby shen didn't get a perfect, then....yeah.", "Solution_3": "Well, what about in 1984? Of course people got perfects.", "Solution_4": "Yeah, even at Nationals. Mr. Crawford did that.", "Solution_5": "DWu got a perfect at states this year.", "Solution_6": "Hard to tell. Some years have easier/harder tests than others. The skill of the participants also varies from year to year.\r\n\r\n@math154: Can't be entirely sure. The nats participants in 1984 were probably not as smart as recent years' participants, partly because people now know what to expect (from being able to study past tests) and because MATHCOUNTS has \"created\" smarter participants. Parent and schools now teach their kids problem-solving math from early ages, to prepare them for contests like MATHCOUNTS. Thus, we can't be sure.\r\n\r\nEDIT: People did get perfect scores.", "Solution_7": "MCrawford got a perfect score for sure.", "Solution_8": "[quote=\"tinytim\"]DWu got a perfect at states this year.[/quote]seriously? i thought the highest was that 3 way tie in texas....wow\r\n\r\n\r\ndwu is beast, looking forward to next year mathcounts to (probably) seeing him and bobby in final round of cd :D that will be exiting and interesting", "Solution_9": "Um no, word got out wrong. DWu got a 42. Texas 3-way tie was definitely top score in the nation. But still, DWu is very beast.", "Solution_10": "[quote=\"mathking123\"][quote=\"tinytim\"]DWu got a perfect at states this year.[/quote]seriously? i thought the highest was that 3 way tie in texas....wow\n\n\ndwu is beast, looking forward to next year mathcounts to (probably) seeing him and bobby in final round of cd :D that will be exiting and interesting[/quote]\r\nActually, it might not be that, since if they're in the same half of the bracket, you'll probably see them sooner.", "Solution_11": "good point, but still it will be interesting.\r\n\r\non that topic, does anyone know where i can find a video of the 2008 countdown round?\r\n\r\ni think mathcounts took it down(?)", "Solution_12": "I think at a national level, people would get it perfect[/i]", "Solution_13": "If you are a test-writer, you ideally want your test to be such that nobody gets a perfect score and nobody gets a score of 0. Think about it. If 3 people had perfect scores at nats, what would you do? How do you decide who gets the 1st place trophy/award? Also, a test in which 25% of people score 0 would be bad as well (not good for comparing skills accurately). The best tests are not the ones on which a perfect score CAN be achieved on or ones that many people fail, but the ones that can accurately seperate the smartest people from the others.\r\n\r\nThis is why the MATHCOUNTS tests are so good. It is as hard to get a score of 0 as it is to get a score of 46 (provided you try your hardest and use all your skills and knowledge). Tests like these are rare and very good for competitions.", "Solution_14": "They would use a tiebreaker round, which considers time. Also, I believe the highest score in California was also a 44." } { "Tag": [], "Problem": "Outline reasonable syntheses of 6-acetyl-1,2,3,4-tetrahydropyridine, the principal compound responsible for the aroma of bread, and of 2-acetyl-1-pyrroline, the principal component of cooked rice.", "Solution_1": "for second one- pyrrole + houben housche or vilsmeier haack reaction to get 2 acetyl pyrrole foll. by birch reduction.\r\nnot sure abt the reduction step.", "Solution_2": "Yes, that reduction procedure doesn't also look reliable to me.", "Solution_3": "for the first one - maybe u cud use an aza diels alder reaction.", "Solution_4": "Between which molecules?", "Solution_5": "ne of these two reaction ofc with the acyl group attached to the substrate." } { "Tag": [ "algebra", "polynomial", "calculus", "integration" ], "Problem": "Let $a_1,a_2,...,a_n$ be distinct integers with $n\\geq1$. Show that the polynomial \r\n$f(x)=(x-a_1)(x-a_2)\\cdots(x-a_n)-1$\r\ncannot be decomposed as a product of two integral polynomials", "Solution_1": "$f(x) = (x-1)(x+1)-1$\r\n$f(x) = x^{2}$\r\n$f(x) = (x)(x)$\r\nbut if you mean that $a_{n} \\ge 1$\r\nthat would be different", "Solution_2": "[quote=\"Altheman\"]$f(x) = (x-1)(x+1)-1$\n$f(x) = x^{2}$\n$f(x) = (x)(x)$\nbut if you mean that $a_{n} \\ge 1$\nthat would be different[/quote]um $(x-1)(x+1)-1=x^2-1-1=x^2-2...$", "Solution_3": "woops...my bad", "Solution_4": "anyone want to try (HINT: the statement is true)", "Solution_5": "anybody want to try? Come on its not too hard", "Solution_6": "Well lets suppose that it can:\r\n\r\nthen $f(x)=p(x)q(x)$ whicn have integral coeficients, but if we evaluate $x$ like $a_1$, $a_2$, ...,$a_n$ we have:\r\n\r\n$f(a_i)=-1=p(a_i)q(a_i)$ but both are integrers so one has to be $1$ and the other $-1$ let\u00b4s say that the grade of $p(x)$ is less of equal than the grade of $q(x)$.\r\nAnd let\u00b4s see that we have n values of $p(x)$ some 1\u00b4s and some (-1)\u00b4s but there are half or more of one....let\u00b4s say there are more 1\u00b4s (the other case is equal).\r\n\r\nThen the number of 1\u00b4s is $\\ge$ than n/2 and that is $\\ge$ than the grade of $p(x)$ but if the both of them arent $n/2$ then $p(x)-1$ would have more roots than its grade so it would be constant wich is impossible...then $p(x)$ is of grade n/2 and it has n/2 q\u00b4s and -1\u00b4s being evaluated in the roots of $f(x)+1$...\r\n\r\nbut then $p(x)+1$ divides $f(x)+1$ because they have the same roots and also $p(x)-1$ but they have different roots so $(p(x)-1)(p(x)+1) divides f(x)+1$ but they have he same grade so $p(x)^2-1=f(x)+1$ $(1)$\r\nand also is the same for $q(x)$ because they have the same grade so $q(x)^2-1=f(x)+1$ and then $p(x)=q(x)$ but then p(x)^2=f(x)$ that\u00b4s absurd becuase of $(1)$", "Solution_7": "sorry about the last thing\r\n\r\nbut then $p(x)^2=f(x)$ whcih is absurd because of the $(1)$", "Solution_8": "JoshuaMex, instead of making a new post to correct something you did wrong in your previous one, you can edit your previous one. Simply click \"edit\" at the bottom of the post you want to edit.", "Solution_9": "I like the first part of the solution, but theres an easier way:\r\n\r\ncall $g(x)=p(x)+q(x)$, and since we established that $p(a_i)=\\pm1$ and $q(a_i)=\\mp1$, then $g(a_i)=0$. This means that either:\r\n\r\nsince there are at least $n$ zeros, $\\deg[g(x)]\\geq{n}$, which means that at least one of $p(x)$ or $q(x)$ is of degree at least $n$ also, which already contradicts.\r\n\r\nor that $g(x)=0$. In this case, $p(x)=-q(x)$, so $f(x)=-q(x)^2$, which can't be true becuase of the leading coefficient of $f(x)$ is positive" } { "Tag": [ "calculus", "integration", "function" ], "Problem": "[url=http://imageshack.us][img]http://img383.imageshack.us/img383/3936/xcyf5.gif[/img][/url]", "Solution_1": "I'm afraid this might take a while....there is no integral for this function!\r\nThis is the \"Bell Curve\", which cannot be integrated (right?).\r\n\r\nRicOh", "Solution_2": "I would say definitely not HSB... but you write that as a maclaurin series and integrate no?", "Solution_3": "They don't even allow calculus problems on the Intermediate board. They definitely won't allow them on HSB.", "Solution_4": "[b]Please post calculus problems in the Calculus Computations and Tutorials forum.[/b]\r\n\r\n[url]http://www.artofproblemsolving.com/Forum/index.php?f=296[/url]", "Solution_5": "someone please move this... :|" } { "Tag": [ "function", "real analysis", "real analysis unsolved" ], "Problem": "Let $ f: [a,b] \\rightarrow \\mathbb{R}$ be monotone. \r\nProve $ f$ has a countabe set of discontinuities.", "Solution_1": "Here's an idea.\r\n\r\nLet's say $ f$ is non-decreasing. Let $ S$ be the set of points where $ f$ is discontinuous. For each $ x$ in $ S$, associate with $ x$ a rational number in between $ f(x\\minus{})$ and $ f(x\\plus{})$. Prove that this correspondence is $ 1\\minus{}1$, using the fact that $ f$ is non-decreasing. $ S$ has now been put into a $ 1\\minus{}1$ correspondence with a subset of the rational numbers, so $ S$ is countable.", "Solution_2": "ok nice. i have one solution that sounds like it starts off the same as this and will post it soon.", "Solution_3": "Well here it goes\r\n[hide] \n Assume WOLOG $ f$ to be increasing and set $ f(a \\minus{} ) \\equal{} f(a)$ and $ f(b \\plus{} ) \\equal{} f(b)$. Then $ \\forall x \\in [a,b]$ $ f(x \\minus{} )$ and $ f(x \\plus{} )$ exist and $ f(x \\minus{} ) \\leq f(x) \\leq f(x \\plus{} )$ with a discontinuity at $ x$ meaning $ f(x \\minus{} ) < f(x \\plus{} )$. \n Now consider the function $ s(x) \\equal{} f(x \\plus{} ) \\minus{} f(x \\minus{} )$ which represents the \"jump\". We say $ f$ is discontinuous at $ x$ iff $ s(x) > 0$. Now we want to look at the set $ s_n \\equal{} \\{x \\in [a,b] : s(x) \\geq \\frac {1}{n}\\}$ for $ n \\in \\mathbb{N}$. Say we have $ x_1,x_2, \\ldots,x_k \\in s_n$ with $ a \\leq x_1 < x_2 \\cdots < x_k \\leq b$. \n Select points $ y_0 \\equal{} a \\leq x_1 < y_1 < x_2 < y_2 \\cdots < x_k < y_k \\equal{} b$. Clearly, we have $ f(y_k) \\minus{} f(y_{k \\minus{} 1}) \\geq f(x_k \\plus{} ) \\minus{} f(x_k \\minus{} ) \\equal{} s(x_k) \\geq \\frac {1}{n}$.\nNow we add them all up,\n\\[ \\left[f(y_1) \\minus{} f(y_0)\\right] \\plus{} \\left[f(y_2) \\minus{} f(y_1) \\right] \\plus{} \\cdots \\plus{} \\left[f(y_k) \\minus{} f(y_{k \\minus{} 1})\\right] \\equal{} f(y_k) \\minus{} f(y_0) \\equal{} f(b) \\minus{} f(a) \\geq \\frac {k}{n}\n\\]\nThis implies that $ s_n$ has at most $ n \\left[f(b) \\minus{} f(a)\\right]$ elements, that is $ s_n$ is finite. \n Now let $ \\mathfrak{D} \\equal{} \\{ x \\in [a,b] : f$ discontinuous at $ x \\}$. If $ x \\in \\mathfrak{D}$ then $ s(x) > 0$ hence, $ \\exists n \\in \\mathbb{N}$ such that $ s(x) \\geq \\frac {1}{n}$ therefore $ x_n \\in s_n$. If $ x \\in s_n$ for some $ n$ then $ s(x) \\geq \\frac {1}{n} > 0$ hence $ x \\in \\mathfrak{D}$. Thus $ \\mathfrak{D} \\equal{} \\displaystyle \\bigcup_{n \\equal{} 1}^{\\infty} s_n$. And since $ s_n$ is finite $ \\mathfrak{D}$ is countable. \n[/hide]", "Solution_4": "I would get rid of that first thing you do where you say $ f(a\\minus{})\\equal{}f(a)$ etc. Notationally that gets confusing because you then say $ f(x\\minus{})\\leq f(x)\\leq f(x\\plus{})$ which is true, but doesn't really cohere with that first part.\r\n\r\nNext. I can't really tell what you did to get $ s_n$ to be finite. It seems that when you \"select points\" you assume that there are only finitely many because you write them down up to $ k$. I could be misunderstanding what you did there, though. :huh:", "Solution_5": "First off, it may be strange that I define $ f(a\\minus{}) \\equal{} f(a)$ and so on, but I do so to make sure the function is well behaved on the interval $ [a,b]$ and it doesn't contradict the statement $ f(x\\minus{}) \\leq f(x) \\leq f(x\\plus{})$ I believe :blush: .\r\n\r\nNext, I know that $ s_n$ must be finite because were dealing with a finite interval and the sum of the jumps cannot exceed $ f(b) \\minus{} f(a)$ and it must be finite. i think? :maybe:", "Solution_6": "I'm having trouble coming up with an counterexample for $ s_2$, say. Let me think about that some more. Maybe it is true. I have a feeling it isn't, though.\r\n\r\nSorry about the other thing. I had forgotten that the interval was labeled $ [a,b]$, I was thinking for some reason that you had just taken two arbitrary letters (I was doing it on $ [0,1]$). :blush:", "Solution_7": "Wow. I'm being really thick-headed for some reason. That fact that I've been trying to dispute is obviously true. Suppose there are infinitely many, then by the same technique used later you have that $ f(b)\\minus{}f(a)\\geq\\infty$ a contradiction. Thus each $ s_n$ is finite.", "Solution_8": "so does this mean you agree with what i said or is there still an error in my work?", "Solution_9": "I like the idea. I haven't seen this done this way before. I think you are trying too hard at parts, though. You are trying to bound the number of elements in $ s_n$ to get it to be finite. This is all you need. So, I would take the sentence $ s_n\\equal{}\\{x\\in [a,b]: s(x)\\geq\\frac{1}{n}\\}$ for $ n\\in\\mathbb{N}$ and alter it to: Let $ s_n\\equal{}\\{x\\in [a,b]: s(x)\\geq\\frac{1}{n}\\}$.\r\n\r\nNow, I'd erase all of the next section to the definition of $ \\mathfrak{D}$ and replace it with:\r\n\r\nClaim: $ s_n$ is finite for each $ n\\in\\mathbb{N}$. Suppose not for some $ s_m$. Then just looking at the \"gaps\" in $ s_m$ we have $ f(b)\\minus{}f(a)\\geq |s_m|\\cdot\\frac{1}{m}\\equal{}\\infty$. (Where $ |s_m|$ is the order of that set). This contradicts our assumption that both $ f(a)$ and $ f(b)$ are finite, so each $ s_n$ is finite.\r\n\r\nYour proof that if $ x\\in\\mathfrak{D}$ then $ x\\in s_n$ for some $ n$ is fine. This gives you that $ \\mathfrak{D}\\subset\\bigcup s_n$, or $ |\\mathfrak{D}|\\leq \\big|\\bigcup s_n\\big|$, so $ \\mathfrak{D}$ is at most countable.\r\n\r\nA weird thing about this is that it seems like you count things too many times. I don't think this leads to error, but it bothers me. A simple redefining of $ s_n$ should do the trick. Here is what I mean: Suppose there is an $ x_0$ such that $ s(x_0)\\equal{}1$, then $ x_0\\in s_1$ and $ x_0\\in s_2$ and $ x_0\\in s_3$ etc.", "Solution_10": "This question has been asked recently, and kenn4000 gives a proof along the same lines here: [url]http://www.artofproblemsolving.com/Forum/viewtopic.php?t=210782[/url]", "Solution_11": "A monotonic function only has discontinuity points of the first kind so you can use [url=http://www.mathlinks.ro/viewtopic.php?t=31694]this problem[/url]. :)", "Solution_12": "Thanks Hilbert for your insight. Sorry ive been away and havent responded but i definitely see where you are coming from. i have very minimal experience writing formal proofs and is something ive been working on just recently with rudin. \r\nthanks again." } { "Tag": [ "inequalities", "geometry", "incenter", "circumcircle", "inradius", "rotation", "geometry proposed" ], "Problem": "Let $ I$ be the incenter of a bicentric quadrilateral $ ABCD$ with diagonals $ AC$ and $ BD.$ Show that, if $R, r$ denote its circumradius and inradius, then we have\n\\[2R^2\\ge IA \\cdot IC \\plus{} IB \\cdot ID\\ge4r^2\\]", "Solution_1": "$ \\boxed{ IA*IC \\plus{} IB*ID \\equal{} 4Rr^2(\\frac{1}{AC}\\plus{}\\frac{1}{BD})}$\n\n1. $ AC \\le 2R$ and $ BD \\le 2R$ => $ IA*IC \\plus{} IB*ID \\ge 4r^2$\n\n2. Let $ AB\\equal{} a$ , $ BC\\equal{} b$ , $ CD\\equal{} c$ , $ DA\\equal{} d$ , $ AC\\equal{} d_1$ , $ BD\\equal{} d_2$\n\n$ \\boxed{4Rrs\\equal{}\\sqrt{d_1d_2}\\sqrt{(ab\\plus{}cd)(bc\\plus{}ad)}}$ ( $ s$ - semiperimeter )\n\n$ \\frac{4Rrs}{\\sqrt{d_1d_2}} \\le \\frac{(a \\plus{} c)(b\\plus{}d)}{2}\\equal{}\\frac{s^2}{2}$\n\n$ IA*IC \\plus{} IB*ID \\equal{} 4Rr^2\\frac{d_1\\plus{}d_2}{d_1d_2} \\le \\frac{s^2}{4} \\le2R^2$ :)", "Solution_2": "Let $ (I,r),(O,R)$ be the incircle and circumcircle of $ABCD.$ $(I)$ touches its side-segments $AB,BC,CD,DA$ at $X,Y,Z,W.$ Simple angle chase gives $ \\angle XAI \\equal{} \\angle YIC$ and also $ \\angle BYI \\equal{} \\angle DIZ.$ Rotate $\\triangle AXI$ about $ I$ by angle $\\angle XIY$ counterclokwise, thus $ X \\mapsto Y$ and we get a right triangle with legs $\\overline{IA}$ and $\\overline{IC}.$ Similarly, rotating $ \\triangle IBY$ about $I$ by angle $\\angle YIZ$ we obtain a right triangle with legs $\\overline{ IB}$ and $\\overline{ ID}.$ Thereby, area of $ABCD$ can be expresed as twice the sum of the areas of the two said right triangles. In other words, $ [ABCB] \\equal{} IA \\cdot IC \\plus{} IB \\cdot ID.$ Now, using the fact that in a fixed circle, the inscribed/circumscribed quadrilateral with maximum/minimum area is the square, it follows that \n\n$2R^2\\ge [ABCD] \\ge 4r^2 \\Longrightarrow 2R^2 \\ge IA \\cdot IC \\plus{} IB \\cdot ID \\ge 4r^2.$" } { "Tag": [ "number theory unsolved", "number theory" ], "Problem": "Hallo,\r\n\r\nI've a problem wich I'm not able to solve it.\r\n\r\nFind all integers for wich\r\n\r\n(x 2 +y 2 )/(xy+1)\r\n\r\nis an integer.", "Solution_1": "$ pgcd (xy, xy+1) = 1$ implies that $\\frac{x^2+y^2}{xy+1}$ is integer ssi $\\frac{x^3y+y^3x}{xy+1}$ is a integer. But, \r\n\r\n$\\frac{x^3y+y^3x}{xy+1} = x^2 + y^2 - \\frac{x+y}{xy+1}$\r\n\r\nThat implies that :\r\n\r\n$\\frac{x+y}{xy+1}$ is a integer\r\n\r\nBut $(x-1)(y-1) = xy-x-y+1$\r\n\r\n=> $x+y==1\r\n\r\n=> There is a unique solution $(x=1, y=1)$", "Solution_2": "http://www.kalva.demon.co.uk/imo/isoln/isoln886.html", "Solution_3": "[quote=\"Labiliau\"](...)$\\frac{x^3y+y^3x}{xy+1} = x^2 + y^2 - \\frac{x+y}{xy+1}$\n[/quote]\r\n??? :?", "Solution_4": "It is known that the expression is integer iff its a perfect square, provable by infinite decent. \r\nBut the general case is crazily hard..\r\n\r\nBomb", "Solution_5": "My short solution for it:\r\n[tex]x^2-kxy+y^2-k=0[/tex]\r\n[tex](x_0,y_0)[/tex] is a root...\r\n1/[tex]x=y->x=y=1[/tex] and k=1\r\n2/WLOG ,assume [tex]x_0+y_0[/tex] is min and [tex]x_0>y_0.[/tex]\r\nThen [tex]x_1=ky_0-x_0[/tex] is another root \r\nEasily prove [tex]x_1>0[/tex]\r\n[tex]-> x_1>x_0>y_0>0[/tex]\r\n[tex]->y_0^2k<0->[/tex](not satisfy)", "Solution_6": "I'm Sorry. I made a mistake. The correct expression must be\r\n\r\n(x\u00b2+y\u00b2)/(xy-1)\r\n\r\nmy conjecture is that (x\u00b2+y\u00b2)/(xy-1) is always equal to 5.\r\nf.e. (x,y)=(67,14)" } { "Tag": [ "trigonometry", "geometry", "inequalities", "inequalities unsolved" ], "Problem": "Prove that : In triangle with $ p\\equal{}\\frac{a\\plus{}b\\plus{}c}{2}$ then$ \\sqrt {3}\\max\\lbrace h_a,h_b,h_c\\rbrace\\ge p$.", "Solution_1": "What about a triangle with angles $ 30, 30, 120$? Then (assuming the short side is 1) $ p \\equal{} 1 \\plus{} \\frac {\\sqrt {3}}{2}$ but the maximal altitude has height only $ \\frac{\\sqrt {3}}{2}$. (and $ \\frac {3}{2} < 1 \\plus{} \\frac {\\sqrt {3}}{2}$.)\r\n\r\nDoes the triangle have to be acute?", "Solution_2": "[quote=\"Hong Quy\"]Prove that : In triangle with $ p \\equal{} \\frac {a \\plus{} b \\plus{} c}{2}$ then$ \\sqrt {3}\\max\\lbrace h_a,h_b,h_c\\rbrace\\ge p$.[/quote]\r\n i forgot .\r\n You're good , not_trig \r\n The problem's true in acute triangle.\r\ni wait to see a nice solution.", "Solution_3": "[hide]\n\nLet $ k$ be the area of $ \\triangle ABC$. Assume WLOG that $ a$, and hence $ \\alpha$, is minimal. ($ a,b,c$ are sides and $ \\alpha,\\beta,\\gamma$ are angles.)\n\nFrom $ 2k = ah_a$, we see that $ h_a$ is maximal. Then, as $ k = rp$, the desired inequality is equivalent to\n\\[ 2\\sqrt {3}r \\ge a\n\\]\nAssume for contradiction that $ 2\\sqrt {3}r < a$. Then we will show that $ a$ is not minimal.\n\nFix $ a$ and $ r$. Then, the maximum of $ \\min(b,c)$ occurs when $ b = c$. (I have yet to prove this; I will edit my post when I do :( )\n\nSo, it suffices to show it for $ b = c$. But then, $ \\angle IBC = \\angle ICB < 30$, so $ \\angle ABC = \\angle ACB < 60$; hence $ AB = BC < BC$, contradiction.\n[/hide]\n\nEDIT: I have proved it, but with a slightly different method.\n\n[hide]\nWe want to show that $ 2\\sqrt{3}r \\ge a$. Considering triangles $ DIC, DIB$, where $ D$ is the point of tangency of the incircle with $ BC$, we see that $ a=r(\\cot \\frac{\\beta}{2} + \\cot \\frac{\\gamma}{2})$. Hence it suffices to show that\n\n$ \\cot \\frac{\\beta}{2} + \\cot \\frac{\\gamma}{2} \\le 2\\sqrt{3}$.\n\nSince $ \\cot$ is convex, the maximum of $ \\cot \\frac{\\beta}{2} + \\cot \\frac{\\gamma}{2}$ for fixed $ \\beta+\\gamma$ occurs when $ \\beta$ and $ \\gamma$ are as far apart as possible. We know that $ \\alpha+\\beta+\\gamma=180$, and the triangle is acute. (We may assume it is just non-obtuse, as $ cot$ is convex and monotonic on $ [0,90]$. Hence, the maximum is either when $ \\alpha = \\beta, \\gamma=180-2\\alpha$, or when $ \\beta = 90-\\alpha, \\gamma=90$.\n\nThe former case is maximal when $ \\alpha \\ge 45$. As $ \\alpha \\le \\beta, \\gamma$, $ \\alpha \\le 60$. So, for $ 45 \\le \\alpha \\le 60$, we want to show that \n\n\\[ \\cot 90-\\alpha + \\cot \\frac{\\alpha}{2} \\le 2\\sqrt{3}\\]\n\nEquality occurs at $ 60$, and when $ \\alpha = 45$, the LHS is $ 2+\\sqrt{2} < 2\\sqrt{3} \\Leftrightarrow 32 < 36$, as $ \\cot \\frac{45}{2} = \\sqrt{\\frac{1+\\cos{45}}{1-\\cos{45}} } = 1+\\sqrt{2}$.. As the LHS is concave in $ \\alpha$, the maximum occurs at the extrema - hence, the inequality is true here.\n\nWhen $ \\alpha \\le 45$, the maximum is when $ \\beta = 90-\\alpha$, $ \\gamma=90$. But then we just need to show that $ \\cot 45-\\frac{\\alpha}{2} + \\cot 45 \\le 2\\sqrt{3}$. As $ \\cot$ is decreasing, it suffices to show that $ \\cot \\frac{45}{2} + \\cot 45 = 2+\\sqrt{2} < 2\\sqrt{3}$. Hence we are done.\n[/hide]", "Solution_4": "I have solved it, in the post above. (I edited it)" } { "Tag": [ "inequalities", "algebra unsolved", "algebra" ], "Problem": "let $x,y>0 ,x+y=1$ , prove \r\n\r\n$\\frac{x}{\\sqrt{y}}+\\frac{y}{\\sqrt{x}} \\geq \\sqrt2$", "Solution_1": "$f(x)=\\frac{1}{\\sqrt{x}}$ is convex in $(0,1)$ so applying Jensen's inequality\r\nwe have: $\\frac{x}{\\sqrt{y}}+\\frac{y}{\\sqrt{x}} \\geq \\frac{1}{\\sqrt{2xy}}$ but\r\n$\\frac{1}{\\sqrt{2xy}} \\geq \\frac{2}{\\sqrt{2}(x+y)}=\\sqrt{2}$, q.e.d.", "Solution_2": "yes ,as you have said it will be easily killed by the power of jensen \r\n\r\ni was trying to find the solution by using the means ,any idea about it?", "Solution_3": "Sure.\r\n\r\nSubstitute $x=a^2$, etc. The LHS is transformed to\r\n\r\n$\\frac {a^3+b^3}{ab}$\r\n\r\nBy Power-Mean, $a^3+b^3\\geq \\frac {\\sqrt {2}}{2}(a^2+b^2)^{\\frac 32}$\r\n\r\nNow $\\frac {a^2+b^2}{ab}\\geq 2$, so we get\r\n\r\n$\\frac {a^3+b^3}{ab}\\geq \\frac {\\sqrt {2}}{2}\\cdot \\frac {a^2+b^2}{ab}\\cdot \\sqrt {a^2+b^2}\\geq \\frac {\\sqrt {2}\\cdot 2}{2}=\\sqrt {2}$", "Solution_4": "thanks i really enjoyed it\r\nthere is no need of using strong tool of jensen\r\nits like killing an ant with a cannon(jensen) , i by myself prefare to kill it by hand(means)", "Solution_5": "having not much time on the olympiad ...I'm not sure whether you would be even aware of killing an ant with a cannon :)" } { "Tag": [], "Problem": "If a,b,c are positive integers such that $ 0 (1) and the proof is completed.\r\n\r\nPS. It is well known that if E, F, are two points on AB, AD respectively, such that the line EF to be antiparallel to BD, with respect to the lines AB, AD, then the line AC bisects the segment EF.\r\n\r\n Kostas Vittas.", "Solution_2": "$AC$ is the symmedian of $\\triangle BAD$ so $\\frac{\\sin{DAC}}{\\sin{BAC}}=\\frac{DC}{BC}=\\frac{DA}{AB}\\rightarrow \\sin{ADC}=\\sin{ABC}$ so either ABCD is cyclic either ABC=ADC in this last case the quadrilateral is a kite :|", "Solution_3": "sorry dears, i dont have much experience to design hard problems :D" } { "Tag": [ "ARML", "AMC", "AIME" ], "Problem": "Does the AMC or the AIME help someone qualify for the ARML???", "Solution_1": "That depends 100% on how your local ARML organization chooses its team-members.", "Solution_2": "Check the [url=http://www.artofproblemsolving.com/Wiki/index.php/How_to_join_an_ARML_team]wiki page.[/url] The information there is not universal, but there is something about selection for quite a few of the teams." } { "Tag": [ "AMC", "AIME", "MATHCOUNTS", "AoPS Books", "\\/closed" ], "Problem": "[quote]Copyright \u00a9 2007 AoPS Incorporated. This page is copyrighted material. You can view and print this page for your own use, but you cannot share the contents of this file with others[/quote]\r\nso i can't post this on the internet, share with friends, etc.? i think this has been asked before, but i just want to double-check.", "Solution_1": "Basically, just don't post it publicly on the web or give it to anyone outside your family.\r\n :P", "Solution_2": "does that mean i cant take problems from the class and use them anywhere else? say...at school?\r\nam i allowed to change the numbers around to make a similar problem? please let me know.", "Solution_3": "Well...it really depends on which class. For example, if it's from the AIME class, you can obviously share those, since they're contest problems. If it's a problem directly from an AOPS book, I think you can change the wording and the numbers around...In general, just don't post things publicly on the internet.", "Solution_4": "You definitely [b]cannot[/b] post it on the internet or make copies of the transcripts or problem sets to give to others. You certainly can discuss problems and ideas from the class with friends.", "Solution_5": "Can you post questions you have on problems in AOPS texts? Or can you post problems from mathcounts books in the mathcounts marathons in the mathcounts forum? Can you modify a problem and then post it?", "Solution_6": "If you have a question about a problem in one of the AoPS books, you can feel free to post it on the AoPS forum to ask about it.", "Solution_7": "[quote=\"rrusczyk\"]You definitely [b]cannot[/b] post it on the internet or make copies of the transcripts or problem sets to give to others. You certainly can discuss problems and ideas from the class with friends.[/quote] \r\nthanks i really appreciate it" } { "Tag": [ "AMC", "USA(J)MO", "USAMO" ], "Problem": "Can anyone post solution official USAMO 2009?", "Solution_1": "The MAA/AMC office will not post the official solutions to the 2009 USAMO until after the grading is completed, on May 11, 2009.\r\n\r\nSteve Dunbar\r\nMAA Director, American Mathematics Competitions", "Solution_2": "Are the solutions out yet?", "Solution_3": "Yes! See http://www.unl.edu/amc." } { "Tag": [ "inequalities", "inequalities unsolved" ], "Problem": "I don't think this is easy and don't sure it is right,but I need it to do a difficult problem:\r\nLet \\sqrt a, \\sqrt b, \\sqrt c are three sides of a triangle.Prove that:\r\n a 2 b+b 2 c+c 2 a-ab 2 -bc 2 -ca 2 \\leq 3/4abc.", "Solution_1": "Dear Anh Cuong\r\n\r\nYour ineq is wrong. Let's take a = 36, b = 9, c = 49 we will find that \r\n\r\na^2b + b^2c + c^2a - ab^2 - bc^2 - ca^2 - (3/4)abc = 2844. \r\n\r\nLet's know your original problem.", "Solution_2": "What a pity!!! :( My original problem is from 'From an IMO problem' in 'Advanced Section -> Inequalities -> Unsolved Problems'.But i want to find a greater condition for that one and it lead me up to this inequality." } { "Tag": [ "inequalities", "trigonometry" ], "Problem": "Let $ x,$ $ y,$ $ z,$ $ \\alpha,$ $ \\beta$ and $ \\gamma$ are real numbers such that $ \\alpha \\plus{} \\beta \\plus{} \\gamma \\equal{} \\pi.$ Prove that\r\n\\[ x^2 \\plus{} y^2 \\plus{} z^2\\geq2xy\\cos\\gamma \\plus{} 2xz\\cos\\beta \\plus{} 2yz\\cos\\alpha\r\n\\]\r\nThere is a short proof. :wink:", "Solution_1": "$ \\sum_{cyc}( x^{2} \\minus{} 2yz \\cos \\alpha) \\equal{} (z \\minus{} (x \\cos \\beta \\plus{} y \\cos \\alpha))^{2} \\plus{} (x \\sin \\beta \\minus{} y \\sin \\alpha)^{2}$", "Solution_2": "Beautiful, pardesi! \r\nWhat about the following problem:\r\nLet $ x,$ $ y$, $ z,$ $ t$ $ \\alpha,$ $ \\beta,$ $ \\gamma$ and $ \\delta$ are real numbers such that $ \\alpha \\plus{} \\beta \\plus{} \\gamma \\plus{} \\delta \\equal{} \\pi.$ Prove that\r\n\\[ x^2 \\plus{} y^2 \\plus{} z^2 \\plus{} t^2\\geq\\sqrt2xy\\cos\\alpha \\plus{} \\sqrt2yz\\cos\\beta \\plus{} \\sqrt2zt\\cos\\gamma \\plus{} \\sqrt2tx\\cos\\delta\r\n\\]\r\nI haven't short proof for this one." } { "Tag": [ "logarithms", "inequalities", "inequalities unsolved" ], "Problem": "", "Solution_1": "Taking logarithms on both sides, the inequality is equivalent to\r\n\r\n$ (x^2 \\plus{} 2yz) log x \\plus{} (y^2 \\plus{} 2zx) log y \\plus{} (z^2 \\plus{} 2xy) log z \\geq (xy\\plus{}yz\\plus{}zx) (log x \\plus{} log y \\plus{} log z)$\r\nSending all terms to the lefthand side, we wish to prove that\r\n\r\n$ (x^2 \\minus{} xy \\minus{} xz \\plus{} yz) log x \\plus{} (y^2 \\minus{} yz \\minus{} yx \\plus{} xz) log y \\plus{} (z^2 \\minus{} zx \\minus{} zy \\plus{} xy) log z \\geq 0$\r\n\r\n$ (x\\minus{}y)(x\\minus{}z) log x \\plus{} (y\\minus{}x)(y\\minus{}z) log y \\plus{} (z\\minus{}x)(z\\minus{}y) log z \\geq 0$\r\n\r\nSince this inequality is symmetric in the three variables $ x,y,z$, assume that $ x \\geq y \\geq z$\r\n\r\nDue to the assumption $ x,y,z > 1$, $ log x , log y , log z > 0$.\r\n\r\nNow, the inequality holds because\r\n$ (x\\minus{}y)(x\\minus{}z) log x \\plus{} (y\\minus{}x)(y\\minus{}z) log y \\plus{} (z\\minus{}x)(z\\minus{}y) log z$\r\n$ \\equal{}(x\\minus{}y)((x\\minus{}y)\\plus{}(y\\minus{}z)) log x \\plus{} (y\\minus{}x)(y\\minus{}z) log y \\plus{} (z\\minus{}x)(z\\minus{}y) log z$\r\n$ \\equal{}(x\\minus{}y)^2 log x \\plus{} (x\\minus{}y)(y\\minus{}z)(logx \\minus{} logy) \\plus{} (z\\minus{}x)(z\\minus{}y) log z \\geq 0$.", "Solution_2": "thanks for help qwerty414", "Solution_3": "$ x^{x^2+2yz}y^{y^2+2xz}z^{z^2+xy}\\geq \\ xyz^{\\frac{x^2+y^2+z^2+2xy+2yz+2zx}{3}}=xyz^{\\frac{(x+y+z)^2}{3}\\geq xyz^{xy+yz+zx}}$\r\nwe can generalise it on this,\r\nfor all nonegative numbers $ x_1,x_2,...,x_n$. prove that:\r\n$ x_1^{x_1}x_2^{x_2}.....x_n^{x_n} \\geq (x_1x_2...x_n)^{\\frac{x_1+...+x_n}{n}}$", "Solution_4": "x\u2260x^2+2yz,...." } { "Tag": [ "calculus", "calculus computations" ], "Problem": "you have a polymerase that is making copies of a single strand of DNA in a closed system (say\r\nthe strand of DNA is double stranded and is 300 base pairs long, though I\r\ndon't think the total length matters for our problem). So, the original\r\ncopy of the double stranded template becomes 2 double stranded molecules\r\nduring the first cycle of PCR, then the 2 double stranded molecules\r\nproduced during the first cycle of PCR become 4 double stranded molecules\r\nduring the second cycle of PCR, and so on. The polymerase makes copies of\r\nthe DNA by inserting the correct complementary base across from each base\r\nin the template strand until it reaches the end of the 300bp template\r\nstrand, but the polymerase also makes mistakes- 1/10,000 times it inserts\r\na base, it will insert the wrong base. Now, say we are interested in a\r\nparticular base- say position 150 out of 300 is a \"T\" that we are\r\nparticularly interested in because it leads to cancer. How frequently\r\nwill the majority product at the end of 40 cycles of PCR be \"wrong\" (i.e.\r\nhave a base other than T at that position). Realize that the A that pairs\r\nwith that T is equally important to consider (the A mutating to some other\r\nbase would also make the strand \"wrong\" at that position).", "Solution_1": "okay i really dont understand exactly whats going on, but im thinking the tools you need for this are\r\n\r\nmarkov chains and the concept of coupling ?" } { "Tag": [ "limit" ], "Problem": "$ S \\equal{} 1 \\plus{} \\frac{5}{11} \\plus{} \\frac{12}{11^2} \\plus{} \\frac{22}{11^3} \\plus{} \\frac{35}{11^4} \\plus{} \\cdots$\r\n \r\nFind the value of $ S$ numerically in the form of $ x \\plus{} \\frac{y}{z}$", "Solution_1": "$ S \\equal{} \\frac {1}{2}\\lim_{n\\to\\infty} \\sum_{k \\equal{} 1}^n (3k^2 \\minus{} k)\\left(\\frac {1}{11}\\right)^{k \\minus{} 1}$\r\n\r\n$ \\equal{}\\frac{1}{2}\\lim_{n\\to\\infty} \\sum_{k\\equal{}1}^n \\{f(k)\\minus{}f(k\\plus{}1)\\}$, where $ f(k)\\equal{}\\left[\\frac{33}{10}(k\\minus{}1)^2\\plus{}\\frac{154}{25}(k\\minus{}1)\\plus{}\\frac{11\\cdot 143}{500}\\right]\\left(\\frac{1}{11}\\right)^{k\\minus{}1}$\r\n\r\n$ \\equal{}\\frac{1}{2}\\lim_{n\\to\\infty}\\left\\{\\frac{11\\cdot 143}{500}\\minus{}f(n)\\right\\}$\r\n\r\n$ \\equal{}\\frac{11\\cdot 143}{1000}$\r\n\r\n$ \\equal{}1\\plus{}\\frac{573}{1000}$", "Solution_2": "[hide]Take $ S\\minus{}S/11$. This is equal to \n$ 1\\plus{}4/11\\plus{}7/11^2\\plus{}10/11^3\\plus{}13/11^4 \\dots \\equal{} X$.\nNow take $ X\\minus{}X/11$. This is equal to \n$ 1\\plus{}3/11\\plus{}3/11^2\\plus{}3/11^3 \\dots \\equal{} 1\\plus{} (3/10) \\equal{} 13/10.$\nSo $ X\\minus{}X/11\\equal{}10X/11\\equal{}13/10$. Hence, $ X\\equal{}143/100$.\n$ S\\minus{}S/11\\equal{}10S/11\\equal{}X\\equal{}143/100$.\nSo $ S\\equal{}1573/1000 \\equal{} 1 \\plus{} 573/1000$\n[/hide][/hide]", "Solution_3": "Our answer is same! :lol:" } { "Tag": [], "Problem": "allora le avete fatte le gare stamani?\r\nio penso di aver fatto una strage dopo un po avevo gi\u00e0 il cervello fuso..", "Solution_1": "anch'io :( :(", "Solution_2": "e cmq si potrebbero anche sbrigare a pubblicare le sol no? :?", "Solution_3": "Quanto pensate di aver fatto all'incirca?\r\nIn effetti l'archimede di quest'anno era pi\u00f9 difficile del solito...", "Solution_4": "ehm diciamo male... 50 pt veramente schifoso come punteggio dato che mi ero allenata su quelli precedenti ed erano andati molto meglio....vabe cmq era la prima volta ;)" } { "Tag": [], "Problem": "1. Find the sum of the squares of the real roots of:\r\n\r\nx^20 - x^18 - x^16 - ... - x^2 - 2\r\n\r\n2. Calculate (without using a calculator) the largest integer less than:\r\n\r\n:sqrt:2500 - :sqrt:2501 + :sqrt:2502 - ... - :sqrt:2999 + :sqrt:3000", "Solution_1": "1. We substitute y=x^2 to get y^10-y^9-y^8-...-2=0, or y^10-1=y^9+y^8+...+y+1=(y^10-1)/(y-1), which can only be equal when y^10-1=0 or y-1=1. In the first case, when y is real, y= :pm: 1, and in the second case, y=2. But y=-1 doesn't work because then x=:pm: i, which isn't real. Thus we have x=:pm: 1 and x=:pm: :rt2:, so the sum of the squares of the real roots is 6.", "Solution_2": "Not quiiite right for the first one. Take a look at it again.", "Solution_3": "Ah yes, sorry. That doesn't work when y=1 because then we have 0=10, which is false. Thus the only solution are :pm: :rt2:, so the sum of the squares of the roots is 4.", "Solution_4": "right. :)\r\n\r\nYou could also refer to Descartes' Rule of Signs to tell that there are only two real roots, one positive and one negative.", "Solution_5": "Just looking back at some old problems, saw nobody tried the second part here..\r\n:sqrt:2500 is 50. If we put the rest of the terms as (:sqrt:2502 - :sqrt:2501)+(:sqrt:2504 - :sqrt:2503)...\r\nWell :sqrt:(a+1)-:sqrt:a = 1/(:sqrt:(a+1) + :sqrt:a).. (just after multiplying by its conjugate)\r\n\r\nSo we get 50 + 1/(:sqrt:2502 + :sqrt:2501) + ... + 1/(:sqrt:3000 + :sqrt:2999).\r\n\r\nEach of those 1/ terms is certainly less than 1/(50+50) = 1/100, so the sum is less than 50 + 250/100 (since there are 250 terms there) which is 52.5.\r\nEach of them is also a lot bigger than 1/(60+60) = 1/120, so the sum is greater than 50 + 250/120 which is bigger than 52..\r\n\r\nSo the answer is 52." } { "Tag": [ "probability", "function", "probability and stats" ], "Problem": "I'm not sure how to go about this - any help would be appreciated:\r\n\r\nSuppose we choose a random point in the interval (-2,1) and denote the distance to 0 by X. Show that X is a continuous random variable.", "Solution_1": "That's a little weird.\r\nShow that any interval in the range of possible values has a positive probability? It's not hard to calculate this probability.", "Solution_2": "The thing is, the phrase \"choose a random point in the interval $ ( \\minus{} 2,1)$\" is a little loose and imprecise, but the most likely interpretation is to make it a well-known type of random variable: let $ U$ be uniformly distributed on $ ( \\minus{} 2,1).$ That's a continuous distribution; its density is $ \\frac13$ on that interval and zero off of it; the cumulative distribution function is a piecewise linear function whose graph includes the points $ ( \\minus{} 1,0)$ and $ (2,1).$\r\n\r\nNow you want to let $ X \\equal{} |U|.$\r\n\r\n[hide=\"Oops. Not sure what I was thinking.\"]Theorem: Suppose $ U$ is a random variable with an absolutely continuous distribution. (You probably don't know why I said \"absolutely continuous,\" but it's the same as saying that it has a density). Suppose $ g$ is a Lipschitz function defined on the range of $ U.$ Let $ X \\equal{} g(U).$ Then $ X$ has an absolutely continuous distribution.\n[/hide]\r\n---\r\n\r\nIn this particular case, we can be quite concrete. We can just go ahead and calculate the cdf and the density of $ X.$\r\n\r\nWhat is $ P(X\\le x)?$\r\n\r\nIf $ x\\le0,$ this is zero.\r\n\r\nIf $ 0 < x\\le1,\\ P(X\\le x) \\equal{} P( \\minus{} x\\le U\\le x) \\equal{} \\frac {2x}3.$\r\n\r\nIf $ 1 < x\\le 2,\\ P(X\\le x) \\equal{} P( \\minus{} 1 < U\\le x) \\equal{} \\frac {x \\plus{} 1}3.$\r\n\r\nIf $ x > 2,\\ P(X\\le x) \\equal{} 1.$\r\n\r\nSo that's the cdf, and it is continuous. Differentiate to find that the density is\r\n\r\n$ f_X(x) \\equal{} \\begin{cases}\\frac23, & 0 < x < 1 \\\\\r\n\\frac13, & 1 < x < 2 \\\\\r\n0, & \\text{otherwise}\\end{cases}.$", "Solution_3": "[quote=\"Kent Merryfield\"]\nTheorem: Suppose $ U$ is a random variable with an absolutely continuous distribution. (You probably don't know why I said \"absolutely continuous,\" but it's the same as saying that it has a density). Suppose $ g$ is a Lipschitz function defined on the range of $ U.$ Let $ X \\equal{} g(U).$ Then $ X$ has an absolutely continuous distribution.\n[/quote]\r\nErm... How about $ g\\equal{}\\text{const}$ :? :o", "Solution_4": "But this leads to an interesting question (possibly more interesting than the original one): what conditions on a map $ f\\colon \\mathbb R^n\\to\\mathbb R^n$ imply that it pushes any absolutely continuous measure to another such?" } { "Tag": [ "geometry", "vector", "complex number geometry", "geometry proposed" ], "Problem": "Hi,\r\nI'm pretty new to complex-number geometry.this is an easy problem on my book. I read the solution but have some troubles understanding it :blush: Ok, here's how it goes\r\nA regular n-gon $A_{1}...A_{n}$ is inscribed in a circle with center O and radius R. X is any point with $d=|OX|$. Then $\\sum^{n}_{i=1}|A_{i}X|^{2}=n(R^{2}+d^{2})$.\r\n\r\nSolution:We have $A_{1}+...+A_{n}=O$, $|A_{i}X|^{2}=A_{i}^{2}+X^{2}-2A_{i}\\cdot X=R^{2}+d^{2}-2A_{i}\\cdot X$, and $|A_{1}X|^{2}+..+|A_{n}X|^{2}=n(R^{2}+d^{2}).$\r\n\r\nIt's this part that confuses me $|A_{i}X|^{2}=A_{i}^{2}+X^{2}-2A_{i}\\cdot X=R^{2}+d^{2}-2A_{i}\\cdot X$ what operation rule is used here? $A_{i}^{2}=R^{2}$ why? $|A_{i}X|^{2}=(A_{i}X)^{2}$ how? :( \r\n\r\nCan anyone help me? Thanks :)", "Solution_1": "[quote=\"andy_tok\"] $A_{i}^{2}=R^{2}$ [/quote]\r\nBecause modul of vector $A_{i}$ equal $R$.", "Solution_2": "[quote=\"N.T.TUAN\"][quote=\"andy_tok\"] $A_{i}^{2}=R^{2}$ [/quote]\nBecause modul of vector $A_{i}$ equal $R$.[/quote]\r\n\r\nThanks. but does $A_{i}^{2}=|A_{i}|^{2}$? for example $A_{i}=1+i$, $A_{i}^{2}=2i$ but $|A_{i}|^{2}=2$", "Solution_3": "[quote=\"andy_tok\"]but does $A_{i}^{2}=|A_{i}|^{2}$? [/quote]\r\nYes, but for vectors!", "Solution_4": "Thanks. ok, the solution is using vectors. Because the chapter before the questions is about complex-number geometry, I though the $A_{i}$ and $X$ are points on a complex plane :blush:", "Solution_5": "[quote=\"andy_tok\"]Thanks. ok, the solution is using vectors. Because the chapter before the questions is about complex-number geometry, I though the $A_{i}$ and $X$ are points on a complex plane :blush:[/quote]\r\n\r\nPlease read carefuly, before send question to everyone. I thinked a lot of :P\r\n\r\nSorry by English of mine :D" } { "Tag": [ "inequalities", "inequalities unsolved", "inequalities proposed", "inequalities open", "convex-concave inequalities" ], "Problem": "Prove that for all real numbers $x,y,z \\in [1,2]$ the following inequality always holds:\r\n\\[(x+y+z)(\\frac{1}{x}+\\frac{1}{y}+\\frac{1}{z})\\geq 6(\\frac{x}{y+z}+\\frac{y}{z+x}+\\frac{z}{x+y}). \\]\r\nWhen does the equality occur?", "Solution_1": "$(\\sum x)(\\sum\\frac{1}{x})-9\\geq 6\\sum\\frac{x}{y+z}-9$\r\n\r\n<=> $\\sum\\frac{(x-y)^{2}}{xy}\\geq 3\\sum\\frac{(x-y)^{2}}{(x+z)(y+z)}$\r\n\r\nAssume $2\\geq x\\geq y\\geq z\\geq 1$, it is easy to show\r\n\r\n$\\frac{(y-z)^{2}}{yz}\\geq \\frac{3(y-z)^{2}}{(y+x)(z+x)}$\r\n\r\n$\\frac{1}{xz}\\geq \\frac{3}{(x+y)(z+y)}$\r\n\r\nThen we just have to prove\r\n\r\n$(x-z)^{2}(\\frac{1}{xz}-\\frac{3}{(x+y)(z+y)})\\geq (x-y)^{2}(\\frac{3}{(x+z)(y+z)}-\\frac{1}{xy})$\r\n\r\n<= $\\frac{1}{xz}-\\frac{3}{(x+y)(z+y)}\\geq \\frac{3}{(x+z)(y+z)}-\\frac{1}{xy}$\r\n\r\n<=> $(y+z)^{2}(y+z)(z+x)\\geq 3xyz(2x+y+z)$\r\n\r\n($(y+z)^{2}\\geq 4yz$) <= $4(y+x)(z+x)\\geq 3x(2x+y+z)$\r\n\r\n<=> $4yz+x(y+z)\\geq 2x^{2}$\r\n\r\n<= $2y\\geq x, 2z\\geq x$ which is obvious.\r\n\r\nThe equality occur that $(x,y,z)=(t,t,t), (1,1,2)$", "Solution_2": "[quote=\"radio\"][quote=\"libra_gold\"]\n\n$\\sum\\frac{(x-y)^{2}}{xy}\\geq 3\\sum\\frac{(x-y)^{2}}{(x+z)(y+z)}$\n\n[/quote]\n\nI don't think this line is correct :wink:[/quote]\r\nI think, it's correct:\r\n$\\sum\\frac{(x-y)^{2}}{xy}\\geq 3\\sum\\frac{(x-y)^{2}}{(x+z)(y+z)}\\Leftrightarrow\\sum_{cyc}(x-y)^{2}S_{z}\\geq0,$ where\r\n$S_{x}=\\frac{1}{yz}-\\frac{3}{(x+y)(x+z)},$ $S_{y}=\\frac{1}{xz}-\\frac{3}{(x+y)(y+z)},$ $S_{z}=\\frac{1}{xy}-\\frac{3}{(y+z)(x+z)}.$\r\nLet's assume $2\\geq x\\geq y\\geq z\\geq1.$ Hence, $S_{x}\\geq0$ and $S_{y}\\geq0\\Leftrightarrow y^{2}+y(x+z)-2xz\\geq0.$\r\nBut $y^{2}+y(x+z)-2xz\\geq z^{2}+z(x+z)-2xz=2z^{2}-xz\\geq2z-xz=z(2-x)\\geq0.$\r\nThus, $S_{y}\\geq0.$ Hence, $\\sum_{cyc}(x-y)^{2}S_{z}\\geq(x-z)^{2}S_{y}+(x-y)^{2}S_{z}\\geq(x-y)^{2}(S_{y}+S_{z}).$\r\nRemain to prove that $S_{y}+S_{z}\\geq0.$ But $S_{y}+S_{z}\\geq0\\Leftrightarrow$\r\n$\\Leftrightarrow\\frac{1}{xz}-\\frac{3}{(x+y)(y+z)}+\\frac{1}{xy}-\\frac{3}{(y+z)(x+z)}\\geq0\\Leftrightarrow$\r\n$\\Leftrightarrow\\frac{y+z}{xyz}-\\frac{3(2x+y+z)}{(x+y)(x+z)(y+z)}\\geq0\\Leftrightarrow$\r\n$\\Leftrightarrow(y+z)^{2}x^{2}+(y+z)^{3}x+(y+z)^{2}yz-6yzx^{2}-3(y+z)yzx\\geq0\\Leftrightarrow$\r\n$\\Leftrightarrow(y-z)^{2}x^{2}+(y-z)^{2}(y+z)x+(y+z)^{2}yz+(y+z)yzx-2yzx^{2}\\geq0.$\r\nBut $(y+z)^{2}+(y+z)x-2x^{2}\\geq4+2x-2x^{2}=2(1+x)(2-x)\\geq0.$ \r\nId est, our inequality is proved. :)", "Solution_3": "this was already discussed [url=http://www.mathlinks.ro/Forum/viewtopic.php?highlight=triangle&t=84210]here[/url]", "Solution_4": "[quote=\"radio\"][quote=\"libra_gold\"]\n\n$\\sum\\frac{(x-y)^{2}}{xy}\\geq 3\\sum\\frac{(x-y)^{2}}{(x+z)(y+z)}$\n\n[/quote]\n\nI don't think this line is correct :wink:[/quote]\r\n\r\nThis one is equivalent to the original one. Do you think so?", "Solution_5": "[quote=radio]Prove that for all real numbers $x,y,z \\in [1,2]$ the following inequality always holds:\n\\[(x+y+z)(\\frac{1}{x}+\\frac{1}{y}+\\frac{1}{z})\\geq 6(\\frac{x}{y+z}+\\frac{y}{z+x}+\\frac{z}{x+y}). \\]\nWhen does the equality occur?[/quote]\n[url=http://www.artofproblemsolving.com/community/c6h84210p487332]Vietnam Team Selection Tests 2006, Day 2, Problem 1[/url]\n[b]Reverse inequality[/b]\nProve that for all real numbers $x,y,z \\in [1,2]$ the following inequality always holds:\n\\[(x+y+z)(\\frac{1}{x}+\\frac{1}{y}+\\frac{1}{z})\\leq 10. \\][b][url=http://www.artofproblemsolving.com/community/c6h478312p2678288]Inference[/url][/b]\nProve that for all real numbers $x,y,z \\in [1,2]$ the following inequality always holds:\n$$\\frac{x}{y+z}+\\frac{y}{z+x}+\\frac{z}{x+y}\\leq\\frac{5}{3}$$\n[url=http://www.artofproblemsolving.com/community/c6h623533p3731246]here,[/url][url=http://www.artofproblemsolving.com/community/c6h422978p2391981]here,[/url][url=http://www.artofproblemsolving.com/community/c6h538481p3096039]here,[/url][url=http://www.artofproblemsolving.com/community/c6h344149p1841164]here,[/url][url=http://www.artofproblemsolving.com/community/c6h398201p2215590]here,[/url][url=http://www.artofproblemsolving.com/community/c4h1061866p4682502]here[/url]", "Solution_6": " If $ x,y $ and $z$ are sides of triangle. Prove that$$(x+y+z)(\\frac{1}{x}+\\frac{1}{y}+\\frac{1}{z})\\geq 6(\\frac{x}{y+z}+\\frac{y}{z+x}+\\frac{z}{x+y}).$$[url=http://www.artofproblemsolving.com/community/c275h116646p662466]Romania[/url]", "Solution_7": "Let $a,b,c>0.$ Prove that\n$$(a+b+c)(\\frac{1}{a}+\\frac{1}{b}+\\frac{1}{c}) \\geq 9\\sqrt{\\frac{a^2+b^2+c^2}{ab+bc+ca}},$$\n$$\\sqrt{\\frac{a(a+c)}{b(b+c)}}+\\sqrt{\\frac{b(b+a)}{c(c+a)}}+\\sqrt{\\frac{c(c+b)}{a(a+b)}}+\\frac{8abc}{(a+b)(b+c)(c+a)} \\geq 4,$$\n$$\\frac{a^3+b^3+c^3}{abc}+36\\left(\\frac{a}{a+b}+\\frac{b}{b+c}+\\frac{c}{c+a}\\right)\\geq57,$$\n$$\\frac{b+c}{a}+\\frac{c+a}{b}+\\frac{a+b}{c}\\geq 2\\sqrt{(ab+bc+ca)\\left(\\frac{1}{a^2}+\\frac{1}{b^2}+\\frac{1}{c^2}\\right)}.$$\n[size=50](Rahim Shahbazov, CYCLIC-INEQUALITY-680 676 675 673)[/size]", "Solution_8": "[quote=sqing]Let $a,b,c>0.$ Prove that\n$$(a+b+c)\\left(\\frac 1{a}+\\frac 1{b}+\\frac 1{c}\\right) \\geq 9\\sqrt{\\frac{a^2+b^2+c^2}{ab+bc+ca}},$$\n[hide=*]$$\\sqrt{\\frac{a(a+c)}{b(b+c)}}+\\sqrt{\\frac{b(b+a)}{c(c+a)}}+\\sqrt{\\frac{c(c+b)}{a(a+b)}}+\\frac{8abc}{(a+b)(b+c)(c+a)} \\geq 4,$$\n$$\\frac{a^3+b^3+c^3}{abc}+36\\left(\\frac{a}{a+b}+\\frac{b}{b+c}+\\frac{c}{c+a}\\right)\\geq57,$$\n$$\\frac{b+c}{a}+\\frac{c+a}{b}+\\frac{a+b}{c}\\geq 2\\sqrt{(ab+bc+ca)\\left(\\frac{1}{a^2}+\\frac{1}{b^2}+\\frac{1}{c^2}\\right)}.$$\n[size=50](Rahim Shahbazov, CYCLIC-INEQUALITY-680 676 675 673)[/size][/hide][/quote]\nLet $a,b,c$ be positive real numbers . Prove that$$(a+b+c)\\left(\\frac 1{a}+\\frac 1{b}+\\frac 1{c}\\right) \\geq 5+\\frac{4(a^2+b^2+c^2)}{ab+bc+ca}.$$\n[b]Proof:[/b]\n$$\\sum_{cyc}\\frac {bc(b+c)}{a}=\\sum_{cyc}\\left(\\frac {c}{b}+\\frac {b}{c}\\right)a^2\\geq \\sum_{cyc}a^2\\iff \\sum_{cyc}\\frac {(b+c)}{a} \\sum_{cyc}ab\\geq 4 \\sum_{cyc}a^2+2 \\sum_{cyc}ab\\iff$$\n$$(a+b+c)\\left(\\frac 1{a}+\\frac 1{b}+\\frac 1{c}\\right) \\geq 5+\\frac{4(a^2+b^2+c^2)}{ab+bc+ca}$$\nLet $a,b,c$ be positive real numbers . Prove that$$(a+b+c)\\left(\\frac 1{a}+\\frac 1{b}+\\frac 1{c}\\right) \\geq 3+6\\left(\\frac{a+b+c}{\\sqrt{ab}+\\sqrt{bc}+\\sqrt{ca}}\\right)^2.$$\n[url=https://artofproblemsolving.com/community/c6h234212p1295275]here[/url]" } { "Tag": [ "probability" ], "Problem": "$ n$ different books $ (n\\geq 3)$ are put at random in a shelf. Among these books there is a book $ A$ and a book $ B$. Determine the probability of which there are exactly $ r$ books between these two books.\r\n\r\n[hide=\"My result\"]\n\n$ \\frac{2}{n(n\\minus{}1)}$[/hide]", "Solution_1": "I dont think you result can be correct because depending on the size of $ r$ there will be a differnet probability.\r\nConsider: $ r \\equal{} n \\minus{} 2$ then the books must be on either end of the shelf. But if $ r \\equal{} 0$ then there are many places ($ n \\minus{} 1$ places to be precise) where this can occur.\r\n\r\nSo I believe the answer is; $ P_n(r) \\equal{} \\dfrac{2(n\\minus{}(r\\plus{}1))}{n(n\\plus{}1)}$", "Solution_2": "[quote=\"ocha\"]So I believe the answer is; $ P_n(r) \\equal{} \\dfrac{2(n \\minus{} (r \\plus{} 1))}{n(n \\plus{} 1)}$[/quote]\r\n\r\n[b]ocha[/b] i think you are right, i saw my mistake.\r\n\r\nBut i got \r\n\r\n[list]$ P_n(r) \\equal{} \\dfrac{2(n \\minus{} (r \\plus{} 1))}{n(n \\minus{} 1)}$[/list]", "Solution_3": "Ah, yes that is what i meant :P", "Solution_4": "And how did you get that answer?", "Solution_5": "There are $ {n\\minus{}2\\choose r}$ ways to choose $ r$ books.\r\n\r\nHence, consider the block with $ r\\plus{}2$ books $ (A,\\,B$ and $ r$ books). There are $ [n\\minus{}(r\\plus{}2)\\plus{}1]!\\equal{}(n\\minus{}r\\minus{}1)!$ permutations considering $ n$ books with the block mentioned. Besides, we have $ 2!\\equal{} 2$ permutations of the books $ A$ and $ B$, and $ r!$ permutations of the $ r$ books.\r\n\r\nThe probability desired is given by\r\n\r\n[list]$ \\frac{{n\\minus{}2\\choose r}\\cdot(n\\minus{}r\\minus{}1)!\\cdot 2\\cdot r! }{n!}$[/list] \n\nRemark:\n[list]$ {n\\choose p}\\equal{}\\frac{n!}{(n\\minus{}p)!p!}$[/list]\r\n\r\nSorry for my poor english.", "Solution_6": "Faster is...\r\n\r\nThere are $ (n\\minus{}2)!$ ways to order all of the books except $ A$ and $ B$. There are $ n\\minus{}r\\minus{}1$ ways to choose the two spots to insert $ A$ and $ B$ so that there are $ r$ books between the two. There are two ways to place $ A$ and $ B$ into these spots ($ A$ first or $ B$ first). Thus the number of ways to place the $ n$ books with $ r$ between $ A$ and $ B$ is just\r\n$ 2(n\\minus{}r\\minus{}1)(n\\minus{}2)!$.\r\nand the desired probability is\r\n$ \\dfrac{2(n\\minus{}r\\minus{}1)(n\\minus{}2)!}{n!} \\equal{} \\dfrac{2(n\\minus{}r\\minus{}1)}{n(n\\minus{}1)}$." } { "Tag": [ "inequalities proposed", "inequalities" ], "Problem": "Let $x, y, z > -1$. Prove that \\[ \\frac{1+x^2}{1+y+z^2} + \\frac{1+y^2}{1+z+x^2} + \\frac{1+z^2}{1+x+y^2} \\geq 2. \\]\r\n[i]Laurentiu Panaitopol[/i]", "Solution_1": "Continue discussion at http://www.mathlinks.ro/Forum/viewtopic.php?t=6150" } { "Tag": [ "Mafia", "pre100" ], "Problem": "Welcome to Middle Earth! This mafia game has 12 players, which is relatively few so we'll hopefully have a quick game. Let me state a few rules for this game. Most will be pretty familiar: [list][*]As in any mafia game, your team wins when all other teams are eliminated (or when your team has a winning strategy).\n[*]It is [b]strictly forbidden[/b] to modify posts once someone has replied to them! \n[*]Everyone is on the block, at any moment. So you can simply vote aye on someone and/or retract your vote later. Voting nay is pointless, but allowed (e.g. to stress your opinion).\n[*]To prevent stalling games, we will have minority lynching. That means: next to the normal lynching, if a day lasts for more than 336 hours (14 days), the day will end. At that point, the person having most ayes is lynched, if that amount is stricly greater than 1. If two or more persons tie for the highest, the person who got his last aye first is lynched. If both are made in the same post, I will take them in the order written.\n[*]Just so you know what to claim, Citizens are called Ents in this game: [quote]You are an Ent. You belong to the good team.\nAt night, you may target one player. If Saruman is targetted by at least half of the Ents alive, he dies.[/quote]And of course you may claim anything else as well.\n[*]Sometimes lynches may fail. This can happen on any team, given the right circumstances. In this case day will simply end without lynch. Same for nightkills.\n[*]Day lynches will be revealed if succesful, nightkills will not.\n[*]If there are any questions, please ask them to me in PM. By asking them here, you spread clues on your identity.[/list]\n\nIt is day 1. Players remaining: [b]12[/b][list][*]142857[*]abacadaea[*]ajai[*]Ars[*]BoesFX[*]CheeseIsGood[*]hunter34[*]miyomiyo[*]mz94[*]pianoforte[*]pieterminate[*]Temperal[/list]\n\nRole PM's are sent. Don't be surprised if you don't understand it completely, [b]mystery[/b] will be part of the fun in this game. :)\n\nYou may start discussing.", "Solution_1": "yo so I believe that during the day we should agree on a player for the ents to target, possibly 2\r\nif we do that it is more likely that we will hit saruman at night.\r\n\r\n the 2 people would work if there is an even number and both have the same amount we could check both\r\n\r\nnow this is all I have to say at the moment", "Solution_2": "Confirming that I'm here. I'm excited for this game! :lol: \r\n\r\nAbout the editing rule, it has been seen in prior games (take Game 21, for example) that when bubka edited his roleclaim post even before anybody responded, it caused an uproar in the course of the game. This gives an unfair advantage to those who have enough time on their hands to be constantly on this thread. Thoughts? Ideas?", "Solution_3": "[quote=\"miyomiyo\"]About the editing rule, it has been seen in prior games (take Game 21, for example) that when bubka edited his roleclaim post even before anybody responded, it caused an uproar in the course of the game. This gives an unfair advantage to those who have enough time on their hands to be constantly on this thread. Thoughts? Ideas?[/quote]I highly encourage not to edit at all, but the software doesn't keep track of editing before anyone replied, so I will not punish it since I have no proof. Do at own risk of making yourself suspicious.", "Solution_4": "[quote=\"Ars\"]yo so I believe that during the day we should agree on a player for the ents to target, possibly 2\nif we do that it is more likely that we will hit saruman at night.[/quote] I would say one target a night (whoever is most suspicious). Two would be too difficult because we don't know who the Ents are. For example, if we think two people are the mafia, we can't evenly divvy up the unknown Ents. Also, don't call me suspicious for saying this, but in this game maybe a few Ents dying is sometimes a good thing. (That way it's easier to get half the Ents to do something.)", "Solution_5": "Heh, I wonder who Sauron is. I wish we knew all the roles...", "Solution_6": "[quote=\"Temperal\"]Heh, I wonder who Sauron is. I wish we knew all the roles...[/quote]We're assuming there is a Sauron (which is rather obvious, I know). Just to get everybody talking and to rise some suspicions, I suggest everybody explain what they think this third power is and what that third power can do. I just don't want this to turn into another endless Day 1.", "Solution_7": "[quote=\"miyomiyo\"][quote=\"Temperal\"]Heh, I wonder who Sauron is. I wish we knew all the roles...[/quote]We're assuming there is a Sauron (which is rather obvious, I know). Just to get everybody talking and to rise some suspicions, I suggest everybody explain what they think this third power is and what that third power can do. I just don't want this to turn into another endless Day 1.[/quote]\r\n\r\nMaybe im just being stupid, but where is this \"third power\" mentioned?...", "Solution_8": "Uh, if you're stupid, then I'm stupid too. Unless it's something in miyomiyo's role PM...?", "Solution_9": "hmm... miyomiyo, explain yourself...", "Solution_10": "Lets just target one person each night, randomly. But what is a Sauron? For all we know, it could be a good role.", "Solution_11": "Sauron was the big evil Eye guy from Lotr, so Id assume he's evil", "Solution_12": "Lol, I never read/watched LOTR. And I noticed we've been spelling Saruman as Sauron.\r\n\r\nSo far, I think miyomiyo is the most suspicious.", "Solution_13": "Sauron is a great evil EYE thing\r\nSarumon is a powerful, but corrupt, wizard\r\nThey're different characters\r\n\r\nAnd I agree, miyomiyo is the most suspicious. But let's give him a chance to defend himself...", "Solution_14": "No, I was just following up on Temperal's guess that there was a Sauron. I wanted to get people talking about what they think this \"third power\" (probably Sauron) is. I do NOT know that there exists a third power, but I'm just guessing (like temperal was). Don't leap to conclusions and assume I'm guilty.", "Solution_15": "What? Ars and Piano were both lying???", "Solution_16": "Well of course I was lying. Honestly, I'm surprised that Temperal was sarumon. I would be surprised if Ars was not gandalf.\r\n\r\nHunter, how many more steps toward Mt. Doom were there?\r\n\r\nDarnit, I probably woulda won too.", "Solution_17": "I did something stupid. the night before hunter was almost lynched i had put on the the ring. i lied about the powers of the ring\r\n1. the ring also protects from lynch\r\n2. the ring can only be used once\r\nso i obviously couldnt reveal 2....and 1. i forgot about it so i didnt really need to roleclaim.\r\n\r\nthat was silly of me........but the answer was 2/4. not sure about last night.", "Solution_18": "eh, i actually targeted you that night (2nd night), though i didn't even notice until i went through my sent pms\r\n\r\nanyways, after hunters death i would be able to kill anyone at night, if no other effects. and there would be no way to kill me.", "Solution_19": "BoesFX was the Witch-King of Angmar. ;) Ars was the real Gandalf.\r\n\r\nI will post the other roles and the triggered effects if you like, when I have more time.", "Solution_20": "[quote=\"pianoforte\"]Well of course I was lying. Honestly, I'm surprised that Temperal was sarumon. I would be surprised if Ars was not gandalf.\n\nHunter, how many more steps toward Mt. Doom were there?\n\nDarnit, I probably woulda won too.[/quote]\r\n\r\n\r\nWith this night included, 1 step.\r\n\r\nI guess I have to pay attention to the other mafia game.", "Solution_21": "no peter I think you got me confused with someone I was a foolish dwarf that wasn't in the game at all and wanted to kill everyone\r\nBoesFX was Gandalf and was my other personality and mafia partner\r\nI made 6 nightkills a night of people not in the game\r\nand couldn't be killed\r\nI also investigated everyone.\r\nWhen killed I would comeback the next day", "Solution_22": "Ok, I am the Witch King of Angmar....\r\nI can kill anyone except the ringbearer, I don't know who my allies are :mad: \r\n\r\nI don't kill in the first day just for making confusion :P\r\n\r\nNice kill, pianoforte :wink:\r\n\r\n[quote=\"pianoforte\"]eh, i actually targeted you that night (2nd night), though i didn't even notice until i went through my sent pms\n\nanyways, after hunters death i would be able to kill anyone at night, if no other effects. and there would be no way to kill me.[/quote]\r\n\r\nDouble Strike :roll:", "Solution_23": "pie probably shoulda just let hunter die that day", "Solution_24": "Actually hunter won't have died, since Pie used the ring and hunter was immortal for the day. It was purely a tactical mistake to reveal themselves.", "Solution_25": "Oh I see.", "Solution_26": "[quote=\"BoesFX\"]Nice kill, pianoforte :wink:[/quote]\r\n\r\nUh thanks?", "Solution_27": "[quote=\"pianoforte's signature\"]I'm Gandalf guys :( [/quote]You can take that off now.\r\n\r\nPeter, I actually would like to see all the roles.", "Solution_28": "yes, that was the mistake i was speaking of....hunter was already immune...and after that i couldnt use the ring any more so it was silly. I DID suspect that someone would go for me that night actually, which was i used the ring :) I think i may have left some hints to some rules that may have provoked some people..and ars also said that i was innocentish so yeah..", "Solution_29": "last post" } { "Tag": [], "Problem": "[youtube]cfgEtmxtz_k[/youtube]", "Solution_1": "It's cute, but in my opinion, not as funny as some of their other stuff.", "Solution_2": "McCain doesn't look realistic; Palin does. :P", "Solution_3": "[youtube]HvqyLC4xMzs[/youtube]", "Solution_4": "Pretty funny. The guy they got to do McCain sounded pretty good, but the voice was a bit high. Palin was too fat.", "Solution_5": "I agree, and the first part is good, but the second part kinda... isn't." } { "Tag": [ "USAMTS", "AMC", "AIME", "AMC 12", "USA(J)MO", "USAMO" ], "Problem": "Hey guys, a really really general question about USAMTS. You know the fourth question on the current round. Well, it says \"describe the strategy and prove it works.\" How exactly do you prove a strategy? Is it sufficient just to explain how you arrived at the strategy? Are there sources where I can read about game theory and stuff on the internet?\r\n And btw, does anyone know how high you have to score on USAMTS to get into AIME? It's my first year doing it see...", "Solution_1": "i think you just have to show that no matter what, the opponent cannot overcome the strategy. its hard to explain it without giving the answer away...\r\n\r\na perfect on USAMTS garners only a 100 on the AMC12, so i wouldnt suggest doing USAMTS only.", "Solution_2": "You should know some terminology for game theory. Suppose we are dealing with a symmetric game, which means that the two players can each make the same moves. (For example, Tic Tac Toe is not a symmetric game, since one marks Xs and one marks Os, but Dots and Boxes is a symmetric game.)\r\n\r\nA position is said to be [b]winning[/b] if it is possible to get from that position to a losing position.\r\n\r\nA position is said to be [b]losing[/b] if it is not possible to get from that position to a losing position.\r\n\r\nYou can usually figure out whether some given position is either winning or losing. For example, often in games such as Nim, the person who takes the last stick loses. (The other variant is the person who takes the last stick loses, but we won't discuss that now.) So suppose we have a simplified version of Nim, where there is one pile of 2003 sticks. Each person may take 1, 2, or 3 sticks. The person who takes the last stick(s) wins. So who wins? Here is a strategy for the first person to win: on the first turn, the first player should take 3 sticks. Then 2000 sticks are left. On every turn from then on, the second player may take 1, 2, or 3 sticks. If the second player takes one stick, the first player should take three. If the second player takes two, the first player should also take two. If the second player takes three, then the first player should take one. Then after the first person has moved, there will always be a multiple of four sticks left. There is no way the second person can change that. Thus the first person wins.", "Solution_3": "I like Simon's answer to your first few questions.\r\n\r\nAnyway, the cutoff to qualify for the AIME has been 90% after 3 rounds for the past couple years. So what is that... 68/75? Yeah. That percentage is then taken to be the number of points that go toward your USAMO index, so the maximum the USAMTS can contribute is 100 points. (So take the AMC even if you've crushed the USAMTS.)" } { "Tag": [], "Problem": "Find the maximum or minimum value of \r\n$ x^3 \\plus{} y^3 \\minus{} 2(x^2 \\plus{} y^2)$ for all $ x,y \\in R.$\r\n\r\nSorry, i make some mistake. i make the correction now.", "Solution_1": "[quote=\"4865550150\"]Find the maximum value of \n$ x^3 \\plus{} y^3 \\minus{} 2(x^2 \\plus{} y^2)$ for all $ x,y \\in R.$[/quote]\r\n\r\n\r\n\r\nYou must have made some mistakes. When $ x \\to \\infty$ ..." } { "Tag": [ "geometry" ], "Problem": "Sanki ikinci soruyu daha \u00f6nce bir erden hat\u0131rl\u0131yorum", "Solution_1": "hat\u0131rlaman\u0131z do\u011fal.. sorular kitapta yazd\u0131\u011f\u0131na g\u00f6re eski 2. a\u015fama sorular\u0131ndanm\u0131\u015f. bu arada 1999-2000 y\u0131llar\u0131 2. a\u015famalar\u0131 elinde olan var m\u0131? varsa erencankizildag@gmail.com adresine yollarsa sevinirim." } { "Tag": [ "algorithm" ], "Problem": "Have someone write down a 13-digit number and then reverse the digits. Subtract the smaller number from the other( if the original number has less than 13 digits then the reverse number is less than 13-digits) then have them draw a circle around anyone of the digits. Asks them to say what the uncircled digits are, after hearing the numbers you will be able to tell them the circled digit. How does the trick work? Why does it work?", "Solution_1": "I'm not sure I understand what you're asking -- what numbers can I choose from to circle? What digits do I read off?", "Solution_2": "I have edited the problem so that you will understand it better. You asks the person to circle anyone of the digits from the\r\nsubtracted sum and then read all the digits that are uncircled so that with this information you should be able to find the circled digit.", "Solution_3": "Oh, okay, thanks!", "Solution_4": "N = a1...a13\r\nN'= a13...a1\r\n\r\nThen |N-N'|=b1..b13 is divisible by 9 and 11 (just look mod 9 and mod 11).\r\n\r\nSo here is a short algorithm which, I think, works :\r\n\r\nLet c1 .. c12 the given (=uncircled) digits. \r\nLet a = sum ci\r\n\r\n. if (a mod 9) != 0 then there is a unique digit d which makes a+d divisible by 9.\r\n. if (a mod 9) = 0 then there is ambiguity mod 9 between 0 and 9 but there is a unique digit d (in {0,9}) which makes sum(-1)^i*ci +/- d divisible by 11.\r\n\r\nIn each case d is the wanted digit." } { "Tag": [ "SFFT" ], "Problem": "The harmonic mean of two positive numbers is the reciprocal of the arithmetic mean of their reciprocals. For how many ordered pairs of positive integers $(x,y)$ with $xb$. Then take the combinations of $2^m3^n$ and add $6^{20}$ and it's done. I'm not too sure how to find whether or not, for example, $3^5 \\cdot 2^{40} ? 3^{35}$ is less than/greater than.[/hide]", "Solution_2": "[quote=\"joml88\"]The harmonic mean of two positive numbers is the reciprocal of the arithmetic mean of their reciprocals. For how many ordered pairs of positive integers $(x,y)$ with $x0 such that abc <=8.\nprove that 1/(a^2-a+1)+1/(b^2-b+1)+1/(c^2-c+1)>=1[/size]", "Solution_1": "I think the result is too short, because you can use AM-GM and Schwartz.\r\nbut we can prove it to>=9/168.", "Solution_2": "[quote=\"Ly Tieu Long\"]but we can prove it to>=9/168.[/quote]\r\nsorry , i dont get you", "Solution_3": "See here:\r\nhttp://www.artofproblemsolving.com/Forum/viewtopic.php?t=35975" } { "Tag": [ "probability", "function", "blogs" ], "Problem": "Four couples are at a party. Four people of the eight are\nrandomly selected to win a prize. No person can win more\nthan one prize. What is the probability that both members\nof at least one couple win a prize? Express your answer as a\ncommon fraction.", "Solution_1": "The probability that both members of at least one couple will win a prize is the same as one minus the probability of each couple winning one prize. So...\r\n\r\n$ 1\\minus{}1*\\frac67*\\frac46*\\frac25\\equal{}\\frac{27}{35}$", "Solution_2": "How'd you get those fractions though?\r\n\r\nOr mayb I'm just being really dumb\r\n>.<", "Solution_3": "I'm sorry. The probability of each couple winning one prize is:\r\n\r\nProbability of first prize going to a couple without a prize: 1\r\nProbability of second prize going to a couple without a prize (3 couples out of 7 total people--since person who won first prize is ineligible): $ \\frac67$\r\nProbability of third prize going to a couple without a prize (2 couples out of 6 total people--since people who won first two prizes are ineligible): $ \\frac46$\r\nProbability of fourth prize going to a couple without a prize (1 couple out of 5 total people--since people who won first three prizes are ineligible): $ \\frac25$\r\n\r\nSo probability of each couple winning one prize is: $ 1*\\frac67*\\frac46*\\frac25 \\equal{} \\frac8{35}$\r\n\r\n$ 1 \\minus{} \\frac8{35} \\equal{} \\frac {27}{35}$\r\n\r\nHope this is more helpful.", "Solution_4": "The last poster's reasoning is probably the best. The important idea to remember is sometimes it's easier to compute the probability that something WON'T happen, and then you know the probability it WILL happen is 1 minus your computation.\r\n\r\nIf the fractions of the last problem are intimidating, you can learn how to do it with the \"n choose k\" function, and the idea that \"probability=(# of chances of success)/(total # of events)\".***\r\n\r\nSo, let's count successes: I need to choose one person from each couple. There are 2 ways to do that for the first couple, 2 ways for the second, 2 ways for the third and 2 ways for the fourth. So in total, 2x2x2x2=16 ways to succeed. How many total events are there? That's where the \"n choose k\" function comes in. It'll tell you how many ways there are total of picking four people out of eight. Grab any basic probability reference, it's in there. In our case: 8 choose 4=8x7x6x5/4x3x2x1. This is quickly reduced to 70 by cancelling. So, using *** we have 16/70=8/35....Then 1-8/35 is 27/35 as above.\r\nBoth methods are probably about the same speed, once you're used to them. So, just pick the one that you remember the fastest :) Definitely learn how to use the \"n choose k\" function though!", "Solution_5": "A note:\r\n\r\n$ n$ choose $ k$ can be written as $ nCk$ or $ \\binom{n}{k}$. These are called combinations. If you do not understand combinations, there's an entry on them in my blog, link's in my signature.\r\n\r\nWhat iin did is called complementary counting. This means we count what we don't want, and subtract this from 1. More on this is my blog, too." } { "Tag": [ "ARML", "geometry", "trigonometry", "trapezoid", "analytic geometry", "ratio", "trig identities" ], "Problem": "Regular polygon $ ABCDEFGHIJKLMNOPQR$ has side length $ 1$. The area of polygon $ ARML$ can be expressed as $ \\frac {3(\\sqrt {3} \\plus{} \\sqrt {2 \\plus{} 2x})}{4}$. What is the value of $ 16x^3 \\minus{} 12x \\plus{} 11$?", "Solution_1": "can someone count the letters for me?", "Solution_2": "18. \r\n\r\nIf calculators are allowed, we could simply bash this with the $ \\frac {s^2 n}{4 \\tan{\\pi / n}}$ formula, find the actual area, solve for $ x$, and then evaluate. If not, then I'm not sure...", "Solution_3": "Ummm, doesn't that formula only apply to regular n-gons so how would it help for finding the area of $ ARML$?\r\n\r\n[hide=\"Another possibility\"]\nUse complex roots of unity. Let $ \\theta\\equal{}\\frac{2\\pi}{18}$ Let the points A through R be the 18th roots of unity $ e^{ik\\theta}, k\\equal{}0$ to $ 17$ and fix B at $ (1,0)$ with the alphabet continuing in counterclockwise order. \n\n$ ARML$ is thus an isosceles trapezoid where the vertices are $ A\\equal{}e^{i17\\theta}$, $ R\\equal{}e^{i16\\theta}$, $ M\\equal{}e^{i11\\theta}$, and $ L\\equal{}e^{i10\\theta}$. The bases of this trapezoid are then $ AL\\equal{}2cos(17\\theta)$ and $ RM\\equal{}2cos(16\\theta)$. The height of the trapezoid is $ sin(16\\theta)\\minus{}sin(17\\theta)$. The area comes straight from the average of the bases times the height. \n\nAll that this would change is the side length of the 18-gon, but that only multiplies all the areas by a scalar anyway. This method might not give the area in the form that we would require to solve for $ x$ though. Maybe a sort of backwards triple angle formula or something would make it in the right form, but my guess is that some fact about the cubic will make this alot easier.\n[/hide]", "Solution_4": "Ooh, trigtastic!\r\n\r\n[hide=\"This is as short as I could make it while still showing a fair amount of work.\"]\nThe figure is a trapezoid whose diagonals are both $ 120^{\\circ}$ cords, so their lengths are $ r\\sqrt {3}$. One can use the LoS to show $ r \\equal{} 1/2\\sin 10^{\\circ}$. The angle between the chords is $ 20^{\\circ}$, so the area of the trapezoid is $ d_1d_2\\sin\\theta/2 \\equal{} \\frac {3}{4\\tan 10^{\\circ}} \\equal{} \\frac {3\\tan 80^{\\circ}}{4}$.\n\nSo it looks like we have $ \\tan 80^{\\circ} \\minus{} \\tan 60^{\\circ} \\equal{} \\sqrt {2 \\plus{} 2x}$. The difference-of-tangents formula can be re-derived by combining fractions:\n\\[ \\tan A \\minus{} \\tan B \\equal{} \\frac {\\sin (A \\minus{} B)}{\\cos A\\cos B},\n\\]\nwhich in our case gives $ \\tan 80^{\\circ} \\minus{} \\tan 60^{\\circ} \\equal{} \\frac {\\sin 20^{\\circ}}{\\sin 10^{\\circ}\\cos 60^{\\circ}} \\equal{} 4\\cos 10^{\\circ}$, so now we have the relation $ x \\equal{} 8\\cos^2 10^{\\circ} \\minus{} 1 \\equal{} 4\\cos 20^{\\circ} \\plus{} 3$. Are you sick of trig yet? Well too bad! We press on...\n\nTriple angle for cosine looks like $ \\cos 3A \\equal{} 4\\cos^3 A \\minus{} 3\\cos A$, which means the final answer is $ 4\\cos 3A \\plus{} 11$, where $ x \\equal{} \\cos A$. Here I get stuck, since our value for $ x$ is clearly larger than $ 5$. My gut instinct is that the problem-writer multiplied by powers of $ 2$ rather than divided whilst contriving this kooky problem. I may be wrong, of course.\n\nFor example, if $ \\sqrt{2\\plus{}2x}$ had been replaced by $ \\sqrt{8\\plus{}8x}$ in the original statement, we would have\n\\[ 4\\cos 10^{\\circ} \\equal{} \\sqrt{8\\plus{}8x} \\quad \\Rightarrow \\quad \\cos 10^{\\circ}\\equal{}\\sqrt{\\frac{1\\plus{}x}{2}} \\quad \\Rightarrow \\quad x \\equal{} \\cos 20^{\\circ},\n\\]\nfrom which $ 16x^3\\minus{}12x\\plus{}11 \\equal{} 4(\\cos 60^{\\circ})\\plus{}11 \\equal{} \\boxed{13}$, which should have been the guess (based on the poser's subject description).\n\nMy initial reaction while working through this problem was I would never wish it on any ARML participant. I've had problems half this hard shot down as \"way too complicated / contrived / obscure / etc.\", and I would argue that in a short time-limit environment there are fairer ways of testing everyone -- not just the smarty-pants AoPS types -- than a problem that relies in part on triple-angle formulas. I would totally use this sorta problem on a team selection test, though :wink: \n[/hide]", "Solution_5": "I dont think its a trapeziod...im probably being retarded though...\r\n\r\nIf you really wanted to bash....\r\n[hide]\nConsider the 18gon formed by the 18 roots of unity on the complex plane(let the verticies be A', B'...). Find the coordinates of A'R'M'L' and use shoelace. However, this is bigger than the original 18gon. to find the ratio between the areas, let the center of the original be Z.(new one is Z') using the similarity between ABZ and ABZ' and using law of sines, we find that(i skipped a TON of algebra, and i hope its right) the new quadrilaterals is $ 2\\sin\\frac{\\pi} {18}$ times bigger than the old one. hence, the area is $ 4\\sin^2\\frac{\\pi} {18}$ times more. So multiply $ BASH*4\\sin^2\\frac{\\pi} {18}$ and youre done.\n[/hide]\r\n\r\nI dare someone to do that." } { "Tag": [ "algebra", "polynomial", "search", "number theory solved", "number theory" ], "Problem": "Prove that $p(x)=(x^2+x)^{2^n}+1$ is irreducible in $\\mathbb Z[x]$", "Solution_1": "This was posted at least five times before... (I recommend search button)\r\nhttp://www.mathlinks.ro/Forum/viewtopic.php?t=76611\r\nhttp://www.mathlinks.ro/Forum/viewtopic.php?t=70003\r\nhttp://www.mathlinks.ro/Forum/viewtopic.php?t=49799\r\nhttp://www.mathlinks.ro/Forum/viewtopic.php?t=14780\r\nhttp://www.mathlinks.ro/Forum/viewtopic.php?t=311", "Solution_2": "in all the links given i don't see a simple solution, what about this one : take X=x+1 => P(x)=(X\u00b2-X+1^)^(2^n)+1\r\nand the result immediately follows from eisenstein criterion for p=2, any mistake?", "Solution_3": "[quote=\"maskman\"]in all the links given i don't see a simple solution, what about this one : take $X=x+1 \\, \\Rightarrow \\. P(x) = (X^2-X+1)^{2^n}+1$\nand the result immediately follows from eisenstein criterion for p=2, any mistake?[/quote]\r\n\r\nYou have $(X^2-X)^{2n}+1$... which brings you nowhere, I think." } { "Tag": [ "calculus", "derivative", "function", "inequalities", "Putnam", "real analysis", "real analysis solved" ], "Problem": "Let $ f: R-->R $ be a function has $3_{th} $ derivative . Prove that : There exists $ a $ such that : \r\n $ f(a)f'(a)f''(a)f'''(a) \\geq 0 $", "Solution_1": "Assume that each $f(x),f'(x),f''(x),f'''(x)$ is positive or negative for all $x$. (If one of them is zero, the ineq holds)\r\nWLOG, assume that $f''(x)>0$ and $f'''(x)>0$. (replace $-f$ by $f$ and replace $f(x)$ by $f(-x)$ if necessary)\r\n$f''(x)>0$ implies that $f'$ is strictly increasing\r\n$f'''(x)>0$ implies that $f'$ is strictly convex : $f'(y) > f'(x) + f''(x)(y-x)$. Fix $x$ and let $y$ tend to $+\\infty$ we know that $f'(y) \\to +\\infty$ hence $f'(x)>0$ for all $x$.\r\n\r\nNow $f'(x)>0$ implies that $f$ is strictly increasing\r\nand $f''(x)>0$ implies that $f$ is strictly convex\r\nA similar argument gives $f(x)>0$ for all $x$\r\n\r\nHence $f(x)f'(x)f\\\"(x)f'''(x)>0$, the inequality holds.\r\n\r\nIt is a Putnam problem but I can't remember the year.", "Solution_2": "[quote=\"liyi\"]It is a Putnam problem but I can't remember the year.[/quote]\r\nSource: 59th Putnam 1998 Problem A3" } { "Tag": [ "combinatorics unsolved", "combinatorics" ], "Problem": "We are given equilateral triangle with side $n$ which is divided into $n^{2}$ congruent equilateral triangles with side $1$. Let's call these small triangles plates. Each plate is either black or white. We perform the following operation:\r\nwe choose a plate which has common sides with at least two plates of diiferent colour from chosen plate and then we change the colour of our plate. For every $n\\geq 2$ decide whether exists a configuration of colours that enables to perform infinite sequence of operations", "Solution_1": "Here is a solution in Polish. Can someone translate it into English?\r\n\r\nNazwijmy odcinkiem granicznym wsp\u00f3lny bok dw\u00f3ch p\u0142ytek, kt\u00f3rych widoczne\r\nstrony maja r\u00f3zne kolory. Oczywiscie liczba odcink\u00f3w granicznych jest\r\nnieujemna i nie przekracza liczby m wszystkich odcink\u00f3w bedacych wsp\u00f3lnym\r\nbokiem dw\u00f3ch p\u0142ytek. Zbadamy, jak zmienia sie liczba odcink\u00f3w granicznych\r\nw wyniku wykonania dozwolonego ruchu.\r\nKazda z n2 p\u0142ytek ma wsp\u00f3lne boki z co najwyzej trzema innymi p\u0142ytkami.\r\nOdwr\u00f3cenie p\u0142ytki jest dopuszczalne, jezeli co najmniej dwa z takich\r\nbok\u00f3w sa odcinkami granicznymi. Przypuscmy, ze odwracamy p\u0142ytke P.\r\nW\u00f3wczas odcinki graniczne moga pojawic sie albo zniknac jedynie na bokach\r\np\u0142ytki P. Ponadto bok p\u0142ytki P jest po odwr\u00f3ceniu odcinkiem granicznym\r\nwtedy i tylko wtedy, gdy przed odwr\u00f3ceniem nie by\u0142 on odcinkiem granicznym.\r\nWynika stad, ze w wyniku wykonania dozwolonego ruchu liczba odcink\u00f3w\r\ngranicznych zmniejsza sie.\r\nUdowodnilismy w ten spos\u00f3b, ze z dowolnego poczatkowego u\u0142ozenia\r\np\u0142ytek mozna wykonac nie wiecej niz m ruch\u00f3w.\r\nOdpowiedz: Dla zadnego n\u00ad2 nie istnieje u\u0142ozenie pozwalajace wykonac\r\nnieskonczony ciag ruch\u00f3w.", "Solution_2": "Here is my solution:\r\n\r\nLet G be a graph with $n^{2}$ vertices. (each vertice assigned to one of the plates in the triangle)\r\ntwo vertices are 'linked' by one edge if they have one side in common and your color is the same.\r\n\r\nnow, see that, after one operation, our number of edges will diminute, because if from one vertice out 2 edges, will out 1, and if out 3, will out zero. (we can do one operation if, from one vertice, 2 edges are outing)\r\n\r\nso, our problem is solved, because the number of edges always will decrease! (and the operations are limited!)" } { "Tag": [ "trigonometry" ], "Problem": "given atriangle ABC , a=4,b=5 , c = 6\r\nshow that C=2A", "Solution_1": "What do you mean by C=2A?\r\n\r\n0 are constants and N a positive integer, what is the value of x that gives the maximum value for f(x)? Note that binom(n,r) is a choose function giving the binomial coefficients.\r\n\r\nThanks.", "Solution_1": "[quote=\"pewthian\"]Hi, I am new to this forum. I hope someone can help me with this problem which I have tried to solve for a long time and still can't obtain the solution. Given a discrete function:\n\nf(x) = binom(a + x, x) * binom(b + N-x, N-x) where x = 0,1,...N\n\nwhere a,b>0 are constants and N a positive integer, what is the value of x that gives the maximum value for f(x)? Note that binom(n,r) is a choose function giving the binomial coefficients.\n\nThanks.[/quote]\r\n\r\n$f(x)=\\binom{a+x}{x}\\cdot \\binom{b+n-x}{n-x}$", "Solution_2": "Binomials have factorials, and factorials are messy, so if possible we'd like to get rid of them. To find potential maxima, we only need to compare adjacent terms; because of the factorials, comparing differences is messy. But comparing ratios of consecutive terms is quite straight-forward -- you just need to know the value of $x$ at which the ratio switches from being $\\geq 1$ (your sequence is increasing) to being $\\leq1$ (your sequence is decreasing), and you end up only having to solve a linear equation for $x$." } { "Tag": [ "function", "floor function", "LaTeX", "ceiling function" ], "Problem": "for which numbers $k\\lfloor x \\rfloor=\\lfloor kx \\rfloor$?", "Solution_1": "if you're talking about an arbitrary $k$ you're only guaranteed that its right if $x$ is an integer. It will work for different values of $x$ based on what $k$ is.", "Solution_2": "It works whenever $x - \\lfloor x \\rfloor < \\frac{1}{k}$.", "Solution_3": "Yeah that seems right.", "Solution_4": "Wait, how do you write the floor and ceiling functions in LaTeX?", "Solution_5": "\\lfloor x\\rfloor\r\n\r\n\\lceil x\\rceil\r\n\r\nYou'll want to use \\left\\lfloor and \\right\\rfloor if the argument is a fraction or something." } { "Tag": [], "Problem": "At a particular school, all 43 students take chemistry, biology, or both. The chemistry class is three times larger than biology class, and 5 students are taking both classes. How many people are in the chemistry class?", "Solution_1": "Well as you see I will show another solution, it has the same basic idea but with some differences..\r\nThe solution ::\r\nSay that the number of students taking Chemistry is [i]x[/i] and the number of students taking biology is [i]y[/i] ,, then we know that the number of chemistry students = 3 times the number of biology students,, so we get that [i]y[/i] = [i]x[/i]/3 . Now we make our equation 43 = x + x/3 -5 (43 is the number of all the students and we subtract 5 since we counted the students who studied both subjects twice )\r\nWe solve the equation for [i]x[/i] and we get that the number of chemistry students =[i]x[/i]=36\r\nThis is how i solved it .. I hope it helps :) .." } { "Tag": [ "algebra", "system of equations", "advanced fields", "advanced fields unsolved" ], "Problem": "How to prove that the system of equations\r\n$\\begin{cases}x \\text{ cos }y=x^{2}+y^{2}-1\\\\ y\\text{ cos }x=\\text{tan }2\\pi(x^{3}+y^{3}) \\end{cases}$\r\nhas a solution $(x_{0},y_{0})$ s.t. $x_{0}^{2}+y_{0}^{2}\\leq 1$. According to the book, one should use the fact that a continuos odd map $f: \\mathbb{D}^{2}\\to \\mathbb{R}^{2}$ on the boundary, (i.e. $x\\in \\mathbb{S}^{1}\\implies f(-x)=-f(x))$ has a point $x_{1}\\in\\mathbb{D}^{2}$ s.t. $f(x_{0})=0$.", "Solution_1": "I made a stupid calculation mistake. If I define $f(x,y)=\\left (x\\text{ cos }y-x^{2}-y^{2}+1,y\\text{ cos }x-\\text{ tan }2\\pi (x^{3}+y^{3})\\right )$ then $f(x,y)=f(-x,-y)$ for all $(x,y)$ inside the unit circle." } { "Tag": [ "real analysis", "real analysis unsolved" ], "Problem": "A set $A \\subset [0,1]$ is called [i]negligible[/i] if it is Lebesgue measurable with Lebesgue measure zero. Let $\\mathcal{F}$ is a totally ordered family of negliglible subsets of $[0,1]$ (totally ordered means that, for arbitrary $A,B$ in the family, we either have $A \\subseteq B$ or $B \\subseteq A$). Must the union $\\bigcup_{A \\in \\mathcal{F}} A$ be negliglble?\r\n\r\n :roll:", "Solution_1": "Start with a well-ordering $x_{\\alpha}$ of $[0,1]$, indexed by an initial segment of the ordinals. For each ordinal $t$, let $S_t=\\cup_{\\alphab>c>d => a+b>b+c>c+d>d+a \r\nby using Chebychev innequality we have : \r\nS>= 1/4(a+b+c+d)((a+b)^3 + (b+c)^3 + (c+d)^3 +(a+d)^3) \r\n>= 1/4(a+b+c+d)(1/16*(2*(a+b+c+d))^3)\r\n= 1/8(a+b+c+d)^4", "Solution_2": "[quote=\"iverson_h3\"]a>b>c>d => a+b>b+c>c+d>d+a \n[/quote]\r\nwrong,take $ a\\equal{}100,b\\equal{}3,c\\equal{}2,d\\equal{}1$ :wink:", "Solution_3": "[quote=\"iverson_h3\"]put a>b>c>d => a+b>b+c>c+d>d+a[/quote]\r\nwhy c+d>d+a :?:", "Solution_4": "sorry :blush: it's false :oops:", "Solution_5": "[quote=\"pvthuan\"]1) Let $ a,b,c,d$ be real numbers. Prove that\n\\[ a(a \\plus{} b)^3 \\plus{} b(b \\plus{} d)^3 \\plus{} c(c \\plus{} d)^3 \\plus{} d(d \\plus{} a)^3\\geq0.\n\\]\n[/quote]\r\n\r\nDid you mean $ a(a \\plus{} b)^3 \\plus{} b(b \\plus{} c)^3 \\plus{} c(c \\plus{} d)^3 \\plus{} d(d \\plus{} a)^3\\geq0.$ ?", "Solution_6": "[quote=\"ringos\"]\nDid you mean $ a(a \\plus{} b)^3 \\plus{} b(b \\plus{} c)^3 \\plus{} c(c \\plus{} d)^3 \\plus{} d(d \\plus{} a)^3\\geq0.$ ?[/quote]\r\n\r\nYes, ringos. I have edited it.", "Solution_7": "If you think they are too easy, the following is hard enough.\r\n\r\nProve that for all real numbers $ a,b,c,d$, we have the inequality\r\n\\[ a(a\\plus{}b)^5\\plus{}b(b\\plus{}c)^5\\plus{}c(c\\plus{}d)^5\\plus{}d(d\\plus{}a)^5\\geq\\frac1{32}(a\\plus{}b\\plus{}c\\plus{}d)^6.\\]", "Solution_8": "Unfortunately, take $ a \\equal{} \\minus{} 1,b \\equal{} 3,c \\equal{} \\minus{} 2,d \\equal{} 1$, we have\r\n\r\n$ a(a \\plus{} b)^3 \\plus{} b(b \\plus{} c)^3 \\plus{} c(c \\plus{} d)^3 \\plus{} d(d \\plus{} a)^3<0$\r\n$ a(a \\plus{} b)^5 \\plus{} b(b \\plus{} c)^5 \\plus{} c(c \\plus{} d)^5 \\plus{} d(d \\plus{} a)^5<0$\r\n\r\n :wink:", "Solution_9": "Thank you, shalex !\r\nRegret that two above nice inequality are wrong.I have mistaken in calculating :oops: \r\nBut we have:\r\nLet $ a$, $ b$, $ c$,$ d$ be positive real numbers.Prove that\r\n\r\n1) $ a(a \\plus{} b)^3 \\plus{} b(b \\plus{} c)^3 \\plus{} c(c \\plus{} d)^3 \\plus{} d(d \\plus{} a)^3 \\geq \\frac {1}{8}(a \\plus{} b \\plus{} c \\plus{} d)^4$;\r\n2) $ a(a \\plus{} b)^5 \\plus{} b(b \\plus{} c)^5 \\plus{} c(c \\plus{} d)^5 \\plus{} d(d \\plus{} a)^5 \\geq \\frac {1}{32}(a \\plus{} b \\plus{} c \\plus{} d)^6$", "Solution_10": "Generally,we also have:\r\nLet $ a$, $ b$, $ c$,$ d$ be positive real numbers.Then for all odd natural number k the inequality\r\n$ a(a\\plus{}b)^k\\plus{}b(b\\plus{}c)^k\\plus{}c(c\\plus{}d)^k\\plus{}d(d\\plus{}a)^k \\geq \\frac{1}{2^k}(a\\plus{}b\\plus{}c\\plus{}d)^{k\\plus{}1}$\r\nis true.", "Solution_11": "That 's a great pro! I'l think of it in some day", "Solution_12": "Assume that $ a\\plus{}b\\plus{}c\\plus{}d\\equal{}1$. Use Jensen Inequality we have\r\n\r\n$ a(a\\plus{}b)^k\\plus{}b(b\\plus{}c)^k\\plus{}c(c\\plus{}d)^k\\plus{}d(d\\plus{}a)^k \\ge [a(a\\plus{}b)\\plus{}b(b\\plus{}c)\\plus{}c(c\\plus{}d)\\plus{}d(d\\plus{}a)]^k$\r\n\r\nIt suffices to show\r\n\r\n$ a(a\\plus{}b)\\plus{}b(b\\plus{}c)\\plus{}c(c\\plus{}d)\\plus{}d(d\\plus{}a) \\ge \\frac{1}{2}$\r\n\r\nWhich is equivalent to\r\n\r\n$ (a\\minus{}c)^2\\plus{}(b\\minus{}d)^2 \\ge 0$\r\n\r\nIt's done, and the equality holds for $ a\\equal{}c, b\\equal{}d$ :wink:", "Solution_13": "that is Ok :)", "Solution_14": "A solution is nice and easy. It is amazing! Congratulation, shalex! :D" } { "Tag": [], "Problem": "When the expression $ 8^{10} \\times 5^{22}$ is multiplied out (written in decimal notation), how many\ndigits does the number have?", "Solution_1": "$ 8^{10} \\times 5^{22} \\implies 2^{30} \\times 5^{22} \\implies 2^8 \\times 10^{22} \\implies 256 \\times 10^{22}$\r\n\r\n256 has 3 digits, so $ 3 \\plus{} 22 \\equal{} \\boxed{25}$.", "Solution_2": "How did you get 2^30 to 2^8?", "Solution_3": "they made the $2^{30}\\times 5^{22}$ turn into $2^8\\times 10^{22}$ " } { "Tag": [ "LaTeX", "number theory unsolved", "number theory" ], "Problem": "Does there exist $ 2002$ distinct positive integers $ k_1, k_2, \\cdots k_{2002}$ such that for any positive integer $ n \\geq 2001$, one of $ k_12^n \\plus{} 1, k_22^n \\plus{} 1, \\cdots, k_{2002}2^n \\plus{} 1$ is prime?", "Solution_1": "No.\nI'll prove inductively the following statement:\nFor $n\\in \\mathbb{N}$ and $k_1,k_2,...,k_n, a\\in \\mathbb{N}$ there exist arbitrarily large $m\\in \\mathbb{N}$ such that all numbers $k_ia^m+1$, $i=1,...,n$ are composite.\n\nFor $n=1$ let $p$ be a prime divisor of $k_1a^t+1$ for some big positive integer $t$. Then $k_1a^{pt}+1\\equiv k_1a^t+1\\equiv 0 (p)$ and $k_1a^{pt}+1>p$, so it is composite.\nNow assume, that this is true for $n2002$\nChoose $X = {F^F \\prod_{i=1}^{F^F} ({p_i-1})}$ to be extremely large where $p_i|k_x+1$ \nBy Fermat's Little Theorem, $2^X k_i +1 \\equiv k_x+1 \\equiv 0 \\pmod {p_i}$,and by the size of $2^x k_i+1>p_i$,so by conclusion no such integers exist.\n[/hide] ", "Solution_4": "The answer is no. The problem is very easy once you realize $2002$ doesn't matter and you can deal with each $k_i$ separately. Simply pick some prime divisor $p_i \\mid 2k_i+1$ for each $k_i$, and set $$n=N(p_1-1)(p_2-1)\\cdots(p_{2002} - 1) + 1.$$ By Fermat's little theorem this implies that $$k_i \\cdot 2^n + 1 \\equiv 2k_i + 1 \\equiv 0 \\pmod p,$$ so all of the terms can be composite." } { "Tag": [ "algebra unsolved", "algebra" ], "Problem": "7/x+3 + 3/6x-2= 4/x+3x-2 solve for x...a problem i have been trying and havent gotten...please show work", "Solution_1": "${\\frac{7}{x}+3+\\frac{3}{6 x}-2 == \\frac{4}{x}+\\text{ }3 x-2}$\r\n\r\nMultiplying both sides by $6 .x$\r\n\r\n${6x\\left(\\frac{7}{x}+3+\\frac{3}{6 x}-2 \\right)= 6 x\\left( \\frac{4}{x}+\\text{ }3 x-2\\right)}$\r\n${42 \\frac{x}{x}+18 x+\\frac{18 x}{6\\text{ }x}-12 x = 24 \\frac{x}{x}+18 x^{2}-12 x}$\r\n${45+6 x = 24-12 x+18 x^{2}}$\r\n${\\left( 24-12 x+18 x^{2}\\right)-(45+6 x ) = 0}$\r\n${-21-18 x+18 x^{2}=0}$\r\n${3 \\left(-7-6 x+6 x^{2}\\right)=0}$\r\n${\\left(-7-6 x+6 x^{2}\\right)=0}$\r\n\r\n${x = \\frac{6 \\pm \\sqrt{(-6)^{2}-4 (6)(-7) }}{2 (6)}}$\r\n${x = \\frac{1}{6}\\left(3\\pm \\sqrt{51}\\right)}$\r\n\r\n\r\n\r\njust aa hint\r\n\r\n\r\n1/xa is not the seme as 1/ (x a)\r\n\r\n1/xa = (1/x) * a\r\n\r\n\r\nso\r\n\r\nuse ((((( and )))))\r\n\r\nfor yours expressions\r\n\r\nbecause i did not understand wat you tipe" } { "Tag": [], "Problem": "Please also post anything else interesting (where you are, how you think you'll do, any pre-comp preparations or foods to eat, etc...)", "Solution_1": "Dream - dadadadlalalalala - in dreamland far, far away. Not really nervous except for Shaunak killing me in chapters lalalalalalala. Other than that nothing ... lalalalalalala.", "Solution_2": "yes...different states take chapters on different days. Chapter competitions are usually held in February where as State competitions are held in March. This year my chapter competition is on V-Day. And state is on March 6.\r\n\r\nI am in Indiana...most of you know that...my team is most likely going to place first in our chapter...we have the past several years. None of the schools in our conference or whatever you call it are a very big threat.", "Solution_3": "why is held on different days? (doesn't that make it possible for cheating ) like you have a cousin in another a state who has taken it and tells yhou the solutions. (i remember that in utah last year several questions were tkaen out because they wer mentioned in teh newspaper in earlier compeitions of other states)", "Solution_4": "I'm in Ohio! On the 28th! This is my first and last year!", "Solution_5": "Chapter is Febuary 14\r\n\r\nState is March 6" } { "Tag": [], "Problem": "a,b and c are positive real numbers such that for each positive integer n there exists a triangle $T_{n}$ with sides $a^{n},b^{n},c^{n}$\r\n\r\nProve that all the $T_{n}$ are equilateral", "Solution_1": "[hide=\"Is it true?\"] Let $b = c = 1$ and let $0 < a < 1$, then we have a series of isosceles triangles with a smallest side approaching $0$. [/hide]" } { "Tag": [ "calculus", "geometry", "3D geometry", "function", "similar triangles", "calculus computations" ], "Problem": "Hi guys. I guess this problem is pretty easy to you. I solved it also. Just need a confirmation. It belongs to an assignment that will pretty much decide my grade.\r\n\r\nWater drains from a conical tank with height 12 ft and diameter 8ft into a cylindral tank that has base 400\u03c0 sqaure feet. The depth, h, of the water in the conical tank is changing at the rate of (h-12) feet per minute.\r\n\r\n(Volume of cone is V=1/3 * \u03c0 * r^2 * h)\r\n\r\na. Write an expression for the volume of water in the conical tank as a function of h\r\nb. At what rate is the volume of water in the conical tank changing at h=3\r\nc. At what rate is the depth of the water in the cylindral tank changing when h=3\r\n\r\nHere is my answer\r\na. [hide]16/3\u03c0h[/hide]\nb. [hide]-144\u03c0[/hide]\nc. [hide]9/25[/hide]\r\n\r\nThanks.", "Solution_1": "Anhnguyen, you have failed to take account of the fact that $r$ is not constant, but rather a function of $h$. Draw a diagram showing a cross-section of the cone and use similar triangles to find out how $r$ is related to $h$. Your expression for $V$, when multiplied out, should involve $h^3.$" } { "Tag": [], "Problem": "If we count by 3s starting with 1, the following sequence is obtained: 1, 4, 7, 10, \u2026 What is the 100th number in the sequence?", "Solution_1": "The $ n$th number is $ 3n\\minus{}2$, so the $ 100$th number is $ 3\\times100 \\minus{} 2\\equal{} 298$." } { "Tag": [ "ARML" ], "Problem": "Please poll your honest opinions.", "Solution_1": "what about ARML :o", "Solution_2": "how[b][u][i]will[/i][/u][/b] you feel?", "Solution_3": "?????\r\n\r\nLocked." } { "Tag": [], "Problem": "We had like 10 posts in a month, so we are sucking now", "Solution_1": "It might be because of:\r\n\r\nWinter break- christmas eve etc. \r\n\r\nVacations-\r\n\r\nMany forums are not posted in often, so it's not uncommon.", "Solution_2": "yeah I guess, I just went to some random ones and saw that there were almost no posts", "Solution_3": "Yup. It is dying. The topics that are only about 6 from the top are like a month old.", "Solution_4": "Many of the state forums are dying since there aren't that many people and there isn't much to talk about; at least from my observations.\r\n\r\n\r\nHUZZZZZZAHHH!! :P", "Solution_5": "There would be more to talk about if everyone posted in their own state forums, not the ones for the competitions.\r\n :P", "Solution_6": "I WILL POST HERE IF OTHER PEOPLE DO", "Solution_7": "Ok, then I'll post. :lol:" } { "Tag": [ "ratio", "calculus", "calculus computations" ], "Problem": "I have this question which I've been struck on for a long time:\r\n\r\nA spotlight on the ground shines on the outside wall of a parking garage 12m away. If a 2-m tall man walks toward the garage at a speed of 0.75m/s, how fast is the height of the man's shadow on the garage wall decreasing when he is 4m from the building?\r\n\r\nI have no clue how to solve it. :oops:\r\n\r\nAll I got was:\r\n\r\ndx/dt = 0.75m/s\r\nIt should be a triangle but y is constant (2)\r\n\r\nBut the triangle size always move because y = 2 always.\r\n\r\nHelp please?", "Solution_1": "There's more than one triangle in the picture- I don't believe you've said anything about the actual shadow yet.", "Solution_2": "Actually, I got it. 12/y = d/2 ratio b/w the two triangles. Thanks anyways" } { "Tag": [ "geometry", "geometry unsolved" ], "Problem": "[img]http://www.mathlinks.ro/Forum/viewtopic.php?mode=attach&id=14444[/img]\r\n\r\n :( \r\n\r\n\r\n[url]http://www.polarprof.org/geometriagon/wesercizio.asp?c=545437&n=867[/url]", "Solution_1": "could you give me an example of a circumscribed square i cant understand", "Solution_2": "It is a square whose sides are each tangent to one circle or the other\r\n\r\n :(" } { "Tag": [ "geometry", "trapezoid", "ratio", "geometry proposed" ], "Problem": "Let $ABC$ be a triangle and $D$ the foot of the altitude from $A$. Let $E$ and $F$ be on a line through $D$\r\nsuch that $AE$ is perpendicular to $BE$, $AF$ is perpendicular to $CF$, and $E$ and $F$ are different from\r\n$D$. Let $M$ and $N$ be the midpoints of the line segments $BC$ and $EF$, respectively. Prove that $AN$\r\nis perpendicular to $NM$.", "Solution_1": "Let $B_1,C_1,M_1$ be the projections of $B,C,M$ respectively on the line through $A$ parallel to $BC$. We have to prove that $N$ lies on the circle $(ADM)$, which coincides with the circle $(ADM_1)$.\r\n\r\nThe four points $A,D,B_1,E$ are concyclic, so the lines $AD,B_1E$ are antiparallel in the angle formed by the lines $B_1C_1,EF$. The in the same manner we deduce the fact that $AD,C_1F$ are antiparallel in the same angle. From these two observations we get $B_1E\\|C_1F$, which means that $M_1N\\|B_1E\\|C_1F$, because $M_1,N$ are the midpoints of the sides $B_1C_1,EF$ of the trapezoid $B_1EFC_1$. This, in turn, means that the lines $AD,M_1N$ are antiparallel in the angle formed by the lines $AM_1,DN$, i.e. $A,D,M_1,N$ are concyclic, Q.E.D.", "Solution_2": "I actually got this problem from the MOC-Book by Titu Andreescu. After I solved it I looked at the \"official\" solution in the book and the first thing I thought after reading it was: \"Hey, I've just found the ultimate IMO Nr 6 Problem: Find a more complicated solution than that one\" :lol: - A problem that I thought to be almost impossible. How wrong I was. When I saw the solution on Kalva, I almost fell off the chair (I did not bother to read it). \r\n\r\nSo here's my (hopefully right) solution:\r\n\r\n[hide]\n\n$AEBD$ and $AFDC$ are concyclic so $\\angle AED = \\angle ABD$ and $\\angle AFE = \\angle ACD$. \nThus $\\triangle ABC\\thicksim \\triangle AEF$. Because $M$ and $N$ divide $BC$ and $EF$ in the same ratio, the triangles $BAM$ and $EAN$ are similar, thus $\\frac{AN}{AM} = \\frac{AE}{AB}$ and $\\angle BAM = \\angle EAN \\Rightarrow \\angle NAM = \\angle EAB$. Thus the triangles $AMN$ and $ABE$ are similar and hence $\\angle ANM = \\angle AEB = 90^\\circ$.\n\n\nWe also see that $M$ and $N$ do not necessarily have to bisect $BC$ and $EF$, they only have to divide them in the same ratio. \n[/hide]", "Solution_3": "Here is my solution:\r\n\r\n\r\n Median AM divides segment between feet of AB and AC on side BC in equal parts, \r\n\r\n so it is quite ordinary and natural for a circle built on AM as diameter cut in half a \r\n\r\n segment passing through one of the intersection points (D) of circles built \r\n\r\n on AB and AC as diameters. Angle ANM looks at the diameter AM.\r\n\r\n\r\n Thank you.\r\n\r\n M.T.", "Solution_4": "Here is a equivalent enunciation (a wellknown problem):\r\n\r\n\"Let $w_1,w_2$ be two fixed secant circles and the (fixed) points $\\{A,D\\}\\subset w_1\\cap w_2$, $B\\in w_1$, $C\\in w_2$ so that $D\\in (BC)$ and $AD\\perp BC$. For the mobile points $E\\in w_1$, $F\\in w_2$ such that $D\\in EF$ we note the middlepoint $N$ of the segment $[EF]$. Ascertain the geometrical locus of the point $N$.\"\r\n\r\n[u]Answer:[/u] The in demand geometrical locus of the point $N$ is the circle with the diameter $[AM]$, where the point $M$ is the middlepoint of the segment $[BC]$.\r\n\r\n[u]A solution.[/u] $\\triangle AEF\\equiv \\triangle ABC\\Longrightarrow \\widehat {ANF}\\equiv \\widehat {AMC}\\Longrightarrow$\r\nthe quadrilateral $ANDM$ is cyclic $\\Longrightarrow NA\\perp NM.$" } { "Tag": [ "geometry", "circumcircle", "ratio", "inradius", "angle bisector", "similar triangles", "geometry proposed" ], "Problem": "1. Let $ABC$ be a triangle, $T$ its centroid and $S$ its incenter.\r\nProve that the following conditions are equivalent: \r\n\r\n(1) line $TS$ is parallel to one side of triangle $ABC$,\r\n\r\n(2) one of the sides of triangle $ABC$ is equal to the half-sum of the other two sides.\r\n\r\n\r\n2. Let $ABC$ be an isosceles triangle with $\\angle A=\\angle B=80^\\circ$. A straight line passes through $B$\r\nand through the circumcenter of the triangle and intersects the side $AC$ at $D$. Prove that $AB=CD$.", "Solution_1": "1. Suppose 2a=b+c. Let AS cut BC at D. By angle bisector theorem, $\\frac{AI}{ID}=\\frac{AB}{BD}=\\frac{AC}{CD}=\\frac{AB+AC}{BC}=\\frac{b+c}{a}=2$. The result follows since the centroid divides a median into 2 segments of the ratio 2:1.\r\n\r\n2. I think it's pretty well known, just construct a point E s.t. triangle EAB is equilateral.\r\n\r\nbtw, these 2 problems should belong to the Intermediate Forum.", "Solution_2": "[hide=\"Solution to Problem 1\"]\nThe first condition is equivalent to the distance from $T$ to one side $c$ equaling the distance from $S$ to that same side. The distance from $S$ is obviously the inradius, which is $\\frac{A}{s}$ ($A$ is the area of the triangle). The distance from $T$ is 1/3 of the altitude to that side because of similar triangles, which is also equal to $\\frac{2A}{3c}$. Setting our two quantities equal, we have $2s = 3c$, or $2c = a + b$. Incidentally, this is equivalent to the second condition, and we are done.\n[/hide]" } { "Tag": [ "search", "combinatorics unsolved", "combinatorics" ], "Problem": "There are nine people attending a meeting.Some of them are friends.\r\nIt is known that there exists no four people such that any of two are friends\r\nProve or disprove,nine people can be partioned into four groups such that any two people in the same group are not friends.", "Solution_1": "I think this follows from [url=http://en.wikipedia.org/wiki/Tur%C3%A1n%27s_theorem]Turan[/url].\r\n\r\nConsider a graph $ G$ with $ k$ vertices and containing no $ \\ell$ - clique. By Turan, in an maximal connected graph with no $ \\ell$ - clique, any two partitions are bipartite. If there exists more than $ \\ell \\minus{}1$ partitions, we can find a clique.", "Solution_2": "[quote=\"Talegari\"]I think this follows from [url=http://en.wikipedia.org/wiki/Tur%C3%A1n%27s_theorem]Turan[/url].\n\nConsider a graph $ G$ with $ k$ vertices and containing no $ \\ell$ - clique. By Turan, in an maximal connected graph with no $ \\ell$ - clique, any two partitions are bipartite. If there exists more than $ \\ell \\minus{} 1$ partitions, we can find a clique.[/quote]\r\nDid you mean if a graph contains no $ K_4$ then it must be a tripartite graph?\r\nBut it is not true...\r\nVery sorry if I misunderstand you :roll:", "Solution_3": "It is known that Ramsey numbers $ R(3,4) \\equal{} R(4,3) \\equal{} 9$, so there must be three persons $ x,y,z$ not knowing each other. For the remaining six, first add edges (edge = people are friends) until any new edge creates a $ K_4$ (thus creating an edge-extremal graph; at the end we will remove those edges, which are only to our disadvantage). Denote the remaining persons $ a,b,c,d,e,f$. There must exist some pair not friends, wlog $ a\\perp b$ (notation for lack of edge). Hence adding this edge must create some $ K_4$, wlog with $ c, d$, so there exist edges $ ac, ad, bc, bd, cd$. \r\nIf also $ ce$ and $ de$, then $ a\\perp e$ and $ b\\perp e$. If now also $ cf$ and $ df$, then $ a\\perp f$ and $ b\\perp f$, and also $ e\\perp f$, so we can take $ \\{ x,y,z \\}, \\{ a,b,e,f \\}, \\{ c \\}$ and $ \\{ d \\}$. Thus we must have wlog $ c\\perp f$ (or $ d\\perp f$), so we can take $ \\{ x,y,z \\}, \\{ a,b,e \\}, \\{ c,f \\}$ and $ \\{ d\\}$.\r\nSimilarly if we assume first $ cf$ and $ df$.\r\nIt remain the cases when $ c\\perp e$ and $ d\\perp f$, when we can take $ \\{ x,y,z \\}, \\{ a,b \\}, \\{ c,e \\}$ and $ \\{ d,f \\}$, and similarly when $ c\\perp f$ and $ d\\perp e$. \r\nFinally, when $ c\\perp e$ and $ c\\perp f$, but $ de$ and $ df$, then the only bad continuation is $ ef, af, be$ and $ a\\perp e, b\\perp f$ (or similar).\r\nThere remain triangles $ dac, dbc, daf, dbe$ and $ def$. If $ x\\perp d, y\\perp d$ and $ z\\perp d$, we are done, since we can take $ \\{ x,y,z,d \\}, \\{ a,e \\}, \\{ b,f \\}$ and $ \\{ c \\}$. So $ dx$ and/or $ dy$ and/or $ dz$. Now it's becoming a little messy, but at least the problem has been reduced to checking some not so large number of cases!\r\n(in all, the triplet $ \\{ x,y,z \\}$ may be now split).", "Solution_4": "I finally found a reference for this problem.\r\nIt is from[url=http://www.mathlinks.ro/viewtopic.php?search_id=281014044&t=170287]Bulgaria TST 2005[/url]" } { "Tag": [], "Problem": "what's better?", "Solution_1": "I haven't seen Madagascar, and i liked Shrek 2 better than Shrek. By the way :welcome: to AoPS. Nice avatar.", "Solution_2": "thanks!", "Solution_3": "What's madagascar? A movie?", "Solution_4": "Its an ok animated movie.", "Solution_5": "[quote=\"chess64\"]What's madagascar? A movie?[/quote]\r\n\r\nIt's highly popular right now. In fact, it's been on the top 10 list for movie profits for like months. :-) Bad movie. Don't go see it.\r\n\r\nAlthough that line \"Dang you, dang you all to heck.\" That was hilarious.", "Solution_6": "Madagascar was a bad movie....it had some REALLY funny parts in it, but overall it was a bad movie.", "Solution_7": "Wow, it seems you guys don't like madagascar. Maybe it was for younger children.", "Solution_8": "only 2 choices?", "Solution_9": "[quote=\"math92\"]Wow, it seems you guys don't like madagascar. Maybe it was for younger children.[/quote]\r\n\r\nNo. It is family appropriate, but it's not exactly a Lion King or a Barney type film. The movie is just stupid, despite the good animation.\r\n\r\nWhy on earth isnt The Incredibles on that poll?", "Solution_10": "I thought that the Incredibles and Finding Nemo were great movies! THey should be on the poll", "Solution_11": "has anyone seen national treasure GREAT MOVIE and pacifier is ok too i heard the madagascar stinks except people were obsessed with i like to move it move it?(no idea what it is) shrek was ok but shrek 2 was definitely better. finding nemo was good but shark tale was a rip off of nemo. the incredibles was a good movie", "Solution_12": "I liked National Treasure but this thread seems to be about animated movies. :P In my opinion, The Incredibles is clearly one of the best out. Shrek is good too, although I must say the lil Fairy Godmother thingy in Shrek II kinda killed some of it.", "Solution_13": "i haven't seen Madagascar yet but from teh votes it seems like Shrek is way better", "Solution_14": "[quote=\"juicybooty911\"]only 2 choices?[/quote]\r\n\r\nIt's just to see which one is better.", "Solution_15": "I think that Shrek was an awesome movie... Saw it like 20 times in a row though... It's still a great (but boring after 20 times) movie.\r\n\r\nAs for madagascar, my sister liked it but she's 12.", "Solution_16": "[quote=\"mathbomberII\"]i haven't seen Madagascar yet but from teh votes it seems like Shrek is way better[/quote]\r\n\r\nYour'e right :P \r\n\r\ni've never seen madagascar, but I've heard from my friends it was a good one, but from you, it sounds awful!", "Solution_17": "shrek! :D :D :D" } { "Tag": [ "conics", "ellipse", "linear algebra", "matrix", "calculus", "derivative", "function" ], "Problem": "Suppose I multiply all points on the unit disc with the matrix $ A$ as defined below:\r\n$ A \\equal{} \\left( \\begin{array}{cc}\r\n2 & 3 \\\\\r\n0 & 4 \\end{array} \\right)$\r\nThis will result in an ellipse. Now I'd like to find out how far the farthest point on this ellipse is from the origin. This problem is supposed to be solvable without using derivatives and without ever multiplying $ A$ with a unit vector.", "Solution_1": "Calculate the greatest eigenvalue of the (positive) matrix $ A^tA$.", "Solution_2": "Thanks! Why is that working?", "Solution_3": "Counter question: Where do you know from that the result is an ellipse? How is it parametrized? ;)", "Solution_4": "Sorry, I dont follow... I know that it's an ellipse since the problem states that :)", "Solution_5": "What you are looking for is the maximum of the function $ f: S^1\\to\\mathbb R$ with $ f(x)\\equal{}x^t(A^tA)x$. Since $ B\\equal{}A^tA$ is symmetric, we can orthogonally diagonalize $ B$. This implies (?) that we can assume WLOG that $ B$ is diagonal where the diagonal elements are the the eigen values of $ A^tA$. \r\n\r\nDoes this help you?", "Solution_6": "Yup, perfectly! Thanks a lot!" } { "Tag": [], "Problem": "hey is anybody here doing engineering who has applied for kvpy in EA or EB stream. please post here or pm me. \r\n i'd like to know what kind of projects have been made and what's the allowed scope and syllabus and so on.", "Solution_1": "This link might be of some use to you:\r\n\r\nhttp://www.orkut.com/CommMsgs.aspx?cmm=453797&tid=2463773829331124275&kw=KVPY+award+winning+projects", "Solution_2": "may be mailing rushil would help" } { "Tag": [ "analytic geometry", "trigonometry", "ratio", "absolute value", "real analysis", "real analysis solved" ], "Problem": "I generally don't enjoy this type of problem, but my college friend needed help with it and it ended up being pretty interesting. I got an answer, but he said the computer was telling him it was wrong. Let me know what you get.\r\n\r\nA clock has a $10\\text{cm}$ long minute hand and a $5\\text{cm}$ hour hand. How fast is the distance between the end of the minute hand and the end of the hour hand changing (in $\\frac{\\text{cm}}{\\text{hr}}$) at $3\\text{ o'clock}$?\r\n\r\nMy answer: $-\\frac{5900\\pi}{\\sqrt{125}}\\frac{\\text{cm}}{\\text{hr}}$", "Solution_1": "I get $\\frac{5}{12} \\sqrt{48^2+1} \\pi$", "Solution_2": "Put the coordinate origin at the clock center, x-axis through 3 o'clock and y-axis through 12 o'clock. Let $A = 5\\ \\text{cm}$, $B = 10\\ \\text{cm}$ be the lengths of the hour and minute hands, $\\omega = 2 \\pi\\ \\text{rad/hr}$ the angular velocity of the minute hand and let the time be $t = 0$ at midnight (or at noon). The endpoints $H, M$ of the hour and minute hands have coordinates\r\n\r\n$H = [x_H, y_H] = [A \\sin{\\frac{\\omega}{12} t}, A \\cos{\\frac{\\omega}{12} t}]$\r\n\r\n$M = [x_M, y_M] = [B \\sin{\\omega t}, B \\cos{\\omega t}]$\r\n\r\nThe distance $HM$ between the hand endpoints is\r\n\r\n$HM = \\sqrt{(x_H - x_M)^2 + (y_H - y_M)^2} = \\sqrt{\\left( A \\sin{\\frac{\\omega}{12} t} - B \\sin{\\omega t} \\right)^2 + \\left(A \\cos{\\frac{\\omega}{12} t} - B \\cos{\\omega t} \\right)^2}$\r\n\r\n$\\frac{dHM}{dt} = \\frac{\\left( A \\sin{\\frac{\\omega}{12} t} - B \\sin{\\omega t} \\right) \\left( \\frac{\\omega}{12} A \\cos{\\frac{\\omega}{12} t} - \\omega B \\cos{\\omega t} \\right) - \\left( A \\cos{\\frac{\\omega}{12} t} - B \\cos{\\omega t} \\right) \\left( \\frac{\\omega}{12} A \\sin{\\frac{\\omega}{12} t} - \\omega B \\sin{\\omega t} \\right)}{\\sqrt{\\left( A \\sin{\\frac{\\omega}{12} t} - B \\sin{\\omega t} \\right)^2 + \\left(A \\cos{\\frac{\\omega}{12} t} - B \\cos{\\omega t} \\right)^2}}$\r\n\r\nObviously, at the time $t = 3\\ \\text{hr}$, \r\n\r\n$sin{3 \\omega} = \\sin{6 \\pi} = \\sin 0 = 0$\r\n$\\cos{3 \\omega} = \\cos{6 \\pi} = \\cos 0 = 1$\r\n\r\n$\\sin{\\frac{\\omega}{4}} = \\sin{\\frac{\\pi}{2}} = 1$\r\n\r\n$\\cos{\\frac{\\omega}{4}} = \\cos{\\frac{\\pi}{2}} = 0$\r\n\r\n$\\frac{dHM}{dt} = \\frac{-\\omega AB \\left( 1 - \\frac{1}{12} \\right)}{\\sqrt{A^2 + B^2}} = -\\frac{2 \\pi \\cdot 5 \\cdot 10 \\cdot \\frac{11}{12}}{\\sqrt{5^2 + 10^2}} = \\frac{11 \\pi \\sqrt 5}{3}\\ \\text{cm/hr}$\r\n\r\nThat makes 3 different results, so here is a sanity check: Let the time be $t = 0$ at 3 o'clock. Consider the endpoint $H$ of the hour hand moving vertically down from the 3 o'clock position at the tangent velocity $u = -\\frac{\\omega}{12} A$ and the enpoint $M$ of the minute hand move horizontally right from the 12 o'clock position at the tangent velocity $v = \\omega B$. Then\r\n\r\n$H = [A, -\\frac{\\omega}{12}At]$\r\n\r\n$M = [\\omega B t, B]$\r\n\r\n$HM = \\sqrt{(x_H - x_M)^2 + (y_H - y_M)^2} = \\sqrt{(A - \\omega B t)^2 + \\left( B + \\frac{\\omega}{12}At \\right)^2}$\r\n\r\n$\\frac{dHM}{dt} = \\frac{-\\omega B (A - \\omega Bt) + \\frac{\\omega}{12} A \\left( B + \\frac{\\omega}{12} At \\right)}{\\sqrt{(A - \\omega B t)^2 + \\left( B + \\frac{\\omega}{12} At \\right)^2}}$\r\n\r\nAt the time $t = 0$\r\n\r\n$\\frac{dHM}{dt} = \\frac{-\\omega AB \\left( 1 - \\frac{1}{12}\\right)}{\\sqrt{A^2 + B^2}} = -\\frac{2 \\pi \\cdot 5 \\cdot 10 \\cdot \\frac{11}{12}}{\\sqrt{5^2 + 10^2}} = \\frac{11 \\pi \\sqrt 5}{3}\\ \\text{cm/hr}$", "Solution_3": "This is what I did.\r\n\r\nedit: i just realized the ratio of the minute hand's angular velocity to that of the hour hand is 12, not 60 :blush: \r\n\r\n\\[D=\\sqrt{(x_m-x_h)^2+(y_m-y_h)^2}\\]\r\nDifferentiate with respect to time.\r\n\\[D'=\\frac{(x_m-x_h)(x_m'-x_h')+(y_m-y_h)(y_m'-y_h')}{\\sqrt{(x_m-x_h)^2+(y_m-y_h)^2}}\\]\r\nNow we need to find equations for $x_m,x_h,y_m,y_h$: ($t$ is in hours)\r\n\r\n$x_m=10\\sin12\\cdot2\\pi{t}$\r\n$x_m'=240\\pi\\cos24\\pi{t}$.\r\n\r\n$x_h=5\\cos2\\pi{t}$\r\n$x_h'=-10\\pi\\sin2\\pi{t}$.\r\n\r\n$y_m=10\\cos12\\cdot2\\pi{t}$\r\n$y_m'=-240\\pi\\sin24\\pi{t}$.\r\n\r\n$y_h=-5\\sin2\\pi{t}$\r\n$y_h'=-10\\pi\\cos2\\pi{t}$.\r\n\r\nThese give the position of the hour and minute hands at time $t$ (hours), where $t=0$ corresponds to $3\\text{ o'clock}$. Thus\r\n\r\n\\[D'|_{t=0}=\\frac{(0-5)(240\\pi-0)+(10-0)(0-(-10\\pi))}{\\sqrt{(0-5)^2+(10-0)^2}}=-\\frac{1100\\pi}{\\sqrt{125}}\\frac{\\text{cm}}{\\text{hr}}=-\\frac{220\\pi}{\\sqrt{5}}\\frac{\\text{cm}}{\\text{hr}}.\\]\r\n\r\nFirst, I noticed that your answers were both positive, but I'm pretty sure the answer should be negative. They're moving closer together because the minute hand is moving to the right and the hour hand is barely moving with respect to it.\r\n\r\n(a few edits in the following few sentences..)\r\n\r\nAlso, I set up a simpler problem, where the hour hand is stationary and the tip of the minute hand is moving to the right (linearly) at a constant speed: $\\left(\\frac{12\\text{ revolutions}}{hr}\\right)\\left(\\frac{2\\cdot10\\text{cm}\\cdot\\pi}{\\text{revolution}}\\right)=240\\pi\\frac{\\text{cm}}{\\text{hr}}$. I graphed $D(t)$ and found $\\frac{dD}{dt}$ with my calculator and got a number that was slightly less (but greater in absolute value) than my answer for the original problem, which is what would be expected.", "Solution_4": "[quote=\"AntonioMainenti\"] Thus\n\n\\[D'|_{t=0}=\\frac{(0-5)(240\\pi-0)+(10-0)(0-(-10\\pi))}{\\sqrt{(0-5)^2+(10-0)^2}}=-\\frac{1100\\pi}{\\sqrt{125}}\\frac{\\text{cm}}{\\text{hr}}=-\\frac{220\\pi}{\\sqrt{5}}\\frac{\\text{cm}}{\\text{hr}}.\\]\n\nFirst, I noticed that your answers were both positive, but I'm pretty sure the answer should be negative. They're moving closer together because the minute hand is moving to the right and the hour hand is barely moving with respect to it.[/quote]\r\n\r\nMy answers are both negative (both expressions have the minus signs even after substituting numbers). Just the final fractions (one a copy of the other) have the minus sign missing, otherwise, the value is correct.\r\n\r\nAnother sanity check: the angular velocity of the [b]minute[/b] hand is $2 \\pi\\ \\text{rad/hr}$ (1 full revolution per hour). The orbital velocity of the minute hand enpoint is $10\\ \\text{cm} \\cdot 2 \\pi\\ \\text{rad/hr} = 20 \\pi\\ \\text{cm/hr}$. Surely, the absolute value of the result has to be less than that. You considered the angular velocity of the [b]hour[/b] hand to be $2 \\pi\\ \\text{rad/hr}$, but it is $\\frac{2 \\pi}{12}\\ \\text{rad/hr}$ ( 1 full revolution per 12 hours).", "Solution_5": "[quote=\"yetti\"][quote=\"AntonioMainenti\"] Thus\n\n\\[D'|_{t=0}=\\frac{(0-5)(240\\pi-0)+(10-0)(0-(-10\\pi))}{\\sqrt{(0-5)^2+(10-0)^2}}=-\\frac{1100\\pi}{\\sqrt{125}}\\frac{\\text{cm}}{\\text{hr}}=-\\frac{220\\pi}{\\sqrt{5}}\\frac{\\text{cm}}{\\text{hr}}.\\]\n\nFirst, I noticed that your answers were both positive, but I'm pretty sure the answer should be negative. They're moving closer together because the minute hand is moving to the right and the hour hand is barely moving with respect to it.[/quote]\n\nMy answers are both negative (both expressions have the minus signs even after substituting numbers). Just the final fractions (one a copy of the other) have the minus sign missing, otherwise, the value is correct.\n\nAnother sanity check: the angular velocity of the [b]minute[/b] hand is $2 \\pi\\ \\text{rad/hr}$ (1 full revolution per hour). The orbital velocity of the minute hand enpoint is $10\\ \\text{cm} \\cdot 2 \\pi\\ \\text{rad/hr} = 20 \\pi\\ \\text{cm/hr}$. Surely, the absolute value of the result has to be less than that. You considered the angular velocity of the [b]hour[/b] hand to be $2 \\pi\\ \\text{rad/hr}$, but it is $\\frac{2 \\pi}{12}\\ \\text{rad/hr}$ ( 1 full revolution per 12 hours).[/quote]I see now. Thanks for clearing that up. This is why I hate doing related rates problems.." } { "Tag": [ "geometry", "trapezoid" ], "Problem": "Q:Prove that a trapezium with non-parallel sides equal is cyclic.", "Solution_1": "An isosceles trapezoid has congruent pairs of base angles and the 2 angles on one leg are supplementary, so the opposite angles are supplementary and the trapezoid is cyclic." } { "Tag": [ "mathleague.org", "AMC", "AIME", "email", "AIME I" ], "Problem": "Yup yet another mathleague.org offering.\r\n\r\nPost how you felt about the contest: I felt the sprint round was disappointingly easy, the target round was good difficulty, and the countdown? Let's just say I got lucky - twice.", "Solution_1": "Sprint - I don't know, maybe Chapterish difficulty\r\nTarget - I don't know, maybe slightly less than Chapterish difficulty\r\n\r\nI guess the difficulty might have been about right for middle schoolers...\r\n[hide]\nbut dang, that was boring\n[/hide]", "Solution_2": "Also, what happened to Miller Team #1?\r\n\r\n@Ube, sure, you are too old :D", "Solution_3": "[quote=\"james4l\"]Also, what happened to Miller Team #1?\n[/quote]\r\n\r\nHmm... I wonder where Kevin Lei and Nikhil Buduma were?", "Solution_4": "oh so that explains why you weren't at Scioly.\r\nwho else was there? i know ccy + bookaholic were at scioly w/ me.", "Solution_5": "nikhil, klei were just gone for some reason\r\n\r\nmieh i failed written AND cd today. *cries*\r\n\r\nwow i added wrong about 5 times on cd today\r\n\r\nlol, 6th-7th-8th scores were pretty funny\r\n\r\n6th:\r\n\r\n1st: 29?\r\n2nd: 28?\r\n3rd: 27\r\n\r\nor something like that\r\n\r\n7th\r\n\r\n1st: 45 (james4l)\r\n2nd: 39 (me) :wallbash_red:\r\n3rd: 38 \r\n\r\n8th\r\n\r\n1st: 45 (miller4math)\r\n2nd: 44 (ecstaticpotter)\r\n3rd: 39 (bowser)\r\n\r\nmieh.\r\n\r\nnotice difference between 6th and 7th/8th scores :D\r\n\r\nhm, ubey saw me fail countdown? not good.", "Solution_6": "[quote=\"bowei\"]oh so that explains why you weren't at Scioly.\nwho else was there? i know ccy + bookaholic were at scioly w/ me.[/quote]\r\nAlbert doesn't do scioly though, doesn't he? I had to explain to him what scioly was just a month ago.\r\nAlso, scioly=silly.", "Solution_7": "AAAAAAA NOW EVERYONES GOING TO KNOW WHO I AM >:[ o well its just ANYONE WHO REMEMBERS ehh just kidding...? me::winner_third: \r\nactually no \r\nvictor and johnny: :winner_first: :winner_second: me: :dry: hahaha\r\n\r\n>.< i failed....hmm.....\r\n3 stupid mistakes on sprint :wallbash: + bombed target round :wallbash_red: :wallbash_red: = 3rd place?? i am amazed\r\nbut i was just lucky that kevin lei or nikhil buduma didnt go\r\nand U B E Y M A Y A M Y M O M S A W Y O U R M O M @_____@ i hope you can read that...theres like 10 O's and 20 A's and 30 M's...HI I THINK THAT YOUR REALLY SMART\r\n\r\nyesterday there was a 1 point gap between 1st and 2nd\r\nAND THEN A 5 POINT GAP BETWEEN 2ND AND 3RD BWAHAHAHA\r\ni just got lucky XD yay now i have a pretty plaque or however you spell it :D \r\n\r\ni hope i dont fail aime!! i practiced a lot and i have one more practice tomorrow....before the real test.....GASP :wow: I WILL NOT FAIL!!! if i tell myself i wont fail then maybe i wont :jump:\r\n\r\nwhat?? how come theres a smiley limit?? >____<", "Solution_8": "yo please limit # of smileys to like\r\n1\r\n\r\nits annoying to read a post spammed with smileys\r\nso i didnt read your post\r\nok\r\n?", "Solution_9": "Well that wasn't really too many smileys... it only becomes spammy when you add 5+ in a row.\r\n\r\nAlso darn I got a 41 on that test... misread 3 on sprint and 1 on target (for example on #7 on target I thought that the vertices of the square were on (-10, 0) and (0, 4), because when I read it my brain automatically assumed \"it couldn't be on (4,0) because they wouldn't make the problem THAT trivial\").", "Solution_10": "What contest are you guys talking about?", "Solution_11": "i failed cd worse", "Solution_12": "lol edgar\r\n\r\nthat is you right?\r\n\r\nwhat grade are you in?\r\n\r\nwhat was your score?\r\n\r\nO_O\r\n\r\ni want to know akshay's score\r\n\r\ntime to email him\r\n\r\nand for countdown\r\n\r\nyou should have seen my scratch paper\r\n\r\ni looked at it afterwards and found 7 adding erros and 2 multiplication errors\r\n\r\nfail?", "Solution_13": "STOP QUOTING 80 LINE POSTS AND REPLYING WITH A ONE LINE POST", "Solution_14": "[quote=\"junggi\"]STOP QUOTING 80 LINE POSTS AND REPLYING WITH A ONE LINE POST[/quote]\r\n\r\njunggi, plz stop with the remarks and stuff, it wasnt an 80 line post, and people have there reasons...\r\n\r\nanyway... what tournament is this???", "Solution_15": "mathleague.org has a better description than what i can give.", "Solution_16": "[quote=\"weird888\"][quote=\"junggi\"]STOP QUOTING 80 LINE POSTS AND REPLYING WITH A ONE LINE POST[/quote]\n\njunggi, plz stop with the remarks and stuff, it wasnt an 80 line post, and people have there reasons...\n\nanyway... what tournament is this???[/quote]\r\nwhat\r\nits like basic forum etiquette \r\nyou dont reply to a page long post with one line\r\nespecially if that one line doesnt concern the whole post\r\nlike cut out the parts you arent responding to\r\ni dont want to scroll down a whole page while rereading a huge post and then read a one line post which is irrelevent to most of the post", "Solution_17": "yes, 7th, -7536725890504451", "Solution_18": "BLARGH", "Solution_19": "Okay, I agree. No more pointlessly long quotations with pointlessly short responses.", "Solution_20": "hi best friend", "Solution_21": "[quote=\"james4l\"]Also, what happened to Miller Team #1?\n[/quote]\r\nwell u see the dumb rules divided our scores by 2...\r\nand we got 41.5 or something...\r\nlol and the third place team was 47.5\r\n :dry:", "Solution_22": "2nd place 7th grade team." } { "Tag": [ "function", "limit", "real analysis", "real analysis unsolved" ], "Problem": "Let $ f_n: N\\rightarrow C$ a sequence of complex functions on the positive integers such that $ \\sum_{m\\equal{}1}^{\\infty}|f_n(m)|<\\infty$ for all $ n$.\r\n\r\nSuppose that $ \\lim_{n\\rightarrow\\infty} \\sum_{m\\equal{}1}^{\\infty}f_n(m)y_m\\equal{}0$ for every bounded complex sequence $ \\{y_m\\}$.\r\n\r\nProve that $ \\lim_{n\\rightarrow\\infty} \\sum_{m\\equal{}1}^{\\infty}|f_n(m)|\\equal{} 0$.", "Solution_1": "I just guess, maybe I am wrong :lol: \r\n\r\nwrite $ f_{n} = f^{1}_{n} + if^{2}_{n}$ so we can assume $ f_{n}$ is a real -valued sequence\r\n\r\nchose bounded sequences and $ ({y_{m})}$, st : $ 0\\leq \\sum_{m = 1}^{k}|f_{n}(m)|\\leq \\sum_{m = 1}^{k}f_{n}(m)y_{m}$\r\n\r\nsince $ \\sum_{m = 1}^{k}f_{n}(m)y_{m}\\rightarrow 0$ when $ k,n\\rightarrow \\infty$ so $ \\sum_{m = 1}^{k}|f_{n}(m)|\\rightarrow 0$", "Solution_2": "I think this is in exact consequance of the uniform boundedness principle or Banach-Steinhaus theorem." } { "Tag": [ "quadratics", "algebra" ], "Problem": "Find the product of the roots of the equation $ 18t^2 \\plus{} 45t \\minus{}500 \\equal{}0$.", "Solution_1": "hello, solving the given quadratic equation we get $ t_1\\equal{}\\frac{25}{6}$, $ t_2\\equal{}\\minus{}\\frac{20}{3}$ and their product is $ \\minus{}\\frac{250}{9}$.\r\nSonnhard.", "Solution_2": "To make it faster (since it's Countdown), we use Vieta's. In a quadratic equation $ ax^2\\plus{}bx\\plus{}c$, the product of the roots is $ \\frac{c}{a}$. So in this case, we do $ \\minus{}\\frac{500}{18}\\equal{}\\boxed{\\minus{}\\frac{250}{9}}$, as stated by Dr Sonnhard Graubner." } { "Tag": [ "inequalities", "inequalities proposed" ], "Problem": "Prove that the sides a,b,c of any triangle suck that $ a \\plus{} b \\plus{} c \\equal{} 3$ satisfy the inequality\r\ni/$ \\sum\\frac {a}{b\\plus{}c^2}>\\equal{}\\frac{3}{2}$", "Solution_1": "you have posted the same problem here,an hour ago:\r\n[url]http://www.mathlinks.ro/viewtopic.php?t=199993[/url]\r\n :huh:" } { "Tag": [ "limit", "algebra unsolved", "algebra" ], "Problem": "[u][i]Problem[/i][/u]\r\n\r\n\r\n [i]Let two sequences $ \\{a_{n}\\}$ , $ \\{b_{n}\\}$ defined by :\n\n\n\n $ a_{0} \\equal{} b_{0} \\equal{} 1$\n\n $ a_{n \\plus{} 1} \\equal{} a_{n} \\plus{} b_{n}$\n\n\n and $ b_{n \\plus{} 1} \\equal{} a_{n} \\plus{} 3 b_{n}$ with all $ n \\in N , n \\geq 1$\n\n\n Find $ \\lim_{n\\to\\infty}\\frac {a_{n}}{b_{n}}$[/i]\r\n\r\n\r\n\r\n [i]Nice problem[/i]\r\n :lol: \r\n\r\n\r\n [i] Love An Forever\n\n H\u00f9ynh V\u00f5 Ph\u01b0\u01a1ng An[/i]", "Solution_1": "Dividing the two equations we get:\r\n\r\n$ \\frac{b_{n\\plus{}1}}{a_{n\\plus{}1}}\\equal{}1\\plus{}\\frac{2 b_n}{a_n\\plus{}b_n}\\equal{}1\\plus{}\\frac{2}{\\frac{a_n}{b_n}\\plus{}1}$\r\n\r\nNow, we know that:\r\n\r\n$ \\lim_{n \\rightarrow \\infty} \\frac{a_n}{b_n}\\equal{}\\lim_{n \\rightarrow \\infty} \\frac{a_{n\\plus{}1}}{b_{n\\plus{}1}}$\r\n\r\nand suppose that the limit actually exists and equals L. Now, let $ n \\rightarrow \\infty$ and get:\r\n\r\n$ \\frac{1}{L}\\equal{}1\\plus{}\\frac{2}{L\\plus{}1}$\r\n\r\n$ L\\plus{}1\\equal{}(L\\plus{}1)L\\plus{}2 L$\r\n\r\nNow, we have:\r\n\r\n$ L_{1,2} \\equal{} \\pm \\sqrt{2}\\minus{}1$\r\n\r\nbut since $ a_0,\\, b_0 \\geq 0$ and $ a_n < a_{n\\plus{}1}$ and $ b_n < b_{n\\plus{}1}$ we conclude that $ a_n,\\, b_n \\geq 0$ so $ L$ must be positive and hence we have:\r\n\r\n$ L \\equal{} \\sqrt{2}\\minus{}1$", "Solution_2": "\\[ \\left\\{\\begin{aligned}a_n&\\equal{}\\frac{(2\\plus{}\\sqrt2)^n\\plus{}(2\\minus{}\\sqrt2)^n}2,\\\\b_n&\\equal{}\\frac{(2\\plus{}\\sqrt2)^{n\\plus{}1}\\minus{}(2\\minus{}\\sqrt2)^{n\\plus{}1}}{2\\sqrt2}.\\end{aligned}\\right.\\]" } { "Tag": [ "probability", "expected value" ], "Problem": "When looking at a book at a hypothetical online book store, you can click \"surprise me\" to see a random page from the book. On average, how many clicks will it take for you to see the entire book?", "Solution_1": "[hide=\"Solution\"]\nSuppose there are $p$ pages. Consider $p$ steps; in each step, the total number of distinct pages viewed increases by 1.\n\nSuppose $n$ distinct pages of $p$ have been viewed. The probability that the next page will be a new one is $\\frac{p-n}{p}$. Hence, the expected number of pages that must be viewed to reach $n+1$ distinct pages viewed is given by:\n$\\sum_{j=1}^{\\infty}j \\left( \\frac{p-n}{p}\\right) \\left( \\frac{n}{p}\\right)^{j-1}= \\frac{p}{p-n}$\n\nNow, we just have to sum this over all of the $p$ steps:\n$\\sum_{n=0}^{p-1}\\frac{p}{p-n}$\nwhich cannot be further simplified.\n[/hide]", "Solution_2": "Wow, that was fast!\r\n\r\nMy friend The Purple Cow came up with a somewhat easier-to-read version of your final formula: [hide]Let n be the number of pages. The expected number of clicks is then\n\n$n \\sum_{k=1}^{n}\\frac{1}{k}$[/hide]" } { "Tag": [ "\\/closed" ], "Problem": "When I look at Resources, I can't download PDF version.\r\nWhat's the matter with that?\r\n\r\nkunny", "Solution_1": "i try to download the jbtst and i failed too. please answer kunny's question.", "Solution_2": "Problem was detected and (finally) fixed :)", "Solution_3": "Thank you! :lol:", "Solution_4": "Yay Valentin's back at last...\r\n\r\nI think this has been an issue for a while, right?\r\nWell thanks for fixing it!", "Solution_5": "THANK YOU VERY VERY MUCH!!!", "Solution_6": "I can't really post anything but does the PDF file database get automatically updated when a new problem is added, or does the physical PDF have to get updated?", "Solution_7": "The PDF file gets updated when someone edits the Resources page for that particular contest (e. g. changes the links to the posts with the problems, or does as if he would change them but changes nothing), [i]not[/i] when the posts with the problems are edited. Hence, the PDF file may be slightly obsolete compared with the HTML resources page.\r\n\r\n@Valentin: thanks a lot on my part!\r\n\r\n darij", "Solution_8": "[quote=\"darij grinberg\"]The PDF file gets updated when someone edits the Resources page for that particular contest (e. g. changes the links to the posts with the problems, or does as if he would change them but changes nothing), [i]not[/i] when the posts with the problems are edited. Hence, the PDF file may be slightly obsolete compared with the HTML resources page\n darij[/quote]Unfortunately you are wrong Darij. The pdf's are updated as you change stuff in the posts themselves ;)", "Solution_9": "I noticed this because there is some trouble with images etc. that should be looked into, did you yourself write the code that generates the pdf?", "Solution_10": "[quote=\"Valentin Vornicu\"]Unfortunately you are wrong Darij. The pdf's are updated as you change stuff in the posts themselves ;)[/quote]\r\n\r\nNice update ;) .\r\n\r\n Darij", "Solution_11": "[quote=\"darij grinberg\"][quote=\"Valentin Vornicu\"]Unfortunately you are wrong Darij. The pdf's are updated as you change stuff in the posts themselves ;)[/quote]\n\nNice update ;) .\n\n Darij[/quote]It's not update, it's the way I designed it from the first time ;)\r\n\r\nme@home: there's a new version coming up soon, so I'm not really working on this one anymore. Have a bit of patience, Resources 2.0 will be here :P", "Solution_12": "thanks, valentin." } { "Tag": [ "trigonometry" ], "Problem": "In any triangle $ABC$. Prove that: \r\n\\[bc(b^{2}-c^{2})\\cos A+ca(c^{2}-a^{2})\\cos B+ab(a^{2}-b^{2})\\cos C=0\\]", "Solution_1": "[hide]Plug $\\cos A={b^{2}+c^{2}-a^{2}\\over 2bc}$ etc. and expand to obtain mass cancelation.[/hide]" } { "Tag": [ "calculus", "integration", "limit", "real analysis", "real analysis unsolved" ], "Problem": "how to integrate\r\n\r\n1/(1+x+x^(2n))?\r\ni know that usually it should be factorized but how...isn't there an easier way\r\nthis is par of\r\n\r\nproblem: a in(0,1). find lim (n->+infinity) of integral(from -a to a) of dx/[(1+x^2+x^(2n))(1+e^x)].\r\n\r\nnotes:the integral in the problem simplifies to integral(from 0 to a)dx/[(1+x^2+x^(2n)) after representing it like integral(-a to 0)+integral(0 to a) and changing x=-t in the first.", "Solution_1": "If I understand your problem corectly you need to calculate the following limit\r\n\r\n$\\lim_{n\\rightarrow \\infty}\\int_{0}^{a}\\frac{dx}{1+x^{2}+x^{2n}}=\\int_{0}^{a}\\frac{dx}{1+x^{2}}=\\arctan(a)$.\r\n\r\nThe limit follows based on the Bounded convergence Theorem, since \r\n\r\n$0 (a-b)(a+b)$ (as $a+a=a-b+a+b$). Multiplying out we get $a^{2}> a^{2}-b^{2}$, which is true (as we know $b>0$). \r\n\r\n[hide=\"Hint\"]\nUse this together with Heron's Formula.\n[/hide]", "Solution_3": "For #1, you just use the same method you would if you were to find the maximum area for a given amount of fence you have.", "Solution_4": "[hide=\"q4\"]\n$\\text{nth}=\\frac{(100)!n}{(100-n)!100^{n}}$\nThen find the maximum...\nI have no clue\n[/hide]\r\nThe diagram is for question 2", "Solution_5": "[quote=\"fizsle\"]I don't know the actual theorem but i think by number theory u can prove it... product of two equal numbers is greater than that of products of two other numbers having the same sum... :maybe:[/quote]\r\nbut it's only true if you prove that $a+b$ is constant-$c$ is constant' but I'm not sure it's true. how can we use heron?\r\nfor question 3, we can work in base 24 and then its esay but I think you didn't meant it.\r\n$Q4.$ in the 99th day?\r\nfor $Q6$ I know it doesn't correct but how do we find a counterwxample in a nice way?", "Solution_6": "6. false\r\nfix a base, then the locus of constant area for the last side is a parallel line, the locus for perimeter is an ellipse (with the endpoints of the base as the foci), their intersection is the third vertex\r\n\r\nhowever if we sufficiently extend the length of the base so it is greater than any side of the first triangle, then we have a different triangle (intersection is guaranteed because the ellipse gets bigger while the line gets closer", "Solution_7": "Does anyone have a solution for the other problems i.e. q4, q3?\r\nThanks" } { "Tag": [ "linear algebra", "matrix", "vector", "linear algebra unsolved" ], "Problem": "Hi all,\r\n\r\nlet A a n*n matrix of range r. Consider the following (n-1)*(n-1) matrix B such that B(i,j)=det( A(1 ,1) A(1 ,i+1)\r\n A(j+1,1) A(i+1,j+1) )\r\nFind the range of B ...\r\n\r\nbye.", "Solution_1": "I'm sure by \"range\" you mean \"rank\", right?", "Solution_2": "of course .. excuse my poor english..", "Solution_3": "the rank ;-)", "Solution_4": "After some computation \r\n$B=a_{11}A' - Z.W^t$ where \r\n\r\n$A'=(a_{ij})_{2\\leq i,j\\leq n}$ and $Z$,$W$ is vector column in $R^{n-1}$\r\n\r\n$W^t$ is the transpose of $W$\r\n\r\nZ=\r\n(a_{12})\r\n(a_{13})\r\n............\r\n............\r\n(a_{1n})\r\n\r\nW=\r\n(a_{2,1})\r\n(a_{3,1})\r\n................\r\n................\r\n(a_{n,1})\r\n\r\n...", "Solution_5": "yeah that's .. and to conclude, think about the LU decomposition ...", "Solution_6": "sorry.. but what's LU decomposition?", "Solution_7": "http://mathworld.wolfram.com/LUDecomposition.html", "Solution_8": "It can;t always be $r-1$. If the first column is $0$, for example, it can be $0$.", "Solution_9": "If all element of the first column of $A$ is zero $a_{i1}=0$ then $B=0$, and \r\n$rank(B)=0$ :D", "Solution_10": "[quote=\"Moubinool\"]\n$B=a_{11}A' - Z.W^t$ where \n\n$A'=(a_{ij})_{2\\leq i,j\\leq n}$ and $Z$,$W$ is vector column in $R^{n-1}$\n\n$W^t$ is the transpose of $W$\n\nZ=\n(a_{12})\n(a_{13})\n............\n............\n(a_{1n})\n\nW=\n(a_{2,1})\n(a_{3,1})\n................\n................\n(a_{n,1})\n\n...[/quote]\r\n\r\n\r\nIf $a_{11}=0$ and if there exist $a_{1,j}\\neq0$ and $a_{i1}\\neq0$ we have $B= -Z.W^t\\neq0$ \r\n => $rank(B)=1$", "Solution_11": "If $a_{11} \\neq 0$ and if there exist $a_{i1}\\neq0, a_{1j} \\neq 0$\r\nthe rank of $B$ is $rank(A)-1$" } { "Tag": [ "number theory unsolved", "number theory" ], "Problem": "Find the minimum natural number $n$ with the following property: between any collection of $n$ distinct natural numbers in the set $\\{1,2, \\dots,999\\}$ it is possible to choose four different $a,\\ b,\\ c,\\ d$ such that: $a + 2b + 3c = d$.", "Solution_1": "Denote by M a set of numbers such that for any four different numbers we have $a+2b+3c\\neq d$.Denote by m the number of its elements which are $a_1a_m-3a_1-2a_2$", "Solution_2": "mmm... \r\n\r\nso wich is the minimun value of $ n$??\r\n\r\n\r\n\r\n:huh:", "Solution_3": "@ElChapin:\r\naccording to xirti the minimum value of $ n$ is $ 999\\minus{}833 \\equal{} 166$", "Solution_4": "mmm... I don't have idea of wath he tried to do then...\r\n\r\n166 non even in dreams...\r\n\r\n833 the maximun without the property, but even 834 doesn't work...\r\n\r\n835 seems to be the answer, but I can't prove that you can alwas find $ a$, $ b$, $ c$, $ d$.", "Solution_5": "elchapin 835 is the answer :lol: \r\nsee [url=http://www.kalva.demon.co.uk/ibero/isoln/isol983.html]here[/url] :wink:", "Solution_6": "Initially, will select the last $k$ integers so that when equality occurs above. Since the smallest element of the set, we have:\n\n$3a +2(a +1) +a+2 < 999 \\iff a<166$\n\nThus, the set $\\{165.166, ..., 999\\}$, $k = 835$, satisfies the conditions of the utterance. Thus, $n \\ge 835$. A good start would be to see if $835$ is the solution or if $n> 835$ ...\n\ndivide the initial set into two subsets $A = \\{1,2, ..., 166\\}$ and $B = \\{167,168, ..., 999\\}$. Clearly, they are selected at least two elements of $A$, so suppose we have selected $n +2$ elements in $A$, which form a subset $A'= \\{x_1, x_2, ..., x_ {n +2}\\}$, and $833-n$ elements in $B$, forming the set $B'= \\{y_1, y_2, ..., y_ {833-n}\\}$, $n \\ge 0$. Are $x_1, y_1$ the smallest elements in $A, B$, respectively. Note that $x_1 \\le 167 - (n +2)$ and $167 + y_1 \\le n$. Thus, $x_p$ is any element in $A$ and different from $x_1$, which follows:\n\n\\[167 \\le y_1+2x_p+3x_1 \\le 167+n+2 \\cdot 166+3(167-(n+2))\\]\n\\[167 \\le y_1+2x_p+3x_1 \\le 994-2n\\]\n\n\nThus, the expression $ y_1+2x_p+3x_1$ is always an element in $B$.Mas note that $x_p$ can take $n +1$ values\u200b\u200b. And $B'$ has $833-n$ elements, at least one of these $n + 1$ is in $B'$! Therefore, $835$ is the smallest value of n with this property." } { "Tag": [ "trigonometry" ], "Problem": "[color=green]1)[/color]Show that a convex polygon is a pentagon if and only if the number of sides is equal to the number of diagonals.\r\n\r\n[color=green]2)[/color] Denote $ a$ and $ d$ the lenght of the side a the diagonal of a regular pentagon, show the relation:\r\n\r\n$ \\frac{d}{a} \\minus{} \\frac {a}{d} \\equal{} 1$\r\n\r\nThanks in advance", "Solution_1": "[hide]1) $ n\\equal{}\\frac{n(n\\minus{}3)}{2}$, so $ n\\equal{}5$. \n\n2) Sine theorem: $ d\\equal{}2a\\cos36^\\circ$\n\n$ d^2\\minus{}a^2\\equal{}ad$ is equivalent to $ 4\\cos^236^\\circ\\minus{}1\\equal{}2\\cos36^\\circ$. This can easily be proven.[/hide]", "Solution_2": "I dont think that is necessary to kill this nice problem with trigonometry. The proportion d/a-a/d=1 follows from being the side of a regular pentagon the golden section of its diagonal." } { "Tag": [ "geometry", "3D geometry", "tetrahedron", "counting", "distinguishability", "rotation", "geometric transformation" ], "Problem": "Each face of a regular tetrahedron is painted either red, white, or blue. Two colorings are considered indistinguishable if two congruent tetrahedra with those colorings can be rotated so that their appearances are identical. How many distinguishable colorings are possible?\r\n\r\nA.) 15 B.) 18 C.) 27 D.) 54 E.)81\r\n\r\n\r\nWhats the fastest/easiest way to solve this problem. The solution in the AMC 12 packet is kind of long.", "Solution_1": "The fastest way to solve this problem is probably with [b]Burnside's Lemma[/b] (see http://en.wikipedia.org/wiki/Burnside's_lemma ), although that isn't exactly elementary. The \"longer\" method is much more elementary and, as long as you are carefully organizing your work, just as good (for this problem).", "Solution_2": "Hopefully this is right\r\n\r\n[hide=\"Solution\"]\nOne color: 3 ways to color all four faces one color.\n\nTwo colors: WLOG let the colors be red and blue. We can have\n\n1 red 3 blue 2 red 2 blue 3 red 1 blue\n\nso only 3 possible ways. There are $ \\binom{3}{2}$ ways to choose two colors, so $ 3\\times 3\\equal{}9$ ways for two colors.\n\nAll Three colors: Clearly we must use the color pattern 1 1 2, so two colors are used once, and one color is used twice. There are 3 ways to pick the color that is used twice, so there are only 3 ways. The final total is\n$ 3\\plus{}9\\plus{}3\\equal{}15$, so A.[/hide]", "Solution_3": "t0rajir0u, that link doesn't work, it should be: http://en.wikipedia.org/wiki/Burnside%27s_lemma", "Solution_4": "What does wiki mean by \"one identity element fixing all 36 elements of X\"? Can someone explain burnside's lemma to me?", "Solution_5": "Burnside's Lemma deals with [url=http://en.wikipedia.org/wiki/Group_action]group actions[/url]. In this case, we are dealing with the group of rotations of a tetrahedron. The [url=http://en.wikipedia.org/wiki/Identity_element]identity element[/url] of this group is the rotation of $ 0^{\\circ}$ in every direction - in other words, it fixes every face. \r\n\r\nThe group-theoretic approach is a little heavy for this problem, and is largely unnecessary. I mention it only as a reasonably fast general technique for problems of this nature. It would be much easier if we wanted to discuss, say, colorings of an icosahedron.", "Solution_6": "but why dosen't this work : \r\n3 choices of paint for each side (including bottom side) so: 3*3*3*3. \r\ndivide this by 3, because you can rotate the tetrahedron three times and the colorings would still be in the same order. so the answer is 27.", "Solution_7": "There aren't exactly $ 3$ rotations corresponding to colorings that are the same. Count more carefully." } { "Tag": [ "geometry", "rectangle" ], "Problem": "One real number is written in each box of a rectangle table . It is allowed to change the sign of all the numbers in any row or column.Prove using these operations one can make the sum of the numbers in the table non-negative.", "Solution_1": "[quote]One real number is written in each box of a rectangle table . It is allowed to change the sign of all the numbers in any row or column.Prove using these operations one can make the sum of the numbers in the table non-negative.[/quote]\r\n\r\nIf the sum is negative, can't we just flip the sign of every number in the table?" } { "Tag": [ "algebra", "polynomial", "calculus", "derivative", "real analysis", "real analysis unsolved" ], "Problem": "A polynomial of deg n (n>3) has all its roots real and distinct. Assume them ordered. The AM of the first 2 and the AM of the last 2 cannot be roots of the derivative of the polynomial.\r\n\r\nPS: All of these were proposed for the 11'th grade.", "Solution_1": "Hey.. isn't this one, a problem of Titu Andreescu? Cause I remember it from somewhere... and I think he was the author! :D\r\n\r\n\r\nthank you grobber for posting them! :D\r\n\r\ncheers!", "Solution_2": "Yeah, it was proposed by Titu Andreescu.", "Solution_3": "Let $r_1, \\dots, r_n$ be the roots of the polynomial $f$. Then it is known that\r\n\\[\\frac{f'(x)}{f(x)} = \\sum_{k=1}^n \\frac{1}{x - r_k} .\\]\r\nFrom there, I think it is pretty quick to finish off the problem." } { "Tag": [ "Putnam", "algebra", "polynomial", "college contests" ], "Problem": "Find all real polynomials $ p(x)$ of degree $ n \\ge 2$ for which there exist real numbers $ r_1 < r_2 < ... < r_n$ such that\r\n\r\n(i) $ p(r_i) \\equal{} 0, 1 \\le i \\le n$, and\r\n\r\n(ii) $ p' \\left( \\frac {r_i \\plus{} r_{i \\plus{} 1}}{2} \\right) \\equal{} 0, 1 \\le i \\le n \\minus{} 1$.\r\n\r\n[b]Follow-up:[/b] In terms of $ n$, what is the maximum value of $ k$ for which $ k$ consecutive real roots of a polynomial $ p(x)$ of degree $ n$ can have this property? (By \"consecutive\" I mean we order the real roots of $ p(x)$ and ignore the complex roots.) In particular, is $ k \\equal{} n \\minus{} 1$ possible for $ n \\ge 3$?", "Solution_1": "in your follow-up you require the roots to be distinct?\r\nbecause even n could be reached with $ x^n$ :)", "Solution_2": "Yes; I forgot to add the condition that the roots be sorted.", "Solution_3": "n-1 can not be reached! $ \\frac{r_i\\plus{}r_{i\\plus{}1}}{2}$ can not be a root of $ p'$ if $ r_i,r_{i\\plus{}1}$ are the two smallest (biggest) roots of $ p$.", "Solution_4": "Find all real polynomials $ p(x)$ of degree $ n \\ge 2$ for which there exist real numbers $ r_1 < r_2 < ... < r_n$ such that\n\n(i) $ p(r_i) = 0, 1 \\le i \\le n$, and\n\n(ii) $ p' \\left( \\frac {r_i + r_{i + 1}}{2} \\right) = 0, 1 \\le i \\le n - 1$." } { "Tag": [ "group theory", "abstract algebra", "superior algebra", "superior algebra unsolved" ], "Problem": "a) Prove that a group of order $ 30$ cannot be simple.\r\nb) Prove that a group of order $ 42$ cannot be simple.\r\nc) Prove that a group of order $ 45$ cannot be simple.", "Solution_1": "b) Show that any group of order $ 42$ has exactly one $ 7$-Sylow subgroup. \r\nc) How many $ 3$-Sylow subgroups does a group of order $ 45$ have? - In fact, it can be shown that there are no simple groups of order $ p^{2} \\cdot q$.\r\n\r\n\r\n The idea underlying both proofs above is [b]simple[/b] to express: if G is a finite subgroup and H is the only subgroup of G of order |H|>1, then H is necessarily a normal subgroup of G. Do you know why?", "Solution_2": "[quote=\"coquitao\"]Do you why?[/quote] Why? :oops:", "Solution_3": "[quote=\"mr.starlight\"]a) Prove that a group of order $ 30$ cannot be simple.[/quote]\n\na) We wish to show that at leat one of the prime divisors of $ 30$ is such that $ n_{p} \\equal{} 1$. If it were not the case, we'd have $ n_{2} > 1, n_{3} > 1,$ and $ n_{5} > 1$... Can you derive a contradiction from the previous assumption now?\n\n[quote=\"mr.starlight\"][quote=\"coquitao\"]Do you know why?[/quote] Why? :oops:[/quote]\r\n\r\nFor any $ x \\in G$ it is known that $ xHx^{ \\minus{} 1}$ is a subgroup of G. Is it clear to you that $ |H| \\equal{} |xHx^{ \\minus{} 1}|$? SInce H is the only subgroup of G that has order |H|, it follows that $ H \\equal{} xHx^{ \\minus{} 1}$ for every $ x \\in G$ and whence the result.", "Solution_4": "[quote=\"coquitao\"]\n\na) We wish to show that at leat one of the prime divisors of $ 30$ is such that $ n_{p} \\equal{} 1$. If it were not the case, we'd have $ n_{2} > 1, n_{3} > 1,$ and $ n_{5} > 1$... Can you derive a contradiction from the previous assumption now?\n\n[/quote]\r\n\r\nIn that scenario we'd have $ 9$ three-Sylow subgroups and $ 6$ five-Sylow subgroups. Since the intersection of any two three-Sylow subgroups is trivial (being in a group or order 30), we have that there are $ 24$ elements of order $ 3$ in our group. By the same token, we can show that the group has at leat $ 8$ elements of order $ 5$, and since $ 24 \\plus{} 8 > 30$, we're done.", "Solution_5": "Thank you! :)" } { "Tag": [ "geometry unsolved", "geometry" ], "Problem": "In convex hexagon $ABCDEF$, triangles $ABC, CDE, EFA$ are similar.\r\n$\\angle BAC = \\angle DCE = \\angle FEA$ \r\n$\\angle BCA = \\angle DEC = \\angle FAE$. \r\nFor what condition of these three triangle, \r\nthat triangle $ACE$ is equilateral if and only if triangle $BDF$ is equilateral?", "Solution_1": "Can anybody find the condition? :maybe:", "Solution_2": "Well... nobody?" } { "Tag": [ "LaTeX" ], "Problem": "Hey. I need some help with two problems: \r\n\r\n1. (square root of 5x) + (square root of x+4)=8\r\n\r\n-i really got stuck after squaring a zillion times and then foiling, a huge pain\r\n\r\n2. \r\n\r\nsquare root of 4x+ square root of 16x+ square root of 64x+3/2 is equal to 2 (square root x) plus 1 (outside the square root)\r\n\r\nif anyone wants to post this using latex and i will reply if you are correct, feel free to. thanks a ton! :D :D", "Solution_1": "[quote=\"Yankzgodzilla55\"]Hey. I need some help with two problems: \n\n1. (square root of 5x) + (square root of x+4)=8\n\n-i really got stuck after squaring a zillion times and then foiling, a huge pain\n\n2. \n\nsquare root of 4x+ square root of 16x+ square root of 64x+3/2 is equal to 2 (square root x) plus 1 (outside the square root)\n\nif anyone wants to post this using latex and i will reply if you are correct, feel free to. thanks a ton! :D :D[/quote]\r\n1. $ \\sqrt{5x}\\plus{}\\sqrt{x\\plus{}4}\\equal{}8$\r\n2. $ \\sqrt{4x}\\plus{}\\sqrt{16x}\\plus{}\\sqrt{64x\\plus{}\\frac{3}{2}}\\equal{}2\\sqrt{x}\\plus{}1$", "Solution_2": "If anyone answers this correctly, I MIGHT give them a free cookie. :]", "Solution_3": "number 1 right \r\n\r\nbut number 2\r\n\r\nsquare root of 4x and everything goes together in one big problem\r\n\r\nlike the square root of 4x+ square root of 16x \r\n\r\nthis is all under one big square root thanks!", "Solution_4": "[hide=\"#1\"]$ \\sqrt {5x} \\equal{} 8 \\minus{} \\sqrt {x \\plus{} 4}$\n\n$ 5x \\equal{} 64 \\minus{} 16 \\sqrt {x \\plus{} 4} \\plus{} x \\plus{} 4$\n\n$ 68 \\minus{} 4x \\equal{} 16 \\sqrt {x \\plus{} 4}$\n\n$ 17 \\minus{} x \\equal{} 4 \\sqrt {x \\plus{} 4}$\n\n$ 289 \\minus{} 34x \\plus{} x^2 \\equal{} 16(x \\plus{} 4)$\n\n$ x^2 \\minus{} 50x \\plus{} 225 \\equal{} 0$\n\n$ (x \\minus{} 45)(x \\minus{} 5) \\equal{} 0$\n\n$ x \\equal{} 45 (extraneous)$ or $ \\boxed{x \\equal{} 5}$[/hide]\n\n[hide=\"#2\"]$ \\sqrt {4x} \\plus{} \\sqrt {16x} \\plus{} \\sqrt {64x} \\plus{} \\frac {3}{2} \\equal{} 2 \\sqrt {x} \\plus{} 1$\n\n$ 2\\sqrt {x} \\plus{} 4 \\sqrt {x} \\plus{} 8 \\sqrt {x} \\plus{} \\frac {3}{2} \\equal{} 2 \\sqrt {x} \\plus{} 1$\n\n$ 12 \\sqrt {x} \\equal{} \\minus{} \\frac {1}{2}$\n\n$ \\sqrt {x} \\equal{} \\minus{} \\frac {1}{24}$\n\n$ \\boxed{x \\equal{} \\frac {1}{576}}$[/hide]", "Solution_5": "[quote=\"modularmarc101\"][hide=\"#1\"]$ \\sqrt {5x} \\equal{} 8 \\minus{} \\sqrt {x \\plus{} 4}$\n\n$ 5x \\equal{} 64 \\minus{} 16 \\sqrt {x \\plus{} 4} \\plus{} x \\plus{} 4$\n\n$ 68 \\minus{} 4x \\equal{} 16 \\sqrt {x \\plus{} 4}$\n\n$ 17 \\minus{} x \\equal{} 4 \\sqrt {x \\plus{} 4}$\n\n$ 289 \\minus{} 34x \\plus{} x^2 \\equal{} 16(x \\plus{} 4)$\n\n$ x^2 \\minus{} 50x \\plus{} 225 \\equal{} 0$\n\n$ (x \\minus{} 45)(x \\minus{} 5) \\equal{} 0$\n\n$ \\boxed{x \\equal{} 45}$ or $ \\boxed{x \\equal{} 5}$[/hide]\n\n[hide=\"#2\"]$ \\sqrt {4x} \\plus{} \\sqrt {16x} \\plus{} \\sqrt {64x} \\plus{} \\frac {3}{2} \\equal{} 2 \\sqrt {x} \\plus{} 1$\n\n$ 2\\sqrt {x} \\plus{} 4 \\sqrt {x} \\plus{} 8 \\sqrt {x} \\plus{} \\frac {3}{2} \\equal{} 2 \\sqrt {x} \\plus{} 1$\n\n$ 12 \\sqrt {x} \\equal{} \\minus{} \\frac {1}{2}$\n\n$ \\sqrt {x} \\equal{} \\minus{} \\frac {1}{24}$\n\n$ \\boxed{x \\equal{} \\frac {1}{576}}$[/hide][/quote]\r\nFor #1 don't forget about extraneous solutions :wink:", "Solution_6": "Errr... Why is the one I had different from yours?\r\n\r\nFor one, I had: *4x + *16x + *64x+3/2 = 2*x + 1\r\n\r\nFor two, I had: *4x + *16x + *64x + *... + *4^n+3/2 = 2*x + 1\r\n\r\nLet * equal the radical symbol. And the first radical symbol of number one extends throughout the first half of the equation. All the radical symbols of number two extends throughout the first half of the equations. Sorry if this sounds extremely confusing.\r\n\r\nAnd btw, the cookie is still up for grabs. :3", "Solution_7": "[quote=\"Yankzgodzilla55\"]number 1 right \n\nbut number 2\n\nsquare root of 4x and everything goes together in one big problem\n\nlike the square root of 4x+ square root of 16x \n\nthis is all under one big square root thanks![/quote]\r\nSo for #2 it should be\r\n$ \\sqrt{4x\\plus{}\\sqrt{16x\\plus{}\\sqrt{64x\\plus{}\\frac{3}{2}}}}\\equal{}2\\sqrt{x}\\plus{}1$\r\nRight or no?", "Solution_8": "yup!!\r\nthats it", "Solution_9": "In that case, let $ y \\equal{} 4x$, then\r\n\r\n$ \\sqrt {y \\plus{} \\sqrt {4y \\plus{} \\sqrt {16y \\plus{} \\frac {3}{2}}}} \\equal{} \\sqrt {y} \\plus{} 1$\r\n\r\n$ y \\plus{} \\sqrt {4y \\plus{} \\sqrt {16y \\plus{} \\frac {3}{2}}} \\equal{} y \\plus{} 2 \\sqrt {y} \\plus{} 1$\r\n\r\n$ \\sqrt {4y \\plus{} \\sqrt {16y \\plus{} \\frac {3}{2}}} \\equal{} 2\\sqrt {y} \\plus{} 1$\r\n\r\n$ 4y \\plus{} \\sqrt {16y \\plus{} \\frac {3}{2}} \\equal{} 4y \\plus{} 4 \\sqrt {y} \\plus{} 1$\r\n\r\n$ \\sqrt {16y \\plus{} \\frac {3}{2}} \\equal{} 4 \\sqrt {y} \\plus{} 1$\r\n\r\n$ 16y \\plus{} \\frac {3}{2} \\equal{} 16y \\plus{} 8 \\sqrt {y} \\plus{} 1$\r\n\r\n$ \\frac {1}{2} \\equal{} 8 \\sqrt {y}$\r\n\r\n$ y \\equal{} \\frac {1}{256}$\r\n\r\n$ 4x \\equal{} \\frac {1}{256}$\r\n\r\n$ \\boxed{x \\equal{} \\frac {1}{1024}}$", "Solution_10": "Here's what I got. For the question. Nikhil stop being a butthead. :]\r\n\r\n[img]http://i263.photobucket.com/albums/ii133/Jcwo137/Mathproblem.jpg[/img]\r\n\r\nAnd it's okay to stare in awe at my epically awesome artistic math diagram.", "Solution_11": "now chang just added another question \r\n\r\n2. \r\n\r\ncan anyone solve that please. thanks. number 1 is already completed. thanks!", "Solution_12": "And trust me, you're REALLY going to want this cookie. :)", "Solution_13": "@modularmarc101: I got $ \\frac {1}{1024}$. I think your error was at the point where you got to \"$ \\frac {1}{2} \\equal{} 8\\sqrt {y}$\". Rather than squaring the expression, you square rooted it.", "Solution_14": "[quote=\"gaussintraining\"]@modularmarc101: I got $ \\frac {1}{1024}$. I think your error was at the point where you got to \"$ \\frac {1}{2} \\equal{} 8\\sqrt {y}$\". Rather than squaring the expression, you square rooted it.[/quote]\r\n\r\nI know, I fixed it like 5 minutes after finishing the solution, cuz I hadn't noticed. Look at it. Thanks anyways :)", "Solution_15": "[quote=\"iEatEmoKids\"]Here's what I got. For the question. Nikhil stop being a butthead. :]\n\n[img]http://i263.photobucket.com/albums/ii133/Jcwo137/Mathproblem.jpg[/img]\n\nAnd it's okay to stare in awe at my epically awesome artistic math diagram.[/quote]\r\n\r\nOkay, the thread starter, who is my friend, has #1 as a completely separate question. His #2 is the same as my #1, and my #2 is also a completely separate question, but we need to know it. Sorry for the confusion, and thanks in advance.\r\n\r\nPS: Cookie. :D", "Solution_16": "1. $ \\sqrt {5x} \\plus{} \\sqrt {x \\plus{} 4} \\equal{} 8$ \r\n$ \\sqrt {5x} \\equal{} 8 \\minus{} \\sqrt {x \\plus{} 4}$ \r\n\r\nSquare both sides; \r\n\r\n$ 5x \\equal{} 64 \\minus{} 16\\sqrt {x \\plus{} 4} \\plus{} x \\plus{} 4$ \r\n$ 4x \\minus{} 68 \\equal{} \\minus{} 16\\sqrt {x \\plus{} 4}$ \r\n$ 68 \\minus{} 4x \\equal{} 16\\sqrt {x \\plus{} 4}$\r\n$ 17 \\minus{} x \\equal{} 4\\sqrt {x \\plus{} 4}$ \r\n\r\nSquare both sides again; \r\n\r\n$ 289 \\minus{} 34x \\plus{} x^2 \\equal{} 16x \\plus{} 64$ \r\n$ x^2 \\minus{} 50x \\plus{} 225 \\equal{} 0$ \r\n$ (x \\minus{} 5)(x \\minus{} 45) \\equal{} 0$ \r\n$ x \\equal{} 5$ \r\nor \r\n$ x \\equal{} 45$\r\n\r\n\r\n2. $ \\sqrt {4x \\plus{} \\sqrt {16x \\plus{} \\sqrt {64x \\plus{} \\frac {3}{2}}}} \\equal{} 2\\sqrt {x} \\plus{} 1$\r\nSquare both sides; \r\n\r\n$ 4x \\plus{} \\sqrt {16x \\plus{} \\sqrt {64x \\plus{} \\frac {3}{2}}} \\equal{} 4x \\plus{} 4\\sqrt{x} \\plus{} 1$ \r\n$ \\sqrt {16x \\plus{} \\sqrt {64x \\plus{} \\frac {3}{2}}} \\equal{} 4\\sqrt{x} \\plus{} 1$ \r\n\r\nSquare both sides; \r\n\r\n$ 16x \\plus{} \\sqrt {64x \\plus{} \\frac {3}{2}} \\equal{} 16x \\plus{} 8\\sqrt{x} \\plus{} 1$\r\n$ \\sqrt {64x \\plus{} \\frac {3}{2}} \\equal{} 8\\sqrt{x} \\plus{} 1$\r\n\r\nSquare both sides; \r\n\r\n$ 64x \\plus{} \\frac{3}{2} \\equal{} 64x \\plus{} 16\\sqrt{x} \\plus{} 1$\r\n$ \\frac{3}{2} \\equal{} 16\\sqrt{x} \\plus{} 1$ \r\n$ \\frac{1}{2} \\equal{} 16\\sqrt{x}$ \r\n$ \\frac{1}{32} \\equal{} \\sqrt{x}$ \r\n\r\nSquare both sides;\r\n\r\n$ \\frac{1}{1024} \\equal{} x$", "Solution_17": "Yes, that's correct, but on #1, x=45 is an extraneous solution.", "Solution_18": "[quote=\"game.guy\"]\n\nSquare both sides; \n\n$ 64x \\plus{} \\frac {3}{2} \\equal{} 64x \\plus{} 16\\sqrt {x} \\plus{} 1$\n$ \\frac {3}{2} \\equal{} 16\\sqrt {x} \\plus{} 1$ \n$ \\frac {1}{2} \\equal{} 16\\sqrt {x}$ \n$ \\frac {1}{32} \\equal{} \\sqrt {x}$ \n\nSquare both sides;\n\n$ \\frac {1}{1024} \\equal{} x$[/quote]\r\n$ 64x \\plus{} \\frac {3}{2} \\equal{} 64x \\plus{} 16\\sqrt {x} \\plus{} 1$\r\n\r\n$ \\frac {3}{2} \\equal{} 16\\sqrt {x} \\plus{} 1$ \r\n\r\n$ \\frac {1}{2} \\equal{} 16\\sqrt {x}$ \r\n\r\n$ \\frac {1}{32} \\equal{} \\sqrt {x}$ \r\n\r\nSquare both sides\r\n\r\n=====================================================================\r\n\r\nWhen your fractions are running into each other (as here one of your denominators is touching \r\none of your numerators, and one of your numerators is touching a denominator), then at least\r\ninsert a horizontal space between affected lines, as I have suggested, for readability." } { "Tag": [ "integration" ], "Problem": "\u0388\u03c3\u03c4\u03c9 $ p>=1$ \u03ba\u03b1\u03b9 $ f,g: [0,1]\\longrightarrow R$ \u03bf\u03bb\u03bf\u03ba\u03bb\u03b7\u03c1\u03ce\u03c3\u03b9\u03bc\u03b5\u03c2 \u03c3\u03c5\u03bd\u03b1\u03c1\u03c4\u03ae\u03c3\u03b5\u03b9\u03c2 . \u039d\u03b1 \u03b1\u03c0\u03bf\u03b4\u03b5\u03af\u03be\u03b5\u03c4\u03b5 \u03cc\u03c4\u03b9 $ (\\int^{1}_{0}\\left|f+g\\right|^{p})^{\\frac{1}{p}}\\leq(\\int^{1}_{0}\\left|f\\right|^{p})^{\\frac{1}{p}}+(\\int^{1}_{0}\\left|g\\right|^{p})^{\\frac{1}{p}}$", "Solution_1": "Ummm, \u03b1\u03c5\u03c4\u03ae \u03b5\u03af\u03bd\u03b1\u03b9 \u03b1\u03c0\u03bb\u03ce\u03c2 \u03b7 \u03b1\u03bd\u03b9\u03c3\u03cc\u03c4\u03b7\u03c4\u03b1 Minkowski \u03b3\u03b9\u03b1 \u03c4\u03bf \u03b4\u03b9\u03ac\u03c3\u03c4\u03b7\u03bc\u03b1 $ [0,1]$ \u03ba\u03b1\u03b9 \u03c0\u03c1\u03bf\u03ba\u03cd\u03c0\u03c4\u03b5\u03b9 \u03b1\u03c0\u03bf \u03bc\u03af\u03b1 \u03b5\u03c6\u03b1\u03c1\u03bc\u03bf\u03b3\u03ae \u03c4\u03b7\u03c2 \u03c4\u03c1\u03b9\u03b3\u03c9\u03bd\u03b9\u03ba\u03ae\u03c2 \u03b1\u03bd\u03b9\u03c3\u03cc\u03c4\u03b7\u03c4\u03b1\u03c2 \u03ba\u03b1\u03b9 \u03bc\u03b5\u03c4\u03ac Holder. A\u03bd \u03b1\u03c0\u03bb\u03ce\u03c2 \u03c8\u03ac\u03c7\u03bd\u03b5\u03b9\u03c2 \u03b3\u03b9\u03b1 \u03c4\u03b7\u03bd \u03b1\u03c0\u03cc\u03b4\u03b5\u03b9\u03be\u03ae \u03c4\u03b7\u03c2, \u03c0\u03b9\u03c3\u03c4\u03ad\u03c5\u03c9 \u03cc\u03c4\u03b9 \u03bc\u03ad\u03c7\u03c1\u03b9 \u03ba\u03b1\u03b9 \u03b7 Wikipedia \u03b8\u03b1 \u03c4\u03b7\u03bd \u03ad\u03c7\u03b5\u03b9 :) \u0391\u03bb\u03bb\u03b9\u03ce\u03c2, \u03c0\u03b5\u03c2 \u03bc\u03bf\u03c5 \u03bd\u03b1 \u03b3\u03c1\u03ac\u03c8\u03c9 \u03ba\u03b1\u03bd\u03bf\u03bd\u03b9\u03ba\u03ac \u03c4\u03b7 \u03bb\u03cd\u03c3\u03b7...\r\n\r\n\u03a0\u03ac\u03bd\u03c4\u03c9\u03c2, \u03b1\u03c5\u03c4\u03ae \u03b5\u03af\u03bd\u03b1\u03b9 \u03b1\u03c0\u03af\u03c3\u03c4\u03b5\u03c5\u03c4\u03b1 \u03c3\u03b7\u03bc\u03b1\u03bd\u03c4\u03b9\u03ba\u03ae \u03ba\u03b1\u03b9 \u03c7\u03c1\u03ae\u03c3\u03b9\u03bc\u03b7 \u03b1\u03bd\u03b9\u03c3\u03cc\u03c4\u03b7\u03c4\u03b1 \u03b3\u03b9\u03b1 \u03cc\u03c3\u03bf\u03c5\u03c2 \u03ba\u03ac\u03bd\u03bf\u03c5\u03bd \u03b1\u03bd\u03ac\u03bb\u03c5\u03c3\u03b7 \u03b1\u03c6\u03bf\u03cd \u03b5\u03af\u03bd\u03b1\u03b9 \u03b1\u03c0\u03bb\u03ce\u03c2 \u03b7 \u03c4\u03c1\u03b9\u03b3\u03c9\u03bd\u03b9\u03ba\u03ae \u03b1\u03bd\u03b9\u03c3\u03cc\u03c4\u03b7\u03c4\u03b1 \u03b3\u03b9\u03b1 \u03c4\u03b7\u03bd $ L^p$ \u03bd\u03cc\u03c1\u03bc\u03b1... \u03ba\u03b1\u03bb\u03ac \u03ad\u03ba\u03b1\u03bd\u03b5\u03c2 \u03ba\u03b1\u03b9 \u03c4\u03b7\u03bd \u03b1\u03bd\u03ad\u03b2\u03b1\u03c3\u03b5\u03c2!\r\n\r\nCheerio,\r\n\r\nDurandal 1707", "Solution_2": "[quote=\"\u0393\u03b9\u03ce\u03c1\u03b3\u03bf\u03c2\"]\n\u0395\u03b1\u03bd \u03b3\u03af\u03bd\u03b5\u03c4\u03b1\u03b9 \u03b3\u03c1\u03ac\u03c8\u03b5 \u03ba\u03b1\u03bd\u03bf\u03bd\u03b9\u03ba\u03ac \u03c4\u03b7 \u03bb\u03cd\u03c3\u03b7 \u03c4\u03b7\u03c2....\n[/quote]\r\n\r\nDum dum dum, sure, \u03b7 \u03bb\u03cd\u03c3\u03b7 \u03c3\u03b5 spoiler, \u03b3\u03b9\u03b1 \u03bd\u03b1 \u03c4\u03b7 \u03b4\u03bf\u03ba\u03b9\u03bc\u03ac\u03c3\u03bf\u03c5\u03bd \u03ba\u03b1\u03b9 \u03cc\u03c3\u03bf\u03b9 \u03b4\u03b5\u03bd \u03c4\u03b7\u03bd \u03be\u03ad\u03c1\u03bf\u03c5\u03bd:\r\n[hide=\"Proof\"]\n\u039a\u03b1\u03c4'\u03b1\u03c1\u03c7\u03ac\u03c2, \u03b1\u03c2 \u03b1\u03b3\u03bd\u03bf\u03ae\u03c3\u03bf\u03c5\u03bc\u03b5 \u03c4\u03b9\u03c2 \u03c0\u03c1\u03bf\u03c6\u03b1\u03bd\u03b5\u03af\u03c2 \u03c0\u03b5\u03c1\u03b9\u03c0\u03c4\u03ce\u03c3\u03b5\u03b9\u03c2 $ p \\equal{} 1$ \u03ae $ \\int |f \\plus{} g|^p \\equal{} 0$. \u0395\u03c0\u03af\u03c3\u03b7\u03c2, \u03b3\u03b9\u03b1 \u03bd\u03b1 \u03bc\u03b7 \u03b3\u03c1\u03ac\u03c6\u03bf\u03c5\u03bc\u03b5 \u03c0\u03ac\u03c1\u03b1 \u03c0\u03bf\u03bb\u03bb\u03ac, \u03b1\u03c2 \u03b8\u03ad\u03c3\u03bf\u03c5\u03bc\u03b5 $ \\|f\\|_p \\equal{} \\left(\\int|f|^p\\right)^\\frac {1}{p}$, \u03cc\u03c0\u03bf\u03c5 $ f$ \u03bc\u03af\u03b1 \u03bf\u03bb\u03bf\u03ba\u03bb\u03b7\u03c1\u03ce\u03c3\u03b9\u03bc\u03b7 \u03c3\u03c5\u03bd\u03ac\u03c1\u03c4\u03b7\u03c3\u03b7. \u0399\u03c3\u03bf\u03b4\u03cd\u03bd\u03b1\u03bc\u03b1 \u03bb\u03bf\u03b9\u03c0\u03cc\u03bd, \u03b8\u03ad\u03bb\u03bf\u03c5\u03bc\u03b5 \u03bd\u03b1 \u03b4\u03b5\u03af\u03be\u03bf\u03c5\u03bc\u03b5 \u03cc\u03c4\u03b9 \u03b7 $ \\|\\cdot\\|_p$ \u03b9\u03ba\u03b1\u03bd\u03bf\u03c0\u03bf\u03b9\u03b5\u03af \u03c4\u03b7\u03bd \u03c4\u03c1\u03b9\u03b3\u03c9\u03bd\u03b9\u03ba\u03ae \u03b1\u03bd\u03b9\u03c3\u03cc\u03c4\u03b7\u03c4\u03b1 \u03b3\u03b9\u03b1 $ p\\geq 1$: $ \\|f \\plus{} g\\|_p \\leq \\|f\\|_p \\plus{} \\|g\\|_p$.\n\n\u0391\u03c0\u03cc \u03c4\u03b7\u03bd \u03ba\u03bb\u03b1\u03c3\u03c3\u03b9\u03ba\u03ae \u03c4\u03c1\u03b9\u03b3\u03c9\u03bd\u03b9\u03ba\u03ae \u03b1\u03bd\u03b9\u03c3\u03cc\u03c4\u03b7\u03c4\u03b1 \u03ad\u03c7\u03bf\u03c5\u03bc\u03b5: $ |f \\plus{} g|^p \\leq \\left(|f| \\plus{} |g|\\right)|f \\plus{} g|^{p \\minus{} 1}$.\n\u0388\u03c4\u03c3\u03b9, \u03c3\u03b5 \u03c3\u03c5\u03bd\u03b4\u03c5\u03b1\u03c3\u03bc\u03cc \u03bc\u03b5 \u03c4\u03b7\u03bd \u03bf\u03bb\u03bf\u03ba\u03bb\u03b7\u03c1\u03c9\u03c4\u03b9\u03ba\u03ae \u03bc\u03bf\u03c1\u03c6\u03ae \u03c4\u03b7\u03c2 H\u00f6lder \u03b3\u03b9\u03b1 $ q \\equal{} \\frac {p}{p \\minus{} 1}\\implies q(p \\minus{} 1) \\equal{} p$:\n\\[ \\int|f \\plus{} g|^p \\leq \\int(|f| \\plus{} |g|)|f \\plus{} g|^{p \\minus{} 1}\\implies \\\\\n\\int|f \\plus{} g|^p \\leq \\|f\\|_p \\cdot \\| |f \\plus{} g|^{p \\minus{} 1}\\|_q \\plus{} \\|g\\|_p \\cdot \\| |f \\plus{} g|^{p \\minus{} 1} \\|_q \\equal{} \\\\\n\\equal{} \\left(\\|f\\|_p \\plus{} \\|g\\|_p\\right) \\left(\\int|f \\plus{} g|^p\\right)^\\frac {1}{q} \\equal{} \\left(\\|f\\|_p \\plus{} \\|g\\|_p\\right) \\cdot\\frac {\\int |f \\plus{} g|^p}{\\|f \\plus{} g\\|_p}\n\\]\n\u03b1\u03c0'\u03cc\u03c0\u03bf\u03c5 \u03ba\u03b1\u03b9 \u03c0\u03c1\u03bf\u03ba\u03cd\u03c0\u03c4\u03b5\u03b9 \u03c4\u03bf \u03b6\u03b7\u03c4\u03bf\u03cd\u03bc\u03b5\u03bd\u03bf.\n[/hide]\r\n\r\nCheerio,\r\n\r\nDurandal 1707" } { "Tag": [ "algebra proposed", "algebra" ], "Problem": "Prove that: With $ F_n$ is Fibonacci number\r\n$ \\sum_{n \\equal{} 2}^{\\infty}\\frac {1}{F_{n \\minus{} 1}.F_{n \\plus{} 1}} \\equal{} 1,\\sum_{n \\equal{} 2}^{\\infty}\\frac {F_n}{F_{n \\minus{} 1}.F_{n \\plus{} 1}} \\equal{} 2$", "Solution_1": "The first sum telescopes:\r\n\r\n$ \\sum_{n\\equal{}2}^{\\infty} \\frac{1}{F_{n\\minus{}1} F_{n\\plus{}1}} \\equal{} \\sum_{n\\equal{}2}^{\\infty} \\frac{1}{F_{n\\plus{}1} \\minus{} F_{n\\minus{}1}} \\left( \\frac{1}{F_{n\\minus{}1}} \\minus{} \\frac{1}{F_{n\\plus{}1}} \\right) \\equal{} \\sum_{n\\equal{}2}^{\\infty} \\frac{1}{F_n F_{n\\minus{}1}} \\minus{} \\frac{1}{F_{n\\plus{}1} F_n} \\equal{} \\frac{1}{F_1 F_2} \\equal{} 1$\r\n\r\nas does the second: since each term is $ F_n$ times the terms of the above, that sum is\r\n\r\n$ \\sum_{n\\equal{}2}^{\\infty} \\frac{1}{F_{n\\minus{}1}} \\minus{} \\frac{1}{F_{n\\plus{}1}} \\equal{} \\frac{1}{F_1} \\plus{} \\frac{1}{F_2} \\equal{} 2$", "Solution_2": "[quote=\"tdl\"]Prove that: With $ F_n$ is Fibonacci number\n$ \\sum_{n \\equal{} 2}^{\\infty}\\frac {1}{F_{n \\minus{} 1}.F_{n \\plus{} 1}} \\equal{} 1,\\sum_{n \\equal{} 2}^{\\infty}\\frac {F_n}{F_{n \\minus{} 1}.F_{n \\plus{} 1}} \\equal{} 2$[/quote]\r\n\r\n[hide]\nNote that $ F_{n\\plus{}1} \\equal{} F_n \\plus{} F_{n\\minus{}1}$ implies\n\n$ 1 \\equal{} \\frac{F_{n\\plus{}1}}{F_n} \\minus{} \\frac{F_{n\\minus{}1}}{F_n}$,\n\nwhere upon dividing both sides by $ F_{n\\minus{}1}F_{n\\plus{}1}$ yields\n\n$ \\frac{1}{F_{n\\minus{}1}F_{n\\plus{}1}} \\equal{} \\frac{1}{F_n F_{n\\minus{}1}} \\minus{} \\frac{1}{F_n F_{n\\plus{}1}}$.\n\nTherefore for any positive integer $ N \\ge 2$,\n\n$ \\sum_{n\\equal{}2}^N \\frac{1}{F_{n\\minus{}1} F_{n\\plus{}1}} \\equal{} \\frac{1}{F_2 F_1} \\minus{} \\frac{1}{F_N F_{N\\plus{}1}}$,\n\nand taking the limit as $ N \\rightarrow \\infty$ gives 1, as claimed.\n\nSimilarly, we find\n\n$ \\frac{F_n}{F_{n\\minus{}1} F_{n\\plus{}1}} \\equal{} \\frac{1}{F_{n\\minus{}1}} \\minus{} \\frac{1}{F_{n\\plus{}1}}$,\n\nfrom which we obtain\n\n$ \\sum_{n\\equal{}2}^N \\frac{F_n}{F_{n\\minus{}1} F_{n\\plus{}1}} \\equal{} \\frac{1}{F_1} \\plus{} \\frac{1}{F_2} \\minus{} \\frac{1}{F_N} \\minus{} \\frac{1}{F_{N\\plus{}1}}$.\n\nAgain, taking the limit as $ N \\rightarrow \\infty$, we find the second sum evaluates to 2.\n[/hide]" } { "Tag": [ "email" ], "Problem": "All competitors please IM me on AIM. My screen name is Mr Chen 484.\r\n\r\n\r\nMeeting will be at 3 PM Eastern Time", "Solution_1": "You're not there :maybe: I am waiting from 2:45-3:00 :maybe:", "Solution_2": "not enough people. I think I might postpone it. Anyone have any ideas?", "Solution_3": "Maybe next weekend?\r\n\r\nSeriously...two of the questions out of 4 were like waaay past mc and I will say that they're minimum target by all menas", "Solution_4": "[quote=\"anirudh\"]Maybe next weekend?\n\nSeriously...two of the questions out of 4 were like waaay past mc and I will say that they're minimum target by all menas[/quote]\r\nHave you ever seen a national target? I mean an official one, not just the first 25 that are distributed, I mean all of the questions. Some of those final questions could be in the last ten of a state sprint easily.", "Solution_5": "Now that I think about it, and because of requests, it probably should be a Saturday 3-hour session. Anyone have any suggestions? Afternoon, morning, etc?", "Solution_6": "AFternoon please before 4:00. Maybe 12-3?", "Solution_7": "aren't people eating lunch?", "Solution_8": "hmmm. maybe...how about 1-2:30?", "Solution_9": "that's even worse from my perspective :P\r\n\r\n-jorian", "Solution_10": "Okay Here are the competitors so far:\r\n\r\nBpms\r\nAnirudh\r\nSplashD\r\nJhredsox \r\nRagnarok23\r\nEliross2\r\nredcomet\r\ndavid li", "Solution_11": "We should have a 3 hr session as Now a ranger said, when we do round robin.", "Solution_12": "can i join in?", "Solution_13": "Wait, can I be a competitor? I think I IMed you.", "Solution_14": "To bpms.\r\n\r\nYou mean countdown right :P?\r\n\r\nYeah, or moreso, have you tried to compete with the people on TV at nationals (or been there for that matter). Unless you can easily beat them, you will not have a chance in the increased pressure of nationals.", "Solution_15": "[quote=\"now a ranger\"]Okay Here are the competitors so far:\n\nBpms\nAnirudh\nSplashD\nJhredsox \nRagnarok23\nEliross2\nredcomet\ndavid li[/quote]\r\n\r\njw, but when did i sign up? :| \r\n-jorian", "Solution_16": "I'd be up for it -- timing is a bit whack as of right now though.", "Solution_17": "[quote=\"kyyuanmathcount\"]To bpms.\n\nYou mean countdown right :P?\n\nYeah, or moreso, have you tried to compete with the people on TV at nationals (or been there for that matter). Unless you can easily beat them, you will not have a chance in the increased pressure of nationals.[/quote]\r\nYes I meant countdown :rlx:\r\nAlso, I agree with what you said unless you are like me and do better under a ton of pressure.", "Solution_18": "can i join?", "Solution_19": "Anyone who wants to CD today, I'm online. Please IM me. Within an hour of this post or less.", "Solution_20": "Is it possible to do something like this by email? Because I have an aim, but... long story short, its not working and its not going to any time soon, and I have no way of getting a new one (really long story :roll: ). But I would really like to do this. :)", "Solution_21": "Use AIM Express :lol: It works fine that way. When is this going to be?", "Solution_22": "[quote=\"anirudh\"]Use AIM Express :lol: It works fine that way. When is this going to be?[/quote]\r\n\r\nDon't use AIM Express. I had to use it on the last countdown, and it went so slow. The message gets to you about a second later, and it takes about a second for your messages to show up.", "Solution_23": "No, see, I had that, but then I got linux wich has gaim. Then my dad disconnected it , but the normal aim doesn't work because it's not supported by my browser. :(" } { "Tag": [ "geometry", "topology", "advanced fields", "advanced fields unsolved" ], "Problem": "Let $ X$ be a topological space and $ Y$ a subset of $ X$. Then $ \\dim Y \\leq \\dim X$ (we use the Krull dimension).\r\nI saw a proof of this statement:\r\nLet $ Y_1 \\subsetneq Y_2$ be two irreducible closed subsets of $ Y$. Let $ X_1,X_2$ be their respective closures in $ X$. Then $ X_i$ are irreducible and $ X_1 \\subsetneq X_2$. It follows that $ \\dim Y \\leq \\dim X$.\r\n\r\nSee (Algebraic Geometry and Arithmetic Curves By Qing Liu, Reinie Ern\u00e9, p 68)\r\n\r\nNow I have questions about the proof. My main question is:Why should the closures of $ X_i$ be irreducible?\r\nWhy is it enough to prove just for a chain with 2 irreducible closed sets? The dimension can be infinitely large! \r\n\r\nTo give a proof I started with writing $ Y_i\\equal{}Y \\cap W_i$ with $ W_i$ a closed set in $ X$ for $ i\\equal{}1,2$. You can see that $ W_1 \\subsetneq W_2$ but I can not see why $ W_i$ is irreducible!", "Solution_1": "Claim 1: If $ Y_1 \\subset X$ is irreducible, then $ \\bar{Y_1}$ is irreducible.\r\nProof: Suppose not, we can write $ \\bar{Y_1} \\equal{} Z_1 \\cup Z_2$, where $ Z_1,Z_2$ are closed in $ \\bar{Y_1}$, and they are not the whole $ \\bar{Y_1}$. Then $ Y_1 \\equal{} (Y_1 \\cap Z_1) \\cup (Y_1 \\cap Z_2)$. Irreducibility of $ Y_1$ implies that $ Y_1 \\cap Z_1 \\equal{} Y_1$ or $ Y_1 \\cap Z_2 \\equal{} Y_1$. Suppose the first case is true (the other is similar), then $ Y_1 \\subset Z_1 \\Rightarrow \\bar{Y_1} \\equal{} Z_1$ (because $ Z_1$ is closed in $ \\bar{Y_1}$). This shows that $ Z_1 \\equal{} \\bar{Y_1}$, contradiction.\r\n\r\nFor your other question, note that whatever chain of irreducible closed sets $ Y_1 \\subset Y_2 \\subset ...\\subset Y_n$ you have in $ Y$, taking closure gives you a corresponding chain $ X_1 \\subset X_2 \\subset ... \\subset X_n$ in $ X$, thus the supremum of length of chains in $ Y$ must be smaller or equal to that of $ X$.", "Solution_2": "if $ X_i$ is the closure of $ Y_i$ in $ X$, then $ X_i \\cap Y$ is the closure of $ Y_i$ in $ Y$, that is $ Y_i$. thus $ Y_i \\equal{} Y \\cap X_i$ and from this you see that $ X_1 \\neq X_2$." } { "Tag": [], "Problem": "Solve for n:\r\n\r\n$ \\frac{3}{2}\\binom{n}{3} \\equal{} \\binom{n\\plus{}1}{2}$\r\n\r\nI expanded into the equation and simplified up to $ \\frac{n^2\\minus{}3n\\plus{}2}{n\\plus{}1} \\equal{} 0$, but how do I solve for n?\r\n\r\nThanks.", "Solution_1": "[hide=\"solution\"]\nI belive the answer is $ 5$.\n\\begin{eqnarray*} \\dfrac{3}{2}\\cdot \\binom{n}{3} & = & \\binom{n + 1}{2} \\\\\n\\dfrac{3}{2}\\cdot \\dfrac{(n)(n - 1)(n - 2)}{3\\cdot 2} & = & \\dfrac{(n + 1)\\cdot (n)}{2} \\\\\n\\dfrac{(n)(n - 1)(n - 2)}{4} & = & \\dfrac{(n + 1)\\cdot n}{2} \\\\\n\\dfrac{(n)(n - 1)(n - 2)}{2} & = & (n + 1)\\cdot n \\\\\n(n)(n - 1)(n - 2) & = & 2\\cdot(n + 1)\\cdot n \\\\\nn^3 - 3n^2 + 2n & = & 2n^2 + 2n \\\\\nn^3 - 5n^2 & = & 0 \\\\\nn^2(n - 5) & = & 0 \\\\\nn & = & 0 \\\\\nn & = & 5 \\end{eqnarray*}\n\nEdit: answers are $ 0,5$\n\n\nAlso if you plug in $ 0$ and $ 5$ it works.\n[/hide]", "Solution_2": "Hmm didn't realize you could subtract, I cancelled out too much.\r\n\r\nThanks.", "Solution_3": "Anymore combination problems..that are a bit harder?\r\n\r\nThanks a lot", "Solution_4": "Actually, I guess you could argue that 0 works, too. There are 0 ways to choose 3 objects when there are not objects, and no ways to choose 2 objects out of only one object.", "Solution_5": "True, so most likely $ 0$ could also be an answer.\r\n\r\nEdit: for fryeggs\r\n\r\nHere are two problems:\r\n\r\n1. Find the value of the sum $ \\binom{n}{0}\\plus{}2\\binom{n}{1}\\plus{}2^2\\binom{n}{2}\\plus{}\\cdots \\plus{}2^n\\binom{n}{n}$\r\n\r\n2. Find the value of the sum $ \\binom{n}{0}\\minus{}\\binom{n}{1}\\plus{}\\binom{n}{2}\\plus{}\\cdots \\plus{}(\\minus{}1)^n\\binom{n}{n}$\r\n\r\n[hide=\"Answers\"]\nThe first one is just $ (1\\plus{}2)^n\\equal{}\\binom{n}{0}\\plus{}2\\binom{n}{1}\\plus{}2^2\\binom{n}{2}\\plus{}\\cdots \\plus{}2^n\\binom{n}{n}\\equal{}3^n$\n\nThe second one is $ (1\\minus{}1)^n\\equal{}\\binom{n}{0}\\minus{}\\binom{n}{1}\\plus{}\\binom{n}{2}\\plus{}\\cdots \\plus{}(\\minus{}1)^n\\binom{n}{n}\\equal{}0$\n[/hide]", "Solution_6": "Problem:\r\nFind the number of zeros at the end of $ \\binom{54}{13}$" } { "Tag": [ "conics", "ellipse" ], "Problem": "Does anyone know what scores the winners or top 10-12 got in previous years? After the format was changed.\r\n\r\nJust curious to see if the score has fluctuated a lot or if earlier/later years seemed to have higher/lower scores.", "Solution_1": "They stopped giving us score information the last couple years, but here is what I have from before that for 1st - 12th written:\r\n\r\n1999 45 - 36\r\n2000 42 - 35\r\n2001 42 - 35\r\n2002 43 - 38\r\n2003 45 - 40", "Solution_2": "[quote=\"frost13\"]They stopped giving us score information the last couple years, but here is what I have from before that for 1st - 12th written:\n\n1999 45 - 36\n2000 42 - 35\n2001 42 - 35\n2002 43 - 38\n2003 45 - 40[/quote]\r\nI thought they gave the bar graphs this year...", "Solution_3": "Thanks frost,\r\n\r\ndoes anyone have anything like around 1993-1994?", "Solution_4": "What was it for 06?", "Solution_5": "it was 42 (neal) to 36 (i believe, but it might of been a tiebreaker for a few 35's)\r\n\r\nwasn't there a perfect in '04\r\n\r\nmy coach knows almost nothing about mc (well..no one really does), but he kept on obsessing about it :D\r\n\r\n-jorian", "Solution_6": "1996: 33-38\r\n2004: 44 was first, 11th had a 37\r\n2005: 30 (I think)-39", "Solution_7": "[quote=\"jhredsox\"]it was 42 (neal) to 36 (i believe, but it might of been a tiebreaker for a few 35's)\n\nwasn't there a perfect in '04\n\nmy coach knows almost nothing about mc (well..no one really does), but he kept on obsessing about it :D\n\n-jorian[/quote]\r\n\r\nNo, Arjun had a 36...I had the highest score of 35 and got 14th Place...", "Solution_8": "How many times have you complained about that?", "Solution_9": "[quote=\"perfect628\"]How many times have you complained about that?[/quote]\r\nDude, he was correcting jhredsox with uncontradictable proof.", "Solution_10": "Ah, but consider the ...\r\nIt seems to me that he combined that with a complaint. Of course, it is rather tough to sense tone with typing...", "Solution_11": "Umm...I wasn't complaining...I was simplying correcting someone...\r\n\r\nI always have ellipses at the end of my sentence... well almost always.", "Solution_12": "[quote=\"bpms\"]\n2005: 30 (I think)-39[/quote]\r\n\r\nNo, Noah Arbesfeld was 14th with a 31, the cutoff was 31." } { "Tag": [ "calculus", "integration", "logarithms", "calculus computations" ], "Problem": "Can anyone demostrate\r\n\r\n$\\int \\frac{x^{2}dx}{\\sqrt{ax^{6}+bx+c}}\\, = \\, \\frac{1}{3 \\sqrt{a}}\\ln \\left| x^{3}\\sqrt{a}+\\sqrt{ax^{6}+bx+c}\\right|+C$\r\n\r\ndirectly, without deriving. Thank you all.", "Solution_1": "It's false. The integral you propose almost certainly has no elementary antiderivative in general.\r\n\r\nIf that were $bx^{3}$ instead, it's a routine exercise. The answer you gave is still wrong (so differentating it won't work), but the integral is doable.", "Solution_2": "[quote=\"jmerry\"]It's false. The integral you propose almost certainly has no elementary antiderivative in general.\n\nIf that were $bx^{3}$ instead, it's a routine exercise. The answer you gave is still wrong (so differentating it won't work), but the integral is doable.[/quote]\r\n\r\nI coudn\u00b4t solve it, then I went to Mathmatica 5.2 and it offerd this result\r\n\r\n[url=http://imageshack.us][img]http://img305.imageshack.us/img305/5702/intpq8.png[/img][/url]", "Solution_3": "Mathematica is interpreting \"$bx$\" as a single object, not $b\\cdot x$. Since we're not integrating with respect to $bx$, it is treated as a constant, just like $c$.", "Solution_4": "[quote=\"jmerry\"]Mathematica is interpreting \"$bx$\" as a single object, not $b\\cdot x$. Since we're not integrating with respect to $bx$, it is treated as a constant, just like $c$.[/quote]\r\n\r\nYes, tks" } { "Tag": [ "number theory", "prime factorization" ], "Problem": "One of the problems in Volume 1: the Basics states that the prime factorization of 36000 is 2^5*3^2*5^3, but isn't it 2^4*3^2*5^2?\r\n\r\nBecause of this, I got a different answer than the book! Ugh! Please try and solve this math problem:\r\n3 radical 36000/243 (the 3 in the beginning is an exponent)\r\n\r\n :yoda: [/b]", "Solution_1": "$ 2^5 \\times 3^2 \\times 5^3$ [i]is[/i] the correct prime factorization. Multiply it out on a calculator.", "Solution_2": "[quote=\"JokoMoko\"]One of the problems in Volume 1: the Basics states that the prime factorization of 36000 is 2^5*3^2*5^3, but isn't it 2^4*3^2*5^2?\n\nBecause of this, I got a different answer than the book! Ugh! Please try and solve this math problem:\n3 radical 36000/243 (the 3 in the beginning is an exponent)\n\n :yoda: [/b][/quote]\r\n\r\nDo you mean symplify the radical?", "Solution_3": "Yes i put it into my calculator and the book is correct about the prime factorization so the problem:\r\n\r\n${ \\frac{\\sqrt[3]{36000}}{243}}$\r\n\r\n${ \\frac{\\sqrt[3]{2^5*3^2*5^3}}{243}}$\r\n\r\n${ \\frac{2\\sqrt[3]{2^2*3^2*5^3}}{243}}$\r\n\r\n${ \\frac{2*5\\sqrt[3]{2^2*3^2}}{243}}$\r\n\r\n${ \\frac{10\\sqrt[3]{36}}{243}}$\r\n\r\nI don't think that you can simplify it any more...", "Solution_4": "yeah, it can't be.", "Solution_5": "[quote=\"JokoMoko\"]One of the problems in Volume 1: the Basics states that the prime factorization of 36000 is 2^5*3^2*5^3, but isn't it 2^4*3^2*5^2?\n[/quote]\r\nHow can there be three 0s on the end if it has only two factors of 5?", "Solution_6": "He already figured out his mistake, if you look at the previous posts." } { "Tag": [ "function", "inequalities", "number theory", "relatively prime", "number theory unsolved" ], "Problem": "Q1>\r\nf:N\u2192N\r\nf is strictly increasing\r\nf(mn)=f(m)f(n)\r\nIf m\u2260n & m^n=n^m, then f(m)=n or f(n)=m\r\n\r\nf(12)?\r\n\r\nQ2> \r\nf:N\u2192N\r\nf(f(m)+f(n))=m+n\r\n\r\nshow that f(n)=n\r\n\r\nQ3>\r\nf:N\u2192N\r\nf is strictly increasing\r\nf(7)=7\r\nIf p,q are relatively prime,then f(pq)=f(p)f(q)\r\n\r\nshow that f(n)=n", "Solution_1": "Q_2, $f(m)=f(n)\\Longrightarrow 2m=m+m=f(f(m)+f(m))=f(f(n)+f(n))=2n\\Longrightarrow m=n$, so $f$ is injective. \r\n\r\n$f(f(n)+f(n+2))=n+(n+2)=(n+1)+(n+1)=f(2f(n+1))\\forall n\\in\\mathbb{N}\\Longrightarrow f(n+2)-f(n+1)=f(n+1)-f(n)$. Now use telescopic sum!", "Solution_2": "I think that Q1 doesn't have any sense, if $m>n$ and $m^{n}=n^{m}$\r\nthen $f(m)=n This is the ugliest solution that I have for now.\r\n\r\nIf we know that $f(n)=n$ and $f(n+k)=n+k$ then it stands that\r\n$f(m)=m$ for all $n14$ so $f(n)=n$ for $n<2p$.\r\nNow there is some prime between $p$ and $2p$ let's say $q$, so\r\n$f(2q)=f(2)f(q)=2q$ so $f(n)=n: n\\leq 2q$, and so on...", "Solution_3": "[quote=\"SpongeBob\"]I think that Q1 doesn't have any sense, if $m>n$ and $m^{n}=n^{m}$\nthen $f(m)=n\nf:N\u2192N\nf is strictly increasing\nf(mn)=f(m)f(n)\nIf m\u2260n & m^n=n^m, then f(m)=n or f(n)=m\n\nf(12)?\n\nQ2> \nf:N\u2192N\nf(f(m)+f(n))=m+n\n\nshow that f(n)=n\n\nQ3>\nf:N\u2192N\nf is strictly increasing\nf(7)=7\nIf p,q are relatively prime,then f(pq)=f(p)f(q)\n\nshow that f(n)=n[/quote]\r\nI think you really should read this->[url]http://www.mathlinks.ro/Forum/viewtopic.php?t=135914[/url]", "Solution_7": "$f(n)$ is $n^{2}$ for all $n$. This is also easy. Choose $r$ such that for $l,n$ we have $2^{r}1$. Simularly we can get a contradiction if $f(n)>n^{2}$.", "Solution_8": "Like I said, I don't like that proof (for Q3), so while I was in school today, I thought of something else.\r\n\r\nFirst paragraph still remains the same: if $f(n)=n$ and $f(n+k)=n+k$ then $f(m)=m$ for all $n\\leq m \\leq n+k$. And this is the main fact we use.\r\nIf we have that $f(n)=n$ and $f(n-1)=n-1$ then $f(n(n-1))=f(n)f(n-1)=n(n-1)$, so for every $m: n\\leq m\\leq n^{2}-n$ it stands that $f(m)=m$. Then take $f((n^{2}-n)(n^{2}-n-1))=f(n^{2}-n)f(n^{2}-n-1)=(n^{2}-n)(n^{2}-n-1)$ and so on, you can always find some number $x$ which is arbitarly big, and $f(n)=n$ for all $n\\leq x$.\r\n\r\nBye" } { "Tag": [ "Alcumus", "articles" ], "Problem": "is there a way to acess the game from the AoPS website instead of this link?", "Solution_1": "[quote=\"abacadaea\"]is there a way to acess the game from the AoPS website instead of this link?[/quote]\r\n\r\nNot currently, except for the link in the Game Information topic.", "Solution_2": "You should make one It would solve alot of trouble", "Solution_3": "Could you put it in the \"My Classes\" Page just like Alcumus?", "Solution_4": "[quote=\"minicon\"]Could you put it in the \"My Classes\" Page just like Alcumus?[/quote]\r\n\r\nWe'll have a public link relatively soon.", "Solution_5": "Where is the alcumus?", "Solution_6": "Oh, did I spill the beans again?\r\n :blush:", "Solution_7": "I just get redirected to http://www.artofproblemsolving.com/Alcumus/invaliduser.php :D", "Solution_8": "[quote=\"minicon\"]Oh, did I spill the beans again?\n :blush:[/quote]\r\nWasn't the description already in the Focus article about AoPS?" } { "Tag": [ "algebra", "polynomial", "function", "real analysis", "real analysis unsolved" ], "Problem": "Let $P \\in \\mathbb{R}[X]$-fixed polynomial of odd degree, $degP$-odd. And $f: \\mathbb{R} \\to \\mathbb{R}$, $f \\in C^{\\infty}$ such that $|f^{n}(x)| \\leq |P(x)|$ for all $n \\in \\mathbb{N}$ and $x \\in \\mathbb{R}$. Prove that $f \\equiv 0$.", "Solution_1": "Let $a$ be a zero of $p$. Then $f^{(n)}(a)=0$ for all $n$. Let $f_n(x)=\\sum_{j=0}^n\\frac{f^{(j)}(a)}{j!}(x-a)^j=0$.\r\n\r\nFor any $r$, there exists $M$ such that $|p(x)|\\le M$ for all $x$ such that $|x-a|\\le r$. Now by Taylor's theorem, $|f(x)-f_n(x)|\\le \\frac{M}{(n+1)!}x^{n+1}\\to0$ whenever $|x-a|\\le r$, and therefore $|f(x)|=0$ in this range. Since $r$ was arbitrary, $f$ is identically zero.\r\n\r\n$P$ can be any continuous function with a zero. If $P$ has no zero, we merely get that the power series for $f$ converges to $f$ on the whole real line.", "Solution_2": "[quote=\"jmerry\"]\n$P$ can be any continuous function with a zero. [/quote]\r\nEven continuity is not needed; it suffices that $P$ be finite at every point. See [url]http://www.artofproblemsolving.com/Forum/viewtopic.php?t=28646[/url]" } { "Tag": [ "LaTeX", "algebra unsolved", "algebra" ], "Problem": "Prove that for n positive integer, n fixed \r\n\r\n\\[x^{2n}+y^{2n}+z^{2n}=x^{2n+1}+y^{2n+1}+z^{2n+1}\\]\r\n\r\n\r\nhas an infinity of solution for x, y, z integer...", "Solution_1": "I swear that this problem has a solution, my teacher haas a book with the solution, but he won't help me!!!!", "Solution_2": "After many tries (and fails :( ) :\r\n\r\nTrying with x=-z < 0 and y = -k*x gives the solutions :\r\n\r\nx = -k*(2*k^(2n)+1), y = 2*k^(2n)+1 and z=k*(2*k^(2n)+1) are solutions :D", "Solution_3": "Thank you very much! Could you please write it again in latex, because i think I didn't understand correctly...I ow you! :D" } { "Tag": [], "Problem": "Any tips on how to improve my violin skills over the summer?", "Solution_1": "Starting from what level, and what's your goal?\r\n\r\nIt's mostly in the etudes. :surrender: :weightlift: I know, i did that last summer.\r\n\r\nSeriously, I'm buckling down this summer, but that's hardcore. Just normal would be lots of scales (30-60 min), etudes (40-60), and whatever pieces your teacher gives you.\r\n\r\nReally good etude books;\r\nFiorillo, Dont (little), Kreutzer, Gavinies (once slow, once fast), Dont (big), then *gasp* Paganini, done in that order. If your done with Paganini, get Ernest 6 etudes. nobody ever gets to those, though. I dream of playing those..... :(", "Solution_2": "Thanks!\r\n\r\n :D", "Solution_3": "for scales, try carl flesch if you havent...\r\nthats what im playing right now.", "Solution_4": "Try Mazas and Wolfhart etudes.\r\nIt's what I did, and I improved\r\nA LOT! :D" } { "Tag": [ "function", "calculus", "integration", "absolute value", "real analysis", "real analysis unsolved" ], "Problem": "Please help me with this problem:\r\n\r\nFind a sequence of functions fn(x) which converge to 0 in L^1 such that fn(x) doesn't converge to 0 for any value of x in R.\r\n\r\nthanks\r\n\r\n(by convergence in L^1 ,which is the space of integrable functions, i mean the integral of the absolute value of fn(x) converge to 0.)", "Solution_1": "Know your standard examples: this calls for the \"rotating tooth\". Take a thin bump of fixed height and translate it to cover the whole interval. Repeat with a thinner bump, and keep going infinitely. The widths and thus the integrals go to zero, but each point has a large value infinitely often.", "Solution_2": "This is what i ended up with myself too. \r\nthanks" } { "Tag": [], "Problem": "Let $z > 1$ be an integer and let $M$ be the set of all numbers of the form $\\dsp\r\nz_k = \\sum \\limits^n_{k=0} z^k,$ $k = 0,1, \\ldots$ Determine the set $T$ of divisors of at least one of the numbers $z_k$ from $M.$", "Solution_1": "Isn't it obvious they are all number coprime to $z$?", "Solution_2": "yeah, if i'm not totally wrong...\r\n\r\nPeter", "Solution_3": "Indeed. Assume $a,b\\in T,\\ (a,b)=1$. Then $a|z_n,b|z_m$. It's not hard to see that $z_m,\\ z_n|z_{mn+1}\\Rightarrow ab\\in T$. Now assume $p$ is a prime which doesn't divide $z$. If $p^k|z-1$ and $p^{k+1}$ doesn't, then $\\displaystyle p^u|z_{\\phi(p^{k+u})-1}=\\frac{z^{\\phi(p^{k+u})}-1}{z-1}$. Combine these two facts and we get the result.\r\n\r\n[TeX question: how do you make \"does not divide\"?]", "Solution_4": "[quote=\"grobber\"][TeX question: how do you make \"does not divide\"?][/quote]\r\n\r\na \\not |\\ b results in $a \\not |\\ b$" } { "Tag": [ "inequalities", "inequalities proposed" ], "Problem": "For $ a,b,c,d$ positive real numbers, prove that\r\n\r\n$ \\frac{a^2}{b(b\\plus{}c)}\\plus{}\\frac{b^2}{c(c\\plus{}d)}\\plus{}\\frac{c^2}{d(d\\plus{}a)}\\plus{}\\frac{d^2}{a(a\\plus{}b)}\\geq2$", "Solution_1": "[quote=\"marin.bancos\"]For $ a,b,c,d$ positive real numbers, prove that\n\n$ \\frac {a^2}{b(b \\plus{} c)} \\plus{} \\frac {b^2}{c(c \\plus{} d)} \\plus{} \\frac {c^2}{d(d \\plus{} a)} \\plus{} \\frac {d^2}{a(a \\plus{} b)}\\geq2$[/quote]\r\n\r\nHello [b]marin.bancos[/b]\r\n\r\nAcording to Am-Gm inequality we have :\r\n\r\n$ \\frac {a^2}{b(b \\plus{} c)} \\plus{} \\frac {b}{b \\plus{} c} \\ge \\frac {2a}{b \\plus{} c}$\r\n\r\nand we 'll prove the inequality :\r\n\r\n$ \\sum_{cyc} \\frac {a}{b \\plus{} c} \\ge \\sum_{cyc}\\frac {b}{b \\plus{} c}$\r\n\r\nwe have :\r\n\r\n$ \\sum_{cyc} \\frac {a}{b \\plus{} c} \\ge \\sum_{cyc}\\frac {b}{b \\plus{} c}$\r\n$ \\Leftrightarrow \\frac {a \\plus{} c}{b \\plus{} c} \\plus{} \\frac {b \\plus{} d}{c \\plus{} d} \\plus{} \\frac {c \\plus{} a}{d \\plus{} a} \\plus{} \\frac {b \\plus{} d}{a \\plus{} b} \\ge 4$\r\n\r\nand we have :\r\n\r\n$ \\frac {a \\plus{} c}{b \\plus{} c} \\plus{} \\frac {b \\plus{} d}{c \\plus{} d} \\plus{} \\frac {c \\plus{} a}{d \\plus{} a} \\plus{} \\frac {b \\plus{} d}{a \\plus{} b} \\equal{} \\frac {(a \\plus{} c)(a \\plus{} b \\plus{} c \\plus{} d)}{(b \\plus{} c)(a \\plus{} d)} \\plus{} \\frac {(b \\plus{} d)(a \\plus{} b \\plus{} c \\plus{} d)}{(a \\plus{} b)(c \\plus{} d)} \\ge \\frac {4(b \\plus{} d)}{a \\plus{} b \\plus{} c \\plus{} d} \\plus{} \\frac {4(a \\plus{} c)}{a \\plus{} b \\plus{} c \\plus{} d} \\equal{} 4$\r\n\r\nHence it 's enough to check that :\r\n\r\n$ \\sum_{cyc}\\frac {a}{b \\plus{} c} \\ge 2$\r\n\r\nWhich is easy By $ CS$ :)\r\n\r\nP/s:Thanks Mathias_Dk :)", "Solution_2": "[quote=\"Abdek\"]\nBy the rearangment inequality we have :\n\n$ \\sum_{cyc} \\frac {a}{b \\plus{} c} \\ge \\sum_{cyc}\\frac {b}{b \\plus{} c}$\n[/quote]\r\nCan you be more specific?", "Solution_3": "[quote=\"Mathias_DK\"][quote=\"Abdek\"]\nBy the rearangment inequality we have :\n\n$ \\sum_{cyc} \\frac {a}{b + c} \\ge \\sum_{cyc}\\frac {b}{b + c}$\n[/quote]\nCan you be more specific?[/quote]\r\n\r\nW.L.O.G $ a\\ge b\\ge c \\ged$\r\n$ \\sum_{cyc}\\frac {b}{b + c} - \\sum_{cyc} \\frac {a}{b + c} = \\sum b(\\frac {1}{b + c} - \\frac {1}{c + d})$\r\n ( + )developed Abel $ \\sum b(\\frac {1}{b + c} - \\frac {1}{c + d}) \\ge 0\\sum_{cyc}$\r\n--->$ \\frac {b}{b + c}\\ge\\sum_{cyc} \\frac {a}{b + c}$", "Solution_4": "Thank you, [b]Abdek[/b]! Your idea is really interesting. Your valuable remark is that\r\n$ \\left.\\frac{a^{2}}{b(b\\plus{}c)}\\plus{}\\frac{b^{2}}{c(c\\plus{}d)}\\plus{}\\frac{c^{2}}{d(d\\plus{}a)}\\plus{}\\frac{d^{2}}{a(a\\plus{}b)}\\geq\\frac{a}{b\\plus{}c}\\plus{}\\frac{b}{c\\plus{}d}\\plus{}\\frac{c}{d\\plus{}a}\\plus{}\\frac{d}{a\\plus{}b}\\geq2\\right.$\r\nIn addition, I'd like to say that my proof is different than yours. I'm waiting for new solutions to the problem.", "Solution_5": "[quote=\"marin.bancos\"]For $ a,b,c,d$ positive real numbers, prove that\n\n$ \\frac {a^2}{b(b \\plus{} c)} \\plus{} \\frac {b^2}{c(c \\plus{} d)} \\plus{} \\frac {c^2}{d(d \\plus{} a)} \\plus{} \\frac {d^2}{a(a \\plus{} b)}\\geq2$[/quote]\r\nBy CS and AM-GM:\r\n$ LHS \\ge \\frac{(\\sum a^2)^2}{\\sum a^2b^2\\plus{} \\sum a^2bc} \\ge \\frac{(\\sum a^2)^2}{2\\sum a^2b^2} \\ge \\frac{(\\sum a^2)^2}{\\frac{(\\sum a^2)^2}{2}}\\equal{}2$", "Solution_6": "Dear [b]CSS-MU[/b],\r\nYou took into account that\r\n$ \\sum a^2b^2\\geq\\sum a^2bc$\r\nCould you explain this inequality? Maybe I'm missing something...", "Solution_7": "[quote=\"marin.bancos\"]For $ a,b,c,d$ positive real numbers, prove that\n\n$ \\frac {a^2}{b(b \\plus{} c)} \\plus{} \\frac {b^2}{c(c \\plus{} d)} \\plus{} \\frac {c^2}{d(d \\plus{} a)} \\plus{} \\frac {d^2}{a(a \\plus{} b)}\\geq2$[/quote]\r\nmarin.bancos, what about the following proof?\r\n$ \\sum_{cyc}\\frac {a^2}{b(b \\plus{} c)}\\geq\\frac {(a^2 \\plus{} b^2 \\plus{} c^2 \\plus{} d^2)^2}{\\sum(a^2b^2 \\plus{} a^2bc)}.$\r\n$ \\frac {(a^2 \\plus{} b^2 \\plus{} c^2 \\plus{} d^2)^2}{\\sum(a^2b^2 \\plus{} a^2bc)}\\geq2\\Leftrightarrow$\r\n$ \\Leftrightarrow a^4 \\plus{} b^4 \\plus{} c^4 \\plus{} d^4 \\plus{} 2a^2c^2 \\plus{} 2b^2d^2\\geq2\\sum_{cyc}a^2bc.$\r\nBut $ 2a^2c^2 \\plus{} 2b^2d^2\\geq4abcd.$\r\nHence, it remains to prove that \r\n$ a^{4} \\plus{} b^4 \\plus{} c^4 \\plus{} d^4 \\plus{} 4abcd\\geq2(a^2bc \\plus{} b^2cd \\plus{} c^2da \\plus{} d^2ab),$ which see here:\r\nhttp://www.artofproblemsolving.com/Forum/viewtopic.php?t=56040", "Solution_8": "Dear [b]arqady[/b],\r\nThanks! Interesting idea... Hmmm... It's not very easy to approach the problem this way... Thanks again.", "Solution_9": "[quote=\"marin.bancos\"]Dear [b]CSS-MU[/b],\nYou took into account that\n$ \\sum a^2b^2\\geq\\sum a^2bc$\nCould you explain this inequality? Maybe I'm missing something...[/quote]\r\n\r\nWe have\r\n\r\n$ a^2 b^2 \\plus{} a^2 c^2 \\ge 2a^2bc$. It's easy\r\n\r\noh", "Solution_10": "[quote=\"marin.bancos\"]Dear [b]CSS-MU[/b],\nYou took into account that\n$ \\sum a^2b^2\\geq\\sum a^2bc$\nCould you explain this inequality? Maybe I'm missing something...[/quote]\r\nSorry,[b]marin.bancos[/b]. I had a mistake.", "Solution_11": "We have from AMGM that $ \\frac{a^2}{b(b\\plus{}c)}\\equal{}\\frac{a^2}{b^2\\plus{}bc}\\geq\\frac{a^2}{b^2\\plus{}\\frac{b^2\\plus{}c^2}{2}}\\equal{}\\frac{2a^2}{3b^2\\plus{}c^2}$\r\nTherefore $ \\frac{a^{2}}{b(b\\plus{}c)}\\plus{}\\frac{b^{2}}{c(c\\plus{}d)}\\plus{}\\frac{c^{2}}{d(d\\plus{}a)}\\plus{}\\frac{d^{2}}{a(a\\plus{}b)}\\geq2\\left(\\sum_{cyc}\\frac{a^2}{3b^2\\plus{}c^2}\\right)$, so the inequality we have to prove is $ \\sum_{cyc}\\frac{x}{3y\\plus{}z}\\geq1$, where $ a^2,b^2,c^2$, and $ d^2$ are replaced by $ x,y,z$ and $ w$ respectively.\r\n\r\nBut from Cauchy-Schwarz, $ \\sum_{cyc}\\frac{x}{3y\\plus{}z}\\equal{}\\sum_{cyc}\\frac{x^2}{3xy\\plus{}xz}\\geq\\frac{(x\\plus{}y\\plus{}z\\plus{}w)^2}{3\\sum_{cyc}xy\\plus{}2xz\\plus{}2yw}$, so we have to prove $ (x\\plus{}y\\plus{}z\\plus{}w)^2\\geq3\\sum_{cyc}xy\\plus{}2xz\\plus{}2yw$, which is equivalent to $ \\sum_{cyc}(x\\minus{}y)^2\\geq0$.\r\n\r\nEquality occurs if and only if $ a\\equal{}b\\equal{}c\\equal{}d$.", "Solution_12": "YES!!! This is the simplest and nicest solution to my problem. Well done, [b]bakerbakura[/b]! I knew you are able to find it! Congrats!\r\nThe proofs given by [b]Abdek[/b] and [b]arqady[/b] are also nice. Thanks to all of you!" } { "Tag": [ "articles" ], "Problem": "Hi,\r\nThis is Steven. I am new member to this site.The given article is really very nice.\r\n=============\r\nJohn\r\n[url=http://www.trainwithmeonline.com]workout programs[/url] [/url]", "Solution_1": "uhhhh... why are u posting workout things in a math forum??" } { "Tag": [], "Problem": "What is the largest natural number that is a solution of $ 13x \\plus{} 8 < 35$?", "Solution_1": "So Natural number $ x$ = $ x$ is Integer, $ x$>0\r\n\r\nNow also $ 13x\\plus{}8<35$ or $ 13x<27$ or $ x<\\frac{27}{13}$ Now $ x$ is integer, so largest is $ 2$." } { "Tag": [ "abstract algebra", "Ring Theory", "superior algebra", "superior algebra unsolved" ], "Problem": "Let $ R$ be a Noetherian ring and x an indeterminate. Then $ dimR[x,x^{\\minus{}1}]\\equal{}dimR\\plus{}1$ where dim means the Krull dimension.\r\n\r\nThe problem is the first exercise of chapter 10 of Eisenbud's Commutative Algebra with a View towards Algebraic Geometry. I've been thinking about it for a bit now and I'm not sure how to do this. My idea is that we can provide a lower bounds of the dimension because $ dimR[x]\\equal{}dimR\\plus{}1$ but I have no idea how to show that the dimension is no larger. Intuitively, I can see that if $ R$ is a field, then the variety $ xy\\minus{}1$ is one dimensional so $ R[x,y]/(xy\\minus{}1)$ should be 1 dimensional, but I'm not sure how to do it in the more general case. Any input is appreciated!", "Solution_1": "[quote=\"LkNsngth\"]Let $ R$ be a Noetherian ring and x an indeterminate. Then $ dimR[x,x^{ \\minus{} 1}] \\equal{} dimR \\plus{} 1$ where dim means the Krull dimension.\n\nThe problem is the first exercise of chapter 10 of Eisenbud's Commutative Algebra with a View towards Algebraic Geometry. I've been thinking about it for a bit now and I'm not sure how to do this. My idea is that we can provide a lower bounds of the dimension because $ dimR[x] \\equal{} dimR \\plus{} 1$ but I have no idea how to show that the dimension is no larger. Intuitively, I can see that if $ R$ is a field, then the variety $ xy \\minus{} 1$ is one dimensional so $ R[x,y]/(xy \\minus{} 1)$ should be 1 dimensional, but I'm not sure how to do it in the more general case. Any input is appreciated![/quote]\r\n\r\nFor the lower bound note that if $ P$ is a prime ideal of $ R,$ then $ P[x,x^{ \\minus{} 1}]$ is a prime (but not maximal) ideal of $ R[x,x^{ \\minus{} 1}].$ \r\n\r\nThe upper bound quickly follows from $ \\dim R[x] \\equal{} \\dim R \\plus{} 1$ and this simple fact that $ R[x,x^{ \\minus{} 1}]$ is the localization of $ R[x]$ at $ S \\equal{} \\{1,x,x^2, \\cdots \\}.$", "Solution_2": "Ahh thank you very much, but just to make sure, we know that $ dimR\\plus{}1$ is an upper bound because whenever we localize, there is an injection from the localized ring into the original ring, in particular $ SpecR[S^\\minus{}1]\\subseteq SpecR$ correct?", "Solution_3": "[quote=\"LkNsngth\"]Ahh thank you very much, but just to make sure, we know that $ dimR \\plus{} 1$ is an upper bound because whenever we localize, there is an injection from the localized ring into the original ring, in particular $ SpecR[S^ \\minus{} 1]\\subseteq SpecR$ correct?[/quote]\r\n\r\nFirst of all $ R_1$ is a subring of $ R_2$ doesn't imply that $ \\dim R_1 \\leq \\dim R_2$. For example $ \\mathbb{Z} \\subset \\mathbb{Q}$ but $ \\dim \\mathbb{Z}\\equal{}1 > 0 \\equal{} \\dim \\mathbb{Q}.$\r\n\r\nSecond of all, when you localize a ring at a multiplicative subset (consisting of regular elements of course!) of that ring, you get a ring which contains the original one not the other way \r\n\r\naround. What I meant was to consider any finite chain of prime ideals of $ S^{\\minus{}1}R[x]$, say: $ S^{\\minus{}1}P_n \\supset \\cdots \\supset S^{\\minus{}1}P_0.$ This will give you a chain of prime ideals of $ R[x]$, i.e. $ P_n \\supset \\cdots \\supset P_0$.\r\n\r\nThus $ \\dim S^{\\minus{}1}R[x] \\leq \\dim R[x]\\equal{}\\dim R \\plus{} 1.$", "Solution_4": "ah yes there was a typo above, I meant that there was an injection in the [i]ideals[/i] of the localized ring into the ideals of the original ring. So yes I believe we are thinking of the same thing. Thanks." } { "Tag": [ "trigonometry", "ratio", "number theory", "least common multiple", "combinatorics unsolved", "combinatorics" ], "Problem": "Four lines are given in the plane such that no two intersect and no three are concurrent. The three intersection points on each line form two segments, giving eight segments in total. Can the lengths of these segments be\r\n\r\n(a) $1,2,3,4,5,6,7,8$?\r\n(b) different positive integers?", "Solution_1": "[quote=\"cancer\"]Four lines are given in the plane such that no two intersect and no three are concurrent. The three intersection points on each line form two segments, giving eight segments in total. Can the lengths of these segments be\n\n(a) $1,2,3,4,5,6,7,8$?\n(b) different positive integers?[/quote]\r\n\r\n(a) : No\r\n(b) : Yes\r\n\r\n(a) ==========================\r\nLet AB the segment with length 1. Let (D1 and D2) the lines containing A and (D1 and D3) the lines containing B (D1 is the line AB). D2 and D3 intersect in C.\r\nAB, AC and BC are all integers and AB=1. Hence AC=BC (else A,B and C would be on the same line).\r\nHence D4 intersects D2 in D between A and C and intersects D3 in E between B and C. Let F be the intersection between D1 and D4. WLOG consider A between F and B.\r\nFor easy writings, let $AB=1,AD=a,DC=b,BE=c, EC=d, FA=u,FD=v, DE=w$. Let $\\alpha$ be the angle AB,AC.\r\n$\\cos(\\alpha)=\\frac{1}{2(a+b)}=\\frac{1}{2(c+d)}$\r\nIn triangle ADF, we have $v^{2}=u^{2}+a^{2}+\\frac{ua}{a+b}$ and $a+b|ua$ and hence $a+b|ub$\r\nIn triangle BEF, we have $(v+w)^{2}=(u+1)^{2}+c^{2}-\\frac{c(u+1)}{a+b}$ and $a+b|c(u+1)$ and hence $a+b|d(u+1)$ (since $a+b=c+d$)\r\n\r\nSo we have $a+b\\in\\{7,8,9,10,11,12,13\\}$ and $a+b|ub$, $a+b|ua$, $a+b|(u+1)c$, and $a+b|(u+1)d$, with $\\{a,b,c,d,u,v,w\\}=\\{2,3,4,5,6,7,8\\}$ and it is easy to check that these conditions are impossible.\r\n\r\n(b) ===========================\r\nLet $\\frac{\\pi}{2}>\\alpha>\\beta>0$ two angles such that $\\sin(\\alpha),\\cos(\\alpha),\\sin(\\beta),\\cos(\\beta)$ are all rational.\r\nOn an line D1, let two points A,B such that $AB=1$\r\nLet D2 the line intersecting D1 in A and such that $(D1,D2)=\\alpha$\r\nLet D3 the line intersecting D1 in B and such that $(D1,D3)=\\pi-\\alpha$\r\nLet C the intersection of D2 and D3.\r\nLet D0 the line such that $(D1,D0)=\\beta$ and such that C belongs to D0. Let M the intersection between D1 and D0. A is between M and B.\r\nLet F any point between M and A such that FA is a rational number.\r\nLet D4 the line such that $(D1,D4)=\\beta$ and such that F belongs to D4. Let D the intersection between D4 and D2 (D between A and C) and E the intersection between D4 and D3 (E between B and C).\r\n\r\nIn triangle ADF, all the angles have their sinus rational and FA is rational, so FA, AD an FD are rational.\r\nIn triangle ABC, all the angles have their sinus rational and AB=1 is rational. So AB, AC and BC are rational.\r\nIn triangle BEF, all the angles have their sinus rationak and FB=FA+1 is rational. So BE and EF are rational.\r\nSo $FA,AB,BE,EC=BC-BE,AD, DC=AC-AD,FD,DE=EF-FD$ are rational.\r\n\r\nSo we just have to increase this figure by a ratio which is the lcm of all the denominators of these rational numbers in order to have a construction in which the eight segments have lengths in natural numbers. And it is easy to choose $\\alpha,\\beta$ and FA such that all these distances are different.\r\n\r\nQ.E.D" } { "Tag": [ "induction", "combinatorics proposed", "combinatorics" ], "Problem": "[b]Mader's Theorem. [/b] Every graph of average degree at least $4k$ has a $k$-connected subgraph.\r\n\r\nIt's very beautiful. :D Enjoy it!", "Solution_1": "Let's prove by inducton the following lemma:\r\nIf a graph $G$ with $m$ vertices contains no $k$-connected subgraph, than it has at most $(2m-1)k+1$ edges.\r\nFor $m\\leq k$ it's obvious.\r\nNow induction step:\r\nSuppose for all $m1$.\r\nTo use induction, this must hold for $m\\leq k$ and it suffices to look only at case $m=k$.\r\nSo we get $(ck-1)k+1=\\frac{k(k-1)}2$ and so $ck\\geq \\frac{k+1}2$ so $c=\\frac{k+1}{2k}$.\r\nSo we can replace $4$ by $\\frac{k+1}k$. \r\n\r\nThis means if a graph has average degree at least $k+1$, it contains a $k$-connected subgraph. This cannot be true, since someone would point it out for sure and would improve it.\r\n\r\nWhat is wrong? :?", "Solution_4": "Actually, here is the general result.\r\n\r\nGiven $k\\geq 1$, denote by $f(n)$ the biggest number of edges a graph with $n$ vertices and no $k$-connected subgraph can have. Then $f(n)=g(n)$ where\r\n$g(n)= \\frac{n(n-1)}2$ for $nk$ then $g(i)+g(j)+k-1\\leq g(i+j)=g(1)+g(i+j-1)+k-1$.\r\nNow the proof is by induction:\r\nFor $n \\leq k$ it's obvious.\r\nFor $n>k$ let $G$ be the graph with $n$ vertices, $f(n)$ the maximal number of edges and no $k$-connected subgraph.\r\nThen there are at most $k-1$ edges whose removal disconnects $G$ into two pieces $A,B$ with \r\n$|A|=i, |B|=j, i+j=f(n)$ and $1\\leq i,j - that doesn't work. Instead, enclose your math expressions between $ \\$$ signs.\r\n\r\n2. I am having trouble deciphering exactly what the problem is here or exactly what you are asking." } { "Tag": [ "MATHCOUNTS", "AMC", "AMC 10", "AMC 8", "geometry", "AIME", "USA(J)MO" ], "Problem": "Mathcounters, what are your AMC 8 scores?", "Solution_1": "Either i forgot mine or i don't no it but i no i didn't get a perfect and i did bad", "Solution_2": "i got a 21", "Solution_3": "[quote=\"Naonao\"]Either i forgot mine or i don't no it but i no i didn't get a perfect and i did bad[/quote]\r\n\r\nPlease do capitalize and not use chatspeak. It makes it easier to read and makes us appear more professional.", "Solution_4": ":wow: :wow:\r\nI got a perfect score!! (just found out from the site because my teacher hasnt gotten the scores back yet)", "Solution_5": "There was only one person from Delaware that got a perfect score... could that be you? :P", "Solution_6": "I got 21 on that test......", "Solution_7": "Possible, there could be some other person with the same name from the same school in the same state :D and if thats true, I didnt get a perfect score :noo: :noo: \r\n\r\nHowd you know I was from Delaware??", "Solution_8": "I actually just found out I was a perfect scorer on the AMC 8 a few days ago (too). You can tell from my username. :o", "Solution_9": "I just realized that your username is your real name :o \r\n\r\nWell, at least us perfect scorers are in good company (Neal Wu) :P", "Solution_10": "I'm not a Mathcount competitor, but I got 22/25.\r\n\r\nquestion for everyone \r\n\r\nWas anyone invinted to AMC10? (it was mentioned in AMC site that high scorers in AMC8 are invited to AMC10.)\r\nBut I don't think they actually invite people to AMC10...", "Solution_11": "tch, i'm taking the AMC10... aiming for a 150 :o ...or w/e the perfect score is :P", "Solution_12": "I got a perfect score on the AMC 8 (from Washington--there were five of us, two (including me) were on the national mathcoutns team last yeawr).\r\nAnd no, you don't have to be invited to take the AMC 10. You can come no matter what your score was on the 8. The first one that you need to be invited to is the AIME.", "Solution_13": "I got a 25 on the AMC 8. Texas. I don't think you get invited to the AMC 10. I think you just take it. There were 9 perfects in Texas. The test was one of the harder ones. I think maybe it is just me.", "Solution_14": "just a question, how do you find out your score from the site? our teacher's been sick so we won't find out our scores for a while.", "Solution_15": "Actually, there's no rule about taking the AMC 8, which is rather separate from the AMC10 or 12 in the sense that it doesn't continue to AIME and USAMO. Anybody can take the 10 without taking the 8.", "Solution_16": "No offense to those who did well (not me) but AMC 8 is relatively useless in terms of qualifying for anything.", "Solution_17": "I took it last year too, and did horribly. I got somewhere around a 17.\r\n[quote=\"binonunquineist\"]The amc8 is superbly easy, even compared to MATHCOUNTS material. I got a 25/25 last year as a sixth grader and a 25/25 this year as a seventh grader. Now I just have to continue that.[/quote]\r\nThanks. :dry:", "Solution_18": "Binonunquneist got a perfect score on the math SAT this year too...", "Solution_19": "[quote=\"pgpatel\"]Binonunquneist got a perfect score on the math SAT this year too...[/quote]\r\n\r\nSAT II? or the math portion of the SAT I. I think the hardest part of the math portion of the SAT I only has very basic Algebra II and all the problems are very straightfoward, not many word ones like mathcounts, not sure though, that's what I've heard.", "Solution_20": "Math portion for duke tip. THat or on the math ACT.", "Solution_21": "800 on the SAT I math isn't that hard. I mean even I did that. Qualifying for National MathCounts is a lot harder, in my opinion.", "Solution_22": "[quote=\"lotrgreengrapes7926\"]800 on the SAT I math isn't that hard. I mean even I did that. Qualifying for National MathCounts is a lot harder, in my opinion.[/quote]\r\n\r\nNot if your from North Dakota or the Virgin Islands, and what does the SAT I cover? Just Algebra I and Geometry?", "Solution_23": "25. Yup.\r\n\r\nI thought the AMC 8 was a really long time ago, though, so why you're asking here and now is beyond me.", "Solution_24": "[quote=\"Ignite168\"][quote=\"lotrgreengrapes7926\"]800 on the SAT I math isn't that hard. I mean even I did that. Qualifying for National MathCounts is a lot harder, in my opinion.[/quote]\n\nNot if your from North Dakota or the Virgin Islands, and what does the SAT I cover? Just Algebra I and Geometry?[/quote]\r\n\r\nI think it is supposed to have algebra II (but I aced it without knowing that), but yeah, it is easier to ace it than make it to nationals. But just because you didn't ace it doesn't mean to can't make nats because 70 or so questions is a lot to not make stupid mistakes it.", "Solution_25": "[quote=\"ehehheehee\"][quote=\"Ignite168\"][quote=\"lotrgreengrapes7926\"]800 on the SAT I math isn't that hard. I mean even I did that. Qualifying for National MathCounts is a lot harder, in my opinion.[/quote]\n\nNot if your from North Dakota or the Virgin Islands, and what does the SAT I cover? Just Algebra I and Geometry?[/quote]\n\nI think it is supposed to have algebra II (but I aced it without knowing that), but yeah, it is easier to ace it than make it to nationals. But just because you didn't ace it doesn't mean to can't make nats because 70 or so questions is a lot to not make stupid mistakes it.[/quote]\r\n\r\nYeah, the SAT is more about doing relatively easy problems accurately. Mathcounts is the real deal.", "Solution_26": "[quote=\"cowpi\"]Yeah, the SAT is more about doing relatively easy problems accurately. Mathcounts is the real deal.[/quote]\r\nPersonally, I think that AMC 10, 12, and AIME (and USAMO) are a better gauge for mathematical ability, while MathCounts requires you to be fast and know lots of tricks for solving problems. Plus, they're harder. :P Maybe I'm just saying this because I'm better at that stuff than MathCounts. :D", "Solution_27": "AMC competitions don't fly you across the country to a nice hotel and vacation in addition to the competition. They just mail you the forms. It is harder, but probably because 1. you get more time 2. there is trig laws, polynomials, logarithms, imaginary numbers, e, and all that fun stuff...\r\n\r\nMy AOPS Book 2 that is supposed to help me will take another 1-2 weeks to come (I have waited one already)", "Solution_28": "i got 23/25 on the AMC 8, 23/25 = 138 on the AMC 10B ;) \r\nSo i would say im better at harder, longer problems than easier, faster problems.", "Solution_29": "[quote=\"ehehheehee\"]AMC competitions don't fly you across the country to a nice hotel and vacation in addition to the competition. They just mail you the forms. It is harder, but probably because 1. you get more time 2. there is trig laws, polynomials, logarithms, imaginary numbers, e, and all that fun stuff...\n\nMy AOPS Book 2 that is supposed to help me will take another 1-2 weeks to come (I have waited one already)[/quote]\r\nUnless you're REALLY REALLY smart and get to go to the IMO. :D" } { "Tag": [ "geometry", "geometric transformation", "reflection", "geometry unsolved" ], "Problem": "$ ABCD$ is a quadrilateral. $ A'BCD'$ is the reflection of $ ABCD$ in $ BC,$ $ A''B'CD'$ is the reflection of $ A'BCD'$ in $ CD'$ and $ A''B''C'D'$ is the reflection of $ A''B'CD'$ in $ D'A''.$ Show that; if the lines $ AA''$ and $ BB''$ are parallel, then ABCD is a cyclic quadrilateral.", "Solution_1": "Denote $\\alpha = \\measuredangle DAB,\\ \\beta = \\measuredangle ABC,\\ \\gamma = \\measuredangle BCD,\\ \\delta = \\measuredangle CDA$. In addition, denote $\\alpha_1 = \\measuredangle DAC,\\ \\alpha_2 = \\measuredangle CAB$, $\\beta_1 = \\measuredangle ABD,\\ \\beta_2 = \\measuredangle DBC$, $\\gamma_1 = \\measuredangle BCA,\\ \\gamma_2 = \\measuredangle ACD$, $\\delta_1 = \\measuredangle CDB,\\ \\delta_2 = \\measuredangle BDA$. Obviously, $\\alpha = \\alpha_1 + \\alpha_2,\\ \\beta = \\beta_1 + \\beta_2,\\ \\gamma = \\gamma_1 + \\gamma_2,\\ \\delta = \\delta_1 + \\delta_2$.\r\n\r\nThe triangle $\\triangle AA''C$ is isosceles with AC = CA'' and\r\n\r\n$\\measuredangle A''CA = \\measuredangle A''CD' + \\measuredangle D'CB + \\measuredangle BCA = \\gamma_2 + \\gamma + \\gamma_1 = 2 \\gamma$\r\n\r\n$\\measuredangle CAA'' = \\frac{180^\\circ - \\measuredangle A''CA}{2} = 90^\\circ - \\gamma$.\r\n\r\nThe triangle $\\triangle BB''D'$ is isosceles with BD' = D'B'' and\r\n\r\n$\\measuredangle B''D'B = 360^\\circ - (\\measuredangle B''D'A'' + \\measuredangle A''D'C + \\measuredangle CD'B) =$ $360^\\circ - (\\delta_2 + \\delta + \\delta_2) = 360^\\circ - 2 \\delta$\r\n\r\n$\\measuredangle D'BB'' = \\frac{180^\\circ - \\measuredangle B''D'B}{2} = \\delta - 90^\\circ$.\r\n\r\nLet the lines BB'' and AA'' meet at a point X and we will calculate the angle $\\measuredangle AXB \\equiv \\measuredangle A''XB''$. The lines AA, BB'' are parallel (and their intersection X at infinity) iff $\\measuredangle AXB'' = 0$.\r\n\r\n$\\angle A''XB'' = \\measuredangle D'BB'' + \\measuredangle CBD' + \\measuredangle ABC -180^\\circ + \\measuredangle CAB - \\measuredangle CAA'' =$\r\n\r\n$= \\delta - 90^\\circ + \\beta_2 + \\beta - 180^\\circ + \\alpha_2 - (90^\\circ - \\gamma) = \\beta_2 + \\alpha_2 - \\alpha = \\beta_2 - \\alpha_1$\r\n\r\nThus we see that the lines AA'', BB'' are parallel iff $\\alpha_1 = \\beta_2$ or $\\measuredangle DAC = \\measuredangle DBC$, which is equivalent to the quadrilateral ABCD being cyclic." } { "Tag": [ "inequalities", "logarithms", "search", "inequalities unsolved" ], "Problem": "Let abc=1 ,a,b,c>o. Prove that:\r\n$ 10(a^3\\plus{}b^3\\plus{}c^3)\\minus{}9(a^5\\plus{}b^5\\plus{}c^5) \\geq 3$", "Solution_1": "Try $ a\\equal{}10^4,b\\equal{}c\\equal{}\\frac{1}{100}$", "Solution_2": "oops! got it wrong", "Solution_3": "What are you going to do, then?\r\n\r\nWe can't subtract both sides directly in this case.", "Solution_4": "I'am sorry but this inequality is wrong, i said that in my first post in this topic. Am i wrong :D", "Solution_5": "This problem should be $ 10(a^3\\plus{}b^3\\plus{}c^3)\\minus{}9(a^5\\plus{}b^5\\plus{}c^5)\\leq 3.$\r\n\r\nLet $ f(a)\\equal{}10a^3\\minus{}9a^5\\plus{}15\\ln a\\ (a>0)$, we have $ f'(a)\\equal{}\\frac{15}{a}(1\\minus{}a)(3a^4\\plus{}3a^3\\plus{}a^2\\plus{}a\\plus{}1)$, yielding $ f(a)\\leq f(1)\\Longleftrightarrow f(a)\\leq 1.\\ \\therefore f(a)\\plus{}f(b)\\plus{}f(c)\\leq 3.$", "Solution_6": "I think he means $ 10(a^{3} \\plus{} b^{3} \\plus{} c^{3}) \\minus{} 9(a^{5} \\plus{} b^{5} \\plus{} c^{5})\\geq1$\r\n\r\nwhere $ a\\plus{}b\\plus{}c\\equal{}1$\r\n\r\nwhich is here\r\nhttp://www.mathlinks.ro/viewtopic.php?search_id=984446516&t=161399\r\n\r\n :)" } { "Tag": [ "algebra", "polynomial", "function", "limit", "calculus" ], "Problem": "How do we determine whether the degree of a polynomial $ h(x)$ is even or odd? [Use the below examples if necessary]\r\n\r\nAlso, suppose a certain root of the polynomial $ f(x)$ has multiplicity $ k$, where $ k > 1$. (i.e. double roots, triple roots etc.) How would this influence the graph of $ f(x)$?", "Solution_1": "This is a pretty detailed treatment:\r\n\r\nhttp://en.wikipedia.org/wiki/Degree_of_a_polynomial", "Solution_2": "It doesn't really answer my problem.", "Solution_3": "A polynomial is even if it is symmetric with respect to the y-axis.\r\n\r\nf(x) is even if f(-x)=f(x)\r\n\r\nA polynomial is odd if it is symmetric with respect to the origin.\r\n\r\nf(x) is odd if -f(-x)=f(x)\r\n\r\n\r\nIf a root has odd multiplicity, then the graph crosses the x-axis at that root. If the multiplicity is odd, then the graph just touches the x-axis at that point. (Like the point (-3,0) in your 1st graph.", "Solution_4": "I think you had some typos in your last paragraph.\r\n\r\nDo you mean that if the degree of the polynomial is even, then the polynomial is even? And if the degree is odd, then the polynomial is odd? I don't think they're related at all! [b]I'm asking about the degree![/b]\r\n\r\nHehe. Maybe I'm wrong.", "Solution_5": "A function is odd iff. it is point-symmetric about the origin. A polynomial function is odd iff. [b]all [/b]terms with non-zero coefficients are of odd degree. Constants are considered the coefficient of the $ x^0$ term.\r\n\r\nA function is even iff. it is symmetric across the y-axis. A polynomial function is even iff. [b]all [/b]terms with non-zero coefficients are of even degree.\r\n\r\nJust looking at your three graphs, none of those polynomials are even or odd.\r\n\r\n-------------------\r\n\r\nLet $ P(x)$ be a polynomial with a root $ r$ of multiplicity $ k$, and let $ Q(x) \\equal{} \\frac {P(x)}{(x \\minus{} r)^k}$ be another polynomial* (with a removable discontinuity). Then in the neighborhood of $ x \\equal{} k$, $ P(x) \\approx Q(r) (x \\minus{} r)^k$.\r\n\r\nSince $ Q(r)$ does not vary with $ r$, we can write this as $ P(x) \\approx c (x \\minus{} r)^k$ for some constant $ c$. Now, what do you know about a function of this form -- what does it look like? Try varying $ c$ and $ r$ and $ k$ on Mathematica, and you'll figure it out :)\r\n\r\nThe neighborhood I refer to can be very small, so you might have to zoom in. It's probably a good idea to pick other numbers that are far away from the root you are observing to create a larger neighborhood.\r\n\r\n\r\n\r\n*- By $ Q(x) \\equal{} \\frac{P(x)}{(x\\minus{}r)^k}$, I really mean that Q is the polynomial $ P(x)$ without the $ k$ factors of $ (x\\minus{}r)$; I don't mean to multiply the factors in and then divide them out, since this would just make $ Q(r) \\equal{} 0/0$. A better way of saying this is that $ Q(x) \\equal{} \\lim_{t \\to x}\\frac{P(t)}{(t\\minus{}r)^k}$", "Solution_6": "Consider $ \\lim_{x \\rightarrow \\pm \\infty} h(x)$ for odd and even degree polynomials. (For odd-degree polynomials, these two sides will tend towards different vertical directions, namely one $ \\rightarrow \\infty$ and one $ \\rightarrow \\minus{}\\infty$, while for even-degree polynomials, they will tend towards the same direction, namely both $ \\rightarrow \\infty$ or both $ \\rightarrow \\minus{}\\infty$) :wink:", "Solution_7": "[quote=\"The Zuton Force\"]A function is odd iff. it is point-symmetric about the origin. A polynomial function is odd iff. [b]all [/b]terms with non-zero coefficients are of odd degree. Constants are considered the coefficient of the $ x^0$ term.\n\nA function is even iff. it is symmetric across the y-axis. A polynomial function is even iff. [b]all [/b]terms with non-zero coefficients are of even degree.\n\nJust looking at your three graphs, none of those polynomials are even or odd.\n\n-------------------\n\nLet $ P(x)$ be a polynomial with a root $ r$ of multiplicity $ k$, and let $ Q(x) \\equal{} \\frac {P(x)}{(x \\minus{} r)^k}$ be another polynomial* (with a removable discontinuity). Then in the neighborhood of $ x \\equal{} k$, $ P(x) \\approx Q(r) (x \\minus{} r)^k$.\n\nSince $ Q(r)$ does not vary with $ r$, we can write this as $ P(x) \\approx c (x \\minus{} r)^k$ for some constant $ c$. Now, what do you know about a function of this form -- what does it look like? Try varying $ c$ and $ r$ and $ k$ on Mathematica, and you'll figure it out :)\n\nThe neighborhood I refer to can be very small, so you might have to zoom in. It's probably a good idea to pick other numbers that are far away from the root you are observing to create a larger neighborhood.\n\n\n\n*- By $ Q(x) \\equal{} \\frac {P(x)}{(x \\minus{} r)^k}$, I really mean that Q is the polynomial $ P(x)$ without the $ k$ factors of $ (x \\minus{} r)$; I don't mean to multiply the factors in and then divide them out, since this would just make $ Q(r) \\equal{} 0/0$. A better way of saying this is that $ Q(x) \\equal{} \\lim_{t \\to x}\\frac {P(t)}{(t \\minus{} r)^k}$[/quote]\r\n\r\nErm, ThetaPi is asking if the degree of the function is even or odd, not if the function is even or odd. Count the amount of changes in the direction of the graph and add 1. If this number is even, the degree of the function is even and vice versa.", "Solution_8": "Oh, I get what he's asking now.\r\n\r\nThe first thing I thought is that \"the degree can easily be found just by looking at the equation, and taking the highest power. Then, is it really that hard to find whether it's even or odd?\"\r\n\r\nI assumed, therefore, that he meant whether the polynomial is even or odd.\r\n\r\nI guess he meant finding the degree by the graph, but I don't see why he asked how to find whether the degree is even or odd, rather than just asking how to find the degree...", "Solution_9": "1)How do we know whether the degree of a polynomial is even or odd if we only have its graph?\r\n2)How would the multiplicity of a root influence the graph of the polynomial?\r\n3)Where do the answers come from/derive from, without invoking something not yet learned? Please explain so I do not have to memorize yet more \"magical\" rules.\r\n\r\nGiven the above questions, I'll try to interpret/summarize (shown) and explain (hidden) the many prior good answers. Feel free to tell me (as a post or by private message) how and where I failed. I also apologize for not giving due credit to everyone.\r\n\r\n1) The sign (positive or negative) at both ends of the graph tells you about the degree of the polynomial $ h(x)=ax^n+bx{}^n{}^-{}^1+ ...$ (and the sign of the leading coefficient $ a$). If the degree, $ n$, is even, at the left and right ends, the graph should go up very sharply (\"through the roof\") for $ a>0$; down very sharply for $ a>0$. If the degree is odd, the graph will head (very sharply) in different directions at both ends.\r\nThe graph 9200.png shows that $ a>0$ and $ n$ is odd.\r\nThe other two graphs are not appropriate for this question because they do not show sharp changes at both ends, like all graphs of polynomials should.[hide]azjps drifts towards calculus and talks about limits, but even without knowing the definition of limit, we can see that as $ |x|$ grows large, at some point $ (|x|>K)$ the first term $ ax^n$ of a polynomial $ h(x)=ax^n+bx{}^n{}^-{}^1+ ...$ will \"overwhelm\" the rest of the polynomial:\n$ |ax^n|>>|bx{}^n{}^-{}^1+ ...|$\nAs a consequence, for those values of $ x$ at the extremes $ ax^n$ is a good approximation of the polynomial, and the values of $ a$ and $ n$ determine the \"behavior\" of the graph at the \"extremes\".\nYou could also think of the factorization $ h(x)=a(x-r)^k(x-s)^l(x-t)^m...$, with real roots $ r$, $ s$, $ t$, and so on, and how the polynomial will change signs only at each root of odd multiplicity. The degree of the polynomial is the sum of all the root multiplicities, and it will be odd if and only if the sum of all the odd multiplicities is odd. You do not need to worry about non-real roots. If there are any, they will come in pairs, and will be reflected in your factorization by a polynomial of even degree that is positive for all values of x. For example, in the full factorization of $ x^3-1=(x-1)(x^2+x+1)$ the factor $ (x^2+x+1)$ is always positive for all real $ x$.[/hide]\n2) A polynomial changes sign (positive to negative or the reverse) only at each odd multiplicity root. Even multiplicity roots will mean $ h(r)= 0$ is a maximum or minimum at root $ r$. There is an even multiplicity root in 9203.png at $ x=-3$. Maybe there's another one in 9201.png at $ x=-3$, but the graph is not very clear. The roots at $ x=1$ in your figures would be multiple roots of odd multiplicity, because of the horizontal tangent to the curve at that point. Other roots look like single roots, where the function changes sign.[hide]If you look at the factorization $ h(x)=a(x-r)^k(x-s)^l(x-t)^m...$, you will see that the polynomial will change signs only at each root of odd multiplicity. If $ r$ is a root of even multiplicity $ (k$ is even$ )$, then $ (x-r)^k$ will be positive on both sides of $ r$. As for $ x=1$ being a multiple root, the explanation I think of would go a bit too much into calculus and mostly repeat part of what The Zuton Force stated.[/hide]", "Solution_10": "[quote=\"ThetaPi\"]It doesn't really answer my problem.[/quote]\r\n\r\nIt does read it carefully or tell me what textbook your using", "Solution_11": "[quote=\"KMST\"]If the degree, $ n$, is even, at the left and right ends, the graph should go up very sharply (\"through the roof\") for $ a > 0$; down very sharply for $ a > 0$.[/quote]\r\n\r\nI believe you meant that if it is even, then the graph goes either up [b]or[/b] down at both ends.\r\n\r\nEither y approaches infinity on both sides, or it approaches negative infinity on both sides. :wink:" } { "Tag": [ "inequalities", "topology", "triangle inequality", "advanced fields", "advanced fields unsolved" ], "Problem": "Let $ X \\equal{} \\{x \\equal{} (x_n) | x_n \\in \\mathbb{R}\\}$ be the set of all sequences of real numbers.\r\n\r\nLet $ d: X\\times X \\rightarrow \\mathbb{R}$ be defined as follows :\r\n\r\n$ d(x,y) \\equal{} \\sum\\limits_{n \\equal{} 1}^{\\infty} \\frac {1}{2^n}. \\frac {|x_n \\minus{} y_n|}{|x_n \\minus{} y_n| \\plus{} 1},$ where \r\n\r\n$ x \\equal{} (x_n), y \\equal{} (y_n)$\r\n\r\nShow that $ d$ is a metric space on $ X$ :?: :|", "Solution_1": "The only real problem is to show the triangle inequality, and this follows from the concavity of $ f(x)\\equal{}\\frac{x}{x\\plus{}1}$.", "Solution_2": "Yes, you're right ....\r\n\r\nThank you :)" } { "Tag": [ "function" ], "Problem": "Prove that the function $d(n)$, the number of divisors of $n$, and the function $\\sigma(n)$, the sum of the divisors, are both multiplicative.", "Solution_1": "[hide=\"For the first one\"]\nLet $m$ and $n$ be coprime positive integers. \nLet the divisors of of $m$ be $m_{1},m_{2},...,m_{d(m)}$ and the divisors of $n$ be $n_{1},n_{2},...,n_{d(n)}$, where all terms are distinct except that $m_{1}=n_{1}=1$. \n$k$ is a divisor of $mn$ iff $k=m_{i}n_{j}$ for some $i,j$. But the number of ways this product can be formed is $d(n)\\cdot d(m)$. Each of these divisors are distinct, so $d(mn)=d(m)\\cdot d(n)$ [/hide]", "Solution_2": "[hide=\"Other solution for 1\"]$d(mn)= d(m)d(n)$ for $m\\bot n$ follows immediately from fact that $d\\left(\\prod_{1\\leqslant i\\leqslant k}p_{i}^{\\alpha_{i}}\\right)= \\prod_{1\\leqslant i\\leqslant k}(\\alpha_{i}+1)$ and all primes in the factorisations of $m$ and $n$ are different. [/hide]\n[hide=\"Solution for 2\"]As in my solution for $d(n)$, we just notice that $\\sigma\\left(\\prod_{1\\leqslant i\\leqslant k}p_{i}^{\\alpha_{i}}\\right)= \\prod_{1\\leqslant i\\leqslant k}\\frac{p_{i}^{\\alpha_{i}+1}-1}{p_{i}-1}$. [/hide]" } { "Tag": [], "Problem": "The sum of the squares of three positive whole numbers is 165. Find the largest of the three squares.", "Solution_1": "Pretty much the only way to do this quickly and easily is to guess-and-check a little. We first see if 144 is one of them. 165 - 144 = 21, and 21 can't be written as the sum of two squares.\r\n\r\nNext let's try 121. 165 - 121 = 44, but it doesn't work either.\r\n\r\nNow we try 100. 165 - 100 = 65, and 65 is $ 8^2 \\plus{} 1^2$! Thus, our answer is $ \\boxed{100}$.", "Solution_2": "Or 65 is $7^2+4^2$. :P" } { "Tag": [ "integration", "logarithms" ], "Problem": "\u039d\u03b4\u03bf\r\n\r\n$ n! > \\left(\\frac {n}{e}\\right)^n$\r\n\r\nps. \u03c3\u03c4\u03bf \u03b2\u03b9\u03b2\u03bb\u03af\u03bf \u03c0\u03bf\u03c5 \u03b2\u03c1\u03ae\u03ba\u03b1 \u03b1\u03c5\u03c4\u03ae \u03c4\u03b7\u03bd \u03b1\u03bd\u03b9\u03c3\u03cc\u03c4\u03b7\u03c4\u03b1 \u03ad\u03c7\u03b5\u03b9 \u03bb\u03cd\u03c3\u03b7 \u03bc\u03b5 \u03bc\u03b1\u03b8\u03b7\u03bc\u03b1\u03c4\u03b9\u03ba\u03ae \u03b5\u03c0\u03b1\u03b3\u03c9\u03b3\u03ae, \u03b1\u03bb\u03bb\u03ac \u03b5\u03b3\u03ce \u03c3\u03ba\u03ad\u03c6\u03c4\u03b7\u03ba\u03b1 \u03bc\u03b9\u03b1 \u03c0\u03b9\u03bf \u03c9\u03c1\u03b1\u03af\u03b1 \u03c4\u03b7\u03bd \u03bf\u03c0\u03bf\u03af\u03b1 \u03ba\u03b1\u03b9 \u03b8\u03b1 \u03b3\u03c1\u03ac\u03c8\u03c9 \u03b1\u03c1\u03b3\u03cc\u03c4\u03b5\u03c1\u03b1 \u03b1\u03bd \u03b4\u03b5\u03bd \u03c4\u03b7\u03bd \u03b2\u03c1\u03b5\u03af\u03c4\u03b5 \u03b5\u03c3\u03b5\u03af\u03c2.", "Solution_1": "\u039a\u03b1\u03b9 \u03bc\u03b5\u03c4\u03b1 \u03bd\u03b1 \u03b4\u03b5\u03b9\u03be\u03b5\u03c4\u03b5 \u03bf\u03c4\u03b9 $ 300! > 100^{300}$ :P", "Solution_2": "\u0393\u03bd\u03c9\u03c1\u03af\u03b6\u03bf\u03c5\u03bc\u03b5 \u03cc\u03c4\u03b9 $ (1 \\plus{} {\\frac {1}{n}})^n < e < (1 \\plus{} {\\frac {1}{n}})^{n\\plus{}1}$.\u0398\u03ad\u03c4\u03bf\u03bd\u03c4\u03b1\u03c2 \u03cc\u03c0\u03bf\u03c5 $ n$ \u03c4\u03bf $ 1,2,3,...,n$ \u03b4\u03b9\u03b1\u03b4\u03bf\u03c7\u03b9\u03ba\u03ac \u03ba\u03b1\u03b9 \u03c0\u03bf\u03bb\u03bb\u03b1\u03c0\u03bb\u03b1\u03c3\u03b9\u03ac\u03b6\u03bf\u03bd\u03c4\u03b1\u03c2 \u03ba\u03b1\u03c4\u03ac \u03bc\u03ad\u03bb\u03b7 \u03ad\u03c7\u03bf\u03c5\u03bc\u03b5 qed. :)\r\n\u03a4\u03bf \u03ac\u03bb\u03bb\u03bf \u03b4\u03b5\u03bd \u03c0\u03c1\u03bf\u03bb\u03b1\u03b2\u03b1\u03af\u03bd\u03c9 \u03bd\u03b1 \u03c4\u03bf \u03ba\u03bf\u03b9\u03c4\u03ac\u03be\u03c9 \u03c4\u03ce\u03c1\u03b1...", "Solution_3": "To \u03ac\u03bb\u03bb\u03bf \u03c0\u03c1\u03bf\u03ba\u03cd\u03c0\u03c4\u03b5\u03b9 \u03ac\u03bc\u03b5\u03c3\u03b1 \u03b1\u03c0\u03cc \u03c4\u03bf \u03c0\u03c1\u03bf\u03b7\u03b3\u03bf\u03cd\u03bc\u03b5\u03bd\u03bf \u03b1\u03bd \u03b2\u03ac\u03bb\u03bf\u03c5\u03bc\u03b5 \u03cc\u03c0\u03bf\u03c5 $ n$ \u03c4\u03bf $ 300$ \u0391\u03c6\u03bf\u03cd $ (\\frac{300}{e})^{300} > 100^{300}$", "Solution_4": "[quote=\"mailo\"]\u039d\u03b4\u03bf\n\n$ n! > \\left(\\frac {n}{e}\\right)^n$\n\nps. \u03c3\u03c4\u03bf \u03b2\u03b9\u03b2\u03bb\u03af\u03bf \u03c0\u03bf\u03c5 \u03b2\u03c1\u03ae\u03ba\u03b1 \u03b1\u03c5\u03c4\u03ae \u03c4\u03b7\u03bd \u03b1\u03bd\u03b9\u03c3\u03cc\u03c4\u03b7\u03c4\u03b1 \u03ad\u03c7\u03b5\u03b9 \u03bb\u03cd\u03c3\u03b7 \u03bc\u03b5 \u03bc\u03b1\u03b8\u03b7\u03bc\u03b1\u03c4\u03b9\u03ba\u03ae \u03b5\u03c0\u03b1\u03b3\u03c9\u03b3\u03ae, \u03b1\u03bb\u03bb\u03ac \u03b5\u03b3\u03ce \u03c3\u03ba\u03ad\u03c6\u03c4\u03b7\u03ba\u03b1 \u03bc\u03b9\u03b1 \u03c0\u03b9\u03bf \u03c9\u03c1\u03b1\u03af\u03b1 \u03c4\u03b7\u03bd \u03bf\u03c0\u03bf\u03af\u03b1 \u03ba\u03b1\u03b9 \u03b8\u03b1 \u03b3\u03c1\u03ac\u03c8\u03c9 \u03b1\u03c1\u03b3\u03cc\u03c4\u03b5\u03c1\u03b1 \u03b1\u03bd \u03b4\u03b5\u03bd \u03c4\u03b7\u03bd \u03b2\u03c1\u03b5\u03af\u03c4\u03b5 \u03b5\u03c3\u03b5\u03af\u03c2.[/quote]\r\n\r\n\u0391\u03c0\u03bb\u03ce\u03c2 \u03bc\u03b5\u03bb\u03ad\u03c4\u03b7\u03c3\u03b5 \u03c4\u03bf \u03bf\u03bb\u03bf\u03ba\u03bb\u03ae\u03c1\u03c9\u03bc\u03b1 $ \\int_1^n \\log x dx$ :)\r\n\r\n\u0391\u03c0\u03cc \u03c4\u03b7\u03bd \u03ac\u03bb\u03bb\u03b7, \u03c5\u03c0\u03ac\u03c1\u03c7\u03b5\u03b9 \u03c0\u03ac\u03bd\u03c4\u03b1 \u03ba\u03b1\u03b9 \u03bf Stirling \u03c0\u03bf\u03c5 \u03b5\u03af\u03bd\u03b1\u03b9 \u03c0\u03bf\u03bb\u03cd \u03c0\u03b9\u03bf \u03c3\u03c6\u03b9\u03c7\u03c4\u03cc\u03c2: $ n! > \\sqrt {2\\pi n} \\left(\\frac {n}{e}\\right)^n$ :)\r\n\r\nCheerio,\r\n\r\nDurandal 1707", "Solution_5": "\u03a9\u03c1\u03b1\u03b9\u03bf\u03c2 Durandal! \u0391\u03c5\u03c4\u03b7 \u03b5\u03b9\u03bd\u03b1\u03b9 \u03b1\u03ba\u03c1\u03b9\u03b2\u03c9\u03c2 \u03b7 \u03bb\u03c5\u03c3\u03b7 \u03bc\u03bf\u03c5.\r\n\r\n\u0391\u03c0\u03bb\u03b1 \u03b5\u03c0\u03c1\u03b5\u03c0\u03b5 \u03bd\u03b1 \u03c3\u03ba\u03b5\u03c6\u03c4\u03b5\u03b9 \u03ba\u03b1\u03bd\u03b5\u03b9\u03c2 \u03bf\u03c4\u03b9 \r\n\r\n$ ln(2) \\plus{} ln(3) \\plus{} ... \\plus{} ln(n) > \\int_{1}^{n}ln(x) \\ dx \\Leftrightarrow$\r\n\r\n$ ln(n!) > (xlnx \\minus{} x)\\Big|_1^n\\Leftrightarrow$\r\n\r\n$ ln(n!) > n\\cdot ln(n) \\minus{} (n \\minus{} 1)\\Leftrightarrow$\r\n\r\n$ n! > \\frac {n^n}{e^{n \\minus{} 1}}\\Leftrightarrow$\r\n\r\n$ n! > \\left(\\frac {n}{e}\\right)^n$ qed :)" } { "Tag": [ "geometry proposed", "geometry" ], "Problem": "Let $ ABCD$ be a quadrilateral has $ AB\\parallel{} CD$.Denote $ M,N,P$ be midpoints of $ BC,AM,DM$\r\nNow,Find the condition of two sides $ AB,BC$ to the intersection of $ BN$ and $ DP$ is inside of $ AMD$\r\n\r\n[hide=\"result\"]$ \\frac {1}{3} < \\frac {AB}{CD} < 3$[/hide]", "Solution_1": "[quote=\"Allnames\"]Let $ ABCD$ be a quadrilateral has $ AB\\parallel{} CD$.Denote $ M,N,P$ be midpoints of $ BC,AM,DM$\nNow,Find the condition of two sides $ AB,DC$ to the intersection of $ BN$ and $ DP$ is inside of $ AMD$\n\n[hide=\"result\"]$ \\frac {1}{3} < \\frac {AB}{CD} < 3$[/hide][/quote]\r\nAny solution,see again my problem,it is my typo mistake and I have just fixed it,sorry" } { "Tag": [ "advanced fields", "advanced fields unsolved" ], "Problem": "Let $\\varphi: \\pi_{1}(\\mathbb{T}^{2})\\to \\pi_{1}(\\mathbb{T}^{2})$ be any homomorphism. Prove that there exists $f: \\mathbb{T}^{2}\\to \\mathbb{T}^{2}$ such that $f_{*}=\\varphi$. Show also that if $\\varphi$ is an isomorphism, then $f$ can be chosen to be a homomorphism.", "Solution_1": "Represent $\\mathbb T^{2}$ as $\\{(z,w)\\colon |z|=|w|=1\\}$, where $z$ and $w$ are complex numbers. The map $(z,w)\\mapsto (z^{a}w^{b}, z^{c}w^{d})$ induces a homomorphism of $\\pi_{1}(\\mathbb T^{2})$ into itself. It remains to check that these are all homomorphisms of $\\pi_{1}(\\mathbb T^{2})=\\mathbb Z^{2}$." } { "Tag": [ "function", "algebra proposed", "algebra" ], "Problem": "find all functions $f:\\mathbb R\\rightarrow\\mathbb R$ such that:\r\n $f(x^n+f(y))=y+(f(x))^n$ for all x,y in real , n is the constant and n is natural\r\nmy solution does very long and boring\r\n I find $f(x)=x$ for all x in real and $f(x)=-x$ for all x in real are two functions satisfy the condition. :D :D :D :D", "Solution_1": "This is from AMM?\r\nvery hard. :D", "Solution_2": "May be this problem can be solved in the same way as IMO 1992 A(2).\r\nWhat is AMM, N.T.Tuan?", "Solution_3": "to solve this pro, we have 2 case: n odd, n even\r\nI think IMO92 very hard because how to find: $f(0)=0$\r\nbut i think similar pro very very hard: find all $f:\\mathbb Q\\to\\mathbb Q$ such that: $f(x^2+f(y))=y+(f(x))^2$\r\n@N.T.TUAN: are you NGUYEN TRONG TUAN in pleiku, iff it true- thanks for your new book :P" } { "Tag": [ "geometry unsolved", "geometry" ], "Problem": "Let $P$ be a point outside the circle $C$. Find two points $Q$ and $R$ on the circle, such that $P,Q$ and $R$ are collinear and $Q$ is the midpopint of the segmenet $PR$. (Discuss the number of solutions).", "Solution_1": "[quote=\"Jos\u00e9\"]Let $P$ be a point outside the circle $C$. Find two points $Q$ and $R$ on the circle, such that $P,Q$ and $R$ are collinear and $Q$ is the midpopint of the segmenet $PR$. (Discuss the number of solutions).[/quote]\r\n\r\n\r\n$2*PQ^2=PQ*PR=PO^2-R^2=const$\r\nSo $PQ=\\frac{\\sqrt(PO^2-R^2)}{2}$\r\nAnd $\\le 2$ solutions\r\n\r\nV:)" } { "Tag": [ "algebra", "system of equations" ], "Problem": "Your local hardwood store sells lawn mowers. Type A, costs 137 dollars and they make a profit of 40 dollars on each one. Type B, costs 120 dollars and they make a profit of 35 dollars on each one. They expect to sell at least 100 lawn mowers this month and they need to make a profit of at least 3,675 dollars on them. If they want to order at least one type of each lawn mower, how many should they order if they want to minimize their cost?\r\n\r\na) 70 -Type A \r\n 30 -Type B \r\n\r\nb) 35 - Type A\r\n 65 - Type B\r\n\r\nc) 65 - Type A\r\n 35 - Type B \r\n\r\nd) 30 - Type A\r\n 70 - Type B", "Solution_1": "[hide=\"solution\"] They earn 40 dollars for each Type A mower and 35 dollars for each Type B mower, and they want to make 3675 dollars. So we have:\n$ 40a\\plus{}35b\\equal{}3675$ \nwhere a is the number of Type A mowers and b is the number of Type B mowers.\n\nWe also know that they expect to sell 100 lawn mowers, which leads to:\n$ a\\plus{}b\\equal{}100$\nSolving the system of equations gives us a=35, b=65. The answer is [b]B[/b].[/hide]" } { "Tag": [ "linear algebra", "matrix", "algorithm" ], "Problem": "For each positive integer n, let k(n) demote the smallest number with the property\r\n\r\nFor any nxn lower triangular matrix A with nonzero diagonal entries, there exist k(n) elementary matrices Esub1, Esub2,... Esubk(n) such that\r\n\r\nEsub1 Esub2 ... Esubk(n) A = Isubn\r\n\r\nprove that k(n)< or = [n(n+1)]/2\r\n\r\nhint: this question asks you to give an upper bound for how many elementary matrices are needed to invert an invertible lower triangular nxn matrix.", "Solution_1": "Describe the algorithm for solving a lower triangular system. How many steps does it take?", "Solution_2": "can you please show me the beginning steps? i just couldn't figure out how to start it. :( And i have no idea the outcome of it will be k(n)< or = [n(n+1)]/2 even i tried very hard to think about the process of doing Lower triangular system.\r\n\r\nThank you very much.", "Solution_3": "whenever you see $ n(n\\plus{}1)/2$ you should think of $ 1\\plus{}2\\plus{}\\ldots\\plus{}n$.\r\n\r\nstart with the $ n$th row. how many elementary row operations does it take to make the $ n$th row of $ A$ agree with the $ n$th row of $ I_n$? Call the resulting matrix $ A_1$\r\n\r\nnow how many elementary row operations, at most, does it take to make the $ n\\minus{}1$th row of $ A_1$ agree with the $ n\\minus{}1$th row of $ I_n$?\r\n\r\nIn general, how many nonzero entries are there, at most, in the $ n\\minus{}k$th row of $ A$?" } { "Tag": [], "Problem": "How many people have acne on AoPS, just curious.", "Solution_1": "ummm occasionally. i mean have u every met someone that has NEVER had acne in their whole entire teenage life?", "Solution_2": "You should've put a \"kinda\" catogory... With only 2 or 3 pimples on my face its kinda hard to say.", "Solution_3": "I do, but I still manage to look fabulous ;)\r\n\r\n[quote=\"guineapiggies\"]ummm occasionally. i mean have u every met someone that has NEVER had acne in their whole entire teenage life?[/quote]\r\nYes, I have.", "Solution_4": "yeah some hot girls just have [i]none at all[/i] (or maybe make-up covers up the minuscule amount but for practical purposes who cares)", "Solution_5": "Yes, and I HATE HATE HATE IT :mad: :mad: :mad:", "Solution_6": "[quote=\"Carnot\"]yeah some hot girls just have [i]none at all[/i] (or maybe make-up covers up the minuscule amount but for practical purposes who cares)[/quote]\r\n\r\nActually, makeup works wonders. I'm just too manly to use it.\r\n\r\nBut like, if I were to, you would barely see anything. Have you ever seen those real-life pictures of some celebs and they have ridiculous acne but you don't see it in the movies?", "Solution_7": "yea..my sister had this horrible acne that was impossible to treat. She eventually had to take this really strong prescription treatment that made her tired and pale. She said it was worth it though.", "Solution_8": "Just use Benzoyl Peroxide whenever a pimple comes up", "Solution_9": "[quote=\"RussianRocket\"]Just use Benzoyl Peroxide whenever a pimple comes up[/quote]\r\n\r\nbenzoyl peroxide works wonders :D", "Solution_10": "[quote=\"crazychinesemaniac\"]yea..my sister had this horrible acne that was impossible to treat. She eventually had to take this really strong prescription treatment that made her tired and pale. She said it was worth it though.[/quote]\r\n\r\nWas it Acutane, or some other form of isotretinoin? If so, I'm interested in more details.\r\n\r\nI have horrendous acne that nothing has succeeded in treating. For the last few years, it's really died down, so its quite sparse, but I can't kill it off. It just won't go away. Like an effing zombie. I'm wondering if I may need something like that, or if time will take care of it.", "Solution_11": "RussianRocket wrote: [quote]\nJust use Benzoyl Peroxide whenever a pimple comes up \n\nguineapiggies wrote: [quote]\nbenzoyl peroxide works wonders[/quote][/quote]\r\n\r\nhaha yay benzoyl peroxide! That's what I use. :)", "Solution_12": "benzoyl peroxide, huh? i've gotta try... SO MUCH ACNE!!!!! (what's worse, i have a habit of picking at it, too. :( )", "Solution_13": "Benzoyl peroxide is basically Proactiv... although Proactiv has some other stuff in it too" } { "Tag": [ "geometry", "incenter", "circumcircle", "trapezoid", "complex numbers", "geometry proposed" ], "Problem": "Let $K$ and $L$ be two points on the arcs $AB$ and $BC$ of the circumcircle of a triangle $ABC$, respectively, such that $KL\\parallel AC$. Show that the incenters of triangles $ABK$ and $CBL$ are equidistant from the midpoint of the arc $ABC$ of the circumcircle of triangle $ABC$.", "Solution_1": "Denote by $I_1$ and $I_2$ the inceters of $\\triangle{AKB}$ and $\\triangle{BLC}$, respectively.\r\nExtend $BI_1$ and $BI_2$ to meet the circumcircle(for the second time) of $\\triangle{ABC}$ at $X$ and $Y$, then obviously $XI_1=YI_2$.\r\nSince $M$- the midpoint of $\\widehat{ABC}$ is found on the perpendicular bisectors of $AC$, $XY$ and $KL$\r\nwe can conclude that $\\triangle{MI_1X}=\\triangle{MI_2Y}$ and consequently $MI_1=MI_2$.", "Solution_2": "I think that the two segments are equal to each other iff B is midpoint of arc ABC. If I am wrong, I'm sorry.", "Solution_3": "CAN YOU EXPLAIN [quote]I1X=I2Y?[/quote]", "Solution_4": "I too think that this equality holds only when $AB=BC$\nLet us assume $YI_2=XI_1$\nIt is easy to observe that $YI_2=YB=YC$ and $XI_1=XA=XB \\Longrightarrow XB=YC \\Longrightarrow BC=XY$\nand $AX=YB \\Longrightarrow AB=XY \\Longrightarrow AB=BC$", "Solution_5": "I'll post 2 solutions.One elementary and one by complex bashing:\nLet $I_1$ and $I_2$ be the incenters of triangles $BKA$ and $BLC$. Now we intersect $BI_1$ and $BI_2$ with the circumcircle and we denote those points as $X$ and $Y$ respectively.Note that $XACY$ is a trapezoid so $XA=YC$.Now, it is well-known that $X$ is the circumcenter of a triangle $AKI_1$ so $AX=XI_1$.By the same reasoning $YC=YI_2$ so we conclude that $XI_1=YI_2$. Now because $MX=MY$ (trivial) and $\\angle BXM= \\angle BYM$ we have that $\\triangle MI_1X= \\triangle MI_2Y$ so $MI_1=MI_2$.", "Solution_6": "Complex bash:\nAsign points $A , B , C, K$ complex numbers: $a^2, b^2, c^2, k^2$. Because $KL \\parallel AC$ we get $l^2=\\frac{a^2c^2}{k^2}$.\n$m=ac$.\n$x=\\frac{m-i_1}{m-i_2}= \\frac{ac+ab+ak+bk}{ac+cb+cl+bl}=\\frac{ac+ab+ak+bk}{ac+cb+\\frac{ac}{k}(b+c)} =\\frac{ac+ab+ak+bk}{\\frac{c}{k}(ak+bk+ab+ac)}=\\frac{k}{c}$.\nNow because $\\frac{m-i_1}{m-i_2}= \\overline{(\\frac{m-i_2}{m-i_1})}$ it follows $MI_1=MI_2$.", "Solution_7": "[b]S[/b]olved with [b]mathscrazy, everythingpi3141592[/b]\n\n[hide = solution]\nLet $X$ be the midpoint of arc $ABC$. Let $I_1, I_2$ be the incentres of triangles $BAK$ and $BCL$ respectively. Further, let $M_1, M_2$ be the midpoint of minor arcs $AK, CL$. Consider the spiral similarity mapping segment $M_1I_1$ to $M_2I_2$. Because $M_1I_1 = M_1B = CM_2 = M_2I_2$, it follows that this spiral similarity is in fact a rotation. Moreover the centre of this rotation lies on $\\odot(AM_1M_2)$. As $XM_1 = XM_2$, it follows that the centre is $X$ and then we have $XI_1 = XI_2$ as well, as desired.\n[/hide]", "Solution_8": "[asy] /* Geogebra to Asymptote conversion, documentation at artofproblemsolving.com/Wiki go to User:Azjps/geogebra */\nimport graph; size(12cm); \nreal labelscalefactor = 0.5; /* changes label-to-point distance */\npen dps = linewidth(0.7) + fontsize(10); defaultpen(dps); /* default pen style */ \npen dotstyle = black; /* point style */ \nreal xmin = -2.6867010823130566, xmax = 12.551672708712484, ymin = -4.521585627577025, ymax = 3.2434767165979923; /* image dimensions */\npen zzttff = rgb(0.6,0.2,1); pen fuqqzz = rgb(0.9568627450980393,0,0.6); \n\ndraw((3.62,2.55)--(0.62,-3.23)--(9.78,-3.17)--cycle, linewidth(0.4) + zzttff); \ndraw((1.3978204105733785,-0.3996390992686019)--(5.160584999681438,2.8173567152998213)--(0.4478262236076711,-1.6606262900810957)--cycle, linewidth(0.8) + fuqqzz); \ndraw((8.61228166294228,-0.7313401678997445)--(5.160584999681438,2.8173567152998213)--(9.931600403828496,-1.5985054985075964)--cycle, linewidth(0.8) + fuqqzz); \n /* draw figures */\ndraw((3.62,2.55)--(0.62,-3.23), linewidth(0.4) + zzttff); \ndraw((0.62,-3.23)--(9.78,-3.17), linewidth(0.4) + zzttff); \ndraw((9.78,-3.17)--(3.62,2.55), linewidth(0.4) + zzttff); \ndraw(circle((5.191709472981987,-1.9343128752501666), 4.751771525504538), linewidth(0.4)); \ndraw((xmin, 0.0065502183406113594*xmin-0.12670123550477158)--(xmax, 0.0065502183406113594*xmax-0.12670123550477158), linewidth(0.4)); /* line */\ndraw((0.7993391990866892,-0.12146538922254435)--(0.62,-3.23), linewidth(0.4)); \ndraw((9.559954775806013,-0.06408144439687188)--(9.78,-3.17), linewidth(0.4)); \ndraw((0.7993391990866892,-0.12146538922254435)--(3.62,2.55), linewidth(0.4)); \ndraw((3.62,2.55)--(9.559954775806013,-0.06408144439687188), linewidth(0.4)); \ndraw((0.7993391990866892,-0.12146538922254435)--(5.160584999681438,2.8173567152998213), linewidth(0.4)); \ndraw((5.160584999681438,2.8173567152998213)--(9.559954775806013,-0.06408144439687188), linewidth(0.4)); \ndraw((3.62,2.55)--(9.931600403828496,-1.5985054985075964), linewidth(0.4)); \ndraw((9.931600403828496,-1.5985054985075964)--(9.78,-3.17), linewidth(0.4)); \ndraw((9.931600403828496,-1.5985054985075964)--(9.559954775806013,-0.06408144439687188), linewidth(0.4)); \ndraw((0.4478262236076711,-1.6606262900810957)--(0.7993391990866892,-0.12146538922254435), linewidth(0.4)); \ndraw((0.4478262236076711,-1.6606262900810957)--(0.62,-3.23), linewidth(0.4)); \ndraw((1.3978204105733785,-0.3996390992686019)--(5.160584999681438,2.8173567152998213), linewidth(0.8) + fuqqzz); \ndraw((5.160584999681438,2.8173567152998213)--(0.4478262236076711,-1.6606262900810957), linewidth(0.8) + fuqqzz); \ndraw((0.4478262236076711,-1.6606262900810957)--(1.3978204105733785,-0.3996390992686019), linewidth(0.8) + fuqqzz); \ndraw((8.61228166294228,-0.7313401678997445)--(5.160584999681438,2.8173567152998213), linewidth(0.8) + fuqqzz); \ndraw((5.160584999681438,2.8173567152998213)--(9.931600403828496,-1.5985054985075964), linewidth(0.8) + fuqqzz); \ndraw((9.931600403828496,-1.5985054985075964)--(8.61228166294228,-0.7313401678997445), linewidth(0.8) + fuqqzz); \n /* dots and labels */\ndot((3.62,2.55),dotstyle); \nlabel(\"$B$\", (3.6644915286740245,2.6599749219490025), NE * labelscalefactor); \ndot((0.62,-3.23),dotstyle); \nlabel(\"$A$\", (0.6684342369186348,-3.1189370827477223), NE * labelscalefactor); \ndot((9.78,-3.17),dotstyle); \nlabel(\"$C$\", (9.82492393756432,-3.0628311409545503), NE * labelscalefactor); \ndot((5.160584999681438,2.8173567152998213),linewidth(4pt) + dotstyle); \nlabel(\"$N$\", (5.20179433380694,2.9068410658389596), NE * labelscalefactor); \ndot((0.7993391990866892,-0.12146538922254435),dotstyle); \nlabel(\"$K$\", (0.8479732506567855,-0.010667907405988926), NE * labelscalefactor); \ndot((9.559954775806013,-0.06408144439687188),linewidth(4pt) + dotstyle); \nlabel(\"$L$\", (9.600500170391632,0.022995657669914325), NE * labelscalefactor); \ndot((1.3978204105733785,-0.3996390992686019),linewidth(4pt) + dotstyle); \nlabel(\"$D$\", (1.4426962336644098,-0.3136399930891182), NE * labelscalefactor); \ndot((8.61228166294228,-0.7313401678997445),linewidth(4pt) + dotstyle); \nlabel(\"$E$\", (8.657920348266341,-0.6390544554895163), NE * labelscalefactor); \ndot((0.4478262236076711,-1.6606262900810957),linewidth(4pt) + dotstyle); \nlabel(\"$F$\", (0.4888952231804841,-1.570413089256173), NE * labelscalefactor); \ndot((9.931600403828496,-1.5985054985075964),linewidth(4pt) + dotstyle); \nlabel(\"$G$\", (9.982020574585203,-1.5143071474630008), NE * labelscalefactor); \nclip((xmin,ymin)--(xmin,ymax)--(xmax,ymax)--(xmax,ymin)--cycle); \n /* end of picture */[/asy]\n Very Easy problem.\n Let $D$ and $E$ denote the incenters of $\\triangle ABK$ and $\\triangle CBL$\n Let $F = BD \\cap ABC$, $G = BE \\cap ABC$\n [b]CLaim:[/b]$\\triangle NDF \\equiv \\triangle NEG$\n[hide=Proof]By Incenter-Excenter Lemma and using $AK = LC$, $FD = KF = FA = GC = GL = GE$,\nClearly $NF = NG$, $\\measuredangle NFD = \\measuredangle NFB = \\measuredangle NGB = \\measuredangle NGE$, hence by SAS these triangles are congruent[/hide]\nBy Claim, we're done." } { "Tag": [ "inequalities", "algebra", "polynomial", "inequalities unsolved" ], "Problem": "Let $a,b,c>0$ and $ab+bc+ca=3$.\r\nProve that: $\\sum \\frac{a}{\\sqrt{\\frac{5}{4}a+b}}\\ge 2$.\r\nIt's true or false :) ?", "Solution_1": "If $ x,y,z$ be nonnegative numbers such that $ yz \\plus{} zx \\plus{} xy \\equal{} 3,$ then\r\n\r\n$ \\frac {x}{\\sqrt {5x \\plus{} 4 y}} \\plus{} \\frac {y}{\\sqrt {5y \\plus{} 4z}} \\plus{} \\frac {z}{\\sqrt {5z \\plus{} 4x}}\\geq 1,$\r\n\r\nwith equality if and only if $ x \\equal{} y \\equal{} z \\equal{} 1.$\r\n\r\n[b]Proof[/b] Using the H\u00f6lder inequality,\r\n\r\n$ \\left(\\sum_{cyc}{\\frac {x}{\\sqrt {5x \\plus{} 4 y}}}\\right)^4\\left[\\sum_{cyc}{x(5x \\plus{} 4y)^2(2x \\plus{} 2y \\plus{} 3z)^5}\\right]$\r\n\r\n$ \\geq\\left[\\sum_{cyc}{x(2x \\plus{} 2y \\plus{} 3z)}\\right]^5,$\r\n\r\nit remains to prove that\r\n\r\n$ \\left[\\sum_{cyc}{x(2x \\plus{} 2y \\plus{} 3z)}\\right]^5\\geq\\sum_{cyc}{x(5x \\plus{} 4y)^2(2x \\plus{} 2y \\plus{} 3z)^5}.$\r\n\r\nBut \r\n\r\n$ 3\\left[\\sum_{cyc}{x(2x \\plus{} 2y \\plus{} 3z)}\\right]^5 \\minus{} \\left(\\sum_{sym}{yz}\\right)\\left[\\sum_{cyc}{x(5x \\plus{} 4y)^2(2x \\plus{} 2y \\plus{} 3z)^5}\\right]$\r\n\r\n$ \\equiv F(x,y,z) \\equal{} F(x,x \\plus{} s,x \\plus{} t)$\r\n\r\n$ \\equal{} 229467\\left(s^2 \\minus{} st \\plus{} t^2\\right) x^8\\plus{} 49\\left(10709s^3 \\plus{} 16308s^2t \\minus{} 10971st^2 \\plus{} 10709t^3\\right)x^7$\r\n\r\n$ \\plus{} 7\\left(78165s^4 \\plus{} 328819s^3t \\plus{} 102881s^2t^2 \\minus{} 116738st^3 \\plus{} 78165t^4\\right)x^6$\r\n\r\n$ \\plus{} \\Big[(2s \\minus{} t)^2\\left(89405s^3 \\plus{} 642467s^2t \\plus{} 713059st^2 \\plus{} 357621t^3\\right)$\r\n\r\n$ \\plus{} s^5 \\plus{} 2s^4t \\plus{} 2649817s^3t^2 \\plus{} 303528s^2t^3\\Big]x^5$\r\n\r\n$ \\plus{} \\Big[(5s \\minus{} 2t)^2\\left(6709s^4 \\plus{} 49310s^3t \\plus{} 145638s^2t^2 \\plus{} 130444st^3 \\plus{} 41934t^4\\right)$\r\n\r\n$ \\plus{} 13s^6 \\plus{} 10s^5t \\plus{} 125919s^4t^2 \\plus{} 791450s^3t^3 \\plus{} 3s^2t^4 \\plus{} st^5 \\plus{} 2t^6\\Big]x^4$\r\n\r\n$ \\plus{} \\Big[(13s \\minus{} 5t)^2\\left(340s^5 \\plus{} 2146s^4t \\plus{} 8715s^3t^2 \\plus{} 14649s^2t^3 \\plus{} 9950st^4 \\plus{} 2302t^5\\right)$\r\n\r\n$ \\plus{} 92s^7 \\plus{} 121s^6t \\plus{} 55s^5t^2 \\plus{} 164s^4t^3 \\plus{} 8555s^3t^4 \\plus{} 16987s^2t^5 \\plus{} 3st^6 \\plus{} 2t^7\\Big]x^3$\r\n\r\n$ \\plus{} \\Big[(7s \\minus{} 3t)^2\\left(271s^6 \\plus{} 1405s^5t \\plus{} 6539s^4t^2 \\plus{} 13380s^3t^3 \\plus{} 15415s^2t^4 \\plus{} 7872st^5\\plus{} 1477t^6\\right)$\r\n\r\n$ \\plus{} 17s^8 \\plus{} 9s^7t \\plus{} 5s^6t^2 \\plus{} 126969s^5t^3 \\plus{} 14939s^4t^4 \\plus{} 6s^3t^5 \\plus{} 4s^2t^6 \\plus{} 2st^7 \\plus{} 3t^8\\Big]x^2$\r\n\r\n$ \\plus{} \\Big[2(2s \\minus{} t)^2\\big(220s^7 \\plus{} 1046s^6t \\plus{} 4545s^5t^2 \\plus{} 13215s^4t^3 \\plus{} 15924s^3t^4 \\plus{} 13664s^2t^5$\r\n\r\n$ \\plus{} 5592st^6 \\plus{} 880t^7\\big) \\plus{} 12837s^6t^3 \\plus{} 57493s^5t^4\\Big]x$\r\n\r\n$ \\plus{} (2s \\minus{} t)^2\\big(24s^8 \\plus{} 124s^7t \\plus{} 418s^6t^2 \\plus{} 1609s^5t^3 \\plus{} 3573s^4t^4 \\plus{} 4272s^3t^5 \\plus{} 2720s^2t^6$\r\n\r\n$ \\plus{} 784st^7 \\plus{} 96t^8\\big) \\plus{} s^6t^4 \\plus{} 1025s^4t^6 \\plus{} 1432s^3t^7\\geq0,$\r\n\r\nwhich is clearly true for $ x \\equal{} \\min\\{x,y,z\\}.$\r\n\r\n\r\nWith the same conditions, \r\n\r\n$ \\frac {x}{\\sqrt {kx \\plus{} y}} \\plus{} \\frac {y}{\\sqrt {ky \\plus{} z}} \\plus{} \\frac {z}{\\sqrt {kz \\plus{} x}}\\geq \\frac {3}{\\sqrt {k \\plus{} 1}}$\r\n\r\nholds if and only if $ 0\\leq k\\leq k_1 \\equal{} 1.3195\\cdots,$ which is the greatest real root of the following irreducible polynomial over $ \\mathbb{Q}:$\r\n\r\n$ 176k^{12} \\plus{} 18048k^{11} \\plus{} 18864k^{10} \\plus{} 2623976k^9 \\minus{} 5649012k^8 \\plus{} 7045137k^7 \\plus{} 5700612k^6$\r\n\r\n$ \\minus{} 5071266k^5 \\minus{} 7421112k^4 \\minus{} 5260687k^3 \\minus{} 2321664k^2 \\minus{} 589824k \\minus{} 65536;$\r\n\r\nwith equality if $ k \\equal{} k_1,x \\equal{} 0, y \\equal{} 1.2429\\cdots$ is the greatest real root of the following irreducible polynomial \r\n\r\n$ 16y^{12} \\minus{} 384y^{10} \\plus{} 1701y^9 \\plus{} 2772y^8 \\minus{} 18954y^7 \\plus{} 36504y^6 \\minus{} 13851y^5 \\plus{} 9072y^4$\r\n\r\n$ \\minus{} 69984y^3 \\plus{} 71928y^2 \\minus{} 26244y \\plus{} 2916,$\r\n\r\n$ z \\equal{} 2.4136\\cdots$ is the second real root of the following irreducible polynomial \r\n\r\n$ 4z^{12} \\minus{} 108z^{11} \\plus{} 888z^{10} \\minus{} 2592z^9 \\plus{} 1008z^8 \\minus{} 4617z^7 \\plus{} 36504z^6 \\minus{} 56862z^5 \\plus{} 24948z^4$\r\n\r\n$ \\plus{} 45927z^3 \\minus{} 31104z^2 \\plus{} 11664.$", "Solution_2": "can anyone provide some cleaner solution without such calculations??", "Solution_3": "[quote=\"Ji Chen\"]If $ x,y,z$ be nonnegative numbers such that $ yz \\plus{} zx \\plus{} xy \\equal{} 3,$ then\n\n$ \\frac {x}{\\sqrt {5x \\plus{} 4 y}} \\plus{} \\frac {y}{\\sqrt {5y \\plus{} 4z}} \\plus{} \\frac {z}{\\sqrt {5z \\plus{} 4x}}\\geq 1,$\n\nwith equality if and only if $ x \\equal{} y \\equal{} z \\equal{} 1.$\n[/quote]\nWe can use also the following Holder.\n$\\left(\\sum_{cyc} \\frac {x}{\\sqrt {5x \\plus{} 4 y}}\\right)^2\\sum_{cyc}x(5x+4y)(x+2y+4z)^3\\geq\\left(\\sum_{cyc}(x^2+6xy)\\right)^3$.\nThus, it remains to prove that $3\\left(\\sum_{cyc}(x^2+6xy)\\right)^6\\geq(xy+xz+yz)\\left(\\sum_{cyc}x(5x+4y)(x+2y+4z)^3\\right)^2$, \nwhich is true.", "Solution_4": "whose computer program is the best?\n\nJi Chen, szl6208, xzlbq or Grotex?", "Solution_5": "[quote=mudok]whose computer program is the best?\n\nJi Chen, szl6208, xzlbq or Grotex?[/quote]\n\n\u90fd\u79d1\uff1a\nJi Chen is very famous\u3002\n\n szl6208 be good at sos,as you see.\n\nxzlbq ,Automatic inequality discovery,\n\nand Grotex is The bright younger generation\u3002\n\n\n ", "Solution_6": "[quote=xzlbq]\nxzlbq ,Automatic inequality discovery [/quote]\n\n:lol: yes! you have the greatest number of inequalities.", "Solution_7": "[quote=xzlbq][quote=mudok]whose computer program is the best?\n\nJi Chen, szl6208, xzlbq or Grotex?[/quote]\n\n\u90fd\u79d1\uff1a\nJi Chen is very famous\u3002\n\n szl6208 be good at sos,as you see.\n\nxzlbq ,Automatic inequality discovery,\n\nand Grotex is The bright younger generation\u3002[/quote]\n\nhow can we get them?", "Solution_8": "[quote=mudok]whose computer program is the best?\n\nJi Chen, szl6208, xzlbq or Grotex?[/quote]\n\nwe needn't their tools at all", "Solution_9": "[quote=xzlbq][quote=mudok]whose computer program is the best?\n\nJi Chen, szl6208, xzlbq or Grotex?[/quote]\n\n\u90fd\u79d1\uff1a\nJi Chen is very famous\u3002\n\n szl6208 be good at sos,as you see.\n\nxzlbq ,Automatic inequality discovery,\n\nand Grotex is The bright younger generation\u3002[/quote]\n\nTheir software can only solve trivial polynomial inequalities. xlzq doesn't even need software and can beat all of these guys with sw easily. ", "Solution_10": "[quote=xlzq][quote=xzlbq][quote=mudok]whose computer program is the best?\n\nJi Chen, szl6208, xzlbq or Grotex?[/quote]\n\n\u90fd\u79d1\uff1a\nJi Chen is very famous\u3002\n\n szl6208 be good at sos,as you see.\n\nxzlbq ,Automatic inequality discovery,\n\nand Grotex is The bright younger generation\u3002[/quote]\n\nTheir software can only solve trivial polynomial inequalities. xlzq doesn't even need software and can beat all of these guys with sw easily.[/quote]\nI believe that\uff0c\n\nI remember your problems,are very very hard!\nas\n\n[quote=xlzq]$a,b,c >0$ and $a+b+c=1$, prove that\n\\[ 2\\geqslant a^{k_ob^2}+b^{k_oc^2}+c^{k_oa^2}\\]\nwhere $k_o=9 \\left( \\frac{ln3-ln2}{ln3} \\right) \\approx 3.32163$\n\nIt is a little bit more challenge than a trivial one posted [url=http://math.stackexchange.com/questions/2176571/if-ab-1-so-a4b2b4a2-leq1]here[/url][/quote]", "Solution_11": "\u662f\u9aa1\u5b50\u662f\u9a6c\u62c9\u51fa\u6765\u6e9c,\u4f60\u505a\u597d\u51c6\u5907\uff0c\n\uff08You're ready for the test....\uff09", "Solution_12": "[quote=xlzq][quote=xzlbq][quote=mudok]whose computer program is the best?\n\nJi Chen, szl6208, xzlbq or Grotex?[/quote]\n\n\u90fd\u79d1\uff1a\nJi Chen is very famous\u3002\n\n szl6208 be good at sos,as you see.\n\nxzlbq ,Automatic inequality discovery,\n\nand Grotex is The bright younger generation\u3002[/quote]\n\nTheir software can only solve trivial polynomial inequalities. xlzq doesn't even need software and can beat all of these guys with sw easily.[/quote]\n\nAre you crazy?" } { "Tag": [ "geometry", "3D geometry", "ratio", "geometry unsolved" ], "Problem": "Consider a cylinder and a cone with a common base such that the volume of the\r\npart of the cylinder enclosed in the cone equals the volume of the part of the cylinder outside the cone. Find the ratio of the height of the cone to the height of the cylinder.", "Solution_1": "Coz the small cone is similar to the large one, so the ratio of the height of these two cones is $ 1: \\sqrt[3]{2}$.\r\nSo the required ratio is $ \\sqrt[3]{2}: \\sqrt[3]{2}\\minus{}1$." } { "Tag": [ "inequalities", "inequalities proposed" ], "Problem": "$ a,b,c>0$ \r\n\r\nprove that !\r\n\r\n$ \\frac{a^2}{b}\\plus{}\\frac{b^2}{c}\\plus{}\\frac{c^2}{a} \\ge \\frac{(a\\plus{}b\\plus{}c)(a^2\\plus{}b^2\\plus{}c^2)}{ab\\plus{}bc\\plus{}ac}$", "Solution_1": "[quote=\"bigbang195\"]$ a,b,c > 0$ \n\nprove that !\n\n$ \\frac {a^2}{b} \\plus{} \\frac {b^2}{c} \\plus{} \\frac {c^2}{a} \\ge \\frac {(a \\plus{} b \\plus{} c)(a^2 \\plus{} b^2 \\plus{} c^2)}{ab \\plus{} bc \\plus{} ac}$[/quote]\r\n\r\n\r\n$ \\Longleftrightarrow \\sum_{cyc} (a^4c^2 \\minus{} a^3b^2c)\\geq 0$.\r\n\r\n$ \\Longleftrightarrow (c^2a^2 \\plus{} ( \\minus{} cb^2 \\plus{} c^3 \\plus{} bc^2)a \\plus{} b^4 \\plus{} c^4 \\minus{} b^3c)(a \\minus{} b)(a \\minus{} c)$ $ \\plus{} (b^2 \\plus{} bc \\plus{} c^2)(ab \\plus{} ca \\minus{} bc)(b \\minus{} c)^2\\geq 0$\r\n\r\nassume $ a \\equal{} max{(a,b,c)}$\r\n\r\nwe can prove\r\n\r\n$ ab \\plus{} ca \\minus{} bc\\geq 0$\r\n\r\n$ f(a) \\equal{} c^2a^2 \\plus{} ( \\minus{} cb^2 \\plus{} c^3 \\plus{} bc^2)a \\plus{} b^4 \\plus{} c^4 \\minus{} b^3c\\geq 0$.\r\n\r\n$ (\\Delta \\equal{} \\minus{} c^2(3c^2 \\minus{} 5bc \\plus{} 3b^2)(b^2 \\plus{} bc \\plus{} c^2)\\le 0)$\r\n\r\nso.........\r\n\r\n :lol:", "Solution_2": "[quote=\"hedeng123\"][quote=\"bigbang195\"]$ a,b,c > 0$ \n\nprove that !\n\n$ \\frac {a^2}{b} \\plus{} \\frac {b^2}{c} \\plus{} \\frac {c^2}{a} \\ge \\frac {(a \\plus{} b \\plus{} c)(a^2 \\plus{} b^2 \\plus{} c^2)}{ab \\plus{} bc \\plus{} ac}$[/quote]\n\n\n$ \\Longleftrightarrow \\sum_{cyc} (a^4c^2 \\minus{} a^3b^2c)\\geq 0$.\n\n[/quote]\r\n\r\nBy AM-GM inequality,\r\n\r\n$ \\sum{a^4c^2} \\equal{} \\frac {1}{2} \\sum{(a^4c^2 \\plus{} b^4a^2)} \\ge \\sum{a^3b^2c}$.\r\n\r\nWe've done. #", "Solution_3": "[quote=\"frankvista\"][quote=\"hedeng123\"][quote=\"bigbang195\"]$ a,b,c > 0$ \n\nprove that !\n\n$ \\frac {a^2}{b} \\plus{} \\frac {b^2}{c} \\plus{} \\frac {c^2}{a} \\ge \\frac {(a \\plus{} b \\plus{} c)(a^2 \\plus{} b^2 \\plus{} c^2)}{ab \\plus{} bc \\plus{} ac}$[/quote]\n\n\n$ \\Longleftrightarrow \\sum_{cyc} (a^4c^2 \\minus{} a^3b^2c)\\geq 0$.\n\n[/quote]\n\nBy AM-GM inequality,\n\n$ \\sum{a^4c^2} \\equal{} \\frac {1}{2} \\sum{(a^4c^2 \\plus{} b^4a^2)} \\ge \\sum{a^3b^2c}$.\n\nWe've done. #[/quote]\r\n :blush:", "Solution_4": "Let $ x_i\\geq 0,i\\equal{}1,2,3,4$\r\n\r\n$ {\\frac {\\left( {\\it x_1} \\plus{} {\\it x_2} \\plus{} {\\it x_3} \\plus{} {\\it x_4} \\right) \\left( {{ \\it x_1}}^{2} \\plus{} {{\\it x_2}}^{2} \\plus{} {{\\it x_3}}^{2} \\plus{} {{\\it x_4}}^{2} \\right) }{{ \\it x_1}\\,{\\it x_2} \\plus{} {\\it x_2}\\,{\\it x_3} \\plus{} {\\it x_3}\\,{\\it x_4} \\plus{} {\\it x_4}\\,{ \\it x_1}}}\\leq {\\frac {{{\\it x_1}}^{2}}{{\\it x_2}}} \\plus{} {\\frac {{{\\it x_2}}^{2 }}{{\\it x_3}}} \\plus{} {\\frac {{{\\it x_3}}^{2}}{{\\it x_4}}} \\plus{} {\\frac {{{\\it x_4}}^{2 }}{{\\it x_1}}}$\r\nBQ\r\n\r\nn-vari:\r\nLet $ x_i\\geq 0,i\\equal{}1,..,n,n\\geq2,x_{n\\plus{}1}\\equal{}x_1$,prove that:\r\n\r\n$ \\sum_{i\\equal{}1}^n{\\frac{x_i^2}{x_{i\\plus{}1}}}\\geq \\sum_{i\\equal{}1}^n{x_i}\\frac{\\sum_{i\\equal{}1}^n{x_i^2}}{\\sum_{i\\equal{}1}^n{x_ix_{i\\plus{}1}}}$\r\n\r\nBQ", "Solution_5": "[quote=\"hedeng123\"][/quote][quote=\"frankvista\"][quote=\"hedeng123\"][quote=\"bigbang195\"]$ a,b,c > 0$ \n\nprove that !\n\n$ \\frac {a^2}{b} + \\frac {b^2}{c} + \\frac {c^2}{a} \\ge \\frac {(a + b + c)(a^2 + b^2 + c^2)}{ab + bc + ac}$[/quote]\n\n\n$ \\Longleftrightarrow \\sum_{cyc} (a^4c^2 - a^3b^2c)\\geq 0$.\n\n[/quote]\n\nBy AM-GM inequality,\n\n$ \\sum{a^4c^2} = \\frac {1}{2} \\sum{(a^4c^2 + b^4a^2)} \\ge \\sum{a^3b^2c}$.\n\nWe've done. #[/quote][quote=\"hedeng123\"][/quote]\r\n \r\n:blush:\r\n\r\n[hide=\"\u70b9\"]\u6211\u77e5\u9053\u5747\u503c\u4e0d\u7b49\u5f0f\u53ef\u4ee5\u505a\uff0c\u6ca1\u60f3\u5230\u8fd9\u4e48\u7b80\u5355[/hide]", "Solution_6": "[quote=\"bigbang195\"]$ a,b,c > 0$ \n\nprove that !\n\n$ \\frac {a^2}{b} \\plus{} \\frac {b^2}{c} \\plus{} \\frac {c^2}{a} \\ge \\frac {(a \\plus{} b \\plus{} c)(a^2 \\plus{} b^2 \\plus{} c^2)}{ab \\plus{} bc \\plus{} ac}$[/quote]\r\n\r\nyou are very good!\r\n\r\nplease see nice and strong:\r\n\r\n$ \\frac{a^3}{b}\\plus{}\\frac{b^3}{c}\\plus{}\\frac{c^3}{a}\\geq (a^2\\plus{}b^2\\plus{}c^2)\\frac{a^3\\plus{}b^3\\plus{}c^3}{a^2b\\plus{}b^2c\\plus{}c^2a}.$\r\nBQ", "Solution_7": "[quote=\"xzlbq\"]Let $ x_i\\geq 0,i \\equal{} 1,2,3,4$\n\n$ {\\frac {\\left( {\\it x_1} \\plus{} {\\it x_2} \\plus{} {\\it x_3} \\plus{} {\\it x_4} \\right) \\left( {{\\it x_1}}^{2} \\plus{} {{\\it x_2}}^{2} \\plus{} {{\\it x_3}}^{2} \\plus{} {{\\it x_4}}^{2} \\right) }{{\\it x_1}\\,{\\it x_2} \\plus{} {\\it x_2}\\,{\\it x_3} \\plus{} {\\it x_3}\\,{\\it x_4} \\plus{} {\\it x_4}\\,{\\it x_1}}}\\leq {\\frac {{{\\it x_1}}^{2}}{{\\it x_2}}} \\plus{} {\\frac {{{\\it x_2}}^{2 }}{{\\it x_3}}} \\plus{} {\\frac {{{\\it x_3}}^{2}}{{\\it x_4}}} \\plus{} {\\frac {{{\\it x_4}}^{2 }}{{\\it x_1}}}$\nBQ\n\nn-vari:\nLet $ x_i\\geq 0,i \\equal{} 1,..,n,n\\geq2,x_{n \\plus{} 1} \\equal{} x_1$,prove that:\n\n$ \\sum_{i \\equal{} 1}^n{\\frac {x_i^2}{x_{i \\plus{} 1}}}\\geq \\sum_{i \\equal{} 1}^n{x_i}\\frac {\\sum_{i \\equal{} 1}^n{x_i^2}}{\\sum_{i \\equal{} 1}^n{x_ix_{i \\plus{} 1}}}$\n\nBQ[/quote]\r\n\r\n\r\n$ n\\equal{}3$ The inequality holds.\r\n\r\ntry \r\n\r\n$ n \\equal{} 4,x_1 \\equal{} \\frac {1}{30},x_2 \\equal{} 150,x_3 \\equal{} \\frac {1}{11},x_4 \\equal{} 1$.\r\n\r\n\r\nexpand(subs(n=4,sum(x[i]^2/(x[i+1]),i=1..n)-sum(x[i],i=1..n)*sum(x[i]^2,i=1..n)/(sum(x[i]*x[i+1],i=1..n))));\r\nsubs(x[4+1]=x[1],%);\r\nxprove(%>=0);", "Solution_8": "Let $ x_1,x_2,x_3,x_4>0$,prove that:\r\n$ \\frac{x_1^3}{x_2}\\plus{}\\frac{x_2^3}{x_3}\\plus{}\\frac{x_3^3}{x_4}\\plus{}\\frac{x_4^3}{x_1}\\geq \\frac{(x_1^2\\plus{}x_2^2\\plus{}x_3^2\\plus{}x_4^2)^2}{x_1x_2\\plus{}x_2x_3\\plus{}x_3x_4\\plus{}x_1x_4}.$\r\nBQ", "Solution_9": "[quote=\"xzlbq\"][quote=\"bigbang195\"]$ a,b,c > 0$ \n\nprove that !\n\n$ \\frac {a^2}{b} \\plus{} \\frac {b^2}{c} \\plus{} \\frac {c^2}{a} \\ge \\frac {(a \\plus{} b \\plus{} c)(a^2 \\plus{} b^2 \\plus{} c^2)}{ab \\plus{} bc \\plus{} ac}$[/quote]\n\nyou are very good!\n\nplease see nice and strong:\n\n$ \\frac {a^3}{b} \\plus{} \\frac {b^3}{c} \\plus{} \\frac {c^3}{a}\\geq (a^2 \\plus{} b^2 \\plus{} c^2)\\frac {a^3 \\plus{} b^3 \\plus{} c^3}{a^2b \\plus{} b^2c \\plus{} c^2a}.$\nBQ[/quote]\r\n\r\n\r\ntry $ a \\equal{} \\frac {1}{11},b \\equal{} \\frac {1}{71},c \\equal{} 1$\r\n\r\n :lol:", "Solution_10": "change to:\r\n$ \\frac {1}{4}\\,{\\frac {\\left( {\\it x_1} \\plus{} {\\it x_2} \\plus{} {\\it x_3} \\plus{} {\\it x_4} \\right) ^{3}} {{\\it x_1}\\,{\\it x_2} \\plus{} {\\it x_2}\\,{\\it x_3} \\plus{} {\\it x_3}\\,{\\it x_4} \\plus{} {\\it x_1}\\,{\\it x_4}}}\\leq {\\frac {{{\\it x_1}}^{2}}{{\\it x_2}}} \\plus{} {\\frac {{{\\it x_2}}^{2 }}{{\\it x_3}}} \\plus{} {\\frac {{{\\it x_3}}^{2}}{{\\it x_4}}} \\plus{} {\\frac {{{\\it x_4}}^{2 }}{{\\it x_1}}}.$\r\nBQ", "Solution_11": "[quote=\"xzlbq\"]Let $ x_i\\geq 0,i \\equal{} 1,2,3,4$\n\n$ {\\frac {\\left( {\\it x_1} \\plus{} {\\it x_2} \\plus{} {\\it x_3} \\plus{} {\\it x_4} \\right) \\left( {{\\it x_1}}^{2} \\plus{} {{\\it x_2}}^{2} \\plus{} {{\\it x_3}}^{2} \\plus{} {{\\it x_4}}^{2} \\right) }{{\\it x_1}\\,{\\it x_2} \\plus{} {\\it x_2}\\,{\\it x_3} \\plus{} {\\it x_3}\\,{\\it x_4} \\plus{} {\\it x_4}\\,{\\it x_1}}}\\leq {\\frac {{{\\it x_1}}^{2}}{{\\it x_2}}} \\plus{} {\\frac {{{\\it x_2}}^{2 }}{{\\it x_3}}} \\plus{} {\\frac {{{\\it x_3}}^{2}}{{\\it x_4}}} \\plus{} {\\frac {{{\\it x_4}}^{2 }}{{\\it x_1}}}$\nBQ\n\nn-vari:\nLet $ x_i\\geq 0,i \\equal{} 1,..,n,n\\geq2,x_{n \\plus{} 1} \\equal{} x_1$,prove that:\n\n$ \\sum_{i \\equal{} 1}^n{\\frac {x_i^2}{x_{i \\plus{} 1}}}\\geq \\sum_{i \\equal{} 1}^n{x_i}\\frac {\\sum_{i \\equal{} 1}^n{x_i^2}}{\\sum_{i \\equal{} 1}^n{x_ix_{i \\plus{} 1}}}.$\n\n\n\n\nBQ[/quote]\r\n\r\nchange to:\r\n\r\nLet $ x_i > 0,i \\equal{} 1,..,n,n\\geq2,x_{n \\plus{} 1} \\equal{} x_1$,prove that:\r\n\r\n$ \\sum_{i \\equal{} 1}^n{\\frac {x_i^2}{x_{i \\plus{} 1}}}\\geq \\frac {\\sum_{i \\equal{} 1}^n{x_i}}{n}\\frac {(\\sum_{i \\equal{} 1}^n{x_i})^2}{\\sum_{i \\equal{} 1}^n{x_ix_{i \\plus{} 1}}}.$\r\nBQ" } { "Tag": [ "limit", "calculus", "calculus computations" ], "Problem": "lim(2arctanx/pi)^x x tends to +inf\r\nlim(e^n-(1+1/n)^n^2) n tends to inf", "Solution_1": "is this what you meant?\r\n$ \\lim_{x \\to \\infty}(2\\frac {arctan(x)}{\\pi})^x$\r\n$ \\lim_{n \\to \\infty}(e^n \\minus{} (1 \\plus{} \\frac {1}{n})^{n^2})$", "Solution_2": "Sorry\r\n[(2arctanx)/pi]^x\r\n sorry not (1-1/n),but (1+1/n),i typed wrongly.", "Solution_3": "The first: \r\nthe limit equals\r\nlimxtoinf e^((ln((2arctanx)/pi))/1/x)=\r\ne^limxtoinf ((ln((2arctanx)/pi))/1/x), \r\n\r\nUsing L'HOPITAL yields\r\n\r\ne^limxtoinf (pi/(2arctanx))(2/pi)(1/(1+x^2))/(-1/x^2)=\r\ne^(-2/pi)", "Solution_4": "$ L \\equal{} \\lim_{x \\to \\infty}\\left(\\frac{2 \\arctan x}{\\pi}\\right)^x \\equal{} e^A$, where\r\n\r\n$ A \\equal{} \\lim_{x \\to \\infty} x\\left(\\frac{2 \\arctan x}{\\pi} \\minus{} 1\\right) \\equal{} \\frac{1}{\\pi} \\lim_{x \\to \\infty} \\frac{2 \\arctan x \\minus{} \\pi}{\\frac{1}{x}} \\equal{} \\frac{1}{\\pi} \\underbrace{\\lim_{x \\to \\infty} \\minus{} \\frac{2x^2}{x^2 \\plus{} 1}}_{using L'Hopital}$\r\n\r\n$ \\equal{} \\minus{} \\frac{2}{\\pi}$, and so $ L \\equal{} e^{\\minus{}2/\\pi}$.", "Solution_5": "is this correct for the second one?\r\n\r\n$ ln\\left(\\left(1\\plus{}\\frac{1}{n}\\right)^n\\right)\\equal{}\\sum_{k\\equal{}1}^{\\infty}\\frac{\\minus{}1^k}{\\minus{}kn^{k\\minus{}1}}$\r\n$ \\left(1\\plus{}\\frac{1}{n}\\right)^n\\approx e^{1\\minus{}\\frac{1}{2n}}$\r\n$ \\left(1\\plus{}\\frac{1}{n}\\right)^{n^2}\\approx e^{n\\minus{}.5}$\r\n$ e^n\\minus{}\\left(1\\plus{}\\frac{1}{n}\\right)^{n^2}\\approx e^{n\\minus{}.5}(\\sqrt{e}\\minus{}1)$\r\nwhich diverges" } { "Tag": [ "geometry", "incenter", "geometry unsolved" ], "Problem": "The orthocentre of the triangle $ ABC$ is $ H$, and its incircle, centred at $ I$, touches the sides $ AC$ and $ BC$ at the points $ P$ and $ Q$. Prove that if $ H$ lies on the line $ PQ$ then the line $ HI$ passes through the midpoint $ K$ of the side $ AB$.", "Solution_1": "let $ BH \\cap AC =E$ and $ AH \\cap BC=D$\r\nby simple agle chasing we observe that $ PQ$ bisects $ \\angle DHB$ and $ \\angle EHA$ also quad $ ABDE$ is cyclic\r\ntherefore, $ \\frac{EP}{PA}= \\frac{EH}{HA}= \\frac{DH}{HB}= \\frac{DQ}{QB} =\\gamma$\r\nnow extend $ HI$ to a point K such that $ \\frac{HI}{IK}= \\gamma$ but, as $ IP \\parallel HE$ and $ IQ \\parallel HD$ we get (by basic proportionality theorem) $ KA$ is perpendicular to $ AC$ and $ KB$ is perpendicular to $ AB$\r\nnow drop perpendiculars to sides $ AC$ and $ BC$ from midpoint of $ AB=M$ at $ S$ and $ T$ respectively \r\nby midpoint theorem\r\nas $ SM \\parallel BE$ we get, $ ES=SB$\r\nas $ TM \\parallel AD$ we get, $ DT=TB$\r\njoin $ HM$ and extend to $ K'$ such that $ HM=MK'$\r\ntherefore by midpoint theorem\r\n $ K'A$ is perpendicular to $ AC$ and $ K'B$ is perpendicular to $ AB$\r\n$ \\Longrightarrow K=K'$\r\n$ \\Longrightarrow H ,I ,M , K$ are colinear\r\nQED", "Solution_2": "gauravpatil , could u please tell me what\r\n$ XY\\cap AB \\equal{}L$ means?? \r\nSorry for such a stupid question.", "Solution_3": "[quote=\"Aravind Srinivas L\"]gauravpatil , could u please tell me what\n$ XY\\cap AB \\equal{} L$ means?? \nSorry for such a stupid question.[/quote]\r\n\r\n$ XY$ and $ AB$ intersect at $ L$ :)", "Solution_4": "thanks Indybar.\r\nNice soln Gaurav INMO topper :P" } { "Tag": [ "geometry", "Princeton", "college", "MIT", "Harvard", "Stanford", "Yale" ], "Problem": "Being an undergrad, choosing graduate program is very hard.\r\n\r\nI am into algebraic geometry, category theory, k-theory, and algebraic topology... sort of thing.\r\n\r\nCould you name some top programs in these areas? \r\nCould you name some schools that I should never go to if I have these interests? (for example, NYU)?\r\n\r\nI haven't met my profs for a while (currently on one-year break), so hopefully someone here will guide me.\r\n\r\nSo far I am looking to apply to only top programs :maybe: , and if that doesn't work out I will probably get master's and try again.\r\n\r\nSchools that I am looking at now are Princeton, MIT, Harvard, Chicago, Berkeley, Stanford, Yale, Columbia, Michigan, and Toronto. I did most of the research by going to the websites, looking at faculty list and funding, and also USNEWS rankings (are these respected? I especially looked at algebra/number theory/algebraic geometry sub-ranking and also topology sub-ranking). If you want these rankings for yourself, you can PM me.", "Solution_1": "I got this answer from another website. I will put it here so other people can have a look.\r\n\r\n[quote]Right now...\n\nHarvard/MIT are best all around choices.\n\nU Penn has some really good people (but I've never liked the enviornment).\n\nToronto recently acquired some really good people so its definitely worth looking at.\n\nNow, if you are absolutely positive that you are looking into the Algebraic Geometry/Topology route, consider (some of these will seem surprising):\n\nColumbia, SUNY-Stony Brook, University of Maryland-College Park, Northwestern, University of Chicago, UC San Diego (more Algebraic Geometry), Berkeley, UC Davis, Princeton, Michigan State University, UIUC (more Category Theory). Ohio state is developing a pretty strong geometry program.\n\nHarvard, Suny-SB, Maryland, Toronto, Berkeley and Princeton each have at least one excellent potential advisor (i.e., very active in the field, producing good work, wide knowledge base, well respected and still accepting \"average\" students) in the field.[/quote]" } { "Tag": [ "inequalities", "rearrangement inequality", "inequalities proposed" ], "Problem": "Let $ a,b,c\\geq 0$, s.t. $ a\\plus{}b\\plus{}c\\equal{}3$. Prove that:\r\n$ (a^2b\\plus{}b^2c\\plus{}c^2a)(ab\\plus{}bc\\plus{}ca)\\le 9$.\r\n\r\nP.S.: could someone explain me the solution here (see post #4):\r\nhttp://www.mathlinks.ro/viewtopic.php?t=228534\r\nThank you very much.", "Solution_1": "[quote=\"Inequalities Master\"]Let $ a,b,c\\geq 0$, s.t. $ a \\plus{} b \\plus{} c \\equal{} 3$. Prove that:\n$ (a^2b \\plus{} b^2c \\plus{} c^2a)(ab \\plus{} bc \\plus{} ca)\\le 9$.\n[/quote]\r\nLet $ a \\plus{} b \\plus{} c \\equal{} 3u,$ $ ab \\plus{} ac \\plus{} bc \\equal{} 3v^2,$ $ abc \\equal{} w^3$ and $ u^2 \\equal{} tv^2.$\r\nHence, $ t\\geq1$ and $ (a^2b \\plus{} b^2c \\plus{} c^2a)(ab \\plus{} bc \\plus{} ca)\\le 9\\Leftrightarrow$\r\n$ \\Leftrightarrow3u^5\\geq(a^2b \\plus{} b^2c \\plus{} c^2a)v^2\\Leftrightarrow$\r\n$ \\Leftrightarrow6u^5 \\minus{} v^2\\sum_{cyc}(a^2b \\plus{} a^2c)\\geq v^2\\sum_{cyc}(a^2b \\minus{} a^2c)\\Leftrightarrow$\r\n$ \\Leftrightarrow6u^5 \\minus{} 9uv^4 \\plus{} 3v^2w^3\\geq(a \\minus{} b)(a \\minus{} c)(b \\minus{} c)v^2.$\r\n$ (a \\minus{} b)^2(a \\minus{} c)^2(b \\minus{} c)^2\\geq0$ gives $ w^3\\geq3uv^2 \\minus{} 2u^3 \\minus{} 2\\sqrt {(u^2 \\minus{} v^2)^3}.$\r\nHence, $ 2u^5 \\minus{} 3uv^4 \\plus{} v^2w^3\\geq2u^5 \\minus{} 3uv^4 \\plus{} v^2\\left(3uv^2 \\minus{} 2u^3 \\minus{} 2\\sqrt {(u^2 \\minus{} v^2)^3}\\right)\\geq0$ because\r\n$ 2u^5 \\minus{} 3uv^4 \\plus{} v^2\\left(3uv^2 \\minus{} 2u^3 \\minus{} 2\\sqrt {(u^2 \\minus{} v^2)^3}\\right)\\geq0\\Leftrightarrow$\r\n$ \\Leftrightarrow u^5 \\minus{} u^3v^2\\geq v^2\\sqrt {(u^2 \\minus{} v^2)^3}\\Leftrightarrow t^3 \\minus{} t \\plus{} 1\\geq0,$ which is true.\r\nHence, $ 6u^5 \\minus{} 9uv^4 \\plus{} 3v^2w^3\\geq0$ and enough to prove that\r\n$ (6u^5 \\minus{} 9uv^4 \\plus{} 3v^2w^3)^2\\geq v^4(a \\minus{} b)^2(a \\minus{} c)^2(b \\minus{} c)^2.$\r\nSince, $ (a \\minus{} b)^2(a \\minus{} c)^2(b \\minus{} c)^2 \\equal{} 27(3u^2v^4 \\minus{} 4v^6 \\plus{} 6uv^2w^3 \\minus{} 4u^3w^3 \\minus{} w^6)$\r\nwe obtain $ (6u^5 \\minus{} 9uv^4 \\plus{} 3v^2w^3)^2\\geq v^4(a \\minus{} b)^2(a \\minus{} c)^2(b \\minus{} c)^2\\Leftrightarrow$\r\n$ \\Leftrightarrow v^4w^6 \\plus{} uv^2(u^4 \\plus{} 3u^2 \\minus{} 6v^4)w^3 \\plus{} u^{10} \\minus{} 3u^6v^4 \\plus{} 3v^{10}\\geq0.$\r\nId est, it remains to prove that $ u^2v^4(u^4 \\plus{} 3u^2 \\minus{} 6v^4)^2 \\minus{} 4v^4(u^{10} \\minus{} 3u^6v^4 \\plus{} 3v^{10})\\leq0.$\r\nBut $ u^2v^4(u^4 \\plus{} 3u^2 \\minus{} 6v^4)^2 \\minus{} 4v^4(u^{10} \\minus{} 3u^6v^4 \\plus{} 3v^{10})\\leq0\\Leftrightarrow$\r\n$ \\Leftrightarrow t(t^2 \\plus{} 3t \\minus{} 6)^2 \\minus{} 4(t^5 \\minus{} 3t^3 \\plus{} 3)\\leq0\\Leftrightarrow(t \\minus{} 1)^2(t^3 \\minus{} 4t \\plus{} 4)\\geq0,$ which is true.\r\nDone! :)", "Solution_2": "[quote=\"Inequalities Master\"]Let $ a,b,c\\geq 0$, s.t. $ a \\plus{} b \\plus{} c \\equal{} 3$. Prove that:\n$ (a^2b \\plus{} b^2c \\plus{} c^2a)(ab \\plus{} bc \\plus{} ca)\\le 9$.\n[/quote]\r\n\r\nLet $ \\{x,y,z\\}\\equal{}\\{a,b,c\\}$ such that $ x\\ge y\\ge z$. By the Rearrangement Inequality we have\r\n$ a^2b \\plus{} b^2c \\plus{} c^2a\\equal{} a(ab)\\plus{}b(bc)\\plus{}c(ca)\\le x(xy)\\plus{}y(zx)\\plus{}z(yz)\\equal{}y(x^2\\plus{}xz\\plus{}z^2)$\r\nUsing AM-GM Inequality we get\r\n$ (xy\\plus{}yz\\plus{}zx)y(x^2\\plus{}xz\\plus{}z^2)\\le y\\frac{(xy\\plus{}yz\\plus{}zx\\plus{}x^2\\plus{}xz\\plus{}z^2)^2}{4}\\equal{}$\r\n$ \\equal{}\\frac1{4}y(x\\plus{}z)^2(x\\plus{}y\\plus{}z)^2\\equal{}\\frac9{8}.2y.(z\\plus{}x).(z\\plus{}x)\\le\\frac9{8}.[\\frac{2(x\\plus{}y\\plus{}z)}{3}]^3\\equal{}9$\r\nwe have done !\r\n\r\n@arqady: Your proof is long but I like it. My congratulations for your patience ! :lol:", "Solution_3": "Very nice proof, dduclam! :lol: \r\nBut it does not always exist. \r\nTry to find a nice proof for the following inequality ( my gift for you ). I am sure that it does not exist although, this inequality seems nice.\r\n\r\nLet $ a,$ $ b$ and $ c$ are positive numbers such that $ abc \\equal{} 1.$ Prove that\r\n\\[ \\frac {a \\plus{} b \\plus{} c}{3}\\geq\\sqrt [10]{\\frac {a^3 \\plus{} b^3 \\plus{} c^3}{3}}\r\n\\]\r\n:P", "Solution_4": "[quote=\"arqady\"]Very nice proof, dduclam! :lol: \nBut it does not always exist. \nTry to find a nice proof for the following inequality ( my gift for you ). I am sure that it does not exist although, this inequality seems nice.\n\nLet $ a,$ $ b$ and $ c$ are positive numbers such that $ abc \\equal{} 1.$ Prove that\n\\[ \\frac {a \\plus{} b \\plus{} c}{3}\\geq\\sqrt [10]{\\frac {a^3 \\plus{} b^3 \\plus{} c^3}{3}}\n\\]\n:P[/quote]\r\nI can prove easily the weaker by Am-Gm:\r\n$ \\frac {a \\plus{} b \\plus{} c}{3}\\geq\\sqrt [11]{\\frac {a^3 \\plus{} b^3 \\plus{} c^3}{3}}$\r\nMy friend, I think the nice proofs always exist. I'll try to find with $ k\\equal{}10$\r\n :)", "Solution_5": "[quote=\"arqady\"]Very nice proof, dduclam! :lol: \nBut it does not always exist. \nTry to find a nice proof for the following inequality ( my gift for you ). I am sure that it does not exist although, this inequality seems nice.\n\nLet $ a,$ $ b$ and $ c$ are positive numbers such that $ abc \\equal{} 1.$ Prove that\n\\[ \\frac {a \\plus{} b \\plus{} c}{3}\\geq\\sqrt [10]{\\frac {a^3 \\plus{} b^3 \\plus{} c^3}{3}}\n\\]\n:P[/quote]\r\nI also killed $ \\frac {a \\plus{} b \\plus{} c}{3}\\geq\\sqrt [10]{\\frac {a^2 \\plus{} b^2 \\plus{} c^2}{3}}$\r\nAnd I will try your problem ,arqady", "Solution_6": "[quote=\"Allnames\"]\nI also killed $ \\frac {a \\plus{} b \\plus{} c}{3}\\geq\\sqrt [10]{\\frac {a^2 \\plus{} b^2 \\plus{} c^2}{3}}$\n[/quote]\r\nThe maximal result here is $ \\frac {a \\plus{} b \\plus{} c}{3}\\geq\\sqrt [5]{\\frac {a^2 \\plus{} b^2 \\plus{} c^2}{3}}.$ It's Vasc's inequality.", "Solution_7": "[quote=\"arqady\"][quote=\"Allnames\"]\nI also killed $ \\frac {a \\plus{} b \\plus{} c}{3}\\geq\\sqrt [10]{\\frac {a^2 \\plus{} b^2 \\plus{} c^2}{3}}$\n[/quote]\nThe maximal result here is $ \\frac {a \\plus{} b \\plus{} c}{3}\\geq\\sqrt [5]{\\frac {a^2 \\plus{} b^2 \\plus{} c^2}{3}}.$ It's Vasc's inequality.[/quote]\r\nBut $ a^3\\plus{}b^3\\plus{}c^3\\ge a^2\\plus{}b^2\\plus{}c^2$ :maybe: \r\nI have just found a proof for your problem in pvthuan's book,It \u00ed also used pqr technique,and of course,not nice", "Solution_8": "[quote=\"Allnames\"][quote=\"arqady\"][quote=\"Allnames\"]\nI also killed $ \\frac {a \\plus{} b \\plus{} c}{3}\\geq\\sqrt [10]{\\frac {a^2 \\plus{} b^2 \\plus{} c^2}{3}}$\n[/quote]\nThe maximal result here is $ \\frac {a \\plus{} b \\plus{} c}{3}\\geq\\sqrt [5]{\\frac {a^2 \\plus{} b^2 \\plus{} c^2}{3}}.$ It's Vasc's inequality.[/quote]\nBut $ a^3 \\plus{} b^3 \\plus{} c^3\\ge a^2 \\plus{} b^2 \\plus{} c^2$ :maybe: \n[/quote]\r\nOf cause! But $ 5<10.$ :P", "Solution_9": "[quote=\"Inequalities Master\"]Let $ a,b,c\\geq 0$, s.t. $ a \\plus{} b \\plus{} c \\equal{} 3$. Prove that:\n$ (a^2b \\plus{} b^2c \\plus{} c^2a)(ab \\plus{} bc \\plus{} ca)\\le 9$.\n\nP.S.: could someone explain me the solution here (see post #4):\nhttp://www.mathlinks.ro/viewtopic.php?t=228534\nThank you very much.[/quote]\nYou can see\nhttp://www.mathlinks.ro/Forum/viewtopic.php?t=146030\n\n[quote=\"arqady\"]Very nice proof, dduclam! :lol: \nBut it does not always exist. \nTry to find a nice proof for the following inequality ( my gift for you ). I am sure that it does not exist although, this inequality seems nice.\n\nLet $ a,$ $ b$ and $ c$ are positive numbers such that $ abc \\equal{} 1.$ Prove that\n\\[ \\frac {a \\plus{} b \\plus{} c}{3}\\geq\\sqrt [10]{\\frac {a^3 \\plus{} b^3 \\plus{} c^3}{3}}\n\\]\n:P[/quote]\nI think it exists, but don't find out yet, that's all :P \n\nBecause in fact, the maximal result which you show also very easy with AM-GM:\n\n[quote=\"arqady\"][quote=\"Allnames\"]\nI also killed $ \\frac {a \\plus{} b \\plus{} c}{3}\\geq\\sqrt [10]{\\frac {a^2 \\plus{} b^2 \\plus{} c^2}{3}}$\n[/quote]\nThe maximal result here is $ \\frac {a \\plus{} b \\plus{} c}{3}\\geq\\sqrt [5]{\\frac {a^2 \\plus{} b^2 \\plus{} c^2}{3}}.$ It's Vasc's inequality.[/quote]\r\n\r\n$ (\\sum a)^6\\equal{}(\\sum a^2\\plus{}\\sum bc\\plus{}\\sum bc)^3\\ge 27(\\sum a^2)(\\sum bc)^2\\ge 81(\\sum a^2)abc(\\sum a)$ \r\n$ \\rightarrow \\frac {a \\plus{} b \\plus{} c}{3}\\geq\\sqrt [5]{\\frac {a^2 \\plus{} b^2 \\plus{} c^2}{3}}.$", "Solution_10": "[quote=\"nguoivn\"]\nI can prove easily the weaker by Am-Gm:\n$ \\frac {a \\plus{} b \\plus{} c}{3}\\geq\\sqrt [11]{\\frac {a^3 \\plus{} b^3 \\plus{} c^3}{3}}$\n[/quote]\r\n\r\nI think somebody want to try :)", "Solution_11": "For the collection. :wink: :) \r\nLet $ a,$ $ b$ and $ c$ are non-negative numbers such that $ (a \\plus{} b)(a \\plus{} c)(b \\plus{} c) \\equal{} 8.$ Prove that:\r\n\\[ \\frac {a \\plus{} b \\plus{} c}{3}\\geq\\sqrt [11]{\\frac {a^2 \\plus{} b^2 \\plus{} c^2}{3}}\r\n\\]", "Solution_12": "[quote=\"arqady\"]\nLet $ a,$ $ b$ and $ c$ are positive numbers such that $ abc \\equal{} 1.$ Prove that\n\\[ \\frac {a \\plus{} b \\plus{} c}{3}\\geq\\sqrt [10]{\\frac {a^3 \\plus{} b^3 \\plus{} c^3}{3}}\n\\]\n:P[/quote]\nThe proof see here:\nhttp://www.artofproblemsolving.com/Forum/viewtopic.php?p=3107205#p3107205", "Solution_13": "[quote=arqady]Very nice proof, dduclam! :lol: \nBut it does not always exist. \nTry to find a nice proof for the following inequality ( my gift for you ). I am sure that it does not exist although, this inequality seems nice.\n\nLet $ a,$ $ b$ and $ c$ are positive numbers such that $ abc \\equal{} 1.$ Prove that\n\\[ \\frac {a \\plus{} b \\plus{} c}{3}\\geq\\sqrt [10]{\\frac {a^3 \\plus{} b^3 \\plus{} c^3}{3}}\n\\]\n:P[/quote]\n\nSee here\n[url]https://www.facebook.com/photo?fbid=2201540009983422&set=gm.2953521171602591[/url]" } { "Tag": [ "inequalities proposed", "inequalities" ], "Problem": "Let be $ a,b,c\\ge 0$ such that $ ab\\plus{}bc\\plus{}ca\\equal{}1$ . Prove that :\r\n\r\n$ \\frac{1}{a\\plus{}b}\\plus{}\\frac{1}{b\\plus{}c}\\plus{}\\frac{1}{c\\plus{}a}\\ge \\frac{5}{2}$", "Solution_1": "[quote=\"alex2008\"]Let be $ a,b,c\\ge 0$ such that $ ab \\plus{} bc \\plus{} ca \\equal{} 1$ . Prove that :\n\n$ \\frac {1}{a \\plus{} b} \\plus{} \\frac {1}{b \\plus{} c} \\plus{} \\frac {1}{c \\plus{} a}\\ge \\frac {5}{2}$[/quote]\r\nSee here:\r\nhttp://www.artofproblemsolving.com/Forum/viewtopic.php?t=31377\r\nhttp://www.artofproblemsolving.com/Forum/viewtopic.php?t=101476", "Solution_2": "Thanks , arqady ." } { "Tag": [ "inequalities", "inequalities unsolved" ], "Problem": "Prove that if a,b,c>0,then:\r\n$ \\sum \\frac{a(b\\plus{}c)}{b^2\\plus{}bc\\plus{}c^2}$\u22652", "Solution_1": "Chebyshev inequality", "Solution_2": "[quote=\"duongptnkvn\"]Chebyshev inequality[/quote]\r\nCan you explain more clearly?I don't think we can use Chebyshev to show this ineq.", "Solution_3": "By multiplying out it equivalents to $ \\sum a^5b \\geq \\sum a^4b^2$ which is obviously true by Muirhead :wink:", "Solution_4": "http://www.mathlinks.ro/Forum/viewtopic.php?t=25553" } { "Tag": [ "factorial", "search" ], "Problem": "How do i do these questions? \r\n1)How can the word DECISIONS be arranged so that the \"N\" is somewhere to the RIGHT of \"D\" (my problem is the two I's in particular) \r\nANSWER: 45360 \r\n\r\n2)Bob is about to hang 8 shirts in the wardrobe. He has 4 different styles of shirt, 2 identical ones of each particular style. How many different arrangements are possible if no 2 identical shirts are next to one another? \r\nANSWER: 864 \r\n\r\n3) A motorist travels through 8 sets of traffic lights, each of which is red or green. He is forced to stop at 3 sets of lights \r\na) In how many ways could this happen? ANSWER: 56 \r\nb) What other number of red lights would give an identical answer to part a)? ANSWER: 5 \r\n\r\n4) Find how many 5 digit numbers are possible from the digits 9,8,7,6,5,4,3,2,1,0 if digits are to be in \r\na) ascending order? ANSWER: 126 \r\nb) descending order? ANSWER: 252 \r\n\r\nAny help on any of these questions really appreciated !!", "Solution_1": "I'll answer your last question.In descending order , the answer is calculated by a ${10 \\choose 5 } = 126 $ .\r\nIn the ascending order , however , it is calculated by a $ {9 \\choose 5 } = 252 $.\r\nHere's the reasoning for that -- In the descending order , if a zero comes first , it can just be put as the last digit , whereas , you cant do that in the ascending order.....", "Solution_2": "I'll take care of number 3. Since we go through 8 lights and 3 are red we have R, R, R, G, G, G, G, G in some order. The way to arrange repeated items in sets is the total number factorial divided by how many times it repeats factorial. In this case we get $8!/(3!5!)$ which equals 56. Note for part b that this answer would be the same if we had 3 green lights and 3 red lights.", "Solution_3": "for number 1\r\nsince there are nine numbers, lets start off w/ 9P9, or 9!\r\n\r\nsince there are 2 S and 2 I (I think this is where you said your problem comes?) divide the expression by 2! (for the 2 S) and another 2! (for the 2 I)\r\nnow we have 9!/(2!2!)\r\n\r\nhalf the time, N is to the right of D, beacause the other half of the time, N is to the left of D. so divide the expression by half again\r\n\r\n9!/(2!2!) * 1/2 = 45360", "Solution_4": "guys, is there a link to permutations problems or some practice sets we can download? \r\n\r\nThanks", "Solution_5": "Why dont you search this forum ? I remember seeing a huge number of permutations and combinations here .....", "Solution_6": "Can someone please answer no.2, im not getting the answer", "Solution_7": "[hide=\"Solution Outline\"]Find the total number of ways to order the shirts without the restriction that identical shirts are not adjacent, then use a massive amount of PIE bash to find the number of ways to arrange the shirts so that at least one pair of identical shirts are adjacent, then subtract the second value from the first. [/hide]\r\n\r\n(Probably not the best solution. )", "Solution_8": "Hint for 2: Notice that 864=2^5*3^3" } { "Tag": [], "Problem": "After noticing that several people could not solve this question, i decided to post it on the intermediate section of this forum\r\n\r\nHere goes the question: what do you think the next 2 numbers in the following arrangement will be? 125, 135, 145, 155, _, _. to find the solution use your inductive reasoning skills. you base your response on what you think will happen. (the answer is not 165 and 175. it is 163 and 168. why?) \r\n\r\nI have tried several things and am not going anywhere, any ideas?", "Solution_1": "There is, in fact, no reason, given only the first few terms of the sequence, why the next few should not be $165, 175$, as that is by far the simplest pattern that can be drawn from the sample data. Any other answer is only a matter of preference, and I think it should be made clearer by assigning the next term $= 163$ and asking to find the next two terms after that, which is much less ambiguous :D \r\n\r\nHowever, given the parentheses, then the problem should really be \"Find a rule for the sequence $125, 135, 145, 155, 163, 168$,\" since this is what the problem is really asking for.", "Solution_2": "Are you sure the rule for this sequence is math related?", "Solution_3": "yea pretty sure it is math related, any thoughts", "Solution_4": "simply put, your answer key is wrong :lol:" } { "Tag": [ "MATHCOUNTS" ], "Problem": "MATHCOUNTS has posted the School and Chapter competitions. You may now discuss any and all aspects of the competition, including scores, questions, solutions, and answers.\r\n\r\nThank you for your patience, as we awaited this moment!", "Solution_1": "The contests are posted [url=http://www.mathcounts.org/webarticles/anmviewer.asp?a=510&z=29]here[/url].", "Solution_2": "Is the MathCounts website down for everyone, or just me?", "Solution_3": "It should be working fine, is it something with your browser?\r\n\r\nYou should try Mozilla Firefox, gets less errors.", "Solution_4": "not working 4 me either.... :?", "Solution_5": "It seems to be always down the Saturday of states competitions." } { "Tag": [ "number theory", "least common multiple", "modular arithmetic" ], "Problem": "What is the smallest number that is divisible by 13 but when divided by the whole nmbers 2, 3, 4, 5, 6, 7, 8, 9, 10, 11, 12 yields a remainder of 1?\r\n\r\n$?r1$\r\n$13)\\overline{abcdef...}$", "Solution_1": "[quote=\"mgao\"]What is the smallest number that is divisible by 13 but when divided by the whole nmbers 2, 3, 4, 5, 6, 7, 8, 9, 10, 11, 12 yields a remainder of 1?\n\n$?r1$\n$13)\\overline{abcdef...}$[/quote]\r\n[hide]The lcm of 2-12 is 27720. So the number is 1 + a factor 27720. 27720 has a remainder of 4. Triple it to get one with a remainder of twelve and add one to that to get [u][b]83161[/b][/u][/hide]", "Solution_2": "[hide]LCM(2,3,...11,12)=27720\nwhich implies that 27721 is the smallest number giving a remaider of 1 when divided by 1-12.\n\n$27721\\equiv 5\\equiv -8\\pmod{13}$\n$27720\\equiv 4 \\pmod{13}$\n\nwe want to add multiples of 27720 to 27721 (since adding 27720 won't change the fact that it is $\\equiv 1 \\pmod{1-12}$) until it is divisible by 13. $-8+2*4\\equiv 0\\pmod{13}$, so we add two of them. 27721+2*27720=$\\boxed{83161}$.[/hide]" } { "Tag": [ "integration", "algebra", "polynomial", "AMC", "AIME" ], "Problem": "1)[img]http://www.artofproblemsolving.com/Images/ClassLaTeX/lx-201681140.gif[/img]\r\n\r\n2)[img]http://www.artofproblemsolving.com/Images/ClassLaTeX/lx-264349124.gif[/img]\r\n\r\n3)[img]http://www.artofproblemsolving.com/Images/ClassLaTeX/lx-161158398.gif[/img]\r\n\r\n4)[img]http://www.artofproblemsolving.com/Images/ClassLaTeX/lx-175366670.gif[/img]\r\n\r\n5)[img]http://www.artofproblemsolving.com/Images/ClassLaTeX/lx-151374445.gif[/img]\r\n\r\n6)[img]http://www.artofproblemsolving.com/Images/ClassLaTeX/lx-84036740.gif[/img]", "Solution_1": "The Abel Summation formula?", "Solution_2": "$\\text{Abel Summation: }$\r\n\r\nLet $a_1,a_2,\\ldots,a_n$ and $b_1,b_2,\\ldots,b_n$ be two finite sequences of numbers. Then \\[ a_1b_1+a_2b_2+\\cdots+a_nb_n = (a_1-a_2)b_1 + (a_2 - a_3)(b_1+b_2)+\\cdots + (a_{n-1} - a_n)(b_1+b_2 + \\cdots+ b_n)+a_n(b_1+b_2+\\cdots+b_n). \\]", "Solution_3": "[hide=\"1\"]\nsquare both sides, nested roots cancel, isolate root, square, squared terms cancel, $\\boxed{x=26}$. Kinda easy... I'll get to the others later..[/hide]", "Solution_4": "[hide=\"2\"]$\\frac{n}{3^{n}}$[/hide]", "Solution_5": "[quote=\"RiemannZetaFunction\"][hide=\"2\"]$\\frac{n}{3^{n}}$[/hide][/quote]\r\nProof please.", "Solution_6": "[quote=\"PenguinIntegral\"][quote=\"RiemannZetaFunction\"][hide=\"2\"]$\\frac{n}{3^{n}}$[/hide][/quote]\nProof please.[/quote]\r\n\r\nJust multiply it by 3 and do what you usually do to find the sum of geometric series.", "Solution_7": "[quote=\"chess64\"][quote=\"PenguinIntegral\"][quote=\"RiemannZetaFunction\"][hide=\"2\"]$\\frac{n}{3^{n}}$[/hide][/quote]\nProof please.[/quote]\n\nJust multiply it by 3 and do what you usually do to find the sum of geometric series.[/quote]\r\nMultiply the sequence by 3? Then you get 1, 2/3, 1/3, which isn't geometric (I think).", "Solution_8": "I have no clue where $n$ got in his answer...\r\n\r\n[hide]\nDefine $S=\\frac 13 +\\frac 29+\\frac 3{27}+\\frac 4{81}\\cdots$. So then $\\frac 13 S = \\frac 19 + \\frac 2{27}+\\frac 3{81}.$ Thus \\[ S-\\frac 13 S=\\frac 13 + \\frac 19 + \\frac 1{27}+\\cdots= \\frac 13\\cdot \\frac 1{1-\\frac 13}=12=\\frac 23S \\Rightarrow S= \\frac 34. \\][/hide]", "Solution_9": "[quote=\"PenguinIntegral\"][quote=\"chess64\"]\nJust multiply it by 3 and do what you usually do to find the sum of geometric series.[/quote]\nMultiply the sequence by 3? Then you get 1, 2/3, 1/3, which isn't geometric (I think).[/quote]\r\n\r\nHe means calculate $3S - S$. :)\r\n\r\nEdit: [hide=\"4\"] This problem and others like it have been posted many, many times before. Let $\\tau$ be a non-unity fifth root of unity. \n\n$f(\\tau) = 0$\n\nHence the remainder is $f( \\tau^5 ) = f(1) = \\boxed{5}$. [/hide]\n\nEdit: Incidentally, RiemannZetaFunction's answer to this problem is nonsense. [hide=\"Generalization of 2\"] Let $G(x) = \\sum_{k=1}^{\\infty} kx^k$. Then\n\n$x G(x) = \\sum_{k=1}^{\\infty} (k-1) x^{k}$\n\n$G(x) - x G(x) = \\sum_{k=1}^{\\infty} x^k = \\frac{1}{1 - x} - 1$\n\n$\\boxed{ G(x) = \\frac{1 - (1 - x)}{ (1 - x)^2 } = \\frac{x}{ (1 - x)^2 } }$. [/hide]\n\nEdit: Abel Summation is not cool. :P [hide=\"3\"] Let $G(x) = \\sum_{k=1}^{n} kx^{k-1}$. Then\n\n$\\int G(x) \\, dx = \\sum_{k=1}^{n} x^k = \\frac{x^{n+1} - x}{x - 1}$\n\nSo that\n\n$G(x) = \\boxed{ \\frac{ nx^{n+1} - (n+1)x^n + 1}{ (x-1)^2 } }$. (I don't know if this has a neater form or not.) [/hide]", "Solution_10": "[hide=\"#5\"]\nFirst let $\\alpha=4, 16, 36, and 64$. Then our equ. becomes $\\frac{x^2}{\\alpha-1}+\\frac{y^2}{\\alpha-9}+\\frac{z^2}{\\alpha-25}+\\frac{w^2}{\\alpha-49}=1$.\nNow, multiply $(\\alpha-1)(\\alpha-9)(\\alpha-25)(\\alpha-49)$ on both sides. Then u would get a fourth degree polynomial in terms of $\\alpha$. Also the roots of this poly are $4,16,36,64$ or $(\\alpha-4)(\\alpha-16)(\\alpha-36)(\\alpha-64)=0$. The coefficients of the two polynomials should be equal. Thus $1+9+25+49+x^2+y^2+z^2+w^2=4+16+36+64$. The answer is $36$. Whew.....\n[/hide]", "Solution_11": "[quote=\"kimby_102\"][hide=\"#5\"]\nFirst let $\\alpha=4, 16, 36, and 64$. Then our equ. becomes $\\frac{x^2}{\\alpha-1}+\\frac{y^2}{\\alpha-9}+\\frac{z^2}{\\alpha-25}+\\frac{w^2}{\\alpha-49}=1$.\nNow, multiply $(\\alpha-1)(\\alpha-9)(\\alpha-25)(\\alpha-49)$ on both sides. Then u would get a fourth degree polynomial in terms of $\\alpha$. Also the roots of this poly are $4,16,36,64$ or $(\\alpha-4)(\\alpha-16)(\\alpha-36)(\\alpha-64)=0$. The coefficients of the two polynomials should be equal. Thus $1+9+25+49+x^2+y^2+z^2+w^2=4+16+36+64$. The answer is $36$. Whew.....\n[/hide][/quote]\r\nI don't think you can set a variable to more than one value.", "Solution_12": "[quote=\"t0rajir0u\"]\n\nEdit: [hide=\"4\"] This problem and others like it have been posted many, many times before. Let $\\tau$ be a non-unity fifth root of unity. \n\n$f(\\tau) = 0$\n\nHence the remainder is $f( \\tau^5 ) = f(1) = \\boxed{5}$. [/hide]\n\n[/quote]\n\nthe idea\n\n[quote=\"t0rajir0u\"][quote=\"SnowStorm\"][b]Problem 253[/b]\nLet $g(x) = x^5 + x^4 + x^3 + x^2 + x + 1$. What is the remainder when the polynomial $g(x^{12})$ is divided by the polynomial $g(x)$?[/quote]\n\n[hide=\"Solution\"]\n$(x-1) g(x) = x^6 - 1$\n\nSo $g(x)$ has as its roots the sixth roots of unity. That is, $g(x^{12}) = g(1)$ for all of these roots. But $g(1) = \\boxed{6}$.\n[/hide]\n\n[hide=\"Longer explanation\"]\n$g(x^{12}) = g(x) Q(x) + R(x)$, where $Q(x)$ is the quotient polynomial and $R(x)$ is the remainder polynomial which must have a degree of $< 5$.\n\nWe have established that $g(x)$ has as its roots the five non-unity sixth roots of unity. Call them $\\omega_1, \\omega_2, ... \\omega_5$. For each of these roots we have $\\omega_k^6 = 1$. Because they are the roots of $g(x)$, obviously $g(\\omega_k) = 0$. Then\n\n$g(\\omega_k^{12}) = g(\\omega_k) Q(\\omega_k) + R(\\omega_k)$\n$g(1) = R(\\omega_k)$\n$6 = R(\\omega_k)$\n\nFor five different values of $k$. Because $R(x)$ must be $4^{th}$-degree or lower, this means\n\n$R(x) = 6$\n\nFor all $x$. Hence the remainder is $6$. QED.\n[/hide][/quote]", "Solution_13": "[quote=\"PenguinIntegral\"][quote=\"kimby_102\"][hide=\"#5\"]\nFirst let $\\alpha=4, 16, 36, and 64$. Then our equ. becomes $\\frac{x^2}{\\alpha-1}+\\frac{y^2}{\\alpha-9}+\\frac{z^2}{\\alpha-25}+\\frac{w^2}{\\alpha-49}=1$.\nNow, multiply $(\\alpha-1)(\\alpha-9)(\\alpha-25)(\\alpha-49)$ on both sides. Then u would get a fourth degree polynomial in terms of $\\alpha$. Also the roots of this poly are $4,16,36,64$ or $(\\alpha-4)(\\alpha-16)(\\alpha-36)(\\alpha-64)=0$. The coefficients of the two polynomials should be equal. Thus $1+9+25+49+x^2+y^2+z^2+w^2=4+16+36+64$. The answer is $36$. Whew.....\n[/hide][/quote]\nI don't think you can set a variable to more than one value.[/quote]\r\n\r\nShe described the process she carried out very poorly. She made the equation $\\frac{x^2}{\\alpha-1}+\\frac{y^2}{\\alpha-9}+\\frac{z^2}{\\alpha-25}+\\frac{w^2}{\\alpha-49}=1$. From the given, this equation has roots at $\\alpha = 4, 16, 36, 64$. So it is the polynomial she claims, and then she equates coefficients. This question is from an old AIME, if I recall -- a very tough one!", "Solution_14": "Who is she? :mad: \r\n\r\nBTW, I'm sorry if my solution was bad. I was writing it on 1am.........", "Solution_15": "[quote=\"kimby_102\"]Who is she? :mad: \n\nBTW, I'm sorry if my solution was bad. I was writing it on 1am.........[/quote]\r\n\r\nSorry! It wasn't bad (actually quite nice) just your first step was poorly described.", "Solution_16": "No pbm. :)" } { "Tag": [ "puzzles" ], "Problem": "My entire class of dedicated nerds (including our teacher) has been trying to solve this for weeks:\r\n\r\n13, 14, 19, 5, 1\r\n\r\nTarget: 8\r\n\r\nFor those unfamiliar with Krypto, the idea is to add, subtract, multiply, and/or divide the first five numbers to reach the target number.", "Solution_1": "8 = 13-(14/((19/5)-1))\r\n8 = 13+(14/(1-(19/5)))\r\n8 = 13-(19/((14/5)+1))\r\n8 = ((13/(19-14))-1)*5\r\n8 = 13-(19/(1+(14/5)))\r\n8 = ((13/5)-1)*(19-14)\r\n8 = (14-19)*(1-(13/5))\r\n8 = (14+((19-1)*5))/13\r\n8 = (14+(5*(19-1)))/13\r\n8 = (14-(5*(1-19)))/13\r\n8 = (14-((1-19)*5))/13\r\n8 = (14/(1-(19/5)))+13\r\n8 = (19-14)*((13/5)-1)\r\n8 = (((19-1)*5)+14)/13\r\n8 = 5*((13/(19-14))-1)\r\n8 = ((5*(19-1))+14)/13\r\n8 = (1-(13/5))*(14-19)", "Solution_2": "Did you use a program? And if so, could you post the code? And if not, nice going, crazy-smart. How long did it take?", "Solution_3": "[quote=\"Hokkage\"]Did you use a program? And if so, could you post the code? And if not, nice going, crazy-smart. How long did it take?[/quote]\r\n\r\nI did use a program, but I didn't write it(I could have though, since I wrote a Maplet that solves 24 problem).\r\n__________________________________________________________________________\r\nEDITED: For the code for 24 problem, go [url=http://www.artofproblemsolving.com/Forum/viewtopic.php?t=38886&postorder=asc&start=14]here[/url]. It's the first complex program I wrote, so it might be inefficient. It's in Maple(only language I know until I learnt java in September) so you might not be able to understand it." } { "Tag": [ "induction", "inequalities solved", "inequalities" ], "Problem": "The sequence is defined as a recurrence relation:\r\n\r\n(1) a_0 = 0, a_1 = 1 and a_2 = 1\r\n(2) a_n+2 + a_n+1 = 2(a_n+1 + a_n)\r\n\r\na.) Prove that all elements of the sequence are perfect squares !\r\nb.) Derive an explicit formula and prove it to be correct !", "Solution_1": "it seems that a3=3", "Solution_2": "I guess the sequence is :\r\n(2) a_(n+2) + a_(n+1) = 2(a_(n+1) + a_n) isn't it?\r\n\r\ncheers!", "Solution_3": "hmm.. I thought that [b] siuhochung [/b] thought that the sequence was like so:\r\n a_n+2 + a_n+1 = 2(a_n+1 + a_n) but now that I also computed a_3 it is 3... something is missing :D", "Solution_4": "There's some problem, with the recurrence given,\r\nthe general term is (1/3)(2^n-(-1)^n)", "Solution_5": "(2) a_{n+2} + a_{n+1} = 2(a_{n+1} + a_{n})\r\n\r\n n 0 1 2 3 4 5 6 7 \r\n a_{n} 0 1 1 4 9 25 64 169\r\n \\sqrt a_{n} 0 1 1 2 3 4 5 13\r\n\r\nPS: How can I make the table look nicer ?", "Solution_6": "The recurrence becomes : a(n + 2) = a(n + 1) + 2a(n) and since a(1) = a(2) = 1, a(3) = 1 + 2.1 = 3. As far as I know 3 is no perfect square not yet :D So there seems to be something wrong with your table orl ! Are you sure this is the right recurrence ? In your table the squares of the Fibonacci numbers appear, but they don't satisfy your recurrence.", "Solution_7": "Yeah, a_3 isn't 4, it's 3..", "Solution_8": "sorry for the typo:\r\n\r\n(2) a_{n+2} + a_{n-1} = 2(a_{n+1} + a_{n})\r\n\r\nDo you have a good site where one can get a description of the all the math symbols available in the forum ....", "Solution_9": "guess nobody took a look at this one after the correction was made! :(\r\n\r\nhere is what I saw\r\n\r\na_n=F_(n+1)^2,n>=1 where:\r\n\r\nF_0=1, F_1=1 and F_(n+1)=F_n+F_(n-1)\r\n\r\nand then induction to prove that we are right! :D\r\n\r\ncheers!", "Solution_10": "part a.) is not too difficult and it can be solved independently from part b.) or try to derive an explicit formula and deduce the fact from it...", "Solution_11": "Well, the problem IS called \"square of Fibonacci\", so I guess it's not hard to see that the terms are squares of the Fibonacci terms! :D", "Solution_12": "Hi grobber,\r\n\r\nI said derive !!! That's the square of the Binet formula is evident from my table and the problem's subject !!! :D" } { "Tag": [], "Problem": "http://www.cybernations.net/\r\n\r\nJoin Cyber Nations today!\r\n\r\nAnd join the \"Federated Allies for Cultural Exchange\" alliance!\r\n(our forums is http://www.face-cn.com/, just register there and post an application to join)\r\n\r\nCybernations is a very enjoyable text-based nation building game. On average at least one login per day is made.\r\n\r\nPlease join!\r\nAlso, my ruler name on Cybernations is \"XRCatD\"\r\n\r\nOn a side note, xscapezaer also plays Cyber Nations.", "Solution_1": "it seems interesting...\r\n\r\nI might join.\r\n\r\nI [i]might[/i].", "Solution_2": "So that's what CN stands for...", "Solution_3": "yeah, most people who play LW start first with Cn.\r\ni play cn too. i'm spaghetti and my nation is lanayru\r\nunfortuantely im already in an alliance (random insanity alliance)\r\nEDIT: would you like to tech trade? :)", "Solution_4": "Tip #1: Join an alliance QUICK if you haven't because there are alliance fishers out there.\r\n\r\nI join and 6 or 7 hours later I have 8 offers to join 8 different clans.", "Solution_5": "darn, i didn't get you\r\nhint: i'm an alliance fisher too.\r\nand i got a lot more spam. random insanity alliance sent me a message in the first 20 seconds and by the first 5 minutes i had 17 invitations" } { "Tag": [ "modular arithmetic", "function", "number theory", "least common multiple", "probability", "MATHCOUNTS" ], "Problem": "I always thought 0 was a multiple of anything because 0 * x = 0, but a CML I took a couple months ago said otherwise. Is 0 a multiple of 3, or any number for that matter?", "Solution_1": "It is disputed over a lot, but I am pretty sure it is not considered a multiple. There was a question on MOEMS in which the answer was affected if 0 was considered a multiple or not. MOEMS considered it a multiple, however it was challenged and the challenge succeeded.", "Solution_2": "i thought it was because -1*3=-3, 0*3=0, 1*3=3. if the other two are, why can't zero times three be a multiple?", "Solution_3": "[quote=\"ragnarok23\"]It is disputed over a lot, but I am pretty sure it is not considered a multiple. There was a question on MOEMS in which the answer was affected if 0 was considered a multiple or not. MOEMS considered it a multiple, however it was challenged and the challenge succeeded.[/quote]\r\n\r\nI remember that question. I spent like 10 minutes debating with myself over that question...\r\n\r\nI think N is a multiple of m, if there is some integer k such that N=km, so yes (k=0).", "Solution_4": "Yeah, I think 0 is a multiple of everything.\r\n\r\nReason 1:\r\n$N$ is a multiple of $N$ (duh)\r\nTo create another multiple, you must add or subtract $N$, and $N-N=0$, so 0 is a multiple.\r\n\r\nReason 2 (similar to pianoforte's):\r\nTo be a multiple of a number $x$, you have $N\\equiv0\\pmod{x}$. This can be expressed as $N=0+Ax$, for some integer $A$. When $A=0$, $N=0$, so 0 is a multiple of $x$\r\n\r\nReason 3:\r\nWhy shouldn't it be a multiple of all numbers?", "Solution_5": "The divisor function (number of divisors) considers 0 to have $\\infty$ factors. ie. It's a multiple of everything.", "Solution_6": "Well, as common sense, if somebody tells you, \"Start counting multiples of 3 from -6.\" \r\n\r\nYou would do, or at least I would do, $-6,-3,\\boxed{0},3,6, \\text{etc.}$. It would be weird if you just skipped zero...", "Solution_7": "I think 0 is the multiple of any number except.\r\n\r\n\r\n\r\n\r\n\r\n\r\n\r\nMath is my life :!:", "Solution_8": "except what", "Solution_9": "Hmmm...so 0 is like the lord of all composite numbers :idea:", "Solution_10": "0 is not a multiple of 3. In Number Theory, if a is divisible by b, then b is a divisor of a, and a is a multiple of b. Note that a and b are all natural numbers (excluding \"0\").", "Solution_11": "Actually it depends on your definition of a multiple.\r\n\r\nIf it means that for $a$ to be a multiple of $b$, then for some INTEGER $k$, $ak=b$, then 0 is a multiple of all integers $b$.\r\n\r\nIf you exchange the integers with natural number, then 0 is NOT a multiple. Dependes on your definition.", "Solution_12": "For this level, I agree with biffanddoc that 0 is a multiple. However, if 0 were a multiple, wouldn't the LCM of any group of numbers be 0? Weird...", "Solution_13": "0 is a multiple of everything. The first multiple of any number is the number. 0 would be the zeroth multiple. Thus, the least common multiple is positive because we don't consider the zeroth multiple.", "Solution_14": "From previous posts, it looks like negatives are also multiples, so that would complicate things earlier. \r\n\r\nCorrect me if I'm wrong, but generally on math competitions 0 is not a multiple and LCM is always positive.", "Solution_15": "1. Well-editted contests would avoid this issue by using 'positive multiples' or 'natural numbers' as descriptors. If not so limited on a contest, 0 should count as a multiple. But....\r\n2. Editors are human. Humans make errors. 'Little' things are easy to miss. :roll:", "Solution_16": "Well, sometimes it's indirect:\r\n\r\nE.G. 2004 Nats #28(?), there's a number line and there's some probability you go there, here, etc. The probability of landing on a multiple of three would differ if 0 was counted as a multiple of 3.", "Solution_17": "I think I counted 0 as a multiple of 3 and got it wrong", "Solution_18": "Well, I'd consider $0$ as a multiple of $3$", "Solution_19": "Kind of late, but no I don't believe 0 is ever a multiple of anything in math competitions. Generally in math competitions, correct me if I'm wrong, it is implied to be the positive multiples unless otherwise stated.\r\n\r\nHey someone else actually does CML? Amazing! What did you get? Does anyone know if you get anything for a perfect score?", "Solution_20": "[quote=\"life=tennis+math\"]Hey someone else actually does CML? Amazing! What did you get? Does anyone know if you get anything for a perfect score?[/quote]\r\nI think I got 1 wrong or something. I don't know what you get, but it's probably not an all-expense paid trip to Washington DC. :D", "Solution_21": "[quote=\"Fanatic\"]E.G. 2004 Nats #28(?), there's a number line and there's some probability you go there, here, etc. The probability of landing on a multiple of three would differ if 0 was counted as a multiple of 3.[/quote]Right, and to get the correct answer of $\\frac{31}{90}$, you had to assume that 0 was a multiple of 3. If you assumed it wasn't, then you would get the wrong answer.\r\n\r\nFor folks who don't have the problem in front of them, here is a paraphrase. You pick a random integer from 1 to 10. Then, with probability $\\frac{2}{3}$, you move one up; otherwise you move one down. For a second time, with probability $\\frac{2}{3}$, you move one up; otherwise you move one down. What is the probability that you end up on a multiple of 3?", "Solution_22": "[quote=\"BFG\"]0 is not a multiple of 3. In Number Theory, if a is divisible by b, then b is a divisor of a, and a is a multiple of b. Note that a and b are all natural numbers (excluding \"0\").[/quote]\r\n\r\n0 can be a multiple by this if you mess with the numbers for a while.\r\nput a as 0, and b as 3. 0 is divisible by 3 (0/3=0) 3 is a divisor of 0 (3 goes into 0 zero times) and 3*0=0. Why does it have to be natural numbers?\r\n\r\ni like to break mathematical laws... :lol:", "Solution_23": "[quote=\"lotrgreengrapes7926\"][quote=\"life=tennis+math\"]Hey someone else actually does CML? Amazing! What did you get? Does anyone know if you get anything for a perfect score?[/quote]\nI think I got 1 wrong or something. I don't know what you get, but it's probably not an all-expense paid trip to Washington DC. :D[/quote]\r\n\r\nwhat is CML?", "Solution_24": "who makes the mathcounts tests??? i want to know their opinion on this so i dont mess up on possible question like this next year.", "Solution_25": "The QWC writes the tests", "Solution_26": "[hide]0 is a multiple of any number. At least that is what I think.\n\ndo you think they might have said positive somewhere in the problem?[/hide]", "Solution_27": "No, it does not say positive in the problem, I checked", "Solution_28": "Right. That means the MathCounts writers agree that 0 is a multiple of everything. Otherwise, they would not have written that $\\frac{30}{91}$ is the answer to the probability question.", "Solution_29": "There is no doubt about it. 0 is a multiple of any number. It said it in AoPS Volume 1. You can always trust it. :D", "Solution_30": "A much better question would be what is a multiple of 0?", "Solution_31": "The only multiple of 0 is 0.", "Solution_32": "CML = continental math league, I got a perfect score =o", "Solution_33": "CML for me:\r\n\r\n6-6-6-6-6-6-5-4... (perfect score is 6/6)\r\n\r\nOur school gives a (small) award to the grade level winners... I thought my 4 would make me not win it but I still had by far the highest score =O", "Solution_34": "I missed like 1 or 2 ( my teacher never told us what we got on the last one...)\r\nI know I ended up with second and what does the first place person get? name on the morning announcements (which second and 3rd people do too) and its almost as bad of a thing as a good thing....(because then your ELA teacher who just likes you too much comes up to talk to you while you are talking to your friends) \r\n\r\nFor the 0 question...Im confused cuz I thought I got it wrong since I put 0 as a not multiple and later my friend was like you did put 1 right and I was like um whoops." } { "Tag": [ "trigonometry", "limit", "calculus", "calculus computations" ], "Problem": "How do you prove the following formula?: \\[\\frac{2}{\\pi}=\\sqrt{\\frac{1}{2}}\\sqrt{\\frac{1}{2}+\\frac{1}{2}\\sqrt{\\frac{1}{2}}}\\sqrt{\\frac{1}{2}+\\frac{1}{2}\\sqrt{\\frac{1}{2}+\\frac{1}{2}\\sqrt{\\frac{1}{2}}}}\\cdots\\]", "Solution_1": "$\\sin \\frac{\\pi}{2^{n}}\\cdot \\prod_{k=2}^{n}\\cos \\frac{\\pi}{2^{k}}= \\frac{1}{2^{n-1}}\\cdot \\sin \\frac{\\pi}{2}= \\frac{1}{2^{n-1}}$, so $\\lim_{n \\to \\infty}\\prod_{k=2}^{n}\\cos \\frac{\\pi}{2^{k}}= \\lim_{n \\to \\infty}\\frac{1}{2^{n-1}\\sin \\frac{\\pi}{2^{n}}}= \\frac{2}{\\pi}\\lim_{n \\to \\infty}\\frac{\\frac{\\pi}{2^{n}}}{\\sin \\frac{\\pi}{2^{n}}}= \\frac{2}{\\pi}$" } { "Tag": [ "calculus", "HMMT", "calculus computations" ], "Problem": "Can anybody recommend an effective book to study from in order to learn HMMT-level calculus?", "Solution_1": "What does HMMT stand for?", "Solution_2": "hello, could it be that he meant this\r\nhttp://web.mit.edu/hmmt/www/\r\nSonnhard.", "Solution_3": "I guess you're right, Sonnhard. \r\n\r\nHaving had a look at the website, I can't think of any book which would be appropriate or useful.\r\n\r\nSorry." } { "Tag": [ "AMC", "AIME" ], "Problem": "[removed] upon the person below's request", "Solution_1": "You should remove this problem for now bugzpodder. I'll explain why later.", "Solution_2": "....... okay i removed it (i will do it right now actually)... why??", "Solution_3": "It was similar to a problem on the AMC-12B. Wait til tomorrow then we can discuss it. It's a nice problem." } { "Tag": [ "calculus", "integration", "function", "derivative", "geometry", "rectangle", "trapezoid" ], "Problem": "Can someone explain to me the Fundamental Theorem of Calculus?\r\n\r\nI can do the exercises in my textbook using it, but I have no intuitive or geometric understanding of it. In particular, I am confused about the change of variables. Why is there both a \"t\" variable and an \"x\" variable? Is the integral a function of x or t? Also, why can you arbitrarily choose a constant on the lower end of the integral and still not affect the answer to a problem?", "Solution_1": "Okay, this is going to be hard to explain without pictures to illustrate - but I'll try my best...\r\n\r\nFirst of all, lets discuss why the Fundamental Theorem of Calculus is so important. Before The Fundamental Theorem of Calculus you have two completely different kinds of integrals. You have indefinite integrals (or anti-derivatives) and definite integrals. They are two very different things. The indefinite integral of a function is a function, whereas the definite integral of a function is a number. And they are defined in two very different ways. The fundamental theorem of calculus ties these two things together - making it possible to actually compute definite integrals without resorting to actually taking the limit of Riemann Sums.\r\n\r\nBasically, the FTC gives you a fancy way of adding things up - without actually having to add them up!!\r\n\r\nOkay, now that the motivation is out of the way, lets talk about what a definite integral is...\r\n\r\nIntuitively you know that the definite integral of a positive function f(t) from t = a to b, tells you the \"area\" under that function f(t) between a and b. For a specific function f(t), and a and b, it gives you a number. Well first of all, there's nothing special about that variable t. It's just a \"dummy variable\". The function would still be the same if we replaced that t by an x or a y or anything else. It's just something used to signify that this is a function of this one thing, t, and wherever else I use it I mean the same thing.\r\n\r\nNow the a and the b are specific set numbers. But if we replace that fixed number b with something that can move, say x. We don't get a number from the definite integral, we get something that changes with x, i.e. a function of x!!\r\n\r\nLets call this function G(x). So G(x) = the integral from a to x of f(t) dt. So what if we wanted to find the derivative of G(x)? Well we apply our old definition of derivative:\r\n\r\nG'(x) = lim h->0 [G(x+h) - G(x)]/h\r\n\r\nBut what is G(x+h) - G(x) graphically? It is the area under f(t) between x and x+h! And as h gets smaller and smaller, this gets closer and closer to being like one of our rectangles from our Riemann Sum - with a height of f(x), and a width of h!\r\n\r\nSo G'(x) = f(x). This is the fundamental theorem of calculus. Lets take a second to think about what it means. It tells us that this thing, G(x), defined by a definite integral of a function, is equal to the anti-derivative (i.e. indefinite integral) of the same function! This is the first thing that you have to says that definite integrals have anything to do with anti-derivatives; and it is very good news! Because taking Riemann Sums is very hard (think Trapeziod Rule and Midpoint Approximation and all of that), but finding anti-derivatives is (well, relatively) easy.\r\n\r\nThe distinction between that x and the t though, is that G is a function of the x, which is in the endpoint of the integral, and t is just a dummy variable. It can be replaced by anything. It's almost like a place holder. And that number that's on the bottom, the a - that can be replaced by anything because all it is going to do is change G by a constant. It's basically the \"constant of integration\" that you get when you take the indefinite integral of f. But since the FTC says that G'(x) = f(x), you see that changing G by a constant doesn't affect the statement of the theorem. So in a sense, the lower endpoint, a, of the integral is sort of like a place holder as well. The x is the thing that \"does all the work\" and ties G and the f together. All the \"action\" happens at x!!", "Solution_2": "That was a really good explanation, Gauss.", "Solution_3": "The weird thing is TI-89s are sort of stupid. If you ever try int(x^2,x,0,x) or anything that has x in both the interval and the name. it will give you an error, although it is legal and should give you a 1/3*x^3.", "Solution_4": "No, the TI is right. While the variable of integration (the thing you integrate with respect to) is just a \"dummy variable\", you should never use that same variable in the endpoints of your integral. For one thing, when you use a variable in a problem it's understood that it should have the same context throughout the problem. You shouldn't use the same variable for two different things.\r\n\r\nSecond of all, from the standpoint of how the calculator computes integrals, you're telling it to use the same memory slot (x) for two different things.\r\n\r\nYou [i]can[/i] use a variable both within the integral and in the endpoints of integration. You just can't use it as the variable of integration. Once it's chosen, that variable has a specific meaning.", "Solution_5": "Thanks! That must mean that Maple is lying to me. :cry:", "Solution_6": "I think I understand it now; the thing that I had the most difficulty understanding was how we went from the expression for G'(x) to just f(x). So-- let me restate it and correct me if I am wrong-- basically, as you said, G(x + h) - G(x) is the area from some number a to x + h minus the area from a to x, which gives us a rectangle with area f(x) * h, and lim (h->0) G(x + h) - G(x) / h = lim (h->0) f(x) * h / h = f(x)? Does this sound right?", "Solution_7": "Exactly! We're leaving out some of the pedantic details about how f(t) needs to be continuous and blah blah blah.. but basically that's it." } { "Tag": [ "LaTeX", "trigonometry", "real analysis", "real analysis unsolved" ], "Problem": "how would i cancel out (2n)! with (2(n+1))!; how would i manipulate the second term to make it easier\r\n\r\nits a part a question which leads to \r\n\r\n{ [(-1)^(n+1) x^(2(n+1))] / [(2(n+1))!] } { [(2n)!] / [(-1)^n x ^(2n) ]\r\n\r\nalso, how do you get the symbols like some people have in their post. when i hover over these symbols it says stuff like what i have above so i'm guessing it's some sort of script they type. it looks much neater that way and i havent been able to figure out how to do it", "Solution_1": "[quote=\"aitch\"]\n\n\nalso, how do you get the symbols like some people have in their post. when i hover over these symbols it says stuff like what i have above so i'm guessing it's some sort of script they type. it looks much neater that way and i havent been able to figure out how to do it[/quote]\r\n\r\nThat's latex. In the end everyone here ends up learning it. There is a button if you scroll up : Latex help and it takes you to a wonderful part of this site explaining it [url]http://www.mathlinks.ro/LaTeX/AoPS_L_About.php[/url]\r\n\r\nthe point basically is that you put dollar signs before and after your math text \r\n$\\sqrt{x}$ is an example of what happens when you put \\sqrt{x} between dollar signs\r\n\r\nnow about your question\r\n\r\n$\\frac{\\frac{ (-1)^{n+1} x^{2(n+1)}} { (2(n+1))! } } { \\frac{ (-1)^n x ^{2n} }{(2n)!}}$\r\nwas this what you meant\r\n\r\nyou probably are proving that the taylor sequence for cosine converges absolutely?\r\n\r\nwell $(2(n+1))!=(2 n) ! (2 n+1)(2n +2)$" } { "Tag": [ "superior algebra", "superior algebra unsolved" ], "Problem": "Prove that the absolute Galois group of a number field has trivial center.", "Solution_1": "And something more difficult: Show that every automorphism of the absolute Galois group of $\\mathbb{Q}$ is inner." } { "Tag": [ "\\/closed" ], "Problem": "Which section of the aops forum would it be appropriate to ask where to find other other cool forums related to academics?", "Solution_1": "Right here would be a great place!\r\n;)", "Solution_2": "Hm.. Okay! :)\r\n\r\nAops has always been the main center for learning new math and such online, and I'm curious whether there are other cool academic websites that are focal points for learning. \r\n\r\nWhat other academic related forums do you like?", "Solution_3": "[url=http://www.biology-online.org/biology-forum/]Biology-Online[/url] is a nice forum.", "Solution_4": "Of course [url=http://www.physicsforums.com/]Physics Forum[/url] is good for Physics." } { "Tag": [ "algebra proposed", "algebra" ], "Problem": "Consider the expressions of the form $ x\\plus{}yt\\plus{}zt^2$ with $ x,y,z$ rational numbers and $ t^3\\equal{}2$.\r\n\r\nProve that:\r\nIf $ x\\plus{}yt\\plus{}zt^2\\neq0$ then there $ u,v,w$ (rational numbers) such that $ (x\\plus{}yt\\plus{}zt^2)(u\\plus{}vt\\plus{}wt^2)\\equal{}1$\r\n\r\nThanks", "Solution_1": "This follows from the fact that $ \\{x + yt + zt^2: x,y,z\\in\\mathbb Q\\}$ is the subfield $ \\mathbb Q(t)$ of the reals and so every nonzero element has a multiplicative inverse. If you want to prove it directly, I suppose you could solve the following simultaneous equations for $ u,v,w$ in terms of $ x,y,z.$\r\n\\begin{align*} xu + 2zv + 2yw & = 1 \\\\\r\nyu + xv + 2zw & = 0 \\\\\r\nzu + yv + xw & = 0 \\end{align*}" } { "Tag": [], "Problem": "Prove that the product of $r$ consecutive integers is divisble by $r!$.", "Solution_1": "hmmm..\r\n\r\nfor r consecutive integers, at least one is divisible by r because from n to n+r (mod r), the range is from 0 to r-1 no matter what. The same can be used with integers from 1 to r, making the product of r consecutive integers divisible by r!", "Solution_2": "thats one way of looking at it, but more generally the formula for the the integer C( n, r ) shows that the product of $r$ consecutive integers is divisible by $r!$." } { "Tag": [ "LaTeX" ], "Problem": "For how many positive integers a does there exist a real number x so that \\sqrt{a-x}+\\sqrt{a+x}= a?\r\n\r\nA. 0\r\nB. 1\r\nC. 2\r\nD. 3\r\nE. 4", "Solution_1": "[quote=\"kimigtsc\"]For how many positive integers a does there exist a real number x so that \\sqrt{a-x}+\\sqrt{a+x}= a?\n\nA. 0\nB. 1\nC. 2\nD. 3\nE. 4[/quote]\r\n\r\nFor $\\LaTeX$, embed your code with dollar signs, as follows:\r\n[code]$\\sqrt{a-x}+\\sqrt{a+x}= a$[/code]\r\n\r\n[hide=\"Anyways,\"]$\\sqrt{a-x}+\\sqrt{a+x}= a$\n\nSquaring both sides,\n$a-x+2\\sqrt{a^{2}-x^{2}}+a+x = a^{2}$\n$a^{2}-a = 2\\sqrt{a^{2}-x^{2}}$\n\nSquaring both sides again,\n$a^{4}-2a^{3}+a^{2}= 4(a^{2}-x^{2})$\n$-a^{4}+2a^{3}+3a^{2}= 4x^{2}$\n\nNotice that if the LHS is bigger than or equal to $0$, then we are done. Thus,\n$-a^{4}+2a^{3}+3a^{2}\\geq 0$\n$a^{4}\\leq 2a^{3}+3a^{2}$\n\nSince $a\\neq 0$ (it is positive), we may divide by $a^{2}$:\n$a^{2}\\leq 2a+3$\n$a^{2}-2a-3\\leq 0$\n$(a-3)(a+1)\\leq 0$\n$-1\\leq a\\leq 3$\n\nSo, the values are:\n$1, 2, 3$\n\nSo $3$, $(d)$[/hide]", "Solution_2": "I got the same answer, but a different set for $a$\r\n\r\n[hide]\n$\\sqrt{a-x}+\\sqrt{a+x}\\leq 2\\sqrt{a}\\implies a \\leq 2\\sqrt{a}\\implies a \\leq 4$ (x=0 case)\n$\\sqrt{a-x}+\\sqrt{a+x}\\geq \\sqrt{2a}\\implies a \\geq \\sqrt{2a}\\implies a \\geq 2$ (x=a case)\n\n$a \\in \\{2,3,4 \\}$\n\n$\\boxed{D. \\; 3}$\n\n[/hide]", "Solution_3": "[hide]Square, $\\implies 2a+2\\sqrt{a^{2}-x^{2}}=a^{2}$.\nRearrange and square, $\\implies 4a^{2}-4x^{2}=a^{4}-4a^{3}+4a^{2}$.\nCancel, rearrange, $a^{3}(4-a)=4x^{2}$.\nSince the right side has to be non-negative, $a\\in\\{1,2,3,4\\}$. $E.4$[/hide]", "Solution_4": "[hide]\n$\\sqrt{a-x}+\\sqrt{a+x}= a$\n$a-x+a+x+2\\sqrt{(a+x)(a-x)}= a^{2}$\n$\\sqrt{(a+x)(a-x)}= \\frac{(a)(a-2)}{2}$\n$\\frac{(a+x)+(a-x)}{2}\\ge \\frac{(a)(a-2)}{2}$\n$2a \\ge a(a-2)$\n$2 \\ge a-2$\n$4 \\ge a$\n$a \\in \\{1, 2, 3, 4\\}$\n$\\boxed{E. \\text{ }4}$\n[/hide]", "Solution_5": "$\\sqrt{1+x}+\\sqrt{1-x}=1 \\implies 2+2\\sqrt{1-x^{2}}=1 \\implies \\sqrt{1-x^{2}}=-\\frac{1}{2}$\r\n\r\nHence $a=1$ does not produce a real solution in $x$.\r\n\r\nRefer to my previous post." } { "Tag": [ "number theory unsolved", "number theory" ], "Problem": "Prove or disprove:\r\n\r\nFor all primes p>=3, there exists a prime between (p-1)/2 and p.\r\n\r\nThe Goldbach conjecture is not allowed.\r\n\r\n\r\n\r\nI thought of this problem while trying to solve another problem. I don't know the solution or whether there is some well known result that trivializes this conjecture.", "Solution_1": "Assume there does not exist a prime in that range. Take the largest prime less than or equal to $ \\frac{p\\minus{}1}{2}$, say prime $ m$. By [b]Bertrand's Postulate[/b] there exists a prime $ l$ between $ m$ and $ 2m$. Therefore, there exists a prime $ l$ greater than $ \\frac{p\\minus{}1}{2}$ (by construction of $ m$) but less than $ p$ as $ m\\leq \\frac{p\\minus{}1}{2}$. Then $ l$ lies in that range from $ \\frac{p\\minus{}1}{2}$ to $ p$ and we get a contradiction. Therefore, there always exists a prime in that range.", "Solution_2": "Thanks a lot.\r\n\r\nI need to learn more number theory.\r\n :)", "Solution_3": "Do you have a proof for Bertrand's Postulate , it is a famous method :)", "Solution_4": "Here is a proof of Bertrand's Postulate: http://www.math.niu.edu/~rusin/known-math/97/bertrand. This is a very famous theorem, so searching the proof in google will give you what you want.", "Solution_5": "Oh yes ,rem . Some one can retype it on the different version (pdf,djvu), in this link, the proof is difficult to pronounce :)", "Solution_6": "You can read on the book:Elementary theory of number' of Author Sierpinsky.", "Solution_7": "Thanks TTsphn. Can you upload this book to the forum ? :D . I don't have much books . :(" } { "Tag": [ "vector", "parameterization", "calculus", "calculus computations" ], "Problem": "Say I have a plane:\r\n\r\n$ 3x \\minus{} 3y \\plus{} 8z \\equal{} 21$\r\n\r\nand it's normal is therefore $ 3\\vec{i} \\minus{} 3\\vec{j} \\plus{} 8\\vec{k}$\r\n\r\nso if I want a vector parallel with the plane I should do a dot product of that normal vector with this unknown vector and it should be 0. However I am quite confused on how to do it.... \r\n\r\nCan anyone help?", "Solution_1": "So you want a vector $ \\left< a, b, c \\right>$ such that $ 3a \\minus{} 3b \\plus{} 8c \\equal{} 0$. It should not be hard to come up with small integers $ a, b, c$ such that this is true; speaking more systematically, you've got an overdetermined system and the set of all solutions has two free parameters, so you're free to set, for example, $ a \\equal{} b \\equal{} 1$ and solve for $ c$.", "Solution_2": "[b]1).[/b]$ 3x \\minus{} 3y \\plus{} 8z \\minus{} 21 \\equal{} 0\\Leftrightarrow 3.(x \\minus{} 2) \\minus{} 3(y \\minus{} 3) \\plus{} 8.(z \\minus{} 3) \\equal{} 0.$\r\n[b]2).[/b]$ \\mbox{Let }P_0(2;3;3), P(x;y;z) \\mbox{ and } \\overrightarrow{v} \\equal{} 3.\\overrightarrow{i} \\minus{} 3.\\overrightarrow{j} \\plus{} 8.\\overrightarrow{k},\\mbox{ then: }$\r\n$ \\overrightarrow{P_0P} \\equal{} \\overrightarrow{OP} \\minus{} \\overrightarrow{OP_0} \\equal{} (x.\\overrightarrow{i} \\plus{} y.\\overrightarrow{j} \\plus{} z.\\overrightarrow{k}) \\minus{} (2.\\overrightarrow{i} \\plus{} 3.\\overrightarrow{j} \\plus{} 3.\\overrightarrow{k}) \\equal{} \\\\\r\n\\equal{} (x \\minus{} 2).\\overrightarrow{i} \\plus{} (y \\minus{} 3).\\overrightarrow{j} \\plus{} (z \\minus{} 3).\\overrightarrow{k}.$\r\n[b]3).[/b]$ \\overrightarrow{P_0P}\\perp\\overrightarrow{v}\\Leftrightarrow\\overrightarrow{P_0P}.\\overrightarrow{v} \\equal{} 0\\Leftrightarrow 3.(x \\minus{} 2) \\minus{} 3(y\\minus{} 3) \\plus{} 8.(z \\minus{} 3) \\equal{} 0.$", "Solution_3": "how did you get the point 2,3,3 from? is it just a random guess??\r\n\r\nI am still confused, if $ 3(x\\minus{}2)\\minus{} 3(x\\minus{}3) \\plus{} 8(x\\minus{}3)$ is the answer, that is an equation of a plane and it's not a vector... Is it the same?", "Solution_4": "One thing is that, if you look at no.1, it doesnt have an y and z, because everything is there", "Solution_5": "$ \\mbox{Ok! My error CORECTED!}$", "Solution_6": "$ 3(x \\minus{} 2) \\minus{} 3(y \\minus{} 3) \\plus{} 8(z \\minus{} 3) \\equal{} 0$ is not 'the answer', it is the equation of the original plane you gave.\r\n\r\nI think you should give some more thought to t0rajir0u's post. The vectors parallel to the plane are those $ \\left < a,b,c\\right >$ satisfying $ 3a \\minus{} 3b \\plus{} 8c \\equal{} 0$. Two such vectors are, for example, $ v_1 \\equal{} \\left < 1,1,0\\right >$ and $ v_2 \\equal{} \\left < 0,8,3\\right >$. All linear combinations of these two vectors (i.e. $ c_1v_1 \\plus{} c_2v_2$ where $ c_1,c_2$ are real numbers not both zero) are also parallel to the original plane (try to think about why this is true from both a geometric and algebraic point of view).\r\n\r\nThe equation that the components of the vector satisfy looks very similar to the original equation of the plane, and this is not by coincidence -- the set of vectors parallel to a plane themselves form a plane passing through the origin and parallel to the original plane. Trying to visualize this might yield some insight that will make the algebra more transparent.", "Solution_7": "$ \\mbox{If the vectors } \\\\\r\n\\overrightarrow{v_1} \\equal{} \\overrightarrow{i} \\plus{} \\overrightarrow{j} \\mbox{ and } \\overrightarrow{v_2} \\equal{} 8.\\overrightarrow{j} \\plus{} 3.\\overrightarrow{k} \\\\\r\n\\mbox{parallels to the plane } \\alpha, \\mbox{ then the vector: } \\\\\r\n\\overrightarrow{v} \\equal{} \\overrightarrow{v_1}\\times\\overrightarrow{v_2} \\equal{} \\left|\\begin{array}{ccc}\\overrightarrow{i} & \\overrightarrow{j} & \\overrightarrow{k} \\\\\r\n1 & 1 & 0 \\\\\r\n0 & 8 & 3 \\end{array}\\right| \\equal{} 3.\\overrightarrow{i} \\minus{} 3.\\overrightarrow{j} \\plus{} 8.\\overrightarrow{k} \\mbox{ is normal to the plane }\\alpha.$" } { "Tag": [ "algebra", "polynomial", "algebra proposed" ], "Problem": "A polynomial $ f(x)\\equal{}x^3\\plus{}ax^2\\plus{}bx\\plus{}c$ is such that $ b<0$ and $ ab\\equal{}9c$. Prove that the polynomial $ f$ has three different real roots.", "Solution_1": "$ f(x)\\equal{}x^3\\plus{}ax^2\\plus{}bx\\plus{}\\frac{ab}{9}$\r\n\r\nLet $ b\\equal{}\\minus{}d$ with $ d>0$\r\n\r\n$ f(x)\\equal{}x^3\\plus{}ax^2\\minus{}dx\\minus{}\\frac{ad}{9}$\r\nIf $ a\\equal{}0$ it is very easy the proof. \r\n\r\nCase 1: If $ a>0$\r\n$ f(\\minus{}a)\\equal{}8ad$\r\n$ f(0)\\equal{}\\minus{}ad$\r\n$ f(\\sqrt{d})\\equal{}8ad$\r\nThen $ f(x)$ has at least two different real roots, then it has three real roots.\r\n\r\nCase2: If $ a>0$\r\n$ f(\\minus{}a)\\equal{}8ad$\r\n$ f(0)\\equal{}\\minus{}ad$\r\n$ f(\\minus{}\\sqrt{d})\\equal{}8ad$\r\nThen $ f(x)$ has at least two different real roots, then it has three real roots.\r\n\r\nWe only have two prove that if it has two different real roots, then it has three different real roots.\r\nBy contradiction, suppose that it has two equal roots. Then:\r\n\r\n$ f(x)\\equal{}(x\\minus{}r)^2(x\\minus{}t)$\r\nBy Viette`s formulas:\r\n$ a\\equal{}\\minus{}(2r\\plus{}t)$\r\n$ b\\equal{}r^2\\plus{}2tr$\r\n$ ab\\equal{}\\minus{}9r^2c$\r\n\r\n$ \\implies \\minus{}(2r\\plus{}t)r(r\\plus{}2t)\\equal{}\\minus{}9r^2t$\r\n$ \\implies (r\\minus{}t)^2\\equal{}0$\r\n$ \\implies r\\equal{}t$\r\nWhich is an contradiction because $ f$ has at least two different roots." } { "Tag": [ "inequalities", "trigonometry", "calculus", "derivative" ], "Problem": "$ x$ is real number such that $ 0 \\le x \\le \\frac{\\pi }{2}.$\r\n\r\nProve this inequality: $ \\frac{{\\sin x \\plus{} 2\\cos \\frac{x}{2}}}{{\\cos x \\plus{} 2\\sin \\frac{x}{2}}} \\ge 1 \\plus{} \\frac{{\\sqrt 2 }}{2}$\r\n\r\nIt's very nice!!! :D", "Solution_1": "Please give me a solution!!! :wink:", "Solution_2": "Clue: You can use derivation to solve th\u00ed problem!!! :wink:", "Solution_3": "[hide=\"Along the lines of...\"]We have equality for $ x=\\frac{\\pi}{2}$ We take the derivative of the expression, and we get ${ \\frac{-2\\cos x (\\sin x+2\\cos \\frac{x}{2})}{(\\sin x + 2\\cos \\frac{x}{2})^2}}$, which is obviously nonpositive for $ 0\\le x \\le \\frac{\\pi}{2}$. Thus the expression is stricly decreasing for $ 0 \\le x \\le \\frac{\\pi}{2}$, and the inequality holds.[/hide]", "Solution_4": "Although this problem is very easy, but can you give me a detail solution? Thank you very much! :oops:", "Solution_5": "Is there a non-calculus approach?", "Solution_6": "yep.\r\n\r\nsince the ineqaulity says $ \\frac{{\\sin x \\plus{} 2\\cos \\frac{x}{2}}}{{\\cos x \\plus{} 2\\sin \\frac{x}{2}}}$ is greater or equal to $ 1\\plus{}\\frac{\\sqrt{2}}{2}$, we must consider the lower bound.\r\n\r\na fraction gets small when the numerator is small and the denominator is large.\r\n\r\nsmallest when $ x\\equal{}\\frac{\\pi}{2}$.\r\n\r\nthen the inequality becomes $ \\frac{1\\plus{}2*\\frac{\\sqrt{2}}{2}}{0\\plus{}2*\\frac{\\sqrt{2}}{2}}\\equal{}\\frac{1\\plus{}\\sqrt{2}}{\\sqrt{2}}\\equal{}\\frac{\\sqrt{2}\\plus{}2}{2}\\equal{}1\\plus{}\\frac{\\sqrt{2}}{2}$\r\n\r\nQ.E.D.\r\n\r\neasy for a pre-olympiad problem", "Solution_7": "That result was not very rigorous. At least the calculus solution clearly demonstrates that it decreases.", "Solution_8": "With $ f(x) \\equal{} \\frac{{\\sin x \\plus{} 2\\cos \\frac{x}{2}}}{{\\cos x \\plus{} 2\\sin \\frac{x}{2}}}$, I solve to $ f'(x) \\equal{} \\frac{{\\sin \\frac{{3x}}{2} \\minus{} 1}}{{\\left( {\\cos x \\plus{} 2\\sin \\frac{x}{2}} \\right)^2 }}$ :wink:" } { "Tag": [], "Problem": "A school is creating a four-digit student code system. For security reasons, the code cannot start with an even number. How many codes are possible?", "Solution_1": "There are $ 5$ possibilities for the first digit and $ 10$ possibilities for each of the other digits. $ 5\\times{10^{3}}\\equal{}\\boxed{5000}$.", "Solution_2": ":)\r\n\r\nExactly, gaussintraining\r\n\r\n5 for first because 1,3,5,7,9 are odd, so 5*10*10*10...\r\nVery weird \"security\", don't you think?" } { "Tag": [], "Problem": "I think Intro to NT has a typo but i'm not sure.\r\nIt states that the binomial expression when $ \\ 10_(b \\plus{} 1)$\r\nWhen b is a positive integer,\r\n$ \\ (b \\plus{} 1)^{2} \\equal{} b^{2} \\plus{} b \\plus{} 1$ I thought it should be $ \\ b^{2}\\plus{}2b\\plus{}1.$\r\nAm i right?", "Solution_1": "[quote=\"cgyao15\"]\nIt states that the binomial expression when $ \\ 10_(b \\plus{} 1)$\n[/quote]\r\n\r\n\r\nwhat does that mean?", "Solution_2": "He means to say $ 10_{b \\plus{} 1}$ which is equivalent to $ 1\\cdot (b \\plus{} 1) \\plus{} 0 \\equal{} b \\plus{} 1$. From there, I assume that you are considering the square of $ 10_{b \\plus{} 1}$ in general which is $ (b \\plus{} 1)^2 \\equal{} b^2 \\plus{} 2b \\plus{} 1.$ This is most likely a typo so I recommend that notify AoPS staff.\r\n\r\nWhat page is this on? I will check to see if I also have that typo. If not, it means that you have an older edition.", "Solution_3": "According to [url=http://www.artofproblemsolving.com/BookLinks/IntroNumberTheory/links.php#errata]this,[/url] it seems like they haven't caught onto it yet.", "Solution_4": "@b555 err yes i put parenthesis instead of curvy brackets.\r\nThx ne ways guys." } { "Tag": [ "\\/closed" ], "Problem": "I wish the board wouldn't automatically close my scripts. That is, when I post italics, which begin with \"[i ],\" I have a habit of typing in the [/i]. It's really annoying when that also appears at the end of my post, because the board auto-closed the script.\r\n\r\nWould it be able to make that feature an on-off feature?", "Solution_1": "I have never enoutered this [i]problem[/i] :) Maybe it's you doing it wrong? :?", "Solution_2": "Yeah, it happens all the time to me also. And I've seen it many times on other peoples post - a [ /spoiler ] or something like that which just appears. Surely it wouldn't be too hard to either stop it doing that altogether or make it actually check they aren't closed in the first place.\r\n\r\nedit - note that it only happens when you click the [b]B[/b] button or any of the buttons above the message but close it by hand yourself. Not if you type both or if you click and unclick both.", "Solution_3": "[quote=\"TripleM\"]edit - note that it only happens when you click the [b]B[/b] button or any of the buttons above the message but close it by hand yourself. Not if you type both or if you click and unclick both.[/quote]Ah wait, that's a totally different story. If you use the button once, you make sure you use it again. \r\n\r\n[code]After all it's harder to type [/b] than [b] isn't it?! ...[/code]" } { "Tag": [ "geometry", "trigonometry", "incenter", "parallelogram", "inradius", "circumcircle", "angle bisector" ], "Problem": "Let $ I_{a}$ be excenter of triangle $ ABC$ wrt $ A$.excircle of triangle $ ABC$ wrt $ A$ is tangent to $ AB$ and $ AC$ at points $ B'$ and $ C'$.Let $ P$ and $ Q$ be intersecttions of lines $ BI_{a}$ and $ CI_{a}$ with $ B'C'$ respectively and let $ M$ be intersection of $ PC$ and $ BQ$. Prove that if $ L$ is the foot of the prependicular drawn from $ M$ to $ BC$ then $ ML \\equal{} r$($ r$=inradi)", "Solution_1": "It's simple to prove that $ a \\sin\\frac{\\gamma}2 \\cos\\frac{\\beta}2 \\equal{} (p\\minus{}b)\\cos\\frac{\\alpha}2$ cause it's equal to $ a \\sqrt{\\frac{(p\\minus{}b)(p\\minus{}a)}{ab}} \\sqrt{\\frac{p(p\\minus{}b)}{ac}} \\equal{} (p\\minus{}b) \\sqrt{\\frac{p(p\\minus{}a)}{bc}}$.\r\n\r\nThen $ CQ \\equal{} \\frac{\\overline{CC'} \\sin\\angle CC'Q}{\\sin \\angle CQC'} \\equal{} \\frac{(p\\minus{}b)\\cos{\\frac{\\alpha}2}}{\\cos\\frac{\\beta}2}\\equal{}a \\sin\\frac{\\gamma}{2} \\equal{} \\overline{BC} \\cos \\angle BCQ$ and so $ BQ \\perp CQ$ and $ BQ \\parallel CI$ and equally we can say that $ CP \\parallel BI$ and so that the triangle $ \\triangle BIC$ and $ \\triangle BI_aC$ are equal.", "Solution_2": "[quote=\"Amir.S\"]Let $ I_{a}$ be excenter of triangle $ ABC$ wrt $ A$.excircle of triangle $ ABC$ wrt $ A$ is tangent to $ AB$ and $ AC$ at points $ B'$ and $ C'$.Let $ P$ and $ Q$ be intersecttions of lines $ BI_{a}$ and $ CI_{a}$ with $ B'C'$ respectively and let $ M$ be intersection of $ PC$ and $ BQ$. Prove that if $ L$ is the foot of the prependicular drawn from $ M$ to $ BC$ then $ ML \\equal{} r$($ r$=inradi)[/quote]\r\n[hide=\"Solution\"]\nLet $ I$ be the incenter of $ \\triangle ABC$, let the incircle be tangent to $ BC$ at $ A'$, and let the midpoint of $ BC$ be $ X$. Also, let $ \\angle BAC \\equal{} 2\\alpha$ and $ \\angle ABC \\equal{} 2\\alpha$, so $ \\angle ACB \\equal{} 180 \\minus{} 2\\alpha$. Also, $ I$ and $ I_a$ lie on the angle bisector of $ \\angle BAC$, so $ A$, $ I$, and $ I_a$ lie on a line. Now, notice that $ \\angle IBA \\equal{} \\angle IBC \\equal{} \\beta$, $ \\angle IAB \\equal{} \\angle IAC \\equal{} \\alpha$, $ \\angle ICA \\equal{} \\angle ICB \\equal{} 90 \\minus{} \\alpha \\minus{} \\beta$, $ \\angle B'BC \\equal{} 180 \\minus{} 2\\beta$, and $ \\angle BCC' \\equal{} 2\\alpha \\plus{} 2\\beta$. It follows that $ \\angle I_aBB' \\equal{} \\angle I_aBC \\equal{} 90 \\minus{} \\beta$ and $ \\angle I_aCB \\equal{} \\angle I_aCC' \\equal{} \\alpha \\plus{} \\beta$. Thus, $ \\angle IBI_a \\equal{} 90 \\minus{} \\beta \\plus{} \\beta \\equal{} 90$ and similalry, $ \\angle ICI_a \\equal{} 90$. Thus, $ ICI_aB$ is cyclic. It follows that $ \\angle BI_aI \\equal{} \\angle ICB \\equal{} 90 \\minus{} \\alpha \\minus{} \\beta$ and $ \\angle II_aC \\equal{} \\angle IBC \\equal{} \\beta$. Since $ AC'$ and $ AB'$ are tangents to the $ A$-excircle, we see that $ AC' \\equal{} AB'\\implies \\angle AB'C' \\equal{} \\angle AC'B' \\equal{} 90 \\minus{} \\frac {\\angle BAC}{2} \\equal{} 90 \\minus{} \\alpha$. It follows that\n\\[ \\angle BB'Q \\equal{} 90 \\minus{} \\alpha \\equal{} 180 \\minus{} (90 \\plus{} \\alpha) \\equal{} 180 \\minus{} (\\frac {\\angle BAC}{2} \\plus{} 90) \\equal{} 180 \\minus{} \\angle BIC \\equal{} \\angle BI_aC\n\\]so $ BRI_aB'$ is cyclic. Thus, $ \\angle BRI_a \\equal{} 180 \\minus{} \\angle I_aB'B \\equal{} 90\\implies BR\\perp I_aC$ and similarly, $ CQ\\perp I_aB$. This means that $ M$ is the orthocenter of $ \\triangle I_aBC$, so $ I_aM\\perp BC$. It follows that $ L$ is the tangency point between the $ A$ excircle and $ BC$. Also, $ BR\\perp I_aC$ and $ IC\\perp I_aC$, so $ MB\\parallel IC$ and similarly, $ IB\\parallel MC$, so $ ICMB$ is a parallelogram, from which it follows that $ IM$ passes through the midpoint of $ BC$, which is $ X$. Since $ L$ is the $ A$-excircle tangency point with $ BC$, we see that $ A'X \\equal{} XL$, $ \\angle MLX \\equal{} 90 \\equal{} \\angle IA'X$, and $ \\angle IXA' \\equal{} \\angle MXL$, so $ \\triangle MXL\\cong \\triangle IXA'\\implies r \\equal{} IA' \\equal{} LM$, as desired. [/hide]", "Solution_3": "[quote=\"Amir.S\"] [color=darkred]The $ A$ - exincircle $ w_a \\equal{} C(I_a,r_a)$ of $ \\triangle ABC$ is tangent to $ AB$ , $ AC$ at the points $ B'$ , $ C'$ . Denote the incircle \n\n$ w \\equal{} C(I,r)$ of $ \\triangle ABC$ and $ \\begin{array}{ccc} P\\in BI_{a}\\cap B'C' & ; & Q\\in CI_{a}\\cap B'C' \\\\\n \\\\\nM\\in PC\\cap BQ & ; & L\\in BC\\ ,\\ ML\\perp BC\\end{array}$ . Prove that $ ML \\equal{} r$ .[/color] [/quote]\r\n\r\n[color=darkblue][b][u]Proof.[/u][/b] \n\n$ \\blacktriangleright$ The point $ I$ is the orthocenter of the triangle $ I_aI_bI_c$ (evidently). Prove that the quadrilateral $ BPQC$ is cyclically.[/color]\r\n\r\n[hide=\"Proof.\"]\n[color=darkblue]$ \\left\\|\\begin{array}{c} m(\\widehat {BB'P}) \\equal{} m(\\widehat {BB'C'}) \\equal{} 90^{\\circ} \\minus{} \\frac A2 \\\\\n \\\\\nm(\\widehat {B'BP}) \\equal{} m(\\widehat {B'BI_a}) \\equal{} 90^{\\circ} \\minus{} \\frac B2\\end{array}\\right\\|$ $ \\implies$ $ \\left\\|\\begin{array}{c} m(\\widehat {QPI_a}) \\equal{} m(\\widehat {B'BP}) \\equal{} 90^{\\circ} \\minus{} \\frac C2 \\\\\n \\\\\nm(\\widehat {BCQ}) \\equal{} m(\\widehat {BCI_a}) \\equal{} 90^{\\circ} \\minus{} \\frac C2\\end{array}\\right\\|$ $ \\implies$ \n\n$ \\widehat {QPI_a}\\equiv\\widehat {BCQ}$ , i.e. $ BPQC$ is cyclically.[/color][/hide]\n[color=darkblue]$ \\blacktriangleright$ Denote $ T\\in I_aM\\cap BC$ . Show that $ \\frac {TB}{TC} \\equal{} \\frac {p \\minus{} c}{p \\minus{} b}$ , i.e. $ \\overline {I_aMT}\\perp BC$ (the point $ T\\equiv L$ and belongs to the circle $ w_a$ ).[/color]\n\n[hide=\"Proof.\"]\n[color=darkblue]$ \\left\\|\\begin{array}{c} \\frac {PB}{PI_a} \\equal{} \\frac {B'B}{B'I_a}\\cdot\\frac {\\sin\\widehat {PB'B}}{\\sin\\widehat {PB'I_a}} \\equal{} \\frac {p \\minus{} c}{r_a}\\cdot\\frac {\\sin\\left(90^{\\circ} \\minus{} \\frac A2\\right)}{\\sin\\frac A2} \\equal{} \\frac {p \\minus{} c}{r_a}\\cdot\\cot\\frac A2 \\\\\n \\\\\n\\frac {QC}{QI_a} \\equal{} \\frac {C'C}{C'I_a}\\cdot\\frac {\\sin\\widehat {QC'C}}{\\sin\\widehat {QC'I_a}} \\equal{} \\frac {p \\minus{} b}{r_a}\\cdot\\frac {\\sin\\left(90^{\\circ} \\minus{} \\frac A2\\right)}{\\sin \\frac A2} \\equal{} \\frac {p \\minus{} b}{r_a}\\cdot\\cot \\frac A2\\end{array}\\right\\|$ $ \\implies$ $ \\frac {PB}{PI_a}\\cdot\\frac {QI_a}{QC} \\equal{} \\frac {p \\minus{} c}{p \\minus{} b}$ . \n\nApply [b]Ceva's theorem[/b] to $ M$ in $ \\triangle BCI_a$ : $ \\frac {PB}{PI_a}\\cdot\\frac {QI_a}{QC}\\cdot\\frac {TC}{TB} \\equal{} 1$ $ \\implies$ $ \\frac {p \\minus{} c}{p \\minus{} b}\\cdot\\frac {TC}{TB} \\equal{} 1$ $ \\implies$ $ \\frac {TB}{TC} \\equal{} \\frac {p \\minus{} c}{p \\minus{} b}$ .[/color][/hide]\r\n[color=darkblue]$ \\blacktriangleright$ Apply[/color] [url=http://www.mathlinks.ro/Forum/viewtopic.php?t=202247][color=red][b]lemma I[/b][/color][/url] [color=darkblue]to the triangle $ BCI_a$ , where $ I_aL\\perp BC$ and for the point $ M\\in I_aL$ the quadrilateral $ BPQC$ \n\nis cyclically. In conclusion, the point $ M$ is the orthocenter of $ \\triangle BCI_a$ , i.e. $ CM\\perp BI_a$ and $ BM\\perp CI_a$ .[/color]\r\n\r\n$ \\blacktriangleright$ [color=darkblue]Apply[/color] [url=http://www.mathlinks.ro/Forum/viewtopic.php?t=202252][color=red][b]lemma II[/b][/color][/url] [color=darkblue] to $ \\triangle I_aI_bI_c$ with the orthocenter $ I$ , where $ M$ is the orthocenter of $ \\triangle BCI_a$ . \n\nIn conclusion, $ ML$ is equally to the inradius of the orthic $ \\triangle ABC$ w.r.t. the triangle $ I_aI_bI_c$ , i.e. $ ML \\equal{} r$ .[/color]", "Solution_4": "Thank you!", "Solution_5": "We denote as $ P,\\ Q,$ the orthogonal projections of $ C,\\ B$ respectively and it is easy to show that the points $ B',\\ P,\\ Q,\\ C',$ are collinear.\r\n\r\n$ ($ as [b][size=100]SpongeBob[/size][/b] wrote in the problem [url=http://www.mathlinks.ro/Forum/viewtopic.php?t=199330]Excircle & collinear[/url] lately, we have $ \\angle C'QC \\equal{} \\angle LQC \\equal{} \\angle LBI_{a} \\equal{} \\angle B'BI_{a} \\equal{} \\angle B'QI_{a}$ $ \\Longrightarrow$ $ B',\\ Q,\\ C',$ are collinear and similarly the points $ B',\\ P,\\ C',$ are collinear $ ).$\r\n\r\nSo, the point $ M\\equiv BQ\\cap CP,$ is the orthocenter of the triangle $ \\bigtriangleup I_{a}BC$ and then, the point $ L,$ as the orthogonal projection of $ M,$ on $ BC,$ is the tangency point of the $ A$-excircle to $ BC.$\r\n\r\nIt is well known that $ KL \\equal{} KL'$ $ ,(1)$ where $ K$ is the midpoint of the side-segment $ BC$ and $ L'$ is the tangency point of the incircle $ (I),$ to $ BC.$\r\n\r\nBecause of $ IB\\perp BI_{a}$ and $ IC\\perp CI_{a},$ we have that the quadrilateral $ IBI_{a}C$ is cyclic with circumcircle $ (A'),$ taken as diameter the segment $ II_{a}.$\r\n\r\nWe denote the point $ M'\\equiv (A')\\cap I_{a}M$ and so, we have $ ML \\equal{} LM'$ $ ,(2)$\r\n\r\nBecause of $ (1)$ $ \\Longrightarrow$ $ LM' \\equal{} IL' \\equal{} r$ $ ,(3)$\r\n\r\nFrom $ (1),$ $ (2),$ $ (3)$ $ \\Longrightarrow$ $ ML \\equal{} r$ and the proof is completed.\r\n\r\nKostas Vittas." } { "Tag": [ "inequalities", "inequalities proposed" ], "Problem": "Given $ a, b, c \\geq\\ 0$ satisfy $ a^2\\plus{}b^2\\plus{}c^2\\equal{}2(ab\\plus{}bc\\plus{}ca)$. \r\nProve that: $ \\sqrt{1\\plus{}\\frac{a}{b\\plus{}c}}\\plus{}\\sqrt{1\\plus{}\\frac{b}{c\\plus{}a}}\\plus{}\\sqrt{1\\plus{}\\frac{c}{a\\plus{}b}} \\geq\\ 1\\plus{}2\\sqrt{2}$\r\n :)", "Solution_1": "[quote=\"nguoivn\"]Given $ a, b, c \\geq\\ 0$ satisfy $ a^2 + b^2 + c^2 = 2(ab + bc + ca)$(1). \nProve that: $ \\sqrt {1 + \\frac {a}{b + c}} + \\sqrt {1 + \\frac {b}{c + a}} + \\sqrt {1 + \\frac {c}{a + b}} \\geq\\ 1 + 2\\sqrt {2}$\n :)[/quote]\r\nlet$ x^2 = a$,$ y^2 = b$,$ z^2 = c$.we have (1)<=>$ 2\\sum_{cyc}{x^2y^2} - \\sum{x^4} = (x + y + z)\\prod_{cyc}{(x + y - z)} = 0$\r\nso sum two numbers of which must by remain number\r\nasume$ x = y + z = 1$\r\nINEQUALITY<=>$ f(y) = \\frac {1}{\\sqrt {1 + y^2}} + \\frac {1}{\\sqrt {1 + z^2}} + \\frac {1}{\\sqrt {y^2 + z^2} } - \\frac {1 + 2\\sqrt {2}}{\\sqrt {1 + y^2 + z^2}} \\ge 0$\r\n${{ f'(y) = y(\\frac {1 + 2\\sqrt {2}}{(1 + y^2 + z^2)^\\frac {3}{2}} - \\frac {1}{(1 + y^2}^\\frac {3}{2}} - \\frac {1}{(1 + z^2}^\\frac {3}{2}})$\r\nwe have$ (1 + y^2 + z^2)^\\frac {3}{2} = [2(y^2 + z^2 + yz)]^\\frac {3}{2} \\ge 2\\sqrt {2}(y^2 + z^2)^\\frac {3}{2}$\r\nso $ \\frac {2\\sqrt {2}}{(1 + y^2 + z^2)^\\frac {3}{2}} \\le \\frac {1}{(y^2 + z^2)^\\frac {3}{2}}$\r\nand $ \\frac {1}{(1 + y^2 + z^2)^\\frac {3}{2}} \\ge \\frac {1}{(y^2 + z^2)^\\frac {3}{2}}$\r\n=>$ f'(y) \\le 0$\r\n=>$ f(y) \\ge f(1) = 0$\r\ninequality hold when $ (a,b,c)=(t,t,0)$", "Solution_2": "[quote=\"nguoivn\"]Given $ a, b, c \\geq\\ 0$ satisfy $ a^2 \\plus{} b^2 \\plus{} c^2 \\equal{} 2(ab \\plus{} bc \\plus{} ca)$. \nProve that: $ \\sqrt {1 \\plus{} \\frac {a}{b \\plus{} c}} \\plus{} \\sqrt {1 \\plus{} \\frac {b}{c \\plus{} a}} \\plus{} \\sqrt {1 \\plus{} \\frac {c}{a \\plus{} b}} \\geq\\ 1 \\plus{} 2\\sqrt {2}$\n :)[/quote]\r\nIt's stronger than the well-known of Le Trung Kien:\r\nGiven $ a, b, c \\geq\\ 0$ and $ ab\\plus{}bc\\plus{}ca\\equal{}1$. Prove that:\r\n$ \\frac{1}{\\sqrt{a\\plus{}b}}\\plus{}\\frac{1}{\\sqrt{b\\plus{}c}}\\plus{}\\frac{1}{\\sqrt{c\\plus{}a}} \\geq\\ 2\\plus{} \\frac{1}{\\sqrt{2}}$\r\n :)" } { "Tag": [ "Support" ], "Problem": "Hi all,\r\n\r\nI am a college student with a very quantitative background. In the past couple years, I have been interested in going down the career path of a quant. However, recently, after doing more detailed investigation into the different possible careers in finance, I have heard that a lot of quants who have been in the industry for a few years decide that they want to move into the role of a trader. I have also heard that quants often play second fiddle to traders (the work that quants do is supposed to support traders after all), and thus end up being less respected and receiving less compensation than traders.\r\n\r\nTherefore, I would like to ask those of you in the know about the comparison between quants and traders (well, the more quantitative traders, that is). What are the pros and cons of each type of job? Is it easy to switch between the two after working in the industry for a couple of years? How is each role generally compensated (of course, presumably we can only compare quants and traders working at the same firm)?\r\n\r\nAny thoughts/input/advice you may have would be appreciated. Thanks!", "Solution_1": "This is going to vary significantly depending on the firm you work for. There's not going to be a straight and simple answer.", "Solution_2": "Well, I don't really have a specific firm in mind to be honest. I guess two different classes of firms I am wondering about would be investment banks (Goldman, Morgan Stanley, etc...) and the larger, more well-known quant shops/hedge funds (Shaw, Jane Street, Citadel, Two Sigma, etc...). However, I do not know whether there is an answer that would apply to all/most of the firms in either of these two categories.", "Solution_3": "Hey guys, I'm double majoring in business(emphasis finance) and mathematics. I'm in my second year and I still don't know what I want to do. Can you tell the difference between a quant and a trader. Also does a trader need to have a mathematical baackgroud? And what is the level of math needed for a quant, and for a trader?\r\n\r\nthank you..." } { "Tag": [ "LaTeX", "trigonometry", "logarithms" ], "Problem": "Can you align equations in this forum?\r\n\r\nIn LaTeX you use the &.", "Solution_1": "You can do it almost the same way as you normally would [b][i]except[/i][/b] put in a couple of carriage returns after the starting dollar sign. Thus\r\n[code]$\\textdollar$\n\n\\begin{align*}\n\\left(x^n\\right)' &= nx^{n-1} & \\left(e^x\\right)' &= e^x & (\\sin x)' &= \\cos x \\\\\n\\left(\\frac{1}{x^n}\\right)' &= -\\frac{n}{x^{n+1}} & \\left(a^x\\right)' &= a^x\\ln a & (\\cos x)' &= -\\sin x\n\\end{align*}$\\textdollar$[/code] produces\r\n\\begin{align*}\r\n\\left(x^n\\right)' &= nx^{n-1} & \\left(e^x\\right)' &= e^x & (\\sin x)' &= \\cos x \\\\\r\n\\left(\\frac{1}{x^n}\\right)' &= -\\frac{n}{x^{n+1}} & \\left(a^x\\right)' &= a^x\\ln a & (\\cos x)' &= -\\sin x\r\n\\end{align*}\r\n\r\n(Example taken from Kopka & Daly's A Guide to LaTeX)", "Solution_2": "[quote=\"Negative_3\"]Can you align equations in this forum?\n\nIn LaTeX you use the &.[/quote]\r\n..\r\nuse equnarry, or you use it in texnincenter and upload the pdf file here.[hide][/hide]" } { "Tag": [ "LaTeX", "videos", "algebra", "binomial theorem", "Pascal\\u0027s Triangle" ], "Problem": "Using the Binomial Theorem Part 4\n\nWe use the Binomial Theorem to prove a pattern in Pascal's Triangle", "Solution_1": "For the first pattern ($ 2^n$), I found another explanation. Let's look at the transition between the 3rd and 4th rows:\r\n\r\n[color=orange]1[/color] [color=violet]3[/color] [color=blue]3[/color] [color=brown]1[/color]\r\n([color=orange]1[/color]) ([color=orange]1[/color]+[color=violet]3[/color]) ([color=violet]3[/color]+[color=blue]3[/color]) ([color=blue]3[/color]+[color=brown]1[/color]) ([color=brown]1[/color])\r\n\r\nWhen we add the numbers in the 4th row, we see that it is twice the 3rd row, because each number in the 3rd row is found twice in the 4th row.", "Solution_2": "For the second pattern (the alternating subtraction), we can simplify each line to (x-y)^n. For example. (x-y)^3 is \r\n(3 c 0)x^3 - (3 c 1)(x^2)y + (3 c 2) (x)(y^2) - (3 c 3)y^3\r\n\r\nLet x = y = 1, and so we have (0)^3 = 1-3+3-1 = 0\r\n\r\nThis will work for all rows of the triangle, because (1-1) ^n = 0\r\n\r\nPlease forgive the lack of Latex", "Solution_3": "[quote=\"trivea\"]For the second pattern (the alternating subtraction), we can simplify each line to $ (x - y)^n$. For example. $ (x - y)^3$ is \n$ \\binom{3}{0}x^3 - \\binom{3}{1}x^2y + \\binom{3}{2}xy^2 - \\binom{3}{3}y^3$\n\nLet $ x = y = 1$, and so we have $ (0)^3 = 1 - 3 + 3 - 1 = 0$\n\nThis will work for all rows of the triangle, because $ (1 - 1) ^n = 0$\n\nPlease forgive the lack of Latex[/quote]\r\n$ \\text{\\LaTeX}$'d :)", "Solution_4": "Yes, that was easy. The harder question is: is there a counting problem that proves it? I can't think of any... :maybe: :wink:", "Solution_5": "Mr. Rusczyk, please make more videos!!!!!", "Solution_6": "Another way we could think about the 2^n power pattern would be to use the binomial theorem verbatim, i.e. with the (x+y) instead of the (x-y) format, but substitute y=-1.\n\nP.S. the videos were great, it's a shame Mr. Rusczyk won't be producing any more :(", "Solution_7": "[quote=\"rashi101\"]Yes, that was easy. The harder question is: is there a counting problem that proves it? I can't think of any... :maybe: :wink:[/quote]\n\nThe entries in the nth row of Pascal's triangle are simply nC0, nC1 ... nCn. Therefore the sum of all of them is nC0 + nC1 + ... + nCn, or the numbers of ways to choose a subset from a set of n elements as the sum represents the number of ways to choose a set of 0 elements plus for 1 element, 2 elements and so on up to n. To choose a set there are two options for each element (to include it or not), therefore there are 2^n ways to choose a subset. Thus the sum in a row will simply be 2^n.\n\nThere's the counting argument for it :)" } { "Tag": [], "Problem": "A number $ x$ is twice its reciprocal. What is $ x^6$ ?", "Solution_1": "We know that:\r\n\r\n$ x \\equal{} \\frac {2}{x}$\r\n\r\nMultiply both sides by $ x$.\r\n\r\n$ x^2 \\equal{} 2$\r\n\r\n$ x \\equal{} \\sqrt {2}$\r\n\r\n$ \\sqrt {2}^6 \\equal{} \\boxed{8}$\r\n\r\nEDIT: Oops. As Dr Sonnhard Graubner said, the other solution is $ \\minus{}\\sqrt{2}$, which, when raised to the 6th power, also yields 8.", "Solution_2": "hello isabella, $ x\\equal{}\\minus{}\\sqrt {2}$ is also a solution of your problem,\r\n$ \\minus{}\\sqrt{2}\\equal{}\\frac{2}{\\minus{}\\sqrt{2}}\\equal{}\\frac{2\\sqrt{2}}{\\minus{}2}\\equal{}\\minus{}\\sqrt{2}$ and\r\n$ \\left(\\minus{}\\sqrt{2}\\right)^6\\equal{}8$.\r\nSonnhard.", "Solution_3": "Oh... I see. $x^6$ can be converted to that." } { "Tag": [ "combinatorics unsolved", "combinatorics" ], "Problem": "There are a finite number of red and blue lines in the plane, no two parallel. There is always a third line of the opposite color through the point of intersection of two lines of the same color. Show that all the lines have a common point.", "Solution_1": "http://www.mathlinks.ro/viewtopic.php?t=198938", "Solution_2": "[quote=\"stvs_f\"]There are a finite number of red and blue lines in the plane, no two parallel. There is always a third line of the opposite color through the point of intersection of two lines of the same color. Show that all the lines have a common point.[/quote]\r\nFrom condition ,for each color there exist at least a line that colored in this color . A hint to consider two lines that have different color and angle between them is smallest . All line must be pass through the intersect of two lines ." } { "Tag": [ "logarithms", "limit", "calculus", "derivative", "calculus computations" ], "Problem": "Does there exist a positive real number k such that for all natural n, log(n) is bigger than $ n^k$?", "Solution_1": "no. by L'Hopital's Rule, $ \\lim_{x\\to \\infty} \\frac{\\log x}{x^k} \\equal{} \\lim_{x\\to \\infty} \\frac{1}{kx^k}\\equal{} 0$.", "Solution_2": "Let $ \\sim$ mean comparison:\r\n$ \\log n \\sim n^k$\r\n$ k \\log n \\sim k n^k$\r\n$ \\log n^k \\sim k n^k$\r\n$ \\log x \\sim k x$\r\n\r\nNote that the right side is always increasing at rate $ k$ w.r.t. $ x$ while the left side's derivative decreases to $ 0$, so RHS eventually surpasses LHS." } { "Tag": [ "inequalities" ], "Problem": "hey hey hey ,Who had chosen OMID HAMATI as the moderator of IRAN community ,\r\n\r\n :P :P Just kidding ,Congratulations OMID for being a moderator ,but who on the earth had chosen you ????? :!: :?: \r\n\r\n\r\nANSWER to my question:\r\n\r\nOne of the ADMINS: we chosed him :mad: \r\n\r\nMy answer:\r\nI was just kidding ,but you should have taked votes for chosing the moderator, :D :D \r\n\r\n\r\nPS.in general just a joke", "Solution_1": "Haha, cool, but the admins ALWAYS decide who will become moderator, you see :)\r\n\r\n[They are ALMIGHTY :D]", "Solution_2": "Arne, I was wondering if i can become a moderator. ;)", "Solution_3": "No comments :D", "Solution_4": "the moderators are cool in math but i think if you use the others too,it will be better and more useful,why don't you test it? :D", "Solution_5": "I don't understand what you are saying...", "Solution_6": "lomos_lupin I have choosen Omid Hatami. I hope you have no problem with that. I highly respect his professionalism. \r\n\r\nPS In choosing the moderators there is no democracy on this board. It's pure evil :diablo: :evilgrin: :P", "Solution_7": "Valentin, I'm interrested in becomming a moderator. Do you need one? :?", "Solution_8": "[quote=\"cezar lupu\"]Valentin, I'm interrested in becomming a moderator. Do you need one? :?[/quote]I doubt the Iranians would like you, Cezar :)", "Solution_9": "Why don't they like me?\r\n :?", "Solution_10": "Like you as a moderator :D", "Solution_11": "I was wondering if you need moderators for Romanian Section, or inequalities or algebra?", "Solution_12": "That is not a discussion relevant for this topic.\r\n\r\nAnd since there isn't any thing left to say about this topic \"who had chosen this moderator?\" (the answer is above :D), I declare topic closed ;)" } { "Tag": [ "limit", "trigonometry", "real analysis", "real analysis solved" ], "Problem": "If $a_{n}={ \\prod_{k=1}^{n}} cos(2^{k-1}*x)$, $x{\\neq}k{\\pi}$ then what is the limit of $ \\lim_{n {\\rightarrow} {\\infty}} a_{n} ?\r\n$", "Solution_1": "does anyone have a clue how it can be solved ?", "Solution_2": "Evaluate the finite products using the identity $\\cos x=\\frac12\\cdot\\frac{\\sin 2x}{\\sin x}$.", "Solution_3": "$a_n=\\prod_{k=1}^n \\cos (2^{k-1} x)=\\left( \\frac{1}{2} \\right)^n \\frac{\\sin 2^n x}{\\sin x} $.\r\n$ \\sin x \\ne 0 \\, \\mbox{is}\\,\\mbox{fixed} \\, \\mbox{here}$.\r\n$a_n=\\left( \\frac{1}{sin x} \\right)\\,\\left( \\frac{1}{2} \\right)^n\\,\\sin 2^n x$.\r\n$\\mbox{Since} \\lim_{n \\to \\infty}\\left( \\frac{1}{2} \\right)^n=0$\r\n$\\mbox{and}\\left| \\sin 2^n x \\right| \\le1$.\r\n$\\mbox{then} \\lim_{n \\to \\infty}a_n = \\left( \\frac{1}{sin x} \\right)\\,*\\mbox{0}=0$.", "Solution_4": "And if $x=k\\pi,$ we have\r\n\r\n$\\cos(k\\pi)\\cos(2k\\pi)\\cos(4k\\pi)\\cdots=(-1)^k\\cdot1\\cdot1\\cdots=(-1)^k.$\r\n\r\nIf $x=2^{-j}k\\pi$ for some odd $k$ and some $j>0$, then franck's work is still valid. In that case, the sequence $a_n$ becomes zero at some point and remains zero afterwards." } { "Tag": [], "Problem": "I am contemplating on whether to take the fermat or the cayley.\r\n\r\nFor cayley, I usually finish the whole contest in less than a half an hour, but I might have trouble if there is a problem I can\u2019t solve or if its a tough contest.\r\nFor fermat, I usually get perfect on the practice tests, but it will definitely be harder.\r\n\r\nShould I take the cayley and try to get perfect or take the fermat for a challenge?", "Solution_1": "Fermat. If you come top in your province you get 200 dollars and an invitation to the Waterloo Seminar.", "Solution_2": "Depends on what your goal is. I believe that in previous years, cayley winners have also been invited to Waterloo seminars. Most students in your position would go with Fermat... it also depends on your geographic location. If you live in Saskatchewan or something... go for it. If you live in Toronto, you'll have a tougher time coming home with cash.", "Solution_3": "lol im not in it for the money (though it would be nice). maybe i should take advantage of the fact that all the brilliant mathematicians are now in gr 12. i live in toronto, so a perfect score is probably needed to become the provincial champion. Do you get an invitation only if your the provincial champion or can ppl who get 144 still get it?", "Solution_4": "They invite on cayley too you know. Everyone seems to think its Fermat only. Its pascal they don't invite on at all. Hehehe its your choice. Good luck on whichever one you decide to take!", "Solution_5": "OMG! Sarah got a new avatar! It's so... infectious!\r\n\r\nAnd in other news... apparently she agrees with me that they also invite on Cayley... so write whatever you feel like.", "Solution_6": "[quote=\"Sunny\"]They invite on cayley too you know.[/quote]Apparently not. Last year I was of your opinion so I asked my math teacher to send an email.[quote]Steven\u2019s score of 150 on the Cayley contest was outstanding. However, we invited very few Cayley contestants to the seminar, and those invitations were based on a number of criteria. \n\nOnce we have processed the results of the Fryer/Galois/Hypatia/Euclid contests, we will be issuing some more invitations. It is possible, though not guaranteed, that Steven could be invited based on those results.\n\nMany of the seminar invitees were Fermat contestants. I hope that Steven will continue with his excellent participation in the contests and write the Fermat contest next year.\n\nSincerely,\nJudith Koeller[/quote]", "Solution_7": "I was invited to Seminar twice based on Cayley (and the Grade 10 invitational when they still had it). Its true that they don't invite as many as Fermat, but more people probably take Fermat. You know...just cuz you get a certain score doesn't mean you'll be invited for sure. No one really knows for sure how people are invited. Honestly, every year there seems to be a few people who really shouldn't have been there, and lots of people who should have. Then again, the organizer changed last year, so maybe that's why priority is given to Fermat. Hehehe, though the Seminar wasn't any different, we still had Ian around to bug.\r\n\r\nBut if they are inviting based on the awards they give out, then much more people should have been invited based on Euclid, especially those in grade 12, since the Euclid is the only contest they can write. Ah well. We'll see what happens this year.", "Solution_8": "Good luck on the Fermat tomorrow everybody! :nhl:", "Solution_9": "If theres no chance at getting an invitation for writing the Cayley then i should probably write the fermat. \r\nLast year about how many people were invited to the seminar and how many were in gr 10?\r\n\r\nBTW, Good luck on the contests 2morw!", "Solution_10": "is there a way to get invited to the Waterloo Seminars with 144 in Fermat, ie. Open score, Hypatia score, etc.\r\n\r\nps have they posted solutions to this year's Fermat yet", "Solution_11": "It's best not to discuss the contest on the forum immediately after it's written, for some people may not have taken it yet (for whatever reason).", "Solution_12": "Approximately how many people are invited to the Waterloo Seminar each year? Do they invite from COMC?", "Solution_13": "[quote=\"chilly_snow\"]Approximately how many people are invited to the Waterloo Seminar each year? Do they invite from COMC?[/quote]\r\n\r\n60 ish people. No, they don't invite based on COMC.", "Solution_14": "Hey, could someone message me how they did #25?", "Solution_15": "[quote=\"Sunny\"][quote=\"chilly_snow\"]Approximately how many people are invited to the Waterloo Seminar each year? Do they invite from COMC?[/quote]\n\n60 ish people. No, they don't invite based on COMC.[/quote]\r\noh ya.. what about CMO? i forgot to ask that... :P\r\n(I didn't do so well on the Fermat contest.. made two really STUPID mistakes... :blush: )", "Solution_16": "[quote=\"chilly_snow\"]\noh ya.. what about CMO? i forgot to ask that... :P\n(I didn't do so well on the Fermat contest.. made two really STUPID mistakes... :blush: )[/quote]\r\n\r\nNot really. If you happen to be there, then they'll give you the awards for CMO. They might give invites to 1st, 2nd and 3rd, but chances are they were invited anyways based on other contests.", "Solution_17": "Write Fermat. If you don't win, you still gain experience to allow you to win next year :D", "Solution_18": "i would write cayley.\r\nbut for some ambitious people they might choose fermat...\r\nso it's up to you \r\nand remember if u have the ability to get perfect, the grade difference won't trouble u" } { "Tag": [ "AMC", "AIME", "USAMTS" ], "Problem": "How and when will my math teacher, who runs the AMC/AIME at my school, be notified that I qualified for the AIME through the USAMTS?", "Solution_1": "If you have completed the CEEB field and the teacher field in your Profile on the USAMTS page, they will be contacted by the AMC some time this week (if you have not filled in those fields, please let me know - I have already passed the info to the AMC, and if you did not have these fields completed, you were not designated as an AIME qualifier)." } { "Tag": [ "number theory", "greatest common divisor", "modular arithmetic", "quadratics" ], "Problem": ":lol: Suppose x,y,z are three integers which are in arithmetic progression. If x is of the form 8n + 4 where n is an integer and each of y,z is expressible as a sum of squares of two integers, show that [b]gcd[/b] (x,y,z) cannot be odd.", "Solution_1": "Since x is even according to the given condition,hence it does not have any odd factor hence ....... G.C.D cannot be odd.", "Solution_2": "The fact that x is even doesn't mean that it won't have any odd factors.", "Solution_3": "[hide]Clearly $x$ is even, and if $y$ is even, then $z$ will also have to because the common difference would be even; this would make $\\gcd (x,y,z)$ even.\n\nThe squares mod 8 are 0, 1, and 4, so $y$ and $z$ must each be 0, 1, 2, 4, or 5 mod 8.\n\nSuppose $y$ were odd. Then $y$ would have to be 1 or 5 mod 8. But $y\\equiv 1\\Rightarrow z\\equiv 6$ and $y\\equiv 1\\Rightarrow z\\equiv 6\\pmod{8}$ so $z$ is not a sum of two squares. Contradiction.\n\nTherefore $y$ is even, so so will $z$, making $\\gcd(x,y,z)$ not odd.[/hide]", "Solution_4": "Let's play with mod :\r\n\r\n$x = 8n+4$ (*)\r\n$y = 8n+4 + r = a^2 + b^2$ (**)\r\n$z = 8n+4 + 2r = c^2 + d^2$ (***)\r\n\r\nNow gcd is odd iff $r$ is odd\r\n\r\n$c^2+d^2$ is even so $mod \\ 8$ it must be $0$ or $2$ (looking quadratic residues $mod\\ 8$). Only possibilty with (***) is $r = 1 \\ mod \\ 8$\r\n\r\nNow an odd sum of two square (**) must be $1$ or $5$ mod 8.\r\n\r\n$8n+4 = c^2+d^2 - 2 (a^2+b^2)$ would give $4 = 0\\ mod \\ 8$ contradiction", "Solution_5": "Sorry,I was a bit hasty :wallbash:" } { "Tag": [], "Problem": "Find the smallest value of x satisfying the condition x^3+2x^3=a, where x is an odd integer, and a is the square of an integer.", "Solution_1": "[code]$x^3+2x^3=a$[/code]", "Solution_2": "[hide]$ 3x^{3}=b^{2}\\Rightarrow (x)^{2}(3x)=b^{2}$, so $ 3x$ must be a square, clearly, $ x=3$.[/hide]" } { "Tag": [ "trigonometry", "algebra unsolved", "algebra" ], "Problem": "Find, with proof, all real numbers $x \\in \\lbrack 0, \\frac{\\pi}{2} \\rbrack$, such that $(2 - \\sin 2x)\\sin (x + \\frac{\\pi}{4}) = 1$.", "Solution_1": "If $y=\\sin (x+\\frac{\\pi }{4})$ then $\\sin{2x}=2y^2-1$.\r\nIt give equation $2y^3-3y+1=0$ with solution $y=1,y=\\frac{-1\\pm \\sqrt 3 }{2}.$" } { "Tag": [ "function", "real analysis", "real analysis unsolved" ], "Problem": "Let $f: [a;b] \\rightarrow [a;b]$ to be a continuous function.Prove that f has a fixed point.", "Solution_1": "Study the function $g(x)=f(x)-x$\r\n :cool:", "Solution_2": "To make it complete: $g(a)\\geq 0$ and $g(b)\\leq 0$. Therefore, because g is continuous (as a difference of 2 continuous functions), we can find a point c in [a,b] such that g(c)=0, i.e. f(c)=c." } { "Tag": [ "induction" ], "Problem": "Let $ e_{1},e_{2},...,e_{r}$ be non-negative integers such that $ e_{1}>e_{2}>...........>e_{r}$. Put $ n=\\sum^{r}_{i=1}2^{e_{i}}$.\r\n\r\nShow that $ \\frac{n!}{2^{n-r}}$ is an odd integer.", "Solution_1": "[hide=\"hint!\"]\nenough to prove that:\n\n$ \\left[\\frac{n}{2}\\right]+\\left[\\frac{n}{2^{2}}\\right]+\\cdots+\\left[\\frac{n}{2^{e_{1}}}\\right]=n-r$\n[/hide]", "Solution_2": "Another way\r\n\r\n[hide] Let $ D(n)=\\alpha$ such that $ \\alpha$ is the greatest non-negative integer such that $ 2^{\\alpha}$ divides $ n$. Prove by induction that $ D(n!) = n-r$ where $ r$ is the number of $ 1$'s in the binary representation of $ n$.\n [/hide]" } { "Tag": [ "function" ], "Problem": "Find all functions: $ f: R \\to R$ such that $ f(x \\plus{} y) \\equal{} x^2.f(\\frac {1}{x}) \\plus{} y^2f(\\frac {1}{y})$for all $ x;y \\in R^*$", "Solution_1": "[quote=\"friendlist\"]Find all functions: $ f: R \\to R$ such that $ f(x \\plus{} y) \\equal{} x^2.f(\\frac {1}{x}) \\plus{} y^2f(\\frac {1}{y})$for all $ x;y \\in R^*$[/quote]\r\nPlease lock this topic. This is problem in this issue in Mathematics and Young Magazine.", "Solution_2": "My solution is very long, if I have time i will post my solution. \r\n\r\nMy result: $ f(x) \\equal{} {\\rm{ax}}$ with $ a \\in R,x \\in R*$" } { "Tag": [ "logarithms", "function", "induction", "calculus", "derivative", "calculus computations" ], "Problem": "Study the convergence of $(x_{n})_{n\\geq0}$ defined by: $x_{0}=\\frac{1}{2}$ and $5^{x_{n}}=2x_{n-1}+3$ for $n\\in [1,\\infty)$.", "Solution_1": "We have that $x_n = \\frac{\\ln(2x_{n-1} + 3)}{\\ln 5}$. Or we could say $x_n = f(x_{n-1})$ where $f(x) = \\frac{\\ln(2x + 3)}{\\ln 5}$. f is obviously a continous strictly increasing function for all $x > -\\frac{3}{2}$. A obvious proof by induction shows for example that $x_n > 0$, so we have no problems with coming into undefined numbers.\r\n\r\n$(x_n)$ is a increasing sequence:\r\n$n=1$: $x_0 = \\frac12$, $x_1 = \\frac{\\ln 4}{\\ln 5} > \\frac12 = x_0$.\r\nAssume that $x_{p+1} > x_{p}$ for some $p$. Then $x_{p+2} = f(x_{p+1}) > f(x_p) = x_{p+1}$ since $f$ is strictly increasing. The principle of induction gives us our conclusion.\r\n\r\nAll possible values that $x_n$ might converge to are roots of the equation $A = f(A), A > \\frac12$. Set $g(A) = A - f(A)$, so that $g'(A) = {\\frac{2\\,\\ln \\left( 5 \\right) A+3\\,\\ln \\left( 5 \\right) -2}{ \\left( 2\\,A+3 \\right) \\ln \\left( 5 \\right) }}$ Knowing that $\\ln5 > \\frac12$ we see that $g'(A) > 0$ for $A > \\frac12$. Therefore, by the intermediate value theorem there is precisely one root to the equation $A = f(A), A > \\frac{1}{2}$. By a lucky guess we see that this root is $A = 1$.\r\n\r\nWe now show that that $x_n < 1$ for all $n$:\r\n$n=1$: $x_0 = \\frac12 < 1$.\r\nAssume that $x_p < 1$ for some $p$. Then $x_{p+1} = f(x_p) < f(1) = 1$ since f is a strictly increasing function.\r\n\r\nSince $(x_n)$ is also increasing the axiom of upper bound (might be called something else in english) shows that $(x_n)$ is convergent, and one of the middle paragraphs shows that is convergent with the value 1.", "Solution_2": "Nice. I don't undersand some parts because there are some things that i haven't leaned yet. Is there a solutino that uses only basic theory?... :?", "Solution_3": "Hmm. You probably have. It's just that I express myself too short sometimes. Here is the idea:\r\n\r\n1) We note that the function by which the sequence is defined is strictly increasing.\r\n\r\n2) Since $x_1 > x_0$ we can make a very simple induction proof that that $x_n$ is an increasing series using\r\n 1). If we had found that $x_1 < x_0$ we could have used the exact same proof to show the opposite. In such sequences where the function $f$ is strictly increasing, the first two values always determine whether the sequence is increasing or decreasing.\r\n\r\n3) For any given recursive sequence of the form $x_{n+1} = f(x_n)$ (where $f$ is continous. disregard if you don't know what this is) we have that if the sequence has a limit A, then it must be the case that $A = f(A)$. This is due to the fact that $x_{n+1} \\to A$ and $f(x_n) \\to A$ (since f is continous). Such a point where $x = f(x)$ is called a fixed point.\r\n\r\n4) We use 3) to determine any such possible limits of the sequence. However, since the equation cannot actually be solved because of the nature of the function $f$, I instead create the difference of $A$ and $f(A)$ and show that the derivative of this difference is always positive. Thus, the difference function is strictly increasing. Therefore there can be at most one solution to the equation, and we can \"guess\" this as $A = 1$. \r\n\r\n5) Now, whenever we have such a fixed point it is easy to show that that the sequence is bounded upwards by it. I think this induction proof is easy to understand.\r\n\r\n6) There is a theorem (actually an axiom), that you hopefully know, that says that if a sequence is increasing (decreasing) it has a limit if and only if it is bounded upwards (downwards). So due to this, we have shown that the sequence converges. Since we have also found that there is only possible limit, i.e. 1, it must converge to 1.\r\n\r\nFeel free to ask about anything. Have you not learned these methods?" } { "Tag": [ "abstract algebra", "superior algebra", "superior algebra unsolved" ], "Problem": "Sorry for the many question. But I finally decided to return to all the things I did not understand previously from about 500 pages commutative algebra ;)\r\n\r\n[b]Firstly:[/b]\r\n[quote=\"Some Theorem\"] Let $ A$ be a Noetherian ring, $ M \\neq 0$ a finite $ A$-module. Then there exists a chain $ 0 \\equal{} M_0 \\subset M_1 \\subset ... \\subset M$ of submodules of $ M$ such that for each $ i$ we have $ M_i/M_{i \\minus{} 1} \\cong A/P_i$ with $ P_i \\in Spec(A)$.\nProof: Choose any $ P_1 \\in Ass(M)$, then there exists a submodule $ M_1$ of $ M$ with $ M_1 \\cong A/P_1$. If $ M_1 \\neq M$ choose $ P_2 \\in Ass(M/M_1)$, then there exists $ M_2 \\subset M$ such that $ M_2/M_1 \\cong A/P_2$. Continuing this way and using the a.c.c. we arrive at $ M_n \\equal{} M$.\n[/quote]\n-I don't see why we need $ A$ noetherian. $ M$ a finite $ A$-module and $ A$ being noetherian implies that $ M$ is noetherian. But why don't we just start the theorem with $ M$ noetherian, and $ A$ any ring or where else do we use that $ A$ is noetherian?\n-If we take another $ P_1' \\neq P_1 \\in Ass(M)$ will we find a chain of the same length?\n\n\n\n[b]Secondly:[/b]\n[quote=\"Some corollary\"]\nIf $ P \\subset Q$ are prime ideals of $ A$, then $ (A_Q)_{P A_Q} \\equal{} A_P$.\n[/quote]\r\n-I don't understand the transitive localization rules very well (also seen in my other post). Does someone want to elaborate?", "Solution_1": "1. existence of associated primes? I don't remember exactly, but I think A noetherian is important.\r\n2. use the universal property.", "Solution_2": "1) Yes indeed. The noetherian property is important:\r\n\r\nLet $ \\mathcal{F} \\equal{} \\{ ann(x) \\ | \\ x \\in M \\}$, then every maximal element in $ \\mathcal{F}$ is prime. Let $ ann(x)$ be a maximal element. Let $ ab \\in ann(x)$, $ b \\notin ann(x)$, then by maximality $ ann(bx) \\equal{} ann(x)$, so $ ax \\equal{} 0$, and $ a \\in ann(x)$.\r\n\r\n2) I'll review that part :)" } { "Tag": [ "geometry", "perimeter", "cyclic quadrilateral", "number theory unsolved", "number theory" ], "Problem": "1.5\r\n\r\nCyclic quadrilateral $ABCD$ has integer side lengths. It is known that $AD =2005$, $\\angle ABC=\\angle ADC=90^\\circ$, and $\\max\\{AB, BC, CD\\}<2005$.\r\n\r\nDetermine that minimum and maximum possible values for the perimeter of $ABCD$.", "Solution_1": "Very Nice problem.. my solution use Ptolomeu\u00b4s Theorem, divisibility and Pitagoras Theorem :)" } { "Tag": [], "Problem": "Question 1\r\n3x+6+2x=4x-3\r\n\r\nI got x= -9\r\n\r\nQuestion 2\r\n-11a-(5-6a)=-(6-3a)+1\r\n\r\nI got a= 0\r\n\r\nQuestion 3\r\n-[p-(4p+2)]=3+(4p+7)\r\n\r\nI got p=1\r\n\r\nQuestion 4\r\nM+M= -9\r\n~ ~\r\n6 4\r\n\r\nThats M over 6 + M over 4\r\n\r\nI got M=-4\r\n\r\nQuestion 5\r\nP-2+p=1\r\n~ ~ ~\r\n3 4 2\r\n\r\nI got P=1\r\n\r\n.06x=.14(x+500)=130\r\n\r\nI got x=300\r\n\r\n.35(140) +.15W=.05(w+1100)\r\n\r\nI didnt find an awnser to this one\r\n\r\n\r\n2[3-4(5-R)]=2(3r-11)\r\nI didnt find an awnser to this one\r\n\r\n2(2y-5)-3(4-y)=7y-22\r\nI didnt find an awnser to this one\r\n\r\n6(3-4x+10=-15x+3(2-3x)\r\nI didnt find an awnser to this one", "Solution_1": "[i][size=134]New here? WELCOME TO AOPS!!![/size][/i]\r\n3x+6+2x=4x-3\r\n\r\nI got x= -9\r\n\r\n[b]right.[/b]\r\n\r\nQuestion 2\r\n-11a-(5-6a)=-(6-3a)+1\r\n\r\nI got a= 0\r\n\r\n[b]right.[/b]\r\n\r\nQuestion 3\r\n-[p-(4p+2)]=3+(4p+7)\r\n\r\nI got p=1\r\n\r\n[b]I got a different answer, check your work again... if you are really sure... then I would check mine.\n\nHere's what I got[/b][hide]-p=8\np=-8[/hide]\n\nQuestion 4\nM+M= -9\n~ ~\n6 4\n\nThats M over 6 + M over 4\n\nI got M=-4\n\n[b]Check your work.\nMy answer[/b]\n[hide]M=21.6[/hide]\n\nQuestion 5\nP-2+p=1\n~ ~ ~\n3 4 2\n\nI got P=1\n\np-2 all over 3 right? if so then...\n\n[b]check your work.\nMy answer[/b]\n[hide]7p=14\np=2[/hide]\n\n.06x=.14(x+500)=130\n\nI got x=300\n\n[b]are you sure there's two equal sign here?[/b]\n\n.35(140) +.15W=.05(w+1100)\n\nI didnt find an awnser to this one\n\n[b]where did you got stuck?\n\nanswer:[/b]\n[hide]0.1w=6\nw=6/0.1=60[/hide]\n\n\n2[3-4(5-R)]=2(3r-11)\nI didnt find an awnser to this one\n\n[b]um... show me where you got stuck[/b]\n\notherwise\n[hide]2r=-12\nr=-6[/hide]\r\n\r\n2(2y-5)-3(4-y)=7y-22\r\nI didnt find an awnser to this one\r\n\r\n[b]Is there an answer AT ALL? or no solution?[/b]\r\n\r\n6(3-4x+10=-15x+3(2-3x)\r\nI didnt find an awnser to this one\r\n[b]I think you missed one paranthesis[/b]\r\n\r\nWARNING: You have to really know how to get to the correct answer. In other words, I don't guarantee you that mine are correct :D . But do check over the ones I marked.\r\n\r\n\r\n[b]For future posts, try to include what you have done for the problem so far. [/b]\r\nsorry, I don't have time to type out full solutions. :(\r\n\r\nIf you have any more questions and are in short of time, AIM me.\r\nmy screenname is probsolving.", "Solution_2": "6(3-4x+10)=-15x+3(2-3x) woops\r\n\r\nYea I had so much trouble with this one, I got to the point after I distributed in the () and added like terms.\r\n\r\n2[3-4(5-R)]=2(3r-11)\r\n\r\n2[3-20-4R=\r\n\r\n6-20+4r=2(3r-11)\r\n\r\nthere is were i got stuck\r\n\r\n :?: 2(2y-5)-3(4-y)=7y-22 :?: \r\n\r\n\r\nAfter working on the others for so long, My brain stopped.\r\n\r\nThis was a pop quiz that i took and didnt count \"thank god\" But My teacher sugested that I go online and get some help. Fridays are test days for all my classes. Math was last period, so I hope you understand. \r\n\r\n\r\nThank you for your help BTW :)", "Solution_3": "6(3-4x+10)=-15x+3(2-3x) woops\r\n18 - 24x + 60 = -15x + 6 - 9x and move the x to one side, the others to the other\r\n-24x + 15x + 9x = 6 - 60 - 18\r\n0y = -72 \r\n\r\nSO\r\n\r\n[b]NO SOLUTIONS[/b]\r\n\r\n\r\nYea I had so much trouble with this one, I got to the point after I distributed in the () and added like terms.\r\n\r\n2[3-4(5-R)]=2(3r-11)\r\n\r\n2[3-20-4R= [b]Remeber to multiply 2 by 20 and by 4r! (DISTRIBUTION)[/b]\r\n6-20+4r=2(3r-11) \r\n\r\n[b]it is actually 6 - 40 + 8r = 6r - 22\nwhich is equal 6 + 22 - 40 = -2r\nso -12 = -2r\nso r = 6[/b]\r\n \r\nWoops, I made a mistake on that earlier :blush: .\r\n\r\nthere is were i got stuck\r\n\r\n :?: 2(2y-5)-3(4-y)=7y-22 :?: \r\nusing distribution property:\r\n4y - 10 - 12 + 3y = 7y - 22\r\n4y +3y - 7 y = numbers...\r\n0 = numbers...\r\n\r\nso [b]NO SOLUTION[/b]\r\n\r\n\r\nAfter working on the others for so long, My brain stopped.\r\n\r\nThis was a pop quiz that i took and didnt count \"thank god\" But My teacher sugested that I go online and get some help. Fridays are test days for all my classes. Math was last period, so I hope you understand. \r\n\r\nRemember... some problems in algebra have no solution. The thing is... you must REALIZE when there is no solution by going through all the normal steps and remember to CHECK YOUR WORK. Math is all about accurary. So make sure ALL of your steps are accurate in order to arrive at a correct response. \r\n\r\n\r\nThank you for your help BTW :)\r\n\r\nNo problem! Happy to help! :)" } { "Tag": [ "LaTeX" ], "Problem": "it says \"the output PDF file is not finished\" and won't let me see it in Adobe Acrobat.\r\nWhat does this mean, and what can I do?", "Solution_1": "[quote=\"hmoonster\"]it says \"the output PDF file is not finished\" and won't let me see it in Adobe Acrobat.\nWhat does this mean, and what can I do?[/quote]\r\n\r\nOK, when and where does it gives you that error?\r\n\r\nIf I assume that: You are taking about a PDF generated with $\\LaTeX$, using TeXNicCenter, and you are trying to open it with the output option of the TeXNic, well, that has happened to me a few times too, and I don't know exactly why that happens. I just either open the file directly with Acrobat Reader (usually I don't have problems with that).\r\n\r\nIt also can be that you tried to compile a file whose PDF output was open, or that you try to open a pdf file that hasn't finished the compilation yet.\r\n\r\nIf you use linux, I strongly recomend to compile for the regular DVI file, and after that use the dvipdf command. Usually the output is a lot smaller, and there is no lose of quality of the digital display." } { "Tag": [ "geometry", "incenter", "circumcircle", "geometry unsolved" ], "Problem": "Dear Mathlinkers, \r\nlet ABCD a square, M a point on segment CD, I, X, Y the incenters wrt MAB, MAD, MBC, \r\nPx the parallel to AD passing through X, Py the parallel to AD passing through Y, Pm the parallel to AD passing through M. \r\nProve that the isogonal lines of Px, Py, Pm wrt triangle MXY are concurrent at I. \r\nSincerely \r\nJean-Louis", "Solution_1": "Let $ M_1, M_2$ be arbitrary points following on the square side $ DC$ in this order, let $ AM_1$ cut $ BM_2$ at $ M_3,$ and let $ M$ be incenter of $ \\triangle M_1M_2M_3.$ Let $ (X), (Y), (I)$ be incircles of $ \\triangle M_1AD, \\triangle M_2BC, \\triangle M_3AB.$ The common external tangent of $ (X), (Y)$ other than $ DC$ cuts $ AM_3, BM_3$ at $ E, F.$ The square $ ABCD$ has an incircle $ (J)$, i.e., the common external tangents $ \\overrightarrow{AB}, \\overrightarrow{BC}, \\overrightarrow{CD}, \\overrightarrow{DA}$ of the points $ A, B$ and the circles $ (X), (Y)$ touch a single circle $ (J).$ Then their other common external tangents $ \\overrightarrow{BA}, \\overrightarrow{AE}, \\overrightarrow{EF}, \\overrightarrow{FB}$ also touch a single circle, the incircle $ (I)$ of the $ \\triangle ABM_3.$ But then $ X, I, Y, M$ are intersections of the external angle bisectors of the general quadrilateral $ M_1EFM_2,$ which means that $ XIYM$ is cyclic with a circumcircle $ (O).$ By the continuity principle, this holds even in the limiting case, when the points $ M_1, M_2$ coincide and the $ \\triangle M_1M_2M_3,$ together with its incenter $ M,$ degenerate into a single point $ M \\in DC.$ See [url]http://www.mathlinks.ro/viewtopic.php?t=247411[/url] for the general case.\r\n\r\nAssume $ M$ is a point on $ DC.$ Let $ (I_m)$ be excircle of the $ \\triangle ABM$ against $ M$ and let $ K, L$ be feet of the perpendiculars from $ I$ to $ MX, MY,$ so that $ KL$ is Simson line of $ \\triangle MXY$ with the pole $ I \\in (O).$ The quadrilaterals $ IKML \\sim I_mAIB$ are centrally similar, having parallel sides and corresponding diagonals $ IM, I_mI,$ therefore their other corresponding diagonals $ KL \\parallel AB$ are also parallel and $ KL \\perp AD.$ But this means that $ I$ is the isogonal conjugate WRT the $ \\triangle MXY$ of the infinite point of any line parallel to $ AD.$ See [url]http://www.mathlinks.ro/viewtopic.php?t=129699[/url], click on \"Show details\" and scroll down to the 3rd paragraph \"Let D be pole of the Simson line $ d \\perp k$...", "Solution_2": "Dear Mathlinkers,\nfor a proof see: \nhttp://perso.orange.fr/jl.ayme vol.5, le r\u00e9sultat de Larrosa Canestro, p.37\nSincerely\nJean-Louis" } { "Tag": [ "symmetry", "geometry", "parallelogram", "angle bisector", "geometry unsolved" ], "Problem": "(F.Nilov, A.Zaslavsky) Let $ CC_0$ be a median of triangle $ ABC$; the perpendicular bisectors to $ AC$ and $ BC$ intersect $ CC_0$ in points $ A'$, $ B'$; $ C_1$ is the meet of lines $ AA'$ and $ BB'$. Prove that $ \\angle C_1CA \\equal{} \\angle C_0CB$.", "Solution_1": "see http://www.mathlinks.ro/Forum/viewtopic.php?t=208506\r\nin my solution ,I obtain \u2220PAD=\u2220QAF \u2220PAF=\u2220QAD (different letters)\r\nthus our problem is solved", "Solution_2": "It\u2019s not a big deal to show that C1 lies on (ABO), \u00d0AC1B=2\u00d0A, then C1C is its angle bisector: the triangles A\u2019CA and B\u2019BC are isosceles, then \u00d0AC1B=\u00d0A+\u00d0A\u2019AC+\u00d0B\u2019BC=2\u00d0A. Further, draw perpendiculars from A and B on CC0, call M and N their feet on it (obviously AM=BN, C0 being the middle of AB) and the perpendiculars from C on AA\u2019 and BB\u2019, call P and Q their feet onto these lines respectively. By symmetry, the triangles ACM and ACP are congruent, therefore AM=CP, similarly BN=CQ ( Here, the sign \u00d0 means \"angle\").\r\n\r\nBest regards,\r\nsunken rock", "Solution_3": "In may solution i made a parallelogram ACBC2 (C2 is a point) and i have a point S so y^k+k/(y^y)$\r\n\r\nIf $k=0$ then $y=x$.\r\n\r\nIf $k\\geq 1$ then $y 5), P(X < 7), and P(X2 \u2212 12X + 35 > 0).\r\n\r\n* With this question, I'm not sure how to approach it to answer the first part. Should C = f(x)/x? That seems not very useful. I'm just starting probability and this notation is kind of confusing to me. \r\n\r\n2) Assume that a new light bulb will burn out after t hours, where t is chosen\r\nfrom [0,infinity) with an exponential density\r\nf(t) = \u0015xe^\u2212(x\u0015t) .\r\n\r\n(a) Assume that X= 0.01, and find the probability that the bulb will not\r\nburn out before T hours. This probability is often called the reliability\r\nof the bulb.\r\n(b) For what T is the reliability of the bulb = 1/2?\r\n\r\n* With this question, I'm uncertain of what f(t) is actually measuring and how probability plays a part here.\r\n\r\n3) Take a stick of unit length and break it into three pieces, choosing the break\r\npoints at random. (The break points are assumed to be chosen simultaneously.)\r\nWhat is the probability that the three pieces can be used to form a\r\ntriangle? Hint: The sum of the lengths of any two pieces must exceed the\r\nlength of the third, so each piece must have length < 1/2.\r\n\r\n* With this question, I'm not sure how to set this up. I believe I should use a cube of [0,1]^3 and perhaps use the idea that A+B+C = 1 and A, B & C are all less than 1/2. But I don't really fully understand the concept. Or perhaps I just graph those two equations and get the area that overlaps? \r\n\r\nThanks!", "Solution_1": "1) $ f(x) \\equal{}Cx$ on [2,10]\r\n\r\n$ 1\\equal{}\\int_2^{10} f(x)dx \\equal{} 48C \\to C\\equal{}\\frac{1}{48}$\r\n\r\n$ P([a,b]) \\equal{} \\int_a^b f(x) dx \\equal{} \\frac{b^2\\minus{}a^2}{96}$\r\n\r\n$ P(X>5) \\equal{} P([5,10]) \\equal{} \\frac{75}{96}, P(X<7) \\equal{} P([2,7]) \\equal{} \\frac{45}{96}$\r\n\r\n$ X^2\\minus{}12X\\plus{}35 \\equal{} (X\\minus{}5)(X\\minus{}7) > 0 \\to X \\in [2,5) \\cup (7,10]$\r\n\r\n$ P([2,5) \\cup (7,10]) \\equal{} P(X<5)\\plus{}P(X>7) \\equal{}$\r\n$ (1\\minus{} P(X>5))\\plus{} (1\\minus{}P(X<7)) \\equal{} \\frac{72}{96}$\r\n\r\n2) i cant read some of the characters, sorry\r\n\r\n3) think of the line [0,1] with two break points $ x,y \\in (0,1)$\r\n\r\nWLOG 0x works, so assume $ x<.5$\r\n\r\nthen we have a triangle only when $ y\\in (.5,x\\plus{}.5)$\r\n\r\nso for fixed x, $ P( y good) \\equal{} \\frac{x}{1\\minus{}x} \\equal{} \\frac{1}{1\\minus{}x} \\minus{}1$\r\n\r\nso$ P( all good) \\equal{} \\int_0^\\frac{1}{2} \\frac{1}{1\\minus{}x}\\minus{}1 dx \\equal{} \\ln 2 \\minus{}\\frac{1}{2}$" } { "Tag": [ "LaTeX", "algebra unsolved", "algebra" ], "Problem": "let the group A={1,2,3,.........,n} such that n is odd.\r\n\r\nX1,X2,X3,X4,...........,Xn are different elements two by two from A.\r\n\r\nprove that it is k elements from A such that Xk-k is even.\r\n\r\nremark: Xk isn't X x k.", "Solution_1": "Please learn to use $ \\text{\\LaTeX}$ , greatestmaths :mad:\r\n\r\nLet the group $ \\text{A}=\\{1,2,3,..,n\\}$ such that n is odd.\r\n\r\n$ X_{1},X_{2},X_{3},X_{4},..,X_{n}$ are different elements two by two from $ \\text{A}$.\r\n\r\nprove that it is $ k$ elements from $ \\text{A}$. such that $ X_{k}-k$ is even.\r\n\r\n\r\n\r\n[code]Let the group $\\text {A}=\\{1,2,3,..,n\\}$ such that n is odd.\n\n$X_1,X_2,X_3,X_4,..,X_n$ are different elements two by two from $\\text{A}$.\n\nprove that it is $k$ elements from $\\text{A}$. such that $X_k-k$ is even.[/code]", "Solution_2": "Do you mean exist $ k$ ,greatestmaths ? . If i am not wrong , this problem very easy. Suppose not exist $ k$ so that $ S_{k}\\minus{}k\\vdots 2$ then $ \\sum_{k \\equal{} 1}^{n}(S_{k}\\minus{}k)\\equiv n(\\text{mod 2})\\Leftrightarrow n\\equiv 0(\\text{mod 2})$contracdiction because $ n$ is an odd number :)", "Solution_3": "[code]Let the group $\\text {A}=\\{1,2,3,..,n\\}$ such that n is odd. \n\n$X_1,X_2,X_3,X_4,..,X_n$ are different elements two by two from $\\text{A}$. \n\nprove that it is $k$ elements from $\\text{A}$. such that $X_k-k$ is even. \n\n \n\n \n[/code]", "Solution_4": "[quote=\"greatestmaths\"][code]Let the group $\\text {A}=\\{1,2,3,..,n\\}$ such that n is odd. \n\n$X_1,X_2,X_3,X_4,..,X_n$ are different elements two by two from $\\text{A}$. \n\nprove that it is $k$ elements from $\\text{A}$. such that $X_k-k$ is even. \n \n[/code][/quote]\r\n\r\nWhat do you want to say , greatestmaths :(", "Solution_5": "Let the group $ \\text{A}\\equal{}\\{1,2,3,..,n\\}$ such that n is odd. \r\n\r\n$ X_{1},X_{2},X_{3},X_{4},..,X_{n}$ are different elements two by two from $ \\text{A}$. \r\n\r\nprove that it is $ k$ elements from $ \\text{A}$. such that $ X_{k}\\minus{}k$ is even." } { "Tag": [], "Problem": "[img]http://i227.photobucket.com/albums/dd269/joedowdle/prob2b-1.jpg[/img]", "Solution_1": "$ \\binom{2n}{n\\plus{}1} \\plus{} \\binom{2n}{n} \\equal{} \\dfrac{1}{2} \\binom{2n \\plus{} 2}{n\\plus{}1}$\r\n\r\n$ \\dfrac{(2n)!}{(n\\plus{}1)!\\times(n\\minus{}1)!} \\plus{} \\dfrac{(2n)!}{n!\\times n!} \\equal{} \r\n\\dfrac{(2n \\plus{} 2)!}{2(n\\plus{}1)!\\times(n\\plus{}1)!}$\r\n\r\n$ \\dfrac{n! \\times n! \\times (2n)! \\plus{} (n\\plus{}1)! \\times (n\\minus{}1)! \\times (2n)!}{n! \\times n! \\times (n\\plus{}1)! \\times (n\\minus{}1)!} \\equal{} \r\n\\dfrac{(2n \\plus{} 2)!}{2(n\\plus{}1)!\\times(n\\plus{}1)!}$\r\n\r\n$ \\dfrac{ (2n)! \\times (n! \\times n! \\plus{} (n\\plus{}1)! \\times (n\\minus{}1)!)}{n! \\times n! \\times (n\\plus{}1)! \\times (n\\minus{}1)!} \\equal{} \r\n\\dfrac{(2n \\plus{} 2)!}{2(n\\plus{}1)!\\times(n\\plus{}1)!}$\r\n\r\n$ \\dfrac{ (2n)! \\times ((n\\minus{}1)!)^2 \\times (2n^2 \\plus{} n)}{n! \\times n!\\times (n\\minus{}1)!} \\equal{} \r\n\\dfrac{(2n \\plus{} 2)!}{2(n\\plus{}1)!}$\r\n\r\n$ \\dfrac{ (2n)! \\times ((n\\minus{}1)!)^2 \\times (2n^2 \\plus{} n)}{(n!)^2} \\equal{} \r\n\\dfrac{(2n \\plus{} 2)!}{2(n^2 \\plus{} n)}$\r\n\r\n$ \\dfrac{ (2n)! \\times (2n^2 \\plus{} n)}{n^2} \\equal{} \r\n\\dfrac{(2n \\plus{} 2)!}{2(n^2 \\plus{} n)}$\r\n\r\n$ \\dfrac{(2n^2 \\plus{} n)}{n^2} \\equal{} \r\n\\dfrac{(2n \\plus{} 1)(2n \\plus{} 2)}{2(n^2 \\plus{} n)}$\r\n\r\n$ 2(n^2 \\plus{} n)(2n^2 \\plus{} n) \\equal{} n^2(2n \\plus{} 1)(2n \\plus{} 2)$\r\n\r\n$ 2n^2(n \\plus{} 1)(2n \\plus{} 1) \\equal{} 2n^2(n \\plus{} 1)(2n \\plus{} 1)$\r\n\r\ntada! I don't know if that's exactly what you wanted, but it works!", "Solution_2": "Well that's not what I was looking for exactly. The part of the question before it asks me to prove it by substitution, which is what you did. Or at least I assumed that's what was meant by \"prove by substitution.\" Although I am glad that we got the same answer. At least I did one thing right.\r\n\r\nAnyway, I think by combinatorial proof is asking me to prove it by using C(n,k) type identities and properties.", "Solution_3": "Two applications of Pascal's Identity can do it for you:\r\n\r\n[b]Pascal's Identity: [/b] $ \\binom{n}{k} = \\binom{n - 1}{k} + \\binom{n - 1}{k - 1}$\r\n\r\nApplying this to the RHS,\r\n\\begin{eqnarray*} \\frac {1}{2}\\binom{2n + 2}{n + 1} & = & \\frac {1}{2}\\left[\\binom{2n + 1}{n + 1} + \\binom{2n + 1}{n}\\right] \\\\\r\n& = & \\frac {1}{2}\\left[\\binom{2n}{n + 1} + \\binom{2n}{n} + \\binom{2n}{n} + \\binom{2n}{n - 1}\\right] \\\\\r\n& = & \\binom{2n}{n + 1} + \\binom{2n}{n} \\\\\r\n\\end{eqnarray*}\r\nwhere the last step follows from the fact that $ \\binom{2n}{n - 1} = \\binom{2n}{n + 1}$.", "Solution_4": "Wow. That was extremely simple. I'm almost mad at myself for not seeing that on my own.\r\n\r\nThank you very much.", "Solution_5": "[hide=\"Another way\"]Consider a set of $ 2n\\plus{}2$ elements, $ 2$ of which are \"special\". (Like the hint says! :P ) Call the rest of the elements \"regular\".\nWe will now consider the amount of ways to choose $ n\\plus{}1$ elements of this set.\n\nMethod 1: Simply choose $ n\\plus{}1$ of the the $ 2n\\plus{}2$ elements. There are $ \\binom{2n\\plus{}2}{n\\plus{}1}$ ways to do this.\nMethod 2: Choose from the special elements, then choose from the regular elements. If we choose $ 0$ special elements, then there are $ \\binom{2n}{n\\plus{}1}$ ways to choose from the regular elements. If we choose $ 1$ special element in one of $ 2$ ways, then there are $ \\binom{2n}{n}$ ways to choose from the regular elements, giving $ 2\\cdot\\binom{2n}{n}$ ways total. If we choose $ 2$ special elements, there are $ \\binom{2n}{n\\minus{}1}$ ways to choose from the regular elements. In total, there are $ \\binom{2n}{n\\plus{}1}\\plus{}2\\cdot\\binom{2n}{n}\\plus{}\\binom{2n}{n\\minus{}1}$ ways to do this.\n\nThese two methods yield the same amount of combinations, so we may equate them.\nWe have \\[ \\binom{2n\\plus{}2}{n\\plus{}1}\\equal{}\\binom{2n}{n\\plus{}1}\\plus{}2\\cdot\\binom{2n}{n}\\plus{}\\binom{2n}{n\\minus{}1}\\]\nSince $ \\binom{2n}{n\\minus{}1}\\equal{}\\binom{2n}{n\\plus{}1}$, this is equivalent to \\[ \\binom{2n\\plus{}2}{n\\plus{}1}\\equal{}2\\cdot\\binom{2n}{n\\plus{}1}\\plus{}2\\cdot\\binom{2n}{n}\\]\nDividing by $ 2$ gives \\[ \\frac{1}{2}\\binom{2n\\plus{}2}{n\\plus{}1}\\equal{}\\binom{2n}{n\\plus{}1}\\plus{}\\binom{2n}{n}\\]\nwhich was what we wanted. :D[/hide]\r\nNote: I believe that the way I proved it (counting the same thing in two different ways) is the kind of proof that the problem asked for. Sorry I wrote it poorly. :P", "Solution_6": "[quote=\"Brut3Forc3\"]\nNote: I believe that the way I proved it (counting the same thing in two different ways) is the kind of proof that the problem asked for. [/quote]\r\n\r\nThat's essentially the same idea behind Pascal's Identity.\r\n\r\nAlso, I think that a simple block walking argument can be used to prove the identity." } { "Tag": [ "geometry", "rectangle", "calculus", "integration", "trigonometry", "calculus computations" ], "Problem": "Hi everyone,\r\n\r\nI've been having trouble with this problem for a few days. I managed to get an expression using a bit of geometry but it doesn't work perfectly in all situations.\r\n\r\nGiven a disk centered on $ (x_{c},y_{c})$ with radius $ r$ and a rectangle with $ x_{r} \\in [a,b]$ and $ y_{r} \\in [c,d]$, how would you express the area of the intersection of the circle with the rectangle ?\r\nSimplification: $ x_{c} \\in [a,b] \\vee y_{c} \\in [c,d]$\r\n\r\nI suppose a double integral would do the trick but I'm not sure how to formulate the problem :(\r\n\r\n\r\nS.", "Solution_1": "It's going to split into an annoyingly large number of cases.\r\n\r\nI would look for a procedure, rather than trying for any sort of piecewise formula.\r\n\r\n1. Find the intersections between the circle and the right side $ x\\equal{}b$ of the rectangle. If there are no intersections, take the point $ (x_c\\plus{}r,y_c)$ (which is inside the rectangle) twice instead. If a corner $ (b,c)$ or $ (b,d)$ is inside the circle, take that corner instead of the lower/upper intersection.\r\n2. Repeat this process for the other three sides in counterclockwise order.\r\n3. The eight points $ (x_i,y_i)$ generate eight wedges (some likely degenerate). The four coming from pairs of points on the same side are triangles with area $ \\frac12(x_iy_{i\\plus{}1}\\minus{}x_{i_1}y_i)$*, and the four coming from pairs on different sides are circular wedges with area $ r^2\\cos^{\\minus{}1}\\left(\\frac{x_ix_{i\\plus{}1}\\plus{}y_iy_{i\\plus{}1}}{r^2}\\right)$*. Add them up.\r\n\r\n* Area formulas stated as if $ x_c\\equal{}y_c\\equal{}0$. Replace $ x_j$ with $ x_j\\minus{}x_c$ and $ y_j$ with $ y_j\\minus{}y_c$ for the generic version. Also, indices are interpreted mod 8, so that $ i\\plus{}1$ is the next point even if it means going back to the first one.", "Solution_2": "Thanks for the answer.\r\n\r\nMy goal is to be able to express the distribution of the area for a fixed $ r$ and a center $ (x,y)$ wandering about a surface where the rectangle lies, so a formula would be more useful if I don't want to spend more time waiting for my computer to compute that numerically.\r\n\r\nHow large is \"annoyingly large\" ? :D \r\n\r\n\r\nS." } { "Tag": [ "function", "calculus", "calculus computations" ], "Problem": "Let $ f: \\mathbb R\\to\\mathbb R$ be continuous such that $ f(x)\\ne0,\\,\\forall\\,x\\in\\mathbb R$ and $ f(0)\\equal{}\\sqrt2.$ Show that $ f$ is positive.", "Solution_1": "Look at the Bolzano-Cauchy intermediate value theorem", "Solution_2": "How'd you do it? Because I don't know how to apply such theorem.", "Solution_3": "Roughly, in order to take a negative value $ f$ must cross (hence intersect) the $ x$-axis, and we are given that this does not occur. Specifically, if $ f(0) \\equal{} \\sqrt{2}$ and $ f(a) < 0$ for some $ a$ then on $ (0, a)$ we know that $ f$ takes every value between $ \\sqrt{2}$ and $ 0$ (this is the content of the Intermediate Value Theorem); in particular, $ \\exists c \\in (0, a) : f(c) \\equal{} 0$, contradiction." } { "Tag": [ "inequalities", "function", "IMO" ], "Problem": "The original problem was :\r\n\r\nIf a>1, find a bounded sequence of real numbers x_i with :\r\n|x_i-x-j|*|i-j|^a>=1 for all distinct i,\r\nj.\r\n\r\nMy question is : what if a=1 ?\r\n\r\nI thought that such a sequence was always unbounded, but the author of the problem told that one could construct such a bounded sequence (and that candidats succeded in doing it during the 1991 olympiads !).\r\nCould someone explain me how to construct it ?", "Solution_1": "Also, contrast problem A2 from ISL 2002: [url]http://www.kalva.demon.co.uk/short/soln/sh02a2.html[/url].\r\n\r\nSketch of a proof for this problem:\r\n\r\nI think something that would work would be to construct it by thinking of it as placing $a_1, a_2, \\dots$ on the real line one by one in sequential order in a manner as below: (the integer n in the diagram represents the relative position of $a_n$)\r\n\r\n1\r\n1 2\r\n1 3 2 4\r\n1 5 3 6 2 7 4 8\r\n... and so on (distances not to scale). When $a_n$ is inserted, say it is directly between $a_i$ and $a_j$, it suffices to make sure the condition holds for the pairs $i, n$ and $j, n$, for all others will be greater than pairs checked before.\r\n\r\nAlso, one should be able to get an upper bound on the the size interval needed to contain the first $2^t$ terms for any $t$: one can then start placing the integers in that interval one by one so as to always assure there is enough space for the integers following.", "Solution_2": "There is a similar problem in ASU 1978.\r\n\r\nhttp://olympiads.win.tue.nl/imo/soviet/RusMath.html\r\n(problem 257)\r\n\r\nProve that there exists such an infinite sequence $\\{x_i\\}$, that for all $m$ and all $k$ ($m\\neq k$) holds the inequality $|x_m-x_k|>\\frac{1}{|m-k|}$.", "Solution_3": "Next question... what if 0 n^{1-a}\r\n\r\nSo the sequence canot be bounded" } { "Tag": [ "trigonometry" ], "Problem": "By a given point $ P$ inside angle $ BAC$ construct the straight line wich cuts $ AB$ and $ AC$ at $ M$ and $ N$ respectively, and wich minimize the sum $ \\frac{1}{MP}\\plus{}\\frac{1}{NP}$\r\n\r\n\r\n :)", "Solution_1": "There is no local minimum; do you mean maximum?", "Solution_2": "Is it asking for an answer, or is it a construction problem? And unless I'm interpreting it incorrectly, there isn't a minimum. You could keep moving the line away from P. There is a maximum when you draw the line through P to make isosceles $ \\triangle MNA$.", "Solution_3": "Perhaps ElChapin intended for P to be outside triangle AMN.", "Solution_4": "I'm sorry it was supposed to say maximize. \r\n\r\n :)", "Solution_5": "In which case you'd just make isosceles $ \\triangle MNA$ where $ P$ is on $ \\overline{MN}$.", "Solution_6": "[quote=\"ZzZzZzZzZzZz\"]In which case you'd just make isosceles $ \\triangle MNA$ where $ P$ is on $ \\overline{MN}$.[/quote]\r\n\r\nNo, that is clearly incorrect. What makes you think that the sum of the reciprocals of the distances will be maximized for the isosceles case? The correct answer is that the sum of reciprocal distances is maximized when the line through P is drawn such that PA is the altitude of triangle MNA.", "Solution_7": "Let $ \\angle BAP\\equal{}y$ and $ \\angle CAP\\equal{}z$. Then $ \\frac{1}{MP}\\plus{}\\frac{1}{NP}\\equal{}AP\\cdot (\\cot y\\plus{}\\cot z)\\cdot \\sin \\angle MPA$. The max occurs when $ \\angle MPA\\equal{}90^\\circ$. We construct $ M$ and $ N$ by drawing the line perpendicular to $ AP$ through $ P$." } { "Tag": [], "Problem": "When a certain number is increased by 4, the result is equal to that number divided by 7 What is the number?\r\n\r\nI think that it's 14/3. I'm not sure if it should be -14/3.\r\n\r\nMy calcs may not make sense. I don't know what I did.\r\n\r\nx + 4 = y = y/7\r\n\r\n(2/3) + 4 = y\r\n\r\n14/3 + 4 = 4 2/3 + 4 = 8 2/3\r\n\r\n(26/3) * (1/7)", "Solution_1": "You only need one variable for this problem.\r\n\r\nLet the number be $ x$.\r\n\r\n$ x\\plus{}4\\equal{}\\frac{x}{7}$.\r\n\r\n$ 7x\\plus{}28\\equal{}x$\r\n\r\n$ 28\\equal{}\\minus{}6x$\r\n\r\n$ \\frac{\\minus{}14}{3}\\equal{}x$", "Solution_2": "[quote=\"cf249\"]You only need one variable for this problem.\n\nLet the number be $ x$.\n\n$ x \\plus{} 4 \\equal{} \\frac {x}{7}$.\n\n$ 7x \\plus{} 28 \\equal{} x$\n\n$ 28 \\equal{} \\minus{} 6x$\n\n$ \\frac { \\minus{} 14}{3} \\equal{} x$[/quote]\r\n\r\nThank you.", "Solution_3": "Oh and the for the first post, y/7 does not equal y.", "Solution_4": "quoting: \"oh and for the first one y does not equal y/7.\" \r\ncouldn't y = y/7 if y was zero? or is that undefined?", "Solution_5": "$ \\frac{y}{7}\\equal{}y$\r\n$ y\\equal{}7y$\r\nOnly works when $ y\\equal{}0$", "Solution_6": "But the point was that y/7 does not ALWAYS equal y. That would be like making a formula that works only for one number, or only one set of points.", "Solution_7": "so how can it SOMETIMES equal y/7? though i think i get what ur saying.", "Solution_8": "Point is that it doesn't always work.\r\n\r\n$ X\\equal{}2x$ when $ x\\equal{}0$. IT DOES NOT HOLD WHEN $ X\\equal{}anything else$" } { "Tag": [ "logarithms", "calculus", "calculus computations" ], "Problem": "Hello everyone... I'm really stuck on some problems testing the convergence of these sequences:\r\n\r\n1) (n>=2)\u2211 [1 / root(n^(2)-1)]\r\n\r\n2) (n>=1)\u2211 [((-1)^(n)*ln(n)) / (1+ln(n)]\r\n\r\n3) (n>=1)\u2211 [(cos(n(pi)/2) / (n^2)]\r\n\r\nHelp would really be appreciated!", "Solution_1": "Think about size, and \"de-clutter.\"\r\n\r\nFor large $n,$, how big is $\\frac1{\\sqrt{n^2-1}}?$ It looks a lot like $\\frac1n,$ doesn't it?\r\n\r\nFor large $n,$ how big is $\\frac{\\ln n}{1+\\ln n}?$ A lot like $\\frac{\\ln n}{\\ln n},$ and that doesn't go to zero, does it?\r\n\r\nIn the third case, you have a fancy variant on a $\\pm$ sign, multiplied by $\\frac1{n^2}.$ What do you know about $\\sum\\frac1{n^2}?$", "Solution_2": "For the second one, you said that ln(n)/ln(n) approaches 0, but does it not approach 1 for all values of n? Also, since it is an alternating series, would it converge to a specific value?", "Solution_3": "Read that sentence again: \"and that [b]doesn't[/b] go to 0...\"\r\n\r\nSince the values alternate between about 1 and about -1, the partial sums jump around; in this case, the sequence of partial sums should have two limit points.", "Solution_4": "Oops, my mistake... well wouldn't it be divergent if it went to two different limit points?", "Solution_5": "Yes, it would be divergent. jmerry provided more information than you really need - the two subsequential limts is a nice touch, but it's more than you were originally asked.\r\n\r\nTheorem: if a series converges, then its terms go to zero.\r\n\r\nContrapositive: if the terms of a series do not go to zero, then the series diverges." } { "Tag": [ "CEMC" ], "Problem": "Hi everyone,\r\nAs the Canadian representative at AoPS, I feel like it is partially my duty to keep this board alive, even though I think it's doing pretty well on its own. So, I'll try to post some more from now on.\r\n\r\nAt AoPS, one thing we'd like to have is more exposure in Canada. I've written to the CEMC and the CMS, so I believe they're aware of us, but I'm wondering if there's anyone else I should be writing to. If you have any ideas, I would love to hear them.", "Solution_1": ":huh: There is a Canadian admin?!?! I didn't know that.", "Solution_2": "[quote=\"lightrhee\"]:huh: There is a Canadian admin?!?! I didn't know that.[/quote]\r\n\r\nIf I'm not mistaken, he was at the National Camp I went to. At least in my year, Naoki? Hehehe, good times those were.", "Solution_3": "[quote=\"Sunny\"][quote=\"lightrhee\"]:huh: There is a Canadian admin?!?! I didn't know that.[/quote]\n\nIf I'm not mistaken, he was at the National Camp I went to. At least in my year, Naoki? Hehehe, good times those were.[/quote]Not only that he is Canadian, but he was your deputy at some point, and won a couple of silvers for you guys ;)", "Solution_4": "[quote=\"lightrhee\"]:huh: There is a Canadian admin?!?! I didn't know that.[/quote]\nYeah, born and raised in Toronto, very proud.\n\n[quote=\"Sunny\"]If I'm not mistaken, he was at the National Camp I went to. At least in my year, Naoki? Hehehe, good times those were.[/quote]\nI remember you Sarah, good to see you on the board, all the way from beautiful downtown Okotoks. That camp was a lot of fun, I kinda wish I could do more of those. Felicitations on your recent engagement to Singular/Alex, I'm sure you'll make a happy couple.\n\n[quote=\"Valentin Vornicu\"]Not only that he is Canadian, but he was your deputy at some point, and won a couple of silvers for you guys ;)[/quote]\r\nWell, only one silver actually, but who's counting? :P I prefer to think I've helped other students win medals, but who knows for sure.", "Solution_5": "hear that sparky? Naoki Sato says we make a good couple! :lol: :lol:", "Solution_6": "[quote=\"Singular\"]hear that sparky? Naoki Sato says we make a good couple! :lol: :lol:[/quote]\r\n\r\n...oh boy....", "Solution_7": "[quote]hear that sparky? Naoki Sato says we make a good couple![/quote]\n\nCongratulation Alex... you seem to have found a soulmate there..........\nAnd also, Sarah? why is he calling you sparky??\n\n\n[quote]I've written to the CEMC and the CMS, so I believe they're aware of us[/quote]\r\n\r\nI'm very sure they're aware of us, considering the last seminar.. :lol: :blush:", "Solution_8": "[quote=\"jpark\"][quote]hear that sparky? Naoki Sato says we make a good couple![/quote]\n\nCongratulation Alex... you seem to have found a soulmate there..........\nAnd also, Sarah? why is he calling you sparky??[/quote]\n\nApparently because I'm bright and slightly evil.\n\n[quote][quote]I've written to the CEMC and the CMS, so I believe they're aware of us[/quote]\n\nI'm very sure they're aware of us, considering the last seminar.. :lol: :blush:[/quote]\r\n\r\nlol...wow that was really scary....they knew everything....", "Solution_9": "[quote][quote]I've written to the CEMC and the CMS, so I believe they're aware of us[/quote]\n\nI'm very sure they're aware of us, considering the last seminar.. :lol: :blush:[/quote]\r\n\r\nWhy, what happened?", "Solution_10": "... no comment on that please.. :blush: :blush:", "Solution_11": "LMFAO!!! Yea seminar was great. Good times all around. This whole forum is getting quite funny.", "Solution_12": "[quote=\"Elyot\"]LMFAO!!! Yea seminar was great. Good times all around. This whole forum is getting quite funny.[/quote]\r\n\r\nI can't believe they found the forum t.t" } { "Tag": [ "geometry", "geometric transformation", "reflection", "geometry proposed" ], "Problem": "Given a triangle $ ABC.$ Let $ A_{1},B_{1},C_{1}$ be the feet of the altitudes of triangle $ ABC$ from the vertices $ A,B,C,$ respectively. Denote by $ Y \\equal{} CA\\cap C_{1}A_{1}, Z \\equal{} AB\\cap A_{1}B_{1}.$ Point $ P$ lies in the plane of triangle $ ABC$. Let $ P_{1},P_{2},P_{3}$ be the reflections of $ P$ in sidelines $ BC,CA,AB,$ respectively. Let $ l_{1},l_{2}$ be lines through $ Y,Z,$ respectively, parallel to $ P_{3}P_{1},P_{1}P_{2}.$ Prove that if $ AP_{1},BP_{2},$ and $ CP_{3}$ are concurrent, then so are $ l_{1},l_{2},$ and $ AP_{1}.$", "Solution_1": "[quote]Let $ P_{1},P_{2},P_{3}$ be the reflections of $ P$ in sidelines $ BC,CA,AB,$ respectively [/quote]\n[quote]$ AP_{1},BP_{2},$ and $ CP_{3}$ are concurrent $\\Longleftrightarrow$ $ \\ell_{1},\\ell_{2},$ and $ AP_{1}$ are concurrent[/quote]\n[hide=\"Hint\"]Locus of P is the Darboux cubic of ABC.[/hide]", "Solution_2": "Very nice problem and ... very hard! \r\n\r\nA synthetic proof for the Sondat theorem is here:\r\n[url]http://pagesperso-orange.fr/jl.ayme/Docs/Le%20theoreme%20de%20Sondat.pdf[/url]\r\n\r\nThe triangles $ ABC$ and $ A_{1}B_{1}C_{1}$ are orthologic and the points $ O$ and $ H$ are the orthologic centers of these \r\ntriangles. The lines $ AA_{1},BB_{1},CC_{1}$ are concurrent at the point $ H$. The Sondat theorem implies that $ OH\\bot YZ$. (1)\r\n\r\nLet $ P^{*}$ be the isogonal conjugate of the point $ P$ wrt $ ABC$, $ \\left\\{Y'\\right\\}\\equal{}P_{1}P_{3}\\cap CA$, $ \\left\\{Z'\\right\\}\\equal{}P_{1}P_{2}\\cap AB$.\r\nThe triangles $ ABC$ and $ P_{1}P_{2}P_{3}$ are orthologic and the points $ P$ and $ P^{*}$ are the orthologic centers \r\nof these triangles. The lines $ AP_{1},BP_{2},CP_{3}$ are concurrent at a point $ Q$. \r\nThe Sondat theorem implies that the points $ P,P^{*},Q$ are collinear and $ PP^{*}\\bot Y'Z'$. (2)\r\n\r\n\r\nNow, $ P_{1},P_{2},P_{3}$ are the reflection points of $ P$ in $ BC,CA,AB$, respectively and the lines $ AP_{1},BP_{2},CP_{3}$ \r\nare concurrent. From here [url]http://pagesperso-orange.fr/bernard.gibert/Exemples/k001.html[/url]\r\nwe have that the point $ P$ lies on the Neuberg cubic, so $ PP^{*}\\parallel{}OH$. (3) \r\nI don't have a synthetic solution to prove that $ PP^{*}\\parallel{}OH$ or $ PQ\\parallel{}OH$. :wink: \r\n\r\n(1),(2),(3) $ \\Rightarrow YZ\\parallel{}Y'Z'$.\r\n\r\n$ \\left\\{M\\right\\}\\equal{}P_{1}P_{3}\\cap YZ$, $ \\left\\{N\\right\\}\\equal{}P_{1}P_{2}\\cap YZ$.\r\n$ MN\\parallel{}Y'Z'\\Rightarrow \\frac{P_{1}M}{P_{1}Y'}\\equal{}\\frac{P_{1}N}{P_{1}Z'}$ (4)\r\nLet $ V'$ be the intersection point of $ AP_{1}$ with the parallel through $ M$ to $ AC$, \r\nand $ V\"$ be the intersection point of $ AP_{1}$ with the parallel through $ N$ to $ AB$.\r\n$ MV'\\parallel{}Y'A\\Rightarrow \\frac{P_{1}M}{P_{1}Y'}\\equal{}\\frac{P_{1}V'}{P_{1}A}$ (5) and $ NV\"\\parallel{}Z'A\\Rightarrow \\frac{P_{1}N}{P_{1}Z'}\\equal{}\\frac{P_{1}V\"}{P_{1}A}$ (6)\r\n(4),(5),(6) $ \\Rightarrow\\frac{P_{1}V'}{P_{1}A}\\equal{}\\frac{P_{1}V\"}{P_{1}A}\\Rightarrow V'\\equal{}V\"\\equal{}V$.\r\n\r\n$ \\left\\{L\\right\\}\\equal{}AP_{1}\\cap YZ$\r\n$ YA\\parallel{}MV\\Rightarrow \\frac{LY}{LM}\\equal{}\\frac{LA}{LV}$ (7) and $ ZA\\parallel{}NV\\Rightarrow \\frac{LZ}{LN}\\equal{}\\frac{LA}{LV}$ (8)\r\n(7),(8) $ \\Rightarrow\\frac{LY}{LM}\\equal{}\\frac{LZ}{LN}$ (9)\r\n\r\n$ \\left\\{W'\\right\\}\\equal{}l_{1}\\cap AP_{1}$ and $ \\left\\{W\"\\right\\}\\equal{}l_{2}\\cap AP_{1}$.\r\n$ YW'\\parallel{}MP_{1}\\Rightarrow\\frac{LY}{LM}\\equal{}\\frac{LW'}{LP_{1}}$ (10) and $ ZW\"\\parallel{}NP_{1}\\Rightarrow\\frac{LZ}{LN}\\equal{}\\frac{LW\"}{LP_{1}}$ (11)\r\n(9),(10),(11) $ \\Rightarrow\\frac{LW'}{LP_{1}}\\equal{}\\frac{LW\"}{LP_{1}}\\Rightarrow W'\\equal{}W\"\\equal{}W$.\r\n\r\nSo, the lines $ l_{1},l_{2},AP_{1}$ are concurrent at $ W$.\r\n\r\nBest regards, Petrisor Neagoe :)", "Solution_3": "Dear luis,\n\nThe points $ P_{1},P_{2},P_{3}$ are the REFLECTION of $ P$ in sidelines $ BC,CA,AB$.\nThe triangle $ P_{1}P_{2}P_{3}$ IS NOT THE PEDAL TRIANGLE of the point $ P$.\n\nBest regards" } { "Tag": [ "vector", "trigonometry", "calculus", "derivative", "calculus computations" ], "Problem": "The position vector of a point on a circular helix is given by\r\n\r\n$ \\mathbf{r}\\equal{} a[(\\cos{t})\\mathbf{i}\\plus{}(\\sin{t})\\mathbf{j}\\plus{}(t\\cot{\\alpha})\\mathbf{k}]$\r\n\r\nFind the unit tangent vector to the helix at any point.\r\nShow that the curvature at any point is\r\n\r\n$ \\kappa \\equal{}\\frac{\\sin^{2}{\\alpha}}{a}$\r\n\r\nFind the unit normal, and show that it always passes through the $ z$-axis.\r\n\r\n(I got stuck at finding the unit normal. You may think I am a bit silly but shouldn't there be a whole plane of normals at any one point?)", "Solution_1": "Back from a long vacation, let me see if I still remember any math... :lol: \r\n\r\nThe unit tangent vector is given by:\r\n\r\n$ T(t)\\equal{}|r'(t)|\\equal{}\\frac{ (\\minus{}a\\sin t, a\\cos t, a\\cot\\alpha)}{a\\csc\\alpha}$\r\n\r\n\r\nThe curvature is given by:\r\n\r\n$ \\kappa(t) \\equal{}\\frac{ |T'(t)| }{|r'(t)|}$\r\n\r\nNow $ T'(t)\\equal{}\\frac{ (\\minus{}a\\cos t,\\minus{}a\\sin t, 0) }{a\\csc\\alpha}$, so we get:\r\n\r\n$ \\kappa(t)\\equal{}\\frac{\\frac{a}{a\\csc\\alpha}}{a\\csc\\alpha}\\equal{}\\frac{\\sin^{2}\\alpha}{a}$\r\n\r\n\r\nThe unit normal vector is given by:\r\n\r\n$ N(t)\\equal{}\\frac{T'(t)}{|T'(t)|}\\equal{}\\frac{\\sin^{2}\\alpha}{a}\\cdot (\\minus{}a\\cos t,\\minus{}a\\sin t, 0)$.\r\n\r\nSince the $ z$-component is always zero, it must always pass through the $ z$-axis.", "Solution_2": "[quote=\"sludgethrower\"](I got stuck at finding the unit normal. You may think I am a bit silly but shouldn't there be a whole plane of normals at any one point?)[/quote]\r\n\r\nYes, there is. However, we often (as edoo did) take the (normalized) derivative of the unit tangent and refer to it as [i]the[/i] unit normal." } { "Tag": [ "number theory unsolved", "number theory" ], "Problem": "Prove that there are infinitely many n such that a^n+1 is divisible by n (with a is an integer, a>1)\r\nIt seem old but i don't know the solution.", "Solution_1": "note that $ a \\plus{} 1 > 2$ which means that this number has a prime divisor,let's call it $ q$;\r\nthen obviously $ q^b |a^{q^b} \\plus{} 1$ where $ b$ is a poistive integer.so there exists infinitely many such natural numbers.", "Solution_2": "shoki: if $ a \\equal{} 3$, then $ q \\equal{} 2$, but $ 2^2\\not | 3^{2^2} \\plus{} 1$", "Solution_3": "sorry u are right i didn't consider the case $ a\\equal{}2^k\\minus{}1$.anyway ,in this case just put $ n\\equal{}2$ then we must have an odd prime dividing it let it be $ q$ then we have $ 2q^b | (2^k\\minus{}1)^{2q^b}\\plus{}1$" } { "Tag": [ "linear algebra", "matrix" ], "Problem": "Prove that addition for matrices is associative. \r\n\r\nIs this too easy?", "Solution_1": "If it's too easy, also prove that the addiction of $n$ matrices, rather than just three, is associative. :D\r\n\r\nIf you think of addition as a binary operation, there will be many ways to associate the $n$ terms with parentheses. But then it's way past getting started level.", "Solution_2": "[quote=\"AntonioMainenti\"]If it's too easy, also prove that the addiction of $n$ matrices, rather than just three, is associative. :D\n\n[/quote]\r\n\r\nThat's what I want, and I'm not addicted to matrices :rotfl:", "Solution_3": "Just look at how the addition of matrices is defined.\r\n\r\nTo add matrices, add the \"items\" in the corresponding places. So adding matrices is, in fact, the same as adding real numbers, and since addition of real numbers is associative, the addition of matrices must be associative as well. \r\n\r\nI know it's not very rigorous, I could perfectly rewrite it in a more mathematical way, but of course we're in \"Getting Started\" here so I don't think that is a good idea.", "Solution_4": "That's how i proved (well, a little more detailed but this is the internet), I just wanted to see if there was a more formal way.", "Solution_5": "No, I don't think a more formal way exists. After all we're proving this straight from the definition. I don't think another \"simple\" proof can be found, but I don't think we should bother since the proof is obvious and easy enough :)", "Solution_6": "For some reason my textbook didn't contain answers to these proofs, and there were like 10 of these similar ones. Of course, if we could prove 1+1=2, that would be fun :P" } { "Tag": [ "LaTeX" ], "Problem": "Find the quotient and the remainder when $ 3x^{4} \\minus{} x^{3} \\minus{} 3x^{2} \\minus{} 14x \\minus{} 8$ is divided by $ x^{2} \\plus{} x \\plus{} 2$.\r\n\r\n[I have quotient $ \\equal{} 3x^{2} \\minus{} 4x \\minus{} 5$ and remainder $ \\equal{} \\minus{} x \\plus{} 2$.]", "Solution_1": "$ 3x^4 \\minus{} x^3 \\minus{} 3x^2 \\minus{} 14x \\minus{} 8 \\\\ \\\\ \\\\\r\n\\equal{} 3x^4 \\plus{} 3x^3 \\plus{} 6x^2 \\minus{} 4x^3 \\minus{} 9x^2 \\minus{} 14x \\minus{} 8 \\\\ \\\\\r\n\\equal{} 3x^4 \\plus{} 3x^3 \\plus{} 6x^2 \\minus{} 4x^3 \\minus{} 4x^2 \\minus{} 8x \\minus{} 5x^2 \\minus{} 6x \\minus{} 8 \\\\ \\\\ \r\n\\equal{} 3x^4 \\plus{} 3x^3 \\plus{} 6x^2 \\minus{} 4x^3 \\minus{} 4x^2 \\minus{} 8x \\minus{} 5x^2 \\minus{} 5x \\minus{} 10 \\minus{} x \\plus{} 2 \\\\ \\\\\r\n\\equal{} 3x^2(x^2 \\plus{} x \\plus{} 2) \\minus{} 4x(x^2 \\plus{} x \\plus{} 2) \\minus{} 5(x^2 \\plus{} x \\plus{} 2) \\minus{} x \\plus{} 2 \\\\ \\\\\r\n\\equal{} (x^2 \\plus{} x \\plus{} 2) (3x^2 \\minus{} 4x \\minus{} 5) \\minus{} x \\plus{} 2$\r\n\r\n\r\n[b]Quotient :[/b] $ 3x^2 \\minus{} 4x \\minus{} 5$\r\n\r\n\r\n[b]Remainder :[/b] $ \\minus{} x \\plus{} 2$.", "Solution_2": "Thanks. You're right: I left out a $ 3$ and have edited it in.", "Solution_3": "Couldn't you just do long division? Although that would be very hard to $ %Error. \"LATEX\" is a bad command.\n$", "Solution_4": "Maybe synthetic division is better?", "Solution_5": "Synthetic division works only for linear divisors, so it's only a practical method when dealing with divisors that can be factored into linear factors. Unfortunately, $ x^2 \\plus{} x \\plus{} 2$ has imaginary roots, which are very hard to work with." } { "Tag": [ "Stanford", "college", "geometry" ], "Problem": "I got into Brown this year and was waitlisted at Stanford. Brown has binding decisions so I am deciding whether to accept at Brown or decline and wait for Stanford waitlist decisions.\r\n\r\nI hope someone can compare both colleges in terms of:\r\n\r\n1) Math and Comp. Sci.\r\n2) Campus life\r\n3) Undergrad Research Opportunities", "Solution_1": "I know little about Brown, but I can give you a quick overview of Stanford for your three points of interest.\r\n\r\n1) The math program is strong -- there is a good honors freshman sequence and plenty of good professors around. I don't know that it especially stands out among the top math programs, but it's definitely up there. As for computer science, it would be tough to find a department better than Stanford's. From my experience the courses have been excellent and very thorough. Many of the professors are famous for their research and teach really awesome classes in their fields. A lot of thought is put into the introductory computer science courses as well. You can expect a lot from a Stanford CS graduate.\r\n\r\n2) The main benefit of campus life at Stanford relative to other academically strong schools is how laid back people are. Of course people take classes seriously, but it's California and you can't help but feel a little less stressful. Of course having never attended another university I can't really compare -- this is just my impression. I really like the environment here overall, though. There are plenty of events to attend and awesome people to meet. I can give you more details if you have specific questions; campus life as a whole is a broad topic.\r\n\r\n3) This is probably one of Stanford's strongest points as a research institution. It is not difficult to find research opportunities as an undergraduate. Although I have not done any research yet, several of my friends are doing research over the summer with professors that are at the forefront of their fields. So not only will you be able to do real research, you'll get to work with both amazing peers and mentors who really know what they're doing. I'm guessing you'd be able to find similar research opportunities at other universities, but I think at Stanford research is much more readily available and advertised to undergraduates.", "Solution_2": "From what I've heard about Brown, its main strength as far as math is in applied math and its pure math department is lacking. I'm afraid I can't give you specifics, though.", "Solution_3": "[quote]Brown has binding decisions[/quote] \r\n\r\nWhat do you mean?", "Solution_4": "What I hear from my grad-school buddies is that Brown is very strong in the areas in which it has professors, but is quite small (so those areas are not all-encompassing). I don't think this should matter too much to an undergrad, though.", "Solution_5": "You should also probably look at the wait list statistics, and see how many people usually get off of Stanford's waitlist.", "Solution_6": "If the original poster had a regular round admission from Brown, he should accept that offer of admission by May 1st by submitting a deposit, and if he is interested in Stanford still he should indicate a willingness to be on the waiting list. Those two actions are consistent. If he had an early decision offer of admission to Brown, he should have withdrawn his application to Stanford long ago. \r\n\r\nIt is routine for applicants who are put on the waiting lists of highly desired colleges to first accept (by May 1st) an offer of admission from some other college, possibly including paying a deposit. If offered admission from the waiting list, you can accept the offer of admission from your preferred college, but may forfeit your deposit--a small price to pay if you really like the waiting list college a lot better. This advice is general, and this situation happens hundreds of times a year.", "Solution_7": "Not on topic but, thanks tokenadult, I was wondering how that works. :)", "Solution_8": "[quote=\"tokenadult\"]If the original poster had a regular round admission from Brown, he should accept that offer of admission by May 1st by submitting a deposit, and if he is interested in Stanford still he should indicate a willingness to be on the waiting list. Those two actions are consistent. If he had an early decision offer of admission to Brown, he should have withdrawn his application to Stanford long ago. \n\nIt is routine for applicants who are put on the waiting lists of highly desired colleges to first accept (by May 1st) an offer of admission from some other college, possibly including paying a deposit. If offered admission from the waiting list, you can accept the offer of admission from your preferred college, but may forfeit your deposit--a small price to pay if you really like the waiting list college a lot better. This advice is general, and this situation happens hundreds of times a year.[/quote]\r\n\r\nYes, I do have a regular round of admission from Brown.\r\nMy problem is that Brown does not ask for any deposit, but once you accept admission to Brown, you are committed to attend Brown and you must withdraw all other pending applications. (That is what I meant by Binding decisions).\r\n\r\nSo either I accept to Brown and forgo Stanford or I decline Brown's offer for admission and wait for Stanford to accept students from the waiting list.\r\n\r\n\r\nAnyway, thanks for you help and advice!" } { "Tag": [], "Problem": "I think a lot of MC problems have shortcuts, and it would be nice to know them. Anyone have any to share? I'll start, I guess. \r\n\r\nYou know those grid problems where you have to count how many ways there are of going from one corner to the other going only in two directions? Well, the old-fashioned way (at least for me) is to count the number of ways to get to each intersection and keep adding. It's easier, though, to say that there are m+n choose n ways for a m by n grid (Can you see why?). \r\n\r\nAny other ideas? It's true that MC can be as shallow as guess and check, but not always...", "Solution_1": "Useful - \r\n3 :^3: +4 :^3: +5 :^3: =6 :^3: \r\n20 :^2: +21 :^2: =29 :^2: \r\nn :^2: +(n :^2: -1)/2=(n :^2: +1)/2\r\n\r\n\r\n\r\n\r\nhttp://www.runescape.com", "Solution_2": "Well, your question is way too broad to answer here, but if you have some specific questions, feel free to ask. Here's my Sprint Round strategy from last year:\r\n\r\n1. If a problem looks like it'll take a while or be brute force, skip it and come back to it if you have time.\r\n2. Try to have gotten to the end of the test by 25:00, even if you skipped a problem or two on the way.\r\n3. Around 35:00, go back and read the last sentence of every problem. Most stupid mistakes (wrong answer forms, incorrect units, ...) can be caught this way.\r\n4. 39:00 - guess.\r\n\r\nProbably a little late, but oh well. Also, MC is not usually as shallow as guess and check - there's almost always a faster way.\r\n\r\n-Adam Hesterberg", "Solution_3": "The question is like a bug is traveling from one corner of a m*n grid to the opposite corner how many way can he do it. \r\nThe answer is (m+n)Cn. Reason, no matter how you move, you need to move up m times and move right n times. Which we can transfer into a a sequence problem. Imagine a conquence consist m Us and n Rs. What we wanted to find out corresponds to total number of way of arranging it, or (m+n)Cn" } { "Tag": [ "inequalities", "geometry", "perimeter", "search", "trigonometry", "geometry proposed" ], "Problem": "Given a triangle $ ABC$.Let $ AM,BN,CP$ be the medians of the triangle $ ABC$. Suppose that$ AM,BN,CP$ intersect the circumircle of the triangle$ ABC$ at $ D,E,F$ respectively .Prove that: $ DE\\plus{}EF\\plus{}FD \\ge AB\\plus{}BC\\plus{}CA$ :)", "Solution_1": "[quote=\"quykhtn-qa1\"]Given a triangle $ ABC$.Let $ AM,BN,CP$ be the medians of the triangle $ ABC$. Suppose that$ AM,BN,CP$ intersect the circumircle of the triangle$ ABC$ at $ D,E,F$ respectively .Prove that: $ DE \\plus{} EF \\plus{} FD \\ge AB \\plus{} BC \\plus{} CA$ :)[/quote]\r\nNO ONE INTERESTED IN THIS INEQUALITY??? :maybe:", "Solution_2": "More than perimeter they are area and inradii see [url=http://www.mathlinks.ro/viewtopic.php?search_id=482334692&t=85211]here[/url], using lemma in the last post $ (A',B',C') > > (A,B,C)$ we can see $ \\sum\\sin A'\\ge\\sum\\sin A$ this mean $ p(A'B'C')\\ge p(ABC)$ or in your post $ p(DEF)\\ge p(ABC)$." } { "Tag": [ "inequalities" ], "Problem": "Let $ a, b, c > 0$. Prove that $ 3x\\plus{}2y\\plus{}4z \\ge \\sqrt{xy}\\plus{}3\\sqrt{yz}\\plus{}5\\sqrt{zx}$.", "Solution_1": "[hide=\"Solution\"] $ \\frac{x \\plus{} y}{2} \\plus{} \\frac{3y \\plus{} 3z}{2} \\plus{} \\frac{5x \\plus{} 5z}{2} \\ge \\sqrt{xy} \\plus{} 3 \\sqrt{yz} \\plus{} 5 \\sqrt{zx}$ by AM-GM. [/hide]", "Solution_2": "[quote=\"t0rajir0u\"][hide=\"Solution\"] $ \\frac {x \\plus{} y}{2} \\plus{} \\frac {3y \\plus{} 3z}{2} \\plus{} \\frac {5x \\plus{} 5z}{2} \\ge \\sqrt {xy} \\plus{} 3 \\sqrt {yz} \\plus{} 5 \\sqrt {zx}$ by AM-GM. [/hide][/quote]\r\n\r\nYes. It's true. Thank you.", "Solution_3": "What's AM-GM?", "Solution_4": "[b]The Arithmetic Mean-Geometric Mean Inequality:[/b] For positive reals $ r_1, r_2, ... r_n$,\r\n\r\n$ \\frac{r_1 \\plus{} r_2 \\plus{} ... \\plus{} r_n}{n} \\ge \\sqrt[n]{r_1 r_2 ... r_n}$.\r\n\r\n[b]For two variables:[/b] $ \\frac{x \\plus{} y}{2} \\ge \\sqrt{xy}$.", "Solution_5": "thanks a lot!", "Solution_6": "[quote=\"t0rajir0u\"][b]The Arithmetic Mean-Geometric Mean Inequality:[/b] For positive reals $ r_1, r_2, ... r_n$,\n\n$ \\frac {r_1 \\plus{} r_2 \\plus{} ... \\plus{} r_n}{n} \\ge \\sqrt [n]{r_1 r_2 ... r_n}$.\n\n[b]For two variables:[/b] $ \\frac {x \\plus{} y}{2} \\ge \\sqrt {xy}$.[/quote]\r\n\r\nSom one say it is [color=darkblue][b]Cauchy innequaliti.[/b][/color]", "Solution_7": "[b]Cauchy[/b], short for [b]Cauchy-Schwartz Inequality[/b],\r\n\r\nstates that\r\n\r\n$ a_1^2\\plus{}a_2^2\\plus{}a_3^2\\plus{}\\hdots\\plus{}a_n^2)(b_1^2\\plus{}b_2^2\\plus{}\\hdots\\plus{}b_n^2) \\geq (a_1b_1\\plus{}a_2b_2\\plus{}\\hdots\\plus{}a_nb_n)^2$.\r\n\r\nEquality occors iff either all $ a_i$ are $ 0$ or there is\r\n\r\na constant $ t$ such that $ b_i\\equal{}ta_i$ for all $ i$.\r\n\r\nThe case the all the $ b_i$ are $ 0$ is covered when $ t\\equal{}0$.", "Solution_8": "Well, some authors [b]do[/b] call AG ineq Cauchy's inequality. That is probably because Cauchy was the first to prove it in its general form.\r\nIt is sometimes a cause of misunderstanding, as most authors call Cauchy-Schwarz-Bunjakowsky inequality just \"Cauchy's\".\r\n\r\nTitle in ru.wikipedia:\r\n\r\n[quote]\u041d\u0435\u0440\u0430\u0432\u0435\u043d\u0441\u0442\u0432\u043e \u041a\u043e\u0448\u0438 (\u043d\u0435\u0440\u0430\u0432\u0435\u043d\u0441\u0442\u0432\u043e \u043e \u0441\u0440\u0435\u0434\u043d\u0435\u043c \u0430\u0440\u0438\u0444\u043c\u0435\u0442\u0438\u0447\u0435\u0441\u043a\u043e\u043c \u0438 \u0441\u0440\u0435\u0434\u043d\u0435\u043c \u0433\u0435\u043e\u043c\u0435\u0442\u0440\u0438\u0447\u0435\u0441\u043a\u043e\u043c)[/quote]\n\nor, translated\n\n[quote]Cauchy's inequality (inequality between arithmetical and geometrical means)[/quote]", "Solution_9": "Is there a particular reason we were told that a,b,and c are all greater than 0???", "Solution_10": "The OP probably meant $ x, y, z > 0$.", "Solution_11": "That makes sense, especially since square roots are involved..." } { "Tag": [ "geometry", "analytic geometry" ], "Problem": "as you can see here [url]http://www.artofproblemsolving.com/Forum/viewtopic.php?t=224442[/url], there is this new program, i made this topic so people can test geometric figures and maybe post geometry problems....\r\n [geogebra]23e4a1f8ec12aec8dddef211d5d5015d24a8f906[/geogebra]", "Solution_1": "Yeah... geogebra is amazing.\r\n\r\nDo you know how to set up coordinate axes on it? I know that I can just draw two lines, but in order to do that, I need the first line to be parallel (and then I can use the altitude feature to draw the second line) to the sides of my computer screen. How can I do that? Also, how come it won't let me name any of the lines I draw with 'y'? I can call it 'a' or 'line', but it won't let me label it as 'y'." } { "Tag": [ "calculus", "integration", "function", "trigonometry", "complex analysis", "real analysis", "real analysis unsolved" ], "Problem": "What is answer for $\\int {Tan(Tanx)} dx$?\r\nThis problem is making rounds in two other math fora claiming this is a very easy problem, whereas my claim is that it is not possible to find by usual methods.\r\n\r\nPlease post your comment with solution.", "Solution_1": "You are correct. There is no closed form. You can check on http://integrals.wolfram.com if you wish. It is a useful tool for verifying this sort of question.", "Solution_2": "its quite simple..\r\n\r\nI = int{ tan(tanx) dx}\r\n\r\nlet tanx =y\r\n\r\ntherefore\r\ndy/dx = sec^2 x\r\n=> dy/dx = 1+tan^2 x\r\n=> dx = dy/(1+tan^2x) = dy/(1+y^2)\r\n\r\ntherefore\r\n\r\nI = int{(tany/(1+y^2))dy}\r\n\r\nyou can solve this by using integration by parts.. with tany as the first function and 1/(1+y^2) as the second...\r\n\r\naditya", "Solution_3": "sorry mistake.. tany would be the second function according to ILATE (inverse, log, arithmatic, trigo and exp)", "Solution_4": "$\\tan \\circ \\tan x \\not= \\tan^2 x$", "Solution_5": "i think you did not read my answer correctly.. \r\n\r\ni did not say... \r\n\r\ntan(tanx) = tan^2 x\r\n\r\ni assumed tanx =y that makes tan(tanx) = tany\r\n\r\naditya :rotfl:", "Solution_6": "The integral you transform it to is not possible to find an antiderivative to.", "Solution_7": "Thinking of definite integrals:\r\n\r\nNote that $\\tan(\\tan x)$ has infinitely many singularities in each period of the function. Thinking of this in complex analysis terms, we have infinitely many simple poles in the interval $\\left(0,\\frac{\\pi}2\\right)$ and an essential singularity at $\\frac{\\pi}2.$\r\n\r\nThe function would not be integrable over any interval whose closure contains any of these singularities. The nearest singularity to zero would be at $\\arctan\\left(\\frac{\\pi}2\\right).$" } { "Tag": [ "calculus", "integration", "number theory", "algebra unsolved", "algebra" ], "Problem": "(1) for what integral value of x such that the expression \r\n(x^2) +13x +44 is a perfect square.\r\n(x^2) means x square.", "Solution_1": "u should post this in the number theory forum. \r\nanyway, u have onlyto find all the integer solutions of\r\n$ \\minus{}7\\equal{}t^2\\minus{}(2y)^2$ which is easy", "Solution_2": "$ \\bigtriangleup \\equal{} 169 \\minus{} 4(44 \\minus{} y^{2})$ $ \\implies$ $ q^{2} \\equal{} 4y^{2} \\minus{} 7$\r\n$ (2y \\minus{} q)(2y \\plus{} q) \\equal{} 7$ $ \\implies$ \r\n$ y^{2} \\equal{} 4$ $ \\implies$ $ x^{2} \\plus{} 13x \\plus{} 40 \\equal{} 0$\r\n$ \\implies$ $ (x \\plus{} 8)(x \\plus{} 5) \\equal{} 0$ $ \\implies$ \r\n$ x \\equal{} \\minus{} 8$ V $ x \\equal{} \\minus{} 5$\r\n$ (x,y) \\equal{} \\{ ( \\minus{} 8, \\minus{} 2),( \\minus{} 8,2),( \\minus{} 5, \\minus{} 2),(5,2)\\}$" } { "Tag": [ "geometric series", "AoPS Books", "geometric sequence" ], "Problem": "$\\frac{1}{3}+\\frac{2}{9}+\\frac{3}{27}+...$\r\npost solutions. ;)", "Solution_1": "[hide] Set the sum equal to S. Multiplying by 3, we get $3S=1+\\frac{2}{3}+\\frac{3}{9}+...$. Then we subtract the original equation $S=\\frac{1}{3}+\\frac{2}{9}+\\frac{3}{27}+...$, to get $2S=1+\\frac{1}{3}+\\frac{1}{9}+...$. Using the formula for infinite geometric series, we get $\\frac{1}{1-\\frac{1}{3}}=\\frac{1}{\\frac{2}{3}}=\\frac{3}{2}$. So $2S=\\frac{3}{2} \\longrightarrow S=\\boxed{\\frac{3}{4}}$. :D [/hide]", "Solution_2": "[hide]We can express this as a bunch of individual infinite series (the vertical columns):\n\n+(1/3)\n+(1/9)+(1/9)\n+(1/27)+(1/27)+(1/27)\n...\n\nThe sum of an infinite series is $\\frac{a}{1-r}$, and all of the infinite series above differ only in the first term, and the first terms makes up a new infinite series. So the sum is\n\n$\\frac{\\frac{1/3}{1-1/3}}{1-1/3}=(\\frac{1}{3})(\\frac{3}{2})(\\frac{3}{2})=\\boxed{\\frac{3}{4}}$.[/hide]", "Solution_3": "Arithmetico-Gemoteric series. :)", "Solution_4": "[quote=\"numbergeek\"]Arithmetico-Gemoteric series. :)[/quote]\r\n\r\nYes but who remembers the general formula.I usually do the question like the others have and the derivation is basically like that as well.", "Solution_5": "[hide]$S=\\frac{a}{1-r}+\\frac{rd}{(1-r)^2}$\n$a=\\frac{1}{3}$\n$r=\\frac{1}{3}$\n$d=\\frac{1}{3}$\n$S=\\frac{3}{4}$[/hide]", "Solution_6": "[quote=\"numbergeek\"]Arithmetico-Gemoteric series. :)[/quote]\r\n\r\nWhere can I find out more about these types of series? Are they in the AoPS books?", "Solution_7": "[hide]This is basically an infinite number of geometric sequences in one sequence.\n\nWe take note that the sum is basically split like this\n\n$\\frac{1}{3}$\n\n$\\frac{1}{9}+\\frac{1}{9}$\n\n...\n\nSo we addup the columns to get\n\n$\\frac{1}{2}+\\frac{1}{6}+\\frac{1}{18}+\\frac{1}{54}+...$\n\nand this equals\n\n$\\frac{3}{4}$[/hide]", "Solution_8": "In case anyone wants more questions on arithmetico-geometric series-\r\n\r\n1)$1+(1+d)\\frac{1}{5}+(1+2d)\\frac{1}{5^2}+(1+3d)\\frac{1}{5^3}+\\cdots=\\frac{15}{8}$.Find $d$.\r\n\r\n2)Generalise the sum as well as $nth$ term of an arithmetico geometric sequence\r\n\r\n3)Find $1+5^2x+9^2x^2+13^2x^3+\\cdots \\infty$,where $|x|<1$\r\n\r\nI'd be glad to help if anyone wants the methods." } { "Tag": [ "floor function", "function" ], "Problem": "$f(x) = |x-\\left\\lfloor x \\right\\rfloor-\\frac{1}{2}|$\r\n\r\nWhat is the period of $f(x)$?\r\n\r\n$A.$ $1$\r\n$B.$ $2$\r\n$C.$ $3$\r\n$D.$ $4$\r\n$E.$ $f(x)$ is not a period function", "Solution_1": "[hide]$f(x)=|x-\\left\\lfloor x \\right\\rfloor-\\frac{1}{2}|$\n$f(x+1)=|x+1-\\left\\lfloor x+1 \\right\\rfloor-\\frac{1}{2}|=|x-\\left\\lfloor x \\right\\rfloor-\\frac{1}{2}|$\n\n$f(x)=f(x+1)$ Period is 1 $\\boxed{A}$[/hide]" } { "Tag": [ "MATHCOUNTS", "AMC", "AMC 12", "AMC 12 B", "ARML" ], "Problem": "Hey guys,\r\n\r\nI'm planning to hold a MathCounts practice for the kids from Dickerson on Wednesday afternoon and was wondering is any of the locals might be available to talk to them. Specifically, I'm looking for someone that has gone to Nationals. (Santosh, Miles, Harrison, someone else?) This is a really talented group. Two of them were the 2 that tied for the Middle School championship at Rockdale and Claus (the 7th grader) believes that he scored a 105 on the AMC12B!) It's quite possible that there may be 3 or 4 potential ARML members (down the road).\r\n\r\nI'm planning to run the practice from about 3PM until about 5PM at Walton.\r\n\r\nThanks,\r\nTom", "Solution_1": "This is the sort of thing I would love to do and feel like I should be doing, but unfortunately, that's a really awkward time for me. Specifically, this Wednesday, I will be on a bus for New York at that time. :( I would also like to help the kids at Mabry (my old middle school). I wonder if maybe there could be small regional MathCounts practices for interested people in the future, perhaps?\r\n\r\n\u2014Miles", "Solution_2": "I'd be happy to host a regional practice. I'm committed to Dodgen and Dickerson of course, but would like to see all of Cobb County grow. My intent is to host an ARML style practice. Basically working through an entire competition. I hope that this won't be the last practice that we have before state and you would be more than welcome at a future practice. I don't remember where Mabry placed at the chapter competition. (Or if they attended.) I'm pretty sure that Hightower Trail is the other Cobb County team going to state.\r\n\r\nFulton" } { "Tag": [ "ratio", "geometry", "analytic geometry" ], "Problem": "The triangle ABC has sides AB=3, BC=4, AC=5. The inscribed circle is tangent to AB in C\u2019, BC in A\u2019, and AC in B\u2019. What is the ratio between the areas of the triangles A\u2019B\u2019C\u2019 and ABC ?", "Solution_1": "[hide]1/5[/hide]", "Solution_2": "How'd you get that? I got this:\r\n[hide]First, the big triangle is a right triangle. Then inscribe the circle and label the parts of the sides of the big triangle that are cut by the circle (they are tangents to the circle) x, x, 3-x, 3-x, 4-x, 4-x. You'll get that the hypotenuse =4-x+3-x, and it also=5, so x=1. Then with that, you can get the coordinates of the little triangle to be (0,1) (1,0) (1.5,2). Use the formula A=.5|determinant of vertices| to get an area of 5/4. That makes the ratio 5/4*6=5/24.[/hide]", "Solution_3": "Can anyone confirm the answer?", "Solution_4": "I agree with Danbert.\r\n\r\n[hide]so AC'=AB', CB'=CA', BC'=BA'\n\nBA'+AC'=4, AC'+C'B=3, AB'+B'C=5. \n\nSolve we have BA'=1, A'C=3, AC'=2.\n\nUse [AB'C']=2(2)(sinBAC)/2, [BA'C']=1(1)/2, [CA'B']=3(3)(sinACB)/2\nsince sinBAC=4/5, sinACB=3/5. Sub it back in you get \n [AB'C']+[BA'C']+[CA'B']=24/5, [A'B'C']=[ABC] - 24/5=6/5. Divide.[/hide]\r\n\r\n[quote=\"Wumbate\"]coordinates of the little triangle to be (0,1) (1,0) (1.5,2)[/quote]\r\n\r\nThe third coordinate is not true.", "Solution_5": "I find the answer of the small triangle to be [hide]6 - 1/2 - 27/10 - 8/5 = 6/5, which makes the ratio 1/5 as Danbert said.[/hide]" } { "Tag": [ "geometry", "trapezoid", "perpendicular bisector" ], "Problem": "[color=darkblue]Let $ABCD$ be a trapezoid for which $AB\\parallel CD$ and $A=D$.\nDenote the middlepoint $M$ of the side $[AD]$.\nDefine the point $P$ so that $PA\\perp MB$ and $PD\\perp MC$.\nProve that $MP\\perp BC$.\n\n[b]Indication.[/b] $X\\in PA\\cap MB\\ ,\\ Y\\in PD\\cap MC\\Longrightarrow$ the points $X,Y,B,C$ are conciclycally.\n\n[b][u]Remark.[/u][/b] Maybe there are more proofs for this nice and easy (in my opinion) problem.\nI have at least three proofs (syntetically, metrically and analytically).\n\n[b]See[/b][/color] http://www.mathlinks.ro/Forum/viewtopic.php?t=100104", "Solution_1": "Could you please give a hint? :maybe:", "Solution_2": "[hide=\"Hint 1 (for syntetical method).\"] $AM^{2}=MX\\cdot MB\\ ,\\ DM^{2}=MY\\cdot MC\\ ,\\ AM=DM\\Longrightarrow$\n$MX\\cdot MB=MY\\cdot MC\\Longrightarrow$ the points $X,Y,B,C$ are conciclycally.[/hide] \n[hide=\"Hint 2 (for metrical method).\"] [b][u]Lemma.[/u][/b] $AB\\perp CD\\Longleftrightarrow AC^{2}-AD^{2}=BC^{2}-BD^{2}\\ .$[/hide]", "Solution_3": "What does A=D mean?", "Solution_4": "Sorry Shobber, $A=m(\\widehat{BAD})$ and $D=m(\\widehat{ADC})\\ .$ Here is a particular case of the problem from http://www.mathlinks.ro/Forum/viewtopic.php?t=100104", "Solution_5": "It seems that now you are doing a research on this kind of diagram, Virgil!\r\n\r\nSince $\\angle{A}=\\angle{D}$, so easily we can get $PA=PD$. But because the locus of the point with equal distance to A and D is the perpendicular bisector of AD, thus P is on the line and thus $PM \\perp BC$.", "Solution_6": "In the problem posted in: http://www.mathlinks.ro/Forum/viewtopic.php?t=100104\r\n\r\nMake $B_{1},A, B_{2}$ be collinear", "Solution_7": "[quote=\"shobber\"]It seems that now you are doing a research on this kind of diagram, Virgil!\n\nSince $\\angle{A}=\\angle{D}$, so easily we can get $PA=PD$. But because the locus of the point with equal distance to A and D is the perpendicular bisector of AD, thus P is on the line and thus $PM \\perp BC$.[/quote]No, $PA\\ne PD$ ! Read with more attention.", "Solution_8": "[quote=\"Virgil Nicula\"][quote=\"shobber\"]It seems that now you are doing a research on this kind of diagram, Virgil!\n\nSince $\\angle{A}=\\angle{D}$, so easily we can get $PA=PD$. But because the locus of the point with equal distance to A and D is the perpendicular bisector of AD, thus P is on the line and thus $PM \\perp BC$.[/quote]No, $PA\\ne PD$ ! Read with more attention.[/quote]\r\n :D I nearly did $PA=PD$ in my solution. Good thing I didn't post it.", "Solution_9": "[color=darkblue][b][u]A nice equivalent enunciation.[/u][/b] Given are a triangle $ABC$ and a line $d$ for which $A\\in d$.\nDefine the points: $M\\in d$, $BM\\perp d$; $N\\in d$ $CN\\perp d$; the point $P$ so that $PM\\perp AB$ and $PN\\perp AC$.\nProve that $AP\\perp BC$ if and only if $AM=AN$.[/color]" } { "Tag": [ "topology", "advanced fields", "advanced fields unsolved" ], "Problem": "If $ G$ is a topological group and $ H\\triangleleft G$, but $ H$ is not closed in $ G$, is it possible that $ G/H$ with the standard quotient topology is not a topological group? I know that closed gets you $ T_2$, but do you lose $ T_1$ sometimes if $ H$ is not closed?", "Solution_1": "OK, let's try an example: $ \\mathbb{R}/\\mathbb{Q}$. The quotient has the indiscrete topology- only two open sets.", "Solution_2": "hm? of course $ G/H$ is a topological group. it is $ T_1$ iff it is $ T_2$ iff $ H$ is closed.", "Solution_3": "Ah, that may be the problem. In my definition of topological group it is required to be $ T_1$, so by those biconditionals, $ H$ not closed implies $ G/H$ not $ T_1$ and thus not a topological group. Without requiring $ T_1$, I knew that the quotient would still satisfy the topological group properties. Thanks." } { "Tag": [ "function", "calculus", "calculus computations" ], "Problem": "apply the intermediate value property of continuous functions to show that the given equation has a solution in the given interval.\r\n\r\nX^3 =5 on (1,2)", "Solution_1": "First you should notice three things:\r\n(1) the function $f(x)=x^{5}$ is continuous;\r\n(2) $f(1)<5$;\r\n(3) $f(2)>5$.\r\n\r\nGiven (1-3), the intermediate value theorem yields that $f(x)=5$ for some $x$ between $1$ and $2$." } { "Tag": [ "inequalities proposed", "inequalities" ], "Problem": "Prove that \\[\\sum_{i=-2007}^{2007}\\frac{\\sqrt{|i+1|}}{(\\sqrt2)^{|i|}}>\\sum_{i=-2007}^{2007}\\frac{\\sqrt{|i|}}{(\\sqrt2)^{|i|}}\\]", "Solution_1": "move the sums, its trivial\r\n\r\n|i+1|>|i|\r\n\r\nsquare root of 2 to the power of |i| is postive", "Solution_2": "[quote=\"Tomaths\"]Its trivial\n\n|i+1|>|i|\n\n[/quote]\r\n$i=-1$.Its not trivial at all." } { "Tag": [], "Problem": "Bonjour tout le monde! :lol: \r\nJe ne peux pas parler le francais tres bien, alors je suis desolee si vous ne pouvez pas bien comprendre ce que j'ecris mais j'ai besoin de votre aide...\r\nJe suis etudiante de dixieme annee dans une ecole canadienne et pour quelque annee maintenant j'etudiais le francais. Pour le projet de l'unite deux \"souvenirs d'enfance,\" on a du (en groupes de six) ecrire un livre d'enfants. Le mienne est 6 pages mais si on ajoute les photos et si on met le font a 14 c'est 30 pages.\r\nDe toute facon...peut quelq'un m'aider a verifier ce que j'ai ecrit?\r\nAssez de paroles...svp le projet est pour le 25 janvier mais je dois aller a l'emprimer devant cette date! Et je dois le memoriser aussi...Y a-t-il quelq'un qui peut m'aider?? Peut on m'envoyer un pm et je vais vous envoyer mon adresse d'email.\r\nMerci,\r\nANI\r\n\r\nP.S. La plupart de gens pensent que les canadiens savent parler le francais tres bien comme l'anglais et le francais sont les langues officieles, mais confiez-moi, la plupart des canadiens DETESTENT le francais...au Toronto au moins... :?", "Solution_1": "Peut-on effacer le derniere message parce que j'ai recu l'aide dont j'avais besoin.\r\nMerci" } { "Tag": [ "LaTeX", "puzzles" ], "Problem": "1. If there were 10 birds on a tree branch, and a hunter shot two thirds of half the birds, how many were remaining?\r\n\r\n2. If a shape had four sides and two corners, would it be considered a polygon?\r\n\r\n3. $\\frac{x}{7}=1$ what is x? lol i dont know why i put this here $\\LaTeX$\r\n\r\n4. Bob walked into a building and sat down in a chair. A nurse knocked him out, and he was in the waiting room when he came around. He was bandaged and walked away without saying anything. Why?", "Solution_1": "[hide]\n1) None, some would be dead and fall off the Branch.\nThe others would fly away due to the gunshots.\n2) No\n3) 7\n4) He had to get surgery. He was knocked out by anesthetics.\nThen surgery was done, and when he came around, he was bandaged from the surgery (assume some incision was made) and was in the waiting room, ready to go. :)\n[/hide]", "Solution_2": "[quote=\".:Logic:.\"][hide]\n1) None, some would be dead and fall off the Branch.\nThe others would fly away due to the gunshots.\n2) No\n3) 7\n4) He had to get surgery. He was knocked out by anesthetics.\nThen surgery was done, and when he came around, he was bandaged from the surgery (assume some incision was made) and was in the waiting room, ready to go. :)\n[/hide][/quote]\n[hide=\"Better one for 3(i think)\"]X is a letter, and the 24th letter of the alphabet.[/hide]", "Solution_3": "[quote=\"ckck\"][quote=\".:Logic:.\"][hide]\n1) None, some would be dead and fall off the Branch.\nThe others would fly away due to the gunshots.\n2) No\n3) 7\n4) He had to get surgery. He was knocked out by anesthetics.\nThen surgery was done, and when he came around, he was bandaged from the surgery (assume some incision was made) and was in the waiting room, ready to go. :)\n[/hide][/quote]\n[hide=\"Better one for 3(i think)\"]X is a letter, and the 24th letter of the alphabet.[/hide][/quote]\n\n[hide]Logic, for number 1 and 2 you are right. Ckck for number 3 you are right. No one has solved number 4[/hide][/hide]", "Solution_4": "4. By 'knocked out' of the chair, he means he got up to let the nun sit down.", "Solution_5": "[quote=\"krums3\"]4. By 'knocked out' of the chair, he means he got up to let the nun sit down.[/quote]\r\n\r\nNope :D This is a hard one", "Solution_6": "Original poster,\r\n\r\nyour first one doesn't make sense for consideration because\r\n(2/3) of (1/2) = (1/3). And you can't hit 1/3 of 10 birds; you must hit a whole number of birds.\r\n\r\n\r\nIt turns out, they are NOT \"easy,\" so you can drop that word. (Leave it to the readers to decide.) You contradicted yourself with your post of:\r\n\r\n\"Nope :D This is a hard one\"", "Solution_7": "ckck, \r\n\r\nfor the 3rd question, you didn't answer the question. \r\n\r\nIt asked about a lower case ex. Your response was about a capital ex. If the poster above you can't get it right, you shouldn't be allowed to get it right either.\r\n\r\nAlso if the basis of this question is that the \"x\" asked about is different than the ex in italics in the equation, the original poster's technicality is as much as mine pointed out in the paragraph above this one, IMO." } { "Tag": [ "algorithm" ], "Problem": "http://mathworld.wolfram.com/CoinProblem.html they say An explicit solution is known for n=3, where can I find it?", "Solution_1": "I don't know the exact definition of 'explicit solution', but I was under the impression that there were quick algorithms for n=3, but no closed form. I could be wrong though. I can't seem to find a link outlining the algorithm either.", "Solution_2": "I found it on CiteSeer:\r\n[url=http://citeseer.ist.psu.edu/cache/papers/cs/11744/http:zSzzSzwww.cs.cmu.eduzSzafszSzcs.cmu.eduzSzuserzSzkannanzSzpubliczSzwwwzSz.zSzPaperszSz.zSzfrobenius.pdf/kannan89solution.pdf]Solution to the Frobenus Problem and Generalizations.[/url]" } { "Tag": [ "linear algebra", "matrix", "complex numbers" ], "Problem": "Let $ \\mathsf{Q}$ be a $ (2n \\plus{} 1) \\times (2n \\plus{} 1)$ orthogonal matrix $ (n \\in \\mathbb{N})$ with $ \\det \\mathsf{Q} \\equal{} 1$. Show that $ \\mathsf{Q}$ has a unit eigenvalue. What is a geometric interpretation of this result for a $ 3 \\times 3$ matrix?", "Solution_1": "$ Q$ can be diagonalized over the complex numbers, and all its eigenvalues have modulus $ 1$. if $ Q$ has real entries, the complex eigenvalues come in conjugate pairs whose products are all 1. thus, since $ |Q|\\equal{}1$, the product of the real eigenvalues is $ 1$, and since the dimension of $ Q$ is odd, at least one of them must be $ 1$.", "Solution_2": "Hence $ Q$ fixes at least one line. The geometric interpretation in the $ 3\\times 3$ case is that that line is the axis of rotation.\r\n\r\n(Real orthogonal matrices with determinant $ 1$ are rotations.)", "Solution_3": "[quote=\"pleurestique\"]$ Q$ can be diagonalized over the complex numbers, and all its eigenvalues have modulus $ 1$.[/quote]\r\n\r\nSorry, but how do we know this?", "Solution_4": "By Schur's Lemma, any complex matrix can be unitarily triagularized. That is, there exists $ P$ with $ P^*P\\equal{}I$ such that $ P*QP\\equal{}T,$ where $ T$ is upper triangular. (I'm using $ A^*$ to mean the Hermitian adjoint, or conjugate transpose, of $ A.$) Then $ Q\\equal{}PTP^*$ and $ QQ^T\\equal{}QQ^*\\equal{}PTP^*PT^*P^*\\equal{}PTT^*P^*\\equal{}I,$ from which it follows that $ TT^*\\equal{}I.$\r\n\r\nBut the inverse of any upper triangular matrix is upper triangular, which means that $ T^*$ is both upper triangular and lower triangular, which means that $ T$ is diagonal.\r\n\r\nLet $ T\\equal{}\\begin{bmatrix}\\lambda_1&&&\\\\&\\lambda_2&&\\\\&&\\ddots&\\\\&&&\\lambda_n\\end{bmatrix}.$\r\n\r\nFrom the fact that $ TT^*\\equal{}I,$ we get that $ \\lambda_j\\overline{\\lambda_j}\\equal{}1$ for each $ j.$\r\n\r\nSo $ Q$ can diagonalized over the complexes, and $ |\\lambda_j|\\equal{}1$ for each eigenvalue $ \\lambda_j.$", "Solution_5": "More generally, any normal matrix is unitarily similar to the diagonal matrix of its eigenvalues ($ A$ is a normal matrix iff $ AA^* \\equal{} A^*A$).", "Solution_6": "[quote=\"Kent Merryfield\"]By Schur's Lemma, any complex matrix can be unitarily triagularized. That is, there exists $ P$ with $ P^*P \\equal{} I$ such that $ P*QP \\equal{} T,$ where $ T$ is upper triangular. (I'm using $ A^*$ to mean the Hermitian adjoint, or conjugate transpose, of $ A.$) Then $ Q \\equal{} PTP^*$ and $ QQ^T \\equal{} QQ^* \\equal{} PTP^*PT^*P^* \\equal{} PTT^*P^* \\equal{} I,$ from which it follows that $ TT^* \\equal{} I.$\n[/quote]\r\n\r\nI learnt something new here, Schur's Lemma is not yet covered in my course. \r\n\r\nThanks Prof Merryfield and Carcul!" } { "Tag": [ "geometry" ], "Problem": "I didn't know where to post this, since it doesn't seem to be what I call a geometry problem. Anyway, here it goes:\r\n\r\nIn the plane lines $\\ell_i,\\ i\\in\\overline{1,n}$ are given, together with a point $P$. We project $P$ on $\\ell_1$, then we project this point on $\\ell_2$ and so on indefinitely (when we reach $\\ell_n$ we go bak to $\\ell_1$). Show that the set of points we have obtained is bounded.", "Solution_1": "The easiest way is to use complex numbers.\r\nLet the points we numbered $P_1, \\cdots P_n \\cdots$ with affices\r\n $z_1, \\cdots z_n \\cdots$.\r\nThe we can see that $z_{i+1}=w z_i+ \\alpha$ where $|w|<1$ for $w, \\alpha$ depending on the line we project.\r\nThe if we tale $M=max\\{|z|, \\frac{\\|alpha\\|}{1-|w|}\\}$ we can see that if $|z_i|0$, $ p_n(x)\\equal{}p_0\\left(x\\minus{}\\dfrac{n\\cdot(n\\plus{}1)}{2}\\right)$\r\n\r\n$ p_{20}(x)\\equal{}p_{19}(x\\minus{}20)\\equal{}p_{18}(x\\minus{}39)\\cdots \\equal{}p_0(x\\minus{}(20\\plus{}19\\plus{}\\cdots \\plus{}1))$\r\n\r\nSo $ p_{20}(x)\\equal{}p_0(x\\minus{}210)\\equal{}(x\\minus{}210)^3 \\plus{} 313(x\\minus{}210)^2 \\minus{} 77(x\\minus{}210) \\minus{} 8$\r\n\r\nTaking each coefficient of $ x$ in each term we have $ (3\\cdot (\\minus{}210)^2)\\plus{}(313\\cdot (2)(\\minus{}210))\\minus{}77\\equal{}\\boxed{763}$", "Solution_2": "Please use Contests (or Resources on MathLinks Skin) to see the answers to previous years' AIME questions.\r\n\r\nhttp://www.artofproblemsolving.com/Forum/viewtopic.php?p=461986#461986\r\n\r\nLocked." } { "Tag": [ "LaTeX" ], "Problem": "factor the following:\r\n$12x^2 \\cdot y^3 \\cdot z +18x^3 \\cdot y^2 \\cdot z^2-24xy^4 \\cdot z^3$", "Solution_1": "$\\displaystyle 12x^2 \\cdot y^3 \\cdot z +18x^3 \\cdot y^2 \\cdot z^2-24xy^4 \\cdot z^3$\r\n\r\n$= 6x{y}^2z \\cdot (2xy + 3{x}^2z - 4{y}^2{z}^2)$\r\n\r\ni dont think it simplifies further than that", "Solution_2": "i agree. I'm too lazy to use Latex[/hide]", "Solution_3": "$\\displaystyle 12x^2 \\cdot y^3 \\cdot z +18x^3 \\cdot y^2 \\cdot z^2-24xy^4 \\cdot z^3= 6x{y}^2z \\cdot (2xy + 3{x}^2z - 4{y}^2{z}^2)$", "Solution_4": "i think that's what i got. so, good.", "Solution_5": "cool, i got it too", "Solution_6": "cool", "Solution_7": "nice same" } { "Tag": [ "inequalities", "inequalities proposed" ], "Problem": "Prove that [tex]\\frac{(\\sum_{i=1}^{n}a_i)^k}{(\\sum_{i=1}^{n}b_i)^p}\\leq{\\sum_{i=1}^{n}\\frac{a_i^k}{b_i^p}[/tex], for [tex]a_i,b_i,k,p>0[/tex].", "Solution_1": "are you sure about it", "Solution_2": "Yeah, I see counterexamples.", "Solution_3": "Yes, i`m not sure about it. I want to know when it\u00b4s right. :lol:", "Solution_4": "[quote=\"probability1.01\"]Yeah, I see counterexamples.[/quote]\r\n\r\n Where are the counterexamples?", "Solution_5": "[quote=\"Lucas Molina\"]Prove that [tex]\\frac{(\\sum_{i=1}^{n}a_i)^k}{(\\sum_{i=1}^{n}b_i)^p}\\leq{\\sum_{i=1}^{n}\\frac{a_i^k}{b_i^p}[/tex], for [tex]a_i,b_i,k,p>0[/tex].[/quote]\r\n\r\n\r\n For [tex]k<1,p+k=1[/tex] it`s right.\r\n\r\n Proof: By Jensen , let [tex]f(x)=x^k[/tex], then [tex]f(\\frac{\\sum_{i=1}^{n}a_i}{n.\\sqrt[k]{(\\sum_{i=1}^{n}b_i)^p}})\\leq\\frac{\\sum_{i=1}^{n}a_i^k}{n.(\\sum_{i=1}^{n}b_i)^p}[/tex], then [tex]\\frac{(\\sum_{i=1}^{n}a_i)^k}{(\\sum_{i=1}^{n}b_i)^p}\\leq\\frac{n^{k-1}.\\sum{i=1}^{n}a_i^k}{(\\sum_{i=1}^n}b_i)^p}\\leq\\frac{n^{k-1}.\\sum_{i=1}^{n}a_i^k}{\\sum_{i=1}^{n}b_i^p}[/tex] (*).\r\n\r\n Let [tex]k<1[/tex], then (*)=[tex]\\frac{\\sum_{i=1}^{n}a_i^k}{n^{1-k}.\\sum_{i=1}^{n}b_i^p}[/tex][tex]\\leq\\frac{\\sum_{i=1}^{n}a_i^k}{\\sum_{i=1}^{n}b_i^p}[/tex][tex]\\leq{\\sum_{i=1}^{n}\\frac{a_i^k}{b_i^p}[/tex]. For [tex]p+k=1[/tex], the proof is equal.", "Solution_6": "[quote=\"zhaobin\"]are you sure about it[/quote]\r\n\r\n YES, now i am!!! :)", "Solution_7": "yeah :) \r\nbut I think it is another form of hoider inequality. ;) \r\nif I rememeber correctly", "Solution_8": "[quote=\"Lucas Molina\"]Prove that $ \\frac {(\\sum_{i = 1}^{n}a_i)^k}{(\\sum_{i = 1}^{n}b_i)^p}\\leq{\\sum_{i = 1}^{n}\\frac {a_i^k}{b_i^p}}$, for $ a_i,b_i,k,p > 0$.[/quote]\r\n\r\n Proof for all $ k > p$:\r\n By Jensen, let $ f(x) = x^k$, then \r\n\r\n$ f(\\frac {\\sum_{i = 1}^{n}\\frac {a_i}{b_i^{\\frac {p}{k}}}}{n})\\leq\\frac {\\sum_{i = 1}^{n}\\frac {a_i^k}{b_i^p}}{n}$. Then we have that \r\n\r\n$ \\frac {\\sum_{i = 1}^{n}\\frac {a_i}{b_i^{\\frac {p}{k}}}}{n^k}\\leq\\frac {\\sum_{i = 1}^{n}\\frac {a_i^k}{\\sqrt [k]{b_i^p}}}{n}$ => \r\n\r\n$ \\frac {\\sum_{i = 1}^{n}\\frac {a_i}{\\sqrt [k]{b_i^p}}}{n^{k - 1}}\\leq\\sum_{i = 1}^{n}\\frac {a_i^k}{b_i^p}$\r\n\r\n Then , we have to prove that $ \\frac {(\\sum_{i = 1}^{n}a_i)^k}{(\\sum_{i = 1}^{n}b_i)^p}\\leq{\\frac {(\\sum_{i = 1}^{n}\\frac {a_i}{\\sqrt [k]{b_i^p}})}{n^{k - 1}}}$\r\n\r\n Then we need to prove that $ \\frac {\\sum_{i = 1}^{n}a_i}{(\\sum_{i = 1}^{n}b_i)^{\\frac {p}{k}}}\\leq{\\frac {\\sum_{i = 1}^{n}\\frac {a_i}{\\sqrt [k]{b_i^p}}}{n^{\\frac {k - 1}{k}}}}$. \r\n\r\n See that $ \\frac {\\sum_{i = 1}^{n}a_i}{\\sum_{i = 1}^{n}\\sqrt [k]{b_i^p}}\\leq{\\sum_{i = 1}^{n}\\frac {a_i}{\\sqrt [k]{b_i^p}}}$, then we going to prove that \r\n\r\n$ (\\sum_{i = 1}^{n}b_i)^{\\frac {p}{k}}\\leq{n^{\\frac {k - 1}{k}}.\\sum_{i = 1}^{n}\\sqrt [k]{b_i^p}}$. It`s proved by Jensen , then we have that $ 2k\\geq{p + 1}$ => $ k > p$.", "Solution_9": "I'm sure there is something wrong with you.\r\nbut I don't know where.\r\nlet $a_i=1,b_i=1$\r\n$LHS=n^(k-p),RHS=n$,if k>p+1 will be not true.\r\nlet $a_i=a,b_i=1$,\r\nI forget the conterexample for $k> , something that is unknow to the most ... preolympiad 'ers.Use very elementary theorems.You ' ll need to drow some auxiliary but very basic segments!!!\r\n\r\n Babis", "Solution_3": "Because the circumcircle of the triangle $ABC$ is the incircle of the triangle $UVW$, where $U,\\ V,\\ W$ are the centers of the circles $k,\\ l,\\ m$ respectively, I offer a equivalent enunciation of this problem:\r\n\r\n\"The incircle of the triangle $ABC$ touchs the its sides in the points $D\\in BC,\\ E\\in CA,\\ F\\in AB$. I note the second intersection $L$ between the line $DF$ with the circle $w=C(C,CD)$. Prove that $EL\\perp EF.\\\"$\r\n\r\n[b]A solution.[/b]\r\n\r\n$m(\\widehat {EFL})=m(\\widehat {EDL})=\\frac 12 (A+B),\\ m(\\widehat {FLE})=m(\\widehat {DLE})=\\frac 12 m(\\widehat {DCE})$ $=\\frac 12 C\\Longrightarrow$\r\n\r\n$m(\\widehat {LEF})=180^{\\circ} -\\frac 12 (A+B+C)=90^{\\circ}\\Longrightarrow EL\\perp EF.$", "Solution_4": "Perfect !\r\n\r\nThis is exactly my solution , too!\r\n\r\n Babis", "Solution_5": "in my solution , u can see( arc AC=arc CD),without using homothecy.\r\n\r\ni said homothecy there to make it clear .", "Solution_6": "in my solution , u can see( arc AC=arc CD),without using homothecy.\r\n\r\ni said homothecy there to make it clear . ;)" } { "Tag": [ "geometry", "rhombus", "incenter", "geometric transformation", "homothety", "similar triangles", "geometry unsolved" ], "Problem": "Let ABCD be a rhombus. A tangent line to its incircle intersects BC at E and CD at F. The incircle touches AD at T. Prove that AE and TF are parallel.", "Solution_1": "Let $ O$ be incenter, $ D_1,D_2,D_3,D_4$ toucing point with $ BC,CD,EF,AB$.\r\nLet $ X$ and $ Y$ be polar points for $ TF$ and $ AE$.\r\n\r\nBy Pascal theorem for degenerate hexagon $ TTD_2D_3D_1D_4$ \r\nwe get $ X,Y,O$ are collinear. \r\n\r\nThen $ AE\\parallel{}TF$", "Solution_2": "I am sorry\r\nI cannot figure out...\r\nI have found on wikipedia what \"Pascal theorem\" is\r\nIs the conclusion of this theorem to the hexagon $ TTD_2D_3D_1D_4$ the following, that the three intersections: $ TT, D_3D_1$, $ TD_2, D_1D_4$ and $ D_2D_3, D_4T$ are on the same line(on the projective plane)?\r\n\r\n[quote=\"Number1\"]Let $ O$ be incenter, $ D_1,D_2,D_3,D_4$ toucing point with $ BC,CD,EF,AB$.\nLet $ X$ and $ Y$ be polar points for $ TF$ and $ AE$.\n\nBy Pascal theorem for degenerate hexagon $ TTD_2D_3D_1D_4$ \nwe get $ X,Y,O$ are collinear. \n\nThen $ AE\\parallel{}TF$[/quote]", "Solution_3": "[quote=\"LaurT\"]Let ABCD be a rhombus. A tangent line to its incircle intersects BC at E and CD at F. The incircle touches AD at T. Prove that AE and TF are parallel.[/quote]\r\n\r\nProbably this will help:\r\n\r\n I have seen a solution(elementary) in Crux ( issues 6 , page 351 ) of this year , using homothecy.\r\n\r\n Babis", "Solution_4": "[quote=\"LaurT\"]Let ABCD be a rhombus. A tangent line to its incircle intersects BC at E and CD at F. The incircle touches AD at T. Prove that AE and TF are parallel.[/quote]\r\nDear [b]stergiu[/b], is this the solution you were speaking of?\r\n[hide=\"Solution\"]\nApply Brianchon's theorem to $ ATDFEB$, so $ BD$, $ AF$, and $ ET$ concur. Furthermore, we have that $ AB$ and $ FD$ are parallel as are $ BE$ and $ TD$. Combining this with the concurrence, we have that a homothety maps triangle $ ABE$ to triangle $ FDT$ and thus $ FT$ and $ AE$ are parallel. [/hide]", "Solution_5": "Let us to present an elementary approach.\r\n\r\nWe denote the point $ P\\equiv BD\\cap ET$ and because of $ BC\\parallel AD,$ we have that $ \\frac {BE}{DT} \\equal{} \\frac {PE}{PT} \\equal{} \\frac {PB}{PD}$ $ ,(1)$\r\n\r\nLet $ T'$ be, the tangency point of the incircle $ (O)$ of $ ABCD,$ to its side-segment $ CD.$\r\n\r\nIt is easy to prove ( by an angle changing in the configuration of the triangle $ \\bigtriangleup CEF,$ with the circle $ (O)$ as its $ C$-excircke ) that $ \\angle BOE \\equal{} \\angle DFO$ and so, from the similar triangles $ \\bigtriangleup BEO,\\ \\bigtriangleup DOF,$ we have $ \\frac {BE}{DO} \\equal{} \\frac {BO}{DF}$ $ \\Longrightarrow$ $ (BE)\\cdot (DF) \\equal{} (DO)^{2}$ \r\n\r\n$ \\Longrightarrow$ $ (BE)\\cdot (DF) \\equal{} (DT')\\cdot (DC)$ $ ,(2)$ $ ($ because of $ (DO)^{2} \\equal{} (DT')\\cdot (DC)$ from the right triangle $ \\bigtriangleup OCD$ $ ).$\r\n\r\nFrom $ (2)$ $ \\Longrightarrow$ $ \\frac {BE}{DT'} \\equal{} \\frac {DC}{DF}$ $ \\Longrightarrow$ $ \\frac {BE}{DT} \\equal{} \\frac {AB}{DF}$ $ ,(3)$\r\n\r\nFrom $ (1),$ $ (3)$ $ \\Longrightarrow$ $ \\frac {AB}{DF} \\equal{} \\frac {PB}{PD}$ $ ,(4)$\r\n\r\nFrom $ (4)$ and because OF $ AB\\parallel DF,$ we conclude the collinearity of the points $ A,\\ P,\\ F$ and so, we have that $ \\frac {PB}{PD} \\equal{} \\frac {PA}{PF}$ $ ,(5)$\r\n\r\nHence, from $ (1),$ $ (5)$ $ \\Longrightarrow$ $ \\frac {PE}{PT} \\equal{} \\frac {PA}{PF}$ $ \\Longrightarrow$ $ AE\\parallel TF$ and the proof is completed.\r\n\r\nKostas Vittas.", "Solution_6": "Hi everyone. \r\nI know that Vittasko,Number 1, Quatto Master and Babis have given brilliant proofs for this problem. However I was just wondering if an inversive solution is feasible here. Since we know that in triangle $ ABE$ and $ TFD$ we have $ AB$ parallel to $ DF$ and $ BE$ parallel to $ TD$ when we invert the whole figure with respect to the incircle of the rhombus we would get a family of circles passing through $ O$(the center of the incircle of the rhombus.) many of which are tangent circles in pairs. Now we can use this to prove that under the inversion lines $ AE$ and $ TF$ are transformed into tangent circles the point of tangency being $ O$ the center of incircle of the rhombus, since this would mean that lines $ AE$ and $ TF$ are parallel.\r\nCould anyone help me with a rigorous proof using this approach or inversion in general.\r\n\r\nThanks in advance.", "Solution_7": "Hi everyone,\r\nCan someone please help me or otherwise give an inversive solution to this problem.", "Solution_8": "Here is my solution:\r\nLet U,V,W,M be the points of tangency of the circle to BC, CD, AB, and EF, O the center of the circle with radius r, let $ WB \\equal{} BU \\equal{} VD \\equal{} DT \\equal{} x$, and let $ \\angle WOB \\equal{} \\angle BOU \\equal{} \\alpha$. Then, we have to show $ \\triangle ABE$ and $ \\triangle TFD$ are similar, or $ \\frac {x}{x \\plus{} VF} \\equal{} \\frac {x \\plus{} UE}{x \\plus{} AW}$, or $ x \\cdot AW \\equal{} x(VF \\plus{} UE) \\plus{} VF \\cdot UE$. But note that $ \\triangle AOB$ is a right triangle with altitude OW, so $ x \\cdot AW \\equal{} r^2$. So we have to show $ r^2 \\equal{} x(VF \\plus{} UE) \\plus{} VF \\cdot UE$.\r\nNote that $ \\angle WOV \\equal{} 180$, so $ \\angle UOV \\equal{} 180 \\minus{} 2 \\alpha$. But $ \\angle UOE \\equal{} \\angle EOM$ and $ \\angle MOF \\equal{} \\angle FOV$, so $ \\angle EOF \\equal{} (1/2) \\angle UOV \\equal{} 90 \\minus{} \\alpha$. Thus, $ \\frac {r}{x} \\equal{} cot \\alpha \\equal{} tan(90 \\minus{} \\alpha) \\equal{} tan EOF \\equal{} tan(EOM \\plus{} MOF) \\equal{} \\frac {\\frac {UE}{r} \\plus{} \\frac {VF}{r}}{1 \\minus{} \\frac {UE \\cdot VF}{r^2}}$. But\r\n$ \\frac {r}{x} \\equal{} \\frac {\\frac {UE}{r} \\plus{} \\frac {VF}{r}}{1 \\minus{} \\frac {UE \\cdot VF}{r^2}}$ is equivalent to what we wanted to show." } { "Tag": [], "Problem": "given that $a^{2}+b^{2}=ab$, find $\\frac{a^{5}}{b^{5}}$\r\n\r\nwait a minute...I don't think I'm doing anything wrong, but plugging the answer back into the equation gives an incorrect eequation ($2a^{2}=-a^{2}???$) :blush: \r\nI started with $a^{3}+b^{3}=0$, then divided by $(a+b)$ to get $a^{2}+b^{2}-ab=0$, which can be rearranged into the original equation. this means that $\\frac{a^{5}}{b^{5}}=\\frac{a^{3}}{b^{3}}=-1$\r\n[b]WHAT AM I DOING WRONG?????[/b]", "Solution_1": "Unless I am wrong:\r\n\r\n$\\frac{a^{5}}{b^{5}}= \\frac{1}{2}\\pm \\frac{\\sqrt{3}}{2}i$", "Solution_2": "ok I figured out what I did wrong :D If you want, I will leave it as a challenge to try and figure out by yourself\r\n\r\nif you just want to know, \r\n[hide]going through my solution backwards is fine, but dividing by $(a-b)$ is equal to dividing by zero when $a=-b$. This invalidates the equation[/hide]", "Solution_3": "Isn't it when $a=b$?\r\n\r\n[hide]Shouldn't the answer be $1,-1$?[/hide]", "Solution_4": "he meant $a+b$" } { "Tag": [], "Problem": "When not on the moving sidewalk, Kelsey can walk the length of the sidewalk in 3 minutes. If she stands on the sidewalk as it moves, she can travel the length in 2 minutes. If Kelsey walks on the sidewalk as it moves, how many minutes will it take her to travel the same distance? Assume she always walks at the same speed, and express your answer as a decimal to the nearest tenth.", "Solution_1": "[hide=\"answer\"]k = speed of Kelsey\ns = speed of sidewalk\nl = length of sidewalk\n$k = \\frac{l}{3}$\n$s = \\frac{l}{2}$\n$k+s = \\frac{l}{3} + \\frac{l}{2}$\n$k+s = \\frac{5l}{6} = \\frac{l}{\\frac{6}{5}}$\nSo she can walk down the sidewalk in 6/5 minutes, or [color=green]1.2 minutes[/color][/hide]", "Solution_2": "[hide]\nWhen walking not on the sidewalk, she goes $\\frac 13$ in a minute.\nWhen standing, or the speed of the sidewalk is $\\frac 12$ per minute.\nThus she can do $\\frac 13 + \\frac 12= \\frac 56$ per minute when walking on the sidewalk moving.\nIt then takes her $\\frac 65$ minutes = $1.2$[/hide]", "Solution_3": "[hide]$d=K*3 \\implies K=d/3$\n$d=S*2 \\implies S=d/2$\n\n$d=(K+S)*x$, solve for x.\n$d=(d/3+d/2)*x$\n$d=(5d)/6 *x$\n$x=(6d)/(5d)=6/5=1.2$ minutes[/hide]", "Solution_4": "I agree with those answers. \r\nA moving sidewalk is just an escalator but flat. \r\nI was on one at the airport. They move really slow.", "Solution_5": "While waiting for our flight from Detroit to Atlanta, I wanted to know how long it would take me to walk backwards on the moving sidewalk without actually doing it. So I used the same process being the genius penguin I am. Then I decided to check my answer by being an idiot. My teacher said that you would never need to do this kind of problems in real life.", "Solution_6": "they could at least say 'moving flat escalator' in the question, i'm like that would be pretty weird if you were standing on the street and the sidewalk started moving. :)", "Solution_7": "It would if you lived in LA. Of course everything would move with it.", "Solution_8": "[quote=\"ritchjp\"]It would if you lived in LA. Of course everything would move with it.[/quote]\r\n\r\nHaha that's where I live. :rotfl:" } { "Tag": [ "algebra", "polynomial", "AoPS Books", "abstract algebra" ], "Problem": "Hi,\r\n\r\nI've recently gotten into problem solving as a hobby. Its really fun (though I'm kinda late to the game, junior in high school :( ). Anyways, I encountered this problem and found the answer (C), but I kinda brute forced it. Does anyone know of a more, how should I put this, elegant way of obtaining a solution? \r\n\r\n10. Which of the following divides $ n^3 \\plus{} 11n$ for all positive integers $ n$?\r\nA. 5 B. 7 C. 6 D. 4 E. 9\r\n\r\n\r\nThanks!", "Solution_1": "I'm not sure what you define as brute-forcing because I just played around with mods and parity to arrive at a solution. First after factoring the expression I obtained $ n(n^2 \\plus{} 11)$. If n is even then the expression always divisible by 2 because the n on the outside is divisible by 2. If n is odd then $ n^2 \\plus{} 11$ is always divisible by 2. Then I considered the residues of $ n modulo 3$. By plugging in the possible residues of 0,1,and 2 modulo 3 into $ n$ and their respective squares into $ n^2$ I found by considering each of the three cases that the expression is always divisible 3, which gives C as an answer. I don't think there is any more elegant way than factoring and using modular arithmetic(which really didn't take me too long). But I could be wrong.", "Solution_2": "Plug in $ n \\equal{} 1$ to eliminate A, B, and E, then $ n \\equal{} 2$ to eliminate D. :D", "Solution_3": "[hide=\"Simple strategy\"] Test out a few values to see what the biggest number [i]could[/i] be. Once you think you've got it, try to prove it. Then you know it can't be a bigger number. [/hide]\n[hide=\"Complicated strategy\"] A polynomial of degree $ k$ has at most $ k$ roots $ \\bmod p$ for $ p$ prime. You might not know what this means, but essentially it means that since the expression you're given has degree $ 3$, the only primes involved in the answer are $ 2$ or $ 3$. [/hide]", "Solution_4": "[quote] Simple strategy\nTest out a few values to see what the biggest number could be. Once you think you've got it, try to prove it. Then you know it can't be a bigger number.\n\nComplicated strategy\nA polynomial of degree k has at most k roots \\bmod p for p prime. You might not know what this means, but essentially it means that since the expression you're given has degree 3, the only primes involved in the answer are 2 or 3. [/quote]\r\n\r\nThanks! \r\n\r\n@Brut3Forc3 and phi-unit\r\nBy brute forcing, I meant what Brut3Forc3 said (haha how ironic :rotfl: ). I was just wondering if there was some way to do it like t0rajir0u posted. I like to see the principles behind solutions, I know that plugging in numbers will give me the same answers, but this is just cooler :P . I don't really know how the complicated solution works but sounds interesting. See, I'm planning to get into problem solving as best as I can but as I said, I am late to the game. How much will the AoPS books help with problems like this? I've borrowed Vol. 1 and Vol. 2 from a friend who bought them but never used them. Do they cover stuff like this? If not, what books do?\r\n\r\nThanks again!", "Solution_5": "@t0rajir0u:\r\n\r\nI considered your strategy, it's pretty interesting. My interpretation for this is: we can always find a congruent polynomial of the original one $ \\bmod p$, suppose it's $ (n \\minus{} x_1)(n \\minus{} x_2)...(n \\minus{} x_k)$, which can only take at most $ k$ distinct roots $ \\bmod p$, i.e. be a multiple of $ p$. Therefore, to make it divisible by $ p$ for all $ n$, we must have $ p\\leq k$. Is this correct?", "Solution_6": "Essentially. The theorem is called [url=http://en.wikipedia.org/wiki/Lagrange%27s_theorem_%28number_theory%29]Lagrange's theorem[/url]." } { "Tag": [ "function", "LaTeX", "parameterization", "trigonometry", "calculus", "derivative", "calculus computations" ], "Problem": "Suppose a smooth curve in $ \\mathbb{R}^2$ passes through the origin. Let $ s$ denote arclength as measured along the curve, with $ s > 0$ to the right of the origin and $ s < 0$ to the left.\r\n\r\nFix a constant $ k > 0.$ Suppose that the curve satisfies the property that $ y \\equal{} ks^2.$\r\n\r\nCan you find a more conventional representation (such as parametric form) for this curve? And does the curve have some familiar name?\r\n\r\nAs a possible intermediate step: Show that there is a particular number (which depends on $ k$) such that the arclength of the entire curve cannot exceed this number. In other words, this is a curve of finite length and bounded extent.", "Solution_1": "The total length of the curve is 1/2k.\r\n\r\ny=ks^2 gives s as a function of y. Differentiating with respect to x, we get an equation in dy/dx \r\n\r\nThe equation is, \r\n\r\n{dy/dx}^2 = 1/{1/4ky - 1}. This puts a bound on y: y<1/4k. this means, s^2 < 1/4k^2. i.e., s<1/2k.\r\n\r\nthe above equation is solved by the substituting, y= 1/{t^2 + 4k}. \r\n\r\nThe final solution turns out to be,\r\n\r\n x= sqr.root{y/4k - y^2 } - (1/4k)tan^-1 {sqr.root(1/4ky - 1)} + pi/8k\r\n\r\nI am not able to get this in the form, y=f(x).", "Solution_2": "One small comment: we actually have $ \\minus{} \\frac1{2k}\\le s\\le\\frac1{2k},$ with the curve extending on both sides of the origin, so the total length of the curve is $ \\frac1k.$\r\n\r\nIt's possible to get a nicer looking form than that. Not $ y \\equal{} f(x)$ but rather a pair of parametric equations, $ x \\equal{} x(t),y \\equal{} y(t).$", "Solution_3": "Please learn a bit of [[LaTeX]]. (See also [url=http://www.artofproblemsolving.com/Forum/viewtopic.php?t=123690]here[/url].) \r\n\r\nWe see that taking $ y$ as a function of $ s$, $ y'(s) \\equal{} 2k s$, so $ |2ks| \\leq 1$. This suggests the parametrization $ s \\equal{} \\frac {\\sin t}{2k}$, so $ y \\equal{} \\frac {\\sin^2 t}{4k}$ and $ \\frac {dy}{dt} \\equal{} \\frac {\\sin t \\cos t}{2k}$. We have (taking all derivatives w.r.t. $ t$) that $ s'(t)^2 \\equal{} y'(t)^2 \\plus{} x'(t)^2$, so $ \\frac {\\cos^2 t}{4k^2} \\equal{} \\frac {\\sin^2 t \\cos^2 t}{4k^2} \\plus{} x'(t)^2$, and this gives $ x'(t)^2 \\equal{} \\frac {\\cos^4 t}{4k^2}$. Then (because I don't want to think about signs right now) we conclude from this that $ x'(t) \\equal{} \\frac {\\cos^2 t}{2k}$, and from here we can integrate to get the formula for $ x$. A few details to be patched up, but is this the sort of thing you were looking for?\r\n\r\n(Admission: I actually didn't think of the parametrization for $ s$ in the way I described in the second sentence, but rather because I started by parametrizing by arclength, which gives $ y'(s) \\equal{} 2k s$ and so $ x'(s) \\equal{} \\sqrt {1 \\minus{} 4k^2 s^2}$. This spits out an arcsine when you integrate, and [i]that's[/i] what got me to the substitution I used above.)", "Solution_4": "It is a pretty famous curve. It got quite a bit of attention throughout the 17th century.", "Solution_5": "substituting cot^2 t = 1/4ky - 1 in the solution mentioned in my previous post, the parametric equations turn out to be:\r\n x = (1-cos t)/8k; y = (t+ sin t)/8k", "Solution_6": "[quote=\"bharathhm\"] x = (1-cos t)/8k; y = (t+ sin t)/8k[/quote]\r\nNow, this is starting to look familiar. Does anyone recognize the curve from that description?", "Solution_7": "Cycloid. :lol:\r\n[url=http://images.google.co.jp/imgres?imgurl=http://www.scitechantiques.com/cycloidhtml/images/cycloid_versus_simple.jpg&imgrefurl=http://www.scitechantiques.com/cycloidhtml/&usg=__RTEEfWBr5aBU6aWbt8DoA6lccFI=&h=317&w=400&sz=43&hl=ja&start=10&sig2=y-0AsdEAAARDz6ZkCDje5A&um=1&tbnid=bkaJwLsjllHL9M:&tbnh=98&tbnw=124&prev=/images%3Fq%3DCycloid%26hl%3Dja%26rlz%3D1T4ADBR_jaJP295JP295%26sa%3DX%26um%3D1&ei=VQ3ESeHDONKAkQWCkOzXDA]Galileo's Pendlum.[/url]", "Solution_8": "Or, the [url=http://en.wikipedia.org/wiki/Tautochrone_curve]tautochrone[/url].", "Solution_9": "This is the cycloid oriented so its cusps point upward. \r\n\r\nAthough the cycloid has many uses and appears in many problems, expressing it in exactly the form I started with here emphasizes the tautochrone property. (Had jmerry not already posted that particular link, I was planning to do so myself.)\r\n\r\nWhat is the motion of a bead sliding on this wire without friction? Gravitational potential energy is proportional to $ y$ and $ y$ is in turn proportional to $ s^2.$ This is precisely the setup needed to have $ \\frac{d^2s}{dt^2}\\equal{}\\minus{}cs,$ which is the equation of simple harmonic motion. And in simple harmonic motion, frequency is independent of amplitude, which is simply another way to state the tautochrone property." } { "Tag": [], "Problem": "hi,\r\n\r\ni am currently doing some coursework of which requires me to need to find out what it means, can you help. the table below is what i have for information.\r\n\r\n[img]http://www.redihot.com/images/english_03.gif[/img]\r\n\r\nthanks alot, pleae click it to see it better.", "Solution_1": "thanks alot for not helping, although i found out, the 100 represents the average so the mirror is less meaning it is less popular than the average newspaper", "Solution_2": "Errr, if you wanted help, youd really have to give more information. What the hell are we supposed to be getting the mean of exactly?", "Solution_3": "just so you know, what you found is stated on the top left corner where it says \r\n\r\n\"index: 100=UK national average\"\r\n\r\nthat's about all i see that you're gonna get from the table, unless if there's some more information you have, i.e specifically what you're supposed to do." } { "Tag": [ "geometry", "inequalities", "geometry unsolved" ], "Problem": "How many triangles $ ABC$ are there if radius of the inscribed circle of the triangle $ ABC$ is $ 1$ and all sides are integer?", "Solution_1": "Let $ BC\\equal{}a,AB\\equal{}c,AC\\equal{}b$ where $ a\\leq{b}\\leq{c}$ are integers. If $ r\\equal{}1$ then $ a,b,c$ satisfy $ (b\\plus{}c\\minus{}a)(a\\plus{}b\\minus{}c)(a\\plus{}c\\minus{}b)\\equal{}4(a\\plus{}b\\plus{}c)$. \r\nMoreover $ a\\leq{12}$. \r\nProof: $ 4(a\\plus{}b\\plus{}c)\\leq{1}2c$ and $ (b\\plus{}c\\minus{}a)(a\\plus{}b\\minus{}c)(a\\plus{}c\\minus{}b)\\geq{ac}$. Thus $ ac\\leq{12c}$ and $ a\\leq{12}$.\r\nFor $ a\\leq{b}\\leq{c}\\leq{3000}$ there is only one solution: $ 3,4,5$.", "Solution_2": "Another note is that when this occurs(inradius as 1)\r\n\r\nK=s where s is the semiperimeter of the triangle", "Solution_3": "Moreover $ c\\leq{b}\\plus{}12$. \r\nProof: $ c\\leq{a}\\plus{}b\\leq{b}\\plus{}12$.\r\nFor $ a\\leq{b}\\leq{c}\\leq{1}0^5$ there is only one solution: $ 3,4,5$.", "Solution_4": "[quote=\"loup blanc\"]Let $ BC \\equal{} a,AB \\equal{} c,AC \\equal{} b$ where $ a\\leq{b}\\leq{c}$ are integers. If $ r \\equal{} 1$ then $ a,b,c$ satisfy $ (b \\plus{} c \\minus{} a)(a \\plus{} b \\minus{} c)(a \\plus{} c \\minus{} b) \\equal{} 4(a \\plus{} b \\plus{} c)$. \nMoreover $ a\\leq{12}$. \nProof: $ 4(a \\plus{} b \\plus{} c)\\leq{1}2c$ and $ (b \\plus{} c \\minus{} a)(a \\plus{} b \\minus{} c)(a \\plus{} c \\minus{} b)\\geq{ac}$. Thus $ ac\\leq{12c}$ and $ a\\leq{12}$.\nFor $ a\\leq{b}\\leq{c}\\leq{3000}$ there is only one solution: $ 3,4,5$.[/quote]\r\nJust let $ x \\equal{} b \\plus{} c \\minus{} a,y \\equal{} a \\plus{} b \\minus{} c,z \\equal{} a \\plus{} c \\minus{} b$ then $ xyz \\equal{} 4(x \\plus{} y \\plus{} z)$.\r\nWlog $ x \\le y \\le z$. If $ x \\ge 4$ then: $ 4(yz \\minus{} 1) \\le x(yz \\minus{} 1) \\equal{} 4(y \\plus{} z) \\iff yz \\minus{} 1 \\le y \\plus{} z \\iff (y \\minus{} 1)(z \\minus{} 1) \\le 2$ which is impossible. So $ x \\in \\{1,2,3\\}$.\r\nSince $ a \\plus{} b \\minus{} c \\equiv a \\minus{} b \\plus{} c \\equiv \\minus{} a \\plus{} b \\plus{} c \\bmod 2$ we see $ x \\equiv y \\equiv z \\bmod 2$. Since $ 4 \\mid xyz$ we see $ 2 \\mid x,y,z$ and therefore $ x \\equal{} 2$. Inserting:\r\n$ 2yz \\equal{} 4(y \\plus{} z \\plus{} 2) \\iff yz \\equal{} 2(y \\plus{} z \\plus{} 2) \\iff (y \\minus{} 2)(z \\minus{} 2) \\equal{} 8$, and therefore $ y \\equal{} 4,z \\equal{} 6$, which in turn gives $ a \\equal{} 5,c \\equal{} 4,b \\equal{} 3$. So the only triangle is the $ 3,4,5$ triangle.", "Solution_5": "Hi Mathias,\r\nyou have not $ x(yz\\minus{}1)\\equal{}4(y\\plus{}z)$.", "Solution_6": "[quote=\"loup blanc\"]Hi Mathias,\nyou have not $ x(yz \\minus{} 1) \\equal{} 4(y \\plus{} z)$.[/quote]\r\nI'm sorry for my silly mistake! It should be $ x(yz\\minus{}4) \\equal{} 4(y\\plus{}z)$. The inequality then becomes $ (y\\minus{}1)(z\\minus{}1) \\le 5$ which is impossible too :)", "Solution_7": "Pretty proof Mathias.", "Solution_8": "[quote=\"Mathias_DK\"][quote=\"loup blanc\"]Hi Mathias,\nyou have not $ x(yz \\minus{} 1) \\equal{} 4(y \\plus{} z)$.[/quote]\nI'm sorry for my silly mistake! It should be $ x(yz \\minus{} 4) \\equal{} 4(y \\plus{} z)$. The inequality then becomes $ (y \\minus{} 1)(z \\minus{} 1) \\le 5$ which is impossible too :)[/quote]\n[quote]Couldnt understand this. $ 4(yz \\minus{} 1) \\le x(yz \\minus{} 4)$[/quote]", "Solution_9": "Hi ertanrock, reread posts #6,#7.\r\n$ 4(yz\\minus{}4)\\leq{x}(yz\\minus{}4)\\equal{}4(y\\plus{}z)$ implies $ (y\\minus{}1)(z\\minus{}1)\\leq{5}$ that is impossible because $ 4\\leq{y}\\leq{z}$.", "Solution_10": "[quote=\"loup blanc\"]Hi ertanrock, reread posts #6,#7.\n$ 4(yz \\minus{} 4)\\leq{x}(yz \\minus{} 4) \\equal{} 4(y \\plus{} z)$ implies $ (y \\minus{} 1)(z \\minus{} 1)\\leq{5}$ that is impossible because $ 4\\leq{y}\\leq{z}$.[/quote]\r\n\r\nok now. Got it.\r\nI thought like that. $ 4(yz \\minus{} 1)\\leq{x}(yz \\minus{} 4)$" } { "Tag": [], "Problem": "Heron triangles are triangles with integer sides and integer area.They are Primitive if their sides taken together have no common factors(not necessarily pairwise) \r\n1) Prove there are infinitely many primitive Heron Triangles two of whose sides have a common factor of 5.\r\n(example:(9,65,70),)\r\n2)determine a primitive heron triangle two of whose sides have a common factor of 65\r\n3)prove or disprove the existence of a primitive heron triangle each of whose sides is a square? :D", "Solution_1": "any one has a solution?", "Solution_2": "[quote=\"anorithdialga\"]any one has a solution?[/quote]\r\n\r\nyes, anyone has a solution", "Solution_3": "coincidentally, i m reading the same only,ill respond in 4 long days!!\r\nedit:wastage of post i think", "Solution_4": "Ok this is the first one:\r\n[hide=\"2\"]Every right triangle is a heron triangle, so their sum or difference should be also a heron triangle, thus forming acute and obtuse heron triangles. Here, we have to get two right triangles with sides multiple of 65. A right triangle with hypotenuse 65 gives options as (63,16,65) and (33,56,65). Choosing any one and creating an isosceles triangle with it does the job.[/hide]" } { "Tag": [], "Problem": "$ \\frac{a}{b}\\equal{}\\sqrt{2}\\plus{}1$. Then the following expression value is ?\r\n\r\n$ \\frac{a^{3}\\plus{}b^{3}}{a^{3}\\minus{}a^{2}.b\\plus{}a.b^{2}}$", "Solution_1": "$ a^3 \\plus{} b^3 \\equal{} (a \\plus{} b)(a^2 \\minus{} ab \\plus{} b^2)$\r\n\r\n$ a^3 \\minus{} a^2b \\plus{} ab^2 \\equal{} a^3 \\minus{} ab(a \\minus{} b) \\equal{} a\\left(a^2 \\minus{} ab \\plus{} b^2\\right)$\r\n\r\n$ \\frac {a^{3} \\plus{} b^{3}}{a^{3} \\minus{} a^{2}.b \\plus{} a.b^{2}} \\equal{} \\frac {(a \\plus{} b)(a^2 \\minus{} ab \\plus{} b^2)}{a\\left(a^2 \\minus{} ab \\plus{} b^2\\right)}$\r\n\r\n$ \\equal{} \\frac {a \\plus{} b}{a}$\r\n\r\n$ \\equal{} b\\left(\\frac {\\frac {a}{b} \\plus{} 1}{a}\\right)$\r\n\r\n$ \\equal{} \\left(\\frac {\\frac {a}{b} \\plus{} 1}{\\frac {a}{b}}\\right)$\r\n\r\n$ \\equal{} \\frac {\\sqrt {2} \\plus{} 2}{\\sqrt {2} \\plus{} 1}$", "Solution_2": "You can simplify the result.", "Solution_3": "$ \\frac{\\sqrt{2}\\plus{}2}{\\sqrt{2}\\plus{}1} \\equal{}\\frac{(\\sqrt{2}\\plus{}2)(\\sqrt{2}\\minus{}1)}{(\\sqrt{2}\\plus{}1)(\\sqrt{2}\\minus{}1)}$\r\n\r\n$ \\equal{}2\\minus{}\\sqrt{2}\\plus{}2\\sqrt{2}\\minus{}2$\r\n\r\n$ \\equal{}\\sqrt{2}$", "Solution_4": "O.K. :lol:", "Solution_5": "Thanks, but both of the answers are correct ?", "Solution_6": "[quote=\"luiseduardo\"]$ \\frac {a}{b} \\equal{} \\sqrt {2} \\plus{} 1$. Then the following expression value is ?\n\n$ \\frac {a^{3} \\plus{} b^{3}}{a^{3} \\minus{} a^{2}.b \\plus{} a.b^{2}}$[/quote]\r\n\r\nHere is my solution:\r\n\r\n$ \\frac {a^{3} \\plus{} b^{3}}{a^{3} \\minus{} a^{2}.b \\plus{} a.b^{2}}\\equal{}\\frac{(a\\plus{}b)(a^2\\minus{}ab\\plus{}b^2)}{a(a^2\\minus{}ab\\plus{}b^2)}\\equal{}1\\plus{}\\frac{b}{a}$\r\n\r\n$ \\equal{}1\\plus{}\\frac{1}{\\sqrt{2}\\plus{}1}\\equal{}1\\plus{}\\sqrt{2}\\minus{}1\\equal{}\\boxed{\\sqrt{2}}$.", "Solution_7": "Ah ok :blush: \r\nThanks.", "Solution_8": ":)" } { "Tag": [ "geometry", "perimeter", "geometric transformation", "dilation", "trigonometry", "number theory", "relatively prime" ], "Problem": "The perimeter of triangle $APM$ is $152,$ and the angle $PAM$ is a right angle. A circle of radius $19$ with center $O$ on $\\overline{AP}$ is drawn so that it is tangent to $\\overline{AM}$ and $\\overline{PM}.$ Given that $OP=m/n,$ where $m$ and $n$ are relatively prime positive integers, find $m+n.$", "Solution_1": "is the answer:\r\n\r\n[hide]\n\n$\\frac{m}{n}= \\frac{7}{8}\\implies m+n = 15$?\n\nwhere is this from?\n\n[/hide]", "Solution_2": "this problem was more: \"can you simplify algebra\" rather than any cool geometry...\r\n\r\nanyway, denote $OP=x$, $PT=y$, dialate by 1/19, denote the circle tangent to $PM$ at $T$, so $OT=OA=1$, and $p=8$\r\n\r\n$\\triangle OTP\\sim\\triangle MAP$, so\r\n\r\n$\\frac{1}{MA}=\\frac{x}{MP}=\\frac{y}{x+1}=\\frac{1+x+y}{8}$\r\n\r\nso $(7-x)y=(x+1)^{2}$, and $1+y^{2}=x^{2}$, so $(x-7)^{2}[(x-1)(x+1)]=(x+1)^{4}$, since this is easily expanded, and solved, we obtain $x=\\frac{5}{3}$, then undo the dialation, so $OP=\\frac{95}{3}\\implies m+n=\\boxed{098}$", "Solution_3": "[quote=\"Altheman\"]this problem was more: \"can you simplify algebra\" rather than any cool geometry...\n[/quote]\r\n[color=red]A cool problem [/color]:D\r\n [hide]\nLet restate the problem. Will just change the letters and will dilate by 19 (see Altheman solution). Triangle $ABC$ is right ($A=90$) with perimeter of 8. Circle with center $D$ on $AC$ touches $AB$ and $BC$ and has radius 1. Find $DC$.\n Construct $GF$ parallel to $AC$ and tangent to the circle. \nLet $AB=x$. The perimeter of $\\Delta BFG=BF+FE+EG+BG=BF+FH+GA+BG=2x$ as $\\Delta BFG \\sim \\Delta BAC$ we have $\\frac{BG}{BA}=\\frac{2x}{8}$ \n$\\frac{x-1}{x}=\\frac{2x}{8}\\Rightarrow x^{2}-4x+4=0$ or $x=AB=2$.\n$\\tan{DAB}=\\frac{DA}{BA}=\\frac12$. As $CD$ is angle bisector $\\tan{ABC}= \\frac43$ (by double angle formula).\nSo $\\frac43=\\frac{AC}{AB}$, $AC=\\frac83$, $DC=\\frac53$ [/hide]", "Solution_4": "excuse me, but what does the p mean in your solution?\r\ndenote the circle tangent to PM at T, so OT=OA=1, and p=8", "Solution_5": "In Altheman's solution, $ p$ represents the [i]perimeter[/i] of $ \\triangle APM$.", "Solution_6": "[hide=Solution]We see that the circle can only be tangent at $A.$ Suppose it it tangent at $X$ instead. Then $OX \\perp AM,$ but $OA\\perp AM.$ Thus, $OA \\parallel OX.$ But they meet at $O,$ so this is a contradiction. Therefore, circle $O$ is tangent to $AM$ at $A,$ and $OA=19.$\n\nCall the point where $O$ meets $PM,$ $Y.$ Note that $OY=19.$ Then, let $x=AM=MY$ ($AM=MY$ because they are tangents from $M$), and $OP=a.$ By Pythagoras, $PY = \\sqrt{a^2-19^2}.$\n\n$\\triangle POY \\sim \\triangle PMA$ by AA similarity because they share $\\angle APM$ and $\\angle PAM = \\angle PYO = 90.$ Hence, $AP/AM=PY/OY,$ or $\\frac{a+19}{x}=\\frac{\\sqrt{a^2-19^2}}{19}.$ Solving for $x,$ we have $x=\\frac{19a+19^2}{\\sqrt{a^2-19^2}}.$\n\nWe know that the perimeter is $152,$ which means that $a+19+2x+\\sqrt{a^2-19^2}=152.$ Substituting our value for $x,$$$a+19+\\frac{38a+2\\cdot19^2}{\\sqrt{a^2-19^2}}+\\sqrt{a^2-19^2}=152.$$Multiplying by $\\sqrt{a^2-19^2},$$$(a+19)\\sqrt{a^2-19^2}+38a+2\\cdot 19^2+a^2-19^2=152\\sqrt{a^2-19^2}.$$Moving the square roots to the LHS and everything else to the LHS and factoring,$$(a+19-152)\\sqrt{a^2-19^2}=-(a+19)^2.$$Squaring and using difference of squares,$$(a-7 \\cdot 19)^2 (a+19)(a-19)=(a+19)(a+19)^3.$$Canceling and expanding (and moving to one side),$$9a^2-570a+19^2\\cdot 50 =0.$$Using the quadratic formula, the only solution is $\\frac{95}{3}\\implies \\boxed{098}.$ [/hide]", "Solution_7": "[hide=Solution]Scale down by $19$; then the radius is $1$ and the perimeter is $8$. Let $T$ be the tangency point between the circle and $\\overline{PM}$. Let $x=OP$. Since $\\overline{OM}$ bisects $\\angle PMA$, we can use the angle-bisector theorem to get $AM=k$ and $PM=kx$. Additionally, $TM=AM=k$.\n\nBecause the perimeter is $8$, we have $kx+k+x+1=8 \\implies k(x+1)=7-x \\implies k = \\frac{7-x}{x+1}$.\n\nSince $PT=kx-k$, we use the Pythagorean Theorem on $\\triangle PTO$ to get:\n$$(kx-k)^2+1=x^2$$\n$$k^2(x-1)^2 = x^2-1$$\n$$k^2(x-1) = x+1$$\n$$\\left(\\frac{7-x}{x+1}\\right)^2(x-1)=x+1$$\n$$(x-7)^2(x-1)=(x+1)^3$$\n$$x^3-15x^2+63x-49=x^3+3x^2+3x+1$$\n$$18x^2-60x+50=0$$\n$$9x^2-30x+25=0$$\n$$(3x-5)^2=0$$\n$$x=\\frac53$$\nScaling up, we get the answer $\\frac{95}{3}\\implies\\boxed{098}$.", "Solution_8": "Scale down by $19$, and reflect $M$ across $AP$ to $M'$ so that said circle is the incircle of $\\triangle MM'P$. Let $(p,m):=(PA,MA)$.\n[list=1]\n[*] The perimeter condition becomes $p+m+\\sqrt{p^2+m^2}=8$. [/*]\n[*] The radius condition becomes, by $A=rs$, \n\\[pm=1(AM+PM)=\\text{Perimeter}(\\triangle APM)-AP=8-p.\\] [/*]\n[/list]\nEquation (1) becomes\n\\begin{align*}\np^2+m^2&=(8-p-m)^2\\\\\n&=(p+m)^2-16(p+m)+64\\\\\n\\implies32&=8(p+m)-pm.\n\\end{align*}\nSubstituting $pm=8-p$ and $m=\\tfrac{8-p}{p}$ from Equation (2),\n\\begin{align*}\n8(p+\\tfrac{8-p}{p})-(8-p)&=32\\\\\n8(p^2+(8-p))-p(8-p)&=32(8-p)\\\\\n9p^2-48p+64&=0.\n\\end{align*}\nSo $p=\\tfrac{8}{3}$ or $OP=\\tfrac{5}{3}$, giving $\\tfrac{95}{3}$." } { "Tag": [], "Problem": "In a room, each 22 liters of air weighs 29 grams. Each liter is 100 cubic centimeters. How much does a cubic meter of air in that room weigh? Express your answer to the nearest whole gram.", "Solution_1": "[quote=\"mr. math\"]In a room, each 22 liters of air weighs 29 grams. Each liter is 100 cubic centimeters. How much does a cubic meter of air in that room weigh? Express your answer to the nearest whole gram.[/quote]\r\nEach liter by the way, it 1000 cubic centimeters since 1 cubic centimeter is 1 milliliter. \r\n[hide] $1 m^{3}=(100)^{3}cm^{3}=1000000 cm^{3}$. We also know that 1 Liter=1000 cc (cc is cubic centimeters), so 1 cubic meter is (1000)(1000) cc=1000 liters. Thus, since each liter weighs 29 grams, so 1000 liters weighs 29 kilograms, which is our answer. [/hide]" } { "Tag": [ "trigonometry", "AMC", "AIME" ], "Problem": "Given that $z$ is a complex number such that $z+\\frac 1z=2\\cos 3^\\circ,$ find the least integer that is greater than $z^{2000}+\\frac 1{z^{2000}}.$", "Solution_1": "[hide]\n\n\n\nLet , then , so , so , so then the given equation evaluates to , and the answer is .\n\n\n\n[/hide]", "Solution_2": "I got that exact same answer, but I didn't know AIME answers can be 0, or can they? :)", "Solution_3": "[quote=\"1234567890\"]I got that exact same answer, but I didn't know AIME answers can be 0, or can they? :)[/quote]\r\n\r\nYeah, they can. It's 000-999. 1000 choice MC test? :P", "Solution_4": "[hide=\"Answer\"]We have $z=e^{i\\theta}$, so $e^{i\\theta}+\\frac{1}{e^{i\\theta}}=\\frac{\\cos \\theta}{\\cos ^2\\theta +\\sin ^2\\theta}+\\cos \\theta +i\\sin \\theta - \\frac{i\\sin \\theta}{\\cos ^2\\theta +\\sin ^2\\theta}=2\\cos \\theta$. Therefore, $\\theta =3^\\circ=\\frac{\\pi}{60}$. From there, $z^{2000}+\\frac{1}{z^{2000}}=e^{\\frac{100\\pi (i)}{3}}+e^{\\frac{-100\\pi (i)}{3}}=2\\cos \\frac{4\\pi}{3}=-1$, so our answer is $\\boxed{0}$.[/hide]", "Solution_5": "the question ask for \"the least integer that is GREATER than......\", not \"not less than\", so -1 can greater than -1, the answer is 0", "Solution_6": "The question is amazingly easy (like 10 sec) if you know that the answer can ONLY be -1, 0 or 1 (AIME only 000 or 001), no matter what inteter power z is taken to and no matter the angle degree (in this situation its 0)...\r\n\r\nthere is a generalized formula in AoPS vol 2 for these types of problems", "Solution_7": "[quote=Elemennop][hide]\n\n\n\nLet , then , so , so , so then the given equation evaluates to , and the answer is .\n\n\n\n[/hide][/quote]\nhuh?\n", "Solution_8": "w0W that revive.\n\nProbably displaying issues from so old. That's why it looks so clipped and stutter-y.", "Solution_9": "[quote=JesusFreak197][hide=\"Answer\"]We have $z=e^{i\\theta}$, so $e^{i\\theta}+\\frac{1}{e^{i\\theta}}=\\frac{\\cos \\theta}{\\cos ^2\\theta +\\sin ^2\\theta}+\\cos \\theta +i\\sin \\theta - \\frac{i\\sin \\theta}{\\cos ^2\\theta +\\sin ^2\\theta}=2\\cos \\theta$. Therefore, $\\theta =3^\\circ=\\frac{\\pi}{60}$. From there, $z^{2000}+\\frac{1}{z^{2000}}=e^{\\frac{100\\pi (i)}{3}}+e^{\\frac{-100\\pi (i)}{3}}=2\\cos \\frac{4\\pi}{3}=-1$, so our answer is $\\boxed{0}$.[/hide][/quote]\n\n \nIsn't Euler identity excluded from most content sets?", "Solution_10": "I just realized I bumped a 9 year old topic oops", "Solution_11": "I think this is much nicer to look at.\n\n[hide]\nLet $z=r(\\cos\\theta+i\\sin\\theta).$ The crucial claim is that $r=1$ (and the rest of the problem follows pretty easily from here).\n\nProof: Notice that $z+\\frac{1}{z}=r(\\cos\\theta+i\\sin\\theta)+\\frac{1}{r}(\\cos\\theta+i\\sin\\theta)=\\cos\\theta(r+\\frac{1}{r})+i\\sin\\theta(r-\\frac{1}{r}).$ Since $2\\cos 3^{\\circ}$ has no imaginary part, $r-\\frac{1}{r}=0\\to r=\\frac{1}{r}\\to r^2=1\\to r=1$ (as $r$ is a positive number).\n\nNow the rest of the problem follows.\n\nPlugging $r=1$ into the equation we see that $2\\cos\\theta=2\\cos 3^{\\circ},$ so $z=cis 3^{\\circ}.$\n\nThus $z^{2000}+z^{-2000}=cis(3\\cdot 2000)+cis(3\\cdot -2000)=-1,$ so the answer is $0.$\n[/hide]", "Solution_12": "Whaaat I didn\u2019t know $0$ could be an answer!\n\n$z + \\frac{1}{z} = 2 \\cos 3^\\circ \\implies z^2 - (2 \\cos 3 )z + 1 = 0.$ So, $z = \\frac{2\\cos 3 \\pm \\sqrt{4\\cos^2 3 - 4}}{2} = \\cos 3 + i\\sin 3.$ By De Moivre's Theorem, $z^{2000} = \\cos 6000^\\circ + i\\sin 6000^\\circ = -\\frac{1}{2} - \\frac{\\sqrt{3}}{2}i$ Thus, $z^{2000}+\\frac{1}{z^{2000}} = 2\\cos 240^\\circ = -1.$ Therefore the answer is the least integer greater than $-1,$ which is $\\boxed{0}.$", "Solution_13": "Let's denote $z = e^{i \\theta}$. Then we have $e^{i\\theta} + e^{-i\\theta} = 2 \\cos 3^\\circ$, and want to find $e^{i2000\\theta} + e^{-i2000\\theta}$. From the first equation, we have $\\cos \\theta + i \\sin \\theta + \\cos (-\\theta) + i \\sin (-\\theta) = 2 \\cos \\theta = 2 \\cos 3^\\circ$, so $\\cos \\theta = \\cos 3^\\circ \\implies \\theta = 3^\\circ$. We want to find $\\cos 2000\\theta + i \\sin 2000\\theta + \\cos (-2000\\theta) + i \\sin (-2000\\theta) = 2\\cos(2000\\theta) = 2 \\cos(6000^\\circ)$. This evaluates to $-1$, so the answer is $\\boxed{0}$." } { "Tag": [], "Problem": "What exactly is a convex hull ?? :?", "Solution_1": "http://en.wikipedia.org/wiki/Convex_hull\r\nhttp://mathworld.wolfram.com/ConvexHull.html\r\n\r\n:)", "Solution_2": "Thanks..... :) ......Just by the way , are there any proofs for that expression on mathworld ?? :?" } { "Tag": [ "topology" ], "Problem": "Suppose that metric space (X,d) is connected. Prove that for every two points a,b $ \\in$ X, and for every positive number $ \\delta$ exists finite sequence of points $ c_1, ... , c_n \\in$ X which satisfy conditions: $ a \\equal{} c_1, b \\equal{} c_n$, and for every k $ \\in$ {1,...,n}: $ d(c_k,c_{k \\plus{} 1})\\le \\delta$", "Solution_1": "Let $ S(a)$ denote the set of points $ b$ reachable in this way. Let $ T(a)$ denote the set of points not reachable. \r\n\r\nThen $ S(a)$ is open, since each $ b \\in S(a)$ has a $ \\delta$ neighborhood that is also reachable. $ T(a)$ is also open. For if $ d \\in T(a)$ is not reachable \r\nin finitely many $ \\delta$-steps, then nothing in a $ \\delta$-neighborhood of $ d$ can be reachable either. Otherwise we may extend a path to such a point to $ d$. \r\n\r\nThen $ X \\equal{} S(a) \\cup T(a)$, while $ S(a) \\cap T(a) \\equal{} \\emptyset$ and each set $ S(a), T(a)$ is open. Therefore either $ S(a) \\equal{} \\emptyset$ or $ T(a) \\equal{} \\emptyset$. Since $ a \\in S(a)$, we conclude $ T(a) \\equal{} \\emptyset$ and so $ S(a) \\equal{} X$." } { "Tag": [ "Ring Theory", "MATHCOUNTS" ], "Problem": "when is nats ths year?", "Solution_1": "I think the date is between May 6 and May 9!! :P", "Solution_2": "sometime in may :P", "Solution_3": "May 7th to May 10th, as found [url=http://mathcounts.org/Page.aspx?pid=294]here[/url].\r\n\r\nIf you require some extra ...uh... \"motivation,\" look at the [url=http://www.swandolphin.com/home.html]hotel[/url]. Don't spend all your time looking at it though; go practice.", "Solution_4": "[quote=\"xpmath\"]If you require some extra ...uh... \"motivation,\" look at the [url=http://www.swandolphin.com/home.html]hotel[/url]. [/quote]\r\n\r\nWhere did you read that? I didn't see that on the mathcounts website... I guess I am still in awe.", "Solution_5": "At [url=http://mathcounts.org/Page.aspx?pid=1336]this page[/url], it's stated that \"The 2009 Raytheon MATHCOUNTS National Competition will be held at Walt Disney World's Swan and Dolphin Resort in Orlando, Florida!\"\r\n\r\nSearching \"Swan and Dolphin Resort\" on Google gives the website." } { "Tag": [ "number theory solved", "number theory" ], "Problem": "Given integers a>b>0 let\r\n\r\nx= \\sqrt a+ \\sqrt b y= \\sqrt a- \\sqrt b\r\n\r\nIf a-b is twice an odd integer prove that both x and y are irrational.", "Solution_1": "Sorry for my typing ; naturally x and y are distinct numbers\r\n\r\nx= \\sqrt a+ \\sqrt b\r\n\r\ny= \\sqrt a- \\sqrt b", "Solution_2": "Suppose x is rational. Then xy=a-b is rational, so y is rational.\r\nThen 2 \\sqrt a=x+y is rational, so \\sqrt a si rational.\r\nSimilarly \\sqrt b is rational.\r\nSince a,b are integers and \\sqrt a, \\sqrt b are rational, it follows that a and b are squares.\r\nLet a=A 2 , b= B2.\r\nSo A 2 -B 2 =4k+2, which is impossible mod 4 (2 divides A-B iff 2 divides A+B)." } { "Tag": [ "number theory open", "number theory" ], "Problem": "For sufficiently large $A, n$, can it be said that $A^n + 1$ is always squarefree?", "Solution_1": "Here is a topic concerning a closely related problem.\r\n[url=http://www.mathlinks.ro/Forum/viewtopic.php?t=30001]http://www.mathlinks.ro/Forum/viewtopic.php?t=30001[/url]", "Solution_2": "Suppose that for all sufficently large $A$, and $n$, $A^n + 1$ is squarefree.\r\n\r\nSince for all $A,n$ [b]sufficently large[/b] $A^n + 1$ is squarefree, we can take $A$ to be even and not divisible by $3$ (we just take an $A$ being a bit larger ;)). Hence $A^n + 1 \\equiv 1 ( \\mbox{mod } 2 )$ for all $n$. Take $n = 2 k$, for some positive integer $k$. Then $A^n + 1 \\equiv A^{2 k} + 1 \\equiv 2 ( \\mbox{mod } 3 )$. This means that we can take sufficently large $A, n$ such that $2 \\nmid A^n + 1$ and $3 \\nmid A^n + 1$ and $A^n + 1$ squarefree. Hence $A^n + 1 = \\prod_{j = 1}^m p_j$, with $p_j \\neq 2, 3$ for all $j = 1, \\ldots, m$. Hence $A^n + 1 \\equiv \\pm 1 (\\mbox{mod } 6 )$, but $A^n + 1 \\equiv A^{2 k} + 1 \\equiv 2 ( \\mbox{mod } 6 )$. Contradiction.\r\n\r\nIf $A$ is given, then remark that if there is some positive integer $k_0$ such that $A^{k_0} + 1 = a \\cdot b^2$ is not squarefree and $A^n + 1$ is squarefree for all sufficently large $n$ then $A^{k_0 \\cdot n} + 1$ is also squarefree for a sufficently large $n$. But $( A^{k_0 \\cdot n} + 1 ; A^{k_0} + 1 ) = A^{k_0} + 1$. Hence $a \\cdot b^2 = A^{k_0} + 1| A^{k_0 \\cdot n} + 1$ and $A^{k_0 \\cdot n} + 1$ is not squarefree. Contradiction.\r\n\r\nAlso I doubt there is some $A$ s.t for all $n$, $A^n + 1$ is squarefree. Hence that would imply that there is no integer $A$ s.t for all sufficently large $n$, $A^n + 1$ is squarefree.\r\n\r\nI hope it is clear.", "Solution_3": "Just take any $A \\equiv -1 \\mod 4$ and any odd $n$ and you get big numbers such that $4|A^n+1$..." } { "Tag": [ "geometry", "ratio" ], "Problem": "Prove that the perpendicular drawn from the point $ (4,1)$ on the join of $ (2,\\minus{}1)$ and $ (6,5)$ divides it in the ratio of $ 5: 13$ . I got its answer $ 5: 8$. I think there is some problem in calculation.So please show your solution with calculation. Thanks a lot.", "Solution_1": "hello, let $ A(2;\\minus{}1)$, $ B(6;5)$ and $ C(4;1)$ then the line say $ g$ through $ A,B$ is $ y\\equal{}3/2x\\minus{}4$, the line which is perpendicular to $ g$ is $ h$: $ y\\equal{}\\minus{}2/3x\\plus{}11/3$ hence the intersection point $ D(46/13;17/13)$ so we get\r\n$ \\overline{AD}\\equal{}10/13\\sqrt{13}$\r\n$ \\overline{DB}\\equal{}16/13\\sqrt{13}$\r\nand we get\r\n$ \\frac{\\overline{AD}}{\\overline{DB}}\\equal{}5/8$.\r\nSonnhard.", "Solution_2": "Thanks. I got the same.It means that it is written wrong in the book.Thanks again. :-)" } { "Tag": [ "inequalities", "inequalities unsolved" ], "Problem": "Let $ a,b,c > 0$, prove that $ \\sum_{cyc}\\frac {a^2}b\\ge3\\sqrt [6]{\\frac{a^6 \\plus{} b^6 \\plus{} c^6}3}$", "Solution_1": "[quote=\"akai\"]Let $ a,b,c > 0$, prove that $ \\sum_{cyc}\\frac {a^2}b\\ge3\\sqrt [6]{\\frac {a^6 \\plus{} b^6 \\plus{} c^6}3}$[/quote]\r\n\r\nPlease help me, Can_hang2007, arqady, dduclam, ... or nother one ??? Thanks !", "Solution_2": "[quote=\"akai\"]Let $ a,b,c > 0$, prove that $ \\sum_{cyc}\\frac {a^2}b\\ge3\\sqrt [6]{\\frac {a^6 \\plus{} b^6 \\plus{} c^6}3}$[/quote]\r\nSee my hint here: http://www.mathlinks.ro/viewtopic.php?t=184602 :)", "Solution_3": "We have a more weak inequality be\r\n$ \\sum_{cyc}\\frac {a^2}b\\ge\\sum\\sqrt [6]{\\frac {a^6 \\plus{} b^6 }2}$" } { "Tag": [ "geometry", "circumcircle", "trigonometry", "geometric transformation", "homothety", "rotation" ], "Problem": "Let $\\omega$ be the nine-point circle of a triangle $ABC$. Prove that there exist precisely three points $P$, $Q$ and $R$ on the circumcircle of triangle $ABC$ whose Simson lines with respect to the triangle $ABC$ are tangent to $\\omega$. Prove also that the triangle $PQR$ formed by these three points is homothetic to the Morley triangle of the triangle $ABC$ (and the triangle with points of touching as the vertices is also homothetic to the Morley triangle).", "Solution_1": "WLOG, assume that the relative size of the angles $\\alpha = \\angle CAB$, $\\beta = \\angle ABC$, $\\gamma = \\angle BCA$ is $\\beta < \\alpha < \\gamma$ (that's how I drew the triangle). Denote the Morley equilateral triangle $\\triangle DEF$, where D is the intersection of the appropriate trisectors of the angles $\\beta$,$\\gamma$, E the intersection of the appropriate trisectors of the angles $\\gamma$, $\\alpha$ and F the intersection of the appropriate trisectors of the angles $\\alpha$, $\\beta$. The angles formed by 2 trisectors from the same vertex of the $\\triangle ABC$ with the sides of the equilateral triangle $\\triangle DEF$ are:\r\n\r\n$\\angle CDE = \\frac{\\beta + \\pi}{3}$, $\\angle CED = \\frac{\\alpha + \\pi}{3}$\r\n\r\n$\\angle AEF = \\frac{\\gamma + \\pi}{3}$, $\\angle AFE = \\frac{\\beta + \\pi}{3}$\r\n\r\n$\\angle BFD = \\frac{\\alpha + \\pi}{3}$, $\\angle BDF = \\frac{\\gamma + \\pi}{3}$\r\n\r\nAn easy proof is to draw an arbitrary equilateral triangle $\\triangle DEF$, take the angles $\\alpha + \\beta + \\gamma = \\pi$, build the above angles on the sides of this equilateral triangle and show, for example, that the angles $\\angle FAB = \\frac{\\alpha}{3}$, $\\angle FBA = \\frac{\\beta}{3}$. Using sine theorem for the triangles $\\triangle AEF$, $\\triangle BDE$,\r\n\r\n$\\frac{EF}{AF} = \\frac {\\sin \\left ( \\frac{\\alpha}{3} \\right )}{\\sin \\left ( \\frac{\\gamma + \\pi}{3} \\right )}$, $\\frac{DE}{BF} = \\frac {\\sin \\left ( \\frac{\\beta}{3} \\right )}{\\sin \\left ( \\frac{\\gamma + \\pi}{3} \\right )}$, $\\frac{AF}{BF} = \\frac {\\sin \\left ( \\frac{\\beta}{3} \\right )}{\\sin \\left ( \\frac{\\alpha}{3} \\right )}$\r\n\r\nSince in addition, $\\angle AFB = 2\\pi - \\frac{\\pi}{3} - \\frac{\\alpha + \\pi}{3} - \\frac{\\beta + \\pi}{3} = \\pi - \\frac{\\alpha + \\beta}{3}$, from the sine theorem for the triangle $\\triangle ABF$, it follows that $\\angle FAB = \\frac{\\alpha}{3}$, $\\angle FBA = \\frac{\\beta}{3}$.\r\n\r\nWe are going to \"construct\" the Simson line perpendicular to the side EF of the Morley triangle $\\triangle DEF$. Simple angle chase reveals the this direction forms with the side AB the angle\r\n\r\n$\\pi - \\frac{\\beta + \\pi}{3} + \\frac{\\alpha}{3} - \\frac{\\pi}{2} = \\frac{\\pi}{6} + \\frac{\\alpha - \\beta}{3}$\r\n\r\nLet a line through the vertex A and forming this angle with the side AB intersect the circumcircle $(O)$ of the triangle $\\triangle ABC$ at a point K and let a normal from the point K to the side BC intersect the circumcircle at another point L. The point L is the pole of the desired Simson line. The line KL perpendicular to the side BC forms the angle $\\frac{\\pi}{2} - \\beta$ with the side AB and consequently, the angle $\\angle AKL$ is equal to\r\n\r\n$\\angle AKL = \\frac{\\pi}{2} - \\beta - \\left ( \\frac{\\pi}{6} + \\frac{\\alpha - \\beta}{3} \\right ) = \\frac{\\pi - (2\\beta + \\alpha)}{3} = \\frac{\\gamma - \\beta}{3}$\r\n\r\nContinuing the angle chase around the circumcircle, the angle $\\angle AKB = \\angle ACB = \\gamma$ (spans the same circumcircle arc AB) and\r\n\r\n$\\angle KBC = \\frac{\\pi}{2} - (\\gamma - \\angle AKL) = \\frac{\\pi}{2} - \\left ( \\gamma - \\frac{\\gamma - \\beta}{3} \\right ) = \\frac{\\pi}{2} - \\frac{2\\gamma + \\beta}{3}$\r\n\r\n$\\angle KBA = \\beta + \\angle KBC = \\beta + \\frac{\\pi}{2} - \\frac{2\\gamma + \\beta}{3} = \\frac{\\pi}{2} - \\frac{2(\\gamma - \\beta)}{3}$\r\n\r\nThe angles $\\angle KLA = \\angle KBA$ are equal, because they span the same circumcircle arc KA, and the angle $\\angle AOL = 2\\angle AKL$ is the central angle of the circumcircle arc AL. Hence\r\n\r\n$\\angle KLO = \\angle OLA - \\angle KLA = \\frac{\\pi - \\angle AOL}{2} - \\angle KLA = \\frac{\\pi}{2} - \\angle AKL - \\angle KBA =$\r\n\r\n$= \\frac{\\pi}{2} - \\frac{\\gamma - \\beta}{3} - \\left ( \\frac{\\pi}{2} - \\frac{2(\\gamma - \\beta)}{3} \\right ) = \\frac{\\gamma - \\beta}{3} = \\angle KLA$\r\n\r\nConsequently, the line AK (having the same direction as the Simson line perpendicular to the Morley triangle side EF) and the line LO (from the pole of this Simson line through the circumcenter O) form the same angles with their transversal KL and they are therefore parallel. As a result, we know that the three Simsons lines perpendicular to the three sides DE, EF, FD of the Morley triangle are parallel to the lines from their poles through the circumcenter O.\r\n\r\nAccording to Honsberger's Episodes (Chapter 11):\r\n[b]1. [/b]The orthopole of a line lies on the Simson line perpendicular to it.\r\n[b]2. [/b]If a line intersects the circumcircle, the two Simson lines corresponding to the two intersections meet at the orthopole of the line. \r\n[b]3. [/b]The orthopole of a line passing through the circumcenter lies on the 9-point circle.\r\n\r\nLet the line LO intersect the circumcircle $(O)$ at a point M diametrally opposite to L. The two Simson lines having poles L and M intersect at the orthopole of this line, which lies on the 9-point circle, and they are perpendicular to each other. Since the Simson line with the pole L is parallel to LM, the Simson line with the pole M is perpendicular to it. Altogether, there are three Simson lines perpendicular to the sides DE, EF, FD of the Morley triangle, respectively, and they mutually form angles $\\frac {\\pi}{3}$. Likewise, there are three Simson lines perpendicular to first three, which meet them on the 9-point circle. All that remains is to show one of the first three Simson lines passes through the center N of the 9-point circle. The Simson line perpendicular to it will then be a tangent to the 9-point circle and similarly for the other two pairs of Simson lines. For this, another theorem from Honsberger's Episodes (Chapter 5) comes handy:\r\n\r\n[b]4. [/b]A Simson line passes through the midpoint of the segment from its pole to the orthocenter and this midpoint lies on the 9-point circle.\r\n\r\nThe 9-point circle is the circumcircle of the medial triangle, hence, its radius is half of the circumradius. In addition, the orthocenter H is the external homothety center of the circumcircle $(O)$ and the 9-point circle $(N)$ (the centroid G is their internal homothety center). Therefore, the midpoint S of the segment HL and the pole L are centrally similar with the same homothety center H and the same coefficient $\\frac 1 2$. Since the line LM and the Simson line perpendicular the EF are parallel and since they pass through the corresponding points L, S, they are themselves centrally similar with the same homothety center and coefficient. Since the line LM passes through the circumcenter O, the parallel Simson line must pass through the center N of the 9-point circle $(N)$.\r\n\r\nQ.E.D.\r\n\r\nEdit: Honsberger's name and some other spelling errors fixed. Thanks.", "Solution_2": ";)", "Solution_3": "to yetti. Nice solution. I had some similar ideas concerning Simson lines passing through the center of nine point circle. And my solution also uses the same well-known facts about the homothety with center in the orthocenter that maps the circumcircle to the nine point circle. But anyway, I suppose that our solutions are different. So, I will post my solution soon.\r\n\r\nOne more solution is in the article of L. Emelyanov and T. Emelyanova \"The Morley theorem. 100 years later.\" In this article the authors give two open questions:\r\n\r\n1. It is easy to prove that the Morley triangle and the original triangle are perspective. What is the center of the perspective?\r\n\r\n2. What is the center of homothety of the triangle PQR (which vertices have Simson lines touching to the nine point circle) and the Morley triangle?\r\n\r\nThese two questions are still open. So, if someone have any ideas...", "Solution_4": "So, here my solution is.\r\n\r\nWe will use the following well-known facts.\r\n\r\n(1) Let $ABC$ and $A'B'C'$ be triangles such that $AB \\parallel A'B'$, $BC \\parallel B'C'$ and $CA \\parallel C'A'$. Then the triangles $ABC$ and $A'B'C'$ are either homothetic or coincide by parallel translation.\r\n\r\n(2) Let $ABC$ be an arbitrary triangle, $H$ be its orthocenter, $\\omega$ be its nine point circle, $\\Omega$ be its circumcircle with center $O$. Then the homothety with center $H$ and coefficient $1/2$ maps $\\Omega$ to $\\omega$. In particular, the center of $\\omega$ is a midpoint of the segment $OH$.\r\n\r\n(3) Let $A'B'C'$ be the Morley triangle of a triangle $ABC$. Then $\\angle{C'B'A}=60^{\\circ}+\\angle{C}/3$.\r\n\r\n(4) Let $A'B'C'$ be the Morley triangle of a triangle $ABC$. Then the angle between the lines $BC$ and $B'C'$ is equal to $|\\angle{C}/3-\\angle{B}/3|$.\r\n\r\n(5) Let $PQ$ be a chord of $\\Omega$ such that $PQ \\perp AC$. Then the Simson line of the point $P$ is parallel to the line $BQ$.\r\n\r\n(6) If $X$ and $Y$ are diametrically opposite points then the Simson lines of these points are perpendicular to each other and $\\omega$ passes through its point of intersection.\r\n\r\n(7) If $P \\in \\Omega$ then the Simson line of $P$ passes through the midpoint of the segment $PH$.\r\n\r\nBack to the original problem, we have to show that there exist precisely three points which Simson lines tangent to $\\omega$. Let $P$ be one of such points and $l_P$ be its Simson line. Then, according to (6), for the diametrically opposite point $P'$ we have: $l_{P'} \\perp l_P$ and $l_P \\cap l_{P'} \\in \\omega$, that is $l_{P'}$ passes through the center of $\\omega$. Observe that the converse is also true, that is if the Simson line of some point passes through the center of $\\omega$ then the Simson line of the diametrically opposite point tangents to $\\omega$. Therefore it is sufficient to prove that there exist precisely three points which Simson lines pass through the center of $\\omega$.\r\n\r\nLet $O$ be the center of circumcircle $\\Omega$ of the triangle $ABC$ and $H$ be the orthocenter.\r\n\r\n[b]Lemma.[/b] Let $P \\in \\Omega$. Then the Simson line of $P$ passes through the center of $\\omega$ if and only if $PO \\parallel QB$, where $Q$ is a point at $\\Omega$ such that $PQ \\perp AC$.\r\n\r\n[i]Proof.[/i] According to (7), the Simson line of the point $P$ passes through the midpoint of the segment $PH$. If it, in addition, passes through the midpoint of $OH$ (that is, according to (2), through the center of $\\omega$), then it contains the medial line of the triangle $OHP$, and hence is parallel to $PO$. Conversely, if it's parallel to $PO$, then it contains the medial line of the triangle $OHP$, and hence passes through the midpoint of the segment $OH$, that is through the center of $\\omega$.\r\n\r\nTo complete the proof it is sufficient to apply the claim (5). $\\square$\r\n\r\nAnd now let $P$ be a point running over the circle $\\Omega$. We are going to investigate the behaviour of the point $Q$ (such that $PQ \\perp AC$). Let $P$ ran from the point $P_1$ to the point $P_2$ the arc with angular measure $\\varphi$, that is the line $OP$ rotated about the point $O$ by the angle $\\varphi$. Let $Q_1$ and $Q_2$ be the corresponding to $P_1$ and $P_2$ positions of $Q$. Then $P_1Q_1 \\parallel P_2Q_2$, and hence the arcs $P_1P_2$ and $Q_1Q_2$ are equal. But it is easy to see that the line $BQ$ rotated about $B$ by the angle $\\varphi/2$, in opposite direction (for instance, if the line $OP$ rotates clockwise, then the line $BQ$ rotates counter-clockwise).\r\n\r\nSo, during the point $P$ was running all the circle $\\Omega$, the line $OP$ rotated by $360^{\\circ}$. During the same time the line $BQ$ rotated by $180^{\\circ}$ (when the points $B$ and $Q$ were the same, we had to consider the tangent to $\\Omega$ at $B$). The question is: \"How many times was there a position when these lines ($PO$ and $BQ$) were parallel to each other?\". The answer is: 3 times. Indeed, observe that the centers of rotations don't matter. Therefore, without loss of generality, we may assume that the centers are the same. Then, instead of rotation both lines in opposite directions, we may consider rotation only one line by the angle $360^{\\circ}+180^{\\circ}=3 \\cdot 180^{\\circ}$. And now it is extremely obvious that the position with $PO \\parallel BQ$ occurs exactly three times. So, we have proved that there are exactly three points which Simson lines tangent to $\\omega$.\r\n\r\nNow let's prove the second part of the problem (about the homotheticness with the Morley triangle). For this goal we will find the location of the points which existance we have just proven.\r\n\r\nWithout loss of generality we may assume $\\angle{BCA} \\geq \\angle{CAB}$. Let the point $B$ be a start position of the point $P$. In this start position the angle between the lines $OP$ and $BQ$ is equal to $\\angle{OBH}=\\gamma-\\alpha$, where $\\alpha=\\angle{CAB}$, $\\beta=\\angle{ABC}$ and $\\gamma=\\angle{BCA}$. During the rotation of the line $OP$ counter-clockwise by the angle $\\varphi$, the angle between the lines $OP$ and $BQ$ increases by $3\\varphi/2$. Therefore by the first desired position of the point $P$, the line $OP$ have to rotate by the angle $2(\\gamma/3-\\alpha/3)$. Call this point $P_1$. Then to obtain the second desired position, the line $OP$ have to\r\nrotate more by the angle $2 \\cdot 180^{\\circ}/3=120^{\\circ}$. And then more by $120^{\\circ}$ to obtain the third desired position. Denote these points by $P_2$ and $P_3$. Clearly, the triangle $P_1P_2P_3$ is equilateral. The rest is to prove that its sides are parallel to the sides of the Morley triangle. But it is trivial. Indeed, the angle between the lines $P_1P_3$ and $BC$ is equal to $\\smile{P_3C}/2-\\smile{BP_1}/2=(\\smile{BA}/2+\\smile{AC}/2-\\smile{BP_1}/2-\\smile{P_1P_3}/2)-\\smile{BP_1}/2=(\\gamma+\\beta-\\gamma/3+\\alpha/3-120^{\\circ})-(\\gamma/3-\\alpha/3)=\\ldots=\\beta/3-\\gamma/3$, that is equal to the angle between the lines $B'C'$ and $BC$ (see the claim (4)), where $A'B'C'$ is the Morley triangle. Consequently, $P_1P_3 \\parallel B'C'$. Similarly we obtain that $P_1P_2 \\parallel A'B'$ and $P_2P_3 \\parallel C'A'$. So, we are done.\r\n\r\nWe have considered the triangle $P_1P_2P_3$ which vertices have Simson lines passing through the center of the nine point circle. And now let us consider the triangle $P_1'P_2'P_3'$ which vertices have Simson lines touching $\\omega$ (this triangle was called $PRQ$ in the problem setting). We have proved earlier that the central symmetry with center $O$ maps $P_1'P_2'P_3'$ to $P_1P_2P_3$. It means that we also have $P_1'P_3' \\parallel B'C'$, $P_1'P_2' \\parallel A'B'$ and $P_2'P_3' \\parallel C'A'$. Applying now the property (1), we can easily obtain desired fact (that is the triangle $P_1'P_2'P_3'$ is homothetic to the triangle $A'B'C'$).\r\n\r\nThe rest is to prove that the triangle with vertices in the points of touching is also homothetic to the triangle $A'B'C'$. Consider a homothety with center $H$ and coefficient $1/2$. According to (2), it maps $\\Omega$ to $\\omega$. Therefore the point $P_1'$ coincide with some point $X_1 \\in \\omega$. On the other hand, by the claim (7), $X_1$ lies at the Simson line of $P_1'$. But the Simson line of $P_1'$ tangents to $\\omega$, and hence $X_1$ is a point of touching.\r\n\r\nSimilarly, we can prove that the points of touching $X_2$ and $X_3$ are the images of the points $P_2'$ and $P_3'$ through our homothety. So, the triangle with vertices in the points of touching is homothetic to the triangle $P_1'P_2'P_3'$, which is homothetic to the Morley triangle as proven earlier.\r\n\r\nSo, all the parts of the problem are proven now.", "Solution_5": "[quote=\"yetti\"]WLOG, assume that the relative size of the angles $\\alpha = \\angle CAB$, $\\beta = \\angle ABC$, $\\gamma = \\angle BCA$ is $\\beta < \\alpha < \\gamma$ (that's how I draw the triangle). Denote the Morley equilateral triangle $\\triangle DEF$, where D is the intersection of the appropriate trisectors of the angles $\\beta$,$\\gamma$, E the intersection of the appropriate trisectors of the angles $\\gamma$, $\\alpha$ and F the intersection of the appropriate trisectors of the angles $\\alpha$, $\\beta$. The angles formed by 2 trisectors from the same vertex of the $\\triangle ABC$ with the sides of the equilateral triangle $\\triangle ABC$ are:\n\n$\\angle CDE = \\frac{\\beta + \\pi}{3}$, $\\angle CED = \\frac{\\alpha + \\pi}{3}$\n\n$\\angle AEF = \\frac{\\gamma + \\pi}{3}$, $\\angle AFE = \\frac{\\beta + \\pi}{3}$\n\n$\\angle BFD = \\frac{\\alpha + \\pi}{3}$, $\\angle BDF = \\frac{\\gamma + \\pi}{3}$\n\nAn easy proof is to draw an arbitrary equilateral triangle $\\triangle DEF$, take the angles $\\alpha + \\beta + \\gamma = \\pi$, build the above angles on the sides of this equilateral triangle and show, for example, that the angles $\\angle FAB = \\frac{\\alpha}{3}$, $\\angle FBA = \\frac{\\beta}{3}$. Using sine theorem for the triangles $\\triangle AEF$, $\\triangle BDE$, $\\triangle ABF$,\n\n$\\frac{EF}{AF} = \\frac {\\sin \\left ( \\frac{\\alpha}{3} \\right )}{\\sin \\left ( \\frac{\\gamma + \\pi}{3} \\right )}$, $\\frac{DE}{BF} = \\frac {\\sin \\left ( \\frac{\\beta}{3} \\right )}{\\sin \\left ( \\frac{\\gamma + \\pi}{3} \\right )}$, $\\frac{AF}{BF} = \\frac {\\sin \\left ( \\frac{\\beta}{3} \\right )}{\\sin \\left ( \\frac{\\alpha}{3} \\right )}$\n\nSince in addition, $\\angle AFB = 2\\pi - \\frac{\\pi}{3} - \\frac{\\alpha + \\pi}{3} - \\frac{\\beta + \\pi}{3} = \\pi - \\frac{\\alpha + \\beta}{3}$, it follows that $\\angle FAB = \\frac{\\alpha}{3}$, $\\angle FBA = \\frac{\\beta}{3}$.[/quote]\n\nAt first, Yetti, thanks a lot for doing this work. The above quoted passage from your solution of the problem actually contains a nice proof of Morley's theorem, so you have done even more than was asked for.\n\nThe only complaint I have to your solution is that \"Hosenberger\" should be \"Honsberger\" throughout the text. :mrgreen:\n\nNow, to Pestich's post:\n\n[quote=\"pestich\"]Mathworld (ref. to Well's book) claims that the envelope\n of Simson lines is a deltoid (oriented as Morley's triangle) and it touches \n the sides of the original triangle in points isotomic to feet of the altitudes.\n\n Is it obvious?[/quote]\n\nWhat? That the envelope is a deltoid? By no means. A proof can be found in\n\n[url=http://racefyn.insde.es/Publicaciones/racsam/art%C3%ADculos/racsam%2095_1/Guzm%C3%A1n.pdf]M. de Guzm\u00e1n, [i]The envelope of the Wallace-Simson lines of a triangle. A simple proof of the Steiner theorem on the deltoid.[/i], Rev. R. Acad. Cien. Serie A. Mat. Geometr\u00eda y Topolog\u00eda 95 (2001) 1, p. 57-64[/url].\n\nI would call the proof anything but simple, but it is probably much simpler than all proofs of this fact that were known before.\n\n[quote=\"pestich\"]What triangle center is the radical point of the 3 circles\n constructed on these isotomic points as diameters?[/quote]\n\nYou mean the three circles whose diameters are the segments joining the feet of the altitudes of triangle ABC with the reflections of these feet in the midpoints of the respective sides of triangle ABC ? The radical center of these three circles is X(185), the Nagel point of the orthic triangle of triangle ABC. This is, by the way, quite easy to prove synthetically.\n\n[quote=\"pestich\"] It also seems that the 3 distances from the midpoints of the original triangle sides\n to the nearby touching points of Simson lines with the 9 PC (which seem to be\n equal to the distances from same midpoints to the intersections of sides with\n Simson lines tangent to 9 PC) \n are linked so that one of the distances is equal to the sum of the other two.[/quote]\n\nIf I correctly understand your assertion, then, unfortunately, it is not true. However, since the points where the Simson lines touch the nine-point circle form an equilateral triangle, for any point P on the nine-point circle, the distance from P to one of these points equals the sum of the distances from P to the other two.\n\nTo the post by Remike:\n\n[quote=\"Remike\"]One more solution is in the article of L. Emelyanov and T. Emelyanova \"The Morley theorem. 100 years later.\"[/quote]\n\nWhere can I find this article? Has it already appeared? I would be very delighted to see it, since I really like Emelyanov's articles!\n\n[quote=\"Remike\"]In this article the authors give two open questions:\n\n1. It is easy to prove that the Morley triangle and the original triangle are perspective. What is the center of the perspective?\n\n2. What is the center of homothety of the triangle PQR (which vertices have Simson lines touching to the nine point circle) and the Morley triangle?[/quote]\r\n\r\nWell, what do you mean by \"What is the center\"? Actually, none of these two centers is one of [i]the[/i] four famous triangle centers ;) . The center of perspective of the Morley triangle and the original triangle is X(357) in Clark Kimberling's ETC; the center of homothety of the triangle PQR and the Morley triangle is, if I'm not mistaken, not in the ETC yet.\r\n\r\nLast but not least, I wanted to say that I also have a solution to the initial problem of this thread. It is different from both Yetti's and Remike's solutions, although, maybe from the advanced viewpoint all three solutions are more or less the same. Whether I will post it here... well... that depends from weather ;) .\r\n\r\n darij", "Solution_6": "[quote=\"darij grinberg\"]\nAt first, Yetti, thanks a lot for doing this work. The above quoted passage from your solution of the problem actually contains a nice proof of Morley's theorem, so you have done even more than was asked for.\n\nThe only complaint I have to your solution is that \"Hosenberger\" should be \"Honsberger\" throughout the text. :D \n[/quote]\nI have seen this proof at the Cut-The-Knot, with a comment that direct proofs are too complicated. I also fixed Honsberger's name. :blush: Thanks.\n\n\n[quote=\"Remike\"]\nNice solution. I had some similar ideas concerning Simson lines passing through the center of nine point circle. And my solution also uses the same well-known facts about the homothety with center in the orthocenter that maps the circumcircle to the nine point circle. But anyway, I suppose that our solutions are different. So, I will post my solution soon. \n[/quote]\r\nThanks. Please, give me some time to go through the details of your solution.\r\n\r\nyetti", "Solution_7": "For other deepenings on Morley's theorem I suggest the following site http://shk.ans.hive.no/\r\n\r\nLeon", "Solution_8": "Reading your proof, I realized that I did not really show that only 3 Simson lines are tangent to the 9-point circle (or only 3 Simson lines pass through the center of the 9-point circle, which is the same thing). It seemed clear to me that at most 3 such Simson lines can exist, because when the pole moves from one vertex of the triangle $\\triangle ABC$ to another, the corresponding Simson line moves from one altitude to another, but I should have said something. Otherwise, the ideas of both proofs are similar, even though I did not see that the homothety of the circumcircle and the 9-point circle WRT centroid $G$ can be used as well. Thanks for posting your solution.\r\n\r\nYou may consider fixing some typing errors:\r\n\r\n[quote=\"Remike\"](3) Let $A'B'C'$ be the Morley triangle of a triangle $ABC$. Then $\\angle{C'B'A}=60^{\\circ}+\\angle{C}/2$.\n...\n[b]Lemma.[/b] Let $P \\in \\Omega$. Then the Simson line of $P$ passes through the center of $\\omega$ if and only if $PO \\perp QB$, where $Q$ is a point at $\\Omega$ such that $PQ \\perp AC$.\n...\nWe have proved earlier that the central symmetry with center $O$ maps $P_1'P_2'P_3'$ to $P_1P_2P_3$.[/quote]\r\n\r\nIn lemma 2, the angle $\\angle{C'B'A}=60^{\\circ}+\\angle{C}/3$.\r\nIn lemma 7, the Simson line of $P$ passes through the center of $\\omega$ if and only if $PO \\parallel QB$, as you proved.\r\nFinally, the homothety center of the triangles $P_1'P_2'P_3'$ and $P_1P_2P_3$ is the centroid $G$ of the triangle $\\triangle ABC$, as you showed.", "Solution_9": "[quote=\"yetti\"]even though I did not see that the homothety of the circumcircle and the 9-point circle WRT centroid $G$ can be used as well. Thanks for posting your solution.[/quote]\nWhat do you mean by \"centroid\"? Is it a common point of all three altitudes (that is the point that I called orthocenter)?\n\n[quote=\"yetti\"]You may consider fixing some typing errors:\n\n[quote=\"Remike\"](3) Let $A'B'C'$ be the Morley triangle of a triangle $ABC$. Then $\\angle{C'B'A}=60^{\\circ}+\\angle{C}/2$.\n...\n[b]Lemma.[/b] Let $P \\in \\Omega$. Then the Simson line of $P$ passes through the center of $\\omega$ if and only if $PO \\perp QB$, where $Q$ is a point at $\\Omega$ such that $PQ \\perp AC$.\n...\nWe have proved earlier that the central symmetry with center $O$ maps $P_1'P_2'P_3'$ to $P_1P_2P_3$.[/quote]\n\nIn lemma 2, the angle $\\angle{C'B'A}=60^{\\circ}+\\angle{C}/3$.\nIn lemma 7, the Simson line of $P$ passes through the center of $\\omega$ if and only if $PO \\parallel QB$, as you proved.[/quote]\nI have edited my post. Thank you.\n\n[quote=\"yetti\"]Finally, the homothety center of the triangles $P_1'P_2'P_3'$ and $P_1P_2P_3$ is the centroid $G$ of the triangle $\\triangle ABC$, as you showed.[/quote]\r\nIf I understand you correctly then it is not true. I meant here that $P_1$, $P_2$ and $P_3$ are the points for which the Simson lines pass through the center of $\\omega$; and $P_1'$, $P_2'$ and $P_3'$ are the points for which the Simson lines tangent $\\omega$. I showed that $P_i$ is diametrically opposite to $P_i'$. So, the central symmetry with center $O$ maps $P_1P_2P_3$ to $P_1'P_2'P_3'$.", "Solution_10": "The intersection of medians is called centroid in English geometry textbooks and usually denoted $G$.\r\n\r\nWhen reading the problem, I thought that $P, Q, R$ were the the tangency points of the Simson lines with the 9-point circle, not their poles on the circumcircle. Similarly when reading your proof, for the points $P_1', P_2', P_3'$ (originally denoted $P, Q, R$) are mentioned at the end, when everything becomes clear. But of course, you denoted the tangency points $X_1, X_2, X_3$. So, the triangles $\\triangle P_1P_2P_3 \\sim \\triangle P_1'P_2'P_3'$ are centrally similar with homothety center at the circumcenter $O$ and coefficient $-1$, while the triangles $\\triangle P_1P_2P_3 \\sim \\triangle X_1X_2X_3$ are centrally similar with homothety center at the centroid $G$ and coefficient $-\\frac 1 2$.", "Solution_11": "[quote=\"darij grinberg\"][quote=\"Remike\"]One more solution is in the article of L. Emelyanov and T. Emelyanova \"The Morley theorem. 100 years later.\"[/quote]\n\nWhere can I find this article? Has it already appeared? I would be very delighted to see it, since I really like Emelyanov's articles![/quote]\nWell, I don't know where you can find it. It was published in Russian journal \"Mathematics in school\" (\"\u041c\u0430\u0442\u0435\u043c\u0430\u0442\u0438\u043a\u0430 \u0432 \u0448\u043a\u043e\u043b\u0435\"), \u2116 9, 2004. If you know Russian, you can see here a content of this journal:\nhttp://www.schoolpress.ru/shop/aposoft.z/StoreFront-Main .\nAnd here the abstract of this paper is:\nhttp://www.schoolpress.ru/shop/SPStoreFront/Share/public/jornal/matematika/short_text/9_2004/08_9_04_62.html .\n\nBy the way, some time ago another article of L. Emelyanov \"Geometrical proof of the Ponsele theorem\" (\"\u0413\u0435\u043e\u043c\u0435\u0442\u0440\u0438\u0447\u0435?\u043a\u043e\u0435 \u0434\u043e\u043a\u0430\u0437\u0430\u0442\u0435\u043b\u044c?\u0442\u0432\u043e \u0442\u0435\u043e\u0440\u0435\u043c\u044b \u041f\u043e\u043d?\u0435\u043b\u0435\") was published in this journal (\u2116 3, 2004). Here the abstract is: http://www.schoolpress.ru/shop/SPStoreFront/Share/public/jornal/matematika/short_text/3_2004/08_3_04_73.html .\n\n[quote=\"darij grinberg\"][quote=\"Remike\"]In this article the authors give two open questions:\n\n1. It is easy to prove that the Morley triangle and the original triangle are perspective. What is the center of the perspective?\n\n2. What is the center of homothety of the triangle PQR (which vertices have Simson lines touching to the nine point circle) and the Morley triangle?[/quote]\n\nWell, what do you mean by \"What is the center\"? Actually, none of these two centers is one of [i]the[/i] four famous triangle centers ;) . The center of perspective of the Morley triangle and the original triangle is X(357) in Clark Kimberling's ETC; the center of homothety of the triangle PQR and the Morley triangle is, if I'm not mistaken, not in the ETC yet.[/quote]\r\nWell, I can quote from the original text of the article (in Russian), but I'm afraid it is out of law. But I suppose I may retell it in English. So, here my translation of the final part of the article is.\r\n\r\n\"It seems that we can finish our consideration of the Morley theorem. However, we want to think that this material will have become a beginning of an investigation of the properties of the Morley triangle, because every fundamental theorem generates more questions than answers. For instance, it is easy to prove that the original triangle and its Morley triangle are perspective, that is the lines AD, BE and CF have a common point. What is that point? Is it a well-known point for the triangles ABC and DEF? Does it lie at any well-known line? And there is one more natural question: what is the center of homothety of the triangles PQR and DEF?\r\n\r\nDream, ask questions, put forward ideas --- modern computer provides an opportunity to verify many and many geometrical conjectures. Fortunately, it (computer) can't prove these conjectures, since we need geometrical intuition and mind to prove it. But it can help us to develop both these things.\"", "Solution_12": "[quote=\"yetti\"]The intersection of medians is called centroid in English geometry textbooks and usually denoted $G$.[/quote]\nYes, in Russian too.\n\n[quote=\"yetti\"]When reading the problem, I thought that $P, Q, R$ were the the tangency points of the Simson lines with the 9-point circle, not their poles on the circumcircle. Similarly when reading your proof, for the points $P_1', P_2', P_3'$ (originally denoted $P, Q, R$) are mentioned at the end, when everything becomes clear. But of course, you denoted the tangency points $X_1, X_2, X_3$. So, the triangles $\\triangle P_1P_2P_3 \\sim \\triangle P_1'P_2'P_3'$ are centrally similar with homothety center at the circumcenter $O$ and coefficient $-1$, while the triangles $\\triangle P_1P_2P_3 \\sim \\triangle X_1X_2X_3$ are centrally similar with homothety center at the centroid $G$ and coefficient $-\\frac 1 2$.[/quote]\r\nI understand you now. There are two homotheties that maps $\\Omega$ to $\\omega$: with positive coefficient and with negative coefficient. You consider a homothety with negative coefficient and with center at $G$. And I consider a homothety with positive coefficient and with center at $H$. You claim that the triangles $P_1P_2P_3$ and $X_1X_2X_3$ are homothetic with coefficient $-1/2$ and center at $G$. Well, it's true. And I claim that the triangles $P_1'P_2'P_3'$ and $X_1X_2X_3$ are homothetic with coefficient $1/2$ and center at $H$. Well, it's also true." } { "Tag": [], "Problem": "Simplify and express your answer as a common fraction.\n\n\\[ \\dfrac{\\frac{1}{1\\plus{}\\frac{2}{1\\plus{}3}}}{\\frac{3}{2\\plus{}\\frac{1}{1\\plus{}1}}}\\]", "Solution_1": "$ \\dfrac{\\frac{1}{1\\plus{}\\frac{2}{1\\plus{}3}}}{\\frac{3}{2\\plus{}\\frac{1}{1\\plus{}1}}}\\equal{}\\dfrac{\\frac{1}{1\\plus{}\\frac{2}{4}}}{\\frac{3}{2\\plus{}\\frac{1}{2}}}\\equal{}\\dfrac{\\frac{1}{1\\plus{}\\frac{1}{2}}}{\\frac{3}{2\\plus{}\\frac{1}{2}}}\\equal{}\\dfrac{\\frac{1}{\\frac{3}{2}}}{\\frac{3}{\\frac{5}{2}}}\\equal{}\\dfrac{\\frac{2}{3}}{\\frac{6}{5}}\\equal{}\\frac{10}{18}\\equal{}\\frac{5}{9}$" } { "Tag": [ "linear algebra", "matrix" ], "Problem": "Find the inverse of the $n$x$n$ \"lower\" triangular matrix. Prove that the matrix you give is indeed the inverse.\r\n\r\nBy lower triangular matrix I mean the matrix with a 1 in every entry below or on the main diagonal, and a 0 in every other entry.", "Solution_1": "perform the following row operations on the matrix:\r\n\r\n$R_{n}\\mapsto R_{n}-R_{n-1}$,\r\n$R_{n-1}\\mapsto R_{n-1}-R_{n-2}$,\r\n...\r\n$R_{2}\\mapsto R_{2}-R_{1}$.\r\n\r\nthe result is $I_{n}$. performing these same operations on $I_{n}$ now gives the inverse of the original matrix: the result is a main diagonal of 1's, the diagonal below that of all -1's, and everything else 0." } { "Tag": [ "linear algebra", "matrix", "vector" ], "Problem": "Prove that the geometric multiplicity of an eigenvalue cannot exceed its algebraic multiplicity.", "Solution_1": "If algebraic and geometric multiplicities are defined via the Jordan form of a matrix, as some authors do, your argument is obvious.\r\n\r\nHere is another treatment: \r\nLet $ k$ be the dimension of the eigenspace $ V_{\\lambda}$ corresponding to the eigenvalue $ \\lambda$. Then extend $ V_\\lambda$ to a basis $ \\mathcal B$ for your vector space $ V$. Clearly the matrix of your operator, say $ \\tau$, with respect to $ \\mathcal B$ has the form:\r\n$ \\begin{pmatrix}\\lambda I_k & A\\\\ 0& B\\end{pmatrix}$.\r\nBut, $ c_{\\tau}(x)=\\det(xI-[\\tau]_{\\mathcal B})=(x-\\lambda)^k\\det(xI_{n-k}-B).$ Here $ n$ is the dimension of $ V$. Hence, the result follows." } { "Tag": [ "geometry", "circumcircle", "geometric transformation", "reflection", "analytic geometry", "trigonometry", "parameterization" ], "Problem": "Let $ \\Gamma$ be the circle with radius $ r$ and center $ O$ and $ \\Gamma'$ be the circle with radius $ r'$ and center $ O'$. Let $ T$ be a variable point of $ \\Gamma$ and the tangent line $ t$ passing by $ T$ intersect the circle $ \\Gamma'$ at two points $ A$ and $ B$. Find the locus of the circumcenter of the triangle $ BOA$ when $ T$ varies on $ \\Gamma$. I know, making a figure, that the locus is a circle of center $ O'$ with a radius depending of $ OO'$, $ r$ and $ r'$. But how to prove it? Thanks in advance.", "Solution_1": "First, invert the figure in the circle $ \\Gamma$ with center $ O.$ Tangent of $ \\Gamma$ at $ T$ goes to a circle $ \\mathcal T^*$ with diameter $ OT.$ Arbitrary circle $ \\Gamma'$ with center $ O'$ (not entirely inside of $ \\Gamma$) goes to a circle $ \\Gamma'^*$, intersecting the circle $ \\mathcal T^*$ at $ A^*, B^*.$ Circumcircle $ \\mathcal C$ of the $ \\triangle OAB$ goes to the line $ A^*B^*$ and its circumcenter $ C$ into the reflection $ C^*$ of $ O$ in $ A^*B^*.$ Let $ E \\in OO'$ be the center of $ \\Gamma'^*$ and $ s$ its radius. Denote $ d \\equal{} OO',\\ e \\equal{} OE.$ Since $ O$ is the similarity center of $ \\Gamma'^*$ and $ \\Gamma'$ with similarity cefficient $ k \\equal{} \\frac {r^2}{d^2 \\minus{} r'^2}$ (power of inversion divided by power of the inversion center to the original circle), we have $ e \\equal{} kd, s \\equal{} kr'.$ \r\n\r\nLet $ O$ be the coordinate origin and $ OO'$ the x-axis. Equations of the circle $ \\mathcal T^*$ with fixed diameter $ r \\equal{} OT$ and passing through the origin $ O$ is\r\n\r\n$ \\mathcal T^*: \\ \\ x^2 \\minus{} xr \\cos \\theta \\plus{} y^2 \\minus{} yr \\sin \\theta \\equal{} 0,$\r\n\r\nwhile equation of the circle $ \\Gamma'^*$ centered on the x-axis at $ E \\equal{} (e, 0)$ and with radius $ s$ is\r\n\r\n$ \\Gamma'^*: \\ \\ x^2 \\minus{} 2ex \\plus{} e^2 \\plus{} y^2 \\equal{} s^2.$\r\n\r\nSubtracting equations of the 2 circles from each other gives equation of their radical axis $ A^*B^*$:\r\n\r\n$ A^*B^*: \\ \\ F(x, y, \\theta ) \\equal{} x(r \\cos \\theta \\minus{} 2e) \\plus{} yr \\sin \\theta \\plus{} e^2 \\minus{} s^2 \\equal{} 0.$\r\n\r\nTransforming the last equation to polar coordinates $ (\\varrho, \\phi)$ by $ x \\equal{} \\varrho \\cos \\phi,\\ y \\equal{} \\varrho \\sin \\phi,$\r\n\r\n$ A^*B^*: \\ \\ \\Phi(\\varrho, \\phi, \\theta ) \\equal{} \\varrho r \\cos (\\phi \\minus{} \\theta) \\minus{} 2 \\varrho e \\cos \\phi \\plus{} e^2 \\minus{} s^2 \\equal{} 0.$\r\n\r\nThis is equation of a family of lines depending on the parameter $ \\theta.$ Equation of their envelope $ \\mathcal K$ (if any) is obtained by eliminating this parameter from the equations $ \\Phi(\\varrho, \\phi, \\theta ) \\equal{} 0$ and\r\n\r\n$ \\frac {\\partial \\Phi}{\\partial \\theta} \\equal{} \\varrho r \\sin (\\phi \\minus{} \\theta) \\equal{} 0.$\r\n\r\nThe 2nd equation yields $ \\phi \\minus{} \\theta \\equal{} n \\pi,\\ \\cos(\\phi \\minus{} \\theta) \\equal{} \\pm 1.$ Substituting this to the polar equation $ \\Phi(\\varrho, \\phi, \\theta) \\equal{} 0$ of the line $ A^*B^*,$ the envelope equation comes out as\r\n\r\n$ \\mathcal K: \\ \\ \\pm \\varrho r \\minus{} 2 \\varrho e \\cos \\phi \\plus{} e^2 \\minus{} s^2 \\equal{} 0,$ or\r\n\r\n$ \\varrho \\equal{} \\frac {e^2 \\minus{} s^2}{r} \\cdot \\frac {1}{\\frac {2e}{r} \\cos \\phi \\pm 1} \\equal{} \\frac {L}{\\epsilon \\cos \\phi \\pm 1}.$\r\n\r\nThis is polar equation of a conic with focus $ O,$ major axis line $ \\phi \\equal{} 0$ (i.e., the line $ OO'$), excentricity $ \\epsilon \\equal{} \\frac {2e}{r}$ and semilatus rectum $ |L| \\equal{} \\frac {|e^2 \\minus{} s^2|}{r}.$ The correct sign, obtained by demanding $ \\varrho > 0,$ depends on radii $ r, r'$ and center distance $ d \\equal{} OO'$ of the circles $ \\Gamma, \\Gamma'.$ For an ellipse $ (\\epsilon < 1$), the correct sign is the same as the sign of $ e^2 \\minus{} s^2 \\equal{} k^2(d^2 \\minus{} r'^2)$ or $ d \\minus{} r'$ ($ O$ outside or inside of $ \\Gamma'$), for a hyperbola ($ \\epsilon > 1$) one sign holds for each branch.\r\n\r\nSince $ A^*B^*$ is tangent to this conic, the foot $ F$ of perpendicular from the focus $ O$ to $ A^*B^*$ lies on its pedal circle $ \\mathcal P$ centered at $ P \\in OO'$ on the conic major axis. Since $ F$ is midpoint of $ OC^*,$ the image $ C^*$ of the circumcenter $ C$ lies on a circle $ \\mathcal Q.$ centrally similar to $ \\mathcal P$ with similarity center $ O$ and coefficient 2, also centered on $ OO'.$ As a result, $ C$ lies on the inversion image $ \\mathcal Q^*$ of $ \\mathcal Q,$ also centered on $ OO'.$ The center $ Q$ of $ \\mathcal Q$ is the other conic focus\r\n\r\nAssume $ \\epsilon > 1$ (hyperbola) and $ d > r'$ ($ O$ outside $ \\Gamma'$). Substituting $ \\phi \\equal{} 0$ into the hyperbola equation, $ q \\equal{} OQ \\equal{} \\frac {L}{\\epsilon \\plus{} 1} \\plus{} \\frac {L}{\\epsilon \\minus{} 1} \\equal{} \\frac {2\\epsilon L}{\\epsilon^2 \\minus{} 1}.$ Radius of $ \\mathcal Q$ is equal to the conic major axis length, $ t \\equal{} \\frac {OQ}{\\epsilon} \\equal{} \\frac {2L}{\\epsilon^2 \\minus{} 1}$ for a hyperbola. Again, $ O$ is the similarity center of $ \\mathcal Q^*, \\mathcal Q$ with similarity coefficient $ m \\equal{} \\frac {r^2}{q^2 \\minus{} t^2} \\equal{} r^2 \\cdot \\frac {\\epsilon^2 \\minus{} 1}{4L^2},$ so that distance of the center of $ \\mathcal Q^*$ from $ O$ is\r\n\r\n$ mq \\equal{} r^2 \\cdot \\frac {\\epsilon^2 \\minus{} 1}{4L^2} \\cdot \\frac {2\\epsilon L}{\\epsilon^2 \\minus{} 1} \\equal{} r^2 \\cdot \\frac {\\epsilon}{2L} \\equal{} r^2 \\cdot \\frac {2e}{r} \\cdot \\frac {r}{2(e^2 \\minus{} s^2)} \\equal{} r^2 \\cdot \\frac {kd}{k^2(d^2 \\minus{} r'^2)} \\equal{} d.$\r\n\r\nSince $ d \\equal{} OO',$ the center of $ \\mathcal Q^*$ containing the circumcenter $ C$ is identical with the center $ O'$ of $ \\Gamma'.$" } { "Tag": [ "algebra", "polynomial", "algebra unsolved" ], "Problem": "Let $P(x)$ be a polynomial with integer coefficents. And let $Q(x) = \\prod \\limits^n_{k=1} P(x^k) + 1.$ Determine whether $Q(x)$ has integer roots dependently on $n$.", "Solution_1": "Is it right that we have to define all natural $n$ s.t. there is $P(x)\\in Z[x]$ that $Q(x)$ has integer root?" } { "Tag": [ "AMC", "USA(J)MO", "USAMO", "AIME", "inequalities", "pigeonhole principle", "AMC 10" ], "Problem": "I'd have to say mine is about 3", "Solution_1": "mine is maybe 1 problem right.. so 7/42 :blush:", "Solution_2": "I'm going to be taking the USAMO unofficially, I'm so mad I didn't qualify because of 1 error, I think I could easily get like a 14 if I made it :mad:", "Solution_3": "lol i clicked 42 just for the heck of it.", "Solution_4": "Hmmm... I clicked 8-14. Since I voted 0-9 in another topic, I guess my score will be 8 or 9.", "Solution_5": "Well that depends on who's doing the expecting. If you ask Delong, I'm getting a 42. If you ask me, I'm getting just enough to make red mop, or 14+ if both #1 and #4 are really easy (read: not geometry).", "Solution_6": "I am so getting a binary number", "Solution_7": "[quote=\"Ignite168\"]I'm going to be taking the USAMO unofficially, I'm so mad I didn't qualify because of 1 error, I think I could easily get like a 14 if I made it :mad:[/quote]\r\n\r\nTake it from the guy who scored a 3 twice in a row Ignite, even if you think you could solve two problems, getting a 14 is far more difficult than you realize. \r\n\r\n--the bubala", "Solution_8": "I'm hoping for a decent improvement, so I said 15-21 (I got 10 last year.) Given the fact that I had a 42-point increase between my first AMC-10 and my second, a 5-point increase between my first AIME and my second, and a 20-point increase between my first AMC-12 and my second, I'm thinking that that might be a reasonable goal.", "Solution_9": "[quote=\"bubala\"][quote=\"Ignite168\"]I'm going to be taking the USAMO unofficially, I'm so mad I didn't qualify because of 1 error, I think I could easily get like a 14 if I made it :mad:[/quote]\n\nTake it from the guy who scored a 3 twice in a row Ignite, even if you think you could solve two problems, getting a 14 is far more difficult than you realize. \n\n--the bubala[/quote]\r\n\r\nHe was joking anyway; he qualified.", "Solution_10": "does he ever not joke?\r\n\r\nhmm...i put 15-21 b/c i hope that that will be enough for blue MOP (at least, the upper end of that range) which is a goal that i have worked EXTREMELY hard for.\r\n\r\nplus i scored 7 last year (2 points from red MOP)", "Solution_11": "uhhh\r\n\r\nhopefully enough to get into red mop\r\n\r\nmeaning i'll be spending my 4.5 hours bashing the living crap out of inequalities with my multivariable calc knowledge and gross misapplications of pigeonhole, CRT, and muirhead\r\n\r\n\r\nbe prepared, usamo graders", "Solution_12": "[quote=\"Treething\"]uhhh\n\nhopefully enough to get into red mop\n\nmeaning i'll be spending my 4.5 hours bashing the living crap out of inequalities with my multivariable calc knowledge and gross misapplications of pigeonhole, CRT, and muirhead\n\n\nbe prepared, usamo graders[/quote]\r\n\r\nJust a tip. If you actually do find yourself writing a proof with Lagrange Multipliers, make sure that you do all the arithmetic (solving the n equations for n variables) in your proof, or else you will get very few points.", "Solution_13": "Zero, since I didn't make it. I'll take it unofficially just to see how I do. Zero, probably, but then again I can''t really know whether I'd get a 0 or a 1 on any of the problems.", "Solution_14": "[quote=\"Treething\"]uhhh\n\nhopefully enough to get into red mop\n\nmeaning i'll be spending my 4.5 hours bashing the living crap out of inequalities with my multivariable calc knowledge and gross misapplications of pigeonhole, CRT, and muirhead\n\n\nbe prepared, usamo graders[/quote]\r\n\r\nIf there are inequalities...hm\r\n\r\nAnd tim's facebook says you don't know the CRT", "Solution_15": "42. obviously. since Delong says so. :lol: \r\nIn reality, hoping for 15-21 so that i at least improved....", "Solution_16": "any positive score would satisfy me", "Solution_17": "[quote=\"undefined117\"]mine is maybe 1 problem right.. so 7/42 :blush:[/quote]\r\n\r\nAs an 8th grader with your AMC10 and AIME scores, solving one problem fully would be very *very* impressive. Unless you've taken practice tests before and *can* solve one problem on each one...?\r\n\r\nAnyways my goal is a non-negative score, although I'd be happy to get a negative score also.", "Solution_18": "Hm I said 13-31 in the last topic. I guess expected score is then $\\frac{13+31}{2}= 22$ assuming a linear probability distribution or symmetric about the median or whatever. I'll be happy if I do in fact get a $22$.", "Solution_19": "[quote=\"probability1.01\"]Hmmm... I clicked 8-14. Since I voted 0-9 in another topic, I guess my score will be 8 or 9.[/quote]\r\n\r\ndude, come on.. there are only 6 problems. Even you can't pull off an 8 :P", "Solution_20": "Good luck to everyone taking it tomorrow", "Solution_21": "I'd say 1-7. I don't think I'll draw blanks on every single question and be able to write nothing. If I get lucky, I'll see a question that I can solve, and maybe get a 7 on that problem." } { "Tag": [ "\\/closed" ], "Problem": "When I move my mouse over the private messages it says that 1 unread message.\r\n\r\nBut I've read all messages.\r\n\r\nWhat is this? :?", "Solution_1": "Looks like a bug :? . What style are you using? Also please tell me if refreshing the window / log in/out clears it ...", "Solution_2": "This is bit weird.\r\n\r\nI had AoPS as my board style and when I changed to MathLinks, I have \"no new private messages.\"\r\n\r\n? :?", "Solution_3": "[quote=\"Silverfalcon\"]This is bit weird.[/quote]I agree :) ... it might have been a cookie issue, or a private message that has been sent to you by someone, and then deleted during the time you spoted the link and when you clicked it. \r\n\r\nDoes returning to AoPS style cause the same problem? ...", "Solution_4": "Thanks Valentin. :D \r\n\r\nBack to ages where I used MathLink Board Style :D :D :D" } { "Tag": [ "superior algebra", "superior algebra unsolved" ], "Problem": "Let $I$ be an ideal of the ring $R$ and $f$ be nonidentity permutation of the set $\\{\\ 1,\\dots ,k\\}$.suppose that for every $0\\neq a\\in R,aI\\neq 0,Ia\\neq 0$ hold;for any elements of $x_1,x_2,\\dots,x_k\\in I$,$x_1x_2\\dots=x_{f1}\\dots x_{fk}$holdsprove that $R$ is commutative.", "Solution_1": "This one has been lying here for quite some time, huh? :)\r\n\r\nIf $f1=1$, the problem is reduced to smaller $k$'s, so we can assume this is not the case. We replace $x_1$ with $ax_1$ for an arbitrary $a\\in R$, and we get $ax_1\\ldots x_k=x_{f1}\\ldots ax_{ff^{-1}1}\\ldots x_{fk}$ on the one hand, and $ax_1\\ldots x_k=ax_{f1}\\ldots x_{fk}$ on the other hand. Equating those, we get $x_{f1}\\ldots ax_{ff^{-1}1}\\ldots x_{fk}=ax_{f1}\\ldots x_{fk}$. In this relation, we can eliminate $x_{ff^{-1}1},\\ldots,x_{fk}$ one by one because of the fact that $tI=0\\rightarrow t=0$, and we're left with a relation of the type $ax_1\\ldots x_u=x_1\\ldots x_ua,\\ \\forall x_i\\in I,\\ \\forall a\\in R$ for some $1\\le u\\le k-1$. If we take $a$ from $I$, this is a relation just like the one we started with, except that the permutation $f$ is a cycle here. This means that from now on we can assume $f$ to be a cycle.\r\n\r\nNow, if $f$ is a cycle and $k\\ge 3$, then either $fk\\ne 1$ or $f^2k\\ne 1$, and if we pull the same trick as before, with the arbitrary $a$ and the elimination of $x_{ff^{-1}1},\\ldots,x_{fk}$, we'll eliminate at least two $x_i$'s. This leaves us with an analogous problem, but for smaller $k$, and we can use induction. This means that it suffices to prove the whole thing for $k=2$ (and $f$ being a transposition).\r\n\r\nWe have $x_1x_2=x_2x_1,\\ \\forall x_i\\in I$. This means that for all $a\\in R$, we have $x_2ax_1=x_1x_2a=x_2x_1a$, so $I(ax_1-x_1a)=0,\\ \\forall x_1\\in I,\\ \\forall a\\in R$. We thus get $I\\subset Z(R)$ (the center of $R$). Finally, for $a,b\\in R,\\ x_1\\in I$, we have $ax_1b=x_1ab=x_1ba\\Rightarrow I(ab-ba)=0$, so $ab=ba$, and we're done.", "Solution_2": "From $ ax_1...x_u\\equal{}x_1...x_ua$ you can finish it faster:\r\n$ abx_1...x_u \\equal{} a(bx_1...x_u) \\equal{} bx_1...x_ua \\equal{} bax_1...x_u$, so $ (ab\\minus{}ba)$ is killed by $ I^u$ which means (by hypothesis) that $ ab\\minus{}ba\\equal{}0$" } { "Tag": [ "AoPSwiki", "articles", "Alcumus", "videos", "Pascal\\u0027s Triangle" ], "Problem": "Pascal's Triangle and the Hockey Stick Theorem are very useful in mathematics, so I heard.\r\n\r\nI want to learn it but the AOPSWiki article confused me.\r\n\r\nCan you guys help?", "Solution_1": "There are Alcumus videos on those things. I find them very useful. Now, everybody can access all the videos, so watch them now before you can't!", "Solution_2": "Hmm.. Did you Google them?", "Solution_3": "[quote=\"Cliu0301\"]Pascal's Triangle and the Hockey Stick Theorem are very useful in mathematics, so I heard.\n\nI want to learn it but the AOPSWiki article confused me.\n\nCan you guys help?[/quote]\r\n\r\nTry intro. to counting&probability book (AOPS), that gives you a great understanding in pascal's triangle. You get to derive the proofs, and all.\r\nIts great fun!" } { "Tag": [ "percent", "MATHCOUNTS" ], "Problem": "A shape that includes the point (3,8) is reflected across the line y = x + 2. At what point does the point (3,8) land?\r\n\r\nA store puts everything on sale for 20% off. If the sales tax is 8%, what percent of the original marked price is the final cost including tax? Round answers to the nearest tenth.\r\n\r\nCHALLENGE: SIG FIG MANIA\r\n\r\nAnswer the following in significant figures.\r\n\r\n1. 2.0 x 3.625\r\n2. 0.0000003 / 1.0\r\n\r\nHow many sig figs do each of these numbers have?\r\n\r\n1. 1.204\r\n2. 0.200000\r\n3. 0.036230\r\n4. 100.0", "Solution_1": "hmm, these look suspiciously like homework problems as opposed to challenge problems (who gives sig figs in a challenge problem?)", "Solution_2": "SIF FIG\r\n\r\nsounds like a sciences class and the problems are too easy to not be hw", "Solution_3": "lol, nice try buddy\r\n\r\nu n00b", "Solution_4": "What's bad about science?\r\n\r\nWhat's a significant figure anyway?\r\n\r\n(I grew up learning in foreign languages; I might know it already)", "Solution_5": "[quote=\"soulzmischief\"]lol, nice try buddy\n\nu n00b[/quote]\r\n\r\n\r\ngoodness, person. i bet you're in math counts and you're in my science class. you're either T-guy, Ma-guy, L-guy, or Mi-guy. Most likely you're T-guy.", "Solution_6": "it has been a day, and we did sig figs three weeks ago.\r\n\r\nanswers: (6,5), 86.4, 7.3, .0000003, 4, 6, 5, 4\r\n\r\nT-guy, shut up. and dont even THINK about talking to me during school", "Solution_7": "[quote=\"pakagawa\"]What's bad about science?\n\nWhat's a significant figure anyway?\n\n(I grew up learning in foreign languages; I might know it already)[/quote]\r\n\r\nThe number of significant figures a number has tells you how accurate that number is. The number of sig. figures in 2.0021 is 5. I ll tell you more if you want but you might already know it so I ll just wait. It's not very hard to learn so anyone can just google it and learn it quickly.", "Solution_8": "[quote=\"soulzmischief\"]lol, nice try buddy\n\nu n00b[/quote]\r\n\r\nare u from hopkins?", "Solution_9": "hey buddy its yuchen.. ur mathcounts tutor\r\n\r\nhave a nice day... and i'll make sure thursday is pretty painful for you \r\n\r\n:)", "Solution_10": "[quote=\"soulzmischief\"]... and i'll make sure thursday is pretty painful for you \n\n:)[/quote]\r\n\r\ni cant wait. har har har", "Solution_11": "btw\r\n\r\nim sure that you know we're past sig figs and all that velocity ****. except for the fact that theres a test this friday. im ready to die.\r\n\r\nand you know all the science hw we get through your sister." } { "Tag": [ "inequalities proposed", "inequalities" ], "Problem": "a)If $ a,b,c > 0$ such that $ abc \\equal{} 1$ then\r\n\\[ \\frac {a^4}{a^3 \\plus{} b^3} \\plus{} \\frac {b^4}{b^3 \\plus{} c^3} \\plus{} \\frac {c^4}{c^3 \\plus{} a^3}\\geq\\frac {9}{2(a \\plus{} b \\plus{} c)}\r\n\\]\r\nb) If $ a,b,c>0$ such that $ \\frac{1}{a}\\plus{}\\frac{1}{b}\\plus{}\\frac{1}{c}\\equal{}1$, then \\[ \\frac {a^4}{a^3 \\plus{} b^3} \\plus{} \\frac {b^4}{b^3 \\plus{} c^3} \\plus{} \\frac {c^4}{c^3 \\plus{} a^3}\\geq\\frac {9}{2}\\]", "Solution_1": "[quote=\"Marius Mainea\"]Let $ a,b,c > 0$. Prove that:\n\n a) $ \\frac {a^4}{a^3 \\plus{} b^3} \\plus{} \\frac {b^4}{b^3 \\plus{} c^3} \\plus{} \\frac {c^4}{c^3 \\plus{} a^3}\\leq\\frac {a \\plus{} b \\plus{} c}{2}$\n[/quote]\r\nMaybe\r\n$ \\frac {a^4}{a^3 \\plus{} b^3} \\plus{} \\frac {b^4}{b^3 \\plus{} c^3} \\plus{} \\frac {c^4}{c^3 \\plus{} a^3}\\geq\\frac {a \\plus{} b \\plus{} c}{2}$ $ ?$ :wink:", "Solution_2": "[quote=\"arqady\"][quote=\"Marius Mainea\"]\n[/quote]\nMaybe\n$ \\frac {a^4}{a^3 \\plus{} b^3} \\plus{} \\frac {b^4}{b^3 \\plus{} c^3} \\plus{} \\frac {c^4}{c^3 \\plus{} a^3}\\geq\\frac {a \\plus{} b \\plus{} c}{2}$ $ ?$ :wink:[/quote]\r\n\r\n Yes ,you are right. :|", "Solution_3": "[quote=\"Marius Mainea\"]Let $ a,b,c>0$. Prove that: \na)$ \\frac {a^4}{a^3 \\plus{} b^3} \\plus{} \\frac {b^4}{b^3 \\plus{} c^3} \\plus{} \\frac {c^4}{c^3 \\plus{} a^3}\\geq\\frac {a \\plus{} b \\plus{} c}{2}$[/quote]\r\nSimilar problem posted previously.\r\nUse $ H\\ddot{o}lder$ on $ L.H.S.$. Followed by some generalizations. :wink:", "Solution_4": "[quote=\"Sunkern_sunflora\"][quote=\"Marius Mainea\"]Let $ a,b,c > 0$. Prove that: \na)$ \\frac {a^4}{a^3 \\plus{} b^3} \\plus{} \\frac {b^4}{b^3 \\plus{} c^3} \\plus{} \\frac {c^4}{c^3 \\plus{} a^3}\\geq\\frac {a \\plus{} b \\plus{} c}{2}$[/quote]\nSimilar problem posted previously.\nUse $ H\\ddot{o}lder$ on $ L.H.S.$. [/quote]\r\nHow we can use Holder? I don't see it. :maybe: Explain please. Thank you!", "Solution_5": "[quote=\"arqady\"][quote=\"Sunkern_sunflora\"][quote=\"Marius Mainea\"]Let $ a,b,c > 0$. Prove that: \na)$ \\frac {a^4}{a^3 \\plus{} b^3} \\plus{} \\frac {b^4}{b^3 \\plus{} c^3} \\plus{} \\frac {c^4}{c^3 \\plus{} a^3}\\geq\\frac {a \\plus{} b \\plus{} c}{2}$[/quote]\nSimilar problem posted previously.\nUse $ H\\ddot{o}lder$ on $ L.H.S.$. [/quote]\nHow we can use Holder? I don't see it. :maybe: Explain please. Thank you![/quote]\r\nIt 's here\r\nhttp://www.mathlinks.ro/viewtopic.php?t=216009&sid=eee3567d29fe4d8663712a426107f404\r\nHi arqady; Can U post your proof ?" } { "Tag": [ "inequalities", "inequalities unsolved" ], "Problem": "Let a,b,c are positive real numbers such that $ \\left\\{ \\begin{array}{l}0 < a < b \\\\b^2 \\minus{} 4ac \\le 0 \\\\ \r\n \\end{array} \\right.$\r\n\r\nFind the min value of $ S \\equal{} \\frac{{a \\plus{} b \\plus{} c}}{{b \\minus{} a}}$", "Solution_1": "hello, prove that $ \\frac{a\\plus{}b\\plus{}c}{b\\minus{}a}\\geq3$.\r\nSonnhard.", "Solution_2": "[quote=\"Dr Sonnhard Graubner\"]hello, prove that $ \\frac {a \\plus{} b \\plus{} c}{b \\minus{} a}\\geq3$.\nSonnhard.[/quote]\r\n\r\nHello!\r\nThis is problem 5 in Gifted school examination of my city. Can u solve this problem detailly? I need a detail solution! :)\r\nThanks so much! ;)\r\n\r\nL_Euler.", "Solution_3": "The answer of [b]Dr Sonnhard Graubner[/b] is right. Now I will solve it in detail.\r\nConsider $ P(x) \\equal{} ax^2 \\minus{} bx \\plus{} c$. Because $ b^2 \\minus{} 4ac\\le 0$ and $ a > 0$ therefore $ P(x)\\ge 0$ for all real number $ x$. Put $ x \\equal{} 2$, we will get $ 4a \\minus{} 2b \\plus{} c\\ge 0$, which is also means that $ a \\plus{} b \\plus{} c\\ge 3(b \\minus{} a)$, or $ \\frac {a \\plus{} b \\plus{} c}{b \\minus{} a}\\ge 3$. Then you will get the result :lol:", "Solution_4": "equality holds if $ a\\equal{}1,b\\equal{}4,c\\equal{}4$" } { "Tag": [ "quadratics" ], "Problem": "I was recently doing a problem that related to quadratic equations.\r\n\r\nI am asked to complete the square and solve.\r\n\r\nSolve: $x^{2}-2x-9 = 0$\r\n\r\nMy answers:\r\n$x$ = $\\sqrt{10}-1$\r\n$x$ = $-\\sqrt{10}-1$\r\n\r\nHowever, those were said to be incorrect.\r\n\r\nThe correct answers are\r\n\r\n$x = 1+\\sqrt{10}$\r\nand\r\n$x = 1-\\sqrt{10}$\r\n\r\nWhat did I do wrong?\r\n\r\n--------\r\n\r\n1) $x^{2}-2x = 9$\r\n\r\n2) $\\frac{1}{2x}*-2x$\r\n\r\n3) $x^{2}-2x+1 = 9+1$\r\n\r\n4) $(x-1)(x-1) = 10$\r\n\r\n5) $\\sqrt{x-1}^{2}= \\sqrt{10}$\r\n\r\n6) $x-1 = \\sqrt{10}$\r\n\r\nFrom there, I started solving.", "Solution_1": "You are mostly correct, but I I have a few comments:\r\n\r\n$\\sqrt[]{{x-1}^{2}}= \\pm \\sqrt[]{10}$, not just $\\sqrt[]{10}$\r\n\r\nIf $x-1 = \\pm \\sqrt[]{10}$, then $x = 1 \\pm \\sqrt[]{10}$, giving the correct answers. Maybe you just made a mistake in solving for x in the last step?", "Solution_2": "[hide=\"hint\"]in step 5,just remeber that\n$\\sqrt{(x-1)^{2}}=|x-1|$\nso \n$x-1=\\sqrt{10}$\nand\n$x-1=-\\sqrt{10}$[/hide]", "Solution_3": "Why is the $1$ put before the $\\sqrt{10}$? \r\nThat is what I do not understand.\r\n\r\nIf I am isolating $x$, then I need to add the one to both sides...\r\n\r\nAh. I subtracted instead of added...\r\n\r\n*Points finger to head*\r\n*Bang*", "Solution_4": "\\begin{eqnarray*}x^{2}-2x-9 &=& 0 \\hfill \\\\ (x^{2}-2x+1) &=& 10 \\hfill \\\\ (x-1) &=& \\pm \\sqrt{10}\\hfill \\\\ x_{1}&=& 1+\\sqrt{10}\\hfill \\\\ x_{2}&=& 1-\\sqrt{10}\\hfill \\\\ \\end{eqnarray*}" } { "Tag": [ "geometry", "3D geometry", "geometric transformation", "rotation" ], "Problem": "There are six uncoloured wooden cubes of equal size and three different colours. On each of the cubes exactly three sides shall be completely covered with one colour. On each of the cubes all three colours shall be used.\r\n\r\nIs it possible, to colour the six cubes as mentioned above, such that each cube can be distinguished from each of the other ones ?\r\n\r\nHint: Two coloured cubes are not to be distinguished if they can be transformed into each other by means of rotations and/or translations.", "Solution_1": "Yes; paint as follows, letting the three colors be red, blue, and green for simplicity. The locations of the colors don't even matter...once again, this seems a bit easy: (spoiler)\n\n\n\n[hide]3 red, 2 blue, 1 green\n\n3 red, 2 green, 1 blue\n\n3 blue, 2 red, 1 green\n\n3 blue, 2 green, 1 red\n\n3 green, 2 red, 1 blue\n\n3 green, 2 blue, 1 red[/hide]" } { "Tag": [ "AMC", "AIME" ], "Problem": "A stone is dropped into a well and the report of the stone striking the bottom is heard $ 7.7$ seconds after it is dropped. Assume that the stone falls $ 16t^2$ feet in $ t$ seconds and that the velocity of sound is $ 1120$ feet per second. The depth of the well is:\r\n\r\n$ \\textbf{(A)}\\ 784 \\text{ ft.} \\qquad\\textbf{(B)}\\ 342 \\text{ ft.} \\qquad\\textbf{(C)}\\ 1568 \\text{ ft.} \\qquad\\textbf{(D)}\\ 156.8 \\text{ ft.} \\qquad\\textbf{(E)}\\ \\text{none of these}$", "Solution_1": "...is AHSME equiv to AMC or AIME?\r\n\r\n[hide=\"solution\"]\n$ d \\equal{} 16t_1^2 \\qquad(1)$\n$ d \\equal{} 1120t_2 \\qquad(2)$\n\n$ t_1 \\plus{} t_2 \\equal{} \\frac {d}{1120} \\plus{} \\frac {\\sqrt {d}}{4} \\equal{} 7.7$\n\nLet $ d \\equal{} a^2$\n\n$ a^2 \\plus{} 280a \\minus{} 8624 \\equal{} 0 \\Leftrightarrow (a \\minus{} 28)(a \\plus{} 308)$\n$ d \\equal{} 28^2 \\equal{} 784$\n\nAnswer $ \\rightarrow \\boxed{A}$\n\nAre you allowed calculators on this exam?[/hide]", "Solution_2": "AHSME was the equivalent of AMC and AIME together until about 1984, when it was \"downgraded\" to only AMC because of the introduction of the AIME.\r\nAnd the first pocket calculator was invented in 1967, so I don't think calculators were allowed. :D", "Solution_3": "We just have to solve for $ t$ in $ 1120(7.7 \\minus{} t) \\equal{} 16t^2$ and then plug that into $ 16t^2$ to get the answer. As a side note, the acceleration of the stone is taken to be $ 32$ feet per second.\r\n\r\nBy the way, how many problems are there on the AHSME?\r\n\r\nEDIT: Sorry, I messed up and have corrected it; I have no idea why I integrated instead of differentiating...", "Solution_4": "mathwizard, the physics equation is $ \\delta x \\equal{} 1/2at^2 \\plus{} v_0 t$ so the acceleration is $ 32$ feet per second, which is commonly used as the Earth's acceleration due to gravity.", "Solution_5": "[quote=\"mathwizarddude\"]We just have to solve for $ t$ in $ 1120(7.7 \\minus{} t) \\equal{} 16t^2$ and then plug that into $ 16t^2$ to get the answer. As a side note, the acceleration of the stone is taken to be $ 32$ feet per second.\n\nBy the way, how many problems are there on the AHSME?\n\nEDIT: Sorry, I messed up and have corrected it; I have no idea why I integrated instead of differentiating...[/quote]\n[quote=\"facis\"]mathwizard, the physics equation is $ \\delta x \\equal{} 1/2at^2 \\plus{} v_0 t$ so the acceleration is $ 32$ feet per second, which is commonly used as the Earth's acceleration due to gravity.[/quote]For the sake of accuracy, acceleration is feet per second [b]squared[/b] or $ \\text{ft}/\\text{sec}^2$", "Solution_6": "oops... stupidity at its best, I guess. ft/s^2 sounds better... And while I'm at it, I meant to use a $ \\Delta$ not a $ \\delta$." } { "Tag": [ "function", "limit", "real analysis", "real analysis unsolved" ], "Problem": "Let $f: X\\rightarrow Y$ be a continuous function and let $E$ be dense in $X$. Prove that $f\\left(E\\right)$ is dense in $f\\left(X\\right)$.", "Solution_1": "Fix $y\\in f(Y)$. Exist $x\\in X$ such that $f(x)=y$ and sequence $\\{e_{n}\\}\\subset E$ such that $\\lim{e_{n}}=x$. Final $\\{f(e_{n})\\}\\subset f(E)$ and $\\lim{f(e_{n})}=f(x)=y$.", "Solution_2": "If we're in a general topological space, note that $f^{-1}(\\overline{f(E)})$ is a closed subset containing $E$, hence containing $\\overline{E}= X$, so $f(X) \\subset \\overline{f(E)}$." } { "Tag": [ "topology", "real analysis", "real analysis unsolved" ], "Problem": "Family {$V_ {\\alpha}$} of open subsets of $X$ is called [i]basis[/i] for $X$, if for arbitrary $x \\in X$ and arbitrary $G \\subset X$ such that $x \\in G$ there exists $V_ {\\alpha}$ such that $x \\in V _{\\alpha} \\subset G$. In other words: every open subset of $X$ is sum of some sets of {$V_ {\\alpha}$}.\r\n\r\nProve that every metric space which contains countable dense subset has countable basis.", "Solution_1": "Take the open balls with radii $\\frac 1n$ centered at the countably many points in the dense countable set.", "Solution_2": ":first: perfect solution :) - it is THAT easy though I like it" } { "Tag": [ "calculus", "integration", "geometry", "trigonometry", "calculus computations" ], "Problem": "In the $ x \\minus{} y$ plane, find the area of the region bounded by the parameterized curve as follows.\r\n\r\n$ \\left\\{\\begin{array}{ll} x \\equal{} \\cos 2t & \\quad \\\\\r\ny \\equal{} t\\sin t & \\quad \\end{array} \\right.\\ (0\\leq t\\leq 2\\pi)$", "Solution_1": "Can anyone solve the problem? :)", "Solution_2": "[quote=\"kunny\"]In the $ x \\minus{} y$ plane, find the area of the region bounded by the parameterized curve as follows.\n\n$ \\left\\{\\begin{array}{ll} x \\equal{} \\cos 2t & \\quad \\\\\ny \\equal{} t\\sin t & \\quad \\end{array} \\right.\\ (0\\leq t\\leq 2\\pi)$[/quote]\r\n\r\nThe curve is not simple, it intersects itself at the point (1,0). So break it up into\r\n\r\n$ C_1: \\vec{r}(t)\\equal{}< \\cos 2t, t\\sin t>,\\mbox{ for }0\\leq t\\leq \\pi$ (Negatively orientated)\r\n\r\n\r\n$ C_2: \\vec{r}(t)\\equal{}< \\cos 2t, t\\sin t>,\\mbox{ for }\\pi\\leq t\\leq 2\\pi$ (Positively orientated)\r\n\r\nGreen's Theorem gives the area bounded by the curve as\r\n\r\n\\[ A\\equal{}\\minus{}\\oint_{\\minus{}C_1\\cup C_2} y\\, dx \\equal{} \\minus{}2\\int_{0}^{\\pi}t\\sin t \\sin 2t\\, dt\\plus{}2\\int_{\\pi}^{2\\pi}t\\sin t \\sin 2t\\, dt\\]\r\n\r\nreplace $ t$ with $ t\\minus{}\\pi$ in the second integral to get \r\n\r\n\\[ A\\equal{} \\minus{}2\\int_{0}^{\\pi}t\\sin t \\sin 2t\\, dt\\minus{}2\\int_{0}^{\\pi}(t\\minus{}\\pi)\\sin t \\sin 2t\\, dt\\]\r\n\\[ \\equal{} \\minus{}4\\int_{0}^{\\pi}t\\sin t \\sin 2t\\, dt\\plus{}2\\pi\\int_{0}^{\\pi}\\sin t \\sin 2t\\, dt\\]\r\n\\[ \\equal{} 2\\int_{0}^{\\pi}(\\pi \\minus{}2t)\\sin t \\sin 2t\\, dt\\equal{} 4\\int_{0}^{\\pi}(\\pi \\minus{}2t)\\sin^2 t \\cos t\\, dt\\]\r\n\r\nnow by parts with $ u\\equal{}\\pi\\minus{}2t$\r\n\r\n\\[ \\equal{} \\left[ \\frac{4}{3}(\\pi \\minus{}2t)\\sin^3 t\\right]_{t\\equal{}0}^{\\pi}\\plus{}\\frac{8}{3} \\int_{0}^{\\pi}\\sin^3 t\\, dt\\]\r\n\\[ \\equal{}\\frac{8}{3} \\int_{0}^{\\pi}\\left(\\sin t\\minus{}\\sin t\\cos^2 t\\right)\\, dt\\]\r\n\\[ \\equal{}\\frac{8}{3}\\left[\\minus{}\\cos t\\plus{}\\frac{1}{3}\\cos^3 t \\right]_{t\\equal{}0}^{\\pi}\\equal{}\\boxed{\\frac{32}{9}}\\]", "Solution_3": "That's correct answer.\r\n\r\nThis problem seems to be difficult for applicants for Tokyo University, because Japanese high school students don't study Green's theorem." } { "Tag": [ "MATHCOUNTS" ], "Problem": "Discuss our plans to win the Spirit Award and win the Team Round... MUAHAHAHAHA :rotfl: :rotfl: :rotfl:", "Solution_1": "You just lost the Spirit Award. Thank you, have a good nats trip.", "Solution_2": "The four MO team people please post your heights here.", "Solution_3": "shhhhhhhhhhhhhhhhhhhhhhhhhhhhhhhhhhh............................\r\nwe don't want to hint at our plan....\r\n\r\nAlthough asking for heights doesn't give away much...", "Solution_4": "Just letting you guys know that I am going to be 10-15 minutes late for Mathcounts Practice at Mewto's house tomorrow night...", "Solution_5": "Okay. I'll have little RomanianGenius go tell mom.", "Solution_6": "Something tells me you guys will win the Spirit Award...\r\n\r\nUnless the CA team puts on some epic rickroll...*COUGH MILLER4MATH COUGH*", "Solution_7": "Although there has been a horrendous amount of spam and useless posts/threads in this forum, I would like to say congratulations to everyone on the Missouri national MC team, being a former Missouri competitor myself. As everyone knows, Missouri's best rank was second as a team; since all four of you are from Ladue, please, [b]please[/b] make Missouri proud (as well as St. Louis) by at least getting first :D . After all, you all live right next to each other, so there's no reason to not get enough practice in, and overall, do your best, congratulations, and make sure at least half the team gets into the Countdown Round.", "Solution_8": "Thanks Michael, we will try to make you proud by being the first team to be kicked out of National MC.\r\n\r\nIs that good?", "Solution_9": "Mewto's response = WIN.", "Solution_10": "I don't think that's what he meant by first...\r\n\r\n...and I hope you were joking.", "Solution_11": "Oh dang I totally just remembered. My first ever official coutndown was when ZzZzZ...Z wiped the floor with me and Sam at chapter. He buzzed in like .5 sec and the fail judge guy read the whole problem before calling on him to give the answer :huh:", "Solution_12": "[size=200]EPIC WIN....KANSAS GOT FIRST ONE YEAR AT NATS\n\nMISSOURI = EPIC UBER PHAILURE\n\nKANSAS = EPIC UBER WIN[/size]", "Solution_13": "So that makes the mean place over your 25 years of MC what, 57x24+1 /25=54.76", "Solution_14": "No Mewto. We definitely beat the Virgin Islands and Guam at least 3 times.", "Solution_15": "Oh. Well then. It would appear my maths is flawed.", "Solution_16": "Perhaps your math is just too beastly for mankind to grasp.\r\n\r\nSO BEASTLY MMM", "Solution_17": "This would explain why I am currently failing pre-algebra...", "Solution_18": "This would explain why I am currently failing pre-algebra...", "Solution_19": "[quote=\"Mewto55555\"]Oh dang I totally just remembered. My first ever official coutndown was when ZzZzZ...Z wiped the floor with me and Sam at chapter. He buzzed in like .5 sec and the fail judge guy read the whole problem before calling on him to give the answer :huh:[/quote]\r\n\r\nAnd then I believe I lost to Shuyang... Strange how being up against Ladue kids gives you motivation to do well, but then with no more strong competition, the mind just goes blank...", "Solution_20": "[quote=\"ZzZzZzZzZzZz\"][quote=\"Mewto55555\"]Oh dang I totally just remembered. My first ever official coutndown was when ZzZzZ...Z wiped the floor with me and Sam at chapter. He buzzed in like .5 sec and the fail judge guy read the whole problem before calling on him to give the answer :huh:[/quote]\n\nAnd then I believe I lost to Shuyang... Strange how being up against Ladue kids gives you motivation to do well, but then with no more strong competition, the mind just goes blank...[/quote]\r\n\r\nYeah and then you lost first round at state too, and then the same thing happened at nationals :rotfl:", "Solution_21": "[quote=\"Mewto55555\"][quote=\"ZzZzZzZzZzZz\"][quote=\"Mewto55555\"]Oh dang I totally just remembered. My first ever official coutndown was when ZzZzZ...Z wiped the floor with me and Sam at chapter. He buzzed in like .5 sec and the fail judge guy read the whole problem before calling on him to give the answer :huh:[/quote]\n\nAnd then I believe I lost to Shuyang... Strange how being up against Ladue kids gives you motivation to do well, but then with no more strong competition, the mind just goes blank...[/quote]\n\nYeah and then you lost first round at state too, and then the same thing happened at nationals :rotfl:[/quote]\r\n\r\nAwful times... Awful times..." } { "Tag": [ "function", "algebra proposed", "algebra" ], "Problem": "Let a differentiable function $f: \\mathbb{R} \\rightarrow \\mathbb{R}$ that follows the conditions:\r\na) $f(0) = 0$ and $f(2) = 2$.\r\nb) For all $a \\in \\mathbb{R} - \\{0\\}$ the tangent line to $f$ on $P(a,f(a))$ meets x-axis on $A$ and y-axis on $B$ such that $A$ is the line-bissector of $\\overline{BP}$.\r\n\r\nFind $f(3)$.", "Solution_1": "Solve the differetial equation $f'(x)=\\frac{2}{x}f(x)$ and $f(2)=2,$ yielding $f(x)=\\frac{1}{2}x^2.$\r\nThus $f(3)=\\frac{9}{2}.$" } { "Tag": [], "Problem": "Be a,b,c roots of $P(x)x^3+px^2+qx+r=0$ . If $S_n=a^n+b^n+c^n$, n is integer and n>3, being\r\n $K=S_n+pS{}_n{}_-{}_1+qS{}_n{}_-{}_2$ Find K", "Solution_1": "what are you asking? $K$ in terms of $S$? if thats the case figure out $S_1, S_2, S_3$ using relationships btwn roots and coefficients.\r\n\r\nthen write $x^3=-px^2-qx-r$\r\nso $x^4=-px^3-qx^2-rx$.\r\n\r\nso then $S_4=-pS_3-qS_2-rS_1$ and continue like this so you have\r\n\r\n$S_n=-pS_{n-1}-qS_{n-2}-rS_{n-3}$, so $S_n+pS_{n-1}+qS_{n-2}=-rS_{n-3}=K$." } { "Tag": [ "probability" ], "Problem": "Is 1.9999999999999999999... recurring a rational number?\r\n\r\n[hide=\"working\"]let\nx=1.999...\n10x=19.9999\n9x=18\nx=2\n2 is rational\nTherefore as x=2, x is rational[/hide]\n\n[hide=\"ambiguity of the solution\"]In this case we proved that 2 is rational and when x is 2, x is rational, but in the original assumption, we said x=1.999...\nso in a way we did not prove that 1.999... is rational at the end.\nif this is the case, we can never prove that it is rational\ntherefore it is not rational (?)\n\nreasoning seems flawless but logic doesn't seem to tie up for either cases???[/hide]", "Solution_1": "It is very well known that $ 0.9999999...\\equal{}1$.\r\nIt is not smaller than 1 by some infinitesimal value; this is a common misconception.\r\n\r\n[hide=\"Proof\"]\nLet $ x\\equal{}0.99999...$\nThen $ 10x\\equal{}9.99999...$\nAnd $ 10x\\minus{}x\\equal{}9x\\equal{}9$.\nDividing both sides by $ 9$, we get $ x\\equal{}1$.\n\nTherefore, $ 0.99999...\\equal{}1$\n[/hide]\r\n\r\nTherefore, $ 1.99999...\\equal{}2$, which is obviously rational.", "Solution_2": "There exists the following remarkable result:\r\n\r\n$ 0.aaaaa...\\equal{}\\frac{a}{9}$, where $ a$ $ \\in$ $ \\{ 0,1,2,3,4,5,6,7,8,9 \\}$.", "Solution_3": "I don't know if I'd call it \"remarkable\" :wink:\r\n\r\nI think you can generalize it for all $ n\\in\\mathbb Z$.\r\nIf $ n \\equal{} \\overline{a_0a_1\\cdots a_m}$, and we want the fractional form of the recurring decimal expansion $ 0.\\overline{a_0a_1\\cdots a_m}$, we let $ x$ be that value, multiply $ x$ by $ 10^{m \\plus{} 1}$, and subtract $ x$ away to get: $ x(10^{m\\plus{}1}\\minus{}1)\\equal{}n$\r\n\r\nAnd therefore, the fractional form is $ \\frac {n}{10^{m \\plus{} 1} \\minus{} 1}$.\r\n\r\nIf $ n \\equal{} a$, $ m \\equal{} 0$, and the fraction is $ \\frac {a}{9}$, as expected.", "Solution_4": "I want to address a different step in your logic.\r\n\r\nWe can never prove that it is rational\r\nTherefore, it is not rational.\r\n\r\nThe above two steps rely on the false assumption that if something is true, it can be proven. See [url=http://en.wikipedia.org/wiki/Godel%27s_incompleteness_theorem]Godel's Incompleteness Theorem[/url].\r\n\r\nIn other words, the provability of a statement does not imply anything about that statement's truth." } { "Tag": [ "modular arithmetic" ], "Problem": "What is the smallest possible value of $\\left|12^{m}-5^{n}\\right|$, where $m$ and $n$ are positive integers?", "Solution_1": "The value can't be zero, by the fundamental theorem of arithmetic. Also, note that $|12^{m}-5^{n}|$ can't be divisible by $2,3$ or $5$, hence the expression can't be equal to $2,3,4,5,6$. Assume it can be equal to $1$. Then either $12^{m}-1=5^{n}$ or $5^{n}-1=12^{m}$. The first case is impossible as LHS is divisible by $12-1=11$ and RHS is not. The second case, for modulo $6$, gives $(-1)^{n}-1\\equiv 0\\pmod{6}\\implies n=2k$. Hence $25^{k}-1=12^{m}$. Since the unit digit of LHS is $4$, that gives $m=4l+2$, hence $25^{k}-1=12^{4l+2}$. Now RHS is divisible by $9$, hence $25^{k}\\equiv(-2)^{k}\\equiv 1\\pmod{9}$, which gives $k=3s$. Therefore $25^{3s}-1=12^{4l+2}$. However, the LHS is divisible by $25^{3}-1=(25-1)(625+25+1)=24\\cdot 651=2^{3}\\cdot 3^{2}\\cdot 7\\cdot 31$, and RHS isn't divisible by either $7$ or $31$. Therefore the initial expression can't be equal to $1$. It can be equal to $7$ for $m=n=1$, thus we conclude that $7$ is its minimal value." } { "Tag": [ "superior algebra", "superior algebra unsolved" ], "Problem": "Let $p$ be a prime number. Show that $\\sigma\\in S_n$ is of order $p$ if and only if $\\sigma$ can be written as products of commuting $p$-cycles.", "Solution_1": "just look at cycle structure." } { "Tag": [ "geometry", "rectangle", "perimeter", "symmetry", "geometry solved" ], "Problem": "A rectangle has a perimeter of 15cm. What is it`s maximum volume if the rectangle starts spinning around one of it`s sides shaping a cylinder?\r\n\r\nI assume one side is: x\r\nI assume one side is: x\r\nThe other side is : (15-2x)/2\r\nThe other side is : (15-2x)/2\r\n\r\nA cylinders volume is : \u03c0*r^2*h\r\nWhere \u03c0=3.14159265359\r\nr: is half of one side as one whole side becomes the cylinders diametre when it starts spinning.\r\nand height is a full opposite site.\r\n\r\nI assume it starts spinning on the side (15-2x)/2 which becomes the cylinders diametre and x its hight:\r\nBut the forumla for a cylinders volume needs the radius which is: ((15-2x/2)/2) = (15-2x)/4\r\n\r\nThe formula for the cylinders volume then is: \u03c0*x((15-2x)/4)^2\r\n\r\nthe Above calculation results in a volume for the cylinder as :\r\n(225\u03c0x) - (60\u03c0x^2)+(4\u03c0x^3) /16\r\n\r\nAs we want to know the maximum volume we derive(typo) it:\r\n\r\nV`=37,699x^2 - 376,991x + 706,858\r\nThis is a simple 2nd degree equation resulting in x1=(7,5) x2=2,5\r\n\r\nNow to get the maximum volume we put 2,5 into the Volume formula since this is a maximum:\r\n\r\n(225\u03c0*2,5) - (60\u03c0*2,5^2) - (4\u03c02,5^3) / 16 = 49,08\r\n\r\nBUT HERE is my problem: The answer ought to be 196 and therefor (225\u03c0*2,5) - (60\u03c0*2,5^2) - (4\u03c02,5^3) / [b]4[/b] \r\nis the correct volume formula for the cylinder.\r\n\r\nBut since one side of the original rectangle is (15-2x/2) then the raidus for the cylinder is ((15-2x/2)/2) \r\n\r\nMy fault accurs as I get the forumla for the cylinder to be split by 16, it should be 4, can anyone see where in the transitiation/calculation from rectangle to cylinder and coverting diametre to a radius I do wrong since the above calculation is correct except the volume is to be split by 4 and not as I get 16.\r\n\r\nThanks in advance, Stephan", "Solution_1": "[quote=\"gossen19\"]r: is half of one side as one whole side becomes the cylinders diametre when it starts spinning...\n[/quote]\r\nNo, cylinder radius is one whole side. The rectangle is not spinning around one of its symmetry axes, but around one of its sides.", "Solution_2": "Ah right, ofcourse! thanks :>" } { "Tag": [], "Problem": "http://www.youtube.com/watch?v=VjgidAICoQI\r\nWhat are your opinions?", "Solution_1": "Neal Adams is a comic book artist, not a scientist.\r\n\r\nhttp://en.wikipedia.org/wiki/Expanding_earth_theory", "Solution_2": "A takedown of Neal Adams is available [url=http://scienceblogs.com/goodmath/2006/12/wacky_physics_it_must_be_right_1.php]here[/url]." } { "Tag": [ "Princeton", "college" ], "Problem": "I just took the SAT 1. \r\nhope to get a 800 in math. does any one remember a mathquestion where a stupid mistake could be done?\r\n\r\nin the past one i got 1 question wrong.... i always do silly mistakes.. :?", "Solution_1": "To be honest, I didn't see any today that may have caused stupid mistakes? I mean...I suppose there was that one question with the chart and it asked for the number of books sold in a certain week...(this better not count as revealing questions) I mean you could've read the chart wrong...but otherwise...eh...", "Solution_2": "true,\r\ni think the answer to this oen was 46,000 if i'm not mistaken...\r\ni think i was careful to read the chart well (i am the kind of person that could have read it wrong)", "Solution_3": "[quote=\"manuel\"]true,\ni think the answer to this oen was 46,000 if i'm not mistaken...\ni think i was careful to read the chart well (i am the kind of person that could have read it wrong)[/quote]\r\n\r\nDifferent problem.", "Solution_4": "What about SAT II, how'd it go for ya!!!", "Solution_5": "i almost missed the $m^2-n^2=12$, find values of $(m-n)$ question. then i realized 1 didn't work.", "Solution_6": "what answer did u selected inthis one..\r\n2 right?", "Solution_7": "[quote=\"Rushil\"]What about SAT II, how'd it go for ya!!![/quote]\r\n\r\nMath IIC was a piece of cake.\r\n\r\nPhysics... was another story. I left 4 blank, which might kill my 800. I hope not, though, there's just too much material to study and formulas to memorize. The problems were easier than the AP test, but they don't give you formulas which is stupid; I forgot some of the waves/sound/optics ones.", "Solution_8": "[quote=\"manuel\"]what answer did u selected inthis one..\n2 right?[/quote]\r\n :( now i realize only 1 is right. yes i did put 2 right. there goes the 800.", "Solution_9": "oh... so u got 790? that's good too,\r\nbut wait i don't get it, what were the possible answers to this question?\r\n\r\nbtw, i wrote the best essay i've done in my life!!! so i hope toget a good writing score too", "Solution_10": "Edit: Oops. Forgot that you can't reveal questions. ;)\r\n\r\nThat retarded essay! Stopped in mid-sentence on my third paragraph!", "Solution_11": "[quote=\"Forsaken\"]Edit: Oops. Forgot that you can't reveal questions. [/quote]\r\ndon't worry. we won't tell anyone. ;)\r\n\r\nand i wonder how they will like an essay that isn't divided into paragraphs. i just couldn't find a spot where my ideas transitioned where paragraphs were needed. o well.", "Solution_12": "[quote=\"paladin8\"]\n\nPhysics... was another story. I left 4 blank, which might kill my 800. I hope not, though, there's just too much material to study and formulas to memorize. The problems were easier than the AP test, but they don't give you formulas which is stupid; I forgot some of the waves/sound/optics ones.[/quote]\r\n\r\nit has a nice curve. i got an 800 after guessing on 4 or 5, and im sure i missed a few more than that.", "Solution_13": "Ugh. Essay. :wallbash_red: I don't GET IT. I wrote for the entire 25 minutes, right? And all I got was a page and a half in normal writing. But the girl next to me? (I glanced over after we were closing the tests up) She wrote a FULL TWO PAGES... and her writing was [size=59]tiny[/size]. HOW? How do people [i]do that????[/i]", "Solution_14": "I wrote two full pages because i did an essay in the same topic for school...that's good luck :D", "Solution_15": "writing two pages shouldn't be that difficult. i spent the first 10 minutes trying to decide which side to argue and then come up with details. so i wrote a page and a quarter in around 15 minutes. 2 pages isn't out of the question at all.", "Solution_16": "If I didn't even start my conclusion, what's the max score I can get? If I had finished, \r\nI think I would have gotten a 10, so how much would that be without a conclusion?\r\n\r\nMan, I just realized that I have at least 2 wrong on math and 3 wrong on CR. I hope I can pull a 2200. :(", "Solution_17": "Lol unless you guys are seniors, there's really no need to worry. Plenty of more opportunities.\r\n\r\nAs for the essay, note that some people just memorize a format and plug in the essay to an outline they've already decided on. Lol. It's very difficult to just see a question and start writing madly for 25 minutes unless you do that, especially if you're trying to get a full 2 pages at reasonable size.\r\n\r\nEssay scoring depends on the length of the essay as well. So unless you've written more than 3/2 pages, an 11-12 is out of the question and a 9-10 is pushing it. Otherwise, it just matters on how coherent your essay is and whether or not you've used correct syntax, good vocab, and supported your thesis. My two, unprofessional cents on that matter...", "Solution_18": "I am a senior. :( \r\n\r\nAt least I know for sure that the 3 paragraphs I wrote this time are better than the ones I wrote on the March test. (Got a 6/12) :blush:", "Solution_19": "[quote=\"Rushil\"]What about SAT II, how'd it go for ya!!![/quote]\r\n\r\nmath II C >> easy(answered every question) > should get 800\r\n\r\nphysics>> easy >> left 2 blank>> should get 800\r\n\r\nchemistry>> Super easy>> answered every question except one which i think was WRONG!! the choices were incorrect, that would be q 57 i think >> should get 800.\r\n\r\n\r\nthats it. :)", "Solution_20": "Which question are you talking about in Chem (Sat II) that you think is wrong. PM me!!!", "Solution_21": "[quote=\"paladin8\"]\n\nPhysics... was another story. I left 4 blank, which might kill my 800. I hope not, though, there's just too much material to study and formulas to memorize. The problems were easier than the AP test, but they don't give you formulas which is stupid; I forgot some of the waves/sound/optics ones.[/quote]\r\n\r\nwell in the AP test they don't give the formulas up to the FR section either.\r\n\r\nbut SAT physics is way easier than AP test (not that AP test is hard, cause its not!). for answering the MC problem all they ask you in the multiple choice problems is to know the basic knowledge of the subject, thats it. :)", "Solution_22": "For people that took the Physics test, out of curiousity did you guys use a review book? If so, which one(s)? And were they any good?\r\n\r\nSame with chem.", "Solution_23": "i took AP physics B, C and AP chem last year. and got 5's in all of them, so i didn't have anyproblem reviewing stuff, \r\nbut i went to the library and checked out the Kaplans SAT review for chem (which wasn't that good!), and also i looked at the SAT physics book by princeton hall the night before the test, that was pretty good, bettar than the Kaplan one.\r\n\r\noh i forgot to mention that the Kaplan book that i said, was for 03-04. so it was a bid too old, but i don't like Kaplan anyways.\r\n\r\n~amir", "Solution_24": "for chem (which i took a couple years ago), i took the AP test in may (got a 5) and then took the SAT II in June. I was totally out of practice, didn't even go over the stuff til two days before the SAT II, and I still got a 760. i used princeton review, which isnt really the best review book, but i like the way theirs is organized. (you have to be very careful when going over math-type problems, because they most definitely WILL make mistakes).", "Solution_25": "[quote=\"amirhtlusa\"][quote=\"paladin8\"]\n\nPhysics... was another story. I left 4 blank, which might kill my 800. I hope not, though, there's just too much material to study and formulas to memorize. The problems were easier than the AP test, but they don't give you formulas which is stupid; I forgot some of the waves/sound/optics ones.[/quote]\n\nwell in the AP test they don't give the formulas up to the FR section either.\n\nbut SAT physics is way easier than AP test (not that AP test is hard, cause its not!). for answering the MC problem all they ask you in the multiple choice problems is to know the basic knowledge of the subject, thats it. :)[/quote]\r\n\r\nTrue; in general the questions were a lot easier. But the AP test is graded on a ridiculous curve, so if you forget like 5 formulas it doesn't hurt your chances of pulling an easy 5. I don't know about the SAT, though. The curve is supposed to be generous, but how generous is the question.", "Solution_26": "Kaplan = bad review books.\r\n\r\nI would suggets PR like most people, but I also like 5 Steps to a 5 for APs. Or barrons." } { "Tag": [ "calculus", "integration", "geometry", "derivative", "function", "geometric series", "calculus computations" ], "Problem": "Find the area under the curve $ f(x)\\equal{}4x^3 \\plus{} 3x^2 \\plus{} 2x \\plus{}1$ from $ x \\equal{} 0$ to $ x \\equal{} 2$.", "Solution_1": "You need to tell us the value of $ f$ and its first derivative at some point otherwise the answer is not unique.", "Solution_2": "Because of the constants and stuff that are canceled out when we find the derivative? I'll just change the problem. Sorry about that, I'm still learning all this calculus stuff.", "Solution_3": "Do you know how to find the antiderivative of $ f$? If so, evaluate it at $ x \\equal{} 2$ and $ x \\equal{} 0$ and then subtract its value at $ x \\equal{} 0$ from its value at $ x \\equal{} 2$.", "Solution_4": "[hide=\"Solution\"]\nFrom the rule $ f(x)\\equal{}x^{n}\\implies f'(x)\\equal{}nx^{n\\minus{}1}$, and noticing the pattern in the function $ f$ given ($ 4x^3$, etc), the antiderivative is pretty obvious.\n\n$ F(x)\\equal{}\\int f(x)\\; dx\\equal{}x^4\\plus{}x^3\\plus{}x^2\\plus{}x\\plus{}C$\n\nBy the fundamental theorem of calculus, the answer is $ F(2)\\minus{}F(0)\\equal{}\\frac{2^{5}\\minus{}1}{2\\minus{}1}\\minus{}1\\equal{}\\boxed{30\\text{ square units}}$.\n\n(That last step used the sum of a finite geometric series to skip a little calculation).\n[/hide]" } { "Tag": [ "trigonometry", "induction", "LaTeX", "linear algebra", "matrix", "geometry", "ARML" ], "Problem": "My hope is that we can get more interest in proofs by doing a proof marathon. The power question score for SC needs to come up. Anyways, here is the first item that needs to be proven. This is a quite famous identity. I will refrain from naming it in hopes that you guys will try to prove it on your own. Prove: \\[{ ( \\ cos\\theta + i\\sin}\\theta )^n = \\cos n\\theta + i\\sin n\\theta , n\\in {\\rm Z} \\]\r\n\r\n\r\n[hide=\"HINT\"] Show that the identity is true for positive integers first by using mathematical induction[/hide]", "Solution_1": "[hide=\"Hint\"]\nHave you even tried induction yet?[/hide]", "Solution_2": "i don't know if this attempt is strong enough to fully prove it, but it's proof by induction\r\n\r\nshow it works for n=1:\r\n$\\cos\\theta+\\i\\sin\\theta=\\cos\\theta+\\i\\sin\\theta$\r\nassume it works for some n=k:\r\n$(\\cos\\theta+\\i\\sin\\theta)^k=\\cos(k\\theta)+\\i\\sin(k\\theta)$\r\nshow it works for n=k+1:\r\n$(\\cos\\theta+\\i\\sin\\theta)^{k+1}=\\cos((k+1)\\theta)+\\i\\sin((k+1)\\theta)$\r\n$(\\cos(k\\theta)+\\i\\sin(k\\theta))(\\cos\\theta+\\i\\sin\\theta)=$\r\n$\\cos(k\\theta)*\\cos\\theta+i\\sin(k\\theta)*\\cos\\theta+i\\sin(\\theta)*\\cos(k\\theta)-\\sin(k\\theta*\\sin\\theta=$\r\ncombining real and imaginary parts and using trig identies:\r\n$\\cos(k\\theta+\\theta)+i\\sin( k\\theta+\\theta)=$\r\n$\\cos((k+1)\\theta)+i\\sin((k+1)\\theta)$\r\nwhich is what you would get by pluging in k+1 into the equation. this satisfies positive integers. \r\nto show negative integers, joe said to use (cis theta)^n where n is negative (i.e. n=-k for positive k) is the same as 1/(cis theta)^k but i don't feel like doing that right now. maybe shobhit will help me out. i'm $\\text{\\LaTeX}$ed out :(\r\nand if there are any errors in the proof, deal with it and assume i meant to put the correct thing :P", "Solution_3": "I thought I told you *not* to use \\i and just put i? I don't even know what those \\i 's are :? \r\n\r\nWow...you posted exactly what i said over IM. If I knew that was going to be posted here I would have said it more clearly ;)", "Solution_4": "$\\text{Problem 2}$\r\n\r\nLet $\\{F_i\\}_{i=1}^\\infty$ denote the Fibonacci sequence.\r\n\r\nProve that \\[ \\begin{pmatrix} 1 & 1 \\\\ 1 & 0 \\end{pmatrix}^n = \\begin{pmatrix} F_n & F_n\\\\ F_n & F_{n-1}\\end{pmatrix} \\]", "Solution_5": "[quote=\"joml88\"]I thought I told you *not* to use \\i and just put i? I don't even know what those \\i 's are :? [/quote]\r\nwell, some of them came out ok :blush: \r\n\r\nand problem 2 is pretty cool. i did it recently so i'll let someone else tackle it (cough*shobhit*cough)", "Solution_6": "Guys,\r\nFurious made a big \"no no\" in the world of proofs. You cannot start off by equating things you want to prove. You need to start with one side until it looks like the other side of the equation. Now, that is not to say that you can't figure out how to write the proof by equating things you want to show, but when you finally write the proof up, you need to work from one side of an equation until you can get to the other. Otherwise, you are assuming that the claim is already true when it may not be, e.g. Prove 1 = 2.", "Solution_7": "He wrote what he wanted to prove first and then worked entirely on the LHS showing that it is equivalent to the RHS. So I think his proof is OK. It could use a lot of work on neatness/clarity/etc though.", "Solution_8": "Yeah, \r\nI should have looked more closely. To be honest, I did not read the whole thing.", "Solution_9": "[quote=\"furious\"]and problem 2 is pretty cool. i did it recently so i'll let someone else tackle it (cough*shobhit*cough)[/quote]\n[quote=\"furious\"] but i don't feel like doing that right now. maybe shobhit will help me out.[/quote]\r\nFor now, I will watch you guys solve these proofs and learn from your solutions. That is atleast how I improve in certain areas of math, I watch before I attempt! Give me about a month, and in April, a month before ARML, I'll start doing these ;)\r\n\r\nBut if you insist, here's how i would do the negative numbers part, let $x=-y$, so therefore, we have:\r\n\r\n$(cis\\theta)^x=(cis\\theta)^{-y}=((cis\\theta)^y)^{-1}$, if we assume demoivres theorem is true, we have $cisx\\theta=cis{-y\\theta}$ but since $x=-y$, then that holds true for negative numbers...is that good enough?", "Solution_10": "[quote=\"Iversonfan2005\"]\nFor now, I will watch you guys solve these proofs and learn from your solutions. That is atleast how I improve in certain areas of math, I watch before I attempt! Give me about a month, and in April, a month before ARML, I'll start doing these ;)[/quote]\r\n\r\nYou should still start trying now. It's easier for us to tell you when you've done something wrong. Additionally, you learn much more from doing it yourself. Watching someone else prove something is far different from actually doing it yourself.", "Solution_11": "Joe,\r\nI do not think the Fibonacci matrix equation is correct. Take a look, and you should see.", "Solution_12": "Hey guys, Jim here. Mr. R. is finally forcing me to deal with computers. (AHHHH!)\r\n\r\nHere is the proof for problem 2 (note correction):\r\nP(n) = $\\begin{pmatrix} 1 & 1 \\\\ 1 & 0 \\end{pmatrix}^n = \\begin{pmatrix} F_{n+1} & F_n \\\\ F_n & F_{n-1}\\end{pmatrix}$\r\n\r\nWe will argue by induction that this holds true for integers $n>1$\r\nFirst, we show that is is true for $n=2$:\r\n\r\nP(2) = $\\begin{pmatrix} 1 & 1 \\\\ 1 & 0 \\end{pmatrix}^2 = \\begin{pmatrix} 2 & 1 \\\\ 1 & 1\\end{pmatrix} = \\begin{pmatrix} F_3 & F_2 \\\\ F_2 &{ F_1}\\end{pmatrix}$\r\n\r\nNow, assume this is true for $n=k$ i.e.\r\n\r\nP(k) = $\\begin{pmatrix} 1 & 1 \\\\ 1 & 0 \\end{pmatrix}^k = \\begin{pmatrix} F_{k+1} & F_k \\\\ F_k & F_{k-1}\\end{pmatrix}$\r\n\r\nNow we plug in $k+1$\r\n\r\nP(k+1) = $\\begin{pmatrix} 1 & 1 \\\\ 1 & 0 \\end{pmatrix}^{k+1} = \\begin{pmatrix} 1 & 1 \\\\ 1 & 0 \\end{pmatrix}^{k} \\begin{pmatrix} 1 & 1 \\\\ 1 & 0 \\end{pmatrix} = \\begin{pmatrix} F_{k+1} & F_k \\\\ F_k & F_{k-1}\\end{pmatrix} \\begin{pmatrix} 1 & 1 \\\\ 1 & 0 \\end{pmatrix} = \\begin{pmatrix} F_{k+1} + F_{k+1} & F_k \\\\ F_k + F_{k-1} & F_k\\end{pmatrix} = \\begin{pmatrix} F_{(k+1)+1} & F_{k+1} \\\\ F_{k+1} & F_{k}\\end{pmatrix}$\r\n\r\nSo $P(k) \\Rightarrow P(k+1)$ and hence, by PMI, the proposition is true for all integers $n>1$", "Solution_13": "Here is Problem 3, sticking with matrices:\r\n\r\nProve:\r\n\r\n$\\begin{pmatrix} 2 & 1 \\\\ 0 & 1 \\end{pmatrix}^n = \\begin{pmatrix} {2^n} & 2^n-1 \\\\ 0 & 1 \\end{pmatrix}$\r\n\r\nfor all positive integer values of n.\r\n\r\nAlso, determine whether or not this holds true for $n=-1$ :ninja:", "Solution_14": "[hide=\"Solution\"]\nWe begin with the base case, $n=1$:\n\n\\[ \\begin{bmatrix} 2 & 1 \\\\ 0 & 1\\end{bmatrix}^1 = \\begin{bmatrix} 2^1 & 2^1-1 \\\\ 0 & 1\\end{bmatrix} \\]\n\nwhich obviously holds.\n\nNow, we assume that the assertion is true for $n=k$:\n\n\\[ \\begin{bmatrix} 2 & 1 \\\\ 0 & 1 \\end{bmatrix}^{k} = \\begin{bmatrix} 2^k & 2^k-1\\\\ 0 & 1\\end{bmatrix}. \\]\n\nWe proceed to show that it holds for $n=k+1$:\n\n\\[ \\begin{eqnarray*} \\begin{bmatrix} 2 & 1 \\\\ 0 & 1\\end{bmatrix}^{k+1} &=& \\begin{bmatrix} 2 & 1 \\\\ 0 & 1\\end{bmatrix}^k\\begin{bmatrix} 2 & 1 \\\\ 0 & 1\\end{bmatrix}\\\\ &=& \\begin{bmatrix} 2^k & 2^k-1\\\\ 0 & 1 \\end{bmatrix} \\begin{bmatrix} 2 & 1\\\\ 0 & 1\\end{bmatrix}\\\\ &=& \\begin{bmatrix} 2^{k+1} & 2^{k+1}-1\\\\ 0 & 1\\end{bmatrix} \\]\n\nSo the assertion is true by the Principle of Mathematical Induction (PMI).[/hide]", "Solution_15": "Let's try something other than induction....\r\n\r\n$\\text{Problem 4}$\r\n\r\nProve that $\\sqrt{11}$ is irrational.", "Solution_16": "Let's say $\\sqrt{11}$ is rational, then it can be written in the form $11=\\frac{a^2}{b^2}$, so therefore $b^2 \\cdot 11=a^2$, because an even divided by an even results in an even number, both $a$ and $b$ are odd. But no odd square divides into another odd square unless they have they can be written with the same base. But in this case, this is impossible since the exponent will always be an even number, therefore, $\\sqrt{11}$ is irrational.....maybe.... :? $\\text{QED?}$", "Solution_17": "An even divided by an even is always even? How about 6/2 or 40/8? That's a really good attempt for not having seen a problem like this before. Try again though, cause you're thinking in the right direction.", "Solution_18": "[quote=\"joml88\"]An even divided by an even is always even? How about 6/2 or 40/8? [/quote]\r\n\r\nwow, :| :huh:\r\n\r\n\r\n\r\nok, just gotta think some more\r\n\r\nEDIT: John, why did u delete your post? i'm not gonna post the answer now cuz i'll just copy you word for word", "Solution_19": "Sorry, I posted that before I saw all this extra stuff, so I was going to get rid of it and let you think more. Seeing as I usually post things really early in the morning, I'm not used to posting at the same time as other people. Anyway...\r\n\r\nSuppose $\\sqrt{11}=\\frac ab$ in simplest terms. Then $11b^2=a^2$ and $11|a$. Let $a=11c$; then $b^2=11c^2$ and $11|b$. Therefore, $\\frac ab$ is not in lowest terms, which is a contradiction. QED\r\n\r\n$\\text{Problem 5}$\r\nProve that e is irrational, assuming $e=1+\\frac 1{1!}+\\frac 1{2!}+\\frac 1{3!}+...$" } { "Tag": [ "superior algebra", "superior algebra unsolved" ], "Problem": "This question appears in the section on transcendence bases in Hungerford's Algebra.\r\n\r\nIf $F = K(u_{1},\\ldots,u_{n})$ is a finitely generated extension of $K$ and $E$ is an intermediate field, then $E$ is a finitely generated extension of $K$.", "Solution_1": "do you know german? :-D\r\n \r\nhttp://www.matheplanet.com/matheplanet/nuke/html/viewtopic.php?topic=49649", "Solution_2": "Sorry but I'm afraid i don't know german." } { "Tag": [ "geometry", "trapezoid", "rectangle" ], "Problem": "[img]http://i165.photobucket.com/albums/u46/137456/polemath.png[/img]\r\n\r\nTwo poles with diameters 10 and 30 are bound together by a metal band. How could I find the length of the metal band?\r\n\r\nKnowing that tangents are perpendicular to the radiuses of the circles, I tried drawing trapezoids and such, but to no success. =(", "Solution_1": "[img]http://ipicture.ru/uploads/080604/a4q1JcZ7Yi.png[/img]\r\nWe have $ AD\\equal{}5$, $ CE\\equal{}15$. Draw perpendicular from $ A$ to $ EC$. THen $ ADEF$ is rectangle and $ AD\\equal{}EF\\equal{}5$, so $ FC\\equal{}15\\minus{}5\\equal{}10$. $ A,B,C$ are colinear and $ AC\\equal{}5\\plus{}15\\equal{}20$. Then by Pythagora $ DE\\equal{}AF\\equal{}\\sqrt{AC^2\\minus{}CF^2}\\equal{}\\sqrt{400\\minus{}100}\\equal{}10\\sqrt{3}$", "Solution_2": "Ah, I can't believe I never thought of that. From that triangle I was able to find the angles to also find what fraction of the circumfrences were being touched by the band and added it to 2(DE) to find the answer. Thanks!" } { "Tag": [ "articles", "geometry" ], "Problem": "I have this idea.\r\n\r\nWhy don't we create a separate page in AoPs wiki for each of the books listed [url=http://www.artofproblemsolving.com/Wiki/index.php/Math_books]here[/url] and then one that separate page, we can let users give their in-depth review of it if they own the book and then give it a rating and maybe other stuff. Over time, more and more reviews will accumulate.\r\n\r\nI think this will be very helpful to those people looking to buy certain books especially if the reviews are from AoPSers. (and it will be very organized).", "Solution_1": "I second that!", "Solution_2": "If you make this I have no objection [i]as long as[/i] you make the page for the book and then make a separate page for reviews.\r\n\r\nFor example, the main article could be:\r\n\r\nChallenging Problems in Geometry\r\n\r\nand the review page would be:\r\n\r\nChallenging Problems in Geometry/Reviews\r\n\r\nA link to the second would be on the first, of course. Using this format should keep things neater.", "Solution_3": "This sounds like a very good idea. . .", "Solution_4": "I fifth that! (I think) :lol: \r\nI may be able to review some of the well-known books (AOPS vol.1 and 2, ACOPS, geometry revisited, etc) once I finsih some of them." } { "Tag": [ "Divisor Functions" ], "Problem": "Determine all positive integers $k$ such that \\[\\frac{d(n^{2})}{d(n)}= k\\] for some $n \\in \\mathbb{N}$.", "Solution_1": "Let $ n\\equal{}\\prod p_{i}^{a_{i}}$, so $ d(n)\\equal{}\\prod (a_{i}\\plus{}1)$ and $ d(n^{2})\\equal{}\\prod (2a_{i}\\plus{}1)$. To have $ d(n^{2})\\equal{}kd(n)$, obviously $ k$ must be odd. \r\n\r\nWe will prove that for any odd $ k$ it works. Obviously $ k\\equal{}1$ works, so let $ w$ be the smallest odd positive integer for which it doesn't, and write $ w\\equal{}2^{m}k\\minus{}1$, with $ kb \\text{ or }b>a$.[/hide]", "Solution_2": "Actually the problem is not hard. As you say, Bezout's Identity application will do it here consdiering that ample information is given in the question that $ (a,b)$ is connected to $ (a, b\\plus{}kab)$ and $ (a\\plus{}kab, b)$ for all $ k \\in \\mathbb{Z}$.So this with this along with Bezout's identity will give you insight into constructing the path from $ (1,1)$ to $ (a,b)$.", "Solution_3": "How to construct a path from (a,b) to (a-b,b) using bezout's theorem..??" } { "Tag": [ "real analysis", "real analysis unsolved" ], "Problem": "Consider the system\r\n$ x_1'\\equal{}x_1\\plus{}x_2$, $ x_2'\\equal{}x_2\\plus{}x_3$, $ x_3'\\equal{}x_3$\r\n\r\nFind the solution satisfying $ x_1(0)\\equal{}2, x_2(0)\\equal{}3, x_3(0)\\equal{}4$", "Solution_1": "[quote=\"countdownmath\"]Consider the system\n$ x_1' \\equal{} x_1 \\plus{} x_2$, $ x_2' \\equal{} x_2 \\plus{} x_3$, $ x_3' \\equal{} x_3$\n\nFind the solution satisfying $ x_1(0) \\equal{} 2, x_2(0) \\equal{} 3, x_3(0) \\equal{} 4$[/quote]\r\n\r\nWe find $ x_3$. Then we find $ x_2$. Then we find $ x_1$.\r\n\r\nWe have $ x_3 \\equal{} 4e^t$, $ x_2 \\equal{} e^t(3 \\plus{} 4t)$, $ x_1 \\equal{} e^t(2 \\plus{} 3t \\plus{} 2t^2)$." } { "Tag": [], "Problem": "Sorry about the wait. I will be busy with school from now on so if I am a little slow with things don't worry too much. \r\n\r\nRound 5 Matchups\r\n\r\nHashBrownie vs. theone853\r\n\r\nwhite_horse_king88 vs. nr1337\r\n\r\ntandjsnell vs. kool_dudy\r\n\r\nRC-7th vs. mlee06\r\n\r\nProblems are due by Saturday. I should send them out today.", "Solution_1": "What are the results so far?", "Solution_2": "Results so far:\r\n\r\nHashbrownie ties theone853 4-4 \r\n\r\nnr1337 vs. white_horse_king88 3-TBD\r\n\r\nkool_dudy vs. tandjsnell 2-TBD\r\n\r\nmlee06 def. RC-7th 2-0(RC-7th forfeited due to vacation)\r\n\r\nThose who still need to take it are:\r\nwhite_horse_king88\r\ntandjsnell\r\n\r\nI will get the tiebreakers out to hashbrownie and theone853 as soon as I put them together ;) .", "Solution_3": "Just a reminder: I need to get the answers from white_horse_king88 and tandjsnell by tomorrow or else they will be disqualified.\r\n\r\n[edit] I am also going to send out the next round of winner's bracket questions since they aren't really affected by what happens in the losers bracket.", "Solution_4": "Sorry - My internet was down for about a week. I will take it today.", "Solution_5": "joml, Are the loser's bracket going to have a round off or are both winners and losers have simultaneous rounds?", "Solution_6": "I ll have the rounds going at the same time.", "Solution_7": "AGH! That was very disappointing. That set was much harder and I wasn't prepared when I took it. I spent so much time on number 2, and I still didn't get the answer. At least I have hopes of tieing nr1337.", "Solution_8": "I just got my first set of tiebreakers. (Were they suppose to be for me or were they sent to the wrong person?)\r\n\r\nI'm not sure if I did well on it but I answered all questions.", "Solution_9": "Those were for you kool_dudy.\r\n\r\nUpdated results\r\n\r\nHashbrownie def theone853 (after tiebreakers)\r\n\r\nwhite_horse_king88 ties nr1337 3-3\r\n\r\ntandjsnell ties kool_dudy 2-2\r\n\r\nmlee06 def RC-7th forfeit\r\n\r\nMan you guys really like tying don't you? Well at least we have good matchups this week.", "Solution_10": "Oh...hehe...just makes you work harder joml....\r\n\r\nHave you got my tiebreaker score?", "Solution_11": "Just if you guys were wondering, I was beaten in the tiebreakers 5 to 4.5... Good luck in future rounds, Hasbrownie!", "Solution_12": "Whew, I get a break for a couple weeks! Yay! :lol: \r\nIt couldn't have come at a better time either, since I'm not having any computer access at all beginning Monday Night EST to at least Friday or Saturday.", "Solution_13": "I just sent in my tiebreaker answers.", "Solution_14": "What's the score so far?", "Solution_15": "I just got home so here is the score of one match. \r\n\r\nwhite_horse_king88 def nr1337 (tie 5-4)\r\n\r\nkool_dudy def tandjsnell (tie 2-0)" } { "Tag": [ "combinatorics proposed", "combinatorics" ], "Problem": "Two players replace in turn the hearts in the formula\r\n\\[ M=\\frac{1\\cdot\\heartsuit+2\\cdot\\heartsuit+\\ldots+1001\\cdot\\heartsuit}{1+2+\\ldots+1001} \\]\r\nstarting from the leftmost heart and ending with the rightmost one\r\nby numbers $1, 2, \\ldots, 1001$ in some order (each number can be\r\nused only once). When all the hearts are replaced by numbers, the\r\nplayers evaluate the integer nearest to $M$ and the player who\r\nreplaced one of the hearts by this integer wins. Which of the\r\nplayers has a winning strategy?", "Solution_1": "[quote=\"rogue\"]Two players replace in turn the hearts in the formula\n\\[ M=\\frac{1\\cdot\\heartsuit+2\\cdot\\heartsuit+\\ldots+1001\\cdot\\heartsuit}{1+2+\\ldots+1001} \\]\nstarting from the leftmost heart and ending with the rightmost one\nby numbers $1, 2, \\ldots, 1001$ in some order (each number can be\nused only once). When all the hearts are replaced by numbers, the\nplayers evaluate the integer nearest to $M$ and the player who\nreplaced one of the hearts by this integer wins. Which of the\nplayers has a winning strategy?[/quote]\r\nthe first player, $1001=2n+1$\r\n$M= \\frac{1\\cdot\\ (n+1)+2\\cdot\\ a_1+3\\cdot\\ ((2n+1)-a_1)+4\\cdot\\ a_2+5\\cdot\\ ((2n+1)-a_2)+\\ldots+2n\\cdot\\ a_n+(2n+1)\\cdot\\ ((2n+1)-a_n)} {1+2+\\ldots+(2n+1)}$\r\n$((n+1)-\\frac{1}{2})$ < $M$< $((n+1)+\\frac{1}{2})$", "Solution_2": "[quote=\"Chen241290\"]\nthe first player, $1001=2n+1$\n$M= \\frac{1\\cdot\\ (n+1)+2\\cdot\\ a_1+3\\cdot\\ ((2n+1)-a_1)+4\\cdot\\ a_2+5\\cdot\\ ((2n+1)-a_2)+\\ldots+2n\\cdot\\ a_n+(2n+1)\\cdot\\ ((2n+1)-a_n)} {1+2+\\ldots+(2n+1)}$\n$((n+1)-\\frac{1}{2})$<$M$<$((n+1)+\\frac{1}{2})$[/quote]\r\n\r\n Why $((n+1)-\\frac{1}{2})$<$M$<$((n+1)+\\frac{1}{2})$? :?" } { "Tag": [ "function", "real analysis", "real analysis unsolved" ], "Problem": "Let $ f(x)$ an integrable function in each limited inteval of R and $ f(x\\plus{}y)\\equal{}f(x)\\plus{}f(y)$ $ x\\in{R}$$ y\\in{R}$.Prove that:$ f(x)\\equal{}f(1)x$", "Solution_1": "No calculus. $ f$ integrable $ \\rightarrow$ bounded, using only this it's easy and well-known.", "Solution_2": "[quote=\"lorincz\"]$ f$ integrable $ \\rightarrow$ bounded.[/quote]\r\nThat's using the Riemann theory of integration. What if we mean (locally) Lebesgue integrable? (Which we could weaken to measurable.)", "Solution_3": "if f is integrable, then pts of discontinuity of f is of measure 0....so you can find $ x\\in\\mathbb{R}$ such that f is cont at x. and a additive function continuous at 1 pt can be proved to be continuous on the whole of R. the formula is easy,do it first for rationals and then by density of Q and continuity conclude it holds for all reals.\r\n\r\n@ dr kent- i havent done lebesgue, so..sorry if i said something stupid", "Solution_4": "http://www.artofproblemsolving.com/Forum/viewtopic.php?t=161327" } { "Tag": [ "email" ], "Problem": "Did you guys hear anything about the team this year? Have you got calls from AAPT or something? I'm on spring break right now and I can't get any information.", "Solution_1": "AAPT sends out letters and packets of forms via overnight mail. I think finalists should already have received their letters, but I can't speak for everyone.\r\n\r\nEdit: I say that because I've received my letter.", "Solution_2": "Not necessarily - they didn't know mailing addresses (or emails) for a lot of people, so I would contact your school teacher?\r\n\r\nedit: oh I said that because even though I turned in the forms, they said they still didn't have any info on me, so I guess they reach you however they can reach you.", "Solution_3": "[quote=\"genericme\"]AAPT sends out letters and packets of forms via overnight mail. I think finalists should already have received their letters, but I can't speak for everyone.\n\nEdit: I say that because I've received my letter.[/quote]\r\nCongrats!! But how do they know your address? Do they just send them to the school?", "Solution_4": "[quote=\"simon_math\"]Congrats!! But how do they know your address? Do they just send them to the school?[/quote]\r\n\r\nThanks. I wrote my address on a form that I e-mailed to AAPT - they mailed me the materials directly. Heck, I don't think my school even knows about it. I'll have to tell my physics teacher.", "Solution_5": "I'm kinda confused. :wacko: But how did they tell you that they wanted your address? Did they have your email or something?", "Solution_6": "AAPT e-mailed my teacher some forms that all semifinalists from my school were to fill out \"in case we were chosen as finalists\" (there being two semifinalists from my school), and then he e-mailed them to us.", "Solution_7": "Oh, I see. Thank you. My teacher didn't tell me anything about the form you just mentioned. BTW, when did your teacher get it? Was it this week or some time earlier?", "Solution_8": "My teacher received the same form through email April 3rd or 4th. It said on the form that it must be returned by April 1st, so they revised the deadline to the 6th online. \r\n\r\nSeeing as how I haven't been contacted about being a finalist, I don't think that form has any special significance. Given the date we received it with relation to the date it was supposed to have been turned in, I'm guessing it's just a slip-up AAPT made where they needed more information about the semifinalists. Remember, semifinalists are supposed to get subscription to Physics Today and some other stuff, so they need your address info regardless of whether you made the team.", "Solution_9": "Thanks. I am not an official semifinalist because my QF paper arrived too late for them to grade, but 2 days before the deadline of SF test administration, AAPT called my teacher saying I could get a chance to take the SF test. Probably that's the reason why I didn't get the form. Also, we are on spring break this week, so I wouldn't know it even if my teacher has got the form.", "Solution_10": "So, genericme, how did you do on the semifinal?" } { "Tag": [ "geometry", "3D geometry", "sphere", "advanced fields", "advanced fields unsolved" ], "Problem": "Any idea of a local diffeomorphism $ f: \\mathbb{R}^2 \\longrightarrow S^2$?", "Solution_1": "Just project onto a portion of the sphere, any way you feel like." } { "Tag": [ "geometry", "geometric transformation", "rotation", "superior algebra", "superior algebra solved" ], "Problem": "Let $G$ be the affine group of a comutative field $k$ (that is, the group of mappings from $g$ from $k$ into $k$ which can be written in the form: $g(x)=ax+b$ where a=a(g) is nonzero), then for each triple $(f,g,h)$ of elements of $G$ such that $j=a(fgh)$ is not the identity and such that $fg$, $gh$, $hf$ are not translations, the following two assertions are equivalent. \r\na) $f^3g^3h^3=1$ (the identity mapping)\r\nb) $j^3=1$ and $\\alpha+j\\beta+j^2\\gamma=0$, where $\\alpha$ is the unique fixed point of $fg$, $\\beta$ that of the $gh$, $\\gamma$ that of $hf$.\r\n\r\nFrom this lemma follows Morley theorem which states that three intersection points of the trisectors of a triangle form an equilateral triangle.", "Solution_1": "please help me with this problem..", "Solution_2": "Well, this is Alain Connes' proof for Morley's theorem. :)\r\n\r\nWith these keywords, you should be able to find it on the web, else, I'll post here my own proof of this result.", "Solution_3": "More exactly,\r\n\r\nhttp://www.cut-the-knot.org/ctk/MorleysRedux.shtml", "Solution_4": "Thank you very much. I found the proof.", "Solution_5": "Napoleon theorem: \r\n\r\nGiven a triange $ABC$ and equilateral triangles $ABM$, $BCN$, $ACP$ erected on sides of $ABC$, Prove that $MNP$ is equilateral. \r\n\r\n\r\nThe problem is from some point of view similar with morley's theorem. I wonder if the same method, (the same lemma) doesn't work for this problem (for well chosen $g_i$). \r\n\r\nI thought about choosing $g_i$ rotation of angle $\\frac{2\\pi } 3$ centers about vertex of $ABC$. Points $M,N,P$ are the fixed point of $g_1g_2$ and $g_2g_3$ and $g_3g_1$. And a relation like that in the point b) of the lemma would be helfull. The problem is that $g_1g_2g_3$ is a translation. \r\n\r\nCan this solution be fixed? I see two ways to fix it : \r\n\r\n1) to extend the lemma\r\n2) to choose another tranformation which have $M,N,P$ as fixed points.\r\n\r\nAt a first look I do not expect to find $g_i$ which are not translations. \r\n\r\nThank you in advance,\r\n\r\nCristi" } { "Tag": [ "inequalities unsolved", "inequalities" ], "Problem": "If a,b,c be Positive real number with:Sum(a.^2)=3,Then Prove That:\r\n (a/b)+(b/c)+(c/a)>=(9/a+b+c)\r\n Enjoy it!", "Solution_1": "Question $\\sum a^{2}=3$ Prove $\\sum_{cyclic}\\frac{a}{b}\\geq \\frac{9}{a+b+c}$\r\n\r\nProof\r\n$LHS=\\sum_{cyclic}\\frac{a^{2}}{ab}$\r\nUse Cauchy, we have\r\n$LHS\\geq \\frac{(a+b+c)^{2}}{ab+bc+cd}$\r\nLet $x=\\sum a$; $\\sum ab=\\frac{x^{2}-3}{2}$\r\nThen it's to prove $\\frac{2x^{2}}{x^{2}-3}\\geq \\frac{9}{x}$\r\n$\\iff 2x^{3}-9x^{2}+27\\geq 0$ \r\n$\\iff (x-3)^{2}(2x+3)\\geq 0$ :)", "Solution_2": "See here:\r\nhttp://www.artofproblemsolving.com/Forum/viewtopic.php?t=33612" } { "Tag": [ "equation", "integer equation", "Germany", "TST", "Team Selection Test" ], "Problem": "Let the positive integers $x_1$, $x_2$, $...$, $x_{100}$ satisfy the equation\n\\[\\frac{1}{\\sqrt{x_1}}+\\frac{1}{\\sqrt{x_2}}+...+\\frac{1}{\\sqrt{x_{100}}}=20.\\]\nShow that at least two of these integers are equal to each other.", "Solution_1": "Let $ xn \\geq ... \\geq x2 \\geq x1 $ \r\nWe have $ 20 \\geq 100/ \\sqrt x1 $\r\nSo $ 5 \\geq \\sqrt x1 $\r\nAgain $ 19 \\geq 99/ \\sqrt x2 => 5 \\geq \\sqrt x2 $\r\nWe can prove that $ 6 \\geq \\sqrt x7$ too.\r\nSo we have seven integer numbers which less than 6.\r\nThat means at least two of these integers are equal to each other. :)", "Solution_2": "A slight different answer : It is easy to prove that $ \\frac 1 {\\sqrt n} < 2 \\sqrt n - 2 \\sqrt{n-1}$, from which we deduce the well-known inequality $ \\sum^n_{k = 1} \\frac 1 {\\sqrt n} < 2 \\sqrt n - 1$.\r\nNow suppose that the $x_i^$'s are pairwise distinct, then \r\n$ 20 = \\sum \\frac 1 {\\sqrt {x_i}} \\leq \\sum^{100}_{k=1} \\frac 1 {\\sqrt k} <2 \\sqrt{100} - 1 = 19$. A contradiction.\r\n\r\nPierre.", "Solution_3": "This was used in the Mathematical Olympiad Correspondence Program in Canada some years ago.", "Solution_4": "Yes , MOCP July 2000 , problem 21." } { "Tag": [ "limit", "inequalities", "algebra", "binomial theorem", "real analysis", "real analysis unsolved" ], "Problem": "Find $ \\lim_{n\\to \\infty}( ^p\\sqrt{(n+a_1)(n+a_2)...(n+a_p)} -n) $ , where $ a_1,a_2,...,a_p >0 $ ?", "Solution_1": "Expand out the product:\r\n\r\n$(n+a_1)\\cdots(n+a_p)=n^p+(a_1+\\cdots+a_p)n^{p-1}+O(n^{p-2})$\r\n\r\n$=n^p\\left(1+(a_1+\\cdots+a_p)n^{-1}+O(n^{-2})\\right)$\r\n\r\nTake the $p$th root and use the binomial theorem:\r\n\r\n$\\left[(n+a_1)\\cdots(n+a_p)\\right]^{\\frac1p}=n\\left(1+\\frac1p\\cdot\\frac{a_1 +\\cdots+a_p}n+O(n^{-2})\\right)$\r\n\r\n$=n+\\frac{a_1+\\cdots+a_p}p+O(n^{-1})$\r\n\r\nSubtract $n$ and take the limit to get the answer of\r\n\r\n$\\frac{a_1+a_2+\\cdots+a_p}p$\r\n\r\nNote that the AM-GM inequality says that we must be less than or equal to this number. As $n\\to\\infty,$ we approach the bound given by AM-GM as the limit. This makes sense, since the condition for equality in the AM-GM inequality is that the terms be equal, and (relatively), as $n\\to\\infty$ the terms get more and more alike.", "Solution_2": "My solution is different . I used Bernuli's ineq : \r\n $ (1+\\frac{a_1}{n})(1+\\frac{a_2}{n})...(1+\\frac{a_p}{n}) \\geq 1+\\frac{a_1+a_2+..+a_p}{n} $ \r\n and the lemma : $ \\lim_{x\\to 0}\\frac{(1+x)^m-1}{x} = m $", "Solution_3": "$\\displaystyle \\lim_{x\\rightarrow \\infty}\\frac{f(x)}{x^p}=1 \\Longrightarrow \\lim_{x\\rightarrow \\infty}(\\sqrt[p]{f(x)} -x) = \\lim_{x\\rightarrow \\infty}\\frac{f(x)-x^p}{px^{p-1}}$\r\n :cool:" } { "Tag": [ "algebra", "polynomial", "algebra unsolved" ], "Problem": "Find all polynomials $ P(x,y)\\in R[x,y]$ such that for all $ x,y\\in R$, \r\n\r\n$ p(x\\plus{}y,x\\minus{}y)\\equal{}2p(x,y)$", "Solution_1": "$ p(2x,2y)\\equal{}p((x\\plus{}y)\\plus{}(x\\minus{}y),(x\\plus{}y)\\minus{}(x\\minus{}y))\\equal{}2p(x\\plus{}y,x\\minus{}y)\\equal{}4p(x,y)$. Therefore $ p(x,y)\\equal{}ax^2\\plus{}bxy\\plus{}cy^2$." } { "Tag": [], "Problem": "Prove that\r\n\\[ \\prod_{n=2}^{\\infty}\\left(1-\\frac{1}{n^2}\\right)=\\frac{1}{2} \\]\r\n\r\nMasoud Zargar", "Solution_1": "[hide]The product is \\[ \\left(1-\\frac{1}{2^2}\\right) \\left(1-\\frac{1}{3^2}\\right) \\left(1-\\frac{1}{4^2}\\right) \\cdots, \\] or \\[ \\left( \\frac{2^2-1}{2^2} \\right)\\left( \\frac{3^2-1}{3^2} \\right) \\cdots, \\] and factoring the numerators, we get \\[ \\frac{(1)(3)\\cdot(2)(4)\\cdot(3)(5)\\cdots}{2^23^24^2\\cdots} , \\] and lots of terms cancel. We're left with \\[ \\frac{1}{2}, \\] as desired.[/hide]", "Solution_2": "it is just (n-1)(n+1)/n^2, and it cancels", "Solution_3": "Yes. I just factored and created two different product functions. Then they could be simplified and multiplied together. Then you get $\\frac{1}{2}+\\frac{1}{\\infty}=\\frac{1}{2}$. I'll be back with a complete solution. :)\r\n\r\nMasoud Zargar" } { "Tag": [ "search", "geometry", "circumcircle", "rectangle", "geometry proposed" ], "Problem": "Let $ABCD$ be a convex cyclic quadrilateral. I note $AB=a$, $BC=b$, $CD=c$, $DA=d$. Prove that for any point $P$ exists the folowing relation:\r\n\r\n$\\frac{bc\\cdot PA^2+ad\\cdot PC^2}{bc+ad}=\\frac{cd\\cdot PB^2+ab\\cdot PD^2}{cd+ab}$.\r\n\r\n[u]Remark.[/u] Search several particular cases ($P\\in \\{A,B,C,D,O,E,\\ldots\\}$), where the point $O$ is the circumcenter and $E\\in AC\\cap BD$.", "Solution_1": "After two years this topic is neatly. It is one from the first my topics on this side. \"Kill\" it, please !", "Solution_2": "We use vectors. WOLOG, assume that points $ A$, $ B$, $ C$, $ D$ lie on a circle centred at the origin with radius $ R$. Then\r\n\\begin{align*}\r\n\\frac{bc \\cdot PA^2 + ad \\cdot PC^2}{bc + ad} &= \\frac{bc |\\vec{P} - \\vec{A}|^2 + ad |\\vec{P} - \\vec{C}|^2}{bc + ad} \\\\\r\n&= \\frac{bc (|\\vec{P}|^2 - 2 \\vec{A} \\cdot \\vec{P} + |\\vec{A}|^2) + ad (|\\vec{P}|^2 - 2 \\vec{C} \\cdot \\vec{P} + |\\vec{C}|^2)}{bc + ad} \\\\\r\n&= \\frac{(bc + ad) |\\vec{P}|^2 - 2 (bc \\vec{A} + ad \\vec{C}) \\cdot \\vec{P} + (bc + ad) R^2}{bc + ad} \\\\\r\n&= |\\vec{P}|^2 - \\frac{2 (bc \\vec{A} + ad \\vec{C})}{bc + ad} \\cdot \\vec{P} + R^2.\r\n\\end{align*}\r\nSimilarly,\r\n\\[ \\frac{cd \\cdot PB^2 + ab \\cdot PD^2}{cd + ab} = |\\vec{P}|^2 - \\frac{2 (cd \\vec{B} + ab \\vec{D})}{cd + ab} \\cdot \\vec{P} + R^2.\\]\r\nHence, it suffices to show that\r\n\\[ \\frac{bc \\vec{A} + ad \\vec{C}}{bc + ad} = \\frac{cd \\vec{B} + ab \\vec{D}}{cd + ab}.\\]\r\n\r\nLet $ X$ be the intersection of $ AC$ and $ BD$. Then by similarity of triangles $ XAB$ and $ XDC$, and $ XBC$ and $ XAD$, $ XA: XC = ad: bc$ and $ XB: XD = ab: cd$. It follows that\r\n\\[ \\vec{X} = \\frac{bc \\vec{A} + ad \\vec{C}}{bc + ad} = \\frac{cd \\vec{B} + ab \\vec{D}}{cd + ab}.\\]", "Solution_3": "If ABCD is a rectangle then we have\r\n PA^2 + PC^2 = PB^2 + PD^2\r\n\r\n For non-rectangle it has to be somewhat adjusted,\r\n so I guess with some help from Ptolemy, the factors \r\n in Virgil's enunciation do this adjustment job \r\n cyclicaly and rather nicely.\r\n \r\n\r\n T.Y.\r\n M.T.", "Solution_4": "[quote=\"Virgil Nicula\"][color=darkred] Let $ ABCD$ be a convex [b]cyclic[/b] quadrilateral. I denote $ \\{\\begin{array}{c} AB = a\\ ,\\ BC = b \\\\\n\\ CD = c\\ ,\\ DA = d\\end{array}$ .\n\nProve that $ (\\forall )\\ P$ we have $ \\frac {bc\\cdot PA^2 + ad\\cdot PC^2}{bc + ad} = \\frac {cd\\cdot PB^2 + ab\\cdot PD^2}{cd + ab}$ .[/color] [/quote]\r\n[color=darkblue][b][u]Proof.[/u][/b] For $ E\\in AC\\cap BD$ denote $ \\{\\begin{array}{c} AC = e\\ ,\\ BD = f \\\\\n\\ AE = x\\ ,\\ BE = y \\\\\n\\ CE = z\\ ,\\ DE = t\\end{array}$ . Prove easily that\n\n$ \\frac {x}{ad} = \\frac {y}{ab} = \\frac {z}{bc} = \\frac {t}{cd} = \\frac {e}{ad + bc} = \\frac {f}{ab + cd} = \\sqrt {\\frac { - p_w(E)}{abcd}} = \\lambda\\ \\ (*)\\ .$ \n\nwhere $ p_w(E)$ is the power of the point $ E$ w.r.t. the circumcircle $ w=\\mathcal C(O,R)$ of $ ABCD$ .\n\nApply the [b]Stewart's relation[/b] to the cevian $ PE$ in the triangles $ APC$ and $ BPD$ :\n\n$ \\{\\begin{array}{c} z\\cdot PA^2 + x\\cdot PC^2 = (x + z)\\cdot PE^2 + xz(x + z) \\\\\n \\\\\nt\\cdot PB^2 + y\\cdot PD^2 = (y + t)\\cdot PE^2 + yt(y + t)\\end{array}$ . Using $ (*)$ obtain :\n\n$ \\{\\begin{array}{c} bc\\cdot PA^2 + ad\\cdot PC^2 = (bc + ad)\\cdot PE^2 + abcd(bc + ad)\\cdot\\lambda ^2 \\\\\n \\\\\ncd\\cdot PB^2 + ab\\cdot PD^2 = (ab + cd)\\cdot PE^2 + abcd(ab + cd)\\cdot\\lambda ^2\\end{array}$ $ \\implies$\n\n$ \\boxed {\\frac {bc\\cdot PA^2 + ad\\cdot PC^2}{bc + ad} = \\frac {cd\\cdot PB^2 + ab\\cdot PD^2}{cd + ab}} =$ \n\n$ PE^2 + abcd\\cdot\\lambda ^2 = PE^2 - p_w(E) = PE^2 - OE^2 + R^2\\ .$\n\n[b][u]Remark.[/u][/b] $ \\{\\begin{array}{ccc} P: = A & \\implies & e^2 = \\frac {ad + bc}{cd + ab}\\cdot (ac + bd) \\\\\n \\\\\nP: = B & \\implies & f^2 = \\frac {ab + cd}{ad + bc}\\cdot (ac + bd)\\end{array}\\|$ $ \\implies$\n\nthe [b]Ptolemaeus' relations[/b] $ \\{\\begin{array}{cc} 1\\blacktriangleright & \\boxed {ef = ac + bd}\\ . \\\\\n \\\\\n2\\blacktriangleright & \\boxed {\\frac ef = \\frac {ad + bc}{ab + cd}}\\ .\\end{array}$[/color]" } { "Tag": [ "real analysis", "real analysis theorems" ], "Problem": "When do we say that two measures are [b]measure theoretically equivalent[/b]?", "Solution_1": "Can someone please give the definition. I am still not getting it!", "Solution_2": "Measures $ \\mu$ and $ \\nu$ are called equivalent if they have the same null sets.\r\n\r\nRead more on [url=http://en.wikipedia.org/wiki/Equivalence_(measure_theory)]wiki[/url].\r\n\r\n(Note: googling on \"measure equivalence\" is your friend.)\r\n\r\nOr is this not what you meant?", "Solution_3": "I think you are right. Thanks!", "Solution_4": "Two meausres on a set $ S$ are equivalent if they both give rise to the same open sets that is iff a set $ A$ in $ S$ is open under measure $ \\mu$ then it should be open under measure $ \\nu$ and vice-versa,then $ \\mu \\equiv \\nu$", "Solution_5": "What do you mean by open under a measure?", "Solution_6": "Pardesi appears to be confusing a \"metric\" with a \"measure.\" The condition listed is then the condition for two metrics to be (topologically) equivalent.", "Solution_7": "sorry :blush:" } { "Tag": [ "geometry", "MATHCOUNTS", "ratio", "inradius", "trapezoid", "trigonometry", "similar triangles" ], "Problem": "http://mcis.jsu.edu/mathcontest/geo02.pdf\r\n\r\n\r\n$42,43,44,46,48,50$\r\n\r\n\r\nI only got 39/50. \r\n\r\nI think I messed up on some similar triangles stuff again. I couldn't find elegant solutions for these. \r\n\r\n\r\nI HATE similar triangle stuff that I can't see. Still getting used to it. iI guess it's part of the learning process.\r\n\r\n25,29,31,39,40 are CORRECTED :lol:", "Solution_1": "I'll do more later, too tired.\r\n[hide=\"29\"]We have similiar triangles which are triangles AED to triangle ACB.So $\\frac{3}{15}$=$\\frac{2}{x}$. Solving for x which is the side length of CB is 10.[/hide]", "Solution_2": "lol...thanks for posting these tests. \r\n\r\n25.\r\n\r\n[hide]$7^{3}=343$.\n\nthere are $5\\times 3=15$ holes drilled through so that makes $15\\times 7=105$ units lost.\n\nhowever, there are 9 places where the holes intersect 3 times so add $9\\times 2=18$. $343-105+18=256$\n\n$\\boxed{D}$[/hide]\n\n31.\n\n[hide]Drawing AC and BC, we see that it is just composed of two 1/6 circles minus $[ABC]$. \n\n$2\\cdot\\frac{4^{2}\\pi}{6}-\\frac{4^{2}\\sqrt{3}}{4}=\\frac{16\\pi-12\\sqrt{3}}{3}$\n\n$\\boxed{A}$.\n\nBTW. if you dont already know, the area of an equilateral triangle is $\\frac{s^{2}\\sqrt{3}}{4}$ where $s$ is the side of the triangle. i dont like memorizing formulas a lot but this is one you should definetly remember (especially if ur in mathcounts)[/hide]\r\n\r\n\r\npost more later. gonna watch the apple :D \r\nHAPPY NEW YEAR!\r\nactually i think it already is new year for the east coast. oh well, $1\\frac{5}{12}$ more hours to go.", "Solution_3": "HAPPY NEW YEAR TO YOU TOO! :lol: \r\n\r\n\r\nWell, I figured out why I missed 29. I thought they were asking for length of a hypotenuse so I just didn't see the answer. \r\n\r\n\r\n31, I think I didn't mean to post that number. oh well. \r\n\r\n\r\nWell I still got a good bit more left.\r\n\r\nAny more solutions? :lol:", "Solution_4": "25 is done\r\n\r\n[hide=\"39\"]since the ratio of the the areas is the square of the ratio of the sides, we have that it is $\\sqrt{9}=3$. so since the side length is 1, the base of the smaller triangle is 1/3. the altitudes have this same ratio too, so we have x+3x=1$\\rightarrow$ x=1/4. the areas of the triangles are (1/3*1/4)/2=1/24 and (1*3/4)/2=3/8 so 3/8+1/24=5/12, E [/hide]\n\n[hide=\"47\"]this is simply the diagonal of a 4 by 5 rectangle. $\\sqrt{41}$[/hide]\n\n[/hide]", "Solution_5": "40.\r\n\r\n[hide]We know $OF=4$ because it is one half of $AC$. $FE=\\sqrt{6^{2}-4^{2}}=2\\sqrt{5}$. $\\triangle AFE~\\triangle OAE$. \n\n$\\frac{4}{2\\sqrt{5}}=\\frac{OA}{6}\\Rightarrow AO=\\frac{12\\sqrt{5}}{5}$.\n\n$\\triangle AOE~\\triangle DPE$. The ratio of the sides is 6 to 1 so $DP$ is one sixth of $AO$. The difference of the radii is just five sixths of $AO$ which is $2\\sqrt{5}$.\n\n$\\boxed{D}$[/hide]", "Solution_6": "note: I was purposefully brief so you have to work at the problem yourself. \r\n\r\n42 [hide=\"hint\"]is a nice problem...show that $\\triangle EAD\\sim\\triangle \\triangle ECA$, and others....you should be able to find AC...[/hide]\n\n43 [hide]find the length of their common external tangent, and use the similarity to find the lengths coming from vertex A, then use the pythagorean theorem to find the tangent from C to the large circle, then use semiperimeter *inradius =area[/hide]\n\n44 [hide]let AF=x, and FB=1, then CD=x+1, and use the trapezoid formula, and solve for x[/hide]\n\n46 [hide]find a relation between the distance between centers, the radii, and the length of the common external tangent...[/hide]\n\n48 [hide]if x is a side of the octagon, then $x=2\\sin \\frac{45}{2}$, , then the area is $8*\\frac{x^{2}}{4}$...use double angle formulas...[/hide]\n\n50 [hide]suppose it took 5 seconds to get from A to E, then the horizontal velocity is 3ft/s, and the vertical velocity is 4ft/s, to hit AD, it must travel 20ft horizontal...20/3 sec, that means teh vertical distance traveled was 80/3, but we have to account for bouncing, we can subtract 10s...to get 20/3 and that going up subtract 5, then going down, 5/3, so the answer is 5-5/3=10/3[/hide]", "Solution_7": "assuming that the ball doesn't lose speed, [hide=\"50\"] We can use similar triangles over an over again until the distance of the ball to C is 1. Then, by similar triangles, when the ball hits BC, it is 5/3 away from C. When the ball hits AB again, it is 2 units away from B. We bounce it off of CDand back to AB, and then it is 2 units away from A. When the ball hits AD, it is 10/3 away from A.\n\n\nC[/hide]" } { "Tag": [ "USAMTS", "AMC", "AIME" ], "Problem": "Just wondering, about how many people participate in USAMTS? Does anyone have a rough figure or something?", "Solution_1": "If you look at the overall statistics, there's about 500 participants this year..", "Solution_2": "What percentage of them make it to AIME?", "Solution_3": "500 is really a lot of people.\r\n\r\nMy impression of the leaderboard right now is that a ton of people have over a 43, which is the minimum right now to meet the 68 cutoff, so I'm guessing 15-20%??" } { "Tag": [ "function" ], "Problem": "Let $ f$ a continous function in $ R$\r\n$ k$ a positive real number\r\n\r\nand: $ |f(x) \\minus{} f(y)|\\geq k|x \\minus{} y|$\r\n\r\nProve that f is a bijection.\r\n\r\nInjective is clear\r\nBut surjective....", "Solution_1": "[hide=\"sketch of a solution\"]\nI don't have a lot of time at the moment, but if you divide each side by |x-y| then you get $ \\left|\\frac {f(x) \\minus{} f(y)}{x \\minus{} y}\\right| < k$. Using this you can show that the function is always increasing or the function is always decreasing. Suppose that $ f$ isn't bijective. In the case for always increasing consider the set $ S$ = {all points that aren't in the image of f}. Since $ f$ is increasing, continuous, if any real is in $ S$ then all smaller reals are also in $ S$. Since $ f$ isn't bijective, $ S$ is non-empty, so $ S$ has a supremum, $ a$. Then since $ f$ is increasing there has to be some real number $ r < a$ such that $ r$ is in the image of $ f$ (I'm not quite sure how to make this rigorous). but because $ r < a$, $ r \\in S$ and because $ r$ is in the image of $ f$, $ r \\notin S$, which is a contradiction. The case for decreasing is similar.\n\nI think that's one approach to take, but it would take some work to make it rigorous... maybe?\n\n[/hide]" } { "Tag": [ "number theory proposed", "number theory" ], "Problem": "Let $a;b$ be real numbers satisfying:$a^{3}+2b^{2}-4b+3=0$ v\u00e0 $a^{2}+a^{2}b^{2}-2b=0$\r\nCaculate the value of: $A=a^{2}+b^{2}$", "Solution_1": "Locked, see http://www.mathlinks.ro/Forum/viewtopic.php?t=135914 ." } { "Tag": [ "number theory solved", "number theory" ], "Problem": "Let q a positive rational numer.Prove that there exists an infinity of natural numbers n such that [ nq] and n are coprimes.", "Solution_1": "The following proof shows that all we need to assume is that q is not an integer. Let P_1=2 p_2=3 and so on the sequence of prime. Suppose the conclusion is false. Then we can find k such that for any n>k we have p_n| [qp_n]. The sequence [qp_n]/p_n tends to q and it's a sequence of integers. So it is stationary and its limit must also be an integer. Therefore q must be an integer.", "Solution_2": "assume that q=a/b such that b>1 and (a,b)=1 so we have x,y s.t:ax-by=1 so [(bm+x)a/b]=[am+y+1/b]=am+y .note that (a,y)=1 so as a result of Drichlit theorem am+y for infinitly many m is equal prime number \r\nso we prove that for infinitly many n ,[nq] is prime number, I think that we are done! am I wrong :D" } { "Tag": [ "algebra", "polynomial", "superior algebra", "superior algebra unsolved" ], "Problem": "Let f(x) in k[x] be an irreducible polynomial with coefficients in a field k. Suppose that f(x) has a root of multiplicity greater than one in some extension of k. We want to prove char(k)= p for some prime p and f(x)=g(x^p) for some g(x) in k[x].\r\n\r\nWe want to find all the values of a in Z for which the polynomial f(x)=x^5-ax-1 is not irreducible in Z[x]. For each such a find the corresponding factorization of f(x) into a product of irreducible polynomials in Z[x]", "Solution_1": "At first, note that having a multiple root at $x=a$ means that $f(a) = f'(a) =0$. So when $f$ is irreducible, so when $f$ is the minimal polynomial of $a$, we can only have $f'(a)=0$ when $char(k) \\neq 0$, since otherwise $f'$ would be nontrivial for sure and would have a lower degree (and characteristics are prime when not $0$).\r\nSimilar, when you look at the derivation $f'(x)$, it has to be the zero-polynomial (otherwise it would again have lower degree than the minimal polynomial, contradiction).\r\nBut when the coefficient $a_n$ of $x^n$ is not zero, then by $(x^n)' = nx^n$ we need that $n=0$ (seen in the field, not in integers), thus $p|n$. This gives the second part of the first question." } { "Tag": [ "inequalities", "Diophantine Equations", "pen" ], "Problem": "Find all integer solutions to $2(x^5 +y^5 +1)=5xy(x^2 +y^2 +1)$.", "Solution_1": "I have an ugly way to solve this problem.But I believe it is not the most beutiful and simple one.I am busy these days,Maybe I can post the complete solution some days.But now only an hint:\r\nNotice that the degree of LHS and RHS is 5 and 4,so we may use some inequality:\r\nCase 1.$x<0,y<0$,then $LHS<0,RHS>0$\r\nCase 2.$x\\ge 3,y\\ge3$,$LHS\\ge RHS$\r\nCase 3.$x>0,y<0$.first we get $x+y\\le-1$,and we can deduce that there must exist $A$ such that when $x\\ge A$ $LHS 0$\nCase 2.$ x\\ge 3,y\\ge3$,$ LHS\\ge RHS$\nCase 3.$ x > 0,y < 0$.first we get $ x \\plus{} y\\le \\minus{} 1$,and we can deduce that there must exist $ A$ such that when $ x\\ge A$ $ LHS < RHS$( I didn't compute carefully,maybe$ A \\equal{} 4$ can work.\nThe other case is easy.(we only need to solve a few equation with one variable.)\nI am sorry for the not complete solution :oops:[/quote]\r\nYour solution is not full.\r\nTry again with result:\r\nIf $ x\\plus{}y\\plus{}z\\equal{}0$ then $ 2(x^5\\plus{}y^5\\plus{}z^5)\\equal{}5xyz(x^2\\plus{}y^2\\plus{}z^2)$\r\nImply that it contain term $ x\\plus{}y\\plus{}1$ .", "Solution_3": "Solution was posted here:\r\nhttp://www.mathlinks.ro/viewtopic.php?t=166171", "Solution_4": "$ 5|x^5 + y^5 +1 $ which is impossible because $x^5$ mod 5 can be just 1 or 0 " } { "Tag": [ "MATHCOUNTS", "AMC", "USA(J)MO", "USAMO", "AIME" ], "Problem": "Now that its April 3, I thought a thread for state results would be good. Feel free to list score, rank, school, name, comments ect.", "Solution_1": "I got 4th at chapter and got 2nd at state.\r\nI live in Huntsville, AL so i competed in the Alabama state competiton.\r\nI originally got 8th at state, and 8th in countdown, but after the competition, I was looking at the scoreshhet and realized that they had switched my scores with the girl who got 2nd place(also from my school) who was busy celebrating her trip to nationals. So unfortunately for her, we showed the problem to the state coordinator, and they finally decided to throw out the countdown round completely, and base the results on the written test, so I got second, and I'm going to nationals. I'm in the 8th grade.", "Solution_2": "I think this is one of the lowest scoring competitions ever in Texas. 32 was the cut off for the top 10 and the best score was a 46 by Mark Zhang.", "Solution_3": "[quote=\"oneal3691\"]I got 4th at chapter and got 2nd at state.\nI live in Huntsville, AL so i competed in the Alabama state competiton.\nI originally got 8th at state, and 8th in countdown, but after the competition, I was looking at the scoreshhet and realized that they had switched my scores with the girl who got 2nd place(also from my school) who was busy celebrating her trip to nationals. So unfortunately for her, we showed the problem to the state coordinator, and they finally decided to throw out the countdown round completely, and base the results on the written test, so I got second, and I'm going to nationals. I'm in the 8th grade.[/quote]\r\nWeird. We had a similar problem where for countdown, two people had the same name and the wrong one went up. I think the name was like Xing-Xao. Anyway, its been a month and I havent gotten my scores back :( :( :( :( :( . But I know my scores were bad. I got a 4 :( :( :( :( :( :( :( :( :( :( :( on target and <24 on sprint :( :( :( :( :( :( . Thats a total of <32 :( :( :( :( :( . Maybe its because I prepared for like 1 hour total and only did school competitions and never got better than 40 and only had 5 (really) practices this school year. O well, at least I got 4th written and 1st countdown :first: official! :first: :first: . See you at nationals!!", "Solution_4": "I was crying after the sprint round because I thought I had lost the chance for nationals, probably barely above 20. Then after that it makes sense that I did so bad at target getting only 5 right. Cut off was a 34 so I probably made a 33 or a 32. When I found out the cutoff was so low, I was very, very, mad. Of course Fort Settlement owned the competition anyway... that's the best team I've ever seen at state...", "Solution_5": "The cut for the countdown was a 34\r\nhehe....\r\nI'm from Fort Settlement \r\nWe got first at state and our individuals scores were 46, 45, 45, and 43 but the people going to nats got 46, 45, 45, and 45. We got a perfect score on the team round. :)", "Solution_6": "[quote=\"tarquin\"]I was crying after the sprint round because I thought I had lost the chance for nationals, probably barely above 20. Then after that it makes sense that I did so bad at target getting only 5 right. Cut off was a 34 so I probably made a 33 or a 32. When I found out the cutoff was so low, I was very, very, mad. Of course Fort Settlement owned the competition anyway... that's the best team I've ever seen at state...[/quote]Did you make Nationals?", "Solution_7": "Well the cutoff for [b]top ten[/b] was a 34. I got caught in some huge tie and made 15th... which is kind of sad...", "Solution_8": "O so thats a \"no\". At least theres next year if your in 6th/7th grade", "Solution_9": "yes i did", "Solution_10": "[quote=\"Magixmaniak\"]yes i did[/quote]Score, state?", "Solution_11": "[quote=\"tarquin\"]Well the cutoff for [b]top ten[/b] was a 34. I got caught in some huge tie and made 15th... which is kind of sad...[/quote]Wow, that must have been a really hard test. I remember two years ago I got a 37 or 38 and ended up 31st. But those nationals scores...46,46,45,45...wow...", "Solution_12": "texas, and i got fourth, so whatever score corresponds...", "Solution_13": "It was more difficult then most. But I swear I should have gotten like a 43 or something in that range... The scores were so messed up...!", "Solution_14": "lol", "Solution_15": "[quote=\"Secret Asian\"][quote=\"mathgeek2006\"]I'm not sure what my score was, (need to ask my coach still) but I know I got 5th individual and 2nd Countdown (Utah)--I think I got like a 32 or something...I'm pretty good at Countdown--If only I had a chance to get in there at nats.... :([/quote]\n\nUm..32?? I tied for first in Utah and I had [hide][hide][hide]30:([/hide][/hide][/hide] \nYou were 5th place. So you couldn't have gotten over my score.[/quote]\n\n[hide=\"top secret information\"][hide]It was a J-O-K-E\nI wanted to see if people actually believed it....[/hide][/hide]", "Solution_16": "In Nevada, I came in 6th in chapter with a score of 31 (21/5). :? However, I came in first at state with a 34 (24/5). :)", "Solution_17": "Massachusetts, my state got a 44, 43, 41, and 39 I think,\r\n\r\nI came in second", "Solution_18": "Well unlike 2003, where you had Adam Hesterberg who was 5th as a seventh grader in 2002, or 2004, where you had Greg Gauthier who was 6th written as a seventh grader in 2003, there aren't any real big favorites. Sergei and David slipped into the countdown, but they got beat by several people at states. This year will be more open, and we'll just see who's better prepared and luckier.", "Solution_19": "Well, Sergei's slow with countdown, as I've beat him twice this year, and on chapter we both got that poorly worded #29 on sprint, problem wrong. On state his mistakes were just two stupid addition mistakes.", "Solution_20": "#29 was not worded badly", "Solution_21": "Don't worry about it. Although I usually don't make too many stupid mistakes on test day, there's a lot of luck involved in MC. You know, two years ago, I was realistically aiming at top 20 at nats, but of course I died on test day and got 50th or something.", "Solution_22": "I think that I got number 29 wrong but not because of the wording. I did 5c2= 10, 6c2 = 15, and then there are 7 left, 7c2 = 21. Then I multiplied these together, but got it wrong. Can someone suggest a reason why this approach is wrong?", "Solution_23": "[quote]nat mc \t\nPostPosted: Mon Apr 11, 2005 4:27 am Post subject:\nMark Zhang will be owned, he won't win[/quote]\r\n\r\nMark never gets owned except once by dennis (chapter)", "Solution_24": "Phelpedo, you did the same thing wrong as I did. The roles of the actors/actresses DID matter, so you wouldn't choose, you would permutate, or whatever the word is. Hope that helps.", "Solution_25": "[quote=\"Phelpedo\"]I think that I got number 29 wrong but not because of the wording. I did 5c2= 10, 6c2 = 15, and then there are 7 left, 7c2 = 21. Then I multiplied these together, but got it wrong. Can someone suggest a reason why this approach is wrong?[/quote]\r\n\r\nsirorange is right, since roles are different, order does matter, so you have to add in duplicates, (if man 1 and man 2 were chosen, you could choose man 2, and man 1 and that would be different), so you could just use the counting principle 5 men, 6 women, 2 male roles, 2 female roles, 2 interchangeable roles... 5 men for first role, 4 for second, 6 for first women role, 5 for second and (5+6-4 roles that have been cast) gives you 7 for the first interchangeable role, and 6 for the second interchangeable role...so basically 5*4*6*5*7*6=25200 ways to cast", "Solution_26": "Yeah.... I got that one wrong too, I thought the roles weren't distinct, so I didn't multiply by that extra 8.", "Solution_27": "That's bad", "Solution_28": "9 people got 46s at state? I hate to think that there are liars on AoPS...", "Solution_29": "Wow, hard to believe, but possible." } { "Tag": [ "analytic geometry" ], "Problem": "Given a line segment where you know both endpoints, and it is not vertical or horizontal, how would you find the midpoint?\r\nThanks,", "Solution_1": "Say the endpoints were $ (a,b)$ and $ (x,y)$. The midpoint would be $ \\left(\\frac{a\\plus{}x}{2},\\frac{b\\plus{}y}{2}\\right)$. Try proving it.", "Solution_2": "Thanks,\r\nHmm...thats odd, its like averaging the x and y values.", "Solution_3": "You can always draw a horizontal and a vertical leg, forcing this line segment to be the hypotenuse of a right triangle. The midpoint in the x direction would be the average of the x values (think of it as a number line ranging from 3 to 7). This also applies for the y direction.", "Solution_4": "[quote=\"eli140\"]\nHmm...thats odd, its like averaging the x and y values.[/quote]\r\n\r\nExactly. If you think about it, that's really what a midpoint is. Directly between the line $ x \\equal{} x_1$ and $ x \\equal{} x_2$, as well as $ y \\equal{} y_1$ and $ y \\equal{} y_2$.", "Solution_5": "izzy is correct, midpoint mainly means like the middle of the point or the point where the distance from the other points is the same so averaging out the x's and the y's coordinates would help u find the midpoint" } { "Tag": [ "function", "\\/closed" ], "Problem": "This is kinda related to the forum...I'm trying to make an avatar, but the program only uses .png, and I want to convert to .jpg to save size. I can't open it in paint to convert it and the program will only let you save as .png (fireworks). Does anyone know of a free program that will do this for me?", "Solution_1": "If you have Fireworks, you can use the 'Export' function to create a gif or jpg or whatever. I think it's in the File menu -> go to Export Wizard (or something like that) and follow the directions.", "Solution_2": "Ah, thanks!" } { "Tag": [ "search" ], "Problem": "The tallest inhabited building in the world is the Sears Tower in Chicago. If the observation tower is 1450 feet above ground level, determine HOW FAR a person standing in the observation tower can see (with the help of a telescope) keeping in mind that the Earth's radius is 3960 miles. \r\n\r\nHINT GIVEN:\r\n\r\n1 mile = 5280 feet.", "Solution_1": "please don't ask for answers to your homework assigments on this board, however you are allowed to ask for tips...\r\n\r\n[hide=\"tip\"]theroritacally, if her looked up, then he could see for millions of miles. Is the question \"what is the furthest thing away he can see on the earth\"? then, I would recomend using triangles.[/hide]", "Solution_2": "How do you know this is homework?\r\n\r\n-especially, since [i]most [/i]schools are out already.", "Solution_3": "[hide=\"More Hints\"]\nThe farthest you can see is the line tangent to the Earth's surface.\n\nLet triangle $ABC$ be the triangle formed by joining the top of the Sear's tower $A$, the farthest you can see $B$, and the center of the Earth $C$. Then, angle $ABC$ is a right angle.\n\nUsing the values you know, solve for the side $\\overline{AB}$. Depending on how precise you want your answer, you can solve for the triangle formed by the base of the Sear's Tower and $\\overline{AB}$, though the difference between the two is probably negligible.\n[/hide]", "Solution_4": "Here's a related [url=http://www.artofproblemsolving.com/Forum/viewtopic.php?search_id=1314609914&t=13321]problem[/url] that I posted several years ago.", "Solution_5": "Whenever I post questions here, I seek tips only and nothing more.\r\n\r\nThanks", "Solution_6": "use tangents to the circle.", "Solution_7": "Can this question be solved using the Pythagorean Theorem?", "Solution_8": "Yes, if you look at my previous explanation, it is evident that $\\overline{AB}^{2}+\\overline{BC}^{2}= \\overline{AC}^{2}$ Plug in the values you know, and you have the answer.\r\n\r\nAB = Distance from person to farthest away he can see (What you are solving for)\r\nBC = Distance from farthest away person can see to center of the Earth (Earth's radius)\r\nAC = Distance form person to center of the Earth (Earth's radius + Sear's Tower)", "Solution_9": "Similar problem here:\r\nhttp://www.artofproblemsolving.com/Forum/viewtopic.php?t=152767", "Solution_10": "Gyan,\r\n\r\nYou link really helped me to fully grasped this question.\r\n\r\nI know how to use a^2 + b^2 = (a + b)^2\r\n\r\nThanks", "Solution_11": "[quote=\"sharkman\"]Gyan,\n\nYou link really helped me to fully grasped this question.\n\nI know how to use a^2 + b^2 = (a + b)^2\n\nThanks[/quote]\r\n\r\nwtf that's not true", "Solution_12": "[quote]I know how to use a^2 + b^2 = (a + b)^2 [/quote]\r\n\r\n$a^{2}+b^{2}= (a+b)^{2}-2ab$. Just in case. :D", "Solution_13": "I was looking at what I'd ever posted in the high school program, and came across this topic from many years ago. I decided to revive it with two comments.\n\n1. The building formerly known as the Sears Tower is now officially called the Willis Tower. (I last lived in Chicago more than 35 years ago, but even so I have a sneaking suspicion that quite a few current residents of Chicago don't use the new name for the building.)\n\n2. But at the same time, the Willis/Sears Tower is now nowhere close to being the tallest inhabited building in the world. So perhaps this problem should be re-posed, but with the building being the Burj Khalifa in Dubai. According to what I could look up:\n\nHeight of observation deck: 555 m (1821 feet)\nHighest occupied floor: 584.5 m (1918 feet)\nTop of spire/antenna: 829.8 m (2722 feet)" } { "Tag": [ "geometry", "parallelogram" ], "Problem": "Take a parallelogram as in the figure. Pick any point in the interior and draw lines from that point to each of the four corners. Color opposite triangles red and blue (as in the figure). Use some basic geometry to show that no matter where the point is placed the total area of the two red triangles will equal the total area of the two blue triangles.", "Solution_1": "You can show that the sum of the areas all equal to half of the paralellogram.", "Solution_2": "yes(it's enough to choose the point on the one of vertices) ,and i think it's very simple but nice problem !", "Solution_3": "Or if you want, draw a horizontal line through the point in question.", "Solution_4": "Same stands for a square. And we can use that property to prove the law of cosines.", "Solution_5": "i think the proof is obvious !\r\ni have posted a same post with some differences for regular triangle i think.", "Solution_6": "I construct altitudes of the triangles through the point......But i think yours is better. :D", "Solution_7": "i think that proof works for every n-regular." } { "Tag": [ "induction", "function" ], "Problem": "Let $S \\in \\mathbb{Q}$ such that:\r\n[list]1. $\\frac{1}{2} \\in S$\n2. If $x \\in S$, then both $\\frac{1}{x+1}$ and $\\frac{x}{x+1} \\in S$[/list]\r\n\r\nProve that if $k \\in \\mathbb{Q}^{(0,1)} $ then $k \\in S$", "Solution_1": "I think it can be done by induction. :)", "Solution_2": "BTW. kueh! Do you have pdf?", "Solution_3": "[quote=\"zhaobin\"]I think it can be done by induction. :)[/quote]\r\n\r\nInduction on?", "Solution_4": "[quote=\"Myth\"]BTW. kueh! Do you have pdf?[/quote]\r\n\r\nThere is are pdfs of all the past british mo 1 and 2 on the official website: http://www.bmoc.maths.org/home/bmo.shtml", "Solution_5": "[quote=\"kueh\"][quote=\"zhaobin\"]I think it can be done by induction. :)[/quote]\n\nInduction on?[/quote]\r\nI have done it for some time.now let me raise my momery.\r\nfor a rational number $\\frac{m}{n}\\in(0,1),mq, therefore c ^2 +1

2, then p+1-2a is even so q is even so q=2, therefore b=1, so 2(a ^2 +1)=c ^2 +1=(p-a) ^2 +1=(a ^2 -a+1) ^2 +1. If a=1, that's impossible.If a=2 that's possible, and from here we obtain the solution (a,b,c)=(2,1,3) (and also (1,2,3), because we supposed that p>q).\r\nIf a>2 then a ^2 -a+1=a(a-1)+1>=2a+1, so (a ^2 -a+1) ^2 +1>=(2a+1) ^2 +1 > 2(a ^2 +1).\r\n\r\nTherefore (1,2,3) and (2,1,3) are the only solutions.", "Solution_2": "The 2nd problem:\r\n\r\nwe presume that: m>n. Let x be their common solution of the two equations. Then x verifies x^n(x+1)(x^(m-n)-1)x=0 and x=-1 is not a solution. Let x be so x^(m-n)=1. We have x<> \\pm 1, x from C\\R, x satisfies: x^(m-1)=1, x^n*(x-1)=-1. We apply the module and get |x|=1 and |x-1|=1 => x cos(pi/3)+isin(pi/3) or x=cos(5pi/3)+i sin(5pi/3), which \r\nmeans x=w or x=w (w has a line on it) , where w^2-w+1=0, w is the 6th order root of unity.\r\nTaking modulo 6 of m and n we get that the only good cases are when m==1(mod 6) and n==1(mod6). So the common roots are given by:\r\n1) the eqs x^2-x+1=0 for m==n==1(mod 6).\r\n2) in other cases it doesn't exist!!!\r\n\r\ncheers mate!" } { "Tag": [], "Problem": "I come across one question and I do it in two ways , both ways \"sounds\" logical but there is something wrong in it ... here it goes\r\n\r\nQuestion : There are 5 couples , 4 person are chosen among them with the condition the must be at least one male and one female . In how many ways can this be done ?\r\n\r\n[b]First Method[/b] : \r\n\r\nAll ways - 4 male - 4 female = $10C4 - 5C4 - 5C4 = 200 $\r\n\r\n[b]Second Method[/b] :\r\n\r\nFirst we chose 1 male and 1 female which is $5C1\\times 5C1$ then for the remaining two vacancies , we can choose it randomly from the remaining 8 person which is $8C2$ . So total ways = $5C1\\times 5C1\\times 8C2 = 700 $ \r\n\r\nWell , second method seems to flow logically but we know it is not true . Where actually goes wrong ? :?", "Solution_1": "to me the second methodmakes more sense; on the other hand:\r\n10C4=210.. ;) \r\n\r\nits 12:50am, i'm too sleepy to think, so i'm wrong", "Solution_2": "your second solution is wrong because you count cases more than one.\r\ni.e: $f_1 m_1 f_2 m_2$ and $f_1 m_2 f_2 m_1$ and $f_2 m_1 f_1 m_2$ and $f_2 m_2 f_1 m_1$ so you count this case $4$ times!!! ;)", "Solution_3": "[quote=\"a_vakilian\"]your second solution is wrong because you count cases more than one.\ni.e: $f_1 m_1 f_2 m_2$ and $f_1 m_2 f_2 m_1$ and $f_2 m_1 f_1 m_2$ and $f_2 m_2 f_1 m_1$ so you count this case $4$ times!!! ;)[/quote]\r\n\r\nHmm .... I guess that is why the second method gives us such big number :rotfl: .\r\nAnyway , could there be any way to get the same answer by using the concept like second method ? Like , could we divide it by some number(or combination) to cancel off the repeated part ? :?", "Solution_4": "yes it is possible. you should divide the problem to $3$ case.when there is $2 woman ,2 man$ and $1 woman , 3 man$ and $3 woman, 1 man$ . ;)", "Solution_5": "[quote=\"a_vakilian\"]yes it is possible. you should divide the problem to $3$ case.when there is $2 woman ,2 man$ and $1 woman , 3 man$ and $3 woman, 1 man$ . ;)[/quote]\r\n\r\nWell , what Im trying to ask is whether we can use the concept in second method , which is like setting a male and female first then the rest can be chosen at random . I know that we can divide it into 3 cases like you mention to solve it but what i want to know is whether the second method can be improve or not to make it right ?", "Solution_6": "I think there is not such a solution that you want. or it is not worth knowing :? :oops:" } { "Tag": [ "geometry", "3D geometry", "sphere", "calculus", "integration", "HMMT", "LaTeX" ], "Problem": "A sphere with a radius of 5 is cut by a plane that is 4 away from the center of the sphere. What is the volume of the larger of the two resulting solids?", "Solution_1": "[hide]I know that you could use calculus (take the equation for a circle, and use the cross sectional method and find the area beneath the graph from -5 to 4).\n\n$\\int_{-5}^{4}{\\pi(\\sqrt{25-x^{2}})^{2}dx}$\n$\\pi\\int_{-5}^{4}{(25-x^{2})dx}$\n$\\pi((25*4-\\frac{4^{3}}{3})-(25*-5-\\frac{(-5)^{3}}{3}))$\n$162\\pi$\n\nAnd you could imagine a hole with a radius of 3 being driven through the sphere. Then if you compare each cross section of that $h$ away from the center with each cross section of a sphere with a radius of 4, $h$ away from the center you notice they are equal. Then you add in the cyclinder that would fill up most of the hole and a sphere that is chopped off by two planes. \n\nSo end the end you would end up with $162\\pi$ like before.[/hide]\r\n\r\nEven if these answers aren't correct I think the process is correct. I just want to find a solution without using calculus or using a crazy idea.", "Solution_2": "I don't believe a non-calculus formula exists for a sectoroid (a sector revolved about its axis) or a segmentoid although I may be mistaken\r\n\r\neven if an antiderivative existed to the integral for the volume of a segmentoid, it would most definitely not be very nice. Calculus is the best method here", "Solution_3": "There is a non calculus way to find the volume of a sectoroid. \r\n\r\nhttp://web.mit.edu/hmmt/www/datafiles/problems/2003/pgeom03.pdf\r\n\r\nProblem #6 is not quite a sectoriod, but you can just add the cylinder in. The answer is in the solutions:\r\n\r\nhttp://web.mit.edu/hmmt/www/datafiles/solutions/2003/sgeom03.pdf\r\n\r\nAnd I am pretty sure I have posted both the calculus way and the noncalculus (from the links) way above. But I am guessing there really is not a way to find the volume without calculus or a great imagination so my question is answered.", "Solution_4": "The solution is perfectly reasonable if you know the geometric derivation of the formula for a sphere.", "Solution_5": "Yeah, I can't think of a non-calculus solution....", "Solution_6": "Hint: Cavalieri's principle \r\n\r\nUse google to find out what it is.", "Solution_7": "The solution Altheman is hinting at (which is essentially what is used in the HMMT solutions linked to above) begins like this:\r\n\r\nSit a cone of radius $r$ and height $r$ down next to a hemisphere of radius $r$ so that the vertex of the cone is in the same plane as the base of the hemisphere and the plane of the base of the cone is tangent to the hemisphere. (If the hemisphere is sitting on a table base-down, the cone is balanced on the same table on its tip.) Now consider the combined area of a cross-section of the two shapes at height $h \\leq r$ above the table. This is a nice way to get the volume of a sphere. It can also be used to get a nice formula for the \"cap\" of a sphere, as in the question here.\r\n\r\nSide note: this requires you to know the volume of a cone, but this can also be derived using a Cavalieri argument.", "Solution_8": "o.O I am trying to force people to be independent and use google. This skill is very valuable. \r\n\r\nWell...I suppose if you were being all rigirous and stuff, you would need to derive the volume of a cone by putting it in correspondence with a pyriamid. But obviously surface area/volume of a cone fall within most first year geometry classes.", "Solution_9": "[quote=\"Altheman\"]I am trying to force people to be independent and use google.[/quote] The effect you are having is of being cryptic and ensuring that most of your audience won't ever find out what you're talking about.\n\n[quote]But obviously surface area/volume of a cone fall within most first year geometry classes.[/quote] But the volume is usually asserted, not proved.", "Solution_10": "[quote=\"JBL\"][quote=\"Altheman\"]I am trying to force people to be independent and use google.[/quote] The effect you are having is of being cryptic and ensuring that most of your audience won't ever find out what you're talking about.\n[/quote]\n\n[hide=\"@JBL\"]Okay. Lets consider an analogous situation. We are given a triangle concurrency problem. I suggest mass points [lets say in the HSB forum]. Then I should expect to get lots of stupid replies asking: \"what are mass points?\" \"where can I learn more about mass points?\" These are stupid questions because you can easiliy find the answer by a quick google search. I know you have this ability. ex. consider your response to mathfanatic in that global warming thread about following fallacies created by those against global warming. Anyway, this situation is similar. You can easily type: \"cavaleri principle sphere\" into google and obtain the link: [url]http://www.walter-fendt.de/m14e/volsphere.htm[/url]. This takes a few seconds and you get a diagram which I find much more helpful than any solution provided in this thread. While you do not get a complete solution in this page, it is enough of a hint to finish the problem. \n\nFurthermore, I think this type of hint is nicer than a full solution. This way the solver can still get a sense of accomplishment from solving the problem. I find that a fundamental part of my enjoyment of problem solving is when I come up with the solution on my own. By offering hints, I let the solver get started on the problem. The full solution should be left to those who have to struggle to solve the problem.\n\nIf I may predict your response, I think that you will think that this is a lot of ---- and I am just being lazy. I am not sure what position you think you are in so you can interpret my actions. I am in a better position than you are to I know what I am thinking.[/hide]\n\n[quote=\"JBL\"][quote]But obviously surface area/volume of a cone fall within most first year geometry classes.[/quote] But the volume is usually asserted, not proved.[/quote]\r\n\r\nReally? I am suprised about that. In freshman year geometry, [i]before I was interested in math[/i] [i.e. so I took not particular action to figure this out], I was aware of this proof:\r\ni) V=1/3 b*h where the base is a triangle; this comes from decomposing a parallelpiped [or w/e you call it]\r\nii) V=1/3 b*h where the base is a polygon; decompose the polygon into triangles and apply i)\r\niii) V=1/3 b*h where the base is a circle; a circle is a polygon with an infinite number of sides; apply ii)\r\n\r\nPerhaps I had a better geometry class than I thought; I thought that this was the standard \"non-calculus proof.\"\r\n\r\nI suppose that is not 100% rigirous since we are dealing with limits. But that is being pretty picky.", "Solution_11": "[hide=\"@Altheman\"]You think saying, \"use mass points\" in the HSB forum is educational -- I think it is pedagogically unsound. Simply telling people to google things will lose most of your audience immediately -- they won't bother. Of those who do google it, they won't know what to google for, and a substantial portion of them will fail to find an appropriate introductory source. Far, far better (in my opinion) is to explain how to do something, and (if it's not obvious) show why it works. (Cavalieri's Principle is something where the \"why\" is really calculus, but the result itself is very intuitively clear.) In general, the general result is something best understood [i]after[/i] particular examples have been dealth with, and knowing the name (or the formula, or whatever) but not understanding it is worthless. Teachers should do more than simply hand their students a textbook for each question, and this is all the more true when the textbook in question is the entire internet.\n\nYou're probably right that I've misinterpreted your intent in the past, so I retroactively apologize. (That does not apply to my complaints about LaTeX non-usage, though :wink: )[/hide]\r\n\r\n[quote=\"JBL\"]i) V=1/3 b*h where the base is a triangle; this comes from decomposing a parallelpiped [[i]maybe prism? -- JBL[/i]]\nii) V=1/3 b*h where the base is a polygon; decompose the polygon into triangles and apply i)\niii) V=1/3 b*h where the base is a circle; a circle is a polygon with an infinite number of sides; apply ii)\n\nPerhaps I had a better geometry class than I thought; I thought that this was the standard \"non-calculus proof.\"\nI suppose that is not 100% rigirous since we are dealing with limits. But that is being pretty picky.[/quote] It could just be that my memory of geometry class is very rusty :). I'm not quite sure what you mean by (i), but it sounds plausible. I don't like (iii) very much, because I think Cavalieri's is a nicer way to go from a single pointy thing to all pointy things (in other words, there's no particular reason to go via (ii) -- one can spit out (ii) and (iii) together without ever using the words \"infinitely many\"). A lot has to do with how well something is explained, I guess.", "Solution_12": "Well...I learn by reading the material first, then asking questions. And the lack of LaTex can be attributed to my procrastinating from school work...but w/e my posts have generally improved now that school has ended.\r\n\r\nAnyway, Here is euclid's proof for $\\frac{1}{3}bh$: http://aleph0.clarku.edu/~djoyce/java/elements/bookXII/propXII7.html\r\n\r\nI think that it is reasonable to consider a circle as a limiting form of a polygon with a large number of sides. If you are particularly picky, you can consider the inscribed and circumscribed regular polygons with n sides. Then you can apply the method of exhaustion [originally an idea from archimedes]. To show that the circle is inbetween these two volumes. This is quite rigirous actually. In the development of integral theory by Tom Apostol in \"Calculus,\" this is one of the 4 fundamental axioms that he takes for area. The analogue to volume is just as acceptable; if one object contains the other, then they have equal volume. Regardless, these are ideas that my dad would understand [and he does psychology]. I think they are sufficiently rigirous for say, the ancient greek geometers." } { "Tag": [ "MATHCOUNTS", "geometry", "AMC", "AIME", "AMC 12", "AMC 10" ], "Problem": "Are they helpful? I am deciding on getting them and I want your thoughts on the books. What are their levels and what contests do they prepare you for? Are they worth the money? Thanks.", "Solution_1": "They definitely are very helpful, at least in my opinion and what seems to be the majority of AoPSers. I haven't really started Volume 2 yet, but Volume 1 is extremely helpful for MathCounts and the AMCs, particularly the geometry section.", "Solution_2": "They are very very very helpful! Vol 1 covers Mathcounts/AMC stuff, Vol 2 is more AIME/AMC/ARML stuff.\r\n\r\nVol 1 is recommended for middle school , Vol 2 is recommended for high school", "Solution_3": "Just make sure that if you get them, you do a lot of the problems and actually learn math rather than being a passive reader.", "Solution_4": "[quote=\"isabella2296\"]They definitely are very helpful, at least in my opinion and what seems to be the majority of AoPSers. I haven't really started Volume 2 yet, but Volume 1 is extremely helpful for MathCounts and the AMCs, particularly the geometry section.[/quote] did you make aime isabella", "Solution_5": "If I was aiming for AIME via AMC12 (failed this year by a few mistakes, motivated to try AoPS now) should I just skip the first volume?", "Solution_6": "Thanks to all. I will buy now.", "Solution_7": "[quote=\"Ktk\"]If I was aiming for AIME via AMC12 (failed this year by a few mistakes, motivated to try AoPS now) should I just skip the first volume?[/quote]\r\nAoPS Vol.1 doesn't go into details about the stuff that's on the AMC 12 and not AMC 10, but it covers basics for the AMCs and just knowing all of that stuff really will help you more than knowing more advanced stuff from Vol. 2. So you should get Vol. 1." } { "Tag": [ "\\/closed" ], "Problem": "Recently, I've found that when I try to use the \"hide\" command, instead of applying the hide format, it will instead display the code. Was this an intentional modification, or a bug?", "Solution_1": "Did you remember to close with (/hide)? (replace () with [])\r\n\r\n[hide=\"Hiding\"]...[/hide] works fine for me.", "Solution_2": "Oops...\r\n\r\nAt least one time it was because I used the wrong slash. :oops: \r\n\r\n[hide=\"This is a test.\"] :maybe: [/hide]\r\n\r\nYep, that must have been it, because the above hide works just fine." } { "Tag": [ "function", "algebra proposed", "algebra" ], "Problem": "Prove that the elements of the set $ A\\equal{}\\{ (x,y)|x \\in \\mathbb{Z},y \\in \\mathbb{Z}, 2xy\\minus{}6x\\plus{}y\\equal{}0 \\}$ are on the graphic of the function $ f: \\mathbb{R} \\rightarrow \\mathbb{R}, f(2x\\minus{}3)\\equal{}8x\\minus{}7.$", "Solution_1": "[quote=\"moldovan\"]Prove that the elements of the set $ A \\equal{} \\{ (x,y)|x \\in \\mathbb{Z},y \\in \\mathbb{Z}, 2xy \\minus{} 6x \\plus{} y \\equal{} 0 \\}$ are on the graphic of the function $ f: \\mathbb{R} \\rightarrow \\mathbb{R}, f(2x \\minus{} 3) \\equal{} 8x \\minus{} 7.$[/quote]\r\n\r\nUnfortunately, this is wrong : $ (\\minus{}1,6)\\in A$ but is not on the required graphic since $ f(\\minus{}1)\\equal{}1\\neq 6$ and $ f(6)\\equal{}29\\neq \\minus{}1$)" } { "Tag": [ "AMC", "AIME", "USAMTS", "USA(J)MO", "USAMO" ], "Problem": "Since this year seems to be easier than usual, what effect will that have on the cutoff? how far up will the cutoff move? Also, how is the cutoff determined? Is it the first score for which >x people got that score or above for some x?", "Solution_1": "[quote=\"solafidefarms\"]Since this year seems to be easier than usual, what effect will that have on the cutoff? how far up will the cutoff move? Also, how is the cutoff determined? Is it the first score for which >x people got that score or above for some x?[/quote]\r\nI think that there are an x number of people who qualify for the AIME. Anyways, if the third round is uber hard than the qualifying score might be as low as 60-63.", "Solution_2": "You start off with an index of 100 if you qualify through USAMTS, right?\r\n\r\nalso, people that qualify for the AIME through the USAMTS can qualify for the USAMO by both their index and the floor score, right?", "Solution_3": "I think the index only matters in USAMO qualification is you are in 11th or 12th grade. Otherwise you go by floor which means if you score at least the score the lowest AIME score that the people that qualified by the index, you are in. So I don't think the USAMO index really does matter for us...\r\n\r\nBut I am interested in how qualifications to AIME is decided. It would feel good if I knew that even if I bombed the AMCs, I would still be in the AIME." } { "Tag": [ "linear algebra", "matrix", "search", "linear algebra unsolved" ], "Problem": "DO any of u know what a \"[b][color=black][size=150]DIRAC MATRIX[/size][/color][/b]\" is?", "Solution_1": "I have two words for you: [b]GOOGLE SEARCH[/b].", "Solution_2": "THanks for that!!!!!\r\n\r\n\r\n@carcul- ia almost forgot that" } { "Tag": [ "combinatorics unsolved", "combinatorics" ], "Problem": "There are n markers, each with one side white and the other side black,\r\naligned in a row so that their white sides are up. In each step, if possible, we\r\nchoose a marker with the white side up (but not one of the outermost markers),\r\nremove it and reverse the closest marker to the left and the closest marker to the\r\nright of it. Prove that one can achieve the state with only two markers remaining\r\nif and only if n - 1 is not divisible by 3.", "Solution_1": "merged with http://www.mathlinks.ro/viewtopic.php?t=90046 .\r\n\r\n darij" } { "Tag": [ "number theory proposed", "number theory" ], "Problem": "find all pairs of natural numbers $ (n,m)$ such that $ n!\\plus{}1\\equal{}m^2$\r\n\r\n\r\n$ (4,5),(5,11),(7,71)$ is this all ?", "Solution_1": "This a very well known problem ( I think it is Brocard's problem) but unfortunately it hasn't been solved yet. I t seem that those are the only solutions but it is not known.\r\nIf someone can solve it I don't think he will post it here without publishing it :P :rotfl: \r\nBut I recommend you to try it, it's very nice\r\n\r\n\r\nDaniel" } { "Tag": [ "calculus", "derivative", "real analysis", "real analysis unsolved" ], "Problem": "Show that $x^{n}+nx-1 =0 \\quad n\\geq 1$, has exactly one real root $a_{n}$. Does the series\r\n\\[\\sum_{n}a_{n}\\]\r\nconverge ?\r\n\r\n(I'm not sure about the problem. Think it should be \"one real [i]positive[/i] root\")", "Solution_1": "It has at least one positive root by the Intermediate Value Theorem; its value at zero is negative, and it goes to $\\infty$ at $\\infty$. If $n$ is even, it has a negative root as well, so it should specify \"positive real root\".\r\nFor uniqueness, show that the derivative is positive for positive $x$; Rolle's theorem finishes the proof.\r\n\r\nThe series diverges: $\\frac1n1$.", "Solution_2": "[quote=\"jmerry\"]\n\nThe series diverges: $\\frac1n1$.[/quote]\r\n\r\nThanks for your reply, but how did you get the bounds for a_n ?", "Solution_3": "Looking again, they're not quite right- I was rushed.\r\n\r\nLet $f_{n}(x)=x^{n}+nx-1$. $f_{n}(\\frac1n)=(\\frac1n)^{n}>0$\r\n$f_{n}(\\frac1{n+1}=\\frac1{(n+1)^{n}}+\\frac{n}{n+1}-1< \\frac1{n+1}+\\frac{n}{n+1}-1=0$\r\n\r\nBy IVT, there is a zero of $f$ between $\\frac1n$ and $\\frac1{n+1}$." }